Вы находитесь на странице: 1из 2049

Skeptical Review Edited by Farrell Till

The Skeptical Review


Volume One - 1990
Farrell Till, editor

• Number 1 Volume One, Issue One


• Number 2 Volume One, Issue Two
• Number 3 Volume One, Issue Three
• Number 4 Volume One, Issue Four

Volume 1990 - 2002 Issue


Page 1 of 2049
Skeptical Review Edited by Farrell Till

The Skeptical Review


Volume One, Number One
January/February/March 1990
Farrell Till, editor

Why Another Religious Publication?


The first cover article for the first issue of The Skeptical Review. Describes the
purpose of TSR and its editorial policy.
The Last Hurrah of the Inerrancy Doctrine
Discusses the verbal inspiration of the Bible, the contradiction between Exodus 6:16-
20 and Exodus 12:40, the original autographs of the Bible.
The Inerrancy Doctrine is Found to be Impregnable
Fundamentalist pastor Jerry Moffit defends the inerrancy of the Bible and responds to
Till's argument about Exodus 6:16-20.
The Flat-Earth Belief of Bible-Writers
Have you ever heard the claim that "the Bible teaches the earth is flat?" Adrian
Swindler surveys the appropriate Biblical evidence to defend this view.
The Unfinished Debate
Information about the written debate between Farrell Till and Church- of-Christ
preacher James H. Laws on the plenary inspiration of the Bible.
Other Debates
An update on the debates with Jerry Moffitt and Bill Jackson.

Why Another Religious Publication?


If there is anything the world already has enough of, surely that would be religious
publications. The pouches of our mail carriers bulge with religious journals, magazines,
periodicals, and bulletins already circulating through our postal system. The Church of Christ,

Volume 1990 - 2002 Issue


Page 2 of 2049
Skeptical Review Edited by Farrell Till
which the publishers of The Skeptical Review were once affiliated with, produces the Gospel
Advocate, Firm Foundation, Christian Chronicle, Reason & Revelation, Power, and at least a
score of smaller, less known publications, and the journalistic production of many other
churches is just as prolific. Why then would anyone want to add anything else to this
mountainous glut?

We could quibble and say that ours is not a religious publication. In the strictest sense of the
word, religion is the expression of belief in and reverence for a supernatural entity, so in that
respect ours will certainly not be a "religious" publication. At no time will we be found
admonishing our readers to "believe in God"; we won't even get involved in the theism-
atheism controversy. Being primarily agnostic in our convictions, we will leave this matter to
those who are far more sure of their positions (on both sides) than we could ever be.

Our purpose will be to promote critical examination of the Bible inerrancy doctrine, and that
will make ours an unusual publication. There are journals that devote some space to this
subject, but only one other we know about that exists only to challenge traditional
assumptions on which this belief has thrived. As our readers get to know us, we suspect that
many will concur in saying that we are far from religious. Some might even insist that
"irreligious" more appropriately describes us. Nevertheless, our editorial purpose will focus in
each issue on an important religious matter--the doctrine of Bible inerrancy--and so to that
extent we have to agree that ours is a religious publication. We just can't think of The
Skeptical Review as being only "another" religious publication. We think we will be quite
different from the others.

An important difference will be our editorial policy of publishing competently written


responses to our lead articles. Not many religious journals do this. Most of them publish only
materials that agree with the particular doctrinal beliefs they were created to promulgate. In
this issue, for example, we are publishing "The Inerrancy Doctrine Is Found To Be
Impregnable," which Jerry Moffitt was invited to write in reply to "The Last Hurrah of the
Inerrancy Doctrine," our lead article that begins on the next page. In future issues, we will
continue to publish dissenting views.

We had a twofold purpose in adopting this policy. First, we firmly believe that no
philosophical opinion, not even our own, is worthy of acceptance if it cannot survive the
scrutiny of public forum. Secondly, we believe that printing responses to our articles will
demonstrate the soundness of our position. If the best qualified minds in the inerrancy camp
cannot refute our arguments--and we believe they can't--that should be a strong indication, at
the very least, that our position deserves further consideration.

With the publication of our first issue, we earnestly solicit your consideration of our views on
this important matter. If you disagree with us, write a response. We will consider it for
publication. If you have questions, send them to us. We will try to answer them.

Volume 1990 - 2002 Issue


Page 3 of 2049
Skeptical Review Edited by Farrell Till

The Last Hurrah of the Inerrancy Doctrine


Farrell Till
Many fundamentalist Christians sincerely believe that the Bible is the verbally inspired word
of God. As believers in verbal inspiration, they see the Bible much differently from those who
respect it as a book with only concepts and ideas that were divinely inspired. Christians who
believe in the doctrine of verbal inspiration think that God directed the writing of the Bible on
a word-by-word basis so that the authors of the original manuscripts were protected from
writing even as much as one word that might inadvertently mislead readers or incorrectly
communicate the truths God wanted man to know.

Dr. George DeHoff, who is widely recognized in the Churches of Christ as an authority on the
subject of verbal inspiration, described its word-by-word process like this:

If God had wanted another "i" dotted or another "t" crossed, He would have had it done. The
writers did not use one word unless God wanted that word used. They put in every word
which God wanted them to put into the Bible, (Alleged Bible Contradictions Explained, p.
23).
Other fundamentalist writers like Jerry Falwell and Pat Robertson have said essentially the
same thing. As far as they are concerned, there is nothing to discuss. The Bible is the word of
God and the only word of God, whose writing he himself divinely inspired on a meticulously
protected, word-by-word basis.

Certain consequences must necessarily follow the postulation of such a rigidly defined
doctrine as verbal inspiration. The most obvious of these would be a requirement to believe
that the Bible is inerrantly perfect in every detail. After all, the God of the Bible is depicted as
an omniscient, omnipotent entity, so if an all-knowing, all-powerful supernatural being
supervised the writing of the Bible on a word-by-word basis anything at all like the process
described above by Dr. DeHoff, it would have to be that the original text of the Bible was
completely free of mistakes of any kind. A perfect God would have guided his chosen writers
to produce a perfect book.

With this conclusion, Christian fundamentalists have no quarrel. In Finding Inner Peace and
Strength (Doubleday, 1982), Jerry Falwell claimed total inerrancy for the Bible:

The Bible is the inerrant... Word of God. It is absolutely infallible, without error in all matters
pertaining to faith and practice, as well as in areas such as geography, science, history, etc.,
(p. 26).
This is not to say that believers in the inerrancy doctrine view the Bible as a textbook in
geography, science, history, or other disciplines; it is simply a recognition that the conclusion
stated above must necessarily follow their claim of verbal inspiration, for if God is ultimately
the author of the Bible, he, knowing everything there is to know about geography, science,
history, and all other secular subjects, would have made no errors--not even little ones--in any
of these matters.

Volume 1990 - 2002 Issue


Page 4 of 2049
Skeptical Review Edited by Farrell Till
The importance of this point has not escaped the notice of Dr. Gleason Archer, a widely
respected spokesman for the inerrancy position:

If the statements it (the Bible) contains concerning matters of history and science can be
proven by extrabiblical records, by ancient documents recovered through archaeological digs,
or by the established facts of modern science to be contrary to the truth, then there is grave
doubt as to its trustworthiness in matters of religion. In other words, if the biblical record can
be proved fallible in areas of fact that can be verified, then it is hardly to be trusted in areas
where it cannot be tested, (Encyclopedia of Bible Difficulties, p. 23).
In this statement, Dr. Archer has made an important admission. He has conceded that
confidence in the divinely authoritative position traditionally assigned to the Bible will be
seriously compromised if erroneous information should be found anywhere within its pages.

Obviously, then, the doctrine of verbal inspiration sets a high standard for the Bible to meet.
Some insist that it is an impossibly high standard, because the existence of contradictions,
discrepancies, absurdities, scientific errors, and other mistakes can be easily established by
anyone willing to subject the Bible to objective textual criticism. This is our position exactly
at The Skeptical Review. Our staff writers believe the same methods of scientific inquiry that
have lifted man to his present state of enlightenment, if applied unbiasedly to the Bible text,
will disprove once and for all the doctrine of verbal inspiration.

Future articles will examine in detail specific examples of textual errors in the Bible, so for
now I will review only briefly a few of the ones that cast serious doubts on the doctrine of
Bible inerrancy. An excellent one to begin with would be the obvious contradiction that
results when Exodus 12:40 is compared to the Aaronic genealogy found in Exodus 6:16-20.
The first passage declares that the Israelites, who were beginning their famous journey to the
promised land, had dwelt in Egypt for 430 years. According to the genealogy in Exodus 6,
however, the Israelite sojourn in Egypt could have lasted no more than 352 years and
probably even considerably less than that.

This genealogy, along with its parallels in I Chron. 6:1-3 and 23:6-13, establishes that Moses
was the great grandson of Levi. Kohath, the grandfather of Moses, had already been born
when Jacob took his sons and their families into Egypt, (Gen. 46:11). If we assume that
Kohath was only a suckling infant in his mother's arms when he was taken into Egypt and if
we further assume that his last act on earth at the age of 133 (Ex. 6:16) was to sire Amram,
the father of Moses, then the very latest date of Amram's birth would have been around 134
years into the Israelite sojourn. If we then make similar assumptions about the birth of Moses,
i.e., that Amram sired him just before dying at the age of 137 years (Ex. 6:20), this would
mean that Moses could have been born no later than 272 years after the Israelite sojourn
began. Since Moses was only 80 years old when Jehovah (Yahweh) called him to lead the
Israelites out of Egypt (Ex. 7:7), the sojourn could have lasted no longer than 352 years.

But to allow even 352 years for the so-journ would require total abandonment of common
sense. For one thing, the custom of listing sons in the order of their births in Jewish
genealogies suggests that the Bible writers understood that both Kohath and Amram had
younger brothers (Gen. 46:11; Ex. 6:16-18), so Kohath was probably older than an infant
when he was taken into Egypt. If he did live to be 133, he undoubtedly fathered Amram,

Volume 1990 - 2002 Issue


Page 5 of 2049
Skeptical Review Edited by Farrell Till
Moses' father, long before he died, because, it is completely unreasonable to assume
circumstances of birth anything at all like those theorized above. The aged Abraham fell on
his face and laughed when Yahweh told him that he would soon father a son. "Shall a child be
born unto him that is a hundred years old?" Abraham asked, (Gen. 17:17). By the same token,
we can ask if it is reasonable to believe Kohath and Amram were able to father children when
they were well past the age of 130.

In the final analysis, however, whether the sojourn lasted as long as 352 years doesn't really
matter. The genealogical data in Exodus 6:16-20 clearly indicate the belief in an Egyptian
sojourn substantially shorter than 430 years, so that puts this Bible passage in unequivocal
conflict with Exodus 12:40, Genesis 15:13, and Acts 7:6, all of which teach that the sojourn
lasted at least 400 years. There is an obvious contradiction in the Bible text.

A hundred articles like this one would not be enough to discuss the numerous other textual
contradictions in the Bible. Many of the same events from Hebrew history reported in the
books of Samuel and Kings were also recorded by the writer(s) of the Chronicles, and the two
accounts often vary significantly in reporting key details. There are contradictions in the
genealogical records in the Bible, in the synoptic gospels (especially their accounts of the
resurrection of Jesus), in the Christology of the New Testament epistles. In a word, the Bible
is a veritable maze of irreconcilable contradictions. Yet fundamentalist preachers never tire of
proclaiming the Bible to be a perfectly harmonious, inerrant record of God's dealings with
man.

Traditionally, purveyors of the Bible inerrancy doctrine have profited from the ignorance,
superstition, and gullibility that characterize societies in which mystical religions thrive, but
recent discoveries and developments in biblical archaeology and criticism, coming in an age
of increased scientific enlightenment, have cut deep inroads into territory once firmly held by
the forces of inerrancy. Early Christian apologists, for example, claimed that not just the
original Bible autographs were inspired of God but also all copies and translations that scribes
and linguists had transmitted to later generations. Such a position was sustainable in a time
when illiteracy was commonplace, Bible manuscripts rare, and textual criticism all but
nonexistent, but with the discovery of Bible manuscripts unknown to previous generations of
Christians, the invention of the printing press and the ensuing proliferation of vernacular
translations, the contributions of archaeology and higher criticism to the field of Bible
research, and the advent of public education, the absurdity of this belief became so obvious
that it could not survive. Today, not even the staunchest fundamentalist would dare claim that
all copies and translations of the Bible have been divinely protected from error.

After losing this decisive battle, the defenders of inerrancy retreated to the only high ground
left for them. They found refuge in claiming that at least the original autographs of the Bible
were inspired of God and so by necessity inerrant. Since none of the original autographs had
survived the passing of the centuries, perhaps the inerrancy advocates thought that they had at
last set up an impenetrable line of defense. After all, if there are no original autographs in
existence, how could anyone possibly prove that they were not inerrant?

The fallacy in this line of reasoning should be obvious to anyone who has even rudimentary
skills in critical thinking. Logicians call it the argument from ignorance. The fact that one

Volume 1990 - 2002 Issue


Page 6 of 2049
Skeptical Review Edited by Farrell Till
cannot disprove an assertion does not prove the truth of the assertion, since the absence of
negative evidence by itself is never conclusive positive evidence. The theist who says, "You
cannot disprove the existence of God, so it must be true that God does exist," is guilty of the
argument from ignorance. One could just as well argue that a failure to disprove the existence
of elves must mean that elves do exist, and with that kind of logic one could prove just about
any fantastic claim.

The argument from ignorance also disregards the burden-of-responsibility principle of logic.
This often ignored principle obligates the claimant of a proposition to prove that his claim is
true. The challenger of the proposition is under no obligation to prove that it is not true.
Accordingly, the one who claims that inerrant autographs of the Bible once existed is
obligated to prove that they did indeed exist. To demand that those who question the
inerrancy doctrine prove that inerrant original autographs did not at one time exist is a resort
to the argument from ignorance. If a believer in Islam should demand proof that the angel
Gabriel did not inspire the prophet Mohammed to write the Koran, even the most radical
Christian fundamentalists would see the fallacy in his reasoning, yet they cannot recognize the
same faulty logic when they apply it to their belief in Bible inerrancy.

Obviously, then, the claim that all original autographs of the Bible were error free is a
postulation that no fundamentalist can ever hope to prove. Furthermore, this claim not only
does not validate the inerrancy position, it makes Yahweh appear even more ridiculous than
some of the stories attributed to him in the Bible, for if God (Yahweh) deemed inerrant
original autographs of the Bible necessary for the people living at the time the originals were
written, then surely he would have considered inerrant copies and translations of the originals
necessary for succeeding generations. To argue that God (Yahweh) carefully protected the
original Bible autographs from error but then left all subsequent transmissions of them to
careless, uninspired scribes and translators is, as I said, to make God look perfectly ridiculous.
In Inspiration of Scripture: Problems and Proposals, Paul Achtemeier very competently
explained the absurd implications of this last-ditch effort of Bible fundamentalists to save
their cherished inerrancy doctrine:

It has been frequently pointed out that if God thought errorless Scripture important enough to
inspire its composition, he would surely also have further inspired its copying, so that it might
remain error free. Surely a God who can inspire error-free composition could also inspire
error-free copying. Since he did not, it would appear he did not think our possession of error-
free Scripture very important. But if it is not important for us, why was it important
originally? (pp. 71-72).
Until inerrancy advocates can give a satisfactory explanation to the problem Achtemeier has
here identified, they would do well to cease talking about "inerrant original autographs." Such
talk only makes their position look even more ridiculous.

We live in an age of advancing technology when Bible scholars (true Bible scholars) are
demonstrating a readiness to combine that technology with recognized principles of scientific
criticism to test old assumptions about the origin of the Bible. Computer analyses of Biblical
manuscripts have been done that cast serious doubts on traditional theories of authorship.
Archaeological studies have completely debunked the old myth that says the Bible is
absolutely inerrant in matters of history, geography, and science, as well as faith and practice.

Volume 1990 - 2002 Issue


Page 7 of 2049
Skeptical Review Edited by Farrell Till
In a word, recent developments in Biblical criticism have not been kind to the inerrancy
doctrine.

Even staunchly conservative churches once regarded as impregnable bastions of Christian


fundamentalism have begun to count their losses. In the April 1989 issue of the Gospel
Advocate, editor F. Furman Kearley described the problem that liberalism now poses to
traditional inerrancy beliefs in the Churches of Christ:

We have college professors who speak unrebuked and un-refuted at Christian college
lectureships affirming that we must accept the results of higher criticism. These professors
reject the Mosaic authorship of the Pentateuch and the doctrine of the inerrancy of Scripture.
They are supported by their administrators....
We have in the church today quite a number occupying prominent pulpits and professorships
who call Genesis 1 and 2 myth and reject the universal flood clearly described in Genesis 6-9
and in 2 Peter 3:1-7. I am concerned about those few who are doing such teach- ing, but I am
far more concerned by the silence of many others who have not spoken out to refute such
false teaching and to make clear that they or their institutions oppose such, ("Unfaithful in
Little; Unfaithful in Much," p. 27).
From the sound of Dr. Kearley's lament, a genie has been released that hard- line
conservatives in the Church of Christ are going to have a difficult time getting back into its
bottle. Truth has a stubborn way of "hanging tough," so if in this fundamentalist body half as
many ministers and college professors as Dr. Kearley implies have finally seen the truth about
the Bible's origin, the situation isn't likely to get any better for the old guard.

The principle at work here is the same as the one suggested by a question asked in an old
World War I song: "How are you ever goin' to keep 'em down on the farm after they've seen
Paree?" Apparently, a lot of preachers are at long last beginning to see the Paree of
responsible Bible criticism, and no one is ever going to get them back down on the inerrancy
farm.

The Inerrancy Doctrine Is Found To Be


Impregnable
A reply to "The Last Hurrah...."
Jerry Moffitt

It is with great pleasure that I lift my pen in defense of God's word. Surely it is an act of grand
benevolence that men can be allowed to so serve an all-powerful God. But God's goodness
toward us is surely equaled by His love and patience toward skeptics in that He allows His
eternal existence and His act of revealing His will to man to be so loudly but ineptly
challenged by what must appear to Him to be but microscopic specks in the universe. Yet, I

Volume 1990 - 2002 Issue


Page 8 of 2049
Skeptical Review Edited by Farrell Till
do not want to take away from the fact that the editor of The Skeptical Review graciously
invited me to respond to the article "The Last Hurrah of the Inerrancy Doctrine." On the
editor's part, it is truly an act of justice, honesty, and fairness, all of which traits Mr. Till
learned from the Bible rather than from skepticism.

As a soldier of Christ (II Tim. 2:3), whose weapons are not of the flesh (II Cor. 10:3-5), I do
not want to appear crass, cold, or vituperative. However, Jesus dealt with different people in
different ways, so in this situation I choose to be bold, frank, and bluntly honest throughout.
Mr. Till's article seems to me to be little more than several pages of orchestrated malarkey. It
is held together with bubble gum and kite string. He comes to us bent over double with wild
assertions, and if he has one keen ability, it is to invent imaginative straw men none of us ever
believed. The article starts off "kind of funny in the head" and then it gets even worse. Let me
quickly deal with all that sort of "stuff," and then I want to deal with the two best of what
could be called arguments.

Assertions
He claims there are discrepancies, absurdities, scientific errors, and contradictions in the
genealogical records and the synoptic gospels that make the Bible a veritable maze of
irreconcilable contradictions. He claims that an age of increased scientific enlightenment has
cut deep inroads into the inerrancy doctrine. Now if such assertions were facts, inerrancy
would be in extremely deep trouble. But such boastful claims are not new. Thomas Paine
thought his The Age of Reason would destroy the Bible. He predicted that within one hundred
years Bibles would be found only in "museums or in musty corners of second-hand
bookstores." He died in 1809, and today the Bible remains a bestseller. Voltaire said it took
centuries to build up Christianity and "I'll show how just one Frenchman can destroy it within
fifty years." Twenty years after his death, the Geneva Bible Society purchased his house to be
used to print the Bible. Later it became the Paris headquarters for the British and Foreign
Bible society. I do not wish to be unkind, but I sincerely believe Farrell Till is no Voltaire,
and The Skeptical Review is no Age of Reason. I challenge Mr. Till to continue to invite me
and his other opponents to review every so-called discrepancy, absurdity, and contradiction he
imagines. We will, by God's help, show that there are no true contradictions proven to be in
the Bible. Further, we will be glad to show just how kind archaeology has been to the Bible.
But this one thing we will admit: the Bible predicted that there would always be apostasy and
false teaching ( II Tim. 3:13; I Tim. 4:1-3; I Jn. 4:1; II Pet. 2:1-3). Liberalism, modernism,
and higher criticism fulfill, in part, those predictions. We do not find it strange that liberals
deny inerrancy. Knowing that they also deny almost every other major doctrine of
Christianity as well makes it useless that Mr. Till the skeptic seeks help from them. Liberals
do not believe in inerrancy, true. They also do not believe in biblical miracles, hell, heaven,
that Jesus Christ was God, or that there is a personal devil. The problem is not the Bible; it is
a problem of liberalism, which is simply the age-old problem of unbelief.

Mr. Till asserts that early Christian apologists claimed all copies and translations were
inspired. We all, even today, speak in similar language of copies and versions. That does not
mean we do not recognize tampering exists, that parts of a version may not have been
rendered faithfully by translators, or that only the originals were inerrant. The apostles knew
their words could be twisted (II Pet. 3:16). They even warned against adding to or taking from

Volume 1990 - 2002 Issue


Page 9 of 2049
Skeptical Review Edited by Farrell Till
them, (Rev. 22:18-19). Pickering (The Identity of the New Testament Text, p. 107) says the
following:

Marcion's truncated canon evidently stirred the faithful to define the true canon. But Marcion
also altered the wording of Luke and Paul's Epistles, and by their bitter complaints it is clear
that the faithful were both aware and concerned...
Dionysius, Bishop of Corinth (168-176), complained that his own letters had been tampered
with, and worse yet the Holy Scriptures also.
Irenaeus defended 666 over 616 as the correct number in Revelation 13:18. He warned of
future punishment for those who changed the text. No, they did not argue that every version
or copy was inspired, inerrant, untampered with or unchanged; they knew certain versions,
like Marcion's, could deviate from the original.

Till asserts that after losing a decisive battle on all copies and translations being inspired, we
retreated to the position that only the original manuscripts were inspired. The above
quotations regarding Marcion's version prove that claim silly, and for lack of a better name we
will call all that "unremittent hokum" and go on toward more responsible claims.

As for higher criticism, the more we study it the less confidence we have in it. Spawned by
German liberalism, which denied the miracles of the Bible, its roots lie in the attempt to
explain away miraculous prophecy. To do that, they must try to prove the prophecy was made
at the same time or after, not before, the event. Thus, they have to claim the prophecy wasnot
made at the time it claims or by the prophet it claims. Having first settled all that in their
minds, they then set out to prove it by higher criticism. Again, I do not want to be
uncharitable, but I suspect that regarding such writers the more warm and overheated their
imagination, the more the editor of The Skeptical Review will cherish them.

He claims that the theist who says, "You cannot disprove the existence of God, so it must be
true that God does exist," is guilty of the argument from ignorance. So he says that to demand
that those who question the inerrancy doctrine prove that inerrant original autographs did not
at one time exist is a resort to the argument from ignorance. Will someone inform him it is
equally true that to demand proof from opponents of his errancy doctrine that errant original
autographs did not at one time exist is a resort to his argument from ignorance? What's sauce
for the goose is sauce for the gander. The truth is, what we say is this: he must take the
multitude of copies that do exist and show that no other explanation is possible than that they
came from errant originals. If he fails to do this, he utterly fails to disprove the Bible is
inspired and inerrant in the original autographs.

One "Argument"
He argues through Paul Achtemeier that the inerrancy doctrine makes God look perfectly
ridiculous for this reason: "If God thought errorless Scripture important enough to inspire its
composition, he would surely also have further inspired its copying, so that it might remain
error free.... If it is not important for us, why was it important originally?"

First, we are not so bold as to decree what is and what is not important to God. For all we
know, He may well have gotten across His will to man through errant originals if He had
chosen to do so. The originals may just happen to be inerrant because in the truest sense of the

Volume 1990 - 2002 Issue


Page 10 of 2049
Skeptical Review Edited by Farrell Till
word they are God's word, and God cannot lie (Tit. 1:2), and neither can the Spirit of truth (Jn.
16:13) affirm error.

Second, God often brings an original thing into existence--as He did Adam and Eve--by
miracle; then He wants that item to carry on and produce naturally, under mere providence.
Third, on top of that, we have a Bureau of Standards on which we may check all copies. God
may have wanted perfect standards to be available to those who really seek. We believe that
in the thousands of manuscripts available today, we have all the original readings. The science
of textual criticism assures us of that very fact. After all, Jesus said, "Heaven and earth shall
pass away, but my words shall not pass away," (Matt. 24:35).

Another "Argument"--Exodus 6:16-20


Mr. Till says according to Exodus 12:40 the Israelites sojourned in Egypt 430 years. But
Exodus 6:16-20 indicates that there were only three generations between Levi and Moses and
Aaron. That would stretch things to get even 352 years from those generations. We agree with
that, if there were only three generations, but the Bible often gives genealogies by listing the
main characters in the genealogies according to the general purpose of the writer. Notice
Matthew 1:1:
The book of the generation of Jesus Christ, the son of David, the son of Abraham.
Between Christ and David 26 ancestors are left out. Between Abraham and David 12
ancestors are left out. That was the way they abbreviated according to their own individual
purposes.

As for Exodus 6:16-20, I Chronicles 6:1-3, and I Chronicles 23:6-16, let us notice it is a
similar case. Gleason Archer (Encyclopedia of Bible Difficulties, pp. 111-112) says:

In common with almost all the genealogies of this type recorded in the Pentateuch (cf.
Numbers 26:28-34), the general practice is followed in Exodus 6 of listing a person's family
tree by tribe, clan, and family group.
Archer further points out that Numbers 3:27-28 says the combined total of Amramites,
Izharites, Hebronites, and Uzzielites came to 8,600. If Amram claimed one fourth of those
and if that same Amram fathered Moses and Aaron, as Till argues, Moses and Aaron (by Till's
argumentation) would have had around 2,150 brothers. That should be hard for even a
dedicated skeptic like Farrell Till to swallow. No, these figures indicate the genealogy of
Exodus 6:16-20 is listing only the main links just as Matthew does in Matthew 1:1. The first
Amram is a kind of clan head of a person's family tree.

Furthermore, Archer points out that other genealogies in I Chronicles indicate that there were
nine or ten generations between the sons of Jacob and the time of Moses. There were ten links
between Ephraim and Joshua (I Chron. 7:25), seven between Bezalel and Jacob (I Chron. 2:1-
20), and nine between Elishama and Jacob (I Chron. 7:22-27). Nine or ten links fit the 430-
year time span perfectly.

Agreeing with all this, Arndt says (Bible Difficulties, p. 80), "It was not at all uncommon in
the Hebrew genealogical tables to omit names which were considered unimportant." In the old
classic work of John Haley (Alleged Discrepancies of the Bible, p. 420), Haley says, "It has
been conclusively shown by Kurtz and others that the omission of several names in a

Volume 1990 - 2002 Issue


Page 11 of 2049
Skeptical Review Edited by Farrell Till
genealogy was common; and the words 'bear' and 'beget' are used with reference to somewhat
remote ancestors."

So all that is the best Mr. Till can bring up, and it has all been answered time and time again,
long, long ago, over and over.

CONCLUSION
The truth is that the Bible is inerrant, absolutely so in all its original autographs. God cannot
lie and scripture is His word ( Num. 23:19; Titus 1:2; Heb. 6:18; I Jn. 2:21; II Tim. 3:16-17).
Notice a few quotations:
"Thy righteousness is an everlasting righteousness, and thy law is truth," (Ps. 119:142).

"The sum of thy word is truth," (Ps. 119:160).

"And now, O Lord Jehovah, thou art God, and thy words are truth," (II Sam. 7:28).

Jesus said not one jot or one tittle would pass away from the law till all things be
accomplished, (Matt. 5:17-19). A jot was not only a single letter; it was the smallest letter in
the Hebrew alphabet. Some Hebrew characters looked alike, so writers distinguished between
them by putting a little horn out on the ends of some of them. That tiny horn was called a
tittle. It is Jesus' way of saying the dotting of the i's or the crossing of the t's would not pass
away until all things were accomplished. Truly, "the scripture cannot be broken," (Jn. 10:35).

(Jerry Moffitt's address is 709 Cass, Harrisonville, MO 64701.)

EDITOR'S NOTE: With a few inches of available space left in this issue, we were tempted to
comment on Mr. Moffitt's "explanations" of the contradictions identified in the article he
replied to, especially the old "skipped-generations" theory he used to reconcile the Exodus-6
genealogy with the Bible claim of a 430-year Israelite sojourn in Egypt. Not wanting to take
unfair advantage in a situation Mr. Moffitt could not immediately react to, we proposed
another exchange on the subject for our spring issue that will be mailed in late March, and he
has accepted the invitation. Farrell Till will show that, contrary to the Archer-Haley-Arndt-
Moffitt theory of skipped generations, a proper interpretation of the Bible proves that the
writer of the Exodus-6 genealogy intended for readers to see it as a complete family tree from
Israel (Jacob) through Phinehas, the grandson of Aaron. Mr. Moffitt will respond. We suggest
that readers keep this issue of TSR available for reference.

In his article, Mr. Moffitt challenged us to continue to invite him and others to "review every
so-called discrepancy, absurdity, and contradiction he (Till) imagines." We hope this second
invitation will convince him that we intend to do just that.

The Flat-Earth Belief of Bible Writers

Volume 1990 - 2002 Issue


Page 12 of 2049
Skeptical Review Edited by Farrell Till
Adrian Swindler
All Christian sects recognize the Bible as the primary source of revelation. This compiled
material was allegedly inspired by God and written by chosen authors to reveal him and his
will to man. The Bible, then, is the foundation of the Christian religion. To Christian
fundamentalists who believe in verbal inspiration, the Bible is an infallible foundation. They
claim that "the Holy Spirit so dominated and guided the minds and pens of those who wrote
(the Bible) as to make their writings free from mistakes of any and all kinds, whether it be
mistakes of history or chronology or botany or biology or astronomy, or mistakes as to moral
and spiritual truth pertaining to God or man, in time or eternity," (Wilbur F. Tillett, "The
Divine Elements in the Bible," The Abingdon Bible Commentary).

Despite the obvious sincerity of those who so view the Bible, the inerrancy doctrine has no
basis in fact. That the Bible contains mistakes in every area mentioned by Mr. Till is a truth
widely recognized by reputable Bible scholars. One of the most consistent scientific errors
that Bible writers made concerned their misconception of the earth's shape. In Psalm 24:2, for
example, it was said that "the world and all that is in it belong to the Lord; the earth and all
who live on it are his. He built it on the deep waters beneath the earth and laid its foundations
in the ocean depths," (GNB).

This passage and others like it in the Bible make no sense until they are interpreted in terms of
the ancient Hebrew conception of the world as represented in the graphic illustrations on the
following page that were published in the New American Bible and The Interpreter's
Dictionary of the Bible. (Similar ones appear in other Bible dictionaries.) If you will study the
graphics and then read the above quotation again, the psalmist's meaning will become quite
clear. He thought the earth rested on foundations or pillars that God had set in the ocean
depths. Needless to say, modern science knows better.

Here are just a few of the many other passages that prove Bible writers were ignorant of
Earth's spherical shape:

Daniel 4:7-8, "I saw a tree of great height at the center of the world. It was large and strong,
with its top touching the heavens, and it could be seen from the ends of the earth." This was
allegedly an inspired dream, yet it conveys a flat-earth concept, because no matter how tall a
tree would be, people on the other side of a spherical earth could not see it.

Matthew 4:8, "The devil took him (Jesus) to a very high mountain and displayed before him
all the kingdoms of the world in their magnificence...." The only plausible reason for the
"very high mountain" was that the altitude would make it possible to see to the ends of the
earth. Only on a flat earth would this be remotely possible, so the New Testament writers
were as ignorant as the Old.

In Genesis 11:4, the people wanted to build a tower up to heaven. If you look at the graphics
above, you can see their concept of heavenly bodies under the dome, not all that far away.
Presumably, the Lord was afraid they would be able to accomplish their plan, so he caused
them to speak various languages. This, of course, is not the reason people speak different
languages, but nothing is too fantastic for the ignorant to believe.

Volume 1990 - 2002 Issue


Page 13 of 2049
Skeptical Review Edited by Farrell Till
The following references show that Bible writers thought there was water above a solid dome
with floodgates (look at the graphics again) that could be opened to make it rain:

Job 38:22, "Have you entered the storehouse of the snow, and seen the treasury of the hail?"
Look at items two and three in the graphic from the Interpreter's Dictionary, and the intended
meaning of this statement becomes very clear.

Psalm 104:3, 13, "You stretch the heavens out like a tent, you build your palace on the waters
above.... You water the mountains from your palace." Here God dwells in a palace above the
waters over the firmament or dome. To water the mountains, he opens the floodgates. Quite
unscientific!

Genesis 1:6-7, "Let there be a dome to divide the water and to keep it in two separate places...
and it was done. So Godmade a dome, and it separated the water under it from the water
above it." So the NAB and The Interpreter's Dictionary of the Bible are quite correct in their
graphic representations of what the Bible writers believed and taught. How many of you
readers believe the earth is flat? The Bible teaches it is!

Christian fundamentalists have used various scriptures to try to prove that Bible writers knew
the earth was round. Since I have already shown that these writers thought the earth is flat, if
some verses actually do teach that it is round, then there is a contradiction in the Bible and the
fundamentalists lose anyway.

Job 38:13-14 is sometimes quoted as a round-earth text: "Hast thou commanded the morning
since thy days; and caused the dayspring to know his place; that it might take hold of the ends
of the earth, that the wicked might be shaken out of it? It is turned as clay to the seal; and they
stand as a garment." Claim is made that the statement about the earth "turn(ing) as clay to the
seal" was a reference to the earth's rotation, but this passage has nothing to do with
movement. The word used was haphak, which meant "to convert, to change, or to make
clear." It is the same word that was used in Exodus 7 in reference to Aaron's rod turning into a
serpent and the waters of Egypt turning to blood, so rather than the word meaning turning in
the sense of movement, it meant turning in the sense of changing. The GNB clarifies the
meaning in Job 38:14: "Daylight makes the hills and valleys stand out like the folds of a
garment, clear as the imprint of a seal on clay." So, far from teaching the revolution of the
earth, this was merely a reference to the effects of sunlight in the morning. Notice also that
the KJV refers here to "the ends of earth." This would indicate a flat earth, since there are no
ends to a globe.

Job 26:7 has also been cited as proof that the writer of this book knew that the earth was a
sphere: "He stretcheth out the north over the empty place and hangeth the earth upon
nothing." An NAB footnote at this verse says, "The North: used here as a synonym for the
firmament, cf. Is. 14:13," (emphasis inserted). Thus, we read, "He stretches out the dome
(firmament) over the empty space." In other words, the dome was unsupported in the middle.
The reference in Isaiah 14:13 says, "You (King of Babylon) were determined to climb up to
heaven and place your throne above the highest stars (see the graphics). You thought you
would sit like a king on that mountain in the north where the gods assemble." The "north" was

Volume 1990 - 2002 Issue


Page 14 of 2049
Skeptical Review Edited by Farrell Till
indeed used as a synonym for the heavens or firmament, so the passage was actually speaking
of a "mountain in the heavens where the gods assemble."

"He... hangeth the earth upon nothing" simply expressed a Hebrew belief that the flat earth,
although supported by pillars, did not rest on the back of Atlas or a turtle or an elephant, as
their pagan neighbors believed. In this Job was right but not because he was inspired;
otherwise, he wouldn't have said in the same context, "The pillars of the heavens tremble (see
the graphics) and are stunned at his thunderous rebuke," (26:11). He thought the thunder was
God's voice!

Fundamentalists use Isaiah 40:22 to argue that Earth's rotundity was known to the writer: "It
is he (God) that sitteth upon the circle of the earth, and the inhabitants thereof are as
grasshoppers; that stretcheth out the heavens as a curtain, and spreadeth them out as a tent to
dwell in." They misunderstand the first half of the verse, which is clarified by the placement
of "God's throne" in the NAB graphic, and they avoid the second half. The NAB gives us a
proper translation of the verse: "He sits enthroned above the vault (dome) of the earth.... He
stretches out the heavens like a veil, spreads them out like a tent to dwell in." See the graphic
illustrations again and check the Hebrew concept of firmament as explained in Eerdmans and
other reliable Bible dictionaries.

The Hebrews were inspired by nothing more than their political and religious motivations.
Thus, being ignorant of scientific facts, they thought the earth was flat, that sick people were
possessed by demons, and that essentially everything was caused by either gods or demons.
Unfortunately, many people are still just as ignorant.

(Adrian Swindler's address is P. O. Box 695, Elmwood, IL 61529.)

The Unfinished Debate


On May 10, 1988, James H. Laws, Jr., a Church-of-Christ preacher and a professor of
apologetics at Tennessee Bible College, challenged Farrell Till to a written debate on the
inerrancy issue. His letter proposed the following propositions:
RESOLVED: I know that the Bible is the plenarily inspired word of God to man.

RESOLVED: I know that the Bible is not the plenarily inspired word of God.

He concluded the letter with a solemn pledge that he would not assume this responsibility in a
"light hearted way." "I will put everything I have into such a discussion (knowledge, ability,
sincerity), " he promised. After preliminary discussions in which Till questioned the phrasing
of the propositions, objecting especially to presumptuous implications in the word "know," an
agreement was signed to use Mr. Laws' propositions essentially as he had submitted them.
The only change was the substitution of verbally for plenarily.

Volume 1990 - 2002 Issue


Page 15 of 2049
Skeptical Review Edited by Farrell Till
The debate began with Mr. Laws defending his proposition. Three manuscript exchanges
were made, and then there was a three-month silence on Mr. Laws' end. After two letters of
inquiry were directed to him, Laws wrote on May 17, 1989, just one year and one week after
he had issued his challenge, to say that he was quitting the debate and would communicate
with Till no further about the matter. Laws refused to accept a letter that Till mailed him on
May 20th in response to the unilateral withdrawal.

Why would Mr. Laws quit a debate that he himself had proposed? Ostensibly, he withdrew
because of increased responsibilities that came with his acceptance of a new position as
minister of the Getwell Church of Christ in Memphis, but we suspect a different reason. By
the end of Till's third rebuttal, the central argument on which Laws had based the defense of
his proposition had been so demolished that he had nowhere to turn, so he chose to withdraw.

We urge readers who might be inclined to accuse us of biased judgment to let the debate
speak for itself. Despite almost frantic efforts by Mr. Laws to block their publication,
Skepticism, Inc., will publish the three exchanges that were made. Printed in the two-column
format you are now reading, this fifty-page volume will be available February 1st for $3.00, a
cost that will barely cover publishing and mailing. Orders should be addressed to P. O. Box
617, Canton, IL 61520.

Other Inerrancy Debates

Farrell Till now has written debates in progress with Jerry Moffitt and Bill Jackson, both
prominent ministers in the Church of Christ. An oral debate between Till and Moffitt will be
held on August 13, 14, 16, and 17, at the 39th Street Church of Christ, 15331 East 39th Street,
Independence, Missouri. Propositions in all the debates are essentially the same. Moffitt's and
Jackson's position is that the Bible is verbally inspired of God and therefore completely
inerrant; Till contends that contradictions and other errors in the Bible disprove its claim to
divine inspiration. We encourage all readers who can to attend the oral debate. The written
debates will be published when finished.

Volume 1990 - 2002 Issue


Page 16 of 2049
Skeptical Review Edited by Farrell Till

The Skeptical Review


Volume One, Number Two
March/April/May 1990
Farrell Till, editor

The Skeptical Review: 1990: Number Two

We Come Not to Destroy the Bible


Till discusses reader reaction to the first issue of TSR and explains that he does not
want to destroy the Bible, but he wants people to understand it.
Holes in the Two-Amrams Theory
Till responds to the Two-Amrams Theory presented by Jerry Moffitt in the previous
issue of TSR.
Plugging "Holes" in the Two-Amrams Theory
Fundamentalist pastor Jerry Moffitt responds to to the above article by Farrell Till.
A Perfect Work of Harmony?
Till discusses "the divergent views of Jehu's slaughter of the royal family of Israel" in
II Kings 9; 10:1-30 and Hosea 1:4, as an example of the inconsistencies in the Bible.

We Come Not to Destroy the Bible


Reaction to our first issue was mainly what we had expected. We received a few letters of
praise, but most of the mail was very negative. A young lady in Louisiana wrote and asked us
to take her name off our mailing list. "As far as I'm concerned," she said, "it (TSR) is trash."
We could only assume that she thought Jerry Moffitt's rebuttal article was trash too, and that's
too bad. We thought that, for the position it defended, it was a well written article.

Volume 1990 - 2002 Issue


Page 17 of 2049
Skeptical Review Edited by Farrell Till
Many copies were returned to us with "refused" stamped or written above the address labels,
and we have to wonder about the objectivity of people who aren't even willing to consider
opinions that disagree with theirs. Do they seriously believe this is the way to discover truth?
Two of those who refused to accept their copies are actively involved in publishing The
Spiritual Sword, a guardian-of-the-faith paper sponsored by the Getwell Church of Christ in
Memphis. If they had mailed us a complimentary copy of their paper and we had refused to
accept it, they would undoubtedly accuse us of being terribly close-minded, yet they probably
see their own actions as some kind of bold stand for the truth. Incidentally, we are not really
expecting to receive a complimentary copy of The Spiritual Sword, because we have twice
sent to its editors subscription checks that have never been acknowledged or cashed. If we
can't even buy it, we aren't very likely to get it free.

Some of the most abusive letters came from relatives and former friends and associates of our
missionary days. Many of these we have not bothered to answer, because we can see nothing
to be gained from trying to reason with people determined to base friendship on matters of
individual conscience. When Peter and the other apostles were charged "not to teach in this
name," they said, "We must obey God rather than men" (Acts 5:28-29), so all that we ask of
former friends and associates is the same kind of consideration: the right to respect our
consciences rather than follow what others think is the correct course for us. After all, what
good would it do us to fill a pew or a pulpit if we just cannot believe what we would hear or
be required to preach?

We have often been accused of wanting to destroy the Bible, but the charge is simply not true.
We have no desire to destroy the Bible; we just want people to understand it. In the sermon on
the mount (if it occurred as recorded), Jesus tried to reassure those who thought he may have
wanted to destroy the law: "Think not that I came to destroy the law or the prophets; I came
not to destroy, but to fulfill," (Mat. 5:17). In the same way, we can honestly say that we are
not publishing this journal or engaging in public debates with the hope that our activities will
destroy the Bible. If any person should seriously advocate that Bibles be collected and burned,
we would raise our voices in opposition to the proposal as loudly as any inerrancy advocate. It
would be the height of folly to destroy a book that has probably had more influence on
western civilization than any other single document.

A better understanding of the Bible is what we hope to accomplish. The problem is that the
average "God-fearing" person doesn't really know much about the Bible; he just knows that
he is supposed to believe it is the inspired, inerrant word of God. At home, his Bible is lying
unopened, collecting dust. If our quarterly journal or any of the debates we participate in
result in just one person like this studying the Bible in a serious, intelligent way to see if the
inerrancy doctrine can be sustained, we will consider our work worth the time and effort we
are putting into it. If after such a study, anyone chooses to continue believing in the inerrancy
of the Bible, he will at least have our respect, but what we can't respect are the many who
believe in Bible inerrancy without knowing why they believe it. They just know that they are
supposed to believe it. This makes them no better than a Moslem who believes in the
inspiration of the Koran, because... well, just because he is supposed to believe it.

Volume 1990 - 2002 Issue


Page 18 of 2049
Skeptical Review Edited by Farrell Till

Holes in the Two-Amrams Theory


Farrell Till
In our first issue of The Skeptical Review, I exchanged arguments with Jerry Moffitt on the
general question of Bible inerrancy. Much of what we said focused on the problem of
reconciling Exodus 12:40, which claims a 430-year Israelite sojourn in Egypt, with Exodus
6:16-20 in which a genealogy gives at least a surface impression that Moses and Aaron were
only three generations removed from Levi, whose family had come into Egypt with Jacob and
Levi's brothers, (Gen. 46:8-11). My position was that a 430-year span would cover many
more than just three generations. To this, Moffitt said, "We agree with that, if there were only
three generations...." (winter issue, p. 8). He went on, of course, to argue that many more
generations of Israelites, probably as many as nine or ten, had actually lived in Egypt during
the period of bondage. He justified this claim by what is sometimes called the skipped-
generation theory. He cited a few examples of where Bible writers had obviously skipped
generations in genealogical listings and from there went on to argue that this was what had
been done in the Exodus-6 genealogy.

Before examining Mr. Moffitt's theory, I first want to thank him for admitting that he agrees
with my conclusion about the length of the sojourn, if there were only three generations,
because he has greatly simplified my task. When I show, as I will, that the Exodus-6
genealogy was presented as a complete Aaronic family tree from Levi through Phinehas, the
grandson of Aaron, Mr. Moffitt will then have to concede that there is a discrepancy in the
"verbally inspired" Bible text.

The crux of Moffitt's argument hinges on Numbers 3:27-28 where a census of the male
Kohathites (so named because they had descended through Levi's son Kohath) put their
number at 8,600. These were in turn divided into Amramites, Izharites, Hebronites, and
Uzzielites, because Kohath, as indicated in Exodus 6:18, had had four sons named Amram,
Izhar, Hebron, and Uzziel. The argument of Mr. Moffitt and the sources he quoted is that the
Amram who was the father of Aaron and Moses could not have been the Amram who was
Kohath's son; otherwise, this would suggest (on the basis of an equal division of the 8,600
Kohathite males into their four clans) that Aaron and Moses had had "around 2,150 brothers,"
(p. 8). "That should be hard," Moffitt said, "for even a dedicated skeptic like Farrell Till to
swallow." For this reason, Moffitt concludes that there had to have been at least two Amrams,
one who was Kohath's son and head of the Amramites and another who fathered Aaron and
Moses by Jochebed, (Ex. 6:20). The writer of the Exodus-6 genealogy had simply "skipped"
some generations between the two Amrams, so the theory goes, and this has caused some
people to wrongly conclude that the Amram who was Moses' father was the same Amram
who was Kohath's son.

It all comes out sounding very pat, but it's a theory with more holes in it than a sieve. For one
thing, unless Moffitt has been living on another planet, he has to know that a major argument
against the Bible inerrancy doctrine is based on the outrageous exaggeration of census figures
in the books of Exodus and Numbers. Exodus 12:37 states that when the Israelites left Egypt
the number of men on foot (not counting women and children) was 600 thousand! When a

Volume 1990 - 2002 Issue


Page 19 of 2049
Skeptical Review Edited by Farrell Till
census was taken in the wilderness (Num. 1:46), it claimed the men of military age (20 years
old and up) numbered 603,550! If we assume an equal number of women in this age group--
and I guess I can do this if Moffitt can assume an equal division of the Kohathites within their
four clans--this would mean the adult population older than 20 numbered around 1,200,000.
Then with the children of both sexes under 20 added on, there would have been a total
population of two and a half to three million! (Since the Israelites had been breeding like flies
in Egypt, we could reasonably assume that the younger, under-the-age-of-twenty group would
have surely represented an equal, if not larger, proportion of the total population.) Regardless,
the fact is that there were an awful lot of people in the exodus, according to the Bible. There
were so many, in fact, that one wonders why, given the relatively small size of the Sinai
peninsula, a few of them at least didn't accidentally stumble onto the promised land before the
end of the forty-year period of wandering, especially since they must have also driven along
with them herds of sheep and cattle numbering in the millions in order to have had enough
lambs to meet the requirements of forty Passover commemorations and to feed the tabernacle
altar the perpetual sacrifices (for three million people) described in Leviticus and Numbers.

I'm having a little fun at Mr. Moffitt's expense, of course, but only to make a serious point. He
would be hard pressed to find a reputable Bible scholar anywhere who will say that the
population figures in Exodus and Numbers were anywhere close to being realistically
accurate. Yet he wants to use one set of those figures as the sole basis for arguing that the
Amram who was Moses' father wasn't the same Amram who was Kohath's son. He said that
even a dedicated skeptic like me would have a hard time swallowing the possibility that
Moses and Aaron could have had 2,150 brothers, but his reasoning here was a little fuzzy
around the edges. At the time of the exodus, Aaron had already had four sons (Ex. 6:23), and
Eleazar, one of those sons, had had at least one son, Phinehas (Ex. 6:25). Moses had also had
at least two sons, (Ex. 18:3). These would have all been Amramites, so since the Israelites, as
we have already noted, were breeding like flies at this time, it doesn't necessarily follow that
an equal distribution of the 8,600 Kohathites into four clans would have meant that Moses and
Aaron had "around 2,150 brothers." Many of them could have been their sons and grandsons
or nephews and great-nephews through their sister Miriam. Since Moffitt established in his
article that probably as many as nine or ten generations of Israelites had lived in Egypt, we
could even imagine that many of these 2,150 Amramite males were their grandsons or great-
grandsons. This theory would certainly fit into the Exodus-6 genealogy as Moffitt sees it. The
writer had just "skipped" some of the generations between Aaron and Phinehas. It might well
be, for example, that Eleazar wasn't actually Aaron's son; he could have been his grandson.
And maybe Phinehas wasn't Eleazar's son; perhaps he was his great-grandson. This is all very
compatible with the skipped-generation theory and makes it quite possible that the 2,150
Amramites had all descended from the same Amram who was the father of Moses.

But I'm not going to swap far-fetched, how-it-could-have-been scenarios with Mr. Moffitt.
That's a game inerrancy believers have to play. I'm going to return Moffitt's favor and say that
I agree with him. If he can establish the reliability of the census figures in Numbers 3:27-28,
then I will agree that the Amram who was Moses' father was not the same Amram for whom
the Amramites were named. Until he can do that, however, he shouldn't expect us to be too
impressed with an argument that relies on one probable Bible discrepancy to explain another
one. The exodus census numbers have long been suspect in scholarly circles, and, quite
frankly, I would find it much easier to swallow the possibility that Moses and Aaron had had

Volume 1990 - 2002 Issue


Page 20 of 2049
Skeptical Review Edited by Farrell Till
2,150 brothers than that two to three million Israelites had wandered around for forty years in
the Sinai desert with immense herds of sheep and cattle.

Another point in Mr. Moffitt's article that we certainly don't want to overlook is his claim that
as many as nine or ten generations of Israelites had lived in Egypt. He used certain
genealogies in I Chronicles to support his claim, and if the sojourn lasted for 430 years, we
would certainly agree that nine or ten generations would be a more reliable estimate than the
four implied in the Exodus-6 genealogy. The tragedy for him, however, is that even if he is
right on this point he still loses. In Genesis 15:13-16, a prophecy about the Hebrew bondage
in Egypt was made to Abram presumably by Yahweh himself: "Know of a surety that thy
seed shall be sojourners in a land that is not theirs, and shall serve them; and they shall afflict
them four hundred years; and also that nation whom they shall serve, will I judge; and
afterward shall they come out with great substance.... And in the fourth generation they shall
come hither again...." So if Mr. Moffitt is right and nine or ten generations of Israelites did
live in Egyptian bondage, he makes his God Yahweh a false prophet. Yahweh said that the
Hebrews would come out in the fourth generation, but they really didn't make it out until the
9th or 10th. I look forward to seeing what "figurative" explanation Mr. Moffitt will devise to
explain away this problem. That's another game the inerrancy defenders play. No matter how
compelling the evidence for textual contradictions and discrepancies may be, they always
manage to come up with some "figurative" interpretation of the problem passage to show how
"it could have been" or what "it may have meant."

With all of Mr. Moffitt's quibbles out of the way, we can now look at evidence that clearly
disputes the claim that generations were skipped in the Exodus-6 genealogy. First, we should
notice that the theory of skipped generations in this genealogy is just that--a theory. It is based
on nothing but pure speculation. As Moffitt has noted, Matthew did call David the son of
Abraham, and other writers at times clearly did skip generations in expressing genealogical
relationships, but in all of these cases we know that generations were skipped because of
information provided outside the genealogical texts. As the Bibletells its story, for example,
we read about Isaac, Jacob, Judah, Boaz, Jesse, and the many other generations that Matthew
skipped in calling David the son of Abraham, but where is the Bible passage(s) that tell(s) of
the generations presumably skipped between the two Amrams of Exodus 6? Unfortunately for
Mr. Moffitt, they simply do not exist. Every time the Levitical genealogies of either Moses or
Aaron are listed in the Bible, they always show the same order: Levi, Kohath, Amram, and
Moses and/or Aaron, ( Ex. 6:16-20; Num. 26:57-59; I Chron. 6:1-3; 23:6-13). As much as the
Bible emphasized genealogies, it seems strange, to say the least, that a complete genealogy of
two of its most important figures--Aaron and Moses--is to be found nowhere in the sacred
text. But this is the conclusion we are driven to if we accept the skipped-generation theory.

Furthermore, this theory ignores a clear intention of the Exodus-6 genealogy, which the writer
began as if he meant to give a complete genealogy of all of Jacob's sons: "These are the heads
of their fathers' houses. The sons of Reuben the first-born of Israel: Hanoch, and Pallu,
Hezron, and Carmi.... And the sons of Simeon: Jemuel, and Jamin, and Ohad, and Jachin, and
Zohar, and Shaul the son of a Canaanitish woman...." (6:14-15). But after Levi, Jacob's
(Israel's) third son through whom the Levitical priests had descended, the writer stopped
listing the Jacobite sons; thereafter, nothing was mentioned of Judah, the son through whom
Jesus descended, or Gad or Asher, etc., etc., etc. Everything suddenly focused on Levi and his

Volume 1990 - 2002 Issue


Page 21 of 2049
Skeptical Review Edited by Farrell Till
sons, and soon thereafter the focus became Aaron and his sons. Even the descendants of
Moses were dropped. Aaron and sons assumed center stage.

Clearly the writer of this genealogy was trying to make what he thought was an important
point about Aaron. What could that have been except to establish that Aaron was clearly a
direct descendant of Levi? To understand why the writer of this genealogy would have had
such an interest, one must know about the struggle that the Aaronic branch of the Levites
waged with other family branches through much of Israel's history to win recognition as the
only divinely recognized Levitical priests. To show that such a struggle did happen would
take more space than I have left, but it is a position I am prepared to prove if Mr. Moffitt or
anyone else wants to challenge it. In the light of such a struggle, the writer's purpose in
Exodus 6 becomes clear. He wanted to establish that the first priests, the ones who had served
the Israelites in the wilderness, had descended from Levi through Aaron. Most reputable
Bible scholars, in fact, believe that the writer of this passage was himself an Aaronic priest. If
that is so, then he had an important point to make--important to him at least--and he couldn't
very well have done it by skipping generations in this genealogy.

He went to elaborate extremes, in fact, to make his point. At the end of the genealogy, he
wrote, "These are the heads of the fathers' houses of the Levites according to their families"
(v:25), so clearly his intention was to present a family tree from Levi through Phinehas and
not a general genealogy of Jacob's family as it appeared when he began it. But why the focus
on Aaron and his sons? Why weren't the other Levite families important enough to extend as
far as the writer went with Aaron's family? He surely had a purpose, and I believe that
purpose has been best explained by scholars who have identified the writer of this genealogy
as a priest who had himself descended from Aaron.

The writer's hand was further tipped as he continued his conclusion of the genealogy: "These
are that Aaron and Moses, to whom Jehovah (Yahweh) said, Bring out the children of Israel
from the land of Egypt according to their hosts. These are they that spake to Pharaoh king of
Egypt, to bring out the children of Israel from Egypt: these are that Moses and Aaron,"
(vv:26-27). Somehow, the writer felt compelled to drive home the fact that the Aaron and
Moses in this genealogy were the very Aaron and Moses famous for having led the Hebrews
out of Egypt. From cover to cover, the Bible mentions no other Aaron and Moses except
these, so why did the writer go to such extremes to make it clear what Aaron and Moses he
meant? Clearly, he wanted it understood that the first Levitical priests to serve Yahweh's
people were descended from Levi through the same Aaron who was Moses' brother. He had a
vested interest in selling that point to his readers.

This writer's extreme care, however, raises another question. Is it reasonable to believe that
someone as redundant as this writer was in identifying which Moses and Aaron he meant
would list one Amram in a genealogy, skip a generation or two (or three), and then resume
listing the generations with a second Amram and not tell his readers the two weren't the same
person! It stretches credibility too far to imagine it. Besides, we have another case where Mr.
Moffitt loses even if he is right. Anyone who knows anything at all about effective writing
will agree that if there really were two different Amrams, then whoever wrote this genealogy
used extremely poor transition, for in the short space of just two verses, he went from one
Amram to another person of the same name without letting his readers know the change was

Volume 1990 - 2002 Issue


Page 22 of 2049
Skeptical Review Edited by Farrell Till
being made. Thus, if Moffitt could actually prove this is not a case of factual error, it would
still be a serious compositional error. Shouldn't an omniscient God know how to direct his
inspired writers to use sound writing practices? But in this case he didn't--if Moffitt is right.

So far all my evidence has been circumstantial. None of it actually proves that only one
Amram was intended in the genealogy, but now that is about to change. The genealogy says
that Kohath had four sons: Amram, Izhar, Hebron, and Uzziel, (v:18). If I am right in saying
that the Amram in this verse was the same Amram identified in verse 20 as the father of
Aaron and Moses, then Izhar, Hebron, and Uzziel were the uncles of Aaron and Moses. Is
there any proof that they were? Unfortunately for the skipped-generation theory, there is. Mr.
Moffitt is no doubt familiar with the story in Leviticus 10:1-2, where Yahweh incinerated
Nadab and Abihu, the priestly sons of Aaron, for using "strange fire" in their censers. I
wouldn't even try to estimate how many you-better-toe-the-line sermons by Church-of-Christ
preachers have been based on this story. Perhaps Mr. Moffitt has preached a few of them
himself. At any rate, after the fire had "devoured them," we read this: "And Moses called
Mishael and Elzaphan, the sons of Uzziel the uncle of Aaron, and said unto them, Draw near,
carry your brethren from before the sanctuary out of the camp," (v:4). Here it plainly says that
Aaron had an uncle named Uzziel.

Was this the same Uzziel as the one in Exodus 6:18 who was "the first Amram's" brother?
Notice that Aaron's Uncle Uzziel had two sons named Mishael and Elzaphan (Lev. 10:4) and
that the Uzziel in the Exodus-6 genealogy (brother of Amram I) had three sons: Mishael,
Elzaphan, and Sithri, (v:22). What will Mr. Moffitt say about this? Will he now come forth
with a skipped-generation, two-Uzziels theory?

Furthermore, we have the fact that Exodus 6:20 states that Amram, the father of Aaron and
Moses, "took him Jochebed his father's sister to wife; and she bare him Aaron and Moses."
Now if Amram's wife Jochebed was his father's sister and if this Amram who married
Jochebed was the same Amram who was Kohath's son, then Jochebed would have been Levi's
daughter, because Kohath was Levi's son. Is there anything in the Bible to indicate that
Jochebed, the mother of Aaron and Moses, was indeed Levi's daughter? In relating the
circumstances of Moses' birth, Exodus 2:1-10 says that his mother was "a daughter of Levi,"
(v:1). Mr. Moffitt will argue that she was a daughter of Levi only in the sense that she was a
descendant of Levi, and he could probably get away with this were it not for Numbers 26:57:
"And the name of Amram's wife was Jochebed, the daughter of Levi, who was born to Levi in
Egypt: and she bare unto Amram Aaron and Moses, and Miriam their sister," (ASV).

Inerrancy believers have desperately tried to deny the clear conclusion this passage leads to,
even to the point of tampering with the text. The NIV renders it like this: "The name of
Amram's wife was Jochebed, a descendant of Levi, who was born to the Levites in Egypt...."
Most versions, however, faithfully represent the Hebrew meaning as it was translated in the
ASV quoted above. Nevertheless, Bible fundamentalists still adamantly insist that Jochebed
wasn't literally Levi's daughter, that she had been "born to Levi" only in the sense that any
Levite woman of Jochebed's time had been born to Levi. Those who so argue have never been
able to explain why the passage states that Jochebed had been born to Levi in Egypt. Why
specify that it was in Egypt that she had been born to Levi? In the maze of genealogical
information in Exodus, Numbers, I Chronicles, and elsewhere, no other person of Jochebed's

Volume 1990 - 2002 Issue


Page 23 of 2049
Skeptical Review Edited by Farrell Till
time was identified as a daughter (or son) of Levi who had been born to him in Egypt. If there
was no special significance to the expression, then it would have been appropriate to say of
any woman of Jochebed's tribe and generation that she had been "born to Levi in Egypt." Yet
it was never said, except in the case of Jochebed. Surely there was a reason why.

We have already noticed the intense interest of the Exodus-6 genealogy in establishing
Aaron's descent from Levi. Scholars generally recognize this genealogy and Numbers 26:57-
59 both as parts of the P Document redacted into the Bible by an Aaronic priest. If so, that
would explain the preoccupation of both passages with establishing Aaron's descent from
Levi. He wasn't just a descendant, specifically a great-grandson, of Levi; his mother was even
a daughter (literally) of Levi, so he was Levi's grandson as well as a great-grandson. That
would have made him about as "Levitical" as anyone could claim. Possibly realizing that
some readers of his redaction would challenge the claim that Jochebed was Levi's daughter on
grounds that no daughters were credited to Levi in earlier genealogies (Gen. 46:11), the writer
took care to specify that she was Levi's daughter because she had been born to him in Egypt.

To say the least, Mr. Moffitt has his work cut out for him. To establish any kind of credibility
in his two-Amrams theory, he must repair the holes the facts in this article have shot into it. In
particular, he must explain away Aaron's uncle Uzziel and his mother Jochebed, who was said
to be the "daughter of Levi" who had been "born to Levi in Egypt." If he can't do this, by his
own admission (as noted earlier), he must agree that the Israelite sojourn in Egypt spanned
only three generations, which would have been considerably less than 430 years.

If he persists in claiming that other genealogies show a span of nine or ten generations during
the sojourn, all he will prove is that intertextual contradictions are in the Bible, and this is
what we have been arguing all along. So even when he wins he loses.

Plugging "Holes" in the Two-Amrams


Theory
Jerry Moffitt
A reply to "Holes...."

Any success, benefits, or accomplishments in any task are directly related to Him in whom we
have complete dependence. We have prayed for His providence and guidance as we approach
this opportunity to serve Him. I said earlier that Mr. Till's first article was held together with
bubble gum and kite string. In his second article I do not see how any skeptic could have done
a better job with the available evidence than Mr. Till has done. He has marshalled his
evidence with precision from the least persuasive toward the most persuasive. Yet his second
article in one way is worse than the first. It seems to me to be held together with not much
more than a strained look and static electricity. For example, when he sent his article, he said

Volume 1990 - 2002 Issue


Page 24 of 2049
Skeptical Review Edited by Farrell Till
it was composed of 3,650 words. He said I could use an equal amount in reviewing it. Out of
his 3,650 words, I want you to inspect 78 of the key ones. Notice these pivotal words by
which the article is sewed together:

"If we assume... and I guess I can assume this... we could reasonably assume that... would
have surely... one wonders why... doesn't necessarily follow... many of them could have
been... we could even imagine... this theory would certainly fit... it might well be... and
maybe... and makes it quite possible... I would find it much easier to swallow... believe that
the writer of this passage... if that is so... is it reasonable to believe... if so... possibly
realizing...."
Now does that sound as if someone is actually proving anything? Does it have the flavor of
logic and the scientific method? It's a pretty "iffy" document for one who claims the Bible is a
"veritable maze of irreconcilable contradictions." Surely when it comes to proof, facts, truth
and error, "assumption don't feed the bulldog."

Let me just go through Mr. Till's article point by point, treating quibbles or major points as
they arise. Remember, I showed that Mr. Till's position required Moses to have 2,150
brothers. Mr. Till mumbled something about the "outrageous exaggeration" of census figures
in the books of Numbers and Exodus. Then he said he couldn't believe two or three million
people wandered about for 40 years in the Sinai peninsula. Why didn't a few of them stumble
accidentally into the promised land, given the small size of the peninsula, he wonders. Now
isn't that foolish? Sinai is over 20,000 square miles and easily double the size of the promised
land. It is about the size of West Virginia, twice the size of Maryland, four times the size of
Connecticut, and half the size of Ohio. New York City alone, in 1989, had more than twice
the population of the whole nation Moses brought out. The Israelites were organized into a
close mass and miraculously sustained with manna, flesh, and water.

Using a little "outrageous exaggeration" of his own, he says I would be hard pressed to find
one reputable Bible scholar who says the figures in Exodus and Numbers are close to being
realistically accurate. Unless the definition of "reputable Bible scholar" is "one who agrees
with Farrell Till's assertions," I find many, and that fact doesn't help Mr. Till's credibility.
Edersheim says the population was upwards of two million (Old Testament Bible History,
Alfred Edersheim, II, 45). Further, The International Standard Bible Encyclopedia (ISBE)
defends both 600,000 for warriors and 2.5 million for the whole congregation. It successfully
argues that seventy souls could multiply into 2.5 million in 215 years, much less 430 years
(IV, 2166). Next, Halley's Bible Handbook, so popular that it has gone through 61 printings,
puts the figure at 3,000,000 (pp. 146-147). Then The Bible Commentary, editor F. C. Cook,
takes the numbers as authentic. It points out that the camp was arranged with a military
precision and would cover an area of around three square miles (I, 660). Also, Commentary
on the Old Testament in Ten Volumes by C. F. Keil and F. Delitzsch says, "Modern critics
have taken offense at these numbers, though without sufficient reasons," (III, 5). We could go
on, but that's enough.

Mr. Till says if I could establish the reliability of the figures in Numbers 3:27-28 he would
agree there were two Amrams as per Archer's Book. Oh? Numbers 3: 27-28 says the
Kohathites in Moses' day numbered 8,600. Is 8,600 reliable? Could they have gotten to be that
number in nine or ten generations. Let's see. Let's say Kohath had only three sons instead of

Volume 1990 - 2002 Issue


Page 25 of 2049
Skeptical Review Edited by Farrell Till
four, and each of those sons had only three sons. A geometric sequence or progression of only
eight generations would be 30 + 31 + 32 + 33 + 34 + 35 + 36 + 37 + 38 = 9,841. This number
is more than enough and consistent with I Chronicles, where I showed several individuals
contemporary with Moses were in the ninth or tenth generation from those who entered
Egypt. And, fatal to Mr. Till, it is consistent with what we have been arguing for Exodus 6. So
the figures are very reliable, even if one counts only the last two or three generations. But we
will not demand that Mr. Till accept Archer's "Two-Amrams Theory" unless he wants to.
Let's move on. Alas, things get even worse.

Remember, if Amram were responsible for one fourth of the 8,600 Kohathites and that same
Amram was the father of Moses and Aaron, then Moses and Aaron would have had around
2,150 brothers, according to Mr. Till's position. But Mr. Till responded, saying that maybe
they were sons and grandsons of Aaron and Moses, and through their sister Miriam they could
have been nephews and great-nephews. Let's say the 8,600 is composed also of sons and
grandsons. Let's help him by supposing that Amram and each of his three brothers had four
sons (which they didn't; I'm being generous to Mr. Till's theory) and each of those four had
four sons and so on. How many males would we arrive at by the time of the grandsons? There
would be the generations of Kohath, Amram, Aaron, Eleazar, and Phinehas. Notice: 40 + 41 +
42 + 43 + 44 = 341. That's 8,259 short, a far cry from 8,600!

Well, what about the nephews and great-nephews? First, if Miriam married a descendant of
Kohath, the children would already have been counted as one of the sons descended from
Kohath. Second, if she married outside of Kohath, Merari for example, her children would not
have been Kohathites. Mr. Till is just not thinking.

But all this led Mr. Till to believe that maybe there are missing links in the genealogy. He is
forced to think this way because of the problem of 2,150 brothers of Moses. So he says maybe
the writer skipped some generations between Aaron and Eleazer and between Eleazar and
Phinehas. Could he be right that the skips are between Aaron and Eleazar and between
Eleazar and Phinehas? Notice Numbers 25:11: "Phinehas, the son of Eleazar, the son of Aaron
the priest...." This clearly says Phinehas was the son of Eleazar and Eleazar was the son of
Aaron. See also Ex. 28:1; Num. 3:4; Lev. 10:16; Num. 4:16; Num. 20:26; Jud. 20:28 Josh.
24:33; and Num. 25:11. But Mr. Till will probably point out that "son" or "daughter" can
mean "descendant," and he would be right. But in this case he would be wrong. Notice two
passages:

And Aaron took him Elisheba, the daughter of Amminadab, the sister of Nashon, to wife; and
she bare him Nadab and Abihu, Eleazar and Ithamar, (Ex. 6:23).

And Eleazar Aaron's son took him one of the daughters of Putiel to wife; and she bare him
Phinehas, (Ex. 6:25).

It is hard to figure out how Eleazar could have been born to the wife of Aaron and yet be
Aaron's grandson. Similarly, it is painful for even a dedicated enemy of the Bible to swallow
that Phinehas was born to the wife of Eleazar but then was Eleazar's grandson. These are the
kinds of things that give skeptics brief moments of religion that they have to bravely fight
down.

Volume 1990 - 2002 Issue


Page 26 of 2049
Skeptical Review Edited by Farrell Till
But now, let's locate the missing links somewhere else. Let's put them between Kohath and
the four: Amram, Izhar, Hebron, and Uzziel. But again, Exodus 6:18 calls them all sons of
Kohath. True, but we have said "son" or "daughter" can be used in the sense of "descendant."
Notice: "of the sons of Kohath, Uriel the chief, and brethren a hundred and twenty," (I Chron.
15:5). Uriel was not the literal son of Kohath. (See also verses 6, 7, 8, 10; I Chron. 23:20;
24:24.) Too, Shubael is called a son of Amram, although Shubael lived in David's day, (I
Chron. 24:20). Similarly, Shebuel is called "the son of Gershom, the son of Moses," (I Chron.
26:24). That was way up in David's time. "Sons" is copiously used in the sense of
"descendant" throughout. See I Chronicles 23:43; II Chronicles 29:14 I Chronicles 2:42-43.

We can see how a man's four literal sons could, because of their relationship with Moses and
Aaron, have divided all Kohath descendants among them. Yet, Uzziel, Amram, Izhar, and
Hebron did not have to be literal brothers but only descendants of Kohath and near kinsmen to
each other. This is a more normal division. But isn't Uzziel called the uncle of Aaron? Yes.
Would not that make him the literal brother of Amram? No! The word in Hebrew for "uncle"
is doud. It is used, according to Theological Wordbook of the Old Testament, around 58 times.
In the King James version, it is translated 38 times as "beloved," eight times "love," and 17
times "uncle," (I, 184). Too, The New Brown-Driver-Briggs-Gesenius Hebrew and English
Lexicon says it can mean "beloved one," "friend," or "kinsman," (p. 187). In Amos 6:10, the
ASV gives a footnote for it as "kinsman." So Mr. Till's line of argumentation is becoming
dreadfully becalmed. Let's continue on. Let's easily handle his Genesis argument.

God said, "Know of a surety that thy seed shall be sojourners in a land that is not theirs, and
shall serve them; and they shall afflict them four hundred years; and also that nation whom
they shall serve, will I judge; and afterward shall they come out with great substance.... And
in the fourth generation they shall come hither again...." Mr. Till points out that they were to
come out of bondage in the fourth generation, not the eighth, ninth, or tenth. Mr. Till
underlined "and in the fourth generation." I underlined "four hundred years." When you put
them together, you can see that in an age when men lived well over one hundred years, God
was using "generation" as equivalent to "century." Four generations are four centuries. The
context clearly argues "generations" were not numbered by counting the number of people in
a lineage line but by counting centuries. Notice what the Theological Wordbook of the Old
Testament says generation can mean: "The circle of a man's lifetime, from birth to death. This
is the apparent meaning at Genesis 15:16 where four generations cover an epoch of 400 years
(cf. Gen. 15:13)," (III, 186). This reference quotes Keil in his commentary, "In the times of
the patriarchs it was reckoned at a hundred years," (216). It adds, "So among the Romans the
word seculum originally signified an age or generation of men and was later transferred to
denote a century." That was the way God was using it, as a century, as the context proves. Mr.
Till claims they were not in Egypt 400 years, but he didn't discuss the "four hundred years" of
Genesis 15:13, if he noticed them, although he quoted the passage. Regardless, his Genesis
argument just went belly up.

Then Mr. Till admitted that there are skipped generations in the Bible, but he said that there
has to be a Bible passage that says they were skipped. Well, if by God's grace I enter heaven, I
will ask Moses if Moses was aware of Mr. Till's new rule of Hermeneutics. In the meantime, I
should point out that Genesis 15:13-16 (400 years) and the genealogies of I Chronicles, which
show 8 to 10 generations, and Numbers 3:27-28 (Moses having to have 2,150 brothers if there

Volume 1990 - 2002 Issue


Page 27 of 2049
Skeptical Review Edited by Farrell Till
are no skipped generations) all clearly show that some generations were elided in Exodus 6
for the sake of brevity. Such eliding in genealogy is biblically common.

Next, Mr. Till sees something astounding in the text. There seems to be a hidden agenda.
Everything focuses on Levi and his sons. The writer is trying to make a point. What could it
be? Why, he is trying to prove that Aaron is a direct descendant of Levi! Mr. Till seems to
think there was a continuing struggle between the Aaronic branch of Levi and other branches,
yet Korah's rebellion did not have a thing to do with whether or not Aaron descended from
Levi but whether all the people could be priests or just Aaron's family. Members of the tribe
of Reuben were even involved, (Num. 16:1). But Mr. Till and his scholars seem to insist
Moses didn't write Exodus 6. Too, they seem, in a kind of hysterical way, to know all kinds of
things about this fairy scribe who supposedly did write this portion. It is a shame to spoil a
really good cock-and-bull story; however, notice a few things they don't know. They don't
know of a scripture or any ancient document that says this man even existed. Do they know
he was a priest? No. Do they know where he lived? No. Do they know his family? No. Do
they know when he wrote? No. Could he be imaginary? Yes! This sly priest, like
Melchizedek, had neither father nor mother, beginning of days or end of life, (Heb. 7:3). One
of the main differences between the two is that Melchizedek had a name while this other
mysterious priest did not. It seems a disgrace that Mr. Till should cruelly force him to parade
before us without even a name, so I think we should give him one. From now on I'll just call
him Skeeter.

Further, Mr. Till says that Moses shifted from one Amram to another with very poor
transition. Mr. Till claims it is a "serious compositional error." Let's have a little Christian
charity, Mr. Till. It was difficult for Moses to get it right. After all, he lived over 3,000 years
before Mr. Till's English Grammar was published. Second, we have shown that there was one
Amram. But Mr. Till says this Amram took a wife, Jochebed, his father's sister, Levi's
daughter ( Ex. 2:1-10; Num. 26:59). Like the demon who was afraid he might be tormented
before his time (Matt. 8:29), Mr. Till said, "Mr. Moffitt will argue that she was a daughter of
Levi only in the sense that she was a descendant of Levi, and he could probably get away with
this were it not for Numbers 26:57: 'And the name of Amram's wife was Jochebed, the
daughter of Levi, who was born to Levi in Egypt....'" He claims the New International
Version tampered with the text, rendering it: "The name of Amram's wife was Jochebed, a
descendant of Levi, who was born to the Levites in Egypt." Mr. Till was slanderously wrong,
although I'm sure he did not intend to be, when he said that the NIV tampered with the
Hebrew text. The text wasn't changed; it was only rendered into English. The rendering was
interpretative, true, but in this case it is correct. Take the word beget (Hebrew, yalad). Even if
Jochebed were a descendant of Levi nine times removed, Levi could still be said to have
begotten her. According to the Hebrew way of thinking, she would have been his daughter.
Notice what Theological Wordbook of the Old Testament says on "bear, beget, bring forth,
travail": "The word [yalad] does not necessarily point to the generation immediately
following. In Hebrew thought, an individual by the act of giving birth to a child becomes a
parent or ancestor of all who will be descended from this child. Just as Christ is called a son
of David and a son of Abraham, yalad may show the beginning of an individual's relationship
to any descendant," (I, 379).

Volume 1990 - 2002 Issue


Page 28 of 2049
Skeptical Review Edited by Farrell Till
Yes, Jochebed was born to Levi, but only in the Hebrew sense. In that sense Jacob told Joseph
that Joseph's two sons were his [Jacob's] (Gen. 48:5). Ruth had a child and the people said,
"There is a son born to Naomi," (Ruth 4:17). Can one, in Hebrew thought, be born to a person
in the sense of tribe, family, or house? Notice: "And he cried against the altar by the word of
Jehovah, and said, O altar, altar, thus saith Jehovah: Behold, a son shall be born unto the
house of David, Josiah by name," (I Kings 13:2). In Numbers 1:18, yalad was used in the
sense of declaring tribal pedigrees. In Numbers 11:12, Moses even used yalad of himself, as if
he could have been thought of as having given birth to Israel. So as a descendant, Jochebed
was born unto Levi, just as Israelites could even bare children unto God, (Ezek. 23:37).

But Mr. Till wonders why it states that she was born in Egypt. The simple answer is obvious;
she was born in Egypt. There is no great mystery behind such an expression. One may as well
make wild speculation and weave mysterious theories out of the fact that it says the same
thing about Ephraim and Manasseh. It says, "and the sons of Joseph, who were born to him in
Egypt," (Gen. 46:27). It is silly to postulate and conjecture fanciful stories that assume some
enigmatic Ephramite wanted to prove Ephraim descended from Joseph and that this statement
is part of a mysterious XYZ document secretly redacted by this undivulged Ephramite later in
the land of Israel. We can all see that Mr. Till is a gifted hypothesizer and that he deeply
enjoys his notions. Yet, if we want fiction, let's turn to Hollywood and not to the Bible.

So we showed that once again there are some elided generations in a genealogy. It was
common. We have answered every objection Mr. Till could come up with against this
position. There is no intertextual contradiction, so Mr. Till's final statement that when I win I
lose is about as likely as walking down the street today and seeing a nun with a harpoon.

Finally, I covered everything Mr. Till brought up. If I missed anything it was an oversight.
The Bible as originally written is inerrant. We wish it were possible to reply to every article
written in The Skeptical Review. This article demonstrates we do have replies if we are given
the opportunity. But we appreciate what we can get. As Jesus said, "The scripture cannot be
broken," (Jn. 10:35). Believing that with all our heart, we stand ready to refute, by God's help,
any article discrediting the Bible that will be published in The Skeptical Review.

(Jerry Moffitt's address is P. O. Box 1275, Portland, TX 78374- 1275.)

EDITOR'S NOTE: Mr. Moffitt called our attention to certain "pivotal words" that make Till's
article "a pretty 'iffy' document." Turn about is fair play, so we suggest that Mr. Moffitt take
another look at his own articles for expressions like the following, which are just as iffy as
anything he quoted from Till's article: "The originals may just happen to be inerrant.... God
may have wanted perfect standards.... If Amram claimed one fourth.... Let's say (in other
words, let's assume) Kohath had only three sons.... If Amram were responsible for one fourth
of the 8,600 Kohathites.... Let's help him by supposing.... Let's locate the missing links
somewhere else. Let's put them between Kohath and the four.... Even if Jochebed were a
descendant of Levi nine times removed, Levi could still be said to have begotten her...." This
all sounds very speculative to us, so if Till's articles lacked "the flavor of logic and the
scientific method," as Mr. Moffitt charged, the same would have to be true of his. So what has
he proven?

Volume 1990 - 2002 Issue


Page 29 of 2049
Skeptical Review Edited by Farrell Till
We commend Mr. Moffitt for effectively showing that the Exodus-6 text required Eleazar to
be the literal son of Aaron and Phinehas the literal son of Eleazar, (p. 8). In so doing, he only
confirmed what we have been arguing: this genealogy was intended to be a complete,
generation-by-generation family tree. All that Mr. Moffitt said in making his point could be
used to show that Aaron and Moses were also the literal sons of Amram, so if this much of the
passage was literal, what is Moffitt's reason for interpreting "sons" to mean "descendants" in
verse 18? If the expression "Aaron's son," as applied to Eleazar (v:25), was literal and if "sons
of Izhar" was literal in verse 21 (compare with Num. 16:1), by what interpretation principle
does Moffitt make "sons of Kohath" (v:18) mean "descendants"? The answer is simple: unless
he distorts the obvious meaning to suit his needs, he is left with a glaring contradiction in the
Bible text, and he must avoid that at all costs.

For the same reason, he must make dod in Lev. 10:4 not mean "uncle" but "kinsman." We
have checked nine translations, and all of them refer to Uzziel as "the uncle of Aaron," but
Mr. Moffitt can't accept this. If he does, his two-Amrams theory will suddenly evaporate.

Or does he in fact believe in the two-Amrams theory? On page ten, he said, "... we have
shown that there was one Amram." We freely admit to being confused, because we thought he
was arguing that there were two Amrams. Perhaps he can clarify this for us in a later issue.

There is much more that we could say about Mr. Moffitt's failure to sustain his position, but
we must get to other matters in future issues. The last word, however, has not been said on
this point. It will be brought up again in the Till-Moffitt debate scheduled to begin August 13.

A Perfect Work of Harmony?


Inerrancy proponents often refer to the Bible's perfect harmony and consistency of theme as
compelling evidence of verbal inspiration. Our reaction to their claim is what harmony? What
consistency of theme? Only someone determined to believe in Bible inerrancy no matter how
convincing the evidence to the contrary could possible claim to see harmony and consistency
in the Bible.

Religious faction existed in Bible times as surely as it exists today, and Bible writers echoed
their partisan beliefs as resolutely as do modern-day religious journalists. We could cite
several examples of factional disagreement in the Bible, but the divergent views of Jehu's
slaughter of the royal family of Israel will illustrate our point as well as any.

Jehu's divine mission was mentioned as early as I Kings 19:15-17 when the word of Yahweh,
as it was prone to do in those days, came to Elijah when he was seeking refuge in a cave on
Mount Horeb. Yahweh commanded Elijah to go anoint Hazael to be king of Syria and Jehu
king of Israel. "And it shall come to pass that him that escapeth from the sword of Hazael
shall Jehu slay; and to him that escapeth from the sword of Jehu shall Elisha slay," (v:17).

Volume 1990 - 2002 Issue


Page 30 of 2049
Skeptical Review Edited by Farrell Till
For reasons that seem not to bother Bible inerrantists, presumably Ahab's repentance (I Kings
21:27-29), the anointing of Jehu was delayed so that "the evil" of Yahweh's judgment would
instead be inflicted on Ahab's son. That, by the way, was quite typical of Yahweh's style (a
point we don't have time to pursue in this article). Anyway, as the story goes, Jehu wasn't
anointed until Elisha sent a "son of the prophets" to Ramoth-gilead to execute the
commandment originally given to Elijah. After being anointed, Jehu took an armed force
against Jezreel and slaughtered not only the king of Israel, his seventy sons, and his mother
Jezebel but also Ahaziah, the king of Judah, and forty-two of his brothers who were all
visiting Jezreel, (II Kings 9; 10:1-17). Afterwards, we are told that Yahweh said unto Jehu,
"Because thou hast done well in executing that which is right in mine eyes, and has done unto
the house of Ahab according to all that was in my heart, thy sons of the fourth generation
shall sit on the throne of Israel," (II Kings 10:30).

If what Jehu did in executing these grisly deeds was so pleasing to Yahweh, why did he
afterwards say to his "inspired" writer Hosea, "For yet a little while, and I will avenge the
blood of Jezreel upon the house of Jehu and will cause the kingdom of the house of Israel to
cease," ( Hosea 1: 4)? Why would Yahweh want to punish the house of Jehu for what was
done at Jezreel if all Jehu had done there was "that which is right in mine (Yahweh's) eyes"?

Perhaps some enterprising inerrantist can explain this to us.

Volume 1990 - 2002 Issue


Page 31 of 2049
Skeptical Review Edited by Farrell Till

The Skeptical Review


Volume One, Number Three
July/August/September 1990
Farrell Till, editor

No Takers
Farrell Till claims there is a contradiction between II Kings 9; 10:1-30 and Hosea 1:4.
He also discusses II Timothy 3:16 and II Peter 1:20-21, which claim the Bible is
inspired.
A Reply to "The Flat-Earth Belief of Bible Writers"
Fundamentalist minister Jerry McDonald responds to Adrian Swindler's flat-earth
article in the first issue of TSR.
The Flat Earth: Still an Embarassment to Bible Inerrantists
Adrian Swindler responds to Jerry McDonald's above article.
Moffitt-Till Debate Postponed

No Takers
Our second issue brought another wave of angry reaction. Some wrote REFUSED on their
copies and had them returned to us, others sent us terse notes calling our publication "junk" or
"trash" and asked to have their names removed from our mailing list, and still others wrote us
long letters expressing concerns ranging from dismay over our "apostasy" to warnings of
eternal damnation. What none of the letter-writers did, however, was to explain away any of
the Bible discrepancies and inconsistencies discussed in our last issue. A short article on the
backside of that issue presented a discrepancy between II Kings 9; 10:1-30 and Hosea 1:4
concerning Jehu's role in the slaughter of the royal families of Israel and Judah. One
"inspired" writer praised Jehu for "executing that which is right in mine (Yahweh's) eyes"; the
other condemned him for the blood he shed at Jezreel. Why? If the Bible is indeed a "perfect

Volume 1990 - 2002 Issue


Page 32 of 2049
Skeptical Review Edited by Farrell Till
work of harmony," as so many fundamentalists claim, how could it contain a discrepancy as
glaring as this one?

Our article ended with a challenge for "some enterprising inerrantist" to explain this
discrepancy, but we had no takers. Again why? If those who wrote the Bible were as
obviously inspired of God as fundamentalists would have us believe, why can't they show us
how that there is no discrepancy here? Gleason Archer, the chief apostle of modern day
inerrantists, made a pathetic try in his Encyclopedia of Bible Difficulties, but, like most
"explanations" of "alleged Bible contradictions," his is too ridiculous to warrant serious
consideration. In II Kings 10:30, Yahweh unequivocally declared that Jehu had executed "that
which is right in mine eyes" and had done "unto the house of Ahab according to all that was
in mine heart," period, no qualifications. No ifs, ands, or buts about it, Yahweh praised Jehu
for destroying the house of Ahab at Jezreel. Archer claims that the condemnation of the house
of Jehu in Hosea 1:4 was for sins that Jehu committed after executing Yahweh's will at
Jezreel, but that "explanation" is about as thin as any you will find in Archer's book. It, at
best, conveniently reads into the text something that was not even implied in it. The passage
plainly said that in a little while Yahweh would avenge "the blood of Jezreel upon the house
of Jehu," so it was for the "blood of Jezreel" (the slaughter of the royal families of Israel and
Judah) that Hosea pronounced judgment upon the house of Jehu. Let's hope somebody out
there can do a better job of "explaining" this problem than Archer did.

Some of our letter-writers took the time to quote to us II Timothy 3:16 and II Peter 1:20-21,
as if they perhaps thought we were unaware that these passages are in the Bible. Well, we
assure you that we are very familiar with these and other biblical passages that claim divine
inspiration for the Bible, but we know too that the Koran and the Book of Mormon also
contain claims of divine inspiration, so there is nothing unique about the Bible inerrancy
doctrine. We simply contend that there is sufficient evidence, both internal and external, to
reject this doctrine, just as most of our readers would agree that there is sufficient evidence to
reject the claims of inspiration made by the Koran and the Book of Mormon. We publish The
Skeptical Review specifically for the purpose of allowing our readers the opportunity to
consider evidence that disputes the Bible inerrancy claim. In so doing, we try to give the
opposition a fair hearing. In this issue, for example, we are publishing in its entirety an article
that Jerry McDonald, a fundamentalist minister, wrote in response to Adrian Swindler's flat-
earth article in our first issue. Do you know of any fundamentalist publications that would
give us the same consideration?

Although we have received compliments and expressions of gratitude for it, there is nothing
particularly noble about our policy of publishing opposition materials. Our motives are
entirely self-serving. We believe our position on the inerrancy issue is a sound one and that its
soundness can best be demonstrated by giving the opposition the uncensored opportunity to
respond to our arguments. Each time we identify a contradiction or discrepancy in the Bible
text and a competent inerrancy spokesman fails to satisfactorily "explain" it, the truth of our
position should become more obvious to our readers. In other words, we think that if we give
the inerrantists enough rope, they will eventually hang themselves.

From our vantage point, they appear to be proving us right.

Volume 1990 - 2002 Issue


Page 33 of 2049
Skeptical Review Edited by Farrell Till

A Reply to "The Flat-Earth Belief of Bible


Writers"
Jerry McDonald
The claim was made by Adrian Swindler (Winter Issue, pp. 9-11) that the writers of the Bible
believed and wrote in the Bible that the earth was flat. Mr. Swindler used several passages to
prove his contention on this matter. In this article, I intend to review those passages and show
what the Bible writers were really saying in them.

First, I want to quote Mr. Swindler's article concerning Bible scholars: "That the Bible
contains mistakes in every area mentioned by Mr. Tillett is a truth widely recognized by
reputable Bible scholars," (emphasis JM). I would like to ask a question: "What reputable
Bible scholars take this position?" Is Mr. Swindler referring to men such as Ian Wilson and
Richard Elliott Friedman? If so, then Mr. Swindler refers to scholars (and I use this word with
hesitation, realizing that anyone can be called a scholar) who believe in higher criticism. I
often wonder why these "scholars" even bother to call themselves Bible believers! Why not
just disregard the Bible completely and become atheists? Higher criticism does away with
miracles and the supernatural happenings mentioned in the Bible. By the time "higher critics"
get through with the Bible, it becomes just an ordinary book. To do this, they completely
disregard the evidence in favor of the Bible's being an extraordinary book. Maybe Mr.
Swindler could tell us which reputable scholars who believe in the inerrancy of the Bible
believe that it contains mistakes in all of the areas mentioned by Mr. Tillett. Now we know
what Mr. Swindler is talking about when he mentions "reputable Bible scholars." He is
talking about scholars who do not believe in the inerrancy or infallibility of the scriptures. No
reputable Bible scholar who believes in the inerrancy of the Bible would take the position that
there are mistakes in the original autographs of the Bible.

Now let us deal with the passages that Mr. Swindler produced to show that Bible writers
believed the earth was flat. The first passage was Psalm 24:1-2: "The earth is the Lord's, and
the fulness thereof; the world, and they that dwell therein. For he hath founded it upon the
seas, and established it upon the floods." Is there anything in this passage that would make
anyone think the earth was flat? Absolutely not! In commenting on these verses, Albert
Barnes says, "The word used here--tebel--is a poetic word, referring to the earth considered as
fertile and inhabited--the habitable globe..." (Barnes on the Old Testament, Psalms I, p. 215).
In the Hebrew dictionary of Strong's Exhaustive Concordance of the Bible, we see the same
meaning for this word: "... the earth (as moist and therefore inhabited); by extens. the globe;
by impl. its inhabitants..." (p. 122). Does this sound as if the psalmist was saying that the earth
is flat? He used a word that recognized the earth is round, a globe. Barnes continues: "As the
earth appeared to be surrounded by water, it was natural to speak of it as founded also upon
the waters.... The earth has been elevated above them, so as to be a residence for animals and

Volume 1990 - 2002 Issue


Page 34 of 2049
Skeptical Review Edited by Farrell Till
for men," (p. 218). This passage does not teach that the earth is flat; rather it teaches that the
earth is round because of the usage of the word tebel.

The next passage Mr. Swindler used was Daniel 4:10: "... I saw, and behold, a tree in the
midst of the earth, and the height thereof was great." When one reads the context of this verse,
it can be easily seen that this was one of Nebuchadnezzar's visions. There is nothing in this
verse to indicate that the writer thought the earth was flat. Why could Nebuchadnezzar not
have seen a vision of the round earth and a tree in the center of it? The tree had reference to
himself, and the vision was showing that he would become great and powerful and then he
would fall. Mr. Swindler thinks this passage proves a flat-earth concept because verse 11
says, "... and the sight thereof (was) to the end of all the earth." We need to remember,
however, that this was a vision, a dream, and many things are possible in visions and dreams
that are not possible in reality. For example, in a dream I once had, some dogs were chasing
me, and I came to a tree that was higher than I could climb, so I jumped and landed in the top
of the tree. Now we all know in reality that I cannot just jump and land in the top of a tall tree,
but in the dream this was possible. So what would have kept it from being possible for
Nebuchadnezzar to see a tree in his dream standing in the center of a round earth and it be so
tall as to be seen by all the earth? Nothing! This was a dream, and the rules of dreams do not
coincide with the rules of reality. The meaning of the tree's being seen from the ends of the
earth was only as Mr. Barnes says: "It could be seen, or was visible in all parts of the earth.
The Greek here for sight is xutos, breadth, capaciousness.... The vision which
Nebuchadnezzar had here, of a tree so conspicuous as to be seen from any part of the world,
was one that would naturally be applied to a sovereign having a universal sway," (Barnes,
Daniel I, p. 250). There is nothing here to prove that this writer had a flat-earth concept.

The next passage Mr. Swindler used was Matthew 4:8: "Again, the devil taketh him up into an
exceeding high mountain, and sheweth him all the kingdoms of the world, and the glory of
them." Mr. Swindler said, "The only plausible reason for the 'very high mountain' was that the
altitude would make it possible to see the ends of the earth. Only on a flat earth would this be
remotely possible." Where does Mr. Swindler get the idea that the word world in this passage
refers to the whole earth? In the Bible, the word world refers to many things. In John 3:18, it
refers to the people on the planet. In 1 John 2:15-17, it refers to the wickedness of men. In
Luke 2:1, the word refers to all of the provinces of the Roman Empire. So why does Mr.
Swindler think that the word world in Matthew 4:8 refers to the whole planet? Because
without that interpretation there would be no way to prove his point. However, let us place a
different interpretation on the verse. Dungan says: "Rule 7. The proper definition of a word
may be used in the place of the word. If the trial be made in this way, and the definition is
wrong, the sense of the passage will be so destroyed as to make it apparent. It need only be
stated that the true meaning of a word will give the same sense that the word would give,"
(Hermeneutics, pp. 188-189). So let us substitute another word in the place of the word used
in Matthew 4:8 and see if it makes any sense. "Again, the devil taketh him up into an
exceeding high mountain, and sheweth him all the kingdoms of Palestine (the world), and the
glory of them." Now, what would be so hard about that? Barnes says that "we need not
suppose that there was any miracle when they (the kingdoms) were shown to the saviour,"
(Barnes, Matthew and Mark, p. 35). If, however, the word world does refer to the whole earth
in the passage, we need to remember that Satan had the power to perform supernatural
happenings. It is possible as H. Leo Boles said that "the devil may have had supernatural

Volume 1990 - 2002 Issue


Page 35 of 2049
Skeptical Review Edited by Farrell Till
power and presented Jesus with a mental vision of 'all the kingdoms of the world, and the
glory of them,'" (Commentary of the Gospel According to Matthew, p. 102). We do not know
from the context if Jesus literally saw the kingdoms of the earth in a vision or the actual
power and the glory of these kingdoms. (See Boles' commentary for an extensive explanation
of this, pp. 102-103.) The context does not bring this out. This passage does not necessitate or
even imply a flat-earth concept.

Swindler's next passage was Job 38:22: "Hast thou entered into the treasures of the snow? or
hast thou seen the treasures of the hail?" Again, I find nothing in this verse to necessitate or
imply the flat-earth concept. Barnes says, "The simple appeal to Job here is, whether he could
explain the phenomena of snow and hail. Could he tell how they were formed? Whence they
came? Where they were preserved and how they were sent forth to execute the purposes of
God? The idea is that all that pertained to snow was distinctly understood by God and that
these were facts which Job did not know of, and which he could not explain," (Barnes, Job II,
p. 202). How does this necessitate or even imply a flat-earth concept?

The next passage was Psalm 104:3, 13: "Who layeth the beams of his chambers in the waters:
who maketh the clouds his chariot: who walketh upon the wings of the wind.... He watereth
the hills from his chambers: the earth is satisfied with the fruit of thy works." Here Mr.
Swindler really stretches the imagination to get the flat-earth concept. How in the world does
this teach a flat-earth concept? Would it be impossible for God to dwell above the earth and
cause it to rain upon the earth and the earth still be round? IF so, then it would be impossible
for the rain to fall on a round earth! Quite ridiculous! This merely means that God made his
abode above the earth and that he is the giver of the rain and all good gifts. Nothing here
necessitates or even implies a flat-earth concept. When the space shuttle goes into outer space,
we speak of it as being above the earth. Does this mean that we have a flat-earth concept? No!
And it does not mean that the psalmist had a flat-earth concept either.

The next passage was Genesis 1:6, 7: "And God said, Let there be a firmament in the midst of
the waters, and let it divide the waters from the waters. And God made the firmament, and
divided the waters which were under the firmament from the waters which were above the
firmament: and it was so." How do these verses show that the writer thought the earth was
flat? They do not! Again Mr. Swindler stretches the imagination to try and prove his point.
Here Mr. Swindler used a translation that uses the word dome in the place of firmament or
expansion. The only problem with the word dome being used here is that people think of a
dome as a bubble that covers something flat. The word expansion or firmament, however, is a
much better translation. This expansion or firmament surrounds the whole earth. Leupold says
that it is "an air space encircling the earth," (Leupold on the Old Testament, Genesis I, p. 59).
What this does is to keep the waters on the earth and the waters (mist, fog, rain, etc.) above
the earth apart so that the sun can shine and we can live upon it. Without this expansion, we
could not live on the earth. How does this necessitate or even imply a flat-earth concept? It
does not!

The next passage was Job 38:12-14: "Hast thou commanded the morning since thy days; and
caused the dayspring to know his place; that it might take hold of the ends of the earth, that
the wicked might be shaken out of it? It is turned as clay to the seal; and they stand as a
garment." Mr. Swindler said, "Notice also that the KJV refers here to 'the ends of the earth.'

Volume 1990 - 2002 Issue


Page 36 of 2049
Skeptical Review Edited by Farrell Till
This would indicate a flat earth, since there are no ends to a globe." We who today know that
the earth is round will say such things as, "That person would follow you to the ends of the
earth," yet we realize that the earth has no literal ends. What is so difficult about
understanding this passage? Does this phrase necessitate a flat-earth concept? No! Does it
even imply it? Not unless our language today implies such. Mr. Swindler, like so many others
today, will do anything possible to prove that the Bible was not inspired by God, even if it
means that they neglect the fact that the Bible speaks in figures of speech in many places.
They refuse to allow the Bible to speak in such a way that man can understand. Why?

The next passage was Job 28:7: "He stretcheth out the north over the empty place, and
hangeth the earth upon nothing." How does this passage necessitate or even imply a flat-earth
concept? It does not! Mr. Swindler says that Job was wrong in verse 11: "The pillars of
heaven tremble and are astonished at his reproof." Does this necessitate the flat-earth concept?
No! Barnes says, "That is, the mountains, which seem (emphasis JM) to bear up the heavens,"
(Barnes, Job II, p. 46). Is there anything difficult about this passage? No! Does it necessitate
or imply a flat-earth concept? No! He is speaking poetically here. What is wrong with that?

The next passage was Isaiah 14:13: "For thou hast said in thine heart, I will ascend into
heaven, I will exalt my throne above the stars of God: I will sit also upon the mount of the
congregation, in the sides of the north." Here the king of Babylon was merely placing himself
above what he ought to be. He envisioned himself as being as powerful (if not more powerful)
than God and reigning in heaven. Now, as for his concept of the earth's shape, I do not know
what that was, but rest assured that the Bible writer here was not teaching the flat-earth
concept. He was merely showing the king of Babylon what his concept was of where he
would be reigning. It says nothing at all about the shape of the earth.

The final passage was Isaiah 40:22: "It is he that sitteth upon the circle of the earth, and the
inhabitants thereof are as grasshoppers; that stretcheth out the heavens as a curtain, and
spreadeth them out as a tent to dwell in." The word for circle in this passage is the Hebrew
word khoog, which, when in its masculine form as it is here, means "a circle, a sphere," (The
Analytical Hebrew and Chaldee Lexicon, p. 249). Therefore, this verse does deal with the
shape of the earth; it shows that the earth is a circle or a sphere. It says that God dwells above
this sphere or circle and the inhabitants of this earth are as grassphoppers in his sight. This
verse does not teach or even imply the flat-earth concept.

In order for Mr. Swindler, Mr. Till, and others of the agnostic/atheistic belief to discredit the
Bible on this and other subjects as well, they must go to faulty translations, place
misinterpretations upon passages, and show pictures from dictionaries about what some
thought about the earth's shape. However, they cannot discredit these verses, because I have
shown that none of the verses that Mr. Swindler brought up teaches or even implies what Mr.
Swindler, Mr. Till, and others of their belief say that they do.

Mr. Till says he is not sure of his position, and I assume that Mr. Swindler takes the same
position. Mr. Swindler seems to be sure, however, that the Bible is not inspired or inerrant. I
wonder why? Mr. Swindler said, "The Hebrews were inspired by nothing more than their
political and religious motivations." I wonder how Mr. Swindler knows that this is true? I

Volume 1990 - 2002 Issue


Page 37 of 2049
Skeptical Review Edited by Farrell Till
wonder how he can be so certain? I have taken his article apart, point by point, and have
shown beyond a shadow of a doubt that his belief is false.

I must insist, therefore, that Bible believers still believe in the inerrancy of the word of God.

(Jerry McDonald's address is 97 Florence Street, Sullivan, MO 63080.)

The Flat-Earth: Still an Embarassment to


Bible Inerrantists
Adrian Swindler
As I proved in an earlier article ("The Flat-Earth Belief of Bible Writers," Winter Issue, 1990,
pp. 9-11), the Hebrews conceived the world as a three-storied structure that included a flat-
earth belief. That they believed in such an unscientific concept should not surprise us, because
they were surrounded by pagan cultures much older than theirs whose cosmologic views were
very similar. The Hebrews had simply borrowed this concept from their pagan neighbors. In
Man and the Cosmos, Lars Thunberg described the pagan cosmology of two of those pagan
neighbors:
The Babylonians thought of heaven as a great vault, immobile and solid, whose foundations
rested on a vast ocean (apsu, meaning "the deep"). Above the vault (dome, firmament) was
the "dwelling of the gods" from which the sun comes through a door every morning and
returns every evening through another door. The earth was supposed to be a mountain, hollow
underneath, also supported by ap-su. The abode of the dead, sheol, the land of darkness and
the shadow of death, was just above the hollow interior but inside the earth' crust. The
Egyptians held similar ideas. As fanciful, and even naive, as these ideas appear now, they
represented the thinking of the day" (1985, pp. 26-27).
The similarity of these pagan ideas to the Hebrew conception of the cosmos should be
apparent to everyone who is familiar with the Old Testament scriptures.

In the lead article of this issue, however, Jerry McDonald, a Church-of-Christ preacher in
Oskaloosa, Kansas, took exception to my first article on this subject. In so doing, he said that
no scholar who believed in the inerrancy of the Bible would take the position that there are
mistakes in the original autographs of the Bible. That was a rather simplistic observation. It
states the obvious and needs no comment. It is a lot like saying a theist would never say that
God does not exist.

But what is this "original autographs" business? There are absolutely NONE, so how could
anyone know that there were no mistakes in them? The copies we do have are obviously not
the same as they once were. In 1958, Professor Morton Smith of Columbia University
discovered in a monastery near Jerusalem a letter containing a missing fragment of the Gospel
of Mark that had been deliberately suppressed by Bishop Clement of Alexandria. It originally

Volume 1990 - 2002 Issue


Page 38 of 2049
Skeptical Review Edited by Farrell Till
followed Mark 10:34 where Jesus, after predicting the approaching death of the "Son of man,"
said, "... and after three days he will rise again":

And they came unto Bethany, and certain woman, whose brother had died, was there. And
coming, she prostrated herself before Jesus and says to him, "Son of David, have mercy on
me." But the disciples rebuked her. And Jesus, being angered, went off with her into the
garden where the tomb was, and straightway a great cry was heard from the tomb. And going
in where the youth was, he stretched forth his hand and raised him, seizing his hand. But the
youth, looking upon him, loved him and began to beseech him that he might be with him. And
going out of the tomb they came into the house of the youth, for he was rich. And after six
days, Jesus told him what to do and in the evening the youth comes to him, wearing a linen
cloth over his naked body. And he remained with him that night, for Jesus taught him the
mystery of the kingdom of God. And thence arising, he returned to the other side of Jordan,"
(Secret Gospel, p. 14ff).
In present versions, this same young man is apparently mentioned in Mark 14:51: "And a
young man followed him, with nothing but a linen cloth about his body; and they seized him,
but he left the linen cloth and ran away naked."

This secret gospel was suppressed by Clement because the Carpocratians were using it to
prove that Jesus approved of homosexual activity. This fragment was about the raising of
Lazarus, and, like most accounts of the gospels, it varies from John's version. Professor Smith
thinks that homosexuality was probably not involved here but rather the description of a
typical mystery school initiation--a ritualized and symbolic death and rebirth of the sort so
prevalent in the Middle East at that time. The point is clear, however, that we don't have all of
the old copies now, so how much more has been excised, added, and altered? No one can tell.

A similar case concerns what the voice in Luke 3:22 said after the baptism of Jesus. Justin
Martyr quoted it as, "Thou art my Son, today have I begotten thee." He said that this was in
the "Memoirs of the Apostles," (Dialogue with Trypho, p. 190 & ciii 6). The Codex Bezae,
the Old Latin, Clement of Alexandria, Augustine, and other western authorities quoted it the
same. But is it that way in your copy? No! So is the Bible complete? Infallible? Inspired of
God? The fact is that we can determine very little about who wrote and when they wrote most
of the Bible.

In Harmony of the Gospels, A. T. Robertson, M. A., LL. D., Litt. D., one of Jerry's old
fundamentalist scholars whom the Church of Christ has used for years, said this about the
Bible writer Luke:

Luke is the first critic of the life of Christ whose criticism has been preserved to us. Others
had drawn up narratives of certain portions of Christ's work. Others still had been
eyewitnesses of the ministry of Jesus and gave Luke their oral testimony. Luke sifted it all
with care and produced an orderly and reasonably full narrative of the earthly ministry of
Jesus. We cannot reproduce all the sources Luke had at his command, but it is clear that he
followed in the main our gospel of Mark, as anyone can see for himself by comparing the two
Gospels in this Harmony. Both Matthew and Luke made use of Mark. But they had other
sources too.

Volume 1990 - 2002 Issue


Page 39 of 2049
Skeptical Review Edited by Farrell Till
So here is your scholar, Jerry! He admits that Luke and the writer of Matthew were about as
inspired as you are. The very fact that Luke wrote his gospel shows that he considered all
others questionable and that he was going to give the straight dope to Theophilus. If Luke had
thought the other gospels were correct and complete, all he would have had to do was point to
them as faithful accounts of the story.

So with the theory of "inerrant original autographs" put to rest, we can now turn to Jerry's
views on scholarship. He didn't care too much for my scholars. Ian Wilson and Richard
Friedman are indeed two of them, but others would include the 100+ translators of the New
American Bible and The Good News Bible. To these can also be added the ones who gave us
The Interpreter's Dictionary of the Bible and the 74 who compiled the New Jerome Bible
Commentary. And that is only a start. My scholars are the professors of philosophy and
religion at the major universities in this country and Western Europe. These are honest people
in their fields, who will not sell out to ignorant and prejudiced fundamentalists. On the other
hand, Jerry's "scholars" are the old manipulators who are adept at using possibility answers to
"explain" the insurmountable inconsistencies, contradictions and absurdities in the Bible.
Possibly this, maybe that, it could be, it might be--this is their strength. My! My! It's so easy
to pull the wool over the sheep's eyes.

Jerry's 19th century scholars have presided over nearly 300 fighting fragments of Christianity
that cannot agree on much of anything. Of course, even the scholars disagree on a multitude
of interpretations. The Church of Christ has been relying on characters like these for almost
two hundred years and has at least 10 divisions, all of which claim to be the "one true
church." The "doctors" of this church are constantly at one another's throats, with J. D. Bales
(Harding College) and Thomas B. Warren (Freed-Hardeman College) disgreeing vehemently
over marriage issues and other interpretative matters. As editor of The Spiritual Sword,
Warren tried with pontifical pronouncements and "definitive" treatment of all subjects to
write the creed for the church. It didn't work!

Jerry even quoted three 19th century scholars whom, if they were still alive, he would not
even allow in his pulpit on Sunday morning, men who believed that baptism is not essential,
that babies should be baptized, that pouring and sprinkling in baptism is acceptable and that a
1000-year literal reign of Jesus is coming. Jerry loves them when he can use their deceptions,
possibilities, and perversions, but he would never give them the right hand of fellowship.

As for the opinion of scholars, I think even Jerry has heard of Bultmann. He, knowing of the
three-tiered structure of the world taught by the Bible, made this cogent observation:

The whole conception of the world which is presupposed in the teaching of Jesus in the New
Testament generally is mythological, i.e., the conception of the world as structured in three
stories, heaven, earth, and hell; the conception of the intervention of supernatural powers in
the course of events; and the conception of miracles, especially the conception of the
intervention of supernatu- ral powers in the inner life of the soul, the conception that men can
be tempted and corrupted by the devil and possessed by evil spirits. This conception of the
world we call mythological because it is different from the conception of the world which has
been formed and developed by science since its inception in ancient Greece and which has
been accepted by all modern men, (Jesus Christ and Mythology, 1958, p. 15).

Volume 1990 - 2002 Issue


Page 40 of 2049
Skeptical Review Edited by Farrell Till
The pre-scientific character of the Bible is obvious to all who will read it objectively.

Jerry spoke of "faulty translations," yet he uses the KJV. "Moreover, brethren, we do you to
wit of the grace of God...." What? "Wot ye not...." This is the nearly 400-year-old language of
the KJV. There are thousands of words that are either mistranslated or obscurely rendered and
several others that are now obsolete. Besides this, the KJV was too highly colored in many
places with the party opinions and ideas of those who translated it to be considered a faithful
record. In the words of Dr. Macknight, "It was made a little too complaisant to the king in
favoring his notions of predestination, election, witchcraft, familiar spirits, and kingly rights,
and these, it is probable, were also the translators' opinions. Their translation is partial,
speaking the language of and giving authority to one sect." And he imparted this not to the
translators alone but to those who employed them, for even some of the translators
complained that they could not follow their judgment in the matter but were restrained by
"reasons of state." So the KJV is not a translation from the oldest MSS--there are no originals-
-but merely a revision of the versions then in use. Those versions had only eight MSS
available, whereas there are about 700 Greek MSS now available. What do these facts say
about the reliability of the KJV?

With Jerry's complaints and quibbles about inerrant original autographs, liberal scholars, and
faulty translations out of the way, I can now address the "rebuttal" arguments in his article.
Space will allow us to reprint only one of the graphics from my first article, so if you have
saved the first issue of TSR, you might want to keep it close at hand for reference purposes as
I analyze Jerry's "counterarguments."

PSALM 24:1-2. Jerry didn't seem to understand why I cited this passage. My point here and
throughout was to show that the graphic illustrations of the NAB and The Interpreter's Bible
were accurate in depicting the Bible writers' conception of a three-tiered universe, with
heaven, hell, and a flat earth. A verse that touches on any one of those three tiers, as these do,
confirms the graphics that contain all three. Jerry said nothing about this.

But one of his scholars has tried to circumvent the obvious embarrassment of this passage.
Scholar Barnes said, "As the earth appeared to be surrounded by water, it was natural to speak
of it as founded also upon the waters...." Natural indeed to the pre-scientific, ignorant
primitive mind but absolutely false nevertheless! In fact, Barnes' statement is an admission
that the writer was wrong and was basing his statements on appearance rather than fact.
Furthermore, Barnes himself is in error in this statement: "The earth has been elevated above
them (the seas)...." That is absolutely incorrect. The earth is not built upon the seas; it contains
the seas. The vast majority of the land is under the water with some rising above it. The Bible
is wrong; the earth (Barnes' and Strong's "globe") is NOT founded upon the seas! Anyone
should easily recognize that Jerry's scholars lived recently enough to know from science that
the earth is round, and that they simply inserted scientific knowledge into their definition of
tebel as meaning "the earth" and by extension "the globe." By their extension, the earth is a
globe, but their extending the meaning of the word results in a vicious lie! There is absolutely
NO Hebrew word for globe in the sense of Earth, because those ancients thought the earth
was flat. By their extensions, Barnes and Strong simply lied. They extended the truth, and that
constitutes lying. Young's Analytical Concordance has every instance where the words

Volume 1990 - 2002 Issue


Page 41 of 2049
Skeptical Review Edited by Farrell Till
translated earth are used, and NOT ONCE does the root mean globe. Jerry's fundamentalist
scholars have been clearly discredited.

DANIEL 4:10. The dream and interpretation of it in this passage were, according to the story,
inspired by God. This was not like one of Jerry's dreams, which is caused by something he
eats, but was a sensible dream with a sensible interpretation. That dream clearly presented a
flat earth with a tree at the center that could be seen from the ends of the earth. This
expression was used over and over in the Bible, as it was here and in Job 38:13-14, and Bible
writers used it because they thought the earth was flat and had ends, just as most people did at
the time of Columbus and before. Jerry said, "We still use that language, even though we
know the earth has no ends." That's true, but the expression originated in a time when people
did think the earth had ends. Our language is filled with unscientific expressions, such as
sunrise and sunset, that originated when people thought they conveyed scientific fact. They
are in our language, because we have a tendency to retain such idioms long after we know
them to be erroneous.

MATTHEW 4:8. The only reason for taking Jesus to an "exceedingly high mountain" was for
a visibility factor that would show him ALL the kingdoms of the world, just as taking him to
the highest point of the temple was to give falling distance. Jerry appealed to Dungan and the
hermeneutic principle of word substitution. I agree wholeheartedly with this principle and
will, to Jerry's embarrassment, shortly use it myself. However, Jerry wants to substitute
Palestine for the world in this passage, so let's look at the consequences of this substitution.
The same word kosmos was used in Mark 16:15: "Go into all Palestine and preach the
gospel...." Well, well, well!

On this verse, Jerry had his scholars contradicting each other and himself. Barnes said, "... we
need not suppose that there was any miracle when they (the kingdoms) were shown to the
savior." But Boles said, "The devil may have had supernatural power and presented Jesus
with a mental vision of 'all the kingdoms of the world....'" So Jerry goes along with Barnes
who claimed only a tiny world was involved and therefore no miracle happened, but then he
quoted Boles who gave the devil credit for a miraculous showing of all kingdoms of the earth
in a vision! This is a fundamentalist nightmare. Their own scholars disagree with each other,
and all they can propose is maybe, possibly, could be, perhaps, it is possible, ad infinitum and
ad nauseam. So please explain something, Mr. Boles et al. Why take Jesus to an "exceedingly
high mountain" only to show him a MENTAL VISION? Hogwash and balderdash! Barnes,
Jerry, et al, how much could you tempt a person by showing him the glory of Palestine?
Glory? What glory? A depressed, primitive, third-worldlike area! You and your pitiful
scholars are batting exactly zero, and you don't get any better.

GENESIS 11:4. The language and context here clearly shows an anthropomorphic god was
afraid that, if he left the people to their own devices, they would reach heaven where his
throne was. Yes, Jerry, those writers were just like you were in your childhood, but they had
no one to teach them science as you had. The only reason you don't feel the same way about
the distance to the stars now is because you have been taught scientific facts. It isn't because
you're an adult but because you have been taught that the earth is not flat and that the stars are
billions of miles away. Science is your teacher and not the Bible. Hold to the Bible and you,
like the Amish and the people of Zion City, Illinois, will believe the earth is flat. We can at

Volume 1990 - 2002 Issue


Page 42 of 2049
Skeptical Review Edited by Farrell Till
least credit them with honesty. They believe this because they believe the Bible, but you are
trying to explain these things away.

Jerry's scholar Leupold carried no weight at all with his foolish comment: "It cannot but work
harm to let this situation continue." How ridiculous! Those people couldn't have hurt a thing
with their ziggurat. Many of them were built in that area at this time. Language didn't come
from a god confounding their speech; it developed from grunts and growls and has been
changing ever since. The English of 500 years ago was so different from ours that, if we were
taken back in time, we wouldn't be able to understand it.

JOB 38:22. This Bible writer had no idea how snow and ice are formed, so he had his god
pose this as a problem for Job. Jerry's Barnes gives us another idiotic and unsupported
statement, but the scholars who translated the NAB, The Interpreter's Bible, and MATC
understood clearly that god was telling Job that he stores up the snow and hail. But we know
very well how hail and snow are formed; it is no mystery at all. Job's god lied to him and told
him he kept the snow and hail "ready for times of trouble, for days of battle and war." We
know, of course, that there was no god involved, merely an uneducated, pre-scientific writer.

Let's look, for example, at the questions this god allegedly asked Job. They are either
questions that little children in school could easily answer or those that are based on
erroneous conceptions. "What holds up the pillars that support the earth? Who laid the
cornerstone of the world?" (v:6). ANSWER: There ain't any, and nobody! "Who closed the
gates to hold back the sea?" (v:8). ANSWER: Nobody, because there are no gates. "Have you
walked on the floor of the ocean?" (v:16). ANSWER: People have, so what? "Do you know
where light comes from or what the source of darkness is?" (v:19). ANSWER: What a
question! It reminds me of an old "little moron" joke. In a class discussing the relative
importance of the sun and the moon, the teacher asked, "Which is more important, the sun or
the moon?" The little moron answered, "Why, the moon is more important! It gives us light at
night when we need it; the sun is there in the daytime when it's already light." This entire
chapter in Job is laughable to anyone educated in science.

GENESIS 1:6-7. Jerry quibbled over the meaning of dome, expansion, firmament, and vault
and then quoted Leupold again, who said that the firmament surrounding the earth is simply
an air space. Now where did Leupold get that? Ipsi dixit will not do. What scripture did he
rely on? It is simply an explanation without evidence. As fundamentalists are so prone to do,
Jerry accepted it and then said that this air keeps the mist, fog, and rain apart from the earth.
So Jerry is still a child. I've seen all of those elements in very close connection with the earth
but never at all separated.

To test the soundness of his theory, let's use Jerry's hermeneutic trick and substitute air for
firmament:

And God said let there be lights in the air of the heavens to separate the day from the night...
and let there be lights in the air of the heavens to give light upon the earth.... And God made
the two great lights, the greater light to rule the day, and the lesser light to rule the night; he
made the stars also. And God set them in the air of the heavens to give light upon the earth,
(Gen. 1:14-17).

Volume 1990 - 2002 Issue


Page 43 of 2049
Skeptical Review Edited by Farrell Till
Now isn't that something? All of those heavenly bodies up there in the air! That means they
are all within 200 miles of the earth. Does this mean the moon is not 238,000 miles away?
The sun is not 93 million miles out in space? How do our space vehicles get past that vast
array of bodies that are all up there in the air?
Let there be air in the midst of the waters, and let it separate the waters from the waters. And
God made the air and separated the waters which were under the air from the waters which
were above the air. And it was so. And God called the air heaven, (vv:6-8).
Now isn't that a shocker? Is there actually water above the AIR? Mind you, that is not in the
air but above the AIR! Not water in the firmament but water above the firmament! Are the
sun and moon and all the stars up there in the AIR? Jerry's Dungan (a real scholar) and
hermeneutics are very useful, eh?

My use of Psalm 104:3, 13 was misunderstood by Jerry (so what else is new?). Check the
NAB graphic illustration again, and you will see that the throne of god is above the dome and
the floodgates (sluices) need merely be opened from god's palace. Also, the expression "who
lays the beams of his chambers in the waters" has reference to his throne established on the
waters above the dome. Since this language supports the graphic illustrations I referred to, it
teaches the three-tiered structure, including the flat earth.

Job 38:12-14. Jerry tried to justify the Bible writers for saying "the ends of the earth" on the
grounds that people still say this. Just a tiny bit of thought should have suggested to him that
the expression originated in a time when people did believe the earth was flat. In my
comments on Daniel 4:10, I addressed the issue of unscientific idioms. The same principle
applies here, so nothing more needs to be said about it.

Job 26:7. Apparently, Jerry didn't realize I was answering an argument that claims this verse
teaches a global earth. It teaches no such thing, and my statement on this should be read again
with that context in mind.

Job 26:11. The fact that the writer speaks of the "pillars of the earth" proves again that my
graphic illustrations are correct. Jerry hangs in there with his discredited Barnes who speaks
of "mountains which seem to support the earth," (emphasis, AS). Can you believe an adult of
even average intelligence would make a statement like that? Job 38:6 asks, "What holds up
the pillars that support the earth?" I suppose Barnes would have said the pillars "seem" to
support the earth. Barnes adds to what is written and deserves the condemnation of Revelation
22:18.

Isaiah 14:13. Jerry misunderstood my use of this passage, even though my argument was
clearly directed against those who use it to prove the writer thought the earth was round. My
original statement should be reread with that context in mind.

Isaiah 40:22 was used for the same purpose as above. I showed that this verse does NOT
teach a round earth. Poor Jerry thinks the old King James per-Version is the correct one, even
with its 20,000 errors. I wonder if those translators were right when they substituted easter for
passover in Acts 12:4? The NAB and GNB translators made Isaiah 40:22 quite clear: "He sits
enthroned above the vault of the earth, and its inhabitants are like grasshoppers; he stretches
out the heavens like a veil, and spreads them out like a tent to dwell in." Isn't that exactly what

Volume 1990 - 2002 Issue


Page 44 of 2049
Skeptical Review Edited by Farrell Till
the graphic illustrations showed? Of course it is! Isn't it nice to know that the heavens are
spread out like a tent? All the tents I have seen were domed over a flat surface. My, my, what
you can learn from the Bible! This is the very verse medieval churchmen quoted to prove the
earth was flat! Now comes Jerry McDonald to tell us it teaches the earth is round.

Those old churchmen understood the Bible much better than Jerry and his deceptive, shifty,
sly, and crooked commentators. Consider, for example, this quotation from Man and the
Cosmos:

We now come to the "dark ages" in the development of cosmology. From Aristotle and
Ptolemy until Copernicus thirteen centuries later, no apparent advance had been made. It even
took until A.D. 1000 for the West to accept a round earth and Ptolemy's system. However, to
understand the background of the Copernican revolution that was to follow, we should know
the important factors of those intervening, nonproductive years, which included political and
religious considerations affecting the study of cosmology.... In its desire to stamp out any
pagan influence, the church soon adopted a rigid interpretation of Scriptures and rejected
anything that might even remotely challenge her influence. Lacantius (A.D. 240 ca.- 320),
writing on the false wisdom of the philosopher, ridiculed the belief in a round earth. His
arguments were the ancient ones about the impossibility of walking upside down and places
where the rain and snow fall upward. He quoted Isaiah 40:22, "It is He that sits upon the circle
of the earth, and the inhabitants thereof are as grasshoppers; he stretches out the heavens as a
curtain, and spreads them out as a tent to dwell in," (pp. 64-65).
Strange, isn't it, that Jerry and Lacantius quoted the same verse, one to prove the earth is flat
and the other to prove it is a globe? Lacantius had a better understanding than Jerry, because
the idea in this verse was the ancient one in which God sat enthroned above a dome (vault,
firmament, expanse) that he had stretched out as a tent to dwell in.

Jerry alleged that Farrell Till is not sure of his position and assumes that I take the same
position. Well, he assumes far too much. I know Farrell quite well, and both he and I are very
sure of our position. Also, I am very sure the Bible is inspired but not by a god. No god would
make those horrendous mistakes.

Jerry bragged that he has shown my position to be false, but in fact he has failed most
miserably in this regard. That kind of attitude reminds me of the little boy whistling to cover
his fear as he goes by the graveyard.

I have clearly established the correctness of the graphic illustrations of NAB, The Interpreter's
Bible, and Man and the Cosmos. A dome over a flat earth, which was built on the seas, with
pillars reaching into the seas to support the earth and sheol deep in that flat earth--this was the
three-tiered world of the Hebrews, the world their writers described in the Bible.

(Adrian Swindler's address is P. O. Box 695, Elmwood, IL 61529.)

Volume 1990 - 2002 Issue


Page 45 of 2049
Skeptical Review Edited by Farrell Till

Moffitt-Till Debate Postponed


At Jerry Moffitt's request, he and Farrell Till have mutually agreed to postpone until next
summer their debate originally scheduled for August 13, 14, 16, and 17 of this year. The
location will remain the same (Independence, Missouri), but the exact date will be announced
later.

Volume 1990 - 2002 Issue


Page 46 of 2049
Skeptical Review Edited by Farrell Till

The Skeptical Review


Volume One, Number Four
October/November/December 1990
Farrell Till, editor

A Year in Review
Till discusses reader reactions to TSR, life as an agnostic or an atheist, and the fact
that no Christian pastors were willing to respond to the article about scientific
foreknowledge in this issue of TSR.
What About Scientific Foreknowledge in the Bible?
Till discusses the claim that "The Bible contains many examples of scientific
foreknowledge in it."
The Jeremiah Dilemma
Discoveries of the Dead Sea Scrolls at Qumran refute the claim that the present day
Bible is essentially the same as what was in the "original autographs."
Textual Contradictions in the Bible
The Bible contains numerous contradictions and is not the work of perfect harmonmy
as inerrancy believers claim it is.
Jackson-Till Debate

A Year in Review
With this issue we complete our first year of publication. The results have been mixed. We
have received compliments, and we have received denunciations. Admittedly, the
denunciations have exceeded the compliments, but we expected this. We knew when we
decided to begin publication that we would be swimming against a headstrong tide of
tradition based on uncritical thinking, and we were not mistaken. The entrenchment of the
inerrancy belief was indicated to us over and over again in the copies of TSR that came back

Volume 1990 - 2002 Issue


Page 47 of 2049
Skeptical Review Edited by Farrell Till
marked REFUSED and the letters from angry recipients asking that their names be removed
from our mailing list. None of these, incidentally, ever attempted to show that our position
was wrong.

This intolerant attitude was more than compensated for by the reactions we received from
open-minded readers. An early subscriber wrote just recently to say, "I have escaped the
religious scene and am now attempting to enjoy life to the full in a moral way." We hope that
this person's testimony will not escape the notice of those who believe that the rejection of
religious faith leads to unhappiness and moral degradation. There is almost universal
agreement among those who have abandoned religious superstition that they are happier and
better adjusted living as agnostics or atheists. Religious fundamentalists will find this hard to
believe; nevertheless, it is true. Our advice to them is simple: Try it; you might like it.
Countless ones who have tried it unhesitatingly declare that they do like it.

We began publication with an open-door editorial policy through which we intended to offer
our readers the opportunity to read competently written responses to our main articles, but in
this issue we have no rebuttal materials. This does not mean that we have abandoned our
policy; it means only that we could find no one willing to respond to our lead article.
Altogether, we sent copies of the article ("What About Scientific Foreknowledge in the
Bible?"), which begins on page 2, to six different fundamentalist writers and invited each to
prepare a response, but none of them accepted the invitation. In fact, we are late going to
press because we waited beyond our deadline to give a late contact to whom we had faxed the
article an opportunity to write a rebuttal. He later called to tell us he would not write it.

Some of these writers we contacted are involved in their own publication projects, so it isn't
as if writing the response we wanted would have been a new venture for them. They simply
refused the chance to have their response published alongside our article in a format that
would have provided our readers the chance to consider both sides of this issue. We have to
wonder why, especially since the very nature of a rebuttal gives a last-word advantage to the
rebutter.

In a monthly publication that he edits, Wayne Jackson, one of the writers who declined our
invitation, quoted a section of the article that he thought he could handle and then proceeded
to give his readers a one-sided response to it, yet he was unwilling to let his prepared response
to everything in it be seen alongside our entire article.

No head-in-the-sand tactics like these will be pursued in TSR. We will continue to invite and
publish responses, and we challenge our fundamentalist adversaries to do the same in their
publications.

What About Scientific Foreknowledge in


the Bible?

Volume 1990 - 2002 Issue


Page 48 of 2049
Skeptical Review Edited by Farrell Till
Any challenge to the Bible inerrancy doctrine will sooner or later encounter the scientific-
foreknowledge argument. "If the Bible is not the inspired word of God," the inerrancy
spokesmen ask, "then how do you explain the many examples of scientific foreknowledge in
it?" The claim implied in this question is that men writing in an age of relative ignorance
indicated in various passages of the Bible that they understood scientific truths that were
completely unknown at the time. The response the question seeks is that these scientific facts
could not have been known to Bible writers without God's having revealed them during the
verbal inspiration process. They see this as a compelling argument for the inerrancy doctrine.

A basic problem with this argument is the same as the one found in the familiar harmonious-
content, unity-of-theme, and fulfillment-of-prophecy arguments so often presented in the
Bible's defense. It is based more on speculation, imaginative interpretations, and wishful
thinking than on verifiable facts. As I write this, I am engaged in a written debate with a
Church-of-Christ preacher who, in trying to use this argument, threw a volley of speculatively
conceived questions at me in his second affirmative manuscript. How did Moses know of
woman's seed being involved in the conception of children, (Gen. 3:15)? How did Isaiah
know in his day that the earth is round, (Isa. 40:22)? How did Job know that the earth rests on
no material foundation, (Job 26:7)? How did Moses know that life is in the blood (Gen. 9:4),
when medical science didn't know it until a late date? How did David know of the moon's
bearing witness (Ps. 89:37) to the sunlight on the other side of the earth? How did David
know that there are paths in the seas (Ps. 8:8) long before oceanography and Matthew Maury's
work found it so?

These are the questions exactly as he fired them at me. Not once did he take the time to
explicate scripture references to show reasonable proof that the writers meant what he was
interpreting them to mean. He just tacked the references onto his questions as if this alone
were enough to establish that the writers had intended the meanings he was attributing to
them. Any verbal communication, however, whether oral or written, must be interpreted
before it can be understood, and this is doubly true of written statements. Participants in oral
communication enjoy the advantage of voice inflections and body gestures to help them
establish or determine meaning, but this advantage is lost in written communication. Written
statements, then, often require careful explication to determine meaning. Without it, the risk
of misinterpretation increases.

But in the volley of questions listed above, not even a hint of explication was in evidence.
What explication, for example, is involved in asking, How did Moses know of woman's seed
being involved in the conception of children, (Gen. 3:15)"? There is none. The intended
impact of the question depends on two assumptions (aside from the assumption that Moses
wrote the book of Genesis): (1) the word seed in this passage refers to the ovum that the
female contributes to procreation and (2) the existence of the ovum was unknown when
Genesis 3:15 was written.

To assess the plausibility of the first of these assumptions, we must examine the passage that
the question alludes to. After their disobedience to Yahweh's command not to eat of the tree
of the knowledge of good and evil, Yahweh pronounced curses upon all parties involved in
the act. To the serpent, he said, "Because you have done this, cursed are you above all cattle
and above all wild animals; upon your belly you shall go, and dust you shall eat all the days of

Volume 1990 - 2002 Issue


Page 49 of 2049
Skeptical Review Edited by Farrell Till
your life. I will put enmity between you and the woman and between your seed and her seed;
he shall bruise your head, and you shall bruise his heel," (Gen. 3:14-15, RSV).

To assert that the word seed in this passage refers to the ova of the woman is almost too
ridiculous to warrant serious comment. For one thing, an ovum is only a female germ cell that
cannot develop into a person unless it is first fertilized by the male counterpart, so if ova were
the intended meaning of the word, how could the "seed" of the woman ever bruise the head of
the serpent?

The Hebrew word translated "seed" in this passage is zera, which could mean both seed, in
the sense of plant ovules, or posterity (offspring or descendants). It is the same word that was
used several times in Genesis 1:11-12 in reference to the creation of vegetation that yielded
seed after "its kind." The meaning of the word here seems rather obvious; it was a reference to
the seed produced by plants like corn, alfalfa, and turnips. The seed of a plant, however, is
something radically different from the ovum of a woman. A plant seed is actually an embryo
(formed from the union of the male and female germ cells) encased in a shell with an
endosperm that will provide the germinating embryo with food until it is mature enough to
survive on its own. A seed, in other words, is the offspring of a plant. It is to the plant what an
embryo in the womb is to a woman, so certainly a woman's ovum alone cannot be considered
biologically parallel to a plant seed, because it is only half of what a seed is. The one is just a
female germ cell; the other an embryo formed from the union of both the female and male
germ cells.

If we are to understand Genesis 3:15, then, we must think of zera as a Hebrew word that most
often meant offspring. In many places in the book of Genesis alone, it was clearly used in this
sense. Yahweh said to Abram in Genesis 12:7, "Unto thy seed [zera] will I give this land." In
Genesis 13:16, Yahweh promised Abram, "I will make thy seed [zera] as the dust of the
earth." After showing a willingness to sacrifice his son Isaac at Jehovah-jireh, an angel of
Yahweh told Abraham, "I will multiply thy seed [zera] as the stars of the heavens and as the
sand which is upon the seashore; and thy seed [zera] shall possess the gate of his enemies; and
in thy seed [zera] shall all the nations of the earth be blessed," (Gen. 22:17-18).

In these and other passages too numerous to cite, the Hebrew word zera was obviously used
to indicate offspring or descendants. Since this meaning also fits appropriately into the
context of Genesis 3:15, only someone desperate to find support for an indefensible position
would ever feel a need to interpret it as a lesson in modern biology by a primitive writer. Most
English translations, in fact, use offspring or descendants in all of these passages as well as
many others in which the King James and American Standard Versions translated zera as
seed.

If these facts leave any doubt about what the Genesis writer meant in referring to Eve's
"seed," Genesis 16:10 should remove it. In her flight from the wrath of Sarah, Hagar,
Abraham's concubine, was visited by an angel of Yahweh, who promised her, "I will greatly
multiply thy seed [zera], that it shall not be numbered for multitude." In the translations
referred to above, the word descendants is used where seed appears in the KJV and ASV. Yet
if zera meant ova in reference to Eve's "seed" in Genesis 3:15, consistency would require the
proponents of this argument to believe that it also meant ova when referring to Hagar's seed.

Volume 1990 - 2002 Issue


Page 50 of 2049
Skeptical Review Edited by Farrell Till
Hence, we would have an angel of Yahweh promising Hagar that she would produce so many
ova that she wouldn't be able to count them. Such is the predicament that inerrancy
proponents get themselves into when they try to manufacture evidence out of nothing.

Isaiah 40:22 speaks of God who "sitteth above the circle of the earth," but there are many
explicative problems that must be resolved before one can present this as proof that Isaiah
knew the shape of the earth in a time when no one else did. For one thing, how can we be sure
that Isaiah was speaking literally in the passage? He also spoke of "the four corners of the
earth" (11:12), but if I should cite this verse as an example of scientific inaccuracy on the part
of a Bible writer who thought the earth was square, inerrancy advocates would demand proof
that Isaiah had intended literal meaning. By the same token, then, they should be prepared to
prove that Isaiah's reference to the "circle of the earth" was meant literally.

Even if they could successfully do this, they would then have to prove that Isaiah meant circle
in the sense of sphere. Plates and disks are circular in shape as well as spheres, and, as
practically any general encyclopedia will confirm, some ancient cultures before and during
Isaiah's time thought that the earth was a flat disk. To find evidence of scientific
foreknowledge in Isaiah 40:22, then, the inerrancy advocates would have to prove that the
passage referred to a spherical rather than a discoid circle. I seriously doubt that they can ever
do that, but until they do, they have no argument.

The main weakness of this argument, however, is the fact that the shape of the earth was
known in Isaiah's time. In discussing the spherical era of Earth's history, the Encyclopedia
Britannica (Vol. 6, 1978, pp. 1-3) explains that ancient astronomers determined that the earth
was round by observing its circular shadow move across the moon during lunar eclipses. The
Egyptians and Greeks as far back as 2550 B.C. (more than a thousand years before Moses)
knew not only the earth's spherical shape but also its approximate size. The Grecian
philosopher Pythagoras, who was born in 532 B.C., defended the spherical theory on the basis
of observations he had made of the shape of the sun and moon. If this information was
generally known by educated Greeks and Egyptians before and during biblical times, how can
anyone say with certitude that Isaiah couldn't have known about it?

If space allowed, I would explicate the other scriptures mentioned earlier that are often cited
as evidence of scientific foreknowledge in the Bible, but these are enough to demonstrate the
problems that the inerrancy proponents must solve before rational-thinking people can take
their argument seriously. If Pythagoras could observe the sun and the moon and thereby
reason that the earth was also spherical in shape, why couldn't Job have looked at the moon or
the sun and concluded that the earth, like them, was suspended in space on nothing? Why
couldn't Moses, if he was indeed the author of Genesis, have observed that when blood is
drained from the body, life flowed out with it so that in some sense life was "in the blood"?
Just why does this have to mean that Moses knew that blood carries oxygen to cells
throughout the body and thereby sustains life? Why does "paths of the seas" in Psalm 8:8
have to be a reference to ocean currents like the Gulf Stream and the North Atlantic Current?
Why couldn't it just as easily have been a reference to ocean trade routes that the ships of that
time traveled? The Hebrew word orach translated paths in this passage in fact meant
"customary road." And even if it was a reference to currents in the oceans, how can anyone
determine today that knowledge of those currents was completely unknown at that time?

Volume 1990 - 2002 Issue


Page 51 of 2049
Skeptical Review Edited by Farrell Till
Simply because it isn't now known that it was known doesn't prove that it wasn't known. So
inerrancy proponents aren't the only ones who can ask questions. Those of us who reject the
inerrancy doctrine have a lot of questions to ask too, especially on this matter of alleged
scientific foreknowledge in the Bible.

Like so much of the other "evidence" that Bible fundamentalists offer as proof of the
inerrancy doctrine, they see scientific foreknowledge in the Bible only because they so
desperately want to see something that can form a rational basis for their faith. In the same
way, they see prophecies and their fulfillments in passages so obscurely written that no one
can really determine what the writers originally intended in the statements. In the face of
unequivocal inconsistencies and contradictions in the Bible text, they see unity of theme
because they so desperately want to see unity of theme.

This approach to Bible interpretation has at times caused them major embarrassment. In 1939,
for example, George DeHoff wrote a biblical apology entitled Why We Believe the Bible. An
entire chapter was devoted to the scientific-foreknowledge argument in which he cited Job
26:7 as supporting evidence, (p. 50):

Astronomers have discovered that there is a great empty space in the North. It contains no
moving planets and shining stars. By turning their telescopes to the South, the East and the
West, men may behold countless millions of stars invisible to the naked eye but when the
telescope is set exactly to the North there is a great empty space. For this, astronomers have
been unable to account. They did not know until recently that there was such an empty space,
yet Job declared, "He stretcheth out the North over the empty places [sic] and hangeth the
earth upon nothing," (Job 26:7).

DeHoff's conclusion was that "Job could not have written by guess. It must be that he wrote
by inspiration of God."

For years, this scripture was cited from Church-of-Christ pulpits as compelling evidence that
the Bible was divinely inspired, but there was just one thing wrong with it. The premise on
which it was based wasn't true. There is no "empty place" in our northern space. Everywhere
astronomers look, they find space filled with galaxies and stars. That includes our northern
space too. So wherever DeHoff got this argument, he didn't get it from science, and he will
find no support for it in scientific circles.

Inerrancy advocates in the Churches of Christ are now admitting that they erred in using Job
26:7 as an example of scientific foreknowledge in the Bible. In the September 1989 issue of
Reason & Revelation, Dr. Bert Thompson summarized the traditional DeHoffian
interpretation of Job 26:7 and then said this, (p. 35):

This writer has so used the verse himself in the past, but does so no longer, because of
problems associated with such interpretations. For example, if we attempt to convince people
that this verse is to be taken literally, how do we then consistently deal with statements in the
chapter which are obviously figurative (such as verse 11: "The pillars of heaven tremble, and
are astonished at his reproof")? Further, there seems to be no empty space in the north.

Volume 1990 - 2002 Issue


Page 52 of 2049
Skeptical Review Edited by Farrell Till
Instead, "billions of stars and galaxies extend outward in all directions," (Donald B.
DeYoung, Astronomy and the Bible).
We congratulate Dr. Thompson for finally recognizing an obvious flaw in a popular inerrancy
argument. It gives us hope that he might someday see the flaws in other inerrancy arguments
too.

Something that has long perplexed me is the way that inerrancy proponents can so easily find
"scientific foreknowledge" in obscurely worded Bible passages but seem completely unable to
see scientific error in statements that were rather plainly written. There are too many to
discuss, but Leviticus 11:5-6 can serve as an example. Here "Moses," after having identified
clean animals as those that "chew the cud and part the hoof," said, "And the coney, because he
cheweth the cud but parteth not the hoof, he is unclean unto you. And the hare, because she
cheweth the cud but parteth not the hoof, she is unclean unto you." Deuteronomy 14:7 also
described the hare and the cony as cud-chewers, but in reality they are not. They do not have
compartmentalized stomachs that ruminants must have in order to be cud-chewers. Inerrancy
champions have stumbled over these passages with various attempts to explain them. Gleason
Archer justifies the classification of hares and conies as cud-chewers on the grounds that they
"give the appearance of chewing their cud in the same way ruminants do," (Encyclopedia of
Bible Difficulties, p. 126). Yet after all has been said on the matter, the fact remains that hares
and conies are not cud-chewers. But "Moses" said that they were.

One would think that if God were going to arm his inspired writers with scientific
foreknowledge about complex matters like the "seed of woman" and the shape of the earth, he
could have easily programmed them to know the simple fact that hares and conies aren't cud-
chewers. That he didn't reveal this to them, as well as other things, certainly doesn't help the
scientific-foreknowledge argument.

The Jeremiah Dilemma


Farrell Till
Before the discovery of the Dead Sea Scrolls, the oldest known manuscripts of the Old
Testament dated from 895 A.D., a fact that raised significant questions about the integrity of
the Masoretic text. Since tradition assigned the authorship of some Old Testament books to
writers who lived as long ago as the 15th century B.C., faith in Bible inerrancy required one
to believe that uninspired scribes had somehow copied these books for more than 2,000 years
without infusing significant error into the texts. That being too much for some to accept, the
inerrancy doctrine lost a lot of followers. Liberal minded Christians began to think of the
Bible as a sacred book whose ideas had been divinely inspired but not necessarily the very
words, not even the words in the long defunct "original autographs" that inerrancy spokesmen
like to talk about.

Volume 1990 - 2002 Issue


Page 53 of 2049
Skeptical Review Edited by Farrell Till
With the discovery of the Dead Sea Scrolls, believers in the inerrancy doctrine thought they
had found cause to rejoice. In Cave One at Qumran was found a manuscript of the book of
Isaiah containing all 66 chapters except for only a few words that were missing where edges
of the scroll had crumbled. Although many spelling variations were found in the text, the
content of the Qumran scroll was found to be remarkably parallel to the Masoretic text of 895
A. D. Translators of the Revised Standard Version in 1952 found only 13 textual differences
in the manuscript that they considered important enough to affect their translation of Isaiah.
When scholars dated the manuscript at circa 100 B.C., Bible fundamentalists believed they
had found in the Qumran text of Isaiah indisputable proof that through the long, silent
centuries Jewish scribes had been scrupulously faithful in transmitting their sacred books.
After all, if a thousand years had brought no significant changes to the text of Isaiah, couldn't
we believe that the same was true of the other Old Testament books?

This would make an impressive argument were it not for subsequent discoveries that were
made at Qumran, which Bible inerrantists have been very reluctant to talk about. In
commenting on these other discoveries, Joseph A. Fitzmyer, professor emeritus of New
Testament at The Catholic University of America, dashed cold water onto the hopes of those
who had hastily concluded too much from the Qumran text of Isaiah:

In Cave 4, 157 fragmentary biblical texts were retrieved, among which is every book of the
Hebrew canon, save Esther (and Nehemiah, which at that time was considered as one book
with Ezra). Eventually, these Cave 4 fragments revealed a different story about the copying
and transmission of Old Testament writings. In some cases, especially 1-2 Samuel, Jeremiah,
and Exodus, the fragments brought to light forms or recensions of biblical books that differed
from the medieval Masoretic tradition. For instance, one text turned out to be a shorter
Hebrew form of Jeremiah, previously known only in its Greek version in the Septuagint. It
now seems that the fuller form of Masoretic tradition represents a Palestinian rewording of the
book. Another from Cave 4, written in paleo-Hebrew script and dated from the early second
century B.C., contains the repetitious expanded form of Exodus previously known only in
Samaritan writings, ("The Dead Sea Scrolls and the Bible: After Forty Years," America,
October 31, 1987, p. 302, emphasis added).

This "different story" told by the discoveries in Cave Four at Qumran is a story that Bible
inerrantists have been conspicuously silent about, probably because it puts to rest all notions
of scrupulously meticulous ancient scribes who counted all the letters in the copies they made
to be sure that no mistakes had occurred. The Cave Four discoveries tell us that mistakes were
not only made but that textual changes were also made with probable deliberation.

A single article can't review all parts of the "different story" told by the Cave Four
discoveries, but an analysis of the Jeremiah manuscript will be sufficient to refute the claim
that we can be reasonably sure the present day text of the Bible is essentially the same as what
was in the "original autographs." Scholars had long known before the Cave Four discoveries
at Qumran that the Masoretic text of Jeremiah differed substantially from the Greek version
found in the Septuagint. Some sections of the Masoretic text were missing entirely from the
Septuagint, and other sections were organized differently. Jeremiah 27:19-22; 33:14-26; 39:3-
14; and 48: 45-47 are sections in the Masoretic and various English texts that were not in the
Septuagint version. The organizational restructuring is too complex to discuss in detail, but

Volume 1990 - 2002 Issue


Page 54 of 2049
Skeptical Review Edited by Farrell Till
some thirty changes in organization have been identified in the Septuagint version. Chapter
25:15-38 of the Masoretic text appears as chapter 32 in the Septuagint, 27:1-19 is chapter 34,
33:1-14 is chapter 40, and so on through more than thirty other changes in organization.

To explain the problem posed by these variations in the Septuagint version of Jeremiah,
proponents of the inerrancy doctrine once attributed the deviations from the Masoretic text to
poor translation, but after the discoveries in Cave Four, this "explanation" became hard, if not
impossible, to defend. Work on the Septuagint version began in Alexandria around 285 B.C.,
and the Jeremiah manuscript found at Qumran, like the Isaiah scroll, was dated in the early
second century B. C. Since the Qumran text of Jeremiah was parallel in content and
organization to the Septuagint version, here was tangible evidence that at one time, for at least
two centuries, a shorter, differently arranged version of the book existed. Hence, variations
from the Masoretic text in the Septuagint version of Jeremiah resulted not from careless
translation but from a radically different Hebrew text that the translators had before them.
More interested in scholarship than the defense of pet theories, Fitzmyer said this about the
Cave Four discoveries:

Such ancient recensional forms of Old Testament books bear witness to an unsuspected
textual diversity that once existed; these texts merit far greater study and attention than they
have been accorded till now. Thus, the differences in the Septuagint are no longer considered
the result of a poor or endentious attempt to translate the Hebrew into the Greek; rather they
testify to a different pre-Christian form of the Hebrew text, (Ibid., p. 302, emphasis added).

Because of the damage these facts inflict on the inerrancy doctrine, Bible fundamentalists
will, of course, resist the obvious conclusion that they lead to, but until the inerrantists can
produce a Masoretic copy of Jeremiah that antedates the Septuagint, they will find it hard to
defend their claim that the Bible text we now have is essentially the same as what was written
in the "original autographs."

The sections missing from the Septuagint and Qumran versions of Jeremiah clearly testify to
what Fitzmyer called "a Palestinian reworking of the book." Let's consider, for example, the
following omission:

Behold, the days come, saith Yahweh, that I will perform that good word which I have spoken
concerning the house of Israel and concerning the house of Judah. In those days, and at that
time, will I cause a Branch of righteousness to grow up unto David; and he shall execute
justice and righteousness in the land. In those days shall Judah be saved, and Jerusalem shall
dwell safely; and this is the name whereby she shall be called: Yahweh our righteousness. For
thus saith Yahweh: David shall never want a man to sit upon the throne of the house of Israel;
neither shall the priests the Levites want a man before me to offer burnt offerings, and to burn
meal-offerings, and to do sacrifice continually.

And the word of Yahweh came unto Jeremiah, saying, Thus saith yahweh: If ye can break my
covenant of the day, and my covenant of the night, so that there shall not be day and night in
their season; then may also my covenant be broken with David my servant, that he shall not
have a son to reign upon his throne; and with the Levites the priests, my ministers. As the host

Volume 1990 - 2002 Issue


Page 55 of 2049
Skeptical Review Edited by Farrell Till
of heaven cannot be numbered, neither the sand of the sea measured; so will I multiply the
seed of David my servant, and the Levites that minister unto me.

And the word of Yahweh came to Jeremiah, saying, Considerest thou not what this people
have spoken, saying, The two families which Yahweh did choose, he hath cast them off? thus
do they despise my people, that they should be no more a nation before them. Thus saith
Yahweh: If my covenant of day and night stand not, if I have not appointed the ordinances of
heaven and earth; then will I also cast away the seed of Jacob, and of David my servant, so
that I will not take of his seed to be rulers over the seed of Abraham, Isaac, and Jacob: for I
will cause their captivity to return, and will have mercy on them, (33:14-26, ASV with
Yahweh substituted for Jehovah).

Obviously intended as a repetition of Yahweh's promise to establish an eternal, perpetual


throne of David over the house of Israel, which promise was first proclaimed in II Samuel
7:12-17, this passage, and ones like it, have proved embarrassing to God's people ever since
the vagaries of history reduced the Yahwistic promises of an everlasting Israelite kingdom to
mere ethnocentric wishes that didn't materialize. To protect the inerrancy doctrine, Bible
fundamentalists have been forced to read figurative meaning into these statements, so rather
than a literal promise to establish David's throne forever, they see this passage, and others like
it, as a Messianic prophecy that was fulfilled in Jesus Christ. Either way, the passage concerns
a central biblical theme and must therefore be considered important, yet it was in neither the
Septuagint version nor the Jeremiah scroll found at Qumran. These omissions have grave
implications for the inerrancy doctrine, because they suggest that significant editing occurred
in at least one Old Testament book after completion of the original manuscript. So what
exactly are we to conclude from this? After verbally inspiring Jeremiah to write his
manuscript, did Yahweh decide he could improve on the original and then direct someone to
reorganize the material and insert the passages that weren't available to the Septuagint
translators or to the scribe who made the Qumran copy? If so, what does this say about the
omniscience of Yahweh that we hear so much about? Or if the changes didn't happen under
Yahweh's direction, did some scribe or committee of scribes just take it upon themselves to do
the editing? Either way again, the proponents of Bible inerrancy have a serious problem on
their hands. They preach a doctrine that simply cannot be squared with known facts.

Questions raised by the Qumran discoveries pose still other problems for the inerrancy
doctrine. A principle of lower criticism states that the older a manuscript is, the more likely its
content will parallel the original version. If this is so, now that we have the Quamran version,
which is a thousand years older than the Masoretic text, to confirm the probable accuracy of
the Septuagint version, what should be done about the book of Jeremiah? Should we scrap the
version that has been published for centuries in English Bibles and substitute the Septuagint
version? If so, what should be done about the past mistake of publishing a corrupted version
of one of God's inspired books? Should Bible fundamentalists simply say, "Oops, sorry about
that," and go on proclaiming the inerrancy doctrine as if nothing had happened? Or should
they just keep the problem swept under the rug and pretend that it doesn't exist? Since they
have now had 40 years to react to the discovery and have done nothing, I suspect they will
choose to keep it swept under the rug. After all, what ignorant pulpit audiences, who probably
have never even heard of Qumran, don't know won't hurt them.

Volume 1990 - 2002 Issue


Page 56 of 2049
Skeptical Review Edited by Farrell Till
Even without the discovery of the Jeremiah scroll at Qumran, the variations between the
Septuagint version of Jeremiah and the Masoretic text posed problems that Bible inerrantists
should have addressed long ago. The New Testament writers, who were presumably guided
by the Holy Spirit in what they wrote, frequently quoted the Septuagint translation when
citing Old Testament scriptures. If these writers were indeed guided by the Holy Spirit as they
composed the New Testament, one would assume that their use of the Septuagint for scripture
references was done not only by the approval of the Holy Spirit but by his explicit direction.
To say the least, then, this would appear to put a stamp of divine approval on the Septuagint
Bible. Why then have Bible inerrantists said little or nothing about the variations between the
divinely approved Septuagint version of Jeremiah and the longer, differently organized
version in the Bibles that they preach their sermons from? Surely this was an incongruity
important enough to warrant an explanation.

The reliance of New Testament writers on the Septuagint scriptures poses still another
problem that goes far beyond variations in the Jeremiah text. That problem concerns the
quality of the Septuagint translation in general. The discovery of the Qumran text of Jeremiah
may have quelled notions that variations from the masoretic text in the Septuagint version of
Jeremiah were primarily due to poor translation, but scholars nevertheless agree that many
sections of the Septuagint were carelessly translated. My own copy of the Septuagint says this
in the introduction:

The variety of the translators is proved by the unequal character of the version: some books
show that the translators were by no means competent to the task, while others, on the
contrary, exhibit on the whole a careful translation. The Pentateuch is considered to be the
part the best executed, while the book of Isaiah appears to be the very worst, (The Septuagint
Version of the Old Testament, Zondervan Publishing House: 1970, p. iii).

The Eerdmans Bible Dictionary gives a similar assessment of translation accuracy in the
Septuagint:

Examination of the text, however, indicates a combination of numerous versions both literal
and free and marked by considerable variance in style, interpretation of the Hebrew, and even
order and contents; the latter suggests a variety of underlying Hebrew texts. The Greek-
speaking authors of the New Testament quoted from the LXX (Septuagint) rather than the
Hebrew text, and the LXX became their authoritative scriptures. Its use by Christians for
proselytizing and in anti-Jewish polemics, as well as the growing Jewish dissatisfaction with
the LXX for being too loose a translation and not based on the current authoritative text (it
varied also from the order of the Hebrew canon), led to the more literal translations of Aquila
(A. D. 130), Theodotion, and the Ebionite Christian Symmachus, (Eerdmans Publishing Co.,
1987, p. 154, emphasis added).
As indicated in an early edition of Scribner's Dictionary of the Bible, the problems of poor
translation in the Septuagint and significant variations from the Masoretic text in manuscripts
that the translators had to work with were generally known even before the Qumran
discoveries:
The second fact that comes to light from a comparison of G (Septuagint) and M (Masoretic) is
that there is a great difference between particular books or sets of books in the OT. This arises
partly from the circumstances that all the books are not due to the same translators, but still

Volume 1990 - 2002 Issue


Page 57 of 2049
Skeptical Review Edited by Farrell Till
more from the different character of the text lying before them. That Isaiah, for instance,
found an interpreter not worthy of this book, was remarked long ago by Swingli; the translator
of Job, says Swete, p. 316, was perhaps more familiar with Greek pagan literature than with
Semitic poetry.... But more important is the fact that already the Hebrew texts used by the
translators differed in varying degrees from the Massoretic text, (1923, Vol. IV, p. 449,
emphasis added).
Incompetent translation and significant variations from the allegedly inspired Masoretic text--
these are serious charges against the Septuagint version that must be resolved if we are to
believe that the Holy Spirit guided the writers of the New Testament to quote the Septuagint
whenever scripture citations were deemed necessary. Anyone can easily verify the divergent
readings between the Septuagint and Masoretic texts by merely using a reference Bible that
will identify the sources of quotations used by the New Testament writers. In Matthew 15:8-9,
for example, Isaiah 29:13 was quoted from the Septuagint Bible: "This people honoreth me
with their lips; But their heart is far from me. But in vain do they worship me, Teaching as
their doctrines the precepts of men." This has been an often quoted statement in Church-of-
Christ sermons aimed at establishing the need of scriptural authority for the structure of
Christian worship. The only problem is that the statement, although faithful to Isaiah 29:13 in
the Septuagint version, is barely recognizable in the Masoretic text. This is readily apparent in
the following juxtaposition of the two versions:
And the Lord has said, This people draw nigh to me with their mouth, and they honor me with
their lips, but their heart is far from me: but in vain do they worship me, teaching the
commandments and doctrines of men, (Septuagint).

And the Lord said, Forasmuch as this people draw nigh unto me, and with their mouth and
with their lips do honor me, but have removed their heart far from me, and their fear of me is
a commandment of men which hath been taught them... (Masoretic).

There are similarities in the first half of each text, but the last halves are markedly different.
Nothing is said in the Masoretic text (which inerrantists say is so close to the "original
autographs" as to make variations irrelevant) about the vanity of worship that is taught after
the commandments and doctrines of men. If this was not in the Masoretic (original?) text,
why did the Holy Spirit guide Matthew to quote it this way? Inerrantists need to explain this.

Another variant reading, among many that I could cite, is the quotation of Psalm 40:6-8 in
Hebrews 10:5-7: "Wherefore when he cometh into the world, he saith, Sacrifice and offering
thou wouldest not, But a body didst thou prepare for me; In whole burnt offerings and
sacrifices for sin thou hadst no pleasure: Then said I, Lo, I am come (In the roll of the book it
is written of me) to do thy will, O God." This quotation is very parallel to the Septuagint
rendering:

Sacrifice and offering thou wouldest not; but a body hast thou prepared me: whole-burnt-
offering and sacrifice for sin thou didst not require. Then I said, Behold, I come: in the
volume of the book it is written concerning me, I desired to do thy will, O my God...."
There is a significant difference, however, in the Masoretic text:
Sacrifice and offering thou hast no delight in; Mine ears hast thou opened: Burnt offering and
sin offering hast thou not required. Then said I, Lo, I am come; In the roll of the book it is
written of me: I delight to do thy will, O my God....

Volume 1990 - 2002 Issue


Page 58 of 2049
Skeptical Review Edited by Farrell Till
What happened to the body that was prepared in the Septuagint account quoted by the
Hebrew writer? It isn't in the Masoretic text, yet we are supposed to believe that the Masoretic
text and the "original autographs" are essentially one and the same. If the two are indeed
essentially the same, why did the Holy Spirit guide the Hebrew writer to quote a corrupted
version of this particular Psalm?

These are legitimate questions to ask in view of the Hebrew writer's application of the
statement present in the Septuagint but missing from the Masoretic text. The reference to the
"body [that] thou didst prepare for me" (v:5) is obviously crucial to his point in verse ten
about Christians having "been sanctified through the offering of the body of Jesus Christ once
for all," but, as already noted, that statement is not in the Masoretic text. So if the Masoretic
text and the "original autographs" are essentially the same, as the inerrantists claim, then the
"inspired" psalmist never said what the Hebrew writer said that he said. Anyone who can't see
the problem this poses for the inerrancy doctrine doesn't want to see it.

In a real sense, then, the Jeremiah dilemma is actually (See JEREMIAH, p. 12) the Septuagint
dilemma, for what is true of the Septuagint version of Jeremiah is generally true of its
versions of the other Old Testament books. They are characterized by faulty translation and
significant variations from the Masoretic text. Skeptics of the inerrancy doctrine have every
right, then, to ask its believers to explain why the Holy Spirit chose a flawed version of the
Old Testament as his primary source of scripture quotations in the writing of the New
Testament. And what should be our position relative to significant variations between the
Septuagint and Masoretic texts, as in the examples cited above? Are we to believe that the
Septuagint was verbally inspired and the Masoretic wasn't? Or should we believe the
Masoretic was inspired and the Septuagint wasn't? Either choice poses major problems for
inerrancy proponents. If they go with the Holy Spirit and choose the Septuagint as the
"verbally inspired" version, they must explain why they have relied for so long on the
Masoretic as their primary textual source. If, on the other hand, they choose the Masoretic,
then they return us to where we started. How do they explain why the Holy Spirit directed
New Testament writers to quote the uninspired Septuagint?

These are questions begging for answers. Perhaps some enterprising inerrantist among our
readers can give us the answers.

Textual Contradictions in the Bible


Is the Bible the work of perfect harmony that inerrancy believers claim it is? Suppose we let
the Bible speak for itself and see what answer we get.

I Kings 6:1 says that work on the temple began 480 years after the exodus from Egypt: "And
it came to pass in the four hundred and eightieth year after the children of Israel were come
out of the land of Egypt, in the fourth year of Solomon's reign over Israel, in the month Ziv,
which is the second month, that he began to build the house of Jehovah (Yahweh)."

Volume 1990 - 2002 Issue


Page 59 of 2049
Skeptical Review Edited by Farrell Till
But the Apostle Paul made a speech in Antioch of Pisidia in which his math contradicted this
statement: "The God of this people Israel chose our fathers, and exalted the people when they
sojourned in the land of Egypt, and with a high arm led he them forth out of it. And for about
the time of forty years as a nursing-father bare he them in the wilderness. And when he had
destroyed seven nations in the land of Canaan, he gave them their land for an inheritance, for
about four hundred and fifty years: and after these things he gave them judges until Samuel
the prophet. And afterward they asked for a king: and God gave unto them Saul the son of
Kish, a man of the tribe of Benjamin, for the space of forty years. And when he had removed
him, he raised up David to be their king; to whom also he bare witness and said, I have found
David the son of Jesse, a man after my heart, who shall do all my will," (Acts 13:17-22).

With nothing else considered, the 40 years in the wilderness and the 450 years that the
Israelites had the land of Canaan for an inheritance before the advent of the judges total 10
years more than the 480 years of I Kings 6:1. Eerdmans Bible Dictionary states that the period
of the judges "could not reasonably be reduced to less than 280 years," (p. 610). Saul, as Paul
noted, reigned as king for 40 years, as did also David who succeeded him (I Kings 2:11). So if
we add the four years that Solomon reigned before work on the temple began, we have 40 +
450 + 280 + 40 + 40 + 4, for a total of 854, a significant variation from the 480 years claimed
in I Kings 6:1. Even if we let Paul's 450 years for the inheritance of Canaan include also the
advent of the judges, as some translations strain to do, his chronology will still total 574
years, almost a century longer than what was claimed in I Kings 6:1.

I Chronicles 2:13-15 in listing the sons of Jesse says that David was the seventh. Yet I Samuel
16:10-11 states that David was Jesse's eighth son: "And Jesse made seven of his sons to pass
before Samuel. And Samuel said unto Jesse, Jehovah hath not chosen these. And Samuel said
unto Jesse, Are here all thy children? And he said, There remaineth yet the youngest, and,
behold, he is keeping the sheep."

Joshua 17:18 promised the Israelites that they would "drive out the Canaanites though they
have chariots of iron, and though they are strong." Yet Judges 1:19 states that Judah's assault
against Canaanites in the lowlands failed because they were equipped with iron chariots:
"And Jehovah was with Judah; and he drove out the inhabitants of the hill-country; for he
could not drive out the inhabitants of the valley, because they had chariots of iron."

Numbers 3 and 4 describe the separation of the Levites for the priesthood and temple service
(including care of the ark of testimony) while the Israelites were camped at Mt. Sinai in the
second year after the exodus, but Deuteronomy 10:7-8 claims that the separation of the
Levites occurred at a place called Jotbathah: "From thence they journeyed unto Gudgodah;
and from Gudgodah to Jotbathah, a land of brooks of water. At that time Jehovah set apart the
tribe of Levi, to bear the ark of the covenant of Jehovah, to stand before Jehovah to minister
unto him, and to bless in his name, unto this day." Verse 6 records the death of Aaron, who
died "in the fortieth year after the children of Israel were come out of the land of Egypt"
(Num. 33:38); hence, this passage also claims that the separation of the Levites took place 38
years after the account in Numbers 3 and 4.

No contradictions in the Bible? It makes good sermon material, but it just isn't so. Only the
gullible will believe it.

Volume 1990 - 2002 Issue


Page 60 of 2049
Skeptical Review Edited by Farrell Till

The Jackson-Till Debate


Farrell Till and Bill Jackson, a Church-of-Christ preacher from Austin, Texas, have
completed a written debate on the Bible inerrancy doctrine. The manuscripts are being
published by the Southwest Church of Christ, 8900 Manchaca Road, Austin, TX 78748 and
will soon be available at $2.25 per copy, postpaid.

This debate clearly exposes the absurdity of the inerrancy doctrine. We enthusiastically
recommend it. All orders should be directed to the above address.

Volume 1990 - 2002 Issue


Page 61 of 2049
Skeptical Review Edited by Farrell Till

The Skeptical Review


Volume Two - 1991
Farrell Till, editor

• Number 1 Volume Two, Issue One


• Number 2 Volume Two, Issue Two
• Number 3 Volume Two, Issue Three
• Number 4 Volume Two, Issue Four

Volume 1990 - 2002 Issue


Page 62 of 2049
Skeptical Review Edited by Farrell Till

The Skeptical Review


Volume Two, Number One - 1991
Farrell Till, editor

The Jannes-Jambres Syndrome


How did Paul know the names of the magicians who opposed Moses (2 Tim. 3:8)?
Farrell Till shows that Paul knew their names from a widely known oral and written
tradition and that, therefore, this verse hardly constitutes evidence for the inspiration
of the Bible.
Yahweh's Failed Land Promise
"On several occasions prophetic statements were made in the Pentateuch about the
land that Yahweh, the tribal god of the Israelites, had promised to Abraham, Isaac, and
Jacob." According to Farrell Till, the Old Testament evidence shows that Yahweh
broke his promise.
Possession of the Promised Land
Charles Coats gives a Christian response to the above article, contending that Yahweh
did not break his promise. According to Coats, Yahweh's promise was a conditional
promise and the Israelites had not lived up to the conditions of his promise.
Scientific Boo-Boos in the Bible
Scientific mistakes in the Bible prove it is not inerrant.
Inerrancy Debates in Print
Deaver-Till Debate
Confidentiality Guaranteed

The Jannes-Jambres Syndrome


Is there no limit to what bibliolaters will resort to in order to defend the inerrancy doctrine?
We thought we had heard just about every desperate grasping for straws possible on this
subject until we received the October 1990 issue of the Christian Courier. On a front-page
article entitled "The God of Infinite Knowledge," editor Wayne Jackson, who is also a staff

Volume 1990 - 2002 Issue


Page 63 of 2049
Skeptical Review Edited by Farrell Till
writer for Apologetics Press of Montgomery, Alabama, made this incredible attempt to prove
the inspiration of the Bible:
How did Paul know the names of the magicians who opposed Moses (2 Tim. 3:8)? This
information is nowhere found in the Old Testament. Obviously the God of history inspired the
writing of this epistle to Timothy.
The passage Mr. Jackson alluded to in this "argument" says in its entirety, "And even as
Jannes and Jambres withstood Moses, so do these (ungodly men) also withstand the truth;
men corrupted in mind, reprobate concerning the truth."

As invariably happens when a bibliolater formulates an inerrancy argument, Mr. Jackson


assumes far more than his "evidence" warrants. For example, Paul (or whoever wrote 2
Timothy) identified Jannes and Jambres only as opposers of Moses; he did not specify that
they were the magicians who had opposed Moses. Since Moses (if we are to believe the
Bible) was often opposed during the wilderness trek by unnamed adversaries, one might as
well, in the total absence of any historical record of who Jannes and Jambres were, identify
them as the leaders of one of those many rebellions as to say that they were pharaoh's
magicians who opposed Moses during the inflicting of the plagues. If not, why not? We will
gladly give Mr. Jackson space in our next issue to explain how he knows that Paul meant for
us to understand that these men were pharaoh's magicians rather than some other adversaries
of Moses.

If he accepts the invitation, he should be able make a good case for his claim that Paul was
referring to pharaoh's magicians, but in building that case, he would reduce his "argument" for
inspiration to nothing. As Mr. Jackson correctly said, the names Jannes and Jambres were
"nowhere found in the Old Testament," but there was a widely circulated tradition in both
secular and apocryphal writings that they were pharaoh's magicians. They were mentioned in
The Gospel of Nicodemus, The Acts of Peter, and The Acts of Paul and were frequently
referred to by early writers like Pliny, Apuleius, and Numenius. In these writings, various
claims were made about them. One source (Yalkut Reubeni) said that they were proselytized
and left Egypt in the Hebrew exodus; another (Tikkunim) claimed that they persuaded Aaron
to make the golden calves while Moses was on Mt. Sinai. In Old Testament Pseudepigrapha,
James H. Charlesworth, a leading authority on apocryphal literature, said this of Jannes and
Jambres:

The names of Jannes and Jambres appear with considerable frequency in ancient and
medieval sources, and traditions about their activity and fate are extant in Hebrew, Aramaic,
Syriac, Arabic, Greek, and Latin. Christian, Jewish, and pagan writers found occasion to refer
to these two magicians at the Pharaonic court who plied the art of magic in opposition to
Moses....

It is now beyond doubt that in antiquity there existed, on the one hand, traditions about Jannes
and Jambres, and on the other, a book that detailed some of their exploits. Not yet entirely
clear, however, is the precise relationship between the loose traditions and the written
composition (Vol. 2, p. 427).

Obviously, then, a widely known oral and written tradition that Jannes and Jambres were
pharaoh's magicians existed before and during the time that 2 Timothy was written. To say

Volume 1990 - 2002 Issue


Page 64 of 2049
Skeptical Review Edited by Farrell Till
that a simple reference to this tradition constitutes wonderful proof that the Bible was inspired
of God is typical of the superficial thinking that characterizes most arguments used to defend
the inerrancy doctrine. Upon careful scrutiny, they are invariably found to be empty of
substance.

Even in the absence of the body of tradition that identified Jannes and Jambres as pharaoh's
magicians, there would still be no proof of divine inspiration in 2 Timothy 3:8. As noted
earlier, without tradition to aid in interpreting this passage, one could never know if the writer
was referring to pharaoh's magicians or to two of the many other adversaries Moses had to
confront as leader of the exodus. Furthermore, without the tradition, the writer could have
whimsically used just any names, and there would have been no criterion to use in evaluating
the accuracy of the information. If, for example, a writer should say that Jeeter and Joomer
were the leaders of the rebellion against Moses at Meribah (Num. 20:2-13), would this prove
(since the names of these leaders are "nowhere found in the Old Testament") that the writer
was inspired of God in so saying or would it prove nothing more than maybe he had just made
up the names? We'll just let Mr. Jackson and those who may have been impressed with his
argument try to escape the cutting edge of Occam's razor on this point.

The real tragedy in this matter, however, is not that bibliolaters like Mr. Jackson would have
no more intellectual pride than to offer such superficial arguments as this one in support of the
inerrancy doctrine but that so many of their readers will gullibly accept them without critical
analysis. It is a syndrome that enables Bible fundamentalism to survive in an era that should
have laid the inerrancy doctrine to rest long ago.

Yahweh's Failed Land Promise


Farrell Till
In their desperate efforts to prove that the Bible was verbally inspired of God, inerrancy
believers often point to prophecy fulfillment. In my debate with Bill Jackson, he referred to
"multiplied dozens of Old Testament prophetic utterances, fulfilled in minute detail in the
New Testament, and in such a manner that there could be no contrivance at all," (Jackson-Till
Debate, p. 3). As is true of all who use the prophecy-fulfillment argument, Jackson could only
claim "multiplied dozens" of prophecy fulfillments; he could not cite a single verifiable
example of a fulfilled OT prophecy.

As I said in the debate, the "prophecy fulfillments" that are invariably cited in support of this
argument never actually "happened except in the fertile imaginations of a few religious
mystics whose fanciful interpretations of certain events have been swallowed hook, line, and
sinker by gullible people like our Mr. Jackson," (Jackson-Till Debate, p. 17). When logical
analysis is applied to these alleged instances of prophecy fulfillment, it quickly becomes
obvious that there is no real evidence of fulfillment. Time would fail me if I tried to analyze

Volume 1990 - 2002 Issue


Page 65 of 2049
Skeptical Review Edited by Farrell Till
the many alleged prophecy fulfillments that inerrantists have pointed to, so instead I will
concentrate on a failed prophecy that they never say much about.

On several occasions prophetic statements were made in the Pentateuch about the land that
Yahweh, the tribal god of the Israelites, had promised to Abraham, Isaac, and Jacob. These
were clearly stated promises that Yahweh would give the land of the Hittites, Girgashites,
Amorites, Canaanites, Perizzites, Hivites, and Jebusites to the seed of Abraham. In
Deuteronomy 7:17-24, for example, Yahweh presumably made this emphatic promise:

If thou shalt say in thy heart, These nations are more than I; how can I dispossess them? Thou
shalt not be afraid of them: thou shalt well remember what Yahweh thy God did unto
Pharaoh, and unto all Egypt; the great trials which thine eyes saw, and the signs, and the
wonders, and the mighty hand, and the outstretched arm, whereby Yahweh thy God brought
thee out: so shall Yahweh thy God do unto all the peoples of whom thou art afraid. Moreover
Yahweh thy God will send the hornet among them, until they that are left, and hide
themselves, perish from before thee. Thou shalt not be affrighted at them; for Yahweh thy
God is in the midst of thee, a great God and a terrible. And Yahweh thy God will cast out
those nations before thee by little and little: thou mayest not consume them at once, lest the
beasts of the field increase upon thee. But Yahweh thy God will deliver them up before thee,
and will discomfit them with a great discomfiture, until they be destroyed. And he will deliver
their kings unto thy hand, and thou shalt make their name to perish from under heaven: there
shall no man be able to stand before thee, until thou have destroyed them," (ASV with
Yahweh substituted for Jehovah).
The substance of this prophecy was repeated in such places as Exodus 23:20-33; Deut. 4:33-
39, Deut. 7:1-2, and Deut. 31:1-8. In some of these passages, the names of the "seven nations
greater and mightier than thou" to be driven out of the land were also specified as they were
above: the Amorites, the Canaanites, the Girgashites, the Hittites, the Hivites, the Jebusites,
and the Perizzites.

When Joshua assumed the leadership of Israel after the death of Moses, the land promise was
renewed in very specific terms:

Now it came to pass after the death of Moses the servant of Yahweh that Yahweh spake unto
Joshua the son of Nun, Moses' minister, saying, Moses my servant is dead; now therefore
arise, go over this Jordan, thou, and all this people, unto the land which I do give to them,
even to the children of Israel. Every place that the sole of your foot shall tread upon, to you
have I given it, as I spake unto Moses. From the wilderness, and this Lebanon, even unto the
great river, the river Euphrates, all the land of the Hittites, and unto the great sea toward the
going down of the sun, shall be your border. There shall not any man be able to stand before
thee all the days of thy life; as I was with Moses, so I will be with thee; I will not fail thee, nor
forsake thee. Be strong and of good courage; for thou shalt cause this people to inherit the
land which I sware unto their fathers to give them, (Joshua 1:1-6, ASV, Yahweh substituted).
Just before crossing the Jordan, Joshua repeated the promise:
And Joshua said unto the children of Israel, Come hither, and hear the words of Yahweh your
God. And Joshua said, Hereby ye shall know that the living God is among you, and that he
will without fail drive out from before you the Canaanite, and the Hittite, and the Hivite, and

Volume 1990 - 2002 Issue


Page 66 of 2049
Skeptical Review Edited by Farrell Till
the Perizzite, and the Girgashite, and the Amorite, and the Jebusite. Behold, the ark of the
covenant of the Lord of all the earth passeth over before you into the Jordan," (Joshua 3:9-11).
To stress the emphatic nature of parts of the land promises that Yahweh made to Israel, I have
underlined certain statements. So when all of the passages I have quoted and listed are
considered, we see that the prophecies included all of the following:
Without fail, God would drive out of the land beyond the Jordan ALL of the people then
possessing it. No man among these people would be able to stand before the Israelites all the
days of their lives. The Israelites would drive out the nations possessing the land and utterly
destroy them and the memory of their name under heaven. They were to make no covenants
with the nations in this land or show mercy to them (Deut. 7:2). Every place that the sole of
their feet would tread upon, God would give to them. Their empire would stretch from the
Red Sea unto the river Euphrates and from the great sea (Mediterranean) toward the going
down of the sun.
To circumvent obvious contradictions that result when Yahweh's promises are compared to
biblical history recorded later, inerrantists contend that the land promises made to the
Israelites were conditional on their good behavior, but there is no support for that dodge in the
Bible. In Deuteronomy 9:3-7, another prophetic passage relating to the land promise, specific
notice was taken of the fact that the Israelites of the then present generation were themselves
undeserving of the land but that it would be given to them for the sake of the promises made
to Abraham, Isaac, and Jacob:
Know therefore this day, that Yahweh thy God is he who goeth over before thee as a
devouring fire; he will destroy them, and he will bring them down before thee: so shalt thou
drive them out, and make them to perish quickly, as Yahweh hath spoken unto thee.
Speak not thou in thy heart, after that Yahweh thy God hath thrust them out from before thee,
saying, For my righteousness Yahweh hath brought me in to possess this land; whereas for the
wickedness of these nations Yahweh doth drive them out from before thee. Not for thy
righteousness, or for the uprightness of thy heart, dost thou go in to possess their land; but for
the wickedness of these nations Yahweh thy God doth drive them out from before thee, AND
THAT HE MAY ESTABLISH THE WORD WHICH YAHWEH SWARE UNTO THY
FATHERS, TO ABRAHAM, TO ISAAC, AND TO JACOB. Know therefore, that Yahweh
thy God giveth thee not this good land to possess it for thy righteousness; for thou art a stiff-
necked people.

So here is another clear statement. God was not giving the land to the Israelites because of
their righteousness; in fact, he considered them a stiff-necked, undeserving people. (See also
Exodus 33:1-6.) He was giving the land to them because of the unconditional promise that he
had made to Abraham, Isaac, and Jacob. Unless he did this, he would have reneged on a
promise made to the patriarchs with no strings attached, (Gen. 12:7; 13:14-16).

The unconditional nature of Yahweh's land promise was restated in Leviticus 26:42-45:

Then will I remember my covenant with Jacob; and also my covenant with Isaac, and also my
covenant with Abraham will I remember; and I will remember the land. The land also shall be
left by them, and shall enjoy its sabbaths, while it lieth desolate without them: and they shall
accept of the punishment of their iniquity; because, even because they rejected mine
ordinances, and their soul abhorred my statutes. And yet for all that, when they are in the land
of their enemies, I will not reject them, neither will I abhor them, to destroy them utterly, and

Volume 1990 - 2002 Issue


Page 67 of 2049
Skeptical Review Edited by Farrell Till
to break my covenant with them; for I am Yahweh their God; but I will for their sakes
remember the covenant of their ancestors, whom I brought forth out of the land of Egypt in
the sight of the nations, that I might be their God: I am Yahweh.
So time and time again, it was specifically said that the Israelites would be given the land of
Canaan, REGARDLESS OF THEIR OWN CONDUCT, so that Yahweh could fulfill the
promise that he made to Abraham, Isaac, and Jacob. Inerrantists who deny this are denying
biblical statements worded just as plainly as anything ever said on the subject of creation, the
resurrection, baptism, final judgment, and other important Christian doctrines.

As proof that the land promise was dependent on the good behavior of the Israelites,
inerrantists like to cite Exodus 23:20-33 where a conditional suggestion was attached to the
promise: "But if thou shalt indeed hearken unto his voice (the angel that was to go before
them, FT) and do all that he speak, then I will be an enemy unto thine enemies and an
adversary unto thine adversaries." In emphasizing the if in this verse, they overlook an
important point. If Yahweh said that he would fulfill the promises made to Abraham, Isaac,
and Jacob regardless of the wickedness of the generation that went in to possess the land, he
could not turn around later and say that he would make good his promise only if the people
were obedient. That would put a contradiction into the scriptures that the inerrantists would
have to explain, because the land promise could not have been both conditional and
unconditional at the same time. And clearly the passages cited earlier were unconditional in
promising the land to the Israelites.

So after Yahweh had unconditionally promised to the Israelites that they would be given the
land beyond the Jordan, under Joshua's leadership they went in to possess it, and initially the
Bible claims that they succeeded. The claim, in fact, was that Joshua thoroughly and
completely subdued the land:

So Joshua smote ALL the land, the hill-country, and the South, and the lowland, and the
slopes, and all their kings: he left none remaining, but he utterly destroyed all that breathed,
AS YAHWEH, THE GOD OF ISRAEL, COMMANDED. And Joshua smote them from
Kadesh-barnea even unto Gaza, and all the country of Goshen, even unto Gibeon. And all
these kings and their land did Joshua take at one time, because Yahweh, the God of Israel,
fought for Israel. And Joshua returned, and all Israel with him, unto the camp to Gilgal,
(Joshua 10:40-43, ASV, Yahweh for Jehovah).
In places, the Bible is almost boringly repetitious, but this writing characteristic of the
"inspired" spokesmen of God often works to the advantage of those who seek to debunk the
myth that God verbally inspired the writing of the Bible. In this case, it makes it easy to
establish that a complete, unqualified fulfillment of the land promises was claimed by the
"inspired" men who wrote the Old Testament. Consider, for example, the clearly stated claim
of the following passages:
And Yahweh said unto Joshua, Be not afraid because of them (the armies of the Amorites,
Hittites, Perizzites, Jebusites, and Hivites poised for battle against the Israelites, FT); for
tomorrow at this time will I deliver them up ALL slain before Israel: thou shalt hock their
horses, and burn their chariots with fire. So Joshua came, and all the people of war with him,
against them by the waters of Merom suddenly, and fell upon them. And Yahweh delivered
them into the hand of Israel, and they smote them, and chased them unto great Sidon, and
unto Misrephothmaim, and unto the valley of Mizpeh eastward; and they smote them, until

Volume 1990 - 2002 Issue


Page 68 of 2049
Skeptical Review Edited by Farrell Till
they left them none remaining. And Joshua did unto them as Yahweh bade him: he hocked
their horses, and burnt their chariots with fire. And Joshua turned back at that time, and took
Hazor, and smote the king thereof with the sword: for Hazor before time was the head of all
those kingdoms. And they smote all the souls that were therein with the edge of the sword,
utterly destroying them; THERE WERE NONE LEFT THAT BREATHED: and he burnt
Hazor with fire. And all the cities of those kings, and all the kings of them, did Joshua take,
and he smote them with the edge of the sword, and utterly destroyed them; as Moses the
servant of Yahweh commanded. But as for the cities that stood on their mounds, Israel burned
none of them, save Hazor only; that did Joshua burn. And all the spoil of these cities, and the
cattle, the children of Israel took for a prey unto themselves; but every man they smote with
the edge of the sword, until they had destroyed them, neither left they any that breathed. As
Yahweh commanded Moses his servant, so did Moses command Joshua: and so did Joshua;
he left nothing undone of all that Yahweh commanded Moses, (Joshua 11:6-15, Yahweh
substituted).
So Joshua took the whole land, according to all that Yahweh spake unto Moses; and Joshua
gave it for an inheritance unto Israel according to their divisions by their tribes. And the land
had rest from war, (Joshua 11:23, Yahweh substituted).

So Yahweh gave unto Israel ALL the land which he sware to give unto their fathers; and they
possessed it, and dwelt therein. And Yahweh gave them rest round about, according to all that
he sware unto their fathers: and there stood not a man of all their enemies before them;
Yahweh delivered all their enemies into their hand. THERE FAILED NOT AUGHT OF ANY
GOOD THING WHICH YAHWEH HAD SPOKEN UNTO THE HOUSE OF ISRAEL. ALL
CAME TO PASS,(Joshua 21:43-45, Yahweh substituted).

These statements are fully as clear as Mark 16:16 and Acts 2:38. Yahweh gave unto Israel
ALL the land that he swore to give to their fathers, and the dimensions of that land were
clearly laid out in such passages as Exodus 23:20-33 and Joshua 1:1-6. Its borders extended
from the Red Sea to the sea of the Philistines, from the wilderness, to Lebanon, and to the
great river Euphrates. Furthermore, the fulfillment claims state that the Israelites left none
alive to breathe and that not a man of all their enemies stood before them. Who were those
enemies? Time and time again, they were named in the land prophecies: the Amorites, the
Canaanites, the Girgashites, the Hittites, the Hivites, the Jebusites, and the Perizzites. Yet
after audaciously claiming in the passages noted above that every aspect of Yahweh's land
promise had been fulfilled, the writer(s) turned around and brazenly admitted that some parts
of the land were not conquered and some of the peoples in these lands were not driven out:

Now Joshua was old and well stricken in years; and Yahweh said unto him, Thou art old and
well stricken in years, and there remaineth yet very much land to be possessed. This is the
land that yet remaineth: all the regions of the Philistines, and all the Geshurites; from the
Shihor, which is before Egypt, even unto the border of Ekron northward, which is reckoned to
the Canaanites; the five lords of the Philistines; the Gazites, and the Ashdodites, the
Ashkelonites, the Gittites, and the Ekronites; also the Avvim, on the south; all the land of the
Canaanites, and Mearah that belongeth to the Sidonians, unto Aphek, to the border of the
Amorites; and the land of the Gebalites, and all Lebanon, toward the sunrising, from Baalgad
under mount Hermon unto the entrance of Hamath; all the inhabitants of the hill-country from
Lebanon unto Misrephothmaim, even all the Sidonians; them will I drive out from before the

Volume 1990 - 2002 Issue


Page 69 of 2049
Skeptical Review Edited by Farrell Till
children of Israel: only allot thou it unto Israel for an inheritance, as I have commanded thee,
(Joshua 13:1-6, Yahweh substituted).
This statement flatly contradicts the claim in Joshua 11:23 that Joshua "took the whole land,
according to all that Yahweh spake unto Moses" so that the land had rest from war. All of the
territorial regions singled out in this passage as land that remained to be possessed lay within
the boundaries that were laid out in Joshua 1:1-6 to specify the scope of the land that Yahweh
would give to the Israelites. So if Joshua had indeed taken "the WHOLE land, according to all
that Yahweh spake unto Moses," as claimed In Joshua 11:23, how could it be said later that
"very much land" remained to be possessed? Perhaps some of our inerrantist readers can
answer this question. They are good at coming up with far-fetched, how-it- could-have-been
scenarios to "explain" obvious contradictions in the Bible.

Most of the rest of the book of Joshua and the better part of Judges contradict all of the
fulfillment claims that I have noted above. Joshua 15:63 says, "And as for the JEBUSITES,
the inhabitants of Jerusalem, the children of Judah could not drive them out; but the Jebusites
dwell with the children of Judah at Jerusalem unto this day." Yet the Jebusites were
specifically named as one of the seven nations "greater and mightier than thou" that would be
utterly destroyed. Joshua 16:10 says, "And they drove not out the CANAANITES that dwelt
in Gezer; but the Canaanites dwell in the midst of Ephraim unto this day, and are become
servants to do taskwork." But the Canaanites were specifically listed as one of the seven
nations that would be utterly destroyed. Joshua 17:12-13 says, "Yet the children of Manasseh
could not drive out the inhabitants of those cities; but the Canaanites would dwell in that land.
And it came to pass, when the children of Israel were waxed strong, that they put the
Canaanites to taskwork, and did not utterly drive them out." Yet the promise had clearly been
that the Canaanites would be utterly driven out, that NO MAN would be able to stand before
the Israelites all the days of their lives. Making servants of them can hardly be considered
fulfillment of a prophecy declaring that they would be "utterly driven out." In fact, it
contradicts a restriction noted on page three that expressly prohibited the Israelites from
making covenants with the inhabitants of their promised land.

In Joshua 16:10; 17:12-13; Judges 1:1-5; 1:9; 1:21; 1:27-36; 3:1-6 and many other places,
references are made to the people that the Israelites could not drive out of the land, and many
of these were specific references to people from the "seven nations greater and mightier than
thou" that Yahweh promised that he would drive out WITHOUT FAIL. But he didn't, and so
the inerrancy champions have some serious explaining to do. IF "Yahweh gave unto Israel
ALL the land which he sware to give unto their fathers" (Joshua 21:43-45) and IF "they
possessed it (the land) and dwelt therein" (same passage) and IF Yahweh "gave them rest
round about, according to ALL that he sware unto their fathers" (same passage) and IF "there
stood not a man of ALL their enemies before them" (same passage) and IF "Yahweh
delivered all their enemies into their hand" (same passage) and IF "there failed not AUGHT of
any good thing which Yahweh had spoken unto the house of Israel" (same passage) and IF
"all came to pass" (same passage), how could it have been that some of the enemies of Israel
were still in the land during the time of the book of Judges and how could it have been that
some of the people of the "seven nations greater and mightier than thou" were still dwelling
with the children of Israel "unto this day"?

Volume 1990 - 2002 Issue


Page 70 of 2049
Skeptical Review Edited by Farrell Till
Someone has a lot of explaining to do, and it isn't those of us who reject the inerrancy
doctrine.

Possession of the Promised Land


Charles Coats
It is my pleasure to be able to write a response to Mr. Till's article concerning "Yahweh's
Failed Land Promise." Since our articles are being printed herein in their entirety, the readers
will be able to compare the arguments given by both of us. Please consider carefully the
things said.

After reading Mr. Till's article, I see that two questions need to be answered:

(1) Did Israel ever possess all the land God promised?

(2) Did God place conditions on this possessing?

Let us begin our study with these questions in mind.

Abraham (then known as Abram) was called out of Ur to go to a land that God would show
him (Gen. 12:1), a land God said would be his for an inheritance (Gen. 15:7-8; 17:7-8).
Abraham and his seed were charged to keep God's covenant (Gen. 17:9ff).

Later, the promise was renewed to Isaac (Gen. 26:1-5) and to Jacob (Gen. 28:1-16). Then, as
Joseph neared death, he reminded the people of the promise in that God would "bring you
again unto the land of your fathers" (Gen. 48:21; 50:24).

It was Moses, as we know, who was given the charge to bring God's people out of Egypt and
take them to the land promised by God (Ex. 3:2-8). Yet, because of the iniquity of the people,
they were not able to go into this land for 40 years (Num. 13 and 14). This event and other
sins of the people are also referred to in Deuteronomy 9.

In Deuteronomy 7-10, Moses spoke to the people about possessing the land. They were to go
in and conquer the land. Note here that while God had promised the land to them, they had to
go in and conquer it, a condition placed by God (Deut. 7:1-2). Also, God told the Israelites to
"make no covenant with them (the people of the land, CC), nor show mercy to them" (Deut.
7:2). Deuteronomy 7:3 forbade marrying into the nations around them.

Further conditions for possession of the land are given in Deuteronomy:

Volume 1990 - 2002 Issue


Page 71 of 2049
Skeptical Review Edited by Farrell Till
ALL the commandments which I command thee this day shall ye observe to do, that ye may
live, and multiply, AND GO IN AND POSSESS THE LAND WHICH THE LORD SWARE
TO YOUR FATHERS, (8:1, emphasis CC).

And now, Israel, what doth the Lord thy God REQUIRE OF THEE, but to FEAR the Lord thy
God, to WALK in his ways, and to LOVE him, and to SERVE the Lord thy God with all thy
heart and with all thy soul, to KEEP the commandments of the Lord, and his statutes, which I
command thee this day FOR THY GOOD? (10:12-13, emphasis CC).

Yes, God was ready to give them the land that he promised to Abraham, Isaac, and Jacob
(Deut. 9:5), but it was conditional upon the people's obeying God. Even to Abraham and his
seed was given the condition of obedience (Gen. 17:9).

When Moses passed away, the leadership of Israel fell to Joshua (Deut. 34:9). It was Joshua
who was to take the people into Canaan and direct the conquering of the land. Joshua was
given a charge in Joshua 1:1-9 concerning this, and in verse 4, he was told the extent of the
land:

From the wilderness and this Lebanon even unto the great river, the river Euphrates, all the
land of the Hittites, and unto the great sea toward the going down of the sun, shall be your
coast.
The readers should also take note of the condition given in verse 3 of this passage:
Every place that the sole of your foot shall tread upon, that have I given you, as I said unto
Moses.
Again, it was going to be necessary for the people to "keep walking" in order to possess the
land. This condition for possession and the condition not to make covenants with the people
of the land were ably pointed out by Mr. Till (in his article) on page 3 of this publication.

God also stated his condition of obedience in Joshua 1:7-9. Note especially verse 8:

This book of the law shall not depart out of thy mouth; but thou shalt meditate therein day and
night, that thou mayest observe to do according to all that is written therein: FOR THEN
THOU SHALT MAKE THY WAY PROSPEROUS, AND THEN THOU SHALT HAVE
GOOD SUCCESS, (emphasis CC).
Joshua began the conquering of the land with the taking of Jericho (Josh. 6) and then lost at
Ai. This loss was brought about by Achan's taking of some of the possessions of Jericho
(Josh. 7:1), a violation of God's will (Josh. 6:18-19; compare with Deut. 7:5). Joshua 10:40-43
shows us that Joshua had conquered the country of the hills, of the south, of the vale, and of
the springs (v:40). These verses do not say that Joshua had conquered all the land of Canaan,
but contextually speak of those lands he had fought against to that time.

Joshua 10:41 mentions that Joshua had conquered unto Gibeon, a land he did not take in
battle but subdued in slavery, because he wrongfully made a pact with them (Josh. 9). This
was a clear violation of Deuteronomy 7:2. It came about because they "asked not counsel at
the mouth of the Lord" (Josh. 9:14).

Volume 1990 - 2002 Issue


Page 72 of 2049
Skeptical Review Edited by Farrell Till
Joshua's conquests continued in chapter 11, and in verse 23 we are told "Joshua took the
whole land." Some would cite this as showing that nothing was left that needed to be
conquered. However, in context, the term "whole land" refers to that land which Joshua had
done battle against. This is evident since verse 22 says that some of the Anakim still remained
in Gaza, Gath, and Ashdod. Since these were not yet conquered, it follows that the "whole
land" included only those lands conquered to this point, which according to Joshua 12:7
stretched from Lebanon to Edom. At this time, they rested from war and divided the land.

Of the land divided, some still needed conquering (Josh. 13:1), and this would have to be
done by those who inherited the land (Josh. 13:6). In Joshua's final charge to the people, he
reminded them of the condition for possession:

Take good heed unto yourselves, that ye love the Lord your God. Else IF YE DO IN ANY
WISE GO BACK, AND CLEAVE UNTO THE REMNANT OF THESE NATIONS, EVEN
THESE THAT REMAIN AMONG YOU, and shall make marriages with them, and go in
unto them, and they unto you: Know for a certainty that the Lord your God WILL NO MORE
DRIVE OUT ANY OF THESE NATIONS FROM BEFORE YOU; but they shall be snares
and traps unto you, and scourges in your sides, and thorns in your eyes, until ye perish from
off this good land which the Lord your God hath given you, (Josh. 23:11-13, emphasis CC).
As Judges 1 shows, the children of Israel did not conquer all their possessions. Judges 2:1-5
explains why. Note the following:
And an angel of the Lord came up from Gilgal to Bochim, and said, I made you to go up out
of Egypt, and have brought you unto the land which I sware unto your fathers; and I said, I
will never break my covenant with you. And ye shall make no league with the inhabitants of
this land; ye shall throw down their altars: BUT YE HAVE NOT OBEYED MY VOICE: why
have ye done this? Wherefore I also said, I will not drive them out from before you; but they
shall be as thorns in your sides, and their gods shall be a snare unto you.
It is obvious that the Israelites made leagues with these people and quit fighting. Thus, they
violated all conditions set forth by God: (1) they quit "walking" (Josh. 1:3), (2) they made
pacts with the enemy (Deut. 7:2), and (3) they disobeyed God's commands (Deut. 8:1).
Therefore, these people could not fulfill all of what God wanted done.

As Biblical history unfolds, however, after several hundred years, we come to the reign of
Solomon, the third king of Israel. In the early part of his reign (when he was right with God),
he was king over all the land that God promised to his people:

And Solomon reigned over all kingdoms from the river unto the land of the Philistines, and
unto the border of Egypt: they brought presents, and served Solomon all the days of his life, (I
Kings 4:21).
Compare this to Exodus 23:31 and Joshua 1:4.

The alleged contradiction stated by Mr. Till is no contradiction at all. Having answered our
questions posed at the outset of this article, I have shown that there were conditions placed on
the possessing of the land and that the land was finally possessed.

Please study this issue carefully.

Volume 1990 - 2002 Issue


Page 73 of 2049
Skeptical Review Edited by Farrell Till
(Charles Coats's address is 709 North Street, Howell, MI 48843.)

EDITOR'S NOTE: We appreciate Mr. Coats's effort to explain away the discrepancy
identified in "Yahweh's Failed Land Promise," but his "explanation" fell far short of his mark.
Mr. Coats seems to believe that he found passages that attached conditions to Yahweh's land
promise, but, as Till pointed out in his article, statements like Deuteronomy 9:3-7 clearly
indicated that Yahweh had to give the land unconditionally to the Israelites in order to fulfill
promises made, with no conditions attached, to Abraham, Isaac, and Jacob. So if Mr. Coats
really has found statements elsewhere in the Bible that affixed conditions to the land promise,
all he has succeeded in doing, as Till warned (p. 4), is establish that a contradiction exists in
the Bible. This leaves him with a problem just as serious as failure of the land promise would
be. So, even if he wins, he loses.

Other than this, a major shortcoming of Mr. Coats's response was his failure to show that the
Bible did not contradict itself in saying that the land promise had been fulfilled and then in
later saying that it had not been fulfilled. As Till pointed out on page 5, Joshua 21:43
explicitly said, "So Yahweh gave unto Israel all the land which he sware to give unto their
fathers; and they possessed it, and dwelt therein." Yet just three chapters later, the book of
Judges began to deny that all the land promised by Yahweh had been taken, and even Mr.
Coats admits that all the land Yahweh promised to give to "the fathers (Abraham, Isaac, and
Jacob)" was not in fact given to the Israelites at this time. Mr. Coats attributed this failure to
conditions that the Israelites did not meet, but even if he is right on this, it in no way explains
why the Bible plainly said that all the land promised had been possessed but then immediately
turned around and said that all the land had NOT been possessed. Any way Mr. Coats cuts it,
he is left with a serious discrepancy in the Bible text.

He thinks that he has found in I Kings 4:21 a complete fulfillment of the promise during the
reign of Solomon, but the readers are urged to look at this passage again (quoted near the end
of his article) and, as Mr. Coats advised, compare it to Joshua 1:4. Notice that the statement in
I Kings exempted the "land of the Philistines" from Solomon's kingdom. His kingdom
extended "unto the land of the Philistines," but did not include this land. However, Joshua
13:1-6 (quoted on page 6), in listing the areas of the promised land that at that time were yet
to be possessed, specifically referred to "all the regions of the Philistines" (v:2). Whoever
wrote this clearly thought that Yahweh intended for the Israelites to possess the land of the
Philistines, but in point of fact the Israelites never did possess this land, not even during the
reigns of David and Solomon when the borders of the Israelite kingdom were pushed to their
farthest extremes.

Mr. Coat's response, it would seem, has raised more problems than it solved. We see, then, no
reason to retract our claim that Yahweh failed to make good his promise that the Israelites
would go into the land beyond the Jordan, completely eradicate "seven nations greater and
mightier" than they, and possess all of their land "from the wilderness to Lebanon" and "from
the river Euphrates to the great sea toward the going down of the sun." By the Bible's own
admission, it just never happened.

Volume 1990 - 2002 Issue


Page 74 of 2049
Skeptical Review Edited by Farrell Till

Scientific Boo-Boos in the Bible


Bibliolaters claim that the Bible is inerrant in every detail, in matters of history, science,
geography, chronology, etc., as well as faith and practice. It is a claim that has won wide
acceptance among fundamentalist Christians, but, as is true of most zealotic tributes that have
been paid to the Bible, it has no basis in fact. As past articles in TSR have clearly shown to
anyone who really wants to know the truth, the Bible is riddled with mistakes. Many of those
mistakes were scientific ones.

The creation account in Genesis divided time into days and the days into evening and
morning for three days before the sun was even created (1:1-19). "There was evening and
there was morning," we are told, "one day... a second day... a third day," but as any
astronomer knows, evening (night) and morning (daylight) result from the earth's rotation
with respect to the sun. With no sun, there would have certainly been evening or night, but
there could have been no morning.

On the fourth day when God created the "two great lights" (the sun and the moon), he created
the stars too. This creation of the rest of the universe was treated by the Genesis writer(s) as if
it were little more than an afterthought: "he made the stars also" (v:16). To the prescientific
mind that wrote this, it probably made sense. To him (her), the earth was undoubtedly the
center of the universe, but today we know better. The solar system of which earth is only a
tiny part is itself an infinitesimal speck in the universe. Surely, then, the creation of the stars
would not have occurred so quickly and suddenly if six days were needed to create the world.
Scientists now know that the creation of stars is an evolutionary process that is still ongoing.
Matter coalesces; stars ignite, shine, and eventually burn out or explode. From the existence
of heavy elements in our solar system, astronomers generally agree that it formed from debris
left over from a supernova that occurred billions of years ago. The prescientific Genesis writer
knew none of this, however, and that is why he viewed the creation of the universe as an
Elohistic afterthought. No modern, scientifically-educated writer would have made that
mistake.

The creation of the stars is the subject not only of scientific error in the Bible but also of
textual contradiction. Clearly, the Genesis writer(s) said that God made the stars on the fourth
day (1:16). By then, the earth had been created, light (somehow without the sun or stars) had
been created, the gathering together of dry land had occurred, and vegetation had been
created. One could surely say that by then the foundations of the world had been laid, yet
Yahweh Elohim presumably told Job that the stars already existed when the foundations of
the earth were laid:

Where wast thou when I laid the foundations of the earth? Declare if thou hast understanding.
Who determined the measures thereof, if thou knowest? Or who stretched the line upon it?
Whereupon were the foundations thereof fastened? Or who laid the cornerstone there-of,
when the morning stars sang together, and all the sons of God shouted for joy? (38:4-7).

Volume 1990 - 2002 Issue


Page 75 of 2049
Skeptical Review Edited by Farrell Till
Granted the "singing of the morning stars" is clearly a poetical expression, but that does not
explain away the problem. How could it be said in any sense, poetical or otherwise, that "the
morning stars sang together" at a time when stars didn't even exist? Obviously, then, the
Genesis writer(s) and the author of Job had different perceptions of when stars were created.

The Genesis writer(s) didn't understand the nature of darkness either. He said that God created
light (somehow before the sun and stars were made) and then "divided the light from the
darkness" (1:3-4). Light, however, is not something that can be separated from darkness.
Light is an electromagnetic radiation from an energy source like the sun or stars, and darkness
is merely the absence of light. Without light, there will automatically be darkness. No god is
needed to separate or divide light from darkness. We know that today; the prescientific
Genesis writer(s) didn't.

The Genesis writer's genetic knowledge was no better than his understanding of astronomy. In
chapter 30, he told of Jacob's scheme to increase his wealth while he was still in the employ
of his father-in-law Laban. The two had reached an agreement whereby Jacob would be given
all striped, spotted, and speckled lambs and kids subsequently born in Laban's flocks. Laban
then removed all the striped, spotted, and speckled animals from his flocks and put them in
his sons' care at a three-day distance from the flock Jacob attended. Not to be outsmarted,
Jacob devised a plan:

Then Jacob took fresh rods of poplar and almond and plane, and peeled white streaks in them,
exposing the white of the rods. He set the rods that he had peeled in front of the flocks in the
troughs, that is, the watering places, where the flocks came to drink. And since they bred
when they came to drink, the flocks bred in front of the rods, and so the flocks produced
young that were striped, speckled, and spotted (30:37-39, NRSV).
The editors of The New American Bible were reputable enough to affix a frankly honest
footnote to this passage:
Jacob's stratagem was based on the widespread notion among simple people that visual
stimuli can have prenatal effects on the offspring of breeding animals. Thus, the rods on
which Jacob had whittled stripes or bands or chevron marks were thought to cause the female
goats that looked at them to bear kids with lighter-colored marks on their dark hair, while the
gray ewes were thought to bear lambs with dark marks on them simply by visual
crossbreeding with the dark goats.
We know today that the color characteristics of animals is purely a matter of genetics, so a
modern, scientifically-educated person would never write anything as obviously superstitious
as this tale of Jacob's prosperity. The Genesis writer(s), however, knew nothing about the
science of genetics, so to him the story undoubtedly made good sense.

One thing the Bible definitely is not is inerrant in matters of science.

Inerrancy Debates in Print

Volume 1990 - 2002 Issue


Page 76 of 2049
Skeptical Review Edited by Farrell Till
Skepticism, Inc., has reissued the Laws-Till Debate. After challenging Farrell Till to a written
debate on the inerrancy question, James H. Laws, a Church-of-Christ preacher in the Spiritual
Sword faction, quit after only three exchanges. The fifty pages that were exchanged prior to
his withdrawal will explain why. Available at publishing cost, $2 ppd. Send orders to the
address on page 2.

The Jackson-Till Debate has been published by the Southwest Church of Christ, 8900
Manchaca Road, Austin, TX 78748. Cost is $2.25 ppd. Direct all orders to the publisher.

Deaver-Till Debate
Mac Deaver, a Church-of-Christ preacher from Austin, Texas, and Farrell Till will conduct an
oral debate on the campus of Southwest Texas State University in San Marcos, Texas, on
March 25-28, 1991. The issue in question will be Bible inerrancy.

Confidentiality Guaranteed

Having experienced the emotional trauma of losing faith in the Bible, we can understand why
anyone in the initial stages of unbelief would be reluctant to let his doubts be made known. If
this should be the case of any readers on our mailing list who are receiving TSR at the request
of friends or relatives, we want to assure them that they can contact us for advice or help with
complete assurance that their inquiries will be kept strictly confidential. Nothing written or
said to us will ever be made public without the consent of the writer (or caller).

Volume 1990 - 2002 Issue


Page 77 of 2049
Skeptical Review Edited by Farrell Till

The Skeptical Review


Volume Two, Number Two
April/May/June 1991
Farrell Till, editor

Playing the Odds


"'If I am right and you are wrong....' How many times have bibliolaters said this to
skeptics after all rational efforts to defend the Bible have failed?" Farrell Till responds
to this oft-given inerrantist argument.
The Blood of Jezreel
Jerry McDonald replies to Farrell Till's article entitled "A Perfect Work of
Harmony?". He claims there is no contradiction between 2 Kings 10:30 and Hosea
1:4.
Ahab, Jezebel, Jehu, and the Blood of Jezreel
Till responds to Jerry McDonald and claims that the contradiction between 2 Kings
10:30 and Hosea 1:4 still stands.
Bible Biology
Did Bible writers demonstrate knowledge of scientific facts that were unknown at the
time the Bible was being written? According to Farrell Till, "there is no more
'scientific knowledge' in the Bible than in any other literature of the same era."
Recommended Publication
Farrell Till advertises Biblical Errancy.
Inerrancy Debates Available
Information on how to get transcripts of the Till-Laws and Till-Jackson debates.

Playing the Odds


Farrell Till

Volume 1990 - 2002 Issue


Page 78 of 2049
Skeptical Review Edited by Farrell Till
"If I am right and you are wrong...." How many times have bibliolaters said this to skeptics
after all rational efforts to defend the Bible have failed? What they are saying is that one
should believe the Bible in order to be on the safe side, just in case it really is God's inspired
word. This argument, if that is what they intend it to be, is merely a variation of Pascal's
wager, a theistic argument made famous by the French philosopher Blaise Pascal. "If you
gain, you gain all," Pascal argued. "If you lose, you lose nothing. Wager, then, without
hesitation that He is."

Aside from the obvious fact that one cannot believe--sincerely believe--a thing just to be on
the safe side, the absurdity of Pascal's wager is seen in the utter impossibility of practicing it.
One should believe in God just in case God really does exist. Okay, what next? After one
wagers on God's existence, what religion does he choose to practice his faith in God? Does he
become a Christian or a Moslem? A Zoroastrian or a Hindu? If he chooses Christianity, what
brand of it does he select? If he becomes a Methodist, how does he deal with the possibility
that Catholicism may be the true religion? To meet the requirements of Pascal's wager, one
would have to simultaneously become a believer in all religions in the world, and this would
be utterly impossible, since many religions forbid beliefs in others.

What does one lose if he accepts Pascal's wager? Pascal said, "You lose nothing," but this is a
questionable premise at best. In Atheism: The Case Against God, George H. Smith exposed
the fallacies in Pascal's wager. On the subject of what one loses by making the wager, he said
this:

What have we got to lose? Intellectual integrity, self-esteem, and a passionate, rewarding life
for starters. In short, everything that makes life worth living. Far from being a safe bet,
Pascal's wager requires the wager of one's life and happiness (Prometheus Books, 1979, p.
184).
Bibliolaters are apparently willing to risk their lives and happiness on the probability that they
have made all the correct choices. They think they have made the right decisions in choosing
theism over atheism, Christianity over all other religions, their particular brand of Christianity
over all the options available to them, and finally the correct variations in doctrines that exist
within the churches selected. But what are the odds that any given Christian has made all the
right decisions in his journey through the religious maze that led him to where he is now?
This is a question that deserves far more thought than most Christians give it.

While considering this, they might also think about how the odds are stacked against the
Bible's being what they believe it is. The doctrine of verbal inspiration logically requires one
to believe that every detail written in the Bible, whether historically, geographical, scientific,
or chronological, must be factually true. The existence of just one mistake of any kind, no
matter how trivial or insignificant, tears the foundation completely from under the doctrine of
verbal inspiration. This is a premise we don't even need to defend, because inerrancy believers
agree that it is true.

What then are the odds that the Bible is the perfectly harmonious and consistent work it
would have to be for the doctrine of verbal inspiration to be true? In his Encyclopedia of Bible
Difficulties, Gleason Archer the chief apostle of the inerrancy doctrine, discussed over 2,100
specific cases of "alleged" Bible contradictions and discrepancies. Even at that, he did not

Volume 1990 - 2002 Issue


Page 79 of 2049
Skeptical Review Edited by Farrell Till
deal with all that have been identified, but 2,100 is more than enough to make a point
inerrancy believers should contemplate. To explain away what they consider to be only
"alleged" contradictions in the Bible, Archer and his inerrancy cohorts have resorted to all
kinds of far-fetched, how-it-could-have- been scenarios of the sort we have analyzed in this
and past issues. For the inerrancy doctrine to be true, they must be right in all of their far-
fetched explanations--every one of them. If they are wrong in just one--only one--the
foundation of the inerrancy doctrine collapses.

What are the odds that they are right in everything--all 2,000+ "explanations"? If bibliolaters
want to play the odds, as their high regard of Pascal's wager would indicate, they should think
about this.

The Blood of Jezreel


Jerry McDonald
In the spring 1990 issue of The Skeptical Review, Editor Farrell Till wrote an article entitled
"A Perfect Work of Harmony?" in which he challenged Bible fundamentalists to answer a
difficulty involving 2 Kings 10:30 and Hosea 1:4. Apparently no one took the challenge, so
he reissued it in the summer issue in an article entitled "No Takers." It is to these challenges
that I address this response.
And the Lord said unto Jehu, Because thou hast done well in executing that which was right
in mine eyes, and hast done unto the house of Ahab according to all that was in mine heart,
thy children of the fourth generation shall sit on the throne of Israel (2 Kings 10:30).

And the Lord said unto him, Call his name Jezreel; for yet a little while, and I will avenge the
blood of Jezreel upon the house of Jehu, and will cause to cease the kingdom of the house of
Israel (Hosea 1:4).

There are several problems with Mr. Till's interpretation of these two passages. In this article,
we shall notice these problems and will show that these two verses have absolutely no
relationship whatever to each other. In summary of Mr. Till's problems, we see that [1] he did
not carefully read what the two passages had to say, [2] he did not read all that was said on
the subject, [3] he did not allow for figures of speech, and [4] he did not allow for the context.
Had he not violated these four principles, he would not have taken the position that he took.

[1] Mr. Till did not carefully read what the passages had to say. "Why would Yahweh want to
punish the house of Jehu for what was done at Jezreel," he asked, "if all Jehu had done was
'that which was right in mine (Yahweh's) eyes'?" (The Skeptical Review, Spring 1990, p. 12).
Did God say that he was punishing the house of Jehu for what Jehu did to the houses of
Jehoram (the son of Ahab) and Ahaziah? No. Did Hosea say this? No. Did anyone make this
charge? Yes. Who? Farrell Till. All God said that he was going to do was "avenge the blood
of Jezreel upon the house of Jehu." Mr. Till wants it to read like this: "I will avenge upon the

Volume 1990 - 2002 Issue


Page 80 of 2049
Skeptical Review Edited by Farrell Till
house of Jehu what Jehu did in Jezreel to the houses of Jehoram and Ahaziah." But it does not
read that way. We must always be careful to read exactly what the writer had to say and not
what we want him to say.

[2] Mr. Till did not read all that was said about the subject. He tried to make it sound as if
Zachariah (the king in power at the time) was a good man who was being punished for Jehu's
(supposed) transgression. However, in looking at Zachariah, we notice that he "did that which
was evil in the sight of the Lord, as his fathers had done: he departed not from the sins of
Jeroboam the son of Nebat, who made Israel to sin" (2 Kings 15:9). So we can see that
Zachariah was an evil king, and we see he was being punished for his wickedness and not for
anything Jehu had done.

Another part of Mr. Till's problem is that he assumed that the blood of Jezreel had to refer to
that which was done by Jehu at Jezreel to the houses of Jehoram and Ahaziah. In making this
assumption, he completely overlooked the story in 1 Kings 21 about the blood of Naboth the
Jezreelite being shed by the order of Jezebel. Naboth had a vineyard that had been given to
him by God. This vineyard was next to Ahab's palace. Ahab saw the vineyard, desired to have
it, and tried to buy it from Naboth. Naboth refused: "The Lord forbid it me, that I should give
the inheritance of my fathers unto thee" (1 Kings 21:3). Now this angered Ahab, and he went
to his room to pout. He pouted so much that he would not eat, and when this came to the
attention of his wife Jezebel, she came to find out why he would not eat. He told her about the
vineyard he wanted, and she told him to eat. She would take care of the matter.

Jezebel sent letters in Ahab's name to the elders and nobles who dwelt in Jezreel and
proclaimed a feast that would set Naboth on high before the people. Then she set two men,
the sons of Belial, before him with instructions to accuse him of blaspheming God and the
king. The instructions included taking Naboth out and stoning him to death. When all these
things happened according to Jezebel's plans, Ahab was able to take possession of Naboth's
vineyard. Because of this crime, God sent Elijah the Tishbite to tell Ahab and Jezebel that
they would be punished because of what Jezebel had done. Jezebel would die and Ahab's son
would die. Jezebel did die, and Jehoram (the son of Ahab) died at the hands of Jehu. Whose
blood did Ahab and Jezebel shed? The blood of Naboth the Jezreelite. Could that not
rightfully be stated as being "the blood of Jezreel"? Yes! Now when one looks at Hosea 1:4 in
this respect, it can be easily seen that this makes much more sense than what Mr. Till
proposed.

[3] Mr. Till's third problem is that he did not allow for figures of speech. The phrase "the
blood of Jezreel" is a figure of speech called synecdoche in which the part is put for the
whole. To be more specific, we have a synecdoche of the species: "Blood is put for murder or
cruelty; or death generally.... So Hos. i.4..." (Figures of Speech Used in the Bible, p. 628). The
avenging of blood, in the Bible, was often used in reference to avenging any innocent blood
that was shed. In this case, the blood of Naboth the Jezreelite was used figuratively to refer to
the vengeance that would be taken upon the one who had shed the spiritual blood of Israel.

[4] Mr. Till did not allow for the context. In Hosea 1:4, God told Hosea to name his firstborn
son Jezreel because in a little while he was going to avenge the blood of Jezreel upon the
house of Jehu. Hosea's son represented the innocent blood that was shed by the wicked king

Volume 1990 - 2002 Issue


Page 81 of 2049
Skeptical Review Edited by Farrell Till
Zachariah. He was probably called Jezreel because of Naboth's innocent blood that was shed
at Jezreel. Hosea and Gomer had a second child, a daughter, and were instructed to name her
Loruhamah, because God would have no more mercy upon the house of Israel. Then they had
a third child, a boy, whose name was to be called Loammi, because Israel would no longer be
God's people. Hosea represented God, and Gomer represented Israel. The three children
represented the three stages of Israel's departure from God. Jezreel represented the innocent
spiritual blood of the Israelites that was shed by the wicked king. Loruhamah represented
Israel following the wicked king and going off into apostasy and even going to the point that
God would no longer have mercy. Loammi represented Israel completely cut off from God.

The phrase "I will avenge the blood of Jezreel upon the house of Jehu" simply meant that God
would avenge the spiritual blood of his people (which was shed by Zachariah when they were
good) upon the house of Jehu. Because Zachariah continued in the sin of Jeroboam, the son of
Nebat, in causing Israel to sin, he was shedding their spiritual blood, and because of this God
was going to punish him. He used the phrase "the blood of Jezreel" as a figure to stand for the
punishment that he would inflict upon Zachariah. He remembered Naboth to show that
injustice will always be punished in the end.

If Mr. Till spent half as much time trying to reconcile these "so-called" difficulties as he
spends finding them, he would find far fewer difficulties in the Bible. There is no problem
with these two passages. I hope this explanation puts to rest any fears that anyone may have
of there being a problem with these two passages. They have absolutely nothing in the world
to do with each other.

(Jerry McDonald's address is 97 Florence Street, Sullivan, MO 63080.)

Ahab, Jezebel, and the Blood of Jezreel


Farell Till
There is apparently no limit to the far-fetched scenarios that Bible fundamentalists are willing
to conjecture in defense of the inerrancy doctrine. Jerry McDonald patently demonstrated this
in his back-to-the-wall attempt to explain away the obvious inconsistency that is seen in the
juxtaposition of 2 Kings 10:30 and Hosea 1:4.

I say that the "blood of Jezreel" that Hosea referred to was Jehu's slaughter of the royal
families of Israel and Judah at Jezreel. McDonald says that I am wrong, that the "blood of
Jezreel" was a reference to the murder of Naboth, which also occurred at Jezreel. Which one
of us is right? The answer is crucial to the matter now before us. If we can find the answer, we
will have determined whether there is a discrepancy in the Bible text.

A widely accepted rule of evidence states that when there are two or more explanations for a
mystery or phenomenon, the least incredible one is the most likely solution. William of

Volume 1990 - 2002 Issue


Page 82 of 2049
Skeptical Review Edited by Farrell Till
Occam, a 14th century English philosopher and logician, enunciated this rule as a principle
that has come to be known as Occam's razor. By applying the rule, we should at least be able
to establish likeliness of truth in the blood-of-Jezreel dispute between McDonald and me. In
this case, we have two biblical passages in apparent conflict. One explanation for the conflict
states that it is exactly what it appears to be--a discrepancy, an inconsistency, a contradiction.
The other, proposed by Mr. McDonald, is that the "blood of Jezreel" in Hosea 1:4 referred not
to Jehu's slaughter of the royal families but to Jezebel's murder of Naboth. Which of the two
explanations provides the least incredible and hence the more likely solution to the problem?
In other words, which of the two explanations is better suited to pass the test of Occam's
razor?

The facts (as related in 1 Kings 21) in the matter of Naboth are as McDonald related them,
except for one important thing. The curse that Elijah pronounced on the house of Ahab
originally included a death sentence for Ahab:

Behold, I will bring evil upon you, and will utterly sweep you away, and will cut off from
Ahab every man-child, and him that is shut up and him that is left at large in Israel: and I will
make your house like the house of Jeroboam the son of Nebat, and like the house of Baasha
the son of Ahijah, for the provocation with which you have provoked me to anger, and have
made Israel to sin.... He that dies of Ahab in the city the dogs shall eat; and he that dies in the
field shall the birds of the heavens eat (1 Kings 21:21-24, Bethel Bible).
Upon hearing the curse, Ahab immediately rent his clothes, put on sackcloth, and fasted.
Because of his repentance, Yahweh, in typical Yahwistic fashion, said that he would not bring
the evil upon Ahab in his days but "in his son's day will I bring the evil upon his house" (1
Kings 21:29). In other words, Yahweh was going to punish someone else for what Ahad had
done.

At any rate, this was the event that Mr. McDonald sees as the "blood of Jezreel" that was later
condemned by the prophet Hosea (1:4). One man--and possibly his sons (2 Kings 9:26)--was
murdered unjustly. How did this act of murder compare to Jehu's infamous deeds at Jezreel?

As related in my other article (Spring 1990), Jehu led an armed force against Jezreel while
Ahaziah, the king of Judah, was visiting King Joram. Both kings, who had ridden out to meet
Jehu in peace, were immediately killed by Jehu (2 Kings 9:21-28). Upon entering the city,
Jehu ordered that Jezebel be thrown from the rampart of the wall. Then he systematically
began a blood bath that dwarfed anything that could be attributed to Ahab and Jezebel in the
matter of Naboth. Jehu gave the elders of Jezreel the choice of fighting for a new king that
they would crown from one of the sons of Joram or else delivering to him the heads of all
seventy sons by the next morning. When the heads were delivered to Jehu in baskets, he
ordered that they be laid in two heaps at the entrance of the gate (10:1-10). He then killed "all
that remained of the house of Ahab in Jezreel, and all his great men, and his familiar friends,
and his priests, until he had left him none remaining" (10:11).

Jehu then left for Samaria, where en route he met forty-two of Ahaziah's brothers, who were
on their way to greet the king. Jehu gave his men orders to take them alive, then kill them, and
cast their bodies into a shearing-house cistern (10:12-14). On his arrival at Samaria, he "smote
all that remained unto Ahab in Samaria" (10:17), but still he wasn't finished. Under the

Volume 1990 - 2002 Issue


Page 83 of 2049
Skeptical Review Edited by Farrell Till
pretense of wanting to sacrifice to the god Baal, he lured all of the priests and worshippers of
Baal into the house of Baal, offered his sacrifice, and then ordered his men to kill everyone in
the temple to rid the land of Baal worship (10:18-28).

By sheer force of numbers, what Jehu did is a far more reasonable event to associate with
Hosea's reference to the "blood of Jezreel" than is Ahab's murder of Naboth. In the latter case,
only one man's blood was shed; in the other, there is no accurate way even to estimate the
number of people who were murdered. Two kings, Jezebel, 70 sons, 42 brothers-- these alone
totaled 115. But how can we determine how many were included in "all that remained of the
house of Ahab, all his great men, his familiar friends, and his priests" (10:11)? How can we
know how many Jehu killed "that remained unto Ahab in Samaria" (10:17)? And how can we
even estimate how many priests and worshippers of Baal were slaughtered in the temple
massacre (10:25-28)? The blood that Jehu shed at Jezreel flowed from hundreds, possibly
even thousands; the blood that Ahab shed at Jezreel flowed from only one--Naboth. In this
respect, the test of Occam's razor obviously favors Jehu's slaughter.

In addition to this is the clear fact that Hosea specifically said that the "blood of Jezreel"
would be avenged upon the house of Jehu (1:4). Now why would Yahweh avenge the blood
of Jezreel upon the house of Jehu if the blood of Jezreel referred to Naboth's blood that Ahab
and Jezebel were responsible for shedding? There is no textual evidence to indicate that Jehu
was involved in the conspiracy to kill Naboth or even present when Naboth was stoned to
death. Neither was Jehu a descendant or relative of Ahab. The descendants of Ahab and the
descendants of Jehu were two entirely separate "houses" or lineages. Doesn't it make sense
that, in Hosea's mind, the "blood of Jezreel" was to be avenged on the house of Jehu because
the blood to be avenged was the very blood that Jehu was responsible for shedding? Clearly,
then, the test of Occam's razor favors Jehu's slaughter on this point too.

Furthermore, the blood of Naboth had already been avenged when Hosea pronounced
judgment upon the house of Jehu. Elijah told Ahab that, for his conduct in the matter of
Naboth's vineyard, the dogs would lick his blood in the same place where dogs licked the
blood of Naboth. That part of Elijah's prophesy was fulfilled in 1 Kings 22:38 when dogs
licked up the blood that was washed from Ahab's chariot after his death, even though Yahweh
had earlier said that, because of Ahab's repentance, he would "not bring the evil in his
(Ahab's) day; but in his son's day will I bring the evil upon his house" (1 Kings 21:29). This is
just a problem incidental to the matter in question that we will let Mr. McDonald wonder
about as he ponders all the other difficulties he must resolve in order to explain away this
discrepancy.

Elijah had also warned that dogs would eat the flesh of Jezebel by the rampart of Jezreel.
When nothing was found but Jezebel's skull, feet, and hands after Jehu ordered her thrown
down from the rampart upon his entry into the city, it was specifically said that this had been
done in fulfillment of Elijah's prophecy (2 Kings 9:36-37). So both events (the dogs licking up
Ahab's blood and eating Jezebel's flesh) were presumably done to fulfill Elijah's prophecy, a
prophecy that concerned the avenging of Naboth's blood.

The third and final part of Elijah's prophecy was the utter destruction of the house of Ahab,
and the writer of 2 Kings obviously viewed Jehu as an agent of Yahweh who had been sent to

Volume 1990 - 2002 Issue


Page 84 of 2049
Skeptical Review Edited by Farrell Till
carry out this part of the prophecy. Even the "son of the prophets" whom Elisha had sent to
anoint Jehu king over Israel said during the ceremony that Jehu had been chosen to execute
judgment against the house of Ahab:

In this manner says Yahweh, the Elohim of Israel, I have anointed you king over the people of
Yahweh, even over Israel. And you shall destroy the house of Ahab your master, that I may
avenge the blood of my servants the prophets, and the blood of all the servants of Yahweh, at
the hand of Jezebel. For the whole house of Ahab shall perish; and I will cut off from Ahab
every man-child, and him that is shut up and him that is left at large in Israel. And I will make
the house of Ahab like the house of Jeroboam the son of Nebat, and like the house of Baasha
the son of Ahijah (2 Kings 9:6-9, Bethel Bible).
After he had killed King Joram (Ahab's son), Jehu gave orders to his captain that clearly
indicated he viewed his actions at Jezreel as fulfillment of Elijah's prophecy and avengement
of the blood of Naboth:
Take up, and cast him in the portion of the field of Naboth the Jezreelite; for remember how
that, when you and I rode together after Ahab his father, Yahweh laid this burden upon him:
Surely I have seen yesterday the blood of Naboth, and the blood of his sons, says Yahweh;
and I will requite you in this plot, says Yahweh. Now therefore take and cast him into the plot
of ground, according to the word of Yahweh (2 Kings 9:25- 26, Bethel Bible).

After ordering the decapitation of Joram's seventy sons, Jehu said in a speech to the people of
Jezreel, "Know now that there shall fall to the earth nothing of the word of Yahweh, which
Yahweh spoke concerning the house of Ahab: for Yahweh has done that which he spoke by
his servant EliYah" (2 Kings 10:10, Bethel Bible).

To the writer of 2 Kings, then, vengeance for the blood of Naboth was completed by Jehu's
actions at Jezreel. As predicted by Elijah, the dogs had already licked the blood of Ahab, and
now, with Jehu acting as Yahweh's agent, Jezebel was thrown to the dogs from the rampart of
Jezreel and the entire house of Ahab was massacred. The house of Ahab had indeed become
"like the house of Jeroboam the son of Nebat and like the house of Baasha the son of Ahijah"
(1 Kings 21:22). With all of this having come to pass, Yahweh had extracted from the house
of Ahab all the vengeance that was humanly or even Yahwistically possible. What else could
have been done on earth to punish Ahab and his family for the shedding of Naboth's blood?
After both Ahab and his wife had died horrible deaths and then every last one of Ahab's
descendants (sons, grandsons, all that remained of the house of Ahab, and even familiar
friends) had also been killed, what possible action could anyone have then taken to further
avenge the blood of Naboth? According to Mr. McDonald, there was yet one more thing that
Yahweh could do to avenge the blood of Naboth. He could punish the house of Jehu for
Naboth's murder. Jehu had nothing to do with Naboth's murder, yet for some reason (known
only to the omniscient inscrutable one himself), Yahweh was going to bring the blood of
Naboth down upon the house of Jehu. Now that is exactly what McDonald wants us to
believe. But who can believe it? I have heard of double jeopardy, but this is ridiculous.

When all details of the two actions at Jezreel (Jezebel's and then later Jehu's) are objectively
considered, which of the two is more likely the one Hosea had in mind when he said, "yet a
little while, and I will avenge the blood of Jezreel upon the house of Jehu"? In every respect

Volume 1990 - 2002 Issue


Page 85 of 2049
Skeptical Review Edited by Farrell Till
and from any angle the events are examined, Jehu's slaughter of the house of Ahab at Jezreel
passes the test of Occam's razor more readily than Jezebel's murder of Naboth.

Mr. McDonald accused me of not reading the two passages (2 Kings 10:30 and Hosea 1:4)
very carefully; however, he is the one who needs to read them more carefully. For the blood
that was shed at Jezreel, Hosea pronounced judgment upon the house of Jehu, NOT the house
of Ahab. By what standard of fairness would judgment be pronounced upon the house of Jehu
for something that had been done by the house of Ahab? That's a question Mr. McDonald
should think about.

As previously noted, judgment was pronounced upon the house of Ahab because of Jezebel's
murder of Naboth, but because of Ahab's repentance, enactment of that judgment was
delayed: "Do you see how Ahab humbles himself before me? Because he humbles himself
before me, I will not bring the evil in his days; but in his son's days I will bring the evil upon
his house" (1 Kings 21:29). What Jehu did at Jezreel was done after the days of Ahab (who at
that time was dead) but in the days of Ahab's son (Joram). To the writer(s) of 2 Kings, then,
Elijah's prophecy was a closed book; the blood of Naboth had been avenged by Jehu whom
the writer(s) obviously viewed as a chosen vessel to execute Yahweh's judgment against
Ahab. After the carnage that Jehu wreaked at Jezreel, Ahab didn't have a son left for "the evil"
spoken of in 1 Kings 21:29 to be brought upon. McDonald must therefore admit that Jehu's
actions at Jezreel fulfilled Elijah's prophecy and thereby avenged the blood of Naboth or he
must say that Elijah's prophecy failed for lack of a son of Ahab to bring "the evil" upon. He
finds himself in a predicament that Bible inerrantists often become trapped in: the invention
of a far-fetched, how-it-could-have-been scenario to "explain" away one problem leaves them
with another one just as serious.

So the problem has not gone away. Yahweh himself praised Jehu for his bloody deeds at
Jezreel:

And Yahweh said to Jehu, "because you have done well in executing that which is right in my
eyes, and have done to the house of Ahab according to all that was in my heart, your sons of
the fourth generation shall sit on the throne of Israel" (2 Kings 10:30, Bethel Bible).
A statement in 2 Kings 15:12 indicates that Yahweh had allowed four generations of the
house of Jehu, from Jehoahaz to Zechariah, to reign over Israel in order to fulfill Yahweh's
promise to Jehu. Obviously, then, the writer(s) of 2 Kings believed that Jehu had served
Yahweh and done his will in massacring the house of Ahab at Jezreel. If that is so, why would
Hosea, presumably another Yahwistically inspired writer, later pronounce judgment upon the
house of Jehu for the Jezreel massacre? That is what McDonald did not and cannot explain.

He had much to say about the "figurative" language in Hosea (Hosea's son represented this,
his daughter represented that, etc.), but his interpretations were purely arbitrary. He gave no
textual proof whatsoever to support them. Everything in some way referred to the evil old
king Zechariah, yet Zechariah's name wasn't even mentioned in the entire book of Hosea. The
truth is that not much of anything was said about Zechariah in the Bible. His entire reign
lasted only six months and was described in just four verses (2 Kings 15:8-11). McDonald
spoke of "the innocent blood that was shed by the wicked king Zachariah" (p. 3), but the Bible
says nothing about this. In fact, Zechariah's own blood was shed by a political assassin.

Volume 1990 - 2002 Issue


Page 86 of 2049
Skeptical Review Edited by Farrell Till
So McDonald has no textual proof whatsoever to support his fanciful interpretation. To get
out of the hole he has dug himself into, he will have to beat that.

Bible Biology
Farrell Till
An earlier article ("What About Scientific Foreknowledge in the Bible?" Fall 1990), debunked
the fundamentalist claim that the truth of verbal inspiration can be verified by places in the
Bible text where writers demonstrated knowledge of scientific facts that were unknown at the
time the Bible was being written. The intent of the claim is to "prove" that Bible writers
"foreknew" these scientific facts because God revealed them through the process of verbal
inspiration, but, as my article showed, scientific foreknowledge in the Bible can be found only
in the eisegetical interpretations of bibliolaters shamelessly bent on clinging to an untenable
view of the Bible. In reality, there is no more "scientific foreknowledge" in the Bible than in
any other literature of the same era.

If it were really true that Bible authors revealed in their works scientific facts that were not
discovered until centuries later, this would indeed be a formidable argument for the verbal
inspiration of the Bible, but the evidence that bibliolaters point to to prove their theory is
entirely too speculative to be convincing. Some inerrantists, for example, have absurdly seen
evidence that the Bible foresaw the potential for using electricity to send messages. In
speaking to Job from the whirlwind, Yahweh asked him, "Can you send forth lightnings, that
they may go, and say to you, Here we are?" (Job 38:35). In Why We Believe the Bible,
George DeHoff made this comment on the verse:

Job could not do this but we are able to do so today as we talk on the telephone and radio and
send our messages by telegraph. Truly the lightning goeth and saith for us (p. 55).
There are so many absurdities in this application of the verse that I hardly know where to
begin commenting on them. For one thing, it violates a principle of common sense that should
tell DeHoff and his inerrancy cohorts that a clear-cut, undeniable case of scientific
foreknowledge would have to be stated in language so obvious in meaning that there could be
no disagreement in interpretation. In my response to Jerry McDonald's article elsewhere in
this issue, I used the rule of Occam's razor to discredit his claim that Hosea meant for "the
blood of Jezreel" to refer to the murder of Naboth. The rule is equally applicable to DeHoff's
claim of scientific foreknowledge in a simple statement about lightning. As long as it is
possible for the statement to mean something less complex than the supernatural insight of a
primitive writer into the physics of transmitting sound by electricity, then there is no force at
all to the claim that this is an example of scientific foreknowledge.

Could the statement have a simpler meaning than what DeHoff assigned to it? It would
certainly seem so. Why, for example, couldn't it mean no more than that lightning announces
its presence by the natural sound it makes? This is a phenomenon we have all witnessed

Volume 1990 - 2002 Issue


Page 87 of 2049
Skeptical Review Edited by Farrell Till
during thunderstorms. In his discourse to Job, Elihu said, "He (God) covers his hands with
lightning, and gives it a charge that it strike the mark. The noise of it tells concerning him, the
cattle also concerning the storm that comes up" (36:32-33). A primitive superstition that God
makes lightning and directs its strike is obviously reflected in this statement (a belief that
hardly qualifies as "scientific foreknowledge"), but the final part of the statement seems to be
saying that lightning announces the approach of a storm. Elihu, then, seemed to know exactly
what Yahweh said in Job 38:35. The lightning goes forth and says, "Here we are." What is so
wonderfully insightful about that?

The problem for bibliolaters who see scientific foreknowledge in the Bible is that none of the
statements they point to can successfully pass the test of Occam's razor. All pose the
possibility of simpler, less complex interpretations than those that attribute supernatural,
scientific insights to the writers. Common sense should again tell us that this is so. If not, then
why didn't those marvelous insights put science centuries ahead of the plodding advancement
it has made? If, for example, Job 38:35 really meant what DeHoff claims it meant, then why
didn't someone among the millions and millions of people who read it during the past 3,000
years recognize its meaning and apply it long before telecommunication systems were finally
invented? The same could be asked of all the other alleged examples of scientific foresight in
the Bible. If these were in fact true cases of foreknowledge, then why didn't Bible readers
apply the scientific principles involved in them long ago? Why did the world have to wait
through the centuries until scientists, working independently of the Bible, discovered the life-
sustaining properties of blood, the female ovum, the water cycle, and the many other
scientific facts that bibliolaters claim were foreknown by Bible writers? There is something
very suspect about after-the-fact biblical interpretations that point to recent scientific
discoveries and gleefully proclaim, "Ah, yes, this was foreseen in the Bible where so-and-so
said thus-and-so!"

Obviously, then, the discoveries of science have been late in coming because they had to be
learned through the long, arduous task of scientific experimentation. The Bible offered no
help, because its authors knew no more about these things than anyone else. In fact, the Bible
probably retarded the process of scientific discovery through the widespread acceptance of
superstitious nonsense found in it. Those who believe and practice superstition aren't the kind
of people who make scientific discoveries. Science advances through the efforts of people
who cast aside superstition and search for truth through application of scientific methods. This
is a characteristic not generally found in Bible believers.

An earlier article ("Scientific Boo-Boos in the Bible," Winter 1991) showed that the Bible,
rather than revealing amazing scientific insights, is riddled with scientific errors. These
mistakes cover a wide range of scientific areas but are most obvious in the field of biology.
The article noted the genetic ignorance of the Genesis writer, who presented Jacob as one who
was able to influence color patterns in Laban's sheep and goats by controlling the environment
in which they bred (Gen. 30:37-43). This is certainly a peculiar mistake for a book that is
supposed to be so wonderfully insightful in scientific matters. It is as if God told his inspired
writers all about the transmission of sound by electricity, the female reproductive system, the
spherical shape of the earth, and a host of other scientific secrets but neglected to reveal a
very basic genetic fact. Strange indeed! Many of the biological mistakes in the Bible were
anatomical in nature. The Leviticus writer (let bibliolaters think this was Moses if they want

Volume 1990 - 2002 Issue


Page 88 of 2049
Skeptical Review Edited by Farrell Till
to) was so unobservant, for example, that he apparently thought insects were four-legged
creatures:

All winged creeping things that go upon all fours are an abomination to you. Yet these may
you eat of all winged creeping things that go on all fours, which have legs above their feet,
with which to leap upon the earth; even these of them you may eat: the locust after its kind,
the bald locust after its kind, the cricket after its kind, and the grasshopper after its kind. But
all winged creeping things, which have four feet, are an abomination to you (Lev. 11:20-23,
BB).
Although the specific references to locusts, crickets, and grasshoppers in this passage indicate
that insects were the creatures under consideration, a curious thing about the Hebrew word
oph that is here translated "winged creeping things" is that it was the same word used six
times in the creation story (Gen. 1:20-30) to refer to birds. It is the same word used twelve
times in the Genesis account of the flood to refer to birds. In the KJV and ASV, the word is
translated birds or fowl(s) in all of these places. The KJV, in fact, even used fowls to open the
Leviticus passage cited above: "All fowls that creep, going upon all four, shall be an
abomination unto you."

Four-legged fowls! That would be a biological blunder indeed, but since the context clearly
indicated insects in this passage, we won't hold bibliolaters responsible for a translation flaw.
They have enough problems to deal with in this passage without adding another one. Suffice
it to say, however, that it does seem strange that a people to whom God routinely gave
insights into complex scientific matters like gynecology, hematology, telecommunications,
and aerodynamics would have no word in their language to distinguish birds from winged
insects. We are supposed to be impressed with the religious musings of a people no more
sophisticated than that?

An immensely greater problem than linguistic and translation flaws in this passage is the fact
that whoever wrote it consistently referred to winged insects as four-legged creatures, a
mistake that practically any modern-day elementary student would know better than to make.
What educated person today doesn't know that insects have six legs? We have to wonder why
God, who so routinely gave scientific insights to his inspired writers, couldn't at least have
opened the eyes of his earthly messenger in this case and had him count the legs on a
grasshopper.

Archer, Haley, Arndt, Torrey, and the other major inerrancy apologists don't even address the
problem of four-legged insects in their works, but knowing inerrancy defenders as I do, I can
almost predict what they will say about it. "Well, insects do have four legs, don't they? Just
because they happen to have a total of six legs doesn't mean that Moses had to include all six
in order to be scientifically correct. He chose to mention only four." Such an "explanation"
may sound strange to readers who are not familiar with the desperation tactics that
fundamentalists resort to to defend the inerrancy doctrine, but they often use this kind of
argument to "explain" numerical discrepancies in the Bible. Mark (5:1-20) and Luke (8:26-
39), for example, mention just one demoniac that Jesus healed in the country of the
Gerasenes, but Matthew, describing the same incident (8:28-34), put the location in the land
of the Gadarenes (several miles away from Gerasa) and said that there were two demoniacs.

Volume 1990 - 2002 Issue


Page 89 of 2049
Skeptical Review Edited by Farrell Till
Gleason Archer dismissed the geographical discrepancy as "scribal error," but of the
numerical discrepancy, he said this:

If there were two of them, there was at least one, wasn't there? Mark and Luke center
attention on the more prominent and outspoken of the two, the one whose demonic occupants
called themselves "Legion" (Encyclopedia of Bible Difficulties, p. 325).
Inerrantists use this same lame argument to explain why Matthew said that Jesus healed two
blind men at Jericho (20:29) but Mark (10:46) and Luke (18:35) mentioned only one who was
healed. As an argument, it grants entirely too much freedom of selection to the writers and
completely ignores the fact that they were presumably being verbally guided by the Holy
Spirit. Why then would the same Holy Spirit decide when he was "inspiring" Mark and Luke
that only one demoniac and blind man needed to be mentioned but when he was "inspiring"
Matthew, he suddenly decided that both demoniacs and blind men should be mentioned?

Whether our inerrantist readers will attempt to apply this line of reasoning to the Bible's four-
legged insects remains to be seen, but if they do, I hope they will address a question we have
every right to ask them. What is there about insects that would warrant writing a description
(like the one in the Leviticus passage) that mentions only four of their six legs? After all, this
was a legalistic description that was intended to let Jews know which insects were clean
(edible) and which were unclean (forbidden), and the description presented the clean locusts,
crickets, and grasshoppers as creatures that "go on all fours." But these insects don't "go on all
fours"; they go on all sixes. That's a strange oversight from an author writing under the
direction of an omniscient deity who routinely gave marvelous scientific insights to his
inspired crew.

But the insect problems aren't over. After declaring all "winged creeping things that go upon
all fours" an abomination, the Leviticus writer then made locusts, crickets, and grasshoppers
exceptions to this restriction. His rationale was that these were creeping things that go on all
fours, "which have legs above their feet" (v:21). So if insects that go about on all fours
(presumably with their other two immobilized) have "legs above their feet," they are clean
and can be eaten. If not, why not? That's the only reason the description gave for exempting
locusts, crickets, and grasshoppers from insects that were unclean or forbidden. Now I want
some enterprising inerrancy defender to give us a list of insects that don't have legs above
their feet. How could any creeping thing "go on all fours" without having legs above those
four (feet)? Feet without legs! It could happen only in Bible biology.

Another anatomical mistake was made by the Leviticus writer in the same context with his
four-footed insects. After stating the two characteristics that clean animals must have (part the
hoof and chew the cud), he declared hares and coneys unclean because they "chew the cud"
but do not part the hoof (vv:3-6). Deuteronomy 14:7 also described hares and coneys as cud-
chewers. The biological facts, however,are these: hares and coneys have no hoofs to part, but
they have no cuds to chew either. The Leviticus writer made a serious biological error in
describing them as cud-chewers.

"What About Scientific Foreknowledge in the Bible?" (Fall 1990) briefly discussed the
Leviticus writer's cud-chewing hares and coneys and the attempts of bibliolaters to explain
them away. Wayne Jackson, one of two staff members at Apologetics Press who were invited

Volume 1990 - 2002 Issue


Page 90 of 2049
Skeptical Review Edited by Farrell Till
to write a response to the article, declined the invitation but reviewed this section of the article
in Reason and Revelation (December 1989) prior to its publication in The Skeptical Review.
He resorted to the usual rationalizations: the words translated hare and coney "are rare and
difficult" in Hebrew; the writer was perhaps using "phenomenal language" to describe what
hares and coneys actually appear to be doing; etc. After all of this was said, however, a
proven biological fact still remained. Hares and coneys do not chew the cud.

In an end-run attempt to circumvent this problem, Jackson resorted to equivocation by


suddenly substituting the word ruminate for "chew the cud":

There is, however, another factor that must be taken into consideration. Rumination does not
necessarily involve a compartmentalized stomach system. One definition of "ruminate" is
simply "to chew again that which has been swallowed" (Webster). And oddly enough, that is
precisely what the hare does. Though the hare does not have a multi-chambered stomach,
which is characteristic of most ruminants, it does chew its food a second time. It has been
learned rather recently that hares pass two types of fecal material. "In addition to normal
waste, they pass a second type of pellet known as a caecotroph. The very instant the
caecotroph is passed, it is grabbed and chewed again.... As soon as the caecotroph is chewed
thoroughly and swallowed, it aggregates in the cardiac region of the stomach where it
undergoes a second digestion" (Jean Morton, Science in the Bible, pp. 179-181).
Unfortunately for Mr. Jackson's end-run, "chew the cud" is the expression that needs defining,
not "ruminate." The Hebrew word translated "cud" was gerah (cud), from garar (to bring up).
The word translated "chew" was alah (to cause to come up). Young's Literal Translation of
the Bible rendered the combination of the two words "bringing up the cud." Obviously, then,
the Leviticus writer was speaking of animals that chew the cud in the literal meaning of the
expression and not some figurative or "phenomenal" manner that bibliolaters might dream up
to protect their precious inerrancy doctrine.

If, however, Mr. Jackson is going to quote Webster's definition of ruminate, he should refrain
from doctoring it to suit his needs. In its entirety, Webster's definition of ruminate is "to chew
again what has been slightly chewed and swallowed." Jackson conveniently omitted the
underlined part of the definition, and in this respect hares certainly don't qualify as
"ruminants," because the caecotrophs of hares consist of materials that have been chewed
once and then passed through the digestive tract. This would hardly be material that has been
"slightly chewed and swallowed." Notice too that Jackson's reference states that "the
caecotroph is chewed thoroughly (by the hare) and swallowed." Are we to believe that hares
thoroughly chew the material in their caecotrophs but only slightly chew it the first time
through?

The main weakness in Jackson's caecotrophic solution to the problem of cud-chewing hares,
however, is its complete failure to explain away the biological error of the Leviticus writer.
After all has been said about what hares appear to be doing and how their reingesting of
caecotrophic materials achieves the same purpose as cud-chewing, the fact still remains that
hares do not chew the cud. Perhaps an analogy would underscore the ineffectiveness of
Jackson's resolution of the problem. The duck-billed platypus, a peculiar egg-laying animal
native to Australia, has been biologically classified as a mammal because the female nurtures
its young with milk. But the female platypus has no teats for her offspring to suck in order to

Volume 1990 - 2002 Issue


Page 91 of 2049
Skeptical Review Edited by Farrell Till
get the milk. There are glands on her stomach that "sweat" the milk, which her young then
suck from strands of hair that it has collected on. This unusual method of nurturing offspring
achieves the same purpose as the mammary glands of other mammals, but if one should say
that a platypus has teats with which she nurtures her young, he would be biologically
incorrect.

In the same way, the Leviticus writer was wrong when he said that hares and coneys "chew
the cud." That he intended this to mean true cud-chewing was indicated in his use of the
camel (11:4) as another example of a cud-chewing animal. Camels are anatomically equipped
with the same Ruminantia as cattle, goats, buffaloes, antelopes, giraffes, llamas, deer, and
bison. Camels are true cud-chewers, and the Leviticus writer's grouping them with hares and
coneys as examples of animals that "chew the cud" leaves little doubt about what he meant.
Perhaps he did superficially look at hares and assume from appearance that they were cud-
chewers, but that is hardly a satisfactory explanation of the problem. After all, inerrantists ask
us to believe that time and time again God gave to his inspired writers amazing insights into
complex scientific matters. He did all that but couldn't reveal to one of his writers a simple
fact about cud-chewing? It's too incredible to believe.

Jackson's final act of desperation was a claim that Grzimek's Animal Life Encyclopedia has
"classified the hare as a ruminant" and "considers the hyrax (coney) as a ruminant." His
reference (1975, pp. 421, 422) did not cite a volume number, but I read these page numbers,
as well as the entire sections about rabbits, hares, and hyrexes, in volume 12 and found no
attempt to classify either the hare or the hyrax as ruminants. If Mr. Jackson will send us a
specific reference and the exact quotation that classifies hares and hyraxes as ruminants, we
will publish it in a future issue. While he is at it, we would like for him to answer this
question: Do hares chew the cud? They either do or they don't, so there is no reason why he
can't give a yes or no answer to the question.

Some errors in Bible biology concerned behavioral misconceptions. Proverbs 6:7-8 described
the ant as an industrious creature, "which having no chief, overseer, or ruler provides her
bread in the summer, and gathers her food in harvest." No one disputes the ant's industry, but
what is this about its "having no chief, overseer, or ruler"? Inerrantists seem to like Grzimek's
Animal Life Encyclopedia, so I suggest that they read what it says about ants (Vol. 2, pp. 441-
453). The various species of this insect are therein presented as members of highly structured
social hierarchies having queens, workers, soldiers, and drones. Clearly, then, ants have
overseers and rulers. If inerrantists wish to dispute this, they should consider slave ants,
because some species of ants actually take captives in war and make them slaves. Surely, it
would be proper to speak of slave ants as having overseers or rulers. The Bible says, however,
that ants have no chiefs, overseers, or rulers. The Bible is wrong. Why didn't God instill in
this inspired writer's mind an insight into the social structure of ant colonies? Perhaps he was
too busy telling Job about the physics of sound transmission.

Even Yahweh himself was a little rusty in his understanding of animal behavior. In speaking
to Job from the whirlwind, he said this of the ostrich:

The wings of the Ostrich wave proudly; but are they the pinions and plumage of love? For she
leaves her eggs on the earth, and warms them in the dust, and forgets that the foot may crush

Volume 1990 - 2002 Issue


Page 92 of 2049
Skeptical Review Edited by Farrell Till
them, or that the wild beast may trample them. She deals harshly with her young ones, as if
they were not hers: Though her labor be in vain, she is without fear; because Eloah (God) has
deprived her of wisdom, neither has he imparted to her understanding (39:13-17, Bethel
Bible).
Reflected in this passage is a primitive, but incorrect, belief that the ostrich is a stupid bird
that lays its eggs on the ground, leaves them to be hatched by the heat of the sand, and then
treats her young harshly after they have hatched. The New American Bible affixes this frankly
honest footnote to what Yahweh said of the Ostrich:
It was popularly believed that, because the ostrich laid her eggs on the sand, she was thereby
cruelly abandoning them.
Modern biologists know better than what the "scientifically insightful" author of Job
mistakenly thought about the ostrich. Both Encyclopedia Americana and Britannica, as well
as Grzimek's (vol 7, pp. 91-95), describe ostriches as very caring parents. The female lays her
eggs on the ground, but so do many other species of birds. The eggs are not abandoned to the
heat of the sand, but in the female's absence, the male incubates the nest. When the young
hatch, they are given watchful care by their mother. As a biological creature, the ostrich has
survived for thousands of years, so obviously it is a successful procreator. Its labor is not in
vain, as the passage above incorrectly declares. Yet Yahweh himself, who presumably created
all living things, didn't know these behavioral facts about the ostrich. He "inspired" Jeremiah
to perpetuate the primitive misconception of the ostrich's careless maternal instincts by having
him write this about the women of Israel:
Even the jackals draw out the breast, they give suck to their young ones: The daughter of my
people has become cruel, like the ostriches in the wilderness. The tongue of the sucking child
clings to the roof of his mouth for thirst: The young children ask bread, and no man breaks it
to them (Lam. 4:3-4, BB).
Amazing scientific foreknowledge in the Bible? Hardly! Bibliolaters should stop trying to
find insightful statements about electronics, oceanography, meteorology, etc. in the Bible text
and worry more about explaining why a divinely inspired, inerrant book has so many obvious
scientificerrors in it. And if the Bible is riddled with scientific errors, they should wonder too
about the truth of that often parroted claim that the Bible is inerrant in all details of history,
geography, chronology, etc., as well as in matters of faith and practice. It just ain't so!

Recommended Publication
Biblical Errancy is a monthly paper that also exposes the myth of Bible inerrancy. Although
we don't agree with everything in it, we do recommend it for people interested in learning
more about discrepancies in the Bible. Subscription cost is $9 per year. Other inerrancy
materials are also available. The address is 3158 Sherwood Park Drive, Springfield, OH
45505.

Inerrancy Debates Available

Volume 1990 - 2002 Issue


Page 93 of 2049
Skeptical Review Edited by Farrell Till
The unfinished Laws-Till Debate, which the fundamentalist James H. Laws withdrew from
after three manuscript exchanges, is available at $2 ppd. A complete debate between Bill
Jackson, a Church-of-Christ preacher, and Farrell Till is now in print at $2.25 ppd. Both can
be ordered at the address on page two.

Volume 1990 - 2002 Issue


Page 94 of 2049
Skeptical Review Edited by Farrell Till

The Skeptical Review


Volume Two, Number Three
July/August/September 1991
Farrell Till, editor

• The Deaver-Till Debate


Farrell Till's account of his debate against Church-of-Christ preacher Mac Deaver on
Bible morality.

• Yahweh, The God of Gods


Were the Hebrews unique in practicing a strict monotheistic religion? According to
Farrell Till, the early Hebrews "believed in polytheism as much as the nations around
them."

• Jehovah, The One and Only God


Can somone speak of other gods without affirming their existence? According to Mac
Deaver, although the writers of the Old Testament spoke of other gods, they were not
polytheistic.

• The Wisdom of Solomon


Was Solomon really as wise as Christians think? Sandra Till says that some of the
biggest absurdities in the Bible concern "the so-called wisdom of Solomon."

• More Problems for Bibliolaters


Is the Bible so harmonious that it can only be explained by divine inspiration? Farrell
Till writes that "bibliolaters cannot harmonize the Bible without resorting to scenarios
so preposterously far-fetched that only the very credulous can believe them."

• Till-McDonald Debate

• Failed Prophecies

Volume 1990 - 2002 Issue


Page 95 of 2049
Skeptical Review Edited by Farrell Till
• Other Debates & Back Issues

The Deaver-Till Debate


On March 25-28, Farrell Till met Mac Deaver, a Church-of-Christ preacher from Austin,
Texas, in oral debate on the campus of Southwest Texas State University in San Marcos. The
issue was Bible morality. Deaver affirmed that none of the acts done or authorized by the God
of the Bible were moral atrocities; Till affirmed that some acts attributed to the God of the
Bible were actual atrocities that disprove the doctrine of divine inspiration.

At no time during the two nights he was affirming did Deaver examine any of the more
infamous Bible events, such as the massacre of the Midianites and Amalekites, to show that
they should not be considered atrocities. The debaters having agreed to exchange five
questions prior to each night's session, Deaver immediately tried to put Till on the defensive
by discussing arbitrarily conceived inconsistencies in Till's answers to the first set of
questions. Specific cases of questionable moral conduct on Yahweh's part had to be
introduced by Till.

The questions directed to Till had been designed to trap him in contradiction concerning his
position on objective or absolute morality. In his answers, Till had rejected Deaver's concept
of objective morality but had nevertheless recognized the existence of moral standards that
man has an obligation to respect. Deaver perceived this as a contradiction and spent much of
his time from then on pressing the point. Till's response to this was to show that determination
of good and bad has to be an intellectual process. He cited Romans 2:14, where it was said
that the Gentiles who "have not the law" (revealed objective morality) "do by nature the
things of the law," to show that even the Bible recognizes that people can by natural processes
discover moral principles without having them divinely revealed.

By the third night when he assumed the affirmative, Till had forced Deaver to take the
position that God can do anything, even kill babies, and it would be morally right. Till urged
the audience to consider the consequences of that position. Rather than to think of the
Midianite and Amalekite massacres in vague, abstract terms, he asked that these events be
considered in the specific, concrete terms of what had to occur if both events happened as
recorded in the Bible. Thirty-two thousand Midianite virgin girls were kept alive for the
sexual whims of their captives; this meant that 32,000 specific cases of sexual abuse of
children had to occur. It also implied that about 32,000 specific male children were killed to
comply to Moses' order to "kill every male among the little ones" (Num. 31:17). The
Amalekite massacre, if it happened according to the Bible account, meant that there were
hundreds or even thousands of specific incidents in which specific Amalekite women,
children, and babies were run through by Israelite swords. Till urged Deaver to tell the
audience if each of these specific killings of women and children was a "good thing."

Deaver would not answer the question directly, but the substance of what he said in response
indicated that he thought these deeds were all expressions of God's perfect morality. Till then

Volume 1990 - 2002 Issue


Page 96 of 2049
Skeptical Review Edited by Farrell Till
pressed the audience to bear in mind that Deaver's position forces one to believe that killing
babies is a morally good act. In the main, Deaver's only defense of this charge was ridicule.
"God didn't do things quite the way that Farrell Till would have," he chided Till, "so therefore
they were wrong."

Audio ($15) and video ($30) tapes of the debate can be ordered from Thomas Gardner, 2229
Lotus, Ft. Worth, TX 76111 (Tel. 817-838-0828). Both men have agreed to debate the same
subject at a later date.

Yahweh, The God of Gods


Farrell Till
Many Bible fundamentalists believe that while the nations around them wallowed in the mire
of polytheism the Hebrews practiced a strict monotheistic religion. Their insight into the
nature of the one true God Yahweh had resulted, of course, from the personal relationships
that Abraham and the other Hebrew patriarchs had experienced with Yahweh, who had
routinely revealed himself to them in dreams, apparitions, and other manifestations. It makes
good sermon material, but there's just one thing wrong with it. It isn't true.

The early Hebrews believed in polytheism as much as the nations around them. They thought
of Chemosh, Molech, Milcom, Baal, Dagon, and the other pagan gods as deities who were
just as real as their own god Yahweh. They just thought that Yahweh was greater and
mightier than the others, a sort of supergod or, in other words, the God of gods (Josh. 22:22).
Monotheism or the belief that Yahweh was the only God was a late development in Jewish
theology.

The evidence for this is too clear to dispute. There is, first of all, the peculiar fact that the
Hebrews, when not referring to him by his personal name Yahweh, generally used a plural
word (elohim) to designate their god. Literally, it meant gods rather than god. In the original
Hebrew, therefore, Genesis 1:1 is actually saying, "In the beginning gods created the heavens
and the earth." It seems strange that a people with a clear concept of monotheism, as
bibliolaters claim that the Hebrews had, would have used a plural word in referring to the one
and only true god. It would be somewhat like an English writer using men to refer to a man.

Bible writers did in fact often use the singular word el (god) in obvious reference to Yahweh.
Genesis 21:23 states that "Abraham planted a tamarisk tree in Beer-sheba, and called there on
the name of Yahweh, the Everlasting El" (Bethel Translation). In Genesis 31:13, an "angel of
God" (elohim) appeared to Jacob in a dream and said, "I am the El of Bethel...." Other
instances when Yahweh Elohim was called El can be found in Genesis 35:1,3; 43:14; 46:3;
48:3; 49:25; Exodus 15:2; 20:5; 34:6 and numerous other places. It happened enough to
indicate that Bible writers had some difficulty deciding whether to call their Yahweh elohim

Volume 1990 - 2002 Issue


Page 97 of 2049
Skeptical Review Edited by Farrell Till
(gods) or el (god). To say the least, this does not indicate a clear grasp of monotheistic
concepts.

Bibliolaters will quickly protest that the Hebrews used the plural word elohim when referring
to their god Yahweh only to show awe and respect. It was "the plural of dignity," they claim,
a way of expressing the majesty and greatness of God. Some even think they see an early
recognition of the triune godhead in the plural term elohim. In Genesis 1:26, Elohim said,
"Let us make man in our image, after our likeness," and after Adam and Eve had sinned,
Yahweh Elohim said, "Behold the man is become as one of us" (Gen. 3:22). What could these
statements be, bibliolaters ask, except the three persons in the one godhead talking?

In this article, I won't get involved in discussing the absurdities of the trinity doctrine except
to say that the Hebrew usage of elohim to designate their tribal god could very well have been
a vestigial expression from their distinctly polytheistic days. One thing is sure: Old Testament
writers often seemed confused about whether they intended the word elohim to mean their
god Yahweh or gods in a definite plural sense. When Yahweh alone was meant, they usually
referred to him as Elohim without the article ha (the), and if Elohim (Yahweh) was the subject
of the sentence, a singular verb was used even though elohim was a plural noun. The creative
god of Genesis 1, for example, is called Elohim, without the article ha (the), some thirty
times. In places like Exodus 12:12, however, where "the gods of Egypt" were referred to, the
same word elohim was used but with the article ha, ha-elohim (the gods). In Genesis 35:7,
English translations state that Jacob built an altar at Bethel "because there God was revealed
to him," but the Hebrew text literally states that the gods (ha-elohim) were revealed (niglu).
The addition of the u sound to a Hebrew verb made it plural much in the same way that the
addition of an "s" to a verb in English makes it third- person singular, so in this case, the Bible
was really saying that the gods were revealed to Jacob, not God was revealed to him. If space
permitted, I could cite many examples like this where English translations have deceptively
rendered haelohim as God and its plural verbs as singulars. Most English readers have not
researched the Bible enough to be aware that these things have been done; hence, they naively
believe that the Hebrews had a consistently monotheistic concept of God all through their
history when in reality monotheism was a late development in their theology.

There are many passages in the Old Testament that indicate belief that the pagan deities were
real gods. Jephthah said in his message to the king of the Ammonites during a dispute over
territory the Israelites had taken on their way out of Egypt, "Will you not possess that which
Chemosh your elohim gives you to possess? So whomever Yahweh our Elohim has
dispossessed from before us, them will we possess" (Judges 11:24, BB). Since there were no
capital letters in Hebrew to show the distinction the translators arbitrarily made in capitalizing
elohim as it referred to Yahweh, it is obvious that Jephthah considered Chemosh of the
Ammonites to be elohim in the same sense that Yahweh was the elohim of Israel. He was
contending that Yahweh, his elohim, had given the Israelites certain territories just as
Chemosh, the elohim of the Ammonites, had given them certain lands and that the two
nations should therefore be content with the arrangements of their respective gods.
Furthermore, we have to wonder at this point if Jephthah intended elohim as a "plural of
dignity" when he applied it to the singular deity Chemosh. If not, why not? If it expressed
dignity and respect when applied to Yahweh, then why would it not mean the same when
applied to another deity? So if there is any merit at all to the plural-of- dignity argument, we

Volume 1990 - 2002 Issue


Page 98 of 2049
Skeptical Review Edited by Farrell Till
have in this passage a clear indication that Chemosh was considered a real god who deserved
respect.

That pagan gods should indeed be respected was often indicated in the Old Testament.
Exodus 22:28 says, "Thou shalt not revile the gods (ha-elohim), nor curse the ruler of thy
people" (KJV). Despite the inclusion of the article ha, as shown in the parentheses, most
translations have tried to hide the fact that gods in general were probably intended by
rendering ha-elohim God (singular) with a capital "G" and no article. Deliberate deceptions of
translation like this have kept English readers from seeing many things that would be
damaging to traditional Judeo-Christian doctrines, in this case an apparent polytheistic
concept in early Hebrew history.

Leviticus 24:10-23 tells the story of the son of an Israelite-Egyptian marriage who had been
heard blaspheming "the Name" during a fight. The man was put in ward until what should be
done to him "might be declared to them at the mouth of Yahweh" (v:12). Upon inquiring,
Moses was told by Yahweh to have the congregation stone the man to death. "And you shall
speak to the children of Israel, saying," Yahweh declared, " Whoever curses his Elohim shall
bear his sin. And he that blasphemes the name of Yahweh, he shall surely be put to death"
(vv:15- 16). The capitalization of elohim in this passage was a purely arbitrary interpretation
of the Bethel translators, because there were no capital letters in Hebrew, so the word could
just as well have been translated gods: "Whoever curses his gods shall bear his sin...."

Is there any reason to believe that the plural concept of gods was intended in the statement?
There very definitely is. Two distinct offenses seem to have been under consideration: (1)
whoever curses his gods shall bear his sin, but (2) he that blasphemes the name of Yahweh
shall surely be put to death. In other words, cursing one's gods was just considered a sinful
offense, but cursing the name of Yahweh was an offense punishable by death. The text
implies that the man who was charged in this case wasn't a Hebrew. Although his mother was
an "Israelitish woman," his father was Egyptian. That he possibly believed in Egyptian gods
was suggested in the last half of verse 16 when Yahweh said that "as well the sojourner, as the
homeborn,when he blasphemes the name of Yah-weh, (he) shall be put to death." This man
may have been a sojourner (foreigner), but notice was being served by his execution that a
more serious penalty would be extracted for blaspheming Yahweh than for cursing other
gods. So whatever dubious value this fanciful little tale might have, it at least seems to be
saying that the Hebrews thought pagan gods were real. If not, why would they have
considered it sinful to curse gods that didn't even exist?

Passages in the Old Testament that show an early Hebrew belief in polytheism are too
numerous to examine in detail. I can cite only a few random ones. After the Israelites had
crossed the Red Sea, for example, they sang a hymn of praise to Yahweh in which they said,
"Who is like unto you, O Yahweh, among the elohim (gods)?" (Ex. 15:11). So obviously was
the word elohim intended in this verse to convey the concept of gods in general that even the
biased Bethel translators have printed it with a lowercase "e," but unless the Hebrews who
sang these words believed that other gods existed, it would have made no sense at all for them
to ask who among the gods was like unto their god Yahweh. In Psalm 95:3, it was declared
that "Yahweh is a great El (god) and a great King above all elohim (gods)." But how could
this psalmist have believed Yahweh was greater than other gods unless he believed that other

Volume 1990 - 2002 Issue


Page 99 of 2049
Skeptical Review Edited by Farrell Till
gods existed to compare Yahweh to? Psalm 86:8 declared, "There is none like you among the
elohim, O Yahweh." However, if the psalmist thought that Yahweh was the only god, his
words of praise were completely meaningless. It would be as if someone said of the Eiffel
Tower, "There are no Eiffel Towers like unto the Eiffel Tower." To say, however, that there
are no towers like unto the Eiffel Tower grants clear recognition that other towers exist, and
so it was when the Hebrews said that there were no gods like their god Yahweh. They were
clearly indicating their belief that other gods existed.

Even as late as Solomon, belief in the reality of pagan gods still persisted. In declaring his
plans to build a temple to Yahweh, Solomon said, "Great is our God above all gods" (2
Chron. 2:5). How could he have thought his god was greater than the other gods unless he
believed other gods existed? Since in this case Solomon himself eventually resorted to
idolatry (1 Kings 11:4-8), he very obviously believed pagan gods were real. In this respect,
Solomon wasn't at all unusual. Throughout the Old Testament, Yahweh was compared to
other gods in ways that showed a belief in the realness of the others. He was called "God of
gods and Lord of Lords, a great God" (Deut. 10:17), but how could he have been God of gods
unless other gods existed? The same comparison was made in Joshua 22:22 and Psalm 136:2-
3. To the Hebrews, Yahweh was simply "God of gods," the greatest and mightiest of many
existing gods. To deny this is to make all the words of Yahwistic praise like those just quoted
completely meaningless.

Fundamentalists will of course point out that many Bible passages clearly teach that Yahweh
was the one and only God. At the dedication of the temple, Solomon said to the people that
"Yahweh is God, and there is none else" (1 Kings 8:60). (This was the same Solomon who
shortly afterwards worshipped other gods, so we have to wonder just how strongly he
believed what he said.) Moses also said that "Yahweh is God; there is no other beside him"
(Deut. 4:35). So no one will dispute that the Bible in many places says that there is only one
God, but trying to disprove that polytheism was believed by some Bible characters and
writers by just quoting passages that clearly teach monotheism is to miss the point entirely.
The contention of The Skeptical Review is that, contrary to what fundamentalists preach from
their pulpits, the Bible is an inconsistent, contradictory book. The conflicting polytheistic-
monotheistic views of its writers is just one example of its inconsistency and contradiction, so
bibliolaters can't satisfactorily explain the problem by simply referring to the passages that
appear to teach monotheism. Pitting scripture against scripture in this way only confirms the
premise on which this publication was founded: there are obvious contradictions in the Bible.
To satisfactorily resolve this matter, they will have to show that the passages I have presented
and explicated in this article don't really teach polytheistic concepts.

I don't think they can do that. In Exodus 12:12, Yahweh said that on the night of the Passover
he would execute judgment "against all the gods of Egypt." But how can judgment be
executed against something that doesn't even exist? This is what bibliolaters must explain,
because whoever wrote Exodus 12:12 clearly believed that the gods of Egypt were real gods.

Volume 1990 - 2002 Issue


Page 100 of 2049
Skeptical Review Edited by Farrell Till

Jehovah, The One And Only God


Mac Deaver
In Farrell Till's article. "Yahweh, the God of Gods," he took the position that "(m)onotheism
or the belief that Yahweh was the only God was a late development in Jewish theology." Mr.
Till affirms that the early Hebrews believed in polytheism.

Let it first be said that Abram was the first person referred to as a Hebrew. In fact, he is "the
Hebrew" (Gen. 14:13). From the time that Jehovah called Abram out of Ur of the Chaldees to
the time of his death, there is no evidence of his adopting a polytheistic stance. He was one
who trusted the pronouncements of Jehovah, and he was characterized by unwavering
obedience (cf. Gen. 15:6; 22:9-19; Rom. 4:16-25). He recognized Jehovah as the Judge of all
the earth (Gen. 18:25).

That some of Abram's descendants came to be influenced by pagan views, this writer would
not begin to deny. At times many Israelites adopted views of their neighbors. But to suggest
that monotheism was a late development in Jewish theology is wrong.

Even Jehovah would speak of "gods." But such did not mean that He was affirming their
actual or ontological existence. He forbade Israel's having "gods" of silver or gold (Ex.
20:23), reference being made not to metaphysical existents but to idols. He would tolerate no
graven images (Ex. 20:4-5). When Israel attempted to identify Jehovah with idols or when she
adopted pagan views regarding the actual existence of some other god or gods other than
Jehovah, she sinned (Ex. 20:3,23; 32:4ff; Num. 25:1-5).

Mr. Till thinks that the use of Elohiym in Genesis 1 indicates that the Bible writer was
polytheistic in thinking. But this is a mistake. Though Mr. Till discounts it, it is still true that
reference is being made to the Godhead (cf. Col. 2:9; Acts 17:29). Verses 26-27 indicate
plural personality sharing divine essence or nature. Clearly, the words "Let us make man in
our image" would be confusing if plural personality were not intended. Elohiym when
referring to Jehovah refers to that plural personality aspect of the One ultimate divine essence.
The word has both singular and plural uses. That is, this plural form word can be used with
reference to several gods (Ex. 18:11), or it can refer to only one (1 Sam. 17:26).

Mr. Till suggests that "Bible writers did in fact often use the singular word el (god) in obvious
reference to Yahweh." But Bible writers also used the same singular form word to refer to a
false god (Is. 43:10) that had no actual existence.

To suggest that Bible writers didn't have a clear concept of the one God because they would
use El and then Elohiym is an inaccurate assessment. The basic idea conveyed by El seems to
be that of power. But Elohiym when referring to Jehovah would capture the idea of plural
personality. God is the sum of His parts. There are three parts which equal one God.

Mr. Till said, "In this article, I won't get involved in discussing the absurdities of the trinity
doctrine except to say that the Hebrew usage of elohim to designate their tribal god could very

Volume 1990 - 2002 Issue


Page 101 of 2049
Skeptical Review Edited by Farrell Till
well have been a vestigial expression from their distinctly polytheistic days." But the "could
have" support for his view is no proof.

The fact that a Bible writer will use El and then Elohiym in the same context in reference to
the same Being shows that the procedure is (1) deliberate, (2) significant, and (3) consistent.

Consider Genesis 35:1-2:

And God (Elohiym, MD) said unto Jacob, Arise, go up to Bethel, and dwell there: and make
there an altar unto God (El, MD) that appeared unto thee when thou fledest from the face of
Esau thy brother. Then Jacob said unto his household, and to all that were with him, Put away
the strange gods (elohiym) that are among you....
Surely, no one wants to contend that the God who talked to Jacob and instructed him to build
an altar was authorizing Jacob to build it to an entirely different God. And yet, without
hesitancy, both El and Elohiym are employed. And then in verse 2, foreign or strange gods
are referred to by elohiym as well. Thus, Elohiym can be utilized in reference to plural false
gods or to plural personalities composing the one true God.

When Mr. Till says, "If space permitted, I could cite many examples like this where English
translations have deceptively rendered ha-elohim as God and its plural verbs as singulars," he
wrongly makes accusation. Consider that in Genesis 35:1 it is said that God El appeared to
Jacob when he fled from Esau. In verse 7 it is said that God (Elohiym) appeared to him when
he fled from Esau. So, both a singular form word and a plural form word are used to refer to
the same divine Being. Since this God claims to be One in actual divine nature (Dt. 6:4), there
would be no point to represent Him in language that would suggest a polytheism. And the
writer of the first five books of the Bible, when referring to God and to gods, must be
understood in the light of his own view that there were actually existing no gods at all.
Jehovah does exist (Dt. 4:35).

Mr. Till suggests that in Exodus 22:28 "the gods" is probably intended to be "gods in
general." I would suggest that it would be more in keeping with other pronouncements of
Jehovah as well as with the remainder of the verse to suggest that "the gods" in this passage
refer to human leaders (cf. Ex. 21:6; Ps. 82:6; Jn. 10:34).

The position is taken by Mr. Till that in Leviticus 24:15 "God" (Elohiym, MD) should be
translated "gods." This writer concurs with the translation "God" as referring to the Lord (cf.
v. 16). If "gods" were the correct translation, it would be a reference to human leaders, not to
false foreign gods.

Mr. Till cites several passages that, according to him, prove an early Hebrew belief in
polytheism. He referred to Exodus 15:11, Psalm 95:3, Psalm 86:8, II Chronicles 2:5, I Kings
11:4-8, Deuteronomy 10:17, Joshua 22:22, and Psalm 136:2-3. And he then pointed out that
certain passages like I Kings 8:60 and Deuteronomy 4:35 affirm the existence of only one
God. So Mr. Till concludes that the Bible contradicts itself. He concluded his article by saying
that "whoever wrote Exodus 12:12 clearly believed that the gods of Egypt were real gods."

Volume 1990 - 2002 Issue


Page 102 of 2049
Skeptical Review Edited by Farrell Till
Without going into a detailed response to each passage, I desire here to merely make several
observations that must be kept in mind when dealing with these passages.

ONE: It is a fact that the Old Testament teaches the actual existence of only one God (Dt.
4:35,39; 6:4; 32:39). Any interpretation of any passage that puts Bible writers in contradiction
with themselves in this basic affirmation is forced, unnecessary, and unwarranted.

TWO: The one true God is composed of three personalities, the sum of whose parts
constitutes God (Gen. 1:26-27; Col. 2:9; Acts 17:29; Matt. 28:19).

THREE: The Bible provides accounts of action that involve men (at times even Israelites)
who are worshippers of a false god or gods (Num. 25:1-5; Acts 14:8-18; 17:16-34).

FOUR: When a Bible writer records the failure of Israelites to faithfully adhere to the worship
of the one true God, such records do not show that the writer himself is involved in such
failure (Jer. 9:14; 17:1-2).

FIVE: Bible writers at times compare God with gods, but such a comparison in language does
not commit the writer to affirming the actual existence of any god. Such a comparison is
made because the "gods" are thought to exist by some, and the comparison made indicates the
powerfulness of God, and the powerlessness of the gods. A Bible writer can do what Elijah
did (I Kings 18). Elijah said to the prophets of Baal, "And call ye on the name of your god,
and I will call on the name of Jehovah: and the God that answereth by fire, let him be God..."
(v. 24). Such terminology does not prove that Elijah actually thought that Baal had real
ontological existence at all, but that a comparison between Baal and Jehovah was intended at
the contest on Mt. Carmel is clear. Some thought Baal was real, and so Elijah's comments
indicated that, but he never affirmed the existence of such (cf. v. 21).

SIX: The failure of pagans (or Israelites even) to realize that Jehovah was the only God and
not merely a superior god in no way implicates Bible writers as participants in the failure. At
times, pagans thought of Israel's God as simply one among many (I Kings 20:23; Num. 14:15-
16; II Chron. 32:9-14). Some non-Jews came to realize that Jehovah was the only God (cf.
Josh. 2:11; II Kings 5:17-19).

SEVEN: If Jehovah could speak of "gods" and not affirm their actual existence, the Bible
writers could do the same. Israel was to have no other gods (Ex. 20:3). This was a prohibition
against all false gods (gods--supposed deities who are not real). There are, according to Paul,
"gods many, and lords many" as far as human thinking goes, and yet there really is one God,
the Father, and one Lord, Jesus Christ (I Cor. 8:5-6). But one can talk of "gods" as concepts in
the minds of men without affirming their actual existence.

The real issue between Mr. Till and me on this topic is not whether at some point non-Jews
and Jews themselves were polytheists but whether when a Bible writer speaks of "gods" he
has automatically committed himself to the affirmation of their actual existence as ontological
beings. If Jehovah can claim to be the only God and yet speak of "gods" (thus not affirming
their existence), then no one can prove that a Bible writer when speaking of "gods" must
mean that they really do exist as actual beings.

Volume 1990 - 2002 Issue


Page 103 of 2049
Skeptical Review Edited by Farrell Till
(Mac Deaver's address is 1200 Bowie, Wellington, TX 79095.)

EDITOR'S NOTE: In reference to my suggestion that the Hebrew usage of the plural elohim
could have been "a vestigial expression from their distinctly polytheistic days," Mr. Deaver
said that "'could have' support for his (Till's) view is no proof" and then proceeded to give us
could-have, would-be, and seems-to-be "explanations" of why apparent polytheistic
applications of elohim didn't necessarily mean that the writers believed in the real existence of
gods. He theorized, for example, that Bible writers in using both el and elohim to refer to
Yahweh seemed to be making a distinction between his power and his plural personality. By
his own rule of hermeneutics, however, he can't resort to this explanation as a way out of the
problem, because "seems-to-be support" for his view is no proof.

Hebrew scholars do think that the word el was derived from a root that meant "strength or
power," but elohim is a plural derivative of the same root. A plural of anything is always more
or greater than a singular, so why would the Hebrew writers have resorted to the singular el to
convey an idea of power when the plural could have conveyed it even better? In this seems-
to- be solution, Mr. Deaver actually has no solution at all. We still must wonder why Bible
writers seemed confused about whether to call Yahweh elohim or simply el.

Mr. Deaver asserts that the usage of el and elohim "in the same context in reference to the
same Being shows that the procedure is... deliberate." In this, he shows a profound
misunderstanding of the mechanics of writing. In twenty-six years of teaching college writing,
I have learned that writers unconsciously make all sorts of mistakes in the struggle to transfer
ideas from the mind to the writing surface. In the same context, they will use both singular
and plural pronouns to refer to the same antecedent. This process is not deliberate; it is an act
of carelessness done while the writer is striving to shape his thoughts into comprehensible
language.

How then can we know that the use of both el and elohim in the same context to refer to the
same entity was the result of deliberation rather than the confusion of writers who lacked
clear concepts of monotheism? Mr. Deaver quoted Genesis 35:1-2 as an example of where
Yahweh was called elohim in one sentence and then el in the next. In the same context,
elohim was used in reference to the "strange gods" in the land Jacob was then living in. Mr.
Deaver doesn't see anything confusing about this?

In this passage, Yahweh was speaking (allegedly) and referred to himself as both elohim and
el. "Surely," Deaver said, "no one wants to contend that the God who talked to Jacob and
instructed him to build an altar was authorizing Jacob to build it to an entirely different God."
Deaver then cited other passages where Yahweh (allegedly) referred to pagan gods as elohim.
"If Jehovah could speak of 'gods' and not affirm their actual existence," Deaver concluded,
"the Bible writers could do the same." However, I must call Mr. Deaver's attention to the
word I have twice used parenthetically in this paragraph. Yahweh allegedly referred to
himself as both el and elohim, and Yahweh allegedly used elohim in reference to pagan gods.
Just because a Bible writer said that Yahweh said thus-and-so doesn't automatically make it
true. It just could be that the ones who wrote these passages only thought that Yahweh had
appeared to so-and-so and said this- or-that. Mr. Deaver wants us to give him the concession
all fundamentalists insist upon. He wants us to assume that everything the Bible says, no

Volume 1990 - 2002 Issue


Page 104 of 2049
Skeptical Review Edited by Farrell Till
matter what it may be, is absolute truth. But we won't concede him that advantage. He cannot
prove Bible inerrancy by simply assuming Bible inerrancy.

He said "(i)t is a fact that the old Testament teaches the actual existence of only one God" and
then concluded from this that &qut;(a)ny interpretation of any passage that puts Bible writers
in contradiction with themselves in this basic affirmation is forced, unnecessary, and
unwarranted," but to take such a position is a resort to the logical fallacy I just noted: trying to
prove Bible inerrancy by simply assuming Bible inerrancy. I have admitted (p. 4) that many
passages in the Bible clearly teach monotheism. That does not mean, however, that other
passages do not convey polytheistic concepts. To argue that no passage in the Bible could
possibly be teaching polytheistic concepts because some passages undeniably teach
monotheism is to assume that the Bible does not contradict itself. This is unsound, illogical
reasoning. The meaning of an idea must be determined within the context of what that writer
said and not by comparison to what another writer said somewhere else. On the matter now in
dispute, it would be possible that one writer was polytheistic whereas the other was
monotheistic.

Of all the shortcomings of Deaver's rebuttal, however, the most damaging was his failure to
explain away statements made by Bible characters and writers that showed an obvious belief
in the reality of pagan gods. To express the majesty and greatness of Yahweh, Bible writers
called him "the God of gods," but if there were no other gods, the comparison is totally
meaningless. Although he sacrificed his own daughter as a burnt-offering, Jephthah was
nevertheless listed as a great hero of faith in Hebrews 11:32. He considered Chemosh to be
elohim to the Amorites as much as Yahweh was elohim to the Israelites. He thought that
Chemosh had given the Amorites certain territory to possess in the same way that Yahweh
had given the Israelites a territory to possess (Judges 11:24). Passages like this pose a serious
problem for Mr. Deaver that he did not and cannot explain away.

The Wisdom of Solomon?


Sandra Till
The Bible is the inspired, inerrant word of God? As a child growing up in the '40s in
Southeast Missouri, completely surrounded by the fundamentalist mentality typical of both
the era and area, I believed so. I had complete faith in the "word of God." Over a period of
several years of conscientious Bible study, however, I became more and more aware of, and
bothered by, the many discrepancies, inconsistencies, and outright absurdities in the book.
Several absurdities concerned the so-called wisdom of Solomon.

Solomon--the man whom God himself declared had no equal when it came to wisdom!
According to the "inspired text," none before him, none after him (presumably this would also
include Jesus Christ) did or would compare to the wisdom God granted to Solomon:

Volume 1990 - 2002 Issue


Page 105 of 2049
Skeptical Review Edited by Farrell Till
Lo, I (God) have given you an understanding heart; so that there has been none like you
before you, neither after you shall any arise like you (1 Kings 3:12, Bethel Bible).

And Elohim gave Solomon wisdom and understanding very much, and largeness of heart,
even as the sand that is on the seashore. And Solomon's wisdom excelled the wisdom of all
the children of the east, and all the wisdom of Egypt (1 Kings 4:29-30, BB).

Well, Solomon's brand of wisdom as revealed in the Bible is totally unimpressive to me and
surely to many other discerning readers too. Let's look at a few examples of the incredible
wisdom of Solomon--the wisest man ever born!

To say the very least, Solomon was a man of excesses. The biblical account of his
conspicuous consumption makes him appear ludicrous and even morally corrupt. His
opulence was totally unfettered. If it was available, Solomon apparently had to have it. The
best woods, the finest metals, precious stones, ivory, exotic animals, the best of everything
from splendid houses to chariots, spices, and gold--nothing luxurious escaped his cravings. In
view of the quantity of food served daily at his table and the excessive (was he anything but?)
sacrificial offerings to his god, one could even say his ostentation extended to what surely
amounted to near decimation of the animal population of that region:

And Solomon's provision for one day was thirty measures of fine flour, and sixty measures of
meal, ten fat oxen, and twenty oxen out of the pastures, and a hundred sheep, besides harts,
and gazelles, and roebucks, and fatted fowl (1 Kings 4:22-23, BB).
His voraciousness was such that he required the services of twelve regional officers just to
provide this amount of food for his household (v:7)!

At the "great high place" in Gibeon, he once offered a thousand burnt offerings on the altar
there (1 Kings 3:4), and at the dedication of the temple, his offerings were so numerous that
they "could not be counted or numbered for multitude" (1 Kings 8:5). The bronze altar in the
newly built temple couldn't even accommodate them:

And Solomon offered for the sacrifice of peace-offerings, which he offered to Yahweh,
twenty-two thousand oxen, and a hundred and twenty thousand sheep. So the king and all the
children of Israel dedicated the house of Yahweh. The same day the king hallowed the middle
of the court that was before the house of Yahweh; for there he offered the burnt-offering, and
the meal-offering, and the fat of the peace-offerings, because the bronze altar that was before
Yahweh was too little to receive the burnt-offering, and the meal-offering, and the fat of the
peace- offerings (1 Kings 8:63-64, BB).
Never mind that we are given a specific number of animals in this passage after having been
told that the offerings were too numerous to be counted; the thing we are interested in at this
point is Solomon's excesses. This is the tale of a man whose daily life makes Lifestyles of the
Rich and Famous look piddling in comparison, and Robin Leach isn't even telling this story.
God's "inspired" writer is! This obnoxious yarn might make acceptable TV entertainment, but
to include it in the book that is supposed to provide mankind with the keys to eternal life and
set an example for us to follow is ludicrous in the extreme. One might rather choose to
emulate Mahatma Gandhi (whose wisdom, of course, didn't come close to Solomon's) than
this immature, egotistical, decadent man to whom God granted supreme wisdom.

Volume 1990 - 2002 Issue


Page 106 of 2049
Skeptical Review Edited by Farrell Till
One has to question too the reason for this grandiose sacrificial ceremony. Was it done to
please God? Did Yahweh really want, not even to say expect, such a vulgar display of
reverence? If so, we have to wonder about the ego of this god we are supposed to love and
serve with all our heart and soul and mind. A god who delighted in seeing the carcasses of
22,000 oxen and 120,000 sheep set ablaze to pay homage to him! Come on, people, let's get
real! This is 1991.

Solomon's wisdom was legendary? Well, let's just take a look at his best known display of
that wisdom--the famous "Case of the Disputed Baby." Two women living together had given
birth to sons within a three-day period. When one of the babies died in the night, the mother
switched his body with the other baby. Naturally, the deception was discovered by the living
child's mother, and the case was brought to Solomon for arbitration:

Then the king said, The one says, This is my son that lives, and your son is dead: and the
other says, No; but your son is dead, and my son is the living. And the king said, Bring me a
sword. And they brought a sword before the king. And the king said, Divide the living child
in two, and give half to the one, and half to the other. Then the woman whose the living child
was spoke to the king, for her heart yearned over her son, and she said, Oh, my sovereign,
give her the living child, and in no wise kill it. But the other said, It shall be neither mine nor
yours; divide it. Then the king answered and said, Give her the living child, and in no wise
kill it: she is the mother of it. And all Israel heard of the judgment which the king had judged;
and they feared the king: for they saw that the wisdom of Elohim was in him, to do justice (1
Kings 3:23-28, BB).
Now let's consider sensibly what happened here. A woman grieving over the death of her
child switched the baby with another woman's. Obviously, this grieving mother wanted a
child to replace the one she had lost. When the case was presented to Solomon, he cleverly
devised a way to determine which mother was being truthful about the incident. Upon hearing
his decision, the true mother immediately offered to give the baby to the other woman rather
than to see him dead. At this point, the woman who had stolen the child had exactly what she
wanted, a baby to replace the one she had lost. Instead of keeping it that way, however, she
said in response to the real mother's offer to relinquish her maternal claim, "It shall be neither
mine nor yours; divide it." So a woman who had the chance to get a living child to replace the
one she had lost-- exactly what she had come to Solomon for-- was turning it down. She now
wanted to see the baby hacked in half! Does this sound like a probable reaction any woman
would have made in a situation like this?

And what would Solomon have done if the second mother had also relinquished claim to the
baby? To have done so would not have been outrageously out of character. After all, these
two women lived together; some degree of friendship and love for the baby surely existed
between them. Of course, we have to realize that God's hand was at work in all of this. He
was orchestrating the scenario to make a point about the magnitude of Solomon's problem
solving skills, so it had to be as it was or we wouldn't have had a story, would we?

Let's just admit for the sake of argument that Solomon's judgment in this case was clever. Did
the cleverness of it warrant the awe-stricken public reaction attributed to it? "And all Israel
heard of the judgment which the king had judged; and they feared the king: for they saw that
the wisdom of Elohim was in him, to do justice." I guess we are supposed to believe that an

Volume 1990 - 2002 Issue


Page 107 of 2049
Skeptical Review Edited by Farrell Till
ordinary man, acting without divine guidance, could not have devised this plan. All the
judgment indicates, however, is that Solomon understood something about maternal instincts.
A man has to have divine help to know that?

But there's still more to consider about Solomon that should make us all grateful that God
didn't choose one of us to be the recipient of the greatest wisdom ever bestowed upon a
human being. Can any discussion of Solomon, for example, be complete without mentioning
his 700 wives and 300 concubines! My, my, even for Solomon this is excess gone to seed. But
it involves much more than mere excess; it involves greed, egotism, inconsiderateness,
injustice, and just about every other character trait that we find despicable in human beings.

Solomon's wisdom is usually associated with his extraordinary ability to dispense justice, but
what kind of justice can possibly be found in this situation? Here was a man who had surely
depleted the regional supply of princesses by taking 700 of them to be his wives (1 Kings
11:3)--wives whom he could not have "gotten around to" if he had done nothing all day, every
day, except get around to them. Add to these the 300 concubines that for some reason he also
felt a need for, and you have a case of 1,000 women trapped in a situation of hopeless misery.
Women are human beings. Bibliolaters may not want to believe this, but women are human
beings. They have emotional needs that must be met if they are to enjoy normal lives, and
Solomon couldn't have possibly provided for the emotional needs of a thousand women.
Could even the wisest man who ever lived have remembered the names of all of them? How
did he keep track of them? Did he have a record-keeper to tell him, "It's so-and-so's turn
tonight"?

Basically, Solomon put these thousand women into a situation where it was virtually
impossible for them to experience normal lifestyles--socially, emotionally, or maritally. No
doubt many of them were young women with normal physical needs, and you can bet that
Solomon, while demanding their complete fidelity, wasn't capable of accommodating those
needs. The world's wisest man! How about the world's most egotistical, inconsiderate jerk?

Another of Solomon's great claims to fame has to do with his outstanding leadership and
abilities as ruler of the united kingdom of Israel. The Bible, in its usual contradictory fashion,
would have us believe that everything was rosy throughout the land. The people were just so
happy "eating and drinking and making merry" (1 Kings 4:20). Solomon made "silver and
gold to be in Jerusalem as stones" (2 Chron. 1:15). Why, life was just a bowl of cherries under
his reign. Or was it?

Rehoboam succeeded his father as ruler of the kingdom, but its unified state wasn't long for
this world. Jeroboam and all Israel came before the king to complain about the misery that
had been heaped upon them while Solomon was king and to petition for relief as Rehoboam
began his reign:

Your father made our yoke grievous: now therefore make the grievous service of your father,
and his heavy yoke which he put upon us, lighter, and we will serve you (1 Kings 12:4; 2
Chron. 10:4, BB).
These men wanted to serve Rehoboam, but he, lacking the wisdom of his father, who had
apparently lacked the wisdom to include all Israel in his grandiose lifestyle, failed to follow

Volume 1990 - 2002 Issue


Page 108 of 2049
Skeptical Review Edited by Farrell Till
the counsel of the old men, who had advised Rehoboam to grant the petition. He chose instead
to implement the advice given by "the young men that had grown up with him." Their advice
was to show the Israelites who was boss, i.e., if they thought it was bad under Solomon, just
let them see what Rehoboam had in store for them. So when the delegation returned for
Rehoboam's decision, he said to them, "My father made your yoke heavy, but I will add to it:
my father punished you with whips, but I will punish you with scorpions" (2 Chron. 10:14,
BB).

Poor Rehoboam, it wasn't really his fault, "for it was brought about by Elohim, that Yahweh
might establish his word" (v:15). One could almost pity Rehoboam and many of the other
pathetic creatures in the Bible who were simply pawns used by God to carry out his
predetermined plans. They seemed not to have any free will of their own. Suffice it to say,
however, that there is every indication here that the wise Solomon was neither just nor
competent as a ruler. His years of sovereignty had actually laid the founda- tion for a division
of the kingdom David had worked so hard to unify. The wisest man who ever lived couldn't
rule competently enough to maintain the good will of his subjects!

Last, but certainly not least, is the biblical denunciation of Solomon's allegiance to God. In his
old age, Solomon turned to idolatry as a result of the influence of his wives (1 Kings 11:4).
There are several noteworthy comments to make about this event. Isn't it strange to believe
that the wisest man on earth could not turn any of his wives from idolatry to worship of the
one true god, Yahweh Elohim of Israel? Isn't it strange that a man who, on at least two
occasions (1 Kings 3:5-14; 9:2-9), was personally visited by Yahweh would turn to idolatry?
Even without the visits, the wisest man on earth wouldn't know not to practice idolatry? What
on earth is so wise about worshipping a stick of wood or a hunk of stone that one has graven
himself? Isn't it strange that a man who was promised and received great wealth would turn
from the source of that wealth? Isn't it strange that the wisest man ever born could not manage
to satisfy God? And if the wisest man ever born couldn't please God, what chance do we
people of ordinary intelligence have to please him?

But maybe we don't have anything to worry about. Perhaps it takes ignorance to maintain a
satisfying relationship with this strange god of the Bible.

(Sandra Till's address is P. O. Box 617, Canton, IL 61520.)

More Problems for Bibliolaters


Bibliolaters want us to believe that the Bible is a work so harmoniously perfect that only
divine inspiration can explain its existence. They never tire of preaching this theme from their
pulpits. As we have repeatedly shown, however, this is a false claim. Try as they may,
bibliolaters cannot harmonize the Bible without resorting to scenarios so preposterously far-
fetched that only the very credulous can believe them.

Volume 1990 - 2002 Issue


Page 109 of 2049
Skeptical Review Edited by Farrell Till
Let's consider, for example, a statement in 1 Kings 15:5, where it was brazenly asserted that
"David did that which was right in the eyes of Yahweh, and returned [sic] not aside from
anything that he commanded him all the days of his life, except only in the matter of Uriah
the Hittite." We know, of course, what "the matter of Uriah the Hittite" was, but to say that
this was the only offense against God that David committed all the days of his life is
flagrantly contradictory to other things that the Bible says about David.

Both 2 Samuel 24 and 1 Chronicles 21 clearly depict David as having sinned for taking a
census of the Israelites. David himself said on this occasion, "I have sinned greatly in that
which I have done" (2 Sam. 24:10; 1 Chron. 21:8). But conducting this census had nothing at
all to do with "the matter of Uriah the Hittite."

While fleeing from Saul, David needed food and weapons. To procure them, he lied to
Ahimelech the priest and said that he was on a secret mission for king Saul (1 Sam. 21:1-8).
Later, when he had gathered around him a band of 600 men and found refuge in Philistia, he
became a guerrilla marauder who raided Philistine villages, killed all of the people in the
villages so that there would be no witnesses to report his activities, and then returned home
and lied to Achish the king about where he had been (1 Sam. 27:8-12). Do these sound like
the activities of a man who never strayed from Yahweh's commandments "except in the
matter of Uriah the Hittite"? About the only thing the Bible says about David that we can
agree with is that he was a "man after Yahweh's own heart" (1 Sam. 13:14).

Genesis 46:8-27 listed "all the souls of the house of Jacob" that went into Egypt. There was a
total of "threescore and ten" souls in the group (v:27), even though the martyr Stephen said
that there were "threescore and fifteen" (Acts 7:14). At any rate, the listing was done
according to the mothers whom the children and grandchildren had descended from. Leah's
family was listed first, and after everyone had been named, it was said, "These are the sons of
Leah, whom she bare to Jacob in Paddanaram, with his daughter Dinah: all the souls of his
sons and his daughters were thirty and three" (v:15). In this, however, we have a problem,
because if you will count the names of everyone listed in Leah's family, you will see that there
were only thirty-two.

To argue that Leah would have made the thirty-third person will not explain the problem,
because verse 26 (in sexist language so typical of the Bible) plainly excluded wives from the
count. Besides, there is a clear indication in Genesis 49:31 that Leah had died before the
descent into Egypt. To say that Jacob should be included in Leah's family is to ignore the
clarity of the statement underlined above. "All the souls of his (Jacob's) sons and his
daughters were thirty and three." Besides, if we include Jacob here, why shouldn't we have to
include him in the count of Zilpah's or Rachel's or Bilhah's children, where after the counts of
sixteen, fourteen, and seven respectively were given, it was said, "These are the sons of
Zilpah (Rachel or Bilhah) who were born to Jacob: all the souls were sixteen (fourteen or
seven) souls" (vv: 18, 22, 25)?

If the counts given in these breakdowns are added together (33 + 16 + 14 + 7), a total of 70
results (the threescore and ten of verse 27), but if Jacob is then added, we have a total of 71.
For that reason, some inerrantists will argue that Jacob should be included in Leah's family,
but the language of that verse will not allow for his inclusion any more than it would permit

Volume 1990 - 2002 Issue


Page 110 of 2049
Skeptical Review Edited by Farrell Till
us to add him to any of the other three groups. Clearly, then, there is a major problem in the
counting of those who went with Jacob into Egypt.

At the very least, bibliolaters will have to admit that there is a problem of ambiguity in this
passage. And why should that be? As we have repeatedly said, don't we have the right to
expect omnisciently inspired literature to be unambiguously written?

TILL-McDONALD DEBATE
Farrell Till and Jerry McDonald will conduct an oral debate at Sullivan, Missouri, on July 29,
30, and August 1, 2, 1991, at the Community Center in Sullivan. McDonald, a frequent
rebutter in TSR, is a Church-of-Christ preacher and the editor of Challenge, a quarterly
journal that devotes much of its space to exchanges on the inerrancy issue.

FAILED PROPHECIES
A study of imaginary and failed Bible prophecies is now available. Farrell Till examines the
popular prohecies (Is. 7:14; Ps. 16:10; Micah 5:2; Jer. 31:15; etc.) so often referred to as
"proof" of inspiration and shows them to be failures or misrepresentations of original
intentions. Double-column layout, 38 pages, $2.50 ppd.

OTHER DEBATES & BACK ISSUES


The Laws-Till Debate ($2) that Till's opponent withdrew from and The Jackson-Till Debate
($2.25) are still available postpaid at the cost indicated. All propositions concerned Bible
inerrancy.

All four 1990 issues of The Skeptical Review are also available at $1 per copy. Send orders to
the address on page two.

Volume 1990 - 2002 Issue


Page 111 of 2049
Skeptical Review Edited by Farrell Till

The Skeptical Review


Volume Two, Number Four
October/November/December 1991
Farrell Till, editor

• Search the Scriptures


Till reflects on the second year of publication of The Skeptical Review.

• If It Walks Like a Duck...


Till writes, "An especially embarrassing myth for bibliolaters is the quaint little story
about `the sons of God' producing a race of terrestrial giants by marrying the daughters
of men."

• Much Ado About Nothing...


Steve Gunter offers a rebuttal to the above article, and claims that "Till has simply
misread the passage."

• Inerrancy Publications
Several materials on Biblical inerrancy from Skepticism, Inc.

• Why Didn't They Know?


According to Till, "just about everyone who had been associated with Jesus knew that
he was supposed to be resurrected except the apostles."

• The Till-McDonald Debate Till's account of the Till-McDonald debate on Bible


morality.

Search the Scriptures


Volume 1990 - 2002 Issue
Page 112 of 2049
Skeptical Review Edited by Farrell Till
What a difference a year makes! When we began publication last year, many of our free trial-
subscriptions were greeted with angry rejections. Copies were returned to us marked
REFUSED, and letters were received calling The Skeptical Review "junk" or "trash." Even
phone calls came from people demanding that their names be removed from our mailing lists.
Some compliments were paid, but they were few and far between, and only a fraction of those
who had received free subscriptions paid to continue them.

This year brought a dramatic change. As we prepare this final article for the last issue of
1991, we still receive negative letters, but the positive ones far outnumber the negative. On a
typical day, the mail will bring letters from new readers ordering back issues and from old
readers suggesting names to add to our mailing list. Subscription requests will come from
people who heard about TSR from this relative or that friend. Sometimes these
recommendations have crossed the country from one side to the other. Just recently a
professor of philosophy and religion at a university in Mississippi wrote and said, "I do not
know how my name got on your mailing list. But today I received a copy of The Skeptical
Review, and I think it is superb. Please keep my name on your mailing list."

Sometimes we even get phone calls from people looking for help as they try to find their way
out of the mess that religious fundamentalism has made of their lives. The last such call came
from a Baptist who, after embarking on a plan to study the Bible and really come to know
what was in "God's word," become disillusioned at the inconsistencies he had found in it. He
called The Freedom From Religion Foundation, and Dan Barker suggested that he contact us.
He is now on our mailing list.

We would like to share with you more of the communications like these that we are now
receiving, but to do so we would have to sacrifice space that we need to discuss the inerrancy
issue. Your letters indicate that you like what you are seeing in TSR, so we will continue to
try to bring you the same kind of articles. We have never solicited financial help, but some
have included contributions with their subscription payments and orders for other materials
that we publish. We appreciate the help and see it as a definite indication that we are
attracting readers who consider The Skeptical Review to be a worthwhile enterprise.

Still there are those who object to what we are doing, especially former friends and associates
who knew us when we were deeply entrenched in the absurdities of Bible fundamentalism.
They continue to write to express their dismay at what they are reading and their personal
disappointment in us. They rarely attempt to refute what we say in our articles; they
concentrate on reminding us of what we once were in an apparent effort to arouse feelings of
guilt. Our position, however, is that one should never feel shame or guilt over taking a stand
for truth, and we believe that we have the truth on our side.

The Jews at Beroea were once commended because they "searched the scriptures daily" to see
if the things Paul was preaching to them "were so" (Acts 17:11). We present the same
challenge to readers who think that we have drifted off into a terrible apostasy. Check to see if
we are right. We don't just make wild assertions; we document every major point we make
with specific scripture references. So when we say that scripture x contradicts scripture y, you
have the opportunity to verify what we are saying. Search the scriptures to see if what we are
saying is so. That's all we are asking for--a fair hearing.

Volume 1990 - 2002 Issue


Page 113 of 2049
Skeptical Review Edited by Farrell Till

If It Walks Like a Duck....


Farrell Till
Like all literature contemporary to the times, the Bible contains mythology. Fundamentalists
vehemently deny this, but it is an inescapable conclusion that all objective Bible students
eventually reach. When one's predisposition and determination to see the Bible as the inerrant
"word of God" are laid aside, the mythological base of many of the Bible stories becomes
readily apparent. An especially embarrassing myth for bibliolaters is the quaint little story
about "the sons of God" producing a race of terrestrial giants by marrying the daughters of
men:
When people began to multiply on the face of the ground, and daughters were born to them,
the sons of God saw that they were fair; and they took wives for themselves of all that they
chose. Then Yahweh said, "My spirit shall not abide in mortals forever, for they are flesh;
their days shall be one hundred twenty years." The Nephilim (giants) were on the earth in
those days--and also afterwards--when the sons of God went in to the daughters of humans,
who bore children to them. These were the heroes that were of old, warriors of renown (Gen.
6:1-4, NRSV with Yahweh substituted for the LORD).
Most versions of the Bible refer to the beings in this passage who took wives of the daughters
of men as "the sons of God," but the expression (beni ha-elohim) in Hebrew literally meant
"sons of the gods" and is so translated in The Revised English Bible. So a definite hint of
mythology is seen in the very language that was used to tell this fanciful tale of angels
marrying earthly women, because any modern reader encountering a story that referred to
gods and the sons of gods would immediately know that it was fantasy fiction or mythology,
especially if it involved gods consorting with earthly women.

Bibliolaters will quickly protest that the Hebrews used the plural word elohim when referring
to their god Yahweh. They call it "the plurality of dignity," a way of expressing the majesty
and greatness of God. Some even think they see an early recognition of the triune godhead in
the plural term elohim. These matters were discussed in my exchange of articles with Mac
Deaver in the summer issue of TSR, so I won't get involved in rehashing them here. Readers
who keep their back issues, however, might want to read the exchange again to review
biblical passages that clearly show the early Hebrews were polytheistic. They believed that
the gods of the nations around them were entities just as real as their own god Yahweh but
that Yahweh was vastly superior to the others, a God of gods, a sort of supergod whose
powers exceeded all the others.

Suffice it to say at this point that Genesis 6:1-4 literally refers to beni ha-elohim (the sons of
the gods) rather than "the sons of God" as it has been deceptively rendered in most English
translations. This fact alone gives us our first clue that this passage is an ancient one that
managed to survive the piecemeal editing of the monotheistic era in which the J, E, P, R, and
D documents were patched together to give the Tanach (Hebrew scriptures) their final

Volume 1990 - 2002 Issue


Page 114 of 2049
Skeptical Review Edited by Farrell Till
structure. It reflects the thinking of a time when people saw the world as a place ruled by gods
(many) rather than Yahweh, the one and only God.

Anyone who has studied the mythology of prescientific times knows that giants figured
prominently in the literature of that era. The Greeks had their Hercules, who was so big and
powerful that he supported the world on his shoulders. Even the relatively modern fairy tale
of Jack and the Beanstalk involved an encounter with a giant. The prescientific Hebrews were
no different from the other nations of superstitious times. They had their mythology, and
giants were part of their mythology.

As indicated in the Genesis 6 passage, the Hebrews called their giants nephilim, which both
Strong and Young define as "fellers," "fallen ones," or "giants." (Why they were perceived as
"fellers" or "fallen ones" will become very clear as I proceed.) After the men whom Moses
sent ahead to spy out Canaan returned to camp, they reported having seen "the Nephilim, the
sons of Anak, who come of the Nephilim: and we were in our own sight as grasshoppers"
(Num. 13:33). Upon hearing this, the people were terrified and wept all night, fearing that
Moses had led them out of Egypt only to face certain death at the hands of the Nephilim
(Num. 14:1-3). To their superstitious minds, these Nephilim, sons of Anak, known also as
Anakim, were "a people great and tall" of whom they had heard said, "Who can stand before
the sons of Anak?"

The Hebrews designated giants by other names too. Deuteronomy 2:10 spoke of "a people
great, and many, and tall" known as Emim, who had "aforetime" dwelt in the land of Moab.
The passage called them Rephaim and compared them to the Anakim. Joshua 11:21-23
credited Joshua with the total destruction of the Anakim from the hill-country of Israel so that
none were left in the land except in Gaza, Gath, and Ashdod, which were all Philistine
strongholds that the Israelites were never able to control. David came to prominence in Israel
as a result of his famous battle with Goliath of Gath, a giant whose height was "six cubits and
a span" or about ten feet (1 Sam. 17:4), and battles with giants (Heb. raphaim) are mentioned
in 2 Samuel 21:16-22 and 1 Chronicles 20:4-8. Other references to giants (rephaim or
anakim) were made in Deuteronomy 2:20-21; 3:11,13 and Joshua 12:4; 13:12; 15:8; 17:15;
18:16. Many post-KJV translations have attempted to veil these superstitious allusions to
giants in "God's inspired word" by transliterating the Hebrew terms Nephilim, Anakim, and
Rephaim rather than translating them to convey the idea of giantism. To unsuspecting English
readers, the terms simply become tribal or nationalistic designations like Syrian or Amorite
rather than mythological allusions to giants.

Wading through all of these references may be tedious, but they do obviously establish that
the Hebrews, like the superstitious nations around them, believed in the existence of giants.
Genesis 6:1-4 appears to be a mythological attempt to explain the existence of giants, just as
Genesis 3:14-15 was an obvious mythological attempt to explain the natural enmity that exists
between man and snakes. To the prescientific Hebrew mind, giants existed because "the sons
of the gods," had seen the fairness of "the daughters of men" and had taken them for wives. In
other words, giants had resulted from the sexual union of angels and human women.

Volume 1990 - 2002 Issue


Page 115 of 2049
Skeptical Review Edited by Farrell Till
In my debate with Bill Jackson, I made brief mention of this passage just to make a point
about another matter, and he built it into a straw man that he could knock around to avoid
dealing with the issue being debated:

His (Till's) view is that Moses believed angels intermarried with earthly women. And he is the
one, mind you, who speaks of the need for evidence! Wonder where he found angels in
Genesis 6:1-4? This from a character who doesn't even know if he believes in God, having
said the greater evidence lies on the side of the atheist (Jackson-Till Debate, p. 19)!
From then on, Mr. Jackson made frequent derogatory remarks about my interpretation of this
passage, as if it were perfectly ridiculous for anyone to see the intermarriage of angels and
women in it. Before his death in April, Mr. Jackson had agreed to write a reply to this article
for simultaneous publication in TSR. I sincerely regret not only his untimely death but the
opportunity he has missed to see just where I found angels in this passage.

They weren't at all hard to find.

The key to understanding the passage is the proper interpretation of beni ha-elohim (the sons
of the gods). Let bibliolaters believe that this expression meant only "the sons of God" if they
want to. That still will not help their case, because usage of the expression, although
infrequent in the Old Testament, clearly establishes it as a reference to celestial beings or
spiritual entities. Besides the Genesis 6 passage, this exact expression was used only three
other times in the OT, all three in the book of Job. In the beginning of this book, reference
was twice made to the day "when the sons of God (beni ha-elohim) came to present
themselves before Yahweh" (1:6; 2:1). On both occasions, Satan "came also among them (the
sons of God) to present himself."

Now where did the writer of Job expect us to believe that these scenes had taken place, on
earth or in heaven? Surely, he didn't intend for us to think that these were scenes that had
happened on earth, for on both occasions, after discussing Job's character with Yahweh,
"Satan went forth from the presence of Yahweh" (1:12; 2:7). To the Hebrew mind, "the
presence of Yahweh" would have been a location in heaven where Yahweh sat on his throne
(1 Kings 22:19; Ps. 11:4; 103:19; Is. 66:1), so whoever these "sons of God" were, they were
creatures who came to present themselves to Yahweh in heaven.

Confirmation of this conclusion can be found in the third reference in Job to the "sons of
God" (beni ha-elohim). Here Yahweh himself was speaking and applied the term to entities
who already existed when he began creating the world:

Where were you when I laid the foundations of the earth? Declare, if you have understanding.
Who determined the measure of it, if you know? Or who stretched the line upon it? Upon
what were the foundations of it fastened? Or who laid the corner-stone of it, when the
morning stars sang together, and all the sons of Elohim (God) shouted for joy (38:4-7, Bethel
Bible)?
Obviously these "sons of God" who shouted for joy on this occasion were not humans,
because man had not yet been created. So whoever these "sons of God" (beni ha-elohim)
were, they were beings who were already in existence when God began his work of creation.

Volume 1990 - 2002 Issue


Page 116 of 2049
Skeptical Review Edited by Farrell Till
The conclusion is unavoidable: the writer of Job intended for us to understand that these "sons
of God" were spiritual or celestial beings.

So if the "sons of God saw the daughters of men that they were fair" and "took them wives of
all that they chose" (Gen. 6:2), whoever wrote this must have believed that he was describing
something more than just ordinary marriages between human men and women, and especially
so since "when the sons of God came in unto the daughters of men" (v:4), the sexual unions
resulted in children who "were the mighty men that were of old"--the Nephilim. To say that
the writer meant nothing more here than when he wrote, "And so-and-so knew his wife and
she bore a son" is to stretch credulity to the limits.

A striking but enlightening parallel to Genesis 6:1-4 can be found in the apocryphal book of 1
Enoch:

In those days, when the children of man had multiplied, it happened that there were born unto
them handsome and beautiful daughters. And the angels, the children of heaven, saw them
and desired them; and they said to one another, "Come, let us choose wives for ourselves from
among the daughters of man and beget us children" (6:1-2, Isaac Translation).
The passage goes on to explain that 200 angels took an oath and bound one another to a curse
to carry out the proposal. Under the leadership of Semyaz, they then descended upon Mount
Hermon to execute their plan:
And they took wives unto themselves, and everyone (respec- tively) chose one woman for
himself, and they began to go unto them. And they taught them magical medicine,
incantations, the cutting of roots, and taught them (about) plants. And the women became
pregnant and gave birth to great giants whose heights were three hundred cubits. These
(giants) consumed the produce of all the people until the people detested feeding them. So the
giants turned against (the people) in order to eat them (7:1-4).
Giants three hundred cubits (450 feet) tall--this is obviously mythology, bibliolaters will no
doubt say. And that is the point exactly; it is obviously mythology. Yet the similarity of
Genesis 6:1 and 1 Enoch 6:1 is so striking that it cannot be lightly dismissed. If both passages
were in secular books, no reasonable critic would deny that the two had been derived from the
same source, either written or oral. Bibliolaters, however, are unwilling to apply to the Bible
the same common sense critical reasoning that they routinely apply to secular works. The
writer of 1 Enoch said that "the angels, the children of heaven, saw them (the daughters of
men) and desired them" and later took them as wives, and bibliolaters agree that this is
mythology. The writer of Genesis said that "the sons of God saw the daughters of men that
they were fair... and took them wives of all that they chose," and bibliolaters say that this is
the inspired truth of God! By what reasoning process does one arrive at conclusions so
patently incongruous?

Inerrantists may react with "What do I care what the book of First Enoch says?" all that they
wish, but to do so is to flagrantly ignore the stamp of approval that New Testament writers put
on this "apocryphal" book. In his introduction to the book, the translator (Isaac) commented
on the esteem that both apocryphal writers and early Christians had for it:

It was used by the authors of Jubilees, the Testaments of the Twelve Patriarchs, the
Assumption of Moses, 2 Baruch, and 4 Ezra. Some New Testament authors seem to have

Volume 1990 - 2002 Issue


Page 117 of 2049
Skeptical Review Edited by Farrell Till
been acquainted with the work, and were influenced by it, including Jude, who quotes it
explicitly (1:14f.). At any rate, it is clear that Enoch- ic concepts are found in various New
Testament books, including the Gospels and Revelation.

... Many church fathers, including Justin Martyr, Irenaeus, Origen, and Clement of
Alexandria, either knew 1 Enoch or were inspired by it. Among those who were familiar with
1 Enoch, Tertullian had an exceptionally high regard for it.... (F)ew other apocryphal books so
indelibly marked the religious history and thought of the time of Jesus.

... (E)ven though Charles (R. H.) may have exaggerated when he claimed that "nearly all" the
writers of the New Testament were familiar with 1 Enoch, there is no doubt that the New
Testament world was influenced by its language and thought. It influenced Matthew, Luke,
John, Acts, Romans, 1 and 2 Corinthians, Ephesians, Colossians, 1 and 2 Thessalonians, 1
Timothy, Hebrews, 1 John, Jude (which quotes it directly) and Revelation (with numerous
points of contact). There is little doubt that 1 Enoch was influential in molding New
Testament doctrines concerning the nature of the Messiah, the Son of Man, the messianic
kingdom, demonology, the future, resurrection, final judgment, the whole eschatological
theater, and symbolism. No wonder, therefore, that the book was highly regarded by many of
the earliest apostolic and Church Fathers (The Old Testament Pseudepigrapha, ed. James H.
Charlesworth, "1 Enoch: a New Translation and Introduction," pp. 8,10).

With a critical reputation like this, whatever is in the book of 1 Enoch cannot be waved aside
with a disdainful, "What do I care what the book of First Enoch says?" Even if one chooses to
think that the preceding quotation greatly exaggerates the influence of 1 Enoch on the
formation of the New Testament, he cannot deny what was twice noted in the quotation, i.e.,
the writer of Jude explicitly quoted the book, and in a way that attributed prophetic powers to
its author:
And to these (ungodly men troubling the church) also Enoch, the seventh from Adam,
prophesied, saying the Lord came with ten thousands of his holy ones to execute judgment
upon all, and to convict all the ungodly of their works of ungodliness which they have
ungodly wrought, and of all the hard things which ungodly sinners have spoken against him
(vv:14-15).
This is obviously a direct quotation from 1 Enoch 1:9, a fact that, to say the least, should give
believers in the divine inspiration of Jude ample reason to respect the book. After all, if the
Holy Spirit deemed the book important enough to direct Jude to quote it, they surely couldn't
ask for a better recommendation than that.

Furthermore, the "inspired" Jude identified the author of this quotation as "Enoch, the seventh
from Adam," so if Jude thought that Enoch had made this statement, he must have been
endorsing the Enochian authorship of the book. And if the Holy Spirit (as the doctrine of
verbal inspiration teaches) was directing Jude in what he wrote, then the Holy Spirit must
have actually known that Enoch had written the book. The inerrantists, then, have nowhere to
go except to conclude that 1 Enoch was actually written by Enoch, the seventh-generation
descendant of Adam who "walked with God" and "was not, for God took him" (Gen. 5:24).
So if bibliolaters are looking for something to give credibility to 1 Enoch, they surely have it
in these facts. They have to believe that the book was written by a man who was so righteous

Volume 1990 - 2002 Issue


Page 118 of 2049
Skeptical Review Edited by Farrell Till
that he was translated directly to heaven without seeing death (Heb. 11:5). Yet with all that to
commend it, the book is not even in the holy canon. But that's another article for another time.

After telling us that angels, the children of heaven, married human women and produced a
race of giants 450 feet tall, this book (which Jude said was written by Enoch, the seventh from
Adam) tells of the earthly corruption that was caused by these celestial half-breeds. Their
angelic fathers taught them "eternal secrets which are performed in heaven" (9:6), and as a
result the earth became corrupt:

(Moreover) Semyaz, to whom you have given power to rule over his companions, co-
operating, they went in unto the daugh- ters of the people on earth, and they lay together with
them--with those women--and defiled themselves, and revealed to them every (kind of) sin.
As for the women, they gave birth to giants to the degree that the whole earth was filled with
blood and oppression (9:7-9).
This corruption became Enoch's explanation for the flood. "The Most High, the great and
Holy One," sent an angel to the son of Lamech (Noah) to warn him of a great deluge that
would destroy all upon the earth except his seed (10:1-3). In this, we see another obvious
parallel between this account and the Genesis record, for immediately after telling of the
"sons of God" marrying the daughters of men, the Genesis writer said that "Yahweh saw that
the wickedness of man was great in the earth" and determined to destroy all life on earth,
except for Noah who had "found favor in the eyes of Yahweh" (6:5-8). Both writers saw the
flood as a consequence of wickedness resulting from the "sons of God" (children of heaven)
marrying "daughters of men," so both obviously relied on the same source or tradition, the
only essential difference being that Enoch gave many more details about the corruption that
had been caused by the intermarriages.

Prior to telling the story of the flood and Noah's salvation in the ark, Enoch devoted much
more space to a description of man's wickedness than did the Genesis writer. Man's corruption
had been caused by the intermarriage of angels and earthly women, so God instructed the
archangels Raphael, Gabriel, and Michael to punish the fallen angels for what they had done:

And secondly the Lord said to Raphael, "Bind Azaz'el (a leader of the fallen angels) hand and
foot (and) throw him into the darkness." And he made a hole in the desert which was in
Duda'el and cast him there; he threw on top of him rugged and sharp rocks. And he covered
his face in order that he may not see light, and in order that he may be sent into the fire on the
great day of judgment.... And to Gabriel the Lord said, "Proceed against the bastards and the
reprobates and against the children of adultery; and destroy the children of adultery and expel
the children of the Watchers (see Dan. 4:13,17,23, FT) from among the people...." And to
Michael God said, "Make known to Semyaza and the others who are with him, who
fornicated with the women, that they will die together with them in all their defilement. And
when they and all their children have battled with each other, and when they have seen the
destruction of their beloved ones, bind them for seventy generations underneath the rocks of
the ground until the day of their judgment and of their consummation, until the eternal
judgment is concluded. In those days they will lead them into the bottom of the fire--and in
torment--in the prison (where) they will be locked up forever. And at the time when they will
burn and die, those who collaborated with them will be bound together with them from
henceforth unto the end of (all) generations" (10:4-6, 9, 11-14).

Volume 1990 - 2002 Issue


Page 119 of 2049
Skeptical Review Edited by Farrell Till
Several times after this, Enoch mentioned the condemnation that God had pronounced upon
the Watchers (angels) for "begetting giant sons" upon wives of "the children of the earth"
(15:4). He told how that they would be "put in bonds" and "imprisoned inside the earth" to be
"detained here forever" until "the great day of judgment in which they shall be judged until
they are finished" (13:2; 14:5; 21:10; 19:2).

God's pronouncement of judgment upon the fallen ones went on and on, and if it all sounds
vaguely familiar, that is because it is familiar to all students of the New Testament. I have
already noted that Jude (vv:l4-15) quoted directly from 1 Enoch 1:9. Prior to this, he referred
to the archangel Michael's contention with the devil over the body of Moses (v:9), an allusion
to a scene in an apocryphal book known as the Assumption of Moses. Obvi- ously, then, Jude
was familiar with apocryphal works, and it was in this context that he warned his readers of
the ever present danger of apostasy by referring to the section of 1 Enoch summarized above:

And the angels who did not keep their own position, but left their proper dwelling, he has kept
in eternal chains in deepest darkness for the judgment of the great day. Likewise, Sodom and
Gomorrah and the surrounding cities, which in the same manner as they, indulged in sexual
immorality and pursued unnatural lust, serve as an example by undergoing punishment by
eternal fire (vv:6-7, NRSV).
That Jude was here referring to angels who had engaged in sexual immorality after leaving
"their proper dwelling" is apparent from his comparison of their sin with the wickedness of
the people of Sodom and Gomorrah. What could he have possibly had in mind regarding
these fallen angels except the situation that Enoch wrote about in the section of his book
summarized above? To deny this is to grasp in desperation for any straw in sight to preserve
an untenable belief in Bible inerrancy.

The writer of 2 Peter made a similar allusion to the damnation of fallen angels:

For if God did not spare the angels when they sinned, but cast them into hell and committed
them to chains of deepest darkness to be kept until the judgment... then the Lord knows how
to... keep the unrighteous under punishment until the day of judgment--especially those who
indulge their flesh in depraved lust (2:4-10, NRSV).
So Enoch told of fallen angels whom God imprisoned in darkness inside the earth to await
judgment for having taken wives from the daughters of men. Both Jude and Peter wrote about
fallen angels who have been imprisoned in chains in deep darkness to await judgment for
having engaged in sexual immorality. The similarity is too striking to deny. The sin of the
angels whom Enoch wrote about had produced a race of giants. Can there be any doubt, then,
that the Genesis writer believed exactly as Enoch did, that angels (sons of God) had come
down from heaven, married human women, and produced giants on the earth?

We have all heard an adage that tells a reliable way to settle controversy. "If it walks like a
duck and quacks like a duck, you can be reasonably sure it is a duck." Genesis 6:1-4 looks
like mythology, sounds like mythology, and reads like mythology. What else is there to
conclude but that it is mythology? Who would believe otherwise if this were in any book but
the Bible?

Volume 1990 - 2002 Issue


Page 120 of 2049
Skeptical Review Edited by Farrell Till

Much Ado About Nothing


Steve Gunter
Farrell Till alleges Genesis 6:1-4 proves the book of Genesis contains a mixture of
mythology. In fact, Till has simply misread the passage. What is the meaning of Genesis 6:1-
4?

Science confirms the unitary origin of the human species in reports of experiments that
validate Genesis 3:20, wherein Eve is named the mother of all living. All women now living
on the earth derive from but one common mother.

In the May 13, 1985, issue of The New Yorker, Alex Shoumatoff demonstrated the "diamond
shape" of human genealogy. This is true since ancient cultures did not have the distance-
conquering automobiles and conveyances that modern man possesses.

Adam Clarke in his admired commentary concludes that the phrase "sons of God" certainly
cannot mean (as skeptics desire) "sons of gods." The polytheistic presupposition Mr. Till
brings to his reading makes him stumble over the passage and so misunderstand its true
meaning.

What is meant by "sons of God"? These are the descendants of righteous Abel. Who are the
daughters of men? These are the offspring of evil Cain. Such a reading makes much more
sense than the skeptical skullduggery Till's essay engages in.

Genesis 6:4 causes much trouble for Mr. Till, especially when it comes to "nephilim in the
earth." The root word here is naphal and means to fall. The King James translators followed
the Septuagint and thus "giants." We do not have inspired translations, but we do have
inspired texts.

Mr. Till cites Numbers 13:32-33 where the word Nephilim is associated with people of great
stature, but the correct meaning of the word is, among other things, to fall upon. Hence, the
"sons of Anak" were men of violence and perhaps inclined to banditry and such like.

Genesis 6:11 says the earth was filled with violence. Thus is explained the Nephilim that Mr.
Till transforms into giants to suit his polytheistic presuppositions.

Furthermore, the rebellion of the angels (Jude 6) must be separated from the sexual crimes of
Sodom in Jude 7. The reason Mr. Till is a skeptic and not a saint today appears to be primarily
due to a massive misreading of the text and too much study of noninspired works such as the
spurious book of Enoch.

I pray the day will come when Farrell Till returns to the faith and again preaches the truth
from the book of books, the Holy Bible.

Volume 1990 - 2002 Issue


Page 121 of 2049
Skeptical Review Edited by Farrell Till
(Steve Gunter's address is 1202 Royal Drive, Bentonville, AR 72712.)

EDITOR'S NOTE: I was tempted to forego my usual editorial rejoinder and just let the
glaring inadequacy of Mr. Gunter's "rebuttal" speak for itself. Of ten fundamentalist preachers
and writers who were sent advanced copies of "If It Walks Like a Duck..." Mr. Gunter was the
only one who accepted our invitation to write a rebuttal. Since the offer was made and he at
least made the effort, we decided to publish his response.

Mr. Gunter sorely needs to explain his allusion to scientific confirmation of "the unitary
origin of the human species" that he claims "validates Genesis 3:20 wherein Eve is named the
mother of all living." If he meant the findings of biochemists Vincent Sarich and Allan
Wilson, whose studies in mitochondrial DNA have traced the ancestry of all humans back to
one woman, he needs to take another look at the data. Their studies were based on the
assumption of a 2 to 4 percent steady rate of mutation in mitochondrial DNA every million
years. Since mitochondrial enzymes are carried in the ovum but not the sperm, Sarich's and
Wilson's DNA "clock" was able to trace human ancestry back to a single woman who had
lived 200,000 years ago. I wonder if Gunter is willing to accept that date. The same DNA
clock indicates that the hominid line that eventually produced "Eve" had diverged 5 million
years ago from the primates that chimpanzees and gorillas evolved from. Is Gunter willing to
accept that finding? If not, he has no business citing scientific "discoveries" whose major
conclusions he rejects. At any rate, the last thing Mr. Gunter should be doing is appealing to
science to prove the accuracy of the Genesis record. He will find himself in a peck of trouble
if he keeps doing that.

I wonder why he even referred to "the unitary origin of the human species." What relevance
does it have to the issue we are supposed to be discussing? All humans have descended from
one woman; therefore, the Genesis writer could not have believed that angels once
intermarried with human women. Is that what he was trying to say? If so, he will have to
explain his logic to me, because I can't see any possible reasoning principle to base that
conclusion on. Surely, he doesn't think that I believe angels and human women actually did
marry. I believe only that the Genesis writer thought that such marriages did occur. But even
if angels and women had in fact married as the Genesis writer obviously believed, the
mitochondrial DNA studies would still be irrelevant to the issue. Mitochondrial DNA can be
transmitted only by females, and in the mixed marriages of Genesis 6, the angels had assumed
the male role in reproduction. So why did Gunter wag this into the debate?

He spoke of Adam Clarke's "admired commentary," but he didn't tell us anything about who
admires it and what that admiration is supposed to mean. He didn't even tell us Clarke's
grounds for concluding that beni ha-elohim in Genesis 6:2 could not mean "sons of the gods";
he just told us that Clarke had concluded this and apparently expected us to accept it with no
supporting explanation. I suspect that Mr. Gunter is too accustomed to speaking to gullible
pulpit audiences who buy everything he says without demanding proof.

Clarke's commentary is widely used by fundamentalist preachers, so that alone speaks


volumes about what one can expect to find in it when the issue of inerrancy is at stake. One
has to wonder, then, just how naive Mr. Gunter is to think that the mere citing of what a
"commentary" says is sufficient to settle an issue as complex as this one. I can cite Bible

Volume 1990 - 2002 Issue


Page 122 of 2049
Skeptical Review Edited by Farrell Till
commentaries that agree with my position, so what does that prove? The Revised English
Bible was translated by a committee of reputable Hebrew, Greek, and Aramaic scholars, and
they translated beni ha-elohim in Genesis 6 as "sons of the gods." That surely carries more
scholastic weight than what a single unabashedly fundamentalist commentator has said on the
matter.

Gunter accused me of "transform(ing)" the Nephilim into giants to suit my "polytheistic


presuppositions." In so doing, he completely ignored my analysis of the word nephilim, which
showed that its usage in the OT was clearly associated with giantism. If I have "transformed"
the Nephilim into giants, I am in good company, because the translators of several versions
(KJV, GNB, Septuagint, Amplified, Confraternity, Revised Berkeley, Lamsa's, and Living
Bible) rendered the word giants. In past articles, I have reminded readers that the Holy Spirit
(according to the inerrancy doctrine) must have held the Septuagint in high regard, because he
frequently directed New Testament writers to quote it. How then can Gunter discredit it as he
did by faulting the KJV translators for "follow(ing) the Septuagint"? If the Septuagint was
good enough for the Holy Spirit, why shouldn't it have been good enough for the KJV
translators?

As for my "polytheistic presuppositions," are we to assume that Mr. Gunter's hermeneutic


approach to this passage was completely free of presuppositions? I suspect that if he
examined his motives closely enough he would find at least a smidgen of predisposition to the
inerrancy doctrine in his denial of angels and giants in this Genesis passage.

Gunter told us that the sons of God in this passage were merely "the descendants of righteous
Abel" and that the daughters of men were "the offspring of evil Cain." But what evidence did
he offer to prove this theory? None whatsoever! It was simply an arbitrary pronouncement.
Anyone with experience in trying to reason with bibliolaters knows that they are good at
making arbitrary pronouncements to resolve discrepancies in the Bible. The only problem is
that arbitrary pronouncements prove nothing.

One has to wonder, however, about certain implications in this pronouncement. If the "sons of
God" were "descendants of righteous Abel" and "the daughters of men" were "the offspring of
evil Cain," does this mean that righteousness and wickedness are hereditary traits? If one had
descended from Abel, he was righteous; if one had descended from Cain, he was evil. Is this
what Gunter is saying? Unless the Church of Christ has changed radically since I was in it, I
don't think he will be willing to accept the idea of hereditary good and evil. Furthermore,
there is nothing in the Bible text to indicate that Cain lived an evil life after his exile for
killing his brother. Certainly, there is nothing to suggest that his offspring were any more evil
than the offspring of Adam's and Eve's other children. So just what is Gunter's basis for
saying that "such a reading (sons of God = descendants of righteous Abel, and daughters of
men = offspring of evil Cain) makes much more sense than the skeptical skullduggery Till's
essay engages in"? If completely groundless assumption makes more sense than documented
critical analysis, I suppose he is right.

Gunter accused me of "stumbl(ing) over the passage," but if anyone has stumbled over it, he
has. Genesis 6:1 does not say, "And it came to pass that when the offspring of Cain began to
multiply on the face of the ground and daughters were born to them, etc." It says that "when

Volume 1990 - 2002 Issue


Page 123 of 2049
Skeptical Review Edited by Farrell Till
men (people, NRSV) began to multiply on the face of the ground and daughters were born to
them...." Gunter is doing a lot of stumbling to get only the offspring of Cain from the word
men in this verse. The truth is that the passage says nothing at all about either Cain or Abel.
Gunter's "explanation," as I said, is purely arbitrary.

Gunter asserted that "the rebellion of the angels (Jude 6) must be separated from the sexual
crimes of Sodom in Jude 7," but why must they be separated? Jude certainly didn't separate
them. After describing the fate of the rebellious angels in verse 6, Jude said, "Likewise,
Sodom and Gomorrah and the surrounding cities, which, in the same manner as THEY,
indulged in sexual immorality and pursued unnatural lust, etc." (v:7). Does Gunter know what
"in the same manner" means? Can't he see that the antecedent of the pronoun they has to be
the angels in verse 6? Jude was obviously saying that the people of Sodom in indulging in
sexual immorality and pursuing unnatural lust had done in the same manner as THEY, the
rebellious angels just referred to. If not, why not?

That indulgence in sexual immorality was the sin of these fallen angels was made very clear
in 1 Enoch and the other apocryphal works cited in my article. Gunter tried to dismiss the
apocryphal references by a simple assertion that the book of Enoch was "spurious," so he
apparently holds the book in considerably less esteem than did Jude who both alluded to its
judgment of fallen angels and quoted it directly: "And to these also Enoch, the seventh from
Adam, PROPHESIED..." (V:14). This introduced a direct quotation from 1 Enoch 1:9, and
the way that Jude quoted it (1) ascribed the authorship of the book to Enoch, the seventh-
generation descendant of Adam, who was translated directly to heaven, and (2) attributed
prophetic abilities to the author of the book. Mr. Gunter cannot dismiss the impact of this by
just crying, "Spurious!" In the first place, if by "spurious" he means "lacking authenticity,"
then 1 Enoch is no more spurious than Genesis, because no reputable Bible scholar believes
that Genesis was actually written by Moses.

Whether 1 Enoch is "spurious" or not is beside the point. I quoted the book and other
apocryphal works simply to show that a legend about the intermarriage of angels and human
women was widely believed when the book of Genesis was written. That fact and the striking
parallel in the wording of Genesis 6:1 and 1 Enoch 6:1 must be dealt with. Gunter dealt with
neither, and he didn't because he can't. Genesis 6:1-4 was deeply rooted in mythology as were
the creation, the flood, the tower of Babel and most of the other sto- ries in the first eleven
chapters of Genesis. To deny this is to deny the obvious.

Inerrancy Publications
We still have copies of the Laws-Till Debate. After challenging Farrell Till to a written debate
on the inerrancy question, James H. Laws, Jr., a Church-of-Christ preacher in the Spiritual
Sword faction, quit after only three exchanges. The fifty pages that were exchanged prior to
his withdrawal will explain why. Available at publishing cost, $2.25 ppd.

Volume 1990 - 2002 Issue


Page 124 of 2049
Skeptical Review Edited by Farrell Till
The Jackson-Till Debate on Bible inerrancy is also available. Sixty-six pages, $2.25 ppd.

Prophecies: Imaginary and Unfulfilled shows that popular "prophecies" cited to prove Bible
inerrancy are failures or misapplications of OT scriptures. Thirty-eight pages, $2.50 ppd.

All four 1990 issues of TSR are available at $1 per copy.

Why Didn't They Know?


Farrell Till
John's account of the resurrection has Peter and another disciple running to the empty tomb
after hearing from Mary Magdalene that the body of Jesus was gone. The unnamed disciple,
outrunning Peter, arrived at the tomb first and waited:
Then Simon Peter came, following him, and went into the tomb. He saw the linen wrappings
lying there, and the cloth that had been on Jesus' head, not lying with the linen wrappings but
rolled up in a place by itself. Then the other disciple, who reached the tomb first, also went in,
and he saw and believed; for as yet they did not understand the scripture, that he must rise
from the dead. Then the disciples returned to their homes (20:6-10, NRSV).
Luke also indicated that the disciples of Jesus had not expected his resurrection, for Luke said
that after Peter looked inside at the linen cloths, "he went home, wondering at that which had
come to pass" (24:12). Numerous references to the apostles' skepticism of a resurrection
appear elsewhere in the New Testament (Lk. 24:11,38; Jn. 20:24-25; Matt. 28:17).

From one perspective, that the disciples did not yet understand the scripture that Jesus must
rise from the dead, as John alleged, is not at all surprising, for the simple reason that there
were no scriptures that said he would rise from the dead. Luke had Jesus telling his disciples
the night of the resurrection that "it is written that the Christ should suffer, and rise again from
the dead the third day" (24:46). The Apostle Paul also alleged that the scriptures said that
Christ would be raised on the third day (1 Cor. 15:4). That is the claim, but the claim and the
reality are two different things. One could search the OT scriptures until doom's day, and he
would find nothing written about a Messiah who would rise from the dead on the third day.

One will find nothing in the OT scriptures about a risen Messiah, period! Bibliolaters like to
point to Psalm 16, which Luke claimed that both Peter and Paul quoted as proof of Jesus's
resurrection (Acts 2:25-31; 13:35-37), but the context of the whole psalm does not support the
application that the apostles gave to the verses they quoted. In my booklet, Prophecies:
Imaginary and Unfulfilled, I have analyzed in detail this psalm and Peter's and Paul's
application of it, so I won't repeat myself here except to say that anyone who reads the
apostles' quotation in context will see a dubious connection at best between it and the alleged
resurrection of Jesus of Nazareth. Despite the often repeated New Testament claim, there just
are no prophecies of a resurrected Messiah in the OT scriptures.

Volume 1990 - 2002 Issue


Page 125 of 2049
Skeptical Review Edited by Farrell Till
From another perspective, however, if the resurrection really did catch the apostles by
surprise, one has to wonder why. Certainly they had been told enough that it would happen. In
the context of the famous passage where Jesus promised Peter the keys of the kingdom of
heaven, it was clearly said that Jesus told his disciples that he would be killed and then
resurrected:

From that time began Jesus to show unto his disciples that he must go unto Jerusalem, and
suffer many things of the elders and chief priests and scribes and be killed, and the third day
be raised up (Matt. 16:21).
Parallels to this passage are found in Mark 8:31 and Luke 9:22. Jesus even repeated the
statement to his apostles at least twice:
And while they abode in Galilee, Jesus said unto them, The Son of man shall be delivered up
into the hands of men; and they shall kill him, and the third day he shall be raised up. And
they were exceeding(ly) sorry (Matt. 17:22-23).

And as Jesus was going up to Jerusalem, he took the twelve disciples apart, and on the way he
said unto them, Behold, we go up to Jerusalem; and the Son of man shall be delivered unto
the chief priests and scribes; and they shall condemn him to death, and shall deliver him unto
the Gentiles to mock, and to scourge, and to crucify: and the third day he shall be raised up
(Matt. 20:17-19).

Some parallel accounts of these passages (Mk. 9:31; Luke 18:32-34) say that the disciples
didn't understand what Jesus was saying, but this is just another case of textual discrepancies
in the Bible, because Matthew clearly indicated that they did understand him. The first time
they were told, for example, Peter took Jesus aside, rebuked him, and said, "God forbid it
Lord! This must never happen to you" (Matt. 16:22). The second time Jesus told them,
Matthew said that they were "exceedingly sorry," but how could they have been exceedingly
sorry about something they didn't even understand? Besides, considering the general
acceptance of the phenomenon of resurrection in those times (Mk. 6:14-16), what was there to
misunderstand when a man said he would "rise again" after he had been killed?

In view of what Jesus said in the last passage cited above, the postcrucifixion conduct of the
apostles is almost impossible to understand. On the way to Jerusalem, he took them aside, told
them that he would be (1) delivered up to the chief priests and scribes, (2) condemned to
death, (3) delivered to the Gentiles to be mocked, (4) scoured, (5) crucified, and (6) raised on
the third day. After their arrival in Jerusalem, the apostles saw Jesus (1) delivered up to the
chief priests and scribes, (2) condemned to death, (3) delivered to the Gentiles and mocked,
(4) scoured, and (5) crucified, yet somehow, after personally witnessing these five specific
fulfillments of Jesus's statement, they didn't expect him to be resurrected. Why? One would
think that if Jesus had really told them to expect all of these things, after witnessing the
precise fulfillment of the first five of his predictions, they would have surely expected at least
a possibility of the sixth. So rather than the women's having to run to tell the apostles about
the empty tomb they had found, one would think that the apostles would have been on the
scene themselves that third-day morning at least waiting to see if Jesus would come forth.

But they weren't there (according to the story). They had to be sought out and told, and even
then they considered the news the women brought to them to be only "idle talk" (Luke 24:11).

Volume 1990 - 2002 Issue


Page 126 of 2049
Skeptical Review Edited by Farrell Till
The women were telling them exactly what Jesus had said would happen, and they thought
their words were just idle talk! At the tomb, the angels said to the women, "(R)emember how
he spake unto you when he was yet in Galilee, saying that the Son of man must be delivered
up into the hands of sinful men, and be crucified, and the third day rise again" (Luke 24:7).
The next verse says that "they remembered his words." So the women were able to remember
that Jesus had said this, but the apostles whom Jesus had taken aside on the way to Jerusalem
expressly for the purpose of telling them to expect his death and resurrection apparently
couldn't remember that he had said it. They just looked into the tomb and went home, "for as
yet they did not understand the scripture, that he must rise from the dead." Is that what we are
supposed to believe?

If the apostles didn't yet understand that Jesus had been destined to rise from the dead, they
were a pretty exclusive club, because just about everybody else knew what to expect. As we
just noticed, the women remembered immediately that Jesus had said that he would rise from
the dead, and they weren't the only disciples (disciples, not apostles) who understood this. In
the conversation that Jesus had with the two disciples on the road to Emmaus on resurrection
day, Cleopas, after summarizing the events surrounding the trial and crucifixion of Jesus,
clearly indicated that he understood a resurrection was supposed to happen the third day:

But we hoped that it was he who should redeem Israel. Yea and besides all this, it is now the
third day since these things came to pass (Luke 24:21).
It seems, then, that just about everyone who had been associated with Jesus knew that he was
supposed to be resurrected except the apostles. Jesus had apparently entrusted the furtherance
of his important cause to a bunch of dimwits who couldn't understand plain language.

Even the enemies of Jesus understood that he had predicted his resurrection. After Jesus had
been put into the tomb, they came to Pilate to ask that precautions be taken to prevent a staged
fulfillment of the prediction:

The next day, that is, after the day of Preparation, the chief priests and the Pharisees gathered
before Pilate and said, "Sir, we remember what that impostor said while he was still alive,
'After three days I will rise again.' Therefore command the tomb to be made secure until the
third day; otherwise his disciples may go and steal him away, and tell the people, 'He has been
raised from the dead,' and the last deception would be worse than the first" (Matt. 28:62-64,
NRSV).
So the women remembered that Jesus had predicted his resurrection, the disciples at Emmaus
remembered it, and the enemies of Jesus remembered it. Everyone apparently remembered it
except Jesus's own handpicked apostles. That's a little hard to believe.

Bibliolaters preach that the Bible is an inerrant work of unity and harmony so perfect that it
can be explained only by the doctrine of verbal inspiration. It makes great sermon fodder to
feed to gullible pulpit audiences, but this discrepancy in what the apostles didn't know but
should have known about an impending resurrection of their leader is a glitch in the Bible that
must be explained before rational people can accept the inerrancy theory.

Volume 1990 - 2002 Issue


Page 127 of 2049
Skeptical Review Edited by Farrell Till

The Till-McDonald Debate


The Till-McDonald Debate on the issue of Bible morality was held in Sullivan, Missouri, on
July 29-30 and August 1-2. It was poorly attended, with a low of 12, besides participants and
moderators, and a high of 19. The audience was obviously comprised of confirmed
fundamentalists, who had come to see an agnostic take a good shellacking, for during the
nightly question-answer period following the speeches, all questions from the audience were
addressed to Till. None were directed to McDonald, although he had taken some absurdly
ridiculous positions.

As fundamentalists always do in debates on this issue, McDonald defended the massacres of


the Midianites, Canaanites, and Amalekites as acts ordered by an infinite God who can do no
moral wrong. In answer to a question Till directed to him, McDonald even said that if he had
lived in biblical times, he would have willingly participated in these massacres by "personally
killing old women, pregnant women, mothers, children, and infants." Apparently without
seeing the absurd inconsistency in it, he later criticized Till for refusing to take a public stand
against abortion.

The position McDonald took in the debate was based entirely on an assumption of the
existence of objective morality, "which emanates from the nature of God." Till emphasized
that proof of this position would require McDonald to prove (1) that God exists, (2) that his
nature is infinitely good, and (3) that this infinitely good god was Yahweh, who had
commanded the Israelites to commit the massacres under consideration. Till repeatedly
challenged McDonald, who claims that the Bible is his only guide in religious matters, even
to cite a single scripture that says that "morality emanates from the nature of God." He was
never able to do it.

Like all Church-of-Christ preachers, McDonald piled assumption onto assumption throughout
the debate and considered mere assertions to constitute proof. Several times he flashed a chart
on the screen that listed 10 elements in his proposition that he had "proven": (1) God does
exist, (2) objective morality does exist, (3) the Bible is of divine origin, (4) God is good, (5)
God is merciful, (6) God is holy, (7) God has the attribute of wrath, (8) God hates sin and is
sinless, (9) God is love, and (10) God is omniscient. Just to list the elements is enough to
show the absurdity of McDonald's claim, because, with the possible exception of number
seven, one would have to spend days and write volumes even to come close to presenting a
reasonable defense of any one of the other nine and especially the first three. His "proof," of
course, consisted primarily of scripture quotations that said the very thing he was trying to
prove, and to the fundamentalist mind this is always sufficient.

Although we were promised audio tapes after each session, we received them only after the
first one, and they were not of good quality. The speakers were often interrupted to allow
operators of recording equipment to correct problems and change tapes that ran out midway in
a speech. As we prepare for press, we have received no word on availability or cost of tapes.
Inquiries should be sent to Jerry McDonald, 97 Florence Street, Sullivan, MO 63080.

Volume 1990 - 2002 Issue


Page 128 of 2049
Skeptical Review Edited by Farrell Till

Volume 1990 - 2002 Issue


Page 129 of 2049
Skeptical Review Edited by Farrell Till

The Skeptical Review


Volume Three - 1992
Farrell Till, editor

• Number 1 Volume Three, Issue One


• Number 2 Volume Three, Issue Two
• Number 3 Volume Three, Issue Three
• Number 4 Volume Three, Issue Four

Volume 1990 - 2002 Issue


Page 130 of 2049
Skeptical Review Edited by Farrell Till

The Skeptical Review


Volume Three, Number One
January/February/March 1992
Farrell Till, editor

• Silence in Fantasyland
According to Till, "The silence of men like Archer and Jackson when they are offered
publishing space to defend their belief in inerrancy only deepns our suspicion that they
know enough to realize their position cannot survive close public scrutiny."

• A Case in Point
Dr. Robert H. Countess, who attended the International Council on Biblical Inerrnacy
(ICBI), discusses a single case in point that will explain why he could not
conscientously sign a document that proclaimed total inerrancy in all matters of the
Bible.

• Sons of God: Just the Godly Lineage of Seth?


Another article about the "sons of God/ daughters of men" myth refered to in Genesis
6:1-4.

• Did They Tarry in the City?


Did Jesus meet the disciples in Galilee after his resurrection? Farrell Till writes that
"this meeting in Galilee poses tremendous credibility problems."

• What Happened to the Resurrected Saints?


Concludes Ed Babinski, "those of us who doubt the story of the many raised saints see
in it a reflection of the kind of blind faith that made the story of Jesus' resurrection
catch on in the first place."

• Non-Profit Status
The Skeptical Review has been recognized by the IRS as a non-profit organization.

Volume 1990 - 2002 Issue


Page 131 of 2049
Skeptical Review Edited by Farrell Till
• Second Deaver-Till Debate Cancelled
Till received a letter from Mac Deaver stating that "I no longer have any interest in
debating you in the near future."

Silence in Fantasyland
On October 26-28, 1978, the International Council on Biblical Inerrancy (ICBI) met in
Chicago to begin a ten-year project designed to define and defend the biblical inerrancy
doctrine. In this issue of TSR, we are featuring an article by Dr. Robert H. Countess, who was
present at the Chicago ICBI summit. Dr. Countess did not sign "The Chicago Statement on
Biblical Inerrancy" that was released to the press at the close of the summit. In the article we
are publishing, he discusses a single "case in point" that will explain why he could not
conscientiously sign a document that proclaimed total inerrancy in all matters that biblical
authors "were moved to speak and write about."

In the article, Dr. Countess quoted a comment about his "case in point" that Dr. Gleason
Archer, the noted inerrancy defender, made in a personal letter. We wrote to Dr. Archer
several times and offered him equal space to reply to Dr. Countess's article. In response, we
finally received a letter from Dr. Archer in which he listed and discussed three "sinister
consequences [that] logically ensue from the assumption that the Bible is not what it purports
to be, namely, the inspired Word of God, revealed to fallen man for his salvation." We regret
to say that none of his three points addressed or even mentioned the "case in point" that Dr.
Countess discussed in his article.

For that reason, we are not publishing Dr. Archer's letter. Our policy is to print only materials
that pertain directly to the issue of Bible inerrancy, and in Dr. Archer's letter, we saw nothing
that related to the issue except passing mention that he made of "predictive prophecies" that
prove divine inspiration. He made no attempt to explicate any alleged prophecy to prove the
merit of his point. Because of Dr. Archer's reputation, however, we will gladly send a copy of
his letter to anyone who requests it. Please include with the request a self-addressed, stamped
business envelope.

Dr. Countess has described himself as a conservative Presbyterian minister "who holds a high
view of Scriptures." By this, he means that one "should say on the scholarly-historical-critical
basis NO MORE and NO LESS for the Scriptures than one would say UPON CAREFUL
EXAMINATION of any other collection of ancient writings." In a letter to us, he said this
about his approach to biblical interpretation:

I seek to CONSERVE a document's historical, philosophical, social, physical character. To


me, this is the "high view" of Scripture.... In my opinion, a Gleason Archer does NOT hold to
a high view when he twists a text's error factor and seeks to make unilaterally an error into
truth.
We are delighted to feature a writer with Dr. Countess's credentials. His "case in point" is
simple but persuasive enough to destroy the illusion of Bible inerrancy.

Volume 1990 - 2002 Issue


Page 132 of 2049
Skeptical Review Edited by Farrell Till
In this issue, we are also publishing another article about the "sons of God/ daughters of men"
myth referred to in Genesis 6:1-4. This one is a response to an article that Wayne Jackson, a
prominent Church-of-Christ preacher, published on the same subject in Christian Courier, a
monthly fundamentalist paper that he edits. We have quoted Mr. Jackson's articles many
times in TSR, and each time we have offered him equal space to respond. Each time he has
refused, as he did this time too.

We have often said that lay people who believe in Bible inerrancy are living in a fantasyland,
but we are not so sure that the same is true of preachers who proclaim the doctrine so loudly
in one-sided situations. The silence of men like Archer and Jackson when they are offered
publishing space to defend their belief in inerrancy only deepens our suspicion that they know
enough to realize their position cannot survive close public scrutiny. So when they are offered
the chance to defend Bible inerrancy in any kind of forum where they will be confronted by
an informed opposition, they refuse it. They prefer to confine their efforts to captive pulpit
audiences and one-sided religious journals.

They prefer silence in fantasyland to open examination of what they preach.

A Case in Point
Dr. Robert H. Countess
One ought to be able to apply the ICBI definition(s) of inerrancy to any page or verse or
phrase or word of the Scriptures and thereupon judge whether or not the absolutist claims
about inerrancy are suitable--IF, that is, one proceeds from the Lindsell-phenomena approach.
(The Van Tillian presuppositionalist, by definition, will not be bothered by problems with the
phenomena!)

The inerrancy debate, in my opinion, proceeds out of the discipline of Systematic Theology, a
highly abstract and subjective discipline--witness the plethora of "systematic theologies" in
Christianity! Instead, the debate ought to center itself in the discipline of Exegetical
Theology, especially since the phenomena confront the student before systematics, although I
must admit that in the brainwashing approach, this order becomes reversed.

I have selected 1 Corinthians 1:14-16 for a case study. Here, Paul asserts with the utmost
clarity whom he did and whom he did not baptize of the believers at Corinth. What follows is
my own highly literal translation:

I am thankful that not one of you did I baptize except Crispus and Gaius, lest anyone might
say that in my name you were baptized. Now I did baptize also the Stephanas household.
Besides, I know not if any other I did baptize.
I chose this passage because there are no problematic variant readings in the manuscript
traditions. There are no difficult grammatical constructions; nor are there any words whose

Volume 1990 - 2002 Issue


Page 133 of 2049
Skeptical Review Edited by Farrell Till
meaning-usage is in great dispute with the obvious exception of any who disagrees with me as
to the historical usage of baptizo!

With the path somewhat cleared, we see "the bottom line": Paul asserted that "I baptized none
of you all there at Corinth EXCEPT for two believers, and their names are Crispus and
Gaius."

The problem for the ICBI definition of inerrancy as a technical term possessing the
characteristics of being "totally, wholly, etc. trustworthy, true, reliable, accurate," etc. is the
word "except." I would not have selected this Pauline passage had he written, "I baptized no
one" (period!) and then had omitted the references to Crispus, Gaius, and Stephanas.

On the other hand, as the facts of this matter apparently were, if he had written to the
Corinthian church "I baptized no one," he would have erred. But even had he erred, I can
imagine Crispus and Gaius musing in a mature manner as follows:

CRISPUS: "Hey, there, Paul, you baptized us!"

GAIUS: (Thoughtfully) "Crispus, he apparently forgot that he baptized us, but it is of little
consequence in light of the point he's trying to make about the centrality of Christ. This
oversight, this lapse of Paul's memory, doesn't make us any less baptized in Christ."

CRISPUS: "You're right, Gaius. Let's rather be glad that he baptized so few people himself.
This way they can't go around boasting about having been baptized at the hand of the great
Apostle Paul."

Now, in point of the manuscript tradition, Paul DID NOT WRITE, "I baptized no one"
(period). On the contrary, he wrote, "I baptized none of you except Crispus and Gaius."

That is a simple assertion. Taken at face value, it is an assertion of universal negation but
having expressly two--and only two--exceptions: Crispus and Gaius. Paul then leaves off
naming the exceptions and goes on to speak of his concern about people who would place an
exaggerated emphasis upon having been personally baptized by him.

It is only after the latter that his memory becomes jogged to the extent that he recalls his
having baptized more than just Crispus and Gaius. He failed to include the Stephanas
household. Paul then CORRECTS his earlier universal-negation-with-only-two-exceptions
assertion. He adds another exception: the household of Stephanas.

The new bottom line reveals that Paul corrected himself. In doing so, he revealed that
humanness to which we all seem to be subject: relative accuracy and relative fallibility. This
new twist is consistent with Berkouwer's remarks on the servant character of Scripture as
"relatable" to phenomena of Scripture. Paul exhibits the phenomenon of making an error in
fact but then correcting it (cf. ICBI point XIV).

The ICBI proponents cannot, I judge, treat this particular phenomenon in 1 Corinthians 1:14-
16 without resorting to some sort of detour around their absolutist, technical definition of

Volume 1990 - 2002 Issue


Page 134 of 2049
Skeptical Review Edited by Farrell Till
inerrancy. I have personally confronted several inerrancists with this passage and have
universally (!) found them to employ a most interesting circumlocution: that inerrancy does
not require (1) omniscience, nor (2) complete precision, nor (3) entire harmonizability. In a
letter to me on this passage, Dr. Gleason Archer insisted that Paul be allowed "the liberty of
expression" that we allow to each other! (Of course, I am willing to do just that, but with that
concession I also allow others to be in error at times and to have to correct themselves.)

Archer concluded his defense of Paul (rather the ICBI's defense of their inerrancy theory):

He imparts this information in an informal manner, to be sure, but by the time he has finished
this item he has given all of the information, and done so with accuracy.
I suggest that an analogy to this "detour-defense" might be a math student who gives
incomplete information on a test, while the teacher, when marking the incorrect response with
red ink, allows the student to redo the work and hand it in later and has--using Archer's
words-- "done so with accuracy." In such an analogy I see both the math teacher and Archer
exhibiting grace toward an erroneous student and an apostle; in Archer's case, grace takes a
back seat to cover up.

Archer recently authored a book treating alleged errors in the Bible. What I wish to note is
that on the passage in question, this defender of a total inerrancy appears to be content with
something less than total (cf. ICBI articles III, VI, XII, XIV). Archer avoids the precise
grammar of Paul's universal-negative-with-only-two-exceptions by means of transforming it
into a universal positive statement to which other positive additions can be made later, and,
which additions, would not point out Pauline fallibility.

The grammar of the text does not, however, allow for such a gracious detouring. One
inescapably must conclude that in verse 14 Paul erred when he wrote that he had baptized no
one but Crispus and Gaius. In verse 16, Paul corrected himself by the addition of the
Stephanas household. I must insist that we readers are not aware of the error of verse 14 until
we read of Paul's correction of that error in verse 16. Without the correction, we probably
would have never become aware of the error.

What is called for by the ICBI and all would-be inerrancists is candor to admit that the
phenomenon of Pauline self-correction CANNOT comport with the abstractly theological
articles that Chicago's summit produced. It is my contention that the ICBI articles were
produced NOT in conjunction with the phenomena of exegesis but, on the contrary, against
the reality of the phenomena of exegesis. As there has been a Realpolitik, there now needs to
be in evangelical circles a "Realexegese."

THE PROBLEM OF
THE ELUSIVE AUTOGRAPHA
The ICBI does have going for it something that cannot be overcome by any critic: non-
existent Scripture originals affirmed to be inerrant. The inherent safety of this position is that

Volume 1990 - 2002 Issue


Page 135 of 2049
Skeptical Review Edited by Farrell Till
no critic can possibly examine for errors that which no longer exists. But a simple application
of logical inference, however, can lead one to conclude that a manuscript tradition with so
many errors/problems at least seems to point out that the autographs probably partook of the
same humanness--unless one insists on the presupposition of immediate dictational inerrancy
as with the Koran.

Even inerrancists themselves admit (with a few hardcore hold-outs) that the present
manuscripts possess real errors of sorts. Thus, in the final analysis, ICBI abandons the
phenomena approach when the subject of the originals comes up and jumps to the
presuppositional approach (shades of Van Til), BECAUSE THE PHENOMENA
APPROACH JUST WILL NOT WORK. Hence, I assert, we see two radically different
methods at work: (1) the phenomena method with its powerful apologetical persuasiveness
over multitudes of people, and (2) the presuppositional method when (1) cannot be employed.

I suggest that we see here at least the problem of inconsistency if not the problem of integrity.
Integrity can be saved only, I believe, if the inerrancist confesses at the outset that he is going
to use two different methods of persuasion, that the implications of the two are at variance,
and that if he cannot persuade by means of empirical data, he will then ask the "persuadee" to
assume the veracity of that which he initially said he would demonstrate empirically. (I do not
know an inerrancist who will do this!)

As a critic of the ICBI position, I do well not to criticize the autographs, at least not beyond
being willing to make a mild inference or two about them, for why should I engage in
laborious effort to justify or attack that which all parties agree does not exist? Critics do well,
I suggest, to limit themselves solely to the present manuscript tradition.

THE CIRCULAR REASONING OF


INERRANCISTS
The evidentialists in evangelical apologetics enjoy attacking the "wretched
presuppositionalists" (John Warwick Montgomery's term) for their circular reasoning. For an
evidentialist to be accused of the latter is tantamount to being accused of being a communist.
However, I have observed the following line of reasoning amongst the ICBI folks:
Our present Bibles have errors of sorts. From these Bibles or manuscript tradition, we have
deduced and/or inferred what we now call inerrancy. Inerrancy applies to the whole of the
Bible because Bible and God's Word are synonyms. However, this total, entire, and complete
inerrancy applies in its fullest sense ONLY to that corpus of writings called "autographa."
Unfortunately, the latter long ago perished, and we are not able to examine them empirically
to measure our theory against them. BUT we are still justified in extrapolating from current
errant Bibles back to our (allegedly) inerrant autographs.
I truly believe that I have presented a fair synopsis of ICBI reasoning.

Critics of ICBI are not all slow to point out that there is no way to demonstrate that the
extrapolation is justifiable or infallible or inerrant itself. Theological extrapolations are no less

Volume 1990 - 2002 Issue


Page 136 of 2049
Skeptical Review Edited by Farrell Till
precarious than astronomical or biological or geographical extrapolations. The ICBI line of
circular reasoning is an example of what I have called "inferential theology" (Journal of
Psychology and Theology, Summer 1977, pp. 220ff). It is not that I fault religionists for
engaging in inferential theological activity; such activity is unavoidable. What I do find fault
with is the dogmatism that frequently surrounds such inferring. Dogmatism is unwarranted by
the nature of the so-called "problem of induction." Rather, some measure of pious agnosticism
is in order.

ICBI thus assumes at the outset that the autographs are inerrant. Then ICBI extrapolates back
to the autographs from a present day errant Bible text and thereupon declares quite
dogmatically that this method is justifiable because faith requires an inerrant source of
recorded revelation. (To me, this line of circularity is reminiscent of the papal infallibility
argument that requires an infallible interpreter to convey an infallible Scripture. Certainly, the
logic of both positions has a tantalizing aspect, lacking, however, in compelling logical
persuasiveness.)

Of signal interest is the evangelical admissions that God can and does work through Bibles
generally regarded to have failings and errors in text. Why cannot these evangelicals take the
further step that God might even be able to have worked from originals with failings and
errors of sorts? These evangelicals--with errant Bibles in hand--continue to preach, teach,
evangelize, and missionize. Why could God not have worked similarly from fallible
autographs (non-absolute documents)? This may seem to be only a rhetorical question, but its
logic impresses at least me.

(Dr. Countess's address is 120 Sagewood Circle, Toney, AL 35773.)

COMPLETE ARTICLE AVAILABLE


"A Case in Point" was excerpted from a longer paper that Dr. Countess presented on March
18, 1983, to the Society of Biblical Literature in Atlanta. The longer paper discussed and
quoted several of the articles in "The Chicago Statement on Inerrancy."

The entire paper is available upon request. Please include a self-addressed, stamped business
envelope. To receive both this paper and a copy of Dr. Archer's letter, please affix 52 cents
postage.

Sons of God: Just the Godly Lineage of


Seth?

Volume 1990 - 2002 Issue


Page 137 of 2049
Skeptical Review Edited by Farrell Till
Farrell Till
In a front-page question-and-answer column in the June 1991 issue of Christian Courier,
editor Wayne Jackson presented a bibliolater's view of Genesis 6:1-4, which was also the
subject of an article exchange in the autumn edition of TSR. Jackson, of course, pooh-poohed
the mere suggestion that this passage in any way implies the consorting of angels with earthly
women. Here is his entire eisegesis of the passage with his original spelling, punctuation, and
grammar retained:
The context of Genesis 6:1f speaks of the "sons of God" who took wives of the "daughters of
men." Subsequently, the record reveals that in those days "the Nephilim were in the earth."
From these phrases, it has been assumed by some Bible students that certain fallen angels
("sons of God") mated with women of the earth ("daughters of men"), and that to these unions
was born a sort of hybrid race called the Nephilim.

For this theory there is no evidence, and it runs counter to numerous biblical facts. Note: (1)
Angels are spirit beings (Heb. 1:14). As such, they do not consist of flesh (Luke 24:39),
hence, they are incapable of a physical relationship. (2) Christ Himself plainly said that angels
do not marry (Mt. 22:30; Mk. 12:25; Lk. 20:34-35). (3) There is, in fact, nothing in Genesis
6:4 that indicates the Nephilim were offspring of the marriages suggested in this context. (4)
The word "Nephilim" usually identified as "giants" (ASVfn), is a term of uncertain meaning.
Likely it suggests the idea of strength and prowess. It is used in Numbers 13:33 of certain
inhabitants of Canaan whom the Israelite spies encountered in the survey of the land. The
context indicates that they were merely "men of great stature" (32), and nothing hints there
[sic] they were the progeny of angels.

The most reasonable view of Genesis 6:1f is that the allusion refers to the fact that some men,
from the godly lineage of Seth, called "sons of God" (an expression denoting those in
covenant relationship with Jehovah--cf. Deut 14:1; 32:5), began to pursue fleshly interests,
and so took wives of "the daughters of men," i.e., those who were unbelievers. (Is there any
principle that we can learn from this?) The subsequent context seems to suggest that it was
this carnal trend that ultimately brought the Flood--which prompts this question. If the "sons
of God" were angels, how did the Flood serve as a judgment upon them? Can angels drown?

Although Mr. Jackson asserts that there is no evidence for the "theory" that Genesis 6:1-4
referred to the intermarriage of angels and human women, a review of my exchange of
articles with Steve Gunter on this same subject (TSR, Autumn 1991, pp. 2-11) will show that
there is not just "evidence" but overwhelming evidence that people living in the biblical era
seriously believed that such marriages had actually occurred in antediluvian times.

Chapters 1-20 in the book of First Enoch are devoted entirely to the premise that angels or
watchers had descended from heaven, married human women, and thereby produced a state of
almost total corruption upon the earth. The language in this section of the book is too specific
and the descriptions too detailed to leave any doubt that the author believed that such
marriages had indeed happened. My first article on the subject pointed out that the "inspired"
writer Jude (vv:14-15) had quoted 1 Enoch 1:9 in a way that both endorsed the Enochian
authorship of the book and attributed prophetic powers to its author. As I said then, these
facts, along with the biblical assertion that Enoch was so righteous that God translated him

Volume 1990 - 2002 Issue


Page 138 of 2049
Skeptical Review Edited by Farrell Till
directly to heaven (Gen. 5:24; Heb. 11:5), leave bibliolaters no recourse but to regard as truth
whatever "Enoch" said about angels and women marrying. To say the least, such facts as
these can hardly be regarded as "no evidence" for the "theory" that some in biblical times
believed angels had once intermarried with earthly women.

But evidence to support the "theory" doesn't end with 1 Enoch. Legends of such marriages
having occurred in antediluvian times abound in apocryphal literature. Before his death in
Testaments of the Twelve Patriarchs, Reuben warned his sons not to be captivated by women
who adorn themselves "to deceive men's sound minds":

For it was thus that they (women) charmed the Watchers, who were before the Flood. As they
continued looking at the women, they were filled with desire for them and perpetrated the act
in their minds. Then they were transformed into human males, and while the women were
cohabiting with their husbands they appeared to them. Since the women's minds were filled
with lust for these apparitions, they gave birth to giants. For the Watchers were disclosed to
them as being as high as the heavens (5:6).
In apocryphal literature--and sometimes even in the Bible--watchers was a term frequently
used for angels. In relating a dream to Daniel, king Nebuchadnezzar said that he saw a
"watcher and a holy one" come down from heaven (Dan. 4:13). The same designation
(watcher) for angels was used two other times in the same context (vv: 17,23). So when
"Reuben" spoke in the above passage about "watchers before the flood" who had been
"transformed into human males" and who then consorted with women who "gave birth to
giants," what could he have been speaking of but the same legend that figured so prominently
in 1 Enoch?

This legend was mentioned also in the book of Jubilees in relating events in the life of Enoch:

And he was therefore with the angels of God six jubilees of years. And they showed him
everything which is on earth and in the heavens, the dominion of the sun. And he wrote
everything, and bore witness to the Watchers, the ones who sinned with the daughters of men
because they began to mingle themselves with the daughters of men so that they might be
polluted. And Enoch bore witness against all of them (7:21).
As the story continues, Noah, after the birth of his grandchildren, bore witness and
admonished them to "preserve themselves from fornication and pollution and from all
injustice":
For on account of these three (things) the Flood came upon the earth. For (it was) because of
the fornication which the Watchers, apart from the mandate of their authority, fornicated with
the daughters of men and took for themselves wives from all whom they chose and made a
beginning of impurity. And they begot sons, the Naphidim, and all of them were dissimilar.
And each one ate his fellow. The giants killed the Naphil, and the Naphil killed the Elyo, and
the Elyo mankind, and man his neighbor (7:21-22).
References to the teachings of the Watchers by which they corrupted the earth (8:3) and their
punishment by confinement within the earth until the day of judgment (10:5) agree essentially
with the legend as it was presented in 1 Enoch. Other allusions to the legend were made in
The Testament of Naphtali (3:5) and Qumran MS4. These contemporaries of the OT authors
consistently spoke of angels or watchers or children of heaven, who married human women,
produced giants, and corrupted the earth to a degree that caused God to send the great flood.

Volume 1990 - 2002 Issue


Page 139 of 2049
Skeptical Review Edited by Farrell Till
With this many references in the literature of Old Testament times to a belief that giants had
been born in antediluvian days to women who had married angels, Mr. Jackson would have to
be brazenly uncritical to argue that Genesis 6:1-4 was not referring to the same myth, and
especially so since "Enoch the seventh from Adam" (Jude 14) had written so specifically and
clearly about the myth.

In the July 1991 issue of Reason and Revelation, an "apologetic" paper that he also edits, Mr.
Jackson began a series on "Josephus and the Bible." The first article almost rhapsodized the
"valuable contribution" that the testimony of the Jewish historian Josephus makes in
confirming the accuracy of "Sacred Scripture." At one point in the article, where the
"testimony" of Josephus didn't quite confirm the accuracy of the Biblical account of the same
story, Jackson observed that "the narrative of the Jewish historian is marred by some
discrepancies" (p. 27). In other words, when Josephus's narrative agrees with the Bible
(obviously the primary source that Josephus relied on to write The Antiquities of the Jews),
Jackson sees this as wonderful confirmation of "Sacred Scripture," but when Josephus
disagrees with the Bible, Jackson dismisses these occasions as "discrepancies" that mar the
narrative of the Jewish historian. It is a familiar game that bibliolaters play. Whatever
extrabiblical materials help their case, they readily accept, but whatever hurts their case, they
reject. They feel no need to critically appraise such materials. The fact that they contradict the
Bible is the only reason they need to reject them.

No doubt, then, Jackson will say that the testimony of Josephus on the subject now under
consideration was "marred" by discrepancies, because Josephus, like the writer of 1 Enoch,
obviously thought that angels had once married earthly women and produced a race of giants.
After stating that men began to show "a double degree of wickedness, whereby they made
God to be their enemy," he said this:

(F)or many angels of God accompanied with women, and begat sons that proved unjust, and
despisers of all that was good, on account of the confidence they had in their own strength; for
the tradition is, That these men did what resembled the acts of those whom the Grecians call
giants" (Antiquities of the Jews, 1.3.1).
The very next statement of Josephus introduced Noah and God's plan to save humanity
through the ark that he commanded Noah to build--in other words, exactly the same premise
and chronological order that was presented in both Genesis and 1 Enoch. For once, then, I am
going to have to agree with Mr. Jackson. Sometimes the testimony of Josephus does make a
"valuable contribu- tion" to confirming the accuracy of the Bible. Genesis 6:1-4 "accurately"
presented the antediluvian myth about angels and earthly women, and the testimony of
Josephus makes a "valuable contribution" to confirming that this was exactly what the
Genesis writer meant.

In addition to the references already noted, the obvious existence of this antediluvian myth
was also confirmed in the rabbinical midrashim or postcanonical commentaries written to
interpret the Tanach or OT scriptures. In Hebrew Myths, Robert Graves and Raphael Patai
have written an excellent critical analysis of midrashic expansions of this antediluvian myth
about the intermarriage of fallen angels and earthly women (pp. 100-107). Anyone who
doubts that early Jewish tradition and scholarship held that Genesis 6:1-4 was a reference to

Volume 1990 - 2002 Issue


Page 140 of 2049
Skeptical Review Edited by Farrell Till
this myth should get the book and read it. It also has some very enlightening information on
other mythological references in the book of Genesis.

According to Mr. Jackson, the theory that angels once intermarried with human women "runs
counter to numerous biblical facts." He then proceeded to play the game of pitting scripture
against scripture to try to show that passages embarrassing to the inerrancy doctrine cannot
mean what the most obvious and least incredible interpretations assign to them. When, for
example, a bibliolater is confronted with a passage clearly showing that God can and
sometimes does lie (1 Kings 22:17-23), he will argue that the passage cannot mean what it
"seems" to mean, because Hebrews 6:18 and Titus 1:2 clearly state that it is impossible for
God to lie. It never occurs to our bibliolater that Bible writers simply had conflicting religious
views, just as thetheologians of all generations have had conflicting religious views. They are
so hung up on their fantasy-land belief that the Bible is perfectly harmonious from cover to
cover that they cannot even consider the possibility that whoever wrote 1 Kings 22 simply
disagreed with the writers of Hebrews and Titus on this particular point.

Jackson used this same scripture-against-scripture tactic to begin his denial that Genesis 6:1-4
was about intermar-riages of angels and human women. It cannot mean that, he argued,
because "it runs counter to numerous biblical facts." Well, what are these "biblical facts" that
it runs counter to? "Angels are spirit beings (Heb. 1:4)," Jackson told his gullible
fundamentalist audience. "As such, they do not consist of flesh (Luke 24:39); hence, they are
incapable of a physical relationship." But how did he arrive at such an unbiblical conclusion
as this? He will surely agree that the Holy Spirit is a "spirit being," and the Bible very clearly
depicted him as a creature capable not just of "a physical relationship" but of sexual
intercourse. In announcing to Mary the impending birth of Jesus, Gabriel said, "The Holy
Spirit will come upon you, and the power of the Most High will overshadow you; for that
reason the holy child to be born will be called Son of God" (Lk. 1:35, REB).

If Mr. Jackson wants to quibble that the Holy Spirit is not an angel, he should consider the
implications of that quaint little biblical yarn about Lot's encounter with angels. After seeing
two angels come into Sodom, Lot invited them into his home. They accepted, and he made
them a feast that they ate (Gen. 19:1-3). This all sounds very "physical" to me. If angels have
mouths and digestive tracts to use in eating and if they have feet that can be washed (v:2),
why would they not have genitals for use in sexual intercourse? As a matter of fact, when a
group of homosexuals in Sodom heard that these "men" were lodging with Lot, they
surrounded his house and demanded that he send the "men" out to them so that "we may
know them" (v:5). Now I assume that Mr. Jackson understands what that means, so at least
these men of Sodom thought that angels were capable of "physical relationship." Just where,
then, does Mr. Jackson get his "biblical fact" that angels were "incapable of a physical
relationship"?

Some bibliolaters, in trying to circumvent the obvious anthropomorphic description of angels


in this passage, contend that these angels were transformed into men to complete their
mission to warn Lot of the impending destruction of Sodom. But if angels could be
transformed for this purpose, why could they not have transformed themselves so that they
could marry the human women they had lusted after. This is, in fact, what the passage quoted
above from Testaments of the Twelve Patriarchs clearly said: "Then they (the watchers) were

Volume 1990 - 2002 Issue


Page 141 of 2049
Skeptical Review Edited by Farrell Till
transformed into human males." That provides a perfectly sensible (in terms of biblical
sensibility) explanation for how the physical aspects of sexual intercourse between angels and
women could have occurred.

"Christ Himself plainly said that angels do not marry (Mt. 22:30; Mk. 12:25; Lk. 20:34-35),"
Jackson went on to say. In this statement, there is an assumption of harmony in the Bible. Mr.
Jackson is completely unwilling to concede that even if Jesus did believe that angels do not
marry, this would not mean that the Genesis writer also believed that they do not marry. It
could simply be that Jesus and the Genesis writer disagreed on this point, and before Jackson
himself would have a valid point, he would have to prove that the two did not disagree.

On closer examination of the passages that Jackson cited in support of this point, we can see
that Jesus did not say that angels do not marry. What he actually said was that angels in
heaven do not marry, but the legend as related in 1 Enoch and other apocryphal works clearly
states that in times past angels did marry. None of them said, however, that angels marry in
heaven. They all depicted them as angels who left heaven to marry earthly women:

And they were altogether two hundred; and they descended into Ardos, which is the summit
of Hermon. And they called the mount Armon, for they swore and bound one another by a
curse.... And they took wives unto themselves, and everyone (respectively) chose one woman
for himself, and they began to go unto them.... And the women became pregnant and gave
birth to great giants whose heights were three hundred cubits (1 Enoch 6:6; 7:1-2).
So no one, including even the Genesis writer, ever even implied that these marriages between
angels and women occurred in heaven. The angels left heaven and came to earth. As Jude said
in obvious allusion to the angels in this legend, "And angels who did not keep their own
position, but left their proper dwelling, he (God) has kept in eternal chains in deepest darkness
for the judgment of the great Day" (v:6).

In the most ridiculous of all his points, Jackson said, "There is, in fact, nothing in Genesis 6:4
that indicates the Nephilim were offspring of the marriages suggested in this context." Oh?
That's about like saying that there is nothing in Mark 16:16 or Acts 2:38 to indicate that
baptism has anything to do with salvation, and we can imagine how quickly Mr. Jackson's
Church- of-Christ mind would reject the mere suggestion of that. "The Nephilim were on the
earth in those days--and also afterward," the passage says, "when the sons of God went in to
the daughters of humans, who bore children to them. These were the heroes that were of old,
warriors of renown" (NRSV). A comparison of translations will show an overwhelming
indication that "the Nephilim were offspring of the marriages suggested in this context":

The Nephilim were on the earth in those days--and also afterward--when the sons of God
went to the daughters of men and had children by them. They were the heroes of old, men of
renown (NIV).

In those days as well as later, when the sons of the gods had intercourse with the daughters of
mortals and children were born to them, the Nephilim were on the earth; they were the heroes
of old, people of renown (Revised English Bible).

Volume 1990 - 2002 Issue


Page 142 of 2049
Skeptical Review Edited by Farrell Till
At that time the Nephilim appeared on earth (as well as later), after the sons of heaven had
intercourse with the daughters of man, who bore them sons. They were the heroes of old, the
men of renown (New American Bible).

The Nephilim were on the earth at that time (and even afterward) when the sons of God
resorted to the daughters of man, and had children by them. These are the heroes of days gone
by, the famous men (Jerusalem Bible).

There were giants on the earth in those days, and later, too, when the sons of God used to
cohabit with the daughters of men, who bore them children, those mighty men of old who
made a name (Revised Berkeley Version).

There were giants on the earth in those days--and also afterward--when the sons of God lived
with the daughters of men, and they bore children to them. These were the mighty men who
were of old, men of renown (Amplified Bible).

There were giants on the earth in those days; and also after that, for the sons of God came in
unto the daughters of men, and they bore children to them, and they became giants who in the
olden days were mighty men of renown (Lamsa's Translation of the Peshitta Text).

In those days, and even later, there were giants on the earth who were descendants of human
women and the supernatural beings. They were the great heroes and famous men of long ago
(Good News Bible).

Of all translations of this verse, however, none is any clearer than the Holy Spirit's favorite
version, the Septuagint, in saying that giants had resulted from the sexual union of angels and
human women:
Now the giants were upon the earth in those days; and after that when the sons of God were
wont to go in to the daughters of men, they bore children to them, those were the giants of
old, the men of renown (Brenton Translation).
If Mr. Jackson can read all these translations of Genesis 6:4 and still say with a straight face
that there is nothing in the verse "that indicates the Nephilim were offspring of the marriages
suggested in this context," then he needs some serious help with basic hermeneutics.

His fourth point was that "(t)he word 'Nephilim,' usually identified as 'giants' (ASVfn), is a
term of uncertain meaning." As noted in "Bible Biology" (TSR, Spring 1991, p. 10), he tried
this same dodge in trying to show that no scientific error occurred in classifying hares and
conies as cudchewers (Lev. 11:5-6). The words translated hare and cony were "rare and
difficult" in Hebrew, he contended, so we can't even be sure that hares and conies were the
animals "Moses" had in mind. Whenever a serious threat to the inerrancy doctrine is seen in
situations like these, fundamentalists will often plead "uncertainty of meaning" in the
language used. Apparently, they never stop to consider the damage that such a defense does to
the very doctrine they are trying to prove. An omniscient God oversaw the writing of the
Bible down to the selection of individual words, yet the language was so vague and uncertain
in places that we can't really be sure what was meant. This is the best that omniscience could
do?

Volume 1990 - 2002 Issue


Page 143 of 2049
Skeptical Review Edited by Farrell Till
At any rate, Jackson claims that the meaning of nephilim was "uncertain" and this despite
numerous OT passages that very plainly associated the word with giantism. The spies sent
into Canaan saw the Nephilim in the land and described them as "men of great stature" (Num.
13:32). The spies said that in the presence of these people they had felt like grasshoppers "in
our own sight" (v:33). These Nephilim were said to be "the sons of Anak," and in other places
were called Anakim. Like Nephilim, the Anakim were always associated with great stature. A
people "great, and many, and tall," who had once dwelt in Moab, were compared to the
Anakim (Dt. 2:10). There are enough references like these in the Old Testament to clearly
establish that nephilim was a word used to mean giants, and except for the absurdity that the
word exposes in the Bible inerrancy doctrine, Mr. Jackson would not argue the point. As for
his claim that "nothing hints that they (the Nephilim) were the progeny of angels," I will just
leave it to the readers to review the various translations of Genesis 6:4 quoted above and let
them decide if there is anything to "hint" that the Nephilim were considered the progeny of
angels.

Jackson arbitrarily said that "(t)he most reasonable view of Genesis 6:1f is that the allusion
refers to the fact that some men, from the godly lineage of Seth, called 'sons of God'... began
to pursue fleshly interests, and so took wives of 'the daughters of men.'" But what did he offer
as proof of this "most reasonable view"? Nothing whatsoever! In "Much Ado About Nothing"
(Autumn 1991, p. 7) Jackson's inerrancy cohort, Steve Gunter, said that the sons of God in
Genesis 6:1 were "the descendants of righteous Abel" (despite the fact that the Bible doesn't
even mention any children that Abel had). So which were they, these "sons of God,"
descendants of Abel or descendants of Seth? One inerrancy defender says one thing, another
something else.

Did either man cite any places in the biblical text where descendants of Abel or Seth were
called "sons of God"? No, they didn't, and they didn't because they can't. There simply are
none. Like Gunter's purely arbitrary pronouncement that these "sons of God" were
"descendants of rightous Abel," Jackson's "most reasonable view" is purely speculative, a
recourse that he must resort to in order to defend the absurd notion that the Bible was verbally
inspired--every word of it-- by the omniscient Yahweh of the Hebrews.

He thought that he had found in Deuteronomy 14:1 and 32:5 an indication that "sons of God"
was "an expression denoting those in covenant relationship with Jehovah," but neither of
these passages used the exact term (beni ha-elohim) that was twice translated "sons of God" in
Genesis 6.The expression and its usage in the Old Testament were analyzed in "If It Walks
Like a Duck..." (TSR, Autumn 1991, pp. 3-4), so I will not repeat myself on this point. A
review of this section of the article, however, will clearly show that the expression was
always applied to celestial beings, who in at least one case (Job 38:4-7) existed before man
was even created.

Jackson can say that the expression denoted "those in covenant relationship with Jehovah" all
that he wishes, but he has no proof of that. The term covenant (Heb. berŒyt) doesn't even
appear in the Bible text until after the "sons of God" had taken wives from the daughters of
men, so why would the descendants of Seth have been in "covenant relationship with
Jehovah" any more than descendants of the other sons and daughters who were born to Adam
and Eve after the birth of Seth (Gen. 5:3)? [As far as actual textual records are concerned,

Volume 1990 - 2002 Issue


Page 144 of 2049
Skeptical Review Edited by Farrell Till
Cain was the only child of Adam and Eve who went bad.] The "sons of God" who came to
present themselves to Yahweh (Job 1:6; 2:1), were these descendants of Seth too who were in
"covenant relationship with Jehovah"? If so, where is that covenant relationship even hinted at
in the book of Job? For that matter, where is their descent from Seth even suggested? If
Jackson is going to make an assertion like this, he should be prepared to support it with more
than two verses that didn't even contain the same Hebrew expression.

Jackson said that "(t)he subsequent context (of Genesis 6) seems to suggest that it (marriages
between the sons of God and the daughters of men) was this carnal trend that ultimately
brought the Flood." I have no problems with this interpretation. It is in complete agreement
with 1 Enoch and the other apocryphal works that elaborated on this myth about the sons of
God or heaven marrying the daughters of men. They all agreed that the flood resulted from
the corruption that the marriages had brought upon the earth. I have all kinds of problems,
however, with a conclusion that he drew from this interpretation. It "prompts this interesting
question," he said. "If the 'sons of God' were angels, how did the Flood serve as a judgment
upon them? Can angels drown?"

These questions simplistically overlook the fact that we are dealing with a definite possibility
of myth, and in mythology anything could happen, even the drowning of angels. Aside from
this, Jackson's questions ignore several important aspects of the myth. First of all, as noted
above in T12P 5:6, these angels "were transformed into human males" and as such would
have been subject to death by drowning as well as any other human males. Jackson may as
well have questioned the death of Jesus on the cross on the grounds that he was God (Jn. 1:1)
and that God could not have been killed. The idea, however, was that Jesus was made human
to suffer in all points as humans do (Heb. 4:15) even to the point of becoming subject unto
death (Phil. 2:5-8). So if God incarnated as a man could die by crucifixion, why couldn't
angels incarnated as men die by drowning? Sometimes Mr. Jackson doesn't seem to be
thinking too clearly, but adherence to a belief as fundamentally absurd as Bible inerrancy will
do that to a person.

Furthermore, the flood was sent not as judgment upon the fallen angels but upon the "children
of adultery" and "children of the Watchers" (1 Enoch 10:9) who had been born to the wives of
the angels. The fornicating angels, after having witnessed "the destruction of their beloved
ones" (v:12), were to be bound "for seventy generations underneath the rocks of the ground
until the day of their judgment" (v:13). So there was nothing in the myth to imply that the
flood was sent to destroy the fallen angels. It was sent to destroy the corruption they had
caused, but they themselves were bound in the earth for judgment of a later date. To this,
Jackson must agree or find himself arguing with the "inspired" writers Jude and Peter:

And the angels who did not keep their own position, but left their proper dwelling, he (God)
has kept in eternal chains in deepest darkness for the judgment of the great Day (Jude 6).

For if God did not spare the angels when they sinned, but cast them into hell and committed
them to chains of deepest darkness to be kept until the judgment... then the Lord knows how
to rescue the ungodly from trial, and to keep the unrighteous under punishment until the day
of judgment--especially those who indulge their flesh in depraved lust, and who despise
authority (2 Peter 2:4-10).

Volume 1990 - 2002 Issue


Page 145 of 2049
Skeptical Review Edited by Farrell Till
As it turns out, then, there are no "biblical facts" to prove that Genesis 6:1-4 was not referring
to the intermarriage of angels and human women.

I realize how embarrassing these things must be to Mr. Jackson. Perhaps that is why he
consistently refuses our offer of space in the pages of TSR (as he has done this time too) to
defend the ridiculous positions he takes in support of Bible inerrancy. However, if he expects
rational people to believe that Genesis 6:1-4 had nothing to do with a mythological belief that
angels had once intermarried with earthly women, he will have to present better evidence than
what he has thus far shown us.

In July 1991, six months before its publication, a preliminary draft of this article was sent to
Mr. Jackson with an offer to simultaneously publish his response to it. He did not accept. (He,
in fact, returned without comment the $3 that I included to renew my subscription to his
paper.) Anyone wishing to contact him about this or any of the other matters on which we
have quoted him in TSR may do so at P. O. Box 55265, Stockton, CA 95205.

Did They Tarry in the City?


Farrell Till
So much depends on the resurrection of Jesus. Without it, the whole superstructure of
Christianity would collapse. To this, agreed even the Apostle Paul, whom some New
Testament scholars consider to be the real architect of Christianity:
(I)f Christ be not raised, your faith is vain; ye are yet in your sins. Then they also which are
fallen asleep in Christ are perished. If in this life only we have hope in Christ, we are of all
men most miserable (1 Cor. 15:17-19, KJV).
As vital to Christianity as the resurrection is, it rests upon the flimsiest of evidence: four
contradictory "gospel" accounts and some scattered references in the New Testament epistles
to a risen Messiah. And that's it. As far as scholars have been able to determine, none of the
gospel accounts of the resurrection were written by anyone who could have been an
eyewitness to the event, and most of the epistolary references to it were made by the Apostle
Paul, who by his own admission did not witness it either. He claimed that he had seen the
resurrected Jesus in a vision on the road to Damascus.

The contradictory nature of the hearsay accounts of the resurrection constitute the most
damaging evidence against it. If alleged eyewitnesses to an event as extraordinary as the
revivification of a dead man should contradict themselves in a court of law as patently as did
the writers of Matthew, Mark, Luke, and John in their telling of the resurrection story, no
intelligent jurors would give a speck of credence to their testimony, yet millions of Christians
have accepted a resurrection story that is riddled with discrepancies.

In future issues of TSR, we will look at several points of discrepancy in the four resurrection
accounts, but for now I will focus on just one: an alleged post-resurrection appearance of

Volume 1990 - 2002 Issue


Page 146 of 2049
Skeptical Review Edited by Farrell Till
Jesus on a mountain in Galilee. If one were to ask a Christian versed in the scriptures if the
disciples of Jesus met him in Galilee after his resurrection, the answer would surely be, "Yes,
they did." After all, Matthew, writing about postresurrection events, clearly said, "But the
eleven disciples went into Galilee, unto the mountain where Jesus had appointed them. And
when they saw him, they worshipped him; but some doubted" (28:16-27).

That seems clear enough, and if Matthew had been the only one to write about
postresurrection events, this meeting in Galilee would certainly be believable to anyone who
could accept the premise that a dead man had been resurrected. Considered in the context of
all four gospel accounts of the resurrection, however, this meeting in Galilee poses
tremendous credibility problems, because Luke said in his gospel that Jesus told his disciples
on the night of his resurrection that they were to stay in Jerusalem until they were "clothed
with power from on high" (24:49). According to the same writer (Luke), this power came to
them about fifty days later when they were baptized in the Holy Spirit on the day of Pentecost
(Acts 1:3-5; 2:1-4), but by then Jesus had already ascended back to heaven, because he had
remained on earth only forty days after his resurrection (Acts 1:3). So if Luke was right and
Jesus did tell his disciples on the night of his resurrection not to leave Jerusalem until they
received "power from on high" and if this power from on high did not come to them until fifty
days later and if Jesus remained on earth for only forty days after his resurrection and if the
disciples obeyed Jesus's command not to leave Jerusalem until they had received power from
on high, how could they have possibly met him on a mountain in Galilee as Matthew
claimed?

Bibliolaters will say that the command to stay in the city until they were "clothed with power
from on high" was not given to the disciples on the night of Jesus's resurrection, but careful
analysis of the text will not support them in this. Luke 24:1-12 described events that occurred
at the empty tomb on the morning of the resurrection. The women went there "at early dawn"
(v:1), found the stone rolled away and the tomb empty (vv:2-3), and encountered two men in
dazzling apparel who told them that Jesus was risen (vv:4-8). The women ran to tell the news
to the eleven (v:9), who considered their words to be only idle talk (v:11), but Peter ran to the
tomb, looked inside, and went back home, "wondering at that which was come to pass"
(v:12).

After all these things were related, Luke said, "And behold, two of them were going that very
day to a village named Emmaus" (v:13). The expression "that very day" surely was intended
to mean the very day of the resurrection, the day all of the events just mentioned had
happened. So the encounter between Jesus and the two disciples on the road to Emmaus
(vv:15-27) had to have happened on the same day that Jesus was allegedly resurrected. If not,
what did "that very day" mean?

Evidence that it was the very day of the resurrection is seen in verse 21. In the conversation
that Jesus had with the disciples, the one named Cleopas stated, with implied doubt, that Jesus
was the Messiah:

But we hoped that it was he who should redeem Israel. Yea and besides all this, it is now the
third day since these things came to pass.

Volume 1990 - 2002 Issue


Page 147 of 2049
Skeptical Review Edited by Farrell Till
Cleopas clearly indicated in this statement that the events being narrated in this passage were
taking place on the third day. What day could that have been except the third day following
the trial and crucifixion of Jesus that Cleopas had just summarized? As any Sunday school
student knows, the resurrection was supposed to have occurred on that third day. So at this
point in Luke 24, everything happening was happening on the day of the resurrection.

When the trio arrived at Emmaus, Jesus "made as though he would go further" (v:28), but the
two disciples "constrained him, saying, Abide with us; for it is toward evening, and the day is
now far spent" (v:29). The "far-spent day" would have been the day the journey started, so
Luke's narrative shows quite clearly that everything he was telling had happened on the third
day, the day of the resurrection.

The insistence of the two disciples prevailed, and Jesus went into Emmaus with them. When
they sat down to "break bread," Jesus blessed the bread and gave it to the others. Until then,
the two disciples had not recognized Jesus, but at the breaking of the bread "their eyes were
opened" and they realized who he was as "he vanished out of their sight" (v:31). "And they
(the two disciples) rose up that very hour, and returned to Jerusalem and found the eleven
gathered together" (v:33).The eleven told the men that "the Lord is risen indeed and hath
appeared unto Simon," and the men told the eleven what they had seen (v:34). Emmaus was
located only seven miles from Jerusalem, so if the two disciples had left "that very hour" after
recognizing the vanishing Jesus and returned to Jerusalem, they would certainly have arrived
there the same night.

While the disciples from Emmaus were telling the eleven what they had seen, Jesus suddenly
appeared out of nowhere:

And as they spake these things, he himself stood in the midst of them, and saith unto them,
Peace be unto you. But they were terrified and affrighted, and supposed that they beheld a
spirit. And he said unto them, why are ye troubled? and wherefore do questionings arise in
your heart? See my hands and my feet, that it is I myself: handle me, and see; for a spirit hath
not flesh and bones, as ye behold me having. And when he had said this, he showed them his
hands and his feet (vv:36-40).
Luke's narrative at this point reads very much like John's account of an appearance that Jesus
made on the night of the resurrection day:
When therefore it was evening, on that day, the first day of the week, and when the doors
were shut where the disciples were, for fear of the Jews, Jesus came and stood in the midst,
and saith unto them, Peace be unto you. And when he had said this, he showed unto them his
hands and his side (John 20:19-20).
The similarity of these two accounts should confirm that the appearance of Jesus to the eleven
that Luke wrote about did allegedly happen the night of the resurrection, because John plainly
said that it occurred on "the first day of the week."

In the continuation of Luke's narrative, Jesus asked for something to eat, and a piece of
broiled fish was given to him (v:41). After eating it, he spoke to the disciples about non-
existent scriptures (as we will see in a later article) that his resurrection had fulfilled (vv:44-
46). Then in giving to them Luke's version of the "Great Commission," he made the statement
that casts serious doubt on Matthew's claim of a postresurrection appearance in Galilee:

Volume 1990 - 2002 Issue


Page 148 of 2049
Skeptical Review Edited by Farrell Till
Ye are witnesses of these things. And behold, I send forth the promise of my Father upon you:
but tarry ye in the city, until ye be clothed with power from on high (vv:48-49).
As related earlier, this "power from on high" that the apostles were to receive presumably
came to them when they were baptized in the Holy Spirit:
(A)nd being assembled together with them, he (Jesus) charged them not to depart from
Jerusalem but to wait for the promise of the Father, which, said he, ye heard from me: for
John indeed baptized with water; but ye shall be baptized in the Holy Spirit not many days
hence (Acts 1:4-5).
The apostles were baptized in the Holy Spirit on the day of Pentecost (Acts 2:1-4), so that
would have been the time that they received Jesus's promise of "power from on high."
Pentecost (from a Greek word meaning fiftieth) fell fifty days after the sacrifice of the
passover lamb (see Josephus, The Antiquities of the Jews, 3.10.5-6), and Jesus was crucified
as the passover was approaching (Matt. 26:1-5, 17-19; Mk. 14:1-2, 12-16; Lk. 22:1-2, 7-13;
Jn. 18:28, 39). John even said that it was during the "preparation of the passover" that Jesus
was taken before Pilate (19:14). So if Jesus was crucified at the time of the passover, he had
already ascended back to heaven when the apostles were "clothed with power from on high"
on the day of Pentecost (50 days after his crucifixion), because Luke claimed that he stayed
on earth only forty days after his resurrection (Acts 1:3).

By now the problem in reconciling Matthew's resurrection account with Luke's should be
obvious. Luke very clearly indicated that Jesus on the night of his resurrection charged the
apostles to stay in Jerusalem until they were "clothed with power from on high" (baptized in
the Holy Spirit, Acts 1:4-5), so that leaves no room for a postresurrection appearance to the
apostles on a mountain in Galilee. As I said earlier, IF Jesus did tell his disciples on the night
of his resurrection not to leave Jerusalem until they received "power from on high" and IF this
power from on high did not come to them until some fifty days later and IF Jesus remained on
earth for only forty days after his resurrection and IF the disciples obeyed Jesus's command
not to leave Jerusalem until they had received power from on high, they couldn't have
possibly met him on a mountain in Galilee as Matthew claimed.

No one can successfully argue that the meeting on the mountain in Galilee happened before
the meeting in Jerusalem on the night of the resurrection, because that would pose other
textual reconciliation problems. For one thing, Galilee was too far from Jerusalem to make
such a meeting logistically possible. The disciples were presumably in Jerusalem the morning
of the resurrection, because the women ran to tell them of the empty tomb (Lk. 24:9). Peter
and another disciple even ran to the tomb, looked inside, and returned home, wondering about
what had happened (Lk. 24:12; Jn. 20:3-9). Are we to believe that after Peter returned home
from the tomb, he and the other apostles journeyed to Galilee (a distance of some sixty or
seventy air miles, even if the mountain was in the southernmost region of Galilee), saw Jesus,
and then returned to Jerusalem ALL IN ONE DAY! The mere suggestion is preposterous (but
perhaps not as preposterous as believing a resurrection story as riddled with contradictions as
the one told by Matthew, Mark, Luke, and John). Besides, Luke said that when the disciples
from Emmaus found the apostles in Jerusalem the night of the resurrection, they were
immediately told, "The Lord is risen indeed, and hath appeared to Simon" (24:34). But if all
of the eleven had met Jesus earlier that day on a mountain in Galilee, they would have surely
told the disciples from Emmaus that Jesus had appeared to all of them.

Volume 1990 - 2002 Issue


Page 149 of 2049
Skeptical Review Edited by Farrell Till
There is just no way to reconcile Matthew's story with Luke's. If Matthew was right about a
postresurrection appearance of Jesus on a mountain in Galilee, then Luke was wrong in at
least some details of his description of a postresurrection appearance in Jerusalem the night of
the resurrection. If Luke was right in all the details he described, then Matthew erred. One of
them had to be wrong. Either that or believers in the resurrection will have to say that the
apostles disobeyed Jesus's command to stay in Jerusalem until they were baptized in the Holy
Spirit. Either alternative they select won't build much confidence in the Bible inerrancy
doctrine.

The principle of falsus in uno, falsus in omnibus (false in one thing, false in everything) is
certainly applicable to the four resurrection stories. This rule of evidence recognizes that
testimony found to be false in one matter should be considered unreliable in other matters. So
if either Luke or Matthew erred in telling the resurrection story, how could it possibly be that
both were protected from error by the Holy Spirit as they wrote? If either of them was wrong
about the when and where of postresurrection appearances, then maybe they were wrong
about the resurrection period. Maybe it didn't even happen. This is surely something for
resurrection believers to think about. Or maybe they would rather go on with their heads in
the sand.

What Happened to the Resurrected Saints?


Ed Babinski
Two short verses in Matthew raise perhaps the most serious questions that can be put to a
literal interpretation of the resurrection stories. Matthew said that at the moment of Jesus'
death "the tombs were opened; and many bodies of the saints who had fallen asleep were
raised; and coming out of the tombs after his resurrection they entered into the holy city and
appeared unto many" (27:52-53). This is an account of a miracle unsurpassed anywhere else
in the gospels. It makes the postresurrection appearing of Jesus "to above five hundred
brethren at once" (1 Cor. 15:6) appear tame in comparison.

In this case, many saints were raised and appeared to many. Unlike the accounts of Jesus
raising Lazarus or the synagogue ruler's daughter or Jesus himself being raised, this depicts
saints dead for way over "three days" being raised. And, from the phrase, "they entered the
holy city and appeared to many," it is possible to infer that these many raised saints showed
themselves to many who were not believers! Yet Josephus, who wrote a history of Jerusalem
both prior to and after her fall, i.e., forty years after the death of Jesus, knew of Jesus but
nothing of this raising of many and appearing to many. Of this greatest of all miracles, not a
rumor appears in the works of Josephus or of any other ancient author. Surely at least one of
the many raised out of those many emptied tombs was still alive just prior to Josephus's time,
amazing many. Or at least many who had seen those many saints were still repeating the tale.
Although people may have doubted that Jesus raised a few people while he was still alive and
although "some doubted" Jesus' own resurrection (Matt. 28:17), who could fail to have been

Volume 1990 - 2002 Issue


Page 150 of 2049
Skeptical Review Edited by Farrell Till
impressed by many risen saints appearing to many? How also could Peter have neglected to
mention them in his Jerusalem speech a mere fifty days after they "appeared to many in the
holy city"? Surely their appearance must have been foremost on everyone's mind. So why
didn't Paul mention such a thing in his letters, our earliest sources? Why did the women who
visited the "empty tomb" on Sunday morning not take notice that many other tombs were
likewise open? Why didn't the visitors to Jesus' tomb mention that they had met or seen many
raised saints in that vicinity, meeting them on the way to Jesus' tomb or on the way back to
town? Why did the apostles disbelieve the first reports of Jesus' resurrection when a mass exit
from the tombs had accompanied his resurrection? Why didn't Matthew know how many
raised saints there were? Why couldn't he name a single one or a single person to whom they
had appeared? How did Matthew know that these saints had come out of their tombs? That
would be more than anyone had seen in the case of Jesus' resurrection.

Let's look at the implications of some of these questions. According to the literal Greek in
Matthew 27:50-53, the tombs were opened and the saints were "raised" at the instant of Jesus'
death, but they entered the city over a day later! Apparently, neither Joseph of Arimathea nor
Nicodemus, while burying Jesus (Jn. 19:38-40), chanced to marvel at all the opened graves
and the raised saints in them waiting patiently for Sunday morning. The women in Matthew's
account were likewise oblivious to the many graves lying opened by the earthquake and the
saints supposedly just beginning to leave the cemeery for town the same morning the women
were arriving. And the other gospels were silent on this major miracle involving many! Paul
was silent on this matter in 1 Corinthians 15, where he discussed the resurrection at great
length! Peter was silent on the matter in his speech recorded in Acts 2, delivered a mere 50
days after the many saints entered the city and appeared to many! Surely the "gift of tongues"
would pale in miraculous significance compared to the "raising of the many who appeared to
many." Yet Peter said nothing about the latter. We are not talking about just the apostles, like
Peter, being witnesses to just the resurrection of Jesus; we are talking about many people who
had witnessed many saints being raised, and some of these "many" witnesses were surely
present in the audience Peter preached to that morning. So why would he have had to speak at
length to convince them that the resurrection of one man had happened? Having witnessed the
resurrection of many, they would have readily accepted the claim that one man had been
resurrected.

And what about the raised saints themselves? Wouldn't they have made terrific evangelists?
But we don't read anything about that; instead, we have silence. We admit that to argue from
silence is not equivalent to disproof; however, it is not the silence of extrabiblical sources that
makes us doubt this account of multiple resurrections. It is the silence of other biblical authors
that is generating our doubt.

A few extrabiblical sources did expand Matthew's tale of the many raised saints. These
expansions were composed over one hundred years after Matthew's gospel was written.
Remarkably, they even mentioned the names of some of the "many saints" raised, like Simeon
and his sons, Adam and Eve, the patriarchs and prophets, etc., names that Matthew neglected
to include. Of course, these expansions of the two extraordinary verses in Matthew and the
list of names are found only in apocryphal gospels, which are full of all sorts of marvelous
miracles that even surpass the ones attributed to Jesus in the four gospels that the church now

Volume 1990 - 2002 Issue


Page 151 of 2049
Skeptical Review Edited by Farrell Till
endorses (like the story of the talking cross that followed Jesus out of his tomb in the Gospel
of Peter).

Perhaps Matthew, like the authors of the apocryphal gospels, collected tales he had heard
from other believers and/or composed gospel fictions. Perhaps when he composed those two
short verses, he was only giving mythical form to the belief that "the resuscitation of the
righteous was assigned to the first appearance of the Messiah, in accordance with the Jewish
ideas" (D. F. Strauss, The Life of Jesus Critically Examined). He was also indulging in
miracle enhancement: multiplying signs and wonders said to accompany Jesus' death and
resurrection, i.e., Matthew's unique account of two earthquakes, one that opened the tombs of
the many saints (at Jesus' death) and one that moved the stone to open Jesus' tomb (Easter
morning). The other gospel writers remarkably neglected to mention that even one earthquake
took place. That leaves Matthew's account on doubly shaky ground. Neither did Matthew use
the most precise words to depict this wonder, because the verses state, literally, that the saints
were raised at the time of Jesus' death and then lay around in their tombs for a day and a
halfbefore entering the city! That absurdity arises from what appears to be a sloppy
interpolation of the phrase "after his resurrection":

And Jesus cried again with a loud voice, and yielded up his spirit. And behold, the veil of the
temple was rent in two from the top to the bottom; and the earth did quake; and the rocks
were rent; and the tombs were opened; and many bodies of the saints that had fallen asleep
were raised: and coming forth out of the tombs after his resurrection they entered into the holy
city and appeared unto many (27:50-53).
The verses make more sense without that phrase than with it. Without it, they would simply
state that the raised saints immediately entered the city upon Jesus' death. But some Christian
copyist, or perhaps the gospel's chief editor, felt obligated to add the phrase "after his
resurrection" to ensure the priority of Jesus' resurrection, regardless of the literal
consequences.

People who believe that many tombs were opened and that many saints appeared to many will
of course have little trouble also believing that Jesus was resurrected. However, those of us
who doubt the story of the many raised saints see in it a reflection of the kind of blind faith
that made the story of Jesus' resurrection catch on in the first place.

(Ed Babinski's address is 109 Burwood Drive, Simpsonville, SC 29681-8768.)

Non-Profit Recognition
Skepticism, Inc., the official publisher of The Skeptical Review, has been recognized by the
Internal Revenue Service as a non-profit organization. This is important to us, because of the
substantial reduction in postage rates that we are now entitled to, but it is also important to
some of our readers. Although we do not make direct appeals for financial assistance, some of
you have included contributions with your orders and subscription renewals. We want you to

Volume 1990 - 2002 Issue


Page 152 of 2049
Skeptical Review Edited by Farrell Till
know that your help is appreciated, but we also want you to know that you can deduct your
contributions on your income tax returns.

Second Deaver-Till Debate Canceled


While arranging their debate in San Marcos, Texas, Mac Deaver and Farrell Till agreed to
conduct a second debate on the same subject (Bible morality) in Central Illinois. After
initiating correspondence to negotiate the time, place, and format of the second debate, Farrell
Till received a letter from Deaver, dated October 17,1991, in which he said, "I no longer have
any interest in debating you in the near future."

In the San Marcos debate, Till forced Deaver to say that every act of killing an Amalekite
baby (1 Sam. 15) was a morally good act. Perhaps he has decided that he does not want to be
put into that position again. At any rate, the second debate has apparently been unilaterally
canceled.

The San Marcos debate was recorded by Thomas Gardner, 2229 Lotus, Ft. Worth, TX 76111
(Tel. 817-838-0828). The last information we had both audio ($15) and video tapes ($30)
were still available. If any TSR readers doubt that belief in Bible inerrancy will force one to
defend the killing of babies, they should listen to these tapes.

Volume 1990 - 2002 Issue


Page 153 of 2049
Skeptical Review Edited by Farrell Till

The Skeptical Review


Volume Three, Number Two
April/May/June 1992
Farrell Till, editor

• There's a Living In It
Why can't fundamentalist preachers simply admit that the Bible is not inerrant?
Because "the Bible is their meal ticket."

• Impossible for God to Lie?


Inerrantists often quote the Bible and then add that "God cannot lie." However, Till
argues that "in many places, the Bible teaches that God not only can but does on
occassion lie."

• Impossible for God to Lie? An Answer


In response to Till, Michael Hughes writes "These passages do not prove that God is a
liar; rather they prove that man has a choice, that God in His wisdom has allowed that
choice regardless of the consequences."

• A Very Great City


Jonah 3:3 implies that Nineveh was 60 miles across. However, Dave Matson argues
that archaeological evidence shows it was not more than 3 miles across.

• The Resurrection Maze


If the four gospel authors were divinely inspired, writes Till, "there would be no maze
of inconsistencies in the juxtaposition of their stories."

• Computer Bulletins
Infomration on posting TSR articles on computer bulletins.

Volume 1990 - 2002 Issue


Page 154 of 2049
Skeptical Review Edited by Farrell Till

There's a Living In It
A subscriber in Florida recently raised an interesting question: "Why can't these preachers
simply admit that the Bible is not the inerrant book it has been proclaimed to be and stop
lying to their congregations?" The question was asked in the context of comments the reader
was making about the thoroughness of TSR's exposure of flaws in the inerrancy doctrine.

I have to admit that I have often wondered the same thing. Ten issues of The Skeptical
Review have now been published containing over 30 major articles and several short ones that
focused on discrepancies in the Bible text. In every issue, we have offered inerrancy
defenders the opportunity to rebut our lead articles, but twice we had to publish without
rebuttals because we could find no one willing to argue the inerrancy view on the subject we
were featuring. We have been especially persistent in challenging Wayne Jackson, the editor
of Christian Courier, to defend his inerrancy views, because he is especially vocal in his
articles about "the uncanny reliablility" of the Bible in even "the smallest details" ("The Bible
Passes the Test," Biblical Notes, Nov./Dec. 1991, p. 9). Jackson, like many of his inerrancy
colleagues, has repeatedly declined our offers of space to defend his claim that the Bible is
inerrant. We find it hard to understand why an inerrancy believer who writes as frequently on
the subject as Jackson does would refuse an offer of free publishing space to write on the
subject if he sincerely believes in what he preaches.

So why do vociferous inerrancy spokesmen like Wayne Jackson and Gleason Archer refuse to
defend publicly what they write books and publish papers to teach in one-sided formats? Our
reader in Florida probably has the answer. "I think that the Bible is simply their meal ticket,"
he went on to say.

For years, I have resisted this conclusion. As a former fundamentalist preacher who sincerely
believed that the Bible was the verbally inspired, inerrant word of God, I have wanted to
believe that those who profess belief in the inerrancy doctrine are just as sincere as I was, but
the more I write on the subject and debate it with fundamentalist preachers, the more difficult
it is to give them the benefit of doubt. Most will not even debate the issue, and those who will
do nothing but evade arguments and parrot worn-out, far-fetched how-it-could-have-been
scenarios that only the naively credulous could possibly believe.

So could it just be that our reader in Florida is right? The Bible is simply a meal ticket to
many of the fundamentalist preachers who proclaim it from their pulpits but won't defend it in
public debate with informed opponents.

If this is so, it would be entirely consistent with Darwinism, which, whether fundamentalists
like it or not, offers a far more likely explanation for the various life forms than does the
Genesis story of creation. In nature, if there is a living to be made in some way, no matter
how unusual it might be, there will always be some creature that will exploit it. This, rather
than creation, probably accounts for the millions of cases of special adaptation that we see in
nature. There was a living to be made in this way selection have evolved the various species
that are "good" at what they do to make a living. Many species are so good at what they do
that some people see them as marvelous evidence of creation, but it isn't that at all. There was

Volume 1990 - 2002 Issue


Page 155 of 2049
Skeptical Review Edited by Farrell Till
a living to be made in all these ways, so the creatures that successfully exploited them
survived and became specialized. Those that didn't didn't.

My point is not to defend the theory of evolution, but to give our fundamentalist readers
something to think about. As long as there is a living to be made at something, there will
always be enterprising life forms that will exploit it. Is it possible that this fact of nature is at
work in the religious affairs of men? One would have to deny the obvious to say that it isn't.
We have seen too many Jim Bakkers and Jimmy Swaggarts exploiting the gullibility of
people to deny that some preachers are preaching only because they have found it an easy
way to make a good living.

"Oh, but that is Jim Bakker and Jimmy Swaggart," some will say. "My preacher would never
do anything that dishonest." Well, don't be too sure of it. If the public suddenly stopped
believing in Bible inerrancy, what would that do to the thousands of fundamentalist preachers
in the land? It would put them all out of work, so don't tell me that they don't have a vested
interest in the outcome of the inerrancy controversy. They have a lot of pressure on them to
say that they believe in Bible inerrancy even if they don't.

When I debate the inerrancy doctrine with fundamentalist preachers, I have no economic
pressure at all on me. If one of them should prove beyond question, so that no rational person
in the world could deny it, that the Bible is the verbally inspired, inerrant word of God, all I
would do is acknowledge that I was wrong (which wouldn't be the first time I've admitted I
was wrong), return home, go back to my job, and draw my salary as an English teacher. I
would have to disband Skepticism, Inc., of course, but that would cause me no economic loss.
It is a nonprofit organization with a decided emphasis on the nonprofit.

On the other hand, if I should prove beyond question, so that no rational person in the world
could deny it, that the Bible is not the verbally inspired, inerrant word of God, what would
that do to my opponents? They couldn't return to their jobs, because there would be no one for
them to preach to, no one to put money into the collection baskets to keep their little empires
going. Their livelihood would be gone with the public's loss of faith in Bible inerrancy.

If you think that they don't know that, you must be very naive.

Impossible for God to Lie?


Farrell Till
"God cannot lie. For this reason alone, we know that whatever he personally has
communicated to mankind is true." This is how Editor Tom Fishbeck introduced a pro-
inerrancy comment in the April 1991 issue of Bible Answers Newsletters (p. 3). It was a
familiar bibliolatry tactic. A Bible passage is quoted and then supported with the claim that

Volume 1990 - 2002 Issue


Page 156 of 2049
Skeptical Review Edited by Farrell Till
God cannot lie. The claim is often "proven" by citing Hebrews 6:18, which says that it is
impossible for God to lie, or Titus 1:2, which says that God can not lie.

Aside from the obvious circular reasoning taking place here, the tactic is flawed by
contradictory statements in the Bible about a much ballyhooed aspect of God's nature. If it is
truly impossible for God to lie, then bibliolaters are going to have to explain some confusing
Bible passages.

One of these is the fanciful little yarn attributed to the prophet Micaiah in 1 Kings 22:19-23.
Ahab and Jehoshaphat were considering an Israelite-Judean joint expedition to go against the
Syrians at Ramoth-gilead. Apparently leery of the plan, Jehoshaphat asked for "the word of
Yahweh" (v:5) on the matter, and Ahab paraded before him 400 prophets who all assured
Jehoshaphat that the alliance would succeed. Still unconvinced, Jehoshaphat wanted to
consult at least one more prophet. Why the word of one more prophet would mean anything if
Jehoshaphat was unconvinced by what the 400 were saying is anyone's guess. At any rate,
Jehoshaphat insisted on consulting at least one more prophet, so Ahab reluctantly sent for
Micaiah, the son of Imlah. "There is yet one man by whom we may inquire of Yahweh,"
Ahab told Jehoshaphat, "but I hate him, for he does not prophesy good concerning me but
evil" (v:8).

When Micaiah was brought before the kings, the 400 prophets were putting on quite a
spectacle. "Go up to Ramoth-gilead and prosper," they were saying, "for Yahweh will deliver
it into the hand of the king." Asked by Ahab if their armies should go up to Ramoth-gilead or
forebear, Micaiah said, "Go up and prosper, and Yahweh will deliver it into the hand of the
king" (v:15). Apparently, this was spoken sarcastically, because Ahab then said, "How many
times shall I command you that you speak to me nothing but the truth in the name of
Yahweh?" In response to this, Micaiah told a story that implicated Yahweh in a conspiracy to
lure Ahab to his death:

I saw Yahweh sitting on his throne, and all the host of heaven standing by him on his right
hand and on his left. And Yahweh said, Who shall entice Ahab, that he may go up and fall at
Ramoth-gilead? And one said on this manner; and another said on that manner. And there
came forth a spirit, and stood before Yahweh, and said, I will entice him. And Yahweh said to
him, With what? And he said, I will go forth, and will be a lying spirit in the mouth of all his
prophets. And he said, You shall entice him, and shall prevail also: go forth, and do so. Now
therefore, behold, Yahweh has put a lying spirit in the mouth of all these your prophets; and
Yahweh has spoken evil concerning you (Bethel Bible).
Yahweh apparently had a penchant for putting lying spirits into people, because a less
detailed, but similar, incident is related in Isaiah 37:7 and 2 Kings 19:7. (For reasons known
only to Yahweh and Bible inerrantists, the wording of these two chapters, Isaiah 37 and 2
Kings 19, is identical.) Threatened by a message from king Sennacherib of Assyria whose
army had laid siege to Jerusalem and other Judean cities, king Hezekiah rent his clothes, put
on sackcloth, and went into the house of Yahweh. The prophet Isaiah sent word from Yahweh
for the king not to be afraid. "Behold, I will put a spirit in him (Sennacherib)," said the
message from Yahweh, "and he shall hear tidings, and shall return to his own land; and I will
cause him to fall by the sword in his own land."

Volume 1990 - 2002 Issue


Page 157 of 2049
Skeptical Review Edited by Farrell Till
The chapters end with an account of Sennacherib's assassination by his own sons, but that is
only incidental to the story. The important thing is that Isaiah, as did Micaiah, depicted
Yahweh as a god who dealt with troublesome men by putting lying spirits into them to
deceive them and lure them to their deaths. How can these two stories be reconciled with the
claim that it is impossible for God to lie?

I asked that question in my debate with Mac Deaver in San Marcos, Texas, and his response
was that God did not lie to Ahab; Satan did. His proof was John 8:44 where it was said that
there is no truth in the devil and that "when he speaketh a lie, he speaketh of his own, for he is
a liar, and the father thereof." As I have pointed out in many prior articles, the pitting of
scripture against scripture is a logically unsound way to protect the inerrancy doctrine,
because it is a tactic that seeks to prove inerrancy by assuming inerrancy. "Your passage can't
mean what you are saying it means," the argument implies, "because a passage over here very
clearly teaches thus-and-so." The inerrantist who reasons like this is saying, "If your passage
means what you claim it means and if mine means what I claim it means, then there is a
contradiction in the Bible, and that can't be, because the Bible does not contradict itself." As I
said, this is circular reasoning or, in this case, proving inerrancy by assuming inerrancy.

Why does it never occur to inerrancy defenders that if one writer made statement "A" and
another writer made statement "B," which appears to contradict "A," then there just might be
a real contradiction in the Bible? If both statements had been made by the same writer, the
inerrantist might then have a sensible basis for suggesting how-it-could-have-been scenarios
to reconcile the meaning of the two, but when the statements have been made by different
writers, the most likely explanation for the apparent discrepancy is that the two writers
disagreed on the common subject they were writing about. Obviously, the writers of Hebrews
(6:18) and Titus (1:2) thought that God could not lie. This fact, however, does not exclude the
possibility that Isaiah (37:7) and the writer of 1 Kings (22:19-23) believed that God did
sometimes lie. So to resolve this problem, bibliolaters must explicate the story of the prophet
Micaiah and the message of Isaiah to king Hezekiah in a way that will show that no lie or
deception on God's part was involved in either case. I don't think they can do that, but until
they do the Bible stands indicted for contradiction in the matter of whether God can lie.

Another problem passage concerns the intertribal dispute between Israel and the Benjamites.
Outraged at the rape and murder of a Levite's concubine at Gibeah by a group of Benjamite
homosexuals, the other Israelites demanded that the tribe of Benjamin deliver up to them the
"base fellows" who had done this thing so that they could be put to death (Judges 20:12-13).
When the Benjamite leaders refused the demand, the Israelites took an army of 400,000
against the Benjamites, who numbered only 26,700. It looked as if it were going to be a
complete rout, so the Israelites, apparently seeing no need to send their entire army out to
battle, went up to Bethel to ask "counsel of God":

Who shall go up for us first to battle against the children of Benjamin? And Yahweh said,
Judah shall go up first (20:18).
Well, Judah did go up first, and lost 22,000 men in a resounding defeat! So what happened
here? The Israelites had asked counsel of Yahweh, and he told them to send Judah out to
battle first. Although inerrantists may quibble (as I have heard them do) that Yahweh did not
specifically say that Judah would be victorious, if the story happened as recorded--and

Volume 1990 - 2002 Issue


Page 158 of 2049
Skeptical Review Edited by Farrell Till
inerrantists will argue that it did--then deception was certainly involved. One would have to
be completely idiotic to think that the Israelites had asked "counsel of Yahweh" to find out
which army to deploy in order to be defeated. Obviously, they wanted to know what army
would secure a victory for them. So if anything like what is related in this story ever
happened, we can conclude only one of two things: (1) Yahweh deceived the Israelites into
thinking the forces of Judah could win the battle or (2) Yahweh is not omniscient. Either way
the inerrancy doctrine suffers irreparable damage.

But this story didn't end with the defeat of the Judean army. In great distress, "the children of
Israel went up and wept before Yahweh until evening; and they asked of Yahweh, saying,
Shall I again draw near to battle against the children of Benjamin my brother?" And what
answer did they receive? "And Yahweh said, Go up against him" (v:23). So on this "counsel"
from Yahweh, the Israelites went to battle the next day, and this time the Benjamites
"destroyed down to the ground of the children of Israel again eighteen thousand men" (vv:24-
25).

To believe that this ridiculous tale is part of the verbally inspired, inerrant word of God is too
absurd to deserve comment, but to argue that if it did happen as recorded no deception was
involved on Yahweh's part would be even more absurd. In profound anguish, the Israelites
had asked their god Yahweh if they should again go to battle against the Benjamites, and he
told them to go. If that was not deception, then someone should explain why it wasn't.

Usually, when the Israelites experienced military defeat, pestilence, famine, or other
calamities, the Bible attributed it to some sin or disobedience. When Joshua's army was routed
at Ai, for example, it turned out that Yahweh was punishing his people for the sin of one man
who had kept some of the spoils for himself after the battle of Jericho (Josh. 7). David sinned
in numbering Israel (2 Sam. 24:1-10), but Yahweh punished all of Israel for it by sending a
pestilence that killed 70,000 people (vv:15-16). As unjust as it is to punish someone for the
"sins" of another, that was clearly the practice in Old Testament times, and bibliolaters
dutifully defend it as justification for Yahweh's having on occasion retracted his promises.
There is nothing in Judges 20, however, that even suggests the Israelites were guilty of some
offense that would have "justified" Yahweh's retraction of his implied promise of victory. To
the contrary, the Benjamites were the offenders. They were harboring a group of men who
had committed a despicable crime. Yet the Israelites were losing all the battles--and that after
they had asked "counsel of Yahweh" and had been told to go against Benjamin! It had to be
either deception or a pathetic lack of foreknowledge on Yahweh's part.

After their second defeat, the Israelites went up to Bethel again "and wept, and sat there
before Yahweh, and fasted that day until evening; and they offered burnt-offerings and peace-
offerings before Yahweh" (v:26). None other than Phinehas, the son of Eleazar, the son of
Aaron, was the priest who stood before the ark of the covenant while all of this counsel-
seeking was going on, and for the third time the Israelites asked Yahweh, "Shall I yet again go
out to battle against the children of Benjamin my brother, or shall I cease?" The answer? "And
Yahweh said, Go up; for tomorrow I will deliver him into your hand" (v:28). They went to
battle the next day, and, by luring the Benjamites into an ambush, finally defeated them, if
suffering 40,000 casualties in order to kill 25,000 Benjamites could in any sense be
considered a victory. Maybe it was the fasting and offering of sacrifices before the third battle

Volume 1990 - 2002 Issue


Page 159 of 2049
Skeptical Review Edited by Farrell Till
that finally brought victory to the Israelites, or maybe it was just that the third time was
charmed. At any rate, the Israelites finally won, according to the story, but at the cost of
considerable damage to Yahweh's reputation for honesty.

An inerrantist once told me (with a straight face) that Yahweh did not specifically say until
the third inquiry was made (v:28) that he would deliver the Benjamites into the hands of
Israel. So to his warped way of reasoning, there was no deception in the answers that Yahweh
gave to the first two inquiries of the Israelites. He had just told them to go to battle without
indicating either way how the battles would go. Can you imagine an inerrancy defense any
lamer than that? If I absolutely knew that John Jones had infallible ability to look into the
future and see what was going to happen and, knowing that, I asked him if I should buy stock
in company A, would Jones be guilty of deception and lying if he said, "Yes, buy it," and
then, after I had bought the stock, the company went bankrupt? To ask the question is to
answer it.

There are other stories in the Bible that are inconsistent with the claim that God cannot lie. In
"Yahweh's Failed Land Promise" (Winter 1991, pp. 2-6), for example, I examined the
numerous OT passages in which Yahweh had unconditionally promised that he would without
fail give all of the land of Canaan to the Israelites but then failed to make good his promise. In
the same issue, a fundamentalist writer tried to rebut the central premise of the article and
couldn't. In a future issue, we will notice that Yahweh once promised (2 Sam. 7:8-17) that he
would establish the throne of David forever and then failed to make good that promise. All
such stories as these are devastating to the claim that God cannot lie. In many places, the
Bible teaches that God not only can but does on occasion lie. This is just one more of many
contradictions in the verbally inspired "word of God."

Impossible for God to Lie? An Answer


Michael P. Hughes
Quite often those who wish to discredit the Bible form an idea, then do their very best to find
"proof" to support that idea. This is what Mr. Till has done many times before, and it is what
he has done in his article "Impossible For God to Lie?"

Mr. Till has decided that the Bible cannot be true. Since it claims to be true, he must find
something that "proves" that it is not true.

In this supposed contradiction, though, I don't believe Mr. Till has made an honest attempt to
explain the verses in question at all. He has simply quoted passages and said, "See, God lies."
This, of course, requires no amount of study whatsoever. I must say that this doesn't seem to
be a method of study that would be used by a "critical thinker," yet here we have it. Farrell,
just a question. What if scientists approached their studies in the same manner that you

Volume 1990 - 2002 Issue


Page 160 of 2049
Skeptical Review Edited by Farrell Till
approached your "study" of the Bible? We would probably still be flying kites to discover
(use?) electricity!

Between his diatribes about circular reasoning and using the Bible to support the Bible, (by
the way is it also wrong to use science to support science), Mr. Till brought into question
three biblical passages that he maintains prove that God lies, therefore proving that the Bible
is not inerrant.

The first of these that I wish to deal with is the account in Judges 20. He is particularly
concerned about two verses, 18 and 23, in which Israel asked counsel of God. God first told
Judah to go against Benjamin, and they were defeated. They then asked God if they should
persist in their actions. God told them to continue, and again they were severely defeated.
This, we are told, is proof that God lies.

Mr. Till is not happy with the idea that God did not say that they would be successful. He uses
the "what if" scenario of someone who can see into the future giving bad advice about stocks.
I hate hypothetical "what if" scenarios like this. They are usually not very relevant, and this
one is no different. Since no one has the ability to see infallibly into the future except God, the
example is totally invalid!

Let me ask one question though. Suppose that Mr. Jones knew that I was going to do
whatever I wanted regardless of his "advice," and his response to me was in a resigned
manner. Would he then still be guilty of lying, if indeed he was to begin with?

This certainly seems to be the case in the account under question. I call your attention to
Judges 21:25, "In those days there was no king in Israel: every man did that which was right
in his own eyes." This was not the first time that the writer of the book of Judges made this
observation. He stated the very same thing in Judges 17:6 and 18:1. Actually, this seemed to
be the attitude of the general populace of Israel all through the book of Judges.

That same attitude seemed to be quite prevalent in this account also. Note the attitude
exhibited in verse nine, "(B)ut now this shall be the thing which we will do to Gibeah; we will
go up by lot against it." One can see the obvious attitude here. There is no counsel with God,
no question of whether they were right or not; they just decided to act.

They did finally seek counsel with God, but not until the eve of the battle. Even then it was
not truly to see what God wanted, but simply to ask Him which tribe should go first. This also
showed their attitude. They needed only one tribe for this little affair, so what did they need
God for except as arbitrator as to who should have the privilege of being the disciplinarians
here?

With this type of attitude it really would not have mattered what God said, since, as so often
happens, Israel would not have listened had they not liked the answer. What better way to
teach them a lesson than to let them go their own way.

This is not an implausible answer to explain the occurrence here and allow the inerrancy of
the Bible to stand. Of course, one can be like those Israelites and believe (or do) that which is

Volume 1990 - 2002 Issue


Page 161 of 2049
Skeptical Review Edited by Farrell Till
right in their own eyes, in which case the only plausible explanation will be that which they
chose to believe, in spite of anything that sincere study might suggest!

The other two accounts that Mr. Till presented as evidence of his allegations concerned a
lying spirit. In these two passages, 1 Kings 22:19-23 and 2 Kings 19:6-7, something was
allowed by God; therefore, says Mr. Till, He is guilty of sin.

We, of course, recognize the significance of these accusations. If God has lied, then (1) He
can't be God, for God can't sin, and (2) if He lied, then the Bible is not inerrant, for it says that
God cannot lie (Titus 1:2; Heb. 6:18).

Mr. Till has already been given one explanation of these two passages, which he did not like.
That explanation is basically that God allowed something to happen and is therefore
accredited with the attribute of having done that thing.

A good example of this is the account of Israel's exodus from their captivity in Egypt. Several
times throughout this account, we are told that God either intended to or actually hardened
Pharaoh's heart.Yet no one who seriously studies the Bible with a desire to know God's truth
would venture forth with the idea that Pharaoh's heart was literally hardened by God so that
he would not do as God wished. It is understood that God allowed Pharaoh to exercise his will
and thus ignored God's wishes.

In 2 Thessalonians 2:10-11, we read, "And with all deceivableness of unrighteousness in them


that perish; because they received not the love of the truth, that they might be saved. And for
this cause God shall send them strong delusion, that they should believe a lie." In this passage,
we read that God will send strong delusions so that people will believe a lie, yet in James we
are told, "Let no man say when he is tempted, I am tempted of God: for God cannot be
tempted with evil, neither tempteth he any man: but every man is tempted, when he is drawn
away of his own lust and enticed" (James 1:13-14).

Now how does one reconcile these two passages and the ones of the Old Testament so that all
is in agreement? Naturally, Mr. Till would say that it cannot be done. I disagree.

God allows things to happen. He allows men to choose which path they will follow. He
allows men to be deceived by others. He allows Satan to have a foothold in this world at this
present time and in the past.

Since God is omnipotent, it is often said that he does something when in fact he has simply
allowed man to exercise choice.

What we read in 1 Kings 22 and 2 Kings 19 is nothing more than that. These passages do not
prove that God is a liar; rather they prove that man has a choice, that God in His wisdom has
allowed that choice regardless of the consequences.

Farrell, you also have made a choice. God allowed that choice, even though it pains him.
Many of us pray that you will repent of that choice and return to the truth that you once
embraced.

Volume 1990 - 2002 Issue


Page 162 of 2049
Skeptical Review Edited by Farrell Till
(Michael Hughes' address is Route 3, Box 924, Camdenton, MO 65020- 9803.)

EDITOR'S NOTE: Mr. Hughes' accused me of first forming an idea about the Bible and then
looking for proof to support that idea. I must remind him that I was once a preacher and
missionary for the same church that he is a member of. At that time, I had an idea about the
Bible, which was the same as the one he presently has. I believed that the Bible was the
inspired, inerrant word of God. Despite what he and his colleagues may accuse me of, I did
not suddenly decide one day to see if I could find proof of errancy in the Bible. In my case,
the Bible condemned itself. I studied it intently. The discrepancies were there for me to see,
and I saw them, to such a degree that my conscience would not permit me to continue
preaching something I knew wasn't true.

Can Mr. Hughes truthfully say that he has examined the inerrancy issue from both sides, as I
have done? I seriously doubt that he can. How then can he accuse me of first forming an idea
and then looking for proof to support it? If anybody has done that, he has. Does he expect us
to believe that his article was objectively researched? Did he not approach the task of writing
that article with a view to finding something--just anything halfway sensible--that he could
say to keep his precious inerrancy doctrine intact? Of course, he did. He knows it, I know it,
and the readers know it too. So until he can present proof that he has objectively and
impartially studied the inerrancy issue, he has no room to accuse me or anyone of
unobjectively trying to prove preconceived ideas and opinions. If scientists approached their
studies in the same manner that I do, he asserted, we would probably still be flying kites to
discover electricity, but I fear that we would still be making stone tools, if scientists had used
the same approach that Mr. Hughes and his inerrancy colleagues apply to biblical
interpretation.

Hughes claimed that the key to understanding the story in Judges 20 that seems to present
Yahweh in a bad light is as simple as understanding the manner in which Yahweh was
speaking when the Israelites asked for his advice. He was speaking with resignation to people
who had already made up their minds and would probably have carried out their plan
regardless of what Yahweh might have said to them. So, according to Hughes' scenario,
Yahweh was simply saying, "Yes, yes, go ahead and go against Benjamin; you're determined
to do it anyway, so just do it."

The problem with Hughes' solution is that it depends upon his having properly interpreted the
tone in which a written statement was said. I have taught college literature courses for over
twenty years, and, as any experienced teacher of literature knows, I know that correct
interpretation of tone is difficult when the writer has not expressly stated that the tone was
sincerity, sarcasm, resignation, etc. Mr. Hughes hates hypothetical scenarios, but his
interpretation of this biblical scene is about as hypothetical as any could be. It depends
entirely upon an arbitrary assertion that the tone of Yahweh's statements was resignation.

The text itself gives no support to this interpretation, because it indicates that the Israelites
had not yet made a definite decision the first time they asked counsel of Yahweh:

Volume 1990 - 2002 Issue


Page 163 of 2049
Skeptical Review Edited by Farrell Till
And the children of Israel arose, and went up to Bethel, and asked counsel of God; and they
said, Who shall go up for us first to battle against the children of Benjamin? And Yahweh
said, Judah shall go up first (V:18, ASV with Yahweh substituted for Jehovah).
Notice that they did not ask, "Shall we go up against Benjamin?" In other words, they were
not asking for divine approval of what they had already decided. They were asking Yahweh to
designate or choose a specific tribe to make the attack. In this respect, they were doing exactly
what was customarily done in Israelite society when a course of action was under
consideration. They asked for advice from their tribal war god, and the advice that he gave
them resulted in the death of 22,000 men.

Hughes sees support for this interpretation in the fact that the book of Judges says three times
that "every man did that which was right in his own eyes" in those days, but the context of
this story hardly supports his theory. If these were people determined to do what was right in
their own eyes, why would they have sought the counsel of Yahweh? The fact that they did,
not just once but three times, indicates that they were typically superstitious Israelites who
believed that the advice of Yahweh should be sought in every important aspect of life, and
especially in a matter as important as a battle plan. (Examples of seeking the counsel of
Yahweh can be found in 1 Samuel 10:22; 22:10,13; 30:8; 2 Samuel 2:1; 5:19,23 and other
passages too numerous to list.) If Phinehas, the priest through whom the counsel of Yahweh
was being channeled in Judges 20 (v:28), had told the people that Yahweh had said not to go
up to battle, they would have superstitiously acquiesced. They in fact indicated a willingness
to do so on their third visit to Bethel: "Shall I yet again go out to battle against the children of
Benjamin my brother, or shall I cease" (v: 28)? If what they were doing was wrong, why
didn't God just tell them to cease? Furthermore, if their conduct was so terribly wrong, why
did God eventually allow them to succeed? These are questions Mr. Hughes' interpretation
fails to answer.

He applied the same principle to the fanciful little yarn about the "lying spirit." God didn't
deceive Ahab and Jehoshaphat; he simply allowed them to fight a battle they had already
made up their minds to fight. Well, his theory won't hold up here either. Jehoshaphat was so
undecided about what to do that he had rejected the advice of 400 prophets. He insisted on
consulting at least one more. That hardly sounds like the conduct of a man whose mind was
already set. Furthermore, the prophet Micaiah plainly said in concluding his story that
"Yahweh has put a lying spirit in the mouth of all these prophets of yours" (1 Kings 22:23). In
the incident involving the king of Assyria, the message of Yahweh clearly said, "I [Yahweh]
will put a spirit in him, and he shall hear tidings, and shall return to his own land; and I
[Yahweh] will cause him to fall by the sword in his own land" (2 Kings 19:7). To argue that
these were simply matters of God letting people do what they had already made up their
minds to do is ridiculous. The obvious intent of both these statements was to convey the
impression that God had intervened in the affairs of these men to lure them by deceptive
means to their deaths.

The last biblical example that Hughes should cite to shore up his theory is the one about the
hardening of Pharaoh's heart. If he thinks that no serious student of the Bible would "venture
forth with the idea that Pharaoh's heart was literally hardened by God so that he would not do
as God wished," then he should read the story again:

Volume 1990 - 2002 Issue


Page 164 of 2049
Skeptical Review Edited by Farrell Till
Then Yahweh said to Moses, "Go to Pharaoh, for it is I who have made his heart and his
courtiers stubborn, so that I could work these signs of mine among them; so that you can tell
your sons and your grandsons how I made fools of the Egyptians and what signs I performed
among them, to let you know that I am Yahweh" (Ex. 10:1, Jerusalem Bible).
If I can understand simple language, this passage is saying that Yahweh himself hardened
Pharaoh's heart to give Yahweh the opportunity to show his stuff and leave the Israelites with
some fireside tales to tell their children and grandchildren.

Hughes resorted to the old fundamentalist ploy of pitting scripture against scripture. "Your
passage can't mean what you say it means, because over here in another place it says thus and
so." We have repeatedly pointed out that this tactic is an attempt to prove inerrancy by
assuming inerrancy. It proves nothing except that we are right when we say that the Bible
contradicts itself.

A Very Great City


Dave Matson
Jonah 3:3 informs us that the old Assyrian capital, Nineveh, was an exceedingly great city,
being three days' journey across! 'Twas a really, really, really big city!

Assyria was the first world-class power to really collide with Israel, and the size of its fabled
capital may have slipped out of focus. At any rate, the Bible gave it the diameter of a three-
day journey. Scholars, using old traveler's accounts, usually reckon a day's journey in ancient
times as 20 miles or thereabouts. The fact that Jonah was probably on foot is irrelevant since
Jonah is not the standard of measurement. For an ancient walker, a day's journey in a well
paved city would have been a picnic.

That gives us a city 60 miles in diameter! Holy flying swordfish! That's bigger than Los
Angeles.

Archaeologists have actually dug up portions of Nineveh, and their findings tell a far different
story. It turns out that Nineveh scarcely exceeded three miles at its greatest stretch. It
extended over some 1,850 acres with a circumference of about eight miles.

How do Bible believers reconcile these facts?

THE DENIAL THEORY


No intelligent writer could make such a dumb mistake, inerrancy defenders say, so the Bible
must have had something else in mind. Unfortunately, Jonah wasn't the only one to assign a
large dimension to Nineveh. The Anchor Bible has this to say:

Volume 1990 - 2002 Issue


Page 165 of 2049
Skeptical Review Edited by Farrell Till
Brewer [J. A. Brewer, Jonah, 1912] gives ample citations from classical sources, most of
which accentuate its inordinate size (Jonah, p. 230).
Ancient embellishments are not that rare. Aristotle himself is said to have speculated that
Babylon had been taken for three days before all of its citizens became aware of the fact.
Herodotus, the famous historian of the ancient world, held that Babylon was in the shape of a
square whose sides were 14 miles long (cited by Josh McDowell, Evidence That Demands a
Verdict, Vol. One, p 304). Thus, Jonah's embarrassing account of Nineveh cannot be
dismissed on the grounds that no one could have taken it literally. Many Bible scholars
believe that Jonah was written several hundred years after the fall of Nineveh, thus allowing
ample time for the growth of legends.

THE PROVINCE "THEORIES"


A Few biblicists claim that in the case of Nineveh's size the Bible is really talking about a
region or a province instead of a city. What do the professionals say?

The translators of The New English Bible, The New Oxford Annotated Bible, Today's
English Version, The New Jerusalem Bible, and other top rated Bibles say "city." That should
settle it.

The Hebrew word here rendered as city (Œyr) was also applied to Jeruslem, Jericho, and
virtually every other city mentioned in the Bible! Thus, if we are to take the Bible at its word,
we are talking about a city. We are not talking about some province that had the same name.

Nineveh is also mentioned in Genesis, 2 Kings, Isaiah, Nahum, Zephaniah, and Matthew.
None of these passages referred to a region. In fact, most of them unmistakably referred to a
proper city. Indeed, Strong's Exhaustive Concordance of the Bible gives only one meaning for
Nineveh: the name of the capital city of ancient Assyria.

Nor is the Bible talking about adjoining suburbs. Ancient Mideastern cities, even the
important ones, were shockingly small by modern standards. The larger ones surrounded
themselves with walls to guard against bands of robbers, passing armies, or what have you.
Those walls marked the limits of the city. There were no suburbs.

Nor can we count the farmland around Nineveh as a kind of "suburbs." Farmlands attached to
Nineveh would certainly lie closer than 30 miles! The poor farmer was not going to spend a
whole day walking to his field! If the Ninevites had needed that much grain, they would have
imported it from nearby regions.

Nor do we have any evidence that Jonah was using the capital of Assyria as a symbol for an
entire nation, as was sometimes done with Jerusalem or Babylon in the Bible. A city spanning
60 miles is, indeed, exceedingly great--but not so a 60-mile-wide nation. Furthermore, Jonah
later removed himself a short distance from the city, built a little hut for himself, and waited
to see what would happen to Nineveh. So Jonah was obviously referring to a city, something
he could keep an eye on while sitting a short distance away. We don't read about Jonah
traveling from town to town, as the disciples of Jesus did, to prophesy to an entire region.

Volume 1990 - 2002 Issue


Page 166 of 2049
Skeptical Review Edited by Farrell Till
Perhaps our would-be revisionists will claim that Nineveh was surrounded by many small
towns and cities that, being closely tied to Nineveh, made up "Greater Nineveh." If that is the
claim, then let us have the evidence. Surely, wild speculation is no grounds for overthrowing
the face value of scripture!

Let our would-be revisionists begin by producing a map showing the vicinity of ancient
Nineveh. Let them show that towns and villages formed a well defined cluster about Nineveh,
one 60 miles in diameter. Let them show that the Hebrews referred to such an arrangement as
"the city" (Jonah 4:5).

Next, our would-be revisionists must turn to the Bible and show us why we can't accept Jonah
3:3 at its face value. Is it because a certain number is a wee bit inconvenient?

Whatever happened to biblical inerrancy? Perhaps biblical inerrancy is just a matter of


reinterpreting the Bible when it gets a little wild! Unless the above steps are taken, however,
our would-be revisionists are just whistling in the dark. Speculation must not be confused
with facts, and facts are needed if we wish to override the face value of a passage.

MISCELLANEOUS "THEORIES"
Another variation of this last defense claims that the passage referred to the circumference of
the city, but a 19-mile-wide city, which would result from this "explanation," is hardly an
improvement.

Another approach views Jonah as walking "into" the city in a roundabout way. Perhaps he
spent his day checking out the bazaars!

Surely, this is not what the ancients meant by "a day's journey," an expression that seems self-
explanatory. That would be a very confusing way to describe the size of a city, and the ruse is
clearly shot down in many of the better translations of Jonah 3:3-4:

Jonah obeyed at once and went to Nineveh. He began by going a day's journey into the city, a
vast city, three days' journey across... (The New English Bible).

So Jonah arose and went to Nin'eveh, according to the word of the LORD. Now Nin'eveh was
an exceedingly great city, three days' journey in breadth. Jonah began to go into the city,
going a day's journey (The New Oxford Annotated Bible).

Jonah set out and went to Nineveh in obedience to the word of Yahweh. Now Nineveh was a
city great beyond compare; to cross it took three days. Jonah began by going a day's journey
into the city and then proclaimed, "Only forty days more and Nineveh will be overthrown"
(The New Jerusalem Bible).

There is one Bible (that I am aware of) that actually supports a variation of this latest defense,
and that is the NIV (New International Version, 1978). We find in it that "Nineveh was a very

Volume 1990 - 2002 Issue


Page 167 of 2049
Skeptical Review Edited by Farrell Till
important city--a visit required three days." Since the NIV is likely to become a standard
among conservatives, we need to examine its treatment of Jonah 3:3.

Dr. Edward P. Blair (The Illustrated Bible Handbook, 1987) informs us that all the translators
of the NIV were expected to subscribe to the "high value of Scripture" set forth in the
Westminster Confession of Faith, the Belgic Confession, and the Statement of Faith of the
National Association of Evangelicals. When translators are hobbled with doctrine, doctrine
will hobble the translation. Indeed, Dr. Blair gives us 12 examples in the NIV where
translation has been so affected. Other examples, such as Isaiah 7:14 and Genesis 2:19, may
be added to that list. Even an honest conservative, such as David O'Brien (Today's Handbook
for Solving Bible Difficulties, 1990) will admit that doctrine has intruded in places in the
NIV.

Take a few moments and look at the modern Bible translations in a large library, if you will.
You will find that the NIV is the odd man out in its handling of Jonah 3:3. Its rendering of
this verse appears to be based on Stuart's translation, a rendition that has little scholastic
support:

There is no merit to Stuart's translation, "requiring a three-day visit," which depends on


Wiseman's incongruous weaving of Assyrian evidence regarding diplomatic visits to royal
cities.... Jonah is hardly sent to Nineveh to negotiate treaties or the like (The Anchor Bible,
Jonah, 1990, p. 230).
Thus, this rendering of Jonah 3:3 may be dismissed. It is one of those places in the NIV where
doctrine has overridden good translation.

NON-LITERAL "THEORIES"
At this point, biblicists may wish to join hands with the liberals and deny the historicity of
Jonah. For once, the biblicists would be right--but at great cost to the fundamentalist
viewpoint! If the straightforward narrative of Jonah can be dismissed, then what about Noah's
flood? What about Jesus's statement in Matthew 12:40-41? Where would the avalanche end?

If the Bible is authoritative in all matters, as fundamentalists claim, then its passages must be
taken at face value. (Of course, if a passage clearly fits another genre, such as poetry or
allegory, then it must be interpreted accordingly.) Thus, if the Bible appears to be making a
factual statement, then the biblicist must accept it as such. To do otherwise is to strip the
Bible of its presumed authority and to acknowledge reason as the higher standard.

That's the last thing the biblicists want to do! Their Bible is supposedly above man-made
standards.

We thus arrive at an important point. If a biblicist advances a claim--such as hyperbolic


usage--that denies the face value of a passage, then the burden of proof is on his shoulders.
Furthermore, he may not invoke such a defense merely because the passage is doctrinally
embarrassing. To do so, once again, is to undercut the very pillars of biblical authority. If the

Volume 1990 - 2002 Issue


Page 168 of 2049
Skeptical Review Edited by Farrell Till
Bible doesn't mean what it says, then what does it mean? Who decides? Biblical authority is
reduced to a matter of human consensus.

The "hyperbolic" and "allegorical" defenses are attractive to the biblicist precisely because
they cover a multitude of sins. They fill in those odd gaps not amenable to normal repair. Let's
see if either applies to Jonah 3:3.

Is the evidence overwhelming? On that point alone rests every claim of hyperbolic or
allegorical usage.

Archer's Encyclopedia of Bible Difficulties affirms the historicity of Jonah, as do apologetic


works in general. If Matthew 12:39-41 is accepted at face value, then Jesus himself supported
the same:

An evil and adulterous generation seeketh after a sign; and there shall no sign be given to it
but the sign of Jonah the prophet: for as Jonah was three days and three nights in the belly of
the whale;so shall the Son of man be three days and three nights in the heart of the earth. The
men of Nineveh shall stand up in the judgment with this generation, and shall condemn it: for
they repented at the preaching of Jonah; and behold a greater than Jonah is here.
By reason of what Jesus allegedly said in this passage, biblicists dare not find evidence of
allegorical or hyperbolic usage throughout the book of Jonah.

That leaves Jonah 3:3, the specific claim about Nineveh's size, for our consideration. Clarke's
commentary on the Bible treated the passage factually. Clarke even cited one or two ancient
accounts attesting to the huge size of Nineveh, although he cautioned his readers about their
reliability.

Consider also, if you will, the various defenses of Jonah 3:3 that are common to all apologetic
works (defenses already noted). They all assume that the passage is factual.

How, then, with such dissension among his ranks, can the biblicist claim overwhelming
evidence for a nonliteral interpretation of Jonah 3:3? Obviously, at least from a conservative
viewpoint, there is nothing about this passage that demands a nonliteral interpretation. In fact,
there is nothing wild about this passage except the size of Nineveh. Thus, the biblicist is
obliged to take it at face value or else admit that reason has overridden the authority of the
Bible.

Therefore, at least here, allegorical or hyperbolic usage is of no help to the biblicist.

SUMMARY OF THE NONLITERAL


OPTIONS
In judging the case of biblical inerrancy, we must accept the face value of a passage unless
another genre (poetry, allegory, etc.) obviously applies. That is, we may not abandon the face
value of the text unless the evidence compels us to do so.

Volume 1990 - 2002 Issue


Page 169 of 2049
Skeptical Review Edited by Farrell Till
We are not compelled to abandon the plain meaning of a biblical passage merely because it is
morally, scientifically, or doctrinally embarrassing. (One might reasonably expect an ancient
work to contain such errors, a possibility that cannot be ruled out a priori.) Therefore, the
burden of proof is on those who claim that a passage fits another genre. Poetic usage,
allegorical usage, hyperbolic usage, and other nonliteral usages must always be justified by
positive, compelling evidence. We are never at liberty to substitute mere speculation for the
plain meaning of a text.

Once the biblicist accepts Jonah as historical, he is stuck with an impossibly big city called
Nineveh.

CONCLUSION
If you accept biblical inerrancy on faith, then you will be blind to biblical error. How can you
recognize something that, to your mind, doesn't exist? If that is your mentality, then biblical
errors will appear as mere puzzles to be explained away. Indeed, the true fanatic doesn't even
worry about the lack of good explanations. Jesus will simply hand out the answers once we
get to heaven!

The heart that rejects reason cannot be called upon to recognize the force of evidence, be it
piled ever so high! Let those minds slumber on. I speak to the man or woman whose mind
still functions.

There is no substitute for reason!

It won't do to claim that God wrote the Bible, thus conferring inerrancy on it. We don't know
a priori what God wrote--if anything. Indeed, in this line of thought we must first establish
God's existence, and, even if we could do that, we would still have to prove that, in fact, God
wrote the Bible. No shortcuts here!

If the Bible is divinely inspired, then how might we know it? God (or his agent) may choose
to personally inform us, we may learn by mystical means, or, after analyzing the evidence, we
may conclude that such is the case. Unfortunately, God is rather stingy on verbal
communication and short on personal appearances. Nor is he in the habit of giving his earthly
messengers special uniforms or badges. Thus, given a world full of liars and lunatics who are
only too happy to pose as God's messengers, one is at risk here.

Of course, if you are one of those rare individuals who speak face to face with God every
morning before breakfast, then you may have some privileged information. You might also be
a nut with a malfunctioning brain!

The second approach is not much better. Mystics do not always agree with one another--
especially when it comes to heavenly messages. How do we know, without appealing to
evidence, who is right? How do we know that feeling right is being right? Thus, if we seek
sound answers to factual questions, we cannot escape the need to analyze the evidence.

Volume 1990 - 2002 Issue


Page 170 of 2049
Skeptical Review Edited by Farrell Till
No matter how you slice it, we can't establish the inerrancy of the Bible--or any book--before
actually studying it. Such a study, of course, cannot begin with the premise of biblical
inerrancy. (You would be astounded at how often biblical inerrancy is assumed in order to
prove biblical inerrancy. Such "proofs" are as common as weeds after a spring rain!)

If the Bible is approached with an objective, open mind--and not with the usual wishful
thinking--then it becomes obvious that its pages contain many errors. I have put before you
just one sampling of the Bible's errors, drawn from a vast ocean.

(Dave Matson's address is 330 South Hill Avenue, Pasadena, CA 91106.)

The Resurrection Maze


Farrell Till
When two or more statements contradict one another, they cannot all be right. If we apply this
rule of evidence to the testimony of the "eyewitnesses" to the resurrection, we have to
conclude that at least some of the testimony was erroneous. If some of it was erroneous, then
the Bible inerrancy doctrine itself must be erroneous.

Inconsistencies in the four gospel accounts of the resurrection are too numerous to discuss in
a single article, so I will limit discussion at this time to just a few. Matthew began his
narrative of the events of that day by telling us that Mary Magdalene and "the other Mary"
went to the sepulcher "as it began to dawn toward the first day of the week" (28:1). The time
factor that he identified contrasts sharply with the testimony of Mark, Luke, and John, who
said, respectively, that the time was "very early on the first day of the week... when the sun
was risen" (16:1) or "on the first day of the week, at early dawn" (24:1) or "on the first day of
the week... while it was yet dark" (20:1). By stretching imagination, perhaps we could
reconcile the testimony of Matthew and Luke. If it was "beginning to dawn toward the first
day of the week," we could maybe grant that this could be called "early dawn," but how could
the sun already be risen if it was only beginning to dawn? And if the sun had already risen, as
Mark claimed, how could it have been "yet dark," as John said? For that matter, how could it
have been "yet dark" if the morning had reached any stage that could be correctly described as
"dawn"? I grew up on a farm, so I can remember getting up many times as it was "beginning
to dawn" or while it was still "early dawn," but it certainly wasn't "yet dark." By looking
outside, I could see clearly enough to recognize objects. Certainly by the time the sun had
risen, it was never "yet dark" outside. So how could it possibly be that John was right in the
time factor that he specified but the other three were also right in their time factors? This is a
chronological discrepancy that bibliolaters have never satisfactorily explained.

As noted, Matthew said that Mary Magdalene and "the other Mary" went to the tomb. Mark
identified the women as "Mary Magdalene, and Mary the mother of James, and Salome"
(16:1). Luke said that the women who went to the tomb were "Mary Magdalene, and Joanna,

Volume 1990 - 2002 Issue


Page 171 of 2049
Skeptical Review Edited by Farrell Till
and Mary the mother of James, and the other women with them" (24:10), and John mentioned
only Mary Magdalene (20:1). We have all heard the Gleason Archer-John Haley type of
explanation for the "variances" in these names. Matthew chose to identify only two of the
women, John chose to identify only one, but their failure to mention the presence of Salome
or Joanna or "the other women" would not mean that they were not there. The same line of
reasoning is usually applied to the variations in the number of angels reported at the scene.
Matthew said that "an angel" (one) descended from heaven (28:2). Mark spoke of "a young
man... arrayed in a white robe" (16:5), who was presumably an angel, but Luke (24:4) and
John (20:12) wrote of two angels. If there were two, then there had to be one, inerrantists will
say, but in a matter as vitally important as the testimony to a resurrection, inspired by an
omniscient, omnipotent deity, this kind of "explanation" has dubious merit at best. However,
it is the stock explanation of inerrancy defenders in matters like these, so I will simply
mention it, leave it to the readers to judge its merit, and go on to other discrepancies that no
stretch of imagination can satisfactorily resolve.

After the women arrived at the tomb, Matthew said that a "great earthquake" occurred and "an
angel of the Lord descended from heaven and came and rolled away the stone and sat upon it"
(28:2). Mark, Luke, and John, however, disagreed. Mark said that the women found the stone
already rolled away when they arrived at the tomb (16:2). Luke agreed with Mark and said
that the stone was "rolled away from the tomb" when the women arrived (24:2), and John said
that the stone had been "taken away from the tomb" when Mary Magdalene arrived (20:1). So
who was right? Matthew or the other three? It simply could not have been that the stone was
both in place and rolled away too when the women came to the tomb? Our rule of evidence
demands one of two conclusions: Matthew was right and the other three were wrong, or the
other three were right and Matthew was wrong. Both versions of the story cannot be right.

Matthew wrote that the angel who descended from heaven and sat upon the stone said to the
women, "He (Jesus) is not here; for he is risen, even as he said. Come see the place where the
Lord lay" (28:6). Luke's angels (plural) also reminded the women that Jesus had said that he
would be raised:

Why seek ye the living among the dead? He is not here, but is risen: remember how he spake
unto you when he was yet in Galilee, saying that the Son of man must be delivered up into the
hands of sinful men, and be crucified, and the third day rise again (24:5-7).
As noted in an earlier article, Luke said that the women, upon hearing this, "remembered his
words" (v:8). Although Matthew didn't specifically say that the women remembered the
words of Jesus that the angel reminded them of, he certainly implied that they did, because he
immediately said that the women "departed quickly from the tomb with fear and great joy,
and ran to bring his disciples word" (28:8). This is essentially what Luke said too. After
saying that the women "remembered his (Jesus's) words," he said that the women "returned
from the tomb, and told all these things to the eleven, and to all the rest" (24:9).

Now if the women "ran to bring his disciples word," as Matthew said, and if they "told all
these things to the eleven," as Luke claimed, then surely they told the eleven about the angel
(angels?) and what he (they?) had told them. In other words, Matthew's and Luke's versions of
the resurrection story both present the women at the tomb as ones who had been informed

Volume 1990 - 2002 Issue


Page 172 of 2049
Skeptical Review Edited by Farrell Till
about the resurrection and convinced that it had occurred before they ran to tell the disciples
what they had seen.

One would never guess this, however, by just reading John's version. John was the one who
chose to mention only the presence of Mary Magdalene at the tomb, and he said that Mary,
upon seeing the stone taken away from the tomb, ran and told Simon Peter and "the other
disciple" (20:2). She did this before seeing the angels, because it wasn't until after she had
returned to the tomb that she saw the angels (vv:11-13). So when John's Mary ran to Peter and
the other disciples, did she run with the "great joy" that Matthew mentioned? And did she tell
the disciples "all these things" that Luke said the angels had told them? Apparently no,
according to John's story! When John's Mary found the disciples, she said, "They have taken
away the Lord out of the tomb, and we know not where they have laid him" (20:2).
Obviously, there was no "joy" in what she was saying here. She was quite distressed.

More perplexing than that, however, is the fact that John's Mary believed that the body of
Jesus had been stolen from the tomb. Why would she have thought that IF she had seen angels
at the tomb (as Matthew, Mark, and Luke claimed), who had reminded her of Jesus's promise
to rise again on the third day, and especially IF she had "remembered his words" after the
angels had jogged her memory? Obviously, John was not aware of any prior contact that
Mary had had with angels or of any assurances she had received from angels that Jesus had
been resurrected. His Mary was a distraught woman who, at the time of her report to Peter
and "the other disciple," had no idea that Jesus had risen from the dead. Even after she
returned to the tomb with Peter and the other disciple, she still thought that the body of Jesus
had been stolen, because the angels that she encountered this time inside the tomb asked her
why she was crying, and she said, "Because they have taken away my Lord, and I know not
where they have laid him" (Jn. 20:13). But why would she have thought that at this time IF
prior to going to get Peter an angel or angels had told her, "He is risen" (Mt. 28:6; Mk. 16:6;
Lk. 24:6)?

Gleason Archer, in his typically far-fetched style, has offered this answer to the question:

She apparently had not yet taken in the full import of what the angel meant when he told her
that the Lord had risen again and that He was alive. In her confusion and amazement, all she
could think of was that the body was not there; and she did not know what had become of it.
Where could that body now be? It was for this reason that she wanted Peter and John to go
back there and see what they could find out (Encyclopedia of Bible Difficulties, pp. 348-349).
That sounds pretty thin to me, but inerrantists are famous for this kind of circumlocution
when they are confronted with obvious discrepancies in the Bible text. Luke flatly stated that
the women at the tomb, who would have included Mary Magdalene, "remembered his
(Jesus's) words" when the angels reminded them that Jesus had predicted his resurrection
while they were "yet in Galilee" (24:7-8). Despite the clarity of this statement, Archer wants
us to believe that the obvious discrepancy between it and John's depiction of Mary in his
resurrection story was only "apparent," because she "had not yet taken in the full import of
what the angel meant when he told her the Lord had risen again." As an explanation of the
problem, it is too transparent to deserve serious comment. What "full import" was there to
take in? The angels said, "He is risen," as he had promised while he was "yet in Galilee," and
the women "remembered his words"!

Volume 1990 - 2002 Issue


Page 173 of 2049
Skeptical Review Edited by Farrell Till
While yet in her state of confusion about what had happened, John's Mary encountered Jesus
by the tomb--and didn't even recognize him! She thought that he was the gardener (20:14-15).
So we are supposed to believe that angels had some time prior to this told her that Jesus was
not "here," that "he is risen," as he had said would happen while he was "yet in Galilee," and
that she had "remembered his words," but when finally she came face to face with the risen
Jesus at the empty tomb, she didn't even know who he was. If a missionary preaching a
religion we had not grown up with should come into our country with a story as inconsistent
as this, who would believe it?

But it gets even worse. Mary's encounter with Jesus at the tomb in John's version clearly
happened after Mary had found the stone rolled away and had run to tell Peter and had
returned to the tomb, but this runs completely contrary to Matthew's version of an encounter
with Jesus near the burial site. Matthew said that the women, upon receiving instructions from
the angel (just one) to go tell the disciples that Jesus would go before them into Galilee,
"departed quickly from the tomb with fear and great joy, and ran to bring his disciples word"
(28:7-8). But look at what Matthew then said:

And behold, Jesus met them, saying, All hail. And they came and took hold of his feet, and
worshipped him. Then saith Jesus unto them, Fear not: go tell my brethren that they depart
into Galilee, and there shall they see me (28:9).
Perhaps Jesus thought that the angel's message had not been clear enough, so he made an
appearance himself to tell the women the same thing the angel had just said to them. Maybe
he knew what he was doing too. A person as confused as Gleason Archer's Mary Magdalene
would probably have to be given a message like this at least twice before she would
understand it. At any rate, Matthew said that the women met the risen Jesus as they were
running to tell the disciples that an angel had told them to meet Jesus in Galilee, and this flatly
contradicts John's version. John said that Mary Magdalene saw Jesus after she had fetched the
disciples and returned to the tomb. Before any inerrantist tries to tiptoe around this problem
by claiming that John's failure to mention an encounter with Jesus while Mary was running to
get Peter doesn't mean that such an encounter did not occur, let him explain why Mary didn't
recognize him in the garden if she had already seen him before this and had even taken "hold
of his feet and worshipped him" (Matt. 28:9). She had done all of this on the way into town
but now back at the tomb again she wasn't even able to recognize Jesus! If I may resort to
slang for just a moment, come on, give me a break!

The holding of Jesus's feet mentioned by Matthew poses yet another problem, for when John's
Mary finally recognized Jesus at the tomb, he specifically forbade her to touch him: "Touch
me not: for I am not yet ascended unto the Father" (20:17). What ascending to the Father
would have to do with a restriction on touching Jesus is anybody's guess (I just read the
scriptures; I didn't write them), but Jesus plainly stated this as a reason why Mary should not
touch him. Yet, in Matthew's story, when the women, who would have included Mary, met
Jesus on the way to "bring the disciples word," they "took hold of his feet and worshipped
him." Now if they took hold of his feet, then they surely touched him. How else could they
have taken hold of his feet? But at this encounter Jesus said nothing about not touching him,
even though at this time he had most certainly "not yet ascended unto the Father." How could
he have done so, since this encounter happened as the women were running into town to find

Volume 1990 - 2002 Issue


Page 174 of 2049
Skeptical Review Edited by Farrell Till
the disciples, and the encounter with Mary, when Jesus told her he had not yet ascended,
happened after she had found the disciples and had returned to the tomb?

The encounter with Jesus as the women were running into town raises yet another problem.
Matthew plainly said that after the women "departed quickly from the tomb with fear and
great joy" (28:8), they met Jesus and "took hold of his feet and worshipped him" (v:9). Now if
the women met Jesus and worshipped him, they surely recognized him and realized that he
had been resurrected. Why then did Mary say what she said when she found Peter and the
other disciple? "They have taken away the Lord out of the tomb, and we know not where they
have laid him" (Jn. 20:2). I earlier noted that this statement made no sense at all in light of
what Matthew, Mark, and Luke said that the angel(s) told the women at the tomb and what
Luke specifically said the women "remembered," but now that we know that the women, on
the way into town to find the disciples, encountered Jesus, touched him, and worshipped him,
Mary's statement to Peter goes beyond making no sense. It becomes an insult to the
intelligence of all rational people who are asked to believe in the resurrection despite glaring
discrepancies like this.

Juxtaposed and considered as a single story, the four resurrection accounts form a veritable
maze of contradictions. So once again I must remind readers of the rule of evidence that says
falsus in uno, falsus in omnibus (false in one thing, false in everything). When alleged
witnesses to an event contradict themselves as flagrantly as did the four gospel writers in their
accounts of the resurrection, everything that they said must be viewed with suspicion. These
writers claimed that a man who was physically dead returned to life. A claim that
extraordinary requires extraordinary proof, but there is certainly nothing extraordinary in the
resurrection testimony of Matthew, Mark, Luke, and John. It is so riddled with inconsistencies
that no one but the hopelessly credulous could possibly believe it.

Last summer, as I was working in my front yard, two Jehovah's Witnesses came by and
engaged me in conversation. As we stood talking, I explained my background and familiarity
with the Bible. I referred to the resurrection stories to illustrate the fact of inconsistency in the
Bible text. One of the JW's nodded at the cars going by and said that if an accident happened
and the police asked us to write reports, they would have inconsistencies in them, but that
wouldn't mean that the accident did not happen. "No, it wouldn't," I agreed, "but if we were
all three inspired by an omniscient, omnipotent deity while we were writing our reports, there
wouldn't be any inconsistencies." The only response to this was a sheepish smile.

So it is with the resurrection accounts in the four gospels. If Matthew, Mark, Luke, and John--
all four of them--had really been guided and directed by an omniscient, omnipotent deity
while they were writing their gospels (as the inerrancy doctrine claims), there would be no
maze of inconsistencies in the juxtaposition of their stories. There would be that perfect unity
and harmony that fundamentalist preachers talk about so much--but which doesn't really exist.
I'm very afraid that the faith of Christians who have put so much hope in the resurrection will
prove to be vain and that they will, unfortunately, never know that "they are of all men most
miserable."

Volume 1990 - 2002 Issue


Page 175 of 2049
Skeptical Review Edited by Farrell Till

Computer Bulletins
We receive frequent requests for permission to use TSR materials in computer bulletins.
Anyone wishing to do this need not write for permission. Nothing in TSR is copyrighted,
specifically because we want the information to be spread as much as possible. We are
already receiving subscription requests from those who have seen our articles in computer
bulletins.

We will copy all TSR articles to high density disks in either Wordstar 6 or WordPerfect 5.1
format for anyone wishing to use them in computer bulletins. Include $1 for return postage
with your blank disk.

Volume 1990 - 2002 Issue


Page 176 of 2049
Skeptical Review Edited by Farrell Till

The Skeptical Review


Volume Three, Number Three
July/August/September 1992
Farrell Till, editor

• Increasing the Advantage


In response to complaints that TSR's editorial policy is unfair, TSR will no longer
publish rejoinders to rebuttal articles.

• Sarah's Power to Conceive


"Living in a time when people believed that females emitted semen during sexual
intercourse, the author of Hebrews wrote in ... [Hebrews 11:11] something that he
thought was a scientific fact. As it turned out, he was swrong. He made a mistake, so
he couldn't possibly have been writing under the direction of an omniscient,
omnipotent deity."

• Sarah's "Power" to Conceive: A Response


Bill Lockwood responds to the above article.

• Squeezing Fifty Years Into Twenty

• More About the Resurrected Saints


Ed Babinski's article, "What Happened to the Resurrected Saints?" generated several
rebuttals. Farrell Till responds to reader criticisms of Babinski's article.

• Did They or Didn't They?


After the women supposedly saw the empty tomb, did they tell anyone what they saw
or didn't they? "That's the problem that inerrantists must resolve."

• Reader Reaction

• A Two-Thousand-Dollar Oppurtunity

Volume 1990 - 2002 Issue


Page 177 of 2049
Skeptical Review Edited by Farrell Till
• Computer Bulletins

Increasing the Advantage


In all issues of TSR, we have offered the opposition equal space to rebut our lead articles.
Although we have asked many inerrancy defenders to write rebuttal articles, only a few have
accepted the offer. Two issues were published without rebuttals, because we could find no
one to represent the inerrancy position. Inerrantists insist that our view of the Bible is wrong,
yet they are unwilling to show us we are wrong. That's a strange attitude to take in a matter as
important as fundamentalists say that Bible inerrancy is.

Recently, we have had a few subscribers who believe in Bible inerrancy tell us that they think
our editorial policy is unfair, because we always give ourselves the final word through
rejoinders added to the rebuttal articles. Apparently those who have made this complaint can't
see the decided advantage that the inerrancy defenders would have if we didn't add these
rejoinders. We write our articles first and send them to inerrancy spokesmen whom we invite
to write responses for simultaneous publication. We offer the invitation without knowing
what they will say in their rebuttal articles, but, of course, they know what we have said
before they begin writing. So if anyone has the advantage, it would clearly be the rebutters.

We seriously wonder about the sincerity of this complaint. As Ralph Nielsen said in his letter
to Wayne Jackson on page 14 of this issue, if TSR is "propagating error," inerrantists should
jump at the chance we offer them to propagate truth. If their position is as right as they seem
to think it is, they shouldn't be deterred by the thought of editorial rejoinders being published
at the end of their articles. What does truth have to fear in editorial rejoinders? We can assure
all fundamentalist editors who receive TSR that they dare not offer us the opportunity to rebut
any articles that they publish in their papers, for if they do, we will react with immediate
acceptance and not worry a bit about whether rejoinders will follow our rebuttals. We are that
confident that truth is on our side, and as we just said, what does truth have to fear in editorial
rejoinders?

To test the sincerity of inerrancy believers who, because of some perceived unfairness in our
editorial policy, have refused our offer of equal space to present their views, we have decided
to increase the advantage they already have. Beginning with this issue, we will no longer
publish rejoinders to rebuttal articles. If we see anything in a rebuttal article that deserves
comment, we will save it until the next issue and simultaneously publish the rebutter's
reaction to it if he cares to submit one. This way the inerrancy position will always have the
advantage of the final word. That is about as fair as any editorial policy could be.

Confidence in our position is the reason why we are willing to make this change. As the
letters in the "Reader Reaction" section of this issue (pp. 14-15) indicate, our readers are
sophisticated enough to see through the flimsy explanations that bibliolaters resort to in their
frantic attempts to preserve the inerrancy doctrine. Readers like these don't need editors to
point out fallacies in fundamentalist logic.

Volume 1990 - 2002 Issue


Page 178 of 2049
Skeptical Review Edited by Farrell Till
In this issue, we are publishing a rebuttal article written by Bill Lockwood, a Church-of-
Christ preacher from Marlow, Oklahoma. In fairness to him, we want our readers to know that
he agreed to write this article before we had announced our new policy. Primarily an ad
hominem attack, it fails to rebut the major points in our lead article, but we may do a follow-
up exchange in the next issue.

We at least commend Mr. Lockwood for showing the courage of his convictions, and we hope
our new policy will encourage other inerrantists to follow his example.

Sarah's Power to Conceive


Farrell Till
Occasionally, we have recommended to our readers other publications and materials that we
believe would assist them in their study of the Bible inerrancy doctrine. Bible Review,
published by the Biblical Archaeology Society, is a bimonthly journal that serious students of
the inerrancy doctrine should find useful. Subscriptions can be obtained at P. O. Box 7027,
Red Oak, IA 51591, for $24 annually. Each issue has 50 pages of scholarly articles on various
biblical subjects.

Bible Review appears to endorse the belief that the Bible is "God's word"; nevertheless, it
obviously respects scholarship above tradition, a policy that often puts it in conflict with
major Christian doctrines. Its "Readers Reply" section frequently has letters from irate
fundamentalists who are threatening to cancel their subscriptions because of liberal positions
that contributing writers have taken on such sacred subjects as the virgin birth, the divinity of
Jesus, the resurrection, etc. It does not openly challenge the inerrancy doctrine, but readers of
The Skeptical Review could take much of the information they will learn by regularly reading
Bible Review and easily apply it to the doctrine.

An excellent example of what I mean can be found in an article by Pieter Willem van der
Horst in the February 1992 issue ("Did Sarah Have a Seminal Emission?" pp. 35-39). In this
convincingly documented article, the author (a professor of New Testament and the Jewish
and Hellenistic milieu of early Christianity at the University of Utrecht) proved that people in
biblical times, as far back as 500 B.C. and probably before, believed that during sexual
intercourse the female emitted semen that mingled with the semen of the male to produce
pregnancy. Van der Horst cited the belief of Alcmaeon (about 500 B.C.) that the sex of a child
was determined by "whose semen was most abundant [during intercourse]" (p. 36). He quoted
Democritus of Abdera (5th century B.C.), Aristotle (4th century B. C.), Galen (2nd century
B.C.), and Lucretius (1st century B.C.), all of whom believed that during sexual intercourse
the female expelled sperm from her ovaries that mixed in the womb with the male sperm to
produce an embryo (p. 36).

Volume 1990 - 2002 Issue


Page 179 of 2049
Skeptical Review Edited by Farrell Till
Van der Horst then turned to Hebrew literature, including the Old Testament scriptures, to
show that these same ideas were "also prevalent in early Jewish circles" (p. 38). He cited one
OT passage in particular that implies that belief in female semen was commonly accepted:

In the Old Testament, Leviticus 12:2 seems to indicate that a woman can produce semen:
"When a woman tazria and bears a male child, then she shall be unclean seven days." The root
of tazria is ZR, which means to sow (a seed). When a form of ZR means "to become pregnant,
to be impregnated," the form tazara (the niphal or passive form) is always used (see, for
example, Numbers 5:28; Nahum 1:14). In Leviticus, however, the causative (hiphil) is used.
The only other place in the Hebrew Bible where the causative form of this root appears, it is
used of plants in the sense of "produce seed, yield seed, form seed" (Genesis 1:11-12--on the
third day of creation God created plants yielding seed). The causative form, used in Leviticus
12:2, cannot mean anything else than "make seed." Commentators have, of course, had
trouble with this verse and have proposed emendations of the text, because they found the
thought expressed impossible (see, for example, Baruch Levine's recent Commentary on
Leviticus, Philadelphia: Jewish Publication Society, 1989). But we cannot avoid at least the
possibility that the author of Leviticus 12 meant what he seems to have written, that is, that a
woman can produce semen (p. 38).
To his credit, van der Horst did not form any premature conclusions from this one passage.
He proceeded to show that clearly stated beliefs in female semen were expressed in both the
Talmud (a collection of Jewish law and teachings compiled in the Palestianian version of the
5th century A.D. and the Babylonian version of the 6th century A. D.) and the midrashim (a
collection of postcanonical rabbinic commentaries on the Old Testament books).

Van der Horst referred to a section of the Babylonian Talmud where the rabbinic
commentators presented an exegesis of Leviticus 12:2 (just referred to above) in conjunction
with Genesis 46:15, which, after listing the children of Jacob that had been born to Leah, said,
"These are the sons of Leah, whom she bore to Jacob in Paddan-Aram, together with his
daughter Dinah." The fact that the text referred to the male children as "sons of Leah" but to
the lone female as "his [Jacob's] daughter Dinah" led the rabbis to reach a rather amusing
conclusion about sex determination. Oddly enough, the "scholars" of this male chauvinistic
society thought that if a man emitted his semen first during intercourse, the offspring would
be a female, but if the woman emitted her semen first, the offspring would be a male. Van der
Horst cited instances where the rabbis "also understood the passage from Leviticus that we
previously discussed to imply, as we did, that women have a seminal emission during coition;
the rabbis took the use of the causative (hiphil) form of ZR (sow) in Leviticus 12:2 to indicate
that women too had a seminal emission" (p. 38).

To support his conclusion, van der Horst cited this passage from the Babylonian Talmud
(Niddah 31a):

Rabbi Isaac citing Rabbi Amni stated: If the woman emits her semen [hiphil of zr, like
Leviticus 12:2] first, she bears a male child; if the man emits his semen first, she bears a
female child; for it is said: "If a woman emits semen and bears a male child" (Leviticus 12:2).
Our Rabbis taught: At first it used to be said that "if the woman emits her semen first, she will
bear a male child, and if the man emits his semen first, she will bear a female," but the Sages
did not explain the reason, until Rabbi Zadok came and explained it: "These are the sons of

Volume 1990 - 2002 Issue


Page 180 of 2049
Skeptical Review Edited by Farrell Till
Leah whom she bore unto Jacob in Paddan-Aram, with his daughter Dinah" (Genesis 46:15).
Scripture thus ascribes the males to the females and the females to the males.
Apparently, it had never occurred to the rabbis that this theory could not explain why, in the
case of fraternal twins, one can be male and the other female. Perhaps they would explain this
as a case of simultaneous orgasm.

Van der Horst's purpose in establishing that people in biblical times erroneously believed that
women emitted semen during intercourse was to shed light on the meaning of a statement in
Hebrews 11:11 that has long troubled Bible scholars and translators: "Through faith also Sara
herself received strength to conceive seed and was delivered of a child when she was past age,
because she judged him faithful who had promised" (KJV). With only slight variations and
modernization of language, the wording of this statement was retained in the ASV, RSV,
NAS, NKJV, and other modern translations. The statement was made in the context of the
famous eulogy to Old Testament heroes whose lives were seen as monuments to their faith in
God.

The problem posed by the statement about Sarah's faith is that it literally said that Sarah had
received power or strength to have a seminal emission. As van der Horst demonstrated in his
article, katabole spermatos, that which Sarah's faith gave her the power to have, was in Greek
"the technical term for a male seminal emission" (p. 35). Strong defines katabole as a
deposition (something deposited) and sperma, the root from which the dative spermatos was
derived, as seed (including the male "sperm"). The meaning of the latter word should be
rather obvious, since our own word sperm was derived from it. So the writer of Hebrews
actually said that Sarah had received power to have a seminal emission or, more literally,
make a "deposit of sperm," a curious statement indeed for a verbally inspired writer who, as
he wrote, was being directed by the Holy Spirit to protect him from all possibility of error,
even in matters that didn't pertain to faith and doctrine. Van der Horst did not pursue this
point in his article, but it is an example of what I meant when I said that much of the
information learned in reading the scholarly articles in Bible Review can easily be applied to
the inerrancy doctrine.

Translators, of course, have hidden from their readers the Hebrew writer's obvious error in
this statement by simply having it read, "Sarah received power to conceive" or something
equivalent to it, but, as van der Horst pointed out, more recent translators have taken a
different approach to concealing the problem by making Abraham the subject of the verbs in
the statement:

By faith Abraham, even though he was past age--and Sarah herself was barren--was enabled
to become a father because he considered him faithful who had made the promise (NIV).

By faith he [Abraham, who was mentioned in the preceding verse] received power of
procreation, even though he was too old--and Sarah herself was barren-- because he
considered him faithful who had promised (NRSV).

It was faith that made Abraham able to become a father, even though he was too old and
Sarah herself could not have children. He trusted God to keep his promise (GNB).

Volume 1990 - 2002 Issue


Page 181 of 2049
Skeptical Review Edited by Farrell Till
Such translations obviously distort the meaning of the Greek text, because it clearly stipulated
that Sarah was the subject of the verbs in this verse. Abraham's name is not even in the
received text of this verse. I retain enough knowledge of Greek from my Bible college days to
see for myself, with the help of a lexicon, that the text states that "Sarah (not Abraham)
received power for a deposit of semen even beyond time of age and gave birth, because
faithful she (not he) deemed the [one] having promised [it]."

Bibliolaters argue that the Bible is inerrant in every detail of history, geography, chronology,
and science, as well as matters of faith and practice, but this one scientific boo-boo in
Hebrews 11:11 is enough to refute that claim. Living in a time when people believed that
females emitted semen during sexual intercourse, the author of Hebrews wrote in this verse
something that he thought was scientific fact. As it turned out, he was wrong. He made a
mistake, so he couldn't possibly have been writing under the direction of an omniscient,
omnipotent deity.

The error in this passage also wreaks havoc on another pet theory of Bible inerrantists. They
like to argue that "scientific foreknowledge" of Bible writers proves that they were divinely
inspired. I examined this argument in the Autumn 1990 issue of TSR ("What About Scientific
Foreknowledge in the Bible?" pp. 2-4) to show that it is completely without merit, but
bibliolaters still use it to dazzle gullible pulpit audiences. Briefly stated, the argument claims
that Bible writers often showed insights into scientific facts that were not known by anyone of
their times, so the only way they could have possessed such information was for God to have
miraculously instilled it in their minds during the inspiration process.

The argument sounds impressive to those who know no better than to believe it, but it is based
entirely on speculation and arbitrary interpretations of obscure biblical statements. Bill
Jackson tried to use this argument in my written debate with him and got nowhere with it. He
saw Genesis 3:15 as an amazing example of "scientific foreknowledge": "(A)nd I will put
enmity between thee and the woman, and between thy seed and her seed...." This was part of
the curse that Yahweh pronounced upon the serpent because of his participation in the first
sin. Jackson, as many of his fundamentalist colleagues still do, thought that the reference to
the "seed" of woman was an indication that the writer of Genesis understood that the female
contributes a "seed" (ovum) during procreation as well as the male. How could the writer
have known this fact, Jackson asked, "when modern science didn't know it until fairly recent
times" (Jackson-Till Debate, p. 3)?

The flaw in such reasoning as this is rather obvious. Van der Horst's research clearly
establishes that knowledge of the female's procreative "seed" was not something that "modern
science didn't know... until fairly recent times." When the Bible was being written, men knew
that the female contributed a "seed" during procreation. They just didn't understand what kind
of seed it was. The Egyptian Hymn to the Sun-God, for example, made a clear reference to
the seed of woman:

Creator of the germ in woman, Who makest seed in men, Making alive the son in the body of
his mother, Soothing him that he may not weep, Nurse even in the womb, Giver of breath to
sustain alive every one that he maketh! (Quoted by James Breasted in Dawn of Conscience,
Scribner's, 1968, p. 283, emphasis added.)

Volume 1990 - 2002 Issue


Page 182 of 2049
Skeptical Review Edited by Farrell Till
This hymn antedated the book of Genesis by several centuries, so just where is the amazing
evidence of "scientific foreknowledge" in the seed-of-woman statement in Genesis 3:15? It's
in the same place where all the other evidence of Bible inerrancy is. It's in a place called
nowhere. It just doesn't exist. Needless to say, the Hebrew writer's reference to Sarah's
reception of power to make a deposit of semen was nothing close to "scientific
foreknowledge."

We can point to even another error in Hebrews 11:11. The writer said that Sarah received
power to conceive seed (have a seminal emission), because she counted him faithful who had
promised." But this flatly contradicts the passage in Genesis 18:9-15 where Yahweh appeared
to Abraham at the oaks of Mamre and renewed his promise that Sarah would have a son:

They [Yahweh and the angels with him] said to him [Abraham], "Where is your wife Sarah?"
And he said, "There in the tent." Then one said, "I will surely return to you in due season, and
your wife Sarah shall have a son." And Sarah was listening at the tent entrance behind him.
Now Abraham and Sarah were old, advanced in age; it had ceased to be with Sarah after the
manner of women. So Sarah laughed to herself, saying, "After I have grown old, and my
husband is old, shall I have pleasure?" [Yahweh] said to Abraham, "Why did Sarah laugh and
say, 'Shall I indeed bear a child now that I am old?' Is anything too wonderful for [Yahweh]?
At the set time I will return to you, in due season, and Sarah shall have a son." But Sarah
denied, saying, "I did not laugh"; for she was afraid. He said, "Oh, yes, you did laugh" (NRSV
with Yahweh substituted for the LORD).
One who reads this quaint little yarn could rightly be excused for not seeing it as an amazing
demonstration of faith on Sarah's part, because there is absolutely nothing in it to indicate that
"she counted him faithful who had promised." As prone as Yahweh was to temper tantrums
and displays of anger toward those who crossed him, Sarah was lucky indeed that he did not
inflict her with leprosy or change her into a pillar of salt. At any rate, there is certainly no
indication of great faith on her part in the story.

In Hebrews 11:11, we have an example of just one little error that strikes at the heart of the
inerrancy doctrine in a variety of ways. It proves that the Bible is not inerrant and demolishes
the claim that amazing examples of scientific foreknowledge can be found in the Bible text.
All of this in just one short verse, yet fundamentalists will continue to proclaim the complete
inerrancy of the Bible. You can count on it.

Sarah's "Power" to Conceive: A Response


Bill Lockwood
I am delighted to take in hand to review Farrell Till's article "Sarah's 'Power' To Conceive."
With appreciation for the space afforded me in The Skeptical Review, I encourage Mr. Till to
open the scope of his paper to defend his ONLY recourse after he rejects the Bible--
agnosticism. Let his readers see what he offers in return.

Volume 1990 - 2002 Issue


Page 183 of 2049
Skeptical Review Edited by Farrell Till
The task before me as far as the "Sarah" article is concerned needs to be emphasized. The
Bible, always on the defense in Till's prosecuting paper, has the presumption of
INNOCENCE and CORRECTNESS of CLAIMS unless Till can PROVE or demonstrate
otherwise. The burden of proof in this case is upon Till's shoulders, since the burden of
"impeaching the credibility lies with the objector." This I say to let the readers know that my
task herein is not to offer proof that the Bible is of divine origin, for that is the
PRESUMPTION. Till grips the laboring oar in this one, and my only occupation shall be to
show that he does not use it well enough to overthrow the Bible. And I must add, after
watching Farrell pull a few licks, that the flotation is too heavy for his puny arms. But he
won't be lonely in his failure, for many have worn themselves out in the same effort.

PRELIMINARIES
First, Till uses an article from Bible Review by Pieter William van der Horst to bring a case
against Hebrews 11:11. Till reads van der Horst and announces that his "convincingly
documented" article proves the Bible is in error. Evidently, Till was more convinced than the
author himself who did the research, for van der Horst's piece was punctuated by the
following: "Leviticus 12:2 seems to indicate that a woman can produce semen.... We cannot
avoid at least the possibility that the author of Leviticus 12:2 meant what he seems to have
written.... The rabbis took this passage to imply (p. 38).... This makes it highly probable (p.
39)...." Regarding Hebrews 11:11, van der Horst said, "It seems evident that the author could
easily have had knowledge of the widely current double-seed theory." Thus, Till, who says he
reads logic textbooks as a pastime, runs from theory to "convincingly documented" and then
to, lo and behold, this move: "It proves the Bible is not inerrant" (final paragraph). Professor,
can you not do any better than that? Why, even atheistic logician Antony Flew would be
ashamed of you for this. From "possibility" to "seems to" to "prove." Your enamored readers
will certainly be disgusted with this breach of logic. I suppose next you will be proving that
since Sarah is in Hebrews 11's great hall of faith, a female authored the book!

A word also about Till's statement that Bible Review "respects scholarship above tradition."
Of course, I am familiar with this silly twaddle. If it is skeptical and questioning, it is
"scholarly." If it defends the Bible, it is "bibliolatry"--an idea that someone out here actually
worships God's book. Pshaw! Till would do well to avoid allowing these foolish ideas to seep
into his head and leak from his pen.

SARAH'S SEMINAL EMISSION


The chief point of van der Horst's attack is Hebrews 11:11, where the phrase katabole
spermatos, "power to make a deposit of semen," seems to indicate that the writer of Hebrews
"had knowledge" of a widely current double-seed theory and was implicitly subscribing to it
when he described Sarah as having a seminal emission" (BR, 2-92, p. 39). The Greek says
"laying down of seed," which is the act of a male, not female, in the reproductive process.

It is amazing to me that neither van der Horst nor friend Till mentioned, let alone dealt with,
the several possible translations offered by textual scholars. The single item that they both

Volume 1990 - 2002 Issue


Page 184 of 2049
Skeptical Review Edited by Farrell Till
mentioned was the translation that makes Abraham the subject of the verb (NIV, NRSV,
GNB). Till pronounced this as an example of translators who "have hidden from their readers
the Hebrew writer's obvious error." Till may be a master BErater, but he is not much of a
DEbater, at least in this case. Will he tell us why this could NOT be a LEGITIMATE
TRANSLATION? Bruce Metzger indicates that the UBS textual committee "understood the
words aute Sarra steira to be a Hebraic circumstantial clause, thus allowing Abraham (v. 8) to
serve as the subject of elaben (by faith, even though Sarah was barren, he-- Abraham--
received power to beget)." Metzger also footnotes Matthew Black in this regard (textual
commentary). These scholars have offered a possible solution, but Till and his source pass
over it with silence! But then van der Horst finds one commentator, Ceslaus Spicq, who says,
"Some ancient Greek authorities believed that women did emit semen," and THIS solution
has become wonderful in their eyes! Why? Who cannot see bias? Readers may applaud this
great "scholarship," but if the Bible is to be proven false, let them demonstrate this is
EXACTLY WHAT THE HEBREW WRITER HAD IN MIND. And while they busy
themselves to patch up this case, consider that there are also other possibilities regarding
Hebrews 11:11.

Second, Moulton and Milligan have shown from papyrus readings that katabalaios is used in
the sense of a "store-place" where deposits are made. (Lexicon, p. 324). Although it is true
that F. F. Bruce believes this to be "the highest degree improbable" as to its usage in Hebrews
11:11, it remains an alternative.

A third possibility, and the one that I personally believe to be the case here, is that the word
seed is used in the sense of "progeny" or "posterity" (see Heb. 11:18; 2:16; or Gen. 4:25).
Katabole is used 11 times in the NT, and upon each occasion, it is translated "foundation"
except in this passage (Wigram-Green, New Englishman's Greek Concordance, p. 471. See
also Berry's Lexicon, p. 52). Bauer, Arndt, and Gingrich's Lexicon reads "received power to
establish a posterity" (p. 409). Therefore, instead of being a stumbling block to inerrancy, at
11:11 we read, "Sarah received power for the 'establishment of posterity'" (offered by
Cremer's Biblical and Theological Lexicon, p. 121; Vincent's Word Studies. Vol. 4, p. 520;
Thayer's Lexicon, p. 330). Do you see, readers, how simple it is to answer Till? This
possibility at Hebrews 11:11 commends itself even more when we remember that the word
seed, beginning in Genesis 3:15, is "regularly used as a collective noun in the singular.... Thus
the word designates the whole line of descendants as a unit..." (Harris, Archer, Waltke,
Theological Wordbook of the Old Testament, Vol. 2, p. 253). If Till would be careful of such
simple lessons, he might save himself much trouble in his little paper.

A fourth solution is to understand the phrase in question as an instrumental of association or


dative of accompaniment (Leon Morris, Expositor's Bible Commentary, 12:119; Bruce
Metzger, NICCommentary, p. 302; Ralph Earle, Word Meanings of the NT; F. F. Bruce,
Ibid., p. 302; see also Dana and Mantey's Greek Grammar, pp. 90-91). This renders the
passage, "By faith he--Abraham--also, together with Sarah, received power to beget a child
when he was past age, since he counted him faithful that promised." Of course, readers of
TSR will be surprised to find these easy alternatives, for, listening to Till, one would think
that there is no solution to any of his questions. Thus, this is a new way of thinking for Till
supporters.

Volume 1990 - 2002 Issue


Page 185 of 2049
Skeptical Review Edited by Farrell Till

GENESIS 3:15
Till referred to his debate with Bill Jackson in which this verse was used by Jackson to show
scientific foreknowledge. Till ridiculed this by pointing to an Egyptian hymn to the sun god,
which, antedating Genesis, spoke of the "germ in woman." Thus, Till concluded, there is no
foreknowledge in Genesis. I marvel at how easily one can pluck the skeptical gamecock's
feathers, but see.

Amenhotep IV, the pharaoh of the "Amarna Revolution," in whose reign this was composed,
lived in the late bronze age (c. 1400-1300). Amenhotep (Akhnaton) lived at the last of this age
(c. 1364-1347, B. W. Anderson, Understanding the OT, p. 52). While it is true that liberal
scholars date the exodus circa 1250 B.C., when did the exodus actually occur? I know that
friend Till would like to date it, and consequently Moses' life and writings, in the 13th century
B. C. In so doing, he would be joined by a host of stu- dents who begin with the naturalistic
assumption that biblical data cannot be counted upon--a thing that not even our court system
would do to criminals on trial. But the Bible gives its own chronology of these events and
dates the exodus in the 15th century B. C., almost two centuries prior to the dynasty of
Akhnaton, in whose reign the poem in question originated. The dating from Solomon's era
and later gives much less variation for dating, and the general consensus is that the 4th year of
Solomon was 966 B.C. First Kings 6:1 is the text that informs Till that it had been 480 years
prior to 966 that Israel had exited Egypt. This places Moses' exodus and writings at about
1446/45 B.C., about 100 years prior to Amenhotep's birth! Hence, Genesis 3:15 will be the
burr under Till's saddle on this one, but he will have to endure it.

But while Till lingers with Akhen-Aton a while, perhaps he will try his hand at telling us why
this ancient Egyptian hymn appears to be monotheistic, so much so, that some have argued
that very thing. Could it be that the exodus, having occurred in Egypt's recent history,
influenced Egyptians to consider the one true God of the universe? The possibility is
interesting. Farrell, let this ancient pharaoh teach you a lesson as he addresses the "ultimate
reality":

How manifold it is, what thou has made! They are hidden from the face (of man). O Sole god,
like whom there is no other! Then didst create the world according to thy desire, Whilst thou
wert alone....
Yes, I enjoy discussing with Till, he brings so many wonderful things to my attention, for it
might have been years before I pulled these ancient Near Eastern texts from my shelf! But
more on Genesis 3:15. Here, in the ancient writings of Moses, the first announcement of the
gospel is found, called the Protevangelium. Satan, in the form of a serpent (Rom. 16:20 is an
allusion to the verse), would be bruised under the feet of the "seed of woman." "One such
seed is the line of woman as contrasted with the opposing seed which is the line of Satan's
followers. And then surprisingly, the text announces a male descendant who will ultimately
win with a crushing victory over Satan himself" (TWOT, Vol. 1, p. 583). No wonder the Jews
from the earliest of times, as per their Targums, recognized this as a Messianic promise. Can
Till explain the fulfillment of a multitude of intricate prophecies, beginning with this one,
without recourse to the divine? "Can he draw out Leviathan with a hook?"

Volume 1990 - 2002 Issue


Page 186 of 2049
Skeptical Review Edited by Farrell Till

FAITHFULNESS OF SARAH
Farrell ferrets out even another error in Hebrews 11:11. It deals with Sarah's faithfulness to
"him that promised," which he says flatly contradicts Genesis 18:9-15 where "Sarah laughed
to herself" when she heard the promise. Why, I was humored more than Sarah at this effort by
Till.

Till, will you allow Sarah to change her mind? By what logical insight do you equate
Hebrews 11:11 with Sarah's laughter? Yes, readers, Till at one time preached the gospel of
Christ but changed his mind about the whole thing and now accuses God of "temper
tantrums." But he will refuse to allow Sarah to change her mind. If Till knew as much about
the female gender as he should, for he is married, he would know women have the right to
change their minds! But I suppose this is the amount of grace that flows from an agnostic.
You can expect no more.

Yes, you can count on "fundamentalists" to continue to proclaim the complete inerrancy of
God's Word, and the more I read such stuff as TSR passes out seeking to dethrone the Bible, I
can only wonder why more readers do not grow tired of Till's "leaps of logic." (1) Sarah
laughed. (2) Sarah had faith. (3) Therefore, her laughter was her faith. Come now, Till, will
you not bone up on your logic? I suppose we have: (1) Till preached the gospel in pulpits of
the churches of Christ. (2) Till founded TSR. (3) Therefore, Till preached skepticism in the
pulpits. Who cannot see through this?

Till wrote to me privately that I should not include a statement herein about my prayers for
him to return to the way he once knew, but in this request, he agrees unwittingly with the
Bible. Jeremiah the prophet was instructed by God regarding his fellow apostate Jews: "Pray
not for this people... for I will not hear thee" (7:16; 11:14). Therefore, as much as I would like
to pray for Till's speedy recovery from his spirit of delusion, I have God's word forbidding it
to be done, and now Till agrees with the Father above that I should refrain. Well, Farrell, that
is a start.

(Bill Lockwood's address is 211 North 5th, Marlow, OK 73055).

Squeezing Fifty Years Into Twenty


Farrell Till
No contradictions in the Bible? That's what bibliolaters say, but the facts say something else.

A simple example of discrepancy can be found in 1 Samuel 7:1-2, which says that the Ark of
the covenant was taken to the village of Kiriath-jearim and kept in the house of Abinadab for
twenty years. Prior to its being taken to this place, it was captured in battle by the Philistines,

Volume 1990 - 2002 Issue


Page 187 of 2049
Skeptical Review Edited by Farrell Till
who afterwards began to suffer all sorts of misfortunes, ranging from the mysterious
destruction of a temple idol of their favorite god Dagon to painful tumors that the people were
afflicted with (1 Sam. 5).

To rid themselves of the ark, which they thought was the source of their problems, the
Philistines accepted the counsel of their priests and diviners who had advised them to send the
ark away on a cart pulled by two undriven milch cows that had never been harnessed to a
yoke (1 Sam. 6:1-9). The wisdom of the priests and diviners was that if the cows took the
route to Bethshemesh, this would be an omen that the evil in their midst had been caused by
the presence of the ark. The cows did indeed pull the cart to Bethshemesh, where the
townsmen rejoiced when they saw the ark (vv:10-13). Unfortunately, some of them looked
inside the ark--an absolute no no--and Yahweh, in typical fashion, struck them with a "great
slaughter" that killed 50,000 men (v:19).

Understandably frightened by this disaster, the survivors sent a message to the inhabitants of
Kiriath-jearim asking them to take custody of the ark. Men were sent to fetch the ark, and
when it arrived in Kiriath-jearim, it was taken to the house of "Abinadab on the hill" (7:1-2),
where it remained for twenty years. Second Samuel 6:1-11 relates how that king David
gathered 30,000 "chosen men of Israel" to go to the house of Abinadab on the hill and
transport the ark to Jerusalem. On the way back, Uzzah, one of the drivers of the cart, touched
the ark to steady it when the oxen stumbled, and Yahweh struck him dead (vv:6-7). Uzzah
was a son of Abinadab, who had conscientiously cared for the ark for twenty years, but, of
course, none of this mattered to Yahweh. Uzzah touched the ark, and apparently because he
wasn't a Kohathite of the priestly tribe of Levi (Num. 3:27-32,38), Yahweh instantly
dispatched him to the nether world for touching a sacred object. Good intentions just never
seemed to matter to the petulant Yahweh, but that is another story for another time.

The point of this article is that all this may make for a quaint little tale, but it is a tale with a
serious discrepancy in it. The ark was captured by the Philistines in 1 Samuel 4, well before
Saul was made king of Israel in chapter 10. It stayed at the house of Abinadab for twenty
years until it was transported to Jerusalem by David, who was Saul's successor to the throne.
Yet we are told that Saul reigned as king for 40 years (Acts 13:21).

Just how did this happen? How could 40 years pass throughout Israel in every place except
Abinadab's house on the hill at Kiriath-jearim, where somehow only 20 years went by? On
closer scrutiny, the problem is even worse. David brought the ark to Jerusalem only after he
had captured the city from the Jebusites, who had maintained control all through Saul's reign.
Thus, David had to reign in Hebron for seven years and six months (2 Sam. 5:5) until
Jerusalem was captured; then he transported the ark from Abinadab's house to the new capital
of Israel. So if, as I have noted, at least some time passed after the ark was captured before
Saul was made king, and then if Saul reigned for 40 years, and then if David reigned for seven
and a half years before the ark was removed from Abinadab's house and taken to Jerusalem,
we must be talking about a span of time equaling almost fifty years. How could it possibly be
true, then, that the ark "abode in Kiriath- jearim" for twenty years (1 Sam. 7:2)?

Volume 1990 - 2002 Issue


Page 188 of 2049
Skeptical Review Edited by Farrell Till
The opportunity to write a response to this article, along with a promise to publish it
simultaneously without editorial comment was offered to three inerrancy spokesmen. None
accepted the offer.

"There are Bible scholars, and there are fundamentalists." Ralph Nielsen, whose letter to
Wayne Jackson appears on page 15, attributes this quotation to Nick Cardell, a Unitarian
minister in Syracuse, New York.

More About the Resurrected Saints


Farrell Till
The attention it has received indicates that Ed Babinski's article about the resurrected saints
(TSR, Winter 1992, pp. 14-15) touched a sensitive spot in the thick skin of Bible inerrancy. A
reader in Georgia wrote a "response" to it as did also Tom Fishbeck in his newsletter The
Bible Answers, which he presents as a bulletin published to express the views of SIG, a
special interest group of MENSA, on biblical issues. A reader receiving a free subscription to
TSR at the request of a friend called to ask that his name be removed from our mailing list.
When asked if he would mind telling us his specific objections to the paper, he cited the
"stupid nonsense" in articles like "the one about the resurrected saints" as the reason why he
preferred not to have TSR "polluting his mail box."

It is one thing to hurl insults at ideas embarrassing to one's personal beliefs; it is another to
refute the ideas with logical arguments. I read Fishbeck's "rebuttal" of Ed Babinski's article
and found it weak as water. He suggested four possible explanations he is "willing to believe"
about the problem of these mysterious, unnamed saints who were resurrected from their
tombs at the moment Jesus died on the cross: (1) Matthew was accurate, (2) Matthew was
accurately reporting the occurrence of false testimony of others without knowing it was false,
(3) the original gospel of Matthew asserted at least one error, or (4) a change was made to one
of the earliest copies of the gospel of Matthew (The Bible Answers, Nov. 1991, p. 4).

The first of these explanations is no explanation at all, because the whole thrust of Babinski's
article was that such an event as this would have been so extraordinary that news of it would
surely have reached contemporary historians and thus been passed down to us in secular
records or, if not that, the other gospel writers would have considered the event to be such
convincing evidence of the divinity of Jesus that they too would have included it in their
accounts of the crucifixion and resurrection. To say, then, that a possible explanation of this
problem is that Matthew was accurate explains absolutely nothing. The mystery of the
exclusion of this stupendous miracle from the other gospels still begs for a sensible
explanation.

Fishbeck's second and third explanations are even worse solutions, because they totally
destroy the Bible inerrancy doctrine. How could Matthew have been inerrantly guided in what

Volume 1990 - 2002 Issue


Page 189 of 2049
Skeptical Review Edited by Farrell Till
he was writing if he reported as truth "the occurrence of false testimony of others"? That he
may have unknowingly done this is beside the point, because the whole purpose of divine
inspiration would have been to protect the inspired writers from error. So if Matthew were in
fact verbally inspired by the Holy Spirit in what he wrote, he wouldn't have made mistakes
unknowingly. Furthermore, this Matthew was presumably one of the apostles who were
present in Jerusalem when Jesus was crucified, so if such an event as this really happened,
wouldn't he have had personal knowledge of it? Unless he was incredibly dense, he couldn't
possibly have been duped by false testimony about a miracle that he would have known from
his own personal experience had not happened. Also, if "the original gospel of Matthew
asserted at least one error," as Fishbeck said he was "willing to believe," then the gospel of
Matthew was not inerrant, and if the gospel of Matthew was not inerrant, how can we believe
that any of the gospels and other allegedly inspired books were inerrant? Fishbeck, who has
often bent over backwards in his newsletter to defend the Bible against error, seemed not to
be thinking too clearly when he offered the possibility of an error as a defense of inerrancy.

His fourth and final explanation was almost as damaging, for if "a change was made to one of
the earliest copies of the gospel of Matthew," that would merely underscore a problem
sensible Bible readers have long recognized: the original autographs of the Bible have been so
corrupted by redactions and copyist errors that no rational-thinking person can have an iota of
confidence in the integrity of the present text. God verbally inspired the original manuscripts
of the Bible, we are told, but then left the transmission of them to error-prone scribes and
translators. That makes about as much sense as belief in astrology and crystal balls.

In a letter to Fishbeck, Ed Babinski pointed out an interesting bit of information that was not
included in his original article or in a written exchange on the same subject that he had earlier
made with Gary Habermas of Jerry Falwell's Liberty University: "Both Mark and Luke
contain in sequence the passages which immediately precede and follow the Matthean 'raising
of the many'" (personal correspondence, April 17, 1992). Perhaps the best way to emphasize
the force of Babinski's point would be to juxtapose Matthew's account with Mark's:

Now from the sixth hour until the ninth hour there was darkness over all the land. And about
the ninth hour Jesus cried out with a loud voice, saying, "Eli, Eli, lama sabachthani?" that is,
"My God, My God, why have you forsaken me?"

Some of those who stood there, when they heard that, said, "This man is calling for Elijah!"
Immediately one of them ran and took a sponge, filled it with sour wine and put it on a reed,
and offered it to Him to drink.

The others said, "Let Him alone; let us see if Elijah will come to save Him."

And Jesus cried out again with a loud voice, and yielded up his spirit.

Then, behold, the veil of the temple was torn in two from top to bottom; and the earth quaked,
and the rocks were split, and the graves were opened; and many bodies of the saints who had
fallen asleep were raised; and coming out of their graves after His resurrection, they went into
the holy city and appeared to many.

Volume 1990 - 2002 Issue


Page 190 of 2049
Skeptical Review Edited by Farrell Till
So when the centurion and those with him, who were guarding Jesus, saw the earthquake, and
the things that had happened, they feared greatly, saying, "Truly this was the Son of God!"

And many women who followed Jesus from Galilee, ministering to Him, were there looking
on from afar, among whom were Mary Magdalene, Mary the mother of James and Joses, and
the mother of Zebedee's sons (Matt. 27:45-56, NKJV).

If one would just omit the underlined part, for all intents and purposes, he would have Mark's
version of the same events, but to make this point as emphatic as possible, I will show the
entire parallel passage from Mark:
Now when the sixth hour had come, there was darkness over the whole land until the ninth
hour. And at the ninth hour Jesus cried out with a loud voice, saying, "Eloi, Eloi, lama
sabachthani?" which is translated, "My God, My God, why have you forsaken me?"

Some of those who stood by, when they heard that, said, "Look, He is calling for Elijah!"
Then someone ran and filled a sponge full of sour wine, put it on a reed, and offered it to him
to drink, saying, "Let Him alone; let us see if Elijah will come to take Him down."

And Jesus cried out with a loud voice, and breathed His last.

Then the veil of the temple was torn in two from top to bottom. So when the centurion, who
stood opposite Him, saw that He cried out like this and breathed His last, he said, "Truly this
man was the Son of God."

There were also women looking on from afar, among whom were Mary Magdalene, Mary the
mother of James the Less and of Joses, and Salome, who also followed Him and ministered to
Him when he was in Galilee, and many other women who came up with Him to Jerusalem
(Mark 15:33-41, NKJV).

I could have started the quotations several verses earlier and extended them several more, and
the results would have been the same. The two accounts are alike detail for detail, except for
Matthew's statement about the earthquake that opened the graves of the resurrected saints.

This startling fact requires bibliolaters to believe that the Holy Spirit in his omniscient
wisdom guided Mark to record such trivial details as the casting of lots for Jesus's garments
(mentioned earlier in both accounts) and the offering of sour wine (vinegar) to Jesus, just as
Matthew reported, but for some reason chose not to have Mark tell about the resurrection of
many saints who later went into the holy city and appeared to many! Only the gullibly naive
could possibly believe that.

Some inerrantists will no doubt argue that the details just mentioned were far from trivial in
that they fulfilled OT prophecies. However, that these alleged prophecy fulfillments were
more imaginative than factual can easily be seen by examining the whole contexts of the OT
scriptures that they referred to (Ps. 22:18; 69:21). On this issue, Babinski scored another
important point in his letter to Fishbeck through several quotations that underscored the
absurdity of believing that a miracle of this magnitude would have been omitted not just from

Volume 1990 - 2002 Issue


Page 191 of 2049
Skeptical Review Edited by Farrell Till
the other gospel accounts but also from alleged prophecies of the crucifixion. A particularly
significant one was from Christianity's old nemesis Thomas Paine:

Matthew concludes his book by saying that when Jesus expired on the cross, the rocks rent
the graves open, and the bodies of many of the saints arose; and Mark says, there was
darkness over the land from the sixth hour until the ninth. They produce no prophecy for this;
but had these things been facts, they would have been a proper subject for prophecy, because
none but an almighty power could have inspired a foreknowledge of them, and afterwards
fulfilled them.

Since then there is no such prophecy, but a pretended prophecy of an old coat ["They parted
my garments among them..."], the proper deduction is, there were no such things... (An
Examination of the Passages in the New Testament... Called Prophecies concerning Jesus
Christ, pam., 1807).

Bible believers boast that Thomas Paine's best known work, The Age of Reason, has been
repeatedly and soundly refuted, but in reality his arguments against belief in divine inspiration
of the Bible have never been satisfactorily rebutted. In 1776, he wrote a political tract that he
entitled Common Sense. Bibliolaters would do well to apply that title to the matters referred to
in Paine's pamphlet just quoted. The omniscient Yahweh had his prophets predict such
piddling crucifixion events as casting lots for the Messiah's garments and giving him vinegar
on a sponge but didn't have the prophets predict an earthquake that would resurrect many
dead saints! Who can believe it?

Inerrantists may cry argument from silence as loudly as they wish, but in all that he has said
about these resurrected saints, Babinski has addressed some very serious problems in the
inerrancy doctrine. They deserve a response, not flippant dismissal.

(Readers wishing to contact Tom Fishbeck about this subject or his newsletter may do so at P.
O. Box 105, Pasadena, MD 21122. Ed Babinski's address is 109 Burwood Drive,
Simpsonville, SC 29681-8768.)

Did They Or Didn't They?


Farrell Till
In the Autumn 1991 issue, we began a series of articles designed to show inconsistencies in
the resurrection accounts of the four gospels. Gallons of ink have been used in attempts to
explain away these inconsistencies, but some of the variations in the accounts are so
discrepant that only the very gullible could possibly believe the far-fetched scenarios that
bibliolaters have resorted to in trying to harmonize them. Of these discrepancies, none is more
obvious than the variations in what the gospel writers said that the women did to spread word
of the empty tomb after hearing from the angel(s) that Jesus had risen from the dead.

Volume 1990 - 2002 Issue


Page 192 of 2049
Skeptical Review Edited by Farrell Till
Matthew and Luke both said that the women hurried from the tomb to tell the disciples what
the angel(s) had told them (Matt. 28:8; Luke 24:9). Even John, whose version of the story
differs significantly from the synoptic accounts, said that Mary Magdalene ran to find Peter
and "the other disciple" to tell them that the body of Jesus had been taken away (20:2). Three
of the gospel writers, then, clearly depicted the eagerness of the women to report to the
disciples what they had found at the empty tomb.

Mark, however, recorded this part of the story in an entirely different way. After telling of
their encounter with an angel, who told them that Jesus was risen and that they should go tell
the disciples to meet him in Galilee (16:6-7), Mark said that the women were too frightened to
tell others what they had seen:

And they went out, and fled from the tomb; for trembling and astonishment had come upon
them: and they said nothing to anyone; for they were afraid (v:8).
The discrepancy is obvious, but it is even more obvious if Matthew's and Luke's accounts are
juxtaposed with Mark's:
And they departed quickly from the tomb with fear and great joy, and ran to bring his
disciples word (Matt. 28:8).

And they remembered his (Jesus's) words and returned from the tomb, and told all these
things to the eleven, and to all the rest (Luke 24:8-9).

Luke's account even recorded an alleged conversation between Jesus and two disciples (on
resurrection day) in which one of the disciples said that the women had reported finding the
tomb empty:
Moreover, certain women of our company amazed us, having been early at the tomb; and
when they found not his body, they came, saying, that they had also seen a vision of angels,
who said that he was alive (24:22-23).
So the facts in this matter are apparent enough: three gospel writers said that the women ran
to report the empty tomb; one said that they were so frightened by what they had seen that
"they said nothing to anyone." A rule of evidence noted in an earlier article in this series
("The Resurrection Maze," Spring 1992, p. 12) states that two or more contradictory
statements cannot all be right. So who was right in the way this part of the resurrection story
was told? Were Matthew, Luke, and John right in saying that the women ran to report the
empty tomb to the other disciples? Or was Mark right when he said that they were so
frightened that they said "nothing to anyone"? Did they tell anyone what they had seen or
didn't they? That's the problem that inerrantists must resolve.

In my debate with Bill Jackson, he presented a completely speculative solution to the


problem:

(T)he women told no man they met by the way, but the accounts are correct in that they told
the apostles (Jackson-Till Debate, TSR edition, p. 63).
I say that this solution is speculative for the simple reason that it assumes something that is
not explicitly stated in the text. Mark did not say, "(A)nd they said nothing to anyone on their
way to find the disciples, for they were afraid"; he said that they said nothing to anyone,
period. The Church of Christ, which Mr. Jackson preached for, prides itself on "speaking

Volume 1990 - 2002 Issue


Page 193 of 2049
Skeptical Review Edited by Farrell Till
where the Bible speaks and being silent where the Bible is silent." However, when Bible
inerrancy is at stake, Church-of-Christ preachers will speak volumes on matters that the Bible
is clearly silent on. This is just one example of their willingness to break biblical silence.

At this point, inerrantists will usually argue that Mark did say that at least one of the women
reported what she had seen at the tomb:

Now when he was risen early on the first day of the week, he appeared first to Mary
Magdalene, from whom he had cast out seven demons. She went and told them that had been
with him, as they mourned and wept. And they, when they heard that he was alive, and had
been seen of her, disbelieved (16:9-11).
Such appeals as this, however, ignore completely the question of authenticity. In scholarly
circles, Mark 16:9-20 is known as the "Marcan Appendix," because there are sound reasons
for believing that the author of Mark did not write this passage. Textual evidence indicates
that as far as original materials are concerned Mark should end at verse 8 with the statement
about the women being too afraid to tell others what they had seen. Verses 9-20 were redacted
by a later scribe.

My own edition of the American Standard Version affixed this footnote at the beginning of
verse 9: "The two oldest Greek manuscripts, and some other authorities, omit from ver. 9 to
the end. Some other authorities have a different ending to the Gospel." My NIV edition has a
bracketed statement between verses 8 and 9: "The most reliable early manuscripts and other
ancient witnesses do not have Mark 16: 9-20." Of the 17 versions of the New Testament in
my personal library, 15 of them have reference notes to tell readers that this ending to Mark
was not in the oldest and most reliable manuscripts.

One of the early manuscripts that did not include the Marcan Appendix was Codex Sinaiticus
(4th-century A.D.), which ended Mark's gospel at 16:8. In Secrets of Mt. Sinai, James Bentley
made this observation about the omission of the Marcan Appendix in Codex Sinaiticus:

The scribe who brought Mark's Gospel to an end in Codex Sinaiticus had no doubt that it
finished at chapter 16, verse 8. He underlined the text with a fine artistic squiggle, and wrote,
"The Gospel according to Mark." Immediately following begins the Gospel of Luke (p. 139).
Codex Sinaiticus is the only ancient Greek manuscript that contains the entire New
Testament. The fact that it did not include the Marcan Appendix clearly suggests that the 4th-
century scribes who copied it had before them a version of Mark that ended with 16:8. In the
foreword to Bentley's book (p. 6), the renown pseudepigraphic scholar James H. Charlesworth
pointed out that Codex Syriacus (a 5th-century translation), Codex Vaticanus (mid-4th
century), and Codex Bobiensis (4th- or 5th-century Latin) are all early manuscripts that
exclude the Marcan Appendix. In addition to these, approximately 100 early Armenian
translations, as well as the two oldest Georgian translations, also omitted the appendix
(Bentley, p. 179). Manuscripts written after Sinaiticus and Vaticanus have been found that
contained the Marcan Appendix but with scribal notes in the margins that said the verses were
not in older copies; others have been found that had dots or asterisks by the verses in the
Marcan Appendix as if to signal that they were in some way different from the rest of the text
(Bentley, p. 179). These facts give us compelling reasons for suspecting that the Marcan

Volume 1990 - 2002 Issue


Page 194 of 2049
Skeptical Review Edited by Farrell Till
Appendix was indeed the redaction of a scribe who considered Mark's omission of
postresurrection appearances to be an inadequate way to end the gospel.

In addition to the Marcan Appendix, some manuscripts ended Mark's gospel with other
variations. Codex Washingtonensis (late 4th or early 5th century A.D.), for example, included
the addition to 16:14 that is known as the Freer Logion. It is the underlined statement added to
the following quotation of verse 14:

Later he appeared to the eleven themselves as they were sit- ting at the table; and he
upbraided them for their lack of faith and stubbornness, because they had not believed those
who saw him after he had risen. And they excused themselves, saying, "This age of
lawlessness and unbelief is under Satan, who does not allow the truth and power of God to
prevail over the unclean things of the spirits. Therefore reveal your righteousness now"--thus
they spoke to Christ. And Christ replied to them, "The term of years of Satan's power has
been fulfilled, but other terrible things draw near. And for those who have sinned I was
handed over to death, that they may return to the truth and sin no more, that they may inherit
the spiritual and imperishable glory of righteousness that is in heaven" (NRSV).
Other manuscripts added to verse 8 still another but much shorter ending than the Marcan
Appendix: "And all that had been commanded them they (the women who had gone to the
tomb--FT) told briefly to those around Peter. And afterward Jesus himself sent out through
them, from east to west, the sacred and imperishable proclamation of eternal salvation"
(NRSV), to which even other manuscripts added Amen.

If anything is clear from all this it should be that the ending to Mark's gospel has undergone
considerable editing. What the original ending actually was may now be permanently lost in
the wake of all this scribal tampering, but the scholastic consensus is that none of the variant
endings-- the Marcan Appendix, the Freer Logion, and the "short ending"--were the work of
the original writer. The reasons for that consensus are summarized in the following quotation
from The New Jerome Biblical Commentary:

The longer ending, traditionally designated Mark 16:9-20, differs in vocabulary and style
from the rest of the Gospel, is absent from the best and earliest mss. now available, and was
absent from mss. in patristic times. It is most likely a 2nd-cent. compendium of appearance
stories based primarily on Luke 24, with some influence from John 20.... The so-called
shorter ending consists of the women's reports to Peter and Jesus' commis- sioning of the
disciples to preach the gospel. Here too the non- Marcan language and the weak ms. evidence
indicate that this passage did not close the Gospel.

The so-called Freer Logion in Codex W at 16:14 of the longer ending is a late gloss aimed at
softening the condemnation of the disciples in 16:14. All the endings attached to Mark in the
ms. tradition were added because scribes considered 16:1-8 inadequate as an ending (p. 629,
emphasis added).

The stylistic and vocabulary differences referred to in this quotation are apparent even in
English translations of the variant endings, but even without this consideration, suspicion is
cast onto their authenticity by (1) the obvious attempt to reconcile Mark's ending with Luke's

Volume 1990 - 2002 Issue


Page 195 of 2049
Skeptical Review Edited by Farrell Till
and John's accounts of postresurrection appearances and (2) the inconsistencies between the
appendix and what Mark had said earlier in the chapter.

As noted in previous articles, Matthew, Mark, and Luke all referred to women (plural) who
went to the tomb and found it empty. Luke mentioned three by name and referred to "other
women" who were on the scene (24:10), and even Mark specified that at least three women
(Mary Magdalene, Mary the mother of James, and Salome) were there. However, after
declaring that the women "said nothing to anyone, for they were afraid" (v:8), chapter 16 of
Mark suddenly begins to read like John's version, which focused on Mary Magdalene's role in
the story: "Now when he was risen early on the first day of the week, he appeared first to
Mary Magdalene, from whom he had cast out seven demons" (v:9). If "Mark" really wrote
this verse, one has to wonder why, after having said that at least three women had gone to the
tomb, and seen the angel, and heard the angel's message to go tell the disciples, he would have
said that Jesus appeared first to Mary Magdalene. If other women were there, why wouldn't
they have all seen him? Indeed, Matthew claimed that they did all see him (28:9). So this 9th
verse reads suspiciously like a statement written by someone wanting to give a twist to the
story, as Mark had begun it, that would make it at least a little more compatible with John's
version.

If this was the redactor's intention, he failed miserably, for he later said that Mary Magdalene
"went and told them that had been with him, as they mourned and wept, (a)nd they, when they
heard that he was alive, and had been seen of her, disbelieved" (vv:10-11). This deviates
significantly from John's version, whose Mary Magdalene went not to tell the disciples that
Jesus was risen and that she had seen him but to say, "They have taken away the Lord out of
the tomb, and we know not where they have laid him" (20:2). Indeed, John's Mary never saw
Jesus until she had returned to the tomb, and even then she didn't recognize him. She thought
he was the gardener (20:14-15)!

The redactor of the Marcan Appendix went on to say, "And after these things, he was
manifested in another form unto two of them, as they walked, on their way into the country"
(v:12). This was surely an allusion to Luke's account of the appearance Jesus made to the two
disciples on the road to Emmaus (24:13-27), at which time he wasn't recognized until he sat
down with the disciples and broke bread with them (vv:28-31). In telling this, however, the
redactor again bungled his attempt to harmonize Mark's ending with other postresurrection
accounts, because he said that after Jesus appeared to these two disciples, "they went back and
told the rest, but they did not believe them" (vv:12-13, NRSV). This disagrees with Luke's
version of the report that the Emmaus disciples made to the apostles. Luke said that when
they realized they had seen Jesus, the disciples from Emmaus "rose up that very hour, and
returned to Jerusalem, and found the eleven gathered together, and them that were with them"
(24:33). However, Luke's eleven did not disbelieve the report of the Emmaus disciples. Even
before the two from Emmaus reported that they had seen Jesus, the eleven said to them, "The
Lord is risen indeed, and hath appeared to Simon" (v:34). Only after this did the Emmaus
disciples tell "the things that happened in the way, and how he was known of them in the
breaking of the bread" (v:34). So why would the apostles have disbelieved the report of the
Emmaus disciples if by then they themselves were proclaiming that Jesus had risen and
appeared to Simon?

Volume 1990 - 2002 Issue


Page 196 of 2049
Skeptical Review Edited by Farrell Till
What we have in the Marcan Appendix, then, is an obviously bungled attempt to harmonize
the ending of Mark's gospel with other accounts of postresurrection appearances. The failure
is so apparent that the authenticity of the appendix must be rejected. So the did-they-or-didn't-
they problem is still with us. As far as we know, "Mark" wrote nothing about postresurrection
appearances and possibly ended his gospel at 16:8. At that point, he said that the women ran
from the tomb so frightened that they "said nothing to anyone." Matthew, Luke, and John all
disputed that. So what is the truth in this matter? Did the women go tell the disciples what
they had seen or didn't they? Both versions of the story can't be right.

Somebody has to be wrong, and the inerrantists can take their pick. The version of Matthew,
Luke, and John or the one by Mark--it doesn't matter. If either version is wrong, then the
Bible is not inerrant. One thing is sure: the four resurrection accounts are certainly not
inerrant.

Reader Reaction
We receive many interesting letters that, regrettably, we have no space to print and often no
time to respond to. In this issue, we are publishing three letters that we believe will be of
particular interest to our readers.

I have been reading your debates, and they are great! You really put your opponents to shame,
and all they can do is use ad hominem attacks. I have a couple of comments to make on your
literature....

In Mac Deaver's article (Vol. 2, Num. 3, p. 5), he said God would not tolerate graven images.
If I read correctly, Moses was instructed, by God, to make the ark of testimony (Ex. 25:10-
22). On the mercy seat were to be "two cherubim of gold." Wouldn't this violate the
commandment against graven images? Moses said that he made the ark (Dt. 10:3), but he also
said that "Bezaleel made the ark" (Ex. 37:1). Who really made it?

"Moses made a serpent of brass" (Num. 21:9), which was destroyed by Hezekiah about 750
years later, and which "the Children of Israel did burn incense to" (2 Kings 18:4). These
cherubim and the serpent sound like violations of Deuteronomy 5:8, don't you think?

Another comment, this one made by Steve Gunter in volume two, number 4, page 7, concerns
Genesis 3:20: "Adam called his wife's name Eve, because she was the mother of all living."
At the time this statement was alleged to have been said, there were only two people on earth.
Adam had been formed from dust, and Eve had been made from Adam's rib. Of whom was
Eve the mother?

On the subject of failed land promises, look at Hebrews 11:8-9. The writer says the land was
to be an inheritance, and Isaac and Jacob were heirs with Abraham. Verse 39 clearly says that
all the heroes of Hebrews 11 "received not the promise." Romans 8:16-17 calls believers

Volume 1990 - 2002 Issue


Page 197 of 2049
Skeptical Review Edited by Farrell Till
"heirs of God, and joint-heirs with Christ." Here is the problem! If you are an heir, you must
wait for the death of the one who made you an heir. If you are an "heir of God," you must
wait for God to die.

One final comment concerns the Book of Daniel. Fundies think it was written in the 6th
century B.C., but scholars say it was written about 165 B.C. The easiest way to resolve it is to
ask why Daniel was not among the Books of the Prophets that became scripture about 200
B.C. The answer is simple: it hadn't been written yet. The Book of Daniel surely would have
been included if it was as old as they believe.

I think the main problem with Christians is in taking the Jewish Bible and presuming to tell
the Jews what it means. If only they would ask the Jews to explain their book, many silly
prophecy fulfillments would disappear.

(This letter was received from Fred Acquistapace, the author of Miracles That Never Were.
The writer and information about his book can be obtained by writing to him at 3505 Stony
Point Road, Santa Rosa, CA 95407.)

In replying to your argument that the Bible makes God out to be a liar, Michael Hughes used
a revealing phrase occasionally employed by Bible-believing Christians. I've underlined the
phrase in question:
Of course, one can be like those Israelites and believe (or do) that which is right in their own
eyes....
At the heart of this phrase (which originally referred to the anarchy prevailing in the time of
the biblical judges) lies an enormous conceit inherent in many religions and especially in
Bible-believing Christianity. In effect, the Bible-believing Christian is saying that his Bible,
or rather his interpretation of it, is the standard by which all truth must be measured. People
who dare to think for themselves are, of course, guilty of doing that which is right in their
own eyes, meaning that they have rebelled against "God's truth."

This subtle put-down is answered by noting that the conscientious, Bible-believing Christian
always does that which is right in his or her own eyes. They are no different from pagans,
atheists or humanists in that respect. If one acts according to conscience, Bible or no Bible,
then one necessarily does what is right in one's own eyes. Thus, as we penetrate to the core of
this little code-phrase, we see that it is a subtle vehicle to put down other beliefs without the
benefit of analysis. Needless to say, the privileged position which Bible believing Christians
generously assign to themselves and their Bible, at the expense of everyone else, is confined
to their own circles. Such claims are rightly rejected by the majority of the world's people.
Neither do they find support among mainstream Bible scholars who know the Bible all too
well.

In asserting that the world's many religions and philosophies are false save his very own, the
Bible believing Christian has, in the final analysis, demonstrated his own intellectual
bankruptcy.

Volume 1990 - 2002 Issue


Page 198 of 2049
Skeptical Review Edited by Farrell Till
(This letter came from Dave Matson, 330 South Hill Avenue, Pasadena, CA 91106. His
article "A Very Great City" appeared in the Spring 1992 issue of TSR.)

The third letter was written not to us but to someone who has frequently been mentioned in
TSR.

Dear Mr. Jackson:

Thank you for the copy of your editorial on Mr. Farrell Till, which you sent me in reply to my
recent letter. I am sorry to say that I am baffled by it.

You give your readers the impression that it is you who is being asked to provide a forum for
Mr. Till's views in your publication. Isn't it the truth that it is Mr. Till who is offering you
room for your views on the Bible in his publication? If Mr. Till is, as you say, "propagating
error," then why don't you jump at the chance to propagate truth?

It is all very well to preach to the converted, as I suppose you do, but if a shepherd has a
hundred sheep and one of them has gone astray, does he not leave the ninety-nine on the
mountains and go in search of the one that went astray (Matt. 18:12)? But this does not apply
to Mr. Till alone; it applies to each and every one of the readers of his publication. If you have
the truth, you should not hide it in a bushel basket or under the bed; you should take
advantage of every opportunity that is given to you to proclaim it to the world--even from
within the camp of the enemy. If God is with you, who can be against you?

I look forward to seeing your presentation of the truth in Mr. Till's publication in the near
future. I hope you are not ashamed of the Son of Man or his words or he will be ashamed of
you when he comes again in his glory (Luke 9:26).

A Two-Thousand-Dollar Oppurtunity
The last letter, of course, was written to Wayne Jackson, the editor of Christian Courier, a
publication that we have often quoted in TSR. It was written by Ralph Nielsen, 334 Lauder,
Moscow, ID 83843-2514, in response to Jackson's attempt to explain to Nielsen why he
refuses TSR's offer of publishing space to respond to our articles about his absurd inerrancy
views.

As a reader of Mr. Jackson's paper, I have noticed that he sometimes solicits contributions to
keep his paper solvent, so we welcome the opportunity to let him know how he can give
Christian Courier a $2,000 shot in the arm. Mr. Nielsen has had a longstanding offer of a
$1,000 reward to anyone who can take all accounts of the resurrection in the four gospels,
Acts, and 1 Corinthians 15, and write a single narrative in which he includes every event and
detail mentioned in the separate accounts and do so without omitting anything or injecting

Volume 1990 - 2002 Issue


Page 199 of 2049
Skeptical Review Edited by Farrell Till
inconsistency, contradiction, or purely speculative materials into the narrative. To sweeten the
pot a little, The Skeptical Review will add another $1,000 to Mr. Nielsen's offer.

Wayne Jackson constantly proclaims the "uncanny accuracy" of the Bible, so this offer will
afford him the opportunity to demonstrate that and at the same time pick up an easy $2,000
that he can apply to his publishing needs or else spend as he sees fit. The offer is open to
anyone who wants to go for it. Wayne Jackson, Bert Thompson, W. Terry Varner, Jerry
Moffitt, Roy and Mac Deaver, Michael Hatcher, Buster Dobbs, Jerry McDonald, Michael
Hughes, Gleason Archer, Steve Gunter, Bill Lockwood, Robert Morey, William Bischoff,
John Robbins, Tom Fishbeck, Charles Provan, James White, Herman Otten--all of you
inerrancy defenders who receive The Skeptical Review, where are you?

Computer Bulletins
We receive frequent requests for permission to use TSR materials in computer bulletins.
Anyone wishing to do this need not write for permission. Nothing in TSR is copyrighted,
specifically because we want the information to be spread as much as possible. We are
already receiving subscription requests from those who have seen our articles in computer
bulletins.

We will copy all TSR articles to high density disks in either Wordstar 6 or WordPerfect 5.1
format for anyone wishing to use them in computer bulletins. Include $1 for return postage
with your blank disk.

Volume 1990 - 2002 Issue


Page 200 of 2049
Skeptical Review Edited by Farrell Till

The Skeptical Review


Volume Three, Number Four
October/November/December 1992
Farrell Till, editor

• Truth Is Its Own Excuse for Being


Many readers have written to say that we have a duty to offer our readers a better
alternative than belief in the divine inspiration of the Bible. But what they don't
understand is that the exposure of error inherently leaves a superior alternative in its
wake-- namely, truth.

• Why I Believe in the Inerrancy of the Scriptures


Dave Miller defends the doctrine of biblical inerrancy.

• Bible Inerrancy: A Belief Without Evidence


Farrell Till's rebuttal to Dave Miller's defense of biblical inerrancy.

• A Typically Inadequate Response


Farrell Till critiques Bill Lockwood's attempt to resolve the problem of Sarah's alleged
power to make "a deposit of semen" in Hebrews 11:11.

• Sarah's Power to Conceive: A Response (II)


Bill Lockwood's attempt to rebut Farrell Till's article, "A Typically Inadequate
Response."

• Our Own Effort


One attempt to win Ralph Nielsen's thousand dollar prize by reconciling the
resurrection accounts ends in defeat.

Volume 1990 - 2002 Issue


Page 201 of 2049
Skeptical Review Edited by Farrell Till

Truth Is Its Own Excuse For Being


In the summer edition, we published Bill Lockwood's response to the problem of Sarah's
seminal emission that was raised by a Greek expression used in Hebrews 11:11. Another
exchange on this subject begins on page 9 of this issue. In his original rebuttal, Lockwood
urged the editor "to open the scope of his paper to defend his ONLY recourse after he rejects
the Bible--agnosticism." "Let his readers see," Lockwood continued, "what he offers in
return."

To say the least, we found this to be a perplexing statement. Why, for example, would
agnosticism be the only recourse to someone who rejects the Bible? Couldn't one reject the
Bible to embrace the Koran or the Avesta or Buddhism or New Age philosophy? We won't
deny that agnosticism is an appropriate and logical alternative to the Bible, but it is certainly
not the ONLY recourse that one has after rejecting the Bible. Apparently, Mr. Lockwood
didn't think this statement through before he passed it along to us.

The intent of the statement, however, was to challenge us to defend agnosticism by offering
our readers something to replace what we urge them to abandon, and he is not the first to
present this challenge. Many others have written to say that we have a duty to offer our
readers a better alternative than belief in the divine inspiration of the Bible. The challenge
implies that the iconoclast has some kind of moral responsibility to formulate an alternative
philosophical system that is superior to the one that he has proven false.

What these critics and challengers don't seem to understand is that the exposure of error
inherently leaves a superior alternative in its wake. That superior alternative is truth, because
it is always better to be right than wrong, to believe a truth than to believe a falsehood. On
this and related issues, H. L. Mencken. the American critic and satirist, once said this:

I believe that religion, generally speaking, has been a curse to mankind--that its modest and
greatly overestimated services on the ethical side have been more than overborne by the
damage it has done to clear and honest thinking.

I believe that no discovery of fact, however trivial, can be wholly useless to the human race,
and that no trumpeting of falsehood, however virtuous in intent, can be anything but
vicious....

I believe that the evidence for immortality is no better than the evidence for witches, and
deserves no more respect.... I believe in complete freedom of thought and speech, alike for the
humblest to the mightiest, and in the utmost freedom of conduct that is consistent with living
in an organized society.

But the whole thing, after all, may be put very simply. I believe that it is better to tell the truth
than to lie. I believe that it is better to be free than to be a slave. And I believe that it is better
to know than to be ignorant.

Volume 1990 - 2002 Issue


Page 202 of 2049
Skeptical Review Edited by Farrell Till
The transcendentalist poet Ralph Waldo Emerson said in his poem "The Rhodora" that "if
eyes were made for seeing then beauty is its own excuse for being." We believe that the same
principle applies to truth. The truth needs nothing to justify it existence. Although falsehood
may be infinitely more appealing and comforting than the truth, falsehood is never better than
truth.

We believe, then, that we have met Mr. Lockwood's challenge. We have told him and our
other readers the truth about the Bible, and no matter how destructive to cherished beliefs that
truth may be, it is far better than the falsehood that deludes people into believing that the
Bible is a divine book that was given to guide man into an eternal paradise. After all, what
could be more cruel than to live a life of expectation only to have that expectation prove
false? The fact that the expecter may never know that the expectation was false changes
nothing. He still lived a life that was based on lies. How could that possibly be better than the
truth?

Why I Believe in the Inerrancy of the


Scriptures
Dave Miller
"Inspiration" refers to the origin of the Bible, i.e., that it is "God-breathed" (II Tim. 3:16).
Several terms are used to describe the nature of this inspiration. "Plenary" means "full" or
"complete" and refers to the fact that inspiration was completely adequate to accomplish the
task of giving God's will to man in written form in all its parts. "Verbal" means that divine
superintendence extended to the verbal expression of the thoughts of the writers. "Infallible"
means that the Scriptures never deceive nor mislead. "Authoritative" means that the Bible is
binding on all people and that all people will give an account for how they lived in light of its
teaching.

THE IMPORTANCE OF BIBLE INERRANCY


The purpose of this article is to discuss the concept of inerrancy. "Inerrant" means "wholly
true" or "without mistake" and refers to the fact that the biblical writers were absolutely
errorless, truthful, and trustworthy in all of their affirmations. The doctrine of inerrancy does
not confine itself to moral and religious truth alone. Inerrancy extends to statements of fact,
whether scientific, historical, or geographical. The biblical writers were preserved from the
errors that appear in all other books.

The original Hebrew and Greek autograph copies of the Bible were inerrant. Certainly the
copies of copies which have come down to us contain errors common to the craft of the
copyist as do all English versions. However, with diligent study, we can ascertain the original
words of the inspired writers. Consequently, the doctrine of inerrancy applies to the biblical
text in our day as well--insofar as the Bible has been accurately translated.

Volume 1990 - 2002 Issue


Page 203 of 2049
Skeptical Review Edited by Farrell Till
Inerrancy is fundamental to the doctrine of biblical authority. Packer wrote, "Only truth can
be authoritative; only an inerrant Bible can be used... in the way that God means Scripture to
be used."1 If the Bible contains mistakes, then it is unreliable as a true guide to matters of
salvation. If mistakes exist in one part, mistakes may just as easily exist in another part. If the
Bible is a mixture of truth and error, then it is like any other book and simply not deserving of
any special attention.

If the doctrine of inerrancy is not true, then the Bible lacks the very criteria and credentials
necessary for authenticating its divine origin. Human beings would be incapable of
distinguishing between it and all other religious books which seek acceptance by men (e.g. the
Koran, Book of Mormon, the Vedas). If the biblical writers demonstrate incompetency and
fallibility in matters of ordinary knowledge where uninspired humans can check their
credibility, then their infallibility in all other areas is discredited. As Archer noted, "If that
revelation is to come in a usable and reliable form... it must come in an inerrant form."

Since many books claim divine origin, we human beings must be able to recognize whether a
book is in fact the word of God. Our reasoning faculties must be sufficiently competent to
consider the evidence of inspiration and inerrancy, applying the law of contradiction and other
self-evident rules of logic in order to ascertain whether the Bible is consistent with a
supernatural origin. If the Holy Spirit is responsible for what the biblical writers wrote, and if
the Bible contains errors in historical details, then the Holy Spirit is the author of error. If the
Scriptures are not inerrant and completely trustworthy, then God himself is equally
untrustworthy.

THE BIBLICAL CLAIM FOR INERRANCY


In order to hold the conviction that the Bible is the word of God, one must also hold that the
Bible is inerrant--for such is the claim made throughout the Bible for itself. Numerous
passages explicitly affirm inerrancy in all apostolic utterances, including both what to say and
how to say it (Matt. 10:17-20; Mk. 13:11; Lk. 12:12; 21:12-15; Jn. 14:16-17,26; 16:12-13;
Acts 1:5,8). Jesus gave his stamp of approval to the entirety of the Old Testament, even down
to the "jot and tittle" (Matt. 5:18).

Passages like II Timothy 3:16, Hebrews 1:1-2, I Peter 1:10-11, and II Peter 1:21 attribute the
utterances and writings found in both Old and New Testaments to God-- though conveyed by
human authors. When Jesus said "scripture cannot be broken" (Jn. 10:35), he was affirming
Scripture's indefectible nature in even its most casual phrases. The Bible makes no distinction
between "moral" or "religious" truth on the one hand and "historical" or "scientific" truth on
the other. Paul alluded to Adam and Eve as literal, historical persons (I Cor. 11:8-9; I Tim.
2:13-14). Jesus treated Jonah in the great fish, the Flood, Adam and Eve, and Abel as
historical fact (Matt. 12:40; 24:38-39; 19:4-5; Luke 11:51). Indeed, the fundamental facts of
the gospel itself are rooted in and inextricably bound up with history!

THE EVIDENCE OF INERRANCY


Since the Bible claims to be inerrant, what proof exists to justify such a claim? No book but
the Bible has weathered the perpetual assaults of infidels and skeptics in their unceasing
efforts to document errors and contradictions within its pages. Despite these relentless attacks,

Volume 1990 - 2002 Issue


Page 204 of 2049
Skeptical Review Edited by Farrell Till
the Bible has consistently been vindicated and demonstrated to possess the unequaled
characteristic of internal harmony, accuracy, and consistency.[3]

No other literary production in the history of the world has been subjected to such meticulous
scrutiny with the expressed purpose of identifying discrepancies. Yet all the critics of all the
centuries have not succeeded in verifying even one legitimate criticism. Many charges have
been advanced, but in every case the alleged contradiction or error has been successfully
explained or, in those areas where adequate information is currently unavailable, sufficient
alternative explanations have been presented to dispel the credibility of the charge.

Examining specific examples of the Bible's unparalleled accuracy demonstrates its


inerrancy.[4] Such an undertaking is expansive, but the investigation is made easier by
classifying alleged discrepancies according to three types.[5]

(1) The first category, the Bible's historical and geographical credibility, has been consistently
validated in every case where sufficient knowledge is available to modern investigation. For
example, Hodge and Warfield noted that the New Testament alludes to the names of some 30
different people, between 40 and 50 countries, about the same number of foreign cities, and
36 Syrian and Palestinian towns.[6] The great majority of these have been eventually
identified, vindicating the Bible's strict attention to accurate detail.

Lewis identifies 44 Old Testament and 17 New Testament persons for whom inscriptional and
coinage archaeological confirmation exists with an additional 11 cited in literary sources for a
total of 63 biblical figures historically authenticated.[7] McGarvey aptly summarizes the
massive amount of confirmatory evidence corroborating biblical inerrancy:

Whether its writers speak of their own or of foreign lands, they always speak with faultless
accuracy, so that their angus-eyed critics for two thousand years have not been able to detect
them in an error. This accuracy extends not only [to] the relative location of places, and to the
points of the compass, but to the most minute details, even to the relative elevations of places
mentioned in the narratives.[8]
(2) A second category is the Bible's internal harmony. Skeptics have long charged that the
writers contradict one another. Yet, once again, in every case the writers have been
exonerated by a more careful examination of the biblical text. The alleged contradictions
between the inspired writers (particularly the synoptic writers) turn out to be supplemental
information concerning the same event which, when put together, forms a cohesive,
harmonious whole; or two different events are being described. Hodge and Warfield
conclude, "It is not rash to declare that no disharmony has ever been proved between any two
statements of the New Testament."[9]

(3) A third category of inerrancy is the New Testament's use of the Old Testament. Objections
to biblical inerrancy in this regard may be summarized as follows:

... quotation appeals to the sense, not the wording, of a previous document and appeals to it
for a definite and specific end; any dealing with the original is therefore legitimate which does
not falsify the sense in the particular aspect needed for the purpose in hand.[10]

Volume 1990 - 2002 Issue


Page 205 of 2049
Skeptical Review Edited by Farrell Till
Lack of space prevents a treatment of sample alleged discrepancies. Suffice it to say that any
honest-hearted person who cares to put in the time and effort to examine the evidence will
come to the same conclusion articulated by Jesus: "Ye shall know the truth, and the truth shall
make you free."

ENDNOTES

1. J. I. Packer, Fundamentalism and the Word of God, (Grand Rapids, MI: William. B.
Eerdmans Publishing Co., 1958), p. 20.
2. Gleason L. Archer, Jr., A Survey of Old Testament Introduction, (Chicago, IL: Moody
Press, 1974), p. 22.
3. For discussion of inerrancy and inspiration, the reader is urged to study the following:
Benjamin B. Warfield, The Inspiration and Authority of the Bible (Philadelphia, PA:
The Presbyterian and Reformed Publishing Co., p. 1974); J. Gresham Machen, The
Christian Faith in the Modern World (1936, rpt. Grand Rapids, MI: William B.
Eerdsman Publishing Co., 1970); R. Laird Harris, Inspiration and Canonicity of the
Bible (Grand Rapids, MI: Zondervan Publishing House, 1969); Bernard Ramm,
Protestant Biblical Interpretation (Grand Rapids, MI: Baker Book House, 1970), pp.
201-214; Wayne Jackson, "Evidence for Bible Inspiration," Reason and Revelation 3
(Feb. 1983), 7-10.
4. For excellent treatments of specific alleged errors, see John W. Haley, Alleged
Discrepancies of the Bible (1874, rpt. Grand Rapids, MI: Baker House, 1977); J. W.
McGarvey, Evidences of Christianity (Nashville, TN: Gospel Advocate Co., 1974);
Archibald A Hodge and Benjamin B. Warfield, Inspiration (1881, rpt. Grand Rapids,
MI: Baker Book House, 1979).
5. See Hodge and Warfield (Inspiration) for the following categories, pp. 45-71.
6. Hodge and Warfield, pp. 45-71.
7. Jack P. Lewis, Historical Backgrounds of Bible History (Grand Rapids, MI: Baker
Book House, 1971), p. 178.
8. McGarvey, p. 28.
9. Hodge and Warfield, p. 55.
10. Hodge and Warfield, p. 64.

(Dave Miller teaches at the Brown Trail School of Preaching, P. O. Box 210667,
Bedford, TX 76095.)

Bible Inerrancy: A Belief Without Evidence


Farrell Till

Volume 1990 - 2002 Issue


Page 206 of 2049
Skeptical Review Edited by Farrell Till
I commend Mr. Miller for an excellent definition of the Bible inerrancy doctrine and perhaps
an even better explanation of the importance of the doctrine to Christianity. Maybe it is a
carry-over from my own fundamentalist background, but I have a much deeper admiration for
Christians who believe in a divinely inspired inerrant Bible than those who believe in a
divinely inspired errant Bible. To the latter, I can only repeat what Mr. Miller said in the
foregoing article: "If the Holy Spirit is responsible for what the biblical writers wrote, and if
the Bible contains errors in historical details, then the Holy Spirit is the author of error" (p. 2).
As Mr. Miller effectively argued, for the Bible to be authoritative, it must be inerrant;
otherwise, man is left with an impractical moral guide, for what good is a moral guide that is
blemished with errors? If the Bible says X, and one can establish that X is an untruth, then
how can he trust anything else it says?

Mr. Miller may have been on track in recognizing the absolute necessity of an inerrant,
"trustworthy" revelation in order to give credibility and authority to a religious system, but he
wandered far afield in his attempt to prove that the Bible provides Christianity with such a
revelation. An entire section of his article was devoted to a discussion of "the biblical claim
for inerrancy," but I have to disagree with his contention that the Bible claims inerrancy,
because it doesn't. Every scripture that Miller cited in this section concerned either promises
to send the Holy Spirit to guide the disciples in what they should say or claims that the
scriptures were inspired of God or that prophets had spoken as they were moved by the Holy
Spirit. However, to say that the Holy Spirit was sent to guide men in what to say or that men
spoke as they were moved by the Holy Spirit is not to say that whatever these men said or
wrote under the direction of the Holy Spirit was inerrant. To arrive at the doctrine of
inerrancy, one must go through the logical process that Miller took us through in his article. A
basic premise of this process is that if the Holy Spirit is omniscient and omnipotent, then any
document that he verbally inspired would have to be inerrant, because an omniscient,
omnipotent deity would be incapable of error.

I would agree with Miller's logic if he could prove three things: (1) an entity known as the
Holy Spirit actually exists, (2) this entity known as the Holy Spirit is both omniscient and
omnipotent, and (3) this Holy Spirit verbally inspired all of the writers of the Bible in
everything that they wrote. Unfortunately for Miller's confidence in Bible inerrancy, these are
all very big ifs, none of which he could actually prove if his life depended on it. This
underscores the major problem in the Bible inerrancy doctrine: it is based on unprovable
assumptions. Any belief founded on assumptions is worthless.

Even if we grant Mr. Miller the first two of his assumptions, he would still have a very high
hurdle to clear in the third one. That hurdle, of course, would be to establish the truth of the
biblical claim that its writers were inspired by the Holy Spirit. A claim is only a claim and
must therefore be examined before its truth can be confirmed. Mr. Miller can never prove the
truth of the biblical claim of divine inspiration. Claims of divinely inspired books are almost a
dime a dozen. The Book of Mormon claims to be a "latter day" revelation from God; the
Avesta claims that it was divinely inspired; the Koran claims that it was revealed to man by
the angel Gabriel. So what evidence can Miller give us to prove that we should accept the
biblical claim of inspiration over all the many others? Christian apologists have tried to give
us such proof, but Miller made no attempt to do so in his article. Like so many Bible
fundamentalists, he just made the claim and expected his readers to accept it. In the

Volume 1990 - 2002 Issue


Page 207 of 2049
Skeptical Review Edited by Farrell Till
publication in which his article originally appeared, he could get away with this, because the
paper is aimed at a predominantly fundamentalist audience. However, more rational readers,
which we believe The Skeptical Review has, will insist on much more than what Mr. Miller
gave them in the reprint of his article.

Miller listed three categories of "alleged"errors in the Bible and declared that the Bible has
"weathered" all attempts by skeptics to prove that these are actual errors. "(T)he Bible has
consistently been vindicated," he boldly asserted, "and demonstrated to possess the unequaled
characteristic of internal harmony, accuracy, and consistency."

This is typical fundamentalist rhetoric. There are hundreds of Bible scholars who would
instantly reject such a claim as this, because their biblical studies have made them aware of
many inconsistencies and discordant themes in the Bible text. Dozens of these have been
identified and discussed in The Skeptical Review, yet Mr. Miller, who is on our mailing list,
has never taken pen in hand to explain to us how that these errors aren't really errors. I have
challenged him to debate the inerrancy issue, but he has never responded to my letters. One
has to wonder why he refuses the opportunity to discuss in public forum a doctrine that he
claims is easily defendable and at the same time absolutely essential to Christianity. Could it
be that his confidence in Bible inerrancy is not as resolute as he pretends when writing to a
sympathetic audience?

Miller asserts that the Bible possesses an "unequaled characteristic of internal harmony" (p.
3). This is a familiar claim that makes good sermon fodder for gullible pulpit audiences, but it
simply isn't true. Admittedly, there is considerable harmony in the Bible, but there is no
reason to see divine intervention in this. The so-called canonical books were selected by
committees and councils of rabbis, clerics, and "church fathers," who discussed and debated
various books and finally selected the ones that were to be considered "inspired" or canonical.
Quite naturally, the theological themes and doctrines of these books were considered before
they were selected, so a high degree of harmony and consistency of themes would be
expected in a compilation that had gone through such a rigid editing process. Anyone who
doubts that the books of the Bible were selected in just a manner as this should read volume
one of The Cambridge History of the Bible. If he should bother to read it, Mr. Miller would
find historical facts about the evolution of the biblical canon that would reduce his miracle of
internal harmony to nothing but sheer ordinariness.

Despite the editing process by which the canonical books were selected, the biblical text is
still fraught with inconsistencies that make Mr. Miller's claim of "unequaled internal
harmony" a myth that is believed only by gullible bibliolaters who haven't bothered to
investigate the claim. As noted in an earlier article ("A Perfect Work of Harmony?" TSR,
Spring 1990, p. 12), whoever wrote 2 Kings 10:30 obviously believed that Jehu's massacre of
the Israelite royal family was the will of Yahweh, but the prophet Hosea just as obviously
disagreed and pronounced a curse upon the house of Jehu to avenge the "blood of Jezreel"
that Jehu shed in the massacre (Hosea 1:4). Apparently, the "inspired" prophets and biblical
writers had their theological and political differences as much as modern-day religious
leaders.

Volume 1990 - 2002 Issue


Page 208 of 2049
Skeptical Review Edited by Farrell Till
Any present day inerrantist would affirm with his dying breath that the book of Ezekiel was
unquestionably inspired of God, yet the rabbis who made the canonical selection were of a
different mind. A bitter controversy surrounded this book before it was finally selected for
inclusion in the Hebrew canon. The rabbis were bothered by chapters 40-48, which contained
information that was difficult to reconcile with the Torah. Ezekiel 46:6 is just one example of
the problems the rabbis had to deal with in these chapters. Here Ezekiel said that the sacrifice
for the new moon should consist of "a [one] young bullock without blemish, six lambs, and a
ram," but the instructions for this same sacrificial ceremony in Numbers 28:11 stipulated two
young bullocks, seven lambs, and a ram." The discrepancy or, if you please, lack of "internal
harmony" is readily apparent to anyone who wants to see it.

At least it was apparent to the rabbis who had to decide whether the book should be
considered canonical. According to Hebrew tradition, Rabbi Haniniah ben Hezekiah retired to
a room with 300 "measures of oil" and worked day and night until he arrived at explanations
that would "dispose of the discrepancies" (The Cambridge History of the Bible, vol. 1,
Cambridge University press, 1970, p. 134). One wonders why such an undertaking as this was
necessary to decide the canonicity of a book that exhibits "unequaled internal harmony."
Could it be that Rabbi Haniniah ben Hezekiah was merely the Bible inerrantist of his day,
who rather than accepting the face value of what was written spent several days searching for
innovative interpretations that would make doctrinally embarrassing passages not mean what
they obvi- ously were intended to mean?

I could discuss many other textual inconsistencies, but these are sufficient to demolish Mr.
Miller's claim of "unequaled internal harmony" in the Bible. This claim has been preached
and preached and preached from fundamentalist pulpits, but it simply is not true.

Mr. Miller placed an exaggerated importance on historical and geographical accuracy in the
Bible. Again, this is a familiar inerrancy argument. Those who point to this feature in the
Bible appear to be arguing that if the Bible has been proven true in some of what it says, then
it must be true in all that it says. The fallacy in this reasoning is evident from the simple fact
that accuracy, historical, geographical, or otherwise, doesn't constitute proof of inspiration.
One can sit in a library all day long and find historical and geographical accuracy in book
after book. No one would assume, however, that their accuracy meant that they had been
divinely inspired. A more reasonable conclusion would be that the authors had researched
their subjects or were by personal experience familiar with the history and geography of the
events and places that they were writing about. Why shouldn't the same be assumed about
historical and geographical accuracy in the Bible?

Despite the fact that there is undeniably some accuracy in the Bible, scholars are now
convinced that many inaccuracies can be found in it. Exodus, Numbers, and Deuteronomy
report that the Hebrews, numbering as many as 2.5 to 3 million, left Egypt, wandered in the
Sinai wilderness for 40 years, and finally invaded and conquered the promised land. Most
biblical scholars and archaeologists doubt the historical accuracy of this biblical story. The
March/April issue of Archaeology magazine declared that neither the exodus nor the conquest
of Canaan happened as recorded in the Bible. "Today's archaeologists are certainly not the
first to challenge the Book of Joshua," said Neil Asher Silberman in the feature article. "Its

Volume 1990 - 2002 Issue


Page 209 of 2049
Skeptical Review Edited by Farrell Till
historical reliability has been a matter of dispute for more than two centuries" ("Who Were
the Israelites?" p. 22).

Bibliolaters love to preach that archaeology has provided amazing confirmation of biblical
accuracy. In my written debate with Bill Jackson, he rashly asserted that "in 140 years of
constant research in the lands of the Bible, archaeologists have yet to find a single fact in
contradiction to what the Bible has said" (Jackson-Till Debate, p. 3), but this claim is a far cry
from reality, as the edition of Archaeology cited above will verify. Speaking at an
archaeological conference at the Royal Ontario Museum, Israeli archaeologist Eliezer Oren
reported that "his efforts at more than 80 sites in the Sinai from 1972 to 1982 had not turned
up any support for the historical accuracy of when the exodus was supposed to have occurred"
(Barry Brown, "Israeli Archaeologist Reports No Evidence to Back Exodus Story," News
Toronto Bureau, Feb. 27, 1988). Oren went on to tell of the discovery of papyrus notes that
reported the sightings of two fugitive slaves. "They were spotted and the biblical account of
2.5 million people with 600,000 of military age weren't?" Oren asked. "This can't be
explained unless you invoke miracles here, and I am a member of the department of
archaeology and not of miracles."

Bible inerrantists have a field day whenever an archaeologist makes a discovery that seems to
confirm something written in the Bible. In complete disregard of archaeological studies that
discredit the biblical record (like those noted above), they leap immediately to the conclusion
that the new discovery confirms the truth of everything in the Bible, when in reality the
discovery proves only that there is some truth in the Bible. Classical scholars once believed
that the Grecian epic of the Iliad was mythology, that no Trojan War had ever occurred, that
the cities of Troy and Mycenae had never even existed. Then the German archaeologist
Heinrich Schliemann, using the Iliad as his guide, discovered and later excavated the sites of
both cities. When this was done, no classical scholars rushed forth to proclaim that the Iliad
was completely inerrant in its original autograph or that it had been inspired by God. They
were intelligent enough to realize that archaeology had proven only that there is some truth in
the Iliad. Bible inerrantists, however, show no such restraint. They want to see confirmation
of one biblical statement as proof that all biblical statements are true. In so doing, they show
their complete lack of objectivity.

Miller asserted that "(m)any charges [of biblical inerrancy] have been advanced, but in every
case the alleged contradiction or error has been successfully explained or, in those areas
where adequate information is currently unavailable, sufficient alternative explanations have
been presented to dispel the credibility of the charge," but the successful explanations that he
alluded to have been successful only to credulously gullible fundamentalists who are
determined to believe in biblical inerrancy no matter how convincing the evidence to the
contrary. Just a look at the endnotes that Miller used to "support" the points he offered in
defense of the inerrancy doctrine is sufficient to discredit his attempt to appear scholarly.
Altogether, he cited ten different books, seven of which were published by Eerdmans, Baker
House, or Zondervan of Grand Rapids, Michigan. All three are notorious for publishing books
slanted to the supposition that the Bible is the "inspired word of God." One of the books was
published by Jack Lewis, who was a professor of Bible and religion at Harding University
when I was a student there. Since he still teaches there, I assume he has not changed the
fundamentalist view of the Bible that he had when I was a student, so I would hardly expect

Volume 1990 - 2002 Issue


Page 210 of 2049
Skeptical Review Edited by Farrell Till
to see anything in a book he authored except the usual inerrancy line. Two of Miller's other
references came from books published by Moody press and the Presbyterian and Reformed
Publishing Company, neither of which has a reputation for unbiased biblical scholarship. The
tenth reference was from an article written by none other than Wayne Jackson, whose illogical
fundamentalist views have been exposed so often in TSR that no further comment is
necessary.

This, then, is the depth of scholarship that Miller finds on his side in the Bible inerrancy
controversy, so when he says that all "alleged" Bible contradictions and errors have been
"successfully explained," this is a claim that must definitely be viewed with suspicion.

As an example of an "alleged" Bible contradiction that has been "successfully explained," let's
just review one that was recently noted in "The Resurrection Maze" (TSR, Spring 1992, p.
13). According to Matthew, Mark, and Luke, Mary Magdalene was in the group of women
who were told by angels at the empty tomb that Jesus had risen "even as he said," and Luke
even said that when the women heard this, "they remembered his words" (24:9). Such
statements as these (aside from the fact that Matthew even claimed that the women saw Jesus,
held him, and worshipped him as they were running from the tomb to tell the disciples what
they had seen, 28:9) definitely indicate that the women left the tomb convinced that Jesus has
risen from the dead. Despite the clarity of these statements, John's account of the resurrection
had Mary saying, after she had found the disciples, "They have taken away the Lord out of the
tomb, and we know not where they have laid him" (20:1).

I suppose Mr. Miller would call this an "alleged" discrepancy that has been successfully
explained, but how has it been explained? In my article cited above, I quoted what Gleason
Archer, a leading apostle of Bible inerrantists, said to explain it:

She [Mary] apparently had not yet taken in the full import of what the angel meant when he
told her that the Lord had risen again and that He was alive. In her confusion and amazement,
all she could think of was that the body was not there; and she did not know what had become
of it. Where could that body now be? It was for this reason that she wanted Peter and John to
go back there and see what they could find out (Encyclopedia of Bible Difficulties, pp. 348-
349).
Mr. Miller may consider this a "successful explanation" of the problem, but I don't and neither
does anyone else whose perspective has not been completely distorted by fundamentalist
nonsense. What is there about a far-fetched, entirely speculative explanation like this that
makes it "successful"? And what about all of the other inconsistencies and discrepancies that
have been noted in the series of resurrection articles that TSR has published. What are the
"successful explanations" to these, and why hasn't some fundamentalist who finds it so easy
to defend Bible inerrancy come forth with them? We will give space to Mr. Miller or any
other fundamentalist who wishes to provide the explanations.

Mr. Miller set out to explain (to an audience already predisposed to agree with him) why he
believed in the inerrancy of the Bible, but all he did was show that he believes in Bible
inerrancy... well, just because he believes in it. He believes that the Bible is God's inspired
word for the same reason that a Moslem believes that the Koran is Allah's inspired word. It is
something that he was taught to believe and that he grew up believing without ever bothering

Volume 1990 - 2002 Issue


Page 211 of 2049
Skeptical Review Edited by Farrell Till
to examine the belief to see if it has any basis in fact. If he wishes to respond to this article,
we will publish it.

A Typically Inadequate Response


Farrell Till
In the summer edition of TSR, Bill Lockwood attempted to resolve the problem of Sarah's
alleged power to make "a deposit of semen," which is the literal meaning of the Greek
expression katabole spermatos that Hebrews 11:11 clearly attributed to Sarah: "By faith even
Sarah herself received power to conceive seed (katabole spermatos) when she was past age,
since she counted him faithful who had promised." As I said in a front-page reference to
Lockwood's response (Summer 1992), it was primarily an ad hominem attack on me that
failed to rebut the major points in my article. I am writing this reply to his "rebuttal" only to
underscore the inadequacy of his "solution" to the problem of Sarah's seminal emission.

Mr. Lockwood tried to make an issue of van der Horst's scholastic reservation in his analysis
of Leviticus 12:2. Because van der Horst used words such as seems, possibility, probable, and
imply in his exegesis of the passage, Lockwood tried to build this into a case of van der
Horst's complete uncertainty about the meaning of the Hebrew original, "When a woman
tazria [produces seed] and bears a male child...." In so doing, Mr. Lockwood conveniently
overlooked the fact that van der Horst clearly said in his exegesis of the passage, "The
causative form, used in Leviticus 12:2, cannot mean anything else but 'make seed'" (emphasis
added). That certainly doesn't sound like doubt.

Mr. Lockwood's problem of course is that he is a Bible fundamentalist, who cannot relate to
the scholar's reluctance to take dogmatic, this-is-what-it-means-and-couldn't-possibly-mean-
anything-else positions that are so typical of fundamentalist mentality. So because van der
Horst introduced his interpretation of Leviticus 12:2 in terms of what it seemed to mean or
probably meant, Lockwood took this as a sign of scholastic pussyfooting. Nevertheless, van
der Horst analyzed the original Hebrew text and concluded that it could not mean "anything
else but 'make seed.'" In response to this, Lockwood said exactly nothing.

Neither did he say anything about van der Horst's references to the Babylonian Talmud that
unequivocally showed that leading rabbis of the time believed that women emitted semen
during sexual intercourse and that Leviticus 12:2 was one of the proof texts that they cited in
support of this belief. These were religious leaders who studied the Jewish scriptures in the
original language, so common sense would indicate that they were eminently more qualified
to pass judgment on what the original text meant than Mr. Lockwood or any of his
fundamentalist colleagues will ever be. As the quotations cited plainly showed, Hebrew
scholars of that time said that Leviticus 12:2 meant that women emitted semen during sexual
intercourse; Mr. Lockwood denies that the text meant this. So whom shall we believe, the
Jewish rabbis interpreting their own scriptures written in their own language or Mr.

Volume 1990 - 2002 Issue


Page 212 of 2049
Skeptical Review Edited by Farrell Till
Lockwood, who maybe took a year or two of Hebrew at a Bible college or school of
preaching?

This traditional Hebrew view of Leviticus 12:2 poses a rather embarrassing problem for Mr.
Lockwood. Even if he could unequivocally prove that Hebrews 11:11 didn't mean to imply
that Sarah received power to make a "deposit of semen" (katabole spermatos), he would then
have to prove that Hebrew scholarship was in error about the meaning of Leviticus 12:2, for
whether a scientific misconception about the female's role in human reproduction was stated
in Hebrews 11:11 or whether it was stated in Leviticus 12:2 doesn't really matter. Either way,
the Bible would have a scientific error in it, and the inerrancy doctrine leaves no room for
errors of any kind. This is the predicament that Mr. Lockwood finds himself in.

In trying to show that katabole spermatos did not mean "a deposit of semen," Lockwood
opened the door to a ton of trouble for his position. He said that the Greek literally meant "the
laying down of seed" (p. 6) and then, astonishingly enough, went on in the same context to
say that "this is an act of a male, not female, in the reproductive process." I couldn't have said
it any better myself, so this admission of his underscores the problem that the passage poses
for the inerrancy doctrine. The depositing of semen or the "laying down of seed" is "the act of
a male, not female, in the reproductive process," but the writer of Hebrews said that Sarah had
received the power to lay down seed or, in other words, to do the reproductive act of a male.
Now perhaps Mr. Lockwood would like to explain to us why someone writing under the
guidance and protection of an omniscient, omnipotent deity could have made such a mistake
as this.

Mr. Lockwood admitted that to translate katabole spermatos as a deposit of semen "remains
an alternative" (p. 6), but he felt that a more accurate translation would be "received power to
establish a posterity." Exactly why he thinks this would be a more accurate translation was
never explained except that he had found some lexicons and commentaries that had so
rendered it. We must wonder, however, about the motives of the lexicographers. Did they
translate the expression this way because linguistic accuracy demanded it, or did they so
translate it to avoid an embarrassment to a presupposed notion that the Bible was verbally
inspired by an omniscient, omnipotent deity? We certainly can't discount the latter.

By Lockwood's own admission, katabole spermatos meant "the laying down of seed," so if
that is what the expression meant in Greek, why would "received power to establish a
posterity" be a more accurate translation? By coincidence, the Bible Review (June 1992, p. 7)
printed a letter from a reader who pointed out that a little known version of the Bible
translated by Julia Smith and published in 1876 had rendered Hebrews 11:11 like this: "By
faith Sarah also herself received power for the laying down of seed, and brought forth during
the time of age, for she deemed him faithful having promised." The letter writer noted that
Smith had stated in the preface of this Bible that her purpose was "to define the Bible word
for word" by providing "the literal meaning of the Bible text" in the original languages.
Furthermore, Miss Smith had translated Leviticus 12:2 to read, "When a woman shall bear
seed and bring forth a male." Here, then, is more scholarly evidence that the Bible, both old
and new testaments, attributed to women a reproductive function that females simply do not
have.

Volume 1990 - 2002 Issue


Page 213 of 2049
Skeptical Review Edited by Farrell Till
To put the issue in perspective, let's suppose that we should discover a book in which it were
said of a man that his faith in God had given him "power to ovulate." We would instantly
know that whoever wrote this book was certainly not inspired by an omniscient, omnipotent
deity, because the author had attributed a reproductive act of the female to a male. If this book
had been written in another language, we could circumvent the problem by translating the
expression so as to have it read "received power to establish a posterity." In so doing, we
would capture at least a general sense of the original and at the same time conveniently
conceal a troublesome scientific error, but we certainly would not have conveyed the exact
meaning of the original, because the poor deluded writer apparently had thought that males
ovulate as females are known to do.

Here then are the obstacles that Mr. Lockwood must surmount in order to give any kind of
credibility to his "solution" to the problem of Sarah's power to have a katabole spermatos: (1)
secular writers in biblical times clearly believed that women emitted semen during sexual
intercourse, (2) prominent Hebrew rabbis shared this belief, (3) prominent Hebrew rabbis
interpreted Leviticus 12:2 to mean that the female emitted semen, and (4) the writer of
Hebrews said that Sarah's faith enabled her to have a katabole spermatos, an expression in
Greek that meant "deposit of semen" or "laying down of seed." All of these together make a
compelling case for believing that the writer of Hebrews made a commonly believed error of
his day in the way that he stated Sarah's reproductive role.

On related matters, Mr. Lockwood said that he had pulled some ancient Near Eastern texts
from his shelves and discovered that the Egyptians had probably learned monotheism from
having witnessed the power of "the one true God of the Universe" during the exodus. Perhaps
he should pull down some more texts and do additional reading. If he takes that advice, he
will learn that the Egyptians had a long tradition in monotheism that antedated by many
centuries the time that the Hebrews were allegedly enslaved in Egypt. Sir Wallis Budge, the
late Keeper of the Egyptian and Assyrian Antiquities at the British Museum, said this in his
book Egyptian Religion (Seacaucus, NJ: University Books, 1959):

A study of ancient Egyptian religious texts will convince the reader that the Egyptians
believed in One God, who was self-existent, immortal, invisible, eternal, omniscient, almighty
and inscrutable; the maker of the heavens, earth, and underworld; the creator of the sky and
the sea, men and women, animals and birds, fish and creeping things, trees and plants, and the
incorporeal beings who were the messengers that fulfilled his wish and word (p. 17).
So when did the Egyptians develop this monotheistic view? Was it after they had witnessed
the power of Yahweh in the plagues that afflicted Egypt? Sir Wallis Budge said no, that "there
is no evidence whatsoever to guide us in formulating the theory that it was brought into Egypt
by immigrants from the East, as some have said.... All that is known is that it existed there at a
period so remote that it is useless to attempt to measure by years the interval of time which
has elapsed since it grew up and established itself in the minds of men..." (p. 18). He went on
to quote Egyptian monotheistic precepts that "were composed as far back as B.C. 3,000" (p.
26). That would be 15 centuries before the time that Lockwood believes Moses led the
Israelites out of Egypt.

On the basis of what this famous Egyptologist claimed about Egyptian religion, if Lockwood
could even prove that the exodus from Egypt ever occurred, we would have every reason to

Volume 1990 - 2002 Issue


Page 214 of 2049
Skeptical Review Edited by Farrell Till
assume that the Hebrews had learned monotheism from the Egyptians rather than vice versa.
If, however, Lockwood should want to try his luck at proving that the Hebrews had instilled
monotheism into Egyptian culture, he might while he is at it try to prove that the exodus even
happened. Archaeology magazine (March/April 1992) reported that modern archaeologists
reject the historicity of the Israelite exodus and conquest of Canaan as they were recorded in
the Bible.

This, then, is the legacy of Mr. Lockwood's fundamentalist view of the Bible. To maintain it,
he must butt heads with practically every branch of biblical scholarship. That is a high price
to pay for a religious belief.

On the matter of scientific "foreknowledge" in the reference to the seed of woman in Genesis
3:15, Lockwood seemed to miss the point entirely. He referred to that "seed" as "the line of
woman" (p. 7), which isn't at all the sense that inerrantists give to it when they claim that
"Moses" demonstrated amazing scientific foreknowledge in the statement. They want us to
believe that the Genesis writer understood by divine revelation that the female contributes a
seed or germ in the act of reproduction and that this is a scientific truth that "modern science
didn't know until fairly recent times" (Bill Jackson, Jackson-Till Debate, p. 3). However, the
many references that van der Horst cited from both secular and Talmudic writings indicate
that this is speculative hogwash. That the female contributes a "seed" in the reproductive
process was widely known at the time; it just wasn't known what kind of seed she contributed.
So even if inerrantists could unequivocally prove that Genesis 3:15 was referring to seed in
the sense of germ cells, there would be no reason to assume that this showed amazing
scientific foreknowledge unless they could prove that "seed" referred to the exact kind of seed
that the female ovulates rather than to the semen that the ancients thought women emitted
during sexual intercourse. I doubt if Lockwood or any inerrantist could prove that.

As for Lockwood's effort to prove that Genesis was written before the Egyptian hymn that
referred to "the germ of woman," two counterpoints are in order. (1) Lockwood's dating of
Genesis runs completely contrary to biblical scholarship. Only hopelessly naive
fundamentalists believe that Moses wrote this book or any of the other four in the Pentateuch.
Traditionally, authorship of the Pentateuch has been credited to Moses, but critical analyses of
the books have identified many problems (too numerous to detail at this time) that are
incompatible with the tradition of a Mosaic authorship. Lockwood wants us to believe that
Moses wrote the book in the 15th Century B.C. Responsible scholarship, however, affixes a
much later date to the final form of the book. (2) Even if Lockwood could prove that the
reference to the "germ of woman" in the Egyptian hymn to the Sun-God was (contrary to all
responsible scholarship) written after the book of Genesis, this would in no way prove that the
idea of a "germ of woman" had come from the seed-of-woman statement in Genesis 3:15.
Neither would it prove that the seed-of-woman statement was an unequivocable reference to
the biological fact of ovulation as the female's role in reproduction. This, however, is the
claim that is made in the scientific-foreknowledge argument based on Genesis 3:15, so this is
what Lockwood would have to prove to salvage the argument. Needless to say, he didn't do
that.

Finally, Mr. Lockwood tried to convince us that there is no inconsistency between the Hebrew
writer's statement that Sarah had "judged him faithful who had promised" that she would

Volume 1990 - 2002 Issue


Page 215 of 2049
Skeptical Review Edited by Farrell Till
make a deposit of semen or conceive or whatever and the story in Genesis 18:9-15, which
presented Sarah as a woman so skeptical that she could have a child at her age that she
laughed when Yahweh made the promise. Why, there was no inconsistency, Lockwood
assured us. Sarah just changed her mind; that's all.

Well, I'd like to know just when this change of mind occurred. When the promise was first
made, even Abraham "fell upon his face and laughed" (Gen. 17:17). "Shall a child be born
unto him that is a hundred years old?" he wondered. "And shall Sarah that is ninety years old
bear?" So the promise made to these near centenarians that they would have a son was viewed
with a great deal of skepticism, not just by Sarah but Abraham too. So if Mr. Lockwood wants
to see skepticism this profound as an indication of great faith, he may of course do so. His
faith in Bible inerrancy requires him to believe things even sillier than this. What I want to
know, however, is why the remarkable change of mind that Lockwood sees as a solution to
this problem was not recorded in the Bible. If the omniscient one saw fit to put Sarah's
skepticism into his holy word, why didn't he tell us about her change of heart, especially if he
was going to have one of his inspired writers make a big point about Sarah's exceptional
faith? Maybe Mr. Lockwood has an answer to that question too.

What always amuses me about off-the-wall solutions like this to textual inconsistencies in the
Bible, when they come from Church-of-Christ sources, is that they run completely contrary to
a cardinal principle that this church stands for. That principle is that one should speak where
the Bible speaks and be silent where the Bible is silent. From having read some of Mr.
Lockwood's writings, I suspect his fundamentalism would make him a subscriber to that
principle. So is offering a completely theoretical explanation for which he has no biblical
support whatsoever his idea of speaking where the Bible speaks? Apparently so, because he,
like Mr. Miller, whose article I reviewed on pages 4-8, belongs to the any-interpretation-will-
do school of hermeneutics. Lockwood really doesn't care how theoretical, speculative, or
unlikely his resolution to the problem of Sarah's apparent lack of faith is. It provides at least a
semblance of "explanation," so as far as he is concerned, it will do.

Bibliolaters who use this approach to defending Bible inerrancy have yet to learn a
fundamental truth: the mere existence of a how-it-could-have-been is no proof that it actually
happened that way. So as far as the Bible record is concerned, Hebrews 11:11 contradicts
Genesis 18:9-15. The one says that Sarah counted God faithful when he promised that she
would "conceive"; the other says that she found the promise downright laughable. Without a
direct statement in the Bible that says Sarah "changed her mind," Mr. Lockwood has no
solution to offer, only speculation.

Sarah's Power to Conceive: A Response (II)


Bill Lockwood

Volume 1990 - 2002 Issue


Page 216 of 2049
Skeptical Review Edited by Farrell Till
Once again I thank Farrell Till for the space to rebut his arguments, and once more I
encourage Till to open his columns to defend the only system that he offers once he has
removed the Bible from the hands of men--agnosticism.

TILL'S DEFEAT
The entire burden of proof in this exchange is upon Till's shoulders. He must demonstrate the
Bible to be in error at Hebrews 11:11. I predicted it would be a load too heavy for his puny
arms, and now we learn that I was not only right but that he has thrown up his hands and
complained that he WON'T DO IT. Well, I am not the least bit surprised. See what he gives
us. He says I need to learn that scholars don't take dogmatic positions-- which is his way of
begging out. When he sees the lexicographers against him, he chirps, "We must wonder about
the motives of the lexicographers." Yes, that's all you agnostics will ever do, run around
"wondering" about everything and trying to induce others to start "wondering"--and then you
leave them, without offering ONE TANGIBLE PROOF of anything. When I poke at Till that
he did not prove his case at Hebrews 11:11, he says that "dogmatic positions are only typical
of fundamentalist mentalities." When he spies all his arrows turned Tillward on the Egyptian
hymn, he challenges: "Lockwood, I want you to prove the exodus even occurred." So you see,
readers, it is good to emphasize at the first of a debate with an agnostic the roles each is to
play, for they always want to "switcheroo" the horses with you. Yes, the "negative horse" is
the only one agnostics like to ride, for when it comes to AFFIRMING a position, they have
not the wherewithal to stay in the saddle. But I'm going to keep setting Till back in his OWN
saddle, no matter how often he must dust off his "breeches."

Notice a couple of examples. On the Egyptian hymn, Till said in his first article that this
"antedates" the book of Genesis and precludes "scientific foreknowledge" of Genesis 3:15. I
showed that this does not antedate Genesis, but here friend Farrell came challenging me to
PROVE that Hebrews instilled monotheism into Egyptian culture and that the exodus even
occurred. Of course, I was not trying to prove all of this but was only rebutting his foolish
assumption that this hymn antedates Genesis and showing that I had a better case of vice-
versa influence on this one.

Now he flees to an Egyptologist and leaves his argument in my hand as far as this hymn is
concerned. Second, I told the readers how it "could have been" on Sarah's faith, offering that
she could have changed her mind for all Till knows about it. But he skedaddles back with this
emphatic point: "The mere existence of a how-it-could-have-been is no proof that it actually
happened that way." No, professor, I never pretended that my suggestion was proof of how it
actually occurred. That's your job. You are to be proving here, and I simply showed everyone
that you have not the ability at this point if the possibility I offered actually exists. But more.
Whether Till realizes it or not, he gave up his entire case right here. Don't miss it. Till's task is
to prove the Bible in error, so I have said from the beginning for all of us to remember that the
mere existence of a possibility of how a passage could read is not proof that it actually IS the
case. But, true to agnostic style, this is all Till has as ammunition--possibilities. But, lo and
behold, he now confesses that possibilities don't prove! Well, if this doesn't beat all! A
promise of a big debate exchange with Mr. Agnostic, and he gives up without a whimper. I
suppose, however, I will stay with it just for the fun of it.

Volume 1990 - 2002 Issue


Page 217 of 2049
Skeptical Review Edited by Farrell Till
Third, Till characterized my piece as "primarily an ad hominem attack" upon him because I
used Farrell's follies as an example to answer ONE small item on Sarah. My guess is that
since this was Till's only argument on Sarah's faith, as far as I could tell, he is just upset about
how easily it was answered. But then he turned right around in the next paragraph and spoke
about Lockwood's problem! If we are to learn from Till, we must suppose that he cannot meet
the argument but wraps himself up in an "ad hominem" attack on me! I declare, I enjoy
debating Farrell.

What's more, I pointed out that van der Horst did not prove his case either, which must be
done to carry the point of this mini-debate, and then Till railed against "fundamentalists" by
saying we just cannot comprehend that scholarship does not take dogmatic positions. Well,
my collegiate professor friend, call it anything you like, paint it any color you wish, berate
some more on how far I am from scholarship, but the fat is in the fire: for if you don't have a
can't-possibly-be-anything-else case, you don't have a case against the Bible. But, of course,
he just doesn't know whether he needs a shut and tight case against the Bible or not, since an
agnostic disclaims certain knowledge. He just cannot know what is a valid argument. Poor
Till, why doesn't he just be good and give up this foolishness of agnosticism? Friends, never
in my wildest dreams did I ever picture Till giving up this way without a good semblance of a
fight, but, alas, he has done so, and everyone can see it.

Till tried to raise from the dead, as far as an argument is concerned, a statement from van der
Horst that said Leviticus 12:2 cannot mean anything else than "to make seed"; thus charging
the account with a falsehood since it referred to the woman, not the man. Do not miss this
point, readers. Till has now changed the focus of his attack. He, for the sake of argument, will
grant me Hebrews 11:11 but said the "embarrassing situation" for me is "to prove that Hebrew
scholarship is in error" when they (anciently) thought this passage proved that women emitted
semen during intercourse. Well, I am indeed embarrassed, to be sure, but only for a man who
wrote an entire piece to prove Hebrews 11:11 is in error but then said on his second round, "If
I were to grant my opponent Hebrews 11:11, he still cannot handle Leviticus 12:2." Do not
forget this change of focus, TSR subscribers. But what is the real problem here? You see, Till
reads an article from an infidelic paper, not doing any of the research himself (while accusing
me of not knowing any Hebrew, to boot!), and the authors that van der Horst quoted to favor
his position on Leviticus 12:2 became the big chiefs and the rest of us just little Indians.

First, I showed already that van der Horst has said his case is NOT PROVABLE, but the
professor then fished out a line that said, "It cannot mean anything else than 'make seed.'"
Naturally, Till overlooked the surrounding sentences and evidently hoped none of you would
read van der Horst for yourself. Van der Horst concluded his analysis of Leviticus 12:2 this
way: "We cannot avoid at least the possibility that the author of Leviticus 12:2 means what he
seems to have written, that is, that a woman can produce semen" (emphasis added). This is a
tentative conclusion, and the professor knows it. Even prior to the "cannot mean" statement,
he said, "Leviticus 12:2 seems to indicate that a woman can produce semen..." (emphasis
added). A POSSIBLE ALTERNATIVE, nothing more, is the CONCLUSION. Now, there is a
good reason for this reservation, as the reader will discover in just a moment, but it is too late
for Till. Little did van der Horst know that an English professor in the north country would
jump up and say, "HE PROVED IT! HE PROVED IT!" But van der Horst did not prove it,
and he tried to warn our little agnostic friend, but he will not listen. Second, Holladay's

Volume 1990 - 2002 Issue


Page 218 of 2049
Skeptical Review Edited by Farrell Till
Concise Lexicon of Hebrew and Aramaic gives "to conceive seed" at Leviticus 12:2 (p. 92).
(See also the ASV and Strong's Concordance.) This is where Till comes in simply to question
the motives of the lexicographers, because he found where some rabbis thought it meant that a
woman "emitted semen."

The Brown, Driver, Briggs, Gesenius Lexicon defines the word zara (Lev. 12:2) in its hipil
form (p. 281-82) as "bear a child." Keil and Delitzsch: "bring forth a child." Thus, it reads, "If
a woman 'brings forth' and bears a man-child, then she shall be unclean seven days." The idea
here includes the birth, not just the conception! This is perhaps why van der Horst remained
more reserved than Till. He saw the other side of the coin that Till refuses to see. But, once
more, we are not finished with the professor yet. In The Pentateuch and Haftorahs (ed. chief
rabbi Dr. Joseph Barry Hertz, Soncino Press, London, Heb. date 5713, 1952, p. 460), the
rendering is, "If a woman be delivered and bear a man-child...." BE DELIVERED! Just as a
plant bears seed, so does the woman. And, incidentally, this is exactly what the verse van der
Horst referred to means (Gen. 1:11-12), bearing seed for further plant life. This means that
Moses at Leviticus 12:2 included for a female the entire process of the conception and
birthing, not just the initial conception. Now, Farrell, do you know more than the rabbis??? Of
course, readers, being an agnostic, Farrell cannot even know about this. Are you not ashamed
of yourself at this point, Mr. Till? And to think, he tried to pit me against the scholars as to
who knows best. Can I not read the books? Can I not comprehend the conclusions stated by
translators? Yes, Farrell has, at least in this debate, reduced himself to proving the Bible
errant because some leading rabbis of the time believed thus-and-so about women. "How art
thou fallen from heaven, O day-star... how art thou cut down to the ground, that didst lay low
the nations!"

HEBREWS 11:11
This was to be Till's main argument, but he used only 1.5 columns out of more than 7 on it.
We need not wonder why. Till tried his favorite ploy, reversing tables on me, by saying,
"Even if you gain Hebrews 11:11, you cannot handle Leviticus 12:2!" After this exchange, I
gather Till will say to his pupils, "Even if Lockwood DOES show that Leviticus 12:2 does not
necessarily imply that women emit semen, he cannot handle this over here." Yes, Till is hard
to pin down at times, because he vacates the lines of defense before the enemy arrives.

Till's first and ONLY effort to rebut my article came this way:

Lockwood opened the door to a ton of trouble for his position. He said the Greek literally
meant "the laying down of seed" (p. 6) and then, astonishingly enough, went on in the same
context to say that "this is an act of a male, not a female, in the reproductive process."
Till admitted, "I could not have said it better myself," and wanted me to explain all of this.

I hate to be such a party-pooper as I remove the jubilant smile from Farrell's lips, this being
the only good point he made in the whole article, but I was not here conceding. I was merely
rehearsing the primaries of van der Horst's work before I reviewed it in the next paragraphs.
Yet this is Till's only defense in the entire review, what he thinks I admitted! When I offered
alternate translations of katabole spermatos, Till ran to this again: "Why would 'receive power

Volume 1990 - 2002 Issue


Page 219 of 2049
Skeptical Review Edited by Farrell Till
to establish a posterity' be a more accurate translation, if Lockwood admits... 'laying down of
seed' is what the Greek expression meant?" Till's illustration is also based on the same
assumption--that I was making a concession of van der Horst's point, when, in reality, I was
merely giving the substance of what van der Horst had said so that I might review it. Try
again, professor.

By now, the readers are wondering about the several possibilities I offered at Hebrews 11:11
and why Till ignored them all. If Till is to carry his point, he must show (1) why these
translations are not legitimate, and (2) that the writer indeed intended to convey the idea that
Sarah emitted semen. Did he do this, readers? He has not even pretended to. First, I showed
that several translations make Abraham the subject of the verb. Bruce Metzger and Matthew
Black do the same. What did Farrell say about this??? Silent as a Texas oyster! But I will add
another one. The Translator's NT reads "the verse could also be trans. w/ reference to
Abraham: 'by faith--though Sarah herself was barren--he received power to deposit seed..."
(Appendix, p. 528). Your case is lost, Till, if this is a possible translation.

Second, "mum" was the word from Till on Moulton and Milligan's "storehouse translation."
Third, "seed" could be used here for "posterity" (Wigram-Green, Berry's lexicon, Bauer,
Arndt, Gingrich, Cremer's; Thayer and Vincent's Word Studies). Thus, Sarah remains the
subject. She had faith to "found a posterity."

Even on a fourth possible solution, Till makes not even a peep. I suggested to him that he be
more careful to learn these lessons so as to save himself such reproof in the future. He
actually saw the lesson, I suppose, for he was careful this time not to bring it up! Will any of
Tlll's agnostic buddies induce him to look at the argumentation here?

GENESIS 3:15
Till thinks I missed the point here, but I was not making an effort on "scientific
foreknowledge" except to expose his "Egyptian hymn to the sun-god" theory that places the
hymn prior to the writing of Genesis 3:15. While Till tries to make up the ground he has lost
at Hebrews 11:11 by grinding me with "hopelessly naive," which, if he says enough, maybe
his agnostic friends will count as a good argument, let's note once again the Protevangelium
herein, which Till avoided mentioning. "The text announces a male descendant who will
ultimately win a crushing victory over Satan himself" (Theological Wordbook of OT, Vol. 1,
p. 253). To this, I add that there is a deeper meaning than just the enmity between Satan and
the woman's seed (line of descendants). Moses went on to prophesy that "he shall bruise thy
head, and thou shalt bruise his heel."
Commencing with Gen. 3:15 the word "seed" is regularly used as a collective noun in the
singular (never plural). This technical term is an important aspect of the promise doctrine, for
Hebrew never uses the plural of this root to refer to "posterity" or "offspring...." Thus the
word designates the whole line of descendants as a unit, yet it is deliberately flexible enough
to denote either one person who epitomizes the whole group (i.e. the man of promise and
ultimately Christ), or the many persons in that whole line of natural and/or spiritual
descendants (Ibid.).

Volume 1990 - 2002 Issue


Page 220 of 2049
Skeptical Review Edited by Farrell Till
Adam Clarke said, "It shall bruise thy head, or rather HE, who? The seed of woman, the
person is to come by the woman, and by her alone, without the concurrence of a man" (Vol. 1,
p. 53). I also point out to Till's agnostic audience that the Targums of Jerusalem and of the
Pseudo-Jonathan both say that this "victory over the serpent will be 'in the days of the
Messiah'" (Cook Commentary, Vol. 1, p. 46). But the point of emphasis is that SO
STRONGLY does this verse point to Jesus Christ, the Messiah, that the Jewish Bible of today
(according to the Masoretic Text, The Menorah Press, Chicago, p. 5) makes it say, "They
shall bruise thy head..." evidently referring it to the Jewish people, not Christ. But Isaac
Lesser's translation (Hebrew Publishing Co., p. 2) follows the original readings: "HE shall
bruise thy head, and thou shalt bruise HIS heel."

I tell you, friends, so great is the case for the miraculous advent of the Messiah at Genesis
3:15 and of the Bible, in general, that efforts are continually made to change the wording of
the Bible and criticize the content with numerous exegetical flip-flops. Why does Till spend
so much effort against the Bible? Because, though he claims to be agnostic about God's
existence, he KNOWS this: the Bible testifies eloquently to the truth upon its pages. And, so
poorly has Till done in this exchange that I kindly suggest to him that from now on he quit
trying to piece together some patchworked anecdotes, sarcasms, and a few shallow exegeses
while sitting at his computer late at night. He has surely made a poor showing in this
exchange, and every reader knows it.

(Bill Lockwood's address is 211 North Fifth, Marlow, OK 73055.)

EDITOR'S NOTE: We have promised not to comment at the end of rebuttal articles, and, as
tempting as it is to point out the many absurdities in Lockwood's article, we will keep our
promise. A response to the major points in this article has been written and mailed to Mr.
Lockwood. A copy will be sent free on request to anyone wishing to read it. In the interest of
fairness, we will include also his reply to it, if he cares to write one. Please include a stamped
envelope with your request.

Our Own Effort


A thousand dollars Is certainly nothing to sneeze at, so we decided to try for Ralph Nielsen's
$1,000 offered in the summer issue. In taking the information from the opening verses of each
account of the resurrection, we hit a snag when we tried to fuse it all into one consistent
statement. Our effort came out like this:
When the sabbath was past, Mary Magdalene, Mary the mother of James, Joanna, Salome,
and the other women bought spices to anoint the body. Late on the sabbath day, at early dawn,
as it began to dawn toward the first day of the week, they came to see the sepulchre when the
sun was risen, very early on the first day of the week, while it was yet dark.
That was obviously no good, because glaring contradictions were already present. The
statement about the purchase of spices was taken from Mark's version, but Luke (23:54-56)
says that the spices were obtained before the Sabbath. More serious than this, however, were

Volume 1990 - 2002 Issue


Page 221 of 2049
Skeptical Review Edited by Farrell Till
the notable time discrepancies that say the women went to the tomb when the sun was risen
but while it was yet dark. So we tried again:
When the sabbath was past, Mary Magdalene, Mary the mother of James, Joanna, Salome,
and the other women bought spices to anoint the body. Late one the sabbath day while it was
yet dark, they came to see the sepulchre at early dawn, as it began to dawn toward the first
day of the week, very early on the first day of the week, when the sun was risen.
The theory behind this second attempt was to test the merits of the inerran- tist claim that
John meant for his readers to understand that the trip to the tomb began while it was yet dark
whereas the references of the other writers to early dawn and the risen sun were alluding to
the arrival time at the tomb. It's a good theory, but it completely ignores the fact that John
said, "While it was yet dark, Mary Magdalene came to the tomb" (20:1, NRSV). "While it
was yet dark" is an adverbial expression of time that obviously modifies the verb came; thus,
Mary came to the tomb while it was yet dark, according to John. Likewise, expressions like
"very early on the first day of the week," "late on the sabbath day," "at early dawn," "as it
began to dawn toward the first day of the week," and "when the sun was risen" are also
adverbial phrases or clauses that modify the verb come in the passages that they are used in.
There is just no other verb in any of the accounts for them to modify. For this reason, any
effort to fuse all of the expressions into one consistent statement is doomed to fail.

"When the sabbath was past" is an adverbial clause in Mark 16:1 that modifies the verb
bought. In other words, the women bought spices "when the sabbath was past," yet came to
the tomb "while it was yet dark." How could both statements be true? And how could the
women have bought the spices "when the sabbath was past" yet had obtained the spices before
the sabbath (Luke 23:54-56)?

With no plausible way to solve this problem, we looked ahead and saw another inconsistency
waiting in the wings. According to Matthew, when the women arrived, an "angel of the Lord"
descended during an earthquake and rolled the stone away from the sepulcher (28:2), but the
other three writers all said that the stone was already rolled away when the women came to
the tomb (Mk. 16:4; Lk. 24:2; Jn. 20:1).

At this point, we gave up and decided that Mr. Nielsen's money is safe.

Volume 1990 - 2002 Issue


Page 222 of 2049
Skeptical Review Edited by Farrell Till

The Skeptical Review


Volume Four - 1993
Farrell Till, editor

• Number 1 Volume Four, Issue One


• Number 2 Volume Four, Issue Two
• Number 3 Volume Four, IssueThree
• Number 4 Volume Four, IssueFour

Volume 1990 - 2002 Issue


Page 223 of 2049
Skeptical Review Edited by Farrell Till

The Skeptical Review


Volume Four, Number One - 1993
Farrell Till, editor

• Living on Borrowed Time


The writing is on the wall, and the days of Christian fundamentalism are numbered.

• A Perfect Work of Harmony?


Inerrantists claim that the Bible "is characterized by such an amazing unity and
beautiful continuity as to be inexplicable on the basis of human origin", but it just ain't
so.

• Was It Morally Right for God to Order the Killing of the Canaanites?
Yes, says Clarence Lavender. In a chilling article that has Lavender comparing
innocent children with cancerous tumors, Lavender calls the massacre "the highest
manifestation of the goodness of God." If that's the highest, I'd hate to see the lowest.

• Suffer, Little Children


Till takes Lavender to task for his moral insensibility.

• Does the Bible Speak of the Brain?


If the authors of the Bible were inspired by an all-knowing God, how come they
believed that the mind was centered in our intestines?

Living on Borrowed Time


Despite all the noise being made by the vociferous religious right and the influence it
currently exerts on the Republican Party, there are clear indications that Christian
fundamentalism is rapidly losing ground. In "The Strange Decline of American

Volume 1990 - 2002 Issue


Page 224 of 2049
Skeptical Review Edited by Farrell Till
Evangelicalism," John Warwick Montgomery cited a recent report by the Princeton Religion
Research Center, which said that a nationwide Gallup poll revealed that "the Average
American's belief in the reliability of Scripture has declined by half in the last 30 years (from
65 per cent in 1963 to 32 percent today)" [Christian News, Sept. 21, 1992, p. 1]. The article
went on to say that "69 percent of U.S. adults now identify with moral relativism." After
citing these statistics, Montgomery declared evangelicalism "a conspicuous failure in our
generation."

That some unusual liberal trends are developing in fundamentalist churches has been apparent
for some time. To stay abreast of what is going on in the religious right, TSR subscribes to
several fundamentalist papers, and the same theme runs through most of them. What can we
do to hold our ground against liberalism? A bitter fight is being waged in the Southern Baptist
Convention between the old-line inerrantists and those who believe that the inerrancy doctrine
is indefensible. The same is true in the Missouri Synod of the Lutheran Church. In the Church
of Christ, the denomination that TSR's editor was once affiliated with, a hard-core guardian-
of-the-faith group constantly pleads for a return to "the old paths." They wage constant battle
with "liberal" preachers who have renounced the we-are-the-only-true-church mentality and
now openly seek accommodation and fellowship with other denominations. They rave against
"modernistic" professors in their "Christian" colleges, who unabashedly teach in their classes
that the Bible contains errors and even mythology. The situation has become so desperate for
some of the diehard congregations that they have given up on their Bible colleges and
established their own "schools of preaching" to train ministers in the old-line doctrines of the
"restoration" preachers who founded the Church of Christ.

Probably the most startling development in this church is the trend to minify the issue of
instrumental music in Christian worship. In 1910, this issue split the Campbellite movement
into the Christian Church and the Church of Christ, and thereafter the latter vehemently
denounced the use of instrumental music in worship as the work of Satan himself. That is
until recently, but now many Church-of-Christ preachers are acknowledging that this view
may be erroneous. They speak of a "new hermeneutics" and express the desire to have
dialogues with the liberal wing of the Campbellite movement to see if some kind of
accommodation can't be mutually agreed upon that would unify the two groups.

To the old-line, guardian-of-the-faith preachers, of course, "new hermeneutics" is a cuss


word, and they will have no part of it. They preach sermons and write articles against it and
issue debate challenges to those who advocate it, probably believing that someday they will
lead their wayward brothers back into "the old paths" when once more the denunciation of
instrumental music, divorce for any reason but adultery and lots of other good things like
these will be preached again in Church-of-Christ pulpits throughout the land.

No one likes to be a party-pooper, but we predict that this just ain't going to happen. The
reason why it will never happen is as simple as the principle implied in the WW I song lyrics
quoted in TSR's first issue: "How are you ever goin' to keep 'em down on the farm after
they've seen Pa-ree?" ("The Last Hurrah of the Inerrancy Doctrine," Winter 1990, p. 3). Just
as the soldiers of World War I who had had their horizons broadened by their experiences in
Europe were unlikely ever again to be content with the routinism of life on the farm so
ministers and Bible college professors whose knowledge has been broadened by exposure to

Volume 1990 - 2002 Issue


Page 225 of 2049
Skeptical Review Edited by Farrell Till
facts about the Bible that in times past were kept from the flock will never again be content to
hide the truth for the sake of preserving something as dubiously important as "the old paths."

Once something is learned, it cannot be unlearned. This is the principle that spells doom for
Bible fundamentalism if not the Bible, period. We live in an age of rapid discovery. In this
century, man journeyed to the moon; in the next century, he will journey to Mars and
probably beyond. Man has conquered many diseases and will conquer even more. Scientists
talk routinely of genetic mapping, gene-splicing, black holes, quarks, and other concepts the
ordinary mind can barely grasp. In such an environment as this, how can people possibly go
on believing that the God who created an endless universe once lived in a tent that nomadic
tribes carried with them in their desert wanderings, spoke to them from a column of fire that
followed them overhead, selected them to be his chosen people "above all peoples on the face
of the earth," and took delight when they slaughtered animals and incinerated them in homage
to him? How can people who will witness the eradication of cancer, AIDS, cystic fibrosis, etc.
through the application of scientific methods continue to believe that "the son of God" once
went about curing diseases by casting out devils? To ask such questions is to answer them.
They won't go on believing such ridiculous nonsense. That kind of superstition is doomed.
Even now, it is running on empty.

We say this knowing in advance how Bible fundamentalists will scoff at it. No doubt, they
will cite men like Thomas Paine and Voltaire, who made similar predictions within time
frames that have now come and gone, yet the Bible, "the word of God," endures. We are well
aware of what Paine and Voltaire rashly predicted. Their primary mistake was that they were
too optimistic. Faith in the Bible will not die overnight; it will not die in the next century or
probably even the century after that. But it will die. The history of religion is one of birth,
development, expansion, decline, and death. It happened to the ancient religions of Sumer,
Babylon, Egypt, Greece, and Persia. It will happen to Christianity, as it will also happen to
Islam, Hinduism, and Buddhism.

Christians who scoff at the notion of a distant future in which no one believes in their "word
of God" should consider the statistic quoted in Montgomery's article. Already within our
lifetime, we have witnessed a 50% loss in faith in Bible inerrancy, and no doubt the 65% who
believed in Bible inerrancy in 1963 was significantly lower than the percentage who believed
in it in 1930. Go back a hundred years before that, and the percentage of believers in Bible
Inerrancy (in Western societies) probably exceeded even 90%. So if Bible inerrancy is not
living on borrowed time (as we believe it is), why does it steadily lose ground? What is going
to happen to thrust it back into the privileged position that it once enjoyed?

People constantly tell us that they are praying for our return to the fold, but this is never going
to happen, no matter how many prayers are uttered. We have learned too much ever to go
back to what we once were. Few laymen devote even a tenth as much time to studying the
Bible as we do, yet they live in an age when they can't help absorbing information that erodes
belief in biblical superstition. That erosion will remain steady until there is nothing left...
except amazement that anyone could have ever taken a book like the Bible seriously.

Volume 1990 - 2002 Issue


Page 226 of 2049
Skeptical Review Edited by Farrell Till

A Perfect Work of Harmony?


Farrell Till
In the second issue of TSR (Spring 1990, p. 14), we raised the question of harmony in the
Bible in a short article with the same title as this one. A frequent argument that bibliolaters
use in their vain attempts to prove the inerrancy doctrine is that the Bible is so harmonious in
content that only divine inspiration could explain its remarkable consistency. In his article
"Why I Believe in the Inerrancy of the Scriptures" (Autumn 1992), fundamentalist minister
Dave Miller said that "the Bible has been consistently... demonstrated to possess the
unequaled characteristic of internal consistency" (p. 3). In Christian Courier, editor Wayne
Jackson rhapsodized the achievement of "some forty different persons" who had written "in
three different languages" over a span of "1,600 years" and yet had "produced a volume of
sixty-six books that is characterized by such an amazing unity and beautiful continuity as to
be inexplicable on the basis of human origin" ("The Holy Bible--Inspired of God," May 1991,
p. 1).

If this often repeated claim were true, it would constitute formidable evidence for the divine
inspiration of the Bible, but, as objective students of the Bible know, it is a claim that doesn't
even come close to being true. Biblical apologists like Gleason Archer, John Haley, William
Arndt, and George DeHoff have published volumes of far-fetched, how-it-could-have-been
explanations to what they sanctimoniously call "alleged Bible discrepancies," the implication
being that the discrepancies aren't real but only "alleged." In his Encyclopedia of Bible
Difficulties , Archer listed in the index over 2,000 "difficult" scriptures that he had addressed
in the book, and there would be no way to estimate the number of independent periodicals and
bulletins like Christian Courier that devote much of their publishing space to defending the
harmony of the Bible. Inerrantists, however, apparently can't see the absurdity of preaching
the "amazing unity and beautiful continuity" of a book that has required the publication of so
many volumes and periodicals to defend and explain its harmony.

The truth is that the Bible, rather than being remarkably consistent and harmonious in its
themes, is a book riddled with discrepancies and divergent theological views. Although unity
of theme, as I noted in my response to Miller's article (Autumn 1992, p. 5), was undoubtedly a
criterion considered by the councils and conferences of rabbis and clerics who arbitrarily
made canonical decisions, the selection processes were nevertheless imperfect in that they
failed to produce a Bible free of discrepancies.

An excellent example of divergent political and theological philosophies is the one that I
presented in my original article on this subject. The writer(s) of 1 and 2 Kings obviously
believed that Jehu's massacre of the royal families of Israel and Judah was sanctioned by
Yahweh, the tribal god of Israel. As early as 1 Kings 19 , Yahweh commanded the prophet
Elijah to anoint Jehu king of Israel (v:16). The actual anointing did not occur, however, until
Elijah's successor Elisha sent a "son of the prophets" to Ramoth-gilead with instructions to
anoint Jehu king of Israel. In executing the command, the prophet said this during the
ceremony:

Volume 1990 - 2002 Issue


Page 227 of 2049
Skeptical Review Edited by Farrell Till
Thus, saith Yahweh, the God of Israel, I have anointed thee king over the people of Yahweh,
even over Israel. And thou shalt smite the house of Ahab thy master, that I may avenge the
blood of my servants the prophets, and the blood of all the servants of Yahweh, at the hand of
Jezebel (2 Kings 9:6-7 ). Jehu then led an armed force against Jezreel and massacred Joram,
king of Israel, and Ahaziah, king of Judah, who at the time was visiting his kinsman Joram (2
Kings 9:21-24 ). After having Jezebel, Joram's mother killed, Jehu then ordered the
decapitation of Joram's seventy sons (10:6-8 ) and the execution of all principals in Joram's
government (10:11 ), as well as 42 servants who had attended Ahaziah on his royal visit
(10:12-14 ). As if this were not enough bloodshed, Jehu later deceived all the prophets,
priests, and worshipers of the god Baal into gathering for a sacrificial ceremony during which
he sent a force of 80 armed men into the house of Baal to massacre everyone in attendance.

After all of this blood had been shed, the writer(s) of 2 Kings said this of Jehu's actions at
Jezreel:

And Yahweh said unto Jehu, Because thou hast done well in executing that which is right in
mine eyes, and hast done unto the house of Ahab according to all that was in my heart, thy
sons of the fourth generation shall sit on the throne of Israel ( 10:30 emphasis added).

This statement is plain enough that anyone should understand it. Whoever wrote it clearly
thought that Jehu had done Yahweh's will in the Jezreel massacres. The writer even went on
to chronicle the reigns of the four generations of the house of Jehu who succeeded to the
throne of Israel after Jehu.

All four of these were described as kings who "did that which was evil in the sight of
Yahweh" (13:1-2 ; 13:10-11 ; 14:23-24 ; 15:8-9 ), yet the clear implication of the writer was
that Yahweh had permitted them to reign to fulfill his promise to Jehu that ended the passage
cited above. So when Zechariah, the fourth generation of these kings, was overthrown in a
rebellion, the writer said, "This was the word of Yahweh which he spake unto Jehu, saying,
Thy sons to the fourth generation shall sit upon the throne of Israel. And so it came to pass"
(15:12 ).

There can be no doubt, then, that the author(s) of 2 Kings approved Jehu's actions in the
massacre at Jezreel and considered him an emissary of Yahweh, who had been sent to execute
judgment against the house of Ahab. A century later, however, the prophet Hosea had a
different view. He opened his book with a pronouncement of judgment upon the house of
Jehu in the latter days of Jeroboam, the third-generation of Jehu's descendants mentioned
above:

(F)or yet a little while, and I [Yahweh] will avenge the blood of Jezreel upon the house of
Jehu, and will cause the kingdom of the house of Israel to cease. And it shall come to pass at
that day, that I will break the bow of Israel in the valley of Jezreel (1:4-5).

Fundamentalists have made their usual how-it-could-have-been attempts to explain what


Hosea meant, but the fact still remains that the author(s) of 2 Kings approved the Jezreel
massacre and Hosea, a century later, pronounced judgment on the house of Jehu for "the
blood of Jezreel." Why would a divinely inspired prophet pronounce judgment on Jehu's

Volume 1990 - 2002 Issue


Page 228 of 2049
Skeptical Review Edited by Farrell Till
descendants for something that another divinely inspired person had praised in terms of
having "execut[ed] that which is right in mine [Yahweh's] eyes" and having done "according
to all that was in my [Yahweh's] heart" (2 Kings 10:30 )?

The only reasonable answers are that (1) neither writer was actually inspired, and (2) the two
writers simply had divergent political and theological views about the Jezreel massacre. The
second of these alternatives certainly explodes the myth of "amazing unity and beautiful
continuity" in the Bible. Another biblical disagreement concerned the importance of
sacrificial ceremonies. Almost from the very beginning, the Old Testament took a very
serious view of sacrifices. The first murder resulted when Yahweh had respect for the bloody
sacrifices of Abel but rejected the "fruit of the ground" that Cain had offered. In an ensuing
argument, Cain killed his brother Abel (Gen. 4:2-8 ). After the Ark had landed on dry ground,
at a time when (if the story is true) every animal on earth was an endangered species, Noah
built an altar, took "of every clean beast and of every clean bird," and offered burnt-offerings
to Yahweh (Gen. 8:20 ). This gesture must have pleased Yahweh who "smelled the sweet
savor" (v:21 ), for he promised "in his heart" that he would not "curse the ground anymore for
man's sake" or "smite anymore everything living" as he had just done in sending the flood
(v:21 ). All through the rest of Genesis, we read how the patriarchs found favor with Yahweh
through their animal sacrifices (12:7-8 ; 15: 8-11 ; 26:25 ; 31:54 ; 33:20 ; 35:1-7 ).

When Yahweh established his covenant with Israel (as the story goes), he included animal
sacrifices as a part of it. After his face-to-face meeting with Yahweh on Mt. Sinai, Moses
gave the Israelites instructions on proper procedures for the offering of sacrifices (Ex. 34:25-
26 ). These laws were expanded later (Num. 6: 13-17 ; 7:84-88 ; 15:1-14 ). The book of
Leviticus contains little more than detailed instructions in proper ceremonial procedures,
many of them pertaining to the offering of animal sacrifices. These instructions were given
with the solemn assurance that they were what "Yahweh had commanded Moses in mount
Sinai, in the day that he commanded the children of Israel to offer their oblations unto
Yahweh in the wilderness of Sinai" (Lev. 7:38 ). That these ceremonies were considered
serious business is indicated in the story of Nadab and Abihu, both sons of Aaron, who tried
to offer sacrifices in a manner not specifically prescribed by Yahweh. Their punishment was
fire that came forth from Yahweh and devoured them (Lev. 10:1-2 ). All of this indicates that
the author(s) of the Pentateuch considered animal sacrifices not just important but a Yahwistic
command that had to be executed according to the letter. Why then did some of the prophets
who lived centuries after the exodus and wilderness wanderings take a different view of
sacrificial ceremonies? Yahweh said through the prophet Hosea, "For I desire goodness, and
not sacrifice; and the knowledge of God more than burnt offerings" (6:6 ). Inerrantists, of
course, will argue that Hosea was speaking of sacrifices offered by those who were
disregarding the importance of inner purity in their personal lives; however, a psalm attributed
to David, a man after God's own heart (1 Sam. 13:14 ), who had done that which is right in
the eyes of Yahweh "all the days of his life" (1 Kings 15:5 ), said this of sacrifices: "For thou
delightest not in sacrifice; else would I give it: Thou hast no pleasure in burnt-offering"
(51:16 ). To want goodness more than sacrifice from the ungodly is one thing, but why would
Yahweh, who was so pleased with Noah's sacrifice that he promised never to destroy man
again, not delight in sacrifices from a man after his own heart? This is hardly unity so
amazing and consistency so beautiful that they could be explained only by divine inspiration.

Volume 1990 - 2002 Issue


Page 229 of 2049
Skeptical Review Edited by Farrell Till
The prophet Jeremiah took the matter of sacrifices even further and denied that Yahweh had
even commanded them:

My people, some sacrifices you burn completely on the altar, and some you are permitted to
eat. But what I, the LORD [Yahweh], say is that you might as well eat them all. I gave your
ancestors no commands about burnt offerings or any other kinds of sacrifices when I brought
them out of Egypt. But I did command them to obey me, so that I would be their God and
they would be my people. And I told them to live the way I had commanded them, so that
things would go well for them ( 7:21-23, Good News Bible).

I have quoted this passage in a modern English version so that the meaning will be clearer
than it is in the versions that use 17th century English. According to Jeremiah, Yahweh
plainly said that he gave no commands concerning burnt offerings and other sacrifices when
he brought the Israelites out of Egypt, and that flatly contradicts passages already cited in this
article and others too numerous to list. Animal sacrifices were the very foundation of the
religion that Yahweh allegedly revealed to the Israelites through Moses. In his typically far-
fetched fashion, Gleason Archer explained the problem posed by Jeremiah's statement by
claiming that the prophet meant only that "no sacrificial requirements were made by God to
the Israelites" until after they were out of Egypt (Encyclopedia of Bible Difficulties, p. 272).
In other words, Archer limits Jeremiah's statement to the time that the Israelites were in Egypt
proper and therefore contends that the prophet was not alluding to sacrificial commands that
Yahweh later gave to the Israelites in the Sinai wilderness.

If ever an inerrantist stretched credulity to the limits to explain away an "alleged Bible
discrepancy," Archer has done so here. For one thing, the territory of Egypt at that time (as it
does today) stretched into the Sinai region, so when the Israelites were in the wilderness,
where the writer of Exodus claimed that Yahweh gave commands concerning sacrifices
(20:24-26 ), they were still in Egypt. It is therefore more probable that Jeremiah was referring
to an era or a period of time that involved both the exodus and the wilderness wanderings.

This interpretation is more consistent with the original text. In Hebrew, Jeremiah literally
said, "I did not speak to your fathers nor command them in the day that I brought them out of
the land of Egypt about the matters of burnt offerings and sacrifices" (Hendrickson's
Interlinear Bible). The expression "in the day that I brought them out of the land of Egypt" is
in the KJV, ASV, NKJV, RSV, NRSV, NAS, RBV, NAB, AB, and other versions including
the Holy Spirit's favorite, the Septuagint. It is customary to use the word day in the sense of
an era rather than a literal twenty-four-hour period. Determining if Jeremiah meant the word
in this sense should be as simple as examining other places where he used the expression "in
the day that I brought them out of the land of Egypt."

A good place to begin would be chapter 11 , where Yahweh said through Jeremiah, "Cursed
be the man that heareth not the words of this covenant, which I commanded your fathers in
the day that I brought them forth out of the land of Egypt" (v:3-4 ). According to Archer's
logic, this statement would have to mean that Yahweh made his covenant with the Israelites
on the very day (twenty-four-hour period) that he brought them out of Egypt, but in fact the
covenant was not made until the events recorded in Exodus 20 , the very chapter that Archer
referred to as the time and place in the Sinai wilderness when Yahweh first spoke to the

Volume 1990 - 2002 Issue


Page 230 of 2049
Skeptical Review Edited by Farrell Till
Israelites concerning sacrifices. Later in the same chapter, Yahweh, speaking through
Jeremiah again, said, "For I earnestly protested unto your fathers in the day that I brought
them up out of the land of Egypt, even unto this day, rising early and protesting, saying, Obey
my voice" (v:7 ). We find no such protest recorded in Exodus that Yahweh made on the very
day or even before the Israelites left Egypt, but Yahweh made many such protests to the
Israelites in the wilderness (Ex. 15:26 ; 19:5-6 ; 23:20-22 ; Dt. 11:27 ; 27:10 ; 30:1-2 ). "In the
day that I brought them up out of the land of Egypt," then, must have referred to an era
associated with the journey from Egypt to Canaan rather than to the actual time it took the
Israelites to leave Egypt proper. In Jeremiah 31:32 , Yahweh again referred to the covenant
that he made with the Israelites "in the day that I took them by the hand to bring them out of
the land of Egypt," and again in 34:13 , Yahweh said, "I made a covenant with your fathers in
the day that I brought them forth out of the land of Egypt, out of the house of bondage." As
noted above, however, Yahweh's covenant with the Israelites was not made on the very day
that they left Egypt but later when they were in the Sinai wilderness. So twice again, the
prophet Jeremiah obviously used the expression in question to mean the period of time
associated with the exodus and the wilderness wanderings. Why then must we believe that he
was referring only to the time that the Israelites were in Egypt proper when he said that
Yahweh did not speak to the Israelite fathers about burnt-offerings and sacrifices in the day
that he brought them out of the land of Egypt?

Another flaw in Archer's theory is his claim that Yahweh made "no sacrificial requirements"
of the Israelites while they were in Egypt. Recognizing the flaw, Archer dismissed the
passover instructions given to Moses on the eve of the exodus on the grounds that the
passover "had nothing to do with offerings on an altar" (Ibid). Nevertheless, the passover was
considered a sacrifice and was so designated in the instructions that Moses gave to the
Israelite elders and in his renewal of the instructions in the wilderness: And it shall come to
pass, when your children shall say unto you, What mean ye by this service? that ye shall say,
It is the sacrifice of Yahweh's passover ( Ex. 12:26-27 , emphasis added). And thou shalt
sacrifice the passover unto Yahweh thy God, of the flock and the herd, in the place which
Yahweh shall choose, to cause his name to dwell there.... Neither shall any of the flesh, which
thou sacrificest the first day at even, remain all night until morning. Thou mayest not sacrifice
the passover within any of thy gates, which Yahweh thy God giveth thee; but at the place
which Yahweh thy God shall choose, to cause his name to dwell in, there thou shalt sacrifice
the passover at even, at the going down of the sun... (Dt. 16:2-6 , emphasis added).

So even if Archer is right in his interpretation of what Jeremiah meant by "in the day that I
brought them [the Israelites] out of the land of Egypt," he is wrong in claiming that God made
"no sacrificial requirements" of the Israelites while they were in Egypt. To argue that
Jeremiah was referring only to sacrifices that involved altars is a resort to arbitrariness,
because he included no such qualification in his statement. He said (without qualification): "I
spake not unto your fathers, nor commanded them in the day that I brought them out of the
land of Egypt, concerning burnt-offerings or sacrifices." If he had intended this as a reference
only to altar sacrifices, then why didn't he stop with burnt-offerings? The fact that he went on
to say "or sacrifices" must mean that he had in mind more than just altar sacrifices. So
Archer's resolution of this problem turns out to be just another futile attempt to preserve the
inerrancy doctrine.

Volume 1990 - 2002 Issue


Page 231 of 2049
Skeptical Review Edited by Farrell Till

Reprinted from The Firm Foundation....

Was It Morally Right for God to Order the


Killing of the Canaanites?
Clarence Lavender
The Bible teaches that God never does anything wrong. God is never out of control but
always does things consistent with his holy and just nature. "Now therefore let the fear of
Jehovah be upon you, take heed and do it: for there is no iniquity with Jehovah our God, nor
respect of persons, nor taking of bribes" (2 Chron. 19:7). There must be good explanation
when God orders the death of people, some of them women and innocent children.

THE DESTRUCTION OF THE CANAANITES


Those who question the ethics of God concerning the destruction of the Canaanites, or any
other judicial decision made by God, fail to take into account six things:

1. They ignore the reason given for the destruction of wicked people, "that they teach you not
to do after their abominations which they have done unto their sons" (Dt. 20:18 ).

2. The only way a person can accuse God of wrong is to be equal with God. No mortal can
make the same judgments God makes because he does not have God's view of things or his
knowledge.

3. The Canaanites were grossly immoral. The justice of God demands punishment for sin.
"The wages of sin is death" (Rom. 6:23). God must show his righteousness in punishing the
wicked or violate his own nature (Rom. 3:23-27 ).

4. A person would have to be omniscient to know that what happened to the innocent children
of guilty parents was not the best thing that could have happened. The alternative here may be
to grow to adulthood and become malignant blights in the society of men like their parents.

5. Punishment here may be in recognition of what the Canaanites had earned. When it became
clear they were past redemption, their destruction occurred.

6. Punishment was deserved by the Canaanites, whereas it was not in the case of the
Holocaust--which was only a vendetta by Hitler and the Nazis against the Jews.

Volume 1990 - 2002 Issue


Page 232 of 2049
Skeptical Review Edited by Farrell Till
It was dedication to continual wickedness that marked the Canaanites for extermination. The
scriptures uphold the justice and righteousness of God even in his command to eradicate the
Canaanites.

Not for thy righteousness, or for the uprightness of thine heart, dost thou go to possess their
land; but for the wickedness of these nations the Lord thy God doth drive them out from
before thee, and that he may perform the word which the Lord sware unto thy fathers,
Abraham, Isaac, and Jacob (Dt. 9:5).

God's call for Israel to destroy "the Hittite, and the Amorite, the Canaanite, and the Perizzite,
the Hivite, and the Jebusite" was not traced to the Jews' moral superiority or their number, but
to fulfill his promise to Abraham and his seed. God was preserving a lineage through which
the Messiah would come and all nations be blessed.

There had been a patient waiting from Abraham's time for the sin of the Amorites to reach its
full measure. "But in the fifth generation they shall come hither again: for the iniquity of the
Amorites is not yet full" (Gen. 15:16 ).

The Canaanites were singled out for severe treatment to prevent Israel and the rest of the
world from being corrupted. "Of the cities of these peoples... thou shalt save alive nothing that
breatheth... that they teach you not to do after all their abominations, which they have done
unto their gods; so would ye sin against Jehovah your God" (Dt. 20:16-18 ). When people
burn their children in honor of false gods, practice sodomy, bestiality, and all forms of
wickedness, the land itself begins to vomit them out (Lev. 18:25-30 ). The modern world
would do well to heed the warning (Rom. 8:22 ). Objection to the fate of these nations is
really an objection to the highest manifestation of the goodness of God. Greene likens this
action on God's part, not to doing evil that good may come, but doing good in spite of certain
evil consequences, just as a surgeon does not refrain from amputating a gangrenous limb,
even though in so doing he cannot help cutting off healthy flesh.

A husband agrees to abort a tubular pregnancy. The purpose of the abortion is not to take the
life of the child, but to save the life of the mother; besides the child could not survive in any
case. The doctor knows that in destroying cancer cells healthy cells will be destroyed. His
purpose is not to kill good cells but bad ones; yet in trying to save the person from the spread
of cancer, good is destroyed, but it could not have survived regardless.

At what instant I shall speak concerning a nation, and concerning a kingdom, to pluck up, and
to pull down, and to destroy it: If that nation against whom I have pronounced, turn from their
evil, I will repent of the evil that I thought to do unto them. And at what instant I shall speak
concerning a nation and concerning a kingdom, to build and to plant it: If it do evil in my
sight, that it obey not my voice, then I will repent of the good wherewith I said I would
benefit them (Jer. 18:7-10).

Canaan had a forty-year countdown. They heard of the events in Egypt, the crossing of the
Red Sea, and what happened to the kings who opposed Israel along the way. We know they
were aware of such events, for Rahab confessed that these same events had terrorized Jericho
and that she, as a result, had placed her faith in the God of the Hebrews (Josh. 2:10-14 ). The

Volume 1990 - 2002 Issue


Page 233 of 2049
Skeptical Review Edited by Farrell Till
destruction of the Canaanites was based on the same principle by which the whole world was
judged in the universal flood and that by which the cities of Sodom and Gomorrah were
judged. Usually those who object to these events are those who deny compatibility of the
doctrine of eternal destruction with the mercy and love of God. God's character and the acts
he requires are fully consistent with everything that both Testaments teach us to expect in our
God. The problem usually centers in a deficiency in our view of things and our inability to
properly grasp the whole of the subject.

(Clarence Lavender's address is P. O. Box 306, Christianburg, VA 24073.)

Suffer, Little Children


Farrell Till
If you couldn't believe what you were seeing while reading the foregoing article, you
probably have no background in Christian fundamentalism. A Christian writer who believes
that the massacre of entire civilian populations in time of war, even to the point of genocide,
is morally good! Is it possible that anyone living in a modern civilized society could really
believe such a thing? Well, I assure you that your eyes weren't playing tricks on you. Bible
fundamentalists really do defend all the bloody deeds that were presumably ordered by the
Hebrew god Yahweh. Standing by this ancient war-god, even to the point of defending his
commands to massacre babies, is an albatross that they must wear around their necks or else
surrender their belief in Bible inerrancy. Apparently unable to bear the thought of life without
their Bible-inerrancy security blanket, they choose to take a stand for killing babies.

Mr. Lavender, of course, played down the fact that babies were slaughtered in the conquest of
Canaan. He twice made passing mention of "children" who were killed, but if the conquest
actually occurred as recorded in the book of Joshua, untold thousands of babies had to have
been included in the civilian populations that were systematically massacred. As I respond to
Mr. Lavender's article, I will focus on the children and babies who were victims of the
Israelite massacres, but as I do, the readers should bear in mind that, if the stories are true,
there would have also been thousands of women and elderly, as well as children, who were
killed.

An idea of the kind of numbers we are talking about can be otained from the story of the
slaughter of the Midianite captives in Numbers 31 . Upon hearing that his army was returning
from the Midianite campaign with women and children captives, Moses went out to meet his
officers and commanded them to "kill every male among the little ones" and to "kill every
woman that hath known man by lying with him" (v:17 ). "But all the women-children that
have not known man by lying with him," Moses went on to say, "keep alive for yourselves"
(v:18 ). The purpose of this article is not to discuss sexual abuse but the massacre of captive
children, so the only comment I will make about Moses' orders to keep the virgin girls "alive
for yourselves" is to urge the readers to keep in mind that belief in Bible inerrancy ultimately

Volume 1990 - 2002 Issue


Page 234 of 2049
Skeptical Review Edited by Farrell Till
forces one to defend not just the massacre of children but also their sexual abuse. What kind
of god would allow his inspired spokesman to tell his soldiers to kill women and children but
keep the virgin girls "alive for yourselves"? The answer to that, of course, is the god of the
Bible that Mr. Lavender and all his inerrantist cohorts defend.

The matter of numbers involved in such massacres as this one is indicated in the fact that
32,000 virgin girls escaped execution on this occasion (v:35 ). If there were that many virgin
girls among the captives, we can reasonably assume that at least this many, and probably
more, male children and nonvirgin women were put to the sword. Thus, when Mr. Lavender
and his inerrantist cohorts speak about "children" who were killed in these military
campaigns, they aren't talking about just a few but THOUSANDS of children--all killed in the
name of Yahweh, the god whom he and every fundamentalist Christian worship and serve.

Isn't Yahweh's bloody history at least a little embarrassing to Mr. Lavender and his
likeminded cohorts? Not at all. They defend it as absolute truth. "Objection to the fate of these
nations," Mr. Lavender said, "is really an objection to the highest manifestation of the
goodness of God" (p. 6). Notice again what he said: THE HIGHEST MANIFESTATION OF
THE GOODNESS OF GOD! That is exactly what Bible fundamentalists believe about the
many Yahwistically ordered massacres in the Old Testament. Such statements as this may
shock some of our readers, but they do not shock me. I have heard them too many times to be
shocked anymore. In my first oral debate on the inerrancy issue, my opponent said that the
massacre of the Amalekites was his "favorite story in the Bible." What happened in this
massacre? Yahweh commanded Saul, the first king of Israel, to "utterly destroy" the
Amalekites "and spare them not, but slay both man and woman, infant and suckling, ox and
sheep, camel and ass" (1 Sam. 15:3 ).

This debate materialized because my opponent, a Presbyterian minister, had led a local effort
to boycott the controversial movie The Last Temptation of Christ. He thought this movie was
obscene yet had the audacity to describe an incident that involved the massacre of "infants
and sucklings" as his favorite story in the Bible. There has to be something fundamentally
obscene in such a position as this.

Nevertheless, it is a position as common as belief in Bible inerrancy. In my debate with the


Church-of-Christ preacher Mac Deaver at Southwest Texas State University, Deaver publicly
took the position that the massacre of Amalekite babies had been a morally good thing. In my
oral debate with Jerry McDonald, who in his articles published in TSR has offered some
rather bazarre solutions to Bible discrepancies, I submitted to him a true or false question that
forced him to say that if he had lived in the time of King Saul, he would have participated in
the Amalekite massacre by willingly killing women, pregnant women, children, and babies.
So there is nothing at all unusual in Mr. Lavender's description of the Canaanite massacres as
"the highest manifestation of the goodness of God." To this, I can only say what I said about
my Presbyterian opponent's opinion of the Amalekite massacre: there has to be something
fundamentally obscene about a position that leads one to proclaim the massacre of babies as
"the highest manifestation of the goodness of God."

Mr. Lavender accused those who question the "ethics of God" in the destruction of the
Canaanites of failing to "take into account six things." The "ethics of God"? The ethics of

Volume 1990 - 2002 Issue


Page 235 of 2049
Skeptical Review Edited by Farrell Till
God! He has the audacity to imply that the massacre of children and babies is compatible with
divine ethics? Indeed he does, because nothing can be too ridiculous or too embarrassing to
keep a fundamentalist from offering it in defense of the Bible. Hence, those who question the
morality of killing babies for purely ethnic and religious reasons are questioning the ethical
conduct of God. Lavender would never put it that candidly, but that is what it amounts to.

So what are "the six things" that we fail to consider when we question the "ethics of God" in
this matter? Well, it seems that the Canaanites were so grossly immoral that they just had to
be exterminated "to prevent Israel and the rest of the world from being corrupted" (p. 6). The
only problem with this theory is that the Israelites weren't exactly paragons of virtue
themselves. After they went in to possess the land they had taken from the Canaanites, their
nation became as corrupt as any of the nations they had driven out. In fact, the story of their
wilderness wanderings on the way to the promised land is filled with incidents of rebellion,
idolatry, and orgy (Ex. 16:2-3 ; 32:1-20 ; Num. 11:1-2 ; 14:1-3 ; 16:1-35 ; 20:1-9 ; 21:4-9 ;
25:1-15 ). Their morality didn't improve after they entered into the promised land. They still
practiced idolatry (Judges 10:6 ; 1 Kings 11:4-8 ; 12:28-30 ; 16:30-33 ), offered human
sacrifices (2 Kings 21:6 ; 23:10 ; Ps. 106:37-38 ; Jer. 32:34-35 ; Ezk. 16:20-21 ), indulged in
orgies and abominations (Jdgs. 19 ; Ezek. 16:44-52 ). These, of course, are just partial listings
that could be greatly expanded if space permitted, for the history of Israel from the time of its
entry into the promised land until it fall to Nebuchadnezzar was, by the Bible's own account, a
history of moral profligacy. What Mr. Lavender is arguing, then, is that the Canaanites were
so wicked that God had to exterminate them so that their land could be possessed by a people
who were just as profligate. It is an argument that makes sense only to a Bible inerrantist.
"When it became clear that they [the Canaanites] were past redemption," Lavender said in
another of his points, "their destruction occurred." Furthermore, he said in still another point,
"The justice of God demands punishment for sin." So we wonder if this is why God was
constantly sending the Israelites into bondage after they displaced the Canaanites (Judges 3:7-
8 ; 4:1-3 ; 6:1-6 ; 10:7-8 ; 13:1 ). Was God, in keeping with his perfect justice, just punishing
them for their sins? If so, did he ultimately destroy the national identity of Israel and send
them into Babylonian captivity because "it had become clear that they were past redemption"?
If so again, then why did the inscrutable Yahweh destroy the Canaanites in the first place only
to fill their land with a people equally as wicked?

This last question poses a serious problem for Mr. Lavender's position that I will address later,
but first let's notice that another of his points was that one would have to be "equal with God"
before he could accuse God of wrong in the Canaanite massacres. This is a variation of the
old God's-ways-are-higher-than-our-ways argument, which is a catch-all dodge that
inerrantists use whenever their arguments make no sense. A major flaw in Lavender's
application of it is the obvious fact that it assumes without proof that God was actually
involved in the Israelite conquest of Canaan. A more probable interpretation of this aspect of
Hebrew history is that they merely thought that their god Yahweh was directing their
conquest of the land. Even today nations have a tendency to think that God is on their side in
time of war. That belief was even more prevalent in biblical times. Each nation had its god(s)
that the people thought rewarded them with victory when they were "good" and punished
them when they were "bad."

Volume 1990 - 2002 Issue


Page 236 of 2049
Skeptical Review Edited by Farrell Till
The Moabite stone, for example, contains an inscription in which the Moabite king Mesha of
2 Kings 3 told of victories that he had won through his god Chemosh who "saved me from all
the kings and let me see my desire upon my adversaries." Later in the inscription, Mesha said
about a victory his forces had won over Israel, "But Chemosh drove him [the king of Israel]
out before me." Pavement slabs in the temple of Urta at Nimud contained an inscription by
the Assyrian king Assur-Nasir-Pal in which he described the massacre of 600 warriors and
3,000 captives he had taken in battle "at the command of the great gods" (Crane Brinton, A
History of Western Morals, p. 48).

If one were to ask Mr. Lavender if he believes that king Mesha had actually been led to
victory by the god Chemosh or that the "great gods" had led Assur-Nasir-Pal in his conquests,
he would no doubt openly scoff at the notion of a pagan god leading an army to victory. How
then does he account for the undeniable fact that inscriptions left behind by these kings
clearly do say that their gods were responsible for their victories? His answer would probably
involve some application of Occam's razor. Chemosh didn't really lead king Mesha to victory.
Mesha just superstitiously believed that it had happened this way. The "great gods" were not
really behind the conquests of Assur-Nasir-Pal. He just thought that they were.

The rule of Occam's razor says that when there are two or more explanations for a
phenomenon, the least incredible one is probably the right one. To apply this principle to the
claims of the pagan kings Mesha and Assur-Nasir-Pal, two possibilities exist: (1) They won
their victories through the intervention of their gods, or (2) they won their victories by means
of superior military forces and tactics and merely thought that their gods had led them to win.
Of these two explanations, the second one is obviously the less incredible and, therefore, the
one rational people would choose to explain the military successes of Mesha and Assur-Nasir-
Pal.

If I were to ask Mr. Lavender to make a choice in the matter--and I am asking him to do that--
I suspect he would choose the second one. If so, why can he not apply the same common-
sense reasoning to the biblical claims that Yahweh led the Israelites to victory in their battles?
King Mesha was a Moabite neighbor to the Israelites and was contemporary to Ahab and
Jehoshaphat. Now when the Bible says that "Yahweh was with Jehoshaphat" and "established
the kingdom in his hand" (2 Chron. 17:3,5 ), inerrantists like Mr. Lavender unhesitatingly
declare their belief that this was absolutely true, yet they scoff at a Moabite inscription that
says the god Chemosh was with Jehoshaphat's neighbor, king Mesha, and established his
kingdom. Why? What is the consistency in such positions as these? If the rule of Occam's
razor makes it unlikely that a primitive war-god was leading Mesha to victory, why wouldn't
the same rule make it just as unlikely that the god Yahweh was helping Mesha's neighbor
Jehoshaphat, just a few miles away, to "establish" his kingdom?

This way of looking at the situation certainly plays havoc with Mr. Lavender's points that
were based on the assumption that God directed the Canaanite massacres. If God had had
nothing to do with these atrocities, as the rule of Occam's razor clearly indicates, then one
doesn't have to be "equal with God" in order to accuse God of wrong in the matter. In fact, the
rational person accuses God of nothing, because he is sensible enough to realize that "God"
was in no way involved in the incidents. The stories simply evolved in a primitive, barbaric
society that believed God was on its side.

Volume 1990 - 2002 Issue


Page 237 of 2049
Skeptical Review Edited by Farrell Till
So we don't have to be omniscient either "to know that what happened to the innocent
children of guilty parents was not the best thing that could have happened" (point 4, p. 6).
Why, if these children had grown to adulthood instead of having been slaughtered in the
Israelite massacres, they might have "become malignant blights in the society of men like
their parents" (p. 6). God couldn't have allowed that, could he? Why, heavens no, he had to
wipe them off the face of the earth before they could grow up to practice idolatry and offer
human sacrifices, as their parents did, so that their land could be occupied by another people
who practiced idolatry and offered human sacrifices (as noted above). About the only thing
Lavender didn't say that inerrantists usually say at this point is that God did these children a
favor by ordering their destruction before the "age of accountability" so that they could go to
heaven rather than hell, where they would have gone had they grown to adulthood. That kind
of thinking is so obscene that it deserves no comment, so I won't offer any.

To return to the serious problem that I skipped above, I have to wonder about the omniscience
of Yahweh. According to Lavender, the Canaanites were so corrupt that Yahweh just had to
vomit them out of the land (p. 6), yet he replaced them with a people who gave him such grief
and disappointment that he eventually had to destroy their national identity too. Why? If
Yahweh were truly omniscient, wouldn't he have known that his plan to establish a righteous
nation in the land would fail and leave him no better off than he had been with the corrupt
Canaanites?

Bibliolaters have an answer to that too. Lavender put it like this: "God was preserving a
lineage through which the Messiah would come and all nations be blessed." What he didn't do
was explain why it was necessary for God to "preserve a lineage" through which he could
give the world a Messiah. Lavender can't give one logical reason why, if God wanted to give
the world a savior, he first had to preserve a lineage "through which the Messiah would
come." At any time in human history, God could have selected a Mayan or an Eskimo or an
Ethiopian or any woman of any ethnic orgin to become the Messiah's mother. It didn't have to
be a Jewish woman. If not, why not?

Even if Lavender could logically explain why not, he would encounter yet another problem. If
the preservation of a lineage was somehow requisite to sending the Messiah, why couldn't
Yahweh have preserved a lineage for him without resorting to massacre and genocide? To
argue that he couldn't preserve the lineage without massacring the Canaanites would be to
argue that God is not omnipotent, because there is at least one thing God can't do. He cannot
preserve a lineage without resorting to massacre. So what all of Lavender's talk about the
preservation of a lineage really amounts to is nothing but desperation theology intended to
explain away an extremely embarrassing problem for the inerrancy doctrine.

One other point that he made requires comment. He said that "(p)unishment was deserved by
the Canaanites, whereas it was not in the case of the Holocaust--which was only a vendetta by
Hitler and the Nazis against the Jews" (point 6, p. 6). Vendetta? Lavender wants to talk about
vendettas? Well, let's just look at a vendetta that was executed Yahweh style. On their journey
to the promised land, the Israelites were resisted by the Amalekites (Ex. 17:8-16 ). This
prompted the infinitely merciful Yahweh to order Moses to "(w)rite this for a memorial in a
book, and rehearse it in the ears of Joshua: that I will utterly blot out the remembrance of
Amalek from under heaven" (v:14 ). Among the final instructions that Moses gave to the

Volume 1990 - 2002 Issue


Page 238 of 2049
Skeptical Review Edited by Farrell Till
Israelites prior to their entry into Canaan was a reminder of Yahweh's promise to exterminate
the Amalekites:

Remember what Amalek did unto thee by the way as ye came out of Egypt; how he met thee
by the way, and smote the hindmost of thee, all that were feeble behind thee, when thou wast
faint and weary; and he feared not God. Therefore, it shall be, when Yahweh thy God hath
given thee rest from all thine enemies round about, in the land which Yahweh thy God hath
given thee for an inheritance to possess it, that thou shalt blot out the remembrance of Amalek
from under heaven. THOU SHALT NOT FORGET (Dt. 25:17-19 , ASV with Yahweh
substituted for Jehovah). Very well, if what the Amalekites had done to the Israelites on this
occasion warranted total extermination as a nation, it should have at least been done to the
generation of Amalekites who had committed the offense. But it wasn't. Not until 450 years
later during the reign of Saul, the first king of Israel, did Yahweh give the orders to massacre
the whole Amalekite nation: Samuel also said unto Saul, Yahweh sent me to anoint thee to be
king over his people, over Israel: now therefore hearken thou unto the voice of the words of
Yahweh. Thus saith Yahweh of hosts, I remember that which Amalek did to Israel, how he
laid wait for him in the way, when he came up from Egypt. Now go and smite Amalek, and
utterly destroy all that they have, and spare them not; but slay both man and woman, INFANT
AND SUCKLING, ox and sheep, camel and ass (1 Sam. 15:1-3 , KJV with Yahweh
substituted for the LORD). I emphasize that this massacre was ordered for something the
ancestors of that generation of Amalekites had done 450 years before! If we could go back in
time 450 years, we would have to wait 65 years for the first permanent European settlement in
North America to be established at Jamestown. Can anyone imagine the moral outrage that
would be expressed if our government should decide to exterminate all Native Americans
thought to be descendants of those who may have in some way resisted the establishment of
the Jamestown settlement? Yet that would be parallel to what was done in the matter of the
Amalekites. So I suggest that if Mr. Lavender wants to talk about vendettas, he forget about
Hitler and the Nazis and give some serious thought to the vendettas that his god Yahweh
executed against the nations in and around Canaan, who had the misfortune not to have been
Israelites, Yahweh's chosen people "above all peoples on the face of the earth" (Dt. 7:6 ).

Of all the attempts that fundamentalists make to defend the inerrancy of the Bible none is
more reprehensible than their insistence that the Old Testament Yahwistic massacres were
morally justified and even "the highest manifestation of the goodness of God." Anyone who
could take such a moral position has earned the contempt of all civilized people--and deserves
it. We are told in the New Testament that Jesus once said to his disciples, "Suffer little
children, and forbid them not, to come unto me..." (Mt. 19:14 ). A rather complex theological
doctrine that I won't attempt to analyze states that Jesus was actually Yahweh in another
"person." If so, he merits commendation for the moral improvement in his character over
what it was in Old Testament times. Back then, he seemed to have a different attitude toward
children, which was simply, "Suffer, little children."

Does the Bible Speak of the Brain?

Volume 1990 - 2002 Issue


Page 239 of 2049
Skeptical Review Edited by Farrell Till
Ed Babinski
Does the Bible or the witless Scarecrow in The Wizard of Oz mention the brain more
frequently? If your answer was the scarecrow, you are right. The Bible mentions a number of
key human organs, such as the heart, blood, bowels, liver, and kidneys, but never mentions
the most important organ of all, the brain. This is not unusual, of course, unless you happen to
view the Bible as an inspired scientific textbook, in which case it would appear to be missing
a bit of vital information.

Of course, it is easy to see how a merely human observer could overlook the brain. It lies
hidden behind a hard bony shell and, even when exposed, maintains a noiseless, placid
appearance. Compare the heart, which beats faster in reaction to anger, love, joy, physical
exertion, etc. Add to this the fact that the heart lies near the center of the body and you arrive
at the ancient conception that it was the primary seat of one's emotions, moral direction (or
misdirection), and according to some, our decision making ability. Indeed, how could early
human observers have avoided being impressed by the pounding heart and racing blood?

In similar fashion, the ancients were also impressed with the "breath"--breathing being an
easily observable external trait associated closely with life, its speed and depth coinciding
with one's emotional and physical state. In fact, both the Hebrew and Greek words for breath
were also used (in their respective languages) to refer to one's soul or spirit.

Historians agree that hundreds or even thousands of years passed before the brain, rather than
the heart, was recognized as the most important organ in the body. An ancient Egyptian
surgical treatise [circa 3,000 B.C.] mentioned how head and neck injuries affected a person's
speech and the use of his limbs. This seemed to surprise the author of the treatise, who
repeated several times that "the injury was in the head." Regardless of the implications of
such observations, the Egyptians, along with the Mesopotamians and the ancient Greek poet
Homer, continued to regard the heart as the primary organ of the soul that harbored
intelligence and feeling. To prepare a pharaoh for mummification, his heart (and other organs
of his torso) had to be embalmed and buried with him, but his brain was removed from his
skull and thrown away.1

Around 460-370 B.C., the ancient Greek philosopher Democritus contested the heart-centered
views found in Homer's Iliad . Democritus wrote: "The brain watches over the upper limbs
like a guard, as citadel of the body, consecrated to its protection," adding, "the brain, guardian
of thoughts or intelligence," contains the principal "bonds of the soul." However, he also
called the heart "the queen, the nurse of anger" and believed that "the center of desire is in the
liver."

The Greek physician Hippocrates (a contemporary of Democritus) enlarged the new brain-
centered theory with clinical observations. He wrote in The Sacred Disease, "Men ought to
know that from the human brain and from the brain only arise our pleasures, joys, laughter,
and jests as well as our sorrows, pains, griefs and tears.... It is the same thing which makes us
mad or delirious, inspires us with dread and fear, whether by night or by day, brings us
sleeplessness, inopportune mistakes, aimless anxieties, absent-mindedness and acts that are
contrary to habit...." That the Hebrews took for granted that the "heart, bowels, and kidneys"

Volume 1990 - 2002 Issue


Page 240 of 2049
Skeptical Review Edited by Farrell Till
were the seats of man's emotional and moral impulses would no doubt have raised a wry smile
on Hippocrates' face. But that is to get ahead of ourselves.

Plato [428-348 B.C.] in his dialogue entitled Timaeus argued that the intellectual part of the
soul was contained in the head. Then Aristotle [384-322 B.C.], Plato's student and successor,
reverted to the heart-centered view of the soul. Aristotle observed the way that blood vessels
from all over the body converged toward the heart and how the heart reacted visibly to being
touched while the brain did not. Furthermore, the lower animals, like worms, insects, and
shellfish, all had pulsating heartlike organs but lacked anything resembling the vertebrate
brain. Lastly, Aristotle pointed out that a chicken's body exhibited a life of its own after being
separated from the head. It was thus obvious to Aristotle that "the seat of the soul and the
control of voluntary movement--in fact of nervous functions in general-are to be sought in the
heart. The brain is an organ of minor importance, perhaps necessary to cool the blood" (De
Motu Animalum).

Three hundred years later, the Roman philosopher Lucretius wrote in a similar Aristotelian
vein, "The dominant force in the whole body is that guiding principle which we term mind or
intellect. This is firmly lodged in the midregion of the breast. Here is the place where fear and
alarm pulsate. Here is felt the caressing touch of joy. Here, then, is the seat of the intellect and
mind" (On the Nature of Things, Book III, circa 55 B.C.).

But even though philosophers like Aristotle and Lucretius were heart-centered, the tradition
of Greek physicians, beginning with Hippocrates, remained brain-centered. In the 3rd century
B.C, Herophilus and Erasistratus dissected thousands of bodies and demonstrated that nerves
were different from blood vessels and that they originated not in the heart, as Aristotle
thought, but in the brain or the spinal cord. Then almost five hundred years after Herophilus'
day, the Greek physician and philosopher Galen [130-200 A.D.] experimented and established
brain physiology as a science. He demonstrated that the brain played the central role in
controlling bodily and mental activity.

However, because of the influence of Aristotle on medieval scholars and the added bonus that
his view of the primacy of the heart agreed more with biblical descriptions than brain-
centered views, heart-centeredness survived until the 16th century. As Martin Luther, the
father of protestantism, put it, "Faith is under the left nipple." (According to "folk anatomy,"
the heart lies under the left nipple.) Or notice the feeling of uncertainty voiced in
Shakespeare's Merchant of Venice: "Tell me, where is fancy bred, in the heart, or in the
head?" And in Balzac's "Heartaches of an English Cat," the "aches" have nothing to do with
heart disease. After the 1700's, new figures of speech arose, based on an appreciation of the
brain's central importance. Today we are more likely to warn someone not to "lose his head"
rather than to be "strong of heart," and we admire people with "brains." Now compare the
Bible. It was written at a time when Hippocrates and other Greek physicians knew better, but
throughout the length and breadth of it, emotional and moral behaviors are related foremost to
the heart, the bowels, and the kidneys, rather than to the brain.

Of course, the Bible speaks of the "head." It is a place to be anointed and crowned and where
the priests wore their miters (turbans), but such practices were shared by heart-centered

Volume 1990 - 2002 Issue


Page 241 of 2049
Skeptical Review Edited by Farrell Till
cultures, so they cannot be used to support any theory that the authors of the Bible recognized
the primary importance of the brain.

There is one verse in the Bible that has been cited as some form of recognition of the brain.
Daniel 2:28 speaks of "visions of thy head upon thy bed." Of course, even if this were an
instance of brain-centeredness, it should be remembered that historical scholarship assigns the
book of Daniel a date of composition later than any other book in the Old Testament. Also,
the verse merely refers to "head-visions," and it could be referring obliquely to the fact that
one's eyes are in one's head or, perhaps, to the fact that one sees images/visions in his head.
However, in neither case does that imply that what is in one's head is anything more than a
screen for visions to play themselves out upon. That hardly gives full recognition to what lies
inside one's head.

Of course, the Bible refers to the "mind." However, aside from implying that the mind is not
identical with the heart (cf. Mt. 22:37 ; Mk. 12:30 ; Lk. 10:27 ), the Bible does not go on to
state where the seat of the mind is located. Neither are "mind" and head used in conjunction
with one another. Other ancient cultures also referred to the mind without specifying to which
organ it was related. It appears to have been an entity like the soul-breath, whose location was
never specified. Nevertheless, certain organs, to them, seemed especially capable of
influencing and directing one's emotions and morality, and those were specified.

The ancients (including the Hebrews) all agreed that one such organ, the primary one, in fact,
was the heart. They used the word heart repeatedly, attaching enormous emotional and moral
significance to its behavior. The Bible emphasizes how the heart "deviseth a man's way,"
"inspires speech" "believes," "is joyful," "is deceitful," "is good" (Prov. 16:9 ; Mt. 12: 34 ;
Rom. 10:10 ; 1 Chron. 16:10 ; Jer. 17:9 ; Lk. 6:45 ). This resembles what the ancient
Egyptians, Babylonians, and Greeks (those Greeks who were not physicians, Homer and
Aristotle) believed and taught. Besides the heart, the Bible also focuses (to a lesser extent) on
the emotional and moral significance of the bowels and kidneys. Here are some of the verses
in the King James Bible in which the Greek and Hebrew terms for bowels and kidneys are
literally translated:

My bowels are troubled for him; I will surely have mercy upon him, saith the Lord (Jer.
31:20).
Be ye straitened [restrained] in your own bowels (2 Cor. 6:12).
I long after you in the bowels [affection] of Christ (Philip. 1:8).
(T)he bowels of the saints are refreshed.... (R)efresh my bowels in the Lord (Philemon 7:20).
(S)hutteth up his bowels of compassion (1 John 3:17).
Thus my heart was grieved, and I was pricked in my reins [Latin, renes, which means
kidneys, a literal translation of the Hebrew] (Ps. 73:21).
My reins [kidneys] also instruct me in the night seasons (Ps. 16:7).
Oh let the wickedness of the wicked come to an end; but establish the just: for the righteous
God trieth the heart and reins [kidneys] (Ps. 7:9).
Yea, my reins [kidneys] shall rejoice when my lips speak right things (Prov. 23:16).
I am He [God] which searcheth the reins [kidneys] and hearts (Rev. 2:23).

Volume 1990 - 2002 Issue


Page 242 of 2049
Skeptical Review Edited by Farrell Till
The Talmud (Berakhoth 61a) says that one kidney prompts man to do good, the other to do
evil. The kidneys (among other organs, yet excluding the brain) were especially reserved for
Yahweh and sacrificed to Him as a burnt offering (Lev. 3:4-5). Even if the Hebrews regarded
this insight into the kidneys as "pure poetry" (which is doubtful, based on historical
comparisons, and since figures of speech have to originate from ideas), it is a poetry that no
longer survives or interests mankind. In fact, in the above verses the Hebrew word for kidneys
has been translated soul in modern English Bibles to avoid cumbersome explanations of why
the ancient Hebrews attributed moral significance to a person's kidneys.

Or consider the twin biblical notions that "the life is in the blood" and "without the shedding
of blood there is no remission of sins" (Lev. 17:11-14 ; Heb. 9:22 ). Because of such ideas, the
ancient Hebrews sacrificed in a bloody fashion many thousands of animals, and Christians
came to view Jesus' "blood" sacrifice as necessary for the forgiveness of sins and the drinking
of Jesus' "blood" (in symbolic and/or mystical fashion) as partaking of his "life." This is in
obvious contrast to scientific consensus, which agrees that human "life" is not primarily "in
the blood" but in the brain and nervous system.

Indeed, one's primary "life" ends with the total and permanent cessation of brain activity, even
if no blood is shed in the process, as in cases of poisoning, asphyxiation, or electrocution. In
fact, as the comedian Lenny Bruce used to jibe, "If Jesus had been executed in the twentieth
century instead of the first, Christians would be wearing little electric chairs around their
necks." Maybe so, but if you were a fundamentalist Christian, I suppose there would still have
to be a spear in the side of an electrocuted Jesus. There's gotta be some "blood" shed for
forgiveness. (I am not seeking to mock religion but merely posing questions to those who
claim the Bible can be interpreted both literally and scientifically.)

"Science" in biblical times was based on apparent , not literal, truths. The earth appeared to be
the flat, firm foundation of creation. The heavens appeared to be stretched out above the earth
like a tent or canopy. The heart, bowels, and kidneys (and not the brain) appeared to be
intimately linked with one's emotions, morality, and decision-making processes. Even ancient
creation accounts (Egyptian, Sumerian, Babylonian, Hebrew, etc.) reflected this attitude
toward "science." They thought that God (or the gods) had created all of the animals and
plants as they then appeared and that their offspring would not appear any different no matter
how many generations should pass. Thus, for the ancient Hebrews, crude "scientific" theories,
based on superficial appearances, dictated the Hebrew view of reality and their subsequent
figures of speech.

According to an article in Psychology: A Journal of Human behavior, Warren Gorman and


Lawrence Edwin Abt questioned 110 females between the ages of eighteen and twenty to
determine the concepts they had of the form and function of their bodies. Their article "Is the
Brain the Most Important Organ?" (Aug. 1964, pp. 2-11) contained some interesting
questionnaire results. For instance, when asked, "In Children, what is the most important
organ? In adults?... In old people?..." the subjects rated the heart the "most important organ"
in old people and the sex organs the most important in adults, whereas in children they rated
the brain, heart, and digestive organs as "equally important." (Only five subjects out of 110
responded "brain, brain, brain" to the three parts of this question!)

Volume 1990 - 2002 Issue


Page 243 of 2049
Skeptical Review Edited by Farrell Till
When asked, "What is the most important function of the body?" the subjects answered
"breathing," with "circulation or heart function" next. These first two questions of the survey
along with their replies demonstrate how easy it must have been for the ancients to have
overlooked the primary importance of the brain. Even 20th century questionnaire respondents
continue to overlook the primary importance of "brain function"! However, the response of
the subjects to the next question left no doubt that a side gap exists between ancient and
modern views of the brain. When asked, "What part or function of the body is most intimately
lined with your emotions?" the brain easily achieved first place. Such a result demonstrates
how deeply modern scientific knowledge has penetrated our culture and superseded
heart/bowel/kidney-linked descriptions of human emotions found in the Bible and other
ancient works.2

ENDNOTES
1 For much of the historical data on how the brain eventually came to be viewed as more
central to one's life and "soul" than the heart, I am endebted to Neuronal Man (New York:
Random House, 1985), an English translation of Jean-Pierre Changeux's book L'Homme
Neuronal.

2 If the results of my inquiry into the question "Does the Bible Speak of the Brain?" are
deemed unsatisfactory by those who adhere to the literal words of the Bible and if those same
adherents should blame my "blasphemous" conclusions on my "hard heartedness" toward
God, then I should like to reply in all sincerity and politeness that my heart is as "hard" as
their heads are soft.

(Ed Babinski's address is 109 Burwood Drive, Simpsonville, SC 29681-8768).

Volume 1990 - 2002 Issue


Page 244 of 2049
Skeptical Review Edited by Farrell Till

The Skeptical Review


Volume Four, Number Two - 1993
Farrell Till, editor

• God Was With Him


When people escape disaster, believers often claim, "God was with him." What, then,
are we to think of those who don't escape disaster? If God gets the credit, shouldn't he
also get the blame?

• An Example of "Prophecy Fulfillment"


An example of the extremes that fundamentalists will go to in their search for fulfilled
prophecy in the Bible.

• Silence in Fantasyland
The Skeptical Review has a policy of giving free space to inerrantists in which they
can respond to rebuttals-- so why aren't there any takers?

• Any Loophole Will Do


The how-it-could-have-been scenario is a common tactic that fundamentalists use to
"explain" passages in the Bible that pose serious problems for the inerrancy doctrine.
But it is an invalid tactic, as this article shows.

• Common Sense and Noah's Flood


Some absurdities in the tale of Noah's Ark.

• What is Wrong With Any-Loophole-Will-Do Hermenuetics


More problems with the implausible how-it-could-have-been scenarios offered by
inerrantists.

• Reader Reaction

• A Virgin-Birth Prophecy
Prophecy is a muddy science, and Bible prophecy more muddy than most. One

Volume 1990 - 2002 Issue


Page 245 of 2049
Skeptical Review Edited by Farrell Till
example of this is the prophecy of Isaiah 7:14, which inerrantists claim is a prediction
of the virgin birth. As it turns out, it is no such thing.

• A Challenge to Josh McDowell

God Was With Him


In January, newspapers in Central Illinois reported the tragedy of a young man who was
seriously maimed in a train accident. Wearing Walkman earphones as he strolled along a
railroad track, the boy was apparently unable to hear a slow-moving train approaching in
reverse from behind him. Despite frantic shouts from people nearby who tried to warn him,
the boy stepped onto the tracks at the last moment and was struck by the train. He survived
but lost his left arm and right leg in the accident.

One of the witnesses who tried to warn the young man said, if quoted correctly in a local
newsarticle, "He could have been killed. God was with him."

A tragedy like this needs no commentary; the sympathy of everyone goes out to the young
man. The comment of the witness, however, is quite another thing. Unfortunately, her
reaction is too typical of those who are parties to incidents like this one. If a tornado or
earthquake devastates a community or if an airplane or train crashes, those who escape death
are often too eager to think that the hand of God reached down especially to aid them. "The
Lord was with me," or, "Somebody up there must like me," are typical comments that the
survivors make.

Reasoning like this (if we can even call it "reasoning") fails to recognize that if the hand of
God reached down to aid the survivors of a tragedy, then it is necessarily true that the hand of
God did NOT reach down to aid those who died. If someone whose home escaped serious
damage in a tornado attributes his good fortune to the providence of God who directed the
funnel around his house, then surely he can see that God cared more for him than those whose
houses were destroyed. In the case of the young man who was maimed in the train accident, if
God was truly with him, then why wasn't God with him in a way that would have spared him
the crippling injuries he suffered? Why couldn't God have influenced him in some way to
remove his earphones a minute or so before the accident? Or why couldn't God have caused a
power failure that would have taken the radio off the air or any one of a dozen things we
could think of that would have enabled the boy to hear the witnesses yelling at him? The logic
of this witness requires her to believe that God wanted this boy to go through life permanently
crippled, because she believes that "God was with him" but not with him enough to spare him
completely from crippling injury.

The purpose of this article isn't to argue the question of divine providence but to take notice of
the appalling lack of logic in the thinking of Bible believers. The woman who thought that
God was with the boy who was maimed in this accident identified herself as a Lutheran, but
she could have been a Baptist or Methodist or Catholic as well, for in matters concerning

Volume 1990 - 2002 Issue


Page 246 of 2049
Skeptical Review Edited by Farrell Till
traditional theistic beliefs, we hear such irrational thinking as hers from all brands of
Christians. The more of it we hear, the more we wonder if there is any hope at all of ever
building a society that is relatively free of superstition.

To say this is not to retract what was said in "Living on Borrowed Time" (Winter 1993, p. 1).
We still believe that Bible fundamentalism will continue to wane until it ceases to exist, but,
as we noted in the article, its extinction will be a long time coming. We live in a tabloid
society that makes astrology and physic predicting profitable enterprises to engage in. The
same mentality that sustains them sustains Bible fundamentalism, so certainly Bible
fundamentalism is going to be with us for a long time.

When we consider the damage that religious fundamentalism inevitably inflicts upon the
societies in which it thrives, it isn't very comforting to think that social progress in our
country will continue to be impeded by those who seek to impose their interpretations of
"God's inspired word" upon us all. Something that rationalists can do that might hasten the
demise of Bible fundamentalism is to get involved in teaching people how to think. A
regrettable characteristic of our society is the emphasis that we put on teaching what to think.
Certain ideas and attitudes become socially, politically, and religiously correct, and children
in particular are indoctrinated in them with little or no attention to logical analysis to
determine if they are indeed the most suitable and beneficial ideas and attitudes to espouse.
For this reason, people grow up to believe that God exists, that Jesus of Nazareth was his son,
that the Bible is his inspired word, that God is responsible for the apparently fortuitous events
that happen to us, etc., etc., etc. They believe these things not for sound rational and well
thought-out reasons but simply because these ideas represent what they have been taught to
believe.

In this publication, we have identified and discussed more than enough problems to convince
rational thinkers that the Bible just cannot be "the inspired word of God." We will continue to
publish articles with that intention in mind, but perhaps it is time to devote at least some space
to articles that discuss how to think and how to apply logical thought processes to the biblical
text. Some of the articles in this issues have been written and selected with that intention in
mind. Similar ones will follow in later issues.

An Example Of "Prophecy Fulfillment"


Farrell Till
What about all of those amazing examples of prophecy fulfillment? This is a question that
fundamentalists almost always resort to in trying to defend their claim that the Bible is the
verbally inspired word of God. Their question can easily be answered with another question.
What prophecy fulfillments? Upon careful examination, these so-called prophecy fulfillments
invariably turn out to be arbitrary distortions or misapplications of vaguely written or highly

Volume 1990 - 2002 Issue


Page 247 of 2049
Skeptical Review Edited by Farrell Till
symbolic OT scriptures. They are "prophecy fulfillments" only in the fertile imagination of
fundamentalists who desperately want them to be prophecy fulfillments.

An example of the extremes that fundamentalists will go to in their search for prophecy
fulfillment occurred in "The Blind Ruler," a short article by Wayne Jackson in the December
1992 issue of Reasoning from Revelation, a simplistic insertion that accompanies its parent
publication Reason & Revelation:

Even though he was a captive in Babylon, the prophet Ezekiel uttered oracles regarding his
brethren who were as yet in the land of Canaan. One of his prophecies had to do with
Zedekiah, who was serving as the "prince in Jerusalem" (Ez. 12:10 ). Zedekiah had been
appointed ruler to replace Jehoiachin, when the latter was taken into Babylon in 597 B.C.
The prophet anounced that the "rebellious house" of Israel, along with the haughty ruler,
would be taken into captivity (vss. 11,12 ). Concerning Zedekiah specifically, Ezekiel
(speaking for God) declared: "My net also will I spread upon him, and he shall be taken in my
snare; and I will bring him to Babylon to the land of the Chaldeans; yet shall he not see it,
though he shall die there" (vs. 13).
This prophecy almost seems to contain a discrepancy. If the king is to be brought to the land,
surely he will see it. That appears to be common sense. Or is it? The fact is, the prediction is
extremely precise.
When the Babylonians came against Jerusalem in 586 B.C., Zedekiah fled the city, hoping to
escape the in- vaders. He was pursued, however, and captured near Jericho. He was then
transported to Riblah (north of Canaan). There he was forced to witness the execution of his
sons. This was the last scene he was to view upon the Earth, for his eyes were put out, and he
was led away to Babylon in chains. Imprisoned there, he finally died in that distant land (II
Kings 25:6-7; Jeremiah 39:7; 52:11).
Ezekiel's prophecy was carried out to the letter. Fulfilled prophecy is a convincing evidence
for the integrity of the Bible (p. 24).

I don't really pick on Mr. Jackson intentionally. It is just that his determination to "prove" the
inerrancy of the Bible provides such an excellent source of fundamentalist nonsense. This
example of prophecy fulfillment that he believes he has found is no exception. Insofar as the
Bible relates them, Mr. Jackson has accurately summarized the circumstances of Zedekiah's
capture and treatment by the Babylonians. His interpretation of them is where he enters the
Never-Never Land of fundamentalist speculation and wishful thinking. He wants to see
prophecy fulfillment in the Babylonian treatment of Zedekiah, yet the alleged prophecy that
he quoted (Ezekiel 12:10 ) doesn't mention Zedekiah. In fact, the entire book of Ezekiel
makes no mention of Zedekiah by name. Jackson's "proof text" refers only to a "prince in
Jerusalem," and at this time there were many other princes in Jerusalem. Jeremiah, another
prophet contemporary to Zedekiah, made many references to "the princes" in Jerusalem
(17:25 ; 24:8 ; 26:10 ; 36:12-19 ). Mr. Jackson's task, then, is to prove, not speculate, that
Ezekiel's "prince in Jerusalem" was in fact Zedekiah and not someone else.

Certainly the circumstances of Zedekiah's capture, treatment, and imprisonment, as they are
related in the passages Jackson cited, appear to "fulfill" what was said in Ezekiel 12:10 about
the "prince in Jerusalem"; however, even if Ezekiel was in fact referring to Zedekiah here, we
have every reason to believe that the statement was written after the fact, and if we allow that

Volume 1990 - 2002 Issue


Page 248 of 2049
Skeptical Review Edited by Farrell Till
advantage, anyone can be a prophet. In the total context of the passages that Jackson cited, we
learn that Nebuchadnezzar appointed Zedekiah to be a puppet king in Jerusalem after the
overthrow of Jehoiachin. If we accept Mr. Jackson's chronology, which appears to be right,
Zedekiah became king in 597 B.C., and reigned for 11 years (2 Kings 24:18 ). In Zedekiah's
ninth year, Nebuchadnezzar besieged Jerusalem again (25:1 ) and finally captured it two years
later (25:2 ). The famous Babylonian captivity of the Jews began in 597 B.C. with the
overthrow of Jehoiachin, and then in 586 B.C., when Zedekiah was defeated, the "rest of the
people" who had remained in Jerusalem were also "carried away captive" (25:11 ).

The prophet Ezekiel was evidently taken to Babylon with the first wave of prisoners, because
he identifies himself at the beginning of his book as one of the captives by the River Chebar
"in the fifth year of King Jehoiachin's captivity" (1:1-2 ). This would have been also the fifth
year of Zedekiah's puppet reign. In other words, by his own admission, Ezekiel did not begin
writing his book until just a few years before Zedekiah's overthrow, and, as we will later see,
Mr. Jackson's own criteria for evaluating prophecy fulfillment would disqualify Ezekiel 12:10
as an example of fulfillment even if he could unequivocally establish that (1) the statement
was referring to Zedekiah and (2) it was written before the fact.

I'll return to the first of these problem areas later to pit Mr. Jackson against himself, but before
doing that, let's notice that problem number two kicks the props right out from under the
claim that "Ezekiel's prophecy [in 12:10 ] was carried out to the letter." As just noted, Ezekiel
began his book in the fifth year of the captivity, but he didn't finish it until at least the 25th
year of the captivity:

In the twenty-fifth year of our captivity, at the beginning of the year, on the tenth day of the
month, in the fourteenth year after the city was captured, on the very same day the hand of
Yahweh was upon me (40:1).

Ezekiel was taken into captivity at the time of Jehoiachin's overthrow. Then eleven years
later, Jerusalem fell to Babylon a second time. The "twenty-fifth year of our captivity," then,
would have been "the fourteenth year after the city [Jerusalem] was captured." However, it
would have also been 14 years after Zedekiah's capture and the treatment he was accorded as
summarized in Mr. Jackson's article. Obviously, then, before he finished his book, Ezekiel
had had the opportunity to know exactly what had happened to Zedekiah. So what assurance
can Jackson give us that Ezekiel did not write his book to make it appear that he had foretold
the fate of Zedekiah? I can show Mr. Jackson prophesies in the Book of Mormon that he
would summarily reject for the same reason that all rational people will reject his claim that
Ezekiel precisely predicted the fate of Zedekiah.

For now, I will just let Mr. Jackson refute his own argument. Writing in the Christian
Courier, Jackson once listed three criteria of valid prophecy: In order for prophecy to be
valid, the following criteria must obtain. It must involve:

(a) Proper timing (i.e., significantly preceding the fulfillment); (b) Specific details--not vague
generalities or remote possibilities; (c) Exact fulfillment--not merely a high degree of
probability" ("The Holy Bible: Inspired of God," May 1991, p. 2, emphasis added).

Volume 1990 - 2002 Issue


Page 249 of 2049
Skeptical Review Edited by Farrell Till
By Jackson's own admission, then, Ezekiel 12:10 cannot be a valid prophecy. It clearly lacks
"proper timing." As already noted, Ezekiel didn't begin writing his book until six years before
Zedekiah's capture, so even if 12:10 was a reference to Zedekiah and even if the statement
was made prior to his capture, it did not "significantly precede the fulfillment." Ezekiel could
have appraised the political climate of the time to make an educated guess of what was likely
to happen to Jerusalem and to its king who was presumptuous enough to rebel against
Nebuchadnezzar (2 Kings 24:20 ).

Likewise, Ezekiel 12:10 was too vaguely stated to meet Mr. Jackson's own standards of "valid
prophecy." To say that "the prince in Jerusalem" would be taken to Babylon but would not see
it, even though he would die there, is hardly my idea of "specific details." It reeks with "vague
generalizations" and "remote possibilities." If Ezekiel had wanted to utter an unequivocal
prediction of Zedekiah's fate, why didn't he say something like this: "Nebu- chadnezzar will
besiege Jerusalem, and king Zedekiah will try to escape by night. The Chaldeans will capture
him in the plains of Jericho and take him before Nebuchadnezzar at Riblah, where he will be
forced to watch the execu- tion of his sons. Then the Chaldeans will blind Zedekiah and
imprison him in Babylon, where he will die"? If just one prophet had made a prediction half
as precise as this, which could be proven to have been made significantly before the fact, that
would have put a lot of punch into the prophecy-fulfillment argument. As it is, the Hebrew
prophets left us nothing but vague, symbolic generalizations that are as meaningless as the
horoscopes that are published daily in our newspapers. That doesn't say much for the
omniscient, omnipotent deity who allegedly inspired those prophets.

Mr. Jackson wants to find prophecy fulfillment in the fate of Zedekiah, but he conveniently
said nothing about an embarrassing prophecy failure that Jeremiah made concerning
Zedekiah. During the Babylonian siege of Jerusalem, the word of Yahweh came to Jeremiah
and led him to make this remarkable prediction about Zedekiah's eventual fate:

Thus says Yahweh, the God of Israel, "Go and speak to Zedekiah king of Judah and tell him,
'Thus says Yahweh: "Behold, I will give this city into the hand of the king of Babylon, and he
shall burn it with fire. And you shall not escape from his hand, but shall surely be taken and
delivered into his hand; your eyes shall see the eyes of the king of Babylon, he shall speak
with you face to face, and you shall go to Babylon."' Yet hear the word of Yahweh, O
Zedekiah king of Judah! Thus says Yahweh concerning you: "You shall not die by the sword.
You shall die in peace; as in the ceremonies of your fathers, the former kings who were before
you, so shall they burn incense for you and lament for you, saying, 'Alas, Lord!' For I have
pronounced the word, says Yahweh" (Jer. 34:2-5, emphasis added).

Zedekiah's fate (according to the Bible) was as Mr. Jackson briefly summarized it. When the
Chaldeans made a breach in the wall of Jerusalem, Zedekiah tried to escape by night. He was
captured and taken before Nebuchadnezzar, who forced him to watch the execution of his
sons. Then the Chaldeans put out Zedekiah's eyes, took him to Babylon, and imprisoned him
till the day of his death ( 2 Kings 25:6 ). I wonder if this is Mr. Jackson's idea of dying in
peace. I wonder too when incense was burned for Zedekiah and lamentations were made for
him as in the ceremonies of his fathers, "the former kings who were before him." Are we to
believe that the Chaldeans permitted this kind of funeral ceremony in Babylon for a captive
king who had been accorded the treatment just described? Are we to believe that the captive

Volume 1990 - 2002 Issue


Page 250 of 2049
Skeptical Review Edited by Farrell Till
Hebrews in Babylon would have even wanted to so honor the king who had presided over the
downfall of their nation?

Prophecy fulfillment indeed! Anyone who sees prophecy fulfillment in the fate of king
Zedekiah must be desperate for something to shore up the badly battered inerrancy doctrine.

Silence in Fantasyland
Although we have a standing offer of free space for all writers who wish to respond to our
rebuttals of their articles reprinted from fundamentalist papers, we have had no takers. In the
last issue, we reprinted Clarence Lavender's article "Was It Morally Right for God to Order
the Killing of the Canaanites?" along with our reply to it, but we have heard nothing from
him. The issue before, we reprinted Dave Miller's article "Why I Believe in the Inerrancy of
the Scriptures" with our rebuttal, but he has not accepted our offer to publish his reply.

On the preceding pages, we reprinted Wayne Jackson's article "The Blind Ruler" with our
response to his claim of prophecy fulfillment in the fate of Zedekiah. Since Jackson has
repeatedly declined our invitations to respond to articles we have published about his
fundamentalist assertions, we predict that he will answer with silence this time too.

Why do these champions of Bible inerrancy conduct themselves in this way? Their inerrant
"word of God" commands them to "contend earnestly for the faith" (Jude 3 ) and to be "ready
always to give answer to every man" that asks them a "reason for the hope" that is in them (1
Peter 3:15 ). Well, we have asked Miller, Lavender, Jackson, and many others to give us their
answers, but they refuse to do so. Why?

The answer is probably as simple as the one suggested in a recent telephone call from a man
who has left the Worldwide Church of God. In referring to our offer of free space to
inerrantists and their consistent refusals to accept, he said that their silence is an admission
that they cannot refute our arguments. "If they thought they could," he said, "then they
would."

We think he is right. Our offer is still open to any inerrantist who will accept it.

Any Loophole Will Do


Farrell Till

Volume 1990 - 2002 Issue


Page 251 of 2049
Skeptical Review Edited by Farrell Till
The how-it-could-have-been scenario is a common tactic that fundamentalists use to "explain"
passages in the Bible that pose serious problems for the inerrancy doctrine. They like to
believe that as long as they can suggest an interpretation that removes the spectre of
contradiction or discrepancy from a problem passage, then they have preserved the inerrancy
doctrine, no matter how far-fetched or unlikely the interpretation may be. They insist that just
as long as the interpretation is not absolutely impossible, they are justified in believing that it
could have happened or could have been the way the interpretation explains away the
problem.

TSR readers have often encountered this tactic in the rebuttal articles of our fundamentalist
contributors. The most recent examples of it occurred in Bill Lockwood's articles written in
response to what I had said about the problem of Sarah's seminal emission mentioned in
Hebrews 11:11 . A problem incidental to this passage was the claim that Sarah "counted him
[Yahweh] faithful who had promised [that she would have a son]." I showed that the Old
Testament record of Sarah's response to the promise had depicted Sarah as anything but a
believer that the promise would be fulfilled, because when Yahweh made the promise, she
laughed at the absurdity of her having a child in her old age (Gen. 18:9-15 ). With no textual
evidence at all to support him, Lockwood proposed, as a how-it-could-have-been way out of
the problem, that Sarah had simply "changed her mind" ("Sarah's Power to Conceive: a
Response, Summer 1992, pp. 14-15). On the actual matter of Sarah's power to "make a
deposit of semen," Lockwood argued that the Greek expression katabole spermatos didn't
have to be interpreted according to its literal or face-value meaning. "(I)f you don't have a
can't-possibly-be-anything-else case," he said, "you don't have a case against the Bible"
(second reply, Autumn 1992, p. 13).

The only thing a statement like this proves is that inerrantists cannot think logically. I don't
have to prove a can't-possibly-be-anything-else case in this or any other matter that challenges
the inerrancy doctrine, because I make no extraordinary claims about the Bible when I
question the inerrancy claim. To the contrary, the inerrantist is the one who must establish
can't-possibly-be-anything-else cases. The matter is as simple as what William Lindley said in
his letter specifically in response to Lockwood's attempt to shift the burden of proof to those
who question the inerrancy doctrine: "(T)he notion that a written text is supernaturally free of
error is so strange that the burden of proof ought to be the other way" ("Reader Reaction," p.
12).

Lindley has merely recognized the principle that says that extraordinary claims require
extraordinary proof. When I say that the face-value meaning of Text A contradicts the face-
value meaning of Text B, I am making no extraordinary claim, because contradictions are
commonplace in written documents. On the other hand, when inerrantists say that a collection
of 66 books containing thousands of words, written by different people, in different
languages, is totally and completely free of mistakes of any kind, that claim is so
extraordinary that it requires extraordinary proof. Since there is nothing at all extraordinary
about how-it-could-have-been explanations, such "proof" really amounts to no proof. An
apologist can't just say, "Well, Text A could have meant this, whereas Text B could have
meant that"; he must present a couldn't-possibly-mean-anything-else case or his argument to
preserve inerrancy fails.

Volume 1990 - 2002 Issue


Page 252 of 2049
Skeptical Review Edited by Farrell Till
When, for example, I point out that Matthew's genealogy of Jesus differs substantially from
Luke's, I am only stating an obvious conclusion that anyone can reach by reading both
genealogies. Inerrantists will say, of course, that Matthew traced the genealogy of Jesus
through Joseph, whereas Luke traced it through Mary, but there isn't a hint of any kind in the
entire book of Luke that he intended his genealogy to be so understood. In the absence of
evidence that this was what Luke intended, the inerrantist accomplishes nothing by merely
saying, "Well, it could have been this way, so to have a case, you must prove that it couldn't
possibly have been this way." No, a thousand times no! Such a position as this is not at all
compatible with the nature of evidence. A lot of things could have been or could have
happened, but just because something could have happened doesn't mean that it did happen. A
copyist could have corrupted the text after Luke wrote it. Joseph could have been orphaned at
an early age and then adopted by another family, and so Matthew traced the genealogy
through the biological father and Luke through the adoptive father. Either one of these would
serve as well to "explain" the inconsistencies as the traditional claim that Matthew traced the
genealogy through Joseph whereas Luke traced it through Mary, but none of the explanations
would work unless couldn't-possibly-mean-anything-else cases could be made for them.

When inerrantists use the how-it-could-have-been tactic, they are simply resorting to
desperation hermeneutics. The quest of those who engage in it is not to discover the intended
meaning of the Bible text but to preserve a cherished belief. In the final analysis, the meaning
of a problem passage isn't important. Importance lies in the ability of an interpretation to
explain away a discrepancy or contradiction, and for that reason, any interpretation will do
that resolves the problem of discrepancy, no matter how far-fetched or ridiculous it may be.
The absurdities that Bible fundamentalists have proposed in the name of this how-it-could-
have-been tactic are too numerous to analyze in one article, but one has only to read a book
like Gleason Archer's Encyclopedia of Bible Difficulties to see this desperation hermeneutics
at work. I have had skeptics and freethinkers tell me that reading such works as this and
seeing the absolute absurdity of the how-it-could-have-been tactics used in them provided the
final impetus that they needed to cross the line separating their belief from skepticism.

In a booklet that examined discrepancies in the four gospel accounts of Peter's denials, Dave
Matson identified nine inconsistencies in the way the story was told. These concerned such
matters as the actual nature of the denials, their locations, their timing, etc. On the matter of
Peter's location when he made the denials, the gospel writers seemed confused about where
Peter was at the time, especially when his second and third denials occurred. Matthew and
Mark, for example, said that Peter left the campfire in the courtyard after his first denial and
went "out into the porch," where his other denials were made (Mt. 26:71-73 ; Mk. 14:68-70 ),
but Luke and John wrote the story as if Peter's second and third denials were also made as
Peter was standing by a fire (Lk. 22:55-60 ; Jn. 18:18-27 ). As a way out of the problem,
inerrantists have suggested that there were two campfires on the scene so that even though
Peter moved about, he was still in the proximity of a fire. In his analysis of this "explanation,"
Matson made some excellent observations about basic fallacies inherent in any-loophole-will-
do hermeneutics.

That section of his booklet has been reprinted in the following article .

Volume 1990 - 2002 Issue


Page 253 of 2049
Skeptical Review Edited by Farrell Till

Common Sense and Noah's Flood


Farrell Till
"The Impossible Voyage of Noah's Ark" by Robert A. Moore (Creation/Evolution, Winter
1983) is the most thorough exposure of absurdities in the universal-flood myth that I have
ever read. Anyone who can read it and still believe that such a flood actually happened is
surely beyond the reach of common sense.

In "Noah's Ark: Fact or Fable," Chris McGowan said, "I find it... hard to believe that I should
be sitting at my desk in 1982 documenting the reasons why the flood could not have happened
according to Genesis" (In the Beginning, Prometheus Books, 1984, p. 54). I feel much the
same way, but because so many sincere people have been duped by fundamentalist preachers
into believing that everything written in the Bible has to be true, it is still necessary at times to
review the absurdities in the flood story.

One of these absurdities concerns the amount of water that would have been required to
produce a flood like the one described in Genesis. The Bible claims that "the waters prevailed
exceedingly upon the earth, and all the high mountains that were under the whole heaven
were covered" (Gen. 7:19 ). The language is explicit enough to discount the theory that the
flood was localized, because a limited flood could not have covered ALL the high mountains
under the WHOLE heaven. After the waters had receded, the ark came to rest on "the
mountains of Ararat" (8:4 ). Since water seeks its own level, it wouldn't have been possible
for a localized mound of water to rise to that height without dropping to a uniform level.

If Bible fundamentalists are honest, then, they will have to admit that the Genesis writer
intended his story to be understood as an account of a flood that had covered the entire earth.
In other words, he expected us to believe that within the space of forty days enough water
rained down or rose from "the fountains of the deep" to cover the highest mountain on earth to
a depth of 15 cubits (7:20 ). In prescientific times when people knew very little about
geography and meteorology, a claim like this could find general acceptance, but in modern
times only the very credulous can believe it.

To understand this, we have only to analyze the story in terms of the number of inches of rain
per minute that would have had to fall on the entire surface of the earth to produce the results
described in Genesis 7-8 . We now know, for example, that Mount Everest is the highest
mountain "under the whole heaven." It reaches an altitude of 29,028 feet, which would be a
height of 348,336 inches. For enough rain to fall in a period of 40 days to reach the peak of
this mountain, the cloud formations would have to drop 8,708 inches of rain per day
uniformly over all the earth. This would amount to 363 inches per hour or six inches per
minute. Can any reasonable person believe that it once rained continuously for 40 days and
nights at an average rate of six inches per minute? A rainfall of six inches in one day is a
veritable downpour. What would six inches per minute sustained for 57,600 continuous
minutes be like?

Volume 1990 - 2002 Issue


Page 254 of 2049
Skeptical Review Edited by Farrell Till
How, in fact, would such a sustained rainfall even be possible? The sun causes water to
evaporate and collect in clouds, and when clouds can no longer retain the water vapor that
they accumulate in the process, rain occurs. Then when the clouds have released their
moisture, the rain stops. So how could a worldwide cloudcover release rain for a period of 40
continuous days without having its supply of water replenished by the process of evaporation?
To argue that it could have happened is to argue that a world-wide cloudcover once retained
and released (without replenishment) several miles of water.

To the prescientific Hebrew mind, such a phenomenon would have made perfectly good sense
because of the cosmogony that was generally believed at the time. A body of water was
thought to exist in the heavens that God had divided from the waters upon the earth with a
"firmament" that he made the second day of creation (Gen. 1:6-8 ). To make it rain, God had
only to open windows or gates in the firmament to release some of the water that was above
the earth (Mal. 3:10 ; 2 Chron. 6:26 ). This cosmogonic misconception was alluded to in the
flood story when it spoke of God's opening and shutting "the windows of heaven" (7:11 ; 8:2
). To modern minds, however, the notion of such a prodigious outpouring of rain is absurd,
because we understand the nature of the water cycle that takes water from the surface of the
earth into the clouds and redistributes it as rain.

To deal with this problem, some diehard inerrantists have actually suggested that the
cloudcover was not worldwide, that part of the earth did receive sunshine that evaporated
water to replenish the clouds. According to this scenario, water, which seeks its own level,
simply flowed from where it was raining into those areas that were receiving sunshine.
However, to take such a position as this is to deny what the Bible says. It says that "the rain
was upon the earth forty days and forty nights" (7:12 ), but if this "explanation" of a major
problem is true, then rain wasn't upon the earth forty days and forty nights, because there were
parts of the earth on which the sun was shining to replenish the supply of water in the clouds.
Furthermore, if the rain was not falling uniformly over all the earth, to reach the height that
the flood did, the downpour would have had to be even more preposterously drenching than in
the examples calculated above. If, for example, the sun was shining on, say, half the surface
of the earth, then 726 inches of rain, rather than 363, would have had to fall each hour of the
40-day period.

This, however, is just a minor difficulty compared to an even greater problem that this
"solution" poses to the flood story. If the sun was evaporating water from one area of the
globe while it was pouring down rain elsewhere, then the sun was undoing the effects of the
cloudbursts. Long before the highest mountains "under the whole heaven" had been covered,
the oceans would have merged and the evaporation of water where the sun was shining would
have tended to maintain a constant level in the oceans even though rain was pouring down
elsewhere. This isn't at all hard to understand if we remember that water seeks its own level.
What would have happened can be compared to filling a swimming pool. If after a certain
quantity of water is pumped into the pool from one end, a siphon is activated at the other end
to drain the water away to feed the pump sending water in, then the pool never would fill. So
it would have been in the scenario just discussed. Water where it was raining would have
rushed to equalize the ocean level that had been momentarily lowered where the sun was
shining.

Volume 1990 - 2002 Issue


Page 255 of 2049
Skeptical Review Edited by Farrell Till
To be as generous as possible to inerrantists who are determined to believe the Genesis flood
story, let's just assume that the highest mountains weren't as high then as they are now. Let's
concede the possibility that through shifts in the earth's crust, the world's highest mountain
has been pushed well above what its height was at the time the flood allegedly happened.
Let's just suppose that the highest peak was only half as high as Mount Everest now is. Still
this would mean that the flood waters had to rise to a height of about 15,000 feet or 180,000
inches. To cover a mountain that high in just forty days, 4,500 inches of water per day or 187
inches per hour would have had to fall or, of course, rise from "the fountains of the great
deep." That still would have amounted to an average addition of three inches per minute to the
water level.

Inerrantists will be quick to remind us that the Bible doesn't claim that all of the flood waters
came from rain, because, as just noted, "the fountains of the great deep" were also opened
(Gen. 7:11 ; 8:2 ). So, they gleefully point out, some of the water came from within the earth
itself rather than from the rainfall. Again, let's be as considerate as possible to them and
assume that as much as three-fourths of the flood waters came from "the fountains of the great
deep." (This is a generous concession indeed, since scientists estimate that only about one
percent of the world's water supply is under the earth.) Then even if we further assume that
the highest mountain at that time reached only 15,000 feet, this would mean that 1,125 inches
of rain PER DAY or 46 inches PER HOUR would have had to fall to reach the heights
claimed in the flood story. This would have been a downpour of three fourths of an inch per
minute, which is still a level of rainfall too unreasonable to believe. If, however, the level of
the highest mountain was approximately the same then as it is today, 2,177 inches per day
would have had to fall in rain while three times that much was gushing from "the fountains of
the great deep." Who besides hopelessly naive Bible fundamentalists can believe it?

There is just so much water on earth, and no reputable scientist would say that there is enough
to raise the level of the oceans above "all the high mountains under the whole heaven." At this
time, we are concerned about the green-house effect that threatens to melt the polar ice caps if
the warming trend continues, but even if it does, only the coastal areas of the continents will
be flooded when the ice caps have melted. Certainly the highest mountains on earth will not
be under water. Where then did all the water in the Genesis flood story come from?

To say that much of it came from "the fountains of the great deep" is no answer at all, because
it shows an incredible ignorance of the mechanics of springs. When water flowing in
underground rivers encounters fissures in the rock beds encasing them, some of the water
rises to the surface to form springs. The pressure of the underground river, however, is what
pushes the water up. The spring water could not keep rising indefinitely unless the water in
the main channel was replenished to maintain a pressure level that would keep the spring
flowing. Once the underground channel had fallen below the level necessary to maintain that
pressure, the spring would "dry up." So this again brings us back to the problem of where all
of the flood water came from. Just to say that much of it came from "the fountains of the great
deep" is too simplistic to provide a satisfactory answer.

To think that several miles of water rose from beneath the earth without some sustaining force
to push it up is to think with the heart and not the head. Such a scenario has to suppose that
several thousand cubic miles of water-filled caverns under the seas just suddenly sent their

Volume 1990 - 2002 Issue


Page 256 of 2049
Skeptical Review Edited by Farrell Till
water spewing up to overcome the tremendous pressure of the oceans already pushing down
on them. What force caused that to happen? And since water seeks its own level, what kept
the water from dropping back into the empty caverns? We are told that when God shut the
windows of heaven and stopped "the fountains of the deep," it still took 150 days for the
water to decrease (Gen. 8:2-3 ). Why? Why on earth why? Why wouldn't the water that had
come up from the underground caverns have rushed back into them in the same way that
water empties from a bathtub when the drain is opened? Furthermore, what kept the caverns
from collapsing under the prodigious weight of millions of tons of water pushing down on
them so that there would have been no place for the water to return to and the surface of the
earth would have remained indefinitely flooded? Am I just crazy or something or don't any
inerrantists ever wonder about some of these things?

Inerrantists, of course, have a stock reply to every objection I have raised. God was behind it
all, they say, and so he could have caused anything, even rainfall at the rate of six inches per
minute for 40 continuous days. He could have provided the force to empty all of the
underground rivers and keep them empty until the flood was over. He could even have created
extra water just for the flood and then taken it away.

Yes, if we concede the existence of an omniscient, omnipotent deity who personally


engineered every phase of the flood, then presumably that would make anything possible. If
we are going to do that, however, we will have to give this deity another characteristic. In
addition to being omniscient, omnipotent, omnipresent, omnibeneficent, etc., etc., etc., we
will also have to make him omnisilly. He could have just said the word or waved his hand or
whatever omnipotent deities do to perform miracles, and thus caused all human life on earth
to die except for Noah's family, and if he had also wanted to include dumb animals in his
carnage, he could have spoken and caused all animal life, except replenishment stock, to die
too. Instead, he chose to have a man build a boat that had to be miraculously stocked with
animal life and then miraculously sustained through a miraculous flood of thousands of inches
of miraculously produced rain supplemented by miraculously emptied "fountains of the
deep."

If one wants to believe that, then the story makes perfectly good sense. If, however, anyone
puts any premium at all on common sense, he will have to consider the story a myth from
prescientific times.

(Creation/Evolution in which Robert A. Moore's article appeared is now published by The


National Center for Science Education, Box 9477, Berkeley, CA. 94709.)

What is Wrong With Any-Loophole-Will-Do


Hermenuetics
Dave Matson

Volume 1990 - 2002 Issue


Page 257 of 2049
Skeptical Review Edited by Farrell Till
One solution, offered by the editor of a Bible newsletter, envisions two campfires in the
courtyard. The rule this apologist is using holds that any loophole destroys the claim of error.

At first glance, the rule appears reasonable, but it is actually the single biggest blunder made
in Bible apologetics! It is the chief source of misunderstanding between those who defend
biblical inerrancy and those who reject it. Therefore, this rule deserves our closest scrutiny.

In its strictest sense, a contradiction consists of the statement "A and not A." That is, if we
state that the earth is spherical and (at the same instant) not spherical, then we are guilty of a
contradiction. Obviously, if the critical parts of a problem passage do not fit the form "A and
not A," then we do not have a contradiction. That is to say, any loophole at all gets one off the
hook. A harmonization--any harmonization--will do just fine.

That is the idea most biblicists have in mind when they offer forced, improbable explanations.
In practice, Bible-believers extend this approach to biblical statements about historical and
scientific matters as well. That is, they count every loophole that does not press the wildest
imagination to the breaking point (and a few that do).

In short, believing that the Bible is something akin to a dictation from God, biblicists assume
that the book is inerrant; they will not budge unless proven 100% wrong. "Don't claim an
error," they say, "unless you're one hundred percent certain."

The problem with this philosophy is that, in the real world, statements often carry hidden or
imprecise meanings. We are not dealing with well- defined mathematical propositions. The
statement, "The earth is spherical," is not, according to how we actually speak, necessarily
contradictory to the statement, "The earth is not spherical." I may offer the statement to you as
a close approximation of the earth's shape as seen from the moon. Later, I may utter the
second statement in recognition of the fact that the earth is slightly flattened at the poles. It is
a matter of perspective.

The statement, "Los Angeles has a population of 2,968,528," is not necessarily in conflict
with the statement, "Los Angeles has a population of three million." The intelligent reader
assumes that the second figure was rounded off, not that the author has contradicted himself.
Likewise, the statement, "It was the best of times; it was the worst of times," is not necessarily
a blunder by some poor half-wit! To apply the hard rules of logic to such statements is to
ignore the abbreviated way that we actually communicate. The real world is full of sentences
that clearly have unstated conditions or implied meanings attached to them. Indeed, the
simplest statements about everyday life are exceedingly difficult to make if rigor is
demanded! If written communication is to function at all, let alone efficiently, then we must
allow for unstated (but identifiable) conditions in a statement.

The upshot of all this should now be clear to you. If a "flat contradiction" (the strongest form
of error) is not necessarily an error in practical discourse, then what is? By carefully juggling
common sense with one hand and the above rule with the other, one can make contradictions
disappear right and left. The biblicists are playing with loaded dice!

Volume 1990 - 2002 Issue


Page 258 of 2049
Skeptical Review Edited by Farrell Till
We must either seek a more realistic definition for "proof" (and, consequently, of "error") or
else admit that we cannot even rule out Santa Claus and Little Red Riding Hood. Perhaps we
should take a lesson from the sciences--that immensely successful enterprise--that put the
moon within our reach. Scientists long ago recognized that our knowledge of the physical
world is tied to inductive reasoning. In principle, inductive reasoning can yield a high degree
of confidence, but it can never confer 100% certainty.

The uncertainty of inductive reasoning follows from the fact that any set of observations can
be explained, in principle, by an infinite number of hypotheses. There will always be
loopholes even to the most rigorous conclusions about the physical world. Strictly speaking,
my own consciousness is the only thing I can know with certainty about the real world.

Take something pretty basic, for example, such as the physical existence of the earth. How
can we prove (with 100% certainty, mind you) that the earth is not an illusion? Perhaps you
are just a brain wired up in some alien laboratory, and the earth is just an illusion programmed
for your benefit. How do you disprove that model with 100% certainty? As far as I know,
nobody has the foggiest idea how to do that. It is just one of an infinite number of logically
possible defenses for a nonexistent earth.

Now you may feel that we are in a hopeless quagmire, but that is true only if we insist on
100% certainty. Once we make the reasonable assumption that our collective senses do give
meaningful data about the real world, once we understand that every proof must contain an
element of uncertainty, then we are back in business. If we err in trusting our collective senses
at their most basic level, then we have lost nothing. We have nothing to lose, and, judging by
the success of the sciences, everything to gain. This is the only "leap of faith" that science
allows itself. It is the bare minimum if we are to understand reality at all.

Truth and error, then, at least in the physical world of atoms and energy, cannot be established
with 100% certainty. Outside the realm of mathematics, of abstract reasoning, proof of error
is always accompanied by loopholes; no amount of data will ever remove all doubt. To put it
another way, in matters of inductive reasoning no one can ever plug the last loophole. There
will ALWAYS be loopholes in every theory about the real world. It is how we handle them
that marks us as seekers of truth or as wishful dreamers.

THE SIMPLE FACT OF POINTING OUT A LOOPHOLE COUNTS FOR NOTHING IN


EVALUATING A CLAIM ABOUT THE PHYSICAL WORLD. For each bone of
contention, we must weigh the hypothesis of biblical inerrancy against that of errancy and
select the better argument. Which argument makes the more credible claims? Both will have
loopholes; that much we already know.

That brings us to the very important concept of objectivity. "Objectivity" is not an easy term
to define, and books have no doubt been written on the subject. In a nutshell, an objective
procedure tries to maximize the chances of being right by employing rules distilled from past
successes.

What might these rules entail? To begin with, an objective argument favors explanations with
an established track record. The most common experiences, the most likely assumptions, are

Volume 1990 - 2002 Issue


Page 259 of 2049
Skeptical Review Edited by Farrell Till
tried first in the evaluation process. Secondly, an objective argument attempts to minimize the
number of assumptions needed to explain a matter fully.

It follows that an objective argument concerning the real world must, at some point, be
grounded in repeatable observations. Judging by the phenomenal success of the sciences, it
appears that nothing more (save logic itself) is needed to explain things. Thus, in keeping with
our concept of objectivity, we must strive to explain the universe in those terms. If someday
that proves to be an oversimplification, then we will adjust our thinking accordingly.

You might object that this procedure is prejudiced in that it rules out the supernatural. But, all
that is really being asked for is that extraordinary claims be backed up by extraordinary
evidence. It is a fool's gamble to bet on improbable explanations when common ones will do.
A miracle, for example, is not established if ordinary explanations are available; the latter are
infinitely more probable. Fraudulent claims and confused witnesses abound the world over
while, as far as I can tell, no supernatural event has ever been proven beyond a reasonable
doubt.

Objectivity means sticking to the face value of a verse (the common meaning of the words)
unless doing so would be a clear cause of error. Again, it is a matter of starting with the
common explanations and working, if necessary, towards the more exotic ones. Objectivity is
not a matter of trying to see if our ideas will fit in but rather of seeing if our ideas should fit
in. "Objectivity" is a fisherman who goes into muddy waters with a big net. "Wishful
thinking" is a fisherman who uses a teaspoon! Now, it is just possible that a fish might leap
into that teaspoon while the big net comes up empty. But who are you going to depend on for
dinner?

An argument that lacks objectivity is like a fisherman who uses a teaspoon to catch fish!

Let us now get back to our two-campfire argument. That two campfires are lit, that they are
conveniently located to support biblical inerrancy, is strictly an ad hoc argument with no
support from the text whatsoever. None of the Gospels remotely suggest such a scene. We
must reject ad hoc arguments, because they are not the fruit of positive evidence. They are the
gods of the gaps, thriving where positive evidence is absent. The fact that something might
have happened is a mighty poor substitute for the claim that it DID happen!

To sum up, we must weigh the merits of inerrancy against those of errancy. On those scales,
what MIGHT have happened is a hollow weight. Thus, the two-campfire defense lacks
objectivity. The solution tries to overthrow the face value of the text without benefit of factual
support. That is excellent grounds for rejecting any interpretation.

(Dave Matson's address is 330 South Hill Avenue, Pasadena, CA 91106. His booklet Bible
Errors: a Sampling from Four Topics from which this article was excerpted can be obtained
from Complimentary Copy Press, 1525 Canterbury Road, Lakewood, NJ 08701.)

Volume 1990 - 2002 Issue


Page 260 of 2049
Skeptical Review Edited by Farrell Till

Reader Reaction
We receive far more letters than we can respond to, but occasionally we print samples that
contribute useful insights to the articles and materials we publish.

The Till-Lockwood debate has been fun to read. The question of burden of proof is not
simple. To prove beyond reasonable doubt that the Bible has errors is tougher than to raise
doubts of its inerrancy, although you and I believe that it is not at all hard to do. I think we're
being a little too easy on the believers. I do not accept the notion that a disinterested party or
"judge" should consider the Bible to be The Inerrant Word of God unless we show beyond
reasonable doubt that it has errors. I think that the notion that a written text is supernaturally
free of error is so strange that the burden of proof ought to be the other way (emphasis added).

In TSR, v3#4, pp. 11-12, part of the discussion is on the antiquity of monotheism. I've seen an
interest ing book: The First Heretic: The Life and Times of Ikhnaton the King, by Fred
Gladstone Bratton. He credits Ikhnaton, who became king in 1375 BC, with the invention of
monotheism. The sun-god Ra is much older, but the universal, immaterial Supreme Being
dates from Ikhnaton. Now, do the inerrantists claim that the Moses Pharaoh ruled before
Ikhnaton?

[William B. Lindley, contributing editor of Truth Seeker, P. O. Box 2872, San Diego, CA
92112.]

At the end of his bizarre debate with you, his every other line being a totally irrelevant slap at
you for the benefit of the anti-intellectual, home crowd, Bill Lockwood finally concluded that
you "... surely made a poor showing in this exchange, and every reader knows it." Either Bill
is a politician at heart, akin to Bush claiming victory in his last days, or else his concept of
your typical reader needs a major overhaul. I reached a different conclusion as, no doubt,
most of your readers did. I concluded that Mr. Lockwood never understood the heart of your
rebuttal. He seemed to be off in an imaginary ring by himself, punching mostly at ghosts of
his own making. The problem is this: Mr. Lockwood has no grasp whatsoever of the role that
proof plays in an inductive argument. Without that reference point one cannot possibly
understand the proper weighing of the evidence. He never even saw your knockout punch.

What Mr. Lockwood needs (and he isn't alone) is some basic tutoring in inductive reasoning.
In the world of atoms and energy, and that includes the interpretation of the Bible, public truth
is ultimately determined through inductive logic. Deductive logic, which can play a
supporting role within a framework of inductive logic, applies to a mathematical or abstract
system that is defined by arbitrary (but hopefully consistent) postulates. Fundamentalists often
confuse (or even abuse) the two forms of logic.

Volume 1990 - 2002 Issue


Page 261 of 2049
Skeptical Review Edited by Farrell Till
You might consider devoting half an issue to educating future debating opponents on the
basics of reason ing. A remedial class seems in order judging by the performance of many of
your opponents. Once done, it will serve as a reference point for future debates.

[Dave Matson, 330 South Hill Avenue, Pasadena, CA 91106. See Mr. Matson's article on
pages 9-11.]

In reading the debates in the back issues, I can feel your frustration in trying to convince the
"fundies" of the many, many mistakes to be found in the bible! I'm reminded of the old
southern saying: Never try to teach a pig to whistle. It just frustrates you, And the pig don't
like it!

In other words, the fundies are not going to admit it, even when you plainly show them they
are wrong. To admit that they're wrong means giving up their social activities (which center
around the church), their entertainment (gospel quartets, snake handling, etc.), their once or
twice per week, sometimes more, chance to "dress up" and show off their finery. Saying
they're mistaken means giving up their bonding (herd instinct) or, as the fundies call it,
"fellowship" with other members, and on and on. I think most fundamental ists know there is
nothing to the bible, but they simply can't let go. Little do they realize it will be the happiest
day of their lives when they finally give it up!

Farrell, I am an avowed atheist, yet I love my wife and son better than life itself. I pay my
bills; I pay my taxes; I serve on community projects (helping the homeless, the poor, etc.); I
do not and will not cheat, lie, or steal; I am a good neighbor, and above all I am tolerant of
other people and their ideals and beliefs. My question: How could I be better? Answer: I can't.
To pursue organized religion would make me less tolerant of my fellow man. I would become
a liar and a hypocrite.

Besides, to be a good atheist, one must read and know the bible better than your ordinary
churchgoer. One must read several hundred other books on freethought, religion, rationalism,
etc. In other words, you can't be an idiot and be an atheist.

[Bob Fincher, 9809 Waters Meet Drive, Tallahassee, FL 32312-3759.]

A Virgin-Birth Prophecy?
Kenneth E. Nahigian
Prophecy is a muddy science, and Bible prophecy more muddy than most. Take those Old
Testament prophecies. Evangelists never tire of telling us that hundreds were fulfilled in the
life of Jesus, far too many to be called coincidence. But how many of these are real, and how

Volume 1990 - 2002 Issue


Page 262 of 2049
Skeptical Review Edited by Farrell Till
many are prophetia ex eventu--prophecies constructed after the fact, products of careful
selection and interpretation?

To get an idea, let's look at the most famous, the prophecy of the child Immanuel as presented
in the Gospel of Matthew:

Now all this was done that it might be fulfilled which was spoken of the Lord by the prophet,
saying, Behold, a virgin shall be with child, and shall bring forth a son, and they shall call his
name Emmanuel, which being interpreted is, God with us (1:22-23 , KJV).

Most good Christians take this at face value, assured that the prophet Isaiah did indeed
describe Jesus' miraculous conception and birth seven hundred years before. But did he?
Authorities are nearly unanimous. The answer is no.

What did Isaiah really say? Turning to Isaiah 7:14 (Masoretic text), we find his precise words:

Therefore the Lord himself shall give you a sign; Behold, ha'almah shall conceive, and bear a
son, and shall call his name Immanuel.

Matthew's interpretation of this passage has several problems, the largest hanging on the
Hebrew word 'almah. Writing in Greek, the gospel author turned almah into parthenos , a
word usually (but not always) meaning "virgin." In fact, he had a precedent for this; the
Septuagint, a translation of the Old Testament used by Greek-speaking Jews of his day, did
indeed use parthenos in the Isaiah passage. But the Septuagint was for the most part a
notoriously sloppy translation, and its version of Isaiah was generally more error-ridden than
the rest. By the Middle Ages, the Jews had abandoned the Septuagint, and later Greek
translations, by Aquila, Theodotion, Lucien and others, did not use the word parthenos. (The
Septuagint, commonly known as the LXX, is still favored by Eastern Orthodox churches.)

Assuredly, the Hebrew Old Testament predating the Septuagint used 'almah, so what did the
word mean? While rare in the Hebrew Bible, almah does occur here and there, notably in
Genesis 24:43 and Exodus 2:8 , but an examination of the contexts of these passages will
show nothing to suggest that the noun imputed virginity.

On the other hand, a male youth in the Old Testament was called na'ar or elem, the feminine
forms of which were na'arah and 'almah respectively. The limited usage of elem (lad or
stripling) in the Old Testament nowhere implied sexual purity; thus an 'almah was an
adolescent female, virgin or not, just as an elem was an adolescent male. In fact, one verse
does seem to use 'almah in reference to a nonvirgin. This is Proverbs 30:19 , which listed four
things too marvelous to understand: the way of an eagle in the air, the way of a serpent on a
rock, the way of a ship in the sea, and the way of a man with a maiden ( 'almah). To say the
least, "the way of a man with an 'almah" would certainly jeopardize a state of sexual purity,
but more damaging than this rather obvious fact is the comparison that the writer went on to
state: "Such is the way of an adulterous woman: she eats, wipes her mouth, and says, 'I have
done no wrong'" (v:20 , NAB). It seems odd writer that the author would use 'almah to denote
sexual purity and then compare it to the ongoing affairs of an adulterous woman. More likely

Volume 1990 - 2002 Issue


Page 263 of 2049
Skeptical Review Edited by Farrell Till
the author's point was that all these things have one element in common: they do not leave
much of a trace.

Aside from this, the Torah does, in fact, have an explicit word for virgin (betulah or bethulah),
which is always used where the context requires virginity. (For confirmation, see Genesis
24:16 , Leviticus 21:14 , and Deuteronomy 22:15-19 ). Even Isaiah used it in 62:5 . Its nonuse
in the "Immanuel" passage is a rather loud hint that Isaiah spoke only of a young woman, not
specifically of a virgin.

More to the point, nearly all modern commentaries agree with Talmudic scholars that Isaiah's
"sign" had nothing to do with a messiah. Reviewing half a dozen for this article, I found only
one dissenter. Significantly, it was one that spouted the fundamentalist party line on every
other issue. Interested readers can jaunt to the library and peruse the massive Interpreter's
Bible (Vol. 5, pp. 217-22), one of the most authoritative works in the field. Or more
succinctly, try the popular Harper's Bible Dictionary (Paul J, Achtemeier, gen. ed., 1985),
page 419, where this statement is found:

It is clear, however, that... Isaiah 7:14 did not speak of the miraculous birth of Jesus centuries
later.... The sign of Immanuel offered by the prophet to Ahaz had to do with the imminent
birth of a child, of a mother known to Ahaz and Isaiah, and signified God's presence with his
people....

Indeed, Isaiah's word for "sign" was 'ot, which in the Hebrew Bible invariably indicated an
imminent sign or omen, not one in the far future. Keep reading, in fact, and you will see
Isaiah's sign appear just a few verses later (Is. 8:3-4 ), when a certain prophetess gives birth to
a son--a child whom God called "Immanuel" in verse 8. By contrast, nowhere in the New
Testament did any character ever call Jesus Immanuel. Why the confusion? Of course, the
author of the Gospel of Matthew had a vested interest in the nascent church and wanted to
ground the new Christian mythos in Jewish prophecy whenever possible. Almost all scholars
agree this "Matthew" was not the apostle but rather a Greek-speaking Christian living in or
near Antioch of Syria, who wrote about A.D. 90, about two generations after the crucifixion.
Very likely, he was familiar with only the Septuagint version of Isaiah. (That Matthew wrote
the first gospel was a tradition started by Bishop Papias of Hieropolos in the second century.)

Also, of course, the early Christians would have liked a virgin-born savior anyway, out of
sheer competitiveness, because so many other rival religions had one. (Mithra, Zoroaster,
Adonis, and Dionysus were just a few.) More- over, we know the gospel writers were not
adverse to massaging and even manufacturing details in order to "flesh out" the Jesus story.
That is why, for example, you find such conflicting genealogies for Jesus in Matthew 1:1-16
and Luke 3:23-38 .

All things considered, it is hardly surprising that "Matthew" would pull Isaiah a bit out of
context and try to wring a new meaning from it. What is surprising is that this literary sleight
of hand grew to become such a cornerstone of Christendom and still has modern
fundamentalists so befuddled. So let's dust off our Bibles (I like the New Revised English
Bible best for clarity and the Revised Standard Version for beauty) and reread the Immanuel
prophecy--in context.

Volume 1990 - 2002 Issue


Page 264 of 2049
Skeptical Review Edited by Farrell Till
The setting is the Syro-Ephraimite war (ca. 734 B.C.). Wicked King Ahaz of Judah was
frantic about Ephraim (another name for the northern kingdom, Israel) and Damascus (capital
of Syria), which were plotting a preemptive strike. Isaiah enters, offering a sign. Ahaz
demurs. Isaiah storms at him for his lack of faith and then provides a sign anyway: A male
child would be born. Before this child is old enough to know to "refuse evil and choose the
good," Assyria would lay waste both Samaria and Damascus (7:16 ). [This sub-prophecy, in
fact, came true in 2 Kings 16:9 ; 17:5-6 .] Then, to punish Ahaz, Assyria itself, with Egypt,
would arise as a far greater threat.

Think about this. If Ahaz was concerned with an imminent attack from Samaria and Syria,
why offer a sign that would not occur for seven centuries? To Ahaz this would be no sign at
all. Also, if the Immanuel child was God incarnate, how could Isaiah speak of a time when
Immanuel would not know enough to choose good over evil? What about divine
omniscience? Note also the striking parallel between verses 7:16 and 8:4 . Here is Isaiah
prophesying almost identically about both children. The more closely you look, the more
difficult to deny that these two are identical. You can hardly blame evangelicals for seeing a
special significance in the name Immanu'el, Hebrew for "God with us," but such language and
imagery was right at home in the world of old Jewish nomenclature, where every other proper
name seemed a reminder of God's presence. Thus we have Isaiah, which means "God's help";
Michael , "Like unto God"; Israel," "Striving with God"; Elihu, "He is my God"; Adonijah ,
"Yahweh Lord"; and a host of others.

Then again, some apologists try to rescue their favored exegesis by equating both Immanuel
and Jesus with the child mentioned a bit later in chapter 9, "Unto us a child is born...." It is
tempting. This section, while obscure, is in fact one of the most powerful and poetic passages
in the Old Testament. It may well be an early messianic prophecy (I like to think it is), but in
fairness, note that most Jewish scholars (who should know better than evangelicals) insist it is
an ode praising Hezekiah, Ahaz's righteous son (2 Chron. 29 ), who came to the throne in 720
B.C. and centralized the worship of Jehovah at Jerusalem. The various titles ascribed to him,
such as "Prince of Peace" and "Everlasting Father," were apparently honorifics used by the
ancient Jews for favorite kings. (You find the same sort of bread-buttering in Egyptian hymns
to the pharaoh and in Babylonian royal eulogies.) Hebrew scholars also remind us, gently,
that the key Hebrew verbs in Isaiah 9:6 are in the past tense.

A moot point. For reasons stated earlier, we cannot use the child in Isaiah 9:6 as a bridge
connecting Immanuel to Jesus. As Old Testament prophecies of the Christian Messiah go, this
one, like so many others, has been overrated.

(Kenneth E. Nahigian's e-mail address is appalonius@netzero.net.)

A Challenge to Josh McDowell

Volume 1990 - 2002 Issue


Page 265 of 2049
Skeptical Review Edited by Farrell Till
In discussing Bible inerrancy with lay believers, I am often asked if I have read Josh
McDowell's book Evidence That Demands a Verdict. As a matter of fact, I did read it, long
ago, and found it to be incredibly simplis- tic, as are most books of its type. McDowell's name
has come up so often that finally, on February 6th, I wrote to him and proposed a debate. As
this edition goes to press, he has not yet responded to the challenge. I predict that he will be
no different from other fundamentalist champions who like the security of partisan audiences,
but I will keep you posted (FT).

Volume 1990 - 2002 Issue


Page 266 of 2049
Skeptical Review Edited by Farrell Till

The Skeptical Review


Volume Four, Number Three - 1993
Farrell Till, editor

• Poisoning the Well


How inerrantists avoid confronting rational arguments, and why.

• Will There Be Spiders in Heaven?


A look at biblical errors in reasoning.

• Sexual Conduct Pentateuchal Style


A look at sexual rules of conduct in the Pentateuch.

• Once Upon a Time


Christian fundamentalists dismiss as liberal nonsense any interpretation of scriptures
that is based on the existence of myths and legends in the biblical text, yet they
themselves often take doctrinal positions that reflect a fairy-tale view of the Bible.

• The Essence of Prophecy


Bobby Liddell defends biblical prophecy in this article.

• The Myth of Prophecy Fulfillment


Farrell Till rebuts Liddell's "The Essence Of Prophecy".

• Correspondence with Gleason Archer

• Et Tu, McDowell?

• Dobbs-Till Debate

Volume 1990 - 2002 Issue


Page 267 of 2049
Skeptical Review Edited by Farrell Till

Poisoning the Well


Logical fallacies of every conceivable kind are much in evidence in apologetic literature
written in defense of the Bible inerrancy doctrine, but few are more evident than the fallacy of
poisoning the well. This fallacy occurs when a debater figuratively offers an audience the
choice of drinking from his "untainted" well or from others that he has unfairly contaminated.
"You can believe Jones and the atheistic philosophy he embraces," a preacher strapped for
evidence to support his inerrancy belief might proclaim, "or you can believe God and his
word." Many in the audience may not even know what "atheistic philosophy" stands for, but
they know that it has to be something bad. Haven't they heard it condemned enough by
preachers like the one in our example? So they fall for the trick and opt to drink from the
"untainted well of God's word." They certainly don't want to be caught sympathizing with
atheistic philosophy.

The poisoned-well fallacy can be a composite of many logical flaws, but it almost always
includes at least two: argumentum ad hominem and begging the question. Our hypothetical
preacher, for example, has declared, "You can believe Jones and his atheistic philosophy,"
(argumentum ad hominem, attacking the opposition rather than his argument), "or you can
believe God and his word," (begging the question, assuming rather than proving major claims,
i.e., God exists and the Bible is his word). As far as actual proof of his claim is concerned, the
preacher has proven nothing, but he has probably persuaded a lot of people already
predisposed to his position to remain sympathetic to it. Persuasive techniques like this can be
effective in the hands of demagogical preachers more interested in obtaining converts than
establishing truth.

For the poisoned-well fallacy to work, it must be applied to a claim for which invincibility is
widely assumed. If Jones in our example should say to his audience, "You can believe my
opponent and the Bible he embraces or you can believe me and my atheistic philosophy," no
appreciable poisoning of the well could result, because there would probably be very little
predisposition in the audience to believe that atheistic philosophy is true. In a typical
audience, however, there would be considerable predisposition to believe that the Bible is
God's inspired word. Inerrancy proponents know this and exploit it for all it's worth.

If inerrancy defenders encounter evidence that clearly disputes their claim, they will never let
a simple thing like facts get in their way. They simply reinterpret the counterevidence, no
matter how overwhelming it may be, to make it appear in some way to support or at least not
contradict their inerrancy claim. Their reinterpretations are quite often very imaginative and at
times even absurdly far-fetched. But the upshot of it all is still the same. To an audience
desperately wanting to drink from the well that says the Bible is God's inspired word, all the
others appear contaminated, so they are left with the same choice. They can believe Jones and
his atheistic philosophy or they can believe God and his word.

"If I can show you how it could have been, then you can't really say that there is a
contradiction." This has become the theme song of inerrancy defenders who are experts at
poisoning the wells when confronted with evidence that clearly disputes their claim. With
techniques that they have almost developed into an art, they can reinterpret any discrepant

Volume 1990 - 2002 Issue


Page 268 of 2049
Skeptical Review Edited by Farrell Till
statements and facts to give them at least a semblance of concordance. The only problem is
that their reinterpretations are almost always incredibly far-fetched.

Long-time readers of TSR have seen this approach over and over again in the articles of
fundamentalist writers seeking to rebut our claim that the Bible contains errors. Sometimes
these writers will confine themselves to the issues and seek only to present their speculative,
how-it-could-have-been scenarios, but often they feel the need to pour at least a little poison
into the well. "The reason Mr. Till is a skeptic and not a saint today," wrote Steve Gunter,
"appears to be primarily due to a massive misreading of the text and too much study of
noninspired works" ("Much Ado about Nothing," Autumn 1991, p. 7). "If Mr. Till spent half
as much time trying to reconcile these 'so-called' difficulties as he spends finding them," said
Jerry McDonald, "he would find far fewer difficulties in the Bible" ("The Blood of Jezreel,"
Spring 1991, p. 3). Such comments as these are obvious attempts to offer two sources of
water to the readers, the well of skepticism or the well of sainthood and the Bible. They do
nothing to prove inerrancy, but admittedly they do impress those who are already predisposed
to believe that the Bible is "God's inspired word." Bibliolaters know this, of course, and that is
why they spend so much time trying to poison the well of rationalism.

Will There Be Spiders in Heaven?


Farrell Till
The inerrancy doctrine is an inevitable consequence of belief in the divine inspiration of the
scriptures. Inerrantists recognize that if the Bible was indeed verbally inspired by an
omniscient, omnipotent, omnimoral deity, then by necessity it would have to be completely
inerrant in every detail written in it. This is why we frequently hear fundamentalists say that
the Bible is inerrant in matters of science, history, geography, chronology, etc. as well as
matters of faith and practice. Their belief in verbal inspiration allows them no other
conclusion but this.

Past articles in The Skeptical Review have exposed many inconsistencies in the biblical
narrative concerning matters of science, chronology, history, etc., and recently we have begun
to focus on another facet relevant to Bible inerrancy--the element of logic. If the Bible was
really inspired by an omniscient, omnipotent, omnimoral deity, then the logic used by its
writers would have to be impeccable, for how could a person being guided by such a deity in
everything he wrote, word for word, be capable of faulty logic?

That biblical writers were not perfect in their use of logic is evident to anyone who is willing
to examine the Bible with an open mind. Faulty logic is most evident in the New Testament,
probably because of a need the writers felt to convince their readers that Jesus was the
Messiah whom God had sent in fulfillment of Old Testament promises. With no logic on their
side, their attempts at argumentation were necessarily faulty. The Apostle Paul, who seemed
to fancy himself as a master of argumentation, made some horrendous mistakes in reasoning.

Volume 1990 - 2002 Issue


Page 269 of 2049
Skeptical Review Edited by Farrell Till
His lack of ability in logic was very much in evidence in his famous defense of the
resurrection in 1 Corinthians 15 . At one point in his narrative, Paul said, "If there is a natural
body, there is also a spiritual body" (v:44 ). Paul's argument could be stated in the form of a
modus ponens syllogism:

MAJOR PREMISE: If there is a natural body, there is a spiritual body.

MINOR PREMISE: There is a natural body.

CONCLUSION: Therefore, there is a spiritual body.

The major premise of this syllogism is a conditional or hypothetical sentence that contains
two parts: an antecedent (the "if" statement) and the consequent (the conclusion derived from
the antecedent). For a modus ponens syllogism to be sound, it must be demonstrably true that
the antecedent undeniably necessitates the consequent, and this is where Paul's logic fails.

There is absolutely nothing in the existence of a natural or physical body that would
necessitate the existence of a spiritual body. I am aware of the deep-seated desire in humans
for life after this one, but desires, wishes, and hopes prove nothing. To arrive at truth in this
matter, we must lay aside dreams and aspirations and examine all the evidence under the
microscope of cold logic. If this is done objectively, one will have to reject Paul's argument,
because it is an obvious non sequitur. The existence of natural or physical bodies does not
necessitate the existence of spiritual bodies. Dogs, cats, fleas, and spiders have physical
bodies, so does this mean that they also have spiritual bodies? Will there be spiders in
heaven? To accept Paul's argument, one would have to believe that there will be, because they
too have "natural bodies." That doesn't leave us much to look forward to, does it? We spend
our lives here tolerating mosquitoes, fleas, spiders, and cockroaches only to find out that we
will also have to put up with them all through eternity.

The same type of non sequitur (conclusion not justified by its premises) occurred in a Pauline
argument intended to show that the fact of baptism proves the existence of a spiritual
resurrection. After stating that in the act of baptism one is buried as Christ was buried in his
death, Paul said, "For if we have been united together in the likeness of His death, certainly
we also shall be in the likeness of his resurrection" (Romans 6:5 ). In this statement, we see
the "if this, then that" elements of a conditional or hypothetical sentence. In other words, Paul
was arguing that if "this" (burial with Christ in baptism), then "that" (resurrection to a new
life).

No one denies that religious experiences can bring about significant changes in people's lives,
but Paul's argument here is purely arbitrary. The ceremony of baptism, as Paul perceived it
here, was symbolic of the death of one's old life and resurrection to a new life only because
Paul arbitrarily declared it was so. Other religious sects of the time practiced baptism as
nothing more than a ceremonial cleansing. They saw nothing at all in it to denote burial of
"the old man" and resurrection of "the new man." Some religious sects (including the
Israelites of the Old Testament) thought that there was advantage to worshiping their gods in
"high places." Had the custom survived into New Testament times, I suppose some "inspired"

Volume 1990 - 2002 Issue


Page 270 of 2049
Skeptical Review Edited by Farrell Till
writer could have argued that one's act of climbing a mountain to worship was a symbol or
likeness of the ascension of Jesus into heaven.

Paul's argument here is just another example of a Bible writer who was long on assumption
and short on proof. If one is resurrected to a new life after baptism, then what do we say about
the person whose religious fervor subsides after the emotionalism of the moment that led him
to be baptized? In backsliding, does he "spiritually" find his "old man," which was crucified
and buried with Christ, and reenter that body? There is simply no logic in Paul's argument. It
is religious whim and caprice--and nothing more. In Romans 3:5-6 , we find Paul at it again;
only this time he is begging the question. In speaking of the righteousness of God, he asked,
"Is God unjust who inflicts wrath?" His answer was, "Certainly not! For then how will God
judge the world?" Begging the question occurs when an argument assumes crucial points that
need to be proved. In this case, Paul was arguing that God cannot be unjust simply because he
inflicts wrath on people, because God is going to judge the world and he could hardly judge
the world justly if he himself is unjust. Paul offered no evidence at all to prove that God is
indeed going to judge the world; he merely presented it as an assumption that he apparently
expected his readers to accept. He undoubtedly did this for the obvious reason that there is no
way that he or anyone else could prove that God is going to judge the world, so at a loss for
logic to prove his point, all he could do was beg the question.

Paul was no better at arguing from analogy. He showed this in an attempt to prove the
abrogation of the law of Moses:

Or do you not know, brethren (for I speak to those who know the law), that the law has
dominion over a man as long as he lives? For the woman who has a husband is bound by the
law to her husband as long as he lives. But if the husband dies, she is released from the law of
her husband. So then if, while her husband lives, she marries another man, she will be called
an adulteress; but if her husband dies, she is free from that law, so that she is no adulteress,
though she has married another man. Therefore, my brethren, you also have become dead to
the law through the body of Christ, that you may be married to another--to Him who was
raised from the dead, that we should bear fruit to God. For when we were in the flesh, the
sinful passions which were aroused by the law were at work in our members to bear fruit to
death. But now we have been delivered from the law, having died to what we were held by, so
that we should serve in the newness of the Spirit and not in the oldness of the letter (Rom.
7:1-6).

Paul's apparent intention here was to compare the abrogation of the law of Moses and its
replacement by a new covenant with the marriage relationship.

His analogy concerned a widow who enters into a second marriage, but he had obvious
difficulty keeping straight what corresponded to the wife and what represented the husband
and even who or what had died. A wife was bound to her husband as long as he lived and,
unless her husband had died, could not marry another without committing adultery. If,
however, her husband died, she was free to contract a second marriage without incurring the
stigma of adultery. Paul seemed able to get that much of it right, but when he tried to
analogize the widow's situation with the relationship between the two covenants, he got
confused and had the widow dying rather than the husband (law).

Volume 1990 - 2002 Issue


Page 271 of 2049
Skeptical Review Edited by Farrell Till
In The Mythmaker: Paul and the invention of Christianity, Hyam Maccoby very effectively
identified the points of confusion in the analogy: It seems that the correspondence intended is
the following:

the wife is the Church; the former husband is the Torah, and the new husband is Christ. Paul
tells us that a wife is released by the death of her husband to marry a new husband; this
should read, therefore, in the comparison, that the Church was freed, by the death of the
Torah, to marry Christ. Instead, it is the wife- Church that dies ("you my friends, have died to
the law by becoming identified with the body of Christ") and there is even some play with the
idea that the new husband, Christ, has died. The only term in the comparison that is not
mentioned as having died is the Torah; yet this is the only thing that would make the
comparison valid (p. 69).

"Dying to sin" was such a recurrent image in Paul's writings that in this passage he apparently
lost sight of who was supposed to represent what in his analogy, so he wound up having his
Christian audience "die" to sin rather than the law die to free Christians to marry another.
Such a mistake would be no big deal in the writings of an ordinary man, but we have to
wonder why Paul, writing under the verbal guidance of an omniscient, omnipotent deity,
could have made a logical error like this.

In Maccoby's commentary on Paul's analogy, he noted another flaw:

On the other hand, there is also present in the passage an entirely different idea: that a person
becomes free of legal obligations after his or her own death. This indeed seems to be the
theme first announced: "that a person is subject to the law so long as he is alive, and no
longer." The theme of the widow being free to marry after the death of her first husband is
quite incompatible with this; yet Paul confuses the two themes throughout... (p. 69).

My reaction to the muddled way that Paul tried to present his argument is the same as
Maccoby's: "Confusion cannot be worse confounded than this" (Ibid). More than that, I have
to wonder why a verbally inspired writer would not have a better sense of logical analogy
than this.

A frequent device of persuasion used by New Testament writers was the a fortiori argument,
but they frequently bungled their attempts to use it. In this kind of argument, a conclusion of
greater necessity is arrived at by comparing it to a generally accepted conclusion of less
significance. "Drunk drivers deserve to have their licenses revoked," one might argue, "so
how much more should those who drive while stoned silly by drugs have theirs revoked?"
Anyone so arguing would be using the a fortiori approach to persuasion, and it was a
common device used by New Testament writers. In Hebrews 10:28-29 , it was used by the
unknown writer of this epistle: "Anyone who has rejected Moses' law dies without mercy on
the testimony of two or three witnesses. Of how much worse punishment, do you suppose,
will he be thought worthy who has trampled the Son of God underfoot, counted the blood of
the covenant by which he was sanctified a common thing, and insulted the Spirit of grace?" In
other words, the writer was arguing that if disrespect for the law of Moses brought death, then
surely open disrespect for the covenant of Christ will merit even harsher punishment.

Volume 1990 - 2002 Issue


Page 272 of 2049
Skeptical Review Edited by Farrell Till
The conclusion that the writer reached, however, violated a basic principle of a fortiori
argumentation, which is that the conclusion (the how-much-more element) cannot validly
exceed the premise it depends on. In my drunk-driving illustration, for example, the
punishment demanded for driving while "stoned silly" on drugs was the same as for drunk
driving--the revocation of one's license. The argument did not attempt to prove that if one's
punishment for drunk driving was the revocation of his license, then the penalty for driving
under the influence of drugs should be a prison sentence or the death penalty; it merely argued
that if driver's licenses are revoked for drunk driving, then the same penalty should be
imposed for the greater offense of driving while "stoned silly" on drugs. The Hebrew writer's
application of the argument, however, deviated from the standard a fortiori pattern and
concluded that a much harsher punishment should be given to those who trample underfoot
the son of God than to those who just disobeyed the law of Moses.

In 9:13-14 , the Hebrew writer made the same reasoning error: "For if the blood of bulls and
goats and the ashes of a heifer, sprinkling the unclean, sanctifies for the purifying of the flesh,
how much more shall the blood of Christ, who through the eternal Spirit offered Himself
without spot to God, cleanse your conscience from dead works to serve the living God?" The
cleansing of the conscience or spirit would be greater than the cleansing of the flesh, so once
again the writer used a fortiori logic to obtain a conclusion that went beyond the premise it
was derived from.

The Apostle Paul also made frequent attempts at a fortiori argumentation, but, like the
Hebrew writer, often misapplied it. In The Mythmaker (p. 65), Hyam Maccoby analyzed
Paul's use of the form in his epistle to the Romans and identified several statements with
conclusions that exceeded the premises they were derived from. Two of them were these:

For if when we were enemies we were reconciled to God through the death of His Son, much
more, having been reconciled, we shall be saved by His life (5:10).
For if by the one man's offense death reigned through the one, much more those who receive
abundance of grace and the gift of righteousness will reign in life through the one (5:17).

Just a cursory examination of both passages will show that Paul tried to derive a fortiori
conclusions that far exceeded the premises he was arguing from, so apparently he was no
more skilled at this type of reasoning than was the Hebrew writer.

Some will no doubt say that the reasoning in the above examples of a fortiori argumentation is
sound, even though it may have technically deviated from the traditional form of the
argument. Does it not make sense to think, one might say, that if the blood of bulls and goats
cleansed the flesh, the blood of Christ would be much more efficacious in the cleansing of the
soul? To so argue, however, is to venture into pure speculation. As Maccoby said of Paul's a
fortiori arguments whose conclusions exceeded their premises, "Such an argument has no
precision about it, for how do we know how much to add to the data given in the premise in
order to arrive at the conclusion" (p. 66)? Besides that problem, we have every reason to
wonder why a writer verbally inspired by an omniscient, omnipotent deity would not know
how to use logic that was as firmly established in Jewish legalism as was a fortiori
argumentation.

Volume 1990 - 2002 Issue


Page 273 of 2049
Skeptical Review Edited by Farrell Till
Another problem in many of the a fortiori syllogisms in the New Testament is the flagrant
resort to begging the question that characterized so many of their premises. In Hebrews 9:13-
14 , for example, the writer's argument was if the blood of bulls and goats cleansed the flesh,
then surely the blood of Christ will be even more efficacious in cleansing the soul. Yes, if...
IF, IF, IF! But what proof did the writer give that the blood of bulls and goats could purify
flesh? He gave none. He simply expected his reader to assume that the statement was true. He
begged the question. Likewise, Paul in Romans 5:10 begged the question in the premise of his
argument: "For if when we were enemies we were reconciled to God through the death of His
Son, much more, having been reconciled, we shall be saved by His life." What was his proof
that the death of Jesus reconciled us to God? He gave none. He merely stated it as a fact to be
assumed without proof and proceeded to draw his conclusion, i.e., the life of Jesus will be
much more efficacious in saving us. Thus, many of the a fortiori arguments in the New
Testament were doubly flawed. They didn't just reach conclusions that far exceeded the
premises they were derived from, but the premises themselves were often no more than mere
assumptions.

Such examples of reasoning as these are flawed through and through. They constitute
formidable proof that the Bible is just another fallible book written by very fallible men. No
omniscient, omnipotent deity had anything to do with writing it. To say otherwise is to insult
the omniscience of the deity who is presumed to be the source of the inspiration.

Sexual Conduct Pentateuchal Style


Robert Countess, Ph.D.
There are several biblical passages that I consider unambiguous in their implications or the
inferences that may be drawn from them concerning sexual beliefs and practices of the times
in which they were written. One is found in Numbers 31 : "But all the young girls who have
not known man by lying with him, keep alive for yourselves" (v:18 , RSV).

The context is the overall conquest of the land after the exodus. Specifically, chapter 31 treats
a war with Midian. Yahweh required revenge against Midian for its previous wrong of Israel
(Num. 25 ). After this military expedition, Moses was to be gathered to his people (31:1 ), a
euphemism for his death.

Paramount here is Yahweh's vengeance (v:3 )--not justice or rehabilitation--and this


vengeance was to take on the form of a nearly total annihilation of men, women, children, and
animals. The cities were to be burned also, but we are not told if the fruit and olive trees and
the vineyards were to be cut down.

Twelve thousand Israelite men warred against Midian and killed every (adult) male (v:7 ). So
far, there was obedience, but we then read that captivity rather than destruction was the end
result for the women, children, animals, and other goods (v:9 ). On the return of his army,

Volume 1990 - 2002 Issue


Page 274 of 2049
Skeptical Review Edited by Farrell Till
Moses upbraided the officers for sparing "all the women" (v:15 ), who were of the same
nationality as the women who had enticed some 24,000 Israelite men to idolatry and sexual
relations in an earlier incident (Num. 25 ).

Moses ordered that "every male among the little ones" be killed (v:17 ), but he did not enjoin
the killing of every Midianite female--only those who had had sexual intercourse with men.
The text does not put this command of Moses in Yahweh's mouth, but the context implies that
Moses was here carrying out Yahweh's word. So now the warriors killed all women who had
had sexual experience.

I find it interesting, even though it might not be all that significant, that those spared were not
called virgins [bethulah] but simply "the young girls who have not known man by lying with
him" (v:18 ). Was the writer implying by this that pagan women could not be virginal in a
high, ethical sense, but that they were only those with a physical qualification called hymen
intactus?

At any rate, the stated purpose for sparing these "intact" females was a sort of dative of
advantage: "for yourselves." And according to verse 35 , there were some 32,000 "intact"
females spared. The formula for dividing up this fortuitous bounty was 50:50 for warriors and
congregation (v:27 ). Thereupon, Yahweh's portion was 1:500 (v:28 ) of the 50% that the
warriors received, whereas the Levites' share was considerably larger: 1:50 of the 50% given
to the congregation.

The "intact" females were divided as follows: 16,000 to the warriors (and since there had been
only 12,000 warriors to begin with, and all had survived, this balanced out to one and one-
third virgins per warrior); 16,000 for the congregation (how they were distributed to the
hundreds of thousands of Israelites was left unsaid); then 32 females for Yahweh and 320 for
the Levites.

At issue here is the question: For what purpose(s) were these "intact" females spared? What
did Yahweh want or need? And what did he actually do with them? Since the Levites were
ministers at the cultic site, it is only logical to conclude that these 32 were turned over to them
for use. In the context of this story, the answer was apparently one of aposiopesis. What kind
of stated answer really could have been given the readers? Here, silence is truly golden.

In the division of the captives, last in the list were the congregation's 15,680 females. This
was the 16,000 less the Levites' 320. Parceling them out to the twelve tribes gave about 306
per tribe and thus not really very many to go around. Again, in the various households to
which they did in fact go, if used as menials, sexual temptation would certainly ensue for
fathers and sons. Question: could these females have ever become full Israelite women
through marriage?

Precedent supports an affirmative answer on the analogy of the "mixed multitude" that
accompanied Israel out of Egypt, even though the matter can be treated only indirectly from
the texts. More directly might be the precedent in which Moses, having taken a Cushite
woman to wife (LXX: aithiops, the common Greek term for "burnt faced" peoples, that is,
Hamitic types). Although Miriam and Aaron were highly critical of this marriage (even

Volume 1990 - 2002 Issue


Page 275 of 2049
Skeptical Review Edited by Farrell Till
though their explicit reason was not given), Yahweh's siding with his servant Moses supports
the inference that a foreign woman could become as legitimate a member of Israel as any
other (Num. 12 ).

On the other hand, because of the immediate association of these Midianite females, it seems
unlikely that they would have been regarded fit for marriage. Hence, the interpretation that I
deem most genial with the context's "for yourselves" (v:18 ) is that these virgins were for
whatever general disposition that comported with the Yahwistic milieu. Namely, that they
could be handled sexually. Analogy for this is Leviticus 19:20-22 , which made provisions for
an Israelite to lie sexually with a female slave even though she was betrothed to another man
(slave?). The two of them were not to be put to death, since "she has not been freed" (RSV).
Thus, I construe that Israelite men could have similarly related to these Midianite females.
CONCLUSION: The thesis question for this study, assuming only the Pentateuch for
guidance, asks about both content and practice for us today.

As for belief content, we might hold that sexual relations on a somewhat casual basis are
acceptable as long as the female is (1) intact, (2) not idolatrous, and (3) not married, even
though perhaps betrothed. We can be (1) warriors, (2) laymen, (3) priests, (4) married, and,
inferentially, (5) Yahweh himself. The latter is most shocking to the sensibility, but we are to
remember that Yahweh was directing Israelite life in great detail during this period, and,
secondly, the people were called upon to be followers, that is, emulators of Yahweh. For
interpretations that seek to give adequate emphasis to the tribal significance of Yahweh, my
inference cannot seem too farfetched.

APPLICATION: Were I, as chaplain, to have taken Numbers 31 for my sermon text and
spoken to a combat arms unit on the eve of a battle, say, in Vietnam, and further, say, My Lai,
Vietnam, I might have said the following: Men, you have read with me this chapter of God's
word to Israel just before attacking the Midianites. Let me reemphasize the salient points.
When you go into battle, you will, with success, reach a village called on our maps My Lai,
and you are to utterly destroy it with napalm and explosives. Kill every male without regard
to age, even those sucking at the breasts of their mothers. Kill the mothers too. Kill also every
female who is not a virgin. Our medics will provide clinical direction about those in doubt as
to their intact status.

You are free to take spoils of sheep, donkeys, and cattle as you have opportunity, as well as
any valuables you come across. After this battle is over--and I pray that all of you will return
safely--our commander will divvy up the virgins and the spoils on an equitable basis. The
virgins will be for yourselves. You who are married, I'm sure, have understanding wives,
because, after all, it is our God who has outlined our mission and its execution. After it's all
over, men and women (of our all- volunteer, nonsexist army), I'll lead you in a postvictory
chapel service of praise and thanksgiving to Yahweh. May God go with you all! Amen.

As distasteful and even ludicrous as the above scenario might sound, I believe that what I
have inferred and reconstructed is an accurate reflection mutatis mutandis when placed within
the context of the thesis of this article. Perhaps in no other way can we enter into the Old
Testament milieu and see what it might have been like to have lived in a certain slice of that
era. One may call my approach "heuristic" in the sense of our trying to emulate the oft

Volume 1990 - 2002 Issue


Page 276 of 2049
Skeptical Review Edited by Farrell Till
repeated dictum: "Follow the scriptures." Are avid preachers of that dictum really willing to
follow this example?

(Dr. Robert Countess is a former army chaplain, who holds the Ph.D. in NT Greek text and is
a history adjunct at Alabama A & M University in Huntsville. His address is 120 Sagewood
Circle, Toney, AL 35773.)

Once Upon a Time....


Farrell Till
Christian fundamentalists dismiss as liberal nonsense any interpretation of scriptures that is
based on the existence of myths and legends in the biblical text, yet they themselves often
take doctrinal positions that reflect a fairytale view of the Bible.

An example would be the miracles-have-ceased doctrine that is taught by all but the
charismatic (Pentecostal and Holiness) churches. The New Testament describes a first-century
church in which Christians could speak in tongues, prophesy, heal the sick, and even raise the
dead. Such charismatic practices were apparently so commonplace in the early church that the
Apostle Paul saw the need to regulate them in 1 Corinthians 14 .

Bible fundamentalists generally reject the claims of the charismatic groups who teach that
miraculous gifts of the Holy Spirit still exist. No inerrantist would say, however, that such
gifts were not given to the early Christians; they simply say that they no longer exist. They
even have a scripture to quote in support of their position: "Love never fails. But whether
there are prophecies, they will fail; whether there are tongues, they will cease; whether there
is knowledge, it will vanish away. For we know in part and we prophesy in part. But when
that which is perfect has come, then that which is in part will be done away" ( 1 Cor. 13:8-10
). Fundamentalists who quote this passage to prove the cessation of miraculous gifts would
argue that "that which is perfect" was the completely revealed word of God or New
Testament, which has now come, so the miraculous gifts present in the early church have now
ceased. In this, they betray their fairy-tale mentality, because they believe that miraculous
gifts existed once upon a time but that now they don't.

Except for the charismatics who believe that miracles still exist, if a modern day
fundamentalist experiences an illness, he will seek medical advice. However, this was not the
way it was done in New Testament times. Jesus went about casting out devils that presumably
possessed people and caused all kinds of physical and mental ailments. Those who were mute
(Mt. 9:32 ), blind (Mt. 12:22 ), epileptic (Mt. 17:14-18 ), and mentally ill (Mk. 5:1-16 ) were
thought to be demon-possessed, and Jesus healed them by casting out the demons.

Again with the possible exception of ultraconservative charismatics, Christian


fundamentalists today do not believe in demon possession. Few would even consider taking a

Volume 1990 - 2002 Issue


Page 277 of 2049
Skeptical Review Edited by Farrell Till
deaf, blind, or epileptic child to an exorcist to cast out the devils causing the affliction. They
do, however, believe that demons were active in human affairs in New Testament times, a
position they must take or else risk making their blessed redeemer look pretty foolish. In this,
their fairy-tale mentality is very much in evidence again. Once upon a time, devils possessed
human beings but not anymore.

This hermeneutic approach to the Bible becomes a convenient catch-all solution to many
problem situations in the Bible. In Genesis 9:13-17 , for example, we are told that after the
flood, God put a rainbow in the sky as a sign of his promise never to destroy the world again
by water. To the prescientific minds of the time when this was written, it probably sounded
pretty convincing. We know today, however, that rainbows are caused by the refraction of
sunlight by rain or mist. Knowing this, the modern mind quite naturally is inclined to ask how
the rainbow in Genesis 9 could have in any sense been understood as a sign of God's covenant
with Noah, because many rainbows must have been seen in the sky before the flood.

Bible fundamentalists have a once-upon-a-time explanation for this problem. They argue that
it had never rained on the earth until the flood. How then did crops grow and people obtain
fresh drinking water? The answer is in the Bible, they tell us. "There went up a mist from the
earth and watered the whole face of the ground" (Gen. 2:6 ), but rain as we know it simply
didn't exist until the flood. So once upon a time, we are assured, it didn't rain on the earth.
One wonders just how the sun shined on the earth for so long without causing that mist on the
ground to evaporate and form clouds that eventually brought rain, but to the fundamentalist
this is nothing to fret about, because anything can happen in fairy tales.

Modern medicine, despite all the progress it has made in recent years, can give us a lifespan
of only 70 some years. Just a relative few live beyond the century mark. Biblical characters,
however, routinely lived much longer than the rare centenarians of our time. The patriarch
Abraham lived to be 175 (Gen. 25:7 ), and his son Isaac 180 (Gen. 35:28 ). Jacob lived 147
years (Gen. 47:28 ), Levi 137 (Ex. 6:16 ), Kohath 133 (Ex. 6:18 ), and the list could go on and
on. These ages, although incredibly long by modern standards, were quite short compared to
the ages that the Bible attributes to their ancestors. The Genesis 5 genealogy contains a list of
patriarchs who routinely lived over 900 years. Methuselah, of course, lived 969 years (v:27),
whereas his son Lamech died a mere youngster at the age of 777 (v:31 ).

If you ask a Bible fundamentalist if these were literal ages, he will tell you without even
batting an eyelash that they were. In "The Bible, Science, and the Age of Patriarchs," an
article in the May 1992 issue of Reason & Revelation , Dr. Bert Thompson discussed various
attempts to assign figurative interpretations to these ages but concluded that they were
intended to be understood as literal ages. Dr. Thompson didn't attempt to explain why people
lived so long in those days when it is a medical struggle for people in modern times to live
even a tenth as long, but this is apparently no problem for the fundamentalist mind. Once
upon a time, people routinely lived for 900 years; now they don't.

We have to consider too the frequent appearances that God made in biblical days. He was
always dropping in unannounced to chew the fat with Abraham (Gen. 15:1-17 ; 17:1-21 ;
18:1-32 ) and the other patriarchs (Gen. 28:10-15 ; 35:1 ; 46:2-4 ) or even to do a little
wrestling (32:24-30 ). God appeared to Moses in a burning bush (Ex. 3:4 ) and again on Mt.

Volume 1990 - 2002 Issue


Page 278 of 2049
Skeptical Review Edited by Farrell Till
Sinai (Ex. 19:20-22 ). If we accept the Bible at face value, divine appearances in those days
were almost as common as dirt. We could fill a page with scripture citations that tell us God
appeared to so-and-so in a dream. If someone today claims that God has appeared to him, we
try to get psychiatric help for him. If someone 3,000 years ago claimed that God appeared to
him, we believe him and call him a prophet.

Inerrantists have a response to all this. They tell us that God had a plan that he was working
on in biblical times that required him to appear to people but that now his plan is complete, so
he doesn't need to do this anymore. However, it all amounts to the same kind of fairy-tale
hooey: once upon a time God appeared to people but not anymore.

In an earlier article ("If It Walks Like a Duck...," The Skeptical Review, Autumn 1991, pp. 2-
6), I said in reference to the sons-of-God/daughters-of-men issue in Genesis 6:1-4 that if a
story looks like mythology, sounds like mythology, and reads like mythology, one is safe in
concluding that it is mythology. The same principle is true of the once-upon-a-time mentality
so characteristic of fundamentalist attempts to explain away serious problems in the Bible
text. Once upon a time happens in fairy tales but not in real life.

We can only conclude, then, that much of the Bible does not reflect what happened in the real
life of the times that we read about in it. After all, if the emissary of an allegedly inspired
book like the Avesta, the Koran, or the Book of Mormon could resolve contrary-to-fact
problems in his "inspired" text only by telling us that once upon a time it was this way but not
anymore, who in our society would believe him? Why then should we accept the same
intellectual insult from those who want to sell us the far-fetched idea of an inerrant Bible.

Reprinted from Defender

The Essence of Prophecy


Bobby Liddell
Prophecy is, by definition, that which is spoken forth. That which is spoken forth, in true
prophecy, is that which could not be known by man unless revealed by God; thus, whether
predictive (foretelling) or declarative (forth-telling), and whether having to do with events
past, present or future, prophecy signifies the speaking forth of the will of God to man. By
prophecy, God gave instruction, information, prediction, warning, comfort and hope. By
prophecy, and its fulfillment, God confirmed His Word to be His Word; that is, divine
revelation; thus, true prophecy is found only in the Bible.

Bible prophecy has certain characteristics (all of which have been attacked by liberal critics),
which, when understood, should heighten one's appreciation for God's inspired Word. Bible
prophecy often involves that which is remote as to time and place, even that which is
nonexistent at the time of the prophecy. To the dreamer, Nebuchadnezzar, Daniel said, "As

Volume 1990 - 2002 Issue


Page 279 of 2049
Skeptical Review Edited by Farrell Till
for thee, O king, thy thoughts came into thy mind upon thy bed, what should come to pass
hereafter: and he that revealeth secrets maketh known to thee what shall come to pass" (Dan.
2:29 ). Daniel spoke of kingdoms and peoples then unknown and unable to be known to him
or any other man by ordinary knowledge, but revealed in prophecy, and whose risings and
fallings are now historically documented to have followed precisely the predicted pattern.

Bible prophecy is often very specific and detailed. With man, the more specific the prediction,
the less likely is the possibility of its coming to pass. With God, accuracy is not accidental,
nor is it incidental, but significant. Of Christ's birth, Isaiah prophesied: "Therefore the Lord
himself shall give you a sign; Behold, a virgin shall conceive, and bear a son, and shall call
his name Immanuel" (Isa. 7:14 ). Isaiah did not prophesy just that the Christ would be born,
nor that He would be born of just any woman (and man), nor that he would be born of just
any "young" woman, but that He would be born of a virgin. Matthew declares the exact
fulfillment of Isaiah 7:14 (Mat. 1:22-23 ).

In an obvious Messianic prophecy, Zechariah noted the price of Christ's betrayal, thirty pieces
of silver, which was later used to buy the potter's field (Zec. 11:12-13 ). Again, in his inspired
writing, Matthew gives the fulfillment (Mat. 26:14-15 ; 27:3-7 ), and declares these actions to
be the specific and accurate fulfillment of prophecy (Mat. 27:9-10 ). Bible prophecy is often
antithetical to human reasoning. What man could reasonably predict the downfall of
impenetrable Edom (Obadiah); of Jerusalem, home of God's temple (Mic. 3:12 ); or of
Nineveh, conqueror of nations (Nah. 1:1 )? Who would dare to predict the Son of God being
born of a virgin (Isa. 7:14 )? What man, on his own, would have predicted the long awaited
and cherished Messiah would be rejected (Isa. 53:3 ; John 1:11 ); forsaken (Zec. 13:7 ; Mat.
26:56 ); scourged (Isa. 53:5 ; Mat. 27:26 ); crucified (Psa. 22:16 ; Mark 15:24 ); and mocked
(Psa. 22:7-8 ; Mat. 27:39-44 )? Such contradictions to that which would accord with the
thinking of men show the prophecies to be miraculously given from God.

Liberal "higher critics" deny the element of prediction or foretelling in prophecy. Their
assumption is that a prophet was a man of "his own time" who spoke only to men "of his own
time." That tells only half the story. God's prophets spoke to men of their own time about
those things which were of concern and significance, but they also spoke of those things
which were future and which would be of concern and significance to all, from those then
living to those who would see the fulfillment of the prophecy and to those who would live
after that fulfillment and read the inspired record of the prophecy and its fulfillment. Contrary
to the "critics," Biblical prophecy was not written after the fact, ambiguous, artificially
fulfilled, nor just a phenomenon common to all religions and peoples. No well-attested
evidence of one miraculous "prophecy" has ever been found outside the Bible!

(Bobby Liddell's address is 4850 Saufley Road, Pensacola, FL 32526.)

The Myth of Prophecy Fulfillment

Volume 1990 - 2002 Issue


Page 280 of 2049
Skeptical Review Edited by Farrell Till
Farrell Till
At The Skeptical Review, we consider articles like the foregoing one to be an excellent source
of materials detrimental to the biblical inerrancy cause. They are incredibly superficial,
naively simplistic, and consistently illogical. We reprint them as often as we do in order to
give our readers a sampling of the best that fundamentalists have to offer in defense of their
belief that the Bible is "the inspired word of God."

Bobby Liddell, the author of the article, is also the editor of Defender, the monthly paper in
which it appeared as an editorial. I receive Defender indirectly through a friend who passes it
along to me, because Mr. Liddell refuses to let me subscribe to it. In the past, I have
challenged Liddell and many of the contributors to his paper to defend the inerrancy doctrine
in public forum, but none has accepted. Liddell, in fact, has been the only one who responded
to my letters, and his reply was merely to tell me that he would no longer send me Defender
and that he didn't want to receive The Skeptical Review anymore.

None of this, of course, is relevant to what Liddell said about prophecy in his article, but it
does give one pause to wonder. In nearly every issue of Defender, a central, guardian-of-the-
faith theme dominates. The men who write for this paper openly boast that they have the
truth, the whole truth, and nothing but the truth on their side, yet they are not at all
enthusiastic about defending that "truth" in any kind of forum that would allow cross-
examination of their beliefs by an informed opposition. They seem to prefer the security of
partisan audiences that gullibly accept just about everything they say.

In Liddell's article, for example, he parroted the old fundamentalist claim that prophecy
fulfillment confirms that the Bible is God's inspired word, a claim that neither Liddell nor any
other Bible fundamentalist can ever prove. As quoted in "An Example of 'Prophecy
Fulfillment,'" (TSR, Spring 1993, p. 4), Liddell's comrade in arms, Wayne Jackson, has cited
three criteria of valid prophecy: (1) proper timing, (2) specific details, and (3) exact
fulfillment. When all of these are applied to the examples of prophecy fulfillment that Liddell
cited in his article, they all fall short of satisfying the criteria.

Liddell referred to Daniel's prophecy of "kingdoms and peoples then unknown and unable to
be known to him or any other man by ordinary knowledge." Well, just what was this prophecy
that has so fired Liddell's enthusiasm? According to the story, Daniel was called into the royal
court to interpret a dream in which Nebuchadnezzar had seen a "great image" whose head was
of fine gold, the breast and arms of silver, the belly and thighs of brass, the legs of iron, and
the feet partly of iron and partly of clay. In the dream, the king had also seen a stone that "was
cut out without hands," which struck the image and broke it into pieces. The pieces became
like chaff and were carried away by the wind, but the stone that broke the image into pieces
grew into a great mountain and filled all the earth (Dan. 2:31-35 ).

It sounds just like another typically obscure biblical vision, but not to worry, because Daniel
was on the scene to give us a divinely inspired interpretation. He explained that the image
represented four kingdoms. Nebuchadnezzar was the head of gold, but after the demise of the
Babylonian empire would arise an inferior kingdom, represented by the breasts and arms of
silver; then after that would appear a third kingdom symbolized by the belly and thighs of

Volume 1990 - 2002 Issue


Page 281 of 2049
Skeptical Review Edited by Farrell Till
brass; and finally would come a fourth kingdom of iron that would be "partly strong and
partly broken," as symbolized by the feet that were part iron and part clay. This fourth
kingdom would break in pieces and be divided (2:36-43 ).

Daniel concluded his interpretation by prophesying that "in the days of these kings [of the
kingdoms arising from the fragmented kingdom of iron] shall the God of heaven set up a
kingdom, which shall never be destroyed: and the kingdom shall not be left to other people,
but it shall break in pieces and consume all these kingdoms, and it shall stand forever" (v:44 ).
To Bible fundamentalists, of course, this kingdom was the church, so they have had to force
an interpretation of Daniel's dream interpretation to make it appear that an amazing prophecy
fulfillment occurred in the birth of Christianity.

Here is how fundamentalists have tried to mold history into fitting Daniel's interpretation of
the dream. The Babylonian Empire was the head of gold (which I won't argue with, because
Daniel made this much of his interpretation quite clear); the Medo-Persian Empire was the
breast and arms of silver; the Grecian Empire under Alexander the Great was the belly and
thighs of brass; the Roman Empire was the legs of iron with the feet part iron and part clay.
"So, aha!" fundamentalists rhapsodize. "The kingdom of God was established in the days of
the Roman Empire, just as Daniel had prophesied."

Well, excuse me, but just where did Daniel say that the kingdom of God would be established
in the days of the Roman Empire? He simply said that it would be established in the days of
"these kings" [of the fragmented kingdom of iron]. Wayne Jackson's second criterion of "valid
prophecy" is specific details, so if Daniel was prophesying by the guidance of an omniscient,
omnipotent deity, why didn't he specifically say that after Nebuchadnezzar would arise the
Medo-Persian Empire, after which would come the Grecian Empire under a ruler named
Alexander the Great, after which would arise the Roman Empire? Then he could have told us
that "in the days of the kings of the Roman Empire the God of heaven shall set up a kingdom
that will never be destroyed." Armed with that kind of specificity, the fundamentalists really
would have had something to crow about, but, as it is, they have nothing but speculation to go
by in determining that these were the specific kingdoms that Daniel meant.

In reality, history has left Christian fundamentalists with too many empires for Daniel's
prophecy to fit the mold that they want to cast it in, so they have had to restructure empires to
fit their need. Actually, the empires and kingdoms that existed between Nebuchadnezzar and
the Roman Empire were so unstable and factional that it is difficult to determine when one
ended and another began. The Median Empire overlapped the Neo-Babylonian Empire that
Nebuchadnezzar II ruled, and they were both absorbed by the conquests of Cyrus the Great,
which gave rise to the Achaemenid or Persian Empire. The conquests of Alexander the Great,
however, absorbed the Achaemenid Empire and gave rise to a worldwide (in terms of the then
known world) Hellenistic Empire. So if the Median and Achaemenid (Persian) Empires are
considered separate entities, as actually they were and (as we will see) Daniel apparently
considered them, then the Hellenistic Empire of Alexander the Great should have been the
fourth or iron kingdom of Nebuchadnezzar's dream. Fundamentalists can't have this, however,
because it doesn't make Daniel's interpretation fit the mold they need, so they conveniently
consolidate these two and call them the "Medo-Persian Empire." That way, the Hellenistic
Empire becomes the brass kingdom so that the Roman Empire can be the iron kingdom. Then,

Volume 1990 - 2002 Issue


Page 282 of 2049
Skeptical Review Edited by Farrell Till
presto, just like that, they have an amazing prophecy fulfillment in Daniel's interpretation of
Nebuchadnezzar's dream.

A flaw in this interpretation is the obvious fact that the writer of Daniel considered the
Median and Persian kingdoms to be separate empires, because he had the Neo-Babylonian
empire falling to "Darius the Mede" (5:30-31 ). This is historically inaccurate (just one of
many historical inaccuracies in the book of Daniel), because reliable records of the time
indicate that Cyrus the Great captured Babylon and ended the Neo-Babylonian kingdom.
Nevertheless, the writer of Daniel told of a reign under "Darius the Mede" that preceded the
reign of the Persian king, Cyrus the Great (6:28 ; 10:1 ). So if the writer believed that the
Neo-Babylonian Empire fell to the Medes and then the Medes fell to the Persians, then the
fourth kingdom in Daniel's interpretation would have been Alexander's Hellenistic empire.

This is exactly how reputable Bible scholars interpret Daniel's interpretation of


Nebuchadnezzar's dream. Certain clues, which I will have to discuss later in a separate article,
indicate that the book was written around the middle of the second century during the
Maccabean wars chronicled in the apocryphal books of 1 and 2 Maccabees. These wars
occurred after the disintegration of Alexander's Empire into four separate kingdoms, so if the
scholars are right in the dating of Daniel, this gives reason to believe that the writer intended
the Hellenistic Empire to be the kingdom of iron, whose instability was symbolized in the
dream by the feet made partly of iron and partly of clay.

In 323 B.C., Alexander the Great died in Babylon at the age of 33, only seven years after he
had conquered the Persian Empire. Almost immediately, his empire fragmented into separate
political entities that were ruled by Alexander's generals. From 312 to 64 B.C., the Seleucids,
a dynasty established by the Macedonian general Seleucus, ruled a kingdom that extended
from India to the Mediterranean Sea with its capital in Antioch of Syria. In opposition to
Seleucid rule, the Greco-Bactrian kingdom arose in the far eastern sector of the old Hellenistic
empire. The rule of the Ptolemies arose in Egypt and other southwestern territories of the old
empire. By the third century B. C., nomadic Iranian tribes had begun to intrude on territory in
the Seleucid kingdom. These intrusions began in the province of Parthia under the leadership
of a chieftain named Arsaces, and slowly but surely new territories were gained from both
Seleucia and Bactria, as well as remnants of the old Median Empire, until a distinctive
Parthian Empire emerged under the leadership of king Mithradates. Eventually, the Parthian
kingdom fell to Roman conquests from the west.

Daniel's interpretation of Nebuchadnezzar's dream, however, recognized only two kingdoms


between the head of gold (Babylon) and the legs and feet of iron, but when recognition is
given to the existence of the kingdoms that arose from the fragmented Hellenistic Empire, we
find that there were at least six separate kingdoms (Persian, Hellenistic, Seleucid, Bactrian,
Ptolemaic and Parthian) between the Neo-Babylonian and the Roman Empires. Obviously,
Liddell's application of Daniel's interpretation conveniently ignores a huge slice of ancient
history so that he can have a "prophecy fulfillment" that will fit a pet theory of his.

Daniel's interpretation of the dream depicted the iron kingdom as one that would "crush and
shatter all these [prior kingdoms]" but would itself become "a divided kingdom" (2:40-41 ) as
symbolized by the feet that were part clay and part iron. In this sense, the Hellenistic Empire

Volume 1990 - 2002 Issue


Page 283 of 2049
Skeptical Review Edited by Farrell Till
fits the description of the iron kingdom much more exactly than does the Roman Empire.
Alexander the Great absorbed into his empire all of the territories in the Neo-Babylonian,
Median, and Achaemenid (Persian) Empires, but the Roman Empire came nowhere close to
doing so. The Hellenistic Empire reached as far east as China, but the Roman Empire, whose
territory was more western, fell far short of crushing and shattering all sectors of its
predecessor empires. Furthermore, the rapidity with which Alexander's empire fragmented
into the kingdoms mentioned above fits the description of the brittle feet of iron and clay
much better than the Roman Empire whose disintegration was much slower.

This is exactly the way responsible scholars, who have no cherished fundamentalist beliefs to
defend, interpret the book of Daniel. They see it as the effort of a second-century author to
instill optimism in beleaguered Maccabean forces by making it appear that a sixth-century
prophet had predicted that their struggle would eventually bring about the establishment of
the long-awaited messianic kingdom.

So this brings us to the factor of "proper timing" in Jackson's criteria of valid prophecy. Bible
fundamentalists like to think that Daniel was written in the sixth century B. C., shortly after
the events that the book closes with during the reign of Cyrus the Great, who had conquered
Babylon in 539 B. C. Few reputable Bible scholars, however, would fix the date that early,
because the book exhibits signs of a much later authorship. Scholars cite the writer's obvious
confusion about political events of the time that a contemporary author would have surely
been familiar with, the linguistic style (especially the section written in Aramaic), and other
factors too numerous to discuss in detail as evidence that the book was written at the extreme
end of the Old Testament period (no sooner than the second century). If this is so, then the
author would have known about the rise of empires that succeeded Nebuchadnezzar's. Since,
in fact, the book even told of Babylon's absorption into the Persian Empire, there can be no
doubt that the writer had first-hand knowledge of the emergence of a second kingdom after
the demise of the Neo-Babylonian Empire. If scholarship is right about a probable second-
century B. C. authorship of the book, then the writer would also have known about the rise
and disintegration of Alexander's Hellenistic kingdom. If that is so, then certainly the writer
did not speak of "kingdoms and peoples then unknown and unable to be known to him or any
other man by ordinary knowledge" as Liddell alleged in his editorial. In fact, there would be a
high degree of probability that the writer was just another second-century believer in the
imminent fulfillment of longstanding messianic promises, and so he was trying to put words
into a sixth-century prophet's mouth to make it appear that the messianic fulfillment that the
writer expected to happen shortly had been foreseen by the prophet. There is certainly nothing
in this likely scenario to get excited about.

Kenneth Nahigian's article "A Virgin-Birth Prophecy?" (Spring 1993, pp. 13-14, 16)
adequately refutes Liddell's claim that Isaiah had foretold the "virgin-birth" of Jesus seven
centuries before it happened, so there is no need for additional comment on Isaiah 7:14 ,
except to note that Isaiah made the statement to king Ahaz as a sign that the Syro-Ephraimite
alliance against Judah would not succeed, but even that part of the "prophecy" failed. Second
Chronicles 28 reports that Pekah of Ephraim attacked Judah and killed 120,000 in one day
and carried away to Samaria 200,000 "women, sons, and daughters" as captives (vv:5-8 ).
One has to wonder just how many men of Judah would have been killed and how many
women, sons, and daughters would have been taken captive had Isaiah prophesied to king

Volume 1990 - 2002 Issue


Page 284 of 2049
Skeptical Review Edited by Farrell Till
Ahaz that the Syro-Ephraimite alliance would succeed. So much for another amazing
"prophecy fulfillment."

"In an obvious Messianic prophecy," Liddell said, "Zechariah noted the price of Christ's
betrayal, thirty pieces of silver, which was later used to buy the potter's field." Oh? Well, let's
just take a close look at Zechariah's "obvious Messianic prophecy":

Then I said to them, "If it is agreeable to you, give me my wages; and if not, refrain." So they
weighed out for my wages thirty pieces of silver. And Yahweh said to me, "Throw it to the
potter"--that princely price they set on me. So I took the thirty pieces of silver and threw them
into the house of Yahweh for the potter (Zech. 11:12-13 , NKJV with Yahweh substituted for
the LORD).

One has to stretch imagination to the limits to see any connection between this and the
payment of 30 pieces of silver to Judas to betray Jesus. The only similarities at all are the
amount of money involved. One can't even argue that the "potter's field," which the chief
priests and elders bought with the 30 pieces of silver that Judas cast back to them, is a
similarity, because in the original statement no field was purchased with the money. Yahweh
simply told Zechariah to throw the money "to the potter," and Zecha- riah threw the pieces
into the house of Yahweh "for the potter."A potter is not a potter's field, so, if Matthew was
indeed referring to this statement in Zechariah, it is simply another case of a New Testament
writer straining to find something in the Old Testament that he could in some far-fetched way
relate to a contemporary event and call it prophecy fulfillment.

An even more contemptible thing about Liddell's claim that Matthew 27:9-10 "declares these
actions to be the specific and accurate fulfillment of prophecy" is that Matthew didn't even
mention Zechariah in the verses Liddell cited. What Matthew actually said was that Judas's
act of casting the pieces of silver down in the sanctuary had "fulfilled that which was spoken
through JEREMIAH the prophet." Jeremiah was not Zechariah, so how could Judas's act be a
fulfillment of what Jeremiah had said if Jeremiah was not the prophet who had said it?
Matthew said that Jeremiah had said; "And they took the thirty pieces of silver, the price of
him that was priced, whom certain of the children of Israel did price, and they gave them for
the potter's field, as the Lord appointed me" (27:9-10 ), but where did Jeremiah say this?
There is no record that he ever said it, and the statement that Matthew attributed to Jeremiah
is only remotely similar to the passage in Zechariah. How then could it be a "specific and
accurate fulfillment of prophecy"?

Liddell spewed out a long list of alleged prophecy fulfillments in a single paragraph that
space will not permit me to reply to, but they all rest on evidence entirely as flimsy as that
which I have examined in the paragraphs above. Liddell can say all that he wants to that
"Biblical prophecy was not written after the fact, ambiguous, artificially fulfilled, nor [sic]
just a phenomenon common to all religions and peoples," but saying is not the same as
proving. If he expects rational people to accept his claim of wonderful prophecy fulfillment in
biblical events, then he is going to have to do more than just assert. He is going to have to
prove.

Volume 1990 - 2002 Issue


Page 285 of 2049
Skeptical Review Edited by Farrell Till
If he would like to try his hand at proving that the birth, rejection, mockery, and crucifixion of
Jesus, as he claimed in his editorial, were prophesied in the Old Testament, we will publish
his article simultaneously with our response. I am going to prophesy that this will never
happen.

Correspondence with Gleason Archer


Farrell Till
Dr. Gleason Archer is undoubtedly the chief apostle of Bible inerrancy. His book
Encyclopedia of Bible Difficulties is the most often quoted reference work in the corre-
spondence that I exchange with Bible inerrantists. In view of his reputation, I decided to
invite him to debate this issue in a setting that would allow the seminary students where he
teaches at Trinity Evan gelical Divinity School to hear both sides. Our exchange of letters to
date is published below.

February 7, 1993

Dear Dr. Archer:

For some time now, you have regularly received our publication The Skeptical Review. If you
have read any of the materials in it, you have undoubtedly noticed that we are dedicated to
exposing flaws in the Bible inerrancy doctrine. Furthermore, we believe that our work speaks
for itself and that we have more than accomplished our goal.

In our work, we constantly see references to your book Encyclopedia of Bible Difficulties in
the letters and articles that we received from defenders of the inerrancy view. I have examined
your book many times, and I personally find it incredibly simplistic. I don't say this to insult
you but merely to express my honest opinion. As a former fundamentalist minister, I can even
understand how that one would go to unjustifiable extremes to try to defend the inerrancy
doctrine, because I once did it myself.

You teach at a divinity school where young men are being trained for the ministry, and I
assume that your staff tries to instill in them the belief that the Bible is the inerrant word of
God. I would like to propose that you and I debate this issue in a public forum that would give
your students and others the opportunity to hear both sides. If you sincerely believe that the
Bible is God's inerrant word, I would think that this proposal would be appealing to you.
Truth has nothing to fear by public examination, so if your position is the right one, you could
strengthen the faith of your students by demonstrating to them that attacks on the integrity of
the Bible cannot withstand public scrutiny.

Volume 1990 - 2002 Issue


Page 286 of 2049
Skeptical Review Edited by Farrell Till
If you are willing to participate in such a debate, I would be happy to enter into negotiations
with you to decide upon specific propositions and a time that would be mutually compatible
with our teaching schedules.

Farrell Till

February 16, 1993

Dear Mr. Till:

Thank you for your invitation to debate me concerning the accuracy and trustworthiness of
Holy Scripture. If I thought it would serve any useful purpose, I would be happy to comply.
But from what I know about you I can only conclude that it would be a mere exercise in
futility.

Let me explain that if you presented yourself as a seeker after truth, or if you were a Muslim
debater, or the exponent of some religion which has a genuine belief in a god of some sort,
and you had not been presented with the strong and compelling evidence for the unique
authority and inerrancy of the Holy Scriptures, then there might be some point in our getting
together.

But as it is, in view of the fact that you have already been confronted with the many infallible
proofs of the truth of Scripture, and were once sufficiently persuaded of them to become what
you term "a Fundamentalist minister," I can only conclude that you have fallen into the
syndrome of a passionate, quarrelsome renegade, and are therefore not really open to reason.
You now consider yourself superior to Moses and Isaiah and Jesus Christ Himself as an
expert on God, and you therefore have super seded them as a supreme authority in all things
metaphysical. You have absolutely nothing to offer to your public but disillusionment and
despair as they face the prospect of eventual death and possible judgment before the Author
and Enforcer of the moral law. You can only leave them without purpose, goal or meaning in
life, and persuade them that life is basically futile and without purpose.

In short, I feel it would be as pointless for me to debate with you as it would be for either of
us to debate with Mary Baker Eddy about the reality of matter or the value of material
medica. I must add that I feel sorry for you that you have lost all hope of salvation because of
your abandonment of your Savior. But the same God who granted me the responsibility of
free choice between Christ and Satan has granted the same to you, and I therefore respect your
prerogative to turn your back upon God if that is what you prefer.

Gleason L. Archer

February 18, 1993

Dear Dr. Archer:

Volume 1990 - 2002 Issue


Page 287 of 2049
Skeptical Review Edited by Farrell Till
I appreciate your prompt response to my letter. To be honest, I must admit that I didn't expect
any answer at all. That expectation had been based on the fact that I have written similar
letters to several inerrantist authors and lecturers only to have them ignored.

In your reply, you said that you would be happy to comply [with my invitation to debate] if
you thought it "would serve any useful purpose." You then went on to say that from what you
know about me you could only conclude that a debate "would be a mere exercise in futility." I
regret that your vision regarding the practicality of a debate is as myopic as your discernment
of biblical discrepancies. Occasionally, I encounter an inerrantist who is willing to defend his
position in public forum, but I never enter into those discussions with any illusions of
converting my opponents. I do not debate the inerrancy issue in order to educate my
opponents, because their minds are almost always anesthetized to reason and logic. I debate
them in hopes of reaching some in the audiences whose minds are still open to honest inquiry.

If you are so certain that the truth is on your side, should you not consider my proposal as an
opportunity not to reach me but to reach some in the audience who are not presently believers
in Bible inerrancy but whose minds might still be open to the "many infallible proofs of the
truth of Scripture," which you referred to in your letter? Viewed in this way, wouldn't a
debate serve some "useful purpose"? Shouldn't you also consider the students at your
seminary? I suppose that they are taught in their classes that the Bible is the inerrant "word of
God," but you surely know that when they leave they will be exposed to other opinions of the
scriptures. If while they are yet students, you should demonstrate to them in public debate that
my position is completely absurd, which you could surely do if there are indeed "many
infallible proofs of the truth of Scripture," would you not be strengthening their faith and
preparing them for what they will encounter after they leave the seminary? Viewed in this
way, wouldn't a debate serve some "useful purpose"?

I trust that you were sincere when you said that you would happily accept my debate proposal
if it would serve "any useful purpose." I have indicated to you at least two useful purposes
that a debate would have, so I hope to receive an acceptance from you by return mail.

Farrell Till

Et Tu, McDowell?

A letter similar to the one written to Archer was also sent to Josh McDowell, suggesting that a
public forum on the issue of biblical inerrancy would be a practical way for an inerrantist of
his reputation to demonstrate that his position is as sound as he claims. As this issue goes to
press, McDowell has not acknowledged the challenge, which was mailed to him early in
February. Could it be that McDowell isn't nearly as confident of his position as he appears to
be in the books that he writes primarily for partisan audiences?

Volume 1990 - 2002 Issue


Page 288 of 2049
Skeptical Review Edited by Farrell Till
Dobbs-Till Debate

As this issue of TSR is making its way to you through the slow process of bulk mail, editor
Farrell Till will be on his way to Portland, Texas, for a debate on the issue of prophecy
fulfillment. His opponent will be H. A. "Buster" Dobbs, editor of The Firm Foundation, a
monthly fundamentalist publication from which we reprinted Clarence Lavender's article
"Was It Morally Right for God to Order the Killing of the Canaanites" (Winter 1993, pp. 6-7).

Mr. Dobbs is reputed to be one of the ablest debaters in the Church of Christ. He will affirm
that fulfillments of Old Testament prophecies prove that Jesus of Nazareth was the son of
God. Till will affirm that New Testament claims of prophecy fulfillment in the person and
deeds of Jesus of Nazareth were fabrications or misrepresentations of Old Testament
Scriptures.

Scheduled for May 23-26, 1993, as part of the Gulf Coast Lectureship sponsored by the
Portland Church of Christ, the debate will consist of four two-hour sessions.

All four sessions will be recorded on both audio and video tapes. For information on cost and
distribution, interested readers should contact Thomas Gardner, 441 Souder, Hurst, TX 76111
(Tel. 817-282-2745).

Volume 1990 - 2002 Issue


Page 289 of 2049
Skeptical Review Edited by Farrell Till

The Skeptical Review


Volume Four, Number Four - 1993
Farrell Till, editor

• The Truth Will Make You Free


This article answers the question, "Why are you evangelical about skepticism?"

• Letter from a Dead Man


Dead men tell no tales-- but they do write letters, according to the Bible. This article
details a contradiction between 2 Chronicles and 2 Kings, in which Elijah writes a
letter to King Jehoram, despite the fact that he wasn't alive (on Earth, anyway) during
Jehoram's reign.

• Letter from a Dead Man: A Response


Jerry Moffitt responds to "Letter From A Dead Man".

• Because the Bible Tells Them So


Till gives his report of the Gulf Coast Lectures, which centered about the theme, "The
scripture cannot be broken". (At least, not if you're willing to bend, twist and mutilate
it.)

• Why Did Matthew Need Dead Babies?


What was Matthew's purpose in telling the tale of the slaughter of the infants in
Bethlehem?

• Prophecy Fulfillment and Probability


What are the odds that statistical arguments concerning biblical prophecy are valid?
Pretty low, as Farrell Till shows.

• How Likely Is It?


Speaking of odds, what are the odds that the Israelites in Exodus could have witnessed
so many of God's miracles, and still have doubted God's power so many times?

• From the Mailbag

Volume 1990 - 2002 Issue


Page 290 of 2049
Skeptical Review Edited by Farrell Till
• Debate on Bible Morality

• Study Aids

• Saul & the Witch of Endor

The Truth Will Make You Free


A question we are frequently asked is, "Why are you doing this?" By "this," the questioners
mean evangelical skepticism, so what they are asking is why we spend so much time
promoting freethought philosophy and especially rationalistic approaches to biblical
interpretation. The question implies that we are doing something wrong or at least something
that we are not entitled to do. Christians can publish papers, preach their religious beliefs over
the airwaves, go door to door trying to convert the unchurched, and maintain a high profile in
the community through various other evangelical activities, and no one wonders why they are
doing this or questions their right to do it. However, if an atheist or an agnostic attempts to
promote his philosophical views, his motives are impugned, and he is viewed with suspicion
and branded a troublemaker.

Why are we evangelical about our skepticism? There is no simple answer to the question. A
skeptic may be evangelical in his attitude for several reasons, not the least of which would be
the value that he puts on truth. If there is intrinsic value in truth--and we believe there is--any
truth that the skeptic may know should be shared with others. If he keeps it to himself, he
denies others the benefits of it. If one knows a medical truth but chooses to keep it to himself,
his morality is suspect. We are where we are today, scientifically and technologically, because
those who discovered truth shared it with the societies they lived in. Where would we be
today if this had not been done?

We should treat religious and philosophical truth no differently. If the truth is that the god
concept is nothing more than wishful thinking, mankind can only benefit from confronting
this reality and dealing with it accordingly. Although we try to convince people that the Bible
is in no sense the "inspired word of God," we do recognize that some excellent philosophical
information can be found in the Bible. One such example is a statement attributed to Jesus:
"You shall know the truth, and the truth shall make you free" (Jn. 8:32). The fact that this is a
biblical quotation doesn't mean that it is unworthy of the skeptic's consideration. A true
skeptic would advise readers of the Bible to accept the value of anything in it that makes good
philosophical sense, and this statement certainly does. The principle stated in it is the
foundation of modern psychotheraphy. It recognizes that the psychologically disturbed
individual begins his journey to recovery when he confronts the truth about the problems
troubling him.

Surely no one can deny that religion is the source of much psychological and social distress in
our society. Anyone inclined to dispute this should think a moment about what happened
earlier this year at Waco, Texas, and in similar episodes most of us can remember. If truth is

Volume 1990 - 2002 Issue


Page 291 of 2049
Skeptical Review Edited by Farrell Till
that the Bible is in no sense the inspired word of God but only another book among many
books purporting to be divine in origin, then think for just a moment about what happened at
Waco. All of those people died for a belief that wasn't true. How much better off would they
be had they been exposed during their formative years to ideas like those we publish in The
Skeptical Review rather than to the radical biblical views that led them down a road to
destruction? If we are right in what we say about the Bible, those people all died for nothing--
absolutely nothing! Isn't the possibility that this is exactly what happened adequate
justification for our doing what we are doing here at The Skeptical Review? If our position is
the truth, we should openly declare it to give others the opportunity to set themselves free of
an erroneous belief.

If reality is that life on earth is the result of evolutionary processes rather than special
creation, wouldn't human beings be better off by accepting that truth rather than clinging to
the belief that we were made in the image of God? If evolution is truth, what is to be gained
by investing our energy and resources into maintaining churches and temples consecrated to
deities that don't exist? If no saviors have been sent into the world to redeem us from our sins,
then what is the value in putting so much time and effort into propagating a belief that isn't
true? The truth is that if The Skeptical Review is right in its position, immeasurable harm has
been done to mankind by the very thing that we oppose. Many schools in our nation have
problems caused by widespread belief in outdated biblical views that few reputable Bible
scholars would agree with. Science courses have been watered down by education boards and
teachers who fear that teaching evolution may offend those in their communities who believe
in creationism. Almost all efforts to put sex education into our school curriculums are
opposed by Christian fundamentalists with simplistic ideas about teaching old-fashioned,
biblical morality in societies that are as different from the cultures that produced the Bible as
daylight from dark. Here again is the matter of evolution versus creationism. If evolution is
reality, then the truth is that the sex drive in our nation's youth is a mechanism that has
evolved through eons of natural selection to insure the survival of our species. In the face of
that truth, one can preach abstinence until doom's day, and it will not change the fact that sex
is a powerful natural urge more apt to follow natural impulses than idealistic sermonizing. If
the controversy continues to rage, a lot of ruined lives will no doubt be left in its wake,
because adolescents were denied knowledge that may have equipped them to make informed
decisions about the role that sex plays in human lives.

Bibliolaters delight in throwing at us some variation of Pascal's wager. "If we are right and
you are wrong, what then?" Well, what is sauce for the goose is sauce for the gander, so we
also have the right to ask, "What if we are right and you are wrong?" If that should be the
case, the damage that religion has done to mankind becomes impossible to assess. All of the
charitable work ever done in the name of religion would not be enough to compensate for that
damage. So if it is possible that the god concept on which religions and holy books are based
is wrong, the skeptic should feel no need to apologize for urging a rational examination of
that concept. In our society, the Bible more than any other single factor is the offshoot of the
god concept that obviously exercises the most influence on our way of life. As long as that is
so, we have every right to demand that those who seem determined to make the Bible an
authoritative standard in our society prove their claim that the Bible is the inerrant, inspired
word of God, because that claim is the only possible rationale they have for their Bible-based

Volume 1990 - 2002 Issue


Page 292 of 2049
Skeptical Review Edited by Farrell Till
political agenda. Without it, there is no reason why a society should concern itself with
biblical precepts any more than Koranic or Vedic precepts.

If, as we have noted, the truth will make one free, being a part of the process that leads to
discovery of truth can be personally gratifying. We receive many letters from people asking
for help in their struggle to break free of Bible fundamentalism. (If you are thinking that if
they want to be free of it, they should just declare themselves free of it, then obviously you
have never been a Bible fundamentalist, at least not the born-again kind.) Such letters delight
us, and we spare no effort seeing that the writers get the help they are asking for.

Just this summer, we received a letter from a young lady in North Carolina, who at the age of
24 had just come to the realization that the fundamentalist beliefs she had been reared in are
erroneous. Her letter described a familiar process of gradualism in her discovery of truths that
will, we hope, eventually set her free. First, she was bothered with the second-class status that
the Bible accords women, and this had produced serious doubts in her mind about the
religious instruction she had received while growing up. Then, rather ironically, a TV
program intended to promote Bible fundamentalism backfired on the producers as far as this
young lady was concerned. This is how she described the experience.

Then, early this year, CBS TV did a special about Noah's ark. When I saw the advertisement,
I rushed to record it. After all, they were going to show pictures of the ark!!! For two hours, I
waited and waited and waited, bored to death of hearing second-hand stories of people who
had supposedly seen the ark. After all, if they had photos of it, then these stories were not
needed. But, after two hours, the best they had was some half-rate satellite imagery photo,
which was not proof to me. I mean, if they can take photos of planets many millions of miles
away, then most surely they could take a photo of something right here on earth. CBS was
trying to help assure the faithful of God, but, ironically, their efforts only served to drive me
farther from belief.
The next stage in her transition was predictable: she began to investigate the other side. In her
investigation, she read books about philosophy, ancient history, the mystery religions, and
even evolution. Anyone who has ever gone through the transition from Bible fundamentalist
to skeptic could, without even reading the rest of her letter, guess what happened, because her
story is a familiar one. Once the mind of a religious fundamentalist is objectively opened to
other ideas, dramatic changes in thinking will begin to occur. This happened in her case when
her studies led to a book familiar to most skeptics, The Bible Handbook. Her letter described
her reaction to it:
But when I read page one of The Bible Handbook, I felt like a ton of bricks were hanging
over my head. I read for hours, until the sun rose, and a thousand thoughts were going through
my mind. Those many religious preachers screaming their threats of how I must be saved, lest
I burn in hellfire, but I saw it was not so, that, in fact, it is them who lacks the truth....

These first 24 hours were painful hours. So many conflicting thoughts and anger for being
lied to all these years of my life. Being taught that if I live my life in a certain way, that I will
inherit an eternal life in paradise, and then seeing the hundreds of biblical contradictions. It
blew that delusion of grandeur away.

Volume 1990 - 2002 Issue


Page 293 of 2049
Skeptical Review Edited by Farrell Till
In this young woman's case, she had a brother who had already rejected the fundamentalist
religion he and his siblings had been reared in, so she had the advantage of having someone
close that she could talk to. She said that in conversations with her brother, she couldn't help
laughing in her dismay about "how all these years, all the many religions fight among
themselves, about who had the true religion, and it only turns out that the one they all oppose
is actually the truth of truths. The atheists, of course."

So why do we do it? Why are we so evangelical about our skepticism? The answer is simple.
There are many people trapped in the throes of Bible fundamentalism who with help can find
their way out of it, as this young woman has done. As long as this is so, we will continue to
put our philosophical views into the free market of ideas and let the shoppers decide if they
want to buy them. Our contention is that we have a better product than the shoddy commodity
being peddled by Bible fundamentalists. From personal experience and the testimony of
others, we know that a message in this young woman's letter is true: If you abandon
superstition and face reality, you will be a happier person. So what is so wrong about helping
people be happy?

Letter From a Dead Man


Farrell Till
An old adage says, "Dead men tell no tales." Well, maybe so, but according to the Bible, dead
men can write letters.

After Jehoram succeeded his father Jehoshaphat as king of Judah, "he did that which was evil
in the sight of Yahweh" (2 Chron. 21:6 ). [So what else is new?] As a result, "a letter came to
him from Elijah the prophet" (v:12 ) telling him that because he had not "walked in the ways
of Jehoshaphat," he would be afflicted with a disease of the bowels so severe that his bowels
would fall out "day by day" (v:15 ). Needless to say, the Bible tells us that Jehoram died
exactly as Elijah's letter had predicted (vv:18-20 ).

The only problem is that the parallel story of Elijah and Jehoram in 2 Kings claims that Elijah
died during the reign of Jehoshaphat before Jehoram succeeded to the throne. Well, of course,
Elijah didn't die. He was "translated" into heaven in a "chariot of fire" (2 Kings 2:11-13 ), but
the point is that the biblical account of Elijah's departure from this life occurred in the reign of
Jehoshaphat.

How do we know this? It's just a simple matter of "rightly dividing the word of truth" (2 Tim.
2:15 ). Ahaziah succeeded his father Ahab as king of Israel in the 17th year of Jehoshaphat's
reign as king of Judah (1 Kings 22:51 ), but Ahaziah "did that which was evil in the sight of
Yahweh" and paid with his life (1 Kings 22:52 ; 2 Kings 1:17 ). Jehoram the son of Ahab (not
to be confused with Jehoram of Judah) succeeded his brother Ahaziah to the throne "in the
eighteenth year of Jehoshaphat king of Judah" (2 Kings 3:1 ). However, the chapter before

Volume 1990 - 2002 Issue


Page 294 of 2049
Skeptical Review Edited by Farrell Till
this, as noted above, reported Elijah's "translation" in the "chariot of fire" (2:11 ), after which
Elisha was recognized as chief of the prophets (v:15 ), a position for which Elijah had been
told to anoint him (1 Kings 19:16 ).

That the writer of 2 Kings believed that Elijah was no longer on the scene in the latter stages
of Jehoshaphat's reign is evident from 3:4-20 , which tells the story of Jehoram of Israel's
attempt to form an alliance with Jehoshaphat against king Mesha of Moab. Jehoshaphat
wanted the opinion of a "prophet of Yahweh" before agreeing to ally himself with Jehoram,
so Elisha (not Elijah) was called in to render a judgment. If Elijah had still been on the scene
at this time, surely he would have been consulted rather than Elisha, because, of all the
prophets of Yahweh, none was more respected than Elijah. Besides that, Elijah's "translation"
was clearly reported in the previous chapter.

It seems rather obvious, then, that the writer of this book thought that Elijah's "translation"
had occurred during the lifetime of Jehoshaphat. Then when Jehoshaphat died, his son
Jehoram became king of Judah in the "fifth year of Joram" (Jehoram) of Israel (2 Kings 8:16
). The names are confusing, so the readers should keep in mind that there was a king Jehoram
of Israel and a king Jehoram of Judah whose reigns overlapped. Suffice it to say that all of
this information properly sorted will show that Elijah was "translated" during the reign of
king Jehoshaphat and that Jehoram (of Judah) became king upon the death of his father
Jehoshaphat.

All of this being true, how did Elijah write a letter to Jehoram after he had succeeded
Jehoshaphat? The only possible explanation would have to be that dead men who can tell no
tales can nevertheless write letters.

Inerrantists, of course, have a how-it-could-have-been scenario that supposedly explains the


problem. Second Kings 1:17 states that Jehoram (of Israel) became king in the second year of
Jehoram of Judah (the son of Jehoshaphat). This clearly contradicts 2 Kings 3:1 (cited above),
which says that Jehoram of Israel became king in the 18th year of Jehoshaphat, but never
mind; inerrantists are not going to let a problem like this deter them from proclaiming the
marvelous harmony and unity of the scriptures. Gleason Archer tells us that a solution can be
found in the old co-regency dodge:

This [the statement in 2 Kings 1:17] appears to be in conflict with the notation in 2 Kings 3:1,
that Jehoram ben [son of] Ahab became king in the "eighteenth year of Jehoshaphat." But the
discrepancy arises from the fact that just prior to joining Ahab in the unsuccessful attempt to
recapture Ramoth-gilead from the Syrians, Jehoshaphat took the precaution to have his son
Jehoram installed as co-regent on the throne of Judah (Encyclopedia of Bible Difficulties, p.
204).

So, presto, just like that, Archer has found a way it could have been. Jehoram of Judah was
made co-regent before the death of his father Jehoshaphat, so it was during Jehoram's co-
regency that Elijah wrote him the letter.

A major weakness of this "explanation" is that Archer cited no book, chapter, and verse as
proof that the writers of Kings and Chronicles understood that Jehoshaphat did indeed install

Volume 1990 - 2002 Issue


Page 295 of 2049
Skeptical Review Edited by Farrell Till
Jehoram (of Judah) as a co-regent, and the reason he didn't is because there is none. Like most
inerrantists looking for a way out of an embarrassing situation, Archer just arbitrarily declared
that this was the way it was.

Another problem with the quest for harmony in this story is the hopelessly confused
chronology in the story of Jehoshaphat as the writer(s) of Kings told it. When Ahab of Israel
was killed in battle, his son Ahaziah succeeded him in the "seventeenth year of Jehoshaphat"
(1 Kings 22:51 ). He reigned for two years (same verse), after which his brother Jehoram (of
Israel) became king in the "eighteenth year of Jehoshaphat" (2 Kings 3:1 ). How could
Ahaziah serve as king for two years while only one year was transpiring in Jehoshaphat's
reign? Inerrantists, of course, will say something about different ways of calculating. One
passage was counting part of a year as a full year, so Ahaziah, who reigned for a full year and
part of a year, reigned for two years; the other passage was written in terms of full or
complete years. Hence, Ahaziah became king in Jehoshaphat's 17th year and reigned one full
year and part of another, so his brother Jehoram (of Israel) became king in Jehoshaphat's 18th
year (in terms of full or complete years). Yeah, right. It's always something like that, isn't it?

Maybe we could swallow a how-it-could-have-been scenario like this were it not for more
confusing chronological problems in this story. (Brace yourself; this stuff gets really
complicated.) After Jehoram of Israel became king in the 18th year of Jehoshaphat, the latter
died, and Jehoram of Judah succeeded him in the fifth year of the reign of Jehoram of Israel
(2 Kings 8:16 ). But wait a minute. Jehoshaphat reigned for 25 years (1 Kings 22:42 ; 2
Chron. 20:31 ). So if Jehoshaphat reigned for 25 years and if Jehoram of Israel became king
in the eighteenth year of Jehoshaphat, how could it be that Jehoshaphat's son (Jehoram)
became king of Judah in the fifth year of Jehoram of Israel's reign? Eighteen plus five equals
only twenty-three, so by necessity, it would have been in the seventh year of Jehoram of
Israel's reign that Jehoshaphat's son Jehoram (of Judah) became king. Either that, or it was in
the twentieth year of Jehoshaphat's reign that Jehoram (of Israel) became king. If not, why
not?

But it gets even worse. After succeeding Jehoshaphat, Jehoram of Judah reigned for eight
years (2 Kings 8:17 ; 2 Chron. 21:5 ). Very well, this would mean that Jehoram of Judah died
in the 13th year of the reign of Jehoram of Israel, because Jehoram of Judah became king in
the fifth year of Jehoram of Israel, but the total length of Jehoram of Israel's reign was only
twelve years (2 Kings 3:1 ). [I told you it was going to get complicated.] Second Kings 8:25 ,
however, states that after the death of Jehoram of Judah, his son Ahaziah became king in the
twelfth year of Joram (Jehoram) of Israel. And if that isn't complicated enough for you,
consider that one chapter later (9:29 ) we are told that Jehoram of Judah's son (Ahaziah)
began to reign in the eleventh year of Jehoram of Israel. About the only thing more confusing
than this chronological mess is the audacity of fundamentalist preachers who insist that they
see no reason in a maze of confusion like this to question the inerrancy of the scriptures.

Let's just put the matter to the cutting edge of Occam's razor. Which is more likely, that all of
this confusion can be explained by undocumented co-reigns and "different methods of
calculation," or that the Bible writers just got some of their "facts" confused? The latter seems
far more likely. As any student of the Old Testament knows, the writers of Kings and
Chronicles relied on various sources for their information: Acts of Solomon (1 Kings 11:41 ),

Volume 1990 - 2002 Issue


Page 296 of 2049
Skeptical Review Edited by Farrell Till
Chronicles of the Kings of Israel (1 Kings 14:19,29 ); Chronicles of the Kings of Judah
(15:7,23 ); Book of the Kings of Israel (1 Chron. 9:1 ); Book of Nathan the Prophet (1 Chron.
9:29 ). The various references are just too numerous to list them all, but the writer(s) of 2
Kings acknowledged that he (they) had drawn information about Jehoram of Judah from "the
book of the chronicles of the kings of Judah" (8:23 ). In the midst of all this borrowing of
information, wouldn't there have been a distinct probability that the end result would be a
patchwork containing at least some discrepancies that escaped the compilers' notice? If
inerrantists want to talk about how-it-could-have-been scenarios, here is one that makes far
more sense than the arbitrary stuff they resort to. At least, there is biblical documentation to
support mine. For theirs, they have nothing but arbitrary theories about unreported co-reigns
and obscure methods of calculation.

So let's put all this confusion on the back burner for a while and return to the matter of Elijah's
letter to Jehoram (of Judah). Gleason Archer wants us to believe that Elijah wrote this letter
while Jehoshaphat was still alive and Jehoram was serving only as his co-regent, but there are
problems with this theory other than the obvious fact that the biblical records mention no
Jehoshaphat-Jehoram co-reign. Time after time, the Bible described the Judean kings as men
who "did that which was evil in the sight of Yahweh," but Jehoshaphat was a rare exception
to this rule. He "walked in the way of David and sought not unto the Baalim but sought to the
God of his father, and walked in his commandments and not after the doings of Israel" (2
Chron. 17:3 ). Because of this, "Yahweh established the kingdom in his hand" (v:5 ) and
Jehoshaphat "was lifted up in the ways of Yahweh" (v:6 ). Later it was said of him that he
"walked in the way of Asa his father and turned not aside from it, doing that which was right
in the eyes of Yahweh" (2 Chron. 20:32 ; 1 Kings 22:44 ). Even when Jehu the seer
reprimanded Jehoshaphat for allying himself with Ahab in the attempt to recapture Ramoth-
gilead, Jehu praised him for putting away the Asheroth (pagan worship) and setting his heart
to seek God (2 Chron. 19:3 ). During his reign, he sent his princes with priests and Levites
into the cities of Judah to teach the people from

"the book of the law of Yahweh" (2 Chron. 17:7-9 ), and he restructured the judicial system
by setting judges in all of the fortified cities of Judah with instructions to "judge not for man
but for Yahweh" (2 Chron. 19:5-7 ). In view of Jehoshaphat's dedication to the laws of
Yahweh, how likely is it that he would have allowed his son Jehoram to serve as co-regent in
such a corrupt manner that the most notable prophet of his religion would have seen the need
to write a letter of reprimand? Archer's speculative theory requires us to believe that
Jehoshaphat cleaned up just about everything in his government except the evil co-regency of
his son. That is asking a bit too much.

A more reasonable interpretation is that Jehoram (of Judah) became king after Jehoshaphat's
death and then abandoned the righteous principles of his father to such a degree that Elijah
sent him the letter of reprimand. This is exactly what the biblical text states:

And Jehoshaphat rested with his fathers, and was buried with his fathers in the City of David.
Then Jehoram his son reigned in his place (2 Chron. 21:1, NKJV).

Volume 1990 - 2002 Issue


Page 297 of 2049
Skeptical Review Edited by Farrell Till
In no sense is this passage describing a co-reign. Jehoshaphat died, and THEN Jehoram his
son reigned in his place. That is about as clear a statement as you will find in this entire
confused record of Jehoshaphat's and Jehoram's reigns.

Immediately after assuming power, Jehoram removed all competition by executing his six
brothers (v:3 ). Now how likely is it that Jehoshaphat stood by and allowed this to happen
during a co-regency that he shared with Jehoram? In point of fact, the text makes it very clear
that this atrocity was committed after Jehoshaphat was dead:

Now when Jehoram was established over the kingdom of his father, he strengthened himself
and killed all his brothers with the sword, and also others of the princes of Israel (v:4).

Certainly Jehoram could not have established himself over the kingdom of his father while a
monarch of Jehoshaphat's power was still alive, so obviously it was after Jehoshaphat was
dead that Jehoram murdered his brothers. If anyone still doubts this, let him consider the full
text of the letter that Elijah sent to Jehoram:

Thus says Yahweh, the God of your father David: Because you have not walked in the ways
of Jehoshaphat your father, or in the ways of Asa king of Judah, but have walked in the way
of the kings of Israel, and have made Judah and the inhabitants of Jerusalem to play the harlot
like the harlotry of the house of Ahab, and also have killed your brothers, those of your
father's household, who were better than yourself, behold Yahweh will strike your people
with a serious affliction--your children, your wives, and all your possessions; and you will
become very sick with a disease of your intestines, until your intestines come out by reason of
the sickness, day by day (vv:12-15).

Could anything be clearer than this? Elijah (if indeed he wrote this letter) knew that Jehoram
had already murdered his brothers. So to believe that Elijah wrote this letter while Jehoram
was simply a co-regent requires one to believe that Jehoshaphat, who had had the power to
institute sweeping political reforms and even maintain garrisons in some of the cities of
Ephraim that Asa had regained from the northern kingdom (2 Chron. 17:2 ; 19:4 ), was unable
to prevent Jehoram from murdering his other sons. Who can believe it?

Besides this, the reference to Elijah's letter is near the end of the chronicler's version of
Jehoram's life, an indication that it was written well into the eight-year reign of Jehoram (21:5
) and not at some time during a hypothetical co-regency. The letter said that Yahweh would
strike Jehoram's people with "a serious affliction" that would include his children, wives, and
all his possessions and that then Jehoram himself would be afflicted in his bowels.
Immediately upon continuation of the narrative, the chronicler reported that Yahweh stirred
up against Jehoram the Philistines and Arabians, who invaded Judah and carried away all the
possessions in the king's house as well as his wives and sons, except for Jehoahaz (vv:16-17 ).

"After all this," the very next verse tells us, "Yahweh struck him [Jehoram] with an incurable
disease." Then after two years, "his bowels fell out by reason of his disease," and he died in
great pain (v:19 ). So the obvious intention of the chronicler was to report that Jehoram's
wickedness became so great during his reign that somewhere close to six years after Jehoram
had become king, Elijah pronounced a curse upon him in a letter, and then he died about two

Volume 1990 - 2002 Issue


Page 298 of 2049
Skeptical Review Edited by Farrell Till
years later, having reigned for a total of eight years. So the problem for the fundamentalists
who still want to insist that the Bible is inerrant in every detail is that they must explain how
Elijah wrote this letter when he was no longer on the scene at this time in Jehoram's reign.

Now there is only one loose end to tie up. If there was no co-regency during the reign of
Jehoshaphat, why did the writer of 2 Kings say in 1:17 that Jehoram of Israel began to reign
in the second year of Jehoram of Judah and then say in 3:1 that Jehoram of Israel began to
reign in the 18th year of Jehoshaphat? Most assuredly, what we cannot do is resort to an
assumption of biblical inerrancy as grounds for arguing that there cannot be a discrepancy in
the two passages. Objectivity requires us to admit the distinct possibility that a fallible human
writer simply got his facts confused.

The likelihood that this is exactly what happened can be found in a fact previously mentioned:
the writers of the books of Kings and Chronicles frequently referred to source materials that
they had used in compiling their histories. The writer(s) of Kings acknowledged that "the
chronicles of the kings of Judah" had been used to compile Jehoshaphat's and Jehoram of
Judah's stories (1 Kings 22:45 ; 2 Kings 8:23 ); the chronicle writer(s) said that the story of
Jehoshaphat had depended on "the history of Jehu the son of Hanai, which is inserted in the
book of the kings of Israel" (2 Chron. 20:34 ). On the other hand, the story of Ahaziah of
Israel was compiled from information in "the chronicles of the kings of Israel" (2 Kings 1:18
). The writers referred to three different source books in telling the stories of Ahaziah and
Jehoram of Israel and Jehoshaphat and Jehoram of Judah, so the glaring discrepancy in 2
Kings 1:17 and 3:1 could easily have resulted from contradictory dates in the writer's sources
that he inserted, probably inadvertently, while compiling his own version of the history of this
period. Anyone who doubts that this could have happened is someone who doesn't have much
experience in researching and writing. To argue that it couldn't have happened because the
writers were inspired is to prove inerrancy by assuming inerrancy.

One thing is obvious. The history of Jehoshaphat and the two Jehorams (one an Israelite, the
other a Judean king) is a confusing mess of chronology from beginning to end. That doesn't
do much to instill confidence in the fundamentalist claim that the Bible is so perfectly
harmonious from cover to cover that only divine inspiration can account for it.

Letter From a Dead Man: A Response


Jerry Moffitt
It is a great joy to write a response to Mr. Till's article "Letter From a Dead Man." It is an
undeserved honor to find myself writing in defense of the Bible, but the God of all mercy may
help me to do that of which He deems me to be capable. Before I close in on the alleged
contradictions, I want to make some preliminary remarks that may be useful.

Volume 1990 - 2002 Issue


Page 299 of 2049
Skeptical Review Edited by Farrell Till
GOD DOES NOT WANT EVERYONE TO
UNDERSTAND
No, I am certain that God does not want all who read His word to understand it. He is self-
revealing, but to those with rebellious, wicked hearts He is also self-concealing. He refuses to
reveal Himself in the Scriptures in a way that is psychologically compelling, so that one
cannot reject the evidence. He presents His word in a way where men or women with certain
sinful traits will weed themselves out, so that heaven may be filled only with honest and good
hearts (Luke 8:15 ). For lack of space, I will give only one quotation. We read, "At that season
Jesus answered and said, I thank thee, O Father, Lord of heaven and earth, that thou didst hide
these things from the wise and understanding, and didst reveal them unto babes: yea, Father,
for so it was well-pleasing in thy sight" ( Matt. 11:25-26 ). To prove the alarming subtitle I
made--"God Does Not Want Everyone to Understand"--read also Job 23:8-9 ; Deuteronomy
28:29 ; Romans 1:21-28 ; 1 Corinthians 1:18-29 ; 1 Corinthians 3:19-21 ; John 7:17 ; John
8:37 ; John 8:46-47 ; John 10:26-28 ; Daniel 12:10 ; Matthew 13:11-16 ; Mark 4:9 ; and John
12:40 .

THE DEFINITION OF A CONTRADICTION


Before we directly analyze Mr. Till's article, J. W. McGarvey has some words that should be
considered:

Two statements are contradictory not when they differ, but when they cannot both be true. If,
on any rational hypothesis, we may suppose them both to be true, we cannot rightfully
pronounce them contradictory. We are not bound to show the truth of the given hypothesis;
but only that it may be true. If it is at all possible, then it is possible that no contradiction
exists; if it is probable, then it is probable that no contradiction exists; and the degree of the
latter probability is measured by that of the former.... It follows, also, that when there is an
appearance of contradiction between two writers, common justice requires that before we
pronounce one or both of them false we should exhaust our ingenuity in searching for some
probable supposition on the ground of which they may both be true. The better the general
reputation of the writers, the more imperative is this obligation, lest we condemn as false
those who are entitled to respectful consideration (Evidences of Christianity, 1886, Part 2, p.
32).

Those of you who have read Mr. Till's article "Letter From a Dead Man" will notice right off
that Mr. Till has failed in regard to McGarvey's comments in several respects. He has decided
that there is a contradiction though he has not proved it. By God's help in all things, I will
show that there are some possible rational hypotheses and even some probable ones that make
Mr. Till's efforts fall short regarding proving a contradiction in the Bible. It seems to me I can
be most clear and concise by distilling from Mr. Till's article certain questions found there.
These, if the Lord will, I will treat one at a time.

QUESTION ONE

Volume 1990 - 2002 Issue


Page 300 of 2049
Skeptical Review Edited by Farrell Till
"If Elijah ascended to heaven in the reign of Jehoshaphat, how could Jehoram, the son of
Jehoshaphat get a letter from Elijah after Jehoshaphat was dead?"

There are several sound hypotheses regarding this conundrum, none of which Mr. Till can
refute. The first one seems more likely to me. It is that Elijah was not dead when he wrote the
letter to Jehoram. Notice this quotation from Albert Barnes regarding Elijah's translation:

The events of this chapter are related out of their chronological order. Elijah's translation did
not take place till after the accession of Jehoram in Judah (2 Chr. 21:12), which was not till
the fifth year of Jehoram of Israel (8:16). The writer of Kings, having concluded his notices of
the ministry of Elijah in chapter 1, and being about to pass in chapter 3 to the ministry of
Elisha, thought it best to insert at this point the final scene of Elijah's life, though it did not
occur till several years later (1 Samuel-Esther, p. 228).

Now all this is not only a rational hypothesis, it answers the crux and core of Mr. Till's article.
All the rest is minor and easily answered, and all in all it must be highly frustrating to Mr. Till
who after all his effort has not found a mistake in the Bible.

However, some put forth the hypothesis that there may have been a co-regency, and Jehoram
got Elijah's letter while he ruled along with his father Jehoshaphat. This hypothesis is
possible, though not as probable in my estimation as the other. But 2 Kings 1:17 and 2 Kings
3:1 indicate that there was a co-regency. Barnes says on the two references, "Jehoram of
Judah perhaps received the royal title from his father as early as his father's sixteenth year,
when he was about to join Ahab against the Syrians; the same year might then be called either
the eighteenth of Jehoshaphat or the second of Jehoram" (Barnes, pp. 227-228).

QUESTION TWO
"If Ahaziah of Israel began to reign in the 17th year of Jehoshaphat and reigned two years,
how could Ahaziah's brother then succeed him in only the 18th year of Jehoshaphat? How
could Ahaziah serve as king for two years while only one transpired in Jehoshaphat's reign?"

It is true that Bible writers reckon parts of years in different ways, and we ourselves do parts
of years in various ways. First Kings 22:51 has two years because part of the second year is
consumed. The way we might say it is "a little over a year," but their way is not unsound to
them. On 2 Kings 3:1 , Barnes, who is good on chronology, says "in the eighteenth year of
Jehoshaphat. This date agrees exactly with the statements that Jehoshaphat began to reign in
the fourth of Ahab (1 Kin. 22:41 ), and Ahaziah in the 17th of Jehoshaphat (1 Kin. 22:51 )."

Someone could ask how long I worked in San Marcos, Texas. Various members of the
congregation might answer "four years," or "four and a half," or "four, going on five," or
"about five," and by our reckoning all would be technically correct. But the ancient quite
often counted any part of a year, no matter how small, as a whole year. We call 33 A.D. as the
first century, though it is only a small way into that century, and it is not yet a full one
hundred years.

Volume 1990 - 2002 Issue


Page 301 of 2049
Skeptical Review Edited by Farrell Till
QUESTION THREE
"If Jehoshaphat reigned 25 years, how could Jehoram of Israel begin to reign in Jehoshaphat's
18th year and five years later see Jehoram of Judah succeed Jehoshaphat? That adds up to 23
not 25 years."

All we have said before applies here. As Barnes says, "Those, however, who regard them [1
Kings 22:42,51 ; 2 Kings 3:1 ; 1 Chronicles 20:31 ] and 1:17 as sound, suppose that
Jehoshaphat gave his son the royal title in his sixteenth year, while he advanced him to a real
association in the empire seven years later, in his twenty-third year. Two years afterwards,
Jehoshaphat died, and Jehoram became sole king" (Barnes, p. 248).

Another hypothesis is that some later scribe accidentally made the mistake of repeating
"Jehoshaphat being then king of Judah" in this place. To be certain in this type of calculation,
we usually need more information. Notice Douglas:

Until about a century ago Old Testament dates were calculated almost entirely from the
biblical statements.... Two difficulties beset this approach. Firstly, the Old Testament does not
provide all the details needed for this task, and some sequences of events may be concurrent
rather than consecutive. Secondly, the ancient versions, e.g., the LXX, sometimes offer
variant figures. Hence schemes of this kind are subject to much uncertainty (The New Bible
Dictionary, p. 212).

So both hypotheses are possible, but we need more information to see which is right.

QUESTION FOUR
"How could Jehoram of Israel reign 12 years if Jehoram of Judah began to reign in the fifth
year of Jehoram of Israel and reigned eight years into the reign of Jehoram of Israel? Eight
plus five equals thirteen, not twelve."

Again recall the previous answer. Barnes notes that the "eight years are counted from his
association with his father in the kingdom. They will end in the twelfth year of Jehoram who
was in Israel."

However, Mr. Till makes the assumption that the Bible has a continuous line of chronology
and that all Bible writers use the same system of dating. Both assumptions, on which Till
bases his contradictions, are wrong. Notice The Eerdmans Bible Dictionary:

The biblical accounts indicate no absolute, continuous chronology by which all events can be
dated, and archaeological findings generally provide only relative correlations. The process of
determining dates of persons and events, and occasionally even of historical sequence, is
made even more complex by the use of various systems of dating and by the nature of the
writings themselves, whose interest is primarily theological rather than precisely historical
(Edited by Bruce M. Metzger, p. 213).

Volume 1990 - 2002 Issue


Page 302 of 2049
Skeptical Review Edited by Farrell Till
So God does not try to equip us with a chronology. Since God's purpose is religious,
extraneous and even necessary chronological details are not deemed by God as essential.
Since information is lacking, we must hypothesize.

QUESTION FIVE
"Ahaziah of Judah became king in the 12th year of Joram of Israel. Then one chapter later we
are told Jehoram of Judah's son Ahaziah began to reign in the eleventh year of Jehoram of
Israel. How do you explain the difference?"

Recalling all we have said before, notice Barnes says, "The discrepancy may be best
explained from two ways of reckoning the accession of Ahaziah, who is likely to have been
the regent for his father during at least one year" (p. 252). This hypothesis, without better
information, is likely because Jehoram was obviously critically ill the last year of his life. See
2 Chronicles 21:19 .

QUESTION SIX
"Is it not easier to accept that they made a mistake rather than that there was a co-regency?"

No. Such a supposition leads to many unanswered, complicated questions. Who made a
mistake? In what source was the date wrong? Did they round off? Did some "sources" count a
co-regency and others not? Was the source copied differently from the original? Did they
reckon years and parts of years the same? Were their numbers specific or rounded off? What
books were used as sources? Were they inspired books? If they were inspired writers, why
would they need sources? Were there copyist errors in the "sources"? Were there copyist
errors in the biblical manuscripts after inspired writers penned them? Till's suggestion is too
complex.

To PROVE a Bible error Mr. Till must definitely answer each of the above questions. But
there is much information regarding co-regency. It is the simplest solution. Co-regencies were
common in Judah. Douglas says, "It is possible to demonstrate, as he [E. R. Thiele in
Mysterious Numbers of the Hebrew Kings, 1951] has done, co-regencies between Asa and
Jehoshaphat, Jehoshaphat and Jehoram, Amaziah and Azariah (Uzziah), Azariah and Jotham,
and Jotham and Ahaz.... This practice of co-regencies in Judah must have contributed notably
to the stability of that kingdom; David and Solomon had thus set a valuable precedent"
(Douglas, p. 217).

QUESTION SEVEN
"Would Jehoshaphat have cleaned up everything but his own co-regency?"

When David was young, his son Absalom almost took the kingdom away from him. When
David was old, Adonijah almost did the same. In the last years of the life of Jehoshaphat,
when he was old, feeble, weak, and maybe even somewhat senile, could not Jehoram have
taken control?

Volume 1990 - 2002 Issue


Page 303 of 2049
Skeptical Review Edited by Farrell Till
CONCLUSION
True, the Bible claims inerrancy, but, no, we never say there is no contradiction because the
Bible is inerrant. We believe the Bible is inerrant in great part because no contradiction has
been proved. But when it comes to making an exact chronology, we do not claim to have
enough information to do so. The Bible as a whole does not try. Bible writers used differing
systems of datings, did not provide enough details, followed no perfectly arranged order of
periods, years, and dates, and they focused on presenting a religious document rather than a
historical one. The student of chronology goes to the Bible well aware that he is severely
handicapped by lack of information and tested beyond his reach because of a lack of details
and a constant lack of agreement in the treatment of dates and years. When it comes to
chronology, only the skeptic thinks he has sufficient data to claim infallibility.

(Jerry Moffitt's address is P. O. Box 1275, Portland, TX 78374.)

Because the Bible Tells Them So


Farrell Till
For four days during the Gulf Coast Lectures in Portland, Texas, I sat patiently listening to
fundamentalist preachers speak on various subjects related to the Bible inerrancy doctrine.
Altogether, I sat through nineteen 35-minute lectures centered around the lectureship theme,
"The scripture cannot be broken," but all I heard was a continuous babble of non sequiturs,
question begging, and circular reasoning. The primary argument of the speakers was that the
Bible is inspired of God, because it says that it is.

When I say this, I am not exaggerating. One speaker lecturing on the reliability of biblical
numbers said that he knew the ancient patriarchs listed in Genesis 5 lived for hundreds of
years, "because the Bible says they did." Another speaker whose subject was "Did Moses
Write the First Five Books of the Bible?" began by saying that the answer was a simple yes.
"Even when our answer was challenged [in the past]," he said, "most would be able to
respond with, 'The Bible tells us so.'" Such was the evidence presented at this lectureship to
prove that "the scripture cannot be broken." Whatever the Bible says was accepted as
irrefutable proof, so, in effect, they were arguing, "The scripture cannot be broken because we
and the Bible say that it can't."

After the lecture on the authorship of the Pentateuch, I had a brief conversation with the
speaker during which I said that he would at least have to admit that Moses did not write the
last chapter of Deuteronomy. His response was, "Oh, why's that?" When I replied that this
chapter recorded the death and burial of Moses and that surely he could not believe that
Moses wrote a chapter of the Pentateuch after he was dead, the speaker said that he would
have to check that. This ended the conversation, because I couldn't see much value in

Volume 1990 - 2002 Issue


Page 304 of 2049
Skeptical Review Edited by Farrell Till
discussing the authorship of the Pentateuch with an "expert" who apparently didn't know that
it ends with a record of Moses' death and burial.

I was at the lectureship to debate the issue of prophecy fulfillment with H. A. "Buster" Dobbs,
a Church-of-Christ preacher and editor of the fundamentalist paper Firm Foundation. He
affirmed that prophecy fulfillment proves that Jesus of Nazareth was the son of God; I
affirmed that New Testament claims of prophecy fulfillment in the person and deeds of Jesus
of Nazareth were fabrications or misapplications of Old Testament scripture. At debates,
people tend to see what they come to see, and the occasional "amens" in response to my
opponent's frequent "plays to the gallery" and the hearty congratulations he received from the
crowd at the end of each session indicated that the largely fundamentalist audience that had
come to see their man give the atheist a good shellacking got what they had come for.

From my perspective, the results were entirely different. I opened the debate with an
affirmative speech that presented five New Testament claims of prophecy fulfillment that are
obvious fabrications. These were John 7:38 where Jesus allegedly said, "He that believeth on
me, as the scripture hath said, from within him shall flow rivers of living water"; Matthew
2:23 where it was claimed that the "prophets" had said that Jesus would be called a Nazarene;
Matthew 27:9 where it was said that Jeremiah had prophesied of the 30 pieces of silver that
Judas cast down before the chief priests; and Luke 24:46 and 1 Corinthians 15:4 where both
Jesus and Paul claimed that it had been written in the scriptures that the Christ would rise
from the dead on the third day.

As each of these fabricated prophecy fulfillments was introduced, I emphasized to the


audience that Mr. Dobbs could "send me packing immediately" by just citing books, chapters,
and verses where any such "prophecies" as these had been made in the Old Testament as
claimed by the New Testament writers. As much as time allowed, I warned the audience of
the quibbles that Dobbs might use in his evasion of the issue. As predicted, these were the
exact quibbles that Dobbs resorted to, and to save face he said that he really had nothing to
reply to because I had rebutted my own arguments. To the quibbles that I predicted, he did
add another: the Old Testament very often gave the "substance" of prophecies without giving
the "particulars," and sometimes prophecies were "acted out" in the Old Testament. As an
example, he cited the exodus as an "acting out" of the calling of Jesus out of Egypt, which
was one of Matthew's many prophecy fulfillment claims.

Another of his quibbles was that Jesus was called a Nazarene in Isaiah 11:1 and 53:2 , which
referred to a "branch" from the stock of Jesse. Dobbs contended that the Hebrew word netser
(branch) is the word from which Nazareth came so that in effect Nazareth could have been
called "Branchtown." In subsequent speeches, I cited Strong and Eerdmans Bible Dictionary,
as well as a Bible dictionary available in the Portland public library, all of which say that the
etymology of the name Nazareth is uncertain, yet Dobbs stuck to this quibble.

During one of the morning lectures, an appeal was made for personal contributions to help
raise $1200 to send a man in the audience to the Memphis School of Preaching to train for the
ministry, so at one point in the debate, I produced my personal checkbook, held it up before
the audience, and said that I would write a personal check to this man for the entire $1200 if
anyone in the audience would produce the exact Old Testament scripture where it was

Volume 1990 - 2002 Issue


Page 305 of 2049
Skeptical Review Edited by Farrell Till
unequivocally said that the Messiah would be called a Nazarene or where the words Nazareth
or Nazarene were even used, period, in the Old Testament. I even stopped speaking at one
point and stood in silence for 30 seconds to allow anyone in the audience (which had many
preachers in it) to produce the scripture. Needless to say, nobody did, yet the crowd
congratulated Mr. Dobbs for a job well done. You figure it out.

An unfortunate aspect of the debate was Mr. Dobbs' imposition of a set of rules that interfered
with open discussion of the issues. Dobbs had adamantly stated that he would not debate
unless we used his rules, so in order to have a debate, I agreed at the last moment to let him
impose his rules. One of the rules permitted the negative speaker to suspend time indefinitely
to question the affirmative, and another rule (after the opening round of 25-minute speeches)
limited the negative speaker to no more time than the affirmant took in his speeches. Thus, in
a time frame that should have allowed me two 25-minute and four 20-minute affirmative
speeches, I got to give only two 25-minute and one 10-minute affirmative speeches, so I was
unable to present most of my materials.

Through abuse of the other rule cited above, Dobbs turned the last session of the debate into a
farce. After my 25-minute rebuttal of his affirmative speech, he would walk to the podium
and make an assertion and then sit down. The time he used to make his assertions ranged from
six to 90 seconds, so about seventy minutes were spent with the two of us walking back and
forth from our tables to the podium, him to make an unsupported assertion and me simply to
remind the audience that assertions are not arguments and therefore cannot possibly be
rebutted in just a matter of seconds. None of his partisans in the audience would have dared
admit it to me, but at this stage of the debate, I think I saw several expressions of
embarrassment in the crowd.

In his first affirmative speech, Dobbs spent most of his 25 minutes playing to the gallery.
Reading from a prepared speech that he finished in 24 minutes and 58 seconds, he made
constant references to my detestable atheism and the despair and hoplessness that it
represented. During all this time, he said nothing that could in any way be construed as a
defense of his proposition. In the final minutes of the speech, he referred to a chart that
presented thirty-two prophecy-fulfillment claims, but refer was all he did. He made no
attempt at all to analyze the Old Testament statements to show that they did in fact mean what
fundamentalists usually interpret them to mean and presented no evidence, other than the
mere New Testament references, to prove that the events actually did happen in the life of
Jesus. Throughout the debate, I pressed Dobbs for extrabiblical evidences that Jesus was an
actual historical character, but he was unable to present any beyond the well known disputed
statements from Josephus and Tacitus. After I presented reasons why critics consider both
statements spurious, he never brought them up again.

Like the fundamentalists in the audience who went away satisfied that they had gotten what
they came to see, I probably cannot evaluate the results of the debate objectively, but in my
opinion, Dobbs had nothing to offer in defense of the prophecy-fulfillment argument but the
usual tripe that has been discredited and rediscredited a thousand times. I have had the
opportunity to view the video tapes of the debate, and my impression is stronger than ever
that Mr. Dobbs failed miserably to produce evidence to support his prophecy-fulfillment

Volume 1990 - 2002 Issue


Page 306 of 2049
Skeptical Review Edited by Farrell Till
belief. From beginning to end, he could do nothing but argue that the Bible says X, so,
therefore, it has to be true.

While the debate was in progress, I complained about Dobbs's abuse of the suspension-of-
time rule mentioned above and particularly his insistence on preaching little sermonettes
rather than answering the questions when I was cross-examining him. At the time, I was
concerned that his tactics were interfering with open discussion of the issues. However, after
having viewed the tapes, I was able to see that, if anything, his strategy backfired on him.
Although diehard fundamentalists will no doubt think that he cleverly manipulated the rules
to give himself additional speaking time, I am sure that objective, open-minded viewers of the
tape will get an entirely different impression. They will see a man with no evidence to offer
beyond "the Bible says," who each time a question was presented to him during cross-
examination rambled for two minutes about anything that popped into his mind, whether it
related to the question or not, and often cut his eyes to his wall chart to cue himself on
something to say.

I will, however, give him credit where credit is due. He skillfully manipulated to his
advantage the rules that he had held out for and, with the added benefit of a biased decision-
making moderator who settled all disputes over interpretations in his favor, he was able to
play a good game of damage control. He thereby kept the audience from hearing arguments
that I had planned to present on the alleged prophecies in Isaiah 7:14 , Micah 5:2 , Jeremiah
31:15 , Deuteronomy 18:17-19 , Genesis 3:15 , and Daniel 9:24-27 , all of which are
frequently cited as amazing examples of prophecy fulfillment.

Video and audio tapes of the debate can be obtained from Thomas Gardner, P. O. Box 865,
Hurst, TX 76053, telephone (817) 282-2745. The video tapes are $25 postage paid, and the
audios are $15 plus $1.35 for postage. My name is misspelled; otherwise the quality of the
tapes is good.

Why Did Matthew Need Dead Babies?


Earle C. Beach
The one biblical story that has been most offensive to me in my adult life is Matthew's tale of
infanticide, sometimes referred to as the slaughter of the innocents. It is found towards the
end of the second chapter of the Gospel according to Matthew, and it tells how all the baby
boys in and around Bethlehem were killed by the order of the Roman governor Herod and
how Jesus was spared by fleeing with his parents to Egypt.

The first realization I had of the offensiveness of this horrible story came to me when I was in
a 24-hour newsstand. It was the Christmas shopping season, and as I was paying for my
magazine, I heard a narration of the infanticide over the commercial radio station being
played in the background on the store's P. A. system. It struck me that such an awful crime

Volume 1990 - 2002 Issue


Page 307 of 2049
Skeptical Review Edited by Farrell Till
would have to be recorded elsewhere: in the other gospels, in secular histories, or in both.
Mankind just was not that primitive, that barbaric, a mere two thousand years ago. And it
would have happened, if it were in fact true, to the people with the greatest literary tradition in
the world of that time. Even if I cannot accept the Jewish scriptures as divinely inspired, true,
or beautiful, I must admit that they are man's earliest known attempt at a comprehensive
history, and as such they are impressive.

Furthermore, this horrible offense was purported to have occurred in a place that is very
important to the Jewish people, in Bethlehem, revered in their scriptures as the birth place of
David, the greatest of their warrior-kings. Josephus, a famous first-century Jewish historian,
for example, made no reference at all to this atrocity, although he chronicled the life of Herod
in Book 18 of his Antiquities of the Jews. And, based upon Matthew's quotation from
Jeremiah, the wailing of the mothers of the dead babies was loud enough to be heard in
Ramah, which was on the other side of Jerusalem, the capital and largest city of Judah. Thus
the inhabitants of Jerusalem, including its scribes who still maintained that great historical and
literary tradition, would have heard this and would have recorded it. But there is no
corroboration from any other source of Matthew's claim of mass infanticide.

On the occasion mentioned above, I was moved to comment on this to the cashier in the store.
He apologized and offered to change the radio station. Well, this surprised me a bit. He
certainly wasn't responsible for what any commercial radio station decided to play, and I
guess I apologized back to him. On my way out, a man came out from behind the somewhat
secluded girlie-book counter, saying something to me that I didn't catch, but he didn't exactly
sound friendly. Not wanting to have to deal with any sort of porno-crazed religious apologist,
I just kept going. I guess you've got to watch what you say about religion. Here I had garnered
an unnecessary apology and interrupted someone's enjoyment of girlie books.

This first problem I had with the infanticide story is what I now call my "humanist" objection.
That story unnecessarily denigrates mankind, causing it to be more barbaric than it is. But
before long I realized I had a second major objection to it, my "rationalist" objection. In the
overall context of the New Testament gospel, this story just makes no sense. If Jesus was sent
to earth to be the ultimate sacrifice so that mankind through him might be saved, then there
should have been no need for humans, human babies at that, to be sacrificed for him. Just
who's doing the saving and who's being saved in such a story? It is plainly contradictory to the
main theme of the gospels.

With these two major problems, what could Matthew's purpose have been in telling this awful
story of killing babies? Are the two quoted passages from the Old Testament necessary in
some way to establish the truth of Christianity? Do the references to these passages establish
that scriptural prophecy (prediction) has been fulfilled?

No, they do not. All that needs to be done with each of these passages to show that they are
not predictions of Jesus is simply to read the next verses. In the passage from Jeremiah that
I've already mentioned, chapter 31, verse 15 , crying is heard in Ramah, and Rachel is
mourning her lost children. But in the next verse (the sixteenth), God is telling Rachel to stop
crying and start rejoicing; her children are not dead but are rather coming home out of
captivity. Instead of being a passage of lamentation, this is one of rejoicing. To twist not just

Volume 1990 - 2002 Issue


Page 308 of 2049
Skeptical Review Edited by Farrell Till
its meaning but its emotional tone in such a manner as Matthew did is as cynical as it is
dishonest.

The other quotation is from Hosea 11:1 in which God said, "When Israel was a child, then I
loved him, and called my son out of Egypt." Now on the face of it, this referred to the exodus
of the Israelites led by Moses. (The singular form of address for Israel in this verse was
obviously used in the collective sense, just as we would use it in the English language today.)
The statement might also have applicability to the return of the majority of the Jewish people
from Egypt after the Babylonian Diaspora, because we are told in 2 Kings 25:26 that not all of
the Jewish nation was taken into captivity in Babylon and that the rest fled to Egypt after the
Babylonian governor over them was murdered. (Second chronicles omits this, and in doing so
would seem to indicate they were all taken to Babylon.) But could it have applied to Jesus
also? Again, the next verse provides the answer, and again the answer is no. Verse 2 states
that Israel then worshiped Baal. This is something the Israelites did throughout their early
history, but nowhere in the Bible does it even suggest that Jesus worshiped Baal. For him to
have done so would have disqualified him from being the unblemished sacrifice that is so
necessary to the Christian story.

Furthermore, it would have been a very inauspicious beginning for the perfect man, the Son
of God, the king of the Jews, to go to Egypt, for it would have been a violation of an
instruction from God for Jewish kings not to go into Egypt again: "Only he [the king] shall
not... cause the people to return to Egypt... forasmuch as Yahweh hath said unto you, Ye shall
henceforth return no more that way" (Dt. 17:15-16 ). Isaiah even pronounced a woe on them
"that go down to Egypt for help" (30:2 ; 31:1 ). Even in the context of the passage Matthew
quoted, Hosea said that Israel "shall not return into the land of Egypt" (v:5 ), so how could it
be that verse one was a "prophecy" of Jesus but verse five wasn't? (Other translations render
verse five differently from the meaning given in the KJV and ASV, but Bible fundamentalists
spend a lot of time condemning the "liberalism" of modern translations, so let them wrestle
with the problem that their beloved versions pose in this passage.)

So if fulfillment of biblical prophecy is not provided in Matthew's story of infanticide, what


purpose could it have had? Well, I recently heard it proposed (at the Dobbs-Till Debate in
Portland, Texas) that this could have been some sort of "prophecy through action," that Jesus's
coming out of Egypt served as an antitype of Israel's deliverance. The trouble with this is that
the Old Testament story that Matthew retold is not that of Israel's going to and coming out of
Egypt. The "dangerous-child" aspect of this story (a familiar theme in many pagan myths)
does not apply there. Jacob and his sons went into Egypt for food, not to keep their babies
from being murdered. And when their descendants were led out by Moses, it was not to
restore a kingdom but to create one, not to recapture what had already been theirs but to have
for the first time their "land of milk and honey."

The infanticide tale more closely parallels a story about the Jews' close cousins, the Edomites,
than it does any biblical story of the Hebrews themselves. In 1 Kings 11:17-25 , the Jews
murdered all the males of Edom, except for a boy named Hadad who fled to Egypt. Hadad,
who had developed a personal relationship with the pharaoh, later came back to his own
country, ruled his people, and exacted some measure of revenge on the Jews. He was
described as an adversary raised up by God. Here is the "dangerous-child"/Messiah tale in its

Volume 1990 - 2002 Issue


Page 309 of 2049
Skeptical Review Edited by Farrell Till
most complete form, and it was told not of a Hebrew but of their enemies! In fact, the
Hebrews acted in the role taken by Herod in the retelling of this story. Herod's massacre
would make Jesus much more an antitype of Edom than of Israel, save for the accident of his
national origin.

Escape-to-Egypt stories were not even limited to nations in Asia Minor. Europeans had them
too. Bullfinch, in his Age of Fable related that the gods fled Olympus and hid in Egypt out of
fear during a war with a race of giants known as Titans. Even the other elements of this story
from Greek mythology have their counterparts in Hebrew and Christian scriptures, for
Genesis 6 tells of angels coming to earth and siring a race of giants with human women. (See
"If It Walks like a Duck..." and "Sons of God: Just the Godly Lineage of Seth?" The Skeptical
Review, Autumn 1991, pp. 2-6, and Winter 1992, pp. 5-10,16.) Presumably these giants were
all drowned in the flood.

In Book Two of The Age of Reason, Thomas Paine, the great American patriot of the
Revolutionary War, pointed out yet another problem with Matthew's infanticide story. John
the Baptist would have been killed in it, since in the Gospel according to Luke he was only a
matter of months older than Jesus (Mary's and Elizabeth's pregnancies overlapped by about
three months, Luke 1:36 ) and was born in a town in the hill country of Judea that had to be
close enough to Jerusalem for his father to perform priestly duties at the temple. Since Herod
ordered the massacre of all male children "from two years old and under" in Bethlehem "and
in all the borders thereof" (Matt. 2:16 ), this would have placed the life of the infant John in
jeopardy, but then Luke had no infanticide narrative and thus no need to have John flee. Not
only did Luke leave the infanticide and flight to Egypt out of his account, he stated that Mary
and Joseph took Jesus up directly to Galilee from Jerusalem, leaving no room for a trip to
Egypt (2:39 ).

This offensive story of Matthew's is left with no redeeming value at all. It denigrates
mankind, it irrationally contradicts the basic message of the gospel, it cynically twists the
meaning and the emotion of Old Testament scriptures, it has Jesus disobeying an instruction
from God, it is a theft from the mythology of neighboring people, and it contradicts the only
other set of biblical infancy narratives. Yet some say that it is divinely inspired.

(Earle Beach's address is 13203 Tamayo Drive, Austin, TX 78729-7403.)

Prophecy Fulfillment and Probability


Farrell Till
Bible apologists love to use probability arguments, and most readers have undoubtedly
encountered them in apologetic literature. Some situation perceived to prove either the
existence of God (life developing from nonlife) or the inspiration of the Bible (prophecy
fulfillment) is analyzed in terms of likeliness or probability. Most of these arguments, of

Volume 1990 - 2002 Issue


Page 310 of 2049
Skeptical Review Edited by Farrell Till
course, are based on purely arbitrary factors selected to make the theistic or biblical position
look good. I have yet to see one that can survive careful scrutiny.

At the debate in Portland, Texas, that Earle Beach referred to in the foregoing article, my
opponent applied probability to the prophecy-fulfillment argument. He mentioned several
times how truly amazing it was that so many Old Testament prophecies had been fulfilled
precisely and exactly in the life of Jesus of Nazareth. His premise was that over 300 such
prophecies were made and later fulfilled. At one point when he was under cross-examination,
he stated that the probability of any 50 of these prophecies being precisely fulfilled was 11
sextillion 250 quintillion to one. The figure written out would look like this:
11,250,000,000,000,000,000,000. Since the statement was made under cross-examination, I
could not respond directly to it without calling for a resumption of time, and at the moment I
was pursuing a line of questioning that I wanted to continue. In reviewing the tapes, I was
reminded that I forgot to return to this issue to show the absurdity of the statement, so I will
do that now. If Mr. Dobbs wishes to respond to my comments, we will gladly publish his
statement in the next issue. My prediction is that he won't respond. If he doesn't, I wonder
what he would say the odds are that I could make a prophecy like this and have it fulfilled.

The major problem with Mr. Dobbs's argument is that it simply assumes that prophecies were
both made and fulfilled, but he has no real evidence to support those assumptions. As I did
point out in the debate, when these fulfillment claims are studied within their original
contexts, one can easily see that most of them had nothing at all to do with the applications
that New Testament writers arbitrarily gave to them. An excellent example would be the one
that Earle Beach cited in his article. Jeremiah 31:15 is a statement that in the original context
was referring to the Jews who had been scattered abroad during the Diaspora. Jeremiah
figuratively referred to this as Rachel weeping for her children, but within the context of the
statement, there was a promise in the very next verse that these children would "come back
from the land of the enemy." Obviously, then, Jeremiah was in no sense talking about a brutal
massacre of Jewish children, so to twist the passage and give it the application that Matthew
did can only be seen as an act of desperation on the part of someone, with no real evidence on
his side, trying to prove that his man Jesus had fulfilled Jewish prophecies of the coming
Messiah. When we add to that the complete lack of reference in contemporary secular
histories to Herod's slaughter of the innocents, we have compelling reason to believe that this
event that Matthew claimed was a prophecy fulfillment never even happened.

In his article, Earle Beach mentioned that the dangerous-child myth on which this story was
obviously based is a common theme in pagan religions that antedated Christianity. Space
won't allow a review of all these myths, but the Hindu version is worth looking at, because it
is strikingly parallel to Matthew's story. According to Hindu literature, when Krishna, the
eighth incarnation of the god Vishnu, was born to the virgin Devaki, he was visited by wise
men who had been guided to him by a star. Angels also announced the birth to herdsmen in
the nearby countryside. When King Kansa heard about the miraculous birth of this child, he
sent men to "kill all the infants in the neighboring places," but a "heavenly voice" whispered
to the foster father of Krishna and warned him to take the child and flee across the Jumna
river. (In this Hindu legend, we recognize many other parallels to the infancy of Jesus other
than the dangerous-child element.) In Bible Myths and Their Parallels in Other Religions,
author T. W. Doane cited a work by Thomas Maurice, Indian Antiquities, vol. 1, pp. 112-113,

Volume 1990 - 2002 Issue


Page 311 of 2049
Skeptical Review Edited by Farrell Till
which described an "immense sculpture" in a cave-temple at Elephanta that depicts the Indian
children being slaughtered while men and women apparently representing their parents are
standing by pleading for the children (p. 167).

A study of pagan mythology would establish similar parallels in the stories of Zoroaster
(Persian), Tammuz (Babylonian), Perseus and Adonis (Greek), Horus (Egyptian), Romulus
and Remus (Roman), Gautama (the founder of Buddhism), and many others, because various
elements of the dangerous-child myth can be found in the stories of all these pagan gods and
prophets. All of these myths antedate, usually by many centuries, Matthew's account of the
massacre of the children at Bethlehem. Krishna, for example, was a Hindu savior who
allegedly lived in the sixth century B. C., so when a study of ancient world literature shows
that an unusual event like the slaughter of the innocents seemed to have happened everywhere
, reasonable people will realize that it probably happened nowhere or, at best, that it happened
only once and was thereafter plagiarized. Since the story occurs many times before Matthew's
version of it, we can only conclude that no such event happened in Bethlehem as Matthew--
and only Matthew--claimed. Just like that, then, Mr. Dobbs finds one of his fifty amazing
prophecy-fulfillments vaporizing right before his eyes.

If space permitted, I could easily establish that many of the other alleged prophecy
fulfillments in the life of Jesus have their parallels in ancient mythology. Mr. Dobbs alleged
that the miracles of Jesus had been prophesied in Isaiah 53:4-5, his crucifixion in Psalm
22:16, his resurrection in Psalm 16:10, and his ascension in Psalm 68:18. Examination of
these passages in context, however, reveal the same problem that Earle Beach and I discussed
above relative to Jeremiah 31:15. The statements are notoriously obscure and become
prophecies only through the arbitrary claims of the New Testament writers who lifted them
out of context and applied them to situations that the original writers were not referring to. So
there is no way that anyone can establish that these "prophecies" were originally intended to
be prophecies. All we have is the mere unsubstantiated word of the New Testament claimants
that they were meant to be prophecies, and that is not a good enough foundation to build a
probability argument on.

To that problem must be added the one cited above. Christianity is not the only religion to
claim that its savior performed miracles, was crucified, was resurrected from the dead, and
ascended into heaven. Hindu writings attributed all of these to Krishna. In fact, the lives of
Jesus and Krishna, as related in the respective literatures of their followers, are so strikingly
parallel that reasonable people can only conclude that the New Testament gospel writers
borrowed many of their ideas from a savior mythology that had evolved long before the first
century. In fact, virgin-born, crucified and resurrected saviors were as common as dirt in
pagan mythology, and if that does not destroy probability arguments (as they pertain to
prophecy fulfillment) in the minds of Mr. Dobbs and all others who see merit in them, then
they are obviously determined to believe the folly of the Christian myth no matter how
compelling the evidence to the contrary.

Another fallacy in this probability argument is that it completely discounts the possibility of
deliberate contrivance. At one point when I was the cross-examiner, I pressed Mr. Dobbs to
tell the audience if it would be at all possible for someone to study the Old Testament
scriptures, interpret a number of obscure passages as prophecies, and then write a biography

Volume 1990 - 2002 Issue


Page 312 of 2049
Skeptical Review Edited by Farrell Till
of a fictional character to make it appear that all of these "prophecies" had been fulfilled in his
life. The tapes will show that Dobbs desperately evaded answering the question, even though
I presented it to him three times.

In a letter to fundamentalist writer Chuck Missler, Jim Lippard very effectively addressed this
same issue in commenting on a probability argument that Missler applied to prophecy
fulfillments:

(Y)ou estimate the probability of a Messiah claimant entering Jerusalem on a donkey based
on how many candidate Messiahs have done this, assuming (without evidence) that it is less
than one in a hundred. Not only is this probably wrong, the correct question to ask is, "How
many prospective Messiahs, knowing of the existence of this prophecy, would bother taking
the trouble to fulfill it?" It's not as though entering Jerusalem on a donkey is beyond the
capacity of a human being to intentionally fulfill. I'd assess the probability as on the order of
one in one (June 8, 1993, p. 2).

My purpose in questioning Dobbs was to show that these alleged prophecy fulfillments never
even happened, that the gospel writers simply went through the Old Testament looking for
statements that they could construe as prophecies and then wrote the biographies of their
Messiah to make it appear that all of the prophecies had been wonderfully fulfilled. Lippard's
approach was to show that, even if the acts of "prophecy-fulfillment" actually did happen,
they could have been done deliberately in order to give the pretending Messiah occasion to
claim that he had indeed fulfilled the Jewish prophecies. Either way of looking at it, there
would be nothing exceptional to claim, in this case, about a man riding into Jerusalem on a
donkey. How many Jews descended from Abraham through David can we suppose rode into
Jerusalem on a donkey at one time or another? Any one of them could have claimed that he
had fulfilled this "prophecy."

With this background established, I can now demonstrate the absurdity of Dobbs's probability
argument. I have not had an expert on probability factors check the argument to verify that the
probability against the fulfillment of "any fifty" of the "more than 300" prophecies about
Jesus would be over 11 sextillion to one. For the sake of argument, I will simply assume that
the math is correct. If the figures are correct, all that Dobbs has accomplished is to show that
the odds against his being able to prove that 50 Old Testament prophecies were fulfilled in the
life of Jesus would be 11 sextillion to one.

To show why this is so, let's return to the slaughter of the innocents. The claim was made that
this event was prophesied in Jeremiah 31:15, so we will let this "prophecy" be number one on
the list of fifty. To begin proving his probability argument Dobbs would have to demonstrate
ABSOLUTELY, beyond any question, that Jeremiah intended the statement as a prophecy of
Herod's slaughter of the innocents. If there is any doubt at all that Jeremiah so intended the
statement, then no fact of prophesy utterance has been established. Since I dispute that this
was what Jeremiah meant and since there are hundreds, even thousands of others like me,
who also dispute it, this is positive proof that Dobbs has not yet established beyond even
reasonable doubt, much less absolute doubt, that Jeremiah's statement meant what it must
mean in order to be a prophecy. Let's assume, however, just for the sake of argument that
Dobbs could prove that Jeremiah did mean for the statement to be a prediction of the

Volume 1990 - 2002 Issue


Page 313 of 2049
Skeptical Review Edited by Farrell Till
slaughter of children at some time in the prophet's future. After he has done that, Dobbs must
then prove ABSOLUTELY that Herod's massacre of the children at Bethlehem can be
established as a historical fact. The complete absence of any reference to such an event by any
other New Testament writer or any secular historian contemporary to the times makes this an
impossible task for Dobbs or anyone else. However, if an event that is allegedly a prophecy
fulfillment cannot be factually established, how can any rational person contend that it was a
prophecy fulfillment?

Again, for the sake of argument, let's assume that Dobbs could somehow prove that Herod's
massacre of the innocents did in fact occur. At that point, all he would have accomplished is
to prove that ONE--just one--prophecy was fulfilled in the life of Jesus. Now he would have
to take the 49 others and go through the same process, one by one, painstakingly proving in
each case that (1) the original statement was indeed intended as a prophecy of something that
would happen in the life of the Messiah and that (2) the event prophesied did in fact happen to
Jesus. This would necessitate taking the prophecy claims about the virgin birth of Jesus, the
miracles he performed, his triumphal entry, his betrayal, his crucifixion, his treatment during
the crucifixion, his resurrection, his ascension, and forty-one other alleged prophecy
fulfillments and proving what was hypothetically proved about the slaughter of the innocents.
No reasonable person can believe that Dobbs or anyone else could possibly do this, because
the very moment that the least element of doubt arose in any one of the 49 remaining steps
(after proving prophecy fulfillment in the massacre of the innocents), the entire probability
argument would collapse like a house of cards.

So if Mr. Dobbs's math is correct in his calculation of probability, the odds against his proving
that Jesus fulfilled 50 different prophecies would be over 11 sextillion to one. So much for
probability and prophecy fulfillment!

How Likely Is It?


Farrell Till
When Pharaoh refused to release the Israelites from Egyptian bondage, the Hebrew god
Yahweh per- formed wonders unlike anything the world had ever seen. Ten plagues were
rained down on Egypt with the implication--and sometimes even direct statement--that the
Israelites were spared the horrors of the plagues. When hordes of flies swarmed over Egypt,
the land of Goshen, where the Israelites dwelt, was "set apart" so that "no swarms of flies
[would] be there" (Ex. 8:22). Likewise, when the plague of murrain decimated the flocks of
Egypt, the livestock of the Israelites was spared (9:6). When the hail came, which was more
grievous than any hail that had ever struck Egypt (9:24), none fell on the Israelites in the land
of Goshen (v:26). When darkness fell over the land, the Israelites "had light in their
dwellings" (10:23), and when the firstborn of Egypt were struck dead, the firstborn of the
Israelites were saved through the Passover ceremony.

Volume 1990 - 2002 Issue


Page 314 of 2049
Skeptical Review Edited by Farrell Till
To say the least, those Israelites witnessed some amazing miracles while Moses and Aaron
worked to gain their release from bondage, but the wonders didn't cease when Pharaoh finally
relented and gave permission for the people to leave Egypt. They saw Yahweh going before
them in "a pillar of cloud" by day and in "a pillar of fire" by night (13:21). And these were not
just occasional appearances that Yahweh made to the people, because "the pillar of cloud by
day and the pillar of fire by night departed not from before the people" as they marched out of
Egypt (v:21).

One would certainly think that the Israelites, having personally seen all of these wonders,
would have been supremely confident that the power of Yahweh was on their side and would
deliver them from all harm on their journey to the promised land, but such was not the case.
They were no sooner under way when their faith began to waver. After letting the Israelites
go, Pharaoh experienced a change of heart and gathered an army to go after them. As the
Egyptian army drew near, the people cried out in protest to Moses:

"Because there were no graves in Egypt, have you taken us away to die in the wilderness?
Why have you so dealt with us, to bring us up out of Egypt? Is this not the word that we told
you in Egypt, saying, `Let us alone that we may serve the Egyptians'? For it would have been
better for us to serve the Egyptians than that we should die in the wilder- ness" (14:11-12).
If there is such a thing as ingratitude, these bellyaching Israelites had to be the all-time
champions of it. Their God had performed unprecedented wonders to obtain their release from
slavery and was journeying with them in the pillars of cloud and smoke, but then at the first
sign of trouble--without even giving Yahweh a chance to do his stuff--they raised their voices
in rebellion.

This reaction of the Israelites, after all the signs and wonders they had personally seen, makes
the exodus story impossible to believe. If they had actually seen the plagues and been
miraculously spared their horrifying effects, and if on their journey to the Red Sea they had
actually seen their almighty god traveling with them in pillars of cloud and fire, a more
probable response to their seeing the Egyptian army overtaking them would have been
incredulous amusement. "Well, just look at those stupid Egyptians," would have been a more
likely reaction. "When are they ever going to learn!"

A critical reader, rather than swallowing stories like these just because they happen to be in
the Bible, will apply the same common sense methods of interpretation to them that he would
use in analyzing any other book. Carl Lofmark expressed the common-sense method to
biblical interpretation like this:

When you lack evidence, the only way to decide whether or not to believe something is to
ask: Is it likely? If you tell me a bird flew past my window, I will probably believe you, even
though I did not see it myself and I have no evidence. That is because such a thing is likely. I
have seen it happen before. It is more likely that a bird flew past my window, than that you
are deceiving me. But if you tell me a pig flew past my window, I will not believe you,
because my past experience tells me that such things do not happen, and so I presume that
what you reported is false. Thus, where there is no evi- dence we have to rely on our own past
experience of the sort of things that really happen (What Is the Bible? pp. 41-42).

Volume 1990 - 2002 Issue


Page 315 of 2049
Skeptical Review Edited by Farrell Till
Lofmark applied this principle to several biblical stories-- Noah's flood, the virgin birth, the
resurrection, the ascension, etc.--to demonstrate that much of what is written in the Bible
cannot pass the test of likeliness. In the absence of corroborating evidence, he concluded, the
rational reader will view such stories to be only myths and legends in the same way that
similar stories in the literature of other nations of that era must be regarded as myths and
legends.

If we apply this principle of likeliness to the events summarized above from the exodus story,
we have to conclude that they aren't very believable. These Israelites had seen Yahweh
perform many wonders just days before and (according to the story) they knew that this
almighty deity was journeying with them out of Egypt, so how likely is it that, under these
circumstances, they would have so soon forgotten all of those signs and wonders and been
reduced to the shivering cowards who cried out in fear and anguish upon seeing the Egyptian
army coming in pursuit? It just doesn't make sense to believe that they would have so reacted
with tangible evidence of Yahweh's presence fresh in their memories and even right before
their eyes in the cloud pillar overhead.

Even if we could somehow convince ourselves that the conduct of the Israelites at this point
in the exodus was believable, as we continued to read, we would immediately encounter a
long string of even more unbeliev- able events. As the Egyptian army approached, Moses
stretched out his hand, and the waters of the Red Sea parted to allow the Israelites to cross on
dry land. The water formed walls on both sides as the people marched across (14:21-22), and
when their pursuers followed them into the midst of the sea, Yahweh "looked down upon the
army of the Egyptians through the pillar of fire and cloud" and caused the wheels of their
chariots to fall off (vv:24- 25). Moses stretched out his hand again, and the walls of water
came back together and engulfed the Egyptians. "Not so much as one of them remained"
(vv:27-28).

Surely, in the entire history of mankind, no one had ever witnessed a miracle as amazing as
the one that those Israelites witnessed on that day. One would think that after seeing the
power of Yahweh wielded so deci- sively on their behalf, the people would have been loyal to
him till death, but, if we are to believe the Bible, it didn't happen that way. The last ripples in
the sea had hardly settled when the people began to bellyache again. They sang a hymn of
praise to Yahweh and turned inland to march across the Sinai, but they had traveled only three
days from the Red Sea when they began to complain because there was no water to drink
(15:22).

So we must again ask ourselves, "How likely is this?" Must we believe that the people who
had witnessed the parting of the Red Sea could so soon forget the power and majesty of their
god Yahweh that they would complain about a shortage of water? Which of them could have
possibly been so utterly dense of intellect that they would not have known that supplying
drinking water would have been next to nothing for a god who could forge a path through the
Red Sea? In fact, supplying them drinking water is exactly what Yahweh did (accord- ing to
the story). At the bitter (nonpotable) waters of Marah, Yahweh showed Moses a tree, which
Moses cast into the waters, and they "were made sweet" (vv:23-24). So, just like that, the
mighty hand of Yahweh had again supplied the people's need.

Volume 1990 - 2002 Issue


Page 316 of 2049
Skeptical Review Edited by Farrell Till
If any of these miraculous events were real, by now the people would surely have realized that
no situation that they encountered would be too difficult for their god Yahweh to take care of,
but such was not the case. On the 15th day of the second month of their journey, the "whole
congregation of the children of Israel mur- mured against Moses and against Aaron in the
wilderness" (16:2):

"Oh, that we had died by the hand of Yahweh in the land of Egypt, when we sat by the pots of
meat and when we ate bread to the full! For you have brought us out into this wilderness to
kill this whole assembly with hunger."
One wonders why Yahweh didn't at this point strike the entire mob of ingrates dead, but, of
course, he didn't. Yahweh said to Moses that he would "rain bread from heaven" so that the
people could go out each day to gather what they would need on that day but on the sixth day
gather a double portion (v:4), presumably so that they would not have to (gasp!) desecrate the
sabbath, which wasn't even consecrated until three chapters later in the narrative. Moses and
Aaron then called the people together, who looked "toward the wilderness" and beheld "the
glory of Yahweh" in the cloud. Yahweh spoke from the cloud to tell them that he would also
pro- vide the people with meat at twilight (vv:9-12).

So how did Yahweh fulfill this promise to provide bread and meat? Well, when evening fell,
quails came in and covered the camp and dew "lay all around" (v:13). When the dew lifted,
the ground was covered with a frostlike substance that the people gathered to eat. This
substance, of course, was the famous manna from heaven that Yahweh provided the people
with throughout their wilderness journey until they reached the border of the land of Canaan.
Although not directly stated, the implication is that the quails provided the people with meat.
The point is that Yahweh came to the rescue of the Israelites again as he had consistently
done before.

Rationality requires us to believe that if this much of the story really happened exactly as
recorded, the people had surely by now learned that they could depend on their god Yahweh
to deliver them from all dangers, no matter how great, and to provide them with nourishment
when their food and water supplies ran out. But such was not the case. Just one chapter later,
the people were complaining again. They journeyed from the Wilderness of Sin and camped
in Rephidim, where there was no water to drink (17:1). Did they say, "Well, not to worry;
Yahweh provided us with water before, and he will do so again"? No, just the opposite! They
"com- plained against Moses" and said, "Why is it you have brought us up out of Egypt, to
kill us and our children and our livestock with thirst" (v:3)? This time, the ever-patient
Yahweh told Moses to take some of the elders of Israel with him and stand before the rock in
Horeb. "Behold, I will stand before you there on the rock in Horeb," Yahweh said, "and you
shall strike the rock, and water will come out of it, that the people may drink" (v:6). Moses
did this, and the people had water to drink.

So surely the people knew by now that they had no need to be concerned about food and
water, because their mighty god Yahweh would provide their every need by sending manna
from heaven, coveys of quails, and water from rocks, but to so assume would be to think
rationally, and in Bible-land the rational just didn't happen. After another outbreak or two of
rebellion while the people were camped at Mount Sinai waiting for Moses to receive the law
from on high, the Israelites broke camp, left the wilderness of Sinai, and traveled for three

Volume 1990 - 2002 Issue


Page 317 of 2049
Skeptical Review Edited by Farrell Till
days with "the cloud of Yahweh over them by day" (Num. 10:12,33-34). Did they journey in
supreme confidence that their god who had delivered them from all adversity and who at that
very moment was traveling with them would continue to protect them? They did not. "(T)he
people were as murmurers, speaking evil in the ears of Yahweh" (Num. 11:1), but this time--
and it was about time--we see a different attitude on Yahweh's part. His anger was "kindled,"
and he sent "the fire of Yahweh" among them and "consumed some in the out- skirts of the
camp" (v:1). The people cried out to Moses, who "prayed unto Yahweh," and the fire was
abated (v:2).

Sensible people under these conditions would surely have said, "Whoa, wait a minute; we had
better watch what we say. One wrong word and we might cross Yahweh again." But that is
what sensible people would have done. The "mixed multitude" journeying with the Israelites
(Ex. 12:38), although having just seen "the fire of Yahweh" consume the other complainers,
"lusted exceedingly" (Num. 11:4). They, and the Israelites too, wept again and said, "Who
will give us meat to eat? We remember the fish which we ate freely in Egypt, the cucum-
bers, the melons, the leeks, the onions, and the garlic; but now our whole being is dried up;
there is nothing at all except this manna before our eyes" (vv:4-5)!

At this point, Yahweh's anger was "greatly aroused" (v:10), but not so much that he didn't
accommodate the ingrates again. He ordered Moses to bring seventy elders to the door of the
tabernacle, where Yahweh would come down and give them a message to tell the people. The
people wanted meat, so he would give them meat. In fact, he would give them meat to eat
until it came out their nostrils and became loathsome to them (vv:19-20). So evidently
Yahweh was just a little peeved after all--and who could blame him? Even an omnibe-
neficent deity can take just so much ingratitude.

A wind "went out from Yahweh" (v:31) and brought quail from the sea. Well, actually, to say
that the wind brought quail from the sea is an understatement. The quail were so numerous
that they were piled two cubits (one yard) high "all around the camp" about a day's journey
(20 miles) "on this side" and "on the other side" of the camp (v:31). If we want to talk about
likeliness, we would have a wealth of material in this little yarn, but that is another story.
Suffice it to say that Yahweh gave the people the meat they wanted. They stayed up "all that
day, all night, and all the next day" gathering the quails. Even the ones who "gathered least"
collected ten homers (v:32). A "homer" was a unit of measurement equal to about 58 dry
gallons, so those who gathered the least still had 580 dry gallons of quails. We can well
imagine that the people indeed had meat coming out of their nostrils by the time they had
consumed all these quails.

The significant point, however, is that the Israelites had again experienced a need and
Yahweh had in an extraordinary exhibition of power provided for their need. Any rational
people who had personally witnessed all of these marvelous displays of power and might
would surely by this time have been convinced that Yahweh was undeniably on their side and
would protect them from all harm. But the Bible tells us that this wasn't the case. Moses sent
twelve men (a representative from each tribe) ahead to spy out the land of Canaan, and at the
end of forty days (a familiar number) they returned with a report of fortified cities inhabited
by "men of great sta- ture" (Num. 13:32). "We were like grasshoppers in our own sight," the
spies reported, "and so we were in their sight" (v:33).

Volume 1990 - 2002 Issue


Page 318 of 2049
Skeptical Review Edited by Farrell Till
Did the people say, "Hey, no problem; we've got Yahweh on our side"? No, they didn't. "So
all the con- gregation lifted up their voices and cried," we are told, "and the people wept that
night" (14:1). The "whole congregation" made a familiar complaint to Moses and Aaron: "If
only we had died in the land of Egypt! Or only if we had died in this wilderness! Why has
Yahweh brought us to this land to fall by the sword, that our wives and children should
become victims? Would it not be better for us to return to Egypt?" They spoke about selecting
a leader who would take them back to Egypt (vv:2-4).

Joshua and Caleb were the only two to speak out on Yahweh's behalf, but the reaction of the
congregation was to threaten to "stone them with stones," even though the "glory of Yahweh
appeared in the tabernacle of meeting before all the children of Israel" (vv:6-11).

Well, Yahweh was really ticked off this time. "How long will these people reject Me?" he
said to Moses (v:11). "And how long will they not believe me, with all the signs which I have
performed among them?" Cer- tainly, that was a good question, the implications of which are
sufficient to prove to any reasonable person that the exodus story isn't accurate history. Just to
restate Yahweh's question, what people who had witnessed all the signs and wonders that
Yahweh had performed among them and who even had the "glory" of his presence right
before them at that very moment would have continued to reject him as the exodus accounts
claim that the Israelites did? It simply doesn't make good sense. If for no other reason, these
people had seen so many dis- plays of his vengeance and wrath by now that they wouldn't
have dared take the chance of crossing Yahweh as this "spy story" has them doing. To face
people of "great stature" who lived in fortified cities would have been no risk at all compared
to facing the wrath of an angry Yahweh, and they would have surely known that by now.

The continuance of the story has Yahweh engaged in dialogue with Moses. Yahweh
threatened to strike the people with "the pestilence" and then make of Moses a "nation greater
and mightier than they" (v:12). Moses, however, proceeded to point out some flaws in
Yahweh's thinking:

And Moses said to Yahweh: "Then the Egyptians will hear it, for by Your might You brought
these people up from among them, and they will tell it to the inhabitants of this land. They
have heard that You, Yahweh, are among these people; that You, Yahweh, are seen face to
face and Your cloud stands above them, and You go before them in a pillar of cloud by day
and in a pillar of fire by night. Now if You kill these people as one man, then the nations
which have heard of Your fame will speak, saying, `Because Yahweh was not able to bring
this people to the land which He swore to give them, therefore He killed them in the
wilderness.' And now, I pray, let the power of Yahweh be great, just as you have spoken,
saying, `Yahweh is longsuffering and abundant in mercy, forgiving iniquity and
transgression; but He by no means clears the guilty, visiting the iniquity of the fathers on the
children to the third and fourth generation.' Pardon the iniquity of this people, I pray,
according to the greatness of Your mercy, just as You have forgiven this people, from Egypt
even until now" (14:13-19).
The cool-headedness of Moses prevailed, and Yahweh decided to spare the people for the
moment. Rather than killing them with "the pestilence," he would allow them to live but
condemn them to wander 40 years in the wilderness until they were all dead, except for

Volume 1990 - 2002 Issue


Page 319 of 2049
Skeptical Review Edited by Farrell Till
Joshua and Caleb and the children. These would be allowed to enter the promised land at the
end of the 40 years (vv:29-35).

As for sheer silliness, this part of the exodus story has few equals in the Bible. We are asked
to believe that Moses, a mere mortal, had to point out flaws in the thinking of the omniscient
Yahweh. If Yahweh is indeed omniscient, then he would know everything and would have
been aware of all of the pros and cons of his plan to kill the Israelite nation with the
pestilence. He would not have needed Moses to tell him of the unfavor- able repercussions
that his image would suffer in the surrounding nations if he carried out his plan to extermi-
nate the Israelites. As the story was written, however, a very anthropomorphic Yahweh was
on the verge of making a grievous mistake until the more level-headed Moses intervened.
Likely? Not very.

There is no need to continue analysis of the exodus narrative, because it is just more of the
same nonsense. Adversity of some kind was encountered, the people complained and even
sometimes openly rebelled, Yahweh responded with a miracle that either supplied the needs
of the people or punished them for their rebellion, but each time nothing was learned from the
experience. Those who sided with Korah in the rebellion against Moses, for example, were
swallowed up, along with their families, when the earth opened beneath them (Num. 16:31-
33). When the water ran out again, the people complained, just as if the incidents at Marah
and Massah and Meribah (Ex. 15:23-25; 17:4-7) had never happened, and Yahweh
miraculously provided them with water again (Num. 20:2-13). Just one chapter later, when
the people "spoke against God and against Moses" in their old familiar refrain--Why have you
brought us up out of Egypt to die in the wilderness?--Yahweh sent the fiery serpents to bite
them and then had Moses forge the serpent of brass for them to look upon and be healed
(21:4- 9). And so the story continues ad infinitum and ad nauseam. Who except the hopelessly
incredulous could possibly believe all this stuff?

The literature of biblical times was written in an age of superstition, and in that respect, the
stories of the Bible are very similar to the stories written in other Near Eastern societies
during the same period. They all tell of national heroes who performed miraculous feats by
the intervention of the gods. If we reject such tales in Babylonian, Egyptian, Moabite,
Canaanite, and Assyrian literature, by what principle of logic do we determine that the same
kinds of stories in the Bible are inerrant truth? If we wouldn't believe the story of the exodus
if we encountered it on tablets of stone in Assyrian archives, does it make any sense to believe
it just because it is found in a book that has "Holy Bible" embossed on the cover? If so, why?
What is the logic behind such reasoning as this?

There is at least one story in the Bible that expresses agreement with the logical principle that
says if it isn't likely, then one should not believe it, and that is the doubting-Thomas story.
According to John's gospel account, Jesus appeared to the apostles on the night of his
resurrection when Thomas wasn't present (20:24). When the others told Thomas that they had
seen Jesus, he said, "Unless I see in His hands the print of the nails, and put my finger into the
print of the nails, and put my hand into His side, I will not believe" (v:25).

Now here is a case where a man refused to believe the incredible, even though he had heard
the testimony of close friends that the incredible was true. Since Thomas was an apostle, we

Volume 1990 - 2002 Issue


Page 320 of 2049
Skeptical Review Edited by Farrell Till
can reasonably assume that he knew the other apostles about as well as anyone could have
known them, yet Thomas did not consider that close association enough to make him believe
the incredible. He demanded more than just the mere word of close friends and associates that
the incredible had happened; he wanted proof that he could see and touch.

The same tale of a resurrected savior that the apostles told Thomas and that Thomas would
not believe has been told to us in the gospels. Two of those gospel accounts were presumably
written by apostles, the same apostles whose claim of a resurrected savior Thomas would not
believe. These facts, which Bible fundamental- ists must claim are indeed facts, should give
pause to every rational person. If Thomas would not believe an unlikely claim that he had
heard directly from the apostles--men with whom he was personally acquainted--then why
should we believe the apostles' claim that Jesus was resurrected? How likely was it that a dead
man came back to life? Does the likelihood that this would happen exceed the likelihood that
men could be mistak- en or even intentionally deceptive? The latter is far more likely than the
former, so on that basis alone, we would be as justified as Thomas was to say, "Unless I see
the evidence myself, I will not believe it."

This same principle applied to other biblical stories makes it impossible for sensible people to
believe them. The story of the exodus is insulting to human intelligence. Incidents involving
talking animals are about as unlikely as anyone can imagine. Disembodied voices speaking
from heaven, angels routinely dropping in for visits, people surviving the intense heat of fiery
furnances, the sun standing still in the sky--how many times have these things happened in
our own lifetime? How likely is it that they ever happened? God so concerned with a people's
effort to build a tower to heaven that he came down and confounded their tongues? Come on,
give me a break! It's too ridiculous to believe.

And that, in effect, applies to many biblical stories. They are unlikely--and so they are
unbelievable. In- credulous fundamentalists will of course argue that with God all things are
possible, but in the case of the exodus story even that argument breaks down. Even if we
concede that an omniscient, omnipotent deity could rain plagues down on a nation, part the
waters of a great sea, send manna from heaven, extract water from rocks, etc., etc., etc., the
context in which those signs and wonders occurred must also be likely. With nothing else
considered, the continual bellyaching and whining of the people who allegedly witnessed all
of these things make the story unbelievable, because it just isn't likely that people would have
behaved in such a way if all those wonders had actually occurred in their presence and the
"glory" of the god who had performed them was per- petually visible to them in overhead
pillars of cloud and fire. That being true, we have to doubt that any of it happened.

From the Mailbag


A lifelong friend of the editor wrote to complain that our standing offer of space for rebuttals
is hypocritical, because he has written letters that haven't been published. The following

Volume 1990 - 2002 Issue


Page 321 of 2049
Skeptical Review Edited by Farrell Till
excerpt from his latest letter (three pages), which attempted to answer every article in the
spring issue of TSR, illustrates what he considers "rebuttals":

Your article about the flood was stupid! What do you or anyone know about the "fountains of
the deep"? Do you find it strange that the "great deluge" is mentioned by so many writers of
God's word? Genesis, Joshua, Psalms, Matthew, Luke, 2 Peter, etc. The "Reader Reaction"
page was one-sided! Why not publish some of the negative letters you receive?

The "Virgin Birth Prophecy" was a farce! Matthew 1:16 : "And Jacob begat Joseph, the
husband of Mary, of whom was born Jesus, who is called Christ." "To a VIRGIN espoused to
a man whose name was JOSEPH, of the house of DAVID; and the virgin's name was Mary"
(Lk. 1:27 ). Where is the problem! In the complete genealogy as given (Lk. 3:23-28 ), there is
just a more detailed list, but no contradiction! I wonder why our scribe did not expound Isaiah
53 , also Acts 8:32-33 !

(John R. Owens, Route 2, Box 437A, Wardell, MO 63879.)

EDITOR'S NOTE: I was born and grew up in Wardell, Missouri. My parents' farm and the
Owens farm were adjoining properties, so I have known Mr. Owens all my life. He once
served as my moderator when I debated as a Bible-believing preacher. I regret that TSR has
so obviously upset him, but as rational readers can easily see, he offers no response to the
materials we publish. Like most fundamentalists, he believes that a scripture quotation should
be sufficient to settle any issue in dispute. We will gladly publish any article he submits that
takes a logical rather than the-Bible-says approach to rebutting our arguments against the
inerrancy doctrine.

Dear Farrell:

I pray you shall come to your senses before it is too late. "The fool hath said in his heart,
There is no God" (Psalm 14:1 ).

(This is a complete letter from Bobby Liddell, 4850 Saufley Road, Pensacola, FL 32526
written in "response" to our rebuttal of his article on prophecy fulfillment reprinted in the
summer issue from his paper Defender.)

Thanks for publishing Gleason L. Archer's wonderful letter declining to debate you on the
grounds that you are not a "seeker after truth" and "not really open to reason." Are these not
remarkable statements from a man notorious for inventing (or at least promoting) falsehoods
and fallacies with which to defend the Bible? In addition to Archer's insufferable arrogance,
note the characteristic "thinking in tongues." He asserts that "you have nothing to offer your
public but disillusionment and despair...." That is nonsense, but suppose it were not? Would

Volume 1990 - 2002 Issue


Page 322 of 2049
Skeptical Review Edited by Farrell Till
that constitute a refutation? Is a message that is genuinely disillusioning and depressing
necessarily false? Think about that, for it is the key. Secretly, the Archers of the world believe
they control reality, that what they want to believe is necessarily true. They rarely say it in so
many words, but they continually betray themselves to the careful listener or reader. ("Well, if
you really want to believe we came from apes.... I'd rather believe....") They equate their
hopes, fears, and prejudices with absolute truth and blindly attack objective reality when it
rears its (to them) ugly head.

Please consider the preceding paragraph for publication. More and more, I've come to believe
that my point therein about controlling reality is the key to understanding the fundamentalist
mind.

(Robert J. Schadewald, 13204 Parkwood Drive, Burnsville, MN 55337. Until recently, Mr.
Schadewald was the president of The National Center for Science Education.)

It was a pleasure to meet in Portland, where I was able to attend the last sessions of your
debate with Mr. Dobbs. Of the debate itself, I can't say it was much of a pleasure; it was about
what I expected. Your opponent used just about every dodge in the book, and the audience
was preconditioned to preaching and could not distinguish such from a formal debate, as
evidenced by the outbursts of "amens." Well, I guess my purpose in going was to see if there
was any difference from the perspective of being there as opposed to reading a transcript of
such a debate later. It came off just about the same to me.

It was the first exposure for this recovered Baptist to your old denomination the "Church of
Christ," and it had a few surprises. In particular, a book in their bookstore was subtitled a
"Debate between a Christian and a Lutheran." And here I'd thought Lutherans counted as
Christians too. Your debate opponent must have thought so when he quoted the World
Almanac's figures on total Christian population.

This led me to research the denomination a bit. My biggest encyclopedia was not much help.
It told of Campbell's Presbyterian and Baptist background and of his and his father's efforts to
unite Christians. This hardly seems consistent with the "we're the only true Christians"
attitude I experienced there.

A better resource turned out to be one that your opponent used, the World Almanac.
However, its entries in the categories of Doctrine and Authority were rather surprising in light
of the experience in Portland. These are respectively stated: "Simple New Testament faith;
avoids any elaboration not firmly based on scripture" and "Where the Scriptures speak, we
speak; where the Scriptures are silent, we are silent."

Perhaps they are talking about another "Church of Christ"? It seems to me that the whole
four-day seminar was nothing more than a series of such elaborations on remarkably unfirm
passages. Seeing Rome in Daniel 2 is extremely far-fetched; equating Nazarene with the
insults to the "suffering servant" of Second Isaiah is simply astounding. From what little I saw

Volume 1990 - 2002 Issue


Page 323 of 2049
Skeptical Review Edited by Farrell Till
of the other lectures and of the preprinted book, this appears to be the modus operandi for the
entire event.

(From Earle C. Beach, whose address is printed at the end of his article on page 8 of this
issue.)

EDITOR'S NOTE: Although Mr. Beach seemed unable to believe what he discovered from
browsing through the books on display at the debate site, members of the Church of Christ do
believe that they are the only true Christians. Perhaps Mr. Beach noticed a popular book by
Thomas B. Warren on display: The Bible Only Makes Christians Only and the Only
Christians. Those "only" Christians, of course, are in the Church of Christ.

"We speak where the Bible speaks and are silent where the Bible is silent" is indeed the
rallying cry of the Church of Christ, but Mr. Beach heard me chiding my opponent several
times on this point for his resort to speaking just about anything that would provide him with
a semblance of a response to nonexistent, fabricated prophecies that I challenged him to give
the book, chapter, and verse for in the Old Testament. Of course, he never did, because they
simply do not exist.

I agree that Romans 3:5-6 begs the question, but Paul makes three assumptions unsupported
by evidence, not just one: (1) the world will be judged; (2) the world will be judged justly;
and (3) God, and no one else, will do the judging. When people beg a question with a
package, I like to look at the pieces.

Your analysis of Romans 7:1-6 , based on Hyam Maccoby, is beautiful! No wonder


Christians' brains get scrambled if they study the Bible too much!

(William Lindley, Associate Editor, Truth Seeker, P. O. Box 2872, San Diego, CA 92112-
2872.)

Mr. Till:

I was particularly impressed with your article "There's a Living in It" (Spring 1992), which
identifies the economic motive for keeping the Bible and the local church together. Another
motive is what I call "credibility." It is difficult to have to retract some of the things adhered
to in the beginning of a ministerial career and say, "I was wrong." It reminds me of the story
of the preacher who had in his sermon margin "weak point here--yell like hell." Look at the
cost involved--loss of face with the congregation who pays the salary, loss of face with
parents who are proud of their preacher son and are now ashamed of his rebellion, loss of face
with professors who taught him, and loss of face with his peers in surrounding churches. It is
a heavy cost, and I am sure that you have experienced some or all of it.

Volume 1990 - 2002 Issue


Page 324 of 2049
Skeptical Review Edited by Farrell Till
But my real motivation with this letter is to offer you some words of encouragement. We both
know that even though there is an outward, negative reaction to your newsletter from the
inerrantists, much of what you say will be retained in the archives of their minds for some
later reflection. I can still remember some of the doubts express by my laymen some 25 years
ago that caused me later to say, "Right on!" So it may take some time on the part of some
inerrantists, but the seed is planted for future harvesting.

I know that sometimes you must feel as if you are beating your head against a wall. Your
weapon of logic is frowned upon by inerrantists. God's reasoning and faith are above logic,
they say. So it is difficult to communicate with them on common ground. Somehow, they
feel, this is superior to logic. So you have a tough battle to fight. But sooner or later, they will
tire of what I call "do-si-do-ing" with words. I can still hear the pastor of my youth
encouraging me not to think so deeply about the text of the Bible; rather I should have the
faith of a little child.

I have personally concluded that most of the clergy I know are either dishonest or dumb.
During my ministerial career, I was "dumb." And for a while, I was dishonest--until I was
able to resolve the economic problem and get on with an honest living.

(Rene Corcoran, ex-Lutheran minister, 1489 West Decatur Street, Decatur, IL 62522.)

Debate on Bible Morality


In the next issue of The Skeptical Review, editor Farrell Till and Lindell Mitchell, a Church-
of-Christ preacher from Livingston, Texas, will begin a written debate. Till will affirm that
the Israelite destruction of the Amalekite nation (as recorded in the Bible) constituted moral
atrocity; Mitchell will deny that it was a moral atrocity.

As indicated in Clarence Lavender's article "Was It Morally Right for God to Order the
Killing of the Canaanites?" (TSR, Winter 1993, pp. 6-7), biblical inerrantists argue that the
massacre of civilian populations, including even children and babies, in time of war was
morally right, if God so decreed it. They must take this position, because the Bible clearly
attributes such actions to the Hebrew god Yahweh, who inerrantists believe was the
omniscient, omnipotent deity who inspired the writing of the Bible. To say that Yahweh erred
morally would be the death knell for the inerrancy doctrine, so inerrantists must defend baby
killing while usually condemning abortion.

Lindell Mitchell refused to defend the massacre of the Amalekites, but he did agree to deny
that it was an atrocity. The debate will continue through the Spring 1994 issue.

Volume 1990 - 2002 Issue


Page 325 of 2049
Skeptical Review Edited by Farrell Till

Study Aids
In addition to The Skeptical Review, Skepticism, Inc., publishes other materials that might be
useful to those wishing to hone their skills on the subject of Bible inerrancy. The following
booklets are available at $2.50 each, postage paid.

The Laws-Till Debate, a 56-page unfinished debate with James Laws, Jr., a professor of
apologetics at Tennessee Bible College. Although Laws challenged, he quit after only three
manuscript exchanges and has since refused to accept mail from Till. Correspondence is
reproduced in the booklet. Jackson-Till Debate, a 64-page debate on the issue of Bible
inerrancy with a Church-of-Christ preacher. Prophecies: Imaginary and Unfulfilled, an in-
depth examination of the most commonly claimed examples of prophecy fulfillment. Recently
revised to expand its scope.

BACK ISSUES of The Skeptical Review from winter 1990 to the current edition (16 in all) are
available at $1 per copy.

Saul & The Witch of Endor


The myth of Bible inerrancy can be easily exposed by two or more writers. Quite often, the
telling of the same story a second or third time resulted in noticeable discrepancies. An
example of glaring inconsistency can be found in the two accounts of the death of Saul, the
first king of Israel, as told by the writers of 2 Samuel and 1 Chronicles . Both writers claimed
that Yahweh engineered Saul's death because of his disobedience, but the two versions of his
disobedience differ significantly. This is how the Chronicle writer summarized Saul's
ignominious death:

So Saul died for his unfaithfulness which he had committed against Yahweh, because he did
not keep the word of Yahweh, and also because he consulted a medium for guidance. But he
did not inquire of Yahweh; therefore He killed him, and turned the kingdom over to David the
son of Jesse (1 Chron. 10:13-15 ).

The statement is too direct to be misunderstood. Saul consulted a medium for guidance but
did not inquire of Yahweh; therefore, Yahweh killed him. The Chronicle writer did not record
Saul's experience with the medium who is commonly called "the Witch of Endor," but the
event was recorded in 1 Samuel 28 in a way that clearly contradicts the statement in 1
Chronicles :

Then the Philistines gathered together and came and encamped at Shunem. So Saul gathered
all Israel together, and they encamped at Gilboa. When Saul saw the army of the Philistines,
he was afraid, and his heart trembled greatly. And when Saul inquired of Yahweh, Yahweh
did not answer him, either by dreams or by Urim or by the prophets.

Volume 1990 - 2002 Issue


Page 326 of 2049
Skeptical Review Edited by Farrell Till
Then Saul said to his servants, "Find me a woman who is a medium, that I may go to her and
inquire of her." And his servants said to him, "In fact, there is a woman who is a medium at
En Dor" (vv:4-7 ). The rest of the chapter records the actual seance in which the witch at
Endor conjured up the spirit of Samuel the prophet, who told Saul that the next day his army
would be defeated by the Philistines and that he and his sons would be killed.

The significant thing to notice in this story is not the absurdity of the writer's apparent belief
that a medium actually conducted a seance in which she conjured up the spirit of a dead man
but his obvious disagreement with the Chronicle writer's version of Saul's death. The latter
said that Saul consulted a medium but did not inquire of Yahweh, and so Yahweh killed him.
The writer of 1 Samuel , however, said that Saul did not consult the medium until after he had
inquired of Yahweh and had received no answer. Both versions of this story cannot both be
right. The discrepancy is obvious, yet in the face of such glaring inconsistency as this,
inerrantists will stubbornly argue that the Bible is a perfectly harmonious book from cover to
cover. Go figure.

Articles Solicited
With the change in our format, we will be publishing more materials. If you have an idea for
an article, we would be interested in reading it. All articles should address specific points of
inconsistency, discrepancy, contradiction, or logical absurdity that illustrate fallacies in the
Bible inerrancy doctrine. Our style is thoroughness, so we will expect in-depth discussion of
major points. Unsupported assertions and generalizations that characterize most
fundamentalist papers should be avoided. We will also consider pro-inerrancy articles with
the understanding that if we accept them, they will be published simultaneously with our
rebuttals.

Volume 1990 - 2002 Issue


Page 327 of 2049
Skeptical Review Edited by Farrell Till

The Skeptical Review


Volume Five - 1994
Farrell Till, editor

• Number 1 Volume Five, Issue One


• Number 2 Volume Five, Issue Two
• Number 3 Volume Five, Issue Three
• Number 4 Volume Five, Issue Four

Volume 1990 - 2002 Issue


Page 328 of 2049
Skeptical Review Edited by Farrell Till

The Skeptical Review


Volume Five, Number One
January/February/March 1994
Farrell Till, editor

• No Morality Without the Bible?


Bibles? Bibles? We don't need no stinkin' Bibles! Humans are more than capable of
determining right and wrong without the aid of holy texts and "words of God".

• Was the Amalekite Massacre a Moral Atrocity?


The opening salvo in the debate over the morality of the Amalekite massacre is a
bloody one, as Till paints a picture of a gruesome, God-order holocaust.

• The Amalekite Destruction a Moral Atrocity?


According to Lindell Mitchell, killing babies is wrong, unless God orders it, in which
case it's just fine. (Not a man I'd want for a babysitter.)

• And God Came to Abimelech in a Dream


Does God actually communicate with humans via divine revelation? Only in your
dreams.

• Did Paul's Men Hear a Voice?


Dan Barker discusses a contradiction between Luke and Paul's account of Paul's
conversion to Christianity.

• The Hovind-Till Debate


A debate over the Genesis flood finds the inerrantist position sinking swiftly.

• Polytheism in Genesis: Baal and Ashtoreth vs. Yahweh


God wasn't always alone up there in the heavens, as an examination of Genesis
reveals.

Volume 1990 - 2002 Issue


Page 329 of 2049
Skeptical Review Edited by Farrell Till
• The Dobbs-Till Debate
A review of the Dobbs-Till debate on biblical prophecy by a Christian who attended
the debate. Till includes his comments on the review.

• To Those Who Worship the Bible-Idol


Dave Matson rebukes inerrantists for allowing their blind enthusiasm for the Bible to
cloud their moral judgement.

• From the Mailbag

• Recommended Reading

• Losing Faith in Faith


A brief discussion of Dan Barker and his book, Losing Faith In Faith.

• Geisler-Till Debate

• Video Library

• A New Column
Farrell Till begins writing a new column for the Secular Humanist Bulletin. Called,
"The Twilight Zone," it will discuss inerrantist claims which are so outrageous, only a
"Twilight Zone" mentality could believe them.

• Article Submissions

No Morality Without the Bible?


Of all the arguments that fundamentalists resort to in their defense of the Bible, none is more
ridiculous than their claim that the Bible is necessary for people to know how to live moral
lives. They arrive at this conclusion through a series of assumptions. Their first assumption is
that God exists, and onto this assumption, they pile another one: morality (and they even
make it an absolute morality) emanates from the nature of God. Then, of course, they assume
that their God, in verbally inspiring the Bible, revealed absolute morality to mankind. Hence,
man must rely on the Bible to know what is moral and immoral. They envision life without
the Bible as a moral chaos reminiscent of ancient Israel before the time of its kings when
"everyone did what was right in his own eyes" (Judges 21:25).

The whole superstructure of this argument is built upon another assumption that is incredibly
cynical on the part of a group that delights in condemning the pessimism of philosophies that
question the existence of God. This assumption is that man is incapable of making moral
decisions without divine guidance. In other words, man must have God's help or else he just
can't determine for sure what is right and what is wrong.

Volume 1990 - 2002 Issue


Page 330 of 2049
Skeptical Review Edited by Farrell Till
Were it not for the seriousness of fundamentalist attempts to impose this belief on society in
general, it would be too ridiculous to deserve comment. We have used human intelligence to
cure diseases, split the atom, and invent a technology that has us reaching for the stars, yet
Christian fundamentalists would have us believe that we are too stupid to discover that lying,
stealing, and killing are harmful enough to the general welfare to be considered morally
wrong. That view of life is about as pessimistic as any that can be imagined, infinitely more
pessimistic than the mental action of a skeptic who questions the existence of an afterlife for
which he can see no verifiable evidence.

This foundation belief of Bible fundamentalism is of course erroneous. It is even contradicted


by the Bible itself. In Romans 2:14, the Apostle Paul said that the Gentiles, who had not
received the law [of Moses] or, in other words, a revelation from God, had nevertheless
sometimes done "by nature the things of the law" and were therefore "a law unto themselves."
If this doesn't mean that Paul believed that the Gentiles who had no divine revelation had
discovered morality on their own, then pray tell what does it mean? So even if the existence
of the biblical god could undeniably be proven, how could bibliolaters, in the face of this
statement from their much revered apostle to the Gentiles, justify their claim that man must
have direct guidance from God in order to live morally?

The fact is that no one can prove the existence of God. Volumes have been written on the
subject, but no theist has yet advanced an argument for God's existence that has not been
adequately answered. Anyone who doubts this should read the information available on the
subject, and a good place to begin would be with George H. Smith's Atheism: the Case
Against God. In this book, one will find logical refutations of all the major theistic arguments.

What this means is that the fundamentalist claim that there can be no morality without a god
to reveal it to us is just an empty shell. It begins with an unprovable assumption and ends with
a conclusion that even the Bible contradicts. What kind of argument is that?

The fallacy of the argument is obvious from its flagrant appeal to wishful thinking. It is
certainly appealing to think that we will live in another world after we die in this one, and so
wishful thinkers spend their lives believing in religions that offer them the hope of gods and
saviors who promise them eternal life in a great beyond. Few of these wishful thinkers ever
bother to subject their otherworldly beliefs to rational examination. They want it, so they
assume that they will get it just on the basis of their wanting it. Nothing could be more
irrational than belief based on a premise no more substantial than this, yet this is exactly how
many theists reason. "I want it, and so I know that I will get it."

If there is no God, fundamentalists are fond of saying, then there can be no standard of
objective or absolute morality. Well, so what? What kind of argument is that? If there isn't,
then there just isn't. What the fundamental- ists are really saying is that it would certainly be
nice if everything on the subject of morality was already decided for us and neatly laid out in
categories of black and white. This is right, and this is wrong, period, end of the discus- sion.
But if it isn't that way, then it just isn't that way, and no amount of wishful thinking or praying
or hoping will ever change the fact that it isn't that way. We (mankind) are just in the world
on our own and will have to get by the best that we can.

Volume 1990 - 2002 Issue


Page 331 of 2049
Skeptical Review Edited by Farrell Till
The thought of that terrifies most theists, but it shouldn't. God wasn't much help to us in
discovering how to cure or prevent smallpox, diphtheria, typhoid, whooping cough, polio,
measles, and dozens of other diseases. We had to do it on our own. God wasn't much help to
us in making the scientific discoveries that led to the technology that now makes life so
comfortable for us. We had to do it on our own. So if we did all these things without God,
surely we can make the moral discoveries that are necessary for society to function in an
orderly, beneficial way.

To the fundamentalists, of course, this is all outrageous heresy. The Bible is the inspired,
inerrant word of God. It just is, and no amount of rational argumentation will remove them
from their fantasy world in which everything is either black or white. There is one thing,
however, that they cannot do. They cannot open their Bibles and demonstrate just how anyone
can know what absolute morality is. They will say that the Bible provides us with a guide to
absolute morality, but they can't show us exactly what absolute morality is.

Is it, for example, morally right for blood to be transfused from one person to another? Most
religions permit it, but the Jehovah's Witnesses argue that biblical principles properly
understood condemn it. Who is right? When the Bible was being written, the technology for
transfusing blood didn't exist, so the Bible did not directly address this problem. The same is
true of numerous other technologies now available to us. The transplantation of body organs
(including even cross-species transplants), artificial insemination, in vitro fertilization,
surrogate motherhood, genetic mapping, gene splicing--these are all technologies that were
developed after the Bible was written, so what is the "correct"" moral position to take on these
issues? Through processes of in vitro fertilization and embryo transplantations, a woman in
South Dakota gave birth to her own grandchildren. Was it morally right for her to do this?
What does the Bible say? Well, of course, the Bible doesn't say anything about this or any of
the other technological procedures mentioned above. If we asked a hundred theologians to
take their Bibles and resolve the moral dilemmas posed by these technologies, we would find
ourselves hopelessly trapped in a maze of confusion when all of their answers were in.

Last summer, when the story about the Lakeberg twins first appeared in the newspapers, the
article was clipped and mailed to several fundamentalist preachers known to believe in
absolute morality. An accompanying letter asked them to explain what the Bible had to say
about the dilemma that the parents of those twins were facing. The twins were joined at the
chest and shared a common heart. Surgery would mean that one of the twins would have to
die, and subsequently this was the decision that the parents made. The absolute moralists who
received that letter were asked to state what their god of absolute morality has revealed to us
in this matter.

Not a one of these preachers has yet answered that letter. Their silence shouts the
inconsistency of their position. The Bible gives us a guide to absolute morality, so they say,
yet they cannot tell us what absolute morality has to say about the difficult moral dilemmas
that we must confront in our modern society.

Elsewhere in this issue, a debate on biblical morality begins. Before it is over, maybe Lindell
Mitchell, the spokesman for the fundamentalist position, will try to explain to us how the
Bible can be an absolute moral guide in problems that didn't even exist in biblical times. If he

Volume 1990 - 2002 Issue


Page 332 of 2049
Skeptical Review Edited by Farrell Till
doesn't attempt to explain it, some of us just may suspect that he isn't nearly as sure of his
position as he would like us to believe.

Was the Amalekite Massacre a Moral


Atrocity?
Farrell Till
The story of the Israelite massacre of the Amalekite nation is recorded in 1 Samuel 15 . The
facts of the case, as claimed in this chapter, are these: Yahweh sent the prophet Samuel to
command Saul, the first king of Israel, to "go and smite Amalek" and to " utterly destroy all
that they have and spare them not" (v:3 ); the command explicitly stated that Saul was to kill
"both man and woman, infant and suckling, ox and sheep, camel and ass" (same verse).
According to the story, Saul took "two hundred thousand footmen and ten thousand men of
Judah" (v:4 ) against the Amalekites and "utterly destroyed all the people with the edge of the
sword" (v:7 ), except for Agag their king, whom he kept alive to take back as a prisoner. This
act of mercy, of course, was a clear violation of Yahweh's instructions, which were to kill
everyone and spare no one. In addition to this act of disobedience, Saul also kept alive "the
best of the sheep, and of the oxen, and of the fatlings, and the lambs, and all that was good,
and would not utterly destroy them" (v:9 ).

Saul's disobedience irked Yahweh, whose word came to Samuel (as Yahweh's word had a
habit of doing in those days) and said, "It repenteth me that I have set up Saul to be king, for
he is turned back from following me, and hath not performed my commandments" (v:10 ).
Apparently when Yahweh said kill everyone and everything and spare no one and nothing, he
meant kill everyone and everything and spare no one and nothing. Samuel left the next
morning and met Saul returning home from battle. Samuel sharply reprimanded Saul for not
executing [no pun intended] Yahweh's word to the letter and informed Saul that Yahweh had
rejected him from being king (vv:17-23 ). Samuel ordered that Agag, the Amalekite king, be
brought out, and then in the presence of Yahweh, Samuel hacked Agag to pieces with a sword
(vv:32-33 ), presumably to show the people that when Yahweh said kill everyone, he meant
kill everyone.

What happened on that day, if indeed it did happen, must by all standards of decency and
morality--except for biblical standards, of course--be considered a moral atrocity. After all,
this is a case where an attacking army went beyond the killing of the soldiers they fought
against to the butchering of women and children and even infants still nursing their mothers'
breasts. Please notice that Yahweh's order was to slay even "infant and suckling" (v:3); no
one--nothing--was to be spared. As we will soon note, it was Yahwistic vengeance at its
bloodiest.

Volume 1990 - 2002 Issue


Page 333 of 2049
Skeptical Review Edited by Farrell Till
My position, which is the position that any humanitarian would take in the matter, is that such
an event as this must be considered a moral atrocity. The American Heritage Dictionary
defines moral as that which is "concerned with the judgment of the goodness or badness of
human action and character; pertaining to the discernment of good and evil." It defines
atrocity as an "atrocious condition, quality, or behavior; monstrousness; vileness." Atrocious
is defined as "extremely evil or cruel; monstrous; exceptionally bad." What degree of
judgment is required for one to determine that killing defenseless babies is "bad"? In the
following article, Lindell Mitchell will deny that the massacre of the Amalekites was a moral
atrocity, so I wonder if Mr. Mitchell will tell us that killing children and babies in his
"judgment" is not "monstrous" or "exceptionally bad," that it can, in fact, be an act of
goodness? Indeed he will, because he is a Bible inerrantist and can take no other position.

The killing of just one Amalekite woman or child or infant, solely because of her, his, or its
nationality, would have constituted moral atrocity by any civilized standard of morality, but to
put into proper perspective the extent of this massacre, I want first to establish a concrete
image of what probably occurred on that occasion. If the story happened as recorded, then by
necessity, hundreds of women, children, and babies were killed by Israelite soldiers. How do I
know this? Well, even though Saul "utterly destroyed" the Amalekites, except for Agag,
whom Samuel quickly took care of, the Amalekites were, inexplicably, still around just a few
chapters later where the guerrilla escapades of David were chronicled. On one occasion,
David raided the Amalekites and "saved neither man nor woman alive" (1 Sam. 27:8-9 ), yet
these twice-utterly-destroyed Amalekites somehow made a speedy comeback, raided David's
camp at Ziklag (30:1 ), and took the women captive. (Apparently, Amalekites were more
humanitarian than the Yahwistic Hebrews.) David pursued the Amalekites, and when he
reached their encampment, he "smote them from the twilight even unto the evening of the
next day" (v:17 ). There "escaped not a man of them, save four hundred young men, who rode
upon camels and fled" (v:17 ). Now if David smote these twice-utterly-destroyed Amalekites
from twilight until evening of the next day, there must have been a lot of them when the battle
started. The extent of their numbers would also be indicated by the way it was said that "there
escaped not a man of them, save four hundred young men." In other words, the writer must
have intended for us to understand that the "four hundred young men" who escaped on camels
represented just a small fraction of the total Amalekite population.

If 400 young men escaped on camels, we can reasonably assume that many other young men
were killed when David was smiting the Amalekites "from the twilight even unto the evening
of the next day." So if that many young Amalekite men were alive at the time of this
encounter with David's guerrillas, we can imagine how many of them there must have been
before Saul and Samuel utterly destroyed them. Be that as it may, we can assume that if there
were at least 400 young Amalekite men, there were probably that many young women too.
Young men able to escape on camels were probably not children, and certainly they were not
infants. A steadiness of tribal growth from year to year, then, would suggest a population of at
least several hundred children and infants at the time of David's battle with the Amalekites, so
before the Amalekites were "utterly destroyed" by Saul and Samuel, there must have been
thousands of children and infants. The point is that, if there is any degree of accuracy at all in
the Bible, inerrantists will have to concede that the Israelite massacre of the Amalekite nation
in 1 Samuel 15 entailed the killing of thousands of women, children, and infants. Mr. Mitchell
wants us to believe that this massacre was morally proper.

Volume 1990 - 2002 Issue


Page 334 of 2049
Skeptical Review Edited by Farrell Till
To put the incident--if indeed the massacre of thousands of women, children, and infants can
properly be called just an "incident"--into a clear perspective, let's try to visualize in specific,
concrete terms what had to have happened on that day, not just once, not just twice, but
hundreds of times. The instruments of warfare at that time were swords, spears, and bows and
arrows, so we can only assume that these were the weapons used to "slay both man and
woman, infant and suckling" (1 Sam. 15:3 ). David, whom the Bible describes as a man "after
Yahweh's own heart" (1 Sam. 13:14 ), made the survivors of a battle with the Moabites lie on
the ground so that he could measure them off in three "lines." Two of the lines were killed,
and the third was kept alive to become "servants of David" (2 Sam. 8:2 ). As brutal as this
was, it was comparatively civilized in terms of the treatment that the Israelite king Menahem
accorded his captives. He "ripped up" the pregnant women captives after a military campaign
against Tiphsah (2 Kings 15:16 ). The point is that these were barbaric times, so we can
reasonably assume that what contemporaries did with civilian captives was probably what the
Israelites did with the Amalekites.

I have often wondered if inerrantists defend biblical events like the Amalekite massacre
because they don't take the time to think in specific terms of everything that such an event
would necessarily have encompassed. I don't know; I'm willing to give them the benefit of the
doubt. Do men like Mr. Mitchell see thousands of Amalekite women, children, and infants as
merely an abstract concept or words on paper rather than specific, living and breathing
individuals with hopes, aspirations, and a desire for life as real as yours and mine... and Mr.
Mitchell's? Does he think that Amalekite children and babies somehow didn't bleed when
spears were thrust through them or feel pain when they were hacked with swords? Recalling
again that the weapons of the time were swords and spears, we can reasonably assume that
these were the instruments of slaughter that the Israelites used to kill the Amalekite women,
children, and infants. So, if the story happened as recorded, Mr. Mitchell has to know that not
just once, not just twice, but hundreds of times, a specific Israelite soldier, wielding a specific
sword or spear, thrust his weapon through a specific Amalekite infant. We can hardly imagine
that all of these children and infants were killed simultaneously. The massacre was surely a
sequential affair. Some were killed first, then others, then others, then others, etc., as the
Israelite army passed through the civilian population. We can imagine, then, the terror of
screaming children who, seeing what was happening as the soldiers advanced toward them,
knew what was soon going to happen to them. How many of them were thrust through from
the back as they tried to flee for their lives? We can never know, but from reports of what
happened at places like My Lai, we can be reasonably sure that it did happen. So these
Amalekite children were not just abstract concepts or words on paper; they were real
individuals, just as real as Mr. Mitchell's children (if he has any). They had individual names
and personalities, just as children today have. They were the Jasons and Brandons and the
Jennifers and Lisas of their time and place--and they were all killed, individually, by specific
Israelite soldiers, presumably acting on orders from the God of heaven. We can imagine too
Amalekite mothers futilely trying to shield their babies from Israelite swords and spears with
their own bodies, because we know from many documented cases that this routinely happens
when mothers sense threats to their babies.

If Mr. Mitchell's inerrant "word of God" is truly inerrant, the scenario I have just described is
exactly what happened on that day. So now I am going to state my position as emphatically as
I know how. If such a scenario as this is not morally wrong, then nothing is! I will repeat it.

Volume 1990 - 2002 Issue


Page 335 of 2049
Skeptical Review Edited by Farrell Till
If such a scenario as this is not morally wrong, then nothing is! And I can't wait to put Mr.
Mitchell's defense of this atrocity into print, because I want the world to see the extremes to
which blind allegiance to the absurd doctrine of Bible inerrancy will lead one. It requires its
believers to defend the killing of children and babies for no other reason than the
circumstance of national birth, and such a belief is repugnant to everything that modern
civilization stands for.

To cut right to the heart of the matter, I am going to ask Mr. Mitchell a question that will let
us know just how sincere his belief in the moral rightness of the Amalekite massacre really is.
I will state it in the form of a true or false question:

If I, Lindell Mitchell, had been born an Israelite in the time of King Saul, I would have
willingly and gladly participated in the Amalekite campaign by killing women, pregnant
women, children, infants, the elderly, the sick, and the feeble.

I sincerely hope that he will answer false, but I know that he won't. To do that would raise
questions highly detrimental to his inerrancy position. Why would he not willingly do the will
of God? Why would he not gladly do the will of God? In all probability, he will somehow
evade the question, but if he does answer it, the only answer he can give, without surrendering
his position, is true. So an honest and forthright answer from him will put this issue in a
crystal clear perspective for our readers. It will enable them to see that belief in Bible
inerrancy will require them to defend the killing of babies.

I know that Mr. Mitchell is not completely without compassion for children, because he
usually plasters his letters to me with "pro-life" stickers that say such things as, "It's a child,
not a choice." Although I don't agree with his position on the issue of abortion, I do recognize
that his position implies a concern for children. So now I will ask him to explain something
that puzzles me. If abortion, even in the very first days of pregnancy is morally wrong (as he
seems to believe), then what made morally right the abortions that the Israelite soldiers
performed on Amalekite women that day? We can certainly assume that in a population of
thousands (as already established) there were many pregnant women, so when the Israelite
soldiers utterly destroyed "man and woman, infant and suckling," they necessarily terminated
many pregnancies. Why was that morally right if abortion is wrong, period? What had those
unborn Amalekite babies done to deserve death? For that matter, what had the babies already
born done to deserve death? Mr. Mitchell has a lot of explaining to do.

Now inerrantists like Mr. Mitchell would have us believe that the massacre was done upon
direct orders from God, so somehow that made everything all right. You see, God can do no
wrong; God knows things we cannot know, so he had special omniscient insights into the
matter that enabled him to understand the necessity of the massacre, no matter how shocking
it may seem to our pathetically finite minds. In the sweet by and by, however, we will
understand God's ways, which are higher than our ways, so then we will know why God had
to do this, and it will make perfectly good sense to us. I have debated this subject with Bible
fundamentalists enough to know that we can expect to hear some kind of doublethink like this
as Mr. Mitchell seeks to explain it all to us. Meanwhile, as we wait for Jesus to pass out
answers to us in the great beyond, we can take solace from knowing that God actually did the
Amalekite children and infants a big favor by commanding Saul to massacre them. Had they

Volume 1990 - 2002 Issue


Page 336 of 2049
Skeptical Review Edited by Farrell Till
been permitted to live, they would have simply grown up to be wicked like their parents. By
having them killed as children living in a state of innocence, he assured them of a place in
heaven, so now they won't have to fry in hell. Yes, expect Mr. Mitchell to say such stuff as
this too. If you don't believe that anyone could possibly advance an argument as asinine as
this, see page 6 of Clarence Lavender's article ("Was It Morally Right for God to Order the
Killing of the Canaanites?" TSR, Winter 1993), where he actually said that the Israelite
massacre of Canaanite children was the "best thing that could have happened." I don't make
this stuff up, readers. I just encounter it in my engagements with fundamentalists and pass it
along to you. So you can expect to see some kind of rebuttal from Mr. Mitchell that is based
on the premise that God did it, so it had to be all right. He will expect us to believe that,
somehow, a deity whom the Bible describes as the "Father of mercies and God of all comfort"
(1 Cor. 1:3 ), "merciful and gracious" (Ex. 34:6 ), and who is "abundant in lovingkindness"
(Ibid.), and who is praised with various other descriptions intended to convey qualities of
goodness and righteousness, was simply manifesting his supreme moral excellence when he
ordered the slaughter of the Amalekites. I have yet to see a logically coherent defense based
on this premise, so I don't expect one from Mr. Mitchell either. Whatever he says, I will
respond to it in our next exchange.

The Amalekite Destruction a Moral


Atrocity?
Lindell Mitchell
Mr. Till agreed to affirm: "The Israelite destruction of the Amalekite nation as recorded in the
Bible constituted a moral atrocity." His first affirmative asserts much and proves nothing.
Clearly, he has no case to make. Mr. Till's proclivity to bluster about the affirmations of
Christians led me to insist that he set forth an affirmative case in this exchange. His first
article is disappointing but not unexpected.

Before responding to Mr. Till's "affirmative," I must address three matters. First, questions
about the existence of Amalekites after the battle recorded in 1 Samuel 15 are not relevant.
Mr. Till obligated himself to show that Israelite conduct in this incident constituted a moral
atrocity. Raising such issues does not establish his case. I refuse to chase that rabbit, except to
say that Amalekite military units that were out of the country when Saul's army swept through
would have escaped destruction. Note further that Saul defied God's instructions concerning
the Amalekite king and the best of the cattle. Who knows what more he neglected to do in
carrying out God's instructions?

Second, Mr. Till posed a series of questions to me. This was not part of our agreement;
therefore, I will not answer his questions. He knows it is the place of the negative disputant to
question the affirmative side. I did not make such a request of Mr. Till because of my desire

Volume 1990 - 2002 Issue


Page 337 of 2049
Skeptical Review Edited by Farrell Till
to grapple with his affirmative. However, he has provided precious little in the way of an
affirmative case.

Third, Mr. Till threw out two incidents in which Jewish rulers engaged in conduct he
considers morally atrocious. Citing these incidents does not establish his case. His job is to
show that Israelite conduct in the battle recorded in 1 Samuel 15 constitutes a moral atrocity.
David's conduct in 2 Samuel 8:2 and Menahem's conduct in 2 Kings 15:16 are not the points
at issue.

Mr. Till's affirmative consists of nothing more than asking us to visualize the Amalekite
experience when the Israelites marched against them. He reminds us of the primitive weapons
used. He asked us to consider what it was like to hear the shrieks and cries of the women and
children as the soldiers of God pursued their grim mission. He reminds us that each of the
Amalekites had individual names and personalities. I would have noted that each one was
created in God's image, but Mr. Till does not believe that. He thinks man ascended from bugs
and other lower life forms. The tragedy is enormous, and we recoil at the thought of such utter
destruction. It is a graphic illustration of the terrible cost paid by a society that persisted in
rebellion against God. It is a powerful reminder that the actions of leaders have enormous
effects on the lives of their people. The scene evokes powerful emotions, but intense emotions
do not establish Mr. Till's case. He is obligated to demonstrate that what took place was a
moral atrocity, not that it was a terrible calamity. No one disputes that fact.

Understanding the background leading to this battle helps us understand God's order to
exterminate the Amalekites. Amalek was a tribe of Bedouins living in the country to the south
of Judah. They were enemies of Israel from the time of the Exodus. They attacked the
refugees from Egypt in Rephidim (Ex. 17:8-16 ) and blocked their entrance into Canaan from
the South ( Num. 14:45 ). In Deuteronomy 25:17-19 , they are condemned for having picked
off stragglers from an Israelite column on the march. The wonder is that God's holiness had
spared the ruthless Amalekites for so long. Decades of patience had not produced any positive
change.

What could better impress upon the Israelites the monstrous nature of sin than being required
to administer punishment for sin? Thus, God declared a "holy war." In such a conflict, the
adversary along with his possessions was placed under a ban (cherem), i. e., no booty was to
be taken from the vanquished foe. All people and animals were to be killed, and all property
of value was to be burned as consecrated to the Lord (Josh. 6:17,21 ). The total destruction of
the adversary and his goods removed all personal profit motives. Those who participated in
battle fought only as avenging agents of God.

Mr. Till, a pro-death advocate of abortion, is concerned about the fate of the Amalekite
children but has no concern for 1.5 million innocents slaughtered in this country annually. He
will not allow the Omniscient God to look down the corridors of time and see that the babes
of Amalek were destined to become vicious beasts like their ancestors. He will not allow the
Lord of creation to mete out justice upon the wicked and call the innocent into the protection
of his heavenly city. He is distressed about the "thousands" of Amalekites, but 25-30 million
Americans murdered by abortion are of no concern to him.

Volume 1990 - 2002 Issue


Page 338 of 2049
Skeptical Review Edited by Farrell Till
He asked us to look with disdain on that horrible day when God punished the sin of Amalek,
but he supports the wanton slaughter of innocents occurring all about him. Mr. Till defends
death by dismemberment via the dilation and curettage abortion procedure. He supports
covering infants with a corrosive, forcing it into their lungs and stomach, and leaving them to
convulse for hours until death mercifully comes. This is accomplished through salt poisoning
abortion. Given a choice, I would much rather die by sword or spear!

Mr. Till will not concede that God stands ontologically on a higher plane than man, which
qualifies him to justly require actions that no man could require. He says there is no evidence
that God exists, but he spends a considerable portion of his time fighting against him. He tells
us that this is done in an attempt to free people from harmful religious superstitions, but Mr.
Till is obviously not free. His life is consumed with rebellion against what he says does not
exist.

Despite the restrictions Mr. Till places on God, he is confortable condoning the brutal death
of over 4,000 infants every day. According to Mr. Till, the population might get out of
control, and he would not have enough to eat.

After countless decades of rebellion, God passed judgment on the Amalekites. Proving
themselves intractable foes and a relentless threat to Israel, they were destroyed. Mr. Till will
not allow justice to be served in this case, yet he defends the slaughter of innocent infants who
are an inconvenience or a source of embarrassment. He is comfortable with tax money
financing the extermination of these unfortunate babes, because he thinks they represent a
threat to him. It is a moral atrocity for God to punish a wicked nation, but modern men can
justly kill people who are inconvenient. The Jewish nation cannot destroy an enemy that has
proved an unrelenting and brutal foe, but it is acceptable to kill people whose presence would
be embarrassing. The legs of the lame are unequal!

Three things make it impossible for Mr. Till to establish his case concerning Israelite conduct
in 1 Samuel 15 . First, he admittedly labors under an enormous burden of ignorance. He does
not know what he is talking about and has candidly admitted it. In his debate with H. A.
Dobbs in Portland, Texas, Mr. Till readily conceded that he knows far less than one percent of
available empirical data. In his debate with Mac Deaver in San Marcos, Texas, he admitted
the possibility of being wrong in affirming that God is guilty of moral atrocities. Until he
knows what he is talking about, Mr. Till should not accuse God of evil conduct.

Second, Mr. Till believes that man's knowledge is restricted to what he gains through one of
his five senses. Given that epistemological posture, he cannot know any- thing he has not
personally tasted, touched, smelled, seen, or heard. He has forever shut himself off from any
evidence that results from testimony. He has no literary historical past. However, Mr. Till
makes no attempt to be consistent in this matter. He readily appeals to history when he deems
it convenient to his cause. When an opponent appeals to history to show the fallacies in his
position, he hides behind his false epistemology limiting knowledge to information derived
through sensory perception. This will not pass. I insist that he be consistent or give ground on
this point. Given his epistemology, we should not be engaged in this exchange! Neither of us
saw, smelled, heard, touched, or tasted the battle recorded in 1 Samuel 15 . In fact, Mr. Till

Volume 1990 - 2002 Issue


Page 339 of 2049
Skeptical Review Edited by Farrell Till
doubts that the events described there ever occurred. Neither of us knows anyone who knew
someone who witnessed the events of 1 Samuel 15 .

Given Mr. Till's view, I am not sure that a man named Farrell Till actually exists. I do not
know that Canton, Illinois, is a real place. Who knows where those long laborious letters
filled with great swelling words, laced with bitter invectives, signed by someone purporting to
be Farrell Till really came from? Such absurdity is the inevitable consequence of the assertion
that one can know only what comes to him through sensory perception. Mr. Till's adherence
to this epistemological posture makes it logically impossible and ethically duplicitous for him
to affirm or deny anything that is not a part of his personal sensory experience. Therefore, he
cannot honestly hope to prove that Israelite conduct in 1 Samuel 15 was morally atrocious.

Third, Farrell Till believes that no absolute objective moral standard exists. If this is true, he
has no philosophical basis for objecting to any action, anywhere, anytime. It is impossible to
have a moral atrocity in the absence of an objective moral standard against which to measure
thoughts, words, and deeds. According to Mr. Till nothing is objectively wrong. Nothing at
all. Hence, he has no right to charge anyone with a moral atrocity.

All that he can logically say, given his philosophical stance, is that certain things offend
Farrell Till. To which I would only reply, if arguing from his perspective, "So what? So what
if you disapprove of my actions? I could not care less." There would be no moral imperative,
no logical intellectual basis for him to object to my actions. Unless he could physically
overpower me (which he cannot do), there would be no basis for him to demand redress of his
grievance. He is incapable of forcing the issue physically, and he has no intellectual route of
appeal in the absence of an objective moral standard. Given Mr. Till's view that there is no
such standard, I could assault him, inflicting multiple contusions and lacerations about his
head and body without being in error. He would not like it, it would be a most unpleasant
experience, but it would not be a moral atrocity. Remember, there is no objective moral
standard and no ultimate judge to enforce it. Hence, it would not be wrong for me,
accompanied by some of my ex-commando buddies, to go to Mr. Till's home to spoil his
goods, kill his children, ravage his wife, and mercilessly torture him because his views are
offensive to me. Thus, we are led by Mr. Till to the law of the jungle! It is vile and repugnant,
but it is the logical consequence of obliterating an objective moral standard.

I trust that in the next exchange the distinguished editor of The Skeptical Review will honor
his commitment to make the case that Israel's conduct in 1 Samuel 15 constituted a moral
atrocity. Emotive descriptions of the scene will not establish his case. Introducing other
incidents he finds objectionable will not establish his assertions. Attempting to shift focus to
the negative disputant by posing perplexing questions will never make Mr. Till's case. What
he must do is produce sensory evidence that compels the conclusion that Israel's conduct in 1
Samuel 15 was morally atrocious. He must show that his conclusion is logically demanded in
the absence of an objective moral standard. Saying it is horrible and offensive to his
sensibilities will not suffice under Mr. Till's philosophical construct. Perhaps he is saving the
best for last. We shall see.

(Lindell Mitchell's address is P. O. Box 411, Livingston, TX 77351.)

Volume 1990 - 2002 Issue


Page 340 of 2049
Skeptical Review Edited by Farrell Till

And God Came to Abimelech in a Dream


Farrell Till
Genesis 20 records one of the doublets that tell of Abraham's habit of passing his wife Sarah
off as his sister when they were living among strangers, this time in the city of Gerar. The
other occasion is recorded in Genesis 12:10-20 when Abraham and Sarah went into Egypt
during a famine. This earlier version of the doublet cited her beauty as the reason why
Abraham presented Sarah as his sister rather than his wife. "Indeed I know that you are a
woman of beautiful countenance," Abraham said to Sarah as they approached the Egyptian
border. "Therefore, it will happen, when the Egyptians see you, that they will say, `This is his
wife'; and they will kill me, but they will let you live. Please say you are my sister, that it may
be well with me for your sake, and that I may live because of you" (vv:11-13 ).

Although the text doesn't say so directly, it nevertheless implies that Sarah's beauty was the
reason for Abraham's duplicity while they were in Gerar, for king Abimelech, who had taken
Sarah with obvious sexual intentions, demanded an explanation when he discovered that
Sarah was actually Abraham's wife. "Because I thought, surely the fear of God is not in this
place," Abraham explained, "and they will kill me on account of my wife" (20:11 ). If
Abraham was afraid that "they" would kill him, as he was afraid that the Egyptians would do
on the other occasion, he must have thought that his life was in danger for the same reason,
i.e., Sarah's exceptional beauty.

I see a major credibility problem in this story. There are textual reasons to assume that Sarah
was no spring chicken even when Abraham lied about his relationship to her when they were
in Egypt. However, with no direct statement of her age at that time, one might concede the
possibility of her beauty being so striking that men of barbaric times might try to kill her
husband. The incident in Gerar, on the other hand, was entirely different. By this time, the
biblical text had described Sarah as a woman in her nineties (17:17 ), who had "passed the age
of childbearing" (18:11 ). The Bible even implies that she was by this time sexually inactive.
When Yahweh appeared to Abraham at the Oaks of Mamre to renew his promise that Sarah
would bear a son, she laughed within herself and said, "After I have grown old, shall I have
pleasure, my lord [Abraham] being old also" (18:12 )? So are we to believe that a woman this
old was so stunningly beautiful that men would actually want to kill her husband so that they
could have her? To raise again a question that we applied to the exodus story in an earlier
issue of TSR, how likely is this?

Sarah's apparent age at the time of this incident is not the only problem in the story. After God
revealed to King Abimelech that he was about to take another man's wife, Abraham's conduct
in this matter was whitewashed and Abimelech came off looking like the bad guy:

But God came to Abimelech in a dream by night, and said to him, "Indeed you are a dead man
because of the woman whom you have taken, for she is a man's wife. But Abimelech had not

Volume 1990 - 2002 Issue


Page 341 of 2049
Skeptical Review Edited by Farrell Till
come near her; and he said, "Lord, will You slay a righteous nation also? Did he not say to
me, `She is my sister'? And she, even she herself said, `He is my brother.' In the integrity of
my heart and innocence of my hands I have done this." And God said to him in a dream,
"Yes, I know that you did this in the integrity of your heart. For I also withheld you from
sinning against Me; therefore I did not let you touch her. Now therefore, restore the man's
wife; for he is a prophet, and he will pray for you and you shall live" (Gen. 20:3-6).

Let's consider what we have here. Abraham had lied about his relationship to Sarah and had
passed her off as his sister, but now Abimelech was the one who was in hot water, the one
who was described as "a dead man," specifically because of Sarah whom he had taken,
believing she was an unmarried woman. Where's the fairness in this? Does the inscrutable
Yahweh expect people to be mind readers? In terms of civilized moral standards, this whole
incident involved nothing that warranted killing anyone, but if anyone deserved to die because
of what had happened, then surely Abraham was the one. Furthermore, if anyone was to pray
for another in this matter, Abimelech should have been instructed to pray for Abraham whose
conduct had precipitated the entire affair.

The story ended with Abraham praying to God, who then "healed Abimelech, his wife, and
his female servants" (v:17 ). And why did Abimelech's wife and female servants need
healing? Well, it seems that Yahweh "had closed up all the wombs of the house of Abimelech
because of Sarah" (v:18 ). Abraham and Sarah both had lied in this affair, but the house of
Abimelech was the one to experience the wrath of Yahweh. Justice and fairness seemed to
have taken a vacation while Abraham and Sarah were living in Gerar.

Furthermore, there is a clear implication in this ending of the story that Abimelech had taken
his female servants as concubines. If Yahweh was so protective of Sarah's honor that he had
divinely intervened to keep Abimlech from "touching" her, we have to wonder why Yahweh
had not kept Abimelech "from sinning against [him]" by not letting him "touch" the female
servants. Are we to understand that Yahweh did not care as much for the honor of the female
servants as he did for Sarah's? If so, what does this do to the biblical claim that Yahweh is not
a respecter of persons (Dt. 10:17 ; 2 Chron. 19:7 ; Rom. 2:11 ; Acts 10:34 ; Gal. 2:6 ; Eph. 6:9
; Col. 3:25 ; 1 Pet. 1:17 )?

Another credibility problem in this story concerns the way that God revealed to Abimelech
the true nature of Sarah's relationship to Abraham. "God came to Abimelech in a dream by
night," we are told (v:3 ). This is the first incidence of divine revelation via a dream recorded
in the Bible, and it established a precedent that biblical writers used and reused thereafter. On
Jacob's journey to Paddanaram, Yahweh appeared to him in a dream and renewed the promise
to bless all nations through the seed of Abraham (Gen. 28:11-17 ).

When interpreting Pharaoh's dream about the seven lean and seven fat cows, Joseph told
Pharaoh that God had shown him "what He is about to do" (Gen. 41:25 ). Yahweh appeared to
Solomon in a dream and told him that he could have whatever he asked (1 Kings 3:5 ). In this
dream, Solomon asked for wisdom and was made the wisest man who had ever lived or ever
would live (v:5 ). An "angel of the Lord" informed Joseph in a dream that Mary's child had
been conceived by the Holy Spirit (Mt. 1:22 ). An angel of the Lord "appeared to Joseph in a
dream" and warned him to flee to Egypt to escape Herod's massacre of the innocents (Mt.

Volume 1990 - 2002 Issue


Page 342 of 2049
Skeptical Review Edited by Farrell Till
2:13 ). In Egypt, Joseph was told by an angel in a dream that Herod was dead and it was safe
to return to the land of Israel (Mt. 2:19 ). On his way back home with his family, Joseph was
"warned by God in a dream" to take the child to Galilee rather than Judea (Mt. 2:22 ).
Matthew seemed particularly fond of the divine-appearance-in-a-dream device.

Like Matthew, Daniel liked the dream device. Probably more than any other biblical writer,
Daniel claimed that he received divine revelations through dreams. When interpreting
Nebuchadnezzar's dream about the great image fabricated from different metals, Daniel said
to the king, "But there is a God in heaven who reveals secrets, and He has made known to
King Nebuchadnezzar what will be in the latter days" (2:28 ). Later, when Nebuchadnezzar
had the dream about the tree that grew until its height reached to the heavens, Daniel
informed the king that the dream meant he would be driven from men to live with the breasts
of the field until he came to realize that "the Most High rules in the kingdom of men" (4:32).
In the seventh chapter, Daniel himself dreamed a dream that was understood to be a divine
revelation of future events.

I could continue with summations of other dreams that biblical writers presented as divine
revelations, but these are sufficient to establish that divine revelation via dreams was a
common biblical scenario. In other words, Bible writers took dreams seriously and apparently
believed that God spoke to people in dreams, either directly or through angels. The fact that
they incorporated this belief into their writings takes us back to the question of likeliness.
How likely was it that God really spoke to Abimelech, Jacob, Pharaoh, Nebuchadnezzar,
Daniel, Joseph, et al in dreams as claimed throughout the Bible? Even if we assume that men
like Abimelech and Jacob actually did have dreams in which "God" spoke to them or that
Pharaoh and Nebuchadnezzar did have the dreams attributed to them or that Joseph did dream
about angels and/or God speaking to him, how likely was it that these were really incidents of
divine revelations via dreams? To apply the principle of Occam's razor to these dreams
(assuming that they actually did occur), which was more likely, that God really was speaking
through these dreams or that the dreamers, living in superstitious times, just thought that he
was? As for the actuality of the dreams themselves, which was more likely, that the dreams
actually occurred as recorded in the various stories or that Bible writers just made up the
stories?

So once again we see that when the test of likeliness is applied in this matter, the many
biblical tales about divine revelations via dreams fail to pass. If a person today should claim
to be in communication with God through dreams, we would view him with suspicion and
recommend psychiatric treatment, yet when we read about the same claim made by people
who lived centuries ago, in highly superstitious times, we venerate them and call them
prophets or men of God. Where is the logic in that?

Did Paul's Men Hear a Voice?


Dan Barker

Volume 1990 - 2002 Issue


Page 343 of 2049
Skeptical Review Edited by Farrell Till
[This article was set-up by Dan Barker on a Macintosh computer. We have copied only the
English text, without the picture and the Greek text that were included in the published article.
All Greek words in the article itself have been transliterated into equivalents in the English
alphabet. FT]

In the 9th chapter of Acts, Luke tells the story of the conversion of Saul, saying that "the men
which journeyed with him stood speechless, hearing a voice, but seeing no man." In the 22nd
chapter of the same book, Luke quotes Paul's own words regarding the same experience:
"And they that were with me saw indeed the light, and were afraid; but they heard not the
voice of him that spake unto me."

There is an apparent contradiction here: Luke says "hearing a voice," but Paul says, "They
heard not the voice." If the translation is correct, then Luke has made a mistake. (We can
assume that Paul, the primary source, is more trustworthy.)

There are two approaches that defenders of the bible have used in an attempt to clear up this
discrepancy. The first approach claims that the word "hear" should be translated "understand"
in Acts 22:9, meaning that although the men heard the voice, they did not hear the voice. The
second defense claims that the word "voice" should really be translated "sound" in Acts 9:7,
meaning that the men heard something but did not know it was a voice.

"Hear" or "Understand"?

I play professional piano part-time. Although I often use electronic keyboards in jazz bands, I
much prefer the acoustic piano, especially for solo work. Nothing matches the beauty of
physically produced tones resonating in a real, acoustic piano of quality wood. The overtones
mix in the air like no computer has been able to duplicate.

The word acoustic comes from the Greek word akouo (pronounced "a-koo-oh*), meaning "to
hear." I pronounce Greek the way a contemporary Grecian would say it. The old, scholarly
pronunciation was just a guess.] To hear physically, acoustically. Both Acts 9:7 and Acts 22:9
use "akouo."

Akouo does not mean "understand." New Testament Greek posses other words for
"understand." The main one is suniemi (sin-EE-ee-mee), which is "to understand in the sense
of putting things together (the word means "to send together"). There are also noeo (no-EH-
oh), from the word for "mind"; ginosko (gi-NO-sko), which means "to know"; and others.
These verbs have noun counterparts: sunesis (understanding). The word akouo has no noun
counterpart for "understanding."

However, this does not mean that "hear" cannot be rendered loosely as "understand" in some
special cases. We do it in English in the phrase, "I hear you." We also use "see" sometimes in
this manner: "Do you see what I mean?" Words denoting physical senses can sometimes be
interpreted in a "mental" way, poetically or loosely.

But the only way to do this is by context. It can't be done by grammar. Akouo always means
"hear," but it might additionally mean "understand," depending on the context.

Volume 1990 - 2002 Issue


Page 344 of 2049
Skeptical Review Edited by Farrell Till
Akouo NEVER means "not to hear."

There is only one instance where the King James Version (KJV) translates akouo as
"understand." First Corinthians 14:2: "For he that speaketh in an unknown tongue speaketh
not unto men, but unto God: for no man understandeth (akouo) him; howbeit in the spirit he
speaketh mysteries." The New International Version (NIV) puts it this way: "Indeed, no one
understands him; he utters mysteries with his spirit." It seems, in context, that "understand"
can be used here because, although there is obviously some physical hearing involved, there is
an ambiguity about how a "mystery" spoken by a "spirit" could be perceived. In any event,
there is no indication that nothing is heard.

The Greek in Acts 22:9 is:"ten de phonen ouk ekousan (EE-koo-san, aorist [past tense] of
akouo, 3rd person plural). The KJV and the New Revised Standard Version (NRSV) say that
the men did not "hear" the voice, but the NIV and Living Bible (LB) say that the men did not
"understand" the voice. On what grounds do the NIV and LB use such a translation? The
passage is not poetic. In fact, in the parallel Acts 9:7, telling the same story, the NIV and LB
do use "hear," from the verb akouo with the same object. There is nothing in the context of
either Acts 9:7 or Acts 22:9 to warrant a looser translation of akouo.

There are a few places in the New Testament where akouo (hear) and suniemi (understand)
act as synonyms, but the connection is explicit. In Matthew 13:13, Jesus reportedly said:
"Therefore speak I to them in parables: because they seeing see not; and hearing (akouo) they
hear (akouo) not, neither do they understand (suniemi)." If the second occur rence of akouo
means "understand," all by itself, then it would not have been necessary for Luke to add,
"neither do they understand." This underscores the fact that grammar is not enough to
determine when akouo might be translated loosely.

The NIV and the LB wish us to think that Paul's men "heard but did not understand" the
voice. But "hear" and "understand" are coupled together all through the New Testament as a
contrast of two different words. Matthew 13:23 says, "But he that received good seed into the
good ground is he that heareth (akouo) the word, and understandeth (suniemi) it." Matthew
15:10: "Hear (akouo) and understand (suniemi)." Mark 4:12: "...and hearing they may hear
(akouo), and not understand (suniemi)." Notice that Mark did not use ouk akouo when he
wanted to say "not understand." For similar constructions see also Matthew 13:15; Matthew
13:19; Mark 7:14; Luke 8:10; Acts 28:26,27; Romans 15:21.

There is nothing in Acts 22:9--nothing grammatical, contextual, or explicit--to indicated that


akouo should be translated anything other than "hear." In fact, the same words (often the very
same phrase: ouk ekousan) occur throughout the New Testament, but neither the NIV nor the
LB use "not understand" in those instances. Look at Matthew 13:17 (NIV): "Many prophets
and righteous people longed to see what you see, but did not see it, and to hear what you hear,
but did not hear (ouk ekousan) it." If Acts 22:9 should be translated "not understand," why not
here?

Look at Mark 8:18 (NIV): "Do you have eyes but fail to see, and ears but fail to hear?" The
phrase "fail to hear" is ouk akouete: "You do not hear." Again, if Acts 22:9 should be "not
understand," then why not here?

Volume 1990 - 2002 Issue


Page 345 of 2049
Skeptical Review Edited by Farrell Till
Other examples are John 5:37 (NIV): "You have neither heard his voice nor seen his form";
and Romans 10:18 (NIV): "Did they not hear (ouk ekousan)?" (See also Mark 6:11, Luke
10:24, Luke 16:31, John 10:8, Acts 9:12, Romans 15:21.) Why is the NIV not consistent?

If Luke had wanted Acts 22:9 to mean "not understand," he should have said so, either
explicitly or contextually. If he had wanted to contrast the two meanings, why didn't he follow
the New Testament practice of pairing akouo and suniemi?

"Voice" or "Sound"?

I once brought up this contradiction on an Arizona radio show where I was debating James
White, a self-styled Christian apologist. White immediately retorted that since phone (voice)
is in two different cases in these verses, it was meant to be understood differently: "voice" in
one in stance, but "sound" in the other.

He is right about the two different cases, but he is wrong about what this means. Greek
scholars who have more than a superficial knowledge of the language would never use this
argument.

Acts 9:7 has tes phones (tees fo-NEES) and Acts 22:9 has ten phonen (teen fo-NEEN) for "the
voice." The first is in the genitive case, and the second is in the accusative case.

Although the KJV and the NRSV use "voice" in both verses, the NIV and the LB translate
phones as "sound" in Acts 9:7. They appear to be suggesting that the genitive case should
change the meaning of "voice." A number of Christian apologists, such as Gleason Archer,
have used this argument.

But they are wrong. In this instance, the genitive case does not change the meaning of the
word in any way.

In many inflected languages, such as Greek, there is a flexibility of case usage (declension).
Some verbs take their direct objects in more than just the accusative case. This is explicitly
true of akouo, which can take either the accusative or the genitive.

If the apologists are right and the genitive cases does change the meaning, then this would
create dozens of contradictions elsewhere in the New Testament where such flexibility of case
is common.

For example, the writers of Matthew and Luke both relate Jesus's parable of the wise man
who built his house upon a rock. Matthew 7:24 quotes Jesus: "[W]hosoever hearth these
sayings (tous logous) of mine, and doeth them, I will liken him unto a wise man...." Luke
6:47, telling the SAME story, quotes Jesus: "Whosoever cometh to me, and heareth my
sayings (ton logon), and doeth them, I will shew you to whom he is like...." Both writers
related the same speech, but they used different cases for the object of akouo. Matthew used
the accusative and Luke used the genitive.

Volume 1990 - 2002 Issue


Page 346 of 2049
Skeptical Review Edited by Farrell Till
This is not a contradiction. There is a tiny inexactness about what declension Jesus might
actually have used when he spoke these words in history (if he indeed spoke them in Greek,
or spoke them at all), but there is no discrepancy. Matthew and Luke, each reconstructing the
scene from memory (or perhaps from notes, or translating from another language), can be
allowed some personal leeway in their choice of declensions. The Greek allows for such
flexibility.

Another example is when Matthew and Mark each report the appearance of Jesus before the
high priest. Matthew and Mark each report the appearance of Jesus before the high priest.
Matthew 26:65 quotes the high priest: "Behold, now ye have heard his blasphemy
(blasphemian)." Mark 14:64 reports the SAME story, quoting the high priest: "Ye have heard
the blasphemy (blasphemias)>" Matthew used the accusative for the object of akouo and
Mark uses the genitive. Again, there is no contradiction--just an impreciseness about what
actual word was spoken by the high priest.

Closer to home are Acts 9:4, Acts 22:7, and Acts 26:14, the story of Saul's conversion itself.
(It is told three different times.) Acts 9:4: "And he fell to the earth and heard a voice (phonen)
saying unto him, Saul, Saul, why persecutest thou me?" (Acts 22:7: "And I fell unto the
ground, and heard a voice (phones) saying unto me...." Acts 26:14: "And when we were all
fallen to the earth, I heard a voice (phonen) speaking unto me...." Notice the different cases.
Paul himself, telling the SAME story, uses two different cases. They cannot have meant two
different things. In fact, the NIV and the LB agree, translating both the accusative and the
genitive as "voice" in all three instances.

If the defenders of Acts 9:7 and Acts 22:9 are correct when they say that the difference in case
changes the mean ing of the word, then the above examples would be contradictory. They
have shot themselves in the foot.

To be fair, there are places where phone can allowably be translated as "sound," but this is
determined by context rather than case. Phone is used 140 times in the New Testament. It is
translated "voice" 131 times in the KJV. The other 9 times it is translated "sound" or "noise,"
but each of these is figurative, referring to something that is not a person: "the noise (phone)
of thunder," "sound (phone) of wind," "wings," etc. Notice that although the KJV translates
Revelation 6:1 as "sound of thunder" (phone, dative case), the NRSV, NIV, and LB use
"voice."

Neither thunder nor wind have a "voice," so it is allowable to use "sound." But in acts 9:7
there is no such context. In fact, Luke goes out of his way to insist that it was a person: "...
hearing a voice, but seeing NO MAN." If Paul's men thought they had heard thunder or wind,
then it would not have been necessary to add the phrase "but seeing no man."

Why should the NIV and the LB use "sound" in this instance? There is no linguistic or
contextual reason. It appears that they have simply wanted to get rid of a troublesome
discrepancy.

Volume 1990 - 2002 Issue


Page 347 of 2049
Skeptical Review Edited by Farrell Till
Why Do the Translations Disagree?

The NIV and LB translators cannot claim a new, more advanced understand of Greek. The
NRSV was published AFTER the NIV and LB, and it uses "voice." (In my opinion, the
NRSV is the best current translation of the bible, although not without problems.)

The motive of the NIV and LB translators are made clear in the preface to each book. The
NIV, translated by a team of evangelical scholars (instigated by the National Association of
Evangelicals), is introduced with these words: "We offer this version of the Bible to him in
whose name and for whose glory it has been made. We pray that it will lead many into a
better understanding of the Holy Scriptures and a fuller knowledge of Jesus Christ the
incarnate Word, of whom the Scriptures so faithfully testify."

If there is a contradiction in the New Testament, then it could not "faithfully testify" anything.

The NIV team was extremely selective in choosing its scholars: "[T]he translators were united
in their commitment to the authority and infallibility of the Bible as God's Word in written
form. They believe that it contains the divine answer to the deepest needs of humanity, that it
sheds light on our path in a dark world, and that it sets forth the way to our eternal well-
being." This is not the agenda of a team of objective scholars! This is evangelism.

If there is a contradiction in the Bible, the NIV translators, committed a priori to infallibility,
could never see it! (Some skeptics might be tempted to use the phrase, "There is none so blind
as he who will not see," but I would never stoop to such ad hominem tactics.)

The Living Bible does not claim to be a strict translation. It is a paraphrase by Dr. Kenneth
Taylor, who admits in the preface: "... when the Greek or Hebrew is not clear, then the
theology of the translator is his guide, along with his sense of logic.... The theological lodestar
in this book has been a rigid evangelical position."

What if an atheistic or skeptical organization were to translate the bible, putting together a
team of staunch materialists, systematically excluding conservative or evangelical scholars,
announcing a "rigid skeptical position,"claiming to be "united in our commitment to the
fallibility of the bible," and advertising the "hope that this translation will lead many astray
from faith into a solid doubt of the reliability of Scriptures?" Such prejudice clearly would
taint the objectivity of the translation.

One of the most popular evangelical verses is Revelation 3:20 (I used this often in my own
evangelism), where Jesus is quoted as saying: "Behold, I stand at the door and knock: if any
man hear my voice (phones), and open the door, I will come into him...." The genitive case is
used here, yet the NIV uses "voice" and the LB says "hears me calling." Neither uses
"knocking" or "sound." They are not consistent. They only employ the "genitive-case
argument" where it suits them.

Acts 9:7 and Acts 22:9 are contradictory. In their confessed missionary zeal, the translators of
the NIV and the Living Bible, and other evangelical apologists, have dishonestly tampered

Volume 1990 - 2002 Issue


Page 348 of 2049
Skeptical Review Edited by Farrell Till
with the meaning of scripture, using an ad hoc argument in order to disguise an embarrassing
discrepancy.

(Dan Barker works for the Freedom From Religion Foundation and is a regular contributor to
Freethought Today, a publication of FFRF. The mailing address of FFRF is P.O. Box 750,
Madison, WI 53701, e-mail dbarker@mailbag.com)

The Hovind-Till Debate


Dr. Karen Bartelt
On September 11, 1993, Kent Hovind, a "creation-scientist" evangelist from Pensacola,
Florida, and TSR editor Farrell Till met in public debate at the Faith Baptist Church in Pekin,
Illinois. Although the topic of the debate was "The Genesis story of the flood is scientifically
accurate in all details," Hovind distributed to the audience a handout that was a generalized
attack on evolution. The paper warned the audience to watch for "desperate measures" and
"illogical ideas" that the evolutionist side could be expected to use during the debate. Among
other things, the paper listed ad hominem arguments, ridicule and scorn, citation of majority
opinion, and various appeals to scholarship.

The debate was supposed to be comprised of a 30-minute opening statement each, 20 minutes
of rebuttal each, and a question-and-answer period (questions sent in by the audience), but
Hovind's opening statement was nothing but his well-travelled (and poor quality, I might add)
slide show. He opened by stating that all evolutionists believe anyone who is a Bible-
believing Christian is ignorant. Some other highlights: an 11-foot skeleton has been found in a
coal mine in West Virginia, proof positive that preflood humans were bigger (so where is this
skeleton now, Kent?); the geologic column doesn't exist anywhere in the world; Noah's ark
contained only babies and only single "kinds." And a relatively new twist: the 23.5 degree tilt
of the earth happened during the flood. A giant "ice meteor" that, because of its low
temperature was magnetic, banged into the north pole and dropped mammoths in their tracks,
the vapor canopy collapsed, and the earth was flooded to a depth of 12,000 feet. As Dave
Barry says, "I am not making this up." The presentation was rapid fire, leaving no time for the
audience to digest a topic before the next slide was flashed. Indeed, for someone who does the
show 700 times a year, the presentation was unpolished, and the slides were of poor quality.

Farrell Till defined science as being outside the realm of the supernatural. He spoke of the
polystrate Specimen Ridge trees and attacked the seaworthiness of the ark. The audience was
asked to consider how probable it was for a lone man or small group to build a huge ark sans
modern tools. All wooden ships have a maximum length far below the purported length of the
ark, and the audience was encouraged to search the references Till provided concerning
shipbuilding. Till asked if such a ship did manage to stay afloat, how did the cargo survive the
rough seas described by other creationists? Finally, if, as Hovind asserted, only a few "kinds"

Volume 1990 - 2002 Issue


Page 349 of 2049
Skeptical Review Edited by Farrell Till
were present on the ark, then Hovind must also admit that all forms of bovids, from bison to
cattle to deer, evolved rapidly after the flood-- something no evolutionist would ever state.

Hovind's rebuttal was to put God in the same bracket as electrons and gravity--natural
forces/particles that cannot be seen either! He stated, without proof, that humans "back then"
were not only bigger and longer-lived but had higher IQ's. Thus they could build the
pyramids, a feat we humans of today could never accomplish. (Something just occurred to me
here: the pyramids had to be postflood; they date roughly from the time of Moses... another
boo boo.) He stated that the Specimen Ridge trees have no roots (flatly false by my geology
books). God brought the animals to Noah and took care of all the little incidentals (like tons of
manure). Hovind encouraged the audience to be polite to Till, because, after all, "He is not the
enemy; he just works for him." Hovind admitted he couldn't prove most of his assertions but
that Till could not prove his either. Refer to the topic of the debate!!!!

Till pressed the issue that since there were seven pairs of each type of clean animal on the ark,
that meant 14 giraffes slopping about the ark on stormy seas, like it or not. How did they
survive? He pointed out that the largest pyramids are in the Americas, not Egypt. He closed
by pointing out that Hovind's assumptions--a vapor canopy, a level antediluvian earth, smarter
people--were not supported by evidence.

About half a dozen questions followed. I wrote two of the ones that were chosen by the
moderator/minister. Briefly....

Concerning a question on missing links, Hovind stated that Lucy was a chimp and was
assembled from bones found at sites miles apart. He said, "I wish I could have seen the train
that hit that chimp." Of course, it is well known that Lucy was recovered over about 50 square
meters of ground.

I asked what the anteaters ate the day the ark landed. Hovind said that they were vegetarians,
preflood and immediately postflood, and that special diets were not necessary then or now.
"Pandas don't just eat bamboo today; they love meat. Ask any zoo keeper." Perhaps that is
why pandas are doing so well in the wild right now! Till replied that you can't have generalist
animals on the one hand, and then, on the other hand, argue that the pronuba moth and yucca
plant were obviously created for each other.

Till pointed out that it was a creationist anachronism that the ark would have been sealed with
pitch, because this is obviously a coal (postflood) by-product. Hovind stated the word pitch
could have meant any oil--corn oil, for instance. I will have to pass this new use for corn oil
on to the Illinois Department of Agriculture. Let's grease up those boat bottoms. Till stood by
the translation of the Hebrew word; it had to be a petroleum product.

Hovind was taken aback by my question (derived from the Soroka and Nelson article) on the
amount of heat that would have been released by 40 days and 40 nights of rain. He talked
about craters as evidence for an "ice meteor" and said that this was all on faith. As I see it, 40
days/nights of rain still releases beaucoup de joules. He simply failed to address it, but Till,
who had the article in hand, expanded the problem and made the reference available to the
audience.

Volume 1990 - 2002 Issue


Page 350 of 2049
Skeptical Review Edited by Farrell Till
Was anyone swayed? Unlikely. The bulk of the audience was clearly unable to understand
how science differs from the supernatural. They became defensive and irritated whenever Till
said, "... but it's just not science." They were for the most part quiet, although a few choruses
of "Amens" resounded when they felt Hovind had made a point. The moderator/minister is to
be complimented. He was polite to both parties and clearly kept a lid on what could have been
a volatile situation.

(Dr. Bartelt is an assistant professor of chemistry at Eureka College. Her address is 22740
Grosenbach, Washington, IL 61571.)

Video Tape of the Debate


The Hovind-Till debate on the Genesis flood story was video taped and is available for
viewing via the library-loan system announced for the Till-Dobbs debate in the Autumn 1993
issue of TSR . To receive a tape on two-week loan, send $1 to cover the cost of mailing. We
ask that borrowed tapes be returned within two weeks so that they will be available for others
to use.

Since the tape is not copyrighted, we can make your own copy for $6. This debate has been
shown on local-access TV in Central Illinois and could be so used in localities where
"creation science" is a public issue. The tape will provide viewers with addresses where
information to combat "creation science" can be obtained.

Polytheism in Genesis: Baal and Ashtoreth


vs. Yahweh
Sol Abrams
Genesis 1:26-27 says, "And God said, `Let us make man in our likeness and let them have
dominion over the fish of the sea....' And God created man in his own image in the image of
God created he him, male and female he created them."

The word man in this text includes male and female . This is confirmed by the word them
whose antecedent is man. So he and his in this sense are both male and female. In fact, the
word him is superfluous, and we could omit the superfluity by stating the passage like this:
"In the image of God, he created them male and female." This means that male and female
were created in the image of God. In other words, man [male and female or mankind] was
created in the image of God.

Volume 1990 - 2002 Issue


Page 351 of 2049
Skeptical Review Edited by Farrell Till
Since man [male and female] was created in the image of God, it logically follows that this
god was both male and female. The word our implies more than one, so, in effect, what we
have is a god-pair consisting of a male god and a female god.

Chapter one of Genesis is from the Elohist source that used Elohim [gods plural] in referring
to "God." Originally, the male god was Baal, and the female god was his consort Ashtoreth.
Orthodox clergymen will argue that the us and our in the creation passage are simply
examples of the "royal we" used by emperors, but this rationalization is false. The book of
Genesis was written before the "royal we" originated. It began with the first Roman emperor,
Augustus, and included the emperor and his loyal civil administrators. Afterwards, it was
sometimes used in pagan religious ceremonies in the pre-Christian Roman Empire, which at
that time was polytheistic.

In Genesis 3:22 , there is further evidence of polytheism as the Hebrew gods are depicted as
saying, "Behold the man has become as one of us to know good and evil, and now lest he put
forth his hand, and take also of the tree of life, and eat and live forever...." Here again the
orthodox clergy will claim that the us is really the LORD God and the angels that were with
him, but this cannot be for a number of reasons. First, there is no mention of angels in Genesis
until Chapter 19 , but even if these angels did exist, they would have been acting upon orders
of the god-pair of 1:26-27 . So the us here was again referring to that god-pair. To further
show that the our and us in these Genesis passages referred to the god-pair of early Hebrew
polytheism, we have only to review the history of the ideological clashes between the
proponents of Baal and those of Yahweh that went on in the Caananite-Israelite lands from
the time of the judges until the fall of Judah and the Babylonian captivity.

During these times, Baal and his consort Ashtoreth were worshiped by many Israelites both in
Samaria (Israel) and Judah even after the captivity, mainly by those who remained in the
conquered lands. Yahwists like Ezra finally purged the Israelites (by then known as Jews) of
all Baal residuals and even forced them to give up their Baalish wives and families (see Ezra
9-10 ). Ezra's purging of Baal appeared to be complete. It was his wish to erase Baal
completely from the Israelite past; however, the residuals in Genesis 1 and 3 continue to
remind us not only of Israel's polytheistic past but of the Canaanite origins of Judaism.

Using archaeological evidence on one hand and biblical between-the-line implications on the
other, the following conclusions support the premises stated above:

(1) Most of the Israelites at the time of the exodus (about 1250 B.C.) were already located in
the Canaanite area, which, incidentally, was at that time a part of Greater Egypt. A relatively
small number, probably only one tribe (Levi), were in Egypt. Exodus 1:15 , for example, says
that only two midwives were needed to attend the births of Hebrew children. Furthermore, the
Israelites needed divine help to defeat a small seminomadic tribe (Ex. 17:8-13 ) in
contradiction to the later editor's estimate of an army of 600,000 men (12:37 ) besides
children (and women?).

(2) This relatively small group of Israelites from the outside (Egypt proper) formed some type
of symbiotic relationship with the much larger inside group (which consisted of Israelites and

Volume 1990 - 2002 Issue


Page 352 of 2049
Skeptical Review Edited by Farrell Till
Canaanites, the so-called mixed multitude) to form the "12 tribes" (when they were not
fighting each other).

(3) The outside group was the Yahwist cult, the inside group the Baal cult. The struggle
between the two groups went on for well over 500 years.

(4) Apparently it was not until the reign of Josiah that the Yahwist group was able to achieve
dominance. The "lost book" of Deuteronomy was discovered in the house of the LORD (2
Kings 22:8 ), and the Passover was reinstituted after a lapse of 500 years (if indeed it even
existed before then). The golden calf (symbol of the Kings of Israel) from the reign of
Jeroboam was suppressed (2 Kings 23:15 ).

(5) Biblical scholars agree on how the Pentateuch was put together. The sources were (E)
Elohist, (J) Yahwist, (P) Priestly, (D) Deuteronomist, and (R) Redactor. The last two were
written to dovetail with the first two, and the writers tried to do two things: (1) eliminate all
contradictions, and (2) eliminate all vestiges of the Israelite primitive past of pagan
polytheisism.

Richard Elliott Friedman noted in Who Wrote the Bible? that after the destruction of the
temple in Jerusalem by the Babylonian king Nebuchadnezzar in 587 B. C., some Jews fled to
Egypt and formed a colony at Elephantine at the first cataract of the Nile (p. 153). They built
a temple there, which was clearly against the law of centralization in Deuteronomy. The
extraordinary thing about the Elephantine temple, however, was that this group of expatriated
Jews worshiped Yahweh and two other gods, one male and one female. This god-pair
apparently was Baal and Ashtoreth. The Yahwist Jews living elsewhere were not happy with
this development, for when the Elephantine temple was destroyed in the 5th century, B.C.E.,
they would not help to rebuild it (p. 154).

The scholarly piecing together of information from archaeological discoveries and overlooked
textual implications of a polytheistic past indicate that the editors failed in both endeavors
listed above. As a result, we know today that monotheism came to Judaism not by divine
revelation but by a process of theistic evolution.

(Sol Abrams' address is 132 Easthampton F, West Palm Beach, FL 33417.)

Study Aids
In addition to The Skeptical Review, Skepticism, Inc., publishes other materials that might be
useful to those wishing to improve their skills on the subject of Bible inerrancy.

The booklets below are available for $2.50 each, postage paid.

The Laws-Till Debate, a 47-page unfinished debate with James Laws, Jr., a professor of
apologetics at Tennessee Bible College. Although Laws challenged, he quit after only three
manuscript exchanges and has since refused to accept mail from Till. Correspondence is
reproduced in the booklet.

Volume 1990 - 2002 Issue


Page 353 of 2049
Skeptical Review Edited by Farrell Till
Jackson-Till Debate, 50 pages on the issue of Bible inerrancy.

Prophecies: Imaginary and Unfulfilled, an in-depth examination of the most commonly


claimed examples of prophecy fulfillment. Recently revised to expand its scope.

Reprinted from the Firm Foundation....

The Dobbs-Till Debate


Dub McClish
On May 23-26, 1993, H. A. (Buster) Dobbs, Christian, and Farrell Till, atheist, engaged in a
public debate in Portland, Texas. The debate was conducted as part of the First Gulf Coast
Lectures sponsored by the Portland Church of Christ. The debate was about the deity of Jesus.
Dobbs affirmed: "Fulfilled prophecy of the Old Testament proves Jesus of Nazareth is the Son
of God." Till affirmed: "The New Testament claims of prophecy fulfillment in the person and
deeds of Jesus of Nazareth were fabrications or misapplications of Old Testament Scripture."

The format of the debate was unusual. It was agreed that the negative speaker could call a
stoppage of his allotted time at any time during his speech and ask his opponent any number
of questions. The affirmative disputant was given a maximum of two minutes to answer each
question. At the conclusion of the questions, time-keeping was then resumed, and the balance
of the time evenly divided between the speakers. This made for numerous very interesting
exchanges that would not have taken place otherwise. Brother Dobbs forced Mr. Till to agree
to the fact that he (Till) has no hope beyond the grave whatsoever. When asked what he
thought about the Judgment, Mr. Till replied that "he never thinks about it." However, at least
for a brief time during the debate, brother Dobbs, with great effect, forced him to think about
it.

The debate was marked by good attendance at each session. Some denominationalists
attended, and three or four atheists were present. One of them made it a point to walk out
during the closing prayer each evening. Brother Dobbs did a creditable job of defending and
expounding the Scriptures. It is likely that no minds were changed by the debate as is often
the case in such proceedings. However, it is good for God's people to cover such ground
again, hear assaults on the faith from a live enemy, and think through such assaults and the
answers to them again. The debate is available in both audio and video tape formats.

(Dub McClish's address is 908 Imperial Drive, Denton, TX 76201.)

Thoughts on the "Final Judgment"

Volume 1990 - 2002 Issue


Page 354 of 2049
Skeptical Review Edited by Farrell Till

Farrell Till
As indicated in the foregoing review of the Dobbs-Till Debate was originally published in the
Firm Foundation, a monthly paper that Mr. Dobbs himself is the editor of. Usually, reviews
of debates published in Church-of-Christ papers revel in the great victory for truth that was
scored by Brother Whoever, so the rather subdued tone of this review is quite remarkable and
undoubtedly more telling than the author, Dub McClish, would willingly admit. Mr. McClish
is doctrinally aligned with the guardian-of-the-faith watchdogs in the Church of Christ who
constantly plead for a return to the "old paths" in opposition to a growing effort in the church
to moderate its hard-line fundamentalist position. Mr. McClish also attended each session of
the debate, so if he had seen anything in Dobbs's performance to gloat about, surely he would
have exploited it. As it was, he had to confine the praise for his side to an unenthusiastic
comment about the effective way that Dobbs had forced me to think about the final judgment.

This is not to say that no one in McClish's corner has praised Dobbs's performance in the
debate. In personal correspondence to me, Lindell Mitchell, whose article appears on pages 4-
5 in this issue, informed me that Dobbs had "spanked [my] britches" and reduced my
arguments to "utter ruin." He did not attend the debate, so I assume that he formed this
opinion from having viewed the tapes or else from a process of wishful thinking. In the total
absence of evidence to prove the inspiration and inerrancy of the Bible, many fundamentalists
apparently believe that fervently wanting it to be the infallible word of God will somehow
make it so. It isn't surprising, then, that someone with this kind of mind-set could find reason
to believe that the evasive tactics and straw-man antics that Dobbs resorted to in the debate
constituted "spanking my britches" and reducing my arguments to "utter ruin."

Is it fair to say that Mr. Dobbs resorted to straw-man antics? Well, the issue in the debate was
prophecy fulfillment. Mr. Dobbs signed on to affirm that "fulfilled prophecy of the Old
Testament proves Jesus of Nazareth is the Son of God," but the tapes of the debate will show
that he spent a disproportionate amount of time attacking my despicable atheism and
philosophy of despair. Advertisements of the video tapes in the Firm Foundation have
boasted that "(t)he degrading nature of atheism was clearly shown by brother Dobbs." I
emphatically dispute this claim. Mr. Dobbs repeatedly proclaimed the degrading nature of
atheism, but he by no means clearly showed that it is degrading. Fundamentalists seem to
have great difficulty recognizing the difference in saying that a thing is true and in proving
that it is true. Undoubtedly this explains why Mr. Dobbs and his audience of sympathizers
seemed to believe that he had proven his proposition by simply showing that the Bible says
that Jesus of Nazareth fulfilled Old Testament prophecies.

Now comes Dub McClish to tell us that "for a brief time during the debate, brother Dobbs,
with great effect, forced [me] to think about" the final judgment. After I had read this review,
I wrote Mr. McClish to inform him that he was mistaken. Dobbs did not force me to think
about the final judgment. As McClish reported in his review, Dobbs merely asked me what I
thought about the judgment, and my reply was that "I never think about it." At that moment,
the biblical doctrine of the final judgment was necessarily in my mind; otherwise, my brain
could not have processed the question and answered it. As soon as my answer was given,
however, all thoughts of a "final judgment" passed from my mind, so my answer to the

Volume 1990 - 2002 Issue


Page 355 of 2049
Skeptical Review Edited by Farrell Till
question still stands. In the sense of lying awake at night worrying about it or going through
my daily activities with nagging thoughts about it on my mind, I never think about having to
stand before some kind of infinite deity in what Christians call a "final judgment."

Does this mean that I do in some sense think about it? Well, of course, I "think" about it in
that it is obviously a biblical doctrine, so since I spend considerable time studying the Bible, I
necessarily encounter passages that teach the concept of a final judgment. That, however, is
the total extent to which I think about "final judgment." I personally regard the doctrine as just
another of the many absurdities in the Bible. It is a finely honed fear tactic that religionists use
to keep the superstitious in line, but there is no proof whatsoever that any such event will ever
occur. The idea of a final judgment is a Persian concept that the Jews apparently encountered
during their Babylonian captivity. It is frequently proclaimed in the New Testament, but so
are numerous other untestable and, therefore, unprovable claims. The virgin birth of Jesus his
divinity, his resurrection, his ascension--all of these are proclaimed in the New Testament,
along with the doctrine of a final judgment, but the truth of these assertions rests entirely on
the word of a bunch of religious mystics, living in highly superstitious times, who claimed
that they were inspired of God. How much credence can rational people give to such claims?
In my case, not much at all, which is why I don't sit around worrying about a final judgment.

For the sake of agument, however, let's just grant two claims that the fundamentalist side has
made about the Portland debate: (1) Dobbs very clearly showed the degrading nature of
atheism, and (2) Dobbs, with great effect, forced me to think about the final judgment. Even
with those concessions, what has the opposition proven, for the debate was about neither
atheism nor the final judgment? It was about the issue of prophecy fulfillment. The record
will show that Dobbs in no way sustained his belief that prophecy fulfillments proved that
Jesus of Nazareth was the son of God. He hardly even addressed the issue. If Dub McClish
wishes to see that as "a creditable job of defending and expounding the Scriptures," I guess he
is entitled to do so, but he is sadly mistaken if he thinks I spent any time during the debate--or
since--thinking about the final judgment.

To Those Who Worship the Bible-Idol


Dave E. Matson
So you think that those antediluvian children may have been unduly influenced by their
parents, part of the cancer that had to be cut out? Poor, limited God had no choice, I suppose.
He had to get all those boys and girls lest their evil ways corrupt the purity of his postflood
generations. Such purity, as exhibited by Noah's drunkenness and debauched state right after
the flood, had to be preserved, no doubt, from the evil influence of those antediluvian
children. No doubt the pure morals of all those idol worshipers and butchers of the postflood
generations, not to mention the pure-minded citizens of Sodom and Gomorrah, were in danger
of being contaminated by pre-flood thinking. Thus, in order to insure the purity of mankind,
all those antediluvian kids absolutely had to be drowned. That's why everyone is so pure of

Volume 1990 - 2002 Issue


Page 356 of 2049
Skeptical Review Edited by Farrell Till
heart today! How horrible it would have been if a few of those antediluvian boys and girls had
been allowed to live, to contaminate us all!

Were the animals and the newborn babies also part of that treacherous cancer? When the
Nazis gassed six million Jews did they lean on a rationalization any more absurd than the one
you're leaning on? I doubt it! When we start justifying evil to preserve our worship of biblical
inerrancy, or any doctrine, then we debase our whole system of values. Our hearts and minds
become warped, numb to atrocity, blind to evil, and we become the perfect cannon fodder for
the next Hitler. The man or woman who can view the children of any age as evil, who can
discover no solution other than killing them all, is not to be trusted with anything more
dangerous than a squirt-gun... or any office higher than dogcatcher. A Stone Age deity, of
course, can get away with such things, being little more than a stand-in for the random, fearful
forces of nature. Thus, the gods of primitive peoples, including the storm-god of the Old
Testament, were potential killers to be feared and placated with sacrifices. When a village was
wiped out by volcano or flood, everyone assumed that their god had been angered or
provoked in some manner. What else could they believe? Scientific enlightenment had yet to
overtake superstition. When they lost a battle, everyone assumed that they were being
punished by their god for some indiscretion. The job of their priests was to identify the
provocation, correct it, and thus humor their god. Keeping their gods in good humor or
sustaining them in symbolic ways (as did the Aztecs) was the all-important job of the ancient
priesthood. Happy gods meant happy times; angry gods sent disaster. If you read between the
lines of the Old Testament, you can see this type of rationalization constantly at work.

It never occurred to the ancients, therefore, to question the morality exhibited by their gods.
The fearful forces of nature spoke powerfully... and man listened. Who could question the
power of lightning or the thumping of giant hailstones or the strength of floods? It never
occurred to the ancients that a truly powerful god would not have to act in such clumsy ways,
for nature was the only god they knew. It never occurred to the ancients that a truly wise and
powerful god would communicate face to face with each man or woman rather than through
nebulous dreams, divinations, or (later) scriptures. The gods of nature never spoke openly but
rather in whispers and dreams and omens, or so the ancients perceived. It never occurred to
them that their god should be concerned with the well being of all peoples, includ in those
days were tribal gods, including Yahweh. They concerned themselves only with their
particular tribes.

Thus nature was the only god the ancients knew, and their gods spoke in terms of
earthquakes, floods, epidemics, volcanic eruptions, droughts, fertility, good crops and bad
crops, victory and defeat. Thus, the gods acted in strange and powerful ways for good and
evil, and it was of the utmost importance to divine their will and placate them. The Hebrews
had their magic "dice," the Urim and Thummim, and the study of animal livers was
practically an industry in that part of the world. God's strange methods were not to be
questioned, but they might be divined with profit.

Today, of course, the better educated among us know better. Once we have separated the
concept of God from the random acts of nature, once we have decided that God must be good
and moral to the highest degree, then such Stone Age nonsense as Noah's flood, the divine
destruction of Sodom and Gomorrah, and the divinely sanctioned rape of Canaan cannot be

Volume 1990 - 2002 Issue


Page 357 of 2049
Skeptical Review Edited by Farrell Till
taken seriously. If a god created this universe, then I imagine that he is too good, too wise,
and too powerful to be limited to Stone Age solutions! Reduce the concept of God to the
random amorality and violence of nature, if you must, but don't sell me the final product as
the highest agent of love, wisdom, and power!

(Dave E. Matson's address is 330 South Hill Avenue, Pasadena, CA 91106.)

From the Mailbag

I read your piece on why Saul died, and it reminded me of the inconsistency of the Bible as to
just who killed Saul. [1] Was it the LORD? Saul "enquired not of the LORD: therefore he
slew him, and turned the kingdom unto David the son of Jesse" (1 Chron. 10:14 , KJV). [2]
Was it the Philistine archers? "And the battle went sore against Saul, and the archers hit him;
and he was sore wounded of the archers" (1 Sam. 31:3 ; 1 Chron. 10:3 ). [3] Was it suicide?
Saul begged his armor-bearer to kill him, but he would n't. "Therefore Saul took a sword, and
fell upon it" (1 Sam. 31:4-5 ; 1 Chron. 10:4-5 ). [4] Was it a young Amalekite? A young
Amalekite man happened by and Saul begged him to "[s]tand, I pray thee, upon me, and slay
me: for anguish is come upon me, because my life is yet whole in me." [The Amalekite told
David], "So I stood upon him, and slew him, because I was sure that he could not live after
that he was fallen" (2 Sam. 1:8-10 ).

Again we have a situation where all versions cannot be right, but they can all be wrong!

(From Fred Acquistapace, author of Miracles That Never Were, Eye-Opener Books, P. O.
Box 1531, Santa Rosa, CA 95402).

It's a fool's errand to fool around with a foolish passage from a discredited book in order to
prove someone a fool. That neither fools the skeptic nor makes a fool of him. Thus, we may
answer Liddell's short letter.

As for Mr. Moffitt, I'm delighted that his heart is neither wicked nor rebellious, thus making
him privy to the inner secrets of the Bible. Breathlessly I read on, hoping to catch the deep
pearls of wisdom which surely must fall from such lips. No doubt, I would learn things that
had eluded those worldly Bible scholars for centuries! But alas! Mr. Moffitt uses the same old
tired crutch so necessary to all wishful thinking.

Mr. Moffitt demands that the skeptic prove biblical errors with 100% certainty or renounce
the claim of biblical error. Such a proof, of course, cannot be had outside of formal systems of
logic, such as mathematics. Let me make clear the unfairness of such a requirement. I
challenge any reader to prove, with 100% certainty, that the earth exists! However clever your

Volume 1990 - 2002 Issue


Page 358 of 2049
Skeptical Review Edited by Farrell Till
reply, I can counter with logically possible alternatives which you can't disprove. Should we
adjust our lives to the possibility that the earth might not exist? Of course not!

In the real world of atoms and energy, and that includes Bible apologetics, error is always a
matter of probability. To say that a book is erroneous is to say that it is erroneous beyond any
reasonable doubt. Once it has been shown that the case for error is much stronger than the
case for no error, then one might rightly claim an error. The rational mind will accept that,
even as it provisionally accepts the existence of the earth, but will always be open to new
evidence. Meanwhile, it does not cling to mere possibility with the idea that someday,
somehow, vindication will be had. If a book cannot be defended on the basis of what is
probable, then it cannot be rationally defended, period. Anyone venturing beyond that point
day. enters the realm of wishful thinking and fanaticism. (I will leave it to Mr. Till to provide
the customary rejoinder to Mr. Moffitt's exercise in wishful thinking.)

In closing, I find Mr. Moffitt's god (small "g" as befitting an inferior concept God)
reprehensible. Any good teacher will go out of her way to help a sincere, failing student; she
will clear away the obstacles to his learning. Mr. Moffitt's god, however, not only ignores the
student but blinds him as well!

(From Dave E. Matson, whose address appears after his article on page 13 in this issue.)

On the matter of absurdity, I've always compared Jehovah to the gods of Zoroaster, who lived
around 2,600 years ago. He had two gods, Ormuzd or Ahura Mazda, the god of light, and
Ahriman, the god of darkness. The Jewish people encountered Zoroastrian ideas in the
Babylonian captivity. Abraham came from Ur of the Chaldees, the hotbed of Magi magic,
astrology, and Zoroastrianism. The "wise men" or Magi reportedly saw Jesus in their
horoscopes and came to see him. Following Jesus' "star," they very well might have believed
in the gods of darkness and light.

Modern Zoroastrians are known as Parsees. They follow the war in heaven and earth via
astrology, and worship the sacred fire, which has the light of the good god, as well as the
god's heat. The heat of the fire and the sun is the life of the god of light, like human body heat.

Like the Jews, the Zoroastrians have clean and unclean animals. The good clean animals, like
cows and sheep, were created by the god of light. They do not kill, and they eat plants to live.
The bad unclean animals, like wolves and tigers, were created by the bad god of darkness.
They kill and eat people and animals alive!

The problem with equal and infinite gods of good and evil is that they can only exist in eternal
stalemate. They neutralize each other. Also, the good god is a lousy fighter. He cannot fight
the war in heaven because he cannot do harm to his enemies.

To overcome this problem, the Jewish worshipers of Jehovah made Jehovah a mixture of
good and evil. Jehovah punishes. He kills. He lies and deceives his enemies and his friends.
There is nothing so vile that Jehovah would not do it or allow it, and nothing so good and

Volume 1990 - 2002 Issue


Page 359 of 2049
Skeptical Review Edited by Farrell Till
perfect that Jehovah would not give it as a gift to his children. The mixture of good and evil in
God confounds the Jews and Christians to this day.

The absurdity of such an object of worship is clear. Like the gods of light and darkness of
Zoroaster, the two sides of schizophrenic, split-personality Jehovah and the Trinity neutralize
each other. The good and evil in the universe from no god, from imaginary gods, or the one
being who can only exist forever in stalemate. This means that such a being cannot act in
either direction, and effectively cannot exist If the being does exist, his (their) wheels
continue to spin, unnoticed, for all after his eternity. The Christian New Testament is faced
with this hellish problem and tries all, a weak, limited devil cannot create or co-create with
God. No Christian is nuts enough to claim that the devil has enough power to create the evil
half of the universe--including tigers, snakes, mosquitoes, and the firmament itself. No, it is
God who created it all, creeping and crawling insects included, and "saw that it was good." So
the devil, bad as he is, is just another creature who can be zapped by God just like humans
and man-eating wolves. The devil created nothing: his creator created the whole works.

And since Jehovah and the Trinity cannot zap the devil because they are tied-together bundles
of goodness and evil, the devil does as he pleases. In this picture, the universe would be
entirely evil under the devil. This is absurd, because even Christians admit there is much
beauty and goodness in the universe. Therefore, neither a mixed good and evil being nor even
the devil can be detected in the universe.

(From Frank Mack, P. O. Box 3012, Chicago, IL 60654.)

I [have] gathered a list of gospel passages that are not found in the oldest extant manuscripts.
This list is based only on footnotes, deletions, and bracketing of the passages in the five post-
KJV bibles (who'd have thought the more militant the atheist you become, the more bibles
you buy!). I've bought both the RSV and NASV since I wrote last....

Part of what led me in this direction, I think, was the lack of recognition from the crowd in
Portland to your dismissal of the Josephus citation as an insert. The term just didn't register
with most of the people there. And I was rather surprised when I developed my list of gospel
inserts to find so many of the most famous and most often quoted passages on the list.

Well, hopefully, I'll have something worth showing to you on that before too long. I've also
started working up a rebuttal to Moffitt's rebuttal of your "Dead Man" article, focusing on the
"good hearts" slur he throws on us from Luke 8:15 ....

Congratulations on the new printer and format. It looks great.

(From Earle C. Beach, 13203 Tamayo Drive, Austin, TX 78729-7403.)

Volume 1990 - 2002 Issue


Page 360 of 2049
Skeptical Review Edited by Farrell Till
The New format with laser print looks great! I continue to excerpt from TSR your articles
(with full credit given) for BBS posting where Bible inerrancy discussed. With excellent
results over all, I am happy to report.

(From Frank Lovell, Jr., 1907 Deer Park Avenue, Louisville, KY 40205.)

Editor's Note: We knew that many subscribers were feeding our materials into computer
bulletin boards, because we receive many inquiries and subscription requests in response to
things that were seen in BBS. Those who wish to post TSR articles or excerpts may do so
without asking permission. We would appreciate receiving credit to assist us in establishing
new contacts.

I want to thank you for your excellent publication. I have enjoyed the year's worth of my
introductory subscription and am renewing it with great enthusiasm. I have made the check
for $10 for a one-year subscription. I know that this is way over your rate. Please use it to
help new freethinkers, like the woman in your lead article Autumn 1993....

I would also like to extend an invitation to you to speak in our area, should you be in the
Minneapolis/St. Paul area. There are several active groups of freethinkers in the metro area.
There is an active atheist group, humanist group, a skeptics group and a growing group of
atheists at the University of Minnesota. I am program chairman for the Humanist Association
of Minneapolis/St. Paul. Our groups often set up joint meetings when nationally known
speakers come to town. Dan Barker of the Freedom From Religion Foundation will be
speaking at a joint meeting in November. Last year Tim Madigan, of Free Inquiry, came to
speak at the university meeting, which all groups helped with. One member of our group has
also started a freethinker BBS, the Freethought Thinktank.

Thank you again for your publication and your tireless pursuit of the truth.

(From Scott Lohman, 3822 Aldrich Avenue N, Minneapolis, MN 55412.)

Editor's Note: The freethought movement must be alive and in Minneapolis. TSR has more
subscribers in the greater Minneapolis region than any other urban area. The invitation to
speak was accepted with the explanation that it would give my wife and me the opportunity to
visit our daughter, who lives in Minneapolis.

I think your publication is worth a lot more than $4. I would love to get a loan copy of the
Dobbs-Till Debate. After I view the tape, I would like to forward it to the Spokane,
Washington, F.F.R.F. [Freedom From Religion Foundation], and they can get it back to
you....

If all the readers who appreciate you were to send a letter, I think you would be deluged.

Volume 1990 - 2002 Issue


Page 361 of 2049
Skeptical Review Edited by Farrell Till
(From Leonard Reitz, 3660 Ninth Drive, Baker City, OR 97814.)

Editor's Note: I do, in fact, receive many letters. I regret that I don't always have the time to
answer all of them, so I will take this opportunity to say that even if you don't receive a
response, I appreciate receiving your reactions to the paper.

Needless to say, the tapes were sent to Mr. Reitz, and permission was granted to send them on
to the Spokane chapter of F.F.R.F. We think that the Bible inerrancy position took a trouncing
in this debate, so we want as many people as possible to see how weak the evidence is for the
prophecy-fulfillment argument that bibliolaters so frequently resort to in trying to defend
Bible inerrancy. The tapes are still available on two-week loan for $2 to cover cost of
packaging and mailing.

From a few perusals of the articles in your newsletter, we feel that you are not at all interested
in the veracity of the word of God in the Bible. We certainly

feel that the Bible is the word of God and your letters are the words of men.

As Roman Catholics, however, we do believe that just as Jesus, the Savior, commissioned 12
apostles to carry on his message to all the earth, he also has apostles today in the name of
Popes and Bishops and they are responsible for promoting the message today. Bible
interpretation is not for individuals but is subject to Church teachings. Most of the known
world believes in the Bible and that it is the word of God. Why would you want to destroy
that belief?

Please do not send us anymore [sic] criticism of the Bible. We really believe that the message
of Scripture and Christ will save the world.

(From Stu & Judy Locklin, 2026 North 3372 Road, Ottawa, IL 61350.)

Editor's Note: To comment on everything that is wrong with the thinking in this letter would
require a special edition of TSR. The Locklins were added to the mailing list because they
wrote a letter to the Ottawa newspaper about the activities of The Freedom From Religion
Foundation (in which I was involved) during the controversy over the display of religious
paintings on public property. Their letter communicates not just an appalling ignorance of
basic facts about religion but also the reason for their ignorance. They see no need to inquire
and investigate. They are perfectly willing to let popes and bishops think for them. How
backward would the world be if everyone had this attitude?

Recommended Reading

Volume 1990 - 2002 Issue


Page 362 of 2049
Skeptical Review Edited by Farrell Till
If you enjoy the articles published in The Skeptical Review, you will be delighted to know that
a paperback reprint of Is It God's Word? by Joseph Wheless is now available. Published
originally by Alfred A. Knopf in 1926, this book thoroughly exposes textual inconsistencies,
contradictions, discrepancies, anachronisms, and mythical content in the Bible. The 500-page
reprint can be obtained from H. H. Waldo, Bookseller, P. O. Box 350, Rockton, IL 61072, for
$33 plus $2 for postage.

Although $35 may seem like a lot of money to pay for a paperback book, I personally
consider it a bargain. In my opinion, this is one of the best works of its kind that I have ever
read. When I am asked to recommend books that discuss biblical discrepancies, this one is
always put at the top of the list.

After securing it through interlibrary loan, I made inquiries at several rare-book outlets trying
to find a copy to buy. When these efforts proved unsuccessful, I requested it again through
interlibrary loan and made myself a xerographic copy at 5 cents a page and was delighted to
have even this. At two outlets, I paid search fees for the book and listed $100 as the price I
was willing to pay for a copy in at least fair condition.

If you want excellent material to use in debunking the Bible when door-to-door missionaries
come knocking, this book will give it to you.

Losing Faith in Faith


With this issue, we welcome Dan Barker as a contributor to our efforts to debunk the myth of
Bible inerrancy. Dan is the public relations director of the Freedom From Religion
Foundation and needs no introduction to most of our readers. An ad for his book Losing Faith
in Faith appears at the end of his article on page 9.

We highly recommend this book to our readers. In addition to exposing many flaws in the
Bible inerrancy doctrine, it has an interesting story line in the parts that tell of Dan's transition
from fundamentalist preacher to freethinker.

Geisler-Till Debate
Norman L. Geisler, a prominent evangelical spokesman for biblical inerrancy, has tentatively
agreed to debate TSR editor Farrell Till on March 29, 1994, at Columbus College in
Columbus, Georgia.

The issue will be the historicity of the resurrection of Jesus of Nazareth.

Volume 1990 - 2002 Issue


Page 363 of 2049
Skeptical Review Edited by Farrell Till

Video Library
In addition to video tapes of the Hovind-Till debate on the Genesis flood, subscribers to TSR
may still borrow tapes of the Dobbs-Till Debate on the issue of prophecy fulfillment.

To obtain the tapes for a two-week loan period, send $2 to cover the cost of packing and
mailing. As stated on page 10, the Hovind-Till tape can be rented for $1. Readers who are
involved in local access TV, may find them suitable for showing.

A New Column
With the Fall 1993 issue of the Secular Humanist Bulletin Farrell Till began a regular column.
Entitled "Journeys into the Twilight Zone," each article will discuss some aspect of the Bible
that only a "Twilight-Zone" mentality could believe. The columns will feature the type of
materials published in The Skeptical Review.

SHB is a 16-page quarterly bulletin published by CODESH (Council for Democratic and
Secular Humanism), the humanist organization that publishes Free Inquiry magazine. The
bulletin is sent to all members of CODESH. The annual membership fee is $15 (individual)
and $25 (family).

Article Submissions
If you wish to submit articles to TSR, please remember that they must in some way address
the issue of Bible inerrancy. We also require careful analysis of the inerrancy problems
addressed in the articles. We try to publish materials that our subscribers will be able to use in
their discussions with Bible inerrantists.

Volume 1990 - 2002 Issue


Page 364 of 2049
Skeptical Review Edited by Farrell Till

The Skeptical Review


Volume Five, Number Two
April/May/June 1994
Farrell Till, editor

• But If There Is No Tooth Fairy...


Theists never tire of painting skepticism and atheism as philosophies of despair and
doom. But it just ain't so, as this article shows.

• No Bastards allowed
The law of Moses barred those of illegitimate birth from entering into the assembly of
God, but this law was apparently not enforced when the descendants of "bastards" had
achieved important social status.

• Touring The Middle East, Jesus Style


A tremendous geographical "oops!" shows that either God works in very mysterious--
even absurd-- ways, or whoever wrote Mark did not know anything about the
geography of the Middle East and certainly was not a disciple of Jesus.

• Just Whose Legs Are Unequal?


In the first exchange between Farrell Till and Lindell Mitchell on the Amalekite
massacre, Mitchell accused Till of poor logic and evasion of the issue. But readers
should have no difficulty seeing whose logic is faulty.

• Mr. Till, Your Legs Are Unequal!


Lindell Mitchell continues to argue that "God stands ontologically on a higher plane
than man", and is therefore morally entitled to order acts of genocide. So much for the
moral superiority of Christianity.

• The Skeptic's Sword


Bill Lockwood claims that Farrell Till "fell on his own skeptical sword" when he
offered justifications for spreading atheism.

Volume 1990 - 2002 Issue


Page 365 of 2049
Skeptical Review Edited by Farrell Till
• As I Lay Dying
Farrell Till responds to Lockwood' article, "The Skeptic's Sword".

• From the Mailbag

But If There Is No Tooth Fairy...


Farrell Till
Theists never tire of painting skepticism and atheism as philosophies of despair and doom.
Immediately after publishing our belief that morality depends on neither God nor the Bible,
the cries of outrage began to arrive, the strongest of which was Bill Lockwood's article "The
Skeptic's Sword" which appears on pages 8-9 of this issue. The message is that it conveys is
typically theistic in outlook: without the security blanket of a god, life is a condition of utter
hopelessness.

The only argument the theist has to offer in support of this position appears to be, "If there is
no God, then..." The then will vary from "then there is no hope for life after death" to "then
there is no absolute standard of morality," but such statements are all rooted in a fallacy of
wishful thinking or belief that reality can be altered by personal hopes and aspirations.

Our response to this line of reasoning is what we have already said, "So what?" If there is no
life after death, so what? If there is no absolute standard of morality, then so what? No life
after death doesn't mean that one cannot make this life satisfying and meaningful. No standard
of absolute morality doesn't mean that we cannot use our intelligence to arrive at moral
principles that will create orderly societies in which we can work to make life satisfying and
meaningful. If theists disagree, then let them continue praying to the sky if it brings them the
"happiness" that Lockwood speaks of in his article, but even they will will have to admit that
all of the prayers uttered in this country haven't created a very orderly society. Our prisons are
filled with people who, if asked, would say that they believe in the god of the Christians, who
is presumably the source of the absolute morality that they have all flung down and danced
upon.

Nothing is true solely because one wants it to be true. Neither is anything true because an
overwhelming majority wants it to be true. If a man is arrested for murder, his mother
undoubtedly wants him to be innocent, but if he isn't innocent, then he isn't innocent no matter
how much the mother hopes that he is. Neither would a million people on the mother's side,
wishing that her son is innocent, make him innocent if he isn't. This principle is so patently
true that anyone should see it, yet millions are apparently unable to see it when the principle is
applied to religion. The faithful continue to say, "But if there is no God..."

To illustrate the absurdity of such thinking, let's apply it to a child who believes in the tooth
fairy. If upon hearing from an older sibling that there is no tooth fairy, what would the child

Volume 1990 - 2002 Issue


Page 366 of 2049
Skeptical Review Edited by Farrell Till
prove by saying, "But if there is no tooth fairy, then when I lose my teeth after I am grown, I'll
have to suffer for nothing"? The child's wish for the tooth fairy to be real would in no way
make the tooth fairy real. The same is true of the Santa-Claus myth. A child hearing for the
first time that there is no Santa Claus could not change reality by saying, "But if there is no
Santa Claus, then I won't get any presents when I'm big."

Mormons want the Book of Mormon to be a modern-day revelation from God, but outside of
the Mormon Church there are few who believe that it really is. In fact, Christian writers have
published tons of literature designed to prove that the Book of Mormon couldn't possibly be a
divine revelation. (Incidentally, they use arguments that, if applied to the Bible, will prove
that the Bible can't possibly be a divine revelation, either.) Moslems want the Koran to be
God's word, but not many Christians would agree that it is, no matter how many wishing and
hoping Moslems want it to be. All religious adherents want their particular holy books to be
God's word, but most Christians have enough common sense to realize that more than
wanting is necessary to make them God's word.

In such matters as these, Christians can all see the absurdity in believing that wishful thinking
can change reality, yet many appear unable to apply the same common sense to the religious
myths that they believe in. Biblical inerrancy, for example, is an obvious myth. We have
published more than ten times enough in The Skeptical Review to banish this myth to the land
of fairy tales, yet its adherents stubbornly refuse to accept reality. They go on believing that
the Bible is inerrant in everything that it says. They want it to be true, so they believe that it is
true.

The same principle applies to the happiness-in-Jesus theme song of Christian fundamentalists.
They have had it drilled into them for so long that they automatically equate lack of faith in
the Jesus myth with despair and gloom. They want skeptics and atheists to be miserable, so
they actually believe that they are. Our own personal experience, however, and what we hear
from those who have also experienced the transition from Christian to skeptic tell us that the
happinness-in- Jesus patter that rolls so effortlessly from Christian lips is just another
unwarranted religious assumption. Many tell us that their faith in Jesus made their lives
miserable.

A few months ago, we received a letter from a former Church-of-Christ preacher who had
learned upon reading his first issue of TSR that the editor is also an ex-preacher from the same
church. He said this about his present state of mind:

After several years of rationalism and realism, I am very comfortable in my beliefs, and this
had led to such peace and serenity that I cannot believe it. I am so much happier in my life.
Those around me, family and friends, constantly remark at how much more at ease I have
become and how much less driven I seem to be. I also feel myself to be a more moral person
now than I ever felt I was as a Christian.

It would appear from the reading of one issue of your newsletter, you probably know exactly
of what I speak. Sorry to bore you with with the needless details, but I am so happy with who
and what I now find myself to be that it's difficult not to share a little of my enthusiasm with
someone who understands it.

Volume 1990 - 2002 Issue


Page 367 of 2049
Skeptical Review Edited by Farrell Till
A longer quotation from this letter, along with the writer's name and address, appears in the
"Mailbag" column on page 13 if any "skeptical" Christian wants to verify its genuineness.

In addition to other letters that we could quote, we have some from writers who have asked us
to respect their privacy until their prepared to cope with the problems of being avowed
skeptics in an intolerant society. They all dispute the theistic claim that skepticism brings
despair and gloom.

No Bastards Allowed
Farrell Till
The law of Moses barred those of illegitimate birth from entering into the assembly of God:
"A bastard shall not enter into the assembly of Yahweh; even to the tenth generation shall
none of his enter into the assembly of Yahweh" (Dt. 23:2). Despite the clarity of this
statement, it was apparently not enforced when the descendants of "bastards" had achieved
important social status. Biblical genealogies show, for example, that David was a ninth-
generation descendant of Perez, the bastard son of Judah and Tamar (Gen. 38:24-30 ; Ruth
4:18 ; 1 Chron. 2:5-14 ). Obviously, though, David was not denied entry into the assembly
despite his descent from one who had been illegitimately born.

Bibliolaters will argue that David was exempted from the restriction, because, if Perez is
considered the first generation, then David was the tenth and was therefore eligible to enter
the assembly. The inerrantist argument is that the restriction extended even to the tenth
generation but did not include the tenth, so the fundamentalists think that they have scored
another victory in their never-ending quest to explain away biblical inconsistencies that we
pesty skeptics continue to point out.

Their explanation, however, is just another grasping for straws, because the expression in the
original Hebrew did not mean even to, as the following literal translation will show:

A bastard doth not enter into the assembly of Jehovah; even a tenth generation of him doth
not enter into the assembly of Jehovah (Young's Literal Translation of the Holy Bible).

The idea of to or until was not in the original. The expression, then, did not mean that
descendants of bastards were to be banished from the assembly up until but not including the
tenth generation. Its obvious intention was to denote a permanent, everlasting banishment.
Descendants of bastards were simply personae nongratae in Yahweh's sacred assembly, no
matter how long ago the indiscretion had occurred--except, as we have noted, in cases of
important social status.

We can expect bibliolaters to insist that the intention of the restriction on bastards was to ban
them only until the tenth generation, which would have allowed David to sneak in just under

Volume 1990 - 2002 Issue


Page 368 of 2049
Skeptical Review Edited by Farrell Till
the wire. For the sake of argument, we will concede them the quibble and then ask them to
explain why the descendants of Aaron, the first high priest of Israel, were not banned from the
assembly "up until" the tenth generation.

According to Exodus 6:23 , Aaron married a woman named Elisheba, who was "the daughter
of Amminadab, the sister of Nahshon." The significance of this can be seen when the
genealogy of the bastard Perez is examined. Amminadab was a fourth-generation descendant
of Perez (Ruth 4:18 ; 1 Chron. 2:5-9 ), so Aaron's wife was the fifth. Aaron's sons born to
Elisheba (Nadab, Abihu, Eleazar, and Ithamar, Ex. 6:23 ) were therefore sixth-generation
descendants of the bastard Perez, and they were all consecrated to serve as priests in the
assembly of Yahweh (Num. 3:3 ). This sounds very much as if they "entered into the
assembly of Yahweh."

Nadab and Abihu disgraced themselves and, in typical Yahwistic fashion, were killed (Lev.
10:1-2), but Eleazar figured prominently in priestly activities all through the 40 years of
wilderness wanderings. Bibliolaters cannot argue that the banishing of bastards from the
assembly occurred after the sons of Aaron were consecrated, because Eleazar served as priest
long after the events recorded in Deuteronomy, where the restriction on bastards was
stipulated. Eleazar is mentioned in priestly services all through Joshua and didn't die until the
very end of the book (24:33). At this point, Eleazar's son Phinehas (only a seventh-generation
descendant of Perez) assumed Eleazar's office and was serving in it as late as Judges 20:29. In
fact, Yahweh had conferred on Phinehas and his descendants "the covenant of an everlasting
priesthood" after Phinehas had thrust a spear through an Israelite man and a Midianite woman
whom he had caught in the act of adultery (Num. 25:6-13). On that occasion, Yahweh seemed
unconcerned about Phinehas's direct descent from the bastard Perez.

Maybe some inerrantist reader would like to explain to us why there is no inconsistency in
any of this.

Touring the Middle East Jesus Style


William Sierichs, Jr.
I recently went to Baton Rouge, Louisiana, from Alexandria, Louisiana, by traveling through
Shreveport and Orleans Parish. You don't believe me? You say it's a geographical
impossibility or else I took a very, very long route to make a short trip?

Well, Jesus did the same thing, according to Mark 7:31, and biblical literalists say everything
in the Bible is true. The passage says, "Then he (Jesus) returned from the region of Tyre, and
went through Sidon to the Sea of Galilee, through the region of the Decapolis" (RSV).

Volume 1990 - 2002 Issue


Page 369 of 2049
Skeptical Review Edited by Farrell Till
Geographically, that's a tremendous Oops! item. Both Tyre and Sidon are on the coast of
Lebanon. Tyre is northwest of the Sea of Galilee, and Sidon is north of Tyre! That's certainly
going the long way around to reach the Sea of Galilee from Tyre.

But it gets worse. According to a map of the first-century Middle East, a coastal road ran
south from Sidon to Tyre, then branched off with a road going to the Sea of Galilee. You
almost could not get from Sidon to the Sea of Galilee except by going through Tyre, not the
other way around.

But the funniest part was the adding of "through the region of the Decapolis." The map shows
the Decapolis as a region to the south(!) of the Sea of Galilee. From Tyre, you don't go to the
Decapolis except by passing the Sea of Galilee, so Jesus went to Baton Rouge from
Alexandria by going through Shreveport through Orleans Parish (from the center of the state
to the far northwest, to the far southeast, then finally to the near southeast). A 125-mile trip in
Louisiana would became a nearly 600-mile trip by my calculation. The middle Eastern
distances, of course, would be much shorter; then again, Jesus didn't have a T-Bird and
couldn't use I-49 or I-10 for his journeys.

Either god works in very mysterious--even absurd--ways or whoever wrote Mark did not
know anything about the geography of the Middle East and certainly was not a disciple of
Jesus. A charitable interpretation might be that "Mark" condensed a longer passage about
traveling around the Levantine coast and that, being ignorant of the geography, garbled his
source material. Less charitable views of this passage and other screw-ups of the laws,
culture, and geography of that particular time and place found in Mark raise serious questions
about the gospel. The possibility that it's entirely fictional can't be ruled out. And, with that, it
must also be noted that Matthew and Luke drew heavily on Mark, so this raises questions
about their authenticity too.

In Matthew 15:21-29, which corresponds roughly to the passage in Mark, the wording is
changed to vague statements that avoid the geographical absurdity in Mark's account. (Among
other things, Matthew changed "Sidon" to the "district of Tyre and Sidon" and left out the
reference to Decapolis.) Perhaps the author of Matthew knew more about Middle Eastern
geography than Mark's author, which doesn't mean that his material is accurate just because
he fudged a problem. Obviously, "Matthew" wasn't a disciple of Jesus either.

As for Luke, it appears to be missing the equivalent of two chapters of Mark (6:45 to 8:26),
which includes the geographical blooper. However, as G. A. Wells noted in Did Jesus Exist?
(p. 90), a new absurdity was introduced. The best explanation for Luke's omission is that he
used for his source a codex of Mark that had some missing pages. At that time, "Mark" had no
name and none of the system of chapter and verse numbers that we have now, so no one could
automatically detect a missing section.

Thus, "Luke" had two unconnected passages at hand, Mark 6:44 and 8:27, which he thought
were sequential. The result shows in the juxtaposition of Mark 6:44 and 8:27 with Luke 9:17-
19, which is the equivalent of the former as it would read with the missing section:

Volume 1990 - 2002 Issue


Page 370 of 2049
Skeptical Review Edited by Farrell Till
MARK 6:43-44; 8:27-28, And they took up twelve baskets full of broken pieces and of the
fish. And those who ate the loaves were five thousand men....

And Jesus went on with his disciples to the villages of Caesarea Philippi; and on the way he
asked his disciples, "Who do men say that I am?" And they told him, "John the Baptist; and
others say. Elijah; and others one of the prophets."

LUKE:9:17-19, And all ate and were satisfied. And they took up what was left over, twelve
baskets of broken pieces. Now it happened that as he was praying alone the disciples were
with him; and he asked them, "Who do the people say that I am?" And they answered, "John
the Baptist; but others say, Elijah; and others, that one of the old prophets has risen."

If Jesus were alone, how could the disciples have been with him? The answer could likely be
that in the version of "Mark" that Luke was using, the missing section left the impression that
Jesus had interrogated his disciples while they were yet in the "lonely place" (Mark 6:35,
RSV) where the multitude was fed. The interrogation of the disciples that follows Luke 19:18
is clearly derived from Mark 6:44 but under circumstances totally unrelated to the events in
Mark 6:44. In Luke, the interrogation follows immediately after 9:17 (the gathering of the
scraps left over from feeding the multitude), but in Mark it takes nearly two chapters and a lot
of wandering to arrive at this point.

Apparently, "Luke" was trying to clean up a mess without knowing how it had happened. He
must have gotten pretty frustrated at being unable to do better. We can only feel sorry for him.

As for "god," if he really inspired the gospels, he screwed up big time. You'd almost think the
Big Guy had never visited the Middle East. What can biblical literalists say about the errors
but "Oops!"

(The address of William Sierichs, Jr., is 1400 East 35th Street, Apartment 98, Texarkana, AR
75502.)

Just Whose Legs Are Unequal?


Farrell Till
In our first exchange on the Amalekite massacre, Lindell Mitchell accused me of poor logic
and evasion of the issue, but the readers should have no difficulty seeing whose logic is
faulty. As for evading the issue, Mitchell's article was a typical fundamentalist masterpiece of
evasion. My position on the Amalekite slaughter was clearly stated in my first article: "The
killing of just one Amalekite woman or child or infant, solely because of her, his, or its
nationality, would have constituted moral atrocity by any civilized standard of morality" (p.
2), if such an event as this did in fact happen. If, however, the massacre involved the killing of
not just one or two but hundreds of women, children, and babies, then its moral repugnance

Volume 1990 - 2002 Issue


Page 371 of 2049
Skeptical Review Edited by Farrell Till
should be all the more apparent, except, of course, to those whose sense of moral decency has
been anesthetized by blind allegiance to the doctrine of Bible inerrancy.

That hundreds or possibly even thousands were killed in the massacre cannot be denied
without admitting that biblical records are sometimes errant. I established the extent of Saul's
massacre by citing passages from Mitchell's own inerrant word of God that indicated
hundreds of Amalekites were killed by David's guerrilla forces after Saul had presumably
utterly destroyed all of the Amalekites except for their king, Agag. Mitchell seemed to
misunderstand my purpose in introducing David's military adventures. I didn't cite David's
massacre of the Amalekites in order to establish a discrepancy between this story and an
earlier one that claimed Saul had already "utterly destroyed" the Amalekites, although my
position is that there certainly is a discrepancy in the two accounts. My only purpose in
bringing David's massacre into the picture was to show that if hundreds of Amalekites were
alive for David to kill, then surely at the time of Saul's slaughter, which was described as an
utter destruction, there had to be several thousands of Amalekites. Of those thousands, surely
many would have been women, children, and infants. Hence, by implication, Saul's massacre
of the Amalekites entailed the slaughter of hundreds of women, children, and babies.

I asked Mr. Mitchell to tell us just how he is able to determine that the massacre of such a
large number of women, children, and babies could possibly be considered "morally proper,"
but, of course, he quickly informed us that he wasn't going "to chase that rabbit." He preferred
instead to chase straw men. The first straw man he went after was my position on abortion, as
if that had any- thing to do with the issue of moral atrocity in the Amalekite massacre.
Admittedly, I mentioned the anti-abortion stickers that Mr. Mitchell affixes to his letters but
only to suggest that they indicated a concern for children that is incompatible with his
position on the many massacres of children recorded in the Bible.

No doubt, he will argue that he was attempting to show an inconsistency in my position, but
he obviously doesn't even know my position on abortion. I once told him in a letter that I
didn't agree with his "pro-life" stance, but that doesn't automatically make me an advocate of
abortion. I am not an advocate of abortion; I am simply anti-prolife in the sense that I cannot
accept the dogmatism of those who look at the issue in the typically black or white way that
Christian fundamentalists assess most moral situations. To me, the fact of abortion is
regrettable, but something even more regrettable is the biological order on this planet, created
presumably by Mitchell's "Omniscient God," that contributes to the social necessity of
abortion.

Let's compare, then, my view of abortion with Mitchell's to see just who is being inconsistent
in the matter of the Amalekite massacre. Mitchell is apparently opposed to abortion, period.
In moral matters, everything is absolute to him. There are no gray areas; everything is either
black or white. So in the matter of abortion, the question is settled. Abortion is wrong at any
stage of pregnancy, and it would be murder to abort a zygote even ten seconds after the
moment of conception.

Yet he refuses to condemn Saul's massacre of hundreds, perhaps even thousands, of


Amalekite women, children, and babies whose lives were not at any stage of embryonic
development. They had already been born! Likewise, he refuses to condemn the many

Volume 1990 - 2002 Issue


Page 372 of 2049
Skeptical Review Edited by Farrell Till
terminations of pregnancy that would have necessarily occurred in the massacre of a complete
society experiencing normal population growth. In his mind, it is horribly immoral to
terminate the development of a day-old zygote that has no nervous system, no brain, no
organs, and has never drawn its first breath, never nursed its mother's breast, never heard its
mother's voice, and never taken its first step, but morally proper to kill hundreds of children
who had undeniably become living, breathing humans. And he had the gall to apply to me that
old saw that's so dear to Church-of-Christ preachers: "The legs of the lame are unequal." Yes,
they certainly are, aren't they? And I make that statement with my apologies to the physically
handicapped and my assurances that I make it not for purposes of ridicule but only to reply to
the sarcasm of my opponent.

Mr. Mitchell, of course, considered the statement appropriate because it is a biblical


quotation, just one among many, incidentally, that are derogatory to the physically
handicapped. Since he introduced it, I will pursue it just long enough to look at the statement
in the entirety of its context: "The legs of the lame are not equal; so is a parable in the mouth
of fools" (Prov. 26:7 ). The latter part of the statement is certainly applicable to Mr. Mitchell,
for only a fool would issue a blanket condemnation of abortion while extolling the virtue of
killing babies who had already been born at the time of their slaughter.

The second straw man Mitchell chased was my epistemological philosophy, which he has
apparently familiarized himself with by watching video tapes of my oral debates.
Epistemology is a high sounding word, but it simply means the branch of philosophy that
investigates the origin of knowledge. In that regard, I am an objectivist, so I believe that all
reality is objective and external to the mind and can therefore be perceived only through the
senses. To discuss epistemology would require much more space than a single article, so I
will have to limit myself to just a brief response to Mitchell's quarrel with this straw man.

Like his fundamentalist cohorts who seem to believe that they have a monopoly on truth, he
ridicules the fact that I have admitted in public debate that (1) the scope of my knowledge is
very limited in terms of all available empirical information and that (2) it is actually possible
for me to be wrong in the propositions that I publicly debate. If he wants to consider these
admissions indications of inconsistency, he is free to do so. I will simply leave it to the
readers to decide how much importance should be attached to such a remonstrance as this
from someone who believes that he is right in every detail of his religious beliefs and couldn't
possibly be wrong.

As for my objectivist philosophy, I challenge him to prove that I am wrong in saying that all
knowledge must be acquired through the senses. To do that, all he must do is tell us one thing-
-just one--that he knows that he did not learn through one or more of his senses. He will
probably label it another rabbit that he doesn't want to chase, but I will ask him a question that
I presented to two of his fundamentalist cohorts in other debates. They couldn't--or wouldn't--
answer it, so I don't expect him to either. Anyway, here it is. If a baby should be born
quintuply handicapped with an absence of all five senses, would it be possible for this person
ever to learn anything?

Mr. Mitchell's problem is that he cannot see the difference in knowledge and reasonable
certitude. I am reasonably certain that Napoleon Bonaparte was a real person, but I don't

Volume 1990 - 2002 Issue


Page 373 of 2049
Skeptical Review Edited by Farrell Till
actually know, in an absolute sense, that he was. I didn't exist when he did, so I never saw
him, heard him, felt him, etc. to know absolutely that he was real. Although extremely
unlikely, it is entirely possible that Napoleon didn't exist and that a colossal hoax was
perpetrated, in the same way that the Jesus hoax was perpetrated, to make people of
succeeding generations believe that Napoleon was a real person.

All of this is really beside the point, because it doesn't address the issue we are supposed to be
debating. If the Israelite massacre of the Amalekites actually happened--and I am not saying
that it did--then it constituted moral atrocity. That is all that I am arguing. I am not arguing
that it did happen in the manner described in the Bible but that if it did so happen, it must be
regarded as a moral atrocity. To argue otherwise is to argue that it is morally right to kill
babies for no other reason than ethnic origin.

Mitchell's only defense, of course, is that "the Omniscient God" looked down "the corridors of
time" and saw that "the babes of Amalek were destined to become vicious beasts like their
ancestors," and so this wonderfully merciful God of his ordered the slaughter of these
innocent "babes" so that they could fly away to "the protection of his [God's] holy city" rather
than growing up to become "vicious beasts" like their parents. Didn't I predict that he would
resort to this? So now that he has retreated to this ignominious position, let's look at the
problems in it.

ONE: God is both omniscient and omnipotent, but he was unable to solve a problem except
by resorting to the massacre of babies.

TWO: To keep them from growing into vicious beasts, the Israelites could have taken the
"babes of Amalek" back as captives and raised them as Yahweh- fearing Hebrews. There are
ample precedents in the Bible for rearing captives as Jews--the law of Moses even made
specific provisions for it (Dt. 21:10-14 )-- so no divine laws would have been violated, but
apparently Mitchell's omniscient God didn't have the intelligence to think of this bloodless
alternative.

THREE: It assumes without proof that the Amalekites were indeed "vicious beasts," but I
would like for Mr. Mitchell to explain exactly how he happens to know this to be the case. A
study of all biblical references to the Amalekites will reveal that they were no worse than the
other nations of that time and area, including even the Israelites. So Mitchell needs to cite the
source of information that he relied on to determine that the Amalekites were "vicious beasts"
who deserved to die.

He cited the Amalekite attack on the Israelite stragglers during their passage through
Amalekite territory at the time of the exodus (Ex. 17:8-16 ) and the Amalekite blockade of the
entry into the promised land (Num. 14:45 ) as justification of the massacre in 1 Samuel 15 ,
but there is nothing in either passage to prove that the Amalekites were "vicious beasts." In
the first place, both of these events, if they happened, would have been normal defensive
actions by a nation concerned about three million foreigners coming into its territory.
Furthermore, the Amalekite blockade in Numbers 14:45 in no way impeded the progress of
the Israelites, because Yahweh, according to this fanciful little yarn, had already decreed that
the Israelites would not be allowed into Canaan because of their fear and rebellion upon

Volume 1990 - 2002 Issue


Page 374 of 2049
Skeptical Review Edited by Farrell Till
hearing the report of the spies who had come back with tales of a land inhabited by "men of
great stature" (Num. 13:32-33 ; 14:1-23 ). So what was so terribly wrong about blocking the
entry of the Israelites into a land that Yahweh wasn't going to allow them to enter anyway?

For the sake of argument, however, let's just concede to Mitchell that these were both
despicable acts that made the Amalekites "vicious beasts." This would prove only that the
Amalekites of that time were "vicious beasts," but the Amalekites that Yahweh ordered Saul
to utterly destroy lived about 450 years later. So what would be moral about killing an entire
nation of people--women, children, and babies included--for something their ancestors had
done 450 years earlier? Yet the Bible states that this is exactly why Yahweh

commanded Saul to destroy the Amalekites: "Thus saith Yahweh of hosts, I remember that
which Amalek did to Israel , how he laid wait for him in the way, when he came up from
Egypt. Now go and smite Amalek, and utterly destroy all that they have, and spare them not;
but slay both man and woman, infant and suckling, camel and ass" (1 Sam. 15:2-3 ). To save
his life, Mitchell could not find a single reason stated in this passage to explain why the
command to slaughter the Amalekites was given, except for the reference to the Amalekite
attack on Israel 450 years earlier. It was purely and simply a matter of longstanding grudge, if
we are to believe the Bible. That is certainly a high plane of morality, isn't it?

It is time, then, for another question for Mitchell to evade. I asked him one in my first article,
but he refused to answer it on the grounds that he is not the affirmant. I have found in my
correspondence with him that he just doesn't like questions, period, because he has yet to
answer any I have submitted to him. Maybe I will have better luck with him this time, so here
goes. Will Mr. Mitchell please tell us why it would be moral to exterminate an entire nation--
women, children, and babies included-- for something that its ancestors had done 450 years
earlier? Now that rabbit shouldn't be too fast for him to chase for a while. He has an audience
of several hundred freethinkers that he will not likely have available to him too often, so I
would think that he would welcome the opportunity to enlighten them on a matter that many,
in his opinion, are confused about.

As he considers whether he wants to chase this rabbit or not, maybe he will want to think
about just how long 450 years would be. If we had a time machine to take us back 450 years,
we would have to wait on the coast of Virginia for 63 years before we could welcome the first
permanent settlers at Jamestown. I know that I am just a lowly, unenlightened atheist, but it
seems to me that it just wouldn't be right to punish people for something done by their
ancestors that long ago. I really don't think the citizens of this country would tolerate it if our
army should attempt to massacre all known descendants of any Native Americans who may
have offered resistance to the Jamestown settlers. I don't think Mr. Mitchell would approve it
either, yet he apparently thinks that it was morally right for his god to order such a massacre.
So whether he realizes it or not, he has a lot of explaining to do to us stupid freethinkers. He
can talk about chasing rabbits all that he wants to, but if he evades this issue, some of us just
might think that he has no reasonable explanation to give.

FOUR: Justifying the Amalekite massacre on the grounds that they were "vicious beasts"
merely has one nation of vicious beasts exterminating another nation of vicious beasts, so
what moral good did Yahweh expect to achieve from such a swap off as this? According to

Volume 1990 - 2002 Issue


Page 375 of 2049
Skeptical Review Edited by Farrell Till
the Bible, from the time that Yahweh brought the Hebrews out of Egypt, he had problem after
problem with their morality. In the book of Judges, Yahweh was constantly punishing the
Israelites with foreign domination for their iniquity, and this continued throughout their
history, until finally (according to the Bible) Yahweh permanently destroyed the nation of
Israel. There were continual problems with idolatry, and even human sacrifices were
commonplace (Ps. 106:37-38 ; Ez. 16:20-21 ; 2 Kings 17:17 ). So if there is any merit at all to
Mr. Mitchell's "explanation" of Yahweh's reason for ordering the Amalekite massacre, he has
a duty to explain to us the justice in using one ungodly nation to eradicate another.

FIVE: The massacre of the Amalekites violated Yahweh's own edict that said iniquity would
be borne by the one who committed it, not his descendants (Ez. 18:20 ; Dt. 24:16 ). Whatever
the Amalekites may have done at the time of the exodus, their descendants during the reign of
Saul, 450 years later, were not responsible for it. Certainly, the children were not guilty of
anything, yet if Mitchell is right, they bore the iniquity of their fathers in a horribly brutal
way.

SIX: Mitchell's position exempts Yahweh from the absolute moral law that presumably
exists. Certainly, it is clear from Mitchell's article that he believes in the existence of an
absolute morality, but how could it be an absolute morality if Yahweh can violate it at whim?
Perhaps this is another rabbit that Mitchell won't want to go after, but many of the readers
would like to see him give it a chase.

This finally brings us to the crux of the matter. Unless he is an unusual fundamentalist,
Mitchell would never say that it would be morally right for any army to massacre civilian
populations. No, certainly not; it would have to be "God's army" acting in a "holy war" under
direct orders from God. In that event, nothing done--no matter how grisly-- could be
considered moral atrocity, because the "Omniscient God" would be directing it all. The
problem with this scenario is that it consists of nothing but assumptions.

ONE: It assumes first of all that this "Omniscient God" even exists, but before anything in
Mitchell's rebuttal can stand, he must prove that there actually is an "Omniscient God." This,
of course, he can never prove. All arguments for the existence of God (ontological,
teleological, cosmological, etc.) are illogical and have been answered hundreds of times.

TWO: It assumes that the "Omniscient God" is morally good, but there is no logical reason
why that would have to be true. Even if an omniscient god did exist, he would not by logical
necessity have to be infinitely good. It would be just as possible for him to be infinitely evil.

THREE: It assumes (after assuming the first two points) that the "Omniscient God" has
revealed absolute morality to mankind, but there is no logical necessity for an omniscient god
to reveal absolute morality. If Mitchell thinks that his "Omniscient God" did make such a
revelation, he is obligated to prove it before he criticizes my position on the grounds that it
excludes the existence of absolute morality.

FOUR: It assumes that Yahweh of the Hebrews is the "Omniscient God," and that would not
logically follow from any proof that an omniscient god does really exist. In fact, many

Volume 1990 - 2002 Issue


Page 376 of 2049
Skeptical Review Edited by Farrell Till
biblical statements about Yahweh could be cited to prove that if an omniscient god does
indeed exist, Yahweh could not be that god.

FIVE: It assumes (after assuming that Yahweh is the "Omniscient God") that Yahweh
actually did order the massacre of the Amalekites. This would not necessarily follow from
proving that Yahweh is the omniscient god, because it would have been possible--not even to
mention probable--that the writer of 1 Samuel 15 just mistakenly thought that Yahweh had
ordered the massacre.

In typical fundamentalist fashion, Mitchell depicted my position as an utterly hopeless one,


because he has absolute morality on his side and I don't. As the above points demonstrate,
however, Mitchell has exactly nothing on his side until he proves unequivocally that absolute
morality exists. Without proof of that, he has nothing but conjecture to offer in support of his
position, and conjecture proves exactly nothing.

He presented a scenario of everything that he could do to me personally-- beat me up, spoil


my goods, kill my children, ravage my wife, and mercilessly torture me--and yet be free of
moral guilt unless there is this thing that he calls "absolute morality." As I read this, I
wondered just how Mitchell expects to convince rational thinkers that his position on morality
is superior to mine. I don't believe that objective (absolute) morality exists, yet I would never
even consider doing any of those things to him. So I have to wonder why, if he should
suddenly realize that objective morality does not exist, he would want to do them to me. Is he
trying to say that he needs some spook-in-the-sky to restrain him from torturing and killing
his fellow man. If so, does he seriously consider this to be a moral position superior to mine?

As ridiculous as Mitchell's position on the Amalekite massacre is, his belief that "(i)t is
impossible to have a moral atrocity in the absence of an objective moral standard against
which to measure thoughts, words, and deeds" is even more ridiculous. In the first place, his
own inerrant "word of God" clearly recognizes that man is capable of making correct moral
decisions without divine enlightenment. In Romans 2:14 , the Apostle Paul referred to
Gentiles who "have not the law" [special revelation] but who nevertheless "do by nature the
things of the law" and are therefore "the law unto themselves." If this doesn't mean that the
chief architect of Mr. Mitchell's religion thought that correct moral decisions could be made
without receiving moral revelations from God, then perhaps Mitchell can chase this rabbit just
long enough to tell us exactly what Paul did mean.

So if Mr. Mitchell wants my authority for classifying as a moral atrocity the killing of babies
nursing their mothers' breasts, he now has it. It is the ability of human beings, by virtue of
common- sense intelligence, to become a moral law unto themselves. That common-sense
intelligence is sufficient for any reasonable person to understand that it is wrong to kill
children and babies solely because of their ethnic origin. The matter is as simple as what I
said in my first article: If killing hundreds of children and babies for no reason but their
ethnic origin is not wrong, then nothing is! If this answer doesn't please Mitchell, I will just
let him argue with his own beloved Apostle Paul.

Mitchell's position on morality is essentially the same as the one that Bill Lockwood takes in
his article on pages 8-9 of this issue. My response to it (pages 9-11,16) clearly shows that

Volume 1990 - 2002 Issue


Page 377 of 2049
Skeptical Review Edited by Farrell Till
even without the existence of objective morality, human intelligence can make proper moral
judgments.

If Mitchell will bother to investigate the facts on this subject--and I seriously doubt that he
will--he will find that human societies formulated moral concepts long before Moses
allegedly trekked up Mount Sinai. The moral principles of the ten commandments existed in
Egypt centuries before the formation of the Israelite nation. They existed in the Code of
Hammurabi, which was written long before the time of Moses. They existed in the
Zoroastrian cultures of ancient Persia. So what is Mitchell's explanation for the pre-Mosaic
existence of moral principles in non-Hebraic societies? Will he argue that the god Horus
revealed to the Egyptians an absolute standard of morality and that Marduk did the same for
the Babylonians? We sincerely hope that he will chase this rabbit long enough to enlighten us
on the matter.

So far Mitchell has refused to tell us what he would have done had he been born an Israelite
in the time of Saul. Would he have willingly and gladly participated in the Amalekite
massacre? Would he have willingly and gladly killed Amalekite women, children, and
babies? Would he have ripped open any pregnant women he encountered, as was the
postbattle custom in those days? If not, why not? Why would he not willingly and gladly do
the will of God?

Until he answers these questions, I am afraid that many of our readers will remain suspicious
of his sincerity. It is one thing to preach theory; it is quite another to practice it. So we want to
know what he honestly thinks his involvement in the Amalekite massacre would have been if
he had had the opportunity to be a participant.

If he doesn't want to chase this rabbit, maybe he will at least fire a shot at it.

Mr. Till, Your Legs Are Unequal!


Lindell Mitchell
My first response to Mr. Till ended with these words: "Perhaps he is saving the best for last.
We shall see." It is now clear that he wasn't! Thoughtful readers know he failed to substantiate
his assertion. He kept up an effort to pull me into a discussion of irrelevant questions.
However, posing such questions did not establish Till's case. As the affirmative disputant, it
was not his place to pose questions. He was obligated to show that Israelite conduct in 1
Samuel 15 constituted a moral atrocity. This he failed to do. If Mr. Till had been able to
sustain his affirmative, he would have been on me like ugly on an ape. Instead, we were
served a large portion of subterfuge--Till style. His article shows the desperation of a defeated
fighter. He lashes out wildly, but the blows have no power. Till lost the first round and knows
it.

Volume 1990 - 2002 Issue


Page 378 of 2049
Skeptical Review Edited by Farrell Till
I thought we were finally going to engage the disputed issue when I read the opening
paragraph of Mr. Till's second "affirmative." He clearly stated his position: "The killing of
just one Amalekite woman or child or infant, solely because of her, his, or its nationality,
would have constituted moral atrocity by any civilized standard of morality." Where is the
unequivocal proof? Where is the objective quantitative data? He offers none! He shows us no
ruined cities, no skeletal remains, no bloody weapons, nothing! He simply makes the assertion
that Israel's conduct constituted a moral atrocity and hangs it on air. His statement grossly
distorts reality. Not one Amalekite was killed solely because of nationality. The Amalekites
were a persistently wicked people; consequently God ordered their execution.

Till's next piece of duplicity was especially juvenile. He claims he was only trying to establish
the extent of Saul's massacre by citing instances where David's guerrilla forces killed
hundreds of Amalekites after Saul had presumably utterly destroyed them. He says that I
mistook his introduction of this material for an attempt to show discrepancies in the biblical
record. Till says he was trying to show that there were a lot of Amalekites in Saul's day. Mr.
Till must think his readers are idiots. No one but Mr. Till disputes that Saul killed thousands
of Amalekites. There is no need to prove it to me. Mr. Till must prove the events of 1 Samuel
15 constituted a moral atrocity. He must produce objective quantitative data to substantiate his
allegation.

In the first exchange, I explained the presence of Amalekites after the events of 1 Samuel 15.
Saul disobeyed God in sparing the life of king Agag. Who knows what else he purposely
neglected to do in executing his orders? Amalekite military units that were out of the country
when Saul descended upon the nation would have escaped destruction. Diplomats on foreign
assignment, and businessmen who were abroad at the time, would also have been spared. Mr.
Till failed to notice these explanations because they blew his quibble apart. Furthermore, the
presence of Amalekites after the events of 1 Samuel 15 does not establish Mr. Till's position.
He is obligated to prove those events constituted a moral atrocity. He must produce objective
sensory data that unequivocally proves his assertion in the absence of an objective standard of
morality.

My refusal to chase Till's rabbits elicited this clever response: "He [Mitchell] preferred
instead to chase straw men." The first alleged straw man was Till's position on abortion. He
opined, "[A]s if that had anything to do with the issue of moral atrocity in the Amalekite
massacre." Since Mr. Till introduced his abortion stance, I noted his glaring inconsistency. He
will not allow God to punish a wicked nation but defends killing inconvenient infants. The
Omniscient God cannot determine that the infants of Amalek were destined to pursue the
wicked course their ancestors relentlessly followed for 450 years. Yahweh could not judge the
wicked and call the innocent into his matchless presence, but it is morally upright to slaughter
25-30 million Americans by abortion. He is horrified by God's justice but supports death by
dismemberment via the dilation and curettage abortion procedure. He defends covering
infants with a corrosive, forcing it into their lungs and stomach, and leaving them to convulse
for hours until death mercifully comes. This is the result of a salt-poisoning abortion. This
man claims to have such respect for all life that he refuses to squash a bug.

Mr. Till refuses to concede that God stands ontologically on a higher plane than man, which
qualifies him to justly demand actions that no man could require. Despite placing such

Volume 1990 - 2002 Issue


Page 379 of 2049
Skeptical Review Edited by Farrell Till
restrictions on God, Mr. Till condones the brutal death of over 400 infants in this country
every day. He is afraid the population will get so large that he will not have enough to eat.

Reeling under the weight of his glaring inconsistencies, this lame defense was offered: "I am
not an advocate of abortion; I am simply anti-prolife in the sense that I cannot accept the
dogmatism of those who look at the issue in the typically black or white way that Christian
fundamentalists assess most moral situations." This is an incredible piece of imbecilic
nonsense. One cannot be "simply anti-prolife" without being pro-abortion. Till attempts to
dodge the bullet by saying it is God's fault for creating a biological order that contributes to
the social necessity of abortion. This is asinine sophistry. Abortion is not a social necessity.
Parents who have their children killed by abortion overwhelmingly do so because the birth
would be an inconvenience or prove embarrassing. Besides, Till claims God doesn't exist and
that evolution produced the biological order.

Mr. Till shows his temper when he roars in disgust at my citation of Solomon's words: "The
legs of the lame are unequal" (Prov. 26:7a ). He shamelessly panders to the "victim classes,"
as a "politically correct" professor. Solomon's words are not derogatory to physically disabled
people. It is a statement of fact. It no more demeans physically challenged people than saying,
"The eyes of the nearsighted are weak" denigrates me.

Mr. Till really warms up when he says, "... only a fool would issue a blanket condemnation of
abortion...." This is a blatant violation of his agreement to maintain civility in our exchanges.
In polite society one does not refer to another as a fool. Mr. Till could not establish his case.
He was incapable of handling the material I presented. If I am a fool, he is less than a fool.
Solomon was right; the legs of the lame are unequal!

Another alleged straw man was Mr. Till's epistemological posture. He believes knowledge is
restricted to what is gained through personal sensory experience. That posture cuts him off
from any evidence resulting from testimony. He has no literary historical past. I noted his
brazen lack of consistency in this regard. He readily appeals to history when it appears to
support his cause. However, when an opponent appeals to history, Till hides behind his false
epistemology insisting on quantitative data as unequivocal proof.

I must plead guilty to chasing a straw man. His name is Farrell Till of Canton, Illinois. He
makes bold assertions he cannot consistently sustain within his own philosophical construct.
He had two opportunities and abysmally failed in both attempts.

Mr. Till attempted to put a Band-Aid on the hemorrhage caused by his "objectivist
philosophy." He wanted me to tell him one thing I have learned that did not come through one
or more of my senses. It is not his place to ask questions as the affirmative disputant. His
responsibility is to produce objective evidence that proves Israel's conduct constituted a moral
atrocity. He must do this in the absence of an objective standard of morality. However, I will
humor him. First, I have never said that you can learn in the absences of using the five senses.
However, knowledge is not restricted to sensory data. Information coming through the senses
and processed by the mind can result in knowledge that transcends sensory experience. I
know that I have a conscience that causes guilt when I violate it and happiness when I behave
properly. In his debate with Mac Deaver, Mr. Till said he also has a conscience. I assure you

Volume 1990 - 2002 Issue


Page 380 of 2049
Skeptical Review Edited by Farrell Till
he has never tasted it. He has never touched it, heard it, smelled it, or seen it, but he knows he
has one. Indeed he does, but knowing that is inconsistent with his epistemological posture.
Consider another example. If I watch someone eat a hamburger and immediately get up and
walk across the street, I know that the hamburger is across the street!

Mr. Till complains that my only defense of the events in 1 Samuel 15 rests on the fact that
"the Omniscient God looked down the corridors of time and saw that the babes of Amalek
were destined to become vicious beasts like their ancestors." He then writes, "Didn't I predict
that he would resort to this?' Till seems to think simply stating matters he is unable to answer
negates their force. However, my statement is more than enough to sink his ship.

Till says I retreated to an "ignominious" position fraught with insurmountable problems. First,
he says, "God is both omniscient and omnipotent, but he was unable to solve a problem
except by resorting to the massacre of babies." Mr. Till assumes God is unable to judge the
situation fairly and punish the wicked. He assumes the giver of life has no right to terminate
life. He assumes the Creator has no right to call innocent Amalekites into the protection of
paradise. Mr. Till is obligated, given his epistemology, to produce objective sensory evidence
that unequivocally proves this.

Second, Till says Israel should have adopted the Amalekite children to prevent their following
an evil path. He says provision was even made for this in the law (Dt. 21:10-14 ). According
to Till no divine laws would have been violated. As usual he is wrong. Deuteronomy 21:10-
14 makes provision for a Jewish soldier to marry a captive woman. It says nothing about
adopting foreign children. Further, a divine order would have been broken had this approach
been taken. The order to execute the Amalekites did not allow for adoption. Saul ultimately
lost the kingdom because he failed to carry out God's instructions. Mr. Till is obligated to
show that Israelite conduct in 1 Samuel 15 constituted a moral atrocity. He must present
objective quantitative data in the absence of any objective moral standard that logically
sustains his assertion.

Third, Till says I assume without proof that the Amalekites were vicious. He says passages
showing them to be bushwhackers, cutthroats, and murderers are insufficient. Their
ambushing the Jews in an unprovoked attack is insufficient for Mr. Till. He asks why God
would punish the Amalekites for something their ancestors did 450 years earlier. In typical
Till fashion, he abuses the context of Numbers 13-14 , refusing to acknowledge the
Amalekites had over four centuries to repent but instead had continued to be an unrelenting
threat to God's people. He is obligated to unequivocally prove the Amalekites were not vile
savages worthy of death. Let him produce quantitative data proving they were enlightened
"free-thinkers" who would not even step on a bug.

Four, Till says it is problematic for God to use an imperfect nation to punish another
imperfect nation. He is obligated to prove it constitutes a moral atrocity, citing objective
sensory evidence in the absence of an objective moral standard. He assures readers that he is a
great Bible scholar. If that were true, he would be familiar with the prophetic literature. He
would know whining Jews lodged this same objection when they were taken captive by
Assyria and Babylon. God's response was, "I'm not through yet, they shall be punished." He
would know God did punish Assyria and Babylon.

Volume 1990 - 2002 Issue


Page 381 of 2049
Skeptical Review Edited by Farrell Till
Five, Till says the massacre of the Amalekites violated Yahweh's own edict that said iniquity
would be borne by the one who committed it, not his descendants (Ezek. 18:20 ; Dt. 24:16 ).
It did not. No Amalekite bore the iniquity of even one sin committed by an ancestor. They
bore the result of their ancestors' rebellion but not the guilt. That happens every time car-
jackers kill an innocent driver. The victims bear no guilt for the crimes committed against
them, but they obviously bear the consequences. This is the price humanity pays for the
ability to choose between good and evil. Till also denies that God stands ontologically on a
higher plane than man, which authorizes him to require what no man could. He is obligated to
unequivocally prove this with objective data in the absence of an objective standard of
morality.

Though I am not affirming anything in this discussion, Mr. Till cites problems with an
affirmation I did not make. You have noticed that he has not produced one tiny particle of
objective quantitative data that unequivocally proves his position. As the affirmative
disputant, it is his responsibility to establish his case in a manner consistent with his
philosophical position.

The first problem he sees with the affirmative I did not make is that it assumes the existence
of the "Omniscient God." He avers that my rebuttal can never stand until I prove God. He
says "all arguments for the existence of God (ontological, teleological, cosmological, etc.) are
illogical and have been answered hundreds of times." We are not debating the existence of
God, and I am not in the affirmative. Mr. Till assumes God does not exist, and his affirmative
can never stand until he disproves God's existence with objective quantitative evidence. He is
unable to do this. He can't even prove a moral atrocity occurred in 1 Samuel 15 . The
ontological, teleological, cosmological arguments for the existence of God are not illogical,
and none of them has ever been successfully refuted.

Second, Till says the affirmative I never made assumes the "Omniscient God" is morally
good. He says there is no logical reason to conclude that God is good. He is wrong. The
existence of Good implies God's existence. Man did not create it. Man would never articulate
principles restraining his impulses as God did. The gods man creates for himself are worse
than he is. Things do not improve when man deifies himself. Till assumes God is not morally
good. This must be proved unequivocally by objective data.

Third, Till says the affirmative I did not make assumes the "Omniscient God" has revealed
absolute morality to mankind. He says there is no logical necessity for an omniscient God to
reveal absolute morality. God is sovereign and does what is consistent with his nature. Mr.
Till assumes God has not revealed absolute morality to man. Given his epistemological
stance, he is now obligated to prove it, citing objective evidence logically compelling the
conclusion. Trying to project the negative disputant into the affirmative does not establish his
case. Only empirical evidence will do that.

Fourth, Till says the affirmative I did not make assumes Yahweh of the Hebrews is the
"Omniscient God," and that it would not logically follow from any proof that an Omniscient
God does really exist. He further asserts that "many biblical statements about Yahweh could
be cited to prove that if an omniscient god does indeed exist, Yahweh could not be that god."
Mr. Till assumes Yahweh of the Hebrews is not the "Omniscient God," and that it does not

Volume 1990 - 2002 Issue


Page 382 of 2049
Skeptical Review Edited by Farrell Till
logically follow from any proof an omniscient God does really exist. No objective evidence
supporting his assertion was produced. Till says many passages in Scripture prove Yahweh
could not be the "Omniscient God." Where are the passages? Besides, Mr. till is bound by his
epistemological stance to produce sensory data unequivocally proving his assertion.

Five, Till says the affirmative I did not make assumes Yahweh actually did order the massacre
of the Amalekites. He assumes Yahweh did not actually order the massacre of the Amalekites.
He is obligated to produce objective measurable evidence unequivocally proving his
assertion. Mr. Till did not appreciate my illustration of what I could do to him with impunity
if there were no objective moral standard. He assures us the "free-thinking" crowd he runs
with would never think of such conduct. He told the people in Portland, Texas, he will not
even step on a bug. Chairman Mao was a "freethinking" objectivist, so was Joseph Stalin, and
they murdered millions. Till refuses to step on bugs but supports the daily killing of 4,000
infants through abortion. His quibble is irrelevant. The point is that there would be no
intellectual basis to charge me with any crime if I chose to severely beat Mr. Till in a world
without an objective standard of morality.

As ridiculous as he thinks my position on the Amalekite massacre is, my belief that it is


impossible to have a moral atrocity in the absence of an objective moral standard against
which to measure thoughts, words, and deeds is even more ridiculous to him. He says the
Bible concedes man's ability to make correct moral decisions without divine revelation.
Romans 2:14 is cited as support for his assertion. Till is wrong. Romans 2:14 does not
substantiate his position. The first three chapters of Romans are devoted to a discussion of
man's universal need for salvation. In chapter one, Paul shows that the Gentiles are guilty
before God. In chapter two, he shows that the Jews are guilty too. In chapter three, he shows
that everyone is guilty. Consequently, everyone is in need of salvation. Any interpretation of
the passage must be consistent with the context.

Paul is not saying the Gentiles made "correct moral decisions" without an objective moral
standard. The law the Gentiles did not have was Moses' Law. Nevertheless, because they were
created in God's image, they had a sense of oughtness. Further, they were subject to God's
moral law that became operative in the Garden and continues until Judgment. It reflects the
same moral verities as Moses' Law.

When Adam and Eve partook of the forbidden fruit, they became aware of good and evil.
With that knowledge came the responsibility to choose the good. This knowledge was passed
from generation to generation. To the extent that early societies reflect proper moral choices,
they evidence compliance with God's initial moral law. To the extent they reflect improper
choices, they evidence rebellion.

Mr. Till has huffed and puffed; he has romped and stomped; he has howled and hollered; but
he has not produced one microbe of objective evidence to sustain his affirmative. Truly, it is
one thing to preach theory and quite another to practice it.

(Lindell Mitchell's address is printed after his letter on page 15.)

Volume 1990 - 2002 Issue


Page 383 of 2049
Skeptical Review Edited by Farrell Till

The Skeptic's Sword


Bill Lockwood
Farrell Till, the turncoat preacher of skepticism, has finally made an effort to justify his
"evangelism." His favorite ploy, of course, is to attack the Bible as a collection of fairy tales
spliced together by man over the centuries. But a number of us have asked him, "Why
bother?" If skepticism--doubting the existence of God--is true, what difference does it make?
Actually, Till's faith is atheistic for he claims there is no evidence for the existence of God.
Again, what benefit shall a man receive for believing what Till or any other skeptic preaches?
Now, he comes forth from hiding among the baggage of criticism to justify or give reasons
for his efforts to spread "the devil's gospel." In so doing, he has fallen on his own skeptical
sword, to use another illustration from the life of Saul.

Till speaks about the "intrinsic value of truth" (TSR, Autumn 1993, p. 1) and asserts that
"Bible believers lack truth." There is "benefit" in "confronting reality." Bible believers not
only "die for nothing," if they die for their faith, but we "waste time" by preaching Jesus. Till
says he "must oppose the Bible for the damage it has done," and besides "truth is gratifying"
(p. 15). He concludes that we "must face reality," for only that will "make you happy."

Truth Is Gratifying?
By saying that "truth is gratifying" and urging us to "face reality" for it will "make us happy"
to do so, Till implicitly assumes a standard--happiness. If we shall reach gratification, we
must face "reality," which, of course, per the professor, means that there is no evidence for the
existence of God.

But this is a subjective standard. What makes one man "happy" does not necessarily make
another happy. "Some may like to help other people and others may enjoy torturing them"
(James Bales, Darwinism, the Devil's Gospel, p. 39). Happiness, in itself, has no moral
content. For example, Till pursues his skeptical dream, because "truth is gratifying" and
"makes him happy," but what if I enjoyed inflicting pain to the bodies of others? One man's
meat is another man's poison, you know. "After explaining away morality by equating it with
happiness, evolutionists [and skeptics, BL] usually then contradict themselves and establish
some sort of moral standard by which to determine what is true or good happiness and what is
bad" (Ibid.).

Second, besides the subjectivity of the standard, if happiness is the goal, there is no ground
upon which to condemn or even criticize the individual who seeks his own pleasure and
preservation at the expense of others. In fact, per Till's logic of "happiness" being the chief
goal, he could not any longer even chastise any group of people that purposefully utilizes
religion to control or master others, let alone the thousands of members of the Lord's church
who go about seeking to do good. Pseudopreachers who are actually powerbrokers, who have

Volume 1990 - 2002 Issue


Page 384 of 2049
Skeptical Review Edited by Farrell Till
indeed plagued society, could not even be consistently criticized by poor Till, since power is
what makes them happy, not seeking "truth." Till cuts his own atheistic heart when he says
"religion ruins lives." Professor Till, upon what basis do you pass such judgment, since
"ruining lives" may be that which gives the most pleasure to so many? Will you tell us now
that we "ought" to seek truth as a goal and find that pleasurable? Will you, as all atheists do,
ascend the judicial bench and give us an edict about what things "ought" to make us happy?

Third, if happiness is the goal of life, and I find happiness in believing the Bible, even though
it is a lie, why have I not achieved life's goal? Why bother with your Skeptical Review ?
Leave me alone, leave the world alone--we don't need you now to come along and make us
unhappy by exposing the fraud in which we have staked our existence.

Fourth, "[i]f the truth about life is that it has no lasting meaning or purpose, and dust is its
destiny, happiness will be found only when one forgets the truth about life" (Bales, p. 44).

Fifth, if pleasure is the ultimate goal, why would Till wish to rescue "Bible believers who die
for nothing"? Till uses the Waco tragedy to illustrate. Setting aside whether or not they were
real "Bible believers," I ask Till, "If those in Koresh's compound were HAPPY dying for what
they believed, why is it a less worthy goal than you have found in skepticism?" This is funny.
In one breath, Till wishes me to believe happiness is the chief goal, but in the next he wants to
mandate to us that there are "good" happinesses and "bad" happinesses. Indeed, those who
reject the one true God wish to become God for the rest of us. Till writes, "all those people
[Koresh's followers in Waco, Texas] died for a belief that wasn't true." So what, my skeptical
friend, if it made them HAPPY?

Till next ponders "how much better off they would have been had they been exposed to
papers like The Skeptical Review in their formative years." Yes, readers, there he goes again,
assuming his little skepticism and its brand of happiness is "better" than Koresh's. But the fact
is, upon Till's standard, Koresh's duped followers are on equal footing with Till. They all
found happiness.

Wastes Time
The professor becomes bold to suggest that all of us Christians should be delighted to
discover that we have been "wasting time" in preaching Jesus. Once more, "SO WHAT?" Why
should it bother Till if we waste time, as long as we are happy in so doing? If life is
purposeless, and Till's thesis of no God allows nothing else, then how has he been able to
assess by value judgments what is the best use of time as opposed to a poor use of time? The
skeptic's view is that "matter in motion" is the sole reality--similar to a rock rolling down a
hill. Since that is so, the way I use my time is no more under my control than how many
bushes the rock smashes on its way down the mountain. Till's view of man is that one rock
rolling down a mountain one day called itself "thinking man" and told the rest of us rocks that
we are traveling down the wrong side of the hill and that his side of the hill is the right one!
Professor, your view of man would never allow you to discover what is the proper use of time
as opposed to an improper, let alone tell us how to be better managers thereof. Besides that, if
I enjoy wasting my time preaching Jesus, "What is that to thee," you who suppose pleasure is
the measure of all?

Volume 1990 - 2002 Issue


Page 385 of 2049
Skeptical Review Edited by Farrell Till
Truth Is Important?
Now, readers, we come to the most absurd blunder of all. Till wrote of the "intrinsic value of
truth" and implied that we are somehow obligated to seek and find it. First, this is the reason,
like the reason of all evolutionists and atheists, for publishing his Skeptical Review. What a
dilemma!

Happiness is the goal of life but if one thinks of the futility of life when viewed in the
framework of evolution, he will become unhappy. However, truth is more important than
happiness, so he should exchange the happiness which faith in God brings him and accept the
truth which undercuts happiness. Then since happiness is the goal of life, he should forget the
truth concerning life's futility in order that he may become happy (Ibid.).

Till evidently cannot spot the pitfalls that he creates for himself. If the truth of life per
skepticism (that there is no God) makes one unhappy in contemplation thereof, what shall we
do? Till thinks we are under some obligation to accept this "truth," even if it makes us
extremely unhappy. Or if religion makes you happy, ought we to keep this greatest happiness
by forsaking Till's truth? Which goal do skeptics rank highest? If we are somehow
"intrinsically" obligated to follow truth, then this may be at the expense of every single thing
Till had to say! Forget being happy, forget "pleasure"--find truth, now says Till. He even
quotes his unfavorite person, Jesus Christ, at John 8:32. But if we are to pursue happiness as
the highest goal, and we may find it in religious belief, why should Till blow about the value
of truth?

Second, and the most telling mistake of Till's thesis, is the notion of the "intrinsic value of
truth." Do not fail to get the point, readers. Till's world view allows only matter in motion--no
God, no purpose in living, nothing but rocks and dirt set in purposeless motion. Thought,
then, which is only the result of our thinking processes, is no more than the same--the
purposeless twitching of gray matter in one's skull. "Thought is matter in motion," wrote
Woolsey Teller (The Atheism of Astronomy, pp. 10-11, quoted by Bales, Man on All Fours, p.
51).

If Till tells the truth about thought, then it is worse than absurd for him to tell us about the
inherent value of truth. Truth could be nothing more than the result of naturalistic forces, and,
hence, per Till's view, there is no reason to speak about even the rationality of thought.
Thoughts are merely the physical reaction to physical forces--haphazard twitching of one's
nervous system. Yet Mr. Skeptic's twitching of gray matter has concluded that his twitching is
more valuable than mine and that there must be somewhere an "intrinsic value" in it. Come
now, professor, can you not do better than this? You have fallen on your own sword.

(Bill Lockwood's address is 211 North 5th, Marlow, OK 73055).

As I Lay Dying
Volume 1990 - 2002 Issue
Page 386 of 2049
Skeptical Review Edited by Farrell Till
(With Apologies to William Faulkner)
Farrell Till

Our old fundamentalist friend Bill Lockwood claims that in my musings on truth I have fallen
on my own sword. I trust, then, that he will accord me the courtesy of considering my final
thoughts as I lie dying from my self-inflicted philosophical wound.

The thought uppermost in my mind at the moment concerns the apparent fragility of this thing
that Lockwood and his fundamentalist cohorts call "objective morality." As long as they think
it exists, they are able to lead exemplary moral lives, but if they should ever be without it,
they see themselves succumbing to the enticements of just about every evil imaginable to the
human mind. In his first article on the Amalekite massacre, Lindell Mitchell described all that
he could do to me personally in the absence of a standard of absolute morality-- assault me,
inflict multiple contusions and lacerations to my head, spoil my goods, kill my children,
ravage my wife, and mercilessly torture me. Now comes Bill Lockwood to tell us that if there
were no objective standard of morality, he might enjoy "inflicting pain to the bodies of
others."

So what is it with these fundamentalist preachers who envision total moral chaos in a world
that doesn't believe in absolute morality? I have established many friendships and associations
with atheists and skeptics, most of whom do not believe in absolute morality, yet none of
them has ever said, "Well, we don't believe in a standard of objective morality, so why don't
we find a preacher and beat him up? It would be fun to inflict multiple contusions and
lacerations to his head and then spoil his goods, kill his children, ravage his wife, and torture
him mercilessly." I can assure all absolute moralists that a suggestion like this would not be
favorably received in a gathering of skeptics and atheists. Anyone making such a suggestion
would be urged to seek professional help.

There aren't many Church-of-Christ preachers in Central Illinois (how lucky can one get?),
but I recall a conversation with one a few years ago in which he told me that he attributed his
personal moral character entirely to God. Without his belief in God, he went on to explain, he
was sure that he would be in prison because he would be unable to resist the temptation to
steal. I told him that I found his statement hard to understand. At the time, we were sitting in
his office, where several objects of value were in view, so I went on to explain that even
though I don't believe in God, if I were left unsupervised in his office the entire day with
every opportunity to pilfer, I would not steal anything. Even if a thousand dollars were left in
the room, I said, and I knew that I could take it with impunity, I still wouldn't do it. I
explained that my morality was based on logic. If a thousand dollars that he had worked for
was not safe in his home, then a thousand dollars that I had worked for would not be safe in
my home, so by respecting his personal property rights, I was acting in my own self-interest.
Every person, then, who doesn't kill or steal or lie is acting in self-interest to help establish an
orderly society in which one can live without fear of losing his life or property. The preacher
seemed unable to understand--or perhaps didn't want to understand--the logical rationale that
morality should be based on, so I assume that he still lives his life in fear that if he steals, God
will get him. He may call that "happiness" if he wants to, but I call it just another example of
the harm that religion inflicts on its believers.

Volume 1990 - 2002 Issue


Page 387 of 2049
Skeptical Review Edited by Farrell Till
Why Bother?
As I lie dying from my self-inflicted wound, I also have to wonder if Lockwood is serious
when he asks why we should bother if there is no God. "Why not sit back and watch the grass
grow?" was the way that H. A. Dobbs put it in my debate with him in Portland, Texas, last
May when we were supposed to be discussing the issue of prophecy fulfillment. I will gladly
accommodate Lockwood and his cohorts with an answer, although I am sure that they will
reject it. Once a fundamentalist has his mind made up on an issue, dynamite can't budge him
from it. The more I talk to them and debate them, the more I wonder how I ever managed to
find my way out of the maze of ignorance in which fundamentalists have imprisoned
themselves.

So here is the answer, Mr. Lockwood. Please read it very carefully. Even though there is no
God, we should "bother," in the sense of being concerned about the way we live our lives,
because if we don't, we squander the only thing we have or will ever have. In the movie
Unforgiven, there is a scene where Clint Eastwood's character said pensively, "It's a terrible
thing to kill a man. You take away all that he has and all he'll ever have." Over 200 years ago,
the American poet Philip Freneau gave a more than satisfactory answer to Lockwood's
question in his poem "To the Honey Suckle":

If nothing once, you nothing lose,


For when you die, you are the same;
The space between is but an hour,
The frail duration of a flower.

Before one is born, he is nothing; he simply doesn't exist. When he dies, he returns to what he
was before he was born--nothing. Dying, however, is not the terrible loss that theists make it
out to be if atheism is true, because by being born we at least gained life (the space between).
Viewed in this way, death becomes nothing to fear. I was not afraid before I was born, so
what is there to fear after I am dead? I will become what I was before--nothing.

Realization of this truth is as old as the Bible itself, whose editors somehow managed to let
some very skeptical sentiments sneak by them. So if Lockwood won't believe me, maybe he
will believe his inspired word of God:

Everything that confronts them [both the righteous and the wicked] is vanity, since the same
fate comes to all, to the righteous and the wicked, to the good and the evil, to the clean and
the unclean, to those who sacrifice and those who do not sacrifice. As are the good, so are the
sinners; those who swear are like those who shun an oath. This is an evil in all that happens
under the sun, that the same fate comes to everyone. Moreover, the hearts of all are full of
evil; madness is in their hearts while they live, and after that they go to the dead. But whoever
is joined with all the living has hope, for a living dog is better than a dead lion. The living
know that they will die, but the dead know nothing; they have no more reward, and even the
memory of them is lost. Their love and their hate and their envy have already perished; never
again will they have any share in all that happens under the sun (Ecc. 9:2-6, NRSV).

Volume 1990 - 2002 Issue


Page 388 of 2049
Skeptical Review Edited by Farrell Till
These thoughts are in full agreement with a belief that Job expressed in his fourth speech.
Preachers frequently lift out of context a question that Job asked in this speech: "If a man dies,
shall he live again?" (14:14). They quote it and then try to make it mean that Job was
expressing faith in an afterlife, but actually he was doing the opposite. Prior to this question,
he clearly expressed a belief that death was the end of man:

For there is hope for a tree, if it is cut down, that it will sprout again, and that its tender shoots
will not cease. Though its root may grow old in the earth, and its stump may die in the
ground, yet at the scent of water it will bud and bring forth branches like a plant. But man dies
and is laid away; indeed he breathes his last and where is he? As water disappears from the
sea, and a river becomes parched and dries up, so man lies down and does not rise. Till the
heavens are no more, they will not awake nor be roused from their sleep (vv:7-12, NKJV).

So that was the answer to Job's question, "If a man dies, shall he live again?" His answer was,
"No, never again!" Yet fundamentalist preachers accuse skeptics of being everything but
human for having the same doubts that some of their own inspired writers expressed.

Life, the "space between" that Freneau spoke of, is valuable; it is the only thing that we can be
absolutely sure of. It is because of the value of life, then, that we should "bother" and do more
with our lives than sit back and "watch the grass grow." It is because of the value of life that I
don't plan to take a gang of commandos to Livingston, Texas, and do all the things to Lindell
Mitchell that he claims he would have the right to do to me in the absence of absolute
morality. My "twitching gray matter" that Lockwood wrote about twitches more than enough
for me to understand that inflicting contusions and lacerations to his head and body would
cause Mitchell pain and suffering. I don't want anybody to do that to me, so I don't want to do
it to him. My head also "twitches" enough to tell me that spoiling one's goods, killing his
children, ravaging his wife, and torturing him mercilessly would cause him severe grief. I
don't want anybody doing this to me, so I won't do it to anyone else. Just what is the big
mystery here? What is so hard to understand about intelligence reaching a degree high enough
to formulate noble thoughts that become moral principles?

As a matter of fact, indications of morality in lower animals has even been observed and
documented. In Shadows of Forgotten Ancestors, Carl Sagan and Ann Druyan related the
results of an experiment with macaques (primates native to Japan) that had been reported
earlier in Psychonomic Science (1964, pp. 47-48):

In a laboratory setting, macaques were fed if they were willing to pull a chain and electrically
shock an unrelated macaque whose agony was in plain view through a one-way mirror.
Otherwise, they starved. After learning the ropes, the monkeys frequently refused to pull the
chain; in one experiment only 13% would do so--87% preferred to go hungry. One macaque
went without food for nearly two weeks rather than hurt its fellow. Macaques who had
themselves been shocked in previous experiments were even less willing to pull the chain.
The relative social status or gender of the macaques had little bearing on their reluctance to
hurt others (Shadows..., p. 117).

What could possibly account for the conduct of these macaques except a level of intelligence
that had enabled them to understand that their personal actions were causing their fellow

Volume 1990 - 2002 Issue


Page 389 of 2049
Skeptical Review Edited by Farrell Till
primates to suffer? The understanding had created in them a desire to avoid causing suffering
to others that they would not want to experience themselves. Now if monkeys are capable of
moral abstraction on this level, why should it surprise anyone that humans, the highest order
of primates, can moralize on an even loftier plane?

Absolutists will argue that humans are able to moralize in the absence of direct divine
revelation of moral commands only because they were created in the image of God, but not
even the Bible will support this view. Adam and Eve were allegedly created in the image of
God (Gen. 1:26-27 ), yet they did not understand the difference in good and evil till after their
experience with the forbidden fruit (Gen. 3:1-5 , 22-23 ). For the sake of argument, however,
let's just grant the fundamentalists their argument: humans can moralize in the absence of
divine revelation because they were created in the image of God. So does this mean that
macaques were also created in the image of God? If not, then it must not be true that the
ability to moralize results from having been created in the image of God.

What then is the origin of moral principles? There is only one logical answer. Concepts of
right and wrong, are intellectual abstractions. The human mind has abstracted concepts of
beauty, loyalty, and fairness, yet there are no gods of beauty, loyalty, and fairness who have
decreed objective standards of beauty, loyalty, and fairness. If Lockwood or Mitchell should
say that it is impossible to have a beautiful sunset in the absence of an objective standard of
beauty against which to measure light, hue, and perspective, they would be soundly ridiculed
by everyone who recognizes that the mind is fully capable of formulating concepts of beauty.
Would they argue that in the absence of an objective standard of loyalty, it is impossible for
anyone to determine if a spouse or friend or employee is loyal? If not, then by what logic do
they argue that morality cannot be conceptualized without a god to issue a standard of
objective (absolute) morality?

Moral absolutists like Lockwood and Mitchell are actually saying that no one can determine
right from wrong unless God speaks from heaven or inspires someone to write in a book that
X is right and Y is wrong, but all reasonable evidence on the subject disputes this premise. I
hate to sound like a broken record, but their own inerrant "word of God" clearly recognizes
that man is capable of making correct moral decisions without divine enlightenment. In
Romans 2:14 , the Apostle Paul referred to Gentiles who "have not the law" [special
revelation] but who nevertheless "do by nature the things of the law" and are therefore "the
law unto themselves." If this doesn't mean that the chief architect of Lockwood's and
Mitchell's religion thought that correct moral decisions could be made without receiving
moral revelations from God, then they should tell us what Paul did mean. Maybe if I urge
them enough, they will eventually get around to dealing with this fly in their absolute-
morality ointment.

Another fly in their ointment is a dilemma as old as Plato. For the absolutist who believes in a
morality that emanates from a god or gods, this dilemma is the problem of why moral
principles are moral. Let's take the proscription against killing. Lockwood would say that it is
morally wrong to kill because God has said, "Thou shalt not kill." Very well, why did God tell
us not to kill? Did he prohibit killing because killing is inherently bad or is killing bad
because God said that it is? If Lockwood says that God prohibited killing because killing is
inherently bad, then he is saying that there is an objective morality that transcends even God

Volume 1990 - 2002 Issue


Page 390 of 2049
Skeptical Review Edited by Farrell Till
himself. In other words, God had no choice; he had to prohibit killing because the inherent
badness of killing had already been predetermined by some "standard" separate and apart
from God, which God could not contravene without compromising his much touted attribute
of "infinite" goodness. If Lockwood chooses this horn of the dilemma, he will have to
concede that God is not supreme, because there is at least one thing superior to him--objective
morality.

Lockwood will find the other horn even less attractive. If he says that killing is bad because
God says that it is, then he completely destroys his precious concept of objective morality. If
things are good or bad only because God says that they are good or bad, then it isn't possible
for anything to be inherently (objectively) good or bad. They are good or bad... well, because
God says that they are. So the same god that declared X good and Y bad could change his
mind and decree X bad and Y good.

Lockwood thought that he was having a lot of fun at my expense when he envisioned people
who think it is fun to inflict pain on others or to burn themselves alive, but turn about is fair
play. Now it's my turn to hypothesize. If Lockwood chooses the second horn of the dilemma,
I ask him to consider the possibility of what the next universe will be like. (He surely doesn't
think that God is through playing around; after the heavens have dissolved in fervent heat [2
Pet. 3:12 ], he will probably want to make himself another toy.) Maybe in that universe,
Lockwood's god will decide to make lying, killing, and stealing good. If so, then there will
probably be absolute moralists like Lockwood and Mitchell somewhere in that universe
arguing that if there is no objective morality, then anything goes. One can tell the truth, save
lives, give money to the needy, and do all other kinds of "evil" deeds.

Perhaps, then, Lockwood will enlighten us in this matter. Why is a given moral principle
moral? Is it because it is just intrinsically moral, or is it because God said that it was moral?
Fundamentalists have pat answers for everything, so he should be able to tell us.

Lockwood accused me of thinking that people have an obligation to accept atheism "even if it
makes [them] extremely unhappy," but I have never said that. I have said that religion makes
people unhappy, often very unhappy, and I think I have in my letter files--not to mention my
own personal experience-- ample testimony that this is so. In those cases, I unhesitatingly
advise people to find the courage to face the disapproval of relatives, friends, and associates
and remove religion from their lives.

As I read Lockwood's claim that atheism will make people "extremely unhappy," I wondered,
"How does he know that this is what will happen if people accept atheism?" Certainly, he
can't be speaking from experience, because he has never been an atheist. On the other hand, I
am an atheist who was once a Bible-believing theist, and I say that there is no comparison
between the two philosophies. Atheism makes one happier, and this has been attested to by
numerous atheists. I have yet to meet an atheist who says, "I am miserable; I was so much
happier when I was a Bible believer." Obviously, then, Lockwood has no qualifications to
claim that atheism causes extreme unhappiness.

The same can be said of James Bales, whom Lockwood quoted on the subject of happiness:
"Happiness is the goal of life, but if one thinks of the futility of life when viewed in the

Volume 1990 - 2002 Issue


Page 391 of 2049
Skeptical Review Edited by Farrell Till
framework of evolution, he will become unhappy" (Bales, p. 44). Bales was a Bible professor
at the college I graduated from. He was a Bible believer then, and when I last had contact with
him by correspondence three years ago, he was still a Bible believer. I don't recall ever
hearing Bales claim that he has personally experienced atheism. How then can he know that
"if one thinks of the futility of life when viewed in the framework of evolution, he will
become unhappy"? How many atheists has he interviewed to determine that this is true? In
other words, what qualifies Bales to make an assertion like this?

I often think of life "in the framework of evolution," and it by no means makes me unhappy.
Of course, I don't think of the "futility" of life in the framework of evolution, and that is
because I don't think that the "framework of evolution" in any sense makes life "futile." To
the contrary, consideration of life in the framework of evolution should produce a sense of
awe when one considers that we are the result of eons of struggle successfully endured by
distant ancestors who brought us from what they were then to what we are now. The suffering
and endurance of the famous heroes of faith in Hebrews 11:32-38 fades to nothing in
comparison to the obstacles that those primitive ancestors overcame.

Lockwood doesn't like to think of thought in terms of "twitching gray matter," and I don't
blame him. The expression suggests a chaotic act, and thought is far from chaotic. The
impulses in the brain that generate thought behave in a very orderly fashion that could hardly
be described as "twitching." This order is the result of eons of organization through natural
selection.

Lockwood ridiculed Woolsey Teller for saying that "(t)hought is matter in motion," but he
should have read the statement in context rather than depending on what "Brother Bales" had
said. After discussing the discoveries that established thinking as a function of the brain,
Teller said this:

As functions cannot exist apart from their organs, it is the height of absurdity to imagine a
function like thinking existing by itself or wandering about the heavens without a material
substratum. A "pure spirit" hovering over matter is pure nonsense. Thought is "immaterial"
only as respiration and digestion are immaterial--we cannot see, weigh, or handle functions
apart from their organs--but thinking is as material as matter itself when we consider it
mechanically, that is to say, as a form of vibration and sensation in the nerve fibers of the
brain and of the nervous system. Matter thinks quite as well as it walks, and talks, and dresses
for the opera; and without matter thinking is impossible (The Atheism of Astronomy, pp. 10-
11).

Teller then went on to say that "thought is matter in motion," and so a "cosmic intelligent
being would have to be made of matter."

Whether this is an accurate way to describe thought may be debatable. Personally, I prefer to
think of thought as the result of matter in motion, just as walking and talking are the result of
matter in motion. However, the basic premise is sound. If Lockwood thinks otherwise, then
perhaps he will accept the challenge to prove to us that thought can exist apart from matter.
Let him establish the existence of a disembodied thought just floating around in space
unattached to any brain, and I will concede the debate to him.

Volume 1990 - 2002 Issue


Page 392 of 2049
Skeptical Review Edited by Farrell Till
What Is Absolute Morality?
I have repeatedly urged Lockwood and Mitchell, along with several other absolute moralists,
to tell us just how we can determine what absolute morality is, but they have consistently
evaded the issue. They say that absolute morality is revealed in the Bible, so why won't they
explain to us just how we can determine, without question, what is moral and what is
immoral?

Is it morally right, for example, for a woman to allow herself to be artificially inseminated by
the semen of a man she is not married to? Is it morally right to transplant organs? Is in vitro
fertilization morally right? Embryo transplants? Gene-splicing? And more recently we have
learned that human embryos can be cloned. Will it be morally right to clone humans? Was the
right moral decision made last summer when Siamese twins were separated in an operation
that surgeons knew would result in the death of one of the twins? What does Lockwood's
guide to absolute morality tell us about this and the other issues mentioned above? He owes
us an answer.

If he will send us a book-chapter-and-verse resolution of these moral problems, we will


publish it in The Skeptical Review. If morality is as simple as he would have us believe, he
shouldn't have any trouble providing us with a document that will satisfactorily resolve all
these problems and set us all on the road to moral happiness.

From the Mailbag


I received your cover letter and a copy of The Skeptical Review, and wish to thank you very
much for your kindness. I read every word of the issue you sent and am very impressed to see
that you touch on so many of the areas that are relevant to me.

You commented in your letter that you were also affiliated with the Church of Christ in the
past, as was I. I had a couple of friends who attended Harding during the '60s, so am
somewhat familiar with the school. I attended Cincinnati Bible Seminary during that same
time period.

For years I really believed that I believed, though some doubts were difficult for me to
assuage. Finally, I fell away from active participation in the church itself and I pursued other
career endeavors and tried not to think too deeply about the doubts I felt. I pretended to still
believe, out of habit and fear, I suppose.

Finally, I began a renewed study of scripture, actually studying it with an open mind for the
first time. I realized that all of my biblical training had been undertaken from a subjective
viewpoint prior to this. My new viewpoint was totally objective, or as near to it as I could
manage.

Volume 1990 - 2002 Issue


Page 393 of 2049
Skeptical Review Edited by Farrell Till
I also began to study other historical documents and viewpoints outside of the bible. The
picture began to become clear very rapidly. I had been misled and misdirected for most of my
life in things spiritual.

After several years of rationalism and realism, I am very comfortable in my beliefs, and this
has led to such peace and serenity that I cannot believe it. I am so much happier in my life.
Those around me, family and friends, constantly remark at how much more at ease I have
become and how much less driven I seem to be. I also feel myself to be a more moral person
now than I ever felt I was as a Christian.

It would appear from the reading of one issue of your newsletter, you probably know exactly
of what I speak. Sorry to bore you with the needless details, but I am so happy with who and
what I now find myself to be that it's difficult not to share a little of my enthusiasm with
someone who understands it.

(Bob Hypes, P. O. Box 264, Leo, IN 46765-0264)

Thanks for the video. Sorry for the return delay. Though I'm a Christian theist, I can't help but
think that "Buster" should be BUSTED! What a mule-brain! Hey, no one should question
things that have already been "decided"! (By who? The Church of Christ? The Catholics?
Constantine?)

Hopefully, you occasionally run into a theist who thinks. There are some, you know. I try to
have an open mind and think rationally; perhaps you've seen my essays in Freethought
Exchange. I do believe in God and Christ but don't believe in Bible inerrancy.

(Ron Labbe, 15 Anson Street, Jamaica Plain, MA 02130)

I would hate to be in your shoes when Jesus comes.

(Anonymous with a Peoria, Illinois, postmark and a tract on the agonies of hell enclosed.)

My wife and I are most indebted to you for the loan of the video tapes of the Till-Dobbs
Debate....

My wife has, until viewing the tapes, shown no interest in The Skeptical Review, considering
them to be too complicated for "ordinary people," that is, those people who know nothing
about the Bible except for Bible references made by the clergy and the "shopping-mall"
preachers.

Volume 1990 - 2002 Issue


Page 394 of 2049
Skeptical Review Edited by Farrell Till
Having sat through a number of hours listening to the debate, my wife is now reading all The
Skeptical Reviews on hand. They are lying all over the house now. In the bathroom, in the
bedroom, even in the hot-tub room. She has gone crazy! She is now reading and talking, and
jumping up and down, and arguing and asking questions and is on a crusade to stamp out
every vestige of superstition from the planet! Boy, you can't believe it! I hope she feels better
tomorrow.

We were stunned, I mean super stunned, that a purported spokesman of Mr. Dobbs' standing
had nothing convincing, not even the slightest evidence in support of his position. For a time,
we felt embarrassed for Mr. Dobbs, but then we became angered by his behavior.

My wife characterizes Mr. Dobbs as an arrogant, overbearing, nasty person without even a
scintilla of fair play in his store of virtues. She thinks Mr. Dobbs is a troubled and tortured
man and will, over time, hurt the cause of Christianity.

My thoughts are that Mr. Dobbs will certainly cause most fence-sitters to fall out on the side
of nonbelief, if not outright atheism. A problem is that it is enormously more dificult to earn
the right to be an atheist than it is to earn the right to be a member of one of the 260 or more
sects in America.

You would have been most proud of yourself if you could have seen the effect the tapes had
on us. You did indeed lay ruin to the Dobbs apologetics. If Mr. Dobbs presents the best that
defenders of the faith have to offer, then perhaps finally we can see a lot more humanity
toward one another and a lot less religious condemnation of all mankind....

(Len & Rita Reitz, 3660 9th Drive, Baker City, OR 97814)

EDITOR'S NOTE: Needless to say, the compliments were appreciated. The video tapes are
still available on two-week loan for anyone who wants to see the utter failure of an esteemed
fundamentalist editor and preacher to offer any reasonable evidence to support the prophecy-
fulfillment argument.

Lindell Mitchell defends the massacre of the Amalekites, in the Winter 1994 issue, as a "holy
war." The holy-war concept in itself is immoral. Contrary to his assertions, morality exists
without reference to the alleged absolutism of any supernatural entity. Like most people who
believe in morality, I can accept the use of force--even lethal force--to defend myself or
another person from an attack, but it is a moral outrage to try to justify killing other people
merely because one thinks they have somehow offended a supernatural belief system.
Whether one's god is the theological entities of the Israelites or those Christians who
committed the crusades, the inquisitions, and the holocaust, or the ideological entities of
Nazis and Communists, whose rationale for their massacres is basically the same as used by
theologians, "holy war" is inherently a moral offense.

(William Sierichs, Jr. See address at the end of Touring the Middle East Jesus Style , p. 12.)

Volume 1990 - 2002 Issue


Page 395 of 2049
Skeptical Review Edited by Farrell Till

The fundamentalist view that there is no morality without God is a view that was challenged
long ago by Plato in the Euthyphro. Plato asked the question: are actions right because they
are commanded by the gods, or do the gods command those actions because they are right? If
the ultimate source of right and wrong is God's will, then it is not informative to say that God
wills what is right. If God wills the killing of Amalekite children, then that is right. If God
wills the torture of children, stealing from the poor, sexual promiscuity, or even the
condemnation of fundamentalists to the fires of hell, then that is right. (And the
fundamentalists cannot object that God wouldn't do this because it would be morally wrong.)
In effect, this view is a form of moral subjectivism which says that God's subjective moral
determinations hold for everyone. There is no deeper explanation of moral truth. By contrast,
virtually all philosophers (theist and atheist alike) have wisely rejected this view in favor of a
standard of morality independent of the will of God. This allows for both deeper explanation
of what makes actions right and wrong (e.g., in terms of utility, hypothetical social contract,
objective reasons, rationality, etc.) and allows the theist to make sense of moral evaluation of
God himself. On this view, it is possible to say something informative when you say, as
traditional theists do, that God is good. On the former view, to say that God is good is to utter
a tautology. Fundamentalists who maintain that there is no morality without God are
contradicting millennia of ethical philosophy, as any introductory moral philosophy textbook
plainly illustrates. (I recommend James Rachels' The Elements of Moral Philosophy, 2nd ed.,
1993, McGraw-Hill.) I should add that I think Till concedes too much when he seems to grant
the fundamentalist claim that morality cannot be objective or absolute without God. The theist
needs an argument to establish this claim--there are certainly a number of purportedly
objective moral theories propounded by theists and atheists alike which are not dependent on
the existence of God (including utilitarianism, Kantianism, and social contract theory).

On another philosophical point, Lindell Mitchell accuses Farrell Till of believing that "man's
knowledge is restricted to what he gains through one of his five senses" and goes on to claim
that Till "has forever shut himself off from any evidence that results from testimony."
Mitchell supposes that the evidence of testimony cannot be justified on the basis of an
individual's perceptions and inductive inference (as is suggested, for example, by David
Hume in section ten of his Enquiry Concerning Human Understanding, "Of Miracles").
While Mitchell is not alone in making this claim, he needs to provide some argument to show
that we cannot be justified in relying on the word of others on the basis of our past
observations that most of what people tell us turns out to be correct when we check it
ourselves. (This happens to be the subject of my doctoral dissertation. While I disagree with
Hume in the details, I think something like what he suggests does allow for us to be justified
in believing a wide variety of testimony, including the products of scientific research.
Religious authority, however, does not fare as well.)

Finally, I'd like to recommend that those who have read George H. Smith's Atheism: The Case
Against God and would like to move on to something more philosophically sophisticated (a
well-read theist can rebut most of Smith's arguments) try J. L. Mackie's The Miracle of
Theism (1982, Oxford University Press; Michael Martin's Atheism: A Philosophical
Justification (1991, Temple University Press); and Richard M. Gale's On the Nature and

Volume 1990 - 2002 Issue


Page 396 of 2049
Skeptical Review Edited by Farrell Till
Existence of God (1991, Cambridge University Press). Be forewarned: the last of these is
quite technical.

(Jim Lippard, doctoral candidate in philosophy [epistemology/cognitive science], University


of Arizona. Personal mailing address is 2930 East First Street, Tucson, AZ 85716.)

Your cover article in the Summer 1993 issue, on some of the logical fallacies employed by
Christian apologists, left a good impression on me and left me looking for more articles in
The Skeptical Review in the areas of logic and critical thinking. I have quite often now seen
fellow freethinkers present a Christian rebuttal and let it stand as the last word, expecting that
we all know what exactly was wrong with the argument. Well, that's not always the case for
some of us. And perhaps for some it is never so. Logic isn't taught much, and those of us who
have had a course in it could from time to time do with a refresher.

For example, I listened to you ask Mr. Dobbs at the debate last summer a question that went
something like this: "If you had an unmarried daughter who became pregnant but told you she
was still a virgin, would you believe her?" Mr. Dobbs refused to answer, saying that this
question was of the same stuff as asking, "Have you stopped beating your wife?"

Of course, it was apparent to me that the comparison was invalid and that your opponent in
the debate was merely dodging the question. But I couldn't state why. I would have liked to
have been able to when, during the next intermission, I overheard two men in the foyer
congratulating each other on the fine job their preacher had done in "answering" your
question. But I just wasn't prepared at the time.

So I bought a textbook on logic and looked it up. The wife-beating question that was Dobbs'
dodge was the second classic example given of a complex question. The book defined
complex questions as those that presuppose that a definite answer has already been given to a
prior question that was not even asked. Your question, though, was a purely hypothetical one.
Every relevant condition was clearly and explicitly stated within the body of the question
itself, save for the assumptions that must always be there--namely, that your opponent was
capable of answering a question and would do so honestly.

So, you see, I appreciate articles that point out the logical flaws and look forward to seeing
more articles like "Poisoning the Well."

(Earle C. Beach, 13203 Tamayo Drive, Austin, TX 78729-7403.)

EDITOR'S NOTE: I have to plead guilty to assuming too much. When Dobbs made the
analogy that Beach mentioned, I immediately realized that he had incorrectly treated my
legitimate question as the fallacy of complex question, but it was so flagrant that I assumed it
would be obvious to everyone in the audience who wasn't already hopelessly duped by
fundamentalist indoctrination. At this point in the debate, Dobbs was groping for just
anything to say to kill the two minutes that the rules allotted him to answer each question.

Volume 1990 - 2002 Issue


Page 397 of 2049
Skeptical Review Edited by Farrell Till
That seemed so obvious that perhaps I made the mistake of assuming that his answers, which
were mainly repetitious nonsense, didn't warrant comment.

Regarding Lindell Mitchell's article "The Amalekite Destruction a Moral Atrocity?" all I can
say is that it has a coefficient of irrationality approaching infinity. He is completely unable to
distinguish between the words infant, fetus, and embryo. Even "Saint" Thomas Aquinas stated
that abortion was not murder in the first trimester. The Hippocratic oath also permits
abortions prior to the "quickening" (first trimester). His article is an excellent example of what
Thomas Jefferson had in mind in the last sentence of a letter that he wrote to James Smith on
December 8, 1822, which reads as follows: "Man, once surrendering his reason, has no
remaining guard against absurdities, the most monstrous, and like a ship without a rudder is
the sport of every wind. With such persons, gullibility, which they call faith, takes the helm
from the hand of reason, and the mind becomes a wreck." I was thinking of writing to L.
Mitchell but decided to wait for signs of a decrease in his coefficient of irrationality.

(Sol Abrams, 1759 Stuart Street, Brooklyn, NY 11229.)

Lindell Mitchell has shown your fallacy and stupidity. His exchanges with you display your
incompetence and explain why you spent the better part of your adult life teaching in a fourth
rate school, and edit a paper with pitifully few subscribers. You, Sir, have spent your life for
nothing. You are a nobody. Your life is a waste. It must be painful to come to the end of the
way and realize that you have subsided into a stale pool. No wonder you bristle when
someone calls to attention your fraud when you were in France. Again, I beg you to repent
and make whole the widow's houses you have robbed. You will come before the judgment bar
of God. I tremble for you. "Repent therefore of this thy wickedness, and pray God, if perhaps
the thought of thine heart may be forgiven thee. For I perceive that thou are in the gall of
bitterness, and in the bond of iniquity."

(H. A. "Buster" Dobbs, Editor, Firm Foundation, P. O. Box 690192, Houston, TX 77269-
0192.)

EDITOR'S NOTE: Apparently the wounds Mr. Dobbs suffered in our debate are smarting
even more than I had suspected. After the debate, I began to hear rumors about misconduct
on my part when I was working in France as a missionary. Among other things, it was being
reported that I had stolen church funds and had been caught and called home in disgrace. In
checking into the rumors, I found that Mr. Dobbs seemed to be the source of some of them.
Despite my request that he produce evidence to support these charges, he has made no
attempt to do so. Yet, as his letter above shows, he continues to make the allegations--all in
the name of Christ, I assume.

Mr. Dobbs seems to find satisfaction in attacking me on a personal level. I'm just a teacher at
a "fourth-rate school," and I edit a paper with "pitifully few subscribers." I am a "nobody,"
and my life is a "waste." Actually, this is not the first letter in which Dobbs has hurled these

Volume 1990 - 2002 Issue


Page 398 of 2049
Skeptical Review Edited by Farrell Till
same insults at me, and each time I read them, I wonder what "inspired scripture" prompted
him to say them. Is this the language of a man who loves his enemy (Matt. 5:44 ) and seeks to
"do good to all men" (Gal. 6:10 )? As he goes about telling lies about my dishonesty as a
missionary, I wonder if he considers himself in compliance with the scripture that says,
"Speak evil of no man" (Titus 3:2 ). But more than any of this, I wonder if in pleading for me
to "repent," he wants me to change and become as "righteous" as he is. Furthermore, if, as he
says, I am in the "gall of bitterness" and in the "bond of iniquity," what terms could possibly
describe the state or condition that he is in? When a man who presents himself to the world as
a "gospel preacher" spreads malicious lies about another person's character in flagrant
violation of scriptures that warn against such conduct (Rom. 1:29-30 ; James 3:3-6 ; Titus
3:2 ; 2 Cor. 12:20 ), then surely his condition is worse than just being in "the gall of
bitterness" and "the bond of iniquity." In his long career as a preacher, I am sure that Dobbs
has had occasion to use some of these very scriptures in his sermons. If so, a paraphrase of
Romans 2:21 would be applicable to him: "You, therefore, who teach another, do you not
teach yourself? You who preach that a man should not steal, do you steal? You who preach
that a man should not lie, do you lie? You who preach that one should not backbite and
gossip, do you backbite and gossip?"

Dobbs labored to associate me with "fourth-rate" schools, but this is strange talk indeed from
a man who is just a preacher for a fundamentalist sect that doesn't even rank as high as
fourth rate. Outside of the Bible-belt environment that spawned it, the "Church of Christ" is
hardly even known, and, even now, it is an anachronism that is steadily disintegrating despite
a coalition of old-guard preachers that is frantically struggling to hold back a tide of
enlightening influences that make it virtually impossible for rank-and-file members to
continue believing the absurdities of the Bible inerrancy doctrine.

As for whether I have "wasted" my life or not, I am willing to let time make that judgment.
Results of the most comprehensive study of religious beliefs yet undertaken in the United
States have been published in Barry Kasmin's and Seymour Lachman's One Nation Under
God (Harmony Books), and the study reveals that only an insignificant minority of Americans
(fewer than 19%) take their church doctrines seriously. As the years pass, this number will no
doubt diminish even more, so when there is nothing left of Dobbs and me but dust; and belief
in the divine inspiration of the Bible no longer exists; and his "Church of Christ," which he
ballyhoos as the "only true church," has also ceased to exist; then there should be no debate
about which one of our lives was a "waste." Meanwhile, the "pitifully few subscribers" to TSR
continue to increase, and at the present rate of growth will easily pass 1,000 before year's
end. Dobbs may consider this number "pitifully few," but I will pit the critical thinking skills
of TSR's 900 subscribers against those of the Firm Foundation's thousands (?) any day.

Elsewhere in Dobbs' letter, he challenged me to prove that important events in the life of
Jesus had their parallels in pagan religions that existed before the Christian era. In response,
I sent a letter to Dobbs proposing that he and I debate this issue publicly. If he will affirm that
the Jesus of the gospels was an actual historical character, I will affirm that the virgin-born,
miracle-working, crucified, resurrected savior was a common belief in pagan religions before
the time that Jesus of Nazareth allegedly lived. Dobbs's reaction to this proposal will let us
know if he has the courage to put his faith where his mouth is. My prediction is that he won't

Volume 1990 - 2002 Issue


Page 399 of 2049
Skeptical Review Edited by Farrell Till
touch this issue with a ten-foot pole, and the reason why he won't can be found in an old
proverb that says, "A burnt dog is afraid of fire."

Enclosed is my second negative, it slightly exceeds the agreed upon length limit. However,
you violated our agreement by using a large portion of your "response" to Bill Lockwood in
an attempt to keep your sinking ship afloat. Therefore, I expect my response to appear in
totality. You also violated our signed agreement when you called me a fool. You were unable
to sustain your position, so you resorted to bluster. They were bold words for a balding old
guy. It is easy to be bold when you are not face to face. I suspect you are a real "woolly-
booger" to a trembling Freshman. Tough talk doesn't scare me Mr. Till, and it doesn't
establish your position.

I took no pleasure in exposing your profound ignorance. It is truly incredible to observe how
intellectually debased a man can become in just one lifetime.

You may continue to send me copies of your paper if you like. However, I am closing our
debate on 1 Samuel 15 at this point. I refuse to continue the discussion for these reasons. First,
we have completed the agreed number of exchanges. Second, you did not keep your end of
the agreement. Third, I have reduced your position to ruin. Fourth, you persistently introduce
irrelevant material and steadfastly refuse to grapple with the issue at hand.

You are obviously trying to make a name for yourself by attacking Christians. Your level of
desperation is seen in the fact that you had to jump on a country preacher like me. When I
trounced you in your own paper, any shred of credibility you had evaporated. Now you stand
before the world poor, blind, and naked. There is little more to commend you than arrogance
and unmitigated gall. How sad.

(Lindell Mitchell, P. O. Box 411, Livingston, TX 77351.)

Editor's Note: Gee, I'm sorry I applied to Mr. Mitchell a scripture that contained the word
fool. I can tell by the tone of the letters he writes to me that he is just an all-around good guy
who would never say anything disrespectful to anyone. A Christian, of course, wouldn't even
consider quoting a scripture, say, Psalm 14:1 , that would suggest the atheist is a fool, so I
can certainly see why Mitchell is upset.

The really frightening thing about his letter is that he probably does believe that he
"trounced" me in my own paper, because he apparently can't see that he begged the question
throughout the exchange and especially in his latest article. On the other hand, if Mitchell
really does believe that he scored a big one for God in this debate, one has to wonder why he
wouldn't want to keep trouncing me, yet he has stated emphatically that he will not continue
the discussion. Our exchange of articles has generated more letters and submission of articles
than anything yet published in TSR. The next issue will contain some of these materials,
advanced copies of which will be sent to Mitchell so that he may respond to them if he wishes
to do so. I predict that he won't. Methinks he just doesn't have the stomach to do any more
trouncings.

Volume 1990 - 2002 Issue


Page 400 of 2049
Skeptical Review Edited by Farrell Till
I suspect some readers will be writing to Mr. Mitchell. If you do, please send me copies.

Volume 1990 - 2002 Issue


Page 401 of 2049
Skeptical Review Edited by Farrell Till

The Skeptical Review


Volume Five, Number Three
July/August/September 1994
Farrell Till, editor

• Is Jesus a Counterfeit?
Christians dismiss the tales of pagan saviors as mythology while adamantly insisting
that it is rational to believe that the story of Jesus's virgin birth and resurrection is
factual. They can offer no reasonable explanation for their inconsistency, and until
they do, skeptics will have to insist that they are worshiping a counterfeit savior.

• Affirmations that Mitchell Did Indeed Make


In the course of the debate over the Amalekite massacre, Mitchell has refused to
answer questions on the grounds that he is not the affirmant. Is Mitchell within his
rights here, or is he using this to dodge questions he would not be comfortable
answering? Till claims the latter is true, and explains why.

• The Law of the Jungle


Dave Matson shows how morality arises naturally as the result of humans interacting
with other humans, not human interacting with God.

• Hare Jesus: Christianity's Hindu Heritage


Could it be that Hinduism played a major role in the development of Christianity?
Stephen van Eck thinks so, and shows several similarities between the two religions
which inerrantists will find hard to explain.

• Another Flaw in the Perfect-Harmony Theory


• Inerrantists boast that the Bible possesses a thematic unity so amazing that it can be
explained only on the basis of divine inspiration, but the facts do not support this
claim. As an example, Till considers Yahweh's promise in Jeremiah 18:7-8, and his
failure to keep this promise for King Josiah.

Volume 1990 - 2002 Issue


Page 402 of 2049
Skeptical Review Edited by Farrell Till
• Evidence That Doesn't Demand a Verdict
Even in the Bible, people rejected Christian testimony and waited for hard evidence
before believing. Who are we to disagree?

• The Geisler-Till Debate

• From the Mailbag

• A Reader's Letter to Mitchell

Is Jesus a Counterfeit?
In the Till-Dobbs Debate, fundamentalist preacher and editor Buster Dobbs asserted that the
Book of Mormon is an obvious counterfeit and then proceeded to parrot the trite old argument
that says the existence of a counterfeit proves the existence of the real thing. A counterfeit
dollar, for example, can exist only because real or authentic dollars exist. To Dobbs, needless
to say, the Bible is the "real" inspired book that the Book of Mormon counterfeited.

The argument is absurd, of course, because entirely theoretical objects can be conceptualized
and made without proving the existence of real or authentic specimens of whatever the objects
are supposed to represent. One could make metal coins and claim that they are replicas of
coins that are used on Mars, but the existence of these "counterfeits" would in no way prove
that Martian coins really do exist. In the same way, the writing of a phony "bible," which a
religious group tries to present as a book that was inspired of God, does not prove that a "real"
divinely inspired book exists. It is far more probable that such a book is just one more
example of a holy book for which sincere but mistaken claims of divine origin are made.

For the sake of argument, however, let's just grant Mr. Dobbs his point: the existence of a
counterfeit known as the Book of Mormon does prove that a "real" divinely inspired book
exists. Even with this concession, his argument begs the question by assuming that the Bible
is the genuine holy book that was divinely inspired. One could just as logically argue that the
Book of Mormon counterfeited the Avesta, an allegedly inspired book that antedates the Bible
by many centuries. Dobbs, however, would never agree that the Book of Mormon
"counterfeited" the Avesta, because he doesn't believe that the Avesta was divinely inspired.
He would ridicule the logic of anyone who would use such an argument to prove the divine
origin of the Avesta, yet he apparently can't see that the argument has no more merit when
applied to the Bible.

Another consequence of the argument that Dobbs and his fundamentalist cohorts would never
accept is that, carried to its logic end, the argument will prove that Jesus was a counterfeit.
This conclusion necessarily follows from the fact that belief in virgin-born, miracle-working,
crucified, and resurrected saviors flourished long before Jesus of Nazareth allegedly lived. So
if such savior-gods were worshiped prior to the time Jesus of Nazareth came onto the

Volume 1990 - 2002 Issue


Page 403 of 2049
Skeptical Review Edited by Farrell Till
religious scene, then Jesus must have been a counterfeit of those who had preceded him. If
not, why not?

Elsewhere in this issue, we are publishing "Hare Jesus: Christianity's Hindu Heritage" by
Stephen Van Eck, who demonstrates similarities in Hinduism and Christianity that are too
striking to attribute to coincidence. Since Hinduism was founded centuries before
Christianity, the only rational conclusion to reach is that the latter borrowed many of its
concepts from the former. This reality inflicts irreparable damage to the fundamentalist claim
that Christianity is a unique religion that was divinely revealed in the first century.

Van Eck rejects the premise (accepted by some scholars of Vedic literature) that Krishna, the
Hindu savior-god, died by crucifixion in some accounts of his death. This is a rather
inconsequential matter, however, for even if this particular similarity is not present in both
religions, there are still enough striking parallels in the lives of the two saviors to discredit the
claim that Jesus was the son of God. Since Krishna allegedly lived centuries before Jesus, this
is sufficient reason to suspect that Jesus was merely a counterfeit of Krishna and the other
savior-gods who were worshiped throughout the pagan world long before Jesus became the
socially correct savior to believe in.

Besides Krishna, there were many other virgin-born pagan saviors who were worshiped
before the time of Jesus. In The Story of Christian Origins, Martin A. Larson traced the myth
of the resurrected savior-god to Osiris, whom the Egyptians worshiped as far back as 3,000 B.
C. (p. 3). Like the many successor saviors who followed him, Osiris was killed by enemies
and then resurrected from the dead. To prevent his resurrection, the conspirators tore the body
of Osiris into fourteen parts and buried them in various regions of Egypt (a tale reminiscent of
the efforts that the enemies of Jesus took to guard his body). However, Isis, the consort of
Osiris, succeeded in finding the parts and putting them together again in Frankenstein fashion,
after which she breathed life into his nostrils (another familiar concept) and resurrected him to
eternal life.

After Osiris came many other virgin-born, resurrected savior gods: Dionysus (Grecian),
Krishna (Hindu), Mithra (Persian), Tammuz (Sumerian-Babylonian), etc., etc., etc. Ezekiel
the prophet referred to women whom he saw weeping for Tammuz at the gate of Jehovah's
house (Ez. 8:14), an obvious reference to the springtime ritual of mourning the death of this
pagan savior who, according to the myth that had fostered the religion, had later risen to life
again. So widespread was belief in virgin-born, resurrected saviors that Justin Martyr, an early
Christian apologist, used the familiarity of the story as an argument designed to convince non-
Christians that it was logical to believe Jesus was the virgin-born son of God:

By declaring the Logos, the first-begotten of God, our Master, Jesus Christ, to be born of a
virgin, without any human mixture, we (Christians) say no more in this than what you
(pagans) say of those whom you style the sons of Jove. For you need not be told what a parcel
of sons the writers most in vogue among you assign to Jove....

As to the son of God, called Jesus, should we allow him to be nothing more than man, yet the
title of "the son of God" is very justifiable, upon the account of his wisdom, considering that
you (pagans) have your Mercury in worship under the title of the word, a messenger of God....

Volume 1990 - 2002 Issue


Page 404 of 2049
Skeptical Review Edited by Farrell Till
As to his (Jesus Christ's) being born of a virgin, you have your Perseus to balance that (First
Apology, vol. 1, chapter 22).

Christians, of course, will dismiss the tales of these pagan saviors as mythology while
adamantly insisting that it is rational to believe that the story of Jesus's virgin birth and
resurrection is factual. They can offer no reasonable explanation for their inconsistency, but
until they do, skeptics will have to insist that they are worshiping a counterfeit savior.

Affirmations That Mitchell Did Indeed


Make
Farrell Till
Throughout Lindell Mitchell's "rebuttal" articles on the issue of the Amalekite massacre, he
refused to answer questions on the grounds that he was not the affirmant. In principle, I agree
with Mitchell's position in this matter. It is indeed the affirmant's duty to affirm, and the
negative's responsibility to respond to affirmative arguments. I myself object to debating
opponents who attempt to put me on the defensive with long lists of questions when they are
the ones who are supposed to be affirming. However, I have yet (with the exception of
Mitchell) to engage a negative opponent who refused to answer questions that arose during
the normal course of the debate.

Knowing Church-of-Christ preachers as I do, I suspect that if Mitchell has ever engaged in
formal debate, he himself has directed questions to his opponents while he was the affirmant.
It is simply routine procedure, and I think that Mitchell knows that it is. Essentially, all that I
asked of Mitchell during our first two exchanges was an answer to this question: If he had
been born an Israelite in the days of King Saul, would he have willingly and gladly
participated in the Amalekite massacre by killing women, children, babies, and pregnant
women?

This is a simple, direct question, and there is no good reason why Mitchell shouldn't answer
it. He has described the Israelite soldiers who participated in the massacre as "God's army"
that was engaged in a "holy war," so if he really believes this, why won't he just say, "Yes, I
would have willingly and gladly participated in the Amalekite massacre. I would have
considered it a great honor to be a part of God's army fighting in a holy war to kill women,
children, and babies"? His inerrantist position would require such an answer as this, so I have
to believe that he won't answer the question because he knows how bad such an answer would
make him look.

In Mitchell's letter published in the Spring 1994 issue (p. 15), he boasted that he had exposed
my "profound ignorance," reduced my position to "ruin," and left me standing before the
world "poor, blind, and naked." I can't help thinking that if he really believed he had
accomplished all this, he would have welcomed the opportunity to answer any question from

Volume 1990 - 2002 Issue


Page 405 of 2049
Skeptical Review Edited by Farrell Till
me so that he could have further exposed my profound ignorance. No, the "thoughtful
readers" that he referred to in his last article will have no difficulty seeing why he met this
question with screaming silence. He knew how ridiculous an answer consistent with his
inerrantist position would have made him look.

To the point of sounding almost like a broken record, Mitchell kept reminding us that he was
"not affirming anything in this discussion" (p. 6) and used that as an excuse to dodge
important issues that I had raised. In six consecutive paragraphs he began with references to
"the affirmative [he] did not make," but the only thing he proved in this section of his article
is that he does not understand basic principles of debating. It is true that the affirmant
advances arguments in a debate, but it is also true that the opponent's responses to these
arguments become negative affirmations that must be defended. For example, if Mitchell
were affirming that Jesus of Nazareth was an actual historical character, as his opponent I
would expect him to argue that the Jewish historian Josephus mentioned Jesus and called him
"the Christ" in Antiquities of the Jews (18.3.1). My response to this argument would be that
this passage is a forgery that Josephus did not himself write. In saying this, I would be
introducing a negative assertion or affirmation into the debate that I would be obligated to
prove. I could imagine what Mitchell's reaction would be if I refused to defend it and simply
said, "Well, I'm not the affirmant, so I don't have to prove anything."

"Thoughtful readers," then, will have no difficulty seeing that all of Mitchell's I-am-not-the-
affirmant talk was just a straw man that he set up to kick around to try to draw attention from
the absurdity of his position in the matter of the Amalekite massacre. However, I don't intend
to let him get away with the evasion. He has made several negative affirmations in this debate
that he must defend before he can make any convincing claim to having exposed my
"profound ignorance."

One negative affirmation he made was that the "Omniscient God" was able to "look down the
corridors of time and see that the babes of Amalek were destined to become vicious beasts
like their ancestors" (Winter 1994, p. 4). This assertion was offered, of course, as a reason
why the Amalekite massacre cannot be considered a moral atrocity; therefore, it is a negative
affirmation that Mitchell should at least attempt to defend.

In my response to this negative affirmation, I pointed out six flaws in it: (1) God is both
omniscient and omnipotent but was unable to solve a problem except by massacring babies,
(2) the Israelites could have taken "the babes of Amalek" back as captives and reared them as
Yahweh-fearing Hebrews, (3) it assumes without proof that the Amalekites were indeed
"vicious beasts," (4) it merely has one nation of "vicious beasts" exterminating another nation
of vicious beasts, (5) it has Yahweh violating his own decree that says iniquity would be
borne by the guilty ones and not by their descendants, and (6) it exempts Yahweh from the
moral law that proscribes killing and thereby proves that the law is not "absolute."

Mitchell's attempt to answer these arguments was a pathetically evasive exercise in question
begging. This was his response to my claim that an omniscient, omnipotent deity should have
been able to solve a problem without massacring babies:

Volume 1990 - 2002 Issue


Page 406 of 2049
Skeptical Review Edited by Farrell Till
Mr. Till assumes God is unable to judge the situation fairly and punish the wicked. He
assumes the giver of life has no right to terminate life. He assumes the Creator has no right to
call innocent Amalekites into the protection of paradise. Mr. Till is obligated, given his
epistemology, to produce objective sensory evidence that unequivocally proves this (Spring
1994, p. 6).

Say what? The whole point of my arguments was to show that the God-did-it-so-it-must-
have-been-okay position is riddled with problems. One of those problems is the obvious fact
that an omniscient, omnipotent god would know everything and be able to do anything. Such
a deity could have easily solved the Amalekite problem (if indeed such a problem even
existed) without ordering the murder of innocent babies, but just where does Mitchell's
comment quoted above even address this problem?

Implicit in his statement are several "negative affirmations": (1) God was able to judge the
situation fairly and punish the wicked, so (2) the Amalekites were wicked, (3) God is the
giver of life, so (4) he has the right to terminate life, and (5) the creator had the right to call
innocent Amalekites into the protection of paradise. None of these statements even remotely
addressed the question of why an omniscient, omnipotent deity could not have found a
solution to the "Amalekite problem" that didn't entail killing the innocent children. Indeed, all
five assertions assume without proof, or even attempt at argumentation, that the Amalekite
massacre was an event ordered by an omniscient deity. Mitchell had a lot to say about my
duty to produce "quantitative objective evidence," but where is his evidence of any kind that
his god Yahweh was able to "judge the situation fairly"? Where is his evidence that "the giver
of life has the right to terminate life"? Where is his evidence that the "Creaor" had the right
"to call innocent Amalekites into the protection of paradise"? For that matter, where is his
evidence that there was even a paradise for the innocent Amalekites to be called into?
Mitchell can't hide behind the fact that he is not the affirmant and so, therefore, didn't have to
prove anything, because if he is going to offer assertions like these as negative arguments
(affirmations), then he has the responsibility to try to prove them. The record clearly shows
that he supported none of the assertions with evidence, so his attempt to explain away the first
flaw in his corridors-of-time affirmation constituted nothing but question begging.

In trying to get around the second flaw in this affirmation, he resorted to both quibbling and
question begging. First, he argued that Deuteronomy 21: 10-14 addressed not the adoption of
foreign children but the marriage of Jewish soldiers and captive women. Be that as it may, the
fact that the women were captives who were marrying Jewish soldiers surely meant that they
were being integrated into Hebrew society rather than being killed. Although there are
despicable implications in the fact that the Hebrew soldiers were allowed to keep the virgin
Midianite girls alive for themselves (Num. 31:18 ), the lives of these captive girls were at
least spared. Exodus 12:48-49 made provisions for "strangers" or "sojourners" to be integrated
into Jewish society through the rite of circumcision, so I have to wonder if Mitchell was
serious when he said that I was wrong "as usual" in saying that there were provisions in the
Mosaic law for integrating foreigners into Hebrew society. If so, he needs to study his
inspired word of God a little more.

After this quibble, the question begging resumed. Why, if the Israelites had brought the
children back as captives rather than slaughtering them, "a divine law would have been

Volume 1990 - 2002 Issue


Page 407 of 2049
Skeptical Review Edited by Farrell Till
broken," Mitchell said, because "(t)he order to execute the Amalekites did not allow for
adoption" (Ibid., p. 6). But what evidence did Mitchell offer as proof that Yahweh actually did
order the Amalekite massacre? He gave none at all, so he begged the question again.

I know that 1 Samuel 15:2 clearly states that Yahweh commanded the massacre, but the only
thing this proves is that 1 Samuel 15:2 says that the orders to destroy the Amalekites came
from Yahweh. The fact that a holy book may say something doesn't prove that what it says is
true, and Mitchell would agree with this in the case of every holy book except his precious
inspired word of God. In my second article (Spring 1994, p. 4), I pointed this out as problem
number five in Mitchell's "holy-war" scenario. This was how he "responded to it:

He [Till] assumes Yahweh did not actually order the massacre of the Amalekites. He is
obligated to produce objective measurable evidence unequivocally proving his assertion (p.
7).

So Mitchell demands that I prove that Yahweh did not order the Amalekite massacre, and this
is the same Lindell Mitchell who said in the letter published simultaneously with his last
article that he had exposed my "profound ignorance" and reduced my position to "ruin"
(Spring 1994, p. 15). He apparently has no sense of what constitutes burden of proof in
argumentation. The one who asserts the extraordinary is the one who finds the burden of
proof hanging on his back. To assert that a deity ordered a primitive, barbaric king to
exterminate an entire tribe of people is an extraordinary claim. The likelihood that such an
event actually happened is practically nil, so if I question that it did happen, I am merely
demonstrating common sense and reasonable skepticism. The responsibility to prove that such
an unlikely event did not happen is about equal to my responsibility to prove that the tooth
fairy does not leave quarters under the pillows of children who lose their baby teeth. If
Mitchell doesn't understand the burden-of-proof principle any better than his statement above
indicates, he should stick to his country preaching and forget about debating.

This is a good place to put to rest Mitchell's I'm-not-the-affirmant quibble. Although


technically he isn't the affirmant, he certainly should be because of the burden-of-proof
principle just stated. He claims that an omniscient, omnipotent deity ordered the massacre of
an entire nation of people, and that is about as extraordinary as any claim could be. The
burden of proving it rests squarely on his shoulders.

So why isn't he the affirmant? Why isn't he affirming, as Clarence Lavender did in an earlier
issue of The Skeptical Review, that the Hebrew massacres of the non-Jewish tribes in Canaan
were "the highest manifestation of the goodness of God" ("Was It Morally Right for God to
Order the Killing of the Canaanites?" Winter 1993, p. 6)? Well, he isn't because he refused to
defend his position. In the negotiations to have a written debate in TSR on the Amalekite
massacre, he flatly refused to defend the moral propriety of the massacre but did agree to deny
that it was a moral atrocity. Now we know why he took this track. He wanted to quibble about
not having to answer questions or prove anything because he isn't the affirmant.

The third flaw that I pointed out in his negative affirmation about the "Omniscient God
looking down the "corridors of time" was that it merely assumes that the Amalekites were
"vicious beasts" who deserved to be exterminated. I repeatedly urged Mitchell to produce a

Volume 1990 - 2002 Issue


Page 408 of 2049
Skeptical Review Edited by Farrell Till
biblical text that even suggests that the Amalekites of Saul's time were any more corrupt than
any other contemporary nation. He did not produce the text! His failure to produce the text,
however, did not keep him from constantly repeating his assertion that the Amalekites were
an evil people that deserved to die. Please notice these direct quotations from his last article:

Not one Amalekite was killed solely because of nationality. The Amalekites were a
persistently wicked people; consequently God ordered their execution (left column, p. 5).
He [Till] will not allow God to punish a wicked nation but defends killing inconvenient
infants. The Omniscient God cannot determine that the infants of Amalek were destined to
pursue the wicked course their ancestors relentlessly followed for 450 years (center column,
p. 5).
Mr. Till assumes God is unable to judge the situation fairly and punish the wicked
[Amalekites] (center column, p. 6).

If the readers will check these statements in context, they will see that not one time did he
quote any biblical or extrabiblical text to support his contention that the Amalekites of Saul's
day were a "wicked" nation. He didn't, of course, because he can't. So this is just one more
example of his question-begging debating tactics.

His question begging reached its peak on this point in column two, page six, of his last article:

Till says I assume without proof that the Amalekites were vicious. He says passages showing
them to be bushwhackers, cutthroats, and murderers are insufficient. Their ambushing the
Jews in an unprovoked attack is insufficient for Mr. Till.

Here Mitchell attempted to build a case against the Amalekites of Saul's time by referring to
events associated with the Amalekites who lived 450 years earlier, but this tactic won't work.
Even the passages he had in mind (Ex. 17:8-16 ; Num. 13-14 ) can't prove that the Amalekites
who had lived 450 years before Saul were "bushwhackers, cutthroats, and murderers," as
Mitchell asserts; the most that these passages can prove is that the Amalekites of that time
took military action against a horde of three million people entering their territory. Mitchell
called this an "unprovoked attack," but according to his inerrant word of God, the Israelite
army at that time numbered 600,000 (Ex. 12:37 ; Num. 1:45-46 ). How can he possibly
consider the Amaekite military action against an encroaching army of 600,00 an "unprovoked
attack"? If 600,000 armed soldiers should enter a foreign country today, what reasonable
person would accuse that country of an "unprovoked attack" if it took military action against
the intruders? Would any reasonable person characterize the defending country as a nation of
"vicious beasts"?

Let's just assume, however, that the Amalekite military action was an "unprovoked attack."
What would that prove about the "wickedness" of the Amalekites who lived 450 years later in
the time of Saul? I repeatedly pressed Mitchell to produce the book, chapter, and verse that
would support his claim that they were "wicked," and this is how he attempted to prove it:

He [Till] asks why God would punish the Amalekites for something their ancestors did 450
years earlier. In typical Till fashion, he abuses the context of Numbers 13-14, refusing to
acknowledge the Amalekites had over four centuries to repent but instead had continued to be

Volume 1990 - 2002 Issue


Page 409 of 2049
Skeptical Review Edited by Farrell Till
an unrelenting threat to God's people. He is obligated to unequivocally prove that the
Amalekites were not vile savages worthy of death (Ibid.).

Say what again! I am obligated to prove unequivocally that the Amalekites were not vile
savages worthy of death? By what logic does he arrive at this screwball conclusion? There is
absolutely no biblical or extrabiblical passage anywhere to support his claim that the
"Amalekites had [had] over four centures to repent but instead had continued to be an
unrelenting threat to God's people"; it is merely something that he asserts without proof. He
begs the question again. And this man had the gall to claim that he had exposed my "profound
ignorance," reduced my position to "ruin," and left me standing before the world "poor, blind,
and naked"! Now what was it that he said about the legs of the lame being unequal?

So I will leave this point with a final appeal for Mitchell to put up or shut up. Where is the
textual evidence, either biblical or extrabiblical, that supports his claim that the Amalekites
had "unrelentingly" pursued a "wicked course" for 450 years? He should either produce the
evidence to prove the Amalekites were "vicious beasts" who deserved to die or else he should
stop begging the question.

The fourth flaw that I identified in Mitchell's negative affirmation about the "Omniscient
God" ridding the world of a nation of "vicious beasts" is that this scenario has one nation of
vicious beasts exterminating another nation of vicious beasts. I asked Mitchell to tell us what
possible moral good Yahweh had hoped to achieve from such a swap off, but of course
Mitchell wasn't in the mood to chase rabbits. All that he said about this was that "whining
Jews" had "lodged this same objection when they were taken captive by Assyria and
Babylon" (p. 6). He said that God's response was, "I'm not through yet; they shall be
punished." Throughout the entire confusing maze of this answer, Mitchell cited no scripture
references, so it was an answer that gave me nothing to respond to. He concluded this
"rebuttal" argument by saying that if I were familiar with prophetic literature, I "would know
God did punish Assyria and Babylon."

What this has to do with why Yahweh would have one "wicked" nation exterminate another
wicked nation (as would have been the case in the Israelite campaign against the Amalekites)
completely alludes me. For one thing, Mitchell's analogy reverses the situation. In the one
case, the "people of God" defeated a pagan nation; in the other case, pagan nations defeated
the "people of God." As for Mitchell's contention that I should know that "God did punish
Assyria and Babylon," I must take issue. I know that the Bible says that God punished
Assyria and Babylon, but we are back to a point that I made earlier. The fact that a holy book
says something doesn't automatically make what it says true. Mitchell has an obligation to
produce "unequivocal" evidence that God really did punish Assyria and Babylon. The fact
that some ethnocentric mystics called "prophets," living in highly superstitious times,
interpreted calamaties that befell Assyria and Babylon as punishment from God doesn't make
it true.

The fifth flaw that I pointed out in Mitchell's main negative affirmation is that the massacre of
the Amalekites had violated yahweh's own edict against the innocent having to bear the
iniquity of the guilty (Ez. 18:20 ; Dt. 24:16 ). This was Mitchell's "answer" to the argument:

Volume 1990 - 2002 Issue


Page 410 of 2049
Skeptical Review Edited by Farrell Till
No Amalekite bore the iniquity of even one sin committed by an ancestor. They bore the
result of their ancestors' rebellion but not the guilt. That happens every time car-jackers kill an
innocent driver. The victims bear no guilt for the crimes committed against them, but they
obviously bear the consequences (p. 6).

One thing is obvious in this debate: Mitchell doesn't seem to know what the fallacy of false
analogy is. Certainly, the victim of a carjacking bears the consequences of the carjacker's
criminal action, but this isn't even remotely parallel to what presumably happened in the
Amalekite massacre. "I remember that which Amalek did to Israel," Yahweh allegedly said,
"how he laid wait for him in the way when he came up from Egypt. Now go and smite Amalek
and utterly destroy all that they have and spare them not; but slay both man and woman,
infant and suckling, camel and ass" (1 Sam. 15:2-3 ). The reason that Yahweh gave for
ordering the massacre--and the only reason he gave--was the Amalekite attack on the
Israelites 450 years earlier. The Amalekites of Saul's day had obviously not participated in the
attack or in any way aided or abetted it. To massacre them for the attack amounts to having
killed them for something that they did not personally do. In this sense, they bore the iniquity
of their ancestors.

Let's suppose that a person killed an entire family during a carjacking and was never caught
and punished for the crime. Then let's suppose that after this person was dead, his identity was
discovered and the execution of all of his surviving relatives was ordered by the government.
Even Mitchell would be able to see the abysmal injustice of such an action. But let's suppose
that the government waited for 450 years and then ordered the execution of all of the
criminal's descendants. That would be even more morally absurd, but it would be much more
parallel to what happened in the Amalekite massacre than was Mitchell's example of the
victim of a carjacking bearing the consequences of criminal action.

There is no evidence--none whatsoever--that the Amalekites of Saul's day were any more
"wicked" than the other regional neighbors of Israel. If such evidence existed, Mitchell would
have jumped on it "like ugly on a monkey." Therefore, one bitterly embarrassing fact remains
unrefuted by Mitchell: the Bible teaches that God ordered the massacre of an entire nation--
including women, children, and babies--for something their ancestors had done 450 years
earlier. I would think that a debater who has exposed my "profound ignorance," reduced my
position to "ruin," and left me standing before the world "poor, blind, and naked" could give a
satisfactory solution to this major problem in his negative affirmation.

At this point in Mitchell's last rebuttal, the intellectual bankruptcy of his position became
painfully conspicuous. I had pointed out six flaws in his negative attempt (affirmation) to
justify the Amalekite massacre on the grounds that God did it, so that made it morally right.
Readers may check my second article to verify that I began discussing these flaws in the
middle column of page 3 and continued the discussion through the left column of page 4.
Each point was highlighted in bold italicized print. In the middle column on page 6 in his
rebuttal, Mitchell began his "response" to these six points. He introduced each response with a
numerical designation, i.e., first, second, third, four, and fifth. A review of his article,
however, will show that there was no "sixth." In other words, he stopped after discussing my
fifth point and made no effort to rebut the sixth.

Volume 1990 - 2002 Issue


Page 411 of 2049
Skeptical Review Edited by Farrell Till
That sixth point was this: "Mitchell's position exempts Yahweh from the absolute moral law
that presumably exists." Absolute, in the sense that it has in the term "absolute morality,"
means "not limited by restrictions, qualifications, or exceptions; unconditional." In other
words, if a moral law is "absolute," then it is universal and unconditional; it can have no
exceptions to it. If it is immoral for Joe to lie, then it is immoral for John to lie. If it is
immoral to lie in Belgium, then it is immoral to lie in Peru. The moral law against lying, if it
is an absolute law, is univerally and unconditionally applicable to everyone.

Inerrantists like Mitchell recognize the implications of absolute as it applies to the moral law
that proscribes lying. They will, in fact, argue that God cannot lie (Titus 1:2 ), that it is even
impossible for God to lie (Heb. 6:18 ). They would argue that the reason why God cannot lie
is because moral goodness emanates from God's nature. Whatever is objectively "good" must
therefore be a part of God's nature, but whatever is objectively evil cannot be a part of God's
nature. It would be impossible for anything objectively "bad" to be a part of God's nature;
hence, God cannot lie.

This makes good sermon fodder, but it conflicts with the issue at hand. If killing is objectively
"bad," then why wouldn't it be impossible for God to kill? The Bible clearly teaches, however,
that it isn't impossible for God to kill. To the contrary, the Bible teaches that God often did
kill and often ordered others to kill, as in the case of the Amalekite massacre. Mitchell
apparently can't see the absurd inconsistency of this. Lying is objectively bad, so it is
impossible for God to lie. Killing is objectively bad, but it is nevertheless possible for God to
kill and very likely that he will kill. No wonder Mitchell skipped this point in his last rebuttal.
He has said that he will not continue this discussion, but if he should change his mind, let's
hope he will give chase to this rabbit. If killing babies is a violation of absolute moral law,
why wouldn't God have been guilty of violating this absolute law when he ordered the
massacre of Amalekite babies? Whether he is the affirmant or not in this discussion doesn't
matter. A basic negative affirmation of his has been that no moral atrocity occurred in the
Amalekite massacre because God ordered it and God can do no wrong. So he must explain
how a law can be both absolute and conditional. How can it be absolute and yet exempt some
from its restrictions?

I also pointed out problems in any negative affirmation that seeks to justify the Amalekite
massacre on the grounds that God commanded it. Such a claim assumes (1) that an
"Omniscient God" exists, (2) that this "Omniscient God" is morally good, (3) that Yahweh of
the Hebrews is the "Omniscient God," and (4) that Yahweh actually did order the massacre of
the Amalekites. Mitchell tried to brush these aside with more negative assertions that he made
almost no attempt to prove. On the matter of God's existence, he tried to shift to me the
responsibility of proving that God does not exist, but the burden-of-proof principle discussed
earlier puts on him the responsibility to prove that God does exist. He who asserts an
extraordinary claim has the duty to prove that claim, and the assertion that an omniscient god
exists despite the fact that he cannot be seen, heard, felt, smelled, or tasted is certainly an
extraordinary claim. The burden of proving this assertion weighs on Mitchell's shoulders, not
on mine to disprove it.

Mitchell said that none of the theistic arguments (teleological, ontological, cosmological, etc.)
has ever been "successfully refuted" (p. 7). If he really believes this, I suggest that he read

Volume 1990 - 2002 Issue


Page 412 of 2049
Skeptical Review Edited by Farrell Till
Michael Martin's Atheism: a Philosophical Justification. I suspect that much of the material in
this book will sail way over Mitchell's fundamentalist head, but if he will give it a try, he
should at least understand some of the reasons why the Judeo-Christian concept of God is too
self-contradictory for rational people to believe.

Even if Mitchell is right in saying that none of the major theistic arguments has ever been
successfully refuted--and I don't for one moment concede that he is right--that would not
prove that Yahweh of the Hebrews is the omniscient God. Without that proof, he has only the
word of ethnocentric Hebrew mystics, writing in abysmally superstitious times, that Yahweh
was the one true God and that he ordered the massacre of the Amalekites. If Mitchell wants to
accept that as "irrefutable" proof of his position, then why not just let him wallow in his
blissful ignorance?

Mitchell's greatest failure in this debate has been his inability to establish the existence of an
absolute (objective) standard of morality. He cannot say that he has no responsibility to prove
this, because a major negative argument of his has been that "it is impossible to have a moral
atrocity in the absence of an objective moral sandard against which to measure thoughts,
words, and deeds" (Winter 1994, p. 5; Spring 1994, p. 7). In arguing this, he has made a
negative affirmation that he must prove in order to sustain his argument. Needless to say, he
hasn't even come close to proving the existence of his beloved standard of absolute morality.

In my response to Bill Lockwood's article ("The Skeptic's Sword"), which Mitchell had a
copy of when he was writing his last rebuttal, I compared concepts of right and wrong to other
intellectual abstractions. I asked if it is possible, in the absence of an objective standard of
beauty against which to measure light, hue, and perspective, to determine if a sunset is
beautiful. I asked if it would be possible, in the absence of an objective standard of loyalty, to
determine if a spouse or friend or employee is loyal. Mitchell said nothing about this. Perhaps
he will say that he didn't address this point because it was in my response to Lockwood rather
than my article on the Amalekite massacre, but I sent him a copy of my response to
Lockwood precisely so that he would have the opportunity to reply to this argument and
anything else that I said to equate moral conceptualization with intellectual abstraction. If
Mitchell chooses to write a response to this article, let's hope that he decides to chase this
pesky little rabbit.

In an effort to extricate himself from the problem posed by the apostle Paul's recognition that
some Gentiles had lived moral lives without having received a divine revelation, Mitchell
gave us a four-sentence exegesis of the first three chapters of Romans: chapter one shows that
the Gentiles are guilty before God, chapter two shows that the Jews are guilty before God, and
chapter three shows that everyone is guilty. "Any interpretation of the passage must be
consistent with the context," Mitchell said. Well, okay, I won't argue with that, but I see
nothing in any of it that negates the fact that Paul clearly said that some Gentiles had done by
nature the things of the law and had thus become a law unto themselves. Let Mitchell tell us if
the following statement is true or false:

When Gentiles that had not the law did by nature the things of the law, these, not having the
law, were the law unto themselves.

Volume 1990 - 2002 Issue


Page 413 of 2049
Skeptical Review Edited by Farrell Till
This statement is a verbatim quotation from the ASV except that I have changed the tense of
the verbs from present to past. So maybe Mitchell will give this rabbit a run for its money.

Another serious failure in Mitchell's article was his refusal to explain to us how we can
determine what absolute morality is. If an omniscient deity has really given the world a
standard of absolute morality, then surely he made it clear enough for all to understand. If not,
then what's the use of having a standard of absolute morality? If we can't determine what it is,
then we are no better off than we would be if we didn't even have it.

Now Mitchell, of course, will argue that the Bible is that standard of absolute morality, so I
renew again my challenge for him to resolve certain perplexing moral dilemmas that our
modern technology has created. In my correspondence with Mitchell, as well as my response
to Lockwood, I have repeatedly asked for a book-chapter-and-verse resolution of certain
modern moral dilemmas:

Is it morally right, for example, for a woman to allow herself to be artificially inseminated by
the semen of a man she is not married to? Is it morally right to transplant organs? Is in vitro
fertilization morally right? Embryo transplants? Gene-splicing? And more recently we have
learned that human embryos can be cloned. Will it be morally right to clone humans? Was the
right moral decision made last summer when Siamese twins were separated in an operation
that surgeons knew would result in the death of one of the twins? What does Lockwood's
guide to absolute morality tell us about this and the other issues mentioned above?

Since these matters were put to Mitchell and Lockwood, network news programs have
reported that scientists in England have succeeded in removing ova from female cadavers and
then successfully fertilizing them in vitro. When this process is perfected, it will be possible
for dead women to become mothers through zygotic transplantation. Will this be morally
right?

We have to wonder why Mitchell and Lockwood refuse to answer questions like these. They
are so cocksure of their absolute-morality position, yet they seem not to have the answer to
much of anything. All they know is that they must defend the Bible no matter what the
intellectual costs. So in the end, they wind up looking ridiculous by defending the killing of
babies on the dubious grounds that a bunch of Hebrew mystics said that God ordered the
babies to be killed. They cling tenaciously to a flimsy position like this rather than conceding
the more likely possibility that the mystics of superstitious times just mistakenly thought that
their god had ordered the killings. They should wake up and realize that the 21st century is
almost here, which will be a time of remarkable scientific advancement when only the
incredibly gullible will be able to believe in the tribalgod nonsense of biblical times.

The Law of the Jungle


Dave Matson

Volume 1990 - 2002 Issue


Page 414 of 2049
Skeptical Review Edited by Farrell Till
Once upon a time, in the bad old days, ancestor Mitchell became annoyed with ancestor Till.
Since there were no moral laws then, ancestor Mitchell decided to act on his anger. Thus, one
fine morning, he gathered together his hunting buddies and savaged ancestor Till's household.
Many of ancestor Till's relatives were killed and ancestor Till was staked out for the vultures.
Since there were no laws then, life went on as before. Then, one fine morning, when many of
ancestor Mitchell's friends were out hunting mammoths, ancestor Mitchell's household was
ravaged. Many of his family were killed and ancestor Mitchell was staked out for the vultures.

In the months that followed, there were raids and counterraids and the violence spread
throughout the campsites and caves. Every man kept one eye on his neighbors and one hand
on his war club, which made it hard to sleep or have any fun. Worse, a neighboring tribe,
sensing their internal disorder, moved in on their favorite mammoth hunting grounds! There
were no happy campers in that neck of the woods. Nobody was benefiting from this
arrangement except the vultures!

One hot day the grumbling got so bad that some surviving sons of ancestor Till and ancestor
Mitchell got together and declared a truce. The importance of upholding the truce was so
great that each side threatened to exile any of its own members who broke it. It was better to
lose one man than to endanger everyone. Thus, the first moral law was made: Thou shall not
kill a fellow tribesman. Soon, another law was made to prevent theft, a situation that often led
to killing. Still other laws were made to allow for cooperation on the hunt and for mutual
defense. Thus, there was less fighting over mammoths and other goods. Soon the old hunting
grounds had been won back, and everyone in that group had the same sense of right and
wrong in large and small matters. Without having to watch their backs constantly, and with
laws and conventions to minimize friction and handle incidents, the Till people and the
Mitchell people worked together efficiently on complex projects and prospered. Their moral
laws were not perfect, but they were good enough to get the job done.

The point of our little story, which is not intended as a scientific reconstruction, is that
morality is the grease that allows a group to function. People simply cannot live together and
do as they please any more than city drivers can ignore all the traffic rules. Chaos would set
in, and the tribe would soon fall apart or be conquered by its neighbors.

Morality was born in efficient communal living, and that is where we must initially seek its
meaning. It's no accident that the very qualities of moral behavior relate to life within a group.
Kindness, sympathy, honesty, generosity, mercy, loyalty, justice, and courage are qualities
that strengthen the group. Even courage, which may apply to a hermit, takes its highest moral
form in a group. Morality is, therefore, concerned with minimizing disruption within the
group and, equally important, promoting cooperation. Efficient cooperation, in turn, requires
justice. Nobody's going to cooperate on the next mammoth hunt if he is constantly cheated of
his fair share! Thus, we see the origins of morality and justice.

Animal communities, each according to their particular needs, must also obey moral "laws"
and conventions in order to function efficiently. Therefore, it should not surprise you to learn
that many animals exhibit some fine moral traits that would put many humans to shame. It is
beyond the scope of this article to go into the fascinating world of animal morality, but its
mere existence is another proof that morality originates in the needs of a group.

Volume 1990 - 2002 Issue


Page 415 of 2049
Skeptical Review Edited by Farrell Till
Whether we're dealing with animal or human societies, keep in mind that not every society
has found ideal moral solutions. Some societies grind along less smoothly than others. Some
limp along, their demise likely. The insights into morality are not in the particulars but rather
in the general landscape. Having briefly traced some of the major threads defining morality,
we see that morality is neither entirely relative nor completely fixed. No doubt there are many
different rules that would work beautifully for any given society. Furthermore, the needs of a
jungle or tropical island society are not the same as those of a European society, and even in
cases where similar rules might apply there is no guarantee that similar conventions would be
adopted if several good solutions exist. Thus, an island society might go about in the buff
whereas a Victorian society would be shocked by such behavior. One society might allow
premartial sex while another condemns it. To that extent morality is relative to specific
societies.

Yet, there is a core of common bedrock to morality. The basic needs of all societies are quite
similar inasmuch as the basic human needs are similar. Murder and theft, with possible ritual
exceptions, have no place in any well ordered society. Harmfulness in any form, within the
group, is invariably frowned upon. Good behavior towards others in the group is an asset.
How you rate with your hunting buddies could be really important the next time you're
cornered by that tiger! Thus, in a healthy society, morality is never a case of "anything goes."
To that extent, even though the boundaries are a bit fuzzy, we may say that morality is
absolute.

Thus, having explored the rudiments of morality, we must extend it to all of mankind. The
great moralists have seen that the tribal boundaries are, in the final analysis, an artificial
division between human beings. Why should morality be limited to one's tribe? If there is a
great truth here it must be a universal one. We must not do to others what is hurtful to
ourselves unless it is to prevent a greater physical harm. Tribal good manners must now apply
to everyone if humanity is to be one, happy family. Let those who would allot happiness to a
select portion of humanity live among the deprived!

On a deeper level, it is ignorance that separates human from human, and humans from
animals. If we could but perfectly understand the feelings and thoughts and dreams of our
fellow man, even as we learn that absolute truth is not our personal property, we would see
that the boundary between him and us is very faint, that to harm him is very much like
harming ourselves, that to help him is very much like helping ourselves. This natural
sympathy, this empathy for our fellow man, is the keystone in the arch of morality whose
foundations are anchored in the needs of the group. It allows us to see morality as a universal
principle that, in its ultimate form, takes in all thinking creatures according to their needs.

This great arch of morality is built out of stones we can see. There are no supernatural
elements in it. "A man who says, `If God is dead, nothing matters,' is a spoilt child who has
never looked at his fellowman with compassion" (Kai Nielsen). To look at our fellowman
with compassion is to understand his feelings, needs, and dreams, to walk a mile in his shoes,
to become as one through empathy.

Being rooted in the human tribal condition, morality can never be simply what God
commands. That is, something is not moral because God commands it, but rather because it

Volume 1990 - 2002 Issue


Page 416 of 2049
Skeptical Review Edited by Farrell Till
successfully fits the needs of a human society. Thus, once man becomes God's chief concern,
moral law is largely fixed and independent of God himself. God, knowing the human
condition, is free to choose among those moral rules that will work, but he can never be the
standard.

Christianity has, itself, split into two traditions on this point. St. Anslem, Ockham, and
Calvin, for example, asserted that God does not discover morality--He creates it. St. Aquinas,
Albert Schweitzer, and others believe that God knows what is good for mankind, even as a
father knows what is good for his children, and acts accordingly.

The idea that God's will sets the standard for morality runs into theological problems in
addition to the secular problems already mentioned.

For one thing, it reduces the Bible to banality. The Bible reminds us constantly that God is
good. But what does this tell us if "good," by definition, means whatever God is? Simply that
X = X (Kenneth Nahigian, personal letter, Jan. 28, 1994).

Is X = X a profound message? Is this the fruit that divine wisdom offers in the Book of
Books, or have we erred in making God the source of morality? As if that were not enough,
Nahigian points out another problem:

[The] Old Testament patriarchs often compared Jehovah to various heathen gods such as
Chemosh and Ba'al, portraying Jehovah in a more positive light, as if Jehovah is better than
these rivals. But this only makes sense if "better" refers to a common standard, a background
of values independent of the contestants. (What if a heathen had reacted by defining "good" as
the will of Chemosh? Would Moses have found this convincing?)

We have yet another theological problem in assuming that God is the standard of morality.
Who are you, dear mortal, to say that God can't change his mind? Good and evil would
become entirely blurred, and there would be no point in calling God "good" anymore. Even if
God didn't change his mind, the fact that he could have chosen any set of rules makes the
whole matter of good and evil entirely arbitrary. With the concepts of "good" and "evil"
blasted from their logical moorings, we might just as well call God "evil" as "good," sincethe
words would have no meaning for us. The consequence is a theological disaster:

Can Mitchell see that this view undermines every Biblical guarantee, every promise, every
covenant? Think about it. No moral guidelines for God. So, if God decides to lie or cheat on
his promises, then for God, lying becomes good, welshing becomes an act of righteousness. ...
No longer can the Christian reason, "God wouldn't lie to me; it'd be wrong." Lying would be
right, if God had a whim to do it. Morally, all bets are off. Anything goes. Would this please
Mitchell? It is the ultimate in moral relativism (Kenneth Nahigian)!

What if God decided to send all good Christians to hell and all atheists to heaven? As the
Bible-believer awakes to find himself boiling in the flames of hell, even as the first scream of
surprise and horror explodes from his throat, he knows he can't complain. God was morally
right in breaking his promise of a heavenly reward. Why? Because God did it, that's why.
Case closed (adapted from Nahigian)!

Volume 1990 - 2002 Issue


Page 417 of 2049
Skeptical Review Edited by Farrell Till
Yes, even God must be bound by certain standards if the concept of morality is to make sense.
Such was the conclusion the great Christian writer C. S. Lewis reached after a lifetime of
wresting with the problem. In a letter to the American writer John Beversluis (July 3, 1963)
Lewis had this to say:

... The ultimate question is whether the doctrine of the goodness of God or that of the
inerrancy of Scripture is to prevail when they conflict. I think the doctrine of the goodness of
God is the more certain of the two. Indeed, only that doctrine renders this worship of Him
obligatory or even permissible.
To this some will reply, "Ah, but we are fallen and don't recognize good when we see it." But
God Himself does not say that we are as fallen as all that. He constantly, in Scripture, appeals
to our conscience: "Why do ye not of yourselves judge what is right?" -- "What fault hath my
people found in Me?" And so on. Socrates' answer to Euthyphro is used in Christian form by
Hooker. Things are not good because God commands them; God commands certain things
because He sees them to be good. (In other words, the Divine will is the obedient servant of
the Divine Reason.) The opposite view (Ockham's, Paley's) leads to an absurdity. If "good"
means "what God wills" then to say "God is good" can mean only "God wills what He wills."
Which is equally true of you or me or Judas or Satan (C. S. Lewis; quoted from Nahigian).

Therefore, we arrive at an important conclusion. The same standards of morality apply


whether God exists or not! That, in turn, leads to several interesting conclusions.

We may consign to oblivion the claim, so often heard, that atheists have no standard of
morality. Indeed, as you can now see, they draw from the same standard as the most devoted
Bible-believer! The chief difference is that the Bible-believer is confused as to morality's true
origin and, as a result, may commit atrocities in "the name of God." Again, morality
originates in the human condition and not in divine edicts, and the man or woman who
realizes that has the clearest compass to travel life's moral highways.

We may consign to oblivion the claim, so often heard, that the Bible sets the standard for
morality. No book can set the standard for morality. At best, a book might illuminate
workable principles that can be discovered by other means. Many cultural roads lead to
morality. The Japanese, for example, are a highly moral people despite being unfamiliar with
the Bible.

We may consign to oblivion the claim, so often heard, that "anything goes" without God.
Since moral rules are rooted in the smooth functioning of a society, they will be enforced
even if there were no God. Do as you please, and you go directly to jail! (Do not collect
$200.) Rulers, even bad ones, would no more dispense with morality in general (inside their
own societies) than they would dispense with traffic rules. Societies, of course, can become
perverted to various degrees. Indeed, past Christian societies have some of the blackest
records of all. A belief in Jesus offers no magical cure here as any good historian of
Christianity can tell you.

We may consign to oblivion the claim, so often heard, that God cannot be judged on moral
matters by mere humans. We might imagine that God is very wise, that he knows far more

Volume 1990 - 2002 Issue


Page 418 of 2049
Skeptical Review Edited by Farrell Till
than we ever could hope to know, but that doesn't mean we know nothing. We do know
something about our world!

Consider a chess game in progress. On one side of the board sits God, a Grandmaster whose
skill we wood pushers could never hope to equal. You (a mere human) are his opponent.
Although your knowledge is nowhere near that of the Grandmaster, it does not follow that
you know nothing about chess. Indeed, there are many positions you could win even against
the Grandmaster. Should this Grandmaster allow such a position, you would rightly judge his
game to be lost. In charges of gross immorality, it is no defense to claim that God is so much
higher than we are that we cannot hope to know what he's up to. We do have a pretty good
idea of what morality is all about! Human morality deals with conditions down here on Earth-
-not those of inscrutable heaven. Thus, if an action appears grossly immoral with no obvious,
compensating circumstances, then there is no adequate defense.

The idea that God might have to do a great harm in order to insure a greater good is no
defense. It makes God out to be a weakling! Mitchell, for example, suggests the possibility
that God might have looked down the long corridor of time and seen "that the babes of
Amalek were destined to become vicious beasts like their ancestors." God winds up with great
eyesight but no brains! Whatever happened to retraining? If God is all-wise and all-good and
all-powerful, then retraining the Amalekites could not have posed a problem to him. Indeed,
we might expect God to do even better than that! There must be dozens of solutions that are
infinitely better than butchering all the women and children.

One might urge that God, being no part of any human society, is free to do as he pleases.
However, an all-powerful being with perfect empathy towards creatures he needlessly
torments can have no claim to morality. Such is the description of a fiend. We don't need a
second opinion about torturing children for fun. If it's evil for us to do it, then it's no less evil
if a king--or a god--does it. The evil of an act lies in its consequences, its hurt, not in who
does it. One's only hope would be that a sufficiently powerful being did it to bring about a
greater good that was hidden from the rest of us. Unfortunately, the greater the agent's power,
the less need there is for a hurt-now-enjoy-later solution. Thus, the theist, whose God is
absolutely powerful, is deprived of his only escape hatch.

If the slaughter of the Amalekites were done without God's orders, then we would
immediately judge it to be a gross act of immorality. That would be a clear violation of a
universal morality whose basic nature we worked out earlier. Indeed, the particular crime
suggests that the participants have not evolved beyond the primitive, tribal concept of
morality.

To make God the author of this act does nothing to lessen its immorality. Since God is all-
powerful, he is deprived of his only possible defense, i.e., needing to commit a harm to reach
a greater good. Therefore, since we have an adequate understanding of morality, to whose
standards even God is not exempt, we may say that if God directed the massacre of Amalekite
men, women, children, and babies--and even the animals--then he stands convicted of gross
immorality.

(Dave Matson, 330 South Hill Avenue, Pasadena, CA 91106.)

Volume 1990 - 2002 Issue


Page 419 of 2049
Skeptical Review Edited by Farrell Till

Hare Jesus: Christianity's Hindu Heritage


Stephen Van Eck
"Can one go upon hot coals, and his feet not be burned" (Prov. 6:28)? But of course!

Objective and open-minded scholars long ago conceded that Christianity is at heart a
revamped form of Judaism. In the process of its development as something distinct from its
mother religion, it became hybridized with so much pagan influence that it ultimately
alienated its original Jewish base and became predominantly Gentile. The source of this pagan
influence is varied and vague in the minds of most advanced Bible critics, but it may owe
more to Hinduism than most people suspect.

The average person does not connect India with the ancient Middle East, but the existence of
some trade between these two regions is documented, even in the Bible. Note the reference to
spikenard in the Song of Solomon (1:12 ; 4:13-14 ) and in the Gospels (Mark 14:3 ; John 12:3
). This is an aromatic oil-producing plant (Nardostachys jatamansi) that the Arabs call sunbul
hindi and obtained in trade with India.

It is axiomatic that influence follows trade, and the vibrant culture of India could not help but
impact on anyone exposed to it. The influence on Judaism came for the most part indirectly,
however, via the Persians and the Chaldeans, who dealt with India on a more direct basis.
(Indeed, the Aryans, who invaded and transformed India over 1500 years before Christ, were
of the same people who brought ancient Persia to its greatest glory. Persia's name today--Iran-
-is a corruption of Aryan.) The ancient Judeans absorbed much of this secondhand influence
during the Babylonian captivity of the sixth century B. C., and during the intertestamental
period, when Alexandria became the crossroads of the world, intellectuals both Jew and
Gentile were exposed to a variety of ideas, some of which originated on the subcontinent.

The precise pattern of influence was neither observed nor documented, but it can be inferred
from the numerous uncanny similarities in concept and expression, not all of which can be
coincidental. Let us examine the telltale evidence (none of which, it may be added, depends
upon any apocryphal account of the alleged "lost years" of Jesus in India).

Most Christians are familiar with Galatians 6:7 , "Whatsoever a man soweth, that shall he also
reap." Less known is Proverbs 26:27 , "Whoso diggeth a pit shall fall therein, and he that
rolleth a stone, it will return upon him." Both express the Hindu principle of karma (the sum
and the consequences of a person's actions during the successive phases of his existence), but
since no direct connection can be deduced, we'll merely consider it an interesting coincidence
and move on.

The concept of a soul that is distinguishable from the body and can exist independently of it is
alien to Judaism. It is first known in Hinduism. Only after the Babylonian captivity did any

Volume 1990 - 2002 Issue


Page 420 of 2049
Skeptical Review Edited by Farrell Till
such concept arise among the Jews, and it is in the epistles of Paul, the "debtor to both the
Greeks and the Barbarians," that the notion receives its first clear expression. (See 2
Corinthians 5:8 and 12:3 .)

The Brahmin caste of the Hindus are said to be "twice-born" and have a ritual in which they
are "born in the spirit." Could this be the ultimate source of the Christian "born again" concept
(John 3:3 )?

The deification of Christ is a phenomenon often attributed to the apotheosis of emperors and
heroes in the Greco-Roman world. These, however, were cases of men becoming gods. In the
Jesus story, the Divinity takes human form, god becoming man. This is a familiar occurrence
in Hinduism and in other theologies of the region. Indeed, one obstacle to the spread of
Christianity in India, which was attempted as early as the first century, was the frustrating
tendency of the Hindus to understand Jesus as the latest avatar (incarnation) of Vishnu.

It is in the doctrine of the Trinity that the Hindu influence may be most clearly felt. Unknown
to most Christians, Hinduism has a Trinity (or Trimurti) too: Brahma, Vishnu, and Shiva, who
have the appellations the Creator, the Preserver, and the Destroyer (and Regenerator). This
corresponds to the Christian Trinity in which God created the heavens and the earth, Jesus
saves, and the Holy Spirit is referred to as a regenerator (Titus 3:5 ). It is interesting to note,
furthermore, that the Holy Spirit is sometimes depicted as a dove, while the Hebrew language
uses the same term for both "dove" and "destroyer"!

The Trinity was a major stumblingblock for the Jews, who adhered to strict monotheism. The
inherent polytheism in the Trinity doctrine cannot be explained away with the nonsensical
claim that three is one and one is three. Besides, Jesus himself undermined any pretense of
triunity (or omnipotence, for that matter) in Matthew 19:17 , "And he said unto them, Why
callest thou me good? There is none good but one, that is God...." Matthew 20:23 ; Mark
14:32 ; John 5:30 ; 7:16 and 14:28 also contradict the Trinitarian concept.

The Hindu scriptures, which are the oldest in the world, contain a number of astonishingly
familiar expressions. The Upanishads mention things like "the blind led by the blind" (Matt.
15:14 ) and God's being "the same yesterday, today, and tomorrow" (Heb. 13:8 ). The path is
said to be "narrow and difficult to tread" (Matt. 7:14 ). They also make reference to "a voice
from out of the fire" (Ex. 3:4 ) and a man's face shining after encountering God (Ex. 34:29 ).
They refer to those who are "wise in their own conceits" (Prov. 34:29 ; Rom. 12:16 ), warn
against "fleshly desires" (1 Pet. 2:11 ), and advise that "it is not by works alone that one
attains the Eternal" (Gal. 2:16 ]), and "to many it is not given" to know of metaphysical truth
(Matt. 13:11 ). They describe the Self as "smaller than a mustard seed" (Matt. 17:20 ), and
they speak of "the highest knowledge, having drunk of which, one never thirsts" (John 4:14 ).
And how about this: "Man does not live by breath alone, but by him in whom is the power of
breath" (Matt. 4:4 )?

Sounds a little too familiar, I'd say!

Then there is the Hindu epic, the Bhagavad-Gita, a story of the second person of the Hindu
Trinity, who took human form as Krishna. Some have considered him a model for the Christ,

Volume 1990 - 2002 Issue


Page 421 of 2049
Skeptical Review Edited by Farrell Till
and it's hard to argue against that when he says things like, "I am the beginning, the middle,
and the end" (BG 10:20 vs. Rev. 1:8 ). His advent was heralded by a pious old man named
Asita, who could die happy knowing of his arrival, a story paralleling that of Simeon in Luke
2:25 . Krishna's mission was to give directions to "the kingdom of God" (BG 2:72), and he
warned of "stumbling blocks" along the way (BG 3:34; 1 Cor. 1:23 ; Rev. 2:14 ). The
essential thrust of Krishna's sayings, uttered to a beloved disciple, sometimes seems to
coincide with Jesus or the Bible. Compare "those who are wise lament neither for the living
nor the dead" (BG 2:11) with the sense of Jesus' advice to "let the dead bury their own dead"
(Matt. 8:22 ). Krishna's saying, "I envy no man, nor am I partial to anyone; I am equal to all"
(BG 9:29) is a lot like the idea that God is no respecter of persons (Rom. 2:11 ; see also Matt.
6:45 ). And "one who is equal to friends and enemies... is very dear to me" (BG 12:18) is
reminiscent of "love your enemies" (Matt. 6:44 ). Krishna also said that "by human
calculation, a thousand ages taken together is the duration of Brahma's one day" (BG 8:17),
which is very similar to 2 Peter 3:8 .

In fairness, however, one purported similarity needs to be discredited. Skeptics sometimes


cite Kersey Graves in Sixteen Crucified Saviors or Godfrey Higgin's Anacalypsis (which
Graves drew from) in asserting that Krishna was a crucified deity. No such event occurred in
the Gita or in any recognized Hindu scripture. Given the pronounced syncretic tendency of
Hinduism, it is safe to assume that any odd tales of Krishna's being crucified arose only after
the existence of Christian proselytism, in imitation of the Christian narrative. It is neither
authentic to Hinduism nor is Hinduism the source of that portion of the Christian narrative.
The same may be said for most of the purported nativity stories. In my opinion, both Higgins
and Graves are highly unreliable sources and should be ignored.

That notwithstanding, the existence of uncanny similarities in concept and phraseology in


those Hindu writings that are both ancient and authentic leaves Christians in a difficult
quandary. With the historical reality of Indian influence on the Middle East being an
established fact, how can they account for these similarities with anything less feeble than
coincidence, or less bizarre than the notion of "Satanic foreknowledge and duplication,"
which is sometimes invoked to explain the similarities of Judeo-Christian precursors?

I'll close with Ecclesiastes 1:10 , another inconvenient and uncomfortable passage: "Is there
anything whereof it may be said, See, this is new? It hath been already of old time, which was
before us."

(Stephen Van Eck, Route One, Box 62, Rushville, PA 18839.)

Another Flaw in the Perfect-Harmony


Theory
Farrell Till

Volume 1990 - 2002 Issue


Page 422 of 2049
Skeptical Review Edited by Farrell Till
Inerrantists boast that the Bible possesses a thematic unity so amazing that it can be explained
only on the basis of divine inspiration, but the facts do not support this claim. As we have
noted in past issues, the biblical writers, like the theologians of all ages, often disagreed in
important doctrinal matters. One such disagreement concerned Yahweh's willingness to
forego promised vengeance when evil-doers turned away from their wickedness.

The prophet Jeremiah taught that when Yahweh pronounced punishment upon a nation for its
sins, the judgment wasn't necessarily final, for if the nation repented and turned from its evil,
Yahweh would relent:

Then the word of Yahweh came to me, saying... "The instant I speak concerning a nation and
concerning a kingdom, to pluck up, to pull down, and to destroy it, if that nation against
whom I have spoken turns from its evil, I will relent of the disaster that I thought to bring
upon it" (Jer. 18:5-8, NKJV with Yahweh substituted for the LORD).

The statement seems clear enough, and inerrantists will even appeal to it to explain why
certain Yahwistic prophecies appeared at times to go unfilled, as in the cases of the
prophecies against Tyre and Egypt (Ez. 26:3-14; 29:8-14). The solution to the problem is as
simple as claiming that the nations prophesied against repented, so Yahweh spared them as he
had done in the case of Nineveh (Jonah 3:4; 4:11).

In the absence of any textual record that these nations did indeed "repent," one can employ
this dodge with at least a dubious degree of success and claim that just because the Bible
doesn't specifically say that the nations repented doesn't mean that they didn't, but there is a
story of national repentance recorded in the Bible that is impossible to harmonize with this
theory that inerrantists have dreamed up to explain why Yahweh's pronouncements of
judgment were not always executed. In this case, the problem concerns Yahweh's execution
of vengeance on a penitent nation in flagrant violation of his promise to show mercy on
nations that turned from their evil ways.

Inerrantists tout Yahweh as a god who is infinite in all of his attributes. He is infinite in
knowledge, power, goodness, mercy, justice, etc., etc., etc. So if his nature was to "relent of
the disaster" that he had pronounced upon a nation that subsequently repented of its
wickedness, then his infinite goodness, mercy, and justice--and especially his justice--would
require him to relent for all penitent nations, wouldn't it?

In one particular case, however, he refused to relent for a people whose penitence and
religious reformation were perhaps unparalleled in biblical history. When Manasseh, the son
of Hezekiah, assumed the kingship in Judah, he "did evil in the sight of Yahweh" (2 Kings
21:2). He rebuilt the altars of Baal that his father had destroyed, offered his own son in pagan
sacrifice, and "did much evil in the sight of Yahweh, to provoke him to anger" (21:3-6). He
even put graven images in the temple (21:5,7).

So provoked was Yahweh that he vowed to destroy the nation of Judah:

And Yahweh spoke by His servants the prophets, saying, "Because Manasseh king of Judah
has done these abominations (he has acted more wickedly than all the Amorites who were

Volume 1990 - 2002 Issue


Page 423 of 2049
Skeptical Review Edited by Farrell Till
before him, and has also made Judah sin with his idols), therefore thus says Yahweh God of
Israel, `Behold, I am bringing such calamity upon Jerusalem and Judah, that whoever hears of
it, both his ears will tingle. And I will stretch over Jerusalem the measuring line of Samaria
and the plummet of the house of Ahab; I will wipe Jerusalem as one wipes a dish, wiping it
and turning it upside down. So I will forsake the remnant of My inheritance and deliver them
into the hand of their enemies; and they shall become victims of plunder to all their enemies,
because they have done evil in My sight, and have provoked Me to anger since the day their
fathers came out of Egypt, even to this day'" (2 Kings 21:10-15).

Manasseh's evil ways continued after the prophetic judgment was pronounced. He "shed very
much innocent blood, till he had filled Jerusalem from one end to another" (21:16), and then
he died, apparently a natural death, and "rested with his fathers" and was buried in the garden
of Uzza (21:18).

Manasseh's son Amon then became king, but he was assassinated after reigning only two
years. In those two years, however, he carried on the tradition of his father and "did evil in the
sight of Yahweh" (21:20) and worshiped and served the idols his father had installed in the
land (v:21). Then on Amon's death, his son Josiah was made king and reigned for 31 years
(22:1).

It was in the reign of Josiah that the phenomenal religious reforms previously mentioned
occurred. The "Book of the law," presumably lost for some time, was discovered in the
temple during renovation work and was presented to Josiah in the 18th year of his reign (22:3-
10). Upon hearing the book read in his presence, king Josiah, realizing that the
commandments of the law had not been observed by his people, rent his clothes in a gesture
of penitence (v:11). He called together all the elders of Judah and Jerusalem and read the
"Book of the Covenant" to them (23:1-2). He then swore to observe the commandments and
statutes of the book "with all his heart and all his soul" (v:3).

There followed, as I said, a religious reformation that was unparalleled in biblical history.
Josiah ordered the removal of all relics of Baal from the temple and burned them outside
Jerusalem "in the fields of Kidron" and carried their ashes to Bethel (v:4). He removed from
office all the idolatrous priests who had been ordained by his predecessors (v:5) and burned
their wooden images. He tore down all the ritual booths in the pagan "high places" and cast
out their priests from Geba to Beersheba (vv:7-8), put an end to human sacrifices to the god
Molech (v:10), and destroyed many other pagan worship sites too numerous to detail (vv:11-
15). He even carried his reforms into the northern kingdom of Israel, where he destroyed all
the pagan shrines and executed all the priests "of the high places" (vv:18-20).

On his return to Jerusalem, he ordered a Passover celebration the likes of which "surely had
never been held since the days of the judges who judged Israel, nor in all the days of the kings
of Israel and the kings of Judah" (vv:21-22). He ordered an end to the cultic practices of
mediums and spiritualists and the worship of "household gods and idols" (v:24). The story of
Josiah's reforms ends with this statement: "Now before him there was no king like him, who
turned to Yahweh with all his heart, with all his soul, and with all his might, according to all
the Law of Moses; nor after him did any arise like him" (v:25). David was a man after
Yahweh's own heart (1 Sam. 13:14), who had done "that which was right in the eyes of

Volume 1990 - 2002 Issue


Page 424 of 2049
Skeptical Review Edited by Farrell Till
Yahweh and turned not aside from anything he [Yahweh] commanded him all the days of his
life" (1 Kings 15:5), so if Josiah's godliness exceeded even David's, his personal character has
to stand without parallel in the Old Testament.

One would think, then, that if any nation were ever entitled to have Yahweh "relent" of the
disaster he had pronounced upon it, Judah under the reign of Josiah would certainly have
qualified. But it didn't. After all the reforms of Josiah had been described in detail, the writer
of 2 Kings made this astonishing announcement:

Nevertheless Yahweh did not turn from the fierceness of His great wrath, with which His
anger was aroused against Judah, because of all the provocations with which Manasseh had
provoked Him. And Yahweh said, "I will also remove Judah from My sight, as I have
removed Israel, and will cast off this city Jerusalem which I have chosen, and the house of
which I said, `My name shall be there'" (23:26-27).

From this one can only conclude that it wasn't always true that when Yahweh spoke
"concerning a nation and concerning a kingdom, to pluck up, to pull down, and to destroy it"
that he would "relent of the disaster" that he had thought to bring upon it if the nation turned
from its evil. Judah certainly turned from its evil, in terms of the biblical sense of evil, in the
reign of Josiah, yet despite this national repentance, Yahweh refused to relent of the disaster
he had pronounced upon it. One chapter later, the writer of 2 Kings concluded his book with
an account of Judah's destruction by the army of Nebuchadnezzar. The repentance of an entire
nation brought no mercy from the infinitely just Yahweh. So what does this do to the claim
that the Bible is so unified in its themes that only divine inspiration can explain its amazing
harmony?

Inerrantists cannot argue that Jeremiah prophesied in a time different from the era of Josiah,
because Jeremiah claimed that he received his revelations from Yahweh "in the days of Josiah
the son of Amon, king of Judah, in the thirteenth year of his reign" (1:2). Jeremiah, then, was
a contemporary of Josiah, so if Jeremiah received his "revelations" in the 13th year of Josiah's
reign, he received them before the reforms that began in the 18th year of Josiah's reign.
Therefore if Jeremiah's words in 18:7-8 were true when he received them from Yahweh, they
should have been applicable to Josiah and the penitent nation of Judah under his reign. But
they weren't. Why?

Perhaps some inerrantist will accept our offer of space to explain how that Yahweh's reaction
to Josiah's reforms was consistent with the promise he had made through Jeremiah to
withdraw his wrath from a nation that turns from its evil. Having seen similar offers in past
issues repeatedly spurned, our readers probably won't hold their breaths until we get a taker.

Evidence That Doesn't Demand a Verdict


Mark Smith

Volume 1990 - 2002 Issue


Page 425 of 2049
Skeptical Review Edited by Farrell Till
Josh McDowell, in his books entitled Evidence That Demands a Verdict, makes a big deal of
the value of testimony within the New Testament in establishing the truth of the message.
Other writers have cited courtroom examples of how testimony is used. Testimony, in the
absence of hard evidence, is sometimes all a jury has to go on.

But how did the people in the New Testament look upon testimony? Did they think it a valid
foundation upon which to base their eternal destinies? Or did they tend to throw out
unfounded assertions and await hard evidence?

The people of Athens rejected the testimony of Paul concerning Jesus's resurrection from the
dead (Acts 17:32). Paul had given them no hard, tangible evidence--just words. Paul, as Saul,
rejected the testimony and visions of the martyr Stephen (Acts 7:56-60, 8:1-3). Stephen had
only his words to offer Paul, and Paul found that kind of evidence to be unacceptable.

The Apostle Thomas, who had been with his fellow apostles for several years, who knew
them on a daily basis, knew enough not to trust them any farther than he could throw them!
When the other 10 apostles told Thomas that Jesus had been raised from the dead, he counted
their personal eyewitness testimony to be insufficient evidence. In other words, he thought
they were lying. He demanded hard evidence, and refused to trust mere testimony (John
20:24-25).

And all eleven apostles flatly rejected as absurd the testimony of the women who claimed to
have spoken to angels at the tomb concerning the resurrection of Jesus. The eyewitness
accounts of the several women "appeared as nonsense," and the apostles refused to believe
without hard, tangible evidence (Luke 24:9-11).

Who are we to break with Bible tradition and example? We should not be so gullible as to
believe outlandish, incredible tales, 2000 years removed and filtered through countless
unknown editors--tales that were rejected by the very people who knew the honesty and
trustworthiness of the witnesses! Thomas rejected mere words and demanded hard evidence.
The 11 apostles rejected the mere words of women and demanded hard evidence. Jesus,
according to the Bible, even offered hard evidence, as found in Luke 24:39.

See my hands and my feet, that it is I myself; touch me and see, for a spirit does not have
flesh and bones as you see I have.
And again, Jesus is reported to have said to Thomas in John 20:37, "Reach here your finger,
and see my hands; and reach here your hand, and put it into my side; and be not unbelieving,
but believing."

There is no reason we should settle for anything less. They refused to believe without hard
evidence, and even Jesus, if the account is true, was sympathetic to their demands for real
evidence. Jerusalem refused to trust Jesus, the apostles didn't trust each other or their female
friends, and Saul (Paul) refused to trust what any Christian said. Maybe that should tell us
something about the quality of the witnesses!

It seems to me as if modern Christians are asking us to swallow a stew without thinking--a


leftover stew that's 2000 years stale, that has gone by dozens of unknown cooks, each

Volume 1990 - 2002 Issue


Page 426 of 2049
Skeptical Review Edited by Farrell Till
throwing in their own concoctions. We are being asked to swallow, whole, stale evidence,
evidence that when fresh was rejected by Saul/Paul, spit out by Thomas, and vomited up by
the rest of the apostles! Thomas rejected the fresh testimony of his best friends! And we are to
believe the same testimony, now 2000 years stale, from people that to us are total strangers???
NO!!!

We're not talking about how history is determined. "Well, all we have are books that say
George Washington existed," Bible believers will argue. "It's the same with Jesus." Oh, no, it
isn't! Washington's best friends never denied he was president or whatever. Jesus's friends did
deny that he had risen from the dead. And Washington never claimed to be a god! Jesus did.
And Washington never asked that you spend your whole life worshiping him or even become
a martyr and die for your belief in his godhood. Jesus did. And Washington never claimed to
walk on water. He used a boat to cross the Delaware! And, most important, Washington
doesn't have millions of deluded fanatics out trying to convince you that he rose from the
dead, can live in your heart, or forgive you of your "sins." Failure to believe in Washington
just means you're stupid. But failure to believe in Jesus, according to the Christians, means
you are doomed to eternal hell fire! Nobody asked you to die for Washington, but they'll ask
you to die for Jesus. A BIG difference!

I stand upon the same principles as those freethinkers in the New Testament, rejecting mere
testimony of such UNbelievable fairy tales. Fantastic claims require extraordinary evidences.
Be like the apostles: don't be swayed by clever sermons or someone's testimony. Demand a
personal, in-the-flesh visit from this all-powerful god of the universe. Christians say that he is
everywhere--even peeping over your shoulder as you read this. Ask! Demand a visit. Don't
settle for anything less.

Know Evidence, Know Belief


NO EVIDENCE, NO BELIEF!!!
(Mark Smith, P. O. Box 3065-192, Garden Grove, CA 92642.)

The Geisler-Till Debate


Farrell Till
On March 29th, I "debated" Dr. Norman Geisler at Columbus College Columbus, Georgia.
The issue was the resurrection of Jesus. Most readers will recognize that Norman Geisler is
one of the premier spokesmen for the Bible inerrancy doctrine. In fundamentalist circles, his
book, When Skeptics Ask, has joined the ranks of Gleason Archer's Encyclopedia of Bible
Difficulties and John Haley's Alleged Discrepancies of the Bible as a handy reference volume

Volume 1990 - 2002 Issue


Page 427 of 2049
Skeptical Review Edited by Farrell Till
for Christians who wish to have a ready-made, how-it-could-have-been explanations of Bible
difficulties to use in their discussions with skeptics.

Because of his reputation, I was expecting Dr. Geisler to be a formidable opponent who
would probably confront me with challenging arguments, but instead I found him to be
incredibly shallow. For one thing, he did nothing but read manuscripts of speeches he had
written prior to the debate. This was true even after my rebuttal of his opening speech. He
simply returned to the lectern and read another previously prepared speech in which, believe it
or not, he referred to my failures to respond to certain points which he had made in his
opening speech (which I had in fact addressed), as if he could have known prior to the debate
what I would and would not respond to in my rebuttal. Even his final two-minute, wrap-up
speech was read from a previously written manuscript.

At the beginning of my second speech I said to the audience that I had several debates but that
this one was the strangest I had ever participated in, because my opponent had responded to
my rebuttal speech by reading a manuscript that he had written prior to the debate. "How
could he know before the debate what I would say and not say?" I asked. At this point, in a
rare moment of spontaneity, Geisler spoke up from his seat and said, "I read your book." To
which I said, "That's strange, because I haven't written a book." He said nothing in response to
this.

Geisler's opening speech consisted of an effort to establish the "reliability" of the New
Testament manuscripts. He referred to over 5,366 copies of "existing" New Testament
manuscripts, which scholars have studied and compared and found to be "ninety-nine percent
free of significant variances." From this, Geisler somehow reached the conclusion of the
"reliability" with which the manuscripts had been copied by ancient scribes proved that
everything the manuscripts said had happened exactly as recorded. In my first rebuttal, I
asked the audience to assume that those 5,000 manuscripts were 100% free of variation. Even
if that were true, that would in no way prove that the events recorded in the manuscripts had
actually happened; it would only prove that the manuscripts agreed in what they said.

I presented three reasons why rational people cannot believe the New Testament resurrection
accounts: (1) resurrected savior-gods were common-place in the pagan religions that
flourished before, during, and after the time Jesus of Nazareth allegedly lived, (2) the claim
that a dead man was restored to life is an extraordinary claim that required extraordinary
proof, and (3) the only proof that Geisler can offer in support of his resurrection claim is
hearsay in nature. Since Geisler spoke entirely from previously prepared manuscripts, he
made no attempts to respond to these points, except when they came up during the 30-minute
period of responses to questions from the audience.

In developing point one, I referred to the widespread pagan belief in resurrected savior-gods
like Osiris, Dionysus, Tammuz, and Krishna, all of whom had had thousands of religious
adherents long before the time of Jesus. The only attempt that Geisler made to rebut this
argument was made during the question-answer session when he incorrectly said that bodily
resurrections had not been claimed for any of the pagan saviors, so they were not "parallel" to
the resurrection of Jesus.

Volume 1990 - 2002 Issue


Page 428 of 2049
Skeptical Review Edited by Farrell Till
In his first speech, Geisler had referred to the apostle Paul's claim that Jesus had appeared to
"500 brethren at once" after his resurrection (1 Cor. 15:6), so in making my second point, I
asked Geisler what was extraordinary about someone saying, particularly at that time when
belief in resurrections was commonplace, that others (even 500 others) had seen a resurrected
man. I asked him if he would believe a modern-day resurrection claim even if 500 people
should say that they had witnessed it. He ignored the question.

In making my third point, I emphasized that the weakness of the testimonial evidence for the
resurrection lies in the fact that all of the testimony was either hearsay in nature or, as in the
case of the apostle Paul, visionary. We pay no serious attention to people today who claim to
have visions, so why should we believe someone who allegedly had a vision 2,000 years ago?
As for the testimony of the other "reliable" witnesses, it was all hearsay. Scholars know that
the apostle didn't write the Gospel of Matthew and that the apostle John didn't write the
Gospel of John, so these writers were not the "eyewitnesses" that Christian apologists claim
that they were. So all that we have is a case of unknown writers saying that certain women
said that they had found an empty tomb and had then seen the resurrected Jesus. "But what
did Mary Magdalene ever write herself?" I asked Geisler. "What did Salome ever write?"
"Who was she anyway?" "What did Joanna write?" "And who was she?" These were
questions that Geisler ignored as well as my demand that he tell us just who those "five
hundred brethren" were that the apostle Paul cited as witnesses of the resurrection. Where did
they live? When did they see Jesus? I challenged Geisler to tell us the name of just one of
those five hundred. He didn't do it, of course.

I usually leave a debate thinking, "This was the weakest opponent I have had yet," but given
Geisler's background and reputation, I have decided that the weakness is not in my opponents.
It just had to be that there is no credible evidence at all to support the views of those whom I
debate. If there were, then surely one of them would have cited some convincing reason to
believe in biblical inerrancy, the resurrection of Jesus, the credibility of prophecy fulfillment,
etc. The evidence just isn't there to support any of those claims, and that is why the opposition
appears so weak.

For $1, a written transcript of the debate can be obtained from Apologetics Press, 230
Landmark Drive, Montgomery, AL 36117. Video tapes are available on two-week rentals
from The Skeptical Review for $1 to cover the cost of mailing.

From the Mailbag


I have never subscribed to Skeptical Review, but I have read one issue. Enclosed is $4 for a
subscription. If I have missed any issues that would be included in this subscription, please
send them.

Do not think me foolish for enclosing cash. If you had been a Christian, I would have sent a
federal money order, registered mail, return receipt.

Volume 1990 - 2002 Issue


Page 429 of 2049
Skeptical Review Edited by Farrell Till
(Adam P. Currie, 3519 22nd Street, Meridian, MS 39301.)

We recently received our first copy of The Skeptical Review. I wasn't sure quite what to
expect when I sent off for it and was therefore knocked for a loop when I read it. Several
weeks later, I am still impressed.

And several weeks and numerous re-readings later, I am still left dazed and reeling by Lindell
Mitchell's incredible dismissal of the Amalekite massacre as possibly a calamity but not an
atrocity. I have trouble believing anyone could seriously argue as he does--or that his friends
and co-religionists are not shocked to hear him put forth such a defense of slaughter.

Please send me the 3 booklets mentioned in the newsletter:

1 Laws-Till Debate....... 2.50


1 Jackson-Till Debate.. 2.50
1 Prophecies.................. 2.50

Also please send me all 16 back issues of TSR. Enclosed is a money order.

This newsletter deserves a much wider distribution. Good wishes on your continued success!

(Elizabeth Dyak, 304 Sunnyland, rear, Pittsburgh, PA 15227.)

EDITOR'S NOTE: Ms. Dyak may rest assured that most of Mitchell's colleagues will not be
at all shocked at his defense of the Amalekite massacre. The same defense that he made is
what they preach to their gullible pulpit audiences.

As I was reading Ms. Dyak's letter, a thought occurred to me. Mitchell did indeed admit that
the Amale- kite massacre was a "terrible calamity" (Winter 1994, p. 4), so I wonder how he is
able to know that it was a calamity in the absence of an objective (absolute) standard of
calamity against which he can measure "thoughts, words, and deeds."

Please RUSH me a copy of your journal. I am surrounded by literalists... daily letters to the
editor in 2 of 3 papers I read. Themes: USA is a Christian nation founded on the Bible.... It is
a lie and a myth that there is separation of church/state.... Secular humanism is a religion and
is being taught in public schools.... There is only one God, our Lord and Savior....

(N. Z. Cassidy, P. O. Box 72, Fallbrook, CA 92088-0072.)

I heard mention of The Skeptical Review on a computer BBS, and I think I'd be interested in
taking a look. Could you add me to your mailing list? Thanks.

(Dave House, P. O Box 111, Essexville, MI 48732-0111.)

Volume 1990 - 2002 Issue


Page 430 of 2049
Skeptical Review Edited by Farrell Till

In the January/February issue of The Humanist, I saw your ad for a quarterly journal refuting
Biblical fundamentalism. I have not had the opportunity to see this journal and would very
much appreciate it if you would put me on your list for that "free first-year subscription"
mentioned in the ad.

My own former fundamentalist perceptions not only failed to redress various personal
problems in my own life but in many ways exacerbated those problems to the point that I now
find myself in prison. Fortunately for me, I have had the past 15 years to reevaluate and sift
through much in my life, including those entrenched fundamentalist beliefs and values. I
should explain that my former beliefs were no small thing in my former life. I was
indoctrinated in fundamentalism since a small child, was reading portions out of the Greek
New Testa- ment before I graduated from high school, and then went on for 3 years of
theological training at one of the most fundamentalist schools in the nation at that time.
Anyway, over those past years I have reformed a good deal of my previous views, including
fundamentalism, its position of "literalism," "inspiration," and bibliology in general. So I
would very much like to read some of your reflections on similar subjects.

Please send that subscription to me at the address below.

(Bob Dornbusch, OTF-160255, P.O. Box 5000, Carson City, MI 48811-5000.)

Thank you for The Skeptical Review Winter 1994. I am very well pleased with it. It is so very
very awful that I never got in contact with people like you sooner, like 1979 or any time
afterwards. I am still very frightened from what the Bible has done to my poor self since 79. I
still kind of believe it, but when you tell me about Abraham and Sarah and Abimelech, I
wonder just how many other stupid things in the Bible I've overlooked! Well done, Mr Till! I
read your letter over and over front to back. I love it!!! I would love to read your Prophecies:
Imaginary and Unfulfilled....

I am still unsure of that Holy (unholy) Bible, but please please by no means, don't you dare
take me off your subscription list. If all your letters are as great as volume five, number one,
winter 1994, then I am going to be on my way to recovery....

(William Kotis, 2199 Kamehameha Highway, Honolulu, HI 96819.)

The Arizona Student Atheists would like all of TSR on this disk [enclosed] so that we can
distribute some articles to hungry minds.

(Erik Mickelson, 9827 Longford Drive, Tucson, AZ 85741-9501.)

I just received my first issue of The Skeptical Review today. I love it!!! In fact, I was so
impressed with it that I would like to order back issues for every other issue ever published. I
have been challenged by some fundamentalist (and I don't mean that in a derogatory sense)

Volume 1990 - 2002 Issue


Page 431 of 2049
Skeptical Review Edited by Farrell Till
Christians to investigate the evidence for the alleged physical resurrection of Jesus. They
maintain that the Bible is the inerrant Word of God and that the New Testament is the most
historically reliable set of ancient manuscripts we possess today. They also happen to be
major fans of Josh McDowell, whom I am sure you are familiar with. Needless to say, I found
your publication a refreshing perspective after all of their one-sided tracts and books. And I
hope you will publish a transcript of your debate with Norman Geisler. That would be worth
its weight in gold to me....

Would you ever be willing to come out to Seattle and do a debate on New Testament
reliability, the resurrection, etc.? In a month, we are sponsoring a debate on New Testament
reliability between an SPU professor and a professor at the University of Washington.
However, both of them claim to be believers of one sort or another. It would be nice to have
someone like you for a future debate so that we could have a believer versus a nonbeliever.

I also read your response to the letter mentioning computer BBSes. In response (and in
gratitude for the free year's subscription), I typed in your article entitled, "No Morality
Without the Bible?" and posted it on the Internet. Specifically, I posted it to the Usenet
newsgroups alt.atheism and talk.religion.misc. And I gave your address so that people may
contact you if they were interested in a subscription.

(Jeffery J. Lowder, Seattle Pacific University, SUB Box 1692, Seattle, WA 98119-1997.)

EDITOR'S NOTE: I will be happy to debate the issue of the resurrection (or other biblical
matters) at Seattle Pacific University if it can be held at a time compatible with my teaching
schedule or after I retire in 1995. May I suggest that the student organization invite Josh
McDowell to represent the Christian position. After securing his agreement, they may then
wish to teach pigs how to fly.

Let me play the prophet. The personal computer, probably more than any other single factor,
will eventually drive the final nail into the coffin of Bible inerrancy. The 18th and 19th
centuries produced enough works in biblical criticism to kill the myth of inerrancy ten times
over, but the general public was unaware of the information because it was suppressed.
Clergymen never told their congregations about it, and few libraries stocked the books that
published it. Whatever was uncomplimentary to Christianity was kept from the sheep.

In the information age that we are now living in, the suppression can no longer be sustained.
Facts about the Bible's origin, its flagrant plagiarism of pagan beliefs, and its inconsistencies
and contradictions now flow freely through computer networks. A revolution in religious
thinking is inevitable.

Now who will be the first bibliolater to tell me that the "word of God" will live and abide
forever?

Thanks for the loan of the tapes of your debate with "Buster" Dobbs. You may receive this
card first, but the tapes are on their way. I enjoyed the debate but found some parts
embarrassing to sit through. In More Than a Carpenter, Josh McDowell says atheists and

Volume 1990 - 2002 Issue


Page 432 of 2049
Skeptical Review Edited by Farrell Till
agnostics resort to irrational methods of argumentation when they reject the supernatural as a
valid explanation of life's mysteries. I felt that Mr. Dobbs worked from the same premise. His
position is not far removed at all from a bumper sticker I saw: The Bible said it, I believe it,
and that settles it. I'm not equipped to critique the whole debate, but Mr. Dobbs seemed very
peeved at your resistance to accepting his generalities as well as your insistence on debating
specifics. I have run into the same tactic, i.e., if I can't explain how the universe came to be
then that "proves" that the only other available explanation is Genesis. I've known people who
consider me a satanist because I do not believe in the existence of a monotheistic god. It's
hard to negotiate with such a point of view....

(Bill Courtney, 5620 183 SW, Apt. 210, Lynnwood, WA 98037.)

I'm sending you something I'm copying from The Workbook on Spiritual Disciplines by
Maxie Dunnam. The piece is called "The Cloud of Unknowing" by an anonymous writer.

"Silence is not God, nor speaking; fasting is not God, nor feasting; solitude is not God, nor
company.... He lies hidden between them and no work of yours can possibly discover Him
save only your heart's love. Reason cannot fully know Him for He cannot be thought,
possessed or discovered by the mind. But loved He may be and chosen by the artless,
affectionate longing of your heart. Choose Him, then, and you will find that your speech has
become silent, your silence eloquent, your fasting a feast, your feasting a fast, and so on.
Choose God in love... for this blind thrust, this keen shaft of longing love will never miss the
mark, God Himself."

Mr. Till, I promise to pray for you every day that you may return to the joy of your First
Love!

(Nikki Kaley, 505 West Washington, Lewistown, IL 61542.)

EDITOR'S NOTE: As a former Christian fundamentalist, I can certainly understand Mrs.


Kaley's motivation, and I know that she wrote this letter with the best of intentions. However,
I hope she will somehow understand me when I say that I would much prefer that she attempt
a logical response to my arguments rather than that she pray for my return to my "First Love."
Something that she probably doesn't realize is that I still to this day have nightmares in which
I find myself sitting on the front pew with Bible in hand, waiting for the congregation to
finish the hymn that will be my cue to step into the pulpit and start preaching. When I wake
up, I experience inexpressible relief from knowing that it was only a dream and that I won't
have to stand in the pulpit and say things I know I don't believe as I did back when I was
struggling to come to terms with my unbelief. So why would I ever consider returning to
something that has left psychological wounds that thirty years have not been long enough to
heal?

Mrs. Kaley's letter probably came in response to a letter of mine that was published in a local
newspaper in which I pointed out parallels in the lives of Jesus and the many pagan savior-
gods who were worshiped long before him. Rather than try to disprove my arguments, she
chose to ignore them completely, just as if they didn't matter, and to her, of course, they don't

Volume 1990 - 2002 Issue


Page 433 of 2049
Skeptical Review Edited by Farrell Till
matter. Nothing matters except her blind allegiance to a faith system that hasn't a shred of
logic in it. All the prayers that she could possibly utter the rest of her life if she did nothing
else but pray continuously could ever make me respect that kind of mentality.

As for the prayers she is uttering for me, I wish I had a nickle for everyone who has told me
he is praying for me. There must be thousands of people out there praying for me at any given
moment. So I wonder when we can expect to see any results from all these prayers. God
wants all men to be saved: "This is good and acceptable in the sight of God our saviour, who
would have all men to be saved, and come to the knowledge of the truth" (1 Tim. 2:3-4).
Therefore, if the Bible is truly the infallible word of God, as Mrs. Kaley no doubt believes,
then she must agree that God certainly wants Farrell Till to be saved. So I see a dilemma for
her when this scripture is considered in conjunction with 1 John 5:14-15: "And this is the
boldness which we have toward him, that, if we ask anything according to his will, he heareth
us: and if we know that he heareth us whatsoever we ask, we know that we have the petitions
which we have asked of him." Now please notice that this passage does not say that if we ask
anything according to God's will, he may hear us and grant it; it flatly says that we can know
that he hears it and will give us what we ask.

From this, I can only conclude that the New Testament promises that God will hear and grant
the prayer of any Christian who asks ANYTHING in accordance with God's will. So since it
is obviously God's will that Farrell Till be saved, then why haven't the many prayers on my
behalf been answered? I once presented this dilemma to a Baptist preacher who had told me
that he was praying for me. His response was that I am not dead yet, so there is still the
possibility that I will yet be "saved." I suppose that he is technically right, but if I should die
without returning to my former beliefs--and I really don't believe there is even a remote
possibility that I will return--would this not constitute logical proof that the Bible is not
inerrant?

We could even extend the dilemma by noting that the passage in 1 Timothy clearly says that
God wants all men to be saved. Since it is the will of God that all men be saved, in order to
have all men be saved, one should only have to pray for the salvation of all men to happen.
Otherwise, the statement in 1 John 5:14-15 is erroneous. However, the salvation of all men
cannot occur without causing other problems for the Bible inerrancy doctrine. The Bible
clearly teaches that some men will be lost; in fact, it teaches that most men will be lost (Matt.
7:13-14). If most men will be lost, then obviously all men can't be saved. If all men can't be
saved, even though a faithful, believing Christian might pray for all men to be saved, then 1
Timothy 2:3-4 and 1 John 5:14-15 cannot both be true statements.

This is just one dilemma that Mrs. Kaley will discover when she applies logic to her studies of
the Bible. There are many more that I could point out. However, since logic is of no concern
to her and those of like persuasion, I don't suppose it will matter to her that she has committed
herself to an activity (prayer on my behalf every day) that is illogical.

Please remove my name immediately from your mailing list. I am a FIRM believer and NO
ONE can take my experiences from me.

Volume 1990 - 2002 Issue


Page 434 of 2049
Skeptical Review Edited by Farrell Till
I'm not interested in your TRASH-- and I have more important things to do in this life than
pick the Bible apart. What do you expect to gain from it anyway? Silly! If you don't believe in
it--DON'T read it. If you don't like what you hear or see on the air waves (if your wrist isn't
broken), turn the dial or switch. You have the option to believe or not. I have no desire to
discuss or argue any of your issues. Life is too precious to waste it in being rebellious or
critical.

I know Christ exists. I've had two personal experiences some years ago-- and no one can
change my mind or my love for Him.

I shall continue to pray for you. Your paper goes straight in the trash.

(Mildred M. Bahn, 636 Pearl Street, Lancaster, PA 17603-5010.)

EDITOR'S NOTE: Ho hum, another Christian who has had a "personal experience"! If I had a
penny for every Christian who has had a personal experience with Jesus, I could retire a rich
man. The only problem, of course, is that "personal experiences" are purely emotional and
psychological, so in terms of evidence or proof they are absolutely worthless. Moslems have
"personal experiences" and so do Mormons, Hindus, Sikhs, and you name it. If it is a religion,
it has its believers who have had personal experiences.

If a Moslem should tell Ms. Bahn that he is a FIRM believer in the prophet Mohammed and
has had two "personal experiences" that NO ONE can take away from him, so therefore he
KNOWS that Mohammed is Allah's prophet, would this mean anything to her? Would she see
this testimony of "personal experience" as proof that Islam is God's true religion? Certainly
not--yet she expects us to accept her testimony of personal experiences as absolute proof that
Christ exists.

Of all the arguments that Christians use, the personal-experience one is by far the most
idiotic. If I should claim that I have had two "personal experiences" in which God informed
me that the Bible is a hoax that Satan perpetrated in order to deceive the world and that NO
ONE can take those experiences from me and make me believe that the Bible is God's word,
how many Christians would accept this as proof that their view of the Bible is erroneous?
None of them would, yet they seem to think that we should accept without question that they
have had "personal experiences" that confirm the truth of their belief in Jesus. That anyone
would even offer such "evidence" as proof of his faith merely shows that he is incapable of
logical reasoning. That is exactly the situation that Ms. Bahn finds herself in.

She wonders why we read the Bible if we don't believe it and why we don't just turn the dial
or switch when we hear something about it on the air waves. Well, I'll make a deal with Ms.
Bahn. If she will get her side to do the same and just not read the books and magazines and
see the movies and TV programs they find objectionable rather than trying to ban them
everywhere for everyone and if she will get her side to stop trying to force its "Christian-
nation" agenda onto our society through the various tactics that Bible fundamentalists are
using to influence legislative and judicial decisions, then I will stop publishing The Skeptical
Review. Until then, we will continue to publish evidence that clearly shows that people like
Ms. Bahn are living in fantasyland.

Volume 1990 - 2002 Issue


Page 435 of 2049
Skeptical Review Edited by Farrell Till
She said that "(l)ife is too precious to waste it in being rebellious or critical," and so what did
she do? She sat down and wrote a critical letter that demonstrates a rebellious attitude toward
the world's best biblical scholarship. No, we are not laying claim to being the world's best
scholarship, but we do claim to have that scholarship on our side. The Bible inerrancy
doctrine has been so thoroughly discredited that few serious scholars believe it anymore.
Reputable seminaries no longer teach it to their students, and their professors freely admit that
the Bible is riddled with myths and legends. In the position that she tries to defend, then, Ms.
Bahn has simply demonstrated that she is willing to waste part of her precious life to rebel
against what the world's foremost scholars say about biblical origins. In this respect, she puts
herself into the position of those who were outraged by Galileo's an- nouncement of a
scientific discovery that conflicted with their view of what the Bible taught. Who knows?
Maybe some of those who opposed Galileo had had "personal experiences" that made them
absolutely certain that the sun revolved around the earth.

I will agree with Ms. Bahn in one thing: life is indeed precious. It is much too precious to
waste clinging to a faith that had its origin in superstitious times when mystics thought that
God routinely talked to them and commanded deeds as ignominious as the murder and rape of
children (Num. 31:17-18). We aren't praying for Ms. Bahn, of course, but we do sincerely
hope that some day she will come to her senses.

I have written a personal letter to inform her that she was receiving The Skeptical Review at
the request of another subscriber who will be notified that she thinks it is TRASH, so if she
should ever attempt to win her friend to Jesus through testimony of her "personal
experiences," he just might tell her what he thinks about her personal experiences. I further
assured her that her name would be removed from our mailing list, because we can see no
need to send TSR to someone who is just going to throw it "straight in the trash." Also, we
can see nothing to gain from casting our pearls before swine.

Thank you so much for existing!

I defected from the Jehovah's Witness cult as a teenager. Most of my family, going back four
generations (which is about as far back as the JW's go), belong to that group. I am 27 now and
find that there still are ridiculous beliefs left over from them that I have to remove one by one
from my brain. While there are many ideas regarding religion that are perhaps unknowable,
whether or not the Bible contradicts itself, is not one of them. The following quote, from a J.
W. tract, stimulated me to begin a search for the best quality and most valid contradictions:

While some may claim the Bible contradicts itself, has anyone ever shown you an actual
example? We have never seen one that could withstand scrutiny. True, there may appear to be
discrepancies in certain bible accounts. But the problem usually is lack of knowledge
regarding details and circumstances of the times (From the tract "Why You Can Trust the
Bible").
I am looking for some more quotes like the one above showing the rigid fundamentalist
position. For over a year now, I have been compiling contradictions that I hope to put into a
small book. I expect your publications and videos will be of great benefit to me.

Volume 1990 - 2002 Issue


Page 436 of 2049
Skeptical Review Edited by Farrell Till
(Darrel Henschell, 3633 East Huntsville Road, Fayetteville, AR 72701.)

EDITOR'S NOTE: We are always gratified to receive letters like this one, and we receive
them quite often. To know that so many are finding their way out of the absurdities of Bible
fundamentalism restores our faith in humanity. It is encouraging to see that at least some who
are indoctrinated in Bible fundamentalism as children refuse to let their adult lives be ruled by
emotionalism.

I just wanted to say that The Skeptical Review is a brilliant piece of work. You include
arguments from both sides which makes your journal stimulating to read. Your exchange with
Lindell Mitchell on the Amalekite massacre was very interesting indeed! After you had made
a strong and very compelling case for your position, I almost couldn't believe that an
intelligent and educated man like Mr. Mitchell went on to defend a total act of barbarism. I
gave a copy of your debate to a Lutheran minister I know. After reading your article and Mr.
Mitchell's, he said it was "embarrassing."

(Jason Munroe, 70 Nottingham Road, Sherwood Park, Alberta, Canada, T8A 5L5.)

A Reader's Letter to Mitchell


Dear Mr. Mitchell:

Having read your exchange with Mr. Farrell Till, I felt I should write and share with you my
thoughts on your discussion.

I grew up in the so-called "Church of Christ" (sat under Leroy Brownlow at the Polytechnic C
of C in Fort Worth and for several years under Roy Deaver at the Brown Trail C of C in
Hurst). I proselytized my (future) wife, while still in high school (she was a hell-bound
Baptist), and we were married in the Glen Garden C of C in Fort Worth by Andrew Connelly,
a missionary to Africa for many years and a close friend. I attended Fort Worth Christian
College for two years (editor of the school paper), worked for Star Bible Publications for
three years (an evangelistic publishing firm supported by C of Cs), then went on to attend
Abilene Christian University. I majored in Bible, with the goal of becoming a preacher in "the
Lord's Church." I was a Sunday School teacher in the Highland Church of Christ in Abilene
(producers of Herald of Truth television program), and my wife was a secretary in the
Highland church for Stanley Shipp, a missionary to South America supported by the Highland
Church.

Only a few credit hours away from graduation and after many discoveries that what I had
been taught as "the truth" were not in the Bible at all--that, indeed, we in the C of C "spoke
where the Bible did not speak and were not silent where the Bible was silent" in many, many

Volume 1990 - 2002 Issue


Page 437 of 2049
Skeptical Review Edited by Farrell Till
areas--I decided that I could not, with a clear conscience, continue in this pursuit, so I dropped
out and moved back to Fort Worth in pursuit of an honest way to support my family.

Since then, I have met with Christians in my home for many years for Bible studies, attended
many rallies and "gospel meetings" and have done everything a good "member of the church"
should do. When it finally dawned on me that all of God's sheep were not all in one corral (the
C of C nondenomination), I broke free and began to have fellowship with many other
believers outside that small, shrinking sect and have never looked back.

I know how you are probably thinking of me at this point. I've read all the books and tracts
that you've read and memorized all the pat scriptures that you've probably memorized. So I
don't expect you to think very highly of me. I would not be surprised if you wrote me a
scathing reprimand, as you did Mr. Till as reprinted in the latest issue of The Skeptical
Review. I assure you [that] it would only confirm your lack of love for "erring brethren" such
as me, even as it showed your lack of true Christian love and concern for Mr. Till. It was
obvious to me and was surely evident to all the "unbelievers" who read Mr. Till's publication.
I imagine many of your brethren who read Mr. Till's paper were very embarrassed by your
letter, not to mention your miserably feeble attempt to answer his proposition on the slaughter
of the Amalekites.

And the fact that you do not wish to continue any further discussion is proof to all of your
total defeat at his able hands. This is pure "chicken-livered," as we say here in Texas. I fear
you'll lose some rewards in heaven for such a tactic, Mr. Mitchell. I'm quite sure the Apostle
Paul would never give up in an attempt to convert such a leading critic of the Holy Scriptures.
In fact, I'm almost positive he would have considered it a special privilege to continue to have
such a vast audience of skeptics and unbelievers as you have had in the pages of TSR. Perhaps
you have better things to do down at the church study each day, like warming over old
sermons for folks who probably long ago tuned you out (you can just generate so much
enthusiasm from the audience with subjects like "Why We Do NOT Use Instruments of
Music in the Church" or "The Five Steps in the Plan of Salvation").

Anyone who really believed with all his heart that he held the Truth that could set men free
would not "turn tail and run" at the first naughty name that was applied to him by a poor, lost
atheist. But you can rest in the knowledge that you are not the first "defender of the faith" to
yell "uncle" in their confrontation with Mr. Till. He is racking up quite an impressive array of
poor country preachers who are out to make a name for themselves by trying to defend the
Bible from his onslaught. And I, for one, am learning how weak our position really is. Is there
no one in the land willing to take a stand against this Goliath of skepticism--not just until they
have "made a name" for themselves among their brethren. Not just so they can add their
"written debate" to their resumes so they can boast of how many giants they have felled when
their flock has tired of them and they are ready to "move up" in the C of C hierarchy. But men
who can continue to fight, no matter how many names they are called, no matter how much
the other guy cheats by not keeping his end of the agreement. I can just hear Paul saying, "We
have completed the agreed number of exchanges, so I'm going to quit now!"

I know, in your pride, Mr. Mitchell, you feel that you "won" the debate. Let me assure you,
you only served to solidify Mr. Till and his "followers" even deeper in their unbelief. And

Volume 1990 - 2002 Issue


Page 438 of 2049
Skeptical Review Edited by Farrell Till
those, who, like myself, have some lingering doubts as to the inerrancy of the Bible are given
further reason to doubt by your weak, inept response. For one, I hope that you do not continue
to publicly embarrass the cause of Christ by continuing. But I pray that someone might step
forward who can answer Mr. Till's arguments satisfactorily--for the sake of other believers
whose faith has been weakened by your and other fame-seekers' impotent attempts at
Christian apologetics.

(Patrick Phillips, 6216 Melinda Drive, Fort Worth, TX 76148.)

Volume 1990 - 2002 Issue


Page 439 of 2049
Skeptical Review Edited by Farrell Till

The Skeptical Review


Volume Five, Number Four
October/November/December 1994
Farrell Till, editor

• God and O.J. Simpson


Until Bible-believers develop the intellectual maturity to assess actions on the basis of
what is done in an act rather than who does it, they will remain intellectually crippled
hero-worshippers crying in the streets of common sense, "Let God go! Let God go!"

• Jairus's Daughter: Was She Dead or Wasn't She?


Inconsistencies in the gospel stories of Jairus' daughter suggest that the gospels are the
product of fallible humans, not an infallible God.

• The Blind, Staggering, Falling-Down-Drunk Luck Debate


"Buster" Dobbs proves that he is a sore loser.

• McBull to the Rescue

• Dobbs Raves On
Farrell Till reviews his debate with Jerry Moffitt.

• Chew on This...Again!
Leviticus 11:6 mistakenly classifies the rabbit as a ruminant or cud-chewer, a problem
for inerrantists, who claim that the Bible is scientifically accurate in every detail. This
article shows why the inerrantist explanation of Leviticus 11:6 is difficult to swallow.

• The Evolution and Devolution of the Bible


Just as modern scientists are certain that humankind evolved from lower life forms so
too are critical scholars certain that the Bible is a product of an evolutionary process of
its own. Much can be learned about Christianity by looking at the "extinct life forms"-
- namely, the apocryphal books-- that this evolutionary process left along the way.

Volume 1990 - 2002 Issue


Page 440 of 2049
Skeptical Review Edited by Farrell Till
• Zoroastrianism: The Forgotten Source
Zoroastrianism is almost unheard of today, yet this religion played a major role in
shaping Christian beliefs.

• From the Mailbag

God and O. J. Simpson


The written debate with Lindell Mitchell has generated an intense interest in the Amalekite
massacre. In this issue, "McBull to the Rescue" (pp. 5-7) continues the discussion. Like
Mitchell, McBull (an AOL user) has tried to justify the Amalekite massacre by arguing that
God is the "giver of life" and so he has the right to take life.

This is a familiar fundamentalist solution to the various Yahwistic atrocities recorded in the
Old Testament. At a debate on the existence of God in Portland, Texas, TSR editor Farrell
Till argued that the character of God depicted in the Bible is as self-contradictory as concepts
of square circles or four-sided triangles. Till used Yahweh's command to massacre Amalekite
women, children, and babies (1 Sam. 15:2-3 ) to argue that acts attributed to Yahweh in the
Bible cannot be logically reconciled with the claim that he is infinitely good, kind, and
merciful. His opponent, Jerry Moffitt, responded in the same way that Mitchell and now
McBull did: God who gives life has the right to take it. As frosting on the cake, Moffitt
assured the audience that the Amalekite children went to heaven rather than growing up to be
wicked like their parents. The audience complacently smiled at the irrefutable response that
their hero had given.

Such reasoning as this is incomprehensible to rational thinkers, but it is nevertheless true that,
no matter how heinous the Yahwistic atrocity, Bible fundamentalists refuse to charge their
god with immoral conduct. God is God, so no matter what he does, it must be right.

Last summer, the nation saw indications that such irrationality may be deeply ingrained in our
society. After the bodies of Nicole Simpson and a male companion were discovered brutally
murdered, the police investigation quickly uncovered evidence that implicated her ex-
husband, the famous hall-of-fame football player O. J. Simpson. Even though the evidence
seemed compelling, Simpson was allowed to remain free long after an ordinary, unknown
citizen would have been arrested. When finally the decision to charge Simpson was made, he
had disappeared, and soon after, the entire nation watched a televised pursuit of his car on
California freeways. During the pursuit, Simpson was cheered on by spectators who had
thronged to the freeways to watch. At Simpson's home, where he finally surrendered, people
stood outside shouting, "Let O. J. go! Let O. J. go!" In Buffalo, New York, where Simpson
had played professional football, people put signs in their yards that said, "Let the juice
loose!"

The purpose of this article is not to judge the guilt or innocence of O. J. Simpson--that is a
matter that will and should be done in court--but to make a point about the apparent inability

Volume 1990 - 2002 Issue


Page 441 of 2049
Skeptical Review Edited by Farrell Till
of people to think rationally. In the days following his arrest, various news programs showed
signs affixed to the gate of Simpson's estate that said such things as, "Free the juice," and "We
still love you, O. J." During a televised interview of curiosity seekers standing outside the
gate, a man said, "I don't care what he did; he'll always be my hero."

Such actions as these are not very complimentary to human intelligence. If the evidence
gathered at the murder scene had pointed to the probable guilt of a person completely
unknown to the general public, how many people would have stood in the streets shouting,
"Let John Doe go"? Would that man on the street have said, "I don't care what he did, I'll
always stand by him"? To the contrary, given the alleged brutality of the murders, there would
have probably been public outcries for swift and harsh punishment in order to send a message
to the lawless elements of our society that such criminal behavior will not be tolerated.

What many people have lost sight of in the charges against O. J. Simpson is exactly what
Bible fundamentalists have lost sight of concerning the many Yahwistic atrocities recorded in
the Bible. The right or wrong of an act is determined by what is done and not by who does it.
If someone takes a child dying of starvation and feeds and nurses it back to health, that is an
act of goodness whether it be done by Saddam Hussein or Mother Theresa. If someone
viciously kills a person out of anger or desire for vengeance, that is an act of evil whether it be
done by Saddam Hussein, O. J. Simpson--or God. To argue otherwise is to empty all such
words as good, bad, merciful, cruel, kind, and evil of meaningful definition.

Blue, for example, is a concept that has meaning only if it is universally and uniformly
applied to all objects that have color. To say that the star-studded field on an American flag is
blue in Colorado but not blue on an identical flag displayed in Ohio would be patently absurd.
To say that the deliberate murder of two people is wrong if John Doe does it but not so wrong
if a popular sports hero does it is just as absurd.

Likewise, it is equally absurd to argue that the killing of women and children was wrong
when American soldiers did it in My Lai, South Vietnam, but not wrong when Israelite
soldiers did it in Amalekite towns and villages. To argue that the latter can be justified on the
grounds that God ordered the Israelites to kill the Amalekite women and children reduces
Bible fundamentalists to the level of hero worshipers shouting in the streets, "Let the juice
loose! Let the juice loose!" The only essential difference is that the fundamentalists are
shouting, "Free God of blame! Free God of blame!"

Those who so rationalize seem unable to comprehend the truth that Dave Matson stated in
"The Law of the Jungle" (TSR, Summer 1994, p. 8): "The evil of an act lies in its
consequences, its hurt, not in who does it." If someone deliberately and viciously kills you, is
it going to be of any special comfort to you if your killer is a popular movie star or famous
athlete rather than a common thug? You will still be just as dead as if the common thug had
killed you. If infliction of pain should be involved in such a murder, the agony that you suffer
will still be just as real as if a common thug had inflicted it on you.

In the Portland debate, Till emphasized that the words good, kind, and merciful mean what
they mean, so goodness, kindness, and mercy must always be determined by what was done
and not by who did it. The point made about as much impact on the predominantly

Volume 1990 - 2002 Issue


Page 442 of 2049
Skeptical Review Edited by Farrell Till
fundamentalist audience as water on a duck's back. However, until Bible-believers develop
the intellectual maturity to assess actions on the basis of what is done in an act rather than
who does it, they will remain intellectually crippled hero-worshipers crying in the streets of
common sense, "Let God go! Let God go!"

Jairus's Daughter: Was She Dead or Wasn't


She?
Farrell Till
All three synoptic gospels relate the story of Jairus's daughter, whom Jesus allegedly raised
from the death. The accounts by Mark and Luke are in substantial agreement, but Matthew's
version differs in one significant point that cannot be reconciled with the other two without
resorting to typically ridiculous fundamentalist "explanations." Although the passages are
quite long, they all need to be read to see the problem, so we will look first at the two
accounts that are in basic agreement. An ellipsis will be inserted in each version to signal
omission of the healing of the woman with the issue of blood, a miracle that allegedly
happened while Jesus was on his way to Jairus's house. All three versions report this miracle,
but it is not relevant to the matter of inconsistency in the story of Jairus's daughter; hence, it
will be omitted in order to save space and focus attention on inconsistencies about the raising
of Jairus's daughter:

MARK'S VERSION: Now when Jesus had crossed over again by boat to the other side, a
great multitude gathered to Him; and He was by the sea. And behold, one of the rulers of the
synagogue came, Jairus by name. And when he saw Him, he fell at His feet and begged him
earnestly, saying, "My little daughter lies at the point of death. Come and lay Your hands on
her, that she may be healed, and she will live." So Jesus went with him, and a great multitude
followed Him and thronged Him.... While He was still speaking, some came from the ruler of
the synagogue's house who said, "Your daughter is dead. Why trouble the Teacher any
further?"
As soon as Jesus heard the word that was spoken, He said to the ruler of the synagogue, "Do
not be afraid; only believe." And He permitted no one to follow Him except Peter, James, and
John the brother of James. Then He came to the house of the rule of the synagogue, and saw a
tumult and those who wept and wailed loudly. When He came in, He said to them, "Why
make this commotion and weep? The child is not dead, but sleeping."And they ridiculed Him.
But when He had put them all outside, He took the father and the mother of the child, and
those who were with Him, and entered where the child was lying. Then He took the child by
the hand, and said to her, "Talitha, cumi," which is translated, "Little girl, I say to you, arise."
Immediately the girl arose and walked, for she was twelve years of age. And they were
overcome with great amazement" (5:21-24, 35-42, NKJV).
LUKE'S VERSION: So it was, when Jesus returned, that the multitude welcomed Him, for
they were all waiting for Him. And behold, there came a man named Jairus, and he was a

Volume 1990 - 2002 Issue


Page 443 of 2049
Skeptical Review Edited by Farrell Till
ruler of the synagogue. And he fell down at Jesus' feet and begged Him to come to his house,
for he had an only daughter about twelve years of age, and she was dying....While He was still
speaking, someone came from the ruler of the synagogue's house, saying to him. "Your
daughter is dead. Do not trouble the Teacher."
But when Jesus heard it, He answered him, saying, "Do not be afraid; only believe, and she
will be made well." When He came into the house, He permitted no one to go in except Peter,
James, and John, and the father and mother of the girl. Now all wept and mourned for her; but
He said, "Do not weep; she is not dead, but sleeping." And they ridiculed Him, knowing that
she was dead.
But He put them all outside, took her by the hand and called, saying, "Little girl, arise." Then
her spirit returned, and she arose immediately. And He commanded that she be given
something to eat. And her parents were astonished, but He charged them to tell no one what
had happened (8:40-42, 49-56, NKJV).

At this point, the significant thing to notice is that both Mark and Luke reported that Jairus's
daughter was yet alive when her father came to Jesus to ask for help. This is not the case in
Matthew's version:

While he spoke these things to them, behold, a ruler came and worshiped Him, saying, "My
daughter has just died, but come and lay Your hand on her and she will live." So Jesus arose
and followed him, and so did His disciples....
When Jesus came into the ruler's house, and saw the flute players and the noisy crowd
wailing, He said to them, "Make room, for the girl is not dead, but sleeping." And they
ridiculed Him. But when the crowd was put outside, He went in and took her by the hand, and
the girl arose. And the report of this went out into all that land (Matthew 9:18-19, 23-26).

The major problem in the story, then, is quite simple: was the girl dead when her father came
to Jesus for help or wasn't she? Mark said Jairus told Jesus that his daughter was lying at the
point of death, and Luke simply said that "she was dying." Matthew, however, had the girl's
father say, "My daughter has just died." It couldn't possibly be true that the girl was both alive
and dead at the time Jairus came to Jesus. She was either dead at the time or she wasn't. If she
was yet alive, then Mark and Luke were right and Matthew was wrong; if she was dead at the
time, then Matthew was right and Mark and Luke were wrong.

Fundamentalists, of course, will not allow a simple thing like glaring contradiction to deter
them from believing that the Bible is the inerrant, inspired word of God, so they have an
explanation to offer. Matthew, you see, was writing the story from the point of view of the
girl's death being so certain (if Jesus didn't intervene) that he had Jairus say, "My daughter has
just died." Inerrantists will usually suggest an analogy like a basketball team whose star center
suffers a serious injury just before the big game. "We have lost the game," someone might say
under the circumstances, not at all meaning that the game has already been played and lost but
that defeat is certain without the star player. This, they will argue, is all that Matthew meant.
Jairus's daughter was so ill that death was certain without divine intervention, so it was
appropriate for him to have Jairus speak figuratively and say, "My daughter has just died."
Mark and Luke, on the other hand, chose to write more literally; hence, they stated that the
girl was in a state of serious illness but still alive when her father came to Jesus. "So, you
see," inerrantists will gleefully (and arbitrarily) declare, "there really is no contradiction here."

Volume 1990 - 2002 Issue


Page 444 of 2049
Skeptical Review Edited by Farrell Till
Although a how-it-could-have-been explanation like this may satisfy Bible inerrantists
desperately looking for a solution to an embarrassing problem, it will not satisfy objective
minds who can't help noticing its failure to resolve all problems involved in reconciling the
three accounts of this story. Inconsistency in what Jairus said to Jesus is just one of several
difficulties in the story. Let's notice, for example, that Mark and Luke both have five elements
in their versions of the story: (1) Jairus came to Jesus to ask him to help his daughter, (2)
Jesus and his disciples then went with Jairus to his home, (3) on the way there, they were met
by some [Mark] or someone [Luke] coming from Jairus's house who announced that the
daughter had died, (4) Jesus nevertheless continued on to Jairus's house, and (5) at the house,
he raised the girl from the dead.

All of these elements are also in Matthew's version except number three. Matthew said
nothing about anyone from Jairus's house meeting Jesus to announce that the girl had died. On
this point, inerrantists will of course argue that omission does not constitute cotradiction, but I
am not citing Matthew's omission of this detail as proof of contradiction or even inconsistency
but rather to challenge the likeliness of the figurative interpretation that inerrantists apply to
what Matthew's Jairus said to Jesus. They insist that the statement could have had the
figurative meaning mentioned above, i.e., the death of the girl was certain and imminent
enough to warrant Jairus's saying, "My daughter is dead." However, Matthew's omission of
the message brought by some[one] from Jairus's house gives sufficient reason to reject their
explanation. Mark and Luke, who began their versions of the story with the premise that the
girl was still alive, stated that Jesus and Jairus were met by some[one] who announced that
the girl had died. Matthew, on the other hand, who began the story with Jairus saying that his
daughter had just died, said nothing about anyone coming to announce that the girl was now
dead. Why? The answer is simple: Matthew was telling a story about Jesus going to raise a
girl who was already dead, so it would have made no sense at all to have some[one] meet him
to announce that the girl was dead. He would have already known that, because it was exactly
what Jairus had said to him: "My daughter has just died." Hence, Matthew's omission of this
detail is sufficient reason to reject the premise that he wanted readers to understand that Jairus
was speaking figuratively in this version of the story.

Another serious fallacy in this figurative "explanation" is the obvious fact that Matthew and
Mark both purported to tell what Jairus said to Jesus, as opposed to Luke's indirect method,
which simply stated that the girl "was dying," without attempting to relate what Jairus had
said to express the facts of her condition. Had all three writers used Luke's method, then
perhaps there would be some merit to the inerrantist attempt to resolve the problem in the
manner stated above; however, the fact that two of the writers (Mark and Matthew) wrote
their accounts as if to convey what Jairus said to Jesus makes the figurative "explanation"
entirely unsatisfactory. What a person says is what he says, and two accounts of what
someone said can't both be inerrant accounts unless both use the exact words to convey what
was said.

Let's imagine that someone named Jones should say, "The sun set yesterday at six-thirty P.
M." That being the case, which of the following statements would be inerrant accounts of
what Jones said?

1. Yesterday, the sun set at six-thirty P. M.

Volume 1990 - 2002 Issue


Page 445 of 2049
Skeptical Review Edited by Farrell Till
2. The sun went down yesterday at six-thirty P. M.
3. Yesterday, the sun went down at six-thirty P. M.
4. The sun set yesterday at eighteen hundred thirty hours.
5. Yesterday, the sun set at eighteen hundred thirty hours.
6. At six-thirty P. M. yesterday, the sun set.
7. The sun set yesterday at six-thirty P. M.

Although one could correctly argue that all seven statements accurately convey the meaning
of what Jones said, number seven is obviously the only statement that could be considered an
inerrant account of what Jones said. This is true because of the fact previously stated: what a
person says is what he says and not something else. If Jones said, "The sun set yesterday at
six-thirty P.M.," then he said, "The sun set yesterday at six-thirty P. M." He didn't say,
"Yesterday, the sun set at six-thirty P. M." or anything else that may accurately state the
meaning of what Jones said.

Fundamentalists will protest that such a position as this demands more than is necessary in
order to have biblical inerrancy. They insist that inspiration could have been verbal without
being dictational, so to "explain" variations in the way that different writers recorded the same
stories, inerrantists will talk about personal choices that the writers exercised. To explain, for
example, variations in the gospel narratives concerning the number of people who went to the
tomb on resurrection morning, inerrancy defenders will say that John chose to tell only about
Mary Magdalene, whereas Matthew chose to give more details about who went to the tomb,
so he informed us that "the other Mary" was with Mary Magdalene. Mark wanted to give even
more information, so he mentioned Salome too, and Luke, the most detailed writer of them
all, added that Joanna and "other women" were also in the company. "However, there is no
contradiction," inerrantists will rhapsodize, "just variations in details that resulted from the
writers exercising personal judgments about which details to include and which to exclude."
They apply this reasoning to various parallel accounts in which details vary from writer to
writer.

What this argument fails to consider is that, no matter what vague lines inerrantists may try to
draw between "verbal" and "dictational" inspiration, the doctrine of verbal inspiration, which
is believed by practically all Bible fundamentalists, virtually eliminates all elements of
individual choices. "No prophecy ever came by the will of man," inerrantists delight in
reminding us, "but men spake from God, being moved by the Holy Spirit" (2 Peter 1:21 ).
What would the "will of man" be except personal choices, and that is precisely what the writer
of this statement said had not been involved in prophesying. Rather than speaking by their
own will, those men spoke "from God" as they were "moved by the Holy Spirit." So
individual and personal choices were not involved.

This conclusion is in complete agreement with what George DeHoff said in his book Alleged
Bible Contradictions Explained as he discussed the full implications of the doctrine of verbal
inspiration:

The Holy Spirit taught the apostles what to say--what to write. We have, therefore, the Word
of God. If God had wanted another "i" dotted or another "t" crossed, He would have had it

Volume 1990 - 2002 Issue


Page 446 of 2049
Skeptical Review Edited by Farrell Till
done. The writers did not use one word unless God wanted that word used. They put in every
word which God wanted them to put into the Bible (p. 23).

If DeHoff is right, then how would variations of any kind be possible in verbally inspired
accounts of the same incidents? If Mark did not use "one word unless God wanted that word
used" and if the same was true of Matthew and Luke, why would there be any variations at all
in the telling of the story of Jairus's daughter? Are inerrantists arguing that the Holy Spirit
wanted writer A to use certain words when recording an event but wanted writer B to use
entirely different words when recording the same event? If so, why? Was the first account that
the Holy Spirit inspired somehow imperfect and thus in need of improvement when the
second account was inspired? This is something that inerrantists absolutely must explain if
they expect anyone but hopelessly indoctrinated fundamentalists to take seriously such
explanations as the one they offer in the matter of Jairus's daughter.

In particular, DeHoff's statement makes variations impossible in what characters said in


verbally inspired parallel accounts of the same incidents. If Jairus said, "My little daughter
lies at the point of death," as Mark indicated, then he couldn't even have said, "My daughter is
about to die," because the two statements are not identical, and he certainly could not have
said, "My daughter has just died," as Matthew indicated in his account. Jairus said what he
said (if the incident actually happened), so no rational person can believe that Mark's and
Matthew's accounts are both verbally inspired, inerrant accounts of what Jairus said. To argue
otherwise is to put one in the ridiculous position of claiming that the omniscient, omnipotent
Holy Spirit verbally inspired Mark to say that Jairus said that his daughter was still alive and
then later verbally inspired Matthew to say that Jairus said that his daughter was dead. So the
problem would remain unresolved even if we could determine beyond question that Matthew
meant for his readers to understand that Jairus was speaking figuratively, because if he did
indeed speak figuratively to Jesus (Matthew's version), then he couldn't have spoken literally
(Mark's version). By necessity, one writer had to have recorded his words inaccurately. Only
a Bible fundamentalist can see merit in such irrational, straw-grasping "solutions" as the one
offered to "explain" the problem in this story.

When this point is applied to other uses of dialogue in the different accounts of the story of
Jairus's daughter, we see the same problem. Mark, for example, had the messenger(s) from
Jairus's house saying, "Your daughter is dead. Why trouble the Teacher any further?" whereas
Luke had them saying, "Your daughter is dead. Do not trouble the Teacher." Although
inerrantists will dismiss this point as a quibble, I must insist that it is a legitimate objection. If
we remember that what a person says is what he says, the legitimacy of the objection is
apparent. If DeHoff is right and Bible writers did not use "one word unless God wanted that
word used," then we must wonder why the Holy Spirit would not have wanted both inspired
writers to use the exact words that the messenger(s) spoke when they announced that Jairus's
daughter was dead. As a matter of fact, Mark and Luke both used the exact same words in
recording what the messenger(s) said in their first sentence: ""Your daughter is dead." So if
the Holy Spirit could inspire this kind of exactness in one sentence, why not in the next? It is
a problem that inerrantists must confront.

A simple analogy will demonstrate the obviousness of this point. Let's suppose that two
reporters at the scene of a fire witness someone in flames running from the house, shouting,

Volume 1990 - 2002 Issue


Page 447 of 2049
Skeptical Review Edited by Farrell Till
"Help me! Help me! Somebody please help me!" Since the reporters are both professionals,
we can reasonably assume that they want to report accurately what happened. However, in
writing her story, one reporter said that the person on fire shouted, "Please help me!
Somebody please help me!" whereas the other one wrote that the person screamed, "Help me
please! Please help me, somebody!" Although both reporters wrote accurate accounts of what
happened, neither one wrote an inerrant account, because neither one reported exactly what
the person shouted. We would understand that the failure to report the exact words of the
person on fire was due to human fallibility. No one, however, would believe that the failure to
report exactly what was said was an intentional matter of choice, for what professional
journalist would purposely report what he knew wasn't accurate? In other words, if the
reporters had been able to recall exactly what the person had screamed, they undoubtedly
would have both reported his exact words.

Now let's apply this to the matter of what Jairus said to Jesus. If Jairus really did come to
Jesus and ask for help, he had to have said something. An omniscient, omnipotent deity would
have known exactly what Jairus had said, so if this deity later verbally inspired three people to
write accounts of what Jairus had said, all three accounts would have attributed the exact
same words to Jairus. If not, why not? So the fact that all three accounts did not use identical
words in reporting what Jairus said to Jesus, what the messenger(s) said to Jairus, what Jesus
said to the mourners at Jairus's house, and what Jesus said to the girl when ordering her to
arise, we can know that the three accounts can't all be verbally inspired.

We can probably expect inerrantists to resort to their old standby: "If three people witness an
accident, the police reports that they file will contain variations...." Yes, this is true, but
inerrantists who use this argument conveniently forget one important thing: the three people
who witness the accident are not verbally inspired by an omniscient, omnipotent deity while
they are writing their police reports. If they were, we would have reason to expect all three
accounts to be in perfect agreement.

Variations in the three versions of the story of Jairus's daughter is just the tip of a biblical-
inconsistency iceberg. The fact that parallel accounts in the Bible so often vary, sometimes
significantly, in reporting what characters allegedly said is more than sufficient to reject the
claim that the Bible was verbally inspired by an omniscient, omnipotent God.

The Blind, Staggering, Falling-Down-Drunk


Luck Debate
by Farrel Till
On May 21-24, I debated Jerry Moffitt in conjunction with the Gulf Coast Lectures at the
Portland, Texas, Church of Christ. At Moffitt's request, the issue was changed from biblical
inerrancy to the resurrection and finally to the existence of God. Perhaps the most accurate

Volume 1990 - 2002 Issue


Page 448 of 2049
Skeptical Review Edited by Farrell Till
way to describe this debate would be to call it the blind, staggering, falling-down-drunk luck
debate, because this was an expression that Moffitt used repeatedly to bolster the design
argument that he focused on throughout the debate.

Subscribers to The Skeptical Review are already familiar with Moffitt's views through several
of his articles that we have published. His latest contribution was a response to my article
"Letter From a Dead Man" (Autumn 1993, pp. 4-6). In his attempt to explain how the prophet
Elijah could have written a letter after he was dead, Moffitt began by declaring that God is
"self-concealing," then went on to say, "He refuses to reveal Himself in the Scriptures in a
way that is *psychologically compelling*" (p. 4). Oddly enough, Moffitt began the Portland
debate the same way. In his first speech, he declared his belief that God will not reveal
himself to man in any "compelling way," because "good and honest hearts" will always be
willing to accept the evidence he has presented. By way of evidence, Moffitt then proceeded
to extol the wonders of the human body, which he of course presented as intelligent design
that had to prove the existence of God. His tactic was to discuss the orderly function of a body
organ and then to ask the audience if anyone could believe that all of this happened by just
"blind, staggering, falling-down-drunk luck."

My response consisted of three points: (1) the Bible is filled with examples of where God
appeared personally to men like Adam, Noah, Abraham, Isaac, Jacob, Moses, etc., so if God
does not reveal himself today in "compelling" ways that makes him a respecter of persons in
direct contradiction to biblical statements like Acts 10:34, (2) the postulation of a God to
explain the mystery of existence explains nothing and only presents a bigger mystery, and (3)
nothing in Moffitt's design argument would require the creator to be Yahweh the god of the
Hebrews.

This third point was important, because, Moffitt and I had signed an agreement that required
us to define "God" in both of our propositions as "the Deity who is described in the Bible in
all His characteristics or attributes." I asked Moffitt to explain how anything that he had said
would require the deity behind intelligent design to be Yahweh of the Hebrews. In what was
undoubtedly Moffitt's biggest blunder in the debate, he tried to prove that Yahweh was the
god of intelligent design by simply asserting that the odds against just 11 of the hundreds of
biblical prophecies being fulfilled were so astronomically high as to be impossible without
intelligent direction. Therefore, he asserted, the God of the Bible had to be the creator.

My response to this was to point out that Moffitt had only asserted the fulfillment of
prophecies but had not specifically identified even one prophecy fulfillment. In answer to my
demand that he identify *and* prove these prophecy fulfillments, he responded that we didn't
have time to discuss them in this debate. I refused to let the issue drop and pointed out that
such an assertion as this was a resort to question begging. This matter came to a head in the
final session when I challenged Moffitt to debate the prophecy issue in a separate debate. He
stood up and accepted the challenge, so an on-the-spot decision was made to have a written
debate on the subject. Whether this was a sincere gesture on Moffitt's part or just a play to the
gallery remains to be seen, because he and I already have a written debate in progress to
which he has not contributed a manuscript in almost three years. My hope is that he will keep
the commitment, because prophecy-fulfillment arguments are very easy to refute. In addition
to the design argument, Moffitt used probability arguments and peppered his speeches with

Volume 1990 - 2002 Issue


Page 449 of 2049
Skeptical Review Edited by Farrell Till
quotations from "scientists" who claimed to believe in creation. His probability arguments
were the typically silly, after-the-fact stuff that we hear from creationists. Most readers have
probably heard them before: the chances of a tornado going through a junkyard and leaving
behind a fully assembled Boeing 707 are greater than the probability that life could develop
from nonlife and such like. To present one "argument," he showed a chart that had a trillion
written on it twenty-seven times and said that if one would multiply a trillion by itself 27
times, he would have the odds against hemoglobin developing by chance. In my response to
this and other probability arguments, I pointed out that no evolutionist believes that any parts
of the human body or any other organism happened by chance or wave-of-the-hand means
that suddenly brought it into existence. I further pointed out that Moffitt had not explained
any of the factors or criteria that he had begun with to arrive at his probability figures. In the
flipping of a coin, for example, one knows exactly what factors he is dealing with to
determine the probability of getting heads twice in succession, but Moffitt never explained the
factors he was working with. He didn't, of course, because he doesn't understand how to
figure probability and even acknowledged in the debate that he was using figures that had
been calculated for him by someone else. I specifically challenged him to explain what factors
he had used to arrive at his astronomical odds against the formation of hemoglobin, but he
never addressed the issue.

In this part of the debate, I turned Moffitt's probability argument back on him and
demonstrated that his own methods will prove that it is impossible for him to exist. I thought I
saw some looks of displeasure in the audience, but I used the fact that a male ejaculates about
250 million individual sperm cells in an act of sexual intercourse. This means that the female
ovum has the chance of being fertilized by 250 million different sperm cells; hence, the
probability of any person being the individual he/she is was about 250 million to one. I
pointed out that this takes into consideration only the factors present during conception and
not the many factors that could have kept any given human being's parents from having met
so that the act of conception could take place. When we consider that these factors would
have been involved not just in the conception of Moffitt but also in the conception of each of
his parents and their parents and their parents and their parents, etc., etc., etc., Moffitt's own
line of argument would prove that it is impossible for him or any other specific individual to
exist.

Not having my personal library with me, I was unable to respond as I would have liked to the
numerous quotations that Moffitt read out of context to try to prove that most scientists
believe in the existence of a personal creator. Albert Einstein, Roger Penrose, Carl Sagan, and
Stephen Hawking were among those that Moffitt tried to present as creationists. I asked him
under cross-examination if he had the gall to say that these scientists believed that the
universe was created by his god Yahweh, at which time he hedged a bit and said that he was
claiming only that they believed in a creator. He evaded my challenge to let the audience
know how many of the scientists he had quoted were believers in evolution.

In assessing the debate, I personally believe that my first negative speech outlined a problem
that Moffitt was never able to resolve. I grouped the design, order, and cause-effect arguments
together and showed that rather than solving the mystery of existence, they merely created a
greater mystery. I argued that if apparent design, order, and cause in nature can be explained
only by postulating a designer, orderer, and causer, then it would necessarily be true that the

Volume 1990 - 2002 Issue


Page 450 of 2049
Skeptical Review Edited by Farrell Till
greater the design, order, and cause, the greater the need to explain them. Since a being who
could design and cause a complex universe would necessarily be more complex than the
universe that he designed and caused, then this being himself would require explanation.
Thereafter, my response to Moffitt's endless comments about the marvels of the human body
was simply to admit that the human body is wonderfully adapted to its environment and then
to ask him to explain the existence of the deity who had created it. He could only manage a
question-begging response to this: time and space began with the Big Bang, according to
scientists like Einstein and Hawking, so God had to exist before time began.

Despite the silliness of his probability arguments and his constant resorts to question begging,
Moffitt was easily the best debating opponent I have faced. He speaks rapidly, so he was able
to overwhelm the audience with a constant volley of scripture quotations, probability
arguments, and out-of-context quotations. He knows exactly what his audience wants to hear,
so he is very good at playing to the gallery. Fundamentalists who watch the tapes of this
debate will find much to applaud. Atheists and skeptics will undoubtedly roll their eyes in
disbelief at many of the things they hear in Moffitt's speeches, especially his probability
arguments and frequent praise of Dr. Hugh Ross the creationist physicist whose book The
Creator and the Cosmos played second fiddle only to the Bible when Moffitt needed a proof
text.

The tapes have eight hours of viewing and can be obtained from The Skeptical Review on
two-week rental for $2 to cover the cost of mailing.

McBull to the Rescue


Farrell Till
Richard Fiori, a TSR subscriber in Fresno, California, has forwarded to me an "explanation"
of the Amalekite massacre that was sent to him by an America On Line (AOL) subscriber
who calls himself McBull. After reading McBull's attempt to justify the Amalekite atrocity,
I'm convinced that he needs to drop the "Mc" from his name. His response, entitled "Yes, a
Moral God!!" is reproduced below exactly as he wrote it, with [sic] inserted where readers
will think that I have surely miscopied.

Yes!! This is a good one! I'll explain it to you and then you can try another one. First though,
you say the killing of an Amalekite woman or child based solely on nationality is an atrocity...
Funny, the Amalekites would not of [sic] thought so!! As a matter of fact, the Amalekites
routinely killed there [sic] own children during sacrifice to idols. Anyway, here is the morality
behind why God decided to destroy the Amalekites:
1. The Amalekites were bent on destroying the Israelites as demonstrated on there [sic]
unprovoked attack upon Israel. The Amalekites were a band a [sic] guerrilla terrorists, they
made a living by raiding others and taking everything they owned, including there [sic] lives.
They killed for pleasure.

Volume 1990 - 2002 Issue


Page 451 of 2049
Skeptical Review Edited by Farrell Till
2. The Amalekites were given plenty to [sic] time to repent from their utter rejection of God,
and their sin against His people, but they refused. They continued to be involved in insest
[sic], molestation, homosexuality, bestiality, child sacrifice, murder, etc... "A moral atrocity
by any civilized standard of morality", wouldn't you agree, Ingersol [Fiori's AOL name].....
The Amalekites made their own choices, by their own free will. God simply held them
accountable for their actions.
3. The entire Amalekite population had to be destroyed due to the gravity of there [sic] sins,
and so that any remnants could not continue their acts of violence against the nation of Israel.
Otherwise, we would have had a situation similar to what now takes place with today's gangs
on our streets. You kill one of ours, we kill two of yours. And so on...
4. While murder is a direct violation of the sixth commandment, not all forms of killing
represents [sic] murder. To kill someone in self-defense or to execute someone for a capitol
[sic] offense is totally justifiable. The fact is, justice demands that war criminals such as Nazis
be put to death. As Walter Kaiser once put it, war is "Gods [sic] ultimate, but reluctant,
method of treating evil that resists every other patient and loving rebuke of God." God used
Israel to bring judgement upon the Amalekites.
4 [sic]. All children who die before the age of accountability are saved (see 2 Sam. 12.23) So,
the act by which God took the lives of children, is far from merciless.
And my final point:
Since God is the author of life, it is entirely His decision as when to give life, as well as when
to take it away. The fact that you and I have our lives, is a blessing. NOT A RIGHT...
Everyone will eventually be taken by God in death - its [sic] only a matter of when....
I look forward to your reply. We can debate this further or you can try another situation or
verse.

Readers who have followed my exchanges with Lindell Mitchell on this subject probably
noticed the striking similarity in Mitchell's and McBull's defenses of the Amalekite atrocity.
Since McBull is not a subscriber to The Skeptical Review, he has probably not seen Mitchell's
articles. The fact that two people working independently of each other would write such
similar articles merely shows that stock answers to Bible problems have been widely
circulated among Christian fundamentalists. Let's notice, one by one, the absurdity of
McBull's "canned" responses.

Their unprovoked attack on the Israelites demonstrates that the Amalekites were bent
on destroying Israel. This argument was soundly refuted in my last response to Mitchell
(Summer 1994). As I pointed out, a horde of about 3 million Israelites (if we are to believe the
census count in the book of Numbers) entered Amalekite territory during their trek to the
promised land (p. 4). The Israelite army alone numbered 600,000 (Ex. 12:37 ; Num. 1:45-46
). How, then, can any reasonable person call the Amalekite action against the Is-raelites an
"unprovoked attack"? I asked this question of Mitchell, but he chose to declare victory and
drop out of the discussion. Perhaps McBull will favor us with an answer.

Bibliolaters try to make a big deal out of the biblical claim that the Amalekites attacked Israel
from the rear (Dt. 25:17-19 ). The fact that the Bible says that the Amalekites attacked in this
manner doesn't prove that it actually happened this way, but for the sake of argument, let's
assume that it did. A possible explanation for this strategy could well be that the Amalekites
did not have a large enough army to confront 600,000 armed Israelite soldiers in conventional

Volume 1990 - 2002 Issue


Page 452 of 2049
Skeptical Review Edited by Farrell Till
battle. Guerrilla tactics are quite common in situations where the defenders of their national
territory have been invaded by numerically superior forces. To say the least, McBull and his
fundamentalist cohorts are stretching reality to call such a situation an "unprovoked attack."
Besides, McBull's point completely ignores the fact that this attack on Israel happened 450
years before Yahweh allegedly ordered Saul to utterly destroy the Amalekites of his
generation, so McBull must show us the justice of massacring an entire nation for something
that their distant ancestors did. Mitchell couldn't, and I doubt if McBull can.

McBull characterized the Amalekites [presumably the Amalekites of Saul's time] as "a band
of guerrilla terrorists," who (1) "made a living by raiding others," (2) "taking everything they
[others] owned, including there [sic] lives," and (3) "kill[ing] for pleasure." In this statement,
McBull has made some assertions that he has a responsibility to prove. I know, for example,
of only one biblical passage (1 Sam. 30:1-20 ) that refers to a raid that was made by the
Amalekites who were contemporary to Saul, and this was done right after David had raided
Amalekite villages and "saved neither man nor woman alive" (1 Sam. 27:8-11 ). In probable
retaliation, the Amalekites then raided David's camp and took the women and children
captive.

All of this happened during the conflict between David and Saul, when David and a band of
loyal followers had taken refuge in the country of the Philistines (1 Sam. 27:1-7 ). David and
his men survived by making guerrilla raids against the Geshurites, Girzites, and Amalekites.
If, then, Guerrilla terrorism was such a horrible sin and making a living by "raiding others and
taking everything they owned, including there [sic] lives" was sufficient reason for God to
order the massacre of the terrorists, then God should have had David killed. But he didn't. He
considered him a "man after Yahweh's own heart" (1 Sam. 13:14 ; Acts 13:14 ) and made him
king over Israel. We have to wonder, then, about the soundness of McBull's argument on this
point. An argument that leaves such inconsistencies in its wake isn't much of an argument.

As for McBull's claim that the Amalekites made their living by raiding others, taking
everything they owned, and killing for pleasure, I must insist that he provide us with proof of
these assertions. As we noted about Mitchell's arguments, asserting is easy; proving isn't quite
so easy. Where, then, is McBull's biblical or extrabiblical proof that the Amalekites of Saul's
time were the despicable, amoral terrorists that McBull claims they were? I predict that he
will have about as much success as Mitchell did in proving this assertion.

The Amalekites were given plenty of time to repent, but they refused and continued to
be involved in insest [sic], molestation, homosexuality, bestiality, child sacrifice, murder,
etc. All I need do here is challenge McBull to cite one-- just one--biblical passage that claims
the Amalekites of Saul's generation were involved in any of these immoralities. No such
passages exist. For example, the only biblical cases of incest that I can recall are the stories of
Lot's impregnation of his daughters (Gen. 19:30-38 ) and the man in the Corinthian
congregation who had taken his "father's wife" (1 Cor. 5:1 ). Incest is, of course, legislated
against in the Bible, but there is absolutely nothing recorded that even suggests that the
Amalekites were an incestuous people. The same is true of McBull's other allegations
regarding Amalekite practices of molestation, homosexuality, and bestiality. In all likelihood,
McBull is just an amateur apologist who is simply parroting something that he has heard
overzealous preachers proclaim from the pulpit, so I will give him a bit of friendly advice.

Volume 1990 - 2002 Issue


Page 453 of 2049
Skeptical Review Edited by Farrell Till
Before he uses pulpit information to defend the Bible, he should check it out. He will very
likely find it to be erroneous or at least overstated.

Prior to introducing his points, McBull said that the Amalekites would not have considered
the killing of children solely for reasons of nationality an atrocity, because the "Amalekites
routinely killed there [sic] own children during sacrifice to idols." Again, however, I must
demand his proof. Where does the Bible say that the Amalekites practiced human sacrifice? If
he doesn't have biblical proof, perhaps he knows of some extrabiblical proof. If so, we would
be interested in seeing it.

Personally, I would not be at all surprised if the Amalekites did indeed practice human
sacrifice, because it was a common custom in the area at that time. The Bible makes frequent
references to it, and archaeologists have discovered evidence of the practice. All I'm saying is
that I want to see McBull's proof of his assertion that the Amalekites "routinely" sacrificed
their children to idols and so this was just one of many moral reasons why God ordered Saul
to exterminate them. After all the asserting by McBull, Mitchell, et al, one stubborn fact still
remains: the only reason given for the orders to massacre the Amalekites was the attack on
Israel made by their ancestors 450 years earlier. "Thus saith Yahweh of hosts, I remember
that which Amalek did to Israel, how he laid wait for him in the way when he came up
from Egypt. Now go and smite Amalek and utterly destroy all that they have and spare them
not; but slay both man and woman, infant and suckling, ox and sheep, camel and ass" (1
Sam. 15:2-3 ).

Even if McBull could prove extrabiblically that the Amalekites did practice human sacrifice, I
would have to ask him, "So what?" The Israelites did too. In condemning the rebelliousness of
Israel, Psalm 106:37-38 says, "Yea, they sacrificed their sons and their daughters unto
demons, and shed innocent blood, even the blood of their sons and their daughters whom they
sacrificed unto the idols of Canaan; and the land was polluted with blood." Second Kings
17:17 ; 21:6 ; Jeremiah 7:31 ; 32:35 ; Ezekiel 16:20-21 ; and 23:37-39 all make references to
the Israelite practice of sacrificing children to idols. So if the Amalekites did indeed practice
human sacrifices, why did Yahweh choose to exterminate them by another nation that
sacrificed children to idols? Perhaps McBull can tell us.

The entire Amalekite population had to be destroyed due to the gravity of there [sic] sins
and so that any remnants could not continue their acts of violence against the nation of
Israel . Why, if this hadn't been done, McBull tells us, "we would have had a situation similar
to what now takes place with today's gangs on our streets. You kill one of ours, we kill two of
yours, and so on...." And so what happened? Yahweh sent Saul to utterly destroy the
Amalekites, and after he did, we somehow have David's guerrilla band raiding the Amalekites
who had already been utterly destroyed by Saul (1 Sam. 27:8 ). Even though David "saved
neither man nor woman alive" on this raid (v:11 ), these twice utterly destroyed Amalekites
managed to regroup and raid David's camp (as previously noted). It would seem, then, that
even though the Amalekite population had to be destroyed "so that there would be no
remnants left to continue their acts of violence against the nation of Israel,"some remnants of
an utterly destroyed people did manage to do the very thing that McBull assures us that
Yahweh exterminated them to keep them from doing. How can anyone believe such
ridiculous nonsense that bibliolaters fabricate to defend an indefensible book!

Volume 1990 - 2002 Issue


Page 454 of 2049
Skeptical Review Edited by Farrell Till
While murder is a direct violation of the sixth commandment, not all forms of killing
represents [sic] murder. I won't argue that there is no such thing as justifiable killing;
however, I will argue that it is not justifiable to kill children and babies for something that
their ancestors had done 450 years ago. This point has already been addressed in my other
articles on this subject, so there is no need to rehash it again. What we do need is for someone
who accepts the inerrantist position to offer a satisfactory explanation of the problem. The
best that Mitchell could do was parrot the old fundamentalist line about the children having
been done a big favor, because they died in a state of innocence and would, therefore, go to
heaven instead of growing up to be "vicious beasts" like their parents. Now McBull gives us a
variation of the same theme.

All children who die before the age of accountability are saved (see 2 Sam. 12.23 ), so the act
by which God took the lives of children is far from merciless. Yeah, right, and if McBull has
any children of his own, I'm sure he would consider it a favor if someone murdered them
before they reach "the age of accountability." That, however, isn't the biggest problem with
McBull's final point. Who is to know whether there really is a heaven or even an afterlife of
any kind? There is certainly no tangible, scientific evidence to prove that one's spirit survives
physical death and lives on in another life. It is a claim that requires proof, so Mitchell,
McBull, and anyone else who offers this as a defense of child killing is shamelessly begging
the question. Let's see Mitchell's and McBull's evidence of an eternally blissful existence in
heaven for those who die in a state of innocence (before what McBull calls "the age of
accountability"), and then we can evaluate the strength of their claim that God did the
Amalekite children a big favor by having them killed.

McBull said to Fiori that they could "debate this [the Amalekite massacre] further" or "try
another situation or verse." He may do as he likes, of course, but I strongly suggest that he
"try another situation or verse," because he didn't fare too well on this one.

Dobbs Raves On
by Farrel Till
Bill Lockwood's article "The Skeptic's Sword," submitted to and published in TSR (Spring
1994, pp. 8-9), was also published in the *Firm Foundation* (April 1994, pp. 10-12) but
without Farrell Till's response, of course. At the end of the article editor H. A. "Buster" Dobbs
added the following editor's note:

Farrell Till has been a teacher of English literature at a little-known college in the
backwoods of Illinois for 28 years. He has no following, no influence, and no
standing, even in the atheistic community. He has spent his life for nothing, and, in his
sunset years, is without hope or purpose. "The fool has said in his heart, there is no
God." Of this we are absolutely certain: There will be no atheists in hell!

Volume 1990 - 2002 Issue


Page 455 of 2049
Skeptical Review Edited by Farrell Till
We won't take the time to correct the factual errors that Dobbs made in this statement.
However, if Dobbs really believes everything he said, we have to wonder why he wasted
space in his paper on someone who has no standing "even in the atheistic community." As for
Till's living in the "backwoods of Illinois," Dobbs just might be surprised if he would visit the
area and review its history. Be that as it may, we do agree with one thing he said. There will
be no atheists in hell, and that is because no one can be in a place that doesn't exist. If we are
right about this, then it's pretty obvious who "has spent life for nothing." Dobbs, in fact, had
better wish that we are right, because we have witnessed conduct from him (including lying)
that his Bible warns will condemn a person to eternal hellfire.

We could say much more about Dobbs's comments, but why bother? Such remarks from a
preacher whose church is barely known outside the Hee-Haw environment that spawned it has
little room to criticize the "backwoods" that Till or anyone else lives in. As time ticks on, it
becomes more and more apparent that Dobbs is still smarting from the humiliation that he
suffered in his debate with Till.

Chew on This... Again!


Jeffrey A. Justice
Food for thought: Does Leviticus 11:6 need rechewing? "The rabbit, though it chews the cud,
does not have a split hoof; it is unclean for you" (NIV). Inerrantists refuse to see this
statement as a biological error that incorrectly classifies the rabbit as a ruminant or cud-
chewer. Instead, they resort to speculation and claim that the statement refers to another
physiological process known as "refection," a little known behavior by which some animals
reingest food that has passed through their digestive tracts. The errantist view, which is the
more likely position, argues that the ancients observed the rabbit's jaw movements, which
mimic rumination or cud-chewing, and mistook them as evidence of cud-chewing. Since the
jaw movements are identical to those of the true ruminants, it is little wonder that the Bible
blunders regarding this animal's biology.

To properly evaluate the inerrantist "explanation" of Leviticus 11:6 , we must examine the
difference in refection and rumination. The former is a behavior by which rabbits eat their
own feces (caecotrophs) for nutritional benefit, whereas rumination or cud-chewing is the
regurgitation of food stored in the first chamber of a multichambered stomach, so that the
food can be further broken down mechanically by chewing before it goes on to the next
chamber of the stomach, where it continues to be broken down by chemical processes. (All
true ruminants have chambered stomachs.)

Are They Equal Theories?


There are many weaknesses in the attempts to make refection equal rumination.

Volume 1990 - 2002 Issue


Page 456 of 2049
Skeptical Review Edited by Farrell Till
The major ones include these:

Facts About Refection: In Science in the Bible, a work that argues the inerrantist position,
Jean Morton makes the following comments:

Caecotroph formations were first described in The French Veterinary Journal in 1882.
Since that date [1882], many zoologists have considered this process [refection] as
rumination. The identification of the hare as a ruminant is based primarily on caecotroph
formation and bacterial decomposition of cellulose (Chicago: Moody Press, p. 181).

Caecotroph formations are the soft fecal pellets reingested by the rabbit during refection.
Since they serve the same nutritional purpose as a cud when rechewed, some inerrantists
argue that the Bible correctly classified the rabbit as a ruminant.

I disagree with the so-called "many zoologists" [which zoologists?] because they define
"ruminant" inaccurately. The American Heritage Dictionary defines ruminant as, "Any of
various hoofed, even-toed, usually horned mammals of the suborder Ruminantia, such as
cattle, sheep, goats, deer, and giraffes, characteristically having a stomach divided into four
compartments and chewing a cud consisting of regurgitated, partially digested food." Thus,
the rabbit does not fit this definition, and trying to classify it as a ruminant is a misuse of the
English language.

Morton further said, "The caecotrophs are generally passed during late night or very early
morning hours, so the average individual may not have observed this process" (p. 181). This
fact raises an interesting question. Did Moses refer to the well-known behavior of the camel
(cud-chewing) in Leviticus 11:5 and then in the very next verse refer to a behavior that
probably no one in biblical times had seen because it usually occurs while most people sleep
or lack of sunlight interferes with visibility? Even today, most rabbit owners are not aware of
this behavior because it is a nocturnal practice.

This is a major weakness of the refection theory. To show how weak it is, we have only to
consider certain relevant questions: Did the Leviticus writer refer to a behavior known to the
average Jew? Which behavior did he most likely refer to? Apparent rumination? Refection?
The jaw movements alone? The writer must have alluded to something the Hebrews could
relate to or else he wrote this passage in vain, because no one would have understood him.

In The Private Life of the Rabbit, R. M. Lockley makes the following observations about
refection from his five-year research on the rabbit:

Our watches overnight were almost totally unproductive, except as negative evidence. On rare
occasions, one [rabbit] might return below for an odd minute, even an hour, to rest and preen,
and reingest. However, so long as we could watch the rabbits underground we had the ideal
opportunity to study the phenomena of coprophagy or reingestion.... Reingestion or eating of
excrement [fecal pellets] was seen occasionally out of doors.... Reingestion was observed both
out of doors [occasionally] by day and below ground; but was most easily studied through the
window of the artificial burrow.... Here it was most frequent between 0800 hours and 1700
hour [sic] GMT; that is, for ten hours or so after the night grazing period.... It is an

Volume 1990 - 2002 Issue


Page 457 of 2049
Skeptical Review Edited by Farrell Till
intermittent, not an incessant action.... Within a twenty-four-hour watch underground, three
rabbits reingested a mouthful of pellets, respectively, on 37, 18, and 16 occasions (Macmillan:
NY, 1964, pp. 101,103-104).

Did the ancients watch the rabbits refect in underground borrows at night? Hardly! If the
rabbits reingested on 37, 18, and 16 occasions below ground, then Lockely's use of the term
"occasionally out of doors" probably means a much lower number. This deceases the odds for
anyone both ancient and modern to observe this behavior. Lockely further says, "It could
easily be overlooked by the casual observer..." (Ibid., p. 103, emphasis added). So if the
casual observer today can easily overlook this, what about the ancient casual observer?

According to Lockely, the rabbit quickly retrieves the pellets from the anus before they hit the
ground (p. 103). He states that the casual observer would probably not catch the rabbit
making this quick movement (p. 103). Even if the ancients had witnessed the chewing of the
feces which can last from one to over a hundred seconds (p. 105), we should ask: what are the
odds for the ancient to have chanced upon the rabbit quickly retrieving the pellets before
chewing? If he had witnessed this action, what would he have thought of it? Would the same
individual witness this frequently enough to think this was typical behavior? Would he think
it was rumination? The possibility seems both humorous and doubtful. If he witnessed only
the chewing, would he have assumed the rabbit was chewing regurgitated bolus? "The
observer could not see the faeces because of the masking action [of the rabbit's mouth
structure] and the closed mouth" (p. 103, emphasis added). Because of this, an ancient would
have had difficulty knowing, if witnessing reingestion, that the rabbit had feces in its mouth.
Since he could not have seen the feces, he would have more likely assumed the rabbit was
chewing cud.

Lockley goes on to say, "Later writers [after Leviticus], even authorities on both wild and
domestic rabbits, have seldom referred to this phenomenon. In general, textbooks on rabbit
physiology and management ignore the subject, not because of its lowly theme but because
the authors apparently were ignorant of it" (p. 102, emphasis added). If, therefore, it is true
that even authors of textbooks on rabbit physiology have been ignorant of refection, why
should we believe the unsophisticated ancients were aware of it? Can we reasonably believe
this behavior went unnoticed for thousands of years even by late rabbit physiologists yet was
common knowledge to the ancient Hebrews? Can the inerrantist believe this more than that
the rabbit's jaw movements were mistaken for rumination? Every rabbit hunter, owner, and
physiologist is familiar with this frequent behavior! On the other hand, if refection was such
an unobvious behavior that people overlooked it for centuries, would it have been an obvious
enough behavior for Moses and those who listened to him to be aware of it? Lockley
ironically quotes Leviticus 11:6 as an example of refection in the Bible (pp. 101-102)!
However, he gives absolutely no reason to justify his claim. Apparently, he overlooked the
overwhelming implications of his own research.

Phillip Eichman, Ph.D., cited his experience with a pet rabbit to support Morton's caecotroph-
formation view:

We had a rabbit once which was house broken and lived entirely in the house with us
humans.... His normal biological clock was altered by living with us. He often ate his night

Volume 1990 - 2002 Issue


Page 458 of 2049
Skeptical Review Edited by Farrell Till
feces during the day. At first we thought that this was just a disgusting habit which he had
developed. However, I was at the time an undergraduate biology major and happened to learn
this was normal, healthy behavior for a rabbit. In fact, the rabbit needs this material and the
vitamins and special factors produced by the microorganisms for its survival" ("Does the
Rabbit Chew the Cud?" Does God Exist? March-April 1990, p. 19).

Dr. Eichman concluded from this that the rabbit practiced a "kind" of rumination. Take note,
however, of the rabbit owner's initial response: he thought this behavior was "just a disgusting
habit which he had developed." Dr. Eichman did not relate this behavior to rumination or a
natural process until he learned that it involved chemistry similar to rumination. Certainly, the
writer of Leviticus could not have believed that rabbits chewed the cud because he knew
about the role of microorganisms that break down cellulose in the rabbit's caecotrophs or
because he was aware of the nutritional value of the caecotroph formations, especially if they
were first described in 1882! Cellulose breakdown is an idea too modern for Moses' time.
Would these characteristics alone be enough to make an animal that has a split hoof clean to
God?

Incidentally, this example of a rabbit's refecting by day, because indoor captivity altered its
biological clock, proves that the casual observer can witness refection in certain
circumstances. How regular this disturbance of the circadian rhythms of rabbits kept indoors
is may be worth further research. However, the Hebrews probably did not keep rabbits as pets
inside their homes.

Like Morton and Lockley, Dr. Eichman, overlooked the significance of the fact that most
people never witness refection in rabbits:

The material from the caecum is then passed on through the intestines as a special type of
feces known as night feces because this normally occurs at night.... Most people have never
seen a rabbit do this" (Ibid., p. 19, emphasis added).

Yet, Dr. Eichman expects us to believe that Moses was talking about refection and that most
of the Hebrews understood what he was saying! Lack of Historical Records: Inerrantists have
presented no ancient records that mention refection, which strongly suggests that it is a recent
discovery to which the writer of Leviticus was not privy.

Ritual Cleanliness: Because of their obsession with ritual cleanliness, it is doubtful the Jews
would have ever thought the eating of waste made an animal clean! Would God have to tell
the Jews not to eat such an animal because the Jew would interpret this reingestion of fecal
matter as rumination and as a sign of cleanliness?

Ancient Taxonomy: In The Encyclopedia of Bible Difficulties Gleason Archer made this
attempt to resolve the problem:

In this technical sense neither the hyrax nor the hare can be called ruminants, but they do give
the appearance of chewing their cud in the same way ruminants do. So convincing is this
appearance that even Linnaeus at first classed them as ruminants, even though the four-
stomach apparatus was lacking" (1982, p. 126).

Volume 1990 - 2002 Issue


Page 459 of 2049
Skeptical Review Edited by Farrell Till
If Archer is correct that even Linnaeus, an inquiring scientist, mistakenly classified the rabbit
as a ruminant because the sidewise jaw movements fooled him, can we believe that anyone
before his time knew better? So it appears that the writer of Leviticus made the same error
Linnaeus did. To claim the contrary requires verifiable evidence.

The Rock Badger: We must not overlook the rock badger mentioned in Leviticus 11:5 . I
could find no discussion of this creature in inerrantist literature, and I think I know why. I
stumbled upon some possible implications presented by this animal for the inerrancy debate.
The rock badger of Leviticus 11:5 may "hold the key" to the validity of the refection
argument for the rabbit of Leviticus 11:6 . Further inquiry on the rock badger may determine
this argument's success or failure. I have been unable to access any material that deals
specifically with the biology of the rock badger, so errantists reading this article may wish to
research this animal to develop a better defense. Fauna and Flora of the Bible identifies the
rock badger as the Syrian coney [Procavia syriacus] of the order Hyracoidea in the group of
subungulates (1972, pp. 69-70). We find this animal also wrongly listed in Leviticus as a
ruminant. Discussing the rock badger, the authors of Fauna and Flora of the Bible point out,
"(A)lthough it does not belong to the ruminants among which it is mentioned in Lv. 11, when
chewing it moves its jaws in a way that resembles a ruminant" (p. 69). So was the writer of
Leviticus deceived by both the rabbit's and the rock badger's sidewise jaw movements?

Fauna and Flora of the Bible has also tried to make the rabbit a ruminant:

It is generally assumed that the Hebrews thought the rabbit was a ruminant through observing
the peculiar movements of its jaws when chewing. Today we know that physiologically the
hare is a rodent [highly debatable], although the zoologists Osen and Madsen [Investigations
on Pseudo-Rumination in Rabbits, 1943] have demonstrated that the hare and the rabbit
perform a kind of rumination (p. 39).

Notice that Fauna and Flora... does not say whether the rock badger refects, although it gives
refection as a possible explanation for the rabbit. Perhaps the writers would have used this
explanation for the rock badger if the rock badger does refect, since they thought it a plausible
alternative explanation for the rabbit. One wonders whether the rock badger does indeed
refect, a matter important to both the inerrantist and the liberal scholar.

The implications are obvious. If it cannot be proven that the rock badger refects, then the
inerrantist claim that the expression "chews the cud" can refer either to refection or
rumination [whichever God secretly had in mind] will not stand, since the rock badger
practices neither! Thus, the inerrantist must either concede that there are scientific errors in
the Bible resulting from ancient misunderstandings or that the Bible is excessively
inconsistent in its use of language, since the phrase "chews the cud" will have to be stretched
to mean jaw movements, true rumination, or refection! Should it come to this, the inerrantists
will have to modify their present position or devise more plausible arguments. Any
modifications will have to be based on evidence, not on the usual resorting to speculation that
is characteristic of most of their arguments.

The Hebrew Language: Now here is the clincher! According to Strong's Exhaustive
Concordance of the Bible, "cud" [gerah] in Leviticus 11:6 means, "the cud [as scraping the

Volume 1990 - 2002 Issue


Page 460 of 2049
Skeptical Review Edited by Farrell Till
throat]: --cud" (word 1625). The American Heritage Dictionary defines cud as, "Food
regurgitated from the first stomach to the mouth of a ruminant and chewed again." Strong
doesn't help the refection theory since his translation is the same as the English definition, and
its root meaning in Hebrew was "scraping the throat." So cud is cud, not rabbit pellets. In
what sense could rabbit pellets be considered a scraping of the throat?

The word chews in Leviticus 11:6 , translated from the Hebrew word `alah, more accurately
means, "[cause to] ascend up" (Strong's, word 5927). Thus, the Hebrew phrase for "chews the
cud" translated literally means, "brings up the cud." This conclusively proves that Leviticus
11:6 gives a description of regurgitation, characteristic of true rumination but in no sense
characteristic of refection. Refection does not involve regurgitation; thus, the phrase "brings
up the cud" cannot refer to the eating of excrement. To claim this would distort the writer's
intended meaning. It would be very inaccurate to say of an animal, e.g., a dog, seen frequently
eating its feces, "Fido has been bringing up his food." This would sound ridiculous and would
even be misleading, because it suggests that "Fido has been vomiting." When the inerrantists
use the refection argument, they assume they need to prove that the rabbit chews, redigests, or
forms cud, but this distracts from the real issue. They need to prove that the rabbit brings up
the cud! The refection argument fails to prove this. Until it does, we may consider it
erroneous, and that is not all!What inerrantists also need to prove is that gerah in this verse
means something other than regurgitated cud and that `alah means something other than
regurgitation. All of this needs to be proven with verifiable evidence. Refection has a few
characteristics similar to rumination but not the most important one: regurgitation, the same
characteristic the Bible refers to! The best the inerrantist can claim is that the phrase "chews
the cud" can apply to refection in some forced, nonsensical, strained, ludicrous, and unnatural
sense. The inerrantist cannot prove it is even unlikely that "brings up the cud" refers to
regurgitation of cud! So one wonders why the writer of Leviticus did not use the Hebrew
word for "dung" if he meant refection!

Does God Lie... Sometimes?


In their desperate attempts to defend the Bible, some inerrantists actually portray God as a
cunning liar. For example, Gleason Archer, who is notorious for his speculative arguments,
attempts to tackle the bunny problem in a way very unflattering to God: But we need to
remember that this list of forbidden animals was intended to be a practical guide for the
ordinary Israelite as he was out in the wild looking for food. He might well conclude from this
sideways movement of the jaws that these animals ruminated like the larger cattle; and since
they fed on the same kind of grass and herbs, they might well be eligible for human
consumption. Thus it was necessary to point out that they did not have hooves at all and
therefore could not meet the requirements for clean food" (p. 126).

Now wait just a minute! Notice what Archer is saying. He is saying effectively that this
passage in Leviticus was written for the common hunter and, therefore, described this
phenomenon from the hunter's mistaken point of view. The hunter might see the jaw
movements and mistake the rabbit for a ruminant and thus consider it clean. Therefore, God
validated this misconception but pointed out that the rabbit did not have hooves! Mr. Archer
doesn't seem to realize he has contradicted himself. An important inerrantist argument is that
the Bible doesn't echo ancient scientific errors, but Archer's argument implies that God just

Volume 1990 - 2002 Issue


Page 461 of 2049
Skeptical Review Edited by Farrell Till
"played dumb" and, to keep them from making a mistake, lied to the gullible Jewish hunters
by telling them the rabbit brings up its cud. Thus, God had good intentions or reasons for
employing this duplicity. So instead of arguing that Leviticus 11:6 does not contain an error,
Archer has admitted that it does contain an error and then proceeded to provide excuses for
God's error! However, excuses or good intentions for making errors do not make errors not
errors!

Some inerrantists will even suggest that God used metaphorical language to give the Jews the
impression that he meant rumination, while he really meant refection. Millennia would pass
before the truth about the rabbit's feeding habits would become known through modern
science, so God just didn't tell his people this scientific truth about rabbits. Does such a
devious God deserve our respect? If the ancients were unaware of refection and if God
secretly referred to this process, God knew in advance that the Hebrews would interpret him
to mean what they perceived as genuine rumination. Why did God fool his own "chosen
people"? Why has he fooled most modern lay Bible students? Why does God bother to speak
to us at all if he does not care about miscommunication? Do the Scriptures get their intended
meaning from the human writer's language or from God who covertly misuses the writer's
language?

Other inerrantists argue that God used "accommodative language" to refer to the rabbit's
alleged rumination. Thus, it is explained, God didn't mean anything literal or scientifically
precise when he inspired the writer to write this phrase. In other words, God didn't necessarily
refer to refection nor did he mean anything literal about rumination.

The difficulty with invoking this explanation is the same as the metaphorical-language
explanation: did God, without making it known that he was using some type of figurative
language, use this deceptive phrase to give the false impression that he validated an ancient
scientific misconception? The Bible was speaking of clean and unclean animals that required
literal characteristics to determine their edibility. How can an animal figuratively classify as
clean or unclean to God? Would it not have been more direct to avoid some weird
metaphorical or accommodative language by saying the rabbit is unclean because it really
doesn't bring up its cud? Surely God knows how to express "truth" this clearly! This, by the
way, would have constituted an awesome proof of "scientific foreknowledge" in the Bible that
inerrantists could have appropriated to use in their evangelistic efforts to convert unbelievers,
not even to mention that God wouldn't have had to go through the trouble of pointing out that
the rabbit doesn't have hooves! Besides all this, accommodative-language and metaphorical
arguments alone could defend practically any religious book no matter how erroneous it was!
These arguments should be replaced with verifiable defenses.

Notice that the inerrantist dares never to define precisely what an irrefutable error is! Since he
does not define what an error is, many gullible people do not know how to find them. Instead,
he arbitrarily argues that particular scriptures are not in error, because they are true in any
sense that he chooses to assign to them. He might argue a scripture is true historically,
chronologically, or scientifically. If the Bible errs in chronology, he may claim that
chronology is not the issue; the Biblical writer's intent was to show events in their "order of
importance." If the Bible errs in matters of science, he might argue that science is not the
issue; the writer's intended meaning was merely figurative. It is as if he is always arguing,

Volume 1990 - 2002 Issue


Page 462 of 2049
Skeptical Review Edited by Farrell Till
"Yes, this may be in error if interpreted that way, but you are really supposed to interpret it
this way." Therefore, he can always ignore the most valid or likely meaning of a passage and
invent another way of interpretation to excuse errors. True, the Bible requires different
methods of interpretation, but these methods should demand certain consistent and
nonarbitrary rules for application. If they do not, the inerrantist cannot tell us what the Bible
says since he really does not know how to interpret it!

Is There a Way Out?


I have formulated below several arguments to support the inerrantist position. Since the
evidence against them is overwhelming and leaves little room for convincing dispute, they
appear weak and unconvincing, but don't be at all surprised if inerrantists appropriate some of
them if you should ever bring up the cud-chewing problem.

[1] Refection involves "bringing up" food technically to the oral cavity but by an external
indirect route rather than an internal direct one, and this was what Moses had in mind.
(However, if this is argued, one should know such things as whether the Hebrews used the
term gerah [cud] interchangeably. Could it mean cud and/or feces? Strong's Exhaustive
Concordance of the Bible gives us but one choice. As one can see, this argument is just
plainly ridiculous for it ignores the context and apparent language of Leviticus 11:6 .)

[2] God preferred to classify the rabbit as a ruminant based on refection, just as some
unidentified zoologists have done.

[3] God used accommodative, idiomatic, or metaphorical language in some strange way
without lying or deceiving.

[4] God used the recent popular concept of "fuzzy logic" (Kosko, Bart, Satoru, "Fuzzy Logic,"
Scientific American, July 1993, pp. 76-81). He meant to say that it is 80% true that the rabbit
is a ruminant. (I would be surprised to hear this explanation from an evangelical inerrantist,
since the idea is influenced by many non-Christian ideas as Buddhism and mysticism and
conflicts with the Socratic Manichaeism of Pauline theology. The inerrantist would have to
redefine his position, since if God can state sentences that are 80% true, then it follows that
such sentences are 20% false. This would also mean that 100% inerrancy does not exist!)

[5] Refection was probably known to ancients, because they would have seen the soft pellets
upon gutting the rabbit to prepare it for eating. (This is just a silly inerrantist argument that I
have fabricated. According to R. M. Lockley, they could have seen these pellets only if they
cut open the stomach and if they killed rabbits early in the evening. It is doubtful, however,
that the ancients would have thought these soft pellets were for reingestion. Instead of
construing the pellets as anything related to rumination or refection, the ancients probably
would have thought that the soft pellets were nothing more than ordinary hard pellets in an
early stage of digestion.)

Comments

Volume 1990 - 2002 Issue


Page 463 of 2049
Skeptical Review Edited by Farrell Till
It is, of course, possible some of the ancient Hebrews were aware of refection, but don't
conclude too much too fast! This wouldn't necessarily imply that Leviticus 11:6 is not a
reference to rumination. Who knows? The ancients could have thought that the rabbit chewed
both regurgitated and excreted food!

I have attempted to give many different facts to interpret Leviticus 11:6 in its historical,
cultural, and linguistic context. I am sure there are many more factors to consider. Thus, I
would strongly urge all who read this article to take my food for thought and chew on it. I
think, however, I have provided skeptics with adequate grounds to question the scientific
accuracy of Leviticus 11:6 . If someone tries to convince you that this passage has already
been explained by the refection argument, "bring up" this issue and tell him he needs to "chew
on this again!"

(Jeff Justice, P. O. Box 454, Wylie, TX 75098; comments and reactions invited.)

The Evolution and Devolution of the Bible


T. Brett Bottger
Just as modern scientists are certain that humankind evolved from lower life forms so too are
critical scholars certain that the Bible is a product of an evolutionary process of its own. How
else can one explain how the greatest god in heaven (Gen. 14:18-22 ) became the "only god in
heaven" in the later books of the Bible? And is there any other way to explain how the cold,
dark and dreary underworld of Sheol became the fiery underworld of Hades except as a
process of theological evolution?

In fact, it was the setting of Sheol on fire by certain radical Jews who were influenced by the
Greek concept of Hades that was instrumental in bringing the evolution of Judaism to a
grinding halt. The Jews believed that the ghosts of everyone who had ever lived were in
Sheol, including all of their saints. The notion that the whole place was on fire was pure
insanity. They canonized the Old Testament books, bringing Jewish evolution to an end.

It was the radical schismatics who now carried on the evolutionary process. They focused on
the Messianic prophecies and added a new mission to the Christ's "list of things to do when I
get to earth." That new item was "get the good people out of Hades." How the Messiah would
accomplish this was heavily debated throughout the intertestamental period.

Any and all claimants to the Messianic crown (John the Baptist for one) were blown out of
the water by the shrewd promoters of Jesus of Nazareth. His movement took the name
"Christianity" and began its own evolutionary process.

Convincing a meaningful number of people that an ordinary mortal had been the Messiah
required a truly hard-sell effort. Accomplishing it required that Jesus be promoted as

Volume 1990 - 2002 Issue


Page 464 of 2049
Skeptical Review Edited by Farrell Till
something far above the ordinary mortal. Rumors of miraculous acts performed by him were
soon circulating on his behalf. Almost a century would pass before the Gospel writers would
record these legends. By that time, the stories about his supernatural deeds had been so
expanded and had been repeated so often that his claim to Messiahship was secure.

But stories of Jesus' miraculous exploits continued to be invented long after the Gospels were
written, and each new story seemed intentionally designed to surpass the last in its
incredibility. Many of these stories were written down and were considered by early
Christians to be as divinely inspired as any of the books of the Bible in our present canon.
Many were read at church services as regularly as we read from the Gospels in today's
services.

And still the legends grew. Even as new books were written, the church fathers wrangled over
which of them were "authoritative" and which were "spurious." It was not until early in the
fourth century that a catalogue of New Testament writings similar to those in the present
canon was first suggested to be authentic by Eusebius of Caesarea.

Deifying Jesus had been essential to Christianity, but somewhere along the line, the
evolutionary process that had made him bigger than life had to be halted lest he become a
transparent collection of superhuman cliches. The little white lies that had laid the snowy
blanket of Christianity were becoming a full-blown blizzard threatening to wipe out the whole
movement.

So the evolutionary process had gone too far. Eusebius and other church fathers realized that a
bit of "devolution" was called for. Thus, books that contained creative history so overly
exuberant as to be immediately transparent were expunged from the canon. As it turned out,
that included practically everything written after c. 150 B. C.

What I have said so far might sound like unsubstantiated hypothesizing from a notorious
Bible-hater, but I would counter that you can see the substance of my argument by examining
some of the expunged books for yourself. When you've seen the incredible proportion of the
lies told in the latest of them and follow the process back to the earliest of them (the Gospels
themselves) you will recognize that they too contain similar untruths with similar intents.

It is an eye-opening exercise that will impress even a "true Christian" with how far early
Christians were willing to go to inflate the image of their Messiah. Here are just a few
examples.

As a boy, Jesus was playing on a river bank on the Sabbath. Another boy came along and saw
young Jesus making fish pools and became righteously indignant over Jesus' breaking of the
Sabbath laws. The boy set about destroying the pools, whereupon Jesus struck him dead. In
the same chapter, Jesus struck another boy dead for bumping into him while running in the
street (I Infancy 19).

After dying on the cross, Jesus descended into hell. There he defeated Death, Satan, and the
Prince of Hell. He then took the saints with him to Paradise (Nicodemas 16-19). [One can
only wonder how anyone was expected to believe that this was an eye-witness account.]

Volume 1990 - 2002 Issue


Page 465 of 2049
Skeptical Review Edited by Farrell Till
When Herod ordered the murder of the infants, Elizabeth, Mary's cousin, fled with her baby,
John the Baptist. Finding no place to hide, she said a prayer to God. Immediately a whole
mountain split open, and Elizabeth and her baby hid inside under the protection of an angel
(Protoevangelion 16).

After Jesus' death, Pilate began to worry that he might have killed the real Messiah. He went
to the temple to have the scribes examine the Scriptures. Having the same worry, the scribes
had already consulted their writings. Sure enough, there was proof in the Old Testament that
Jesus was the Messiah (Nicodemus 22).

The passage cited as this proof was Exodus 25:19 , "And they shall make an ark of shittim
wood: two cubits and a half shall be the length thereof, and a cubit and a half the breadth
thereof, and a cubit and a half the height thereof." The scribes had interpreted this as a
prophecy that five thousand five hundred years (a thousand years per cubit) after the creation
of Adam, the Christ would come.

Obviously, this author was no mathematician. According to the complete biblical genealogy,
the number of years from Adam to Jesus was just slightly more than 4,000. So if we accept
the dimensions of the ark of the covenant as a prophecy in the manner reckoned by these
scribes, the true Messiah could not have arrived until several centuries after the time Jesus
allegedly lived.

Fundamentalists decry the use of the "lost books" as a tool for disputation of the canonical
books. They claim that God himself inspired information in the New Testament canon (a
distasteful concession fundamentalists are forced to make to canonical selections made by the
Catholic Church). But they should take a lesson from what every spy and detective knows:
you can learn an awful lot from what you find in a person's trash.

(T. Brett Bottger, Bible Origins Research Institute, 5938 South Albany Avenue, Chicago, IL
60629.)

Editor's Note: According to ages given in the patriarchal genealogies of Genesis 5 , there
were 1,656 years from the time of Adam's creation to Noah's flood. The postdiluvian
genealogies of Genesis 11 list 297 years from the flood to the birth of Abraham, who was100
years old when Isaac was born (Gen. 21:5 ). Isaac was 60 years old when Jacob wasborn
(Gen. 25:26 ), and Jacob was 130 years old when Joseph presented him to Pharaoh shortly
after the Israelite migration into Egypt (Gen. 47:9 ). Hence, 2,243 years passed from Adam to
the descent into Egypt. The Israelites sojourned in Egypt 430 years (Exodus12:40 ), and
Solomon, in the fourth year of his reign, began to build the Temple 480 yearsafter the Israelite
exodus (1 Kings 6:1 ). Therefore, the Bible's own chronology teaches that 3,153 years
transpired from Adam to the fourth year of Solomon's reign, when construction on the temple
began. Bible chronologists assign 930 B.C. as the year of Solomon's death (see SOLOMON,
Eerdmans Bible Dictionary, 1987, p. 959). Since Solomon reigned for 40years (2 Chron. 9:30
), construction on the temple would have begun 36 years earlier in 966B. C., so if the 966
years that separated the fourth year of Solomon's reign from the birth ofJesus are added to the

Volume 1990 - 2002 Issue


Page 466 of 2049
Skeptical Review Edited by Farrell Till
above 3,153 years, we have only 4,119 years. Since Jesus was about33 when he was allegedly
crucified, we see that Bible chronology allows for only 4,152 yearsfrom Adam to the
crucifixion. Hence, Bottger was exactly right in saying that the temple scribes and the author
of the apocryphal book Nicodemus were poor mathematicians.

Zoroastrianism: The Forgotten Source


Stephen Van Eck
The composition Also Sprach Sarathustra by Richard Strauss featured in 2001 is a piece of
powerful drama, rich in majesty, awe-inspiring, and devastatingly portentous. It is an
appropriate memorial to the Persian prophet Zarathustra, whom the Greeks called Zoroaster.

Zarathustra's influence on Judeo-Christianity and all of western civilization is little known but
should not be underestimated. His life and words changed the nature of civilization in the
west, setting it on a course that departed from the static cultures of the ancient Middle East.
Without his impact, Judaism would be unrecognizable, and Christianity would probably have
never existed.

Western civilization owes mainly to Zarathustra its fundamental concept of linear time, as
opposed to the cyclical and essentially static concept of ancient times. This concept, which
was implicit in Zarathustra's doctrines, makes the notion of progress, reform, and
improvement possible. Until that time, ancient civilizations, particularly Egyptian, were
profoundly conservative, believing that the ideal order had been handed down to them by the
gods in some mythical Golden Age. Their task was to adhere to the established traditions as
closely as possible. To reform or modify them in any way would have been a deviation from
and diminution of the ideal. Zarathustra gave Persian (and through it, Greek) thought a
teleological dimension, with a purpose and goal to history. All people, he declared, were
participants in a supernatural battle between Good and Evil, the battleground for which was
the Earth, and the very body of individual Man as well. This essential dualism was adopted by
the Jews, who only after exposure to Zoroastrianism incorporated a demonology and
angelology into their religion. Retroactively, what was only a snake in the Genesis tale came
to be irrevocably associated with the Devil, and belief in demonic possession came to be a
cultural obsession, as amply reflected in the Gospels.

Zarathustra claimed special divine revelation and had attempted to establish the worship of
one supreme God (Ahura Mazda) in the 7th century B. C., but after his death, the earlier
Aryan polytheism reemerged. Many other features of his theology, however, have endured to
the present time, through the religions that eventually superseded it.

The Babylonian captivity of the 6th century B. C. transformed Judaism in a profound way,
exposing the Jews to Zoroastrianism, which was virtually the state religion of Babylon at the
time. Until then, the Jewish conception of the afterlife was vague. A shadowy existence in

Volume 1990 - 2002 Issue


Page 467 of 2049
Skeptical Review Edited by Farrell Till
Sheol, the underworld, land of the dead (not to be confused with Hell) was all they had to
look forward to. Zarathustra, however, had preached the bodily resurrection of the dead, who
would face a last judgment (both individual and general) to determine their ultimate fate in
the next life: either Paradise or torment. Daniel was the first Jewish prophet to refer to
resurrection, judgment, and reward or punishment (12:2 ), and insofar as he was an advisor to
King Darius (erroneously referred to as a Mede), he was in a position to know the religion
thoroughly.

The new doctrine of resurrection was not universally accepted by the Jews and remained a
point of contention for centuries until its ultimate acceptance. The Gospels (Matthew 22:23 )
record that the dispute was still going on during the time of Christ, with the Sadducees
denying and the Pharisees affirming it. It may be a mere coincidence, but note the similarity
between the names Pharisee and Farsi or Parsee, the Persians from whom the doctrine of
resurrection was borrowed. In addition to incorporating the doctrines of resurrection and
judgment, exposure to Zoroastrianism substantially altered Jewish Messianism as well.
Zarathustra predicted the imminent arrival of a World Savior (Saoshyant), who would be born
of a virgin and who would lead humanity in the final battle against Evil. Jewish Messianism
grafted these conceptions onto their preexisting expectations of a Davidic king who would
redeem the Jewish nation from foreign oppression.

It was at this time, as a response to their captivity, that the era of apocalyptic literature
commenced in Judaism, based on Babylonian models and patterned after their symbology.
This was to have a strong influence on later Christian thinking. With the key elements of
resurrection, judgment, reward or punishment, a Savior, apocalyptism, and ultimate
destruction of the forces of Evil, it can be concluded that Jewish and Christian eschatology is
Zoroastrian from start to finish.

The similarities don't end with eschatology either. A lot of the tradition and sacramental ritual
of Christianity, particularly Catholicism, traces back to Zoroastrian precursors. The
Zoroastrian faithful would mark their foreheads with ash before approaching the sacred fire, a
gesture that resembles Ash Wednesday tradition. Part of their purification before participating
in ritual was the confession of sins, categorized (as Catholics do) as consisting of thought,
word, or deed. Zoroastrians also had a Eucharistic ritual, the Haoma ritual, in which the god
Haoma, or rather his presence, was sacrificed in a plant. The worshipers would drink the juice
in expectation of eventual immortality. Finally, Zoroastrians celebrated All Souls' Day,
reflecting, like the Catholics, a belief in intercession by and for the dead. We should also note
that the story of the Magi, who were said to have visited the newborn Jesus, resembles an
earlier story of Magi who looked for a star foretelling the birth of a Savior, in this case
Mithras. Magi were not kings but Zoroastrian astrologers, and the birthday of Mithras on
December 25th was deliberately appropriated by the church to be that of their Christ, whose
actual date of birth is unknown and undocumented.

Christianity may also have borrowed the story of the temptation in the desert, since an earlier
legend placed Zarathustra himself in that situation. The principal demon (Ahriman) promised
Zarathustra earthly power if he would forsake the worship of the supreme God. Ahriman, like
Satan when tempting Jesus, failed.

Volume 1990 - 2002 Issue


Page 468 of 2049
Skeptical Review Edited by Farrell Till
A final interesting parallel is the three days that Jesus spent in the grave. This concept may
have been derived from a Zoroastrian belief that the soul remains in the body for three days
before departing. Three days would have established death yet left his soul in a position to
reanimate his body. As a Messiah, Jesus functioned purely along Zoroastrian lines. While
purportedly of the Davidic line, he offered only redemption from sin, rather than national
salvation for the Jews. He was a world savior rather than a Jewish Messiah. Jews did not
recognize him as their Messiah, and in a real sense he wasn't. Their Messianic expectations,
which preceded any foreign influence, went unfulfilled; in fact, their nation was ultimately
destroyed. Neither did Jesus effect a final triumph over Evil. This has been reserved for a
second coming in conjunction with the last judgment and the rewards and punishments of
either Heaven or Hell.

Although Zoroastrianism is almost extinct today, it lives on in its spiritual descendants.


Zarathustra, a prophet beyond any in the Old Testament, still speaks today, unrecognized by
his children.

"Let us worship Zarathustra,


Just the way we used ta.
I'm a Zarathustra boosta--
He's good enough for me."

(Joseph Campbell, with a tongue-in-cheek parody.)

(Stephen Van Eck, Route One, Box 62, Rushville, PA 18839-9702.)

From the Mailbag


Dear Brother Dobbs:

I recently subscribed to The Skeptical Review and am pleased to see how well you and
Brother Mitchell are refuting the errors and misconceptions of its Editor, Farrell Till. I learned
about the Review through an ad in The Humanist. I'm a member of the American Humanist
Association, joining to get a first-hand idea of what they're all about, and have proposed twice
to serve as their "resident fundamentalist" if they would publish my views in The Humanist.
My assumption is that, since the AHA considers itself to be composed of such "free thinkers,"
they would certainly be open to an opposing point of view, but, alas, they never responded. I
wonder why!?

I can understand how you and Brother Dobbs [sic] find Editor Till difficult with which [sic] to
reason, but you need to remember that it's often those who once believed who turn into the
most vigorous opponents. Consider that Madelaine [sic] Murray O'Hare [sic] and B. F.

Volume 1990 - 2002 Issue


Page 469 of 2049
Skeptical Review Edited by Farrell Till
Skinner were raised as Presbyterians, Hugh Hefner was raised in a conservative Methodist
home, Joseph Stalin was a seminary student, and Adolph Hitler sang in the church choir as a
boy.

People rarely lose their faith for intellectual reasons but because they suffered some personal
hurt (as with the foregoing examples), even though they assert--and may even have convinced
themselves--that they really just got smart. They will then go on ad nauseum [sic] in an effort
to justify their loss of faith, as demonstrated by Skinner's behavioristic ideology and Hefner's
hedonistic philosophy. Thus whatever personal hurt Editor Till suffered, let us pray that he
may yet be led back to the Lord through love.

I recently concluded conducting a series of ten lessons on "Religious and Moral Authority in a
Do-Your-Own Thing Age" at Fairfax in which I traced the evolution of religious and secular
authority and its destruction in modern times, thus accounting for views such as Editor Till
now professes. But as the Apostle Paul tells us in Galatians 6:7 , "Be not deceived, God
cannot be mocked, for whatsoever a man sows, that shall he also reap."

(Name and address withheld for reasons stated below.)

EDITOR'S RESPONSE: This letter was written to Lindell Mitchell and H. A. Dobbs by a
Church-of-Christ preacher in Fairfax, Virginia, who sent a copy to us. Upon receiving an
advanced copy of the letters column for this issue, he wrote back immediately and declined
the offer to be our "resident fundamentalist." He said that he was refusing the offer, even
though he recognized "the risk of appearing to [us] to having had [his] bluff called." (Now
why would he think we would think that?) He went on to say that he "would also ask that [we]
not publish [his] letter." Because of his request, we are withholding his name. The letter itself
is being published in order to demonstrate something that I know that many readers probably
don't know. Bible fundamentalists often speak very boldly in the pulpit and in their personal
conversations and correspondence with others who are like-minded, but they aren't nearly as
bold about defending their claims in a forum that would allow equal time to an informed
opposition.

This letter-writer is exactly right. I think he backed down after his bluff was called. If he was
so anxious to be the "resident fundamentalist" for a magazine that focuses primarily on the
humanist view of social and political issues, why would he refuse the opportunity to be the
"resident fundamentalist" for a publication whose chief focus is on Bible fundamentalism?
What else should we believe except that he backed down when his bluff was called?

The following comments were written and sent to Mr. X along with the above setup of his
letter. The only change we have made is the substitution of X for the writer's name whenever
it was used....

Mr. X's letter confirms something that I have often told Bible fundamentalists: they are their
own worst enemies. From its inception, the policy of TSR has been to give Bible inerrantists
equal space to respond to the articles that we publish. Our reason for this policy is quite
simple: we want to give inerrantists the rope to hang themselves. The far-fetched, ridiculous
"explanations" that they resort to in trying to resolve obvious discrepancies and

Volume 1990 - 2002 Issue


Page 470 of 2049
Skeptical Review Edited by Farrell Till
inconsistencies in the biblical text merely confirm that the traditional view of Bible inerrancy
cannot be logically defended. The more we expose their failures to our readers, the more we
help our case. That is why articles from bibliolaters are always welcomed at TSR. This letter
from Mr. X also effectively illustrates the inability of bibliolaters to think logically, so we are
happy to publish it.

X tells us that since subscribing to The Skeptical Review he has been pleased to see how well
Dobbs and Mitchell are "refuting the errors and misconceptions of its (e)ditor, Farrell Till."
Oh? Well, I must have missed something somewhere along the way, because I have an
entirely different impression. For one thing, Dobbs has written nothing for TSR, although he
has been invited and even challenged to do so. We did quote from a letter in which Dobbs
launched an ad hominem attack on me, but there was certainly nothing in this brief quotation
that could even remotely be considered a refutation of my "errors and misconceptions." If X
was referring to my oral debate with Dobbs, perhaps he will be kind enough to point out to us
exactly what Dobbs said in it that refuted my "errors and misconceptions," because my mail
has been running decidedly in the opposite direction of X's assessment. Even some Church-of-
Christ members have said that, although they don't agree with my position, they consider
Dobbs's performance in the debate an embarrassment. Is this what X considers a creditable
job of refutation?

As for Mitchell, why did he unilaterally declare victory and drop out of the debate if he was
doing such a marvelous job of refuting my "errors and misconceptions"? One would think
that he would have welcomed the opportunity to stay in the fray and refute my "errors and
misconceptions" even more. But he didn't. He quit. Why? Furthermore, Mitchell has rejected
my challenge to debate the Bible-morality issue publicly on the grounds that he has no desire
to humiliate me. Yeah, sure, we can all believe that, can't we? He's just too good of a guy to
have a desire to humiliate an atheist in public.

Mr. X wonders why The Humanist did not leap at his offer to be their "resident
fundamentalist." Well, perhaps he will find the answer to his question if he will just read his
letter with a critical eye. He asserted that "it's often those who once believed who turn into
the most vigorous opponents [of Christianity]" and asserted that such people "rarely lose
their faith for intellectual reasons but because they suffered some personal hurt." He advised
Dobbs and Mitchell to pray that "whatever personal hurt Editor Till suffered," he "may yet be
led back to the Lord through love."

Such a maze of muddled thinking hardly deserves a response, but I have a serious personality
flaw. When a debating opponent puts his foot into his mouth, I experience an irresistible urge
to shove it in even farther. X has made a claim that is familiar to most ex-fundamentalists who
have become atheists or agnostics: the decision to become a skeptic is usually not an
intellectual one but one that results from "some personal hurt" that makes the injured person
claim atheism in order to have an excuse to oppose God. X cited Madalyn Murray O'Hair, B.
F. Skinner, Hugh Hefner, Joseph Stalin, and Adolph Hitler as examples. He doesn't even
know how to spell O'Hair's first and last names, yet he expects us to believe that he knows her
atheism is the result of some personal hurt she experienced that caused her to rebel against
her Presbyterian upbringing. What is his proof? He cited none; he simply asserted it and went

Volume 1990 - 2002 Issue


Page 471 of 2049
Skeptical Review Edited by Farrell Till
on. Apparently, he is so used to the fundamentalist approach to argumentation that he can't
even recognize when evidence is needed.

I wouldn't even try to estimate the number of times someone has asked me, "What terrible
tragedy happened to you that turned you against God?" or words to that effect. These are
people who have no conception of what critical thinking is, so because they are logically
illiterate, they assume that those who reject the religious doctrines they were taught in their
childhood are people who have experienced some kind of personal "hurt" or tragedy. They
reject outright the possibility that the many who reject their early religious training do so
simply because they have critically examined religion and found it too ridiculous to continue
believing.

As I look back on my own life, I can see that I have been rather fortunate. All of my children
were born healthy and have remained healthy, so I have never experienced the tragedy (as
many have) of having a child die or suffer from some permanently crippling or debilitating
disease or injury. Now I have grandchildren, and they too are all normal and healthy. My
father did die suddenly at a relatively young age, but when this happened, I had already made
my decision to reject my early religious beliefs. So just what is this great "hurt" that I
presumably suffered that caused me to "turn against God"? Maybe Mr. X can enlighten me.

If Mr. Dobbs answers X's letter, maybe he will set X straight in this matter. It wasn't any great
"hurt" that I experienced that turned me against religion. Why, no, nothing as innocuous as
that. When I was a missionary, I got caught stealing church funds, was called home, and was
fired. I was so angered that I turned against the church. This is the scoop on me according to
Dobbs, and he seems to get a lot of "Christian" pleasure from making this accusation.
Another preacher, who was in diapers when I was a missionary, has yet another explanation
for my fall from grace. I lost my financial support when I was a missionary and had to come
home. (At least he had the courtesy not to make me a thief.) In my anger, I just quit the church
entirely and made a career out of attacking it.

A lot of fundamentalists who didn't even know me 30 years ago nevertheless seem to know all
about why I abandoned my religious beliefs. Their pride just won't let them admit the
possibility that I made a rational decision, so they imagine all sorts of ignominious reasons
for my loss of faith. None of them, however, seems to have an explanation for why I sat on my
anger for 20 years before I took revenge by attacking God and the church that had hurt me so
badly.

I personally know nothing about the reasons why Madalyn Murray O'Hair, B. F. Skinner, and
the others on X's list abandoned their religious beliefs; I don't even know if all of them did
abandon their religious beliefs. I certainly don't think of Hugh Hefner as any great opponent
of religion. However, I do know that X made an incredible blunder by including Adolph Hitler
in his list. Hitler was a practicing Catholic, and there is no indication that he ever abandoned
his faith. So X made the same mistake that Time magazine made when it represented Nazism
as "an officially atheistic regime." When Joseph C. Sommer, vice president of the Humanist
Community of Central Ohio, called the error to the editors' attention, the following letter of
retraction was issued:

Volume 1990 - 2002 Issue


Page 472 of 2049
Skeptical Review Edited by Farrell Till
We appreciate your noting that we were mistaken in characterizing Nazism as an officially
atheistic regime. The National Socialists indeed did acknowledge a deity and, however anti-
Christian their tenets and methods, aimed to established the German supremacist faith they
called "positive Christianity." And too, as you point out, Hitler claimed life-long allegiance to
the Catholic religion. We have notified the appropriate staff so that they may correct our files
(quoted in The Human Quest, May-June 1994, p. 22, emphasis added).

The commonness of X's mistake in attributing religious opposition to Hitler speaks volumes
about the way that fundamentalists think. Hitler committed monstrous deeds; therefore, he
must have been an atheist, because no believer in God would have done the despicable things
that he did. Such thinking shows an incredible ignorance of religious history and a complete
inability to think rationally. If X has no better critical-thinking skills than to assume
automatically that all despots are opponents of religion, is it any wonder that he would falsely
assume that vocal opponents of religion are just people who have suffered "some personal
hurt"?

Mr. X impugned the motives of The Humanist for not agreeing to "publish [his] views." "I
wonder why?" he asks. The question implies that the editors of The Humanist are afraid to
allow his views to be heard, but if that is a reasonable assumption, then he would have to
agree that Mr. Dobbs's refusal of my offer to submit articles to the Firm Foundation means
that he is afraid to let his readers have access to my views. I have suggested many times that
Dobbs and I exchange articles in both TSR and the Firm Foundation, but, alas, he has never
responded. I wonder why! Perhaps Mr. X can tell us.

X said that he had recently conducted a series of lessons on "Religious and Moral Authority
in a Do-Your-Own-Thing Age" that would account for the views that I now profess. (Oh, gee,
and I missed them!) He seems to crave a little limelight, so I am going to give it to him. He is
now TSR's "resident fundamentalist." If he will favor us with articles that refute materials we
have published in The Skeptical Review, we will publish them. I suggest that he begin with a
refutation of my short article "No Bastards Allowed" (Spring 1994, pp. 7,12,16). If he will do
that, we will publish it simultaneously with our response. Or if he prefers to select another
article to use in refuting my "errors and misconceptions," we will give him his choice. Or if he
would like to submit an article that attempts a logical defense of the Bible inerrancy doctrine,
we will publish it with our response. He is the resident fundamentalist, so the choice is his.

I further urge X not to stop with articles in TSR. He seems to enjoy giving public lectures, so I
propose that he and I debate the Bible inerrancy issue in Fairfax, Virginia, to give all of the
people who attended his lectures the opportunity to see him refute the "errors and
misconceptions" of an atheistic editor. If he doesn't feel up to the task, perhaps he wouldn't
mind finding a qualified spokesman to represent the inerrancy view. I suggest that he contact
Mitchell and Dobbs and invite either or both of them to debate me in Fairfax. Since they have
done such a marvelous job of refuting my "errors and misconceptions" in The Skeptical
Review, he should be glad to have them as my opponents. Surely one of them won't mind
publicly humiliating me.

We will all anxiously wait to hear what our resident fundamentalist decides.

Volume 1990 - 2002 Issue


Page 473 of 2049
Skeptical Review Edited by Farrell Till

Dear Mr. Till:

Thank you for sending me the tapes. I have to admit it was a miserable experience watching,
and I had many flashbacks while watching, but watch it I did, and thank goodness for the fast
forward on the remote when it came to Dobbs' exaggerated rhetoric. As a child, I sat through
this type of rambling and ranting preaching by preachers of Dobbs' ilk on a regular basis. By
the way, what a perfect name for Dobbs, Buster; he sure is a Buster all right, and the red-neck
he is he could very well have been a Bubba. Both names are perfect for him.

I had no control over this type of exposure until I was a little older, but the damage had
already worked on me. I was a believer until I was older and away from home and had time to
think and reason, and ask why this, and why that.

Then I started reading, and at that time I admit I did not see any real harm in religion. Now I
see the irreparable harm to the human race and on so many different levels. I wonder if
humanity will ever climb out of this religious pit.

It took me years before I found Freedom From Religion, their books and their paper,
Freethought Today. I also found Truth Seeker, and this led me to your material, The Skeptical
Review, and I have found many others and have been reading ever since.

Hovind is a mystery to me, as he is supposedly an educated man or so he bragged. He did not


sound educated. Are the brains of this type of man set in concrete?

Goodness, this was just going to be a very brief note to say thank you and get your tapes back
as soon as possible so that other people will be able to view them. THANK YOU.

(Kova Lancaster, 2468 Baywood Court, Orange Park, FL 32065-6253.)

P.S. I was so busy being irritated with Buster that I forgot to tell you what a grand job you did
by knocking the props out from under all of Buster's arguments. He couldn't answer you, so
he went off on those preaching tangents, as if he had a tape in his head that kept going off.
Your answers were brief, concise, and to the point. You were superb. Keep up the good work.
Count me in your audience that appreciates you. Only one little flaw--try to remember not to
turn away from your mike, as I don't want to miss anything you have to say!

EDITOR'S NOTE: I asked Ms. Lancaster for permission to publish her letter not because of
her compliments (which I appreciate) but because of her comments about the harm that
religion does. Long-time readers will recall that I have said the same: religion is harmful to
society. I know this to be true not from just my own personal experience but from studying the
history of persecution and inquisition that religion has left in its wake. Ms. Lancaster is just
one of many who have told me about the psychological harm Bible fundamentalism inflicted
on them in their youth. I realize that religion does do some things that are beneficial to
society, but, as H. L. Mencken said, the bad far outweighs the good. This is so obviously true

Volume 1990 - 2002 Issue


Page 474 of 2049
Skeptical Review Edited by Farrell Till
to those willing to look objectively at the history of religion that additional comment isn't even
necessary. A corroborative view is stated after the next letter.

We have just finished reading a copy of your publication The Skeptical Review, given us by
Kova Lancaster of Orange Park, FL, and find it makes more sense than most. We will
appreciate your including us on your mailing list.

(Beth Perry, 7926 Praver Drive West, Jacksonville, FL 32217.)

I would like to receive The Skeptical Review as offered in the Freedom From Religion
Foundation paper for November 1993....

Thank you very much for this favor. I will consider subscribing, of course. I am very much
interested in this kind of material after spending 40 years in a dogmatic religion but am now
liberated by reading history, philosophy, and religious history.

(Ruth C. Riales, 1205 Forest Circle, Altemonte Springs, FL 32714.)

The next issue of the review, which I hope I will receive in a few weeks, will end my free
annual subscription to it. Although I think you pay too much attention to the Old Testament
and the review seems intended to fight fundamentalism, I find informations and comments
which are useful to a Roman Catholic like myself who likes studying atheism and compared
history of religions.

Therefore, I am enclosing a cheque of US $12 to pay for my subscription for two further years
when the present free subscription ends. Many thanks for these first issues and kind regards.

(Antonio Casa Ibanez, Apartado 882, 50080 Zaragoza, Spain.)

I might have a beer and pizza party with my non-Christian/Christian friends and show the
resurrection tape. Geisler is their hero. You murdered him.

And the two tapes on prophecy fulfillment--does Buster Dobbs really get away with that kind
of ignorance and issue evasion? Are there no members of the C of C left with half a brain to
reel that airhead back to earth? You did good. You stuck to your questions like glue, didn't let
him slip out of them.

(Mark Smith, P. O. Box 3065-192, Garden Grove, CA 92642.)

Volume 1990 - 2002 Issue


Page 475 of 2049
Skeptical Review Edited by Farrell Till
EDITOR'S NOTE: Mark Smith, whose article "Evidence That Doesn't Demand a Verdict"
appeared in the summer issue of TSR, is a former member of the Church of Christ and
therefore additional proof that Bible fundamentalists do occasionally come to their senses--
not very often, admittedly, but sometimes they do.

Thanks for the tape [Geisler-Till Debate ]. You presented several cogent and salient lines of
argument. In total, I believe you did quite well. Still, you failed to note (as did Dr. Geisler)
some categorical differences between the seeming syncretisms of Christianity and other
ancient "virgin savior" based belief systems. Like Mormonism, their followers didn't die for
their assertions (as opposed to renouncing).

(Ross E. Nunes, 1805 Broadway, Pekin, IL 61554.)

EDITOR'S NOTE: We have never claimed that the "Jesus story" is exactly parallel to the life
of any one of the many pagan savior-gods who were worshiped before Jesus was allegedly
born. We do, however, claim that there are many striking parallels in the gospel stories and
the myths of those pagan gods. Our position is that the parallels are too striking and too
numerous for rational people to dismiss them as mere coincidences. After all, if saviors were
born of virgins, then they were born of virgins, and the exact manner of conception, whether
by any of the various ways that the pagan saviors were conceived or by the "Holy Ghost"
coming upon and "overshadowing" the Virgin Mary (Luke 1:35 ) is incidental. The fact that
virgin births were, as I said in my debate with Geisler, "a dime a dozen" at the time Jesus was
allegedly born of a virgin is sufficient to make any rational person suspicious of the claim. If
Mr. Nunes knows of "categorical differences" that would make it reasonable to believe that
the virgin birth of Jesus really happened whereas the virgin births of the pagan savior-gods
were merely myths, we would certainly be interested in hearing what they are.

More puzzling than any of this, however, is Mr. Nunes's final sentence. Followers of the other
pagan saviors weren't willing to die for their beliefs? I suggest that Mr. Nunes bone up a little
on religious history. Religious martyrs existed long before Christianity came on the scene. As
for his reference to Mormons as proof that other religious adherents "didn't die for their
assertions," he couldn't have picked a worse example. Living where he does, only twenty
miles from my home, he should be more familiar with Mormon history than he is. Nauvoo and
Carthage, Illinois, are located just a few miles west of here in the region where the Mormons
came to escape the persecutions they had experienced in the Eastern states. The peace they
were looking for wasn't found there, because Joseph Smith was taken by mob force from the
Carthage jail and lynched. Later, the Mormons moved on, going first to Western Missouri
and then eventually settling in Utah. The facts of Mormon history simply do not support Mr.
Nunes's claim, and if he will, as I suggested, review religious history, he will find that people
were dying for what they believed long before the Christian era began. It is ridiculous to try
to defend the basic claims of Christianity on the grounds that the apostles and early
Christians were willing to die for their beliefs. Martyrdom was not only not unique to
Christianity but its scope has been greatly exaggerated. The violent ways in which most of the
apostles allegedly died are largely matters of unsubstantiated tradition, and the extent of
Christian persecutions in general has been misrepresented in Christian propaganda. What

Volume 1990 - 2002 Issue


Page 476 of 2049
Skeptical Review Edited by Farrell Till
Christians suffered hardly compares to the persecutions inflicted by Christians once they
enjoyed majority status. If, then, my arguments in the Geisler-Till Debate were "cogent and
salient," Mr. Nunes would do well to consider them and stop grasping at any straw he can
find to try to defend a ridiculous belief system.

Volume 1990 - 2002 Issue


Page 477 of 2049
Skeptical Review Edited by Farrell Till

The Skeptical Review


Volume Six - 1995
Farrell Till, editor

• Number 1 Volume Six, Issue One


• Number 2 Volume Six, Issue Two
• Number 3 Volume Six, Issue Three
• Number 4 Volume Six, Issue Four
• Number 5 Volume Six, Issue Five

Volume 1990 - 2002 Issue


Page 478 of 2049
Skeptical Review Edited by Farrell Till

The Skeptical Review


Volume Six, Number One - 1995
Farrell Till, editor

• The Sound of Silence


The Skeptical Review has an editorial policy that grants equal space to inerrancy
believers who want to respond to our articles-- so why are more and more
fundamentalists running away from the opportunity to present their case?

• Mitchell Rides Again


After arbitrarily declaring victory and dropping out of the debate on the Amalekite
massacre, Lindell Mitchell is back, and he's whinier than ever.

• Those Amazing Biblical Numbers: Taking Stock of the Armies of Ancient Israel
An examination of biblical records relating to the size of ancient armies poses still
more problems for the inerrantists.

• What About Casualty Numbers?


Another article in the same vein as the above one, this time focussing on the number
of casualties instead of the number of soldiers.

• God Can't Be Tempted?


Can God be tempted? Depends on which verse of the Bible you happen to be reading
at the time.

• Religion and How I Lost It


The story of one man's journey from religion to reason.

• God is Omniscient?
Is God a know-it-all? If so, then he must know the reason why the Bible doesn't give a
coherent answer to that question.

• From the Mailbag

Volume 1990 - 2002 Issue


Page 479 of 2049
Skeptical Review Edited by Farrell Till

The Sound of Silence


An old Simon and Garfunkle song spoke of the sound of silence, a term that is becoming
more and more descriptive of the way that Christian fundamentalists are choosing to respond
to the evidence that disputes the Bible inerrancy doctrine. Long-time subscribers to The
Skeptical Review know that we have always had an editorial policy that grants equal space to
inerrancy believers who want to respond to our articles. In view of their dogmatic claim that
the Bible is the inerrant word of God, one would think that there would have been a mad rush
by fundamentalists to take advantage of our offer so that they could enlighten us silly skeptics
who reject the inerrancy doctrine, but instead they have consistently run from the opportunity
to present their evidence to the hundreds of atheists and skeptics who subscribe to our paper.

First, there was Wayne Jackson, editor of Christian Courier and co-editor of Reason and
Revelation, whose pro-inerrancy articles in these publications were often quoted, reviewed, or
reprinted simultaneously with our rebuttal articles. We always sent him advanced copies of
any such article with an offer of equal space if he wished to respond. All of these offers were
met with the sound of silence. He has the truth, and there isn't a chance in the world that his
position is wrong, yet he has no interest in using a forum that would give him an audience of
hundreds that would otherwise be unavailable to him. He prefers preaching to the choir in
journals that go primarily into homes that are already convinced that his inerrancy view is
right. That doesn't make a lot of sense.

Next came Lindell Mitchell, who tried his luck at vindicating God for ordering the massacre
of babies. We all remember what happened to him. After two articles, he unilaterally declared
victory and dropped out in a flurry of insults and assurances that he would not continue the
discussions ("From the Mailbag," Spring 1994, p. 15). Well, he is back in this issue with
letters that editor Farrell Till has responded to (see pages 2-4), which response Mitchell has
again declined the opportunity to reply to. In a letter to Till after receiving an advanced copy
of this article, he made it clear that his reaction will be only partial silence:

It is tempting to let you sink back in the cesspool of obscurity you deserve. However, because
other "free-thinkers" accept some of your absurd quibbles uncritically, I plan to deal with
them in the Firm Foundation occasionally. Your name and location will not be used, but you
will find it interesting (August 9, 1994).

See what we mean about preferring to preach to the choir? Mitchell is vitally concerned about
"free-thinkers" who uncritically accept Till's absurd quibbles, but he chooses to "deal with
them" in a religious journal that is read primarily by members of his own little faction rather
than in TSR, where the impeccable logic in his arguments would be seen by hundreds of
"uncritical freethinkers." Does he seriously believe that freethinkers are so "uncritical" that
they won't see right through a rationalization as transparent as this one? The man doesn't have
a leg to stand on, he knows that he doesn't, so he is desperately groping for some face-saving
way to get out of a situation he wishes he had never gotten into.

Volume 1990 - 2002 Issue


Page 480 of 2049
Skeptical Review Edited by Farrell Till
Viewers who have watched the tapes of the Moffitt-Till Debate saw Moffitt's moderator,
Marion Fox, assure the audience that he was going to write an article "in the next few
months" that would respond to the contradiction pointed out in "No Bastards Allowed" (TSR,
Spring 1994, pp. 7,12,16). This announcement came after Till had presented the problem
during the debate and challenged the bevy of preachers in the audience to give a satisfactory
explanation of the problem before the debate was over. Not a single preacher spoke to Till
about it, but at the beginning of the third session, Fox announced that he would write the
article and went on to say that there were some "good answers" to the problem. Although Till
spoke up from his seat and assured the audience that he would publish the article, as this issue
goes to press, nothing has been heard from Fox concerning these "good answers" to the
problem, even though more than just "the next few months" have gone by. So the sound of
silence continues here too.

In the same debate, Moffitt interrupted one of Till's speeches and announced that he would
debate the prophecy issue. An on-the-spot agreement was reached to have a written debate on
the subject, and the tapes show Moffitt assuring Till that he would arrange his work schedule
so that he could resume a written debate that he and Till already had in progress on the
inerrancy issue, finish it, and then begin the one on prophecy. At that time, it had been over
two years since Moffitt had last contributed a manuscript to the debate already in progress. As
this edition of TSR goes to press, six months after the oral debate, Moffitt still has not written
another manuscript for the inerrancy debate, so, needless to say, the prophecy debate is
nowhere close to starting. The sound of silence is beginning to scream in Portland, Texas.

So what is happening here? Why would Mitchell choose to expose Till's "absurd quibbles" for
the benefit of "uncritical freethinkers" but do so in a journal that few freethinkers will ever
read? Why would Fox announce to an audidience of fundamentalists that he would write an
article that would give some "good answers" to a contradiction Till had alleged and then not
write the article? Why would Moffitt stand up before the same audience and announce that he
would debate Till on the prophecy issue and then not do it? Well, logicians have an
expression that explains it. It is called "playing to the gallery," i.e., saying what the debater
knows will have a strong emotional appeal to his audience. When Mitchell publishes his
"exposure" of Till's "absurd quibbles" in a one-sided religious journal, he will be preaching to
the choir. His readers will lay the paper aside thinking, "Wow, Brother Mitchell sure ripped
that atheist to pieces," but they will never hear the evidence that supports Till's position.

The fundamentalists in the audience who saw Fox's and Moffitt's performances left the debate
thinking that their preachers were going to put another atheist in his place. Most of them will
never realize that their beloved preachers merely played to the gallery and then responded
with the sound of silence when it was time to make good their promises. Like Mitchell's Firm-
Foundation readers, they will never hear the other side of the issue or even know that their
men copped out.

So what are these inerrancy champions up to? They would never admit it, but it seems to be
what was, strangely enough, expressed in another old song. They want to "hang on to what
[they've] got." They know that their position is too untenable to persuade objective minds to
accept it, so they have adopted a holding-action strategy. They will take on an adversary now
and then in public debate but not to try to reach the undecided or the openly skeptical; they

Volume 1990 - 2002 Issue


Page 481 of 2049
Skeptical Review Edited by Farrell Till
will just play to the gallery and hope to hang on to what they've got. It's a free country, so
they may do as they please, but our mail suggests that their strategy isn't working. Little by
little, they're losing ground.

Mitchell Rides Again


Farrell Till
After arbitrarily declaring victory and dropping out of the debate on the Amalekite massacre
(Spring 1994, p. 15), Lindell Mitchell quickly fired off a letter to me after receiving the
Summer 1994 issue. I thought that his letter, which was addressed to Farrell Swill, deserves a
page or two of its own rather than just a column in "From the Mailbag":

I just received the most recent issue of your excuse for a journal. As always, it is filled with
distortion and deception. You know that [you?] did not present an affirmative case in our
debate. We both know that you are unable to make an affirmative case given your
epistemological posture. You looked even more silly than you normally do; and you were
exposed in your own paper! The fact that you have devoted so much space to our exchange
indicates how badly wounded you were.
I still want to see you produce objective quantifiable data, in the absence of an objective
standard of morality, which proves that God was guilty of a moral atrocity in 1 Samuel 15.
You did not do it; your buddy Dave Matson did not do it; no one can do it, because it is an
asinine position.
You ran Patrick Phillips [sic] whiney [sic] letter but did not carry my response, which you
surely had. Are you scared Mr. Till? What does his snotty letter prove? Did he produce the
data which would establish your position? I asked him to, but he has not seen fit to produce
the information required.
Your lead article is typical of the tripe you trot out to the public. You assert that Jesus is a
counterfeit and seek to establish your assertion by telling us that Martin A. Larson says that
there were many virgin-born savior gods. So? What does that prove? Where is the objective
data? Where is the measurable information that establishes the claim? If your claim were
granted how would that establish your assertion that Jesus is a fraud? Where do you sustain
your position in the absence of a standard?
Until you discover the concept of honesty and make some effort to keep your word, I will not
take you seriously.

Mitchell knows that I offered him an opportunity to respond to my third article (Summer
1994) on the Amalekite massacre. The article was sent to him in April to allow him time to
write a response before the printer's June 1st deadline, but he chose not to accept the offer.
Now that he has seen my article, he wants to reopen the discussion, although the letter
accompanying his last submission clearly said that he was "closing the debate on 1 Samuel 15
at this point" and that he "refuse[d] to continue the discussion" (Summer 1994, p. 15).
Anyone, of course, is entitled to change his mind, and now that Mitchell has reconsidered, I

Volume 1990 - 2002 Issue


Page 482 of 2049
Skeptical Review Edited by Farrell Till
am more than happy to publish his letter. There is absolutely nothing in it that I am "scared"
to let TSR readers see. On the other hand, I don't see the editors of the fundamentalist papers
that Mitchell writes for rushing to give skeptics the opportunity to express their opinions on
the pages of those journals. I wonder why.

Mitchell referred to TSR as my "excuse for a journal"; however, this "excuse for a journal"
does give a forum to opposing views, and that is more than the Firm Foundation does.
Mitchell is a regular contributor and a member of the advisory staff of this paper, so perhaps
he will be willing to advise my favorite editor Buster Dobbs to feature an exchange of views
in the pages of the Firm Foundation. If Dobbs is too proud to let me be the representative of
the biblical errancy view, I know others who would gladly do it. However, until the religious
papers that Mitchell is affiliated with show a willingness to publish both sides, he certainly
has no room to criticize the editorial policy of The Skeptical Review.

Mitchell asserted that "the most recent issue" [Summer 1994] was "filled with distortion and
deception"; however, he made no attempt to identify those distortions and deceptions except
for a brief reference to the lead article that I will comment on shortly. Although some TSR
readers may be perplexed by Mitchell's refusal to discuss specifics, his tactic is very familiar
to me. I have debated enough Bible fundamentalists to know that their tactics are based on
question begging and unsupported assertions. Mitchell has harped about my duty to "produce
objective quantifiable data," but he seems to think that he is under no such obligation, that he
can just assert anything he wishes and expect everyone to accept it on his mere word. The
mind of the Bible fundamentalist is indeed a marvel.

Mitchell complained that I "did not present an affirmative case in our debate," but I have to
wonder where he has been. His complaint that I cannot prove moral atrocity "in the absence
of an objective standard of morality" has been shown to be without merit. How many times
did I show that concepts like good, bad, right, wrong, sorrow, happiness, beauty, ugliness,
etc., etc., etc. are merely abstractions that the human mind has formulated from intellectual
experiences? How many times have I begged Mitchell, Lockwood, et al to tell us if legitimate
concepts of sorrow, happiness, or beauty can exist without the existence of gods of sorrow,
happiness, or beauty to issue objective standards by which these can be measured? This
argument has been put before them repeatedly, but they have made no attempt to respond to
it. So until they do logically refute it, we are justified in assuming that the same human
intellect that can conceptualize abstractions of beauty and sorrow can also conceptualize
abstractions of good and bad, right and wrong. If not, why not? Mitchell has never answered
that question.

In the "Editor's Note" affixed to Elizabeth Dyak's letter (Summer 1994, p. 13), I pointed out
that Mitchell admitted in his first article that "(n)o one disputes" that the Amalekite massacre
was a "terrible calamity" (Winter 1994, p. 4). Note that Mitchell didn't admit that it was just a
"calamity"; he said that it was a "terrible calamity" and that "(n)o one disputes that fact."
Earlier in the same paragraph, he referred to it as an "enormous tragedy," which was a graphic
illustration of the "terrible cost paid by a society that persisted in rebellion against God"
(emphasis added). In this paragraph, the abstractions were running wild in Mitchell's article.
Tragedy, calamity, terrible--these are all abstract concepts, so what I want to know is how
Mitchell arrived at these descriptive conclusions about the Amalekite massacre in the absence

Volume 1990 - 2002 Issue


Page 483 of 2049
Skeptical Review Edited by Farrell Till
of objective standards of tragedy, calamity, and terribleness against which to measure the
events of the Amalekite massacre? If somehow he was able to determine that it was a
calamity, when he has no objective standard of calamity to use as a measuring instrument,
then why can't I or any other intelligent person determine that it was an atrocity, even though
we may not have an objective standard of atrocity (morality) to use in evaluating the act?
Before Mitchell talks about silliness, he should first take a good long look at the absurdity of
his own position.

If Mitchell wants to respond to this article, I will give him the opportunity, only this time the
offer is being extended with a condition attached. To have his article published, he will have
to address the subject of the preceding paragraph. In other words, he will have to tell us how
he is able to determine tragedy and calamity in the absence of objective standards of tragedy
and calamity. Then he will have to explain why human concepts of good or bad, right or
wrong, are invalid, whereas his concepts of tragedy and calamity are valid. If he is willing to
go this far, then he shouldn't mind addressing another issue he has evaded. He should explain
to us how his objective standard of morality [the Bible] can be used to find solutions to the
many moral dilemmas that our advancing technology is creating. If he isn't willing to confront
these issues, then his ideas deserve no further consideration.

Mitchell seems to believe that Martin A. Larson is the only authority I could quote to prove
that virginborn savior-gods flourished long before Jesus allegedly lived. In reality, Larson was
just one source of many whom I could have quoted. The fact that virgin-born, miracle-
working, resurrected savior-gods were widely worshiped in pagan religions long before the
time of Jesus is so commonly known in scholarly circles that I have to wonder what planet
Mitchell has been living on if he doesn't know this rather elementary fact. Has he assumed
that a "country preacher" has no need to acquaint himself with religious history? Apparently
so. I suggest that Mitchell make a trip to his local library and check out a book on ancient
mythology or go to the card catalog and look for books by Joseph Campbell. In the general
encyclopedias, he can look up specific names like Osiris, Dionysus, Perseus, Mithras, Apollo,
Tammuz, and Krishna. If he is serious about it, he will have no trouble at all finding materials
on the virgin-born savior-gods whom pagans worshiped long before Christianity made its
appearance on the religious scene.

If he wants specific references that prove the mythological background of the Jesus story, he
might try Mythology's Last Gods: Yahweh and Jesus by William Harwood, Jesus: God, Man,
or Myth? by Herbert Cutner, The Myth of the Resurrection by Joseph McCabe, How Did
Christianity Really Begin? by Howard Teeple, Bible Myths and Their Parallels in Other
Religions by T. W. Doane, and The Diegesis by the Reverend Robert Taylor.

He will find plenty of material in these books to open his eyes to the real origins of
Christianity, and when he has finished with these sources, I have plenty more to recommend
to him. Mitchell might be surprised at what he could learn if he should ever open his mind
enough to permit objective investigation.

I have to wonder why Mitchell focused on Martin A. Larson and ignored the other references
mentioned in this article. Did he not notice the quotation from Justin Martyr, the early church
father who tried to convince the emperor Hadrian to believe in Jesus on the grounds that

Volume 1990 - 2002 Issue


Page 484 of 2049
Skeptical Review Edited by Farrell Till
virgin-born sons of the gods were commonplace in the pagan religions? In view of the
widespread acceptance of these pagan saviors, Justin wondered why anyone should consider
it unreasonable to believe in just one more virgin-born savior-god. Did Justin Martyr simply
make this up or was it indeed a fact that faith in virgin-born sons of the gods was very
common in his time?

The same article referred to Ezekiel's reference to the women at the gate of Jehovah's house
weeping for Tammuz (Ez. 8:14), but Mitchell's letter didn't refer to this. Why? Since this is a
biblical reference to the worship of a Babylonian savior-god, shouldn't it qualify as "objective
data"? Or does Mitchell now want to argue that the Bible doesn't represent objective truth?

Mitchell wanted to know how the granting of my claim that virgin-born pagan saviors were
worshiped long before Jesus would "establish [my] assertion that Jesus is a fraud"? Is he
serious? Does he really expect us to believe that he can't see the damage this would do to the
claim that Jesus was the virgin-born son of God? The matter is as simple as what I said on this
subject in an earlier issue of TSR: "(W)hen the study of ancient world literature shows that an
unusual event... seemed to have happened everywhere, reasonable people will realize that it
probably happened nowhere" ("Prophecy Fulfillment and Probability," Autumn 1993, p. 9).
So the problem with Mr. Mitchell's Jesus is that he was just a Johnny-come-lately, who came
along much too late for people informed in ancient mythology to believe in him. Mitchell
complained that I did not publish his answer to Patrick Phillips' "snotty letter," even though I
surely had had his response in hand at the time. The truth is that I did not receive a copy of
Mitchell's response to Phillips until after the Summer 1994 issue was ready for press. I had set
up this issue early so that I could take it to the printer's before I left for my debate in Texas
with Jerry Moffitt. To include Mitchell's letter, I would have had to pull some of the material
to make space for it, and, as any editor knows, that takes time, which I didn't have to spare,
especially for something no more substantial than the content of Mitchell's response to
Phillips. However, to prove that I am not trying to evade anything, I'm going to publish
Mitchell's letter. Here it is:

Your tender loving letter arrived today. It was most touching and veritably dripping with
gentle loving kindness. Reading it made me feel so loved! Your concern for me was
overwhelming.
You whined a lot about how good you are and how bad the Lord's church is. You asserted that
there are saved people who are not a part of Christs' [sic] church, but you didn't offer any
evidence supporting that assertion.
It appears that one of your biggest mistakes was attending ACU [Abilene Christian
University]. They have produced confused misshapen people for some time. I regret that you
have been thus tainted.
I could not help but notice your failure to produce any empirical data supporting traitor Till's
assertions. Till agreed to offer an affirmative case proving God guilty of moral atrocity in 1
Samuel 15. I obligated myself to deny his assertion. We agreed to have two exchanges of
3000 words each. It was also agreed that we would narrow the discussion to this topic. Till
did not keep his word (he never does), nor did he offer any affirmative case. Instead he spent
his time trying to muddy the water by posing perplexing ethical questions having no bearing
on the events of 1 Samuel 15. He tried to force me into the affirmative where he could take

Volume 1990 - 2002 Issue


Page 485 of 2049
Skeptical Review Edited by Farrell Till
pot-shots, which I refused to do. I insisted on seeing his objective evidence for his assertion,
and I still insist that he produce it.
I pressed Mr. Till to make his case in a manner consistent with his philosophical construct. He
made no attempt to do so. You should be able to see that. If he had been able to make his case
he wouldn't have hid [sic] behind a smoke-screen. I'm still waiting for him to produce one
shred of empirical evidence supporting his position. Since you are so impressed with Till's
position perhaps you will take up the challenge and provide the data that Till could not
produce?
The point demonstrated in the exchange is that Till's epistemology is flawed. The issue is
meta-physical [sic], but he refuses to admit the existence of anything beyond the material
universe. Yet he can't consistently follow his own philosophy. Further, asking perplexing
ethical questions does not establish his case. I would have to be god to answer all questions.
The fact that I can't answer a question does not mean that there is no answer, nor do such
questions establish Till's case. Questions do not establish anything. Please enlighten me
concerning how asking questions about things not remotely related to God's actions in 1
Samuel 15 somehow strengthens Till's case.
Till uses this ploy to cover his ineptitude. In fact, the questions Till posed can be dealt with,
but we were not debating those issues. We agreed to debate the propriety of God's actions in 1
Samuel 15, which he did not touch. Why? He didn't touch it because he could not. If you
think my response is weak and inept surely you can produce the data proving that God is
guilty of moral atrocity in ordering the destruction of the Amalekites in 1 Samuel 15. All that
I've seen from you thus far is condescending arrogance. Please produce the empirical
evidence for Till's case. If you can do that I'll surrender my sword and retreat from the field.
However, the drivel you have produced thus far does not move me.
I hope that you do not continue to debase yourself by pandering to traitors like Mr. Till. I pray
that you will cease your rebellion and return to the Lord's church. You now occupy the
position of a traitor to the Christian cause. Nothing could be more shameful. We see the
fundamentalist mind at its strangest in Mitchell's response to Phillips. In the first paragraph,
Mitchell began by criticizing the tone of Phillips' letter, but he did this far more sarcastically
than anything Phillips had said. If writing a letter that can be sarcastically described as "most
touching" and "veritably dripping with gentle loving kindness" is wrong, then what makes
Mitchell believe that he can write the same kind of letter and not commit the same offense for
which he faulted Phillips?

Many readers will not understand why Mitchell believes that Phillips had made a grievous
mistake in attending Abilene Christian University. Mitchell's church is being greatly troubled
today by a widespread trend to mollify its hardline doctrinal position. At one time, all
members of this church arrogantly believed that they were the only true Christians on earth.
All others--Baptists, Pentecostals, Methodists, Lutherans, Congregationalists, Adventists,
Presbyterians, Episcopalians, and you name it--were all hell-bound. Even those in the First
Christian Church, who were doctrinally similiar except that they worshiped with instrumental
music rather than just a cappella singing, were going to hell too. In recent years, however,
many Church-of-Christ members have begun to question the narrowness of their position and
have suggested that there may be saved Christians in other denominations too. Some
professors at Abilene Christian University have taken the liberal view in this controversy, so
Mitchell, a preacher in the guardian-of-the-faith wing of the church, quite naturally thinks that
Phillips first went astray when he enrolled at ACU. To TSR readers who have little or no

Volume 1990 - 2002 Issue


Page 486 of 2049
Skeptical Review Edited by Farrell Till
familiarity with the doctrinal silliness of the Church of Christ, I will simply say that you
should not try to understand the workings of a mind like Mitchell's. I am just trying to explain
the meaning of some of the things he said in his letter.

In his letter to Phillips, Mitchell claimed that the "perplexing ethical questions" I had asked in
my articles did not "establish anything." However, I beg to differ with him. The only way that
he or anyone can justify the Yahwistic atrocities of the Old Testament is to argue that there is
an objective morality that emanates from the perfect nature of God, who has revealed that
objective morality to us in his word so that we will know how to live moral lives. Since the
perfect nature of God is the source of objective morality, it would be impossible for God to do
anything wrong. All such talk, however, is speculative nonsense. He harps about my duty to
produce "objective quantifiable data" to prove that the Amalekite massacre was a real
atrocity, but what about his duty to produce objective, quantifiable data to prove the major
negative argument (affirmation) that he has resorted to in the debate? All he has done is assert
and then run on to the next assertion. He likes to talk about "objective quantifiable data," so
let's talk about it. Let's see his objective, quantifiable data that proves his god Yahweh even
exists. If he doesn't exist, then there can be no objective standard of morality emanating from
the perfect character of this deity. So what is Mitchell's objective, quantifiable data that
establishes unequivocally that an objective standard of morality exists? What is his objective,
quantifiable data that will prove the Bible is where that objective standard of morality has
been recorded? Where? Where? If he has such data, let's see it. If he doesn't, then case
dismissed. The matter is that simple, but he just can't see it.

As for the relevance of my "perplexing ethical questions," I was simply trying to get through
his thick skull to show him the absurdity of his belief that the Bible is our guide to objective
morality. No two people can read the Bible and see it alike. That can't even be done in the
Church of Christ, which claims to be the only true church. This church has split into so many
warring factions that it has become a joke, and each faction has resulted from inability of the
general membership to agree on the meaning of some verse or two of scripture. Outside of the
Church of Christ, Christians are almost as badly divided. There are Baptists, Methodists,
Catholics, Presbyterians, Pentecostals, etc., etc., etc., and within the denominational
groupings, other divisions are very much in evidence. There are United Methodists, Free
Methodists, Episcopal Methodists, and there are Southern Baptists, American Baptists,
Independent Baptists, Missionary Baptists, Bible Baptists, Free Will Baptists, etc., etc., etc.
All these divisions exist because the members of these denominations can't agree on biblical
teachings. Yet they want us to believe that God revealed to us in the Bible an objective
standard of morality. This is the absurdity that I have been trying to get Mitchell and
Lockwood to see. If God did indeed reveal to us an objective standard of morality in the
Bible, why wouldn't he have done so in a manner so clear that we could all understand it the
same way? The purpose of my "perplexing ethical questions" was simply to demonstrate the
absurdity of believing that a book written many centuries ago contains an infallible guide that
can solve all of humanity's moral problems. If such a moral code exists, why can't Mitchell
refer to it to answer my questions? He claimed in his letter to Phillips that my questions "can
be dealt with," so why doesn't he deal with them and humiliate me even further than he seems
to think he already has? How long does he expect rational people to buy his claim that he
didn't answer the questions because "we were not debating those issues." In truth, we were
debating "those issues," because we were debating a topic concerned with moral conduct. If

Volume 1990 - 2002 Issue


Page 487 of 2049
Skeptical Review Edited by Farrell Till
someone is going to argue that killing babies nursing their mothers' breasts isn't morally
wrong because of some obscure notion dependent on the existence of "objective morality,"
then he has an obligation to demonstrate that objective morality does in fact exist. Mitchell
has evaded every demand that he address this issue. Who's Really Scared?

Mitchell insinuated in his letter to me that I did not publish his response to Phillips because I
was scared, so let's take a look at the situation to see just who is really scared. I have
challenged Mitchell to an oral debate on the issue of Bible morality or any subject related to
Bible inerrancy, but he has refused the challenge. He doesn't want to humiliate me publicly.
Yeah, right!

Besides Mitchell, I have challenged many other preachers in his church known for their
debating skills, and only four of them have accepted. I debated all four of them, and have tried
to negotiate return engagements with all of them. Only one has accepted.

In addition to these Church-of-Christ preachers, I have debated Kent Hovind, a Baptist who
lectures nationally on "creation science," and Dr. Norman L. Geisler, whose reputation as an
inerrancy spokesman crosses denominational lines. I have challenged both Gleason Archer
and Josh McDowell to debate, and they have both refused. Does this debating record sound as
if I would be afraid of someone like Lindell Mitchell? To put to rest any notion Mitchell
might have that I am afraid of him, I will renew my debate challenge. If he will agree to
debate the issue of Bible morality or any topic related to Bible inerrancy or the existence of
God, I will gladly oppose him. The moment he sends me an acceptance of this challenge, we
can begin negotiating the time and place.

If he refuses this challenge, we will then have a pretty good idea just who is afraid to defend
his beliefs publicly.

Those Amazing Biblical Numbers:


Taking Stock of the Armies of Ancient Israel
William Sierichs, Jr.

The survival of ancient Israel must have often been in doubt, since armies numbering in the
hundreds of thousands repeatedly attacked the nascent state. The Israelites once stood off an
Egyptian army of a million. Miraculously, for several centuries, Israel survived attacks by
armies larger than those commanded by Napoleon, Alexander the Great, Genghis Khan, the
generals of the American Civil War, and even the massive forces of Prussia and France in
1870.

Let's look more closely at this miracle. We'll start by putting it in a historical context.
According to Herodotus, Xerxes' Persian army numbered 1.7 million when it invaded Greece

Volume 1990 - 2002 Issue


Page 488 of 2049
Skeptical Review Edited by Farrell Till
(The Persian Wars, Book VII, Section 60), but no reputable historian accepts this figure. The
Persians could not possibly have supplied such a horde, given the transportation and food
handling technology of the day. For instance, Donald W. Engels calculates that Alexander's
army of 65,000 personnel of all types needed, at a minimum, 1,500 pack animals--and as
many as 8,400 whenever it had to cross a dry or desert area--just to carry one day's supplies
(Alexander the Great and the Logistics of the Macedonian Army, p. 19). That figure increases
to 50,400 pack animals for a four-day march in a desert. In a fertile area, Alexander still
needed 40,350 pack animals at a minimum to carry his supplies for 10 days of marching
(Ibid.). Thus the more generous historians cut the figure for Xerxes' army to a tenth or about
180,000 troops. More skeptical historians think that even this figure is much too high and cut
it to 100,000 or so.

Some historians suggest that Herodotus misunderstood his sources and counted the entire
military muster of the Persian Empire. For an empire that controlled a region stretching from
Western India and South Central Russia, across Pakistan, Afghanistan, Iran, Iraq, Syria,
Lebanon, Israel, down into Egypt, back up to Anatolia and into what used to be Yugoslavia,
that figure strikes historians as about right.

Certainly, when Alexander the Great invaded the Persian Empire, he faced huge Persian
armies at Issus and Gaugamela, forces possibly as high as 100,000 or even as many as
200,000 troops--but these figures are much debated. Alexander, who controlled Greece,
Macedonia, Thrace (Southern Yugoslavia), and a little bit of Western Anatolia, was able to
raise between 90,000 and 100,000 troops total, with about half remaining in Macedonia when
he invaded the Persian Empire.

Epic battles like these, however, were infrequent in ancient history. More common were
fights like Marathon, where about 10,000 Greeks met a slightly larger Persian force in 490 B.
C. E. or the 13th century B. C. E. campaign in which the Hittite king Tudhaliyas IV faced an
army of 10,000 infantrymen and 600 chariots in Western Anatolia.

Even more common were raids between tribes of cities, involving a few hundred, maybe a
few thousand. Xenophon's A History of My Times records numerous instances of these small-
scale campaigns: 1,000 emancipated Spartan helots and 4,000 other troops from the
Peloponnese, aided by 300 mercenary cavalrymen; 300 light infantrymen, 200 cavalrymen,
and 200 Greek hoplites raiding an area; 800 light infantrymen and a comparable force of
hoplites ambushing another force; etc.

Great battles with hundreds of thousands of combatants, as we think of them, did not become
possible until the 19th century and the development of railroads. In the 18th century,
Frederick the Great had an army of 83,000 troops when he became king of Prussia. Other
states--Austria, France, and Russia-- fielded larger armies, but rarely did they approach
100,000 troops. Frederick's greatest victories--Rossbach and Leuthen--involved about 75,000
and 115,000 troops respectively on both sides. In the American Revolution, battles rarely
exceeded 10,000 combatants total and were usually far fewer in number.

Napoleon's greatest victory--Austerlitz--involved about 150,000 troops total. So did


Gettysburg, America's greatest Civil War battle. Napoleon raised 500,000 troops for his

Volume 1990 - 2002 Issue


Page 489 of 2049
Skeptical Review Edited by Farrell Till
invasion of Russia--scattered in columns up and down the Russian border for hundreds of
miles. Supply problems took many lives. The Union kept about 500,000 troops at arms in the
Civil War, supplied by railroads and steamships and scattered across half the nation, so no
one area had to support such a horde.

The exception to all of these historical military events is ancient Israel, according to Jewish
scripture. For example, in 2 Chronicles 14:9, Zerah the Ethiopian brought one million men
and 300 chariots against King Asa of Judah (908-868 B. C. E.). The ten tribes of Israel had
earlier split from Judah, so Asa commanded only 300,000 warriors from the tribe of Judah
and 280,000 from the tribe of Benjamin (2 Chron. 14:8). Nevertheless, we are told that Asa
defeated the Ethiopians and killed "so many that they could not recover themselves" (v:13). It
helped, of course, that Asa cried unto "Jehovah his God" before the battle (v:11), and, quite
expectedly, "Jehovah smote the Ethiopians before Asa" (v:12). It always helps an army to
have an omniscient, omnipotent war-god on its side.

Earlier, King Shishak of Egypt (945-924 B. C. E.) had also attacked Judah with 1200 chariots,
60,000 cavalrymen, and "infantry without number" (2 Chron. 12:3) in the time of King
David's grandson, Rehoboam (926-910). Shishak carried off a lot of loot from many cities in
both Israel and Judah. He left a list of his trophies in Egypt.

These battles must have been spectacular, like scenes from a Cecil B. DeMille movie, because
Rehoboam had 180,000 troops from the tribes of Judah and Benjamin (1 Kings 12:21) to
defend the land of Judah. Rehoboam might have defeated Shishak if he had had the army of
his forefathers, but, unfortunately for him, the kingdom had split in a civil war at King
Solomon's death. According to a census of men over 20 ordered by David, his army was
huge--800,000 warriors in Israel and 500,000 in Judah, or 1.3 million "mighty men of valor"
(2 Sam. 24:9), unless we want to believe a record of the same census that put the total number
of David's army at over 1.57 million (1 Chron. 21:5).

Nothing in scripture suggests that King Solomon's army was weaker than David's, although 2
Samuel 24:15 does note that 70,000 Israelites died in a plague right after David's census.
Solomon himself had 40,000 stalls for his horses and 12,000 cavalrymen (1 Kings 4:26). For
clarification, chariots of that period commonly were pulled by two horses, with a third horse
kept as a "spare." Thus, Solomon must have had about 9,300 chariots--28,000 horses--plus his
cavalry, although 1 Kings 10:26 says he had only 1,400 chariots but 17,000 cavalrymen.

The mention of Egyptian and Israelite cavalries is a little strange, inasmuch as cavalries seem
to have originated in the Russian steppes and moved south into the Middle East around the
end of the 10th century (John Keegan and Richard Holmes, Soldiers: a History of Men in
Battle, pp. 79-80). The Assyrians almost certainly had cavalry before the Israelites and the
Egyptians, and the first mention of cavalry in Assyrian annals is in the 9th century. [Before
that time, boys or young men rode horses as messengers, but horses of the Bronze and Early
Iron Ages generally were too small to carry fighting men. Bigger horses came from the north
(Ibid.).] You would almost think that the chroniclers of Israel introduced an anachronism into
their histories, but that couldn't be, because Yahweh himself wrote these stories. Right?

Volume 1990 - 2002 Issue


Page 490 of 2049
Skeptical Review Edited by Farrell Till
Anyway, the Israelite army was also tough. David's chief of the captains, Josheb-bas-sheboth
killed 800 men in a battle (2 Sam. 23:8). Another commander, Abishai, killed 300 men in a
fight (2 Sam. 23:18), and Jashobeam, another "mighty man" of David, killed 300 in one battle
(1 Chron. 11:11). By comparison, I don't think Conan the Barbarian ever killed more than a
few dozen men in any battle, according to the chronicles of his deeds by Robert Howard, but,
of course, Conan worshiped Crom, not Yahweh, so the paltriness of his feats is
understandable.

The civil war that divided the kingdom really hurt Israel, because Rehoboam could muster
only 180,000 warriors in the late 10th century (compared to David's 1.3 or 1.57 million). The
Israelites were fast breeders, however, because a few decades later, 2 Chronicles 17:14-18
tells us that King Jehoshaphat (868-847 B. C. E.) had five commanders with a total of 1.16
million troops from Judah and Benjamin. There must have been a plague, though, because by
2 Chronicles 25:5, Judah and Benjamin could raise only 300,000 warriors over the age of 20,
and King Amaziah (801-773?) had to hire 100,000 mercenaries from his neighbor Israel. By 2
Chronicles 26:12-13, however, King Uzziah (Amaziah's son, 787-737 B. C. E.) was back up
to 310,000 troops. [Reading Chronicles is like following the stock market: the Israelite army
is up 100,000 today, down 200,000 tomorrow, up 120,00 by the end of the month.]

The state of Judah fell on hard times in 2 Chronicles 28:5-8, because first the Syrians killed
many men, which may have been revenge for that time in 1 Kings 20:30 when the Israelites
killed 100,000 Syrians in a battle, after which the surviving Syrians retreated into the city of
Aphek, where a wall fell and killed 27,000 more. (This wasn't the Great Wall of China, was
it?) Anyway, back to 2 Chronicles 28. Israel next invaded Judah and killed 120,000 troops
and carried off 200,000 civilian captives, who later were generously released. These captive
figures sound suspiciously inflated. When the Babylonians under Nebuchadnezzar carried off
the Israelites (2 Kings 24:14-16), he got only 10,000 captives--all the "men of valor," princes,
craftsmen and smiths. Of course, he didn't get the poorest people, but then they weren't
"mighty men of valor." So the Babylonians must have slaughtered the bulk of the Judean
army, several hundred thousand perhaps. Evidently, the Babylonians were even mightier men
than the Israelites. [On the Israelite army stock market, this was the Great Depression.]

Captivity must have had a debilitating effect on the Israelites. Although they had bred
gigantic armies earlier, those 10,000 Israelite captives in Babylon multiplied to only 42,360
people (Ezra 2:64) by the time of their return to Judah seventy years later. Of course, they
also had 7,337 servants and 200 singers.

Let's contrast that with the Exodus population of 603,550 tribesmen over 20 years of age
(Num. 1:46). The Levite males one month and older, who were counted separately, totaled
22,300, if the separate figures in Numbers 3:22,28,34 are added, but 22,000 according to
Numbers 3:39. God should have invented the calculator 3,000 years earlier. Of course,
women and children probably tripled or even quadrupled these figures, and this was after a
long period of brutal slavery and the Egyptian slaughter of the Hebrew male infants.
Considering that it had all begun from only 76 "souls" that went into Egypt with Jacob (Gen.
46:26), the Israelite exodus population should have been much smaller. Apparently, the
Israelites really knew how to be fruitful and multiply in those days but not centuries later
when they really needed a large population. [By the way, the Egyptian massacre of Hebrew

Volume 1990 - 2002 Issue


Page 491 of 2049
Skeptical Review Edited by Farrell Till
male infants occurred only after two Israelite midwives refused to kill the babies themselves
(Ex. 1:15-17). How could only two midwives have served a body of pregnant women giving
birth fast enough to have produced such a large exodus population?]

The prenational Israelites needed all those fighting men. When the Midianites, a nomadic
tribe living in the deserts of Jordan--which today support a Bedouin population numbering
only in the tens of thousands--invaded Israel, their army was so large that they could lose
120,000 troops to the Israelite defense and still have 15,000 left (Judges 8:10). And the
wasteland of Moab produced a large army, which lost 10,000 in a battle (Judges 3:29). On
Israel's behalf, Gideon raised 32,000 men (Judges 7:3), ultimately selecting only 300 to tackle
the 135,000 Midianites, who had apparently made a remarkable recovery from complete
annihilation inflicted by an earlier Israelite army (Num. 31:1-18).

Similarly, Judge Deborah raised 10,000 Israelite warriors from only two tribes (Judges 4:6) to
tackle an invading Canaanite army. They defeated Sisera, who had 900 chariots (carrying two
men each) and an undisclosed number of infantrymen. But wait! Only a few decades earlier,
the Israelites of the exodus had had 600,000 warriors, yet after Sisera's defeat, Judges 5:8 says
Israel had a mere 40,000 troops available to Deborah. Why, Joshua's advanced guard in the
invasion of Judea had alone numbered 40,000 (Josh. 4:13), [That ancient Israelite army stock
market could go from bull to bear and back again in the blink of a divine eye.] Those wild
stock fluctuations must be why, when the era of the judges came to a close some time after
Deborah, Saul was able to raise 300,000 men from Israel and 30,000 from Judah (1 Sam.
11:8). Saul's successor David fared even better. He raised 340,000 troops plus the muster of
Issacher immediately after coming to power (1 Chron. 12) and then later, of course, had 1.3
million or was it 1.57 million? Whatever.

So Saul had plenty of troops to protect his land. That's why, when the Philistines invaded,
Saul raised a massive army of... 3,000 troops? (1 Sam. 13:2). He faced a Philistine horde of
30,000 chariots, 6,000 cavalrymen, and countless infantrymen (1 Sam. 13:5). [By the way,
where did the Philistines get their cavalry, since this was the 11th century B. C. E. and real
cavalry was another 150 years or so away? Egyptian drawings of 12th century battles with the
Philistines show the Philistines with only chariots-- and not too many of those. Surely,
Yahweh didn't commit another anachronism! Say it ain't so, Jeho.]

Evidently, the Philistines were the greatest militarists of history. The Hittite empire, a
superpower of the Late Bronze Age, could field only 3,500 chariots from an area covering
most of modern Turkey and Syria.

The Philistines, who were confined to the coastland of what is now Israel, had more than eight
times that number, plus that mysterious cavalry. The Philistines must have done nothing but
build chariots and train every man, woman, and child for combat. Nothing else could explain
that fantastic army they fielded. So why didn't Saul call out his 330,000 troops? It's a real
mystery, because the people of Israel had to hide in caves and on hills and in cisterns and
tombs (1 Sam. 13:6). You'd think they would turn out by the hundreds of thousands to defend
their country. Poor Saul had only 600 men left of his mighty army (1 Sam. 13:15). Israel's
army was definitely in a bear market at the time. Remember, only a few years later, David had

Volume 1990 - 2002 Issue


Page 492 of 2049
Skeptical Review Edited by Farrell Till
1.57 million troops... or 340,000 plus the muster of Issacher... or 1.3 million. Whatever. David
was riding a bull market in Israelite army futures.

Saul eventually hit a bull market too, for after a long war with the Philistines, he could field
210,000 troops (1 Sam. 15:4). God certainly opened the wombs of the women of Israel. Each
woman must have had quintuplets every year, with a child mortality rate of zero.

As you can see by now, the dry scrubland of Judea--populated by scattered villages and small
settlements-- could raise gigantic armies, larger than the Persian Empire's when it faced
destruction at Alexander's hand, larger than Frederick the Great's, larger than Napoleon's in all
his battles save the invasion of Russia, larger than the Union's and Confederacy's in their epic
struggle. Truly remarkable!

So Israel had the ability to take on its powerful neighbors, such as the Assyrians, who were
stopped by 65,000 infantrymen, 4,000 chariots, 1,200 cavalrymen, and 1,000 camel-riders at
the battle of Qarqar in 853 B. C. E. The Assyrians killed 14,000 enemy warriors, which
undoubtedly included many troops of King Ahab of Israel, along with armies from his allies--
Damascus, Hamath, Cilicia, and six other Middle Eastern states. Ahab himself had brought
2,000 chariots and 10,000 infantrymen to the battle. [I wonder if Qarqar was before or after
Ahab and his army slaughtered 127,000 Syrians as recounted in 1 Kings 20:29-30?]

Only 14,000 casualties? After all those gigantic battles in which hundreds of thousands of
warriors from the nomadic and semi-nomadic peoples of dry Jordan and dry Judea were
slaughtered in epic struggles, the Assyrians-- with their giant cities and dense population--
could manage to kill only 14,000 enemy troops? No wonder the Assyrians didn't win at
Qarqar!

Now for a little reality check on all these numbers that the Bible throws around. In the ancient
world, only the really great powers, such as the Bronze Age Egyptians, Hittites, Assyrians,
Babylonians, and later the Persians, fielded armies upward of 50,000 or more. Around the
19th century B. C. E., the Assyrian King Shamshi-Adad reported having 60,000 troops for a
siege. A typical army might be the 6,000 troops fielded by several Mesopotamian city-states
of the 19th century during a period of prosperity. Their combined populations likely exceeded
Israel's at the time of Saul and David. The kings of these states also kept a close eye on
logistics. Even Shamshi-Adad worried about taking care of 400 troops on an expedition, and
one king questioned another about a frequent problem inplanning a military campaign:
"Where would such a numerous force of men find enough water to drink?" (Stephanie Dalley,
Mari and Karana: Two Old Babylonian Cities, 1984, pp. 141-147).

When Ramesses II fought the Hittites at Kadesh in about 1285 B. C. E., he recorded their
force as 37,000 infantrymen and 3,500 chariots--most with three-man crews--and said that the
Hittites mustered much of the military power of their empire, which covered most of
Anatolia, Syria, and a bit of Iraq (Sir Alan Gardiner, The Kadesh Inscriptions of Ramesses II,
1975, pp. 41-42). Ramesses himself had four Egyptian divisions--possibly the largest army
Bronze Age Egypt fielded. Ramesses' father, Seti, had used only three divisions in a major
campaign.

Volume 1990 - 2002 Issue


Page 493 of 2049
Skeptical Review Edited by Farrell Till
It's worth noting that in his poetic account of Kadesh, Ramesses claimed to have personally
killed hundreds of thousands of Hittites and their allies (Ibid., pp 10-13), and Ramesses
probably lost the battle. Propaganda was invented long before the Israelites appeared.

Smaller states fought with smaller armies. When the 330 princes from Canaan to Syrian
raised a great army in the 15th century B. C. E. to stop the Egyptian Pharaoh Thothmosis III,
they had about 1,000 chariots. Thothmosis captured most of their army at Megiddo, in what is
now Israel, but recorded fewer than 2,000 captives. He probably did not count the common
troops taken captive, but their numbers were in the few thousands, not the hundreds of
thousands.

Egypt was almost certainly the most populous single state in the Middle East because of the
tremendous agricultural bounty provided by the regular floods of the Nile. In the first
centuries B. C. E. and C. E., Egypt had 7 million or so people, according to the census reports
then. The great states of Mesopotamia--relying on the fertile lands around the erratic Tigris
and Euphrates rivers--were close in population to Egypt. Babylon, counting its suburbs, may
have reached a population of 500,000 in the Iron Age.

Israel, by comparison, was tiny in both land and population. Jerusalem and Shechem were out
in the boondocks during the Bronze Age. The 14th century B. C. E. population of the region
that is now Israel is estimated at no more than 250,000, based upon archaeological discoveries
and the analysis of historians ("The Amarna Letters from Palestine," The Cambridge Ancient
History, Vol. 2, Part 2A, 1975, p. 108). The Egyptian garrison commander at Jerusalem
requested only 50 troops to guard the area around that city. Other garrisons in 14th century
Palestine ranged from 50 to 100 (Ibid.). The population of the western lowlands possibly
shrank at the end of the Late Bronze Age and then grew as the Sea Peoples (Philistines)
moved in (William H. Stiebing, Jr., Out of the Desert, p. 94). According to archaeological
surveys, the population of the hill country (early Israel) increased during the Early Iron Age,
but people lived in small settlements, not large cities (Carol Meyers, Discovering Eve:
Ancient Israelite Women in Context, 1988, pp. 51-55).

Ms. Meyers described the evidence for expansion of the population of early Israel in small
villages at this time. She also noted the difficulties the Israelite pioneers faced in working an
area of poor soil and erratic water supplies. It was hardly a landscape that could support
armies numbering tens of thousands, much less hundreds of thousands. In Who Were the
Israelites? Gosta W. Ahlstrom makes similar points about the small size of Israelite
settlements and the difficult nature of the land (1986, pp. 19-22).

Archaeologists have located 23 settled sites for highland Israel in the Late Bronze Age, an
area of about 1,622 square miles. For the Early Iron Age, 114 settlements are known (Meyers,
pp. 51-55). This region was relatively dry. Farming it was hard work, which is probably why
it was not fully settled until after the lowlands were occupied. In essence, their surplus
population--or perhaps fugitives fleeing Egyptian raids and the invasion of the Sea Peoples--
overflowed into the hill country.

Volume 1990 - 2002 Issue


Page 494 of 2049
Skeptical Review Edited by Farrell Till
Nomadic and semi-nomadic peoples were already crisscrossing the area, carrying trade goods,
buying agricultural produce of the settlers in exchange for the animals the nomads raised, and
sometimes raiding the lowlands and retreating into the hills when pursued.

So what about those nomadic Midianites, who supposedly lost 120,000 in a single battle and
still had 15,000 left (Judges 8:10)? Gideon nevertheless had defeated the Midianites with only
300 men (Judges 7:19-22). The allegedly great slaughter of the Midianite army in Judges 8
came as all the Israelites joined in pursuit. What can we make of such inconsistencies? The
only thing reasonable minds can assume: the Midianite numbers were sheer fiction. Gideon's
300 is realistic for guerrilla warfare common to the hills. Remember how the Egyptian
garrison commander of Jerusalem needed only 50 troops to reinforce his control over that
region?

The independent Early Iron Age state of the hill country of Judea was hemmed in by the more
populous and powerful city-states of lowland Canaan and Philistia to the west and strong
Syrian states to the north. The 3,000 Israelite warriors of 1 Samuel 13:2 are realistic. Note that
when Saul was pursuing David, David had a mere 400 supporters (1 Sam. 22:2, everyone "in
debt" or "discontented") or 600 (1 Sam. 23:13).

Saul, David, and Solomon united these growing settlements, conquered areas, and molded
nomads into a nation, but if the Hittites could raise only 48,000 or so troops from their large
empire and if Pharaonic Egypt couldn't do much better from the densely populated Nile
valley, then the Israelite nation never had 330,000, much less 1.57 million, warriors. (Note:
archaeologists have never found any record with the names of Saul or Solomon or the
"judges" of Israel. No one really knows if any of these people actually existed. According to a
wire-service story, a reference to the "House of David" was found in 1994 in association with
the name of Asa, a 9th century king of Judea.)

A comparison with modern Israel will show the absurdity of the biblical claims of armies
numbering into the hundreds of thousands. Israel today has about 6 million people, but it
occupies a larger area than its progenitor state, including the richer coastal region, and uses
modern farming methods, supported by mechanization. In the 1967 war in which Israel
defeated the combined forces of its Arab invaders, Israel's population of 2 million provided
only 264,000 soldiers.

One more reality check: the troops of modern Israel and the forces of all modern armies are
supplied by railroads, motorized vehicles, aircraft, and powered ships. In the ancient world,
armies had supply trains of ox-drawn carts, donkeys and camels, even sometimes humans
carrying packs, and meat traveling on the hoof. Ancient cargo ships were sometimes
comparatively large-- perhaps up to 400-500 tons in the Bronze Age--but they served only
coastal regions. Not until mechanization and improved methods of food preservation arrived
in the 19th century could armies readily exceed 100,000 troops. World War I is the first time
in history when armies of more than 1 million troops met in combat.

People who claim that the Bible is literally true from beginning to end are shameless liars,
openly insulting the intelligence of all of us by defending these absurd numbers. (Either that
or they are incredibly naive and simplistic.) The Bible's convoluted and contradictory figures

Volume 1990 - 2002 Issue


Page 495 of 2049
Skeptical Review Edited by Farrell Till
are propaganda written centuries after the fact by rulers and priests who had ideological axes
to grind. The stories are fantasy, like Ramesses' slaughter of "millions" of Hittites.

The lies amount to one thing: the faithful want us to believe sheer nonsense about the past so
that we will accept equal nonsense about the present. History shows that the claims of the
ambitiously driven superstitious add up to nothing but calculated attempts to make fools of us
all.

(Bill Sierichs, Jr., 316 Apartment Court Drive, Apartment 44, Baton Rouge, LA 70806.)

What About Casualty Numbers?


Farrell Till
William Sierichs' article on biblical armies should convince any objective reader that
Yahweh's inspired writers had a penchant for hyperbole. If the great armies of fairly modern
times, such as those that fought in the Napoleonic wars, numbered only in the tens of
thousands, what reasonable person can believe that tiny Israel and its neighboring nations
could have fielded armies that numbered in the hundreds of thousands? Obviously, then,
biblical writers were prone to exaggeration.

In their zeal to extol the greatness of their God Yahweh, the Hebrews often resorted to
exaggeration. The plagues that Yahweh brought against Egypt, for example, were described
as the severest that had ever happened. The hail was "very grievous, such as had not been in
all the land of Egypt since it became a nation" (Ex. 9:18,24). The locusts were likewise very
grievous; "before them there were no such locusts as they, neither after them shall be such"
(Ex. 10:10). Yahweh granted Solomon wisdom that none before him had had and that none
after him would have (1 Kings 3:12). It isn't surprising then that Bible writers would inflate
numbers to make their nation and their god appear greater than surrounding nations and gods.

The writer of 1 Chronicles claimed that David prepared 3,000 talents of gold from Ophir and
7,000 talents of silver to overlay the walls of the temple (29:3-4). To this, the "princes of the
fathers" contributed 5,000 talents and 10,000 darics of gold and 10,000 talents of silver (29:6-
7). These contributions of gold and silver were to be used only to overlay the walls of the
temple. Altogether (so we are told), David had prepared 100,000 talents of gold and
1,000,000 talents of silver to be used in building the temple and all of its vessels (1 Chron.
22:14). A talent in biblical times varied from 75 to 98 pounds. Thus, if only the lower weight
is used in calculating, we determine that over 8 million pounds of gold and over 76 million
pounds of silver were used in building the temple--or so the Bible claims. Using Encyclopedia
Britannica (15th Edition, 1980, Vols. 8 & 16, pp. 237, 778) as their source, footnotes in The
Bible Handbook point out that this is "nearly three times the total amount of gold produced
worldwide between 1492 and 1700" and "more than five times the worldwide production of
silver in 1970" (Reprinted by American Atheist Press, Austin, Texas, 1986, p. 82). In terms of

Volume 1990 - 2002 Issue


Page 496 of 2049
Skeptical Review Edited by Farrell Till
today's market values, the Israelite temple had $50 billion of gold and $6.5 billion of silver in
it, but what reasonable person can believe that a tiny desert kingdom in the 10th century B. C.
had such huge reserves of precious metals? Obviously these were exaggerations designed to
magnify the majesty and greatness of Yahweh and his specially chosen people of Israel.

That biblical writers exaggerated the size of armies was clearly demonstrated in Sierichs'
article, but the exaggerations extended to battlefield casualties too. To understand this, we
need only compare the casualties recorded in famous battles of more modern times to those
claimed in the Bible. Eighty-five thousand Union and 70,000 Confederate troops clashed at
Gettysburg, Pennsylvania, in a three-day decisive battle of the Civil War that cost the North
23,000 lives and the South 20,000 (Encyclopedia Americana, 1991, Vol. 12, pp. 707-710). In
contrast to this, as Sierichs noted, we are told that Gideon's army of 300 men killed 120,000
Midianite soldiers in a battle recorded in Judges 7. This small band achieved this phenomenal
victory by creating a night-time confusion that caused the Midianites to "set every man's
sword against his fellow" (v:22). Only 15,000 Midianite troops were left alive after the battle
(8:10). If only General Hooker had thought to use this tactic, he could have decimated Lee's
army and saved the lives of thousands of his own soldiers.

On December 2, 1805, Napoleon's army of 70,000 (a paltry number in biblical terms) met
80,000 Austrian-Russian troops in battle at Austerlitz. The allied armies suffered 25,000
casualties, but Napoleon lost only 9,000 (Ibid., Vol. 2, p. 700). This, however, was nothing
compared to a stunning defeat that King Ahab's army inflicted on the Syrians at Aphek, where
"the children of Israel slew of the Syrians a hundred thousand footmen in one day" (1 Kings
20:29). As Sierichs related in his article, the remnant of the Syrian forces fled to the city of
Aphek, where a wall fell on them and killed 27,000 (v:30). Why, the collapse of this wall
killed 25,000 more than the Americans lost during the D-Day assault on Omaha Beach (Ibid.,
Vol. 29, p. 402), and that was one of the bloodiest battles fought in Word War II.

To show the utter absurdity of biblical casualty figures, we need only compare them to the
official numbers of those who died in the WWII battles for Normandy. After General Omar
Bradley's forces established a beachhead on Utah Beach with surprisingly light casualties
(compared to the other beaches), the famous Battle of the Hedgerows began on July 1, 1944,
and lasted until St. Lo was captured on July 8th. Bradley's forces suffered 40,000 casualties
during those eight days (Ibid., p. 404), but this was nothing compared to the losses that
Judah's King Abijah inflicted on the forces of Jeroboam, who reigned over the northern
kingdom after Israel had broken away from Judah following Solomon's death. In a single
battle, Abijah's army "slew [the Israelites] with a great slaughter, so there fell down slain of
Israel five hundred thousand chosen men" (2 Chron. 13:17).

Isn't that amazing? In all of the battles that U. S. forces fought in during World War II, only
292,131 were killed in combat (Ibid., p. 529), yet the army of a tiny desert kingdom 3,000
years ago inflicted almost twice that number of casualties in a single battle! Of course, we are
told that when Jeroboam's forces attacked Judah from both the front and the rear, "they [the
Judeans] cried unto Yahweh, and the priests sounded with the trumpets. Then the men of
Judah gave a shout, and as the men of Judah shouted, it came to pass, that God smote
Jeroboam and all Israel before Abijah of Judah" (vv:14-15). So undoubtedly that explains the
reason for the phenomenal success of Abijah's army on that day. If U. S. troops at Omaha

Volume 1990 - 2002 Issue


Page 497 of 2049
Skeptical Review Edited by Farrell Till
Beach had had a few priests blowing on trumpets, no doubt they could have shouted such
heavy losses on German forces that the war would have ended that day.

The Battle of the Bulge was another decisive engagement of World War II. On December 16,
1944, the Germans in a surprise assault, broke through Allied lines near Bastogne, Belgium.
German and Allied forces fought until the Germans were driven back to the original battleline
at the end of January. This was an engagement of several battles, which inflicted about
100,000 German and 76,000 Allied casualties (Ibid., p. 407). In biblical terms, however, the
six-week Battle of the Bulge was a mere skirmish. In a battle that Sierichs mentioned,
Israelite forces under King Pekah invaded the southern kingdom and "slew in Judah a hundred
and twenty thousand in one day" (2 Chron. 28:6). Just imagine that! The German and Allied
forces, equipped with heavy tanks, artillery, mortars, machineguns, hand-grenades,
landmines, fighter planes, bombs, etc., fought for six weeks in one of the fiercest
engagements of World War II, and only 176,000 were killed on both sides, yet a primitive
army equipped with only swords and spears inflicted over two thirds that many casualties in a
single day!

Few biblical battles were fought that didn't allegedly inflict thousands of casualties. Bradley's
forces landed at Utah Beach on D-Day at a loss of only 200 men (Ibid., p. 402), but when
David engaged the Syrian army of King Hadarezer, he killed 40,000 horsemen (2 Sam.
10:18). At Waterloo, on June 18, 1815, Napoleon's army of 80,000 engaged Wellington's
allied forces of 68,000 (David Chandler, The Campaigns of Napoleon, pp. 1065-1066). This
was the battle that ended Napoleon's military career, but in it he sustained only 25,000 losses.
Wellington's forces suffered 19,000 casualties. However, the confrontation between the army
of Judah and Ethiopia's forces of one million (2 Chron. 14:9) ended with the Judeans
completely routing the Ethiopians. In this case, the biblical historian was restrained enough
not to give specific Ethiopian casualty figures, but he did imply that they were extremely
heavy: "(A)nd there fell of the Ethiopians so many that they could not recover themselves; for
they were destroyed before Yahweh" (v:13). If indeed so many fell that the Ethiopians could
not "recover themselves," then surely they had lost hundreds of thousands from their army of
a million. To assess the probability that primitive armies were able to inflict losses far beyond
those of modern armies, we need only to apply the principle of critical analysis that we noted
in the Autumn 1993 issue of TSR (p.11):

When you lack evidence, the only way to decide whether or not to believe something is to
ask: Is it likely? If you tell me a bird flew past my window, I will probably believe you, even
though I did not see it myself and I have no evidence. That is because such a thing is likely. I
have seen it happen before. It is more likely that a bird flew past my window, than that you
are deceiving me. But if you tell me a pig flew past my window, I will not believe you,
because my past experience tells me that such things do not happen, and so I presume that
what you reported is false. Thus, where there is no evidence we have to rely on our own past
experience of the sort of things that really happen (Carl Lofmark, What Is the Bible? pp. 41-
42).

Any objective person who applies this principle to the battlefield-casualty numbers in the
Bible can only conclude that they are gross exaggerations. If modern armies, equipped with
the latest weaponry mentioned above, even in fierce battles were able to inflict casualties

Volume 1990 - 2002 Issue


Page 498 of 2049
Skeptical Review Edited by Farrell Till
numbering only a relatively few thousand, then reasonable people will understand that the
primitive armies of the Bible couldn't have killed as many as 500,000 in a single day.
Furthermore, if, as Sierichs pointed out, the army of modern Israel numbered only 264,000 in
a time of critical national emergency, who can believe that primitive Israel was able to field
the gigantic armies that biblical writers claimed?

Bible fundamentalists sorely need to read the Bible with their heads and not their hearts.

God Can't Be Tempted?


Proponents of the perfect-harmonytheory have another problem on their hands. If we are to
believe James 1:13, God cannot be tempted: "Let no one say when he is tempted, 'I am
tempted by God'; for God cannot be tempted by evil, nor does He Himself tempt anyone." Yet
if we are to believe other biblical passages, God can indeed be tempted.

To establish this, let's first notice that (according to the inerrant word of God) Jesus was God:
"In the beginning was the Word, and the Word was with God, and the Word was God" (Jn.
1:1). The 14th verse of this passage states that "the Word became flesh and dwelt among us."
Obviously, John meant for us to understand that Jesus was the "Word" who was with God in
the beginning and was himself God. Therefore, if Jesus was God and if God cannot be
tempted, then Jesus could not have been tempted.

This, however, is not what the Bible teaches. After Jesus was baptized, he "was led up by the
Spirit into the wilderness to be tempted by the devil" (Matt. 4:1; Luke 4:1-2). There is no need
to discuss the particulars of this incident in the life of Jesus, because his temptation is a story
that all Sunday school students have heard many times. In anticipation of possible inerrantist
quibbles, however, we should notice that the gospel accounts of this story do not say that the
devil tried to tempt Jesus and failed; they plainly say that the devil tempted Jesus:
"Immediately the Spirit drove Him into the wilderness. And he was there in the wilderness
forty days, tempted by Satan" (Mark 1:12-13).

The writer of the Hebrew epistle certainly agreed that Jesus had been tempted: "For in that He
Himself [Jesus] has suffered being tempted, He is able to aid those who are tempted" (2:18).
In fact, the Hebrew writer didn't just say that Jesus had been tempted but that he had been
thoroughly tempted: "Seeing then that we have a great High Priest who has passed through
the heavens, Jesus the Son of God, let us hold fast our confession. For we do not have a High
Priest who cannot sympathize with our weaknesses, but was in all points tempted as we are"
(5:14-15).

So Jesus was God, God cannot be tempted, but Jesus was tempted in all points as we are. In
the face of flagrant contradictions like this, inerrantists persist in proclaiming the Bible
perfectly harmonious from cover to cover.

Volume 1990 - 2002 Issue


Page 499 of 2049
Skeptical Review Edited by Farrell Till
Go figure!

Religion and How I Lost It


Bob Hypes
"Jesus loves me this I know, for the Bible tells me so." These lyrics constitute one of my
earliest memories of religious instruction or the concept of religion. They may formulate the
base experience for many others as well.

Even if the song itself does not elucidate such a memory, the concept implied in these lyrics
may. This may comprise the primary religious training of the preschool child, a training based
on unqualified love directed from this brotherly figure, Jesus, to the lowly little child, a source
of warmth and comfort, a contrast to the child's own fragility. No matter where we go or what
we do the rest of our lives, that image will remain in some part of our being. It may be the one
feeling that is hardest to shake when we grow to question and doubt this religion called
Christianity.

We next learn that God is the creator of all that we behold and all that we will never
understand. He is the grandfather many of us never knew or an extension of the grandfather
on whose knee we sat when young. We also become aware of God's propensity for wrath, and
we are told not to tempt him or displease him. Then we are introduced to the Holy Spirit and
the unfathomable tale of the Trinity. That three can equal one is totally outside of our ability
to understand. In fact, few, if any, adults can comprehend this one. The story continues to
become more muddled and confusing, and yet we are told we must believe, and we oblige.
Belief becomes a habit driven by fear of the unknown or the fear of rejection if we doubt or
question, so our questions are internalized, and we begin to feel guilt.

We now learn a more rigid set of moral values. We learn that thinking a wrong thing is the
same as committing the act. Our guilt grows, and our ability to deal with it overwhelms us.
The feelings of inadequacy wash over us, challenging the depth and the coldness of the
baptismal immersion. Thoreau said it well: "They think they love God! It is only his old
clothes, of which they make scarecrows for the children. Where will they come nearer to God
than in those very children?"

Theists base their belief on faith, belief based on emotion and culturalization. When reason
and rationale challenge that faith, then the reason can have no value and the rationale must be
incorrect. Faith is irrefutable and errorless because it must be in order to validate all in which
they believe. They then raise their children into the habit of accepting absurdities, mysteries,
convoluted thinking, and supplication. They do this while the children's minds are supple and
moldable. They know that the habits of thought thus formed stand a good chance of lasting a
lifetime.

Volume 1990 - 2002 Issue


Page 500 of 2049
Skeptical Review Edited by Farrell Till
Belief existing in such a vacuum serves to alienate the faithful of each new generation from
the world around them. They either live in judgment of anyone who does not believe as they
do, or they begin to question their own values. The following poem by John Dryden may best
express this phenomenon:

By education most have been misled;


So they believe, because they were so bred.
The priest continues what the nurse began,
And thus the child imposes on the man.

What I thought of as an honest and critical look at the religion I had embraced all of my life
had gone on for years as a halfhearted effort. I wanted to find the truth, yet I wanted that truth
to support that in which I had always believed. In other words, I was front-loading my search
by trying to find corroborating evidences, not by searching for the real truth.

As I delved into the questions raised by rational thought, I increasingly found more questions.
Each answer ended up raising dozens of other questions. I finally had to face the fact that the
only way I would ever find the answers I sought would be to let the truth lead me to its
destination. I then stumbled onto the following quotation. It is known as the Maxim of
Freethought: "He who cannot reason is defenseless; he who fears to reason has a cowardly
mind; he who will not reason is willing to be deceived and will deceive all who listen to him."
This struck home. I realized my cowardice and resolved to overcome it. I threw myself anew
into research but with a new approach.

Biblical literalism and inerrancy appear to be enemies to the truth, and subsequent study on
my part has led me to believe this to an absolute degree. Biblical literalism, as defined and
interpreted by various denominations and individuals, has produced such things as the Amish
shunning of modern lifestyles and snake handling to prove one's faith and refusing medical
treatment to oneself or one's family. Biblical literalism has led to prejudicial actions against
nonbelievers, including imprisonment, censure, torture, death, and even wars. Religion, says
Feuerbach, is self-estrangement. There is the separation of the world into one spiritual and
one earthly. Man sees himself, first, as an individual with limitations, then as a self without
limits, empowered by his God.

A major purpose of fundamentalist religions is to supply a safe harbor for those who are
insecure, fearful, lost or lonely, by justifying a way of life with narrow, defining principles
and prejudices. The authority of the Bible is the final arbiter of any question. The inerrancy of
the Bible is the final argument to justify or indemnify, becoming the central focus of such a
life. The main philosophy of fundamentalists is one of constancy in which they find solace
against an outside world filled with questions. They insulate themselves against such assaults
by finding answers in these words and ideas, no matter how flawed they may prove to be.

To be human means we are doomed to explaining our world, not simply and directly, but only
indirectly, through these interpretations. We dwell in our interpretations. In explicating a
phenomenon, we always put it in terms limited by our ability to understand, always based in
our own prejudices and preconceptions. This means that we will understand things partially
and inadequately, through language rather than a godlike omniscience. Therefore, we

Volume 1990 - 2002 Issue


Page 501 of 2049
Skeptical Review Edited by Farrell Till
internalize our belief structure, i.e., that which causes and enhances our beliefs. At the same
time, we externalize its effects on our lives and that of those about us. This duality of nature
does not lead us to understanding or knowledge but to faith. Faith in an improperly arrived at
conclusion based on illconceived thought processes becomes so entrenched that it is often
thought to be the truth even when it flies in the face of reality.

No reasonable person can believe that the guesses of preliterate man, upon which the myths of
gods and the supernatural are based, were true. The beliefs of these primitives, however, were
more reasonable in terms of their limited and insignificant knowledge, than the beliefs of
today's religionists who have masses of information available to them.

It is apparent that such faith is based upon emotion, rather than reason. Emotion needs no
proof and rejects all questioning. Reason demands answers, questions conflicts, and
objectively studies the issues from every available source and viewpoint. Reason is fearless
thought, undeterred by legal, spiritual, or social penalties. Dissenting viewpoints do not alarm
those who seek truth. The knowledge seeker who has a passion for truth fears nothing except
error.

I have found the average skeptic to have a much broader knowledge of the Bible and
theological issues than the average Christian. Whether led to skepticism by knowledge or led
to the knowledge by their skepticism, the truth of the skeptic is that he is ultimately led by a
search for truth.

Few Christians can delineate the reasons and evidences for their faith. Almost any attempt to
elucidate qualitative responses on the subject elicit catch phrases and incoherent babbling. If
one believes, based on naivety or innocence, it may appear charming or quaint, such as a child
believing in Santa Claus. If one believes culturally, because he was raised to believe certain
things, it can be understood, even if there is no other basis. If one believes as a result of
erroneous information or faulty study, it is lamentable. When one defends, propounds, and
propagates such error as fact and refuses to examine other information objectively, it is
intellectually reprehensible, and I will challenge that type of belief every time.

Biblical literalism presents more questions than answers. It offers a god we cannot respect or
understand, a god who changes vastly from passage to passage and event to event, a lack of
consistency in what should be consistent if our faith is not to be shaken. What is impossible
for our minds to believe our hearts cannot worship.

(Bob Hypes, P. O. Box 305, Howe, IN 46746.)

EDITOR'S NOTE: Bob Hypes' letters have appeared in previous issues of The Skeptical
Review. He is a former Church-of-Christ preacher, and he tells a familiar story. He grew up
believing what he had been taught in his childhood, but when he engaged in serious Bible
studies as an adult, he found things in it that made it impossible to continue believing what he
had been taught as a child. Many former fundamentalists will say that the Bible is its own
worst enemy. If we could just get more Christians to study this book that they claim to believe
in so much, the inevitable result would be fewer Christians. The Christian religion thrives on
ignorance of the very book that is its foundation.

Volume 1990 - 2002 Issue


Page 502 of 2049
Skeptical Review Edited by Farrell Till

God Is Omniscient?
Elsewhere in this issue (pp. 9, 16), we discussed inconsistencies in the biblical claim that God
cannot be tempted (James 1:13) and the fact that the Bible also claims that although Jesus was
God (John 1:1, 14), he was nevertheless tempted (Mark 1:12-13; Heb. 5:14-15). If we accept
the premise that Jesus actually was God incarnate, we have another inconsistency between the
passages that teach that God is omniscient or all-knowing (Ps. 147:5) and other passages that
indicate that Jesus didn't know certain things.

In summarizing the childhood of Jesus, Luke said that he "increased in wisdom and stature,
and in favor with God and men" (2:3), but if Jesus were truly an omniscient deity, how could
he possibly have "increased in wisdom"? To increase one's wisdom is to increase one's
knowledge, so if Jesus increased in wisdom, he couldn't have been omniscient when he began
to increase his wisdom. Yet we are asked to believe that Jesus was the omniscient god who
made all things (Col. 1:16).

In speaking to the apostles about the time of his second coming, Jesus said, "But about that
day and hour no one knows, neither the angels of heaven, nor the Son, but only the Father"
(Matt. 24:36). The problem in this statement is obvious. If Jesus were an omniscient deity,
how could it possibly be that he would not know the day and hour of his second coming? The
issue of omniscience aside, we might also ask how it could possibly be that the Father would
know something that the Son wouldn't know if the Father and the Son were one and the same
as the absurd doctrine of the trinity teaches.

Some scribes must have recognized the problem that this passage poses, because some Greek
manuscripts of the New Testament made no reference to "the Son" in this verse. The ASV,
RSV, NRSV, NIV, and other versions of the New Testament have a footnote to inform
readers that "some ancient authorities omit 'nor the Son,'" and the KJV and NKJV omit the
reference entirely. However, the fact that "some ancient authorities" don't mention "the Son"
in this verse is of no benefit to inerrantists, because the exact statement is in Mark's parallel
account of Jesus' discourse on his second coming (13:32), and there are no footnotes in Mark
to inform us that "some ancient authorities" omitted the reference to "the Son." Hence, the
problem remains. God is omniscient, Jesus was God, yet Jesus didn't know when he would
come again, even though the Father, who is the same God that Jesus is (was) did know. Was
Peter wrong then when he said to Jesus, "Lord, You know all things" (John 21:17)? And if
Peter was right, were Matthew and Mark wrong when they said that Jesus said there was at
least one thing he didn't know? And if there was even one thing that Jesus didn't know, how
could he have been an omniscient god?

Maybe some enterprising inerrantist can explain all this to us dumb skeptics.

Volume 1990 - 2002 Issue


Page 503 of 2049
Skeptical Review Edited by Farrell Till

From the Mailbag


What an honor to be published by your fine magazine. I enjoyed your article entitled "Mcbull
to the rescue". Its [sic] me, Mr. Till! I am the mighty McBull!!! I was sure to thank Richard
Fiori for forwarding our little debate to you. In my defense however, I was unaware that he
was going to do that. So it comes as both an honor and a surprise, that I find myself gracing
the pages of your periodical. I do have to admit that my response was not as complete as I
would have liked, had I known it was going to be picked apart by someone of your caliber, I
could have provided you with references and further background information. The thing you
have to remember, Mr. Till is that not too many people will stand in and debate with me on
America Online [sic]. Most people are not as informed (or deceived) as you are, and as a
result never respond back to me. That being the case, if I were to write a detailed response to
every person, then I wouldn't have time to eat, sleep, or take a dump, for that matter.

On the issue of the Amalekites, I would be more than happy to provide you with my sources
and information, then perhaps we could debate the issue further to the enjoyment of your
readers. However, I picked up on an nference [sic] in your article that the subject had become
tedious to you. Perhaps you would rather let it go? We can just call it a draw. By the way,
what's the story with all the "sics'? All I could picture was my cat when he gets a hair ball,
going "sic....sic....sic". Ever have a cat do that? It's sort of entertaining to watch, if you know
what I mean. The most profound thing to me in the piece you mentioned "...The fact that two
people working independently of each other would write such similar articles..." I found that
to be most profound, for reasons I don't expect you to understand. I'll assume Mr. Mitchell
knew he was being published? Anyhow, I'll leave this one up to you, I'll defend every point I
made, refute all of yours, or move on. More on than [sic] in a moment.

Please accept my subscription, enclosed you will find $4 cash for my first year. Ingersol told
me that was the cost, I assume he is correct. Please mail it too [sic] the Mighty McBull care of
the address above on this letterhead.

Now, I had a thought. Would you consider me for a position in your publication? I got to
thinkin' I would be interesting [sic] in being your resident whipping boy. Perhaps we can have
a debate for the sake of the reader's [sic]. I'll take you on - in any subject. I'll even let you
choose the topics and I don't even want to be paid for it! How about that!!! Maybe, I could
have an editorial column, The Christian Perspective, or something like that. I could get the
readers all riled up, and get them interested in the debate process. What do ya think Farrell,
I'm game for it? I even have a great sense of humor, and I'm a really nice guy to boot. Drop
me a line.

By the way, could I get a copy of the issue that I am going to be published in, I'll pay for
whatever it costs. My mommy will be so proud of me.

(Matthew C. Bullard, The Mighty McBull, 15520 Rockfield Boulevard, Suite E-2, Irvine, CA
92718.)

Volume 1990 - 2002 Issue


Page 504 of 2049
Skeptical Review Edited by Farrell Till
EDITOR'S NOTE: Gee, I don't know what to do. Should I take the Mighty McBull on and
risk humiliation from the blows of his impeccable logic or should I crawl into a hole and hide
until he goes away? Oh, well, what the heck, I always have been the foolhardy type, so I think
I'll take him on.

Let's begin with that impeccable logic of his. He tells us that "not too many people will stand
in and debate" him on AOL. Well, okay, if that's the case, then I would think that he would
have all the time he needs to eat, sleep, and attend to his bodily functions. He informs us,
however, that since few will take him on, if he were to "write a detailed response to every
person [who did], then [he] wouldn't have time to eat, sleep, or take a dump, for that matter."
Somehow, the coherence of his reasoning eludes me. Perhaps he can explain why not having
very many AOL opponents infringes on his time for eating, sleeping, and toilet duties.

The Mighty McBull assured me that he would have put forth a better effort had he known that
his response to Ingersol (Fiori) was going to be published in The Skeptical Review. Am I to
believe that he didn't mind if tens of thousands of AOL users saw a sloppy performance but
he would have preferred TSR's 1,200 readers to see him at his best? What I suspect is that we
really have seen McBull at his best and that he is just desperately trying to put a better face on
a pathetic performance that he thought was pretty good until he saw his silly pulpit arguments
critically analyzed. An informed analysis of his Gleason Archer-Norman Geisler echoes was
probably a new experience for him.

For reasons that he doesn't expect me to understand, McBull thought it "most profound" that I
had pointed out that "two people working independently of each other would write such
similar articles," so let me see if I do understand what McBull sees profound in this. I suspect
that he wants us to think that the similarity wasn't really coincidence but merely the result of
truth being easy to recognize. However, until he makes good his promise to "defend every
point [he] made" and "refute all of [mine]," I'll choose to believe what is probably the real
reason for his and Mitchell's agreement in the matter. They both relied on a pathetically weak
"explanation" that has been widely disseminated in fundamentalist churches.

So should I make McBull our "resident whipping boy"? Well, first I have problems with the
title he wants to appropriate. A whipping boy was someone designated to stand in and take
the punishment due another, but McBull has personally earned every whiplash I have
administered. He has simply parroted the absurdly illogical "explanations" of the apologetic
professionals, and he did it on the writing level of a junior high school student. Did he
seriously think that I would tremble in fear at the prospects of crossing verbal swords with
someone who doesn't know what a sentence is or the difference in there and their and to and
too? Does he expect us to believe that someone who doesn't know the basic rules of
punctuation nevertheless knows the answers to questions that have puzzled Bible readers for
centuries?

Although we can't make McBull our "resident whipping boy," we are certainly willing to give
him space to demonstrate his forensic skills. He boasted that he can defend every point that he
made and refute all of mine, so we're going to give him the opportunity to make good that
boast. If he will send us a reply to "McBull to the Rescue," we will publish it. We will expect
him to make good his boast to defend every point that he made and refute all of mine. This

Volume 1990 - 2002 Issue


Page 505 of 2049
Skeptical Review Edited by Farrell Till
means, of course, that he will present proof that (1) the Amalekites of Saul's day were
"guerrilla terrorists" who had been given "plenty [of] time to repent of their utter rejection of
God," (2) the Amalekites of that time practiced incest, molestation, homosexuality, bestiality,
child sacrifice, and murder, (3) the Amalekites of that time had to be destroyed because of the
gravity of their sins and so that the "remnants could not continue their acts of violence against
the nation of Israel," and (4) the act by which God took the lives of the Amalekite children
was far from merciless because the children went to heaven. In support of this final point, we
hope that McBull will provide us with photographs of children frolicking in heaven. If he
lives up to his expectations, we will certainly look forward to publishing him again on another
subject.

Dear Mr. Dobbs:

As a university student, I am relatively new to the intellectual scene of religion and


skepticism. Recently, I have watched the tapes of your debate with Farrell Till, which took
place in Portland. There are many things that I can comment on about the debate, but I have
chosen one topic in particular to discuss in my letter.

I would like to comment on your argument for the veracity of "metaphorical prophecy" from
the Old Testament. I believe that Farrell Till was correct in stating that this was a rather
"weak" argument and that you "stretched the meaning by a country mile." For example, you
stated that the story of Jonah living three days in the whale gave the "substance" of prophecy
for Jesus rising out of his tomb after three days. Don't you see the fallacy you are making?
This allows Christians to snatch practically anything out of the Old Testament, twist it around
to fit their own theology, and proudly declare it a glorious "fulfilled" prophecy. This is exactly
what the writers of the Gospels were guilty of doing.

Let's just take a hypothetical case. With your definition of a Bible prophecy, I can easily say
that the story of the Genesis Flood was a messianic prophecy. How you might ask? Well,
Yahweh wanted to flood the world so that he could destroy the wicked, but Noah was
righteous in Yahweh's eyes, so he instructed him to build an ark so that he could be saved
from the floodwaters. Hence, the wicked symbolize all those who are unsaved and thus will
perish in the final judgment. The ark is a metaphor for Christ who will save all those who are
righteous before God.

It took me only about forty seconds to think up this metaphor. I am willing to predict that if I
stood in front of your congregation, told them I was a Christian apologist, and stated the
above metaphorical "prophecy," I would probably receive full acceptance and maybe a chorus
of "amens" too. In Dan Barker's book Losing Faith in Faith: From Preacher to Atheist, in his
chapter on Bible prophecy he states, "Maybe this kind of thing is a 'proof' to those who
already believe, but they should certainly be able to understand why the rest of us raise our
eyebrows" (p. 191).

In closing, I would like to say that when I watch public debates I try to look past my biases
and the rhetoric to determine the winner. Mr. Dobbs, I can honestly say that you were not the

Volume 1990 - 2002 Issue


Page 506 of 2049
Skeptical Review Edited by Farrell Till
victor in Portland. If you wish to convince us skeptics that Christianity is true, arguments like
"metaphorical prophecy" will certainly not establish your case.

(Jason Munroe, 70 Nottingham Road, Sherwood Park, AB, Canada, T8A 5L5.)

EDITOR'S NOTE: As soon as the last session of my second debate in Portland ended (May
24, 1994), Mr. Dobbs, who had served as one of Jerry Moffitt's assistants, angrily confronted
me and called me a liar for saying that I had distributed video tapes of the Till-Dobbs Debate
to various TSR subscribers. The longer he ranted, the angrier he became, saying among other
things that The Skeptical Review was a big nothing and had nowhere close to a thousand
subscribers. I was understandably delighted just a month later to receive Jason Munroe's
letter, which proves not only that I have made the tapes available to TSR viewers but have
even sent them out of the country. I have had several debating opponents, but Mr. Dobbs has
easily proven himself the sorest loser of them all.

I recently received a copy of your Skeptical Review and read it cover to cover.

How you got me on your mailing list I'll probably never know, but I am grateful. Although I
am not as yet an atheist, I am certainly a skeptic and found your arguments profound and
titillating.

Enclosed is a check for $5 for copies of the Laws-Till Debate and Prophecies: Imaginary and
Unfulfilled. I will look forward to receiving them

Thank you for the copy of your publication, and I trust you will continue to send them.

(Lola R. Reams, 412 Memphis Way, Vancouver, WA 98664.)

EDITOR'S NOTE: Ms. Reams was put on our mailing list at the request of a friend. If you
have friends or relatives whom you think would like to receive The Skeptical Review, send us
their names and addresses. We will send them TSR for a year at no cost.

I am writing to inform you of a change of address, but first I just want to say how much I have
enjoyed your publication. My only complaint is that the issues are too small and too
infrequent. I read and reread each issue I receive, eagerly anticipating the next one. Keep up
the great work. The world needs more clear thinking and rationalism such as yours to make it
a better place and to free people from intolerance, guilt and hatred such as is engendered by
religious superstitions.

(Bob Hypes, P. O. Box 305, Howe, IN 46746.)

Volume 1990 - 2002 Issue


Page 507 of 2049
Skeptical Review Edited by Farrell Till
Thank you for allowing me to view your taped debate with Mr. Dobbs. I think you did an
excellent job. Mr. Dobbs is mistaken with his analogy between testimonies of the life of Jesus
and other secular figures. The first involves events of a totally unique nature; the second are
particular instances of general phenomena, which may or may not be true but are possible or
probable.

I am very interested in Bible inerrancy (or errancy?) and contradictions, but so far I have read
only Thomas Paine's Age of Reason. What books do you recommend? Which Bible
translation do you think is best?

Thank you again, and I hope to hear from you soon.

(Richard G. Rich, Route One, Box 89A, Decatur, AL 35601.)

EDITOR'S NOTE: Isn't it strange that simple truths are so obvious to some people but not to
others? Mr. Rich has no difficulty distinguishing ordinary historical information from
extraordinary religious claims. Now why can't Bible fundamentalists see the difference too?

The Christ by John Remsburg, Is It God's Word? and Forgery in Christianity by Joseph
Wheless, The Diegesis by Reverend Robert Taylor, How Did Christianity Begin? by Howard
Teeple, The Story of Christian Origins by Martin A. Larson, The Myth of the Resurrection or
just about any other book by Joseph McCabe would be excellent reading materials for those
interested in researching biblical origins. For research in basic biblical contradictions and
discrepancies, The Bible Handbook (G. W. Foote, W. P. Ball, et al) and Dennis McKinsey's
monthly paper Biblical Errancy (3158 Sherwood Park Drive, Springfield, OH 45505) would
be good sources to study.

Emmett Fields, 514 Eastern Parkway, Louisville, KY 40217, has scanned numerous
freethought classics and recorded them on computer disks. A set of 10 HD disks on which
most of the books listed above and many others have been copied in ASCII files can be
obtained from him for only $10. If there ever was such a thing as a bargain, this is definitely
it.

I use primarily the American Standard Version but probably only because it is the version I
used when I was a preacher. I have 27 different versions in my library. They all have their
strengths and weaknesses. Modern versions, such as the NRSV or NIV, can make meaning
much clearer than the old King James Version, but one must use them carefully. In some
cases, these versions seem to mistranslate deliberately in order to conceal inconsistencies in
the original texts. The NIV, for example, changed Ahaziah's age from 42 to 22 in 2
Chronicles 22:2 to make it agree with a parallel statement in 2 Kings 8:26.

You asked for permission to use my original letter in the winter 1995 issue of TSR. You
certainly do have my permission. If its publication could lend courage or support to even one
other person, pastor, missionary or spouse, then I would want it printed in bold letters.

Volume 1990 - 2002 Issue


Page 508 of 2049
Skeptical Review Edited by Farrell Till
May I take this opportunity to relate to you just a bit more of my--that is, our--story? You
might find it interesting.

Both my wife Virginia and I were raised in conservative Christian homes during the '40s and
'50s. Charles Fuller's Old-Fashioned Revival Hour was a regular in both our homes, as were
the broadcasts of Donald Grey Barnhouse and many others. We met in our first year at
college (the former National Bible Institute, renamed Shelton College; this eventually became
one of the strongholds of Carl McIntire, founder of the American Council of Christian
Churches) and have been together ever since. I graduated from Gordon Theological Seminary
(now Gordon-Conwell) in Wenham, MA, and served churches in both the Baptist and
Presbyterian denominations, all strictly evangelical/ fundamentalist.

During those years, I always preached the standard gospel message of the "necessity of the
new birth," based on the foundation of an "inerrant" Bible. I gave altar calls, held evangelistic
and missionary conferences, sent the young people to Christian camps, etc., etc. But despite
all this, which of course included an active personal prayer life, I became more and more
concerned about the truth and validity of what I was doing and what I was saying.

To help quiet my ever-growing uneasiness, I decided to return to formal studies, eventually


graduating from Westminster Theological Seminary (Philadelphia, PA) with a Th. M. degree.
Two years later, I again returned to the books, entering the Ph.D. program of the University of
Nebraska at Lincoln ("Big Red" territory). My major was ancient history; my hopes were to
find a way to bolster my sagging faith. (Yes, I was serving a church at the time.) I became a
"teaching fellow" and was well on my way to a degree, but time began to run out in the sense
that I was losing in the faith department faster than I was gaining in the doubt department.

More knowledge--knowledge about the Bible and Christian history and other religions, etc.--
only made the situation worse for me; now I could clearly see the fallacies, problems, cover-
ups, denials, inconsistencies, exaggerations, and misrepresentations, which at one time I only
suspected existed. To her eternal credit, it was my wife who had the guts to say, "This is
crazy; you can stay in the church if you want, but count me out." Without her, I fear it would
have taken me twice as long to do what needed to be done years earlier: I resigned never to
pastor another church again. And--I want it clearly understood--not only did I resign from my
particular church, but we also left the entire Christian system behind.

Have we ever regretted this decision? Never. Not once. I admit, for a while, we didn't know
what, if anything, to do about beliefs, or god, and such things. But this should not be mistaken
for sorrow for leaving either the pastorate or the "faith." We have never, at any time, been
sorry. We feel free, unburdened, relieved... and happier than we have ever felt in our entire
lives. The initial concern that god would "get us" for this is gone. Now, together we explore
ideas, seek new realms, experience wisdom previously lost to us. We look at social, political
and spiritual issues in an entirely new way, invariably diametrically opposed to our old
"Christian" views.

As we consider the condition of the evangelical church of today, see their tactics, listen to
their "logic," understand their agenda, hear their rationalizations, we are indeed glad that no
longer are we caught up in this mindset, which once so powerfully "had us."

Volume 1990 - 2002 Issue


Page 509 of 2049
Skeptical Review Edited by Farrell Till
Well, I'm not finished expressing myself by a longshot, but I'll stop, appreciative of this
opportunity to have my say. There's a lot more to relate, about our present "beliefs," and what
we are endeavoring to do, but I know you're a busy man. If you would like to hear more some
time I'll be happy to comply. In the meantime, we do wish you success in your work. We feel
a close kinship to people like you and are glad someone is doing something to refute the
claims of that so-called "mighty army."

One final question: do you know of anyone who is seeking to help former preachers and
missionaries and their spouses on an emotional or psychological level through a support
network or newsletter or something? ...I'm thinking of former Christian workers only, not lay
people.

(Ransom L. Webster, 121 Jaffrey Road, Marlborough, NH 03455.)

EDITOR'S NOTE: Mr. Webster sent an earlier letter in which he said that he had been a
fundamentalist preacher for 20 years and now "shudder[s] at those days and the amount of
energy spent leading so many people astray." Although his second letter was much longer, I
couldn't resist publishing it instead of his first. Again, we have personal testimony of the
happiness and intense satisfaction that comes from breaking the shackles that enslave the
mind to religious superstition and nonsense. In my debates, the opposition continually charges
that I have nothing to offer but doom and despair, but those who have been down the same
road I have traveled consistently testify to the tremendous joy and relief that accompany
abandonment of the ridiculous belief system that Christian fundamentalism represents.

Although I am pleased to publish this ex-preacher's story, there is really nothing new in it.
Long-time subscribers to TSR will remember similar letters that we have published, and they
all tell essentially the same story. A preacher begins to study higher criticism, world history,
religious history, philosophy, etc., and eventually, he begins to see that he has been misled.
Christianity is not a unique religion that God revealed in the first century but only a
patchwork of pagan beliefs fraught with "fallacies, problems, cover-ups, denials,
inconsistencies, exaggerations, misrepresentations," and many other flaws that Mr. Webster
didn't mention.

Each letter like his makes me wonder how many more fundamentalist preachers are out there
wallowing in misery, who want to get out but don't know how. If by chance any read this and
want whatever assistance I can give, please be assured that your letters will be held strictly
confidential. I have been where you are, so I know what you are going through.

I received the Spring 1994 issue of The Skeptical Review and read the whole issue the same
day. My wife snagged it next; then I took it to work where I have been involved in the
occasional argument over Biblical errancy. We enjoyed it very much, and I would like to
express our appreciation of your efforts in making this available to others.

I run an electronic bulletin board on my personal computer, and the articles in The Skeptical
Review look as if they will make excellent reference material for the debates I am involved in

Volume 1990 - 2002 Issue


Page 510 of 2049
Skeptical Review Edited by Farrell Till
regarding Judeo/Christian religions. I have accumulated a variety of information and opinions
which I store electronically to facilitate rapid searching and quoting when needed, and I was
curious if I could get the past issues of The Skeptical Review on disk so as to utilize in the
same manner. I have enclosed the fee requested for the 17 back issues and three booklets
listed on the last page of the Spring 1994 issue (The Laws-Till Debate, Jackson-Till Debate,
and Prophecies: Imaginary and Unfulfilled). If this request is not possible to be fulfilled,
please just send the paper versions instead.

Assuming I can get the information electronically, there are several options open for me. I
have an IBM compatible, with a 1.44 meg 3.5" floppy drive, or my bulletin board is operating
24 hours a day with a 14.4 US Robotics modem running at 57,600. I have a guest account
which requires no password to log on, and a compressed file containing all of the information
can be attached to e-mail to the sysop (me). I can also call out to Illinois to pick up the data if
there is a board which is willing to act as an intermediary for us (which I could quite easily
arrange; I know several sysops out there.)

If there is any other information you need, or if you have any questions or even want my
reference materials (don't know if they would benefit you or not, but you are certainly
welcome to them), my home phone is (619) 667-1223, and the BBS phone number is (619)
697-9635.

(Dante Ridley, 5639 Amaya Drive, Apartment 305, La Mesa, CA 91942.)

EDITOR'S NOTE: We sent Ridley not just the paper versions of all back issues but also all of
the articles in ASCII. This will be just one more of many BBS's sending out TSR and other
biblical errancy materials. As I said in an earlier "Mail-bag" column, computer technology
will undoubtedly hasten the demise of Bible fundamentalism. In the past, Christians could
suppress information detrimental to their cause, but this is no longer possible. The lay public
is going to learn a lot about Christianity that is not complimentary to it. If other BBS
enthusiasts would like to have our articles on disk, please let us know.

I hate to see you waste your time or our time by sending your "junkmail" to us. We will not
read it or show it to any of our friends as it is offensive to us. So be advised we wish to never
see another copy of this sent to us. We trust we will not hear from you again.

Matthew 24:4, "Watch out that no one decieves [sic] you." Mark 13:13, "All men will hate
you because of me, but he who stands firm to the end will be saved." This is our stand!

(Randy and Candy Eckstein, 695 Hillcrest Road, Milan, IL 61264.)

EDITOR'S NOTE: Since we have no desire to send TSR to people with the Ecksteins'
attitude, we opened our address files to remove their name and found that they were not listed.
We have no idea who sent them the copy that raised their hackles.

Volume 1990 - 2002 Issue


Page 511 of 2049
Skeptical Review Edited by Farrell Till
Some scriptures give good advice, and the Ecksteins quoted an excellent one in Matthew
24:4. It contains advice that we would urge them to heed, but, unfortunately, it is too late for
them. They have already been deceived. The world's best biblical scholarship rejects the Bible
inerrancy doctrine, but apparently the Ecksteins continue to allow preachers to deceive them
into believing superstitious nonsense.

We are publishing their letter so that whoever sent them TSR will know how close-minded
they are. If they should ever flaunt their righteousness or attempt to evangelize the one who
sent them TSR, he/she will then have every right to tell them that their "junk-belief" is
offensive.

While picking Lindell Mitchell's arguments to pieces, you overlooked a simple flaw at the
heart of his logic. Mitchell argued that god "looked down the corridor of time" and foresaw
that "the babes of Amalek were destined to become vicious beasts like their ancestors."

Reality check! The babes of Amalek were not destined to become vicious beasts. Obviously,
they were destined to die as babes, slaughtered by marauding Israelites. How can anyone,
speaking normal English, say an event was predestined yet did not happen? Can anything be
more self-contradictory? Just what does predestined mean in Mr. Mitchell's vocabulary
anyway?

Well, perhaps he thinks God "changed destiny." Then destiny is flexible. "Destined" events
are not inevitable. So why bring it up, except to fog the issue?

Lindell Mitchell is a master of evasion. Don't let him get away with it.

(Kenneth E. Nahigian, 2411 Tyrolean Way, Sacramento, CA 95821-4809.)

EDITOR'S NOTE: Nahigian makes an excellent point, and I must admit that I'm a little
embarrassed for having let it slip by me. The fact that a reader picked it up just may be one of
the reasons why Mitchell and his inerrantist cohorts are reluctant to have their articles
published in TSR. They realize that they are writing for a much more critical audience than
what they are accustomed to in their religious journals. Like newly-hatched nestlings, pew-
warmers tend to swallow everything poked down their throats, but Mitchell has seen enough
from our readers to realize that this isn't the type of audience he will be dealing with when he
is published in The Skeptical Review.

I enclose $10. Four dollars is for this year, and since it is likely that the postal rates will go up
next year, the six bucks is to cover the second year's subscription. If you do not raise the
subscription rate next year... well, you really ought to raise it. Four bucks is too cheap.

(Jacques A. Musy, P. O. Drawer 69, Valrico, FL 33594.)

Volume 1990 - 2002 Issue


Page 512 of 2049
Skeptical Review Edited by Farrell Till
EDITOR'S NOTE: The subscription rate is $5 this year but only because we will be
publishing five issues rather than four. Postage rates did increase for nonprofit bulk mail, but
we are trying to absorb the increase without raising the subscription rates.

After Reading my first two issues, I'm so favorably impressed that I want to order all the back
issues from Winter 1990 through Winter 1994. I think that comes to 17, so here's my check
for that many dollars. Your responses to the letters from Nikki Kaley and Mildred Bahn were
especially well put. As you say, though, it does amount, as far as they are concerned, to
casting pearls before swine. Those people are so deluded as to very likely be past any hope of
enlightenment. As the saying goes, "There are none so blind as those who will not see." I
know, because I grew up around that kind and was once almost one myself. Fortunately, I
eventually came to my senses.

(Jerry Brown, 8929 Hubbard Street, Culver City, CA 90232.)

Having just sampled your excellent publication, Skeptical Review, Winter '94, I would now
hate to miss another debate. Please put me on your mailing list A.S.A.P., and I will gladly pay
any postage or fees.

I currently subscribe to Freethought Today, Humanist in Canada, and American Atheist. It


took me 37 years to discover others like me (M. O'Hair on Donahue, 1992), who are anxious
for information and determined that critical thinking should have more exposure and be much
more accessible; we can't help each other or make a difference by helplessly letting others
dictate to us.

(Judy Loewen, Box 429, Debolt, Alberta, Canada T0H 1B0.)

I enjoyed your summer issue, particularly your discussion of Buster Dobbs' argument that the
existence of a counterfeit proved the existence of a real article. I like Mr. Dobbs' logic. Many
of the sayings attributed to Jesus are copied from Jewish scripture, so he clearly destroys
Christianity. It's only a counterfeit of Judaism. But... much of the material in Jewish scripture-
-outside of the historical chronicles--is copied from even earlier works. For instance, Genesis
1 is counterfeited from the Mesopotamian origin work the "Enuma Elish." Genesis 2 is
plagiarized from a separate Mesopotamian origin story, which explains the contradictions
between the two accounts.

Also, the Noah story is counterfeited from the earlier Mesopotamian story of Atrahesis and
the ark, which is copied from the earlier (about 2000 B.C.E.) story of Utnapishtim and the
ark, found within the Gilgamesh epic, which is copied from the 3rd millennium B.C.E. story
of the Sumerian king Ziusudra, who rode an ark to safety atop a worldwide flood. Many more
examples of counterfeiting from pre-Israelite societies in the Middle East can be cited.

Volume 1990 - 2002 Issue


Page 513 of 2049
Skeptical Review Edited by Farrell Till
Thus, Judaism is only a counterfeit of the Mesopotamian religion. Dobbs' logic makes the
origins of the Bible easy to explain.

Your list of dying/resurrected gods, from whom Jesus was counterfeited, was good, but left
out one significant deity. The Sumerians worshiped the goddess Inanna as the Queen of
Heaven, the source of the fertility of life, the original sex goddess, and the creator of
civilization. Later Semitic peoples joined in worshiping her under the name Ishtar, aka Anath
and Astarte to the Canaanites and Israelites. (Jeremiah 44:17-19 is a reference to Israelite
worship of her with a sacrament of wine and cake). She was regarded as the power behind all
thrones, the source of victory in war, the healer of the sick, and the inspiration of prophets and
visionaries.

Inanna/Ishtar also was a dying/ resurrected deity. She supposedly visited her twin sister (or
the dark side of her psyche) Ereshkigal, who was ruler of the underworld. Ereshkigal had
Inanna/ Ishtar killed, and her body was hung up, either upon a peg in a wall or upon a wooden
stake (almost like a crucifixion). After three days, the goddess was resurrected. She then
appeared to various people on the earth, looking for a substitute for her in the underworld. She
finally selected her lover--Dumuzi in Sumerian stories, Tammuz in Semitic ones. He then had
to spend part of each year in death. That's why the women of Israel, among others, wept for
Tammuz, as you cited in your article (Ez. 8:14).

Scholars in recent years have gotten away from earlier theories that Christianity was simply
copied from pagan ideas and are focusing instead on Christianity's inspirations from Judaism.
So I can't say that Jesus is just a transvestite version of Inanna/Ishtar, but it's obvious that if
Jesus' parents were Jewish, his grandparents were pagan. This is why Christians, every Easter,
prominently feature such pagan fertility symbols as fresh flowers, egg-carrying rabbits and
bright green grass. They're honoring the ancient fertility deities--mother goddess
Inanna/Ishtar; grain god Osiris; and rain god Adad, aka Baal, the dying/resurrected son of the
Canaanite/Israelite high god El, who is revived through a grain-sowing ritual.

Truly, some things under the sun never change.

(William Sierichs' address is printed on page 8 at the end of his article.)

Volume 1990 - 2002 Issue


Page 514 of 2049
Skeptical Review Edited by Farrell Till

The Skeptical Review


Volume Six, Number Two - 1995
Farrell Till, editor

• The Wisdom of the World


Fundamentalists scoff at "the wisdom of the world", but the wisdom of the world has a
far better track record than the "wisdom" of the Bible.

• More Trouble for the Perfect-Harmony Theory


This article cites several examples of biblical contradictions to show that the "perfect
unity" of the Bible is anything but perfect.

• Let Us Pray
Prayer is a practice than cannot survive the harsh spotlight of logic. This article shows
why.

• The I'm-Not-Certain-I-Exist Debate


Jerry Moffitt, for reasons known only to him, wants Farrell Till to debate on the issue,
"I (Farrell Till) may not exist." Till may or may not exist, but it's pretty clear that Jerry
Moffitt isn't all there.

• Yes, View the Tapes! View the Tapes!


Farrell Till and Jerry Moffitt seem to agree on one point: People should view the tapes
of the Moffitt-Till debate.

• What Was a "Mamzer"?


A closer look at the use of the word mamzer in Deuteronomy 23:2 reveals more
problems for biblical inerrantists.

• Still Chewing: Chasing Rabbits With Dr. Geisler


Jeffrey A. Justice responds to Dr. Geisler's attempt to explain away the biological
error in Leviticus 11:5-6, in which it is said that rabbits chew their cud.

• From the Mailbag

Volume 1990 - 2002 Issue


Page 515 of 2049
Skeptical Review Edited by Farrell Till

The Wisdom of the World


Have you ever thought about the implications of Adam's and Eve's sin? They ate of the tree of
knowledge (Gen. 3:6), an act that opened their eyes (v:7), and so God drove them from the
Garden of Eden, because "the man is become as one of us, to know good and evil" (v:22). In
other words, their sin was the acquisition of knowledge. It was a sin so great that the petulant
Yahweh not only banished them from the garden but pronounced an everlasting curse on them
and all their descendants.

The condemnation of knowledge implied in this story established a policy that was generally-
-but not always-- followed by the other biblical writers. That policy was to discourage and
even sometimes to condemn the acquisition of knowledge. Perhaps no single biblical writer
ridiculed knowledge any more than the apostle Paul, who is considered by many scholars to
be the real founder of Christianity. His strongest denunciation of knowledge was made in his
first letter to the Corinthian church:

For the preaching of the cross is to them that perish foolishness, but to us who are saved it is
the power of God. For it is written, "I will destroy the wisdom of the wise, and will bring to
nothing the understanding of the prudent." Where is the wise? Where is the scribe? Where is
the disputer of this world? Has not God made foolish the wisdom of the world? For after that
in the wisdom of God the world by wisdom knew not God, it pleased God by the foolishness
of preaching to save them that believe. For the Jews require a sign, and the Greeks seek after
wisdom, but we preach Christ crucified, unto the Jews a stumblingblock, and unto the Greeks
foolishness. But unto them who are called, both Jews and Greeks, Christ the power of God,
and the wisdom of God. Because the foolishness of God is wiser than men; and the weakness
of God is stronger than men. For you see your calling, brethren, how that not many wise men
after the flesh, not many mighty, not many noble, are called. But God has chosen the foolish
things of the world to confound the wise, and God has chosen the weak things of the world to
confound the things that are mighty; and the base things of the world that are despised has
God chosen, yes, and things that are not, to bring to nothing the things that are: that no flesh
should glory in his presence (1:18-29).

Ever since Paul penned this statement, Bible believers have used it to justify every form of
ignorance imaginable. If anyone dares suggest that the idea of a human sacrifice for the
vicarious atonement of the sins of mankind is ridiculous, the Christian will merely see this as
proof that his belief is right. "Yes," he will say, "that is exactly what Paul said. It pleased God
to save the world through the foolishness of preaching. You can't understand it because you
seek after worldly wisdom, but God chose the foolish things of the world to confound the
wise, and this is exactly what has happened to you. You have allowed the wisdom of the
world to blind you to the truth. God's wisdom is greater than man's wisdom, and when you
understand that, you will understand why the sacrifice of his son for the sins of mankind was
necessary."

Volume 1990 - 2002 Issue


Page 516 of 2049
Skeptical Review Edited by Farrell Till
Yeah, right. And if the Christians who parrot such nonsense as this ever learn to recognize
circular reasoning, they might begin to see the foolishness in their own "wisdom." With that
type of reasoning, one could justify any kind of belief. Flat-earthers--and believe it or not,
there still are some--dismiss all scientific evidence for the rotundity of the earth as just "the
wisdom of the world." Creation-scientists (an oxymoron if ever there was one) reject as
"man's wisdom" all scientific findings that dispute their young-earth, creationist views. Just
cite any kind of scientific or scholarly information that conflicts with what the Bible teaches,
and bibliolaters will pooh-pooh it as "the wisdom of the world."

In the "Mailbag" column of this issue (p. 12), we are publishing the letter of a subscriber in
Texas, who boasts of having a faith that is stronger than ever after having read "all issues of
TSR, plus other publications, debates, etc. of Mr. Till and other skeptics, current and past."
Readers will see that he takes refuge in the wisdom-of-the-world bastion of Bible
fundamentalism. He dismisses Till and the other skeptics who write for TSR as
"wisdomolaters," who have "sold their souls for a mess of pottage--man's wisdom." A detailed
response follows this letter in the "Mailbag" column, so here we will merely note that history
has proven repeatedly that this "mess of pottage" that skeptics have sold their souls for is
superior to the "wisdom of God" that bibliolaters have put their trust in. Presumably by the
wisdom of God, Jesus attacked human illness by casting out devils, but the wisdom of the
world invented microscopes, discovered microbes and viruses, and then conquered diseases
with vaccines and drugs. The wisdom of God rebuked Galileo, but the wisdom of the world
has long confirmed that he was right about the heliocentric nature of our solar system. The
wisdom of the world discovered that smallpox could be prevented by vaccination, but through
the wisdom of God, preachers opposed it as witchcraft and the work of Satan. Whenever the
wisdom of the world has clashed with the "wisdom of God," the wisdom of the world has had
a consistent way of proving itself right.

So let bibliolaters scoff at the "wisdom" of the world all that they want to. In this controversy,
we will put our trust in the side with the better track record. Flat-earthers, for example, are
absolutely right when they say that the Bible teaches a flat-earth cosmology, but one would
have to be hopelessly naive to accept that view over the compelling scientific evidence that
proves the earth is spherical. Bible fundamentalists are also right when they say that the Bible
teaches that life on earth resulted from acts of special creation that God performed over a
period of six days about 6,000 years ago, but geology, archaeology, paleontology,
microbiology, chemistry, astronomy, and various other branches of science indicate an
entirely different scenario. According to Richard Dawkins, whose scientific credentials are
known worldwide, "Darwin's theory is now supported by all the available relevant evidence,
and its truth is not doubted by any serious modern biologist" ("The Necessity of Darwinism,"
New Scientist, April 15, 1982, p. 130). To Bible fundamentalists, however, this is merely "the
wisdom of the world" speaking, and no matter how many world-class scientists like Dawkins
say that Darwin's theory is supported by all the available scientific evidence, they are going to
continue pressing to have their nonsense taught as "science" in our public schools. Ron
Patterson pegged them right when he said, "So-called scientific creationism is really nothing
more than an attempt to give credence to an ancient Hebrew myth by trying to prove that
virtually all the world's biologists, geologists, and paleontologists are a bunch of incompetent
buffoons" (The Freethought Exchange, September/October 1994, p. 50).

Volume 1990 - 2002 Issue


Page 517 of 2049
Skeptical Review Edited by Farrell Till
The right course to take in controversies like these can easily be determined by application of
a principle that Bob Hypes stated in a recent TSR article: "No reasonable person can believe
that the guesses of preliterate man, upon which the myths of gods and the supernatural are
based, were true" ("Religion and How I Lost It," Winter 1995, pp. 11).

This, of course, is a difficult worldview for Bible fundamentalist to accept, but perhaps they
should be more attentive to something that the apostle Paul said in the passage quoted earlier:
"For you see your calling, brethren, how that not many wise men after the flesh... are called"
(1 Cor. 1:26). Sociological studies have confirmed the truth of what Paul said here. The more
educated people are, the less likely they are to practice orthodox religion. Religious
orthodoxy, therefore, depends on ignorance. Many beliefs of today's most radical Bible
fundamentalists would have been considered rank heresy by the most educated Christians of a
thousand years ago. Knowledge caused the change and will continue to change religious
thinking, because not even the most fanatical Bible fundamentalist can live in our scientific
era without absorbing at least some of the "wisdom of the world" that is so despised in
fundamentalist circles.

Meanwhile, knowing the threat that knowledge poses to them and the institutions from which
they derive their livelihood, clergymen will continue to ridicule the "wisdom of the world" in
order to cultivate the colossal ignorance that is necessary for fundamentalist religion to
survive.

More Trouble for the Perfect-Harmony


Theory
Farrell Till
The Bible is so perfectly harmonious from cover to cover that only divine inspiration can
explain its unity. You don't believe it? Well, just ask any Bible fundamentalist, and he will
assure you that it's true.

Critical works of the past two centuries have shot the perfect-harmony theory so full of holes
that by now it should be lying rusted out at the bottom of an ocean of biblical scholarship.
Instead, Christian fundamentalists continue to proclaim to gullible pulpit audiences that there
are no contradictions or inconsistencies in the Bible. As we have shown repeatedly in past
issues of The Skeptical Review, this claim is patently false. Let's take as an example the fact
that the Bible plainly teaches that God is no respecter of persons: "(F)or there is no respect of
persons with God" (Rom. 2:14). Acts 10:34, Ephesians 6:9, Colossians 3:25, and 1 Peter 1:17
all claim that God judges all men fairly without respect of person. In boasting of having stood
his ground against the pillars of the Jerusalem church who wanted to force Titus to submit to
circumcision, the apostle Paul said that the positions of prominence held by his opponents in
the dispute didn't matter to him, because "God shows personal favoritism to no man" (Gal.

Volume 1990 - 2002 Issue


Page 518 of 2049
Skeptical Review Edited by Farrell Till
2:6, NKJV). There is no doubt, then, that the Bible teaches that God is impartial toward all
men.

Well, okay, let's see how consistent the Bible is in presenting God as an impartial deity. We
could begin by pointing out that God at one time favored an entire nation, because he selected
the Israelites to be his chosen people "above all peoples that are upon the face of the earth"
(Dt. 7:6). That certainly sounds like favoritism to me. If a teacher should select Bobby to be
her chosen student above all students that are in the class and even proclaim to the world that
she had done so, who would argue that she was not showing favoritism?

Inerrantists are fond of arguing that God had a plan of redemption for mankind that required
him to select a special people. Bible fundamentalists constantly use this marvelous "plan of
redemption" to cover a multitude of divine shortcomings, and they apparently can't see that an
omniscient, omnipotent deity would not have been required to select a plan of redemption that
necessitated racial favoritism, because such a deity could have redeemed mankind in any one
of several ways that would not have entailed racial favoritism and the various atrocities
committed against the non-Hebraic people of biblical times. To argue otherwise is to argue
that God is not omniscient and omnipotent. At any rate, this is the quibble that inerrancy
defenders resort to in this matter, so I'll just let the readers evaluate the merits of it so that we
can go on to other examples of divine favoritism that will give the inerrantists plenty more to
cavil about.

We could cite the case of Aaron's sons being ordained as priests of Israel (Num. 3:2-3) when
God's law specifically banned bastards and their descendants from entering the assembly of
Yahweh for at least ten generations (Dt. 23:2). Aaron's wife, as noted in an earlier article ("No
Bastards Allowed," Spring 1994, pp. 7,12,16), was a fourth-generation descendant of the
bastard son of Judah, so her sons would have been fifth-generation descendants. Nevertheless,
they were ordained as priests (so says the inerrant Bible) by specific instructions from
Yahweh himself (Num. 3:5-10). Bastards and their descendants for at least ten generations
were to be excluded from Yahweh's assembly, yet the fifth-generation bastard sons of an
important Hebrew official were not only made exceptions but were appointed to important
positions of leadership in the assembly. This certainly seems like favoritism.

At the Portland, Texas, debate last May, Marion Fox, Jerry Moffitt's moderator, announced to
the audience that he was going to write an article to show that Deuteronomy 23:2 is not
inconsistent with the appointment of Aaron's sons to the priesthood. I'm still waiting for that
article. Fox made a smart play to the gallery to get Moffitt out of a jam, but now it's time for
him to make good his promise. In Numbers 12:1, Miriam and Aaron spoke out in public
opposition to their brother Moses, who had married a Cushite (Ethiopian) woman. They had
every right to make this complaint, because Yahweh had forbidden the Hebrews to marry
foreigners (Ex. 34:15-16; Dt. 7:3). However, rather than taking Moses to task for breaking
this command, Yahweh made Miriam and Aaron the heavies, and especially Miriam. Yahweh
came down in a "pillar of cloud" and stood in the door of the tabernacle (that must have been
a sight to behold) to announce in no uncertain terms that Moses was his prophet to whom he
spoke face to face, so how dare they speak against his servant Moses (vv:4-8). Yahweh's
anger had been so aroused that when he departed, Miriam was left standing white as snow,
stricken with leprosy (v:10).

Volume 1990 - 2002 Issue


Page 519 of 2049
Skeptical Review Edited by Farrell Till
Well, okay, if Miriam had challenged the leadership of Yahweh's prophet whom he spoke
with face to face, then by all means teach her a good lesson; go ahead and afflict her with
leprosy. But why not afflict Aaron too? He was in on the rebellion as much as Miriam was.
Read the passage and see for yourself that both of them had complained about Moses'
marriage to the Cushite woman. So why wasn't Aaron afflicted with leprosy too? Whatever
happened to this wonderful impartiality that the Bible attributed to Yahweh, the god of gods?
For that matter, whatever happened to the wonderful harmony and unity that is supposed to
permeate the Bible?

Well, in the matter of Yahweh's alleged impartiality, there is no unity at all in the Bible. King
Ahab and his wife Jezebel were so notoriously wicked that their names have become
synonyms for evil. When Jezebel conspired to have Naboth murdered so that Ahab could
have his vineyard (1 Kings 21:1-16), Yahweh was so outraged when Ahab took possession of
the vineyard that he sent the prophet Elijah to denounce the deed and pronounce a sentence of
death and destruction upon the whole house of Ahab (vv:17-24). When Ahab heard the curse,
he rent his clothes, put on sackcloth, and fasted (v:27), after which Yahweh said to Elijah,
"See how Ahab has humbled himself before Me? Because he has humbled himself before Me,
I will not bring the calamity in his days. In the days of his son I will bring the calamity on his
house" (vv:28-29).

Whoa, wait a minute! Ahab was the culprit here, but because he put on sackcloth and fasted,
Yahweh decided not to destroy his family but to wait until Ahab was dead and then do it "in
the days of his son." Where's the fairness in that? Is this Yahweh's idea of impartiality? If
Yahweh had rewarded Ahab's repentance by simply forgetting the whole matter, that would
have been commendable and quite compatible with the character of an omnibenevolent god,
but to let the guilty live and later punish his son, that's downright tacky. We might also
mention that it flagrantly contradicts Yahweh's decree that said the son would not bear the
iniquity of the father (Ezek. 18:20). One just doesn't have to look very hard at all to find holes
in the perfect-harmony theory.

Yahweh's law decreed death for the sin of adultery: "The man who commits adultery with
another man's wife, he who commits adultery with his neighbor's wife, the adulterer and the
adulteress, shall surely be put to death" (Lev. 20:10). Now that's plain enough that anyone
should understand it. Certainly, an omniscient deity should have no difficulty understanding
what he had meant when he "verbally" inspired the writing of this law, yet we learn that
Yahweh apparently did have problems understanding it when David committed adultery with
Bathsheba, the wife of Uriah the Hittite (2 Sam. 11:2-5). No sentence of death was executed
against David, not even after he had murdered Bathsheba's husband Uriah (vv:21), another
offense for which Yahweh's divine law demanded the death penalty: "But if a man acts with
premeditation against his neighbor, to kill him by treachery, you shall take him from My altar,
that he may die" (Ex. 21:14). In cases of murder by treachery, then, one could not even seek
sanctuary in the tabernacle by clinging to the altar (see 2 Kings 1:50-51), but if ever there was
a case of "murder by treachery," David was guilty of it. He sent written orders for his
commanding general to put Uriah (Bathsheba's husband) in "the forefront of the hottest
battle" and then to withdraw the other soldiers and leave Uriah to die (vv:14-17). So if
adultery was a capital offense, surely David had deserved to die far more than some poor sap
who in a moment of ardent indiscretion had had the misfortune to be caught in a

Volume 1990 - 2002 Issue


Page 520 of 2049
Skeptical Review Edited by Farrell Till
compromising position with his neighbor's wife. We can well imagine that such as this had
happened many times in the primitive eye-for-an-eye society of the ancient Hebrews, yet no
death sentence was pronounced on King David for a far more flagrant case of adultery.
According to the story, while walking on the palace roof, he saw Bathsheba bathing, inquired
about her, and sent messengers to bring her to him, so this was no moment-of-weakness
indiscretion. It was a flagrant, premeditated case of adultery, but, for reasons presumably
known only to the inscrutable Yahweh, the death penalty was waived for David--and for
Bathsheba too, whom David added to his harem after her husband had been put out of the
picture.

To those who might try to argue that Yahweh was not to blame for waiving the penalty
demanded by the law in this case but the religious leaders who lacked the courage to apply the
law to their king, I will simply urge them to read the story again. After Bathsheba's mourning
was over (and it was downright decent of her to go through a proper period of mourning),
David sent for her, brought her to his house, and married her. When all of this was done,
Yahweh "sent Nathan [the prophet] to David" (2 Sam. 12:1). Why? To tell David that he had
broken divine law and so now he would have to pay with his life? Hardly. "Why have you
despised the commandment of Yahweh to do evil in His sight?" Nathan asked. "You have
killed Uriah the Hittite with the sword; you have taken his wife to be your wife, and have
killed him with the sword of the people of Ammon." There is no doubt that Yahweh
considered David guilty of murder, because the prophet that he sent to David plainly said,
"You have killed Uriah the Hittite." So why wasn't David sentenced to death as the law of
Yahweh had commanded? According to that law, David and Bathsheba both should have
been executed. Instead, Yahweh let them both off and, in clear violation of his decree that
"the wickedness of the wicked shall be upon him" (Ezek. 18:20; Dt. 24:16), proceeded to
pronounce punishment upon innocent parties for the sin of David and Bathsheba. The divine
message that Nathan delivered to David contained this curious provision:

Thus saith Yahweh: "Behold, I will raise up adversity against you from your own house; and I
will take your wives before your eyes and give them to your neighbor, and he shall be with
your wives in the sight of this sun. For you did it secretly, but I will do this thing before all
Israel, before the sun (2 Sam. 12:11-12).

Now besides this not being the punishment that Yahweh's perfect law had decreed for
adultery, it was a punishment directed against others rather than David and Bathsheba, who
were the guilty parties. In other words, Yahweh was saying to David, "You committed
adultery, and now I am going to subject your wives to the humiliation of rape." Admittedly,
David would have suffered embarrassment and humiliation if this decree had been executed,
but that would not have compared to the humiliation that his wives would have experienced.
How could the mind of a fair and impartial god "who shows personal favoritism to no man"
have conceived of such a scheme as this?

The rape of David's wives never occurred, of course, because David repented. "I have sinned
against Yahweh," he said in response to Nathan's message (v:13), and apparently that was all
it took to get him off the hook. "Yahweh also has put away your sin," Nathan said. "You shall
not die." Here is the only suggestion in the story that Yahweh had at any time considered
imposing the full penalty of his law for David's offense, and all that it had taken to obtain

Volume 1990 - 2002 Issue


Page 521 of 2049
Skeptical Review Edited by Farrell Till
divine pardon were five little words: "I have sinned against Yahweh." We can only wonder
how many adulterers before David had also said, "I have sinned against Yahweh," before the
stones had started pelting them. If repentence had not earned them Yahweh's mercy, then by
what twist of logic can anyone argue that sparing David's life for the same offense was not a
flagrant case of Yahwistic favoritism?

This story didn't end with Nathan's telling David that Yahweh had put away his sin, because it
just didn't seem to be in the nature of Yahweh to let a thing like this pass without showing his
temper in some way. However, instead of punishing the offenders, he chose instead to punish
the child conceived during their adulterous relationship. "(B)ecause by this deed you have
given great occasion to the enemies of Yahweh to blaspheme," Nathan said to David, "the
child also who is born to you shall surely die" (2 Sam. 12:14). So David and Bathsheba had
violated a divine law that decreed death, but rather than killing either David or Bathsheba,
Yahweh chose instead to kill their child. We will admit that such a punishment would have
inflicted considerable grief on David and Bathsheba, but that is beside the point. They were
the ones who had violated Yahweh's law, but an innocent baby was the one who paid with his
life for an offense committed by others. It's so... well, so typically Yahwistic.

We see the appalling partiality in Yahweh's character when we compare the ease with which
he forgave David and Ahab to the harsh judgment that he pronounced on others, who were far
more deserving of mercy. All David and Ahab had to do was say, "I have sinned," and
Yahweh immediately forgave them and then later punished their offspring. As we saw earlier
in "Another Flaw in the Perfect-Harmony Theory" (Spring 1994, pp. 10-11), such was not the
case when Josiah repented and instituted a religious reformation that was unparalleled in
Israel's history. Like Ahab, Josiah " rent his clothes" in a gesture of penitence (2 Kings
21:11), but unlike the generosity that Yahweh showed to Ahab, he did not show mercy to
Josiah. Even after Josiah had eradicated idolatry in the kingdom, destroyed all the pagan
altars, and celebrated a Passover the likes of which "surely had never been held since the days
of the judges who judged Israel, nor in all the days of the kings of Israel and the kings of
Judah" (vv:21-22), Yahweh still "did not turn from the fierceness of His great wrath, with
which His anger was aroused against Judah" (2 Kings 23:26). One would have to be
completely devoid of intellectual objectivity to argue that these are not inconsistencies in the
biblical depiction of Yahweh's character.

David, of course, was no stranger to the appalling unfairness that Yahweh often demonstrated
in the treatment of his "chosen people." In 2 Samuel 24:1-9, David ordered a census of Israel
that for some reason greatly offended Yahweh. In this case, Yahweh sent the prophet Gad to
give David his choice of three punishments: (1) seven years of famine, (2) three months of
flight while his enemies pursued him, or (3) three days of plague in the land (vv:11-13). A
parallel account of this story in Yahweh's perfectly harmonious word (1 Chron. 21:12) has
Gad offering David three years of famine rather than seven, but we will let this inconsistency
pass for the moment to make a point about Yahweh's idea of impartiality.

Eventually Yahweh settled on three days of plague in the land as a way of punishing David
for presuming to number Israel (for whatever reason this was so horribly sinful). As biblical
plagues go, it was typically Yahwistic in its scope. "From Dan to Beersheba, seventy
thousand men of the people died" (v:15). Now let's try to put this story into perspective. For

Volume 1990 - 2002 Issue


Page 522 of 2049
Skeptical Review Edited by Farrell Till
some reason known only to the inscrutable Yahweh, David sinned by ordering a census of
Israel, and as punishment for the sin, Yahweh sent a plague that killed seventy thousand men.
Meanwhile, not a hair on David's head was touched. Is this what inerrantists call impartiality?
Apparently so, because in the face of passages like this, they will still argue that the Bible is
perfectly harmonious from cover to cover. Don't try to figure it out, because you won't.

The absurdity of their position is underscored by the fact that not even David would agree
with them, for after Yahweh had finally relented just short of allowing his death angel to
destroy Jerusalem (v:16), David, who had watched the angel striking the people, spoke to
Yahweh and said, "Surely I have sinned, and I have done wickedly; but these sheep, what
have they done?" (v:17).

That's a good question, and inerrantists should give it some serious thought. David was the
one who had sinned in this matter, but Yahweh killed 70,000 others for something they had
not done. What is the fairness of that? By what stretch of imagination, can anyone say that a
silly yarn like this is perfectly harmonious with the biblical passages that say "God shows
personal favoritism to no man"?

This story obviously conflicts with Yahweh's law that decreed individual responsibility and
punishment for one's own sins (Ezek. 18:20; Dt. 24:16), and even David was apparently able
to see the injustice of it. After asking Yahweh, "What have these sheep done?" he went on to
say, "Let your hand, I pray, be against me and against my father's house" (v:17).

Well, it seems that even David's understanding of justice was a little tarnished too, for even he
thought that it would have been proper to punish his father's house for the sin he had
committed. Nevertheless, the point is crystal clear: the Bible teaches an abysmally
inconsistent concept of fairness and impartiality in the character of the Hebrew god Yahweh.
This character flaw was effectively verbalized in a question that Genesis 18:25 attributes to
the patriarch Abraham. According to the story, Yahweh had informed Abraham that he was
going to destroy Sodom, upon which Abraham asked, "Would you also destroy the righteous
with the wicked?" In other words, Abraham recognized the truth that we are focusing on in
this article: it is inherently unfair to punish innocent people.

As the story continued, Abraham attempted to bargain with Yahweh in an effort to save the
city. He asked Yahweh if he would spare Sodom if 50 righteous people could be found in it.
"Far be it from You to do such a thing as this," Abraham said, "so that the righteous should be
as the wicked; far be it from You!" Obviously, then, Abraham recognized that it would be
fundamentally unjust to kill everyone in the city, the righteous along with the unrighteous.
"Shall not the Judge of all the earth do right?" he asked Yahweh. Today, Bible
fundamentalists lack the basic common sense that Abraham demonstrated in this story
(which, needless to say, is undoubtedly fictional), because they continue to defend the
appalling moral conduct that the Bible attributes to the god Yahweh. They do so, of course,
because they have to. Numerous biblical stories attribute utter moral depravity to their god
Yahweh, so they are stuck with the embarrassing task of having to defend them with the same
old tired arguments that have failed over and over again to explain the problem: "If God can
give life, then he has the right to take life." "Those heathen children and babies went to
heaven instead of growing up to be evil like their parents."

Volume 1990 - 2002 Issue


Page 523 of 2049
Skeptical Review Edited by Farrell Till
Is there any hope for mankind as long as presumably intelligent people sit in pews and
gullibly swallow such drivel as this?

Reprinted from Freethought Today...

Let Us Pray
Judith Hayes
Not only Christianity, but most religions, urge prayer. Prayer is a practice that cannot survive
the harsh spotlight of logic.

The three most often cited reasons for prayer are worship, confession, and petition.
(Differences exist. Choose your experts.)

The most popular form of prayer, petition, poses some complicated problems. At first glance,
asking a god to do something or other seems perfectly logical. Who better to ask? But the
only way that such asking makes sense is if there is a chance that you might receive a positive
response. What would be the point of having billions of prayers offered beseechingly to a god
who never intended at any time to answer a single one of them?

A more pointless, time-wasting, soul-draining exercise is difficult to imagine, and a god who
would demand such a practice would have to be sadistic. Such bait-and-switch tactics are
difficult to attribute to any god, even the one who sent the Flood. On the other hand, if prayer
is encouraged because there is a chance that requests will be granted, you run headlong into
the unavoidable requirement to explain the seemingly capricious nature of some of these
boons.

For example, a high-school student prays that he will pass a math exam even though he hasn't
studied for it, and when he does pass, he attributes this to God's intervention. Most religious
leaders would agree with this. (Differences exist. Choose your experts.) But if it is true, we
are faced with a god who answers a single petition from a single person in the matter of a
10th-grade algebra test, but who chose to ignore the millions of prayers for liberation from
concentration camps during World War II. There is a selection process at work here that is
extremely difficult to grasp.

According to the "Lord's Prayer," people are supposed to ask, "Give us this day our daily
bread." Why? If you ask, will it be done? If it won't be done, why should you ask? Since war
and famine have brought death by starvation to many True Believers, this asking for daily
bread seems pointless. If starvation happens to those who ask as well as to those who don't,
then the explanation for starvation must lie in factors wholly unrelated to the asking. In other
words, asking God for your daily bread has nothing to do with whether or not you'll get it. So
why are you supposed to ask for it?

Volume 1990 - 2002 Issue


Page 524 of 2049
Skeptical Review Edited by Farrell Till
Likewise, prayers of thanksgiving intrinsically impute to God complete control over your
well-being. If you thank God for the food on your table, you are saying that he put it there. A
necessary component of this premise, the other side of this coin, is that if there is no food on
your table, God is responsible for that, too. The power to give necessarily includes the power
to withhold. When you thank someone for a gift, it is because you understand that he or she
had the choice of not giving it to you but chose to do so anyway. Thanking God for your food,
then, is the same as saying thank you for not withholding food. You are offering thanks for
not being allowed to starve.

Just as it would make no sense to thank your neighbors for a much needed rain shower, since
they could not have played any role in producing the rain, so it would make no sense to thank
God for the food on your table unless he definitely plays a role in getting that food to your
table. And if he does, we are presented with the vexing question of just how he chooses to
feed some while starving others. If the choice to put food on your table is God's, then the
choice not to put food on someone else's table is also God's. So, then, why doesn't God feed
all of us?

Starving babies are an awkward consideration on Thanksgiving Day, as we sit down to


sumptuous turkey dinners, but if God puts the turkey on your table, he withholds it from
countless others. Why? If God feeds only "his own," that would mean that the babies of those
other than his own could starve without his caring, a heartless proposition. It would also mean
that his own have never starved, which is certainly not true. Nor can it be said that all atheists
starve

So how does God decide whom to feed? This question of God's priorities cannot be
sidestepped if his participation in daily events is posited. If God has the power to feed all of
us but chooses not to, his reluctance must be explained in a way that is compatible with his
purported omnipotence (all-powerfulness) and omnibe- nevolence (all-goodness). No one has
yet managed to proffer such an explanation.

Trying to explain starvation by saying that "God helps those who help themselves" is a cruel,
callous way to regard victims of crop failures from floods, drought, or pestilence. And what
about the babies? How can babies help themselves?

Likewise, trying to explain starvation by saying that we just can't understand the ways of God
is a contradiction of all the rest of Christian doctrine. Christians claim to know precisely how
God wants his "children" to worship, how they should pray, how they should dress, what they
should eat, how they should address their elders and so on, implying quite clearly that God's
ways are indeed understood. But questions about the terrible reality of starved-to-death babies
are met with vague shrugs as if such trivia did not need to be understood.

But someone must accept responsibility for the haunting specter of starving children. If food
production and distribution on this earth are solely the result of human activities, with no
participation by God, then giving thanks to God for food is a misplaced, meaningless gesture.
He has done nothing to deserve thanks, and we alone must answer for the cruel inequities. If,
on the other hand, God does participate in the process, then you should give thanks to him for
your chocolate bars and imported cheeses, and he has a lot of starving babies to answer for.

Volume 1990 - 2002 Issue


Page 525 of 2049
Skeptical Review Edited by Farrell Till
All this talk of starvation is of course representative of and interchangeable with all human
conditions. Whether you are considering illness, injury, persecution or whatever, if you pray
for deliverance from any of them, the results will be the same as with starvation--random and
inexplicable.

So let's consider again prayers of supplication. Ending world hunger, a most admirable
request, has yet to become a reality, in spite of countless prayers. So people are encouraged to
pray, instead, for more easily achieved goals, like having Aunt Helen get over her cold soon,
or for the kids to do well in school, or whatever. Football players actually get on their knees
and thank God for touchdowns. In a world that contains starvation, disease, murder, and rape,
such mundane considerations trivialize the role of a supposedly omnipotent god. For every
"miraculous" recovery by a seriously ill person that is attributed to God, there is a seriously ill
person who is prayed for but dies anyway. Soldiers are prayed for and die, and soldiers are
not prayed for but live. Bad things happen to good, prayed-for people, bad things happen to
bad people, good things happen to bad people and good things happen to good people. In
other words, the laws of probabilities are quite clearly in control here.

All things are not made well for those who trust in God, and life can be very pleasant for
those who do not. If judged only by the results that challenge the laws of probabilities, then
the power of prayer is nil.

(Judith Hayes is a granddaughter and great-granddaughter of Lutheran ministers. She became


a freethinker after reading the Bible as an adult, and she believes that a thorough reading of
the Bible is the surest path to atheism. Ms. Hayes is a regular columnist in Freethought
Today. Her articles alone, which usually examine religious doctrines and practices from a
logical point of view, are worth the subscription price. Freethought Today is published by the
Freedom From Religion Foundation, P. O. Box 750, Madison, WI 53701. Ms. Hayes' address
is P. O. Box 77, Valley Springs, CA 95252.)

Copyrights
The preceding article is copyrighted by Judith Hayes; however, she has permitted us to reprint
it under the same conditions that apply to all materials published in The Skeptical Review.
Any of our articles may be copied for local distribution or reproduced electronically for
computer bulletin boards provided that the names and addresses of the authors and the
original source of publication are included on all copies.

The I'm-Not-Certain-I-Exist Debate


Jerry Moffitt

Volume 1990 - 2002 Issue


Page 526 of 2049
Skeptical Review Edited by Farrell Till
I enjoyed reading Farrell Till's review of our debate, which he called "The Blind, Staggering,
Falling-Down-Drunk Luck Debate." Of course, you would expect that our view of what
happened would differ. He said I did not specifically identify prophecy fulfillment regarding
the Creator being the God of the Bible, that my probability arguments were silly, that my
quotes of scientists were out of context, and that I begged the question. My defense to all that
is simply this: "View the tapes; view the tapes." The videos can be purchased for $25 from
Thomas A. Gardner, P. O. Box 865, Hurst, TX 760543; (806) 282-2745.

But in his review of the debate, Mr. Till left out the most exciting part and that which some
said settled things for them. Below is a sequence of excerpts which make up in abbreviated
form an exchange between us. It started off when I displayed this chart containing a letter Mr.
Till wrote to me.

Existence, By Farrell Till


You seem to think that my mind is not made up, but for all intents and purposes it really is. I
am 99.9% sure that an infinite deity does not exist. However, I refuse to assert that I know
positively and absolutely (and couldn't possibly be wrong) that God does not exist. I wouldn't
even say that I am 100% sure and couldn't possibly be wrong that I exist... (Letter, Oct. 18,
1993).

The following are excerpts from the interrogation:

Moffitt: "Why do you have doubt that you exist?"

Till: "Uh, how do you or how do I know that, uh, my mind is not something that exists from,
uh, wiring into a computer that has been programmed to make me think that I exist?"

Moffitt: "Mr. Till, are you saying that maybe you are not really here?"

Till: "Jerry, I don't really believe that. I don't even come close to believing it."

Moffitt: "Then why did you say it to me in a letter?"

Till: "Because I said I am not one hundred percent sure!"

Moffitt: "Ok, ok, that's enough. I just wanted to show you the thinking of a man who does not
believe God exists."

As I said, the above is abbreviated. You need to view the tapes and see the whole exchange. I
want to make a proposal. Just to set things straight, I propose that Mr. Till and I have a brief
series of exchanges in TSR on the subject. Following are two propositions:

RESOLVED: "I may not exist."


(Affirm: Farrell Till)

Volume 1990 - 2002 Issue


Page 527 of 2049
Skeptical Review Edited by Farrell Till
RESOLVED: "Farrell Till Exists."
(Affirm: Jerry Moffitt)

I personally believe this issue must be decided before Mr. Till debates or edits his paper
anymore. As a debate opponent of Mr. Till's, I ask you how you would like to spend eight
long hours debating a man who actually may not have ever existed? Too, don't you feel
strange sending articles or letters to an editor who possibly is not even there?

Now toward the end of his article regarding my skill as a debater, Mr. Till said some of the
nicest things ever said to me by an opponent. He concluded by saying, "Fundamentalists who
watch the tapes will find much to applaud." It is all very flattering, but I cannot accept. I
sincerely believe Mr. Till recognizes that he had a problem in our debate and wants to get the
readers braced for it if they view the tapes. But if fundamentalists who watch the tapes of this
debate find anything to applaud, I sincerely believe it will be the evidence for the existence of
God so readily available and in such massive amounts that it can make even a below-average
debater look good.

(Jerry Moffitt, P. O. Box 1275, Portland, TX 78374.)

Yes, View the Tapes! View the Tapes!


Farrell Till
The day I received the preceding article from Jerry Moffitt, I wrote to tell him that I would
put it in the next available issue. I'm sure Mr. Moffitt took the time to write and submit the
article because he knew that it would be published. I didn't bother to send my review of the
debate to any Church-of-Christ publication, because I knew that it would have been a waste of
time and postage. In the interest of fairness, I hope Mr. Moffitt will urge the "gospel" papers
that he writes for to allow their subscribers to read his opponent's view of this debate, but I
really don't expect that to happen.

On one point, we both seem to be in agreement, because I too urge TSR readers to view the
video tapes of this debate. You won't even have to pay $25 to get them either. For $2, I will
send them to you on a two-week rental. When you watch them, you will see Mr. Moffitt
pursuing the usual theistic approach to "proving" the existence of God. He confuses the
results of billions of years of adaptation through natural selection with "design" and argues
that all of these marvels prove that everything was created. What you will not see in the
debate is any more than fleeting efforts on his part to explain why if "design" in nature can be
explained only through creation by a Great Designer the Great Designer himself would not
have required creation. To comment on how gingerly he tiptoed around this challenge would
take too much space, so view the tapes! View the tapes!

Volume 1990 - 2002 Issue


Page 528 of 2049
Skeptical Review Edited by Farrell Till
Neither did Moffitt try to prove that the "Great Designer" was Yahweh of the Hebrews, even
though our signed propositions obligated him to do so. The extent of his effort here was a
claim that biblical prophecy fulfillments exceeded reasonable probabilities, but he refused to
present in evidence even one example of verifiable prophecy fulfillment, even though I
challenged him several times to do so. View the tapes! View the tapes!

Now comes Mr. Moffitt to challenge me to debate an insignificant, not to mention irrelevant,
point that we discussed (at his instigation) probably no longer than two or three minutes. He
wants to debate whether I exist. The offer was made in jest, of course, but I am going to
surprise him and accept the challenge. (Never let it be said that I refused a debate challenge.)

Before the average TSR reader can understand why I answered Moffitt as I did during the
interrogation he has transcribed, a bit of background information is necessary. Jerry Moffitt is
a Church-of-Christ preacher, and Church-of-Christ preachers will look you right in the eyes
and say without blinking that they absolutely know that they are right in their religious
beliefs. They know (absolutely) that God exists. They know (absolutely) that this god who
exists is Yahweh of the Hebrews. They know (absolutely) that the Bible is the verbally
inspired word of God. They know (absolutely) that the Church of Christ is the one and only
true church, and they know (absolutely) that Catholics, Baptists, Methodists, Adventists,
Presbyterians, Pentecostals, and even liberal Church-of-Christ members are going straight to
hell. In other words, they just know absolutely.

As a former Church-of-Christ preacher, I am familiar with this belief, so when Moffitt asked
me if I know whether I exist, I knew that he was using the word in the sense of absolute
knowledge. Prior to the debate, he had insisted that I sign a proposition that would require me
to affirm that "I know God does not exist." I had refused, because that would have put me in
the position of affirming a universal negative. My letter that he quoted was simply my attempt
to explain why I couldn't agree to such a proposition. It would have required me to claim
absolute knowledge on the question of God's existence, and I lay no claim to absolute
knowledge on any subject. The readers will notice, however, that I did say "I am 99.9% sure
that an infinite deity does not exist," and that should have been enough certainty to satisfy
anyone except, of course, a Church-of-Christ preacher.

Well, can't I at least be 100% sure of my own existence? I don't think so. Dave Matson
addressed this issue in the Spring 1993 edition of TSR ("What Is Wrong with Any-Loophole-
Will-Do Hermeneutics," pp. 9-10), so I will simply refer readers to that article for a possible,
although not probable, reason why an individual can't be 100% sure that he exists. Anyone
familiar with transcendental philosophy--and some very intelligent people have accepted this
philosophy--know that transcendentalism teaches that material existence is only imaginary. I
wonder if Mr. Moffitt has heard of "virtual reality," a new technology that can take people
into imaginary worlds so realistic that participants gain the impression of being part of those
worlds. So how does Jerry Moffitt know that we are not just part of some virtual reality that
makes us think we are real? Can he prove absolutely that such a scenario just isn't at all
possible? If he claims that he can, what is his evidence? Let's see it.

Do I seriously believe that I am part of some virtual-reality world? No, not at all, but
remember that we are talking about 100% absolute certitude. Mr. Moffitt thought my

Volume 1990 - 2002 Issue


Page 529 of 2049
Skeptical Review Edited by Farrell Till
response to his question was absurd, but, unfortunately, he himself is unable to see the
absurdity in his own claim that he has acquired absolute certitude about the existence of God
and all the other accouterments of religion by reading a book that was written thousands of
years ago by semi-literate, prescientific, superstitious desert nomads who thought that the
creator of the universe lived in a tent that they carried around with them. If Moffitt wants to
contemplate absurdity, perhaps he should meditate on his own situation for a while.

Rather than debating the issue of my existence, I have a far more sensible proposal. Why
doesn't Mr. Moffitt deliver on his promise to debate me on the issue of prophecy fulfillment.
Those of you who do view the tapes of our debate will notice that when I was pressing Moffitt
to give specific proof that prophecy fulfillments prove that Yahweh is the god who created the
universe, he stood up and said that he would debate the prophecy issue with me in a written
format if I would agree to publish it, and I assured him that I would publish it. At the time, we
were already involved in a written debate on the general issue of Bible inerrancy, although
Moffitt had not submitted a manuscript to me in over two years. The tapes will show that he
promised, in the presence of everyone in attendance, that he would arrange his work schedule
so that he could complete the debate in progress and begin the other. Well, guess what? That
was almost a year ago, and I have yet to receive anything from Moffitt except the foregoing
article. He has not resumed the inerrancy debate, so we are nowhere close to beginning the
prophecy debate.

I might also say that Moffitt's moderator, Marion Fox, who assured the audience that he was
going to write a response to my article "No Bastards Allowed" (Spring 1994, pp. 7, 12, 16)
hasn't kept his promise either. These fundamentalist preachers are quick to say the things they
know will have strong audience appeal, but they aren't at all eager to keep their word when
there is no gallery for them to play to. I'll offer Jerry Moffitt a deal. If he will agree to an oral
debate on the prophecy issue in Portland, Texas, at a mutually agreeable date, I will let him
off the hook on his commitments to the written debates. Meanwhile, I suggest that he forget
about debating whether I exist so that we can spend our time on more serious matters about
which we are in substantial disagreement.

In response to a letter from me in which I suggested that we debate the issue of God's
existence again, Moffitt informed me that there is no need for another debate, because I was
"soundly whipped" in our first one. Somehow, his logic eludes me, because if he truly
believes that he soundly whipped me in the other debate, I would think that he would be eager
to do it again to demonstrate to another audience that the truth of his god-belief simply cannot
be impeached. I honestly believe that I defeated Moffitt's principal arguments in our debate,
and that is exactly why I would like to debate him again before another audience. I want as
many people as possible to see that belief in the existence of the Hebrew god Yahweh is an
absurdity that cannot be successfully defended. An old adage says that actions speak louder
than words, so I have to believe that Moffitt's reluctance to act means that he isn't half as sure
of his position as his words indicate.

Moffitt did say that if I could guarantee him an "audience of skeptics and atheists," he would
give another debate his prayerful consideration, so perhaps readers who are members of
freethought groups would like to invite us to debate in a setting where Moffitt could realize
his wish. If such a debate should ever occur, I predict that the harsh critical analysis to which

Volume 1990 - 2002 Issue


Page 530 of 2049
Skeptical Review Edited by Farrell Till
the audience subjects his ridiculous design and probability arguments will very quickly make
him wish he were back in Portland speaking to a partisan audience that would greet just about
anything he said with "oohs" and "ahs" of uncritical admiration.

At any rate, Moffitt has indicated that he will consider a return engagement if it is held in a
skeptical or even an academic setting, so I hope interested readers will make inquiries in their
communities so that we can possibly give Mr. Moffitt something to pray about. Personally, I
predict that there will be no more oral debates between me and Jerry Moffitt and probably no
written ones either, but I hope I'm wrong. He represents a religious view that is detrimental to
public welfare and the progress of humanity, and it needs to be publicly opposed.

What Was a "Mamzer"?


Farrell Till
Despite various challenges we have issued for someone to write an article in response to "No
Bastards Allowed" (TSR, Spring 1994, pp. 7, 12, 16), no one has yet accepted the challenge.
This article focused on problems for the Bible inerrancy doctrine raised by Deuteronomy
23:2, which prohibited bastards from entering the assembly of Yahweh "even to the tenth
generation." A central point of the article focused on the issue of Aaron's sons, whom Yahweh
ordained to serve as priests in his assembly (Num. 3:1-3), even though they were fifth-
generation descendants of the bastard Perez through their mother Elisheba (Gen. 38:24-30,
Ruth 4:18; Ex. 6:23). At my debate with Jerry Moffitt last May, Marion Fox, Moffitt's
moderator, told the audience that he would write an article that would present some "good
answers" to the problem, but we have yet to hear from Mr. Fox. With so many "good
answers" at his disposal, we have to wonder why he wouldn't have submitted his article by
now. Perhaps he, like Lindell Mitchell, doesn't want to humiliate me publicly.

Despite the loud silence on this issue from those we have directly challenged to explain why
Deuteronomy 23:2 doesn't destroy the myth of Bible inerrancy, we did receive two letters
from subscribers who proposed similar solutions to the problem. Their explanation centered
on the meaning of the Hebrew word mamzer, which is translated "bastard" in this verse.
According to both letter-writers, the word didn't mean "bastard" in the sense that we assign
the word in English. A mamzer, we were assured, was someone "born of a forbidden
marriage."

Both letter-writers cited two authorities in support of their position, Strong's Concordance and
the NIV Bible. Strong states that mamzer is "from an unused root, meaning to alienate; a
mongrel, i.e., born of a Jewish father and a heathen mother:--bastard," and the NIV renders
Deuteronomy 23:2 like this: "No one born of a forbidden marriage nor any of his descendants
may enter the assembly of the LORD, even down to the tenth generation." Hence, the letter-
writers concluded, only those born of forbidden marriages were banned from the assembly
and not "bastards" in the English sense of the word. Both Elisheba and Aaron were of Jewish

Volume 1990 - 2002 Issue


Page 531 of 2049
Skeptical Review Edited by Farrell Till
descent; therefore, Elisheba's sons were not mamzers and were qualified to enter into the
assembly of Yahweh.

Obviously, a lot depends upon the meaning of mamzer, so we need to take a careful look at
the word. It appears only twice in the Old Testament (Dt. 23:2 and Zech. 9:6), and both times
it is translated bastard in the KJV and ASV. Most other versions translate it bastard or
equivalent in Deuteronomy 23:2, but some of them give it the sense of "strangers," or
"mongrels" in Zechariah 9:6. In defining "bastard," Eerdmans Bible Dictionary also indicated
a degree of uncertainty:

A name given to those begotten in adultery or incest (Heb. mamzer, Deut. 23:2; NIV "born of
a forbidden marriage"). This violation of marriage was such a serious offense that such
persons and their descendants were denied admission to the assembly of the LORD, first in
the temple and later in the synagogue, to the "tenth generation" (Deut. 23:2)--i.e., forever. At
Zech 9:6 "a mongrel people" [KJV, JB "bastard"; NIV "foreigners"] refers to a nation of
mixed population (1987, p. 129).
Scriber's Dictionary of the Bible takes the position that mamzer "means a child of incest, not
simply an illegitimate child" (Vol. 1, 1923, p. 257).

So what did the Hebrew word mamzer mean? Scholarship offers us three choices: a person
born of adultery, a person born of incest, or a person born of a forbidden or mixed marriage.
Unfortunately for our inerrantist friends, we can give them their choice of definitions and it
won't matter, because all of the definitions pose serious problems for the inerrancy doctrine.
Elsewhere in this issue, my article "More Trouble for the Perfect-Harmony Theory" shows
serious inconsistency between the biblical claim that God doesn't show partiality to
individuals and several passages that indicate he did on various occasions show partiality. As
an example of partiality, I noted the very passage that we are now discussing--Deuteronomy
23:2. If an individual were banned from the assembly because of a circumstance of birth, it
really wouldn't matter whether the person being banned had been born of adultery, incest, or a
"forbidden (mixed) marriage." Any one of these reasons would make Yahweh guilty of
showing partiality, because people simply cannot help the circumstances of their birth. If they
are born of adultery, incest, or mixed marriages, there is nothing they can do about it. So what
business does an infinitely impartial god have banning people from his assembly because of
circumstances related to birth? This is something inerrantists need to explain before any other
problems related to Deuteronomy 23:2 are even considered.

Something else they should try to explain is why an omniscient, omnipotent deity would
choose to reveal his divine truths to mankind in an "inspired" book that no one can be sure of
understanding. What does mamzer mean? If the meaning of the word is indeed obscure, this
would be just the tip of an iceberg of linguistic confusion that one will encounter in studying
the Bible. One may as well ask what was meant in Leviticus 6:21 or 1 Samuel 13:21 or
Jeremiah 11:15 or Proverbs 22:20. These are just some of many Old Testament passages
where detailed reference Bibles append footnotes to explain that some words in the Hebrew
text are too obscure for translators to be sure of the meaning. In countless other cases, even
when literal word-meanings are known, the language of the Bible is so vaguely and abstractly
written that no one can be sure what the writers meant. Consider, for example, this statement
from the prophet Ezekiel:

Volume 1990 - 2002 Issue


Page 532 of 2049
Skeptical Review Edited by Farrell Till
Then I looked , and behold, a whirlwind was coming out of the north, a great cloud with
raging fire engulfing itself; and brightness was all around it and radiating out of its midst like
the color of amber, out of the midst of the fire. Also from within it came the likeness of four
living creatures. And this was their appearance: they had the likeness of a man. Each one had
four faces, and each one had four wings. Their legs were straight, and the soles of their feet
were like the soles of calves' feet. They sparkled like the color of burnished bronze. The hands
of a man were under their wings on their four sides; and each of the four had faces and wings.
Their wings touched one another. The creatures did not turn when they went, but each one
went straight forward... (1:4-9).
This vision of the four living creatures goes on for three chapters--in fact, it's questionable
where this vision ended and the next one began--and no one really knows what any of it
meant. Such stuff as this has provided an endless supply of sermon fodder for preachers on
cable TV who think they see signs of the end in current events--and no two ever see exactly
the same thing.

In reading such passages as this, one wonders what the writers were smoking or drinking
when they wrote them, and we have every right to ask if this is the best job of verbal
inspiration that an omniscient, omnipotent deity could do. Whoever wrote Deuteronomy 23:2
had to mean something by the words he used, so if he were truly inspired by an omniscient,
omnipotent deity, why couldn't he have said what he meant in terms so clear that no one could
have misunderstood him?

The problems of divine partiality and obscurity in an inspired text aside, the statement in
Deuteronomy 23:2 is incompatible with the biblical inerrancy claim regardless of which of
the three previously mentioned meanings is given to the word mamzer. If the writer meant
bastard in the sense of someone born of an adulterous relationship, then unquestionably, the
appointment of Aaron's sons to the priesthood was inconsistent with Yahweh's decree that
"bastards" would be banned from the assembly "even unto ten generations." That point has
already been established, so there is no need to rehash it.

To claim that a mamzer was the offspring of an incestuous relationship will not remove the
problem with respect to Aaron's sons, because Perez, the great-great-grandfather of Aaron's
wife Elisheba, was, according to Jewish law, born of an incestuous relationship. Tamar, the
mother of Perez, was the daughter-in-law of Judah, the father of Perez, and Leviticus 20:12
specifically prohibited "lying" with one's daughter-in-law, as did also Leviticus 18:15. So if
Perez was not a mamzer in the sense of mere illegitimacy, he was a mamzer in the sense of
having been born of an incestuous relationship.

Inerrantists can't take refuge in the third definition of mamzer either, for if the offspring of
mixed marriages was what the writer meant in Deuteronomy 23:2, the problem of
inconsistency will merely shift from Aaron's sons to David, the Israelite king who was a man
after Yahweh's own heart (1 Sam. 13:14). Ruth, David's great-grandmother, was a Moabitess
(Ruth 4:13-22), so if a mamzer was the offspring of a mixed marriage, then David's
grandfather Obed was a mamzer, and David, only the second-generation descendant of a
mamzer should have been banned from the assembly. Obviously, though, David wasn't
banned from the assembly, because the last seven chapters of 1 Chronicles describes activities
in the assembly in which David participated.

Volume 1990 - 2002 Issue


Page 533 of 2049
Skeptical Review Edited by Farrell Till
Furthermore, if a mamzer was indeed the offspring of a mixed marriage, then David's
parentage was a double problem for him. Not only would the restriction on descendants of
mamzers have disqualified him from entering the assembly, but the same passage specified
that Moabites and their descendants "shall not enter into the assembly of Yahweh, even to the
tenth generation" (Dt. 23:3). So poor David would have had a double whammy pronounced
on him. However, the fact that neither of these restrictions was applied to David simply shows
how inconsistent the biblical text is.

Sometimes I almost feel sorry for inerrantists. They dig and dig to try to get out of the hole
they are in only to find themselves in a deeper one after they have presented all of their how-
it-could-have-been scenarios.

Still Chewing: Chasing Rabbits With Dr.


Geisler
Jeffrey A. Justice
Even though I addressed a similar argument by Gleason Archer in my article, "Chew on This .
. . Again!" (TSR, Autumn 1994, pp. 8-11), it will be worthwhile to discuss a more elaborate
variation of Archer's argument that Dr. Norman Geisler made in his popular fundamentalist
reference book When Critics Ask. So here is some more "food for thought" to chew on
concerning the biological error of Leviticus 11:5-6. First, let's look at Geisler's version of the
argument:

Leviticus 11:5-6, How can the Bible say that the hyrax and the rabbit chew the cud when
science now knows that they do not?...

Although they did not chew the cud in the modern technical sense, they did engage in a
chewing action that looked the same to the observer. Thus, they are listed with other animals
that chew the cud so that the common person could make the distinction from his or her
everyday observations.

Animals which chew the cud are identified as ruminants; they regurgitate food into their
mouths to be chewed again. Ruminants normally have four stomachs. Neither the rock hyrax
(translated "rock badger" in the NASB) nor the rabbit are ruminants and technically do not
chew the cud. However, both animals move their jaws in such a manner as to appear to be
chewing the cud. This action was so convincing that the great Swedish scientist Linneaeus
originally classified them as ruminants.

It is now known that rabbits practice what is called "reflection [sic], in which indigestible
vegetable matter absorbs certain bacteria and is passed as droppings and then eaten again.
This process enables the rabbit to better digest it. This process is very similar to rumination,

Volume 1990 - 2002 Issue


Page 534 of 2049
Skeptical Review Edited by Farrell Till
and it gives the impression of chewing the cud. So, the Hebrew phrase "chewing the cud"
should not be taken in the modern technical sense, but in the ancient sense of a chewing
motion that includes both rumination and reflection [sic] in the modern sense.

The list of clean and unclean animals was intended as a practical guide for the Israelite in
selecting food. The average Israelite would not have been aware of the technical aspects of
cud chewing, and may have otherwise considered the hyrax and the rabbit as clean animals
because of the appearance of cud chewing. Consequently, it was necessary to point out that,
although it may appear that these were clean animals because of their chewing movement,
they were not clean because they did not divide the hoof. We often follow a similar practice
when talking to those who are not familiar with more technical aspects of some point. For
example, we use observational language to talk about the sun rising and setting when we talk
to little children. To a small child the daily cycle of the sun has the appearance of rising and
setting (see comments on Josh. 10:12-14). The description is not technically correct, but it is
functionally useful for the level of understanding of the child. This is analogous to the use
here in Leviticus. Technically, although the hyrax and the rabbit do not chew the cud, this
description was functional at the time in order to make the point that these animals were
considered unclean (pp. 89-90).

What Geisler implies is that the average Israelite would have been unaware of the "technical
aspects" of cud chewing; therefore, the Hebrew Biblical phrase "chews the cud" should not be
interpreted in or associated with the modern technical sense of rumination. It should instead
be interpreted strictly in the ancient sense of describing a chewing motion. But which
"technical aspects" is Geisler referring to? Is he referring to [1] regurgitation of cud and/or [2]
possession of four stomachs?

Regardless, the Biblical Hebrew does not support this argument. Contrary to what Geisler
says, the Hebrew phrase [does he mean the Hebrew based on a lexicographical study of the
Hebrew language of the OT or the KJV's inaccurate English translation?] is not "chewing the
cud." As I pointed out in my previous article, the phrase consists of two words: gerah and
`alah. Strong's Exhaustive Concordance of the Bible translates `alah [word 5927] literally as
"[cause to] ascend" and gerah [word 1625] as "cud." Therefore, the complete literal phrase
translates as "brings up the cud." So is Geisler saying that the Hebrew phrase "brings up the
cud" does not refer to or was not intended for the ancients to interpret in the sense of
characteristic [1]? Since "brings up the cud" as used in Leviticus 11:5-6 obviously does refer
to regurgitation of cud and since rabbits and rock badgers do not regurgitate cud, we may
consider both verses as biologically erroneous. What is so "technical" about regurgitation? I
don't know, but, nonetheless, this is the same so-called "technical aspect" of rumination that
Leviticus wrongly applied to the rabbit and rock badger.

I have already refuted at great length in my previous article what Geisler said about the need
to interpret "chews the cud" in Leviticus 11:6 as a reference to refection. Therefore I do not
want to rehash everything I have already discussed on the issue but will simply recommend
the article to those interested in studying this subject. The main thing to remember is that the
ancients would not have thought that eating feces was "bring[ing] up the cud." Only a modern
sees what little similarity exists between refection and rumination because of his knowledge
of the chemistry of rumination. An ancient would not have had this knowledge, and for

Volume 1990 - 2002 Issue


Page 535 of 2049
Skeptical Review Edited by Farrell Till
reasons I stated in the prior article, the ancients were probably not even aware of refective
behavior. Therefore, this verse could not have been a reference to this behavior. It had to refer
to an observable behavior known to the ancients or it would not have made sense to them.
Why does Dr. Geisler say that we must not interpret this passage in the modern sense of
rumination but later says that we should interpret this passage in the modern sense of
refection? "Consistency thou art a jewel!"

Notice that Geisler offered absolutely no verifiable evidence or documentation [other than his
reference to the NASB] to support his theory. He merely speculated. I don't even have to
discuss anything else about his argument. The fact that almost all of his explanation is only
speculation is enough reason to dismiss Geisler's argument as unreliable. If Geisler's book
When Critics Ask is aimed at convincing intelligent and rational critics that the Bible is
inerrant, then he and his cohorts need to stop offering speculation to justify their beliefs
because speculation is not going to convince the skeptic and should not in itself convince
most rational people, who believe that speculation is very unreliable evidence. Incidentally,
Geisler misspelled refection as reflection, a mistake that makes me wonder just how
thoroughly he researched his argument.

The matter of researched and documented claims is one of the primary reasons why skeptics
abhor many inerrantist arguments, because most skeptics [as I would define the term] usually
try to research and examine their arguments thoroughly to ensure that we avoid the logical
fallacies that are defined in practically any logic or critical thinking textbook and offer
information that can be investigated and verified. If such information cannot be investigated
and verified, we willingly and without hesitation, when necessary, admit that such
information is mere speculation. This is why we have trouble putting up with or tolerating
most inerrantist claims, because after we have researched and documented our claims, many
of the inerrantists want to get lazy and offer mere speculation, which cannot be verified and
which took only a few minutes of fanciful thinking to fabricate, compared to the amount of
time we have spent researching our claims. Unlike us, many of these inerrantists do not like to
admit that such information is pure speculation that has not been or cannot be verified or
investigated. This sums up and pin points in plain language, for the uninformed average
church member who is wanting to know about this, the main reason for the conflict between
the inerrantist and errantist. If one is really concerned about what is trustworthy, he will see
the justification for our complaint and the need for such literature as The Skeptical Review.

So what is skepticism? I like to think of skepticism as the philosophy that emphasizes


investigation and justification of one's beliefs. We believe that undocumented claims, logical
fallacies, speculation, subjective feelings, and hearsay testimony [things many inerrantists like
to use] are not reliable justifications for belief, as do also most logicians and rational-thinking
people. We believe that sound logic, verifiable evidence, and documented claims are the most
reliable guides for truth that we presently have, as do most logicians or rational-thinking
people. You see, skepticism is just rational thinking with a different name. There's nothing
dishonest or mysterious about it at all.

Geisler also stated, "This action was so convincing that the great Swedish scientist Linneaeus
originally classified them [the rabbit and the hyrax] as ruminants." However, Geisler never
tells us if Linneaeus made an error when he classified the rabbit as a ruminant because he had

Volume 1990 - 2002 Issue


Page 536 of 2049
Skeptical Review Edited by Farrell Till
misinterpreted his observations of the rabbit's chewing action from a lack of modern technical
knowledge of rumination and rabbit physiology. Did the "ordinary Israelite" that Leviticus
was written for make an error, for the same reason Linneaeus did, in thinking that rabbits and
rock badgers ruminated? If one answers yes to this question, then he must agree that clearly
an ancient Biblical writer must have also been in error when he stated the ordinary Israelite's
misconception as if it were true.

Ultimately, Geisler had to resort to a completely unsupported assertion: "Consequently, it was


necessary to point out that, although it may appear that these were clean animals because of
their chewing movement, they were not clean because they did not divide the hoof." The
Biblical writer did not "point out," at least not explicitly, that the rabbit and rock badger
appeared to ruminate. He merely asserted that they did ruminate. Therefore, how is Geisler so
sure of his interpretation of Leviticus 11:5-6? Certainly, anyone can add words to scripture
[example: "Although you [the ancients] believe that rabbits chew the cud..." or "Although the
rabbit may appear to chew the cud..."] to force an interpretation that is true in some sense, but
if one is creative, he can do this with any sentence, scripture or not, that is in error; therefore,
we need to be cautious. If I say, "The grass in most yards is purple," most people would agree
that this sentence is false. If somebody really liked me and was very gullible and wanted to
defend me the way some inerrantists defend the Bible, he could say, "Well, you see, the
sentence is true. Jeff dreamed that the grass in most people's yards is purple. Therefore, you
should interpret his statement to mean that in paraphrase, 'I dreamed that the grass in most
people's yards is purple,' and so that is what Jeff was trying to imply. He just failed to say this
explicitly." Adding words to an interpretation is sometimes helpful to test how plausible
different interpretations sound in comparison to the context of the words being interpreted;
however, as we have seen, this is not enough to establish actual meaning and is therefore a
very unreliable method of literary interpretation.

Since Geisler's position is that God inspired the words of the Bible and knew in his
omniscience that rabbits only appeared to ruminate [whereas the ordinary Israelite wrongly
thought that the rabbit really did chew cud], did Geisler mean to imply that we should
interpret the words in Leviticus from God's perspective of referring only to the appearance of
cud chewing? If so, we must ask, why we should so interpret it if "God" was not concerned
whether the ordinary Israelite interpreted this verse to mean that rabbits really do ruminate?
Was not "God" depending upon the ancient's misconception of the chewing motion so that
these verses would make sense to them? This verse would not make sense if "God" wanted
the ancients to interpret it as a reference only to appearance or illusion, because just the
appearance or illusion of rumination would have disqualified the rabbit and rock badger as
clean because illusion or appearance is not genuineness, and genuine rumination was a
necessary qualification of cleanness. Therefore, if the ordinary Israelite understood this as just
an "apparent" characteristic, it was absurd and very unnecessary for the writer to go on and
point out another characteristic [does not possess split hooves] to disqualify the rabbit and
rock badger from being considered clean animals. Pointing out that these animals only appear
to [but really don't] ruminate would have been all that was necessary to disqualify them.
Besides, the fact that this verse refers to regurgitation [not a chewing motion] demolishes this
theory.

Volume 1990 - 2002 Issue


Page 537 of 2049
Skeptical Review Edited by Farrell Till
Since Geisler and many inerrantists like to assert arbitrarily that many Biblical passages that
skeptics point out as scientifically incorrect are actually examples of
observational/phenomenological language without giving us any justified reasons or reliable
criteria for distinguishing scientific errors from observational language, they need to inform
us on this. If they cannot, then we can safely conclude that when they assert that an alledged
scientifically incorrect passage is actually just some observational language, they are
guessing. So perhaps Geisler, or at least some inerrantist, could enlighten us and tell us how
he can determine by provable means when a passage is using either [O] nonscientific,
nonliteral, "observational," "phenomenological" language--the use of language to refer to the
illusion or appearance of something and not a misconception about the illusion or appearance.
Example: using the term "falling star" to refer to "a bright trail or streak that appears in the
sky when a meteoroid is heated to incandescence by friction with the earth's atmosphere"
[American Heritage Dictionary] because the object appears from the person's limited
perspective from the earth to be a star and not because one believes or misconceives the
object is "one of the many visible self-luminous celestial bodies consisting of a mass of gas
held together by its own gravity in which the energy generated by nuclear reactions in the
interior is balanced by the outflow of energy to the surface, and the inward-directed
gravitational forces are balanced by the outward-directed gas and radiation pressures"
[American Heritage Dictionary], or [S] a scientific error--language that states or reflects one's
misconception about the illusion [and not to the illusion alone or observable appearance] of
something because one was unknowingly deceived by the illusion and had no modern
technical knowledge about the illusion. Example: an ancient saying the earth is flat because
he believed that the entire earth is a two-dimensional flat disk because he was deceived by his
limited ground level perspective of the earth's appearance and had no "modern technical
knowledge" of geology.

Even if Geisler could give us a reliable criterion for deciding what is [O] and what is [S], he
concedes that even [O] can be technically incorrect! Do my eyes deceive me? Did Geisler
actually say that our use of observational language, when describing the sun rise and set to a
child, is not technically correct [What is he talking about here? I personally would never
deliberately make a child think that the sun actually moves around the earth, because that
would be deceptive] but has a function nonetheless and that this technically incorrect
language is analogous to the descriptions in Leviticus 11:5-6? Is he saying that although
Leviticus 11:5-6 is technically erroneous, it still had a function? I understand that sometimes
people unintentionally give the wrong impression of what they are trying to say [a
shortcoming that none of us are innocent of], so I'll give Geisler the benefit of the doubt. But
if I understood him correctly, I think he is saying that the Bible can, after all, have errors and
still be "inerrant" as long as those errors have functions. I don't believe it! We finally get the
inerrantists to admit that errors are in the Bible. Now if we can just get them to admit that
whether an error has a function or not, it is still an error. My main conflict with inerrantists is
not over the issue of whether "God" or a Biblical writer had an intentional purpose or function
for a Biblical error; my disagreement is over whether any kind of error [with the exception of
such errors that both of us agree do exist, such as copyist errors], functional or unintentional,
exists in the Bible. Could the Bible say something else that is not scientifically correct, as
long as saying this served some important function?

Volume 1990 - 2002 Issue


Page 538 of 2049
Skeptical Review Edited by Farrell Till
Just as adding words or details not stated in a passage to an interpretation can be applied to
any error to make it true in some sense [which is why inerrantists like to do this, as they do
with their attempt to "harmonize" the Gospels' various conflicting narrative accounts of
particular events], so can the "observational/phenomenological language" argument excuse
practically any scientific error no matter how erroneous it was. This is because all you have to
do is assert; no proving is involved in this approach. However, if you are a rational person,
you will not rely on assertions alone. This "argument" [I hesitate even to call it an argument]
is the inerrantists' favorite last resort argument when no other argument can be fabricated.
Indeed, Geisler even uses this same "argument" on page 140-141 of When Critics Ask to
excuse the scientific error in the book of Joshua when "God" halted the sun's movement
across the sky. Here he totally ignored the fact that when ancients said that the sun moved
across the sky, they meant that the sun really moved across the sky. Geisler ignored this and
added the word appear to his interpretation, forcing the passage to be true in a different sense,
and asserted that the Bible is only saying implicitly that the sun appeared to move across the
sky. Although I do not want to debate that particular issue, I wanted to show that what most
people would ordinarily consider an ancient scientific error, the inerrantists will assert without
proof is really only observational language that described only the appearance and not a
misconception of phenomena. And many people let them get away with this!

Let's notice how Geisler's argument "works" on even the most evident scientific errors. Pick
any passage of any religious literature that contains any genuine scientific error of antiquity
that you want. You can use an already existing passage or you can make one up. After you
make your selection, fill in the parentheses, as I have done, with the appropriate statements:

1. Although this (any ancient scientific error that the passage reflects, such as a flat-earth
view) is not true in the modern technical sense, to the ancient perspective, it would appear that
(the earth is flat).

2. Therefore, since (pick any hypothetical religious book that contains the error of your
choice) was written for ancients without any modern technical knowledge of (any science that
pertains to the error of your choice, such as geology) and since the only experience of reality
these people had was their limited perception or the observable appearance of things, it was
necessary for (pick any "god" as the author who inspired the writing of your hypothetical
religious book) to describe the appearance [don't forget to add the word "appear"] of
(whatever the error of your choice is, such as the shape of the earth) with words that the
ordinary people used to describe their limited perspective of the observable appearance of
(whatever the error of your choice is, such as the shape of the earth) so that they would
understand him/her/it [whichever you prefer].

3. Therefore, the language of (your hypothetical religious book) is merely observational and is
not an attempt to state something as true in the modern technical sense.

You should see that if this "argument" can make even legitimate errors look good, we can
know that by itself it is not a reliable way to explain biblical "difficulties."

I challenge any inerrantist reading this article to respond and tell us with precise words how or
under what provable conditions he would expect Leviticus to say that rabbits chew the cud so

Volume 1990 - 2002 Issue


Page 539 of 2049
Skeptical Review Edited by Farrell Till
that he would cease to assert that the statement is not erroneous. In fact, we skeptics should,
with every passage we point out that inerrantists deny is in error, always demand that they tell
us how the Bible in such cases would have to state something for us to be sure that it is
erroneous. In other words, just what does the inerrantist define as an error? He never tells us.
If he did, he would run the risk of someone finding an error that fits his criteria, giving him no
escape by speculating another wild interpretation to make the passage correct in some sense.
For him to be convinced, does he expect the Bible to say something like, "Although it is
absolutely, positively literal scientific fact that the rabbit literally, and I mean literally and not
just apparently, regurgitates the cud, it does not split the hooves"? If this were the case, I think
the inerrantist ridiculously expects just a wee bit too much. I think too that even if the Bible
were this explicit, inerrantists would still find some way to explain that it didn't really mean
this.

(Jeffrey A. Justice, P.O. Box 454, Wylie, TX 75098, reactions and comments invited.)

From the Mailbag


I just watched your program on TV [Atheists Talk produced by Minnesota Atheists] and felt
such relief that these questions were of interest to someone else.

Please put me on the mailing list for the Skeptical Review, and if you have some copies of
back issues, I would like to read them too.

(Cindy Daly, 411 West Whipple, B, Prescott, AZ 86301.)

EDITOR'S NOTE: I appeared as a guest on Atheists Talk, a program produced by Minnesota


Atheists, and when the tape was sent to me, I copied it and sent it to Charlotte Barkley, who is
active in local access TV in Prescott. Ms. Daly's letter is proof that it got results. I guess what
I'm trying to say is that if freethinkers will do what little they can in their respective
communities, the results will soon add up to significant numbers.

Having read all issues of TSR, plus other publications, debates, etc. of Mr. Till and other
skeptics, current and past, I find nothing to move me from holding to a strong belief that the
Bible is the Holy Spirit inspired, inerrant Word of God. Indeed such writings have made
stronger my belief that the Bible is the only revelation in which we may have absolute
confidence and trust, and the Bible will survive any attacks of its critics. To me, such a belief
in its inspiration is much more "logical," "sane," and "reasonable" rather than rejecting it as
such because man's "wisdom" and "logic" cannot prove the Bible to be inspired. Too often
man has thought by his "logic" and "wisdom" that he has proven or also disproven things only
for another generation to have to "rethink" what man thought he once knew or was more
logical. In every field, man is constantly "rethinking" what previous generations held. One

Volume 1990 - 2002 Issue


Page 540 of 2049
Skeptical Review Edited by Farrell Till
example is how many skeptics have gone to their graves thinking certain people, cities, etc.
found "only in the Bible" did not exist, because such were found nowhere else, and man's
wisdom could not logically prove the existence of such. Thus, many "freethinkers" and
"seekers of truth" were denied wonderful facts because they trusted in man's logic and
wisdom.

These skeptics and atheists have sold their souls for a mess of pottage-- man's wisdom.
Mankind's God-given means to advance, grow, and progress, have vastly benefited us.
Wisdom, properly used, makes a good servant, but it is a cruel master. While these skeptics
bow before the altar of wisdomolatry and worship it as their god with all their pomp and
ceremony, their so-called logic is shattered with the passing of time, and the Word of God
endures. Now Mr. Till plays a prophet and predicts the personal computer will be the
downfall of belief in Bible inerrancy. How simple minded! I do not have to be a prophet to
believe such an instrument can only confirm our belief in Biblical inerrancy. It seems each
past generation of skeptics since the Bible was completed in the first century A. D. has
boasted they were more "modern" than past "primitive" generations and would predict the
demise of Christianity. Ho-hum, what else is new? The Bible still endures. Bless the Lord, o
my soul!

Wisdomolaters ignorantly refer to us as Bibliolaters, not realizing we worship and bow to


what the Bible points us to, the Living God, and indeed never before the altars of science
falsely so-called nor any of man's ever-changing wisdom. So as Mr. Till would ridicule us for
believing something "just because it is in the Bible," a book that for centuries still endures
despite so many attacks, both from unbelievers as well as "alleged believers" who would have
us believe the Bible to be no more inspired than the works of Shakespeare, we confidently
believe merits our trust. So "just because it is in the Bible"? A million times, yes! "Which
Bible or version?" skeptics would reply. What a blessing to have "many" to confirm the word.

To show how wisdomolatry has dulled and warped the minds of skeptics, one needs to read,
as a small example, Mr. Till's reply (TSR, Summer 1994) to Nikki Kaley or an article, same
issue, entitled "Evidence that Doesn't Demand a Verdict" by Mark Smith, both of which twist
scripture and reasoning. Does Mr. Till, in publishing TSR, think he is in the college classroom
and the world should humbly accept the professor's teachings as "truth" or "probabilities" he
and cohorts have discovered via Mr. Occam's razor and such like? Lump all such skeptics of
all centuries together into one "brain," and we still affirm the inspired Word of God is sharper
and [more] reliable.

If TSR and such is the best skeptics, atheists, etc. can do to shatter our belief in Biblical
inerrancy and inspiration, then perhaps mankind would benefit more if Mr. Till and cohorts
put their "massive" wisdom to use helping solve man's physical problems and leave Biblical
matters to us country folk. Yes, skeptics have the right to their missionaries, publications, etc.,
as Bible believers do, but while skeptics ridicule our zeal, surely they can offer something far
greater to mankind than by spending their labor, time, and monies in attempting to refute
something as primitive as Christianity. But we wait in vain for something better, and these
wisdomolaters, at best, despite all their "hot-air" can offer us only "a choice," no factual
proof, and not a very good choice at that. We offer, by faith, what we believe [is] a better

Volume 1990 - 2002 Issue


Page 541 of 2049
Skeptical Review Edited by Farrell Till
choice and confidently, joyfully, still sing, "Give me the Bible.... Holy message shining...."
Bless the Lord, o my soul!

Such is my conclusion after reading all published issues of TSR. Enclosed is a check for
another year.

(Hershel V. Davis, 42 South Abbott Street, Hillsboro, TX 76645.)

EDITOR'S NOTE: Mr. Davis and I have conducted a private correspondence for over a year,
which began when he wrote to me after viewing the tapes of my debate with Buster Dobbs. I
have always appreciated Mr. Davis's cordiality and willingness to read the views that are
published in TSR. Needless to say, these qualities are often absent in other Church-of-Christ
members, as evinced by letters that we have published from Dobbs, Mitchell, and others. I am
happy to give him a forum to express his opinions.

Unfortunately, Mr. Davis did not make very good use of the space that we have given him.
Like so many other fundamentalists, he seems to believe that asserting is equal to proving.
For instance, he cited the article by Mark Smith ("Evidence That Doesn't Demand a Verdict")
and my response to Nikki Kaley's letter as examples of how we had twisted "scripture and
reasoning"; however, he made no attempt to analyze either of them to show exactly where the
twisting of scripture and reasoning had occurred. I fear that Mr. Davis has spent so many
years listening to sermons filled with unproven assertions that he now thinks that mere belief
claims constitute proof. He wondered if I think that the world is a college classroom in which
everyone should "humbly accept the professor's teachings as 'truth,'" but he fails to apply the
principle to himself. Must we humbly accept his mere word as truth? Apparently, he thinks
that we should, because he made no attempt to prove his assertions. I honestly believe that I
make a serious attempt to support the major points of my articles with clearly delineated
arguments. In fact, some have criticized me for being too thorough or technical at times, but
my goal is to leave the opposition with no room to quibble. In contrast to this, Mr. Davis says
essentially nothing to prove his assertions and then accuses me of expecting the world to
accept humbly whatever I say. I urge him, then, to heed his own advice and give us some kind
of proof that the Bible is "the Holy Spirit inspired, inerrant Word of God." Ipse dixit just
won't do.

Mr. Davis could make a much better impression on me and most TSR readers too, I'm sure, if
he would write an analytical response to just one article that we have published and show us
how that the reasoning in it is "twisted." To assist him, I'm going to suggest that he respond to
"Jairus's Daughter: Was She Dead or Wasn't She?" (Autumn 1994, pp. 2-4). During the week
of the Moffitt-Till Debate last May, I sat through several lectures that were supposed to
explain "fancied contradictions in the New Testament," and none of the speakers who were
assigned Matthew, Mark, and Luke even mentioned "fancied" problems in the three versions
of the raising of Jairus's daughter. I sent advanced copies of my article to all three of these
speakers and others who had "unraveled fancied contradictions" during this lectureship, but
none of them accepted the offer of equal space to publish simultaneously a response to this
article. I now make the same offer to Mr. Davis and urge him to accept it. It is easy to say that
he has found "nothing to move [him] from holding to a strong belief that the Bible is the Holy
Spirit inspired, inerrant word of God"; it is quite another to respond logically to the evidence

Volume 1990 - 2002 Issue


Page 542 of 2049
Skeptical Review Edited by Farrell Till
that disputes that belief. I contend that the three accounts of the raising of Jairus's daughter
are irreconcilably contradictory, and I invite him to prove me wrong in this claim.

Like all good Bible fundamentalists, Mr. Davis expressed his contempt for "man's wisdom."
Since this subject has been adequately discussed in the front-page article of this issue, we
need not comment on it again. Suffice it to say, that the comforts that we enjoy in our society
are entirely the benefits of "man's wisdom." They did not come from people who sat around
reading the Bible and praying for God to cure our diseases and invent radios, telephones, air
conditioners, automobiles, airplanes, printing presses, computers, etc., etc., etc. All of these
resulted from the work of scientists who used dedication and "man's wisdom" to make our
lives much better than they were when people believed that praying for the sick, anointing
them with oil, and casting out demons were ways to cure diseases. In some of his letters, Mr.
Davis has mentioned personal health problems, which I hope have since improved, so I
wonder if he has at any times sought the wisdom of man to help him with these problems. If
so, how can he conscientiously ridicule "man's wisdom" as he did in his letter?

Davis ridiculed the constant revisions that "man's wisdom" has to make as new generations
make discoveries that prove past generations wrong. "In every field," he said, "man is
constantly 'rethinking' what previous generations held," but he is wrong. There is one field
where this is not done, and that is the field of Biblical fundamentalism. This perhaps is the
only field where people take the stubborn position that they don't need to "rethink" their
positions, because they have the truth and couldn't possibly be wrong. As the little ducklike
character in a political cartoon by Oliphant said last summer, "Blessed are the self-righteous,
for they are always right." Now Mr. Davis may consider such an attitude as this a virtue, but I
consider it both deplorable and dangerous. How many people have been executed by
religious zealots who just knew that they were right and couldn't possibly be wrong? The fact
that scientists and freethinkers are willing to review their ideas and revise them if necessary
is a quality that has brought mankind to its present state of technology. So Mr. Davis is a
beneficiary of the very thing that he claims to despise.

And speaking of "Ho-hum," I know of no better response than "ho-hum" to all of Davis's talk
about the endurance of the "Word of God." The 2,000 years that Christianity has existed is
nowhere close to how long some religions have flourished. Hinduism, Buddhism, and
Zoroastrianism are older by many centuries than Christianity, and so is Judaism. Does their
longevity prove that they are "truer" than Christianity? The pagan religions and the ancient
religion of Sumer thrived for several millennia. Does this prove that they were "true"
religions? Since when does the age of an idea prove its truth? Actually, the older an idea is,
the more likely it is to be untrue, because it originated in superstitious, unscientific times.

Mr. Davis said that he and other Christians have waited in vain for skeptics to give them
something better than the truth of "God's word," but who can give something better to anyone
who stubbornly insists that he doesn't need to examine his beliefs because he knows he is right
and couldn't possibly be wrong? If, however, he wants skeptics to give him something better
than Christianity, I will certainly accommodate him. What about the self-respect, intellectual
integrity, and personal confidence that comes from facing reality and accepting obvious truth
no matter how stark and unflattering it may be? We have filled our "Mailbag" column with
the testimony of those who once wallowed in the superstition and guilt of Bible

Volume 1990 - 2002 Issue


Page 543 of 2049
Skeptical Review Edited by Farrell Till
fundamentalism but are now free of it, and they all agree that their lives are much better than
they were before. Mr. Davis, of course, won't believe this, but at least we can say that we
offered him a better alternative.

To set the record straight, I want to assure Mr. Davis that I have no illusions that the demise
of Christianity is imminent, because I know that religious superstition is deeply ingrained in
human societies. I have never been so rash as to predict that within 50 years or 100 years
belief in the Bible will be nonexistent, because religions die slowly. However, they do die, just
as surely as institutions, empires, and people die. In "The Legendary Triumph," Joseph
McCabe cited the widespread ridicule of paganism by its own thinkers and philosophers as
one of the major factors that enabled Christianity to supplant it (The Myth of the Resurrection
and Other Essays, Buffalo: Prometheus Books, p. 126). Paganism had become so illogical for
the times that, as McCabe noted, "(a) very large number of people were ready for
alternatives" (Ibid.). Let's compare the status of Paganism then with the present situation of
Christianity. It too has lost the respect of our best scholars. No serious biblical scholar
believes in the inerrancy doctrine any more, and the findings of higher criticism are being
openly taught in most seminaries and even some Bible colleges. Just recently, a group of
biblical scholars published The Five Gospels, a work that concluded that probably as many
as 80% of the sayings attributed to Jesus in the gospels were never said by him. These were
not wicked old atheists saying this to discredit Christianity but clergymen and Bible scholars
who were willing to put scholarship above Christian traditions. In "The Great Gulf Between
Scholars and the Pew," Michael D. Cougan discussed the widespread disparity in what is
generally known by the clergymen in the pulpits and the people sitting in the pews (Bible
Review, June 1994, pp. 44-48,55). The clergymen understand that the Bible is not inerrant
and that many stories in it are not historically accurate and in many instances are even
legendary or mythological, but they can't teach such things openly because of the intellectual
gulf between them and their audiences. So they tell them what they expect and want to hear.

With time, that gulf will narrow and religious leaders will have more freedom to teach facts
about the Bible that present church audiences wouldn't tolerate from their preachers. When
the knowledge in the pew more or less equals the knowledge in the pulpit, profound changes
in Christianity will have to occur. It is my belief that personal computers will be instrumental
in the dissemination of information that will narrow the gulf between pulpit and pew.
However, I would never be stupid enough to believe that this is going to happen overnight.
For Mr. Davis and me to argue whether any scenario like this is going to happen would be an
exercise in futility, because no one could ever prove that it will or will not happen. We can
only wait and see. Computers, however, obviously facilitate the dissemination of knowledge,
and knowledge is no friend of religion. I wonder if this could be why Bible believers are so
quick to ridicule worldly wisdom.

Ernest Renan (1823-1892) said, "Belief in the supernatural is the shame of civilization."
Personally, I am convinced beyond a reasonable doubt that anyone who believes in the
supernatural has lost touch with reality, is irrational, and is in fact mentally ill.

Volume 1990 - 2002 Issue


Page 544 of 2049
Skeptical Review Edited by Farrell Till
Under the term supernatural, we may lump God or gods, any religious system, mysticism, the
New Age rot of Shirley McLaine, telekinesis, ESP, UFO's, astrology, and any belief system
not supported by concrete evidence, experience, reason, and common sense.

When we look around us, all we see is nature and the natural order. Do we see any gremlins,
gods, elves, spooks? We do not for the simple reason that there are no such entities; they are
all imaginary beings. We can have a concept of a unicorn, yet we know this creature belongs
in the realm of myth and fantasy.

By discounting the supernatural realm, we are left with the real world, nature and the natural
order. We can then be faithful to the earth and strive to make life better for all and to lend
nature a helping hand. Anyone who believes in the supernatural is insane and should be
confined to a mental institution for his/her own welfare and safety.

(Jesse Baily, 2149-B 16th Avenue South, Birmingham, AL 35205-5020.)

EDITOR'S NOTE: What about for the welfare and the safety of the rest of us too, Jesse?

I am enclosing a check of $20 for two copies of Prophecies: Imaginary and Unfulfilled and as
a contribution to cover expenses. (Your excellent publication costs your subscribers too little.
You should charge more.)

After again reading your article "No Morality Without the Bible" (Winter 1994), I was
motivated to "search the scriptures" to try to find the word moral or the word morality in the
Bible. I could not find it, either in the Old Testament or the New Testament. I am reasonably
certain that the word is not there. Another word, righteousness, regarded by Jews and
Christians as synonymous with morality, appears ad nauseam. However, in the biblical
contexts these words are not synonyms. Righteous, in the biblical sense, means being a
believer in the biblical god. If you are a believer, then you are righteous; otherwise, you are
not and are classified as infidel, heathen, pagan, etc. King David, Gideon, Moses, etc. are
therefore classified as righteous despite the fact that they committed war crimes and other
atrocities.

It is quite apparent that Lindell Mitchell in his article "The Amalekite Destruction a Moral
Atrocity?" does not know the definition of morality. His defense is the biblical definition of
righteous. He also assumes that the commandment to kill the Amalekites came from "God"
and overlooked the fact that it was really Samuel who gave the order, claiming (falsely) to be
speaking for "God."

(Sol Abrams, 132 Easthampton F, West Palm Beach, FL 33417.)

Volume 1990 - 2002 Issue


Page 545 of 2049
Skeptical Review Edited by Farrell Till
I received my first issue of The Skeptical Review today, and I'm totally impressed! Enclosed
is a money order for $17 for all back issues from Winter 1990 to Spring 1994. Keep up the
good work. I feel as though I've struck gold discovering this newsletter!

Thank you.

(John Sears III, 17 Forest Avenue, Millinocket, ME 04462.)

Thank you for the free issue of The Skeptical Review. I hope that you enjoyed reading Gospel
Fictions by Randel Helms.

I am so impressed with The Skeptical Review that I would like a 10-year subscription. Ergo,
enclosed is $40.

(Bruce Mazet, 11011 North 92nd Street, Apt. 1025, Scottsdale, AZ 85260.)

I have already completed the first "free year" of your publication, and I am interested in
subscribing for a second year. A check for $4 is included. Your magazine is very informative
and is perfect ammunition against Bible thumpers who go around like mindless robots
parroting the same stuff their church tells them without even bothering to think. I have
especially enjoyed the debate between you and Mr. Mitchell concerning the genocide of the
Amalekites. I didn't much like Mr. Mitchell's weasely and slimy way of rebutting. What
difference does it make if you yourself were a pro-abortionist or not? That wasn't what the
debate was about. I also enjoyed the article by Stephen Van Eck that dealt with Christianity's
heritage in Hindu religion. Christianity is not a divinely unique religion, as would be believed,
but just a smorgasbord of other religions. There's a little helping of this religion and a little
helping of that religion, and it's all mixed together into an inedible concoction.

(Kevin M. Kral, 2171 Dusty Mile Road, Landers, CA 92285.)

Here is a check for a new subscription to The Skeptical Review. Also $2.50 of it is for the
Jackson-Till Debate. Could you also send me Spring of 1994 with the debate? I never
received it.

Please keep up the good work! I am a former Catholic, and I still suffer from some of the
stupid phobias that they had me believe in, like the devil "getting me," hell, and all the other
weird stuff. It might sound funny, but it isn't. I wonder how many other Catholics still belong
to the church because of fear.

(Elizabeth Glodzik, 84 Rosemere Road, Pawtucket, RI 02861.)

Volume 1990 - 2002 Issue


Page 546 of 2049
Skeptical Review Edited by Farrell Till
EDITOR'S NOTE: Mail sometimes gets lost. If this should happen to you, please let us know.
We will send you replacement copies.

All fundamentalist subscribers who pooh-pooh those of us who say that religion is harmful to
society, please notice that here is another ex-Christian testifying to the psychological harm
that her childhood indoctrination in religion inflicted on her.

I passed along the copy of The Skeptical Review to friends, and they have asked me to write
to you to put them on your mailing list. (Names and address given.)

Yours is a much needed publication in this age of fundamentalism. May you continue to grow
fantastically!

(Lola Reams, 412 Memphis Way, Vancouver, WA 98664.)

EDITOR'S NOTE: Our subscription list does continue to grow at what we consider a fantastic
rate. Could this mean that Ms. Reams' wish has been granted by some unseen cosmic
providence? After all, Christians see such occurrences as answers to their prayers.

The letters section of TSR is beginning to rival the "technical" content as to which I like
better. The letter by Mr. Phillips was an absolute gem! One would expect that Mr. Mitchell's
response to it may be a long time in coming.

(Douglas Palmer, 3420 Hartford Street North, St. Petersburg, FL 33710.)

EDITOR'S NOTE: Well, such as it was, Mitchell's response to Phillips was published in the
Winter 1995 issue (pp. 3-4).

Thank you for The Skeptical Review. You are doing good work.

I was raised in the Church of Christ, and it took me many years to discard their babble. They
have the ability to rationalize the entire Bible to mean their narrow version of its
interpretation. Good riddance! I'm much happier an atheist. Find enclosed $10 check for two
years subscription.

(Dillard W. Henderson, 4989 Crooks Road, Royal Oak, MI 48073.)

I want you to know that I appreciate what you are doing. Yours is a noble effort. For a
"backwoods teacher" with "no standing," you seem to be having quite a noticeable impact....

Volume 1990 - 2002 Issue


Page 547 of 2049
Skeptical Review Edited by Farrell Till
At one time my wife and I were very involved with a fundamentalist Bible church: I was on
the board of elders and chairman of the Christian Education Committee, my wife taught
Sunday School, our kids went to a Christian school, we tithed and then some. We "ate up"
Christianity with a passion.

I always said that one day I was going to look into the foundation of Christianity. What got
me started was that I read the Bible enough to begin to notice significant problems. I did a lot
of reading, and I now have an extensive library of critical material. I'm happy to say that I am
a "born-again atheist" (and have been for about fifteen years).

I too enjoy debunking Christianity. Although I have been accused of "bashing" Christianity
and religion for no "good" reason, I feel that those of us in the know have a responsibility to
do so. To remain silent about something this important and not tell what you know is both
cowardly and uncharitable.

(Donald Morgan, 366 Wahkiakum Place, La Conner, WA 98257-9534.)

EDITOR'S NOTE: I don't want to sound like a broken record, but here are more former
Christians testifying to the happiness that they found by rejecting their fundamentalist beliefs.
We have yet to hear a former fundamentalist say, "Skepticism has made my life miserable; I
was so much happier when I was a Christian fundamentalist." Nevertheless, bibliolaters
continue to insist that atheists and skeptics have doomed themselves to lives of gloom and
despair.

You are responsible for me becoming a heavy-duty Bible reader (studier). Since I received the
first issue, I have purchased three Bibles and Strong's Concordance. I had no idea just how
ignorant I was. I am also experiencing emotions that are really traumatic. Anger, anger--it's
like someone else has said, "Reading the Bible is like wading through a sewer to find a gem."

I have a question. I was looking up a word in my concordance and accidentally ran across the
word Easter in Acts. How can that be? My other Bibles say Passover. My first thought is that
Easter was not yet invented when Acts was written, but what do I know?

I also have a comment. It is as you have said: Having a discussion with a Bible thumper can
be really frustrating. They have all the answers. I asked a bibliolater that I work with why no
one else wrote about Herod's killing all the boy babies. A puzzled look came across his face
for a moment and then his answer... (about four seconds later)... "Clandestine operation." I
said, "Gee, I should have known that; it sounds so logical."Anyway, keep up the good work. I
love it.

(Chris Long, 4156 Kenneth Drive, Suite 1, Gibsonia, PA 15044.)

EDITOR'S NOTE: The Greek word in Acts 12:4 is pascha, which is the same word translated
Passover whenever the Jewish holy day was referred to. The KJV translators simply
mistranslated the word in Acts 12:4. Despite this and other problems in the KJV, there are

Volume 1990 - 2002 Issue


Page 548 of 2049
Skeptical Review Edited by Farrell Till
fundamentalists who will argue that it is an inerrant translation. The Baptist preacher Peter
Ruckman in Pensacola, Florida, is an advocate of KJV inerrancy. (As Dave Barry would say
in his humor column, I'm not making this up.)

As for whether Easter existed at the time of the incident recorded in Acts 12:4, pagan
commemorations of the resurrected savior-god were celebrated long before the alleged
resurrection of Jesus. In that sense, "Easter" did exist at that time, so the Christian Easter is
merely another example of pagan influence on Christianity.

Thank you for editing a thought-provoking and informative publication. I believe that every
skeptic/freethinker/ atheist needs (1) the firm conviction in his/her ideas (which must come
from within) and (2) intellectual tools and facts to answer the (often stupid) claims made
against their ideas. TSR goes towards fulfilling the second need admirably. Many religion-
fanatics are prudent enough to avoid getting into logical arguments with skeptics (realizing
the flimsy to nonexistent foundations for their own ideas), but if one of them does try a
conversion, they must be ruthlessly dealt with, with every tool at the disposal of the skeptic.

In the Autumn '94 issue, Mr. X claimed that atheism arises through personal hurt." I would
claim instead that theism arises through personal incompetence. When people are unwilling or
afraid to face life's challenges and accept the full consequences of their actions, they turn to
abstract entities like god or religion, hoping to absolve themselves of their duties and errors. I
do not think that the atrocities committed in the name of religion will completely disappear
even if the whole notion of god and religion disappears from our cultures, but at least the
perpetrators of the crimes will not derive a cloak of legitimacy for their actions by appealing
to divine dictum (as the defenders of the Amalekite massacre are now doing, and the massacre
is just one example of the many disgusting justifications for misdeeds).

I have enclosed a check towards my subscription, back issues, and transcripts of debates (with
details on a separate form).

(Krishna Kunchithapadam, Computer Sciences Department, 1210 West Dayton Street,


Madison, WI 53706.)

I've been meaning for months to write to tell you how much I enjoyed your eloquent article
"As I Lay Dying (With Apologies to William Faulkner)," in the Spring issue. I suppose the
hardest concept to get across to believers is that all morality comes from within ourselves--
individually and collectively. As with so many other aspects of life, we adults must learn to
take personal responsibility for our own morality. How sad that anyone could preach--or even
believe--that only superstitious belief imposed from authority can make one moral. Good luck
to you as you accelerate to 5 and then 6 issues in 1996. I'll be with you!

(David Bartley, 10235 Echo Ridge Court, Dallas, TX 75243-2534.)

Volume 1990 - 2002 Issue


Page 549 of 2049
Skeptical Review Edited by Farrell Till

Since Norm Geisler has a radio program here and is "Dean" of Southern Evangelical
Seminary, I'd like the transcript of your debate with him.

When they were discussing the evils of abortion on their radio program, I called with a
question. I said, "If life is so sacred to God, why are there so many spontaneous abortions?" In
the usual fashion with which you are very familiar, he explained it away as "acts of nature,"
etc. They never fail to squirm out with some sort of twisted logic to fit their beliefs! In a
nutshell, we should praise and thank God for everything good in life and blame the poor ol'
devil, mother nature, or liberal Democrats for all the evils in the world. And this is the God
they claim is "in control" of everything?

I've almost reached the point where my chagrin and frustration have turned into amusement.

(Nina Griffin, 6110 Grove Park Boulevard, Charlotte, NC 28215.)

Thanks for the letter. We continue to get a good response from those who listen to the tape.

Sorry, my busy schedule as Dean won't allow me time to do more debates. Contact Dr. Gary
Habermas at Liberty University, Lynchburg, VA.

(Norman L. Geisler, Dean, Southern Evangelical Seminary, 5801 Pineville-Matthews Road,


Charlotte, NC 28226-3447.)

EDITOR'S NOTE: This letter was received from Dr. Geisler in response to a letter from me
suggesting that we debate a second time, preferably in the Charlotte area where the students
at his seminary would have the opportunity to observe a spokesman of his reputation defend
either the inerrancy of the scriptures or the historicity of Jesus. I sent the following answer to
Dr. Geisler:

I regret that your busy schedule will not allow you time for more debates. I do trust, however,
that your schedule will allow you enough time to write more books, because they are an
excellent source of information to demonstrate the absurdity of the inerrancy position. I could
tell you how busy my schedule is, but I'm sure that wouldn't move you to reconsider your
decision.

I noticed that the envelope in which you mailed your letter has John 10:35 printed under the
return address: "The scriptures cannot be broken." I do wish that you or someone of your
reputation would show a willingness to defend that position in public debate, but I don't have
much hope that that will happen. Your stationery listed Josh McDowell as a member of the
"Advisory Board" for Southern Evangelical Seminary. Do you suppose that he would be
willing to represent the inerrancy position in a debate in Charlotte? I would appreciate your
asking him to consider my proposal. I have already challenged him to debate, but he never

Volume 1990 - 2002 Issue


Page 550 of 2049
Skeptical Review Edited by Farrell Till
even responded to my letter. Perhaps he would be more receptive to the proposal if it came
from you.

I will contact Dr. Habermas at Liberty university, which I have already done, to no avail, but
perhaps if I include a copy of your letter, he will give my proposal more consideration.
Meanwhile, if you should ever find any spare time in your busy schedule for a debate, please
contact me.

Volume 1990 - 2002 Issue


Page 551 of 2049
Skeptical Review Edited by Farrell Till

The Skeptical Review


Volume Six, Number Three - 1995
Farrell Till, editor

• What About Their Missing Links?

• Answers to a Claim of Errancy of the Scriptures

• All Prophets Were False!

• Creationists Bugging You?

• Evaluating Historical Claims

• Leaning Over Backwards for God


Farrell Till uses the Bible to prove that bastards aren't allowed in God's asssembly.

• From the Mailbag

• Back Page

What About Their Missing Links?


Creationists never tire of talking about the transitional forms or "missing links" that must be
found in the fossil record in order to prove the theory of evolution but which, unfortunately
for their theory, scientists have yet to find. We lay no claim to expertise in the field of
evolution, but from the books and articles that we have read on the subject by those who are
recognized authorities in the field, creationists are wrong when they say that no transitional
fossils have ever been discovered. In Evolution and the Myth of Creationism, Tim M. Berra
refuted this creationist claim and identified some of the transitional fossils that have been

Volume 1990 - 2002 Issue


Page 552 of 2049
Skeptical Review Edited by Farrell Till
found (Stanford University Press, 1990, p. 127). Among others, he discussed the
Australopithecines, some species of which were ancestral to homo sapiens (pp. 100-119), as
well as transitional forms in the evolution of other species (pp. 39-46).

Our purpose is not to discuss data that prove the theory of evolution but to point out an
inconsistency in the way that creationists reason. They seem to be arguing that if fossil
evidence of something (in this case, transitional forms or "missing links") cannot be found,
then we know that it never existed. In using such "logic" as this, they apparently don't realize
the damage that it will do when applied to some of their own claims. Creationists claim, for
example, that the Bible gives an accurate account not just of creation but also of Mideastern
history from the time of creation through the first century A. D. In order to prove this claim,
however, they must produce some "missing links" of their own that no one has yet found.

An obvious example would be fossils of people whose life spans covered centuries rather than
just the few decades that modern humans can expect to live. Genesis 5 contains a genealogy
from Adam to Noah, and the average life span of the nine generations listed was 847 years.
(The average would have been even higher, but Enoch "walked with God" and was translated
when he was just a mere kid of 365.) After the flood, so we are asked to believe, the life span
of humans began to decrease. Noah died when he was 950 ( Gen. 9:29), but his son Shem
lived for only 600 years ( Gen. 11:10-11). Shem's son Arphaxad lived only 438 years (vv:10-
11), and within two generations, the average life span had decreased by still another 200 years
( Gen. 11:14-24).

After this, things just went from bad to worse. Abraham had to settle for a life span of only
175 years ( Gen. 25:7), and his son Isaac and grandson Jacob lived to be just 180 and 147,
respectively ( Gen. 35:28; 47:28). Still, at the time of the exodus, people routinely lived for
130+ years ( Ex. 6:20) --or so the Bible claims.

So now it is time to apply the logic of bibliolaters to their own situation and ask them for
archaeological or paleontological proof that human life spans ever reached anywhere near the
incredible ages that the Bible attributes to the people who lived before and for at least a few
centuries after the flood. According to the scenario that creationists ask us to believe, most of
the fossil record resulted from catastrophic conditions that accompanied the flood, so if
people routinely lived for 800 or 900 years at that time, why haven't we found some fossilized
proof of this biblical claim? Where are these missing links?

Probably more archaeological digging has taken place in that part of the world than anywhere
else, yet no one has ever found human remains that expert examination has determined to be
anywhere close to the ages that the Bible attributes to the antediluvian patriarchs, yet if the
Bible record is true, there had to have been thousands of people buried in the silt of the flood
who were several centuries old. Why haven't we found any fossils of these people? Where are
these missing links?

The truth is that evidence unearthed so far indicates that prehistoric man had a very short live
span compared to ours. The consensus of experts who have examined prehistoric human
remains is that 40 was a ripe old age. In February 1992, the wire services reported the
discovery of a cemetery near Giza, Egypt, that dated from the 26th to the 21st century B. C.,

Volume 1990 - 2002 Issue


Page 553 of 2049
Skeptical Review Edited by Farrell Till
so the people buried in it would have been contemporaries of the ante and postdiluvians who
had routinely lived from 175 to 900+ years. However, expert examination of the skeletal
remains in this cemetery revealed that those who had lived the longest had died in their
midthirties. Isn't that curious? Just a few miles to the northeast, people were enjoying life
spans measured in centuries, but their Egyptian contemporaries could manage to hang on for
only a few decades. When will Bible fundamentalists come forward with fossilized "missing
links" that will fill in the gaps between present day life spans and those that presumably
existed in anteand postdiluvian times?

We can ask the same about the Hebrew exodus from Egypt. According to census figures in
Exodus and Numbers (too complex to discuss here) 2.5 to 3 million Israelites left Egypt and
wandered for 40 years in the Sinai wilderness. Because of their disobedience, all of them but
the children were condemned to die in the wilderness without ever seeing the promised land (
Num. 14:20-24; Dt. 1:37-39). So we are asked to believe that 3 million nomads wandered
from encampment to encampment for 40 years in a relatively confined area no bigger than the
Sinai peninsula, that two million of them died and were presumably buried there, but
archaeologists have never found any trace of evidence that they left behind. Speaking at a
conference at the Royal Ontario Museum, Israeli archaeologist Eliezer Oren reported that
more than 80 sites excavated in the Sinai by his team between 1972 and 1982 "had not turned
up any support for the historical accuracy of when the Exodus was supposed to have
occurred" (Barry Brown, "Israeli Archaeologist Reports No Evidence to Back Exodus Story,"
carried by News Toronto Bureau, Feb. 27, 1988). Oren reported that "(o)ther research groups,
digging just as long in the central and southern Sinai, have found no traces of human activity
at all from the Exodus time, except for some small mining sites that were under strict
Egyptian control." Brown's article reported that "(o)ther archaeologists who have worked in
the Sinai agree no evidence has been found of any encampments indicating where the children
of Israel stayed in the desert." How much trash would three million people have left behind at
these encampments to be buried in the shifting desert sands? And why hasn't any of it been
found?

Numbers 33 lists some forty separate encampments where the Israelites allegedly stayed
during their wanderings. The book of Leviticus details the daily sacrificial ceremonies that
required the slaughtering of bullocks, sheep, lambs, goats, etc. to appease the wrath of
Yahweh. Three million people participating in such sacrifices over a period of 40 years would
have necessarily slaughtered millions of animals whose remains had to be carried outside of
the camp and burned ( Lev. 4:11). Where is the archaeological evidence that sacrifices on any
such scale as this ever occurred in the Sinai? Where is this missing link?

We could continue this and ask about many other "missing links" that Bible inerrantists have
yet to produce, but these are sufficient to show an inconsistency in fundamentalist thinking.
Logic recognizes that the lack of positive evidence does not constitute negative evidence, and
fundamentalists recognize it too. We can guarantee you that some of them will write to inform
us that the absence of archaeological proof of the exodus or of antediluvian life spans of
several centuries doesn't mean that neither happened--and they are right. If they can recognize
this in matters that concern their outrageous beliefs, why can't they see that all of their talk
about "missing links" (even if it were true) would prove nothing about evolution?

Volume 1990 - 2002 Issue


Page 554 of 2049
Skeptical Review Edited by Farrell Till

Answers to a Claim of Errancy of the


Scriptures
In his debate with Jerry Moffitt, Farrell Till made the claim that the Scriptures contain at least
one error. His claim was based upon the assertion that the prohibition of Deuteronomy 23:2
would have prevented David from serving as king of Israel. This assertion will be examined
in this brief treatise.

Common Errors Made by Atheists and Agnostics


Many of the arguments made for claims of errancy in the Scriptures are of two different
forms, the first one being if p then q, not p, therefore not q. This argument contains the fallacy
of denying the antecedent. The following argument illustrates this fallacy:

• First Premise: If Farrell Till has climbed Mount Everest, then Mount Everest exists.
• Second Premise: Farrell Till has not climbed Mount Everest.
• Conclusion: Therefore, Mount Everest does not exist.

Both the first and second premises are true in this hypothetical syllogism, but the conclusion
is not true. Some of the arguments of those who claim the Bible is errant are of this form.
They argue the following:

• First Premise: If the Bible says X, then X is true.


• Second Premise: The Bible does not say X.
• Conclusion: Therefore, X is not true.

The argument contains the fallacy of denying the antecedent.

The second basic argument of those who claim the Scriptures are errant is of the following
form:

• First Premise: If Christian "X" can refute my claim of an error in verse "Y," then there
is no error in verse "Y."
• Second Premise: Christian "X" cannot refute my claim of an error in verse "Y."
• Conclusion: Therefore, there is an error in "Y."

This argument also contains the fallacy of denying the antecedent. This is the kind of
reasoning being done by many skeptics and modernists when they approach the Bible.

This fallacy was committed for a number of years in the form of the following argument:

• First Premise: If there is evidence that the Hittite nation existed, then the Hittite nation
existed.

Volume 1990 - 2002 Issue


Page 555 of 2049
Skeptical Review Edited by Farrell Till
• Second Premise: There is no evidence that the Hittite nation existed.
• Conclusion: The Hittite nation did not exist.

Linked with this argument was the following argument:

• First Premise: If the Hittite nation did not exist, then the Bible is errant.
• Second Premise: The Hittite nation did not exist (conclusion of prior argument).
• Conclusion: the Bible is errant.

It is evident that the Hittite nation existed and that the conclusion of the first hypothetical
syllogism is unsound.

A Consideration of Deuteronomy 23:2


Farrell Till claims that the entrance of both David and Phinehas into the assembly of Yahweh
were violations of Deuteronomy 23:3 (TSR, Spring 1994, pp. 7,12,16). There are several
possible explanations of this passage that would not cause Yahweh to be violating His word
by placing these men into leadership roles in the assembly.

The first explanation is that David might have been the generation that was allowed to enter
the assembly (the tenth generation). Farrell claims that "the expression in the original Hebrew
did not mean even to..." (Ibid.). Several translations render it as "even to" or an equivalent
expression. These are the KJV, ASV, NASV, NIV, and NRSV. But, for the sake of argument,
let it be given that the words "even to" should not be in the translation. Farrell must know that
there are no gaps in the genealogies of 1 Chronicles 2:3-15. Farrell makes the following
argument:

• First Premise: If there are no gaps in the genealogies of 1 Chronicles 2, then David
serving as king is a violation of Deuteronomy 23:2.
• Second Premise: There are no gaps in the genealogies of 1 Chronicles 2.
• Conclusion: David serving as king is a violation of Deuteronomy 23:2.

This argument is an excellent example of the fallacy of begging the question. In order to
know that his argument is sound, Farrell must know that there are no gaps in the genealogy.
Since gaps are common in the genealogies of the Bible, it is certainly possible that Farrell is
wrong on this point.

The second explanation is derived from applying the logical principle of contraposition to the
first premise of the argument of the prior paragraph. The first premise becomes, "If David
serving as king is not a violation of Deuteronomy 23:2, then there are gaps in the genealogies
of 1 Chronicles 2." This argument is based upon the assumption that Perez was a bastard. If it
can be proven that Perez was not a bastard, this argument vanishes. The reader should note
that the union of Boaz and Ruth is likened to the union of Judah and Tamar ( Ruth 4:10-12).
Since the union of Boaz and Ruth was a levirate marriage (cf. Deuteronomy 25:5-10) and the
union of Judah and Tamar was a parallel to the union of Boaz and Ruth, the union of Judah
and Tamar was also a levirate marriage. According to the book of Ruth, the levirate marriage
included more than a brother; it related to the near kinsman ( Ruth 2:20, 3:9-13, and 4:1ff.).
Onan refused to perform the duty of the levirate marriage ( Gen. 38:8-10) and was punished

Volume 1990 - 2002 Issue


Page 556 of 2049
Skeptical Review Edited by Farrell Till
by God. Judah promised Tamar that his other son Shelah would perform the duty ( Gen.
38:11), but he feared that Shelah would also die and did not keep his word. By this law,
Tamar had the right to have a child by the next of kin, and if she had this right the child was
not a bastard. Judah recognized that Tamar had the right to a child ( Gen. 38:26). In fact, the
expression "knew her" is used for their sexual relations ( Gen. 38:26). This expression is
commonly used to refer to sexual relations in marriage ( Gen. 4:1, 17, 25, etc.). A bastard, by
definition, is one who is illegitimate (one born from an unlawful sexual union). In no instance
does the Bible refer to their sexual union as adultery. I offer the following argument to prove
that the argument of Farrell Till is unsound:

• First Premise: If the union of Judah and Tamar was lawful, then Perez was not a
bastard.
• Second Premise: The union of Judah and Tamar was lawful (Tamar was righteous to
have the child, Genesis 38:26).
• Conclusion: Perez was not a bastard.

The third explanation is that God is not unrighteous in that He does not have any ex post facto
(retroactive) laws. This type of law is prohibited by the constitution of the United States of
America (Article I, Section 9). The interpretation of this passage set forth by Farrell Till
requires that it be an ex post facto law. I offer the following evidence that Far- rell has begged
the question in this matter:

• First Premise: All those who make ex post facto laws are those who are unrighteous.
• Second Premise: Yahweh is one who has made ex post facto laws.
• Conclusion: Yahweh is one who is unrighteous.

The major premise is axiomatic, and the conclusion is what Farrell is attempting to prove. In
order to prove the conclusion, Farrell must prove his minor premise. Where is his proof? He
assumed it without a proof! He begged the question.

The fourth explanation is that Farrell must assume that the expression "enter into the
congregation of the LORD" means to become a leader. Matthew Henry, in his commentary on
Deuteronomy, lists other explanations of this expression.

The fifth explanation of Farrell's charge that Phinehas should have come under the ban is that
Phinehas might not be a descendant of Perez. Farrell claims that Phinehas was a descendant of
Perez. In order to know that Phinehas should not enter the assembly, Farrell must know, not
just think, that Amminadab and Nahshon are the same persons in the genealogy of David.
There were at least three men named Amminadab ( 1 Chron. 2:10, 6:22, and 15:10). Is it
possible that the Amminadab of Exodus 6:23 is a fourth Amminadab? Or is it possible that the
Amminadab of Exodus 6:23 is the same Amminadab of 1 Chronicles 6:22? In addition,
Farrell must know that there are no gaps in the genealogy of Phinehas and Perez.

In conclusion, the argument set forth by Farrell Till is unsound. Perez was not a bastard, and
all of Farrell's arguments on this matter are moot. He is guilty of the fallacies of begging the
question and denying the antecedent. There is no contradiction in this passage, and a rational
interpretation of this passage demonstrates that Farrell Till has failed to consider all possible

Volume 1990 - 2002 Issue


Page 557 of 2049
Skeptical Review Edited by Farrell Till
solutions to the problem. In order to know that a contradiction exists, Farrell Till must know
that all of the above explanations are wrong, not just think they are wrong or wish them to be
wrong.

(Marion Fox, OKC School of Biblical Studies, 11826 S.E. 59th, Oklahoma City, OK73150.)

All Prophets Were False!


According to the Bible, false prophets were to die ( Dt. 18:20; Zech. 13:3). Such were to be
determined by the simplest of tests: whether or not their prophecies came true ( Dt. 18:22).
Well, according to the Bible, the label of "false prophet" should be applied to Isaiah,
Jeremiah, Ezekiel, and even Jesus himself! All of them issued prophecies that turned out to be
false.

The role of the prophet among the ancient Hebrews was to serve as a national scold, to
denounce the people, even the temporal rulers themselves, for their sins, principally their
frequent deviations from the strict religious standards the priests were struggling to establish.
Their role was not primarily to predict the future like an archaic Jeane Dixon. This
misconception of the prophet is the product of the wholesale abuse of the prophetic scriptures
by the fabricators of the New Testament. Thomas Paine is but one of the honest, objective
Bible scholars who have pointed out that the so-called Messianic prophecies of the Old
Testament were an extreme misunderstanding. In brief, they were not issued as prophecies.
They were taken out of context, and they were distorted to make Jesus appear to be the
fulfillment of prophecies--prophecies that were not even issued as such.

The process of scriptural distortion started in the popular imagination as obsessed Jews read
the scriptures in an oracular manner and came to possess a detailed consensus on what the
ever-coming Messiah would be like. Somewhere along the line, these popular conceptions
were systematized into a narrative account of the Messiah as if he had already come, and thus
a popular legend became a gospel "reality" that took on a life of its own.

In addition to rendering denunciations against the sins of the people, the prophets frequently
issued threats of dire consequences for them if they did not reform (as in Jeremiah 6:19;
11:11, and 22:6-7). A frequent warning was that foreign powers would conquer them and take
them into captivity. This was a historical reality, thanks to the Assyrians (who destroyed
Israel) and the Babylonians (who conquered Judah, although taking only an estimated 10,000
captives). The apparent fulfillment of these prophecies has wowed Bible readers for centuries,
helped inculcate a deep faith in the validity of the Old Testament, and made people less
inclined to raise questions about the dubious nature of the alleged Messianic prophecies. And
they would indeed be considered authentic prophecies, were it not that they were written after
the fact!

Volume 1990 - 2002 Issue


Page 558 of 2049
Skeptical Review Edited by Farrell Till
Objective Biblical scholars agree that up to the time of the Babylonian Exile, there was no
Old Testament per se. The scriptures existed primarily as an oral tradition and were
comprised of the Psalms and the Proverbs, along with the most prominent prophetic rantings.
The captivity provided the impetus for the scriptures to be formalized and recorded, impelled
by the fear that the Jews in Babylonia might otherwise lose their religion, as had the lost tribes
of Israel before them, who lost their very identity as well. This gave the priests the
opportunity to take whatever liberties with the text would suit their interests. Just one of the
things that they could do was "predict" things that had already come to pass, namely the
Babylonian conquest of Judah and the captivity of the Jews. Historical accounts of events
were recorded in the form of prophecies.

This was not necessarily done as a con game but to show the hand of God in all the events
that befell them and to offer a guilt-laden superstitious explanation for them. Actually,
though, the only explanation was that the ancient Hebrews were relatively small and weak,
and were surrounded by more powerful neighbors who acted typically for the time and place.
All the piety in the world wouldn't have made any difference to the fate of their nation. A
strategic mistake of the prophets was to give in to wishful thinking and desires for revenge
and, thus, to solace themselves by issuing predictions of ruin against their neighbors. Despite
the ceaseless warfare of the times and the inevitable waxing and waning of cultures, these
prophecies did not come to pass in the excessive manner in which they were issued and may
therefore be dismissed as false prophecies (a fact that Christians all too conveniently ignore).

Isaiah condemned Babylon, "predicted" its overthrow, and said that it would never be
inhabited again ( 13:19-20), but the Persians had already conquered it by the time the final
version of Isaiah was actually written. Thus, it is doubtful that this was a valid prophecy. And
while Babylon as an important city may no longer exist, to state that it should be so
uninhabited that you couldn't find even so much as a shepherd in the vicinity is contradicted
by historical fact. The town of Hilla exists today in that location and was built in part from the
ruins of Old Babylon.

In 19:4, Isaiah seemed to correctly predict that the Egyptians would be conquered. This,
however, has happened several times, both before and since the time of Isaiah, and as a
prediction is less than impressive. (It's like predicting that Tampa Bay will miss the playoffs
next season.) But when he predicted the Nile would dry up and the reeds and rushes and
everything sown by the river would wither and be no more ( 19:5-7), he obviously went too
far. Likewise, in 19:18, when he predicted the Egyptians would be speaking the now dead
language of Canaan, he prophesied something that never happened--and never can. He was
also wrong when he predicted a highway from Egypt to Assyria, which would bring the
Assyrians into Egypt and take the Egyptians into Assyrian bondage ( 19:23).

Next Isaiah denounced Idumea ( chapter 34). Predicting the dissolving of the mountains in
blood ( v:3) was a blatantly absurd hyperbole, and prophesying that it would become a
wasteland that none would ever pass through ( v:19) has been contradicted by historical
reality.

We can't leave Isaiah without pointing out that the famous "Immanuel" prediction of 7:14 was
not only not about Jesus but was instead a self-fulfilling prophecy whose fulfillment Isaiah

Volume 1990 - 2002 Issue


Page 559 of 2049
Skeptical Review Edited by Farrell Till
himself guaranteed by impregnating the "young woman" (not virgin) in 8:3. This child was
not only not named "Immanuel," but his birth was offered as a sign that Israel and Syria
would not prevail against Judah ( 7:5-7, 14-16). Well, Isaiah was dead wrong about that!
Second Chronicles 28 reveals that what he said wouldn't happen is exactly what did happen.
So much for Isaiah as a prophet.

In much of his book, Jeremiah argued against other prophets who disagreed with him ( 5:31;
23:16, 25, 26; 27:9; 29:8-9). Written after the fact, his predictions were made to appear
prescient that Babylon would conquer his people, but he failed the acid test when he assured
Zedekiah that while the Babylonians would take him hostage, he would die in peace ( 34:5),
and in 38:17, Jeremiah said that the city would be burned but that Zedekiah and his house
would live. In Chapter 39, Zedekiah fled Jerusalem and was captured by the Babylonians,
who killed his sons before him and then gouged out his eyes ( vv:6-7). He was bound in
fetters and carried away to Babylon, where he died in prison ( 52:11) (That's not "dying in
peace" to me.) In 34:22, Jeremiah said that the cities of Judah would be a desolation without
an inhabitant, which was hardly the case. After Jerusalem fell to the Babylonians, only some
of the Jews were taken captive to Babylon, and Gedaliah was appointed by Nebuchadnezzar
to be "governor over the cities of Judah" ( 40:1-5). Jeremiah was allowed to "dwell with him
[Gedaliah] among the people that were left in the land" ( v:6). Finally, Jeremiah predicted,
when God was through using Babylon as his instrument against Judah, he would turn around
and punish Babylon with a similar desolation ( 51:28-29), which was not only dastardly of
him but just more hyperbole as well.

If all this weren't bad enough, in Lamentations 4:22, Jeremiah stated that God would no more
send his people into captivity. Obviously, he was not prophet enough to foresee the final
Diaspora at the hands of the Romans (something the Gospels were able to hint at, after the
fact) or the concentration camps in our own century.

Ezekiel, the maniac prophet, blew it when he predicted the restoration of Sodom and Samaria
to their former status ( 16:55). Like Jeremiah, he predicted the desolation of Edom (Idumea]
"by the hand" of Israel ( 25:13). The worst that happened to the Edomites was their
amalgamation into another culture, as was often the case in the Middle East's stewpot of
ethnicities.

Most grandiose was Ezekiel's prediction of the destruction of Tyre ( 26:3-4, 19-21). Zeke
claimed it would be scraped away "like the top of a rock" and would "never be found again,"
yet the city was later known and referred to in the Bible itself ( Mark 3:8; Acts 21:3). [D.
James Kennedy, in his book, Why I Believe, actually credited this prophecy as a fulfilled one,
despite Tyre's New Testament appearance, citing its destruction by Alexander the Great as
proof. But Ezekiel predicted that Nebuchadnezzar would be the one to destroy it ( 26:7) and
more thoroughly and finally than was the case when Alexander captured it. Tyre was famous
in later Roman times for its university, and the Lebanese town of Sur is located there at this
very day.]

Ezekiel surpassed Isaiah in his scenario for the destruction of Egypt ( 29:10-13). The oft-
conquered Egypt was not only never made desolate, but its population was never dispersed (
30:23, 26), so any "regathering" ( 29:13) thus becomes moot. Chapter 30 issued more

Volume 1990 - 2002 Issue


Page 560 of 2049
Skeptical Review Edited by Farrell Till
captivity predictions for various countries and cities, none of which is known to have
occurred. And how about 32:5-6, wherein Ezekiel predicted that dead bodies would fill up the
valleys, and blood would rise to the tops of the mountains? To believe this would render one
as much an idiot as a lunatic, for it could not come true even if one tried to make it so.

Ezekiel went on to predict that the mountains of Israel would be made so desolate that no one
would pass through them ( 33:28). Another clear prophetic failure! He echoed Jeremiah's (
30:9) little-known prophecy of the resurrection of David in 34:23-24 and 37:24, who would
be immortal ( v:25). He predicted that the lost tribes of Israel would be regathered ( 36:24;
37:21), an utter impossibility once that they had been absorbed into neighboring cultures. His
fanciful imagination even had their dead bodies coming out of the graves in 37:4, 12, an
image later appropriated in Matthew's unbelievable, solitary, and unsubstantiated account of
resurrected saints on the day Jesus was crucified ( Matt. 27:52-53).

Jesus, although primarily a moral teacher, also issued a few prophecies. Most significant was
the expectation that his second coming would happen in short order ( Mark 9:1) and before
the original generation of his disciples had died off ( Matt. 23:36; 24:34). This was a false
expectation, plain and simple, that cannot be made true at any future time, and it is sufficient
to prove that Jesus was neither infallible nor God.

It's amazing that with every opportunity to cook the books, the Bible still possesses so many
prophetic failures. Equally amazing is that these failures go unnoticed by believers. But by the
Bible's own criteria, the most revered prophets of all were false. Their track record is an
abysmal one. This should be enough to disabuse any rational mind from the notion that the
so-called prophets were inspired by God, but fundamentalists are notoriously resistant to
letting go of untenable ideas. If fundamentalists need a handy excuse for the prophet's shoddy
performance, they might take solace in Ezekiel 14:9 --"And if the prophet be deceived when
he hath spoken a thing, I the Lord have deceived that prophet." The prophet is thereby off the
hook. But, of course, that only makes God look mighty fishy himself.

(Stephen VanEck, Route One, Box 62, Rushville, PA 18839-9702.)

Creationists Bugging You?


Dave Matson, whose articles have frequently appeared in TSR, has written a thorough
refutation of the major young-earth arguments used by Kent Hovind, a creationist lecturer and
debater. Since other young-earth creationists use some of the same "arguments," this would be
a useful book to have. It is a comb-bound book with 103 pages and a seven-page
bibliography. Matson has thoroughly documented his information, which has been written
simply enough for scientific laymen to understand his rebuttal arguments. The book is priced
at $14.50, but Matson will give a free copy to those who sell four of them. If you want a disk
copy in ASCII, this is available for $7.

Volume 1990 - 2002 Issue


Page 561 of 2049
Skeptical Review Edited by Farrell Till
In addition to this book, Matson has written booklets on other subjects related to evolution,
radiometric dating, and biblical errors. These are too numerous to list, but additional
information and prices will be sent on request. Dave Matson's address is P. O. Box 61274,
Pasadena, CA 91116. His e-mail address is 72234.330@compuserve.com.

[Note: Dave Matson's refutation of young-earth arguments is available at


http://www.infidels.org/library/modern/dave_matson/young-earth/]

Evaluating Historical Claims


Farrell Till
The biblical characters presented as emissaries of God were almost always miracle-workers.
They parted the water of seas and rivers; they walked on water; they commanded the sun to
stand still in the sky; they healed the blind and the deaf; they raised the dead. Nothing, it
seems, was impossible for them to do.

The Bible is filled with tales of such fabulous deeds as these, and fundamentalist Christians
believe that every one of these stories of miraculous achievements is literally true. If the Bible
says that the prophet Elisha retrieved an iron axe head that had fallen into the Jordan River by
making the axe head float ( 2 Kings 6:7), then fundamentalist Christians insist that this
literally happened. If the Bible says that the body of a dead man whom a band of Moabite
marauders cast into Elisha's tomb revived and stood upon its feet when it touched the bones of
Elisha ( 2 Kings 13:20-21), then fundamentalist Christians insist that this literally happened.
If the Bible says that a donkey conversed with its owner in a human voice ( Num. 22:28-30),
then fundamentalist Christians insist that this literally happened. If the Bible says that an
earthquake opened the graves in a cemetery after which the dead people in the opened graves
revived and went into the city of Jerusalem ( Matt. 27:51-53), then fundamentalist Christians
insist that this literally happened. If the Bible says... but why continue? We could fill this
entire issue with examples of other events just as fabulous as these that the Bible presents as
actual historical occurrences--all of which fundamentalist Christians believe literally
happened exactly as recorded.

In accepting the literal truth of stories like these, fundamentalist Christians accord the Bible a
privileged status that they deny the literature of other nations contemporary to biblical times.
Belief in the supernatural was commonplace back then, and so the literature of the times
reflected that belief. The Jewish historian Josephus, for example, claimed that during the feast
of unleaven bread just before the Jewish-Roman wars, a light so bright shined around the
temple altar at the ninth hour of the night that it gave the appearance of "bright day time" for
the space of half an hour (Wars of the Jews 6:5.3). He reported that a heifer being led to the
altar at the same festival gave birth to a lamb in the midst of the temple and that the eastern
gate of the temple, which was so "vastly heavy" that 20 men had been needed to close it, was
seen to open "of its own accord about the sixth hour of the night" (Ibid.). He went on to report

Volume 1990 - 2002 Issue


Page 562 of 2049
Skeptical Review Edited by Farrell Till
that a few days after the feast, just before sunset "chariots and troops of soldiers in their armor
were seen running about among the clouds and surrounding the city" (Ibid.).

Not even radical fundamentalists believe that these events actually happened, even though the
works of Josephus contain some of the same miraculous claims that are in the Old Testament.
His Antiquities of the Jews told of Noah's ark through which life on earth was saved from a
great flood (1:3.2-5), of God's confusing of tongues at the tower of Babel (1:4.3), of God's
appearance to Moses in a burning bush (2:12.1), of Moses' parting of the Red Sea (2:16.3),
and of many other miraculous acts that are also recorded in the Bible. Fundamentalists, of
course, believe that if Josephus recorded stories of miraculous deeds that have their parallels
in the Bible, then they should be believed insofar as they agree with the biblical accounts, but
if Josephus wrote about miraculous deeds that don't have parallels in the Bible, like those
mentioned earlier, then they may be rejected.

To say the least, there is an inconsistency in this approach to evaluating historical claims. For
one thing, Josephus was far removed from the time of Noah and Moses and, therefore, had no
opportunity to investigate firsthand the miraculous feats that allegedly happened in those
days. On the other hand, Josephus, an actual participant in the Jewish-Roman wars, was alive
and in Jerusalem at the time that the altar in the temple allegedly shined like the light of day at
the ninth hour of night, when the heifer gave birth to a lamb in the temple, and when the
heavenly chariots and soldiers were seen running about in the clouds. He had the opportunity
to interview witnesses and gather firsthand information about the events. Whether he actually
did so is not known, but at least these were events that allegedly happened right in his midst,
so if one is faced with the choice of believing either the miraculous claims that Josephus made
for his own time or those that had presumably happened centuries before, it would be more
reasonable to believe the claims that he had at least had the opportunity to investigate
personally.

The truly rational person, of course, will accept none of the fabulous deeds that Josephus
wrote about, whether they have their parallels in the Bible or not, because rational people
realize that the Bible is no different from the other literature of its time. The people of those
times, in all nations, believed that miracles happened routinely. The Roman historian
Suetonius, for example, recorded as a fact that while Roman magistrates publicly argued
about where to take the body of Julius Caesar to be cremated, two "divine forms" came down
with torches and set fire to the bier on which Caesar's body was lying in state (The Twelve
Caesars, Penguin, 1979, p. 52). He reported that Caesar's "soul" was seen as a comet for
seven consecutive days about an hour before sunset (Ibid., p. 53). He reported that some had
seen the spirit of Augustus Caesar ascending to heaven in the crematory flames (Ibid., 111).
Suetonius told of a woman named Claudia, who to prove "her perfect chastity" prayed to
refloat a boat grounded in a mud-bank on the Tiber river, "and did so" (Ibid., p. 114). A
footnote in the Penguin version of the book dates this event at 204 B. C., so it is unlikely that
a woman living in Rome at that time would have been praying to Yahweh, the Hebrew god.
Bible fundamentalists, therefore, would say that if the boat in this story did actually float free
from the mudbank after Claudia's prayer, the pagan prayer had had nothing to do with it. On
the other hand, they would argue with their dying breath that Peter's prayer to the "true" God
was directly responsible for raising Dorcus from the dead (Acts 11:40), a feat that would be
far more difficult to accomplish than floating a boat free of a mud-bank.

Volume 1990 - 2002 Issue


Page 563 of 2049
Skeptical Review Edited by Farrell Till
Bibliolaters like to talk about the "uniqueness" of the Bible, but actually it is a rather ordinary
collection of writings for the times that produced it. The Old Testament often speaks of
Yahweh's leading the Israelites to victory over their enemies, but the literature of surrounding
nations tells of gods who led their people to victory too. The Moabite Stone, discovered in
1868 east of the Dead Sea, recorded the victories that the god Chemosh had led Mesha, a
Moabite king mentioned in 2 Kings 3, to win over his enemies. The text reads much like a
page from the Old Testament:

I am Mesha, son of Chemosh..., king of Moab, the Dibonite. My father was king over Moab
thirty years and I became king after my father. And I made this sanctuary for Chemosh at
Qrchh, [a sanctuary] of salvation; FOR HE SAVED ME FROM ALL THE KING AND LET
ME SEE MY DESIRE UPON MY ADVERSARIES. Omri, king of Israel, he oppressed
Moab many days, FOR CHEMOSH WAS ANGRY WITH HIS LAND. And his son
succeeded him and he too said, "I will oppress Moab." In my days he spoke [thus], and I saw
my desire upon him and upon his house, when Israel perished utterly forever. And Omri had
taken possession of the land of Medeba and [Israel] dwelt in it his days and half the days of
his son, forty years; BUT CHEMOSH DWELT IN IT IN MY DAYS. And I built Baal-Meon
and made it in the reservoir, and I built Qaryaten. And the men of Gad had long dwelt in the
land of Ataroth, and the king of Israel had built Ataroth for himself. But I fought against the
town and took it and I slew all the people of the town, A SPECTACLE FOR CHEMOSH and
Moab. And I brought back from there the altar-hearth of David and I dragged it before
Chemosh at Qeriyoth. And I settled there the men of Sharon and the men of Mchrt. AND
CHEMOSH SAID TO ME, "GO, TAKE NEBO AGAINST ISRAEL." And I went by night
and fought against it from the break of dawn till noon; and I took it and slew all: seven
thousand men, boys, women, and [girls] and female slaves, FOR I HAD CONSECRATED IT
TO ASHTAR-CHEMOSH. And I took from there the vessels of Yahweh AND DRAGGED
THEM BEFORE CHEMOSH. And the king of Israel had built Jahaz and he dwelt in it while
fighting against me. BUT CHEMOSH DROVE HIM OUT BEFORE ME. And I took from
Moab two hundred men, all of them leaders, and led them up against Jahaz and took it to
annex it to Dibon. I built Qrchh, the walls of the parks and the walls of the mound; and I built
its gates and I built its towers; and I built the king's house; and I made both the reservoirs for
water inside the town. And there was no cistern inside the town of Qrchh, so I said to all the
people, "Make yourselves each one a cistern in his house." And I had ditches dug for Qrchh
by prisoners of Israel. I built Aroer and I made the road by the Arnon. I built Beth-bamoth, for
it was destroyed; I built Bezer, for it was in ruins, with fifty men of Dibon, for all Dibon is
under my authority. And I reigned [over] hundreds of towns which I had annexed to the
country. And I built... Medeba and Beth-Diblathen and Beth-Baal-Meon, and I led up there
the breeders of the sheep of the land. And as for Hauronen, there dwelt in it.... CHEMOSH
SAID TO ME, "GO DOWN, FIGHT AGAINST HAURONEN." And I went down... [and
there dwelt] in it Chemosh in my days... (D. Winton Thomas, Documents From Old
Testament Times, Harper & Row, pp. 196-197, emphasis added).
There are gaps in the text, represented by the ellipses, and the absence of vowels in the
Semitic dialect in which it was written makes the pronunciation of some place names
undeterminable, but the text itself is sufficient to show that the Moabite religion was very
much like the Hebrews'. The Hebrews thought that their god Yahweh was actively involved in
their daily affairs, but the sections of the Moabite text emphasized in bold print show that the
Moabites believed the same about their god Chemosh.

Volume 1990 - 2002 Issue


Page 564 of 2049
Skeptical Review Edited by Farrell Till
The Hebrews believed that their god saved them and their kings from their enemies: "Thus
Yahweh saved Hezekiah and the inhabitants of Jerusalem from the hand of Sennacherib the
king of Assyria, and from the hand of all others, and guided them on every side" ( 2 Chron.
32:22; see also Ex. 14:30; 1 Sam. 10:17-18; Ps. 44:7). But as the first point of emphasis in the
Moabite text shows, King Mesha believed that Chemosh "saved [him] from all the kings and
let [him] see [his] desire upon [his] adversary." In times of adversity, the Hebrews thought
that they were being punished by Yahweh: "And the children of Israel did that which was evil
in the sight of Yahweh, and Yahweh delivered them into the hand of Midian seven years" (
Judges 6:1; see also Judges 2:11-12; 4:1; 10:7-8). King Mesha of the Moabites likewise
believed that his god punished the people when he was miffed at them: "Omri, king of Israel,
he oppressed Moab many days, for Chemosh was angry with his land." According to the Old
Testament, Yahweh at times even talked directly to the Israelite kings to give them battle
instructions: "Therefore David inquired of Yahweh, saying, 'Shall I go and attack these
Philistines?' And Yahweh said to David, 'Go and attack the Philistines, and save Keilah'" ( 1
Sam. 23:2). In the text of the Moabite Stone, however, we see that the god Chemosh at times
talked directly to kings too: "And Chemosh said to me, 'Go, take Nebo against Israel.'"

According to King Mesha's account engraved on the Moabite Stone, he "went by night and
fought against it [Nebo] from the break of dawn till noon." Mesha claimed that he took Nebo
and "slew all: seven thousand men, boys, women, and [girls] and female slaves," so, yes,
indeed, this reads very much like a page out of the Old Testament, right down to the details of
the massacre of the captives. It just shows how very much alike the nations of that
geographical region were at that time. They all thought in terms of "our god can lick your
god," and when they won, they all dealt quite harshly with the losers. The shame of Bible
fundamentalists is their absurd belief that slaughters like the one King Mesha described were
immoral acts of superstitious heathens, but massacres executed by the Israelites were morally
right because they were commanded by Yahweh. The Israelites were separated from the
Moabites by only the Dead Sea and in some places by just the Jordan River, so reasonable
people, living in enlightened times, should have enough common sense to realize that if the
Moabite god Chemosh wasn't real, then neither was the Hebrew god Yahweh, who was so
much like Chemosh in temperament and character.

If we assume that the battle for Nebo happened as Mesha described it, one might wonder why
the Israelites living in Nebo didn't question why an omniscient, omnipotent deity like Yahweh
would have allowed an upstart pagan like Mesha to defeat his specially chosen people, but the
Hebrew prophets would have had a perfectly sensible explanation. They had done something
to displease Yahweh, and so he had used Mesha to punish them for their evil. You have to
give the sorcerers and prophets of that era credit where credit is due, because they knew how
to cover all of their bases. If something fortuitous happened, usually victory over their
enemies, it was a sign that they were pleasing Yahweh; if something calamitous happened, it
meant that Yahweh was punishing the people for wrongdoing. Either way Yahweh came out
smelling like a rose, and it is all so ridiculous that I find it absolutely incredible that intelligent
people today can take such stuff as this seriously.

All of this brings us back to a point that I have made in past articles: the only sensible way to
evaluate claims that one has no firsthand knowledge of is to apply a rule of evidence that Carl
Lofmark explained in What Is the Bible?

Volume 1990 - 2002 Issue


Page 565 of 2049
Skeptical Review Edited by Farrell Till
When you lack evidence, the only way to decide whether or not to believe something is to
ask: Is it likely? If you tell me a bird flew past my window, I will probably believe you, even
though I did not see it myself and I have no evidence. That is because such a thing is likely. I
have seen it happen before. It is more likely that a bird flew past my window, than that you
are deceiving me. But if you tell me a pig flew past my window, I will not believe you,
because my past experience tells me that such things do not happen, and so I presume that
what you reported is false. Thus, where there is no evidence we have to rely on our own past
experience of the sort of things that really happen (pp. 41-42).
If we do not apply this rule of evidence to historical claims, we will be driven to an extreme
that Thomas Paine described in Age of Reason:
As to the ancient historians, from Herodotus to Tacitus, we credit them as far as they relate
things probable and credible, and no farther; for if we do, we must believe the two miracles
which Tacitus relates were performed by Vespasian, that of curing a lame man and a blind
man, in just the same manner as the same things are told of Jesus Christ by his historians"*
(Part Two, 200th Anniversary Edition, p. 62).**
This method is the only rational way to evaluate historical claims. If we apply it impartially,
we will have to reject the fantastic claims in the Bible for the same reason that we reject the
many fantastic claims that are in other ancient documents contemporary to biblical times.

*Actually, Suetonius rather than Tacitus attributed these miracles to Vespasian (The Twelve
Caesars, p. 284). For additional information on the topic of evaluating historical claims, see
Thomas Wheeler's letter on pages 13-14 of this issue.)

** This paperback edition of Paine's book can be purchased for only $5 from Stephen VanEck
whose address is published at the end of his article on page 9 of this issue.

Cost Increases

As a nonprofit organization, Skepticism, Inc., has been entitled to mail all editions of The
Skeptical Review at the special nonprofit third-class rate, and to mail debate transcripts, video
tapes, back issues, etc. at a specially low library rate. January 1st these rates were raised
substantially. The library rate was increased 75% so that now it is actually cheaper to mail
some items first-class. Accordingly, we will have to raise our price on debate transcripts from
$2.50 to $3.00 and the rental fees on video tapes to $3.00 for the 8-hour (2-cassette) debates
and $2 for the shorter singlecassette debates. Please apply these prices to all future orders.
The cost of back issues will remain $1 per copy.

New Address

Because of the increased volume in mail, we had to change to a larger mail box. This new
address has been printed on the last four issues of TSR, but we still receive a lot of mail
addressed to the old box number. As the return address on the back of this issue indicates, our
new box number is 717.

Volume 1990 - 2002 Issue


Page 566 of 2049
Skeptical Review Edited by Farrell Till

Leaning Over Backwards for God


by Farrel Till
For some time, I have been planning an article with the title I have given to this one. My
purpose was going to be to examine the extremes that bibliolaters resort to in their defense of
the untenable belief that the Bible is the inerrant word of an omniscient, omnipotent deity who
lived in a tent that desert nomads carried around with them 3,500 years ago (Ex. 25:8, 22;
29:42-45; Num. 5:3). We are all familiar with the colloquial expression "leaning over
backwards," which is often applied to those who go to uncommon, and sometimes drastic, ex-
tremes to achieve a goal or perhaps to accommodate someone who is hard to please. "I lean
over backwards to satisfy you," an exasperated wife might say to a critical husband. She
means, of course, that she exceeds reasonable expectations in trying to please her husband.

Having had the title "Leaning Over Backwards for God" in mind for some time, when Marion
Fox's long-awaited defense of Deuteronomy 23:2 arrived, I decided to brush the title off and
finally write my article. Mr. Fox's "resolution" of the "bastard problem" provides an excellent
background to show how that Christian fundamentalists go to unreasonable extremes in their
efforts to prove that the Bible is the inerrant word of God. They truly "lean over backwards
for God."

Let's notice first that Fox's article really solved nothing, because he arrived at no firm
conclusions about what Deuteronomy 23:2 means. Like most Bible fundamentalists, he
apparently belongs to the any-interpretation-will-do school of hermeneutics. In other words,
when it comes to resolving a Bible contradiction, he is satisfied with any explanation that
sweeps the problem under a rug of could- have-beens. Whether any of the explanations
actually capture the intended meaning of a disputed passage doesn't matter. If the inerrancy
doctrine is techni- cally preserved by a how-it-could-have-been explanation, that is the
important thing. Thus, truth becomes incidental to doctrine and tradition, and that alone
speaks volumes about the validity of this familar method of "resolving" biblical "difficulties."

This hermeneutic approach cheapens the Bible far more than the "liberal" interpretations of
scripture that fundamentalists openly despise. So-called liberal theologians reject the
inerrancy view of the Bible but still contend that it has an overall value on "higher levels" that
transcend archaic inerrancy views. To a liberal, determining the writer's intended meaning of
a given passage should be a primary goal of the Bible reader. If the meaning that honest and
objective methods of interpretation lead to should conflict with another scripture, the liberal
theologian will try to understand the reason for this, perhaps see it as a normal development in
the course of human efforts to discover God. A fundamentalist, on the other hand, must
believe that the intended meaning of a "given passage" isn't nearly as important as an
interpretation that doesn't disagree with what was written anywhere else in the Bible.
Ultimately, this approach to hermeneutics strips the Bible of all value. If it is indeed the "word
of God," what does it matter, because it means not what the writers intended as they were

Volume 1990 - 2002 Issue


Page 567 of 2049
Skeptical Review Edited by Farrell Till
writing but what- ever the cleverest apologist wants it to mean? Such an apologist could
decree that the "given passage" under consideration means X today, but if biblical criticism
should afterwards discover that X conflicts with another text that had hitherto gone unnoticed,
then a clever apologist of that time will go to work and decide that the "given passage" didn't
mean X after all but "could have meant" Y, which would eliminate the conflict. This type of
hermeneutic juggling has been going on for generations, and if it gives Mr. Fox and his
inerrantist cohorts comfort, all I can say is to each his own. Intellectual integrity happens to be
more important to me than that.

In the matter of Deuteronomy 23:2 , Mr. Fox has presented five how-it-could-have-been
solutions to the problems posed by Yahweh's ban on bastards entering his assembly. He used
the usual hit-and-run tactic of fundamentalists, which is long on assertions but short on
supporting evidence, so all five of his possible solutions were presented in little more than a
single page with about a third of that space devoted to some general comments about
"common errors made by Atheists and Agnostics." In responding to Mr. Fox's article, I will
not use his assert- without-proof approach. I will instead examine his points in depth, and to
do this, some rather tedious analyses of genealogical passages will be necessary to show that
many of Mr. Fox's "arguments" are baseless quibbles. Complete responses to all five of his
"solutions" cannot be done in the space available at this time, so this article will focus only on
the genealogy of David to show that Israel's most famous king should have fallen under the
ban on bastards stated in Deuteronomy 23:2 . A follow-up article will be published in the next
issue to examine the other "solutions" Mr. Fox proposed.

His first argument was that "David might have been the generation that was allowed to enter
the assembly (the tenth generation)." This familiar attempt to circumvent the problem has
been proposed by those who have noticed that David was the 10th generation in the lineage of
Perez (Ruth 4: 18-22), if Perez is considered the first generation. Since Deuteronomy 23:2
says, "(E)ven to the tenth generation shall none of his [the bastard's] enter into the assembly
of Yahweh," this could mean that David, who was a 10th-generation bastard, would have
been entitled to enter into the assembly. However, such an interpretation of the statement
ignores obvious intent. The expression "even to the tenth generation" didn't mean that the
generations descended from a bastard should be counted so that the tenth generation could be
admitted to the assembly. It was simply an intensifying expression that meant "forever." In its
definition of the word "bastard," Eerdmans Bible Dictionary agrees with this view:

A name given to those begotten in adultery or incest (Heb. mamzer, Deut. 23:2; NIV
"born of a forbidden marriage"). This violation of marriage was such a serious offense
that such persons and their descendants were denied admission to the assembly of the
LORD, first in the temple and later in the synagogue, to the "tenth generation" (Deut.
23:2)--i.e., forever. (1987, p. 129, emphasis added).

The meaning of "forever" will be more apparent in the expression if we keep in mind that the
word to in most English translations was not in the original Hebrew text. Literally, the
Hebrew meant, "A bastard enters not [Hebrew didn't have a future tense] into the assembly of
Yahweh, even the tenth generation of him enters not into the assembly of Yahweh," and this
is the way that Young's Literal Translation of the Bible renders it. Hendrickson's Interlinear
Bible translates it the same way with to enclosed in parentheses, which according to the

Volume 1990 - 2002 Issue


Page 568 of 2049
Skeptical Review Edited by Farrell Till
preface is intended to identify words that may be implied but were not actually in the Hebrew
text.

The misimpression (if indeed it is a misimpression and not just an attempt to circumvent a
textual embarrassment) results from the fact that we are accustomed to "even to" or "even till"
having the meaning of "up until," but even in English it doesn't always have that meaning. If a
young man said to his girlfriend, "I will love you even till I'm old and gray," he obviously
would mean that he would love her forever. What young lady hearing a pledge like this would
understand it to mean that her boyfriend would love her until he was "old and gray" and then
he would stop loving her?

Of course, no inerrantist defense of a problem like this would be complete without a gaps-in-
the-genealogy argument, so Mr. Fox didn't disappoint us. "In order to know that his argument
is sound," Fox said, "Farrell must know that there are no gaps in the genealogies." For those
who may be new to the inerrancy controversy, I should explain that several genealogy-based
discrepancies exist in the Bible. What is said in genealogy A disagrees with what is said in
genealogy B or passage C or both, and what is said in genealogy D disagrees with what is said
in genealogies A and B and passage C, etc. Exodus 12:40, for example, says that the Israelites
had "sojourned" in Egypt 430 years, but this is hard to reconcile with a genealogy in Exodus
6:16-20, which indicates that Moses and Aaron were grandsons of Kohath, a son of Levi, who
had already been born when Jacob took his family into Egypt (Gen. 46:11). The problem here
is obvious. If Kohath, who had already been born before the Egyptian sojourn began, lived to
be only 133 (Ex. 6:18) and if Kohath's son Amram (the father of Moses and Aaron) lived only
137 years (Ex. 6:17-20), then how could 430 years have passed between the entry of the
Israelites into Egypt and their exodus under the leadership of the 80-year-old Moses (Ex.
7:7)? "Simple," say the inerrantists, "there are gaps in the Exodus 6 genealogy and all others
that indicate that Moses and Aaron were only grandsons of Kohath." There were some
generations that the writers just didn't list in their genealogies. Jerry Moffitt and I debated this
very issue in the Winter and Spring 1990 editions of TSR, and this is exactly the position he
took.

So what Marion Fox is arguing in the matter before us is that David could possibly have been
more than 10 generations removed from Perez, but we just don't know it because the
genealogists who recorded the Judah-Perez lineage could have skipped some generations. In
other words, Fox's strategy is to claim a "possible" solution for which there is no proof and
then to challenge me to prove that his "solution" is not true. In this case, I must prove that the
genealogists who recorded the Judah-Perez lineage did not skip any generations.

I find it hard to understand why a rational person would cling to beliefs that drive him to such
improbable defensive tactics as this. Let's imagine a small community in Illinois where
everybody knows John Jones, who has just retired after having served as the local high school
principal for 30 years. Now let's imagine that an unmasked man walks into the bank in this
town, pulls a gun, tosses a black gym bag onto the counter, orders the five tellers on duty to
fill the bag with money, and then flees in a blue, 1991 Chevrolet Cavalier bearing the license
number HGE-692.

Volume 1990 - 2002 Issue


Page 569 of 2049
Skeptical Review Edited by Farrell Till
The police investigate and find that the car just described was registered to John Jones, and
the tellers, questioned individually, all say that they recognized the robber as John Jones.
Three of the tellers are graduates of the high school where Jones was principal, and the other
two had personally served Jones many times when he was in the bank either to deposit or
withdraw money. Ten other employees and five customers who had been in the bank at the
time of the robbery testify that they all had known John Jones personally for several years and
that they had recognized the robber as Jones. The tapes recorded by the surveillance cameras
were viewed by four different police officers who had personally known Jones for several
years, and they all agreed that Jones was the man shown on the tapes wielding a gun and
perpetrating the robbery. An audit revealed that $52,540 had been taken in the robbery.

That night, John Jones is arrested at O'Hare Airport in Chicago as he was buying a ticket to
Brazil. In the airport parking lot, the police find a blue 1991 Chevrolet Cavalier with the
license number HGE-692. In the trunk, they discover a black gym bag and a .38 caliber pistol,
and in carry-on baggage that Jones had with him, the police found $52,000. In his wallet,
Jones had $320.

The situation certainly looked bad for Jones, but after all of the evidence and testimony was
presented at his trial, Jones's lawyer argued that unknown to the people who had testified
against him, Jones had an identical twin brother. The lawyer told the court that Jones and his
twin had been born in Timbuktu, while his parents were serving as missionaries. Shortly after
the birth of the twins, the parents had been killed in a plane crash and the twins separated by
adoption. All contact between the twins had been lost until the week of the robbery when
Jones's twin brother had suddenly shown up in a surprise visit. The twins had not left Jones's
house during the visit, so that was why no one had seen them together. On the morning of the
robbery, Jones and his brother had started for Chicago in Jones's car so that the brother could
catch a flight to Katmandu, Nepal, where he was now living. Along the way, the brother
discovered that he had left his passport at Jones's house, or so he had said, and Jones, not
wanting to make the tiring drive back home, had let his brother use his car to drive back for
the passport, while Jones himself had attended an early afternoon movie in Joliet. Obviously,
the twin brother had taken advantage of Jones's trust and had committed the robbery. So why
was Jones trying to go to Brazil at the time of his arrest? Well, the lawyer explained, he was
simply fulfilling a life-long ambition. He had always wanted to visit Brazil but never could
because of his job. Now that he was retired, however, the opportunity was there, and he was
simply taking it. Since Jones had had to drive his brother to Chicago anyway, he had decided
that it would be as good a time as any to go to Brazil. After all, is there any crime in wanting
to visit Brazil? And the money in the carry-on bag? "That's not hard to explain," the lawyer
said. "Mr. Jones had squirreled the money away over the past several years expecting
someday to make his dream trip." The lawyer explained that Jones had kept the money at
home rather than depositing it in a savings account, because having it where he could count it
each night encouraged his saving habits more so than if he had deposited it in a bank.
Eccentric, yes, but there is no law against eccentricity. "Is it unreasonable," the lawyer
concluded, "to think that a man who had never married and therefore had never had the
financial responsibility for anyone but himself could save fifty-two thousand dollars over a
period of several years?" The lawyer concluded with a challenge for the prosecution to prove
that the details in Jones's alibi had not happened exactly as they had been presented to the

Volume 1990 - 2002 Issue


Page 570 of 2049
Skeptical Review Edited by Farrell Till
court. "To convict my client," the lawyer said, "the prosecution must know, not just think or
wish but know, that the details in his alibi are wrong."

I apologize to the readers for the tediousness of this example, but I have intentionally made it
tedious in order to make a serious observation about the apologetic tactics of Mr. Fox and his
inerrantist cohorts. As compelling as the evidence was against Jones in my example, who
could prove that the details of Jones's defense were absolutely not true? Nothing that Jones's
lawyer said in his defense was absolutely not possible, yet a juror would have to be gullibly
naive to accept such an unlikely story over the compelling evidence against Jones. Who
would dare base important daily decisions on such unlikely scenarios?

I submit that this example is parallel to what Bible inerrantists do. Confronted with
contradictions or discrepancies when the face-value meaning of the biblical text is accepted,
inerrantists will postulate far-fetched, how-it-could-have-been scenarios and then challenge
critics of the Bible to prove absolutely that the how-it-could-have- been "solutions" are
absolutely not possible. Mr. Fox resorted to this tactic when he said at the conclusion of his
article, "Farrell Till must know that all of the above explanations are wrong, not just think
they are wrong or wish them to be wrong" (emphasis added).

To show the absurdity of Mr. Fox's apologetic methods, I will issue to him the same challenge
that I have issued to other inerrantists, none of whom has ever accepted. If he will send me an
example of what he perceives to be a contradiction or discrepancy that he has found in any
written document, I will use his "apologetic" methods to prove that no contradiction or
discrepancy exists. His example may come from the Koran, the Book of Mormon, the Avesta,
or any other holy book, or it can come from any secular document--the Iliad, the Aenid, a
Shakespearean play; it doesn't matter. It can be an example from any written document whose
author(s) is now dead or otherwise unavailable to explain his/her intention. If he will send me
the alleged contradiction from any such document, I will use his methods to show that no
contradiction exists. I will do this by simply postulating "possible" figurative meanings or
how-it-could-have-been scenarios, such as copying or printing errors, different methods of
calculating or the rounding off of figures, the nonexistence of original autographs,
phenomenological or accommodative language, etc., etc., etc.-- and thereby prove that no
contradiction or discrepancy exists. Mr. Fox's task will then be to prove that he knows, not
just wants or wishes but knows, that none of my explanations is possible.

Mr. Fox wants us to consider some "common errors made by atheists and agnostics," but can
any errors in the reasoning of atheists and agnostics be more ridiculous than the "logic" that
bibliolaters use to protect their precious inerrancy doctrine? Their logic makes contradiction
and discrepancy impossible, but anyone with common sense knows that contradictions and
discrepancies are hard realities.

As for "gaps" in David's genealogy, it really doesn't matter whether there are gaps in it or not.
The intent of the language in Deuteronomy 23:2 was to ban a bastard's descendants from the
assembly permanently or forever, as noted in Eerdman's Bible Dictionary, so whether David
was 10 generations or 50 generations removed from Perez is immaterial to the issue. The ban
would still have applied had it really been a law written by Moses that was meant to apply to
all Israelites impartially. Bible scholars know, of course, that the book of Deuteronomy wasn't

Volume 1990 - 2002 Issue


Page 571 of 2049
Skeptical Review Edited by Farrell Till
even written until after David's reign, but that is another issue that Mr. Fox may want to
debate later. For now I will limit myself to analyzing Deuteronomy 23:2 on the assumption
that it was a law that Yahweh decreed through Moses.

The ban on bastards if evaluated in context should convince any reasonable person that the
writer did not mean that descendants of bastards should be banned for ten generations and
then with the tenth or eleventh be allowed to enter. They were to be banned forever. To
determine this obviously intended meaning, let's look at the passage in context:

He that is wounded in the stones [testicles] or hath his privy member [penis] cut off
shall not enter into the assembly of Yahweh. A bastard shall not enter into the
assembly of Yahweh; even [to] the tenth generation shall none of his enter into the
assembly of Yahweh. An Ammonite or a Moabite shall not enter into the assembly of
Yahweh; even [to] the tenth generation shall none belonging to them enter into the
assembly of Yahweh forever: because they met you not with bread and with water in
the way, when ye came forth out of Egypt, and because they hired against thee Balaam
the son of Beor from Pethor of Mesopotamia to curse thee. Nevertheless Yahweh thy
God would not hearken unto Balaam; but Yahweh thy God turned the curse into a
blessing unto thee, because Yahweh thy God loved thee. Thou shalt not seek their
peace nor their prosperity all thy days forever. Thou shalt not abhor an Edomite; for he
is thy brother: thou shalt not abhor an Egyptian; because thou was a sojourner in his
land. The children of the third generation that are born unto them shall enter into the
assembly of Yahweh (vv:1-8).

Although this is technically off subject, I feel compelled to say that this passage is an insult to
the intelligence of civilized people. It depicts Yahweh, Mr. Fox's omniscient, omnipotent,
omnibenevolent god, as an outrageously prejudiced grudge-bearer. The passage in its entirety
banned people from the assembly for acquired physical imperfections (crushed testicles and
amputated penises, v:1), circumstances of birth (illegitimacy, v:2), and ethnic origin
(Ammonites, Moabites, Edomites, and Egyptians specifically singled out for exclusion, vv:3-
8). Rather than recognize this passage as the primitive, barbaric tripe that it is, Mr. Fox leans
over backwards to defend it. If he sees virtue in that, let him. I will pity him but certainly not
envy him.

In support of my position that saying "even a tenth generation of him [the bastard] enters not
into the assembly of Yahweh" was intended to mean forever, I ask the readers to compare the
expression with the next verse, which banned Ammonites and Moabites from the assembly:
"(E)ven to the tenth generation shall none belonging to them enter into the assembly of
Yahweh forever." By directly adding the word forever at the end of the statement, the writer
clearly showed that the expression "even [to] the tenth generation" didn't mean "up until" ten
generations and then after that the ban would be lifted, but rather that the ban was to be
permanent, one that was to last forever. To emphasize that the ban was to last forever, he
repeated it after bigotedly explaining why Yahweh wanted Ammonites and Moabites
excluded from his assembly: "Thou shalt not seek their peace nor their prosperity all thy days
forever" (v:6). Yahweh was a veritable paragon of compassion and tolerance, wasn't he?

Volume 1990 - 2002 Issue


Page 572 of 2049
Skeptical Review Edited by Farrell Till
The ban on Edomites and Egyptians (v:7) is also significant, because it shows that when
restrictions for a specific number of generations were intended, the limitations were expressly
stated: "The children of the third generation that are born unto them [proselytized Edomites
and Egyptians] shall enter into the assembly of Yahweh" (v:8). Surely, then, if the writer had
meant that bastards were to be banned from the assembly only up until ten generations, he
would have been as specific about that as he was in stating the limitations on Edomites and
Egyptians. So Mr. Fox's argument is without merit. Even if there were "gaps" in the Perez-
David genealogies, David would still have fallen under the ban on bastards, because the ban
was to last forever.

As for the actual number of generations between Perez and David, I'm going to surprise Mr.
Fox and admit that if there actually was a person named Perez, who was born at the time
claimed in Genesis 38:29, and if King David, who lived at the time claimed in the books of
Samuel, was a direct descendant of this Perez, then there surely were more than just ten
generations separating Perez and David. In Mythology's Last Gods: Yahweh and Jesus,
William Harwood shows how that Bishop Ussher's chronology of the Old Testament would
have required each of David's ancestors from Perez through Jesse to have "reached the age of
eighty before fathering an heir" (p. 221). That is so unlikely that we can reasonably assume
that it just didn't happen.

So am I conceding this point to Mr. Fox? Not at all, because the biblical writers obviously
thought that only ten generations separated Perez and David or else they were so careless in
their writing that no rational person can believe that they were verbally inspired by an
omniscient, omnipotent deity. Either way the inerrancy doctrine suffers, and Mr. Fox loses.

The genealogy of Perez through David is listed four times in the Bible: Ruth 4:18-22, 1
Chronicles 2:4-15, Matthew 1:3-6, and Luke 3:31-33. Each time, the following generations
were listed: Perez, Hezron, Ram, Amminadab, Nahshon, Salmon, Boaz, Obed, Jesse, and
David. Luke's genealogy has Arni for Ram, but a footnote in most reference Bibles explains
that "many ancient authorities" have Aram, an apparent variation of Ram. We can well
imagine what inerrantists would say if anyone tried to argue that Luke's use of Arni or Aram
for Ram constituted a discrepancy, so no inerrantist is going to argue that Luke's genealogy of
David disagrees with the other three. So at this point, I will enter into evidence Exhibit A:
Whenever Bible writers recorded genealogies that extended from Perez through David, they
always listed the same ten names as those noted above, no more and no less. If biblical
genealogists had been aware that there were more than ten generations between Perez and
David, surely at least one of them would have filled in a gap or two that the others had left in
their records. Or did the omniscient, omnipotent Holy Spirit, who verbally inspired those
genealogists, intentionally complicate his "revealed truth" in this matter in order to test our
faith and identify those who have "good and honest hearts" (as if an omniscient deity wouldn't
already know those who have good and honest hearts)?

In addition to the four complete genealogies, there are partial genealogies that, considered
collectively, confirm that biblical writers believed that the ten generations listed above
constituted David's complete genealogy back to his ancestor Perez. The book of Ruth clearly
teaches that Ruth's levirate husband, Boaz, was the father of Ruth's son Obed, so there would
be no "gap" between Boaz and Obed. Ruth 4:17, after stating that Ruth's son was named

Volume 1990 - 2002 Issue


Page 573 of 2049
Skeptical Review Edited by Farrell Till
Obed, says that he [Obed] was "the father of Jesse, the father of David." The Bible so
explicitly presents David as the literal son of Jesse (1 Sam. 16:8-13) that Mr. Fox can't dare
argue that there was a "gap" between Jesse and David, so if Jesse was literally David's father,
then the author of Ruth surely meant that Obed was literally the father of Jesse. Hence, we
can reasonably conclude that the biblical writers did not think that there were any "gaps"
between Boaz and David. Otherwise, Mr. Fox must argue that the author of Ruth, within the
space of eight words, equivocated on the meaning of the word father, and that would be a
curious writing error for a verbally inspired author to make.

Genesis 46:12 listed Hezron and Hamul as sons of Perez who went into Egypt with Jacob, and
this agrees with a partial genealogy in Numbers 26:19-22, which says that Shelah, Perez, and
Zerah were sons of Judah. That the word sons was being used literally is confirmed by
Genesis 38. Verse 5 says that Judah's Canaanite wife bore him a son and called his name
Shelah, so Shelah was literally Judah's son. Verses 13-30 relate the story of the deception that
Tamar used to trick Judah into impregnating her. She gave birth to twin sons (vv:29-30), who
were named Perez and Zerah, so these twins were literally the sons of Judah. Therefore if the
word sons in Numbers 26:20 was used literally in reference to the relationship between Judah
and Shelah, Perez, and Zerah, we must conclude that the word sons was intended literally in
the next verse, which declared that the sons of Perez were Hezron and Hamul. Otherwise, Mr.
Fox must again argue that a verbally inspired writer equivocated over the space of just two
verses of scripture.

If Mr. Fox tries to argue that there was a "gap" between Perez and Hezron, he will find
himself in even more trouble than he now is. Biblical scholars have long recognized that the
chronology of Genesis 37 through 46 doesn't allow enough time for Perez to have fathered
even two sons, much less grandsons or great-grandsons, before the Israelite descent into
Egypt. This subject will be discussed in detail in a debate that will be published in a future
issue of TSR, so I will present the problem just very briefly here. Joseph was seventeen when
his brothers sold him into Egypt (Gen. 37:2), and he was 30 when pharaoh appointed him
chief food administrator (Gen. 41:46). After the seven years of plenty in Egypt and two years
of famine (or nine years later), Joseph was reunited with his brothers (Gen. 45:6), whom he
then brought into Egypt with his father's extended family. So Joseph was only about 39 or 40
when the Israelites went into Egypt. In other words only 22 or 23 years passed between the
selling of Joseph and the migration of the Israelites into Egypt, at which time Hezron and
Hamul were listed as two sons of Perez who went into Egypt (Gen. 46:12).

The chronological problem arises out of the fact that immediately after the selling of Joseph
was recorded by the Genesis writer, he said in 38:1 that "it came to pass at that time" that
Judah married his Canaanite wife, an event that precipitated all of the events recorded in the
chapter. Judah's wife bore him three sons. The first two sons grew up and married Tamar,
died without producing offspring, and then Judah's wife died, and Tamar tricked Judah into
impregnating her. [What this morally corrupt country needs is wholesome literature on our
bookshelves, like these stories we find in the Bible.] Tamar gave birth to the twins, Perez and
Zerah, and when the Israelites went into Egypt Perez had two sons, Hezron and Hamul. Can
any sensible person believe that all of this happened within the space of 22 or 23 years?

Volume 1990 - 2002 Issue


Page 574 of 2049
Skeptical Review Edited by Farrell Till
Confronted with a problem like this, surely Mr. Fox won't try to argue that there could have
been a "gap" between Perez and Hezron, so he is forced to admit that the Genesis writer at
least thought that Hezron was Perez's son and not his grandson or an even more distant
descendant.

Exodus 6:23 says that Aaron married Elisheba, "the daughter of Amminadab, the sister of
Nahshon." So if Elisheba was the daughter of Amminadab and the sister of Nahshon, that
would have made Amminadab and Nahshon father and son. If not, why not? Nahshon was an
important leader of the tribe of Judah during the exodus and wilderness wanderings, and he
was often referred to as "Nahshon the son of Amminadab" (Num. 1:7; (2:3; (7:12, (17;
(10:14). When the writer of Numbers called Nahshon the son of Amminadab, did he mean
son in a literal sense? According to Strong's Exhaustive Concordance, the word son [Hebrew,
ben] was used 143 times in Numbers. I have checked them all and found none that gives any
reason to believe that it was used in a broader sense than the literal meaning of son. In
(Matthew 15:22, for example, Jesus was called the "son of David," in a context in which the
word son obviously could not have been literal, but in Numbers there are no examples like
this. Instead, the writer repeatedly wrote in terms of so-and-so being the son of so-and-so. In
(1:20, for example, Reuben was called "the eldest son of Israel [Jacob]," which (Genesis
29:32 tells us that he literally was; Eleazar was called the son of Aaron ((4:16; (20:25, 26, 28),
and according to (Exodus 6:23, Eleazar literally was the son of Aaron. I could cite numerous
examples like these among the many other times that the writer used son, so the evidence
from Exodus and Numbers plainly indicates that the writer(s) thought that Nahshon was
literally the son of Amminadab.

So let's look at the complete genealogy again with certain of the names emphasized in bold
print: Perez, Hezron, Ram, Amminadab, Nahshon, Salmon, Boaz, Obed, Jesse, and David. I
have established that biblical writers at least thought that literal father-son relationships
existed between all of the names in bold print, so if there are any "gaps" in the genealogy,
they would have to fall between Hezron and Ram or Ram and Amminadab or between
Nahshon and Salmon or Salmon and Boaz. The biblical text leaves no reasonable cause to
claim that gaps could have occurred anywhere else.

(First Chronicles 2:3-13, however, dashes all hope that Mr. Fox may have of finding gaps on
either side of the positions occupied by Ram and Salmon in the Perez-David genealogy. Let's
notice first that the writer began by identifying the "sons of Judah" and then named Er, Onan,
and Shelah, who had been born to Judah's Canaanite wife ((Gen. 38:3-5), and then named
Perez and Zerah, who had been born to Tamar. "All the sons of Judah were five," he
concluded (v:4), so clearly he was using the word sons literally. Next he listed the "sons of
Perez," Hezron and Hamul (v:5). Since we have established that Hezron was surely the literal
son of Perez, we must conclude that the word sons was being used literally in this part of the
genealogy too.

In verse 9, the writer listed the "sons also of Hezron that were born unto him." Of the three
sons that were "born unto him [Hezron]," Ram was listed as the second one, so any
reasonable person must conclude that the genealogist thought that Ram was literally the son
of Hezron. Right before Mr. Fox's eyes, then, another possibility of a gap has vanished.

Volume 1990 - 2002 Issue


Page 575 of 2049
Skeptical Review Edited by Farrell Till
As the genealogy continued, the writer said, "And Ram begat Amminadab" (v:10). Does this
necessarily mean that Ram literally "begat" Amminadab or could he have begotten
Amminadab only in the sense that Amminadab was a descendant, possibly only a grandson or
a great-grandson? Well, the same verse says, "(A)nd Amminadab begat Nahshon," and we
have already established a literal father-son relationship between Amminadab and Nahshon.
So if "Amminadab begat Nahshon" meant that Amminadab was literally Nahshon's father,
why wouldn't "and Ram begat Amminadab," used in the same context, mean that Ram was
literally Amminadab's father. The same argument would extend to verse 11 in the genealogy
where it says, "and Nahshon begat Salma [Salmon], and Salma [Salmon] begat Boaz." Unless
begat means here what it meant in the preceding verses, a verbally inspired writer
equivocated, and that is a recognized logical error.

So where are the "gaps" in this genealogy? All recognized rules of literary interpretation lead
to only one conclusion: the biblical writers believed that only ten generations had separated
Perez and David. Mr. Fox's quibble is only a desperate grasp for some straw that he can cling
to in his frantic effort to defend a view of the Bible that has been abandoned by virtually all
serious Bible scholars. Mr. Fox even concocted a syllogism intended to prove that I was
begging the question on the matter of how many generations there were between Perez and
David, but actually I had never considered the number of generations in the genealogy a
serious point, because, as the evidence above clearly shows, the intent of Deuteronomy 23:2
was to ban descendants of bastards from the assembly forever. So whether David was 10
generations or 50 generations removed from Perez isn't relevant. He was a descendant of
Perez, and that is all that matters. Nevertheless, I have established by Mr. Fox's own inspired
word of God that biblical genealogists obviously thought that only ten generations separated
Perez and David, so now we will wait for Mr. Fox to show just where I am begging the
question.

I will respond to Fox's comments about "common errors made by atheists and agnostics" in
the next issue, as well as his other "solutions" to the problem that Deuteronomy 23:2 poses to
the inerrancy doctrine. However, one of his remarks about errors in reasoning that skeptics
make was so silly that I will answer it in this article. This occurred when he presented his
second syllogism as an example of how skeptics reason:

First Premise: If the Bible says X, then X is true.


Second Premise: The Bible does not say X.
Conclusion: Therefore, X is not true.
I wish Mr. Fox would tell us what skeptic he knows who would say that the first premise in
this syllogism is true. The foundation of biblical skepticism and higher criticism rests on the
premise that nothing is automatically true just because it is written in the Bible, so rather than
this being an"error" that skeptics make, it is an error that Bible fundamentalists make. This
very common mistake in fundamentalist logic can be stated in the following adaptation of the
syllogism:
First Premise: If the Bible says X, then X is true.
Second Premise: The Bible says X.
Conclusion: Therefore, X is true.
This change in the syllogism expresses the exact reason why people like Mr. Fox go to such
ridiculous extremes to try to find some way to explain obvious discrepancies in the Bible.

Volume 1990 - 2002 Issue


Page 576 of 2049
Skeptical Review Edited by Farrell Till
They reason from the false premise that if the Bible says X, then X must be true. This is
exactly why Mr. Fox has leaned over backwards in his search for some way-- just any way--to
reconcile Deuteronomy 23:2 with other biblical passages whose face-value meaning
obviously contradict it. He cannot accept the face-value meaning of any passage if it is going
to put a crack in the armor of Bible inerrancy that he and fundamentalists in general wear with
such illogical pride. He believes that if the Bible says X, then X is true, and that is a more
flagrant reasoning error than any that he charges to atheists and agnostics. This is a premise
that he must prove, and until he does he has no right to accuse anyone of begging the
question.

From the Mailbag


I have been very pleased with The Skeptical Review. However, I must express a small amount
of alarm over some of the logic in your article "Jairus's Daughter: Was She Dead or Wasn't
She?" On page three, you write, "However, Matthew's omission of the message brought by
some[one] from Jairus's house gives sufficient reason to reject their explanation," and "Hence
Matthew's omission of this detail is sufficient reason to reject the premise that he wanted
readers to understand that Jairus was speaking figuratively in this version of the story." I think
you are clearly overstating your case. Necessary, yes. Sufficient, no.

It is not impossible that Matthew meant for Jairus's words to be interpreted figuratively, and
for him to have omitted mention of anyone delivering news of the girl's death. This possibility
is hardly outlandish in the world of fundamentalist thought. Surely you can produce a more
convincing argument on this point or find an issue where the woolly-mindedness necessary to
maintain inerrancy is more clear-cut.

(Richard Trott, 78-A Phelps Avenue, New Brunswick, NJ 08901.)

EDITOR'S NOTE: I'm not sure I understand the distinction that Mr. Trott tries to make in
"necessary" and "sufficient"; however, I will admit that my statement was a little excessive.

Perhaps I should have said that Matthew's omission of the message that the girl had died was
sufficient reason to doubt the fundamentalist "explanation" of this discrepancy.

As for the possibility that "Matthew meant for Jairus's words to be interpreted figuratively,"
we should remember a major point that I made in the article. What a person says is what he
says, so if figurative meaning was intended in what Jairus said to Jesus, then Jairus, not
Matthew, would have been the one to determine whether figurative meaning was intended in
his statement. If Matthew was verbally inspired by an omniscient deity, his task was to
produce an inerrant account of the incident, and he could not have done that by putting
figurative meaning into a statement that the speaker had intended to be literal in its meaning.

It will do no good for inerrantists to quibble on this point and argue that for all we know
Jairus did speak figuratively to Jesus, because that would merely shift the problem to Mark's

Volume 1990 - 2002 Issue


Page 577 of 2049
Skeptical Review Edited by Farrell Till
version of the story. As the article pointed out, Mark had Jairus saying, "My little daughter
lies at the point of death" (5:23). So the problem for inerrantists is simple: Jairus either spoke
figuratively or he didn't. Their "explanation" of the apparent discrepancy in this matter in no
way resolves the problem because it just isn't possible that two writers could have inerrantly
reported what a man said, and one of those writers had the man speaking figuratively while
the other had him speaking literally.

I believe, then, that my article did expose the "woolly mindedness" of fundamentalist
thinking. This opinion has apparently been confirmed by the conduct of some preachers who
were asked to write a response to this article for simultaneous publication. At the Gulf Coast
Lectureship last May (where I also debated Jerry Moffitt), I sat patiently and listened to three
different preachers who had been assigned the task of resolving "fancied contradictions" in
Matthew, Mark, and Luke, but not a one of them even mentioned the problem in the story of
Jairus's daughter. I talked to two of them individually about this after their lectures and later
sent them advanced copies of my article with invitations to write responses. Another preacher
at the lectureship had said, probably for my benefit, that atheists and skeptics reject the Bible
only because they want to live evil lives, so I sent the article to him too. Not a one of these
preachers accepted the invitation to respond to my article. If they are so right about the Bible
being inerrant, why would they not have jumped at the opportunity to show the hundreds of
TSR subscribers that the problem discussed in my article was only a "fancied contradiction"?
Their silence in this and other matters we have discussed in TSR gives us sufficient reason to
doubt that they are as sure of their position as they claim when speaking to partisan audiences.

You are such a beautiful example of why freedom of the press is so necessary and why church
and state must stay separate. If the fundamentalists of any religion were allowed to govern.... I
shudder at the thought.

I have read only about 80% of the NKJV once. I picked it up and read it as I would any other
piece of literature, trying to put together the pieces of the puzzle to make a complete picture.

I too found far too many inconsistencies for it to form a rational, logical, factual piece of
work. I even went to many bible studies thinking there must've been something I missed, for
these other people had something totally different from what I got out of the "book." I always
ran into the same lines of question begging and definitions that were pure fantasy whenever I
critically questioned the bible. From my little bit of education in psychology, I believe there is
something too painful in a fundamen talist life to deal with objective reality. I pity them.

Keep up the brilliant essential work. Enclosed is $10 for the next year's subscription. The
extra $5 is a donation to help keep the press rolling.

(Tony Powell, 41044 179th Street East, Lancaster, CA 93535.)

I thoroughly enjoyed my copy. All of it!

Volume 1990 - 2002 Issue


Page 578 of 2049
Skeptical Review Edited by Farrell Till
I particularly got a hoot out of X's letter to Dear Brother Dobbs (p. 13). Especially amusing is
his contention that people rarely lose their faith for intellectual reasons but because they
suffered personal hurt. As another of the horde who escaped religion, I must say that being
brought up in church is abuse in it self, and personal hurt is the name of the game.

But long before I studied the histories of religion, with the ignorance, cruelty, and control
therein contained, I knew it was crap! As a child, sitting through endless sermons, I sure
wondered about this god people worship, who decreed death for thousands of offenses and
who had prepared a boiling lake of fire for nearly everyone so that they could suffer forever
and ever. It took me many years to break away. After being abused as a child by a
churchgoing father, I had no self-esteem to break away. In fact, I married a preacher and
stayed in that crummy marriage for 25 years. He, I must add, sexually abused our daughter,
and was extremely unfaithful as well, but I didn't believe in god or any of the dogma for years
before I escaped.

Mr. X's opinions, answers, and explanations are certainly run of the mill. Ho hum! Why is it
religionists never concoct anything new? They just parrot what they hear someone else say.

(Name withheld for obvious reasons.)

Please renew my subscription. I'm so pleased that you are increasing the number of issues in
'95 and '96. I get pretty depressed between issues of TSR. I live in an area of fundamentalists,
and I'm terribly close (in miles) to Pat Robertson and the Christian Coalition. Even Jerry
Falwell isn't all that far away. [At least the voters in Virginia didn't elect Ollie North.] Here in
North Carolina, we have that bundle of joy, Jesse Helms. See why I get depressed? I would
move back north, but it's too damned cold up there.

Keep up the good work, and would you please include some reading lists in TSR for those of
us who want to learn more about atheism but are afraid to ask.

I'm 67 years old and have been a nonbeliever for 50 years. Your publication has been a
lifesaver (or a mind-saver). Thank you.

(Barbara Briggs, 230 Ocean Winds Drive, Kitty Hawk, NC 27949.)

EDITOR'S NOTE: We have published a few suggested reading lists in recent issues.
Subscribers who are interested in further study of atheism/agnosticism and the issues
addressed in TSR should ask to be put on the mailing list of H. H. Waldo, Book seller, P. O.
Box 350, Rockton, IL 61072-0350. The telephone number is (800) 66-WALDO. Most of the
books cited in TSR articles are available at Waldo's.

Most local libraries now have interlibrary-loan systems, which give patrons access to books
that aren't actually in stock at the library. Just go to the library with the book title and author's
name and ask if it can be obtained on loan. The lending time, usually two or three weeks, will

Volume 1990 - 2002 Issue


Page 579 of 2049
Skeptical Review Edited by Farrell Till
give you the opportunity to decide if this is a book you would like to purchase for your
personal library.

Internet Infidels' Note: We have an on-line bookstore which includes an atheism section,
at http://www.infidels.org/infidels/products/books/.

Listening just now to the Till-Hovind Debate on the flood, I hear the audience saying,
"Amen," "Yes," "Yes," when Professor Till says their answer to all the problems in the story
is that "God could do it."

According to Hovind, miracle after miracle had to take place in preparation for this so-called
worldwide flood. So my question is, "Why didn't God use up some miracles doing good
things for people?" If they were so bad, couldn't he have helped them in some way? Why did
he use all these miracles for the purpose of drowning every man, woman, and child on earth
except for eight and to drown most of all animals, birds, and insects that he him self had
created and pronounced good???

Why did the audience consider those miracles so wonderful when they were for the purpose
of mass murder? These same Christians always say, "God is love," "God is merciful," "God
forgives," yet the Bible is filled with mass murder, slavery, infanticide, torture, rape (even
gang rape), etc., etc., etc. How do they have the nerve to be insulted when they are called
ignorant?

Hovind said that the animals and people were vegetarian then, so Noah didn't have to provide
meat for their diets, because they ate vegetables and fruits. So I suppose vegetables and fruits
fell into the window from heaven, kind of like manna, during that year in water.

Well, Professor Till, listening to you in all the debates was interesting, entertaining, and
educational, but as for your opponents, listening to them was painful! You are greatly admired
for your patience.

(Dr. Dorothy Thompson, P. O. Box 562, Bandon, OR 97411.)

A year ago I ordered a free one-year trial subscription as advertised in Freethought Today.
The trial was over and the verdict rendered before the second page was turned. Today I am
enclosing $5 for renewal, plus $4 as belated compensation for the first year. As Ayn Rand
noted, a rational man neither desires nor accepts the unearned. The value of your product is
too great to accept for free. In fact, I believe you could and should raise the rate.

Your work is outstanding, your presence reassuring, and your courage inspiring. You have my
support and admiration. Keep up the assault.

(Dutch Williams, 3325 Hunter Lodge Road, Marietta, GA 30062.)

Volume 1990 - 2002 Issue


Page 580 of 2049
Skeptical Review Edited by Farrell Till
EDITOR'S NOTE: I just can't resist rubbing the noses of those who say that The Skeptical
Review has no following in letters like this to prove that they are as wrong about this as they
are their ridiculous inerrancy belief.

It was with great joy and satisfaction that I read my first copy of The Skeptical Review. I
received my first copy after requesting it from you after seeing your advertisement in The
Humanist. I have been looking for a publication like yours for a long time.

I am 42 years old. I grew up a Southern Baptist, and my father was a Southern Baptist pastor.
He pastored in Orange and Los Angeles Counties for over 20 years. When I was in my late
20's, I began to experience doubts concerning the things I had learned about God, the Bible,
Jesus, heaven, hell, and salvation. I wrestled with my doubts for a long time, because the
tenets of orthodox Christianity were so much a part of my psychological make-up and
identity. I can now say with conviction that I do not believe in the fundamentals of
Christianity or in the inerrancy of the Bible. I am basically an agnostic. I began to look
honestly at all I believed to see if it made sense and was logical. I found much of it was not. I
see that what most people call faith is but credulity. I see most people do not want to be
challenged to think for themselves. It seems most people accept the status quo and what is
most comfortable to them rather than have the strength to think for themselves. They are
afraid of what people, especially relatives, will think of them if they expressed aloud their
doubts and misgivings of what they hear ever Sunday. Think of the pastors who often get a
flash of enlightenment on the idiocy that they teach the people but immediately shut off those
skeptical thoughts because expressing them would cost them their careers and financial
securities.

(Glenn W. Wood, 24806 Walnut, Apartment 1, Newhall, CA 91321.)

Here's my renewal check. Five beans can't cover much more than the cost of printing and
postage.

As one of your fans, I can't resist the urge to toss a few words and ideas in your direction. I
warn you; I have a problem with brevity.

I like the quotation on your banner: "It is wrong, always, and for everyone to believe anything
upon insufficient evidence" (W. K. Clifford). That reminds me.... One way to attack
nonsensical religious ideas seems all too obvious to me, yet it's one I have never heard. It's
highly intuitive and goes like this:

ONE: We can be quite sure Jack Ruby shot Oswald. A number of people were present and
saw it happen. Millions of people saw it on TV as it happened. Video tapes are available for
re view. It's a nonarguable point.

Volume 1990 - 2002 Issue


Page 581 of 2049
Skeptical Review Edited by Farrell Till
TWO:We can be only partly sure that Bruno Hauptmann kidnapped the Lindbergh baby, since
there were no eyewitnesses. There was a lot of circumstantial evidence, such as the ladder,
which pointed to his guilt. A jury convicted him, yet some say he was inno cent. It's an
uncertain matter, at best.

THREE: We really can't be confident that young George Washington chopped down a cherry
tree. There is no supporting evidence and only he and his father, so the story goes, would have
had direct knowledge of the event. It's an appealing story but really can't be relied upon. It's a
popular legend.

FOUR: The story of Jesus of Nazareth being born of a virgin has no supporting evidence
whatsoever. It arises out of hand-me down-stories. Only one person could have had direct
knowledge of the event, and the only written accounts were drafted many years after her
death. Like the cherry tree story, it has wide appeal for those predisposed to believe it, and for
that reason it has endured. It has all the earmarks of a myth. Now think of it. One of the major
religions of the world turns on an event for which there is no supporting evidence. Christians
love to beat on the Mormons and the Joe Smith tales and the Mormons' silly ideas of
advanced civilizations in the Americas many centuries ago. They sneer because there is
neither supporting evidence for Smith's claims nor for the civilizations. Yet the same folks
will swoon and rattle their beads over the event recounted as #4 above.

TSR beings me the recurring idea that extreme religiosity destroys the reasoning power of the
minds of the believers. Isn't it a good thing that Jonas Salk was born into a Jewish home that
respected learning and inquiry instead of a Christian "Science" household? Has a Christian
fundamentalist ever made an information -based contribution to the world? I can't think of
any....

(Thomas T. Wheeler, 322 Highland Street, Rochester, MI 48307.)

EDITOR'S NOTE: Mr. Wheeler is making essentially the same point about historical
information as Richard Rich did in an earlier "Mailbag" column (Winter 1995, p. 13): people
tend to accept ancient records of ordinary events, which are possible or probable, even though
they aren't necessarily true. As I noted in "Evaluating Historical Claims," pp. 9-11 (this issue),
Thomas Paine made this same point in Age of Reason: reasonable people accept ordinary
claims that were recorded by ancient historians but reject the fantastic or extraordinary. As the
article also noted, the works of early historians like Tacitus, Suetonius, Josephus, etc., all
contain accounts of miraculous events that no rational person can believe really happened
even though they are no more fabulous than many biblical stories. This should tell Bible
fundamentalists something, but of course it doesn't. Nothing can budge a confirmed
bibliolater from his irrational belief that all events written in the Bible happened exactly as
recorded.

Thanks for the Moffitt-Till debate tape loan. I mailed them back to you by "priority" this
morning.

Volume 1990 - 2002 Issue


Page 582 of 2049
Skeptical Review Edited by Farrell Till
The debate was interesting, although I was tempted to fast forward when Mr. Moffitt spoke. I
forced myself to listen (I tune out when people start quoting Bible scripture). Just think, I
wouldn't have learned that God didn't require creation because he lives in a different
dimension where time flows backward as well as forward. Please!

After viewing this debate, I had a scary thought. I wonder if the 52 GOP congresspersons that
received 100% ratings from the Christian Coalition this November election (Freedom Writer,
Institute for First Amendment Studies, Jan. 95, p. 4) would agree with Mr. Moffitt and his ilk
that the Amalekite slaughter of babies was morally justifiable if Yahweh commanded it. I
learned last night on a "Bible prophecy" show on a Christian Cable station that Ted Turner of
CNN is possibly the anti-Christ....

(John Sears III, 17 Forest Avenue, Millinocket, ME 04662.)

Although I am skeptical of biblical skeptics, I shall request a free subscription to The


Skeptical Review, as a result of "strong urging" from a foremost biblical skeptic in this neck of
the woods, Ralph Nielsen (whom I suppose you are already familiar with since I am to "feel
free to mention [his] name").

However, you are hereby forewarned that if you offend my conservative Christian
sensitivities I shall be forced to invoke an ancient Hebrew curse upon all of you: "May a
camel urinate in your drinking well." And I'm sure you wouldn't want that to happen, would
you?

I am the type of "whacko" that actually believes the events in the Bible really happened. You
know, the kind who thinks that Daniel really wrote the book of Daniel and there was only one
person named Isaiah who wrote a warning to Israel. Yes, I'm the kind of person who believes
that Jesus walked on water with out it being frozen solid. That the Red Sea parted and the
Israelites walked across on dry land. That Sodom and Gomorrah was [sic] burnt to crispy
critters for all their deeds in and out of the closet. (But you can breathe easy now because I
didn't vote for the Republicans, nor for Clinton for that matter.)

So go ahead, send me your skepticism! I dare you to make me a believer in skepticism!

(Corey Wicks, 1620 Chestnut, Apt. 25, Clarkston, WA 99403.)

EDITOR'S NOTE: In a letter to Mr. Wicks, I asked him why he believed that Daniel and
Isaiah really wrote the books that bear their names. Was it because he had carefully examined
all the evidence on both sides of the issue and decided that logic and reason were on the side
of the fundamentalist view of authorship? I asked why he believed that Jesus had walked on
water and that the Red Sea had parted for the Israelites? Was it because he had examined
evidence that indicated a high degree of probability that these exceptional events had actually
happened? In all probability the answer to both questions is no, for unless he is an unusual
Bible believer, he believes the Bible because... well, just because he believes the Bible. He
was taught to believe it as a child, and so he believes it. I would be very surprised if Mr.

Volume 1990 - 2002 Issue


Page 583 of 2049
Skeptical Review Edited by Farrell Till
Wicks had ever seriously investigated biblical claims to see if there is any kind of reasonable
evidence to support those claims.

Let's take the parting of the Red Sea as an example. What evidence is there to indicate that
such an event ever happened? None whatsoever beyond the fact that the event is recorded in
the Bible, a book that is filled with fabulous tales that Mr. Wicks would instantly reject if they
were recorded in any other book.

I refer again to "Evaluating Historical claims" (pp. 9-11), which notes that historians like
Suetonius and Tacitus recorded as facts fantastic events that no reasonable person can believe
actually happened. I am sure that Mr. Wicks does not believe that such events as those are
historical facts, even though they are no more fantastic than many biblical stories. If Wicks
had been taught from early childhood that he must believe everything that Suetonius, Tacitus,
and Josephus wrote, he would probably accept as truth every fabulous event that they
recorded.

So I really don't have much hope that I will convince Mr. Wicks that his view of the Bible is
naively credulous, but I hope I am wrong. Certainly, I will not try to make Mr. Wicks "believe
in skepticism," because skepticism is not something that one be lieves in. Skepticism is
simply a critical thinking process by which one evaluates information and rejects claims that
have no reasonable evidence to support them.

As for the ancient Hebrew curse, why should I object to a camel urinating in my drinking
water? I am a part of the humanity that Christianity has been pissing on for almost 2,000
years. Compared to that, a little camel urine in the drinking water would be nothing.

I want to thank you for being courageous enough to speak the truth and willing to take the
time to do so. Until four months ago, I was a hard-headed fundamentalist, certain that I had all
the answers. I'm sure my friends and family would have voted me least likely ever to become
an agnostic. Indeed, no one was more surprised than I when I did so. You mentioned in TSR
that computers will hasten the demise of Christianity. They certainly did so in my case. I
started writing on a secular humanist bulletin board with the intention of converting those
"poor souls" who didn't have the truth. At the least, I would be able to say to God that I'd done
my part in spreading his word. In the end, they ended up converting me!

The case against Christianity is airtight. No one who is aware of the evidence can honestly go
on believing the lie that Christianity is. Unfortunately, Christians, even intelligent, scientific,
studious Christians, aren't aware of the truth. When I did find out the truth, I went through the
stages of grieving: first denial, then intense sadness, then intense anger at having been so
deluded, and then the best stage, acceptance. Now that my eyes are open, now that I can look
at what the Bible actually teaches, I would never want to go back to blind faith. The God of
the bible is a hateful God, a lying, untrustworthy God.

Again, please keep up the good work. When you are talking to Christians, it must seem as if
you are talking to a brick wall. Some of my agnostic/ atheist friends said they felt that way

Volume 1990 - 2002 Issue


Page 584 of 2049
Skeptical Review Edited by Farrell Till
when they talked to me. But truth cannot be denied. And even brick walls can be penetrated
by truth.

Please send a copy of your newsletter to my fundamentalist friends (names and address
given).

Thank you very much.

(Rhonda Diane Jockisch, 1018 South 13th Street, Pekin, IL 61554-4956.)

EDITOR'S NOTE: This is a letter that hardly needs comment, except for my expression of
delight at Ms. Jockisch's awakening to rational thought. I suspect that fundamentalist
Christianity is going to suffer the loss of many Rhonda Jockisches in coming years. In the
Spring 1995 issue of TSR, letter-writer Herschel Davis ridiculed me for saying that the
personal computer would hasten the demise of fundamentalist Christianity (p. 12), but in Ms.
Jockisch's story we have an example of what Bible fundamentalists will have to contend with
in the future. There was a time when the truth about the Bible could have been suppressed and
kept from Ms. Jockisch, but that is no longer possible. The information age offers very little
for Bible fundamentalists to be optimistic about.

I really enjoyed reading the first copy of TSR. I also enjoyed reading the copies I found at a
WWW site maintained by Jeff Lowder, namely,
http://www.infidels.org/library/modern/magazines/tsr/. I've enclosed a check for $26.50 for
the following materials: (materials listed).

Thank you very much. Keep up the fine work!

(Rychard Bouwens. 1750 Liberty, Apt. 8, El Cerrito, CA 94530.)

EDITOR'S NOTE: Jeff Lowder's support via computer bulletins has Introduced TSR to many
new subscribers. I am publishing Mr. Bouwens' letter to illustrate the contribution that
computers can make to the advancement of rational thought. As I said above, computer
bulletins are not good news for Bible fundamentalism.

In one of the back issues of TSR you offered to provide all past issues on floppy disks. As I
recall, two high-density disks are required. I have enclosed two 1.44 MB disks for this
purpose.

Several months ago I ordered a set of all back issues (through Volume Five, Number Two:
Spring 1994). I have enclosed $2 for Numbers Three and Four of Volume Five. I have viewed
tapes of your debates with Kent Hovind, Buster Dobbs, and Jerry Moffitt. Although I disagree
with you on the propositions involved in those debates, I still found them profitable for study.
Are there any other debates you have available for loan (at a modest charge, of course)? If so,

Volume 1990 - 2002 Issue


Page 585 of 2049
Skeptical Review Edited by Farrell Till
and you do not mind shipping them with the items mentioned above, I will be glad to mail the
loan fee. If you would rather send them after the loan fee is paid, just let me know which ones
are available and how much the fee is.

Incidentally, in reading all of the past issues of TSR (except the last two which I am ordering
above), I have appreciated the sense of fairness that you seem to manifest. Not many editors
of periodicals allow the open discussion opportunities that you do. I think this is
commendable. Although I disagree strongly with most of what you write, and (to be perfectly
frank) I believe that you are headed for eternal torment in your present course, I also think
you are more open and fair in some respects than some so-called Christians. As a gospel
preacher for nearly 30 years, I would be interested in our participating in a two night public
debate of the issues related to Biblical inerrancy discussed in TSR. If you would be willing to
come to the Huntsville, Alabama, area in early August 1995 (about the earliest I could get to
such a discussion in view of my present commitments), I will be happy to secure facilities and
do extensive advertising. If we can agree upon terms for a debate, I will assure you that I will
conduct myself as a gentleman and attempt to deal fairly with your arguments.

(Thomas N. Thrasher, P. O. Box 1941, Decatur, AL 35602 1941.)

EDITOR'S NOTE: I commend Mr. Thrasher for the attitude that he conveyed in this letter. It
was a refreshing change from the venomous letters that I am accustomed to receiving from
other Church-of-Christ preachers. Mr. Thrasher, by the way, belongs to a faction of the
Church of Christ whose members are considered apostates by guardian-of-the-faith preachers
like Jerry Moffitt, Buster Dobbs, and Lindell Mitchell, yet Mr. Thrasher, who preaches
"error," is willing to defend the Bible inerrancy doctrine in public debate while most of the
"faithful" preachers aren't.

Sometimes it's hard to figure out just who the apostates are. For the moment, the only serious
suggestion I have for Mr. Thrasher is that he should reexamine the religious belief that makes
him think that I am "headed for eternal torment" when by his own admission I am "more open
and fair in some respects than some so-called Christians."

Needless to say, I welcomed Mr. Thrasher's debate proposal. I have accepted it, so people in
the Huntsville, Alabama, area should contact either him or me for details about time and
place.

The Skeptical Review is certainly the best bargain around. I look forward to each issue and
usually read most of the articles. I found the articles by Sierich and Till on the size of armies
and battles to be particularly interesting in the winter 1995 issue. It was always difficult for
me to read the passages that were discussed without feeling that they just had to be grossly
exaggerated.

It's obvious that debating many of your antagonists is pointless from the standpoint of
changing their thinking. It is difficult to imagine that anything, except possibly rejection by
denominational peers, would ever cause some of them to admit limitations in their reasoning.

Volume 1990 - 2002 Issue


Page 586 of 2049
Skeptical Review Edited by Farrell Till
But at least the chance exists that a few people less committed to absolutist viewpoints
(including those in the rationalist camp) might take a more objective look at the basis for their
convictions. If nothing else, they could conclude that people who differ from them in their
beliefs might possibly be as reasonable as they. Providing a forum for the exchange and
defense of ideas sets The Skeptical Review apart from fundamentalist publications. It's
difficult to imagine the folks at ICR, for example, letting an evolutionist present the scientific
evidence relating to one of their Acts and Facts tracts.

(Robert Norton, 1709 West Queen's Court, Peoria, IL 61614.)

The winter issue of The Skeptical Review came today. It has some interesting items, but it is a
bit tiring to read all of it.

Might I suggest that the answer to all fundamental inerrantists is a good lesson in history. The
first six books of the Old Testament were written much later than the FI people assume.
Deuteronomy was written ca. 600 B.C.E. by the priests to get King Ahaz to bring the
Hebrews back to JHWH. The lst, 2nd, 3rd, and 4th books of the OT were written later at the
time of the Babylonian captivity. Ezra and his scribes gathered a lot of folk lore and gave
fanciful authenticity to them to unite the Hebrews who were enjoying life on the banks of the
Euphrates. Even as many New York Jews give lip service to the state of Israel but prefer the
Hudson River to the Jordan, so the Hebrews needed a unifying document. Compare Joshua's
alleged version of the conquest of Canaan with that of the book of Judges. Most of the
conquest was imaginary.

All together the first seven books are a collection of folk tales, and numbers of people as well
as historical anomalies are to be expected.

Now as to the New Testament, the strongest material is found in the Pauline epistles. Saul
(Paul) never saw Jesus. The gospels had not yet been written. Whence came his authority?
The prophecies in Matthew are nonexistent. Isaiah did not forecast a virgin birth; Daniel was
written a couple of centuries after the fact. How could the writers miss in their prophecies? So
let the inerrantists stew in their own silly myths. Answer them with historical facts about the
writings.

(Eliel F. Kirkpatrick, 326 West Elting Street, Macomb, IL 61455-1132.)

EDITOR'S NOTE: I just wish it were as easy to answer Bible fundamentalists with historical
facts as Mr. Kirkpatrick seems to believe. The problem with the approach that he suggests is
that no fundamentalist will accept historical evidence when that evidence contradicts his/her
interpretation of the Bible. Let's take the creation-evolution controversy as an example.
Geological evidence that the earth is billions of years old is overwhelming, yet Bible
fundamentalists reject this evidence because it disagrees with their creationist beliefs, which
require a young earth view. Biology, microbiology, geobiology, paleontology, archaeology,
chemistry, etc., etc., etc. all support the evolution ary view, but Bible fundamentalists reject

Volume 1990 - 2002 Issue


Page 587 of 2049
Skeptical Review Edited by Farrell Till
all of the findings of any branch of science that disagrees with the biblical claim that Yahweh
created the earth and all life on it about 6,000 years ago.

So if Bible fundamentalists have this attitude toward science, we can imagine how they would
react to the "historical facts" that Mr. Kirkpatrick summarized in his letter. Bibliolaters, in
fact, believe that the only truly accurate ancient history is that which is recorded in the Bible
or that which seems to confirm the Bible version of history. They would scoff at the notion
that the book of Deuteronomy was written around 600 B. C., because they know that Moses
wrote Deuteronomy. How do they know this? Well, you see, Jesus (Matt. 19:8) and Paul
(Rom. 12:10) indicated a belief that Moses wrote Deuteronomy, so bibliolaters must insist
that he did or else admit that the two most important people in the founding of Christianity
made judgment errors about the authorship of the Old Testament, and no fundamentalist can
do that without irreparably damaging the inerrancy doctrine. In Mark 12:26, Jesus said, "Have
you not read in the book of Moses..." and proceeded to tell about Yahweh's appearance to
Mos es in the burning bush, which is recorded in Exodus 3:1-6. According to Mark, then,
Jesus thought that Exodus was "the book of Moses," so bibliolaters must believe that it was or
else admit that either Jesus or Mark made a mistake. It's easy to guess which choice they
going to make.

As for the historical facts about the authorship of the Bible that Mr. Kirkpatrick related in his
letter, they aren't really historical facts in the sense that Napoleon's defeat at Waterloo or the
assassination of Julius Caesar are historical facts. The latter are facts that can be reasonably
verified by reliable historical records; the former are only sensible conclusions that have been
arrived at by painstaking--and often controversial-- methods of critical analysis about the
authorship of documents in a time when corroborative records were scarce. In other words,
we have no 6th century B. C. records from Babylonian archives that say, "Yesterday in the
third month of the reign of King Cyrus, the Hebrew priest Ezra released a scroll that he has
written, in collaboration with a group of scribes, about the exodus of the Hebrews from Egypt
and their subsequent wanderings in the Sinai wilderness." No, the scholarly information that
tells us who wrote the books of the Bible and when they were written was obtained through
meticulous methods of higher criticism, and anyone who has had any experience at all with
Bible fundamentalists knows that they will simply wave aside the scholarly conclusions of
higher critics and make them go away with the magic word--liberalism.

None of this is intended to dispute what Mr. Kirkpatrick said in his letter, with the possible
exception of his associating the authorship of the book of Deuteronomy with the reign of
King Ahaz. Most higher critics believe that the "book of the law" discovered during repairs to
the temple in the reign of King Josiah (2 Kings 22:8) was actually the book of Deuteronomy,
which some priest had written at that time and passed off as a book written by Moses. Josiah
reigned from 640 to 609 B. C., a time that is compatible with the date that Mr. Kirkpatrick
assigned to Deuteronomy, but Ahaz reigned a hundred years earlier from 732 to 716 B. C. For
additional information on the authorship views that Mr. Kirkpatrick has summarized, readers
should consult Who Wrote the Bible? by Richard Elliott Friedman, but don't expect the
conclusions of higher criticism discussed in this book to make any kind of impression on
bibliolaters. When it comes to choosing between "the wisdom of the world" or "the word of
God," any Bible fundamentalist worth his salt will take the good book any time. This is why
we use the overkill method that you so often see in The Skeptical Review. We simply take the

Volume 1990 - 2002 Issue


Page 588 of 2049
Skeptical Review Edited by Farrell Till
same Bible that fundamentalists use in their churches, examine in detail passages that
common sense and face-value meaning of the texts clearly indicate are points of discrepancy,
and defy the inerrantists, with offers of free space, to show that they are not discrepant.
Through this approach, we expose the utter inability of fundamentalist spokesmen to defend
the inerrancy doctrine and give skeptics on our subscription list information that they can use
in their contacts with door-to-door missionaries and other bibliolaters.

What About Kobe?


Without belief in God and the Bible, there can be no morality. How many times have we
heard this from Lindell Mitchell, Bill Lockwood, and other fundamentalists whose articles
and let ters we have published? It's a common claim but one that has no basis in fact, as the
earthquake in Kobe, Japan, proved. Although the city was devastated, the Japanese
"heathens," who as a nation have no faith in either the Bible or its god Yahweh, did not
exploit an excellent opportunity to loot. Who reading this seriously believes that no looting
would have occurred if an earthquake like this struck a city in our "Christian" nation? Perhaps
some fundamentalist can explain where the Japanese got their morality.

Back Cover

Address Changes
If you change your address, you must notify us in order to receive The Skeptical Review
without interruption. The post office will not forward third-class mail, so if you move without
notifying us, your copy will be discarded, and we will have to pay fifty cents for a change-of-
address notification. When our subscription list was much shorter, we would send
replacement copies via first-class mail to those who moved without notifying us, but our list is
now too long and first class postage rates are too high to continue this service. The cost of
continuing the service would be $1.60 for each person who moves without notifying us.

If you do move and neglect to notify us, we will correct your address and continue your
subscription with the next issue. For $1, we will also replace the copy you missed.

E-Mail Addresses
TSR subscriber Edward Brunelle is compiling an e-mail directory of skeptics. If you have an
e-mail address and would like to have it listed in the directory, notify him at
eddy@prizm.com.

Volume 1990 - 2002 Issue


Page 589 of 2049
Skeptical Review Edited by Farrell Till

Another Apology
We must apologize again for the mail that we never have time to answer. We always have
good intentions, but each mail delivery puts us a little further behind. The responses from our
ad in Bible Review, which will run also in the next issue, has put us even further behind. We
hope you will understand.

If you have an e-mail address, we can more conveniently reply to your letters. At the Free
Inquiry conference in Minneapolis last summer, Molleen Matsumura told editor Farrell Till
that he should put his computer on line. She said that e-mail was faster, more convenient, and
cheaper. We found out that she was right. As noted elsewhere in this issue, our e-mail address
is jftill@midwest.net. We are behind on our e-mail too, but we get to it much quicker than
"snail" mail.

The Horner-Till Debate


On May 30th at Seattle Pacific University, Farrell Till and Michael Horner debated the
resurrection of Jesus. Horner is an official in the Campus Crusade for Christ.

Since SPU is a Methodist university, the crowd was understandably partisan, so comment
cards collected from the audience unsurprisingly indicated that Horner's position had
prevailed. With a printing deadline to meet, we didn't have time to write a review of the
debate, but we hope to publish one in the next issue of TSR.

A video tape of the debate will be available for rental at the new cost of $2. Those who want
their own tapes will have permission to copy it. Don Morgan is transcribing the debate from
audio tapes, so a transcript will also be available for $2.

Did We Miss Anyone?


Our volume of mail has been unusually large in recent months. Although we welcome the
increases in subscriptions and distributions of other printed matter that accompanied the
increase, we realize that the chances of clerical mistakes also multiplied. In double checking
our files, we discovered some mistakes that we have corrected. However, if you ordered
something that you did not receive, please let us know, and we will send the materials to you.

We also receive reports from subscribers whose copies aren't delivered. This, of course, is the
fault of postal errors. However, we will replace at no cost all subscription copies that are lost
in the mail system. If ten weeks after the delivery of this issue, you have not received the
special fifth edition that we are adding this year, please let us know. In 1996, we will be

Volume 1990 - 2002 Issue


Page 590 of 2049
Skeptical Review Edited by Farrell Till
publishing bimonthly, so the scheduled mailing dates will be the first weeks in January,
March, May, July, September, and November.

The Mailbag
Your letters tell us that "From the Mailbag" has become a popular column. If you write to us,
please let us know if you do not want your letter published. This would save us the time of
having to write for permission to publish. After we print this notice again, if you don't ask us
not to publish your letter, we will assume that you have no objections.

Volume 1990 - 2002 Issue


Page 591 of 2049
Skeptical Review Edited by Farrell Till

The Skeptical Review


Volume Six, Number Four -1995
Farrell Till, editor

• The Historicity of Jesus

• Just Another Far-Fetched How-It-Could-Have-Been

• The Fivefold Challenge

• The "Testimony" of Mara Bar-Serapion

• The 430-Year Sojourn

• "God's" Opinion of Woman

• From the Mailbag

The Historicity of Jesus


Did Robin Hood exist? Possibly, there was a person whose exploits were exaggerated over
time until the legendary character known as Robin Hood emerged in English folklore, but few
people would claim that the Robin Hood in these legends was an actual historical figure who
possessed incredible archery skills and went about rescuing Maid Marian and robbing the rich
to give to the poor. At best, then, Robin Hood was a quasi-historical person who became the
legendary hero of Sherwood Forest through exaggeration and embellishment of his real life
accomplishments.

The same is probably true of William Tell, King Arthur, and other famous legendary
characters. Through exaggeration and embellishment over time, the lives of exceptional

Volume 1990 - 2002 Issue


Page 592 of 2049
Skeptical Review Edited by Farrell Till
leaders were transformed into the legendary figures we read about in folkloric literature. In
fairly recent times, we have seen the same process at work in our own country. Wyatt Earp,
Wild Bill Hickok, Buffalo Bill, Jesse James, Billy the Kid--these were frontier marshals,
heroes, and outlaws whose names are familiar to all of us, but their exploits were so
exaggerated and embellished by word of mouth, by 19th-century dime novels, and then later
by 20th-century movies that it would be difficult, if not impossible, to determine the real
historical accomplishments of any of them. In this sense, it would be proper to say that the
Wyatt Earp and Jesse James of the dime novels and movies were not real historical characters.
Men by these names once lived, but they were not the men portrayed in the many
fictionalized accounts of their lives. The real Wyatt Earp and Jesse James have probably been
lost to us in a hopeless maze of legendary embellishments.

The same is true of Jesus of Nazareth. A few scholars seriously argue that no such person ever
existed, and their arguments are certainly thought provoking and deserving of consideration.
Other biblical scholars (many of them professing Christians) acknowledge the existence of a
man named Jesus but quite frankly admit that the New Testament gospels greatly embellished
his life and that the actual achievements of the real Jesus were nothing like those attributed to
the Jesus of the gospels. The quasi-historical Jesus may have been born to a woman named
Mary, but certainly she was not a virgin at the time.

This is the stuff that myths and legends are made of, and folklore of the times was filled with
tales of great men who had been born to virgins. Even Christians consider those folk tales to
be nothing but quaint legends, so by what rule of logic do they insist on making Jesus an
exception to the general rule? They have no reasonable answer to this question.

Likewise, the quasi-historical Jesus may have attracted a following, but it isn't reasonable to
believe that vast multitudes thronged to him in the manner claimed for the New Testament
Jesus. Mark said that "a great multitude from Galilee... and from Jerusalem, Idumea and
beyond the Jordan, and... from Tyre and Sidon" once followed him to the Sea of Galilee (3:7-
8). So huge was the multitude that Jesus told his disciples to keep a boat ready for him to
board, "lest [the multitude] crush him" (v:9). Matthew claimed that "great multitudes followed
[Jesus] from Galilee, and from Decapolis, Jerusalem, Judea, and beyond the Jordan" (4:25). In
the verse before this, Matthew said that "his fame went throughout all Syria" so that the
people there "brought to him all sick people who were afflicted with various diseases and
torments, and those who were demon-possessed, epileptics, and paralytics."

All of this presumably happened, but no one in Syria, Idumea, Tyre, or Sidon left any record
of the mass hysteria that the Jesus of the New Testament created. Only the New Testament
gospels mention the huge crowds that he attracted. As Rob Berry points out in his article The
Fivefold Challenge (p. 10, this edition), historical silence in some matters is quite telling, and
such is the case in the matter of public attention that the Jesus of the New Testament
presumably attracted. If these gospel accounts are even reasonably close to being accurate,
why did no one in the regions from which the multitudes came ever mention the crowds that
thronged around Jesus? Why did no one in the places where the crowds gathered (with the
exception of the biased gospel writers) mention these huge crowds? The answer is that such
multitudes probably never existed, because the quasi-historical Jesus wasn't nearly so popular
with his contemporaries as the gospel writers allege for their Jesus.

Volume 1990 - 2002 Issue


Page 593 of 2049
Skeptical Review Edited by Farrell Till
The gospel writers claim that Jesus made a triumphal entry into Jerusalem just before his
crucifixion and that "a very great multitude spread their clothes on the road" and "others cut
down branches from the trees and spread them on the road" (Matt. 21:7-8; Mark 11:8; Luke
19:36) and that multitudes went before and after him shouting, "Hosanna to the son of
David!" Such vast multitudes as these welcomed Jesus into the city and then just a short time
later crowds were screaming for Pilate to crucify him. Who can believe it? There may have
been a quasi-historical Jesus who was crucified during Pilate's administration, but it is
unreasonable to believe that this Jesus was welcomed into Jerusalem so enthusiastically by
huge crowds only to have mobs demanding his crucifixion just a few days later. In this sense,
we can assume that the Jesus of the gospels never existed.

If there was a quasi-historical Jesus who was crucified by the Romans, certainly his execution
did not occur as recorded in the New Testament. All three synoptic gospels claim that while
Jesus was on the cross, darkness fell "over all the land" from the sixth hour until the ninth
hour (Matt. 27:45; Mark 15:33; Luke 23:44). In all three accounts of this event, the word land
has been translated from the Greek word ge, which can mean "earth," so it is quite possible
that all three gospel writers intended to say that the three hours of darkness covered the whole
earth. In fact, the KJV even translates the word as earth in Luke's version: "(T)here was
darkness over all the earth."

Whether the synoptic writers intended to say that darkness covered the whole earth for three
hours is really immaterial, because their language is such that they obviously didn't mean that
this was only a phenomenon that was localized to the city of Jerusalem. They claimed that
darkness covered "all the land" for a period of three hours, beginning at midday, so this would
have been at least a regional event that would have been noticed and mentioned in the
contemporary records of other nations. Who can seriously imagine a three-hour period of
darkness happening in midday without references to it being recorded in Egypt, Greece, Syria,
Arabia, Persia, and the other nations that would have experienced it? Even if it were merely a
regional darkness, we can reasonably expect that other writers of the time would have referred
to it. The fact that no such records exist is reason to believe that this midday darkness was
simply another part of the legends and myths that evolved as Christianity grew.

We can say the same about Matthew's reference to the "many saints" who were resurrected
after an earthquake opened their tombs at the moment of Jesus's death and who later went into
the city and appeared unto "many" (27:52-53). Such an event as this would have attracted far
more attention than the resurrection of Jesus, because its results would have been witnessed
by far more people, but no one else besides Matthew (not even Mark or Luke) mentioned this
remarkable event. Rationality, then, requires us to interpret this story as just another legend
that developed along with Christianity. A quasi-historical Jesus may have been crucified, but
certainly his death was not accompanied by a mass resurrection. Such an event simply would
not have passed unnoticed by historians of the time.

Bible fundamentalists, of course, will contend that these are all arguments from silence, but
sometimes silence can scream to those whose minds have not been numbed by religious
indoctrination. Since Rob Berry discusses this point quite well in his article (p. 10), there is no
need to comment further on it. Suffice it to say that there are many good reasons to assume
that the Jesus of the gospels never existed.

Volume 1990 - 2002 Issue


Page 594 of 2049
Skeptical Review Edited by Farrell Till
Some will also dismiss these points as just the rantings of a cynical atheist, but the average
churchgoer doesn't realize that radical revision is taking place in modern Christian thought.
Many seminaries teach their students some of the same things that we publish in The
Skeptical Review, so it isn't at all uncommon to find Christian scholars who agree that the real
Jesus was very different from the fictionalized Jesus of the gospels. After its March meeting
in Santa Rosa, California, the Jesus Seminar, a group of Christian scholars dedicated to
identifying the real historical Jesus, announced their belief that the "story of the historical
Jesus ended with his death on the cross and the decay of his body." The group concluded that
"whatever Jesus' followers experienced after the crucifixion, it happened in their hearts and
minds, not as a matter of history." Speaking for the group, Stephen J. Patterson, an associate
professor of New Testament at Eden Theological Seminary in St. Louis, said, "`God raised
Jesus from the dead' is a statement of faith, not historic fact."

These quotations have been taken from an article from Religion News Service that was
published in various newspapers last March, so their accuracy can easily be verified. That
they represent conclusions reached by conscientious Christian scholars rather than atheists
and skeptics indicates the transition that is presently occurring in Christian thought. The
average church member who doubts the major points in this article has simply not kept up
with the latest scholarship.

Just Another Far-Fetched How-It-Could-


Have-Been
Farrell Till

Although it was five years late in arriving, I appreciate Mr. Hutchinson's attempt to explain
the discrepancy between the length of the Israelite sojourn in Egypt and the Exodus-6
chronology in an Aaronic genealogy. TSR subscribers who have all back copies would profit
from reading the Winter and Spring 1990 issues, for this was where Jerry Moffitt and I
originally debated the discrepancy that Mr. Hutchinson is now proposing a solution to. A
cover letter from Mr. Hutchinson explained that he had just read my original article on the
Internet, so his delay in writing on the subject is certainly understandable.

I appreciate Mr. Hutchinson's calling his solution to the discrepancy a "scenario," because that
it is exactly what it is, a scenario for which he can offer only assumptions for proof. I hope
that readers noticed that Mr. Hutchinson even used the word assumption several times in his
article, so I appreciate his candor in that too. I will show that his "scenario" is too far-fetched
to be credible and his assumptions too unlikely to carry any force of argument.

Volume 1990 - 2002 Issue


Page 595 of 2049
Skeptical Review Edited by Farrell Till
Readers of the original exchange between Moffitt and me may recall that Moffitt proposed a
"skipped-generations" theory to try to prove that there were two Amrams in Aaron's
genealogy and that the Amram who was Aaron's father was not the Amram who was Kohath's
son. I think anyone who reads the earlier exchange on this subject will see that Mr. Moffitt
failed to offer any credible proof that the Exodus writer skipped some generations in Aaron's
genealogy, and, most certainly, Moffitt failed to prove that there is any logical reason to
believe that the writer(s) of the Pentateuch was aware of another Amram who was the "father"
of the branch of Kohathites who were called Amramites (Num. 3:27-28). Failing to establish
the existence of this "other Amram" left Moffitt's explanation with nothing but conjecture to
support it, and, of course, mere conjecture doesn't prove anything. Now five years later, Mr.
Hutchinson offers another version of the skipped-generations theory, which, as we will soon
see, isn't any better than Moffitt's.

This latest theory of skipped-generations attempts to make Levi, Kohath, Amram, and Aaron
simply represent or symbolize the four generations whom Yahweh said, in a revelation to
Abraham, would serve as strangers "in a land that is not theirs" and be afflicted for 400 years
before coming out of servitude in the "fourth generation" (Gen. 15:13-16). A scenario such as
this must be assumed, of course, because anyone with common sense knows that more than
just four generations will live and die over a period of 400 years. As Moffitt pointed out in his
first article on the subject, a study of the genealogies in 1 Chronicles 2 through 7 will show
lists of as many as ten generations who had to have lived in Egypt ("The Inerrancy Doctrine Is
Found to Be Impregnable," Winter 1990, p. 8), so both Moffitt and Hutchinson are using a
familiar inerrantist tactic: when literal interpretation of a passage threatens the Bible inerrancy
doctrine, claim figurative meaning. Hence, we now have Mr. Hutchinson arguing that the
Hebrew word ben (son) didn't always mean son in the literal sense. It could have meant
"offspring" or "descendant." Such an "assumption," he claimed, makes it possible that Kohath
wasn't Levi's "immediate son" but only a "direct descendant of Levi," and likewise "Amram
could have been a direct descendant of Kohath rather than his immediate son, and Aaron
could have been a direct descendant of Amram." I wouldn't turn around for the difference in
Hutchinson's theory and Moffitt's. They are both based on the assumption that generations
were skipped in the Exodus 6 genealogy, an assumption for which Hutchinson offered no
proof, but against which an abundance of evidence can be found in the Bible. Moffitt did at
least try to support his claim by arguing that the 8,600 Amramites, Izharites, Hebronites, and
Uzzielites counted in the census taken in Numbers 3:27-28 could not have descended from
Amram, Izhar, Hebron, and Uzziel if one assumes that "sons of Kohath" in Exodus 6:18
means that they were literal sons of Kohath.

Moffitt's mistake in advancing this argument was that he was trying to prove inerrancy by
assuming inerrancy. He wanted the privilege to argue that what was said in the book of
Numbers couldn't possibly contradict what was said in the book of Exodus, and so, by
necessity, the genealogy in Exodus 6 had to be understood in some figurative sense. The only
possible grounds that one could have for making such an argument as this is that the Bible is
inerrant in its entirety. That, however, was the very issue we were debating, so I couldn't
allow Mr. Moffitt to beg the question. He had to prove that the census figures in Numbers
3:27-28 were accurate, and, of course, he could not do that. Therefore, he had no basis for
using this census figure to prove that generations had been skipped in Exodus 6, especially
when there are so many sound reasons for believing that the census figures in Numbers and

Volume 1990 - 2002 Issue


Page 596 of 2049
Skeptical Review Edited by Farrell Till
elsewhere in the Pentateuch were greatly inflated, but that is a subject that would require a
separate article, which I would be happy to write if Moffitt, Hutchinson, or any other
fundamentalist wishes to argue that the Israelite nation was as large as census numbers in the
Pentateuch claim.

Besides this problem, there is a widely recognized hermeneutic principle that plays all kinds
of havoc with Mr. Hutchinson's "assumption" that "son of" in Exodus 6 could have meant
"descendant" rather than "immediate son." This principle was stated by Mr. Moffitt himself in
a written debate that we have in progress. "Sound hermeneutics," Mr. Moffitt said, "teaches us
that words have their normal import unless the context inhibits the normal use." I
wholeheartedly accept that principle and appeal to it now to demand that Mr. Hutchinson
show us how the context of Exodus 6:14-25 gives any reason to "assume" that the expressions
"son of" and "sons of" should not be understood to have their "normal import." Verse 4 lists
Hanoch, Pallu, Hezron, and Carmi as "the sons of" Reuben, and Genesis 46:8-9 also identifies
them as the "sons of" Reuben. Numbers 26:5-6 also lists the same four as "the sons of"
Reuben. What is there in the context of any of these passages that gives us any reason at all to
think that these were not the literal sons of Reuben?

The Exodus 6 genealogy lists Jemuel, Jamin, Ohad, Jachin, Zohar, and Shaul as "the sons of"
Simeon (v:15) and even mentions that Shaul was "the son of a Canaanitish woman." Genesis
46:10 lists these same six as "the sons of" Simeon and also identified Shaul as the son of a
Canaanitish woman. What is there in the context of either passage that gives any reason to
suspect that the writers did not mean that these were the literal sons of Simeon?

In the interest of space, I won't analyze every listing in the Exodus 6 genealogy to show that
each time the "sons of" whoever were listed, the same names were used that were listed in
Genesis 46 and elsewhere when the genealogies of Jacob's sons and their sons were given. If
Mr. Hutchinson questions that this is true, then I urge him to investigate by doing a little
comparison of genealogies himself. The limited analysis that I have done is enough to
challenge Mr. Hutchinson to show us what there is in the contexts of any of these genealogies
to suggest that "sons of" did not mean literal sons. If he can't do that, then he finds himself in
violation of a recognized hermeneutic principle. I should remind him too that he cannot argue
that "sons of" must have been used figuratively some of the times in the Exodus 6 genealogy
or otherwise this genealogy contradicts Exodus 12:40-41, which states that the Israelites
sojourned in Egypt 430 years. To so argue is to make the same mistake that Mr. Moffitt did in
his articles on this same issue, i. e., trying to prove inerrancy by assuming inerrancy.

If Mr. Hutchinson is going to argue that "son of" in Exodus 6 could have meant that Kohath
was only a "descendant" of Levi, and Amram was only a "descendant" of Kohath, and Aaron
was only a "descendant" of Amram, then he must also argue that every time the expressions
"son of" or "sons of" was used in this genealogy, they had the same meaning, i.e., descendant
or descendants. Otherwise, he must deal with the problem of why a writer who was verbally
inspired by an omniscient, omnipotent deity would have so flagrantly equivocated in such a
short space as this genealogy occupies in the Bible. Equivocation is the act of using the same
word in different senses in the same context without informing the audience that a change of
meaning is intended, and that is recognized as a serious writing flaw. Application of this
hermeneutic principle means that if Hanoch was a literal son of Reuben, then Pallu, Hezron,

Volume 1990 - 2002 Issue


Page 597 of 2049
Skeptical Review Edited by Farrell Till
and Carmi were also literal sons of Reuben; otherwise, the writer of this verse was guilty of
equivocation. Likewise, if Hanoch was only a "descendant" and not a literal son of Reuben,
then Pallu, Hezron, and Carmi had to be just "descendants" and not literal sons of Reuben.
Otherwise, the writer of this verse was guilty of equivocation.

If we continued analyzing the genealogy, we would see nothing to suggest that the writer was
using "sons of" in the next verse, when listing the "sons of" Simeon, in any sense different
from the meaning given the word in the previous verse. Thus, if the names listed in Reuben's
genealogy were literal sons, the names listed in Simeon's genealogy were literal sons, and if
the names listed in Reuben's genealogy were just "descendants," then all the names listed in
Simeon's genealogy were "descendants." Otherwise, a verbally inspired writer equivocated.

Again, in the interest of saving space, I won't carry this analysis through the rest of the
genealogy, but I have taken it far enough to show that Mr. Hutchinson is forced to argue that
all of the names listed as "sons" in Exodus 6 were literal sons or they were all just
"descendants." To argue otherwise is to charge a divinely inspired writer with the serious flaw
of equivocation.

Those who have not read my original exchange with Jerry Moffitt on this subject may not
have understood why Mr. Hutchinson tried to show that Uzziel, who was described as "the
uncle of Aaron," in Leviticus 10:4, wasn't really Aaron's uncle but only a "relative." For the
benefit of those readers, I will quote a section of my original article to help clarify what Mr.
Hutchinson was trying to do:

So far all my evidence has been circumstantial. None of it actually proves that only one
Amram was intended in the genealogy, but now that is about to change. The genealogy says
that Kohath had four sons: Amram, Izhar, Hebron, and Uzziel (v:18). If I am right in saying
that the Amram in this verse was the same Amram identified in verse 20 as the father of
Aaron and Moses, then Izhar, Hebron, and Uzziel were the uncles of Aaron and Moses. Is
there any proof that they were? Unfortunately for the skipped-generation theory, there is. Mr.
Moffitt is no doubt familiar with the story in Leviticus 10:1-2, where Yahweh incinerated
Nadab and Abihu, the priestly sons of Aaron, for using "strange fire" in their censers. I
wouldn't even try to estimate how many you-better-toe-the-line sermons by Church-of-Christ
preachers have been based on this story. Perhaps Mr. Moffitt has preached a few of them
himself. At any rate, after the fire had "devoured them," we read this: "And Moses called
Mishael and Elzaphan, the sons of Uzziel the uncle of Aaron, and said unto them, Draw near,
carry your brethren from before the sanctuary out of the camp" (v:4). Here it plainly says that
Aaron had an uncle named Uzziel.
Was this the same Uzziel as the one in Exodus 6:18 who was "the first Amram's" brother?
Notice that Aaron's Uncle Uzziel had two sons named Mishael and Elzaphan (Lev. 10:4) and
that the Uzziel in the Exodus-6 genealogy (brother of Amram I) had three sons: Mishael,
Elzaphan, and Sithri (v:22). What will Mr. Moffitt say about this? Will he now come forth
with a skipped-generation, two-Uzziels theory? ("Holes in the Two-Amrams Theory," Winter
1990, pp. 5-6).

Mr. Hutchinson, of course, isn't arguing for any two-Amrams or two-Uzziels theory; he is
arguing that Amram was simply a symbol of a generation but not literally the "immediate"

Volume 1990 - 2002 Issue


Page 598 of 2049
Skeptical Review Edited by Farrell Till
son of Kohath and that Aaron was also just a symbol of the fourth generation but not literally
the "immediate" son of Amram. Hence, the description of Uzziel in Leviticus 10:4 as "the
uncle of Aaron" should not be understood literally but figuratively. To think of Uzziel as only
a "relative" of Aaron, Hutchinson said, "would be consistent with the proposed scenario" that
he is arguing for.

A frustrating thing about Mr. Hutchinson's approach to argumentation is his habit of making
assertions for which he offers no proof. For example, he said that the Hebrew word dod,
translated uncle in Leviticus 10:4, is "also translated as love, beloved, and well beloved
elsewhere in the Bible," but he cited no examples. He could have made my task much easier
had he cited book, chapter, and verse where dod was so translated. Before I discuss what my
own research uncovered, I must point out that it really doesn't matter how many times dod
was translated love, beloved, or well beloved; to give credibility to his argument, Mr.
Hutchinson needs to find a place where dod was translated relative, and I think if he had
known of such a place, he would have cited it.

In a sense, Mr. Hutchinson is right in saying that dod did have meanings other than uncle,
although it is questionable that dod in those other places was the same word as the dod used in
Leviticus 10:4. Simply because words are spelled alike and pronounced alike doesn't make
them the same word. We can use mean in English as an example. To say that a person is mean
is not to use the same word as in either of the following sentences: (1) I know what you mean,
and (2) the mean distance from Earth to the sun is 93 million miles. Each sentence uses a
different word, and the technical designation for such words as these is homonyms, words that
are spelled alike and pronounced alike but have different meanings.

The people who speak and read a language are able to determine from context which
homonym is being used. For example, what English speaking person hearing someone say
that the mean distance from Earth to the sun is 93 million miles would think that mean was
the word that meant "lacking qualities of kindness or goodness"? He would know that the
sentence was using the mean that "means" middle point or average. So it is with dod in
Hebrew. The context determines what was meant by dod, and this is where Mr. Hutchinson's
scenario runs into deep trouble.

A check of Strong's concordance will show that when dod was used in the sense of love,
beloved, or well beloved, it almost always referred to concepts or objects but not persons.
When the reference was to a person, it was used in the sense of an object of romantic love.
The following quotations will illustrate that this is so:

Come, let us take our fill of love (dod) until morning (Prov. 7:18).
For your love (dod) is better than wine (Song of Solomon 1:2).
We will remember your love (dod) more than wine (Song of Solomon 1:4).
How fair is your love (dod), my sister, my spouse (Song of Solomon 4:10).
(I)ndeed your time was the time of love (dod); so I spread My wing over you and covered
your nakedness (Ezek. 16:8).
My beloved (dod) is to me a cluster of henna blooms (Song of Solomon 1:14).
Like an apple tree among the trees of the woods, so is my beloved (dod) among the sons
(Song of Solomon 2:3).

Volume 1990 - 2002 Issue


Page 599 of 2049
Skeptical Review Edited by Farrell Till
A bundle of myrrh is my beloved (dod) to me (Song of Solomon 1:13).

As you can see, dod used as Mr. Hutchinson said appears almost exclusively in the Song of
Solomon, a book with strong sexual overtones, and those same overtones are also present in
the passages in Proverbs and Ezekiel. Let's compare these uses of dod to the places where it
has been translated uncle:

If a man lies with his uncle's wife (dowdah, "aunt"), he has uncovered his uncle's (dod)
nakedness (Lev. 20:20).
After he [a slave] is sold, he may be redeemed again. One of his brothers may redeem him; or
his uncle (dod) or his uncle's son may redeem him (Lev. 25:49).
Then Saul's uncle (dod) said to him and his servant, "Where did he go?" (1 Sam. 10:14).
And Saul's uncle (dod) said, "Tell me, please, what Samuel said to you" (1 Sam. 10:15).
So Saul said to his uncle (dod)... (1 Sam. 10:16).
And Mordecai had brought up Hadassah, that is, Esther, his uncle's (dod) daughter, for she
had neither father nor mother (Esther 2:7)
Now when the turn came for Esther the daughter of Abihail the uncle (dod) of Mordecai, who
had taken her as his daughter, to go in to the king, she requested nothing... (Esther 2:15).
Also Jehonathan, David's uncle (dod) was a counselor, a wise man, and a scribe (2 Chron.
27:32).

There are other passages where dod was used in reference to a specific male person, rather
than an abstract concept or object of romantic love, and each time, it has been translated uncle
in the major English translations. When these passages are compared to the places in the Song
of Solomon where dod was used to refer to the emotion of romantic love or to a person who
was the object of that emotion, the difference in the meaning of the two words is obvious.
This is all I need to say about Mr. Hutchinson's quibble that Uzziel was just a "relative" of
Aaron and not his uncle. Obviously, the Leviticus writer meant that Uzziel was the brother of
Aaron's father, and Uzziel is listed in the Exodus 6 genealogy as a brother of Amram (v:18).
So if Amram and Uzziel were brothers and if Uzziel was Aaron's uncle, then Amram was
Aaron's literal father, not just an ancestor, and Mr. Hutchinson's "scenario" vanishes into thin
air.

We have Mr. Hutchinson's own testimony to the inadequacy of his "scenario," because he
himself said, "If we find one scripture that cannot be reconciled with this context, we will
have to reject this particular scenario and look for another explanation." I see no way for Mr.
Hutchinson to reconcile his theory that Levi, Kohath, Amram, and Aaron were merely
symbols or representatives of generations with the fact that Leviticus 10:4 was clearly
intended to mean that Uzziel was Aaron's uncle, so Mr. Hutchinson must reject his "scenario"
and look for another explanation.

I wonder, however, why Mr. Hutchinson feels the need to "look for another explanation." I'm
afraid he betrays a bias so typical of Christian fundamentalists. They interpret the Bible on the
assumption that it is inerrant in everything it says, so if a "scenario" can't be reconciled, they
"look for another explanation." When a "scenario" that they devise cannot be reconciled with
another scripture, why don't they consider the possibility that reconciliation isn't possible for
the simple reason that discrepancies and inconsistencies may actually exist in the biblical

Volume 1990 - 2002 Issue


Page 600 of 2049
Skeptical Review Edited by Farrell Till
text? They never want to accept that "scenario"; they just look for other "explanations" that
will give at least a semblance of respectability to their irrational inerrancy belief.

Just as Hutchinson's scenario required him to argue that the word uncle was not to be
understood literally where Uzziel was described as "the uncle of Aaron," so it must also
assign a far-fetched figurative meaning to Exodus 6:20, which says that Amram's wife
Jochebed "bore" him Aaron on Moses. Without citing any biblical references to support his
claim, Hutchinson simply said that "(i)n Hebrew thought, an individual giving birth to a child
becomes a parent to all who are descended from that child." Hence, he tells us, "Jochebed can
be said to have borne Aaron and Moses even though she may have actually given birth to
their great-grandfather." Oh, I see, and then, of course, when two verses later, we are told that
Aaron married Elisheba, who "bore him Nadab, Abihu, Eleazar, and Ithamar," these four
were not really the "immediate sons" of Aaron; they were perhaps his great-grandsons. Then
when verse 25 says that Eleazar, "Aaron's son," married one of the daughters of Putiel, who
"bore him Phinehas," we are to understand that Eleazar wasn't really Aaron's "immediate son"
and that Phinehas wasn't Eleazar's "immediate son." All of this is what Hutchinson must
believe or else take us right back to the problem of a verbally inspired writer flagrantly
equivocating within the space of just a few sentences.

Hutchinson wants to claim that "Hebrew thought" could give a broad general meaning to
yalad (bear), but, as I pointed out, he cited no examples to support his case. This required me
to do his research for him, and what I found wasn't at all friendly to his "scenario." The word
yalad appears dozens of times in the Hebrew text, but I could find no examples that would
give even a scintilla of credibility to Hutchinson's theory. The listings I found were invariably
like the following:

Now Adam knew Eve his wife, and she conceived and bore (yalad) Cain (Gen. 4:1).
Then she bore (yalad) again, this time his brother Abel (Gen. 4:2).
And Cain knew his wife, and she conceived and bore (yalad) Enoch (Gen. 4:17).
And Adah [Lamech's wife] bore (yalad) Jabal (Gen. 4:20).
So Hagar bore (yalad) Abram a son (Gen. 16:15).

And so it goes, on and on, case after case, where yalad was used to signify the obvious act of
a woman giving birth to her own literal child. I won't say that there are absolutely no cases in
the Hebrew text where yalad was used in a broader sense, but for Hutchinson's "scenario" to
carry any weight, he would have to show that the word was so used often enough to give
credibility to the claim that yalad in Exodus 6 had a meaning broader than women giving
birth to their own sons. If Mr. Hutchinson can cite enough examples to give his claim
credibility, we would like to see them.

In the absence of such evidence, the Pentateuch gives us every reason to believe that the
Exodus genealogy was using yalad (bear) literally in reference to the sons that were borne by
Jochebed, Elisheba, and Eleazar's wife. All through the Pentateuch, Nadab, Abihu, Eleazar,
and Ithamar were called the "sons of Aaron" (Ex. 28:1; Num. 3:2, 32; 4:16; 16:37; 20:25.)
There are many other references, too numerous to list, in which one or more of these were
referred to as the son or sons of Aaron, and Phinehas was called the son of Eleazar. Numbers
26:60 even says that Nadab, Abihu, Eleazar, and Ithamar were "born" to Aaron. What

Volume 1990 - 2002 Issue


Page 601 of 2049
Skeptical Review Edited by Farrell Till
possible reason can anyone find to suspect that "sons of" and "born to" in these many
references were not intended to have their "normal import"? So if literal meaning was
intended when the Exodus 6 genealogy said that Elisheba and Eleazar's wife "bore" sons, why
should we assume that the same word, in the same context, was being used in a broader sense
when it said that Jochebed "bore" Aaron and Moses to Amram?

Even a passage that Hutchinson referred to in laying out his "scenario," if considered in its
entirety, establishes beyond reasonable doubt that the writer(s) of the Pentateuch understood
that Nadab, Abihu, Eleazar, and Ithamar were the literal sons of Aaron. In Leviticus 10:1-2,
Yahweh devoured Nadab and Abihu with fire because of an act of disobedience. After their
bodies were carried out of the camp by the sons of Uzziel, the uncle of Aaron, Moses warned
Aaron and "his sons," Eleazar and Ithamar, not to mourn the deaths by uncovering their heads
or tearing their clothes or they would die (v:6). The reason why the Leviticus writer had
Moses directing such a warning to Aaron and Eleazar and Ithamar seems rather obvious to
anyone who isn't looking for how-it-could-have-been ways to shore up an unlikely theory.
Aaron was the father and Eleazar and Ithamar were the brothers of the two men whom
Yahweh had devoured by fire, so there was a stronger likelihood that they would express
disapproval, through mourning, of Yahweh's harsh punishment than would others in the
congregation who were more distantly related to Nadab and Abihu. Thus, Moses warned them
not to express sorrow or grief over what Yahweh had done.

If this isn't enough to convince Mr. Hutchinson that his "scenario" is completely without
merit, he should consider another fact. Exodus 6 is not the only place where the genealogy of
Aaron was recorded, and whenever it was recorded, Levi, Kohath, and Amram were listed as
the three generations who had preceded Aaron (Num. 3:17-32; 1 Chron. 6:1-2, 16-18). In
addition to these complete genealogies from Levi through Aaron, there are some partial ones,
which all indicate that Kohath was Levi's "immediate son" or Amram was Kohath's
"immediate son." There simply is no reason to think that the biblical genealogists thought that
there were any "skipped generations" in the Levi-Kohath-Amram-Aaron lineage. What Mr.
Hutchinson is doing, then, is proposing a "scenario" for which there is no proof and
challenging me to prove that it is not true.

Mr. Hutchinson prepared a chart in support of his "scenario," which is just as flawed as his
general theory is. For reader convenience in following my analysis of the chart, I will
reproduce it.

Years Israelites Lived in Egypt

Levi 77 years
Kohath 133 years
Amram 137 years
Aaron 83 years

Total 430 years

The chart proposes that Levi spent as many as 77 years of his life in Egypt, and so the
generation represented by Levi accounted for 77 of the 430 years of Egyptian bondage;
however, the time that Levi spent in Egypt is immaterial to the "scenario" Mr. Hutchinson
wishes to present, because Kohath was born before the Israelite descent into Egypt (Gen.

Volume 1990 - 2002 Issue


Page 602 of 2049
Skeptical Review Edited by Farrell Till
46:11). By what distorted logic does Mr. Hutchinson think that he can assign 77 of the 430
years to the generation represented by Levi and then 133 years to the generation represented
by Kohath, when Kohath had to have lived in Egypt all through the 77 years that Levi spent
there? Hutchinson is trying to count 77 years twice, and with that kind of juggling of numbers
one could prove just about any kind of cockamamie chronological theory. Furthermore, at
least some of Kohath's 133 years were spent in Canaan before the Israelite descent into Egypt,
and this spells more trouble for Mr. Hutchinson's scenario. Even if Kohath had been an infant
in his mother's arms at the time of the Israelite descent into Egypt, he would not have lived his
entire life there. Please notice in the passage from The Testament of the Twelve Patriarchs
quoted below that by Jewish tradition, Levi was 40 and Kohath about five years old when the
Israelites went into Egypt. If there is any validity to this tradition, then Mr. Hutchinson's
"scenario" falls 82 years (77 + 5) short of the 430 it needs.

Speaking of cockamamie hypotheses, I want to be as kind to Mr. Hutchinson as possible, but


his "scenario" is about as cockamamie as any I have seen on this particular issue. It requires
such far-fetched hypotheses as "uncle" not really meaning uncle, "son of" not really meaning
an "immediate" son, and the verb bear (yalad) meaning remote rather than immediate
parentage."While many people dogmatically assert that Moses' parents were Amram and
Jochebed," he argued, "such a conclusion is basically speculative. The Bible leaves the door
open for a different conclusion." I think I have presented enough evidence to prove that
biblical writers obviously thought that Amram and Jochebed were the literal parents of Moses
and Aaron, but to settle the issue, I am going to suggested that we allow Jewish scholars to
interpret their own literature.

To do this, let's first notice that Exodus 6:20 clearly says that Jochebed "bore" Aaron and
Moses to Amram. The same statement is repeated in Numbers 26:59.

The name of Amram's wife was Jochebed the daughter of Levi, who was born to Levi in
Egypt; and to Amram she bore Aaron and Moses and their sister Miriam.

This passage not only repeats the claim that Jochebed bore Aaron and Moses to Amram, but it
gives an additional bit of information that poses serious problems for any attempt to reconcile
the Exodus 6 genealogy with the claim of a 430-year Israelite sojourn in Egypt. Levi's
daughter, whom Amram married, was born after Levi had gone into Egypt, so even if Levi's
final act on earth was the siring of Jochebed, it wouldn't have been possible for her to have
borne Aaron and Moses just 83 and 80 years before the end of a 430-year sojourn. If she had
been born even a few months after Levi's 77-year stint in Egypt ended in death, she would
have been around 270 years old when she gave birth to Aaron. This figure is easily arrived at
by subtracting 77 (the number of years the Israelites spent in Egypt before Jochebed's birth)
and 83 (the number of years from Aaron's birth to the exodus) from 430 years (the time that
the Israelites allegedly spent in Egyptian bondage). So perhaps Mr. Hutchinson would be
interested in debating the problem of the 270-year-old mother that the Exodus-6 genealogy
poses.

Hutchinson, of course, will argue that Jochebed wasn't the literal daughter of Levi, so this is
why I suggest that we allow Jewish scholars (rather than Christian fundamentalists with a pet

Volume 1990 - 2002 Issue


Page 603 of 2049
Skeptical Review Edited by Farrell Till
theory to defend) to interpret their own literature. The following quotation is from Levi's
section of The Testament of the Twelve Patriarchs:

XI. When I married I was twenty-eight years old, and my wife's name was Melcha. [2] And
she conceived and bore a son, and she called him Gershon because we were living as
foreigners in our land (for Gershon means "living as a foreigner"). [3] And I saw in a vision
about him that he would not be in the front rank. [4] And Kohath was born in my thirty-fifth
year, towards sunrise. [5] And I saw in a vision that he was standing raised above the rest of
the congregation round about him [6] (that is why I called him Kohath, which is "beginning of
greatness" and "reconciliation"). [7] And as a third son she bore me Merari in my fortieth
year; and it was a difficult birth (that is why his mother called him Merari, which is "my
bitterness"--and his life too was in danger). [8] And Jochebed was born in my sixty-fourth
year, in Egypt; for by then I was much esteemed among my brothers.
XII. And Gershon married, and his wife bore him Libni and Shimei. [2] And Kohath's sons
were Amram, Izhar, Hebron, and Uzziel. [3] And Merari's sons, Mahli and Mushi. [4] And
in my ninety-fourth year Amram married my daughter Jochebed, for they were born the
same day, he and my daughter. [5] I was eight years old when I went into the land of Canaan
and eighteen when I killed Shechem, and at nineteen I became a priest; and I was twenty-eight
when I married and forty when I came into Egypt. [6] And you, my children, are a third
generation. Joseph died in my hundred and eighteenth year.

The Testament of the Twelve Patriarchs is, of course, a pseudepigraphal work, a fact I'm sure
Mr. Hutchinson would not want us to overlook. However, this work dates from the 2nd
century B. C. and is considered in scholastic circles to be the work of Hellenic Jews, who
wanted to pass along Jewish traditions concerning the final remarks or "testaments" of the
sons of Jacob. The collection seems to reflect an effort to relate the last words of the
patriarchs to the prophetic remarks that Jacob made about each of his sons before his death in
Genesis 49.

Since I do not accept the premise of biblical inerrancy, I certainly won't try to argue that The
Testament of the Twelve Patriarchs is in any sense inerrant. However, it is reasonable to
argue that this pseudepigraphal work does represent what 2nd century B. C. Jews thought
about their traditions. With that in mind, let's notice that the author of Levi's testament
obviously thought that Levi was Kohath's literal father, that Kohath was Amram's literal
father, and that Jochebed was Levi's literal daughter, who had been born to him after the
Israelite descent into Egypt. So Mr. Hutchinson's far-fetched scenario finds no support at all
in Jewish tradition.

One final testimony from Jewish scholarship should put this matter to rest. Josephus, a Jewish
historian whom most Bible fundamentalist rely on whenever they think they have found
something in his works to support their case, clearly thought that Amram and Jochebed were
the literal parents of Moses. Hutchinson argued that no specific names were given to the "man
of the house of Levi" who "took to wife a daughter of Levi," and so "(t)his leaves open the
possibility that the unnamed man and woman in Exodus 2 [presented as the parents of Moses]
were not Amram and Jochebed, which must be the case for the above scenario to work." We
want to remember the statement I have emphasized in italics, because we are going to see that
the Jewish historian Josephus offered no room at all for Mr. Hutchinson's scenario "to work."

Volume 1990 - 2002 Issue


Page 604 of 2049
Skeptical Review Edited by Farrell Till
In The Antiquities of the Jews (2:9.4-7), Josephus told an expanded version of the story of the
infant Moses who was put adrift in an ark of bulrushes in order to save the child from
pharaoh's command to kill all male Hebrew children. In his version of the story, Josephus
identified the parents of Moses as Amram and Jochebed and even identified the child's sister
as Miriam. After relating how that pharaoh's daughter selected Moses as the name for the
child, Josephus said, "For Abraham was his ancestor of the seventh generation. For Moses
was the son of Amram, who was the son of Caath [Kohath], whose father, Levi, was the son
of Jacob, who was the son of Isaac, who was the son of Abraham" (2:9.6). Was Josephus
using the words son and father literally in this genealogy? Well, he clearly said that Abraham
was Moses' "ancestor of the seventh generation," and all we have to do is count the names in
the genealogy to see that only seven were listed from Moses back to Abraham.

Josephus was the son of Matthias, a Jewish priest who lived in Jerusalem during the time that
Jesus was allegedly tried and crucified. Coming from such a background as this, Josephus
would be expected to be well educated in Jewish thought and literature, so his opinion is
certainly worthy of our serious consideration. The passage I quoted above leaves no doubt
that he thought that Moses was the "immediate" son of Amram and Jochebed.

Philo Judaeus, another famous Jewish historian, didn't mention the names of Moses' parents in
telling the story of the great Hebrew emancipator. However, he did say that Moses was "the
seventh generation in succession from the original settler in the country who was the founder
of the whole race of the Jews" (The Works of Philo, Hendrickson: Peabody, MA, 1993, p.
459). The "founder of the whole race of the Jews," of course, was Abraham, so again I will
point out that if we count down seven generations from Abraham in succession, we have the
following genealogy: Abraham, Isaac, Jacob, Levi, Kohath, Amram, and Moses. Obviously,
then, this famous Jewish historian thought that Amram was the "immediate son" of Kohath
and that Moses was the "immediate son" of Amram. Since Moses and Aaron were brothers,
Philo would have thought that Aaron was the "immediate son" of Amram.

There is simply no evidence to support Mr. Hutchinson's scenario or any of the other far-
fetched, skipped-generation attempts to reconcile Exodus 12:40 with the Aaronic genealogy
in Exodus 6. The writer obviously intended readers to understand that the genealogy was
complete from Levi through Phinehas, Aaron's grandson, and the genealogy simply doesn't
leave enough room for a 430-year Israelite sojourn in Egypt. I sincerely hope that, rather than
looking for "another explanation," Mr. Hutchinson will give serious consideration to the
distinct probability that there is at least one discrepancy in the Bible.

The Fivefold Challenge


Introduction

Fundamentalist Christians claim that the Bible is a historically accurate work in every detail.
They delight in showing how "modern archaeology" has verified this little biblical detail or

Volume 1990 - 2002 Issue


Page 605 of 2049
Skeptical Review Edited by Farrell Till
that minor biblical event. But something they don't talk about much is the failure of modern
archaeology to confirm some major events in the Bible.

Specifically, there are five major miraculous events in the Bible which are completely
unconfirmed by modern archaeology. These miracles are:

1. The parting of the sea by Moses (Exodus 14:21-31)


2. The stopping of the sun by Joshua (Joshua 10:12-14)
3. The reversal of the sun's course by Isaiah (Isaiah 38:7-8)
4. The feeding of thousands of people by Jesus using only five loaves of bread and two
fishes (Mark 6:34-44; see also the parallel accounts in Matthew 14:14-21, Luke 9:12-
17, and John 6:1-14)
5. The resurrection of the saints, and their subsequent appearance to many (Matthew
27:52-53)

The Argument From Silence

When skeptics point out that some event in the Bible is unconfirmed by non-biblical records,
fundamentalists usually respond by claiming that this is an argument from silence, and that
just because nobody else confirms it doesn't mean it didn't happen. Sometimes, this is a
legitimate response-- the argument from silence is not always valid. If the event is an ordinary
event which attracted little attention, or a private event not witnessed by others, than the
argument from silence cannot be used to show the event never happened.

But in the case of the above five miracles, the argument from silence is perfectly valid. All
five of these miracles were allegedly witnessed by thousands of people-- indeed, two of these
miracles would have been visible worldwide. Hence, fundamentalists cannot claim that the
events were simply not noticed by others. Furthermore, all five of these events were of an
extraordinary nature. They are the most impressive miracles in the Bible, more impressive
than even the resurrection of Jesus. It would be absurd to claim that other people could have
witnessed a change in the sun's course, or the resurrection of a large number of long dead
people, without having been amazed by it. Such events would have attracted widespread
attention and generated dozens of documents concerning them. Take the resurrection of the
saints, for instance. Other first-century Christians would have used this event as further proof
of Jesus' divinity-- Paul and the other gospels would certainly have mentioned it, for instance.
Or how about the sun turning backwards? This would have been visible worldwide, and thus
other cultures active at the time would have noticed the event and offered their own
explanations, in keeping with their own cultural and religious beliefs. And so forth. Hence,
the argument from silence is valid in the case of these miracles. If no other evidence can be
found to support them, we are justified in concluding that they never happened, and thus that
the Bible is wrong in at least five points.

The Challenge

With this in mind, I present The Fivefold Challenge. The challenge is this: Pick any one of the
five miracles listed above, and provide one piece of documentary evidence that confirms this
miracle. The evidence must conform to the following requirements:

Volume 1990 - 2002 Issue


Page 606 of 2049
Skeptical Review Edited by Farrell Till
1. The evidence must be contemporary. It should have been written shortly after the
miracle in question.
2. The evidence must be independent. The author must have obtained his information
about this miracle from a source other than the biblical book(s) in which the miracle is
described.
3. The evidence must be unambiguous. The document should clearly describe the
miracle in question. I will not accept claims which argue from double entendre or
hidden meanings, unless you can prove that a person living in the time and place in
which the document was written would have clearly understood the hidden reference.
4. The evidence must be reliable. The evidence must have been written by a person
known to be reliable. If the document's author is unknown, the rest of the document
must be shown to be reliable. Other miraculous claims within the document will
disqualify the document unless these additional miracle claims can themselves be
independently verified.

Reward (Sort Of)


I wish I could offer a monetary reward for anybody meeting the challenge, a la Ralph Nielsen
or the Skeptical Review. Alas, I just don't have $1,000 to toss around-- not that I'm worried
about losing it, but I believe it's wrong to offer a reward you can't pay, even if you know
you'll never need to pay it. I'm also not interested in getting sued over the matter, which has
happened to some skeptics who offer rewards. So instead, I'll offer something more valuable--
a chance at saving my eternal soul. If somebody manages to meet the challenge successfully, I
will either read three books of that person's choice, or attend a church of the denomination of
that person's choice for three months. To a fundamentalist, that kind of reward is probably
more valuable than money anyway.

Conclusion

Those wishing to take me up on The Fivefold Challenge may contact me at the following
address:

Robby Berry
2496 Hard Road
Dublin, OH 43016
E-Mail: berry@nebulis.net

Unless you specifically ask me not to, I reserve the right to publish your answers, with your
name intact. So to those fundamentalists who truly believe that archaeology confirms the
Bible (and who would like a shot at impressing, and maybe even converting, an atheist) here's
your chance to prove it. I wish you luck-- you'll need it.

The "Testimony" of Mara Bar-Serapion

Volume 1990 - 2002 Issue


Page 607 of 2049
Skeptical Review Edited by Farrell Till
The absence of extrabiblical evidence of the historicity of Jesus of Nazareth has long been an
embarrassment to Christian apologists. In their defense of Christianity, early church fathers
like Justin Martyr, Origen, Tertullian, and Clement of Alexandria cited from both Jewish and
pagan sources evidences that they thought were supportive of Christianity, but they were
unable to produce any secular testimony of the actual existence of Jesus. The frustrating truth
was (and is) that secular records contemporary to the time that Jesus allegedly lived were
completely silent about this man who presumably was so famous that great multitudes
followed him during his personal ministry ( Matt. 4:24-25, 8:1, 18, 13:2, 15:30, 19:2, 21:9;
Luke 5:15, Luke 14:25). Inasmuch as word of Jesus's works spread at least as far as Syria
(according to the New Testament record) and attracted people from there to be cured of
"divers diseases and torments" ( Matt. 4:24), one would think that historians of the time would
have heard about the man and recorded at least some of this part of his life. But it didn't
happen. Contemporary secular records don't even mention the man Jesus. It is as if historians
would examine all records of the 4th century B. C. and find no references to Alexander the
Great.

In our modern age of enlightenment, the fact that no indisputable testimony to the life of Jesus
can be found outside of the New Testament and apocryphal documents (all of which were
penned by obviously biased writers) continues to trouble Christian apologists, possibly more
so than at any time in church history. In their frustration, they have resorted to some rather
imaginative efforts to find Jesus of Nazareth in ancient secular records. One such effort has
focused on a letter that may have been written toward the end of the first century.

The writer, Mara Bar-Serapion, was in prison at the time, and he wrote the letter to exhort his
son to seek wisdom. The following quotation from the letter is sometimes cited as evidence
that Jesus of Nazareth was mentioned in contemporary secular documents:

What advantage did the Athenians gain from putting Socrates to death? Famine and plague
came upon them as a judgment for their crime. What advantage did the men of Samos gain
from burning Pythagoras? In a moment their land was covered with sand. What advantage did
the Jews gain from executing their wise king? It was just after that that their kingdom was
abolished. God justly avenged these three wise men: the Athenians died of hunger; the
Samians were overwhelmed by the sea; the Jews, ruined and driven from their land, live in
complete dispersion. But Socrates did not die for good; he lived on in the teaching of Plato.
Pythagoras did not die for good; he lived on in the statue of Hera. Nor did the wise king die
for good; he lived on in the teaching which he had given (quoted by F. F. Bruce, The New
Testament Documents: Are They Reliable? Eerdmans Publishing Co., Fifth Revised Edition,
p. 114).
Bruce, an often-quoted Christian apologist, noted that this letter was "written some time later
than A. D. 73, but how much later we cannot be sure" (Ibid.). He, of course, wants to see this
letter as proof of the historicity of Jesus, but by his own admission, the document was written
at least 40 years after the time that Jesus allegedly lived and possibly even later. Since Bar-
Serapion made no claim in his letter that he had personally witnessed the execution of the
"wise king" or had ever even seen him, his statement cannot in any sense be considered
firsthand testimony of the historicity of Jesus, as Bruce and other apologists would like us to
believe that it is.

Volume 1990 - 2002 Issue


Page 608 of 2049
Skeptical Review Edited by Farrell Till
We can't even be sure that Bar-Serapion was referring to Jesus. He didn't identify the "wise
king" by name, as he did in the case of both Socrates and Pythagoras, so one merely
speculates when he says that this is a first-century secular reference to Jesus. How does one
make that determination? Messianic pretenders in Judea were a dime a dozen during the era of
foreign domination. Josephus referred to some of them, and even the New Testament
mentioned two of them in Gamaliel's speech to the Jewish council ( Acts 5:35-36). In Bandits,
Prophets, and Messiahs: Popular Movements at the Time of Jesus (Harper & Row, 1985),
authors Richard Horsley and John Hanson tell of several Messianic prophets of this period
besides Theudas and Judas of Galilee, whom Gamaliel mentioned in his speech. Some of
these Messiahs were even named Jesus, and most of them came to ignominious ends at the
hands ofeither the Romans or their own countrymen. How, then, do Bruce and other
apologists who cite Mara Bar-Serapion's reference to a "wise king" who was executed by the
Jews know for a fact that this was an allusion to Jesus of Nazareth and not to some other
Messianic prophet of those times?

One could just as easily assume that Mara Bar-Serapion was referring to the Essene "Teacher
of Righteousness," who was often mentioned in the Dead Sea scrolls found at Qumran. The
person in these documents called the teacher of righteousness was presented as a Messianic
figure who suffered vicariously for the people. Since Essene teachings were widely circulated
before and after the time Jesus allegedly lived, one could argue that this teacher "lived on in
the teaching which he had given." The point is that Mara Bar-Serapion simply did not identify
the "wise king" whom the Jews had "executed," and in the absence of that information, one
can only guess who this was supposed to be. F. F. Bruce and his inerrantist cohorts can't just
arbitrarily make a vague statement like this into a reference to Jesus. They need proof that
Bar-Serapion definitely meant Jesus, and they simply don't have it.

We should notice too the historical time frame of the characters that Bar-Serapion specifically
identified by name. Socrates lived in the 5th century B. C. and Pythagoras in the 6th. One
may legitimately wonder, then, if Bar-Serapion's "wise king" was someone who had lived
about the same time as Socrates and Pythagoras rather than someone who had been
"executed" centuries after their time. Bar-Serapion did say that "just after" the execution of
their "wise king," the Jews had their kingdom abolished and were "ruined and driven from
their land" to live "in complete dispersion," so he could very well have been referring to
events close to the time of Pythagoras and Socrates when the Jews' kingdom was abolished by
Nebuchadnezzar, after which the Jews were taken into Babylonian captivity and later
dispersed in various countries, or he could even have been referring to a king in the northern
kingdom of Israel, which was abolished by the Assyrian conquest of 722-721 B.C., after
which the northern tribes were dispersed to Mesopotamia and Media, where they eventually
lost their national identity. At any rate, if Bar-Serapion was using the word king literally, he
had to be referring to someone who lived before the Babylonian conquest of Judea, because
there simply were no "Jewish kings" after Zedekiah, whom Nebuchadnezzar installed as a
puppet until he finally sacked Jerusalem in 586 B. C. and "abolished" the Jewish kingdom. So
if the Jews had literally executed a "wise king," it had to have been someone who lived before
that date.

There is just as much biblical evidence to support this view as there is for the apologetic claim
that Bar-Serapion was referring to Jesus. The Bible itself recorded the political assassinations

Volume 1990 - 2002 Issue


Page 609 of 2049
Skeptical Review Edited by Farrell Till
of Jewish royalty that occurred close enough to Nebuchadnezzar's capture of Jerusalem to
consider the conquest of either Israel or Judea as an event that had happened "just after" the
murder of one of these kings. Josiah's father, King Amon, for example, was assassinated less
than 50 years before Nebuchadnezzar's siege of Jerusalem ( 2 Kings 21:23). Admittedly, the
Bible didn't speak very flatteringly of Amon or of most of the other assassinated Jewish kings
either for that matter, but in the absence of corroborating records, who can really know how
objective the writers of biblical history were? At times, they seemed very political in their
reporting. Who can know either what records and traditions Bar-Serapion may have known
about that would have given him a different perspective of Jewish history than the
ethnocentric views of the biblical writers? Furthermore, if Christian apologists are going to
quibble that some biblical details do not fit the theory that Bar-Serapion's "wise king" was one
of the kings of Israel or Judea assassinated before Jerusalem's fall to Nebuchadnezzar, they
should admit that some biblical details don't fit their Jesus theory either. Where, for example,
does the New Testament say that the Jews executed Jesus? It says that he was executed by the
Romans. The New Testament also teaches that Judea was under foreign domination during
the life of Jesus, so the Jews' kingdom wasn't abolished after Jesus was executed. It had been
abolished long before then.

For the sake of argument, let's just assume that Bar-Serapion's letter was beyond all
reasonable doubt referring to Jesus of Nazareth. Still it could not be considered reliable
evidence that such a person had actually lived, because the letter contains factual errors.
According to Bar-Serapion, the "men of Samos" had "burn[ed]" Pythagoras, an implication
that he had been killed by his countrymen. In reality, Pythagoras left the island of Samos in
530 B. C. and emigrated to the Greek colony of Croton in Southern Italy. He later died in
Metapontum, which is now Metaponto, Italy (Encyclopedia Americana, 1994, Vol. 23, p. 45).
The men of Samos did not "burn" Pythagoras, so if Mara Bar-Serapion's letter was incorrect
in such an important detail as this, how can it be considered reliable proof of the historicity of
anything, much less the existence of a person who wasn't even mentioned by name in the
document?

We might also ask what Mara Bar-Serapion meant in saying that Pythagoras "lived on in the
statue of Hera." What statue of Hera? Hera was the sister and wife of Zeus, so there were
undoubtedly many statues of her in the Greek culture of the 6th century B. C. What exactly
did Bar-Serapion mean? When was the island of Samos covered with sand "in a moment,"
and when did the famine and plague come upon the Athenians after their crime against
Socrates? History knows of no events as calamitous as these that Bar-Serapion referred to. To
say the least, there is a lot of vagueness and uncertainty in this document.

Christian apologists will have to keep looking, because they have certainly found no proof of
the historicity of Jesus of Nazareth in Mara Bar-Serapion's letter.

EDITOR'S NOTE: Sharon Williams with whom I have debated the historical Jesus and
other biblical subjects through several letters published in The Pekin (Illinois) Daily Times,
was sent an advanced copy of this article and invited to respond to it. She declined on the
grounds that she did not want to be mistreated in TSR.

Volume 1990 - 2002 Issue


Page 610 of 2049
Skeptical Review Edited by Farrell Till

The 430-Year Sojourn of Israel in Egypt


Roger Hutchinson
Among the examples cited in an effort to disprove the doctrine of Biblical inerrancy is an
apparent contradiction between Exodus 12:40-41 and Exodus 6:16-20 :
Now the sojourning of the children of Israel, who dwelt in Egypt, was four hundred and thirty
years. And it came to pass at the end of the four hundred and thirty years, even the selfsame
day it came to pass, that all the hosts of the LORD went out from the land of Egypt.
(Ex.12:40-41).

And these are the names of the sons of Levi according to their genera tions; Gershon, and
Kohath, and Merari: and the years of the life of Levi were an hundred thirty and seven
years.... And the sons of Kohath; Amram, and Izhar, and Hebron, and Uzziel: and the years of
the life of Kohath were an hundred thirty and three years.... And Amram took him Jochebed
his fa ther's sister to wife; and she bare him Aaron and Moses: and the years of the life of
Amram were an hundred and thirty and seven years.
(Ex. 6:16-20).

Exodus 12 states clearly that Israel dwelt in Egypt 430 years. Exodus 6 provides a genealogy
that does not appear, at first glance, to cover 430 years. For there to be no contradiction
between these scriptures, there must be at least one explanation that will harmonize the two
scriptures. This article proposes one such explanation.

Four key people are identified in Exodus 6. They are Levi, Kohath, Amram, and Aaron. We
know that Levi was among the group that originally entered Egypt.

Now these are the names of the children of Israel, which came into Egypt; every man and his
household came with Jacob. Reuben, Simeon, Levi...
(Ex. 1:1-2)
Exodus 6 then tells us that Levi died at the age of 137, which would have been some time
after he entered Egypt. We are also told that Aaron was among the group of Israelites that left
Egypt with Moses and that Aaron was 83 years old when the Israelites left Egypt.
And Moses was fourscore years old, and Aaron fourscore and three years old, when they
spake unto Pharaoh.
(Ex. 7:7)
Between Levi and Aaron, Exodus 6 tells us that Kohath lived 133 years and Amram lived 137
years.

The Hebrew word ben translated as son in Exodus 6 can also be translated as offspring or
descendant. So, in Exodus 6, we can take "son of" to mean either that Kohath was Levi's
immediate son or that he was a direct descendant of Levi. Likewise, Amram could have been
a direct descendant of Kohath rather than his immediate son, and Aaron could have been a
direct descendant of Amram.

Volume 1990 - 2002 Issue


Page 611 of 2049
Skeptical Review Edited by Farrell Till
Because Levi died in Egypt at 137 years, we are able to speculate with some confidence that
he was less than 137 when he entered Egypt. We can assume that Levi was 60 when he
entered Egypt and that he lived in Egypt 77 years before his death at 137. The assumption that
Levi was 60 seems consistent with other information we find in the Bible. More important,
this assumption allows us to develop the following scenario in an effort to harmonize Exodus
6 and Exodus 12.

Years Israelites Lived in Egypt

Levi 77 years
Kohath 133 years
Amram 137 years
Aaron 83 years
Total 430 years
In this scenario, we have one possible explanation that describes how Exodus 6 and Exodus
12 fit together. Whether we will be able to explain fully the use of genealogies and ages as
portrayed in this scenario is not critical at this point. The critical need is to determine whether
this scenario is consistent with all other information we find in the Bible.

At this point, it is important to understand that translating from one language to another is not
an exact science. Translating a language like Hebrew is particularly difficult, because there
are few words in the lan- guage and Hebrew words can have many nuances. It is not unusual,
then, for translators to consider the context in which the word is found and to choose that
meaning which is consistent with that context. Therefore, we will allow the above scenario to
dictate the context in which we are to translate and interpret other scriptures. If we find one
scripture that cannot be reconciled with this context, we will have to reject this particular
scenario and look for another explanation.

We see how context affects our understanding of the Bible when we look at Genesis 15:13-
16:

And he [the LORD] said unto Abram, Know of a surety that thy seed shall be a stranger in a
land that is not theirs, and shall serve them; and they shall afflict them four hundred years....
But in the fourth generation they shall come hither again: for the iniquity of the Amorites is
not yet full.
(Genesis 15:13-16)
Based on our hypothesized scenario, Levi would represent the first generation, Kohath the
second, and Amram the third. Finally, Aaron would represent the fourth in which Israel would
leave the land that was not theirs. Thus, under the above scenario, we draw the conclusion
that generations can be measured by the lives of certain individuals. A generation would
begin with the birth of some unique individual and end with his death. A new generation
would then be identified with a new unique individual who would be born immediately after
the death of the prior generation figure. With this view of a generation, we see how it is
possible for God to say that Israel would leave Egypt in the fourth generation.

Since our scenario covers 430 years, it is consistent with the census recorded in Numbers 3. If
the original population of 70 individuals that entered Egypt doubled every 20-25 years, then it
would easily exceed one million after 430 years.

Volume 1990 - 2002 Issue


Page 612 of 2049
Skeptical Review Edited by Farrell Till
There are several scriptures whose interpretation is directly influenced by the context
established by this scenario. Leviticus 10:4 states:

And Moses called Mishael and Elzaphan, the sons of Uzziel the uncle of Aaron, and said unto
them, Come near, carry your brethren from before the sanctuary out of the camp.
(Leviticus 10:4)
The Hebrew word dod translated as uncle in this verse is also translated as love, beloved, and
well beloved elsewhere in the Bible. The word appears broad enough in scope to refer to any
relative, not just an uncle. Thus, Leviticus 10:4 could legitimately be translated as, "Uzziel,
the relative of Aaron," which would be consistent with the proposed scenario. Limiting the
range of dod by requiring that it be translated uncle and nothing else is not a supportable
position even though that translation is found in several versions of the Bible. The translation
of dod as uncle reflects the conclusion of the translator that the proper context was that Aaron
was the immediate son of Amram. Had the translator believed that Aaron was a descendant of
Amram, he would not have translated dod as uncle. He would have chosen the proper English
word that reflected the meaning of dod within the different context. Thus, Leviticus 10:4 is
not inconsistent with the above scenario.

Exodus 6:20 is a little more difficult, but the same basic argument applies:

And Amram took him Jochebed his father's sister to wife; and she bare him Aaron and Moses:
and the years of the life of Amram were an hundred and thirty and seven years.
(Exodus 6:20)
In Hebrew thought, an individual giving birth to a child becomes a parent to all who are
descended from that child. The Hebrew word yalad, translated as bear, can also denote
paternity, so that either the wife or the husband can be said to bear a child. The word
encompasses many ideas, and its translation relies on the context in which it is used. Jochebed
can be said to have borne Aaron and Moses even though she may have actually given birth to
their great-grandfather. Because of this, it is not possible to assert dogmatically that Jochebed
was the physical mother of Moses and Aaron just because yalad is used. Again, yalad has a
breadth of meaning that would allow it to be translated in a manner that supports the context
of the proposed scenario.

Exodus 2 describes the birth of Moses and can be confusing. Here we read, "And there went a
man of the house of Levi, and took to wife a daughter of Levi" ( Ex. 2:1). Many people
assume that the unidentified man and woman in Exodus 2 were Amram and Jochebed. At the
same time, we see that these people are not specifically identified. This leaves open the
possibility that the unnamed man and woman in Exodus 2 were not Amram and Jochebed,
which must be the case for the above scenario to work. While many people dogmatically
assert that Moses' parents were Amram and Jochebed, such a conclusion is basically
speculative. The Bible leaves the door open for a different conclusion.

Clearly, the scriptures dealing with the genealogy between Levi and Aaron are confusing.
However, because of the nuances in meaning of critical Hebrew words, we are able to propose
an explanation to harmonize Exodus 6 and Exodus 12. The validity of this explanation
requires that we make certain assumptions. The assumptions that we made fall within the
realm of acceptable possibilities. This makes the proposed explanation valid. It does not mean

Volume 1990 - 2002 Issue


Page 613 of 2049
Skeptical Review Edited by Farrell Till
that there are no problems with the explanation. However, it does show that there is at least
one way to reconcile Exodus 6 and Exodus 12.

(Roger Hutchinson, 11904 Lafayette Drive, Silver Spring, MD 20902.) For a response to this
article, see "Another Far-Fetched How-It-Could-Have-Been" (4far95).

"God's" Opinion of Women


When I was an active Christian minister and missionary, I noticed that women were generally
more zealous church workers than men. As I learned more and more about the Bible, I began
to wonder why. In my younger days, I had often heard preachers cite the elevated status of
women as one of the wonderful achievements of the Judeo-Christian religions, but this was
not what I was seeing in my own private Bible studies. I was finding instead a shockingly
disdainful attitude toward women in a book that had been presumably written by divinely
inspired men.

Time would fail me if I tried to cite every biblical example of contemptuous attitudes toward
women, so I will have to limit myself to just a few. King David's affair with Bathsheba while
her husband Uriah was away on military duty produced an embarrassing pregnancy. David
first tried to conceal his indiscretion by bringing Uriah home on furlough apparently so that
he would sleep with Bathsheba and later think that the child was his. When Uriah's loyalty to
his unit proved so strong that he refused to indulge in the pleasures of a conjugal visit, David
sent him back to the front with a letter ordering the commander of his unit to put Uriah in "the
forefront of the hottest battle" and then withdraw so that he would be killed. The order was
executed, and when word of Uriah's death reached David, he took Bathsheba and added her to
his harem ( 2 Sam. 11).

If there is such a thing as contemptuous conduct, then David's actions in this matter certainly
qualify. One would think that if this deed called for divine wrath, David would have been the
rightful target of it. But the Bible tells us that God chose to punish David only by inflicting
pain and death on the members of his family, beginning with his wives. Nathan the prophet,
sent to reprimand David for his sin, delivered this message from God: "I will take thy wives
before thine eyes, and give them unto thy neighbor, and he will lie with thy wives in the sight
of this sun" 2 Sam. 12:11). One could imagine that this would not have been a pleasant thing
for David to witness and would have in that sense constituted "punishment," but we must look
past that to the fact of what God was threatening to do. David had sinned grievously, but God
was going to punish him by having some unnamed "neighbor" rape his wives "before all
Israel and before the sun" ( v12). According to the story, David repented and so his wives
were spared the indignity of public rape, but that is beside the point. The fact that David's god
would even threaten such a thing raises serious doubts about the Bible's claim to be the
verbally inspired word of an omnibeneficent deity. Certainly, this story in no way reflects the
"elevated status" that preachers say the Bible has brought to women. If it does, I have to admit
that I can't see it.

Volume 1990 - 2002 Issue


Page 614 of 2049
Skeptical Review Edited by Farrell Till
Women today often complain that our society has relegated them to second-class citizenship.
I agree and would even say that perhaps third-class citizenship would more accurately
describe the condition of women in modern America. Why then do they, as a group, have
such high respect for the Bible? Modern news coverage has made us aware of the shocking
treatment accorded women in the Mideastern Islamic societies that now occupy what was
once the ancient biblical lands, but this attitude toward women was frequently reflected in the
Bible and is merely a social custom that has survived biblical times. Women in western
societies fare somewhat better, but they do so despite the influence of the Bible on our
culture, not because of it, for if our laws were based on strict adherence to biblical precepts to
the degree that Moslem nations base their laws on Koranic precepts, women in our society
would fare no better than their Islamic sisters.

Just consider these examples of biblical attitudes toward women. A woman was considered
impure for seven days during her menstrual period, and anything she touched was also
considered unclean ( Lev. 15:19-24). After a woman had given birth, she was considered
impure for 40 days if her child was male, and 80 days if her child was female ( Lev. 12:1-5).
The fact that her period of uncleanness was twice as long for giving birth to a female hardly
suggests an "elevated" opinion of women in the biblical society of "God's chosen people." If a
woman was taken captive in battle, any Israelite male who found her appealing was entitled to
take her to be his "wife" ( Dt. 21:10-14), but nothing was said in this passage about securing
the woman's consent. If a jealous husband suspected his wife of adultery, he had the right to
make her submit to a trial by ordeal to prove her innocence. The ceremony was too elaborate
to summarize here, but part of it required her to drink a concoction called the "water of
bitterness" that was derived by mixing holy water with dirt swept up from the tabernacle
(temple) floor. If her body did not swell and her thighs rot after the water of bitterness had
entered the woman's bowels, she was declared innocent. In this event, however, the accusing
husband was declared "free from iniquity." This trial by ordeal is described in Numbers 5:11-
31 and should be read by all who believe that the Bible has accorded women an elevated
status. One wonders how many wrongly accused wives among "God's chosen people"
suffered great pain and even death from having had to submit to this trial by ordeal. I must
also note before leaving this point that the Bible required no comparable trial for husbands
suspected of adultery.

The attitude toward women reflected in this trial by ordeal pervaded Israelite culture. The
prophet Zechariah, looking for a symbol of wickedness in one of his prophecies, selected a
woman to be that symbol ( Zech. 5:5-8). In the book of Ecclesiastes, the celebrated wisdom of
Solomon expressed this exalted opinion of women: "This is what I have found, reasoning
things out one by one, after searching long without success: I have found one man in a
thousand worthy to be called upright, but I have not found one woman among them all" (
7:27-28). We could hardly expect a better opinion than this from a man who had no more
respect for women than to take 1,000 of them to be his wives and concubines ( 1 Kings 11:3),
but one would expect an omniscient, omnibeneficent God to have this man indicate a slightly
better attitude toward women at least while he was being divinely inspired to record God's
eternal truth. Since he didn't, are we to assume that God's eternal truth is that women are to be
considered evil?

Volume 1990 - 2002 Issue


Page 615 of 2049
Skeptical Review Edited by Farrell Till
The New Testament era brought no improvement in "divine" opinion of women. Needing a
symbol of apostate evil, the Holy Spirit presumably inspired the writer of Revelation to make
that symbol a "whore" ( 17:1-5). The Apostle Paul, perhaps the most chauvinistic of all New
Testament writers, ordered women to "keep silence in the churches" and to "ask their own
husbands at home" if they wanted to know anything, "for it is shameful for a woman to speak
in the church" ( 1 Cor. 14:34-35). He said to the young preacher Timothy, "I permit not a
woman to teach... but to be in quietness" ( 1 Tim. 2:12). He ordered wives to be in strict
subjection to their husbands ( Eph. 5:22 ; Col. 3:18 ; Titus 2:5), as did also the Apostle Peter (
1 Pet. 3:1). One must assume that this injunction would apply even to a woman of high
intelligence who may have made the mistake of marrying a fool. After all, who are we to
dispute God's eternal truth?

A friend and associate of mine at the college from which I recently retired once told me that
she had quit attending church. When I asked why, she said that she had been upset by some
things the minister of her church had said in a recent sermon. After she had explained what
the minister had said about the woman's place in the family and church, I obtained a Bible and
showed her that the man had said nothing except what the Bible teaches. Her reaction was
first surprise and then a frank assertion that she didn't care what the Bible said; she still didn't
like it. I have to wonder how many women there are like her who attend church faithfully and
work zealously on its behalf without ever realizing that their faith is based on a book that is
very uncomplimentary to women.

From the Mailbag


The receipt of your two debates and the essay on Biblical prophecies was timely in that I had
over a week off from work between the holidays. I had more time than usual to read them. I
benefited greatly in reading an articulate and logical person counter the claims of inerrancy so
well as you did. I became annoyed and grieved by the sarcastic attitude of Bill Jackson.
Surely this person was an embarrassment to the Churches of Christ denomination. His lack of
intelligence and logic was readily apparent by his appalling writing skills. I analyzed his style
of communicating and could not help coming to the conclusion that logical thinking is all but
foreign to the fundamentalists. A minister of the gospel proclaims and heralds his message,
especially as in a sermon. They are not comfortable with dialog or being challenged to defend
what they say. They proclaim the gospel and people accept or reject. In the homiletic style,
they have very little need to defend. So when they enter into a debate, they continue to assert
and herald as if that is enough to convince people. I'm glad you did not let him off the hook or
stoop to his level of sarcasm and vilifying.

Dr. Laws was a more sensible debater, and it is a real shame he did not continue the debate. I
would have loved to read your affirmatives in addition to the rebuttals that the publication
includes. I agree with you that you tore him up in your rebuttals. There is no comparison in
the logic, scholasticism, or communication skills between you and Jackson and Laws.

Volume 1990 - 2002 Issue


Page 616 of 2049
Skeptical Review Edited by Farrell Till
I appreciate your explanations of how to debate. By that I mean how to formulate an
argument into verifiable premises that lead to a valid conclusion. You repeatedly say it is not
enough to assert; one must have a truthful, verifiable, and sound basis upon which to assert....

By the way, the subjects of the early stages of the Yahweh religion from which Judaism
evolved and the history of the conflict between the religions of the Canaanites and the
Yahwists greatly interest me. Similarly, the history of monotheism also interests me. Can you
steer me to some good reading? I have no doubt that you have thought considerably on these
subjects.

(Glenn W. Wood, 24806 Walnut, Apt. 1, Newhall, CA 91321.)

EDITOR'S NOTE: Like Mr. Wood, I have often thought that the apologetic performances of
men like Bill Jackson, Jim Laws, and Buster Dobbs were surely embarrassing to the general
membership of their church, but apparently they aren't. When Bill Jackson died, he was
eulogized as one of the best debaters in the Church of Christ. I think the reason for this is in
something Mr. Wood said in his letter: "(L)ogical thinking is all but foreign to the
fundamentalists." If their preachers can't think logically, we can hardly expect the general
members to. Books like Mythology's Last Gods: Yahweh and Jesus by William Harwood and
Who Wrote the Bible? by Elliott Richard Friedman would be useful references to consult on
the evolution of Yahwism.

What a deal! All the TSRs in ASCII. Thank you so much for the offer. Enclosed are 2 HD
formatted floppies.

We would like your permission to upload them to various BBSs (computer bulletin boards) in
the area. We are sure you would not mind, but we just wanted to check first. But if you do
have a problem with that, we will respect your wishes.

We have thoroughly enjoyed the two papers we have received so far. We sit down and read
them to each other, then discuss them. It is amazing how we were kept so ignorant when we
were biblical inerrantists.

William Sierichs, Jr., is funnier than hell. We were rolling. Hope to see his articles again.

(Mike & Annie Marion, 4516 132nd Pl. NE, Marysville, WA 98271-7802.)

EDITOR'S NOTE: Any article published in The Skeptical Review can be copied or
reproduced electronically for educational purposes. We ask only that the names and addresses
of the authors and the source of publication be included so that readers of the materials will
know where to inquire for additional information.

Volume 1990 - 2002 Issue


Page 617 of 2049
Skeptical Review Edited by Farrell Till
Back in December, my older son did me the favor of installing American Online in my
computer, and I enjoyed romping through the information exchanges in the religion, ethics
and philosophy, etc. sections in which correspondences were exchanged. Late in the month
my hard drive crashed, so I have been out of contact since then. I do not recall seeing
anything attributable to "The Mighty McBull" as Matthew C. Bullard implies his nom-de-
keyboard to be. I would be happy to meet his challenge when I get restored to AOL, if he
would let us know in what section he posts his messages.

Your answer to his question, "(W)hat's the story with all the 'sics'?" was a bit indirect and may
have gone over his head. I would have said, "A sic given in parentheses after a particular
word or phrase in a quote is the publisher's way of declaring that this is the way it was written
by the author and is not a typographical error and is not the responsibility of the publisher to
correct. It pertains to mispellings [sic, "misspellings"], misuse of punctuation's [sic,
"punctuations"], wrong worlds [sic, "words"], poor uses to grammar [sic, "of" not "to"], etc."

It is strange that a "good Christian," such as he, would select as his nom-de-monitor the
epithet of the supreme god of the Canaanites, the most high El, the Mighty Bull. This, of
course, only demonstrates the cultural ignorance of so many Christians who believe that all
they need to know is contained in "The Good Book"! One name prominent in AOL is that of
Molleen Matsumura, whose nom-de-pixtels is MolleenM." She is network project director for
the National Center for Science Education and an editor of Free Inquiry. She was a speaker at
the recent Humcon held aboard the Queen Mary in Long Beach, California, last fall. I plan to
accumulate noms-de-bytes so that they will be available to AOL people who want to know
who is "on."

(Kenneth H. Bonnell, Khbonnell@aol.com, P. O. Box 65706, Los Angeles, CA 90041.

EDITOR'S NOTE: This spring we streamlined our office with a new computer system and
went on line too. jftill@midwest.net is our e-mail address.

I thoroughly enjoyed reading the winter, 1995, edition of TSR. I am new to the swirling Till-
Mitchell debates, but I think I get the gist of what's going on. Mitchell brings new meaning to
the phrase, ad hominem argument." His childish, sarcastic name-calling reminded me of the
notes we used to write to each other in the 7th grade when we were having fights. I admired
your refusal to get down to his level and sling meaningless mud. Your only slip, it seems, was
in referring to Mitchell's "thick skull," but considering the almost unbelievable provocation, it
is certainly understandable.

Matthew Bullard's letter was a stunning hodgepodge of meandering sarcasm, a grammatical


nightmare, and an excellent example of the type of graffiti you might find on the walls of the
boys' restroom in junior high school. Adolescent boys do get such a kick out of "grossing out"
girls with stories about feces and hair balls and so on. But I cannot understand how such
topics could possibly end up in a letter to TSR about biblical inerrancy. Nor can I fathom the
state of Bullard's mind while he was writing such uninspiring prose.

Volume 1990 - 2002 Issue


Page 618 of 2049
Skeptical Review Edited by Farrell Till
First of all, don't these men understand that such writing is boring? Secondly, if one is going
to try to get cute with one's quill pen, one should first learn to spell and punctuate. Never have
I seen so many run-on sentences in print in one place. And finally, one undermines one's own
case when one rants and raves. My grandfather, a minister, used to joke about one of his
sermon notes: Argument weak, holler like hell.

More important, I found the Mitchell/Bullard letters quite cheering. If that's all that's out there
to defend the Bible's supposed inerrancy, then the Bible's end cannot be long in coming.
Illiterate claims about inerrancy are oxymora to begin with. "The Bible is, duh, gooder. Yeah,
gooder." Who can take such drivel seriously? Mitchell claimed, "Questions do not establish
anything." They do if they can't be answered. I would like to see something in writing
defending biblical inerrancy that is (a) written in acceptable English, (b) devoid of useless
name-calling, and (c) focused solely on the issues at hand. Am I asking too much?

(Judith L. Hayes, P. O. Box 77, Valley Springs, CA 95252-0077.)

EDITOR'S NOTE: Having recently retired after 32 years of teaching English and college
writing, I can't express how much I agree with Ms. Hayes' comments about Bullard's letter.

However, he is downright literate compared to some fundamentalists who have written to me.
Unfortunately, some freethinkers who write to us also have problems with English grammar,
spelling, and punctuation, but overall they fare much better than most fundamentalist letter-
writers. Ms. Hayes probably can't imagine what I see in my mail. I get letters from people
who obviously don't know the first thing about basic English grammar, yet they want me to
believe that they know all about the Bible, who wrote it, when, why, how, etc. My favorite
letter-writer is the kind who wouldn't know a noun from a verb but wants me to believe that
he can explain what controversial biblical statements meant in the Hebrew and Greek
originals. I recently finished a written debate with such a man. Sometimes I had to read
sections of his manuscripts several times in order to understand even what he was trying to
say. Why did I bother with him? I had already made the commitment before I realized how
woefully unequipped the man was to participate in such an undertaking.

Before anyone writes to tell me that the truth of a claim is not dependent upon the claimant's
grammatical skills, I will readily acknowledge that that is true. However, I have carefully
examined the "evidence" that the presumably best qualified spokesmen for inerrancy have
presented in support of their case, and have repeatedly demonstrated in The Skeptical Review
just how pathetically weak and absurdly illogical it is. So when a Matthew Bullard rehashes
that same evidence on the linguistic level of an elementary school student, that just makes it
look all the more ridiculous.

Needless to say, I also agree with Ms. Hayes' belief that "the Bible's end cannot be long in
coming" if the likes of Mitchell and Bullard are "all that's out there to defend the Bible's
supposed inerrancy." I have had quite a bit of experience debating fundamentalists, so I feel
qualified to say that really the best in the inerrancy camp aren't a whole lot better than
Mitchell and Bullard.

Volume 1990 - 2002 Issue


Page 619 of 2049
Skeptical Review Edited by Farrell Till
Like Ms. Hayes, I too would like to see someone on the inerrancy side defend this doctrine in
"acceptable English" without resorting to "useless name-calling" and deviations from "the
issues at hand." How about it Lindell Mitchell, Jerry Moffitt, Buster Dobbs, Marion Fox, Mac
Deaver, Wayne Jackson, Norman Geisler, Josh McDowell, Gleason Archer, Gary Habermas,
and Mr. X in Fairfax, Virginia? Any of you interested?

Yet another argument that could be used in refuting the fundamentalist justification of the
Bible god's orders to kill infants (that all who die before reaching the "age of accountability"
get to go to heaven) is that such actions are unfair to those who die after reaching that "age."

Unlike the rest of us, those babies whom the Bible god wanted killed got to go to heaven for
eternity without having to make the conscious decision to accept the right religious creeds.
They had no trial period that could've jeopardized their eternal fate. Since all of us were
babies before, and since babies don't have much control over what happens to them, it is an
absolute travesty of justice that some have gotten to go to heaven simply because they were
lucky enough to die during their infancy. It is simply a matter of chance that those of us who
didn't die during infancy didn't.

Hope we can set up another debate with you. Keep up the good work.

(Sanjay Lal, 4900 Hearthstone Drive, Columbus, GA 31909-4112.)

EDITOR'S NOTE: Sanjay makes a point similar to what I have argued with fundamentalists
who try to justify the Yahwistic massacres of children and babies in the Old Testament on the
grounds that they went to heaven instead of growing up to become wicked like their parents. I
suggest to those who make this asinine argument that instead of being so eager to send out
preachers and missionaries to save the lost, churches should organize hit squads to go into
maternity and pediatric wards and kill all of the innocent babies and children so that they will
go to heaven instead of growing up to become wicked like their parents. They would probably
send far more souls to heaven than they have been doing through evangelization methods. I
might also add that rather than bombing abortion clinics and assassinating those who work
there, fundamentalists should set up their own clinics and encourage women to use them. Just
think how many more souls they would be able to send to heaven. As for the eternal destiny
of those who participated in such baby massacres, we would think that a just, benevolent god
would surely reward them for their good deeds.

You might wish to inform your readers that the Deaver-Till Debate is available in print (the
oral debate was transcribed and is available in three ring binders). The price is $14.00 plus
$2.00 for postage. This can be ordered from Biblical Notes, 7401 Glenhaven Path, Austin, TX
78737.

(Mac Deaver, address same as above.)

Volume 1990 - 2002 Issue


Page 620 of 2049
Skeptical Review Edited by Farrell Till
EDITOR'S NOTE: This debate was conducted at Southwest Texas State University in March
1991. The subject was Bible morality, and Mr. Deaver defended the massacre of the
Amalekite children on the grounds that Yahweh did them a favor by having them killed while
they were still young enough to be taken to "glory."

In earlier issues of TSR, availability of video tapes of this debate was advertised, but I stopped
promoting it because of the poor quality of video production. The sound was lost several
times, and it was necessary for one entire speech of Mr. Deaver to be re-created. I found the
accuracy of the re-creation to be doubtful in places. I was asked to supply audio reproductions
of some sections of one of my speeches, but I couldn't honestly verify that the "dubs" I sent to
the producer were actually what I had said. For this reason, I stopped promoting the tapes.

At my first debate in Portland, Texas, I learned that the Deaver-Till Debate had been
transcribed by Vicki Hightower, the wife of a Church-of-Christ preacher in San Antonio.
Having worked at transcribing part of my debate with Norman Geisler, I appreciate the time
and effort that Ms. Hightower put into this work, but I have checked sections of her
transcriptions while listening to the tapes and have found several mistakes. None of the
mistakes substantially altered the content of the debate, so anyone reading it will get a fairly
accurate account of the video version of the debate.

I have tried to find time to publish a more accurate transcript of this debate to sell at a much
lower cost, but the remarkable growth of The Skeptical Review has kept me too busy. If any
subscribers have scanning equipment and would be willing to scan the Biblical Notes version
into ASCII files, I could more easily compare the files to the tapes, correct mistakes, and then
publish a more accurate version. If anyone is interested in helping me on this project, please
let me know.

Before this debate, Mac Deaver wrote to tell me that he would expect me to reciprocate by
agreeing to debate the same issue in Central Illinois. I immediately assured him that I would
do so, but after the debate was over, I found that he didn't have much enthusiasm for a return
engagement. The subject of Bible morality is an important one that both Christians and
skeptics seem to be interested in, so I would urge Mr. Deaver to give serious consideration to
the second debate that we previously agreed on. After sending him an advanced copy of this
column, he responded with an indication that he is now ready to consider doing the second
debate. Discussions are now in progress, so perhaps another debate on Bible morality will
occur after all. Meanwhile, if any subscribers would like to view tapes of the first debate, The
Skeptical Review will make them available at our regular two-week rental rate of $3.

Are you still offering the first year's subscription free? If so, would you please send The
Skeptical Review to me? I think I started being a skeptic as a young child, when I thought,
"But how could they get all of those animals on the ark, anyway?" Thank you for your work. I
just read something about TSR and got your address from the Worldwide Web on the Internet.

(Susan Brown Cocherl, 706 A West 30th, Austin, TX 78705.)

Volume 1990 - 2002 Issue


Page 621 of 2049
Skeptical Review Edited by Farrell Till

Thanks very much for your complimentary subscription for one year. It is a bit of a culture
shock in this day and age to be given something valuable for free by a small business
enterprise, and it is much appreciated.

An important bit of suppressed information that helps show the close relationship between
Christianity and Hinduism such as attempted by Stephen Van Eck in his article "Hare Jesus:
Christianity's Hindu Heritage," Summer 94, is that the words Christ, Christian, etc. are merely
alternative spellings of the word Krishna. So modern Christians are inadvertently singing the
praises of the Hindu god Krishna and not the poor deluded Jew they think they are idolising,
who wasn't known by that label. Jesus (which also incidentally wasn't his real name either but
was a modified version of the names of a number of ancient pagan gods such as Zeus,
Dionysus, Adonis, etc.) was only named after the Hindu god as a vain attempt to make him
seem more important and to acknowledge the true origins of some of his deceptive teachings.

Thanks again for what you are doing.

(John Glue, C/O P. O., Cooktown Q4871, Australia.)

Sometime last summer I sent you a check for all of the back issues of The Skeptical Review. I
received them and have read them fairly thoroughly, highlighting the many parts of articles I
want to recall for future reference. I am in no way a Bible scholar, so I do enjoy these articles;
however, I really became excited when you decided to take on other issues besides Biblical
inerrancy, such as "God Was with Him," "The Truth Will Make You Free," and "No Morality
without the Bible?" (Several years ago, in a nearby town, a tree limb in a park crashed on two
toddlers. One lived; one died. The mother of the surviving child said, "God was with him." I
was horrified for the parents of the dead child. Assuming they are believers, how could they
not be crushed by such an insensitive comment?) Thank you for the things you write about.
I'm beginning to see, through your publication and others I have found, that I am not alone in
my horror at religion's impact!

The last issue I received was Winter 1995. Because I am not certain if I have received all
issues of the first-year free subscription and because I do not want to miss Spring 1995, I am
enclosing a check for $5 and leave you to sort out that matter accordingly. I certainly do
believe your publication is a bargain and assume you do not therefore have the resources to
notify subscribers when it is time for renewal.

I was wondering if you might eventually delve into the subject of religion and the bible and
its impact upon women. It was one of the factors in my realizing I'd always been an atheist
but by no means the only one. That would have to be when I realized how preposterous the
whole god concept was! Except for Sandra Till's article, which touched upon the "woman"
question, I haven't seen as much as I might like, especially considering how dismissive of
women, how sexist, the bible writers were. It seems to me religion would be mortally
wounded if women would stop supporting it. Why they do, I do not know.

Volume 1990 - 2002 Issue


Page 622 of 2049
Skeptical Review Edited by Farrell Till
In closing, I just want to note the change in attitude I saw in your position from Winter 1990,
wherein you described yourself as "agnostic." In the Spring 1994 issue, you said you are "an
atheist who was once a bible-believing theist." Certainly, you gave out clues early on, but as
an atheist who has described herself that way for years, I was heartened to read it. To me, it
means quite simply one who is without a belief in god. We need to say it proudly, to de-
demonize it, and stop letting others define us.

My compliments to you and your contributors for the obvious enthusiasm and scholarship you
bring to this issue. I too care desperately about this issue and am reading everything I can get
my hands on that is written by nonbelievers, and I'm finding there is a lot available, but you
really have to hunt for it.

(Sandra Feroe, 1400 Pheasant Run, Berthoud, CO 80513.)

EDITOR'S NOTE: Jesus allegedly said, "The kingdom of heaven is like a mustard seed,
which a man took and sowed in his field, which indeed is the least of all the seeds; but when it
is grown it is greater than the herbs and becomes a tree, so that the birds of the air come and
nest in its branches" (Matt. 13:31-32). Fundamentalists quote this as if Christianity were
unique, but the statement merely expresses an ideological truth. One doesn't abandon belief in
an idea to accept another the first time he hears a new idea presented. The change occurs
gradually over a period of time. That was what happened to me. I didn't reject the Bible the
first time I encountered flaws in the inerrancy doctrine; the seed of doubt had to germinate
and grow. When I did step away from fundamentalist Christianity, I didn't do it as an atheist
but as an agnostic. With time, however, I realized that the "god concept" that Ms. Feroe
mentioned in her letter simply could not survive logical examination, so I declared myself an
atheist. I take great pride in my atheism and consider it the biggest step that I ever took in my
intellectual growth.

Sandra Till's article ("The Wisdom of Solomon," Summer 1991, pp. 8-10) received more
favorable comments than any single article we have published. I assume that most readers
know, or at least suspect, that she is my wife. I have encouraged her to write more on the
Bible and women's issues, but she doesn't share my enthusiasm for writing. This issue
contains "God's Opinion of Women" (pp. 11, 16), and we plan to publish more on this subject.
We do notify subscribers with a center insert when it is time to renew their subscriptions.
Until you see one in your copy, you don't have to wonder if your subscription is current.
Those who send renewal checks before receiving notification will have their subscriptions
extended another year.

There is indeed a lot of material available to debunk the Bible, but one has to know where to
look for it. Readers who are interested in such materials should contact Prometheus Books (59
John Glenn Drive, Buffalo, NY 14228-2197) and H. H. Waldo, Bookseller (P. O. Box 350,
Rockton, IL 61072) and ask to be put on their mailing lists. Those who have computers
should contact Emmett Fields, 514 Eastern Parkway, Louisville, KY 40217. For $10, you can
obtain 10 HD disks on which he has scanned numerous freethought books and publications
that are out of print or hard to find.

Volume 1990 - 2002 Issue


Page 623 of 2049
Skeptical Review Edited by Farrell Till
My son, Daniel Anderson, recently subscribed to your magazine The Skeptical Review. I
would like you to cancel his subscription immediately and refund any remaining amount.
Please send to me at the above address any amount owed back.

(Margene Anderson, 250 East 600 South, Bountiful, UT 84010.)

EDITOR'S NOTE: I informed Ms. Anderson that her letter contained insufficient information
to honor her request, because her son's age was not given. If he is old enough to make
responsible decisions about what he wishes to read, he should request the cancellation. The
fact that she expected a refund on a free subscription indicated that she had written without
even having consulted her son.

Less than a week ago while checking out the skeptical pages on the world-wide-web
(Internet), I had the great fortune of finding a site containing the text of all TSR issues. I
downloaded one issue from 1990, to check what it was all about, read it in one go then went
back, downloaded, printed and read all 20 of them. The question is, what am I going to do
until the next one arrives?

The $50 bill included in this letter will hopefully be enough to cover the costs of all published
issues as well as mailing costs to Sweden. I also hope you could put me on your subscription
list for the next few decades (got more dollars in a shoe box somewhere). In one of the last
issues, you've mentioned the great news that you would be retiring sometime during the year.
I say it's great news because I suppose you'll have more time to devote to writing, and perhaps
we'll see more than 4 issues per year. I subscribe to a great number of skeptical and
humanist/secular journals (such as The Skeptical Inquirer, Free Inquiry, etc.), but the material
in your review surpasses everything else in terms of clarity of thought and razor-sharp logic.
I'm referring, of course, to the skeptical material, because the contributions of the
"apologates" (McDonald, Dobbs, Mitchell, etc.) range from the pathetic through the
ridiculous to outright moronic. While debating the above mentioned gentlemen amounts to
nothing less than shooting dead fish in a barrel, I do hope they could be persuaded to keep
sending in their material, which, besides giving us all a good laugh, might help science in
documenting the depths of human stupidity and moral bankruptcy....

Finally, for all of your readers with access to Internet, I highly recommend the following
World Wide Web address: http://www.infidels.org/, which is maintained by Mr. Brett
Lemoine. It is a gold mine of (mostly) out-of-print freethought literature as you can see from
the list of contents I have included in this letter.

Mr. Lemoine writes, "Most of the files available here were painstakingly scanned in from
freethought literature, for which the copyright had expired, by Emmett F. Fields. Mr. Fields
gave copies of his work to Frank Prahl of the American Humanist Association. Mr. Prahl, in
turn, provided copies to me. I am making this literature available to the public so that more
people might be made aware of the depth and duration of the freethought movement in this
country and abroad. These files may be freely reproduced, copied, printed out, and given
away but may not be sold."

Volume 1990 - 2002 Issue


Page 624 of 2049
Skeptical Review Edited by Farrell Till
Notice that from this site you can, with a click of the mouse-button, download the complete
text of the book Is It God's Word? by Joseph Wheless (which you recommended in TSR, vol.
5, issue 1) as well as his other book from 1930, Forgery in Christianity. Let's face it, whether
they like it or not, they are "living on borrowed time."

(Kiriakos Papadopoulos, Jungfrudansen 100, 171 56 Solna, Sweden. E-mail:


Kiri.Papadopoulous@haninge.trab.se)

EDITOR'S NOTE: Where the ellipsis (...) appears in Mr. Papadopoulos's letter he had some
words of praise for Ruth Hurmence Green's book The Born Again Skeptic's Guide to the
Bible, which he incorrectly assumes is out of print. This book is still published by The
Freedom From Religion Foundation (P.O.Box 750, Madison, WI 53701) and can be
purchased for $10.

Subscribers with computers not connected to the Internet may wish to contact Emmett Fields
directly (address given above) to purchase a set of disks containing all of the freethought
materials that Brett Lemoine has available at the WWW address mentioned in Mr.
Papadopoulos's letter. A set of 10 HD disks may be purchased for only $10. Several entire
out-of-print freethought books are on these disks.

We have printed Mr. Papadopoulos's letter to emphasize a point we have made before: the
information age, which is only in its infancy, will undoubtedly bring startling changes in
religious thinking. The time is gone when church leaders can suppress information they don't
want their sheep to know. Anyone sitting at his computer practically anywhere in the world
can, by stroking just a few keys, access the Internet and all the information it contains. Much
of it is unfriendly to Bible fundamentalism.

While watching your debate on the existence of God with Jerry Moffitt, I was intrigued by his
use of probability arguments. This seems to be a constant theme with theists and creationists.

I think to assume that all natural processes are totally random is a mistake. Take the example
of atoms joining to form specific molecules. If you treat the joining of different atoms to form
all possible compounds as random, like flipping a coin or tossing dice, with each combination
as likely as any other, then any specific outcome is highly unlikely. But surely atoms do not
operate like coins or dice. Specific compounds would form more readily than others because
of the properties of atoms like carbon. The randomness that nature exhibits may not be the
pure mathematical randomness that creationists assume. Snowflakes would be another
example. Would a theist call snowflakes a miracle? I think not. Yet they are not random.
Order from chaos? Hardly!

Creationists can see only two alternatives: nature, which they think is random, and God,
which cannot leave room for evolution. They cannot see the possibility of complexity
evolving from simplicity. The chance of life's origin may have been a small finite one, but it
was an open question as to if it was totally random.

Volume 1990 - 2002 Issue


Page 625 of 2049
Skeptical Review Edited by Farrell Till
Finally, I do not see why they insist on placing the cause of things outside the universe, in an
unknown. Why not instead look for the explanation in the operation of known forces? They
ask, "What caused the Big Bang?" To which I reply, "Why would its cause be a God (much
less the God of the Bible) instead of a series of causes without end?" I think an eternal
universe is more likely than an unknown, eternal God. We know the universe with its various
forces exists; we cannot say the same for God.

(Richard G. Rich, 813 Fifth Avenue SE, Decatur, AL 35601.)

Editor's Note: I have received a lot of letters that scoffed at the absurd probability arguments
that Moffitt used in this debate. One of them, written by someone in attendance at the debate,
actually beat me home from Texas.

I am a recent skeptic, just liberated from fundamentalist Christianity. Praise reason!

Please send me a subscription to The Skeptical Review. I would be happy to pay for any back
issues you have.

(Andy Traiger, 10219 Stratford Avenue, Fairfax, VA 22030.)

Volume 1990 - 2002 Issue


Page 626 of 2049
Skeptical Review Edited by Farrell Till

The Skeptical Review


Volume Six, Number Five - 1995
Farrell Till, editor

• A Special Message From The Editor


The Skeptical Review has grown from zero to 1700, and new subscription requests
arrive daily. We will be publishing six times a year but we need your help.

• Bent Over Double For God


In the summer issue of TSR (pp. 2-3), we published an article in which Marion Fox
attempted to show that the banning of bastards from the assembly of Yahweh (Dt.
23:2) does not contradict any other biblical passages. Farrell Till counters that Fox's
explanations have little or nothing in the Bible to support them.

• Just Another Far-Fetched How-It-Could-Have-Been: a Rejoinder


Roger Hutchinson debates Farrell Till's response to his explanation of the discrepancy
between the length of the Israelite sojourn in Egypt and the Exodus 6 chronology in
the Aaronic genealogy.

• Murder Most High


William Sierichs, Jr. indicts God for crimes against humanity. God should be at the
top of the FBI's 10-Most-Wanted list. Christians often chide nonbelievers because we
supposedly are incapable of morality. How dare Christians demand that nonbelievers
bow down to such a murderous god!

• Can the Bible (or Any) God Support an Absolute Morality?


Tim Gorski, M. D., observes that there is much that is terrible and unjust in the world,
so that if there be a God, He can not be both all-good and all-powerful. Because if He
were, He would put an end to such things.

• Not Much To Do
Why doesn't Roger Hutchinson just admit that in this case we have an obvious
example of biblical discrepancy?

Volume 1990 - 2002 Issue


Page 627 of 2049
Skeptical Review Edited by Farrell Till
• American Society for Religious Concern
An unusual letter from an unusual group.

• A Response to Marion Fox


E. E. Brennaman disputes Fox's inerrancy doctrine.

• From the Mailbag


A collection of our most interesting letters.

• Announcements
Our back page tells you how to order videos and back issues.

Special Fifth Edition, 1995


An Important Message from the Editor
At the end of our first year of publication (in 1990), we issued a progress report but have
made none since then. We started in January 1990 with *no* subscribers. For that first issue, I
compiled a list of 200 or so names, primarily from an alumni newsletter I had recently
received from the Bible college I had attended, sent out a mailing, and waited. The main
response I received was a lot of copies that were returned marked REFUSED (which some of
you ordering back issues are now seeing, since that 1st edition has sold out and been reprinted
in loose-leaf format). There were also some angry, please-remove-my-name-from-your-list
letters and phone calls.

That has all changed--radically. In our six years of publication, the subscription list has grown
from zero to 1700, and new subscription requests arrive daily by regular mail, e-mail, and
phone calls. Only rarely do we receive demands to cancel subscriptions, and these usually
come from people who were sent the free first-year subscription at the request of friends or
relatives.

The first 8 issues of TSR contained 12 pages, but, never having enough space for everything
we wanted to publish, we switched to a 16-page format with the first issue of 1992. We still
didn't have enough space, but at least we were giving our readers more with no increase in the
subscription cost. The first 15 issues were set up on a dot matrix printer, but we switched to a
laser printer with the Autumn 1993 edition. That gave us a much more attractive format and
made it possible to include almost twice as much material on the 16 pages. We have since
upgraded the computer system, bought a better laser printer, and added e-mail and a fax
machine.

Each improvement was intended to make my workload easier, but it didn't always work out
that way. When I first started TSR, I received letters and requests for information only
occasionally, but as the subscription list grew, both of these increased. At first, I was able to

Volume 1990 - 2002 Issue


Page 628 of 2049
Skeptical Review Edited by Farrell Till
answer each letter (and even wish for more to answer), but that changed long ago. By 1994, I
was receiving more mail than I could possibly answer and had to publish occasional notices to
let subscribers know why some letters were not being answered. E-mail has made this
problem even worse. However, it is easier to answer e-mail than ordinary letters, so I urge
subscribers who have e-mail to use this medium to contact me. I can respond to these
messages by merely typing an answer and clicking onto a "send" icon, and this saves the time
of having to print a hard copy, address an envelope, put the letter inside, affix a stamp, and
take it to the post office. It is also much cheaper.

The main purpose of this special editorial message, however, is to announce another
important change. Beginning in 1996, TSR will be published bimonthly rather than quarterly.
Many of you have written or called to say that you wish the paper were published more often,
and I have decided to do this. Since I have never been able to publish everything I would like
to (including some good articles submitted by subscribers), I decided that my retirement,
which began last July, would give me enough time to add two more issues per year. The two
additional issues would accommodate your requests for more and would enable us to publish
articles that are on our waiting list.

Your help will be needed to make the bimonthly format successful. The first way you can
help would be by renewing your subscription. The two additional issues next year will add
several hundred dollars to our operational costs. In the past, I personally covered all losses
when we were operating in the red, but I may not be able to do that very often on a retirement
budget. TSR had just reached a break-even point when I decided to change to a bimonthly
format, so we will need to maintain this year's paid-subscription level at the new $6 rate (still
$1 per issue) through next year in order to remain solvent.

Most subscriptions (but not all) expire with this issue. If your copy has a renewal notice
inserted inside (between pages 10 and 11), this means that your subscription expires with this
issue. To renew (and assist us in meeting our goals), just complete the form and mail it back
with your $6 renewal check. If no insert is in your copy, your subscription has not yet expired.

Because subscriptions have increased steadily since we began publication, we can reasonably
expect this to continue. If so, my workload will also increase, because The Skeptical Review is
essentially a one-person operation. In addition to writing many of the articles, I set up all
articles submitted from outsiders, keep the subscription list current, package and mail all
orders for back issues and other materials, answer as much correspondence as I can, run
address labels, affix them to all copies, and complete the forms for bulk mailings. Needless to
say, all of these tasks make heavy demands on my time.

If you want to submit articles, I would encourage you to do so. Some of our best articles have
been written by our subscribers. However, you can help lighten our load here if you will send
ASCII copies of your articles along with your hard copies. With only a hard copy of an
article, I must type it and then set it up in our publishing format. Depending on the length of
an article, this could take up to two or three hours, but with an ASCII copy, I can have it
formatted within a few minutes. Of course, if you have no computer (and few writers don't
these days), we will still consider typed or even handwritten articles. We would never let a
little work keep us from publishing a deserving article.

Volume 1990 - 2002 Issue


Page 629 of 2049
Skeptical Review Edited by Farrell Till
For those who do want to submit articles, we ask you to remember that our publishing focus is
Bible inerrancy. We have received many articles on religious subjects not related to the
inerrancy issue, political subjects, and even works of fiction, but we cannot consider these for
publication.

Finally, we would urge our inerrantist readers to take advantage of our offer to publish
responses to any of our articles and to publish competently written defenses of the inerrancy
doctrine simultaneously with our responses. This policy gives our readers the opportunity to
hear both sides of the issue and gives inerrantists an audience of skeptics they would not
likely find in their own publications.

Bent Over Double for God


by Farrell Till
In the summer issue of TSR (pp. 2-3), we published an article in which Marion Fox attempted
to show that the banning of bastards from the assembly of Yahweh (Dt. 23:2) does not
contradict any other biblical passages. Mr. Fox presented five "solutions" to the problem, all
of which consisted of how-it-could-have-been explanations that have little or nothing in the
Bible to support them. Because I want to expose as thoroughly as possible the flimsiness of
inerrantist methods of resolving Bible "difficulties," I have had to divide my response into
two installments. This time I will reply to Fox's "solutions" that I didn't have space for in my
first response.

One of his "explanations" was an attempt to show that Perez, a direct ancestor of both David
and the sons of Aaron, was the offspring of a lawful levirate marriage, and so he was not a
bastard. In support of his case, Fox argued that "the union of Judah and Tamar was parallel to
the union of Boaz and Ruth," and so "the union of Judah and Tamar was also a levirate
marriage."I have already devoted enough space to demolishing Mr. Fox's quibbles, so I will
not quote in their entirety the passages in Ruth that give the details of Ruth's levirate marriage
to Boaz. However, I urge the readers to review chapters 3 and 4 in the book of Ruth to get
those details. Ruth, a widow with no children, made herself sexually available to Boaz and
urged him to "spread his skirt over her" (a euphemism whose meaning should be obvious) on
the grounds that he was "a near kinsman" (3:9 ). Boaz refused the offer because there was
another man whose kinship to Ruth's dead husband was closer than his, and not until Boaz
had discussed the situation with that other kinsman and received a waiver of this man's right
to Ruth did Boaz consummate the levirate marriage.

Now Mr. Fox apparently expects us to believe that this case was "parallel" to Judah's and
Tamar's escapade. Under levirate law, Tamar was entitled to be impregnated by Judah's third
son Shelah, but Judah had withheld him from Tamar. Before he had sexual relations with
Tamar, Judah did not consult his son Shelah for a waiver of levirate rights. He didn't even
know who Tamar was, because she had disguised herself as a prostitute in order to trick Judah

Volume 1990 - 2002 Issue


Page 630 of 2049
Skeptical Review Edited by Farrell Till
into the sexual relationship. Judah left a "signet, cords, and staff" with Tamar as a pledge that
he would later send her a kid from the goats of his flock as payment for her services, so he
obviously thought that he was striking a deal with a prostitute. Tamar left before Judah sent
the kid back to redeem his pledge, so she was able to use the signet, cords, and staff later to
prove that he was the man who had impregnated her.

So Mr. Fox has a strange concept of what constitutes parallelism. Boaz went to great lengths
to make his relationship with Ruth legal in terms of the laws of the society they lived in.
Judah was simply interested in a quick roll in the hay. Fox said that "Tamar had the right to
have a child by the next of kin, and if she had this right the child was not a bastard," but that is
precisely the point. She didn't have her twins by her dead husband's next of kin. That next of
kin was Shelah, her husband's brother, not Judah, her husband's father. Jewish law, in fact,
prohibited sexual relations between a man and his daughter-in-law (Lev. 20:12). So just what
is there in the story of Judah and Tamar that makes it so parallel to Boaz and Ruth?

Perhaps Mr. Fox is reading too much into a statement attributed to the people who had
witnessed the transaction between Boaz and the "nearer kinsman" who waived his right to
Ruth: "And all the people who were at the gate, and the elders, said, "We are witnesses.
Yahweh make the woman who is coming to your house like Rachel and Leah, the two who
built the house of Israel; and may you prosper in Ephrathah and be famous in Bethlehem.
May your house be like the house of Perez, whom Tamar bore to Judah, because of the
offspring which Yahweh will give you from this young woman" (Ruth 4:11-12)."

Mr. Fox apparently wants to twist this passage to mean that God considered the levirate
marriage of Boaz and Ruth to be parallel to Judah's and Tamar's relationship. Let's notice,
however, that the passage doesn't claim that God made this statement; it was merely
something that the witnesses to the transaction allegedly said. So if Mr. Fox is going to claim
that the statement was inspired truth, then he puts himself in the position of having to say that
whenever the Bible reported what people said, those statements were always true, and I doubt
if he wants to do that.

For the sake of argument, let's just assume that the statement was "inspired truth." That
wouldn't prove anything, because the witnesses who made the statement said nothing to
suggest their approval of Tamar's and Judah's liaison; they merely expressed hope that Boaz's
"house" (descendants) would become like the "house of Perez," which had been the branch of
the tribe of Judah that produced David and other Judean kings. This book was obviously
written after the time of David, because the genealogy in 4:18-22 listed him. So if Mr. Fox
wants to play a how-it-could-have-been game, we can meet him on his own turf. It very well
could have been that the witnesses to Boaz's transaction (if it ever even happened) never said
anything like what the writer of Ruth attributed to them. He could have just put into the
mouths of the witnesses words that would pay homage to the famous king who had descended
from Perez. Be that as it may, if the witnesses to Boaz's transaction with the "nearer kinsman"
did actually make a statement like this, it can be seen as nothing more than an expression of
hope that Boaz's marriage to Ruth would be a fruitful one. Mr. Fox is really grasping for
straws when he uses the passage to try to prove that the offspring of Judah's and Tamar's
liaison was just as "legitimate" as the son produced by the levirate marriage of Boaz and Ruth.

Volume 1990 - 2002 Issue


Page 631 of 2049
Skeptical Review Edited by Farrell Till
Mr. Fox tried to argue that "Tamar was righteous to ave the child" and cited Genesis 38:26 in
support of the claim. In this verse, there is nothing but a statement allegedly made by Judah
when Tamar surprised him with the signet, cords, and staff that he had left with her as a
pledge to send her a kid later. When Judah saw the tokens he had pledged, he acknowledged
them and said, "She has been more righteous than I, because I did not give her to Shelah my
son." Let's notice, then, that Judah did not say that Tamar was righteous. He said that she had
been more righteous than he, because he had withheld his son Shelah from her. In other
words, Judah was saying that his violation of the levirate law in withholding his son from
Tamar was worse than what she had done. How does Fox get from this that Tamar's
disguising herself as a prostitute and tricking her father-in-law into impregnating her was an
act of "righteousness"?

Let's just assume, however, that Judah did mean to say that what Tamar had done was
righteous. That would prove no more than that Judah *said* that Tamar was righteous, so we
are right back where we were a moment ago. Is Mr. Fox going to take the position that
everything said by biblical characters was true? He needs to reexamine an important
hermeneutic principle accepted by almost all Bible inerrantists. This principle says that when
the Bible reports what characters said, it is inerrant in stating what was said but what was said
wasn't necessarily inerrant. An example often cited to illustrate this principle is found in G
enesis 3:4, where the serpent said to Eve, "Ye shall not surely die." Fundamentalists would
say that the Bible is inerrant in reporting that the serpent *said* this, but what the serpent said
was not inerrant, because eating the forbidden fruit did bring death into the world. So with
this heremeutic principle in mind, maybe Mr. Fox would like to explain why what Judah may
have thought about the righteousness of Tamar's actions would prove anything.

The extent of Fox's desperation is shown in his attempt to prove that the expression "knew
her" meant that Judah's and Tamar's sexual relations constituted marriage. "This expression is
commonly used to refer to sexual relations in marriage ( Gen. 4:1)," he said. I suppose, then,
that he will argue that when the men of Sodom surrounded Lot's house and told him to bring
out the men (angels) visiting him "that we may *know* them" (Gen. 19:5), they were actually
making a marriage proposal. The same would apply to the men of Gibeah in Judges 19:22,
who called for the Levite to be brought out so "that we may *know* him." *Know* was used
in the Bible in reference to conjugal sex, but it was also used to refer to sex outside of
marriage. Genesis 38:26 said, "And he [Judah] *knew* her [Tamar] again no more," so the
word wasn't used to describe sexual relations within marriage but sexual relations that never
occurred again after Judah's "business transaction" with Tamar. Mr. Fox is desperate, and I
think he knows it. On this point, he isn't just leaning over backwards for God; he has bent
over double in search of a quibble-- just any quibble--to shore up a conspiculously weak spot
in the inerrancy doctrine.

Mr. Fox's third "explanation" concerned *ex post facto* or retroactive law, and he was so
vague about it that I'm really not sure I even understand what he meant. He said that my
"interpretation of this passage [Dt. 23:2?]... requires that it be an *ex post facto* law." What is
the antecedent of *it*? I'm not sure. Does he mean that the ban on bastards was given after the
sons of Aaron were appointed priests, and so this law cannot be applied *ex post facto* to
them? If so, then perhaps he should reconsider his argument. As he noted, the levirate
marriage law was given in Deuteronomy 25:5-10, yet he tried to apply that law retroactively

Volume 1990 - 2002 Issue


Page 632 of 2049
Skeptical Review Edited by Farrell Till
to Judah and Tamar to prove that their sexual relations were legal. Isn't he just a little bit
inconsistent?

He even formulated another syllogism to prove that I was begging the question. The "First
Premise" in his syllogism said, "All those who make *ex post facto* laws are those who are
unrighteous." He then said that this premise is "axiomatic," so if I can show that Yahweh has
enacted *ex post facto* laws, I assume that Mr. Fox will agree that Yahweh is "unrighteous."

Numbers 15:32-36 tells us that Yahweh once commanded an *ex post facto* punishment. A
man who was found gathering sticks on the sabbath day was taken to Moses and Aaron and
the whole congregation, and they "put him in ward, *because it had not been declared what
should be done to him*" (v:34). Yahweh then said to Moses, "The man shall surely be put to
death: all the congregation shall stone him with stones without the camp" (v:35). The next
verse states that the sentence was carried out. So here is the case of a man on whom Yahweh
pronounced an *ex post facto* punishment of death. By Mr. Fox's own admission, then,
Yahweh is axiomatically unrighteous, so why are we debating?

Mr. Fox's fourth "explanation" can be handled very quickly. He seems to think I have been
arguing that the expression "enter into the assembly of Yahweh" meant to serve as a leader,
but I have never even implied that. "Entering into the assembly" meant to participate in
religious ceremonies, and the entire context of Deuteronomy 23 quoted in my first response
makes that rather clear. Banning Ammonites, Moabites, Edomites, and Egyptians from the
assembly didn't mean that they couldn't become leaders (although the bigotry expressed in
this law would undoubtedly have prevented it); it meant that they couldn't enter into the
assembly where religious ceremonies took place. Likewise, banning bastards and those with
mutilated genitals meant that they could not participate in these ceremonies. So it wasn't a
matter of whether David and the sons of Aaron could become "leaders." This law should have
prohibited them from "entering the assembly," but it didn't. Mr. Fox has yet to give us a
satisfactory reason why they were exempted from the ban.

In offering his fifth "explanation," Mr. Fox understated my position. I have never said that
only Phinehas should "have come under the ban"; I have said that Phinehas's father, Eleazar,
as well as Aaron's three other sons, Nadab, Abihu, and Ithamar, should have been banned as
well. Their mother, Elisheba, was a great-grandaughter of Perez, so all of her sons would have
been fifth-generation descendants of a bastard, and Phinehas, Elisheba's grandson, would have
been a six-generation descendent. Nevertheless, Yahweh appointed all of them to be priests of
Israel (Num. 3:1-3, 10; 18:7), and he was so pleased with Phinehas for running a spear
through a couple caught in the act of adultery that he bestowed an "everlasting priesthood" on
Phinehas and "his seed after him" (Num. 25:7-13). Now why did Yahweh bestow honors that
required these men to perform specific duties in the assembly (Num. 4) if descendants of
bastards were to be banned from the assembly of Yahweh "even to the tenth generation"?

Well, of course, Mr. Fox, as any self-respecting inerrantist will always do, has come up with a
possible solution to this problem. Aaron's wife Elisheba was described as "the daughter of
Amminadab" and "the sister of Nahshon" ( Ex. 6:23), so Mr. Fox has declared that I "must
know, not just think" but know, that the Amminadab and Nahshon in E xodus 6:23 were the
same Amminadab and Nahshon who were listed as descendants of Perez in the genealogy of

Volume 1990 - 2002 Issue


Page 633 of 2049
Skeptical Review Edited by Farrell Till
David. Fox further declared that at least three men in the Bible were named Amminadab (1
Chron. 2:10; 6:22; 15:10). "Is it possible," he asks, "that the Amminadab of E xodus 6:23 is a
fourth Amminadab?" He even suggested that the Amminadab of Exodus 6:23 could have been
"the same Amminadab of 1 Chronicles 6:22."

I do wish that inerrantists would occasionally show a little originality. Whenever they
encounter difficulties involving conflicting details that the Bible attributes to a particular
person, they invariably trot out the same-name-but-different-persons dodge that Mr. Fox has
resorted to in offering us his fifth "explanation." Exodus 12:40 states that the Israelites
sojourned in Egypt 430 years, a time period that is hard to reconcile with a genealogy in
Exodus 6: 18-20 that indicates Moses and Aaron were grandsons of Kohath, a son of Levi
who was born before the Israelites went into Egypt (Gen. 46:11). In the very first issue of
*TSR* ("The Inerrancy Doctrine Is Found To Be Impregnable," Winter 1990, p. 8), Jerry
Moffitt offered an unlikely two-Amrams theory in an attempt to show that the Amram who
was the father of Moses and Aaron ( Ex. 6:20) was not the same Amram who was Kohath's
son ( Ex. 6:18). Moffitt's theory didn't float, and we will see that Fox's won't either.

Although I haven't directly asked him, I am sure Mr. Fox, as a loyal inerrantist, believes that
the Pentateuch was written by Moses. The best biblical scholarship in the world will disagree
with this fundamentalist belief, but for the moment we are going to assume that Moses did
indeed write the Pentateuch. If he was the author of these five books, then basic hermeneutics
should require us to interpret them according to perspectives we would expect someone on the
scene at the time of the exodus would have had in recording events he was personally
involved in.

With that in mind, let's notice that "Moses" described Aaron's wife Elisheba as "the daughter
of Amminadab" and "the sister of Nahshon" ( Ex. 6:23). This description was given in a
matter of fact way as if "Moses" thought that his readers would understand exactly who
Amminadab and Nahshon were. The Exodus writer (Moses?) didn't always write so matter-
of-factly. Even in the same context where Elisheba was identified as "the daughter of
Amminadab" and "the sister of Nahshon," the writer went to great lengths to explain exactly
who Moses and Aaron were: "These are the same Aaron and Moses to whom Yahweh said,
'Bring out the children of Israel from the land of Egypt according to their armies.' These are
the ones who spoke to Pharaoh king of Egypt, to bring out the children of Israel from Egypt.
These are the same Moses and Aaron" (vv:26-27). We could hardly expect a writer as specific
as this to have used the names Amminadab and Nahshon just three verses earlier without
clarifying their identities if he had thought that his readers might not understand exactly
whom he was referring to.

For some reason, the Exodus writer thought it was important to identify Elisheba as the
daughter of Amminadab and the sister of Nahshon, so the sensible conclusion to reach is that
he thought that these names needed no explanation. Mr. Fox quibbled about the three different
Amminadabs who are mentioned in the Bible. Of these three, the one in 1 Chronicles 15:10
was a contemporary of David, so he could hardly have been the Amminadab who was the
father of Aaron's wife. Of the other two, only one was ever identified as the father of a son
named Nahshon, and he figured so prominently in the affairs of the exodus (which happened
under the leadership of Moses and Aaron) that common sense should be sufficient to convince

Volume 1990 - 2002 Issue


Page 634 of 2049
Skeptical Review Edited by Farrell Till
any objective person that this was the Nahshon who was Aaron's brother-in-law (the brother
of Elisheba).

In Numbers 1:1-4, Yahweh told Moses to take a census of the tribes of Israel and to appoint a
"man of every tribe" to assist him in the census. For the tribe of Judah, Moses selected
Nahshon, the son of Amminadab ( v:7). In Num bers 2, Moses designated the camping
locations of the different tribes with reference to the tabernacle. The tribe of Judah was to
camp on the east side, and Nahshon the son of Amminadab was chosen to be "the prince of
the children of Judah" ( v:3). When the princes of the tribes offered their first oblations at the
dedication of the altar in the tabernacle, Nahshon the son of Amminadab made the offering
for the tribe of Judah (Num. 7:12,17). Nahshon the son of Amminadab also led the tribe of
Judah when "the children of Israel set forward according to their journeys out of the
Wilderness of Sinai" (Num. 10:12-14). In summarizing this information, I have emphasized
that the Nahshon involved in all of these matters was a leader in the tribe of Judah. This is
important to remember, because several genealogies in the Bible claim that Nahshon of Judah
had descended from Judah through the bastard Perez. Furthermore, this Nahshon is the only
Nahshon mentioned anywhere in the Bible, *unless the Nahshon who was Elisheba's brother
was indeed a different Nahshon*.

To exclude this as a reasonable possibility, let's consider the extremes that Mr. Fox would
have to resort to if he should try to defend this premise. (1) He would have to argue that
"Moses" took the time to identify his sister-in-law Elisheba as the daughter and brother of
men who are mentioned nowhere else in the Bible. (2) He would have to argue that the
Nahshon who was Elisheba's brother by sheer coincidence was the son of a man who had the
same name as the father of the Nahshon who figured so prominently in the tribal affairs of
Judah at the time of the exodus. (3) He would have to argue that the verbally inspired
"Moses" didn't receive enough guidance from the omniscient, omnipotent Holy Spirit to avoid
a writing gaffe that would leave his readers throughly confused. One doesn't even have to be
inspired by an omniscient, omnipotent deity to realize that just mentioning the names
Amminadab and Nahson in passing, with nothing more said to explain who they were, would
be confusing to readers if this Amminadab and this Nahson were not the same Amminadab
and Nahshon who would later be mentioned often in recording affairs of the tribe of Judah
during the wilderness wanderings.

The mere suggestion that the Amminadab and Nahshon who were the father and brother of
Elisheba were not the same Amminadab and Nahshon of the tribe of Judah runs contrary to
the best biblical scholarship. I have consulted several Bible dictionaries, and they all describe
Amminadab as the father of Nahshon and Elisheba. Likewise, they describe Nahshon as the
brother of Elisheba and a leader of the tribe of Judah. The following quotations from
*Eerdmans Bible Dictionary* are typical of what Bible dictionaries say about these two:

AMMINADAB: The father of Nahshon ( Num. 1:7) and of Elisheba the wife of Aaron
( Exod. 6:23). *He was an ancestor of King David* (Ru th 4:19) and is mentioned in
the genealogies of Jesus (M att. 1:4; Luk e 3:33; KJV follows Gk. Aminadab*).
NAHSHON: A son of Amminadab and brother of Elisheba the wife of Aaron (E
xodus 6:23; KJV "Naashon"). *He represented Judah in assisting Moses with the first
census ( Num. 1:7) and led that tribe during the wilderness wanderings* (2 :3; 10:14;

Volume 1990 - 2002 Issue


Page 635 of 2049
Skeptical Review Edited by Farrell Till
cf. 7:12, 17). He is reckoned among the ancestors of David (Rut h 4:20; 1 Chron. 2:10-
11) and Jesus (M att. 1:4; Luk e 3:32; KJV "Naasson").
Without a doubt, compilers of the major Bible dictionaries considered the Amminadab and
Nahshon who were father and brother to Elisheba to be the same Amminadab and Nashon
who were listed as father and son in the genealogy of King David. And why shouldn't they so
consider them? Only someone trying to shore up an untenable position like biblical inerrancy
could possibly read the books of Exodus and Numbers and reach any other conclusion but
that the names Amminadab and Nahshon, when used in these books, always referred to the
same two individuals.

Only one other point needs to be made about Mr. Fox's five "solutions" to the problem posed
by Deuteronomy 23:2. He has quibbled that the Amminadab who was the father of Elisheba (
Ex. 6:23) could have been the Amminadab referred to in 1 Chronicles 6:22. In this verse,
Amminadab was identified as a son of Kohath. We have already noted that Kohath was the
grandfather of Moses and Aaron (Ex. 6:18-20), so what Mr. Fox has done is suggest that
Aaron may have married his uncle's daughter. An analysis of 1 Chronicles 6:22 in context will
show that this is a quibble that only desperation could have driven Mr. Fox to: " The sons of
Kohath were Amram, Izhar, Hebron, and Uzziel. The sons of Merari were Mahli and Mushi.
Now these are the families of the Levites according to their fathers: of Gershon were Libni his
son, Jahath his son, Zimah his son, Joah his son, Iddo his son, Zerah his son, Jahath his son,
Zimmah his son, and Jeatherai his son. The sons of Kohath were Amminadab his son, Korah
his son, Assir his son, etc. (1 Chron. 6:18-22)."

I realize how tedious--and even outright boring--genealogical analysis can be, but if we look
carefully at this entire passage and compare the genealogical listings to the Exod us 6
genealogy, we can only conclude that the Chronicle writer miscued in saying that Kohath had
a son named Amminadab. Either that or some scribe copying the book made a "copyist error."

Let's notice that the first sentence of the passage stated that Kohath (the grandfather of Moses
and Aaron) had four sons: Amram, Izhar, Hebron, and Uzziel. This agrees perfectly with E
xodus 6:18, which listed Amram, Izhar, Hebron, and Uzziel as the sons of Kohath.
Nevertheless, the Chronicle writer just four verses later said that Kohath had a son named
Amminadab. Why?

To answer that question, I'm going to ask the readers to imagine that Deuteronmy 23:2, which
banned bastards from the assembly of Yahweh, was not in the Bible. In that case, we would
not be discussing the issue now under consideration. If that were the case and I should try to
argue that the genealogies in Exod us 6 and 1 Chronicles 6 are contradictory, because the
latter listed an "Amminadab" as the son of Kohath, yet Exodus 6 did not refer to any
Amminadab in listing Kohath's sons, what do you suppose Mr. Fox would say about this?
Undoubtedly, he would say the same thing that I am going to say to answer his quibble. The
"Amminadab" listed as Kohath's son in 1 Chronicles 6:22 was the same person as the Izhar
listed as his son in E xodus 6:18.

Let's notice that the "Amminadab" of 1 Chronicles 6:22 had a son named Korah, but Kohath's
son Izhar also had a son named Korah: "The sons of Izhar were Korah, Nepheg, and Zichri" (
Ex. 6:21). In Numbers 16:1, Korah was called "the son of Izhar." Everywhere the sons of

Volume 1990 - 2002 Issue


Page 636 of 2049
Skeptical Review Edited by Farrell Till
Kohath were named in genealogies, with the one exception of 1 Chronicles 6:22, they were
always listed as Amram, Izhar, Hebron, and Uzziel (E x. 6:18; Num. 3:19; 1 Chron. 6:18, 38;
23:12). When the family names of the Kohathites were listed, they were always listed as the
Amramites, the Izharites, the Hebronites, and the Uzzielites ( Num. 3:27; 1 Chron. 26:23).
Reference was never made to a family of Amminadabites, and that is because the correct
name for the "Amminadab" of 1 Chronicles 6:22 was Izhar.

Further proof of that can be found in the same context where Izhar was called Amminadab
(probably inadvertently). The generations in the genealogy of this Amminadab were listed as
Korah, Ebiasap, Assir, and Tahath (1 Chron. 6:22, but just a few verses later, the Bible in its
uniquely repetitious style listed the generations in reverse: Tahath, Assir, Ebiasap, Korah, and
Izhar (vv:37-38).

Can anyone in the face of such evidence deny that the "Amminadab" of 1 Chronicle 6:22 was
the son of Kohath known everywhere else in the Bible as Izhar? A footnote affixed to 1
Chronicles 6:22 in my reference Bible says of this Amminadab, "In verse 2, 18, 38,
*Izhar*."*Eerdmans Bible Dictionary* says of this Amminadab, "A Levite, the son of Kohath
and father of Korah (1 Chron. 6:22); *called Izhar in parallel lists* (E x. 6:21; Num. 16:1; 1
Chron. 6:38)."

Mr. Fox may want to argue that even if the Amminadab of 1 Chronicles 6:22 was the same
person called Izhar elsewhere in the Bible, he was called Amminadab at least once, so perhaps
"Moses" also called him Amminadab in E xodus 6:23 where Elishiba was described as the
daughter of Amminadab. To believe this, one must think that "Moses" referred to one of his
uncles as Izhar in E xodus 6:18 but just five verses later, without informing his readers,
switched and called this uncle Amminadab and then nowhere else in the entire Pentateuch
used this name again in referring to this uncle. If Mr. Fox can believe this, he may be
interested in buying the Brooklyn bridge from me. I'll let him have it cheap.

In my first article, I promised that I would comment on Mr. Fox's discussion of "common
errors made by atheists and agnostics," so I will look briefly at some of his "syllogisms." He
accused skeptics of committing the fallacy of denying the antecedent, but as Jim Lippard
points out in his letter (p. 13, this issue), Mr. Fox has in some of his examples distorted
inductive reasoning processes used by some skeptics into "deductive straw men." Mr. Lippard
explains the error far better than I can, so his letter should be read in conjunction with this
paragraph. I am, however, going to surprise Mr. Fox and at least admit that skeptics do
sometimes induce too much from the silence of extrabiblical records (as was done in the case
of the Hittite nation). I deplore this as much as anyone, but I am not responsible for what
others do. I wish that he had been more specific and cited an example of when I have done
this. I can't respond to a charge like this when no specific example of the mistake (in my
articles) is cited.

I have been honest enough with him to admit that some skeptics do commit errors in
reasoning, so now I wonder if he will reciprocate and admit that some of his cohorts do the
same when they try to defend the inerrancy doctrine. Let's look at his first syllogism:

First Premise: If Farrell Till has climbed Mount Everest, then Mount Everest exists.

Volume 1990 - 2002 Issue


Page 637 of 2049
Skeptical Review Edited by Farrell Till
Second Premise: Farrell Till has not climbed Mount Everest.
Conclusion: Therefore, Mount Everest does not exist.
I don't recall ever reasoning in this way, but if I have, perhaps he would be accommodating
enough to cite a specific example. Meanwhile, I will ask him to consider a parallel syllogism:
First Premise: If Farrell Till has found an error in the Bible, then there is an error in
the Bible.
Second Premise: Farrell Till has not found an error in the Bible.
Conclusion: Therefore, there is no error in the Bible.
Now I wonder if Mr. Fox is willing to admit that this is exactly the way that most inerrantists
reason. Will he admit that it was the way that he reasoned all through his article? He proposed
a second "syllogism" that he sees as the way that atheists and agnostics reason:
First Premise: If Christian "X" can refute my claim of an error in verse "Y," then there
is no error in verse "Y."
Second Premise: Christian "X" cannot refute my claim of an error in verse "Y."
Conclusion: Therefore, there is an error in "Y."
I recognize the logical principle that says that the absence of negative evidence does not
constitute positive evidence and vice versa, so I don't think that his syllogism represents the
way that I reason. If I am wrong about this, perhaps he will cite a specific example of when I
have violated this principle. Meanwhile I will counter this syllogism with one that represents
the way that he and his cohorts reason:
First Premise: If skeptic "X" can prove an error in verse "Y," then there is an error in
verse "Y."
Second Premise: Skeptic "X" cannot prove an error in verse "Y."
Conclusion: Therefore, there is no error in verse "Y."
Is it fair to say that inerrantists reason in this way? Well, the essence of everything Mr. Fox
said in his article was that my failure to prove that Deuteronomy 23:2 conflicts with other
biblical passages proves that Deuteronomy 23:2 does not conflict with other biblical passages.
All through his article, he played the old how-it-could-have been game that fundamentalists
invariably resort to in their defense of Bible inerrancy. He said, "David might have been the
generation that was allowed to enter the assembly (the tenth generation)." He said that the
sexual union of Judah and Tamar could have been a legal levirate marriage. He said that the
Amminadab and the Nahshon who were Elisheba's father and brother could have been a
different Amminadab and Nahshon than the ones in David's genealogy. Yes, he offered us a
lot of could-have-beens but not a single scrap of proof that any of his could-have-beens were
what really happened.

I get tired of pointing out to Bible fundamentalists that this approach to proving biblical
inerrancy ultimately eliminates the entire logical concept of contradiction, because no matter
how obviously the face-value meaning of statements within a document may seem to
contradict each other, one can always dream up a "possible" explanation that would remove
the contradiction. To argue that such an approach to resolving contradictions is valid is to say
that absolute proof of contradiction (in any document) is impossible. On the other hand, I
have accomplished two things in this series of articles: (1) I have shown that there is
reasonable proof that Yahweh's ban on bastards conflicts with other biblical statements, and
(2) I have given reasonable proof that Mr. Fox's "solutions" to the problem are baseless
quibbles. If he disagrees, I will give him space to reply.

Volume 1990 - 2002 Issue


Page 638 of 2049
Skeptical Review Edited by Farrell Till

"Just Another Far-Fetched How-It-Could-


Have-Been: A Rejoinder,"
by Roger Hutchinson
The significance of Farrell Till's response to my explanation of the discrepancy between the
length of the Israelite sojourn in Egypt and the Exodus 6 chronology in the Aaronic genealogy
should not be underestimated. He did his homework well, conducted an excellent analysis,
and reached what, I believe, were logical conclusions. After all that, is it possible that Mr. Till
could have missed something?

To investigate this, we need to look at the points on which Mr. Till and I agree. We agree that
the Bible clearly states that Israel sojourned in Egypt for some 430 years. This is shown in
three scriptures:

And he [the LORD] said unto Abram, Know of a surety that thy seed shall be a
stranger in a land that is not theirs, and shall serve them; and they shall afflict them
four hundred years (Gen. 15:13).
Now the sojourning of the children of Israel, who dwelt in Egypt, was four hundred
and thirty years. And it came to pass at the end of the four hundred and thirty years,
even the selfsame day it came to pass, that all the hosts of the LORD went out from
the land of Egypt (Ex. 12:40-41).
And God spake on this wise, That his seed should sojourn in a strange land; and that
they should bring them into bondage, and entreat them evil four hundred years (Act s
7:6).

We also agree that the genealogy in Exod us 6, repeated in 1 Chronicles 6, lists the
descendants of Levi in the following order: Levi, Kohath, Amram, and Aaron. We agree that
Levi entered Egypt at the beginning of Israel's sojourn and that Aaron left Egypt at the end of
the sojourn.

We both agree that Genes is 15:16 speaks of Israel's coming out of Egypt in the fourth
generation.

Finally, I think we both agree that there are problems inherent in any effort to reconcile the
genealogy in Exod us 6 with the 430 years in Exo dus 12. Till described these problems in his
analysis.

The real issue is not what Mr. Till or I think about these scriptures but what the writer(s) of
Exodus meant to convey to the reader. If the writer considered both Exodus 6 and 12 to be
accurate and consistent, then Till's analysis has failed to discover what the writer was saying.
If the writer knew that an inconsistency existed, then Till's analysis is correct, and either the

Volume 1990 - 2002 Issue


Page 639 of 2049
Skeptical Review Edited by Farrell Till
writer of Exodus was lazy in not correcting the information or he deliberately sought to
mislead the reader. Another possibility is that the writer did not proofread well and missed the
inconsistency altogether. So what was the writer of Exodus seeking to tell us when he wrote
Exodus?

The writer(s) of Exodus stated clearly that Israel spent 430 years in Egypt. That number is
repeated twice and emphasized by the language "even the selfsame day it came to pass...."
Was he lying? Stephen certainly believed the statement was accurate because he referred to
the "four hundred year" sojourn in Acts 7. We have good reason to assume that the writer of
Exodus believed that Israel sojourned 430 years in Egypt, and that he sought to convey that
information in as clear and direct manner as possible so that it would not be misunderstood.
The writer succeeded. I understand it. Till understands it. Everyone understands it. Of course,
not everyone believes it, and that has led to the current discussion.

The writer(s) of Exodus also went into great detail to describe the Aaronic genealogy. Exod
us 6 gives such detail that, again, there is no room to misunderstand what is said. The writer
of Chronicles certainly believed that this was the correct genealogy, because it is repeated
there. Were these people confused? Given the importance of genealogies in the Bible, it
seems plausible to assume that very careful records were kept and that this was one of those
records. Not only do we understand the basic genealogy presented, but there seems to be no
reason to think that the information recorded in Exod us 6 would be wrong. Both Till and I
accept this genealogy and incorporated it into our arguments.

Considering the above, I see only two scenarios that rate serious consideration. Either the
writer(s) of Exodus understood Exodus 6 and 12 to be accurate and consistent, or the writer(s)
deliberately perpetrated a hoax by introducing false information. I do not see any support for
the position that the writer purposely set out to deceive the reader or sought to provide
information that was not true. On the contrary, if the writer had sought to enhance the
reputation of God, as promise keeper or whatever, then he would have been careful to make
sure that everything he said was consistent. After all, if Mr. Till can look at these scriptures
and see a problem, how much more would people in that day have seen the same problem?
There seems to be nothing that the writer would have gained by doing so. Considering the
reaction by Mr. Till and others, introducing an inconsistency would have proven disastrous
long before now.

What are we to make of this? If Till's analysis is correct, then we must conclude that it was
impossible for Israel to have spent 430 years in Egypt. If Israel did indeed spend 430 years in
Egypt, then we must conclude that Till's analysis has missed something. Where could Till's
analysis have gone wrong? One point on which his analysis could be wrong is in his
conclusion that the key Hebrew words he analyzed are too narrow in meaning to allow for a
reconciliation of Exodus 6 and 12. Here, Till has merely looked at how the same word is used
elsewhere in the Bible and drawn conclusions based on that use. Till's methodology certainly
cannot be faulted. At the same time, there are occasions where the key Hebrew words are
used in a broader sense. For example, when Till says that "anyone with common sense knows
that more than just four generations will live and die over a period of 400 years," he is
defining generation as the average span of time between the birth of parents and their
offspring. Thus, Till defines four generations to encompass a father/son/ grandson/ great-

Volume 1990 - 2002 Issue


Page 640 of 2049
Skeptical Review Edited by Farrell Till
grandson span that would cover about 100 years. However, if we define one generation to
encompass one complete father/son/grandson span or more, then it becomes possible to
conclude that four generations could span 430 years. We have one example of this broader
sense in Nu mbers 32:

And the LORD's anger was kindled against Israel, and he made them wander in the
wilderness forty years, until all the generation, that had done evil in the sight of the LORD,
was consumed v:13). The "generation that had done evil" would include all those above
twenty years old (Num. 14:29). Within this number would have been a son, his father, and
conceivably even a grandfather. Generation as used here includes all those people sharing a
common characteristic.

Another example is the following: And Joseph died, and all his brethren, and all that
generation.... Now there arose up a new king over Egypt, which knew not Joseph (Ex. 1:6-8).
Here, the term generation can refer to all those who knew Joseph personally or by reputation.
Today, we have a generation of people who experienced the holocaust. So long as those
people live, and to the extent that they are able to convey the horror of that experience to their
children and grandchildren, their influence spreads beyond their generation and encompasses
a broader generation. It is only after all those have died who have some personal connection
to those days that we could say that their generation had ended. The point here is that it is
possible for the word *generation* to have a greater range in meaning than that which Till
concluded from his analysis to be the norm.

It is curious that the Exodus writer(s) provided us with the ages of Levi, Kohath, Amram, and
Aaron but no one else. The ages of these men must have been important to the writer. If we
fail to discover the significance of these ages, don't we risk misunderstanding what the writer
has sought to convey to us? Is it coincidence that we can add up the individual ages of
Kohath, Amram, and Aaron, subtract from 430, and get a result that says that Levi lived in
Egypt for 77 years, which is entirely plausible? Is it possible that the writer of Exodus was
providing specific information that would have been readily understood by the historians of
that day? Since the ages of people are rarely given in the scriptures, isn't it altogether possible
that those ages that are given are there to give us an accurate chronology of events dating
from the creation of the universe? The point is that we may err by not understanding all the
information available that would allow us to interpret correctly the contested passages.

The Till analysis contains one particular point for which there are no ready explanations. It is
the apparent equivocation that occurs when the writer shifts from a father-descendant list to a
father-son list, all within a few verses. It is possible for there to have been something within
the Hebrew language to distinguish one situation from the other that is not readily apparent to
present scholars who attempt to translate these verses. For example, by expressing the years in
the life of the person, as the writer did for Levi, Kohath, and Amram, he may have been
indicating that the children born were actually descendants so that the writer is providing a
chronology rather than a true father-son genealogy. Because of the vagaries of translation
from one language to another and the relatively few words in the Hebrew language, there is
room for translators to miss key distinctions.

Volume 1990 - 2002 Issue


Page 641 of 2049
Skeptical Review Edited by Farrell Till
The central issue in this discussion is whether the writer(s) of Exodus is credible. The writer
clearly stated that Israel spent 430 years in Egypt and then provided a genealogy to cover that
period of time. If the writer is credible, then the two scriptures must be accurate. If they are
accurate, then they must be consistent. If they are consistent, then Till's analysis must err by
failing to take into account some information that is not brought out in the available
translations or is otherwise hidden from view.

(Roger Hutchinson, 11904 Lafayette Drive, Silver Spring, MD 20902.)

Murder Most High


by William Sierichs, Jr.
Based on the Bible, god should be at the top of the FBI's 10-Most-Wanted list. Christians
often chide nonbelievers because we supposedly are incapable of morality. Why? We do not
believe in a god and, therefore, cannot partake of the higher morality of the divine.

What exactly does this supremely moral being do, according to the scriptures of the Judeo-
Christian-Muslim deity? Here are a few examples.

In 2 Samuel 6:6 and also 1 Chronicles 13:9, the oxen hauling the ark of God stumbled, and a
man named Uzzah, in order to protect the ark from falling, quite naturally put out his hand to
steady it. Most people would consider this a good deed. So what happened next? "And the
anger of the LORD was kindled against Uzzah; and God smote him there because he put forth
his hand to the ark."

No good deed shall go unpunished, it seems.

In 2 Samuel 12:15, because David committed a crime in order to marry Bathsheba, god killed
their firstborn child--not the offending adulterers! In fact, the poor baby suffered seven days
before dying. It was kind of strange that god should do this, since in Deuteronomy 22:22 god
clearly commanded that adulterous couples should be executed and in Deuteronomy 24:16
said that children should not be put to death for the sins of their fathers. God's ways are
mysterious beyond all insanity.

In Joshua 7:15 and 7:23-25, a man named Achan, during the battle of Jericho, took a pretty
mantle and some silver and gold that god desired. (The scriptures, which teach that god
already has the whole wide world in his hands, does not explain why god desired gold, silver,
and a pretty cloth.) God told the Israelites to find the culprit and burn him. (Kind of strange
the omniscient deity didn't already know whodunit.) When Achan confessed, not only he--the
actual thief--but his entire family ("his sons and his daughters") and even his livestock (oxen,
asses, and sheep) were stoned and burned to death at god's command.

Volume 1990 - 2002 Issue


Page 642 of 2049
Skeptical Review Edited by Farrell Till
God, by the way, had allowed the people of the city of Ai to slaughter the Israelites in a battle,
because Achan had secretly committed this "sin," of which the other Israelites were guiltless.
God never warned the Israelites of their danger; he simply let them be killed to prove to them
he was angry. God's anger ended only after Achan and his family and livestock were
butchered ( Josh. 7:26). *"Oh, give thanks to the LORD, for He is good! For his mercy
endures forever" (1 Chron. 16:34).*

In the *Iliad*, the Greeks also suffered a bloody defeat from the wrath of the gods, but at least
they knew Zeus was unhappy and that it was the pride and anger of Achilles and Agamemnon
that were behind the Trojans' successes. The Olympians didn't seem as mischievous as the
Judeo-Christian-Muslim god.

There is a strange story in 1 Kings 20:35 about a prophet who told another man, "by the word
of the LORD," to strike him (the prophet). Ancient legal texts show that assault and battery
were illegal and were punished by courts, so naturally the man refused to strike the prophet.
At god's command, the prophet then put a curse of death on the man who had refused to hit
him. Sure enough, as soon as the man left the prophet's presence, god sent a lion to kill him.

Invoking god after a group of children mocked his baldness, the prophet Elisha in 2 Kings
2:23-24 called two bears out of the woods to rip apart 42 of the young miscreants. Apparently
god doesn't like people who are "hairline- challenged" to be ridiculed, since he (allegedly)
created baldness along with everything else on the earth, such as the well known and long-
standing irreverence of small children. It's a wonder that god hasn't killed every child ever
born. Maybe some power protects children from divine wrath. (Could it be... Satan? Could
that be the real reason god does not like the devil? He's humanity's guardian angel? We need
the Church Lady to answer this one.)

Of course, in the book of Job, god made a casual bet with Satan that Job, described as a
"blameless and upright" man, would remain so even under severe adversity. Satan was then
allowed to ruthlessly slaughter the innocent servants and family of Job to test his uprightness.
Sometimes, it's kind of hard to tell the good guys from the bad guys in the Bible.

In Numbers 12:1-2, Moses' sister Miriam and brother Aaron criticized Moses for marrying a
Kushite (black) woman, and god punished them by afflicting them with leprosy. That is, he
punished Miriam with leprosy; for some reason, Aaron was let off the hook. Anyway, they--
or at least Miriam--were not punished for being despicable racists, ancient versions of Adolf
Hitler, Orval Faubus, George Wallace, and William Rehnquist, but rather because they dared
to criticize Moses. No mention was made of the worse immorality of bigotry.

One of the stranger scriptural stories is in E xodus 4:24. Moses (allegedly) had been given his
holy marching orders at the burning bush to go back to Egypt and free the Israelites. As he
returned to Egypt... well, let the Bible speak for itself: "At a lodging place on the way the
LORD met him (Moses) and sought to kill him." Moses' wife cut off their son's foreskin with
a flint knife (ouch!) and touched it to Moses' feet, calling him "a bridegroom of blood--
because of the circumcision." And "so he (god) let him (Moses) alone."

Say what?

Volume 1990 - 2002 Issue


Page 643 of 2049
Skeptical Review Edited by Farrell Till
Christians like to claim that theirs is a god of love and mercy, replacing the vengeful divine
patriarch of the Jewish scriptures. Is this true? Well, let's see.

Acts 2:43-46 described how early Christians lived in a communistic fashion, all selling their
possessions and jointly sharing everything in common. But A cts 5:1-10 tells how a couple,
Ananias and Sapphira, sold a piece of their property and gave some, but not all, of the income
to their Christian comrades. Peter confronted them, accused them, in essence, of not being
good Christian communists and thereby offending god. God killed both Ananias and Sapphira
(no surprise, we would say after compiling this list of his homicides).

One wonders what the real story was in this early Christian purge. Were Ananias and
Sapphira a noncommunist opposition to Peter that were condemned in a Christian show trial?
Did Peter have them murdered as Stalin had Trotsky eliminated?

As glasnost and other political reforms allowed the former Soviet Union to acknowledge
communism's crimes and rehabilitate its victims, it's time Christians consider restoring the
reputations of Ananias and Sapphira.

God can be quite ungrateful. According to the scriptures, Judas fulfilled a vital role in
Christian history, for if he had not betrayed Jesus to be crucified, there would have been no
resurrection and no Christianity. Judas's reward for doing his assigned task in history? Act s
1:18 says that he died a painful death for his "wickedness." (Not by suicide, as Matthew
independently claimed.)

In Revelation 14:1-5, we are told that of all the billions of people who have inhabited the
earth, each of whom allegedly was or is the creation of god, only 144,000 males "who have
not defiled themselves with women" are to be saved from the innumerable horrors of
Judgment Day. Revelation 17:8 tells us their names were written in the book of life from the
beginning of the world, and no one else can be saved. Revelation 20:15 tells us that anyone
whose name is not in the book will be thrown into a lake of fire. Why won't god give the rest
of us a chance? It seems that Mother Teresa must burn in hell with everyone else, since she is
not a man.

But Jesus would rather threaten people with horrible torture. For example, "The Son of man
will send his angels, and they will gather out of his kingdom all causes of sin and all
evildoers, and throw them into the furnace of fire; there men will weep and gnash their teeth"
(Matt. 13:41-42). *"How precious is your loving kindness, O God!" ( Ps. 36:7).*

Skeptics today still question those unrecorded "mighty works" of Jesus (John 20:30). If god
really cared about people in the scientific era, you'd think he would send some solid,
verifiable proof of all these miraculous claims that would pass modern scientific scrutiny.
Dreams and revelations are hard to verify with Geiger counters, electron microscopes, X-rays,
or chemical analysis. Scientists are open-minded. Show them that a miracle is a miracle is a
miracle is a miracle, and a lot of agnostics and atheists would believe. Oral Roberts and Pat
Robertson are not particularly convincing miracle workers.

Volume 1990 - 2002 Issue


Page 644 of 2049
Skeptical Review Edited by Farrell Till
If Paul was telling the truth in 1 Timothy 1:19-20, then acting under god's power, he
condemned two men--Hymenaeus and Alexander--to Satan for blasphemy. Were they
removed from the book of life, leaving only 143,998, or did others take their places to give us
the full 144,000?

The Bible contains many other divinely backed atrocities. The Israelites allegedly massacred
the entire populations of Canaanite cities with god's blessings. Israelites who worshiped other
gods were brutally murdered, rather than being mercifully shown the truth. The firstborn
children of the Egyptians were slaughtered prior to the exodus, even though most Egyptians
would have been ignorant of the clash between Moses and Pharaoh. Under that monarchy,
few Egyptians could have changed Pharaoh's will anyway, especially after god had, according
to the scriptures, "hardened Pharaoh's heart."

God can be gruesomely comprehensive in his brutality, as the terms of his covenant with
Israel make clear in Leviticus 26:14-39 and Deuteronomy 28:15-68, which vowed to smite the
Israelites with famine, invasions, plagues, and numerous other horrors, including cannibalism.
Even the Nazis were not as vicious as the vengeance promised to Israel if an iota of its
covenant was violated. The penalty far exceeded the crime.

We could even ask why god, in Mat thew 2, made such a fuss over the birth of Jesus, with a
spectacular star of Bethlehem and other signs guaranteed to tip off the paranoid King Herod,
who then massacred the children of Bethlehem in a vain attempt to stop the coming of the
Messiah. If god is all-knowing, surely he was aware of Herod's homicidal predilections and
could have made Jesus's appearance less threatening to the king, thereby saving all of those
poor babies.

Fortunately for Herod's reputation, neither Mark, nor Luke, nor John, nor history (period)
knew anything about the slaughter of the innocents, so actually Herod was probably guiltless
of this particular charge.

If the Bible was written by men, as skeptics suspect, then what kind of mentality would
conceive of these horrors, much less worship a god who allegedly commits them? And how
dare Christians demand that nonbelievers bow down to such murderous ideas!

If these really are the deeds of the Judeo-Christian-Muslim god, then who is this deity, Adolf
Hitler?

(William Sierichs, Jr., 316 Apartment Court Drive, Apt. 14, Baton Rouge, LA 70807.)

Can the Bible (or Any) God Support an


Absolute Morality?

Volume 1990 - 2002 Issue


Page 645 of 2049
Skeptical Review Edited by Farrell Till
by Tim Gorski, M. D.
The world is in moral decay, say the theists, because of "moral relativism." Only a divine
power makes possible an absolute standard of right and wrong, they say. And yet, entirely
aside from the evil that men (and women) do, there is much that is terrible and unjust in the
world, so that if there be a God, we realize, He can not be both all-good and all-powerful.
Because if He were, He would put an end to such things.

But I'm afraid the situation is much, much worse even than that. Four hundred years before
Jesus Christ is supposed to have been born, Socrates asked "whether the pious or holy is
beloved by the gods because it is holy, or holy because it is beloved of the gods." Socrates
also observed that the gods--plural-- argued and disagreed about right and wrong as much as
human beings. He got around this by supposing that that which all the gods approved was the
good, and that which they all objected to was the evil, and that all else was neither good nor
evil. He might just as well have considered the problem of a single god-- like that of the
Christian Bible--who's inconsistent about what is beloved. But, as we know only too well,
there simply is no honest way out of contradictions like that.

So let's just consider a strictly theoretical situation. Just for the sake of argument, let's suppose
there's a God, and that He, She, or It is the absolute standard of morality. Is right and wrong
then simply no more than this God's say-so? Or is what is right loved by this God and what is
wrong hated by this God because of what right and wrong are in themselves?

In the first instance, if good and evil are no more than the product of the will of a divine
power, and if that will is truly free, then such a God could, with a thought, cause what we
consider to be the most repugnant and heinous criminal act to become the highest virtue. Now
the further question would arise, of course, as to whether if this happened we would know it.
Why? Because of "the moral law within us," as the philosopher Immanuel Kant put it, or "the
work of the law written in our hearts," as "Saint Paul" acknowledged ( Romans 2: 15). If
morality is the say-so of a God, then presumably, like the gravitational effects of a massive
body, any change in His (or Her or Its) will would cause our own consciences to be
instantaneously altered. I've never heard of this happening, though.

At any rate, if there is a God, and if this God's will determines what is right and wrong, then
this supposed God's being all-good is no more than His (or Her or Its) being all-powerful. Is
that an absolute morality? I don't think so. Rather, it's a morality that's completely relative to
His (or Her or Its) desire. In a word--well, three actually--it's *might makes right*. It's another
version of the law of the jungle. How's that for an admirable system of morality?

The only uncertainty remaining is whether it's more or less pathetic than the alternative
situation of a God who is Himself (or Herself or Itself) subject to a logically anterior or prior
standard of morality. That would be the case in the second instance of things that are good
being beloved by God because they're good, because, of course, that puts God on the same
level with human beings. It makes Him (or Her or It) irrelevant.

Well, we know He--or She or It--is irrelevant. That's why we're revolted by such Biblical
stories as that of Yahweh asking Abraham to sacrifice his son Isaac as a burnt offering--as if

Volume 1990 - 2002 Issue


Page 646 of 2049
Skeptical Review Edited by Farrell Till
an all-good God could be pleased by a criminal act. Did Abraham really think he was
flattering Yahweh to agree to do such a thing? It's curious that this same God is also supposed
to have issued orders of mass extermination, orders that "The Good Book" tells us were
actually carried out with less hesitation than Abraham had in preparing to kill his own son.

Well, so much for theistic "absolute morality." It's anything but.

(Dr. Gorski practices medicine in Arlington, Texas, and is president of the Dallas/Fort Worth
Council Against Health Fraud. He is a founding member and director of the North Texas
Church of Freethought and is the editor/publisher of The Freethought Exchange and The
Freethought Observer, P. O. Box 202447, Arlington, TX 76006.)

Not Much to Do
by Farrell Till
Roger Hutchinson has left me with very little to do in responding to his second attempt to
explain why Aaron's genealogy in Exodus 6:16-20 doesn't conflict with the biblical passages
that say the Israelites spent 430 years in Egypt (Gen. 15:13; Ex. 12:40-41; Acts 7:6). He said
that I "did [my] homework well, conducted an excellent analysis, and reached what [he
believes] were logical conclusions." He further said that he agrees "that there are problems
inherent in any effort to reconcile the genealogy in Exod us 6 with the 430 years in Exo dus
12." So why doesn't he just admit that in this case we have an obvious example of biblical
discrepancy?

In our correspondence, I have never put this question directly to him, but I suspect I know
why he cannot bring himself to make such an admission. He is a Bible inerrantist, and as a
rule, Bible inerrantists will go to just about any extremes to preserve their precious inerrancy
doctrine. I remind the readers that in his first article, Mr. Hutchinson actually said that if I
found one scripture that could not be reconciled with the scenario he was proposing to explain
the discrepancy, he would have "to reject this particular scenario and look for another
explanation" (Autumn 1995, p. 2). My response to this was to ask why inerrantists feel
compelled to accept the Bible with such blind loyalty. "When a `scenario' that they devise
cannot be reconciled with another scripture," I asked, "why don't they consider the possibility
that reconciliation isn't possible for the simple reason that discrepancies and inconsistencies
may actually exist in the biblical text?" They don't do this, of course, because their allegiance
to a discredited idea has too strong a grip on them. Once they admit that errors may be in the
Bible, they have lost their base of authority. They can no longer say, "You must do what the
Bible says, because it is God's word," for they realize that a book with errors in it just cannot
be the "word" of an omniscient, omnipotent deity. Hence, their desire to have a base of
authority as an excuse to impose their views on others forces them to resort to all sorts of
mental contortions to try to prove that errors aren't really errors.

Volume 1990 - 2002 Issue


Page 647 of 2049
Skeptical Review Edited by Farrell Till
We have seen this factor at work in Mr. Hutchinson's determination to show that the Exodus
writer(s) made no mistake in the genealogy of Aaron (6:16-20) and a later claim that the
Israelites had sojourned in Egypt 430 years (12:40). As already noted, he admits that my case
was strong, that I did my homework well, that I conducted an excellent analysis, and that I
reached what he believes were logical conclusions, but rather than admit that there just might
be a real discrepancy between these two passages, he went back to the drawing board and
looked for "another explanation."

The best I could tell, he seemed to be arguing that even though we may not understand
everything about this problem, "there seems to be no reason to think that the information
recorded in Exod us 6 would be wrong." Well, why isn't there reason to believe that the
information is wrong? If Kohath was born before the Israelites went into Egypt (Gen. 46:11)
and if the combined lifetimes of Kohath and Amram totaled only 270 years (6:18, 20), and if
Aaron was only 83 at the time of the exodus (7 :7), how can anyone possibly find 430 years in
these figures, especially when common sense tells us that Amram's life surely overlapped
Kohath's and Aaron's overlapped Amram's?

Mr. Hutchinson argued that "if Mr. Till can look at these scriptures and see a problem, how
much more would people in that day have seen the same problem?" I can't see this as a
legitimate way out of the corner he has painted himself into, because he surely knows that
textual criticism is a fairly recent development, and certainly it didn't exist in prescientific
Israel to the degree that it does now. In fact, the literature of ancient times indicates that the
prescientific minds of that era were apparently not as concerned with textual inconsistencies
and discrepancies as we are today. I recall how that in my days as a believer in the inerrancy
of "God's word," inconsistencies somehow went unnoticed when I was reading the Bible.
Surely, this was because I was reading the Bible with the assumption that it was the word of
God, and so I read it uncritically and just didn't notice the obvious. How much more likely
would readers have used this approach in prescientific, superstitious times? Furthermore, Mr.
Hutchinson seems to be arguing that if past generations did not notice a particular discrepancy
in the Bible, then surely it isn't a discrepancy, and I can't see any logical basis for such a
conclusion as this.

Most of Mr. Hutchinson's attempt at argumentation this time centered on the meaning of the
word *generation*, but this is really an irrelevant issue. Whether four generations or ten
generations of Israelites resided in Egypt during their sojourn is beside the point. Exodus
12:40 says that the length of the sojourn was 430 years, and even Mr. Hutchinson said he
agrees that this is what the Bible "clearly states." The problem, then, is that he must explain
how that the lifespans of Kohath (who was born *before* the Israelite descent into Egypt and
lived only 133 years) and Amram (who lived only 137 years) and Aaron's age (83 years at the
time of the exodus) could possibly allow for a sojourn of 450 years. The figures simply do not
add up, and Mr. Hutchinson has even admitted "that there are problems inherent in any effort
to reconcile the genealogy in Exod us 6 with the 430 years in Exodus 12." He just won't admit
that the problems are so great that they cannot be rationally resolved.

Volume 1990 - 2002 Issue


Page 648 of 2049
Skeptical Review Edited by Farrell Till

American Society for Religious Concern


Farrell Till
We get a lot of unusual mail at The Skeptical Review, but none has been more surprising to us
than a letter that came from a group with the same name as the title of this article. Prior to this
letter, we had received a subscription request, and we sent it thinking that this was a group
concerned about religious attempts to influence political decisions. magine our surprise, then,
when the following letter arrived some weeks after the subscription to ASRC had started:
" We realize that this may come rather as a shock to you, but this fundamentalist
Christian organization would like to offer its "praise" to both you and your
publication. Being at odds with one another is quite understandable. We can also
understand anger, sarcasm and the like.
" For several months now, our Society has scouted numerous freethinker publications.
The one thing woefully lacking in every one of them, except The Freethought
Exchange and The Skeptical Review, is caring. As much as is humanly possible for an
atheist, your caring attitude shines through in abundance. Each and every one of our
staff members "salute" you.
" For the past few years (under a different name) our Society has published a
newsletter for Christians. After a survey, and over 3300 responses in favor of it, we
have recently changed our format to deal exclusively with freethinkers. Our first issue,
The Stupid Christian Chronicle, is enclosed for your enjoyment. We are certain that
there are several articles (if not all) that you will totally disagree with. Were that not
the case, you would not be a "good" freethinker, and we would not be "good"
Christians. We do, however, hope to extend to you the same kind of caring that we
witness in the Review. I draw your attention to article #3 and to the back cover. There
is no reason to be skeptical; it is there in black and white; we are very sincere; it is our
pleasure to give you prominent space and we will do our part to see that our Christian
brothers and sisters take advantage of your superb publication.
" We look forward to a caring and exciting business relationship with you. Our articles
and differing philosophies may offend. It is our hope and prayer that our direct
relationship with you will never be offensive."

The letter was signed by H. Jate, Editor. I'm not really sure, but I seem to remember from the
letter requesting a subscription (which I have thrown away) that the editor would be Ms. Jate.
With a disclaimer of agreement with their contents, the back cover referred to in Ms. Jate's
letter "highly recommends" The Freethought Exchange and The Skeptical Review to ASRC
subscribers. Article #3, entitled "Freethinker Papers," by Joy Murdock, states that various
freethinker papers have been examined but that "with the exception of The Freethought
Exchange and The Skeptical Review, the "love" just wasn't there. Ms. Murdock then went on
to say this:

"Our Lord Jesus is certainly not afraid of "facts"; we, as born-again Christians, are
certainly not afraid of "facts" either. As a matter of fact, this very chronicle solicits
your "facts." Why? Because they are filled with love. The Christian "knows" what
love is all about; it is quite evident, the "freethinker" has no conception whatsoever.

Volume 1990 - 2002 Issue


Page 649 of 2049
Skeptical Review Edited by Farrell Till
" Take, for example, "equal time." Read any paper that I've described for you. I
understand that an "atheist" paper is going to give more time to atheistic words; a
"Christian" paper does the same. But, see what's happening: whenever a Christian
responds to an opposing publication, who gets the article long before this issue of TSR
hits the mail in November. As for the prediction that atheists will "fall by the wayside
every time" when confronted with "a real opportunity for religious and ethical
exposure," I'm going to predict that the dropouts will be on the Christian side.
" However, I do thank the editors of this publication for recommending The Skeptical
Review to their readers."

A Response to Marion Fox


by E. E. Brennaman
My copy of the Summer 1995 issue of The Skeptical Review arrived a week or so ago. I have
enjoyed every issue I have received because of the excellent information each one contains. I
also like to read those few attempted rebuttals by inerrantists who challenge the errantist
view.

Mr. Marion R. Fox's "Answers to a Claim of Errancy of the Scriptures" was a puerile attempt
to show that skeptics, atheists, and other nontheists cannot reason well, and to save the
inerrancy doctrine. He never had a solution to the problem Farrell Till presented to any
inerrantist who might want to explain why there was not inconsistency in certain of Perez's
descendants holding offices that they should not have been qualified to hold, if Perez was
indeed considered a bastard.

After reading all of the pertinent biblical passages, I wondered why Mr. Fox did not answer
Till directly instead of asserting the ridiculous arguments he ascribed to atheists and
agnostics. He could have pointed out that the persons involved in the Perez case all lived
before Moses received the Law from God. It was a time when the various Israelitish clans
used situation ethics. For instance, Cain did not commit incest when he took one of his sisters
or nieces as a wife, because they were the only females available to him. If the race of Adam
was to multiply, it was necessary for Adam's sons to marry close relatives.

Judah only suggested to Onan that he "go in to your brother's wife, and perform your duty as a
brother-in-law to her, and raise up offspring for your brother" (Gen. 38:8). Notice that it was
his duty to raise up offspring for his brother, and it seems he could have refused as easily as
Judah refused to force Shelah, the younger son to assume the "duty." There was no law to
compel Onan to perform a duty; it apparently was a custom of the tribe of Judah or perhaps of
all the Israelites. Er, Judah's oldest son, "was evil in the sight of the Lord, so the Lord took his
life" (v:7). There is no hint as to how Er irritated the all-loving God, and after Onan died,
Judah asked Tamar to remain a widow in her father's house until his younger son Shelah had
grown up, but when Shelah reached marriageable age, Tamar was not given to him to perform
the "duty" of a brother-in-law.

Volume 1990 - 2002 Issue


Page 650 of 2049
Skeptical Review Edited by Farrell Till
The law not having been promulgated, there was no rite for Shelah to refuse to marry Tamar,
and no rite for her to loose his shoe and spit in his face, the Law promulgated by the all-loving
God (Dt. 25:9).

The story of Ruth and Boaz, also supposedly before the law of Moses was formalized,
indicates that there still was no regulation concerning the childless wife of a dead husband.
Boaz was not a brother or the nearest relative of Elimelech, Mahlon, and Chilion; an unnamed
relative was. There was nothing written whether Orpah's husband died childless. Apparently,
it was not a matter of importance or the author would have included this information. Perhaps
the purpose of the story was to give David a genealogy. There still was no rite to show
displeasure toward a brother reluctant to perform the "duty" of a brother-in-law.

Mr. Fox did not deal with Tamar's acting the whore. She dressed as a temple prostitute, whom
fundamentalists disbelieve Israelites had. What is not mentioned specifically in the Bible
either is not true or is a possibility to avert an error; it is not positive evidence of anything.
Such is the way inerrantists argue.

It seems to me that what had been a tribal custom was eventually enacted into law with
regulations governing the levirate marriage, or, in the event of a refusal by the brother of the
deceased, a rite to embarrass him. Marriage was required, and there was no provision for
another near relative of the deceased to replace the brother.

Inerrantists do not recognize gaps in genealogies or missing information concerning events to


be errors in the Bible. Someone needs to explain why these lapses do not constitute errors.

Although moderns may consider Perez a bastard, the ancients apparently did not. If Perez had
been looked upon as a pariah, his descendants would have become nonentities, and no one
would know the names of his progeny. What member of the congregation of Israel would
have married his daughter to Perez or any of his progeny? Perez's only choice for a mate
would have been the daughter of another pariah or some foreigner.

Although I believe that Perez was not considered to be a bastard by his contemporaries, no
one should even begin to imagine that I agree that the Bible has no errors, because I don't
have to read very far in the Bible to be confronted by them.

It is unfortunate that inerrantists refuse to defend their doctrine with evidence, and only
attempt to make critics inept logicians. That tactic is dishonest and useless.

(E. E. Brennaman, 1601 Airline Road, Apt. 62, Corpus Christi, TX 78413-4517.)

EDITOR'S NOTE: Ernie Brennaman served as my moderator for both of the debates that I
had at the Portland, Texas, Church of Christ, so he has not only read Mr. Fox's article, but he
also was present at the Moffit-Till Debate and heard Jerry Moffitt say several times that Mr.
Fox had formulated the probability arguments that were being presented in the debate.
Readers should keep this in mind as they read Krishna Kunchithapadam's letter on pages 13-
14 of this issue.)

Volume 1990 - 2002 Issue


Page 651 of 2049
Skeptical Review Edited by Farrell Till

A Message From Mr. Fox

Would it be possible for you to inform your readers that you received my letter in January?
Your articles (Vol. 6 # 1, Winter 1995, and Vol. 6 # 2, Spring 1995) leave the false impression
with your readers that I did not reply "in the next few months." Would it not be fair to inform
your readers when you received my article?

I am enclosing $2 for the cost of the rental on the Hovind-Till Debate. Incidentally, there was
no looting in Oklahoma City after the bomb, but the United States is not a Christian nation.
Some nations kill looters! I will probably send you a couple of high density disks for ASCII
files of your publication "in the next few months."

I have enclosed an article on abortion for you to publish if you think it worthy of
consideration.

(Marion R. Fox, 4004 Twisted Trail Road SE, Oklahoma City, OK 73150-1910.)

EDITOR'S NOTE: Mr. Fox did indeed send his article in January, but when we received it,
the spring (mid-March) edition had already been sent to the printer. We therefore had to wait
until the summer edition to run it. I might also add that the article was received almost eight
months after the debate in Portland, Texas, where Mr. Fox announced to the audience that he
would submit the article to TSR within "a few months." I hardly consider eight months to be
just a few months, and even now I wonder if Mr. Fox would have written the article if we had
not goaded him in the two issues he referred to in his letter. At any rate, we did publish his
article in less time than it took him to submit it, and we have offered and still offer him space
to respond to anything that I have published in response to his articles. If he is as fair as he
expected us to be, I think he will have to admit that no Church-of-Christ publication would
give me the forum that I have given to him and his inerrantist cohorts.

We were certainly glad that Mr. Fox has asked to see the video tape of The Hovind-Till
Debate. We have sent this to him along with complimentary disks containing ASCII copies of
all articles that have been published in TSR. Now he won't have to wait until "the next few
months" to receive them.

The biblical inerrancy doctrine is the focus of The Skeptical Review, and for this reason we
will not be publishing Mr. Fox's article on abortion. I am not discriminating against Mr. Fox
anymore than I have against skeptics whose articles I have rejected because they did not
address the inerrancy issue. I once wrote an article on abortion too, but I didn't publish it in
TSR. I sent it to Truth Seeker, because it was inconsistent with our editorial purpose. ("God Is
Prolife?" was published in Volume 120, Number 5, 1993, pp. 15-17, of Truth Seeker.)
However, to head off any insinuations from Mr. Fox that I am evading an issue I am afraid to
confront, I have written a response to Mr. Fox's article and will send a copy of both articles
(his and mine) to anyone who requests them. (Please include an SASE.) I think that this is as
fair as he can expect me to be.

Volume 1990 - 2002 Issue


Page 652 of 2049
Skeptical Review Edited by Farrell Till
For the benefit of those who may not have understood why Mr. Fox said that the United
States is not a Christian nation, I should explain that he believes the only Christians are in the
Church of Christ, which he naively believes is *the* church of Christ. The members of all
other churches are apostates in his opinion, and so he doesn't consider them Christians. Since
the Church of Christ, according to Information Please Almanac, has only 1,770,000 members,
which is less than 1% of the population, Mr. Fox cannot consider the United States a Christian
nation.

All of us certainly sympathize with the people of Oklahoma City, and they obviously acted
heroically in the face of a great tragedy. As horrible as the Oklahoma City disaster was, the
bomb destruction was limited to a relatively small area of the city, so civil order would have
been much easier to maintain than in a disaster like the one in Kobe, Japan, which leveled an
entire city that is almost three times as populous.

From The Mailbag


Mr. [Hershel] Davis, speaking for fanatical Bible-believers, boasts that if TSR (and such) is
the best that we skeptics, atheists, etc. can do to shatter their belief in Biblical inerrancy, then
we ought to take up another vocation. However, that is not a very fair test. Fanatical believers
are disconnected from the reasoning process altogether, and no amount of evidence, piled
however high, would be enough to convince them. Consequently, the seasoned skeptic does
not expect to shake their faith, be it in an inerrant Koran, an inerrant Bible, flying saucers, or a
flat earth. We write for the mind that is still open. Once a mind has snapped shut and rusted
solid, the head being buried in concrete as it were, then there is not much point in reasoning
with it. The true fanatic (defined as a person with extreme and uncritical enthusiasm or zeal,
as in religion, politics, etc.) cannot be reasoned with by definition. No doubt, Mr. Davis will
continue reading TSR, and he will go safely to his grave with his beliefs intact. That is his
privilege, his rightful prerogative, and his error.

But there is always hope. Many of us skeptics were once in a similar condition.

(Dave Matson, P. O. Box 61274, Pasadena, CA 91116.)

EDITOR'S NOTE: Long-term subscribers will recognize Dave Matson as a writer of several
articles that TSR has published. As noted in the Autumn issue, he has recently published a
102-page analysis of the "scientific" arguments used by creationist Kent Hovind to prove his
young-earth theory. The book is thorough in its examination and refutation of Hovind's
arguments. It can be ordered at the above address for $14.50. If creationism is an issue in your
local or area school districts, this would be a good reference to have in your personal library.

Volume 1990 - 2002 Issue


Page 653 of 2049
Skeptical Review Edited by Farrell Till
I really look forward to your Skeptical Review each quarter and am excited to receive all of
your past issues on disk. Your work is commendable. After over 50 years of religious
entrapment, all it took for me to get shocked off my biblical inerrancy perch was two days
into Thomas Paine's Age of Reason four years ago. Since then, I've read all issues of Dennis
McKinsey's Biblical Errancy, the past 2 years of your Skeptical Review, Steve Allen's Steve
Allen on the Bible, Religion, and Morality, plus a couple of dozen more books on atheism,
deism, agnosticism, etc.

I became involved in Mormonism at age 22 in 1961 and spent 22 years involved as an Elder
and finally a High Priest. Then, due to some B.A.C. friends of mine, I "saw the light," left
Mormonism, became an active anti-Mormon, B.A.C., and spent the next nine years on a
crusade to win Mormons from Mormonism to the real "true faith," i.e., born- again
Christianity. What a trip it has been for my family and me! We all together (my wife and I
and our nine children) came out of Mormonism in 1982; then in 1991 came out of religious
nonsense entirely. I consider myself a free thinker-agnostic, who has finally been completely
rid of religion/superstition/ ignorance.

Thanks to you and the others I mentioned, I can really enjoy my newfound freedom. Please let
me know if I can be of any service to you in any way.

(Walt Noble, 436 East Tenth Street, Mesa, AZ 85203-4728.)

EDITOR'S NOTE: Needless to say, this is the kind of letter we like to receive. It suggests that
rationality is gaining a stronger foothold in the land. A paperback edition of Paine's book Age
of Reason can be purchased for only $5 from Stephen VanEck, Route One, Box 62, Rushville,
IN 18839-9702. A free sample copy of Biblical Errancy can be obtained by writing to Dennis
McKinsey, 3158 Sherwood Park Drive, Springfield, OH 45505 or e-mail
klo_mckinsey@k12.mec.ohio

I have just received my second issue of The Skeptical Review. I'm impressed with the fine
editing of your publication, the only one I read from cover to cover. The incisive
commentaries and intelligent articles are a delight. Keep up the good work.

I've evolved from conservative Catholic to liberal Catholic to agnostic, and it's been an
interesting journey. I've recently read The Bible Handbook, published by the American
Atheist Press, which places biblical texts side by side so that the contradictions are undeniable
to someone with common sense. What an eye opener! The real miracle of Judeo-Christianity
is that the bible, with its contradictions and accounts of atrocities attributed to God, gets as
much unquestioning respect as it does from some believers.

I would love to share your publication with religious friends and family members, but I don't
think they're ready. My experience with Christians tells me that some are so emotionally
fragile that for them to question their faith would be to undermine it, to undermine it would be
to lose it, and to lose it would be to turn their worlds upside down. Fear of death and
damnation aren't the only reasons some close their minds. There are social, economic, and

Volume 1990 - 2002 Issue


Page 654 of 2049
Skeptical Review Edited by Farrell Till
cultural dimensions to religion that would be imperiled if one adopted a skeptical mindset.
Facing facts after a lifetime of indoctrination requires integrity and involves the same courage
it takes for a child to relinquish a beloved security blanket, but the freedom is worth it. Your
publication shows me I'm not alone, and that helps.

I'm enclosing a check to cover the cost of your booklet Prophecies: Imaginary and
Unfulfilled, as well as back issues of TSR 1993 and 1994. Thanks for giving me a forum to
express my views.

(Irene Jones, 16585 Barryknoll Way, Granger, IN 46530.)

EDITOR'S NOTE: "The freedom is worth it." I know that this is true from my own personal
experience of finding the courage to reject religious nonsense and face reality, and I hear the
same message over and over from those like Ms. Jones who take the time to write about their
journeys from superstition to rationality.

The Bible Handbook can be ordered from H. H. Waldo, Bookseller, P. O. Box 350, Rockton,
IL 61072-0350. The telephone number is (800) 669-2536 (66WALDO).

It seems that religionists never tire of showing their ignorance of logic. A few years back you
printed my letter in which I pointed out that in his debate with you Buster Dobbs had
incorrectly referred to a hypothetical question as a complex question. Since then, in an
exchange stemming from your printing my article "Why Did Matthew Need Dead Babies?"
Jerry McDonald attempted to deflect my charge that he had committed the fallacy of converse
accident by confusing its definition with that of accident. Now in your Summer 1995 issue, I
find Marion R. Fox completely missing the mark in his attempt to define begging the
question.

In the second column of his article (p. 2), Fox seems to be defining begging the question as
failure to adequately support the minor premise in one's syllogism. At first, I thought I might
be misreading him, but on the next page he makes it clear when he writes, "In order to prove
the conclusion, Farrell must prove his minor premise. Where is his proof? He assumed it
without proof! He begged the question."

But that is not at all what the term means. In the words of Copi in his introductory text: "If
one assumes as a premise for his argument the very conclusion he intends to prove, the fallacy
committed is that of pettitio principi, or begging the question." This has nothing to do with the
acceptance or level of development of a premise, only with whether one of the premises
assumes the conclusion.

Fox did at least correctly define denying the antecedent. He took some pains to do it, as
you've pointed out, with the rather strange sample premise of the Biblical errantist arguing
that what the Bible says is true. But there he drops the ball. Although accusing you of
committing this fallacy, he in no way ties it in to your argument. This particular fallacy is an

Volume 1990 - 2002 Issue


Page 655 of 2049
Skeptical Review Edited by Farrell Till
invalid syllogistic form. Yet his two reductions to syllogistic form of what he sees as your
positions are simple AAA syllogisms, as is his own syllogism. These are valid.

For all his effort in carefully defining this fallacy, he nonetheless demonstrates that he really
doesn't understand it after all. What's more, he shows that he cannot distinguish between the
validy of an argument (whether it is of a valid form) and its soundness (whether its premises
and conclusion are correct). It is unfortunate that, signing his school's name to his article, he
could not present us with a bit more in the way of academic excellence.

(Earle C. Beach, P. O. Box 16519, Austin, TX 78761.)

EDITOR'S NOTE: Several readers have written or called to comment on various logical
problems in Mr. Fox's article. Two other letters on the subject appear below.

Marion Fox erred in his discussion of supposed logical errors by inerrancy skeptics. He
claimed that skeptics are guilty of the fallacies of denying the antecedent and begging the
question. Actually the logic is as follows: If the Bible is true, then X is true. The logical
equivalent of this is its contrapositive: If X is not true, the Bible is not true (if p then q; not q,
therefore not p).

This reasoning is how all rational people test ideas against reality and how scientists know
when to modify their hypotheses. Fundamentalists never modify their "theory," since they
never test it against reality. They in fact beg the question as Mr. Fox accuses skeptics of
doing.

We have hundreds of statements in the Bible to choose from, including facts of astronomy,
geology, biology, mathematics, logic, ethics, history, anthropology, and other disciplines, in
order to test against reality and decide whether the Bible is literally true. The Bible may be a
literary genre, or an attempt by a superstitious desert tribe to find meaning in their existence,
or any number of things, but the Bible cannot be literally true.

(Ed Unger, 1380 Toonigh Road, Canton, GA 30115.)

Marion R. Fox's "Answers to a Claim of Errancy of the Scriptures" (TSR, Summer 1995)
seems to presume that all arguments are deductive arguments. This presumption leads him to
press inductive arguments into deductive straw men, then complain that they are invalid. For
instance, Fox claims that an argument for the nonexistence of the Hittite nation from the lack
of evidence of such a nation is an example of denying the antecedent. It is far more likely that,
rather than engaging in the invalid deductive reasoning Fox gives, adherents of this view
reasoned as follows:

Most civilizations leave evidence of their existence.


There is no trace of the Hittite nation.

Volume 1990 - 2002 Issue


Page 656 of 2049
Skeptical Review Edited by Farrell Till
Therefore, it is probable that there was no such nation.

This is an inductive argument of the form known as "statistical syllogism." It is not


deductively valid, but rather is inductively cogent and subject to counterexample as later
evidence comes along. The revisability of the conclusions of inductive arguments is why such
reasoning is known in the artificial intelligence community as "nonmonotonic reasoning." A
knowledge base of information is not always increasing (monotonic), but can contract as
conclusions have to be discarded. Rules of reasoning that are nonmonotonic in character more
accurately represent human knowledge and reasoning than does the oversimplification of
deductive logic.

For more information, I recommend Gilbert Harman's Change in View (1986, MIT
Press/Bradford Books), John Pollock's Nomic Probability and the Foundations of Induction
(1990, Oxford University Press), and Howard Kahane's Logic and Contemporary Rhetoric:
The Use of Reason in Everyday Life(6th edition, 1992, Wadsworth). The Harman and Kahane
books require no prerequisite knowledge in logic or philosophy; the Pollock book is fairly
technical.

(Jim Lippard, 2020 West Glendale Avenue, Apt. 20, Phoenix, Arizona 85021, e-mail,
mailto:lippard@skeptic.com

EDITOR'S NOTE: Jim Lippard is well known in the freethought movement. When he was
still a philosophy student at the University of Arizona, we published a letter from him on the
subject of Lindell Mitchell's logic (Spring 1994, pp. 13-14). In the 1989-1990 issues of
Creation/Evolution, he conducted a written debate with creationist Walter Brown, who was a
star spokesman on Sun Pictures' now infamous "documentary" on The Incredible Discovery of
Noah's Ark. Jim is now writing investigative articles for Skeptic magazine. Two that have
already been published are "Sun Goes Down in Flames: The Jammal Ark Hoax" (vol. 2, no. 3,
1994) and "Scientology v. the Internet" (vol. 3, no. 3 , 1995, co-authored with Jeff Jacobsen).
He also contributed a chapter to Ed Babinski's Leaving the Fold (Buffalo, NY: Prometheus
Books, 1995). I have learned to value his opinion.

I am an atheist, and I must admit that Moffitt was better prepared for this debate than you
were. Yes, he is wrong, but he is a fast talker. Also, I think Hawking has done some fine
work, even if he seems to imply a cause/God. (Greg Lemery, P. O. Box 168, Lebanon, CT
06249-0168.)

EDITOR'S NOTE: I have often said that Jerry Moffitt is the best debating opponent I have
faced. He is indeed a fast talker, and so he gets lots of information into the record. Also, he
made an honest effort to debate as opposed to Buster Dobbs, who spent much of the time
trying to run and hide, and Norman Geisler, who ignored my arguments and read previously
prepared speeches.

On the other hand, I think that an objective examination of Moffitt's performance will show
that he didn't even try to defend his proposition. The propositions we signed obligated him to

Volume 1990 - 2002 Issue


Page 657 of 2049
Skeptical Review Edited by Farrell Till
prove that Yahweh, the god of the ancient Hebrews, exists, but Moffitt spent most of his time
arguing that design in nature proves that a creator exists. The record will show that when I
pressed him to show how that this would in any way prove that the creator was Yahweh of the
Hebrews, he refused to address the issue beyond making a quick, unsupported assertion that
prophecy fulfillment in the Bible proves that the creator was Yahweh. In my opinion, this was
a critical failure on his part that makes it very difficult to consider him the "winner" no matter
how glib he may be as a public speaker. Much of my mail agrees with me on this point.

At any rate, I appreciate Mr. Lemery's evalutation of the debate. The next letter expresses a
different opinion.

I received the Summer issue of TSR this afternoon, and had a lot of fun reading it. I am
impressed by your patience in dealing with completely stupid arguments from inerrantists. If
only these people would give up either their claim for inerrancy and/or literalism, the debate
about the existence of god would become all the more interesting and intellectually
rewarding.

I mailed back the Moffitt-Till debate tapes earlier this week, and I have to admit that I had a
very hard time sitting through Moffitt's seemingly interminable rants about the "wonderful
design of the human body." You may have seen a Woody Allen movie where Woody and his
girl-friend are at a theater waiting to buy tickets. A man standing in front of them launches
into a rant about the theories of Marshall McLuhan. Woody then stops this guy and tells him
that he is wrong, and that he (Woody) can even produce McLuhan to refute the nonsense.
And, incredibly enough, McLuhan makes a cameo and totally trashes the wise-guy's
misinterpretations. Woody then makes a classic ad-lib towards the screen saying, "Don't you
wish real life were like this?" And I felt exactly like that listening to Moffitt quote Hawking
and Sagan ---totally misinterpreting their statements and quoting them out of context or
quoting them selectively to his advantage. How wonderful it would have been to have
Hawking or Sagan come on stage and say "Jerry, stop crapping!" (Pardon my language, but I
get very upset thinking about the debate).

I also happened to be reading John Allen Paulos' book on Innumeracy at about the time I was
watching the debate tapes and found that Jerry (and his minions, Fox et al) are quite
innumerate and lacking in the knowledge of basic physics (and to watch Jerry Moffitt
standing there smiling and lustily expounding on one physical theory after another about God
and YHWH).

For example, there was this large board with those huge numbers on it. The number of atoms
in the universe is 10^100 and the number of electrons in the universe is 10^102 and so on.
Now, I don't know if the number of atoms in the universe is 10^100 or not (likewise for the
number of electrons), but I certainly know that the two numbers cannot be simultaneously
correct. The best astrophysical calculations we have today have determined that the universe
is mostly hydrogen (about 75%) and most of the remaining matter is helium (~25%). The
heavier atoms we living beings are made of and which compose the planets are so minuscule
in number that they just do not figure into the total. Now, hydrogen atoms have 1 electron and

Volume 1990 - 2002 Issue


Page 658 of 2049
Skeptical Review Edited by Farrell Till
helium atoms have 2 electrons. So the total number of electrons per atom in the universe
would be 0.75 + 2*0.25, or about 1.25. The figures that Moffitt quoted make this ratio 100:1,
which by no stretch of the imagination is close to 1.25. Maybe Moffitt has his own fantastic
theory about free protons and free electrons floating around the universe in a place where time
flows forwards and backwards.

Which brings me to the next point. Moffitt was saying that God could exist (without any sort
of proof or justification, of course) in a dimension where time flows both forwards and
backwards and almost immediately was claiming that time and space were created at the Big
Bang (and did not exist before) and that it was God that created time. Now, if God created
time then he/she obviously could not exist in a dimension where time flows (backwards or
forwards or eastwards or up Jerry's you-know-what). [It] seems like such nutcases want their
audiences to swallow any nonsense to justify their stupidity.

Another number I found interesting was the "Borel limit" for impossibility. Borel would be
turning in his grave (assuming he was buried) if he heard his theory of probability being so
badly mutilated at the hands of super-mathematician Jerry Moffitt and his trusty sidekick
Marion Fox. Moffitt was making great news of the "fact" that if the chances against an event
are greater than 1 in 10^50, even the great Borel had said that this would not happen. Well, if
one took a deck of 52 cards, shuffled it (or did not even shuffle it), and one by one opened out
the cards, the probability of that exact sequence occurring is 1 in about 10^68---which
according to Jerry could not have happened. Since it did happen, maybe we all really exist in
a dimension where time flows upwards and downwards, while Elvis and the UFOs move
sideways and zig-zag.

Finally, Jerry (supreme Nobel-prize caliber physicist) was also claiming that we live in a
dimension where time moves in only one direction---the so-called half-dimension of time.
Sadly, most physicists would claim that the notion of a "flow of time" is purely a human
fabrication. The laws of physics are completely unbiased as to the direction of the arrow of
time. Even the Second Law of Thermodynamics (so often quoted by creationists) is not totally
inviolate---mathematical physicists have proved that if one waited sufficiently long (the
Poincare cycles) the universe would revisit any arbitrarily chosen state. While it is true that
the Poincare cycles are so unimaginably large (much much larger than anything Jerry could
ever dream of writing down on a board and showing his audiences), it is also true that the
Second Law is only a local, short-term (relatively speaking), statistical rule for a closed
system. The past and the future are in no way special. Of course, I would not expect Jerry to
understand or accept this, or nontechnical audiences to appreciate the truly amazing theories
that physicists have built over centuries of time, through hard work (and not just through
contemplation on some divine entity).

What really cracked me up, made me realize the wickedness of my ways, and repent was
Jerry's quote from Carl Sagan that chances against the formation of the universe were 1 in
10^(2-billion). This was the single point during the whole debate that I got past my
frustrations and had a really good laugh. This number is so large that even Jerry is satisfied
using the smaller numbers from such eminent physicists as Hugh Ross or Duane Gish or
Henry Morris.

Volume 1990 - 2002 Issue


Page 659 of 2049
Skeptical Review Edited by Farrell Till
Speaking of Ross, there is another nutcase of his ilk, Frank Tipler who has written in his
recent book The Physics of Immortality that general relativity can be used to prove that the
Bible is the one true word. I imagine we shall soon begin seeing and hearing the bible-
thumpers start to thump Tipler as well in future debates.

When I first started to watch the debate, I had some paper and a pen, ready to take notes.
After 10 minutes of Moffitt I simply gave up all hope of hearing one sensible phrase from
him. And then we now have Marion Fox's "explanation" of the flaws in the thinking of
atheists and agnostics (in TSR). His "explanation" had me doing triple and quadruple takes
just deciphering what the hell he was "explaining." I sincerely admire you for even agreeing
to debate such people.

(Krishna Kunchithapadam, 1210 West Dayton Street, Madison, WI 53706. E-mail


krisna@cs.wisc.edu)

I must admit that after receiving only three issues of TSR, I'm hooked! Anxiously awaiting
each issue, I devour it in one evening and ponder the stimulating articles for days. So I would
now like to order every back issue from Volume 1, #1, to Volume 5, #4, inclusive. Enclosed
is $20 for the materials requested.

I also wanted to mention that the WWW site maintained by Jeff Lowder
(http://paul.spu.edu:80/library/modern/jeff_lowder/sr/) has been moved to a site maintained
by the Internet Infidels (http://www.infidels.org/library/magazines/tsr/).

And, by all means, keep up the excellent work. The more people are exposed to it, the more
we can shed our supersitions and get on with living.

(Thomas G. Concannon, 1105 Highway 54 Bypass, 8-D Kingswood Apartments, Chapel Hill,
NC 27516.)

Thank you so very much for your uplifting magazine. I, as I'm sure many of your readers are,
am isolated in a sea of religion. I've been a member of the AHA for a while now but have
never been so uplifted as I was after reading your readers' letters. It is true [that] I have been
much happier since becoming a skeptic than I was when I was a believer. The problem has
been that I have had to go "underground" with my feelings because of friend/family alienation
and discrimination. It's nice to find other people whom I can relate to.

I found out about you through the internet, so I guess that lends credence to your belief that
computers will mean the destruction of fundamentalism. I agree with you in general.
Certainly many people are not skeptical simply because they have never been exposed to the
other side of the coin. We accept many things that are never questioned. I remember when I
became a skeptic it had never dawned on me to question the existence of god. The question
had always been, what was he? It was never, is he?

Volume 1990 - 2002 Issue


Page 660 of 2049
Skeptical Review Edited by Farrell Till
I am enclosing a check for $25 for the back issues of your magazine: 1990-1994. Thank you
so much for the great work that you are doing! I know you could go around deluding yourself
and others thumping a bible and making lots of money, Oh, well, the truth may set you free,
but it won't make you rich.

(John Sandstrum, 1805 Devine, Jackson, MS 39202.)

Enclosed you will find a money order for $6 for a subscription to The Skeptical Review.
Sending me those free sample issues is what convinced me to subscribe to your informative
newsletter. I will be looking forward to receiving my first issue from you.

(Roger La Porte, 148 Freund Street, Buffalo, NY.)

EDITOR'S NOTE: We are often asked why we give a free first-year subscription. The answer
is that most people who request the free subscription become paid subscribers after they have
had the opportunity to read a few issues at no cost to them.

The God concept is a fantasy, pure and simple. Why? There is not a scintilla of evidence to
support belief in a deity. How can anyone believe in an entity that no one has seen or heard?

Why believe in an imaginary creature? Who needs a deity? What good is it? What can it do?
It is impossible to believe in something that does nothing and hides from everyone. Is this
imaginary being ashamed of something? Why doesn't he reveal himself to everyone? Why
should some think he did many years ago in a backward country in the Near East?

Jesus Christ has been dead nearly 2,000 years and no one has seen hide nor hair of him. Why?
He is a creature of myth and legend. Albert Schweitzer said, "There is nothing more negative
than the result of the critical study of the life of Jesus."

Some theologians say the burden of proof for God's nonexistence rests on the atheist. They
are in error. Atheists want believers to do what should be, in principle, much easier. If their
God exists, bring him up for all to see.

Believers talk about a "creator" or "designer." The cosmos contains only matter and energy
that has always existed and always will. There was no first cause. Whatever looks as though it
was designed is the result of millions of years of evolution.

A thousand years from now, most people will be atheists. Christianity will principally be
remembered for bringing about the Dark Ages.

All the gods are dead. The truth is that they never existed except in the imagination of early
man and today in the minds of lunatics.

Volume 1990 - 2002 Issue


Page 661 of 2049
Skeptical Review Edited by Farrell Till
(Jesse Bailey, 2149-B 16th Avenue South, Birmingham, AL 35205-5020.)

Please renew my subscription. Enclosed is $25 for this year and for the back issues that I have
missed.

I wish to let you know how very much I and my associates enjoy your newsletter. It is like a
beacon of light in a world full of ignorance and supersition. I look forward to the day when
the Bible will be regarded as simply another work of art rather than a guide to morality, and
when it will take its rightful place next to the Iliad and Faust as an elaborate work of fiction
and nothing more.

Allow me to share a very telling "Christian" memory I have. Last year my brother was
brainwashed into becoming a born-again fundamentalist. For the next three months, he
became obsessed with his religion, giving all of his money away to the church. Everything
became a sin to him, and I, as a hardcore atheist, became the devil incarnate. Despite all
attempts to show him the Bible's obvious errors and inconsistencies, he insisted on being
faithful. His fanaticism would have undoubtedly lasted longer than three months if he had not
been murdered at the Circle-K store where he worked. I had never seen a more fanatically
devoted Christian, and the reward for his faith was to be shot down like a dog at 20 years of
age.

I suppose all of his fundamentalist friends were really happy for him, getting to meet Jehovah
so soon and all, but to those of us who knew the truth, it was a real tragedy. Every Christian
who said to me, "He's surely in a better place now," insulted me deeply. As of yet, I have seen
no photographs of him partying in heaven, and I don't expect to any time soon. This ridiculous
idea of a richer life beyond the grave needs to be blotted out. It simply creates a mindset that
discourages people from improving things here and now. Fundamentalists have no idea how
much damage they are doing to the human race by their infernal theologies--and what's worse
is that most of them don't even seem to care.

Keep up the excellent work.

(Randall Le Jeune, P. O. Box 323, Plaquemine, LA 70765-0323.)

EDITOR'S NOTE: I have sent my sympathy to Mr. Le Jeune in a separate letter in which I
asked him for permission to publish his letter. It focuses on a problem that theists have never
given a satisfactory response to, i.e., the existence of evil in a world presumably created by an
omniscient, omnipotent, omnibenevolent deity. The argument is simple but unanswerable,
except of course with theistic gibberish. If God is omniscient, then he must know that evil
exists in the world he created. If God is omnipotent, he has the power to eliminate the evil that
he is aware of. If God is omnibenevolent, then he would have the desire to eliminate the evil
that he knows about and has the power to eliminate. Since evil continues to exist in the world,
we must conclude that God cannot be simultaneously omniscient, omnipotent, and
omnibenevolent, and any deity who is not simultaneously omniscient, omnipotent, and

Volume 1990 - 2002 Issue


Page 662 of 2049
Skeptical Review Edited by Farrell Till
omnibenevolent cannot be the Christian God. Furthermore, if God is omniscient and
omnipotent but not omnibenevolent, then he does not deserve our respect.

I'm writing to request the Gleason Archer letter and the entire paper by Dr. Robert Countess
that was referred to in the Winter 1992 issue p. 4....

Thank you for TSR. It has helped me to confirm my departure from born- again Bible believer
to freethinker. My journey started when I had general questions on the four gospel accounts of
the resurrection of the NT Jesus and the so-called "Messianic prophecies" being fulfilled in
the NT Jesus. I put off the studies, looked a little here and there, put if off mostly, but the
questions kept returning, so I finally decided to really dig in. My former pastor said that when
I studied the issues I had questions about, I should go only to the text of the Bible and if I
went outside the Bible, I should read only inerrancy defenses. On my first study, I expanded it
to the crucifixion, resurrection, and ascension accounts of the NT Jesus. I didn't use gospel
parallels and other sources that were like them, only the biblical text. I had many more
problems after this than before I began. When I read the inerrancy defenses, they really didn't
give me solid, intellectually honest answers. I read Josh McDowell on the issue of Jesus's
resurrection, defenses by Gleason Archer, Norman Geisler, and others. When I read the other
side, it made more sense. (I didn't read a lot on this side.) Then I tackled the "Messianic
prophecies." I went to the Jewish scriptures, OT passages that Christians use, and then to the
Christian NT passages where they are supposed to be fulfillments. I saw more problems and
things that just didn't add up. I went to inerrantist Christian defenses of the position, like
Norman Geisler and others, and to those sources on "the other side," many or most being
Jewish views of the "Messianic prophecies" and the whole issue of the Messiah doctrine. The
Christian side just did not have answers that proved the NT dealt honestly with the passages
(rationalizing notwithstanding). My two studies surely didn't show me overwhelming,
harmonious, it-could- only-have-been-God evidence for the Bible's supposedly divine origin.
There is real evidence against Christian Bible belief that the Christians need to face up to.
McDowell, Archer, and the others have not done this. The "verdict," I believe, is that the
Bible simply doesn't stand.

This decision hasn't come to me without searching, hoping to find answers, much studying,
and crying in church. Thanks for TSR and the debate videos. Keep up the good fight of
reason.

(Harry Ricker, Jr., P. O. Box 6269, Haverhill, MA 01830.)

I can't thank you enough for your wonderfully informative and entertaining publication. As a
recovering fundamentalist, I think that you are indeed a breath of fresh air! Keep up the good
work. Enclosed is a check for my renewal.

(Jamie Smith, 3500 East Park Boulevard, Apt. 104, Plano, TX 75074.)

Volume 1990 - 2002 Issue


Page 663 of 2049
Skeptical Review Edited by Farrell Till

Your response to my letter (Summer 1995, p. 12) seems to successfully address my objection
to (and misunderstanding of) your article [concerning the raising of Jairus's daughter]. You
are right to be confused by my distinction between necessary and sufficient. I misread your
argument.

Thank you for your commendable publication. Although you may think this letter is
superfluous, I do hope you print it. Admitting error is important.

(Richard Trott, 78-A Phelps Avenue, New Brunswick, NJ 08901.)

Announcements
Horner-Till Debate

To have room for the article about the American Society for Religious Concern (p. 11), we
had to delay publication of a review of the Horner-Till Debate on the resurrection. The video
tape has missing sections, so we will not be copying it for rental, but a transcript prepared
from both video and audio tapes can be obtained for $3. If you would like to have your own
video tape (with the understanding that sections are missing), we will send you one for $6.

Back Issues & Other Videos

All back issues of TSR from Winter 1990 through Autumn 1995 can be obtained for $1 per
copy. Some of these will be loose-leaf reprints. Besides the five issues published this year, 20
issues were printed from 1990-1994.

Video tapes of oral debates and transcripts of written and oral debates are also available. For
information on ordering these, please refer to "Study Aids" on page 7 of the Spring 1995 issue
or check earlier back issues for details.

Volume 1990 - 2002 Issue


Page 664 of 2049
Skeptical Review Edited by Farrell Till

The Skeptical Review


Volume Seven – 1996
Farrell Till, Editor

• January/Feburary Volume Seven, Issue One


• March/April Volume Seven, Issue Two
• May/June Volume Seven, Issue Three
• July/August Volume Seven, Issue Four
• September/October Volume Seven, Issue Five
• November/December Volume Seven, Issue Six

Volume 1990 - 2002 Issue


Page 665 of 2049
Skeptical Review Edited by Farrell Till

Skeptical Review
Volume Seven, Number One
Jauary/February 1996
Farrell Till, editor

• God, Captain Scott O'Grady, and the Atlanta Braves


A discussion of the special relationship with God that Captain Scott O'Grady
apparently thinks that he enjoys.

• Fulfilled Prophecy: Evidence for the Reliability of the Bible


Dr. Hugh Ross attempts to prove the infallibility of the Bible by stating some unusual
probability statistics.

• Prophecy Fulfillment: An Unprovable Claim


Farrell Till counters Dr. Ross' claims.

• "Killed Their Own Prophets": New Testament Libel of the Jews


Stephen Van Eck suggests that the Jewish people are getting a bum rap by being
blamed for the killing of their own prophets.

• Another Preacher Who Never Ceases to Be Amazed


Farrell Till never ceases to be amazed at the extremes that religionists will go to find
inerrancy in God's word.

• Christianity and Hindu Influence


Jim Dew explains the case for the incorporation of Hindu ideas into early Christian
writings.

• From The Mailbag


Recovering fundamentalists sing our praises.

Volume 1990 - 2002 Issue


Page 666 of 2049
Skeptical Review Edited by Farrell Till
• The Back Page
Study aids, submission guides and general announcements.

God, Captain Scott O'Grady, and the


Atlanta Braves
Farrel Till
In an earlier issue (Autumn 1994), we discussed some public attitudes concerning God and O.
J. Simpson, and now it seems appropriate to discuss the special relationship with God that
Captain Scott O'Grady apparently thinks that he enjoys. Captain O'Grady, as most readers will
remember, was the American pilot whose plane was shot down during a mission over Bosnia.
He parachuted to safety and, after eluding capture for several days, was finally plucked out of
Serbian-occupied territory in a daring rescue mission. O'Grady had no sooner set foot on
friendly territory (in this case the deck of a U. S. aircraft carrier) when he began to attribute
his survival and rescue to God. Since then he has incessantly praised God for his dramatic
rescue.

Through all of the publicity that the various news media have given O'Grady and his rescue,
not once did any reporter or interviewer ever ask O'Grady an obvious question that must have
been on the minds of all freethinkers who heard the captain thanking God for bringing him
safely back. It would have been so gratifying if just once a reporter had said something like
this: "Captain O'Grady, each day on television, we see pictures of Bosnian civilians, many of
them children, who have been killed by sniper fire or blown apart by mortar shells. Many of
these casualties, which now number in the thousands, are people whose faith in God is just as
strong as yours, so why do you think that God would have such a special interest in you that
he would protect you from just being captured when apparently he has no interest in keeping
children from being maimed and slaughtered in the daily carnage occurring throughout
Bosnia?"

We might also add that if O'Grady is right in this matter, then God apparently likes American
pilots better than those of other nations, for after O'Grady's rescue, a French fighter plane was
shot down over Bosnia and, although its two crewmen were seen parachuting to safety, God
hasn't yet seen fit to rescue them. We have to wonder why this disparity in divine providence
continues to go unnoticed by journalists who say nothing about it as Captain O'Grady
continues his praise-God-for-saving-me rhetoric.

The reason for this journalistic silence, of course, is no mystery. Even if any reporters have
had such thoughts, they probably understand that they had better not express them, because
there is a "religion good/skepticism bad" mentality in this country that could harm a reporter's
career if he dared to challenge it too openly. So undoubtedly some reporters go along with the
game out of professional necessity.

Volume 1990 - 2002 Issue


Page 667 of 2049
Skeptical Review Edited by Farrell Till
Another likely reason why God-saved-me remarks like Captain O'Grady's receive so much
media fanfare is that reporters know that such comments are exactly what the American
public wants to hear. Writing on this same subject, Tim Gorski, editor of *The Freethought
Exchange*, said this about O'Grady's comment:

"I'm just sorry that the downed airman hadn't instead been Captain Moshe Goldstein
who might have said, "The God of Abraham and Isaac saved me from the Serbs! Only
because I wore my yarmulke and prayed with my prayer shawl was I saved!" Or how
would Americans have reacted if Captain Mohammed Faisal had been the hero who
proceeded to tell Americans that "Allah alone is responsible for my rescue! Allah
Akbar! Allah Akbar!" Or [choke] just think of the entertainment value we would have
had if it had been *an atheist* who, upon being rescued, said, "I certainly didn't waste
my time praying to nonexistent god(s); I'm here today because of the bravery and
determination of the well-trained people of the U. S. military, among whom I proudly
count myself" (July/ August 1995, p. 137). In the event of the latter scenario, this
matter would have been shushed up long ago. As it is, Captain O'Grady continues to
exploit the situation. Recent news reports indicate that he plans to leave the air force
and is considering book and movie offers. Ah, yes, there is money to be made in the
God-was-with-me business. Skeptics and atheists just haven't yet learned which side
their bread is buttered on."

If these comments seem unduly cynical, just stop and consider how often comments like
O'Grady's are made. When a hurricane or a tornado strikes an area, some people's property
inevitably escapes serious damage. So what do we see during the news coverage? Reporters
will thrust their microphones in the face of someone standing in front of his relatively
undamaged house with chaos and destruction all around him, and the guy will say, "The Lord
was with me." When an airplane crashes, if any passengers survive, sooner or later we will
see them on TV thanking God for saving them. Apparently, it never occurs to these people
that if God is to be thanked for saving one house from destruction, then he must be blamed for
all of the houses that were destroyed. If an omnipotent God saved one or two from death in a
plane crash, then he could have saved all of the passengers, or, even better, he could have kept
the plane from crashing. So if there is any truth to the survivors' claims that God saved them,
then friends and relatives of the victims of tragedy should curse God for showing a flagrantly
unbenevolent partiality.

One of the absurdest examples of this kind of reasoning (or rather lack of the same) occurred
after the Atlanta Braves had defeated the Cincinnati Reds to win the National League
pennant. Reporters and TV cameras were in the dressing room to record the pandemonium as
the Braves celebrated. While a player was being interviewed with trite questions about his
reaction to the victory, a voice in the background was heard to say, "The Lord sure works in
mysterious ways." Yeah, sure, we are supposed to believe that the creator of the universe so
wanted some of the World Series games to be played in Atlanta that he intervened to have the
Braves beat the Reds. Apparently, God just doesn't care anything about the feelings of
baseball fans in Cincinnati. Perhaps they should repent and turn to God so that the World
Series can be played in Cincinnati next year.

Volume 1990 - 2002 Issue


Page 668 of 2049
Skeptical Review Edited by Farrell Till
Did the reporter holding the microphone turn to the player who had made this comment and
ask him to explain why he thought that God had favored the Atlanta Braves? No, he didn't.
Did the reporter ask the player who had said this if he thought it was fair of an omnipotent,
omniscient God to put himself on the side of one of the teams and pull strings in such a way
as to enable it to win? No, he didn't. Reporters who record comments like these just never
seem to have the perception or the courage to put people like this on the spot and demand that
they justify their extraordinary claims of having an omnipotent deity watching over them.

Journalists are supposed to be professionals, and a true professional would want to get the
whole story, but as long as reporters allow statements like these to go unchallenged and
unexplained, they are not getting the whole story. They are selling out to popular opinion.

Meanwhile, we have to pity the poor Indians. They didn't have a chance to win the World
Series with God on the side of the Braves. They did, however, prove themselves to be the
better team by winning two games with God playing for the other side.

Fulfilled Prophecy: Evidence for the


Reliability of the Bible
Dr. Hugh Ross
Unique among all books ever written, the Bible accurately foretells specific events--in detail--
many years, sometimes centuries, before they occur. Approximately 2500 prophecies appear
in the pages of the Bible, about 2000 of which already have been fulfilled to the letter--no
errors. (The remaining 500 or so reach into the future and may be seen unfolding as days go
by.) Since the probability for any one of these prophecies having been fulfilled by chance
averages less than one in ten (figured very conservatively) and since the prophecies are for the
most part independent of one another, the odds for all these prophecies having been fulfilled
by chance without error is less than one in 10^2000 (that is 1 with 2000 zeros written after it)!

God is not the only one, however, who uses forecasts of future events to get people's
attention. Satan does too. Through clairvoyants (such as Jeanne Dixon and Edgar Cayce),
mediums, spiritists, and others, come remarkable predictions, though rarely with more than
about 60 percent accuracy, never with total accuracy. Messages from Satan, furthermore, fail
to match the detail of Bible prophecies, nor do they include a call to repentance.

The acid test for identifying a prophet of God isrecorded by Moses in Deuteronomy 18:21-
22. According to this Bible passage (and others), God's prophets, as distinct from Satan's
spokesmen, are 100 percent accurate in their predictions. There is *no* room for error.

As economy does not permit an explanation of all Biblical prophecies that have been fulfilled,
what follows [is] a discussion of a few that exemplify the high degree of specificity, the range

Volume 1990 - 2002 Issue


Page 669 of 2049
Skeptical Review Edited by Farrell Till
of projection, and/or the "supernature" of the predicted events. Readers are encouraged to
select others, as well, and to carefully examine their historicity.

(1) Some time before 500 B. C. the prophet Daniel proclaimed that Israel's long-awaited
Messiah would begin his public ministry 483 years after the issuing of a decree to restore and
rebuild Jerusalem (Daniel 9:25-26). He further predicted that the Messiah would be "cut off,"
killed, and that this event would take place prior to a second destruction of Jerusalem.
Abundant documentation shows that these prophecies were perfectly fulfilled in the life (and
crucifixion) of Jesus Christ. The decree regarding the restoration of Jerusalem was issued by
Persia's King Artaxerxes to the Hebrew priest Ezra in 458 B. C., 483 years later the ministry
of Jesus Christ began in Galilee. (Remember that due to calendar changes, the date for the
start of Christ's ministry is set by most historians at about 26 A. D. Also note that from 1 B. C.
to 1 A.D. is just one year.) Jesus's crucifixion occurred only a few years later, and about four
decades later, in 70 A. D. came the destruction of Jerusalem by Titus. (Probability of chance
fulfillment=1 in 10^5.)

*The estimates of probability included herein come from a group of secular research
scientists. As an example of their method of estimation, consider their calculations for this
first prophecy cited:
Since the Messiah's ministry could conceivably begin in any one of about 5000 years,
there is, then, one chance in about 5000 that his ministry could begin in 26 A.D.
Since the Messiah is God in human form, the possibility of his being killed is
considerably low, say less than one chance in 10.
Relative to the second destruction of Jerusalem, this execution has roughly an even
chance of occurring before or after that event, that is, one chance in 2.
Hence the probability of chance fulfillment for this prophecy is 1 in 5000 x 10 x 2,
which .lh6.2 is 1 in 100,000, or 1 in 10^5.

(2) In approximately 700 B. C. the prophet Micah named the tiny village of Bethlehem as the
birthplace of Israel's Messiah (Micah 5:2). The fulfillment of this prophecy in the birth of
Christ is one of the most widely known and widely celebrated facts in history. (Probability of
chance fulfillment=1 in 10^5.)

(3) In the fifth century B. C. a prophet named Zechariah declared that the Messiah would be
betrayed for the price of a slave--thirty pieces of silver, according to Jewish law--and also that
this money would be used to buy a burial ground for Jerusalem's poor foreigners (Zechariah
11:12-13). Bible writers and secular historians both record thirty pieces of silver as the sum
paid to Judas Iscariot for betraying Jesus, and they indicate that the money went to purchase a
"potter's field," used--just as predicted--for the burial of poor aliens (Matthew 27:3-10).
(Probability of chance fulfillment=1 in 10^11.)

(4) Some 400 years before crucifixion was invented, both Israel's King David and the prophet
Zechariah described the Messiah's death in words that perfectly depict that mode of execution.
Further, they said that the body would be pierced and that none of the bones would be broken,
contrary to customary procedure in cases of crucifixion (Psalm 22 and 34:20; Zechariah
12:10). Again, historians and New Testament writers confirm the fulfillment: Jesus of
Nazareth died on a Roman cross, and his extraordinarily quick death eliminated the need for

Volume 1990 - 2002 Issue


Page 670 of 2049
Skeptical Review Edited by Farrell Till
the usual breaking of bones. A spear was thrust into his side to verify that he was indeed,
dead. (Probability of chance fulfillment=1 in 10^13).

(5) The prophet Isaiah foretold that a conqueror named Cyrus would destroy seemingly
impregnable Babylon and subdue Egypt along with most of the rest of the known world. This
same man, said Isaiah, would decide to let the Jewish exiles in his territory go free without
any payment of ransom (Isaiah 44:28; 45:1; and 45:13). Isaiah made this prophecy l50 years
before Cyrus was born, 180 years before Cyrus performed any of these feats (and he did,
eventually, perform them all), and 80 years before the Jews were taken into exile. (Probability
of chance fulfillment=1 in 10^15.)

(6) Mighty Babylon, 196 miles square, was enclosed not only by a moat, but also by a double
wall 330 feet high, each part 90 feet thick. It was said by unanimous popular opinion to be
indestructible, yet two Bible prophets declared its doom. These prophets further claimed that
the ruins would be avoided by travellers, that the city would never again be inhabited, and that
its stones would not even be moved for use as building material (Isaiah 13:17-22 and
Jeremiah 51:26,43). The description is, in fact, the well-documented history of the famous
citadel. (Probability of chance fulfillment=1 in 10^9.)

(7) The exact location and construction sequence of Jerusalem's nine suburbs was predicted
by Jeremiah about 2600 years ago. He referred to the time of this building project as "the last
days," that is, the time period of Israel's second rebirth as a nation in the land of Palestine
(Jeremiah 31:38-40). This rebirth became history in 1948, and the construction of the nine
suburbs has gone forward precisely in the locations and in the sequence predicted.
(Probability of chance fulfillment=1 in 10^18.)

(8) The prophet Moses foretold (with some additions by Jeremiah and Jesus) that the ancient
Jewish nation would be conquered twice and that the people would be carried off as slaves
each time, first by the Babylonians (for a period of 70 years), and then by a fourth world
kingdom (which we know as Rome). The second conqueror, Moses said, would take the Jews
captive in ships, selling them or giving them away as slaves to all parts of the world. Both of
these predictions were fulfilled to the letter, the first in 607 B.C. and the second in 70 A.D.
God's spokesmen said, further, that the Jews would remain scattered throughout the entire
world for many generations, but without becoming assimilated by the peoples or of other
nations, and that the Jews would one day return to the land of Palestine to re-establish for a
second time their nation (Deuteronomy 29; Isaiah 11:11-13; Jeremiah 25:11; Hosea 3:4-5 and
Luke 21:23-24). This prophetic statement sweeps across 3500 years of history to its complete
fulfillment--in our lifetime. (Probability of chance fulfillment=1 in 10^20.)

(9) Jeremiah predicted that despite its fertility and despite the accessibility of its water supply,
the land of Edom (today a part of Jordan) would become a barren, uninhabited wasteland
(Jeremiah 49:15-20; Ezekiel 25:12-14). His description accurately tells the history of that now
bleak region. (Probability of chance fulfillment=1 in 10^5.)

(10) Joshua predicted that Jericho would be rebuilt by one man. He also said that the man's
eldest son would die when the reconstruction began and that his youngest son would die when
the work reached completion (Joshua 6:26). About five centuries later this prophecy found its

Volume 1990 - 2002 Issue


Page 671 of 2049
Skeptical Review Edited by Farrell Till
fulfillment in the life and family of a man named Hiel (1 Kings 16:33-34). (Probability of
chance fulfillment=1 in 10^7.)

(11) The day of Elijah's supernatural departure from Earth was predicted unanimously--and
accurately, according to the eye-witness account--by a group of fifty prophets (2 Kings 2:3-
11). (Probability of chance fulfillment=1 in 10.)

(12) Jahaziel prophesied that King Jehoshaphat and a tiny band of men would defeat an
enormous, well-equipped, well-trained army without even having to fight. Just as predicted,
the King and his troops stood looking on as their foes were supernaturally destroyed to the
last man (2 Chronicles 20). (Probability of chance fulfillment=1 in 10^8.)

(13) One prophet of God (unnamed, but probably Shemiah) said that a future king of Judah,
named Josiah, would take the bones of all the occultic priests (priests of the "high places") of
Israel's King Jeroboam and burn them on Jeroboam's altar (1 Kings 13:2 and 2 Kings 23:15-
18). This event occurred approximately 300 years after it was foretold. (Probability of chance
fulfillment=1 in 10^13.)

Since these thirteen prophecies cover mostly separate and independent events, the probability
of chance occurrence for all thirteen is about 1 in 10^138 (138 equals the sum of all the
exponents of 10 in the probability estimates above). For the sake of putting the figure into
perspective, this probability can be compared to the statistical chance that the second law of
thermodynamics will be reversed in a given situation (for example, that a gasoline engine will
refrigerate itself during its combustion cycle or that heat will flow from a cold body to a hot
body)--that chance=1 in 10^80. Stating it simply, based on these thirteen prophecies along,
the Bible record may be said to be vastly more reliable than the second law of
thermodynamics. Each reader should feel free to make his own reasonable estimates of
probability for the chance fulfillment of the prophecies cited here. In any case, the
probabilities deduced still will be absurdly remote.

Given that the Bible proves so reliable a document, there is every reason to expect that the
remaining 500 prophecies, those slated for the "time of the end," also will be fulfilled to the
last letter (see *signs of the End, Tribulation Dooms,* and *The Rapture Promise*). Who can
afford to ignore these coming events, much less miss out on the immeasurable blessings
offered to anyone and everyone who submits to the control of the Bible's author, Jesus Christ?
(See the paper entitled *Grace* for a list of those benefits.) Would a reasonable person take
lightly God's warning of judgment for those who reject what they know to be true about Jesus
Christ and the Bible, or who reject Jesus's claim on their lives?

Now is the day of salvation! Now is the time of God's favor! Please do not let it go by without
further consideration.

(Dr. Hugh Ross, P. O. Box 5978, Pasadena, CA 91107.)

Volume 1990 - 2002 Issue


Page 672 of 2049
Skeptical Review Edited by Farrell Till

Prophecy Fulfillment: An Unprovable


Claim
Farrell Till
Just a superficial reading of Dr. Ross's article about the application of probabilities to biblical
prophecy fulfillment is enough to see that it is just another attempt to shore up a discredited
apologetic argument. Biblical inerrantists have repeatedly tried to prove the divine origin of
the Bible through various prophecy-fulfillment claims, but the truth is that no one has ever
been able to verify a single example of biblical prophecy fulfillment. I have read many such
attempts, and Dr. Ross's effort differs only in that it is, in my opinion, not nearly as good as
some I have seen.

What is really ludicrous about Dr. Ross's article is his arbitrary probability figures. They are
reminiscent of the ridiculous probability arguments that creationists bandy about to "prove"
that life could not have happened without divine creation. Before anyone can determine the
probability of any event, he must know all of the factors that would be involved in the
occurrence of the event, and this is where the absurdities begin in most creationist and
prophecy-fulfillment probability arguments. No one knows how many factors would be
involved in the formation of life, and nobody knows how many factors would be involved in
the occurrence of a specific event years in the future. Yet biblicists constantly yak about how
the probability of such-and-such happening would be one in so many millions or billions or
trillions. It's all too ridiculous to deserve serious comment, but because so many
fundamentalists use such arguments to impress the gullible, we, unfortunately, have to
comment on their "arguments" from time to time.

In order to prove--and I mean *prove*, not just surmise--prophecy fulfillment, one would
have to establish four things: (1) the claimant of a prophecy fulfillment is properly
interpreting whatever text he is basing his claim on, (2) the prophecy was made *before* and
not after the event that allegedly fulfills the prophecy, (3) the prophecy was made not just
*before* an event but far enough in advance of it to make educated guesswork impossible,
and (4) the event that allegedly fulfilled the prophecy did in fact happen. When Dr. Ross's
claims of prophecy fulfillment are examined in terms of these four characteristics of valid
prophecy, we will see that none of his alleged prophecy fulfillments can pass muster.

In a continuing series of articles, I will examine all thirteen of Ross's claims but not
necessarily in the order he presented them. Let's begin, then, with number two: the birth of
Jesus in Bethlehem fulfilled a prophecy about where the Messiah would be born (Micah 5:2).
Ross said that "(t)he fulfillment of this prophecy is one of the most widely known and widely
celebrated facts in history," but is it? I say that this prophecy-fulfillment claim fails tests 1 and
4 in the list above.

Did Micah unquestionably predict that the Messiah would be born in the town of Bethlehem?
That's what most biblicists believe, but it is certainly not something that they can prove
definitively enough to claim that they have an unquestionable prophecy fulfillment in the

Volume 1990 - 2002 Issue


Page 673 of 2049
Skeptical Review Edited by Farrell Till
place of Jesus's birth. To evaluate the claim, let's first look at the statement that Ross alleges
was a prophecy of the Messiah:

"But you Bethlehem Ephrathah, though you are little among the thousands of Judah,
yet out of you shall come forth to Me the One to be Ruler in Israel, Whose goings
forth are from of old, from everlasting (Micah 5:2, NKJV).

In the context of the passage in which Micah made this statement, he was speaking of "many
nations [that] have gathered against you [Israel]" (4:11). Inparticular, there seemed to be
concern about "the Assyrian com[ing] into our land" (5:5), so it makes good sense to assume
that Micah, rather than predicting the coming of a Messiah in the distant future, was talking
about a "ruler" who would arise to help Israel during the present threat to its national security.
There is serious doubt, then, that Micah even intended his statement to be a Messianic
prophecy beyond the sense of someone arising to lead Israel through its present crisis.

For the sake of argument, however, let's just assume that Micah did intend this to be a
prophecy of the long-awaited Jewish Messiah. Even if this were so, there would still be
serious problems to overcome before Ross or anyone could prove that a birth in the town of
Bethlehem fulfilled this prophecy. First, it is questionable that Micah 5:2 was even referring
to the *town* of Bethlehem. Several translations suggest that he meant a tribal clan and not a
town. Many people who cite Micah 5:2 as a case of amazing prophecy fulfillment don't
realize that a person named Bethlehem was an Old Testament character who had descended
from Caleb through Hur, the firstborn son of Caleb's second wife, Ephrathah ( 1 Chron. 2:18;
2:50- 52; 4:4). Young's Analytical Concordance, p. 92, identifies Bethlehem as the name of
this person as well as the name of two different villages.

The NIV translates the relevant part of Micah 5:2 like this: "But you, Bethlehem Ephrathah,
though you are small among the *clans* of Judah...." A clan, of course, is not a town, so if
this translation is accurate, the prophet was speaking not of a place but a tribal family that
would give rise to a ruler. The RSV, NRSV, NAS, NAB, NEB, REB, the Amplified Bible, the
Jerusalem Bible, and others give similar renditions that agree that Micah was referring to a
family clan rather than a town.

Young's Literal Translation of the Holy Bible refers to the Bethlehem Ephrathah in this
passage as something that is "little to be among the chiefs of Judah," another implication that
the prophet was speaking of a person or clan, but even more damaging to Matthew's attempt
to apply this statement to the town of Bethlehem is the Septuagint translation of the passage:

"And thou, Bethlehem, *house of Ephrathah*, art few in number to be reckoned


among the thousands of Judah: yet out of thee shall one come forth to me, to be a ruler
of Israel "(Brenton Translation).

Three statements in this version are significant. First it was said that Bethlehem was "few in
number to be reckoned among the thousands of Judah." If this was a reference to the town of
Bethlehem, then indeed it would have been "few in number." *One* is about as "few" as you
can get. Secondly, this Bethlehem was too few to be "reckoned among the thousands of
Judah. The towns in Judah could not have been reckoned in terms of thousands; the country

Volume 1990 - 2002 Issue


Page 674 of 2049
Skeptical Review Edited by Farrell Till
was just too small to have "thousands" of towns. The statement, however, makes sense if
interpreted as a reference to the family clans in Judah. Finally--and this is the clincher--this
Bethlehem was described as the "house of Ephrathah." When we encounter expressions like
"house of David" or "house of Levi" in the Bible, we immediately recognize them as
references to family clans, not towns or cities, and we should do the same here. Matthew
obviously distorted Micah 5:2 to try to make it a reference to the town of Bethlehem, and this
becomes more apparent when we consider that Matthew usually quoted the Septuagint
version when he referred to the Old Testament. But he didn't here, possibly because the
Septuagint shows too clearly that Micah 5:2 was referring to a family group or clan.

Again, for the sake of argument, let's just assume that Micah was indeed referring to the town
of Bethlehem in his famous "prophecy." Dr. Ross still would not have a verifiable case of
prophecy fulfillment, because neither he nor anyone else can prove that Jesus of Nazareth was
born in Bethlehem. As noted earlier, Ross said that "the fulfillment of this prophecy [the birth
of the Messiah in Bethlehem] is one of the most widely known and widely celebrated facts of
history." Now if he had said that the birth of Jesus in Bethlehem was one of the most widely
known and widely celebrated "claims" or "traditions," I would have no quarrel with him, but
to say that it is a widely known and celebrated "fact of history" that Jesus was born in
Bethlehem is to make an assertion that cannot be established as fact.

Ross has simply assumed that the New Testament records are historically accurate; therefore,
if they say that Jesus was born in Bethlehem, it is a historical fact that he was born in
Bethlehem. Unfortunately for his probability argument on this point, he has no way of
proving that Jesus was actually born in Bethlehem. There are no contemporary documents or
records of any kind to corroborate the New Testament claim that Jesus was born in
Bethlehem. As "The Historicity of Jesus" (The Skeptical Review, Autumn 1995, p. 1) pointed
out, despite the New Testament claims that Jesus was so popular that great multitudes
followed him and even brought the sick and the lame to him from as far away as Syrian to be
cured by his miraculous powers, no contemporary writers even mentioned the man. He was
known only in the New Testament documents and some apocryphal writings that even
Christianity has rejected. Since the gospel writers were obviously biased in their zealous
attempts to sell Jesus as the Messiah, how do we know that Matthew and Luke didn't just say
that he was born in Bethlehem in order to give the appearance that even his birth had fulfilled
Old Testament prophecy? So if Ross wants to talk about probabilities, I could ask him to
calculate what the odds would be that a writer like "Matthew" could make up details about the
birth of Jesus to make it appear that he had fulfilled prophecy by being born in the town of
Bethlehem. I would say that the odds for that would be about one in one.

From this point on, I would urge readers to keep in mind that most of Ross's arguments
assume the historical accuracy of the Bible, and that is certainly an improper way to argue for
prophecy fulfillment. One would have to be incredibly ignorant of the Bible not to know that
it contains numerous examples of both prophecy and prophecy-fulfillment claims. So whether
the Bible *claims* prophecy fulfillment is a point not relevant to this debate, because it
certainly does contain such claims. What Dr. Ross must do to sustain his position is prove that
the Bible's many claims of prophecy-fulfillment are true, and that is something he won't find
as easy as formulating arbitrary probability figures.

Volume 1990 - 2002 Issue


Page 675 of 2049
Skeptical Review Edited by Farrell Till
In his third example of prophecy-fulfillment, Dr. Ross claims that Judas's betrayal of Jesus for
30 pieces of silver was predicted by Zechariah, a prophet who lived in the 5th century B. C.
Of all of his prophecy-fulfillment claims, this is undoubtedly Dr. Ross's most disingenuous,
because he failed to inform his readers that the only New Testament claim that prophecy was
fulfilled when Judas was paid 30 pieces of silver for betraying Jesus was made by Matthew,
and he alleged that the payment fulfilled a prophecy that was made by Jeremiah, not
Zechariah (27:9). I have to wonder why Dr. Ross kept this controversial point from his
readers.

Zechariah did make a reference to 30 pieces of silver, but it was clearly not the statement that
Matthew in his inimitable way tried to quote. To see this, we have only to look at the two
statements together:

Matthew 27:9--Then was fulfilled what was spoken by *Jeremiah* the prophet,
saying, "*And they took the thirty pieces of silver, the value of Him who was priced,*
whom they of the children of Israel priced, *and gave them for the potter's field, as the
LORD directed me"* (Italics inserted as used in the NKJV).
Zechariah 11:12-13- -Then I said to them, "If it is agreeable to you, give me my
wages; and if not, refrain." So they weighed out for my wages thirty pieces of silver.
Yahweh said to me, "Throw it to the potter"--that princely price they set on me. So I
took the thirty pieces of silver and threw them into the house of Yahweh for the
potter."

One has to stretch imagination to assume that Matthew was actually quoting this passage from
Zechariah, but even if he was, Dr. Ross will have to explain why God's inspired writer said
that this was something Jeremiah had predicted. If Ross wants to think in terms of
probabilities, perhaps he can do a little figuring and let us know what would be the probability
that a writer who was verbally inspired by an omniscient, omnipotent deity would attribute to
Jeremiah a prophecy that had really been made by Zechariah. I would think that the odds
against this happening would be so astronomical that even the universe itself couldn't contain
all the zeroes that would follow the number. So the fact that Matthew made such an error (as
Ross has tacitly admitted by correcting Matthew's statement) is more than reasonable
evidence that he wasn't divinely inspired. To say the least, the mistake doesn't give readers
much confidence to believe that Matthew knew what he was talking about when he made this
prophecy-fulfillment claim.

Examination of the context in which Zechariah's statement was made also shows that it
obviously referred to a contemporary situation rather than an event that would happen five
centuries later. Zechariah was a prophet at the time that the repatriates from Babylonian
captivity were rebuilding Jerusalem under the leadership of Ezra and Nehemiah. The first half
of the book consists of "visions" that Zechariah had about the restoration of Jerusalem, but the
second half is an apocalyptic, doom-and-gloom/ day-of-the-Lord work, which many scholars
believe is a later redaction to Zechariah's optimistic predictions that Yahweh would lead his
people to restore Jerusalem to its former glory. Whether this view of the last part of the book
is correct or not, the passage that Dr. Ross alluded to is found in the apocalyptic section of the
book.

Volume 1990 - 2002 Issue


Page 676 of 2049
Skeptical Review Edited by Farrell Till
In the chapter where reference was made to 30 pieces of silver, Yahweh had commanded
Zechariah to "feed the flock for slaughter," because Yahweh would "no longer pity the
inhabitants of the land" (12:4-6). In other words, Yahweh was ticked off at his people again,
and so someone was going to have to pay a price. After a period of feeding the flock,
Zechariah found the work so disagreeable that he quit and demanded that the leaders pay him
wages for his work. The 30 pieces of silver were weighed out in payment to him, and Yahweh
then told him to throw them "into the house of Yahweh for the potter." The meaning of the
word translated *potter* in this statement is controversial, and it is translated *treasury* in
some versions (RSV, NRSV, NEB, NAB, GNB, JB, etc.). The context of the passage suggests
that Zechariah was ordered by Yahweh to "feed his flock" and that the leaders of "the flock"
had considered this work worth only 30 pieces of silver. This price was considered so
insulting that Yahweh ordered Zechariah to throw the money back into the treasury,
apparently as a gesture of contempt, and this is the passage that Ross sees as a prophecy of
Judas's betrayal of Jesus. Of course, whether Matthew saw it this way or not is debatable,
because he made no reference to Zechariah. At any rate, to see prophecy in a statement as
vague and uncertain as this only shows how desperate biblicists are for evidence to support
their claim that the Bible was divinely inspired.

Once again, however, let's suppose for the sake of argument that Dr. Ross is right and that
Zechariah did intend this statement to be a prophecy that someone would betray the Messiah
for 30 pieces of silver. That would not prove that it was fulfilled by Judas, because we have
only Matthew's word that Judas was paid 30 pieces of silver to betray Jesus and then
afterwards brought the money back and cast it down in the sanctuary. To argue that this one
account, by a biased disciple of Jesus, is reliable enough that we can know an Old Testament
prophecy was fulfilled is, again, to assume the complete historical accuracy of the New
Testament documents. At the very least, Dr. Ross will have to produce contemporary records
to corroborate Matthew's account. I don't think he will be able to do that, although he said in
his article that "Bible writers *and secular historians* both record thirty pieces of silver as the
sum paid to Judas Iscariot for betraying Jesus." Now he may have meant by this that secular
historians have uncritically accepted Matthew's word that this happened and have recorded it
as historical fact, but he won't be able to find any secular historians who recorded it from
nonbibical sources such as Roman archives or temple records. *It is an unverifiable story, and
that is bad news for Dr. Ross's prophecy-fulfillment claim.* An unverifiable event simply
cannot be considered a fulfillment of prophecy.

Dr. Ross's fourth claim of prophecy fulfillment alleges that "King David and the prophet
Zechariah described the Messiah's death in words that perfectly depict that mode of
execution." Although Ross didn't specify any specific statements in Psalm 22 that he
considered prophetic in terms of describing the death of Jesus, he undoubtedly meant verses
16 and 18, which allude to pierced hands and feet and the casting of lots for garments. The
gospels all claim that Jesus was crucified, an act that would have required the piercing of his
hands, and that the Roman soldiers cast lots for his garments ( Matt. 27:35; Mark 15:24; Luke
23:34), but John (19:23-24) specifically said that the casting of lots was done "that the
scripture might be fulfilled that says, `They divided My garments among them, and for My
clothing they cast lots.'" Very obviously, then, John was alleging that the casting of lots for
Jesus's garments was a fulfillment of a prophecy in Psalm 22:18.

Volume 1990 - 2002 Issue


Page 677 of 2049
Skeptical Review Edited by Farrell Till
So is this proof that prophecy was fulfilled when the hands of Jesus were pierced during his
crucifixion and when lots were cast for his garments? Not at all! To so argue is again to
assume the historical accuracy of the New Testament documents. There are absolutely no
contemporary nonbiblical records of the crucifixion of Jesus, so to accept as absolute fact
everything reported in the gospels, which flagrantly admit that they were written for
propaganda purposes to further the belief that Jesus was the son of God (John 20:30), is
certainly an uncritical approach to proving a claim as extraordinary as prophecy fulfillment.
The gospel writers all claim that lots were cast for Jesus's garments, but how do we know that
this actually happened? Dr. Ross can find no nonbiblical contemporary records to confirm
that any such incident as this happened, so to claim a prophecy fulfillment solely on the basis
of what the gospel accounts say about this is to assume that they are unbiased, reliable,
accurate historical records. This claim completely ignores the possibility that disciples of
Jesus, desperately wanting the world to believe he was the son of God, could sit down and
write biographies of Jesus that would deliberately try to leave the impression that certain
events happened in his life that were fulfillments of prophecy. After all, how difficult would it
have been for a writer to see a statement in Psalm 22:18 about lots being cast for the
psalmist's garments and then to put such an incident into the life of the central character in his
book in order to claim prophecy fulfillment? Again, I would say that the odds that this could
happen would be about one in one. Therefore, Dr. Ross's claim that prophecy was fulfilled
when Roman soldiers cast lots for the garments of Jesus fails test #4 in our list of rules
governing valid prophecy fulfillment: he cannot prove that this event even happened.

If Jesus was crucified, then quite probably his hands and feet were pierced, because this was
standard procedure during crucifixion. However, that is no proof that the reference in Psalm
22:16 to pierced hands and feet was a prophecy of the crucifixion of Jesus. Contextually,
there is nothing in this psalm to indicate that the writer intended the statement to be so
understood. An objective reading of the psalm should be enough for any reasonable person to
see that the writer was referring to himself and certain abuses that he was suffering at the
hands of his enemies.

"Many bulls have surrounded Me," the psalmist said in verses 12 and 13. "Strong bulls of
Bashan have encircled Me. They gape at Me with their mouths, like a raging and roaring
lion." Was this some kind of prophecy of the suffering that Jesus would endure or was it a
reference to some personal abuse that the psalmist felt he was experiencing in his present
condition? Not even the overly imaginative mind of the writer of Matthew in his endless quest
for prophecy fulfillments tried to relate this statement to the life of Jesus, yet the gospel
writers took the reference to pierced hands and feet just three verses later and exclaimed,
"Aha, prophecy fulfillment!" What is the rationale for distorting the scriptures so flagrantly?
Well, the answer, of course, is obvious: the gospel writers were desperate to prove that their
man Jesus was the Messiah who had been promised in the Old Testament. Since there really
were no prophecies of a virgin-born, crucified, resurrected Messiah in the Old Testament,
they had to twist and distort to give the appearance that Jesus was the long-awaited one.

Absurdity in the claim that the reference to pierced hands and feet in this psalm was a
prophecy about Jesus becomes even more evident when the obscurity of the statement is
considered. A footnote in many reference Bibles will point out that use of the word *pierce*
in Psalm 22:16 follows the Septuagint, Syriac, and Vulgate versions but that the original word

Volume 1990 - 2002 Issue


Page 678 of 2049
Skeptical Review Edited by Farrell Till
is pointed in the Hebrew Masoretic text to read *lion*, and despite the loss of an important
prophecy-fulfillment text some English translations recognize the uncertainty of the text. The
REB and NEB, for example, render the statement like this: "Hounds are all about me; a band
of ruffians rings me round, and they have bound me hand and foot." The GNB says, "A gang
of evil men is around me; like a pack of dogs they close in on me; they tear at my hands and
feet." This translation has a footnote to point out that the last statement in the Hebrew reads,
"Like a lion, they tear at my hands and feet." Some reference Bibles also have footnotes to
indicate that the latter statement may mean, "They tie my hands and feet," as the REB and
NEB actually translate it.

The point is that the original text is very uncertain in its meaning, and on the basis of the
Septuagint translation of a controversial word, the gospel writers have twisted this statement
into a prophecy of the crucifixion of Jesus. What Dr. Ross is actually claiming, then, is that
the odds are 1 in 10^13 that a controversial, uncertain text in a psalm about the writer's
personal suffering was a prophecy of the crucifixion of Jesus. It is just such nonsense as this
that biblicists must resort to in their desperate search for something--*anything*--to support
their irrational belief that the Bible is the "inspired word of God."

The same objections can be applied to Ross's claim that Psalm 34:20 and Zechariah 12:10
were prophecies of the manner in which Jesus would die. When both passages are examined
in context, it is clear that the writers had contemporary situations in mind and not events far in
the future. Let's look first at Psalm 34:19-20:

Many are the afflictions of the righteous, but Yahweh delivers him out of them all. He
guards all his bones; not one of them is broken."

The key statement in the text is the final sentence: "Not one of them [bones] is broken." The
writer of John claimed that when the Roman soldiers were breaking the legs of the three who
had been crucified, they saw that Jesus was already dead, and so they didn't break his legs.
According to "John," this was done, of course, "that the scripture should be fulfilled, `*Not
one of his bones shall be broken*.'" This is recognized in most footnoted Bibles as a reference
to Psalm 34:20, so, of course, this is enough to convince Dr. Ross that we have an amazing
example of prophecy fulfillment.

Let's notice, however, that the statement about no bones being broken was preceded by an
assurance that Yahweh will deliver the righteous out of all his "many afflictions." So by what
rule of logic would the last part of this text be a prophecy but the first part wouldn't? I would
certainly think that someone "who committed no sin nor was deceit found in His mouth" (1
Peter 2:22) would be a "righteous" person, and I would further think that nailing such a person
to a cross would be an affliction. So if the writer of this psalm was really prophesying about
the Messiah, why wasn't the sinless Jesus delivered from his affliction as the "prophecy"
promised? He wasn't, because nothing in the context of this passage was intended as a
prophecy of a Messiah's death. It is just another example of desperation to find something to
shore up an untenable belief.

Even if we assume that Psalm 34:20 was intended as a prophecy, Dr. Ross cannot prove that it
was fulfilled in the death of Jesus, because there is nothing in contemporary records,

Volume 1990 - 2002 Issue


Page 679 of 2049
Skeptical Review Edited by Farrell Till
including even the other gospel accounts, to corroborate John's claim that the legs of Jesus
were not broken on the cross. As I have already pointed out, it would have been very simple
for a disciple of Jesus to search the Old Testament for statements like Psalm 34:20 and then
write a biography that intentionally included events that never happened just to make it appear
that (1) vague statements in the scriptures were actually prophecies, and (2) the life of Jesus
had fulfilled them. So Dr. Ross is actually claiming in this case that an unverifiable event
fulfilled an unverifiable prophecy.

There is no need to analyze Zechariah 12:10 with the same detail I have applied to the other
alleged prophecies in this section. When the passage is examined in context, the objective
reader will clearly see that the prophet was apocalyptically referring to a contemporary
situation. In verse 9, for example, just before the reference to looking upon "me whom they
have pierced," Yahweh declared that he would "destroy all the nations that come against
Jerusalem." At the time of Jesus's crucifixion, the land of Judah was under foreign
domination, and just a few years later Roman legions devastated the city. Certainly, Yahweh
did not destroy all the the nations that came against Jerusalem at the time that Jesus was
"pierced." How then can any sensible person consider Zechariah 12:10 to be a prophecy of the
crucifixion of Jesus? We are simply seeing desperation theology at work in a futile attempt to
prove the discredited belief that the Bible is "God's Word."

Dr. Ross's other claims of prophecy-fulfillment will be examined in subsequent issues of TSR.
If he wishes to reply to this or any of the other articles, we will publish his response.

"Killed Their Own Prophets": New


Testament Libel of the Jews
Stephen Van Eck
Although originally a reform movement in Judaism, the new religion of Christianity quickly
ran into Jewish opposition. This was understandable, since its pagan-derived innovations
offended their religious sensibilities. As the movement became dominated by Gentiles and the
Jews refused to go along with it, anti-Jewish sentiments started appearing in the Gospels and
epistles. Of all these negative passages, perhaps none has been as damaging historically as 1
Thessalonians 2:14,15: "*For ye also have suffered like this of your countrymen, even as they
have of the Jews; who both killed the Lord Jesus Christ and their own prophets, and have
persecuted us; and they please not God, and are contrary to all men*."

We can only imagine how such a harsh statement affected the mentality of Christendom and
inspired, almost justified, the long history of Jewish oppression. Although I'd been well aware
of the decisive arguments that Jesus had really been executed by the Romans (though with the
acquiescence of the high priests), when it came to the accusation of killing the prophets, I'd
taken it for granted that this was correct. I should have known better than to take anything the

Volume 1990 - 2002 Issue


Page 680 of 2049
Skeptical Review Edited by Farrell Till
New Testament says at face value. The accusation proves to be virtually unsupported by
scriptural evidence, which should not surprise anyone experienced in critical reading of the
scriptures.

At least 34 prophets are mentioned by name in the Bible, besides the occasional obscure
prophetesses. Half have books in their names, some do not, and at least five (Shemiah, Iddo,
Nathan, Gad, and Jehu) have books mentioned (2 Chron. 12:15; 1 Chron. 29:29; 20:34) that
are nowhere to be found, a fact that raises the question of why God would allow his
revelations to faithful prophets to vanish completely. It can't be that they were too
insignificant, since relatively insignificant ones like Obadiah and Haggai were preserved.
Besides, no revelation of God could truly be deemed insignificant.

Of these prophets, no record of their deaths is given for most of them, so there is no scriptural
indication that they were killed. The deaths of a few prophets are mentioned (Moses, Samuel
in [First] 25:1, and Elisha in 2 Kings 13:20) without any indication that their demises were by
anything other than natural causes. One prophet (Elijah in 2 Kings 2:11) did not even die at all
but was assumed bodily into heaven... or so it says. There were instances where false prophets
were put to death, as when Ahab and Zedekiah were roasted to death by Nebuchadnezzar in
Jeremiah 29:21. Presumably, the Babylonian king was doing the Lord's work here, but that's
not what Paul had in mind in 1 Thessalonians. We might also include Jeremiah's death curse
on Hananiah (28:15-17) for the horrendous impiety of giving the people hope in the face of
foreign oppression, but, again, that's not what Paul had in mind either.

No, we need the deaths of "true" prophets, not "false" ones. In my investigation of the
prophet-killing charge, I found only three who actually were killed: John the Baptist, Balaam,
and the obscure Urijah. The Baptist was killed not by the Jews but at the behest of Herodias,
the wife and former sister-in-law of Herod, who took offense at John's denunciation of her. It
is highly unlikely she was a Jew but rather an Edomite.

As far as Balaam is concerned, while Numbers 31:8 records his death at the hands of the
Israelites, it is important to realize two things. First, he was not one of "their" prophets
anyway (although he set the pattern subsequent prophets followed) but was hired by the king
of Moab, whom he double-crossed by refusing to curse Israel. Second, he was judged to be
evil (Numbers 31:16; Rev. 2:14; Jude 11; 2 Peter 2:15), just the sort of prophet Yahweh
would conceivably *want* the Jews to kill, despite his use of Balaam against Moab.

The only fully legitimate prophet I could find who was killed by his own people was Urijah, a
small-time Jeremiah parrot, who was tracked down, dragged back, and killed by King
Jehoiakim himself (Jere. 26:20-23). This was the deed of one Jew and his flunkies and not a
collective act.

Elsewhere in the Old Testament I could find only a vague reference to the Jews' killing their
own prophets: 2 Kings 19:10, 14. These redundant passages make reference to Jewish
apostasy, including that they had "slain thy prophets by the sword," but we should note that
the only prior account of a general prophet-slaying by the sword was that of Elijah, who slew
450 prophets of Baal (1 Kings 18:40). As for prophets of the Lord, Jezebel is casually
mentioned, almost as an afterthought, as having killed an unspecified number of them in 1

Volume 1990 - 2002 Issue


Page 681 of 2049
Skeptical Review Edited by Farrell Till
Kings 18:13. However, Jezebel, as a princess of Sidon (1 Kings 16:31), was a Phoenician, not
a Jew. Any attribution of blame to the children of Israel collectively for the act of their foreign
queen is unjustified, and the subsequent killing of the prophets of Baal may have been a
retaliatory massacre by Elijah. At any rate, he was considered a hero for doing it.

In the final analysis, there is insufficient basis to substantiate the charge that the Jews killed
their own prophets. All we have in the Old Testament is a solitary killing by a solitary king,
and the unsupported indictment in 2 Kings 19. In the New Testament, we have Paul's bald
assertion in 1 Thessalonians, plus Hebrews 11:32, 36-37, which makes the killing of prophets
seem like a common occurrence. The source of the notion in Hebrews is not even scriptural
but derives from later Jewish traditions that Isaiah and Jeremiah had both been killed.
Hebrews 11:37, without even mentioning his name, reflects a folk belief that Isaiah had been
sawed in half. Another tradition had Jeremiah killed (method unknown) by exilic Jews who
found him profoundly annoying, that part of the tradition being highly credible. However,
since both of these traditions are noncanonical, no Bible literalist can accept them as evidence
and must conclude that on the basis of scriptural information alone, Paul's accusation in 1
Thessalonians is unfounded.

Further refutation might be found in Jeremiah 1:8, which seems to indicate that Jeremiah was
under divine protection: "Be not afraid of their faces; for I am with thee to deliver thee, saith
the LORD." It isn't recorded that this protection was withdrawn, which God could not have
done anyway without being a fink. So any notion of Jeremiah's being killed would present a
severe apologetic dilemma: either God is ineffectual or unreliable. Jeremiah 27:11-19, 24
provides additional support against this notion by relating how the false prophets and priests
could not persuade the princes or (most important) the people that Jeremiah should die.

But even if we add Jeremiah to the single name of Urijah, only three prophets killed of a guild
that operated for centuries, shows remarkable restraint, given how incendiary the prophets
strived to be... and murderous as well. Recall the mass killings by Elijah (1 Kings 18:40; 2
Kings 1:10,12) and the deaths of 42 children following Elisha's curse, for a crime tantamount
to yelling, "Hey, Baldo!" (2 Kings 2:23-24). Why this vile and unwarranted overreaction did
not lead to the elimination of Elisha by a rightfully aggrieved parent is one of the Bible's
unremarked miracles. That is, if this bizarre moral lesson for children ever actually occurred,
which sane minds will strongly doubt.

Investigating Paul's charge led to an unexpected nugget of some interest. When Jesus (Luke
11:15) referred to murder victims from A to Z (Abel to Zechariah), Matthew (23:35)--or even
Jesus himself--confused the latter with the prophet Zechariah, referring to him as "the son of
Barachiah," when the Zechariah who was killed was the son of Hehoiada (2 Chron. 24:20-22),
and not the Old Testament prophet. This is yet another scriptural discrepancy that the
inerrantists will undergo extreme contortions trying to explain away.

No doubt they will also devise some imaginative explanation to show that Paul was right
when he said that the Jews had killed their own prophets, but they won't find any support for
the claim in the Jewish scriptures.

(Stephen Van Eck's address is Route One, Box 62, Rushville, PA 18839-9702.)

Volume 1990 - 2002 Issue


Page 682 of 2049
Skeptical Review Edited by Farrell Till

Another Preacher Who Never Ceases to Be


Amazed
Farrell Till
If you contend with Bible fundamentalists very long, sooner or later you will hear one of them
pooh-pooh your examples of biblical discrepancies by saying something like, "I never cease
to be amazed at the extremes that skeptics will go to find errancy in God's word." An article
that I published on page 8 of the Summer 1992 edition of *The Skeptical Review* has elicited
such a reaction from Jerry McDonald, a Church-of-Christ preacher whose articles in defense
of biblical inerrancy appeared in some of the early issues of TSR. To put his comment in
perspective, we will need to review my original article, which was entitled "Squeezing Fifty
Years into Twenty." It was brief enough to quote in its entirety:

No contradictions in the Bible? That's what bibliolaters say, but the facts say
something else. A simple example of discrepancy can be found in 1 Samuel 7:1-2,
which says that the Ark of the covenant was taken to the village of Kiriath-jearim and
kept in the house of Abinadab for twenty years. Prior to its being taken to this place, it
was captured in battle by the Philistines, who afterwards began to suffer all sorts of
misfortunes, ranging from the mysterious destruction of a temple idol of their favorite
god Dagon to painful tumors that the people were afflicted with (1 Sam. 5).
To rid themselves of the ark, which they thought was the source of their problems, the
Philistines accepted the counsel of their priests and diviners who had advised them to
send the ark away on a cart pulled by two undriven milch cows that had never been
harnessed to a yoke (1 Sam. 6:1-9). The wisdom of the priests and diviners was that if
the cows took the route to Bethshemesh, this would be an omen that the evil in their
midst had been caused by the presence of the ark. The cows did indeed pull the cart to
Bethshemesh, where the townsmen rejoiced when they saw the ark (v:10-13).
Unfortunately, some of them looked inside the ark--an absolute no no--and Yahweh,
in typical fashion, struck them with a "great slaughter" that killed 50,000 men (v:19).
Understandably frightened by this disaster, the survivors sent a message to the
inhabitants of Kiriath-jearim asking them to take custody of the ark. Men were sent to
fetch the ark, and when it arrived in Kiriath-jearim, it was taken to the house of
"Abinadab on the hill" (7:1-2), where it remained for twenty years. Second Samuel
6:1-11 relates how that king David gathered 30,000 "chosen men of Israel" to go to the
house of Abinadab on the hill and transport the ark to Jerusalem. On the way back,
Uzzah, one of the drivers of the cart, touched the ark to steady it when the oxen
stumbled, and Yahweh struck him dead (vv:6-7). Uzzah was a son of Abinadab, who
had conscientiously cared for the ark for twenty years, but, of course, none of this
mattered to Yahweh. Uzzah touched the ark, and apparently because he wasn't a
Kohathite of the priestly tribe of Levi (Num. 3:27-32,38), Yahweh instantly
dispatched him to the nether world for touching a sacred object. Good intentions just

Volume 1990 - 2002 Issue


Page 683 of 2049
Skeptical Review Edited by Farrell Till
never seemed to matter to the petulant Yahweh, but that is another story for another
time.
The point of this article is that all this may make for a quaint little tale, but it is a tale
with a serious discrepancy in it. The ark was captured by the Philistines in 1 Samuel 4,
well before Saul was made king of Israel in chapter 10. It stayed at the house of
Abinadab for twenty years until it was trans-ported to Jerusalem by David, who was
Saul's successor to the throne. Yet we are told that Saul reigned as king for 40 years
(Acts 13:21).
Just how did this happen? How could 40 years pass throughout Israel in every place
except Abinadab's house on the hill at Kiriath-jearim, where somehow only 20 years
went by? On closer scrutiny, the problem is even worse. David brought the ark to
Jerusalem only after he had captured the city from the Jebusites, who had maintained
control all through Saul's reign. Thus, David had to reign in Hebron for seven years
and six months (2 Sam. 5:5) until Jerusalem was captured; then he transported the ark
from Abinadab's house to the new capital of Israel. So if, as I have noted, at least some
time passed after the ark was captured before Saul was made king, and then if Saul
reigned for 40 years, and then if David reigned for seven and a half years before the
ark was removed from Abinadab's house and taken to Jerusalem, we must be talking
about a span of time equaling almost fifty years. How could it possibly be true, then,
that the ark "abode in Kiriath-jearim" for twenty years ((1 Sam. 7:2)?"

The opportunity to write a response to this article, along with a promise to publish it
simultaneously without editorial comment was offered to three inerrancy spokesmen. None
accepted the offer. Mr. McDonald publishes an on-again, off-again "quarterly" paper that he
calls Challenge, and whenever he puts out an issue, which is by no means done on a quarterly
basis, his favorite pastime is attacking me and The Skeptical Review. In the Summer 1994
issue, which I received almost a year late, McDonald published a reply to my article quoted
above. I will also quote his article in its entirety and then respond to the ridiculous
explanation that he proposed as the solution to the discrepancy that I identified. I am
reprinting the article exactly as he published it, so the spelling, grammar, and punctuation
mistakes are his. I will insert an occasional [*sic*] to emphasize that the mistakes are
McDonald's and not mine.

"I never cease to be amazed at the lengths to which skeptics and atheists will go in
order to prove an untenable position. The article that you have just read from Mr. Till
is just one of the many absurdities that these people commit in their attempts at
sustaining their position; that the Bible is not inspired by God. Those who have had
dealings with Mr. Till, in the past, are well aware of his ability to read at scripture
rather than reading scripture. His article "Squeezing Fifty Years Into Twenty" is but
one example of that which I speak.
Mr. Till informs us that there is a problem with 1 Samuel 7:1,2 because he says that
the scripture says that the ark of the covenant dwelt in the house of Abinadab for
twenty years from the time that the men of Kirjath-jearim took it from Bethshemesh
until the time that David sent men to bring it to Jerusalem.
His argument is this: The men of Kirjath-jearim took the ark from Bethshemesh and
immediately took it to the house of Abinadab, who dwelt in Kirjath-jearim, on the hill.
This of course was before Saul was chosen as king. It remained there throughout

Volume 1990 - 2002 Issue


Page 684 of 2049
Skeptical Review Edited by Farrell Till
Saul's reign and continued to remain there for seven and one half years of David's
reign, which amounted to at least 47 and 1/2 years. Then he tells us that while the ark
dwelt in Abinadab's house that, "somehow only 20 years went by", according to the
Bible.
Mr. Till did not consider the overall context of the subject under consideration, but
this is nothing new for him. Furthermore, he did not carefully read that which he did
consider. He tells us that "(m)en were sent to fetch the ark, and when it arrived in
Kiriath-jearim, it was taken to the house of `Abinadab on the hill' (7:1-2). Let us look
at the errors that Mr. Till made in his article concerning this passage.
The Bible nowhere says that the ark of the Covenant spent "twenty years at the house
of Abinadab." It simply says: "*And it came to pass, while the ark abode in Kirjath-
jearim, that the time was long; for it was twenty years: and all the house of Israel
lamented after the Lord"* (7:2).
Mr. Till assumed that because the scriptures said, "*And the men of Kirjath-jearim,
came and fetched up the ark of the Lord, and brought it into the house of Abinadab in
the hill, and sanctified Eleazar his son to keep the ark of the Lord"* (v:1), that
Abinadab lived in Kirjath-jearim. However, (2 Samuel 6:4,5 tells us that, "*they set
the ark of God upon a new cart, and brought it out of the house of Abinadab that was
in Gibeah... (a)nd they brought it out of the house of Abinadab which was at
Gibeah..."*. From this we learn that Abinadab did not live in Kirjath-jearim, but rather
in Gibeah. Nowhere does the Bible say that Abinadab's house was "on the hill" in
Kirjath-jearim. It says that his house was *in the hill.* The word "Gibeah" means "the
hill." Thus when it said that Abinadab's house was "in the hill" it meant that
Abinadab's house was "in Gibeah."
Now Till might argue that Gibeah was sort of a suburb of Kirjath-jearim or visa
[*sic*] versa. However the maps show us that Kirjath-jearim was on the northern
boundary of Judah and on the western and southern boundaries of Benjamin. Also we
find three Gibeah's [*sic*] mentioned in the Old Testament. One was located in the
mountain district of Judah. One was a city of Benjamin's territory, which was the
birthplace of king Saul. And one was a city of the territory of Ephraim. We don't know
for sure, which one Abinadab lived in, but it was likely the city in the territory of
Benjamin. It was probably close to the city of Kirjath-jearim. However, the two cities
were not one and the same.
Saul's birthplace (probably where Abinadab lived) was four miles north of Jerusalem
while Kirjath-jearim was west of Gibeah. They were in close proximity to one
another, but they were two different cities. Apparently the ark was taken to Kirjath-
jearim where it spent 20 years before being taken to Gibeah where it spent the next 37
[*sic*] and 1/2 years.
The Bible doesn't state that the ark spent 20 years in the city of Gibeah or in the house
of Abinadab in 1 Samuel 7:2, but it states that it spent 20 years in Kirjath-jearim. The
mistake Farrell made was in mis-reading [*sic*] verses 1 & 2. Verse one does not
state that upon arrival at Kirjath-jearim they took the ark to the house of Abinadab. It
just states that the men of Kirjath-jearim fetched the ark, and brought it to the house of
Abinadab. The word "and" there does not denote any time span between the fetching
the ark [*sic*] and bringing it to the house of Abinadab.
Now if we only had verse 1 to give us the information on this, we might rightfully
assume that there was no time lapse between the fetching of the ark and the bringing it
to the house of Abinadab. However, verse 2 states that the ark spent twenty years in

Volume 1990 - 2002 Issue


Page 685 of 2049
Skeptical Review Edited by Farrell Till
Kirjath-jearim. Thus the proper interpretation should be as follows: The men of
Kirjath-jearim came, and fetched up the ark of the Lord, and after it had spent twenty
years in Kirjath-jearim, they brought it into the house of Abinadab in the hill, (in
Gibeah) and sanctified Eleazar his son to keep it.
Now there is not a thing in the world wrong with that interpretation (except for the fact
that it doesn't fit Mr. Till's conclusion that the Bible is not inspired by God). When the
overall context is considered one can see that the men of Kirjath-jearim went to
Bethshemesh and took the ark and brought it to Kirjath-jearim where it spent the next
20 years. Then they brought it to the house of Abinadab in Gibeah where it remained
until David, some 37 [*sic*] and 1/2 years later, sent men to bring it to Jerusalem.
Contradiction? Where?
I have said it before, and I will say it again: If Mr. Till spent half as much time trying
to harmonize these accounts as he spends trying to find contradictions, he would find
far fewer places to complain about. It's not a matter of there being contradictions in the
word of God, it is merely a matter of heart and attitude. If one wants to find
contradictions in the Bible, that is exactly what he will find (2 Thess. 2:10- 12), but
that does not mean that the Bible actually contradicts itself.
I have shown you how simple it is to answer Mr. Till's objections. He boasts of
sending his article to three inerrancy spokesmen and offered them the opportunity to
write a response without his editorial, and none of them accepted the offer. Well, I
don't know who he sent it to, but one thing is for certain, he didn't send it to me. I hope
and pray that you will find the truth regarding this and other subjects which fall under
attack by the atheists and skeptics of the world. "
In response to McDonald's claim that he will publish any of my replies to his article, I sent
him the following article.
"In replying to my article "Squeezing Fifty Years into Twenty" (*Challenge*, Summer
1994, pp. 5-6), Editor Jerry McDonald said, "I never cease to be amazed at the lengths
to which skeptics and atheists will go in order to prove an untenable position." Well,
perhaps he should take a good hard look at the ridiculous extremes that he resorts to in
his effort to defend Bible inerrancy. If there was ever a position that is completely
untenable, it is the doctrine of biblical inerrancy.
The thrust of McDonald's response to my article was that Abinadab who was given
custody of the ark of the covenant did not live on a hill in Kirjath- jearim but in the
town of Gibeah. His "evidence" consisted of two points: (1) Second Samuel 6:3 (KJV)
states that David brought the ark "out of the house of Abinadab that was in Gibeah"
and (2) the word *gibeah* in Hebrew means "the hill." Hence, he reasoned that 1
Samuel 7:1, which states that the men of Kirjath-jearim brought the ark to the house of
Abinadab "on the hill," didn't mean that they took the ark to Abinadab's house on a hill
in Kirjath-jearim but to Abinadab's house that was in Gibeah. In other words,
McDonald is arguing that "on the hill" was mistranslated in 1 Samuel 7:1; it should
have been translated to read that the men of Kirjath-jearim took the ark to the house of
Abinadab *in Gibeah.*
One wonders, of course, why McDonald doesn't recognize the possibility that
mistranslation in the KJV version occurred in 2 Samuel 7:3, where it says that the
house of Abinadab, from which David took the ark, was "in Gibeah." Since *gibeah*
in Hebrew meant "the hill," why isn't it possible that the KJV translators erred in
saying that Abinadab's house was "in Gibeah" rather than "on the hill"?

Volume 1990 - 2002 Issue


Page 686 of 2049
Skeptical Review Edited by Farrell Till
The evidence indicates that this is exactly what happened. I have checked the ASV,
NIV, RSV, NRSV, JB, REB, NEB, NAB, NCV, GNB, NWT, the Amplified Bible, the
New Berkeley Version, Moffatt's, Young's Literal Translation, Hendrickson's
Interlinear Bible, the Septuagint, Brenton's Translation of the Septuagint--all of these
versions render the location of Abinadab's house in 2 Samuel 6:3 as "on the hill" or
equivalent. Even the NKJV says, "So they set the ark of God on a new cart, and
brought it out of the house of Abinadab, which was on the hill." Obviously, Bible
scholars don't agree with McDonald's far-fetched theory of mistranslation in 1 Samuel
7:1. Instead, they recognize that mistranslation in the KJV occurred in 2 Samuel 6:3,
not ( 1 Samuel 7:1. Abinadab's house wasn't in the town named Gibeah (the hill) but it
was located on the hill in Kirjath-jearim.
I won't play McDonald's game that consists of speculating without offering proof, so
let's now look at the textual evidence that supports my claim and discredits
McDonald's. Let's notice first of all that 2 Samuel 7:1 says that David "arose and went
with all the people who were with him from *Baale Judah* to bring up from THERE
the ark of God." Now why did David go with all of the people from Baale Judah to
bring from THERE the ark of God if the ark of God was in Gibeah? The answer is
simple: the ark was at Kirjath-jearim, and Baale Judah was another name for Kirjath-
jearim. Let's just let McDonald's inerrant word of God prove that I am right about this.
Joshua 15:9 refers to the town of Baalah and then adds parenthetically, "The same is
Kiriath-jearim."(Verse 60 of this same chapter refers to the town as Kiriath-baal and
then adds parenthetically, "The same is Kiriath-jearim." The same name and the same
parenthetical explanation are repeated in Joshua 18:14-15. *Eerdmans Bible
Dictionary* gives the following definition of Kiriath- jearim:

"A city on the border of Judah and Benjamin, near where those territories adjoined
Dan (KJV "Kirjath-jearim"). It was reckoned among the possessions of Judah ( Josh.
15:9, 60; 18:14-15), although at verse 28 some versions assign it to Benjamin.
Probably originally known as Kiriath-baal (Josh. 15:60; 18:14), the city is also
referred to as Kiriath (18:28), Kiriath-arim (Ezra 2:25), Baalah (Josh. 15:9; 1 Chr.
13:6, *and Baale-judah* (2 Sam. 6:2) [1987, p. 628].

Later in this same context, *Eerdmans* said that the Philistines returned the ark of the
covenant to the Israelite inhabitants of Kiriath-jearim who enshrined it in the house of
Abinadab under the care of his son Eleazar (1 Sam. 6:21-7:2); "HERE it remained for twenty
years before David transported it to Jerusalem."

Is this enough to convince McDonald that he is wrong? Certainly not. I have had enough
debating experience with the man to know that nothing will budge him from his Bible-
inerrancy position. However, for the benefit of those who are more open-minded let's look at
2 Samuel 6:1-4 alongside the parallel version of David's removal of the ark from the house of
Abinadab in 1 Chronicles. I will quote both passages from McDonald's beloved KJV:

"2 Samuel 6:1-4, Again, David gathered together all the chosen men of Israel, thirty
thousand. And David arose, and went with all the people that were with him from
*Baale of Judah*, to bring up from thence the ark of God, whose name is called by the
name of the LORD of hosts that dwelleth between the cherubims. And they set the ark

Volume 1990 - 2002 Issue


Page 687 of 2049
Skeptical Review Edited by Farrell Till
of God upon a new cart, and brought it out of the house of Abinadab that was in
Gibeah.... "
"1 Chronicles 13:5-7, So David gathered all Israel together, from Shihor of Egypt even
unto the entering of Hemath to bring the ark of God from Kirjath-jearim. And David
went up, and all Israel, to Baalah, that is, to Kirjath-jearim, which belonged to Judah,
to bring up thence the ark of God the LORD, that dwelleth between the cherubims,
whose name is called on it. And they carried the ark of God in a new cart *out of the
house of Abinadab....*"
Now what could be clearer than this? David gathered the people together "to bring the ark of
God from Kirjath-jearim," *exactly where 1 Samuel 7:1-2 says that the ark was taken to*.
After the people were gathered together, they went up to Baalah, which is explicitly identified
as another name for Kirjath-jearim. Why did they go to Kirjath-jearim? Well, they went to
"bring up thence" the ark of God. Does McDonald know what *thence* means? If so, perhaps
he will tell us why they went to Kirjath-jearim to bring the ark "thence," if the ark wasn't in
Kirjath-jearim but in a town named Gibeah.

Let's notice also that this place where David and the people went to get the ark "belonged to
Judah" (v:6), but the town named Gibeah (the hill) belonged to Benjamin. This is where the
rape of the Levite's concubine occurred (Judges 19:12-30), which caused the bitter intertribal
war between Benjamin and the other tribes of Israel (Judges 20). All of the textual evidence
objectively considered shows that McDonald has it all backwards. Mistranslation did not
occur in 1 Samuel 7:1-2 but in 2 Samuel 6:1-4. Abinadab's house was on a hill in the town of
Kirath-jearim, and the men of this town took the ark to Abinadab's house, where it remained
for 20 years, or so we are told in 1 Samuel 7:2. We are supposed to believe that somehow the
ark was taken to Abinadab's house before king Saul began his reign but that the ark wasn't
removed from Abinadab's house until after Saul had reigned for 40 years (Acts 13:21) and
David over seven. If this isn't squeezing 50 years into 20, what would you call it?

McDonald concluded his imaginative attempt to explain away this problem by saying, "Now
there is not a thing in the world wrong with that interpretation (except for the fact that it
doesn't fit Mr. Till's conclusion that the Bible is not inspired by God)," but I beg to differ with
him. There is something seriously wrong with his interpretation besides the fact that it
disagrees with my position that the Bible was not inspired by God. It simply doesn't agree
with the overwhelming textual evidence that I have cited above. If we accept the face- value
meaning of the Bible text, we will have the following facts:

1. The ark of God was taken to Abinadab's house on the hill in Kirjath- jearim (1 Sam.
7:1).
2. This happened before Saul became king (1 Sam. 9). years (1 Sam. 7:2).
4. Saul reigned as king for 40 years (Acts 13:21).
5. After Saul's reign was over and David was king, he took men to Kirjath- jearim "to
bring up 6), Editor Jerry McDonald said, "I never cease to be amazed at the lengths to
which skeptics and atheists will go in order to prove an untenable position." Well,
perhaps he should take a good hard look at the ridiculous extremes that he resorts to in
his effort to defend Bible inerrancy. If there was ever a position that is completely
untenable, it is the doctrine of biblical inerrancy.

Volume 1990 - 2002 Issue


Page 688 of 2049
Skeptical Review Edited by Farrell Till
The thrust of McDonald's response to my article was that Abinadab who was given
custody of the ark of the covenant did not live on a hill in Kirjath- jearim but in the
town of Gibeah. His "evidence" consisted of two points: (1) Second Samuel 6:3 (KJV)
states that David brought the ark "out of the house of Abinadab that was in Gibeah"
and (2) the word *gibeah* in Hebrew means "the hill." Hence, he reasoned that 1
Samuel 7:1, which states that the men of Kirjath-jearim brought the ark to the house of
Abinadab "on the hill," didn't mean that they took the ark to Abinadab's house on a hill
in Kirjath-jearim but to Abinadab's house that was in Gibeah. In other words,
McDonald is arguing that "on the hill" was mistranslated in 1 Samuel 7:1; it should
have been translated to read that the men of Kirjath-jearim took the ark to the house of
Abinadab *in Gibeah.*
One wonders, of course, why McDonald doesn't recognize the possibility that
mistranslation in the KJV version occurred in 2 Samuel 7:3, where it says that the
house of Abinadab, from which David took the ark, was "in Gibeah." Since *gibeah*
in Hebrew meant "the hill," why isn't it possible that the KJV translators erred in
saying that Abinadab's house was "in Gibeah" rather than "on the hill"?
The evidence indicates that this is exactly what happened. I have checked the ASV,
NIV, RSV, NRSV, JB, REB, NEB, NAB, NCV, GNB, NWT, the Amplified Bible, the
New Berkeley Version, Moffatt's, Young's Literal Translation, Hendrickson's
Interlinear Bible, the Septuagint, Brenton's Translation of the Septuagint--all of these
versions render the location of Abinadab's house in 2 Samuel 6:3 as "on the hill" or
equivalent. Even the NKJV says, "So they set the ark of God on a new cart, and
brought it out of the house of Abinadab, which was on the hill." Obviously, Bible
scholars don't agree with McDonald's far-fetched theory of mistranslation in 1 Samuel
7:1. Instead, they recognize that mistranslation in the KJV occurred in 2 Samuel 6:3,
not ( 1 Samuel 7:1. Abinadab's house wasn't in the town named Gibeah (the hill) but it
was located on the hill in Kirjath-jearim.
I won't play McDonald's game that consists of speculating without offering proof, so
let's now look at the textual evidence that supports my claim and discredits
McDonald's. Let's notice first of all that 2 Samuel 7:1 says that David "arose and went
with all the people who were with him from *Baale Judah* to bring up from THERE
the ark of God." Now why did David go with all of the people from Baale Judah to
bring from THERE the ark of God if the ark of God was in Gibeah? The answer is
simple: the ark was at Kirjath-jearim, and Baale Judah was another name for Kirjath-
jearim. Let's just let McDonald's inerrant word of God prove that I am right about this.
Joshua 15:9 refers to the town of Baalah and then adds parenthetically, "The same is
Kiriath-jearim."(Verse 60 of this same chapter refers to the town as Kiriath-baal and
then adds parenthetically, "The same is Kiriath-jearim." The same name and the same
parenthetical explanation are repeated in Joshua 18:14-15. *Eerdmans Bible
Dictionary* gives the following definition of Kiriath- jearim:

"A city on the border of Judah and Benjamin, near where those territories adjoined
Dan (KJV "Kirjath-jearim"). It was reckoned among the possessions of Judah ( Josh.
15:9, 60; 18:14-15), although at verse 28 some versions assign it to Benjamin.
Probably originally known as Kiriath-baal (Josh. 15:60; 18:14), the city is also
referred to as Kiriath (18:28), Kiriath-arim (Ezra 2:25), Baalah (Josh. 15:9; 1 Chr.
13:6, *and Baale-judah* (2 Sam. 6:2) [1987, p. 628].

Volume 1990 - 2002 Issue


Page 689 of 2049
Skeptical Review Edited by Farrell Till
Later in this same context, *Eerdmans* said that the Philistines returned the ark of the
covenant to the Israelite inhabitants of Kiriath-jearim who enshrined it in the house of
Abinadab under the care of his son Eleazar (1 Sam. 6:21-7:2); "HERE it remained for twenty
years before David transported it to Jerusalem."

Is this enough to convince McDonald that he is wrong? Certainly not. I have had enough
debating experience with the man to know that nothing will budge him from his Bible-
inerrancy position. However, for the benefit of those who are more open-minded let's look at
2 Samuel 6:1-4 alongside the parallel version of David's removal of the ark from the house of
Abinadab in 1 Chronicles. I will quote both passages from McDonald's beloved KJV:

"2 Samuel 6:1-4, Again, David gathered together all the chosen men of Israel, thirty
thousand. And David arose, and went with all the people that were with him from
*Baale of Judah*, to bring up from thence the ark of God, whose name is called by the
name of the LORD of hosts that dwelleth between the cherubims. And they set the ark
of God upon a new cart, and brought it out of the house of Abinadab that was in
Gibeah.... "
"1 Chronicles 13:5-7, So David gathered all Israel together, from Shihor of Egypt even
unto the entering of Hemath to bring the ark of God from Kirjath-jearim. And David
went up, and all Israel, to Baalah, that is, to Kirjath-jearim, which belonged to Judah,
to bring up thence the ark of God the LORD, that dwelleth between the cherubims,
whose name is called on it. And they carried the ark of God in a new cart *out of the
house of Abinadab....*"
Now what could be clearer than this? David gathered the people together "to bring the ark of
God from Kirjath-jearim," *exactly where 1 Samuel 7:1-2 says that the ark was taken to*.
After the people were gathered together, they went up to Baalah, which is explicitly identified
as another name for Kirjath-jearim. Why did they go to Kirjath-jearim? Well, they went to
"bring up thence" the ark of God. Does McDonald know what *thence* means? If so, perhaps
he will tell us why they went to Kirjath-jearim to bring the ark "thence," if the ark wasn't in
Kirjath-jearim but in a town named Gibeah.

Let's notice also that this place where David and the people went to get the ark "belonged to
Judah" (v:6), but the town named Gibeah (the hill) belonged to Benjamin. This is where the
rape of the Levite's concubine occurred (Judges 19:12-30), which caused the bitter intertribal
war between Benjamin and the other tribes of Israel (Judges 20). All of the textual evidence
objectively considered shows that McDonald has it all backwards. Mistranslation did not
occur in 1 Samuel 7:1-2 but in 2 Samuel 6:1-4. Abinadab's house was on a hill in the town of
Kirath-jearim, and the men of this town took the ark to Abinadab's house, where it remained
for 20 years, or so we are told in 1 Samuel 7:2. We are supposed to believe that somehow the
ark was taken to Abinadab's house before king Saul began his reign but that the ark wasn't
removed from Abinadab's house until after Saul had reigned for 40 years (Acts 13:21) and
David over seven. If this isn't squeezing 50 years into 20, what would you call it?

McDonald concluded his imaginative attempt to explain away this problem by saying, "Now
there is not a thing in the world wrong with that interpretation (except for the fact that it
doesn't fit Mr. Till's conclusion that the Bible is not inspired by God)," but I beg to differ with
him. There is something seriously wrong with his interpretation besides the fact that it

Volume 1990 - 2002 Issue


Page 690 of 2049
Skeptical Review Edited by Farrell Till
disagrees with my position that the Bible was not inspired by God. It simply doesn't agree
with the overwhelming textual evidence that I have cited above. If we accept the face- value
meaning of the Bible text, we will have the following facts:

1. The ark of God was taken to Abinadab's house on the hill in Kirjath- jearim (1 Sam.
7:1).
2. This happened before Saul became king (1 Sam. 9). years (1 Sam. 7:2).
4. Saul reigned as king for 40 years (Acts 13:21).
5. After Saul's reign was over and David was king, he took men to Kirjath- jearim "to
bring up from THERE the ark of God" (1 Chron. 13:6).
6. David's men "set the ark of God on a new cart, and brought it out of the house of
Abinadab, which was on the hill" (2 Sam. 6:3).
Now as I said before, if this isn't squeezing 50 years into 20, I don't know what you would call
it. McDonald accused me of not reading the Bible carefully enough, but the predicament he is
now in is the result of his own careless research. He dreamed up a what-it-could-have-meant
hypothesis and then rushed it to print in his on-again, off-again paper without first testing it
against all of the relevant passages in the Bible. So we have to wonder just who needs to read
the Bible more carefully. McDonald never tires of saying that if I would "spend half as much
time trying to harmonize these accounts as [I] spend trying to find contradictions, [I] would
find far fewer places to complain about." But look who's talking. If he would spend just half
as much time trying to understand the face-value meaning of the biblical text as he does
looking for how-it-could-have-been scenarios to "explain" away flagrant textual
inconsistencies, he just might begin to see that the Bible is nowhere close to being the
uniquely consistent work of perfect harmony that he constantly claims it is. At any rate, we
will all look forward to watching him try to climb out of the hole he has dug himself into this
time, because if he wants to reply to this article, I will publish it.

Several months ago, McDonald sent me a letter in which he said that he was going to publish
my article and expose the absurdity of my position. However, I have seen no indication that
he has ever published my response. I think the readers of this entire exchange can see why.

(Jerry McDonald's address is 97 Florence Street, Sullivan, MO 63080.)

Christianity and Hindu Influence


Jim Dew
I very much enjoyed reading Stephen Van Eck's "Hare Jesus: Christianity's Hindu Heritage"
(TSR, Summer 1994, pp. 8-9). Twenty years ago, I was a member of a fundamentalist Hindu
sect, the International Society for Krishna Consciousness, aka the Hare Krishnas. I have
continued a critical interest in Hindu literature and philosophy and more recently a similar
interest in Christianity. It seems to me that Van Eck makes a good initial case for the
incorporation of Hindu ideas into early Christian writings. There are very many doctrinal

Volume 1990 - 2002 Issue


Page 691 of 2049
Skeptical Review Edited by Farrell Till
similarities between Christianity and Hinduism that help substantiate the theory of religious
borrowing. A great many of these can be found in Christ and Krishna: The Path of Pure
Devotion (Moundsville WV: Palace Publishing, 1987) written by Kirtananda Swami
Baktipada (who may still be serving time for racketeering). However, we must remember that
deep down, members of these two religions believe that the other religion has got it all wrong!

I do wish Van Eck had better documented the specific verses from the Upanishads that
correspond to the biblical verses he cited. Also, some of Van Eck's scriptural comparisons
seem to stretch the limits of confirmatory reasoning. For example, he compared the Hindu
belief that the soul is smaller than a "mustard seed" to the mustard-seed-of- faith reference in
Matthew 17:20, and he compared the notion of man's not living by bread alone but by "every
word that proceeds from the mouth of God" (Matt. 4:4) to the Hindu claim that "man does not
live by breath alone, but by him in whom is the power of breath."

It should be obvious to those who have knowledge of both religions that the doctrinal and
scriptural differences far outweigh the similarities, even if one compares the modern
"monotheistic" Vaisnava literature of Krishna with Christianity. Consider other comparisons
Van Eck makes. First, he states that Revelation 1:8 ("I am Alpha and Omega, the beginning
and the ending, saith the Lord, which is, and which was, and which is to come, the Almighty")
is extremely similar to the Bhagavad Gita verse 10:21 ("I am the Self, O Gudakesa, seated in
the hearts of all creatures. I am the beginning, the middle, and the end of all beings"). I've
cited more of the verses than did Van Eck to show that although there are similarities, there
are also some notable differences. A major difference not evident in the verses is the Bible's
singular and linear view of time with a definite beginning and end versus the Hindu spiral
view of time that involves a recurring creation and destruction.

Second, consider the comparison Van Eck makes between the Biblical assertions that "God is
no respecterof persons" (Rom. 2:11) and the Bhagavad Gita verse(9:29) where Krishna states,
"I envy no one, nor am I partial to anyone. I am equal to all." Yes, there is a similarity but a
crucial difference lies in Krishna's impartiality to "sarvabhutesu," that is "to all living
entities," not just humans! Hindus believe that all animals and plants have souls containing
the quality but not quantity of God. Certainly, Christianity did not borrow this concept.
Finally, consider Van Eck's comparison between 2 Peter 3:8, "But, beloved, be not ignorant of
this one thing, that one day is with the Lord as a thousand years, and a thousand years as one
day," and the BG 8:17 verse, "By human calculation, a thousand ages ["yuga" or
millenniums] taken together is the duration of Brahma's one day. And such also is the
duration of his night." The difference is that the BG verse doesn't say 1000 years are one day
for God; it says 1000 millenniums are one day!

Nevertheless, it seems reasonable that some transfer of ideas, analogies, and practices
occurred between these cultures. It is clear that the Bible is a conglomeration of myths (see
Randel Helms' Gospel Fictions, Buffalo, NY: Prometheus, 1989), but the source of these
myths is still open to scholarly debate. While it is appealing to attribute Biblical ideas to
Hinduism, it becomes circular only if, as Stephen Van Eck's article promotes, we try to
confirm that the persons who wrote the Bible borrowed some of their ideas from Hinduism.
This possibility deserves further attempts at both confirmation and falsification.

Volume 1990 - 2002 Issue


Page 692 of 2049
Skeptical Review Edited by Farrell Till
Some functional similarities between Judeo-Christian and Eastern religions exist and may
facilitate the involvement of Americans in Eastern religions or "cults." In both traditions,
there is a general opposition to drug and alcohol use, except in religiously prescribed rituals.
They both promote a general attitude that makes sex a sinful focus. Western religious sects
have many similar rituals such as chanting and praying that seem equally effective in turning
off the rational mind in order to attain some "higher state of being."

These religions prescribe the separation of oneself psychologically, if not physically, from
both the material world and the nonbelievers in it, and they prescribe and proscribe certain
thoughts and behaviors.Finally, there are the doctrinal similarities, such as the valid ones
mentioned by Van Eck and Baktipada, as well as many stories that offend common sensibility
and describe God's violation of natural laws and sane human ethics. Thus, if one believes that
some god will solve the problems of the world and that the prevailing religions or
philosophies are ineffective, then one may "discover the Truth" from some imported religious
cult. The result of such conversions is that one's own problems and the problems of the world
are inverted into blessings or magically erased by the unconscious (see Edmund Cohn's The
Mind of the Bible- Believer and John Schumaker's Wings of Illusion: The Origin, Nature and
Future of Paranormal Beliefs, both published by Prometheus Books, 1988 and 1990). One
final note: I'd like to compliment Van Eck on his criticism of the assertions made by some
authors about the similarities between Krishna and Jesus. For example, some have said
Krishna was "crucified," but the Upanishads say he "disappeared" and is waiting for us in a
spiritual abode that appears as boring as heaven. I'd like to add my criticism to those writers
who perpetuate the myth that Krishna was born on December 25th. This is not Krishna's
birthday according to scripture, nor is it when Hindus celebrate Krishna's "appearance." This
is the birthday that Christians stole from the Persian sun god, Mythra, and adopted for Jesus!
It seems that some authors, in their attempt to show a common origin for religious myths,
tenaciously adopt misinformation to make their case. We must remember to be critical and
always check our sources.

(Jim Dew, 138 lakewood Gardens, Madison, WI 53704-4661; e-mail jimdew@macc.wisc.edu)

From The Mailbag


I can't thank you enough for your wonderfully informative and entertaining publication. As a
recovering fundamentalist, I think that you are indeed a breath of fresh air! Keep up the good
work. Enclosed is a check for my renewal.

(Jamie Smith, 3500 East Park Boulevard, Apt. 104, Plano, TX 75074.)

Your response to my letter; (Summer 1995, p. 12) seems to successfully address my objection
to (and misunderstanding of) your article [concerning the raising of Jairus's daughter]. You

Volume 1990 - 2002 Issue


Page 693 of 2049
Skeptical Review Edited by Farrell Till
are right to be confused by my distinction between necessary and sufficient. I misread your
argument.

Thank you for your commendable publication. Although you may think this letter is
superfluous, I do hope you print it. Admitting error is important.

(Richard Trott, 78-A Phelps Avenue, New Brunswick, NJ 08901.)

I received 3 copies of your publication and read the;winter of 95 issue. I hope Mr. Sierichs
realizes that our Bible as it is published today is not in chronological order nor is it in its
original order. See the book The Original Bible Restored by Ernest L. Martin. The numbers
that he gives both ancient and modern are not the real concern in matters of faith, although I
believe the Bible to be correct.

This is a very volatile subject and wars have been fought over less than his [Sierichs']
statement, "People who claim that the Bible is literally true from beginning to end are
shameless liars, openly insulting the intelligence of all of us by defending these absurd
numbers." Of course, Mr. Sierichs' statement is his statement and his right to think this way.
The fact is he has no right to tell me what to believe or not to believe. I do not call people
names with whom I disagree. Shame on him if he would cause anyone to lose faith because of
his views.

In the article "What about Casualty Numbers" by Farrell Till, he states, "Bible
fundamentalists sorely need to read the Bible with their heads and not their hearts." Mr. Till is
wrong in telling me how to read the Bible. I'll read as I please... thank you! You people must
truly feel threatened by what the Bible has to say and cannot understand it without God's help,
so lashing out is your thing.

I am a believer in the Lord and his Son the Savior Jesus Christ. I have believed in and served
the Lord most of my life, raising my son who now is a deacon in his church and preaches on
occasion with his family in attendance. My life is full, pleasant, active and worthwhile. My
church group is small and serves our home area.

Sorry to hear that you folks are so angry because there are believers in the world! Don't send
any more of your papers. I got the gist of your message, and it's sour grapes to me! Bob
Hypes says Christians are "incoherent babblers." I've heard that it takes one to know one.

A believer in Christ now and forever....

(Charlotte Doubravsky, 79 Maryhill Highway, Goldendale, WA 98620.)

EDITOR'S NOTE: Mrs. Doubravsky responded to the ad that we published in Bible Review
last summer. Although we offered only a sample issue, we maintained our policy of giving a
free first-year subscription; hence, we started her subscription with the beginning of the year
and sent her three issues. What she found in the first one offended her, so we dropped her

Volume 1990 - 2002 Issue


Page 694 of 2049
Skeptical Review Edited by Farrell Till
subscription as she requested. We have no desire to send The Skeptical Review to people who
don't intend to read it. There are too many people who will to waste copies on those who have
Mrs. Doubravsky's inflexible attitude.

We have published her letter in order to respond to accusations that indicate she is guilty of
some of the very things she attributes to us. We have charitably corrected several spelling and
grammatical mistakes in her letter.

In her comments about William Sierichs' article ;("Those Amazing Biblical Numbers...,"
Winter 1995, pp. 5-8), she informed Sierichs that the "Bible as it is published today is not in
chronological order," a statement that is puzzling indeed, since nothing in Sierichs' article
pertained to chronology. He simply showed, with ample documentation to support his case,
that the sizes claimed for biblical armies were greatly exaggerated. Mrs. Doubravsky said
nothing to prove that any of Sierichs' arguments were wrong. She simply said that the
numbers given were "not the real concern in matters of faith." Oh? If the Bible reported
incorrect information, that is not a "real concern in matters of faith"? Evidently, she has no
use for the recognized rule of evidence called falsus in uno, falsus in omnibus. Simply stated,
this rule recognizes that when a witness makes an error in one point, those judging his
testimony are entitled to doubt anything else he says whose truth cannot be established
independently of the witness's testimony. Perhaps, then, Mrs. Doubravsky would accept my
invitation to publish any article that she will submit to explain why if biblical numbers
(especially as they were applied to the size of armies) were obviously inflated, we can
reasonably trust anything else that was reported by writers who used exaggerated numbers.

Mrs. Doubravsky contended that she still believes the Bible to be correct, even though she
thinks that numbers are no "real concern in matters of faith." So she seems unable to see the
forest for the trees. If numbers in the Bible have been consistently exaggerated, as I believe
they have in other matters as well as in the issue that Sierichs and I discussed, then how can it
possibly be "correct"? Mrs. Doubravsky would have made a much better impression on me
and other readers, I'm sure, if she had tried to prove that the sizes of biblical armies were
correctly reported. Instead, she was satisfied simply to declare arbitrarily that "the Bible is
correct," and that is an all-too-familiar tactic of would-be biblical apologists. If she cannot
show reasonable proof that Sierichs and I erred in our conclusions about biblical numbers,
then how can she consider the Bible to be a reliable basis for faith? The apostle Paul said, "So
then faith comes by hearing, and hearing by the word of God" (Rom. 10:17), but if a book that
purports to be "the word of God" contains obvious exaggerations that insult my intelligence,
how can I possibly acquire faith from reading it? Perhaps Mrs. Doubravsky has an answer to
this question.

She informed us in no uncertain terms that she will read the Bible as she pleases, and by the
time she said this, I had read far enough to know this without her directly saying it. She thinks
that "(y)ou people must truly feel threatened by what the Bible has to say and cannot
understand it without God's help, so lashing out is your thing." I suspect, however, she has the
situation reversed. She is the one who feels that her precious Bible is being threatened in a
way that she cannot effectively respond to, and so she lashes out at us.

Volume 1990 - 2002 Issue


Page 695 of 2049
Skeptical Review Edited by Farrell Till
This "believer in Christ now and forever" enclosed with her letter a church bulletin that
explained her congregation was founded as a result of the Millerite movement of the past
century, which predicted a specific date (October 22, 1844) for the second coming of Jesus.
The bulletin shamelessly stated that the failed prophecy did not end the movement but instead
gave rise to five different denominations, one of which is the church that Mrs. Doubravsky is
a member of. I think that says quite enough about her rationality, so no further comment on
her letter is necessary.

Concerning the historicity of Jesus, you commented that it was unlikely that a mob should
first welcome Jesus, then kill him. If one mob liked him, and the other mob was controlled by
the power establishment, this could have indeed been possible. Today we have mobs
operating under less duress. In those days, everyone knew that if you disagreed with the
champions too strenuously, then they would just kill you. Today we have mob behavior that is
very similar, yet most of the mob (at least in the USA) do not realize this danger, and believe
themselves to be free. Is the "progress" of civilization merely the craftsmanlike polishing of
this illusion of freedom in a slave colony?

(David Briars, Route 1, Box 45, Craftsbury, VT 05826; e-mail dbriars@ world.std.com)

In "The Historicity of Jesus," you (presumably Farrell, though I didn't see a byline) write:
"Such vast multitudes... welcomed Jesus into the city and then just a short time later crowds
were screaming for Pilate to crucify him. Who can believe it?"

Actually, it's not at all hard to believe. Consider, for example, that George Bush's approval
ratings exceeded 75% right after the Persian Gulf War, yet he pulled in less than 40% of the
votes a year later in the Presidential election of 1992. People can change their minds.
Furthermore, it's perfectly believable that the J-fans shouting hosannas on Palm Sunday
represented an entirely different subset of the population of Jerusalem from those shouting,
"Crucify him!" a few days later. This is no different from a modern-day demagogue speaking
to the cheers, applause, and adulation of an auditorium full of the faithful, then having to run a
gauntlet of placards, boos, and taunts while leaving the place.

In short, I wouldn't use this illustration in an effort to convince anyone that the Bible is
illogical or doesn't have a handle on human nature

(Richard S. Russell, 2642 Kendall Avenue, Madison, WI 53705-3736; e- mail


RSRMadison@aol.com)

EDITOR'S NOTE: The view expressed in the letters above would certainly be meritorius (1)
if more time had passed between the "triumphal entry" of Jesus and (2) if his trial and
crucifixion had occurred in a larger city than 1st-century Jerusalem. As it was, less than one
week separated his entry into Jerusalem from his trial. If the New Testament record is correct,
he was welcomed into the city on the first day by "multitudes" (Matt. 21:7-8) that threw

Volume 1990 - 2002 Issue


Page 696 of 2049
Skeptical Review Edited by Farrell Till
branches into his pathway and went before him shouting, "Hosanna to the son of David!
Blessed is he that comes in the name of the Lord! Hosanna in the highest!" After he had
"come into Jerusalem," all the city was moved and asked, "Who is this?" The multitudes
answered, "This is Jesus, the prophet from Nazareth of Galilee" (v:13).

On day two, Jesus went into the temple, overturned the tables of the money changers, and
drove them out of the temple. The chief priests and the scribes were "moved with indignation"
(Matt. 21:15) and "sought how they might destroy him," but "they feared him, because all the
people were astonished at His teaching" (Mark 11:18). Luke claimed that the chief priests,
scribes, and leaders of the people wanted to destroy him but "were unable to do anything, for
all the people were very attentive to him" (19:47). So clearly the gospels teach that Jesus
enjoyed the support of the general population during those first two days in Jerusalem before
his trial and crucifixion.

Jesus spent the third day teaching in the temple. (This covers two and a half chapters in Mark
and three and a half in Matthew.) After Jesus had finished all of these sayings, he said to his
disciples, "You know that after two days is the Passover and the Son of Man will be delivered
up to be crucified" (Matt. 26:1). That night he was betrayed by Judas and arrested, "and when
morning came, all the chief priests and elders of the people plotted against Jesus to put him to
death" (Matt. 27:1). This would have put the trial of Jesus about five days after his triumphal
entry, so we are supposed to believe that the enemies of Jesus, consistently described as the
chief priests, scribes, and elders (leaders of the people), had spent four days wringing their
hands over the activities of Jesus, unable to move against him because they feared the
multitudes, but suddenly they were able to take him into custody and produce a screaming
mob that demanded his crucifixion. What had happened to bring about the change? The
gospels give no plausible explanation for it, just as they give no plausible explanations for a
lot of things.

This, then, is an entirely different scenario from George Bush's loss of popularity over a
period of more than a year, because the alleged change in public attitude toward Jesus
occurred over a space of only four or five days. We are asked to believe that the chief priests
and leaders of the Jews were afraid to move against Jesus because of his popularity with the
people but that just 4 days later something had happened to remove that obstacle. To suggest
that there were multitudes in Jerusalem who adjulated him but also multitudes that hated him
is an explanation that doesn't take into consideration the size and population of Jerusalem.
The Jerusalem of this time encompassed an area that was only 800 yards wide from east to
west and 1200 yards from north to south, and its population was estimated at about 25,000. If
the chief priests and scribes feared the reaction of the multitudes if they tried to take action
against Jesus the day after his triumphal entry, we can hardly imagine how they could have
kept their plot against Jesus from these multitudes in such a relatively small area, even if we
can imagine their succeeding in stirring up another mob against Jesus in a city of only 25 or
30 thousand where "the people all hung upon him, listening" (Luke 19:48). It is more
reasonable to believe that this is just another case of inconsistency in the gospel accounts of
biased Christians overzealously exaggerating the exploits of their "Messiah."

Volume 1990 - 2002 Issue


Page 697 of 2049
Skeptical Review Edited by Farrell Till
Please send me all your back issues up to the spring of 95--at least 19 issues. I'm send $20 to
cover them ($1 extra in case). I don't expect any credit, if there is any. That's okay; your
publications are worth every penny and more for the truth and honest approach to your
articles. It's time for us to come out of the dark ages.

(Barbara Knapp, 25527 Via Dolarita, Santa Clarita, CA 91355-2827.)

In reference to your article, "The Testimony of Mara Bar-Serapion," I have historical


information that makes his letter even more inaccurate. You state that no historical record
existed of a plague in Athens, but actually such a calamity did strike them, in 430 B. C., on
the heels of the Peloponnesian War, as detailed by Thucydides. Unfortunately, Socrates was
executed in 399 B. C., after the plague had irreparably decimated Athenian society. Bar-
Serapion, in his ignorance, was confusing cause and effect. It has been argued that a major
motivation for killing Socrates was the plague and its aftermath. The more superstitious could
not abide his perceived impiety toward the traditional gods, and were fearful of what they
might do next to their city if he were allowed to continue. Socrates was a sacrifice, then, who
died for their "sins" of irrationality.

Moving to another matter, I have a problem with John Glue's letter in the same issue. He
passes on some of the same bogus derivations found in questionable sources as Godfrey
Higgins and Kersey Graves, when names like Christ and Jesus have no demonstrable
linguistic connection to the exotic sources some attribute. Christ comes from the Greek
translation of the Hebrew meshiach (messiah), both meaning "the anointed one." Jesus is
nothing more than a Greek transliteration of Yeshua (English mutation, Joshua), meaning
savior, which may suggest the fictional nature of the character (among many others with
symbolic names). Far from being the source of the term Christ, Krishna means "dark" and
reveals the subtext of the Mahabharata: resistance of the Aryan invasion by the darker
indigenous element of India. The fact that he is the second person of the Trimurti in human
incarnation is what may be significant.

We do not aid our cause when we seize upon anything that appeals to us as damaging
religion. We ought to utilize the most scrupulous standards lest they discredit us, and not
worry that we're missing an opportunity. Our case is solid enough to be content with nothing
less.

(Stephen Van Eck, Route 1, Box 62, Rushville, PA 18839.)

EDITOR'S NOTE: There was indeed a plague in Athens that (as Van Eck points out)
occurred *before* Socrates was executed, which certainly would disqualify it for the plague
that Bar-Serapion declared was punishment for the Athenians having "put Socrates to death."
I have checked but have not found any plague that struck Athens after Socrates' death, so as
Van Eck suggested, Bar-Serapion was probably confused in his chronology, which doesn't do
much for the fundamentalist Christian claim that his letter was testimony to the historicity of
Jesus. Let's notice also that Bar-Serapion said that plague *and* famine came upon the

Volume 1990 - 2002 Issue


Page 698 of 2049
Skeptical Review Edited by Farrell Till
Athenians because of their execution of Socrates. Does anyone know of any such famine? I
have not been able to find any record of it.

We also have to wonder what Bar-Serapion meant about the island of Samos having been
"covered with sand" as punishment for their having "burned" Pythagoras. Of course, we know
that Pythagoras wasn't burned by the "men of Samos"; he died in Italy. I wonder, though, if
anyone has any idea what the allusion to the sand that covered Samos might be. I have not
found any record of a calamity that meets that description.

As for the similarity in the names Christ and Krisha, my understanding is that Van Eck is
right. They have different origins. However, isn't it true that the popular spelling of Krishna
used to be Chrishna? Does anyone know if there was any Christian effort expended to change
the spelling to its present form?

After 26 years of bondage to religious ideas (Pentecostalism, charisma, fire, end times, and
circumcision), I'm free. Thank you for your free offer per the ad in Humanist.

(Bob Homac, 2808 Old St. Augustine Road, Tallahassee, FL 32301-5122.)

EDITOR'S NOTE: This is the kind of letter we love to get.

Enclosed please find a disk that needs to be updated with the latest editions of The Skeptical
Review. I am quite pleased with the publication, and have distributed the back issues not just
around my office but also around California on various bulletin boards.

My own BBS, Dante's Inferno, now has thousands of freethought text files available to be
downloaded free of charge, and your publication has been among the most popular. The
phone number is 619-540-BURN, and anyone may log on quickly to access the files by using
my GUEST account, which requires no password. I have a 28.8 USR V.everything, so
anybody should be able to connect with no problems.

I have run across a few books that I feel would come in handy for those of your readers who
enjoy debating or just reading up on the subject of reality rather than fantasy. The first one,
The Encyclopedia of Biblical Errancy (ISBN 0-87975-926-7), is by C. Dennis McKinsey,
editor of the fine monthly publication Biblical Errancy. I have found this to be very well
organized and extremely useful when debating inerrantists. Also, Atheism: The Case Against
God (ISBN 0-87975-124-X), by George Smith, while a little more technical from the
philosophical standpoint, is well worth reading. Both of them are available from Prometheus
Books (1- 800-421-0351). A third book is Ain't Nobody's Business if You Do (ISBN 0-
931580-53-6) by Peter McWilliams, published by Prelude Press. This covers a much broader
range of topics than the previous two but really looks at the social impact religion has had on
our society.

Volume 1990 - 2002 Issue


Page 699 of 2049
Skeptical Review Edited by Farrell Till
On another subject, you mentioned in volume six, number two, that Jerry Moffitt would
consider debating again if he can be guaranteed an audience of skeptics. I can pretty much
guarantee you an audience of at least 30, probably as many as 50, atheists and skeptics
anytime you wish. I would be most interested in helping arrange a debate if indeed Moffitt is
willing to do so. Please feel free to contact me with any questions.

I would also like to rent the Till-Dobbs Debate, the Hovind-Till Debate, and obtain a copy of
the transcript of the Geisler-Till Debate.

(Dante Ridley, P. O. Box 2415, La Mesa, CA 91943- 2415, phone (619) 670-7062, e-mail
dante@daffy.cgl.com)

EDITOR'S NOTE: Mr. Ridley's offer to arrange a debate that would attract an audience of
skeptics has been passed along to Jerry Moffitt. Whether the debate occurs will now depend
upon Moffitt's decision.

For those interested in the books Mr. Ridley recommended, the address of Prometheus is 59
John Glenn Drive, Buffalo, NY 14228-2197. They can also be obtained from H. H. Waldo,
Bookseller, P. O. Box 350, Rockton, IL 61072.

It was by chance that I saw a notice in The Seattle Times newspaper regarding your debate on
the resurrection at Seattle-Pacific University. While there, I signed up for a free one-year
subscription to TSR. I have received the 3 issues of 1995 so far, thank you, and I feel they are
so well done that I would like all the back issues. Enclosed is my check for the 20 back issues
and Prophecies: Imaginary and Unfulfilled.

(Lando J. Derksen, 4030 Williams Avenue W, Seattle, WA 98199.)

Please send me a year's subscription. If possible, I'd also love to see whatever back issues are
available. I have enclosed a check to cover costs.

I am 24 years old, and I was raised in the Assemblies of God. Several years ago, I debated
Messianic prophecy with a Jewish friend (handle=Svee) whom I met over a computer
network called Q-link (for Commodore users). It lasted months and drew quite an audience.
Besides arguing for the fulfillment of prophecy, I also tried to demonstrate how the OT
required the Messiah to be the divine son of God in a trinitarian sense.

Well, four years ago, I progressed to being a Unitarian in the classical sense--Bible believing
but asserting Jesus was a man and did not preexist. I still believe this is the proper way to
understand the NT.

Two years ago, I dropped Christianity entirely to become an atheist, secular-humanist, and
libertarian. Every day that passes makes me more confident my decision was correct.

Volume 1990 - 2002 Issue


Page 700 of 2049
Skeptical Review Edited by Farrell Till
Evolution is true; God is not necessary for the world to exist; the idea of God is absurd;
religion is itself a sociological phenomenon; Genesis is a compilation of stories with
divergent points to make; the gospels contain historical mistakes, scientific errors, and
contradictions.

I won that Q-link debate, because I was willing to learn from my opponent and learn I was
wrong. Through that debate, I got the foolish to realize the truth- -even if it took years to sink
in. If I knew where Svee was now, I'd congratulate him too.

(Wayne VanWeerthuizen, P. O. Box 206, Centralia, WA 98531- 0206.)

EDITOR'S NOTE: Need I say how much we enjoy receiving letters like this? The way that
Mr. VanWeerthuizen learned the truth about the Bible points to what I have said many times
in the past. The internet is bad news for Christianity. So much discussion is taking place on
the various newsgroups that it is bound to have long-term effects on traditional religious
beliefs, not just here but all over the world, and the nice thing about it is that the free
exchange of information is something that the religious establishments can no longer
suppress. Any government that keeps personal computers from its citizens will be plunged so
far back into a dark-age mentality that it will not be able to survive in the modern world. So
Christians who have computers will now have access to information that was kept from the
sheep in the past.

When I was still a child, I believed in God simply because my parents told me so. As a soldier
in WWII, I seriously began to doubt the existence of God. Later, after reading many books on
the subject of God (including the Bible) and after much contemplation I came to the
conclusion that the Judeo-Christian God is manmade. The fable of Jesus deserves to be
included in the world's best fairy tales.

I enclose a two-year subscription to your excellent magazine. "Christianity" needs


enlightenment more than ever.

(J. Jaschimow, R2, C-12B, Lansdowne, BC, Canada V0E 1B0.)

EDITOR'S NOTE: Mr. Jaschimow's story is a familiar one. He started reading and.... Any
Christians on the subscription list who may be interested in seeing what Jaschimow and others
mean when they say that reading books about the Bible changed their lives, I offer an
experiment that you can use to test whether reading about your religion will in any way
change your attitude. First, you should read the Bible (many Christians don't) and develop at
least a fair understanding of what is in it. Then take your time and read these books: The
Seven Mighty Blows to Traditional Beliefs by A. J. Mattill, Jr., The Myth of the Resurrection
and The Forgery of the Old Testament by Joseph McCabe, Is It God's Word and Forgery in
Christianity by Joseph Wheless, Who Wrote the Bible? by Richard Elliot Friedman, The Story
of Christian Origins by Martin A. Larsen, Out of the Desert by William H. Stiebing, Jr.,
Gospel Fictionsby Randel Helms, The Trial of Jesus of Nazareth by S. G. F. Brandon, The
Case Against Christianity by Michael Martin, and Beyond Born Again by Robert Price. I

Volume 1990 - 2002 Issue


Page 701 of 2049
Skeptical Review Edited by Farrell Till
could extend the list far beyond this, but these would be enough to keep you busy for a year if
you read them as carefully as you should. Such books as these will give the serious biblical
reseacher a perspective of the Bible that is very different from what people hear in churches.
The reason why so many people still cling to old, discredited religious views is that (1) they
don't even read the Bible, and (2) if they do other religious studies, they rely on books usually
published in Grand Rapids, Michigan, or by their own churches.

Your publication is a gust of fresh air cleaning away the cobwebs of fundamentalism. I
greatly appreciate your civility and humor comingling with sweet reason. A former Wisconsin
Synod Lutheran, parochially educated from kindergarten through 12th grade, I've labored for
the last 25 years to free myself from the "truths" of such indoctrination. It has often been a
lonely struggle, but spiritually speaking, there's simply no going home again.

Please add my name to your subscription roster, and enclosed is a check for back issues. I
relish the prospect of catching up on my reading as I continue to put away childish things.

(Kathi Houghton, 44 Peninsula Trail, Traverse City, MI 49686.)

EDITOR'S NOTE: Ms. Houghton's allusion to Thomas Wolf's famous novel You Can't Go
Home Again is an effective way to express the transformation that occurs when one abandons
religious superstition. So many well meaning people have expressed hope that I will one day
"come back"; many tell me, of course, that they are praying for me. What they don't
understand is the simple truth that Ms. Houghton expressed in her letter. When one frees his
mind from the silliness and absurdity of Bible belief, he simply cannot "go home again." It
just isn't going to happen.

Thank you so much for the sample issue of The Skeptical Review, which I received and read
today. It's excellent and "truth revealing." If my first year is free, then I'm sending you a check
for the 2nd- and 3rd-year subscriptions. I recently read The Book Your Church Doesn't Want
You to Read and got your address from it.

I'm grateful to men like you and Dennis McKinsey for your dedication and service to
enlighten mankind, for you're very much needed and appreciated. If it wasn't for freethinkers,
we'd never learn anything to remove the shackles of bondage or slavery of the "mind
polluters" or "mind corruptors." The older I become, the more I read, and the more I learn, the
less I want to be called a "Christian" and the more proud I become of being called an atheist.

(Albert Monette, 60353 Latham Tr., Joshua Tree, CA 92252.)

I have always had a philosophical disagreement with the premise of a religion that has a
reward system that is based on faith and not the true spirit of a person. Frankly, I find the

Volume 1990 - 2002 Issue


Page 702 of 2049
Skeptical Review Edited by Farrell Till
whole notion to be so lacking in true justice that I'm appalled. Thanks to The Skeptical
Review, I now have factual resources at my disposal to help me debate the many Christians
who are a part of my everyday life. You have no idea how much this has benefited me
psychologically.

I was raised in a fundamentalist Christian home and was forced to endure unimaginable
psychological abuse because of the extremely "hardline" Christianity that my parents
enforced. I was constantly reminded that my rebellious nature was "as the sin of witchcraft." I
was brought to endless numbers of "revival" meetings and "crusades." Even now, my parents
dare to say that the blood of other people will be on my hands because I'm willing to share my
skepticism.

I want to arm myself with as much knowledge as you can give me, so I am hereby sending
you a check for Prophecies: Imaginary and Unfulfilled and also the Geisler-Till Debate and
as many back issues as this money can go to. I believe skepticism to be the most humanitarian
point of view. May our children live in a world free of superstitious nonsense.

(David Friesen, 902 South Loop 499, Apt. E-8, Harlingen, TX 78550.)

I just received the '94 issues, and I just want to thank you for responding so quickly after you
found out I didn't receive my back issues. I really appreciate it. I have a lot of respect for your
publication. It frees me from the myths and legends that create fear and guilt, which has
haunted me from those insane "born-again-Christian" days. I wish I could express myself
better in the written word. There is so much more I could say. Thank you for your ability to
express yourself so well.

I was thinking that if God knows the future and wanted to spread the word, why didn't he wait
for the technology we have today--with TV, faxes, etc.? Why do it in an age with limited
means of communication? Now everyone would have a better chance to get all the facts-- like
Thomas. Why did God sacrifice his son (himself) in a primitive age? What difference would it
make, then or 2,000 years later, since time means little to "God"? One day to God is like
1,000 years to us; at least that's what I have heard.

I know you could do a debate on this. I'd love to hear what fundamentalists would say.

(Barbara Knapp, 25527 Via Dolarita, Santa Clarita, CA 91355-2827.)

EDITOR'S NOTE: Ms. Knapp's first paragraph refers to an order of back issues that were
mailed but not received. If this should ever happen to you, please let us know.

I too have often wondered why God would choose to reveal his "word" to a backward,
barbaric people in primitive times. If he felt it was necessary to do this at the particular time
he did, why doesn't he take advantage of the technology that Ms. Knapp referred to and re-
reveal his word in a way that is unequivocal so that he could quell all of the controversy, for
all times, about what was in "the original autographs" and how certain words in the original

Volume 1990 - 2002 Issue


Page 703 of 2049
Skeptical Review Edited by Farrell Till
languages should have been translated. A simultaneous revelation direct from God, recorded
instantly in all of the languages on earth that have computer technologies, would be so
remarkable and unequivocal that it should end all of the controversy. No more disputes about
which book is really his word, which sect is really his "true religion"--why would an
omniscient, omnipotent, omnibenevolent deity, who doesn't want anyone to perish, not want
to do this?

Who will be the first fundamentist to respond by telling us that God chooses to reveal himself
in parables and "dark sayings" so that he can test us and determine who have good and honest
hearts (as if an omniscient deity would have to perform such tests to make this
determination)?

Back Page

Study Aids
The following booklets are available for $3 each, postage paid:
The Laws-Till Debate, a 47-page unfinished debate with James Laws, Jr., a professor
of apologetics at Tennessee Bible College. Although Laws challenged, he quit after
only three manuscript exchanges and has since refused to accept mail from Till.
Copies of correspondence are included in the debate.
Jackson-Till Debate, 50 pages on the issue of biblical inerrancy.
Prophecies: Imaginary and Unfulfilled, an in-depth examination of the most
commonly claimed examples of prophecy fulfillment.

Back Issues
All back issues of The Skeptical Review, from Winter 1990 through the special 5th edition
1995 (25 altogether) are available at $1 per copy. Some issues will be loose-leaf reprints.

Unsolicited Materials
We will be glad to consider unsolicited articles. However, the focus of The Skeptical Review,
which is the doctrine of biblical inerrancy, should be kept in mind before you send any
materials to us. As our circulation grows, we receive more and more manuscripts that do not
relate to our publishing purpose. We cannot consider poetry or other works of fiction, book
reviews, or articles about the existence of God, the political threat from the Christian right, or

Volume 1990 - 2002 Issue


Page 704 of 2049
Skeptical Review Edited by Farrell Till
biblical doctrines that do not in some way relate to the inerrancy issue. Some of our best
articles have come to us unsolicited, so we encourage submissions. We ask only that you keep
our publishing theme in mind.

If you submit an article, you would greatly reduce our work load by including an ASCII copy
on a 3.5 HD disk, or if your computer is on line, you may transmit it to us at
jftill@midwest.net.

Beware
In the last issue of 1995, we published an article about The Stupid Christian Chronicle, a
paper that rather arrogantly challenged skeptics to submit articles on atheistic subjects that
would be considered for publication if accompanied by a $1 reading fee. We now have reason
to suspect the legitimacy of this organization and advise our subscribers to investigate before
making submissions.

Recommended Reading
Dr. A. J. Mattill, Jr., has published a revised edition of his book The Seven Mighty Blows to
Traditional Belief. We highly recommended this book, which can be obtained for $15 from
The Flatwoods Free Press, Route 2, Box 49, Gordo, AL 35466-9517.

Volume 1990 - 2002 Issue


Page 705 of 2049
Skeptical Review Edited by Farrell Till

The Skeptical Review


Volume Seven, Number Two
March/April 1996
Farrell Till, editor

• The Editor Goes to Church


"What A Friend We Have in Jesus" is debunked.

• Fulfilled Prophecy: An Unprovable Claim (2)


Farrell Till continues his rebuttal to Dr. Hugh Ross's article claiming that prophecy
fulfillments prove the divine origin of the Bible.

• What Third-Day Prophecy?


Where in the Old Testament is a prophecy whose face-value meaning was so obvious
that no reasonable person could deny that the prophets were indeed predicting that the
Messiah would rise from the dead?

• Additional Errors Committed by Farrell Till (2)


Marion R. Fox picks apart Farrell Till's biblical errancy arguments.

• What Farrell Really Said


Farrell responds to Marion Fox's accusations with a few challenges of his own.

• New Biblical Archaeology


Louis Rushmore condemns biblical archaeologists because their findings disprove
common bible stories.

• The Old Logic


Farrell wonders why Louis Rushmore doesn't recognize the findings of his own
people. Isn't it time for progress?

Volume 1990 - 2002 Issue


Page 706 of 2049
Skeptical Review Edited by Farrell Till
• From The Mailbag
The letters we receive are truly inspirational.

• The Back Page


Change of address notices and a challenge to our inerrantist readers.

The Editor Goes to Church


by Farrel Till
Yes, it's true. I confess. I recently went to church. A friend, who constantly reminds me that
he is praying for me and seems supremely confident that someday I will see the error of my
way, repent, and return to the fold, asked me to attend his church on the occasion of its special
Friendship-Day Services. In a moment of weakness, pity, or something, I accepted the
invitation, and the following Sunday found me sitting beside him in a pew. Except for the few
times I have had debates scheduled as part of church services, this was my first time to "go to
church" since September 1963 when, after preaching a sermon on the Navajo Reservation in
Arizona, I decided that I could no longer endure the hypocrisy of preaching what I knew I
didn't believe anymore. Well, this wasn't really my first time to go to church since then, but
another occasion when I tried to go can't be counted. On a Sunday perhaps four or five years
ago, I went to the local Church of Christ hoping to meet in person its preacher, whom I had
been corresponding with. When he learned I was in the audience, he stood in the pulpit and
announced that the services would not continue until I had left. So much for seeking and
saving the lost.

At any rate, I recently went to church with my friend, and it was an experience worth telling
about. The sermon was about friendship, a subject the preacher had no doubt selected to fit
the occasion. It began with the reading of a text in 1 Samuel 18:1-5, ((1 After David had
finished talking with Saul, Jonathan became one in spirit with David, and he loved him as
himself.
2
From that day Saul kept David with him and did not let him return to his father's house.
3
And Jonathan made a covenant with David because he loved him as himself.
4
Jonathan took off the robe he was wearing and gave it to David, along with his tunic, and
even his sword, his bow and his belt.
5
Whatever Saul sent him to do, David did it so successfully that Saul gave him a high rank in
the army. This pleased all the people, and Saul's officers as well.)) which relates an incident in
the friendship of David and Jonathan, the son of King Saul. The preacher elaborated on the
depth of the friendship between David and Jonathan and related some of their experiences to
illustrate what true friends are willing to do for each other.

Volume 1990 - 2002 Issue


Page 707 of 2049
Skeptical Review Edited by Farrell Till
Of course, it didn't take a genius to guess where the preacher was going with his sermon topic,
so I reached for a hymnal, checked the index, then opened the book to "What a Friend We
Have in Jesus," and showed it to my friend, who grinned weakly. Sure enough, the preacher
eventually got around to assuring us that we have a friend in Jesus, whose friendship is
greater than any we could ever expect to experience. How can we know this? Well, Jesus
himself told us, "Greater love has no one than this, than to lay down one's life for his friends"
(John 15:13). Jesus laid down his life for us, so what better friendship could we ask for? Of
course, this part of the sermon was spiced with references to the pain and agony that Jesus
suffered on the cross and the great love that God must have had for mankind to allow his only
son to endure such an experience.

From a religious point of view, it was an impressive and emotional sermon, and there was
even one "altar call" before the preacher had finished. I probably was the only person in the
audience who wasn't impressed, and during a friendship luncheon following the services, the
preacher sat with my friend and me, so I had the opportunity to talk to him and explain why I
wasn't impressed.

I pointed out that the entire sermon had been based on anthropomorphic premises that
assumed what is true of people must also be true of God. I used myself as an example and
asked the preacher to imagine a scenario in which he is about to be executed by a despotic
government. If in such a case, I went to the leader of this government and offered myself as a
substitute for the preacher and my offer was accepted, one could truly say that my act would
constitute a remarkable expression of friendship and love. "But what if I knew that I was
eternal and omnipotent," I asked the preacher, "and that my death would be only a temporary
thing and less than three days later, I would be restored to life never to die again. Wouldn't
that take something away from the remarkableness of my gesture on your behalf?" Indeed, if I
knew that I possessed eternalness and omnipotence, it would be rather despicable of me if I
refused to offer myself as a substitute for a friend who was about to be executed. Even so,
"Kill me instead" in such a scenario would not be a noble gesture at all; it would actually be
sort of an obligation that the omnipotent one should feel duty bound to discharge. I suggested
to the preacher that these are ideas that seem to escape gullible pulpit audiences, and I can't
recall that he had any satisfactory response to make to my comments.

I was reminded of that sermon just the day before I sat down to write this article. On the way
to K-Mart, I had remembered to check out the latest message on the yard sign of a church that
always has some simplistic religious platitude posted. You have probably seen these
yourselves, something like, "God Sent His Son to Man to Make Men God's Sons" or such
like. That day the message was, "God's Xmas Gift to Men Was His Son." My first reaction to
the message was surprise that it had taken Christ out of Christmas, and then it reminded me of
the friendship sermon on the day I went to church. Here again was the idea that God's gift of
his only begotten son to die for the sins of mankind was some supremely noble gesture, but I
just can't buy the idea.

I have two sons, and I would never agree to offer either of them as a substitute for anyone
under sentence of death. However, let's just suppose that I were an eternally omniscient and
omnipotent entity, and so I would necessarily know that if I offered one of my sons as a

Volume 1990 - 2002 Issue


Page 708 of 2049
Skeptical Review Edited by Farrell Till
substitute for someone else, his death would be only temporary and three days later he would
be alive again nevermore to die. What would be the big deal about my gesture?

Let's further complicate this scenario by assuming that my son is a chip off the old block who
possesses my same characteristics of eternalness, omniscience, and omnipotence. These
characteristics would necessarily remove any reason for him to be concerned about my
decision to offer him as a substitute in death for others. If he were truly omniscient, then he
would know that he was also eternal and omnipotent. Therefore, he would know that his death
would be merely temporary. He would also know that he was incapable of suffering any real
harm, because omnipotence would not be subject to physical harm. This logical consequence
of omnipotence, in fact, often makes me wonder how those who crucified Jesus managed to
kill him. How could something eternal and omnipotent be killed even temporarily? I suppose
the same inerrantists who tell us that in the nature of deities it is possible for 1+1+1 to equal
one will now tell us something about Jesus's being "wholly God" and "wholly man," and so it
was the "wholly man" part of him that the ordeal of the cross killed, as if it isn't ridiculously
contradictory to talk about something being "wholly" one thing while simultaneously being
"wholly" something else. When I hear such as this, I have to wonder if theologians ever study
logic.

So just what is the big deal that theologians make about the "supreme" sacrifice that God
made for man in offering up his son? In the scenario hypothesized above, there wouldn't be
anything to write home about if I, as an eternal, omniscient, omnipotent person, should offer
up one of my sons if he too possessed the same characteristics. So why get all teary-eyed and
grovel in guilt and shame when we hear preachers wail about the supreme love that God
showed for mankind in sending his son to die for the miserable creatures that we are?

At any rate, I went to church, and all that the experience did was confirm that I had made the
right decision 32 years ago when I walked away from a belief system that couldn't be any
more illogical if someone had deliberately tried to make it so. If Christians want to go to
church and weep over their sins when they hear preachers wailing about an omniscient,
omnipotent deity who for some inexplicable reason prayed feverishly in the Garden of
Gethsemane (while sweating "as it were great drops of blood") to be spared the ordeal of
something that wasn't going to hurt him all that much anyway (and if it hurt him at all, it was
his own fault, because an omnipotent person could anesthetize himself to pain), that's their
privilege. There is no law against superstitious ignorance. As for me, I have better things to
do with my time.

Fulfilled Prophecy: An Unprovable Claim


(2)
by Farrell Till

Volume 1990 - 2002 Issue


Page 709 of 2049
Skeptical Review Edited by Farrell Till
In the previous issue, I began a response to Dr. Hugh Ross's article claiming that prophecy
fulfillments prove the divine origin of the Bible. In continuing my rebuttal of Ross's
individual prophecy claims, I will state again the obvious factors that must be present before a
prophecy fulfillment can be proven:

In order to prove--and I mean *prove,* not just surmise--prophecy fulfillment, one would
have to establish four things:
(1) the claimant of a prophecy fulfillment is properly interpreting whatever text he is
basing his claim on,
(2) the prophecy was made *before* and not after the event that allegedly fulfills the
prophecy,
(3) the prophecy was made not just *before* an event but far enough in advance of it
to make educated guesswork impossible, and
(4) the event that allegedly fulfilled the prophecy did in fact happen (TSR,
January/February 1996, p. 3).

As I also said in my first rebuttal article, when Dr. Ross's claims of prophecy fulfillment are
examined in terms of these four characteristics of valid prophecy, we will see that none of his
alleged prophecy fulfillments can pass all points of this test. That being true, he has no logical
basis for claiming that any of his examples were real prophecy fulfillments.

In his fifth example, for instance, Ross argued that 150 years before the birth of Cyrus, Isaiah
specifically named him in prophecies predicting the destruction of Babylon and the defeat of
Egypt. The relevant passages here are Isaiah 44:28 ((28 who says of Cyrus, 'He is my
shepherd and will accomplish all that I please; he will say of Jerusalem, Let it be rebuilt, and
of the temple, Let its foundations be laid.')) and 45:1,13. (( 1 This is what the LORD says to
his anointed, to Cyrus, whose right hand I take hold of to subdue nations before him and to
strip kings of their armour, to open doors before him so that gates will not be shut: 13 I will
raise up Cyrus in my righteousness: I will make all his ways straight. He will rebuild my city
and set my exiles free, but not for a price or reward, says the LORD Almighty.)) These
verses do name Cyrus in a long context predicting the defeat of Babylon and Egypt, but in
order to prove the unlikely claim that Isaiah mentioned a king 150 years before his birth, Ross
will have to show that these passages can pass test number 2 in the list above. He must prove,
in other words, that Isaiah really wrote these verses and that they were not redactions that
someone added after the fact to make it appear that Isaiah had had remarkable prophetic
abilities.

Needless to say, Ross cannot do this, because the original copy of Isaiah's work does not
exist, so we have no way of knowing how much editing his book underwent before it was
finally standardized by the invention of the printing press. Through critical analysis, however,
responsible scholarship has determined that the book of Isaiah in its present form is not the
work of a single author. The literature on this subject is too extensive to review at this point,
but the following statement from Eerdman's Bible Dictionary presents the view that is held by
most scholars concerning the authorship of the book of Isaiah:

From a literary standpoint, the book can be divided into two major sections, Isa. 1-39
and 40-66, on the basis of content and, concomitantly, theological concerns. Indeed,

Volume 1990 - 2002 Issue


Page 710 of 2049
Skeptical Review Edited by Farrell Till
the majority of critical scholars accept the view suggested as early as Abraham ibn
Ezra (twelfth century A. D.) that only the first portion can be ascribed to the eighth-
century B. C. prophet Isaiah, a contemporary of Amos, Hosea, and Micah. The second
section is attributed to an unknown prophet, commonly designated Second or Deutero-
Isaiah, living among the Jews in Babylon toward the end of the Exile (ca. 550-538).
Many scholars further identify chs. 56-66 as the work of a Third or Trito-Isaiah,
addressed to the restoration community perhaps in the period immediately preceding
Ezra and Nehemiah. More extreme critics posit even more "Isaiahs" (Grand Rapids,
MI, 1987, p. 531).

Ross, of course, mentioned nothing about this widely held critical view of the book of Isaiah,
and this speaks volumes about his intellectual integrity. Biblical scholars have suspected for at
least 800 years that Isaiah did not write the last half of the book bearing his name, but in
making the unlikely claim that a king was mentioned by name 150 years before his birth,
Ross said nothing about the widely held critical view that the part of Isaiah that mentions
Cyrus was written well after Cyrus had become king of Persia. Ross has resorted to the
familiar fundamentalist tactic of depending on the ignorance of his audience not to know any
better than to believe an outrageous claim. What is really sad is that the tactic works, because
most Christians lack the incentive to check such claims.

Timing is also a problem in some of Ross's other prophecy-fulfillment examples. He cited the
case of the prophet who allegedly foretold that a king named Josiah would one day burn on
Jeroboam's altar the bones of all the priests of the "high places." This "prophecy" is recorded
in 1 Kings 13:2 ((2 He cried out against the altar by the word of the LORD: O altar, altar!
This is what the LORD says: 'A son named Josiah will be born to the house of David. On you
he will sacrifice the priests of the high places who now make offerings here, and human bones
will be burned on you.')) in a setting that was approximately 300 years before Josiah was even
born, and the "fulfillment" is recorded in 2 Kings 23:15-18. ((15 Even the altar at Bethel, the
high place made by Jeroboam son of Nebat, who had caused Israel to sin--even that altar and
high place he demolished. He burned the high place and ground it to powder, and burned the
Asherah pole also.
16
Then Josiah looked around, and when he saw the tombs that were there on the hillside, he
had the bones removed from them and burned on the altar to defile it, in accordance with the
word of the LORD proclaimed by the man of God who foretold these things.
17
The king asked, What is that tombstone I see? The men of the city said, It marks the tomb
of the man of God who came from Judah and pronounced against the altar of Bethel the very
things you have done to it.
18
Leave it alone, he said. Don't let anyone disturb his bones. So they spared his bones and
those of the prophet who had come from Samaria. ))

If a prediction this specific had actually been made, it would certainly have been remarkable,
but unfortunately for Dr. Ross, there are some obvious problems that he didn't even bother to
address. For one thing, the prophet who made this alleged prediction wasn't even identified by
name, as Ross acknowledged in parenthetically noting that the man was "unnamed but
probably Shemiah," so he is apparently arguing that it is reasonable to believe that an
"unknown" person remarkably predicted *by name* the reign of a king who wouldn't even be
born until 300 years later. Would Ross be willing to believe the genuineness of such a

Volume 1990 - 2002 Issue


Page 711 of 2049
Skeptical Review Edited by Farrell Till
prophecy if it were found in any book other than the Bible? I seriously doubt if he would, but
since this little yarn is in the Bible, he expects everyone to accept it without question.

In order to prove that this is a case of prophecy fulfillment, Ross would have to prove that an
"unnamed" prophet actually did make this prediction at the time claimed (300 years before the
fact), and I see no way for him to do that except through an a priori assumption that if the
Bible says that it happened, then it has to be true. This, however, would be a flagrant resort to
question begging, because the whole purpose of Ross's prophecy-fulfillment claims is to
prove that the Bible is true. Therefore, he cannot assume that something as extraordinary as
this particular prophecy claim is true until he has proven his original premise, i.e., the Bible is
God's word and therefore true in what it says.

In the absence of any corroborating evidence that an unnamed prophet predicted *by name*
the reign of a king 300 years before the fact, I feel entitled to question that this happened.
Common sense tells me that it would have been quite simple for a copyist or editor of 1 and 2
Kings to fabricate an "unnamed" prophet or at least to put into his mouth the words he
presumably spoke about Josiah in 1 Kings 13:2 and then later to insert a claim in 2 Kings
23:15-18 that Josiah's actions in burning bones on the altar at Bethel was what "the man of
God had proclaimed" years before. In other words, to prove that this is a case of prophecy
fulfillment, Ross must prove that everything happened exactly as recorded and that the story
wasn't deliberately written to give an appearance of remarkable prophecy fulfillment.
Needless to say, he cannot do this, because he has no other records to corroborate the biblical
account. This claim of prophecy fulfillment fails, then, for the simple reason that Ross cannot
prove that the prophecy was written before the fact or for that matter that the alleged
fulfillment event ever happened. Ross is simply assuming that if the Bible says that it
happened, then it happened.

This same problem occurred throughout Ross's article. Ross cited the alleged curse
pronounced by Joshua after the destruction of Jericho (an event that many reputable scholars
say the archaeological evidence doesn't support) upon the man who tried to rebuild the city.
Joshua said that the man would lay the foundation at the cost of his eldest son and the gates at
the cost of his youngest son (Josh. 6:26). ((26 At that time Joshua pronounced this solemn
oath: Cursed before the LORD is the man who undertakes to rebuild this city, Jericho:
At the cost of his firstborn son will he lay its foundations; at the cost of his youngest will he
set up its gates. )) Well, guess what happened? About five centuries later lived a man named
Hiel, and 1 Kings 16:34 ((34 In Ahab's time, Hiel of Bethel rebuilt Jericho. He laid its
foundations at the cost of his firstborn son Abiram, and he set up its gates at the cost of his
youngest son Segub, in accordance with the word of the LORD spoken by Joshua son of
Nun.)) assures us that "Hiel of Bethel built Jericho" and "laid its foundation with Abiram his
firstborn, and with his youngest son Serug he set up its gates, *according to the word of
Yahweh, which He had spoken through Joshua the son of Nun.*"

Now isn't that truly amazing? Biblicists probably read this and wonder how anyone can deny
that the Bible is the inspired word of God. How would it have been possible for Joshua to
know 5 centuries before the fact that these deaths would occur? Well, I have some better
questions. How do Ross and his biblicist friends know that Joshua actually pronounced this
curse on Jericho? Through painstaking scientific analysis of the Bible, critics know that many

Volume 1990 - 2002 Issue


Page 712 of 2049
Skeptical Review Edited by Farrell Till
of its books are actually collages put together by many writers and editors (as in the case of
Isaiah just cited), so that being the case, does Ross not understand how simple it would have
been for an editor of the book of Joshua to insert in the text a statement that would make it
appear that Joshua had predicted deaths that were known to have occurred in the
reconstruction of Jericho, or that it could have happened the opposite way, i.e., the "original
autographs" of the book of Joshua could have contained the statement about Jericho, and so
the writer of 1 Kings just made up the deaths of Hiel's sons in order to fabricate a fulfillment
of what Joshua had predicted? Or it could easily be that Joshua never made the statement and
that the deaths never happened. Biblical writers just made up the whole thing. Ross put the
odds against fulfillment of this curse-on-Jericho prophecy at 1 in 10^7, but I would say that
the odds would be about 1 in 1 that the writers and editors of Joshua and 1 Kings could have
used any of the above methods to make it appear that prophecy fulfillment had happened in
the rebuilding of Jericho.

Ross must remember that in order to prove prophecy fulfillment, he has to establish beyond
reasonable doubt that the event alleged to be a fulfillment actually happened. So what is his
proof that (1) Joshua did in fact make the prediction, and (2) the deaths of Hiel's sons did
actually happen? There is no corroborating evidence for either one in nonbiblical records.
Ross must return to the drawing board and look for another case of prophecy fulfillment,
because this one has fizzled out on him.

Even more simplistic than the curse-on-Jericho prophecy is Ross's claim that "(t)he day of
Elijah's supernatural departure from Earth was predicted unanimously--and accurately,
according to the eyewitness account--by a group of fifty prophets" (2 Kings 2:3-11). ((3 The
company of the prophets at Bethel came out to Elisha and asked, Do you know that the LORD
is going to take your master from you today? Yes, I know, Elisha replied, but do not speak of
it.
4
Then Elijah said to him, Stay here, Elisha; the LORD has sent me to Jericho. And he replied,
As surely as the LORD lives and as you live, I will not leave you. So they went to Jericho.
5
The company of the prophets at Jericho went up to Elisha and asked him, Do you know that
the LORD is going to take your master from you today? Yes, I know, he replied, but do not
speak of it.
6
Then Elijah said to him, Stay here; the LORD has sent me to the Jordan. And he replied, As
surely as the LORD lives and as you live, I will not leave you. So the two of them walked on.
7
Fifty men of the company of the prophets went and stood at a distance, facing the place
where Elijah and Elisha had stopped at the Jordan.
8
Elijah took his cloak, rolled it up and struck the water with it. The water divided to the right
and to the left, and the two of them crossed over on dry ground.
9
When they had crossed, Elijah said to Elisha, Tell me, what can I do for you before I am
taken from you? Let me inherit a double portion of your spirit, Elisha replied.
10
You have asked a difficult thing, Elijah said, yet if you see me when I am taken from you, it
will be yours--otherwise not.
11
As they were walking along and talking together, suddenly a chariot of fire and horses of
fire appeared and separated the two of them, and Elijah went up to heaven in a whirlwind.))
It's almost embarrassing to respond to such simplistic nonsense as this. In the first place,
where is the proof that Elijah departed from the earth supernaturally? I know the Bible says
that "a chariot of fire appeared with horses" and separated Elijah and Elisha as they walked

Volume 1990 - 2002 Issue


Page 713 of 2049
Skeptical Review Edited by Farrell Till
along and that Elijah was then taken up into heaven "by a whirlwind," but to argue that he can
know, with enough certitude to claim prophecy fulfillment, that such an extraordinary event
as this really happened is to assume the inerrancy of the scriptures, and isn't that why Ross is
citing his cases of prophecy fulfillment, i.e., to establish the accuracy and divine origin of the
scriptures? He needs to bone up on circular reasoning.

So what Ross is claiming is that on the day that this unverifiable event allegedly happened a
group of 50 prophets predicted it would happen on that same day. To evaluate this claim, we
need to look at the entire passage:

And it came to pass, when Yahweh was about to take up Elijah into heaven by a
whirlwind, that Elijah went with Elisha from Gilgal. Then Elijah said to Elisha, "Stay
here, please, for Yahweh has sent me on to Bethel." But Elisha said, "As Yahweh
lives, and as your soul lives, I will not leave you!" So they went down to Bethel. Now
the sons of the prophets who were at Bethel came out to Elisha, and said to him, "Do
you know that Yahweh will take away your master from over you today?" And he
said, "Yes, I know; keep silent."
Then Elijah said to him, "Elisha, stay here, please, for Yahweh has sent me on to
Jericho." But he said, "As Yahweh lives, and as your soul lives, I will not leave you!"
So they came to Jericho. Now the sons of the prophets who were at Jericho came to
Elisha and said to him, "Do you know that Yahweh will take away your master from
over you today?" So he answered, "Yes, I know; keep silent!" (2 Kings 2:1-5)

To make a long story short, Elijah and Elisha continued their journey together, and wherever
they went "sons of the prophets" would come out to meet them and tell Elisha that Yahweh
was going to "take" Elijah that day. The remaining verses in this fanciful yarn tell us that
altogether these prophets numbered 50 and that as Elijah and Elisha continued on their
journey, sure enough, just as the prophets had predicted, a chariot of fire suddenly appeared
and separated Elijah and Elisha, and then Elijah was taken into heaven in a whirlwind.

One does not have to be a rocket scientist to see the absurdity in claiming this as a case of
amazing prophecy fulfillment. In the first place, there is nothing in the narrative that even
remotely suggests that this is an "eyewitness" account. Not a one of these "sons of the
prophets" was ever identified by name, and there is nothing in the text that even implies that
Elisha, the only person who was with Elijah when the event allegedly happened, was the
author of the text. In fact, we don't know who the author of 2 Kings was, but we do know that
it could not have been Elisha, because 2 Kings records events that happened after Elisha was
dead. So in the matter at hand, we know only that some unknown writer SAID that "Elisha
saw it (the taking of Elijah in the whirlwind)" (v:12). That is hardly eyewitness testimony.

Secondly, the story is as transparent as cellophane. Anyone could sit down and write a
narrative in which he claims that extraordinary events happened, and indeed such fantastic
claims as these were characteristic of the national literatures of biblical times. They can be
found in Egyptian, Canaanite, Sumerian, Babylonian, Greek, Roman, and other literature of
the times. If this story were in any book besides the Bible, Dr. Ross would laugh it into the
next county, but because it is in the Bible, he expects us to accept it without question. This
argument, like so many of his others in the article, is rooted in the assumption that the Bible is

Volume 1990 - 2002 Issue


Page 714 of 2049
Skeptical Review Edited by Farrell Till
accurate in everything it says. This is an assumption that he must prove before he can claim
the story of Elijah's translation as an example of prophecy fulfillment.

A twin sister to this one is the story Ross cited about Jahaziel's prophecy that King
Jehoshaphat would win a great victory over his enemies without even lifting a hand in battle.
Such a story is in the Bible (2 Chron. 20), and it indeed asserts that "the spirit of Yahweh"
came upon Jahaziel, who then assured Jehoshaphat that he would win a victory without
having to fight a battle (v:17). The next day, according to the story, Jehoshaphat took his
army out, appointed certain ones to sing to Yahweh, "Praise Yahweh, for his mercy endures
forever," and his enemies turned on each other and "helped to destroy one another" (v:23). So,
you see, Jehoshaphat won the war without even lifting a hand in battle.

Do I even need to say what is wrong with this wonderful example of "prophecy fulfillment"?
Not really, but just for the heck of it, I think I will. (1) There is no evidence of any kind to
corroborate the Chronicle writer's claim that a prophet named Jahaziel ever made such a
prediction as this. Not even the writer(s) of 1 & 2 Kings who also recorded the history of
Jehoshaphat's reign told any such story as this, although one would think that if such an
extraordinary event as this had really happened, Yahweh would have had his other inspired
historian(s) to include it in his accounts of Jehoshaphat's exploits. The fact is that the parallel
accounts of Jehoshaphat's reign in 1 & 2 Kings don't even mention anyone named Jahaziel.
Curious indeed! (2) There is no extrabiblical corroboration to confirm that such a
confrontation as this ever occurred between Jehoshaphat and the armies allied against him. (3)
It is a story that any literate person could sit down and write and, furthermore, is just the type
of fantastic story that literate writers of biblical times in all nations liked to sit down and
write. One would think that Dr. Ross would have more professional pride than to claim such a
silly story as this as an example of prophecy fulfillment.

A favorite of prophecy-fulfillment buffs is Isaiah's prophecy that mighty Babylon, "the glory
of kingdoms and the beauty of the Chaldeans' pride" would become "as when God overthrew
Sodom and Gomorrah" and never again be inhabited (Isaiah 13:19), and I see that Dr. Ross
likes this one too, as well as Jeremiah's prophecy that Babylon would be destroyed and
become "desolate forever" (Jer. 51:26). Both prophets predicted the utter and complete
destruction of Babylon forever. Isaiah said that "(i)t shall never be inhabited, neither shall it
be dwelt in from generation to generation: neither shall the Arabian pitch tent there; neither
shall shepherds make their flocks to lie down there" (13:20). Jeremiah predicted that her cities
would become "a desolation, a dry land, and a desert, a land wherein no man dwells, neither
does any son of man pass thereby" (51:43).

This is pretty impressive stuff, and it would be even more impressive if it had all come true as
these prophets predicted. Let's notice first that Jeremiah was obviously using "Babylon" in the
sense of the nation of Babylonia, because he said that her CITIES (plural) would become a
desolation and a land where no one dwells. The area occupied by the Babylonian kingdom is
now part of the nation of Iraq, and cities exist on the same territory that was once Babylonia,
so obviously people pass through the area and dwell there.

But what about the site that was occupied by the city of Babylon? Isaiah said that it would
never be inhabited again and that Arabs would not pitch their tents there or shepherds make

Volume 1990 - 2002 Issue


Page 715 of 2049
Skeptical Review Edited by Farrell Till
their sheepfolds there. As a child, I recall a sermon by a preacher who had recently returned
from a trip to the biblical lands, and he told about visiting the site of ancient Babylon with a
guided tour. According to his story, the group arrived late in the afternoon when there wasn't
enough daylight left to complete the tour, so the tourists began to make camp for the night
with tents that had been brought along for such occasions. And guess what? Some of the tour
guides were Arabs, and they refused to stay in the camp. They went out into the desert away
from the city and set up their tents.

I can remember thinking how amazing it was that Isaiah had been able to prophesy with such
accuracy as this. Could there be any doubt that the Bible was the inspired word of God?
Having since learned to look at matters such as this more critically rather than just gullibly
believing everything I hear, I see a lot of problems in this story. For one thing, I have had
enough experience with preachers and the stories they use to embellish their sermons to
realize that it's not at all unlikely that such an incident as this never even happened. For the
sake of argument, however, let's assume that it did. It would prove only that the Arabs on this
tour refused to camp overnight at the site of ancient Babylon; it would not establish that no
Arabs have ever pitched their tents there since the time of Babylon's destruction.

Also, I now wonder if it never occurred to this preacher that his very presence there had
established the inaccuracy of at least part of the prophecies against Babylon, because
Jeremiah had said, "Neither does any son of man pass thereby" (51:43), but in the story the
preacher was relating several people had passed thereby. Isaiah had said that the site would
never be inhabited, and Jeremiah had said that no man would dwell there, but for at least one
night, a whole company of tourists had dwelt there. In addition to this, extensive
archaeological excavations have been done at the site of ancient Babylon, and I seriously
doubt if all this work has been done without crews having lived on the site during the
diggings. These are points that preachers apparently never think of when they're trying to
prove biblical inerrancy.

There is a final nail to drive in the coffin of these amazing prophecies about Babylon. As
anyone can confirm with a little research at a local library, Saddamn Hussein, the present
dictator of Iraq, has been engaged for over a decade in the reconstruction of ancient Babylon.
Articles in World Press Review (Feb. 1990, p. 74) and U. S. News & World Report (Sept. 25,
1989, p. 37) both discuss this venture and show pictures of projects that have been completed
as well as those still in progress. Hussein's intention is not just to build a city on the site but to
reconstruct the ancient city as archaeological findings indicate that it once existed. The actual
foundations of some of the old buildings are being used in the reconstruction work. This
project, incidentally, is being carried out much to the consternation of archaeologists who
deplore the history of ancient Babylon that will be lost in the process. If Ross hopes to find an
example of accurate prophecy in the Bible, he will have to look elsewhere, because the facts
do not support the prophetic claims that Babylon would be destroyed and remain forever
desolate, a place that no one would pass through.

Akin to Isaiah's and Jeremiah's prophecy against Babylon is Jeremiah's and Ezekiel's
prediction that "the land of Edom would become a barren, uninhabited wasteland" (Jer. 49:15-
20; Eze. 25:12-14). The same problem surrounds these prophecies as the ones spoken against
Babylon by Isaiah and Jeremiah. The latter said of Edom, "No one shall remain there, nor

Volume 1990 - 2002 Issue


Page 716 of 2049
Skeptical Review Edited by Farrell Till
shall a son of man dwell in it" (49:18), and of the Edomite city of Bozrah, he said that "it shall
become a desolation, a reproach, a waste, and a curse" (v:13). Ezekiel declared that Yahweh
would make the land of Edom "desolate from Teman even unto Dedan" (25:13). Despite these
tirades, the region that once was Edom is inhabited today by Jordanians who now incorporate
the territory into their country. The modern city of Buseirah now occupies the site where
Bozrah was located, and Tawilan is the modern name for Teman. The prophetic descriptions
of what was to become of Edom do not "accurately tell the history of that now bleak region,
as Ross claims.

If one investigates prophetic literature, he will see that the Hebrew prophets constantly
spewed out invectives and tirades against the nations around Israel. Ezekiel predicted that the
stronghold of Tyre would be destroyed and never built again (chapter 26) and that Egypt
would be laid waste for 40 years and that "no foot of man or beast" would pass through it
(29:11-12). These prophecies never materialized, just as Jeremiah's and Ezekiel's prophecies
against Edom never happened.

Three more of Dr. Ross's 13 claims of prophecy fulfillment remain, and I will discuss them in
the next issue. Meanwhile, If Dr. Ross wishes to respond to my rebuttal articles, we will
publish his reply.

What Third-Day Prophecy?


by Farrell Till
New Testament writers claimed that the resurrection of Jesus was prophesied in the Old
Testament. The weakness of this claim is apparent in the fact that none of these writers ever
cited an Old Testament prophecy whose face-value meaning was so obvious that no
reasonable person could deny that the prophets were indeed predicting that the Messiah would
rise from the dead. The best they could do was distort a statement in Psalm 16:8-11 to try to
make it mean something that the psalmist never intended:

I have set Yahweh always before me; because He is at my right hand, I shall not be
moved. Therefore, my heart is glad, and my glory rejoices. My flesh also will rest in
hope. For you will not leave my soul in Sheol, nor will You allow Your Holy One to
see corruption. You will show me the path of life; in Your presence is fullness of joy:
at your right hand are pleasures forevermore.

Pardon my ignorance, but if there is a clear cut prediction of a resurrection in this passage, I
simply cannot see it. Yet the apostle Peter, in a sermon that Luke put into his mouth, quoted
this scripture on the day of Pentecost and said that it was a prophecy of the Messiah's
resurrection. "Men and brethren, let me speak freely to you of the patriarch David, that he is
both dead and buried, and his tomb is with us to this day. Therefore, *being a prophet,* and
knowing that God had sworn with an oath to him that of the fruit of his body, according to the

Volume 1990 - 2002 Issue


Page 717 of 2049
Skeptical Review Edited by Farrell Till
flesh, He would raise up the Christ to sit on his throne, he, *foreseeing this,* spoke
concerning the resurrection of the Christ, that His soul was not left in Hades, nor did his flesh
see corruption" (Acts 2:29-31).

One would think that if Yahweh's eternal plan was for his son to die and then rise from the
dead, he could have had the prophets predict this in terms far more distinct than what was said
in the passage Peter allegedly quoted. If someone from a culture not familiar with the Bible
simply read the 16th Psalm, what would be the likelihood of his thinking that the verses Peter
cited were speaking of a resurrection from the dead? He would be much more likely to think
that the first-person narrative in this poem was referring to the writer's own state or condition.
It took the imagination and desperation of someone trying to establish a foothold for a new
religion to see a prophecy of resurrection in the statement.

For the sake of argument, however, let's just assume that the intention of this psalm was to
speak of a resurrection from the dead. Even if that were so, why wouldn't the reader
understand that the writer was speaking of his own resurrection? After all, the writer spoke
consistently in terms of "I," "me," and "my" throughout the passage Peter quoted, and first-
person pronouns are clearly references to the person speaking or, in this case, writing. To
apply the statement to a third-person party who would live hundreds of years later is to take
unwarranted liberties with the text. It is the kind of liberty that could prove just about
anything from any written text. With such interpretative methods, Moslems could prove that
their Koran is God's word, and Latter Day Saints could prove that their Book of Mormon is
inspired.

If Peter really made the speech that Luke attributed to him, he no doubt recognized this
problem, because he made a strained attempt to prove that David (who may or may not have
been the author of this psalm) was not speaking of himself. His argument was that "David is
both dead and buried, and his tomb is with us to this day" (Acts 2:29). Well, if that were so
and if it could be determined beyond question that "David" was speaking of a resurrection
from the dead in this passage, why wouldn't it be reasonable to argue that David prophesied of
his own resurrection and that this prophecy had failed because "his tomb is with us to this
day"? In other words, why must we assume that Peter was right in the slant that he gave to the
16th Psalm? Why couldn't it be that Peter was mistaken in his interpretation of what "David"
had written? To show that he was not mistaken, inerrantists will have to explain why it is
logical to believe that the first-person pronouns ("I," "me," and "my") of a 10th-century B. C.
psalmist were in actuality references to someone who would not be born for another thousand
years.

Inerrantists cannot argue that they can know Peter's application of the passage was correct
because he was speaking by the inspiration of the Holy spirit, for that would be a flagrant
attempt to prove inerrancy by assuming inerrancy. Biblicists must first prove that Peter spoke
by the inspiration of the omniscient, omnipotent Holy Spirit, and then they can argue that this
divinely guided insight is proof that he was right in what he said that "David" meant in Psalm
16. So the task for inerrantists who support Peter's view is to analyze the text of Psalm 16 and
give compelling reasons why the language of the passage gives sufficient reason to
understand that it was speaking about a resurrection from the dead.

Volume 1990 - 2002 Issue


Page 718 of 2049
Skeptical Review Edited by Farrell Till
I don't think that inerrantists can give any compelling reasons in support of Peter's view, but I
can definitely give some compelling reasons to reject his view. First, there are the facts
already noted: (1) the psalm was written in the first person and so the situations spoken about
can best be understood as personal references to the writer's own condition, and (2) there is
simply no language in the psalm that can be interpreted only as references to a resurrection
from the dead. In addition to all this, there was a strategic error that Peter made in his zeal to
prove that the 16th Psalm was speaking of the resurrection of Jesus. After saying that David
was "both dead and buried" and that "his tomb is with us to this day," Peter went on to say,
"Therefore, being a prophet, and knowing that God had sworn with an oath to him that of the
fruit of his body, according to the flesh, *He would raise up the Christ to sit on his throne,* he
foreseeing this, spoke concerning the resurrection of the Christ..." (vs:30-31).

To prove Peter's argument, inerrantists must show us where the Old Testament says that God
swore with an oath to David that he would "raise up the Christ to sit on his throne." The clear
implication of Peter's statement is that God had sworn with an oath to David that he would
resurrect the Christ to sit on David's throne; otherwise, Peter made no sense when he said,
"He [David] foreseeing this [that the Christ would be resurrected to sit on his throne], spoke
concerning the resurrection of the Christ." So just where in the Old Testament did Yahweh
ever speak with an oath to David that a Messiah from "the fruit of his body" would be
resurrected to sit on his throne"?

The best that reference Bibles can do in support of Peter's claim is list Psalm 89:3; 132:11;
and 2 Samuel 7:12. If we examine them individually, however, we will see that they do not
refer to the resurrection of any of David's descendants for the purpose of having them sit on
David's throne. Psalm 89:3 says, "I have made a covenant with My chosen, I have sworn to
My servant David: your seed I will establish forever; your faithfulness you shall establish in
the very heavens." Establishing David's seed forever is at best a promise to establish his
throne through a process of having a natural descendant of David occupy it in each
succeeding generation, so where is the oath in this statement that God would resurrect a
descendant of David to sit on his throne? It isn't there, except in the minds of those who are
desperate to prove an untenable position.

Admittedly, the prophets promised that David's throne would be established forever, but they
clearly meant the literal throne of David that would be maintained by an endless line of
David's descendants. In their fanatical ethnocentrism, the Hebrew prophets thought that their
little nation was favored of Yahweh, who would always protect them and see that their
kingdom lasted forever, but in no sense were the Jews looking for the establishment of some
"spiritual kingdom." This was an idea that was hatched up by the New Testament writers as a
way of presenting an allegedly resurrected Jesus as the long-awaited Messiah. This can
clearly be seen by analyzing Psalm 132:11, the second reference-Bible proof text for Peter's
claim. "Yahweh has sworn in truth to David; He will not run from it: `I will set upon your
throne the fruit of your body.'" Again, there is nothing in this statement that even implies that
the psalmist meant that a descendant of David would be resurrected from the dead to sit on
David's throne. It was simply a promise that the throne of David would be established through
his descendants. That this was the clear intention of the statement is shown by the very next
verse: "If your sons will keep my covenant and my testimony which I shall teach them, their
sons also shall sit upon your throne forever."

Volume 1990 - 2002 Issue


Page 719 of 2049
Skeptical Review Edited by Farrell Till
What could be clearer than this? Yahweh promised to establish the throne of David "from the
fruit of [David's] body" and if these sons [plural] of David kept Yahweh's covenant and his
testimony, their sons also would sit upon David's throne forever. Obviously, this was not a
promise that just one person (Jesus) would be resurrected from the dead to sit on David's
throne. It wasn't a promise of a resurrection (period); it was simply a promise that Yahweh
would establish David's throne forever through his sons and then their sons if they kept
Yahweh's covenant. So what is the compelling reason for us to believe that Peter was right
when he said that God had sworn with an oath to David that he would resurrect one of David's
descendants to sit on his throne?

Both of the quotations from the Psalms appear to refer to 2 Samuel 7:12- 14. Yahweh,
speaking to David through the prophet Nathan, said, "When your days are fulfilled and you
rest with your fathers, I will set up your seed after you, who will come from your body, and I
will establish his kingdom. He shall build a house for My name, and I will establish the throne
of his kingdom forever. I will be his Father, and he shall be My son. If he commits iniquity, I
will chasten him with the rod of men and with the blows of the sons of men." It cannot be
claimed that this "son" who would come from the seed of David was Jesus, because Yahweh
said, "If he commits iniquity, I will chasten him with the rod of men," but Jesus was allegedly
without iniquity. Whoever this "son" was, he was going to "build a house for My Name," and
this sounds very suspiciously like a reference to Solomon. The first part of this chapter (as
well as the part after the above quotation) discussed the building of a house for Yahweh so
that he would not have to dwell in a tent (the tabernacle). It is simply a matter of Old
Testament record that Solomon was the one who built this house or temple (1 Kings 6-8), so
clearly this promise of a son who would sit on David's throne was a reference to Solomon and
not some descendant who would be born a thousand years later.

There is simply no Old Testament support for Peter's claim that Yahweh had sworn with an
oath to resurrect one of David's descendants to sit on his throne, but there is a New Testament
statement attributed to the apostle Paul that makes it logically impossible for the Old
Testament to contain any prophecy of the resurrection of Jesus. In a speech allegedly made in
the synagogue at Antioch of Pisidia, Paul said of the crucifixion of Jesus, "Now when they
had fulfilled all that was written concerning Him, they took Him down from the tree and laid
Him in a tomb" (Acts 13:29). Please notice what Luke attributed to Paul in this sermon. He
said that all that had been written concerning Jesus had been fulfilled when they took him
down from the tree and laid him in a tomb. Now if all that had been written of Jesus was
fulfilled when they took him down from the cross (tree), then the resurrection could not have
been written about in the Old Testament, because the resurrection allegedly happened after
Jesus was taken down and laid in a tomb.

Some may point to verses 33-37 and say, "That can't be right, because Paul went on to quote
the same passage that Peter did from Psalm 16 as a prophecy of the resurrection." That's true,
but Paul can't have it both ways. If everything that had been written of Jesus when they took
him down from the cross had been fulfilled, then there could have been no prophecies of
Jesus's resurrection, but if there were prophecies of Jesus's resurrection, then everything that
had been written about him could not have been fulfilled when he was taken down from the
cross. Either way, inerrantists have a problem, and I would be happy to see them satisfactorily
explain away either one.

Volume 1990 - 2002 Issue


Page 720 of 2049
Skeptical Review Edited by Farrell Till
So all of the evidence points to misrepresentation or distortion of Old Testament scriptures by
the New Testament writers who claimed that the prophets had foretold the resurrection of
Jesus. There are simply no reasonable grounds for claiming that there had been prophecies in
the Old Testament of the Messiah's resurrection, and there are certainly no grounds for the
VERY specific prophecy claim in Luke 24:46. Here it is alleged that Jesus said the night of
his resurrection, "Thus it is written, and thus it was necessary for the Christ to suffer and to
rise from the dead *the third day.*" So we have Jesus claiming that the resurrection had not
just been written about but that it had been written that it would occur on the *third* day. The
apostle Paul made a similar claim in 1 Corinthians 15:3-4, "For I delivered to you first of all
that which I also received: that Christ died for our sins according to the Scriptures, and that
He was buried, and that He rose again the third day *according to the scriptures.*"

Here are two passages that claim the scriptures had spoken of the resurrection of Jesus on the
*third* day. And here is a challenge to inerrantists who claim that the Bible contains no
mistakes. Produce an Old Testament statement that prophesies of the resurrection of the
Messiah in language too clear to be misunderstood, and I will publish it on the masthead of
*The Skeptical Review* from now on as long as this paper continues to be published.
Produce an Old Testament prophecy of the resurrection of the Messiah on the third day, and I
will immediately cease publication of *The Skeptical Review.* I would think that this is an
offer that loyal inerrantists would find hard to refuse.

EDITOR'S NOTE: The preceding article was written especially for Paul Nanson to reply to.
This name will be instantly recognized by most subscribers who are active on the internet.
Nanson maintains a list called Apologia-l on which he tolerates very little dissension. Those
who disagree with his views of the Bible are likely to be expelled from his list. Nanson
expelled me, even though I informed him when I requested a subscription that I was a
professional debater and the publisher of a paper devoted to debunking biblical inerrancy.

Although Nanson tolerates little dissent on his list, he likes to lurk on other lists and post
insults. After daring me to give him space in *TSR,* I wrote the article above and sent it to
him for rebuttal. He declined the opportunity.

Additional Errors Committed by Farrell


Till(2)
by Marion R. Fox
Farrell made several errors in his reply to my article (TSR, Vol. 6, No. 3). Farrell said, "(H)e
[Fox] arrived at no firm conclusions about what Deuteronomy 23:2 means." I certainly did
arrive at a firm conclusion: "God is not unrighteous in that He does not have any *ex post
facto* (retroactive) laws" (p. 2), and "(T)he argument set forth by Farrell Till is unsound.
Perez was not a bastard, and all of Farrell's arguments on this matter are moot" (p. 3).

Volume 1990 - 2002 Issue


Page 721 of 2049
Skeptical Review Edited by Farrell Till
Farrell apparently belongs to the any-objection- will-do school. He claims that I "assert
without proof," but I set forth simple and sound arguments to prove my case. It does not
always take a book to set forth a sound argument. Farrell admitted that there are more than ten
generations between Perez and David. Farrell said, "I'm going to surprise Mr. Fox and admit
that if there actually was a person named Perez, who was born at the time claimed in Genesis
38:29, and if King David, who lived in the time claimed in the books of Samuel, was a direct
descendant of this Perez, then there surely were more than just ten generations separating
Perez and David" (TSR, Vol. 6, No, 3, p. 6). Farrell then retreats to his claim that "even to the
tenth generation" means "forever." He retreats by making an accommodative argument: "For
the sake of argument, we will concede them [those who argue that the ban ended with David,
who was a 10th generation descendant of Perez] the quibble and then ask them to explain why
the descendants of Aaron, the first high priest of Israel, were not banned from the assembly"
("No Bastards Allowed," TSR, Vol. 5, No. 2, p. 12). Notice how Farrell slides from one
objection to another when he cannot sustain his objection. Farrell says, "(S)o this article will
focus only on the genealogy of David to show that Israel's most famous king..." (TSR, Vol. 6,
No. 3, p. 3) and then proceeds to ramble around and discuss Aaron (p. 4).

I am convinced that a rational person would read the Bible and discern that there are more
than ten generations from Perez to David. The reader should be honest enough to ask, "Does
the Bible use the word 'son' in a different way than we use it today?" I have been called "son"
by my grandfather and did not think he was improperly using the word "son." This was a
common usage in the Scriptures and is the obvious import of the genealogies. This is the kind
of reasoning that Farrell might use to claim that Matthew thought Jesus was the son of David,
in a literal sense (Matt. 1:1). It is obvious that the word "son" is used in a broader sense in the
very context of the genealogies (Matt. 1:1, 20). The writer of Matthew does not have to tell
the reader what is obvious (that Jesus is not the literal son of David, Matthew 1:1). Farrell
probably accuses Matthew of equivocating (cf. TSR, Vol. 6, No. 3, p. 6) when it is obvious
that he used the word "son" to refer to a descendant (whether a son, grandson, etc.).

Farrell should remember that an analogy does not prove anything; it only illustrates. His
analogy about John Jones (TSR, Vol. 6, No. 3, p. 4) does not prove anything. Why not just get
to the point instead of flaying about and diverting the issue from David and Aaron? The
analogy used by Farrell causes problems for him. A legal analogy requires that the testimony
of a witness be accepted unless it is shown that his claims CANNOT be true. He is presumed
innocent until he is proven guilty. But Farrell does not allow the Bible to be right until it is
proven beyond any reasonable doubt to be guilty of having errors.

My article was brief because I was taught to get to the point and say what you mean as briefly
as possible. Farrell, being the editor, can ramble as much as he wants, but a good writer will
get to the point and deal with the issue. Farrell, quit taking so many side excursions and deal
with the issue. In scholarly journals, writers are required to be brief and get to the point.
Farrell says, "Although this is technically off subject..." (TSR, Vol. 6., No. 3, p. 5) and admits
to having a rambling style.

Volume 1990 - 2002 Issue


Page 722 of 2049
Skeptical Review Edited by Farrell Till
Farrell's Accommodative Argument

When Farrell responds to my syllogism (below), he says, "I wish Mr. Fox would tell us what
skeptic he knows who would say that the first premise in this syllogism is true" (TSR, Vol. 6,
No. 3, p. 7).

First Premise: If the Bible says X then X is true.


Second Premise: The Bible does not say X.
Conclusion: Therefore, X is not true.

Farrell used this argument in an accommodative sense in his reply to me. Farrell argued:

First Premise: If the Bible used the word "son" in a broader sense than a literal son in a
genealogy, then the word "son" is used in a broader sense than a literal son.
Second Premise: The Bible did not use the word "son" in a broader sense than a literal
son in a genealogy.
Conclusion: The word "son is not used in a broader sense than a literal son.

I certainly know that skeptics deny the integrity of the Bible.

Farrell's Misrepresentation of My Position

Farrell misrepresents my position on how I know something is true. Farrell constructs the
following syllogism and represents it as my position:

First Premise: If the Bible says X, then X is true.


Second Premise: The Bible says X.
Conclusion: Therefore, X is true.

If this was my argument, I would be guilty of "begging the question" or assuming what I was
required to prove. My basic argument is:

First Premise: If God exists and the Bible has evidence of being inspired of God, then
the Bible is inerrant.
Second Premise: God exists and the Bible has evidence of being inspired of God.
Conclusion: Therefore, the Bible is inerrant.

The conclusion of this syllogism is the basis for believing the first premise of Farrell's
syllogism. Christians are required to prove the second premise.

I do not have to know all things to know some things are true. For example, I might sit on a
jury and hear unimpeachable witnesses claim they saw a man kill another man, hear
testimony from expert witnesses that his fingerprints were on the murder weapon, hear
testimony from expert witnesses that some of his tissue was identified under the fingernails of
the victim from DNA, and that some hair samples from the body of the victim were from the
defendant. The defendant might claim to have been in jail at the time of the murder with the
jail record showing that he was in jail but that the jailer had "bed check" two hours before the

Volume 1990 - 2002 Issue


Page 723 of 2049
Skeptical Review Edited by Farrell Till
murder and two hours after the murder. Other jurors might ask me how he got out of jail and
back into jail, and I reply, "I do not know, but the evidence is overwhelming that he is guilty."
This is the same reason I do not have to know the answer to all objections to know that the
Bible is the Word of God. The argument made by Farrell seemed reasonable until I
considered it more closely. Perez was not a bastard, and the law of God is not retroactive. I
sincerely doubt that he has properly interpreted the word "forever" in this passage but will not
deal with this in this treatise.

Conclusion

I have given answers to all of Farrell's arguments that related to Deuteronomy 23:2 and the
argument concerning David. I have not followed him on his ramblings. Farrell has committed
all the errors I set forth in my first article, in the section entitled "Common Errors Made by
Atheists and Agnostics."

(Marion R. Fox, 4004 Twisted Trail, Oklahoma City, OK 73150- 1910.)

What Farrell Really Said


by Farrell Till
In publishing Mr. Fox's response to my rebuttal article, I omitted the first page of his
manuscript, because it pertained entirely to "mistakes" that Fox alleged that I had made in my
debate with Jerry Moffitt. Many readers of The Skeptical Review have seen this debate via the
rental tapes that we make available to interested subscribers. However, the majority of our
readers have not seen the tapes, and since the debate has not been published in TSR, most of
Fox's comments about the debate would not be understood by those who have not viewed the
tapes. For these reasons, I omitted that section from Mr. Fox's article and began with his
comments about the "errors" I made in responding to his first article.

Before, I address the points in his latest article, I will suggest to Mr. Fox that if he thinks I
made mistakes in my debate with Jerry Moffitt, he should try to convince Moffitt to debate
me again. I have asked Mr. Moffitt to debate the same proposition (the existence of the
Hebrew god Yahweh) in another location, and all that he has said is that he will consider it if I
will guarantee him an audience of atheists. Atheistic organizations in both Ohio and
California have agreed to sponsor a second debate on this issue, and although that information
has been communicated to Mr. Moffitt, he has not yet accepted either offer. Since he and Mr.
Fox, who was Moffitt's moderator in the debate, seem to believe that I was soundly defeated, I
can't understand why Moffitt wouldn't want to give me a second thrashing before another
audience, this time on my own turf, i.e., before an audience of skeptics and atheists.

TSR readers may remember that Jerry Moffitt submitted an article about the debate that I
published in the Spring 1995 edition. In this article, Moffitt focused on a side issue that came

Volume 1990 - 2002 Issue


Page 724 of 2049
Skeptical Review Edited by Farrell Till
up in the debate, which really had nothing to do with the propositions we debated, and
indicated his belief that I had pretty much made a fool of myself. Now along comes his
moderator, who tries to sneak into TSR his comments about "mistakes" that he thought I had
made in the debate with Moffitt. It just seems to me that Moffitt and Fox are spending a lot of
time trying to convince TSR readers that Moffitt scored a big one for God in that debate. Well,
if they really believe that, maybe they will consider doing it again. The challenge has already
gone out to Moffitt, so if he doesn't want to accept it, maybe Fox would consider debating the
same issue.

WHAT WAS FOX'S CONCLUSION? I said in my rebuttal article that Fox's article "arrived
at no firm conclusion about what Deuteronomy 23:2 means." Now Mr. Fox comes back to
assure us that he "certainly did arrive at a firm conclusion." He had concluded that "God is not
unrighteous in that He does not have any *ex post facto* (retroactive) laws." Oh, really?
That's what Deuteronomy 23:2 means? A passage that banned bastards from entering the
assembly of Yahweh unto the 10th generation meant that God does not have any *ex post
facto* laws?

We know what Mr. Fox was trying to say, of course; he just seems to have trouble saying
what he means. He seems to think that there is some kind of virtue in being brief to the point
of obscurity, so he frequently makes statements whose meanings can only be guessed at.
Having taught college writing for 30 years, I certainly know that one should never take more
words than necessary to say what he means, but there is a flip side to this rule of economy.
One should never sacrifice concreteness of expression just for the sake of brevity. Fox seems
to think that the use of specific, concrete details in support of one's argument constitutes
"rambling," but a major flaw in his writing is a failure to support his assertions and rebuttal
statements with specific details; consequently, we sometimes have to guess at what he meant.

In saying that his conclusion about the meaning of Deuteronomy 23:2 is that "God is not
unrighteous in that He does not have any *ex post facto* (retroactive) laws," I assume that
what he actually meant was that my application of this text to Judah's son Perez and the sons
of Aaron is inappropriate because it would be applying the law to people who were born
before the law was given. If this is what he meant, I remind him that my second rebuttal
article established that Yahweh has been known to make *ex post facto* laws (Special Fifth
Edition, p. 3). Here, I cited the case of the man who was caught picking up sticks on the
sabbath day while the children of Israel were still in the wilderness (Ex. 15:32-36). The man
was taken to Moses and Aaron, who put him in ward, "because it had not been declared what
should be done to him." Yahweh promptly told Moses that the man should surely die, and he
was taken before the congregation and stoned to death. In other words, Yahweh pronounced
an *ex post facto* sentence of death on this man, so Fox's premise that Yahweh "does not
have any *ex post facto* laws appears to contradict what the Bible says. Fox said exactly
nothing about this, but I suppose he didn't want to follow me in my ramblings.

I also pointed out that Fox's attempt to prove that Perez was the offspring of a levirate
marriage between Judah and Tamar was predicated entirely on an *ex post facto* application
of the levirate marriage law that was given in Deuteronomy 25:5-10, long after the time of
Judah's and Tamar's sexual escapade. In telling the fate of Onan, Judah's son who spilled his
seed on the ground rather than risk impregnating Tamar, Genesis 38 certainly implies the

Volume 1990 - 2002 Issue


Page 725 of 2049
Skeptical Review Edited by Farrell Till
existence of a levirate custom at this time, but nowhere before Deuteronomy 25 will Fox find
it stated as a law. His attempt to apply it to Judah and Tamar in order to give legitimacy to
their offspring is, therefore, an *ex post facto* application of the law, the very thing he
accused me of doing with the ban on bastards.

I further pointed out in my second rebuttal (5thEdition, p. 3) that the levirate marriage law
pertained to the rights of a widow to be impregnated by her dead husband's oldest surviving
brother and in cases where there was no surviving brother, the "next of kin." Judah was not
his dead son's next of kin, because he had another son Shelah whose duty it would have been
under this law to impregnate Tamar. I showed by the case of Boaz and Ruth that there were
ceremonial obligations that the next of kin had to attend to before consummating a levirate
marriage but that Judah didn't do this in the case of Tamar. He intentionally withheld his only
surviving son from her and then impregnated her himself when he thought that he was just
striking a deal with a prostitute. All of these things I pointed out in my rebuttal, and Mr. Fox
said... nothing. We can only assume that he didn't wish to follow me in my ramblings.

Mr. Fox began his second article by restating an assertion from his first: "Perez was not a
bastard, and all of Farrell's arguments on this matter are moot," but he cannot make a matter
"moot" by simply asserting that it is. He must examine my counterarguments point by point
and show that they are moot. Let's hope that we can convince him to engage in a little
rambling long enough to do that. Until he does, he has refuted nothing, and my argument still
stands.

HOW MANY GENERATIONS FROM PEREZ TO DAVID? Mr. Fox seems to find
inconsistency in my admission that "if there actually was a person named Perez, who was
born at the time claimed in Genesis 38:29, and if King David, who lived in the time claimed
in the books of Samuel, was a direct descendant of this Perez, then there surely were more
than just ten generations separating Perez and David," but he has missed the point entirely. I
can recognize the obvious fact that there would have to be more than ten generations in a line
of descent spanning more than 800 years, but I can also read the Bible and see that the writers
who cited David's genealogy always did so in a way that indicated they thought that there had
been only 10 generations from Perez to David. So where is the inconsistency? I am not
responsible for what the writers thought, and if they couldn't recognize an obvious mistake
like this, it must have been the fault of the omniscient, omnipotent Yahweh who guided them
to write such a flawed genealogy. Fox should blame Yahweh for this and not me. All I'm
doing is pointing out errancy in the Bible where errancy obviously occurs.

Fox tried to salvage this point by citing places in the Bible where the word "son" was used in
obviously figurative senses, but I have never denied that the word was so used in the Bible. In
the "Editor's Note" after Douglas Smith's letter (p. 12, this issue), I cited the same figurative
use of "son" in Matthew 1:1 that Fox did as I tried to explain to Mr. Smith the skipped-
generation argument that inerrantists use to "explain" genealogical inconsistencies in the
Bible. Since the "Mailbag" columns are always set up two to three months before the other
articles in an issue, I wrote this statement long before I had received Mr. Fox's rebuttal, so
obviously I am aware of figurative usage of the word "son" in the Bible.

Volume 1990 - 2002 Issue


Page 726 of 2049
Skeptical Review Edited by Farrell Till
Whenever the word is used figuratively, this can be easily determined by the context. In
Matthew 1:1, for example, Jesus was called "the son of David," and David was called "the son
of Abraham." Since Matthew proceeded to give a genealogy of Jesus that listed 14
generations between Abraham and David and 28 (27 actually) between David and Jesus, one
doesn't have to be a genius to see that the context of this passage indicates that the word "son"
was being used figuratively in the first verse of the chapter. Now I defy Mr. Fox to find any
genealogical information in the Bible that will show the writers were using the word "son" or
the verb "begot" figuratively in any of David's genealogies so that we can determine by
biblical context that the writers were speaking figuratively when they said that Hezron was
the son of Perez, Ram was the son of Hezron, etc., or that Perez begot Hezron, Hezron begot
Ram, etc. On pages 6-7 of my first rebuttal article (Summer 1995), I did a detailed,
generation-by-generation analysis of all of the complete genealogies from Perez to David that
are given in the Bible and showed that the usage of "son" or "begot" in all of them require us
to understand that the words were being used literally. Mr. Fox has said exactly NOTHING
about this, but, of course, he probably didn't want to follow me in my ramblings.

This has been his tactic throughout this discussion. I showed that the expression "even unto
the tenth generation" in Deuteronomy 23:2 really meant forever, and so it really didn't matter
if there were ten or fifty generations between Perez and David. If the ban was to apply to all
generations forever, David should not have been allowed in the assembly. I sustained my
argument that the expression really meant forever by pointing out that in the very next verse
Ammonites and Moabites were also banned from the assembly "even unto the tenth
generation *forever,*" and Fox's only response to the argument was this: "I sincerely doubt
that he has properly interpreted the word *forever* in this passage *but will not deal with this
in this treatise.*" So this is apparently the way that Fox refutes arguments. He simply asserts
that they are erroneous and then chooses not to deal with them.

An article from Mr. Fox would not be complete without a syllogism to illustrate how he
thinks atheists reason, so he didn't disappoint us. He said that the following premise represents
the way that I reasoned in my reply to him:

If the Bible used the word "son" in a broader sense than a literal son in a genealogy,
then the word "son" is used in a broader sense than a literal son.

All I can say is that I certainly hope that this was not the case. Beginning an argument like
this would be like saying, "If robins can fly, then robins can fly." I suggest that Mr. Fox
reconsider my articles and try to see that I was really arguing like this:

First Premise: If it is the case that the words "son" and "begot" were used literally
throughout all of the biblical genealogies of Aaron and David, then no gaps are in the
listings of the generations in the Aaronic and Davidic genealogies in the Bible.
Second Premise: It is the case that the words "son" and "begot" were used literally
throughout all of the biblical genealogies of Aaron and David.
Conclusion: Therefore, no gaps are in the lists of the generations in the Aaronic and
Davidic genealogies in the Bible.

Volume 1990 - 2002 Issue


Page 727 of 2049
Skeptical Review Edited by Farrell Till
As I pointed out above, my first response to Fox contained very detailed analyses of these
genealogies to show that there is no basis at all for assuming that the writers were at any time
using the words "son" and "begot" figuratively in listing the generations in the genealogies.
Mr. Fox has complained about my "ramblings," but he has not even attempted to show that
my conclusions about this were inaccurate.

Mr. Fox defended his own reasoning with the following syllogism:

First Premise: If God exists and the Bible has evidence of being inspired of God, then
the Bible is inerrant.
Second Premise: God exists and the Bible has evidence of being inspired of God.
Conclusion: Therefore, the Bible is inerrant.

If this is indeed his position, I would dearly love to see him prove the second premise of this
syllogism. Since Mr. Moffitt is showing little enthusiasm to accept the invitations from Ohio
and California to debate the existence of Yahweh, perhaps Mr. Fox would like to do it. As for
his claim that "the Bible has evidence of being inspired of God," if he will present that
evidence in an article or even a series of articles, I will publish them with my simultaneous
responses. His refusal of this invitation will tell us just how much he really believes this
claim.

If he thinks the evidence for the divine inspiration of the Bible is as overwhelming as the
evidence of guilt in his hypothetical example at the end of his article, I would think he would
welcome the opportunity to present it.

New Biblical Archaeology


by Louis Rushmore
Reprinted from Therefore Stand....

Not unlike hermeneutics which is being supplanted by the infamous "new hermeneutics,"
biblical archaeology is being revamped with a "new" biblical archaeology. In both cases, the
old, tried, proven and biblical defensible standards are being wholly undercut and discarded--
by "scholars" who demonstrate their agnosticism or worse at every juncture.

Strangely, many "biblical archaeologists" today are open enemies of the Bible. That being so,
the term "biblical" should be dropped. Further, the propositions of both "new hermeneutics"
and "new biblical archaeology" contradict the respective topics of hermeneutics and biblical
archaeology.

James K. Hoffmier, a professor at Wheaton College, summarizes the perspective of new


biblical archaeologists.

Volume 1990 - 2002 Issue


Page 728 of 2049
Skeptical Review Edited by Farrell Till
The result of their scholarly investigations has been that virtually all that the Bible has
to say about the early history of Israel has been rejected.... Gone are Abraham, Isaac,
Jacob, Joseph, Moses, Joshua and now even David. The Egyptian sojourn and exodus
stories, along with Sinai wanderings and Joshua's military entry into Canaan, have
been reduced to retrojections or inventions by later Biblical writers ("Of Minimalists
and Maximalists," Biblical Archaeology Review, Vol. 21, No. 2, March/April 1995, p.
22).

These and other biblical events, places and people are touted as "fictional" (Ibid.). New
biblical archaeologists picture "... the emergence of Israel from varied groups of pastoral
nomads, sedentary farmers and possibly even urban families, mainly of local Canaanite
origin" (Shlomo Bunimovitz, "How Mute Stones Speak: Interpreting What we Dig Up,"
Biblical Archaeology Review, Vol. 21, No. 2, March/April 1995, p. 67). The Old Testament is
equated with "superstition" and "folk religion" (Ibid., pp. 67, 96).

New biblical archaeologists do not believe that the Bible is the inspired Word of God (at least
not by the *ordinary definitions* to which we subscribe). Therefore, they are predisposed
NOT to find correlation between archaeological discoveries and the Bible. Of course, new
biblical archaeologists accuse "old" biblical archaeologists of being predisposed to find
correlations between archaeological discoveries and the Bible-- even if they must forge those
comparisons. God's Word does not require fraudulently devised eternal [sic] evidences to
defend it. However, legitimate external evidences of the Bible's veracity--which may be
uncovered through biblical archaeology-- deserve fair consideration.

John H. Morison, Professor of New Testament Studies and Winn Professor of Ecclesiastical
History at Harvard Divinity School, author and Lutheran minister, has a loathsome view of
God's Word. In his recent article in Biblical Archaeology Review, he:

(1) shudders at the thought: ...that the Hebrew Bible... can be understood properly by
both Jews and Christians only if it is acknowledged that its ultimate meaning is seen
as a prophecy for the fulfillment in Jesus Christ... that all Jews should be converted to
Christianity... there is neither a historical nor a theological justification for such claims
("Historic Mistakes Haunt the Relationship of Christianity and Judaism," Biblical
Archaeology Review, Vol. 21, No. 2, March/April 1995, p. 26).
(2) Morison quotes a fellow Harvard colleague and proceeds to defend the statement:
"Both the canon of the New Testament and the Mishnah are a tragic historical
mistake" (Ibid.). He adds: It is a simple historical fact that Jesus was an Israelite from
Galilee, and that he understood himself to be nothing else but a prophet in Israel and
for Israel--a venerable tradition, and he was not the first of these prophets of Israel
who was rejected and persecuted-- though he was tried and executed by the Romans,
not by the Jewish authorities (Ibid.).
(3) According to Morison, Christianity and modern Judaism resulted from an
unauthorized disruption in Judaism. A new religion (Christianity) was never intended
by God or the apostles. "The Pharisee Paul," though he did not envision a new
religious group, through his teaching inadvertently drove a wedge between Jews and
Jewish-Gentile groups, by which "the heirs of the Pharisaic tradition" caused two
distinct groups to form: Jews and Christians (Ibid., pp. 26-27).

Volume 1990 - 2002 Issue


Page 729 of 2049
Skeptical Review Edited by Farrell Till
Even the editor of the magazine Biblical Archaeology Review is critical of "belief in inerrancy
of the Bible" ("A Short History of BAR," Biblical Archaeology Review, Vol. 21, No. 2,
March/ April 1995, p. 38). Four sentences within two paragraphs by another renown
archaeologist show the link between new archaeology and new hermeneutics-- which
produces liberal theology. Strikingly, the following sentiments are echoing in our brotherhood
too.

I think that it is time for us to stop fooling the people, making them think that there is just one
Bible and that OUR Bible committee got closer to it than THEIR committee did.... Must we
continue to pretend that only our group is right denominationally and others are not right, and
it is just too bad about others?... The Hebrew text is still in the process of standardization, but
I wonder if it would not be proper for there to be an effort afoot to provide our people with the
differences where they exist and let them see that there have been differences all along. I have
been told by some that that would just destroy the Bible because lay folk still want to think of
the Bible as somehow "inerrant" (James A. Sanders, "Understanding the Development of the
Biblical Text," The Dead Sea Scrolls After Forty Years, pp. 70-71).

Funny, if it were not so tragic, biblical archaeology has become the very thing that when it
initially came forth it was intended to counteract. Biblical archaeology sprang from
Palestinian archaeology as a calculated effort to offset German "higher criticism" of the Bible.

Archaeologists working in Palestine between the end of the 19th century and the First World
War are generally regarded as the "Founding Fathers" of Biblical archaeology.... The "Golden
Age" of Palestinian archaeology--the period between the two World Wars--was dominated by
American-style "Biblical archaeology" as championed by William Foxwell Albright and his
disciples (Bunimovitz, op. cit., pp. 60-61).

J. W, MacGarvey was a valiant opponent of higher criticism--especially in the Lord's church.


His book, *Lands of the Bible*, is an enduring testimony of his appeal to external evidence
with which to confirm the Bible against liberal German theology.

However, a new generation of "scholars" in "the 1960's and early 1970's" arose which dubbed
themselves "new biblical archaeologists". They dismissed the archaeological work of their
predecessors as "simplistic" or otherwise faulty. Former biblical archaeologists were ridiculed
for engaging in "circular reasoning" for their acceptance of "... both archaeology and the bible
as essentially trustworthy sources of historical information" (Ibid., pp. 59, 62).

(Louis Rushmore, 4325 Southeast Drive, Steubenville, OH 43952- 3353.)

EDITOR'S NOTE: For a response to this article, see the next page.

The Old Logic

Volume 1990 - 2002 Issue


Page 730 of 2049
Skeptical Review Edited by Farrell Till
by Farrell Till
I think we can safely assume that Louis Rushmore didn't think very much of the March/April
1995 edition of Biblical Archaeology Review. Before I say anything else about Rushmore's
denunciation of modern archaeology, I should explain his comments about the "new
hermeneutics." Rushmore is a preacher in the old-guard faction of the Church of Christ that
opposes a liberal movement within the denomination to reexamine some of its hardline
doctrinal positions that have isolated it from fellowship with other churches. As a first step
toward mending the breach that divided the old Campbellite movement into the Church of
Christ and the Disciples of Christ, this liberal movement has in particular advocated taking
another look at the scriptures on which the Church of Christ has based its exclusion of
instrumental music from worship. The old-guard contemptuously refers to this movement as
"the *new* hermeneutics."

From what I have read on the subject (and also heard in lectures that were held in conjunction
with the Portland, TX, debates I participated in), the old- guard preachers aren't willing even
to consider the possibility that their doctrinal beliefs have been based on incorrect
interpretations. They simply assume that the "old" way is the right way. They declare the
"new hermeneutics" wrong, no ifs, ands, or buts about it.

In Louis Rushmore's article, we see the same attitude toward modern archaeology. Biblical
archaeology was begun at a time when the divine inspiration of the Bible was taken for
granted. Higher criticism had dared to challenge traditional views of the Bible, but, like all
ideas that break with the past, these critical works had not received much attention. Religious
institutions had also done their best to keep these liberal ideas from their general membership,
so it should not be surprising that the first biblical archaeologists were people who believed in
a divinely inspired Bible. Having entered the field with the presupposed opinion that the
stories in the Bible were historically accurate, they tended to see what they expected to see. It
is understandable, then, that most of their conclusions were influenced by what the Bible said
and not necessarily by what the evidence indicated.

With time, that changed. The conclusions of higher criticism gained respect and withstood
attempts to discredit them. More and more, archaeologists began to see in their excavations
indications that the Bible record wasn't always accurate in what it had reported, until it
became rather commonplace to see archaeologists openly disputing biblical accounts that had
once been revered as sacred history.

This turnabout is what Mr. Rushmore contemptuously calls the "new" archaeology. An
objective person, seeing this turnabout, would advocate juxtaposing the conclusions of both
the "old" and the "new" archaeologists and applying scientific methods of analysis to the
evidence unearthed by both to see which view is more likely the correct one, but Mr.
Rushmore showed no willingness at all to do this. He just summarily pronounced the "new"
archaeology wrong. He ridiculed Professor John H. Morison, who had apparently said in
Biblical Archaeological Review that he shuddered at certain traditional views of the Bible that
some Christians have, but I am not at all embarrassed to say that I shudder to think of what
kind of world we would be living in if our human ancestors had tenaciously clung to the old
and stubbornly resisted the new. Progress can occur only when some dare to try the new.

Volume 1990 - 2002 Issue


Page 731 of 2049
Skeptical Review Edited by Farrell Till
If Mr. Rushmore would dare to give the "new" archaeology a chance, he might learn that
modern archaeologists have good reasons for challenging the accuracy of some biblical
accounts of history. Joseph Callaway had been a conservative Southern Baptist and former
professor at Southern Baptist Theological Seminary before he went to Israel to engage in
archaeological work. He excavated the ruins of the ancient city of Ai, whose destruction by
Joshua's army is alleged in Joshua 8. According to biblical chronology, the sacking of Ai
would have occurred around 1426 B. C., but Callaway's work indicated that the Bible was
inaccurate in this claim. Callaway's findings were reported in Mr. Rushmore's favorite
publication, Biblical Archaeology Review:

The evidence from Ai was mainly negative. There was a great walled city there
beginning about 3000 B. C., more than 1800 years before Israel's emergence in
Canaan. But this city was destroyed about 2400 B. C., after which the site was
abandoned.
Despite extensive excavation, no evidence of a Late Bronze Age (1550- 1200 B. C.)
Canaanite city was found. *In short, there was no Canaanite city here for Joshua
to conquer* ("Joseph A. Callaway: 1920-1988," November/December 1988, p. 24,
emphasis added).
This article, written in memory of Callaway's dedication to archaeological research, quoted
what Callaway himself had announced after nine years of excavations at the Ai ruins:
Archaeology has wiped out the historical credibility of the conquest of Ai as reported
in Joshua 7-8. The Joint Expedition to Ai worked nine seasons between 1964 and
1976... only to eliminate the historical underpinning of the Ai account in the Bible
(Ibid., p.24).

The matter before us, then, is quite simple. A Southern Baptist professor and archaeologist
spent nine years in painstaking excavations at the site of ancient Ai and concluded that the
biblical account of Joshua's destruction of this city never happened. Louis Rushmore has
spent no time digging at Ai (I venture to say), but he will probably dismiss Callaway's
conclusion as nothing but "new archaeology." That's all I need to say about whose opinion in
this matter deserves the more serious consideration.

The books of Exodus, Numbers, and Deuteronomy claim that 2.5 to 3 million Israelite slaves
left Egypt with pharaoh's army in pursuit, crossed the Red Sea through a path that Moses had
miraculously cut through the water, and then spent 40 years wandering in the Sinai wilderness
before finally entering Canaan. Along the way, ALL of the adults who came out of Egypt died
in the wilderness because of a rather petty offense that had angered their god Yahweh.
Anyway, that's the story... according to the Bible. If there is any truth to it, one would think
that archaeologists could rather easily find evidence in the Sinai that 2.5 million people had
spent 40 years there camping, breaking camp, moving on, making camp again, etc. For one
thing, two million Israelites died in the wilderness if this story is true, so surely serious
archaeological work would unearth some of their grave sites.

But that has not been the case, even though serious archaeological work has been done in the
Sinai region. Eliezer Oren is an Israeli archaeologist who spent 9 years digging in the Sinai
for evidence to support the biblical account of the wilderness wanderings. In a lecture at the
Royal Ontario Museum in Toronto, Canada, he reported that "his efforts at more than 80 sites

Volume 1990 - 2002 Issue


Page 732 of 2049
Skeptical Review Edited by Farrell Till
in the Sinai from 1972 to 1982 had not turned up any support for the historical accuracy of
when the exodus was supposed to have occurred" (Barry Brown, News Toronto Bureau,
February 27, 1988). Referring to "papyrus notes" he had found that mentioned the sighting of
two runaway slaves, Oren said, "They were spotted, [but] 2.5 million people with 600,000 of
military age weren't."

To Mr. Rushmore, of course, this is all just "new" archaeology, but I wonder what evidence
he can cite from "old" archaeology that would give logical reasons to believe the exodus did
happen?

From The Mailbag

Please renew my subscription to The Skeptical Review. I'm very impressed with what you do
and how well you do it. I'm really exhausted by all the lies that we've been fed over the years,
and it's nice to know that you're there.

(David C. Mitten, 543 Sicklerville Road, Williamstown, NJ 08094- 1214.)

EDITOR'S NOTE: It's also nice to know that some are waking up to the lies, and our mail is
telling us that their numbers are growing.

From my experiences with Christian fundamentalists, I had come to the conclusion that it was
an exercise in futility to attempt to carry on a meaningful dialogue with anyone who was
firmly convinced that they had a monopoly on the absolute truth (especially when such a
conviction is thought to be prerequisite to the attainment of blissful immortality). After
reading the testimonials of several reformed bibliolaters in my first three issues of The
Skeptical Review, it was reassuring to see that at least some religion addicts are capable of
rehabilitation. It is inconceivable that any thinking person could fail to experience at least a
twinge of doubt after reading your thorough and persuasive arguments in opposition to the
inerrancy of the scriptures. One can only hope that you will have many more such success
stories to relate in the future.

(Jack DeBaun, 635 Vedelwood Drive, Sandpoint, ID 83864).

EDITOR'S NOTE: At times, it does seem pointless to try to reason with Bible
fundamentalists. However, I like to remind skeptics who despair of ever getting through to
any of them that sometimes fundamentalists do change. I'm sure that any rational person who
had tried to discuss the Bible with me in the '50's (during the heyday of my activities on
behalf of fundamentalism) would have thought that I was a hopeless case. One reason why I

Volume 1990 - 2002 Issue


Page 733 of 2049
Skeptical Review Edited by Farrell Till
give publishing preference to letters from ex-fundamentalists is to show those who were never
fooled by it that there is hope of getting through to some biblicists, yet I admit myself that the
odds are certainly against getting them to see the folly of their beliefs.

One thought about Mr. Fox's commentary. He really put his foot into it. He states as a matter
of fact that the genealogists didn't get the genealogies correct. Isn't that a mistake? End of
argument. You win. There is at least one mistake, and Mr. Fox has so graciously pointed it
out.

I mean, didn't he? If he questions the Bible and the names and the verses, and calmly states
that these aren't the same and that the copyist put the wrong genealogies in, I mean--well, I
mean, what the hell did he mean?

Well, thanks for everything. Keep up the good work.

(Douglas L. Smith, P. O. Box 513, Tiffin, OH 44883.)

EDITOR'S NOTE: Admittedly, the inerrantist position in matters like this is hard to
understand, but in fairness to them, we should try to understand what they are arguing. They
believe that God inspired Bible writers to communicate his "truths" in the idioms of their
time. The contemporaries of biblical writers considered bats birds, so it was appropriate for
the Leviticus writer (11:19) to classify bats as birds. In biblical times, the Red Sea and its
arms now known as the Gulf of Aqabah and the Gulf of Suez were all called the "Red Sea";
hence, it would be incorrect to accuse the Bible of error for saying that the town of Ezion-
geber was located on the Red Sea (1 Kings 9:26) on the grounds that it was really situated on
the Gulf of Aqabah. If the people of that time called the Red Sea and all of its arms the Red
Sea, then it was appropriate for Bible writers to say that Ezion-geber was by the Red Sea. I
personally think that these arguments are valid and that we should avoid citing such examples
as cases of Bible errors. Indeed, if the Bible had said that Ezion-geber was located on the Gulf
of Aqabah, we would have grounds to claim a biblical error on the grounds that Aqabah was a
name that was not used when 1 Kings was written, and so it would have to be an anachronism
that indicates the text had been tampered with at a later date.

So what does all this have to do with Fox's claim that generations were omitted in the Exodus
6 genealogy of Aaron? The claim is that this was a common practice of the times in recording
genealogies. Genealogists would sometimes list only the names of important figures in the
history of Israel, and so since the Bible was written in accordance with the language and
customs of the common people, it would not have been erroneous to say that Aaron was the
"son" of Amram when possibly he wasn't a literal son but only a grandson or even a great-
grandson. Inerrantists who so argue will cite such references as Matthew 1:1 where Jesus was
called the "son of David," and David was called the "son of Abraham" in a context where the
word "son" was obviously not being used literally.

I find less merit in this argument than the examples above, because (1) when "son" was used
figuratively in the Bible, the context usually made the figurative use quite obvious, (2) there is

Volume 1990 - 2002 Issue


Page 734 of 2049
Skeptical Review Edited by Farrell Till
nothing in the context of the Exodus 6 genealogy that implies the figurative use of the word
"sons," so no one reading the text would see a need to interpret the word figuratively were it
not for embarrassing inconsistencies with other passages that result from interpreting it
literally, and (3) there are many genealogies in the Old Testament that list minor figures that
are mentioned nowhere else in the Bible except the genealogies. One has only to examine the
genealogies in the opening chapters of 1 Chronicles to see that this is true.

Besides these problems with Mr. Fox's skipped- generation argument, my two-part response
to Fox's article (Fall & Special 5th Editions of 1995) contained very detailed information from
the Bible that showed the writer at least thought he was skipping no generations in the Exodus
6 genealogy. Mr. Fox never made an attempt to deny this information.

I have been freeloading (electronically) your excellent publication for years. It's about time I
actually subscribed, so please find enclosed a check for next year, plus a sum that I hope
covers all back issues (if it does not, please let me know what it will take). The Skeptical
Review is worth far, far more than the subscription rate.

Concerning one issue's article (second issue, 1995, I think), entitled "More Trouble for the
Perfect-Harmony Theory," you questioned the allegation of Yahweh's anger at David for
taking a census. Despite the conflict between verses (2 Sam. 24:1; 1 Chronicles 21:1) that
state that both Satan and God "Moved David to number his people"-- and I grant you that it is
often difficult to tell the two apart--the anger of Yahweh can be "understood" (if that word
can be applied here) in the context of Genesis 15:5, 22:7, and 32:12. Since God said the
people were as uncountable and numerous as the stars, I would think that a zealot or two
would have been filled with self-righteous anger for Yahweh's sake. Also, a census would
probably have shown how many pagan foreigners were living in the nation, contrary to
Yahweh's dicta. After all, since Yahweh was said to have given the land to his chosen people,
how could the resident pagans be explained?

Currently, I am browbeating an inerrantist over the issue of the forged Josephus paragraphs in
Antiquities. He asserted that no scholars and historians dispute the authenticity of the
passages, so I gave him a very brief list of scholars that do (if I am not very much mistaken:
Paul Winter, Louis H. Feldman, S. G. F. Brandon, Morton Smith, James H. Charlesworth,
Carlo M. Martini, Wolfgang Trilling, A. M. Dubarle). Of course, the validity of an argument
does not rest upon the number of people who make it: however, that does place the burden of
evidence upon the nay-sayers who claim the passages mentioning Jesus in Josephus' texts are
authentic.

Given that Christians in previous centuries (just as in this century) tended to reconstruct
history according to their political agenda, I think it is valid to assume that any historical texts
that conflict with the Bible are probably authentic.

(Rev. David Michael Rice, 34041 "B" Street of the Silver Lantern, Dana Point, CA 92629-
2660, e-mail shy.david@edenbbs.com)

Volume 1990 - 2002 Issue


Page 735 of 2049
Skeptical Review Edited by Farrell Till
EDITOR'S NOTE: Although we appreciate Rev. Rice's paying for back issues he had read on
the WWW (Worldwide Web), it wasn't necessary to do so. I cooperate with Jeff Lowder and
Brett Lemoine to make all TSR articles available for posting on the web. My desire is to
dispense information, so for that reason, I don't copyright materials or try to make a profit on
subscription fees. Those who have access to the web will find all TSR articles at
http://www.infidels.org/library/magazines/tsr

You and your associates are doing a magnificent job in getting the word out concerning
biblical errancy. Having endured both the Baptist and Pentecostal indoctrinations, I am now a
card-carrying freethinker. Your publication is an arsenal for rationality and deserves a far
wider circulation than Christianity Today or The National Enquirer. Too bad "enquiring"
minds aren't a bit more skeptical. Put me down for a two-year subscription.

(Larry Forest, 1019 East 35th Pl., Tulsa, OK 74105.)

You argued long and well with Marion Fox to show that David should not "enter into the
assembly of Yahweh" because he was within ten generations of Judah and Tamar. While I am
satisfied your argument is valid, Fox finds he may have a loophole. Wouldn't it be easier to
say David was not fit to "enter the assembly of Yahweh" simply because his great-
grandmother was Ruth, the Moabitess? Deuteronomy 23:3 excludes Ammonites and Moabites
"to their tenth generation," and Ruth was only four generations back from David.

Isn't it true that heritage came through the mother because her identify was certain, while the
identify of the father might be in doubt? Certainly Fox can't argue against the idea that David
was at least part Moabite.

(Fred Acquistapace, 3505 Stony Point Road, Santa Rosa, CA 95407.)

EDITOR'S NOTE: Yes, as I pointed out in my responses to Fox's articles, David was given a
double whammy by the restrictions in Deuteronomy 23. He was both a 9th-generation
descendant of Judah's bastard son Perez, and he was just three generations removed from the
Moabitess Ruth. The latter whammy alone would have banned him from the assembly
*forever* (Dt. 23:3). This is just another example of biblical inconsistency that inerrantists
like Fox bend over backwards to keep from admitting.

I continue to enjoy The Skeptical Review. It's well written, polite, stimulating, education, and
has all those other "traditional values." (grin)

I'm writing to suggest that it might be useful or beneficial to readers if you devoted a small
section of each issue to letters discussing the psychology of inerrantists and fundamentalists.
Indeed, it seems to me that several comments I've seen in TSR cry out for such a section.

Volume 1990 - 2002 Issue


Page 736 of 2049
Skeptical Review Edited by Farrell Till
Consider, for example, the consistent way that fundamentalists seek to avoid public debate or
discussion of their views. On a strictly logical basis, you'd think they would welcome such an
opportunity, since it would allow them to reach an audience they ordinarily don't reach and
possibly make converts among the folks who (from *their* perspective) are most in need of
hearing the salvation message--those of us who laugh at fundamentalists and at the idea of a
Bible that has any truth or usefulness at all.

Yet not only do they not seek those opportunities, they actively shun them! Why? Could it be
that they understand very clearly that in an honest debate in public view they will come out
looking very foolish? If so, doesn't that suggest that perhaps some other agenda is at work
here besides simply "preaching the gospel"? For example, could it be that the agenda is to
avoid public discussion of their views, which carries the risk that believers might see that
there is another side to the story, and maybe as well to keep shoring up the beliefs of the
faithful? Moreover, in my observations of fundamentalists, they seem especially concerned
about the issue of "control." It's a concept that keeps coming up time and again in their
speeches and writings. Could it be that part of their agenda is manipulating others?

It seems to me that some discussion of this aspect of fundamentalism might shed some light
on the other material in TSR.

(Howard A. Karten, 40 Woodland Parkway, Randolph, MA 02368, e-mail


howard.karten@channell.com )

EDITOR'S NOTE: I appreciate Mr. Karten's letter, because it states a view that I have
expressed many times in past issues of TSR. The primary goal of evangelical Christianity is to
evangelize or in the words of Jesus to "preach the gospel to every creature." The Skeptical
Review offers Bible fundamentalists free space for their articles and an audience of several
hundred subscribers that they would be unlikely to contact in any other medium, certainly not
in their own publications, yet it is difficult to find a Bible fundamentalist who is willing to
accept our offer.

Having reached a point in the growth of TSR where heavy demands are made on my time, I
am less cynical about this than I once was, because I understand what it is like to be too busy
to do everything one would like to do for a cause he/she is devoted to. However, I am
confident that if religious publications made space available to us, there would be at least
some skeptics and atheists who could find the time to accept the offer. I can only believe that
Mr. Karten has put his finger on the reason why most inerrantists avoid public forums that
require them to defend their positions against informed opponents. They recognize that such
forums carry with them the risk of having the weaknesses in their position exposed.

In my opinion, Mr. Karten is exactly right in what he says about the desire of fundamentalists
to be in control. I have participated in several debates with fundamentalists, and in each one
of them my opponent insisted upon dictating the rules of discussion. Almost always they want
something in the rules that will give them the advantage.

Volume 1990 - 2002 Issue


Page 737 of 2049
Skeptical Review Edited by Farrell Till
I can only guess how busy you are publishing your newsletter, preparing for debates, etc., so I
will be brief in my comments. You will never know just how extremely grateful I am for your
paper! I am another former fundamentalist, and I too have never been so happy as when I
finally removed the last vestiges of guilt resulting from the dogma of the Christian religion.

Please find a donation enclosed as a small token of my appreciation for your paper. It is worth
every penny. (Please understand that this is a donation, and I am not prepaying my
subscription for future years. Bill me per your normal schedule for next year's subscription.)

Again, thank you so much. Please send a subscription to my friend at the address below.
(Name and address omitted.)

(Chuck Casey, 2841 Wren Avenue, Elida, OH 45807- 1469.)

EDITOR'S NOTE: Mr. Casey found out just how busy I am when on December 14, 1995, I
found his letter of July 31, 1995, mislaid in a stack of mail to answer rather than the stack
with subscription requests and checks. His letter came amidst a flood of subscription requests
from the ads a subscriber ran in Bible Review last summer. I have been playing catch-up for
several months now, and I have just about despaired of ever getting my mailed cleared away.

Although we do not directly solicit them, many subscribers do send contributions as Mr.
Casey did. Those of you who do may be interested in knowing that Skepticism, Inc., is
recognized by IRS as a nonprofit organization, so all contributions are tax deductible.

I am gravely concerned about you, since I have not gotten word back from you. I've written
you twice since the summer. Have you received those other two letters? Volume 6, Number
Two, Spring 1995, "The Wisdom of the World," was the last issue I ever received. Can you
please bring me up to date? You know I truly treasure your publication.

I am genuinely concerned about your health and well being. Please now take the time out to
write me so that I know you're all right or write and inform me if you're not up and at them
presently. Thank you for your help.

(William Kotis, 2199 Kamehameha Highway #4, Honolulu, HI 96819.)

EDITOR'S NOTE: I don't know if I can say that I am "up and at them presently." Sometimes I
wonder, but the note after the letter above explains why much of my mail has to go
unanswered. One of my chief regrets over the success of TSR is that I no longer have time to
answer letters as I once did. In addition to the paper, I also own a "list" on the internet where
biblical errancy is discussed and debated. This is another opportunity that I feel that I have to
give priority to, but I would like for all subscribers to understand that if you write to me and I
don't answer, it isn't because I don't want to. It is a matter of finding the time.

I regret that Mr. Kotis failed to receive the last two issues of TSR. The problem has to be local
delivery, because his name is still on our address list. The issues that he didn't receive have

Volume 1990 - 2002 Issue


Page 738 of 2049
Skeptical Review Edited by Farrell Till
been sent to him by first-class mail. If this should happen to you, please let us know. We will
send replacement copies at no charge.

I am writing you today to renew my subscription to your magazine. I received the current
issue (Vol. 6, No. 5) yesterday evening (Nov. 9) and have just finished reading it for the
second time (Nov. 10). I don't want to miss a single issue, and I look forward to receiving The
Skeptical Review every other month in the coming year. Three months between doses of
sanity is just too long of a dry spell.

I was particularly interested in your article discussing the American Society of Religious
Concern. As president of Pittsburgh Secular Humanists, a local humanist group associated
with the Council for Democratic and Secular Humanism, I had received a rather strange
subscription offer from them accusing me of stupidity and offering me a chance to pay money
for additional insults and humiliation. A somewhat different approach to salesmanship, I must
say.

I was planning to send you a copy of the ad, asking if you knew anything about them. I had
even thought of sending them a letter telling them about your publication and pointing out
how much more "Christian" your free first-year trial subscription was. Your article saved me
the effort.

By the way, as an atheist, I am sometimes a little hazy on what is in the Bible, but I do seem
to remember something about Joshua ben Joseph (aka Jesus) making dire predictions
concerning the fate of people who say to their brother, "You fool!" Am I remembering this
correctly? And considering their attitude, I was wondering just how do they define the word
"love."

(Victor E. Bernard, 405 Nike Drive, Penn Hills, PA 15235.)

EDITOR'S NOTE: All subscribers to TSR who have had letters published in the Mailbag
column have probably received letters from the American Society of Religious Concern. Just
before the deadline to send the Special 5th Edition of 1995 to press, we received a letter from
this organization challenging us in its inimitable, insulting way to take on its staff of Christian
"apologists." The organization claimed to be the publisher of a paper that had 3,300
subscribers, and the literature we (and some of you, no doubt) received indicated that it was
beginning a new paper that would be devoted to showing how stupid atheists were. All of this
information was accompanied by a letter from "H. Jate," who purported to be the editor of the
new paper. Since I had no reason to doubt the "sincerity" of the letter, I pulled an article that
had already been set up for the Special 5th Edition in order to make room for the article about
ASRC that was published on page 11 of that issue.

Since I was busy at the time trying to cope with an influx of subscriptions that were coming
from the ads in Bible Review, I wasn't immediately able to write an article for this new
publication that called itself The Stupid Christian Chronicle. This resulted in another letter
from the editor, only this time the "editor" was someone named Dennis Conley. This

Volume 1990 - 2002 Issue


Page 739 of 2049
Skeptical Review Edited by Farrell Till
prompted me to write to Conley and ask for clarification of some points: Just who was really
the editor of this paper? What was the name of the parent publication that allegedly had 3,300
subscribers?

A reply from Conley dated October 16th contained some information that led me to suspect
that this was a bogus organization. Conley's letter said that "(t)he parent organization that
controls ASRC decided to disband and reorganize." I had to wonder why an organization as
successful as the previous communications had claimed would want to disband. "That
organization controls five different divisions," Conley said, "all larger than ASRC." The letter
went on to claim that "(b)ecause the reorganization would automatically put a halt to
publication of the *Chronicle,* a vote was taken among the twenty-one members of ASRC to
decide if we should go along with them or strike out on our own and continue publication."
Well, it seems (according to Conley) that he was the only one who voted to continue with the
*Chronicle.* "To make a long story short (yes, please do)," Conley said, "I am now ASRC.
My request for the name of the parent publication that had allegedly had 3,300 subscribers
was denied. "I was given permission to keep the `names' and continue publication on my own
if I wished," Conley said. "Their only request was that I not divulge any information about
them nor [sic] the previous newsletter." Yeah, sure. If anything ever smelled rotten in
Denmark, this fishy story does.

So it seems that this organization that was going to take on the atheists of the world and give
us the benefit of its 3,300 subscribers has been reduced to just one lone voice that constantly
belittles and insults atheists for being afraid of him. Not one to give up so easily, Conley
claimed that he has worked diligently (written letters, sent out fliers, beaten the bushes) and
obtained 1500+ subscribers for his paper, which he intends to call The Believers Chronicle.
What this all means is that The Stupid Christian Chronicle folded after just one issue.

In all of this talk that is coming from Conley, I see too much evidence of phoniness to take
him seriously. He claims that he sends out hundreds of letters on behalf of his organization,
hand addresses 1500 copies of *TBC,* all by himself, mails them, and still has time for his
job and family responsibilities. Yeah, sure, and both the Republican and Democratic parties
are frantically wooing me to run for president in the next election to save the country from
disaster. In telling me all of this, Conley failed to consider that my experience in editing a
paper with only a few more subscribers than he claims enables me to understand that he isn't
being exactly truthful. I print 1600+ address labels via computer, peel them, and put them on
the papers to be mailed, and I know how much time this takes. Hand addressing that many
copies would take a lot longer, and this is the way that Conley addresses his. What editor of a
newsletter with that many subscribers would not get himself a computer or at least engage the
services of an organization that can provide computerized labels at reasonable cost?

The copies of Conley's paper that I have seen were mailed at first-class rates, an extravagant
waste of money for a paper that could qualify for bulk- rate mailing, but, of course, to mail at
bulk rate a minimum of 200 copies is required. Could it be that Conley sends his paper by
first-class mail because he doesn't have 200 subscribers? I suspect that this is the case,
because all copies I have seen look as if they have been produced on a photocopy machine. I
think if the truth were known, we would find that Conley is making copies as they are needed
to fill the demand, and his claim of having 1500+ subscribers is greatly exaggerated.

Volume 1990 - 2002 Issue


Page 740 of 2049
Skeptical Review Edited by Farrell Till
Meanwhile, he continues to send out insulting letters that seem designed to arouse the anger
of atheists and skeptics to goad them into subscribing to his paper and then sending him
money for the "privilege" of seeing their articles published. My suspicions having been
aroused by the facts that I have presented above, I have sent neither a subscription check nor
an article to Conley, and for this reason, he sends out letters insinuating that I have put my tail
between my legs and run away from his challenge to refute what he publishes in his paper.
Well, in the first place, I have found nothing to refute in his paper, because I have seen
nothing that even remotely resembles an argument. The paper contains nothing that I can see
but strings of assertions for which the writers offer no supporting evidence. If Mr. Conley
would show me an argument, I might consider responding to it.

I send a free subscription to The American Society of Religious Concern, so the least Conley
could do is reciprocate. However, I don't receive Mr. Conley's paper, presumably because I
haven't sent him a paid subscription. Furthermore, if Conley wants to rebut an article in The
Skeptical Review, I will read his article without being paid to do so, and I will publish it
without being paid to do so. If Conley really wants to get a dialog started, let him show a little
willingness to reciprocate. Quite honestly, I don't think that is his desire. I think he wants to
sucker a lot of angry atheists into sending him money to publish their articles. No doubt, I
have some of the facts wrong in this case, but this is my honest assessment of Conley's
publishing enterprise based on the evidence I have seen. If I am wrong, he can set the record
straight.

Meanwhile, I would like for all subscribers to TSR to know that I sent a letter to Mr. Conley
informing him of the demands that The Skeptical Review is now making on my time but
assuring him that if he wishes to organize a debate between him and me or any qualified
representative of the inerrancy doctrine, I will somehow find the time to go to Columbus,
Ohio, to participate in the debate. I don't mind finding time to discuss issues that an audience
of reasonable size will have the opportunity to hear, but with my work schedule being what it
is, I can't build up much enthusiasm for writing an article that 1500 people may, may see but
that probably only a handful at the most will read. In response to my debate challenge, Conley
wrote back and said, "I don't debate. No writer in the *Chronicle* debates." Well, I do, and I
want to make it clear to Conley & Company that I have a standing offer to go to his
hometown to debate him or any representative he may choose. In his letters to skeptics, he
dares them to come out and "play with the big boys." Well, if Conley doesn't debate and if no
writer for the *Chronicle* debates, perhaps he should stop daring us to come out and "play
with the big boys." I have played with some of the biggest, so if Conley doesn't want to play
with me, he has little room to criticize skeptics who haven't fallen for his little con game.
When he gets ready to play, all he has to do is let me know.

The scripture that Mr. Bernard inquired about is Matthew 5:22, where Jesus allegedly said,
"Whoever says, `You fool!' shall be in danger of the judgment." Some translations say "hell
fire" instead of "judgment."

Volume 1990 - 2002 Issue


Page 741 of 2049
Skeptical Review Edited by Farrell Till
I have been receiving The Skeptical Review for the last year. Please continue my subscription.
Also send back issues from 1990 through and including Summer 1994. My check is enclosed.
If there is an error, I will adjust accordingly.

I was a totally sold-out fundamentalist Christian for the past 15 years. I was not a hypocrite. I
totally believed and put into practice the precepts of the Bible. I was honest, never cheated,
faithful, forgave my neighbors, prayed unceasingly, attended church several times a week,
tithed. I've read at least ten versions of the bible and spent much more time in intense study. I
read only Christian books, listened only to Christian music, and spent much time
evangelizing.

The problem was that the precepts of the Bible weren't panning out. There were troublesome
spots that were hard to ignore, since I spent so much time in the bible. The preaching I heard
didn't work in real life. Explanations I heard for the "hard" things in the Bible didn't always sit
well, especially the genocide. I did a report in college on the genocide in Cambodia
committed by the Khmer Rouge, and it made me sick to realize all the atrocities that occurred.
I could see no reason why any humane god would ever allow any such occurrences. Neither
could I see a whole lot of difference between the genocides of the 20th century and those of
biblical times.

My own life was difficult. I came to Christianity with a lot of hard life behind me. I was
sexually abused and raped as a child. I became involved in habitual drug use for ten years,
and I became a teenage, unwed mother, among other things. Now I had five children I was
trying to raise and rather than helping me to keep my family together, the church always
seemed to find ways to take away our time from each other. Added to that, I am manic-
depressive, which in fundamentalist eyes is akin to demon oppression, but when I became a
Christian, supposedly all these things were done away, and I was now a "new" creature in
Christ. In other words, bury it, push it under the rug, it never happened, it isn't happening. I
tried to live for Jesus, to be like Jesus, but Jesus was never satisfied, and Jesus never really
satisfied.

So one day, as I sat by the railroad tracks, having taken an overdose of drugs, thinking about
throwing myself in front of the next train, because after all I was really a demon in need of
going back to hell--as I waited for the train, crying out to God, I suddenly realized that he
wasn't there. I couldn't depend on him, I couldn't trust him, I couldn't "give it all to Jesus,"
because there was no one there.

Things have changed since then. I started to think for myself. It was scary at first. Would God
send me to hell for daring to question his wisdom? Sometimes I'm still not sure of so many
things, but I am sure that the Bible is not the word of any god; it is full of myths and
superstitions. Jesus is not the son of god. I tried him; it isn't true. Christianity is not the way of
God, if there be a god.

I am thankful for the energies you expend in debunking the myths of the bible and
Christianity. Your articles are very well researched and an encouragement to me. Someday I
hope I might be able to discuss some of these topics with my Christian friends and family.
Right now, they don't know what to make of me (of course, they expect that soon I will be

Volume 1990 - 2002 Issue


Page 742 of 2049
Skeptical Review Edited by Farrell Till
back in the fold, despite my assertions to the contrary), and it makes our relationships
difficult.

Fifteen years believing a lie. Pretty sad, isn't it?

If you wish to use my letter, you may do so, but because of its personal nature, please do not
use my real name. Thanks.

(Writer's name withheld as requested.)

EDITOR'S NOTE: Normally, I don't publish letters without giving the name and address of
the writers. In this case, I think it is understandable why I have respected the privacy of this
writer. Those whose letters are published in this column tell me that they receive quite a bit of
mail from other readers. If anyone would like to correspond with this letter writer, we will
forward all mail sent to our address.

Back Page

Address Changes
With the number of subscribers we now have, address-correction fees are becoming a major
operational expense. From the last mailing, we have received 57 notifications for which we
must pay $0.50 each, and they are coming back daily. For this reason, we have dropped our
policy of sending replacement copies by first-class mail to those who move without notifying
us. To avoid interruption in your subscription service, please send us notification if you move.

To Our Inerrantist Readers


The front-page editorial in the Special 5th Edition of 1995 and editorial comments on letters
in the "Mailbag" column of this and previous issues have pointed out the increased demands
on the editor's time that have come with the growth of The Skeptical Review. Ads that ran in
Bible Review last summer brought in several hundred new subscribers, and messages about
our work that various readers have posted on the internet have brought in other subscriptions
as well as requests for information. Many readers, including even a few inerrantists, are
sending articles for publishing consideration, and we have already received more of these than
we can possibly publish. Reading them to look for the ones appropriate for publication puts
another demand on the editor's time.

Volume 1990 - 2002 Issue


Page 743 of 2049
Skeptical Review Edited by Farrell Till
Our attempts to explain this problem have been graciously received by most readers. This is
indicated by the increased number of letter writers who tell us that they just want to share
their thoughts or opinions with us but don't expect personal replies. Needless to say, we
appreciate that understanding.

There are, however, some inerrantist subscribers who see their unanswered letters as an
indication that they have "stumped" us with arguments that cannot be answered. A typical
letter from these writers will be 5 or 6 or even more single-spaced, type-written pages in
which they string together assertion after assertion (just as fundamentalist debaters do in
public forums), which would require several hours to write a point-by-point to response to.
There just aren't enough hours in the day to answer them all. When they go unanswered, the
writers send additional letters in which they gloat over their victories. This has been
especially true of Dennis Conley, whose accusations were addressed in the "Mailbag" column
of this issue, and a group of creationists/ fundamentalists in Pekin, Illinois. No amount of
explanation seems to satisfy them.

To those with this attitude, we offer what we believe is a reasonable proposal. If they will
arrange some kind of public forum in their communities where others will have the
opportunity to hear both sides of the inerrancy issue discussed, the editor of TSR will
somehow arrange his schedule to participate in the forum. This would be a far more practical
use of time than spending hours writing letters that will be read by only one or two people,
whose position on biblical inerrancy has already been shown to be hopelessly inflexible.

If this offer isn't satisfactory, perhaps our inerrantist subscribers who think that their letters
have left us hopelessly defeated would like to consider the opportunity explained in the
following notice.

errancy@infidels.org

The editor of TSR owns a list on the internet on which issues concerning Bible inerrancy are
discussed. (This, by the way, is another activity that requires the editor's personal attention
each day.) The present address of this list is errancy@infidels.org, but this site may be moved
to another address by the time this issue is mailed. To obtain up-to-date information on how
to subscribe to this list, send inquiries to our e-mail address, jftill@midwest.net. For those
who are newcomers to the internet, "subscription" may be a misleading word. A subscription
merely puts you on the list so that you can receive copies of all postings and post your own
messages, but it does not entail cost beyond the expense of being on line long enough to send
and receive messages. This cost is very nominal, but TSR receives nothing from subscribers to
the list.

Your postings on this list, for or against inerrancy, will receive priority from the editor over
regular letters, because answering e-mail is far simpler than responding to "snail mail." At the
same time, everyone on the list will be able to follow all exchanges between subscribers. By
subscribing, any of our inerrantist readers who think that we are dodging their questions and
arguments will have the opportunity to test that opinion before an on-line audience.

Volume 1990 - 2002 Issue


Page 744 of 2049
Skeptical Review Edited by Farrell Till

Volume 1990 - 2002 Issue


Page 745 of 2049
Skeptical Review Edited by Farrell Till

Skeptical Review
Volume Seven, Number Three
May/June 1996
Farrell Till, editor

• How-It-Could-Have-Been Scenarios
A plea for more rational arguments from inerrantists.

• Fulfilled Prophecy: An Unprovable Claim


Farrell Till shows the fallacy in Dr. Ross' attempts to prove the infallibility of the
Bible.

• Jairus's Daughter: Dead but Raised to Live Again


Roger W. Hutchinson creates hypothetical scenarios to back his claim of biblical
inerrancy.

• More How-It-Could-Have-Been Speculation


Farrell Till proposes a *common sense* explanation for biblical discrepancies.

• Hutchinson-Till Debate Is Merely a Tempest in a Teapot


Wilhelm Schmitt makes it perfectly clear that Roger Hutchinson and Farrell Till are
wrong about calculating dates for the 430-year sojourn of Israel in Egypt.

• More Like "As You Like It"


Farrell Till disputes Wilhelm Schmitt's reasoning.

• From The Mailbag


Farrell Till challenges Dennis Conley, editor of The Believer's Chronicle to a public
debate.

• The Back Page


Debates, phonies and internet connections.

Volume 1990 - 2002 Issue


Page 746 of 2049
Skeptical Review Edited by Farrell Till

How-It-Could-Have-Been Scenarios
by Farrel Till
Although previous issues of The Skeptical Review have exposed fallacies in the how-it-could-
have-been hermeneutic tactics that inerrantists invariably resort to in their never-ending quest
to "prove" that the Bible is perfectly harmonious, they still persist in their determination to
use this discredited method of argumentation. First, we had Jerry Moffitt, Jerry McDonald,
and Lindell Mitchell bombarding us with highly unlikely but possible scenarios that,
according to them, remove inconsistencies and discrepancies that result from accepting the
face-value meaning of certain biblical passages. More recently, Roger Hutchinson and Marion
Fox have come on the scene to argue from the simplistic assumption that the mere postulation
of how-it- could-have-been scenarios is sufficient to defend the doctrine of biblical inerrancy.
On pages 4-5 of this issue, we have Hutchinson's latest venture into the Never-Never Land of
how-it-could-have-beens, and joining him on pages 7-8 is Wilhelm Schmitt, who seeks to
shore up the earlier attempts of Moffitt and Hutchinson to prove that there is no discrepancy
in the Exodus 12:40 claim of a 430-year sojourn in Egypt and the Exodus-6 genealogy of
Aaron, which lists only four generations from the time of the Israelite descent into Egypt until
their exodus under the leadership of Moses and Aaron.

An interesting aspect of this 430-year matter is that Moffitt, Hutchinson, and Schmitt have all
offered explanations, yet all of their explanations are different. In other words, Moffitt
proposed one solution, Hutchinson another, and Schmitt still another. It isn't possible for all
three "explanations" to be right, so aren't we entitled to arch a mental eyebrow at a
hermeneutic method whose advocates insist is valid even though they can't even use it
themselves to arrive at a consensus on the real meaning of controversial biblical passages? In
the past, we have pointed out that biblical inerrantists seem to be infected with an any-
interpretation-will-do virus when confronted with the challenge to explain away biblical
discrepancies, and the performance of Moffitt, Hutchinson, and Schmitt confirms that. They
can't agree on what the Exodus writer meant, but they can agree that their contradictory
solutions have removed the discrepancy. In other words, they don't really know what the
Exodus writer meant, but they know that he did not contradict himself. How much sense does
that make?

To those who belong to this any-interpretation-will-do school of hermeneutics, truth isn't


important. The only thing that matters to them is the inerrancy doctrine, and it must be
protected at all costs, even at the cost of common sense and intellectual integrity. That
inerrantists do compromise their intellectual integrity when they defend the Bible with how-it-
could-have-been scenarios is evident in the fact that anyone can use this method to remove
any kind of inconsistency from any written document, secular or religious. To all inerrantists
who doubt this, we issue the following challenge. If they will send us examples of
contradiction or inconsistency in any written documents whose authors are not alive to
explain the meaning of the texts, we will use how-it-could-have-been methods to present

Volume 1990 - 2002 Issue


Page 747 of 2049
Skeptical Review Edited by Farrell Till
possible scenarios that will remove all discrepancies or inconsistencies, and we guarantee that
our scenarios will be no more absurd than some of those that inerrantists have proposed as
solutions to biblical contradictions. This, of course, will give us a great deal of leeway,
because some of the "solutions" to biblical contradictions that inerrantists have proposed
border on the supremely ridiculous. If, however, the method is valid for them to use in
resolving biblical contradictions, it must be permitted for anyone who wishes to remove
textual contradictions from other written documents.

The end result of this how-it-could-have-been approach to hermeneutics is a sort of *reductio-


ad-absurdum* situation, because it leaves us with a mountain of holy books, which teach
conflicting theological doctrines, yet none of the books contains inconsistency or
contradiction, because how-it-could-have-been hermeneutics can be applied to each to
"prove" that all of the books are perfectly harmonious in what they teach. A primary
inerrantist argument is that the inerrancy of the Bible (a claim arrived at through application
of how-it-could-have-been scenarios) proves the divine inspiration of the Bible, because only
a divinely inspired book as complex as the Bible could be inerrant. The entire argument is
deeply rooted in circular reasoning, but for the moment let's concede the point to them. If such
an approach is a legitimate way to prove the divine inspiration and inerrancy of the Bible,
then it would be equally legitimate to use the same approach to prove the divine inspiration
and inerrancy of the Koran or the Book of Mormon or any other holy book. So that leaves us
with a world filled with conflicting religions, all of which can justify their existence by
appeals to inerrant books.

This *reductio-ad-absurdum* situation is a classic example of the logical principle that says
what proves too much proves nothing at all. It just cannot be that all of the conflicting
religions of the world are founded on books that can be shown to be inerrant, so it must be
that the how-it-could-have-been method that bibliolaters use to prove Bible inerrancy is
fundamentally unsound. Its unsoundness, of course, is due to the falsity of the premise on
which it is based. The postulation of a how-it-could-have been scenario does not remove
textual contradiction. It merely poses a hypothesis whose truth would provide a solution, but
in order to prove that it is a real solution, the hypothesis must be tested and proven true. This
is where how-it-could-have-been hermeneutics fail, because inerrantists rarely attempt to test
their hypotheses; they simply assert that the existence of a hypothesis solves the problem of
inconsistency. This is as ridiculous as a scientist who would assert that posing a hypothesis
that might cure disease X is equal to curing the disease, and anyone with a shred of common
sense knows that this isn't true. The disease will be cured only if the testing of the hypothesis
proves that it is true.

This is why the articles that inerrantists like Moffitt and Hutchinson have submitted to The
Skeptical Review have done nothing to resolve the problem of biblical inconsistencies. They
merely assert that such and such could have happened or could have been the intention of a
writer who penned a statement whose face-value meaning results in biblical inconsistency or
discrepancy, but they offer no tangible evidence to support their hypotheses. An established
principle of hermeneutics that is taught in all reputable seminaries and Bible colleges says that
language is to be interpreted literally unless there are obvious or compelling reasons to assign
figurative meaning. Poetic forms like most of the psalms provide obvious and compelling
reasons to interpret language poetically. When, for example, the psalmist said that Yahweh "is

Volume 1990 - 2002 Issue


Page 748 of 2049
Skeptical Review Edited by Farrell Till
my rock and my fortress" (Ps. 18:2), we have every reason to understand that he was not
saying that Yahweh is literally a rock or a fortress made out of brick and mortar. In such
cases, we know to assign figurative meaning to the words. On the other hand, when we read
in Exodus 6 that Kohath was the son of Levi (v:16) and that Amram was the son of Kohath
(v:18), there is no reason not to interpret these statements literally, especially when we find
numerous other genealogies in the Bible that use the same language to describe these
relationships. When we can take many individual listings in this genealogy and confirm by
other passages that biblical writers understood that Amram, Izhar, Hebron, and Uzziel (v:18)
were literal sons of Kohath and that Mishael and Elzaphan (v:22) were literal sons of Uzziel,
it is utter foolishness for inerrantists like Moffitt, Fox, and Hutchinson to insist that *son* was
sometimes being used figuratively in this text. They take this position only to avoid a glaring
discrepancy between this genealogy and the Exodus 12:40 claim that the Israelites sojourned
in Egypt 430 years, but the desire to eliminate embarrassing textual discrepancy is not a
compelling reason to assign figurative meaning to a passage that would otherwise be
interpreted literally. To so argue is to prove inerrancy by assuming inerrancy. We have
pointed this out time and time again.

We are more than glad to give publishing space to inerrantists who want to put their case
before our readers. We do this to try to give our subscribers a balanced view, but there isn't
much balance to be found in a hermeneutic method that has been discredited time and time
again. We hope our inerrantist contributors will take notice of this complaint and in future
submissions try to support their arguments with sound evidence rather than mere speculation.
If there are compelling reasons to assign figurative meaning to a passage whose face-value
meaning will result in inconsistency or discrepancy, let them present those reasons, and we
will consider them. However, if they continue to resort to unverifiable how-it-could-have-
been scenarios, we will see this as evidence that they have no evidence.

Fulfilled Prophecy: An Unprovable Claim


(3)
by Farrel Till
In two earlier installments, I have responded to 10 of the 13 prophecy-fulfillment claims that
Dr. Hugh Ross presented as evidence that the Bible was inspired of God. Before addressing
the remaining claims, we should notice again the characteristics of valid prophecy fulfillment
as they were presented in my first article:

In order to prove--and I mean *prove,* not just surmise--prophecy fulfillment, one


would have to establish four things: (1) the claimant of a prophecy fulfillment is
properly interpreting whatever text he is basing his claim on, (2) the prophecy was
made *before* and not after the event that allegedly fulfills the prophecy, (3) the
prophecy was made not just *before* an event but far enough in advance of it to make

Volume 1990 - 2002 Issue


Page 749 of 2049
Skeptical Review Edited by Farrell Till
educated guesswork impossible, and (4) the event that allegedly fulfilled the prophecy
did in fact happen. When Dr. Ross's claims of prophecy fulfillment are examined in
terms of these four characteristics of valid prophecy, we will see that none of his
alleged prophecy fulfillments can pass muster.

To these criteria, we could also add the obvious requirement that an event allegedly fulfilling
a prophecy must not be one that could have been deliberately contrived in an attempt to bring
about fulfillment. If, for example, the Old Testament prophesied that the Messiah would ride
into Jerusalem on a donkey, as Matthew (21:4-5) claimed, then one would have to prove that
the fulfillment event was completely independent of any deliberate attempt to give the
impression that prophecy was being fulfilled. After all, if someone believed that such an event
had been prophesied, the odds would be one in one that he could get a donkey and
deliberately stage the event in order to claim prophecy fulfillment.

This factor poses a big problem for Dr. Ross's seventh example of prophecy fulfillment. He
alleges that Jeremiah 31: 38-40 correctly predicted "(t)he exact location and construction of
Jerusalem's nine suburbs," the building of which began in 1948, 2600 years after the
prophecy." To assess Ross's claim, we should look at the alleged prophecy:

The days are surely coming, says Yahweh, when the city shall be rebuilt for Yahweh
from the tower of Hananel to the Corner Gate. And the measuring line shall go out
farther, straight to the hill Gareb, and shall then turn to Goah. The whole valley of the
dead bodies and the ashes, and all the fields as far as the Wadi Kidron, to the corner of
the Horse Gate toward the east, shall be sacred to Yahweh. It shall never again be
uprooted or overthrown.

Did construction work that began in the suburbs of Jerusalem in 1948 follow the pattern laid
out in Jeremiah 31: 38-40? I don't really know, but even if it did, to prove his case, Ross
would have to show that it wasn't done by religious zealots (which Israel has no shortage of)
in a deliberate attempt to give the appearance of prophecy fulfillment. Otherwise, it could
well be that a planned effort was made to have the construction work proceed according to the
Jeremiah text so that prophecy fulfillment could be claimed. In such a case, this would be no
more of a prophecy fulfillment than if someone should deliberately contrive to ride into
Jerusalem on a donkey and claim that he had fulfilled Zechariah 9:9 (which wasn't even a
prophecy anyway). In other words, even if it should be true that construction in and around
Jerusalem had proceeded in the pattern presented by Jeremiah, Dr. Ross would have a lot of
proving to do before this could be considered a verified prophecy fulfillment.

That isn't the only problem in this particular prophecy-fulfillment claim. Dr. Ross said that
Jeremiah "referred to the time of this building project as `the last days,' which Ross went on to
interpret as the time period of "Israel's second rebirth as a nation in the land of Palestine";
however, an examination of the text shows that Jeremiah made no reference to "the last days";
he simply said that "the days are surely coming" when the city would be rebuilt. How Ross
gets the "last days" from this is a mystery to me. However, even if Jeremiah had said that this
would happen in "the last days," it would be purely arbitrary on Ross's part to say that this
was a reference to Israel's "second rebirth" in the 1940's, because the New Testament
identifies "the last days" with an epoch that began shortly after the ascension of Jesus (Acts

Volume 1990 - 2002 Issue


Page 750 of 2049
Skeptical Review Edited by Farrell Till
2:17; Heb. 1:2). So clearly Ross is not even meeting the first criterion of valid prophecy
fulfillment, which is the requirement to give reasonable evidence that he is correctly
interpreting the passage on which he is basing a prophecy- fulfillment claim.

A second problem in Ross's claim is the well known fact that Jerusalem was rebuilt during the
time of Ezra and Nehemiah. Since Jeremiah said only that the days were surely coming when
Jerusalem would be rebuilt, why shouldn't the accuracy of Jeremiah's prediction be evaluated
in terms of how the city was rebuilt the first time? Did the construction then follow the pattern
laid out in the Jeremiah prophecy? If not, why not? Since Jeremiah was so obviously
concerned with the time when the Jews would return from 70 years of Babylonian captivity
(Jere. 25:11; 29:10), why should we not believe that Ross's text was actually intended to refer
to the rebuilding of Jerusalem at that time? Why would Jeremiah have skipped over that time
to speak of a rebuilding so far in the distant future that it would offer no hope to the Jews of
his generation? On the other hand, if the rebuilding of Jerusalem under the leadership of
Nehemiah did follow the pattern laid out in Jeremiah 31, that would raise an interesting issue.
Scholars know that the book of Jeremiah underwent considerable revision and redaction (see
"The Jeremiah Dilemma," TSR, Autumn 1990, pp. 6-10). That being true, how can we know
that an editor who was familiar with the reconstruction work at the time of Ezra and
Nehemiah did not revise the Jeremiah text to leave the impression that prophecy had been
fulfilled in the first rebuilding of Jerusalem? In a word, there are far too many unanswered
questions in this matter to accept Ross's claim that reconstruction work in 1948 had fulfilled a
2600-year- old prophecy.

Dr. Ross claims that certain prophecies made by Moses, Isaiah, and Jeremiah were fulfilled in
the dispersion of the Jews. Ross sees the passages he cited as predictions that the Jews would
be taken captive Romans. Ross alleges that the prophecies predicted that the Jews would be
gathered again "to the land of Palestine to reestablish their nation for a second time." The
proof-texts that Ross cited were Deuteronomy 29, Isaiah 11:11-13, and Jeremiah 25:11. The
text in Isaiah speaks of a second recovery of the Jews from "Assyria and Egypt, Pathros and
Cush, Elam and Shinar, and Hamath and the islands of the sea." Since the restoration of the
nation of Israel was accomplished in the late 40's by Jewish refugees who had immigrated to
Palestine primarily from Europe following the Nazi holocaust, I fail to see how Ross can
connect this with a prophecy that predicted a second gathering of the Jews from the locations
cited in Isaiah 11:11-13. None of these places were located in Europe. Indeed, most of them
had lost their geographical identities long before the 1940's.

Since Jeremiah 25:11 refers to a 70-year bondage in Babylon, the only other text cited by
Ross that could possibly refer to a Jewish return to Palestine in the 1940's would be
Deuteronomy 29. Ross alleges that Moses predicted that "the second conqueror... would take
the Jews captive in ships, selling them or giving them away as slaves to all parts of the
world." Ross cited no specific verses in support of this claim, so apparently he meant to say
that this prophecy of Moses was in chapter 28 of Deuteronomy as well as chapter 29. In both
chapters, a long harangue against the Israelites was delivered, allegedly by Moses, but neither
chapter states that they would be taken "captive in ships" and sold or given away "as slaves all
over the world." The only reference to ships in either chapter is in 28:68, but here the
prediction was that the Israelites would be taken "back *to Egypt* in ships." I know of no
event that Ross can cite as a fulfillment of this prediction. Furthermore, this verse declared

Volume 1990 - 2002 Issue


Page 751 of 2049
Skeptical Review Edited by Farrell Till
that when the Israelites were taken back to Egypt in ships to be offered for sale as slaves, "No
one will buy you." Can Ross cite historical evidence that the Jews were ever taken to Egypt in
ships to be sold as slaves but that no one would buy them? I personally know of no historical
events that could be cited as evidence that such a prophecy as this was ever fulfilled.

Dr. Ross further asserted that this chapter prophesied of a captivity by "a fourth world
kingdom," which he then identified parenthetically as Rome, but where does Moses' tirade in
these chapters mention a "fourth world kingdom"? I suspect that Dr. Ross is merely engaging
in wishful thinking, wanting to give his readers the impression that Moses was able to see
thousands of years into the future when Rome would defeat the Jews in the wars of A.D. 66-
70, but if such details as these were present in the prophecy, exactly where can they be found?
This part of Ross's article was conspicuously lacking in specific references to Moses' speech
that would give reasonable evidence that this is what he meant. When all things are
considered, we can only conclude that Deuteronomy 28-29 prophesies of a Roman conquest
of Palestine and subsequent Jewish captivity simply because desperate inerrantists arbitrarily
declare that it does.

Another problem in Ross's interpretation of this "prophecy" is his complete failure to mention
popular scholarly opinions about the dating of the book of Deuteronomy. Biblical inerrantists
cling to the traditional belief that the book was written by Moses, but reputable scholars reject
this view. Most scholars date the preliminary writing of the book at around the 8th century B.
C. and then an editing of it at *circa* 622 B. C. when "the book of the law of Yahweh given
by Moses" was "discovered" during the renovation of the temple (2 Chron. 34:14). Evidence
is seen of a final editing that occurred following the fall of Jerusalem in 587 B. C., and the
long tirade of Moses in chapters 28 and 29 is one of the clues to this final reediting. Many of
the things that "Moses" said in this speech fit conveniently into events that accompanied the
fall of Jerusalem and the subsequent captivity in Babylon. Many proponents of prophecy
fulfillment have cited statements in Deuteronomy 28 as examples that were fulfilled at the
time of Babylon's overthrow of Jerusalem, and they make a much better case for their position
than Ross did for his. They can do this, of course, because Deuteronomy 28 was written after
the fact in the final editing of the book. At any rate, we have to be suspicious of prophetic
interpretations that have been applied to both the Babylonian and the Roman destructions of
Jerusalem. They can hardly be considered resounding evidence that the Bible is a divinely
inspired book.

Probably more printer's ink has been wasted on Daniel 9:24-27 than any other Old Testament
prophecy. It has been interpreted and reinterpreted and applied to various historical events in
which Bible believers have wanted to see evidence of prophecy fulfillment. This widespread
disagreement about the meaning of the passage is by itself enough to cast suspicion on Ross's
interpretation of it or, for that matter, all of the other interpretations too. If this text were
indeed a divinely inspired prophecy, surely the god who inspired it would have stated it
clearly enough to avoid the controversy that has surrounded it. To see that this "prophecy"
was not clearly worded, we have only to read it:

Seventy weeks are decreed for your people and your holy city: to finish the
transgression, to put an end to sin, and to atone for iniquity, to bring in everlasting
righteousness, to seal both vision and prophet, and to anoint a most holy place. Know

Volume 1990 - 2002 Issue


Page 752 of 2049
Skeptical Review Edited by Farrell Till
therefore and understand: from the time that the word went out to restore and rebuild
Jerusalem until the time of an anointed prince, there shall be seven weeks; and for
sixty-two weeks it shall be built again with streets and moat, but in a troubled time.
After the sixty-two weeks, an anointed one shall be cut off and shall have nothing, and
the troops of the prince who is to come shall destroy the city and the sanctuary. Its end
shall come with a flood, and to the end there shall be war. Desolations are decreed. He
shall make a strong covenant with many for one week, and for half of the week he
shall make sacrifice and offering cease; and in their place shall be an abomination that
desolates, until the decreed end is poured out upon the desolator (NRSV).

In Ross's article, he devoted only one paragraph to his explication of this obviously complex
passage and concluded that "(a)bundant documentation shows that these prophecies were
perfectly fulfilled in the life (and crucifixion) of Jesus Christ," yet he said nothing to explain
the many symbolic and figurative expressions in the text. He merely declared arbitrarily that
the life of Jesus "perfectly fulfilled" all details of the prophecies.

The literature on this passage is too extensive to review in a single article, but a good source
of information would be The Interpreter's Bible, which gives a detailed analysis of the book
of Daniel to show, first of all, that it was not written by its namesake who allegedly lived in
Babylon during the captivity but by an unknown author living during the time of the Seleucid
Empire, which arose from the partitioning of Alexander's kingdom after his death. This
writer's purpose was to give his countrymen reason to believe that centuries earlier a prophet
of Yahweh had foreseen the rise of the Seleucid Empire and had predicted the triumph of the
Maccabean struggle for independence against Antiochus Epiphanes, who in 175 B. C. had
succeeded his brother Seleucus IV as ruler of the empire. Textual clues for fixing the date of
authorship at this time are discussed in detail but are too complex to review in this article.
Suffice it to say that a book written under such a pretense as this would be an unlikely place
for a morally perfect deity to put a prophecy of the Messiah's coming.

The Interpreter's Bible also analyzes Daniel 9:24-27 in detail and assigns very sensible
interpretations to the many symbols and figurative expressions in the text. Specific events
from the life of Antiochus Epiphanes are identified to give reasonable evidence that the writer
of Daniel wanted his readers to understand that the "prince" who destroyed "the city and the
sanctuary" with his troops was Antiochus, and this analysis cites passages from the
apocryphal Maccabean books to confirm that these events had occurred. This interpretation
shows how that for a period of three and a half years (half of a week) the holy place was
defiled with sacrifices of swine and other unclean animals. Hence, the conclusion is that
Antiochus Epiphanes was the "abomination of desolation" of this prophecy, and it makes
much more sense than does Ross's arbitrary assertion that "Daniel" was prophesying events
that would be fulfilled in the life of Jesus Christ. One thing at least is sure: the lack of
consensus on this passage doesn't give Dr. Ross a very convincing example of prophecy
fulfillment. The first characteristic of a valid claim of prophecy fulfillment is that the claimant
must show that he is correctly interpreting the text on which he is basing his claim. Dr. Ross
has not done that.

Volume 1990 - 2002 Issue


Page 753 of 2049
Skeptical Review Edited by Farrell Till

Jairus's Daughter: Dead but Raised to Live


Again
by Roger W. Hutchinson
In the Autumn 1994 issue of The Skeptical Review, Farrell Till described a problem he
encountered when comparing the three accounts of the meeting between Jairus and Jesus
recorded in Matthew, Mark, and Luke. He made some statements about inerrancy that could
lead a person to a misunderstanding of "inerrancy" and its application to the Bible. A basic
understanding of "inerrancy" is necessary if we are to resolve the problem that Till has
described. Consequently, we will need to define inerrancy before we can address Till's
problem.

We who believe that the Bible is inerrant claim that it provides us with an accurate record of
historical events. Consequently, when we read about Jairus in the book of Matthew, we claim
that what we read is an accurate or inerrant record of those events. When we read further
about Jairus in Mark or Luke, we know that we have additional accurate records of the events
surrounding Jairus's meeting with Jesus. When we combine all the information from the three
accounts, we will still have an accurate account. If we do not, then we did not have inerrant
records to begin with.

In the individual accounts of Jairus's meeting with Jesus, we can have variations among the
accounts only if one account provides additional information not contained in the other
accounts. However, where two accounts provide the same information, there cannot be
variations among the facts presented. If one account says that Jairus's daughter was twelve
years old, then the other accounts must agree with this. We would not have an inerrant
account if we were told that the daughter was twelve in one place and thirteen in another
(unless the daughter's age were being measured by two different and acceptable methods of
recording a person's age).

The particular problem that Till identified concerned the actual words that Jairus spoke to
Jesus when he first approached him. In Mark, we find Jairus stating that his daughter was at
the point of death. In Matthew, he is recorded as saying that she had died. The original Greek
words used by the authors to describe the events are different in each case, so we have Mark
recording that Jairus said one thing and Matthew recording that Jairus said something similar
but clearly and obviously different. Even though both accounts convey the same basic
message, each account is unique. What do we do with this?

Till described the problem this way:

What a person SAYS is what he SAYS, and two accounts of what someone said can't
both be inerrant accounts unless both use the exact words to convey what was said
(TSR, Autumn 1994, p. 3).

Later in the article, Till noted that...

Volume 1990 - 2002 Issue


Page 754 of 2049
Skeptical Review Edited by Farrell Till
Jairus SAID what he SAID (if the incident actually happened), so no rational person
can believe that Mark's and Matthew's accounts are BOTH verbally inspired, inerrant
accounts of what Jairus said (Ibid.).

Till is right and then wrong. Because the Bible is inerrant, we can be very certain that both
versions of Jairus's meeting with Jesus as recorded in Mark and Matthew are historically
accurate. If both Mark and Matthew are recording what Jairus said, then their accounts must
agree. On this point, Till is correct. Till errs in concluding that either the account in Mark is
accurate or the account in Matthew is accurate but that both cannot be accurate. On that basis,
Till says that at least one account is in error. However, if the Bible is inerrant, then we are
forced to conclude that both accounts must be true and accurate statements.

If we are correct in our conclusion that both accounts are inerrant, then Jairus must have
spoken the words that Matthew recorded, and he must have spoken the words that Mark
recorded. Thus, Matthew and Mark could not have been paraphrasing one unique statement
made by Jairus, as Till wrongly assumes. Instead, they must have recorded two unique and
different statements made by Jairus. In other words, Jairus came to Jesus and, in the course of
the conversation that followed, he stated that his daughter was dying as recorded by Mark,
and then, elsewhere in the conversation, he also stated that his daughter was dead as recorded
by Matthew. There can be no other outcome under the inerrancy doctrine.

One of the problems that we encounter in the Bible is that we are usually given very few
details about the events that are described. Much can be and usually is left out. For example,
in the account of Jairus's meeting with Jesus, we have no idea how long the conversation
between Jairus and Jesus lasted. We are not told whether Jesus said anything to Jairus in that
conversation nor are we told how Jesus reacted to Jairus. We can only imagine what tone of
voice might have been used and how this might have conveyed additional information. All we
know is that Jairus approached Jesus seeking to persuade him to heal his daughter and, in the
course of that initial meeting, made two statements that God wanted recorded in the Bible.
One of these statements was recorded by Matthew and the other was recorded by Mark.
Nothing else was recorded, so we do not even know which statement was made first or the
specific context in which the statements were made other than in the broadest terms.

At this point, we can only speculate on what might have happened. Before doing so, however,
we must be sure that we have our facts straight. One factor leading to Till's conclusion that a
problem exists may be the particular translation of the text that he used. In the KJV, Matthew
quotes Jairus as saying, "My daughter is even now dead." The NKJV, quoted by Till, has
Jairus saying, "My daughter has just died." The KJV translation suggests that Jairus may have
been drawing a conclusion about his daughter's condition. The NKJV translation is more
emphatic and suggests that Jairus may have received specific knowledge about his daughter's
condition. Obviously, translation is important, and an inaccurate translation can lead us to a
false conclusion.

If we combine the three accounts from Matthew, Mark, and Luke, we can build the following
hypothetical scenario to illustrate how Jairus's initial meeting with Jesus could have played
out.

Volume 1990 - 2002 Issue


Page 755 of 2049
Skeptical Review Edited by Farrell Till
Jairus comes to Jesus, falls at his feet and says, "My little daughter lieth at the point of death:
I pray thee, come and lay thy hands on her, that she may be healed; and she shall live." Jesus
says nothing, even seeming to be indifferent toward Jairus's plea. He may even have turned to
talk to those around him. In desperation, Jairus pleads, "My daughter is even now dead: but
come and lay thy hands upon her, and she shall live." At this demonstration of faith, Jesus
goes with him.

Obviously, we cannot claim that the above occurred. We use it merely to illustrate that it was
possible for Jairus to have made what appear to be two inconsistent statements about his
daughter's condition and to have done so in a consistent, rational manner. His daughter was at
the point of death as he leaves to find Jesus. So critically ill was his daughter that, in the short
time since he left her, it is entirely possible that she could have died. So great is his love for
his daughter and his faith in Jesus's ability to heal that he is certain that Jesus can heal his
daughter even if she were already dead.

To illustrate what could have happened as Jesus and Jairus are walking to Jairus's house, we
can propose the following, again hypothetical, scenario incorporating the information given to
us by the various accounts.

While Jesus was speaking to the woman, a man came from Jairus's house. He came up to
Jairus and said, "Thy daughter is dead; trouble not the Master." He then turned to go
expecting Jairus to follow. Jesus, overhearing what was said, leaned close to Jairus and said,
"Fear not: believe only, and she shall be made whole." Encouraged by this, Jairus continued
walking with Jesus to his house. The man from his house, not hearing what Jesus had said,
and thinking that Jairus had not understood the import of his message, grabbed Jairus by the
arm and said, "Thy daughter is dead: why troublest thou the Master any further?" When Jesus
heard this, he turned to Jairus and said again, "Be not afraid, only believe."

Again, we know little about what really happened as Jesus and Jairus walked to Jairus's
house. All we know is that Mark records certain statements that were made and Luke records
others. Because Luke had stated in the beginning of his gospel that he had set out to
investigate the life of Jesus, it is possible that he has paraphrased what people described Jesus
to say. In other words, Luke talked to several people and based on what he was told, he wrote
that such and such was said, but he was only paraphrasing what they said based on what he
had been told. For our purposes, however, we are assuming that Luke, under the inspiration of
the Holy Spirit, is recording the exact words spoken. From the above scenario, we can again
see how the accounts of Mark and Luke can be reconciled.

Continuing on, we can suggest a scenario to illustrate what could have happened at Jairus's
house.

They came to Jairus's house and entered. They saw the tumult, and them that wept and wailed
greatly. Jesus said to the crowd as he stood in the doorway, "Why make ye this ado, and
weep? The damsel is not dead, but sleepeth." As Jesus walked into the room, he said to others,
"Weep not; she is not dead, but sleepeth." Some, who had seen the child, said that she was
dead. Jesus desiring that all should leave called out so that all can hear, "Give place: for the
maid is not dead, but sleepeth." And the people laughed him to scorn, because they knew that

Volume 1990 - 2002 Issue


Page 756 of 2049
Skeptical Review Edited by Farrell Till
the child was dead. After the crowd was removed, they went to where the child lay. Jesus took
the child by the hand and said, "Talitha cumi," which is, being interpreted, "Damsel, I say
unto thee, arise" or "Maid, arise." Then her spirit came again and she arose right away.

The above scenarios serve to illustrate how events could have played out in a manner
consistent with the scriptures. Obviously, we do not know what actually happened. However,
just because we do not have all the facts of the case does not mean that the facts that we do
have must be erroneous. We should not let a lack of information drive us to false conclusions.
We do know that we have inerrant or accurate records of certain events that occurred. Our
scenarios are consistent with the available facts and demonstrate that there was a reasonable
and logical way for all of these events to have happened as they were described.

It is important that we apply the inerrancy doctrine correctly as we seek to explain the events
we find recorded in the Bible. If we do this, then we come a long way in our efforts to
discover what God is telling us in the Bible.

(Roger W. Hutchinson, 11904 Lafayette Drive, Silver Springs, MD 20902, e- mail


rhutchin@aol.com)

More How-It-Could-Have-Been Speculation


by Farrel Till
The flagrant resort to speculation in the foregoing article was not at all surprising to readers
who are also subscribers to errancy@atheist.tam.edu, because they have seen Roger
Hutchinson post utterly ridiculous speculative "solutions" to Bible discrepancies that have
been discussed on the list. The best way to begin a response to Mr. Hutchinson's article, then,
is to notice his frequent admissions that he is arguing from the assumption that the Bible is
inerrant.

In concluding his article, Hutchinson said, "We do know that we have inerrant or accurate
records of certain events that occurred." Well, pardon me, but I know no such thing and
neither does Mr. Hutchinson. He simply insists on the right to argue from this assumption.
This is not just an unfounded allegation I am making, because earlier in the article, he said,
"Because the Bible is inerrant, we can be very certain that both versions of Jairus's meeting
with Jesus as recorded in Mark and Matthew are historically accurate." At the beginning of
the article, he said, "We who believe that the Bible is inerrant claim that it provides us with an
accurate record of historical events. Consequently, when we read about Jairus in the book of
Matthew, we claim that what we read is an accurate or inerrant record of those events. When
we read further about Jairus in Mark or Luke, we know that we have additional accurate
records of the events surrounding Jairus's meeting with Jesus." So when we understand that,
from the beginning to the end of his article, Mr. Hutchinson was arguing from the assumption
that the Bible is inerrant, we can also understand that he resorted to a very familiar logical

Volume 1990 - 2002 Issue


Page 757 of 2049
Skeptical Review Edited by Farrell Till
fallacy that inerrantists use to prove their case, i.e., circular reasoning. The following scenario
illustrates the way he is trying to prove the inerrancy of the Jairus accounts:

Hutchinson: I know that everything the synoptic gospels say about the raising of
Jairus's daughter is inerrant.
Till: How do you know this?
Hutchinson: Everything the accounts say has to be accurate, because the Bible is
inerrant.
Till: How do you know the Bible is inerrant?
Hutchinson: The Bible has to be inerrant, because it is inspired of God.
Till: How do you know?
Hutchinson: The Bible has to be inspired of God because it is inerrant, and only a
divinely inspired book could be inerrant.

As ridiculous as this scenario may seem to some readers, it will not appear ridiculous to those
who have seen the hundreds of postings that Mr. Hutchinson has made on the internet. He
openly admits that he argues from the assumption that the Bible is inerrant and that this
entitles him to speculate in order to resolve biblical discrepancies.

For reasons no inerrantist has ever explained, Hutchinson assumes that an assumption of
biblical inerrancy entitles the inerrantist to resolve discrepancies by simply postulating how-
it-could-have-been scenarios. I first encountered this inerrantist tactic as a ministerial student
in 1952 when a Bible professor pointed out a scripture that seemed to contradict a passage the
class had been assigned to read for that day. After identifying the problem, the professor
dazzled the class by presenting a how-it-could-have been explanation, after which he told us
always to remember that whenever "apparent" inconsistencies are presented to us, as long as
we can show a possible interpretation of one of the passages that would resolve the
inconsistency, then we have eliminated the skeptic's right to claim errancy in the Bible.

I was impressed with the professor's demonstration that day, but having long since passed the
stage of impressionable student preacher, I can now see all kinds of problems in this method
of resolving biblical discrepancies. First of all, it really proves nothing except that an
imaginative person can always think of some kind of what-it-could-have-meant interpretation
to "explain" away inconsistency. What those who use this method stubbornly fail to see--or
perhaps I should say refuse to admit, because I really think they do see it--is that in any
situation, biblical or nonbiblical, a "possible" explanation is merely a hypothesis that doesn't
prove anything unless there is reasonable evidence to establish that it is probably true.

We can use any unsolved mystery to illustrate this. One of the great mysteries of this century
was the disappearance of the American aviatrix Amelia Earhart during a flight around the
world in 1937. Various "solutions" to the mystery have been proposed, all of which fall within
the realm of possibility, but none has ever been proven. Hence, Earhart's disappearance
remains a mystery till this day. If John Jones should fail to return home from a hunting trip
and never be seen again, one could propose several "possible" explanations for his
disappearance: (1) he was murdered and his body disposed of in a place where it was never
found, (2) he encountered a bear in the woods who killed him and completely devoured his
body, (3) he drowned while crossing a river in the woods, and his body became permanently

Volume 1990 - 2002 Issue


Page 758 of 2049
Skeptical Review Edited by Farrell Till
snagged on something under the water, (4) he was kidnapped in the woods and was taken to a
place where he is still being held captive, or (5) he was unhappy with his home life, so he
simply ran away and assumed a new identity. Any of these "solutions" to the mystery of
Jones's disappearance are clearly possible, and others could also be proposed. However, none
of them would constitute real solutions to the mystery until some kind of evidence is
uncovered to support one of the hypotheses.

So it is with Mr. Hutchinson's solutions to inconsistencies in the gospel accounts of the raising
of Jairus's daughter. In his article, Hutchinson has merely proposed how-it- could-have-been
explanations, none of which is as likely as the probability that different writers telling the
same story told it in such a way that inconsistencies and contradictions result when the three
versions are compared. We all know that when different people tell the same story, no two
accounts are exactly alike, and this fact is so well known that there is really no need for me to
waste space discussing how likely it is that three different accounts of a narrated event would
contain inconsistent variations.

Were it not for Hutchinson's irrational belief that an omniscient, omnipotent deity *verbally*
inspired all three synoptic gospels, he would feel no compulsion at all to propose unverifiable
postulations that would reconcile all three versions of the Jairus story. He would simply
recognize that the variations in the accounts are consistent with what we encounter all of the
time: different writers recording the same events get some of the details wrong, and so
inconsistencies result.

Hutchinson can't make this concession without doing serious damage to his belief that the
Bible is the verbally inspired, inerrant "Word of God," so he has to do the same thing that we
said of Marion Fox's attempt to reconcile Deuteronomy 23:2 with other biblical passages: he
must lean over backwards to look for some way out of a sticky situation.

In doing so, just look at the predicament that Hutchinson has gotten himself into. He has
agreed with my claim that whatever a person *says* is what he *says,* and so if Mark
reported that Jairus said one thing and Matthew reported he said something different, then
Jairus had to have said both statements word for word in order for the Bible to be inerrant.
Does he have any idea how many far-fetched scenarios he will have to invent in order to
harmonize the many parallel texts in the Bible that report dialogues? For example, in
Matthew's account of the temptation of Jesus, the devil said to Jesus, "If you are the Son of
God, command *these stones* to become bread" (4:3), but according to Luke, the devil said,
"If you are the Son of God, command *this stone* to become a *loaf* of bread" (4:3). So
which statement did the devil make? Did he challenge Jesus to turn STONES (plural) into
bread, or did he challenge him to turn A stone (just one) into a LOAF of bread? Well,
Hutchinson has taken a position that requires him to say that the devil made both statements,
so he must concoct some scenario that would have the devil making both statements. An
examination of both parallel passages will show that in the other two temptations there are
variations in what either Jesus or the devil said in their conversations, so this will require
Hutchinson to postulate a temptation scenario that would have the devil and Jesus saying
essentially the same thing twice but in different words each time. Who can believe such
nonsense?

Volume 1990 - 2002 Issue


Page 759 of 2049
Skeptical Review Edited by Farrell Till
The temptation scene is just one example of dozens of parallel accounts that I could cite that
would require Hutchinson to dream up scenarios that have various biblical characters
hemming and hawing and saying twice (in different words each time) what the parallel
accounts attribute to them in recorded conversations. This is such a preposterous solution to
the problem of variations in the allegedly inspired accounts of conversations recorded in the
Bible that it doesn't even deserve serious consideration.

It is far more reasonable to believe that such parallel accounts were written by uninspired
writers who had only oral traditions to rely on, and so quite naturally their accounts of what
was said and what happened varied from writer to writer. The variations, however, are clear
proof that the writers were not verbally inspired by an omniscient, omnipotent deity, because,
as I noted in my original article about Jairus's daughter, an omniscient, omnipotent deity
would have known exactly what words were spoken in a conversation and would have guided
the men writing under his influence to use those exact words. This is a problem that
inerrantists cannot explain away, no matter how much they talk about God's allowing his
inspired writers to inject their own personalities into their writing. What was said was what
was said, and no personality differences in Mark and Matthew would justify having one of
them say that Jairus said, "My little daughter is at the point of death," and the other one say
that he said, "My daughter has just died." Inerrancy would have required both of them to say
exactly what Jairus said.

Hutchinson has proposed some how-it-could-have-been scenarios, but as the front- page
article of this issue has pointed out, such scenarios prove nothing at all until the inerrantists
produce some kind of tangible evidence to show that their scenarios are what probably
happened. The fact that the scenario is in the realm of possibility does not prove anything. If it
is more likely that fallible humans were imprecise in their language or mistaken in their
understanding of what happened or what was said, that would be a more reasonable
explanation for the many variations in the various parallel accounts in the Bible, so skeptics
have no obligation to prove that Hutchinson's hypothetical scenarios did NOT happen; he
must prove that they DID happen. Until he does, the inerrancy doctrine is in serious trouble.

Only one other observation needs to be made about Hutchinson's how-it-could- have-been
scenarios. If all of the fanciful scenarios that inerrantists have proposed as solutions to biblical
discrepancies really happened, why didn't the omniscient, omnipotent Holy Spirit just direct
his verbally inspired writers to so record the stories? Their method of resolving Bible
discrepancies reduces God to the level of an inept simpleton who didn't have the intelligence
that the Roger Hutchinsons of the world claim to have. Roger Hutchinson, John Haley,
Norman Geisler, Gleason Archer, William Arndt, and such like are able to relate Bible events
in a way that eliminates inconsistencies, but somehow an omniscient, omnipotent deity just
didn't seem to have the talent to do that.

In the matter of Jairus's conversation, Hutchinson said, "All we know is that Jairus
approached Jesus seeking to persuade Jesus to heal his daughter and, in the course of that
initial meeting, made two statements *that God wanted recorded in the Bible*" (p. 4,
emphasis added). Okay, will Mr. Hutchinson please answer a simple question for us? When
Mark was writing his account, what circumstances existed that made God want Mark to
record only one of the statements, and when Matthew was writing his account, what

Volume 1990 - 2002 Issue


Page 760 of 2049
Skeptical Review Edited by Farrell Till
circumstances existed that made God want Matthew to record only the other statement? If
God really wanted both statements allegedly made by Jairus to be recorded in the Bible, why
didn't he just direct Matthew, Mark, and Luke (all three) to record both statements? Better yet,
why didn't God look for three people with Roger Hutchinson's ability to be his inspired
writers so that this story could have been recorded in such a way that no confusion would
have resulted? Still even better, why didn't God just find *one* Roger Hutchinson of that
generation to write just *one* gospel account that would have included EVERYTHING "that
God wanted recorded in the Bible"? If the job had been done right the first time, there would
have been no need for a second or third or fourth try at telling the world the story of God's
only begotten son. Would Mr. Hutchinson or someone please tell us why it wasn't done that
way?

Hutchinson-Till Debate Is Merely a Tempest


in a Teapot
by Wilhelm E. Schmitt
The Skeptical Review (Vol. 6, No. 4) carried an article entitled "The 430-Year Sojourn of
Israel in Egypt" by Roger Hutchinson and a rebuttal by Farrell Till. While Mr. Till's rebuttal
is well written and responsive (as are many of his written efforts I have seen), I am of the
opinion that Hutchinson made a fundamentally incorrect assumption near the beginning of his
article, which makes his effort not only unnecessary but the entire exchange moot.

The erroneous assumption occurred in Hutchinson's first paragraph, where he asserted,


"Exodus 12 states clearly that Israel dwelt in Egypt 430 years." Actually, it states no such
thing; the text does not state that the 430 years covered only the time in Egypt. The words
"who dwelt in Egypt" form an appositional phrase relating to "sons (of Israel)," NOT to
"sojourning." Therefore, it is exegetically incorrect to maintain that the dwelling in Egypt
covered 430 years. A more careful reading of the text yields the proper understanding that the
430-year period commenced with the *beginning* of the Israelites' "sojourning." The
determination of this "beginning" point will clear up the controversy, and that is the purpose
of this article.

Since it is my conviction that God is the author of the Bible, it is therefore my contention that
the only errors the Bible may contain are those of copyists, printers, or translators. Once these
errors are eliminated, every statement of the Bible must agree with every other. However, this
harmony is not always found on the surface of the text; it is sometimes deeply hidden; but if it
is patiently, reverently and diligently sought, harmony will eventually emerge. Dr. Ivan Pain
wrote, "The Bible makes upon the reader certain inexorable demands. To those who comply,
it readily yields its secrets, treasures ere long, according to the capacity of each. To those who
do not comply, it abides sealed in proportion to their lack of compliance" (Biblical
Chronology, 1922, p. 7).

Volume 1990 - 2002 Issue


Page 761 of 2049
Skeptical Review Edited by Farrell Till
One of the foremost canons of interpretation of the Bible (or any written document) is that a
clear statement in one part of the Bible is to be adhered to against others not so clear in other
parts. The light from the clear is to be thrown on the obscure, rather than the obscure to
darken the light. A mere difficulty is not to annul a certainty. Furthermore, fairness requires
observance of another canon: the Bible alone is the final authority for its own data.

With these two canons (there are several others, but need for brevity prohibits my citing them)
in control of our inspection of the matter at hand, I call the reader's attention to a clear
statement by the Apostle Paul recorded in Galatians 3:16-18:

(T)he promises were spoken to Abraham.... But this I say: A covenant confirmed afore
by God the Law which came *four hundred and thirty years after,* disannulleth not
unto the voiding of the promise. For if the inheritance is of Law, it is no longer of
promise. But to Abraham God hath granted it by promise (emphasis added).

This statement is abundantly clear: from the covenant with Abraham (whenever it was) to the
Law (given at Sinai in the year of the exodus) is 430 years. Alongside this stands Exodus
12:40-41:

And the sojourning of Israel's sons who sojourned in Egypt land was thirty years and
four hundred years. And it was at the end of thirty years and four hundred years (and it
was the self-same day) that all the hosts of the Lord went out from Egypt land.

A superficial reading of this passage could give the impression that from the going down of
the sons of Jacob with their father to sojourn in Egypt to the giving of the Law was 430 years;
Galatians 3:16 is thus apparently contradicted. The covenant with Abraham was, at the latest,
the year before Isaac was born, when Abraham was 99. Isaac was 60 when Jacob was born,
and Jacob was 130 when he went to Egypt. Thus, 1+60+130+430 yields 621 years from the
covenant with Abraham to the Law, if the data from Exodus 12 are thus to be understood--and
Paul's assertion stands contradicted.

Note that Galatians is clear, but Exodus is not so clear. The terms in Exodus *sojourned,*
*Israel's sons,* *Egypt land* (to which the regions where Abraham and Isaac lived may have
been subject at some period of their lives)-- each of these terms needs considerable
elucidation before they can be allowed to annul the clear statement in Galatians. Thus, our
first canon makes Galatians the standard by which Exodus is to be understood, rather than the
reverse. The clear overrides the obscure, and the passage from Exodus must submit to the
clarity of Galatians.

The solution to the 430 years question begins with the determination of the year of Abraham's
birth, which affects all subsequent biblical dates. The first Christian martyr, Stephen, is
recorded in Acts 7:2-4 as saying that when Abraham left Haran, his father, Terah, was dead.

The God of the Glory appeared to our father Abraham... and said to him, Come out of
thy land.... Then came he out of the Chaldeans' land and dwelt in Haran; and thence
*after his father's death* [God] r moved him into this land wherein ye now dwell.

Volume 1990 - 2002 Issue


Page 762 of 2049
Skeptical Review Edited by Farrell Till
Genesis 11:32 states "all the days of Terah were five years and two hundred years." In
Genesis 12:4, Abraham is said to be a "son of five years and seventy years when he departed
from Haran." Therefore, according to Stephen, who then was expressly said to have been
"filled with the Holy Spirit," Terah was 205 less 75 when Abraham was born, or 130. Note
that this deduction is clear and obvious. However, in Genesis 11:26 we read, "Terah lived
seventy years, and begat Abram, Nahor and Haran"--which at first glance implies they were
triplets of whom Abram was firstborn. But in the clear light of Stephen's statement in Acts
7:2-4, the statement in Genesis 11:26 requires clarification: if Abraham was born when his
father was 70, he was at Terah's death 135, not 75. Our first canon thus requires that Stephen's
clear statement be held to against Genesis 11:26, where it is possible to derive the meaning
that Terah became a father at 70, and his children were Abraham, Nahor, and Haran, of whom
Abraham is not necessarily the oldest but named first for some reason that is not obvious here.
Thus, we should hold to Stephen and keep Genesis 11:26 in abeyance pending further light.
This treatment is paralleled by the case of Shem, Ham, and Japhet, sons of Noah, where it is
demonstrable that Shem, although named first, was not the oldest of triplets but was two years
younger than Japhet.

I give now (without developing the proof, which would frustrate the objective of brevity, but
if proof of this development is demanded and space for printing in The Skeptical Review is
offered, I will supply it) the chronology from Abraham's birth to the exodus. The first date
given is the number of years that elapsed since the creation week (A. M. = Anno Mundi =
years of the world). The figure following is the same year in terms of B. C. (Before Christ):

Event A. M. B. C.

Terah born 1878 2125

Terah 70, Haran born 1948 2055

Abraham born 2008 1995

Sarah born 2017 1986

Terah (205) dies 2094 1920

Abram 86, Ishmael born 2083 1909

The covenant given 2107 1896

Isaac born, Abraham 100 2108 1895

Sarah dies 2144 1859

Isaac marries Rebecca 2148 1855

Esau & Jacob born 2168 1835

Abraham (175) dies 2183 1820

Volume 1990 - 2002 Issue


Page 763 of 2049
Skeptical Review Edited by Farrell Till
Jacob (77) with Laban 2245 1758

Joseph born 2259 1744

Joseph (17) sold 2276 1727

Joseph in prison 2287 1716

Isaac (180) dies 2288 1715

Joseph before pharaoh 2289 1714

End of 7 years of plenty 2296 1707

Year 2 famine, Jacob 130 2298 1705

Jacob 147, dies 2315 1688

Joseph 110, dies 2369 1634

Aaron born 2454 1549

Moses born 2457 1546

Moses 40 2497 1506

Exodus, giving of law 2537 1466

The exodus having occurred in 1466 B. C., going back in time 430 years would bring us to
1896 B. C., which is the year of God's covenant with Abraham recorded in Genesis 17:1-27.
That Abraham was at this time 99 years of age is established in verses 1 and 20. In addition,
verse 25 states that Ishmael was 13, having been born when Abraham was 86 (Gen. 16:16).
This is the only covenant from which Paul could begin the 430-year period to the Law: the
covenant made with Abraham when he was 99, in 2107 A. M. (1896 B. C.). [Two other dates
have been suggested as commencing the 430 years that culminated in the exodus: (1) Abram's
call, at 75 (2083 A. M.; 1920 B. C.); (2) the land covenant made with Abram (Gen. 15:18).
The former was not a covenant but a "call" and does not meet the 430-year period to the
exodus in 1466 B. C.; the second cannot be dated but was not later than Abram's 85th year, so
it also fails the requirement of the 430-year period reaching 1466 B. C.]

In summary, the 430-year "sojourn" commenced with the Abrahamic covenant recorded in
Genesis 17:1-27, when Abram (renamed "Abraham" by God at that time) was 99: 2107 A.M.
or 1896 B.C. Application of Occam's Razor is appropriate here; the simplest explanation is
usually the correct one. There is no need to find 430 years during the events of Exodus 6:16-
20. Therefore, the semantic gyrations used by Mr. Hutchinson and rebutted by Mr. Till are
rendered unnecessary and may be seen to be, in Shakespeare's words, "full of sound and fury,
signifying nothing."

Volume 1990 - 2002 Issue


Page 764 of 2049
Skeptical Review Edited by Farrell Till
(Wilhelm E. Schmitt, 14575 Louisiana Avenue, PriorLake, MN 55372.)

More Like "As You Like It"


by Farrell Till
Wilhelm Schmitt has described the debate over the 430-year sojourn of the Israelites as a
"tempest in a teapot," which, in the words of William Shakespeare, has been "full of sound
and fury, signifying nothing." After reading his defense of inerrancy in the matter of the
Israelite sojourn in Egypt, I took a cue from Shakespeare too and decided that a better title for
Schmitt's article would have been "As You Like It," because he has argued not for a
conclusion the evidence will support but for one that he would like to be true. As I will show,
all of Mr. Schmitt's arguments were predicated on the assumption that the Bible contains no
mistakes, so he has made the mistake that most inerrantists make, i. e., trying to prove
inerrancy by assuming inerrancy.

Before I address this matter, however, let's first look at another fundamental error in his
reasoning. He argued that "(o)ne of the foremost canons of interpretation of the Bible... is that
a clear statement in one part of the Bible is to be adhered to against others not so clear in other
parts," but this is a popular rule of hermeneutics that cannot be logically defended for the
simple reason that the Bible is not just one book written by a single writer. It is a collection of
66 different books written by an estimated 40 different writers over a period of several
centuries, so Mr. Schmitt is arguing that if writer "A" made an obscure statement in, say, 500
B.C., and writer "B" over 500 years later made a clearer statement on the same subject, we
must allow the clear statement to explain or clarify the obscure one. In so arguing, he is
assuming that the Bible is inerrant in everything that it says, and so it would not be possible
for the obscure statement of writer "A" to contradict the clearer statement of Writer "B." This,
however, is a premise that he needs to support with reasonable evidence, and evidence to
support this premise was conspicuously absent in Mr. Schmitt's article.

To illustrate the fallacy in the reasoning of inerrantists who argue as Mr. Schmitt has, let's
suppose that we should find a statement in volume two of *Encyclopedia Americana* that
contradicts a statement on the same subject in volume ten. If the statement in volume ten is
worded more clearly than the one in volume two, would Mr. Schmitt argue that the clear
statement must be allowed to explain or clarify the obscure one in order to remove
contradiction from the encyclopedia? Wouldn't he more likely recognize that since the
statements were written by different authors, they simply disagreed with each other on this
particular subject? If Schmitt wishes to talk about Occam's razor, here is an appropriate
situation to use the principle.

To apply what I am saying to the issue at hand, I have only to point out that the book of
Exodus was not written by the apostle Paul, so it is entirely possible that Paul and the writer
of Exodus, who were separated by several centuries, disagreed on the length of the Israelite

Volume 1990 - 2002 Issue


Page 765 of 2049
Skeptical Review Edited by Farrell Till
sojourn in Egypt. The reason why Paul thought it was much shorter than 430 years is known
to biblical scholars, but I will have to address that matter later. Here I simply want to notice
that Mr. Schmitt's primary "canon" that he expects us to apply to biblical hermeneutics is a
familiar attempt to give a privileged status to the Bible that even he would not be willing to
give to any other source of information. I'm sure, for example, that if inconsistent statements
were found in the Koran or the Book of Mormon, Schmitt would not argue that "the light
from the clear is to be thrown on the obscure." Why then should he argue such a principle as
this for the Bible except for the obvious fact that he expects the Bible to enjoy privileged
status among the books of the world? This is a concession we cannot allow him. He must first
prove that the Bible is indeed the inspired, inerrant "word of God," and then we will be
willing to consider the validity of this "foremost canon of interpretation" on which he has
based his defense of biblical inerrancy.

Numerous examples can be cited of biblical writers who, writing at different times, expressed
contradictory opinions. One such case involved Jehu's massacre of the Israelite royal family at
Jezreel, which is recorded in 2 Kings 9-10. After Jehu had killed Joram, his family,
government officials, and even Ahaziah, the Judean king visiting Joram at the time, the writer
of 2 Kings stated his approval of the massacre:

And Yahweh said to Jehu, "Because you have done well in doing what is right in My
sight, and have done to the house of Ahab all that was in My heart, your sons shall sit
on the throne of Israel to the fourth generation" (2 Kings 10:30).

Chapters 13-15 record the reigns of four generations of Jehu's descendants. Each is described
as "doing evil in the sight of Yahweh," yet these four generations were allowed to reign until
the assassination of Zechariah, the fourth generation, at which time, the writer of 2 Kings
said, "This was the word of Yahweh which He spoke to Jehu, saying, `Your sons shall sit on
the throne of Israel to the fourth generation.' And so it was" (15:12).

The writer of 2 kings said that Yahweh was so pleased with Jehu's role in the Jezreel massacre
that he promised to let four generations of Jehu's descendants sit on the throne of Israel, and
then subsequently allowed these descendants to reign even though they "did that which was
evil in the sight of Yahweh." In the face of such evidence, can anyone deny that the writer of
2 Kings approved the massacre?

About a hundred years later, in the time of King Zechariah, the fourth and last of Jehu's
descendants to reign over Israel, the prophet Hosea expressed an entirely different opinion of
the Jezreel massacre. Hosea alleged that the word of Yahweh came to him and pronounced
judgment on the house of Jehu for the "blood of Jezreel":

For yet a little while, and I will avenge the blood of Jezreel on the house of Jehu, and
will cause the Kingdom of the house of Israel to cease (1:4).

So much did the prophet Hosea disagree with the writer of 2 Kings in the matter of Jehu's
massacre that he predicted Yahweh would destroy the Kingdom of Israel in order to extract
vengeance for the blood of Jezreel.

Volume 1990 - 2002 Issue


Page 766 of 2049
Skeptical Review Edited by Farrell Till
Obviously, then, biblical writers had their theological differences just as we can find today in
any religious institution. In the matter of Jehu's massacre of the Israelite royal family, it isn't a
matter of letting "the light from the clear [passage] to be thrown on the obscure [passage],"
because both passages are quite clear in their meanings. The writer of 2 Kings approved
Jehu's actions; the prophet Hosea denounced them.

This "foremost canon of interpretation" that Mr. Schmitt touts as a solution to the discrepancy
under consideration might have some validity when inconsistent statements (one obscurely
worded, the other clearly worded) appear in a single document that was written by one author.
Then we could see some merit in arguing that a person would not likely contradict himself in
a single document (even though experience tells us that this isn't necessarily true), and so we
should allow the clear statement to "throw light" on the obscure one. However, when we have
documents written by different individuals, who lived centuries apart, this "foremost canon"
isn't nearly so likely as the assumption that people can easily have different opinions on a
subject. We have mountains of empirical evidence that support the latter assumption, but the
only thing that supports the former is an irrational desire for the Bible to be the inerrant "word
of God."

That Mr. Schmitt was arguing from the assumption that the Bible is inerrant was apparent
throughout his article. The age of Terah (Abraham's father) when Abraham was born was
crucial to Schmitt's case, but the book of Genesis isn't too clear about this. The Genesis
account says that Terah was 70 when he begot Abraham and his brothers Nahor and Haran
(11:26) and that Terah lived to be 205 years old (11:32), but it also says that Abraham was 75
when he left Haran (12:4), so the chronological problem in this matter is rather apparent. If
Terah was 70 when he begot Abraham and if Terah lived to be 205 and if Abraham didn't
leave Haran until "after his father's death," as Stephen claimed (Acts 7:4), then Abraham had
to be at least 135 when he left Haran. To resolve the problem, Schmitt appealed to the clarity
of Stephen's claim that Abraham left Haran "after his father's death." To validate his
chronological chart, however, Schmitt had to establish that this was a true statement, so he
proved the truth of Stephen's statement by assuming the inerrancy of the scriptures. Let's
notice the subtle way that Schmitt tried to sneak this assumption by us. After summarizing the
above problems in the Genesis record, Schmitt concluded, "Therefore, according to Stephen,
*who then was expressly said to have been `filled with the Holy Spirit,'* Terah was 205 less
75 when Abraham was born, or 130." Schmitt concluded that the "clear light of Stephen's
statement" therefore clarifies the ambiguity in Genesis 11:26, so he is demanding that we
accept the inerrancy of Stephen's statement on the grounds that it was "expressly said" that he
was "filled with the Holy Spirit." In other words, Schmitt is arguing that if the Bible says X
(in this case that Stephen was filled with the Holy Spirit), then X has to be true. But why
should he be entitled to this assumption? I see no reason to allow him such an assumption
anymore than I would allow a Moslem to assume the truth of what the Koran may say in a
disputed matter, so I will insist that Schmitt prove to us the truth of Stephen's claim that Terah
was dead when Abraham left Haran. Until he can produce that proof, he cannot establish that
Abraham was born in the year 2008 A. M. (Anno Mundi) or 130 years after Terah's birth in
1878 A. M., and if he cannot establish that, his chronological chart collapses at its third level.

In fact, even if Mr. Schmitt could prove that Stephen's statement was correct and that
Abraham did not leave Haran until *after his father's death,* this would not establish that

Volume 1990 - 2002 Issue


Page 767 of 2049
Skeptical Review Edited by Farrell Till
Terah was 130 when Abraham was born. Schmitt has argued that Abraham was 75 when he
left Haran (Gen. 12:4) and that he left Haran "after his father's death" (Acts 7:4), so since
Terah lived 205 years (Gen. 11:32), we can determine Terah's age when Abraham was born
by subtracting 75 from 205. Hence, Terah was 130 when Abraham was born. Let's notice,
however, that Stephen said only that Abraham left Haran "after his father's death"; he did not
say that Abraham left Haran *immediately* after his father's death. If Abraham had stayed in
Haran 10 or 15 or, let's say, 20 years after his father's death and then left for Canaan, it would
still have been true that Abraham left Haran *after his father's death.* Such a delay in
Abraham's departure, however, would kick the props right out from under Mr. Schmitt's
chronological chart. It would have made Haran 150, not 130, when Abraham was born, so
from the third step in the chart (the date of Abraham's birth) all entries would have to be
pushed forward 20 years.

Since both the dates when the covenant was given and the exodus occurred would also be
pushed forward 20 years, this wouldn't significantly affect Schmitt's claim that 430 years
separated these two events, but it would show the uncertainty involved in trying to establish
exact biblical dates. I can illustrate this by beginning at an established time in the postexodus
history of Israel and counting backwards to the exodus. By comparing the biblical dates given
for the reigns of the kings of Israel and Judea with Babylonian records that establish the
destruction of Jerusalem (which signaled the end of the Judean kingdom) at 586 B. C., we can
establish that Solomon began his reign in 970 B. C. With that in mind, we can use 1 Kings 6:1
to fix the date of the exodus from the perspective of the author of 1 Kings. This verse states
that construction began on the temple in the 480th year after the children of Israel came out of
Egypt and that construction began during the 4th year of Solomon's reign, which would have
been 966 B. C. So if we add 480 to 966, we get a date of 1446 B. C., which is 20 years after
the date in Schmitt's chronological chart.

But the situation gets even worse. If we use the chronology of the apostle Paul, we get a much
greater discrepancy. To see this, let's notice what Paul said in a speech at Antioch of Pisidia:

The God of this people Israel chose our fathers, and exalted the people when they
dwelt as strangers in the land of Egypt, and with an uplifted arm He brought them out
of it. Now for a time of about *forty years* He put up with their ways in the
wilderness. And when He had destroyed seven nations in the land of Canaan, He
distributed their land to them by allotment. After that he gave them judges for about
*four hundred and fifty years,* until Samuel the prophet. And afterward they asked
for a king; so God gave them Saul the son of Kish, a man of the tribe of Benjamin, for
*forty years.* And when He had removed him, He raised up for them David as king,
to whom also He gave testimony and said, "I have found David the son of Jesse, a man
after my own heart, who will do all my will'' (Acts 13:17-22).

Just look at the chronological mess that Paul left us with here. He didn't say how long that
David reigned, but 1 Kings 2:11 states that David reigned for 40 years, which always seemed
like a good round number for biblical writers to use. So if we take the 40 years of David's
reign and add it to the 40 years that his predecessor Saul reigned and then add the 450 years
that Paul said the judges ruled and finally add on the 40 years of the wilderness wanderings,
we have a total of 570 years from the exodus to the end of David's reign, *without even

Volume 1990 - 2002 Issue


Page 768 of 2049
Skeptical Review Edited by Farrell Till
considering the time that it took Joshua to conquer Canaan and distribute the land to the
various Israelite tribes* (v:19). Since construction on the temple began in the fourth year of
Solomon's reign (1 Kings 6:1), if we now add 574 years to 966 B. C. (the fourth year of
Solomon's reign), we arrive at 1540 B. C. as the date of the exodus. This would be 74 years
before the date in Mr. Schmitt's chart, the chronology of which he assures us he can provide
proof for if it is demanded.

My purpose in developing this point has been to show the futility of trying to fix precise
biblical dates solely on the basis of what the Bible says. The end result of such efforts is going
to be inconsistencies like the ones just presented, so if Mr. Schmitt wants to talk about
"semantic gyrations," I suggest that he direct all discussions of this subject to the semantic
gyrations that inerrantists must resort to in order to give a semblance of harmony to the
biblical text. As we shall soon see, Exodus 12:40 very clearly states that the Israelites spent
430 years in Egypt, and the only way Schmitt can get around this fact is through his own far-
fetched semantic gyrations.

Mr. Schmitt wants us to allow him the right to let "clear" statements in the Bible "throw light"
on obscure ones, but there is a logical inconsistency in this principle that he calls "the
foremost canon" of biblical interpretation. When we consider the inerrantist claim that the
Bible was written through a process of *verbal* inspiration by which an omniscient,
omnipotent deity guided the writers in what words to use in conveying his eternal truths, we
have to wonder how that obscurity in the Bible would even be possible. Let's take as
examples the statement in Exodus 12:40 and Paul's statement in Galatians 3:17, which
Schmitt insists is a "clear" statement that we should allow to clarify the obscure statement in
Exodus 12:40. The doctrine of verbal inspiration claims that the same omniscient, omnipotent
deity guided both Moses (the alleged writer of Exodus) and the apostle Paul in everything that
they wrote, so why should a statement by Paul on the subject of the Israelite sojourn be any
clearer than one that Moses wrote, because in the final analysis (according to the doctrine of
verbal inspiration) the same omniscient, omnipotent deity made both statements? This is a
problem that Mr. Schmitt sorely needs to address before he can expect us to give any
credence to his "foremost canon of interpretation" that says clear biblical statements should be
allowed to thrown light on obscure ones. Verbal inspiration by an omniscient, omnipotent
deity should exclude the very possibility of obscurity.

As far as obscurity is concerned in Exodus 12:40, I would like for Mr. Schmitt to point out
where the obscurity is. He has resorted to a strained interpretation of the verse that he bases
on a translation similar to the KJV:

Now the sojourning of the children of Israel, who dwelt in Egypt, was four hundred
thirty years.

Mr. Schmitt is claiming that this verse is not saying that the Israelites sojourned IN EGYPT
430 years but merely that they sojourned 430 years. The dependent clause "who dwelt in
Egypt" is merely appositional, according to Schmitt, so its intention is to point out that the
Israelites who sojourned for 430 years did at one time dwell in Egypt but not necessarily for
430 years. The 430 years actually referred to the time that the Israelites sojourned both in

Volume 1990 - 2002 Issue


Page 769 of 2049
Skeptical Review Edited by Farrell Till
Canaan and Egypt, from the time that Yahweh made a covenant with Abraham until the time
that they left Egypt, so Schmitt says.

As I said, this is a strained interpretation, and it is fraught with more problems than Mr.
Schmitt could ever hope to resolve. First, it makes the Exodus author rather simplistic in his
writing style, because the first 11 chapters of Exodus tell all about the troubles that the
children of Israel had during their stay in Egypt, so it was rather obvious by the 12th chapter
that they had "dwelt in Egypt." If, then, Exodus 12:40 was intended to mean that the length of
the Israelite sojourn, altogether from the time of Abraham to the exodus, had been a period of
430 years, why would the writer bother to add parenthetically the obvious fact that they had
dwelt in Egypt part of that time? Anyone who wouldn't know this by the 12th chapter would
be a very careless reader.

More serious than this problem is the fact that both the Hebrew text and various English
translations of that text do not support Mr. Schmitt's strained interpretation of Exodus 12:40.
The best way to show this is by quoting various translations:

American Standard: Now the time that the children of Israel dwelt in Egypt was four
hundred and thirty years.
New American Standard: Now the time that the sons of Israel lived in Egypt was four
hundred and thirty years.
Revised Standard Version: The time that the people of Israel dwelt in Egypt was four
hundred and thirty years.
New Revised Standard Version: The time that the Israelites had lived in Egypt was
four hundred and thirty years.
New International Version: Now the length of time the Israelite people lived in Egypt
was 430 years.
Revised English Bible: The Israelites had been settled in Egypt four hundred and
thirty years.
New American Bible: The time the Israelites had stayed in Egypt was four hundred
and thirty years.
New Century Version: The people of Israel had lived in Egypt for 430 years.
Good News Bible: The Israelites had lived in Egypt 430 years.
Amplified Bible: Now the time the Israelites dwelt in Egypt was 430 years.
Jerusalem Bible: The time that the sons of Israel had spent in Egypt was four hundred
and thirty years.
Revised Berkeley Version: The period of the Israelites' stay in Egypt was 430 years.

I have 27 translations of the Bible in my personal library. I have consulted them all, and none
of them support Mr. Schmitt's strained interpretation of Exodus 12:40. Only two of them, the
KJV and Lamsa's translation from the Peshitta, put "who dwelt in Egypt" in apposition to "the
children of Israel," but that alone does not support Schmitt's interpretation, because even with
this translation, it is possible for the verse to mean what the above translations clearly indicate
that the statement meant: the Israelites dwelled or sojourned in Egypt for 430 years.

Young's Literal Translation of the Holy Bible renders the verse like this: "And the dwelling of
the sons of Israel which they have dwelt in Egypt IS four hundred and thirty years."

Volume 1990 - 2002 Issue


Page 770 of 2049
Skeptical Review Edited by Farrell Till
Hendrickson's Interlinear Bible gives this literal translation of the verse: "And the living-time
sons Israel's which lived in Egypt [was] thirty years and four hundred years." The marginal
translation, which attempts to express the meaning in idiomatic English that captures the
intention of the original, renders the verse like this: "And the time of the dwelling of the sons
of Israel, which they dwelt in Egypt, WAS four hundred and thirty years."

There just is no support in the original text or the various English translations for Mr.
Schmitt's very unlikely explanation of what Exodus 12:40 really meant. When we read the
above translations, we encounter no ambiguity at all, so we must reject Schmitt's premise that
Paul's statement in Galatians 3:17 is clearer than Exodus 12:40. They are both quite clear. The
Exodus writer said that the Israelites sojourned in Egypt 430 years, and Paul said that 430
years passed between the promise made to Abraham and the giving of the law of Moses. Both
statements can't be right, so that spells a peck of trouble for Mr. Schmitt's "conviction that
God is the author of the Bible." I suggest that he reexamine his basis for that conviction.

What could possibly explain such a glaring discrepancy as this? Why would Paul think that
430 years passed between the covenant made with Abraham and the giving of the law,
whereas the Exodus writer thought that the Israelites had spent 430 years in Egypt? A
possible answer to this question may be in the Septuagint version of the Old Testament:

And the sojourning of the children of Israel, while they sojourned in the land of Egypt
and the land of Chanaan, WAS four hundred and thirty years (Exodus 12:40, Brenton's
translation).

The Septuagint version, then, says exactly what Mr. Schmitt wishes that the various English
translations (derived from the Masoretic text) had said. The New Testament writers most
often quoted the Septuagint translation when they referred to Old Testament scriptures, so this
would explain why Paul believed that there had been 430 years between the land promise
made to Abraham and the giving of the law. He was merely saying what he believed was true
from his familiarity with the Septuagint. This may also explain why Stephen said that 75 souls
had come into Egypt with Jacob (Acts 7:14), when all English versions derived from the
Masoretic text put this number at 70 in Genesis 46:27. However, this verse in the Septuagint
says that 75 souls went into Egypt with Jacob, a tally that was derived from a variation in
verse 20, where five grandchildren of Joseph (two born to Manasseh and three to Ephraim)
were listed, who are not listed in the Masoretic text. Stephen--or Luke, who could very well
have put the words into Stephen's mouth--was probably stating what he knew the Septuagint
version had said on this subject.

So does this mean that no discrepancy exists between Exodus 12:40 and Paul's statement in
Galatians 3:17, that the misunderstanding has simply resulted from Paul's reliance on a
version of the Old Testament that read differently from the Masoretic version of Exodus
12:40? Well, Mr. Schmitt may choose to see it this way if he wishes, but doing so will create
more problems than it will solve. If Paul were inspired by the Holy Spirit when he wrote
Galatians 3:17, then we must conclude that the Septuagint version of Exodus 12:40 is correct
and the Masoretic incorrect. Likewise, if Stephen was indeed "filled with the Holy Spirit"
(Acts 6:10; 7:55), then his reliance on the Septuagint concerning the number of souls that
went with Jacob into Egypt would amount to the Holy Spirit's stamp of approval on that

Volume 1990 - 2002 Issue


Page 771 of 2049
Skeptical Review Edited by Farrell Till
version of Genesis 46:20-27. This would be compelling reason for Mr. Schmitt to believe that
he too should accept the Septuagint text over the Masoretic where differences in meaning
exist.

I seriously doubt if Mr. Schmitt would want to do that. The differences in the Masoretic and
the Septuagint versions are too numerous to discuss here, but any inerrantist who bothers to
research this subject will discover information that is hard to reconcile with his belief that the
Bible is the inerrant word of God. He will find a longer version of the book of Esther, a much
shorter version of the book of Jeremiah, apocryphal books that aren't in the Masoretic canon,
and numerous other variations. So the fact that their reliance on the Septuagint text led Paul
and Stephen to make statements that are inconsistent with the Masoretic is hardly a boon for
the inerrancy doctrine. It leaves rational people wondering (1) why the version of the Old
Testament that we have today is so different from the one that the omniscient, omnipotent
Holy Spirit inspired New Testament writers to quote, and (2) why God would verbally inspire
a book and then allow it to be so flagrantly tampered with.

Mr. Schmitt charged that the debate over the 430-year sojourn has been characterized by
semantic gyrations "full of sound and fury, signifying nothing," but after reading his attempt
to resolve this biblical discrepancy, we find that it is just more inerrantist flapdoodle, which in
the words of Shakespeare could be aptly described as "much ado about nothing." The
inerrancy of the Bible still stands very much in doubt.

From the Mailbag

I just received my first issue of The Believers' Chronicle. The editor's article about you was
quite revealing. And, are you really running from him? >From the elapsed time, it would
appear so. When do you plan on "chewing him up and spitting him out"? We Christians know
that the Holy Spirit will win out every time.

Well, I sure would like to see how you handle his article. And, how long it will take. Because
of his endorsement of TSR, please enter my free subscription. If you run from "ignorant"
Christians, however, I don't know what you could possibly teach me.

(J. D. King, 110 East 4th Avenue, Columbus, OH 43201.)

EDITOR'S NOTE: The following response was mailed directly to J. D. King, whom I suspect
from the style of the handwriting is Ms. J. D. King:

I have added your name to our mailing list to receive a free subscription to The
Skeptical Review. The next issue will be mailed on January 2nd if the printer has the

Volume 1990 - 2002 Issue


Page 772 of 2049
Skeptical Review Edited by Farrell Till
project finished on schedule. I send by bulk mail, so you should expect a delay of
seven to ten days for delivery.
For your information, I am sending you a copy of a letter and my editorial comments
on it that will be published in the March/April issue of The Skeptical Review. I thought
that sending it to you would be the easiest way to respond to your accusation that I am
"running" from Dennis Conley. My comments on the letter explain why after I
received my first communication from ASRC, I came to suspect that it is a bogus
organization that has falsified information about the size of the audience that it has.
Each communication from Mr. Conley simply increases that suspicion.
When I was first contacted by Mr. Conley--well, another "editor," to be more exact--I
had just retired from teaching college writing, so I naively envisioned myself having
enough time on my hands to do just about anything. About this same time, one of my
subscribers ran display ads in two consecutive issues of Bible Review, and this brought
a flood of inquiries and subscription requests that I still haven't cleared away. In
addition to this, a group of young skeptics who are heavily involved with the internet
put The Skeptical Review on the worldwide web and organized a "list" for me where
internet users could post arguments and rebuttals concerning biblical inerrancy issues.
This also added to my workload (far more than you can imagine if you have no
experience with the net). In other words, I am simply saying that I am just one person,
I do *everything* involved with publishing The Skeptical Review with the exception
of the actual running of the printing presses and stapling equipment that puts the
papers together. This is done by a nearby publishing company. Nevertheless, all of the
other activities draw heavily on my time. In addition to these duties, I write articles for
an area newspaper and also a regular column for The Secular Humanist Bulletin. All
of this takes time, and all of it generates a lot of mail that requires my personal
attention. If you know of any solutions to my time problems, I would be glad to hear
them.
Your insinuation that I am afraid of Mr. Conley is ludicrous to people who know me. I
have publicly debated preachers who are reputed to be the best that their
denominations have to offer. One of my opponents in a public debate was Norman
Geisler, a name you will instantly recognize if you know anything at all about
fundamentalist apologetics. I have proposed debates to Josh McDowell, who has
refused them with silence, and I have challenged Dr. Gleason Archer to debate. I can
produce a letter from him in which he refused the challenge. Just today, I discussed
proposals via telephone and internet with two different fundamentalist preachers that
will probably result in three public debates in 1996: one in Alabama, one in Georgia,
and one in Texas. I pay all of my expenses when I participate in these debates, and I
usually have two per year.
Does all of this sound as if I am "afraid" of Christians or that I "run" from them. As
the attached letter from the "Mailbag" column of TSR shows, I have challenged
Dennis Conley to debate publicly, but he has informed me that he doesn't debate and
that, in fact, no one in his organization debates. Well, I do debate, and if I can have
reasonable assurance of an audience to listen to the issues under discussion (rather
than the virtual nonaudience that I suspect I would have for any articles that I may
write for Conley's paper), I will gladly arrange my schedule so that I can come to
Columbus, OH, to confront whomever the Christians may designate to represent their
inerrancy view. Perhaps you would like to devote some of your time to make such a
debate a reality.

Volume 1990 - 2002 Issue


Page 773 of 2049
Skeptical Review Edited by Farrell Till
Finally, just let me say that I have seen nothing from Conley's publication or its
predecessor that even remotely resembles an argument. If I am wrong about this,
please send me a copy of the article in which you think a legitimate argument was
published, highlight it for me, and I will make the time to write a response to it. Isn't
that about as reasonable as you could expect me to be?
As for when I intend to chew up Mr. Conley and spit him out, if Ms. King will
persuade him to defend the doctrine of Bible inerrancy in public debate, I will go to
Columbus, Ohio, to oppose him. That will give her the opportunity to see me chew
him up and spit him out. I say this not because I think I have invincible debating
talents but because I know that biblical inerrancy is impossible to defend. I think that
the inevitable defeat Mr. Conley would suffer in such an encounter, if he is foolish
enough to accept the challenge, could properly be called a "chewing up and spitting
out."

Enclosed is a letter from ASRC [American Society for Religious Concern]. I sent for a copy
of their December issue of The Believers Chronicle, because I wanted to read your reply to
them on, I believe, a challenge to biblical inerrancy. The attached letter states that they have
not received any response from you.

I am wondering if, as they say, it's true that you have not responded to them? Is the reason
you did not respond to them because this organization is not worth the effort, or you sent them
a response but they said you did not just to embarrass you, as well as other skeptics that
would look for your article in The Believers Chronicle?

By the way, ASRC wrote me a letter saying I wrote an interesting letter in the last issue of
The Skeptical Review. What could have been so interesting in a letter that was requesting a
subscription to TSR is beyond me, and I basically told ASRC so.

(Roger La Porte, 148 Freund Street, Buffalo, NY 14215.)

EDITOR'S NOTE: For reasons noted above in my reply to J. D. King's Letter, at the time Mr.
La Porte received the letter from The Believers Chronicle I had not responded to anything
published about me in this paper or its short-lived predecessor The Stupid Christian
Chronicle. That, however, is no longer true. When I finally cleared away the backlog of
subscription requests and renewals, I wrote a response to Mr. Conley's article "The Progress
of Sin" and mailed it to him along with a renewal of my challenge for a public debate in
Columbus, Ohio, on the issue of biblical inerrancy. What happens now is up to Mr. Conley. If
he will accept my debate challenge, I think he will very definitely see that I am not "running"
from him as he has alleged in his paper.

Thank you for putting out such a wonderful publication. I have used up my first year's free
subscription to The Skeptical Review and am forwarding a check to continue my subscription
to the new bimonthly TSR. I would also like to order a complete set of TSR back issues from
Winter 1990 through the end of 1994. You indicate that 20 issues were printed during this

Volume 1990 - 2002 Issue


Page 774 of 2049
Skeptical Review Edited by Farrell Till
time and their cost is $1 each. The enclosed check includes $6 for my subscription, plus $20
for the back issues. Again thank you for your time and effort in making this publication
available and for increasing to a bimonthly schedule.

I enjoyed the pious stance taken by the folks at the American Society for Religious Concern
in V6, #5 of TSR. The sanctimonious people at ASRC seem to think that we atheists are
"literally too ignorant of the facts to even bother with intelligent interaction." Apparently they
think we nonbelievers have never been exposed to the so-called *real* truth! I also like the
way they believe we atheists "will fall by the wayside" when we are confronted by religious
and ethical exposure.

Because of these statements, I would like to know how often atheists have deserted their
beliefs to become Christians. Have any TSR readers, or ASRC readers for that matter, ever
known an intelligent, well read, and committed atheist who became a devout Christian? I'm
not talking about an unchurched nonbeliever; I mean an astute atheist who made a conscious
decision to convert. Does anyone know of such a conversion?

Personally, I have encountered several former Christians who are now atheists. These people
were not just "show up at church on Christmas and Easter" Christians; they were enlightened
fundamentalists and even preachers. On the other hand, I have not yet encountered one
formerly committed atheist who now considers himself a Christian. My observations and life
experiences emphatically support Farrell's statement that "the dropouts will be on the
Christian side."

I'll keep an eye out for the masses of broken atheists who fall to the so-called "facts and
intelligent interaction" put forth by ASRC in The Stupid Christian Chronicle. Intuition tells
me the more likely case will be other former Christians who ask themselves the same question
I finally asked myself: How could I have been so stupid!

(Mike Ulm, 3102 Allis, Springfield, IL 62703.)

EDITOR'S NOTE: Mr. Ulm raises a good question. I personally don't know a single person
who was once a committed atheist but is now a Christian, but I know several atheists who
were once very committed Christians. All one has to do is read the Mailbag column in TSR to
see evidence of that.

I do know that it is rather commonplace for Christians, especially preachers, to claim that they
were once atheists or agnostics but when they took the time to investigate the Bible, they saw
such compelling evidence that it was inspired of God that they became Christians; however, I
have never met a preacher that makes this claim whose reputation in the freethought
movement was established before he "converted" to Christianity. Josh McDowell, for
example, alleges that he was an atheist until he took the time to investigate the Bible, but I
don't know of a single article or book that McDowell wrote or a single lecture he presented on
the subject of atheism before his alleged conversion happened. I suspect that these claims of
atheism before conversion to Christianity are exaggerations or else honest delusions of people
who had no firm commitments to skepticism before becoming Christians. The closest that I
can come to the name of someone who changed from atheism to Christianity is Austin Miles,

Volume 1990 - 2002 Issue


Page 775 of 2049
Skeptical Review Edited by Farrell Till
the author of Don't Call Me Brother Anymore, and I don't actually know if Miles ever
considered himself an atheist. He made the transition from Christian to at least skepticism (at
which time he wrote the above book) and back to Christianity, but in all sincerity I have to
wonder how much he was committed to freethought. I personally find it hard to understand
how that any skeptic who takes the time to research biblical errancy and really become
informed on the subject could possibly return to believing that it is the "inspired word of
God."

As for ASRC's boasting that atheists will fall by the wayside if they dare to accept its
challenge to debate in the pages of its publication The Believers Chronicle, I think that this
group is in for a big surprise. I have often said to my fundamentalist debating opponents that
although changes in personal beliefs are rarely seen after a debate, the long-term effects favor
the side of skepticism. If an audience of a thousand people, evenly divided between Christians
and skeptics, should regularly attend debates between informed representatives of the errantist
and inerrantist views, the Christian side would lose far more of its people over the long run
than would the skeptics.

If anyone has any statistical information on the subject Mr. Ulm has raised, I will publish it in
a later issue.

Please renew my subscription to your interesting and informative paper for two more years.
Along with McKinsey's periodical and his Encyclopedia, it gives good ammunition to counter
the local JW's and assorted other holy-roller types. Of course, their minds are made up, but it
is interesting to confuse them with facts.

(Hugo Niemi, Box 66, Birsay, SA, Canada S0L 0G0.)

EDITOR'S NOTE: The publication by McKinsey that Mr. Niemi referred to is Biblical
Errancy, a monthly publication that also focuses on fallacies in the Bible inerrancy doctrine.
A free sample can be obtained from Dennis McKinsey, 2500 Punderson Drive, Hilliard, OH
43026. The "encyclopedia" that Mr. Niemi mentioned is The Encyclopedia of Biblical
Errancy written by McKinsey and published by Prometheus Books. Information about this
reference book can also be obtained at the above address.

I am delighted that you will be publishing The Skeptical Review six times a year rather than
just four. I certainly do appreciate the time and effort that you put into the answering of these
inerrantists. They look so stupid when you get through with them....

I was "into" religion for most of my life, and then I discovered things in the bible and my
church's teachings that did not make sense. Little by little, I began to read a set of books
called The Encyclopedia of Philosophy, and I had so much fun reading about those early
freethinkers and was yet sad to see how they were treated, rejected and persecuted (even
killed).

Volume 1990 - 2002 Issue


Page 776 of 2049
Skeptical Review Edited by Farrell Till
I always questioned when I would read about the life of a "thinker" and then saw that he
"repented" just before he died. Could it have been that religion had such a hold on his mind or
that death was such an awful thing to face that he might as well let the priest perform the "last
rites"? Of course, some very intelligent people were loyal to their church, but I find that so
inconsistent with reality.

(Ruth C. Riales, 1205 Forest Circle, Altamonte Springs, FL 32714-2820.)

EDITOR'S NOTE: Has there ever been an atheist or freethinker who died without believers
spreading tales of deathbed repentance? Such tales were told about Darwin, Ingersoll,
Voltaire, and others, even though people present at the times of death denied that any such
changes occurred. This is just another example of religious leaders pulling the wool over the
sheep's eyes.

As for how stupid inerrantists look when we have finished with them, I regret that most of
them apparently can't look stupid enough to turn them away from their absurd beliefs.

Help! During the time of my loss of faith, I learned doubt--doubt in God's existence, doubt in
my own thinking, and now doubt in what I once considered to be true. Now I am tired of the
unmitigated skepticism that has crept into my everyday thoughts. I seek something
constructive. Therefore, I ask readers of The Skeptical Review to help a neophyte agnostic by
recommending some books and authors in the following two areas.

First, I feel I need to know how to think: How can I come to reliable conclusions about
things? How can I justify the methods I have used in coming to my conclusions? What
presuppositions, if any, are necessary for knowledge?

Second, I want to know how language and logic are used in debate and discussion. Where and
how do things go wrong in discussions? How can I go about preventing and eliminating those
same problems from my own written and oral discussions?

I am looking for comprehensive works in these two areas that would normally have been used
as textbooks or reference books at the undergraduate level. Please do not recommend any
partisan atheistic books.

The reason I am asking the readers of The Skeptical Review for their recommendations is
because they seem to have a deep and abiding concern for knowledge and truth. The problem
I have is that the nontheistic community is not the only community that makes claims to
knowledge and truth. Other (theistic) communities propose their own ways to "truth," so I am
on a search to find reliable, justifiable methods to distinguish between the true and the false.
Anything that I believe after that would be built upon that foundation. Would you please help
me in my search by recommending some books and authors?

(John Mehlberg, 8026 Frederick Street, St. Louis, MO 63147-1837.)

Volume 1990 - 2002 Issue


Page 777 of 2049
Skeptical Review Edited by Farrell Till
EDITOR'S NOTE: There are several books that Mr. Mehlberg might find useful. Although
publishing companies sent me many textbooks about critical thinking during my teaching
career, I have kept in my personal library one I thought was particularly good. Published by
McGraw-Hill in 1978, Argument: a Guide to Critical Thinking by Perry Weddle may be out
of print now, but I'm sure it could be located through interlibrary loan. It was written for use
in freshmen writing classes, so its language and examples are easy to understand.

I posted Mr. Mehlberg's request on the internet, and Robby Berry, who is actively involved in
the www.infidels.org lists, suggested that I recommend A Rulebook for Arguments by
Anthony Weston. Don Morgan recommended Logic and Contemporary Rhetoric by Howard
Kahane and Beyond Feeling: a Guide to Critical Thinking by Ryan Ruggerio. The latter was
selected by the English department where I taught freshman composition as a textbook for the
second- semester writing courses, so I am familiar with it and would also recommend it.

If Mr. Mehlberg will check these out, I'm sure he will find one or more of them to be helpful.
He is to be commended for the objective attitude that was shown in his letter.

I have found many of the "From the Mailbag" letters to be beautifully written and very
inspiring. The personal stories of individuals (and families) struggling to free themselves from
fundamentalism or religion in general have renewed my determination to break through the
formidable Watchtower wall that has imprisoned my husband's mind and life for the past
seven years. Some of those letters have also helped reinvigorate the joy I felt (about a year
ago) when I was liberated from the irrationality and deception of religion.

I send my sincere thanks to you for creating TSR and to the letter writers for sharing their
feelings and experiences.

(Kristine Jackson, 825 129th Avenue NE, Blaine, MN 55434.)

Religion bashing is a controversial subject among humanists. For years I have felt torn. On
the one hand, religion bashing seems immature and negative. One might ask when we're
going to get over our oppressive indoctrination and get on with our lives. On the other hand,
when I see religionists seeking to get rid of religious freedom and diversity and seeking to
establish theocracy, I feel I need to protest and to fight for my right to exist. But protest and
fighting get us labeled as strident troublemakers. It's common for people who protest
government endorsement of religion to be driven out of town. Yet to do nothing is to invite
the violations to continue and become precedent for more. There's got to be a better way. I
keep thinking diplomacy is the answer.

I am asking you this because I have been impressed by the way that you seem to maintain
cordial relationships with even your most strident opponents.

(Heidi Johnson, 12221 Berry Street, Wheatland, MD 20902.)

Volume 1990 - 2002 Issue


Page 778 of 2049
Skeptical Review Edited by Farrell Till
EDITOR'S NOTE: Sometimes my relationships with my fundamentalist opponents are not so
cordial. As I look back on those relationships that have soured, I see that it was the attitudes
of my opponents that caused the rifts. By the very nature of their temperaments,
fundamentalists have to believe that they are right, so when their positions don't fare so well
in public debate, it isn't unusual for them to lash out in anger. On the other hand, I have been
able to maintain cordial relationships with some of my opponents. Perhaps this is because I
was once a fundamentalist myself and understand the way that they think, especially their
need to be right, so I don't allow their inflexibility to bother me.

On the matter of whether skeptics should engage in religion bashing, I think it is very obvious
where I stand on this issue. The reason for my position was stated in Ms. Johnson's letter. If
we don't oppose the militant religious right, whose roots are firmly planted in biblical
inerrancy, we may find some important personal freedoms being taken away from us via
legislative processes controlled by this vociferous minority.

I have delayed too long in writing to thank you for your work, and for the subscription to The
Skeptical Review. Enclosed is a check for $25 to pay for my first year's subscription and to
renew it for the coming year. I have also included two computer disks for the ASCII files of
previous issues. If there is any balance after deducting these costs, I'd appreciate if it could be
applied to the rental of a debate video or two. I have no particular preference, but since I
enjoy the God Squad's flailing attempts at disengaging itself from the ridiculous positions it
worships, send me that which presents them at their silliest. I guess that doesn't narrow it
down much.

From a recreational standpoint, I must admit I enjoy the comedic efforts of the many
preachers, healers, and defenders of the dogma. Unfortunately, after the laughter subsides, as I
watch or listen to them, the sobering realization sets in that their followers (believers seems
too strong a word) accept their assertions, and send them money--lots of it!

Living in Central Pennsylvania, which seems to be a hotbed of fundamentalist sentiment, it is


impossible not to be confronted by their message. The essence of the logic (?) I am
confronted with, it seems, is to make a ridiculous and unverifiable claim in the positive,
challenge someone to "prove" the equally unverifiable negative, which of course he or she
can't, thereby "proving" the positive. It seems a bible-believer's best defense of the bible's
contradictions is to ignore them and pose a claim in the manner I described.

Although I don't seek out discussions with fundamentalists, I also don't shy away from them.
It seems that each conversation with them generally gets around to their finally admitting that
god was lonely and looking for someone to love him out of the exercise of free will. That
immediately begs the question. Couldn't an omnipotent, all- knowing god create a race of
beings that did so without the obvious consequences that the exercise of mankind's free will
has thus far engendered? Shouldn't "free will" be uncoerced?

I will close by posing a challenge to bible-believers that has yet to be answered. I make the
challenge in response to their claim that morality is impossible without god. I generally

Volume 1990 - 2002 Issue


Page 779 of 2049
Skeptical Review Edited by Farrell Till
answer that claim by saying that, in my own experience, the two [god and morality] are
mutually exclusive. One can do what is moral or one can obey god. The challenge? Find one
example of absolute morality in action in the bible. That is, show me one situation of any
positive or negative behavior that is, in and of itself, right or wrong. I have found none, nor
has any ever been pointed out to me. Thou certainly shall lie, murder, enslave, steal, etc., etc.,
etc., * if god says so.*

(Mark A Glazewski, 7309 Sandy Hollow Road, Harrisburg, PA 17112.)

EDITOR'S NOTE: I would like to add another challenge to Mr. Glazewski's. If objective
(absolute) morality exists, how can anyone possibly find out what it is? This certainly can't be
done by reading "God's inspired word," because there never have been two people living on
the face of the earth who understood the Bible exactly alike.

Since you changed my article by editing some of my arguments out of it, I respectfully
request that you not print my article. Is it not true that you have printed at least one article (or
a letter) making references to the probability argument that you edited out of my article? In
fact, did you not make a reference to an article from a person in Texas? Incidentally, I have
never received the article from the person in Texas. Have you not printed at least one article
(or a letter) that referred to at least one of the other arguments I addressed in the material you
deleted from my article? If you answer "yes" to these questions, why did you edit my article
in such a manner?

I have for years contended that the atheist has no objective system of ethics to guide his
actions. What system do you use to determine that you can edit my article in such a manner?

I am truly sorry that you see fit to conduct yourself in this manner. For this reason I do not
think I should devote any more of my time to making any replies to your paper The Skeptical
Review.

(Marion R. Fox, 4004 Twisted Trail Road SE, Oklahoma City, OK 73150- 1910.)

EDITOR'S NOTE: This letter from Mr. Fox was received on Thursday, February 29, 1996,
and on Saturday, March 2, the printer called to tell me that I could pick up the March/April
issue of TSR in which Mr. Fox's article appeared. Obviously, he waited too long to inform me
that he did not want his article to be published. There was nothing that I could do to stop it.

Mr. Fox has accused me of dishonesty, so in the July/August issue of TSR, I will publish the
opening paragraphs that I omitted from his article. This will be sufficient for readers to see
why I deleted them. They refer to two arguments that were discussed in my debate with Jerry
Moffitt in which Fox served as Moffitt's moderator. Fox's article gave no background
information concerning what either Moffitt or I had said in the debate about probabilities. He
simply launched into a discussion of the differences in the probability of related and unrelated
events, so what he said will be incomprehensible to those who have not seen the debate tapes.
For that reason, I deleted this section of his article.

Volume 1990 - 2002 Issue


Page 780 of 2049
Skeptical Review Edited by Farrell Till
I did publish a letter from a reader who viewed the debate tapes and sent to me his comments
on the absurdity of Moffitt's use of probability in the debate, but that writer gave sufficient
background for his comments to be understood by people who had not seen the debate tapes.
Fox, on the other hand, seems unable to express himself clearly in writing, so he has
repeatedly made the mistake in his articles of giving insufficient details for his comments to
be understood. In reading his articles, I often had to back up and read two or three times what
he had said just to make an educated guess at what he meant. In the case of his comments
about the *Moffitt-Till Debate,* Fox made the mistake of assuming that because he knows
what Moffitt and I said about probability factors, his readers will too without having it
explained to them. So, as everyone will see in the next issue when I publish his explanation of
the difference in the probability of related and unrelated events, he at no time stated what I
had said in the debate that would indicate that I don't understand the difference. How much
sense is that going to make to readers who have not seen the debate?

As for the "article from a person in Texas" that Fox referred to, this was actually a letter from
a statistician who attended the debate and wrote a letter to Moffitt to point out his misuse of
probability arguments. I received a copy of the letter, and Moffitt sent me a copy of a reply
that he sent to this person. Moffitt's reply was quite brief. He said that he was not a
mathematician, and so he had sent the letter to Fox, who would write a reply to it. If Fox
didn't receive this letter, perhaps he will want to contact Jerry Moffitt to find out why.

At any rate, for Fox or any inerrantist to accuse me of intellectual dishonesty is ludicrous. I
defy him to send me the name of a single fundamentalist publication that would give skeptics
the same kind of forum that The Skeptical Review accords to believers in biblical inerrancy.
We literally beg inerrantists to refute the articles we publish and to submit their articles in
defense of the inerrancy doctrine.

I always look forward to receiving TSR. I live in a bible-belt region and have a fundamentalist
wife and families. It is never easy not to believe. I was a "born again" Christian until the age
of 25 and still describe myself as a "recovering Christian."

Do you know of any forums where former believers discuss their views and experiences of
dealing with loved ones who still are believers (as opposed to the inerrancy discussions in
TSR). I am looking for more general dialogue with others in similar situations: how to deal
with raising children in a "mixed" marriage, etc.

(Kevin Berry, Box 16, Site 1, Route 1, Fredericton Junction, NB, Canada E0G 1T0.)

EDITOR'S NOTE: There is a site on the internet where ex- and recovering Christians discuss
issues that Mr. Berry is interested in. Anyone may subscribe by sending the following
message to listproc@infidels.org: "Subscribe ex-tian ." The subscriber's own name should go
where is. The lists on the tamu.edu system may be moved to another server by the time this
issue appears. If your subscription request is returned or receives no reply, contact me at
jftill@midwest.net.

Volume 1990 - 2002 Issue


Page 781 of 2049
Skeptical Review Edited by Farrell Till

Thank you for the free issues of The Skeptical Review that I have received. Enclosed is $6 for
the coming year.

I loved "God, Captain Scott O'Grady, and the Atlanta Braves." It is one of those articles I
wish I'd written, and, indeed, exactly expresses my thoughts on the subject. I am another
reformed fundamentalist Christian, but, unlike some imperturbable types, I'm mad as hell and
having a hard time with my anger. Not only did I waste years of my life and talent on
religious lies, but my self-esteem was involved in the fellowship and in various positions of
authority within the church.

It's lonely out here, fellow skeptics. Are there others around the Great Falls/Helena, Montana,
area who would like to meet a 59-year-old woman for discussion, sharing materials, and help
me work off my anger with a hike in our beautiful mountains?

(Jane Basta, 26 Willington Lane, Cascade, MT 59421.)

EDITOR'S NOTE: I think anger is a natural stage that one goes through after realizing that
he/she has been duped into devoting a big chunk of life to a lie. I recall going through that
stage until finally I just shrugged it off. "Oh, well," I told myself, "at least I got to see a lot of
the world and learn another language when I was working as a missionary." Still it's a shame
that so many people are wasting their time and resources on a colossal falsehood. What would
humanity have achieved by now if all of the people who threw their lives away on religious
nonsense had devoted their time and energy to making life better?

I greatly appreciate your sending me a copy of The Skeptical Review. I want to make sure that
I receive each issue. I was raised in a fundamentalist environment and eventually even
graduated from Oral Roberts University. How- ever, I did a dangerous thing a while back. I
started thinking. This is not encouraged within Christianity. Eventually that thinking led me to
embrace reality, not mysticism. It has cost me a lot in terms of lost friends, family members
hurt, and cost me my marriage. However, it has been well worth it to live life embracing truth
and not relying on hocus- pocus.

Enclosed you will find a check for $100 to cover my subscription to The Skeptical Review for
however long $100 will keep it going. Just drop me a note and let me know how long I can
expect to receive your wonderful publication.

(Terry Brock, 3481 Lakeside Drive, Suite 3202, Atlanta, GA 30326; e-mail,
terrybrock@aol.com)

EDITOR'S NOTE: "I did a dangerous thing.... I started thinking." How many times have I
heard that? I like to hear stories like Mr. Brock's, although I regret the personal difficulties
that followed his abandonment of religious superstition. I wish him well.

Volume 1990 - 2002 Issue


Page 782 of 2049
Skeptical Review Edited by Farrell Till
In compliance with his request, I have entered a 16-year subscription for Mr. Brock. Others
have sent checks for 5- or 10-year subscriptions, so I think it only right that I should inform
everyone that I will be 63 years old by the time this issue is mailed. I just may not be around
long enough for these people to get their money's worth.

Back Page

Thrasher-Till Debate
A debate on prophecy fulfillment will be conducted at Huntsville, AL, on June 24th and 25th.
The disputants will be Thomas N. Thrasher, a Church-of-Christ preacher in Decatur, AL, and
Farrell Till, editor of The Skeptical Review. People living in the Huntsville- Decatur area may
obtain information about the debate by contacting Mr. Thrasher at P. O. Box 1941, Decatur,
AL 35602. His e-mail address is tnthrash@traveller.com.

Debating Opponent Wanted


We have been contacted by a graduate of a Bible school who is interested in defending the
issue of the resurrection or the existence of God in public debate. He is presently doing
additional religious studies at Southern Christian University. He is 27 years old and would
like to have an opponent who is about his age and has the same level of education and
experience. He has already had one debate and has two more scheduled. Preferably, he would
like to have the debate on a college campus in the southern part of the United States.

Anyone interested in this proposal should contact John Lewis at P. O. Box 65, Lone Grove,
OK 73443, or call him at (405) 657-4455.

Our Suspicions Confirmed


In the Mailbag column of this issue, as well as earlier issues, we expressed a suspicion that
the American Society for Religious Concern was a phony organization. That suspicion has
now been confirmed. In January, "editor" Dennis Conley mailed a letter announcing the
demise of The Believers' Chronicle and laid the blame for his paper's failure on "Satan and
Farrell Till." Just recently he sent us another letter in which he claimed the whole thing had
been a joke. In reality, Conley claimed, he is a "freethinker" who "left fundamentalist
Christianity 15 years ago." He further said, "I agree with everything you say," and admitted

Volume 1990 - 2002 Issue


Page 783 of 2049
Skeptical Review Edited by Farrell Till
that his claim of having 1500+ subscribers was an exaggeration. Now he says that he had
actually had only "a handful of subscribers... 12 to be exact."

Conley said that he had just wanted to have a little fun and that he had succeeded by getting
so much attention from atheists. We really think that Conley is just trying to save a little face,
but in case he is finally telling the truth, all we can say is that the freethought community
hardly needs friends like him.

Errancy@infidels.org
If your computer is on line, you may subscribe to errancy@infidels.org, an internet list owned
by the editor of The Skeptical Review, and participate in the discussion of issues related to
biblical inerrancy. Information on how to subscribe can be obtained from jftill@midwest.net.
Your only cost will be the fee you pay for e-mail service.

Volume 1990 - 2002 Issue


Page 784 of 2049
Skeptical Review Edited by Farrell Till

The Skeptical Review


Volume Seven, Issue Four
July/August 1996
Farrell Till, Editor

• Did Marco Polo Lie?


Biblicists often argue that there is as much proof for the historicity of Jesus as there is
for the existence of Julius Caesar and other historical characters. A recent controversy
surrounding a well known historical character will illustrate the erroneous thinking of
those who so argue.
• The Resurrection of Christ: Myth or Reality?
Matthew Perman argues for the historicity of the resurrection of Jesus.
• Put Me Down for Myth
Farrell Till rebuts Perman's article.
• Daniel in the Historians' Den
Daniel may have survived meat-hungry lions, but fact-hungry historians have ripped
him to shreds.
• Cretinism or Evilution?
Information on how to subscribe to Ed Babinski's biannual journal, Cretinism or
Evilution?
• The Omitted Section
Marion R. Fox points out two errors made by Farrell Till in his debate with Jerry
Moffitt. This section was originally omitted from an earlier article by Fox.
• So What's the Point?
Till explains his original decision to omit a section of Fox's article, and why he
decided to go ahead and publish it.
• Marion Fox's Logic
Dave Matson exposes an error in Fox's apologetic technique.

Volume 1990 - 2002 Issue


Page 785 of 2049
Skeptical Review Edited by Farrell Till
• From the Mailbag
Letters to the editor.

Did Marco Polo Lie?


by Farrell Till
We often hear biblicists argue that there is as much proof for the historicity of Jesus and the
events attributed to him in the New Testament as there is for the existence of Julius Caesar
and other historical characters. They claim that those who reject biblical characters and events
have no logical basis for accepting anything that we have learned through historical records.
A recent controversy surrounding a well known historical character will illustrate the
erroneous thinking of those who so argue.

The March 24, 1996, issue of the Peoria (Illinois) Journal Star ran an Associated Press story
about recent challenges to the historical accuracy of Marco Polo's tales of his travels in China.
In 1295, Marco Polo returned to Venice after 24 years of travels and told of his adventures in
China. British historian Frances Wood, however, has recently published a book Did Marco
Polo Go to China? in which she seriously questions whether the Venetian explorer's travels
ever took him into China proper. "It is a terrific story," she says. "The only trouble is that
there is no evidence to support it. Like so many other great historical legends, the story is a
myth."

Marco Polo reported that he had spent years exploring China for the Mongol ruler Kublai
Khan, but Wood found evidence that this is at best an exaggerated claim. Among other flaws
that she found in her critical analysis of Marco Polo's tales was his failure to mention the
Great Wall of China, the Chinese tea-drinking ceremonies, and the widespread practice of
foot-binding. The Associated Press article stated that other historians have also questioned
whether Polo actually went to China but that it has been difficult to make inroads against the
"prevailing view."

This challenge to a deeply en- trenched "historical" view illustrates how wrong biblicists are
when they assume that scholars accept everything that has been taught as historical facts. True
scholars constantly subject history to critical analysis and make intelligent judgments about
which claims are credible and which ones aren't. The authorship of the plays traditionally
attributed to William Shakespeare has been challenged by several scholars of English
literature. In "Those Amazing Biblical Numbers" ( TSR, Winter 1995, pp. 5-8), William
Sierichs, Jr., cited historians who have questioned the accuracy of the numbers that Herodotus
attributed to the armies of the Persian emperor Xerxes. (See also Sierichs's letter on page 12
of this issue.) In our own times, we have witnessed challenges to the "official" versions of
what happened in Dallas, Texas, the day President Kennedy was assassinated and at Waco,
Texas, at the Branch Davidian Compound. In other words, historical scholars don't just pick
on the Bible out of some devious "liberal" conspiracy to discredit a book that is important to

Volume 1990 - 2002 Issue


Page 786 of 2049
Skeptical Review Edited by Farrell Till
Christianity. They apply their principles of scientific investigation to all historical claims in a
scholarly search to determine what probably did and what probably did not happen.

When biblicists object to historical scholars who apply to the Bible the same investigative
methods that they use in evaluating the accuracy of secular history, they are actually
demanding that the Bible be granted some kind of privileged status. In the case of Marco
Polo's silence on such matters as the Great Wall of China and the practice of foot-binding,
historians like Frances Wood are recognizing the improbability of Marco Polo's not
mentioning such unusual landmarks and customs if he had indeed traveled as extensively in
China as he claimed. This silence is recognized as a legitimate reason to question the accuracy
of his other reports. If such a method is proper in the critical analysis of secular history, why
would Bible believers consider it improper to use the same method in the critical analysis of
biblical history?

The answer is simple: biblicists know that critical analysis of the Bible will destroy its
credibility. Let's consider a situation somewhat like Marco Polo's silence on Chinese
landmarks and customs that surely would not have escaped his notice had he traveled in
China as extensively as he claimed. The gospel of Matthew states that an earthquake struck
Jerusalem at the moment Jesus died and that graves were opened after which "many saints
who had fallen asleep were raised" (27:51-53). These many saints went into the city and
"appeared to many." If such an event had actually happened, surely any other person present
on the scene at the time would have included it in any account of that day that he himself
might later write, yet the gospel of John (traditionally ascribed to the apostle John who was
allegedly present at the crucifixion) said nothing at all about this remarkable phenomenon.
Neither did he mention the three hours of darkness at midday, when the sun's light failed, that
all three synoptic writers put into their gospels (Matt. 27:45; Mark 15:33; Luke 23:44). How
likely is it that references to events as remarkable as these would have been omitted in any
historical accounts written by people who were present when they happened?

There are only two reasonable conclusions to reach in this matter: (1) If the apostle John was
on the scene of the crucifixion, his failure to mention the three hours of darkness at midday
and the earthquake that opened the tombs of "many" saints is reason to believe that neither
event happened, or (2) if either or both really did happen, their omission from John's gospel is
reason to believe that it was not written by an eyewitness to the phenomenal events of that
day. In saying this, we are being no more critical of the Bible than historians are when they
weigh what Marco Polo did not say against what he did say in order to evaluate the accuracy
of his records.

The problem posed by the omission of these phenomenal events in some of the New
Testament documents is further complicated by the failure of contemporary secular writers to
mention either the midday darkness or the resurrection of "many" saints. One would think that
the resurrection of "many saints" and their subsequent appearance to "many" in the city would
have created such a stir that word of it would have spread abroad even in a time that had no
rapid communication systems. When this event allegedly happened, Jewish communities
existed all over the known world, so surely some of the "many" in Jerusalem who saw these
"many" resurrected saints would have told about it in their visits and letters to relatives and

Volume 1990 - 2002 Issue


Page 787 of 2049
Skeptical Review Edited by Farrell Till
friends in other countries, yet there is no known mention of it outside of Matthew's gospel and
apocryphal works whose authenticity is rejected even by the church.

The three hours of darkness at midday poses an even bigger critical problem. One might
conceivably imagine how that the resurrection of the "many" managed to go unmentioned in
contemporary secular records, because knowledge of it in other regions would have depended
on contacts with the "many" witnesses in Jerusalem who had seen these resurrected saints.
Although it is highly unlikely that all who had had contacts with those "many witnesses"
would have failed to leave written records of such a remarkable tale, we will concede that
such historical silence could at least fall within the range of far-fetched possibility. The term
many would not denote all, so let's try to imagine that these "many" and the friends and
relatives they talked to about the event just all happened to be people who did not leave
written records. Such a scenario, although very unlikely, may have been possible, and we will
concede that point for the sake of argument.

The matter of the mysterious darkness at midday would have been entirely different. This
would have been something witnessed not just by "many" but by all who were up and about
that day. Furthermore, it would have been a phenomenon noticed not just by the people in
Jerusalem but by people in other regions. The synoptic writers described this as a darkness
that fell over "all the land" (Matt. 27:45; Mark 15:33). The Greek word for land in both texts
was ge, which was the same word used to denote the entire earth, in the same way that the
French language has one word, terre, for both "land" and "earth." It was entirely possible,
then, that the synoptic writers meant to say that the darkness fell over all the earth. Indeed, the
word ge in Luke's account (23:44) was translated earth in both the KJV and the NKJV.

At the very least, we have to conclude that the synoptic writers meant that this was a regional
darkness. That being true, we must wonder why no one but the synoptic writers referred to it.
In superstitious times when comets and brief eclipses often produced widespread panic, such
a remarkable phenomenon as three hours of darkness at midday would have created such
mass hysteria that records of it would have been left all over the hemisphere that had
experienced it. But no such records exist. The Jewish historian Josephus, whose father was a
priest in Jerusalem at the very time Jesus was allegedly crucified (The Life of Flavius
Josephus, 2:7), mentioned nothing about a midday darkness that was followed by an
earthquake and a mass resurrection from the dead. If such events as these had actually
happened, surely Josephus's father would have talked about them enough in the family setting
for Josephus to have known about them.

Inerrantists have made unconvincing attempts to claim that the midday darkness was
mentioned by a writer named Thallus, but in actuality, the writings of this person have not
survived. In A. D. 221 (circa), a Christian writer named Julius Africanus wrote the following
statement that contains a reference to Thallus, who allegedly alluded to the midday darkness
at the time of the crucifixion: "Thallus, in the third book of his histories, explains away this
darkness as an eclipse of the sun--unreasonably, as it seems to me...."

This fragment from Thallus embedded in the writings of Africanus is much too abbreviated to
determine for sure if it was indeed a reference to the alleged midday darkness mentioned in
the synoptic gospels. Did Thallus indeed refer to a period of darkness at the time of the

Volume 1990 - 2002 Issue


Page 788 of 2049
Skeptical Review Edited by Farrell Till
crucifixion, or did Africanus merely take a reference to an eclipse at some unspecified time
and assume that Thallus was referring to a period of darkness when Jesus was crucified? Did
Thallus declare as a matter of fact that such an event had happened or was he merely trying to
offer a sensible explanation for something that had by that time (circa A. D. 52) become a
part of Christian legend? We have no way of answering these questions, because we do not
have the works of Thallus to examine the reference in context.

Christian "apologists" like Josh McDowell also cite a first-century writer named Phlegon as
another witness to the midday darkness on crucifixion day. As it turns out, however, the
writings of Phlegon have not survived either, and the only evidence we have that he ever
wrote anything about this event are claims by Christian writers like Julius Africanus that
"Phlegon mentioned the eclipse." Aside from the fact that the secondhand references from
Africanus give only fragments of the original contexts, two other observations about these
alleged testimonies seem in order: (1) it is rather strange that if such writers as Thallus and
Phlegon really had referred to a three-hour period of midday darkness, the Christian
community did not zealously preserve these works as non-Christian testimony to a significant
event in the life of Jesus. (2) Both secondhand references say that Thallus and Phlegon called
the darkness an "eclipse," but anyone who knows much at all about eclipses would know that
their duration is measured in minutes, not hours. This would suggest that whatever these
secular writings were referring to, they were not talking about a three-hour darkness.

In view of how remarkable a three-hour period of darkness at midday would have been, the
uncertain testimony of secular historians whose works didn't even survive the vagaries of time
seems flimsy indeed. Such an event would have been so phenomenal that firsthand testimony
to it would surely have survived in various parts of the world that experienced the
unexplained darkness. In closing the 14th chapter of his famous work The Decline and Fall of
the Roman Empire, Edward Gibbon said this about the midday darkness alleged in Matthew's
gospel:

Under the reign of Tiberius, the whole earth, or at least a celebrated province of the
Roman Empire, was involved in a preternatural darkness of three hours. Even this
miraculous event, which ought to have excited the wonder, the curiosity, and the
devotion of mankind, passed without notice in an age of science and history. It
happened during the lifetime of Seneca and the elder Pliny, who must have
experienced the immediate effects, or received the earliest intelligence, of the prodigy.
Each of these philosophers, in a laborious work, has recorded all the great phenomena
of nature, earthquakes, meteors, comets, and eclipses, which his indefatigable
curiosity could collect. Both the one and the other have omitted to mention the
greatest phenomenon to which the mortal eye has been witness since the creation of
the globe. A distinct chapter of Pliny is designed for eclipses of an extraordinary
nature and unusual duration; but he contents himself with describing the singular
defect of light which followed the murder of Caesar... (emphasis added).

In a footnote, Gibbon said that the celebrated passage of Phlegon has been "wisely
abandoned." Nevertheless, diehard inerrantists like Josh McDowell still resort to it in their
desperation to find secular testimony to the phenomenal claims of the New Testament
gospels. Critical students of the gospels, however, will apply to these phenomenal claims the

Volume 1990 - 2002 Issue


Page 789 of 2049
Skeptical Review Edited by Farrell Till
same standards that historians like Frances Wood have applied to the tales that Marco Polo
told about his travels in China. Through application of these critical methods, we can
reasonably conclude that the failure of Seneca and Pliny, the two greatest natural scientists of
that age, to mention a three-hour period of darkness at midday is compelling evidence that it
did not happen.

To accord the Bible privileged status is intellectually dishonest. If any of its claims cannot
pass critical scrutiny, they should not be believed.

The Resurrection of Christ


Myth or Reality?
Matthew Perman

Imagine that you have been hiking all day in the mountains of Colorado, and you are lost. An
enormous snowstorm is fast approaching, and if you do not find a way out soon, it will cost
you your life. Up ahead there is a fork in the road, where you see two people. One is lying
down on the ground--dead. The other is standing up and wearing a park ranger's uniform--
alive. Whom would you ask for directions? Obviously, the living one.

A similar situation surrounds the questions of life after death. Many religions claim to have
the answers, but they all contradict one other (Mortimer J. Adler, Truth in Religion: The
Plurality of Religions and the Unity of Truth, Macmillan, 1990). They cannot all be true. So
how can we know whom to believe? Christianity seems to be unique. Its founder and leader,
Jesus Christ, not only experienced death, but it is claimed that He also rose from the dead and
remains alive. If this is true, whom would you believe concerning matters of eternal destiny--
one who is lying in his grave, or one who has risen from the grave?

If Jesus has risen, it would seem reasonable to consider His claim to be the only way to God:
"I am the way, and the truth, and the life; no one comes to the Father, but through me" (Jn.
14:6). But, according to the apostle Paul, "If Christ has not been raised, [the Christian's] faith
is worthless" (1 Cor. 15:17). So the question arises: Did Jesus really rise from the dead?

To investigate this issue, we will examine six facts that virtually all scholars--even critical
non-Christian scholars--who address Christ's resurrection accept as historical (Antony Flew
and Gary Habermas, Did Jesus Rise from the Dead? pp. 19-20). We will also see that the
Bible is not the only source of evidence for Christ's resurrection. John Singleton Copley,
recognized as one of the greatest legal minds in British history, sums up the matter well: "I
know pretty well what evidence is; and I tell you, such evidence as that for the resurrection
has never broken down yet" (Wilbur M. Smith, Therefore Stand, p. 425).

Volume 1990 - 2002 Issue


Page 790 of 2049
Skeptical Review Edited by Farrell Till
1. Jesus Christ died from the rigors of crucifixion and was buried in a tomb.

A. Jesus was crucified. Extrabiblical sources (sources apart from the Bible) confirm this fact.
Of particular interest is a reference by Thallus, a non-Christian Samaritan historian. He
regarded the crucifixion of Jesus as so significant that he included it in his History of the
World, which he wrote about A. D. 52. Thallus tried to explain away the darkness that fell
when Jesus died on the cross as an eclipse of the sun (F. F. Bruce, The New Testament
Documents: Are They Reliable? Intervarsity Press, 1972, p. 113). Jewish sources also refer to
Jesus's crucifixion at Passover (The Babylonian Talmud, Sanhedrin, 43a).

B. Jesus was dead. The nature of crucifixion ensures death. After analyzing the medical and
historical evidence leading to Jesus's death, an article in the Journal of the American Medical
Society concludes: "Accordingly, interpretations based on the assumption that Jesus did not
die on the cross appear to be at odds with modern medical knowledge" (March 21, 1986, p.
1463).

2. Jesus's tomb was empty just a few days later.

Professor of philosophy Dr. G. R. Habermas in his Ancient Evidences for the Life of Jesus
writes: "Our study [of the extrabiblical sources] has shown that Jesus taught in Palestine and
was crucified and buried in Jerusalem under Pontius Pilate. These sources assert that
Christianity had its beginnings in the same location." Christ's apostles did not go to some
obscure place to begin preaching about His resurrection but instead went back to the city of
Jerusalem, the very place of Jesus's execution and grave. If what the apostles were preaching
had been false, it would have been evident to the people in Jerusalem and Christianity more
than likely would not have begun.

This situation therefore demands that Jesus was no longer in His tomb. Paul Althaus writes
that the resurrection proclamation "could not have been maintained in Jerusalem for a single
day, for a single hour, if the emptiness of the tomb had not been established as a fact for all
concerned" (Josh McDowell, Evidence that Demands a Verdict, Thomas Nelson Publishers,
1979, p. 217).

Second, early Jewish testimony admits the empty tomb. Matthew 28:11-15 makes reference to
the Jewish assertion that the disciples stole the body. The author then adds that this story was
still being spread at the time when he was writing. This text could not have been written
unless there really was a Jewish counterargument to the empty tomb; otherwise, this passage
would have been exposed as a fraud. Also, the passage would have been pointless, since its
main purpose was to refute the Jesus allegation. The significance of this is that the early Jews
did not deny the empty tomb but rather admitted the empty tomb by trying to explain it away.
Additionally, Josh McDowell points out that a compilation of 5th-century Jewish writings,
called the Toledoth Jeshu, acknowledges that the tomb was empty. Dr. Paul Maier calls this
"positive evidence from a hostile source, the strongest kind of historical evidence. In essence,
if a source admits a fact that is decidedly not in its favor, the fact is genuine." That is exactly
the case with the empty tomb.

Volume 1990 - 2002 Issue


Page 791 of 2049
Skeptical Review Edited by Farrell Till
Because of the strong case for the empty tomb, there are many natural theories that attempt to
explain it away in order to deny Christ's resurrection.

A. Did the disciples go to the wrong tomb? This cannot be the case because the Jewish
authorities, since they were against Christianity, would have wasted no time producing the
body of Jesus from the proper tomb, putting an end to Christianity. Surely someone would
have discovered this "mistake."

B. Did the disciples steal the body? If so, then the men who delivered to the world the highest
moral standards it has ever known were frauds, liars, and hypocrites. Is this credible to
believe? Paul Little asks, "Are these men, who helped transform the moral structure of
society, consummate liars or deluded madmen? These alternatives are harder to believe than
the fact of the resurrection, and there is not a shred of evidence to support them" (Paul Little,
Know Why You Believe, Scripture Press Publications, Inc., 1971, p. 63).

C. Did the Jews or the Romans steal it? Dr. John Warwick Montgomery dispels this
possibility: "It passes the bounds of credibility that the early Christians could have
manufactured such a tale and preached it among those who might easily have refuted it by
producing the body of Jesus" (John Warwick Montgomery, History and Christianity,
Intervarsity Press, 1972, p. 78). If they had the body, why didn't they put the corpse on a cart
and wheel it through Jerusalem, thus eliminating for all time any belief in Christ's
resurrection?

D. What about grave robbers anonymous? They steal what's on the body, not the body. Who
would want to steal a dead corpse?

In addition, most scholars today reject these natural theories because they all fail to explain
another crucial factor:

3. The disciples had real experiences with one whom they believed was the risen Christ.

This fact is not widely disputed today, even among critical scholars (Carl Braaten, History
and Hermeneutics, p. 78) because of the firsthand testimony supporting it. The gospels, which
record these appearances, claim to have been written by eyewitnesses to the life of Jesus and
by those who recorded eyewitness testimony. These internal claims are confirmed by external
sources (History and Christianity, pp. 31-35). In addition, the reliability and trustworthiness
of the New Testament has been confirmed by extrabiblical sources and archaeology (Evidence
that Demands a Verdict, pp. 65-74). For these reasons, the conclusion that the gospels record
eyewitness testimony, as they claim, cannot be denied.

In these reliable eyewitness documents, Jesus is reported to have appeared physically alive to
His disciples after His crucifixion. This testimony is verified by 1 Corinthians 15:3-8. In this
passage, Paul is recording an early creed concerning the resurrection appearances, which, the
majority of scholars believe, he received from Peter and James within six years of the
crucifixion. Since Peter and James are both mentioned in this creed as having seen Jesus alive
after His death, we may agree with Jewish scholar Pinchas Lapide who says that this creed
"may be considered the statement of eyewitnesses" (Pinchas Lapide, The Resurrection of

Volume 1990 - 2002 Issue


Page 792 of 2049
Skeptical Review Edited by Farrell Till
Jesus, p. 99. For a more in-depth treatment of 1 Cor. 15:3-8, refer to footnote 15, pp. 67-68).
Because the gospels and this creed are the early testimony of eyewitnesses (not to mention
that these eyewitnesses have been shown to be trustworthy), the theory that the resurrection is
a myth or legend can be ruled out. There are only three options: the disciples hallucinated,
lied, or really encountered the bodily risen Christ.

The disciples could not have been hallucinating because this theory is flatly contradicted by
certain psychological principles governing the appearances of hallucinations. Also, the
disciples record touching Jesus and giving Him food (Luke 24:39-43), which cannot be done
with a hallucination. In addition, this theory fails to account for the empty tomb. The next
option is that the apostles were lying. But...

4. Jesus's disciples were transformed into bold witnesses who died for their belief in the
resurrection.

Of the twelve disciples, ten died for their belief in Christ's resurrection and their belief in Him
as the Son of God. This is significant because if Jesus had not risen from the dead, His
disciples knew it. People may die for something they believe to be true, but is in fact false.
But if the resurrection did not happen, the disciples did not just die for a lie which they
mistakenly believed to be true, but died for a lie that they knew to be a lie.

Josh McDowell puts this well: "Jesus's followers could not have faced torture and death
unless they were convinced of His resurrection. The unanimity of their message and course of
conduct was amazing.... If they were deceivers, it's hard to explain why one of them didn't
break down under pressure" (Josh McDowell, More Than a Carpenter, Tyndale House
Publishers, 1977, p. 67). After witnessing events such as Watergate, can we reasonably
suppose that the disciples could have totally covered up such a lie?

5. The existence of the Christian Church.

Christianity requires a historic cause. It did not exist until about A. D. 30, when it suddenly
burst to life, spread like wildfire, and changed the world. What could have started this if not
the resurrection, as the early Christians claimed?

Josh McDowell writes, "The Church was founded on the resurrection, and disproving it would
have destroyed the whole Christian movement. However, instead of any such disproof,
through the 1st century, Christians were threatened, beaten, flogged and killed because of
their faith" (Evidence, p. 218). It would have been much simpler to silence Christianity by
putting forth evidence disproving the resurrection, but this could not be done.

6. The conversion of Paul.

If there was no resurrection, then Paul deceived the other apostles of an appearance of Christ
to him, and they in turn deceived Paul! "Even worse, what could have motivated him to `sell
out' to his former `ministry' of persecuting the Christians when he was convinced that it was
God's will? From his point of view, why would he risk the damnation of his own soul by
converting to what he perceived as anti-Jewish beliefs?" (Gary Habermas & J. P. Moreland,

Volume 1990 - 2002 Issue


Page 793 of 2049
Skeptical Review Edited by Farrell Till
Immortality: The Other Side of Death, Tyndale House Publishers, 1992, p. 58). Paul says that
it was an appearance of the risen Christ that convinced him that Christianity is true.

Based on the evidence, my conclusion is that the Christian faith is a reasonable faith (not a
blind faith), based not on myth or legend but on a solid historical event--the resurrection of
Jesus. What do you think? Would you agree with George Ladd, who said, "The only rational
explanation for these historical facts is that God raised Jesus in bodily form?" (George Ladd, I
Believe in the Resurrection of Jesus, William B. Eerdmans Publishing, 1975, p. 141). How
else would you explain all of these facts? Perhaps before you come to a conclusion, you
should consider one more reason.

Jesus has transformed millions of lives throughout history.

My reasons for believing in Christ's resurrection are not simply based on historical facts, as
important as they are. I believe that Jesus rose from the dead because He lives in me and I
have experienced the abundant life He offers. Millions of others have experienced this, too,
which leads us to the most important question of all: What is the significance of Christ's
resurrection?

First, we can be sure that life does not end at the grave. Second, we can be certain that Jesus is
who He claimed to be -- fully God and fully man. Therefore, Jesus is the only one who can
speak with certainty and final authority on matters of eternal destiny. This verifies Jesus's
claim to be the only way to God and the claim that Christianity is true. Third, there is genuine
hope. Through the risen Jesus, we can enter into a personal relationship with the living God,
have the certainty of eternal life, and experience His abundant life.

Several years ago, I began this relationship with God. I understood that God loved me and
created me to know Him personally. But I was also aware that before a holy and just God I
was morally guilty, i.e., sinful, and deserving of His judgment. I came to understand that a
relationship with God could not be restored unless the penalty for my sin was paid -- eternal
death (Rom. 6:23).

The good news is that, on the cross, Jesus died in our place to pay the death penalty for our
sin (Rom. 5:8). That's what it means to say, "Christ died for us." That is also why He is the
only way to God -- only Jesus has died to provide our forgiveness. If there was any other way,
Jesus wouldn't have died. His resurrection demonstrates that He conquered death and sin.

The Bible says that this relationship with God and eternal life are gifts and therefore cannot be
earned by good moral behavior (Eph. 2:8-9). Like any other gift, I knew that I had to accept it
before it would become mine (John 1:12). So I admitted to God that I was guilty of rebellion
toward Him and made a decision to put my trust in Jesus to forgive me and to give me eternal
life. I have been encouraged by His promises to come into my life (Rev. 3:20), to give me
eternal life (John 5:24), and to make me a new person (2 Cor. 5:17). This gives me hope not
only for the hereafter but also for the here and now.

No one wants to believe in something that isn't true, especially me. The resurrection of Jesus
has given me substantial reason to believe that my faith is not in vain.

Volume 1990 - 2002 Issue


Page 794 of 2049
Skeptical Review Edited by Farrell Till
The evidence is clear: Mohammed's tomb -- occupied. Buddha's tomb -- occupied. Confucius'
tomb -- occupied. Jesus's tomb -- EMPTY. What is your verdict?

(Matthew Perman, 1120 Dancer Hall, Cedar Falls, IA 50614-0166; e-mail


permanm4888@uni.edu)

Put Me Down For Myth


Farrell Till

On April 2, 1996, I attended the Horner-Barker Debate at Northern Iowa University, at which
time I had the opportunity to meet Matthew Perman with whom I had been corresponding by
e-mail for several weeks. Prior to the debate, I spent about four hours with Matthew and two
of his friends talking about various issues related to biblical inerrancy. I found them to be
particularly intense about their Christian beliefs, and it was very evident throughout our
conversations that they had been thoroughly indoctrinated in fundamentalist Christian
ideology. In many ways, Matthew, who was the most vocal of the three, reminded me of how
I was at that age, i.e., young and idealistic and completely entrenched in inerrantist thinking.

At the debate, survey cards were distributed that among other things offered a complimentary
article about the resurrection. I indicated that I would like to receive it, and two days later, the
foregoing article arrived in the mail. When I requested it, I had no idea that it had been written
by none other than Matthew Perman. It was a pleasant surprise.

There is certainly nothing new in the article, which is simply a rehashing of the same
discredited arguments of Josh McDowell, F. F. Bruce, Gary Habermas, and other evangelical
"apologists," so an exhaustive rebuttal of Perman's arguments should assure readers of his
article that the very best evidence Christians can present for the foundation doctrine of their
religion is woefully inadequate to convince rational people that the resurrection of Jesus is a
historical fact.

Close to the end of his article, Perman asked, "How else would you explain ALL of these
facts?" By using the word *facts* rather than *claims,* Perman begged an important
question, because he is asking his readers to assume that certain claims that the New
Testament and Christians make about the resurrection are facts, when in reality they are
merely matters of belief for which there is nothing close to the kind of evidence that would be
necessary to establish them as facts. The death of Jesus, the empty tomb, the postresurrection
appearances, the willingness of the apostles to die for their beliefs--these are all touted as
historical facts when in reality they are nothing more than beliefs that Christians base on an
unjustified assumption that the New Testament documents are historically accurate in
everything they reported.

Volume 1990 - 2002 Issue


Page 795 of 2049
Skeptical Review Edited by Farrell Till
I will have more to say about this later, but first I want to answer Perman's question and show
how all of his "facts" can be explained more rationally than does his assumption that Jesus
literally died and was bodily resurrected to life. This explanation was very ably presented as
Perman, his friends, and I sat together during the Horner-Barker Debate. Dan Barker devoted
most of his first speech to the development of the argument, which Horner largely ignored the
rest of the night. Perhaps Mr. Perman would like to respond to the argument. If so, I will
publish his reply in a later issue.

When they are debating the resurrection issue, inerrantists like to forget the principle of
argumentation that says, "He who asserts must prove." They insist that their opponents have
an obligation to prove that the resurrection did NOT happen or at least to offer a better
hypothesis for the "data" than their claim that the resurrection literally happened. In his first
speech, Barker met this challenge and presented an alternative hypothesis that explains the so-
called "data" much better than the Christian claim that a resurrection literally occurred.

Starting with an analysis of 1 Corinthians 15:3-8, Barker showed that the Christian belief in a
bodily resurrection was a result of doctrinal evolution that had begun with belief in only a
spiritual resurrection. He noted that Paul's statement in this passage is recognized by most
biblical scholars, including evangelical apologists, to be the earliest known statement about
the resurrection, which had probably been transmitted orally, possibly in hymns or poems,
until Paul finally wrote it down in his famous defense of the resurrection. Barker noted that
this earliest account of the resurrection makes no references to many of the elements that are
found in the gospel accounts, which were written much later. There were no references to an
earthquake and empty tomb, to women, to angels, etc. Why did this earliest account of the
resurrection leave out such important events as these if they were so widely known as part of
the resurrection story? It isn't at all unreasonable to assume that they were not mentioned for
the simple reason that they were traditions that had not yet developed when Paul wrote his
first epistle to the Corinthians. This, however, is merely a hypothesis for which reasonable
evidence should be presented, and Barker more than satisfied this requirement of logical
argumentation.

He focused our attention on the words *buried, raised,* and *appeared* in Paul's text and
analyzed each as they were used in the Greek text of the New Testament. The word *thapto*
(bury) meant to inter or bury and carried no necessary connotations of entombment, so this
would be entirely consistent with the known practice of taking the bodies of crucifixion
victims and burying them in a common grave. The word translated *rose* or *raised* in
English translations of this passage was *egeiro,* which meant to "arouse" or "awaken."
Barker noted that this was the word that Paul used in referring to the resurrection in such
passages as 2 Corinthians 5:15 and that it was the word used in Ephesians 5:14, where Paul
said, "Awake (*egeiro*), thou that sleepest and arise (*anistemi*) from the dead." The latter
word, which meant "arise" or "raise up," is the word used in reference to resurrection, but
*egeiro* (awake) is the word that Paul used in 1 Corinthians 15:4, 12 in speaking of Christ's
arising.

*Egeiro* (awake) was used by Paul eleven other times in 1 Corinthians 15: 15-52, as he
spoke about the apostles being false witnesses if the dead are not *raised,* faith being dead if
the dead are not *raised,* and seed and bodies being sown in corruption but *raised* in

Volume 1990 - 2002 Issue


Page 796 of 2049
Skeptical Review Edited by Farrell Till
incorruption, etc. That Paul believed only in a spiritual resurrection of Jesus is very evident
from a careful analysis of this passage:

But someone will say, "How are the dead raised up? And with what body do they come?"
Foolish one, what you sow is not made alive unless it dies. *And what you sow, you do not
sow that body that shall be,* but mere grain--perhaps wheat or some other grain" (vv:35-37).

We have to give Paul an "F" in botany for what he said here, because practically any
elementary science student today would know that nothing can grow from a dead seed. The
important thing to notice, however, is that in analogizing the resurrection of the dead with the
planting of seeds, he clearly said, "*You do not sow that body that shall be.*" That is a view
diametrically opposed to the subsequent doctrine of a bodily resurrection that was based on
the gospel accounts. This doctrine proclaims that the physical body that was "sown" (buried)
was the same body that was resurrected, but in Paul's analogy he said the body that was sown
was not the body that would be.

More evidence that Paul believed only in a spiritual resurrection of Jesus is found farther
along in the text. After declaring that there are different kinds of flesh, he argued that there are
also different kinds of bodies:

There are also celestial bodies and terrestrial bodies; but the glory of the celestial is one, and
the glory of the terrestrial is another. There is one glory of the sun, another glory of the moon,
and another glory of the stars; for one star differs from another star in glory. *So also is the
resurrection of the dead. The body is sown in CORRUPTION, it is raised in
INCORRUPTION. It is sown in dishonor, it is raised in glory. It is sown in weakness, it is
raised in power.* IT IS SOWN A NATURAL BODY, IT IS RAISED A *SPIRITUAL*
BODY. There is a natural body, and there is a spiritual body (vv:40-44, emphasis added).

The statements emphasized in italics and bold print are clear enough to show that Paul
believed that the body that is raised is different from the body that was buried. Since the
statements were made in a context in which Paul was arguing that Christ had been raised, it is
entirely reasonable to conclude that Paul believed that the natural body of Christ was "sown"
when he was buried but that a spiritual body was resurrected.

A favorite expression of Mr. Perman seemed to be "most scholars" as he talked about what
"most scholars" think or believe. In his case, of course, he meant "most fundamentalists,"
because, as we will see, most scholars--and by this, I mean most true scholars and not
evangelical apologists--decidedly do NOT believe many of the things that he attributed to
"most scholars." At any rate, since he brought up the subject of what "most scholars" think, he
can't object if I point out that most scholars--and I mean recognized scholars and not
evangelical preachers whose books are published in Grand Rapids, Michigan--have dated the
gospels anywhere from 15 to 70 years after Paul's first epistle to the Corinthians. By that time,
legend had built the spiritual awakening that Paul taught and preached into a resurrection of
the natural or physical body, which is exactly the kind of resurrection that Paul said had not
happened.

Volume 1990 - 2002 Issue


Page 797 of 2049
Skeptical Review Edited by Farrell Till
This doctrinal transition from a spiritual arising to a bodily resurrection led to the writing of
the gospels, which put the resurrection into a specific historical setting with tales of an empty
tomb and postresurrection appearances in which disciples actually touched the physical body
of the resurrected savior. Perman asked how else ALL of "these facts" could be explained,
and this hypothesis explains it much more sensibly than his irrational belief that all of the
fabulous claims in the resurrection narratives literally happened. More likely, they didn't
happen. They are simply legends that developed around the story of a savior-god who had
arisen spiritually after his death. If Perman wants to dispute this, he can argue with his
beloved apostle Paul, who clearly said, "It is sown a *natural* body; it is raised a *spiritual*
body" (1 Cor. 15:44). Then he can try to explain to us why if Paul said that Jesus had risen
spiritually, the gospel writers later said that he had risen bodily.

Part of Dan Barker's argument in the debate was an analysis of the word "appear" to show that
Paul and other New Testament writers had used it in visionary senses. In Matthew 17:3,
Moses and Elijah "appeared" at the time of the transfiguration, and the Greek word here is the
same one that Paul used in 1 Corinthians 15:3-8 in listing the appearances that Jesus made to
Cephas, to the twelve, to the 500 brethren, to James, and finally to Paul himself. There is
nothing in the text of Matthew that even remotely hints that Moses and Elijah had been bodily
resurrected in their appearances at the transfiguration. In Acts 16:9, "a vision *appeared*
[same word as in 1 Corinthians 15:3-8] to Paul in the night in which a man from Macedonia
stood praying for Paul to come there to help them. Since the same word for "appear" was used
in 1 Corinthians 15:8, where Paul said, "And last of all, as to the child untimely born, he
*appeared* to me also," Barker argued that there is sufficient reason to assume that the other
appearances were like the appearance to Paul. Barker then showed that the only records that
exist of the appearance of Jesus to Paul show clearly that this was just a vision that Paul had
and that he had actually not even seen Jesus in the vision. He heard only a voice speaking
from a bright light (Acts 9:3-8; 22:6-11; 26:12-18), and the men who were with him saw only
the light but didn't hear the voice (according to one of the accounts). So if this was the way
that Paul "saw" Jesus, and since the same word for *see* or *appear* (depending on
translation) was used for all of the appearances that Paul mentioned in 1 Corinthians 15, why
should we believe that Paul considered these "appearances" any more than just the same kind
of visionary appearance that he had experienced on the road to Damascus?

If there is still doubt that Paul had experienced only a visionary appearance of Jesus, we can
let him settle the matter for us. In Acts 26, he related to King Agrippa the story of Christ's
appearance to him, after which he said, "Therefore, King Agrippa, I was not disobedient to
the HEAVENLY VISION" (v:19), so if Luke told the story right, Paul himself said that he
had seen Jesus in a heavenly vision, and as noted above, it was a type of appearance that none
of the other men with Paul were able to see. That doesn't sound very much like a bodily
appearance.

So the "facts" of the New Testament records indicate that Christians first believed in a
spiritual resurrection of Christ and that the claims of bodily appearances came much later in
the gospel narratives. Even then the gospel narratives presented these appearances in a
composite manner. In some ways, these postresurrection appearances of Jesus were physical
in that he had a body that could be touched and examined, but in other ways, it was a spiritual
body that could be teletransported, appear suddenly, and even pass through closed doors. The

Volume 1990 - 2002 Issue


Page 798 of 2049
Skeptical Review Edited by Farrell Till
most rational explanation for these divergent views of the resurrection depicted in the New
Testament is the one that Barker presented in his debate with Michael Horner. Christians first
believed in a spiritually resurrected Jesus, but this doctrine gradually evolved into a belief in a
physically resurrected savior. So if Perman is really sincere in asking if the resurrection of
Christ was myth or reality, he should see that Occam's razor favors those of us who think it
was myth. This will become even more obvious as I take the points in his article and show
that none of them proves anything.

Having satisfied Mr. Perman's request for a better explanation of the "facts," I can now
discuss his arguments individually and show that they prove exactly nothing. His first major
point was that "Jesus Christ died from the rigors of crucifixion and was buried in a tomb."
The only way that anyone can know this is to assume the historical accuracy of the New
Testament records, because there are certainly no contemporary extrabiblical records that
Perman can cite to corroborate the gospel claims, and even if such records did exist to
confirm that Jesus died from crucifixion and was buried, they would prove nothing except
that Jesus died from crucifixion and was buried. Lots of people were crucified in those times,
but I doubt if even Perman would argue that their deaths and burials in any way constituted
proof that they were later resurrected.

Of course, Perman thinks he has found "extrabiblical sources" in the discredited argument that
Thallus, "a non-Christian Samaritan historian," verified the three-hour period of darkness that
the synoptic writers claimed in their gospel accounts. The front-page article in this issue of
*TSR* ("Did Marco Polo Lie?") discusses this alleged nonbiblical reference, so there is no
reason to rehash it. Suffice it to say that Perman will have to find more than dubious
secondhand references to darkness in nonextant writings before he can hope to convince
rational people that any such event actually occurred. Edward Gibbon is a highly respected
historian, and we have already seen his assessment of Matthew's claim that three hours of
darkness covered "all the land" while Jesus was on the cross.

As further "evidence" that Jesus died on the cross, Perman cited the notorious article in the
*Journal of the American Medical Society.* This article, of course, had to assume that the
New Testament documents were accurate in what they reported, because there simply are no
references to the death of Jesus in the nonbiblical records of that time. So if one is going to
assume the absolute accuracy of the New Testament documents, what is there to argue about?
They obviously claim that Jesus died on the cross. However, an honest investigation of an
issue like this would not beg a question that needs to be proven. How can we know that
blatantly biased documents like the gospel accounts were accurate in everything they
reported? Perman would not assume that the Koran is historically inerrant. He would not
assume that the Book of Mormon is historically inerrant. He would not assume that the
Zoroastrian Avesta is historically inerrant. So why does he insist on according the Bible
privileged status? He does so because of a wishful bias that makes him want to believe that
his religion is the "true" one. This bias was evident several times in his article as he labored to
explain why Christianity must be considered "unique" and Jesus the "only way to God." Is
this any way to conduct honest historical research?

As for the medical-journal article itself, I will just ask Perman if he knows of any competent
forensic pathologist who thinks that death can be determined without examining the body, yet

Volume 1990 - 2002 Issue


Page 799 of 2049
Skeptical Review Edited by Farrell Till
this is exactly what the authors of the article in question have presumed to do. Without
examining the body of Jesus, they have concluded that he had to have been dead. The only
"evidence" they could possibly base that conclusion on is what the New Testament says, and
so that brings us right back to square one: they have assumed the accuracy of the New
Testament documents.

We have all read in newspapers stories about people who were pronounced dead by attending
physicians but later were found to be alive when movement was noticed as the bodies were
lying in the morgue. If such as this can happen in modern times after competent physicians
have declared people dead, how can Perman or anyone argue that it is possible for physicians
today to know definitively that a man who was crucified almost 2,000 years ago was dead
when he was taken down from the cross? Of all the arguments for the resurrection that I have
heard, this one ranks high on my list of the silliest.

*Jesus's tomb was empty just a few days later.* Again we have an argument that is based on
the assumption that the New Testament records are historically accurate. Perman can cite no
contemporary nonbiblical records that refer to an empty tomb, and even if he could, they
would not prove that Jesus was resurrected from the dead. An empty tomb could have several
explanations, and probably the last one that would occur to a rational person would be that the
body buried in it had been resurrected.

Perman said, "If what the apostles were preaching had been false, it would have been evident
to the people in Jerusalem and Christianity more than likely would not have begun," but I
must keep reminding readers that Perman always argues from the assumption that the New
Testament records are historically accurate. Except for what the New Testament says, how
does Perman know when the apostles started preaching? For that matter, how does he know
that the apostles were actual historical characters? Secular history is strangely silent about
these apostles who, according to the New Testament, played such prominent roles in founding
Christianity. Perman spoke about how Christianity "suddenly burst to life, spread like
wildfire, and changed the world," but for some reason secular history was silent about the
men who were responsible for this spreading wildfire. Everywhere Paul went, he seemed to
stir up civil and religious unrest. According to Acts 14, he and Barnabas were overwhelmed
by a mob intent on worshiping them as the gods Hermes and Zeus. In Antioch of Pisidia, Paul
preached a stirring sermon in the synagogue, and "the next sabbath almost the whole city was
gathered together to hear the word of God" (Acts 13:14-44). At Philippi, Paul and Silas were
arrested and thrown into prison for having driven a "spirit of divination" from a young slave
girl who was thereby rendered unprofitable to her masters (Acts 16:16-40). In the province of
Asia, Paul was almost lynched by a mob that was angry because his many conversions had
almost destroyed the worship of Diana and Jupiter (Acts 19:22-41). On his return to
Jerusalem, the captain of the temple had to rescue him from an angry mob (Acts 21:27-36).
Paul was arrested and made appearances before Felix and Agrippa and eventually was taken
to Rome, where one tradition says that he was tried and executed, but according to another
early church tradition, Paul was released and went to Spain.

All of these remarkable events surrounded Paul, but for some reason (known perhaps to Mr.
Perman) secular history left no record of his exploits. We find him mentioned only in biased
New Testament and early Christian documents. Secular records don't refer to any of the other

Volume 1990 - 2002 Issue


Page 800 of 2049
Skeptical Review Edited by Farrell Till
apostles either, so what are we to make of this? Could it just possibly be that the spread of
Christianity was not exactly the wildfire that Perman claims it was?

We know only that the New Testament claims that the apostles began to preach the
resurrection on the day of Pentecost following the crucifixion of Jesus (Acts 2). This would
have been 50 days later. Because of the long delay between that time and the writing of the
gospels and the book of Acts, we must recognize the possibility that some accounts of the
apostles' activities were at least exaggerations if not outright legends. However, even if we
assume a high degree of accuracy in the book of Acts, this would mean that Jesus was
crucified 50 days before the apostles began to preach that he had been resurrected. How then
would it have been possible for the Jewish authorities "to waste no time in producing the body
of Jesus from the proper tomb"? After a body has been dead that long, a forensic expert would
have been needed to establish the identity with certitude.

We must also recognize the distinct possibility that the early Christian movement was so
insignificant in its influence that no organized opposition to it even developed at that time.
Mr. Perman likes to think in terms of rapidly spreading wildfire, but he can find no historical
support for this theory. As noted in the front-page article of this issue, the father of Josephus
was a priest in Jerusalem at the very time the book of Acts presented the Jewish leaders as a
hand-wringing opposition anguishedly wondering what it could possibly do to counteract the
influence that the preaching of the apostles was having on the people. If that picture is even
halfway accurate, why wouldn't Josephus have known about it? He made references in his
works to various Messianic groups and rebels whom history has almost completely forgotten,
but he didn't even mention the Christian movement that, according to Perman, was spreading
like "wildfire." How reasonable is it to believe that? It would be as if someone should write a
socio-political history of mid-19th century America and not even mention the abolitionist
movement and its underground railroads.

Furthermore, if Barker's doctrinal-evolution argument is true, then it is likely that a bodily


resurrection of Jesus wasn't even preached until long after the time that he was allegedly
crucified. That being the case, there would have been no "corpse to put on a cart and wheel
through Jerusalem." Since the gospels were written in Greek for Hellenistic readers well after
the time the crucifixion presumably happened, it is likely that the bodily resurrection of Jesus
was first proclaimed in places far removed from Jerusalem. This is why there is no merit at all
to Paul Althaus's claim that the resurrection proclamation "could not have been maintained in
Jerusalem for a single day, for a single hour, if the emptiness of the tomb had not been
established as fact for all concerned." Without assuming the historical accuracy of the New
Testament documents, Althaus cannot even know when the bodily resurrection of Jesus was
first preached or even if it was first preached in Jerusalem.

*Early Jewish testimony admits the empty tomb.* Perman asserted this and tried to prove it
by Matthew's claim that the chief priests bribed the Roman guard into saying that the disciples
had stolen the body of Jesus and that "this story was still being spread at the time when he
[Matthew] was writing." Yes, this is exactly what Matthew reported (28:11-15), but Perman's
appeal to this passage is a flagrant assumption that Matthew's record is historically accurate.
If we assume that the crucifixion and burial really did happen, how does Perman know that
the disciples didn't steal the body and that Matthew fabricated a story about the bribing of the

Volume 1990 - 2002 Issue


Page 801 of 2049
Skeptical Review Edited by Farrell Till
guard as a way of counteracting the truth about what had happened? Perman seems to be an
admirer of John Wenham's *Easter Enigma,* which is a highly speculative attempt to
reconcile inconsistencies in the resurrection narratives, and in this book Wenham admits that
Matthew's story of the Roman guard "bristles with improbabilities at every point" (*Easter
Enigma,* Academie Books, 1984, p. 79). Wenham cited the guards' reporting to the chief
priests and their accepting a bribe to tell their officers that the body had been stolen while
they had fallen asleep on duty as major improbabilities in the story.

So did Wenham find the improbabilities too hard to swallow? Certainly not, because the aim
of his book was to defend the accuracy of the resurrection narratives despite their "apparent"
inconsistencies. "It is a worthless piece of Christian apologetic at whatever date it was
written," Wenham concluded, "*unless it happens to be undeniably true*" (Ibid., original
emphasis). So there you have it. "It is so absurd that it just has to be true," Wenham was
arguing. Well, I hope he and Matthew Perman will forgive me for saying that it is so absurd I
cannot believe it, and that is a much more rational reaction to an implausible story like this..

*Did the disciples steal the body?* "If so," Perman argued, "then the men who delivered to
the world the highest moral standards it has ever known were frauds, liars, and hypocrites. Is
this credible to believe?" I'm sorry, but I can't share Perman's enthusiasm for Christian
morality. I see things in Christianity that I would consider being far from "the highest moral
standards" the world has ever known. Be that as it may, let's consider the merits of his claim.
Mormonism requires an extremely high moral code of its members. They can't use tobacco or
drink beverages that contain alcohol or caffeine, and their "clean" lifestyle probably explains
the increased longevity that they enjoy. Do the high moral standards of this religious group in
any way prove that Joseph Smith's claims of special revelations from God have to be true? If
not, then are we to believe that a "fraud, liar, and hypocrite" gave to the world an extremely
high moral standard? Or should we just recognize the complexity of human nature that results
in almost all people having both their good and bad qualities? Why should we think that the
early leaders of Christianity would have been any different? I would say that the burden is on
Matthew Perman to prove that it just isn't possible for "frauds, liars, and hypocrites" to teach
high moral standards. Indeed, we have witnessed many times "frauds, liars, and hypocrites,"
who preach high standards of morality from the pulpit but are later caught in acts that show
them to be frauds, liars, and hypocrites. Does anyone really consider this a convincing
argument?

*The disciples had real experiences with one who they believed was the risen Christ.* Again,
I must point out that this is an argument that is based on an assumption that the New
Testament is historically accurate in everything it says. These "real experiences" that Perman
referred to were the appearances that Jesus allegedly made to his disciples after his
resurrection. Perman claimed that the gospel accounts of these appearances are reliable,
because they "claim to have been written by eyewitnesses to the life of Jesus and by those
who recorded eyewitness testimony." Oh, is that so? Just where did the writer of Matthew
claim that he had been an eyewitness of the things that he wrote about? Where did the writer
of Mark claim that he had been an eyewitness? The author of Mark didn't identify himself,
and only tradition says that it was written by John Mark. If this tradition is true, then he wasn't
an eyewitness, so everything that he said has to be recognized as hearsay testimony. John
Singleton Copley may think that he knows "pretty well what evidence is," but if his "legal

Volume 1990 - 2002 Issue


Page 802 of 2049
Skeptical Review Edited by Farrell Till
mind" was half as great as Perman claimed it was, then he surely must have known that
hearsay testimony is not even permitted in most courtroom situations.

If Perman wants to talk about what "critical scholars" believe, why doesn't he consider that
formidable body of critical works that reject the notion that the gospels were written by the
individuals whose names have been associated with them? If, for example, the apostle
Matthew really wrote the book that bears his name, then why did he feel the need to copy
90% of the gospel of Mark, which was written by someone who wasn't even an eyewitness?
Does Perman have a logical explanation for why someone who was an eyewitness to the life
and deeds of Jesus felt the need to copy much of his gospel from the work of someone who
wasn't an eyewitness? These are matters that Mr. Perman needs to give serious consideration
to.

Mary Magdalene, Mary the mother of James, Salome, Joanna, Cleopas, the "twelve," Cephas,
James, the 500 brethren--none of these left any firsthand testimony of their alleged
experiences with the risen Christ. All of their "testimony" has been filtered to us through
second- and thirdhand sources, and Paul admitted that he had experienced Jesus only in a
vision in which he saw a bright light and heard a voice. That is hardly solid evidence
regardless of what John Singleton Copley may think.

*The disciples of Jesus were transformed into bold witnesses who died for their belief in the
resurrection.* Says who? Well, of course, the New Testament indicates that the disciples were
transformed into "bold witnesses," but James was the only apostle whose death was
mentioned in the New Testament (Acts 12:1-2). As I noted earlier, stories about the
martyrdom of the apostles "for what they believed" are largely legendary and highly
inconsistent. So now we have Perman arguing from the assumption that both the New
Testament records and church traditions are historically accurate.

For the sake of argument, let's just assume that the traditions are true and that the apostles all
died horrible deaths as martyrs for what they believed. What would this prove? If martyrdom
proves the truth of what a martyr dies for, then practically every religion on earth can lay
claim to being the only "true" religion. Martyrdom is as old as religion itself, so the logical
axiom that says, "What proves too much proves nothing at all," shows that there is no merit at
all to the argument that the martyrdom of the apostles and earlier disciples proves the truth of
what they believed.

Perman made other points that I would like to comment on, but space will not allow it.
Perhaps I can address them in a future article. Meanwhile, if he wants to respond to this
article, we will publish it.

Daniel in the Historians' Den


by William Sierichs, Jr.

Volume 1990 - 2002 Issue


Page 803 of 2049
Skeptical Review Edited by Farrell Till
The Book of Daniel is one of those "Oops" items in the Bible. It's partly a historical novel and
partly an Arabian Nights fairy tale, with a disembodied hand writing on a wall (perhaps the
prophet could see The Addams Family before it was made), a man surviving unscathed in a
den of starving lions, and three men left unsinged after a trip through a blazing furnace.

The history portions center around King "Nebuchadnezzar" of Babylon, who captures King
Jehoiakim of Judah and carries off some Israelites. Nebuchadnezzar becomes a patron to
Daniel, presented in the story as a prophet.

Nebuchadnezzar later undergoes a period of insanity before his death. He is succeeded by a


"son," Belshazzar, presented as the last king of Babylon. The "thing" [disembodied hand]
appears writing on the wall to warn Belshazzar of doom. Daniel 5:30 says Belshazzar died
that night when "Darius the Mede" captured Babylon.

Daniel may have survived meat-hungry lions, but fact-hungry historians have ripped him into
shreds. Very little is left of him. Archaeologists have pieced together the genuine history of
Babylon from records recovered in the last century and a half. The following account is drawn
from Babylon by Joan Oates; The Bible as History by Werner Keller; The History of Ancient
Israel by Michael Grant; articles on Daniel, Babylon, Belshazzar, and Nebuchadrezzar in The
Oxford Companion to the Bible; and chapters 23-25 of The House of Seleucus by Edwyn
Bevan. Nebuchadrezzar -- the historians' spelling -- reigned from 604 to 562 B.C.E. (Before
the Common Era). He was succeeded by his son, Amel-Marduk, who ruled one year, then
Neriglissar, who ruled three years. Labashi-Marduk ruled next, for less than a year.

The last king of Babylon was Nabu-na'id, commonly called Nabonidus. He ruled from 555 to
539 B.C.E., when Babylon was captured by the Persians under Cyrus the Great -- not Darius
and the Medes. Nabonidus fled, although he apparently was later captured and killed in
Babylon.

Nabonidus had a son -- Belshazzar (or Bel-shar-usur in Babylonian)--who apparently ruled


for a decade as crown prince while Nabonidus was in Arabia. What Nabonidus did in Arabia
is unknown, but the story of "Nebuchadnezzar's" insanity may be a reference to a bout of
insanity or lengthy depression in Nabonidus, who apparently was very unpopular in Babylon -
- or so the victorious Persians later claimed.

But Nabonidus returned to Babylon before its capture by Cyrus, so Belshazzar was not the
ruler as the Book of Daniel claims, and he was never king.

Scholarship points to the reason for Daniel's bloopers. It was written during the period of the
Maccabees, in the middle of the 2nd century B.C.E., or about 400 years after the events it
describes. Its origin is betrayed in chapter 11, when Daniel supposedly prophesies about the
future. He refers in verse 3 to a "mighty king," identifiable as Alexander the Great. The "king
of the south" in verse 5 is the Ptolemaic dynasty of Egypt, while the stronger prince -- "king
of the north" -- is the Seleucid dynasty of the Middle East. Both the Ptolemaic and Seleucid
kings (the "Syrians" of the Book of Maccabees) were descended from two of Alexander's
officers.

Volume 1990 - 2002 Issue


Page 804 of 2049
Skeptical Review Edited by Farrell Till
Daniel's description of the 2nd century struggle of these two dynasties is historically useful,
much more so than the account of the 6th century dynasty of Babylon, when Daniel
supposedly lived. Daniel 11:21 onward is a partial description of the Seleucid King Antiochus
Epiphanes' actions which precipitated the Maccabean War.

The Seleucid Empire was under considerable pressure in 167 B.C.E. from the expanding
Roman empire, which had inflicted major defeats on the Seleucids; Ptolemaic Egypt -- which
formerly had ruled Judea and whose rulers wanted it back; and the Parthians, who were taking
Iran from the Seleucids. Thus, international events as well as internal Judean politics played a
role in the Maccabean War. The Book of Daniel reflects events in that war, beginning in 167
B.C.E. with a rebellion against a pro-Seleucid Jewish group that ruled Judea and ending in
164 B.C.E., when a native Maccabean army captured Jerusalem.

Daniel can't get Babylonian history straight, but he does pretty well by the Hellenistic era.
Obviously, whoever wrote the book was a very solid citizen of the 2nd century B.C.E., whose
"prophecies" were wholly retroactive.

To err is human, of course; not to make mistakes would be divine. Daniel's blunders betray a
very human origin. Hardly a case of divine inspiration, as the literalists proclaim. An
intriguing footnote: In Matthew 24:15 Jesus calls Daniel a "prophet." It's funny that Jesus --
purportedly one-third of an omniscient deity -- should make such a mistake. Maybe Jesus was
the Uncle Fester of the Trinity; god didn't figure common humanity needed a visit by a deity
with a good memory. Or perhaps it's just that to err is human, indeed.

(William Sierichs, 316 Apartment Court Drive, Apt 44, Baton Rouge, LA 70806.)

EDITOR'S NOTE: In addition to the works that Sierichs cited, The Interpreter's Bible is an
excellent reference. Its analysis of the 11th chapter of Daniel will leave little doubt that
Sierichs was correct in identifying the king of the north as the Seleucid dynasty and the king
of the south as the Ptolemaic dynasty. It also provides other analyses that show the book was
a 2nd-century B. C. forgery intended to leave the impression that a 6th-century B. C. prophet
had foreseen the Maccabean uprising and had predicted victory for the Jews.

Cretinism or Evilution?
Ed Babinski publishes a biannual journal entitled Cretinism or Evilution. It would be an
excellent source of information for anyone interested in debunking the major claims of
creationists. The latest issue (Winter/Spring 1996) discusses many of the tall tales about
archaeological discoveries, such as human and dinosaur footprints side by side, steel tools
embedded in solid coal, and fossils of human giants, which creationists like Kent Hovind
continue to use in their literature, lectures, and debates despite their having been discredited
many times. This issue contains copies of letters that Babinski sent to museums and

Volume 1990 - 2002 Issue


Page 805 of 2049
Skeptical Review Edited by Farrell Till
paleontologists around the world to see if there is any truth to these claims, and the answers to
the letters (indicating that the claims have never been verified) are also published. Babinski
analyzed several quotations that creationists have used to leave the impression that leading
scientists are abandoning their acceptance of evolution. These analyses show very clearly that
creationists quote these authorities out of context and distort what they really meant.

The annual subscription cost to this publication is $15. For additional information, contact Ed
Babinski, 109 Burwood Drive, Simpsonville, SC 29681, e-mail Ed.Babinski@furman.edu.

[Secular Web Editor's Note: As of June 1998, Mr. Babinski is no longer selling subscriptions
to Cretinism or Evilution but is offering it free over the Internet.]

More Creationist Materials

Other materials on creationism can be obtained from Dave Matson (address on page 10). He
has published a particularly good refutation of Kent Hovind's major arguments. The cost is
$14.50.

The Omitted Section


by Marion R. Fox
In my former article I demonstrated one error committed by Farrell Till in his debate with
Jerry Moffitt. In this article I will set forth two additional errors that Mr. Till committed in his
debate with Jerry Moffitt and will respond to his article in Volume 6 # 3 Skeptical Review.

Farrell Till's First Error

Farrell Till made a serious error in his misunderstanding of mathematical probability. Mr. Till
failed to recognize that there are two kinds of probability. First, there is the probability of
unrelated events. Second, there is the probability of related events. In order to illustrate the
difference between these two types of probability we will assume that we have six balls with
the numbers 1 through 6 on them. These six balls are inserted into a hat and drawn out at
random. The probability of an event occurring is p = s/(s + f) where p is the probability of the
event, s is the number of ways the event can successfully occur, and f is the number of ways
the event can fail to occur. The probability of pulling out the number three ball on the first
attempt is p = 1(1 + 5) = 1/6 (one chance in six).

If the first ball drawn is not a three ball and it is placed back into the hat the probability of
drawing a three ball on the second draw is p = 1/(1 + 5) = 1/6 (one chance in six again). If
this is repeated until it has been attempted six times the probability of drawing the three ball
on the sixth attempt is 1/6 (the same as on the first attempt). No matter how many times this

Volume 1990 - 2002 Issue


Page 806 of 2049
Skeptical Review Edited by Farrell Till
procedure is repeated the probability will always be 1/6 (one chance in six). This illustrates
the probability of unrelated events (the second event is unrelated to the outcome of the first
event).

If the first ball drawn is not the number three ball and it is not placed back into the hat the
probability of pulling out the number three ball on the second attempt is p = 1/(1 + 4) = 1/5
(one chance in five). If the number three ball is not drawn on the second attempt and the ball
drawn is not put back into the hat the probability of drawing the number 3 ball on the third
attempt is p = 1/(1 + 3) = 1/4 (one chance in four). If the process is repeated until five balls
have been removed from the hat without drawing the number 3 ball the probability of drawing
the number 3 ball on the sixth attempt is p = 1/1 = 1 ((one chance in one or a 100% chance of
drawing the ball). This illustrates the probability of related events (the second event is related
to the outcome of the first event).

Farrell Till's Second Error

Farrell Till made the same argument that was long ago refuted when he argued for
pseudogenes being useless. This argument is essentially the same as the vestigial organ
argument which has been proven to be unsound. For a number of years atheists argued that
the tonsils, appendix, thyroid, etc. were vestigial organs. The presence of vestigial organs was
used as an argument to prove that organic evolution occurred. All biologists know that these
"vestigial organs" have been proven to be fully functional in the last few years. Physicians
once, routinely, removed these organs when they performed surgery.

This argument is based upon the fallacy of "denying the antecedent" which was discussed in
my first article. Mr. Till's argument is:

First Premise: If Christians can give a reason for the existence of pseudo-genes then
pseudo-genes have a purpose (are not vestigial).

Second Premise: Christians cannot give a reason for the existence of pseudo-genes.

Conclusion: Pseudo-genes have no purpose (they are vestigial).

(Marion R. Fox, 4004 Twisted Trail Road SE, Oklahoma City, OK 73150-1910.)

So What's the Point?


by Farrell Till

Volume 1990 - 2002 Issue


Page 807 of 2049
Skeptical Review Edited by Farrell Till
In a letter published in the May/June "Mailbag" column, Marion Fox complained about the
deletion of some paragraphs from his article that appeared in the March/April issue. Those
paragraphs have now been published. I deleted them from his original article for three
reasons: (1) the article was supposed to reply to what I had said in response to his attempt to
show that the exclusion of bastards from the assembly of Yahweh (Deut. 23:2) would not
have necessitated banning David and the sons of Aaron from the assembly, (2) the paragraphs
I deleted related not to the topic under discussion but to an issue involving my debate with
Jerry Moffitt, and (3) Fox failed to give sufficient background for his comments about this
debate to be understood by those who have not seen the debate tapes.

My decision to omit the paragraphs was based primarily on the third reason just listed. Now
that the part I deleted has been published, I'm sure readers will agree that Fox made an
assertion that no one could possibly understand without first viewing the debate tapes. Exactly
what did I say in my debate with Jerry Moffitt that would indicate that I "made a serious error
in [my] understanding of mathematical probability"? Fox didn't say. He didn't even hint at
what this serious error was. He simply asserted that I had made such an error and then
launched into an explanation of the difference in the probabilities of related and unrelated
events. Even though I was a participant in the debate, I don't recall anything I said that would
lead Fox to believe that I don't understand the difference in the probabilities of related and
unrelated events, so I certainly don't expect readers who have never seen the debate to
understand what he means. If he thought that I made a "serious error," he should have
explained exactly what I said.

I do remember stating that his and Moffitt's attempts to calculate the probability of life
beginning on earth were foolish, and I still maintain that position. Creationists are fond of
pointing out that scientists do not know how to make life, but at the same time, they like to
talk about the "probability" of life beginning without intelligent creation. Apparently, they
can't see the incompatibility of the two positions. If humans can't "make" life, it is because we
don't yet understand all of the factors that are necessary to have life, so if all of the factors
necessary to life are unknown, how can anyone calculate the probability of life beginning by
natural means? Mr. Fox was able to explain the chances of drawing the number three ball out
of a hat precisely because he knew all of the factors involved. He knew that there were six
balls in the hat, but could he calculate the probability of drawing the number three ball if he
didn't know how many balls were in the hat? I think he is intelligent enough to know that he
couldn't, yet he seems to think that creationists can calculate the probability of life forming
naturally on the earth when neither he nor anyone else knows how many factors would be
involved in the formation of life. So I wonder just who doesn't understand "mathematical
probability."

On the matter of vestigial organs, Fox seems to be arguing that because scientists were wrong
in postulating that tonsils and appendices were vestigial organs, the whole concept of vestigial
organs is incorrect, so pseudogenes must have a purpose. In other words, wrong in one thing,
wrong in everything seems to be his argument. Is this the fellow who is trying to lecture us on
the logical errors in atheistic thinking?

Scientists have long known that whales have vestiges of pelvic girdles and hindlimbs that
were interpreted to mean that modern whales are descendants of land animals that once

Volume 1990 - 2002 Issue


Page 808 of 2049
Skeptical Review Edited by Farrell Till
moved about on legs (Michael J. Novacek, "Whales Leave the Beach," Nature, April 28,
1994, p. 807). This assumption has since been verified by the discovery in Pakistan of an
amphibious ancestor of whales that lived 52 million years ago. The fossil shows clearly
defined front and hindlimbs that probably enabled it to leave the water and move about on
land, awkwardly undoubtedly, because the body was more adapted to swimming (Ibid.; Philip
D. Gingerich et al, "New Whale from the Eocene of Pakistan and the Origin of Cetacean
Swimming," Nature, April 28, 1994, p. 844). A subsequent discovery of whale fossils in
Egypt has shown that descendants of the amphibious "whales" possessed hindlimbs but no
front limbs, and even today some whales are born with external vestigial hindlimbs. I'm sure
Mr. Fox wishes that he could cite even a fraction as much evidence from the fossil record to
support his creationist view.

It isn't the purpose of The Skeptical Review to discuss evolution, but since Mr. Fox's
complaint about the omission of paragraphs from his earlier article brought up the subject, I
will simply suggest that he put aside the materials that fundamentalist preachers have
published on the subject of evolution and try to do some reading in scientific journals. He
might be surprised at what he will learn.

Marion Fox's Logic


by Dave Matson
Marion Fox, in arguing the number of generations between Perez and David ( TSR, 7:2, p.7),
makes a fatal mistake which is so very common in Bible apologetics. He assumes that one can
pick and choose among the various meanings that a word may have. Thus, he selects a
meaning for son that does not force a contradiction.

This defense is a branch of that great, central crutch of apologetics, which states that if the
skeptic can't disprove every single loophole then the skeptic has no proof of error in the Bible.
In this instance, Fox is daring the skeptic to prove that son could not be used in the other
sense.

Let us briefly examine the central crutch of apologetics before continuing with Fox.

In the real world of atoms and energy, one can never rule out every loophole no matter how
good the evidence or the theory. (Believe it or not, there are logical loopholes to the claim that
the earth exists!) This follows directly from the nature of inductive logic, which is central to
the acquisition of knowledge in the real world. Consequently, loopholes can always be found;
the better the theory, the more exotic (less believable) the loopholes. In other words, pointing
out the existence of a loophole does not rule out the case for error in any meaningful sense.

It is ludicrous, or at least contrary to good sense, to take the position that no idea about the
real world can be proven wrong. Every day we make judgments based on things being right or

Volume 1990 - 2002 Issue


Page 809 of 2049
Skeptical Review Edited by Farrell Till
wrong. Thus, the mere citing of a loophole as an adequate means of defense must be dropped.
Either that, or we must give up the idea of ever proving that an error exists anywhere. Such is
the error, briefly put, of that great, central crutch of Bible apologetics.

What must be done, in order to get a fair and meaningful answer, is to weight the case for
error against the case for no error -- and to go with the better argument. If the better argument
clearly points to error, then the objective mind will draw that conclusion. Objective minds
will remain open to new data, of course, since no case for error can be 100% certain.
However, in all but borderline cases, the proper and meaningful conclusion must be "error,"
and not the biased view that time and further knowledge will eventually remove that
conclusion of error. More likely, time and further knowledge will increase our confidence in
that conclusion of error, even as it has increased our confidence in the sphericity of the earth.

The concept of error above is none other than the everyday meaning stripped of its mistaken
views of certainty. When we talk about error in the real world, no matter how obvious, it
always has an element of uncertainty about it. A sufficiently imaginative mind would always
find loopholes, but that does not stop us from calling an obvious error an error. We call the
flat-earth idea an "error." We don't call it a discrepancy that might be resolved in the course of
time! In principle, such a description might not be wrong, but without supporting evidence it
is worthless speculation. We simply call the flat-earth view an error, and rightly so. "Error"
means no more and no less in our world of atoms and energy. (In the abstract world of
mathematics, things are a little different.)

Consequently, when we find the case for an error in the Bible to be overwhelming, according
to the above view of error, we have found an error as real as any of its counterparts in the
world about us. We rightfully call it an "error." That is what we mean by "error." The
everyday concept of error includes loopholes, provided that they are so small as to present no
serious challenge to the case for error.

In the case of Fox, he has adopted an inferior solution with the idea that it might be right.
"Prove me wrong," he would probably say. He has produced a loophole and is under the
illusion that it defends against the claim of error. As we have seen, a loophole is not an
adequate defense against error -- unless it is more believable than the case for error. So, that
brings us to the central question. Which meaning of son is the more believable in the context
of the detailed genealogies?

The most common meaning, by far, of the word son is in the literal sense. That must be our
starting point in keeping with the principles of good interpretation. The meaning with the best
track record is most likely, all things being equal, to be right. We then look for evidence of
alternate meanings in the surrounding context. We overthrow the common meaning only if
the evidence is such that using the common meaning would constitute a clear error.

Fox has produced evidence for his alternate meaning in a genealogy that has been obviously
abbreviated. Jesus was clearly not a literal son of David. However, in the Perez to David
genealogy, there is no clear evidence for intended abbreviation. It is given as though it were a
factual genealogy, and that is how it must be taken. Fox's argument is irrelevant.

Volume 1990 - 2002 Issue


Page 810 of 2049
Skeptical Review Edited by Farrell Till
Obviously, there is nothing in the surrounding context that challenges the normal meaning of
son. The main account, given in 1 Chronicles 2:4-15, is beautifully consistent with the
common meaning of son. That, and some additional points by Till, make it clear that there is
only one intelligent interpretation of son in this context, and that is the literal, common
meaning.

Therefore, if God is the author of the Bible, he has contradicted himself by allowing David to
be a part of his congregation. No bastard was to be admitted, even through the 10th
generation.

(Dave Matson, P. O. Box 61274, Pasadena, CA 91116, e-mail


103514.3640@compuserve.com)

EDITOR's NOTE: Dave Matson and I have both written extensively on the how-it-could-been
hermeneutics that inerrantists use to "harmonize" the Bible. In the May/June 1996 issue of
TSR, we published "How-It-Could-Have-Been Scenarios" (p. 1) and "More How-It-Could-
Have-Been Speculations" (pp. 5-6). In the Spring 1993 issue, my article "Any Loophole Will
Do" (pp. 8-9) and Matson's "What Is Wrong with Any-Loophole-Will-Do Hermeneutics" (pp.
9-11) were published. All of these articles have pointed out the fallacies in arguing that the
mere postulation of possible, even if unlikely, scenarios resolves the problem of biblical
discrepancies. Although TSR offers space to inerrantists who want to refute our arguments or
present their own case for inerrancy, no one has tried to respond to Matson's and my argument
that a textual inconsistency or discrepancy resulting from the face-value meaning of language
remains an inconsistency or discrepancy until inerrantists offer plausible evidence that a
hypothesis based on a how-it-could-have-been scenario is not just theoretically possible but
very likely true. We invite them to confront this issue.

ASCII Copies

In addition to all TSR articles being accessible at


http://www.infidels.org/library/magazines/tsr/, we can copy them to your disks in ASCII files
for your personal use. To receive yours, send three (3) formatted DOS disks and $1 for
postage to P. O. Box 717, Canton, IL 61520-0717.

From the Mailbag


Thank you for your comments on behalf of my article after Mrs. Doubravsky's criticisms.
Please allow me to complement your response.

Normally, name-calling and invective in analyses are in bad taste, at best, and often
counterproductive. Occasionally, however, a situation calls for a stronger response than
simple, cold analysis. The main problems of the biblical numbers for Israel's armies cross that

Volume 1990 - 2002 Issue


Page 811 of 2049
Skeptical Review Edited by Farrell Till
line. They remind me of a saying: "Don't urinate on my shoes and try to tell me it's raining."
The Bible's figures are such obvious lies that anyone who insists they are accurate must have
such utter contempt for me as to think I'm a complete idiot.

The reasons are simple: Israel was not big enough to provide anywhere near that many troops.
If it somehow raised that number, its economy could not have supported them in the field, and
such an army could not have been supplied by any means available to any society in the pre-
mechanized era.

Even the Roman Empire, at the height of its size and power, fielded at most 200,000 troops
for regular duty. This number increased during the civil wars but was reduced as soon as the
wars ended because of the strain on the empire's economy. Similar situations can be found in
the Persian, Seleucid, and other great empires of that era. Rarely do we find mention of even
100,000 troops in an army.

When historians confront a figure of one million troops -- such as Herodotus' account of the
Persian army that invaded Greece in 480 B.C.E. -- they scoff for the reasons cited two
paragraphs above. The Persian empire possibly had one million men of military age within its
borders, but it could never have called more than a fraction of them to arms at any one time
and probably could not have supported 100,000 troops in one army for more than a very short
period. Read Donald W. Engels' study Alexander the Great and the Logistics of the
Macedonian Army for a thorough review of ancient military supply systems.

I know that some people will argue that not everyone has the time or resources to research
historical material. But if you're going to insist upon the accuracy of historically verifiable
claims in the Bible, you have an obligation to make at least a minimum effort to check the
facts. This is not like an analysis of the Bible's miracle claims, which in theory at least are
extraordinary events beyond ordinary methods of verification. I cannot prove that Jesus did
not walk on water or raise the dead. Of course, the burden of proof remains upon the
claimants, and extraordinary claims require extraordinary proofs. I am not required to accept
such claims without the appropriate level of proof.

If someone wrote an article claiming that the ancient Romans won their empire because they
had tanks and bombers, I am free to call that person a liar. Historians have ample material
available showing that the Romans did not have the level of technology or industry to produce
tanks and aircraft, no remains of such equipment have ever been found in Roman sites, and
the surviving literature and artwork show nothing remotely like tanks or aircraft. Such things
simply did not exist in the Roman world, and it would be an insult to my intelligence to try to
convince me they did.

The musters are flat out impossible. The wildly fluctuating, contradictory numbers over time,
as listed in the Hebrew scriptures, simply add to the evidence that these figures are pure
propaganda or the exaggerations of oral legends. They're blatant falsehoods.

So my response to anyone who tries to defend these figures as correct is, "My shoes are wet;
the sky is dry. What you've done is very dirty and disgusting, and don't lie to me about your
action. I'm not that stupid."

Volume 1990 - 2002 Issue


Page 812 of 2049
Skeptical Review Edited by Farrell Till
(William Sierichs, 316 Apartment Court Drive, Apt 44, Baton Rouge, LA 70806.)

EDITOR'S NOTE: A frequent contributor to TSR, William Sierichs has another article on
page 8 of this issue. If Mrs. Doubravsky would like to respond to this letter, we will publish it.

I just wanted to thank you for the prompt reply to my inquiry regarding The Skeptical Review.
I devoured the first two issues of 1996, so now I need more freethought brain food. Enclosed,
you'll find a check for the 25 back issues and the booklet Prophecies: Imaginary and
Unfulfilled.

Keep up the good work. We need people like you on the front lines to battle that mental
disease of irrational, illogical, relativistic thinking that passes for religious faith, which seems
to be spreading like a cancer across our nation. I am a former "born-again, evangelical
Christian." Now I guess I'm an agnostic (I can't as yet rule out the possibility of a supreme
being), and I am against all organized religion because of its horrible history. I pray to
whatever god there may be that the U. S. never becomes a "Christian Republic." The two
examples of history's theocracies are the Old Testament's Israel and the dark ages' "Holy
Roman Empire." Both were inhumane and intolerant, causing untold suffering of multiple
millions of people, despite the prevalence of "the word of God" and its teachings (in places)
on love, mercy, forgiveness, etc.

I hope that the internet will be a tool for spreading the "good news" of humans having the
ability to be autonomous, rational, moral, ethical, good people based upon a philosophy of
empathy, social response, ability and accountability to our fellow man. I believe in personal
freedom to believe anything one wants, but it should be kept private and discussed only when
invited to. I feel that militant proselytizing of any religion (or political cause) is offensive.

(Jeff A. Schmura, 808 Apple Lane, Shoemakersville, PA 19555.)

I cannot speak for others, but I am one Christian who looks forward to receiving each issue of
The Skeptical Review. Obviously, I disagree with much of your material, but I respect your
scholarship. I find TSR to be very interesting, thought provoking, and challenging.

In your article, "The Historicity of Jesus" ( Autumn 1995, p. 3), you said this about the 3
hours of darkness during the crucifixion of Jesus: "The fact that no such records exist is
reason to believe that this midday darkness was simply another part of the legends and myths
that evolved as Christianity grew." But in his book The Day Christ Died, Jim Bishop said
there were historical records of this darkness. Enclosed are photo copies of the title page and
the page where he has an interesting footnote on the darkness. Please respond to Bishop's
information in a future issue of TSR. Thank you.

(Paragraph ordering debate tapes deleted.)

Volume 1990 - 2002 Issue


Page 813 of 2049
Skeptical Review Edited by Farrell Till
I am a former member of the Church of Christ. I received "the right foot of fellowship" (my
term) because of doctrinal differences, such as, a nontrinitarian view of the godhead and
conditional immortality. (There are some Christian Freethinkers.)

(Don Robertson, 644 Walnut Street, Rock Hill, SC 29730.)

EDITOR'S NOTE: The pages from Jim Bishop's book that accompanied Mr. Robertson's
letter contained the same references to secular historians that Josh McDowell included in
Evidence That Demands a Verdict, so the front-page article of this issue is an adequate
response to Bishop's claim that secular history contains references to the alleged midday
darkness when Jesus was crucified.

I appreciate Mr. Robertson's interest in TSR and hope that he continues to read it. As a former
Church-of-Christ preacher, I can certainly understand why the doctrinal beliefs that Mr.
Robertson mentioned in his letter would have earned him "the right foot of fellowship" from
the Church of Christ. (See Galatians 2:9 for an explanation of the pun.)

Your article "The Editor Goes to Church" is terrific. I have always had those exact thoughts
about the ridiculous substitutionary explanation of Jesus's death on the cross. You should
know that there are others, even actual ministers, who feel the way you do about this. I'm sure
you have read Bishop John Shelby Spong's Resurrection: Myth or Reality? If not, you should.
His Rescuing the Bible from Fundamentalism is also highly recommended. I myself do not
believe in a literal interpretation of the stories of Jesus's virgin birth or resurrection, or many
of the other miraculous tales. Do you think at all that these stories could have been myths, in
the tradition of the Greeks, designed to draw the world into the revolutionary message of a
man called Jesus? I have come to view these gospels in the light of such mythology and come
away with much more applicable meanings: that his virgin birth most likely symbolizes that
his message was unique, that his resurrection most likely symbolizes that his message rose
after his death, that his walking on the water most likely symbolizes that he is the master of
spiritual knowledge, that his healing of the blind man is most likely symbolic of his teaching
that opens the eyes to important truths (what John called the Logos) of Judaism.

We've been blessed, if I can use this term, in recent years with new translations of the Bible,
translations that will ultimately transform Christian doctrine. We all know by heart the
traditional King James reading of John 3:16: "For God so loved the world that he gave his
only begotten son, that whosoever believeth in him should not perish, but have everlasting
life." But read the Jesus Seminar translation of this famously out-of-context verse (the
"Scholar's Version" found in the book The Complete Gospels) and you get a whole new, more
believable (even to agnostics) sense of the text. Jesus is teaching about God's love for the
world -- that is, everyone -- equating that love to that of a father and an only son. He's not at
all referring to himself: "This is how God loved the world: God gave up an only son, so that
everyone who believes in him will not be lost but have real life." [It's] not a belief in the
figurative son, but in the literal God, of whom we are all sons and daughters.

Volume 1990 - 2002 Issue


Page 814 of 2049
Skeptical Review Edited by Farrell Till
In this same book, it is easier to understand the first chapter of John. The KJV gives the
impression that the writer of the fourth gospel believes that Jesus the man or spirit was
preeminent, existing since the dawn of time, indeed, creating time and the world and universe.
Now, with the help of the Scholar's Version, I understand that this writer is saying that Jesus's
teaching -- his divine truth, wisdom and understanding (the Logos) -- had always been alive
in the Jewish tradition, but that the man known as Jesus brought the teaching to life in a
unique way, in a way these young followers of Jesus had never seen before.

The mystic view of Jesus is that he was a man of conviction who embodied the spirit of truth,
threatened the existing Roman empire by condemning the tax on atonement, and was
crucified. It would be preposterous to think he came into the world any other way than what is
normal. A virgin birth [What is a virgin birth anyway? Did God have sex with this teenage
girl named Mary? How silly!] is not really implied in the scriptures. Mary was pregnant,
Joseph didn't know how she got that way, but had a dream that told him to marry her, and that
the boy was a gift from God. People have such dreams all the time.

It would be preposterous to think Jesus transcended death. Even though these stories are in the
gospels, the details of the resurrection differ greatly from gospel to gospel. [It's] better to look
at them as allegory. His message transcended death.

But it would be equally preposterous to think his life is to be ignored. The stories of Jesus's
life and his message brought the soul of Judaism (the Logos) to pagans, the Greeks and
Romans, and--mixed with the best of all religions and philosophies--have given the world so
much in terms of what we call freedom and democracy. Do mystics praise Jesus? In a way,
but as the man who revolutionized the world, not as a God. We celebrate his life as the Master
of wisdom and are encouraged to behave as he did, with compassion, with conviction, with
courage in the face of adversity.

Christian tradition tells us, "In Him there is no sin," and, "He took away the sins of the
world." Mystic understanding is that Jesus's message was one that destroyed the concept of
guilt, sin, and shame. He only shamed the pious religious leaders whose job it was to keep the
masses feeling like wretches, and the political leaders who capitalized on such guilt.

Christian tradition tells us Jesus was the only son of God. Mystic understanding is that we are
all children of the most high and that Jesus is important for cluing us in to this knowledge.

(Gary E. Headley, 1814 NE Miami Gardens Drive, Apt. 204, Miami, FL 33179.)

EDITOR'S NOTE: With a great deal of reluctance, I have published Mr. Headley's letter in its
entirety. I appreciate his interest in The Skeptical Review, and I hope he continues to read it.
I'm certainly glad he recognizes the unlikeliness that miraculous events like the virgin birth
and the resurrection of Jesus actually happened, but I regret that he still seems to think that
there is some "mystical" importance to these stories that transcends the literal interpretations
that most Christians believe in. In saying this, I don't mean to imply that the New Testament
has no importance at all, because there is always value to be found in considering the
philosophical ideas of others. I suspect, however, that Mr. Headley sees far more importance

Volume 1990 - 2002 Issue


Page 815 of 2049
Skeptical Review Edited by Farrell Till
in Christian philosophy than in Islamic or Hindu or Buddhist philosophy, and this is an
arbitrary position that I find hard to understand.

Quite honestly, I have never understood the person whose intellectual growth leads him to see
the legendary and mythological bases of the Christian religion but who can't quite grow tall
enough to reject it entirely. Leaders in the controversial Jesus Seminar are examples of what I
mean. They have publicly proclaimed, for instance, that the resurrection of Jesus never
happened "except in the hearts and minds of early Christians." On the resurrection issue, they
have even said that if Jesus was really crucified, his body was probably thrown into a garbage
dump and later eaten by dogs. If that is true, then I can see no reason at all to cling to
Christianity, for, like Paul, I would have to say that if Jesus did not rise from the dead, the
Christian faith is vain and those who believe in it are "of all men most miserable" (1 Cor.
15:17-19). Liberal Christians, however, will deny the virgin birth, the miracles of Jesus, and
his resurrection but still contend that these stories have a "spiritual" importance in that they
symbolize the hopes and aspirations of people.

I see this kind of thinking in Mr. Headley's letter, and I find it hard to understand. If Jesus did
not rise from the dead, then I have no reason to hope that I will. Personally, I find much more
satisfaction in facing reality, accepting the fact that this life is probably all that I have and
ever will have, and living my life accordingly.

Mr. Headley publishes The Mystic and Gnostic Newsletter, which has some good ideas in it
sometimes, but most of it is the kind of material suggested by the title. Anyone interested in
reading more of what he has to say may contact him at his address listed above.

[May 26, 2001, INTERNET INFIDELS NOTE: Mr. Headley has contacted us and requested
that we link to Mr. Headley's web site by way of reply to Mr. Till, above. Mr. Headley may
also be reached by e-mail.]

I once had a professor who often piously cited St. Paul's dictum that "faith is the evidence of
things unseen" as unimpeachable wisdom. He was a pantheist and mystic, who still clung to
the carapace of Christianity. At the time, I accepted the professor's words without challenge.
It was a time when one did not dispute professors with impunity.

Accepting the ontological argument that "things" can be immaterial, I gather that the "things"
meant by Paul are, among others, love, kindness, compassion, truth, honesty. etc. In Paul's
context, however, it appears that faith is belief in the Pauline version of Christianity. Other
theologies (Hinduism, Buddhism, Islam, etc.) would not be evidence of things unseen within
the narrow scope of Paul's Christian orthodoxy. However, divorced from human conduct, the
virtues of love, kindness, etc. are meaningless abstractions. Kindness, for example, cannot
exist qua kindness, like some platonically ideal chair, existing only in the philosopher's brain.
The deed is nonexistent, absent the doer. One must then ask the believer if an atheist or
agnostic is capable of love, kindness, compassion, truth, and honesty. The answer is apparent.
If the believer answers that the atheist or agnostic is incapable of any of these virtues, he or
she is singularly lacking in the last of the series.

Volume 1990 - 2002 Issue


Page 816 of 2049
Skeptical Review Edited by Farrell Till
In short, "faith" is entirely unnecessary to validate the presumed virtues adduced by Paul. But
if "faith" extends to belief in angels, devils, possessed swine, virgin birth, human gods raising
the dead, changing water to wine, walking on water, and other hocus-pocus, then Martin
Luther's favorite "whore," i.e., reason, must intervene. Reason tells us that faith in the
aforementioned beliefs of many Christians is gullibility chasing after chimera.

(Fred L. Ehrstein, 9 Westwood Drive, Belleville, IL 62223-6407.)

Care to hear from another grateful, recovering ex-fundamentalist? I strongly identified with
the writer of the anonymous letter in the March/April issue. You may forward her a copy of
this letter if you'd like; and she may feel free to correspond with me if desired. Apparently,
there are a lot more of us out there than I ever expected. I've often considered starting a new
organization, "Fundamentalists Anonymous," if only I had more free time. (I'm graduating
from medical school in May 1996, and beginning residency training in psychiatry.)

My evolution into a free-thinking "Jeffersonian Deist," if you will, began at the end of my
first year of medical school, when faced with a life-threatening cancer diagnosis. I was forced
to reevaluate my priorities in life and reexamine my whole way of thinking. Well, I'm healthy
as a horse now and much better off mentally for having gone through the experience!

I'm amazed now at what a sick religious addict I was. You know, fundamentalist Pharisees
don't actually read their "Holy Word of God." It was only a short time ago that I realized that
the contradictions begin on page one. The first two chapters of Genesis contain contradictory
creation myths. One clearly states that plants and animals were created before man, and the
next one says man was created first.

In reality, fundamentalists are idolaters; they blindly worship a book. Not "a book," per se but
a 2000- to 4000-year-old collection of books, written by various authors to various diverse
cultures, far removed from our own, that contradicts itself on every other page. To "literally
interpret" such an ancient collection of literature as "absolute truth" is clearly delusional and
borders on psychotic.

One good thing about being an ex-fundamentalist, I can proof-text with the best of them. I'm
always happy to point out that they are condemned by their own "Holy Word of God."
Matthew 7:22-23 says, "Many will say to me on that day, `Lord, Lord, did we not prophesy in
your name... cast out demons in your name... do many miracles in your name?' And I will
declare to them solemnly, `Depart from me you evil doers, I never knew you.'" Red letter
print and all! Now, who is it today who claims to prophesy, cast out demons, and do miracles
"in the name of Jesus"? Why, the fundamentalist "church," of course!

See how the religious-hate game works? Delve into that wicked, self-contradictory book, and
you can proof-text to condemn anyone. "God's Word" is truly an equal-opportunity offender.

(Dr. Woodrow Coppedge, P. O. Box 691021, San Antonio, TX 78269.)

Volume 1990 - 2002 Issue


Page 817 of 2049
Skeptical Review Edited by Farrell Till
EDITOR'S NOTE: Dr. Coppedge's letter was forwarded to the anonymous letter-writer as he
requested. When I read letters like his, I recall my own experiences and wonder how people
who read the Bible can keep from seeing the contradictions that Dr. Coppedge referred to.
They are so obvious that I can't believe there was a time when I didn't see them, but there was.
Religion has a remarkable power to blind believers to obvious truths and realities. As Dr.
Coppedge noted, to believe that literature as ancient as the Bible could possibly be "absolute
truth" is clearly delusional and borders on psychotic, but there are millions of people who
have been deluded into believing it. And people actually ask me why I do what I do. Could
there be a better reason than this?

Sorry I lapsed it. Please renew my subscription, starting with the March/ April 1996 issue.

You're welcome to join the TFE [The Freethought Exchange] fray. Otherwise, your silence to
my comments is telling. Where are the minds that can grapple with the unheard other facts?

(Willard Small, P. O. Box 705, Picabo, ID 83348-0705.)

EDITOR'S NOTE: The "TFE fray" that Mr. Small alluded to is an issue that he raised in The
Freethought Exchange and tried to get TSR involved in through an article he submitted. It was
the kind of article that leaves the reader scratching his head and wondering, "What the hell is
this guy trying to say?" Unfortunately, I get articles like this much too often, almost always
accompanied with I-dare-you-to-publish-this comments. The first one I received came from a
man in Belize (formerly British Honduras), who had determined from a far-fetched
interpretation of scripture that Jesus was skinned alive before he was nailed to the cross. He
sent me long, rambling incoherent articles that I finally stopped even trying to read, but the
mailing of each issue of TSR was sure to bring a letter from him gloating over my fear to
publish the truth that he had discovered and that the world at large seemed afraid of. He
would stuff the envelopes with copies of challenges that he had sent to popular televangelists
and talk-show hosts daring them to put him on the air. I guess the guy actually thought that he
had confounded me with arguments that I couldn't refute, and in a sense I suppose he was
right. How does one refute sheer stupidity?

Now comes Mr. Small with startling insights that he dares me to publish. The best I can tell,
he has determined that Jesus was the reincarnation of King David and that a third incarnation
of David is supposed to occur. He bases his peculiar belief on Matthew 16:21 where Jesus
said that he would be killed and raised on the third day. Small argues that the Greek word
aion, which could mean day in the sense of a 24-hour period, most often meant age in the
sense of an eon or long period of time. Hence, Mr. Small has concluded that David was
destined to be reincarnated a third time at an age distant from the second incarnation and that
this time has at last arrived, so David (Jesus) is here again. Guess who this third incarnation
is? That's right. None other than Willard Small himself, and he has even sent computer-
scanned pictures of himself to The Freethought Exchange to prove his resemblance to Jesus.
Has no one ever told this fellow that we have no pictures or portraits of David or Jesus, and so
there is no way to know if the resemblance is real? (And this guy wonders why I didn't
publish his article?)

Volume 1990 - 2002 Issue


Page 818 of 2049
Skeptical Review Edited by Farrell Till
I also have some very bad news for him. Someone in the very county I live in has beaten him
to the draw. King David lives here, and as a matter of fact, he was once married to my
daughter-in-law's grandmother. This "David" even proclaimed his "kingship" on his checks,
which listed his name as King David with no other identifying moniker. As indicated above,
my daughter-in-law's grandmother is no longer married to him. I have never bothered to ask
why. At any rate, what's a guy like me supposed to do? Here is someone right under my nose
proclaiming himself to be King David, and then comes an announcement from Idaho that the
real incarnation of King David lives there. What to believe? It's a real dilemma.

An even bigger problem for Mr. Small (King David? Jesus?) is the fact that the Greek word
aion was not used in Matthew 16:21 when Jesus said that he would be raised up on the third
day. The actual word was hemera, which meant "the time space between dawn and dark" (see
Strong # 2250). I hate to be the bearer of bad news, but if Mr. Small will check in a
concordance, I think he will find that this is the case. Perhaps he would like to tell us what
this (if anything) does to his belief that he is the reincarnated Messiah.

When Mr. Small sent the letter published above, I had to pay the post office three cents in
postage due, because he had put only a 29-cent stamp on the letter. I don't really mind the
three cents I had to pay, but I just couldn't help thinking that a reincarnated Messiah should
know what it costs to mail a first-class letter. Also I have to wonder about some of the
spelling and grammatical mistakes in the article he sent to TSR and The Freethought
Exchange. Wouldn't an omniscient, omnipotent deity know better than to make such
mistakes? Maybe not, because he couldn't keep mistakes out of his "inspired word" either.

I have now joined "the fray," and this is my way of informing Mr. Small that I will devote no
more space to his Messianic-reincarnation claims. The purpose of The Skeptical Review is to
publish competently written articles either for or against the inerrancy doctrine, not to give a
forum to every nutty religious idea that comes along.

I received a free subscription to The Skeptical Review in 1995, and I'm so impressed.
Professor Till's logical arguments are strong to ignore. Please add my name to your list of
subscribers. Also, please send me the following materials. (Order deleted.)

I was formerly a strong fundamentalist Christian. I still consider myself a Christian, but with
your magazine's help, I'm studying whether the Bible truly is inerrant. My biggest question is
whether it is logically sound to believe that God let only unimportant errors creep into the
Bible, the kind of errors that won't affect any of the foundations of Christianity (salvation,
baptism, heaven, hell, afterlife, loving one another, etc.).

Keep up the good work you're doing.

(Timothy J. Jones, 41 Robie Street, Bath, NY 14810.)

EDITOR'S NOTE: The doctrine of verbal inspiration states that God was responsible for the
very words that the biblical writers used, so its logical consequence would be absolute

Volume 1990 - 2002 Issue


Page 819 of 2049
Skeptical Review Edited by Farrell Till
inerrancy in everything contained in the Bible. There can't be even insignificant errors,
because an omniscient, omnipotent deity would be incapable of error of any kind. Most
Christian fundamentalists will avow that they believe in this doctrine. Nevertheless, I am
happy to see that Mr. Jones is reexamining his beliefs. If only more Christians would do the
same....

Thank you for the samples of your publication. I liked them very much and would like to be a
subscriber.

I wonder if some inerrantist in one of your future debates could clear up something for me
that I asked when I was beginning to be an errantist. This was when I was 10 years old; I am
now 68.

When Jesus went to the Garden of Gethsemane, he went away from his companions, who
promptly fell asleep, and while away from them, he prayed to his father to take the cup away
from him. What I want to know is, who was there to hear this prayer, an owl? A squirrel?
How does anyone know what he prayed? Not even his "Father" was listening.

(Sonia Coble, 3737 SW 117th, Apt. 13, Beaverton, OR 97005.)

EDITOR'S NOTE: I've never heard an inerrantist address this particular issue, but I suspect
that inerrantists would say that the gospel writers were divinely inspired, and so God told
them what to write. If this is their "explanation" of the problem, I would respond by telling
them that they can't have it both ways. When significant (even inconsistent) variations in
parallel accounts of the same event (which occur many times in the synoptic gospels) are
presented as reasons to believe that the writing of the Bible was not directed by an
omniscient, omnipotent deity, inerrantists will argue that even though God inspired the
writers, he left them free to express their individual personalities in the way that they wrote.
So which way was it? Did God decide on the very words that the writers used (as inerrantists
will argue when advancing an argument that depends on the meaning of a particular word),
and so that is how the gospel writers knew what Jesus said in a prayer that was spoken when
he was alone, or did God leave the writers free to express their individual personalities in
what they wrote? If the latter, then how could they have known anything that they didn't
personally witness or hear? I think Ms. Coble's letter raises a legitimate issue.

In two recent issues of The Skeptical Review, we discussed Israel's 430-year sojourn in Egypt.
I think that we have adequately described both sides of the issue, but you and I disagree on
key points. It seems obvious that neither of us is going to move the other from his position.
From my perspective, you are being unreasonable and wrong in requiring that the Aaronic
genealogy in Exodus 6 describe a father to son to grandson to great-grandson descent. You
reject my arguments against your position, in my opinion, more because you disagree with
them than because they are illogical. Each of us believes that we have made a case to prove
our position. You believe that this case offers an obvious example of Biblical discrepancy. I

Volume 1990 - 2002 Issue


Page 820 of 2049
Skeptical Review Edited by Farrell Till
see these scriptures providing us with a timetable for historical events and I see no
discrepancy. If there were an obvious example of Biblical inerrancy, it is not this one. The
only additional action we might take now would be to find an impartial observer to review the
arguments and determine which are valid and which need further support.

At the very least, we have shown how two different people can interpret the scriptures in two
entirely different ways. Others can read the arguments we presented and decide for
themselves what they will believe. I think we have shown that genealogies do not have to be
dry and boring. I appreciate the tremendous amount of work you put into this effort, both in
researching the issues and writing your articles.

(Roger Hutchinson, 11904 Lafayette Drive, Silver Springs, MD 20902, e-mail


rhutchin@aol.com)

EDITOR'S NOTE: I have often said that it is difficult, if not impossible, for anyone to be
completely objective, so I'm sure that I am no exception to this human weakness. However, I
would like to remind Mr. Hutchinson, as I have said to him before on the internet, that I was
once a stubborn Bible fundamentalist like him. The fact that I changed dramatically in my
thinking is proof that I am not a person who is entirely unobjective. Can Mr. Hutchinson point
to anything in his personal life that would indicate a comparable willingness to consider
ideologies that are in conflict with his own?

As for finding a disinterested party to decide the issue, I'm sure that that would not work.
Both of us would probably be unwilling to accept a decision that disagrees with our respective
positions. I'm afraid that we will have to leave the decision up to the individual readers, and I
am perfectly willing to do that. In fact, what else can we do?

Volume 1990 - 2002 Issue


Page 821 of 2049
Skeptical Review Edited by Farrell Till

The Skeptical Review


Volume Seven, Issue Five
September/October 1996
Farrell Till, Editor

• Family Values
We hear a lot these days about family values, but if Christians would actually read
their Bibles for a change, they might be horrified at the type of "family values" they
would encounter.
• A Response to Alleged Difficulties in Matthew 27:9-10
According to Wilhelm E. Schmitt, Mr. Till has "once again wasted much space, ink,
and energy fulminating on a nonexistent problem in his attempt to impugn the
inerrancy of Scripture."
• A Matter of Literary Style
God could use a good course in basic writing, says Farrell Till.
• Till's Errors Concerning Tyre
Farrell Till claims that Ezekiel's prophecy against Tyre failed, but in truth, the
"failure" that has taken place is Till's failure to distinguish between plural pronouns
and singular pronouns. At least, that's what Matthew Hogan claims in this article.
• Hogan's Errors Concerning Pronouns
Like all others in the fundamentalist repertoire of fulfillment claim, the alleged
prophecy in Ezekiel 26 offers no plausible evidence that the Bible was divinely
inspired.
• How Many Women Went to the Tomb?
According to Roger Hutchinson, no contradictions exist concerning the number of
women going to the tomb following the crucifixion of Jesus.
• Not Enough to Make the Stories Convincing
Till shows that Hutchinson's attempted to resolve the contradictions concerning the
number of women at Jesus' tomb is not sound, and points out a deeper problem that
Hutchinson has not addressed.

Volume 1990 - 2002 Issue


Page 822 of 2049
Skeptical Review Edited by Farrell Till
• The Internet for Nontheists
Jeff Lowder, now President of the Internet Infidels, offers some pointers to
freethinkers about the Internet.
• Complaints About A Lot Of Things
An angry reader sounds off.
• Another Frustrated Inerrantist?
Till responds to the disgruntled reader above.
• From the Mailbag
Letters to the editor.

Family Values
by Farrell Till
We hear a lot these days about family values. The Christian right has made this such an issue
that political candidates have been conditioned to believe that mentioning family values
frequently in their campaign speeches is an easy way to score points with voters. Of course, if
a candidate also associates "family values" with the so-called biblical principles on which our
country was founded, he is certain to score even more points with the Christian right. It has
become a cheap way to get votes.

The duplicity of this political tactic is twofold: (1) our country was not founded on "biblical
principles," and (2) the "family values" found in the Bible are completely contrary to what the
religious right supposedly stands for. One has only to read unbiased histories of the
Revolutionary War period to learn that the so-called founding fathers of our country were
steeply entrenched in Deistic philosophy and even in some cases openly hostile to
Christianity, and if some of those in Christian fundamentalist groups would just dust off their
Bibles and actually read them for a change, they might be horrified at the type of "family
values" they would encounter on the pages of their inspired word of God.

Let's take the family of Abraham as an example. This man was considered to be the father of
all the faithful (Rom. 4:16-17), and both he and his wife Sarah were listed among the great
heroes of faith in Hebrews 11:8-12. So how did they measure up in terms of the type of
family values that the Christian right claims to stand for? Not very well, it seems. In Genesis
16, we learn that Abraham and Sarah after struggling with the problem of Sarah's infertility
solved it by having Abraham impregnate Sarah's handmaiden Hagar (vs:2-4). As a result,
Hagar gave birth to Abraham's first son, Ishmael (v:15).

Now let's suppose that the religious right should learn that a political candidate whose wife
was infertile had impregnated his wife's maid in order to have children. Would such a man be
considered worthy of endorsement by the Christian Coalition? If news of his private life
became public knowledge, it would probably end his public career, but a biblical character

Volume 1990 - 2002 Issue


Page 823 of 2049
Skeptical Review Edited by Farrell Till
who did this very thing has been lauded for thousands of years as a great hero of faith. Go
figure.

It gets even worse. After Hagar became pregnant, she despised Sarah for having given her to
Abraham, so this caused friction in the family. When Sarah complained to Abraham, he said,
"Your maid is in your hand; do with her as you please" (v:6). Say what! A man impregnated a
servant and then told his wife to do with her whatever the wife wanted. How's that for family
values?

What Sarah pleased to do was deal with Hagar so harshly that Hagar finally fled from Sarah's
presence (v:8). In the wilderness that she had fled into, Hagar encountered Yahweh (as people
routinely did in those days), and he advised her to return to Sarah and submit to her (v:9). In
other words, Yahweh's advice was for Hagar to go home and eat dirt. It's called "family
values."

After Sarah had borne her own son Isaac, the conflict between Hagar and her intensified, and
Sarah asked Abraham to "cast out the handmaiden and her son" (Gen. 21:10). To Abraham's
credit the request grieved him "because of his son" (v:11), but apparently Yahweh wasn't so
easily distressed. He told Abraham to listen to whatever Sarah had said to him (v:12), and so
Abraham rose early the next morning, gave bread and a bottle of water to Hagar, and sent her
and the boy away (v:14). Another triumph for family values, biblical style!

Now let's imagine this story in a modern setting (if it is at all possible to do so). Our
hypothetical political candidate (a champion of family values, of course) after impregnating
his wife's maid, advises his wife to deal with her jealousy in whatever way she wishes, and so
the wife treats the maid in such a way that she runs away and puts herself into a situation that
endangers both her and her unborn child. Then when the maid returns home submissively and
finally gives birth to her baby, the wife orders her husband to throw the maid and her child
out to survive the best way they can. Such a scenario would reduce some of the complaints
that the Christian right makes about the morality of Bill and Hillary Clinton to mere
trivialities.

If their treatment of Hagar and Ishmael had been the only flaw in the moral character of
Abraham and Sarah, we could overlook it. After all, no one is perfect. The biblical saga of
Abraham and Sarah, however, indicates other flaws in their character that can hardly be
considered "family values" that civilized people would want to emulate. Genesis 12:10-20
records a famine in the land that forced Abraham and Sarah to go into Egypt. Sarah, it seems,
was such a raving beauty that Abraham feared the Egyptians would kill him in order to get
her, so a plan was devised. He asked Sarah to tell the Egyptians that she was his sister so that
his life would not be in jeopardy while they were in Egypt (v:13). This was apparently what
they told the Egyptians, and, sure enough, Sarah's beauty was immediately recognized. The
princes of Egypt commended her to Pharaoh, who took her into his palace and rewarded
Abraham with sheep, oxen, donkeys, camels, and female servants (vs:14-16).

What a paragon of virtue! Here is a biblical hero of faith who concocted a lie about his
relationship to his wife and then, to put it bluntly, pimped her to the king of Egypt for flocks
and herds and female servants. In the case of Hagar, we have an example of how Abraham

Volume 1990 - 2002 Issue


Page 824 of 2049
Skeptical Review Edited by Farrell Till
treated his female servants, so we can well imagine that if any truth is in this story, Abraham
didn't lack for female companionship while Sarah was cavorting in the palace with Pharaoh.
In today's society, Abraham and Sarah would be called "swingers." Is this the kind of "family
values" that the religious right wants our country to practice?

Christian fundamentalists will no doubt quibble that technically Abraham and Sarah didn't lie
when they said that she was his sister, because she was biologically Abraham's half-sister
(Gen. 20:12), but intention is the primary factor in determining if a statement is a lie, and
Abraham and Sarah had clearly conspired to deceive the Egyptians about their relationship. If
that isn't lying, what is?

But there were other questionable escapades in the biblical story of Abraham and Sarah. In
Genesis 20, we learn that Abraham and Sarah pulled this same trick again, this time on
Abimelech, the king of Gerar. The results were pretty much the same. Abimelech learned that
Sarah was really Abraham's wife and returned her to Abraham with a reprimand for their
deception. Nevertheless, they again came out of the deal smelling like a rose. Abimelech gave
Abraham sheep, oxen, and servants and gave Sarah a thousand pieces of silver. Family values
paid rather handsomely in those days.

A Response to Alleged Difficulties in


Matthew 27:9-10
by Wilhelm E. Schmitt
I have received and read several recent issues of The Skeptical Review. While I appreciate Mr.
Till's efforts to state his positions clearly, frequently I find that his arguments -- otherwise
sound -- fail to take into account matters that either invalidate his arguments or render them
moot. (I am the first to concede that some of his opponents' arguments suffer from the same
malady; often they do not sufficiently close up loopholes in their arguments, or they fail to
write in a nonequivocal manner, thus exposing themselves to Mr. Till's counterattacks.)

An example of this with which I wish to deal here involves one of the points of contention in
Mr. Till's exchange with Dr. Hugh Ross on the subject of fulfilled prophecy, published in the
January/February 1996 Skeptical Review. The controversy involved the question as to whether
the Gospel writer Matthew incorrectly attributed the prophecy of the 30 pieces of silver to
Jeremiah, and Mr. Till's argument that the passage in Zechariah 11:12-13 cannot reasonably
support the notion that Matthew was actually quoting Zechariah. I offer the following
observations that were not addressed by either Dr. Ross or Mr. Till.

Two difficulties arise in connection with Matthew 27:9-10, and a number of solutions have
been proposed to meet them.

Volume 1990 - 2002 Issue


Page 825 of 2049
Skeptical Review Edited by Farrell Till
The first difficulty is that the words quoted from and attributed to Jeremiah are not found in
his written prophecy. Solutions that have been suggested include the following: (1) that
"Matthew quoted from memory" (Augustine and others), (2) that the passage was originally in
Jeremiah, but the Jews cut it out (Eusebius and others); however, no evidence for this has
been located or produced, (3) that it was contained in another writing by Jeremiah, which is
now lost (Origen and others), (4) that Matthew intended to put Jeremiah for the whole body of
the prophets (Bishop Lightfoot and others), but again no evidence of such a tactic can be
found in the other prophets, (5) that it was "a slip of the pen" on the part of Matthew (Dean
Alford), (6) that the mistake was allowed by the Holy Spirit deliberately, to teach us not to
concern ourselves as to who the writers were but to faithfully accept all prophecy as coming
directly from God, Who spoke by human prophets (Bishop Wordsworth), (7) that some
annotator wrote "Jeremiah" in the margin and it "crept" into the text (Smith's Bible
Dictionary).

However, the foregoing "solutions" create difficulties far more grave than the one that they
attempt to eliminate. All of them may be shown to be unnecessary by observing the simple
fact that Matthew does not state that the prophecy was written by Jeremiah but that it was
"spoken" by him. I suggest that herein lies the answer to the alleged problem.

The second difficulty is created by the suggestion that the prophecy attributed to Jeremiah is
really written in Zechariah 11:10-13. For some time I accepted this notion, and on one
occasion promoted it in writing, but further study has convinced me that this idea cannot be
correct, for the following reasons:

(1) Zechariah 11:10-13 contains no reference either to a "field" or its purchase. In fact, the
word "field" (shadah) occurs only in chapter 10, verse 1, which in no way relates to our
subject at all.

(2) Regarding the "thirty pieces of silver," the passage in Zechariah speaks of them with
approval, while in Matthew they are not so spoken of. "A goodly price" (`eder hayekar)
signifies "sufficiency," and the verb yakar carries the meaning of being "precious." The text
does not indicate that the amount was paltry or that the offer of it was in any sense an insult.
However, this latter sense is conveyed in Matthew 27:9-10.

(3) The givers were "the poor of the flock," and this enhanced its value, very much as in the
case described in Mark 12:43-44.

(4) The waiting of "the poor of the flock" was not characterized by hostility but friendliness.
The Hebrew word shamar occurs more than 450 times in the Old Testament, of which only
about 14 express hostility.

(5) Concerning the disposal of the silver, the sense of the verb cast must be determined by the
context in which it is used, not by the verb itself. In Zechariah 11, the context shows it to be
in a good sense, as in Exodus 15:25, 1 Kings 19:19, 2 Kings 2:21, and others.

(6) The "potter" is a fashioner; his work was not necessarily confined to fashioning "clay," but
his work extended also to metals. For confirmation of this, compare Genesis 2:7-8; Psalm

Volume 1990 - 2002 Issue


Page 826 of 2049
Skeptical Review Edited by Farrell Till
33:15; Isaiah 43:1, 6, 10, 21; and others. The verb yazar occurs in the Old Testament about 62
times, and in about 45 of these it has nothing whatever to do with a "potter." Moreover, a
"potter" in connection with the temple or its service is unknown in fact or to Scripture.
Furthermore, silver would be useless to a potter, but necessary to a fashioner of metallic
things or for payment to such artisans. One might as well cast clay to a silversmith as silver to
a potter.

(7) The Septuagint, and also its revision by Symmachus, reads in verse 13, "cast them, i.e., the
30 pieces of silver) into the furnace" (Greek: eis to choneuterion). This shows that before the
Gospel of Matthew was written, yotzer was interpreted as referring not to a "potter" but to a
fashioner of metals.

(8) Note that the persons mentioned in the two passages are different. In Matthew we have
"they took," "they gave," "the price of him"; in Zechariah one reads "I took," "I cast," "I was
valued." In addition, Matthew names three parties as being concerned in the transaction;
Zechariah speaks of only one.

(9) In Matthew the money was given "for the field"; in Zechariah it was cast "unto the
fashioner."

(10) Matthew not only quotes Jeremiah's spoken words but names him as the speaker. This is
parallel to Matthew 2:17-18, where the figure of speech known as "metonymy of cause" is
employed, referring to a prophecy of Jeremiah that was spoken as well as written.

On the basis of all the foregoing, I suggest that the passage in Matthew 27:9-10 cannot have
any reference to Zechariah 11:10-13. But the following considerations must also be kept in
mind:

(1) It is not inconceivable that Matthew quoted Jeremiah's spoken words (of which he had
knowledge either from some historical source or by direct revelation from the Holy Spirit
through the process referred to as "inspiration" in 2 Timothy 3:16) and interjected other words
by way of parenthetical explanation. These are not to be confused with the quoted words.
They may be considered to have been combined thus:

Then was fulfilled that which was spoken by Jeremiah the prophet, saying: `And they
took the thirty pieces of silver [the price of him who was priced, whom they of the
sons of Israel did price], and they gave them for the potter's field, as the Lord
appointed me.'"

Thus Matthew quotes that which was spoken by Jeremiah the prophet, and combines with the
actual quotation a parenthetical reference to the price at which the prophet Zechariah had
been priced.

(2) Had the sum of money been twenty pieces of silver instead of thirty, a similar remark
might have been interjected thus:

Volume 1990 - 2002 Issue


Page 827 of 2049
Skeptical Review Edited by Farrell Till
Then was fulfilled that which was spoken by Jeremiah the prophet, saying: "And they
took the twenty pieces of silver [the price of him whom his brethren sold into Egypt],
and they gave them for the potter's field," etc.

(3) Or had the reference been to the compensation for an injury done to another man's servant,
as in Exodus 21:32, a similar parenthetical remark might have been introduced thus:

Then was fulfilled that which was spoken by Jeremiah the prophet, saying: "And they
took the thirty pieces of silver [the price given in Israel to the master whose servant
had been injured by an ox], and they gave them for the potter's field," etc.

By not taking into account the literary device known as "parenthetical interjection," Mr. Till
has once again wasted much space, ink, and energy fulminating on a nonexistent problem in
his attempt to impugn the inerrancy of Scripture. To summarize briefly: a designed
parenthetical insertion by the inspired Evangelist of a reference to Zechariah, interjected in a
direct quotation from the prophet Jeremiah, is very different from a "mistake," or "a slip of the
pen," "a lapse of memory," or "a corruption of the text," any of which needs an apology.

The quotation itself, as well as the parenthetical reference, are both similarly exact.

Much of the above material is not due to my original research. It was noticed by many
scholars of preceding generations to whom I am deeply indebted.

(Wilhelm E. Schmitt, 14575 Louisiana Avenue, Prior Lake, MN 55372.)

A Matter of Literary Style


by Farrell Till
Mr. Schmitt has simplified my task by admitting the unlikeliness of various attempts that have
been made to explain why Matthew attributed to Jeremiah a "prophecy" that can be found
nowhere in the book of Jeremiah (Matt. 27:9-10). Schmitt very wisely recognizes that these
various solutions "create difficulties far more grave than the one they attempt to eliminate."
Since there is no controversy between us on this point, all I need to do is show that Schmitt's
solution is also fraught with difficulties.

Mr. Schmitt believes that the solution to this problem is as simple as noticing that Matthew
said only that the prophecy had been spoken by Jeremiah but did not say that the prophecy
had been written. This, by the way, is the same "explanation" that inerrantists resort to when
confronted with Matthew's claim that Joseph took his family to Nazareth in order to fulfill a
prophecy that Jesus would be called a Nazarene. Here too Matthew said that this had been
"spoken" by the prophets, and so inerrantists explain the absence of any such prophecy in the
Old Testament by quibbling that Matthew had said that the prophets had "spoken" this. He did

Volume 1990 - 2002 Issue


Page 828 of 2049
Skeptical Review Edited by Farrell Till
not say that they had written it. So now Mr. Schmitt is making this same appeal as an
explanation of the problem in Matthew 27:9-10, where the claim is that Jeremiah had spoken
a prophecy that cannot be found anywhere in the book of Jeremiah.

Although commonly used, this explanation fails to consider the fact that Matthew, who
constantly saw prophecy fulfillment in events associated with Jesus, consistently introduced
his various prophecy-fulfillment claims by saying that such and such an event fulfilled that
which had been spoken by prophet so and so. The following examples will illustrate that this
was simply Matthew's literary style.

The birth of Jesus: "So all this was done that it might be fulfilled which was spoken by the
prophet, saying: `Behold, the virgin shall be with child and bear a Son, and they shall call His
name Immanuel,' which is translated, `God with us'" (Matt. 1:22). Although Matthew said that
this prophecy had been spoken, it was written in Isaiah 7:14.

Jesus in Capernaum: "Now when he [Jesus] heard that John had been put in prison, He
departed from Galilee. And leaving Nazareth, He came and dwelt in Capernaum, which is by
the sea, in the regions of Zebulun and Naphtali, that it might be fulfilled which was spoken
by Isaiah the prophet, saying: `The land of Zebulun and the land of Naphtali, by the way of
the sea, beyond the Jordan, Galilee of the Gentiles; the people who sat in darkness have seen a
great light...'" (Matt. 4:12-16). Matthew said that this prophecy had been spoken by Isaiah, but
it is actually a quotation of a written statement in Isaiah 9:1-2.

The chosen servant: "But when Jesus knew it [a plot by the Pharisees to kill him], He
withdrew from there. And great multitudes followed Him, and He healed them all. Yet He
warned them not to make Him known, that it might be fulfilled which was spoken by Isaiah
the prophet, saying: `Behold! My servant whom I have chosen, my Beloved in whom My soul
is well pleased! I will put My spirit upon Him, and He will declare justice to the Gentiles...'"
(Matt. 12:15-18). This "prophecy" that Matthew said had been spoken by Isaiah was actually
written in Isaiah 42:1-4.

The parables of Jesus: "All these things Jesus spoke to the multitude in parables; and
without a parable He did not speak to them, that it might be fulfilled which was spoken by the
prophet, saying: `I will open My mouth in parables; I will utter things kept secret from the
foundation of the world'" (Matt. 13:34-35). Matthew said that this "prophecy" had been
spoken, but it is actually a quotation of a statement written in Psalm 78:2.

These examples are sufficient to establish that Matthew's literary style was to introduce a
prophecy-fulfillment claim by saying that the event had been spoken by a prophet. This wasn't
just a literary device that he occasionally used; he always introduced his own prophecy-
fulfillment claims in this way. Sometimes, in quoting prophecy-fulfillment claims that had
been made by Jesus, John the Baptist, and others, he had them say that the prophecies had
been written (Matt. 2:5; 4:6; 11:10; 21:42), but Matthew himself always introduced his own
prophecy-fulfillment claims by saying that they had been spoken. That being true, we can
conclude that in reference to prophecy-fulfillment, Matthew used the word spoken in the
sense of written, and that is unfortunate for Mr. Schmitt's explanation of Matthew 27:9-10,
where a prophecy that appears nowhere in the book of Jeremiah was nevertheless attributed to

Volume 1990 - 2002 Issue


Page 829 of 2049
Skeptical Review Edited by Farrell Till
Jeremiah. An analysis of Matthew's writing style gives us every reason to think that Matthew
thought that Jeremiah had written about the purchase of Judas's burial place for 30 pieces of
silver. So Mr. Schmitt's explanation is really no explanation at all.

Certainly, it is no explanation for Schmitt to claim that Matthew, either through "some
historical source or by direct divine revelation from the Holy Spirit, could have had
knowledge of words that had been spoken by Jeremiah but not written." Such an argument
assumes the very point that biblicists want to prove, i.e., the Bible is inerrant because it was
inspired by the Holy Spirit. The fact that Matthew quoted a prophecy that may never have
been made by Jeremiah is a plausible reason to doubt the doctrine of inspiration, so Schmitt
resorts to question begging when he says that Matthew could have known by direct
inspiration that Jeremiah had spoken this prophecy. That writers make mistakes in quoting is
a known fact. That writers are inspired by invisible deities is an unverifiable conjecture. So
Schmitt needs more than a could-have-been guess. He needs solid evidence that Jeremiah
really did make the statement.

Schmitt's belief that Matthew combined a statement that Jeremiah had only spoken with a
parenthetical reference to the price that Zechariah had been paid for his shepherding services
is really a moot point. If Matthew attributed a prophecy to Jeremiah that we have every reason
to suspect was never made by Jeremiah, what difference does it make if a parenthetical
reference to Zechariah was included? He still erred in attributing to Jeremiah a prophecy that
was never made. So I will remind Schmitt of a point that I have made before. We have every
reason to expect an omniscient, omnipotent deity to inspire clear, unequivocal writing. If such
a deity did inspire Matthew to write the statement as it was recorded, he could profit from a
good course in basic writing.

Till's Errors Concerning Tyre


by Matthew Hogan
In his booklet Prophecies: Imaginary and Unfulfilled, Farrell Till claims that Ezekiel's
prophecy against Tyre failed to materialize. In truth, the "failure" that has taken place is Till's
failure to distinguish between plural pronouns and singular pronouns -- among other things,
but more on that later.

Briefly, here are a few facts concerning Tyre. Tyre (Hebrew, Sor) was a dual-city. Part lay on
the coast, and part lay offshore on an island. Tyre still exists today but is a minor port, and its
wealth is gone.

In the year of Jerusalem's fall, God gave Ezekiel the following information concerning Tyre's
future:

Volume 1990 - 2002 Issue


Page 830 of 2049
Skeptical Review Edited by Farrell Till
Therefore thus says Yahweh God: "Behold, I am against you, O Tyre, and will cause
many nations to come up against you, as the sea causes its waves to come up. And
they shall destroy the walls of Tyre and break down her towers; I will scrape her dust
from her, and make her like the top of a rock. It shall be a place for spreading nets in
the midst of the sea, for I have spoken," says Yahweh God; "it shall become plunder
for the nations. Also her daughter villages which are in the fields shall be slain by the
sword. Then they shall know that I am Yahweh."

For thus says Yahweh God: "Behold, I will bring against Tyre from the north
Nebuchadnezzar king of Babylon, king of kings, with horses, with chariots, and with
horsemen, and an army with many people. He will slay with the sword your daughter
villages in the fields; he will heap a siege mound against you, build a wall against you,
and raise a defense against you. He will direct his battering rams against your walls,
and with his axes he will break down your towers. Because of the abundance of his
horses, their dust will cover you; your walls will shake at the noise of the horsemen,
the wagons, and the chariots, when he enters your gates, as men enter a city that has
been breached. With the hooves of his horses he will trample all your streets; he will
slay your people by the sword, and your strong pillars will fall to the ground. They
will plunder your riches and pillage your merchandise; they will break down your
walls and destroy your pleasant houses; they will lay your stones, your timber, and
your soil in the midst of the water. I will put an end to the sound of your songs, and
the sound of your harps shall be heard no more. I will make you like the top of a rock;
you shall be a place for spreading nets, you shall never be rebuilt, for I Yahweh have
spoken" (26:3-14).

At this point, it is important that the reader carefully note chapter 26:7-11, where God says
that he would bring upon Tyre King Nebuchadnezzar of Babylon, who would bring horses
and "(come) with chariots, and with horsemen, and an army," and so forth. Here is where Till
commits a major blooper. Verse 11 ends the prophecy about Nebuchadnezzar's siege against
Tyre! Up until verse 11, the singular forms he and his are used. In verse 12, the prophecy
shifts to the plural form they.

Why the sudden change in pronouns -- from he and his to they? Because it is not
Nebuchadnezzar who is to fulfill verse 12. The they in verse 12 refers to the "many nations"
of verse 3, where it is mentioned that God would bring "many nations against (Tyre)."

Nebuchadnezzar didn't destroy Tyre because he wasn't the one prophesied to do so. Indeed,
Ezekiel mentions (29:18-19) that the Babylonian king would receive "wages from Tyre." The
they of verse 12, or the "many nations" of verse 3, were the ones prophesied to lay siege
against Tyre -- and not in one generation. Remember, verses 7-11 refer to Nebuchadnezzar's
siege -- not permanent destruction -- of Tyre.

The plural pronoun they, used in verse 12, refers to Alexander the Great and his army, and
numerous others: Greeks, Romans, Crusaders, the Mamelukes from Egypt, and so forth. All
of the armies above made a "plunder" of Tyre's riches -- gradually -- and not in one
generation, thus fulfilling verse 12.

Volume 1990 - 2002 Issue


Page 831 of 2049
Skeptical Review Edited by Farrell Till
Now notice that in Ezekiel 26:13-14 the pronouns shift again, this time from they to I. The
singular pronouns in verses 13-14 refer to God. Tyre will finally and totally be destroyed
when Christ returns.

In summary, Farrell misinterpreted Ezekiel's prophecy about Tyre in a number of ways: (1)
He failed to note the changes in pronouns, from he and his (Nebuchadnezzar) to they in verse
12 (Greeks, Romans, Crusaders, etc.) and then to I (God) in verses 13-14. (2) Verse 11 ends
the prophecy about Nebuchadnezzar. Also, verses 7-11 refer to a siege -- not to the
destruction of Tyre. (3) The they in verse 12 refers to various countries, including Romans,
Crusaders, the Mamelukes of Egypt, Alexander the Great and his army, etc. These nations
would make a "plunder" of Tyre's riches, thus fulfilling verse 12. (4) The singular pronoun I,
in verses 13-14, refers to God himself. Final and total destruction awaits Tyre at the hands of
Christ when he returns!

(Matthew Hogan, 177 Salisbury Street, Rochester, NY 14609-4137.)

Hogan's Errors Concerning Pronouns


by Farrell Till
Although Matthew Hogan's article was written in response to material that did not appear in
The Skeptical Review, I have decided to publish it anyway. Ezekiel's prophecy against Tyre is
often cited as an amazing example of prophecy fulfillment, so it should be a subject of interest
to TSR readers. As we will see, this "prophecy," like all others in the fundamentalist repertoire
of fulfillment claim, offers no plausible evidence that the Bible was divinely inspired.

In my booklet Prophecies: Imaginary and Unfulfilled, I showed that Ezekiel's prophecy


against Tyre failed, because Nebuchadnezzar's siege of the island city did not succeed (a fact
easily verifiable by reliable historical records) and because, quite obviously, the city was not
permanently destroyed, "never to be rebuilt" (Ezek. 26:14). This too is a fact verifiable not
just by reliable historical records but by the actual existence of Tyre today.

Now comes Matthew Hogan to inform us that I have misinterpreted the prophecy because of
my failure to notice a change from the singular pronouns he and his (used in reference to
Nebuchadnezzar) to the plural form they, which Hogan argues was referring to nations in the
future and not just to Nebuchadnezzar. It may surprise Hogan to learn that I was well aware of
this switch in pronouns when I addressed the Tyre prophecy in my booklet, but I was also
aware of something else that has apparently escaped Hogan's notice. Such pronoun shifts are
quite common in the Bible, especially in the book of Ezekiel. From having taught college
writing for 30 years, I learned that it is apparently difficult for some writers to maintain strict
adherence to the rules of pronoun-antecedent agreement. The average person, for example,
would think nothing of writing, "The team lost their game last night," but the "correct" way to
write it would be, "The team lost its game last night." The word team is grammatically

Volume 1990 - 2002 Issue


Page 832 of 2049
Skeptical Review Edited by Farrell Till
singular, so strict adherence to the rules of pronoun-antecedent agreement requires that the
singular pronoun its be used in reference to team. Most people, however, aware that a team
consists of more than one person, will carelessly use the pronoun their in a sentence like this.
This is similar to what happened in Ezekiel's prophecy against Tyre.

Before we examine this prophecy, let's first notice that ungrammatical pronoun shifts were
characteristic of Ezekiel's writing. In the following examples from Ezekiel, to show how
obvious the shifts are, I will italicize the antecedent and emphasize in bold print the pronoun
referring back to the antecedent:

Lie also on your left side, and lay the iniquity of the house of Israel upon it. According
to the number of the days that you lie on it, you shall bear their iniquity (4:4).

Son of man, write down the name of the day, this very day -- the king of Babylon
started his siege against Jerusalem this very day. And utter a parable to the rebellious
house, and say to them ... (24:2-3).

Syria was your merchant because of the abundance of goods you made. They gave
you for your wares emeralds, purple, embroidery, fine linen, corals, and rubies
(27:16).

And the land of Egypt shall become desolate and waste; then they will know that I am
Yahweh (29:9).

Behold, I am against you, Sidon; I will be glorified in your midst; and they shall
know that I am Yahweh (28:20).

I could cite many other examples, but these are enough to establish that Ezekiel had a habit of
ungrammatically shifting pronoun references in his writing. In the last example, when Ezekiel
had Yahweh addressing Sidon in the second person (you and your), he inadvertently shifted
and had the statement finish with a third person plural (they) reference back to the singular
Sidon.

With this background in Ezekiel's pronoun usage established, we can now see just how weak
Hogan's defense of the Tyre prophecy is. Let's look at the place in the prophecy where Hogan
claims that Ezekiel stopped referring to Nebuchadnezzar individually and switched to "many
nations" in general. Keep in mind that even Hogan admits that Nebuchadnezzar was the point
of reference in the beginning of the text, which for brevity's sake I will skip:

He [Nebuchadnezzar] will slay with the sword your daughter villages in the fields; he
will heap up a siege mound against you, and raise a defense against you. He will direct
his battering rams against your walls, and with his axes he will break down your
towers. Because of the abundance of his horses, their dust will cover you; your walls
will shake at the noise of the horsemen, the wagons, and the chariots, when he enters
your gates, as men enter a city that has been breached. With the hooves of his horses
he will trample all your streets; he will slay your people by the sword, and your strong
pillars will fall to the ground. They will plunder your riches and pillage your

Volume 1990 - 2002 Issue


Page 833 of 2049
Skeptical Review Edited by Farrell Till
merchandise; they will break down your walls and destroy your pleasant houses; They
will lay your stones, your timber, and your soil in the midst of the water... (26:8ff).

The point where the shift from singular to plural occurs is obvious, but does the shift warrant
the significance that Hogan attaches to it? If in the entire book of Ezekiel, we could not find
an incorrect pronoun shift, we would agree that Hogan has a point, but when we are able to
find numerous pronoun shifts in Ezekiel's writings, we have to conclude that Hogan's point is
weak indeed.

The reason for the shift from singular to plural in this text was probably the same as in the
other examples of shifts that we noted in the book of Ezekiel. In verses 10 and 11, Ezekiel
predicted that horses (plural), horsemen (plural), wagons (plural), and chariots (plural) would
enter the city and thunder through the streets. Hence, the shift to they in the very next verse
could have been as inadvertent as Ezekiel's usage of they to refer to the house of Israel and to
Syria and Jerusalem. All of the antecedents were singular in these cases, yet Ezekiel used the
plural they in reference to them. This is a more plausible explanation for the abrupt shift in the
Tyre prophecy than Hogan's claim that Ezekiel suddenly changed the subject of the prophecy
from Nebuchadnezzar to "many nations."

There are good reasons to believe that the shift from singular to plural in the Tyre prophecy
was just another case of Ezekiel's careless use of pronouns, but if not and if Ezekiel actually
did intend the shift in pronoun reference to be a shift in subject, Hogan will have to tackle a
problem that would be just as damaging to biblical inerrancy as an obvious prophecy failure.
Why would a writer who was verbally inspired by an omniscient, omnipotent deity to
prophesy the destruction of a powerful contemporary city-state communicate his message as
confusingly as Ezekiel did? The singular pronouns he and his ended abruptly, and in the very
next verse shifted to the plural they without any attempt at all to notify the readers. Is that
any way for an inspired prophet to write? Should readers who are expected to believe that a
book was verbally inspired by an omniscient, omnipotent deity not have a right to expect
clear, unequivocal writing in that book? One would certainly think so, but if Hogan is right
about the Tyre prophecy, then Ezekiel didn't understand a very elementary writing principle:
changes of direction in writing, and especially sudden changes, should be signaled with
transitional devices. Hogan can look forever between verses 11 and 12 in the Tyre prophecy,
but he will find no transitional markers. Without such a marker, we have no reason at all to
think that Ezekiel intended to switch the focus of his prophecy from Nebuchadnezzar to other
nations.

In support of his assertion that "Nebuchadnezzar didn't destroy Tyre, because he wasn't the
one prophesied to do so," Hogan said, "Indeed, Ezekiel mentions (29:18-19) that the
Babylonian king would receive `wages from Tyre.'" Apparently, Hogan has difficulty
understanding fairly plain language, because this is not what Ezekiel 29:18-19 actually said.
The best way to show this is to look at the passage:

Son of man, Nebuchadnezzar king of Babylon used his army to labor strenuously
against Tyre; every head was made bald, and every shoulder rubbed raw; yet neither
he nor his army received wages from Tyre, for the labor which they expended on
it (29:18-19).

Volume 1990 - 2002 Issue


Page 834 of 2049
Skeptical Review Edited by Farrell Till
The passage actually says the exact opposite of what Hogan said. As history records,
Nebuchadnezzar put considerable labor into a prolonged siege of Tyre but was unable to
penetrate the island defenses. In this text, Ezekiel, astonishingly enough, is recognizing that
despite his prophecy that Tyre would fall to Nebuchadnezzar, the Babylonian king had
actually received no "wages" for his effort. As compensation for Nebuchadnezzar's failure to
profit from the siege of Tyre, Ezekiel was promising that Yahweh would reward the
Babylonians with Egypt:

Therefore, thus says Yahweh God: "Surely I will give the land of Egypt to
Nebuchadnezzar king of Babylon; he shall take away her wealth, carry off her spoil,
and remove her pillage; and that will be the wages for his army.

Why Hogan thinks that this passage teaches that Nebuchadnezzar was to receive "wages from
Tyre" is a mystery to me. It actually teaches that Yahweh would give him Egypt as
compensation for the wages that he failed to receive from Tyre. As I noted in my prophecy
booklet, this passage confirms that even Ezekiel himself recognized that his prophecy against
Tyre had failed. It is difficult to understand the minds of biblical prophets, but one would
think that Ezekiel would have had the common sense not to include this admission of a
prophecy failure in his book.

Hogan claims that Ezekiel 26:7-11 prophesied only that Nebuchadnezzar would lay siege to
Tyre, not that he would destroy it, but I suggest that he read the text again. He admits that the
he in these verses was Nebuchadnezzar, so the prophecy was clearly predicting that
Nebuchadnezzar would come from the north (v:7) and first slay with the sword Tyre's
"daughter villages in the field" (v:8). As Hogan correctly noted, Tyre had been built as an
island stronghold but had expanded onto the mainland with "daughter villages." We know
from historical records that invaders often sacked these mainland villages, and
Nebuchadnezzar did the same during his siege of Tyre. He destroyed the mainland villages,
but he was never able to penetrate the island defenses.

Destruction of the mainland villages, however, was not the extent of what Ezekiel had
prophesied for Nebuchadnezzar, and the text makes that very clear. After destroying the
"daughter villages," he would "heap up a siege mound against" Tyre (v:8). He would direct a
battering ram against the walls of Tyre and with his axes break down the towers of Tyre (v:9).
He would enter the city with his horses, horsemen, wagons, and chariots (v:10). HE would
trample all of Tyre's streets with the hooves of his horses, and he would slay Tyre's people
with the sword (v:11). All of these things, Ezekiel prophesied that he would do, but in reality
Nebuchadnezzar accomplished none of them, because he was never able to capture the island
city.

That Ezekiel intended for these predictions to apply to the island stronghold of Tyre and not
to the mainland villages is quite evident from verse 8: "He will slay with the sword your
daughter villages in the field; he will heap up a siege mound against you, and raise a defense
against you." Clearly, the "daughter villages in the field," i.e. the villages on the mainland,
were not included in the pronoun you, which was a reference to Tyre proper, the island
stronghold. Ezekiel predicted that the mainland villages would be destroyed and then
Nebuchadnezzar would turn his army against Tyre proper, batter down its walls and towers,

Volume 1990 - 2002 Issue


Page 835 of 2049
Skeptical Review Edited by Farrell Till
and enter the city with his horses, horsemen, wagons, and chariots. But it just didn't happen.
The prophecy failed.

A distinctive part of the Tyre prophecy is Ezekiel's prediction that Tyre would be destroyed
and never built again. Furthermore, this prediction was in the same context with the prophecy
that Nebuchadnezzar would destroy the city, so we can chalk this up as another failure in the
Tyre prophecy: "I will make you like the top of a rock; you shall be a place for spreading nets,
and you shall never be rebuilt, for I Yahweh have spoken" (26:14). Ezekiel was so confident
that Tyre's destruction would be permanent that he repeated this prediction twice (26:21;
27:36), but even Hogan admits that the permanent destruction of the city never happened.
Various attempts have been made to explain away this failure, but Hogan's is perhaps the
most ridiculous of them all. He argues that the final and permanent destruction of Tyre will
occur when Christ returns.

To see the absurdity of Hogan's position, we need only examine what the New Testament
teaches about the second coming of Jesus. At this time, so the New Testament claims, the
earth itself will be destroyed:

But the day of the Lord will come as a thief in the night, in which the heavens will
pass away with a great noise, and the elements will melt with fervent heat; both the
earth and the works that are in it will be burned up. Therefore, since all these
things will be dissolved, what manner of persons ought you to be in holy conduct and
in godliness (2 Peter 3:10-11)?

I certainly don't believe there will ever be a second coming of Jesus (assuming even that there
was a first coming), but I would think that Hogan does. If so, he surely believes what the
passage above says: the earth and the things that are in it will be dissolved with fervent heat
when Jesus returns. If this is so, it would have been utterly absurd for Yahweh to predict
through Ezekiel that he would destroy Tyre permanently when Jesus returns, because not just
Tyre but every other city on earth would also be destroyed permanently at that time (if New
Testament eschatology is true). Furthermore, Ezekiel had Yahweh declaring, "I will make you
[Tyre] like the top of a rock; you shall be a place for spreading nets, and you shall never be
rebuilt" (26:14). Obviously, if the earth and all of the elements in it have been dissolved, then
it wouldn't be possible for Tyre to be rebuilt, and it couldn't very well become a place for
spreading nets, because if the ground on which it had been built didn't even exist, nets
couldn't be spread on it. Nets, in fact, wouldn't exist either. So if Yahweh knew that Tyre
would not be permanently destroyed until the end of time, why did he make such inane
statements as these?

The fact is that Ezekiel's prophecy against Tyre makes no mention of the end of time or the
second coming of Jesus. All of this is simply an arbitrary interpretation that is conveniently
unfalsifiable, because if the earth should stand for billions of years, people like Hogan could
argue that the Tyre prophecy will yet be fulfilled when Jesus returned. I would like to think
that humanity will have progressed beyond crass superstitious nonsense by that time, but the
more I deal with biblical fundamentalism, the more I wonder if it will ever disappear
completely.

Volume 1990 - 2002 Issue


Page 836 of 2049
Skeptical Review Edited by Farrell Till
As for the switch to the first-person pronoun I in the Tyre prophecy, Hogan makes entirely
too much of this. It was not a switch but merely a return to the first person with which the
prophecy had begun: "Therefore, thus says Yahweh God, `Behold, I am against you, O Tyre"
(v:3). After then describing in third person the devastation that Nebuchadnezzar would bring
upon Tyre, the narrative turned to the first person ("I will put an end to your songs") as a
means of showing that Nebuchadnezzar would merely be the agent through whom Yahweh
would destroy Tyre. If Hogan will take the time to examine the book of Ezekiel more
carefully, he will see that this narrative style was characteristic of Ezekiel's prophecies.
Ezekiel 28:6-10 and 30:10-12 are just two examples of where Ezekiel began a prophecy with
Yahweh speaking in the first person after which he shifted to third-person narrations to
describe the agencies of punishment that he would use, and then he returned to first-person
narration by Yahweh.

Hogan's interpretation of the Tyre prophecy is just another frantic grasping for straws by a
biblicist who can't admit that the Bible is not inerrant. If he wants to, Hogan may respond to
this article.

How Many Women Went to the Tomb?


by Roger Hutchinson
To the casual reader, the Gospel accounts can appear confusing when they describe events
following the resurrection of Jesus. The confusion is understandable. Each Gospel describes
unique aspects of the resurrection story and each provides information not contained in the
others. None of the Gospel accounts ties everything together in one neat little bundle.

One difficulty people encounter concerns the number of women who went to the tomb to
prepare the body of Jesus for burial following the crucifixion. In Matthew, only Mary
Magdalene and the other Mary are explicitly identified as going to the empty tomb. John
names only Mary Magdalene. Luke tells us that a group of women went to the tomb.
Harmonizing the Gospel accounts is the purpose of this article.

Luke begins his Gospel stating that "It seemed good to me also, having had perfect
understanding of all things from the very first, to write unto thee in order...That thou mightest
know the certainty of those things, wherein thou hast been instructed" (Luke 1:3-4). Since
Luke provides an orderly, historical account of the life of Jesus, we will start our investigation
there.

Early in his Gospel, Luke tells us that certain women followed Jesus.

And it came to pass afterward, that [Jesus] went throughout every city and village...
and the twelve were with him, And certain women, which had been healed of evil
spirits and infirmities, Mary called Magdalene...And Joanna the wife of Chuza Herod's

Volume 1990 - 2002 Issue


Page 837 of 2049
Skeptical Review Edited by Farrell Till
steward, and Susanna, and many others, which ministered unto him of their substance
(Luke 8:1-3).

Luke tells us that Mary Magdalene, Joanna, and Susanna were among many other women
who ministered to Jesus' needs. Some or all of these women followed Jesus everywhere that
He went. As Jesus was led away to be crucified, Luke states that "there followed him a great
company of people, and of women, which also bewailed and lamented him" (Luke 23:27).
Later, as Jesus hangs on the cross, Luke tells us that "all his acquaintance, and the women that
followed him from Galilee, stood afar off, beholding these things" (Luke 23:49).

Matthew and Mark confirm what Luke has said on this point.

Matthew 27:55-56: And many women were there beholding [the crucifixion] afar off,
which followed Jesus from Galilee, ministering unto him: Among which was Mary
Magdalene, and Mary the mother of James and Joses, and the mother of Zebedee's
children.

Mark 15:40-41: There were also women looking on [the crucifixion] afar off: among
whom was Mary Magdalene, and Mary the mother of James the less and of Joses, and
Salome; (Who also, when he was in Galilee, followed him, and ministered unto him;)
and many other women which came up with him unto Jerusalem.

Matthew, Mark, and Luke each state that many women observed the crucifixion and that
among these women were those who had followed Jesus from Galilee. It is unlikely that the
women who had followed Jesus and ministered to His needs throughout His ministry could
suddenly become disinterested and exhibit no concern for His proper burial following the
crucifixion. These women followed Jesus as He was taken to be crucified, watched as Jesus
was crucified, saw those who took the body of Jesus from the cross, observed where He was
taken, and then returned to the tomb following the Sabbath to provide Jesus with a proper
burial. To suggest that all but a very few of these women would lose interest in Jesus at His
crucifixion and begin to drift away without concern for where He would be buried or that He
receive a proper burial makes no sense. We should expect, therefore, that most, if not all, of
these women either went to the tomb to prepare the body of Jesus for burial on the Sabbath
following the crucifixion or contributed to that effort.

In Matthew, Mark and Luke, we find several women identified. They are Mary Magdalene,
Joanna, Susanna, Mary (the mother of James and Joses), and Salome, who may also be the
mother of Zebedee's children. In all, five or six women are identified, in some manner, from
among the "many" women who followed Jesus. In addition, Mark tells us that Mary
Magdalene, and Mary the mother of James the less and of Joses, and Salome followed Jesus
from Galilee. From Luke's account, this group may also have included Joanna and Susanna.
There may have been others. It is the women who followed Jesus from Galilee who become
the focus of attention in the Gospel accounts of events following the crucifixion.

When the body of Jesus was removed from the cross, Luke records:

Volume 1990 - 2002 Issue


Page 838 of 2049
Skeptical Review Edited by Farrell Till
And the women also, which came with him from Galilee, followed after, and beheld
the sepulchre, and how his body was laid. And they returned, and prepared spices and
ointments; and rested the sabbath day according to the commandment (Luke 23:55-
56).

The women whom Mark specifically identified as those who were from Galilee and
ministered unto Jesus are described by Luke as observing the place were the body of Jesus
was placed. It is they who prepare or oversee the preparation of the spices and ointments.
Luke does not tell us how many there were. However, it is these women to whom Luke refers
when he begins his account of the events occurring after the resurrection of Jesus.

Now upon the first day of the week, very early in the morning, they came unto the
sepulchre, bringing the spices which they had prepared, and certain others with them
(Luke 24:1).

The women who followed Jesus from Galilee are those who now come to the tomb bringing
spices. In addition, certain other women came with them. Luke identifies two unique groups
of women going to the tomb: the women that followed Jesus from Galilee and certain others
with them.

Later, after Jesus had appeared to the women and they had returned to the apostles, Luke says,
"It was Mary Magdalene, and Joanna, and Mary the mother of James, and other women that
were with them, which told these things unto the apostles" (Luke 24:10).

Still later, Luke tells us about two men on the road to Emmaus who meet Jesus, but not
recognizing Him, recount what had happened.

Yea, and certain women also of our company made us astonished, which were early at
the sepulchre; And when they found not his body, they came, saying, that they had
also seen a vision of angels, which said that he was alive. And certain of them which
were with us went to the sepulchre, and found it even so as the women had said: but
him they saw not (Luke 24:22-24).

Within Luke's Gospel, we find a consistent reference to many women during the key events
surrounding the resurrection. Jesus healed some of these women, and they followed Him
during His ministry and were present at His death. They and others came to the tomb
following the crucifixion and Jesus met them soon after. All of these women then told the
apostles about the empty tomb and how they had seen Jesus alive.

As noted earlier, Matthew and Mark confirm Luke's account that many women followed
Jesus. Matthew and Mark also provide us with additional information not included by Luke
concerning the resurrection.

And [Joseph of Arimathaea] bought fine linen, and took him down, and wrapped him
in the linen, and laid him in a sepulchre which was hewn out of a rock, and rolled a
stone unto the door of the sepulchre. And Mary Magdalene and Mary the mother of
Joses beheld where he was laid. And when the sabbath was past, Mary Magdalene,

Volume 1990 - 2002 Issue


Page 839 of 2049
Skeptical Review Edited by Farrell Till
and Mary the mother of James, and Salome, had bought sweet spices, that they might
come and anoint him. And very early in the morning the first day of the week, they
came unto the sepulchre at the rising of the sun (Mark 15:46-16:2).

And when Joseph had taken the body, he wrapped it in a clean linen cloth, And laid it
in his own new tomb, which he had hewn out in the rock: and he rolled a great stone to
the door of the sepulchre, and departed. And there was Mary Magdalene, and the other
Mary, sitting over against the sepulchre. In the end of the sabbath, as it began to dawn
toward the first day of the week, came Mary Magdalene and the other Mary to see the
sepulchre (Matt 27:59-61; Matt 28:1).

Mark tells us that Mary Magdalene, Mary the mother of Joses, and Salome prepared the
spices and carried the spices to the tomb to anoint the body of Jesus. These are the women
that Mark earlier identified as having followed Jesus from Galilee (Mark 15:41). Luke agrees
telling us that the women from Galilee brought the spices (Luke 23:55-24:1). Luke added that
other women also came to the tomb. Mark said that their purpose was to anoint the body of
Jesus. Luke implies this (Luke 23:56).

Mark and Luke agree in their accounts. Mark, however, chose to name certain women who
came to the tomb from among the group of women that Luke said were there. This does not
mean that there were no other women present. If we read in the newspaper that the president
visited Bosnia, we do not understand that he went alone. The president is just the key person
on whom we are to focus attention. Although not stated, we know that the president does not
travel alone but that he travels with staff and secret service agents. In the same way, Mark
draws our attention to certain women. Earlier Mark had said that other women were present at
the crucifixion (Mark 15:41), so the presence of other women to help prepare Jesus' body for
burial would be realistic.

Mark also tells us that Jesus appeared first to Mary Magdalene after His resurrection (Mark
16:9). He provides no further detail. Only John provides further information about this. This
indicates that Mark was not describing every event that had happened even though he had
knowledge of other events.

Matthew's account is somewhat different from Mark and Luke. Matthew makes no mention of
the spices or of the desire of the women to anoint the body of Jesus. Like Mark and Luke,
Matthew says that many women followed Jesus and were present at the crucifixion. He then
specifically identifies Mary Magdalene, and Mary the mother of James and Joses, and the
mother of Zebedee's children (Matt 27:56), who is likely Salome from Mark's account (Mark
15:40) as being among those women.

Matthew, like Mark, draws our attention to certain women, in this case, Mary Magdalene and
the other Mary. Matthew says that they watched where the body of Jesus was placed (Matt
27:61). Later, while other women have come to the tomb to anoint the body of Jesus with
spices, it is these two women whom Matthew identifies as having an additional purpose.
Matthew describes them as wanting to "see" the tomb. Why does Matthew do this? Matthew
seems to be providing proof that Jesus was dead. To do this, Matthew identifies two
eyewitnesses to the events. It was on the testimony of two witnesses that truth would be

Volume 1990 - 2002 Issue


Page 840 of 2049
Skeptical Review Edited by Farrell Till
established (Matt 18:16; 2 Corinth 13:1). Thus, Matthew's account of the two eyewitnesses
establishes the truth of Christ's death and therefore, of the resurrection. This ties in with
Matthew's account of the guard being placed at the tomb, the guard becoming as dead men
when the angel appeared, and of the story spread by the guards that the disciple's had stolen
the body of Jesus. Only Matthew provides this information (Matt 28:11-15). His purpose
seems to be to counteract the stories being spread about the disappearance of the body of
Jesus.

Commentators indicate that Mark was the first Gospel written and that Matthew probably
wrote his Gospel with the knowledge of Mark's Gospel and with the intent of complementing
what had already been written. Similarly, Luke was likely aware of Matthew and Mark when
he wrote his Gospel. All three Gospels agree that many women followed Jesus and were
present at the crucifixion. Luke describes how two groups came to the tomb to prepare the
body of Jesus for burial. Mark identifies Mary Magdalene, Mary the mother of James and
Joses, and Salome as the group who prepared the spices for the burial. Matthew identifies two
women, Mary and the other Mary, as eyewitnesses. From these accounts, we can be certain
that Jesus died on the cross and that He was resurrected three days later.

In the Gospels, we have different perspectives on the events following the resurrection of
Jesus. Each Gospel provides a focus on unique aspects of those events. It is only after we
combine all the accounts into one that we are able to understand how these events unfolded.
In doing this, we see that no contradictions exist concerning the number of women going to
the tomb following the crucifixion of Jesus.

(Roger Hutchinson, 11904 Lafayette Drive, Silver Spring, MD 20902; e-mail


rhutchin@aol.com)

Not Enough to Make the Stories Convincing


by Farrell Till
Roger Hutchinson returns to our pages with a question: "How many women went to the
tomb?" Before addressing the points he made in a labored attempt to show that the
resurrection narratives are consistent concerning the number of women who went to the tomb
on resurrection morning, I want to answer his question. How many women went to the tomb?
Not nearly enough to make the stories convincing.

The problem for Hutchinson and every other defender of the resurrection is not just the
inconsistencies in the story but the very nature of the claim itself. They are claiming that a
man died, literally, and that he was revivified after lying stone-cold dead in his tomb for two
days. Such a claim is so extraordinary that no rational person can believe it without good solid
evidence. So whether one woman or two women or four or five or six went to the tomb and
found it empty and later claimed that they saw this dead man alive again is not good solid

Volume 1990 - 2002 Issue


Page 841 of 2049
Skeptical Review Edited by Farrell Till
evidence, especially when the testimony of these women exists only in the form of hearsay
evidence. If, for example, we should be told by someone who was not present when it
allegedly happened that five or six people had had direct contacts with alien beings in a flying
saucer, would we be willing to accept hearsay testimony that such an extraordinary event had
actually occurred? The claim is so exceptional that only the very gullible and credulous would
be willing to believe it.

Briefly, this is the real problem that confronts Roger Hutchinson and all who would expect us
to buy the story of their resurrected savior. They must show us strong evidence that this
extraordinary event really happened, and the mere resolution of discrepancies in the accounts
of how many women went to the tomb is not enough to do it.

Hutchinson, nevertheless, has chosen to address this rather minor issue, so I will follow him
where he has led us. He has, in effect, gambled everything on an old fundamentalist quibble
that argues that if there were three or four or more, then there was one. Here is an example of
how it works. Mark (10:46-52) and Luke (18:35-43) claimed that Jesus healed a blind beggar
at Jericho, but Matthew (20:29-34) said that Jesus healed two blind men at Jericho. When this
is presented to inerrantists, they will brush it aside with, "Well, if there were two, then there
was one, and Mark and Luke just chose to tell about one of them." So in the case of the
resurrection stories, they argue that there is no inconsistency in the number of women who
went to the tomb. Matthew mentioned two women: Mary Magdalene and the other Mary;
John mentioned only Mary Magdalene. Mark mentioned three women: Mary Magdalene,
Mary the mother of James, and Salome; and Luke mentioned at least five: Mary Magdalene,
Joanna, Mary the mother of James, and "other women." The inerrantist argument is that if
there were five, then there were three; if there were three, then there were two; if there were
two, then there was one. Their conclusion is that there is no contradiction.

Not surprisingly, Mr. Hutchinson used this same argument. "If we read in the newspaper that
the president visited Bosnia," he said, "we do not understand that he went alone," but the
fallacy of false analogy is inherent in this example. Anyone who knows anything at all about
the presidency of the United States knows that wherever the president goes, he is always
accompanied by an entourage of advisors, translators, secretaries, secret service agents, etc.
The presence of other people, then, is necessarily implied when one refers to a trip that the
president made. Let's suppose, however, that someone should say that Farrell Till went to
Huntsville, Alabama, last June. How many would know if I had gone alone or if other people
had accompanied me? The fact is that I traveled alone. There is nothing in the statement that
implies in any way that other people went with me, so anyone who heard this statement would
probably assume that I made the trip alone. This example is more analogous to the
resurrection stories than is Hutchinson's example of the president's trip to Bosnia. I know
nothing in the New Testament that would necessarily imply that wherever Mary Magdalene
went, she was always accompanied by someone else.

Even Hutchinson understands the difference between a reference to the president and a
reference to just an ordinary person, because he went on to say, "Although not stated, we
know that the president does not travel alone but that he travels with staff and secret service
agents." He then said that Mark, in the same way, "draws our attention to certain women," but
this is a resort to the fallacy of false analogy, because "certain women" who lived two

Volume 1990 - 2002 Issue


Page 842 of 2049
Skeptical Review Edited by Farrell Till
thousand years ago are not the president of a modern country. People today know that the
constant threat of terrorism and assassination makes it necessary for traveling presidents and
prime ministers to be protected by body guards and to be accompanied by staff members to
assist them with work that they couldn't possibly do by themselves. That certainly wasn't true
of three women who decided to go to a cemetery two thousand years ago. Had Mark been the
only one to write a gospel account, no one today would have any reason even to suspect that
more than three women had gone to the tomb. If Matthew had been the only gospel account,
no one would have suspected that more than two women had gone to the tomb, and, of course,
if John's account had been the only one ever written, everyone would think that only Mary
Magdalene had gone to the tomb. That is the hard reality of this matter, and no amount of
twisting and distorting and speculating can change that fact.

When biblicists say that Matthew "chose" to focus on just two women but Mark "chose" to
focus on three, this is apologetic sophistry gone to seed, because it ignores a central point of
their inerrancy doctrine. The Bible is inerrant, they argue, because it was verbally inspired of
God, and God cannot make errors. Very well, if there is any truth to this inspiration claim, it
would be improper to argue that Mark "chose" to do such and so, whereas Matthew "chose" to
do otherwise, because if they were both verbally inspired by the same omniscient, omnipotent
deity to tell the same story, there would be no reason why, in the case of the resurrection
accounts, this deity would have Matthew "focus" on just two women but would have Mark
focus on three. The fact is that there is nothing in Matthew's, Mark's, or Luke's accounts that
would give readers any reason to suspect that special attention was being given to any of the
women they mention. Matthew mentioned Mary Magdalene and the other Mary, and Mark
included these same two plus Salome. However, Mark mentioned not a single thing that
Salome did that made his "focus" on the story such that it was necessary to include her in the
list of women that went to the tomb. The same is true of Luke's account. He included Joanna,
who wasn't mentioned in either Matthew or Mark, but he said nothing about Joanna beyond
the mere listing of her name to give any indication that his slant to the story made it essential
to include Joanna. I defy Mr. Hutchinson to show us anything attributed to Salome and
Joanna that made it imperative for Mark and Luke to mention these two women whom
Matthew omitted from his account.

I suspect that Hutchinson recognized this problem, because he said this of Matthew's "choice"
to mention just two women:

Later, while other women have come to the tomb to anoint the body of Jesus with
spices, it is these two women whom Matthew identifies as having an additional
purpose. Matthew describes them as wanting to "see" the tomb. Why does Matthew do
this? Matthew seems to be providing proof that Jesus was dead. To do this, Matthew
identifies two eyewitnesses to the events. It was on the testimony of two witnesses that
truth would be established (Matt 18:16; 2 Cor. 13:1).

Here is another case of a Christian apologist leaning over backwards to try to explain away a
glaring problem in the biblical text. Matthew just wanted to make sure he satisfied the biblical
law that two witnesses would be needed to establish the truth of a claim. Hence, he chose to
mention only two women. Well, let's just take a look at the scriptures that Hutchinson cited.

Volume 1990 - 2002 Issue


Page 843 of 2049
Skeptical Review Edited by Farrell Till
Matthew 18:15-16, "Moreover, if your brother sins against you, go and tell him his
fault between you and him alone. If he hears you, you have gained your brother. But if
he will not hear, take with you one or two more, that `by the mouth of two OR three
witnesses very word may be established."

2 Corinthians 13:1, "This will be the third time I am coming to you. `By the mouth of
two or three witnesses every word shall be established.'"

We see that both of these texts state that two or three witnesses were necessary to establish
the truth of a statement. Both of these passages were referring to the Old Testament law that
said, "One witness shall not rise against a man concerning any iniquity or any sin that he
commits; by the mouth of two or three witnesses the matter shall be established" (Dt. 19:15).
This principle is also alluded to in 1Timothy 5:19 and Hebrews 10:28, both of which mention
the need of two or three witnesses to establish truth.

In this respect, we see that Mark, who mentioned three women, came closer to fulfilling the
requirements of this law than did Matthew. If Matthew was indeed trying to respect this law,
why did he "focus" on just two women if he knew that as many as three had gone to the
tomb? The fact is that Mr. Hutchinson is merely grasping straws again to try to find some way
to explain problems in a text that he believes is inerrant. Anyone with common sense would
know that exceptional claims, like the resurrection of a dead man, need exceptional proof, so
if there are four or five witnesses to such an event, who would try to convince others that the
event really happened by intentionally failing to mention some of those who were present
when it happened? Let's apply this to the claim of an encounter with alien beings. If five
people were on the scene when a flying saucer landed and alien beings disembarked, what
rational person, trying to convince others that this had happened, would cite only one person
as a witness or two persons as witnesses if he knew that five people had actually been on the
scene?

In the matter of witnesses to extraordinary events, we can say that the more the better is
definitely an important factor. What prosecutor would rather have just one eyewitness to
testify in a criminal trial rather than five or six? Even the apostle Paul recognized the
importance of this principle, because in his famous defense of the resurrection, he listed those
who had seen the resurrected Jesus individually and in small groups and included a claim that
"five hundred brethren" had also seen him (1 Cor. 15:6). If Roger Hutchinson had written this
passage, I suppose he would have mentioned only two people who had seen Jesus after his
resurrection and would have considered that sufficient to establish his case.

There is just no logic to Hutchinson's apologetic effort to make sense of this discrepancy in
the resurrection narratives. If five women were on the scene when the events recorded in the
resurrection narratives allegedly happened, any rational writer trying to convince the world
that the exceptional events claimed in those narratives had actually happened would surely
have mentioned all five of the witnesses. That the four writers were all just uninspired men
writing from the perspective of different traditions and legends is a much more logical
explanation of this discrepancy.

Volume 1990 - 2002 Issue


Page 844 of 2049
Skeptical Review Edited by Farrell Till

The Internet for Nontheists


Jeffery Jay Lowder
I never cease to be amazed when I hear a fundamentalist Christian inform me that "there is a
secular humanist conspiracy to take over the government and the schools" or that "the United
States is an anti-Christian nation." These arguments have been discussed before;
consequently, I will not give a refutation of them here. Rather, I shall assume that the
converse is true and that the United States is dominated by Christians who collectively have
resources in orders of magnitude greater than that of nontheists. My contention is that, given
their relative lack of resources, the Internet is the one medium where nontheists can compete
with theists on an equal footing.

When Brett Lemoine and I started Internet Infidels in the Spring of 1995, the religious
landscape on the Internet was relatively bare. Sure, there were some Internet resources for
Christians and other theists, but nothing like what we find today. Since that time, we have
witnessed the proliferation of theistic sites on the World Wide Web (WWW). Now, virtually
every religion imaginable either has or will soon have a home page on the World Wide Web.
Christianity is especially prolific. Those Christian ministries which already have a home page
include Campus Crusade for Christ, the Society of Christian Philosophers, the Southern
California Center for Christian Studies, the Institute for Creation Research, Ravi Zacharias
International Ministries, and Reasons to Believe, to name just a few. Moreover, these sites
often give the impression that only Christians have home pages on the World Wide Web;
theistic sites which acknowledge secular sites are extremely rare.

Are there secular resources on the Internet? Yes. Rather than list all of the numerous home
pages, e-mail lists, and other resources available, I will simply mention the Secular Web,
maintained by Internet Infidels. Starting from the Secular Web (http://www.infidels.org), one
can easily find his/her way to any other secular resource on the Internet. With over 250
megabytes of freethought information, the Secular Web is easily the largest site on the World
Wide Web. Along with the complete collection of freethought works scanned in by Emmett
Fields, I am proud to declare that this magazine, The Skeptical Review was one of the first
items to be uploaded to our site (TSR is available on the Web at
http://www.infidels.org/library/magazines/tsr/). The Secular Web now hosts an impressive
array of national organizations, periodicals, and authors. In conjunction with designated
representatives, the Secular Web has hosted or still hosts American Atheists, the Council for
Secular Humanism, and the Freedom From Religion Foundation. From the Secular Web, one
can also find Free Inquiry, The Skeptical Review, and Truth Seeker magazine. Our library
boasts the writings of many famous freethought writers, both past and present. In our
historical section, one can find pointers to the writings of Charles Bradlaugh, Charles Watts,
Bertrand Russell, Joseph McCabe, and Thomas Paine; we also have the complete works of
Robert Ingersoll. In our modern section, we have works by Dan Barker, Fred Edwords, Annie
Laurie Gaylor, Jim Lippard, Michael Martin, Quentin Smith, Vic Stenger, Farrell Till, and

Volume 1990 - 2002 Issue


Page 845 of 2049
Skeptical Review Edited by Farrell Till
Frank Zindler. (In the future, we plan to add material by Keith M. Parsons, Robert M. Price,
and George H. Smith.)

Whereas nontheists cannot compete with theists on TV, radio, or in print, we do have a level
playing field on the Internet. For as little as $20/month, one can establish a home page on the
WWW that is accessible worldwide by anyone with Internet access. Thus establishing an
Internet presence is not only an expense even nontheists can afford; it is an opportunity we
cannot afford to miss. All of the organizations, periodicals, and individual authors we host
have reported a huge increase in contacts since they were first listed on the Secular Web. (I
know from personal experience that I forwarded so many requests for a free subscription to
TSR to Farrell Till that I have lost count.)

If you are not a theist and do not yet have an Internet account, I promise you that it will be
well worth the time and expense to get one.

Footnotes

[1] Berry, Robby Alyn and Jeffery Jay Lowder, eds. Life In Our Anti-Christian America.
http://www.infidels.org/misc/humor/lioaca.html.

[2] Lowder, Jeffery Jay. Theistic Sites Which Link to Secular Sites. Internet Infidels
Newsletter, April 1996. http://www.infidels.org/infidels/newsletter/1996/april.html.

(Jeffery Jay Lowder, 4440 Burton Way #815, Colorado Springs, CO 80918; e-mail,
jlowder@frii.com)

EDITOR'S NOTE: The growth of The Skeptical Review is unquestionably due more to the
work of Jeff Lowder and Brett Lemoine than any other single factor. Ever since TSR was
posted on the freethought web site, subscription requests have come in from all over the
world. The secular web deserves the support of all freethinkers who are tired of the influence
that fundamentalist Christianity has in our country.

The Jury Is In

Another of Jeff Lowder's contributions to the secular web is a response to Josh McDowell's
book Evidence That Demands a Verdict. The chapters were written by various skeptics and
will be a useful resource for freethinkers who have contacts with McDowell admirers. The
Jury Is In can be accessed at http://www.infidels.org/library/modern/jeff_lowder/jury/.

Complaints About a Lot of Things


David Conklin

Volume 1990 - 2002 Issue


Page 846 of 2049
Skeptical Review Edited by Farrell Till
Please remove my name from your mailing list. I do not find that your publication meets my
expectations or needs. I had been led to believe by a supporter of yours on AOL that your
publication would point out Bible "contradictions," etc. Instead, I find that your publication
seems to be having a running controversy with the Biblical inerrantists. I am also very greatly
disappointed at the level at which the argumentation takes place, especially yours, since you
claim to be more rational and logical than those who believe in the Bible.

For instance, in your latest issue you lead us to believe, in the first column, on the first page,
that you are going to present 3 articles by believers defending the 430 year sojourn in Egypt
vs. the alleged discrepancy with Exodus 6. Instead, I find only two articles and only one of
these deals with the 430 year sojourn and doesn't even deal with Exodus 6! And you have the
gall to complain about their supposed "discredited method of argumentation"! Okay, Pa
Kettle, whatever you say!

You claim that "all offered explanations, yet all are different" and yet you fail to even present
all of the ideas so that we can judge for ourselves.

In my dealings with so-called Bible critics I find that it isn't just the biblical inerrantists [who]
seem to be infected with an any-interpretation-will-do virus." And you are quite correct that
"to those who belong to this any-interpretation-will-do school of hermeneutics, truth is not
important. The only thing that matters to them is the inerrancy doctrine, and it must be
protected at all costs, even at the cost of common sense and intellectual integrity." Even those
who criticize the Bible fall under this criticism. For if you deny the inerrancy of the Bible
their whole argument falls flat. In fact, when you ask the critics to define the terms according
to Scripture they never return or respond. Or, on rare occasions, they will persist in attempting
to use 20th century, Western man's definitions which is ludicrous. Just ask any cultural
anthropologist whose definitions and meanings they use when attempting to understand a
different culture. And in this case, we are dealing with a culture that is also widely separated
from us in space and time.

Also, on page 9 you cite Acts 13:17-21. Then on page 10 you claimed that Paul said the
judges ruled for 450 years. If instead of taking the overly simplistic approach of assuming that
the KJV is the only reliable, or "real," translation (which is typical of Bible critics), or, even
better if you had checked out the text in some scholarly commentaries, you would have found
that the judges came some time after, not during, the 450 years (which is also given only as a
"round number," an approximation). And, on page 10 you point out that 40 years "always
seemed like a good round number for biblical writers to use" and then you use it as if it were
set in concrete! Your whole argument is based on the assumption that the "round" numbers
were absolutely infallible and can therefore be used to establish an accurate chronology.
Would you attempt a launch into space if you knew that the rocket design and computer
calculations were based on the assumption that pi is equal to 3?

By the way, your whole argument against the inerrancy of the Bible is based on the false
assumption that the text constitutes, or is, the Bible. The text is all we have to work with, but,
it is not logically necessarily "the Bible."

(David Conklin, 1235 Reaney Avenue, St. Paul, MN 55106-4011.)

Volume 1990 - 2002 Issue


Page 847 of 2049
Skeptical Review Edited by Farrell Till

Another Frustrated Inerrantist?


Farrell Till
Usually, I publish letters in the mailbag column, but Mr. Conklin has registered certain
unwarranted complaints that win him a page of his own. First, I must apologize to him,
because I somehow neglected to take him off the mailing list, so he received another issue
after requesting that his name be removed. Evidently, my oversight didn't disturb him too
much, because he read the other issue and wrote an article that I will publish later.

In his letter above, he complained that the front-page article of the May/June issue said that
three articles were being published in that issue in defense of the 430-year sojourn in Egypt,
but in reality there was only one. I have to wonder if Mr. Conklin can understand plain
language, because no such promise was made. As anyone can see by checking this article, the
paragraph he was referring to was merely summarizing what had so far been published in TSR
on this subject. It pointed out that articles on this subject by Jerry Moffitt and Roger
Hutchinson had already been published and that the current issue was presenting another one
by Wilhelm Schmitt. Mr. Conklin is either a careless reader or else he was just looking for
something to complain about. If he will read that paragraph again, I think he will see that it
reads exactly as I have just summarized it.

The second column of Mr. Conklin's letter left me thoroughly confused. He said that "if you
deny the inerrancy of the Bible their [errantists] argument falls flat," but I fail to see that this
is true. Many subscribers to TSR deny the inerrancy of the Bible, but I'm pretty sure they do
this because they see the validity of the arguments against inerrancy. How could denial of
biblical inerrancy possibly make an argument against inerrancy invalid? Does he mean by this
that he denies inerrancy? If so, why the burr under his saddle? Why would someone who
denies biblical inerrancy object to arguments that demonstrate errancy?

In this same confusing paragraph, he said that when critics are asked to "define [their] terms
according to Scripture, they never return or respond." Was he implying that I have done this?
If so, when? I can hardly respond to a complaint as ambiguous as this. If he will cite a specific
case of when I have been asked to define my terms "according to scripture," I will be glad to
address his complaint.

He complained that I had quoted the KJV on page 9 when I referred to Paul's speech at
Antioch of Pisidia [to show a discrepancy in Paul's math and the claim in 1 Kings 6:1 that
construction on the temple had begun 480 years after the exodus], but if he had looked at the
passage carefully, he would surely have seen that the language in the quotation is too modern
to be from the KJV. It is, in fact, from the New King James Version. I certainly don't think
that the KJV is the only "reliable" translation. As a matter of fact, I am aware of many flaws
in this translation, and Conklin may be surprised to learn that even when I was a
fundamentalist preacher in the '50s, I didn't use the KJV. My preferred version was the ASV.

Volume 1990 - 2002 Issue


Page 848 of 2049
Skeptical Review Edited by Farrell Till
Conklin said that if I had "checked out the text in some scholarly commentaries," I would
have found that the judges had come "some time after, not during, the 450 years." Well, first
of all, I wonder if Mr. Conklin thinks that I just got up one morning and decided that I would
start publishing The Skeptical Review, not having given any time at all to researching the
subject. The truth is that I put about 25 years into seriously researching biblical inerrancy
before I began publishing The Skeptical Review. That research involved reading the
apologetic works of such "scholars" as Josh McDowell, Gleason Archer, William Arndt, John
Haley, and others, so I am well aware that there is no such thing as a biblical discrepancy that
confirmed inerrantists have not "explained" with some kind of how-it-could-have-been
hypothesis. The discrepancy between Paul's speech and 1 Kings 6:1 is no exception. I know
what the experts have said, but I know too that their "explanations" are unsatisfactory.

I'm not so sure that Mr. Conklin knows what the apologists have said about this particular
discrepancy, because he contends that the rule of the judges "came some time after, not
during, the 450 years." If that is so, then the discrepancy is even greater than if we include the
rule of the judges in the 450 years. I'll leave it to him to think about it and try to figure out
how that his explanation makes the discrepancy even greater.

Finally, he complained that I had used the number 40 in calculating how long Saul had
reigned, but I used it only because Paul said in his speech that Saul had reigned 40 years
(Acts. 13:21). Is it my fault that the Bible says what it says? I really suspect that the real
problem here is that Mr. Conklin is just another frustrated inerrantist who can't refute
arguments against inerrancy, and so he chooses to complain and quibble. If he has reasonable
evidence that the Bible is inerrant, why doesn't he let us see it. I will gladly publish it.

From the Mailbag


The Skeptical Review arrived today and, I have read most of it already. I enjoyed your lead
article "Did Marco Polo Lie?" and have a few comments about the article that appeared on
page 2, "The Resurrection of Christ: Myth or Reality?" Mr. Perman is apparently a Josh
McDowell clone as I see nothing in his article I haven't seen elsewhere in McDowell's
writings. His quotation of John Singleton Copley commenting on the evidence for the
resurrection of Jesus is very similar to the quotation attributed to Dr. Simon Greenleaf in one
of McDowell's books. Greenleaf made a statement to the effect that there is more historical
evidence for the death and resurrection of Jesus than there is for any other event or person in
history. Greenleaf's credentials were then cited to add further weight to his statement. When
confronted with such a statement, all I do is ask, "Well, then, what is the evidence?"
Christians then either bring out the traditional apologetic arguments that are easily disposed of
or they have no further comeback but are no less perturbed by their lack of evidence. I point
out that we have in our possession letters written by John F. Kennedy (by his own hand),
photographs of this man called President Kennedy, film of the same, eyewitness testimony by
friends and family of the man (most of whom are still alive today) as well as contemporary
accounts written by his political enemies while he was still alive. I then ask the Christian to

Volume 1990 - 2002 Issue


Page 849 of 2049
Skeptical Review Edited by Farrell Till
produce similar evidences of Jesus of Nazareth. Of course, they cannot do this. Why then, I
ask, does Dr. Greenleaf and others go unchallenged by Christians when they make such wild
and unsubstantiated assertions? Don't Christians ever think about the claims made by their
leaders?

Mr. Perman made reference to a popular Josh McDowell claim that all the Pharisees had to do
was produce the body of Jesus and parade it down the streets in Jerusalem in a cart to refute
the disciples' claim of the resurrected Jesus. This is funny. Elsewhere in his book The
Resurrection Factor McDowell asserts that tampering with a tomb was a very serious offense
in Jewry and there were laws that would discourage would- be grave robbers. This part was
written as a refutation to the skeptics' claim that the body of Jesus might have been stolen, but
McDowell forgets this when defending the resurrection with the ridiculous story that the
Pharisees needed only to parade the body of Jesus down the streets of Jerusalem. You pointed
out, correctly, that by the time Peter allegedly began to preach publicly, the body of Jesus
would have been so badly decomposed it would have been unrecognizable. I'm glad you did
this because I have also pointed this out to fundamentalists, who nevertheless reuse the same
argument with someone else, as though nothing has ever been said about it. John Warwick
Montgomery obviously overlooked this possibility when he made his remark in the quoted
History and Christianity.

In regards to grave robbers not stealing the bodies, I beg to differ with Mr. Perman's claim. I
have heard of documented cases of body removal from grave sites, although for what
purposes I can only guess. However, I have a theory on this too. What about a handful of
disciples not related to the inner circle (say, some secret admirers, some of whom had
celebrated Jesus's entry into Jerusalem a few days before) taking it upon themselves, even at
great risk, to remove the body to preempt any further disgrace from befalling their hero and
reburying it elsewhere and then, knowing the truth of the matter, dropped out of the
movement and lived the rest of their lives quietly, never daring to tell what had really
happened because of the reprisals that would surely have come, from either the Christians
themselves or the Sanhedrin or Romans?

In regards to the stories not being hallucinations, the stories themselves are suspect, so it is
unnecessary to discuss whether the events themselves are hallucinations or not. (By the way,
thousands have seen Elvis since his supposed interment at Graceland. Of course, we could dig
him up and run some tests, but these methods of testing were unknown in Jesus's day.) Your
comments to Mr. Perman on the bold transformation of the disciples were well written. I have
pointed out the same facts to others as well and the typical response is that no one dies for
what they know is a lie. If the original disciples knew the resurrection stories were make-
believe, would they have died willingly? Well, as you made clear, there is no evidence the
disciples really were martyred (except James); there is only uncertain tradition. But even if
there were "original" disciples in the first place (as opposed to a gradual evolving of belief
among early believers), how do we know they died willingly? How do we know they didn't
go to their deaths kicking and screaming, yanking and struggling, begging for forgiveness and
renouncing their beliefs? How do we know it was for their beliefs they died and not some
political charges? Is it not possible that the magistrates may have sympathized with their
religious views but because of some political positions have been forced to act? However, had
the "disciples" died unwillingly, do you believe one minute the faithful would have been

Volume 1990 - 2002 Issue


Page 850 of 2049
Skeptical Review Edited by Farrell Till
allowed to know this? Would this information have been shared with the church members or
would a more acceptable version have been given out? It has happened before (for example,
Adolf Hitler died fighting the communists to his last breath, according to popular Nazi
propaganda). The existence and perseverance of the church was due to its popular status with
Constantine beginning A. D. 325 and upheld by threat and intimidation for the next 1500
years. Until Constantine, the church was not nearly as powerful and promising as it was once
it received the blessing of the state. There is no way we can know what would have happened
to Christianity had it not been so blessed. It may have died a slow death, but we'll never
know. You have covered the arguments well, as usual. I just had to unload a little because the
article by Mr. Perman reeks of so much sophistry I just had to say something too, even though
you did well enough on your own.

EDITOR'S NOTE: This letter was written by David Mooney who travels so frequently that a
permanent address cannot be published. Anyone wishing to correspond with him may address
letters to TSR, and we will forward them to him whenever he checks in with us. Mr. Mooney
is an ex-fundamentalist whose deconversion resulted from reading the apologetic works of
writers like Josh McDowell and Gleason Archer. He consulted them for answers to the
arguments of inconsistency that he had heard from skeptics, but in reading the "explanations"
of the apologists, he found them so illogical and far-fetched that he became a confirmed
skeptic.

Perman's article apparently struck a responsive chord with our readers. Other opinions of the
article are published below.

It is amazing to read someone like Matt Perman and discover that he still has not discovered
himself. In his lengthy article, "The Resurrection of Christ: Myth of Reality?" he has drifted
through a maze of fundamentalist theology to convince himself and others that a mortal man
(supposedly named Jesus) was crucified, died, and somehow was resurrected bodily into the
atmosphere.

He attempts to rely on certain authors of biblical obscurantism, as well as the flimsy and
definitely unreliable accounts recorded in the New Testament. If what he has written in his
article is convincing proof of a resurrected god-man Jesus, then this heretic (myself) has
become even more convinced that those who are mentally disposed to believe in the
supernatural are a threat to the sanity of this planet. Anyone could arrive at the same
conclusions as Perman, if they have invested countless hours in ruminating on invisible
demons, a nonexistent devil, fluttering angels, and a heaven overcrowded with evangelical
clergy who have lost their employment as mystical pundits of the cloth.

First of all, Mr. Perman must have an entrenched faith in the miraculous, as well as a belief
that the physical composition of matter can be altered to the extent that a supposedly dead
body can assume characteristics that allow it to defy gravity and flitter upward into the
atmosphere. A magician might attempt this maneuver of a physical body, but only with the
aid of an assistant and by a clever manipulation of props and illusion. Deception in this regard
is no different from the delusions of the religious mystic who convinces by resorting to all

Volume 1990 - 2002 Issue


Page 851 of 2049
Skeptical Review Edited by Farrell Till
manners of nonphysical impossibilities to deceive a naive public into believing the
improbable and inconceivable.

Two or three thousand years ago, such a primitive and Neanderthal perspective of existence
may have seemed normal. At present, there is no evidence even to slightly suspect differently
that those who are dead remain dead physically. As to what happens to our consciousness or
perception, I would assume that it also expires with the brain. Since no one has died, returned
to planet earth, and related their experiences in that realm of existence or nonlife, I must
remain skeptical of all religious claims to an eternal heaven or hell.

What exactly was the purpose of the Jesus-man to exhibit his body in a renewed state of
physical resurrection? And if he was capable of this impossible feat, why not stay put and
continue to convince the public that he was both the father-son and the holy ghost? Why say
goodbye and tell his disciples to continue where he left off? If a general left his troops on the
battlefield and said, "So long; you are on your own," how many would be convinced that any
further bloodshed was necessary? Most of them would throw their weapons down and walk
off the battlefield.

I also find Mr. Perman's reference to the fact or assumption that we owe allegiance to a God
because we are born with sin an irrational concept. Since when did a God (the Christian one)
find it impossible to create humans without sin, especially since the religionist attributes to
this God all the perfections and intelligence of an almighty creator? If after millions of years,
this God is still producing humans with an exposure to sin and created only one human (man-
Jesus) with immunity to this vice, I can only assume that it is imperfect. Or is this a deliberate
attempt by this God to subjugate entire populations to the rituals of Christianity?

To believe as Mr. Perman requires either a depletion of oxygen to the brain or an inclination
to paranoiac delusion and an occasional bout of psychopathic religiosity. No Jesus-man,
constructed on fiction and myth, could obtain for any living being a salvation based on the
irrational and absurd. No resurrection of a dead body is capable of physical regeneration,
whether it is a Jesus-man or any other presently dead body.

(Leland W. Ruble, 833 Orchard Street, Toledo, OH 43609).

In the July/August issue of TSR, Matthew Perman attempted to prove the resurrection of Jesus
by proffering as one of his "reasons" (#5) that Christianity required a historic cause (the
resurrection), because it "suddenly burst into life and spread like wildfire, and changed the
world."

Wrong! Christianity did not spread like wildfire; it was a very gradual process. Christianity
took centuries to develop. It wasn't until Constantine with the Edict of Milan in A. D. 312,
close to 300 years after the supposed resurrection, that Christianity was even given the status
of state religion and a full 70 years later that Theodosius made it the only recognized church.
Prior to the Edict of Milan, the hierarchy of priests and bishops of the church had had at least
two centuries to evolve. Being far removed from the actual time of the supposed resurrection,

Volume 1990 - 2002 Issue


Page 852 of 2049
Skeptical Review Edited by Farrell Till
they had plenty of time to discuss, debate, and decide what the doctrine and dogma of the
church was to be. All those not agreeing, like the Gnostics, Donatists, Arians, Monophysites,
and "god" knows how many others, were deemed heretics and eliminated from the body of
the church with the knowledge that they had even existed being extinguished by the church
"fathers" when possible.

As for the persecutions Perman speaks of, it is true that Christians were persecuted, at times
severely, but not continually. It was sporadic according to individual emperors, some of
whom were severe, but others would lift the restrictions. Actually, the Romans usually prided
themselves on tolerance of diverse religions. Then, of course, anyone that knows anything at
all about history can't deny that once the Christians were given preferred status by
Theodosius, they turned the tables and severely persecuted the other religious elements. Later,
they would persecute their own if they did not consider them "holy" enough.

As for Christianity "changing the world," it certainly did, but whether it was for the better is
very debatable. An immediate change was Rome, which was facing many problems internally
as well as externally. The northern invasion, the class struggle between the rich and poor,
being saddled with large military expenditures, were only a few of the problems facing the
Roman Empire. Some historians consider the new influence from the Christians, who were
more concerned about the afterlife than solving the problems Rome was facing at the time, to
be a major contributing factor to the decline and fall of Rome and the beginning of the Dark
Ages. This is only one small slice of how Christianity "changed the world."

Can the problems of America and the problems of Rome be compared with the rise of
fundamentalist Christianity in our time being more concerned with an afterlife and hiding
their collective heads in that sand rather than attempting to solve the problems we face today?

(Marie Micheletti, P. O. Box 335, Tremont, IL 61568-0335.)

EDITOR'S NOTE: All of these letter writers are trying to tell Mr. Perman that he didn't make
a good case for his claim that Jesus was resurrected from the dead. To call Perman's
arguments "good evidence" is ridiculous. If he wants to believe in the resurrection of a man
who may or may not have existed, that is his right, but why can't he be honest enough to
admit that he believes it because he wants to believe it rather than trying to fabricate
reasonable evidence where there is no evidence? Also, as Ms. Micheletti's letter shows,
Christians sorely need to learn a few facts about the history of their religion rather than just
parroting what they hear from the pulpit and read in the books of apologists like Josh
McDowell, Gleason Archer, and Norman Geisler.

At any rate, Mr. Perman has indicated that he will write a reply to my response. If it is
received in time, it will be published in the next issue.

I saw your address in The Book Your Church Doesn't Want You to Read and am interested in
purchasing a subscription to The Skeptical Review. I would really appreciate it if you could
send me a subscription card or let me know how to get a subscription. I was very excited to

Volume 1990 - 2002 Issue


Page 853 of 2049
Skeptical Review Edited by Farrell Till
find out that there are other people who think along the same lines I do concerning religion. I
look forward to your reply.

(Linda Vincent, 1504 Rockwood Boulevard, Mulvane, KS 67110.)

Shame on Dr. Ross [ click here for his article] for his biblical idolatry! Alas, he is using the
Bible as a ouija board or a crystal ball.

The great period of Old Testament prophecy was around 750 B. C. and included such greats
as Amos, Ezekiel, Jeremiah, and Isaiah. These men were speaking to their fellow
(contemporary) citizens that God would send down another nation to conquer them as
punishment for disobedience to God. That's the extent of their prophecy. Their central
message was righteous living, ethical conduct, and justice. The only time the future was even
considered was in a general way ("God will punish the people and nations for their sins"). It
was about moral pronouncement, not fortune-telling, Dr. Ross! We all need to become "Bible
literate" and not devoted to our own mental chatter.

When Isaiah foretold the coming of the Messiah to be called Immanuel, perhaps Dr. Ross
could take that as literally true. Then he might see that the name Jesus has been placed in our
Bible by Satan in a demonic trick!

(James Warren, 4835 California Avenue SW, Apt 304, Seattle, WA 98116.)

I've been quite amused by the "pee wee" hermeneutics of Roger Hutchinson et al, and I
thought, "Hey, I can play that game too." Let's apply the how-it-might-have-been technique to
one of the key fundamentals, i.e., the resurrection. Christians seem quite firm that it actually
happened and that the Bible evidence (which is the only evidence) is all the proof we need.
But let's consider the following scenario: Joseph of Arimathea claimed the body of Christ and
told Pilate he had a nice tomb for him but never actually placed him there. Based on the Old
Testament precedent of Moses being buried in an undisclosed location, Joseph decided to do
the same thing with Jesus. So when the women went to the tomb, they found it empty,
because he had never been there to begin with. Why not? Nobody was specified as having
actually witnessed his entombment, so Joseph might have lied. Unless fundamentalists can
prove that this didn't happen, then it must be considered a possible explanation for how belief
in the resurrection came to be.

As for all the subsequent reports of Jesus's sightings, when you consider how many people
have reported seeing Elvis in this modern day, these can likewise be discounted as the wishful
delusions of people who wouldn't accept that their idol was dead. But he was and remains
dead. If only the misbegotten movement that others founded around him would be buried as
well!

(Stephen Van Eck, Route One, Box 62, Rushville, PA 18839.)

Volume 1990 - 2002 Issue


Page 854 of 2049
Skeptical Review Edited by Farrell Till
EDITOR'S NOTE: Actually Luke did claim that "the women" followed Joseph of Arimathea
and saw the tomb and where the body was laid (23:55); however, this does not substantially
affect the point Van Eck was trying to make. Anyone could imagine all sorts of how-it-could-
have-been scenarios to explain how that belief in the resurrection of Jesus developed when
actually no resurrection took place. I'm sure that Christians, confronted with such scenarios,
would demand some reasonable evidence to support them. They certainly wouldn't agree that
since the scenarios are possible, this is sufficient to prove that the resurrection did not happen.

Marion Fox, in his discussion of vestigial organs, has made a serious error. Vestigial organs
are the remains of organs that no longer serve their original function because evolution no
longer selects for them. (The remains of hind legs in some whales are a good example.) Fox
errs in assuming that a vestigial organ cannot have a useful function. Evolution quite often
gives an old part a new function. Feathers, for example, probably evolved as a means for
insulating small animals against a cold environment, and only later became adapted for flight.
That tonsils, appendix, or thyroid may be fully functional is not an argument against their
being vestigial organs, at least in the sense that we are interested in. Fox needs to show that
these organs contain no evidence of an earlier use, a point often suggested by studies in
comparative anatomy. Pseudogenes are nonfunctional copies of working genes and, not being
preserved by the selective forces of evolution, often collect random changes. That such a unit
might later become useful in some way does not eliminate the vestigial nature of it.

After considerable e-mail with Roger Hutchinson, I have come to the conclusion that one
remarkable difference between his and Till's positions is the use of accepted rules to ensure
objectivity. Though not always stated, there are a number of rules that all good scholars
depend upon to find the most objective interpretation of a passage. One such rule requires that
we use the most common meaning of a word or phrase unless it constitutes a clear error. If
there are no other clues at our disposal, the most probable guess is the interpretation with the
best track record. The genealogies in question may certainly be taken at their most common
meaning (father-son succession) without any obvious error. Therefore, objectively speaking,
they must be so interpreted.

Another important difference between Hutchinson and Till is what is at stake. If Mr.
Hutchinson is wrong, then he must discard any idea that the Bible is the inerrant word of God.
The very existence of his view of a heavenly reward, or even of an afterlife, is threatened. His
whole world must ultimately collapse. By comparison, if Till is wrong then public
embarrassment is the worst that he might suffer. However, that would be greatly offset by his
realization that he could, with little effort, obtain an everlasting heavenly bliss. If Farrell Till
is anything like me, he is more interested in the truth than in a particular belief. If the
evidence so warranted, I could give up evolution, atheism, and all the rest. I would be greatly
surprised, and initially embarrassed, but I would welcome the truth as a friend. By
comparison, I doubt that Roger Hutchinson is presently in any position to do so. He cannot let
go of his world view, at least for now, meaning that he cannot acknowledge the power of
Till's arguments or the need to adhere to certain rules to help ensure objectivity.

Volume 1990 - 2002 Issue


Page 855 of 2049
Skeptical Review Edited by Farrell Till
I brought out these points because Hutchinson left us with the impression that he and Till are
two equally reasonable men who are at total loggerheads with no means for objectively
settling the issue. In fact, as noted, there are some striking differences between the
fundamentalist and skeptical positions, differences that leave Hutchinson on an unequal and
inferior footing.

(Dave Matson, P.O. Box 61274, Pasadena, CA 91116, e-mail


103514.3640@compuserve.com)

EDITOR'S NOTE: As usual, Dave Matson makes some excellent points. The only thing I can
add to what he has said is to point out another advantage that my position has over
Hutchinson's: I can be wrong and still be right. If, for example, Hutchinson should prove
beyond question that I am wrong in claiming errancy in the book of Exodus on the matter of
the 430-year sojourn in Egypt, he would have proved only that I am wrong on this point. He
would not have proved that I am wrong in the many other claims of biblical errancy that I
have published in this paper and argued in public debates. If I am right in any of those claims-
-just one-- then I have established that the Bible is not inerrant. On the other hand, if I prove
that Hutchinson is wrong in declaring that there is no inconsistency in the book of Exodus in
the matter of the 430-year sojourn (and I believe that I have done that), then his entire position
collapses, because the claim of biblical inerrancy cannot allow for a single discrepancy of any
kind in the Bible. Obviously, if there is an error in the Bible -- just one -- then the Bible is not
inerrant. I won't even bother to discuss the odds against Hutchinson's being right in every
position that he has taken on biblical inerrancy.

I will remind readers again that some good materials on evolution and creationism can be
obtained from Dave Matson at his address indicated above.

Please take us off your mailing list. You've misspelled Sid Roth's name (Jed), you're totally
misinformed, and one day, your [sic] gonna realize you're mistaken.

(Messianic Vision, P. O. Box 1918, Brunswick, GA 31521-1918.)

EDITOR'S NOTE: Sid (Jed) Roth was on our mailing list for a year, and I have no idea how it
got there. He either requested the subscription, or someone else requested it for him. As to
how his name was misspelled, I don't know how that happened either. It probably resulted
from illegible writing in the letter that requested the subscription. At any rate, I apologize for
the error, but I suspect that the misspelling of the name is not the reason why the writer of this
note was so upset. He seems so certain that I am mistaken, but in the year that he was
receiving TSR, he didn't attempt a single time to show exactly where I was mistaken, even
though the policy of this paper is to allow anyone who can write a competently organized
article to have free space to rebut our arguments. We never heard from Mr. Roth or
"Messianic Vision."

Volume 1990 - 2002 Issue


Page 856 of 2049
Skeptical Review Edited by Farrell Till
Finally, I get the distinct impression that if I am mistaken, the writer of this note will be
thoroughly delighted when his barbaric Yahweh begins roasting me in hell. What can I say
except that it's so ... Christian of him?

As a subscriber I feel the need to respond to the letter Dr. Woodrow Coppedge, a recovering
ex-fundamentalist sent. Dr. Coppedge expressed an idea of a Fundamentalists Anonymous
Association to help former fundamentalists recover. Approximately 15 years ago my son and
daughter, both college age at that time, got heavily involved in the born-again fundamentalist
movement. As a father, I was devastated and could not understand how two bright college
kids could blindly follow and fall for the fundamentalist line. To make a long story short,
somehow I found an organization out of New York, I believe, named Fundamentalist
Anonymous. They sent me a great deal of information on how to deal with my problem and
gave me the names of recovered and recovering Fundamentalists around the Indianapolis area
where I lived. I believe the founder was Oriental. I did in fact make contributions of several
hundreds of dollars to them, but it was worth every penny of it for my mental stability. I don't
know if they still exist or even if they were a fraud; all I can say is they were very helpful and
the idea of such an organization is a worthwhile thought to explore. I enjoy your publication
very much. The world needs you.

EDITOR'S NOTE: The writer of this letter is the president of a large retail outlet in Central
Illinois, and he has allowed me to print his letter anonymously. He wishes to avoid the
possibility of Christian boycotts of his company if his skepticism became public knowledge.

I have known about Fundamentalists Anonymous for several years, because the New York
chapter once subscribed to The Skeptical Review. About two years ago, a copy was returned
to us with a notice that the organization had moved and left no forwarding address. Does
anyone know if it still exists?

TSR has printed nothing from me but the five sentences preceeding [sic] your two column
attack in July-Aug '96. I believe your argument of the meaning of `hemera' falls, and I do
want to explain my side to TSR readers, as you said I might. Do I get not even these rebuttal
words to your attack in TSR, as your last paragraph seems to say? The following is submitted
as rebuttal for TSR:

Any that peruse my "Two Hard Challenges" article (in The Freethought Exchange #22 Jan-
Feb '96, POB 202447, Arlington, TX 96006, and subsequent issues) will surely notice it's
hardly just "an issue" I raise. It's more like hundreds, but granted, many relate to a pivotal
issue.

You have greatly misrepresented me, saying my "peculiar belief" is based on Matt 16:21, and
misled your readers by omitting the rest of the meaning of the Greek word hemera, so I
request this brief rebuttal letter be printed in TSR.

Volume 1990 - 2002 Issue


Page 857 of 2049
Skeptical Review Edited by Farrell Till
As my article noted, the basis for expecting a peculiar human who was David, and was Jesus,
and will be similar again, is primarily drawn from the obvious and direct statements of the
main "prophetical" books: Psalms, Isaiah, Jeremiah, and Ezekiel.

You stated `hemera' only meant the time between dawn and dark (or 24 hours), but my
concordance says the context should indicate if hemera is used literally or in its figurative
sense, which it lists as meaning a period or age. As Jesus apparently used it in that sense many
other places, it is reasonable to consider that possible usage of it at Matt 16:21.

What seems to be Till's main complaint with me has just disolved [sic] . What else have we?
He thinks I should be devastated he has a neighbor calling himself King David! Psst, Mr. Till,
there are more: one David Koresh, deceased, a loose [sic] dozen in NYC and Israel at any
given moment, and hundreds of others all over. I've met a number of them even thru [sic] my
reclusive studious times. What do you suggest this proves and/or disproves? I think little,
except for the curiosity of why so many connect the David name into it.

For those curious, the appearance and other issues Till and others have been raising have been
or are being addressed in TFE where we get sufficient space, and I'm trying to get a `netable'
computer.

(Willard Small, P. O. Box 705, Picabo, ID 83348.)

EDITOR'S NOTE: Usually, I correct spelling and grammatical mistakes in letters and also
revise clumsy wording, but I have published Mr. Small's as it was submitted in order to make
a point. Small wants us to believe that he is the third incarnation of King David, yet he has
problems with spelling and is apparently unable to write coherently. One would think that
God would have selected for his third incarnation of David someone who could make a more
convincing presentation of his credentials.

In the matter of what the Greek word hemera meant, Mr. Small's comments are irrelevant,
because he alleged that Jesus had used the word aion in Matthew 16:21, but in reality hemera,
not aion (age or era), was used. One would think that the third incarnation of King David
would not have made this mistake. The word hemera did have the figurative meaning of "age"
or "era," but Strong (#2250) in citing this definition said that this sense of the word was
"always defined more or less clearly by the context." In Matthew 16:21, Jesus allegedly told
his apostles that he would be killed and "raised the third day." Now would Mr. Small please
point out to us what there is in the context of this statement that "more or less clearly" defines
the word day (hemera) to mean "age" or "era"?

Small informed us that he is trying to get a "netable" computer, presumably to present his case
on the internet. My reaction to this is, "Oh, goody, now the internet lists that discuss biblical
issues will have another fruitcake to put up with." Although this comment may sound unduly
harsh, readers who are on the internet will understand what I mean. As I am writing this, for
example, subscribers to the Errancy list are having to contend with someone who bills himself
as the world's only biblical evolutionist. Central to his evolutionist theory is the notion that the
Bible is "basically inerrant" but universally misunderstood. The writers were trying to present
evolution to us in the best way they knew how. Eve was literally a primitive reptile, the

Volume 1990 - 2002 Issue


Page 858 of 2049
Skeptical Review Edited by Farrell Till
Amalekites were pterosaurs, the Amorites, Perizzites, Hittites, Hivites, and other nations that
Israel drove from the land of Canaan were not humans but populations of all sorts of animals
ranging from long-necked dinosaurs to ground-dwelling squirrels, rabbits, and badgers. (As
Dave Barry frequently says in his column, I'm not making this up.) After many attempts to
reason with the man, most subscribers on the list have decided that our only choice is to
ignore him. Like Small, however, he is absolutely convinced that he is right and the rest of us
are appallingly ignorant.

I have tried to explain to Mr. Small that I simply do not have the time to humor every
religious quack that comes along. That is why I have ignored his "Two Hard Challenges," and
it is why I will not waste any more publishing space on his nonsense.

Volume 1990 - 2002 Issue


Page 859 of 2049
Skeptical Review Edited by Farrell Till

The Skeptical Review


Volume Seven – Issue Six
November/December 1996
Farrell Till, Editor

• More About Family Values


• The Resurrection Stands Firm: A Response to Farrell Till
• Standing on Quicksand
• Jacob an Old Geezer?
• The Thrasher-Till Debate
• The Sins of the Fathers
• From the Mailbag
Letters to the editor.

More About Family Values


by Farrell Till
In our first article on family values, we left Abraham and Sarah in Gerar collecting the
proceeds from Abraham's second attempt to pass Sarah off as his sister. Each time, she caught
the eye of a king, and each time Abraham was given sheep and oxen and servants, both male
and female, but the second time King Abimelech gave him a thousand pieces of silver as a
"covering of the eyes to all that are with you" (Gen. 20:16). Hmmm, it sounds a lot like hush
money, doesn't it? But such was the way of "family values" in those days.

Volume 1990 - 2002 Issue


Page 860 of 2049
Skeptical Review Edited by Farrell Till
By and by, Abraham and Sarah had a child of their own if one can believe that a 100-year-old
man and a 90-year-old woman (Gen. 17:17) could become parents, but why not? Abraham
was 75 when he left Haran (Gen. 12:4), and the passage just referred to claimed that Sarah
was 10 years younger than Abraham. Therefore, when they went into Egypt, where pharaoh
took Sarah into his palace, she had to have been at least 65 years old (if there is any truth to
the Bible record). So if Sarah was such a raving beauty at 65 that kings lusted over her, why
not believe that she could bear a child when she was 90?

Anyway, Isaac was born to Abraham and Sarah, and when he grew up, they betrothed him to
Rebekah, the daughter of Abraham's brother (Gen. 24:15). That's the way it was done in those
days. Brother married sister; cousin married cousin. As noted in "Jacob an Old Geezer?" (pp.
7-9, this issue), Isaac's and Rebekah's son Jacob, the offspring of first cousins, even married
the daughters of Rebekah's brother Laban. (Maybe it was all of this inbreeding that made
these people think that they were always seeing God.) At any rate, Isaac and Rebekah
married, after which we find in the Bible a story that illustrates the old like-father-like-son
adage, only in this case, it was a matter of like father and mother, like son and daughter-in-
law, because Isaac and Rebekah pulled the same trick that Ma and Pa did. Not only that, they
did it in Gerar, the same place where Abraham and Sarah had pulled the wool over
Abimelech's eyes. In fact, Abimelech was still king, even though it had to have been at least
40 years later, because Isaac had not been born yet when Abraham and Sarah were in Gerar,
and Isaac didn't marry Rebekah until he was 40 (Gen. 25:20). Anyway, Isaac and Rebekah
went to Gerar, and "when the men of the place asked him of his wife, he said, `She is my
sister,' for he was afraid to say, `She is my wife,' because he thought, `Lest the men of the
place kill me for Rebekah, because she is beautiful to behold'" (Gen. 26:7). Yes, indeed,
beauty was definitely a genetic trait in this family. After all, if Sarah was Abraham's sister,
then Sarah was Rebekah's aunt. So isn't it quite believable that both women could have had
beauty that men would kill for? At least, Abraham and Isaac thought so.

Apparently Abimelech didn't learn a thing from his experience with Abraham and Sarah,
because he bought the same lie from their son and daughter-in-law. Oh, well, since it was at
least 40 years later, maybe the guy was getting senile. At least, he didn't make a play for
Rebekah himself, but he did believe Isaac's lie until one day he looked out the window and
saw Isaac sporting with Rebekah (v:8). Hmmm, Isaac was "sporting" with Rebekah in a place
public enough that the king could see them from a window? Is that any kind of example in
family values for the promised seed of Abraham to be setting for all generations to read
about?

There has been a lot of debate over what Isaac's "sporting with Rebekah" meant. Some think
that it meant... uh, well, that they were "doing it" right in public, but others argue that they
were only "displaying affection." Whatever they were doing, it was enough for King
Abimelech to guess that they were married (v:9), so it had to have been more than just a hug
or a kiss. Anyway, if kids today want an appropriate response to parents and other adults who
criticize them for "making out" in public, they can just point to this story and say, "What's the
big deal? We're just practicing old-fashioned family values that we read about in the Bible."

So Abimelech called Isaac before him, just as he had summoned his father years before, and
raked him over the coals for having lied about his relationship to Rebekah. "What is this you

Volume 1990 - 2002 Issue


Page 861 of 2049
Skeptical Review Edited by Farrell Till
have done to us?" he said. "One of the people might have lain with your wife, and you would
have brought guilt on us" (v:10). He then warned his people that anyone who touched
Rebekah would surely be put to death (v:11). There is no explanation for why a king who kept
concubines in his palace would have thought it so tragic if some of the men of the country had
"lain" with Rebekah

Times had changed somewhat since Abraham's and Sarah's sojourn in Gerar, because there is
no indication in the story that Abimelech gave any goodies to Isaac as he had to his father
Abraham. Maybe he felt that he didn't need to; after all, he hadn't made a direct play for
Rebekah as he had for her mother-in-law/aunt Sarah. Age is a terrible curse!

Isaac and Rebekah didn't exactly wind up empty-handed, though, because Isaac "sowed in
that land and reaped in the same year a hundredfold, and Yahweh blessed him" (v:12). So the
moral of this story seems to be that in those times when Yahweh routinely visited and chatted
with his chosen ones, if they lied and "sported" in public, he would bless them with
prosperity. Perhaps Christians, who go about preaching a need to return to biblical values,
should not be surprised that bribery and sex scandals so frequently disgrace our public
officials. Maybe they are just doing what they so often hear from the Christian right and are
making an honest effort to return to the family values of biblical times.

In due time, Isaac and Rebekah had children too, although it took them 20 years (Gen. 25:26).
Twin sons (Esau and Jacob) were born to Rebekah, and right away problems developed
between the brothers that make ordinary sibling rivalry seem like brotherly love. The fault
wasn't really the children's, because Isaac loved Esau, but Rebekah loved Jacob (Gen. 25:28).
Parents today might keep this in mind. If they want to return to those wonderful family values
of yesteryears, they must show favoritism to their children. It's the biblical way.

The sibling rivalry culminated one day when Esau came in from the field exhausted with
hunger. Jacob, who had prepared pottage, refused to give his brother any until Esau vowed to
give up his primogenitive rights (acquired by being the firstborn of the twins). Faint with
hunger, Esau agreed to the demand in order to get food from Jacob (Gen. 25:27-34). This was
a supreme manifestation of brotherly love biblical style.

Rebekah, who loved Jacob, wanted to make sure that he retained the birthright that Esau had
relinquished, so she conspired to disguise Jacob as his hairy brother Esau in order to deceive
Isaac (whose eyesight was now failing) into pronouncing the blessing of the firstborn on him.
This involved killing two goat kids to prepare a sumptuous meal for Isaac. Two kids? Isaac
must have been hungrier that day than Esau ever was. Anyway, Rebekah took some of Esau's
clothing and put it on Jacob, so that he would smell like his brother, and then put the goat
skins on Jacob's hands and neck (Gen. 27:15-17). To make a long story short, when Isaac
smelled Jacob and felt the goat skins on his hands and neck, the poor old guy thought that
Jacob was his hairy brother Esau and pronounced upon him the blessing. If Isaac couldn't tell
the difference in feeling goat skins and a hairy human body, his eyesight must not have been
the only thing that was failing. Anyway, that's how the story is told in God's inspired word, so
it must be true. The lesson for us is that conspiracy, deception, and lying were very much a
part of family values in biblical times. Many Christians today seem to be following this
example only too well.

Volume 1990 - 2002 Issue


Page 862 of 2049
Skeptical Review Edited by Farrell Till
When Esau learned what had been done, he vowed that when his father was dead, he was
going to kill his brother. Boy, talk about biblical family values! Rebekah heard of this (how is
anybody's guess) and arranged for Jacob to go visit her brother in Paddanaram. By this time,
Esau had married two Hittite women, who were "a grief of mind to Isaac and Rebekah" (Gen.
26:35). Ah, so we see that ethnic prejudice was also a family value in biblical times, as was
polygamy too, of course. Anyway, Rebekah used this as an excuse to get Jacob out of the
country and away from Esau. She said to Isaac, "I am weary of my life because of the
daughters of Heth. If Jacob takes a wife of the daughters of Heth, like these who are daughters
of the land, what good will my life be to me" (27:47)? That's the spirit, Becky; show contempt
for all who are not as ethnically pure as you. It's the biblical way.

Once this idea had been put into Isaac's head, he too became concerned. He called Jacob into
him and said, "You shall not take a wife of the daughters of Canaan. Arise, go to Paddanaram,
to the house of Bethuel your mother's father, and take yourself a wife from there of the
daughters of Laban your mother's brother" (28:1-2). So Jacob went to Paddanaram and
engaged in the sexual escapades described in "Jacob an Old Geezer?" on pages 7-9 of this
issue. If Isaac, Rebekah, and their two sons were living today, psychologists would call them
a dysfunctional family. But don't knock it; that's just the way family values were in biblical
times.

The Resurrection Stands Firm: A Response


to Farrell Till
by Matthew Perman
After reading Till's response to my article defending Christ's resurrection (both appeared in
the July/August issue of TSR), I was surprised to see him reuse Dan Barker's argument for the
"evolution of a myth" (Barker argued this in a debate with Michael Horner). He argued that
"the Christian belief in a bodily resurrection [of Christ] was a result of doctrinal evolution that
had begun with belief in only a spiritual resurrection." Thus, Till argued that my data are
better explained by this myth theory, not an actual, bodily resurrection of Christ. But before
responding to the Till/Barker claim for the "spiritual resurrection myth," I first want to
address some of Till's general comments concerning my article.

Till repeatedly claimed that I was "always argu[ing] from the assumption that the New
Testament records are historically accurate" when presenting the evidence to back up my six
historical "facts" that critical scholars accept. (My points included the empty tomb, the
resurrection appearances, etc.) First, we must note Aristotle's dictum that "the benefit of the
doubt should be delegated to the document itself, not arrogated by the critic to himself." What
right does Till have to operate from the extreme assumption that the New Testament is
fundamentally unreliable? He says that the gospels are "blatantly biased documents."
However, all historical records are "biased" -- no one wants a disinterested historian. But this

Volume 1990 - 2002 Issue


Page 863 of 2049
Skeptical Review Edited by Farrell Till
doesn't render their records completely unreliable. A Jew who was writing about the events of
the Holocaust would not be discredited just because he was a victim of it and passionate over
the issue, would he? In fact, this would seem to establish his credibility. There are also
numerous good reasons that do establish the general reliability of the New Testament (but
that's another article).

Second, and most important, I am emphatically not arguing from the premise that the New
Testament documents are reliable in everything. In each case where I assert something that
the New Testament claims (such as that the tomb was empty), I give specific reasons why we
should accept what it says on this specific point. For example, I appeal to Matthew 28:11-15
to show that the earliest Jewish propaganda admits the empty tomb. Till says that this is a
"flagrant assumption that Matthew's record is historically accurate." But in my article I give
two specific reasons for accepting the accuracy of this specific report from Matthew.

This leads to my next point of clarification. I specifically wrote the article to show that one
can give good evidence for the resurrection without establishing that the New Testament is
inerrant, or even very trustworthy. That's why I said that I would "examine six facts that
virtually all critical scholars... accept." Till seems to think that by "critical scholars" I mean
"most fundamentalists," but that is not the case. My article clearly says that even virtually all
"critical non-Christian scholars" who address Christ's resurrection accept my six points. I
mean that these data are accepted by serious scholars (whether Christian or not) across a
broad spectrum of beliefs who "apply to the Bible the same investigative methods that they
use in evaluating the accuracy of secular history" (this quote is from "Did Marco Polo Lie?"
on page 1 of the July/August issue, where it is claimed that Christians are afraid to apply to
the Bible the same historical methods applied to secular history). This puts my case on firm
basis, not an "unwarranted assumption" that the New Testament is "always trustworthy"
(though I believe it is). Now I can turn to Till's attempt to explain away my six facts with his
myth theory.

I'm glad that Till is willing to admit that the creed recorded by Paul in 1 Corinthians 15:3ff is
very early. We will return to this later. But he asks why such an early account would leave out
so many of the details that the gospels record. The reason is that the formula of this creed was
simply meant to be a summary, or brief outline, of the core of Christian beliefs. There are
many early, short creeds recorded in the New Testament (Rom. 1:3-4; 10:9; Phil. 2:6ff, 1 Tim.
3:16, etc.), and none of them go into great detail because they are only intended to be concise
summaries of beliefs, not detailed records as the gospels were.

Till points out that Paul used the word thapto, which means "burial." He insists that thapto
carries with it no connotation of sepulchre, or tomb, and that this probably meant that Paul
understood the body to have been thrown into a common grave. But let's look at Acts 2:29. It
reads "Brothers, I can tell you confidently that the patriarch David died and was buried
(thapto), and his tomb is here to this day (NIV)." The KJV replaces "tomb" with "sepulchre."
So for Till (following Barker) to assert that there is no connection between sepulcher, tomb,
and thapto is simply incorrect. Of course, Paul does not use mnema (tomb) or mnemeion
(sepulchre)! Would the following make sense: "He died for our sins in accordance with the
Scriptures, and was sepulchered"? Till's theory also fails to account for the wealth of evidence
supporting the burial of Jesus in a tomb. (1) Archaeology supports that the crucifixion victims

Volume 1990 - 2002 Issue


Page 864 of 2049
Skeptical Review Edited by Farrell Till
were buried in tombs. Look, for example, at the case of Yohanon, who was discovered in
Tomb #1 at Giv'at ha-Mitvar, As el-Masaref, by Tzaferis. Yohanon was from the first-
century, and was found to have his heel bones transfixed by a large iron nail and his shins
broken. "Death by crucifixion" reported Dr. N. Haas after examining him. Further, (2) Jewish
holy men (as was Jesus) were buried in tombs so their grave could be preserved, (3) the burial
story lacks legendary development (as even Bultmann agrees in The History of the Synoptic
Tradition, 2 ed., trans. John Marsh, p. 274, and most agree with him), (4) archaeology
confirms the description of Christ's tomb in the gospels and (5) the phrase "first day of the
week" reveals an early date for the story since it fell out of use by the late 30s (or so) to be
replaced with "on the third day." Lastly, (6) the inclusion of Joseph of Arimathea strongly
supports the burial record (we will see why later). This all suggests that the burial story is
very early and accurate. Since it is so early, there is simply not enough time for legend to
replace the historical core.

Till's second point was that anistimi means "to be raised," while egeiro means "to awaken"
and sometimes lacks physical connotations. Paul used egeiro, so Till reasons that Paul is
implying a spiritual resurrection. If Paul had intended a physical resurrection, says Till, Paul
would have used anistimi. Now, whether or not we agree with the gospels on the resurrection
or not, both skeptic and Christian can be certain that Matthew, Luke, and John record that
Jesus appeared physically and bodily. Matthew 28:9 says that "they clasped his feet." Luke
24:39 says, "Touch me and see, a ghost does not have flesh and bones as you see I have
them." John 20:27 says, "Put your fingers here; see my hands." Whatever may be said about
his, they are reporting physicality. However, these same gospels also describing raising as
egeiro, which is the same term Paul used. Matthew 28:6-7 says "He is not here, he has risen
(egeiro) just as he said. Come and see the place where he lay. He has risen (egeiro) from the
dead." Luke 24:6 says, "He is not here, he has risen (egeiro)!" And John 21:14 says "...he was
raised (egeiro) from the dead." The point is clear: the writers of Matthew, Mark, and Luke
saw no contradiction in affirming that Jesus was alive bodily and physically with the word
egeiro the same word that Paul used.

Later on Till examines Paul's use of the word opethe. He argued that this word is used to
describe visions in the New Testament, and therefore Paul was not recording physical
appearances in 1 Corinthians 15. However, let's go back once again to the gospel writers who
did believe that He appeared physically. "There you will see (opethe) him" (Matt. 28:7);
"(T)he Lord has risen (egeiro) and has appeared (opethe) to Simon" (Luke 24:24). "(T)hey
will look (opethe) on the one they have pierced" (John 19:37). Did John believe they were
having a vision of Jesus on the cross? Luke 24:24 is sufficient evidence alone to show that
egeiro can mean both raised or waken, and that opethe can refer to a vision or a physical
appearance.

Till also tried to show, mainly from his exegesis of 1 Corinthians 15:40-44, that Paul believed
Christians would experience only a "spiritual resurrection," not a physical one, and therefore
Christ was only raised spiritually, not physically. But when Paul says of a believer's body in
verse 42 that "it is sown in corruption, it is raised in incorruption," he is not saying that our
bodies will be taken from materiality, but from mortality. Till thinks he has found further
support in verse 44. However, this verse most forcefully teaches the traditional doctrine of the
resurrection. "It is sown a natural body, it is raised a spiritual body." Clearly Paul teaches a

Volume 1990 - 2002 Issue


Page 865 of 2049
Skeptical Review Edited by Farrell Till
continuity between the "natural body" and the "spiritual body," for it is the same "it" in both
cases. He is referring to the same physical body in different states, not a change in substance.
And virtually all commentators agree that "spiritual" does not mean "made out of spirit," but
"directed by and orientated to the Spirit." It is just like when we say someone is a "spiritual"
person. Paul uses the word in this way in 1 Corinthians 2:15: "The spiritual man judges all
things...." Clearly Paul does not mean "immaterial, invisible man" here but "man oriented to
the Spirit." And look at 10:4, where Paul refers to a "spiritual rock." Does Paul mean
"immaterial rock"?

In verses 35-37, where Till also finds "support," Paul says, "What you sow is not made alive
unless it dies. And what you sow, you do not sow that body that shall be, but mere grain." But
he is not even referring to the substance of our future bodies. And Paul does teach a continuity
between our bodies now and in the future in saying that the thing that is sown is made alive if
it dies; there is continuity between the seed to the plant (they are the same organism), yet
there is also change. And certainly both the seed and the plant are both physical! Christians do
not have "the body that shall be" because we are not immortal yet. In support of Paul's view
of a physical resurrection, look at Romans 8:21-23. He teaches that the whole creation will be
transformed into freedom, but not nonmateriality. He then says our bodies will likewise be
"redeemed." Since creation will be transformed, yet remain physical, and our bodies are part
of creation, they will also be transformed, yet still be continuous with our old bodies and
remain physical. Finally, Paul's belief in the personal return of Christ (1 Thess. 4:14-17) also
implies that he believed in a physical resurrection.

It is also difficult to see, using Till/Barker's hypothesis, why the following "legendary
developments" about the burial, empty tomb, and appearances would take place: (1) The use
of women to discover the tomb. The testimony of women was not considered credible in first
century Judaism. So if the resurrection is simply a large legendary evolution, why didn't the
early Christians have the disciples discover the tomb instead? Also, (2) why was Joseph of
Arimathea used in the burial story? The members of the Sanhedrin were too well known for
someone to place a fictional member on it or to spread a false story about one of its members
burying Jesus. And (3) why weren't the "hopeless contradictions" in the resurrection
appearances harmonized if it was all a legend? While it does harmonize, if the whole thing
was made up, shouldn't it harmonize a little easier?

Lastly, Jewish New Testament scholar Pinchas Lapide has examined Jewish thought of the
first century and found that all schools of thought held to a notion of a physical resurrection
(Lapide, The Resurrection of Jesus: A Jewish Perspective, pp. 44-65). The early Christians
were Jewish (including Paul). A resurrection without the body would have been nonsense to
them. This in itself should put an end to any notion of a "spiritual resurrection" myth.

Clearly Till's attempt at my challenge to "explain all these facts" with his myth theory has
failed. If, as some atheists are saying, this is the best argument against the resurrection, then
Christianity will remain intellectually strong. Now we will briefly turn to his specific
treatment of my six facts.

Till says that no non-Christians sources corroborate the New Testament record of the
crucifixion. Till disputed my reference to the non-Christian Thallus on this point, but did not

Volume 1990 - 2002 Issue


Page 866 of 2049
Skeptical Review Edited by Farrell Till
deal with the fact the in the Babylonian Talmud, Sanhedrin 43a does record the crucifixion of
Christ. He also thinks that my belief in the empty tomb must be based on the assumption that
the New Testament documents are reliable, so let me give some more reasons to believe in
this specific New Testament point. First, my arguments supporting Jesus's burial also apply
here, for if the burial story is accurate, the empty tomb story likely is accurate as well, since
linguistic ties indicate that they form one continuous narrative from the pericope. Second, in
his record of the empty tomb, Mark used a source which originated before AD 37. Scholars
know this because the high priest is referred to without using his name. Caiphas (the high
priest during Christ's death) must have therefore still been the high priest when this story
began circulating since there was no need to mention his name in order to distinguish him
from the next high priest. Caiphas's term ended in 37, so that is the last possible date for the
source's origination. Thus, the evidence for the empty tomb is so near to the events
themselves that it is hard to argue that legend could sweep in and replace the hard core
historical facts. This also confirms early Christian belief in a bodily resurrection, for clearly
the gospel of Mark teaches a bodily resurrection. Lest Till claim that the empty tomb is only
accepted by "fundamentalists," I have a list of 47 critical scholars in front of me
(nonfundamentalists) who accept the empty tomb (such as Blank, Delorme, Lapide, Schwank,
Strobel, etc.). Till of course will say that an empty tomb proves nothing, but all natural
attempts to explain it have been rejected by critics. As Craig says, "They are self-confessedly
without any theory to explain it."

There are also sources outside the New Testament which support the empty tomb. In his
Dialogue with Trypho the Jew, Justin Martyr states, without fear of being disputed by his
Jewish opponent, that the Jewish leaders had sent men around the Mediterranean to further
the teaching that the body had been stolen. This presupposes an empty tomb.

Concerning the resurrection appearances, Till calls into question the traditional authorship of
the gospels, claiming that they were not written by eyewitnesses. But even if this were true, it
would not hurt my case. First, there is still the eyewitness testimony of 1 Corinthians 15:3ff to
establish the resurrection appearances (which Till admits is very early). Second, while it is
true that there is a "formidable body of critical works" that reject the traditional authorship of
the gospels, most of these same scholars still find a large amount of eyewitness testimony
behind the gospels (Robert Grant, Cranfield, Hunter, Brown, John A.T. Robinson. Who
would call these guys "fundamentalists"?). Even Raymond Brown, a skeptic who wrote a
significant commentary on John, held that the apostle John was a major source behind the
gospel of John.

Till calls into question the apostles' martyrdom and very existence. He then says that the rapid
spread of early Christianity is only recorded in "biased" Christian sources, and so he seems to
reject the notion (and the sources) altogether. But on what basis can he just reject Christian
sources because in his opinion they are biased? Again, no historian is disinterested, but as I
said earlier, this is no reason to discredit the possibility of finding genuine history from them.
Till is not just accusing the early Christians of bias; he is accusing them of blatant dishonesty
(especially when he rejects the main events of the New Testament record)! But on what
grounds can he do this, especially since the early Christians considered moral integrity and
honesty more important than life itself? As secular governor Pliny the Younger said around
AD 112: "They... bound themselves by a solemn oath, not to any wicked deeds, but never to

Volume 1990 - 2002 Issue


Page 867 of 2049
Skeptical Review Edited by Farrell Till
commit any fraud, theft or adultery, never to falsify their word..." (Pliny, Letters, vol. II,
X:96). Now we can consider this early quote from Tertullian: "But go zealously on.... Kill us,
torture us, condemn us, grind us to dust. . . . The oftener we are mown down by you, the more
in number we grow; the blood of Christians is seed." And even the secular Tacitus (AD 55-
120) "speaks of an "immense multitude" of Christians, who were murdered in the city of
Rome alone during the Neronian persecution in 64. To this must be added the silent, yet most
eloquent testimony of the Roman catacombs... and are said to contain nearly seven millions of
graves..." as historian Philip Schaff reports (History of the Christian Church, pp. 79-80). The
catacombs alone are clear, reliable evidence to the rapid spread and resulting great
persecution of the early church. As to whatever doubt some may have about the martyrdom of
the apostles, we must ask this question: if the students were willing to die for their faith, how
much more their teachers? Further, Eusebius is considered generally accurate in what he
reports (see Schaff), and he records how each apostle died. Also, in a passage almost
universally considered authentic, Josephus records the martyrdom of James. Critical scholars
even acknowledge that the apostles were willing to die for their faith.

Apparently Till thinks that I said that just because the apostles were martyred, Christianity
must be true. But that is not my argument. This is my argument: 1 Corinthians 15:3ff is an
early creed in which Paul records the resurrection appearances. Even Till agreed to this. We
have also seen that these appearances were clearly physical. Paul received this creed from
Peter and James (as virtually all critical scholars agree), who are listed in this creed as
eyewitnesses to the resurrection of Christ. So, in other words, Jesus's original disciples
claimed to have seen Him alive again after His death. Even the most skeptical New Testament
scholars admit that the disciples really believed they saw Jesus (see Bultmann, Theology of
the New Testament, 44-45; Fuller, Resurrection Narratives, 27-49; Pannenberg, Brown, etc.).

We have two options if Jesus did not rise: either the disciples believed their claim that they
saw Him and were mistaken, or they did not believe their own claim and were therefore
deliberately lying. I clearly showed in my first article how the disciples could not have been
mistaken. But the only other option forces us to concede that the apostles died, not just for a
lie they mistakenly believed to be true, but for a lie that they knew was a lie! While martyrs of
other religions have died for what they sincerely believed was true, the difference is that the
disciples would have been dying for what they sincerely knew was a lie! As I said in my
article, "Ten people would not all give their lives for something they know to be a lie."
Therefore, we must conclude that the disciples believed that Jesus rose and appeared to them
because Jesus really did rise and appear to them! That is the only explanation for their claim.
Taken along with our evidence for the early belief in Christ's physical resurrection and our
evidence for the empty tomb, we can conclude that the resurrection still stands tall. The
testimony of 1900 years of history bears this out, and Christ continues to say today, "Whoever
hears My word, and believes Him who sent Me, has eternal life, and does not come into
judgment, but has passed out of death into life" (John 5:24).

(Matthew Perman, 1120 Dancer Hall, Cedar Falls, IA, 50613; e-mail
permanm4888@uni.edu)

Volume 1990 - 2002 Issue


Page 868 of 2049
Skeptical Review Edited by Farrell Till

Standing on Quicksand
by Farrell Till
I welcome Matt Perman's second attempt to prove the historicity of the resurrection. He is to
be commended for having the courage to defend his belief in a forum like this. He is not to be
commended, however, for the quality of his arguments. Most of them are as transparent as
cellophane, and, like many fundamentalists, his primary defenses of the resurrection are based
on (1) unsupported assertions, (2) the fallacy of the appeal to authority, and (3) an
unwarranted assumption that the New Testament documents are historically accurate. His
unsupported assertions are so many that I will not be able to respond to all of them in a single
article, so I will publish a series of two, possibly three, responses, after which Perman may
respond to them if he wishes.

There are some general observations that need to be made about Perman's latest article before
I address his "arguments" specifically. First, there is the very nature of the resurrection claim
that makes it untenable, and Perman and his apologetic cohorts seem completely unable to
grasp the significance of this point. There is a widely accepted rule of evidence that says
extraordinary claims require extraordinary evidence. In my debate on the resurrection with
Michael Horner, he tried to impose the meaning of "miraculous" on the word extraordinary to
make it appear I was arguing that it would take a miracle to prove a miracle, but this is only a
straw man set up to give the appearance of responding to an argument that is very damaging
to the resurrection claim. The frequent application of the word extraordinary to situations that
are explainable by natural laws shows that the word extraordinary is not commonly
considered just a synonym for miraculous. In reality, the word extraordinary simply means
"beyond the ordinary," and events that are "beyond the ordinary," without being miraculous,
certainly happen. As I write this, the Olympic games are in progress, and those who have
been watching them have seen many athletic achievements that can properly be called
"beyond the ordinary" or extraordinary. Each time an Olympic contestant breaks a world
record, this can be properly called an extraordinary feat, because breaking world records
doesn't happen routinely. One swimmer, for example, broke the world record in the
preliminaries and then turned around and broke that record in the final event. Who could
argue that this was not an extraordinary feat, i.e., an event that was beyond the ordinary?

If, however, resurrection apologists want to quibble, we can think of "extraordinary evidence"
as simply being unusually good or unimpeachable evidence. Hence, I will argue that
exceptional or extraordinary claims require unusually good evidence that is so convincing it
cannot be rationally impeached. I would assume that even Perman is willing to admit that a
resurrection from the dead would be an event that is well "beyond the ordinary"; otherwise,
we could expect resurrections from the dead to occur at least a few times in the life of the
average individual, just as the average person can expect world records in athletic events to be
broken at least a few times during his/her life span. Hence, Perman certainly should not object
if I say that before rational people will accept the claim that a man rose from the dead, they
will demand "unusually good" evidence that cannot be impeached. As we will see, the claim
that Jesus of Nazareth rose from the dead doesn't even come close to being supported by
unimpeachable evidence.

Volume 1990 - 2002 Issue


Page 869 of 2049
Skeptical Review Edited by Farrell Till
Eyewitness Testimony. Perman has tried to prove the resurrection by claiming eyewitness
testimony, but in my first rebuttal article, I showed that a scholarly consensus rejects the
notion that the gospels were written by actual eyewitnesses of the events recorded. I further
showed that even if we accept that the apostles Matthew and John wrote the gospels
traditionally attributed to them, this still would not make people like the women who went to
the tomb or the disciples on the road to Emmaus or Simon Peter eyewitness testifiers to the
resurrection, because none of them left any firsthand testimony that they had seen Jesus alive
after his crucifixion. The gospel writers merely said that these people said they had seen
Jesus alive after his burial. So even on the assumption that the apostles Matthew and John
wrote the gospels attributed to them, this would reduce the sum total of eyewitness testimony
in all four of the gospels to just two people. That is hardly unimpeachable evidence for a
claim as extraordinary as a resurrection. Far more people now living have claimed that they
personally saw Elvis Presley after his reported death, yet no rational person gives any serious
credence to such reports. Such testimony is impeachable by the very nature of what is being
claimed.

To illustrate what constitutes unimpeachable evidence, let's look again at the Olympic
swimmer who broke a world record in the preliminaries and then broke that record in the
finals. There is unimpeachable evidence to establish that this extraordinary event happened,
because it was not just witnessed on TV by millions of people around the world but was
recorded by the best electronic equipment available, which was in turn verified by other back-
up systems. Let's suppose, however, that this remarkable feat had happened under different
circumstances. Suppose that the swimmer had done this in another place where the only
witness was a person who had timed her with a hand-held stopwatch. Who then would be
willing to accept her feat as a new world record? The question is rhetorical, because any
sensible person would know that the probability for error or outright false testimony would be
too high to be accepted as fact. Well, let's suppose that 15 or 20 or even more people should
come forward and claim that they too had witnessed the event. That wouldn't make the claim
any more credible, because rational people would realize that it is more likely that people
poorly equipped to judge would be mistaken than that an exceptional feat like this could
happen twice in the space of just two days.

Now let's make the feat really remarkable. Let's suppose that someone should claim that she
had swum 800 meters in only 15 seconds. Who would believe her? Well, what if someone
claimed that 500 people had witnessed her alleged accomplishment? Would that make it more
believable? Not to rational people, because rational people would realize that it is far more
likely that the claim of 500 witnesses was a lie or an honest mistake than that such an
extraordinary feat had actually been accomplished. Even if the 500 people should come
forward and swear under oath that they had witnessed the feat, rational people still would not
believe it, because rational people would still understand that it is far more likely that 500
people would all lie or could be honestly mistaken than that a person could actually swim 800
meters in only 15 seconds.

Even if resurrection apologists could prove beyond reasonable doubt that they had personal
firsthand testimony from Mary Magdalene, the other Mary, Salome, Joanna, Simon Peter, and
the other apostles that they had seen Jesus alive after his burial, that still would not constitute
unimpeachable evidence that Jesus literally died and rose from the dead, because there would

Volume 1990 - 2002 Issue


Page 870 of 2049
Skeptical Review Edited by Farrell Till
be too many natural explanations for their testimony: (1) they all lied in order to further the
cause of their Christ beliefs, (2) the cognitive dissonance they were struggling with in their
extreme grief caused them to rationalize or imagine that they had seen Jesus alive again, or (3)
they were mistaken in thinking that Jesus had actually died. Any of these would be a more
likely explanation for firsthand testimony to a resurrection from the dead than the assumption
that a resurrection had actually happened.

The historical reliability of the New Testament Documents. I have argued, and still do,
that basically all of the so-called evidence for the resurrection so highly touted by Perman and
his fundamentalist cohorts assumes the historical accuracy of the New Testament documents.
He objects to this and says, "I am emphatically not arguing from the premise that the New
Testament documents are reliable in everything," but this is only a smoke screen intended to
make him appear to be the very epitome of scholarly objectivity. Perman is a biblical
inerrantist, but he wants to leave the impression that this irrational position has in no way
influenced the formation of his belief that a man was once restored to life after lying dead for
about two days. To his credit, he did at one point admit, as a parenthetical aside, that he does
believe the New Testament is "always trustworthy." So let's lay this fact out on the table and
keep in mind while we are discussing this issue that Perman really believes that Jesus rose
from the dead for no reasons except that the New Testament says that he did. If the New
Testament had said that Jesus urinated watermelon juice, Perman would believe it. Nothing
could be so outrageously absurd that Perman would not believe it if the New Testament had
said it. This is the kind of mentality that I am presently trying to reason with.

Perman, of course, will not appreciate my characterizing him as a slavish advocate of


whatever the New Testament says, because he wants us to believe that his faith in the
resurrection of Jesus has been based on reliable history rather than mere gullibility, but let's
look at some of Perman's "reliable historical claims" and just let the facts speak for
themselves.

The empty tomb. Perman said that he had "appeal[ed] to Matthew 28:11-15 to show that the
earliest Jewish propaganda admits the empty tomb," but apparently he can't understand that
appealing to the New Testament is not an appeal to Jewish propaganda but to Christian
propaganda. All that any appeal to the New Testament accomplishes is to show what early
Christians believed, not what non-Christian Jews believed. Perman expects us to believe that
this passage proves that contemporary Jewish leaders knew that Jesus had risen from the dead
but circulated a rumor that the body had been stolen in order to curtail the growth of the Jesus
movement, but in my first response to Perman, I pointed out that an interpretation of Matthew
28:11-15 can lay no claim to objective analysis unless it considers the distinct possibility that
this passage was written into the text as a means of countering a legitimate claim that was
circulating at that time. In other words, if the death of a man named Jesus actually did occur,
it is entirely possible that the body had been stolen, and Jewish leaders knew this had
happened and were so informing their people. If such were the case, then it is easy to see what
Matthew's purpose was in this passage. He wanted to give his readers an explanation for why
the rumor had been circulated, so he concocted an unlikely scenario that had the Roman
guards reporting to the chief priests and then accepting a bribe to say that while they had been
asleep on duty, the disciples had come and stolen the body.

Volume 1990 - 2002 Issue


Page 871 of 2049
Skeptical Review Edited by Farrell Till
In accepting this tale as historical fact, Perman shows an incredible lack of critical ability.
How likely would it have been that Roman guards would have reported to a group of Jewish
officials rather than to their own officers? How likely is it that these Roman guards would
have claimed that they had fallen asleep on duty, an offense that could have brought them
severe penalties? How likely is it that Jewish priests could have persuaded the Roman
governor to rid the guards of blame, as they promised the guards they would do if charges
were brought against them (v:14)? This whole scenario is so unlikely that John Wenham took
the position in Easter Enigma that its very absurdity must mean that it is true. This is the way
that Christian apologists have to reason to find support for their resurrection belief. They have
to assume that the New Testament documents are historically reliable no matter how absurd
their claims may be. So what Perman is actually arguing on this point is that he knows that
Jewish propaganda admitted there was an empty tomb, because the New Testament says that
this is so. If that is not assuming the historical reliability of the New Testament documents, I
would like for Perman to explain to us why it isn't.

Perman argues that "serious scholars (whether Christian or not) across a broad spectrum of
beliefs" accept the six points (one of which was the empty tomb) that he presented in support
of the resurrection doctrine. Is he right about this? Do some "serious scholars" accept the
empty tomb as historical fact? Of course, they do, but what Perman conveniently left
unmentioned is that probably even more "serious scholars" reject the empty-tomb claim as
well as the resurrection story in general. No matter how many "serious scholars" Perman may
produce who believe in the historicity of the empty tomb, one stubborn fact remains: no
Jewish records, no Roman records, no contemporary records of any kind mentioned an empty
tomb in Jerusalem at the time Jesus was alleged crucified and buried. The only records that
mention an empty tomb are the New Testament and apocryphal gospels, which were all
written well after the fact, when Christianity was becoming an established religion, so if no
other records of the time mention an empty tomb in Jerusalem, how could anyone believe that
such a tomb did exist except by assuming that the New Testament documents are historically
reliable? Even if Perman should produce a million "serious scholars" who agree with him on
this point, it would have to be true that they formed their opinion the same way Perman did.
They assumed the historical reliability of the New Testament documents. If there are no
contemporary secular references to Jesus, then there are no contemporary secular references
to the alleged empty tomb. So how else could anyone believe that there was an empty tomb in
Jerusalem at this time except by assuming that the New Testament documents are accurate in
their claim that there was an empty tomb? This is so obvious that I can't believe Perman is
even disputing it, so what he needs to do is produce some "serious scholars" who accept the
historicity of the empty tomb on grounds other than that this is what the New Testament
documents claim. We have every reason to believe that if Perman could produce evidence that
corroborates the New Testament on this point, he would have done so long ago.

The Benefit of the Doubt. As a reason why the New Testament documents should be
considered historically reliable, Perman cited Aristotle's dictum that "the benefit of the doubt
should be delegated to the document itself, not arrogated by the critic to himself." I have
heard this dictum before, but I have never seen it quoted in its full context. Exactly what was
Aristotle saying? I would like to know if he was contending that this principle should be
followed no matter what the document says or if he was only stating it as a principle that
should be followed in evaluating claims of ordinary, everyday events that one's common

Volume 1990 - 2002 Issue


Page 872 of 2049
Skeptical Review Edited by Farrell Till
sense says could very well have happened. If the latter, then why is Perman quoting it in
support of documents that, among many other fabulous claims, reported that a dead man
returned to life? If the former, then I beg to differ with Aristotle, who, after all, was just
another fallible human being, so in that case, I see no reason to evaluate historical information
by a standard that would require me at times to surrender my common sense, even if Aristotle
thought otherwise.

Perman asked, "What right does Till have to operate from the extreme assumption that the
New Testament is fundamentally unreliable?" Extreme assumption? What right does Perman
have to label a conclusion arrived at by recognized methods of historical analysis as an
"extreme assumption"? When I read in the Book of Mormon all sorts of fantastic claims, such
as voices speaking from heaven, the postresurrection appearances of Jesus in the Land of
Bountiful, multitudes of sick Nephites being healed by Jesus, etc., am I making an "extreme
assumption" of fundamental unreliability when I doubt that such events actually happened?
Should I follow Aristotle's dictum and give the benefit of the doubt to the Mormon documents
rather than arrogating it to myself? This is a question that Perman and other Christian
apologists who are so enamored with the "historical reliability" of the New Testament need to
answer. If we assume that the outrageous claims of the New Testament deserve the "benefit of
the doubt," just where does this process of according the benefit of the doubt end?

As just noted, the Book of Mormon makes many fantastic claims that rational people simply
cannot accept as historical facts, yet in my opinion the evidence for the reliability of the Book
of Mormon is much better than any evidence that Perman can cite in support of the New
Testament. The Book of Mormon is prefaced with the firsthand testimony of Joseph Smith
and the three and the eight witnesses, who tell what they claimed to have seen, experienced,
and personally witnessed when the golden plates on which the book was allegedly inscribed
were revealed to and translated by Smith. The following affidavit, with emphasis added, was
sworn to by the three witnesses:

Be it known unto all nations, kindreds, tongues, and people, unto whom this work
shall come: That we, through the grace of God the Father, and our Lord Jesus Christ,
have seen the plates which contain this record, which is a record of the people of
Nephi, and also of the Lamanites, their brethren, and also of the people of Jared, who
came from the tower of which has been spoken. And we also know that they have
been translated by the gift and power of God, for his voice hath declared it unto us;
wherefore we know of a surety that the work is true. And we also testify that we have
seen the engravings which are upon the plates; and they have been shown unto us by
the power of God, and not of man. And we declare with words of soberness, that an
angel of God came down from heaven, and he brought and laid before our eyes, that
we beheld and saw the plates, and the engravings thereon; and we know that it is by
the grace of God the Father, and our Lord Jesus Christ, that we beheld and bear record
that these things are true. And it is marvelous in our eyes. Nevertheless, the voice of
the Lord commanded us that we should bear record of it; wherefore, to be obedient
unto the commandments of God, we bear testimony of these things. And we know that
if we are faithful in Christ, we shall rid our garments of the blood of all men, and be
found spotless before the judgment-seat of Christ, and shall dwell with him eternally

Volume 1990 - 2002 Issue


Page 873 of 2049
Skeptical Review Edited by Farrell Till
in the heavens. And the honor be to the Father, and to the Son, and to the Holy Ghost,
which is one God. Amen (emphasis added).

This document was signed by the three witnesses: Oliver Cowdery, David Whitmer, and
Martin Harris. In addition to their firsthand testimony, the Mormons also have the testimony
of the eight witnesses:

Be it known unto all nations, kindreds, tongues, and people, unto whom this work
shall come: That Joseph Smith, Jun., the translator of this work, has shown unto us
the plates of which hath been spoken, which have the appearance of gold, and as
many of the leaves as the said Smith has translated we did handle with our hands;
and we also saw the engravings thereon, all of which has the appearance of ancient
work, and of curious workmanship. And this we bear record with words of soberness,
that the said Smith has shown unto us, for we have seen and hefted, and know of a
surety that the said Smith has got the plates of which we have spoken. And we give
our names unto the world, to witness unto the world that which we have seen. And we
lie not, God bearing witness of it (emphasis added).

This document was signed by the eight witnesses: Christian Whitmer, Jacob Whitmer, Peter
Whitmer, Jun., John Whitmer, Hiram Page, Joseph Smith, Sen., Hyrum Smith, and Samuel H.
Smith.

Christian apologists would be deliriously happy if they had even a tenth as much evidence to
support the New Testament resurrection accounts. They talk about eyewitness testimony,
which isn't eyewitness testimony at all but only second- and thirdhand accounts of alleged
postresurrection appearances, but the Mormons do have the firsthand testimony of eleven
witnesses who swore that they actually saw and handled the golden plates, and three of those
witnesses swore that the angel laid the plates down right before their eyes. The closest thing
to firsthand testimony to the resurrection that Christians can produce is the apostle Paul's
imprecise claim that Jesus had appeared to him (1 Cor. 15:8), and according to Luke's
secondhand account of this appearance, Paul called it a "heavenly vision" (Acts 26:19). That's
pretty weak evidence compared to the sworn detailed testimony of the Mormon eyewitnesses.

Perman, of course, rejects in toto these Mormon claims. Although I have not discussed with
him his reasons for this rejection, I am sure if he were asked why he doesn't believe the
testimony of the Mormon witnesses, he would say such things as (1) the claim is too unlikely
to believe without better evidence, (2) the eyewitnesses were biased followers of Joseph
Smith, (3) the surnames indicate that most of the witnesses were related and therefore likely
to have had self-serving family interests, (4) the Book of Mormon itself is too flawed by
inconsistencies and discrepancies to believe that it is divine in its origin, etc. In other words,
Perman undoubtedly rejects the Mormon claims for reasons that he arrived at by rational
methods of critical analysis. Yet he denounces skeptics who by parallel reasoning reject the
fantastic claims of the New Testament. There seems to be more than just a slight
inconsistency in Perman's methods of evaluating historical claims.

Furthermore, the same basic "arguments" that Christian apologists use in defense of the
resurrection can be applied with equal legitimacy to prove the divine origin of Mormonism.

Volume 1990 - 2002 Issue


Page 874 of 2049
Skeptical Review Edited by Farrell Till
The origin of Christianity requires an explanation, apologists say, so that is a good reason to
believe that the resurrection was a crucial activating circumstance in the Christian movement.
Mormons, however, can adapt this same "argument" to their religion. The Book of Mormon
requires an explanation, so that is a good reason to believe that it was revealed in the manner
sworn to by Joseph Smith and the three and eight witnesses. The early disciples died for their
claim that Jesus had risen, Christian apologists say, and they surely would not have died for
what they knew was a lie. But Mormons also suffered persecution. I live in Western Illinois
where Mormons fleeing persecution in the East once tried to establish a settlement. Only
about 65 miles from my home, Joseph Smith was lynched at the Carthage, Illinois, jail, which
is now a Mormon shrine, and eventually Mormons were driven by persecutions farther west
from their settlement at Nauvoo, Illinois. So if the Christian argument based on persecution
proves the truth of the resurrection, why wouldn't the same argument prove the truth of
Mormonism? Would Mormons have suffered and, in some cases, died for what they knew
was a lie?

Virtually all "critical non-Christian scholars" accept Perman's "six points."


Undoubtedly, Perman considers a "critical non-Christian scholar" to be a non-Christian who
accepts his six points, and any who don't just aren't "critical scholars," no matter how
impressive their academic credentials may be. It is telling to note, however, that Perman,
aside from dropping a name or two, didn't bother to list these "critical non-Christian scholars"
who agree with his position. Are there really non-Christian scholars who agree with at least
some of Perman's points? Of course, there are, but Perman is a gullible victim of apologetic
propaganda if he really believes that "virtually all" non-Christian scholars agree with him.
Common sense should tell him that if virtually all non-Christian scholars agree with his
position, then they would be Christian rather than non-Christian scholars. The truth is that not
even virtually all Christian scholars are in agreement with Perman's basic points. The Jesus
Seminar, whose opinions Perman will certainly ridicule, consists of seminary professors and
other biblical scholars, who as a group even reject the resurrection claim itself.

I have no desire to get into a shooting match with Perman in which he fires the name of a
scholar who agrees with him, and in response I fire back the name of a scholar who agrees
with me. Such an approach would be a resort to the fallacy of the appeal to authority, and
would prove exactly nothing. No matter what religious opinion one may have, he can always
find books that agree with him, so I will insist that we discuss the merits of our respective
arguments without appealing to what alleged "scholars" may believe.

My position is that belief in the resurrection is irrational for reasons that I have noted in my
responses to Perman, so if he intends to continue this discussion, I urge him to confront those
arguments and explain to us why they are unreasonable. In particular, I wish Perman would
address the very nature of the resurrection claim and explain to us in logical terms why
rational people should give any more credence to the resurrection claim than they would give
to any other fabulous claim, because this is really the crux of the matter. Rational people,
even Christian apologists themselves, routinely reject miraculous claims, but Perman just
can't seem to understand that Christian claims of the fantastic and supernatural warrant no
more serious consideration than other such claims. A slogan of the Democrats in the 1992
presidential election was, "It's the economy, stupid," so in the matter of the resurrection claim,
I think it is appropriate for skeptics to keep reminding Christians of the primary reason why

Volume 1990 - 2002 Issue


Page 875 of 2049
Skeptical Review Edited by Farrell Till
so many rational thinkers just can't believe in their resurrected savior-god. It is the very nature
of the claim, stupid, that makes it so unbelievable. Will some Christian apologist please try to
deal with that problem?

In the next issue, I will respond to Perman's specific counterarguments, especially those he
presented in denial of the spiritual resurrection that I believe the apostle Paul argued for in 1
Corinthians 15. After that, Perman may respond to my articles if he wants to.

Jacob an Old Geezer?


by Farrell Till
There is no scarcity of fanciful tales in the Bible. An especially quaint one tells about the duel
of the handmaids when Jacob, the grand patriarch of Israel, was siring his sons who
subsequently became heads of the twelve Israelite tribes. It is a story that simply cannot pass
the how-likely-is-it? test that was discussed in the Autumn 1993 issue and applied to the
Israelite exodus.

The story began in Genesis 28 when Jacob left Canaan and journeyed to Paddanaram to look
for a wife. Before his departure, Jacob had forced his twin brother Esau, in a moment of
hunger, to sell his birthright for a mere mess of pottage and then had tricked his father Isaac
into pronouncing a primogenitive blessing on him rather than Esau, the firstborn of the twins.
Fearful of what Esau might do to avenge Jacob's deception, their mother Rebekah convinced
Isaac to send Jacob to Paddanaram to seek a wife among the daughters of her brother Laban
rather than the women of Canaan. Esau had already married two Hittite women who were "a
grief of mind to Isaac and to Rebekah" (Gen. 26:34-35), so this gave Rebekah an excuse to
feign concern for whom Jacob might marry. "I am weary of my life because of the Hittite
women," she said to Isaac. "If Jacob marries one of the Hittite women such as these, one of
the women of the land, what good will my life be to me?" (Gen. 27:46).

So Isaac sent Jacob to Paddanaram with instructions to select a wife from among his mother's
people. On his arrival, Jacob saw his uncle Laban's daughter Rachel keeping her father's sheep
and instantly fell in love with her. For the right to marry Rachel, he struck a deal with Laban
to work for him seven years. At the end of Jacob's seven years of service, however, Laban
tricked Jacob and gave him his elder daughter Leah instead. Jacob spent his wedding night
with his bride, and "it came to pass in the morning that, behold, it was Leah" (Gen. 29:25)
rather than Rachel.

If ever there was an absurdity in the Bible, this part of the story certainly qualifies. Are we to
believe that Jacob spent seven years working for Laban and living in his house (Gen. 31:41), a
period in which he must have had many personal contacts with Rachel, the woman he loved
enough to offer himself as a servant in order to obtain permission to marry her, and yet he
spent his entire wedding night doing what men do on their wedding nights and didn't even

Volume 1990 - 2002 Issue


Page 876 of 2049
Skeptical Review Edited by Farrell Till
know until the light of day that the woman he was with was not the woman he thought he was
marrying! It is an insult to human intelligence to expect anyone to believe that.

Anyway, as the story goes, Laban explained to Jacob that "(i)t is not so done in our place, to
give the younger before the firstborn" (Gen. 29:26). Laban then offered to give Jacob Rachel
in exchange for seven more years of service. Jacob accepted the terms, and Laban gave him
Rachel (vv:27-29). Leah began to bear children, and after she had given birth to four sons,
Rachel, who had not yet conceived, gave Jacob her handmaid Bilhah and asked him to sire
children for her on Bilhah. Apparently no arm-twisting was necessary to get Jacob's
cooperation in going along with Rachel's plan, for Bilhah promptly bore Jacob two more sons
(Gen. 30:1-8).

Meanwhile, Leah, who had temporarily stopped bearing children, gave Jacob her handmaid
Zilpah, who in the duel of the handmaids, also bore Jacob two sons (Gen. 30:9-13).
Eventually, "God hearkened unto Leah" (Gen. 30:17), and she bore two more sons and Dinah,
the only daughter ever born to Jacob during all of this sexual rampage (vv:17-21). "God
remembered Rachel" too and "opened her womb" (v:22), and she bore Joseph. So when all of
the passions had cooled in Paddanaram, Jacob had had eleven sons and one daughter. Later,
on Jacob's journey back to Canaan, Rachel gave birth to Benjamin at the cost of her life. In
all, according to the story, twelve sons and one daughter were born to Jacob.

All of this had happened over a period of 20 years, because "it came to pass when Rachel had
borne Joseph" that Jacob, having finished his 14 years of service to Laban, asked permission
to take his wives and children and return to his country (Gen. 30:25-26). Laban, however,
persuaded Jacob to stay and work six more years in exchange for all the spotted and speckled
lambs and kids born in his flocks (Gen. 30:27-43), so altogether Jacob spent 20 years in
Paddanaram (Gen. 31:41).

This story has been read countless times by the millions of people who have professed
Christianity, but, unfortunately, not very many of them have critically examined it. If they
had, they would have recognized it for what it is -- just a fanciful Hebrew folk tale.

I know that such a statement is blasphemy to those who have been reared to respect the Bible
as the verbally inspired, inerrant word of God, but inconsistencies existing between this story
and other incidents related in the Bible about Jacob necessitate this conclusion. For one thing,
if the Bible is inerrant in all details, as fundamentalists adamantly insist that it is, then we
have to believe that Jacob was 83 years old when his sexual adventures in Paddanaram began.
Such nonsense might make good reading material for The National Enquirer and other
sensationalistic tabloids, but it is asking just a little too much to expect critical readers to
believe that an 83-year-old man could have been capable of such sexual vigor while working
long hours in the day tending to large flocks of sheep and goats. When Yahweh appeared to
Abraham to tell him that his wife Sarah would bear him a son, Abraham was 99 at the time,
and his reaction was to laugh: "Then Abraham fell on his face and laughed, and said to
himself, `Can a child be born to a man who is a hundred years old?'" (Gen. 17:17). Of course,
after Sarah died, when Abraham was 137, he married another woman (Keturah) and
apparently with no difficulty sired six other children (Gen. 25:1-2), but that is just another
example of nonsensical inconsistency in the biblical text. The point is that rational people will

Volume 1990 - 2002 Issue


Page 877 of 2049
Skeptical Review Edited by Farrell Till
realize that the sexual escapades attributed to Jacob while he was in Paddanaram far exceed
the capabilities of a man who was 83 to 96 years old, and even the Bible in at least one place
recognizes the unlikeliness of it.

How can we know that Jacob was 83-96 years old when all this was happening? It is a matter
of simple arithmetic. When Joseph presented his father to Pharaoh after the arrival of Jacob's
family in Egypt, Jacob was 130: "Pharaoh said to Jacob, `How many are the years of your
life?' Jacob said to Pharaoh, `The years of my earthly sojourn are one hundred thirty; few and
hard have been the years of my life'" (Gen. 47:8-9). This age was confirmed later in the
chapter, at the time of Jacob's death, when it was claimed that "Jacob lived in the land of
Egypt seventeen years; so the days of Jacob, the years of his life, were one hundred forty-
seven years" (v:28). So if we can determine the age of any of his sons at the time when Jacob
was presented to Pharaoh, it would be possible to know how old he was when his sons were
being born to him in Paddanaram.

In the case of Joseph, we can make that determination. After his brothers sold him into Egypt,
Joseph rose to prominence through his dream-interpretation skills. Having interpreted with
accuracy the dreams of Pharaoh's butler and baker while they were in ward with him, Joseph
was called out of prison to interpret two disturbing dreams that Pharaoh had had. In his
interpretation, Joseph predicted seven years of plenty to be followed by seven years of
famine. So pleased was Pharaoh with Joseph's performance, that he elevated Joseph to the
second highest political position in Egypt to supervise the storage of food during the seven
years of plenty so that the country would have food supplies during the famine. At the time of
his promotion, Joseph was thirty: "Joseph was thirty years old when he entered the service of
Pharaoh king of Egypt" (Gen. 41:46).

Joseph's task was to gather excess food during the seven years of plenty and store it for use
during the seven years of famine (Gen.41:34-43). The Bible implies that the 14-year period
foreseen in Pharaoh's dreams (seven of plenty and seven of famine) began immediately after
Joseph was made food administrator, because Joseph had said when interpreting Pharaoh's
dreams, "God will shortly bring it to pass" (Gen. 41:32). Also, the verses following the
passage quoted above to establish his age when Joseph "entered the service of Pharaoh king
of Egypt" said that "Joseph went out from the presence of Pharaoh, and went through all the
land of Egypt" to gather up "all the food of the seven years when there was plenty in the land
of Egypt" (41:46-48). Obviously, then, the Bible teaches that Joseph was thirty years old
when the seven years of plenty began.

This would mean that Joseph was 39 when he identified himself to his brothers who had come
into Egypt to buy grain. They came after the seven years of plenty had passed and "the
famine was sore in all the earth" (41:57; 42:1-3, ASV). Although he had immediately
recognized his brothers, Joseph did not reveal his identity to them. They bought grain and
returned home, and when the grain was consumed, Jacob ordered them to return to Egypt to
buy more (Gen. 43:1-2). On this second trip, Joseph made himself known to his brothers, and
by then, the famine was in its second year (45:6,11). So at this point, Joseph had to be nine
years older than when he had first entered into the service of Pharaoh at age 30.

Volume 1990 - 2002 Issue


Page 878 of 2049
Skeptical Review Edited by Farrell Till
Joseph sent his brothers back into Canaan to bring their father and relatives into Egypt where
they would have food to sustain them during the five remaining years of famine (45:11). If it
had taken no more than a year for the brothers to return home and bring Jacob and their
families to Egypt, Joseph would have been about 40 years old, according to the biblical
narrative. So if Jacob was 130 at this time, as the Bible claims, that would mean that he was
90 years old when Joseph was born.

Inerrantists will insist that men lived longer in those days and were therefore more vigorous in
their eighties than men of today, but that is an assumption that they have no proof for but
what the Bible says. To make such a defense as this, then, would be a resort to the familiar
tactic of circular reasoning or trying to prove inerrancy by assuming inerrancy. Besides, they
would still have the problem of explaining Abraham's statement quoted earlier, which plainly
indicated that even in biblical times it was considered unlikely that a man in his nineties could
sire children. If Jacob was 90 when Joseph was born, as the chronology cited in support of
this conclusion certainly showed, then he was at least 96 when Benjamin was born. When he
learned that Laban had given him Leah instead of Rachel, Jacob agreed to work seven more
years if Laban would give him Rachel too. Then "it came to pass when Rachel had borne
Joseph" that Jacob asked Laban for permission to take his wives and children and return to his
country... "for thou [Laban] knowest my service wherewith I have served thee" (Gen. 30:25-
26). As previously noted, however, Laban persuaded Jacob to stay and continue working in
exchange for all the spotted and speckled lambs and kids that would be born in the flocks.
When Jacob finally left Paddanaram, he said to Laban, "These twenty years have I been in
your house; I served you fourteen years for your two daughters, and six years for your flock"
(Gen. 31:41, NRSV). So if Jacob had felt free to go after Joseph's birth because he had served
Laban well, this must mean that Joseph was born at the end of Jacob's second seven-year term
of servitude or, in other words, 14 years after Jacob arrived in Paddanaram. Rachel died
giving birth to Benjamin en route to Canaan after Jacob had spent 20 years in Paddanaram, so
if Jacob was 90 when Joseph was born (as already shown), then he would have been at least
96 when Benjamin was born. At age 99, Abraham laughed when told that he would father a
son within a year, so the claim that Abraham's grandson Jacob sired a son when he was 96
should warrant at least a chuckle.

There is also one direct textual statement in the Bible that is inconsistent with the chronology
that fixes Jacob's age at 76-96 when he was in Paddanaram. As previously noted, Rachel's
concern for what Esau might do to Jacob for having deceived their father Isaac into
pronouncing a primogenitive blessing on him instead of the firstborn Esau was why she urged
Isaac to send him to Paddanaram to find a wife. However, just before the story of Jacob's
deception of Isaac was told in Genesis 27, the 26th chapter ended by saying that "when Esau
was forty years old," he married two Hittite women. If Esau was 40 at this time, then Jacob
was the same age, because they were twin brothers. After saying this about Esau, the very
next verse in the Genesis narrative begins to tell the story of Jacob's trickery in getting Isaac
to bless him. That story runs throughout the 27th chapter and ends by telling of Rachel's fear
that Esau might kill Jacob for his duplicity. Pretending then to Isaac that she was afraid Jacob
might do as his brother Esau had done and marry a Hittite woman, Rachel used this as an
excuse to urge Isaac to send Jacob to Paddanaram. Surely, then, thirty-six years had not
passed since Esau had married his Hittite wives. How convincing would her fear have been to
Isaac if Rachel had reached back thirty-six years for something to express concern about? The

Volume 1990 - 2002 Issue


Page 879 of 2049
Skeptical Review Edited by Farrell Till
fact that she used this as an excuse to get Jacob out of the country clearly implies that the
Genesis writer thought that Esau's marriages to the Hittite women and Jacob's deception of
Isaac were sequentially close events. Further indication that the two events were sequentially
close is evident in Esau's reaction to Jacob's flight:

Now Esau saw that Isaac had blessed Jacob and sent him away to Paddan-aram to take
a wife from there, and that as he blessed him he charged him, "You shall not marry
one of the Canaanite women," and that Jacob had obeyed his father and his mother and
gone to Paddan-aram. So when Esau saw that the Canaanite women did not please his
father Isaac, Esau went to Ishmael and took Mahalath daughter of Abraham's son
Ishmael, and sister of Nebaloth, to be his wife in addition to the wives he had (Gen.
28:6-9, NRSV).

To believe that Jacob was 76 when he went to Paddanaram, as chronological information


cited earlier indicates, requires us to believe that Esau was married to his Hittite wives for
thirty-six years before he realized that "the Canaanite women did not please his father Isaac."
That, of course, is ridiculous, because recognizing that one's parents are dissatisfied with his
choice of a wife would typically occur much sooner than that. So this spiteful marriage that
Esau entered into can only be seen as another indication that his marriage to the Hittite
women when he was 40 and Jacob's flight to Paddanaram both happened within a relatively
short period of time.

Furthermore, to say that Jacob was 76 when he left for Paddanaram requires one to believe
that 36 years in his life were skipped in the Genesis narrative. The writer detailed Jacob's life
more specifically than any other Genesis character, including even Abraham, yet he passed
over entirely 36 years of this central character's life, from age 40 to 76, if Jacob was indeed 76
when he went to Paddanaram. That seems very unlikely.

Obviously, then, whoever wrote this part of the story of his life thought that Jacob was much
closer to 40 than 76 when he went to Paddanaram. Certainly, the sexual escapades attributed
to him while he was there would be far more believable in a man in his forties than one in his
eighties.

Once again, we must conclude that the Bible narrative cannot sustain critical analysis. To
argue that the Bible is historically and chronologically accurate in every detail, as bibliolaters
claim, one must believe that Jacob, the grand patriarch of Israel, was an old geezer, who spent
the better part of his twenty years in Paddanaram engaging in sexual escapades with younger
women.

Reprinted from The Huntsville (Alabama) Times...

The Thrasher-Till Debate

Volume 1990 - 2002 Issue


Page 880 of 2049
Skeptical Review Edited by Farrell Till
Howard Thompson
Huntsville had its own cultural warfare recently (June 24-25). It was one of those little noticed
confrontations between the reason of secular society and the biblical faith of Christian
evangelicals.

Our local skirmish was a debate hosted by the North Huntsville Church of Christ. At issue
was the proposition that the fulfillment of biblical prophecy proves the Bible is the inerrant,
inspired word of God. Church-of-Christ preacher Tom Thrasher defended Bible inerrancy.
Atheist Farrell Till, a former Church-of-Christ preacher, challenged the validity of prophecy.

About 60 mostly older people attended the four-hour debate. The way to tell the true believers
from the skeptics was when 20 percent of the audience did not bow their heads in prayer.

Thrasher told of the prophecies Daniel made in Babylonia about 539 B.C. Thrasher
interpreted symbolic language about the horns of animals as proof that Daniel predicted the
fall of Babylon and the rise of four new kingdoms. His use of computerized displays was well
organized.

Till told the audience about historical evidence for the book of Daniel being written after the
events allegedly predicted. He pointed out mistakes in Daniel's inerrant words from God. It
seems Daniel wrongly identified a king's father four times and thought that Babylon fell to
Darius the Mede instead of Cyrus. For good measure, Till threw in Jesus saying that scripture
foretold the resurrection on the third day. Thrasher freely admitted that no such scripture
appears in the Bible. The audience was left to figure out how Jesus could have erred.

Till utilized reason and evidence to debunk biblical mythology. He challenged the audience to
wonder why an all-knowing deity would have his prophets write symbolically when specific
predictions were within his power. He challenged the audience to read the Bible for
themselves instead of relying on the interpretations of others.

Thrasher insisted that no disproof of his interpretation of Daniel was offered. He cited
scripture to show that writers of the time sometimes used words not in a literal sense. He
defended the lack of scriptural references on the basis that prophets could have said what was
attributed to them with the scriptures having been lost later.

Rare Confrontation: Public confrontations between believers and atheists are fairly rare.
More common are confrontations between those that believe the Bible is the inerrant word of
God and those who believe God transcends man's attempt to limit him to a single book.
Events like a debate on biblical prophecy are part of the cultural war evangelical Christians
wage against what they call humanism. It's a reflection of the evangelical campaign to inject
God's word into everything.

Biblical inerrancy is important to evangelicals because the Bible is the center of their private
and public lives. It is important to others because evangelicals are serious about making
biblical law supreme in the land. Evangelical successes in the Republican Party make their
dream of establishing evangelical dominion over everyone more than a vain hope.

Volume 1990 - 2002 Issue


Page 881 of 2049
Skeptical Review Edited by Farrell Till
The handful of attendees at this debate on biblical prophecy at least gives understanding of an
opposing view in a way they could later verify. No minds were change. At most, a few people
may have walked away with the seeds of change.

Welcomed Change: I must say that the debate was a welcome change from the strident,
opponent-bashing intensity of talk radio ax-grinders. How different things appear when two
sides speak opposing views calmly. The North Huntsville Church of Christ is to be
commended for its courage in providing a forum for the free exchange of ideas.

(Howard Thompson, P. O. Box 1782, Georgetown, TX 28627; e-mail gofreemind@aol.com)

EDITOR'S NOTE: Howard Thompson also attended my debate with Jerry Moffitt in
Portland, TX, in May 1994. After the debate, he sent a letter to Moffitt pointing out errors in
his probability arguments. Moffitt said that he would send the letter to Marion Fox, Moffitt's
moderator and the acknowledged formulator of the arguments, but Mr. Thompson informed
me at the debate with Thrasher that he had never received a response to the letter.

Our policy of renting tapes has been discontinued, but a video tape of the Thrasher-Till
Debate can be purchased for $5 postage paid.

The Sins of the Fathers


by Sol Abrams
It is most unfortunate that the people who claim that the Bible is inerrant have never taken the
time to read it carefully and to think about what they have read. If the creator or the creative
force gave man a brain, it was intended to be used for critical and logical thinking and not just
for memorization. The claim of biblical inerrancy can be disproved beyond any reasonable
doubt by the power of logical deductive reasoning. The disproof involves the examination of
two contradictory statements wherein if one is true the other must be false. The following
may be used to illustrate the argument.

Question: According to the Bible, does a son inherit the sins of his father? The answer to this
question must be either yes or no; it cannot be both. If the answer based upon a passage in the
Bible is yes, then any other passage in the Bible that says no must be false. Similarly, if the
answer based upon a passage in the Bible is no, then any other passage that says yes must be
false.

It so happens that there are some passages in the Bible that say the answer to the above
question is yes and others that say the answer is no. The passages that say yes are Genesis
9:20-25; Exodus 34:7; 2 Samuel 12:14; Isaiah 14:21; and Romans 5:19. The passages that say
no are Deuteronomy 24:16; Jeremiah 31:30; and Ezekiel 18:20.

Volume 1990 - 2002 Issue


Page 882 of 2049
Skeptical Review Edited by Farrell Till
I will compare 2 Samuel 12:13-14 and Deuteronomy 24:16, because these two passages
illustrate this contradiction most clearly, and both involve putting someone to death as
punishment for sin.

Second Samuel 12:13-14, And David said to Nathan, "I have sinned against the
Lord." Nathan said unto David, "The Lord hath put away thy sin, thou shalt not die.
Howbeit, because by this deed thou hast given great occasion to the enemies of the
Lord to blaspheme, the child that is born unto thee shall surely die."

Deuteronomy 24:16, The fathers shall not be put to death for the children, neither
shall the children be put to death for the fathers; every man shall be put to death
for his own sin.

It is interesting to note that David was not only spared from death even though he had sinned
but that his son was clearly put to death because of the sin of his father, thereby contradicting
the verse in Deuteronomy. The son, in effect, became the sacrificial lamb to atone for the sins
of his father, David.

In short, if the passage in 2 Samuel is true, then the verse in Deuteronomy is false and vice
versa. In any event, this proves that the Bible contains at least one false statement and is
therefore not inerrant (quod erat demonstrandum).

The above example is but one example of more than 200 directly contradictory passages in
the Bible, any pair of which can be used to disprove inerrancy. Furthermore, if the verse in
Deuteronomy is correct, it also proves that the concept of original sin is incorrect, because
this doctrine holds that the entire human race is responsible for the alleged "sin" of Adam.
Thomas Jefferson, who did not believe in the doctrine of original sin, stated in a letter to
James Smith (December 8, 1823), "Man once surrendering his reason, has no remaining guard
against the most monstrous absurdities and like a ship without a rudder is the sport of every
wind. With such persons, reason and the mind becomes a wreck." In this matter of biblical
inerrancy, many people have regrettably surrendered their reason to a claim that is not true.

(Sol Abrams, 132 Easthampton F, West Palm Beach, FL 33417-1922.)

From the Mailbag


You recently sent me the first three issues for 1996, and I can hardly believe my eyes. It is so
comforting to know that there are so many other people out there who have beliefs similar to
my own. Thank you!

I am 21 years old and was raised Catholic. I live in a small religious town in the middle of the
bible belt. People my age go to church functions for fun, families give a percentage of each
paycheck to their churches, and even go to church to vote! About five years ago, I decided the

Volume 1990 - 2002 Issue


Page 883 of 2049
Skeptical Review Edited by Farrell Till
Catholic religion was a joke and set out to find the truth about God and the "right" religion for
me. After reading the bible and applying some common sense, I came to the conclusion that
all religions are basically the same and God does not exist. This was a great relief to me, and I
have been (happily) an atheist ever since. In fact, I am kind of embarrassed that I believed in
something for so long just because other people told me to.

When I told my family that I no longer believed in God, they said I was going through a
phase, rebelling, etc. and assured me that I would snap out of it when I grew up. I hope they're
not holding their breaths.

Thanks again for The Skeptical Review. I thought I was alone for so long and you have shown
me differently. Enclosed is a check for a 2-year subscription.

(Linda Vincent, 1504 Rockwood Boulevard, Mulvane, KS 67110.)

EDITOR'S NOTE: Letters like this are a delight to receive, and they confirm something that I
have said many times: the Bible is its own worst enemy. If people would just read it and learn
what is in it, more and more people would be doing exactly what Ms. Vincent did. At the end
of my public debates, I usually say to the audiences that I hope my presence in the community
has at least created an interest in reading the Bible. I then urge the people to go home and read
it, because if the day ever comes when people actually read their Bibles rather than letting
them be dust-collectors in their homes, there will be many more departures from the faith.

In people like Ms. Vincent, I also see hope for the future. If any substantial change is to
happen in religious thinking, it will have to come through her generation. Those who are my
age are usually too set in their ways to change, but the young are almost always the ones
responsible for changes in society. I really can't imagine people her age and younger growing
up in a highly technological society and yet continuing to believe primitive superstition and
nonsense. The Internet Infidels is an organization made up primarily of people who went
through high school and college during the computer age, and their work on behalf of
freethought on the internet will surely bring about profound changes in religious thinking.

I can certainly understand Ms. Vincent's embarrassment when she reflects on what she used to
believe. That is a perfectly normal reaction, which everyone who has been down that road
experiences. How would she like to know that she spent 12 years of her life preaching
religious nonsense, as I did? Now, that's really embarrassing!

I look forward to each issue and find them very informative. I especially enjoy the debates
that you have with the inerrantists.

Bertrand Russell was my mentor many years ago, which is when I would say I became a
confirmed atheist. I've had no reason since to change my mind. However, I have a problem. In
the Autumn 1995 issue, your lead article is "The Historicity of Jesus." I agree with your main
points and especially with the fact that there are no contemporary accounts of Jesus's
activities, which there should have been if "tens of thousands flocked to him." I find it

Volume 1990 - 2002 Issue


Page 884 of 2049
Skeptical Review Edited by Farrell Till
inconceivable that there would not be a written record. Therefore, I took the position that
probably Jesus never did exist. After all, the life of Jesus from the nonsensical virgin birth,
walking on water, etc. is so irrational.

Here is my problem. I am a Jew, a Jewish atheist. If I believe that Jesus is a fictional character
because there is no contemporary confirming evidence, then to be consistent I must say the
same about Moses. There are no contemporary Egyptian records of Moses, and I find it hard
to believe that there would not have been records of a person of his stature as leader of the
Hebrew exodus. Aside from the hocus-pocus that Moses was supposed to have done, I
considered him a historical figure. Now placing Moses in the same category as Jesus, i.e.,
both are fictional, I feel a sense of loss.

(Arthur Robins, 69 Millersgrove Drive, Willowdale, Ontario, Canada M2R 3S1.)

EDITOR'S NOTE: Mr. Robins' sense of loss is certainly understandable. I suspect that it is
akin to the feeling that Christians experience when they come to realize that they have been
duped. As for the historicity of Jesus, I personally suspect that no such man ever existed, but I
also recognize that there could have been a Jesus whose life was simply exaggerated and
legendized beyond recognition. I think the same would be true of Moses and other biblical
characters. If a man named Moses ever existed, certainly he wasn't the Moses who parted the
Red Sea, talked to Yahweh on Mount Sinai, and performed the other extraordinary feats
attributed to him.

Don't let the losers define the terms! Who controls the words controls how we think about the
debate. Perhaps because of the way humans think, i.e., in words, clever use of loaded words
manipulates the firing of synapses in our brains, driving us to involuntary or unthinking
conclusions.

I bring this up, because it seems that atheism relies on disproving biblical literalism. Isn't that
like letting losers define the word god and then proving there is no God because the definition
is preposterous?

(David Nixon, Box 562, Marmora, NJ 08223.)

EDITOR'S NOTE: Some think that it is improper to debate the Bible at all because it dignifies
an obvious collection of primitive superstition and leaves the impression that it is important
enough to warrant public debate. I can't agree with this position. No matter how absurd belief
in the Bible is, those who believe in it exert an influence on our society that needs to be
opposed. Otherwise, we stand to lose freedoms and retard social and technological progress. I
have often said that if the Mother Goose Tales were believed in our society to a degree that
influenced social and legislative matters, rational people would have no choice but to oppose
Mother Goose believers. On the other hand, I agree with what I think Mr. Nixon was basically
trying to communicate: he who asserts must prove. Too often we allow the believers in
ridiculous premises to put on us the burden of proving that their premises are not true, so we
should constantly remind them that they are the ones who should prove that god exists, the

Volume 1990 - 2002 Issue


Page 885 of 2049
Skeptical Review Edited by Farrell Till
Bible is inspired of god, the Bible is inerrant, etc. When they refuse to do so, however, I think
it is appropriate for skeptics to state the reasons why they do not believe in ridiculous
religious premises if for no other reason than to try to enlighten the few believers in religious
superstition who may be willing to listen to common-sense reasoning. That is different from
assuming the burden of proving that ridiculous premises are not true.

Somehow my name was placed on your snail mailing list and I have received several copies
of your newsletters The Skeptical Review. After reading through these, I am quite baffled.
You seem to think that refuting the claims of Biblical Inerrancy constitutes a refutation of
Christianity, even God's existence. If you can't see the illogic of this idea, you have a serious
blind spot in your thinking or understanding of Christianity. (And please, leave out the
testimonials about having been a brainwashed inerrantist before you saw the light. Just
because you didn't use your brain then is no reason for someone to think you are using it
now).

The closest you come to addressing a fundamental point of Christianity is your response to
Matthew Perman's highlighting of historical facts (July/ August 1996). There you reveal
clearly that you do not understand what critical scholarship is nor what they have established.
I could list such scholars for you by name but you obviously do not know one from another.
Your "points" (made by others and reiterated by you) are irrelevant to the established history;
You then have the audacity to say that the established history is "based on the assumption that
the NT records are historically accurate." How embarrassing it is for someone to have
assumed that you were much more studied than you obviously are. How insulting it is for you
to say such stupid things when you are the one who has chosen to ignore or not learn what
critical scholars have to say about the history of Jesus.

You obviously don't want to tackle fundamental issues (the existence of God, the history of
the resurrection claim, etc.) in a rational way so Biblical Inerrancy is handy a straw man you
have set up to bait Christians who don't realize the waste of time the issue is. But how about
being real. You don't believe in God so you have no objective morals or ethics (if you think
you do, please explain). You have no basis for condemning or applauding anyone other than
personal preference so what is your motivation for your "mission"? You obviously think that
people should not promote certain beliefs and claims and that there is a value to you opposing
these; I sense that the answer to why will betray your hypocrisy.

(Jeffery McCain, 303 Navajo Springs Road, Diamond Bar, CA 91765; e-mail
jmccain@cris.com.)

EDITOR'S NOTE: Mr. McCain's e-mail letter has been copied to this column exactly as he
sent it. I don't recall how his name came to be on TSR's mailing list. People will sometimes
ask us to add the names of friends and relatives who they think will profit from receiving the
paper. Obviously, whoever thought that McCain needed to receive the paper was right, but
whether he will profit from it or not is another matter. Many readers who also subscribe to
errancy@atheist. tamu.edu will immediately recognize his name. He came onto the list, at my
invitation, apparently thinking that he was going to blow the errantists away with his

Volume 1990 - 2002 Issue


Page 886 of 2049
Skeptical Review Edited by Farrell Till
invincible logic, but very quickly he saw that he was the one whose position was in serious
trouble. Despite several direct challenges that he defend wild assertions like those he made in
his letter above, he has yet to state a clearly recognizable position and then defend it. As I am
writing this, he is known on the errancy list as an asserter of unsupported claims and an
evader of arguments, who seems to think he has the right to do what Mr. Nixon criticized in
the letter immediately before this one, i.e., insist that skeptics and atheists assume the burden
of proving that major theistic claims are not true.

He seems to have read just enough philosophy to bandy philosophical terms around, as if he
thinks that sounding informed will make readers believe that he is. From the best I have been
able to determine from his postings on the inerrancy list, McCain believes in the
presuppositionalist premise that nature is amoral and so it would be impossible for morality to
exist unless a deity "outside of time," who is transcendent to the amoral universe, infused into
humans the ability to think and to behave as moral entities. If this is a misrepresentation of his
belief, then he is to blame for refusing to clarify his position to the many Errancy subscribers
who have asked him to.

If he wants to think that I am stupid, he is of course free to do so. He won't be the first one to
so think. If, however, he should ever decide to take a clear stand on any issue related to
biblical inerrancy, I would be more than glad to direct all of my stupidity toward responding
to any evidence that he thinks he can present in support of the biblical inerrancy doctrine.

[Here are] a few interesting nuggets to add to your "Family Values" piece: (1) Ishmael was 14
years old when Abraham sent him away (Gen. 16:16; 21:5). Are we to believe that Hagar
carried him piggyback (21:14)? Obviously, the author didn't grasp the chronology here and
erroneously thought Ishmael was a small child. (2) Sarah was at least 65 when Abraham first
trafficked in her honor. (Genesis 17:17 establishes that she was 10 years younger than her
husband, and 12:4 that he was 75 when he left Haran before going into Egypt.) And she was
still a "looker"? And this was even before facelifts! (3) Both versions of the story have the
innocent dupes incurring God's wrath, not the duplicitous Abraham, who was God's pet, even
though 20:5-6 admits that Abimelech was really blameless. (4) The author of Genesis had
three versions of the same legend, and rather than determine which one was the real one, he
simply ran all three. In Genesis 26, Isaac's the culprit, Abimelech is still king, and he sees Ike
and Becky copulating in public (26:8)! (I got your "Family Values" right here!) The reference
to the Philistines is an anachronism, since the Philistines were not to settle in Palestine for
another seven centuries. They were, however, there when the actual author (not Moses) wrote
Genesis, which explains his mistake and disproves Mosaic authorship.

Hello! Incest, people! That's a Yahwistic no-no! And these were the holy, chosen ones of
God???

(Stephen Van Eck, RR 1, Box 62, Rushville, PA 19939.)

Volume 1990 - 2002 Issue


Page 887 of 2049
Skeptical Review Edited by Farrell Till
EDITOR'S NOTE: Van Eck's letter arrived after the front-page article had already been
written. It's a good supplement to the series on "Family Values" that will continue through at
least the next issue.

Thank you for your excellent rebuttal to Hogan's defense of the Tyre prophecy. In that
context, Hogan makes an error that is worth mentioning in some detail as it is used so often
by Bible-believers.

When Hogan brings in Alexander the Great, as an explanation for a shift in pronouns, he is
invoking an explanation that is not in the Bible. Such an explanation is seized upon, of course,
because it makes sense in the light of what we know today, namely that Alexander the Great
did indeed destroy Tyre. However, isn't this just another way of assuming that the prophecy
must be right? The prophecy must be right, the believer reasons, so we are justified in
attaching whatever historical event we know of that will make the prophecy work. If a
prophecy has not been fulfilled, then it will be fulfilled in the future because it must be right.
This whole line of thinking is based on the premise that biblical prophecy cannot err! If one is
going to make that kind of assumption up front, one should not even pretend to be engaged in
a rational, impartial discussion of the matter.

The error above is better understood if we look at the other side of the coin. Suppose Till
assumed (without any evidence) that the Tyre prophecy was false and supplied a historical
event (to explain the shift in pronouns) that made that prophecy false. What would Bible-
believers think of those apples? No question about it, they would scream "foul." Yet, Till
would be doing no worse than Hogan did in his defense of the Tyre prophecy. Hogan showed
that there is a possible solution, however far- fetched. In our hypothetical example, Till shows
that there is a possible error, however far-fetched. Reject one side of the coin, and you reject
both sides of the coin. How then can we proceed?

An impartial analysis requires that we do not add new information to the text unless we can
rule out all competing information. If a scenario is advocated, then all competing scenarios
must be eliminated beyond a reasonable doubt -- using only the context of the passage. That is
to say, we seek the best explanation. Thus, Hogan may not bring in Alexander the Great to
explain the shift in pronouns unless he can eliminate, beyond a reasonable doubt, all other
scenarios. After all, the author of that passage may have had something else in mind! Horrors!
He might even have been wrong! Obviously, Hogan cannot do this without first assuming that
the prophecy is correct.

That only leaves Nebuchadnezzar as the named conqueror. Given Ezekiel's propensity to shift
pronouns, as aptly shown by Till, given the fact that the shift occurs in the middle of a
sequence that would otherwise envision a sweeping destruction of Tyre by Nebuchadnezzar,
and given that the "they" could easily apply to the men, horses and chariots of
Nebuchadnezzar's army, we can see that Hogan's defense is not much of an argument. I
suspect that the main prophecy was made while Nebuchadnezzar seemed invincible, and that
Ezekiel's admission of his failure was later added so that scripture would acknowledge a
"done deed." One cannot easily erase scripture already in circulation, but one can add to it in

Volume 1990 - 2002 Issue


Page 888 of 2049
Skeptical Review Edited by Farrell Till
order to save face. The above reasoning also applies to prophecy that was obviously intended
to be fulfilled during those days but was not. The claim that such prophecy might yet be
fulfilled in some distant future draws its sole justification from the assumption that biblical
prophecy cannot err. We might just as easily, and with much more justification, claim that
such prophecy was intended for biblical times and failed. Such prophecy must be judged in a
context where it is meaningful, and many of these "punishment" prophecies become an absurd
joke when deferred to the indefinite future.

(Dave Matson, P. O. Box 61274, Pasadena, CA 91116; e-mail


103514.3640@compuserve.com)

EDITOR'S NOTE: Dave Matson is a frequent contributor of both articles and letters to TSR. I
always appreciate his comments. I will remind readers again that Dave Matson publishes and
distributes through The Oak Hill Free Press some useful materials on evolution, creationism,
and other subjects of interest to freethinkers. His own book that answers the creationist
arguments of Kent Hovind would be useful in combating the widespread attempts to require
the teaching of creationism in public school. Inquiries should be directed to his address above.

I find it interesting that The Skeptical Review, July/August 1996, should begin with the
following quote by W. K. Clifford, "It is wrong, always, everywhere, and for everyone, to
believe anything upon insufficient evidence." This quote reveals two absolute presuppositions
in which the author has placed his faith: 1) that there is such a thing as "sufficient" evidence,
and 2) that finite individuals are ontologically and epistemologically capable of perceiving
and processing "sufficient" evidence. Moreover, consider the morally absolute nature of such
a statement, "It is wrong, always, everywhere and for everyone..." and that from a detached
skeptic!

He demonstrates that all knowledge is creedal in its nature and foundation. We all believe
certain presuppositions, through which we then interpret and know "objective" data. This is
the fatal flaw in the defense of the fundamentalist; he has agreed to work with a system given
to us by Descartes; one in which we divide objective and subjective knowledge, supposing
thus to reveal indubitable certainty.

The Cartesian pursuit of certainty has ironically produced a culture in which nothing is certain
or beyond the tenacious grasp of doubt. The fundamentalist believes, along with the skeptic,
that the two above presuppositions are true, thus guarenteeing [sic] a lost battle. Your
magazine proves the ineffectiveness of his defenses. The unstated truth, however, is that the
skeptic begins with precisely the same creed (i.e. he begins with the two above
presuppositions, or beliefs) that the fundamentalist uses. The fundamentalist is wrong; the
skeptic is unaware. Both of them in faith accept the Cartesian model as truth; was he right? Is
there sufficient evidence for an answer? Such evidence can only be had on faith, and the circle
is now complete.

(John Boonzaaijer; e-mail jboonz@tums.org)

Volume 1990 - 2002 Issue


Page 889 of 2049
Skeptical Review Edited by Farrell Till
EDITOR'S NOTE: And so we hear from another presuppositionalist theist, who seems to
have read just enough philosophy to throw out a few names and impressive-sounding
expressions that will fool the uninformed into thinking that he knows his subject, but when we
strip all of this jargon away, what do we have? In Mr. Boonzaaijer's case, not much. The fact
is that these pseudophilosophers seem to have a great deal of difficulty stating their positions
in clear, concise language. Readers spend half their time backing up to read again and wonder
what it is these guys are trying to say.

At any rate, I sent Mr. Boonzaaijer (this may not be his real name) a response in which I
pointed out that W. K. Clifford was not making any claims about absolute knowledge; he was
simply commenting about irrational belief. Mr. B may think that it is never possible to have
sufficient evidence to know anything, but Clifford was not talking about "knowing" things; he
was talking about "believing" things. Does Mr. B think that it is impossible to have sufficient
evidence to form reasonable beliefs? I have never been to Tokyo, Japan, but I believe that
there is a Tokyo, Japan, and I think I have sufficient evidence to justify that belief.

Mr. B's letter came by e-mail. I sent him a response in which I pointed out that Clifford was
not talking about knowing but believing, and I never received a reply. I sent him a message
asking for his mailing address, but he didn't answer.

My granddaughter just gave me the January/February 1996 copy of The Skeptical Review. I
was especially interested in the Horner-Till Debate. I started at that school [Seattle Pacific
University] in 1945 when it was still Seattle Pacific College.

I enjoyed the time I spent there and, in fact, made a lot of friends. They are good and sincere
people. Reading about the debate reminded me of an experience I had in physics class. One
day, they were knocking around the theory of evolution. The instructor was an intelligent and
educated man and well qualified in his field. He explained that evolution couldn't be possible
because if you saw a row boat next to an ocean liner, you would not assume that one evolved
from the other. I don't need to list the reasons that it was a poor analogy. It was only one of
the experiences at SPC that hastened me on the way to becoming a skeptic.

If I am not too late, I would like to receive the tape and the transcript of the debate. I am
enclosing $12. If these materials are not available, please add the money to the subscription of
my granddaughter. [Granddaughter's name and address deleted.]

(Claire Elliott, 3099 Erskine, Apt. 515, Kodiak, AK 99615-6392.)

EDITOR'S NOTE: The video tape of the debate at Seattle Pacific University contained gaps,
so we are not distributing it. From both the audio and the video tapes, a complete, accurate
transcript was made that has been approved by both Michael Horner and me. It is available
for $3, postage paid. The issue was the resurrection of Jesus.

Volume 1990 - 2002 Issue


Page 890 of 2049
Skeptical Review Edited by Farrell Till
I have received a copy of Skeptical Review with pleasure and appreciation. I wish more
people have [the] chance to read these marvelous works. I considered myself a lucky man for
knowing the truth provided by you.

I have picked up [the] Bible and tried to read it many times, but when they say that the stories
in [the] Bible must be accepted as true stories, I cannot make my brain accept it, so I put them
down. How can a man read a fairy tale childish story and accept them as truth? Not me, I have
a feeling as Albert Einstein had when he said, "There are two things that are indefinite to me;
the first is the universe, the second is the stupidity of mankind, and I am not sure about the
first one."

So when I get your booklet, I feel very happy and released. I realized that among [a] heavily
stupid society, there still be a wise, brave, selfless hero who is loveful and sincere enough to
lit the light in this dark world so that the men with good eyes can see what is right, what is
wrong, what is harmful.

Those people who say love, love, [a] hundred times, thousand times, even million times, but
they take away the wisdom, the reason, and [the] ability to think, to discover the truth freely.
How can I say that they love mankind? To me, they are destroyers who bring the darkness to
this world, not the light as they say.

So please keep doing what you are doing now. The wise men appreciate and honor you.
Enclosed is a check [for] $20; it is too small compared to the value of your work. Thank you
very much.

(Anek Imphita, 11603 East Virginia Drive, Aurora, CO 80012.)

EDITOR'S NOTE: I appreciate Mr. Imphitak's compliments and support. Obviously, his
native language is not English, and as one who once had to struggle with writing in another
language when I was a foreign missionary, I appreciate how well he has learned English. I
published the letter essentially as he wrote it with only occasional brackets to supply words he
had omitted. I am happy to have him as a new subscriber.

As a very satisfied subscriber, I love your product! As an associate of Westar Institute, a


"layman" backer of the Jesus Seminar, I like what's being done.

The Skeptical Review, July/August issue, had two statements attributed to the Jesus Seminar:
(A) the resurrection of Jesus happened in the minds and hearts of early Christians, and (B) if
Jesus was crucified. his body was likely thrown into a garbage dump and later eaten by dogs.
The votes taken by the fellows of the Jesus Seminar on the passion story don't quite match
these statements. Attached are the votes of the fellows. Items 57, 58, and 59 come closes to
the disposition of the body of Jesus. I hope you will select a couple of the items voted on and
comment on them in an issue of The Skeptical Review, especially where you disagree with the
vote.

Volume 1990 - 2002 Issue


Page 891 of 2049
Skeptical Review Edited by Farrell Till
When the prime mover of the Jesus Seminar speaks about the overall results of the J. S. vote,
it's his view of the proceedings. The fellows aren't all in lockstep with Dr. Robert Funk. Bob
knows what will grab an audience and the press. I like what he has to say.

John Dominic Crossan certainly has his own ideas, and they are not all accepted by his peers
in the seminar. I've been told Dominic had a session with Bill Buckley where Buckley won!
I'm really sorry the program didn't make it here to the west coast.

I like your statement, "If Jesus did not rise from the dead, then I don't have much hope at all,
so Ill live my life accordingly." Didn't Lin Yutang have the same approach? In a book he
published in 1938, he stated a belief in living life to the full. I agree! Yesterday, I found a
1959 book by Dr. Lin, From Pagan to Christian, which is the personal account of a
distinguished philosopher's spiritual pilgrimage back to Christianity. In a sentence, Dr. Lin
now believes there are limits of the knowable for the mere rationalist, and those who accept
God have a more mature intelligence. I don't know of his notion of God -- yet.

At a Thursday evening session with Dr. Funk, I quoted a sentence from an article: "We come
from the void, and we return to the void." Bob didn't agree. His goal is to find what Jesus saw
in the parables. I've told Dr. Funk that I'm a mole for Cardinal Joseph Ratzinger! Now I'm a
mole for Farrell Till. (Bob laughed at me on Ratzinger.)

If the passion votes grab you, I have back issues of the house magazine and can provide the
voting results on past events. Holler if you want them.

(Tom Healy, 1345 Felder Road, Sonoma, CA 95476-7806.)

EDITOR'S NOTE: Mr. Healy's letter is a composite of a letter and a postcard. The postcard
was sent merely to correct some information in his letter about the Chinese author Lin
Yutang.

The Jesus Seminar materials that Mr. Healy enclosed with his letter contain 85 different
beliefs associated with just the trial and crucifixion of Jesus. These are too numerous for me
to comment on. It was interesting to notice that none of the points listed included a direct
consideration of whether Jesus actually rose from the dead. Anyone interested in a copy of
them should contact Mr. Healy or else the Westar Institute, P. O. Box 6144, Santa Rosa, CA
95406; telephone (707) 523- 1323; fax (707) 523-1350.

The quotation in TSR was derived from a newspaper article that quoted several seminar
members after the resurrection meeting was over. I believe that John Dominic Crossan was
the one who said, according to the article, that if Jesus was really crucified, his body was
probably thrown into a garbage dump and eaten by dogs. I believe that Dr. Robert Funk was
the one who said that Jesus was resurrected only in the minds and hearts of early Christians.
At any rate, the gospel resurrection accounts are obviously viewed with much skepticism by
members of the seminar. That the trial account recorded in Mark 14:53-65 was authentic
received only a weighted average of .05% from the seminar.

Volume 1990 - 2002 Issue


Page 892 of 2049
Skeptical Review Edited by Farrell Till
Anatole France has given us the wittiest and best definition of theology that I've ever read:
"Theology is that science which treats of the unknowable with infinitesimal exactitude."

From my Roman Catholic youth, I remember that "Holy Mother Church" was obsessed with
details and numbers: so many joyous mysteries, so many sorrowful mysteries, so many deadly
sins, so many commandments, so many holy days and saints, all neatly polished and
packaged. It gave a sense of comfort, like primly stacked fruit in a supermarket. One had to
accept the entire consignment, but pull one apple from the bottom layer, and the entire edifice
came tumbling down. So it is with theology. The grand design seems impressive enough, but
examine the details of any Christian theology and conflicting vexations abound, vexations
that led the church doctor Tertullian to the absurd conclusion, "I believe because it is absurd."
How would that stand up in a court of law or a science lab?

Christian theology maintains that we mortals are, in fact, immortal. If the faithful truly
believed this, there would be no tears at Christian funerals. Fundamentalists insist that the
bible is clear and unequivocal, that it says what it means. Yet when the scripture spins myth
as in the Genesis creation stories, "what it means" becomes remarkably elastic.

Some liberal apologists claim that "day" in this biblical context means eons! Fundamentalists,
conversely, are left shamefacedly defending an earth no older than about six thousand years.
Accepting the fundamentalist dogma that man and dinosaur lived at the same time, can you
imagine the carnage as Noah welcomed Tyrannosaurus Rex and Triceratops aboard his ark,
not to mention the monumental task of dinosaur dung removal? Despite the bible's "clarity,"
fundamentalism has spawned innumerable sects, all squabbling over the "true meaning" of
holy writ.

When religious zealotry rears its head, we reap intolerance, alienation and brutality. Witness,
for example, the madness in Northern Ireland, Bosnia, and the Middle East, which certainly
can't be trace to secular humanism.

So let us be about the business of understanding and improving this world and its creatures.
And the one medium of our disposal for achieving this purpose is science with conscience.
We can affirm this earth, warts and all, with joy and compassion, honesty and love, but
without theology and without illusion. After all, theology is much like Noah's ark -- full of
holes.

(Fred Ehrstein, 9 Westwood Drive, Belleville, IL 62226.)

EDITOR'S NOTE: Well, what can I really say about Mr. Ehrstein's letter except, "Why can't
the religiously superstitious see this too?"

Volume 1990 - 2002 Issue


Page 893 of 2049
Skeptical Review Edited by Farrell Till

The Skeptical Review


Volume Eight – 1997
Farrell Till, Editor

• January/Feburary Volume Eight, Issue One


• March/April Volume Eight, Issue Two
• May/June Volume Eight, Issue Three
• July/August Volume Eight, Issue Four
• September/October Volume Eight, Issue Five
• November/December Volume Eigh, Issue Six

Volume 1990 - 2002 Issue


Page 894 of 2049
Skeptical Review Edited by Farrell Till

Skeptical Review
Volume Eight, Issue One
January/February 1997
Farrell Till, editor

• Still More Family Values


Till writes more on the 'Family Values' found in the Christian Old Testament

• Still Standing on Sinking Sand


Till's second reply to Perman's defense of the Jeremiah prophecy

• A Reply to Wilhelm Schmitt


Yoel shows that Schmitt's knowledge of Hebrew isn't all it's cracked up to be

• A Straw House Amid 10-Foot Waves


Matthew Hogan defends a prophecy in Ezekiel 26

• The Romans, Greeks, and So Forth


Till replies to Hogan's article on the prohecy in Ezekiel

• Distinguishing Fact From Fiction in the Man from Galilee


Van Eck attempts to tackle the problem of the historical Jesus

• From the Mailbag

Still More Family Values

Volume 1990 - 2002 Issue


Page 895 of 2049
Skeptical Review Edited by Farrell Till
Our last quest for biblical family values left us contemplating the public promiscuity,
deception, sibling rivalry, parental favoritism, and ethnic prejudice of the family of Isaac and
Rebekah. When we left them, Rebekah, fearing that her favorite son Jacob might be killed by
his twin brother Esau for having cheated him out of his birthright, had convinced her husband
Isaac to send Jacob to live with her brother Laban in Paddanaram, ostensibly to look for a
wife among the daughters of Laban but actually to protect him from Esau, who had threatened
to kill him.

We find, then, that half-truths were evidently a part of family values in biblical times, for in
telling Isaac that her life would be ruined if Jacob married "a wife of the daughters of Canaan"
as Esau had done (Gen. 27:46), Rebekah wasn't actually telling a lie, because Esau's Hittite
wives had been "a grief of mind" to her (Gen. 26:35), but she used this truth only to conceal
her real motives. She didn't want Esau to kill Jacob, and she didn't want Isaac to know about
the family rift that her own deception and favoritism to Jacob had precipitated.

By the way, we should notice in passing that when Esau "saw that the daughters of Canaan
did not please his father Isaac," he went to Ishmael (his father's half-brother whom Abraham
had sired during his escapade with Sarah's handmaid Hagar) and took Ishmael's daughter
Mahalath as another wife "in addition to the wives he had" (Gen. 28:8-9). Spite, then, seemed
to be another "family value" in biblical times, as was also polygamy, which will be addressed
later.

Anyway, Isaac sent Jacob to Paddanaram, where he had the various sexual adventures
discussed in "Jacob an Old Geezer?" (TSR, November/December 1996, pp. 8-9,11). From this
tale, of course, we can only conclude that polygamy was a family value in those days, because
Jacob didn't just marry the two sisters, Leah and Rachel, but also accepted their handmaids as
concubines with whom he sired four sons. The fact that Yahweh selected Jacob's sons born of
these polygamous relationships to be the tribal heads of his specially "chosen people" must
mean that Yahweh didn't consider polygamy to be morally improper. In fact, Yahweh's
approval of Jacob's cozy little arrangement was at least implied where the Bible declares that
"God remembered Rachel," who until this time had been unable to bear children, "and opened
her womb" (Gen. 30:22). Prior to this, Leah had experienced temporary sterility (a
contributing factor to the duel of the handmaids, as each sister tried to outdo the other by
producing children through their handmaids Bilhah and Zilpah), and "God hearkened to Leah"
and ended her sterility (v:18). Apparently, it never occurred to the rivaling sisters that at least
part of their "sterility" problems was surely due to spreading the sexual activities of an 85-
year-old husband a bit too thin by having him try to sire children by four different women, but
who are we to question the inspired word of God? If it says that God hearkened to Leah and
Rachel and opened their wombs, then they must have been genuinely sterile, and God must
have intervened to remove their sterility. In so doing, God was surely conveying that he didn't
see anything wrong with Jacob's cozy little setup and that he even encouraged it by
intervening to have the arrangement produce children to become the fathers of his specially
chosen people.

Jacob's little love nest in Paddanaram comes about as close to "free love" without actually
crossing the line as one can imagine, but the sexual part of Jacob's family life isn't the only
"value" that we should look at. During Leah's and Rachel's rivalry for their husband's sexual

Volume 1990 - 2002 Issue


Page 896 of 2049
Skeptical Review Edited by Farrell Till
attention, Leah's eldest son Reuben "found mandrakes in the field and brought them to his
mother Leah" (Gen. 30:14), and Rachel asked Leah for some of the mandrakes. "Is it a small
matter that you have taken away my husband?" Leah responded. "Would you take away my
son's mandrakes also?" Rachel then struck a deal with Leah. She would give up her sexual
turn with Jacob that night in exchange for some of the mandrakes (v:15). So when Jacob came
home from the field, Leah went out to meet him and said, "You must come in to me, for I
have surely hired you with my son's mandrakes." And so Jacob "lay with her that night"
(v:16). There always seemed to be plenty of Jacob to go around.

Jacob's sexual prowess, in fact, must have been something indeed. Usually men are the ones
who are willing to pay for sexual favors, but even after the 85-year-old Jacob had spent a
whole day working in the field, at least one woman in his life was willing to pay for his stud
services. At any rate, this deal that Leah struck with Rachel warrants our attention. Mandrakes
of the Mediterranean region produced a narcotic effect when they were eaten, and even as late
as the Middle Ages, they were used to dull sensitivities in patients undergoing surgical
procedures. Mandrakes belong to the family of plants known as Solanaceae (sometimes
called Nightshade), and although some of them are popular food plants, many of them, such
as jimsonweed, tobacco, morning glory, and mandrake, are sources of powerful and even
dangerous drugs.

The mandrake was also superstitiously considered both an aphrodisiac and a charm for
pregnancy, so considering the competition for Jacob's sexual attention that dominates the
story of his sojourn in Paddanaram, the Genesis writer may have intended readers to
understand that Leah and Rachel were using mandrakes as aphrodisiacs or pregnancy charms.
Nevertheless, their use for either purpose would have required that they be consumed, and so
the sisters still would have experienced the narcotic properties of the mandrakes. In effect,
then, Leah and Rachel transacted a drug deal, and those who clamor in today's society for a
return to biblical family values would soundly condemn parents who use and traffic in drugs.
In Bible classes, however, they honor Leah and Rachel as the mothers of God's chosen
people, apparently never recognizing the inconsistency.

Like mother, like son proved to be true in the case of Jacob's relationships with his children,
for as his mother Rebekah had shown favoritism to him, Jacob had a favorite son whom he
"loved more than all his children" (Gen. 37:3). This was Joseph, of course, whose favored
treatment by his father caused such jealousy in his brothers that they threw him into a pit,
waited for a caravan of traders to come by, sold him into slavery (vs:23-28), and then dipped
Joseph's tunic in goat's blood to make it appear that he had been devoured by wild animals
(vs:31-33). What we find in the Bible, then, are three successive patriarchal families that
showed favoritism to certain ones of their children. Abraham favored Isaac over Ishmael,
Rebekah favored Jacob over Esau, and Jacob favored Joseph over all of his children. It seems
safe to conclude, then, that parental favoritism was very much a part of "traditional family
values" in biblical times, but for some reason we don't hear Christians advocating it today.
They seem to be lost in a sort of biblical limbo, wanting on the one hand a return to family
values of biblical times but on the other hand not wanting a return to those biblical values that
time and experience tell them are really not good ideas.

Volume 1990 - 2002 Issue


Page 897 of 2049
Skeptical Review Edited by Farrell Till

Still Standing on Sinking Sand


Farrell Till
In his continuing effort to defend the unlikely New Testament claim that Jesus rose from the
dead, Matthew Perman, in typical apologetic fashion, made so many unsupported assertions
that I could not adequately respond to them in a single article. This time I will address his
"arguments" that I didn't have space to comment on in my last article. Then if Perman wants
to respond to my rebuttals, he may do so in a later issue.

Perman expressed surprise that I would "reuse Dan Barker's argument" that the doctrine of a
bodily resurrection had evolved from an earlier belief in just a spiritual resurrection. Well, in
the first place, this is not "Dan Barker's argument"; it was an alternative view of some
scholars long before Barker used it in his debate with Michael Horner. I am not saying this to
take away any credit that is due Dan Barker but simply to point out a fact that Perman would
surely know if he had researched the resurrection issue as thoroughly as he apparently wants
us to believe he has. If Perman would tear himself away from the fundamentalist works that
he obviously spends a lot of time reading, he just might find that there is much more to this
issue than the simplistic, illogical apologetic arguments that the likes of Josh McDowell,
Norman Geisler, and Gleason Archer keep recycling for the benefit of Christians looking not
for truth but for something-- anything--to extenuate their irrational presuppositions.
Furthermore, I suspect that Perman's "surprise" that I would "reuse" this argument is more a
matter of dismay than surprise, because he was at the debate when Dan Barker used the
argument against Michael Horner, so Perman knows that Horner's only response to it was to
quibble that a statement about a "spiritual book" would not be intended to mean that the book
was made of spirit. A flagrant equivocation like this hardly constitutes a refutation of the
argument.

To Perman's credit, he has at least tried to refute the spiritual-resurrection argument. Central
to this argument is the fact that the apostle Paul, in his famous defense of the resurrection in 1
Corinthians 15, made no references at all to an earthquake, an empty tomb, women, angels, or
other elements that figured so prominently in the gospel accounts of the resurrection. Perman
explained Paul's silence on these matters by claiming that Paul was merely reciting a
"formula" of a "creed" that was meant to be only "a summary, or brief outline, of the core of
Christian beliefs." For the sake of argument, let's just assume that this is so. How would that
explain the fact that the whole of Paul's writings, which constitute the major part of the New
Testament, made no references to any of these? Nowhere did Paul mention Roman guards,
women who visited the tomb and found it empty or an angel that descended in an earthquake,
rolled the stone away from the tomb, and announced to the women that Jesus had risen. In
fact, Paul nowhere mentioned the virgin birth of Jesus or the place of his birth or gave any
chronological indication of when his resurrected savior-god had lived. Perman may want to
cite 1 Timothy 6:13 as a text that put Jesus's trial in the time of Pontius Pilate, but the Pauline
authorship of 1 Timothy and the other pastoral epistles is too much in dispute in scholarly
circles for anything they say to be considered convincing evidence of what Paul may have

Volume 1990 - 2002 Issue


Page 898 of 2049
Skeptical Review Edited by Farrell Till
thought about the "historical Jesus." In none of the undisputed Pauline epistles will Perman
find any indication that Paul knew anything about the Jesus who is described in the gospels.
Paul preached only a resurrected Jesus, but he knew nothing or at least indicated nothing
about where this Jesus was born, when he was born, where he lived, or when and where he
died and was resurrected.

Is it reasonable to believe that in all that Paul wrote about Jesus, he wanted to give only a
"summary, or brief outline, of the core of Christian beliefs" and so "chose" not to mention any
of the biographical information that is recorded about Jesus in the gospels? Paul's silence in
these matters, considered in the context of the time between the writing of Paul's epistles and
the gospels, is certainly compatible with the spiritual-resurrection argument that I presented in
an earlier response to Perman. Paul put Jesus into no particular historical setting, but after
Christianity had developed legends about when and where Jesus lived, the gospels, written
decades after Paul's epistles, filled in the gaps that he had left. With the historical setting that
evolved, the spiritual resurrection that Paul described in 1 Corinthians 15 evolved into a
bodily resurrection.

Some Christian scholars, by the way, do not think that Paul was reciting an "early creed" in
this chapter. They argue that verses 3-11 are a post-Pauline interpolation, which interrupts the
continuity that flows logically from verse two to verse twelve and also contradicts what Paul
said in Galatians 1:1, 11-12 about the manner in which he had received the gospel. Christian
apologists, however, argue that the disputed passage is an early creed, and so this is a position
that requires them to defend their claim of the bodily resurrection of Jesus against statements
in 1 Corinthians 15 that clearly indicate Paul was speaking about spiritual rather than physical
resurrection.

That both Christological and ecclesiastical evolution occurred during the decades that the
New Testament was being written should be apparent to all who have no fundamentalist axes
to grind, so it would not be at all unreasonable to assume that evolution also occurred in the
most vital of all Christian doctrines, i.e., the resurrection. As I showed in my first response to
Perman, an early belief in the spiritual resurrection of Jesus is very much in evidence in 1
Corinthians 15:35-44, which I will review before addressing Perman's attempt to make this
passage not mean what it clearly says.

But someone will say, "How are the dead raised up? And with what body do they come?"
Foolish one, what you sow is not made alive unless it dies. And what you sow, you do not
sow that body that shall be, but mere grain--perhaps wheat or some other grain....

There are also celestial bodies and terrestrial bodies; but the glory of the celestial is one, and
the glory of the terrestrial is another. There is one glory of the sun, another glory of the moon,
and another glory of the stars; for one star differs from another star in glory. So also is the
resurrection of the dead. The body is sown in corruption, it is raised in incorruption. It is
sown in dishonor, it is raised in glory. It is sown in weakness, it is raised in power. It is sown
a natural body, it is raised a spiritual body. There is a natural body, and there is a spiritual
body (emphasis added).

Volume 1990 - 2002 Issue


Page 899 of 2049
Skeptical Review Edited by Farrell Till
In this passage, Paul presented an analogical argument by pointing out that even in nature, the
body that is sown (a grain of wheat) is not the body (a stalk of wheat) that "shall be." Anyone
who has had any horticultural experience at all knows that this is true. The seed that is planted
bears no resemblance at all to the plant that "rises" from the seed, so if Paul was not arguing
that the body that is resurrected is radically different from the body that is planted (buried),
his analogy makes no sense at all.

Let's notice also that Paul's analogy was actually an answer to a question: "But someone will
say, `How are the dead raised up, and with what body do they come'" (v:35)? In other words,
Paul understood that to sell his claim that Jesus was resurrected, he had to convince people
who were skeptical about the possibility of dead bodies returning to life that resurrection from
the dead was plausible. To those skeptics who questioned how it would be possible for dead
bodies to return to life, Paul was, in effect, saying, "That's not at all hard to explain. The body
that is buried is not the same body that is resurrected, just as a seed that is planted is not the
same as what comes from the seed." As noted in my first response to Perman (TSR,
July/August 1996, p. 5), Paul incorrectly believed that a seed, when planted, dies and from the
dead seed a new body, i.e., the plant, comes forth, and so he argued by analogy that this is
what happens when a physical body dies. From the dead physical body, a new body will arise.
Paul wasn't much of a botanist, but, nevertheless, this was his argument, and so Christians are
stuck with it.

The important thing to note in this passage is that Paul thought resurrected bodies were not
the same as the bodies that were buried. The body that was buried was a corruptible body; the
body that was raised was an incorruptible body (v: 42). The body that was sown (buried) was
a natural body; the body that was raised was a spiritual body (v:44).

In response to this, Perman argued, "But when Paul says of a believer's body in verse 42 that
`it is sown in corruption, it is raised in incorruption,' he is not saying that our bodies will be
taken from materiality, but from mortality" (TSR, November/December 1996, p. 3). Oh,
really? Well, it would have been nice if Perman had made a hermeneutic attempt to show that
this is a reasonable way to interpret the statement, but he made no such effort. He just
arbitrarily declared that this was the meaning of the statement and went on to something else.
Does he seriously expect us just to accept his arbitrary claim that this was what the statement
meant, when it so obviously disagrees with the import of Paul's argument?

To settle the matter of what Paul meant, I will just let Perman argue with what Paul said later
in verse 50: "Now this I say, brethren, that flesh and blood cannot inherit the kingdom of God;
nor does corruption inherit incorruption." So we have Paul saying that flesh and blood cannot
inherit the kingdom of God, but we have Perman saying that Paul never meant that "our
bodies will be taken from materiality." However, if "flesh and blood" cannot inherit the
kingdom of God, Perman needs to explain how that the resurrected bodies of Christians will
be able to inherit the kingdom of God without first being "taken from materiality." If they
aren't "taken from materiality" but spirited [no pun intended] away to heaven as material
bodies, then Paul was wrong when he said that flesh and blood cannot inherit the kingdom of
God. If not, why not? Unless Perman can explain why not, he is stuck with yet another textual
problem that demands explanation.

Volume 1990 - 2002 Issue


Page 900 of 2049
Skeptical Review Edited by Farrell Till
Further evidence that Paul believed that material bodies are planted (buried) from which
spiritual bodies are resurrected is seen in verses 51-53: "Behold, I tell you a mystery: We shall
not all sleep, but we shall all be changed--in a moment, in the twinkling of an eye, at the last
trumpet. For the trumpet will sound, and the dead will be raised incorruptible, and we shall
be changed. For this corruptible must put on incorruption, and this mortal must put on
immortality." This is just one of the many New Testament passages that expressed belief in an
imminent return of Jesus. Thus, those who would not "sleep" (in the mystery that Paul was
explaining) would be those who were still alive when Jesus returned (see 1 Thess. 4:15-17).
They would not die or sleep, but they would be changed, after the dead had been raised
incorruptible. Why would they have to be "changed"? Because they would be living in flesh
and blood (physical bodies) when Jesus returned, and Paul had just said that flesh and blood
cannot inherit the kingdom of God. Therefore, a "change" would be absolutely essential
before they could be taken to heaven.

In the "change" that those who were alive at the time would experience, their corruptible
bodies would have to put on incorruption, and for this to happen, some change in their bodies
would be necessary, just as the bodies of the dead, which had been planted as natural,
"corruptible" bodies, would be raised as spiritual, "incorruptible" bodies. Otherwise, these
bodies would not be able to "inherit" the kingdom of God.

So let's juxtapose two major statements that Paul made in this passage: (1) His conclusion
from his analogy of the seed was that "the body (physical) is sown in corruption; it is raised in
incorruption. (2) Those who were alive at the time of Jesus' second coming would be
"changed" by having "this corruptible" (the physical body) put on "incorruption" (a spiritual
body). After stating this, Paul then went on to say, "So when this corruptible (physical body)
has put on incorruption (spiritual body), then shall be brought to pass the saying that is
written: `Death is swallowed up in victory'" (v:54). By arguing that a "change" from
"corruptible" to "incorruptible" would take place in the physical bodies of those living when
Jesus returned, Paul removed all reasonable doubt about what he had meant earlier when he
said that "(t)he body [of a dead person] is sown in corruption, it is raised in incorruption." He
obviously meant that the resurrected body is a spiritual, "incorruptible" body as opposed to
the physical body that was buried.

Despite language as explicit as this, Perman contended that "this verse [it is sown a natural
body, it is raised a spiritual body] most forcefully teaches the traditional doctrine of the
resurrection," because "it is the same it in both cases." In other words, Perman is arguing that
the body that is sown (buried) has to be the same body that is raised, because the pronoun it
was used in reference to both bodies, but a simple illustration will suffice to show that Perman
is wrong. Let's suppose that someone says, "My profession has changed twice in my lifetime.
First, it was preaching, then it was plumbing, and now it is teaching." In such a statement, the
pronoun it is the same pronoun, but it obviously refers to three different professions:
preaching, plumbing, and teaching. So there is absolutely nothing in the way that Paul used
the word it that would require "it" to refer to the same "physical body." What Paul really
meant was that it [the body] is sown [buried] as a physical body, but it [the body] is raised a
spiritual body. To argue that it (the natural body) was the same as it (the spiritual body) would
make Paul's argument completely meaningless. If the two "its" were the same, then Paul

Volume 1990 - 2002 Issue


Page 901 of 2049
Skeptical Review Edited by Farrell Till
would have had to say, "It is sown a natural body, and it is raised a natural body." If not, why
not?

Perman argued that the word spiritual doesn't have to mean "made out of spirit" but can mean
"directed by and orientated to the Spirit." This is true, but the issue is not what the word
spiritual can mean but what it did mean in the context where Paul was using it. Perman said
that "virtually all commentators" agree that the word didn't mean "made out of spirit," but
when Perman's case is weak, he likes to talk about how many "commentators" agree with his
position, as if that is supposed to prove he is right. The issue in a debate should never be the
number of scholars or commentators who agree with one's position but the quality and force
of the arguments, and the force of argument is against Perman on this issue. Let's notice that
Paul equated the body corruptible with the body natural and the body incorruptible with the
body spiritual: "The body is sown in corruption, it is raised in incorruption.... It is sown a
natural body, it is raised a spiritual body" (vs:42-44). For Perman to argue that Paul simply
meant that the body is raised "orientated to the Spirit" is ludicrous, because, as noted above,
he went on to explain that those who were living when Jesus returned would have to be
"changed," i.e., their corruptible (bodies) would have to put on incorruption, before they could
inherit the kingdom of God. Since Paul used incorruptible synonymously with spiritual, he
couldn't have meant that these people would have to undergo a change that would "orientate
them to the Spirit," for the clear teaching of the New Testament is that unless one is already
"orientated to the Spirit" when Jesus comes, he/she would not be qualified for entry into
heaven. The "change," then, would be a change from flesh and blood to spirit, i.e., from
corruptible to incorruptible, which would then entitle them to go to heaven, where flesh and
blood cannot enter (v:50).

That the New Testament teaches this can be seen in a similar passage in 1 Thessalonians:

"But I do not want you to be ignorant, brethren, concerning those who have fallen asleep, lest
you sorrow as others who have no hope. For if we believe that Jesus died and rose again, even
so God will bring with Him those who sleep in Jesus. For this we say to you by the word of
the Lord, that we who are alive and remain until the coming of the Lord will by no means
precede those who are asleep. For the Lord himself will descend from heaven with a shout,
with the voice of an archangel, and with the trumpet of God. And the dead in Christ will rise
first. Then we who are alive and remain shall be caught up together with them in the clouds to
meet the Lord in the air. And so shall we forever be with the Lord" (4:13-18, emphasis
added).

Notice that Paul was saying in this passage that it was only those who were dead in Christ
who would be resurrected to meet the Lord. In Christ is a New Testament idiom that denoted
a state of salvation, which would have required a prior "orientation to the Spirit," so those
whom Paul thought were going to be resurrected to meet Jesus at his return would be those
who were "asleep (dead) in Christ." If Paul thought that only those who had died in Christ
would be resurrected to meet Jesus at his return, we would hardly expect that he believed just
anyone living at the time would also be caught up to "meet the Lord in the air." Surely, Paul
thought that those so honored would be those who were living in Christ at the time of his
return, but if they were living in Christ, there would certainly be no need to "change" them so
that they would be "orientated to the Spirit," because such orientation would have already

Volume 1990 - 2002 Issue


Page 902 of 2049
Skeptical Review Edited by Farrell Till
occurred. So what else could be the "change" that Paul mentioned except a change from
corruptible or natural bodies to incorruptible or spiritual bodies?

Completely ignoring the wording of Paul's analogy, Perman said, "(T)here is continuity
between the seed to [sic] the plant (they are the same organism), yet there is also change. And
certainly both the seed and the plant are both physical! Christians do not have `the body that
shall be' because we are not immortal yet" (TSR, November/December 1996, p. 3, emphasis
added). If Perman would just read Paul's analogy, he will see that Paul did not use "the body
that shall be" in reference to the resurrection of the dead but in reference to the seed that is
sown: "And what you sow, you do not sow that body that shall be, but mere grain--perhaps
wheat or some other grain" (v:37, emphasis added). Certainly, both the seed and the plant are
physical, but Paul was merely making an analogy. One thing is planted; another thing comes
from what is planted. Paul believed that this illustrated to his readers how that the dead could
be resurrected. They are buried as natural, corruptible bodies; they are raised as spiritual,
incorruptible bodies.

In support of the theory that the Christian doctrine of the resurrection had begun as only a
belief in a spiritual resurrection, I stated that the Greek word for "buried" in 1 Corinthians
15:4 was thapto and that it "meant to inter or bury and carried with it no necessary
connotations of entombment" (TSR, July/August 1996, p. 4, emphasis added), but Perman is
apparently trying to distort my statement into meaning that thapto could not be used in
reference to a burial in a tomb. This, however, is not what I meant. Perman's argument--
which really isn't his but only something he is parroting from fundamentalist apologetic
works--is that Paul's statement in 1 Corinthians 15:3-4 that Jesus was buried but rose again on
the third day implies an empty tomb, but if there is nothing in the word thapto to imply
entombment and if, as I have already noted, Paul never made any references to when Jesus
lived, where he lived, where he died, where he was buried, how he was buried, etc., exactly
why would saying that someone was buried and rose again necessarily imply an empty tomb,
especially if that statement was made in a context in which spiritual resurrection was being
discussed? If someone should say (as superstitious people sometimes do) that a certain
individual died and was buried and that his ghost has appeared to several people who knew
him before his death, would such a claim as this imply an empty grave? Anyone hearing such
a claim would understand the claimant was saying only that the person's ghost or spirit had
returned from the dead and not that the body had left the grave. So if Paul thought that Jesus
had risen as a spiritual entity (as I believe I have clearly shown that he did), that would not in
any way imply that he believed an empty tomb had been found, but even if it did, this would
prove nothing more than that Paul believed an empty tomb had been found. What Perman
would then have to do is prove that Paul was right in believing that an empty tomb had been
found. Otherwise, he becomes just another fundamentalist arguing that if the Bible says
something, it must be true.

Perman also misunderstood my point about egeiro, which was the Greek word that Paul used
in listing the "appearances" that Jesus had made. I merely quoted Ephesians 5:14, where Paul
had urged the Ephesians to awake from sleep and arise from the dead, and pointed out that the
word for "awake" in this verse is egeiro, whereas anistemi was the word he had used for
"arise." This is simply a fact, which Perman didn't dispute, so my point was that if the word
was used in the sense of awaking in this text, it could have carried this meaning in 1

Volume 1990 - 2002 Issue


Page 903 of 2049
Skeptical Review Edited by Farrell Till
Corinthians 15:5-7 too. I cited other passages where the word egeiro was obviously used in
the sense of visionary appearances, but that is not the same as arguing (as Perman tried to
make it appear I had done) that the word was never used in the sense of actual physical
sightings. My intention was to show that the meanings of the Greek words that Paul used
were completely consistent with the view that Paul was arguing in 1 Corinthians 15 only for a
spiritual and not a bodily resurrection.

Perman must consider burial by entombment crucial to his case, because he claimed that there
is a "wealth of evidence supporting the burial of Jesus in a tomb" (TSR, November/December
1996, p. 2). He cited six points from which he concluded that the entombment of Jesus is a
historical fact:

Archaeological support: Perman alleged that archaeologists have discovered the tomb of
"Yohanon, who was discovered in Tomb #1 at Giv'at ha-Mitvar, As el-Masaref by Tzaferis,"
and his "heel bones [were] transfixed by a large iron nail and his shins broken," evidence that
Perman sees as proof that "Yohanon" was crucified and then buried in a tomb. I have to
wonder why Perman didn't document this information so that I could evaluate both it and his
source. I'm not denying that such a discovery was made, but does Perman just expect me to
accept his word that this tomb was found, with the remains exactly as he (or his source)
described them? I really doubt that Perman knows any more about this discovery beyond the
bare information that he cited, which I suspect that he has taken from an apologetic source
that was just as sketchy with details as Perman was. At any rate, there is no way to critically
evaluate this information, because Perman didn't say when this discovery was made, who this
Tzaferis is who allegedly made the discovery, what exactly was meant by heel bones that
were "transfixed by a large iron nail," whether the death of this "Yohanon" occurred under a
Roman administration or some other regime, the estimated dating of the death, etc. In other
words, the information is too sketchy to determine if it would materially affect other
information that indicates victims of Roman crucifixion were thrown into garbage heaps or
buried in common graves. Furthermore, if the archaeological information about this person
Yohanon should conclusively indicate that he was crucified under a Roman administration, at
about the time Jesus allegedly lived, and then buried in a tomb, it would prove only that this
person was so buried. It would not prove that Jesus was, and so Perman would still face the
task of having to prove that the disposal of Jesus's body was done in a manner different from
the usual practice.

Jewish holy men (as was Jesus) were buried in tombs so their graves could be preserved.
This is the claim exactly as Perman made it, and beyond this mere assertion he said nothing to
try to prove the claim. However, even if the claim is true, Perman must show that this practice
was done even in cases of Roman executions where disposal of the bodies by entombment
would have been dependent upon permission from Roman authorities. The New Testament
does indicate that such permission was granted in the case of Jesus, but to accept this as
conclusive proof that Jesus was entombed would be to assume the historical accuracy of the
New Testament. If the historical accuracy of the New Testament is going to be assumed, then
there is nothing to debate. I admit that it says that Jesus was buried in a tomb. However, I
don't admit that this is necessarily a historical fact just because the New Testament says it.

Volume 1990 - 2002 Issue


Page 904 of 2049
Skeptical Review Edited by Farrell Till
The burial story lacks legendary development. Again, this is just an unsupported assertion
that Perman made. If he thinks it is true that the burial story lacks legendary development,
then he must at the very least do a literary analysis of the story to explain what he finds in it to
support this claim. When I read the burial story, I find it embedded in narratives that tell about
three hours of darkness at midday, an earthquake that resurrected "many saints," an angel that
descended in an earthquake to roll away the stone sealing a tomb, angels that talked to
women, a resurrected man who was apparently able to teletransport himself and to pass
through material objects like doors, etc. If Perman can't see "legendary development" in such
literary devices as these, then he has a serious inability to interpret literature. He said that the
New Testament critic Bultmann agrees that the burial story lacks "legendary development,"
but this would prove only that Bultmann thought that the story lacked legendary development.
At any rate, if the entire body of Bultmann's critical works were analyzed, I suspect there
would be very little in them that Perman would agree with. What we have in Matthew
Perman, then, is a would-be apologist using smorgasbord research in his quest to defend the
resurrection. If he finds critics who agree with him, he will cite them on their points of
agreement, but he will leave uncited the many details and points on which the critics do not
agree with him. Of course, Perman just has to say that "most [critics] agree with him
[Bultmann]," but Perman conveniently omitted the names of these "most" who agree.

Archaeology confirms the description of Christ's tomb in the gospels. This assertion, for
which Perman offered no supporting evidence, is as meaningless as the old inerrantist
argument that geographical accuracy in the Bible proves that it was inspired of God.
Inerrantists will point to the many geographical places mentioned in the Bible and argue that
the accuracy of the writers in these details must mean that they were inspired of God, as if
uninspired writers, familiar with the locales in which they set their stories, would have been
unable to report geographic and topographic details accurately unless they were divinely
inspired. Libraries are filled with books in which such accuracy is contained, but simplistic
biblical apologists think that a feature like this in the Bible should be accorded special
significance.

So let's suppose that "archaeology has confirmed the accuracy of the description of Christ's
tomb." All this would prove would be that the New Testament writers accurately described a
tomb as tombs were known to exist in those days. What would be so extraordinary about a
writer accurately describing a tomb? If I should write a narrative in which I accurately
describe a grave site as graves are now known to exist but then continue in my story to claim
that the person buried in this grave later rose from the dead, what reader in his right mind
would think that my accurate description of the grave would give credence to the
extraordinary claim that the person buried in the grave later rose from the dead? I do wish that
Christian apologists could think a little more rationally, but I've just about given up hope of
ever finding one who can.

The phrase `first day of the week' reveals an early date for the story, because, Perman
claims, "it [the phrase] fell out of use by the late 30s or so to be replaced with `on the third
day.'" Again, Perman has simply made an assertion for which he gave no supporting evidence,
and, quite frankly, I get a little tired of his regurgitation of simplistic "apologetic" arguments
that he finds in Josh McDowellian books and bounces back to us without even attempting to
prove their validity. There are two problems with this claim: (1) the phrase did not "fall out of

Volume 1990 - 2002 Issue


Page 905 of 2049
Skeptical Review Edited by Farrell Till
use" by the late thirties as evinced by its use in New Testament books that were written well
after the 30s (Acts 20:7; 1 Cor. 16:2). (2) "The third day" was a term that came to be applied
to the time that Jesus was presumably resurrected. He was resurrected on "the third day" (1
Cor. 15:4; Acts 10:4), and this was even the phrase that was used when the gospel writers
reported the many times that Jesus told his disciples he would be resurrected (Matt. 16:21;
20:19; Mark 9:31; Luke 9:22). So "the third day" wasn't an expression that came into use after
the late 30s. If we are to believe that the gospels accurately reported the conversations of
Jesus, this was an expression that was used even during the personal ministry of Jesus to
denote the time that he would rise again.

What Perman apparently can't see-- or doesn't want to see--is that the phrase "the first day of
the week" wasn't used in the gospel narratives to denote the time that Jesus was resurrected
but the time when the women went to the tomb (Matt. 28:1; Mark 16:2: Luke 24:1; John
20:1). The sole exception to this is a statement in the Marcan Appendix, which says, "Now
when he was risen early on the first day of the week, he appeared first to Mary Magdalene"
(Mark 16:9), but even this statement disputes Perman's claim that the phrase "the first day of
the week" had fallen out of use by the late 30s. There is a scholarly consensus that Mark 16:9-
20 was added to this gospel at a later date. The literature on this subject is too extensive to
review, but since Perman likes to talk about what "most scholars" think, he might want to
research what New Testament scholars think about the date of this addition to the gospel of
Mark. The statement that Jesus "rose early on the first day of the week" in this late addition to
Mark's gospel demolishes Perman's claim that the phrase "fell out of use by the late 30s."

The inclusion of Joseph of Arimathea strongly supports the burial record. Why would
this be so? Well, Perman argued that "(t)he members of the Sanhedrin were too well known
for someone to place a fictional member on it or to spread a false story about one of its
members burying Jesus." Such an argument as this gives no consideration at all to the time
that separated the alleged burial of Jesus and the writing of the gospels. Mark, the earliest of
the four gospels, is generally dated no sooner than A. D. 70, except of course by
fundamentalist apologists, so after the alleged crucifixion and burial of Jesus, 40 to 70 years
passed before the gospel accounts were written. In a time when radio, television, newspapers,
and other modern means of communication didn't exist, it isn't difficult at all to imagine that a
fictional member of the Sanhedrin could have been created and passed unnoticed. Even today,
how many people would know who the vice-presidents of the United States were 40 to 70
years ago? How many would know who the supreme court justices were that long ago? Who
was the speaker of the house of representatives in 1950? 1940? 1930? Furthermore, the
gospels were written in Greek by Hellenistic Christians, whose audiences would not have
been Jews personally familiar with the religious/ political milieu in which Jesus had allegedly
lived. If a writer today should fictionalize a member of the Israeli supreme court as it was
constituted 40 years ago, how many Jews who are citizens of other countries would notice it?
This argument is completely without merit, and the only thing it proves is the desperation of
Christians who have no real evidence to support their resurrection belief.

In addition to his "wealth of evidence supporting the burial of Jesus in a tomb," Perman made
several other baseless claims, none of which can be proven and none of which would prove
anything even if their truth could be established. They have been repeatedly discredited, but
Christian apologists continue to recycle them.

Volume 1990 - 2002 Issue


Page 906 of 2049
Skeptical Review Edited by Farrell Till
Scholars know that Mark used a source that originated before A.D. 37. How does Perman
know this? Are you ready for the answer? He knows, because Mark did not use the name
Caiphas when referring to the high priest during the trial of Jesus! From Mark's silence on
this matter, Perman concludes... well, to be more exact, the apologetic source from which
Perman gets his material has concluded that A. D. 37, the last year of Caiphas's term, was the
last possible date for the origination of the source that Mark used. I suppose it has never
occurred to Perman that "Mark," a Hellenistic writer, just may not have known the name of
the high priest at that time, and so that was why he gave no name. Who was the prime
minister of Israel 40 years ago? Who was the chief justice of the Israeli supreme court 40
years ago? How many people today would not know these things? A modern writer could
easily get such information as this from the internet or a local library, but information wasn't
so easily accessible when the gospel of Mark was written. A writer back then who didn't
know the name of a Jewish high priest who had lived 40 or 50 years earlier would have had
no choice except to tell his story without specifically naming the priest. Regardless, Mark's
silence on the name of the high priest is certainly no evidence of a source earlier than A. D.
37 for his burial story.

Just for the sake of argument, let's assume that Mark did indeed use a source that dated before
A. D. 37. Let's even assume that his source dated even earlier than that. What would it prove?
It would prove only that Mark had used an early source, but it would not prove that the
information in that source was historically accurate. Anyone could write a fictional narrative
and plot it with a specific contemporary setting, but this would be no guarantee that the
information in the narrative is historically accurate. Surely, Perman and his apologetic cohorts
can see that.

Even if the gospels were not written by actual eyewitnesses, there is still the eyewitness
testimony of 1 Corinthians 15. Either Perman is not reading my responses carefully or he is
being intentionally evasive, because I discussed at length in my first response (TSR,
July/August 1996, p. 5) that Paul never claimed that he had seen the resurrected Jesus in
bodily form. The only records we have of Paul's experience with Jesus are Luke's three
accounts in Acts 9, 22, and 26, and they are contradictory on some points. They do agree that
nobody with Paul saw Jesus, and so this wonderful eyewitness testimony that Perman is so
excited about was a visionary sighting that only Paul experienced. If Luke's accounts are
accurate, then Perman must acknowledge that Paul himself described it as a "heavenly vision"
(Acts 26:19). Perman is free to consider this amazing evidence if he wants to, but I consider it
no more convincing than someone's claim that he was told that another person saw Elvis
Presley in a vision. What we all want to see from Perman is real evidence, but he seems
unable to produce any.

The use of women to discover the tomb establishes the credibility of the resurrection
story. This worn-out apologetic argument shows an incredible ignorance of the culture that
produced the gospel accounts. It is true that in Jewish society, women had very little social or
political status, but Perman ignores that the gospels were Hellenistic productions. If their
authors were Jews, they were Hellenistic Jews who had been exposed to cultural influences
that were not quite as unkind to women as was the Mosaic law, which considered women to
be mere chattel. Hellenistic literature and mythology had goddesses as well as gods and
heroines as well as heroes. Hera was the queen of heaven and the consort of Zeus; Athena was

Volume 1990 - 2002 Issue


Page 907 of 2049
Skeptical Review Edited by Farrell Till
the goddess of wisdom, and Athens, the center of Greek learning, was her namesake; Artemis
was the goddess of the moon and also the protector of wildlife; Aphrodite was the goddess of
love.

These are only a few of the many goddesses and nymphs who were worshipped and respected
by the Greeks, and it would be unreasonable to think that generations of Hellenistic Jews
could have lived in Greek societies without having been influenced by their fascination with
goddesses. Greek culture, for example, had Sibyls, who were aged women that uttered
prophecies thought to have been revealed to them by the gods. They functioned much in the
same way as the "oracles," like the famous one at Delphi, and Hebrew society didn't escape
the influence of the Sibyls as evinced by writings like The Sibylline Oracles, a
pseudepigraphic work purporting to contain the prophetic utterances of several Hebrew Sybils
dating as far back as Sambethe, a daughter-in-law of Noah. Such works as these could not
have been produced in a culture that put no credence in the testimony of women.
Furthermore, a study of the Old Testament shows that the Hebrews had their prophetesses,
such as Deborah and Huldah in Judges 4 and 2 Kings 21, and that the worship of goddesses
like Ashtaroth was often a religious problem in early Israel (Judges 2:13; 10:6; 1 Sam. 7:3-4;
12:10). The Hebrews also had their heroines like Ruth and Esther in the canonical books and
Judith in the apocryphal. To argue that the testimony of women would have commanded no
respect in first-century Judaism is a claim that cannot be substantiated, and it ignores evidence
that indicates the gospels were Hellenistic in origin.

Finally, I must ask Perman to explain why the apostle Paul didn't even mention that women
were the first to find the empty tomb and to see the resurrected Jesus. If "the use of women to
discover the tomb" is so forceful in "establish[ing] the credibility of the resurrection story,"
then why didn't Paul realize it and include this information in 1 Corinthians 15? He labored
long and hard in this chapter to convince the Corinthian Christians that Jesus had risen. In so
doing, he listed six separate postresurrection appearances Jesus had allegedly made, including
even the famous claim that he had appeared to "about five hundred brethren at once," but Paul
left out entirely what Perman believes is information that "establishes the credibility of the
resurrection story." That was very negligent of Paul, but it was even more negligent of the
omniscient, omnipotent Holy Spirit not to have given Paul an inspirational nudge to let him
know that the experiences of the women was information that he definitely needed to include.
Perman's god apparently didn't think this information was quite as important as desperate
Christian apologists think it is.

Perman attaches undue importance to the opinion of the Jewish author Pinchas Lapide that the
resurrection of Jesus did actually occur. In the next issue, I will address this point and a few
other loose ends. After that, if Perman insists on believing in the resurrection of his crucified
savior-god, I know of nothing else to do but extend to him my sympathy. He, of course, may
reply to my articles if he wishes, but unless he can show us something better than the
fundamentalist flapdoodle that has characterized his other two articles, I will see no need to
dignify his apologetic desperation with another response.

Volume 1990 - 2002 Issue


Page 908 of 2049
Skeptical Review Edited by Farrell Till

A Reply to Wilhelm Schmitt


Yoel Wasserman
In response to Wilhelm Schmitt's article that replied to "alleged difficulties in Matthew 27:9-
10" (September/October, pp. 2-3), I will use his section numbers when referring to his article.

Anyone can look up a few Hebrew/ Greek words in Strong's concordance and make it seem as
if he is an expert. I previously asked someone (a former roommate) who knows nothing about
the Bible or Hebrew to write an article quoting from Strong's to make it seem as if he knew
what he was talking about. The result was a very professional sounding document, which
made no sense at all.

Since I know Hebrew fluently, I know the following to be true: Strong compiled his
concordance under the assumption that the Bible in general and the KJV in particular were
true and contained no mistakes. Strong's method was this: If the KJV says that a certain
Hebrew word has a certain meaning, it must have that meaning somewhere, somehow, so he
listed that KJV meaning, whether or not the translation had any linguistic accuracy. He did
not do a linguistic study of the word in question, nor did he alter any of the words in the KJV.
He did not even resort to the supposed cross-referencing with the Septuagint, which most
modern Christians claim to rely on. What he did was make a sort of biblical thesaurus, not a
dictionary or a critique. Strong actually claimed that certain Hebrew words had meanings that
they do not have in any manner whatsoever. Strong's source document was the KJV and not
the Masoretic text, so Strong is a very faulty and unscholarly work.

(1) The Hebrew word for field is sadeh, not shadah, as Schmitt claimed. It doesn't appear in
Zechariah 11, because, just as in so many other cases, the gospel writer of Matthew, when
misquoting, was forcing a Messianic meaning into an Old Testament verse that wasn't even
originally a prophecy.

(2) Schmitt claims that eder hayekar (translated "a goodly price [or value]" in the KJV
"signifies sufficiency." Where does he get this? Literally, eder hayekar means"the mightiness
[of] the price/value." The context of the phrase in the surrounding verses signifies that the
meaning of the verse is an obvious sarcasm. In the previous verse, Zechariah had asked his
employers whether or not he was going to be paid. When his payment of 30 pieces of silver
was weighed out to him, he sarcastically declared the 30 pieces of silver as eder hayekar
asher yakarti mealeihem ("the mightiness of the price [or value] at which I was priced/valued
[in Hebrew the same word is used in a verb form] by them"). Schmitt claimed that the verse
did not in any way indicate that the "amount was paltry or that the offer of it was in any way
an insult." The opposite is true! The very fact that Zechariah immediately threw the money
away shows unarguably that the verse was meant in sarcasm and that the 30 pieces did not in
any way "meet with his approval."

(4) The Hebrew word in question is hasomrim (the keepers/they who keep), not shamar as
Schmitt stated. Shamar is the root (to keep), so Schmitt is not totally incorrect, but,
nevertheless, he did not use the correct conjugation. This is because Strong's does not list an

Volume 1990 - 2002 Issue


Page 909 of 2049
Skeptical Review Edited by Farrell Till
exact word-for-word translation but rather lists the verb root. In so doing, he often chose the
wrong root to list.

True, "to keep" doesn't connote anything negative, but what's the point? Schmitt is conceding
that what Matthew claims is in Jeremiah is not in Jeremiah. He is also admitting that there is a
similar verse in Zechariah that seems as if it may have been the source for Matthew's
"prophecy." In spite of all this, he is stubbornly claiming that if Matthew said that the verse
about the field existed and was spoken of by Jeremiah, then it cannot be derived from this
verse in Zechariah. So now he is trying to "prove" that Zechariah 11:13 is not the verse that
Matthew referred to and, therefore, brings up the possibility that Matthew was referring to a
verse that exists/existed somewhere but doesn't exist anymore. He overlooks the fact that the
New Testament repeatedly tries to "prove" from the scriptures that Jesus Christ was the
Messiah. What purpose would it serve to quote a nonexistent verse that no one had heard of?
Schmitt also overlooks the fact that there are many other similarly misquoted verses. Which is
more probable and likely? That the gospel writers lied and twisted the meaning of verses,
made mistakes, or that the Bible is a perfect jewel of a book, unlike any other book in the
world, and that Wilhelm Schmitt knows all the answers and the creator of Andromeda and
every other galaxy and star in this and any other universe?

(5) See #2 above to see the context in which va'ashlich, "I cast/threw," was used. The context
in no way shows it to be "in a good sense," as Schmitt claimed. Also, how do you throw
something away in a good sense? Especially money? The fact that he threw instead of "gave"
or "handed" the money definitely shows that the connotations were negative.

(6) In ancient Hebrew, yotgser, which the KJV translates as "potter" (actually
"former/creator/fashioner"), also meant "treasurer," In this sense, it is derived from the
Hebrew otsar, which means "treasury" instead of from yatsar, which means "to form." The
phrase used in Zechariah is beit hayotser, which literally means "the house of the treasurer."
This is a very common Hebrew construction of a house for public use. For example, beit
haknesset (house of the assembly) is the word for "synagogue"; beit hasefer (house of the
book) is the word for "school"; beit hakise (house of the chair) or beit hashimush (house of
use) are two Hebrew terms for bathroom. Also, it is obvious from the context that Zechariah
was referring to a treasury because (1) there is no such service as "potter" in the temple, (2)
there is such a service as treasurer, since money is involved in the running of any institution,
(3) since it is money that was given to the institution, is it more probable that the money was
given to a nonexistent potter or to an obviously existent treasurer?

Schmitt used the word yatser instead of yotser, the actual word used in Zechariah. Why?
Because this is what Strong did! I don't know whether Strong did this purposely to bypass the
fact that what was spoken of was a treasury, or whether it was an honest mistake.
Nevertheless, Schmitt's use of Strong's yatsar (root of "to form"; the root for treasury is atsar)
shows that all that Schmitt did was flip through a Strong's concordance and nothing else.

(7) Who cares what the Septuagint says? The Septuagint is just another translation and is
renowned for being a bad one. The only reason its existence is in any way seemingly relevant
in modern theology is because most early Christians did not know Hebrew but did know
Greek. Otherwise, it would be nothing more than another old document. The language of the

Volume 1990 - 2002 Issue


Page 910 of 2049
Skeptical Review Edited by Farrell Till
Jews was Hebrew, not Greek, and the language of the Old Testament was Hebrew, not Greek.
That's just the way it is.

(Yoel Wasserman, P. O. Box 998, Aspen, CO 81612; e-mail core@rof.net)

A Straw House Amid 10-Foot Waves


Matthew Hogan
In the September/October issue of The Skeptical Review, I informed the readers that Farrell
Till misinterpreted Ezekiel's prophecy concerning Tyre in the following ways: (1) He failed to
note the changes in pronouns, from he and his (Nebuchadnezzar) to they in verse 12 (Greeks,
Romans, etc.) and then to I (God) in verses 13-14. (2) Verse 11 ends the prophecy with
respect to Nebuchadnezzar. Also, verses 7-11 refer to a siege--not the destruction of Tyre. (3)
The they in verse 12 refers to various countries--Romans, Greeks, and so forth. These nations
would "plunder" Tyre's riches, thus fulfilling verse 12. (4) The singular pronoun I refers to
God himself.

Till responded by writing that pronoun shifts were common in the book of Ezekiel. He then
cited five examples in an attempt to back up this claim.

My response: So what!

Till is erecting a straw house on an ocean beach amid ten-foot waves because how we
interpret the Tyre prophecy is determined by the words "many nations" (verse 3)--not by what
is written elsewhere in the book of Ezekiel.

Moreover, why would Ezekiel mention "many nations" if he meant that Nebuchadnezzar
would be the one to destroy Tyre?

Till asks, "Why would a writer who was verbally inspired by an omniscient, omnipotent diety
[sic] to prophesy the destruction of [Tyre] communicate his message as confusingly as
Ezekiel did?"

Strange, I don't have any trouble understanding the Tyre prophesy [sic]! The only confusion
regarding the Tyre prophesy [sic] is Till's.

Till also writes that "changes of direction in writing, and especially sudden changes, should be
signaled with transitional devices."

In truth, no transitional device is needed because the "many nations" of verse 3 requires that
the they in verse 12 be interpreted as refering [sic] to the Romans, Greeks, and so forth.

Volume 1990 - 2002 Issue


Page 911 of 2049
Skeptical Review Edited by Farrell Till
Thus, Till's allegations regarding "transitional devices" is just another straw house built on an
ocean beach amid ten-foot waves.

Moreover, I could also perhaps argue that the plural pronoun they of verse 12 is a transitional
device itself because of the words "many nations" (verse 3).

Till wastes much ink and copy space when he writes that I have "difficulty understanding
fairly plain language, because that is not what Ezekiel 29:18-19 actually said."

What Till is refering [sic] to is the fact that I left out the word no immediately before "wages
from Tyre" because I wrote that "Nebuchadnezzar didn't destroy Tyre because he wasn't the
one prophesied to do so."

Likewise, Till should realize that the they (verse 12) refers to the Romans, Greeks, and so
forth, because of the words "many nations."

Am I finally getting through to you Farrell?

On page six, Till writes much literary diddle.

"Destruction of the mainland villages, however, was not the extent of what Ezekiel had
prophesied for Nebuchadnezzar," he writes.

Till's historical revisionism not withstanding, verses 7 through 11 refers [sic] to


Nebuchadnezzar's siege against Tyre. This is why Ezekiel writes that Nebuchadnezzar would
"heap a siege mound" against Tyre.

Verses 7 through 11 refer to Nebuchadnezzar's assault, in greater detail, against the "daughter
villages" of Tyre. The you in verse 8 refers to the daughter villages" [sic] of tyre [sic].

Moving on, Till cites 2 Peter 3:10-11 in order to negate my claim that verses 13-14 refer to
the final destruction of Tyre. Farrell, Ezekiel 26:20 makes it plain that Tyre will not be finally
uninhabited until Christ returns.

Anyway, why do you cite 2 Peter 3:10-11? This assumes that I believe that these verses are
inspired?

So what if Ezekiel doesn't mention Jesus specifically! Jesus' name is implied.

Besides, Till seems to think that the second coming of Christ is implied in the Peter citation
above--so why can't I interpret that verses 13 through 14 refers [sic] to, or implies [sic], Jesus'
second coming? So much for another straw house erected by Farrell.

At the bottom of page six till [sic] quotes verse 3 which says, "Therefore, thus says Yahweh
God, `Behold, I am against you, O Tyre."

Till erroneously thinks that the "I" is "merely a return to the first person..."

Volume 1990 - 2002 Issue


Page 912 of 2049
Skeptical Review Edited by Farrell Till
Oh Mary, here we go again! The I refers to "Yahweh God." This interpretation makes much
more sense than Till's bizzare [sic] "return to the first person" idea.

If Farrell can explain what Ezekiel meant when he wrote about the "many nations" that would
come up against Tyre then perhaps I would admit defeat.

(Matthew Hogan, 177 Salisbury Street, Rochester, NY 14609-4137.)

The Romans, Greeks, and So Forth


Farrell Till
Normally, I edit all articles and letters published in The Skeptical Review to correct errors in
grammar, spelling, and punctuation and to polish syntax and style, but sometimes I leave
articles unedited in order to let the way they were written make a point that the readers would
otherwise miss. This is the case with Matthew Hogan's article. His main argument for the
inerrancy of Ezekiel's Tyre prophecy depends upon the usage of the pronoun they, yet his
article leaps from the page and screams that he is far from being linguistically qualified to
base arguments on grammatical principles involving the use of a single word. His article is
filled with mistakes in spelling, grammar, and punctuation; the organization of the article
shows that he doesn't understand basic principles of paragraphing; yet he apparently expects
us to believe that he is expert enough in linguistics to know that the pronoun they in verse 12
of Ezekiel's prophecy referred to "many nations" and not to the horses, horsemen, wagons,
and chariots of Nebuchadnezzar's army, which in the verses just before this, Ezekiel had
depicted raging through the streets of Tyre.

All of my analyses of passages in Ezekiel where plural pronouns were obviously used to refer
to singular antecedents Hogan summarily dismissed with a simple, "So what!" and an
arbitrary assertion that the pronoun they referred to the Romans, Greeks, and so forth. When
confronted with incisive counterarguments like this, what can I say?

To refute Hogan's assertion that the pronoun they in this passage must refer to a plural
antecedent, we have only to compare it to another passage in Ezekiel that is strikingly similar
in structure. First, let's consider again the passage in dispute:

Ezekiel 26:10-12: Because of the abundance of his horses, their dust will cover you; your
walls will shake at the noise of the horsemen, the wagons, and the chariots, when he enters
your gate, as men enter a city that has been breached. With the hooves of his horses he will
trample your streets; he will slay your people by the sword, and your strong pillars will fall to
the ground. They will plunder your riches and pillage your merchandise; they will destroy
your pleasant houses....

Volume 1990 - 2002 Issue


Page 913 of 2049
Skeptical Review Edited by Farrell Till
By all rules of grammar, Hogan is certainly right. The plural pronoun they in the last verse
grammatically calls for a plural antecedent, and if Ezekiel was referring to Nebuchadnezzar in
verse 12, he should have continued with the singular pronoun he, which, as indicated in bold
print, was used throughout the passage. However, what is grammatically required is not
always what writers do. Everyone who writes much at all will make mistakes, and mistakes in
pronoun-antecedent agreement are very common in writing. This is a fact I know from having
taught college writing for 30 years.

As I demonstrated in my first response to Hogan, Ezekiel's writing clearly shows that he was
inclined to this same grammatical mistake. Hogan apparently didn't think too much of the
examples I used, but another passage, very similar to the one just quoted, will show that
Ezekiel was too careless in his usage of pronouns for Hogan's argument to have any
credibility:

Ezekiel 29:18-20: Son of man, Nebuchadnezzar king of Babylon caused his army to serve a
great service against Tyre; every head was made bald, and every shoulder rubbed raw; yet had
he no wages, nor his army, from Tyre, for the service that he had served against it. Therefore
thus says Yahweh God: `Behold, I will give the land of Egypt to Nebuchadnezzar king of
Babylon; and he shall carry off her multitude, and take her spoil, and take her prey; and that
will be the wages for his army. I have given him the land of Egypt as his recompense for
which he served, because they worked for me....

Here we notice that Nebuchadnezzar was the subject of another of Ezekiel's prophecies, this
one clearly admitting the failure of the earlier prophecy that Nebuchadnezzar would totally
destroy Tyre, and so Yahweh was going to give him Egypt as compensation for the prior
prophecy failure. I have italicized the appropriate pronouns to show that all through the text,
Ezekiel used the masculine singular pronouns he, him, and his, which were grammatically
consistent with the fact that the antecedent of the pronouns was Nebuchadnezzar (a singular
masculine antecedent). However, at the very end of the prophecy, Ezekiel, for no grammatical
reason, shifted to the third person plural pronoun they: "I have given him the land of Egypt as
his recompense for which he served, because they worked for me" (v:20). There is no better
explanation for this shift than the assumption that Ezekiel had in his mind that the siege of
Tyre had been the work of not just Nebuchadnezzar but an entire army. Since a concept of
plurality was in the writer's mind, he inadvertently shifted to a plural pronoun, even though
grammatical consistency should have required him to use the singular pronoun, as he had
done throughout the text.

This is the type of carelessness that we often see in writing and hear even more often in
conversations. Someone might say, "The committee finished its report last night. They
worked past midnight." In a statement like this, the speaker is aware that committee is
singular, and so he/she uses its as the first pronoun but apparently forgets, because of an
awareness that a committee consists of several people, and shifts to the plural pronoun they.
In my responses to Hogan, I have analyzed several passages in Ezekiel to show that this type
of carelessness in pronoun usage was characteristic of Ezekiel's style. This is a point that
demands a response from Hogan. He answers nothing by just brushing it aside with a curt,
"So what!"

Volume 1990 - 2002 Issue


Page 914 of 2049
Skeptical Review Edited by Farrell Till
Hogan thinks that my position is "strange," because "[he doesn't] have any trouble
understanding the Tyre prophesy [sic]." Well, I don't have any trouble understanding it either,
and it is precisely because I do understand it that I consider it an obvious example of
prophecy failure. That Hogan claims to understand the prophecy is unimpressive to me,
because he has demonstrated that he has difficulty understanding rather simple statements. In
my first response, for example, I made the following statement in reply to Hogan's claim that
the shift to the personal pronoun "I" at the end of Ezekiel's tirade against Tyre meant that
Tyre's final destruction would happen "when Christ returns":

As for the switch to the first-person pronoun "I" in the Tyre prophecy, Hogan makes entirely
too much of this. It was not a switch but merely a return to the first person with which the
prophecy had begun: "Therefore, thus says Yahweh God, `Behold, I am against you, O Tyre"
(v:3). After then describing in third person the devastation that Nebuchadnezzar would bring
upon Tyre, the narrative [re]turned to the first person ("I will put an end to your songs") as a
means of showing that Nebuchadnezzar would merely be the agent through whom Yahweh
would destroy Tyre. If Hogan will take the time to examine the book of Ezekiel more
carefully, he will see that this narrative style was characteristic of Ezekiel's prophecies.
Ezekiel 28:6-10 and 30:10-12 are just two examples of where Ezekiel began a prophecy with
Yahweh speaking in the first person after which he shifted to third-person narration to
describe the agencies of punishment that he would use, and then he returned to first-person
narration by Yahweh" (TSR, September/October 1996, p. 6).

Now it should be rather obvious to anyone that in this statement I was clearly recognizing that
the personal pronoun "I" referred to Yahweh and that the literary device that Ezekiel used was
to begin his prophecies as if Yahweh himself were speaking, to switch within the text of the
prophecy to the third person as Yahweh described the agents, such as Nebuchadnezzar, whom
he would use to administer punishments, and then at the end of the prophecy to return to the
first person as Yahweh concluded his tirades with warnings like, "For I Yahweh have spoken
it" (26:14). Despite the clarity of my explanation, Hogan made this astounding statement in
his second article:

Till erroneously thinks that the "I" is "merely a return to the first person..."

Oh Mary, here we go again! The I refers to "Yahweh God." This interpretation makes much
more sense than Till's bazzare [sic] "return to the first person" idea.

How can anyone read the statement I quoted above and think that I don't understand that the
pronoun "I" in the disputed passage referred to Yahweh God? Didn't I say that Ezekiel
typically used a literary device in which he "began a prophecy with Yahweh speaking in the
first person"? So, yes, absolutely, the "I" in the text is a first-person singular pronoun whose
antecedent was Yahweh God. This, however, is not to say that the use of the pronoun "I" in
this passage "makes it plain that Tyre will not be finally uninhabited until Christ returns."

To argue that the "I" at the conclusion of the Tyre prophecy meant that the final destruction of
Tyre wouldn't happen "until Christ returns" [who at that time had not even come the first
time] would require rather absurd interpretations of some of Ezekiel's other prophecies. In
30:10-12, for example, Ezekiel prophesied the total destruction of Egypt by Nebuchadnezzar.

Volume 1990 - 2002 Issue


Page 915 of 2049
Skeptical Review Edited by Farrell Till
This prophecy is structured exactly like the one against Tyre. It begins with Yahweh speaking
in the first person: "I will also make the multitude of Egypt to cease by the hand of
Nebuchadnezzar, king of Babylon" (v:10). Then after Nebuchadnezzar was introduced as
Yahweh's agent of punishment, the prophecy shifted to a third-person description of what
Nebuchadnezzar would do to Egypt: "He and his people with him, the terrible of the nations,
shall be brought in to destroy the land, and they shall draw their swords against Egypt and fill
the land with the slain" (v:11). Then at the end of the prophecy, Yahweh returned to first-
person narration: "I will make the rivers dry and will sell the land into the hand of evil men,
and I will make the land desolate and all that is therein, by the hand of strangers. I, Yahweh,
have spoken it" (v:12).

Nothing like this and the long tirades before it, predicting utter and complete destruction of
Egypt, ever happened, so it is just another example of a failed prophecy. To be consistent,
then, Hogan would have to argue that this was just a prophecy of the final destruction of
Egypt when "Christ returns," even though the prophecy was directed against "Pharaoh, king
of Egypt" (29:2-3; 30:21-25;31:2), and Nebuchadnezzar was clearly identified as the agent
that would bring about the destruction (29:17-20; 30:10, 25). This is the extreme that people
like Hogan are driven to when they try to defend the accuracy of Old Testament prophecies
that obviously failed to materialize. Even if we assume the existence of the Hebrew god
Yahweh, we would have to consider him a very stupid deity if he sent prophets to warn
nations of an utter destruction that was going to come upon them at the final coming of a
Messiah at some remotely distant time in the future. If this god should send a prophet to warn
a country today that it would be totally destroyed when "Christ" returns at some unknown
time that could be thousands of years away, what effect, if any, would it have on the people?

Hogan's entire argument, which he has obviously borrowed from apologists like Gleason
Archer and Norman Geisler, depends upon a claim that the pronoun they in 26:12 refers to
"many nations" in verse 3. In other words, he is claiming that the antecedent of they is to be
found nine verses before the pronoun was used. I think I know enough about principles of
writing to say that if this is what Ezekiel really intended, he was an extremely careless writer,
because no effective writer would separate a pronoun from its antecedent with that much
verbiage, especially when there were so many words like horses, horsemen, men, wagons, and
chariots in between that could be confused as the antecedent of they. So if Hogan (read
Archer and Geisler) is right, we have to wonder why an omniscient, omnipotent deity didn't
do a better job of verbally guiding his "inspired" agent. As I have pointed out before, even if
Hogan could prove his case--and he can't--he would then have to explain this problem, which
is just as damaging to biblical inerrancy as a failed prophecy.

On the matter of the "many nations," Hogan said, "If Farrell can explain what Ezekiel meant
when he wrote about the `many nations' that would come up against Tyre, then perhaps I
would admit defeat." Well, I do have an explanation, which I consider far moresensible than
the strained interpretation that Hogan has resorted to in an attempt to salvage this prophecy.
This interpretation is based on the fact that the Hebrew language had no future tense. Thus,
Ezekiel did not actually say, "I am against you, O Tyre, and will cause many nations to come
up against you." Instead, he said, "I am against you, O Tyre, and have caused to come up
against thee many nations" (Young's Literal Translation of the Holy Bible). In the absence of a
future tense, which was characteristic of Semitic languages, future action was denoted by

Volume 1990 - 2002 Issue


Page 916 of 2049
Skeptical Review Edited by Farrell Till
using the past tense, and in the introduction to his literal translation, Young discusses at
length the controversy that this linguistic feature has generated among Bible translators
("Battle of the Hebrew Tenses," Revised Edition, Baker Book House).

To people whose native languages have a future tense, as English does, it may seem strange
that some languages don't have this tense, but even in English we sometimes express future
intention the same way. In a heated dispute, someone might react to the other's insult with a
threat like this: "Okay, buddy, you've had it." The object of the threat hasn't received any kind
of retaliation yet, but he would understand that his adversary is threatening that he will take
some kind of action in the future, just as surely as if the threat had been, "Okay, buddy, you're
going to get it." In such cases, the meaning is determined by context.

Without a future tense, this was how it always was in Hebrew, so it is possible that some past-
action statements in the Bible were misinterpreted by translators and rendered in the future
tense when futurity wasn't intended. A comparison of English versions will show several
places where translators disagreed on how tenses should be rendered.

With this tense problem in Hebrew, it may have been that Ezekiel was talking in verses 3-6
not entirely about what would be done to Tyre but what had already been done, at least in
part, because the mainland villages of Tyre had been frequent victims of invading armies.
However, at verse 7, Ezekiel obviously focused his prophecy on what Nebuchadnezzar would
do to both the mainland villages and to Tyre proper. That Ezekiel was predicting action by
Nebuchadnezzar against both the mainland villages and the island city is evident to those who
want to see it. First, Nebuchadnezzar would "slay your [Tyre's] daughters in the field [the
mainland villages]," and then he would "set his battering engines against your [Tyre's] walls,
and with his axes he will break down your [Tyre's] towers" (vs:8-9). Surely, if Ezekiel had
been referring only to walls and towers of the mainland villages, he would not have used the
second-person singular pronoun your but the third-person plural their. If a modern dictator
should threaten a nation, he might say, "I will destroy all of your allies, and then I will send
my army against your cities and break down your fortifications." Who hearing a statement
like this would not understand that the first clause was a threat against only the nation's allies
and that the second clause was a threat against the nation itself? This is exactly the situation in
Ezekiel's prophecy against Tyre. The prophecy quickly addressed the fate of Tyre's daughter
villages (v:8), which had always been sitting ducks, and then turned to Tyre proper, the island
fortification that posed the greatest problem to an invading army.

The extensive allusions to Tyre as a city associated with the sea shows that the prophecy was
concerned with Tyre proper and not the mainland suburbs, which had often been attacked and
destroyed by invading armies. These allusions are too extensive to examine in a single article,
but Ezekiel said that after the complete destruction of Tyre, "Shall not the isles shake at the
sound of your fall, when the wounded groan, when the slaughter is made in your midst?"
(v:15). All the "princes of the sea" would come down from their thrones and tremble and
"take up a lamentation" in which they would say, "How are you destroyed that was inhabited
by seafaring men, renowned city, that was strong in the sea, she and her inhabitants" (vs:14-
17). All of this was obviously applicable to the island city of Tyre but not to the mainland
villages. They had been overrun so often that no nation would have expressed surprise that

Volume 1990 - 2002 Issue


Page 917 of 2049
Skeptical Review Edited by Farrell Till
they had been destroyed one more time. It was the island city of Tyre that had always proven
to be impregnable.

Tyre still exists today despite Ezekiel's prophecy that it would be destroyed and never rebuilt,
but it exists only as a shadow of its former glory. The causeway that now links the city to the
mainland, not to mention modern weaponry that would be available to an invading army, has
forever removed the natural protection that Tyre enjoyed in Old Testament times, so for
Hogan to think that nations will express astonishment at the final destruction of Tyre when
"Christ returns" is an utterly ridiculous attempt to salvage this prophecy from the garbage
heap of other prophecy failures that it must be relegated to. A modern army could attack Tyre
today and wipe it from the face of the earth, and no one would see anything astonishing about
it. So it turns out that Hogan is the one who has erected a straw house.

Distinguishing Fact from Fiction in the Man


from Galilee
Stephen Van Eck
Opinions among skeptics about the historicity of Jesus range from a verdict of totally
fictitious to a suspicion that the gospels may be based to some degree on historical reality.
Determining what degree of fact or fiction, however, is the real challenge. How much of the
story is true and how much is based on legendary amplification, doctrinal distortion, and
evangelical embellishment have perplexed anyone who examines the matter beyond an idiot-
fundamentalist level of total mindless acceptance. The complete absence of historical
information about Jesus beyond the gospels and the corresponding lack of inside information
about the origins of Christianity make textual analysis the only avenue of approach for serious
critical examination.

If Jesus actually existed at all, perhaps the surest authentic detail about him was that he was a
Galilean. This is evidenced by the divergent ways Matthew and Luke dealt with his Galilean
origin. Luke (2:1-7) contrived to get Jesus's parents from Galilee to Bethlehem so that he
could be born there in fulfillment of the Messianic expectations. Matthew (2:1-2,11) had
Jesus born in a house in Bethlehem, where his parents apparently had been living all along
and then contrived to have them subsequently settle in Galilee to avoid a purported royal
threat (2:19-23).

While Matthew and Luke resorted to totally different contrivances, both point to an
underlying awareness of the Galilean origin of Jesus and a need to account for this somehow.
The fact that there was no Messianic oracle that predicted a Galilean Messiah made this all
the more essential to account for and less likely to be a fictionalization.

Volume 1990 - 2002 Issue


Page 918 of 2049
Skeptical Review Edited by Farrell Till
Providing further support is the ironic reference (John 7:52) to the standard belief that no
prophet would come out of Galilee, a notion refuted by the presence of a Galilean prophet as
it was being repeated. Only for an actual Galilean would his origin need to be so addressed.
Were the story entirely fictional, they wouldn't have made him Galilean to begin with and
would not have needed to resort to contrivance to explain it.

However, the very existence of contrivance and contradiction underscores the fictional nature
of perhaps most of the gospel details, and the difficulty of isolating any authentic information.
It is easier to identify and account for false information than true. Most pertinently, it is likely
that the story of Jesus's being born in Bethlehem is purely fictional, since this idea had been
suggested by Micah 5:2. This passage created an expectation that the Messiah would be born
in Bethlehem, and the gospelers, knowing this, may well have inserted it into their narratives,
assuming it must have been the case. Since the scriptures were assumed to be the word of
God, the events they purportedly foretold and the details they suggested had to have occurred,
according to the gospelers' line of thinking.

What makes fabrication the more likely here is the fact that this particular Messianic oracle--
like virtually all others--was badly misunderstood. According to the standard Christian
interpretation, Micah is predicting the town where Jesus would be born. But Bethlehem here
actually referred to a person, not a place. First Chronicles 2:51, 54 both contain references to
a man named Bethlehem. Matthew, who wrote from the Septuagint, took unwarranted
liberties with the Septuagint text, changing the actual phrase "Bethlehem of the house of
Ephrathah" to "Bethlehem, in the land of Judah." The fact that there was a house of Ephrathah
is confirmed by 1 Chronicles 2:50, and supports the proper reading of the text of Micah.

The alleged Messianic passages were further distorted by taking them out of context, and this
was the case for Micah as well. One need only read further (vs:5-6) to know that Micah was
referring to a contemporary situation with the Assyrians and predicting a great military leader
who would wreak vengeance on their oppressor. The fact that this did not occur is bad enough
for Micah's prophetic credibility, but it clearly cannot be referring to Jesus--unless you take
the position that "Assyria" is actually referring to Rome, which would be typical exegetic
fudging (also known as "pee wee hermeneutics"). But then, Jesus didn't effect any triumph
against the Romans either, so he comes out a loser regardless.

The degree of distortion involved regarding the alleged Bethlehem prophecy can only lead an
objective mind to conclude that Jesus's birth in Bethlehem is almost certainly a fictional
element. The character may well be based, however loosely, on an actual person, and that he
was Galilean is quite plausible, based on how Matthew and Luke contrived their material, but
what else about the story may be true, obscured amidst a thicket of legendary accretions,
bogus scriptural fulfillments, and dogmatic modifications, will be even harder to establish
than that.

(Stephen Van Eck, Rural Route 1, Box 62, Rushville, PA 18839-9702.)

Volume 1990 - 2002 Issue


Page 919 of 2049
Skeptical Review Edited by Farrell Till

From the Mailbag


Not a Name-Caller...

Uh, I'm not a name caller or anything, but, you know, I really don't know what else to say.
You're a college professor.... I'm not a religious man, but you really are a loser.

(Anonymous phone message)

EDITOR'S NOTE: I've never published phone messages before, but I thought I should make
an exception for this one. It was left on the office message machine sometime between 1:00
A.M., when I shut down the computer and went to bed, and 8:00 P.M., when I returned to the
office the next morning. Since the message was left on this machine, I assume that the call
was made by someone who is a subscriber to TSR, because I have never advertised this
number locally, and it is listed in the directory under The Skeptical Review, rather than my
own name. Probably, then, the call was made by a frustrated subscriber (who isn't a religious
man, of course) wanting to make a sneak attack that he lacks both the courage and the
knowledge to make in a direct confrontation. I'm used to anonymous messages like this, but
they usually come by mail with the obligatory religious tract enclosed to describe the agonies
of hell that I'm sure to endure someday. Each time I get them, I know there is a frustrated
biblicist somewhere, steaming over his/her inability to confront me with logical
argumentation. Those who send such messages may as well end them with a comment like
this: "Well, you win. I can't refute your arguments with logic, but I can insult you in a
cowardly anonymous way."

I'm a loser? If so, what would you call someone who is so cowardly that he leaves anonymous
messages on a phone machine in the middle of the night when he is reasonably sure that no
one will be around to take his call?

A Small World...

I first became acquainted with your writing in the Secular Humanist Bulletin. I was delighted
to learn that you were the same Farrell Till who taught at Spoon River College. Your
understanding of the believer's mind and dissection of Christianity are right on the mark.

[List of materials ordered and names submitted for gift subscriptions deleted.]

(Nathan Robertson, 4101 West 45th Avenue, Apt. 3314, Amarillo, TX 79109.)

EDITOR'S NOTE: It's a small world after all. When Nathan Robertson sent me his
subscription request from Amarillo, Texas, I didn't associate him with the Nathan Robertson
whom I had had as a student several years ago, but I remember him very well. Student essays
on religious subjects are very common, but 99.9% of them are so shallow that they make
religious tracts look downright profound. One day I read an essay that discussed the writer's
reasons for not believing in God, and it took a logical approach to the subject that was like a
gust of fresh air in a musty dungeon. Instead of platitudes and stale cliches, it actually

Volume 1990 - 2002 Issue


Page 920 of 2049
Skeptical Review Edited by Farrell Till
contained ideas and thoughts that I considered unusually insightful for an 18- or 19-year-old
college student. That essay was written by Nathan Robertson.

It also led to private discussions outside of the classroom that contributed to some significant
changes in my life. Until that time, I had confined my activities on behalf of religious
skepticism to a few letters-to-the-editor in the local newspaper, but for the most part I had
bought the idea that it just wasn't nice to criticize people's religious beliefs. After all, someone
might be offended by comments that suggest religion is illogical, and so I kept my thoughts to
myself much more than I do today. One day Nathan showed me some copies of a publication
that his brother subscribed to, and this was how I was introduced to Free Inquiry magazine. I
liked what I saw (which was a more direct criticism of religious absurdities than its present
editorial approach) and subscribed to it. In reading Free Inquiry, I found information about
organized opposition to religion and soon became more directly involved in it myself. I don't
know if this would have happened if I had not met Nathan Robertson, but I've always thought
that the experience helped. Anyone who has ever been a professional teacher will tell you that
teaching isn't just a one-way street. Teachers can also learn from their students as I did from
Nathan Robertson.

As far as I am concerned, your articles on religion and the fallacies in the bible are pure prose!
Your essays are well written and can be backed up. I am a 60-year-old business man here in
Amarillo. My wife and friends are devout Catholics and my opinions, which coincide with
yours, are not too well received. The older I get, the more I am convinced of the shortcomings
of religion.

Please apply this $100 as far as it will go on my subscription to The Skeptical Review. If you
are ever coming through Amarillo, please let me know ahead. I would be honored to meet you
and visit with you. Keep up the good work.

(Joe H. Virden, P. O. Box 7160, 2821 Mays Street, Amarillo, TX 79109.)

EDITOR'S NOTE: When Nathan Robertson sent his subscription request, I believe he said
that he sometimes thinks he is the only skeptic in Texas, so I am publishing Mr. Virden's
letter along with his to show that skeptics are not as alone as they think. Besides Mr. Virden,
there is another subscriber to TSR in Amarillo. He has a D.D. degree, and his correspondence
very definitely shows that he rejects traditional claims about the Bible. I don't know how
public he wants to be about this, so I will leave it to him to contact the other two birds of a
feather in Amarillo.

I appreciate Mr. Virden's support, but I appreciate even more his optimism. I have extended
his subscription for 16 more years, but... will an editor who is now 64 be around that long? I
hope so.

If I ever pass through Amarillo, I will let him know. I would enjoy meeting him and seeing
Nathan Robertson again.

Atheists Who Converted...

Volume 1990 - 2002 Issue


Page 921 of 2049
Skeptical Review Edited by Farrell Till
I await with interest every issue of the review, which I read with pleasure, especially the
letters from readers and your replies to them.

However, in the May/June 1996 issue, I find two opinions from you which I cannot accept
because I feel they are not accurate. Voltaire asked for pardon to God and to the Church when
he felt next to death. He signed a statement, on March 2, 1778, in which he assured to have
confessed (his sins) to "abbe" Gaultier, a Catholic priest, and his wish to die in the Saint
Catholic Church in which he was born. Maybe I should mention that Voltaire was never an
atheist, in spite of what many people think. Quite the contrary, Voltaire wrote: "Atheists, in
their greatest part, are scholarly persons, daring and lost, who reason badly, and who, being
unable to understand the creation, the origin of evil and many other difficulties, have come to
the hypothesis of the eternity of things and to the hypothesis of their need." (By the way, I am
astonished to learn that many American freethinkers and atheists consider Tom Paine their
Patron Saint. Probably they are unaware of the fact that his book Age of Reason aimed, as he
said, to keep France from atheism by proving that Newtonianism, showing the orderly design
of nature and the cosmos, presupposed a Divine Designer, one God. In fact, Paine began the
Age of Reason with this statement: "I believe in one God, and no more; and I hope for
happiness beyond this life.") I hope you will now change your opinion about Voltaire's
repentance.

I can offer you several names of famous atheists who later changed to Christianism. Charles
Foucault and Jacques Maritain (and his wife), both of them French philosophers, Andre
Frossard, son of a former Secretary of the French Communist Party, Vittorio Messori, the
interviewer and collaborator of John Paul II's Crossing the Threshold of Hope, Max Jacob, the
Jewish poet and friend of Picasso, Evelyn Waugh. The latest case I have learnt is that of
Sheldon Vanauken, a professor of history in Virginia, USA.

Last but not least, have you ever wondered why so many fundamentalists, like yourself or
Dan Parker [Barker probably intended], later become atheists? Maybe it is a matter of
attitude. Some people need affirming or denying faith categorically. In your words, they need
to be right.

(Antonio Casao Ibanez, Apartado 882, 50080 Zaragoza, Spain.)

EDITOR'S NOTE: I was delighted to hear from Mr. Ibanez. He has been a long-time
subscriber to TSR. and if I am not mistaken, he was the first in Europe to subscribe. We now
have subscribers in France, Italy, Belgium, the Netherlands, England, Ireland, Northern
Ireland, Norway, Sweden, Finland, Germany, Iceland, and on every inhabited continent. Most
of this interest from abroad has been the result of putting our articles on the WWW.

I had to check the May/June 1996 issue to see what statement Mr. Ibanez was referring to,
and the best I can tell, he was responding to my note at the end of Mike Ulm's letter on page
13. This was where after commenting on the many claims of former atheism that we hear
from apologists like Josh McDowell, I went on to say that I don't know of a single firmly
committed atheist who converted to theism. I was not claiming that such had never happened;
I was just stating that I couldn't think of any. Almost everywhere I go to debate, I am
confronted by frustrated Christians who try to tell me that Darwin, Russell, Hume, Huxley,

Volume 1990 - 2002 Issue


Page 922 of 2049
Skeptical Review Edited by Farrell Till
and such like renounced their atheism or skepticism on their deathbeds. Such tales as these are
commonplace, even though most of them have been debunked by people who were present at
the times of death, but to hear these people, one would think that it is practically impossible
for an atheist to die without getting cold feet and frantically pleading for divine mercy with
his last breath. I am reasonably sure that most stories like these are claims that were fabricated
by insecure theists who are desperately hoping to give some kind of shot in the arm to their
own anemic beliefs. At any rate, I would never claim that no atheist has ever renounced his
atheism at the end of his life, because it has undoubtedly happened, just as there have
undoubtedly been theists at the end of their lives who realized there was nothing to their god-
belief and renounced it. Examples like these would really prove nothing at all, and I made the
comment that Mr. Ibanez wrote about only to express my doubt that preconversion atheism in
the ranks of Christian apologetics was as widespread as they claim.

I am a bit puzzled by Mr. Ibanez's citation of Voltaire as an example of an atheist who


converted to theism. He referred to a confession of faith that Voltaire allegedly signed before
his death (although Mr. Ibanez didn't document his source) but then immediately proceeded to
claim that Voltaire was never an atheist. If he were never an atheist, then he couldn't very well
be an example of an atheist who converted to theism before his death, could he?

I certainly know that Thomas Paine was not an atheist, and a major irritation with me is the
uninformed theist who knows no better than to call Paine an atheist. Just recently I had a
discussion with a person in the community who called Paine an atheist and then resisted all of
my efforts to correct her wrong impression. Thomas Paine was a Deist, and I'm sure that most
skeptics and atheists know this. It was the freethinking ability of the man, at a time when it
was unpopular to swim against the current of Christianity, that we admire. Also, his refutation
of the popular belief that the Bible is God's word remains a classic to this day, and no
Christian apologist has ever successfully responded to it. Pain did indeed begin Age of Reason
exactly as Mr. Ibanez quoted him, but after his profession of faith in God, he immediately
went on to say, "But, lest it should be supposed that I believe in many other things in addition
to these, I shall, in the progress of this work, declare the things I do not believe, and my
reasons for not believing them."

It was what Paine said, in the progress of this work, that he did not believe that has made him
a permanent enemy of organized religions. Among other things, he said, "I do not believe in
the creed professed by the Jewish church, by the Roman church, by the Greek church, by the
Turkish church, by the Protestant church, nor by any church that I know of. My own mind is
my own church" (200th Anniversay Edition, p. 2). Later, he said in reference to the stories of
murder and atrocity that abound in the Bible, "Whenever we read the obscene stories, the
voluptuous debaucheries, the cruel and torturous executions, the unrelenting vindictiveness,
with which more than half the Bible is filled, it would be more consistent that we called it the
word of a demon, than the word of God. It is a history of wickedness, that has served to
corrupt and brutalize mankind; and, for my part, I sincerely detest it, as I detest everything
that is cruel" (Ibid., p. 12). When we consider the times in which Paine made such statements
as these, is it any wonder that he would be considered the "patron saint" of modern
skepticism?

Volume 1990 - 2002 Issue


Page 923 of 2049
Skeptical Review Edited by Farrell Till
I certainly can't speak for Dan Barker or other fundamentalists who became skeptics and left
the Christian ministry, but I will say that I do subscribe to what President Harry Truman
allegedly said: "I would rather be right than president." I can't imagine why anyone would put
religious or philosophical beliefs above the desire to be right, but obviously many people do.
Was Mr. Ibanez suggesting that Barker and I should not feel the need to be right?

Anyone who is interested in a very reasonably priced paperback copy of Age of Reason
should write to Stephen Van Eck. RR 1, Box 62, Rushville, PA 18839. He published the
200th Anniversary Edition and distributes it for only $5 per copy.

Another Opinion from Afar...

The other day I received the September/October 1996 issue of TSR, and as usual, I read it
from cover to cover upon arriving home. As has been the case with the previous issues of
TSR, all of the articles in this issue were rather thought provoking. Predictably, the inerrantists
continue to grasp at straws, use weak logic and maintain double-standards. Your excellent
article concerning Ezekiel's failed prophecy about the city of Tyre needs no further comment.
The Bible seems to be screaming loud and clear that it's only of rather dubious human origins.
It amazes me that the inerrantists just can't seem to hear. As they say, there is one miracle
about the Bible--that there are actually people who believe that it's God's Word!

Additionally, I think Roger Hutchinson's article "How Many Women Went to the Tomb?"
really proved the opposite of what he intended. Frankly, it stands out as another glaring
example of how weak the inerrantist position really is. One of the things that really struck me
is that he hardly even mentioned the (what-he-considers-to-be) "facts" from the Gospel of
John. Besides saying that John only mentions that there was one visitor to the (allegedly)
empty tomb, Mr. Hutchinson left out the other evidence from this gospel almost altogether. I
think the reason for this is that some of the information in John would make Mr. Hutchinson's
task increasingly difficult. That is assuming, of course, that something that is already
impossible can be made more difficult. As usual, when trying to harmonize the gospel
narratives, Christians have to leave out many "facts" that just refuse to fit in. The so-called
"Resurrection Story" would not be believable even if we limit ourselves to the Synoptic
Gospels, and throwing in John's account just makes matters worse.

In his article, Mr. Hutchinson emphasized that the women visited the tomb the morning after
the Sabbath in order to anoint the body of Jesus with spices. However, on this point Mr.
Hutchinson conveniently forgot to mention the fact that the Gospel of John says that the body
had already been anointed on the day of the crucifixion (read "cruci- fiction")! According to
John 19:38-42, Joseph of Arimathea and Nicodemus "took the body of Jesus, and bound it in
strips of linen with the spices" and then put it in a tomb. Also, John's account of Mary
Magdalene's visit to the tomb early Sunday morning does not mention that she came there to
anoint the body of Jesus. Well, at least this part of John's narrative is internally consistent,
since according to him, the anointing had already been done. To make matters worse,
however, John's gospel places Mary Magdalene at the tomb earlier than any of the other three
gospels, i.e., "while it was still dark"--but the stone had already been removed! Matthew and
Mark say that the women came to the tomb around the rising of the sun, but the tomb was still
closed!!!

Volume 1990 - 2002 Issue


Page 924 of 2049
Skeptical Review Edited by Farrell Till
If "harmonizing the Gospel accounts" was really the purpose of Mr. Hutchinson's article, then
we're still waiting. Actually, if I thought it were reasonably possible, I would suggest that you
consider trying to verify the credentials and beliefs of the writers who submit rebuttal articles.
As others before me have said, maybe they're really closet freethinkers just trying to make the
inerrantist position look untenable! Thanks again for your great service. I'm anxiously await
the next issue of TSR.

(Robert Squires, P.O. Box 23906, Safat 13100, Kuwait; e-mail rws@kuwait.net)

EDITOR'S NOTE: Roger Hutchinson will recognize Robert Squires as a former subscriber to
the errancy list on the internet, who defended Islam as a religion superior to Christianity, so
Roger may be inclined to view Squires' letter as merely the comments of a convert to Islam
who wants to bash his former religion. However, in my private e-mail messages with Mr.
Squires, I have learned that he is not a convert to Islam but a religious skeptic who was only
using the errancy list to test Christian claims against what he has learned from living in an
Islamic nation. His observations represent the flaws that he now sees in the religion he was
indoctrinated in when he was growing up in this country. His comments deserve Hutchinson's
serious consideration.

Are Inerrantists Really Sincere?...

Do inerrantists really believe the Bible is inerrant, or do they just maintain it for the sake of
appearance? I wonder.

(E. E. Brennaman, 1601 Airline Road, Apt. 62, Corpus Christi, TX 78412-4434.)

EDITOR'S NOTE: I am convinced that many inerrantists really know that the inerrancy
doctrine is untenable but still adhere to it for emotional reasons. This may be an unfair
accusation, but I am basing it on my own experience. Even when I knew beyond any
reasonable doubt that the Bible just couldn't be "the inspired word of God," I couldn't bring
myself to admit it. What will my friends and relatives think? How foolish will I look? How
will I support my family? These were some of the emotional questions that I grappled with
before I made my decision to do what I knew was the right thing and leave a profession that
required me to be untrue to my conscience. I can't believe that I was unique, and neither can I
believe that all fundamentalists who come to the same realization that I did always do what
their consciences tell them to do. I also think that some preachers just enjoy being big fishes
in little ponds, and they are willing to achieve this at the cost of their personal integrity.

The Values of Abraham & Sarah...

I thoroughly enjoyed and applauded your "Family Values" piece regarding the questionable
ethical behavior and moral values demonstrated by Abraham and Sarah. However, over the
years, whenever I have mentioned the despicable immorality of Old Testament characters,
both Christians and Jews have promptly defended such behavior by pointing out that in those
days people were living under very different laws and very different social traditions. They
quickly point out that incestuous pregnancies and marriages were very common and were
considered both legal and moral. Further, that servant women frequently filled the role of

Volume 1990 - 2002 Issue


Page 925 of 2049
Skeptical Review Edited by Farrell Till
surrogate mates was again both socially and legally acceptable. Still further, they argue that it
was considered appropriate for the patriarch to sell or trade his wife or children for livestock
or other material gain, and they even argue that deliberate deceitfulness was perfectly moral
and ethical under certain circumstances, such as for the purpose of surviving, tricking an
enemy, fulfilling prophecy, and achieving the greatest good for the greatest number. As for
their treatment of Ishmael and his mother, even that was considered acceptable for the
prevailing standards and traditions of the time.

While I personally agree with you, fully, that Abraham and Sarah were a prime example of
terrible "family values," I don't think we can properly or fairly judge the actions of ancient
and primitive people by the moral and ethical standards of 20th-century America. You wrote
that "in today's society, Abraham and Sarah would be called `swingers,'" but you are much
too kind. By our contemporary standards, Abraham was nothing more than a pimp, a liar, an
incestuous sex offender, an abusive cold-hearted bastard, and a deluded, greedy, passive-
aggressive, cowardly, common, low-life, criminal scumbag. But can we judge him only by the
laws and traditions of his time, his culture, and his religion?

Sir, if there is anything in the Bible to indicate that the actions of Abraham and Sarah were
illegal, unethical, or immoral by the laws, standards, and traditions of that time, culture, and
religion, then I would dearly love to know about it! Are the Christians and Jews correct or
incorrect when they claim that Abraham was a very moral man by laws and standards
applicable to him at that time?

(Grant H. Hendrick, A-131851, P. O. Box 5000, Carson City, MI 48811-5000.)

EDITOR'S NOTE: I certainly agree that the morality of Abraham and Sarah was probably
typical of the time they lived (if indeed they were actual historical persons). This was a time
when, as Mr. Hendrick correctly pointed out, polygamy, incestuous relationships, slavery,
barbarity, and other customs repugnant to modern civilizations were commonplace, but that is
exactly the point that I wanted to make in the "family-values" articles currently running in
TSR. How can the Christian Right advocate as openly as it does a return to biblical values in
morality when they would soundly condemn the morality of biblical characters like Abraham
and Sarah, Isaac and Rebekah, Jacob and his wives and concubines, etc. if it were practiced
today? Furthermore, a fundamental claim of biblicists is that morality is absolute, but if
morality is absolute, then moral principles must remain constant and never change.
Consequently, what was moral in the time of Abraham and Sarah must be considered moral
today, or else the entire structure of absolute morality collapses. So biblical moralists need to
explain why they are pressing to impose on society a moral system that they don't really
believe in themselves.

On Track...

Nice job on The Skeptical Review. You are right on track all right. The Holy Bible and the
entire Christian doctrine are nonsense. I also read The Skeptical Inquirer, Freethought Today,
and other publications. All of them are good.

Volume 1990 - 2002 Issue


Page 926 of 2049
Skeptical Review Edited by Farrell Till
The human race has existed for roughly 3 million years. During this time, many millions of
people have lived and died, yet Christianity has existed for less than 2 thousand years. So
most humans lived before Jesus (if indeed he ever existed). What about the zillions of people
who lived before Jesus? How are they judged? Even today, how are the millions judged who
know nothing of the Christian doctrine? Do believers ever think about these questions? Surely
not many. Keep up the good work.

(Doug Tracy, 5345 Rector Street, Toledo, OH 43615-2811.)

EDITOR'S NOTE: Do believers ever think about these things? As Mr. Tracy said, "Surely not
many." Those who do would probably say that people who lived and died before Christ will
be judged by the moral standards of their eras, and perhaps some will know enough about the
New Testament to quote the apostle Paul, "For as many as have sinned without the law shall
also perish without the law, and as many as have sinned under the law shall be judged by the
law" (Rom. 2:12), as if this would prove anything except that this was what Paul thought. As
for those who have died in the Christian era without ever hearing of Christ, their fate will, of
course, depend upon which Bible-believer you ask. Some will be calloused enough to say,
"Tough bippy, they went to hell," but others will make the "sacrifice" of Jesus completely
meaningless and say that those who die without hearing the gospel will be saved if they were
moral people in the societies they lived in. That's the nice thing about Christianity. It's
somewhat like a smorgasbord. Believers can look everything over and pick whatever appeals
to them.

Mr. Tracy's reference to the age of the human race made me think about a comment that
Christians often make about those who challenge the authenticity of the Bible. They delight in
referring to skeptics of the past who predicted that belief in the Bible was doomed for
extinction, and those who know a little bit about the scriptures may even quote Matthew
24:35, "Heaven and earth shall pass away, but my words shall not pass away." Of course, they
conveniently ignore--or, more likely, don't even know--that this statement was made in the
context of a prediction that the second coming of Jesus would occur in the lifetimes of the
generation he allegedly lived in (Matt. 24:29-34). At any rate, those who cite the age of
Christianity as some kind of evidence of its divine origin show an incredible ignorance of the
history of religion. Records of the past show that religions are born, thrive sometimes for
centuries, but eventually wane and die. One would have to be very naive to think that
Christianity will be any different.

From Agnostic to Atheist...

In the past year, since I started receiving The Skeptical Review, I have read all of your back
issues, two of your debates, Prophecies: Imaginary and Unfulfilled, and a good portion of the
Bible. I have also read works by Bertrand Russell, Madalyn Murray O'Hair, Dan Barker,
Robert Ingersoll, and Thomas Paine, all of whom directly confronted Christianity. To shore
up my very weak knowledge of science, I have read books by Paul Davies, John Gribbin,
Stephen Jay Gould, Carl Sagan, and Isaac Asimov. And, finally, to appease the wishes of my
parents, I have read pro-Christian books by Josh McDowell, Ravi Zacharias, Donald Ford,
and R. C. Sproul.

Volume 1990 - 2002 Issue


Page 927 of 2049
Skeptical Review Edited by Farrell Till
I used to be agnostic, and that was based mostly on my negative emotional response to
fundamentalism. Thanks to the past year of intense study, I am happy to be more confident
about my rejection of fundamentalist Christianity, but there's more. I am now confident
enough to say that none of the world's religions could possibly be true and, therefore, I am an
atheist.

I have become quite activist about the issue because I can now see just how much religion
affects society. I have managed to convince my only brother and my best friend [names
deleted] to start down the road that I've been on and receive your publication. My wife has
also moved from an agnostic to atheist position. I have many fundamentalist friends and
family who are no longer "safe" in my presence and can be assured that I will confront their
idealism if the situation warrants. For the first time in my life, I feel utterly at peace regarding
the "meaning" and "origin" of it all. Life is what matters, and I'm living it and enjoying it.
Thanks for your part in enlightening me.

As a favor to me, for reading all the Christian books they have sent me, my parents have
offered to receive and read your publication. [Names and address deleted.]

Also, for some reason, I have not yet received Volume 7 #2 and Volume 7 #5. Could you
send me replacement copies? For some reason, I received all other copies for this year but not
those two.

(Dave Friesen, 902 South Loop 499, Apt. 3-8, Harlingen, TX 78550.)

EDITOR'S NOTE: Letters like this are always a joy to receive. As a preacher, I "converted"
several people, but I can't remember those experiences being nearly as satisfying as knowing
that I may have helped someone see the folly of religious superstition. I'm also glad to see that
Mr. Friesen, as so many others have done in this forum, testified to the personal happiness
that he found in facing reality rather than using the false security of religious belief as a crutch
to help him hobble through life. Those who have traveled down both roads know that
accepting reality brings more personal satisfaction. No "personal experience with Jesus" can
compare to the pleasure of knowing that one has found the courage to look reality in the face,
accept it, and live life accordingly. Theists who gamble the only thing they can ever be sure
of, i. e., the life they now have, on wishful thinking will never know this kind of personal
satisfaction.

Another Satisfied Reader...

I'm sending back payment for the issues already received. They are well worth it. The
Skeptical Review is beyond any doubt the most informative publication on the idiocy of
religion, not only the Christian religion but the general mentality of all believers in gods,
devils, angels, hell, and you name it. Keep up the excellent work.

(Melvin Leui, 4900 South Sunnyside Drive, Rapid City, SD 57701.)

How Many Resurrections?...

Volume 1990 - 2002 Issue


Page 928 of 2049
Skeptical Review Edited by Farrell Till
Please send me any information concerning how many and who were supposed to have risen
from the dead. I know there were stories of others prior to the Lazarus and Jesus characters.

(John Rye, 311481-A, P. O. Box 396, Hardwick, GA 31034.)

EDITOR'S NOTE: Besides the resurrections of Lazarus and Jesus, the Bible claims that Elijah
raised the widow of Zarephath's son (1 Kings 17:8-24); that Elisha raised the Shunammite
woman's son (2 Kings 4:17-37); that Jesus raised Jairus's daughter (Mark 5:22-43; Matt. 9:18-
26; Luke 8:41-56) and the widow of Nain's son (Luke 7:11-16); that an earthquake shook
open the tombs of "many saints," who were later resurrected and appeared to "many" in the
city of Jerusalem (Matt. 27:51-54); that Peter raised Dorcas, who was also known as Tabitha
(Acts 9:36-43); and that Paul raised Eutychus, who went to sleep during Paul's sermon, fell
from a third story window, and killed himself (Acts 20:7-10). Ezekiel's vision of the valley of
dry bones could also be construed as a resurrection (Ezek. 37:1-10), as could also an incident
where a band of Moabite marauders cast one of their dead into the cave where the prophet
Elisha had been buried. When the man's body touched the bones of Elisha, "he revived and
stood upon his feet" (2 Kings 13:20-21). In other words, claims of resurrection from the dead
were not at all unusual in the Bible, a fact that should--but won't--make Christians wonder
about their faith in a savior whose resurrection was recorded only in a book in which
resurrections were fairly commonplace. Since resurrections from the dead are never witnessed
today, how likely is it that all of the resurrections claimed in a collection of ancient
documents, written in highly superstitious times, actually happened? How likely is it that any
of them happened?

Volume 1990 - 2002 Issue


Page 929 of 2049
Skeptical Review Edited by Farrell Till

Skeptical Review
Volumen Seven, Number Two
March/April 1997
Farrell Till, editor

• A Final Look at Family Values


Till's last article on family values in the old testament.

• Prophecy of Seventy Years of Servitude to Babylon


Price presents Jeremiah 25:1-4 as a fulfilled prophecy.

• A Bad Example of Prophecy Fulfillment


Till replies to Price's defense of the Jeremiah prophecy fulfillment.

• Perman Wrap-Up
Till finishes his response to perman. The scarcity of extrabiblical references to Jesus
are discussed.

• The Nature of the Claim


Till explains why the nature of the resurrection claim makes it improbable

• From The Mailbag

A Final Look at Family Values


The Christian Right touts the need for biblical family values in modern society, but when the
lives of famous biblical characters are examined, as was done in previous articles in this

Volume 1990 - 2002 Issue


Page 930 of 2049
Skeptical Review Edited by Farrell Till
series, we quickly see that they were not the kind of role models that most parents today
would like for their children to emulate. Abraham, Sarah, Isaac, and Jacob were all listed in
Hebrews 11 as great "heroes of faith," but their individual stories in the Old Testament depict
them as people who lied and practiced various other types of deception, who granted sexual
favors for personal gain and sometimes engaged in polygamous relationships, who showed
favoritism to their children, and did many other things that no decent family today would
consider morality worthy of emulation.

This series would have to be continued indefinitely in order to examine the personal lives of
all of the biblical characters whom Christians consider great servants of their god, so this final
installment in the series will summarize only the "family values" of other notables whom the
writer of Hebrews also listed as heroes of faith.

"By faith, Noah, being warned of God concerning things not seen as yet, prepared an ark to
the saving of his house" (Heb. 11:7). In Genesis 6:9, Noah was described as a "righteous
man," who was "perfect in his generation," so one would think that if any biblical character
was worthy of imitation, certainly this man who was chosen of God to save human life from
extinction would be the one. But one of Noah's first activities recorded after the flood was the
planting of a vineyard, whose produce was used to make wine. Noah consumed so much of
the wine that he became drunk and lay naked in a drunken stupor in his tent. When he later
learned that his son Ham had seen him naked, he pronounced a curse upon Ham's descendants
(Gen. 9:20-27). There just wasn't very much in the character of this man to admire.

"And what shall I more say? For the time would fail me if I tell of Gideon and Barak and
Samson and Jephthah, also of David and Samuel and the prophets, who through faith subdued
kingdoms, worked righteousness, obtained promises, stopped the mouths of lions, quenched
the violence of fire, escaped the edge of the sword..." (Heb. 11:32-34). The Bible may
attribute such deeds as these to some of these men, but it also attributes to them some deeds
that are not quite so admirable.

To consider Jephthah a man who practiced "family values," new meaning would have to be
given to the term. His mother was a harlot (Judges 11:1), which, of course, was no fault of
his, but his treatment of the only child he had has to be one of the most despicable stories in
the Bible. As he was preparing to battle the Ammonites, he vowed to Yahweh that if he were
successful against them, whatever he first saw upon returning home, he would offer it as a
burnt offering to Yahweh. After defeating the Ammonites, his daughter (his only child) came
out to greet him when he returned to his house. He was kind enough to allow her two months
to "bewail her virginity," after which he "did with her according to his vow" (Judges 11:29-
40). What father today would consider this man's family values the kind to imitate?

According to a quaint little story in Judges 6-8, Gideon delivered the Israelites from Midianite
bondage with an army of only 300 men, who lapped water like a dog (7:6-7), and then Gideon
judged Israel for 40 years, during which time he sired 70 sons with his "many wives" and a
concubine named Shechem (8:29-30). He hardly seemed the type of father the Christian Right
would urge men to emulate.

Volume 1990 - 2002 Issue


Page 931 of 2049
Skeptical Review Edited by Farrell Till
Samson was the famous strong man of Israel, who killed a thousand men with the jawbone of
an ass (Judges 15:15). Another feat of his was the "plucking up" of the gate and gateposts of
Gaza, which he then carried away to the top of a mountain, but Sunday school lessons don't
mention the fact that he did this after having visited a harlot in Gaza (16:1). After this, he
developed a relationship with a beautiful Philistine woman named Delilah with whom he
spent a lot of time in her "chamber." Although sexual activities between Samson and Delilah
were never explicitly stated, they were certainly implied. Bribed by an offer of 1100 pieces of
silver from each Philistine lord to betray him, Delilah twice begged Samson to reveal the
secret of his strength, and both times he lied to her about its source. Each time, Delilah then
betrayed her intentions by binding him in the manner Samson had described in his lies. If
stupidity were a family value, then Samson would easily qualify as the all time hero of family
values, because after twice seeing Delilah's real intentions, he foolishly succumbed to her
wiles on a third attempt and told her that the real secret of his strength was in his hair. Delilah
used this information to subdue him and deliver him to the Philistines for the money she had
been promised. So Samson was a liar who consorted with harlots. If this is the Hebrew
writer's idea of a "hero of faith," so be it, and if the Christians want him as an example of
family values, they're welcome to him.

David was a man after Yahweh's own heart (1 Sam. 13:14: Acts 13:22), so what better way to
conclude this discussion of biblical family values than with the legacy that David left? Like
many other heroes of faith, David practiced polygamy but apparently wasn't satisfied with the
sexual diversity to be found in having at least six wives (2 Sam. 3:2-5), for after becoming
undisputed king over a united Israel, he demanded that Saul's daughter Michal be given to
him as another wife. Before the rift developed between Saul and David, Saul had promised
Michal to David for the foreskins of 200 Philistines, whom David then went out and killed in
order to pay the dowry Saul had demanded (1 Sam. 18:27). By the time David had become
king, Michal had married a man named Paltiel, but this didn't deter David from claiming her
as another wife. Michal was taken by force from her husband, who followed along behind her
weeping, until Abner (David's agent) turned and ordered him to return home (2 Sam. 3:12-
16). How's that as an example of how to hold a family together with good old-fashioned
biblical moral values? But such a display of petty vanity should only be expected from a man
after Yahweh's own heart? Surely, when Christian men today are looking for role models in
family values, David has to be far down on their list.

David, who now had at least seven wives, still wasn't satisfied, for one day when he saw from
the roof of his house Bathsheba, the wife of a soldier in his army, bathing, he sent messengers
to take her and bring her to his house. The result was an embarrassing pregnancy, which
David tried to pin on Bathsheba's husband Uriah by calling him home on furlough to spend
some time with his wife, but when Uriah refused to enjoy the pleasures of a conjugal visit
while his unit was fighting a war, David sent him back with a letter to his commanding officer
to put Uriah in a dangerous battle position and then withdraw from him so that he would be
killed. When these orders were executed and Uriah was dead, David then took Bathsheba to
be his eighth wife.

David ruled Israel for 40 years (2 Sam. 5:4) as did also Saul (Acts 13:21), since 40 seemed to
be a good round number in those days. When he was "old and stricken in years," he was
covered with clothes but could not be warmed (1 Kings 1:1), and so his servants suggested

Volume 1990 - 2002 Issue


Page 932 of 2049
Skeptical Review Edited by Farrell Till
that a "young virgin" be brought to the king so that she could "cherish him" and "lie in [his]
bosom" to keep him warm. They then searched for "a fair damsel throughout all the borders
of Israel and found Abishag the Shunammite and brought her to David" (1:3). No explanation
was given for why the servants were so picky-picky and why the damsel had to be a fair one
if her duty was just going to be keeping the king warm, but, anyway, Abishag was selected;
she was "fair," and she cherished the king and ministered to him, but the king knew her not
(1:4). Nothing is said about whether the king wanted to know her but was prevented from
doing so by reason of his age and health. At any rate, this is how David, the man after
Yahweh's own heart, spent his final days. It's enough to make skeptics also yearn for a return
to the family values of biblical times.

Prophecy of Seventy Years of Servitude to


Babylon
Dr. James D. Price
This is the first Biblical prophecy that I present as fulfilled. The prophecy is contained in
Jeremiah 25:1-14. Before I begin, certain preliminary details are discussed. First, the
generally accepted definition of a genuine prophecy consists of at least three components: (1)
a prediction of an event with specific details that go beyond a general forecast; (2) an element
of the prediction, that goes beyond the prophet's natural ability to foresee, and/or a time delay
of that sort; (3) an event that fulfills the predicted details. The prophecy under discussion
contains both elements of (2)--historical elements that are beyond the prophet's natural ability
to foresee, and a specified time delay significantly beyond the prophet's lifetime and beyond
his natural ability to forecast.

Second are qualified authorities to validate the available evidence. In this instance, I use two
authorities--one a highly qualified secular historian, and one a highly qualified Biblical
historian. As the secular historian I use A. T. Olmstead the eminent Professor of Oriental
History at the Oriental Institute of the University of Chicago. Of his numerous publications he
has written the History of Palestine and Syria originally published by Charles Scribner's Sons
(1931), and the History of the Persian Empire published by the University of Chicago Press
(1948). These are the classic works cited by most other authors on the subject. Professor
Olmstead is not a fundamentalist or a biblicist but a world class secular historian.

Since some of the evidence comes from the Bible, I use John Bright, the Cyrus H.
McCormick Professor of Hebrew and Interpretation of the Old Testament at Union
Theological Seminary in Richmond, VA. Dr. Bright was trained in Union Theological
Seminary and Johns Hopkins University; he is not a fundamentalist, nor one who believes in
the inspiration and inerrancy of the Bible. In fact, he is liberal in his theological perspective
and skeptical of the supernatural elements in Scripture. He is a professional linguist and

Volume 1990 - 2002 Issue


Page 933 of 2049
Skeptical Review Edited by Farrell Till
historian. His publications include the Commentary on Jeremiah in the Anchor Bible series,
and A History of Israel, 2nd ed. (Philadelphia: Westminster Press, 1972).

No doubt Till will respond here that Olmstead and Bright do not accept the idea of fulfilled
prophecy, and thus I cannot use them in a discussion of prophecy. It is true that these
historians share Till's invalid presupposition against fulfilled prophecy. But I must advise the
audience that I do not use these historians for their theological views but for their expertise in
history. I use them to demonstrate that the dates and other historical evidence for the pertinent
events discussed here are regarded by reputable authorities as valid. These historians disagree
with me in the arena of theology, not in the arena of history. If Till wants to dispute the
validity of the historical evidence, then he stands against the authority of world class
historians, not merely against me.

The Date of the Prophecy: The date of the prophecy is given by the prophet himself at the
beginning of the passage. "The word that came to Jeremiah concerning all the people of
Judah, in the fourth year of Jehoiakim the son of Josiah, king of Judah (which was the first
year of Nebuchadnezzar king of Babylon), which Jeremiah the prophet spoke to all the people
of Judah and to all the inhabitants of Jerusalem, saying" (Jeremiah 25:1-2).

The prophet Jeremiah dated this prophecy in the fourth year of Jehoiakim which was the first
year of Nebuchadnezzar. According to John Bright (History, p. 325), this occurred in 605 BC,
the year Nebuchadnezzar's father died and Nebuchadnezzar ascended to the throne of
Babylon. That year, Nebuchadnezzar and the Babylonian army were engaged in the conquest
of the Palestinian area. The conquest was temporarily interrupted by the death of his father,
but the following year he returned and completed the conquest. Jehoiakim became the vassal
of Nebuchadnezzar, and although there were some times of short-lived rebellion, for all
practical purposes Judah was under the servitude of Nebuchadnezzar or the Babylonians for
seventy years, some of that time as captives exported from their native land to Babylon.

The Reason for the Prophecy: The reason for this prophecy of judgment is given in
Jeremiah 25:3-7. The people had refused to repent from their evil deeds and their idolatry
after a long period of God's patient and consistent warning by His prophets.

The Content of the Prophecy: The principal content of the prophecy is contained in
Jeremiah 25:8-14.

"Therefore thus says the LORD of hosts: `Because you have not heard My words, behold, I
will send and take all the families of the north,' says the LORD, `and Nebuchadnezzar the
king of Babylon, My servant, and will bring them against this land, against its inhabitants, and
against these nations all around, and will utterly destroy them, and make them an
astonishment, a hissing, and perpetual desolations. Moreover I will take from them the voice
of mirth and the voice of gladness, the voice of the bridegroom and the voice of the bride, the
sound of the millstones and the light of the lamp. And this whole land shall be a desolation
and an astonishment, and these nations shall serve the king of Babylon seventy years. Then it
will come to pass, when seventy years are completed, that I will punish the king of Babylon
and that nation, the land of the Chaldeans, for their iniquity,' says the LORD; `and I will make
it a perpetual desolation. So I will bring on that land all My words which I have pronounced

Volume 1990 - 2002 Issue


Page 934 of 2049
Skeptical Review Edited by Farrell Till
against it, all that is written in this book, which Jeremiah has prophesied concerning all the
nations. (For many nations and great kings shall be served by them also; and I will repay them
according to their deeds and according to the works of their own hands.)'"

Jeremiah 29:10 contains a more specific detail the prophet added about ten years later: "For
thus says the LORD: After seventy years are completed at Babylon, I will visit you and
perform My good word toward you, and cause you to return to this place."

The details of the prophecy consist of the following:

(1) Nebuchadnezzar and the Babylonian armies [the families of the north] would conquer the
entire territory, destroy the property, make the inhabitants an international disgrace, and take
them into captivity. The term "perpetual desolations" is understood to refer to the
international disgrace, because the promise of return to the land after seventy years indicates
that the word "perpetual" does not apply to the land but to the people's disgrace. The conquest
includes Judah and the surrounding nations.

(2) The Jews and the peoples of the surrounding nations would be in servitude to the King of
Babylon for seventy years. The period of seventy years is beyond Jeremiah's natural ability to
foresee or anticipate. It would extend far beyond his death, given his age at the time of the
prophecy. Thus the fulfillment of this detail would ordinarily be regarded as of supernatural
origin.

(3) At the end of the seventy years the Babylonian nation and its king would be judged and
the Jews would be restored to their lands. In all history known to Jeremiah, no nation taken
into captivity by the Assyrians, the Babylonians, or any of the preceding world powers had
ever been released to return to their native land. The foretelling of such an unprecedented
restoration would ordinarily be regarded as of supernatural origin, because it was something
that Jeremiah's natural experience could not anticipate.

The rest of chapter 25 contains a figurative elaboration of God's judgment of Judah and the
surrounding nations in poetic terms. Such figurative language is not to be interpreted beyond
the reasonable way Jeremiah's ancient readers would have understood it.

The Fulfillment of the Prophecy: The above significant details were fulfilled according to
the following evidence:

(1) Concerning the conquest of Judah and Jerusalem, Olmstead stated: "On the ninth day of
the fourth month of the eleventh year, 586, the walls were breached. Zedekiah and his men of
war fled that night by the gate between the two walls at the southeast corner near the king's
garden and the pool of Siloam; they hoped to reach the Arabah and so pass to the east Jordan
country, but were overtaken at Jericho. Zedekiah was carried to Nebuchadnezzar at Riblah,
where his sons were slain before his eyes and then he was blinded that his last sight might be
the end of his hopes of posterity" (History of Palestine, pp. 526-27). Bright provided added
details: "Having witnessed the execution of his sons, he [Zedekiah] was blinded and taken in
chains to Babylon, where he died (2 Kings 25:6f.; Jer. 52:9-11). A month later (2 Kings 25:8-
12; Jer. 52:12-16) Nebuzaradan, commander of Nebuchadnezzar's guard, arrived in Jerusalem

Volume 1990 - 2002 Issue


Page 935 of 2049
Skeptical Review Edited by Farrell Till
and, acting on orders, put the city to the torch and leveled its walls. Certain of the
ecclesiastical, military, and civilian officers, and leading citizens, were hauled before
Nebuchadnezzar at Riblah and executed (II Kings 25:18-21; Jer 52:24-27), while a further
group of the population was deported to Babylon. The state of Judah had ended forever"
(History, p. 330).

It should be noted that both of these professional historians regarded many historical details
recorded in 2 Kings and Jeremiah as valid. The conquest of Jerusalem is also verified by
archaeological evidence [see Kathleen M. Kenyon, Jerusalem (New York: McGraw-Hill
Book Company, Inc., 1967), pp. 78-104, 107f.]. Some details of the Babylonian invasion of
Judah are given in the Lachish Letters [see Albright in Prichard, Ancient Near Eastern Texts,
1f.]. Unfortunately, few Babylonian records for this era have survived. However, Josephus
described this event in great detail, deriving much of his information from the Bible, but also
from such ancient historians as Berosus, Menander, Megasthenes, Abydenus, and Alexander
Polyhistor. Few, if any, modern historians question Nebuchadnezzar's conquest and
destruction of Jerusalem and the surrounding territory.

(2) Concerning the seventy years, the interval is calculated from the date of the prophecy (605
BC) to the date of Cyrus' decree (536 BC). This may seem to be only sixty-nine years from
our western perspective, and thus not an exact fulfillment. However, the number seventy is a
round number that is sufficient for the facts. According to the way the ancient Jews reckoned
time, and according to the calendar then in use, the first event may have occurred late in the
first year and the second event early in the seventieth year, which would appear to span only
sixty-nine years according to western convention. Further, as much as a year may have
elapsed from the time the decree was issued until the time the Jews actually arrived in Judah.
We do know that the contemporary Jews regarded the interval to be seventy years, and to be a
fulfillment of Jeremiah's prophecy (2 Chron 36:21-23; Ezra 1:1-4).

Others have regarded the seventy years to begin with the destruction of Jerusalem and the
temple in 586 BC and to conclude with the completion of the second temple in 516 BC.
Evidently some of the Jews in the days of the completion of the second temple regarded
Jeremiah's seventy years to be fulfilled then (Zech. 1:2; 7:5). In either case the interval of
seventy years was fulfilled.

Now Till may want to quibble over the precision of the dates, perhaps citing some historian
with differing figures. The truth is that these ancient dates cannot be established with rigorous
precision; the various authorities differ by a year or two. However, the intervening interval
remains approximately seventy years, sufficient to fulfill the requirements of a round number.

Further, Till may assert, as some radical critics have done, that a later scribe altered
Jeremiah's prophecy by changing his original number to the number seventy in order to make
Jeremiah's prophecy appear fulfilled. If so, he will be making an assertion for which there is
absolutely no objective evidence. Such an assertion is not based on objective evidence but
solely on the presupposition that long-range, specific prophecy is impossible, a presupposition
that I have previously demonstrated is inadmissible in this current debate. It involves
assuming the conclusion, begging the question, arguing in a circle. It is not sufficient for him
to cite critical scholars, who share his faulty theological presupposition. If Till wants to make

Volume 1990 - 2002 Issue


Page 936 of 2049
Skeptical Review Edited by Farrell Till
this assertion, then let him first produce convincing objective evidence that such a forgery
took place. Otherwise, let him admit by default that the number is authentic.

(3) Concerning the restoration of the Jews and the surrounding nations to their native lands,
an ancient Persian record known as the Cyrus Cylinder, dated about 536 BC, and composed
by Cyrus himself, records the fact that Cyrus granted release to the nations taken captive by
Nebuchadnezzar, and granted them permission to return to their native land and restore their
temple worship. It states in part:

"I returned to (these) sacred cities on the other side of the Tigris, the sanctuaries of which
have been in ruins for a long time, the images which (used) to live therein and established for
them permanent sanctuaries. I (also) gathered all their (former) inhabitants and returned (to
them) their habitations.... May all the gods whom I have resettled in their sacred cities ask
daily Bel and Nebo for a long life for me and may they recommend me (to him)... All of them
I settled in a peaceful place.... I endeavored to fortify/repair their dwelling places" (Source:
James B. Prichard, ed., Ancient Near Eastern Texts Relating to the Old Testament, 2nd. ed.
[Princeton: Princeton University Press, 1955], p. 316; translated by A. Leo Oppenheim).

The general decree of Cyrus was followed by a specific decree for the Jews (and perhaps
specific decrees for the other nations). Various parts of this specific decree are recorded in
four places in the Bible (2 Chron 36:23; Ezra 1:2-4; 5:14-15; 6:3-5). Two of these sources
(Ezra 5:14-15; 6:3-5) are written in Aramaic, the language in which the original decree would
have been written. Bright stated that "the authenticity of which need not be questioned"
(History, p. 361). The other two (2 Chron 36:23; Ezra 1:2-4) are in Hebrew and thus are
translations. Concerning the Ezra record, Bright asserted that "it contains no intrinsic
improbability that might cast doubt upon its essential historicity" (History, p. 362).

Both Olmstead and Bright (and almost all other historians of that era) record the fact that a
large company of Jews returned from the Babylonian captivity and rebuilt their temple. The
contents of the Cyrus Cylinder indicate that Cyrus let all the nations west of the Tigris river
return to their native lands. Thus the citizens of those lands must have been taken captive by
the Babylonians, and they, like the Jews, must have returned to their native lands in
fulfillment of Jeremiah's prophecy. There is no reasonable evidence to doubt this fact.

Till will respond by quibbling over some minute details that are irrelevant to the main issue--
the real fulfillment of the central details of one specific prophecy. He will raise potential
difficulties for which he has no objective evidence. Before he questions the accuracy of any
detail, let him produce objective evidence to the contrary; otherwise, let him accept the
validity of my evidence by default. He will again (!) raise the issue of errancy; but the current
question is the fulfillment of this one specific prophecy, not the errancy of Scripture or the
validity of any other prophecy. Let him stick to the subject at hand.

Conclusion: I believe I have presented common sense, rational, objective, historical evidence
that Jeremiah's prophecy of the seventy year captivity and restoration of the Jews was indeed
fulfilled. The evidence is sufficient for rational people to accept the prediction as a genuine
prophecy that has been fulfilled in its pertinent details. Only radical skeptics who insist on the

Volume 1990 - 2002 Issue


Page 937 of 2049
Skeptical Review Edited by Farrell Till
validity of their theological presuppositions in spite of objective evidence will deny this
conclusion.

(James D. Price, Ph. D., Professor of Hebrew and old Testament, Temple Baptist Seminary,
Chattanooga, TN 37404; e-mail drjdprice@aol.com)

A Bad Example of Prophecy Fulfillment


Farrell Till
I appreciate the opportunity to reply to Dr. Price's claim of prophecy fulfillment that he
believes he has found in the book of Jeremiah. Dr. Price and I were brought together in this
discussion at the request of two subscribers to The Skeptical Review, who asked me to try to
arrange a debate with Dr. Price on the subject of prophecy fulfillment. Upon receiving my
proposal, Dr. Price informed me that he is not a public debater, but he indicated a willingness
to debate in writing. The article above is Dr. Price's effort to prove the reality of biblical
prophecy fulfillment. My reply is being published simultaneously, after which Dr. Price may
respond if he wishes, and I will reply again if deemed necessary. The exchanges will continue
until we both mutually agree that the subject has been exhausted.

To say that I was delighted when I received Dr. Price's article would be an understatement,
because, with the possible exception of some prophecies in the book of Ezekiel that failed
miserably, he could not have selected a worse example than the one he has chosen to defend. I
suspect that Dr. Price is aware of the weaknesses in his defense, because throughout his
article, he made references to points that I would probably "quibble over" and then said just
before his conclusion, "Till will respond by quibbling over some minute details that are
irrelevant to the main issue--the fulfillment of the central details of one specific prophecy."

He is absolutely right in assuming that I will raise questions about some of the details in
Jeremiah's 70-year prophecy and his defense of it, but I will leave it to the readers to decide if
these are just "quibbles." If Dr. Price is going to claim prophecy fulfillment, then he is
obligated to defend all details of the alleged prophecy and not just those that he considers
"central," for if Jeremiah was indeed inspired by an omniscient, omnipotent deity, we have
every right to expect fulfillment in ALL details and not just those that Dr. Price considers
"central." He has predicted that I will quibble in my response, but he himself has already
resorted to a familiar inerrantist quibble, by implying that some biblical details are more
important than others and that we need to be concerned only with the details that are "central."
This is the approach that some apologists use in defending the maze of contradictions in the
resurrection narratives. They argue that if there is disagreement in some details in the
narratives there is nevertheless agreement in one central detail, which is that Jesus rose from
the dead. On the basis of this quibble, many inerrantists claim that the resurrection narratives
can be trusted when they say that Jesus rose from the dead, no matter how inconsistent they
may be in other details.

Volume 1990 - 2002 Issue


Page 938 of 2049
Skeptical Review Edited by Farrell Till
I consistently find that inerrantists accuse skeptics of quibbling over minor details when they
are confronted with problems in their inerrancy defenses for which they have no sensible
explanations, but apparently can't see their own quibble, not to mention inconsistency, when
they try to rank biblical discrepancies in terms of importance, for if the Bible is inerrant, then
it can have no mistakes of ANY kind, no matter how trivial they may seem. In other words,
there is no such thing as "central details" in an inerrancy debate. They are all central and must
all be reconcilable, or else the Bible is not inerrant. Anyone who disputes this should check a
dictionary for the definition of inerrancy.

At the beginning of this exchange, I want to assure Dr. Price that I will not sit idly by and
tolerate this smorgasbord approach to apologetics in which the defender of the Bible chooses
details that he considers "central" and disregards those that are damaging to his position.
Inerrantists claim that the Bible is the inspired, inerrant word of an omniscient, omnipotent
deity, and if that claim is true, there is no such thing as a trivial inconsistency. Inerrantists
must defend EVERYTHING and not just the details that they consider "central."

With my position clarified, I will begin my rebuttal of Dr. Price's article by citing first the
criteria of valid prophecy fulfillment that were first published in my response to Dr. Hugh
Ross's article on biblical prophecy fulfillments ("Prophecy Fulfillment: An Unprovable
Claim," TSR, January/February 1996, pp. 3-6). Here they are as originally published:

In order to prove--and I mean PROVE, not just surmise--prophecy fulfillment, one would
have to establish four things: (1) the claimant of a prophecy fulfillment is properly
interpreting whatever text he is basing his claim on, (2) the prophecy was made BEFORE and
not after the event that allegedly fulfills the prophecy, (3) the prophecy was made not just
BEFORE an event but far enough in advance of it to make educated guesswork impossible,
and (4) he event that allegedly fulfilled the prophecy did in fact happen.

As was true in the case of Dr. Ross's prophecy fulfillment claims, we will see that Dr. Price's
example also cannot past this test for valid prophecy fulfillment.

THE DATE OF THE "PROPHECY": The most serious problem for Dr. Price is the second of
these criteria. He must prove that this "prophecy" was made BEFORE its "fulfillment" and
was not just retrojection or editing by a scribe or redactor who, after Jeremiah had been long
dead, altered the text of this book to make it appear that he had made an amazing prediction of
future events. The mere suggestion that such could have happened to a book revered by
millions of Christians is shocking heresy to Dr. Price, but the evidence for widespread textual
editing in the Bible and especially the book of Jeremiah is so overwhelming that no
intellectually honest person can deny it.

Before I present the evidence to support my claim that the biblical text has been repeatedly
edited, let's look first at a very simple rebuttal of Dr. Price's claim of prophecy-fulfillment in
Jeremiah 25. Most adherents of religions that have holy books cite prophecy fulfillments as
proof of the divine origin of their particular books. Mormons, for example, claim that
amazing prophecy fulfillments in the Book of Mormon are undeniable evidence that it is
another testament of Jesus Christ, inspired of God just as the Bible was. Some examples of
Mormon "prophecy fulfillment" can be found in the books of Nephi, who was allegedly the

Volume 1990 - 2002 Issue


Page 939 of 2049
Skeptical Review Edited by Farrell Till
son of a man named Lehi, who during the reign of Zedekiah, the rebellious puppet king whom
Nebuchadnezzar put on the throne of Judah, was warned to take his family and flee into the
wilderness by the Red Sea, from where Lehi's descendants later migrated to a new "promised
land" on a ship they built. Lehi's son Nephi saw visions of a "book" that would go out among
the Gentiles (1 Nephi 13:19-20), and in 2 Nephi 27 he made a lengthy prophecy about the
"discovery" of this book:

And it shall come to pass that the Lord God shall bring forth unto you the words of a book,
and they shall be the words of them which have slumbered. And behold the book shall be
sealed; and in the book shall be a revelation from God, from the beginning of the world to the
ending thereof. Wherefore, because of the things which are sealed up, the things which are
sealed shall not be delivered in the day of the wickedness and abominations of the people.
Wherefore the book shall be kept from them. But the book shall be delivered unto A MAN,
and he shall deliver the words of the book, which are the words of those who have slumbered
in the dust, and he shall deliver these words unto another; but the words which are sealed he
shall not deliver, neither shall he deliver the book. For the book shall be sealed by the power
of God, and the revelation which was sealed shall be kept in the book until the own due time
of the Lord, that they may come forth; for behold, they reveal all things from the foundation
of the world unto the end thereof.

And the day cometh that the words of the book which were sealed shall be read upon the
house tops; and they shall be read by the power of Christ; and all things shall be revealed unto
the children of men which ever have been among the children of men, and which ever will be
even unto the end of the earth. Wherefore, at that day when the book shall be delivered unto
THE MAN of whom I have spoken, the book shall be hid from the eyes of the world, that the
eyes of none shall behold it save it be that three witnesses shall behold it, by the power of
God, besides him to whom the book shall be delivered; and they shall testify to the truth of
the book and the things therein. And there is none other which shall view it, save it be a few
according to the will of God, to bear testimony of his word unto the children of men; for the
Lord God hath said that the words of the faithful should speak as if it were from the dead.

Wherefore, the Lord God will proceed to bring forth the words of the book; and in the mouth
of as many witnesses as seemeth him good will he establish his word; and wo[e] be unto him
that rejecteth the word of God! But behold, it shall come to pass the Lord God shall say unto
him to whom he shall deliver the book: Take these words which are not sealed and deliver
them to another, that he may show them unto the learned, saying: Read this, I pray thee. And
the learned shall say: Bring hither the book, and I will read them.

And now, because of the glory of the world and to get gain will they say this, and not for the
glory of God. And the man shall say: I cannot bring the book for it is sealed. Then shall the
learned say: I cannot read it. Wherefore it shall come to pass that the Lord God will deliver
again the book and the words therefore to him that is not learned; and the man that is not
learned shall say: I am not learned. Then shall the Lord God say unto him: The learned shall
not read them, for they have rejected them, and I am able to do mine own work; wherefore
thou shalt read the words which I shall give unto thee. Touch not the things which are sealed,
for I will bring them forth in mine own due time; for I will show unto the children of men that
I am able to do mine own work.

Volume 1990 - 2002 Issue


Page 940 of 2049
Skeptical Review Edited by Farrell Till
Wherefore, when thou hast read the words which I have commanded thee, and obtained the
witnesses which I have promised unto thee, then shalt thou seal up the book again, and hide it
up unto me, that I may preserve the words which thou has not read, until I shall see fit in mine
own wisdom to reveal all things unto the children of men (vs:6-22, emphasis added).

I apologize for this lengthy quotation, but it was necessary to present the full context in order
to show obvious reasons why Mormons see this as a clear prophecy of the coming of their
holy book. This "prophecy" speaks of a book that would be delivered to "a man" but hidden to
the word except for "three witnesses," who would testify to the truth of the book, and no one
else save it be a few according to the will of God, who would also "bear testimony of his word
unto the children of men." Every Book of Mormon is prefaced with the testimony of the three
witnesses, who swear that they saw with their own eyes a book of golden plates delivered to
Joseph Smith, and the eight witnesses, who swear that they handled the plates with their own
hands. So this prophecy is far more explicit than any biblical prophecy that Dr. Price can cite,
but does he believe that this is in any sense a genuine prophecy fulfillment? I seriously doubt
it, because he is intelligent enough to see through a prophecy claim as transparent as this one.
Any rational person not blinded by Mormon indoctrination can see that it is far more likely
that the author of the Book of Mormon purposely wrote this passage to give it an appearance
of prophecy fulfillment than that a man who lived 2,500 years ago actually predicted the
coming of the Book of Mormon. As I will show, this is the problem that faces Dr. Price. He
must prove beyond reasonable doubt that Jeremiah's prophecy was written seventy years
before its "fulfillment" and not altered after the fact to make it appear that Yahweh had
enabled Jeremiah to see far into the future. I predict that he will not be able to do that.

That some Old Testament writers resorted to prophetic retrojection just as the author of the
Book of Mormon did can be seen by just common-sense interpretation of the alleged
prophecies. A prime example would be the prophecy of Josiah's religious reformation. In this
fanciful little tale, a "man of God" came from Judah to confront Jeroboam at the dedication of
his pagan altar at Bethel. As Jeroboam was standing at the altar to burn incense, the man of
God cried out, "O altar, altar, thus says Yahweh, Behold, a son shall be born to the house of
David, Josiah by name, and upon you will he sacrifice the priests of the high places that burn
incense on you, and men's bones will they burn on you" (v:2). This "prophecy" was allegedly
spoken around 920 B. C. in the early years of Jeroboam, and, sure enough, a king from the
house of David, who ruled Judah from about 640 to 609 B. C. did the very things that the
"man of God" had predicted almost three centuries earlier. Josiah's "fulfillment" is recorded in
2 Kings 23:17-19.

Is anyone impressed? If so he/she must be very naive, because anyone with common sense
should be able to see that the author(s) of the books of Kings or some scribe editing them
could have very easily retrojected the "man of God's" prediction of Josiah's reforms into the
text to make it appear that an amazing prophecy had been made and fulfilled, or it is entirely
possible that a writer just made up the whole story (both the prophecy and the fulfillment).
Either alternative is far more likely than the simplistic assumption of biblical inerrantists that
Yahweh enabled this unnamed "man of God" to see three centuries into the future and predict
exact details like these.

Volume 1990 - 2002 Issue


Page 941 of 2049
Skeptical Review Edited by Farrell Till
Another example much like this one can be found in Joshua 6:28, where Joshua pronounced a
curse upon anyone who would rebuild Jericho: "Cursed be the man before Yahweh who rises
up and builds this city Jericho. With the loss of his firstborn will he lay the foundation
thereof, and with the loss of his youngest son will he set up the gates of it." Well, guess what?
That's right. Centuries later, a man named Hiel the Bethelite rebuilt Jericho, and "laid the
foundation with the loss of Abiram his firstborn, and set up the gates thereof with the loss of
his youngest son Segub, according to the word of Yahweh, which he spoke by Joshua the son
of Nun." It's in the book (1 Kings 16:34).

Impressive? Hardly, except to people who are gullible enough to believe about anything. With
no extrabiblical records from contemporary times to corroborate tales like these, it is more
likely that such stories as these were written to give the appearance of prophecy fulfillment
than that the prophecies were spoken and fulfilled exactly as claimed. In my personal
correspondence with Dr. Price, I learned that he doesn't like to confront this type of rebuttal
argument, which he calls an "anti-supernatural bias," and even tried to establish a ground rule
that would prohibit my using it. I rejected his demand, because I consider it a legitimate
response to miracle claims that have nothing to support them but the biased testimony of
biblical writers, who lived in superstitious times and were very deeply committed emotionally
to belief in their miracle-working god. It is an argument that he cannot escape from in a
discussion like this. Hence, Dr. Price must present sensible reasons why anyone should
believe in biblical miracle claims any more than the miracle claims in holy books of other
religions.

THE SEPTUAGINT PROBLEM: A major thorn in the flesh of biblical inerrantists is the
many variations in the Septuagint and Masoretic texts of the Old Testament. The Septuagint, a
Greek version of the Old Testament that was translated in the 3rd century B. C., has more
than 6,000 variations from the Masoretic text, which most English versions were derived
from. The oldest complete version of the Masoretic text dates from the late 9th century A. D.,
so there is a span of about 12 centuries between these two versions. This many variations is a
rather clear indication that substantial altering of the Hebrew text occurred between the 3rd
century B. C. and the 9th century A. D. Admittedly, many of these variations are minor, but
some of them aren't so minor. This can best be illustrated by comparing the two versions of a
specific passage. First Samuel 17 (1 Kings 17 in the Septuagint) records the quaint story of
David's battle with the Philistine giant Goliath. Verses 12 through 31 in the Masoretic text and
consequently most English versions, which give genealogical information about David and
the circumstances that caused him to be present when Goliath was challenging the Israelites,
are missing from the Septuagint, an indication that the Hebrew text did not contain them when
the Septuagint was translated. Jeremiah 27:19-22; 33:14-26; 39:3-14, and 48:45-47 in the
Masoretic text are missing completely from the Septuagint, and, as we will see, the Septuagint
did not have some of the very statements in chapter 25 on which Dr. Price is basing his
argument for biblical prophecy fulfillment. In the Septuagint, chapter 32 is chapter 25:15-18
of the Masoretic version, chapter 34 is 27:1-19 of the Masoretic, chapter 40 is 33:1-14 of the
Masoretic, and so on through more than 30 other changes in organization. In the face of such
variations and omissions as these, who can honestly deny that substantial tampering with the
text of Jeremiah was done after the Septuagint was completed?

Volume 1990 - 2002 Issue


Page 942 of 2049
Skeptical Review Edited by Farrell Till
Biblical "apologists" often argue that scribes took meticulous care in copying biblical
manuscripts. These apologists tell tales about scribes who counted letters in their copies to
make sure that they had made no errors in transcribing the text, but the actual evidence
disputes this popular tale that preachers tell to credulous pulpit audiences. In cave four at
Qumran, famous for the Dead Sea Scrolls discovered there, excavators have found fragments
of Jeremiah that match the Septuagint version of this book. Since scholars have dated these
fragments at some time in the second century B. C., here is evidence that the book of
Jeremiah at that time was substantially the same as what the Septuagint translators had used.
Writing on this subject, Joseph A. Fitzmyer, professor emeritus of New Testament at the
Catholic University of America said this about these discoveries at Qumran:

In some cases, especially 1-2 Samuel, Jeremiah, and Exodus, the fragments brought to light
forms or recensions of biblical books that differed from the medieval Masoretic tradition. For
instance, one text turned out to be a shorter form of Jeremiah, previously known only in its
Greek version in the Septuagint. It now seems that the fuller form of Masoretic tradition
represents a Palestinian rewording of the book ("The Dead Sea Scrolls and the Bible: After
Forty Years," America, October 31, 1987, p. 302, emphasis added).

Information like this is not at all friendly to Dr. Price's claim that he has found an amazing
case of prophecy fulfillment in the book of Jeremiah, because if this book has been revised,
edited, and tampered with as much as the evidence indicates, no reasonable person can have
confidence in the integrity of the book.

The best way to show the extent to which the Hebrew text of Jeremiah has been compromised
would be to compare with the Septuagint version the Masoretic passage in which Dr. Price
thinks he has found a prophecy that was made 70 years before its fulfillment. Here is the
Septuagint version of Jeremiah 25:1-14 in the Masoretic version, where Dr. Price claims he
has found an amazing prophecy. The citation has been taken from Brenton's translation of the
Greek text:

The word that came to Jeremias concerning all the people of Juda in the fourth year of Joakin,
son of Josias, king of Juda; which he spoke to all the people of Judah, and to the inhabitants
of Jerusalem, saying, In the thirteenth year of Josias, son of Amos, king of Juda, even until
this day for three and twenty years, I have both spoken to you, rising early and speaking, and I
sent to you my servants the prophets, sending them early; (but ye hearkened not, and listened
not with your ears;) saying, Turn ye everyone from his evil way, and from your evil practices,
and ye shall dwell in the land which I gave to you and your fathers, of old and for ever. Go ye
not after strange gods, to serve them, and to worship them, that ye provoke me not by the
works of your hands, to do you hurt. But ye hearkened not to me.

Therefore, thus saith the Lord; Since ye believed not my words, behold, I will send and take a
family from the north, and will bring them against this land, and against the inhabitants of it,
and against all the nations round about it, and I will make them utterly waste, and make them
a desolation, and a hissing, and an everlasting reproach. And I will destroy from among them
the voice of joy, and the voice of gladness, the voice of the bridegroom, and the voice of the
bride, the scent of ointment, and the light of a candle. And all the land shall be a desolation;
and they shall serve among the Gentiles seventy years.

Volume 1990 - 2002 Issue


Page 943 of 2049
Skeptical Review Edited by Farrell Till
And when the seventy years are fulfilled, I will take vengeance on that nation, and will make
them a perpetual desolation. And I will bring upon that land all my words which I have
spoken against it, even all things that are written in this book.

Actually, this is only the quotation of 25:1-13, because the Septuagint has no verse 14. It is
just one of the many passages missing from the Septuagint that are in the Masoretic. For
convenience in comparing the two version, I will quote in its entirety the Masoretic-dependent
version that Dr. Price cited in his article. The sections enclosed in asterisks (*) are either
different from or missing in the Septuagint.

The word that came to Jeremiah concerning all the people of Judah, in the fourth year of
Jehoiakin the son of Josiah, king of Judah (*which was the first year of Nebuchadnezzar king
of Babylon*), which Jeremiah the prophet spoke to all the people of Judah and to all the
inhabitants of Jerusalem saying: "From the thirteenth year of Josiah even to this day, this is
the twenty-third year in which the word of the Lord has come to me; and I have spoken to
you, rising early and speaking, but you have not listened. And the Lord has sent to you all his
servants the prophets, rising early and sending them, but you have not listened nor inclined
your ear to hear. They said, `Repent now everyone of his evil way and his evil doings, and
dwell in the land that the Lord has given to you and your fathers forever and ever. Do not go
after other gods to serve them and worship them, and do not provoke me to anger with the
works of your hands; and I will not harm you.' Yet you have not listened to me," says the
Lord, "that you might provoke me to anger with the works of your hands to your own hurt.

"Therefore thus says the LORD of hosts: `Because you have not heard My words, behold, I
will send and take *all the families* of the north,' says the LORD, `*and Nebuchadnezzar the
king of Babylon, My servant,* and will bring them against this land, against its inhabitants,
and against these nations all around, and will utterly destroy them, and make them an
astonishment, a hissing, and perpetual desolations. Moreover I will take from them the voice
of mirth and the voice of gladness, the voice of the bridegroom and the voice of the bride, the
sound of the millstones and the light of the lamp. And this whole land shall be a desolation
and an astonishment, and these nations shall serve *the king of Babylon* seventy years. Then
it will come to pass, when seventy years are completed, that I will punish *the king of
Babylon and* that nation, *the land of the Chaldeans, for their iniquity,' says the LORD;*
`and I will make it a perpetual desolation. So I will bring on that land all my words which I
have pronounced against it, all that is written in this book, *which Jeremiah has prophesied
concerning all the nations. (For many nations and great kings shall be served by them also;
and I will repay them according to their deeds and according to the works of their own
hands.)*'"

One thing immediately obvious in comparing these two versions is that Nebuchadnezzar and
the nation of Babylon were not even mentioned in the 3rd-century B. C. Septuagint version.
Where the 9th-century Masoretic text has Jeremiah dating this prophecy in the fourth year of
Jehoiakim and the "first year of Nebuchadnezzar, king of Babylon," the Septuagint made no
mention of Nebuchadnezzar and put the date of the prophecy only at the "fourth year of
Joakin [Jehoiakin]" (v:1), and in verse 8, where the Masoretic has Jeremiah predicting that
Yahweh would send "all the families of the north" and "Nebuchadnezzar, the king of
Babylon" against Judah and its inhabitants and "utterly destroy them," the Septuagint said

Volume 1990 - 2002 Issue


Page 944 of 2049
Skeptical Review Edited by Farrell Till
only that Yahweh would send "a family from the north" against Judah. The Masoretic has
Jeremiah predicting that the people of Judah would "serve the king of Babylon seventy years,"
but the Septuagint said only that they would "serve among the Gentiles seventy years." These
variations indicate that when the Septuagint was completed in the 3rd century B. C., the
translators were working with a Hebrew text that didn't even mention Nebuchadnezzar in this
prophecy. Between then and the 12 centuries that separate this version from the earliest
Masoretic text, some editor(s) evidently altered this passage to make it appear that Jeremiah
had made a specific prediction about Nebuchadnezzar's part in the conquest and captivity of
Judah.

Nebuchadnezzar was mentioned by name in the Septuagint but not nearly as much as in the
Masoretic. That some post-Jeremiah editor(s) had an intense interest in increasing
Nebuchadnezzar in this book is evident when other Masoretic passages are compared to the
Septuagint:

MASORETIC 21:7, "And afterward says Yahweh, I will deliver Zedekiah king of Judah, and
the servants, and the people, even such as are left in this city from the pestilence, from the
sword, and from the famine, into the hand of Nebuchadnezzar king of Babylon, and into the
hand of their enemies, and into the hand of those that seek their life: and HE shall smite them
with the edge of the sword; HE shall not spare them, neither have pity, nor have mercy."

SEPTUAGINT 21:7, "And after this, thus saith the Lord; I will give Sedekias king of Juda,
and his servants, and the people that is left in this city from the pestilence, and from the
famine, and from the sword, into the hands of their enemies, that seek their lives; and THEY
shall cut them in pieces with the edge of the sword: I will not spare them, and I will not have
compassion upon them."

Comparison of the places emphasized in italics and bold print shows where the text from
which the Septuagint was translated was later altered in order to inject Nebuchadnezzar into
Jeremiah's prophecy. In the 3rd century B. C., the Hebrew text said only that the Lord would
give Sedekias {Zedekiah] and the people of Jerusalem into the hands of their enemies, but
somewhere between then and the 9th century A. D., the text was altered to make
Nebuchadnezzar the specific enemy into which Judah would be delivered. In the 3rd century
B. C., this verse read that Yahweh, speaking in the first person, would not spare them or have
compassion on them, but 12 centuries later the text had been changed to a third-person
narrative that had Nebuchadnezzar specified as the one who would not spare them or have
mercy on them.

There are many passages in the Septuagint version of this book where Jeremiah referred only
to "the king of Babylon," but many of them in the 9th century A. D. Masoretic text had been
altered to refer specifically to Nebuchadnezzar by name, so obviously some post-Jeremiah
editor(s) rewrote this book to increase the role that the prophet gave to Nebuchadnezzar. With
evidence like this staring Dr. Price in the face, it is inconceivable that he would argue that
people today can be assured that Jeremiah 25:8-14 is a prophecy that was written before the
fact and afterwards fulfilled exactly as Jeremiah had written it. The text of this book has been
compromised far too much for rational people to buy this argument. Hence, there is no way
that Dr. Price can show that this prophecy satisfies the second criterion of valid prophecy.

Volume 1990 - 2002 Issue


Page 945 of 2049
Skeptical Review Edited by Farrell Till
THE SEVENTY YEARS: The evidence does suggest that the 3rd century B. C. Hebrew text
contained a seventy-year prophecy, because the Septuagint said that the people of Judah
would "serve among the Gentiles seventy years," but it isn't chronologically possible to
establish a seventy-year exile for the Judeans who were taken into captivity during the reign
of Nebuchadnezzar. Both the Bible and the Babylonian Chronicles recorded the captivity, and
even with the most generous interpretation of these records, the captivity cannot be stretched
beyond 67 years, and a maximum of 60 years is a more likely interpretation. The Babylonian
Chronicles, written on cuniform tablets, are in the British Museum, and the Jewish account of
Judah's defeat and subsequent captivity is, of course, recorded in the Bible (2 Kings 24-25; 2
Chron. 36). The chronological information in the biblical text is tedious to wade through, but
the following quotation from Eerdmans Bible Dictionary ("Exile," 1987, pp. 361-362)
presents an easy to follow summation of the biblical record:

DEPORTATION OF JUDAH: Far more significant [than the deportation of the Northern
Kingdom] was the exile of Judah, the southern kingdom (the tribes of Judah and Benjamin).
Because they had been able to preserve a margin of autonomy following Sennacherib's siege
of Jerusalem in 701 (2 Kings 18:13-19:37 par[allel] Isa. 36-37), the people of Judah believed
that they could regain their freedom from the crumbling Assyrian Empire, in dissolution since
the fall of Nineveh in 612. But Judah had to face another powerful foe, the Neo-Babylonian
Empire, whose capable leader, Nebuchadnezzar II, quickly defeated Judah's ally and overlord,
Egypt, at Carchemish in 605.

According to 2 Kings 24, Nebuchadnezzar laid siege to Jerusalem in 598 and, following
capture of the city, exiled to Babylon King Jehoiachin, his family, the nobles, a large number
of valiant soldiers (a smaller number is given at Jer. 52:28), and craftsmen. Nebuchadnezzar
also seized the temple treasure as booty (2 Kgs. 24:11-16; 2 Chr. 36:6; Dan. 1:1-2). The
Babylonian monarch made Zedekiah his vassal in Jerusalem, and when Zedekiah refused to
pay tribute, Nebuchadnezzar returned to Judah in 587, besieged the city again, and finally
leveled it. He took the remaining Jewish rebels, except for the very poorest, to his capital (2
Kgs. 24:20-25:17; 2 Chr. 36:15-21; Jer. 52:3-16). A third deportation took place ca. 581
following the murder of Gedaliah, whom Nebuchadnezzar had appointed governor over Judah
(2 Kgs. 25:22-26; Jer. 41:2-3). (Meanwhile a sizable number of Hebrews had taken refuge in
Egypt [Jer. 43:5-7].)

Unlike their northern counterparts, the people of the southern kingdom RETURNED IN 538
when Cyrus the Achaemenid, who had conquered Babylon the preceding year, issued an edict
that anyone who wished to assist in building a sanctuary for Yahweh in Jerusalem could go
there (2 Chr. 36:22-23).

Dr. Price tried his best to get 70 years by dating the captivity from 605 B.C. and the issuance
of Cyrus's decree in 536 B.C., but even then he could get only 69 years, which he said I would
probably quibble about (an indication that he was well aware of a problem in the way he had
calculated the beginning of the captivity). As stated in Eerdmans summation above,
Nebuchadnezzar first captured Jerusalem in 598 B.C., at which time he took King Jehoiachin,
his family, and some nobles and soldiers back to Babylon, but it wasn't until his destruction of
Jerusalem in 587 B.C. that he took captive "the residue of the people" who had been left in the
city and "the residue of the multitude" (2 Kings 25:10). So even if we accept Price's dating of

Volume 1990 - 2002 Issue


Page 946 of 2049
Skeptical Review Edited by Farrell Till
Cyrus's decree at 536 B.C., this would leave only a maximum of 62 years of captivity (598
minus 536 = 62). Dr. Price tried to sneak 605 B.C. by us as the beginning of the captivity, but
Jeremiah said only that the word of Yahweh, through which this "prophecy" had presumably
been revealed, had come to him in the "first year of Nebuchadnezzar"; he did not say that the
captivity would begin that same year. Nebuchadnezzar became king in 605, but it wasn't until
598, seven years later that he made Judah a vassal state and took some captives to Babylon, so
this is when the exile actually began.

Dr. Price claims that the Babylonian Exile ended for the Jews in 563 B. C., but the
Babylonian Chronicles date Cyrus's conquest of Babylon at what would have been October
16, 539 B. C. (New Bible Dictionary, Inter-Varsity Press, 1994, p. 258). Second Chronicles
36:22 states that Cyrus issued his decree in the first year of his reign, so if Dr. Price's inspired
word of God is historically accurate, the Jewish exiles were given their freedom to return to
Judah in 539 B. C. Eerdmans dates the return to Jerusalem of a "great number" of exiles at
538 B. C. ("Dispersion," p. 286), as it also did in the quotation above. If this date is accepted
as the end of the exile, its maximum duration could have been only 60 years. This is hardly a
minor discrepancy, and one would think that a prophet speaking by the inspiration of an
omniscient, omnipotent deity could have been more exact.

Dr. Price, of course, knows the precarious position he is in on this matter, because after taking
us through calculations beginning with the date the prophecy was allegedly written and
ending with his 536 B. C. date (in order to get 69 years so that he could claim the prophecy
"fulfill[ed] the requirement of around number"), he then went on to say that some "have
regarded the seventy years to begin with the destruction of Jerusalem and the temple in 586 B.
C. and to conclude with the completion of the second temple in 516 B. C." In other words, Dr.
Price seems to be arguing that if his first interpretation of the prophecy is unsatisfactory,
maybe the second one will be more palatable so that "(i)n either case the interval of seventy
years was fulfilled" (p. 3, this issue). So which was it? Did the captivity begin in 605 B. C.,
when Nebuchadnezzar came to power, and end in 536 B. C. for a length of 69 years that can
be rounded off to 70, or did it begin in 586 B. C. and end in 516 B. C., even though waves of
exiles had returned to Jerusalem more than 20 years before the second temple was completed?
Dr. Price seems to regard Jeremiah 25:8-14 as an amazing example of prophecy fulfillment,
but after taking us on a long and winding road to arrive at a strained interpretation of a 69-
rounded-off-to-70-year captivity, he ends up telling us that this may not be the right
interpretation of the prophecy after all. Maybe Jeremiah meant that the captivity would begin
19 years after Price's first date and end 20 years after the first exiles had actually returned
from Babylon. As if this were not wishy-washy enough, after stating his alternative
interpretation, Dr. Price went on to say, "The truth is that these ancient dates cannot be
established with rigorous precision." Well, pardon me, but if the dates cannot be established
with rigorous precision, how can Dr. Price claim that even the "central detail" of the prophecy
(the 70 years) was fulfilled. What he seems to be saying is that he knows that Jeremiah's 70-
year prophecy was fulfilled even though the dates of the fulfillment "cannot be established
with rigorous precision." Needless to say, I'm not impressed, and I suspect that critical-
thinking readers won't be either.

THE "FIGURATIVE ELABORATION OF GOD'S JUDGMENT: In the same context with


Jeremiah's 70-year prophecy, there are some other predictions (vs:15-38) that by no stretch of

Volume 1990 - 2002 Issue


Page 947 of 2049
Skeptical Review Edited by Farrell Till
the imagination anyone can claim were fulfilled. Jeremiah listed various nations that Yahweh
would "send the sword" among so that they would "fall and rise no more" (v:17). Some of
these nations have ceased to exist, but there is no historical evidence to justify believing that
Yahweh's judgment had anything to do with their demise. Others in the list, Egypt and Arabia,
still exist, and Gaza has recently "risen again." The prophecy said that Yahweh would "call
for a sword upon ALL the inhabitants of the earth" (v:29). He said that "evil shall go forth
from nation to nation" and "a great tempest will be raised up from the uttermost parts of the
earth" and "the slain of Yahweh shall be at that day from one end of the earth even unto the
other end of the earth." He further said that "they [the slain] shall not be lamented, neither
gathered, nor buried" but would be "dung upon the face of the ground" (vs:32-33). Dr. Price
can find nothing in history that any remote stretch of imagination can claim as fulfillments of
these ravings of the Hebrew mystic Jeremiah. Knowing full well that he couldn't produce
evidence of fulfillment for this part of the prophecy, Dr. Price resorted to a preemptive strike
and declared that the "rest of chapter 25 contains a figurative elaboration of God's judgment
of Judah and the surrounding nations in poetic terms" but that "(s)uch figurative language is
not to be interpreted beyond the reasonable way Jeremiah's ancient readers would have
understood it" (p. 3). Of course, the "reasonable way" that Jeremiah's "ancient readers" would
have understood it is the way that Dr. Price wants it to be understood in order to eliminate a
serious problem from the text, but such smorgasbord tactics are unacceptable. If Dr. Price is
going to buy the biblical inerrancy doctrine, he will have to buy the whole bill of goods that
go with it. He just can't pick and choose the parts he wants and dismiss the troublesome parts
as just unimportant, "figurative" trivia that are "irrelevant to the main issue[s]."

There is much more that could be said about Dr. Price's claim of prophecy fulfillment, but the
points I have made are sufficient to establish that there are no good reasons to see Jeremiah 25
as an example of fulfillment. Dr. Price cannot show that Jeremiah ever even made a prediction
that the Judeans would be taken into captivity by Nebuchadnezzar, and he can point to
nothing that fulfills any of the details that he tried to dismiss as just irrelevant "figurative"
language. If he wishes to respond to my rebuttal, he may do so in a later issue.

Perman Wrap-Up
Farrell Till
The Jewish author Pinchas Lapide has become the darling of Christian apologists, because
even though he himself is not a Christian, he has stated a quasi-belief in the historicity of the
resurrection. For some reason, apologists cite this as if it were some kind of conclusive
evidence that should end controversy over the foundation doctrine of Christianity, but I find it
inconsistent, to say the least, that Christians would use an argument that they would instantly
reject if anyone should present it as evidence for a position contrary to what they believe. If,
for example, a Christian writer should state, his belief that Muhammad was a genuine prophet
of God, Christians would rightly see this as proving no more than that this particular Christian

Volume 1990 - 2002 Issue


Page 948 of 2049
Skeptical Review Edited by Farrell Till
thinks that Muhammad was a real prophet. So if such argumentation proves nothing about
truth in other religions, it cannot be used to prove anything about Christianity.

This is all that I really need to say about the "testimony" of Pinchas Lapide, but there are other
facts about this writer that shed considerable light on possible reasons why he has stated
publicly a reserved belief that the resurrection did happen. In his book, The Resurrection of
Jesus (Translated by W. C. Linss, London, 1984, pp. 32-34), Lapide reveals an interest in
promoting dialogue and unity between Christians and Jews, a goal that would hardly be
promoted if he accused Christianity of having been founded on historical falsehood or self-
delusion in the first Christians. This fact alone is sufficient to make us wonder if Lapide's
position on the resurrection is a matter of sincerity or expedience. I find it hard to imagine that
a non-Christian would investigate the Christian resurrection claim, decide that it is a true
claim, yet not convert to Christianity. To say the least, this does not sound like a very firm
belief in the resurrection.

I said earlier that Lapide's belief in the resurrection was only a quasi-belief, and this is evident
from the many problems that he noted in the resurrection story. He noted, for example, that a
resurrected savior is not unique to Christianity, because there were "deities, heroes,
philosophers and rulers who, all long before Jesus, suffered died and rose again on the third
day" (p. 40), and he noted that resurrection from the dead was a belief that was familiar to
Jews as well as pagans (pp. 46ff). He pointed out that the only people who claimed that they
had encountered Jesus were those who were already believers in him. He acknowledged that
the New Testament is "the only source of the resurrection" and described the gospel accounts
as narratives that contain "much legend" and "glaring inconsistencies" (p. 32). A more
complete discussion of the problems that Lapide pointed out in the resurrection doctrine is in
The Jesus Legend by G. A. Wells, pp. 56-63.

Extrabiblical testimony: The embarrassment of not having any nonbiblical contemporary


records to corroborate the New Testament claims of amazing events that accompanied the
death and resurrection of Jesus has forced apologists like Perman (who is actually only
parroting the simplistic "arguments" of Josh McDowellian writers) have tried to manufacture
contemporary testimony where none really exists. Perman said, for example, that I had
"disputed [his] reference to the non-Christian Thallus" on the matter of the midday darkness
while Jesus was on the cross. What Perman was alluding to here was a statement made by
Julius Africanus, a 3rd-century Christian writer, whose work survives only in fragments. In
one of the fragments, he made this brief comment: "In the third book of his history, Thallus
calls this darkness an eclipse of the sun--wrongly in my opinion." On the basis of this
truncated quotation, Christian apologists have argued that a non-Christian contemporary of
Jesus testified to the midday darkness, but there are serious problems with their claim. (1) The
surviving fragment in which Africanus made this statement has the first letter(s) of the name
missing, so in actually, we know only that Africanus referred to someone named (X)allus.
Whether this was a reference to a writer named Thallus is, therefore, merely conjecture. (2)
Even if the name in this fragment was "Thallus," we can't really know who he was. (3) The
actual writings of this "Thallus" have not survived; we have only the brief allusion that
Africanus made to "Thallus's" reference to an eclipse, so it isn't possible to examine the
statement in its original context.

Volume 1990 - 2002 Issue


Page 949 of 2049
Skeptical Review Edited by Farrell Till
Steven Carr, a subscriber to TSR's errancy list on the internet, has posted some excellent
comments about early church writings. In a posting dated March, 30, 1996, he addressed these
and other problems in the reference that Thallus allegedly made to the three-hour darkness at
midday when Jesus was crucified: "I posted the full text of Africanus's citation of Thallus a
while back. It is not very convincing. It is, as far as we can tell, Africanus, not Thallus, who
identifies this eclipse with Jesus's crucifixion.... "What do we know about this Thallus? We
have two possible additional references to him. One, Eusebius tells us that this Thallus wrote
in Greek an account of world history from the fall of Troy down to the midfirst century--ca.
52. Thallus' work is generally believed to have been written in the period A. D. 50-100
[Murray Harris, JSOTGP5:344]. Eusebius wrote, `From the three books of Thallus in which
he collects (material) from the fall of Troy to the 167th Olympiad)....'

"Two, Josephus possibly refers to a certain Thallus as a wealthy Samaritan freedman of


Tiberius who had lent a million drachmas to the bankrupt Herod Agrippa (Antiquities,
18.167):

Now there was one Thallus, a freedman of Caesar's of whom he borrowed a million of
dracmae, and thence repaid Antonia the debt he owed her; and by spending the surplus in
paying his court to Caius, became a person of great authority with him

If these two are the same Thallus, then it would explain how he had both time to write a
history and how he had access to records (being a close associate of Tiberias) and how he had
knowledge of events in Palestine (being a Samaritan). The identification of these two
individuals is made by Emil Schuerer (HJP: 2.2.241).

"In Volume 3, Section 33a,10 (page 543-544), Schuerer writes as follows:

Thallus, according to Julius Africanus, mentioned a solar eclipse... so either Eusebius did not
hand down correctly the number of Olympiads or Thallus' work must have been extended at a
later date.... The reasons for believing Thallus to have been a Samaritan are two-fold. First, he
wrote about the history of Syria. Second... if Thallus is correctly reported by Africanus as
having written about the eclipse of A. D. 29, his work goes up to at least the time of Tiberius,
and it may therefore be possible to identify him with a Samaritan Thallus, whom Josephus
may (depending on the text) have mentioned.... However, Thallus in Josephus's text is only a
conjecture from "x"allus in the manuscripts (x is not theta), and although the original name is
difficult to understand in this context... it is possible. In that case the evidence for Thallus as a
Samaritan historian would disappear.... In favour of identification with Josephus' Samaritan
Thallus is the fact that the name occurs may times on Roman inscriptions.... The conjecture is
certainly reasonable and should be accepted with caution.

It hardly seems convincing to me. After all there were lots of Thalluses, and the manuscript in
Josephus does not say Thallus but only something close to Thallus. It is not clear to me at all
that Thallus, whoever he was, was writing about Jesus."

Carr has pointed out some serious problems in the Christian attempts to find in Africanus's
fragmented quotation an allusion to the midday darkness that was made by a writer
contemporary to the time of the alleged event, but there is another problem that must also be

Volume 1990 - 2002 Issue


Page 950 of 2049
Skeptical Review Edited by Farrell Till
considered. If Christian "apologists" are correct in their identification of this (X)allus, they
would still not have their contemporary witness, because this Thallus's birth is estimated at
about A. D. 50. At best, he could be considered only a near contemporary, and so if he did
allude to the midday darkness in the disputed quotation, he could only have been reporting
what he had heard but not what he had seen himself.

Perman complained that I did not "deal with the fact that in the Babylonian Talmud,
Sanhedrin 43a does record the crucifixion" (TSR, November/December 1996, p. 3), but
Perman surely knows that the references to Jesus in the Babylonian Talmud have been dated
no earlier than the 2nd century A. D., so these could not be considered contemporary
references either but merely reactionary statements to the principal claims of Christianity. The
same is true of Tacitus, Suetonius, Pliny, and other early writers whom Christians often cite as
"contemporary" witnesses to the historicity of Jesus, but none of them were contemporaries.
They all were born after the alleged crucifixion of Jesus, so the few brief and disputed
references that they made to him can only be considered allusions to what the Christians of
their time were known to believe. Perman is going to have to keep looking for a non-Christian
witness who was a contemporary of Jesus.

The Nature of the Claim


Farrell Till
Christian apologists argue that skeptics are unreasonably illogical when they reject biblical
miracle claims. They disdainfully refer to this as an "anti-supernatural bias." In my debate on
the resurrection with Michael Horner, he said in concluding his first speech, "One cannot rule
out the resurrection because of a prior assumption that miracles are impossible," and went on
to say, "As long as it's even possible that God exists, miracles are possible" (Horner-Till
Debate, Skepticism, Inc., 1995, p. 8). Of course, Horner himself was arguing from "a prior
assumption," because he was assuming that if a god exists, it is a god who intervenes in
human affairs to perform miracles. Such a view would be contrary to Deism, a religious
philosophy that believes in a creator who made the world to operate according to the natural
laws instituted at the time of creation, so if assumptions are not allowed skeptics, we have to
wonder why Christians think that they should be entitled to argue from an assumption that a
god does exist and that he/it is their particular god. There is an inconsistency here that they
need to explain.

The fact is that Christian apologists accuse skeptics of having an "anti-supernatural bias,"
when they themselves have the same bias. The only difference is that their bias is a selective
one in that they reject nonbiblical supernatural claims and then uncritically accept all miracle
claims in the Bible. In my debate with Michael Horner, I cited various nonbiblical miracle
claims during the cross-examination periods and asked if he believed that they are historical
facts. I referred to the Mormon claim that an angel delivered to Joseph Smith golden plates on
which the Book of Mormon had been transcribed in ancient Egyptian script, and Horner said

Volume 1990 - 2002 Issue


Page 951 of 2049
Skeptical Review Edited by Farrell Till
that he didn't believe that this had happened. I cited the various claims that Elvis Presley has
been seen alive, and Horner said that he considered them so unbelievable that he was
surprised I would even bring them up. I cited various miracle claims that Josephus recorded in
Wars of the Jews, 6:5.3, and Horner, apparently realizing by then the impression his answers
could be leaving with the audience, said that he didn't know about these; he hadn't yet
"checked into that one" (p. 9). I cited Suetonius's claim that two "divine forms" came down
and set fire to the funeral couch of Julius Caesar while Roman officials were arguing over
where to cremate the body, and Horner said, "Same response, Farrell. I don't know where this
is leading" (Ibid.). I suspect, however, that Horner knew exactly where this was leading, and
so he was trying to give the audience at least an appearance of consistency by stating that he
would have to "check into" these matters before he could state an opinion.

Finally, I ended this line of questioning by asking Horner if he could give us an example of
even one nonbiblical miracle that he accepted as historical fact, and he answered with one
word-- "No" (p. 16). By then, Horner's selective bias was rather evident. If a miracle claim
was recorded in the Bible, he routinely accepted it; if a miracle claim was recorded in
nonbiblical literature, he either rejected it or didn't consider it important enough even to
"check into." Yet he criticized me for dismissing miraculous claims on the grounds that
miracles don't happen.

Now along comes Dr. Price with the same complaint: "Only radical skeptics who insist on the
validity of their theological presuppositions in spite of objective evidence will deny this
conclusion" [that Jeremiah made a valid prophecy]. I suppose it has never occurred to Dr.
Price that if radicalism is present in a controversy like this, it is to be found in the inerrantists
and not in the skeptics. A skeptic who rejects a biblical miracle claim is doing nothing more
than applying to the Bible the same standard he would apply to miracle claims in the Book of
Mormon, the Qur'an, and other documents. If there are no sound reasons to believe the claims,
the skeptic rejects them regardless of whether they are recorded in the Bible or some other
documents. Inerrantists, on the other hand, apply a double standard. If a miracle is recorded in
the Bible, they accept it as historical fact; if it is recorded anywhere else, they, with few
exceptions, reject it.

If Dr. Price or any other inerrantists dispute this, they may be interested in accepting a
challenge that has gone unanswered, although it has been posted on the internet several times
where inerrantist subscribers have surely seen it. The challenge is to list just one miracle
claim in nonbiblical literature that inerrantists accept as historical fact. I will even suggest
some that they may wish to consider. In The Twelve Caesars, Suetonius stated that the
emperor Vespasian once healed a blind man and a lame man by just touching them (Penguin,
1979, p. 284). This miracle is parallel to some of the miracles attributed to Jesus in the New
Testament, so it would be interesting to see if any of our inerrantist readers accept this claim
as historical fact and if not, why not. In defending the resurrection claim in the gospel
narratives, Matthew Perman has argued that I should accept Aristotle's dictum that "the
benefit of the doubt should be delegated to the document itself, not arrogated by the critic to
himself" (TSR, November/December 1996, p. 2), so I wonder if he is willing to give the
benefit of the doubt to Suetonius and accept Vespasian's miracle cures as historical fact. Is
Price willing to do the same? Are other inerrantists willing to do the same? If not, they should
explain why not.

Volume 1990 - 2002 Issue


Page 952 of 2049
Skeptical Review Edited by Farrell Till
If they don't like this example, I can cite others. In Wars of the Jews, Josephus made these
miracle claims: (1) a heifer being led to the temple altar gave birth to a lamb, (2) one night the
temple altar glowed with such brilliance that it gave the appearance of daylight for half an
hour, (3) the eastern gate of the temple, which had taken twenty men to shut "with difficulty,"
opened one night of its own accord, and (4) soldiers and chariots were seen surrounding the
city of Jerusalem in the clouds (6:5.3). Of this last "certain prodigious and incredible
phenomenon," Josephus said, "I suppose the account of it would seem to be a fable, were it
not related by those who saw it" (v:297).

We can analogize these miracles with the New Testament resurrection claim. Christian
apologists argue that it is rational to believe the resurrection happened, because we have the
testimony of witnesses. The "testimony" of these witnesses is flimsy indeed, but be that as it
may, the Christian claim is that witnesses testified to the resurrection of Jesus, and so this
claim should be accepted. Josephus likewise stated that his miracle claims had been
witnessed, so shouldn't Christians be consistent and say that Josephus's claims should also be
accepted as historical fact? It will be interesting to see if any inerrantists will respond to the
challenge to tell us whether they accept or reject nonbiblical miracle claims like these.

The glaring inconsistency in the way Christian apologists defend biblical miracles can best be
illustrated by comparing the claims of two major religions. Like Christianity, Islam is a
religion with a holy book. Muslims believe that their Qur'an is, in fact, the verbatim word of
Allah, and also like the Bible, the Qur'an contains various miracle claims. Suran 54:1 refers to
a time when the moon was "rent asunder," and Muslims believe that this is an allusion to an
actual historical event, when the prophet Muhammad rent the moon into two pieces. In my
English version of the Qur'an, there is a long footnote affixed to this verse, which identifies
several Muslims who testified that they actually saw this event. They agreed that when it
happened, they could see the peak of Mount Hira "interposing between the two parts." I
receive e-mail messages from Muslims who have accessed some TSR articles at its internet
site, and one of them sent me a long commentary on this event that they claim is a historical
fact. Muslims, then, apparently consider this claim as much a historical fact as Christians
consider the resurrection a fact of history.

Both of these alleged events are such that if they actually happened, they would have received
hemispheric notice. If the light of the sun failed for three hours at midday as Luke said
(23:44), then by necessity this would have been noticed all over the hemisphere, which the
sun had been shining on when it suddenly went out. Likewise, if the moon was rent asunder,
this should have been an event that was witnessed throughout the hemisphere. In this case, I
say should, because we can imagine the possibility that if this occurred at night, people could
have been inside their homes and not observed it, although it would certainly be unlikely that
people throughout the hemisphere would have been inside homes, where the event would
have passed unnoticed. With the midday darkness, however, whether people were inside or
outside, the event would have been noticed, because people in their homes couldn't have
helped seeing that the light of the sun had suddenly vanished. So of the two alleged events,
we must consider the midday darkness to be the one that would have been more widely
noticed.

Volume 1990 - 2002 Issue


Page 953 of 2049
Skeptical Review Edited by Farrell Till
So we have two very extraordinary claims that both have their roots in holy books considered
by their respective adherents to be the divine word of God or Allah, yet neither religion
considers the other's miracle claim to be historical fact. Muslims don't believe that a midday
darkness occurred when Jesus was crucified, and Christians don't believe that Muhammad
once rent the moon asunder. Neither claim has anything to support it except the dubious
"testimony" of the biased followers of each religion. Christians can produce no non-Christian
contemporary witnesses who claimed that they personally experienced the midday darkness,
and Muslims can produce no non-Islamic contemporary witnesses who claimed that they
personally saw the moon divided into two pieces. Each rejects the other's claim.

The reasons why each religion rejects the other's claim is essentially the same as the skeptic's
reason for rejecting both. I have repeatedly stated that reason in past articles, and Perman and
Price have both ridiculed it as an "anti-supernatural bias." If, however, it is an anti-
supernatural bias to reject a miracle claim in the absence of exceptionally good evidence--and
mere testimony, especially the testimony of religious adherents, is not exceptionally good
evidence-- on what grounds do they justify their rejection of the Qur'anic claim that
Muhammad once rent the moon asunder? Apologists like Perman and Price argue that if the
resurrection is rejected, then skeptics have no grounds for accepting anything from the distant
past as valid history, but this is a ridiculous position. When Christians reject the claim that
Muhammad rent the moon asunder, do they surrender all grounds for accepting anything else
from ancient documents as valid history? Certainly not, for with the exception of what they
read in the bible, they evaluate historical claims the same way that skeptics do. If a historical
document makes a mundane claim that common sense tells the reader is something that could
have easily happened because it or something similar to it has been known to happen many
times, he/she simply applies Aristotle's dictum and accords the benefit of the doubt to the
document. If, however, the claim is something that the reader knows is completely out of the
ordinary, he/she rejects it if there is nothing but the mere word of the writer to support the
claim.

In the case of the Qur'anic claim, I have noted that it is an event that could have happened
without the widespread attention that the midday darkness would have necessarily received,
because it could have been an event that happened at night when many people were inside
their homes and possibly even sleeping. Yet Christians reject the more believable claim but
accept the less believable claim of their own holy book. So who is it that has a "bias"?

It is the very nature of the claim that makes the resurrection and prophecy fulfillment
unbelievable. Christians are the biased ones, not skeptics. Why can't they see that?

__________________________________

Lockwood-Till Debate

The Lockwood-Till Debate has been scheduled for March 24-25, 1997, at the Barnes Church
of Christ, 11826 South East 59th, Oklahoma City, OK 73150. Lockwood will affirm that
"God exists," and Till will affirm that "matter and the forces and laws that must necessarily
accompany the existence of matter are the only provable realities in the universe."

Volume 1990 - 2002 Issue


Page 954 of 2049
Skeptical Review Edited by Farrell Till
We are not involved in the organization of the debate, so for additional information, you
should contact Marion Fox at 4004 Twisted Trail Road SE, Oklahoma City, OK 73150-1910
or Bill Lockwood, 25 Cherrywood Circle, Marshall, TX 75670. Lockwood's phone number is
(903) 935-7448 or (903) 938-4422.

From the Mailbag


Failure of the Tyre Prophecy...

I have been thinking carefully about the comments you made in response to my article, which
dealt with the Tyre "prophecy" (TSR, September/October 1996). Farrell, I agree with you that
the Tyre "prophecy" failed miserably. Please do not print the rebuttal article I sent you.

I also want to set the record straight: I am not an inerrantist. I am just not good at refuting the
religious flimflam masquerading as truth. For seven years, or thereabouts, I subscribed to The
Plain Truth magazine, which is published by the Worldwide Church of God. This cult almost
totally brainwashed me. Herbert W. Armstrong, who ruled the church until his death in 1986,
was a master at propaganda. I still have a couple of the church's booklets, and this is where I
got the idea for my article about Tyre in The Skeptical Review.

I hope that you take on the Worldwide Church of God in a future issue of The Skeptical
Review.

(Matthew Hogan, 177 Salisbury Street, Rochester, NY 14609-4137)

EDITOR'S NOTE: Unfortunately, the January/February issue of TSR had already been sent to
press by the time Mr. Hogan's letter arrived, so I could not remove his article. Having heard
from people who were once members of the Worldwide Church of God, I can understand the
difficulty Mr. Hogan has experienced in trying to break away from its influence. The position
that Hogan presented in the Tyre matter is basically the same as other inerrantists have used in
trying to defend this prophecy, so perhaps our exchanges on the subject will still be beneficial
to readers who encounter Christians who see a wonderful example of prophecy fulfillment in
Ezekiel's tirade against Tyre.

Response to Perman...

Matthew Perman commits a fascinating defense of the resurrection of the physical body. So
explain again how it works. I've been dead, say, for 6,000 years (the limit, to biblical
literalists). My body has been eaten by worms and spread throughout the soil, or perhaps I've
been cremated, blown on the winds, and captured by the falling rains. My bodily molecules
have become nutrients absorbed in the leafy greens you ate for dinner. You are what you eat.
So come the bodily resurrection, whose body will it be?

Volume 1990 - 2002 Issue


Page 955 of 2049
Skeptical Review Edited by Farrell Till
(David Nixon, P. O. Box 562, Marmora, NJ 08223)

EDITOR'S NOTE: Biblical fundamentalists will no doubt retreat to their old stand-by to
answer Mr. Nixon's question: God can do anything, and so he will be able to reassemble every
body of every human who ever lived. This is the way they explain away all of the problems in
the story of Noah's ark or any other objection that may be raised against the integrity of the
Bible. In other words, they will argue that the unprovable existence of their deity will explain
all problems that can be identified in the biblical text. They are apparently unruffled by the
obvious fact that the same assumption would prove the truth of the Qur'an, the Book of
Mormon, the Avesta, or any other holy book.

At any rate, Mr. Nixon has raised a question to which Christians should give serious thought,
but probably very few will. Perman's article has brought numerous reactions from readers,
none of which were in support of his position. We are printing some of them below.

The Testimony of Women...

I just finished reading the Matt Perman article in the latest issue of The Skeptical Review. I
have not read your "answer" to it yet, but I wanted to reply to a rhetorical question he asks
near the end of the article before I read your reply. His question is (and I'm paraphrasing),
"Since women's testimony had no standing in that culture, wouldn't it make more sense to
have two of the disciples find the empty tomb, rather than Mary Magdalene and another
woman?"

The fact that the word of women had no standing in that culture is exactly the point. If you
were going to tell an outlandish story and wanted to give the story a certain appearance of
authenticity, wouldn't you include a couple of people in the story who were alleged witnesses,
but who couldn't call you liar if the story got back to them before they died? I certainly
would! Who better to include in your story than the entire U.S. Congress as witnesses,
followed by the caveat, "Of course, we all know what liars politicians are, so darn it all,
there's no dependable witness who can verify my story one way or the other!"

The inclusion of two women in the story was a masterpiece of a snow job. There's no way
they would have named any witness who could call the gospel writers liars and be believed!
This way, they got the best of both worlds: two witnesses, and witnesses who wouldn't be
believed if they denied the truthfulness of the story. Furthermore, the two witnesses they
named would have known that merely opening their mouths and calling the storytellers liars
would have made them persona non grata for the rest of their lives!

(Marilyn Burge, 6925 SE Knight Street, Portland, OR 97206-5956; e-mail


Marilyn_Burge@msn.com)

EDITOR'S NOTE: Besides the possibility that the gospel writers may have purposefully
selected women for the express purpose of creating the very situation that Perman sees as
credibility, they would have had the added advantage of knowing that these women (if indeed
they ever even existed) were probably dead by the time the gospels were written, and so they

Volume 1990 - 2002 Issue


Page 956 of 2049
Skeptical Review Edited by Farrell Till
would not have been able to deny the role that the gospels attributed to them. The axiom that
says dead men tell no tales would be equally true of women.

More Questions for Perman...

If Matthew Perman believes or has concrete evidence pertaining to the birth of Jesus as a
supernatural phenomenon, that the same was conceived either by the visit of the angel
Gabriel, the Holy Ghost, or the biblical God, then his belief in the resurrection according to
his reasoning is accurate. Only a supernatural entity could die a physical death and by some
esoteric alchemy reconstitute an inanimate body back to its original state of being. But the
only concrete evidence to prove that Jesus was a supernatural being rests on the assumptions
of the unknown authors of the New Testament who composed the biblical text many years
after the pregnancy of Mary was conceived.

Did the authors of the New Testament interview Mary and get firsthand information on her
insemination by the God of the Bible? Did they have a conversation with Joseph, his earthly
father, and try to ascertain whether or not this event actually took place? The only source of
verification for Jesus's existence as a supernatural being is the unknown authors of the New
Testament.

Mr. Perman believes the resurrection was plausible, possible, and probable, because some
unknown authors attributed this event to a being who could no longer verify the accuracy of
this event and was no longer available for comment, since his physical presence as a witness
was unavailable. Of course, we can still pretend that the authors were in divine
communication with God when they wrote the New Testament. In a court of law, one cannot
convict a person on the frivolous, flimsy evidence of people who did not participate as
spectators to a crime, but merely made assumptions on the fictional accounts of others who
may have wanted to believe that such an event took place. The authors of the New Testament
were enthusiastic converts and defenders of what eventually evolved into the Christian
religion. Deception in the proliferation of a myth based on evidence that is not verifiable, that
is not mentioned by other learned men of that time, is the mystical propaganda of Christian
demagogues.

How, if it is true that Jesus was supernatural in origin, capable of miracles, a god-man, and a
divinity, was it even possible to cause his death? Would the devout with a fundamentalist
perception subscribe to the fact that a man-god can be murdered (crucified) when it exists
both physically and supernaturally? If he was a miraculous being, then he was incapable of
death by any physical means. He therefore could not die, because his body was structured as a
supernatural entity rather than as an entirely physical and material being. If Perman believes
that Jesus was supernatural in origin, then he must ask himself the question: "How can a man-
god die a physical death, when it is composed of supernatural elements that are able to exist
independently of and outside the natural laws of nature?"

If, as the New Testament has established, Jesus was the deified son of God, he could not have
died on the cross or by being run over by a chariot or by being pushed over a cliff. He
therefore would not have needed to rise from the dead, because his body would have been

Volume 1990 - 2002 Issue


Page 957 of 2049
Skeptical Review Edited by Farrell Till
incapable of physical death. For a resurrection to occur, he would have had to exist as a
mortal man, capable of reconstructing a physical, material self. This, according to the New
Testament, was not possible, since he was born as the supernatural son of God and not as a
mortal man.

Finally, since it is written that he ascended into the upper atmosphere, just how far up did he
ascend in order to reach his heavenly destination? Was it beyond Mars, and if so, how did the
physical part of his body (flesh and blood) withstand the extreme variations of heat and cold?
A man composed of myth or an imaginary creature could defy the elements, but if this was
the condition of his essence and being, then there was no way he could have died on the cross.
For a resurrection to occur, he would have had to fake his death by acting for several days as a
deceased mortal, and then reappear to baffle a few enthusiastic spectators by exhibiting a
reconstructed body. Hopefully, Mr. Perman can respond to this in a future article by offering
more evidence and conclusive proof to support his undeniable and certain belief that a
resurrection actually occurred.

(Leland W. Ruble, 833 Orchard Street, Toledo, OH 43609)

Basic Rules of Evidence...

The first rule of good reasoning is not to accept a claim without compelling evidence. We
want to keep our central stock of knowledge reasonably free from error. Therefore, like a
jealous guardian, we set up stiff standards that a claim must pass before it earns the title of
"true knowledge." Until it passes such a test, a claim is at best a working hypothesis.

The role of the benefit of the doubt cannot be to undermine the first rule of good reasoning.
Thus, it cannot require us to lower those standards.

The second rule of good reasoning is that extraordinary claims require extraordinary proof. If
you tell the boss that you had a flat tire, at least that is believable. It is a reasonable working
hypothesis even if it is not actually confirmed by a careful check of he facts. However, if you
tell your boss that you were kidnapped by Martians, that's not going to go over very well. If
you want to be believed, you had better bring in some stunning evidence, some extraordinary
proof as it were.

We may give the benefit of the doubt to a work provided that it allows for a good working
hypothesis. The Martian story, however, just doesn't make a good working hypothesis,
because there is no established track record for such events. (Compelling evidence is still
required if we wish to consider a work part of established knowledge.)

Neither does Jesus's resurrection story lend itself to a good working hypothesis. Not only are
there no established track records to go by, but scientific understanding strongly argues
against such a thing. Thus, the benefit of the doubt does not apply to the Gospels.

The New Testament, as Till and others have shown, does not supply the extraordinary
evidence needed to support Jesus' supposed resurrection. The general consensus among

Volume 1990 - 2002 Issue


Page 958 of 2049
Skeptical Review Edited by Farrell Till
mainstream scholars, after much study and analysis over the years, is that the Gospels cannot
even be treated as historical documents. They were apparently written to shore up diverse
theological viewpoints within the early church.

At first, there was no need for any written material. All good Christians expected the world to
come to an end in the first century, including Jesus according to the Gospels, so why go to the
trouble and expense of writing things down? Later, as doubts arose and as competing views
evolved or were encountered, a need to define and fix one's theology developed.

From Mark to the latest noncanonical gospels, we have a continuous spectrum of evolution
that reflects part of this development. The story gets more detailed, more complete, and
names become attached to people who were formerly not named.

The oldest copies of Mark (the oldest Gospel) begin with Jesus's baptism and end with the
confusion of the empty tomb. Internal evidence suggests that Mark was built around various
bits of tradition. That is, a fictional history of Jesus's ministry was probably developed by
Mark to tie together the various Jesus stories. Later, different postresurrection scenes were
added to Mark. Still later, Matthew and Luke copied extensively from Mark, improving
Mark's grammar in some places. However, each added a different infancy story. The
postresurrection scenes were expanded. And, in the noncanonical gospels, the final empty
spaces, Jesus's childhood and Jesus's manhood, were filled in.

In the Gospels no one actually witnesses Jesus' resurrection. Even the guards supplied by
Matthew have fainted away and are useless for that purpose. That must have bothered at least
a few of the church fathers, because the Gospel of Peter makes it a point to come up with
some witnesses. That small addition, though noncanonical, is a good example of how Gospel
"history" was actually created.

Perhaps, you can begin to see why mainstream scholars have long ago rejected the Gospels as
eyewitness accounts. The Gospels are an evolved phenomena of the early church, and their
purpose was to support church doctrine--not to record history. There were several gospels
because of the divisions within the church, both in time and place.

Though some mainstream scholars hold out the hope that bits and pieces of a genuine Jesus
might still be retrieved from the Gospels, they rightly reject the Gospels as historical
accounts. (The purpose of The Jesus Seminar, for example, was to sift out the genuine Jesus
from the evolved growth of the Gospels.) Other scholars argue that even that modest goal is a
vain effort, that the real Jesus (if any) has been lost forever.

Into this complex picture strolls the fundamentalist who is blissfully unaware of these
problems, who naively sallies forth with arguments based on the empty tomb, Matthew's
guards, the arrangement of Jesus' grave clothes and other "facts" associated with the Gospels.
They seem incapable of understanding that their tightly constructed arguments are worthless
because the whole cloth is rotten. Such a monumental fact is beyond their comprehension, all
the more so as many of them deliberately wear blinders.

Volume 1990 - 2002 Issue


Page 959 of 2049
Skeptical Review Edited by Farrell Till
It is beyond the poor powers of a mere letter to bring the full light to those who willingly live
in intellectual darkness. All I can do is shed some light on why educated skeptics reject the
Gospels, and I leave it to the energetic mind to pursue the matter to its proper end. A few will
catch the ray and outgrow the need for darkness.

(Dave Matson, P.O. 61274, Pasadena, CA 91116; e-mail 103514.3640@CompuServe.com)

EDITOR'S NOTE: "It is the nature of the claim, stupid!" This was the central idea that I tried
to present in my second response to Perman. The very nature of the resurrection claim is such
that its verification requires much more than a few biased documents (which have obviously
been tampered with over the centuries) and early Christian traditions about apostles who died
for their beliefs. This is a point that seems to elude Matthew Perman and his resurrectionist
cohorts, who see amazing evidence where, in reality, no evidence at all exists. I have
discussed this point further in "The Nature of the Claim" on pages 10-11 of this issue. Let's
hope that Perman or the next resurrection apologist who submits an article will try to address
this issue rather than just regurgitating discredited apologetic arguments.

What Did the Apostles Die For?

First of all I'd like to thank you for the work you do in rebutting the fantastic claims of Bible
inerrantists. In the current political climate, I believe it is of the utmost necessity for skeptics
to speak out firmly against irrationality.

In Matthew Perman's response to you he says, "While martyrs of other religions have died for
what they sincerely believed was true, the difference is that the disciples would have been
dying for what they sincerely knew was a lie! As I said in my article, `Ten people would not
all give their lives for something they know to be a lie.' Therefore we must conclude that the
disciples believed that Jesus rose and appeared to them because Jesus really did rise and
appear to them!"

First of all, it is not necessary to conclude this because Perman is making some assumptions
that may or may not be erroneous. The first is that there were disciples at all who died
because they believed they had seen the risen Jesus. What has come down to us through
history are the names of men who are the same as those of the supposed disciples to Jesus,
who were supposedly killed because they didn't renounce their belief in Christ. Is it accurate
to assume that because they had the same names, those who were supposedly executed were
indeed the same men as described in the New Testament? It is suspected with a good deal of
evidence that later Christians were not above altering historical documents to portray their
religion in a better light. Redactions in the writings of Josephus are the best example of this
sort of alteration. The assumptions that Perman makes are that the men executed were
disciples of Jesus and that a Jesus existed, so these men could have been his disciples.

Perman also makes the assumption that these men who would have had to have died for what
they "knew" was a lie were rational and not hallucinatory or deluded. Perman presumes to
know that men do not die for causes which they know to be a lie. If Perman were in a similar
situation and felt that he was going to be executed anyway, I wonder if he would recant even

Volume 1990 - 2002 Issue


Page 960 of 2049
Skeptical Review Edited by Farrell Till
if he knew that he was dying for a lie. I submit that Perman might be more than willing to
martyr himself for what he knew to be a lie if he felt that in so doing it would cause others to
stand and fight for whatever his cause du jour might be.

In essence, Perman is professing to be able to read the minds of those long dead when he
makes the assertion that no one would die for what they knew to be a lie. If there are
examples anywhere of men or women who have died for a lie, this renders Perman's argument
invalid. Unfortunately, it is nearly impossible to really know what any individual truly
believes. But isn't this fact also a problem with Perman's argument?

(Stephen Sommers, 67 West Indian Sage, The Woodlands, TX 77381; e-mail


ssommers@ix.netcom.com)

Rationalist or Rationalizer?

Your refutation of Matthew Perman's article on the resurrection was masterful. I don't see
how people of his ilk can possibly take such a story seriously, but then again inerrantists
would take Jack and the Beanstalk as "revealed truth" if it were found in scripture! There's a
big difference in being a rationalist and a rationalizer.

(Jon Nelson, 15225 Cantara Street. Panorama City, CA 91402.)

Vivid Imagination...

I notice that Anek Imphita, ("From the Mailbag") considers himself a "wise man." As well, he
considers himself as one who subscribes to approaching issues with "wisdom, reason and the
ability to think and discover the truth freely." He dismisses "those people" who do not, from
his perspective, apply these things to their thoughts and logic.

It is nice to see that, finally, a skeptic has shown a real contradiction. Note what Anek states
at the beginning of his letter when discussing his approach to reading the Bible: "I have
picked up the Bible and tried to read it many times, but when they say that the stories in the
Bible must be accepted as true stories, I cannot make my brain accept it, so I put it down.
How can a man read a fairy tale childish story and accept them as truth?"

First of all, please note: Anek says he has never even read the whole Bible; he just "puts it
down." He states that he relies on second-hand (at the least) interpretations and opinions of
the Bible to make up his mind--hardly a "reasonable" way to reach a conclusion. He states
that he is one "who discovers the truth freely," yet he doesn't even read [what] the Bible says
before deciding it is untrue. He states that the stories in the Bible are "fairytale/childish" yet
he has not even read them or the context they were written in! He implies that those who
subscribe to such "stories" are "destroyers, who bring darkness to this world, not the light as
they say," yet he hasn't even read what "they" say.

Volume 1990 - 2002 Issue


Page 961 of 2049
Skeptical Review Edited by Farrell Till
Anek states that "I cannot make my brain accept it"--so, Anek, do you make your brain do
things and think things? If so, what is it in you that performs this function? Do you force
yourself to believe things? If so, then the truth really has little meaning to you; you are only
interested in what you want, or "force yourself," to believe. Is this what you term the "free
discovery of truth"?

Conjecture, opinion, feeling and assumptions have given Anek his conclusions: his "truth" if
you will. Is it reasonable to dismiss something as a falsehood without even looking at it? Is a
person "wise" who makes a conclusion without even considering any evidence or making
observations? Is one "discovering" anything when one isn't even looking and hasn't even
found anything?

Anek encourages TSR to "keep doing what you are doing"--and what exactly is that? Do
Anek's comments reflect the sole purpose and meaning for TSR: to falsely represent an idea,
or a religion, "putting it down" before even honestly considering it? Do skeptics make a habit
of forming irrational conclusions such as Anek's? Or is his an isolated incident?

Anek states that he "wishes that more people have the chance to read these (TSR) marvellous
works." If everyone was like him, they would put it down before reading it anyway, so it
would make little difference what TSR said!

No wonder the atheistic and skeptical argument does little convincing: it argues the lack of
substance in Christianity with fluff of its own. As the old saying goes: the skeptical position
has its feet firmly planted in mid-air.

And the Editor "supports" this way of thinking. Knowing the editor as I do, you'll have to
pardon me for not being surprised.

Anek, I suggest being honest and giving the Bible a fair read and a fair and introspective
consideration; it deserves at least that much before you dismiss it.

(David Court, 28 Mitchem Drive, Etobicoke, ON, Canada M8W 2S8; e-mail
hoover1@newtcom.ca)

EDITOR'S NOTE: The name David Court will be familiar to all TSR readers who are also
subscribers to the errancy list on the secular web. He tries to respond to every posting that
presents a biblical inconsistency or discrepancy, and his chief way of responding is simply to
deny that a contradiction has been proven, no matter how compelling the evidence. He is the
typical Jack-and-the-Beanstalk type that Jon Nelson referred to above.

Space restrictions will not allow me to review Court's fallacious apologetic methods that he
has displayed on the Errancy list, but we can see in his letter that he is not above distorting
statements to mean what he wants them to mean. He began his letter by saying he had noticed
"that Anek Imphita... considers himself a `wise man,'" when in reality all that Mr. Imphita
said was that "wise men appreciate and honor you." This was intended as nothing more than
an obvious compliment that he was extending to me, so it is certainly a distortion to try to
make it mean that Mr. Imphita was boasting that he was a wise man.

Volume 1990 - 2002 Issue


Page 962 of 2049
Skeptical Review Edited by Farrell Till
Because Mr. Imphita has tried many times to read the Bible but finds the stories insulting to
his intelligence, Dave Court distorts this to mean that Imphita relies on "secondhand
interpretations and opinions." If a Christian should try "many times" to read the Qur'an or the
Book of Mormon but found its content too fantastic to maintain his/her interest, I'm sure Mr.
Court would not call this Christian's rejection of these books reliance on "secondhand
interpretations and opinions," but if a non-Christian finds it difficult to swallow the nonsense
in the Bible, this is "secondhand interpretation and opinion." Well, I'm sure many Errancy
subscribers will agree with me when I say that this is typical David Court logic. I wonder if he
would mind telling us how idiotic ancient literature would have to be before one would be
justified in considering it nothing more than myth and superstition.

Mr. Imphita said that he had tried "many times" to read the Bible, so a reasonable person
would recognize that many attempts to read the Bible could bring enough familiarity with its
content to justify labeling it a collection of myths and fairy tales, so it is hardly accurate of
Mr. Court to say that Mr. Imphita hasn't even read what "they" say. After all, how much of
the Book of Mormon would Mr. Court have to read before he would be entitled to reject it as
a book that is too incredible to believe? At any rate, if Court thinks that Mr. Imphita has been
unfair in his opinion of the Bible because he may not have read it all, I will issue a challenge
that Court may want to accept. I have read the Bible--many times--and I don't hesitate to say
that it is a book riddled with ancient myths and superstitions, inconsistencies, contradictions,
discrepancies, absurdities, and failed prophecies. If Mr. Court thinks this is an unfair
representation of the Bible, he is free to respond to any articles that I publish in this paper or
to submit his own defense of the Bible. I will gladly publish it. The only problem for him is
that I will publish it simultaneously with my rebuttals (which he will then be free to rebut). I
really don't expect Mr. Court to take advantage of this opportunity.

Peace and Contentment...

Please keep me on your mailing list. I received the recent copy yesterday and, as usual, I
devoured very word!

I admitted to myself and others that I am an atheist, after 40 years of living, 20 married to a
preacher. Ugh! At last, I found happiness, peace, and contentment, never possible in the
fetters of religion. I love The Skeptical Review.

(Dr. Dorothy B. Thompson, P. O. Box 562, Bandon, OR 97411.)

An Internet Contact...

I ran across your article on the internet; then I read that you were an ex-Church of Christ
preacher. I was a Church of Christ member all of my life.

When I was at Freed-Hardeman in '69-71, I decided to let Christ control my life and use me to
spread the gospel. After all, if we had the truth and the world would be saved only if we gave

Volume 1990 - 2002 Issue


Page 963 of 2049
Skeptical Review Edited by Farrell Till
them that truth, every Christian would have to get serious about salvation. Well, guess what?
Bullshit!

It's good to know there's someone who had the courage say, "Look, we've been lied to and the
real truth is important. Please send me the Skeptical Review.

Thank you and God Bless (Oops). Keep up the good work. Were you ever a speaker at the
Freed-Hardeman Lectures? You'd be good in Open Forum.

(Rex A. Mobley, 501 County Road 513, Rienzi, MS 38865.)

EDITOR'S NOTE: By coincidence, I am an alumnus of Freed-Hardeman too. My first


ministerial training was done there a decade before Mr. Mobley's time, when it was only a
junior college. I'm sure that I would not be welcome there as a speaker. I have, in fact, tried to
arrange a debate there, but the proposals have received no responses.

I consider it significant that Mr. Mobley contacted me through the internet. The posting of all
TSR articles on the secular web has brought hundreds of subscription requests. I consider the
internet one of the greatest opportunities for freethinkers to combat religious irrationality. If
you have a multimedia computer, you should seriously consider putting it on line so that you
can receive information and participate in on-line discussions of biblical issues. TSR articles
can be accessed at http://www.infidels.org/library/magazines/tsr/. To subscribe to the Errancy
list owned by TSR, send the two words "subscribe errancy" to major-ii@infidels.org. This will
add your name to the list to receive copies of all postings and entitle you to respond to
postings of interest to you.

Tedious Details...

These arguments over details can get tedious but nonetheless are enlightening. What I would
like to see more of is what is known--in the scholarly sense-- about the origins of the various
books in the Bible and their authors. Pick a book, any one, and tell us how and why it was
written, and who the real authors might be, and what editing was done, and when and why.
And what was the agenda of the authors and editors?

(Wayne Yoder, 5249 Mountain Crest Drive, Knoxville, TN 37918.)

EDITOR'S NOTE: The subjects Mr. Yoder suggested are certainly fascinating, but the
average Christian fundamentalist would summarily dismiss such articles as "liberal"
nonsense. In my opinion, success is improbable with them, at best, but more likely when the
Bible is taken, just as they study it in their church classes, and shown to be contradictory and
inconsistent in its content. All they can do then is scream, "You're taking it out of context!"

In case anyone is interested in the subjects Mr. Yoder mentioned, I would recommend Who
Wrote the Bible? by Richard Elliot Friedman as a good source of information on the origin of
the Old Testament, and Who Wrote the New Testament? by Burton L. Mack for the same type
of information on the New Testament. Motives behind the editing of New Testament books is

Volume 1990 - 2002 Issue


Page 964 of 2049
Skeptical Review Edited by Farrell Till
discussed in The Orthodox Corruption of Scripture by Bart D. Ehrman. These books can be
obtained from H. H. Waldo, Bookseller, P. O. Box 350, Rockton, IL 61072. The telephone
number is (815) 624-4593.

Volume 1990 - 2002 Issue


Page 965 of 2049
Skeptical Review Edited by Farrell Till

Skeptical Review
Volume Eight, Issue Three
May/June 1997
Farrell Till, editor

• New Testament Family Values


Till discusses the family values taught by Jesus and other New-Tesament figures

• Solving the Jeremiah Problem


Price's second defense of the 70-year servitude prophecy in Jeremiah

• Whatever Happened to the Burden-of-Proof Factor?


Till claims that the burden of proof is on Price, and that he has proven nothing

• Holy Bible by Anonymous: A Book Review


Judith Hayes (a.k.a The Happy Heretic) imagines what an unsuspecting reviewer
would think of the bible

• Schmitt Replies to Wasserman


Schmitt's side in the Hebrew language debate

• Conspicuous by Its Absence


Yoel expands his argument that the hebrew "yotser" means treasurer, not potter

• From The Mailbag


Readers respond to The Skeptical Review

Volume 1990 - 2002 Issue


Page 966 of 2049
Skeptical Review Edited by Farrell Till

New Testament Family Values


By request, the series on biblical family values is being extended another issue so that we can
look at what the New Testament teaches us on the subject by both example and decree. For
some reason, Christians seem to believe that Old Testament accounts of atrocious conduct on
the part of God and famous biblical characters are unimportant, because, after all, those things
were written in the Old Testament and not the New, as if the two testaments are not
inextricably bound together in the traditional Christian claim that everything in the Old
Testament happened to lead the way to God's plan of salvation in Jesus Christ. Christianity,
then, must be held accountable for the god and biblical heroes in whom their own religion is
firmly rooted. Christians can't just wave aside matters like those noted in previous articles in
this series by saying, "Oh, well, that was in the Old Testament."

Even if they could evade the problem that easily, they would still have to explain why so-
called family values don't fare much better in the New Testament. One would think, for
example, that if proper family values could be found anywhere in the New Testament they
would be evident in the conduct of Jesus himself and the couple whom God chose to be the
earthly parents of his only begotten son, but such is not the case. In Luke 2:41-49, we read a
story that certainly doesn't present Joseph and Mary as exemplary parents. When Jesus was
12, the family traveled from Nazareth to Jerusalem to keep the feast of the Passover, and
when it was over, his parents were returning home and had gone "a day's journey" before they
noticed that Jesus was not in the company they were traveling in. It's hard to imagine how
responsible parents could travel for an entire day and not know that their minor child had been
left behind in the city they had visited. Ordinarily, a couple doing this would be considered
parentally negligent, but if the couple should be the ones whom God chose as the parents of
his son, that apparently made it all right, because the inspired writer said nothing critical
about the favored couple.

This little yarn concludes with the claim that after Jesus was found by his parents, he returned
to Nazareth with them and "was subject to them" (v:51). He may have been "subject" to his
parents, but he wasn't always polite to them. According to John, the first miracle that Jesus
performed was at a wedding in Cana of Galilee. When the wine ran out, Mary went to Jesus
and said, "They have no wine," and in response, her perfect son said, "Woman, what does
your concern have to do with me? My hour has not yet come" (4:1-4). A man today who
would speak in this manner to his mother would be considered disrespectful, and rightly so,
but maybe Jesus in his omniscient wisdom thought that once a man had reached adulthood, he
was no longer obligated to be polite to his mother. If so, most men today would disagree.

The incident at Cana wasn't an isolated one. On an occasion when Jesus was speaking to a
crowd, he was told that his mother and brothers "stood without, seeking to speak to him."
When he was told this, Jesus said, "Who is my mother and who are my brothers?" He then
gestured at his disciples and said, "Here are my mother and my brothers, because whoever
does the will of my Father in heaven is my brother and sister and mother" (Matt. 12:46-50). A
lot of preachers see noble concepts of brotherhood in this statement, but it is just as easy to
see flagrant disrespect for close family relationships.

Volume 1990 - 2002 Issue


Page 967 of 2049
Skeptical Review Edited by Farrell Till
Telling the truth is a recognized family value, but the son of God wasn't above lying to his
family. Once when the feast of tabernacles was "at hand," his brothers, who presumably didn't
yet believe in him, challenged Jesus to go to Jerusalem and prove himself by doing his signs
openly. Jesus then said to them, "My time has not yet come, but your time is always here....
Go to the festival yourselves. I am not going to this festival, for my time has not yet come"
(John 7:6-8). His brothers then left for the festival, and after they were gone, Jesus "also went,
not publicly but as it were in secret" (v:10). It's hard to see lying as a family value, but what
else could this be called? At the very least, it has to be considered deliberate deception. Is that
a value that families today should emulate?

In his personal relations with his family, Jesus demonstrated an attitude that is hardly
commendable, but in his public teachings, he fared even worse. This is the man who said, "Do
not think that I came to bring peace on earth. I did not come to bring peace but a sword. For I
have come to set a man against his father, a daughter against her mother, and a daughter-in-
law against her mother-in-law, and a man's enemies will be those of his own household. He
who loves father or mother more than me is not worthy of me, and he who loves son or
daughter more than me is not worthy of me" (Matt. 10:37). He even said that anyone who
came to him and did not hate his father and mother, wife and children, brothers and sisters
could not be his disciple (Luke 14:26). Jesus taught his followers to demonstrate a cultist
fanaticism toward him, which, if actually practiced, would destroy not just family values but
the family institution itself.

On another occasion when a disciple asked leave to go bury his father, Jesus said to him, "Let
the dead bury the dead" (Luke 9:60), and when another one told Jesus he would follow him
but first wanted to "bid farewell" to them that were in his house, Jesus said, "No man, having
put his hand to the plow and looking back is fit for the kingdom of God" (vs:61-62). Jesus
showed little interest in family relationships and demanded that his disciples do the same. It's
hard to see in this man's teachings much of anything that could be considered "family values."
If parents today should practice the family values that Jesus taught, they would save no
money for tomorrow (Matt. 6:19-20) and apparently show no interest in providing for the
needs of their families. They would not worry about what they would eat or drink or wear
(Matt. 7:25-31), and in so doing would lead their families into poverty and even total
destitution. If people today should make a serious attempt to follow the teachings of Jesus, the
family institution would be destroyed, yet the Christian Right keeps demanding that our
government force upon the nation a return to "biblical family values." We can only hope that
it never happens.

Solving the Jeremiah Problem


Dr. James D. Price
Mr. Till began his rebuttal with the common tactic of poisoning the pot. He tried to prejudice
the audience by referring to the prophecy under discussion as a bad example. Far from being a

Volume 1990 - 2002 Issue


Page 968 of 2049
Skeptical Review Edited by Farrell Till
bad example, the prophecy was chosen specifically because its authorship is widely accepted
by the most reputable authorities, and it is clearly dated long before the fulfillment occurred.
The date of the prophecy cannot be denied by any objective evidence--the kind of evidence
that Till allegedly demands. Those few who would presume to deny the date do so on the
basis of subjective theories. Further, the fulfillment of the prophecy is documented by
contemporary extra-Biblical historic records, the validity of which is denied by none. As I
predicted, Till indeed quibbled over some of the details, but the reader will observe that for
the most part he quibbled over details that are irrelevant.

Inerrancy Ploy: Mr. Till apparently thinks we are debating "inerrancy." No, Mr. Till, we
agreed to debate fulfilled prophecy! Remember? As I predicted, Till insisted on bringing in
the question of inerrancy. Of course, the consequences of this debate will have some bearing
on the question of inerrancy. But this debate is on the topic of fulfilled prophecy, not
inerrancy. To demonstrate the existence of fulfilled prophecy, all that must be done is to meet
the four requirements that Till outlined in his rebuttal. These requirements do not include a
proof of the inerrancy of Scripture. I have demonstrated that the prophecy under discussion
was accurately fulfilled, and I did so without invoking an appeal to the inerrancy of Scripture.
After all, it is possible for a prophecy recorded in the Bible to be accurately fulfilled without
the Bible being necessarily inerrant in nonrelevant details. The inerrancy of the Bible depends
on the truth of fulfilled prophecy, but fulfilled prophecy does not depend on the Bible being
inerrant in every detail; it depends only on the truth of the relevant details of the given
prophecy. Till seems unable to understand this significant difference. At least, he will not
admit it. That is why I keep insisting that we stick to the one question at hand and not becloud
the issue by continually chasing irrelevant rabbit trails. The issue of inerrency is a non
sequitur in this debate.

Smorgasbord approach: Mr. Till likened my defense of Jeremiah's prophecy to a


smorgasbord, and asserted that he would not tolerate it. Likewise, at the beginning of this
response, I want to assure Mr. Till that I will not tolerate his "smorgasbord approach" to a
rebuttal of the prophecy under discussion. Mr. Till's approach to rebuttal is to bring up
apparent discrepancies that are irrelevant to the topic under discussion. Apparent
discrepancies indeed must be resolved individually in order to defend inerrancy, but unless an
alleged discrepancy is directly related to the topic under study it remains irrelevant to the
investigation. In this debate, the topic is Jeremiah's prophecy of Israel's 70-year servitude.
Anything not directly related to that topic is irrelevant--a non sequitur. I insist that Till limit
his rebuttal to that topic only.

Date of the prophecy: Mr. Till presented four requirements necessary for the proof of
fulfilled prophecy. The requirements are satisfactory, and I have met them in the proof of the
fulfillment of Jeremiah's prophecy of the 70-year servitude of the Jews.

Mr. Till began his rebuttal by challenging the validity of the date Jeremiah attached to the
prophecy. He did so by hypothesizing that the date was changed by a late redactor to make the
prophecy appear fulfilled. Since the burden of proof lies with the one who challenges the
validity of the date given in the Book of Jeremiah, it is interesting to note that Till provided
no convincing objective evidence that such textual manipulation took place. Indeed, I will
show that Jeremiah's date is consistently supported by all textual traditions from diverse

Volume 1990 - 2002 Issue


Page 969 of 2049
Skeptical Review Edited by Farrell Till
sources at the earliest dates. All the objective evidence consistently supports the validity of
Jeremiah's date.

Mormon ploy: Mr. Till digressed from the topic by introducing a lengthy ploy about
potential religious fraud. Mr. Till indeed should apologize for such a lengthy rabbit trail. This
is an old illustration that he has used before in other debates. Surely his readers are becoming
bored with such useless repetition. As in the previous debates, this example has no direct
relationship with the topic under discussion. The fact that some Mormon elders evidently
falsified records does not give Till the right to conclude that such falsification has necessarily
happened in all religious literature. This is a logical fallacy known as a hasty generalization.
Its falsity is demonstrated by the following fallacious analogous generalization: (1) The
Piltdown Man was proven to be a hoax; therefore, all human fossils are hoaxes; (2) Some
atheists are known to be liars; therefore, all atheists are liars. Let Till refrain from using such
fallacious generalizations. Each case must be evaluated on its own merits. According to Till's
theological presuppositions it may seem likely that Jeremiah's date is a late redaction. But
subjective speculations are not objective facts, and the less likely event does actually happen
at times. Let him produce convincing objective evidence that Jeremiah's date is fraudulent, or
let him refrain from unsubstantiated slander and accept the date as accurate by default.

More alleged fraud: Twice again Mr. Till digressed from the topic at hand, by alleging,
without any verification, that two Biblical prophecies were fraudulent. I am not impressed
with Till's naive belief that common sense demands that such predictions are impossible.
Common sense merely suggests that such predictions are unlikely. But common sense does
not demand that unlikely events never happen. History is full of unlikely events, and everyone
with common sense knows that. Of course it is possible that the Biblical author made up these
stories to accomplish some theological purpose. On the other hand it is also possible that the
author recorded as history what he found written in the court records he used as his sources;
after all, that is what common sense suggests was the author's usual practice for most of the
history he wrote. What is the objective evidence that these prophecies are hoaxes? None! Till
merely appealed to his subjective theological presupposition that fulfilled prophecy is
impossible. According to him it must be a forgery because his presupposition demands it,
regardless of the evidence. Circular reasoning, pure and simple! In a scientific investigation
one must prove the null hypothesis; it is insufficient to merely assert it.

Mr. Till complained that I do not like that kind of reasoning. Of course, I don't like that kind
of reasoning because it is fallacious, and I have clearly pointed this out to Till. Jeremiah's
prophecy is not without "extrabiblical records from contemporary times to corroborate" its
validity. With such reasonable objective evidence in support of Jeremiah's prophecy, Till's
appeal to this presupposition amounts to simplistic rationalization. I am not impressed.

Now, even if these prophecies were forgeries--and there is no objective evidence that they
were--it would have no direct bearing on the present debate. Jeremiah's prophecy must be
evaluated on its own merits, and the evidence for its validity is of a different stripe. We know
who wrote the prophecy. We know when he lived. The date he appended to the prophecy is
supported by strong textual and historical evidence. Those who question the validity of the
date do so on the basis of subjective theories, not on the basis of objective evidence. So Till is
guilty of another logical fallacy--false analogy.

Volume 1990 - 2002 Issue


Page 970 of 2049
Skeptical Review Edited by Farrell Till
The Septuagint problem: Mr. Till introduced what he called "the Septuagint Problem" in
which he denied the validity of the text of the Book of Jeremiah based on the witness of the
Greek translation known as the Septuagint. Here the discussion became serious, and Till
committed three serious blunders: (1) exaggeration, (2) misinformation, and (3) withholding
information.

Exaggeration: Till tried to impress his readers with the alleged gross corruption of the
Biblical text by parading before their eyes a large number (6,000) of textual variations that
exist between the Hebrew Masoretic text and the Greek translation known as the Septuagint.
Such a large number sounds serious until it is placed in proper perspective with the size of the
Old Testament text. That number amounts to about 6 variations per chapter, about one
variation in four verses, or about one variation in 100 words. Now, since Till himself admitted
that "many of these variations are minor," and since most of them have little effect on the
significant facts of the message, this leaves only about one variation per chapter that may
amount to a problem. But Till would have his readers to believe that these textual variations
compromise the integrity of the text. Actually, the presence of these variations (and others
from other textual sources) enables textual scholars to determine the correct reading of the
text in all but the most difficult places.

Next Till paraded the book with the most textual variations (Samuel) as an example,
supposedly to give the readers the impression that all the books of the Old testament
experienced the same degree of textual variation. While it is true that the text of Jeremiah has
a greater proportion of variations than most of the other Old Testament books, it is not true
that such variations compromise the integrity of the book, and in particular, the text of the
prophecy under discussion. This is discussed later.

Misinformation: Next Till engaged in misinforming his readers. He may be guilty of not
being well informed himself and of not doing proper research, or he may be guilty of
deliberately misrepresenting the facts. I give him the benefit of the doubt and assume that he
is just out of touch with the real world. In the first place, Till asserted: "The oldest complete
version of the Masoretic text dates from the late 9th century A.D." Here he must mean a copy
of all the books of the Masoretic text bound together in one volume. Otherwise, the statement
is inaccurate. There are individual manuscripts of portions of the Masoretic text (including the
Prophets) that date earlier than the 9th century. In fact, before about the 2nd century A.D.,
none of the books of the Old Testament were bound together in a common volume. Instead,
each book existed as an independent scroll. Still today the Hebrew Bible used in synagogues
is in the form of handwritten scrolls. Further, Till seems ignorant of the fact that numerous
scrolls of nearly every book of the Old Testament found in the caves of the Dead Sea, dating
from 1st century A.D. to as early as the 4th century B.C., verify that the Masoretic text in all
its essential details was in existence at that time. The main difference between the early form
of the Masoretic text found in the Judean desert and the form found in medieval manuscripts
is that the later texts contain vowel signs, accent marks, and marginal notes. Actual textual
variations between the early form of the Masoretic text and its medieval form are of very
minor importance. Emanuel Tov, one of the leading experts on textual criticism of the Old
Testament, stated:

Volume 1990 - 2002 Issue


Page 971 of 2049
Skeptical Review Edited by Farrell Till
The Masoretic Text (M), sometimes called the "received text," is strictly speaking a medieval
representative of an ancient text of the Bible which already at an early stage was accepted as
the sole text by the central stream of Judaism. As a result, the slightly different forms of this
text were copied and circulated more than other texts. The final form of this text was
determined in the Middle Ages, and it is that form which is usually called the Masoretic Text,
while earlier forms found in the Judean Desert, lacking the later vocalization [vowel points]
and accentuation, are called proto-Masoretic (Textual Criticism of the Hebrew Bible, Fortress
Press, 1992, p. 19).

For the Book of Jeremiah six scrolls exist from that era, some dating in the 2nd century B.C.

Next Till referred to the Greek Septuagint, translated in the 3rd century B.C., as though
manuscripts of that text now exist from that time. Thus he alleged that "there is a span of
about 12 centuries between these two versions. This many variations is a rather clear
indication that substantial altering of the Hebrew text occurred between the 3rd century B. C.
and the 9th century A. D." Thus he presents the picture that careless copying together with
some deliberate altering of the text took place over a 1200 year period. The truth is: the Dead
Sea Scrolls verify that the Septuagint text and the Masoretic text existed side by side at that
time. So whatever differences exist between the two text traditions, they originated sometime
before the days of the Dead Sea Scrolls. Since the days of the Dead Sea Scrolls the Masoretic
text has remained essentially the same, except for very minor variations.

Withholding information: Till either withheld information important to this discussion, or he


is ignorant of such information. He was eager to introduce the Greek Septuagint translation of
the 3rd century B.C. because it contains textual variations that he thinks disproves my
position. However, he was careful not to mention the Aramaic translation which dates from
the 4th or 5th century B.C. and corresponds with the Masoretic Text in the Book of Jeremiah.
Two other ancient translations--the Syriac translation (2nd century A.D.) and the Latin
Vulgate (4th century A.D.) also validate the antiquity of the Masoretic Text. Concerning the
Aramaic translation, Ernst Wurthwein, another leading authority on the text of the Old
Testament, stated: "The Jewish tradition associating it with Ezra (cf. Neh. 8:8) may well be
correct" (The Text of the Old Testament, Eerdmans, 1979, p. 75). Common sense dictates that
a translation would have been made from an authoritative Hebrew text; therefore, the
Masoretic Text from which the Aramaic translation was made must have dated near the time
of the return from the Babylonian Captivity, or earlier. Thus, while Till would have the
readers to believe that the Masoretic text of Jeremiah is very late, and that long after Jeremiah
was dead the text was fraudulently altered to make it appear to be a fulfilled prophecy; on the
other hand, the objective evidence contained in ancient manuscripts and ancient translations
demonstrates that the date (and all the other essential details of the prophecy) was soundly
intact very shortly after Jeremiah's death. Later I show why common sense demands that the
date of this prophecy (and other details) must have come from the hand of Jeremiah himself.

Textual variations: Next Mr. Till introduced evidence from the popular magazine America
that the Book of Jeremiah has a diverse textual history. This explains why Till evidently is ill
informed. He seems to rely on articles in popular magazines that are necessarily limited and
slanted, rather than on scholarly publications that treat the subject in depth. Of course, it is
important that two of the six Qumran manuscripts of Jeremiah contain the Hebrew text

Volume 1990 - 2002 Issue


Page 972 of 2049
Skeptical Review Edited by Farrell Till
tradition that lies behind the Greek Septuagint. However, it is equally important that the other
four manuscripts contain the early form of the Masoretic Text. Common sense dictates that
the textual sources behind all six of these manuscripts are equally ancient.

It is true that the Masoretic Text of Jeremiah contains a number of short sections not found in
the Septuagint, and that its material was organized according to a different pattern. However,
the other differences between the texts consist primarily of the expansion of proper nouns and
formulae on the basis of context. Tov referred to the two text traditions as Edition I and
Edition II (page 321). Because the objective textual evidence points to a very early date of the
Masoretic tradition, it is likely that both editions came from the hand of Jeremiah, apart from
minor textual variations due to later influences. The first (shorter) edition was likely produced
in Egypt soon after Jeremiah was taken there by the Jews, and it became the text used by the
Egyptian community of Jews which eventually produced the Septuagint.The second
(expanded) edition was likely produced by Jeremiah just before his death; this edition was
taken to Babylon by Baruch where it became part of the Masoretic tradition.

Septuagint evidence: Next Mr. Till attempted to show from Septuagint evidence that
Jeremiah's 70 year prophecy was seriously different in the Septuagint and the Masoretic Text.
Till's original quibble was about the validity of the date of this prophecy. The reader is invited
to examine the text of the Septuagint and of the Masoretic Text presented by Till to see
clearly that both texts give the same date in two complementary forms: (1) in the 4th year of
Jehoiakim, and (2) 23 years after the 13th year of Josiah. The Masoretic Text contains
Jeremiah's added comment that Jehoiakim's 4th year corresponded with the 1st year of
Nebuchadnezzar--a detail that Jeremiah himself would have known. Both text traditions also
state that the land of Judah would be attacked by a nation from the north, that the land of
Judah would be made desolate, and that after 70 years God would judge the northern nation
and the Jews would return to their land. That is, both text traditions are in full agreement with
respect to all the essential details of the prophecy.

Now, let's use some common sense right here, not Till's brand (because it is tainted with
subjective theories), but common sense that deals with objective evidence. Suppose, for a
moment, that it was true that long after Jeremiah was dead some redactor changed the date in
the text in order to make it appear as though the prophecy was fulfilled. Till seems to imply
that the change took place in the middle ages sometime before the 9th century, but anyone
with common sense can see the flaw in that conclusion. Let's suppose it was changed
sometime shortly after the return from the captivity, say in the days of Ezra. But by that time,
Jeremiah's book would have been copied hundreds of times and circulated among the Jewish
communities throughout the Middle East, in Egypt, Babylon, Assyria, etc. Now here is an
interesting question for common sense evaluation: How did that fraudulent redactor manage
to change all the textual exemplars and all the copies in all those places so that all textual
witnesses in all forms, including manuscripts, translations, and commentaries (such as Daniel
and Josephus) have the same readings? Common sense consideration of Till's hypothesis
reveals how ridiculous such an idea is, and it leads one to conclude that the only way all
extant witnesses to the text could agree on these details would be that they were from the
hand of the original author, Jeremiah. This is a widely accepted law of textual criticism: when
all the textual witnesses agree, the reading is original. Those who would question the validity
of this date would not dare to do it on the grounds of the objective textual evidence, but must

Volume 1990 - 2002 Issue


Page 973 of 2049
Skeptical Review Edited by Farrell Till
resort to subjective theories. Therefore, common sense demands that the date of this prophecy
is authentic on the basis of the objective evidence.

Now what has been said about the date also holds true for the other essential details of the
prophecy; all textual witnesses agree. The main detail that the Masoretic Text adds is an
expansion of the identity of the nation from the north and its king. In Jeremiah's last years he
would have known more fully these details, and that would have been a reasonable expansion
to make. Such an expansion made no essential difference in the prophetic message, and did
not make the passage have an appearance of a fulfilled prediction that was not already in the
earlier edition. Verse 14 is obviously an added parenthetical summation that adds no new
elements to the prophecy.

Quibble over Nebuchadnezzar: Next Mr. Till introduced a lengthy quibble over the
introduction of Babylon and King Nebuchadnezzar into the Masoretic Text as an expansion of
the identity of the northern nation and its king. In this lengthy quibble, Till made much of the
fact that Jeremiah expanded the identity of the northern nation to include the fact that the
nation was Babylon and that its king was Nebuchadnezzar. But everyone familiar with the
history of Jeremiah and the last days of the kingdom of Judah knows that in the last years of
Jeremiah that identity was well known. Jeremiah lived to see the destruction of Jerusalem and
the captivity of the Jews by Nebuchadnezzar, king of Babylon. He was the most likely one to
have expanded the prophecy to include those details. This expansion added nothing to the
predictive aspects of the prophecy. Till's quibble is just that, irrelevant haggling.

Seventy-year chronology: Mr. Till objected that "it isn't chronologically possible to establish
a seventy-year exile for the Judeans who were taken into captivity during the reign of
Nebuchadnezzar." Mr. Till has made another of his usual blunders at this juncture. He has
mistaken vassalage for captivity. Jeremiah's text says "these nations shall serve the king of
Babylon seventy years." While it is customary to refer to this period as a captivity or an exile,
because most of it includes the time in which the Jews were in exile as captives, yet the text
says "serve" not "be captives" or "go into exile." Technically, the text refers to vassalage, not
captivity. Judah and the surrounding nations became vassals of Nebuchadnezzar in 605 B.C.

The Eerdmans Bible Dictionary, the same dictionary Till cited as an authority on the dates of
the Exile, states the following regarding the reign of King Jehoiakim: "After the battle of
Carchemish (605), in which Nebuchadnezzar defeated Neco, Jehoiakim became a Babylonian
vassal (Jer 46:2) ["Jehoiakim," p. 559; see also Josephus, Antiquities, 10:5.2]. That was the
year Nebuchadnezzar first besieged Jerusalem, confiscated some of the Temple treasures, and
deported a number of young royal hostages to Babylon (Dan. 1:1-4; Josephus, Antiquities,
10:10.1). That was the year in which Jeremiah proclaimed the prophecy under discussion--the
year the servitude of Judah began.

As a secondary remark, the alleged difficulty that Mr. Till pointed out here is evidence to
support the validity of the date Jeremiah attached to this prophecy. If the date had been
supplied by some late redactor, as Till hypothesizes, the redactor surely would have selected a
date that would have been free from difficulties. He would have selected a date that fulfilled
the seventy years without possible question. As it stands, the date bears all the marks of
authenticity.

Volume 1990 - 2002 Issue


Page 974 of 2049
Skeptical Review Edited by Farrell Till
536 verified: Mr. Till objected to my date of the Jews' return from the Babylonian captivity in
the year 536 B.C. He did so on the grounds that Cyrus conquered Babylon in 539 and that the
Jews must have returned in 538. However, in this he displays an ignorance of the cultural
practice among the Babylonians and Persians for dating regnal years of their kings. Among
these ancient neareastern people, the regnal year of a king was counted from the New Year's
day following the kings ascension to the throne. This is different than the practice of western
cultures by which Till erroneously judged the present situation. Cyrus conquered Babylon on
October 16, 539 B.C., and thus became the sovereign of the Medo-Persian Empire. However,
the following New Year's day was not until March 24, 538. So his first regnal year extended
from March 24, 538, to March 23, 537 B.C. Sometime within that year, Cyrus issued the
decree for the captive nations to return home (2 Chron. 36:22). It is likely that the decree was
issued late in the year, because administrative duties would have been heavy for the months
immediately following the conquest of Babylon; less important details, like the affairs of
foreign captives, would naturally be postponed. Taking into account the slow pace at which
government business took place, the amount of time required to summons, assemble, and
organize a large company of returnees, and the time for such a large group to travel the long
distance from Babylon to Jerusalem, it is reasonable to expect that the convoy did not arrive
in Judah until some time in 536.

Lack of precision: Mr. Till objected that my explanation of the 70 years of servitude was not
established with rigorous precision. In this he demonstrates his ignorance of the hermeneutics
of historic research. Reasonable common sense expects that an ancient text should be
interpreted according to how it would have been understood by the people to whom it was
addressed, not according to 20th century scientific precision. From extant contemporary
literature from that period, I provided two examples of how the ancient Jews understood the
70 years of Jeremiah's prophecy. In both cases, the Jews regarded the elapsed time to be
consistent with their understanding of 70 years. In insisting on 20th century scientific
precision, Till is neither reasonable, nor is he exercising common sense.

Irrelevant quibble: Mr. Till took exception to my interpretation of Jeremiah 25:15-38 as


"figurative elaboration of God's judgment of Judah and the surrounding nations in poetic
terms." Likewise, he objected to my statement that "(s)uch figurative language is not to be
interpreted beyond the reasonable way Jeremiah's ancient readers would have understood it."
Of course, this statement is in full harmony with the standard laws of hermeneutics. Till wants
to use an ultra-literal interpretation on ancient figurative language with 20th century scientific
precision. He knows quite well that such interpretive techniques are contrary to the commonly
accepted hermeneutics of the historical method.

The detail that completely destroys Till's critique is that in the first edition of Jeremiah's book
as reflected in the Septuagint version, the version Till used as the authority for the early form
of Jeremiah, verses 15-38 appear in chapter 32, an entirely different section of the book. In
that chapter, the oracle has its own heading in the Greek text. This indicates that verses 15-38
and verses 1-14 constitute completely different oracles-- independent prophecies. Till
conveniently withheld this information from his readers. Whatever reason Jeremiah had for
juxtaposing the two oracles in his second edition, it was not because the two constituted a
single prophecy. Rather, it is likely that he did so because the two oracles have similar
elements. Thus, whatever Till's complaints about the oracle of verses 15-38 may be, they are

Volume 1990 - 2002 Issue


Page 975 of 2049
Skeptical Review Edited by Farrell Till
completely irrelevant to the prophecy in verses 1-14, and must be treated in a separate
discussion. They have no bearing on the veracity of the prophecy under discussion.

Prophecy was fulfilled: As expected, Mr. Till quibbled over some of the details of Jeremiah's
prophecy of the 70 year servitude of the Jews. However, his quibble over the validity of the
date Jeremiah attached to the prophecy turns out to be just that, nothing but empty haggling.
All the objective evidence indicates that date must have come from the hand of the original
author, not from a late redactor. All the objective evidence supports the validity of the number
70 in the text, and the details foretold about the servitude. The fulfillment of the prophecy is
confirmed by contemporary secular evidence. Till's quibble over rigorous precision regarding
the duration of the 70 years is unreasonable and inconsistent with the hermeneutics of historic
research. Finally, Till's quibbles over the details of verses 15-38 turns out to be completely
irrelevant. Thus Jeremiah's prophecy of the 70 year servitude of the Jews has been verified as
fulfilled, and Till's theological presupposition that fulfilled prophecy is impossible has been
demonstrated to be fallacious.

(James D. Price, Ph.D., Professor of Hebrew and Old Testament, Temple Baptist Seminary,
Chattanooga, TN 37404; e-mail drjdprice@aol.com)

Whatever Happened to the Burden-of-Proof


Factor?
Farrell Till
Space constraints in this issue will require me to divide my response to Dr. Price's second
defense of Jeremiah's prophecy into at least two parts. On page 15 of this issue is a letter from
Dave Matson that helps me to abbreviate a response to Dr. Price's persistent objections to my
insistence that extraordinary claims, such as the one he is defending, require exceptionally
good, and even extraordinary, evidence. Matson's letter and my article "The Nature of the
Claim" (TSR, March/April 1977, pp. 10-11) provide more than reasonable proof that Dr.
Price's constant complaining about an "anti-supernatural bias" is merely a straw man he has
set up to beat on and divert attention from the obvious fact that his case is weak. I have
repeatedly asked Dr. Price to prove me wrong in my suspicion that he has the same so-called
"anti-supernatural bias" about everything but the Bible, and he has refused to address the
issue. Does he, for example, accept the miracle claims of ancient nonbiblical documents? If
not, why not? Does he have an "anti-supernatural bias" toward all miracle claims except
biblical ones? If so, what is his rationale for this highly selective bias? If he expects to have
any credibility with rational people, he needs to tell us. My article previously cited ("The
Nature of the Claim") gives him an excellent opportunity to address this issue. In it, I listed
several specific miracle claims from various ancient nonbiblical documents. I challenge Dr.
Price to read this article and tell us which, if any, of these claims he accepts. If he accepts
none of them, what are his reasons for the rejections? Until he addresses this issue, it is going

Volume 1990 - 2002 Issue


Page 976 of 2049
Skeptical Review Edited by Farrell Till
to hang around his neck like an albatross all through the debate on this subject. So I will put
the question to him pointblank. Dr. Price, do you accept the miracle claims in the Book of
Mormon, the Hindu Vedas, the Qur'an, Josephus's Wars of the Jews, Suetonius's Twelve
Caesars, and such like? If not, why not? If it is reasonable to question such claims as these,
why is it unreasonable to question comparable claims in the Bible on the same grounds that
have led you to reject nonbiblical miracles? My position is that exaggerations, deliberate
falsifications, mistaken impressions, etc., in documents of prescientific times were far more
likely than the suspension of recognized natural laws? What is so wrong about that position?
We need an answer, and continual harping about an anti-supernatural bias is not an answer. It
is an indication of a far more unreasonable bias.

Other straw men: In addition to his complaints about an "anti-supernatural bias," Dr. Price
has set up other straw men to give a false impression that he is debating the issue that divides
us. He accused me of "poisoning the pot" by entitling my rebuttal article "A Bad Example of
Prophecy Fulfillment," but if he will consult the textbooks used in writing courses at the
seminary where he teaches, he should see that my title did exactly what writing experts say
that a good title should do: it encapsulated the central theme of my article. As any writing
instructor knows, the title is an integral part of an essay or article, so if Dr. Price is going to
set a writing standard that would prohibit any statement in a title that would inform readers of
the author's opinion on the subject of the article, consistency would demand that nothing be
said anywhere in the article that would express the author's opinion. Such a standard would
make discussion of controversial issues impossible, so his complaint is without merit.

He accused me of "continually chasing irrelevant rabbit trails" by referring to the biblical


inerrancy doctrine, but this too is a straw man that was set up to divert attention from what I
was actually arguing. He had predicted that I would "respond by quibbling over some minute
details that are irrelevant to the main issue--the fulfillment of the central details of one
specific prophecy" (March/ April, p. 4), and I replied to this by arguing that in a prophecy-
fulfillment claim, there can be no such thing as "minute details," for if a genuine prophecy,
inspired by an omniscient, omnipotent deity, was ever made, it would necessarily be fulfilled
in every single detail, including the "minute" ones. To argue otherwise would reduce
"divinely inspired" prophecy to a hit-and-miss proposition somewhat like the tabloid
"prophecies," which sometimes seem to hit on some details but are incorrect in others and
most often incorrect in everything.

I further said that "(i)f Dr. Price is going to claim prophecy fulfillment, then he is obligated to
defend all details of the alleged prophecy and not just those that he considers `central,' for if
Jeremiah was indeed inspired by an omniscient, omnipotent deity, we have every right to
expect fulfillment in all details and not just those that Dr. Price considers `central'" (p. 4,
emphasis added). I then compared Price's argumentation on this point to the way biblicists try
to defend inerrancy by accusing skeptics of "quibbling over minor details when [inerrantists]
are confronted with problems in their inerrancy defenses" (p. 4). I don't deny my belief that
biblical inerrancy is inextricably related to the issue of prophecy fulfillment. At no time,
however, did I state that the Bible would have to be inerrant before it could contain a valid
prophecy, and at no time did Dr. Price address my claim that a genuine prophecy fulfillment
would have to include fulfillment of all details and not just those that someone arbitrarily
declares "central" ones. Let's hope that Dr. Price will address this issue.

Volume 1990 - 2002 Issue


Page 977 of 2049
Skeptical Review Edited by Farrell Till
The relevance of inerrancy: Having just stated my belief that biblical inerrancy cannot be
separated from the issue of prophecy fulfillment, I will try to show Dr. Price what the
relationship is. My position is that lack of faith in a claim, especially an extraordinary claim,
is always justified when there are no good reasons to believe the claim. From beginning to
end, the Bible is filled with fabulous claims, none of which can be verified by reasonable
evidence. The Bible text, however, is a tangible existent that can be critically examined, and
when it is so examined, we find that it is riddled with inconsistencies, discrepancies, and
outright contradictions. (Space won't allow me to discuss in this article the many proofs of
biblical errancy, but if Dr. Price cares to challenge me on this, I am prepared to debate it when
this one is finished.) Verifiable flaws like these give rational readers every reason to suspect
that the many fabulous, unverifiable claims probably didn't happen. In other words, the
obvious errancy of the Bible in matters that can be verified gives probable cause to doubt the
accuracy of its fabulous claims that cannot be verified. This is a simple application of the rule
of evidence known as falsus in uno, falsus in omnibus, i. e., false in one thing, false in
everything. This rule does not mean that if one verifiable error is found in a document or the
testimony of a witness, everything else in the document or testimony is necessarily false; it
simply means that if one error is discovered, the credibility of the document or testimony is
thereby impeached, and one then has sufficient cause to question anything else in the
document or testimony.

So it is in the matter of the prophecy claim that Dr. Price is defending. It is found not just in a
book within a collection of books that contain numerous inconsistencies and discrepancies;
the specific book in which the prophecy is found is itself riddled with inconsistencies. The
fact that two very different versions of Jeremiah have long existed (as explained in my first
rebuttal article) is enough to call into question the traditional claim that this is a document that
was inspired by an omniscient, omnipotent deity, but I will address this matter and Dr. Price's
response to it later (probably in the next issue of TSR). For now, I'll limit my comments to
problems in Jeremiah that show, whether inspired or not, it is not a book that instills
confidence in the prophetic talents of its writer.

The credibility of the prophet Jeremiah: In my booklet Prophecies: Imaginary and


Unfulfilled, I presented the following example of Jeremiah's shortcomings as a prophet:
"Zedekiah to die in peace: In predicting Jerusalem's fall to Babylon, Jeremiah prophesied
that Zedekiah, the king of Judah, would `die in peace':

Thus says Yahweh, the God of Israel: `Go and speak to Zedekiah king of Judah and tell him,
"Thus says Yahweh, `Behold, I will give this city into the hand of the king of Babylon, and he
shall burn it with fire. And you shall not escape from his hand, but shall surely be taken and
delivered into his hand; your eyes shall see the eyes of the king of Babylon, he shall speak
with you face to face, and you shall go to Babylon.' Yet hear the word of Yahweh, O
Zedekiah king of Judah! Thus says Yahweh concerning you: "You shall not die by the sword.
You shall die in peace; as in the ceremonies of your fathers, the former kings who were before
you, so they shall burn incense for you and lament for you, saying, `Alas, lord!' For I have
pronounced the word, says Yahweh'" (Jer. 34:2-5).

"The prophecy was that Jerusalem would fall to Babylon and Zedekiah would be captured and
taken to Babylon but would `die in peace.' So what happened? Jerusalem was besieged by

Volume 1990 - 2002 Issue


Page 978 of 2049
Skeptical Review Edited by Farrell Till
Nebuchadnezzar for two years and finally fell. Zedekiah and all his men of war fled the city
by night but were pursued and overtaken on the plains of Jericho. Read what happened next:

But the army of the Chaldeans pursued the king, and they overtook Zedekiah in the plains of
Jericho. All his army was scattered from him. So they (the Chaldeans) took the king
(Zedekiah) and brought him up to the king of Babylon at Riblah in the land of Hamath, and he
pronounced judgment on him. Then the king of Babylon killed the sons of Zedekiah before his
eyes. And he killed all the princes of Judah in Riblah. He also put out the eyes of Zedekiah;
and the king of Babylon bound him in bronze fetters, took him to Babylon, and put him in
prison till the day of his death (Jer. 52:9-11).

"If this was Jeremiah's idea of `dying in peace,' one can only surmise how horrible Zedekiah's
fate would have been had the prophet predicted a dreadful death. Also, there is no indication
that incense was burned in memory of Zedekiah as Jeremiah had predicted. Indeed, how could
there have been with Judah defeated and its people scattered abroad" (pp. 20-21)?

Other discrepancies in the book of Jeremiah could be noted if space permitted, but this is
sufficient to show that the prophet on whom Dr. Price is betting so much in this debate had a
prophetic track record that, if he were alive today, would qualify him for inclusion in National
Enquirer's annual article about psychic predictions. Besides that, we have to wonder about the
man's intelligence if he indeed wrote the book attributed to him. In one passage, he predicted
that Zedekiah would die in peace and enjoy the respect and veneration of his people; then
later the same prophet (presumably) wrote that Zedekiah was captured and treated as recorded
in the passage just quoted. One would think that if Jeremiah had predicted a peaceful death
for Zedekiah and then subsequent events proved the prediction wrong, he would have had the
good sense to omit the prophecy about Zedekiah's death or else rewrite it to make it
compatible with the account of how Zedekiah died or even fabricate a version of his death
that would have been consistent with the prophecy. Since he chose none of these options, we
have to wonder about the man's rationality.

Jeremiah a liar by his own admission: In chapter 37, Jeremiah was accused of being a
Chaldean sympathizer (a charge that was not without some merit, as we will see later) and
was imprisoned in Jonathan's house by the princes of Judah. As superstitious kings living in
superstitious times so often did, Zedekiah wanted a prophet's prediction of the future, which at
the time looked dismal with Nebuchadnezzar's army having laid siege to Jerusalem, so
Zedekiah had Jeremiah brought to him (38:14-16). When the prophet told him that he could
save his life only by going out of the city to Babylon's princes, Zedekiah then said, "Let no
man know of these words, and you shall not die. If the princes hear that I have talked with
you, and they come to you and say to you, `Declare to us now what you have said to the king
and also what the king said to you; do not hide it from us, and we will not put you to death,'
then you shall say to them, `I presented my request before the king that he would not make
me return to Jonathan's house to die there'" (vs:24-26). The princes did come to Jeremiah and
ask about his conversation with the king, and Jeremiah "told them according to all these
words that the king had commanded" (v:27).

In other words, if Jeremiah's account of this incident is accurate, he admitted that in certain
situations he would and did lie. It is also noteworthy that in answering Zedekiah's question

Volume 1990 - 2002 Issue


Page 979 of 2049
Skeptical Review Edited by Farrell Till
about what the future held for him, Jeremiah did not tell him that if he went out of the city to
meet the Babylonian princes, all of his sons would be killed in his presence and he himself
would be blinded and imprisoned for the rest of his life. We have to wonder if Zedekiah
would have left the city if he had known this. To say the least, if Jeremiah was a real prophet,
who could see into the future, he was a bit deceptive in answering Zedekiah's question about
the outcome of Nebuchadnezzar's siege.

To summarize, then, I have identified three facts that impeach the credibility of the book of
Jeremiah: (1) the existence of two very different versions of the book gives sufficient reason
to suspect that it was edited and rewritten at least once, (2) it contains identifiable
discrepancies and inconsistencies, and (3) its author admitted that he sometimes practiced
deception and falsified information. Dr. Price, of course, will insist that these are
inconsequential "quibbles" that have no bearings on whether the 70-year prophecy was
fulfilled. "To demonstrate the existence of fulfilled prophecy," he said, "all that must be done
is to meet the four requirements that Till outlined in his rebuttal," and he further said that "the
requirements are satisfactory." One of those requirements, however, was that the prophecy
must have been made before and not after the alleged fulfillment. So is Dr. Price going to
argue that the unverifiable word of an admittedly deceptive prophet recorded only in a
document that is contradictory in places and shows obvious signs of having been edited and
rewritten is sufficient to prove that this prophecy was actually made before the fact? If so, he
has a strange concept of what constitutes reliable evidence.

What do I mean by the unverifiable word of an admittedly deceptive prophet? I mean that
there is no way for Dr. Price or anyone to prove that Jeremiah 25:1-14 (the passage containing
the prophecy in question) was written by Jeremiah in 605 B. C., the date that Dr. Price claims
for the prophecy. In fact, Dr. Price can't even prove that Jeremiah himself made this
statement. In biblical times, there were no copyright agencies or archives that registered and
kept original works, which could be consulted if questions later arose concerning the reading
of the original text and the date of its authorship. All Dr. Price has to offer as evidence are the
existing text of Jeremiah and a tradition that the entire book was written by a 7th- and 6th-
century prophet named Jeremiah. In other words, Dr. Price is asking us to believe that the
prophecy was made before the fact because an ancient book containing the prophecy says that
it was made before the fact. That is hardly sterling evidence that satisfies a criterion of valid
prophecy that even Dr. Prices says is "satisfactory."

Let's notice some additional facts about the book of Jeremiah that Dr. Price needs to consider.
At the close of the book, a reference is made to events that allegedly happened at least 37
years after the captivity began: "And it came to pass in the thirty-seventh year of the captivity
of Jehoiachin king of Judah, in the twelfth month, on the twenty-fifth day of the month, that
Evil-Merodach king of Babylon, in the first year of his reign, lifted up the head of Jehoiachin
king of Judah and brought him out of prison" (52:31). This would mean that the manuscript
was not completed until at least 45 years after "the word of Yahweh came to Jeremiah" in
chapter 25, where he made the "prophecy" that Dr. Price is so excited about. The number 45
is arrived at by calculating from the 4th year of Jehoiakim's reign and the first year of
Nebuchadnezzar's reign, which was the year that Jeremiah claimed that the word of Yahweh
came to him (25:1). As Dr. Price has already noted, the first year of Nebuchadnezzar's reign
was 605 B. C. (2 Kings 24:1-7; 2 Chron. 36:5-10). Jehoiakim continued to reign as a vassal

Volume 1990 - 2002 Issue


Page 980 of 2049
Skeptical Review Edited by Farrell Till
king for 7 more years, so the total length of his reign was 11 years (2 Kings 23:36), which
would have ended in 598 B. C. He was succeeded by his son Jehoiachin, who reigned for only
3 months, at which time (597 B. C.) he was taken as a captive to Babylon with his family (2
Kings 24:6-12). Thus, Jehoiachin's captivity began eight years after "the word of Yahweh
came to Jeremiah" (25:1), so if Jeremiah knew that Jehoiachin was released from prison in
Babylon in the 37th year of his captivity, he could not have completed his book until 45 years
after the word of Yahweh allegedly came to him and revealed the 70-year prophecy.

But the problem becomes even more complex for Dr. Price. The very last verse of Jeremiah
states that the king of Babylon gave Jehoiachin a continual allowance" every day "until the
day of his death, all the days of his life" (52:34). This expression indicates that the writer
thought that Jehoiachin's favored treatment lasted for an extended period of time, so if the
conclusion of Jeremiah was actually written by the prophet himself (or even by his scribe
Baruch), which Dr. Price will surely claim, we have no way of knowing just how many years
of the captivity had passed before this book was completed. If we assume that "all the days of
[Jehoiakim's] life" after his release from prison were no more than 10 years, this would mean
that the book of Jeremiah in its present form was not completed until 55 years into the
captivity (as Dr. Price dates it). This would put the completion of the book late enough (even
if we assume the authenticity of everything in it) close enough to the end of the captivity to
pose a conflict with the third criterion of valid prophecy in my list: The prophecy must be
made not just before an event but far enough in advance of it to make educated guesswork
impossible.

This would pose another serious problem for Dr. Price, because the book of Jeremiah does not
claim that the prophecy in question was written in the 4th year of Je hoiakim's reign (605 B.
C.) but only that the the word of Yahweh had come to Jeremiah at that time. So if the book
wasn't even completed until 45 to 55 years into the captivity, how can Dr. Price know exactly
when Jeremiah wrote the prophecy? Also, how can Dr. Price know that a prophet who, by his
own admission, lied and practiced deception would not have seen in the political climate at
that time events that indicated a softening of Babylonian attitudes toward the captives, which
would have enabled him to guess that the captivity would end soon? After all, if Jehoiakim
had been released from prison and was being favored above all the other captive kings in
Babylon, that would have certainly suggested that better times were in store for the captives.
If such a change in the political climate had occurred before the book was finished, a devious
prophet would certainly not have been above gambling on a retrojected prediction that the
captivity would last only "seventy" years. As we will see in my next article, the number 70
could have been intended not literally but only in a figurative, round or perfect number.

Many biblical scholars have identified various sections of Jeremiah that were undoubtedly
written by a Deuteronomist editor after the exile was over, but with no more space available
in this issue, I will have to discuss this point in my next article. I think unbiased readers will
find the evidence for this conclusion quite compelling. Even without that evidence, however, I
have already pointed out sufficient reason to question the in tegrity of the book of Jeremiah to
an extent that obligates Dr. Price to prove with reasonable certainty that the 70-year prophecy
was made before and not after the fact. Dr. Price used a lot of ink talking about my
responsibility to produce "objective evidence" to prove that the prophecy was not written or
altered in any way after the fact, but just who has the burden of proof in this matter?

Volume 1990 - 2002 Issue


Page 981 of 2049
Skeptical Review Edited by Farrell Till
Dr. Price is the one who is making an extraordinary claim, so he is the one who must prove
that the 70-year prophecy meets the criteria that he says are "satisfactory." He, then, must
establish beyond doubt that this prophecy was actually made 70 years before its fulfillment.
Dr Price seems to think it is my responsibility to produce "objective evidence" to prove that
the prophecy was not written or altered in any way after the fact, but the burden of proof in
this matter doesn't rest on me. He is the one who is making an extraorfdinary claim, so he is
the one who must prove that the 70-year prophecy meets criteria that he says are
"satisfactory." He, then, must establish beyond doubt that this prophecy was actually made 70
years before its fulfillment. We won't settle for anything less than unimpeachable evidence.

Holy Bible by Anonymous: A Book Review


Judith Hayes
Not for the faint of heart, the Bible is a very long read. Almost 1,300 pages of tiny print, it is a
mixture of fierce, bloody battles, scores of twenty-generations-long, mind-numbing
genealogies, and quite a bit of indecipherable, symbolic secret code of some sort. There is
also a pinch of poetry thrown in for flavor. All in all, though, it is for the most part boring, too
often frightfully violent, and at all times very confusing. It passeth human understanding.

I was unable to track down the actual author(s) of the book. Some of the sections have titles
like "Mark" and "Joshua," but nowhere could I find who these people were or where or when
they had lived. This is why authorship must remain anonymous.

The book is divided into two main sections, the Old Testament and the New Testament,
"testament" apparently meaning that the authors were "testifying" to the contents of the book.
The problem, though, is that most of these writers never claimed to have witnessed,
personally, any of the described events. So I can't really say if this is a work of fiction,
nonfiction, or both.

For example, several of the earliest sections, for reasons unclear to me, are believed to have
been written by someone named Moses. However, these sections always refer to Moses in the
third person--Moses did this, and Moses said that--and there is also an account of Moses' own
death. Since the personal pronoun "I" is almost universally used to describe personal thoughts
and experiences, and since it is impossible to write an account of your own funeral, we can be
fairly sure that whoever wrote these sections was not someone named Moses.

I encountered the same problems in the New Testament. A man named Jesus supposedly did
all manner of astonishing things. Yet not one of the authors ever claimed actually to have seen
any of those astonishing things personally. (This was a disturbing theme throughout the whole
book.) Obviously, some unknown, unnamed third parties had to have told the writers about
these feats by Jesus. But this forces the reader to wonder why the firsthand witnesses weren't
moved to put pen to parchment and describe what were clearly wonders to behold.

Volume 1990 - 2002 Issue


Page 982 of 2049
Skeptical Review Edited by Farrell Till
As an example, Jesus supposedly performed miracles (such as literally walking on water),
healed very ill people with only a few uttered words, and, most astounding of all, came back
to life after being dead for about two days! These are great wonders indeed, but with the
glaring omission of any firsthand accounts, the reader will soon find himself doubting all of it.
I, for one, find Agatha Christie's murder mysteries far more likely to depict real events.

Discerning readers usually dismiss secondhand accounts of anything, preferring to get it


straight from the horse's mouth, like a good reporter. But I'm afraid the Bible cannot be called
good reporting. No newspaper editor worthy of the name would run stories such as these
without reliable, confirming sources. This book provides none.

So, the reliability of the Bible as history is almost nil, even though undisputed nations and
leaders are mentioned. The fact that historically correct names and places are often used in the
Bible is the basis for the argument that the entire book is valid. However, this argument is
unconvincing because anyone could write an account of, say, a flying saucer landing on a
rooftop. Just because the writer might place this improbable event against the backdrop of a
correctly depicted 1970s USA, amid the swirling Watergate scandal, would not authenticate
the saucer landing. The Bible suffers from this same credibility problem.

I found what I think are far too many grisly massacres and murders, described in gruesome
detail, in the Bible. This book should definitely carry a warning that it is not suitable for
children. I could see no point in describing such bloody details other than to claim bragging
rights for an allegedly mighty group of warriors. But this brings up another problem.

So many battles described in the Old Testament include impossibly large numbers of soldiers.
In the section named "Judges" (8:10), there is a battle in which 120,000 Midianites are killed,
and in the section named "2 Chronicles" (13:17), 500,000 men were killed. And there are
dozens more just like this. These numbers are difficult to accept, and for a very simple reason.
In an arid region such as Palestine (Israel), the supply lines for such massive armies would
have been impossible to maintain. That arid, scrubby territory could never have produced
enough food to maintain hundreds of thousands of soldiers, let alone the civilian population as
well. Even if bountiful Egypt (the Israelites' sworn enemies) had agreed to supply those
armies, how could the supplies have reached them? And how did they reach them in any
case?

Even with a complex railroad system, which these soldiers did not have, or motorized
vehicles, which they did not have, such enormous armies could never have been kept
supplied. Yet we are asked to believe that battlefields of nearly a million soldiers were able to
be fielded, in an arid climate, using only pack mules, horses, and camels. This reviewer found
such an untenable scenario impossible to believe. So, although some of the biblical battles
may actually have taken place, the numbers mentioned are a human impossibility. And this
brings up another problem.

Many people claim that this book is inerrant somehow, that it is the divinely inspired book of
God. (There is worldwide disagreement as to the name and nature of "God," but that's another
problem for another time.) If this is true, then everything I've written thus far should not be
possible. I've checked and rechecked the meanings of "inerrant" (incapable of error; infallible)

Volume 1990 - 2002 Issue


Page 983 of 2049
Skeptical Review Edited by Farrell Till
and "divine" (superhuman), and I am at a loss to understand why anyone would believe the
Bible is either. In addition to the problems I've already mentioned, the Bible is often
contradictory, often silly (with talking animals) and always confusing.

Moreover, the various groups claiming inerrancy do not agree with each other as to the
meaning of this book, which makes their (contradictory) claims of inerrancy even more
baffling. For example, one group claims that the Bible teaches, without a doubt, that birth
control is a sin. Countless other groups disagree. One group asserts that the Bible is clear in
forbidding blood transfusions. Countless others disagree. One group claims that the Bible was
"expanded" when God inspired one Joseph Smith to write an addendum. Countless others
disagree. And so on. With such irreconcilable disagreements still raging about the meaning of
the Bible, "inerrant" and "divinely inspired" must be ruled out.

However, setting aside historical accuracy and inerrancy, how does the book measure up in
prose, style and storylines? Well, it's a mixed bag. There are some lyrical elements that are
pleasing to the ear when read aloud. These are especially pleasing when read in the
Elizabethan English of the King James Version, which is still preferred by most of the book's
adherents. However, when weighed against the graphic violence, the inconsistencies, the
vagueness of authorship and incomprehensibility of large sections of the book (such as the
entire section named "Revelation"), the Bible is not worth the hundreds of hours required to
plow through it. In this reviewer's opinion, almost anything on Public Television is a far better
investment of one's time.

(Judith Hayes, P. O. Box 77, Valley Springs, CA 95252; e-mail jhayes@goldrush.com)

EDITOR'S NOTE: The word "testament" can also mean "will" or "covenant," which is
probably the sense it was intended in calling the two divisions of the Bible the Old and the
New Testaments. However, this does not materially affect Ms. Hayes's point about many
biblical writers' not having witnessed what they wrote about, because, as she noted, much of
what was written in the Bible wasn't and couldn't have been personally experienced by the
writers. Thus, there is no good reason to believe that their "testimony" was any more reliable
than other history that was passed by word of mouth through several generations before
finally being written down.

ABOUT THE AUTHOR: For almost three years, Ms. Hayes was a regular columnist in
Freethought Today, published by the Freedom From Religion Foundation, Inc. Her article
"Let Us Pray," originally published in Freethought Today was reprinted in the the Spring
1995 issue of The Skeptical Review. She currently has a monthly column on the Internet, The
Happy Heretic, which can be accessed on the Secular Web (http://www.infidels.org). She is
also a senior writer for The American Rationalist. In God We Trust: But Which One? was
published in 1996, and she is presently working on a second book, His Holiness?, which will
trace the history of the papacy.

Copyrights

Judith Hayes copyrights all of her articles, but she agrees to allow all of her articles in The
Skeptical Review to have the same reproduction rights accorded to all materials that we

Volume 1990 - 2002 Issue


Page 984 of 2049
Skeptical Review Edited by Farrell Till
publish. Skepticism, Inc., grants the right to reproduce its materials for local distribution and
postings on the internet provided that nothing is marketed for profit and that the names and
addresses of the authors are noted on all copies.

Schmitt Replies to Wasserman


Wilhelm E. Schmitt
In the January/February 1997 Skeptical Review, one Joel Wasserman commented on my
article dealing with alleged difficulties in Matthew 27:9-10, published in the
September/October 1996 edition. In his response, he accused me of merely looking up "a few
Hebrew/ Greek words in Strong's concordance" in order to "make it seem" [as if] I am an
expert. Perhaps Mr. Wasserman will explain how I was able to do this in view of the fact that
I do not own a Strong's nor did I have a copy available when I composed the article in
question.

The Hebrew terms referenced in my article are given according to Dr. C. D. Ginsberg's
Massoretico-Critical Text of the Hebrew Bible. The transliteration of the Hebrew words
follows a system generally approved by oriental scholars, which may differ from that favored
by Mr. Wasserman. Also, it is common practice to give Hebrew words not in the inflection
found in the text, but in their root form. This makes it easier to locate them in the lexicons.

Regarding the significance of the Septuagint, competent students of Semitic languages often
can determine, with a high degree of probability, what the Hebrew word must have been that
the Septuagint translators were looking at, from the Greek word or words used to translate
them. It was this very technique that enabled Champollion to "crack" Egyptian hieroglyphics
by studying the Rosetta Stone, which had the same message inscribed in hieroglyphics,
demotic, and Greek characters. That is why we should "care about what the Septuagint says,"
and also all other ancient documents that bear on Biblical texts. One never knows when even
an obscure source will shed light for scholarship.

In his disposition to focus on details of transliteration and vocabulary, Mr. Wasserman lost
sight of the principal points I raised: (1) Matthew stated that the prophecy was "spoken" (not
written) by Jeremiah. Matthew's general practice of using the word spoken in the sense of
written is an example of a literary figure of speech known as "metonymy of cause," but it
does not necessarily follow that this is the case in 27:9-10, where it is conceivable that
Matthew literally meant "spoken." When two interpretations of a passage are equally possible,
one of which causes a problem while the other does not, who has the right to insist on the one
that creates the problem? (2) It is possible that Matthew quoted Jeremiah's spoken words and
interjected other portions from the Old Testament by way of parenthetical explanation. These
are not to be confused with the quoted words. This is another literary device known as
"parenthetical interjection," and in my article I gave three possible examples. Regardless of
one's interpretation of the various statements in Zechariah 11:12-13, the fact remains that the

Volume 1990 - 2002 Issue


Page 985 of 2049
Skeptical Review Edited by Farrell Till
two considerations above obviate the alleged difficulty. Only by forcing acceptance of an
alternate interpretation that is guaranteed to create the difficulty can the skeptical view be
enthroned.

It is also regrettable that Mr. Wasserman was so quick to question my motives and accuse me
of plagiarism and pompous pretense. Such a spirit does not minister to attainment of truth,
and it interferes with true scholarship.

(Wilhelm E. Schmitt, 4500 West Old Shakopee Road, Bloomington, MN 55379.)

Conspicuous By Its Absence


Yoel Wasserman
In response to Wilhelm Schmitt's article, an article in which he seemed to claim some
knowledge of the Hebrew language and in which he at times incorrectly threw around a
smattering of Hebrew words, I gave very good linguistic and idiomatic proof that the word
yotser (which the author of Matthew translated as "potter" when misdirecting New Testament
readers to Jeremiah in reference to a misquoted verse that actually appears in Zechariah
11:12-13) really means "treasurer." In this article, I will reaffirm my reasons for claiming this,
and I will give Biblical references to an ecclesiastic "treasury." I will also apologize profusely
to Mr. Schmitt for assuming that he possessed a Strong's concordance. Instead, I will humbly
suggest that his source is some other document that uses the KJV as its source and that
although in his initial article, he presented himself as knowledgeable in Hebrew, he doesn't
seem to be at all knowledgeable in it.

For example, Schmitt says: "Also, it is common practice to give Hebrew words not in the
inflection found in the text, but in their root form. This makes it easier to locate them in the
lexicons."

Yes, but shadah is not the "root form" of sadeh (field). Root forms are listed in 3rd-person
singular past tense for ease of search in a dictionary. It is the simplest form, usually consisting
of just the root. Sadeh is not a verb, so it has no 3rd-person "root" form. Also, it is the letter
sin and not shin at the beginning of the word (they may look the same, but are not
interchangeable). If anything, shadah would mean "her breast" (shad, [breast] plus the heh
ending which signifies 3rd-person female possessive).

In the previous article I pointed out that yotser (translated "potter") could very well have been
conjugated from otsar (treasury) instead of yatsar (to form or shape). I spelled out in detail
the logical linguistic reasoning that would lead one knowledgeable of Hebrew consonantal
and vowel shifts to believe that yotser means "treasurer" and not "potter." I said that nowhere
in the Bible is the function of potter mentioned in connection to the temple institution, nor is
the need for such a function obvious, but I did claim that the temple or any institution

Volume 1990 - 2002 Issue


Page 986 of 2049
Skeptical Review Edited by Farrell Till
religious or otherwise by necessity needs a treasurer. The Old Testament itself mentions
ecclesiastical treasurers (Joshua 6:24, "to the treasury of the house of the Lord," Joshua 6:19,
Nehemiah 13:13, and elsewhere), but as far as I can see, nowhere in the administratively
exhaustive theocratic Old Testament is the temple-potter duty mentioned. Maybe Wilhelm
can give us good reasons for believing that this institution did exist, that is, reason other than
some drivel that adds up to "it exists because I believe it exists."

And even if it did exist, apart from trying to justify Matthew's alleged "prophecy-fulfillment"
claim, a "treasurer" translation fits into the context much more accurately than a "potter." If
Zechariah had meant "potter," then his mention of the potter begs an explanation of why he
gave the money to a potter. But the fact that the prophet gave money to the treasury of the
temple makes sense in and of itself without any further explanation. If Zechariah gave money
to the potter... why? What was his reason? What did this temple-potter do with the money?
Isaiah and Jeremiah bring up potters (which is probably why Matthew made his Jeremiah
mistake), and these verses draw analogies ("We are the clay, and thou the potter," Isaiah 64:8;
Jeremiah 18 compares Israel to clay in a potter's hand). But according to Schmitt's
interpretation, Zechariah just inserted the word "potter" into the sentence without any obvious
intention and did not elaborate at all. On the other hand, the scenario of an indignant religious
man giving unwanted money to the temple treasury makes perfect logical sense and begs no
question. It fits the context perfectly.

If you ask why they used yotser instead of otsar (treasury) or otser (treasurer), there is a
simple and very possible explanation: The Hebrew letter yod, or y, appears and disappears in
various places even in modern Hebrew. It often displaces the letter which was originally
there. Example: nasa (to travel) in the future is yisa (will travel). The yod has replaced the
nun, but the three-letter root of the word is still nun-samekh-ayin. The yod is not part of the
root, the nun is still part of the root. The same has happened in Zechariah with the root alef-
tsadi-reysh. In conjugating the noun, the yod has replaced the alef, but the root is still alef-
tsadi-reysh otsar (treasury). The letter yod can also disappear, but still be understood as part
of the root. Consider Yehoshua (Joshua or "God saves/will save"); the Yeho- YHW is god's
name. The root "to save" is yod-shin-ayin, but the yod is missing. This is because yod appears
and disappears in Hebrew, sometimes misplacing the original letter that was there. This letter
switching does not occur with the root consonant at the middle of the word or at the end
(except at times for either a heh or tav). Nor can this be done with all consonants at the
beginning of a word, but alef appears as the first letter in otser, and alef is one of the prefix
letters, which appear and disappear in various Hebrew conjugations, as is yod! Whether
Zechariah's use of yotser instead of otser is because of linguistic dialectic variation in Hebrew
prevalent at the time or because of a spelling mistake, I cannot say, but his meaning is clearly
obvious by context, and his Hebrew linguistic precedent for so doing is clear. (By the way,
the Masoretic text has many Hebrew spelling mistakes, while the KJV and other translations
have none! The Masoretic is not a perfect document.)

So the simple and common word for "potter" in Hebrew is yotser. However, if one is
acquainted with the Hebrew spelling variances which is at its most idiosyncratic in Biblical
Hebrew, and if one is familiar with the root and prefix/suffix systems of the language and
consonantal and vowel shifts in Hebrew, one would then have to be quick to admit that it is
very possible, and I would say probable and likely, that the word translated "potter" in

Volume 1990 - 2002 Issue


Page 987 of 2049
Skeptical Review Edited by Farrell Till
Zechariah actually meant "treasurer," especially in the context in which the word appears.
Apart from a desire of some scholars to keep the Christian interpretation intact, there is no
reason to believe that this is not the case. In addition, for any scholar or translator not to list
the very real possibility that this may be the case is dishonest. I can definitely see and
appreciate why a simple reading of the text may lead one to believe that the word "potter" is
intended, but I'll bet few Christians would be willing to even admit the validity of the other
interpretation, which is linguistically defendable as well as much more probable and
consistent with the text, albeit idiomatic.

By the way, Zechariah makes no reference to a potter's field.

Matthew 27:9-10, "Then was fulfilled that which was spoken by Jeremiah the prophet
saying, `And they took the thirty pieces of silver, the price of him that was valued, whom they
of the children of Israel did value; and gave them for the potter's field, as the Lord appointed
me.'"

Zechariah 11:12-14, "And I said to them, `If it is good in your eyes, give me my wages and if
not, refrain.' And they weighed out my wages thirty-silver. And YHWH said to me, `Throw to
the treasurer the mightiness of the value of which I was valued by them.' And I took the
thirty-silver and I threw it to the House of The Lord, to the Treasurer" (Hebrew Masoretic text
direct translation).

In Mr. Schmitt's scathing rebuttal, one thing is conspicuous by its absence: He did not respond
to any of the above points that I made. He also did not adequately explain the obvious error in
Matthew which misdirects readers to Jeremiah, where no such verse even remotely exists,
instead of to Zechariah, where the misquoted verse exists. Instead he claimed that Matthew
was in fact not attempting to refer the readers to a prophecy "fulfillment" in the book of
Jeremiah (which sort of exists in Zechariah) but rather to a spoken (but not written, mind you)
prophecy. So what kind of prophecy was it? How were people to know about it so that it
could be a fulfillment? Was Jeremiah's prophetic voice hanging in the air for hundreds of
years as some sort of nebulous and auditory divine flatulence so that the gospel writer could
point to it as a "prophecy" which was fulfilled in Jesus? Wilhelm Schmitt is really
convoluting logic here just to desperately grasp onto the point he was trying to make. If
Matthew, in an eternal book is citing prophecy fulfillment by quoting a source, then that
prophecy would have to exist somewhere so that it could indeed be found and considered
prophecy fulfillment. Since all of mankind's salvation supposedly depends on a clear
understanding of who Jesus was, wouldn't Jeremiah have written down this most important
prophecy in the book bearing his name? He mentions many trivialities, but leaves out this
important "prophecy."

Also, Schmitt indignantly "rebuked" me for claiming that he had culled his justification for
insisting that the term means "potter" and not "treasurer" from Strong's. Schmitt claims that he
does not even own a Strong's concordance. Well, I must apologize! Perhaps I was mistaken,
and he in fact did not glean his understanding from Strong's, even though every
fundamentalist Christian I have ever known does own a Strong's. I myself have a Strong's,
because as far as finding words goes, it's the best source around. The dictionary definitions in
the back may at times be incorrect (they rarely, if ever stray from the KJV, even if the KJV is

Volume 1990 - 2002 Issue


Page 988 of 2049
Skeptical Review Edited by Farrell Till
incorrect), but the word guide in the front is very useful. But, okay, Schmitt says he does not
own a Strong's. Fine, I concede, admit my error, and sincerely apologize. I would also advise
him to buy a Strong's.

However, I will insist that his source is faulty and shows a lack of knowledge in Hebrew, or
perhaps instead, a lack of initiative to stray from accepted Christian reading. Many English
Bibles and translations--even Jewish ones--use the KJV as a source document. Here is an
example. A common Jewish English Bible translated Genesis 1:28 as "replenish" (as does the
KJV), which signifies "to fill again," even though the prefix re- is a feature of Latin languages
(and English is influenced by Latin) but not of Hebrew. The Hebrew reads "fill," not
"replenish." Not only is the prefix re- not in the Hebrew text, it is not even a Hebraic feature!
This leads me to logically surmise that this Jewish translation was based on the KJV which
was probably based on the Latin Vulgate. The NIV, at times the most scholarly of Christian
bibles, (I say "at times," because the NIV keeps 'alma as "virgin" and yotser as "potter," as do
other popular Christian mistranslations. Zondervan knows on which side its bread is buttered)
and other recent translations have corrected the error. You may think that this is a minor
point, until you realize that there are Christians who have a vague doctrine of a pre-adamic
race of men, angels, demons, etc. just based on that simple KJV "replenish" mistranslation!
So mistranslations can obviously lead people astray.

I know nothing about Dr. Ginsberg, and I have no way to test his knowledge of Hebrew or his
intentions for publishing an English translation of the Masoretic. I don't know who his "target
market" was. If his target market was Christians, then he had a vested interest in keeping his
translation in line with the accepted Christian reading, and since yotser does mean "potter,"
though from context most probably not in this instance, then he had some linguistic reason or
excuse as well as monetary motivation for keeping the reading in line with Christian dogma.
I'll tell you one thing. If I was publishing a book and my targeted market was Christians (a
huge market), I would base my translation on the KJV and Christian dogma. It would be
financially detrimental not to. Most Christians that I have met will chose a questionable
"scholarly" document that reinforces their argument over a truly scholarly document that
contradicts their adopted interpretation. Did Mr. Ginsberg mention at all the possibility that
"treasurer" might be a possible translation? If he didn't, then he was not being very scholarly.
I know that I am not the only Hebrew speaker who has noticed this probable interpretation.

I must admit an error. In part 6 of my letter (which turned out to be the previous article), I
pointed out that beit ha... is a very common form for a house of public use. This is true. beit
ha-knesset (house of the assembly) is "synagogue," beit ha-sefer (house of the book) is
"school," beit ha-shimush (house of the use) or beit ha-kise (house of the chair) both mean
bathroom, beit ha-noar (house of youth) is a youth club, and beit ha-taklit (house of the
record) is a chain of record stores in Israel. I incorrectly mentioned beit ha-yotser in the
context of Zechariah as being "house of the treasurer," but in fact beit ha-yotser does not
appear at all in Zechariah, but rather in Jeremiah 18. What appears in Zechariah 12:13 is beit
YHWH el ha-yotser, which means "the house of YHWH to the treasurer."

Beit ha-yotser does not appear in Zechariah at all. Instead, it appears in Jeremiah 18:3. I
suppose I made the same mistake that Matthew made when misdirecting the faithful to
Jeremiah, where it does indeed mean "house of the potter." But the Jeremiah verse is even

Volume 1990 - 2002 Issue


Page 989 of 2049
Skeptical Review Edited by Farrell Till
further removed from Matthew than the Zechariah verse is and bears no resemblance to
Matthew's verse. It is obvious that Matthew made the same mistake I made and misdirected
readers to Jeremiah when he meant to direct reader to Zechariah, which Matthew very
liberally paraphrases but with some major mistranslations and misrepresentations. So I was
wrong in stating that beit ha-yotser appears in Zechariah. It doesn't.

But I was right, because as I detailed above, there is good linguistic and Biblical precedent to
conclude that yotser in Zechariah can and does mean "treasurer." Furthermore, the beit-
Hebrew structure does indeed appear in Zechariah 11:13 as a house of public use: "And I
threw it to the house of the LORD, to the treasurer." House of the LORD in Hebrew is beit
YHWH, which is the same compound structure (semikhut in Hebrew) as the beit ha-yotser. It
signifies a house of public use, which clearly points to the fact that what is spoken of here was
the temple, which probably does not include a potter but definitely does include a treasurer.
(For biblical references, see above.) The reason that ha or the does not appear in the text is
because YHWH is a proper name, and just as in English, Hebrew proper names do not carry
the definite article the (ha prefix). So what is spoken about, and what I originally meant to
imply, is that "the house of the Lord" is the temple, which by biblical precedent includes a
treasury but for lack of Biblical precedent, and for lack of any obvious purpose includes no
such function as "temple potter." Matthew, as I did, made a mistake and cross referenced
readers to the wrong verse. I gave good linguistic reasons to conclude that yotser in this case
means "treasurer" and not "potter."

As to my alleged "assertions to plagiarism," Mr. Schmitt has yet to prove that he knows
Hebrew at all. Yet he incorrectly threw around Hebrew words as if he were an expert. So
where is he getting his information?

(Yoel Wasserman, P. O. Box 998 Aspen, CO 81612; e-mail core@rof.net)

From the Mailbag


Personal Relationships with Jesus...

I have enclosed a money order for another year of The Skeptical Review with the
January/February issue. I would also like to know if you have the answer to a question that
has been on my mind for the past three or four years. I would like to know when the doctrine
of "having a personal relationship with Jesus" started. It does not seem to have been taught
until recently. Can it be considered an authentic Christian teaching if it was not taught by the
early church? If it's not, then how can the biblicists get away with promulgating idiotic
nonsense like this? I'd sure like to know. Keep up the good work.

(Patrick Shiflett, 1521 Charrington Drive, Midlothian, VA 23113.)

Volume 1990 - 2002 Issue


Page 990 of 2049
Skeptical Review Edited by Farrell Till
EDITOR'S NOTE: When I was growing up in the 30s and 40s and preaching in the 50s, I
don't recall ever hearing this expression. As Mr. Shiflett stated, it seems to be a recent
"doctrine." The words "personal" and "relationship" don't even appear in the Bible. There are
some New Testament passages that speak of "knowing" certain things about Jesus, such as
Philippians 3:10 where Paul said that he had "suffered the loss of all things" that he might "be
found in him [Christ]" and "know him and the power of his resurrection," but this certainly
doesn't echo the maudlin strains of modern Christians who flaunt their claims of having a
"personal relationship with Jesus." Anyone who has talked to personal-relationship-with-Jesus
Christians knows that they use the term as if Jesus was a visible buddy who walked and talked
with them every moment of the day, and I know of no scripture that teaches that this is an
aspect of Christianity that the "saved" can expect to experience. One could make a much
better scriptural case for having a personal relationship with the Holy Spirit.

My experience has been that Christians who parrot this expression do so as a defensive
maneuver when they are confronted with arguments against their beliefs that they can't
respond to, so they retreat to their personal-relationship-with-Jesus bastion. "Well, I know that
I have a personal relationship with Jesus," they will say, "and you can't tell me that I don't
know what I know I know." This is merely a resort to emotionalism when they cannot
logically defend their beliefs. All religionists are emotionally convinced that theirs is the true
religion, and all of the fanatical ones I have ever talked to simply turn deaf ears to any
attempts to show them that their emotional apologetics can be used to show the "truth" of any
religion. This situation simply underscores the insidiousness of religion. People are going to
believe what they have been indoctrinated to believe, no matter how compelling the evidence
against it may be. So when confronted with facts, Christians can always whine about having a
personal relationship with Jesus.

A Threat from the Opposition?

I saw this add [sic] in the Blackstone paper, and find it hard to believe that someone would go
to such extremes to defy the word of God. I doubt very seriously that you can show me any
real contradictions or errors in the old 1611 King James Bible that cannot be explained with a
little study and common horse sense. But then of course this thing of spiritual dicernment [sic]
that is the most needful of must play the largest role.

But then again, if you are using some of the new versions taken out of the corrupt Hort &
Westcott Greek Text, than [sic] I can understand to the fullest why you would think as you
do. All these new modern man translations of the Word of God are absurdities within
themselves.

As you have probably figured out by now, I am a Born again, fundamentalest [sic], and a
BIBLE BELIEVER. But never-the-less [sic], I do want to take advantage of this "The
Skeptical Review" (First year FREE). Who knows it just may cause me to dig deeper into
the Word of God, for I have found everywhere there appears to be a contradiction, is where
most of the Golden Nuggets are found.

Volume 1990 - 2002 Issue


Page 991 of 2049
Skeptical Review Edited by Farrell Till
I hope there will be no problem in you standing true to your add [sic], seeing there are a lot of
my neighbors have brought this to my attention, and the penalties of false advertising is [sic]
devastating.

Looking forward to see what you can come up with.

(Rev. Garry L. Creed, Route 2, Box 925, Crewe, VA 23930-9639.)

EDITOR'S NOTE: The add [sic] the Reverend Creed referred to was a classified ad that a
subscriber to The Skeptical Review ran in his local newspaper. The first issue of Reverend
Creed's subscription was sent to him the day his envelope was opened. For some reason, he
seemed to think that the ad wasn't legitimate, but his fears were unfounded. Since neither I nor
the subscriber who ran the ad is a Christian, he need not fear that there was any intention to lie
or in any way mislead or deceive anyone. The offer is exactly what the ad stated: a free first-
year subscription to The Skeptical Review. Literally thousands of people can testify to the
legitimacy of this offer, because in the seven years of its publication, almost every subscriber
has been introduced to it through this offer. A lawyer once sent us a subscription request with
a cynical note saying that nothing is free, and so he would have to see it to believe it. Well, he
saw it, so I guess he believed it, although he never sent an apology.

Why do I make this offer? Because the majority of the people who request the free
subscription become paid subscribers after their first year. Some have been with us from the
beginning, and many who heard about us after we began publication have ordered all back
issues to read what was published before they became subscribers. If Reverend Creed would
send us the names of his neighbors who called the ad to his attention, I will gladly put their
names on the mailing list, but since he has now seen one issue, I don't expect him to do that.
Neither do I expect him to become a paid subscriber.

When he has seen enough to realize that claims of biblical discrepancies are not as superficial
as he imagines, he will undoubtedly try to climb as gracefully as possible out of the hole he
has dug himself into, but I don't intend to make it easy for him. In the past, I have admitted to
having a personality flaw. When a biblical inerrantist sticks his foot into his mouth, I can't
seem to resist the temptation to shove it in a little farther. If the reverend is so certain that
"Golden Nuggets" are to be found in the very places where biblical contradictions appear to
be, I am going to offer him the opportunity to mine enough nuggets to make him a
millionaire. Under separate cover, I am mailing to him a complete set of all back issues
(FREE). If he will read them and "dig deeper into the Word of God," I will publish for him
any "Golden Nuggets" that he finds if he wishes to respond to any of those articles. The
penalty, of course, is that my reply to his response will be published simultaneously. As
always, however, he will have the opportunity to respond to my reply, if he wishes. In other
words, I am giving the reverend a chance to put his faith where his mouth is. If he thinks that
there is no contradiction in the King James Bible "that cannot be explained with a little study
and common horse sense," he should have no objection to accepting my offer. I would think
that this is an opportunity he can't refuse, because TSR would give him a worldwide audience
that he would otherwise have no chance to reach. It has subscribers in every state in the union,
including Alaska and Hawaii, several Canadian provinces, and many foreign countries: China,
Japan, Hong Kong, the Philippines, Australia, New Zealand, Indonesia, Malaysia, Thailand,

Volume 1990 - 2002 Issue


Page 992 of 2049
Skeptical Review Edited by Farrell Till
India, Lebanon, Israel, South Africa, Italy, France, Spain, Belgium, Germany, England,
Ireland, Iceland, Denmark, Sweden, Norway, Finland, Mexico, El Salvador, Venezuela,
Argentina, Brazil, Uruguay, and probably others I can't recall without checking the mailing
list.

I'll even give Reverend Creed an opportunity to embarrass me publicly before all of his
neighbors who brought the ad to his attention. I usually conduct two or more public debates
each year, at my own expense, so I am challenging Reverend Creed to defend the inerrancy of
the Bible in a public debate at Crewe, Virginia, or some community nearby. If he will agree to
do so, I will come to Virginia (at my own expense) to meet him at a time and date convenient
to him. Since I am now retired, scheduling would be no problem for me. An ideal location for
the debate would be right in the very church building where he preaches to his congregation.
This would give him the opportunity to show to his congregation some of the "Golden
Nuggets" that can be found when one "digs deeper into the Word of God." If, however, that
would not be an acceptable location, we could conduct the debate in a neutral setting. I will
pay half of the rental fee on such a location.

And I'm still not through shoving on the reverend's foot. If he refuses this offer, I will send
money to the TSR subscriber in that area of Virginia to pay for publishing in the Blackstone
paper a notice that Reverend Creed was challenged to defend the integrity of the Bible in
public debate and declined the opportunity. It's going to be hard for him to climb out of his
hole gracefully.

Another Skeptic...

Do you want to know what I'm skeptical about? Well, I'll tell you anyway! I'm skeptical
about an offer for a free one-year subscription to The Skeptical Review I saw in the internet.
This puts you in a dilly. If I do get the free subscription, I will lose my skepticism and will
take up flower sniffing, and if I do not, I will be skeptical about skeptics. That's a double-
edged sword if I've ever struck one in someone's figurative gut; I recommend you mail me the
subscription, because that way at least I'll be happy. (I love flower sniffing!)

(Michael "Jubal" Gardner, 39273 Sutter Drive, Fremont, CA 94538.)

EDITOR'S NOTE: By the time this is published, Mr. Gardner will be sniffing flowers,
because his subscription started with the March/April issue. As I said in commenting on the
preacher's letter above, there is no catch or fine print in our offer of a free subscription. We
have been offering it ever since our first issue was published in January 1990. Thousands of
free subscriptions have been given over that time. By the way, I hope Mr. Gardner sticks to
flower-sniffing and doesn't try to give comedy a try.

Religion and Psychological Health...

Volume 1990 - 2002 Issue


Page 993 of 2049
Skeptical Review Edited by Farrell Till
I want you to know that I very much appreciate having had the opportunity to read your
Skeptical Review. It has been a difficult experience and I have been very dismayed by the
approaches your critics take to explain away the contradictions you have addressed.

What I appreciate is the care and effort you have devoted in your study, when it seems that
the natural thing would have been to just dismiss the Bible wholesale. As I understand it, you
were yourself engaged in missionary work. What a shock it must have been for you to make
these discoveries.

I am a mental health professional and daily see people whose spiritual beliefs are literally
"spoon fed" to them by family or clergy. They never even think-- and are prohibited--to
question any of it. I can understand why.

I recently wrote the pastor of the church my wife and I attend for a meeting regarding some of
these bothersome contradictions. He keeps apologizing for being so busy or for having
forgotten to schedule some time.

With so many contradictions, I cannot foresee anyone explaining them away. When
apologists attempt to do so, I find myself filled with disgust. In my heart of hearts, I long for
these discrepancies to just go away. The more I read, the more discrepancies are apparent.

As a mental health professional I must say that religion seems to be associated in some way
with so many of my clients' difficulties. Yet, they are stuck. I think of one dear fundamentalist
woman who is lonely after divorcing an abusive husband, yet who could not have a husband
as she would be committing adultery if she did so. Her family and church would disown her.
She is attractive and in her 30s with normal needs and desires, but is denied fulfillment of
those because of this belief.

I have counseled gay clients who desired some religious faith and human contact, but felt the
coldness of Christian persons after revealing themselves. I see many adult clients who were
abused as children, yet feel ostracized by their respective religious faiths and feel that others
somehow blame them for what happened to them as children. Not infrequently, I struggle by
telephone with these clients in a suicidal crisis and am dismayed that so many local churches
do not understand the long-term effects of child abuse, and typically tell my clients to "forget
the past," go to their knees in prayer, get a life, and everything will be okay. What my clients
are needing is some human contact in the present by someone who accepts, listens, and
understands. For several years, I have been quite bitter about this.

I am rambling on. Again, I appreciate your efforts and your integrity.

(Dan Dunlap, 1925 Melrose, Apt. 35, Walla Walla, WA 99362, e-mail dunlapd@wwics.com)

EDITOR'S NOTE: I remember reading once in a source that I can't recall that the Swiss
psychologist Carl Gustav Jung once said that in the majority of patients he had treated, he had
found that their mental illnesses had been caused by their religious beliefs. I'm not a
psychologist, so I can't speak with any authority on the subject, but it does seem to me that
religion, which is supposed to bring comfort and happiness, leaves in its wake wrecked lives

Volume 1990 - 2002 Issue


Page 994 of 2049
Skeptical Review Edited by Farrell Till
and deep psychological wounds. How many times do we see in the news stories about
violence and self-mutilation committed by those who heard voices or thought that their
victims were the devil or demons? Even churches deplore the existence of Satanism in our
society (which has probably been greatly exaggerated), but if Satanism does exist, who is
responsible for it? Certainly not skeptics and atheists, who advocate rational approaches to
life and its problems. The fault must lie with the religious institutions that instill in children
lifelong superstitious beliefs in spirits and demons.

The Bloodthirsty God...

Thank you very much for the sample copy of The Skeptical Review. I love it! Enclosed is a
check for two years of issues.

The Skeptical Review is just the type of material I read all the time. I constantly read books,
magazines, and other literature covering biblical criticism, rationalism, philosophy, etc. I refer
to myself as an "evangelical atheist." I "witness" to people all the time, but I preach against
the gospel and against Christianity.

This certainly doesn't mean that I hate God, or the gods, or the Goddess or goddesses, or
whatever deities may exist. It means I despise the imaginary God of the Bible, because he is a
bloodthirsty butcher and a mass murdered, and a liar who broke his own commandments and
promises, and did other beastly things.

I am currently working on my own book on religion (Christianity mainly). I also will be


attending the March 24-25 Till-Lockwood debate in Oklahoma City. The Skeptical Review
will bolster my "career" as a student of religion. Thank you very much for adding me to your
mailing list.

(Christopher Edsall, 555 North Council Road, Apt. F, Oklahoma City, OK 73127.)

EDITOR'S NOTE: I had the opportunity to meet Mr. Edsall at the Oklahoma City debate.
After the first session, six of us gathered in a motel room and talked until 2:00 A. M. In
addition to these five, I met three other TSR subscribers for the first time. Some preachers
complain that they won't debate skeptics, because they don't want to give them an audience.
The attendance at this debate, however, easily showed that an atheistic opponent will draw an
audience above the proportion of skeptics in the general population.

Too Much Minutia?

Enclosed is a check for another year's subscription to The Skeptical Review. I enjoy the letters
more than the articles most of the time. The articles for debating the Church of Christ cronies
get bogged down with minutia of details that really aren't necessary for refuting their religious
claims. Expounding on your major points of difference, to a degree, is understandable, but
your freethought readership can fill in the microscopic blanks themselves. I realize that your
goal for all the exhaustive analyses is to leave your "opponents" no room to quibble.

Volume 1990 - 2002 Issue


Page 995 of 2049
Skeptical Review Edited by Farrell Till
Unfortunately, this goal is not achieved, due to the fact that the religionists continue to
quibble. I feel you owe it to your freethinking readership (which in my opinion is the majority
and most loyal of TSR subscribers) to cover more ground per issue or maybe print more of the
submitted articles (which I enjoy more than the debating articles). Covering more topics
would create a salvo that religionists could never argue with, without revealing the depravity
of their irrational thought processes. Committing so much printed space for the religionists'
articles is usually an exercise in redundancy, because the same old lame assertions and
"explanations" are used.

I just finished watching the NFL playoffs and regurgitating from my mind are all the idiotic
religious babbling of the athletic superstars for Jesus. This country is inundated with
superstition, mostly propagated through the Christian religion's official and unofficial
spokespersons that seem always to give "glory to the Lord" at every opportunity. They are on
the offensive, and the media are catering to them. When was the last time you heard an
interviewer scoff at the notion that god helped them win or saved them from death, etc.? It is
unpopular (and therefore rarely done) to attack religion. The amalgamation of mythologies
called Christianity in America have been with us since the founding of our nation, and so is
deeply entrenched in our culture. Even though in our more scientific day, emphasis on
religion in our public schools, criminal justice system, and other institutions had declined (the
religionists call it secularization), most people still cling to the basics of religion, i. e.,
god/devil, heaven/hell, life after death, miracles, etc....

(Jeff Schmura, 808 Apple Lane, Shoemakersville, PA 19555.)

EDITOR'S NOTE: Mr. Schmura's letter gives me an opportunity to restate the editorial policy
of The Skeptical Review. In our very first issue in January 1990, this policy was stated in the
front-page article: "Our purpose will be to promote critical examination of the Bible inerrancy
doctrine." The article further stated that we would not even get involved in the "theism-
atheism controversy." In a word, our policy from the beginning has been to expose fallacies
and absurdities in the biblical inerrancy doctrine. The reason for this policy is rooted in the
present socio-political climate in the United States. Conservative Christian organizations are
working feverishly to promote their claim that the Bible is the inspired, inerrant word of God
on which our civil laws should be based. Some freethought organizations choose to oppose
these movements in our courts and have won some significant victories, but I personally think
that these legal battles will have to be fought as long as a significant percentage of our
population believes the ridiculous claim that what the Bible says is actually what God has
said. If that premise can be proven false, the Christian Right will lose the cornerstone of its
political agenda. In the battle against the Christian Right, then, our strategy at TSR has been to
put the axe to the root of the tree and attack the cause of the problems rather than trying to
pick off its fruit one at a time. Some who have not understood our editorial policy have sent to
us some interesting articles on the broader subject of religion, but they do not relate directly to
the biblical inerrancy doctrine. Hence, we have not published them.

As for the attention to minutia, I'm sure that those who have never been under its influence
don't understand the extent to which Christian fundamentalism is preoccupied with minutia.
Details that are considered trivial to anyone else are often magnified by fundamentalists to
supreme importance. Whereas rational people would not think that singing hymns to the

Volume 1990 - 2002 Issue


Page 996 of 2049
Skeptical Review Edited by Farrell Till
accompaniment of a musical instrument would matter to an omnibenevolent deity, some
fundamentalists make this a test of fellowship. Examples like this that I could cite would fill
an entire page, so obviously "minutia" is important to Christian fundamentalists. Anyone who
hopes to make headway in reasoning with them must understand this.

This is one reason why articles in TSR are so thorough in their examination of inerrancy
issues. If no stones are left unturned, then, as Mr. Schmura noted, inerrantists will be left with
little room to quibble. Another reason for the thoroughness is that it benefits skeptics on the
subscription list by arming them with information they can use when discussing the Bible
with fundamentalist acquaintances. I get many letters, phone calls, and e-mail messages from
people who ask for help in dealing with biblicists that they are having private discussions
with. They tell me what their contacts say to explain certain biblical discrepancies that have
been brought up during these discussions, and then ask me what could be said in response to
these "explanations." Good advice for everyone who engages is such discussions is not to
bring up an issue until it has been thoroughly studied. What may appear to be an unresolvable
discrepancy to a skeptic will most assuredly not be considered unresolvable to a Christian
fundamentalist. One should always bear in mind that there is no such thing as an
"unanswerable" argument for biblical errancy, because a skilled inerrantist will always know
about how-it-could-have-been scenarios that have been used to "explain" the major biblical
discrepancies. So skeptics who intend to discuss biblical errancy with Christian
fundamentalists should make sure that they understand the issues and know in advance what
"explanations" their opposition will probably give. One way to prepare for these discussions
is to become familiar with the explanations that apologists like Gleason Archer and Norman
Geisler have given to the discrepancies that will probably be brought up in discussions with
fundamentalist associates. If an "explanation" is encountered in these books that may not
seem convincing to the skeptic but, nevertheless, cannot be rebutted by the skeptic, it would
be best not to bring the issue up. It makes an unfavorable impression if one has to say, "I'll
check into that and get back to you." The fundamentalist will leave the discussion thinking he
has won.

Most subscribers are indeed freethinkers, although there are many Bible believers who receive
TSR and renew their subscriptions each year. Articles that examine biblical discrepancies in
detail, then, serve as double-edged swords. They show inerrantist subscribers that their
position is really indefensible, and they help skeptics to understand the issues well enough to
talk to their fundamentalist friends about them. At any rate, the purpose of TSR has always
been to refute the doctrine of biblical inerrancy, and to do this, we have to meet inerrantists on
their own turf. This requires giving attention to "minutia."

What Is a Miracle?

What is a miracle? Is it not, in its fullest sense, a violation of scientific law? What is a
scientific law? Is it not a description, within an appropriate range, of what physically happens
under certain conditions? Has it not been established by the most careful observations again
and again without fail? Which is more likely, therefore, a miracle reported secondhand, the
circumstances of its observation being unknown, or the failure of an established scientific
law?

Volume 1990 - 2002 Issue


Page 997 of 2049
Skeptical Review Edited by Farrell Till
The answer to that question should be obvious to anyone acquainted with reason, and that is
why biblical miracles are rejected. It is far more likely that the reporter of a miracle has either
lied, been deceived, hallucinated, misunderstood, reported erroneous information, or
incorrectly conveyed the event in question than that scientific law failed.

Where prejudice is involved, group sightings are no better. One occasionally reads about them
in the papers and discovers just how credulous people can be within a supporting group. Even
the agreement of a long list of respectable observers with impeccable credentials, who are not
members of a supporting group, mean nothing if a selective factor is involved. If one
thousand people see something in the night sky, and if 73 of them (all respectable) believe
they saw an alien craft, that does not constitute a favorable testimony. What do we do with the
other 927 people (equally respectable) who reached a different conclusion? Thus, a simple list
of respectable observers is never enough. We must have a representative sampling, which is
never easy to obtain since those who see nothing unusual report nothing. Consequently,
scientists are not impressed with group sightings of what are supposed to be miracles. Group
prejudices and selective reporting must be weighed before any weight can be attached to a
group sighting. For events that occurred 2000 years ago, that is clearly an impossible task.
Consequently, the number of people in the Bible, who supposedly witnessed a miracle, add
little in the way of credibility.

The skeptic need not be pig-headed and flatly reject all miracles out of hand. However, he is
clearly within the bounds of reason to ask that the evidence in support of a miracle be at least
as good as the evidence supporting the scientific law that was supposedly violated. In other
words, extraordinary claims must be accompanied by extraordinary proof if they are to be
taken seriously. That means good documentation, which is clearly impossible for biblical
miracles that have been handed down for generations, whose original circumstances are
beyond any kind of reconstruction.

Thus, you have the reason for our skepticism concerning biblical miracles. There is nothing
unfair or illogical about it. Indeed, when Bible-believers get away from their Bibles, so that
their brains can air out a bit, they instinctively apply the same reasoning in rejecting the
miracles of other religions. Therefore, we skeptics don't need this whining about
presuppositional bias. We do acknowledge that our rejection of biblical miracles is based on
the lack of proper evidence and not on a certain knowledge of their falsehood. We have
applied the best standards of reasoning and reached the most rational conclusion that can be
reached concerning biblical miracles, namely that they should be rejected in general. (In a few
rare cases, there might actually exist a physical explanation for the "miracle" in question. It is
known, for instance, that the flow of the Jordan River is occasionally interrupted by landslides
or what have you. In those cases no scientific law has been violated.)

Why do so many people go to such pains to establish a scientific basis for the miracles of the
Bible? It is because they sense (or understand) that miracles need extraordinary evidence to be
rationally believed, evidence that is lacking. What better strategy is there than making the
"miracle" scientific so that no scientific law has been violated? In one stroke a plausible
explanation is offered and the standards for admission greatly lowered. The drawback, of
course, is that one must deny that supernatural events play a role in the Bible, and the Bible
clearly calls for violations of scientific law.

Volume 1990 - 2002 Issue


Page 998 of 2049
Skeptical Review Edited by Farrell Till
The skeptic's rejection of biblical miracles is based on solid reason and not some unfair bias
against all things supernatural. The best reasoning has been applied and the best conclusions
reached. The rational mind must reject biblical miracles for the same reason that it rejects the
miracles of other cults and religions. They are rejected for lack of adequate evidence.

(Dave Matson, P. O. Box 61274, Pasadena, CA 91116; e-mail


103514.3640@compuserve.com)

EDITOR'S NOTES: As usual, Dave Matson makes some excellent comments. His letter was
particularly timely for inclusion in this issue, because it is an excellent supplement to my
rebuttal of Dr. Price's claim that it is unreasonable to question prophecy on the grounds that
miraculous events are unlikely.

Dave, who owns the Oak Hill Free Press, announces that he will shortly have a new booklet
in print (The Bible, Common Sense and the American Way), which will have about 72 pages
and sell for $4.95. He states that it is based on powerful, common-sense observations, which
reject the Bible as a divine product, and that each of the common-sense arguments will be
fully developed with special emphasis on the usual apologetics.

Resurrection & the Wright Brothers...

I read with interest the debate [Horner-Till] about Christ's resurrection. I make no claim to a
great knowledge of debates but I was interested in your argument that extraordinary claims
require extraordinary evidence. The "Elvis legend" is a good point. Mr. Horner stated that you
used circular reasoning with this argument, and I suppose it is possible he is right.

Would it be possible to use a correlation between the resurrection and the flight of the Wright
brothers? In 1903, they made the extraordinary claim of the capability of heavier than air
flight, yet there was very little belief among the so-called experts and the general public. For
years there were reports of their flights yet few took them seriously. What was needed was
extraordinary evidence. When their flights became commonplace after 1908, the people
received the extraordinary evidence. They saw an airplane with their own eyes, and reporters
documented this extraordinary evidence. Of course, today nobody considers flight
extraordinary so I'm sure this couldn't be circular reasoning. The only fault I can see in this
argument is that an apologist might compare the former non-believers of flight to the
"Doubting Thomas."

(Jim Bragge, 612 Christopher Drive, Clovis, NM 88101; e-mail jimb@3lefties.com)

EDITOR'S NOTE: I recall a childhood experience that illustrates Mr. Bragge's point. A friend
told me that his family had just bought a console radio that had a place to install a new
invention that would enable people to see the people speaking as well as hearing them.
Everyone laughed at him, but several years later, when I was a freshman in college, I went
downtown and saw television for the first time through the display window of an appliance
store.

Volume 1990 - 2002 Issue


Page 999 of 2049
Skeptical Review Edited by Farrell Till

Rampant Ignorance...

Waging war against obscurantism is a rather thankless task, so please accept my thanks for
your latest incisive and lucid attack on the forces of darkness ("The Nature of the Claim"
March/ April 1997).

It seems to me that one of the Christian apologists' greatest blunders is the insistence that faith
is a virtue and doubt is a vice. If all humanity had clung to this tenet, we would still be
padding about in caves wearing loincloths. It is precisely our natural curiosity, probing of
alternatives, skepticism of received "truths," and question of conventional wisdom that makes
us humans rather than robots. Inventors, scientists, and scholars thrive on doubt. It's the
mother's milk of democracy and progress.

I'm reminded of the propaganda (quite appropriately, the word derives from congregatio de
propaganda fide) TV programs of the Catholic Church called Eternal Word Television
Network, which, among other things, features a segment called "Visionaries, Mystics, and
Stigmatists." The hosts, Polly and Jim Lord, spend most of their time illustrating and peddling
their book of the same name. The pair, with an IQ of about 120 (combined), breathlessly
recount levitations, incantations, flagelations, apparitions and, inevitably, canonizations of
their favorite saints, most of whom would be considered certified psychotic whackos by an
unbiased panel. Any doubt of these wonders is dismissed as arrant rationalism and flat-out
heresy. Polly seems to parrot hagiographics that have actually become an embarrassment for
some thinking Catholics since Vatican II.

Near my home is a Presbyterian Church whose marquee features a slogan du jour. The current
one reads, "Faith can move mountains. Doubt can create mountains." Rally around the altar,
boys!

In closing today's homiletic, brothers and sisters, let me quote from the poet Robert
Browning: "Rather I prize the doubt/Low forms exist without/Finished and finite
clods/Untroubled by a spark."

(Fred L. Ehrstein, 9 Westwood Drive, Belleville, IL 62226.)

Volume 1990 - 2002 Issue


Page 1000 of 2049
Skeptical Review Edited by Farrell Till

Skeptical Review
Volume Eight, Issue Four
July/August 1997
Farrell Till, editor

• How Did the Apostles Die


Till surveys the inconclusive evidence about early Christian ``martyrs''

• Testing the Null Hypothesis


Till continues his demonstration that the Jeremiah prophecy is a failure

• The Sins of the Fathers: Another View


Roger Hutchinson defends the view that the Bible does not punish the children for the
sinds of their fathers

• A Problem of False Analogy


Till dissects the analogy used in Hutchinson's article

• Why Did the Apostles Die?


Matson addresses the claim that the Apostles would not have died for something they
knew was false

• From the Mailbag


What people are saying about The Skeptical Review

• Back Page
Errancy-related tidbits of information

Volume 1990 - 2002 Issue


Page 1001 of 2049
Skeptical Review Edited by Farrell Till

How Did the Apostles Die?


Christian apologists, both real and would-be, argue that the willingness of the apostles to die
for their faith is proof that the resurrection of Jesus was a real experience in their lives. People
will die for what they believe to be true, the argument goes, but they would not die for what
they know is not true. In this issue (pp. 10-11), Dave Matson has rebutted this argument by
showing how that the postresurrection appearances of Jesus could well have been only
imaginary or psychological experiences of those who allegedly claimed that they saw Jesus
alive after his death. If so, then the apostles who were martyred (if indeed any were) would
have died not for what they knew to be true but only for what they thought they knew was
true. There's a big difference.

To have a cogent argument, then, Christian apologists would have to prove the unprovable
and establish that the apostles did actually know that their postresurrection experiences were
real and not merely psychological, and with all of the apostles long dead, there is no way that
any apologist could do this. There is even another hurdle in the path of this argument that is
impossible for resurrection proponents to clear. They must show convincing evidence that the
apostles did indeed suffer martyrdom for what they preached.

Christians bandy this argument about so much that many will no doubt think it strange that
anyone would question that the apostles died as martyrs, but the truth is that the evidence of
widespread martyrdom in the early church is very weak. The claim assumes the historical
accuracy of the New Testament, which makes some scattered references to persecutions of
early Christians (Acts 8:1; 11:19; 13:50; 2 Thess. 1:4), but if the accuracy of the New
Testament is to be assumed, then it would be pointless to debate any of the major apologetic
claims, because the New Testament does claim that Jesus was born of a virgin, that he worked
many miracles, that he was resurrected from the dead, that he ascended into heaven, etc.
Outside of the New Testament, however, evidence of wholesale persecutions of early
Christians is primarily a tradition that has been foisted on an unsuspecting Christian public. In
his debate with Celsum, Origen, as late as A. D. 240-250, said that the number of Christian
martyrs was "few" and "easily numbered":

For in order to remind others, that by seeing a few engaged in a struggle for their religion,
they also might be better fitted to despise death, some, on special occasions, and these
individuals who can be easily numbered, have endured death for the sake of Christianity
(Contra Celsum, Book 3, Chapter 8, emphasis added).

So if more than two centuries after Christianity had its beginning, an important "church
father" like Origen could say to a doubter that those who "have endured death for the sake of
Christianity" could be "easily numbered," that gives little support for the traditional view of
the apostles and early Christians dying in droves for their beliefs.

In the matter of martyrdom suffered by the apostles, there is a larger question that needs to be
resolved. Were the apostles even real historical persons? There are reasons to suspect that at
least some of them were merely legendary figures. Here presumably were men who took the
gospel into various countries and provinces, but the only records of their activities are to be

Volume 1990 - 2002 Issue


Page 1002 of 2049
Skeptical Review Edited by Farrell Till
found in the traditions and writings of early church leaders, who had a special interest in the
growth of Christianity. According to the book of Acts, for example, the apostle Paul stirred up
public controversy almost everywhere he went on his missionary tours. In Philippi, Paul and
Silas were allegedly beaten and thrown into prison for having cast a "spirit of divination" out
of a young lady who had brought considerable gain to her masters through fortune-telling
(Acts 16:16-24). While they were in prison, a great earthquake struck (as earthquakes did so
often in those days when Christian activities were going on), opened the doors, and shook off
the bonds of Paul and Silas (v:26). In Lystra, Paul and Barnabas were mobbed by a crowd and
worshiped as the gods Jupiter and Mercury (Acts 14:11-13). Later Paul was stoned in the city,
dragged outside, and left for dead (v:19). While preaching in the province of Asia, a pagan
mob rioted in protest of Paul's preaching and would have lynched him and his companions
except for the intervention of a town clerk (Acts 19:23-41). Everywhere Paul went
controversy like this allegedly followed him, yet there are no records outside of the New
Testament of any of his activities.

Since the New Testament is relatively silent on postresurrection activities of the other
apostles, we "know" even less about their evangelistic work. What we do know is mainly a
matter of tradition, which is all that Christians can offer in support of their claim that the
apostles died for their beliefs. The problem with these traditions is that they are (1)
unverifiable and (2) contradictory. One tradition, for example, says that the apostle Paul was
tried in Rome and executed, but another tradition says that he was released and went to Spain
to do more missionary work. So which tradition do we accept? When traditions are in
conflict, how do we determine which, if any, is the truth?

In The Search for the Twelve Apostles, Dr. William Steuart McBirnie examined the maze of
traditions about the fate of the apostles, and although he seemed to retain his belief that the
apostles were real historical characters who had suffered persecution and often martyrdom, he
admitted that the traditions were sometimes so inconsistent and contradictory that it cannot
now be determined how all of the apostles died. He referred to Tertullian's claim that the
apostle John was tortured and "boiled in oil but was delivered miraculously," and then
admitted that "(t)his story does not seem to have much foundation in historical fact," even
though tradition says that the Church of San Giovanni "has been built on the spot in Rome" in
honor of the apostle's escape (Tyndale House, 1977, pp. 116-117). McBirnie concluded that
the best traditional evidence indicates that John died in Ephesus of old age. If this is so, John
would not have been an example of an apostle who died for what he knew was right.

McBirnie had no better luck in trying to determine the fate of other apostles. He found
Matthew to be an especially confusing case. Various traditions had Matthew preaching in
places as far flung as Ethiopia, Persia, Parthia, Isidore, and Macedonia (p. 176). The traditions
relate preposterous accounts of attempts that were made to kill him, which he, like John,
miraculously escaped from. In one tradition, a jealous king tried to have Matthew burned
alive, but the flames flew out, took the form of a dragon, and curled around the king.
McBirnie concluded that "(t)here are too many stories of Matthew's death to be certain just
where he died" (p. 182), but even though he had earlier cited Heracleon and Clement of
Alexandria (The Miscellanies, 4, 9), who had both said that Matthew died a natural death (pp.
175-176), McBirnie would not give up so easily on his desire to find martyred apostles. "It is

Volume 1990 - 2002 Issue


Page 1003 of 2049
Skeptical Review Edited by Farrell Till
perhaps possible that Matthew was martyred in Egypt upon his return from Ethiopia in
Africa," he said, "but this conclusion is not certain" (p. 182, emphasis added).

Uncertainty was what McBirnie seemed to find everywhere in his research. He found
traditions that said Bartholomew was "flayed alive and crucified in agony" in India after
banishing a demon from the idol of a king (p. 135). He found other traditions that said
Bartholomew was martyred in Armenia. To reconcile the conflicting traditions, he cited Edgar
Goodspeed, who had suggested that "India" was a "term very loosely used by the ancients" (p.
133).

McBirnie's search for the fate of the other apostles uncovered traditions that were just as
inconsistent and uncertain as those noted above. He claimed that his research took him three
times to the island of Patmos (where John allegedly wrote Revelation) and to the locations of
the seven churches of Asia cited in Revelation (p.7). He traveled to Germany, Rome, Greece,
Lebanon, and "almost every Middle Eastern country." The other locations he visited and
libraries and archives he claimed to have used are too numerous to list here, but the results of
his research were as noted above, i. e., too much inconsistency and contradiction to determine
with certainty how and where the apostles died.

Despite the uncertainty he found in his research, apparently McBirdie still retained his belief
that the apostles were real historical people who had suffered persecution and martyrdom for
their faith. No doubt many Christians who read this article will lay it aside and continue like
parrots to ask the same question: "Why would the apostles have died for something they knew
was false?"

Testing the Null Hypothesis


Farrell Till
In alleging that Jeremiah accurately prophesied that the Judeans would be in Babylonian
captivity for 70 years, Dr. Price has said that "(i)n a scientific investigation one must prove
the null hypothesis" ("Solving the Jeremiah Problem," TSR, May/June 1997, p. 2). What Dr.
Price meant, of course, is that when biblicists make extraordinary assertions (no matter how
outrageous and far-fetched they may be), those who question the assertions have the
responsibility to prove that they are not true. Aside from the fact that there is about as much
"science" in the hermeneutic methods of prophecy-fulfillment proponents as there is in
astrology and fortune-telling, Price's continual efforts to shift the burden of proof onto me in
the matter of Jeremiah's alleged prophecy runs completely contrary to a recognized principle
of argumentation that says the one who asserts must prove.

Debaters who try to shift the affirmant's burden of proof onto their opponents are nothing new
to me. I encounter the tactic almost every time I debate a biblical inerrantist. In my debate on
the resurrection with Dr. Norman Geisler, he told the audience that it was "incumbent" upon

Volume 1990 - 2002 Issue


Page 1004 of 2049
Skeptical Review Edited by Farrell Till
me to prove that Jesus did not rise from the dead (TSR Transcript, p. 7), and so did Michael
Horner in our debate on the same subject (TSR Transcript, p. 14). In my debate with Bill
Lockwood, who was affirming that "God is," he charged late in the session that I had not
proven that God does not exist. Now along comes Dr. Price, asserting that a prophecy made
by Jeremiah was remarkably fulfilled, but when the weakness of his position is exposed, he
says that it is my responsibility to "prove the null hypothesis." Apparently, I must remind Dr.
Price again that he is the one who is asserting the extraordinary claim of prophecy fulfillment,
and so it is his responsibility to prove all of the facts necessary to establish that prophecy
fulfillment did indeed occur. Needless to say, he hasn't even come close to doing that.

Criteria 2 and 3: In my first response to Dr. Price, I listed criteria of valid prophecy
fulfillment, which Dr. Price has described as "satisfactory" ("Solving the Jeremiah Problem,"
May/June 1997, p. 2). The second of these criteria requires the claimant of a prophecy
fulfillment to prove that the alleged prophecy was made before and not after the alleged
fulfillment. The third one requires the claimant to prove that the prophecy was made not only
before the alleged fulfillment but far enough in advance to preclude the possibility of
educated guessing. These are the two criteria that spell doom for Price's position, because
there is no way that he can satisfy either criterion.

I have shown that the existence of two very different versions of the book of Jeremiah (the
Septuagint and the Masoretic) is sufficient to cast suspicion on the integrity of the text. How
else can these two versions be reasonably explained except by postulating a time when there
was no standardized canon and the scribes and clerics of different theological communities
felt free to alter texts to conform to their particular views? This is certainly a more reasonable
assumption than the ridiculous claim that Yahweh inspired Jeremiah to write two infallible
versions.

The Septuagint problem, however, doesn't even begin to scratch the surface of evidence that
shows extensive editing in the book of Jeremiah. Biblical scholars with no fundamentalist
axes to grind have long recognized that the book of Jeremiah is the work of not one but
several writers. The introduction to Jeremiah in The Interpreter's Bible described the disorder
and confusion that one encounters in reading Jeremiah and then explained why the train of
thought is so hard to follow.

This seeming lack of order arises from the fact that this book, like many others in the Old
Testament, is not the product of one person or of a small group of persons. It is the product of
growth over a long period of time, to which many contributed (Vol. 5, p. 787, emphasis
added).

If Dr. Price is a typical fundamentalist-- and I suspect a professor at a Baptist seminary would
have to be--he will very likely dismiss this as "liberal" rubbish, but that would not explain
why so many biblical scholars view the book of Jeremiah as a composite put together by
several writers rather than the work of a single religious mystic who lived in the 6th century
B. C. Surely, Dr. Price won't argue that this view of Jeremiah is held only by liberal
theologians who purposely look for ways to discredit the Bible, because that doesn't explain
anything. There must be good reasons why so many scholars reject the claim that Jeremiah in

Volume 1990 - 2002 Issue


Page 1005 of 2049
Skeptical Review Edited by Farrell Till
its present form was written by a single author, so to make his case, Dr. Price must show that
those reasons are without merit.

In Who Wrote the Bible? Richard Elliott Friedman explained that careful examination of style,
grammar, syntax, vocabulary, theme, and literary structure enables critics to recognize where
redacting and editing occurred in the biblical text (pp. 130, 137). These methods require an
expertise in biblical languages that laymen do not have, but sometimes the evidence of textual
editing and tampering is so obvious that even a layman can see it. The final chapter of
Jeremiah, for example, is so similar to the conclusion of 2 Kings that even a layman can see
that either one was copied from the other or they were both copied from another source. The
sections are too long to examine in their entireties, but a comparison of the final verses of
both books will illustrate the almost verbatim similarities.

Jeremiah 52:31-34, In the thirty-seventh year of the exile of King Jehoiachin of Judah, in the
twelfth month, on the twenty-fifth day of the month, King Evil-merodach of Babylon, in the
year he began to reign, showed favor to King Jehoiachin of Judah and brought him out of
prison; he spoke kindly to him, and gave him a seat above the seats of the other kings who
were with him in Babylon. So Jehoiachin put aside his prison clothes, and every day of his
life he dined regularly at the king's table. For his allowance, a regular daily allowance was
given him by the king of Babylon, as long as he lived, up to the day of his death.

2 Kings 25:27-30, In the thirty-seventh year of the exile of King Jehoiachin of Judah, in the
twelfth month, on the twenty-seventh day of the month, King Evil-merodach of Babylon, in
the year that he began to reign, released King Jehoiachin of Judah from prison; he spoke
kindly to him, and gave him a seat above the other seats of the kings who were with him in
Babylon. So Jehoiachin put aside his prison clothes. Every day of his life he dined regularly in
the king's presence. For his allowance, a regular allowance was given him by the king, a
portion every day, as long as he lived.

As an incidental aside, I will call attention to the fact that although these passages are almost
identical, one says that Evil-merodach released Jehoiachin from prison on the 25th day of the
month, whereas the other says that the release occurred on the 27th day. This is a curious
discrepancy to find in a book that is supposed to be the verbally inspired, inerrant word of an
omniscient, omnipotent deity, and the problem becomes even more complicated when both
are compared to the Septuagint account, which says that Jehoiachin was released on the 24th
day.

Dr. Price will dismiss these differences on the grounds that inerrancy is not the issue, but, as
pointed out in my second rebuttal, in the consideration of prophecy claims, the issue of
errancy in the Bible can't be just waved aside. To believe Price's fulfillment claim requires a
high degree of confidence in the integrity of the Jeremiah text, but such confidence is hard to
maintain when obvious discrepancies, revisions, additions, and plagiarisms are evident in the
text. Such flaws as these, which can be proven, give sufficient cause to wonder how many
other textual flaws are in the book that cannot be proven.

At any rate, the striking similarities between the two passages quoted above certainly don't
support the fundamentalist claim that a book written in the 6th-century B. C. by the prophet

Volume 1990 - 2002 Issue


Page 1006 of 2049
Skeptical Review Edited by Farrell Till
Jeremiah has been preserved over the centuries essentially as he wrote it. A comparison of
Jeremiah 49 with the book of Obadiah will also show textual similarities that cast further
suspicion on the integrity of the Jeremiah text.

Jeremiah 49:15-16, For I will make you least among the nations, despised by humankind.
The terror you inspire and the pride of your heart have deceived you, you who live in the
clefts of the rock, who hold the height of the hill. Although you make your nest as high as the
eagle's, from there I will bring you down, says Yahweh.

Obadiah 1:2-4, I will surely make you least among the nations; you shall be utterly despised.
Your proud heart has deceived you, you that live in the clefts of the rock, whose dwelling is in
the heights. You say in your heart, "Who will bring me down to the ground?" Though you
soar aloft like the eagle, though your nest is set among the stars, from there I will bring you
down, says Yahweh.

Jeremiah 49:9, If grape-gatherers came to you, would they not leave gleanings? If thieves
came by night, even they would pillage only what they wanted.

Obadiah 1:5, If thieves came to you, if plunderers by night--oh, how you have been
destroyed!--would they not steal only what they wanted? If grape-gatherers came to you,
would they not leave gleanings?

The fundamentalist view of biblical authorship is that Moses, Joshua, Isaiah, Jeremiah, David,
Solomon, etc. wrote certain books whose texts have been accurately preserved through the
centuries, but biblical criticism paints instead a picture of textual evolution over extended
periods of time. Did Jeremiah write both the final chapter in his book and the conclusion of 2
Kings? Did both writers use a common source, did one copy from the other, or did a late
revisionist add the same ending to both books? Did Jeremiah copy Obadiah, did Obadiah
copy Jeremiah, or did the similarities result from the revisions of later editors? How can we
ever know for sure what really happened? But until we do know, Dr. Price's naive view of
biblical authorship must be called into question.

Dr. Price may argue that textual variations such as those that exist between the Septuagint and
Masoretic versions of Jeremiah do not materially affect his 70-year prophecy, because the
prophecy is in both the Septuagint and the Masoretic versions. That may be true, but the most
that this can prove is that the 70-year prophecy has been in the Jeremiah text since the 3rd
century B. C. (when the Septuagint was translated). Dr. Price cannot know if it was in the text
significantly prior to that time. Since Jeremiah was a 6th-century B. C. character, there would
have been three centuries for editors to insert the 70-year prophecy into the text, and this is
exactly what some textual critics believe happened.

The literature on the subject is far too extensive to review, but The Interpreter's Bible
discusses the evolution of the Jeremiah text from the simple scroll that Baruch wrote "from
the mouth of Jeremiah," which was short enough to be read publicly three times in one day
(36:10-22), to the Deuteronomic edition that was compiled toward the end of the Babylonian
exile and even later postexilic editions that followed the Deuteronomic version (Vol. 5, pp.
787-791). If this critical theory is correct, it is quite easy to see how that "Jeremiah" could

Volume 1990 - 2002 Issue


Page 1007 of 2049
Skeptical Review Edited by Farrell Till
have made his 70-year prophecy. It would have been made through a simple process of
retrojection by a postexilic editor, and as long as this possibility has wide critical support, Dr.
Price cannot make a convincing case for his claim that Jeremiah predicted the exact duration
of the captivity.

The Interpreter's Bible identifies sections in chapters 1, 3, 5, 7, 9, 11, 13, 14, 15, 16, 17, 18,
19, 21, 22, 24, 26, 32, 34, 35, 36, 37, 38, 39, 40, 42, and 45, that were added or revised by the
Deuteronomic editor(s). Chapters 30-31, 33, and other passages are identified as postexilic
additions. These are critical determinations that scholars have made through the methods
previously quoted from Friedman (examination of style, grammar, syntax, vocabulary, theme,
and literary structure), so to prove his case, Price will have to demonstrate that these critical
methods are invalid. Otherwise, his prophecy-fulfillment claim stands impeached by his
failure to provide unequivocal proof that the prophecy was made before and not after the
alleged fulfillment

In listing above the Deuteronomic sections of Jeremiah, I purposely omitted chapter 25, in
which textual critics claim that there is a Deuteronomic addition to the book. Verses 1-13 in
this chapter contain "Jeremiah's" prophecy that Dr. Price is defending, but The Interpreter's
Bible attributes this part to the Deuteronomic editor.

In ch. 25, vss. 1-13a in their original form were written by D to provide the conclusion to
what he believed was Baruch's scroll of 604 B. C. The words "this book" in 25:13 refer
backward rather than forward. The Septuagint form of 25:1-13 is closer to the original, as
written by D, than the Masoretic Text. It does not have the references to Babylon and
Nebuchadrezzar, and makes clear that the condemnation was originally directed against Judah
and Jerusalem (Vol. 5, p. 789).

If this were not bad enough for Dr. Price's position, the ditors point out that chapter 29, verses
10-20, were also written by the Deuteronomic editor, and this passage is the other place that
contains "Jeremiah's" prediction that the captivity would last seventy years. None of this is
good news for Dr. Price, and if he wishes to see evidence that supports what he calls a "null
hypothesis," he might want to investigate what textual critics have written about the
Deuteronomic and postexilic editions of the book of Jeremiah. Enough has been published on
the subject to keep him busy for a lifetime.

To conclude this part of my rebuttal, then, I would simply ask Dr. Price to show us undeniable
evidence that satisfies criteria 2 and 3. What is his proof that Jeremiah himself, significantly
before the alleged fulfillment, actually predicted that the Babylonian captivity would last for
70 years? It isn't sufficient that Dr. Price would like for it to be this way. He must prove that it
was this way.

Criterion 1: Even if we conceded for the sake of argument that the 70-year prophecy in
Jeremiah is an authentic utterance of the 6th-century prophet, Dr. Price would still not be out
of the woods. The first criterion of valid prophecy fulfillment, which Dr. Price has said is
"satisfactory," requires that the claimant of a prophecy fulfillment prove that he is properly
interpreting the alleged prophecy, but Dr. Price hasn't even come close to doing that. His own
dating of the beginning and end of the captivity stretched the rules of literary interpretation

Volume 1990 - 2002 Issue


Page 1008 of 2049
Skeptical Review Edited by Farrell Till
beyond the limits of probability but still fell short of 70 years. Obviously anticipating that I
would point this out, he said in his first article that "(t)his may seem to be only sixty-nine
years from our perspective, and thus not an exact fulfillment; however, the number seventy is
a round number that is sufficient for the facts" (March/April 1997, p. 3). You see, in the
Never-Never Land of biblical fundamentalism if a forced interpretation of a biblical passage
solves a discrepancy, the interpretation is always "sufficient for the facts," and this appears to
be the Procrustean game that Dr. Price is playing. If the facts don't fit his theory, he will
simply twist them, stretch them, or truncate them to make them fit. Let's notice how he has
been doing that.

To get even 69 years so that he can claim a "round number that is sufficient for the facts," Dr.
Price dated the captivity from 605 B. C. (the first year of Nebuchadnezzar's reign) to 536 B.
C. This last date was three years after Cyrus conquered Babylon in 539 (Eerdmans Bible
Dictionary, 1987, p. 251), but both 2 Chronicles 36:22 and Ezra 1:1 state that in the first year
of his reign, Cyrus issued his proclamation that released the captives to return to their
homeland. In order to fudge three more years into his scenario, Price argued that (1) the first
year of Cyrus's reign didn't really count, (2) Cyrus's decree probably wasn't issued until late in
the first "real" year of his reign, and (3) a large group of captives probably wouldn't have
arrived in Judah until "some time in 536." Let's examine the merits of these arguments.

When biblicists encounter chronological discrepancies in the Bible, they can usually be
expected to argue that ancient cultures dated events and reigns differently from modern
societies, so Dr. Price has not disappointed us. He said that "(a)mong these ancient
neareastern people, the regnal year of a king was counted from the New Year's day following
the king's ascension to the throne" (May/June, p. 5). Then he "explained" that Cyrus captured
Babylon on October 16, 539, but that the "following New Year's day was not until March 24,
538." So, just like that, Price has lopped five months off the first year of Cyrus's reign and put
us into 538 B. C. I assume that those who have been following this discussion noticed, as I
did, that Price merely asserted, but cited no references in support of his claim, that the
Persians calculated regnal years in the manner he described. I couldn't help wondering how
the Persians would have calculated the first year of Cyrus's reign if he had conquered Babylon
on March 25, 539. Would he have reigned one day short of a year without the Persians
counting this as the first year of his reign?

I'm not saying that Price is wrong about the Mideastern method used in counting regnal years,
but even biblicists themselves will sometimes argue that this method wasn't always used. An
apparent discrepancy exists between Jeremiah 25:1 and Daniel 1:1. The first says that the 4th
year of Jehoiakim's reign was the first year of Nebuchadnezzar's reign, but the other says that
the 3rd year of Jehoiakim was the first year of Nebuchadnezzar. How do biblicists resolve this
problem? You guessed it. They say that Jeremiah was calculating from the actual time that
Jehoiakim began to reign but that Daniel was calculating from the first full year of
Jehoiakim's reign. If their solution to the problem is correct, then it isn't true that Mideastern
writers always counted regnal years in the way that Price claims. According to 2 Kings 8:25,
Ahaziah of Judah began to reign in the 12th year of Joram of Israel, but 2 Kings 9:29 says that
he began to reign in the 11th year of Joram. To resolve this discrepancy, Gleason Archer
argued that the "nonaccession-year system" was used in 2 Kings 8:25 and the "accession-year
system" in 2 Kings 9:29 (Encyclopedia of Bible Difficulties, 1982, p. 206). Hence, biblicists

Volume 1990 - 2002 Issue


Page 1009 of 2049
Skeptical Review Edited by Farrell Till
want to have it both ways. When the accession-year system will help their case, they argue for
the accession-year system (as Dr. Price has done), but when the accession-year system works
against them, they argue for the nonaccession-year system. To prove his case in the matter of
when Cyrus issued his proclamation, Dr. Price needs to present evidence, rather than
speculation, that the writers of 2 Chronicles and Ezra used the accession-year system when
they said that the proclamation was made "in the first year of Cyrus, king of Persia." Without
that evidence, Price's claim is merely unsubstantiated conjecture.

After arbitrarily making the first year of Cyrus's reign his second, Price further argued that
"(i)t is likely that the decree was issued late in the year, because administrative duties would
have been heavy for the months immediately following the conquest of Babylon," so Price
concluded that "less important details, like the affairs of foreign captives, would naturally be
postponed" (p. 5). At this point, speculation was running wild in Price's attempt to shore up
his position, so let's notice exactly what he was asking us to believe. By moving the beginning
point of Cyrus's first year as king from October to March, Price gained a leeway of five
months. Then after this maneuver, he asked us to believe that pressing administrative duties
during this delayed-action first year of Cyrus's reign probably deferred issuance of the decree
until "late in the year." If we assume that "late in the year" was, say, ten months into the year,
then this would give Price ten more months to work with, but the problem is that Price has
offered us nothing but speculation to support his claim. Where is the proof?

If speculation is allowed, I can play that game too and argue that freeing the captives would
"likely" have been a pressing administrative concern of Cyrus and not the "less important
detail" that Price says it was. Although Cyrus had conquered Babylon, Egypt was a
potentially powerful enemy sitting on his southern border, so freeing the Jewish captives
would have reestablished Judah as a loyal buffer state between the Persian Empire and Egypt,
an administrative matter that Cyrus would surely have attended to more expeditiously than
Price contends. Indeed, this is exactly how many historians interpret Cyrus's motives
(Eerdmans Bible Dictionary, 1987, p. 252). According to the Cyrus Cylinder (now in the
British Museum), Cyrus didn't just allow foreign captives to return to their homelands; he also
permitted the Babylonians to continue worshiping their deities (Ibid., p. 251). These
enactments indicate that Cyrus considered civil peace and secure borders a much higher
priority than Dr. Price imagines. Since Cyrus left Babylon in 538 and returned to his palatial
residence in the Median city of Ecbatana (Ibid., p. 251), it is hardly likely that he delayed
issuing his decree until late in 538. At any rate, Dr. Price is arguing for prophecy-fulfillment,
so to make his case, he needs unequivocal supporting evidence. Mere speculation won't
satisfy that requirement.

The long journey home: Dr. Price also speculated that a "large company of returnees" could
not have arrived in Judah "until some time in 536." Ezra indicated that the captives were back
in their cities "when the seventh month was come" (3:1), so if Cyrus issued his decree
reasonably soon after conquering Babylon, the captives could have been back in Judah as
early as 538, so this would have reduced the captivity (if calculated from 605) to only 67
years, a number that wouldn't be quite as "round" as 69. When the captives may have returned
to Judah is irrelevant, however, because the prophecy that Dr. Price is so excited about said
that the nations taken captive would "serve the king of Babylon seventy years" (Jere. 25:11).
After the captives had been released, however much time it may have taken them to journey

Volume 1990 - 2002 Issue


Page 1010 of 2049
Skeptical Review Edited by Farrell Till
back to Judah wouldn't have mattered, because once they had been released, it couldn't very
well be argued that they were still "serv[ing] the king of Babylon." Furthermore, once
Babylon fell in 539 B. C., the captives were no longer serving the king of Babylon, period.
Whatever time they may have remained in captivity after Babylon had fallen would have been
in service to the king of Persia.

Another problem for Dr. Price's pat little scenario of captives straggling back to Judah is in
the repetition of the prophecy in 29:10, where the "prophet" said, "For thus says Yahweh,
after seventy years are completed in Babylon, I will visit you and perform my good word
toward you, and cause you to return to this place." If the captives were released in 538 or 537
and didn't reach Judah "until some time in 536," then it couldn't very well be said that
Yahweh "visited" the captives after 70 years were completed in Babylon. The "visiting"
would have happened after only 67 or 68 years in Babylon had been completed. Dr. Price,
however, will no doubt argue that 67 or 68 would be close enough to a "round number" to be
"sufficient for the facts," but it does seem to me that a prophet predicting by the inspiration of
an omniscient, omnipotent deity could have spoken with a little more precision than this. Why
didn't Yahweh have Jeremiah hit the prophetic nail right on the head so that there would be no
room for nasty skeptics like me to dispute that he was a prophet inspired of God to see into
the future?

The 605 date: There are also serious problems on the other end of Dr. Price's dating of the
fulfillment. As noted earlier, he has contended that the captivity should be dated from 605 B.
C., because this is the year that Jeremiah dated his prophecy: "The word that came to
Jeremiah concerning all the people of Judah in the fourth year of Jehoiakim the son of Josiah,
king of Judah (the same was the first year of Nebuchadnezzar king of Babylon" (25:1). Dr.
Price assured us in his first article that it was in 605 B. C. that "Nebuchadnezzar's father died,
and Nebuchadnezzar ascended to the throne of Babylon" (March/ April, p. 2). It's interesting
to note, however, that with reference to the first year of Nebuchadnezzar's reign, Dr. Price has
said nothing about Mideastern methods of calculating "regnal years," so we have to wonder
why in his dating system, Dr. Price thinks that the first year of Cyrus's reign didn't really
begin until five months after he had conquered Babylon but that the first year of
Nebuchadnezzar's reign should be calculated from the moment that he became king. Well, no,
we don't really have to wonder why Dr. Price calculates in this manner, because we know
why he does. If Dr. Price dated the first year of Nebuchadnezzar's reign from the new year
following his accession to the throne, that would move the beginning of his alleged
fulfillment down to 604 B. C., so he has to stretch the years a bit on each end in order to get a
figure close enough to a "round number" to call it "sufficient for the facts."

According to extrabiblical records Nebuchadnezzar did indeed accede to the throne in 605 B.
C., but this doesn't help Dr. Price's case. The prophecy quoted above says only that the word
of Yahweh came to Jeremiah in the first year of Nebuchadnezzar; it does not say that this
should be considered the beginning of the 70-year captivity he was predicting. In commenting
on Nebuchadnezzar's "conquest of the Palestinian area," Dr. Price said that "(t)he conquest
was temporarily interrupted by the death of his father, but the following year he returned and
completed the conquest" (March/April, p. 2). Price then said that "Jehoiakim became the
vassal of Nebuchadnezzar, and although there were some times of short-lived rebellion, for all

Volume 1990 - 2002 Issue


Page 1011 of 2049
Skeptical Review Edited by Farrell Till
practical purposes Judah was under the servitude of Nebuchadnezzar or the Babylonians for
seventy years, some of that time as captives exported from their native land to Babylon."

In response to this, I have only to note that if Nebuchadnezzar did not complete his conquest
until the year following his accession to the throne, the captivity must be dated from 604 B.
C., not 605, and that will make Price's number a little less round. Also, the prophecy stated
that 70 years would be completed in Babylon, so it hardly matters how much "for all practical
purposes" the Judeans may have served Nebuchadnezzar in Jerusalem. Those years could not
be counted toward fulfillment of a prophecy that said 70 years would be completed in
Babylon.

The really big problem for Dr. Price's beginning date, however, is the fact that "Jeremiah"
clearly indicated that the captivity he was prophesying had begun not with the subjugation of
Judah as a vassal state during the reign of Jehoiakim but from Nebuchadnezzar's capture of
Jerusalem in 597 B. C. To see this, we have only to compare a few scriptures. First, let's look
at a letter that Jeremiah sent to the captives in Babylon:

Jeremiah 29:1-2, These are the words of the letter that the prophet Jeremiah sent from
Jerusalem to the remaining elders among the exiles, and to the priests, the prophets, and all
the people, whom Nebuchadnezzar had taken into exile from Jerusalem to Babylon. This was
after King Jeconiah, and the queen mother, the court officials, the leaders of Judah and
Jerusalem, the artisans, and the smiths had departed from Jerusalem.

The dating in this passage is quite clear. It was written to the captives after king Jeconiah, his
mother, and various Judean officials had been taken "into exile from Jerusalem to Babylon." I
assume that Dr. Price will not dispute the fact that Jeconiah was an alternate name for
Jehoiachin, so I won't waste time showing that the two names were used interchangeably for
the same person. With that in mind, we can look at the passage that describes Jeconiah's
capture:

2 Kings 24:8, Jehoiachin was eighteen years old when he began to reign; he reigned three
months in Jerusalem. His mother's name was Nehushta daughter of Elnathan of Jerusalem. He
did what was evil in the sight of Yahweh, just as his father had done. At that time the servants
of King Nebuchadnezzar of Babylon came up to Jerusalem, and the city was besieged. King
Nebuchadnezzar of Babylon came to the city, while his servants were besieging it; King
Jehoiachin of Judah gave himself up to the king of Babylon, himself, his mother, his servants,
his officers, and his palace officials. The king of Babylon took him prisoner in the eighth
year of his reign.

This text clearly claims that Jeconiah, his family and officials, and other leaders were taken
captive in the eighth year of Nebuchadnezzar's reign. If we accept 605 or 604 B. C. as the
first year of Nebuchadnezzar's reign, then his eighth year would have been in 597 or 596 B. C.
Verse 15 in this passage states that Nebuchadnezzar "carried away Jehoiachin to Babylon; the
king's mother, the king's wives, his officials, and the elite of the land, he took into captivity
from Jerusalem to Babylon." So clearly this was the group to whom Jeremiah addressed his
letter mentioned in 29:1. The full context of this "letter" shows that Jeremiah's 70-year
prophecy was directed to this group.

Volume 1990 - 2002 Issue


Page 1012 of 2049
Skeptical Review Edited by Farrell Till
29:3-10,The letter was sent by the hand of Elasah son of Shaphan and Gemariah son of
Hilkiah, whom King Zedekiah of Judah sent to Babylon to King Nebuchadnezzar of Babylon.
It said: Thus says Yahweh of hosts, the God of Israel, to all the exiles whom I have sent into
exile from Jerusalem to Babylon: Build houses and live in them; plant gardens and eat what
they produce. Take wives and have sons and daughters; take wives for your sons, and give
your daughters in marriage, that they may bear sons and daughters; multiply there, and do not
decrease. But seek the welfare of the city where I have sent you into exile, and pray to
Yahweh on its behalf, for in its welfare you will find your welfare. For thus says Yahweh of
hosts, the God of Israel: Do not let the prophets and the diviners who are among you deceive
you, and do not listen to the dreams that they dream, for it is a lie that they are prophesying to
you in my name; I did not send them, says Yahweh. For thus says Yahweh: Only when
Babylon's seventy years are completed will I visit you, and I will fulfill to you my promise
and bring you back to this place.

So the 70-year prophecy was directed to those who had been taken captive in 597 B. C., and
the use of "you" in the last sentence makes that very clear. Although Daniel 1:1-3 claims that
some of "the royal seed and nobles" of Israel were taken captive when Nebuchadnezzar made
Judah a vassal state in the third year of Jehoiakim's reign (605 or 604 B. C.), the account in 2
Kings 24 states that it was in the 8th year of Nebuchadnezzar that the Judeans were taken to
Babylon in great numbers:

2 Kings 24:13-17: He [Nebuchadnezzar] carried off all the treasures of the house of Yahweh,
and the treasures of the king's house; he cut in pieces all the vessels of gold in the temple of
Yahweh, which King Solomon of Israel had made, all this as Yahweh had foretold. He carried
away all Jerusalem, all the officials, all the warriors, ten thousand captives, all the artisans
and the smiths; no one remained, except the poorest people of the land. He carried away
Jehoiachin to Babylon; the king's mother, the king's wives, his officials, and the elite of the
land, he took into captivity from Jerusalem to Babylon. The king of Babylon brought captive
to Babylon all the men of valor, seven thousand, the artisans and the smiths, one thousand, all
of them strong and fit for war.

Although there is undoubtedly exaggeration in describing the size of the multitude taken
captivity, if there is any truth at all in this passage, then it is reasonable to date the captivity
from the events described here, which would have occurred in 597 B. C. So we have this
description of a massive captivity, and we have "Jeremiah's" letter that was directed to the
people taken captive at this time, to whom he had Yahweh saying that he would visit them
after 70 years had been completed. They both dispute Dr. Price's claim that the captivity
should be dated from 605 B. C. So even if we accommodate Dr. Price and allow 537 B. C. as
the end of the captivity, it would have lasted only 60 years (597 - 537 = 60). Sixty is a nice
round number, but in this case it isn't close enough to 70 to be "sufficient for the facts."

A figurative view: To satisfy criterion 1 in the list that Dr. Price has agreed to, he must prove
that the 70-year prophecy means what he claims it means, but some scholars think that the
number 70 was never intended to represent a literal 70-year period but only a long period of
time. Isaiah prophesied the destruction of Tyre, but, unlike Ezekiel, he didn't rashly predict
that Tyre would never be rebuilt. He said only that it would be "forgotten seventy years" but
that "after the end of seventy years... Yahweh will visit Tyre, and she shall return to her hire

Volume 1990 - 2002 Issue


Page 1013 of 2049
Skeptical Review Edited by Farrell Till
and shall play the harlot with all the kingdoms of the world on the face of the earth" (Isaiah
23:15-17). There were periods in history when Tyre lay in destruction, but it was always
rebuilt and still exists today, so unless biblicists want to admit to a prophecy failure here, they
must claim that Isaiah was using the number 70 only in a figurative sense to represent a
period of time.

In Daniel 9:24, "seventy weeks" was used in an obviously figurative sense to represent a
period of time for which biblicists have had more interpretations than Carter has pills. So if
the number 70 was used figuratively elsewhere in prophetic literature, how does Dr. Price
determine that it was intended in a literal sense in Jeremiah's prophecy? Anyway, if I were he,
I would think about arguing for a figurative application of the number 70 in Jeremiah's
prophecy, because the literal interpretation has obviously backfired in his face.

The Mormon ploy: Dr. Price waved aside my example of a "prophecy" in the Book of
Mormon as just a "ploy" and "an old illustration that [I have] used in other debates"
(May/June 1997, p. 2). To be sure, I have used this example in other debates, but I have yet
for any opponent to respond to the argument that I base on this Mormon prophecy. The
intention of the example is to show that even biblicists are able to see the transparency of
fabricated prophecies in other religious books, but for some reason they can't apply the same
common sense to biblical prophecies.

Price said that because "some Mormon elders evidently falsified records," this does not "give
Till the right to conclude that such falsification has necessarily happened in all religious
literature" (Ibid.). No, but the fact that the falsification of records has happened rather
routinely in religious literature should cause people to think more critically than they do when
reading their holy books. I have certainly presented enough evidence for tampering in the
book of Jeremiah to give Dr. Price pause, so instead of waving the Mormon example aside, he
should explain to us why biblical prophecies that could easily be fabrications and
falsifications deserve more consideration than the prophecies of other religions.

Averaging textual variations: Dr. Price accused me of exaggerating because I mentioned


that over 6,000 textual variations exist between the Septuagint and the Masoretic Text. He
tried to minimize this by pointing out that this would be an average of only 6 variations per
chapter. That may be true, but even 6 variations per chapter in a work purported to be the
inspired word of an omniscient, omnipotent deity is certainly nothing to sneeze at. After all, a
problem is a problem is a problem, whether 6 per chapter or 60 per chapter. Furthermore,
averaging the variations as Price has done merely clouds the picture. Some chapters, and even
some books, have long sections without variations, so this means that other sections have far
more than just 6 variations per chapter, as in the cases of the books of Samuel and Jeremiah.
Omissions that make a book in the Septuagint 15% shorter than the Masoretic version can
hardly be considered an inconsequential matter. What we need is less effort to gloss over the
problem and more effort to offer a reasonable explanation for why an omniscient, omnipotent
deity could not have done a better job of preserving his inspired word so that we could have a
text that has zero variations per chapter.

Numerous Dead Sea Scrolls: Dr. Priced accused me of resorting to misinformation, but in
the same section where he made this charge, he said that "numerous scrolls of nearly every

Volume 1990 - 2002 Issue


Page 1014 of 2049
Skeptical Review Edited by Farrell Till
book of the Old Testament found in the caves of the Dead Sea, dating from the 1st century A.
D. to as early as the 4th century B. C., verify that the Masoretic text in all its essential details
was in existence at that time" (May/June 1997, p. 3). Of course, Dr. Price didn't specify what
the "essential details" of the Masoretic text are, but if misrepresentation is at all possible, we
certainly have it in Price's claim that the Dead Sea Scrolls verify that the Masoretic text was
intact in its "essential details" as early as the 4th century B. C. I assume that Dr. Price will
agree that Frank Cross of Harvard University is a recognized biblical scholar and authority on
the Dead Sea Scrolls. In Understanding the Dead Sea Scrolls, he said this about textual
variations in the Old Testament:

(T)he history of the text of the Hebrew Bible has been confused and obscured by an
assumption, or rather a dogma, on the part of the ancients--rabbis and Church Fathers alike--
that the Hebrew text was unchanged and unchanging, unaltered by the usual scribal realities
that produce families of texts and different recensions in works that have survived over long
periods of transmission ("The Text Behind the Hebrew Bible," Random house, 1992, p. 143).

Prior to this, in the same chapter, Cross had addressed the problem of variations in ancient
versions of the Old Testament.

In the medieval Hebrew manuscripts, there are hundreds, even thousands of differences,
mostly minor, rarely major. In the old versions, especially in the Old Greek version (which
was written beginning in the third century B. C. and is commonly called the Septuagint), there
are thousands of variants, many minor, but also many major (Ibid., p. 143, emphasis added).

What about a 4th-century B. C. verification of the "essential details" in the Masoretic text?
Was such verification obtained from the Dead Sea Scrolls? In the introduction to Cross's
chapter, editor Hershel Shanks certainly gave no support to Price's claim.

The earlier group (from Qumran), however, shows wide variations, sometimes even different
editions of the same book. Cross is able to identify among these different texts three families
of texts that appear to have originated in different geographical localities--Palestine, Egypt,
and Babylonia (emphasis added).

There's no consolation here for Price either. Furthermore, he has probably misled readers into
believing that numerous complete copies of almost every book in the Old Testament have
been found at Qumran, but in reality many of the discoveries have been fragments. In his
chapter quoted above, Cross said that "some 170 manuscripts of biblical books" have been
discovered in the 11 caves of Qumran but that "most of them [are] in a highly fragmented
state" (p. 144). I fail to understand how that "highly fragmented" manuscripts could verify the
Masoretic "in all its essential details," because unless there were fragments of every verse in
the Old Testament in which Price thinks there are "essential details," verification of all these
details simply would not be possible.

Popular Magazines: In my first rebuttal, I quoted an article that Joseph A. Fitzmyer


contributed to America magazine. In the article, Fitzmyer stated that the discovery of a
Hebrew text of Jeremiah at Qumran that was parallel to the Greek (Septuagint) version was
evidence that the "fuller form of Masoretic tradition represents a Palestinian rewording of the

Volume 1990 - 2002 Issue


Page 1015 of 2049
Skeptical Review Edited by Farrell Till
book" (October 31, 1987, p. 302). Rather than addressing Fitzmyer's statement, Dr. Price
dismissed it with the charge that I seem "to rely on articles in popular magazines that are
necessarily limited and slanted" (May/ June 1977, p. 4). Such a comment would be parallel to
dismissing all that Carl Sagan had to say on scientific subjects that he wrote about in his many
articles published in Parade magazine, but surely Price knows that the truth or falsity of a
statement is always independent of its source. Does it matter that Fitzmyer published his
opinion in America magazine? He is a recognized authority on the Dead Sea Scrolls, and in
Understanding the Dead Sea Scrolls, the contributing writers cited Fitzmyer's opinion four
times. I haven't noticed a single reference that any of them made to Dr. Price's opinion of the
scrolls.

Clearly, Dr. Price has failed to make a convincing case for his prophecy claim. Although it
was not my duty to formulate a "null hypothesis" and defend it, I believe that I have, in effect,
done that by showing that the text of Jeremiah is too unreliable to hang on it one's claim that
an ancient tribal deity spoke prophecy through a 6th-century B. C. Hebrew mystic. I have
tried to address all of Dr. Price's major points, and if any were skipped, that was an oversight.
If he wishes to respond to my rebuttals, I will publish his reply in a later issue.

The Sins of the Fathers: Another View


Roger Hutchinson
According to the Bible, does a son inherit the sins of the father? A recent article ("The Sins of
the Fathers," TSR, November/December 1996, pp. 10-11) sought to answer that question.
Because the Bible says nothing directly about a son inheriting the sins of the father, that
question is misleading.

The real question that the author sought to answer was: According to the Bible, can a son be
punished for sins committed by his father? The Bible provides a very clear answer to this:
"The fathers shall not be put to death for the children, neither shall the children be put to death
for the fathers: every man shall be put to death for his own sin" (Deut. 24:16). In case
Deuteronomy was too difficult to understand, God provided us with an example of the law in
action as implemented by Amaziah, a king of Judah.

"In the second year of Joash son of Jehoahaz king of Israel reigned Amaziah the son of Joash
king of Judah.... And it came to pass, as soon as the kingdom was confirmed in his hand, that
[Amaziah] slew his servants which had slain the king his father. But the children of the
murderers he slew not: according unto that which is written in the book of the law of Moses,
wherein the LORD commanded, saying, The fathers shall not be put to death for the children,
nor the children be put to death for the fathers; but every man shall be put to death for his own
sin" (2 Kings 14:1-6).

Volume 1990 - 2002 Issue


Page 1016 of 2049
Skeptical Review Edited by Farrell Till
This illustrates one of the major, consistent themes of the Bible. Every individual is
personally accountable for his/her own sin and this accountability is certain. This can be seen
in the following Scriptures.

"It is appointed unto men once to die, but after this the judgment" (Heb. 9:27).

"For we must all appear before the judgment seat of Christ; that every one may receive the
things done in his body, according to that he hath done, whether it be good or bad" (2 Cor.
5:10).

"The soul that sinneth, it shall die. The son shall not bear the iniquity of the father, neither
shall the father bear the iniquity of the son: the righteousness of the righteous shall be upon
him, and the wickedness of the wicked shall be upon him" (Ezek. 18:20).

Further evidence of this theme is found in an event involving Israel in the Wilderness:

"And it came to pass on the morrow, that Moses said unto the people, Ye have sinned a great
sin: and now I will go up unto the LORD; peradventure I shall make an atonement for your
sin. And Moses returned unto the LORD, and said, Oh, this people have sinned a great sin,
and have made them gods of gold. Yet now, if thou wilt forgive their sin--; and if not, blot
me, I pray thee, out of thy book which thou hast written. And the LORD said unto Moses,
Whosoever hath sinned against me, him will I blot out of my book" (Ex 32:30-33).

The Bible makes it clear that each person is responsible for and accountable to God for his
own sin. No person is accountable for the sin of any other person. The question, then, is: Do
we find a contradiction between this clear expression of accountability and other Scriptures?

To answer this, we must distinguish between punishment for sin and suffering as a
consequence of sin. Suppose a person robs a bank killing two people in the process. He
attempts to escape by jumping into a car that is waiting at a traffic light outside the bank and
forcing the driver to speed off. Bank guards fire at the car as it speeds away causing it to crash
which results in the deaths of both the bank robber and the driver. Was the driver punished for
the bank robber's sin or did the driver, being an innocent party, die as a consequence of that
sin?

In the above example, we see that sin can influence events far beyond the person who
commits the sin. Many illustrations of this can be found in the newspapers. Two men use the
highway for a drag race. One driver loses control of his car, crosses the medium strip, and hits
an oncoming car head on. An innocent family in the oncoming car is killed. A person smokes
in bed and falls asleep. A resulting fire leads to the deaths of the smoker as well as his family.
A man gets married, but after a few years, he walks out leaving his wife with the financial
burden of caring for their children all by herself. As a result, the man's wife and children live
in poverty.

We would not conclude that the innocent people in these examples were punished for another
person's sin. Certainly, a young boy living in poverty because his father walked out on his
family is not being punished for his father's sin. The boy suffers as a result of, or as a

Volume 1990 - 2002 Issue


Page 1017 of 2049
Skeptical Review Edited by Farrell Till
consequence of, his father's sin but he is not being punished as a substitute for the father. It is
evident from these examples that people can suffer as a consequence of another person's sin.

Sometimes people can appear to suffer from the consequences of sin, but they are not exactly
innocent bystanders. This situation is described in the following Scriptures:

"Remember, O LORD, what is come upon us: consider, and behold our reproach. Our
inheritance is turned to strangers, our houses to aliens. We are orphans and fatherless, our
mothers are as widows... Our fathers have sinned, and are not; and we have borne their
iniquities" (Lam. 5:1-7).

In this, we find the lamentation of the children of Israel. Their fathers had sinned against God.
Now, they appeared to face the consequences of that sin. There is more to the story, however,
as we find in the following:

"And God spake all these words, saying... Thou shalt have no other gods before me... Thou
shalt not bow down thyself to them, nor serve them: for I the LORD thy God am a jealous
God, visiting the iniquity of the fathers upon the children unto the third and fourth generation
of them that hate me" (Exodus 20:1-6).

"The LORD is longsuffering, and of great mercy, forgiving iniquity and transgression, and by
no means clearing the guilty, visiting the iniquity of the fathers upon the children unto the
third and fourth generation" (Num. 14:18).

A superficial reading of these passages might lead one to conclude that the Bible is saying
that God will punish the children of the third or fourth generation for the sins of the fathers.
Because the Hebrew word translated as "visit" can be translated as "punish" when a different
form of the Hebrew word is used, many people interpret it to mean punish in the above
verses. That is not the case, however.

Instead, God is saying that He will tolerate the iniquity introduced by the fathers for no more
than three or four generations. In other words, God will allow a father to teach his children to
disobey His commandments, and then allow those children to teach their children to disobey
God's commandments. However, in the third or fourth generation, God will visit the children
with the intent of taking action against the sins of the fathers. God will look to see if the
children are following after those same sins or whether they have turned away from them to
obey God.

"Them that hate me" in Exodus 20:6 refers to the children of the third or fourth generation. If
the children have continued in their father's sins, God implies that He will punish the children,
not because their fathers hated God, but because the children also hate God. Here, we find that
one consequence of a father's sin is the imitation of the father's actions or the adoption of the
father's sins by the children so that the children do the same sinful deeds. People sometimes
complain that they suffer because of what others have done when they really suffer because
they do the very same things.

Volume 1990 - 2002 Issue


Page 1018 of 2049
Skeptical Review Edited by Farrell Till
The TSR article focused its attention on the account of the death of the baby born to David
and Bathsheba as a result of their adulterous relationship. "And Nathan said to David...
Wherefore hast thou despised the commandment of the LORD, to do evil in his sight? Thou
hast killed Uriah the Hittite with the sword, and hast taken his wife to be thy wife, and hast
slain him with the sword of the children of Ammon.... And David said unto Nathan, I have
sinned against the LORD. And Nathan said unto David, The LORD also hath put away thy
sin; thou shalt not die.... Howbeit, because by this deed thou hast given great occasion to the
enemies of the LORD to blaspheme, the child also that is born unto thee shall surely die. And
Nathan departed unto his house. And the LORD struck the child that Uriah's wife bare unto
David, and it was very sick" (2 Sam. 12:7-15).

Was the baby punished for David's sin or was the baby's death a consequence of David's sin?
When confronted with his sin, we are told that David responded saying, "I have sinned against
the LORD." As a result of David's admission of guilt, Nathan responds, "The LORD also hath
put away thy sin; thou shalt not die." In other words, Nathan declares that God had forgiven
David of his sin. With the sin forgiven, God can no longer punish David for that sin, much
less the baby or anyone else. Thus, we know that the baby's death cannot be a punishment for
David's sin.

Here we must distinguish between forgiveness of sin and the effect of forgiveness on the
consequences of sin. David arranged for Uriah to be killed. God later forgave David for that
sin. Did that forgiveness bring Uriah back to life? Of course not.

When a person sins, that sin can set in motion a chain reaction of consequences. Sin earns
God's punishment. That punishment can be avoided through forgiveness. However,
forgiveness does not keep the consequences of sin from continuing or from affecting innocent
bystanders. A man may start a forest fire for which he is later forgiven. However, the
consequences of the fire would continue. The destruction of the trees cannot be undone. With
the loss of the trees, spring rains can cause soil erosion and flooding of streams and rivers that
damages the homes of innocent families. Sin can set in motion a series of events that continue
even when the sin is forgiven.

The Bible agrees with this when it says that people must reap what they sow. "Even as I have
seen, they that plow iniquity, and sow wickedness, reap the same" (Job 4:8). "Be not
deceived; God is not mocked: for whatsoever a man soweth, that shall he also reap. For he
that soweth to his flesh shall of the flesh reap corruption; but he that soweth to the Spirit shall
of the Spirit reap life everlasting" (Gal. 6:7-8).

What we see, then, is that forgiveness of sin does not restrain the consequences of sin. One
consequence of David's sin was that he gave "great occasion to the enemies of the LORD to
blaspheme." David set out only to satisfy his fleshly lusts. He ended up providing the enemies
of God an opportunity to blaspheme. As a further consequence, God told David that the baby
must die, and we are told that God struck the child and the child died. The child died as a
consequence of David's sin.

This brings us to the issue of original sin which God explains in Romans 5. "Wherefore, as by
one man sin entered into the world, and death by sin; and so death passed upon all men, for

Volume 1990 - 2002 Issue


Page 1019 of 2049
Skeptical Review Edited by Farrell Till
that all have sinned.... Therefore as by the offence of one judgment came upon all men to
condemnation; even so by the righteousness of one the free gift came upon all men unto
justification of life. For as by one man's disobedience many were made sinners, so by the
obedience of one shall many be made righteous" (Rom. 5:12-19).

Adam's sin had deadly consequences. One consequence was death for "the wages of sin is
death" (Rom. 6:23). Not only did Adam become subject to death but as a consequence, all
Adam's progeny would inherit death from him. Since there can be no death where there is no
sin we see a further consequence. Adam's nature was altered. He became a sinner and his
progeny would also inherit his sinful nature. They would be sinners even from conception.

The doctrine of original sin, however, goes even further. It says that Adam acted as mankind's
agent, so to speak. This is like a sports agent who negotiates a contract that an athlete then
signs. Whatever the agent requests is viewed as a request from the athlete. As mankind's
agent, Adam's sin became mankind's sin and every individual became subject to punishment
as if each individual had committed the sin instead of Adam. Mankind was a full partner with
Adam in that first sin.

The Scriptures tell us that the son cannot be punished for the sins of the father. The son can be
punished, however, if he commits the same sin as his father even if he does so only because
that is what his father taught him to do. The son can also suffer from the consequences of his
father's sin. Sin can be pervasive and its effects far reaching.

(Roger Hutchinson, 11904 Lafayette Drive, Silver Spring, MD 20902; e-mail


rhutchin@aol.com)

A Problem of False Analogy


Farrell Till
The only thing Roger Hutchinson proved in the foregoing article is that he can't recognize
false analogy. In trying to make the death of David's infant son a consequence of, rather than
a punishment for, his father's actions, Hutchinson tried to equate it with such incidents as a
father who smokes in bed and causes a fire that kills members of his family, or drag racers
who crash and kill third parties. In such cases, the innocent do indeed suffer consequences
caused by the irresponsibility of others, but these are not even remotely parallel to the death
of David's and Bathsheba's son.

In the case of the father who smoked in bed, the cause-effect relationship is obvious. The act
of smoking caused a fire that killed other family members. The irresponsible act of the father
clearly triggered the circumstances that caused the innocent to die, but that was certainly not
the case in the matter of David's and Bathsheba's act of adultery. Hutchinson cannot claim a
cause-effect factor in their sexual act that linked it directly to the death of their son. There

Volume 1990 - 2002 Issue


Page 1020 of 2049
Skeptical Review Edited by Farrell Till
may be cases where adultery results in the transmission of a fatal venereal disease from one or
both parents to the offspring, and these would be proper examples of children who suffer the
consequences of their parents acts. In the story of David and Bathsheba, however, there is
nothing to indicate that the death of their son resulted from a cause-effect factor. To the
contrary, the biblical text clearly states that "Yahweh struck the child that Uriah's wife bore to
David" (2 Sam. 12:15). If Yahweh had not "struck" the child, he would not have died, so the
child's death was not a consequence of David's and Bathsheba's "sin" but rather the result of
Yahweh's intervention for the express purpose of killing the child. This is radically different
from a child's dying from a fire caused by a parent who was smoking in bed or a passing
motorist's being killed as a result of a chain of circumstances triggered by a bank robbery, and
only someone desperate to shore up an illogical belief in the total inerrancy of books written
in ancient, superstitious times would be driven to such analogical extremes as this. To have a
case, Hutchinson would have to establish a cause-effect relationship between adultery and
infant mortality. In other words, he would have to show that just as syphilitic parents can
transmit their disease to their offspring, there is something inherent in the act of adultery that
would very likely cause the death of children born from adulterous relationships, but
Hutchinson knows as well as I that such is not the case. Untold millions of children have been
born of adulterous relationships without experiencing any health problems, much less death.
If we are to believe the biblical account, the cause of death in the case of David's son was the
result of divine intervention: "However, because by this deed you have given great occasion
to the enemies of Yahweh to blaspheme, the child that is born to you shall surely die.... And
Yahweh struck the child that Uriah's wife born to David, and it became very sick" (2 Sam.
12:14-15). The text states very clearly that the sickness, which eventually caused the child to
die, resulted from Yahweh's striking the child. The way that this story is told, if Yahweh had
not "struck" the child, he would not have died, and so there was nothing inherent in David's
and Bathsheba's adulterous act that caused the child to die.

Mr. Hutchinson seems to like analogies, so let's look at one that would be more parallel to this
biblical story than the deaths of innocent people who died from someone else smoking in bed
or drag racing. Let's imagine someone who was not the king of Israel but just an ordinary man
of the times living in Moab, who met a married woman and had a relationship that resulted in
the birth of a son. Now let's suppose that this act of adultery had angered the king of Moab,
who reprimanded the man and then ordered the execution of the child. If that were the case,
would Hutchinson be able to see that the child was punished for the sin of his father? What
we have here is a simple case of a diehard biblical inerrantist who refuses to attribute
unfairness or injustice to the Hebrew god Yahweh no matter how obvious biblical accounts
are in describing Yahwistic actions that are unfair and unjust by any reasonable standard of
conduct.

Besides his false analogies, Hutchinson appealed to other scriptures to try to prove that
David's son was not punished for his father's sin, but all that he accomplished was to
strengthen the errantist position by showing that other scriptures teach that children should
not be punished for the sins of their fathers. If this is clearly what the Bible teaches, then
Yahweh's striking an infant with a fatal illness would have been a violation of his own moral
code, and this is exactly the point that Sol Abrams made in his article: the Bible contradicts
itself by saying in some places that children will not be punished for the sins of their fathers
but showing in other places that Yahweh did sometimes punish children for their fathers' sins.

Volume 1990 - 2002 Issue


Page 1021 of 2049
Skeptical Review Edited by Farrell Till
Hutchinson's ploy in this part of his article is an old inerrantist tactic, which says that
"scripture must interpret scripture," but this is fallacious hermeneutics. If a clear statement in
one biblical passage plainly says X, then it doesn't matter how many other biblical passages
may plainly say not X. Two biblical passages that plainly contradict themselves constitute
contradiction and not a reason to resort to verbal gyrations about "letting scripture interpret
scripture" to try to weasel out of acknowledging that the Bible is inconsistent. In the first
place, this claim that scripture should be allowed to interpret scripture flagrantly begs the
question of biblical inerrancy by assuming that there are no errors in the Bible, and so when
two passages appear to contradict one another, it must be true that one of them does not mean
what it clearly says. Inerrantists themselves would never accord this same consideration to
any other written documents. In other words, they would never agree that if two clear
statements in the Qur'an appear to contradict themselves, one of them must not mean what it
plainly says. No, they reserve this privileged status only for the Bible, and that alone is
enough to show the unsoundness of their hermeneutics.

Hutchinson cited the example of King Amaziah, who secured his hold on the throne of Judah
and then killed those who had assassinated his father but did not kill their children because of
the commandment in the law of Moses that said that children should not be put to death for
the sins of the fathers, and from this example, Hutchinson concluded that David's son could
not have been put to death for David's sin. His line of reasoning seems to be that because
there was consistency between Amaziah's actions and the law that prohibited killing children
because of their fathers' sins, Yahweh's actions in the matter of David's son must have also
been consistent with that law. He doesn't seem to understand that no one is arguing that the
Bible is completely inconsistent. The errantist position contends only that the Bible is
inconsistent and contradictory in some places. The example of David's son, then, must be
evaluated only on the bases of what the Bible says about this specific case and what it says
about the punishment of children for the sins of their fathers. If the case of Amaziah's sparing
the children of his father's assassins conformed to the law of Moses, then there is consistency
between this story and the law. If the case of David's son does not conform to the law, then
there is inconsistency between this story and the law. The matter is that simple, and no
demand to let scriptures interpret scriptures can change the inconsistency one whit.

Hutchinson tried to explain away the death of David's son by arguing that David said to the
prophet Nathan, "I have sinned against Yahweh," to which Nathan replied, "Yahweh has put
away your sin; you shall not die" (2 Sam. 12:13). How convenient! It's too bad, isn't it, that
David's infant son couldn't talk? Perhaps he could have said a word or two that would have
moved the omnimerciful Yahweh to spare his life too. At any rate, Hutchinson has reached
into his bag of inerrantist tricks and pulled out the same old discredited assumption that the
Bible can't contradict itself, because the Bible is inerrant. Just because the Bible teaches that
one will not be punished for sins that have been forgiven does not automatically make this
story consistent with the doctrine. The fact is that Hutchinson has only identified another
problem in the story. Nathan proclaimed that David's sin had been forgiven but then
immediately proceeded to pronounce upon David a punishment for that pardoned sin. I would
assume that Hutchinson will agree that killing a man's son for something that he did would
constitute a type of punishment on the man himself, but whether he agrees or not doesn't
matter. The biblical account clearly indicates that David suffered extreme grief for the
affliction that Yahweh brought upon his son. David "fasted, and went in, and lay all night

Volume 1990 - 2002 Issue


Page 1022 of 2049
Skeptical Review Edited by Farrell Till
upon the earth" (v:16). That hardly sounds as if David thought that Yahweh had rewarded him
for having repented.

At any rate, the matter is not punishment that David may have experienced because of the
death of his son. It is a question of whether Yahweh took the life of an infant for something
that the parents had done, and the story plainly indicates that he did. If Hutchinson still
disagrees, perhaps he would like to answer a question: Is there anything in the story to
indicate that this child would have died if he had not been born of an adulterous act? If not,
then what else can we call this but a clear example of a child being killed for the sins of his
parents?

In a final act of desperation, Hutchinson wagged in the doctrine of original sin to draw an
analogy between the consequences of Adam's sin that all humans suffer and, presumably, the
consequences of David's sin that his son suffered. Many Christians doubt that the New
Testament even teaches the doctrine of original sin, but for the sake of argument, I will
assume that it does. All that this doctrine does is put before biblical inerrantists another
contradiction to explain, because it poses the same problem for Hutchinson as the case of
David's infant son. It teaches that the children of Adam must all die for something that Adam
did, and that is inconsistent with Deuteronomy 24:16 and Ezekiel 18:20, which both say that
the sons shall not bear the iniquity of their fathers. The doctrine of original sin teaches that
Adam sinned, and so all of Adam's descendants had to die. If that isn't a case of children
being punished for something their father did, what is it?

Hutchinson has wandered here and there all through the Bible and wound up explaining
nothing. In other words, Hutchinson is a typical Bible inerrantist.

Why Did the Apostles Die?


Dave Matson
A favorite argument for Jesus's crucifixion claims that his disciples would not have risked
their lives for the cause had they known that the resurrection was a hoax. Christian lore has it
that most of Jesus's disciples were martyred. Accordingly, we should accept his resurrection
as the truth. After all, Jesus's own disciples must have believed it, since they pressed on in the
face of persecution. Liars or pranksters would have folded their tents and fled rather than be
persecuted for something they didn't believe in.

As Stephen Sommers noted in his letter, the claim that Jesus's disciples were martyred is
unproven and highly questionable. We have only the word of early Christian writers, many of
whom were given over to pious fabrication. At best, early Christian martyrdom was greatly
exaggerated. Thus, for all we know, the disciples of Jesus may well have been scoundrels of a
sort who thrived in the limelight. It would not have been the first time that a cult stretched the
truth for its own glory. After their leader was crucified, they could have simply spread the

Volume 1990 - 2002 Issue


Page 1023 of 2049
Skeptical Review Edited by Farrell Till
word that he had arisen on the third day. Mighty Jesus had arisen, and they were his special
disciples! Being an inconsequential group, nobody would have bothered to investigate them.
Several minor cults were probably making similar claims. Later, as the cult gained in size and
became "respectable," any serious investigation was out of the question. True believers would
have assumed positions of authority and filled in Jesus's history according to their own
doctrinal understanding.

The above scenario assumes intentional dishonesty on the part of Jesus's disciples, an
unnecessary assumption from the skeptic's point of view. Neither is it necessary to challenge
the assumed historicity of the martyr stories. Perhaps the disciples initially believed Jesus's
claims and were later too entrenched in their belief to admit that they had been wrong.
Psychologically, they may have had too much at stake to simply back out. Perhaps, after a
period of initial depression and confusion, they had forced themselves via group
reinforcement to believe that Jesus had risen even though none of them had actually witnessed
the event. One or more of the group may have mistakenly identified a perfect stranger at a
distance as Jesus, only to lose him to a crowd. Perhaps one of them was out fishing and saw
Jesus amidst the waves. They might have been seeing Jesus under every tree and behind every
bush, only to have their hopes dashed--until a fateful combination of events "confirmed" a
sighting. Within a cult group, the flimsiest evidence can easily evolve into proof, so that the
group ends up convincing the individual of his own tentative hallucinations or delusions! As
absurd as this scenario may seem to the reader, it is wholly consistent with known group
psychology. The power of true believers to fool themselves, especially in a cult group, can
never be underestimated. Reverend Robert M. Price (Beyond Born Again, 1993) has probed
this scenario. By quoting scientific studies and pulling together historical information on
messiahs who arose after Jesus, Price has shown just how easily this sort of thing can happen.
It has historical parallels. Look at the Jehovah's Witnesses for a quick example, that of their
invisible return of Jesus! (See also pages 34-35 in G. A. Wells' Who Was Jesus? for a
discussion of witness psychology.)

Consider the recent tragedy at Rancho Santa Fe, just north of San Diego. There 39 people
committed suicide, apparently in the belief that their physical death was a necessary step
before being picked up by a spaceship traveling in the tail of comet Hale-Bopp! You might
ask how anyone in their right mind could possibly believe such a thing, but is their belief any
more absurd than a belief in biblical inerrancy? There is absolutely nothing about the Bible
that even remotely justifies it as a divine, inerrant work. Yet, millions of people have been
brainwashed into believing that it is the greatest thing that was ever written!

The main difference between the two beliefs is that Bible-belief usually doesn't prove fatal to
the believer. Its damage is chiefly limited to our educational system and to good government
(church-state separation). Nor can we dismiss those 39 men and women as retarded idiots,
since they were computer programmers. Note the heavy use of small, group environments by
both Bible-believers and this UFO cult, which is ideal for restricting contact with the outside
world even as it reinforces the group's beliefs in the individual. Thus, Bible-believers have a
lot more in common with those UFO folks than they would care to admit, and, at times, it
shows in crazy individual or group actions.

Volume 1990 - 2002 Issue


Page 1024 of 2049
Skeptical Review Edited by Farrell Till
The above two scenarios, of course, assume that we have an eyewitness account that has been
passed down to us more or less intact. Perhaps Jesus was originally a teacher with no
pretensions of godhood, who taught that the world was coming to an abrupt end. The entire
idea of Jesus's resurrection may have been added to deify him, to put him on par with the
popular savior gods of that era--the competition as it were. Such doctrines as the resurrection
may have been fixed decades later in distant cities, for the most part, by people who had their
own agendas. We tend to forget that our standard view of Jesus today is a product of later
centuries. Movies and books have given us a false familiarity with those times; what we
actually know about Jesus, if anything, is what has survived the purges of the first few
centuries. If we could actually go back in time, we might find that some early Christian
communities viewed Jesus only as a teacher, that his crucifixion played no doctrinal role for
them.

It is naive to think that the truth would have gotten back from a few elderly disciples to
destroy such developments. We know of historical groups where legend developed despite the
active protests of the group's living founder! It is not a matter of memories going bad but
rather of enthusiastic supporters, out of touch by way of geography or time, who are only too
happy to write their own version. Popular views often persist even in the face of repeated
denials by the key players. Thus, Jesus's few surviving disciples may have actually denied that
Jesus was resurrected when the matter came up, their voices a feeble protest amongst the
increasing din of popular approval.

Finally, a minority of Bible scholars believe that Jesus never existed. The historical evidence
is essentially non-existent as G. A. Wells and others have shown, adding plausibility to this
view. We may be dealing with a myth that accumulated historical justification over time. It
would not be the first time such a thing has happened. Perhaps the mythical Jesus came into
being as a popular savior god 130 years before the time the "historical" Jesus was supposedly
born. How odd, that the earliest literature on Jesus, written by Paul, takes a dry, doctrinal
view devoid of the glorious miracles that Jesus supposedly did. Perhaps the myth grew
"historical legs" as the competition got tougher. The larger the Christian circle became the
more contact it would have had with the competing religions. Thus, Jesus's disciples may
have never existed.

The above argument for Jesus's resurrection has more holes in it than a slab of Swiss cheese.
That is to say, we have several plausible scenarios that undermine its supposed proof. The
burden of proof is obviously on the Christian apologist. It is he who claims to have a good
argument for Jesus's resurrection. Therefore, the skeptic is not obliged to disprove the
Christian argument; he or she need only show that it has no force, that there are other
plausible scenarios that could easily circumvent its chain of reasoning. And that is exactly
what I have done, to the extent that a letter will allow.

(Dave Matson, P. O. Box 61274, Pasadena, CA 91116; e-mail,


103514.3640@compuserve.com)

Editor's Note: Because of its length and its relevance to my article "How Did the Apostles
Die?" (p. 1), Dave Matson's letter has been published as an article. The new booklet by The
Oak Hill Free Press mentioned in the last issue of TSR will soon be available. The Bible,

Volume 1990 - 2002 Issue


Page 1025 of 2049
Skeptical Review Edited by Farrell Till
Common Sense, and the American Way, 72 pages, is priced at $4.95. It is based on powerful,
common-sense observations, which reject the Bible as a divine product. Each of the
arguments is fully developed with special emphasis on the usual apologetics. It should prove
to be a very handy and devastating booklet, just the type of thing you always wanted to lend
your favorite Bible-believer.

From the Mailbag


A Possible Debate...

An atheist wrote to me and said that you debate Bible-believers on the subject of "inerrancy,"
and I'd like to challenge you right now to a public debate--with someone else! His name is
Vin Lewis, and he is founder and president of All Roads Ministry. You can contact him
yourself (address deleted), or let me act as go-between to set up this debate. Please let me hear
from you A.S.A.P., as I am anxious to see who wins! Thank you.

(Bob Holmes, 3200 North Vermont Street, Arlington, VA 2207.)

EDITOR'S NOTE: This letter, dated January 24, 1996, was found when I recently reorganized
my desk. The volume of regular mail I receive, in addition to e-mail and the editing and
business responsibilities associated with publishing The Skeptical Review, sometimes results
in mail getting lost under the paperwork on my desk. This happened with Mr. Holmes' letter,
and I have already sent him my apologies.

I also notified him that I would be glad to debate biblical inerrancy with Mr. Lewis, and pay
my own expenses to the debate, as I normally do. I predicted that Mr. Lewis would not agree
to debate, but about a week after I mailed the letter, Mr. Lewis called to say that he is willing
to debate. We have agreed to have the debate in Virginia, somewhere near Washington, D. C.,
and we are now negotiating propositions and format.

Are Christians Rational?

I never seem to find fault in your arguments, but I must admit all rules can be broken. On
page 5, of the January/ February issue of The Skeptical Review, I find an oxymoron: "I do
wish that Christian apologists could think a little more rationally...." Rationally is not an
adjective that can be used with Christian!

Just about given up hope? I suppose if Christians can believe in a god, then I guess it's okay to
hope to find a rational one.

As for Perman's "tomb" theory, of course, they buried their dead in tombs. The land is just a
rock field, so any digging could be considered a tomb. It was probably just a cave. If all dead

Volume 1990 - 2002 Issue


Page 1026 of 2049
Skeptical Review Edited by Farrell Till
in that era were in tombs, wouldn't they be everywhere? After all there were millions of
people living, and they all had to die.

The stated fact that the "legs [of Jesus] were broken" is typical of crucifixions. It made 'em die
faster.

A personal thanks for your insightful reviews. Question? Do you tutor students? You don't
want to take all that knowledge to the grave, do you? (Or is it a tomb?)

(Larry Laird, 7500 Creager Road, Waynesboro, Pa 17268-9748; e-mail


sofrank@atheism.com)

EDITOR'S NOTE: I tutor no one except to the degree that this can be done through TSR. For
one thing, I have no time for tutoring. (Why didn't someone tell me how much fun retirement
would be?) I don't want to take my knowledge to the grave, but I don't have much choice.

The Emperor's Clothes...

I have recently learned about your publication through your web site. Oddly enough, I learned
about it through Greg Koukl's Stand to Reason web site, www. str.org, a Christian apologetics
organization. Whichever way, I am happy to have found your articles. I do have some
reservations about some of your articles, however. My main reservation is that biblical
errancy efforts seem somewhat like criticizing the clothes the emperor is no longer wearing.
The issue isn't just whether this verse or that is flawed; I find scholarly textual analysis of the
Bible is miles beyond this.

I was a thoroughly serious, studious Christian up to the age of 24, that is, until I started doing
my undergraduate engineering studying on the third floor of the University Research Library
at UCLA. I would procrastinate by picking up a book or two on Biblical analysis. Almost the
entire floor is dedicated to research on the Bible as if it were some sort of archeological find.

What I found is that for more than 70 years, biblical scholars have been amassing a growing
consensus on a core set of theories on the evolution and development of the biblical texts that
are all but incontrovertible. In very general terms, I derive from this consensus the following
set of near-facts: (1) The synoptic gospels (Matthew, Mark and Luke) and the gospel of John
simply cannot be reconciled in their details of the storyline of Jesus's ministry. (2) As for the
synoptic gospels, Matthew and Luke agree in order of events only when the events are also
contained in Mark. These agreements reflect textual agreement rather than thematic
agreement, which indicates they copied the text of a document [Matthew and Luke copied
Mark or its precursor document] rather than gathering independent facts and accounts. In fact,
scholars seem to agree that the copying was of a written document not the copying of an oral
tradition. (3) To the extent the rest of Matthew and Luke agree in subject matter [albeit out of
order], the agreements again reflect the copying of another single written document, termed
Quelle or Q, which was identified in 1838 by Christian Weisse. Today, Q is well accepted by
biblical scholars. (4) The rest of Matthew and Luke disagree irreconcilably. Mostly, this is the
nativity and the resurrection, I believe. The strong implication is that the synoptic gospels

Volume 1990 - 2002 Issue


Page 1027 of 2049
Skeptical Review Edited by Farrell Till
contain agreement only because Matthew and Luke copied two existing texts, Mark and Q.
Apart from this large-scale copying, at least one of the copying authors (Matthew and/or
Luke) didn't know the first thing about Jesus's life and times. (5) In the midst of all this
literary copying, the amount of literary license that the authors of Matthew and Luke granted
themselves, altering the meaning of Jesus's words and deeds, should by itself be sufficient
reason to doubt every verse of the Bible. (6) Mark is a set of short stories which themselves
may contain some degree of accurate oral tradition, but the connective tissue between each
story, the overall storyline of Jesus's ministry, was shown to be the author's own creation as
early as Karl Ludwig Schmidt in 1919. (7) Q is an interesting document, parts of which can
be pieced together from Matthew and Luke. It appears to have had its own evolution, but the
earliest version of Q may well have been accurate, even nearly contemporaneous, reflections
of Jesus's sayings. (8) The gospel of John also contains facts and subtleties that indicate that
its earliest version may have contained reliable and accurate information of Jesus's life; at
least it enjoys more corroboration than Mark. In John, however, Jesus talks just like the author
writes when the author is writing in his own voice. Much has been written on this point. It is
possible, however, that the author was a close follower of Jesus and so aligned in thought with
Jesus that he sounds like Jesus. There is evidence that Johnlike cosmic discourse was alive
and well in the Essene sect, which was centered near the desert region where John the Baptist
and Jesus were claimed to have begun. (9) The Acts of the Apostles is a classic deification of
the apostles and Paul. This was a common occurrence in literature about nonbiblical figures
throughout the Roman empire in that era. Few scholars address Acts as much more than
whole-cloth fiction. Unlike the Acts story, Paul never mentioned any blinding revelation from
Jesus when he was desperately trying to justify his authority in Galatians, 1 Thessalonians and
elsewhere. Also, his description of his compromise with the Jerusalem "pillars" pales in
comparison with his near coronation found in Acts. (10) The authentic Pauline letters [1
Thessalonians, 1 & 2 Corinthians, Philippians., Philemon, Galatians and Romans] are
probably the most reliable documents in the New Testament. They show a quickly evolving
mind. Some of his earliest thoughts were very scary, e.g., 1 Thessalonians 4-5; others were
just screwy, e.g., Galatians 3-4. Generally, however, it appears Paul never knew the first thing
about Jesus's teachings, nor did he get along with hardly anyone who knew Jesus. In fact, Paul
reduces Jesus's entire life and ministry to simply the idea that he was sinless, was crucified,
and was resurrected for our sins. Paul's letters include some later editorial insertions, such as 1
Thessalonians 2:14-16. (11) Several books attributed to Paul sound nothing like him, such as
Colossians, Ephesians, Hebrews, 1 & 2 Timothy, and Titus. (12) Second Peter includes nearly
all of Jude, which is an entirely unnoteworthy book of its own. (13) Revelation is the rantings
of a madman.

There is certainly much more that can said about any one of these points, and other points not
included above. I would just turn the attention of anyone interested in these points to authors
like Burton L. Mack, John Dominic Crossan, and Paula Fredriksen, whom I find most
accessible to most of us. Alternatively, the reader could go to the nearest university research
library. However, in such libraries, you'll be hard pressed to find anyone that seriously
analyzes simple questions of errancy.

(Kelly Cunningham, 1610 Granville Avenue, Apt. 1, Los Angeles, CA 90025; e-mail
acandkc@aol.com)

Volume 1990 - 2002 Issue


Page 1028 of 2049
Skeptical Review Edited by Farrell Till
EDITOR'S NOTE: The points that Kelly Cunningham raised are too numerous to discuss
individually, but they are all problems that biblical scholars have long recognized. Many
major seminaries now teach their students this realistic view of the Bible, and the traditional
biblical inerrancy doctrine survives primarily in fundamentalist Bible colleges. It is interesting
to note that Mr. Cunningham was a Christian until he took the time to research the Bible
seriously. In the process of his research, he saw things that are incompatible with the inspired,
inerrant view of the Bible. This is a familiar story.

Well, Maybe Not All Seminaries...

Please remove my name from the mailing list.

(George Faull, Summit Theological Seminary, 198 Airport Road, Peru, IN 46970.)

EDITOR'S NOTE: Mr. Faull's name was removed. He was added to the list at the request of a
subscriber to whom he had given religious literature. Apparently, he doesn't believe in
reciprocity, so Summit Theological Seminary may not be one of the enlightened institutions
that teach a scholarly view of the Bible.

Happiness Found "out of" Christ...

I have enjoyed reading The Skeptical Review since 1994. Your articles are timely,
informative, and satisfy my need for a clear and logical systematic refutation of the claims of
the Bible and of its believers. Please find enclosed a money order to cover the cost of
resubscribing for another year.

If I am "addicted" (as some will say) to Farrell Till's "Review," it is because of the good effect
it has had on my life over these past few years. The more of Till's philosophy I adopt, that
much more is my behavior positively modified. I can attribute much of my success at
becoming more unfettered in the mind and less demanding on my physical body to Farrell
Till's contribution. For this, I wish to thank him publicly in this column, i.e., "The Mailbag."

I was a Bible-believer of the Jehovah's Witness type from 1987 to 1992. In 1992, I came to
the realization that my leaders were liars and that I had to tell my "brothers and sisters" that
this was so. I started writing letters to them to warn them of the lies being told by the
governing body, only to find myself excommunicated two weeks into my campaign.

From 1992 to 1994, I searched for the "true truth" of the Bible. For a while there, I thought I
had found it in the Fundamentalist Protestant Baptist Christian Church, but then I met Farrell
Till in his devastatingly logical publication, The Skeptical Review. Now I know what that
"true truth" of the Bible really is: It is not to be trusted.

(Rob Savarie, 206-518 Morris Street, Sudbury, ON, Canada P3B 1C1)

Volume 1990 - 2002 Issue


Page 1029 of 2049
Skeptical Review Edited by Farrell Till
EDITOR'S NOTE: A familiar Christian claim is that happiness can be found only "in Jesus,"
but as long-time subscribers know, we have published many letters from former Christians
who consistently tell us that escaping from the fetters of religious superstition brought them a
satisfaction and happiness that they had never found in Christianity. Mr. Savarie's letter is just
one of the latest.

A Christian Subscriber...

Thank you for the complimentary subscription to The Skeptical Review.. I have enjoyed
receiving it and look forward to the well written articles that you include in each issue. I am
enclosing a check in the amount of $12 for a two-year renewal of this subscription.

A short bit about me. I am a Christian, a preacher, and someone who does take the Bible to be
the Word of God. For the past year, I have been challenged by your newsletter and forced to
take a close look at the truths that I have held for so long with so little thought. As I have
looked at these truths, I have become stronger and better equipped for the preaching of the
Word. I have you and your newsletter to thank for part of that growth. That is why I am
renewing my subscription; so the Lord can continue to use it to cause growth in my ministry
and spiritual walk.

Thank you for the work that you do in preparing such an excellent newsletter, and for your
own challenging articles. Best wishes to you and to your family. Rest assured and rejoice that
you are being used of God.

A good friend of mine resides in Canton, Illinois. He is named (deleted). Just curious if
perhaps you know him.

(Richard L. Johnson, 1820 Highland Avenue, Burlington, IA 52601-4528)

EDITOR'S NOTE: I appreciate Mr. Johnson's letter, and all I ask of him is that he continue to
read TSR and give the articles in it a fair hearing. I'm not being sarcastic when I say this, but if
he will give the articles an unbiased appraisal, I think he will eventually change his opinion of
how God is using me. As Rob Savarie did immediately above, too many people have testified
to the help that TSR has given them in throwing off the fetters of religious superstition for me
to believe that God is "using me" to help his cause.

I did not know Mr. Johnson's friend, but I would be glad to talk to him if Mr. Johnson wants
to make the arrangements. As a matter of fact, I live only about 90 miles from Burlington,
Iowa, so if Mr. Johnson would ever like to talk personally about his beliefs, biblical inerrancy,
my views, etc., I would be glad to go to Burlington for private discussions.

Maybe God Can Use Me Here Too...

Greetings from Texas. I've thoroughly enjoyed your most insightful publication, and now I
would like very much to take this opportunity to request a subscription for a lady friend of

Volume 1990 - 2002 Issue


Page 1030 of 2049
Skeptical Review Edited by Farrell Till
mine who is presently on the downside of throwing off her Christian belief. I think she will
read your publication with great interest, and, finally having been thoroughly convinced, will
discard her Christian beliefs once and for all. This is just the kind of information she needs.
I've noted her address below (deleted).

Thank you for your time and assistance here, as it is genuinely appreciated. I know she'll look
forward to hearing from you.

(Steve Armistead, #465704 Stiles, Route 4, Box 1500, Beaumont, TX 77705)

EDITOR'S NOTE: Hmmm, perhaps God can use me here to restore this lady's faith. I'm
sorry, Mr. Johnson, but I just couldn't resist.

A Postcard from Japan...

In your reply to Mr. Boonsaayer (November/December 1996), you said, "I have never been to
Tokyo, Japan, but I believe there is a Tokyo, Japan." Since we are visiting here, I just wanted
to send you a postcard to let you know I checked and there is indeed a Tokyo, Japan. Keep up
the good work.

(Bob Oliver, 607 North 38th Street, Nixa, MO 65714)

EDITOR'S NOTE: The postcard showed scenes from Tokyo on one side, with the scenes
identified on the back. Now if only someone could produce evidence just as convincing that
Moses parted the Red Sea or that Jesus was born of a virgin, raised Lazarus from the dead,
and then rose from the dead himself, I could close down my operations here and do something
else with the rest of my life.

More about Converted Atheists...

Thank you for your comments to my first letter in the January/February issue. Yes, I was
referring to your note at the end of Mike Ulm's letter on page 13 of the May/June 1996 issue:
"I personally don't know a single person who was once a committed atheist but is now a
Christian." I offered you several names of famous atheists later converted to Christianity
(Charles Foucault, Jacques Maritain and wife, Andre Frossard, Vittorio Messori, Max Jacob,
Evelyn Waugh). A few days later, I wrote to you again with two more cases: Joseph Joubert, a
philosopher, disciple and collaborator of Diderot and a self-confessed atheist before his
conversion, and Henri Gheon, who changed from a Nietzschean atheism to Catholicism.
Maybe, I added, there are more atheists who converted to Christianity, such as Louis Veuillot
or Huysman.

With this information, I only intend to explain to you that conversion is not such a strange
phenomenon as your note could invite to think. As far as I know, these conversions did not
happen on the deathbed. Recently, I learned about Dr. Bernard N. Nathanson, who has
performed more than 60,000 abortions. After being an atheist for the largest part of his life, he

Volume 1990 - 2002 Issue


Page 1031 of 2049
Skeptical Review Edited by Farrell Till
has become a Catholic in December 1996. The news appeared in The New York Times, The
Washington Post, etc.

In my letter, I did write that "Voltaire was never an atheist, in spite of what many people
think," so it is I who am most puzzled to read that I cited him as an example of an atheist who
converted to theism. I only said that "Voltaire asked for pardon to God and to the Church...."
If I referred to Voltaire, it was because you mentioned him as an example of the atheists or
freethinkers about whom tales are told, "even though people present at the times of death
denied that any such changes occurred." Instead of your attempts to find faults in my
reasoning, I would have preferred learning about those witnesses who, according to your note,
denied Voltaire's repentance. But, quite surprisingly, it is you who complain because I did not
document my assertion. Well, I invite you to read Voltaire: A Biography by Haydn Mason,
Granada Publishing, Ltd., London, 1981. His sources are Voltaire et la Societe au XVIII
Siecle (Voltaire and Society of the 18th Century) by Desnoiresterres, Paris, 1867-76, La
Religion de Voltaire, by R. Pomeau, Reflections at the Death Bed of Voltaire: The Art of
Dying in Eighteenth Century France, Oxford, 1975.

I already knew the reason why so many atheists and freethinkers admire Tom Paine, his
opposition to organized religions mainly, but I keep on astonished, as I told you, because
Paine, your Patron Saint, was also against atheism, and his book Age of Reason aimed, as he
said and I repeat now, "to keep France from Atheism." As to Voltaire, after having read
Systeme de la Nature, by d'Holback, [he] decided that he had to fight in two fronts: "against
l'Infame [the infamous] and against atheism, as the threat of the latter is a danger so great as
that coming from the first one" ("l'infame" could be the Catholic Church or organized
religions in general; critics are not sure). Is there any atheist who can explain to me why they
pay no attention to what their "patron saint" or their admired Voltaire wrote about them?

It was yourself who wrote that "by the very nature of their temperaments, fundamentalists
have to believe that they are right" and that "because I was once a fundamentalist myself and
understand the way they think, especially their need to be right...." No, I am not suggesting
that Dan Barker or you would not feel the need to be right. In fact, with your words, you are
confirming my opinion that some people (fundamentalists) need affirming faith categorically
while others (atheists and freethinkers) need denying it so categorically as they. In other
words, if I am right, belief or unbelief would be a matter of temperament rather than of
reasoning.

Many thanks again for your comments, but please note that my first family name is Casao, not
Ibanez.

(Antonio Casao, Apartado 882, 50080 Zaragoza, Spain)

EDITOR'S NOTE: Whew, where should I begin? Well, I will first apologize for incorrectly
assuming that Mr. Casao's surname was Ibanez, but I trust that he will understand that the
English system puts the person's surname last. Next, I think a comment on one of Mr. Casao's
final remarks is in order. He contends that fundamentalists by temperament have a need to be
right and that atheists feel a need to "deny"; however, it seems to me that if anyone feels a
compelling need to be right, it would be Mr. Casao, because Mike Ulm asked me if I knew of

Volume 1990 - 2002 Issue


Page 1032 of 2049
Skeptical Review Edited by Farrell Till
any atheists who had converted to Christianity, and I said that I personally knew of no
"committed atheists" who had converted. Mr. Casao may believe what he wants to, but that is
a true statement. I personally know of no committed atheists who have converted to
Christianity. I know of some Christians who claim that they were once atheists. In fact, it has
become rather fashionable for Christians to make this claim, but of all the Christians who
have told me that they were once atheists, not a single one of them had prior histories of
having shown dedication and commitment to atheism.

Perhaps Mr. Casao just didn't understand what I meant by "committed atheists." When I refer
to committed atheists, I mean atheists who have studied the major arguments for the existence
of God and the many refutations that have been published and use this knowledge to try to
educate others to the fallacies in theistic thinking. Since I don't even know the backgrounds of
the converted atheists that Mr. Casao has listed in his letters, I have no way of knowing if they
were ever committed atheists in the sense that I use the term. Of all the Christians who have
said to me that they were once atheists, I had never heard of any of them. Since I have done
extensive reading in freethought and atheistic literature, I'm sure that I would have
encountered their names if they had been committed atheists before their conversions. It
seems to me that Mr. Casao is trying to make an issue where none exists. I was asked if I
knew of any committed atheists who have converted to atheism, and I said that I didn't. And
that was the truth.

Once again, Mr. Casao expresses wonderment at the respect that freethinkers and atheists
have for Thomas Paine, whom he refers to as our "patron saint." He pointed out that Paine
said that he wrote Age of Reason to try "to keep France from atheism." I taught American
literature for 30 years at a nearby college, and sections from Age of Reason were were always
studied during the unit on literature from the Revolutionary War period, so I certainly am
familiar with Paine's position on atheism. Despite what he thought about atheism, however,
Paine said relatively little in the book to oppose atheism. Primarily, it was an attack on the
ridiculous belief that the Bible is "God's word," and Paine demolished this cherished belief
with compelling arguments that Christians have yet to respond to satisfactorily. Since
freethinkers and atheists in America recognize that fundamentalist Christianity, which
champions belief in biblical inerrancy, poses real threats to our personal freedoms, that is one
of the main reasons why we admire Thomas Paine. That admiration, however, doesn't require
us to accept everything that Paine said. If Mr. Casao wants to think that we have made Paine
our patron saint, he is certainly entitled to that belief. As for me, if I may take liberties with
one of Paine's better known statements in Age of Reason, I will simply say that I don't believe
in patron saints. My own mind is my own patron saint. If Paine and Voltaire thought that
atheism was a philosophy harmful to society, the freedom of my mind entitles me to disagree
with them. Why would Mr. Casao think that this needs explanation?

I'd Rather Be Right...

No doubt about it, you are from Missouri! I remember a fair amount of Truman's presidency,
and he indeed said a lot of quotable things. He and Bess were marvelously colorful... and
gutsy. Henry Clay, who kept running for president, and losing, is generally credited with

Volume 1990 - 2002 Issue


Page 1033 of 2049
Skeptical Review Edited by Farrell Till
saying, "I'd rather be right than president." This has routinely been perceived as sour grapes
on his part.

We are in 100% agreement. I can't imagine why anyone would put religious or philosophical
beliefs above the desire to be right, but obviously many people do. Oh, yes, I would guess
upwards of 90%, because of the power and wealth involved, would prefer to be president
rather than be right. A Clinton joke would be appropriate here, but I'll refrain.

I don't know about all these so-called atheists who became deathbed Christians. One must
share your skepticism, but the skeptic I think would be most thought of by you personally
would be T. S. Eliot, the great one. Surely he was minimally an agnostic. Super poem, "The
Hippopotamus"! But eventually he went over to the church he so criticized. A fellow
Missourian, I simply do not recall you or any of your readers mentioning him. He has a big
fan here, for over 40 years.

(Robert J. Zani, Michael Unit, 12-F-18, Route 1, Box 4500, Tennessee Colony, TX 75886.)

EDITOR'S NOTE: I checked in two volumes of famous quotations, and Mr. Zani is right.
Henry Clay, a senator who ran for president three times and lost, was the one who said, "I'd
rather be right than president." He said it in a senate speech in 1850. Although Mr. Zani
probably said in jest that I must be from Missouri, I am in fact from Missouri. I was born in
the boot heel and lived there until I left for college in Tennessee when I was 18. I am so
Missourian, in fact, that I actually know how to pronounce the name of the state, which is
miz-ZOUR-uh rather than miz-ZOUR-ee, which is used only by outsiders and a few Northern
Missourians who have fallen under the influence of Yankee phonetics. At any rate, I was
wrong, and Mr. Zani was right. The quotation should have been attributed to Henry Clay. I
had always heard that Truman had said it, an error that probably resulted from Missourians
who attributed too much to their native son. The important thing anyway is the thought in the
statement, and I can't at all understand people who would put more importance on popularity,
public opinion, and religious tradition rather than truth (being right).

I agree that T. S. Eliot was a great poet, who did indeed convert to Anglicanism (as well as
renounce his American citizenship). Although I taught American literature for many years, I
always skipped Eliot in the anthology, because he went to Europe when he was 22 and
eventually became more British than American. Anthologies of British literature also claim
him as one of their own.

An Internet Contact...

I struggled out of fundamentalism about 15 years ago, and I just started searching the internet
for ex-fundamentalist information and was very happy to find your journal. Please enter a
subscription for me.

(Rochelle Berner, 77 Moore Street, New Hyde Park, NY 11040.)

Volume 1990 - 2002 Issue


Page 1034 of 2049
Skeptical Review Edited by Farrell Till
Another One...

My name is Jason Wilson, and I have been reading on the subject of is the Bible the infallible
word of God. I have seen much or your material on the internet. I would like your free one-
year subscription to The Skeptical Review if possible. I would also like to know how to get
any debate tapes people from The Skeptical Review have participated in. I would have e-
mailed, but I have the internet only at school. I would also like to ask a personal question. Is it
possible the U. S. Government pays money (secretly) to missionary groups to stop the spread
of rival ideologies?

(Jason Wilson, 607 West Main, Iron Mountain, MI 49801.)

EDITOR'S NOTE: Letters like these are a delight to receive, and they lend support to a
prediction I have made several times in TSR, which is that the information age ushered in by
the internet will dramatically change the face of religion worldwide. In he past, the religious
establishment could control public access to undesirable information by simply not
mentioning it in pulpits or Bible classes and by exerting influence to keep such materials off
the shelves of public libraries. Obviously this is no longer possible, because anyone with an
on-line computer can execute key-word searches to browse the internet and find information
on a wide range of subjects. Churches and other religious organizations also make extensive
use of the internet, but they must do it in a forum that requires them to compete with informed
opponents. No longer can they just preach and teach one-sided views and expect their
audiences never to encounter dissenting views. In the long run, the owners of religious pages
on the internet will find themselves fighting windmills, because the silly, superstitious stuff
that once found gullible audiences will be unable to compete with reason and logic that are
being helped along by advancements in science and technology.

Jason Wilson's letter made me realize just how rapidly the information age is advancing.
When I was in school, computers didn't even exist, and students looking for diversion from
classroom studies had to be content to read comic books hidden in their geography books.
Now students can browse the internet.

I have no knowledge of government support to missionary organizations, but, of course, there


is an organized effort under way to secure government support for religion in different ways,
especially the plan to fund religious schools through voucher programs.

Because of the time that was involved in filling orders for video tapes, I stopped publishing
the notice of debates that were available. Since the backlog of orders has almost been filled, I
am running the notice again on the backside of this issue.

Personal Visitors...

It was absolutely wonderful meeting with you during our visit to Illinois. We are so grateful
for the input you have given us in our efforts to overcome our 30 years of fundamentalist
indoctrination.

Volume 1990 - 2002 Issue


Page 1035 of 2049
Skeptical Review Edited by Farrell Till
We have spent most of our lives like the characters in The Wizard of Oz--believing in the
omniscient, inerrant word of God--told never to look behind the curtain in fear of Satan
jumping out and grabbing us. Now we realize the persons pulling God's strings were mere
humans. Thank you for helping us to "get" a brain, a heart, and courage. The freedom we feel
is overwhelming.

Thank you for the back issues. We are enclosing a check to cover the next 2 years'
subscription.

(Denny and Kristina Clark, P. O. Box 13, Evanston, WY 82931.)

EDITOR'S NOTE: The Clarks are professional musicians who stopped by while they were on
a gig in Central Illinois. They came with a friend who subscribes to TSR and lives only 25
miles from my home. I believe he was the one who put them in contact with TSR. While they
were in my home, they mentioned several times how refreshing it has been to escape from the
grips of the Southern Baptist beliefs that they were reared in.

As a former literature teacher, I appreciated the Wizard-of-Oz imagery in their letter. It very
appropriately expresses the way religion works to stifle the intellectuality of its adherents.

Back Issues
We receive requests from many recent subscribers for a listing of back issues. From 1990
through 1994, TSR was a quarterly paper, so four issues per year were published then and
were numbered 1 through 4. Altogether there were 20 quarterly issues. In 1995, five issues
were printed in the transition from quarterly to bimonthly publication. In 1996, there were six
issues published, and this is the fourth issue of 1997. The total number of issues, including
this one, is 35, so recent subscribers should be able to determine how many issues they need
to have a complete set.

All back issues are available at $1 each. Some will be loose-leaf reprints. Orders will be filled
as promptly as possible, but backlogs may sometimes cause delays.

Debate Tapes

We receive many requests for a listing of debate tapes. We currently have the following tapes
available: Geisler-Till (resurrection), Hovind-Till (Genesis flood), and Thrasher-Till
(prophecy fulfillment) are single-cassette debates, which cost $5 each. The Till-Dobbs
(prophecy fulfillment), and Moffitt-Till (existence of Yahweh) are double-cassette debates,
which cost $8 each. Prices include postage. If you order all of the tapes at once, expect a delay

Volume 1990 - 2002 Issue


Page 1036 of 2049
Skeptical Review Edited by Farrell Till
in filling the order, because it takes over 20 hours to copy all of them. Shipment is also done
at book rate, which is slower than first-class delivery.

Study Aids

In addition to The Skeptical Review, Skepticism, Inc., publishes other materials that might be
useful to those wishing to improve their skills on the subject of Bible inerrancy. The booklets
below are available for $3 each, postage paid.

The Laws-Till Debate, a 47-page unfinished debate with James Laws, Jr., a professor of
apologetics at Tennessee Bible College. Although Laws challenged, he quit after only three
manuscript exchanges and has since refused to accept mail from Till. Correspondence is
reproduced in the booklet.

Jackson-Till Debate, 50 pages on the issue of Bible inerrancy.

Prophecies: Imaginary and Unfulfilled, an in-depth examination of the most commonly


claimed examples of prophecy fulfillment.

Geisler-Till Debate, transcript of an oral debate with Dr. Norman Geisler on the resurrection,
held at Columbus College in Columbus, Georgia.

Horner-Till Debate, transcript of an oral debate with Michael Horner on the resurrection, held
at Seattle Pacific University in Seattle, Washington.

ASCII Copies

All articles previously published in The Skeptical Review have been converted to ASCII text
and can be obtained by sending 3 HD DOS formatted disks and $1 for postage. Skepticism,
Inc., grants permission for these articles to be recopied and used for local or internet
distribution and posting as long as the post office and e-mail addresses of Skepticism, Inc., are
noted on all copies.

errancy@infidels.org

The Errancy list on the secular web is still active with both skeptics and Bible inerrantists
participating. You may join this list by sending to major-ii@infidels. org the command
"subscribe errancy" in the message window. If you encounter problems, contact us at
jftill@midwest.net for instructions.

Volume 1990 - 2002 Issue


Page 1037 of 2049
Skeptical Review Edited by Farrell Till
Writers Wanted

A new quarterly journal Pagan Palaver is looking for writers. The purpose of the publication
is to inform and to create a forum for open-minded discussions. The editor-publisher is
interested in book reviews, essays, and articles on subjects that give "something to think about
in our pursuit of freedom from religion." Anyone interested should contact Jack Truett, at P.
O. Box 935, Somerville, TN 38068.

Volume 1990 - 2002 Issue


Page 1038 of 2049
Skeptical Review Edited by Farrell Till

Skeptical Review
Volume Eight, Number Five
September/October 1997
Farrell Till, editor

• Keep Them Barefooted and Pregnant


What the bible has to say about women

• The 70-Year Prophecy: Undeniable Evidence


Price claims that Till is a `radical skeptic' to deny the 70-year prophecy

• Responding to the Undeniable


Till's response to Price's latest article on the 70 years of servitude prophecy in
Jeremiah

• Editorial Potpourri
Till catches readers up on all the boring stuff they should know

• From the Mailbag


The monthly feature

Keep Them Barefooted and Pregnant


A popular claim of Christianity is that the Bible has given women a dignity and status far
superior to societies that are dominated by other religions. That the ranks of Christianity
include so many women is evidence that there must be some truth to the axiom that says if a

Volume 1990 - 2002 Issue


Page 1039 of 2049
Skeptical Review Edited by Farrell Till
lie is told enough times, some will believe it, because only someone who is relatively ignorant
of the Bible could believe that it is in any sense complimentary to women. From beginning to
end, the Bible insults women and speaks of them with a disdain that one would think women
in modern times would no longer tolerate. But tolerate it they do, for the membership rolls of
churches probably include many more women than men.

The men who wrote the Bible wasted no time getting down to one of their favorite themes: all
the pain and suffering, sorrow and grief that the human race has to endure is the fault of
women. Right in the opening chapters of God's inspired word, the first woman ate a fruit that
God had told her not to eat, and that's why men have to earn their living by the sweat of their
brows. Never mind that this grievous offense also caused women to have to endure the pain of
childbirth (Gen. 3:16). What are a few labor pains compared to the men's ordeal of having to
till soil that brings forth thorns and thistles (v:18)?

Ever since the Genesis writer put the blame on Eve, God's emissaries have continued to lay it
on thicker. Paul, the chief apostle not just of Christianity but of blatant sexism too, used Eve's
sin as an excuse to put women into the basement of Christianity, which they have yet to climb
out of. Writing by the inspiration of the Holy Spirit, who is presumably another person in the
one "godhead," Paul told women that they were welcome in the churches as long as they kept
their mouths shut: "Let your women keep silent in the churches, for they are not permitted to
speak; but they are to be submissive, as the law also says" (1 Cor. 14:34). Well, gee, if women
are not permitted to speak in the churches, how can they be expected to learn things they may
need to know? Not to worry; Paul had the answer to that: "And if they want to learn
something, let them ask their own husbands at home; for it is shameful for women to speak in
church" (v:35). It's hard to see dignity and respect for women in any of this, but obviously
many women have bought it and meekly acquiesce to the sexist rantings of a religious mystic
whose denigration of half the world's population has been rivaled only by other religions that
have enshrined the same primitive, male-chauvinist nonsense.

Christians can't say that Paul was just in a bad mood when he wrote his epistle to the
Corinthians, because if he was the author of the pastoral epistles, as most fundamentalists
claim, then he gave orders for his sexist views to be taught as ecclesiastic law: "Let a woman
learn in silence with all submission. And I do not permit a woman to teach or to have
authority over a man, but to be in silence" (1 Tim. 2:12). Why, heavens, no, never permit a
woman to teach, even if she has a Ph. D. in theology and the only alternative is a male with
the IQ of a doorknob. The guy has to get the nod. Presumably, then, Paul believed that a penis
and a pair of testicles somehow qualified a male over a female for teaching assignments no
matter what. Maybe God just has a penchant for testosterone, or maybe he considers sex
organs more important than brains. After all, this God is a "he" too, isn't he?

And what was God's reason for giving men the preeminence in his church? "Paul" told us
why: "For Adam was formed first, then Eve. And Adam was not deceived, but the woman
being deceived, fell into transgression" (vs:13-14). So there it is. It all goes back to Eve. She
ate the forbidden fruit, and so everything is her fault. Not only that, but everyone born since
then with the misfortune to have two X chromosomes has had to bear the brunt of Eve's
mistake. Talk about injustice, this has to be the granddaddy of all injustices.

Volume 1990 - 2002 Issue


Page 1040 of 2049
Skeptical Review Edited by Farrell Till
"Paul" never bothered to explain why Adam was so blameless in this affair of the forbidden
fruit. The Bible says that Eve ate the fruit all right, but it also says that she gave some to
Adam and he ate it too (Gen. 3:6). There isn't any arm-twisting even implied in the story, so if
eating the fruit was all that bad, why did men get off with just having to sweat to earn a
living? If given the choice of either working to earn a living by the sweat of his brow or
carrying a child to full term and then giving birth to it with the pain roughly equivalent to
expelling a watermelon from his abdomen, even the most ardent male chauvinist would
choose the sweaty brow.

"Paul" can't be accused of completely lacking empathy for women, because in the very next
verse of his ecclesiastic instructions to Timothy, he did assign women a role in the church:
"Nevertheless, she will be saved in childbearing if they continue in faith, love, and holiness
with self-control" (v:15). So it all ties together. A woman brought grief and suffering upon
humanity, and so all of her descendants of the same gender have to pay for it with no recourse
except to redeem themselves from something they didn't do by bearing children. What woman
wouldn't be deliriously happy to have a role like that in the church that God's omniscient
wisdom foreordained before the foundation of the world (Eph. 1:4), while the male
descendants of Eve's co-conspirator get to be top dogs in the plan? It's hard to find fault in
such wisdom as this.

No one can claim that Paul was just a maverick voice on the subject of the place for women in
God's grand scheme of things, because all through the Bible women were denigrated by the
earthly vessels whom God allegedly chose to write his inspired, inerrant word. In Genesis 19,
God sent two angels to Sodom to warn Lot of the impending destruction of the city so that he
could flee to safety. When a group of men surrounded Lot's house and demanded that he send
out the "men" so that "we may know them" (v:5), Lot tried to save the men from gang rape by
offering his daughters to the mob: "So Lot went out to them through the doorway, shut the
door behind him, and said, `Please, my brethren, do not do so wickedly. See now, I have two
daughters who have not known a man; please, let me bring them out to you, and you may do
to them as you wish; only do nothing to these men, since this is the reason they have come
under the shadow of my roof'" (vs:6-8). Do you get the message implied in this? Sexually
abusing men would be doing "wickedly," but abusing women wouldn't be so bad.

If anyone thinks that such a despicable proposal as this made Lot an abomination to the
infinitely good Yahweh, then think again. Yahweh saved Lot from the fire and brimstone that
he rained down on Sodom and then later "inspired" a New Testament writer to describe Lot as
a "righteous" man who was "sore distressed by the lascivious life of the wicked" (2 Peter 2:7-
8). So if a man who would offer his daughters to a mob of men to appease their sexual
demands could in any sense be described as a "righteous" man, we can only wonder how
depraved the rest of the people in Sodom were. At any rate, this story makes very clear the
attitude of the Bible god. If it is necessary to sacrifice the honor of women, in order to protect
the honor of men, then so be it. This is the god that millions of women trek to churches each
week to oooh and ahhhh over.

There is more--much more--that could be said on this subject, but it will have to wait till the
next issue.

Volume 1990 - 2002 Issue


Page 1041 of 2049
Skeptical Review Edited by Farrell Till

The 70-Year Prophecy: Undeniable


Evidence

Dr. James D. Price


In his May/June 1997 response to me, Mr. Till complained that I did not fulfill the
requirement of "the burden of proof." By that I assume that he meant my evidence was not
sufficiently compelling to convince him. I'm not surprised. His type of skepticism would not
be persuaded, no matter how compelling the evidence. However, in this complaint he has
misused the term "burden of proof." It is true that the person who defends the truth of a
debated proposition must present convincing evidence to support its truth. But the person who
denies the truth of the proposition has the burden to convincingly disprove the proposition--
that is the real burden of proof. The proof of a proposition is in the failure to prove the denial.
If the defender of a proposition must produce objective evidence to prove it, then the
opponent of the proposition must present objective evidence to disprove it. It is insufficient
for the challenger to appeal to a fallacious presupposition as his sole objection. Why should
the burden of proof be one-sided. In this debate, I am supposed to present evidence supporting
the proposition that Jeremiah's prediction of the seventy-year captivity of the Jews is indeed a
valid fulfilled prophecy. Till is supposed to discredit my evidence and prove that the prophecy
is a fraud. I have presented verifiable objective evidence that Jeremiah's prophecy is valid and
has been fulfilled. All Till needs to do is provide the objective evidence to the contrary--not
extraordinary evidence, just ordinary evidence will be sufficient. So far Mr. Till has failed
miserably on both counts.

Extraordinary Proof: Mr. Till appealed to an alleged law of evidence that states:
"Extraordinary claims require extraordinary proof." On the basis of this "law" he complained
that my evidence was not sufficiently extraordinary. But how extraordinary must the claim be
before the "law" applies, and how extraordinary must the proof be before he will accept it?
Should it be greater than what is accepted in a court of law? Should it be greater than what is
required by the canons of historical research? Life is full of extraordinary events that we
believe without extraordinary proof. Robert Ripley recorded thousands of extraordinary
people and extraordinary events which we believe without extraordinary proof. Mr. Till does
not intend for his "law" to apply to this type of extraordinary event; ordinary evidence is
sufficient for him there. The bottom line is that Till's "law" really applies only to supernatural
claims, not to extraordinary claims. So this "law" becomes rather ridiculous. If it is restated
according to how Till intends it to be applied, it would read: "Supernatural claims require
supernatural proof." So what kind of supernatural proof would Till accept? The witness of an
angel? A notarized affidavit from God Himself? No. Till's anti-supernatural presupposition
rejects the possibility of any supernatural proof. Thus, Till's "law" is just a clever restatement
of his anti-supernatural presupposition in disguise. It means that Till will not accept any proof
of fulfilled prophecy, no matter how extraordinary it may be.

Volume 1990 - 2002 Issue


Page 1042 of 2049
Skeptical Review Edited by Farrell Till
Mr. Till declared: "My position is that exaggerations, deliberate falsifications, mistaken
impressions, etc., in documents of prescientific times were far more likely than the suspension
of recognized natural laws?" My question to Mr. Till is: What law of nature does prediction
violate? Nearly every law of nature includes the capacity for predicting future events. In fact,
the predictive capacity of a law of nature is one characteristic used to validate it. The more
well defined the law, the greater its predictive power. Thus Newton's laws of motion enable
astronomers to predict the exact position of the sun, moon, planets, and comets thousands of
years in advance; and their predictive power enables astronauts to land on the moon, and to
sent a space craft to a distant planet like the recent landing on Mars. Prediction is a necessary
ingredient of normal living. Without the capacity for prediction, life would be hopelessly
perilous. No one could safely drive a car, fly an airplane, or even cross the street. We would
have no sports, business, or agriculture; in fact, animate life can scarcely survive without
prediction.

So prediction is not what radical skeptics deny. They only object to predictions for which no
natural law is known to them. In other words, their objection is based on ignorance, not
knowledge. Indeed, many things involving prediction occur regularly in nature for which no
one has yet determined a law. One must therefore assume that some such law exists. Till does
not object to such assumptions nor to such unexplainable extraordinary predictions, nor does
he demand extraordinary proof for them. Why? Merely because the phenomena are
observable, not explainable. He does not even have to observe the extraordinary predictive
ability himself; he is willing to accept the witness of someone else, perhaps someone like
Robert Ripley, or even the report of an ancient observer. The only kind of prediction Mr. Till
objects to, or for which he demands extraordinary proof, is a prediction attributed to God.
Clearly this is a theological objection, not a logical one.

What radical skeptics like Mr. Till fail to realize is that their own anti-supernatural
presupposition is based on a sequence of unverifiable fabulous claims, claims that they say
should have extraordinary proof. Mr. Till's reasoning states: a common sense explanation is
more likely than a fabulous claim. Now, most of the classical arguments favoring the
existence of God are based on known laws of nature with which they are in harmony, whereas
opposing arguments involve accepting unverifiable fabulous claims. For example, the
argument from the law of cause and effect involves accepting one of two alternate claims: (1)
a sufficient first cause based on the second law of thermodynamics which extrapolates to a
beginning in the past and predicts an end in the future; or (2) an infinite sequence of causes
contrary to the second law. Now, while neither claim can be proved or disproved rigorously,
common sense and reasonableness say the first is more likely because it is conceivable, and is
in harmony with perceived reality and with two verifiable laws of nature; whereas the second
has characteristics of a fabulous claim--infinity (by definition an inconceivably large value)
and the contradiction of a verifiable natural law. Now, the same observation is true for all the
classical arguments. Those who choose to believe in God regard the joint consensus of the
classical arguments to be far more likely and more than sufficient reason to believe, whereas
atheists choose to believe the joint witness of a sequence of unverifiable fabulous claims. In
the final analysis, it is a matter of choice. Mr. Till has chosen to accept the witness of a
sequence of fabulous claims contrary to his own declared method of reasoning.

Volume 1990 - 2002 Issue


Page 1043 of 2049
Skeptical Review Edited by Farrell Till
To those who believe in an omniscient God, the idea of fulfilled prophecy attributed to Him is
not a fabulous claim, but what one would reasonably expect. To them an instance of such
fulfilled prophecy requires only ordinary, not extraordinary, evidence as sufficient reason to
believe it. I acknowledge that this debate is not about the existence of God, but the above
discussion was necessary in order to let the reader see the true foundation of Till's anti-
supernatural presupposition. It is not based on common sense and reason, but on a theological
choice supported by a string of unverifiable fabulous claims that common sense and
reasonableness reject. Logically it involves the fallacy known as a hasty generalization, and
when appealed to in debates like this, it involves the fallacy of assuming the conclusion--
circular reasoning. An unbiased skeptic will admit the possibility of fulfilled prophecy no
matter how unlikely it may seem, and let the evidence decide the case. Many unlikely events
are recorded in ancient records the validity of which historians accept on the basis of good
(but not extraordinary) evidence. Not Mr. Till! He has repeatedly refused to debate under
those conditions.

What Mr. Till means by "extraordinary proof" is evidence that is superior to what is possible
to obtain from ancient records. He wants to evaluate historical evidence by a double standard,
one that admits the possibility of natural events but denies the possibility of supernatural
events. In other words, no matter how valid and convincing the ancient evidence may be
otherwise, if it involves a possible fulfilled prophecy, the evidence automatically will be
insufficient by Till's standard. He has insisted that the debate proceed, not on a level field, but
with this theologically motivated double standard. Debating under this restraint is a distinct
disadvantage because Till will claim a victory on the basis of this double standard no matter
how convincing the evidence is otherwise. But I will continue to present the valid evidence,
and to point out this double standard whenever Till imposes it. Discerning readers will see
through this tactic and be able to evaluate the evidence in an unbiased manner without letting
Till do their thinking.

Radical Skepticism: Mr. Till complained that I object to his appeal to an anti-supernatural
presupposition. He does this on the basis that he thinks I have the same presupposition in
areas outside the Bible. In this complaint Mr. Till is monotonously repetitive. We have
discussed this issue in previous e-mail exchanges where I demonstrated that in a debate like
this such an anti-supernatural presupposition is fallacious. In those exchanges, I have already
answered his questions. In one such exchange I stated: "I do not advocate accepting all
reported supernatural events as historical events. I advocate healthy skepticism, a skepticism
that evaluates the evidence and accepts what is verifiable, a skepticism that is willing to give a
reputable witness the benefit of the doubt." That means that I am willing to accept the
historicity of reported supernatural events if they are verifiable, regardless of whether the
source is the Bible, secular history, or a non-Christian religious historian. Also, I am willing
to withhold judgment on some unverifiable supernatural events reported by reputable sources-
-after all, the events may have occurred. Without objective evidence to the contrary, who can
be sure? Till can be sure only because he has accepted without proof his anti-supernatural
presupposition. Till cannot justify his own radical skepticism by an appeal to my more
moderate and reasonable skepticism. They are not the same.

It is irrelevant for him to insist that I waste the readers time by identifying what extra-Biblical
miracles I would accept as historical. That has nothing to do with the debate at hand. That is

Volume 1990 - 2002 Issue


Page 1044 of 2049
Skeptical Review Edited by Farrell Till
another diversionary tactic to distract attention from his failure to adequately deal with the
evidence I presented. In a previous e-mail exchange I documented two modern verifiable
miracles. The evidence is undeniable and is available for Till to examine. Till has completely
ignored these and has refused to comment on them. This indicates to me that Till is blindly
committed to a radical skepticism that refuses to accept the possibility of supernatural events
regardless of the evidence, extraordinary or otherwise.

All the Details: Mr. Till has insisted that "all the details" of the prophecy must be fulfilled.
He stated that "at no time did Dr. Price address my claim that a genuine prophecy fulfillment
would have to include fulfillment of all details and not just those that someone arbitrarily
declares `central' ones." By that I suppose he means that all the minor details must be verified
with objective evidence, not just the principal significant details. He is not satisfied that I
validated the details of a fulfilled prophecy that he himself defined as essential for a genuine
prophecy--the date of the prophecy, the duration of the 70 years, and the return of the Jews
from captivity--now he insists that I must provide validating evidence for all the minor
details. However, this is where the real burden of proof comes on the scene. It is Till's
responsibility to provide objective evidence that the minor details were not fulfilled. In the
absence of contrary evidence, common sense and reasonableness demand that if the essential
details of a reported historical event are verified as true, then the minor unverified details can
be regarded as true also. No canon of historical research denies this. This is true of prophecy
also: in the absence of contrary evidence, if the essential details of a prophecy are verified as
fulfilled [the details Till specified as essential], then the unverified details may be regarded as
fulfilled also. What law of evidence would deny this? Let Till produce the objective evidence
to the contrary or stop quibbling about unverified minor details.

Irrelevant Errancy: Because Till cannot provide objective evidence against the details of the
prophecy under debate, he launched into an unsubstantiated attack on the integrity of the
Bible as a whole, on the validity of the book of Jeremiah, and on the veracity of the prophet.
This was his attempt to justify a charge of fraud. Concerning the Bible, he stated: "The Bible
text, however, is a tangible existent that can be critically examined, and when it is so
examined, we find that it is riddled with inconsistencies, discrepancies, and outright
contradictions." He then indicated that he has many proofs of this statement which, of course,
space did not permit him to provide.

Since Mr. Till introduced unsubstantiated claims, I am permitted to introduce some of my


own. Radical skeptics like Mr. Till are unable to critically examine the text of the Bible in a
neutral, unbiased manner. They approach the text with hostility and a theological bias that
guarantees that every apparent inconsistency, discrepancy, or contradiction invariably will be
labeled as an error. Any reasonable resolution of such apparent problems, which is
appropriate in responsible criticism, is rejected by them, and is categorized as uncritical
rationalization. This is not an accepted practice in a court of law or in reasonable historical
research. Reasonable people regard such skepticism as radical dogmatism, not genuine critical
thinking. Many proofs of this exist, but space does not permit their presentation other than
those that appear in this debate.

Now all this wrangling over errancy is irrelevant to the topic under debate, namely, does the
Bible contain a fulfilled prophecy? I have presented a prophecy that I claim was fulfilled. Till

Volume 1990 - 2002 Issue


Page 1045 of 2049
Skeptical Review Edited by Farrell Till
has admitted that an errant text may still contain valid accounts of historical events. So let him
restrict his criticism to the prophecy under discussion. Let him produce objective evidence
that this prophecy contains errors and was not fulfilled, and let him cease quibbling over the
irrelevant topic of errancy in general. I have not appealed to the inerrancy of Scripture in the
defense of this prophecy, so let Till cease appealing to his alleged doctrine of errancy in his
attempt to disprove the prophecy. Let him produce an undeniable error in the debated
prophecy, not a doctrine about errancy.

Jeremiah's Credibility: Since Mr. Till could not produce objective evidence that the debated
prophecy was not fulfilled, he resorted to an attack on the prophet's credibility. He did this in
the hope that he can produce a case for fraud. To do this he introduced another of Jeremiah's
prophecies that he regards as lacking credibility. Of course this is another diversionary tactic,
because, for the sake of discussion, Jeremiah could have been right in one prophecy and
wrong in another. We are not debating Jeremiah's inerrancy, nor the implications of errancy
on the doctrine of divine inspiration; we are debating whether or not the one prophecy under
discussion was fulfilled. Nevertheless, because Jeremiah's credibility has some bearing on the
reliability of his book in general, I address Mr. Till's credibility attack.

Evidently this attack represents Till's best shot. But I am surprised that he would make such
an issue over such a minor difficulty--a difficulty to him perhaps, but not to reasonable
people. Mr. Till presented Jeremiah's prediction of the death of King Zedekiah (Jeremiah
34:2-5) and an alleged contradiction of that prediction in the record of his death (52:9-11).
Although the prediction contained specific details, and the record of his death verified most of
those details, Mr. Till was disturbed that whereas Jeremiah predicted Zedekiah would "die in
peace," the latter record indicated that he died in prison. Mr. Till regards this as a
contradiction.

Evidently Mr. Till imagines that to "die in peace" meant that the king would die in freedom,
honor, and dignity, and that to die in prison meant dying as a result of being treated with
dishonor and torture. But this is not necessarily the case. It is more likely that there was some
degree of chivalry among ancient monarchs, and even though captured kings were enemies,
they were treated with dignity and respect. For example, in later years King Jehoiachin was
released from prison, given a lifetime pension, and promoted to a prominent seat among the
other released kings (Jeremiah 52:31-43). This demonstrates that the Babylonian kings treated
captive monarchs with some measure of respect. The expression "to die in peace" is
commonly understood to mean a natural death as opposed to a gruesome death resulting from
war, crime, violence, or suffering. This is verified by Jeremiah's own words that clarify his
intended meaning in this passage: "You shall not die by the sword. You shall die in peace"
(34:4-5). There is nothing contradictory in the idea of one dying peacefully in prison. Mr.
Till's hostility toward the Bible has created an illusion of contradiction where none exits.
Since it is much more likely that the term was used in its common, natural sense rather than in
the unnatural, contradictory sense imagined by Mr. Till, no reason exists for questioning
Jeremiah's credibility.

Next Mr. Till quibbled that "there is no indication that incense was burned in memory of
Zedekiah as Jeremiah had predicted." Of course this is a fallacious argument from silence, for
Mr. Till has no contrary evidence to present. His rationalization that the Jews were widely

Volume 1990 - 2002 Issue


Page 1046 of 2049
Skeptical Review Edited by Farrell Till
scattered is not factual. All the records indicate that the Jews were taken to the province of
Babylon, that many of them built houses, raised families, and engaged into business; some
even became high ranking government officials. Their standard of living in Babylon became
so prosperous and satisfactory that most of them chose not to return to Israel at the end of the
seventy years. There is no reason to believe that the Jews would have been denied the
privilege of giving their fallen king an honorable burial. This is supported by Josephus:
"When the king was come to Babylon, he kept Zedekiah in prison until he died, and buried
him magnificently, and dedicated the vessels he pillaged out of the temple of Jerusalem to his
own gods, and planted the people in the country of Babylon, but freed the high priest from his
bonds." (Antiquities of the Jews, 10:9:7).

Thus Mr. Till's best shot at Jeremiah's credibility missed the target and exposed his own
hostile imagination. If this is the best that Till can do, then Jeremiah's credibility remains
unscathed.

Jeremiah's Rationality: Next Mr. Till questioned Jeremiah's rationality for somehow not
correcting the imagined contradiction between the prophecy and the actual death of Zedekiah.
Of course, as I indicated above, no real contradiction exists. So no reason existed for altering
either the prophecy or the record of Zedekiah's death. The problem exists only in the mind of
Mr. Till who fails to read correctly. If he had read the last verse of chapter 51, he would know
that Jeremiah's material stopped there. Chapter 52 is a historical appendix written by a later
hand, probably Jeremiah's young assistant Baruch. So if Mr. Till wants to question the
rationality of someone it should be that of Baruch not Jeremiah. But evidently Baruch saw no
contradiction with Jeremiah's prophecy in the idea that Zedekiah died peacefully in prison--
good reasoning as far as I'm concerned.

Jeremiah's Veracity: Next, since Mr. Till could not produce objective evidence that the
debated prophecy was not fulfilled, he resorted to questioning Jeremiah's veracity by accusing
him of lying to the princes of Judah (38:27). Actually what Jeremiah did was withhold
information that would have resulted in his death. No one is expected to relate information
that would jeopardize his own life. Jeremiah had done nothing worthy of death. All he had
done was to advise the king on how to avoid the destruction of the city and the death of his
household and many of his officials. Having been so advised, the king charged Jeremiah,
under threat of death, not to disclose their discussion to the princes. Instead, the king
commanded him to tell the princes: "I presented my request before the king, that he would not
make me return to Jonathan's house to die there" (38:26). When the princes interrogated
Jeremiah under threat of death, his life was in double jeopardy. If he disclosed what he
advised the king, he would die; and if he remained silent, he would die. So he protected his
life by obeying the king's command, but in so doing he did not lie. Jeremiah indeed had
previously made that request of the king: "Therefore please hear now, O my lord the king.
Please, let my petition be accepted before you, and do not make me return to the house of
Jonathan the scribe, lest I die there" (37:20). Thus what Jeremiah told the princes was true. So
Mr. Till has falsely accused Jeremiah of lying. What reasonable person would question
Jeremiah's veracity under those circumstances? Would Mr. Till have handled the situation any
differently? Far from being untrustworthy, Jeremiah's veracity as a historian is to be
commended for recording the events as they actually happened without glossing over the
gritty details.

Volume 1990 - 2002 Issue


Page 1047 of 2049
Skeptical Review Edited by Farrell Till
Mr. Till further accused Jeremiah of deceiving Zedekiah. He stated: "Jeremiah did not tell him
that if he went out of the city to meet the Babylonian princes, all of his sons would be killed
in his presence and he himself would be blinded and imprisoned for the rest of his life." Mr.
Till is correct, Jeremiah did not tell him that. But in being correct, Till exposed his failure to
read correctly and to get the facts straight. Perhaps he has a comprehension problem. He
incorrectly led his readers to believe that Zedekiah was treated that way as a result of his
surrender to the Babylonian princes; but Zedekiah did not surrender, he tried to escape.
Jeremiah made the options clear to Zedekiah; he said: "Thus says the LORD, the God of
hosts, the God of Israel: `If you surely surrender to the king of Babylon's princes, then your
soul shall live; this city shall not be burned with fire, and you and your house shall live. But if
you do not surrender to the king of Babylon's princes, then this city shall be given into the
hand of the Chaldeans; they shall burn it with fire, and you shall not escape from their hand'"
(38:17-18). He further warned Zedekiah of the dire consequences of attempting to escape
(38:21-23). Thus Zedekiah knew his alternatives, but he disbelieved Jeremiah and refused to
surrender. Instead, he held out until the Babylonians breached the walls and entered the city;
then he escaped with some of his troops. But they were pursued and captured by the
Babylonians in the plains of Jericho (39:1-5). Contrary to Till's "revised version," Zedekiah
did not surrender to the Babylonian princes, but suffered the consequences of trying to escape
in spite of Jeremiah's clear warning. Mr. Till has erroneously accused Jeremiah of deception
in this incident. In his best efforts to impugn the veracity of the prophet, Mr. Till has failed
miserably. Instead, he has demonstrated his own fallibility.

Valid Text: Next, Mr. Till again questioned the validity of the text of the Book of Jeremiah.
He stated: "In biblical times, there were no copyright agencies or archives that registered and
kept original works, which could be consulted if questions later arose concerning the reading
of the original text and the date of its authorship." Of course, this is not a problem unique to
the text of the Bible, but to all ancient historical texts. If such evidence is necessary before an
ancient text has validity, then all ancient history is invalid--something Mr. Till is not willing
to concede. Indeed, the canons of historical research do not require evidence from sources
known to never exist. This is absurd! In reality, what Mr. Till means is that he regards the text
of the Book of Jeremiah to be much less reliable than other historical documents, so that one
cannot believe what it has recorded. He stated: "All Dr. Price has to offer as evidence are the
existing text of Jeremiah and a tradition that the entire book was written by a 7th- and 6th-
century prophet named Jeremiah." But all that we have of any ancient historical document is
its existing text and the traditions about its authorship. So again, if that invalidates an ancient
document, then all ancient history is invalid. No one can take these objections of Till
seriously. Other means are available for assessing the validity of an ancient text.

The question about an ancient historical text is: How does one determine its validity and
authorship? The validity of a text is determined by the consensus among existing manuscripts
and other witnesses, such as translations and ancient quotations. The greater the number of
independent witnesses, the more ancient the witnesses, and the greater the consensus among
the witnesses, the greater is the probability that the authentic text can be determined. This is
the accepted method for evaluating the objective evidence for textual validity. It is known as
the science of textual criticism. It applies to all ancient documents.

Volume 1990 - 2002 Issue


Page 1048 of 2049
Skeptical Review Edited by Farrell Till
Mr. Till has called attention to some general problems regarding consensus among the
witnesses to the text of Jeremiah; in comparing it with the texts of other biblical books, the
text of Jeremiah has more variations. But this problem does not hinder a reasonable recovery
of the authentic text. In my earlier discussion of Jeremiah's text, I demonstrated that the text
of the debated prophecy is valid, but how does the text of Jeremiah compare with the texts of
ancient secular historical documents? In spite of its textual variations, compared with any
ancient secular text, the witnesses to the text of Jeremiah far exceed them in number,
diversity, antiquity, and consensus; that is, the evidence is extraordinary. Let Mr. Till provide
the witnesses to any ancient secular historical document that come anywhere near those for
the Book of Jeremiah with respect to number, diversity, antiquity, and consensus; or let him
cease quibbling over textual validity.

Authorship: In a court of law and the canons of historical research the authorship of an
ancient document is determined by the internal claims of authorship and by internal and
external witnesses to that authorship. According to the laws of evidence, such claims to
authorship are authentic unless proven otherwise by valid evidence. In this regard, the Book
of Jeremiah fares as well or better than any secular document. The prophet Jeremiah is
mentioned 117 times in his book; the expression "the word of the LORD came to Jeremiah"
(or its equivalent) occurs 29 times (1:1, 4, 11, 13; 2:1; 13:3, 8; 14:1; 16:1 ; 18:5; 24:4; 25:3 ;
28:12; 29:30; 32:6, 26; 33:1, 19, 23; 34:12; 35:12; 36:27; 37:6; 39:15; 42:7; 43:8; 46:1; 47:1;
49:4); in addition, the expression "The LORD said to Jeremiah (or its equivalent) occurs
another 16 times (1:7, 9, 12, 14; 3:6, 11; 11:6, 9; 13:1, 6; 14:11, 14; 15:1; 17:19; 24:3; 32:25).
Often Jeremiah referred to himself by first person pronouns that indicate he was the
narrator/writer. Four times the book records that God commanded Jeremiah to write portions
of the book (22:30; 30:2; 36:2, 28); three times it records that Baruch wrote portions of the
book at the dictation of Jeremiah (36:4, 18, 32); and once it records that Jeremiah himself
wrote a portion (51: 60). Finally, the entire collection, from chapter one through fifty-one, is
attributed to Jeremiah: "Thus far are the words of Jeremiah" (51:64). Other forms of internal
evidence point to Jeremiah as the narrator/writer.

Thus the internal evidence is strong, and is supported by three independent external witnesses
(2 Chronicles 36:12, 21, 22; Ezra 1:1; Daniel 9:2). True, these witnesses are in the Bible, but
they are indeed external to and independent of the Book of Jeremiah. If Mr. Till wants to
classify as mere tradition these external references to Jeremiah's authorship, then he must also
do the same for external references to the authors of secular historical documents, because
none come from more reliable sources. This does not include references to the authorship of
Jeremiah in Josephus, in the New Testament, and in the Jewish Mishnah and Talmud. All
ancient witnesses agree that the prophet Jeremiah wrote the book; and there are no dissenting
witnesses. In other words, as compared with the evidence for ancient secular documents, the
objective evidence for the authorship of Jeremiah is extraordinary. Mr. Till will continue
quibbling over authorship based on denials written by radical critics. But let the readers be
forewarned that these denials are not based on objective evidence, but on subjective theories
built on theological presuppositions like Till's anti-supernatural one.

Valid Date: Next, Mr. Till again challenged the date of the debated prophecy. He stated: "the
book of Jeremiah does not claim that the prophecy in question was written in the 4th year of
Jehoiakim's reign (605 B.C.) but only that the word of Yahweh had come to Jeremiah at that

Volume 1990 - 2002 Issue


Page 1049 of 2049
Skeptical Review Edited by Farrell Till
time." Of course, this is an obvious quibble to avoid admitting that the prophecy has a valid
date. But this again demonstrates that Mr. Till does not read carefully. Not only is the date
recorded for when God gave the prophecy (25:1), but Jeremiah devoted a whole chapter to
record when and how he actually wrote the early chapters of the book (36:1-32). In the fourth
year of Jehoiakim (605 B. C.), the Lord commanded Jeremiah to write all the words that He
had given him up to that time (36:1-2). Jeremiah immediately obeyed. He acquired a
parchment scroll and dictated all the words to his amanuensis Baruch, who accurately wrote
them down. The scroll was read to King Jehoiakim who then proceeded to burn the scroll leaf
by leaf. That the scroll contained the prophecy under debate is indicated by the king's
complaint to Jeremiah: "Why have you written in it that the king of Babylon will certainly
come and destroy this land, and cause man and beast to cease from here?" (36:29). Note that
both the Hebrew text and the Greek Septuagint translation contain the reference to the king of
Babylon. Thus the Septuagint also understood that Jeremiah's prophecy referred to the king of
Babylon. Shortly after King Jehoiakim burned the first edition of the book, the Lord
commanded Jeremiah to rewrite the scroll, which Jeremiah did with expansions (36:27-32).
Thus the date of the actual writing of the prophecy is clearly stated as the fourth year of King
Jehoiakim (605 B.C.). All Hebrew manuscripts and ancient versions validate that date, so no
objective evidence exists that gives reason to doubt it.

Mr. Till presented the events recorded in chapter 52 as evidence that the book, and thus the
debated prophecy, was written near or after the end of the seventy year captivity, at least in its
final form. This contradicts the evidence I presented above, and the evidence that attributes
the content of chapters one through fifty-one to Jeremiah: "Thus far are the words of
Jeremiah" (51:64). This statement clearly attributes chapter 52 to a later hand such as
Jeremiah's young assistant Baruch.

The evidence indicates that Jeremiah wrote an initial edition of his book in 605 B.C.
(including the prophecy under debate) which was destroyed and immediately rewritten; this
was followed by one or more additions of Jeremiah's oracles and historical essays which were
completed shortly after the destruction of Jerusalem (586 B.C.). This latter edition of the
book, which included chapters one through fifty-one, Jeremiah took to Egypt when Johanan
forced him to go there. It is likely that Jeremiah edited and expanded the book before he died
in Egypt, and that Baruch took the expanded edition to Babylon after Jeremiah's death.
Finally, Baruch, or another scribe who inherited Jeremiah's manuscripts, wrote a historical
essay that was appended to the text as chapter 52. The shorter edition (including chapter 52)
became the tradition used in Egypt and used for translating the Greek Septuagint. The longer
edition became the tradition used by the Jews in Babylon which became the proto-Masoretic
text.

Apart from a difference in the order of the chapters, the two editions are in essential
agreement. Their differences are much like what one would find in comparing two different
editions of a text book, one an expanded revision of the other. No textual differences exist that
indicate that the author of the historical appendix made any significant changes to Jeremiah's
portion of the text. This is certainly true of the prophecy under debate. As I demonstrated in
my earlier article, the text of the prophecy is valid for all the important details; all the textual
evidence supports the date that the prophecy was given and written, and the seventy year
duration of the captivity. The fulfillment was supported by external evidence. Mr. Till's

Volume 1990 - 2002 Issue


Page 1050 of 2049
Skeptical Review Edited by Farrell Till
appeal to the historical appendix has no bearing on these details of the prophecy. He has
suggested the possibility that the date could have been fraudulently changed. But a possibility
is not a reality. Let him produce objective evidence that the date was changed, or let him
cease quibbling over what he cannot verify. A charge of fraud is invalid without objective
proof, and a date is not invalid just because it disagrees with Mr. Till's theological theories.

Undeniable evidence: Mr. Till challenged me to present "undeniable evidence." I have


presented valid objective evidence that Jeremiah's prophecy was fulfilled. In an attempt to
discredit my evidence Mr. Till unsuccessfully attacked Jeremiah's credibility, rationality, and
veracity. He unsuccessfully attacked the validity of the text, and the authorship and date of the
book and the prophecy. In all these attacks, Mr. Till failed to present valid objective evidence
that successfully denies the evidence I presented. I was able to show that Mr. Till engaged in
unnatural interpretation of common expressions, in misrepresentation of the facts, and in
exaggeration of alleged textual problems. In so doing, I was also able to show that the
objective evidence for the fulfillment of Jeremiah's prophecy is extraordinary in comparison
with similar evidence for ancient secular historic documents. Since Mr. Till was unable to
successfully deny the validity of my evidence, I am free to claim that I presented undeniable
evidence for a fulfilled prophecy.

(James D. Price, Ph.D., Professor of Hebrew and Old Testament, Temple Baptist Seminary,
Chattanooga, TN 37404; e-mail drjdprice@aol.com)

Responding to the Undeniable


Farrell Till
By responding to my May/June article before the second part was published in the
July/August issue, Dr. Price has created a problem that I will have to work around. Responses
to most of what he said in the foregoing article have already been made in part two of my
rebuttal, "Testing the Null Hypothesis" (July/August, pp. 2-6, 11), so rather than rehashing
those points, I will be referring readers to that article throughout this one. Following my
rebuttal points would be easier if you would have the July/August issue at hand as you are
reading this.

Failure to prove the denial: Dr. Price began his latest article with the astonishing claim that
"(t)he proof of a proposition is in the failure to prove the denial." I have to wonder just where-
-if anywhere--Dr. Price studied logic, because all logic textbooks that I have ever read
recognize that the lack of negative evidence does not constitute positive evidence, just as the
lack of positive evidence does not constitute negative evidence. In other words, if I cannot
prove that an alien spacecraft did not crash near Roswell, NM, 50 years ago, my failure to
prove that this did not happen in no way constitutes proof that it did. This is so elementary
that I can't believe Dr. Price is not aware of the universal recognition of this logical axiom,
but desperation has apparently driven him to claim that his proposition must be true because I

Volume 1990 - 2002 Issue


Page 1051 of 2049
Skeptical Review Edited by Farrell Till
have not proven that it is not true. Such an obvious resort to logical absurdity must result
from Price's painful recognition that he has not proven his prophecy-fulfillment claim, and so
his only recourse is to declare victory by default.

The negative's responsibility: Dr. Price said that "(i)f the defender of a proposition must
produce objective evidence to prove it, then the opponent of the proposition must present
objective evidence to disprove it," so it would appear that in addition to being uninformed in
basic principles of logic, Dr. Price also knows very little about debating principles. The
affirmant's role in a debate is to establish the truth of his proposition; the negative's role is to
rebut the affirmant's arguments to show that they are insufficient to prove his proposition. In
discharging his responsibility, the negative may well prove that his opponent's proposition is
not true, but he doesn't have to. If he establishes sufficient reasons to doubt the truth of the
affirmant's proposition, then he has accomplished what the negative side should do in a
debate. Let's consider the Roswell controversy again. The person who affirms that an alien
spacecraft crashed near Roswell 50 years ago would have the burden of proving that this
incident is historical fact; the opposition wouldn't have to prove that the incident did not
happen in order to demolish the affirmant's case. If the negative demonstrates that the
evidence is insufficient to establish the extraordinary claim that the incident did happen, then
the affirmative side has lost the debate. In the matter before us, I have clearly demonstrated
that Dr. Price's evidence is conspicuously insufficient to establish his prophecy-fulfillment
claim. Therefore, I have done what I am supposed to do in this debate.

Why should the burden of proof be one-sided? In debating an unextraordinary proposition,


both sides should share the burden of proof. Would raising taxes cause an economic
recession? Two politicians debating this question would both have a responsibility to defend
their positions, but in the matter of extraordinary claims, the burden of proof rests entirely on
the claimants by virtue of the nature of the claims. Such claims engender doubts and denials
only because the claims have been made. If, for example, no one had ever claimed that a man
named Jesus of Nazareth rose from the death, there would never have been anyone to deny the
claim. If no one had ever claimed that Joseph Smith was visited by an angel who gave him
golden plates on which a revelation from God had been inscribed, there would never have
been anyone to deny the claim. If no one had ever claimed that the prophet Muhammed
ascended into heaven, there would never have been anyone to deny the claim. It is the
extraordinary claim itself that fosters doubt, so the ones who create the controversy have the
duty to prove their extraordinary claims; those who reject them have no duty to prove that the
claims are not true.

I have been over and over this, so rather than continually harping about my insistence that he
prove his case, Dr. Price should accept the recognized burden-of-proof principle or else
present an argument that will establish logical reasons why the one who questions an
extraordinary claim should bear the responsibility of proving that the claim is not true. I really
don't believe that Dr. Price is willing to allow others to impose on him the same duty that he is
trying to push onto me. If this were done, he would have the responsibility of proving that
various claims of Mormons, Muslims, Hindus, Zoroastrians, etc. are not true, and I certainly
don't believe that he is willing to assume that responsibility. What all of this amounts to, then,
is that Dr. Price realizes that he cannot prove his proposition, and so he seeks to draw
attention away from his failure by demanding that I prove that his prophecy-fulfillment claim

Volume 1990 - 2002 Issue


Page 1052 of 2049
Skeptical Review Edited by Farrell Till
is not true. His tactic is not going to work, because the critical-thinking skills of most readers
of this paper are too sharp to fall for that ploy. If Dr. Price wants to talk about "rabbit chases,"
he should take a long look at this one, which he has been trying to divert me to ever since this
discussion began.

Do supernatural claims require supernatural proof? Dr. Price has evidently been reading
my debate with Michael Horner, because he has taken a cue from Horner and argued that
when I say that extraordinary claims require extraordinary proof, I am actually say that
"supernatural claims require supernatural proof." I'm going to surprise him and admit that in
the cases of some extraordinary claims, this is exactly what I am saying. The claim that a dead
man returned to life or that a man was born to a virgin mother are such that nothing short of
supernatural evidence would be sufficient to establish them as historical facts. In the case of a
virgin-birth claim, for example, there is the obvious fact that only the woman who allegedly
had such an experience as this could possibly know that she was a virgin at the time. The
probability that a woman would tell a lie like this in order to spare herself social
embarrassment is far greater than the possibility that she could have actually conceived a
child without having had sexual relations, so I can't imagine how a claim like this could ever
be proven without supernatural evidence. Dr. Price intended his statement as a reproach, but I
see nothing at all disgraceful about agreeing that some extraordinary claims are such that only
supernatural evidence could establish them as facts. Price asked what kind of supernatural
evidence I would accept, whether the witness of an angel or an affidavit from God himself.
Well, why not at least one or the other or something comparable? Angels rather routinely
visited biblical characters, and God himself even dropped by occasionally to chat with
Abraham and Moses. If it happened then, why would it be unreasonable to expect the same
type of confirmation today? Rather than ridiculing those of us who demand unequivocal
evidence before we accept fabulous claims, Dr. Price should show why such a demand is
unreasonable. He hasn't done that. He has ranted about "anti-supernatural bias," but he has yet
to explain why it is in any way irrational to demand unimpeachable evidence before accepting
fabulous claims like those that characterize the Bible.

What law of nature does prediction violate? Desperation has apparently driven Dr. Price to
argue that "prediction" does not violate natural laws. In so doing, he has resorted to
equivocation on the word prediction. He said that "(n)early every law of nature includes the
capacity for predicting future events," but in equating a biblical prophecy like Jeremiah's to
Newton's laws of motion, he has committed the fallacy of false analogy by comparing two
entirely different kinds of prediction. Events that happen as a result of fixed laws of physics
cannot be compared to events that are dependent on whimsical human decisions. A scientist
who understands the laws of planetary motion can accurately predict the positions of moons
and planets at given moments, even thousands of years into the future, but this is because they
are objects that cannot make decisions and exercise options. They have no choice but to
follow the orbital paths determined by fixed laws of motion. Events that result from the policy
decisions of political rulers, however, are entirely different. For example, in the matter of the
"prophecy" under consideration, there are no fixed laws of human behavior by which
Jeremiah could have known what specific events would occur over an extended period and
how Nebuchadnezzar and his successors would react to them, and so there are no "laws" that
would have enabled Jeremiah to predict the end of the Babylonian captivity at a certain time.
If such a prediction as this were actually made and fulfilled, as Price claims, it could have

Volume 1990 - 2002 Issue


Page 1053 of 2049
Skeptical Review Edited by Farrell Till
happened only by one of two ways: (1) it was supernaturally revealed to Jeremiah, or (2) it
happened by sheer coincidence. The first alternative is what Dr. Price must prove, because if
it happened by sheer coincidence, this would reduce biblical prophecy to nothing better than
the tabloid prophecies, which "psychics" make by the hundreds, in vague, imprecise language,
and then brag if one of them appears to happen. That Dr. Price would resort to such an
obviously false analogy as this indicates that he knows just how weak his position is.

Radical Skepticism: Eight times in his latest article, Dr. Price used terms like "radical
skeptics," "radical critics," and "radical dogmatism" to refer to those who reject biblical
miracle claims, but he described his own position as "moderate and reasonable skepticism" (p.
3). This is a common tactic in debating known as "poisoning the well," which occurs when a
debater attempts to discredit his opponent in a way that will make the opponent's position
appear unattractive. What Dr. Price is doing, in effect, is saying to our readers that they can
accept my position and become "radicals" or else they can accept his position and be
"moderate and reasonable skeptics." Whether one's position on an issue is "radical" or
"moderate," however, has nothing to do with whether the position is true or false, and so such
rhetoric as this is out of place in a debate.

My own unverifiable fabulous claims: Dr. Price charged that my "own anti-supernatural
presupposition is based on a sequence of unverifiable fabulous claims" (p. 2) and used the
issue of God's existence to try to prove the charge. This, by the way, has come after his
objections to a few references I have made to biblical inerrancy. He complained that inerrancy
has nothing to do with the accuracy of a prophecy written only in a book that is allegedly
inerrant, but he seems to think that the question of God's existence is relevant. I am going to
be brief in my response to this point, because in the very first issue of TSR, I stated that
biblical inerrancy would be the focal point of this paper and specifically mentioned the
question of God's existence as an issue that would not be discussed. I personally find the
subject fascinating, but if I should publish articles on the subject, they would divert attention
from the purpose of the paper, which is to discuss the inerrancy doctrine.

If Dr. Price is interested in discussing the existence issue, I would be glad to participate on
one of the many internet sites where this subject is debated, but here, in this paper, I am going
to confine my remarks to a rebuttal of his claim that my position on this question entails
acceptance of "a sequence of unverifiable fabulous claims." As many Christians and theists on
internet sites can confirm, I have stated many times that my position on the existence of God
is simply that there is insufficient evidence to warrant belief in a deity. I do not assert that a
deity does not exist; I simply say that I do not believe in the existence of a deity. In the same
way, I do not assert that alien beings are not visiting earth in spacecrafts known as UFOs; I
merely say that I do not believe that this is happening. In both cases, my opinion is based on
insufficient evidence to establish belief. Dr. Price may see this as a presupposition "based on a
sequence of unverifiable fabulous claims," but I consider it sound common sense. It is a
common sense that he himself uses to evaluate and reject fabulous nonbiblical claims.

As for his claim that the second law of thermodynamics requires a "beginning in the past" for
the universe and an "end in the future," he may want to take this matter up with the many
physicists who see nothing in the second law of thermodynamics to require the extrapolation
of a deity to explain the origin of the universe. If the laws of thermodynamics so obviously

Volume 1990 - 2002 Issue


Page 1054 of 2049
Skeptical Review Edited by Farrell Till
require the existence of a deity to explain the universe, then why do so many physicists reject
the God-did-it theory and continue to look for naturalistic explanations? If Dr. Price is
interested in checking into the subject of naturalistic explanations of the universe, I suggest
that he read "What Happened Before the Big Bang?" (Astronomy, May 1996, pp. 34-41) by
Michio Kaku, professor of theoretical physics at the City University of New York. He will see
a reputable physicist proposing a "multiverse/superstring" theory, which in effect makes
matter eternal and our universe only one bubble of many fluctuating from "an infinite ocean
frothing with universes" (p. 37). The article names physicists, including Nobel laureates, who
are giving this theory their serious consideration. Kaku said of this theory that the "sane
proposals" for a theory of everything [which would unify both general relativity and the
quantum theory] have all been proven "mathematically inconsistent," and so the only
candidate left at the present time is the multiverse/ superstring theory (p. 40). This theory was
the subject of "The Self-Reproducing Inflationary Universe," Scientific American, (November
1994), in which the author, Adrei Linde of Stanford University, said, "If my colleagues and I
are right, we may soon be saying goodbye to the idea that our universe was a single fireball
created in the big bang" (p. 48). After explaining over the space of seven pages the
inflationary model of the universe envisioned by this theory, Linde said, "In this scenario the
universe as a whole is immortal. Each particular part of the universe may stem from a
singularity somewhere in the past, and it may end up in a singularity somewhere in the future.
There is, however, no end for the evolution of the entire universe" (emphasis added, p. 54).
Dr. Price may think that my rejection of his God-did-it explanation of the universe is "based
on a sequence of unverifiable fabulous claims," but if it is, I find myself in some very good
company.

What theologically motivated double standard? Because of my insistence on


unimpeachable evidence to support his prophecy-fulfillment claim, Dr. Price has accused me
of wanting "to evaluate historical evidence by a double standard, one that admits the
possibility of natural events but denies the possibility of supernatural events" (p. 3). The
"possibility" of natural events? Does anyone seriously question the possibility of natural
events? Once again, Dr. Price is comparing apples to oranges by trying to equate that which is
known and recognized by everyone with that which has never been and cannot now be
verified as reality.

He complained that my insistence on extraordinary evidence for his extraordinary claim


would require the debate to proceed "not on a level field" but with a "theologically motivated
double standard." However, if there is any "theologically motivated standard" at work in this
debate, it is entirely on his side. For one thing, I have no theology, because I have no belief in
gods, and as I have repeatedly pointed out, he is the one demanding that we play on an
unlevel field, which would allow him to argue from the assumption that biblical miracles, in
every case without exception, happened exactly as recorded, whereas my insistence has been
only that biblical miracles should and must be evaluated by the same standards that are
applied to other literary works. Just where would that make the field unlevel? If I would not
accept claim X if it were recorded in a book written by someone named Tacitus or Suetonius,
there are no logical reasons why I should accept it just because it was recorded in a book that
has Holy Bible embossed on the cover. This is the standard that I have insisted on, and Dr.
Price has yet to show us why this is an unreasonable standard.

Volume 1990 - 2002 Issue


Page 1055 of 2049
Skeptical Review Edited by Farrell Till
In the March/April issue, I published "The Nature of the Claim" (pp. 10-11) to explain why
rational people do not accept supernatural claims. The reason is simple. We see the natural
repeating itself daily, but we do not see the supernatural occurring at all. Therefore, it is
irrational to believe that the supernatural happened with the routine regularity claimed in the
Bible and other ancient books. The mere fact that we do not see supernatural events occurring
in our enlightened, scientific age is a strong reason to assume that the many claims of
miraculous events in ancient literature resulted from the ignorance and superstition of the
times.

Dr. Price could easily end this point of controversy by just giving us unequivocal evidence
that the supernatural does happen, but he refuses to discuss this beyond simply asserting that I
have an "anti-supernatural bias" because I won't accept extraordinary claims without
extraordinary evidence. In "The Nature of the Claim," I listed several supernatural claims
recorded in the works of Josephus, Suetonius, the Qur'an, and Mormon literature, and asked
Dr. Price to address these and tell us why he does or does not accept them. In his latest article,
Dr. Price just waved at this issue on his way by it. He said that he had discussed this issue
with me "in previous e-mail exchanges" and that he had informed me that he advocates
"healthy skepticism, a skepticism that evaluates the evidence and accepts what is verifiable, a
skepticism that is willing to give a reputable witness the benefit of the doubt" (p. 3). So Dr.
Price's standard seems to be that if one accepts biblical miracle claims, he has a "healthy
skepticism" and is "willing to give a reputable witness the benefit of the doubt," but if he
evaluates biblical miracle claims by the same methods used to evaluate nonbiblical miracle
claims, he is a "radical skeptic." I fail to see any logic at all in such a position.

Dr. Price said that it is irrelevant of me to "insist that [he] waste time by identifying what
extrabiblical miracles [he] would accept as historical." He argued that this "has nothing to do
with the debate at hand" (p. 3), but why isn't it relevant? He accuses me of imposing a double
standard on the debate and of having an "anti-supernatural bias," so he should feel some
obligation to show us that his standards are consistent and that they are consistent in all
situations and not just in those that involve biblical claims. Otherwise, we have a case of the
pot calling the kettle black, for if his standards for evaluating miracle claims are not consistent
in all situations, biblical and nonbiblical alike, it will be very obvious just who has a bias.

Price claimed that in a previous e-mail exchange with me, he had documented "two modern
verifiable miracles" for which "the evidence is undeniable" (p. 3). I didn't remember any such
exchange, and so I asked Price to send the information to me again. When it arrived, these
modern miracles turned out to be healing-by-prayer incidents. In one case, the father of one of
Dr. Price's students was diagnosed with terminal cancer and had been given less than one
month to live, but the members of his church "fervently prayed for his healing." Within "a
short time," his "vigor returned," and he went back to the doctors, who ran new tests and
"found that his body was completely free of cancer." The man is still alive today to verify the
story.

The second case was "a beautiful 19-year-old young woman by the name of Marolyn Ford,"
who in 1960 "began to have trouble with her eyesight." She was examined by an
ophthalmologist and told that she had "macular degeneration, the deterioration of the retina in
the area of central vision." The condition is untreatable, so "over a period of time her

Volume 1990 - 2002 Issue


Page 1056 of 2049
Skeptical Review Edited by Farrell Till
blindness advanced until she became totally blind." She was told by medical experts that her
retinas were hopelessly deteriorated and the optic nerves were dead." She remained blind until
the night of August 25, 1972, when she was 31, and her husband and she prayed fervently that
God would restore her sight." To make the long story short, her vision was completely
restored that night to the point that she was able to read even fine print in newspapers. She
returned to her eye specialist "who examined her and found that her retinas were still
deteriorated and full of holes." Despite this condition, she still is able to see, and eye
specialists can't explain why. Dr. Price cited These Blind Eyes Now See by Marolyn Ford and
Phyllis Boykin as documentary evidence that this miracle actually happened.

Dr. Price was able himself to see a problem in claiming these cases as genuine miracles,
because he immediately made this comment:

Now Mr. Till will attempt to explain away these verifiable accounts by claiming that on rare
occasions such recoveries occur naturally. But this is merely a radical skeptic's form of
rationalization, because the fact remains that there is no scientific explanation for them, and
the recoveries cannot be reproduced in a laboratory: they do indeed meet the definition of a
supernatural event, and some medical experts honestly regard them as miracles.

So even though Dr. Price apparently recognizes that cases like these occur naturally on rare
occasions, he states that it would be "merely a radical skeptic's form of rationalization" if I
should attempt to explain them in this way. He is very quick to pin the label of "radical
skeptic" on anyone who disagrees with his supernatural inclinations, but if he admits that such
cases as these occur naturally, how does he know that these two were not just rare natural
occurrences? Does he think that spontaneous remissions of cancer are always miracles? If not,
then on what basis does he conclude that the student's father experienced a miracle and not
just a natural spontaneous remission?

Marolyn Ford's case is a different matter, and I view it with considerable suspicion. First, I
couldn't help noticing that her book These Blind Eyes Now See was published by Christ For
the World Publishers in Orlando, Florida. I'm not familiar with the company, but it sounds
very much like a small religious press, and such publishing companies are notorious for
printing poorly researched books. One would think that if such an experience as Ms. Ford
claims had really happened and could be documented by incontestable medical records, this
book would have easily been accepted by major publishing companies, where it would have
found a much larger audience. I would be very interested in knowing why it wasn't submitted
to such publishers and if it was, why it was rejected. This is information a "radical skeptic"
would want to know before deciding to accept this case as a genuine miracle.

For the sake of argument, let's just assume that both of Dr. Price's examples can be confirmed
by incontestable medical records to the extent that even a "radical skeptic" would have to
agree that these were both supernatural events. We would then have two extraordinary claims
that were confirmed by extraordinary evidence, and that is what I have been arguing all along:
extraordinary claims require extraordinary evidence. Now if Dr. Price could just provide us
with unimpeachable evidence to prove his prophecy-fulfillment claim, we could end this
discussion and go on to other tasks.

Volume 1990 - 2002 Issue


Page 1057 of 2049
Skeptical Review Edited by Farrell Till
In the case of Jeremiah's prophecy, the evidence would not have to be supernatural. If Dr.
Price could produce undeniable corroborative evidence that Jeremiah wrote the prophecy in
605 B. C. and that the text of his book was never revised or edited after that date, that the
Judean captives whom Jeremiah was referring to in chapter 25 were taken to Babylon that
same year, and that they were released from captivity in 535 B. C., exactly 70 years later, then
we would have the evidence that would be necessary to confirm that an extraordinary
prophecy-fulfillment did happen. But where is Dr. Price going to find that kind of evidence?
There are no nonbiblical records from Syria or Egypt or Babylon that mention a Hebrew
prophet named Jeremiah, who prophesied during the first year of Nebuchadnezzar's reign that
the Jews would be taken into bondage but released after 70 years. There were no archives of
that time where original documents were recorded so that they could be referred to later to
verify the accuracy of subsequent copies. In a word, there is simply no way to establish that
Jeremiah made his prophecy at the time that Dr. Price alleges. Hence, there is no way for him
to satisfy criterion number 2 in my list of valid prophecy requirements, which states that one
claiming a prophecy fulfillment must prove that the prophecy was made before and not after
the event. Since Dr. Price has said that these criteria were "satisfactory," I suggest that he get
busy and try to satisfy this one.

The allegedly valid Text: Dr. Price trotted out the same old saw that we hear so often from
biblicists who cannot support the historicity of their claims: If we can't trust the Bible, then
"all ancient history is invalid" (p. 5). Price doesn't seem to understand that historians don't just
automatically accept everything that ancient documents say. They evaluate the information
and make judgments based on sound critical methods. If historians accepted everything that
ancient documents say, then history books would be filled with tales of men who were born of
virgins, performed various miracles, received visions, and foretold future events, but even
though these claims permeate ancient literature, they have not been presented as facts in
reputable histories, because they were rejected by common-sense critical methods of
evaluation. Price apparently expects the Bible to be exempted from the same type of scrutiny.

The authorship of Jeremiah: "The authorship of an ancient document is determined by the


internal claims of authorship," Dr. Price said, and then paraded before us four lines of
scripture references in Jeremiah where the text says "the word of the LORD (or its equivalent)
came to Jeremiah," but if such as this proves anything about authorship, then a Mormon could
prove that the books of Nephi were written by a man named Nephi, who lived in Jerusalem
during the reign of Zedekiah; that the book of Jacob was written by a man named Jacob 55
years after Nephi left Jerusalem; that the book of Enos was written by a man named Enos,
etc., etc., etc. Dr. Price is approaching this subject as if he thinks that in a time when there
were no copyrights or national archives to protect the integrity of documents, it would have
been impossible that someone could have altered or revised a text after its author had
completed it. To make his case, however, he must present to us unimpeachable evidence that
scholars are wrong when they say that this is exactly what happened to the book of Jeremiah,
which is the work of many writers over a long period of time.

The three external witnesses: Dr. Price argued that there are also three "external" witnesses
to the 70-year prophecy, because 2 Chronicles 36:12, 21-22; Ezra 1:1; and Daniel 9:2 all refer
to Jeremiah's prophecy. However, all of these books are recognized as postexilic works. As
such, they were written after the Babylonian captivity, so they would not constitute evidence

Volume 1990 - 2002 Issue


Page 1058 of 2049
Skeptical Review Edited by Farrell Till
that Jeremiah's 70-year prophecy was in his original work. The most they could prove would
be that the prophecy was in the Jeremiah text at the time that the books of Chronicles, Ezra,
and Daniel were written. What Dr. Price must find is clear, incontestable evidence in external
sources that Jeremiah made this prophecy before the fact and that it was not put into the text
by revisionist scribes and editors.

In "Testing the Null Hypothesis," I discussed the scholarly evidence that indicates that the
book of Jeremiah "is the product of growth over a long period of time, to which many
contributed" (The Interpreter's Bible, vol. 5, p. 787). I explained how that textual critics have
identified Deuteronomic sections of the book, which were added toward the end of the
captivity, and postexilic sections that were written even later, after the exile had ended. Both
places where the 70-year prophecy appears are in sections that have been identified as late
editions to the book. These are identifications that have been made through analyses of style,
grammar, syntax, vocabulary, and theme, and the methods are recognized as legitimate
critical devices. For a more complete discussion of this subject, readers and Dr. Price may
consult pages 2 and 3 of the July/August issue of TSR. The evidence is sufficient to establish
doubt that Jeremiah himself actually wrote the 70-year prophecy, and as long as there is
reasonable doubt, Dr. Price doesn't have a case.

He was aware of this textual problem, of course, because he tried to make a preemptive strike
against it: "Mr. Till will continue quibbling over authorship based on denials written by
radical critics." Thus, we see again how Dr. Price responds to opinions that differ from his. He
simply calls them "radical," as if a single buzz word is sufficient to refute them. He went on to
say that "these denials are not based on objective evidence, but on subjective theories built on
theological presuppositions like Till's anti-supernatural one" (p. 5). I guess we are supposed to
believe that Dr. Price's position is not based on a theological presupposition that the Bible is
infallibly true in everything it says. If he would bother to examine the literature on this
subject, he will find that these "radical" opinions about the authorship of Jeremiah are based
on careful analyses of style, grammar, vocabulary, syntax, and themes. It is far more objective
than his claim that Jeremiah must have made this prophecy because it is in a book that bears
the name "Jeremiah."

In the issue of Jeremiah's 70-year prophecy, Price could help his case if he could just present
some good solid ordinary evidence to support key elements of his argument. A crucial
element is the matter of when the prophecy was written. In the July/August edition of TSR (p.
5), I showed that the exiles to whom the prophecy was directed were those who were taken
captive in Nebuchadnezzar's destruction of Jerusalem in 597 B. C. and not in 605 B. C., as Dr.
Price has been claiming. Thus, the release of the captives in 537 would have constituted a
captivity period of only about 60 years rather than 70. Since there is nothing extraordinary
about a prophecy that failed, we really don't have an extraordinary claim in the case of
Jeremiah's 70-year prophecy. At the very least, however, Dr. Price would have to present
clear evidence that Jeremiah made the prophecy in 605 B. C., and I can't think of any way he
could do that without either producing a copy of Jeremiah that he can show was written and
completed by 605 B. C. or else some kind of corroborative nonbiblical record that would
confirm that Jeremiah made such a prophecy in 605 B. C.

Volume 1990 - 2002 Issue


Page 1059 of 2049
Skeptical Review Edited by Farrell Till
Baruch's role: As I pointed out, the 52nd chapter of Jeremiah shows internal evidence of a
late authorship, but Dr. Price dismisses this chapter as just a late edition by Baruch, which he
determines by citing 51:64, which says, "Thus far are the words of Jeremiah." So Price argued
that the internal evidence "attributes the content of chapters one through fifty-one to
Jeremiah" (p. 6). However, in the Septuagint, which Price has claimed was just a short edition
of Jeremiah, chapter 51 is chapter 28, and 51:64 isn't even in the text. Furthermore, chapter 52
in the Septuagint begins with verses that aren't in the Masoretic: "The word which Jeremias
the prophet spoke to Baruch son of Nerias, when he wrote these words in the book from the
mouth of Jeremias...." After two more verses assuring readers that these are the words that
"the LORD said to thee, O Baruch," the text goes on to present the last chapter of the book
with essentially the same content that is in the Masoretic text. Hence, the Septuagint claims
that this chapter contains words that were from the mouths of both Jeremiah and Yahweh. The
concluding paragraph refers to events that happened in the 37th year of the captivity, so if the
words in this chapter are really words that came from the mouth of Jeremiah, then Jeremiah
didn't complete the book until at least 37 years into the captivity. If this is the case, did
Jeremiah live long enough to feel safe in putting into the book a prediction that the captivity
would last 70 years?

This doesn't really matter, because I showed in "Testing the Null Hypothesis" (July/August, p.
5) that Dr. Price's 605 B. C. dating of the prophecy is clearly wrong and that "Jeremiah" had
directed the prophecy to the exiles who had been captured in 597 B. C. and that these were the
ones who would serve Babylon for 70 years. Readers and Dr. Price should review the section
subtitled "The 605 Date" to see that even the internal evidence of Jeremiah and extrabiblical
records will allow for only a captivity of 60 years. Hence there was no fulfilled prophecy.

Evidence of textual tampering: Dr. Price argued that "(n)o textual differences exist [in
Jeremiah] that indicate that the author of the historical appendix [Baruch's additions] made
any significant changes to Jeremiah's portion of the text" (p. 5). In response to this, I will
simply refer readers to my July/August article in which I pointed out several striking
similarities in the text of Jeremiah and other biblical passages. (See the section subtitled
"Criteria 2 and 3," pp. 2-3). These examples, which were by no means exhaustive, are
sufficient to cast serious doubt on Price's apparent belief that Jeremiah wrote all but the end of
this book and that it has remained unchanged through centuries of transmission by
handwritten copies.

The debate has come down to three questions: (1) Is there "undeniable" evidence that
Jeremiah himself made the prophecy attributed to him? (2) Is there "undeniable" evidence that
Jeremiah intended for this prophecy to date from 605 B. C.? (3) Did the captives whom
Jeremiah addressed in the prophecy serve 70 years in Babylon? We all understand by now
that Dr. Price thinks that I have an "anti-supernatural bias" and that I am a "radical skeptic"
and that all scholars who disagree with him on this issue are "radical critics," so if he chooses
to continue this debate, I will insist that he drop the ad hominem rhetoric and address the three
questions above.

Volume 1990 - 2002 Issue


Page 1060 of 2049
Skeptical Review Edited by Farrell Till

Editorial Potpourri
Farrell Till
For some time, I have needed to publish an article about the growing pains that The Skeptical
Review has been steadily experiencing, but the space always seemed to be needed for
something else. With this issue, I decided to solve the problem by writing the article first and
then forcing everything else to fit into the space left.

Unanswered Correspondence: I am quite frankly embarrassed by the amount of unanswered


mail that is piled in bundles on and under my desk. When I first started publishing TSR, I
thought that maybe I could eventually attract one or two hundred subscribers, but I never
expected it to reach ten times that many. In those days, the letters that came in now and then
always received my immediate and personal attention. Those who ordered back issues and
debate transcripts received same-day service, but those days are long gone. There are letters
piled up that I would like very much to answer, but I just don't have the time.

Part of the problem is the nature of the letters. Many of them ask for assistance that would
require hours at my computer typing adequate responses. Whenever I undertake to answer a
letter, I have on my mind that I am investing time in writing a letter that will be read by only
one person. This was one of the reasons why I added "From the Mailbag" to the paper. This
column enables me to select letters and answer them, knowing that several people will benefit
from what both the letter writer and I may have to say.

Contributions: What is really embarrassing is the number of contributions that go


unacknowledged. TSR has never actively solicited contributions, but subscribers sometimes
voluntarily give to assist the work. I have always tried to send an acknowledgement of these,
but their increase in number has made it difficult to keep up with them. I plan to correct this
with a form letter that I can quickly send to contributors. I realize that this is impersonal, but it
seems the best solution. I want to assure those who have contributed that their generosity has
been appreciated. With the number of new subscribers that are now being attracted, it helps
me to continue the free first-year policy. Contributors may like to know that Skepticism, Inc.,
the official publisher of TSR has been recognized by IRS as a nonprofit 501(c) (3)
organization, so all contributions are legitimately deductible.

How You Can Help: The mail problem can't be eliminated, but it could be alleviated by
reducing the number of unnecessary letters. Although subscription information is always
printed at the bottom of page two, I get letters from people who want to know how much
money they should send to renew their subscriptions when all they would have had to do was
refer to this section of the latest issue. Information about back issues, ASCII copies of articles
on computer disks, written debates, video cassettes of oral debates, and other materials is not
included in every issue, but it is published two or three times per year, so if subscribers would
keep their back issues for at least a few months, they could consult them for this kind of
information. That would save me the time of having to answer letters that request information
that is regularly published.

Volume 1990 - 2002 Issue


Page 1061 of 2049
Skeptical Review Edited by Farrell Till
The convenience of e-mail: If you have e-mail, please use it to make inquiries about the
status of your subscription or to request information, because it will be much easier and
quicker for me to send you a reply. There have been times when people whom I know have e-
mail sent me letters by "snail mail" and didn't include their e-mail addresses. If for some
reason you prefer to send a regular postal letter, please include your e-mail address if you
have one, because I will want to answer by this faster, easier way.

Late Deliveries: I try to mail each issue during the first week of the first month of that issue.
Thus, the January/ February issue is due to be mailed the first week of January, the
March/April issue the first week of March, etc. However, this deadline cannot always be met.
Sometimes I may be a few days late getting an issue into the mail, but I don't recall ever being
over a week late. All mailings are sent at nonprofit bulk rates, so delivery is much slower than
first class. If you have not received your copy by the middle of the month in which it was
mailed, this is not at all unusual, especially if you live on the east or west coast. If you fail to
receive a copy, even though it is the fault of the mail service, I will send you a replacement
copy at no cost by first-class mail. However, I would like for you to wait until at least the
third and preferably the fourth week of the posting month to inquire about copies that haven't
yet been received. I have sometimes sent replacement copies only to learn later that the
originals had finally arrived. In March, the test copy that I always send myself took a month
to get into my mail box at the same post office where it was mailed. That's the disadvantage
of bulk mailing. It's cheap, but it doesn't get a high priority.

Interpreting the expiration dates: On your address labels, the expiration date of your
subscription is printed, but some have been misinterpreting the date. The first number is the
number of the issue and not the month of the year. Hence, 6-97 meant that your subscription
will expire with the sixth issue (November/December) of this year and not in June of this
year. Some, not understanding this, have sent their renewals six months in advance. These
subscriptions have been renewed for another year, and so 6-98 will appear on their labels until
they renew again.

If you wonder when your subscription will expire, all that you have to do is look at the three
numbers on the righthand side of your address label and remember that the first number,
which will range from 1 through 6, is the number of the issue, and the last two numbers are
the year in which the subscription will expire. Some have paid their subscriptions in advance
for several years, so they will see numbers like 2-00 (second issue of the year 2000), 3-01
(third issue of the year 2001), etc.

No expiration date on the label: If there is no expiration date on your label, this means that
your subscription is free for an indefinite period (probably the lifetime of the publication).
People in this group would include those whose articles (not letters) have been published in
TSR, people who perform services for the paper that I want to repay in some small way,
people known to be unable to pay for their subscriptions, etc.

Inmate Subscribers: In the "mailbag" column of this issue, there is a letter from a prison
inmate in Kentucky. When this issue went to press, there were 48 inmate subscribers in
various prisons across the country. Some of them pay the subscription cost, but most of them
don't, because they have informed me that they don't have the money to pay but would still

Volume 1990 - 2002 Issue


Page 1062 of 2049
Skeptical Review Edited by Farrell Till
like to receive the paper. I have kept many of them on the subscription list for several years,
but now I would like to ask a favor of them. At the end of each year, I would like for them to
write me at least a postcard to let me know if they want to continue receiving the paper. If so,
I will continue their subscriptions; if not, dropping them from the subscription list would help
reduce the cost of keeping the publication going.

Submissions for Publication: Articles submitted for publication should address biblical
inerrancy. TSR doesn't publish fiction, poetry, book reviews, or articles about doctrinal
matters that are not related to the inerrancy issue.

From the Mailbag


What's Wrong with Jamie?

You are indeed a breath of fresh air in a world of fundamentalist smoke and mirrors. As a
recovering--no, make that "recovered"--Church of Christer with some Bible Church thrown
in, I can't tell you how much your publication has meant to me. The first time I read it, I
thought, "I'm not crazy after all! `They' are!"

Thanks for giving me back my brain. Enclosed is a check to renew my subscription.

(Jamie Kay Smith, 3500 East Park Boulevard, Apt 104, Plano, TX 75074)

EDITOR'S NOTE: What's wrong with Jamie Smith? Doesn't she know that without God it's
just not possible for people to be happy? That's what we constantly hear from the "born
again," yet those who follow this column know that former Christians, especially the
fundamentalist kind, have written many times to express their happiness--no, not happiness;
joy--at having escaped from the shackles of religious insanity.

Another Born-Again Atheist...

Thank you for your enlightening articles in the Review. Since I started reading your articles,
I'm more convinced, through reason and the use thereof, that there is no God. The Bible,
which I was taught growing up as a Methodist, is a myth. I'm glad to know that there are
thousands like me, who are unbelievers and infidels. It's hard for my family members to
accept that I am an atheist, but I know the truth has set me free from religious dogmatism and
fiction.

Again, thanks to The Skeptical Review. I am enclosing a check to cover a 3 years' subscription
and a copy of each of the following (order deleted).

(Rene Eugenio, 19402 Elmira Court, Cerritos, CA 90703)

Volume 1990 - 2002 Issue


Page 1063 of 2049
Skeptical Review Edited by Farrell Till

Women without a Clue...

I do appreciate the way you explained the buzz words family values that the political religious
right loves to use (and countless others). You have the knowledge and expertise to explain the
ridiculous, boring, and idiotic to people who have been spoon fed religious and biblical
ramblings since birth. Please, do not stop at family values. There are so many women out
here in the U. S. who do not have a clue how or why women have been placed in second-
class citizenship. A few issues dedicated to this subject would be very enlightening and very
needed. As you know, religion has historically provided society with ideas and beliefs based
on the moral teachings of the Bible, which have shaped our culture and traditions and, sadly,
our views of what it means to be male and female. Christian religion condones and has
contributed to male domination, and actually encouraged the physical chastisement of
women, which long ago bloomed into domestic violence and continues today.

(Kova Lancaster, 2468 Baywood Court, Orange Park, FL 32065-6253)

EDITOR'S NOTE: "God's Opinion of Women" was published in the Autumn 1995 issue of
TSR, (pp. 11, 16), but in response to Ms. Lancaster's request, the front-page article in this
issue was written. Her suggestion is a good one, so I will try to include more on this subject.
My experience when I was a minister was that women formed the core of most congregations.
I found that when work needed to be done, women were generally more dependable than
men. After I began to see flaws in the fabric of the biblical inerrancy doctrine, I wondered
why so many women allowed themselves to be treated as inferiors of men. I still haven't
figured it out. I suppose it must be that people can be indoctrinated in their childhood to
believe anything.

Science and Religion...

I saw some excerpts of The Skeptical Review, on the internet and quite enjoyed them.
Although my main interests are in the physics area, I have been puzzling for 30 years about
why most people seem rather oblivious to the manifold serious errors of belief. Years ago, I
read Volume 2 of A. D. White's A History of the Warfare of Science with Theology and was
fascinated. Recently, I came across a grossly erroneous book attacking biological evolution
from a Christian viewpoint; then I encountered your Review and would very much like to
receive a year's subscription at no cost if possible, but I will pay if required.

(Michael O'Brien, 14932 SW 155 Terrace, Miami, FL 33187)

EDITOR'S NOTE: All first-year subscriptions are free. This is a policy I have maintained
from the beginning of TSR, because I believed that most people who give it a try will like it
and remain as paid subscribers. The policy has obviously worked.

The "warfare" between science and religion is indeed a puzzle. We have a situation where
those who enjoy unparalleled prosperity and comforts from the fruits of those who have
devoted their lives to science will gladly take all that science has to offer except anything that

Volume 1990 - 2002 Issue


Page 1064 of 2049
Skeptical Review Edited by Farrell Till
conflicts with their religious beliefs. This is especially true of the theory of evolution.
Scientists can make space probes, put them into rockets that they have programmed for trips
to other planets, and succeed in getting them to the exact destinations intended. Scientists can
now grow in laboratories organs from the tissue of animals and successfully transplant them
into the animals that donated the tissue, and it is just a matter of time till they will be able to
do the same in humans. Then people urgently needing new hearts, kidneys, livers, etc. will be
able to have their defective organs replaced with new ones grown from their own body tissues
so that rejection will no longer be a problem. I have no doubt that when this becomes a
reality, preachers and Christians will take advantage of it, but when that time comes, I also
have no doubt that upon leaving hospitals completely recovered, they will still oppose the
scientific community's worldwide acceptance of the theory of evolution. Biologists,
paleontologists, archaeologists, chemists, geologists, etc. spend their entire lives working in
laboratories and field observations and experiments, and conclude that evolution is the
explanation for the origin of species. Preachers and Christians read the Bible, generally
engage in no research or experimentation, and they conclude that the scientists are all wrong.
It is as if someone seriously ill should be told by leading specialists that he has cancer, and he
refused to accept the diagnosis because of something that he had read in a document
thousands of years old. So why does religion make people so oblivious to reality? I wish I
knew.

Biblical Holocausts....

The Skeptical Review is wonderfully absorbing to me. I have never read the Bible until
recently when I was looking specifically for all the holocausts it contains. I found Joshua's
very fine holocausts in his book right away, but I have bogged down looking for the others I
know they are there if I can only find them. So now I turn to The Skeptical Review for help. It
seems to me holocausts are worthy of some attention.

I know I cannot hope to find those outstanding examples by Attila the Hun, Genghis Khan,
and Tamerlane in the Bible, but I feel sure those in the Bible are comparable and were enacted
by kindred spirits.

Here in the very capital of the Bible Belt, I find intellectuals are a rarity, and so I hope this
will bring me contact with others who speak my language.

(John Coffin, 1959 West Division Street, Springfield, MO 65802)

EDITOR'S NOTE: I'm not familiar with many details of atrocities that accompanied the
military conquests of Attila the Hun and Genghis Khan, but I doubt if they could have been
more brutal than the Yahwistic massacres of the Old Testament. In addition to the ones
recorded in the first 11 chapters of Joshua, the most notorious ones were the massacres and
rape of the Midianite captives in Numbers 31 and the Amalekite women, children, and infants
in 1 Samuel 15.

If the head count of the virgin Midianite girls who were kept alive for their captives (vs:17-
18) is accurate, there were 32,000 of them. Since the text states that all of the male children

Volume 1990 - 2002 Issue


Page 1065 of 2049
Skeptical Review Edited by Farrell Till
and nonvirgin females were killed, there would have surely been 60,000 captives who were
killed. This calculation is based on the assumption that the number of male children would
have approximately equaled the number of virgin girls and that the nonvirgin women would
probably have equaled the number of virgin girls.

There isn't much information on which to base an estimate of the Amalekite massacre, except
that the text states that Saul "smote the Amalekites" from Havilah to Shur, an expression that
indicates the massacre extended over a wide range of territory. There are textual data,
however, that would make Joshua's massacres of the Canaanites greater than Hitler's massacre
of Jews in World War II. This conclusion is based on census figures given in the book of
Numbers for the Hebrews who had come out of Egypt. There were over 600,000 foot soldiers,
who were 20 years old or older (1:45-46). If there were this many males of military age, it
would be reasonable to assume that there would have been an approximate number of females
of the same age, so there would have been 1.2 million in this one age group. If there were this
many who were 20 or older, then it would not be unreasonable to assume that there would
have been that many adolescents, children, toddlers, and infants. The Levites were not
counted in the census, and we could hardly imagine that the military group would have
included the elderly and the infirm. So when all census numbers are taken into account and
reasonably interpreted, a total population of 2.5 to 3 million is arrived at. All but conservative
Bible scholars agree that the population numbers were probably exaggerated (as were many
other things in the Bible), but biblical inerrantists are stuck with the conclusions that are
necessitated by what is said in a book that they think is inerrant.

In Deuteronomy 7:1-2, Yahweh commanded the Israelites to go into Canaan and "utterly
destroy" seven nations that were "greater and mightier" than they, and Joshua states in several
places that this was done "as Yahweh had commanded" (10:40; 11:11, 15, 23). So if these
seven nations (listed in Deuteronomy) were greater and mightier than the 2.5 million
Israelites, there must have been over 17.5 million people living in Canaan. If, as Yahweh had
commanded, Joshua "utterly destroyed" them and "left nothing alive to breathe," then the
Israelites would have massacred three times as many as the estimated 6 million who were
killed in the Nazi holocaust. Every time I think about this, I wonder why the horrible
atrocities that the Jews experienced in Europe in the 30s and 40s do not cause them to
renounce their own genocidal history. This must be just another example of how religious
indoctrination makes people oblivious to the obvious.

Price and the Dead Sea Scrolls...

In Dr. Price's rebuttal in the May/June issue, I read the following sentence: "Further, Till
seems ignorant of the fact that numerous scrolls of nearly every book of the Old Testament
found in the caves of the Dead Sea, dating from the 1st century A. D. to as early as the 4th
Century B. C. verify that the Masoretic text in all its essential details was in existence at that
time" (p. 3).

Now I am totally lacking in any scientific or educational credentials, but I have read some
books on the Dead Sea Scrolls. It is my understanding that very little material from the Old
Testament has been found there. I have read that a large fragment of the book of Isaiah has

Volume 1990 - 2002 Issue


Page 1066 of 2049
Skeptical Review Edited by Farrell Till
been found but only small fragments of other books and that the bulk of the material is not
related to the Old Testament. It also seems that though all these writings date from before the
destruction of Jerusalem by the Romans in 70 C. E. and were therefore written before, during,
and shortly after the alleged life span of the biblical Jesus, no mention of events related to the
life of Jesus have been found. It therefore seems to me that Dr. Price's statement is inaccurate.

If I am mistaken here, I guess I will have to go back and reread some book to see where I
went astray.

(Dave Nelson, 3944 Chickory Road, Racine, WI 53403)

EDITOR'S NOTE: I'm no Dead Sea Scrolls scholar either, but my understanding is that Mr.
Nelson is essentially right. Most of the scrolls are not copies of Old Testament books but
writings of the Essene sect that produced them. The accuracy of saying that "the bulk of the
material is not related to the Old Testament" would depend upon what is meant, because
many of the scrolls concerned religious views based on the Jewish scriptures, and some of the
scrolls were commentaries on the Jewish scriptures. On page 6 of the July/August issue, I
responded to the same statement that Mr. Nelson has quoted from Price's article and cited
scholarly sources to show that most of the scrolls were "in a highly fragmented state." I
understand that even the scrolls that were in the best condition, such as the Isaiah scroll, were
not complete, because sections on the edges had crumbled with age. I think that Dr. Price will
have a great deal of difficulty proving his claim that the Dead Sea Scrolls "verify that the
Masoretic text in all its essential details was in existence" as early as the 4th century B. C.

Why Not Be Done with the Whole Mess?

In the interests of reason, logic, and common sense, why not just admit that the failed
"parousia" [second coming] thoroughly and forever destroyed the Christian faith and be done
with the whole damned mess? Am I missing something? Are not the statements absolute and
therefore not subject to any other interpretation? Please respond.

(E. L. Robbins, 2700 East Rapids Road, Apt. 17, Lansing, MI 48911-6312)

EDITOR'S NOTE: The only thing Mr. Robbins is missing is the Christian mindset, which
instills determination to believe that Jesus died and is coming again, no matter how
compelling evidence for the failure of the promise may be. Christians--at least the
fundamentalist kind--would argue that the New Testament statements about the second
coming are indeed absolute, but interpretation is another matter. Throughout its history,
Christianity has been adept at reinterpreting the scriptures whenever confronted with evidence
that indicated error in the Bible. When undeniable evidence that the earth is a sphere was
discovered, the faithful reinterpreted the flat-earth passages in the Bible and claimed that
biblical writers had taught a spherical earth all along. Even with the evidence for a spherical
earth being as overwhelming as it is, there are a few diehards who still cling to the biblical
view, and as incredible as it may seem, a Flat-Earth Society maintains its headquarters in
Lancaster, California.

Volume 1990 - 2002 Issue


Page 1067 of 2049
Skeptical Review Edited by Farrell Till
It isn't surprising, then, that when too much time had passed to make the promises of an
imminent return of Jesus credible, Christians shuffled to reinterpret the New Testament
passages that said he was returning soon. Many biblical scholars believe that 2 Peter was
written primarily to gloss over the promises in 1 Peter (4:7, 17; 5:4). The second epistle put a
different spin on the subject and advised Christians to understand that "one day is with the
Lord as a thousand years, and a thousand years as one day" (3:8) and went on to assure
Christians that "the Lord was not slack concerning his promises" but is just "longsuffering"
(v:9).

So that's why we can't just be done with it. The mindset of most Christians is so unreasonable
that we have to keep piling evidence onto evidence until their faith collapses under the weight
of too much nonsense. The first two letters in this issue show that this does happen
sometimes.

The Anti-Supernatural Bias...

I have a few thoughts on a concept that recurs often in The Skeptical Review. Regarding "anti-
supernatural bias," I think that for one who understands the nature of supernaturalism, the
concept is an oxymoron, and that you are therefore being accused of something which is
logically impossible.

Conclusions about alleged supernatural entities are based on mysticism, which is intentionally
nonrational. The word "bias" implies the inappropriate rejection of an assertion due to
nonrational factors, for instance a prior irrevocable commitment to an opposing viewpoint.
Supernaturalism is therefore founded upon bias and would not exist without it. If one
recognizes this and wishes to avoid bias, one must therefore reject supernaturalism.

What is being misunderstood as "anti-supernatural bias" is nothing more than "rational


methodology," in other words an attempt to reach conclusions that are distinguishable from
lies and delusions.

I welcome e-mail or other correspondence.

(John Burton, 7246 Jefferson Road, Magna, Utah 84044; e-mail JrhoBurton@aol.com)

EDITOR'S NOTE: Dr. Price, of course, is the one who has been bandying about the charge of
"anti-supernatural bias, as if it were an incontestable sign of irrationality. What Mr. Burton
says is true and is recognized even by Dr. Price, who applies rationality himself to evaluate
the merits of unlikely claims. As indicated in his article in this issue, he apparently accepts
some modern miracle claims, but he has conspicuously sidestepped the issue of miracle
claims in the nonbiblical literature of ancient times. From all indications, Dr. Price accepts all
miracle claims in the Bible, without exception, but rejects all similar claims in the nonbiblical
literature of ancient times. How can anyone who holds to a position like that expect anyone to
take him seriously?

Volume 1990 - 2002 Issue


Page 1068 of 2049
Skeptical Review Edited by Farrell Till
Publicizing TSR Locally...

For every critical remark you receive, there are thousands that praise you for freeing them.
Please tell me the best possible way to have TSR known throughout my area. Are there any
other books that I can read that will give me more insight into the Bible from a critical
perspective? I look forward to your future articles.

(Jason Wilson, 607 West Main, Iron Mountain, MI 49801)

EDITOR'S NOTE: I wish I knew the most effective way to publicize TSR locally. Some have
tried various methods: running classified ads in area newspapers, sending subscriptions to
local libraries, copying articles for hand-to-hand distribution, etc. The effectiveness of these
methods is at best questionable, because they have resulted in only a handful of inquiries. The
most effective method of publicizing TSR by far, in my opinion, has been the internet. Its web
cite brings inquiries almost daily, and subscription requests have come in from all over the
world. I would think, then, that if there is a local computer bulletin board in your area that this
might be your best bet.

I have already suggested Is It God's Word? by Joseph Wheless to Mr. Wilson. In addition to
that book, I would recommend Who Wrote the Bible? by Richard Elliott Friedman. Who
Wrote the New Testament? by Burton L. Mack, Forgeries in Christianity, also by Wheless,
and practically any book by Joseph McCabe, especially The Myth of the Resurrection and The
Forgery of the Old Testament.

Hobnobbing with Some Big Ones...

The Skeptical Review arrived yesterday, and as usual was quickly devoured by me. Of all the
papers I receive, TSR is the best. I love to read anything by Farrell Till, Judith Hayes, and
Dave Matson, plus many others.

Rev. Garry L. Creed's letter was a hoot. I can say that, since I was married to one of those
fundamentalist preachers for 25 years. We hobnobbed with an awful lot of them, including
Billy Graham, I might add. Religion is very big business, so they pull out all stops when it
comes to ignorance fertilization. When I first met Billy Graham, at least 35 years ago,
everyone was awe struck by him. I, ever the skeptic, saw him as a pompous, very rich,
religion machine. I think he could spell, though, and use English adequately.

I don't agree with Jeff Schmura about "too much minutia" in TSR. Till's explanations thrill and
educate me. This is what sets TSR far apart (and way above) all other atheist papers.

I am enclosing one of my satirical lyrics because Dave Matson's letter "What Is a Miracle?"
reminded me of it, so I dug it up.

Enclosed is payment for my next year's subscription to TSR.

(Dorothy B. Thompson, P. O. Box 562, Bandon, OR 97411)

Volume 1990 - 2002 Issue


Page 1069 of 2049
Skeptical Review Edited by Farrell Till
EDITOR'S NOTE: TSR does not publish fiction and poetry, much to the regret of many who
have sent these kinds of submissions; however, I am going to make an exception for Ms.
Thompson's poem, because it makes some observations about biblical inerrancy that are just
as incisive as some of the articles we have published.

Miracle Madness

Bible folks worked miracles,


According to "the book."
They parted waters of the seas,
All axioms forsook.
They raised the dead and talked to snakes,
Turned water into wine.
A stick became a serpent,
The sun refused to shine.
The dead were raised and donkeys spoke,
And rivers became blood.
Polar bears in Palestine,
And rabbits chewed the cud.
The mountains moved, the sun stood still,
A virgin girl gave birth.
The rain came down for forty days,
And flooded all the earth.
A person walked on water and
Didn't even sink.
A man was dead for half a week,
And then was in the pink.
But if nature's laws were voided,
By religious tittle tattle,
The universe would be unhinged,
And up a creek without a paddle.

Three Observations...

Three observations were inspired by the latest issue: (1) It was interesting to see you cite W.
S. McBirnie. I wonder if you know that he was a veteran figure in the Extreme Right/Crypto-
Fascist element in this country. He kept files on several thousand Americans whose opinions
were deemed "suspicious" by extremist standards, based mostly on rumor, innuendo,
conjecture, distortion, and paranoia. He was even allowed to take over the old subversive files
of the LAPD in 1975, once they realized they hadn't been using them (and they were
manifestly unfair to begin with). Last I heard of him, he was brought to court on a charge of
bilking old ladies, a standard tactic in Christbiz. With all of his political activity, I was
surprised to learn that he even bothered to do any real research into religion. Perhaps you
should have included a brief disparaging remark in your citation of him, for in him we see the
detrimental effects on one's character of taking the Bible seriously.

(2) The recent debate over how the apostles could have come to believe in the resurrection, if
it were not actually true, has surprisingly not brought up the "swoon theory": that Jesus wasn't
really dead when brought down from the cross, but later revived for long enough to create a
powerful impression of a miracle. This, though not original with him, was described in detail
by Hugh Schonfield in his best-selling book, The Passover Plot, which also focused on the

Volume 1990 - 2002 Issue


Page 1070 of 2049
Skeptical Review Edited by Farrell Till
"purposeful planning" by Jesus, a theory earlier suggested by Bolingbroke. I found the book
worthy of a good deal of consideration. Is the failure to refer to the "swoon theory" due to the
rejection of it among skeptics, or the fact that the book, being a best-seller, is hence
unacceptable?

(3) I recently came across a book by Hyam Maccoby entitled The Mythmaker: Paul and the
Invention of Christianity. Maccoby presents a highly persuasive case for an idea I've long
held, that Paul was the real inventor of Christianity. Maccoby also claims that Paul was
eventually considered a false apostle by the Jerusalem followers of Jesus, who virtually
excommunicated him for his bizarre conceptions. They, "the Nazarenes," had actually known
Jesus, and knew that he never claimed to be God, or that his death on the cross would save us
from hell. Jesus must be seen as a Messiah in the Jewish meaning of the term, as a king in the
mundane sense, but Paul made him into a metaphoric king and world savior. Paul further cast
Jesus in a Gnostic image, as an emissary from "above," who had come with salvific
knowledge, and simultaneously in the guise of the dying and rising god of the mystery cults
(particularly Attis, who died a bloody death on a tree and rose again, and whose cult was
strong in his native Tarsus). These ideas were easily accepted by a Gentile world already
familiar with them. Jews quite understandably rejected such concepts, but when the Romans
destroyed Judea in 70 A. D., the way was left clear for the Pauline interpretation to become
Christianity, with virtually no one left to dispute its authenticity. And history, as we know, is
written by the winners. Christianity is Paulism, and its scriptures were written from a Paulist
perspective that has little resemblance to "the historic Jesus," a man who was a Messianic
pretender and who did not make too big a stir in the world, certainly nowhere nearly as big as
the mythic image of Jesus.

What I want to know is, how did this book escape my awareness for so long? Why isn't it
being shouted about from the rooftops as the best explanation of the mysterious origins of
Christianity.

(Stephen Van Eck, Rural Route 1, Box 62, Rushville, PA 18839)

EDITOR'S NOTE: I don't know how Maccoby's book escaped Van Eck's attention. I read it
several years ago and have it in my personal library. I have seen it recommended in
freethought publications, and, although I would have to check to be sure, I believe that I once
recommended it in TSR. At any rate, I will recommend it now. It's a book well worth having
for anyone interested in studying the origins of Christianity.

I have not discussed the "swoon theory" in my articles on the resurrection, because in my
opinion it detracts from the real issue, which is the Christian obligation to prove that the
crucifixion of a man named Jesus ever occurred in the first place. There is no credible
historical evidence that it did, but certainly the lack of evidence to corroborate the
extraordinary events that allegedly surrounded the crucifixion (the three hours of midday
darkness and the resurrection of the many saints) constitutes sufficient reason to doubt that it
did. All discussions of the swoon theory that I have read approached the gospel accounts of
the crucifixion with the assumption that they are basically accurate reports written by men
who simply misinterpreted what happened. Thus, the theory supposes that Jesus was
crucified, taken down from the cross, and put into a tomb but that he wasn't actually dead. He

Volume 1990 - 2002 Issue


Page 1071 of 2049
Skeptical Review Edited by Farrell Till
was just mistakenly assumed to be dead, and so he later revived, came out of the tomb, and
was seen by his disciples, who thought that he had come back from the dead.

A problem that I see with this theory is that it obligates its proponents to concede the basic
accuracy of the whole crucifixion-resurrection scenario presented in the gospels. Thus, the
debate over the swoon theory usually gets bogged down in demands from the Christian side to
explain how that blood and water could have come out when Jesus's side was pierced with a
spear if he wasn't really dead or how disciples of Jesus could have wrapped his body in a
hundred pounds of spices without recognizing some sign that he was still alive, etc. So many
elements of myth and legend are in these accounts that I consider it the responsibility of
Christians to prove that any of it happened rather than my responsibility to prove that what
happened was simply misinterpreted.

In defense of the swoon theory, however, I will concede that it is a much more sensible view
than the Christian claim that Jesus was literally dead and returned to life. We know from our
own experiences that confirmation of death often requires expert verification, which has
sometimes even been known to err. We have all read newspaper accounts of people who were
pronounced dead by physicians, taken to morgues, and later discovered to be alive. If this can
happen today, how much more likely would it have been 2,000 years ago? The gospel of
Mark claims that Pilate wanted verification that Jesus was dead before he would release the
body to Joseph of Arimathea, and so he inquired of the centurion in charge of the crucifixion
and was told that Jesus was dead (15:44-45). But what medical qualifications would a
centurion have had to confirm death? Admittedly, a man in his position had no doubt seen
many dead bodies, but that wouldn't have necessarily qualified him to verify that Jesus was
dead. If a police officer today should report that he had shot and killed someone, but then the
body turned up missing and the person was later seen alive, what person in his right mind
would believe that this person had risen from the dead? Rational people would recognize the
obvious: the police officer, who had undoubtedly seen many dead bodies before, had
nevertheless simply made a mistake in declaring the person dead.

I had not heard of McBirnie's political activities and subsequent legal problems, but I don't
see this as a reason not to consider what he said in his book about the fate of the apostles. If I
had known about these allegations, I would not have thought it necessary to "include a brief
disparaging remark" in my citation of him, because the truth or falsity of information is
always independent of its sources. I have published articles by a writer whom I later learned is
a holocaust revisionist and probable anti-Semitist. Had I known this about him at the time, I
would have published his articles anyway, because they presented what I considered sound,
logical arguments against the biblical inerrancy doctrine. To follow any other editorial policy
would be to play the biblicists' game of declaring information false because it came from
sources, such as "liberals" or "atheists," whom they do not approve of.

To Whom It May Concern...

I am an inmate here at F. G. I., Ashland, KY, at the mercy of our government. I have always
been a God-fearing, Bible-believing person, but only because of morals and hand-me-down
traditions. Just like a lot of other people, I took it for granted that everything was true, but
here lately I have been searching for the truth, and what I have found out is shocking. It's like

Volume 1990 - 2002 Issue


Page 1072 of 2049
Skeptical Review Edited by Farrell Till
my mind is going through changes, reprogramming itself, by sorting out all the lies and
trickery that our laymen have laid on it. I'm not saying that I'm an atheist all the way. It's just
that I'm using logic and common sense. I read and study everything with an open mind (not
like brainwashed Christians, who read with a closed mind).

I read a book called Man and His Gods by Homer W. Smith, who also wrote Kamongo and
End of Illusion, and it really helped me to understand a lot. If you haven't read it, I would like
to invite you and all your readers to check it out. It's published by Little, Brown, and
Company, 200 West Street, Waltham, MA 02154 or call 1-800-759-0190. I also checked out
The Skeptical Review. So far, so good.

I would like to be put on your mailing list to receive your paper and also any other material
that you may want to send to help my studies in the truth, including any back issues that you
have. Keep up the good work. By the way, I read the March/April issue. One article spoke
something about family values. I liked it.

If you have anything on the Lockwood-Till Debate, please send it to me.

(James Morris, 92483-088 HA, P. O. Box 6001, Ashland, KY 41105-6001)

EDITOR'S NOTE: As I explained in "Editorial Potpourri" on page 11 of this issue, almost 50


prison inmates nationwide subscribe to TSR. The number has been steadily growing.

As this issue goes to press, I still have not received tapes of my debate with Bill Lockwood,
although telephone inquiries have been made and gone unanswered.

Volume 1990 - 2002 Issue


Page 1073 of 2049
Skeptical Review Edited by Farrell Till

Skeptical Review
Volume Eight, Number Six
November/December 1997
Farrell Till, editor

• The 'Error' of Balaam


The editor shows that the Mesopotamian prophet Balaam received a "bad press" from
New Testament writers, which is not justified by the story of Balaam asrelated in the
Old Testament.
• Biblical Miracles
Inerrantist Roger Hutchinson defends the historicity of biblical miracles onthe grounds
that an a priori belief in the accuracy of biblical accountswill enable one to know not
just that the miracles happened but that Godalso had to exist in order to make them
happen.
• Inerrantist Tail-Chasing
Farrell Till compares Hutchinson's circular reasoning to a dog chasing histail and
shows that Hutchinson's logic could be used to prove thehistoricity of all ancient
miracle claims.
• On the Matter of Proof
Dave Matson reacts to Dr. James D. Price's attempts to shift the burden of proof to
Editor Farrell Till in the debate about Jeremiah's 70-year prophecy and shows that the
skeptic's insistence on extraordinarily good evidence tosupport miracle claims is a
sound principle.
• Two More Criteria of Valid Prophecy Fulfillment
Richard Russell proposes two additional criteria that should be added to Till's list that
was presented to Dr. Price in the discussion of Jeremiah'sprophecy.
• Which Bible?
Steven Carr discusses variations in the New Testament text and the disagreements of
early Christians in matters of "orthodox" doctrines.
• Serpentine Logic
Dan Barker reviews the part of his debate with Dr. Walter Kaiser that concerned

Volume 1990 - 2002 Issue


Page 1074 of 2049
Skeptical Review Edited by Farrell Till
biblical claims about talking animals and shows that such storiesby definition must be
considered fables.
• From the Mailbag
Letters from readers are published on a variety of issues along with the editor's
comments on many of the letters.

The "Error" of Balaam


Farrell Till
If any biblical character ever got a raw deal from God's "inspired" writers, it was the
Mesopotamian prophet Balaam, who is famous for the conversation with an ass that Dan
Barker alluded to in his article on pages 9-11 of this issue. New Testament writers spoke of
Balaam as if he were the very epitome of evil. The writer of 2 Peter compared false teachers,
who "walk according to the flesh in the lust of uncleanness and despise authority," to Balaam:
"They have forsaken the right way and gone astray, following the way of Balaam the son of
Beor, who loved the wages of unrighteousness, but he was rebuked for his iniquity: a dumb
donkey speaking with a man's voice restrained the madness of the prophet" (2:15-16). In his
denunciation of ungodly men troubling the church, the writer of Jude made a similar
comparison: "Woe to them! For they have gone in the way of Cain, have run greedily in the
error of Balaam for profit, and perished in the rebellion of Korah" (v:11). In Revelation 2,
Jesus disparagingly compared false teachers in the church at Pergamos to Balaam: "But I have
a few things against you, because you have there those who hold the doctrine of Balaam, who
taught Balak to put a stumbling block before the children of Israel" (v:14). For some reason,
New Testament writers obviously held Balaam in very low esteem, but as we will see, the Old
Testament record painted an entirely different picture of him.

The passages cited above made two charges against Balaam: (1) he loved the "wages of
unrighteousness" and was "greedy for profit," and (2) he taught Balak to put a stumbling
block before the children of Israel. An examination of Balaam's story as it is told in chapters
22- 24 of Numbers, however, shows that there is no basis at all for characterizing him as New
Testament writers did.

Was Balaam greedy for profit? According to the Old Testament account, the opposite was
true. Balak, the king of Moab, was concerned about the Israelite hordes who had encamped on
his territory, and so he sent elders of both Moab and Midian to Balaam with "the rewards of
divination in their hand" (Num. 22:7) to pay Balaam to come to Moab and pronounce a curse
on the Israelites. Upon receiving the delegation and learning what they wanted, Balaam
invited them to stay the night so that Yahweh could speak to him (as Yahweh was so prone to
do in those days) about their proposal. In the night, Yahweh told Balaam not to go with the
men, and so Balaam informed them that Yahweh had refused him permission to go (v:13).

Volume 1990 - 2002 Issue


Page 1075 of 2049
Skeptical Review Edited by Farrell Till
That hardly sounds like the actions of a man who was greedy for the "wages of
unrighteousness," but if there was any doubt about whether he was motivated by a desire for
"profit," that question should have been settled by what allegedly happened next. Balak sent a
more "honorable" delegation of princes back to Balaam, who immediately informed them that
even "though Balak were to give me his house full of silver and gold, I could not go beyond
the word of Yahweh to do less or more" (v:18). That sounds even less like the words of a
money-hungry prophet.

Inerrantists try to resolve the discrepancy in the images of Balaam as he is depicted in the two
testaments by quibbling that Balaam did not end the matter at this point but invited the
delegation to spend the night with him while he inquired of Yahweh a second time. It is true
that the story claims that Balaam made the second inquiry, but it also states that Yahweh told
him on this night that "if the men are come to call you, arise and go with them, but only the
word that I speak to you--that you shall do" (v:20). So the next morning, Balaam saddled his
ass and went with the princes of Moab (v:21).

Inerrantists argue that Balaam's "error" was in going with the Moabite delegation after
Yahweh had told him not to go, but the story clearly states that on the occasion of the second
delegation's visit, Yahweh told Balaam to go. Balaam did exactly what Yahweh commanded,
yet the story says that "God's anger was kindled because [Balaam] went" (v: 22). That is
curious indeed. Yahweh told Balaam to go, Balaam did exactly what Yahweh told him to do,
and then Yahweh got angry and sent an angel to block the way so that Balaam's ass would
have to turn aside. Such obvious inconsistency as this could happen only in fabulous
literature, just as the tale of an ass talking to rebuke Balaam could only happen in fables (as
Dan Barker has ably pointed out in his article).

The story of Balaam continued through two more chapters, but nothing can be found there to
even suggest that Balaam "loved the wages of unrighteousness." As the story was told by the
Old Testament writer, whom inerrantists will claim was inspired of Yahweh, when Balaam
finally met the Moabite king, he restated what he had told the delegation, that he would speak
only the words that Yahweh put in his mouth (22:38). Balak repeatedly urged Balaam to
pronounce a curse on the Israelites, but each time Balaam would pronounce only blessings on
them. Finally, when Balak's "anger was kindled against Balaam," the prophet reminded him
that he had said all along that even "if Balak were to give me his house full of silver and gold,
I could not go beyond the word of Yahweh" (24:13). So just what was the "error" of Balaam,
whom the Old Testament presented as a man who always did exactly what Yahweh told him
to do?

As for Balaam's love for the "wages of unrighteousness" alleged by the writer of 2 Peter, the
Old Testament text implies that Balaam received no money at all from Balak. After three
failures to curse the Israelites, Balak angrily said to Balaam, "Now therefore flee to your
place. I said I would greatly honor you, but in fact, Yahweh has kept you back from honor"
(24:11). So just where was Balaam's greedy love of money?

Did Balaam teach Balak to lay a stumbling block before the Israelites? This charge is
even more unfair than the other, because if we are to believe the presumably inspired Old
Testament account, Balaam never spoke a single word against the Israelites. Everything he

Volume 1990 - 2002 Issue


Page 1076 of 2049
Skeptical Review Edited by Farrell Till
said was described as a "blessing" on them. Furthermore, this whole affair was presented as
Balak's idea, so he was the one desperately trying to get Balaam to lay a stumbling block
before the Israelites, not the other way around. The inconsistent views of Balaam by different
biblical writers is just one more reason why reasonable people cannot believe that an
omniscient, omnipotent deity had anything to do with the writing of the Bible. Certainly, no
reasonable person can believe that the Bible is inerrant.

Biblical Miracles
Roger Hutchinson
In "The Nature of the Claim" (TSR, March/April, 1997), Farrell Till discussed miracles. His
comments tend to confuse the issue, especially with regard to Biblical miracles. It would be
advantageous for future discussion to clarify how one should view miracles.

To do this, we first define a miracle. A miracle is an event that contradicts natural law. In
other words, a miracle does not and cannot occur on its own initiative or under circumstances
that could be explained by natural phenomena. Somebody outside the natural system must
interfere with the laws governing that system in order to cause a miracle. Within a natural
system, many rare, strange, or unique things can happen, which people can label as miracles
even though no outside force has interfered with the system to cause the event. These would
not be miracles, especially not from a Biblical perspective.

Basically, there can be no true miracles in a natural system, since all naturally occurring
events can be explained by natural laws no matter how mysterious they may seem. Within the
Biblical context, miracles occur only if God interferes with natural laws and causes the
miracle to occur. In the broader religious context, a miracle is a supernatural event
precipitated by a supernatural agent.

By defining miracles in this way, a person is not required to believe in either God or miracles.
All that is required is that one recognize the interdependence between a miracle and the
miracle worker.

Now, let's apply this to Till's arguments. In commenting on his debate with Michael Horner,
Till says that Horner "was arguing from `a priori assumption,' because he was assuming that
if a god exists, it is a god who intervenes in human affairs to perform miracles." Till
misunderstood the argument. The "a priori assumption" underlying Horner's argument is not
that God exists or that God interferes in human affairs. The assumption is that the historical
account of God's actions, as recorded in the Bible, is true.

If we make the a priori assumption that the Biblical account is true, then the miracles it
describes require a supernatural agent to cause them. For example, if the account of an ax
head floating on water is true, then someone had to interfere with natural law to cause iron to

Volume 1990 - 2002 Issue


Page 1077 of 2049
Skeptical Review Edited by Farrell Till
float on the water. Consequently, we have Horner drawing the conclusion (not assuming) that
a supernatural agent, God, exists based on the a priori assumption that the Biblical account of
miracles is true.

The reason we must argue in this direction rather than taking Till's approach and first
assuming the existence of God is that we begin with a written record of miracles. Nobody has
seen God, but people have witnessed miracles (e.g., parting of the Red Sea, walls of Jericho,
raising of Lazarus). The parting of the Red Sea was a supernatural activity witnessed by the
writer of Exodus. Assuming that it is a true account, we can conclude that a supernatural
agent exists who caused it. Not surprisingly, this agrees with the Biblical record.

Till adds that Horner assumes that God intervenes in human affairs to perform miracles. No
such assumption is necessary because the Bible clearly states that this is exactly what God
does. So, again, the initial (and a priori) assumption that Horner and other Christians make is
that the Bible provides a true record of historical events. Building on this assumption,
Christian apologists are able to conclude that God exists and that God intervenes in human
affairs.

Till later argues that, "If, however, the claim [made in a historical document] is something
that the reader knows is completely out of the ordinary, he/she rejects it if there is nothing but
the mere word of the writer to support the claim." Is this true? Should we reject an eyewitness
account of a historical event if all we have is the written account of the writer? For example,
we have a historical account of some Jews who said that they saw Jesus alive after He was
crucified. We also have a historical account of some Jews who tell of death camps and gas
chambers in the 1940's. Should we reject these accounts if all we have is the mere word of the
writer and no other corroboration? Till says, "Yes."

Till may justifiably use such a standard with regard to the Bible so long as he consistently
applies that same standard to all other historical documents (and even all issues). However,
that standard does not tell us whether a historical account is actually true or false. It simply
provides a criterion for rejecting the historical account regardless of whether it is true or false.
Certainly, it is possible to accept false accounts as truth by accepting historical accounts at
face value without obtaining corroborating evidence. However, it is also possible to reject true
historical accounts in the absence of corroborating evidence.

Consistent application of this standard could cause Till to reject information that is true just as
it can cause people to accept information that is false. Of the many standards that could be
used to determine the validity of historical documents, this one seems the least useful. It can
lead people to reject claims of the Bible, or other historical documents, without first
determining whether those claims are true. Perhaps Till can devise a more responsible and
accurate standard.

Till further states that Christians are biased because there are many religions in the world each
of which claims its own miracles but Christians reject the miracle claims of all religions save
their own. In comparing Christians and Muslims, Till states that "(t)he reason why each
religion rejects the other's claim is essentially the same as the skeptic's reason for rejecting
both." This is not really true.

Volume 1990 - 2002 Issue


Page 1078 of 2049
Skeptical Review Edited by Farrell Till
Members of one religion reject the claims of all other religions simply because they believe
that there can be only one true God, and He is their god. If the Christian god is the true and
living God, then all other gods must be false. Any claims of miracles attributed to false gods
must also be false. If people believe the miracle claims of a religion, they also must believe in
the existence of that religion's god(s) who caused the miracles. A person who believes that
God is Jesus Christ must conclude that the Muslim "Allah" is a false god, and that the
miracles attributed to "Allah," such as splitting the moon in two, could never have happened.
For the religious person, there can only be one true religion and one true God; all other Gods,
and the miracles attributed to them, must be false.

If the skeptic wants to reject every miracle claim of every religion, he must first be able to
prove that there can be no God and thus no God able to perform miracles. While atheists
arbitrarily reject the notion that God exists, I am not aware that skeptics have devised a proof
that allows them to say that God cannot exist. While there is no definitive proof of God's
existence other than the Biblical accounts and common sense, there is also no definitive proof
that God cannot exist. Thus, the skeptic, while he may require proof of God before he will
accept the existence of God, cannot discount the possibility that God can exist or that God can
cause miracles. The skeptic can reject miracle claims one at a time through scientific research,
but he cannot reject any miracle claims of a religion in the absence of such research.

Skeptics, reasoning like religious folk, could reject the miracle claims of all religions except
one based on the impossibility of there being two Gods. However, the problem for skeptics is
that they cannot determine which god is the real God. Thus, they cannot reject the miracle
claims of any one religion unless they can show that the god(s) of that religion are false. Both
religious people and skeptics can reject miracle claims, but they do so for entirely different
reasons.

Does the skeptic have a basis for disbelieving miracles? Till notes at the beginning of his
article that, "Christian apologists argue that skeptics are unreasonably illogical when they
reject biblical miracle claims." Later, he states, "A skeptic who rejects a biblical miracle claim
is doing nothing more than applying to the Bible the same standards he would apply to... other
documents." What are these "standards," and do skeptics apply them reasonably and logically
with respect to the Bible?

Basically, skeptics reject miracles either because they do not believe direct eyewitness or
secondary accounts of the miracle and/or because the miracle runs counter to personal
knowledge of and experience with scientific laws. In other words, the "standards" that
skeptics apply are nothing more than judgments based on the current base of knowledge.

The issue, then, is whether it is reasonable and/or logical to reject something that one has not
personally witnessed and that cannot be corroborated in some scientific manner. So long as
the skeptic is consistent in rejecting any and all accounts that he has not personally witnessed
or that no one has been able to corroborate through other means, then the rejection of miracles
is reasonable. In doing such, the skeptic may actually reject truth, but that is the price to be
paid for guarding against the acceptance of something that is false.

Volume 1990 - 2002 Issue


Page 1079 of 2049
Skeptical Review Edited by Farrell Till
It is the perception of an inconsistent application of such "standards" by skeptics that leads
Christian apologists to argue that skeptics are unreasonable and illogical in rejecting Biblical
miracle claims. In other words, some people who call themselves skeptics appear to believe
things they want to believe (e. g., macro-evolution) and not things they do not want to believe
(e. g., Biblical miracles) when consistent application of standards would result in both being
rejected. Also, some people like to think that science is able to explain what it observes when
science is usually only able to measure what it observes without the foggiest notion of why it
occurs. Scientists are prone to speculation and hypothesis that many take for fact. So much for
standards.

Till puts it in more favorable terms. He states that "(i)f there are no sound reasons to believe
the claims, the skeptic rejects them regardless of whether they are recorded in the Bible or
some other book." Since there are no sound reasons to reject the idea that God exists, we
should expect those who call themselves skeptics to be either Deists (at a minimum) or liars.

(Roger Hutchinson, 11904 Lafayette Drive, Silver Springs, MD 20902; e-mail,


rhutchin@aol.com)

Inerrantist Tail-Chasing
Farrell Till
One of the main reasons why I renounced biblical fundamentalism was that I could no longer
abide the intellectual embarrassment that it forced upon me. After years of trying to defend
my beliefs with double-talk and hair-splitting of the type that we have just seen in Roger
Hutchinson's article, I finally decided that no position is worth the shameful compromise of
one's intellectual integrity that belief in the biblical inerrancy doctrine requires. In order to
believe in the inerrancy of the Bible, one must convince himself that far-fetched, unlikely
scenarios are legitimate explanations of highly probable contradictions, inconsistencies, and
other discrepancies, and one must believe that there was a time when supernatural events such
as the parting of the Red Sea, the sun standing still in the sky, humans walking unharmed
through a fiery furnace, angels dropping in for visits, the resurrecting of the dead, and such
like happened with routine regularity. As I have said repeatedly in past articles, skeptics who
reject such claims as these are merely being critically consistent in the way that they evaluate
ancient literature. They do not accept such biblical claims for the same reason that they do not
accept similar nonbiblical claims. Supernaturalism is completely contrary to what is observed
today, and so there is no rational reason to believe that supernaturalism occurred in ancient
times, especially not with the frequency that is claimed in the literature of the past.

Now along comes Roger Hutchinson double talking and splitting hairs in a vain attempt to
give a semblance of respectability to the irrational position that biblical miracle claims should
be believed for no other reason than that they are recorded in the Bible. He began by accusing
me of misunderstanding Michael Horner's argument. "The `a priori assumption' underlying

Volume 1990 - 2002 Issue


Page 1080 of 2049
Skeptical Review Edited by Farrell Till
Horner's argument is not that God exists or that God interferes in human affairs," Hutchinson
said. "The assumption is that the historical account of God's actions, as recorded in the Bible,
is true." It was at this point that the double talk and hair- splitting began, for Hutchinson went
on to say, "If we make the a priori assumption that the Biblical account is true, then the
miracles it describes require a supernatural agent to cause them." He then concluded that
Horner wasn't arguing that miracles happened because God exists but that Horner knows that
God exists because miracles happened, and Horner knows that miracles happened because of
his "a priori assumption" that what the Bible says is true.

If Hutchinson were a dog, he would probably entertain himself by chasing his tail, so let's
work our way though his mumbo jumbo by first noticing that he is begging a question he is
obligated to prove. He said that "the historical account of God's actions, as recorded in the
Bible, is true," so he apparently was not content to begin with just one a priori assumption, i.
e., what the Bible says is true, but he tried to sneak another one in too, i e., the Bible is a
"historical account of God's actions." By the same "logic," a Mormon could prove that the
Book of Mormon is a "historical account of God's actions," and so all miracles recorded in it
actually happened, and a Muslim could prove the same thing about the Qur'an. Like most
biblical inerrantists, Roger Hutchinson seems unable to grasp the truth of the logical axiom
that says what proves too much proves nothing at all. I have yet to hear a biblical inerrantist
explain the value of "a priori assumptions" that could be used to prove the historical accuracy
of any and all miracle claims, both ancient and modern. Perhaps Hutchinson would like to
give it a try.

Did I misunderstand Horner's argument? Although Hutchinson wasn't even at the Seattle
debate, he seems qualified to say that I didn't understand Horner's argument, and so I need
him to explain it to me. After all, I was only Horner's opponent in the debate, so what do I
know? Well, for starters, I know that Horner's argument was exactly what I said, i. e., the
existence of God would make miracles possible. As he was concluding his first speech,
Horner said, "As long as it's even possible that God exists, miracles are possible. What one
should do, then, is try to honestly answer the question: What does the evidence suggest is the
most plausible explanation for the data?" (transcript published by Skepticism, Inc., p. 8).
Horner's "data" were simply the New Testament accounts of what had allegedly happened, so
if his argument was what Hutchinson claims, this would have been an ideal place to say, "If
we make the a priori assumption that the Biblical account is true, then the resurrection it
describes required a supernatural agent to cause it; hence, we can know that God existed to
cause the resurrection, because the New Testament account of the resurrection is true, and if
the resurrection account is true, then God must exist." To his credit, Horner was a bit more
sophisticated in his debating style than to chase his tail in this fashion. My advice to
Hutchinson is that he keep his day job, because I don't see much of a career for him in public
debating.

If we make the a priori assumption that the Biblical account is true.... I'd like for
Hutchinson to explain to us why anyone should make such an assumption as this. When I read
a history book, I don't read it with the assumption that everything it says is true, and I
seriously doubt that Hutchinson would accord this special consideration to any other book.
Books, after all, are written by humans, and humans are prone to make mistakes. I recently
acquired a copy of The Histories by Cornelius Tacitus, a 1st-century Roman senator and

Volume 1990 - 2002 Issue


Page 1081 of 2049
Skeptical Review Edited by Farrell Till
historian during the reigns of Vespasian, Titus, and Domitian, the time that most New
Testament critics believe the synoptic gospels were written. I am not reading this book with
the a priori assumption that it is a completely accurate account of 1st-century Roman history.
Why should I? However, if I did read it from that perspective, I would have to believe some
things that no doubt seemed rational to people in the superstitious times in which Tacitus
lived but which would be completely imbecilic for people of enlightened times to believe
really happened.

The book tells of a prophecy that Basilides, a priest of the "local god" on Mount Carmel--yes,
the Mount Carmel of the Bible--made by reading the entrails of an animal that Vespasian
sacrificed. The prophecy was that whatever Vespasian was "planning," it would be granted
him, and he would have a "great mansion, far-flung boundaries and a host of people"
(Translation by Kenneth Wellesley, Penguin Books, 1995, p. 129). Vespasian was, of course,
aspiring at the time to become emperor, and so if one assumes that this work of Tacitus is an
accurate report of 1st-century Roman history, he could prove (by Hutchinson's logic) that this
accurate prophecy made by reading entrails actually happened.

Tacitus also told of a case of teletransportation that involved another Basilides, this one a
"leading Egyptian," who appeared to Vespasian as he was in the "holy house of Serapis" to
"consult the god on matters of state." After this experience, Vespasian asked the priests of
Serapis if Basilides had been in the temple that day and then inquired of others if Basilides
had been seen in the city. Finally, he sent a "party on horse, and ascertained that at the
relevant time [Basilides] had been eighty miles away" (p. 273). Vespasian's conclusion was
that he had experienced a divine vision, whose significance lay "in the meaning of the name
Basilides," which in Greek meant "king's son." Hence, Vespasian was convinced that this was
another omen that he was destined for greatness.

Does Roger Hutchinson believe that such miracles as these actually happened? I seriously
doubt it, but why wouldn't consistency require him to so believe? After all, he accepts as
completely accurate a New Testament story of teletransportation when Jesus allegedly
appeared suddenly out of nowhere to his disciples, who were in a room with closed doors
(Luke 24:36; John 20:19). If Hutchinson can assume the accuracy of this story and thereby
know that God, the causative agent, exists, then why should I not assume the accuracy of
Tacitus' history of Vespasian and thereby know that the god Serapis exists? After all,
reasoning like Hutchinson's should cut both ways. If not, why not?

In John 9:1-7, there is an account of a miracle that Jesus performed by mixing clay with saliva
and anointing the eyes of a man born blind, who was then healed by washing away the clay at
the pool of Siloam. The synoptic gospels tell of another miracle of Jesus in healing a man
with a withered hand (Matt. 12:9-13; Mark 3:1-5; Luke 6:6-10). Hutchinson's "logic" will
lead him to assume that these are accurate accounts of "God's actions," and so he can know
from this assumption that God, the causative agent, exists. However, Tacitus told of two
strikingly similar miracles that emperor Vespasian performed by putting the "water of his
mouth" on the cheeks and eyeballs of a blind man and by treading with his foot on the
withered hand of another man (p. 272). These two miracles were also attested to by the
Roman historian Suetonius, who claimed that the two men had been sent to Vespasian by the
god Serapis (The Twelve Caesars, translation by Robert Graves, Penguin Books, 1989, p.

Volume 1990 - 2002 Issue


Page 1082 of 2049
Skeptical Review Edited by Farrell Till
284). Perhaps Hutchinson can explain to us why he does not assume the accuracy of the
reports by these two Roman historians and thereby conclude that the god Serapis also exists.

There are many more examples of miracle claims in the works of Tacitus and Suetonius that I
could cite, but these are sufficient to make a point that Hutchinson needs to address. He said
that "(n)body has seen God, but people have witnessed miracles" and that "(t)he parting of the
Red Sea was a supernatural activity witnessed by the writer of Exodus." How does
Hutchinson know that the parting of the Red Sea was witnessed by the writer of Exodus?
How else? He assumed it. "Assuming that it is a true account," he said, "we can conclude that
a supernatural agent exists who caused it," and then he added, "Not surprisingly, this agrees
with the Biblical record."

Well, duh, can't Hutchinson see that this same way of reasoning could prove the "truth" of any
religious miracle claims and the existence of the god(s) who caused them to happen? Let's
take Hutchinson's "logic" and apply it to the miracle claims of Tacitus and Suetonius. Nobody
has seen Serapis, but people have witnessed miracles. People witnessed Vespasian's healing
of the blind man and the man with the withered hand. Tacitus, in fact, said that "(b)oth these
incidents are still vouched for by eye-witnesses, though there is now nothing to be gained by
lying" (p. 273). Suetonius claimed that both miracles were done "in the presence of a large
audience" (p. 284). Assuming, then, that these are true accounts, we can conclude that a
supernatural agent exists who caused them. Not surprisingly, this conclusion agrees with the
historical records of Tacitus and Suetonius. Maybe now Hutchinson will have a little more
appreciation for the logical axiom that says what proves too much proves nothing at all, but I
doubt it.

Building on the assumption: There seemed to be no end to Hutchinson's convoluted logic.


He objected to my saying that "Horner assumes that God intervenes in human affairs to
perform miracles," but I have quoted above where Horner said exactly that. Hutchinson
argued that "(n)o such assumption is necessary because the Bible clearly states that this is
exactly what God does," but can't Hutchinson see that this begs the question of biblical
inerrancy? How willing would he be to accept the premise that one can know that the god
Serapis intervenes in human affairs to perform miracles because Tacitus and Suetonius clearly
stated that this is exactly what Serapis does? He would never accept this as a convincing line
of reasoning, yet he apparently said with serious intention that Christian apologists can build
on the assumption that "the Bible provides a true record of historical events" and thereby
"conclude that God exists and that God intervenes in human affairs." Yes, and Roman history
buffs could build on the assumption that Tacitus and Suetonius provided a true record of
historical events and thereby conclude that Serapis exists and that Serapis intervenes in
human affairs. This kind of argumentation may be convincing to Hutchinson, but I suspect he
would be hard pressed to find very many Christian apologists who see any merit in it.

Why not make the opposite assumption? Arguments are no better than the assumptions on
which they rest, so for arguments to be sound, they must be based on assumptions for which
reasonable evidence of their truth can be shown. This is what makes Hutchinson's argument
ludicrously unsound. Of all the millions of books that have ever been written, Hutchinson has
taken only the Bible and given it a special status that he will accord no other book. He has
idolized it with the assumption that everything--not just some things but everything--the Bible

Volume 1990 - 2002 Issue


Page 1083 of 2049
Skeptical Review Edited by Farrell Till
says is true. But what are his grounds for beginning with this assumption? He has none except
for his obvious desire that it be this way. The unlikeliness that his assumption is true is
evident in all of the controversy that surrounds the biblical inerrancy doctrine. Christian
apologists have built an entire industry around publishing books intended to "explain"
thousands of inconsistencies, discrepancies, and contradictions that critics have identified in
the Bible. That some of their resolutions of these discrepancies are correct is undoubtedly
true, but one would have to be hopelessly naive to believe that all of these explanations--
many of them ridiculously far-fetched--are correct. Thus, the evidence against the truth of
Hutchinson's assumption is overwhelming, but, being a biblical inerrantist, he will not let this
deter him in his belief that the Bible is a completely accurate "account of God's actions."

The absurdity of Hutchinson's assumption can be demonstrated by imagining an argument


based on the opposite assumption, i. e., the Bible is errant in everything it says. If someone
should seriously propose this as an assumption by which the Bible should be evaluated, he
would accuse him/her of total unobjectivity, but he sees no unobjectivity in assuming the
opposite, that the Bible is inerrant in everything it says. He might argue that the absurdity of
assuming complete biblical errancy can be demonstrated by the obvious fact that the Bible
contains information whose correctness can be verified. In this, I believe he would be correct,
but if someone didn't mind appearing ridiculous, he could use inerrantist apologetic methods
and explain away every verifiable fact recorded in the Bible just as "effectively" as
inerrantists resolve biblical discrepancies.

Space won't permit me to apply inerrantist methods to established facts in the Bible to show
how easily this could be done, so I will just modify the assumption. Let's suppose that
someone should seriously argue that all information in the Bible that cannot be verified is
erroneous. We cannot verify that Adam, Noah, Abraham, Moses, etc. were real people;
therefore, the Bible is wrong in saying that they once existed. We cannot verify that Judas
betrayed Jesus for 30 pieces of silver; therefore, the Bible is wrong in saying that this event
happened. We assume that everything recorded in the Bible that cannot be verified as truth is
erroneous. If we did this, Hutchinson would accuse us of an incredibly radical prejudice, yet
he expects us to consider him the epitome of objectivity as he assumes that everything in the
Bible is inerrant, an assumption that is every bit as unreasonable as its opposite.

If the Christian god is the true and living God, then all other gods must be false. Ah, yes,
but there's the rub. A conclusion is only as sound as the assumptions on which it is based, and
the simple act of assuming that the Christian god is the true and living God doesn't make it so.
No doubt Hutchinson is thoroughly convinced that his god is the true and living God, but he is
no more convinced of this than a Muslim is that his god is the true god. Hutchinson boasted
that the skeptic "cannot discount the possibility that God can exist or that God can cause
miracles," but a Hindu could just as validly argue that Hutchinson cannot discount the
possibility that Vishnu can exist or that Vishnu can cause miracles. So what is the value in an
argument that can be used to prove just about anything or everything? Rather than chasing his
tail as Hutchinson did throughout his article, his time would have been better spent if he had
offered some convincing evidence that his god does exist and that he inspired the writing of
the Bible. He didn't do that, though, because he knows that no such evidence exists. Thus, he
spent his time demanding, in effect, that skeptics prove that his god does not exist and that the
Bible is not inerrant.

Volume 1990 - 2002 Issue


Page 1084 of 2049
Skeptical Review Edited by Farrell Till
The price we pay. Even Hutchinson seemed to recognize the strength of the skeptical
approach to evaluating miracle claims, because he admitted at the end of his article that it is a
"reasonable" method. He thinks, however, that the method comes with a price. By rejecting
miracles because they "cannot be corroborated in some scientific manner," he said, "the
skeptic may actually reject truth, but that is the price to be paid for guarding against the
acceptance of something that is false." It has probably never occurred to Hutchinson that by
accepting unverifiable fabulous claims just because they are found in a particular book, he has
probably accepted falsehood. That, of course, is the price to be paid for excessive gullibility,
and if I have to choose between the two, I would much rather err on the side of caution than
gullibility. Needless to say, I believe that when I reject claims about talking animals, dead
people returning to life, people once living for centuries, and such like, the chance that I am
rejecting truth is so minimal that it doesn't even deserve consideration.

The current base of knowledge. A favorite inerrantist defense of biblical miracles is that one
cannot assume that miracles did not happen in the past because they don't happen now, and so
at the end of a discourse that proved absolutely nothing, Hutchinson finally retreated to this
sanctuary of last resort. "(T)he `standards' that skeptics apply are nothing more than
judgments based on the current base of knowledge," he said, but what better standard can one
use in evaluating historical claims, especially fabulous claims, than the "current base of
knowledge"? Does he have any evidence at all that the past was not like the present in terms
of what can and cannot happen? If so, he should present it. If not, he should recognize that
this is a poor defense of biblical miracle claims.

"Macro-Evolution": Evolution falls outside the scope of the publishing purpose of TSR, so I
will not comment on Hutchinson's citing of "macro-evolution" as an example of skeptics
"believ[ing] what they want to believe," except to say that he can only wish that he had a
fraction of the evidence for miracles that science has for what he calls "macro-evolution." If
he had anywhere near that much evidence for biblical miracles, this discussion would never
even have started, because I would be a strong advocate of biblical miracles.

He did, however, show a flagrant inconsistency by bringing this up on the heels of his
disparaging comment about the "current base of knowledge" that skeptics rely on, for if
miracles should not be evaluated by our "current base of knowledge," then evolution shouldn't
either. Creationists, however, will invariably attack evolution on the grounds that life cannot
come from nonlife, which is an argument that the past should be evaluated by our "current
base of knowledge." His attack on evolution is therefore as inconsistent as his defense of
biblical inerrancy. "Liar," then, is a label better suited for his side.

On the Matter of Proof


Dave Matson

Volume 1990 - 2002 Issue


Page 1085 of 2049
Skeptical Review Edited by Farrell Till
Despite numerous articles on the subject, courtesy of Till and me, creationists still submit
essays demonstrating a profound misunderstanding of the reasoning process. How many times
do we have to repeat ourselves?

In particular, Dr. James D. Price begins his article with this statement: "It is true that a person
who defends the truth of a debated proposition must present convincing evidence to support
its truth. But the person who denies the truth of the proposition has the burden to convincingly
disprove the proposition-- that is the real burden of proof. The proof of a proposition is in the
failure to prove the denial.... Why should the burden of proof be one-sided?"

The first portion of Dr. Price's statement is technically correct, but neither Till nor any other
skeptic need take that position. We are simply saying that we won't buy his goods unless he
convinces us of their value. We are not saying that we know beyond a shadow of a doubt that
his goods are rotten. The burden of proof is clearly on his shoulders.

The second portion of his statement is, of course, absurd. A proposition is not proven by a
failure to disprove it! It is not my job to prove that giant, green spiders are not hiding out in
San Diego! The great, green-spider conspiracy does not become acceptable public knowledge
just because I can't disprove it! Wake up and smell the coffee!

Thus, all that Till needs to do, and has done, is to show that Dr. James Price's thesis is far
from compelling. Strong disproof is hardly necessary. The reason that the burden of proof is
one-sided is that reliable public knowledge must be safeguarded against nonsense. If we are to
weed out the giant-green-spider theories, then we must insist upon positive, compelling
evidence--and not the lack of strong disproof--as the criteria. Otherwise, public knowledge
becomes a total joke where anything goes.

Dr. James Price refers to the matter of extraordinary proof as an"alleged" law of evidence.
Whether it is a formal law or not is of no interest to me; it is, however, absolutely necessary
for the accumulation of reliable knowledge. It is based on the concept of accepting the
probable over the improbable, which makes good sense if we are trying to collect sound
knowledge on any subject.

Dr. Price asks how extraordinary a proof must be before it is accepted. In matters of science,
we must ask whether it is more probable that a thoroughly tested law, which has successfully
explained a great deal of nature, has failed or whether claims of its failure, which can often be
traced to lies, confusion, distorted third-hand information, etc., are unsound. In that case, the
proof must be so extraordinary that it is easier to believe that the law of science has failed
than that a lie, confusion or faulty report has been involved. Clearly, that means the kind of
scientific documentation that is available for all to see, one that forcefully rules out the
alternatives.

Historical claims involving miracles, in the full supernatural sense of the word, are claims that
scientific law has failed. Generally, the circumstances involved in those claims are beyond
investigation, making extraordinary proof impossible. Rather than saying that such claims are
clearly false, the skeptic need only note that scientific law is far more believable. That is, we
reject the miracle because it is very likely a false or distorted claim. Accumulating sound

Volume 1990 - 2002 Issue


Page 1086 of 2049
Skeptical Review Edited by Farrell Till
public knowledge means rejecting claims that, in principle, might be right. We must accept
the probable over the very improbable, and that's the bottom line.

Consequently, the skeptic will continue to demand that extraordinary claims be accompanied
by extraordinary proof. There is no alternative if we seek to accumulate sound knowledge.

The data collected by Robert Ripley does not generally, if ever, run contrary to scientific law.
Consequently, the standards of reasonable proof are much lower. Do strange oddities occur in
nature? Absolutely! Thus, the real question is whether Ripley accurately recorded his data.
Since many of his claims are backed by numerous photographs, records or even direct
physical evidence, enough to fill a museum and make it interesting, we are well within
reasonable bounds to accept most of it without further proof. The "bias" that Dr. Price lays
against the principle of extraordinary proof is revealed as nothing more than good sense.

Miraculous prediction, as in biblical prophecy, does not necessarily violate scientific law, but
it does involve the acquiring of knowledge in a manner that has never been verified. Nor has
science given us a theoretically sound framework to explain how such prophecy might
reasonably occur. To the extent that mediums have been competently investigated, in the past
as well as the present, the claim of miraculous prophecy has always proven false or
unconvincing. Thus, we have neither the theoretical groundwork nor a single, proven example
of miraculous prophecy.

We must ask which is more probable, that a few individuals in the Mideast had special
powers during biblical times, men associated with a particular religious cult, or that the usual,
faulty claims, lies, misunderstandings and later manipulations of history have occurred.
Clearly, the odds are heavily in favor of the latter. Any proof of miraculous prophecy must be
so strong, so extraordinary, as to make the latter the less likely case. Not only is such proof
lacking, but in many cases we have good evidence that biblical "prophecies" were written
down after the event in question! There is also good evidence that many of the events and
characteristics attributed to Jesus have been lifted from the Old Testament by New Testament
authors! Thus, it is hardly surprising that there are some parallels between the Old Testament
and the Jesus story.

Dr. James Price attempts to give the principle of extraordinary proof this strange twist: "To
those who believe in an omniscient God, the idea of fulfilled prophecy attributed to Him is
not a fabulous claim, but what one would reasonably expect. To them an instance of such
fulfilled prophecy requires only ordinary, not extraordinary, evidence as sufficient reason to
believe it."

The principle of extraordinary proof has nothing to do with what some individual or group
believes! Believers in the great, green-spider conspiracy may find claims that giant, green
spiders have been spotted in San Diego to be the expected, ordinary thing. However, they are
not exempt from the need to present extraordinary proof. Similarly, just because Dr. Price
belongs to a group that views the Bible as God-breathed does not exempt him from the need
to present extraordinary proof for miraculous prophecy.

Volume 1990 - 2002 Issue


Page 1087 of 2049
Skeptical Review Edited by Farrell Till
The criteria that determines whether an argument needs extraordinary proof or not is that old
question: "Which is more probable, given actual track records and the results of careful
investigation?" The less probable an event, compared to the alternatives, the more
extraordinary the required proof. No unreasonable presupposition is involved here, only a
straightforward application of good reasoning.

Arguments involving supernatural miracles, whether in the Bible or elsewhere, require


extraordinary proof. Miraculous prophecy is in a similar position. In both cases the burden of
the proof is squarely on the shoulders of the believer. To win his or her case, the skeptic need
only show that the arguments in its favor are not compelling. The skeptic does not have to
disprove the claims. The skeptic's claim is that supernatural miracles and miraculous
prophecy should presently be rejected as extremely unlikely, not that they have been proven
false beyond any possible doubt.

(Dave Matson, editor, The Oak Hill Free Press, P.O. Box 61274, Pasadena, CA 91116; e-
mail 103514.3640@compuserve.com)

Two More Criteria of Valid Prophecy


Fulfillment
Richard S. Russell
Farrell Till has been having a good time eating biblical literalists for lunch on the issue of
prophecy fulfillment. For all of that, though, I think he's been too easy on them. He's listed 4
tests which a "fulfilled" prophecy needs to meet. I think he should add 2 more to the list.

Let me lead up to the 1st of these with a story. It was told to me as true, but it sure has all the
earmarks of an urban legend. Regardless, it doesn't need to be true in order to illustrate the
point I'm trying to make. Say that you're a relatively well-to-do professional person. One day
you receive a plain envelope in the mail. In it is a single sheet of paper bearing only 2
sentences: "The closest game in the NFL this week is Cincinnati vs. Tampa Bay. The Bengals
will win." "Huh!" you think to yourself as you throw the paper into the recycling bin. But, out
of curiosity, you check the sports scores Sunday night and, sure enough, the Bengals did win.

Same thing next week, as the mystery letter names the Vikings to beat the Lions, and they do.
The 3rd week's letter carries another prediction, but this time there's an added paragraph:
"You have probably figured out that I'm working on a system for predicting winners of sports
contests, and I'm sure you understand the implications of being able to do so consistently. It is
in your interest as well as mine to keep these letters confidential."

The letters continue to arrive every week. In week 5, it's pointed out that you are free to place
bets based on the predictions.

Volume 1990 - 2002 Issue


Page 1088 of 2049
Skeptical Review Edited by Farrell Till
In week 7, the letter predicts that the Raiders will beat the Chargers. As it turns out, the game
is tied at the end of regulation time, and the Chargers win on a field goal in overtime. The
Week 8 letter acknowledges that the system isn't perfect but goes on to correctly call the
winner of that week's game.

Eventually an explanation arrives: "Five years ago I was a college student who lived for only
two things: sports and computers. I found that the best way to combine these passions was by
playing computer-simulation football games, but, as a computer engineer, I was sure I could
write better simulation routines than the commercial software then on the market, so I started
doing my own programming. After I graduated, I got a job with a company that has its own
supercomputer, and I've been using it after hours for the tremendous amount of number-
crunching needed to run my very elaborate simulation program. I've been testing and
improving it for the last three years, and I think it's up to the task of calling winners of even
the tightest football games over 90% of the time.

"My problem is that my use of the supercomputer has technically been illegal, and if my
employer ever found out about it, I'd lose my job. I'm looking for financial backing so that I
can legitimately rent the computer time I need, and I'm coming to you because, to be blunt,
you have lots more money than I do. I'm sure you'll understand when I say I'd like to remain
anonymous."

The letter goes on to propose that you front the computer geek the money needed to go legit,
and in return the geek will keep providing you with predictions. You have some reservations,
but there's that fabulous 90% track record to go on, so you come up with the requested
amount of money.

That's the last you ever hear of the geek, who actually turns out to have been a con artist.
When eventually you swallow your pride and contact the cops, you find out exactly what the
scam was. All that was necessary was a list of 1024 names of people as well off as you. The
first week, 512 of them got a letter just like yours, predicting the Bengals would win; the
other 512 got a letter predicting the exact opposite, that the Buccaneers would win. Needless
to say, in week 2 it was only the people in the 1st half of the original list who got the next
letter, and 256 of them were told, like you, that the Vikings would win, while the other 256
letters named the Lions. Keep dividing by 2, and you drop to 128, 64, 32, 16, and 8 suckers in
successive weeks. The con artist could string the entire 8 along for an extra week on the
strength of the overtime game.

Eventually, though, the list is whittled down to a couple of people who will be absolutely
convinced that this is the fabled "sure thing"--so convinced that they will fork over thousands
of dollars. The con artist, meanwhile, has invested about 2000 letters and stamps in the scam,
so any return over about a thousand bucks represents a profit.

In fact, the so-called "fabulous track record" reflects nothing more than the workings of sheer
chance. If 1024 people independently flip a coin 10 times, the expectation--the norm, the
typical result-- is that one of them will get 10 straight heads. And a good statistician (or a
good con artist) can tell you that this happens about once in a thousand tries--no big deal,
certainly not a great rarity. But, if you don't understand the math behind it, it seems like a big

Volume 1990 - 2002 Issue


Page 1089 of 2049
Skeptical Review Edited by Farrell Till
deal. Guess which one of the coin-flippers will be talking about it at the office for the next
week. (Actually, there'll probably be another one as well, the 1 out of 1024 who got 10
straight tails.)

Same story with prophecy fulfillment. It could well be that there were a thousand "prophets"
making predictions about how long the Chosen People would remain in captivity in Babylon.
Some said 5 years, some said 100, some said forever. Which one gets remembered? The one
who came closest.

So I propose Rule 5, which is designed to eliminate the lucky guess: ALL of the prophet's
predictions, not just a few of them, must be shown to be true. After all, if infallible inspiration
from God is the source of this wisdom, why should there be any exceptions to the ability to
foretell the future? Anything less than perfection could otherwise be explainable by sheer
chance.

Moving along, consider the opinion of most linguists that almost every sentence you speak or
write is unique. Neither you nor anyone else has ever spoken that exact combination of words
before or is likely to do so again. That's because there are over 800,000 words in the English
language and a bazillion different ways of combining them. And yet I'd be willing to lay even
money that everyone reading this article has, at one time or another, uttered the sentence, "To
be or not to be; that is the question." Why? Because this is the beginning of the most famous
soliloquy in the most famous play in English literature. The words come from a script, people
are expected to recite them verbatim every time, and they do. Plays, quotations, and rituals are
all examples of exceptions to the "unique sentence" generalization.

This leads us to Rule 6, which can be stated as a metaphor: The "fulfillment" cannot be a
cake. You know how to bake a cake, right? Start with a recipe, an exact list of ingredients and
procedures, and do exactly what it tells you to do. Voila! A cake!

Want a messiah? Start with a recipe, a script, and follow all the steps. Voila! A savior!

Now, I'd be willing to believe that Nebuchadnezzar had no particular reason to go out of his
way to fulfill a prophecy of some obscure Hebrew fanatic that he'd probably never even heard
of. But how about Jesus and his merry band? They certainly knew what the messianic
prophecies were and what it would take to fulfill them. All they had to do to appear messianic
was to follow the script.

Of course, not just anyone was likely to do this, especially given the prophecies involving
suffering and death, but we've certainly seen plenty of recent examples of people in the
Middle East willing to commit suicide for their favorite cause, so there's every reason to think
that their counterparts existed 2000 years ago as well.

So does it count as prophecy fulfillment if someone says, "The messiah will do so-and-so,"
and a couple of centuries later someone comes along, wanting to be thought of as that very
messiah, and says, "By golly, then, I'd better do so-and-so"? Obviously not. Unless, of course,
you believe without questioning that the Lord works in mysterious ways.

Volume 1990 - 2002 Issue


Page 1090 of 2049
Skeptical Review Edited by Farrell Till
(Richard S. Russell, 2642 Kendall Avenue, Madison, WI 53705-3736; e-mail
RSRMadison@aol.com)

Which Bible?
Steven Carr
When somebody says that the Bible is inerrant, a good reply is to ask "Which Bible?" The
first task of translators of the Bible into English is to decide which verses they want to put
into each of the 27 New Testament books. There are more than 5,000 Greek manuscripts, but
not one has the same collection of verses as any popular English Bible. Indeed, there is no
Greek manuscript before A. D. 800 which has 27 books in its New Testament. The Codex
Sinaiticus, from about A. D. 350, comes closest, but it also contains the Epistle of Barnabas
and the Shepherd of Hermas.

How did this textual variety come about, and what significance does it have?

The King James Version was produced on the basis of relatively few late manuscripts. Since
then, many early manuscripts have been found. The impression is often given that these early
manuscripts help us get back to what the authors originally wrote. Instead, they reveal that we
can only guess what was originally written and that what was written was changed within
decades, often for purely doctrinal reasons.

Let me give just a few examples, from many which could have been chosen. Take Luke
22:43-44, "And being in an agony, he prayed more earnestly, and his sweat became like great
drops of blood falling down upon the ground." This is not present in the very earliest Greek
manuscripts p66 and p75 from the third century. (P66 stands for papyrus number 66, p75 for
papyrus number 75). Will they be dropped from future printings of the King James Version
because they are not in the earliest manuscripts we have? Somehow, I doubt it.

These verses are also omitted by Codex Alexandrinus, Codex Vaticanus (4th century), Codex
Washingtonensis (5th century), etc., but are in Sinaiticus (4th century) and the great majority
of later manuscripts. They are cited by the early church father Justin (c. A. D. 130). Whenever
these verses were added or dropped, it must have been very early.

We know that these verses were quoted, not always exactly, in the second century by the early
church fathers to counter the heretical view that Jesus was not a real human being and only
quoted for that doctrinal purpose.

For example, Justin said, "(I)t is recorded that His sweat fell down like drops of blood while
He was praying... in order that we may perceive that the Father wished His Son really to
undergo such sufferings for our sakes, and may not say that He, being the Son of God, did not
feel what was happening to Him and inflicted on Him" (Dialogue with Trypho, 103).

Volume 1990 - 2002 Issue


Page 1091 of 2049
Skeptical Review Edited by Farrell Till
Irenaeus (c. A. D. 170) wrote, "(N)or would He have wept over Lazarus, nor have sweated
great drops of blood; nor have declared, `My soul is exceeding sorrowful'; nor, when His side
was pierced, would there have come forth blood and water. For all these are tokens of the
flesh which had been derived from the earth, which He had recapitulated in Himself, bearing
salvation to His own handiwork" (Against Heresy, 3, 22, 2).

Hippolytus (c. A. D 190) said, "Thus then, too, though demonstrated as God, He does not
refuse the conditions proper to Him as man, since He hungers and toils and thirsts in
weariness, and flees in fear, and prays in trouble. And He who as God has a sleepless nature,
slumbers on a pillow. And He who for this end came into the world, begs off from the cup of
suffering. And in an agony He sweats blood, and is strengthened by an angel..." (Against
Noetus, 18).

These verses were cited only for doctrinal purposes and are missing from the earliest
manuscripts of Luke's Gospel. What more proof is needed that Luke's Gospel was altered no
more than decades after being written and that this was done for doctrinal reasons?

The biggest doctrinal issue in Christianity is whether Jesus is God and what the Trinity
means. The earliest Christians held all kinds of views about the status of Jesus. Even as late as
A. D. 200, the majority of Christians did not believe in the Trinity, as Tertullian freely
conceded: "The simple, indeed, who always constitute the majority of believers, are startled at
the dispensation (of the Three in One), on the ground that their very rule of faith withdraws
them from the world's plurality of gods to the one only true God; not understanding that,
although He is the one only God, He must yet be believed in with His own `economy'"
(Against Praxeas, Chapter 3).

These sorts of disputes led to many alterations of the New Testament. At the time Tertullian
wrote, many Christians believed that Jesus was God the Father. Noetus based this belief on
John 14:9, "Whoever has seen me has seen the Father." The orthodox Christian Hippolytus
had to go through contortions to show that this does not mean that Jesus was the Father. Is it
any surprise that just a few years later, one of the very earliest manuscripts of John's Gospel
we have was altered to read, "Whoever has seen me has seen the Father also"? A very useful
change for doctrinal reasons.

Of course, people who said that Jesus was God the Father were also altering manuscripts. The
earliest manuscript of 1 and 2 Peter is called p72. It states, in 1 Peter 5:1, that Peter was a
witness of the sufferings of God, not a witness of the sufferings of Christ. This is not an
accidental slip. Second Peter 1:2 in English Bibles today says, "May grace and peace be
multiplied to you in the knowledge of God and of Jesus our Lord." P72 removes the "and" so
that it becomes "God, our Lord Jesus." P72 also alters Jude 5 to say that the person who saved
the people out of the land of Egypt was "the God Christ." Christians nowadays draw back
from the idea that God the Son and not God the Father led the Israelites out of Egypt.

The earliest manuscript of 1 and 2 Peter and of Jude is distinctly heretical, which is why
English translations often give its readings as mere footnotes, and later readings are put in the
main text.

Volume 1990 - 2002 Issue


Page 1092 of 2049
Skeptical Review Edited by Farrell Till
Another 3rd century manuscript, which differs from what is selected to go into Bibles today,
is called p46. It includes Hebrews 1:8, which today says about Jesus, "Your throne, O God, is
forever and ever and the righteous sceptre is the sceptre of your kingdom." P46 says about
Jesus, "God is your throne forever and ever; the righteous sceptre is the sceptre of his
kingdom." The earliest manuscript of Hebrews that we have says that the kingdom is God's,
not Jesus's. This reading is also backed up by the earliest Great Codices, Sinaiticus and
Vaticanus. Naturally, English Bibles prefer later manuscripts.

The evidence of the earliest manuscripts is that Christianity was split into many factions.
Orthodox views did not win out until the fourth century or later. Until then, people wrote and
rewrote the New Testament books, trying to put the correct spin on the texts. This happened
extremely early. By about A. D. 110, Polycarp was quoting, in his letter to the Philippians, a
version of Acts called the "Western" version, which is about 10 percent longer than the other
version.

If the New Testament books really were inspired by an omnipotent God, why was he unable
to prevent their being changed within decades of being written, by people acting in his name?

(Steven Carr, Flat C, 28 Keighley Road, Bradford BD8, England; e-mail,


steven@bowness.demon.co.uk; web page, http://www.bowness.demon.co .uk/

Serpentine Logic
Dan Barker
On May 14 of this year I participated in a debate in Sheboygan, Wisconsin, on the topic of
God and the Bible: Fact or Fable? My opponent was Dr. Walter Kaiser, a well-known
evangelical scholar, professor of Old Testament, who is now President of Gordon-Conwell
Theological Seminary in Massachusetts.

Just to be certain what we were debating, I looked up the word fable in the dictionary. I was
somewhat surprised to learn that the primary definition involves talking animals. Webster's
New Universal Unabridged Dictionary gives "fable, n. 1. a fictitious narrative intended to
teach some moral truth or precept, in which animals and sometimes inanimate objects are
represented as speakers and actors." The other dictionaries agree.

Not all stories with talking animals are fables (only moral tales), and not all fables contain
talking animals (so even without them, the Bible could still be a fable); but there is an
assumption in the dictionary definitions that conforms to common knowledge: animals do not
speak human language. If there is any evidence that the Bible is a fable, or contains fables,
then the presence of talking animals would certainly be a part of such evidence.

Volume 1990 - 2002 Issue


Page 1093 of 2049
Skeptical Review Edited by Farrell Till
I mentioned these facts in my opening statement, and then I read from Genesis 3, which
relates the most crucial moral tale of the entire Bible, the story of the fall of the human race:
"Now the serpent was more subtil [NRSV: `crafty'] than any beast of the field which the Lord
God had made. And he said unto the woman, Yea, hath God said, Ye shall not eat of every
tree of the garden" (Gen. 3:1)?

This story of the temptation of Eve, right at the beginning of Scripture, is the cornerstone of
Jewish, Christian, and Moslem theology. Without it, there is no need for salvation, Moses,
Jesus, or Mohammed. It is clearly a "moral tale," and it contains a talking animal. I could have
claimed victory and sat down, but having been a preacher, it's hard for me to quit, especially
when I still have time on the clock. So I did what most preachers do: I kept talking. I opened
the Bible and read about an ass that didn't know when to stop talking: "And Balaam said unto
the ass, Because thou hast mocked me: I would there were a sword in mine hand, for now
would I kill thee. And the ass said unto Balaam, Am not I thine ass, upon which thou hast
ridden ever since I was thine unto this day? was I ever wont to do so unto thee" (Num. 22: 29-
30)?

This story may not be as morally crucial as the Garden of Eden, but that makes it no more
believable.

The Bible describes more talking animals: "...four beasts full of eyes before and behind. And
the first beast was like a lion, and the second beast like a calf, and the third beast had a face as
a man, and the fourth beast was like a flying eagle. And the four beasts had each of them six
wings about him; and they were full of eyes within; and they rest not day and night, saying,
Holy, holy, holy, Lord God Almighty" (Rev.4:6-8).

Most Christians think Satan is a spiritual creature, but the Bible vividly describes him as a
talking animal: "Behold a great red dragon, having seven heads and ten horns, and seven
crowns upon his heads. And his tail drew the third part of the stars of heaven, and did cast
them to the earth.... And the great dragon was cast out, that old serpent, called the Devil, and
Satan, which deceiveth the whole world: he was cast out into the earth, and his angels were
cast out with him.... And he opened his mouth in blasphemy against God, to blaspheme his
name" (Rev. 12:3,4,9; 13:6).

The Bible contains moral tales that involve speaking animals. Therefore, the Bible is a fable.

After the opening statements, Kaiser and I engaged in a period of cross- examination, which
was timed by the moderator. During my opening statement, I had said: "Christians are afraid
people will think for themselves. Atheists are afraid they won't."

Kaiser: You say that most Christians are afraid of human thought. I would not have thought
that. Some of the best thinkers we have had in the whole of this--since the A.D. times-- have
been Christian thinkers.

Barker: Who believe in talking snakes.

Kaiser: Who believe in giving the evidence its first due.

Volume 1990 - 2002 Issue


Page 1094 of 2049
Skeptical Review Edited by Farrell Till
Barker: Do you believe there was a talking snake?

Audience member: Yes.

Barker: I'm asking this guy. He's... do you believe a snake spoke human language?

Kaiser: I don't think you understand what Genesis 3 is talking about. I'm surprised at your
understanding .

Barker: But just tell me: did a snake speak human language? Yes or no?

Kaiser: It said "cha nachash," the "serpent" spoke, yes. Uh, huh. Which is a whole lot
different than what you're trying to put the construction on it.

Barker: But, what is a "serpent"?

Kaiser: Uh--that's a translation of "cha nachash." Do you know Hebrew?

Barker: I know a little. I don't know that word.

Kaiser: Uh, huh. Well, I thought so, because the whole point there is, I think that's a title
being given to the Evil One, and I don't think that it in any way, in the text itself, indicates
that it was a reptile.

Barker: So this was the Devil, Satan talking?

Kaiser: I think it is.

Barker: You mean the red dragon with the seven heads and the ten horns and all that? That's
who was talking in the garden?

Kaiser: No. First of all, stick to the text here, in Genesis, what it claims. Don't mish-mash and
go from one to the other and therefore confuse it.

Barker: Don't take it in context?

Kaiser: If we're really wanting to know, we've got to stick to the context, sure.

Barker: So, what was it? What was that thing that was talking to Eve? It says the beast--"The
serpent was more subtil than any other beast of the field which the Lord God had made." He's
comparing--in context, he's--

Kaiser: In the English translation, it was there- -"cha nachash" was more subtle, "mi kol"--
"from all"--and you're taking it in a partitive sense, "from any of the beasts of the field," which
could also, and I think more accurately be translated, "than the beasts of the field." It's a
comparative translation there.

Volume 1990 - 2002 Issue


Page 1095 of 2049
Skeptical Review Edited by Farrell Till
Barker: Okay, is the God--

Kaiser: He's talking about all of God's creation, and putting it over against that one of his
other creatures, the Devil.

Barker: If you're right about this--I don't think you are, but if you are--does the god you
believe in, is he capable of causing an animal to speak human language? Can he do that? Did
Balaam's ass speak human language to Balaam?

Kaiser: Uh, you're asking me what God is capable of, and God is capable of everything
except contradicting himself.

Barker: So, he could cause an animal--he could cause Balaam's ass to speak to Balaam. Do
you believe that a donkey spoke human language?

Kaiser: God can't make ropes with one end.

Barker: But I'm asking about talking donkeys. Can God cause a donkey to speak?

Kaiser: We have--I just wrote an article on an archaeological find of that particular episode,
from the 9th century B. C., with Balaam, son of Beor, and it's from a schoolboy copy's text,
and yes, they talk about the fact that this was a most unusual event in which God spoke
through the mouth of an animal.

Barker: So did the animal speak human language, yes or no?

Kaiser: You heard me, sure. Yes.

Barker: Okay, so you believe. So, you believe in the dictionary definition of "fable," then,
right? A story which has talking animals is a fable.

Kaiser: Now you are confusing two things, because in the definition of a fable, you do have
a[n] animal speaking, but that's not your only criteria for what constitutes a fable. Fables are a
part of figure of speech. We use figures of speech: "You hit the ceiling," but I don't look for
you on the ceiling. You do various kinds of things that are in terms of figures of speech, and
you must allow that also to be operative in ancient language as well as modern language.

Barker: So, the Bible contains a lot of allegory with pretending speaking animals, which, to
me--that's a fable. Animals who are pretending to speak, or that you are- -

Kaiser: Those were your words: "lot of allegory." Now you're putting a value judgment on
that, and your value judgment is clouding your ability to do rational thought. I thought you
were a rationalist. Now you're becoming an emotionalist. (laughs)

Barker: This morning, when I was driving up here, I stopped to get gas. I went in to get a
coffee break, and as I was going into the little shop--

Volume 1990 - 2002 Issue


Page 1096 of 2049
Skeptical Review Edited by Farrell Till
Kaiser: Yeah.

Barker:...this cat came up to me and spoke to me in human language, and said, "Stay away
from the tuna sandwiches." Do you believe that? Do you believe that happened to me?

Kaiser: I don't know your cats that well. (laughing)

Barker: But do you believe that that happened to me?

Kaiser: Uh, I have my doubts about it.

Barker: Why?

Kaiser: I need more evidence.

Barker: Why would you doubt that a cat could speak?

Kaiser: Because, we need evidence; that's why.

Moderator: Time.

Barker: Ah! Well, we have no evidence for the truth of the Bible.

Even Kaiser, who takes the Bible at face value, acts like a normal human being when it comes
to stories of talking animals outside of his own religion. He knows that animals do not speak.
Or at least he knows enough to be highly suspicious and to demand strong evidence for such
an unusual claim. But what is the difference between the biblical talking snake and my story
of a talking cat? Neither of them can boast any evidence outside of the tale itself. Without
proof, they are both fables.

Kaiser's contradiction is pretty obvious. The only reason Christian scholars believe that the
Hebrew-speaking serpent was Satan is because two New Testament writers thought so. There
is nothing in Genesis, or its immediate context, to suggest that the nachash was anything but a
snake. Kaiser accused me of "mish-mash" when, at his prompting, I tried to connect Genesis 3
with the "red dragon" of Revelation 12:3; yet he supposedly is not interpreting it out of
context when he does the same thing with the "Devil" of Revelation 12:9. He wants to have
his cake and eat it too. (Devil's Food.)

After the debate, I investigated Kaiser's claims about nachash. I read Genesis 3 in 11 different
English versions--including the KJV, a Jewish translation from the Masoretic text, the NRSV,
as well as the less-than- scholarly, though popular, NIV and Living Bible-- and none of them
call nachash the Devil: they all use "serpent" or "snake." I also looked up all the other places
in the Hebrew Scriptures where nachash appears, and it is always rendered "serpent" or
"snake," an animal that "biteth the horse heels," that moves "upon a rock," that eats dust and
moves "out of their holes like worms of the earth."

Volume 1990 - 2002 Issue


Page 1097 of 2049
Skeptical Review Edited by Farrell Till
Nachash is frequently paired poetically with "adder" in such verses as, "Dan shall be a serpent
by the way, an adder in the path" (Gen. 49:17), "Their poison is like the poison of a serpent:
they are like the deaf adder that stoppeth her ear" (Psalm 58:4), "They have sharpened their
tongues like a serpent; adder's poison is under their lips" (Psalm 140:3), "At the last it biteth
like a serpent, and stingeth like an adder" (Proverbs 23:32).

I am not an expert in Hebrew, but this cursory examination appears to prove Kaiser wrong.
For a scholarly opinion, I wrote and asked Dr. Hector Avalos for some help. Avalos, who is a
professor in the Religious Studies program at Iowa State University, has a Ph.D. in biblical
languages from Harvard, and is the author of "Animals" and "Satan" in the Oxford
Companion to the Bible (New York: Oxford, 1993). He was gracious enough to prepare a
helpful paper on this topic, in the form of a letter, which I have excerpted below. (I have
eliminated or transliterated Avalos's Hebrew characters, which I can't reproduce here, and I
have embedded his footnotes in square brackets.)

Dear Dan,

If [Kaiser's] contention is that nachash in Genesis 3:1 should not be regarded as an


animal because of the use of a "partitive" construction, then he would be in error, and
in opposition to about 99% of all biblical scholars (conservative, liberal, or secular)
and translations of which I am aware. [There is one respected scholar, A. W. Sjoberg,
who argues that nachash in Genesis 3:1 is really a chameleon, not a serpent, but that
would still make it "an animal."]
I would also like to know what sort of creature the nachash is according to Kaiser. If it
is not "an animal," then does he believe it is some sort of creature like a cherub or
seraph? How is he defining "an animal"? It is true that nachash can describe
creatures that we would call mythological. For example, Isaiah 27:1, "On that day the
LORD with his cruel and great and strong sword will punish Leviathan the fleeing
serpent, Leviathan the twisting serpent, and he will kill the dragon that is in the sea."
[All English quotes, unless noted otherwise, are from the NRSV.]
However, this is not the case in Genesis 3. Aside from its ability to speak, nachash in
that chapter is an animal, in the sense of nonhuman creature which normally inhabits
the earth and would be readily recognizable. Besides Genesis 3:1, we have the
following statement in the same chapter.
Genesis 3:14, "The LORD God said to the serpent, Because you have done this,
cursed are you among all animals and among all wild creatures; upon your belly you
shall go, and dust you shall eat all the days of your life.
Genesis 3:15, I will put enmity between you and the woman, and between your
offspring and hers; he will strike your head, and you will strike his heel. This verse
indicates that the nachash of Genesis 3 is a creature that 1) is expected to interact with
human beings on a continuing basis, and 2) crawls on its belly. Thus, the nachash here
is not ostensibly a supernatural creature, but one which we ought to be able to
recognize as a normal animal according to the author. A serpent is consistent with
this description of the nachash, and a serpent is an animal in our normal sense. The
same Hebrew word is used for a creature that "bites" people in Numbers 21:5ff.

Volume 1990 - 2002 Issue


Page 1098 of 2049
Skeptical Review Edited by Farrell Till
One objection that might be made is that the characteristics of the nachash changed
after it was cursed, but then that would only pose new problems for conservative
Christians (e.g., was there a change in species--evolution?).
Other biblical authors also apparently understand nachash in Genesis 3 to be a
serpent in our normal sense. For example, 2 Corinthians 11:3, "But I am afraid that
as the serpent deceived Eve by its cunning, your thoughts will be led astray from a
sincere and pure devotion to Christ." The author here uses the Greek ophis, the
normal word for snake or serpent (though it can be used for a creature we would call
mythological as in Revelation 12:15). The Hebrew speaking Jews that translated
Genesis 3:1 into Greek in the version known as the Septuagint also used ophis.
The occurrence of a so-called partitive construction in Genesis 3:1 cannot be used to
conclude that nachash is not an animal or "beast of the field." To understand this, let's
review the basic grammar of the relevant part of the verse (using my literalistic
English translations).

mkol (min + kol): "all" + "from" or "than"

arum: "crafty" (adjective)


hayah: "was" (verb)
cha nachash: "(and) the serpent" (noun + waw)
cha sadeh: "the field" (noun + def. article)
chaiyah: "beast(s) of" (noun in construct)

[NRSV: "wild animals." I am using the generic plural "beasts" though other
translations may use the more proper singular. "Noun in construct" refers to the
particular form that a Hebrew noun takes when used to express a genitive relationship
with a following noun, and usually translated as "X of Y" in a more schematic form.]

Here the basis for seeing a partitive construction is the use of "from all" ("than any
other" in the NRSV). Gesenius' Hebrew textbook does regard the preposition ["from"
or "than"] in Genesis 3:1 as part of a partitive expression. [W. Gesenius, E. Kautszch,
and A. E. Cowley, Genesius' Hebrew Grammar (2nd English Edition: Oxford:
Clarendon, 1910) paragraph 119 v(1).]
A partitive construction is one that expresses separation, but one must be specific
about the nature of the separation. Genesis 3:1 should be seen as describing a
separation regarding a particular feature of the serpent within the category of nouns
which follows "from all" in the construct phrase. The nachash is separated from all the
"beasts of the field" as to a particular feature expressed by the adjective "crafty," but
nachash is not separated from "beasts of the field" as a whole. The serpent is still one
of the "beasts of the field" which follow "from all."
This is a complicated way of saying that it is best to see "from all" in Genesis 3:1 as
part of a comparative or superlative expression as do Bruce Waltke and M. O'Connor
in their An Introduction to Hebrew Syntax [Paragraph 14d. 44, Winona Lake,
Indiana: Eisenbrauns, 1990, p. 270.] In such cases, the subject of comparison is
included in the category of nouns which follows "from all." For example, in 1 Samuel
10:23, this is used to state that Saul is "the tallest of his people." Just as the subject
(speaking of Saul) in the comparative expression, "He was the tallest of his people"

Volume 1990 - 2002 Issue


Page 1099 of 2049
Skeptical Review Edited by Farrell Till
would not mean that Saul is not a person or part of "his people," so too the subject in
the comparative expression "the serpent was the craftiest of all the beasts of the field"
does not mean that the serpent is not one of the beasts of the field.
Consider also 1 Kings 10:23: "Thus Solomon excelled all the kings of the earth in
riches and in wisdom." The verse clearly means that Solomon was the richest and
wisest of all the kings of the earth (separate insofar as riches and wisdom are
concerned), and not that Solomon was not a king at all. Solomon is still included in
the class of things (i.e., "kings of the earth") which follow "from all" as a construct
phrase.
Similarly, in Genesis 3:1 the serpent belongs to the class of things (beasts of the field)
which follow "from all" in a construct phrase, but it is only in a particular feature
expressed by the adjective ("crafty") that it is in any sense distinct.
Conclusion: Dr. Kaiser may know some Hebrew, but, as does any scholar, he needs to
show good empirical arguments and examples for his conclusion. Did Dr. Kaiser
adduce any examples elsewhere in the Hebrew Bible where the subject in such a
"partitive construction" cannot be included in the general category of nouns following
"than all" in a construct phrase? Based on analogies with similar Hebrew
constructions and the Greek translations made by Hebrew speakers themselves, it is
best to see reference to the craftiness of the nachash in Genesis 3:1 as part of a
comparative expressions which would still include that creature among "the beasts of
the field." These beasts of the field may be properly described as "animals" in our
normal sense. But even if one describes this passage in question as a partitive
construction, such a construction does not warrant the exclusion of the nachash from
general category, "the beasts of the field," that follows "from all."
And even if the nachash is something "other than an animal," it would still mean that
the biblical authors, if interpreted literally, believe in creatures that have
anthropomorphic features and abilities that are not scientifically verified; or believe
in creatures that bear anthropomorphic features and abilities ascribed to animals in
stories that Christians might otherwise call fables or myths. There is, of course,
nothing to indicate that the author of Genesis is equating nachash with the Devil as
described in the New Testament (see Revelation 20:2 for that equation by Christian
authors).
I hope that this helps unravel the convoluted grammatical arguments that some
Evangelicals like to use.

In May, I sent Kaiser a copy of Avalos's wonderful analysis, asking if he would like to
clarify his comments. I also mentioned that it appears from his "figures of speech"
remarks that he actually supports my claim that the Bible contains allegory, such as
talking animals, that were not meant to be taken literally. And even if the talking
serpent in the Garden can be imagined as a spiritual "Evil One" rather than a physical
animal, there is no way around the fact (as Kaiser admits) that Balaam's talking ass
was an animal.

I am still waiting for an answer from Kaiser. (Dan Barker, P. O. Box 750, Madison,
WI 53701; e-mail, dbarker@mailbag.com)

Volume 1990 - 2002 Issue


Page 1100 of 2049
Skeptical Review Edited by Farrell Till
EDITOR'S NOTE: Dan Barker is the author of Losing Faith In Faith: From
Preacher To Atheist, and is on the staff of the Freedom From Religion Foundation.
This debate was later broadcast on local television. For a videotape of the event, send
a $20.00 check to "TV 8" at TV8-WSCS, Attn: Kerry Kautzer, 1 University Drive,
Sheboygan WI 53081.)

From the Mailbag


A Guiding Light in the Darkness...

I am enclosing a check for $12, $6 for the past year's subscription and $6 for the upcoming
year. Why am I paying for something that was clearly stated as free, you ask? Because the
work of enlightening mankind is worthy of financial support as well as words of
encouragement. Farrell, I appreciate the work you and your staff do by explaining the truth
about religious claims, which most of us in this country have grown up believing to some
degree or another to be somewhat true, somehow. The question floating around in my
generation seemed to be, "What do I feel and why do I feel it?" The idea being, I suppose, to
get in touch with our feelings, the I'm OK, you're OK feeling. Well, feeling good is good, but
is what I feel good about true? Ah, therein was the problem. I didn't know what was true and
what was, well, a lie. I had to ask myself a more important question, "What do I know and
how do I know it?"

This being the information age, I turned to the Internet, typed in Christianity, religion,
atheism, and my thinking light instantly came on and has stayed on ever since. I discovered
The Skeptical Review, requested a subscription and was soon engrossed in article after
article explaining the ins and outs of many Biblical claims and how they misrepresent truth
and why. Although I have expanded my search for truth, your journal fills a much needed
void. The fear of knowing need not be a fear of learning. It is a challenge to all "believers" to
seek the guiding light of truth through honest inquiry, not through the blindness of faith. Truth
is truly a guiding light in the darkness and that is the meaning and value of enlightenment.
May that light forever shine bright.

(Gloria Nyquest, 10500 Highway 10 East, Missoula, MT 59802; e-mail,


nyquest@Montana.com)

EDITOR'S NOTE: For two years now, I have been expressing my belief that the internet will
bring about dramatic changes in religion, because the internet ended the ability of the
religious establishment to control information. Religious leaders and advocates can no longer
suppress information by just not making anything detrimental to common beliefs available in
churches and public and school libraries. Now anyone, almost anywhere in the world, who
can use a keyboard can access information that even a decade ago could have been screened
from the public. Ms. Nyquest's letter is just one of many that we have received from people
who discovered TSR by browsing the internet. Not a week goes by that we don't receive

Volume 1990 - 2002 Issue


Page 1101 of 2049
Skeptical Review Edited by Farrell Till
subscription requests from people who discovered our web site. I want to express my
appreciation to the Internet Infidels who have made the secular web available to publications
like The Skeptical Review.

Jewish Atheism...

Thank you for sending me The Skeptical Review. I'm an atheist who was raised on the Bible
until the age of 24. I was born in Israel, and I'm an ex-Jewish Israeli since coming to the USA
in 1979. As a child, you are never able to doubt the information that you are given in school.
The seeds of doubts start when you are a little older and wiser and realize that some of the
stuff they fed you is part of brainwashing. I thank my fortune that I was able by myself to sort
it out and come to the conclusion that you don't have to believe everything grown-ups tell
you. As a Jew (ex- Jew), I still am fascinated by the Bible as far as history is concerned, plus
other things in it. That is why I'm happy to read your magazine. It helps me to understand a
lot of things that I never before was skeptical about. My eyes are now more wide open!!!
Thank you again.

I'd like to mention that I read the book by Dennis McKinsey The Encyclopedia of Biblical
errancy. It's most illuminating and educational. Half of it is dedicated to the New Testament,
which as an ex-Jew I'm not familiar with, so it sounds strange to me. I do prefer to read about
the Old Testament, especially your article about "prophecies." It was educational for me.
Enclosed you will find a check for back issues and The Jackson-Till and Geisler- Till
debates.

I belong to Atheists United, which I joined in 1995, and I'd like to mention that I have met a
lot of American Jewish atheists. Quite interesting!

(Jesse Illous, 15240 Kittridge Street, Apt. 222, Van Nuys, CA 91405)

EDITOR'S NOTE: The Encyclopedia of Biblical Errancy that Mr. Illous referred to was
written by Dennis McKinsey, the editor of Biblical Errancy, and published by Prometheus
Books. Information about the book can be obtained from McKinsey at 2500 Punderson Drive,
Hilliard, OH 43026; e-mail, klomckin@ infinet.com.

An Inerrantist's view of Miracles...

Dave Matson defines a miracle, in his letter to TSR, as a "violation of scientific law." The
issue of miracles is a major--if not the major--area of contention between the believer and the
non-believer, the inerrantist and the skeptic errantist.

According to Dave, a violation of scientific law can be termed a "miracle," yet where I
completely disagree with Dave's comments is that a "miracle" does not, at all, have to violate
scientific law to be considered a "miracle."

Volume 1990 - 2002 Issue


Page 1102 of 2049
Skeptical Review Edited by Farrell Till
A miracle is based on perception--not necessarily in reality. What is a miracle to one may not
be to another. It is, in the broad sense of the term, something which is beyond our own
personal understanding and/or acceptance. The ignorant will find much more "miraculous"
than the learned.

If I took my cellular phone back to ancient Greece and began making calls (assuming
someone else went back with me with a phone), there would, no doubt, be many Greeks
totally dumbfounded by what they were seeing. Surely many would consider this contraption
a "miracle." And if they found out they could fly, or found out that light could be used for
healing, or found out that we have visited the moon, they would continue to be amazed by
these "miracles." To us, these things may be incredible but not "miracles"--but to the
Greeks...?

Failure to understand can be the progeny of a "miracle," as can ignorance. Only an omniscient
being, should it exist, would consider nothing a "miracle."

No, Dave, a miracle is not a "violation of scientific law." One area where we do agree is that
scientific laws, as far as we know, cannot be broken--so, if this is your definition of a
"miracle," then I do agree with you--there are no miracles. The real question lies in whether or
not we are aware of all the "scientific laws" we may be subject to.

Dave, as many skeptics do, is judging events in the Bible as being implausible based on the
assumption that he has absolute knowledge of all scientific laws. As a result, anything beyond
his/ our comprehension, or "outside" of that understanding, is unknowable, and therefore
impossible. He labels these "miracles" and dismisses them and, as a result, dismisses their
"source," in its entirety.

But the real truth is that Dave, like all of us, may not know all there is to know concerning our
existence and that there may be other scientific laws still undiscovered (unless, of course, he
is that omniscient being I spoke of above). This suggests that there may be rational
explanations for seemingly "impossible" occurrences lurking out there waiting for science to
discover them. And there are countless examples of this occurring throughout history.

I agree with Farrell's comments on Dave's letter. It is a good letter, logically presented. But it
starts out with a false and mistaken premise, and we all know logical arguments can be built
on false premises.

Dave, in the second sentence of his letter, asks the question: "Is not it (a miracle), in its fullest
sense, a violation of scientific laws?" To that, I answer, "No, it is not. Scientific laws, to my
knowledge, cannot be violated."

(Dave Court, 25 Mitchem Drive, Etobicoke, ON, Canada M8W 258; e-mail,
hoover1@netcom.ca)

MATSON'S REPLY: You have brought up a good point! Your definition of a miracle is
certainly a practical one, a miracle being an event that is totally beyond the understanding of

Volume 1990 - 2002 Issue


Page 1103 of 2049
Skeptical Review Edited by Farrell Till
the observer to the point of awe. As you pointed out, such a miracle need not violate scientific
law. A computer would certainly strike the ancient Greek mathematicians as a miracle.

The central core of my reasoning, however, remains unchanged. The rational mind is still
obliged to choose the probable explanation over the improbable. The thrust of your argument
is as follows: "Dave, as many skeptics do, is judging events in the Bible as being implausible
based on the assumption that he has absolute knowledge of all scientific laws. As a result,
anything beyond his/our comprehension, or `outside' of that understanding, is unknowable,
and therefore impossible; he labels these `miracles' and dismisses them and, as a result,
dismisses their `source,' in its entirety."

The flaw in your argument is that I don't have to assume that my knowledge of scientific law
is complete or correct. Therefore, it does not follow that I am claiming that anything outside
of such laws is impossible. All I know is that "law X" has been tested rigorously over the
years by numerous, trained individuals, under various circumstances and extreme conditions,
and has always proven true within a certain range of the variables under consideration. Now,
it's theoretically possible that one more test under unusual circumstances might overthrow
those data in favor of a miracle. But, which is more likely true, a well documented scientific
law (having survived rigorous testing over the years) or a third-hand claim that such a law is
in error, a claim whose circumstances are lost forever to us?

My reason for rejecting biblical miracles, in my last letter and now, is not that they are a
priori impossible. It is that they lack the kind of documentation that is so necessary to
compete with scientific laws. In this case, a miracle (to the modern mind), your definition or
mine, boils down to a claim that our understanding of a well tested principle(s) is wrong. That
is an extraordinary claim given the degree to which that principle(s) has been tested and found
to be true. On one hand, we have our understanding of a principle that is backed by a huge
amount of research and observation. On the other hand, unless one makes unjustified a priori
assumptions about the Bible, we have what can only be described as totally undocumented
claims of miracles. The rational choice is obvious, especially given that similar miracle claims
invariably accompany most religions.

Extraordinary claims always require extraordinary evidence. If you wish to argue that a well
established view of science is wrong, in favor of a miracle, then you need some awesome
evidence! A few biblical miracles lend themselves to plausible scientific explanations, but
most, such as the claim that a metal axe head can be made to float by a magical spell, run
squarely against our understanding of scientific principles. You are free, of course, to believe
in the undocumented miracle at the expense of the well established scientific principle, on the
idea that the miracle might be true, but don't fool yourself by thinking that you have made a
rational choice. The rational mind notes the total lack of any serious documentation for the
miracles of the Bible and compares that with the heavily tested and documented principles of
science. Under those circumstances, it rejects the conflicting biblical miracles as an inferior
hypothesis. We reject the inferior hypothesis as improbable; we do not say that it is
impossible.

Therefore, we reject Bible miracles (where they run counter to well tested principles) as
improbable scenarios.

Volume 1990 - 2002 Issue


Page 1104 of 2049
Skeptical Review Edited by Farrell Till

The Importance of Inerrancy...

After reading Dr. James Price's article in the May/June issue of The Skeptical Review, I felt
inspired to write a letter for the "Mailbag." A key sentence in the doctor's article is, "The
inerrancy of the Bible depends on the truth of fulfilled prophecy, but fulfilled prophecy does
not depend on the Bible being inerrant in every detail; it depends only on the truth of the
relevant details of the prophecy."

What a mouthful. And what a cop-out!

In other words, Dr. Price, prophecy is the only aspect of the Bible that matters? In the
controversy over divine inspiration and inerrancy, or lack thereof, all the contradictions,
cruelties, incongruities, plagiarisms, mythological transferences, absurdities, and obvious
untruths in the Bible are insignificant. The only criterion that counts is whether a prophecy
came true. That offsets all of the unholy attributes of the Bible and its deity.

Let's consider the first one third of Dr. Price's mouthful: "The inerrancy of the Bible depends
on the truth of fulfilled prophecy...." Dr. Price cannot accuse me of taking this out of context,
because it is a separate point and can be written as a free-standing sentence. Saying biblical
inerrancy is dependent on fulfilled prophecy is analogous to saying only the debits in a
bookkeeper's ledger amount to any importance, but the credits have no value. What about the
Bible's recording of past events? Shouldn't a Bible author recounting the story of Adam and
Eve be just as accurate in looking back at that past event and putting the information into the
sacred page as a prophet is in writing about the future? The author of the events concerning
the creation and fall of man was not a spectator at those events, just as Jeremiah was not
around at the time his prophecies were to be fulfilled. The Bible logically must be as reliant
on "reverse prophecy" for its inerrancy as it is upon correct predictions of the future.
Therefore, Price's claim that "the inerrancy of the Bible depends on the truth of fulfilled
prophecy" is incomplete and fallacious. Furthermore, by emphasizing prophecy so strongly,
Dr. Price deemphasizes not only the retelling of past events but other elements of the "word of
God" as well, such as family values, social ethics, morality, promise-keeping, historical
accuracy, and harmony of content in the books of The Book.

Now for the second part of the key mouthful: "(F)ulfilled prophecy does not depend on the
Bible being inerrant in every detail." To the skeptic, I'm afraid it does. After learning of the
many failed prophecies and contradictions in the Bible, the skeptic cannot acknowledge a
"fulfilled" prognostication as anything but a coincidence or a product of editing ex post facto.

My final question for Dr. Price regarding this middle third: "Does `in every detail' encompass
other prophecies besides the one in Jeremiah 25:11?" If yes, Dr. Price implicitly admits that
some prophecies can be errant, can be unfulfilled, without affecting the validity of other
prophecies. In other words, the totality of prophecy is true even if details of particular
prophecies never come to pass. This is illogical to the max. If divine, the Bible is
automatically a work of harmony.

Volume 1990 - 2002 Issue


Page 1105 of 2049
Skeptical Review Edited by Farrell Till
The last section of Dr. Price's mouthful says prophecy fulfillment "depends only on the truth
of the relevant details of a given prophecy." Restated and reduced to a simpler state, this
example of convoluted logic redundantly says that a fulfilled prophecy depends on the truth of
itself. We can also fairly ask, "Well, then, who determines what details are relevant?"

I get the impression that Dr. Price deems a detail relevant if he finds historical correlation to
support it or another passage of the Bible that harmonizes with it, but discards a detail as
irrelevant if he finds no supporting evidence. Dr. Price seems to "pick and choose" his pet
prophecy details, just as Christians pick and choose their own pet verses and commands of
God by selecting those that comfortably fit into their lifestyle and socio-economic niche and
conveniently ignoring those that they find uncomfortable or that would involve sacrifice or
change.

In the initial paragraph of "Solving the Jeremiah Problem," we discover another of many
weaknesses in Dr. Price's argument. This weakness reads, "Further, the fulfillment of the
prophecy is documented by contemporary extrabiblical historic records, the validity of which
is denied by none."

I'm afraid this is simply not true. Three well known authorities on the Bible who do deny
validity come to mind. These authorities give different durations for the Babylonian captivity
than the duration Dr. Price so stubbornly insists upon, and the fact that one of these authorities
is a Bible apologist certainly doesn't help resurrect Price's sagging argument. Two of these
authorities were atheists, so such a balance in biases certainly puts a kink in Price's hose.

The first of these is Isaac Asimov, who was an expert in many things, including the Bible. In
1967, Asimov wrote, "(T)he period of exile was not seventy years. From the destruction of the
temple in 586 B. C. to Cyrus's proclamation in 538 B. C. was a lapse of only 48 years.

"Of course, Jeremiah and the chronicler may not have thought of seventy years as
representing a precise length of time.... Seventy years may merely have meant the `lifetime of
a man' to them.

"On the other hand, the seventy years that were accomplished at Babylon may refer to the
duration of the Chaldean Empire, which from the accession of Nebuchadnezzar to that of
Cyrus lasted sixty-seven years" (Guide to the Bible, Volume 1, The Old Testament).

As if one were needed, here's another blow to Dr. Price's insistence on a literal fulfillment of
seventy years, by eminent Bible scholar Robin Lane Fox: "In 597 B. C., the king of Judah was
sent east as a royal captive to Babylon. Some of his subjects went with him, and ten years
later, in 587, the city of Jerusalem fell to a second invasion by Babylonian troops"(The
Unauthorized Version: Truth and Fiction in the Bible).

The third authority who is at variance with Dr. Price's opinion of a 70-year exile is Werner
Keller, author of The Bible as History. This work, which is most definitely pro-Bible, lists
the years of the Babylonian exile as 586 to 538 B. C., concurring with Dr. Asimov and falling
short of Dr. Price by 22 years.

Volume 1990 - 2002 Issue


Page 1106 of 2049
Skeptical Review Edited by Farrell Till
(Christopher Edsall, 555 North Council Road, Apt. F, Oklahoma City, OK 73127.)

EDITOR'S NOTE: Mr. Edsall points out a problem that I have tried unsuccessfully to get Dr.
Price to see. He cannot claim amazing prophecy fulfillment when the event alleged to be the
fulfillment is fraught with as much controversy as this one is. If even those who are Bible
believers cannot agree that the Judeans spent 70 years in Babylonian captivity, how can Dr.
Price reasonably expect rational people to accept this as a genuine case of prophecy
fulfillment? Dr. Price obviously has a bias that makes him predisposed to accept everything
claimed in the Bible, but rational people cannot and will not do this.

The Most Effective Weapon...

I read with interest the letter, "The Emperor's Clothes," by Kelly Cunningham in the
July/August 1997 issue of TSR. He complains that "Biblical errancy efforts seem somewhat
like criticizing the clothes the emperor is no longer wearing. The issue isn't just whether this
verse or that is flawed; I find scholarly textual analysis of the Bible is miles beyond this."
Well, Mr. Cunningham is certainly right on all counts, and I am in full agreement with his
points challenging belief in Bible inerrancy. However, he either ignores or is unaware of an
important fact of life. Let me try to make my point with the following example. Last year I
attended the annual conferences of both the American Humanist Association and the Freedom
From Religion Foundation. Each meeting was well attended by their standards. There were
between one hundred fifty and two hundred attendees at each. However, at the recent annual
convention of the Southern Baptist Association, there were around fourteen thousand
attendees. As freethinkers we must never forget that we remain a feared and hated minority
awash in a sea of Bible-believing Christians.

Yes, the Bible was thoroughly discredited long ago, as Mr. Cunningham points out, and it's all
there in the literature for anyone to read. But, except for some Bible scholars and a few
isolated freethinkers like us, who bothers? Most pulpit audiences remain blissfully ignorant of
these works. Their preachers or priests may be vaguely aware of them, but they are not about
to reveal it. If you asked the average churchgoers who Burton L. Mack is, they would
probably identify him as a wide receiver for the Dallas Cowboys.

We freethinkers may be small in number; we certainly do not have access to huge sums of
money; we lack political clout, but we do have one important thing on our side--the facts.
Therefore, we must continue to drive the point of biblical errancy home again and again and,
if necessary, again. To the scholar and to the informed freethinker such repetition of the
obvious may be downright boring, as Mr. Cunningham suggests, but it remains the most
effective weapon in our limited arsenal.

(Louis W. Cable, 102 Spyglass Dr., Lufkin, TX 75901; e-mail, lwcable@inu. net)

EDITOR'S NOTE: I am repeatedly asked, "Why do you do this [oppose biblical


inerrantists]?" My answer has always been the same. History has shown that when religion
controls government, enlightenment and freedom suffer. If biblicists are not opposed by those
who know the absurdity of their claims, they will continue to hoodwink political bodies into

Volume 1990 - 2002 Issue


Page 1107 of 2049
Skeptical Review Edited by Farrell Till
giving them concessions and a favored status that will erode personal rights and freedoms. If
people who know the absurdity of their position won't oppose them, who will?

To Roger Hutchinson...

Whether or not god "visits the iniquity of the fathers upon the children unto the third and
fourth generation" just doesn't matter, because that lousy, vengeful, murderous god of the
bible is a creation of primitive, vulgar, cruel people who thought the earth was flat.

Atheism is not a religion. An atheist is free of any gods, demons, heavens, hells, or dogmas of
any kind. No manmade superstitions! We love our brains and our reasoning power. Voltaire
was one of us, up to and including his dying day. (Attention, Mr. Casao.) The lies made up
after his death, that he adored Christ and repented, are another self-serving attempt by
Christians to convince us that no one could really be free of religion.

To god, a really bad sin is not murder, torture, incest, wife-beating, theft, or corruption. It is
worshiping some other god. Oh, puleeez! Blasphemy is an unpardonable sin as well, as you
point out, Mr. Hutchinson. I say to you that you can be a "sinner" if you want to, but I am a
fine human being, and it is thanks to my character, not some bloody sacrifice.

(Dorothy B. Thompson, P. O. Box 562, Bandon, OR 97411.)

EDITOR'S NOTE: Ms. Thompson made a point that I wonder about every time I read the
Bible, especially the Old Testament. I wonder why it took me so long to see that the Bible is
obviously the work of superstitious people living in primitive times when gods were thought
to be active in human affairs. Once one is able to see this, he can't read the Bible without
thinking, "How stupid I was!"

The Historical Background...

After reading "Keep them barefooted and pregnant" Sept/Oct 1997, I find that you are not
knowledgeable in the historical background as to why "women are to keep silent in the
churches."

The part where women are to "ask their husbands at home" was said because in church, men
sat on one side of the building and women sat on the other side. Whenever a woman did not
understand what the minister was saying, she would ask her husband, who was sitting on the
other side of the sanctuary, what did the minister mean. So it was disruptive to others. This is
the historical background of this.

Woman [sic] are permitted to teach and encouraged to do so. Acts 2:17-18 indicates that "sons
and daughters shall prophecy [sic]." The word "prophecy" [sic] means "to preach" or
"foretell." Phillip [sic] the evangelist in the book of Acts had 4 daughters who were
prophetess[es] (Women preachers).

Volume 1990 - 2002 Issue


Page 1108 of 2049
Skeptical Review Edited by Farrell Till
Woman [sic] are not given a bad rap in the Bible as you would have your readers to believe. It
was Adam's fault that Eve fell into temptation. He was given charge over his family. The man
is to oversee the affairs of the family to make sure they are in order, since woman [sic] are to
be treated as a weaker vessel. Yet, they are one in the sight of God as husband and wife in
marriage (and not shacking up either).

You must really be reaching for straws to make the Bible look bad.

(SuprDave@lightspeed.net)

EDITOR'S NOTE: When "SuprDave's" letter arrived by e-mail, I sent a reply asking him for
his mailing address so that I could publish it with his letter. He refused to send it, and so I am
printing his letter with only his e-mail address. In refusing to send his mailing address,
"Suprdave" said, "I'm really not interested in submitting my name and address since it will
probably result in [a] stampede." If I had written this letter, I don't think I would want my
name attached to it either.

He arbitrarily declared a "historical background" for Paul's commandment that women keep
silent in the churches and apparently expects us to accept it, even though he gave no
supporting evidence. Only a cursory examination of the text is sufficient to see that
"SuprDave's" explanation is unlikely. In the passage in question, Paul said, "As in all the
churches of the saints, let the women keep silence in the churches," so this was obviously a
commandment that Paul expected all churches to observe and not just some where there may
have been a custom of women sitting on one side and men on the other. Furthermore,
"SuprDave's" explanation is even more sexist than Paul's probable meaning, because it
supposes that women would be so intellectually inferior to men that they would have to
constantly ask their husbands what the preacher meant. The scenario that this explanation
presents is also too ridiculous to deserve serious comment, because it supposes situations,
where wives would be calling across the room to say, "Hey, Stephanas, what did the preacher
mean by that?" What we have in "Suprdave's" historical explanation is just another absurd
effort to circumvent an obvious textual embarrassment in the Bible. It's no wonder he didn't
want his name published with this letter.

It was so nice of "SuprDave" to put some of the blame for the "fall" on Adam, but that does
nothing to remove the fact that the Bible is obviously sexist in its teachings. "SuprDave," for
example, said nothing about Paul's statement to Timothy, "Let a woman learn in quietness
with all subjection. But I permit not a woman to teach, nor to have dominion over a man, but
to be in quietness" (1 Tim. 2:11- 12). It's hard to find in this only a scenario in which Paul was
trying to regulate women who were speaking to their husbands across the room during
sermons. It's an obvious command for women to shut up and be in subjection to men.

"SuprDave" is certainly right about other biblical passages that refer to women prophesying,
but these are all references in non-Pauline books. The only thing they would prove is that
there was disagreement between Luke (in this case) and Paul. The former seemed to think that
women could sometimes assume teaching duties in the church; the other thought that they
should keep quiet and stay in their places. This is just another case of discrepancy in the
biblical text, but biblicists like "SuprDave" would never admit it. They apparently want us to

Volume 1990 - 2002 Issue


Page 1109 of 2049
Skeptical Review Edited by Farrell Till
believe that their omniscient, omnipotent deity was so stupid that he couldn't inspire clear
writing, and so he has to depend on the "SuprDaves" of the world to explain what this
omnimax deity really meant.

More Possibilities to Consider...

I just read Farrell's article on the supposed martyrdom of the apostles where he explained that
not only the martyrdom but the very existence of many of the apostles is highly questionable.
Farrell concluded, "No doubt many Christians who read this article will lay it aside and
continue like parrots to ask the same question: `Why would the apostles have died for
something they knew was false?'"

There are a few possibilities/realities that were overlooked: (1) Many people have died for
many false beliefs. This includes people from various religions and cults--the Mormon
founder Joseph Smith, for one. (2) There is no proof that the "apostles" were in fact 1st-
generation witnesses of the ministry of Jesus. Except for Paul who was admittedly not an
apostolic witness to Jesus, and whose very conversion story has inconsistencies, it is difficult
to ascertain whether or not the gospels and epistles attributed to the various "apostles" were in
fact written by the people whose names they carry. The gospel writers never identified
themselves. Jesus, important as he is supposed to be, never wrote anything down, and the
gospel stories about his ministry contain many contradictions. There is no proof that the
people "spreading the good news" in fact knew Jesus personally. Most of the Jewish prophets
gave their names to their prophetic books and identified themselves by name, and still the
authorship of their final forms is questionable. This cannot even be said about any of the
gospels, whose authorship is highly questionable. The "apostles" themselves may have been
nothing more than duped believers. (3) Even if the apostles did know Jesus and were in fact in
love with his persona and charisma and were loyal to him, they might well have been willing
to risk their freedom and lives for him, or for the claims about him, especially if they were
making their living off the stories about him. (4) The apostles, once they were arrested, had
very little say in the matter. They were at the mercy of their executioners. They didn't "die for
what they believed"; rather, they were murdered, and it seems unlikely that once they were in
the sadistic hands of their murderers they had any say in the matter.

(Yoel Wasserman, P. O. Box 998, Aspen, CO 81612; e-mail, core@rof.net)

Unable to Believe in "the Book..."

You never cease to amaze me with your knowledge of the Bible and your ability to refute
religionists in such an in-depth and polite manner. I grew up in a Christian family and later
became an extreme Bible-thumper. After praying and reading the "Good Book" every day, a
funny thing happened: I started to know the Bible. I started to realize how savage everyone in
the Old Testament was, so I avoided reading it. It tested my faith too much. Then I noticed the
contradictions in the New Testament (specifically the gospels), but I couldn't very well read
the Bible every day and avoid the Old Testament and the New Testament.

Volume 1990 - 2002 Issue


Page 1110 of 2049
Skeptical Review Edited by Farrell Till
After a great deal of "soul-searching," I finally had to be honest and admit to myself that I just
don't believe. I've had many doubts about that decision, but I have to be as honest as I can. I
can't believe in that book.

Your publication has helped so much in keeping me on the straight and narrow. I can't thank
you enough. Enclosed is $12 for two more years.

(J. C. Douglas, P. O. Box 5546, Ocala, FL 34478-5546.)

EDITOR'S NOTE: Mr. Douglas's experience was similar to mine. I first became aware of
glaring inconsistencies in the Bible by studying parallel accounts in the synoptic gospels, but
once my mind was opened to the probability that errors were in the Bible, I was able to see
the absurdity of claiming that God's ways are higher than our ways as a defense of the many
Old Testament atrocities. These stories soon bothered me so much that I went through a
period when I refused to read the Old Testament, but that did nothing to help my condition. In
reading the New Testament, I continued to see problems there too. The journey toward an
enlightened view of the Bible begins with the initial recognition that it just may contain
errors.

Volume 1990 - 2002 Issue


Page 1111 of 2049
Skeptical Review Edited by Farrell Till

The Skeptical Review


Volume Nine – 1998
Farrell Till, Editor

• January/Feburary Volume Nine, Issue One


• March/April Volume Nine, Issue Two
• May/June Volume Nine, Issue Three
• July/August Volume Nine, Issue Four
• September/October Volume Nine, Iss Fiveue
• November/December Volume Nine, Issue Six

Volume 1990 - 2002 Issue


Page 1112 of 2049
Skeptical Review Edited by Farrell Till

Skeptical Review
Volume Nine, Number One
January/February 1998
Farrell Till, editor

• The Biblical Way to Treat Women with Dignity


As noted in previous articles, a popular claim of Christianity is that the Bible has
accorded women a status superior to that in societies dominated by other religions.
Those who make this claim are either very ignorant of what the Bible teaches or else
have no scruples against misrepresenting facts to try to further the cause of
Christianity.
• Did the Judge of All the Earth Always Do What Was Right?
The killing of David's son for his parents' act of adultery was a clear violation of the
principle in Deuteronomy 24:16.
• Sins of the Fathers and other Matters
Roger Hutchinson responds to Farrell Till's "The Problem of False Analogy".
• Is Roger Hutchinson for Real?
The principle of hermeneutics that says scripture should be allowed to interpret
scripture is fundamentally unsound.
• Reply to Hutchinson
A reply to Hutchinson's article, "Sins of the Fathers: Another View" by Sol Abrams.
• The Man with No Heart: Miracles and Evidence
The party who makes a positive assertion in court has the responsibility of offering
sufficient evidence for his assertion. If he fails to do so, his assertion is not proven and
will be disregarded. This is the "burden of proof." What is "sufficient" evidence? That
depends on the assertion and its importance in the case.
• How Much Can Historical Documents Prove?
Biblical criticism has raised a fundamental problem for the Christian believer. If the
Bible is examined as a historical document, then the central question arises whether
one can ascertain what, if anything, in the Bible contains revealed religious
knowledge.

Volume 1990 - 2002 Issue


Page 1113 of 2049
Skeptical Review Edited by Farrell Till
• If Dr. Price Denies Zeus, He's Irrational
Dr. James D. Price's arguments in the September/October 1997 issue of The Skeptical
Review have convinced me that the anti-supernatural bias of skeptics is wrong. I
hereby renounce my former irrational anti-supernaturalism and take up the cause of
the divine powers that created and govern this universe: the gods of Mount Olympus.
• From the Mailbag
Letters from readers are published on a variety of issues along with the editor's
comments on many of the letters.

The Biblical Way to Treat Women with


Dignity
Farrell Till
As noted in previous articles, a popular claim of Christianity is that the Bible has accorded
women a status superior to that in societies dominated by other religions. Those who make
this claim are either very ignorant of what the Bible teaches or else have no scruples against
misrepresenting facts to try to further the cause of Christianity. Many of the deplorable
attitudes toward women found in the Bible have already been examined, but none were more
flagrantly sexist in their scope than a "test" in Numbers 5 that Yahweh required of women
accused of adultery.

If any man suspected his wife of "going astray" but had no evidence to confirm his suspicion,
he was entitled to take her to the priest, "if the spirit of jealousy comes upon him" (vs:11-15),
and the priest would subject her to a trial by ordeal, which she had to pass in order to prove
her innocence. The man was required to bring a meal offering, which the priest would put into
the woman's hands. He would then take a concoction of "holy water" and dust from the
tabernacle floor, which was called "the bitter water that brings a curse," and say an incantation
over the woman:

If no man has lain with you, and if you have not gone astray to uncleanness while under your
husband's authority, be free from this bitter water that brings a curse. But if you have gone
astray while under your husband's authority, and if you have defiled yourself and some man
other than your husband has lain with you... Yahweh himself make you a curse and an oath
among your people, when Yahweh makes your thigh rot and your belly swell; and may this
water that causes the curse go into your stomach and make your belly swell and your thigh
rot" (vs:19-22).

At this point, the woman was required to say, "Amen, so be it" (v:22). Who says that Yahweh
didn't have a sense of fairness?

Volume 1990 - 2002 Issue


Page 1114 of 2049
Skeptical Review Edited by Farrell Till
The priest would then write "these curses" into a book and "scrape them off into the bitter
water" (v:23), at which point the "bitter water" would contain not only dirt from the
tabernacle floor but apparently any other contaminants that may have been in the "ink" and on
the surface of the book that was scraped. .cb After this, the woman was required to "drink the
bitter water that brings a curse, and the water that brings the curse shall enter her to become
bitter" (v:24).

The priest would take the "grain offering of jealousy" from the woman's hand and "wave it
before Yahweh" and then take it to the altar. He would take a handful of the offering as a
"memorial portion" and burn it on the altar. (None of this is being made up; it's all in the
inspired word of God.) Apparently, the woman would have to take another swig of the bitter
water at this point, just in case the first dose had not done enough damage. The inspired text
tells us that after the woman had drunk the bitter water, "then it shall be, if she has defiled
herself and behaved unfaithfully toward her husband, that the water that brings a curse will
enter her and become bitter, and her belly will swell, her thigh will rot, and the woman will
become a curse among her people" (v:27). As suggested earlier, however, Yahweh did have a
sense of fairness, for "if the woman has not defiled herself, and is clean, then she shall be free
and may conceive children" (v:28).

Now wasn't that decent of the merciful Yahweh? Living in enlightened times, we can all
guess what happened during these ceremonies. Some women would appear to pass the test,
and others would appear to fail. We know, however, that the factors of guilt or innocence
would not have been involved. In the same way that some people today are resistant to
bacteria and viruses and can live in the midst of epidemics without being infected, while
others fall victim to the diseases they cause, so it undoubtedly was with the women who were
subjected to this trial by ordeal to test them for adultery. It was all a matter of individual
constitution. Some women could drink the contaminated "water of bitterness" and experience
no adverse effects, whereas others probably did experience internal infections that caused
their "bellies to swell" and their "thighs to rot." Their guilt or innocence, however, had
nothing to do with it. Individual resistance to infection would have been the chief determining
factor. In his magnanimity, Yahweh decreed that if the woman's belly did not swell and her
thighs did not rot, then she was innocent and would be free to "conceive children." Yes, after
all, that's the whole duty of women, isn't it, to be free to conceive children? It's a vital part of
the dignity and lofty status that the Bible has accorded women.

Well, what happened when women "passed the test"? Were their husbands held accountable
in any way for having falsely accused their wives? Those who would even ask the question
have undoubtedly developed a sense of fairness from having been subjected to too much
modern philosophy about the equality of the sexes. Such equality just wasn't Yahweh's way,
so we are told that when the trial by ordeal was over, "the man shall be free from iniquity"
(v:31). However, the woman who didn't pass muster had to "bear her guilt."

An interesting issue is raised by the reference to conceiving children that women who did
pass muster were "free" to do. We can imagine that in a superstitious society that used a trial
by ordeal like this to test for adultery, there would have been many pregnant women brought
before priests to drink the so-called "water of bitterness," and no doubt many of them,
whether guilty or innocent, didn't pass the test. If the concoction of "holy water" and

Volume 1990 - 2002 Issue


Page 1115 of 2049
Skeptical Review Edited by Farrell Till
contaminants produced anything at all like the swelling bellies and rotting thighs described in
the ceremonial instructions, we can also imagine that many pregnant women experienced
abortions that were induced by the "water of bitterness" and that some women were left sterile
by its effects. This point is made neither to defend nor condemn the practice of abortion but to
suggest that this "test for adultery" poses a problem for those who argue so fervently that
abortion violates "God's law," for those who so argue are presumably referring to the same
god who ordained this "test for adultery" that would have presented obvious threats to the life
of any fetus whose mother was forced to submit to it. Would a god who is so vitally
concerned about the welfare of unborn children have devised a plan like this, or was it just a
practice that originated with primitive, superstitious people who thought that their god was
directing the outcome? It's something biblicists should think about as they ponder the
problems that this trial by ordeal poses for their claim that the Bible has elevated the dignity
of women much higher than it is in other religious societies.

Did the Judge of All the Earth Always Do


What Was Right?
Farrell Till
Elsewhere in this issue (pp. 4-5), Roger Hutchinson continues his losing battle to prove that
when Yahweh killed David's son, there was no violation of the biblical principle that children
should not bear the iniquity of their fathers. This principle was stated in Deuteronomy 24:16,
"The fathers shall not be put to death for the children, neither shall the children be put to death
for the fathers: every man shall be put to death for his own sin." It was repeated in Ezekiel
18:20, "The soul that sins, it shall die: the son shall not bear the iniquity of the father, neither
shall the father bear the iniquity of the son; the righteousness of the righteous shall be upon
him, and the wickedness of the wicked shall be upon him." My position has been that the
killing of David's son for his parents' act of adultery was a clear violation of this principle, but
Hutchinson has resorted to all sorts of verbal maneuvers to deny this conclusion.
I have decided that the best way to bury this issue is to let my rebuttal of Hutchinson's
latest effort (pp. 5-7) and its predecessor articles speak for themselves as I take readers
through the Bible to show other examples of where Yahweh violated his own moral
code and held the innocent accountable for the "sins" of others. I hope that Hutchinson
will go along on the trip and see for himself that he is trying to ride a dead horse.
First, I will need to establish that the Hebrew god Yahweh declared another moral
principle that will be important in showing that he sometimes punished the innocent
for the offenses of the guilty. That other principle was enunciated when Moses was
reminding the Israelites of a rebellion against Yahweh when the spies who had been
sent into Canaan had returned with a report of giants in the land. The people had
trembled so at the report that Yahweh, in one of his typical tantrums, decreed that
everyone but the children and Joshua and Caleb (who had urged the people not to
listen to the spies) would be doomed to wander in the wilderness for 40 years until

Volume 1990 - 2002 Issue


Page 1116 of 2049
Skeptical Review Edited by Farrell Till
they were all dead, after which those who had been children at the time of the incident
would be allowed to go into Canaan and possess the land. In Deuteronomy 1:39,
Moses explained why Yahweh had spared the children the punishment of death in the
wilderness that he had pronounced upon the adults: "Moreover your little ones and
your children, who you say will be victims, who today have no knowledge of good
and evil, they shall go in there; to them will I give it, and they shall possess it." The
other principle, then, was a fair and just one: those who were too young to know right
from wrong would not be held accountable for their actions or the actions of their
parents. We will see that Yahweh violated this principle time and time again.
The most obvious example to begin with would be the Genesis flood by which
Yahweh destroyed all life on earth except for that which was aboard the ark. Yahweh's
complaint was that "the wickedness of man was great in the earth and that every
imagination of the thoughts of his heart was only evil continually" (Gen. 6:5).
Therefore, Yahweh selected Noah, who had found favor in the eyes of Yahweh (6:8),
and his family to build an ark by which life on earth could be saved from destruction.
For sheer silliness, few stories in the Bible rival this one, but for this article, I won't
concentrate on biblical absurdities. I will just take the Bible at face value and note that
if this story happened as recorded in the Bible, then there were necessarily thousands
of children all around the world who drowned in the flood. Even if we assume that the
wickedness of their parents was "great" and that "every imagination of the thoughts of
[their] hearts was only evil continually," this would not have been true of the children,
who would have been just like the Israelite children mentioned above, who did not
know the difference in good and evil. In killing them via the flood, Yahweh punished
them because of the sins of their parents or adult guardians and in so doing violated
his own principle that children should not bear the iniquities of their fathers.
The same is true of all the children who were killed when God destroyed Sodom and
Gomorrah. Genesis 19:24-26 says that Yahweh rained fire and brimstone from heaven
onto these cities and overthrew "all the inhabitants," except for Lot and his two
daughters. If anything like this happened, then there would have been children and
infants in these cities who, like the Israelite children in the wilderness, would not have
known the difference in good and evil. So if the wickedness of these cities was
"grievous," as Yahweh claimed in Genesis 18:20, it would have been due to the
wickedness of the adults. The children, like all children who don't know good from
evil, were innocent. In killing them, Yahweh violated his own law that prohibited
children from bearing the iniquity of their fathers.
Another case in point was the massacre of children and infants during Joshua's
invasion of the "promised land." Yahweh himself had told Moses that nothing should
be saved alive to breathe in these cities but to utterly destroy them (Dt. 20:16). Joshua
10:40 and 11:11 claim that this commandment was carried out and that nothing was
left alive to breathe in the cities that the Israelites destroyed in Canaan. Joshua 11:15
and 20 state that all this was done according to what Yahweh had commanded Moses.
The image of an invading army massacring everyone in the conquered territory is
repugnant to all modern standards of morality, yet the Old Testament depicts the god
Yahweh as a barbaric national deity who commanded his "chosen people" to take no
prisoners in their march through the "promised land." Rather than recognizing such
stories for what they are, i. e., the superstitions of a primitive people who had created
their god in their own barbaric image, biblicists resort to all sorts of desperate
rationalizations to try to explain this embarrassing moral problem in their sacred

Volume 1990 - 2002 Issue


Page 1117 of 2049
Skeptical Review Edited by Farrell Till
scriptures. Some actually argue that God did these children a favor by having them
killed, because they died sinless and went to heaven rather than growing up to be
wicked like their parents. Inerrantists also talk about a "plan of redemption" that
required Yahweh to eradicate entire nations of people who were so completely evil
that unless they were utterly destroyed, they would have been a threat to corrupt the
chosen "seed" through whom the redeemer would come, but even if we conceded that
there was any merit at all to this "argument"--and there isn't--it would not explain why
Yahweh ordered the massacre of even children and infants. Once the adults in these
nations had been exterminated, there would have been no one left but the Hebrews
themselves to corrupt the children and infants, who at that time did not know the
difference in good and evil, so surely biblicists will not argue that keeping the children
alive to be educated in the religion of the Hebrews would have posed any kind of
threat to a "plan of redemption" that Yahweh may have had in mind. So if the
Canaanite nations were killed because they were "evil," killing the children and infants
too would have been a case of children bearing the iniquity of their parents. If not,
why not?
In another case, Yahweh even held the entire nation of Israel accountable for the sin of
just one man. After the defeat of Jericho, the Israelites turned their attention to the city
of Ai, but even before the battle began, the Bible states that "the anger of Yahweh was
kindled against the children of Israel" (Josh. 7:1). The source of the anger turned out
to be the offense of a single man named Achan, who during the sacking of Jericho had
found some precious artifacts and kept them for himself, buried in his tent. Such
things were to become a part of the spoils of war that were to be "devoted to Yahweh,"
which is just another way of saying that they were supposed to be put into coffers that
were controlled by the priests. Rather than informing Joshua that one of his soldiers
had committed this offense, Yahweh allowed an Israelite force to attack Ai and suffer
defeat as punishment for Achan's sin (7:2-5). In great anguish, Joshua then tore his
clothes and fell upon his face before the ark of Yahweh to ask why this terrible
calamity had happened. Only then did Yahweh tell Joshua what the problem was.
"Israel has sinned," Yahweh said. "Yes, they have even transgressed my covenant that
I commanded them. Yes, they have even taken of the devoted thing and have also
stolen and dissembled also; and they have even put it among their own stuff" (7:11).
Well, pardon my insolence, but just where did Yahweh get all of this "they" and
"their" stuff, because as it turned out, the man Achan was the only one who had kept
artifacts for himself during the sacking of Jericho. Yet Yahweh was blaming all the
Israelites for it and had caused them to suffer a humiliating defeat. He informed
Joshua that this sin was why the Israelites had been unable to stand before their
enemy, and announced that he would not be with them any more unless the "accursed
thing" in their midst was destroyed (v:12). So Joshua began a family-by-family
investigation, and when he confronted Achan, he somehow knew that this was the
guilty party. "My son," Joshua said to Achan, "give, I pray you, glory to Yahweh, and
make confession to him, and tell me what you have done. Hide it not from me" (v:19).
Reading this, we have to wonder why, if Joshua had such insight, he had not
recognized Achan's guilt before his troops had gone out to be defeated. Thirty-six men
had died in that encounter (7:5), all because Yahweh was upset at one man.
At any rate, Achan came clean and told what he had done. Whoever said that
confession is good for the soul probably never heard of the punishment that Achan
received for the crime he confessed to. Achan and all of his sons and daughters, along

Volume 1990 - 2002 Issue


Page 1118 of 2049
Skeptical Review Edited by Farrell Till
with "his oxen, and his asses, and his sheep, and all that he had" were taken to the
valley of Achor, where "all Israel" stoned them and burned them with fire and "raised
over them a great heap of stones" (7:25-26). That must have been some sight when
"all Israel," which according to census figures in the book of Numbers would have
totaled about 2.5 to 3 million people, stoned this family and all their livestock to death.
The ones in the back must have encountered considerable difficulty getting a clear
shot at the victims. Anyway, this story is an example of Yahweh's not only allowing
but requiring the punishment of children for an offense of their father.
Some may argue that the members of Achan's family knew about his theft of the
artifacts and therefore shared his guilt, but that would be pure speculation. The biblical
text gives no indication at all that anyone was involved in the "crime" except Achan
himself. Certainly his oxen, asses, and sheep couldn't have known anything about it.
This story is simply indicative of the barbarity of the times when it was believed
proper to punish a man by killing his entire family in order to abolish his "name"
forever by leaving him no descendants. If space permitted, I could cite other biblical
examples of this barbarous practice. This was exactly the mentality behind massacres
of civilian populations during the Israelite invasion of Canaan. The disgrace is that
Bible believers accept this as appropriate conduct and bend over backwards to defend
the claim that "God" had decreed it to happen.
Probably the most flagrant example of Yahweh's requiring the innocent to be punished
for the "sins" of their fathers is found in his command to utterly destroy the Amalekite
nation. During their trek across the Sinai wilderness, the Israelites were attacked by
the Amalekites (Ex. 17:8-16). As a result of this battle, Yahweh swore that he would
"have war with Amalek from generation to generation" (v:16). In his farewell address
to the Israelites, Moses mentioned this incident and told them to "(r)emember what
Amalek did to you on the way as you were coming out of Egypt" (Dt. 25:17). He went
on to say that it would be "when Yahweh your God has given you rest from your
enemies all around, in the land which Yahweh your God is giving you to possess as an
inheritance, that you will blot out the remembrance of Amalek from under heaven.
You shall not forget" (v:19).
All of this happened in the wilderness during the 40-year wanderings of the Israelites,
but about 450 years later, when Saul was king of Israel, Yahweh decided it was time
to make good his prior threat and "blot out the remembrance of Amalek from under
heaven." He sent the prophet Samuel to Saul with this message: "Thus says Yahweh of
hosts, I will punish Amalek for what he did to Israel, how he ambushed him on the
way when he came up from Egypt. Now go and attack Amalek and utterly destroy all
that they have, and do not spare them. But kill both man and woman, infant and
nursing child, ox and sheep, camel and donkey" (1 Sam. 15:2-3, emphasis added). If
ever there was an example in the Bible that shows the utter futility of trying to depict
the Hebrew god Yahweh as a just deity who did not require descendants to bear the
iniquity of their fathers, we certainly have it in this passage. Inerrantists have tied
themselves into verbal knots trying to explain this text. They argue that the Amalekites
at this time were a morally depraved nation, and so it was for their own iniquity that
Yahweh ordered their destruction. But what does the text say? It has Yahweh clearly
saying that he would punish the Amalekites for attacking the Israelites on their way
out of Egypt. The KJV has Yahweh saying, "I remember that which Amalek did to
Israel," so the only reason that he gave for these orders was the Amalekite attack on
Israel about 450 years earlier. One could reasonably argue that the Amalekites had

Volume 1990 - 2002 Issue


Page 1119 of 2049
Skeptical Review Edited by Farrell Till
simply reacted to protect their territory from the intrusion of a nomadic horde of about
3 million, who had an army of 600,000 foot soldiers, but that is really beside the point.
Whatever the Amalekites may have done to the Israelites in the wilderness, the
Amalekites living 450 years later were not responsible for it. To hold them
accountable by ordering their "utter" destruction can be seen only as a clear case of
Yahweh's violating his own moral standard that had exempted children from
responsibility for the sins of their fathers.
The extent of Yahweh's grudge against the Amalekites is seen in the rest of this story.
Verse 9 states that Saul carried out Yahweh's instructions by attacking the Amalekites
and "utterly destroy[ing] them" with the exception of Agag, their king. When Saul
kept Agag alive to bring back as a prisoner, Yahweh sent Samuel the prophet to meet
Saul and strip him of his kingship for disobeying the command to "utterly destroy" the
Amalekites. Samuel then took a sword and hacked Agag to pieces (v:33) as an
apparent indication that when Yahweh said "utterly destroy," he meant utterly destroy.
This is the god that Roger Hutchinson has been defending in a vain attempt to show
that the Bible "consistently teaches that children are not to be punished for the sins of
their fathers" (p. 4, this issue). Needless to say, the Bible does not consistently teach
this. In the next life, Hutchinson should try to see how much luck he can have in
convincing the Amalekites that Yahweh never punished the innocent for the sins of
their fathers. Earlier I mentioned Yahweh's destruction of Sodom and Gomorrah as
examples of his holding children accountable for the sins of their fathers. In a famous
biblical scene in Genesis 18, Abraham tried to persuade Yahweh to spare Sodom. He
pleaded with Yahweh to spare the city if 50 righteous people could be found in it.
"Will you consume the righteous with the wicked?" Abraham asked Yahweh, and we
see now that the answer to that question is yes. On many occasions Yahweh consumed
the righteous with the wicked. It didn't seem to matter to him. In this same context,
Abraham asked Yahweh, "Shall not the judge of all the earth do right?" If we assume
that this Yahweh exists and that he is the "judge of all the earth" and that the Bible is
an accurate account of his affairs with humanity, we are forced to say that the answer
to Abraham's second question is, "No, the judge of all the earth won't necessarily do
what is right, because he sometimes kills children for the offenses of their parents." If
inerrantists think that this is "right," then they need more help than I can give them.

Sins of the Fathers and other Matters


Roger Hutchinson
Farrell Till, responding to the article on Sins of the Fathers (TSR, July/ August 1997), did
little to advance a constructive dialog on a basic Biblical issue. I want to address the
confusion in Till's article because it affects how we look at Biblical interpretation broadly and
then focus on the specific arguments that he made.

The confusion begins with a method to interpret Scripture. In his article, Till refers to "an old
inerrantist tactic, which says that `scripture must interpret scripture,' but this is fallacious

Volume 1990 - 2002 Issue


Page 1120 of 2049
Skeptical Review Edited by Farrell Till
hermeneutics.... In the first place, this claim that scripture should be allowed to interpret
scripture flagrantly begs the question of biblical inerrancy by assuming that there are no errors
in the Bible, and so when two passages appear to contradict one another, it must be true that
one of them does not mean what it clearly says. Inerrantists themselves would never accord
this same consideration to any other written documents."

Interpreting Scripture with Scripture is a method no different from defining a word in a


sentence by the context in which it is used. We all do it. This has nothing to do with inerrancy
as Till would have one to believe; it has everything to do with consistency.

Consistency in writing does not eliminate error. If two statements are true, then they must also
be consistent. However, two statements can be consistent even if one or both of the statements
is false. Consequently, we do not have to assume anything about the truthfulness of a
document in order to understand what it says. When we interpret Scripture with Scripture, we
are seeking to understand one passage of Scripture within the context established by all
Scriptures. This method is used to determine what Scripture says, not whether it is inerrant.
Till confuses the issue with the reference to inerrancy.

Additionally, inerrantists want other written documents to be evaluated in the same manner.
Till misleads the reader to think otherwise. When we require that a document be consistent in
its presentation of ideas and in its use of language (particularly metaphors), we are
establishing a basis for determining exactly what the document is telling us. Once we
determine what a document says, then we can determine its accuracy. We should do this for
all documents, not just the Bible.

If I want to claim that the Bible is inerrant, then I must, among other things, be able to show
that the Bible is internally consistent. However, while internal consistency is a necessary
condition for proving inerrancy, it is not sufficient, by itself, to do this. We would have to do
much more to prove inerrancy than just show consistency.

Let's use an example. Genesis 1 describes how God created the heavens and the earth in six
days and it uses the Hebrew word yom for day. Is Genesis 1 telling us that God created
everything in six 24-hour days or that He did it over some longer time period? To determine
what Genesis 1 is telling us, we interpret Scripture with Scripture. This requires that we do
two things. We look at every instance in which the Hebrew word yom is used in the Bible
outside Genesis 1 to see if there is one consistent definition that can be determined by the
context in which the word is used. We also look at all other references to the creation in the
Bible to see if a consistent theme is expressed with regard to the length of a day.

When we do this, we find that yom is always used outside Genesis 1 to mean a 24-hour day
(with one or two exceptions that do not affect our conclusion) and that all other references to
the creation also point to a 24-hour day. By interpreting Scripture with Scripture, we conclude
that Genesis 1 is telling us that God created the heavens and the earth in six literal 24-hour
days. By interpreting Scripture with Scripture, we find that the Bible consistently expresses
this theme. This consistency does not prove that Genesis 1 provides an inerrant account of
creation. Further, we do not have to assume inerrancy in order to understand what Genesis 1,
or the Bible, says about the creation. Interpreting Scripture with Scripture is a methodology

Volume 1990 - 2002 Issue


Page 1121 of 2049
Skeptical Review Edited by Farrell Till
for determining what the Scriptures say. It is not fallacious hermeneutics. Till's reasoning in
this instance is fallacious and not good hermeneutics.

In the original article, I compared Scripture with Scripture to show that the Bible consistently
teaches that children are not to be punished for the sins of their fathers. Even Till conceded
this point. This brings us to the situation involving David and Bathsheba which Till has
muddled unmercifully. It is often the manner in which issues are defined that lead to
confusion. An example is where Till states, "In the story of David and Bathsheba, however,
there is nothing to indicate that the death of their son resulted from a cause-effect factor. To
the contrary, the biblical text clearly states that `Yahweh struck the child that Uriah's wife
bore to David' (2 Sam. 12:15). If Yahweh had not `struck' the child, he would not have died,
so the child's death was not a consequence of David's and Bathsheba's `sin' but rather the
result of Yahweh's intervention for the express purpose of killing the child."

This is an interesting argument. Are we to believe that the death of the child born to
Bathsheba was not somehow related to the adulterous affair between David and Bathsheba?
Of course not. There is a definite relationship. One issue, then, is whether we can use the
term, "consequence," to describe this relationship.

Had David and Bathsheba not sinned, no child would have been conceived. The sin of the
parents laid the foundation for a series of events that included the conception, birth, and death
of that child. Till wants us to believe that the death of the child would have to have been a
direct, physical effect arising from the sexual act in order to be labeled a consequence of that
sinful act. He argues that intervention by a third party, such as God, in reaction to that sin
cannot be described as a consequence of that sin. Till's argument is ludicrous and contributes
nothing to the issue.

The basic issue is not whether the death of the child was a consequence of David and
Bathsheba's sin. It was. The issue is whether the death of the child was also a punishment for
that sin. As Till puts it, "What we have here is a simple case of a diehard biblical inerrantist
who refuses to attribute unfairness or injustice to the Hebrew god Yahweh no matter how
obvious biblical accounts are in describing Yahwistic actions that are unfair and unjust by any
reasonable standard of conduct."

Regardless of Till's inference that God's action in this case was unfair, the issue is whether it
was unjust and whether we should conclude that, as Till says, "(T)he Bible contradicts itself
by saying in some places that children will not be punished for the sins of their fathers but
showing in other places that Yahweh did sometimes punish children for their fathers' sins."

Later in his article, Till asks the rhetorical question, "Is there anything in the story to indicate
that this child would have died if he had not been born of an adulterous act?" Here Till argues
that the child would not have died if no adultery occurred, so the child should not have died
even though the act was adulterous. In other words, the death of the child was not tied to the
sexual act so whether the act was adulterous makes no difference. He then asks, "(W)hat else
can we call this but a clear example of a child being killed for the sins of his parents?" It turns
out that there is more to the story than Till wants to address.

Volume 1990 - 2002 Issue


Page 1122 of 2049
Skeptical Review Edited by Farrell Till
As we read the account of David's sin, we see that, "Nathan said unto David, The LORD also
hath put away thy sin; thou shalt not die.... Howbeit, because by this deed thou hast given
great occasion to the enemies of the LORD to blaspheme, the child also that is born unto thee
shall surely die. And Nathan departed unto his house. And the LORD struck the child that
Uriah's wife bare unto David, and it was very sick" (2 Sam. 12:7-15). Here, we see that the
Bible links the death of the child to the "occasion for God's enemies to blaspheme" and not to
the adultery of David and Bathsheba.

With respect to this, Till states, "The text states very clearly that the sickness, which
eventually caused the child to die, resulted from Yahweh's striking the child. The way that
this story is told, if Yahweh had not `struck' the child, he would not have died, and so there
was nothing inherent in David's and Bathsheba's adulterous act that caused the child to die."
We can all agree with this. We can also agree that the point is irrelevant. There was nothing
inherent in the sexual act that caused the child to die either from a physical medical viewpoint
or from the Biblical viewpoint. The Scriptures tell us that the death of the child was not tied to
the adulterous act but to the "occasion to blaspheme." The adulterous act merely provided the
foundation for a series of events that included the death of the child. These events would not
have occurred if David had not sinned.

We now summarize what happened. David and Bathsheba sinned by committing adultery.
One consequence of that sin was that a child was conceived and born. Another consequence
was that it provided an opportunity to the enemies of God to blaspheme. God then reacts to
the "occasion to blaspheme," and takes the child away from David. The death of the child is
linked solely to the opportunity for God's enemies to take advantage of David's sin in order to
blaspheme God; it is not linked to David's adultery.

The most that we can conclude is that the death of the child was a punishment of David for
giving the enemies of God an opportunity to blaspheme. There is no basis to conclude that the
death of the child was a punishment for the sin of adultery committed by David and
Bathsheba. Consequently, there is no contradiction in the Biblical account.

The allegations of false analogies and diatribes about inerrantist tactics in Till's article were
side shows that accomplished nothing. Till's time would have been spent better in addressing
the issues.

Roger Hutchinson, 11904 Lafayette Drive, Silver Spring, MD 20902; e-mail,


rhutchin@aol.com)

Is Roger Hutchinson for Real?


Farrell Till

Volume 1990 - 2002 Issue


Page 1123 of 2049
Skeptical Review Edited by Farrell Till
Although I have had a long association with Roger Hutchinson through letters, internet and e-
mail exchanges, and his articles that have been published in TSR, when I read material from
him like the foregoing article, I have to wonder if he is a real person or at least a prankster,
who sends me his articles and then laughs when I take them seriously enough to publish and
respond to. He has been receiving TSR for several years, but he must not be reading the
articles or he would know by now why the principle of hermeneutics that says scripture
should be allowed to interpret scripture is fundamentally unsound.

He claimed that "(i)nterpreting Scripture with Scripture is a method no different from defining
a word in a sentence by the context in which it is used." If this were all that was involved in
the principle, it would, of course, be a sound one, but biblicists carry it far beyond this. They
insist on making the entire Bible the "context" of a disputed statement and straining the
principle to the point of letting passages that were written centuries apart "interpret" each
other. In the May/June 1996 issue, we saw Wilhelm Schmitt use this hermeneutic principle to
try to remove the chronological discrepancy between the four-generation genealogy of Aaron
in Exodus 6 and the claim in Exodus 12:40 that the Israelites had sojourned in Egypt 430
years. Schmitt argued that a statement by the apostle Paul in Galatians 3:17 should be allowed
to shed light on the "not so clear" passages in Exodus. In this text, Paul said that 430 years
had passed from the covenant that God made with Abraham until the giving of the law, so
Schmitt argued that the discrepancy was solved. Paul's statement was clearer than the other
one, and so the Exodus writer couldn't have meant what he appeared to be saying. The only
problem was that Schmitt was arguing from an unproven assumption of total biblical
inerrancy and was unwilling to give any consideration at all to the possibility that Paul, who
lived centuries after the Exodus writer, simply had a different opinion on the duration of the
Egyptian bondage.

In my response to Schmitt, I pointed out that biblicists would not consider this an appropriate
critical method to apply to any other book. If volume two of Encyclopedia Americana should
contain a statement that conflicts with a statement on the same subject in volume ten, no one
would argue that the statement in volume ten is clearer than the one in volume two, and so it
should be allowed to "interpret" the earlier statement and remove the contradiction.
Reasonable people would understand that a multivolume encyclopedia, like the books in the
Bible, was not written by the same person and that different writers often can and do have
opposing views, so therefore inconsistency in such a work is not only possible but probable.

This highly touted hermeneutic principle is as if a literature teacher should tell his/her class
that all of the literary selections in the textbook are inerrant, and so if a story by Ernest
Hemingway should appear to contain a discrepancy, the students should consult the writings
of John Steinbeck, Washington Irving, Walt Whitman, et al to find "clearer" statements on the
same subject and use them to remove the discrepancy in Hemingway's work. It is equally
absurd to apply this principle to the biblical anthology. The prophet Isaiah and the apostle
Paul were two different men, who lived centuries apart, and so it is just as likely that they
would have conflicting religious views as any other two people. To say that the apostle Paul
should be allowed to interpret Isaiah, or vice versa, is no more hermeneutically sound than
arguing that Martin Luther should be allowed to interpret Ezekiel.

Volume 1990 - 2002 Issue


Page 1124 of 2049
Skeptical Review Edited by Farrell Till
Hutchinson cited the usage of the Hebrew word yom (day) as an example of how the entire
context of the Old Testament should be allowed to define words. He said that "with one or
two exceptions that do not affect our conclusion," the word yom was always used "outside
Genesis 1 to mean a 24-hour day." Boy, did Hutchinson put his foot in his mouth on this one!
I happen to agree that the Genesis writer meant for his readers to understand that God created
the heavens and the earth in six 24-hour days, but the word yom was used many times-- not
just once or twice--in the Old Testament to mean "eras" or periods of time longer than 24
hours. One has to go no further than the Genesis account of creation to find an example:
"These are the generations of the heavens and of the earth when they were created, in the day
(yom) that Yahweh made earth and heaven" (Gen. 2:4) Since chapter one states that God
created heaven and earth in six days and rested on the seventh, the word yom in "the day that
Yahweh made earth and heaven" has to mean more than just a 24-hour period. It obviously
conveyed the sense of a period of time rather than just 24 hours. Genesis 19:37 stated that
Moab, the son of Lot's firstborn daughter, was "the father of the Moabites until this day
(yom)." Surely, Hutchinson won't claim that the Genesis writer meant that Moab was the
father of the Moabites only up until the very 24-hour day that this statement was written, but
afterwards the statement didn't apply and Moab somehow ceased to be the father of the
Moabites. That would put an absurd meaning on the word yom (day), which was obviously
being used in this verse to mean the time period in which the Genesis writer lived and wrote.
Other examples are too numerous to note, but anyone can use a concordance to find many
places where yom was used to mean a longer period of time than just 24 hours. Job 18:20,
Psalm 20:1, Ecclesiastes 7:14, Isaiah 17:4, Jeremiah 4:9, and Ezekiel 30:3 are just a few of
many examples I could cite, so I suggest to Hutchinson that if he is going to use an example
in support of his position, he should at least use one that is tenable. This one fizzled on him.

After taking us through his yom excursion, Hutchinson concluded that his methodology "is
not fallacious hermeneutics." He said, "Till's reasoning in this instance is fallacious and not
good hermeneutics." Well, my reasoning is that different people could and did have divergent
views on religious issues and that the more widely separated they were in time, the more
likely they were to be in disagreement. Let Hutchinson explain to us what is "fallacious"
about that reasoning. What is the fallacy in assuming that writers who lived centuries apart
could very well have disagreed on some subjects that they wrote about?

Good Hermeneutics? Hutchinson needs to explain why the scripture-should-interpret


scripture is "good" hermeneutics. What is his rationale for assuming that the author of Amos
or Mark or Daniel could not have written anything that conflicted with something the author
of Genesis wrote? The only possible answer he can give to this question is that he assumes
that all books that were selected for inclusion in the biblical canon were inerrant, so he must
explain to us why it is reasonable to believe this. We just want simple, verifiable reasons why
one should believe that the Bible-- but no other collection of writings-- should be accorded
such a privileged status. I suspect we will wait a long time to get those simple, verifiable
reasons.

Hutchinson claimed that he "compared Scripture with Scripture to show that the Bible
consistently teaches that children are not to be punished for the sins of their fathers," but he
did no such thing. He compared scripture with scripture in order to claim that if King
Amaziah did not kill the children of his father's murderers because it was written in the law of

Volume 1990 - 2002 Issue


Page 1125 of 2049
Skeptical Review Edited by Farrell Till
Moses that the children should not be put to death for the sins of their fathers (2 Kings 14:1-
6), then it must be true that David's son was not killed for a sin that David had committed.
That "comparison" in no way demonstrated that the Bible "consistently teaches that children
are not to be punished for the sins of their fathers." It demonstrated only that the Bible in
some places does indeed teach that children should not be punished for the sins of their
fathers, and it demonstrated that some biblical characters, as in the case of Amaziah, honored
this law; however, if other biblical passages, such as the story about the death of David's son,
indicate that children were sometimes punished for the sins of their fathers, then we have a
point of contradiction in the Bible. Hutchinson cannot make that contradiction go away by
arguing that if passage X clearly states that children should not be punished for the sins of
their fathers, then if passage Y states that a child was punished for the sin of his father,
passage Y must not have meant what it says. Such reasoning as this attempts to prove biblical
inerrancy by assuming biblical inerrancy and completely ignores the probability that biblical
writers simply had divergent opinions on this subject.

A resort to equivocation: Hutchinson asked if we should believe that the "death of the child
born to Bathsheba was not somehow related to the adulterous affair between David and
Bathsheba." In answering his own question, he said, "Of course not," because "(t)here is a
definite relationship." From there, he went on to say that the issue is "whether we can use the
term consequence to describe" the relationship between the parents' act of adultery and the
death of the child. He argued that if David and Bathsheba had not sinned, then no child would
have been born, and so the act of adultery had "laid the foundation for a series of events that
included the conception, birth, and death of that child." In that sense, Hutchinson concluded,
the death of the child can properly be considered a "consequence" of adultery.

We see, then, that Hutchinson has had to resort to equivocation to defend his claim that the
death of David's son was a consequence of and not a punishment for David's sin. In his
original article on the issue, Hutchinson had compared the death of David's son to the death of
an innocent party in an accident that resulted from two men drag racing on a public highway.
When one of the drivers lost control, his car crashed into another one and killed an innocent
third party. In this case, the death of the innocent person was a direct consequence of the
reckless conduct of the drag racers. Here the word consequence was being used to denote an
effect that resulted directly from a wrongful act without the intervention of any other agent,
who was not involved in that act, to bring about additional effects, but now Hutchinson is
trying to extend the meaning of consequence to include the intervention of a third-party
causative agent. It may be true that no child would have been born for Yahweh to kill had
David and Bathsheba not committed adultery, but it remains embarrassingly true that had
Yahweh not afflicted him by direct intervention, the child would not have died. This act (if it
happened) was fundamentally different from the death of an innocent party caused by the
reckless conduct of drag racers. To make the difference clear enough for even Hutchinson to
see, let's give names to the parties involved in the drag racing incident. We will call the drag
racers Joe and Bill, and the person killed in the accident will be Jim. Surely, Hutchinson can
see that Joe's and Bill's actions killed Jim. No third party intervened. However, in the case of
David's son, nothing that either David or Bathsheba did killed the child. Yahweh, a third
party, intervened and "struck" the child, who then died. To have a parallel to this, we would
have to imagine that Joe and Bill recklessly engaged in drag racing during which Joe's car
careered out of control and killed Jim, after which the police went to Joe's house and killed his

Volume 1990 - 2002 Issue


Page 1126 of 2049
Skeptical Review Edited by Farrell Till
son. Not even Hutchinson would claim that killing Joe's son in this scenario was justifiable,
but his beloved Yahweh can do no wrong. Therefore, if the Bible says that Yahweh killed a
baby, Hutchinson must rationalize some justification for it.

In so doing, he did an about face and wound up flatly contradicting himself. As just noted, he
first argued that the death of the child must be considered a "consequence" of David's and
Bathsheba's act of adultery, for if the adulterous act had not occurred, no child would have
been born, and so "(t)he sin of the parents laid the foundation for a series of events that
included the conception, birth, and death of that child" (p. 4). In other words, if no adultery,
then no child, and if no child, then no death of the child; therefore, the death of the child was
a "consequence" of the adultery. As he was ending his article, however, he argued that
Yahweh could not allow the child to live because "by this deed thou [David] hast given great
occasion to the enemies of the LORD to blaspheme" (2 Sam. 12:14). At this point,
Hutchinson switched horses in midstream and said, "The Scriptures tell us that the death of
the child was not tied to the adulterous act but to the `occasion to blaspheme'" (p. 5). So now
which was it? Was the death of the child a "consequence" of the adulterous act, or was the
death of the child "not tied to the adulterous act"? When one must contradict himself as
flagrantly as this to defend a position, that's a good reason to believe that the position is weak.

We see too that there is apparently no end to the number of hairs that Hutchinson is willing to
split to avoid admitting that there are discrepancies in the Bible. To say that this child was
killed not because of David's adultery but because of the great occasion that the "deed" had
given the enemies of Yahweh to blaspheme is about as flagrant a case of inerrantist hair
splitting as I have seen. Whether the child was killed to punish David for the sin of adultery or
whether he was killed because of the "great occasion" the "deed" had given the enemies of
Yahweh to blaspheme is a quibble that may give Hutchinson some consolation, but the fact
will still remain that the story clearly indicates that it was because of the "deed" that Yahweh
killed the child. Since the child had nothing to do with the performance of the "deed," it
would still remain true that the Bible claims that Yahweh killed a baby for a deed that his
parents had done.

Let's return to the hypothetical drag racers to help Hutchinson see the obvious. If the police
should kill Joe's son after the crash in which Joe had killed Jim, would Hutchinson think that
killing Joe's son was justifiable if the police should argue that Joe's reckless conduct had
given the enemies of the police "great occasion" to complain about the enforcement of traffic
laws? I doubt it. Furthermore, even if David's and Bathsheba's "deed" had given the enemies
of Yahweh "great occasion to blaspheme," killing David's son would not have undone the
"deed." Once it had happened, it had happened, and nothing could ever change that. The
people who had known that David had consorted with another man's wife would still have
known it after the child was dead. If they had had occasion to blaspheme before the child's
death, they would still have had occasion to do so after the child was dead. The writer who
concocted this as an excuse for Yahweh's conduct in this matter wasn't thinking too clearly,
but that's not at all unusual for biblical writers, just as it isn't unusual for biblical inerrantists
to quibble as Hutchinson is doing.

Beginning on page 2 of this issue, another article discusses biblical examples of people whom
Yahweh killed or ordered killed for offenses committed by others. If Hutchinson reads it,

Volume 1990 - 2002 Issue


Page 1127 of 2049
Skeptical Review Edited by Farrell Till
perhaps he will see that even if he could show that David's son was not killed for a sin
committed by his parents, the Bible would still teach that Yahweh on many occasions killed
the innocent for "sins" that others had committed.

Reply to Hutchinson
Sol Abrams
In the article "Sins of the Fathers: Another View" (TSR, July/August 1997), Roger
Hutchinson, using the artistic verisimilitude of semantics, tried valiantly to rationalize the
irrational. His biblical beliefs are so deeply ingrained that he has painted himself into a corner
of denial. I can only hope that someday reason will remove his biblical chains and that he will
recognize Farrell Till, Dan Barker, and Thomas Paine as men of great courage and
intelligence, who succeeded in breaking those chains.

Now there are three types of statements in the Bible: explicit, implicit, and inferential. In 2
Samuel 12:13-14, Yahweh is very explicit. David's son had to die as punishment for David's
sin. Roger tried to obfuscate the issue by splitting hairs on the difference between punishment
and consequence. In this case, there is no difference. Farrell Till made this crystal clear in his
article "A Problem of False Analogy," and I thank him for emphasizing my point that the
Bible says contradictory things in different places. Roger is locked in on Deuteronomy 24:16
(each person dies for his own sin). He implies that this is Yahweh's official position, and
either ignores or plays down any passages to the contrary.

In my original article, TSR, November/December 1996, I disproved inerrancy by using


contradictory verses dealing with the sins of the fathers. There are, however, at least 200
contradictions in other areas. I could have zeroed in on Genesis 1 vs. Genesis 2, where El
Shaddai (later known as Yahweh, I Am, and Jealous) explains creation to Moses. His
explanation of the order in which he created things in Genesis 2 contradicts the order in
Genesis 1 and in addition contradicts basic scientific principles, as we see below. In one
account, the writer (identified in scholarly circles as the "Elohist") referred to the creator as
Elohim; in the other, the writer (considered by scholars to be a different person from the
Elohist) referred to the creator as Yahweh Elohim.

Genesis 1 (Elohist): Order of creation in the first account.

1. The heaven and light were made (vs:1-5).

2. The firmament was constructed and the waters divided (vs:6-8).

3. The waters gathered into seas-- and then dry land, grass, herbs, and fruit trees created (vs:9-
13).

Volume 1990 - 2002 Issue


Page 1128 of 2049
Skeptical Review Edited by Farrell Till
4. The sun and moon created and the stars made also (vs:14-17).

5. Fishes, fowls, and great whales created (vs:20- 23).

6. Beasts, cattle, every creeping thing, man and woman created (vs:24-27).

Summary: Heaven and earth were created before the sun; all animals created, and then man
and woman (both sexes) were created simultaneously.

Genesis 2 (Yahwist): Order of creation in the second account.

1. The heavens and the earth created (v:4).

2. A mist went up from the earth and watered the whole face of the ground (vs:5-6).

3. Man (male only) made out of dust and named Adam (v:7).

4. A garden planted eastward in Eden and man put into it (vs:8-17) .

5. Beasts and fowls created (vs:18-20).

6. Woman created from one of the man's ribs (vs: 21-24).

Summary: The man (male only) was created, then all the plants, beasts, and fowls, and
finally the woman.

Conclusion: The two creation accounts are in obvious conflict, and the different names by
which God was called in the two accounts indicate separate authorships.

(Sol Abrams, 132 Easthampton F, West Palm Beach, FL 33417)

The Man with No Heart: Miracles and


Evidence
Richard Packham
The problem of evidence for miraculous occurrences has frequently been discussed in TSR. It
sometimes takes the form of an exchange between Farrell Till and some inerrantist. Till:
"Extraordinary event require extraordinary evidence." Inerrantist: "Then miraculous events
require miraculous evidence, and since you don't accept anything miraculous, you preclude
the offering of any evidence in support of a miracle!" This puts Till in an unnecessarily
awkward position.

Volume 1990 - 2002 Issue


Page 1129 of 2049
Skeptical Review Edited by Farrell Till
As a retired (and recovering) attorney, I sometimes wonder why this is such a problem.
Perhaps a review of some of the rules of evidence, particularly as they are used every day in
thousands of American and British courtrooms, would be helpful.

The party who makes a positive assertion in court has the responsibility of offering sufficient
evidence for his assertion. If he fails to do so, his assertion is not proven and will be
disregarded. This is the "burden of proof." What is "sufficient" evidence? That depends on the
assertion and its importance in the case. If it is a minor point and asserts an ordinary,
everyday kind of fact, very little evidence is required. If the issue in the case is where Bob
was yesterday, a friend of Bob who saw him yesterday can testify that he saw Bob eating a
ham sandwich, but if Bob died of poisoning from bad chicken, more than casual testimony
would be required to establish definitely that Bob ate ham, not chicken for lunch. Again, if
someone asserts that Bob, who weighs 180 pounds, ate an eight-pound ham and four loaves of
bread for lunch yesterday, that is so unusual that it would require much more evidence than
the casual testimony of one witness.

In law there are generally three degrees of sufficiency of evidence. They are, in ascending
order: 1) preponderance, 2) clear and convincing, and 3) beyond a reasonable doubt. To prove
something by a preponderance of the evidence is to weigh the evidence on each side of an
issue, declaring the side with the most evidence to be proven. This is the standard applied in
most noncriminal trials. The clear and convincing standard is often required when one party is
trying to prove something that is out of the ordinary, that is, something which doesn't
ordinarily occur. An example in law would be trying to prove that someone who signed a
deed or will did so against his will. That is so rare in the ordinary course of things that
whoever makes such a claim must provide considerable evidence to support it. The beyond
reasonable doubt test is used primarily in criminal trials, where the prosecution is required to
prove its case with so much evidence that no reasonable person could doubt the accused's
guilt. (The difference between the reasonable-doubt test and the preponderance test was
vividly displayed in the two recent trials of O. J. Simpson.)

Other rules of evidence require us to disregard some things offered as evidence, such as
hearsay. These rules are not arbitrary, but have developed over centuries in the courts as
pragmatic and reliable methods to come as close as possible to the truth in disputes over facts.
Hearsay evidence--that is, the witness stating that something is true because some other
person told him it was true--is generally not admissible because it is inherently so unreliable.
Documentary records of events that are offered as evidence are not generally admissible
unless they were made at the same time as the events they describe, because otherwise their
reliability is suspect.

An explanation of an event that is probable is more acceptable than an explanation that is


improbable. In fact, improbabilities are not even admissible as evidence if there is a probable
explanation. This rule alone would make the miracles of the Bible unprovable, since they can
all be explained as reports of gullible writers reporting tales of gullible people.

So does an extraordinary event require extraordinary evidence? If "extraordinary evidence"


means "clear and convincing" evidence or evidence "beyond a reasonable doubt," then the
answer is clearly "yes." But that requirement is a statement about the sufficiency of the

Volume 1990 - 2002 Issue


Page 1130 of 2049
Skeptical Review Edited by Farrell Till
evidence, not its nature. The evidence itself can be very ordinary, and, in fact, must be (since
improbable explanations are inadmissible). But if a miracle really happens, there is no reason
why there should not be evidence to prove it.

Let me illustrate with an imaginary miraculous event. Suppose that a man is undergoing
surgery for a heart transplant. As the surgeons remove his diseased heart, and before they can
connect the replacement, the man dies. The surgeons stop the procedure, and the corpse is
taken to the mortuary, where the embalmer begins to do his work. Suppose that the corpse sits
up and says he wants to go home. The astonished embalmer calls an ambulance, which
returns the living corpse to the hospital, where the surprised doctors examine this man, who
has no pulse, no heart, no blood, and a still open chest cavity. They examine him, test him,
photograph him, feed him, and finally send him home. He is contacted by news media, and he
appears on talk shows, where he displays his empty chest. He posts a schedule of visiting
hours at his home, where anyone for five dollars can see him and put a hand inside his chest.
He survives with no heart for ten years.

Now, this event, if it actually happened, would indeed be a miracle, with no ready "natural"
explanation. (Perhaps it would be a miracle for an inerrantist only if the man were brought
back to life as a result of a minister's prayer to God.) The question then is, what evidence
would "prove" that this "miracle" occurred? The evidence would be very convincing both in
amount and weight: the testimonies of the doctors who performed the surgery and other
disinterested doctors who examined him, the x-rays and other medical records, the video tapes
and photographs, the testimony of the thousands of people who personally put a hand into his
chest. Yet all these items of evidence are very ordinary, nonmiraculous things.

I would believe in the virgin birth of Jesus or in his resurrection and bodily ascension into
heaven, if we had such evidence for the actual occurrence of those alleged events. But we do
not. Inerrantists may object, "But they did not keep medical records in those days! They did
not have video cameras! Nobody wrote down the events as they happened!" Yes, and that is
precisely the point. That is why we are not justified in believing any of the miracles reported
in the Bible (or in Tacitus, or in Homer): there is no admissible evidence that they occurred as
reported.

There are undoubtedly many things that are true even though we cannot prove that they are
true. For instance, there may well be a city of twenty million purple people on Planet X in the
Andromeda galaxy. I really don't care whether that is true or not. However, if someone asserts
that such a city exists and that the mayor of that city has ordered that I am supposed to obey
and assist the person now making that assertion, I a justified in requiring some evidence to
establish all his assertions beyond reasonable doubt, before I accede to his demands. That is
all we ask of Biblical inerrantists, who insist that the Bible is such a message from some
invisible being in space.

(Richard Packham, 2145 Melton Road, Roseburg, OR 97470-9320; e-mail


packham@teleport.com)

Volume 1990 - 2002 Issue


Page 1131 of 2049
Skeptical Review Edited by Farrell Till

How Much Can Historical Documents


Prove?
Robert Lockwood
A major philosophical question that has become central as the result of Bible scholarship and
the growth of modern science is whether historical and scientific evidence can establish if
God has revealed any information to mankind. This would also include whether God has
intervened in history. This question is central for the Judeo-Christian and Islamic traditions,
since they center on certain historical documents, monuments, and institutions.

Bible critics have shown that when these items are treated as parts of human history, one can
find out the best estimates of their dates, origins, original meanings and uses, and so on. But
what does all of this establish in terms of the religious significance of these entities? A crucial
matter that one would want to know is whether the documents, no matter what their history,
do or do not convey special knowledge that God has imparted to man. Whether the documents
are older or younger than people used to think does not settle this. But would any historical
information accomplish a resolution of the question?

Let's consider some hypothetical cases. First, let us suppose that someone discovered a very
ancient copy of the Pentateuch. Let us further suppose that by carbon-14 testing this
manuscript could be dated around 1300-1400 B.C. Let us finally suppose that sufficient
evidence could be found to lead historians to conclude that it was in the handwriting of Moses
himself. Does that show that what Moses reported really happened, or only that the historical
Moses reported that it happened?

If other corroborating evidence were found, the broken tablets with the Ten Commandments
written on them, an account by Aaron of what his brother told him, and a journal by a
disinterested bystander present during the period at Mount Sinai, would this settle the matter?
Again the question would be, do these human artifacts show that God spoke to Moses, or
again merely that Moses said that He did? The crucial feature of the situation is not in the
documents but in whether there is a relationship between God and human history. The
documents are human historical ones. Their revelatory status depends upon knowing not just
that the documents report an alleged revelation but rather that a revelation took place. Can any
historical information tell us this?

But what could all this research ever establish? It might settle interesting historical questions,
such as was there a historical figure named Jesus of Nazareth, what actually happened to him
during his lifetime, what doctrines did he actually teach, and were these doctrines original
with him? But would any answers to these questions establish whether Jesus is or was God?
At best they would tell us that Jesus said he was God, that be acted accordingly, and that some
people believed him.

But could any information gleaned from human history tell us if Jesus actually was... God?
Let us assume that a crucial document is found, the diary of Jesus of Nazareth. Let us suppose

Volume 1990 - 2002 Issue


Page 1132 of 2049
Skeptical Review Edited by Farrell Till
that it can be authenticated. This certainly would then be a more interesting document for
finding out the message of Christianity than the accounts of Matthew, Mark, Luke, John, and
Paul. But no matter what claims appear in the diary, they will not establish that Jesus is or was
God, but only that he wrote certain claims. The historical information will not confirm or
deny what is essentially not visible in human history. One can find the historical data that may
or may not be the results of divine affairs, but to know that it is or is not related to God is
more than the data can tell us.

Bible critics have shown the extraordinary amount of information one can gain about a text by
employing all sorts of scientific and historical analyses, but in so doing, they have raised the
problem of whether one can find the Bible's revelatory content by means of historical and
scientific study.

Not all Biblical scholars have come to such negative or skeptical conclusions. One present-
day scholar, John Warwick Montgomery (see Where is History Going? 1969), has tried to
combat the results issuing from the works of Bible critics. Montgomery bases his
counterattack on taking the Bible at face value, and insisting there are no good reasons to
doubt the claims and to doubt the eyewitness accounts. In the case of Jesus, one has several
independent accounts of the same events. There is no reason to believe that all of the Biblical
writers would lie. (Montgomery points out that it is standards like this that lead us to accept
any historical account, religious or secular.) Regarding an event like Jesus's death, there were
hundreds of eyewitnesses (according to the Bible). Similarly, the miracles of feeding the
flocks with a few loaves of bread were witnessed by multitudes. So if these miracles were
seen by reliable witnesses, claim Montgomery and the English theologian and science fiction
writer C. S. Lewis, we then have direct factual knowledge of the supernatural events recorded
in the Bible.

Skeptics view this as an overcredulous acceptance of all claims in this ancient text as being
true accounts. It would be as if a historian were to read in an ancient text an account of how a
certain ancient prince slew a dragon with his pocket knife and concluded from this that
dragons really existed.

Biblical criticism has raised a fundamental problem for the Christian believer. If the Bible is
examined as a historical document, then the central question arises whether one can ascertain
what, if anything, in the Bible contains revealed religious knowledge. Some have concluded
that since the Bible is a historical document, it cannot also have a privileged status of
containing information that goes beyond human history. One result of this view has been to
denigrate the Bible to being just a compilation of documents of the early Hebrews and early
Christians, having no particular import about the nature of God. Others have rejected the
results of Biblical criticism, either denying the claims of Bible scholars or insisting that the
historical interpretation does not deny that there is another dimension to Biblical material,
namely its revelatory content.

As Bible scholars have accumulated more and more information about the historical context
of the Biblical documents, the question of delineating what is the revealed information has
become a central issue within the community of Christian believers, and has led to a wide
variety of interpretations of Judaism and Christianity, ranging from a denial that either of

Volume 1990 - 2002 Issue


Page 1133 of 2049
Skeptical Review Edited by Farrell Till
them contains any special religious knowledge, to modern presentations reevaluating these
religions in terms of modern scholarship, to fundamental rejections of the findings of the
Bible critics. Another view was offered by the Danish philosopher of religion Soren
Kierkegaard (1813-1855). He concluded that the basis for religion could never be found in
history but could only come from faith. If one had faith, then certain historical materials
would have religious significance. This has led to the development of fideism, i. e., a
complete reliance on faith and rejection of science and philosophy in religious matters.

The development of Bible criticism and the findings and theories of modern science have thus
raised grave difficulties for Christian believers. I was reminded of this upon the reissuance of
one of the best books on this problem: The Historian & the Believer: The Morality of
Historical Knowledge and Christian Belief (Westminster Press, 1966). I recommend it to
readers wanting to research the subject.

(Robert Lockwood, 3955 Bigelow Blvd., Apt. 106, Pittsburgh, PA 15231; e-mail
RLock81626@aol.com)

If Dr. Price Denies Zeus, He's Irrational


William Sierichs, Jr.
Dr. James D. Price's arguments in the September/October 1997 issue of The Skeptical Review
have convinced me that the anti-supernatural bias of skeptics is wrong. I hereby renounce my
former irrational anti-supernaturalism and take up the cause of the divine powers that created
and govern this universe: the gods of Mount Olympus.

Although he does not say it, I suspect Dr. Price is one of those atheists who deny the existence
and power of Zeus, the King of the Gods; Hera, the Queen of Heaven; Poseidon, Earth-
Shaker; Herakles, the son of Zeus and the mortal virgin Alcmena, who was resurrected after
his death to save the gods and humanity from evil; and the other deities who have done so
much for us all. "The glory that was Greece and the grandeur that was Rome" are a clear
testament to the gifts from Olympus.

So I'm ready to debate Dr. Price on the proposition: Are the gods of Mount Olympus our
creators and guardians? We'll debate under the rules of logic he lays out in his article. I will
present positive proof for my thesis, and, as the opponent, he "must present objective
evidence to disprove it."

I have no doubt of my victory. Dr. Price himself admits that "(m)any unlikely events are
recorded in ancient records the validity of which historians accept on the basis of good (but
not extraordinary) evidence."

Volume 1990 - 2002 Issue


Page 1134 of 2049
Skeptical Review Edited by Farrell Till
Thus, we have the many fulfilled prophecies found in Homer, acknowledged even by many of
those Zeusless atheists called Christians to be one of the greatest poets of all time. Homer
records the prophecies of the deaths of Hector and Achilles and the ruin of Troy, plus the
many troubles that beset Odysseus and crew. Furthermore, the contemporary 13th-century B.
C. E. records of the Hittite empire provide independent confirmation of the existence of Troy,
its war with the Greeks, and of Paris of Troy (also known as Alexander), the lover of Helen,
who signed a treaty under the name Alexander with the Hittite King Muwatallis.

Dr. Price cannot deny this undeniable evidence for a fulfilled prophecy, which is more than
those atheists called Christians can present for their god Jesus. We have no independent,
contemporary proof of his existence, only the later, noncontemporary Christian texts with
their contradictory statements about the alleged events of his life.

Beyond Homer is the acknowledged genius of Greek civilization, and its later Roman
imitator. Christianity, by contrast, presided over the collapse of Roman civilization and the
descent of the western world into what is now called the Dark Ages. The West certainly paid
a terrible price for abandoning the Olympian gods. Only the revival of the literature and
philosophy of the Greco-Roman civilization, during what is properly called the Renaissance,
pulled the western world out of its malaise. The gods of Mount Olympus gave us back
civilization when we began paying attention to them again. What's Jesus done for us lately?

I will note, as an indirect proof, that Fox TV honors the gods of Olympus with a popular show
(Hercules) about the dying-and-resurrected son of Zeus, at the same time that the United
States is undergoing its biggest economic and scientific boom ever. Coincidence? Or a gift
from Olympus? Considering what the Olympian gods did for Greece and Rome, I will argue
the latter.

Dr. Price must agree with me on this, since he says (using an incorrect singular title to refer to
the Olympian gods), "To those who believe in an omniscient God, the idea of fulfilled
prophecy attributed to Him is not a fabulous claim, but what one would reasonably expect."
As Farrell Till points out, no evidence exists for any prophecy in the Bible save the statements
in the Bible. Homer has the Hittite records to support him as well as statements by later
writers attesting to his authorship and verifying his authenticity.

I know Dr. Price will agree because he points out that "(i)n a court of law and the canons of
historical research the authorship of an ancient document is determined by the internal claims
of authorship and by internal and external witnesses to that authorship." I can find more
references to Homer, Troy, and the prophecies regarding Troy among Greek writers than he
can find for Jeremiah in biblical sources and Josephus, the Mishnah, and Talmud. All ancient
witnesses agree Homer wrote the poems, and there are no dissenting witnesses, to paraphrase
Dr. Price.

Other contemporary historical records support the fact that the Olympian gods told Croesus of
Lydia that if he attacked the Persians, an empire would collapse. He launched his invasion,
and an empire collapsed in 547 B. C. E.-- ironically, his own, at the hands of Cyrus of Persia.
We have a contemporary, independent record of Cyrus' invasion in the Babylonian Chronicle,
and the 5th century B. C. E. Greek historian Herodotus passes on the prophecies Croesus was

Volume 1990 - 2002 Issue


Page 1135 of 2049
Skeptical Review Edited by Farrell Till
given, which Herodotus researched in contemporary 6th-century B. C. E. records. That's a
more certain dating of the prophecies than can be provided for Jeremiah, leaving aside the fact
that as Jeremiah did not claim his prophecy came from the Olympian gods, it automatically
must be a false prophecy.

The gods of Olympus very clearly spoke, and their prophecy was accurate. And the gods also
prophesied Rome would become a great power. We know from our history books how that
prophecy turned out.

Indeed, the conquests of the Greek and Roman generals were greater than any Christians ever
achieved. Alexander the Great, who was directly counseled by Zeus, conquered an area far
greater than the Christian Crusaders did. His successors held the area he conquered, and even
expanded it in Asia, until displaced by the Romans, who acknowledged that their gods were
merely the gods of Olympus under other names. No Christian empire achieved the size of the
Roman realm. (England and Spain built their larger empires only after the Renaissance return
to the Olympian gods.)

The evidence that Zeus favored Alexander, that Troy was doomed, that Croesus of Lydia was
doomed, and that Rome was divinely favored is far more massive, diverse, and objectively
verifiable than most of the material in the Bible.

How can Dr. Price deny this plain evidence?

Even those atheists called Christians indirectly admit the truth of what I say, since many of
their so-called Bible miracles are merely copied from the true stories of the gods of Olympus.
Jesus's death and resurrection, as well as the alleged ascension of his mother to heaven, are
copied from the stories of Herakles and his mother, who was taken to Mount Olympus with
her son, according to the Greek scriptures.

Jesus's water-to-wine miracle imitates Dionysius' real miracle, attested by eyewitnesses.


Jesus's healing of a blind man imitates the miracle in Alexandria that the Olympians (acting
under the local name of Serapis) performed in A. D. 69 for the Roman Emperor Vespasian,
who also made a crippled man walk. Vespasian did his gods-given miracles in front of a large
audience, as reported by the Roman historian Suetonius in The Twelve Caesars. Although the
Christian gospels contain similar stories about Jesus, they were not written until after
Vespasian had performed his miracles. The plagiarism is obvious.

Jesus's raising of dead people and other healings are similar imitations, and the visions of the
Virgin Mary are obviously stolen from the Greeks' reports of real visions of their gods.

Furthermore, polytheism is clearly more rational than monotheism. The many problems of
this world are best explained by conflicts among multiple supernatural entities. Even those
Zeus-less atheists called Christians admit this indirectly, since they claim that their "one god"
is really three gods, and that he's at war with an evil god, and that both their three gods and
the evil god command lots of lesser gods, known as angels and demons, and that a goddess
named Mary helps out the trio. Just as Zeus sometimes sent Hermes to carry messages to
mortals for him, so Christians claim their gods send messages to mortals via his angels.

Volume 1990 - 2002 Issue


Page 1136 of 2049
Skeptical Review Edited by Farrell Till
Yeah, sure. Accept no imitations. Only the real thing-- straight from Olympus--speaks to this
world.

Well, as Dr. Price himself states, "I am willing to accept the historicity of reported
supernatural events if they are verifiable, regardless of whether the source is the Bible, secular
history, or a non-Christian religious historian."

So I prophesy, under the inspiration of Apollo, the god of reason, that by the end of our
debate, Dr. Price will admit that Christianity is a blasphemous form of atheism, and he will
come to Zeus, begging forgiveness for his transgressions against the divine laws.

I didn't really need help from Apollo to reach this conclusion. Based on Dr. Price's arguments,
the gods of Mount Olympus are undeniable.

(William Sierichs, Jr., 316 Apartment Court Drive, Apt. 44, Baton Rouge, LA 70806)

From the Mailbag


Renouncing Jewish History...

Your response to John Coffin's request for biblical holocausts was both enlightening and
disturbing. You wonder "why the horrible atrocities that the Jews experienced in Europe in
the 30s and 40s do not cause them to renounce their own genocidal history." We can agree
that the 17.5 million that you calculated constituting the seven nations living in Canaan is
undoubtedly exaggerated. Such a large population in a small area 3,300 years ago is
unimaginable. Also, no rational person believes that Joshua "utterly destroyed" them as
described in the Bible. Under instruction from Yahweh, Joshua and the Israelites did kill men,
women, and children in city after city. When one kills after capturing a city, as was done, the
killing constitutes genocide, a flagrant violation of Yahweh's sixth commandment against
murder. This is one example of many where people act upon the belief that God is speaking to
them. The Christian crusades and inquisition, shooting doctors at abortion clinics, and the
Islamic concept of jihad (holy war) are some others.

This kind of fundamentalist thinking and the consequent militant impassioned action for one's
God can be traced back to Numbers 25, where Phinehas, the grandson of Aaron, upon seeing
an Israelite man and a Midianite woman engaging in sexual intercourse, took a spear and
stabbed both of them through their genitals. For this, Yahweh rewarded Phinehas with a pact
of [everlasting] priesthood.

As a secular humanistic Jew I find the atrocities in the Hebrew Bible a total embarrassment.
Since I do not attribute the actions to God's commands but rather to what people thought God
commanded, I can only blame those who committed the crimes "in God's name." John Salvi
III was not forgiven for shooting at an abortion clinic because he believed God instructed him.

Volume 1990 - 2002 Issue


Page 1137 of 2049
Skeptical Review Edited by Farrell Till
Baruch Goldstein cannot be forgiven for murdering Muslims praying in a mosque because he
believed God instructed him. Neither can anyone in the Bible or not in the Bible use such a
defense. The consequent results of such fundamentalist thinking are one of the most
frightening and dangerous aspects of our society today.

In answer to your challenging question, I hereby renounce all genocidal aspects of Jewish
history as unacceptable and challenge other Jews to do the same. I also challenge all peoples
of the earth to likewise renounce all genocidal aspects of their particular histories.

(Edward J. Klein, Attorney at Law, 179-54 80th Road, Jamaica Estates, NY 11432-1402.)

EDITOR'S NOTE: Religion in and of itself seems to foster intolerance toward those who are
not "believers." Sikhs and Hindus can't get along, and both persecute Zoroastrians. Muslims
fight their jihads, which are usually directed against non-Muslims. The Bible-based religions,
however, have a particularly violent history of genocide, inquisitions, crusades, and witch
hunts. Although no scientific study of its causes has been made or is even possible to make,
one has to wonder if the violent history of the Judeo-Christian religions is rooted in the
Bible's depiction of their God. After all, when believers in Yahweh read in his "holy word"
that he is a jealous god, who commanded the utter destruction of nations who stood in the
way of his "chosen people" and the execution of all who worshiped other gods (Dt. 13:6- 10;
17:2-7), why would they not think that in massacring heretics, witches, and infidels, they were
doing the will of the god they worship? One of the more puzzling aspects of Judeo-Christian
beliefs is that Bible fundamentalists who are shocked by modern-day holocausts and
atrocities will nevertheless wink at the Yahwistic massacres of the Bible and bend over
backwards to justify them with such tortured excuses as, "God's ways are higher than ours,"
"God had a plan that required it," "The babies and children who were massacred went to
heaven," etc., etc., etc. They tolerate in their god conduct that they would despise in their
neighbors and national leaders.

Taking Issue with Till & Hutchinson....

Having just gotten the July/August issue in the mail yesterday (Thanks for mailing it out
specially, Till) I take keyboard in lap and prepare myself to write critically about what I read
within.

No matter how much I like Till, or how right he usually is, I must foremost take exception to
the discussion on Page 14 re: "More about Converted Atheists..." Till has ranted in the past
that some Christians say of other Christians---particularly ones who converted to another faith
or to atheism---that he "wasn't a real Christian." Now I read Till saying the same thing: They
were not "committed atheists." Till's whole reply reads just like a Christian explaining how no
real Christian has ever converted to atheism.

I guess I don't understand even the point of trying to establish that no "committed atheists"
have ever reverted/converted to Christianity. So what? If nobody ever converted from
Christianity to atheism, I might argue that it was merely because Christians were so closed-
minded that such a thing was impossible, but that wouldn't really do anything to prove

Volume 1990 - 2002 Issue


Page 1138 of 2049
Skeptical Review Edited by Farrell Till
atheism. Similarly, a Christian could point at the lack of atheist converts and say that it's
because we're too closed-minded about the idea of God or of "the supernatural."

Furthermore, when Till says he's never heard of these guys who were said to be atheists but
are now Christians, I don't know that he had ever heard of Dan Barker the Christian before he
had read/ knew/heard of Dan Barker the former-Christian atheist. I certainly hadn't, and even
today I tend to think of Bob Barker (The Price is Right gentleman, I believe) before I think of
Dan.

So, Till, how is your dismissal of those who have converted from atheism to theism as not
"committed atheists" different from Christians saying of others that they aren't or weren't real
Christians? Even if there is some relevant difference, what does this demonstrate about the
superiority of the atheist position?

Aside from this quibble---but what is left in the debate over errancy but quibbles---I found the
issue as good as I have learned to expect from the year and a half I've been a subscriber.
Particularly off target was Roger Hutchinson's article, and his suggestion that
Adam:humanity: agent:player [was] particularly laughable. I would certainly not have chosen
him for my agent, but I had no choice in the matter.

One might make the claim that David's son died of "natural causes," which were incorrectly
understood by a primitive people to be an "act of god," and while this reply gets one out of the
woods on the child-punishment issue, it practically admits biblical errancy. I doubt that the
inerrantist readers of TSR would wish to do this.

One thing I itch to see in TSR is a noninerrantist Christian defend Christianity in light of an
errant Bible. I agree with Theodore Drange when he said of his Argument from the Bible (an
argument against Christianity from the errancy of the Bible) that some might say he was
knocking down a straw man, but that "I believe there are millions of such `strawpeople' out
there." If it could be shown that the establishment of biblical errancy was not such a blow to
Christianity everywhere, then things are great for Christians, they needn't worry about this or
that contradiction, and they wouldn't appear so foolish when they dismiss as unimportant to
their faith what TSR says.

Just what is it that keeps still more millions of people from losing their faith in the face of an
errant bible? Is it the claim that they were inerrant in the original autographs (but god stopped
caring after that)? Is it due to being taught happy-happy/joy-joy versions of Christianity that
are so pleasant that questioning them is like taking a disgusting-tasting medicine? Is it
because of a personal religious experience? Or is it just a matter of being in a really deep rut?

(Jeff Epler, 2332 NW 51, Lincoln, NE 68524; e-mail, jepler@inetnebr.com)

EDITOR'S NOTE: Drange's argument against Christianity can be accessed at


<URL:http://www.infidels.org/library/modern/theodore_drange/bible.html>. I will reserve
comment on Jeff Epler's letter until after the one below from Antonio Casao Ibanez, whose
previous letters initiated the discussion about converted atheists.

Volume 1990 - 2002 Issue


Page 1139 of 2049
Skeptical Review Edited by Farrell Till

More about Converted Atheists...

Thank you for your comments to my letter in the July/August issue. Yes, I understand the
differences between English and Spanish systems, so I give no importance to your mistake
with my surname. Regretfully, I make much more mistakes in my letters.

You know that I appreciate your good work as editor of the Review, a publication which I feel
interesting and well-documented in general. I told [you] so on renewing my subscription for
two years and in my first letter of this discussion.

Of course, I am sure you are being true on stating that you know of no committed atheists
who have converted to Christianism, and I have never doubted about it. If I listed the names
of famous converted atheists, [it] was to reply [to] Mr. Ulm's question: "Have any TSR
readers... ever known an intelligent, well read, and committed atheist who became a devout
Christian?" As a TSR reader, I thought I could add something to the knowledge of facts
related to Mr. Ulm's question. My surprise and my deception came [when] I read that I could
be "trying to make an issue where none exists." Probably my troubles with [the] English
language, which force me quite often to use a direct, almost aggressive style could give this
impression. If this was the case, I very much regret it and apologize for it.

Coming to my list, I feel that, at least, Joseph Joubert, a philosopher, disciple and collaborator
of Diderot, and Henry Gheon, a Nietzschean philosopher, seem good examples of committed
atheists, even in your restrictive exception of "committed." The first, Joubert, had written with
regard to Saint Sulpice Church in Paris: "In temples like this one, men become religious and
[en]slaved; before these altars, they initiate themselves into servitude, because every religious
feeling is servile." Maybe their names do not appear in freethought and atheist literature, but I
fear that, similarly, we are not to find in a history of Christian thought names of committed
Christians who have become atheists.

I hope you will now acknowledge receipt of the reference sources about Voltaire's repentance,
which I offered in my previous letter. I can add to it that the confession of faith signed by
Voltaire was published in Volume 12 of Correspondance Littteraire, Philosophique et
Critique, an old French review, published by Grimm, Diderot, and other encyclopedists. It
appeared in pages 87 and 88 of the April 1778 issue. Besides, they reproduced two further
documents in the June 1778 issue, proving that the confession was authentic.

Many thanks for your explanations about Paine. If I called him the patron saint of some
American unbelievers is because they treat [him] this way in their literature. For instance, [in]
Freethought Today, March 1995, we can read two articles, "Let's Revive Thomas Paine
Birthday Memorials" and "Celebrate Paine's Age of Reason," which I think confirm my
opinion. On my part, I admire Paine as a fighter for human rights, although I disagree with
him in some of his attacks on organized religions.

(Antonio Casao Ibanez, Apartado 882, 50080 Zaragoza, Spain.)

Volume 1990 - 2002 Issue


Page 1140 of 2049
Skeptical Review Edited by Farrell Till
EDITOR'S NOTE: The last thing Mr. Casao should feel compelled to do is apologize for his
English. The syntax is a bit off in places, but otherwise he has a good command of English
grammar. I suspect that many TSR readers wish that their command of Spanish were half as
good.

I'm glad he now understands that I was only answering Mr. Ulm's question, which was
whether I knew of any committed atheists who had ever converted to Christianity. I don't
know of any, and so I answered accordingly. I was not trying to argue, as Jeff Epler seemed
to think, that no committed atheists had ever converted to Christianity. I hope I am more
rational than to try to argue something that I couldn't possibly know.

I do know Christians who claim that they were once atheists, but in talking to them, I formed
the opinion that they had been unchurched rather than atheists. I found none of them who
seemed to know much about responses that have been made to the traditional theistic
arguments. Mr. Casao has reached back into the 18th century for some examples of what he
considers committed atheists who converted to Christianity. As far as I know, he may be right
about them, because I am not familiar with the lives of either Joubert or Gheon. Mr. Casao
identified them both as philosophers, so I will make a cynical comment about whether
philosophers should be considered committed to anything except their philosophical ideas.
Whenever I read philosophical works, I sometimes find myself wondering if even the writers
know what they're trying to say. I'm willing, however, to concede that Joubert and Gheon
were atheists who converted, but as Jeff Epler pointed out, what does it matter? I didn't know
either one of them, and so I can still say that I have never known a committed atheist who
converted to Christianity.

Before making a decision in the matter of Voltaire, I would want to see the documents that
Mr. Casao referred to, because tales of deathbed conversions almost always circulate after
famous atheists and skeptics die. They inevitably turn out to have no substance to them.
Anyway, I'm still confused about why Mr. Casao keeps wagging Voltaire into this dispute,
because he argued at length in his first letter that Voltaire was never an atheist. If he wasn't
an atheist, then Voltaire would not be an example of an atheist who converted to Christianity,
no matter what may have happened when he was on his deathbed.

A Former Pastor...

Within the past few months, I've been sitting in my study, reading, thinking, and absorbing
material found on the internet. Presently, I'm on infidels.org and I have been left speechless.
Night by night, I read the material presented, and I find myself digging out all the unanswered
questions I had put upon the "mental shelf," realizing that someday I would begin to pull them
down, one by one, seeking answers to all my unanswered questions.

Why am I writing this? Well, I'm a former pastor, who preached his last sermon in July of this
year. I have been keeping track of the "theological contradictions" contained within scripture
hoping that someday I will be able to share my "notes" with those on the internet. The nature
of "Jehovah God" has always been puzzling at best. Even the kids I taught in Confirmation
(Lutheran) would point out God's "paranoid-schitzo" qualities that included a heavy hand in

Volume 1990 - 2002 Issue


Page 1141 of 2049
Skeptical Review Edited by Farrell Till
murder. I nearly drove myself crazy trying to bring a reconciliation of the two natures
possessed by God and His Son Jesus (God).

I no longer can. I no longer have "all the answers" that I once possessed as a "Christian
Apologist" (note email address). The mental hold that "Christianity" has upon my soul runs
deep--I often wonder if I'll ever escape its guilt, fear, and shaming qualities as it lays a
question mark across my eternal destination. Now, I find the internet to be a very useful tool,
in finding material that I can read, and resources that can help me see through problems that I
cannot escape on my own. I found your ad for a free one-year subscription to The Skeptical
Review. Please, if you think this resource can help me, could you put me on your list? Also,
on the issue of "Biblical Inerrancy," can you recommend a very detailed book that explores
the nature of the "Biblical God?"

It's hard for me to request such a thing. I and my wife have spent hours and hours discussing
this issue. After requesting a removal from the clergy roster, I found out how many friends I
have left in this world. None! You wouldn't believe the treatment I've received from the
brethren, and you would really be amazed at how denominational heads react. I could just as
well have an "H" tattooed upon my forehead for "heretic" or the proverbial "A" for apostate.

Thanks for hearing me out!!

(Douglas R. Larson, 20861 467th Avenue, Brookings, SD 57006; e-mail


berean@brookings.net)

EDITOR'S NOTE: Mr. Larson was, of course, added to the subscription list. He has also
joined errancy@infidels.org and is making some valuable contributions to the discussion of
inerrancy issues. The mental anguish and turmoil that I went through when I quit the ministry
is now 34 years into my past, so Mr. Larson has brought fresh insights into the inner conflict
and other problems that a minister experiences when he can no longer practice the profession
that he entered with youthful optimism and idealism. I know all too well what he means by the
loss of friendship that accompanied his decision. The New Testament teaches Christians to
love their neighbors, but they don't have much love for preachers whose doubts force them to
resign from the ministry.

Readers with e-mail may be interested in joining the Errancy list. To do so, just send
"subscribe errancy" in the message window to major-ii@infidels.org.

Tired of Price's Droning...

As always, your September/October issue was outstanding, but you really outdid yourself
with the lead article on the Bible's attitude toward women. It was long overdue in TSR, and
you did a marvelous job with it. I'm looking forward to the second part.

I must say, however, that I'm tiring of Dr. Price's droning. Your responses have been quite
concise and revealing. He just doesn't seem to get it, though.

Volume 1990 - 2002 Issue


Page 1142 of 2049
Skeptical Review Edited by Farrell Till
My subscription expires with 6-97, so here's an extension. I also enclose three HD formatted
disks and add another $1 in my check for ASCII copies of TSR. Hopefully, this compilation is
up-to-date with the most recent issue, as I want to forward your article on women to a couple
of friends via e-mail.

Keep up the fantastic work. Like most of your readers, I devour each issue as soon as it
arrives.

(Dave Howard, 38281 Mountain Home Drive, Lebanon, OR 97355-9367)

EDITOR'S NOTE: The compilation was up-to-date, and thanks to Jeff Epler, who has been
doing the HTMLizing, the web site is also up-to-date for the first time in almost two years.
Those who want to access the web site to find past articles published in TSR may do so at
http://www.infidels.org/library/magazines/ tsr/. Those who want the articles on disk should
send three HD DOS formatted disks and $1 for postage to our address on the backside of this
issue.

A Good Piece of Christian Desperation...

I am glad as always to receive your publication. I thought that I would get a head start and
renew my subscription for another two years. Enclosed is a check for $12. I also would like to
submit the address listed below for a one-year trial subscription. (Name omitted.)

I was delighted to read the prophecy defense article by Dr. Price. It is the best piece of
Christian desperation I have seen in quite a while. By his understanding of the burden of
proof, the earth must be flat if I am not able to discredit completely the flat-earth theory to the
satisfaction of a flat-earth believer. I grew tired of hearing anti-supernatural bias. I have only
one question for Dr. Price that would cut through all of the smoke and mirrors that he
employs to rationalize his viewpoint. Is there even so much as one prophecy/fulfillment
scenario in the Bible that Price would possibly consider erroneous? I suspect his answer is
that there is not. If that is so, then what kind of position would that be for a man who claims
to advocate "healthy skepticism"? Could Dr. Price have an anti-errancy bias?

Thank you again for the work that you do. It is needed now more than ever.

(Chad D. Kelly, P. O. Box 183, Greenwell Springs, LA 70739-0183)

EDITOR'S NOTE: I not only suspect that Dr. Price would say that there are no erroneous
prophecy-fulfillment claims in the Bible; I'm very sure that this is his belief. He couldn't retain
his position at a Baptist seminary unless he subscribed without reservation to the doctrine of
biblical inerrancy. The height of inconsistency is for someone like Price to yell "anti-
supernatural bias" or "radical skepticism" at us when he uncritically accepts the inerrancy of
everything recorded in the Bible, no matter how fantastic it is.

Is Hutchinson Being Honest?

Volume 1990 - 2002 Issue


Page 1143 of 2049
Skeptical Review Edited by Farrell Till
You have done an excellent job of responding to Roger Hutchinson's ridiculously weak
defense of the idea that the biblical God does not punish sons for the sins of their fathers
("The Sins of the Fathers: Another View," TSR, July/August 1997).

I simply cannot believe that Hutchinson is being intellectually honest on this topic. As you
noted, the Bible clearly features stories of innocent children who were punished for the sins of
their fathers, and perhaps the most well-known example is the killing of all the firstborn in
Egypt (Exodus 11:5). Is Hutchinson willing to argue that these children were not deliberately
killed by God only because of the sins of their fathers? What other "reason" could there have
been?

There are other problems with Hutchinson's piece. He writes that "God will allow a father to
teach his children to disobey His commandments..." until the third or fourth generation,
whereupon God will punish the offending children for their sins, but this is absurd. First, it
does not make sense to punish children for following the ways of their parents. Indeed, it is
only natural for children to follow the ways of their parents. After all, how are they going to
follow God if they have not been properly introduced to him?

Second, why does God wait until the third or fourth generation to handle his business? To
paraphrase Robert G. Ingersoll, would some kindly Christian genius please come forward and
explain this to me? It just seems disingenuous of Hutchinson to suggest that those who realize
"that the Bible is saying that God will punish the children of the third or fourth generation for
the sins of the fathers" are reading the Bible superficially. The Bible says what it says, no
matter how disgusted or embarrassed Hutchinson might be by its ideas.

Finally, Hutchinson asserts that "Adam acted as mankind's agent, so to speak." Thus, original
sin was imputed to humanity. But no human being elected Adam as his or her agent. God
appointed that idiot as our representative, and we have been cursed ever since! This is just
another excellent example of God's unjust, undemocratic, and utterly irrational ways of
dealing with human beings. When are fundamentalists going to admit that the Bible simply
does not stand up to critical examination? "God only knows!"

(Norm R. Allen, Jr., Executive Director, African Americans for Humanism, P. O. Box 664,
Buffalo, NY 14226)

EDITOR'S NOTE: At times, it is hard to believe in the intellectual honesty of Bible


inerrantists who peddle the kind of nonsense we have seen in Roger Hutchinson's articles, yet
as a former preacher, I know that biblical fundamentalism can warp one's ability to think
rationally. The situation, however, is not hopeless. I eventually recognized the stupidity in the
Bible inerrancy doctrine, and so did Dan Barker, another former preacher whose article on
biblical fables was published in the issue before this one. On pages 13-14 of this issue is a
letter from Douglas Larson, who just recently resigned from the Lutheran ministry.

Whether Roger Hutchinson will ever see the folly of his position certainly looks doubtful at
this point. Readers may have noticed that he is back again with an article on pages 4-5 of this
issue. Another opinion about his "Sins of the Fathers" articles appears directly below.

Volume 1990 - 2002 Issue


Page 1144 of 2049
Skeptical Review Edited by Farrell Till

Hutchinson's Selective Citations...

Roger Hutchinson's selective scripture citations are so typical of bibliolatry tactics. In this
case, it's the idea that god is totally fair--always has been--and will not-- never did--punish
descendants for the sins of their ancestors ("The Sins of the Fathers: Another View," TSR,
July/ August 1997). The title of his article actually reveals the truth of the whole matter. It's
another view, a view that is diametrically opposed to the many scriptures that are
unquestionably literal (and need no inventive, subjective "interpretation" to explain "what it
really means") and teach that god does indeed punish the children for the sins of the fathers.

Mr. Hutchinson and his ilk quote scriptures favorable to their "view," and ignore or interpret
away scriptures that contradict, and use false, irrelevant analogies to state their case. These
practices reveal that bibliolaters either have retarded, depraved thought processes or have
cunning, deceitful and deviated natures. It is almost impossible for religionists to be objective
concerning their "matters of faith," since they have so much of their self-worth, self-
validation, self-importance, self-love, self-acceptance, and self-preservation (eternal life)
invested in it. To admit they're wrong about their religious beliefs or even to be unsure about
them would be like committing spiritual suicide. I know it's hard because I used to be a
fundamentalist of the Pentecostal variety. Fortunately for me, I'm not a fantasy-prone
personality, so my religious experience was uneventful, boring and without tongues, signs,
wonders, miracles, etc. Only my intense Bible study with its resultant "revelations" and
"insights" kept me believing that god was "in my life" and "teaching me for a special purpose,
when he will use me in his timing," etc., but with reason, "all things are possible"--well,
maybe not all things. So it was easier for me to renounce superstition completely than for the
poor self-deluded/clergy-deluded religionist who has a history of "supernatural experiences"
between his ears.

As for Mr. Hutchinson's article, I'll let his god Jesus settle the issue. Jesus is allegedly god and
therefore "inspired" the writers of the whole Bible, which would make him omniscient and
intimately acquainted with its contents and concepts. In Matthew 23:31-35, he said,
"Therefore you [scribes & Pharisees] are witnesses against yourselves that you are sons of
those who murdered the prophets. Fill up, then, the measure of your fathers' guilt. Serpents,
brood of vipers! How can you escape the condemnation of hell? Therefore, indeed, I send you
prophets, wise men, and scribes: some of them you will kill and crucify, and some of them
you will scourge in your synagogues and persecute from city to city, that on you may come all
the righteous blood of Zechariah, son of Berechiah, whom you murdered between the temple
and the altar" (NKJV). Verses 34 and 35 should be read together with no break, because
Jesus was saying that his motive for sending the prophets to the Pharisees was for them to be
killed by the Pharisees in order to increase ("fill up") the guilt of murder and bloodshed to a
point that the Pharisees could finally be guilty, judged, convicted, and punished for all the
righteous blood shed on the earth.

Wow! So Jesus admitted to conspiring to send his pawn servants (prophets, wise men, and
scribes) to their sure murder just to increase the Pharisees' guilt/punishment to a hyperbolic
degree! How sadistic! Unnecessary too, because if, as Jesus said, the Pharisees were already
guilty and condemned to hell for the murder of past prophets, then why increase their guilt,

Volume 1990 - 2002 Issue


Page 1145 of 2049
Skeptical Review Edited by Farrell Till
which wouldn't increase their punishment? Hell is hell, or would a religionist invent "levels of
punishment in hell," red hot, white hot, blue hot, whatever?

The answer to the whole debate is right in these verses above. An excuse of this being "before
the cross" and the "age of grace" while the Jews were still "under the law" won't work either,
since the whole debate is over whether or not god punishes descendants for their ancestors'
sins. To say that at one time god did, but later he didn't, and sometimes (as in Matthew) he
does for special reasons means that god does not have absolute moral values and that "he"
does have "variableness" and "shadow(s) of turning" (James 1:17).

(Jeff Schmura, 808 Apple Lane, Shoemakersville, PA 19555)

Hutchinson's Audacity...

Alas! Roger Hutchinson's recent article (TSR, 11/12 1997) is another example of our Bible-
believing, guest authors not doing their homework. If they would but study the principles of
good reasoning expounded upon in the past issues of TSR, materials developed chiefly by Till
and me, a great deal of commentary could be skipped. TSR is on the Internet, leaving precious
little excuse.

Mr. Hutchinson begins by assuming that the Bible is correct in all its historical details. This is
inappropriate, because TSR is dedicated to proving that the Bible contains numerous errors. If
one wishes to meaningfully challenge Till's ongoing study, it cannot be on the basis of
assuming, a priori, what the conclusion must be! Mr. Hutchinson's attitude seems to be, "I
believe that the Bible is right, which leads to my belief in the biblical god as well as the
miracles in the Bible. Prove me wrong!"

If that is his personal worldview, then so be it. However, if he wishes to convince the rest of
us that he has something, then the burden of proof is his. His failure to fully grasp that
concept has led (I suspect) to his faulty presentation in no small way. Furthermore, it is not
enough to advance an idea that may possibly be true; you must show that it is the best
argument in order to win your case. Thus, Hutchinson's failure is also tied to a
misunderstanding of the concept of "absolute proof and error."

The above two principles, and the principle of needing extraordinary proof for extraordinary
claims, answer virtually all of Mr. Hutchinson's points. A little study of past TSR issues could
have saved a lot of ink! For instance, he seems puzzled that skeptics reject the Gospels while
accepting the account of Hitler's atrocities against the Jews. But the two are hardly equal!
Strong evidence says that the Gospels are not first-hand accounts. Rather, Jesus' "history"
appears to have been largely invented by the church to support certain doctrinal views. It
seems to have been manufactured from the Old Testament, the concept of a savior god, the
concept of a sun god, astrology, and other sources. The extraordinary claims for Jesus not
only lack extraordinary proof, but they run counter to the best conclusions!

Compare that to the numerous, firsthand witnesses to Hitler's atrocities (including former
Soviet and U.S. soldiers who liberated those camps). Add to that the surviving German

Volume 1990 - 2002 Issue


Page 1146 of 2049
Skeptical Review Edited by Farrell Till
records, the massive photographic documentation, and the other physical evidence. What a
difference! Thus, the skeptic rejects the first and accepts the second.

Mr. Hutchinson seems to think that the skeptic cannot reject miraculous claims without
rejecting the associated god. Again, just apply the principle of extraordinary proof for
extraordinary claims. Such miracles are extraordinary claims lacking extraordinary proof.

Had Mr. Hutchinson done his homework, he would not need to ask, "What are these
`standards,' and do skeptics apply them reasonably and logically with respect to the Bible?"
Nor would he have found fault with our current state of knowledge. (The skeptic doesn't claim
to be 100% certain, only that he has the best argument, that it is likely to prevail in one form
or another. Check out the concept of "absolute proof and error.")

At one point Mr. Hutchinson fears that our principles of reasoning will lead us to reject true
information. Mr. Till nicely addresses that point: "I would much rather err on the side of
caution than gullibility." Erroneously rejecting some truths, usually for a time, is the price
paid to insure that our stock of knowledge is sound. The alternative, that of accepting
everything until proven otherwise, is to lose any hope of developing a sound stock of
knowledge.

Mr. Hutchinson accuses us of prejudice in accepting macro-evolution while rejecting most of


the biblical accounts. Apparently, he has forgotten (or never understood) the solid ground
beneath macro-evolution. The order of the fossil record, the structure of certain biological
molecules, and comparative anatomy not only scream "macro-evolution" but independently
yield similar evolutionary sequences. Add the other major lines of evidence, such as the
distribution of plants and animals worldwide, vestigial organs, etc., and we can begin to
understand why scientists speak of evolution as a fact. The idea that life has undergone
modification with descent over the ages is not a scientific controversy; it is a social or
religious controversy born out of ignorance and prejudice. (We must distinguish between the
fact of macro-evolution and the various theories of macro-evolution, the latter being subject to
legitimate criticism.)

Compare the above to biblical miracles, which have no documentation whatsoever, which
resemble the tall tales of many Mideastern religions, which often violate scientific principles,
and are, in short, extraordinary claims without extraordinary evidence. The choice should be
obvious to anyone who understands the evidence and has an elementary grasp of the
principles of good reasoning.

Finally, Roger Hutchinson has the audacity to suggest that we skeptics are really Deists or
liars! "Since there are no sound reasons to reject the idea that God exists, we should expect
those who call themselves skeptics to be either Deists (at a minimum) or liars." Ideas are
rejected for a lack of good evidence; they are not accepted for a lack of good disproof. Once
again, we have a misunderstanding of the concept of "the burden of proof." (Aside from that,
there are many reasons for rejecting the biblical god.)

(Dave Matson, editor, The Oak Hill Free Press, P. O. Box 61274, Pasadena, CA 91116; e-
mail: 103514.3640@compuserve.com)

Volume 1990 - 2002 Issue


Page 1147 of 2049
Skeptical Review Edited by Farrell Till

Another Happy Atheist...

A friend passed his May/June 1997 issue of The Skeptical Review on to me, and I read it with
great delight. This is a wonderful publication. I was especially pleased to find an article by
Judith Hayes. Certainly I want a year's free subscription. As a matter of fact, I would
appreciate it if you would also send a subscription to my son, whose name and address I will
place at the bottom of this letter.

You quoted Jung as saying that a majority of mental illnesses are caused by religion. I believe
it! I was raised in a fundamentalist home. Although I changed denominations several times, I
was a churchgoing and dedicated Christian for many years. From my middle teens onward, I
suffered depression. Sometimes the only thing that kept me from committing suicide was the
fear of hell. From time to time, I suffered from totally irrational fears. I remember a time
when I, a grown woman and mother of three school age children, would run and hide under
the bed if the phone rang when I was home alone. I went to doctors, a clinical psychologist,
and clergymen for help. I took various medications, and I prayed--and prayed! Nothing
helped. In January 1995, both my husband and I (I was 56 and he was 61) found the courage
and honesty to give up religion altogether. After a little reading, we realized we were atheists
and have proclaimed it proudly ever since. And guess what? Since then I have not had one
second of depression or irrational fear. I didn't know life could be so enjoyable. I now believe
with all my heart that teaching a child to fear hell is the very worst form of child abuse.

Thank you for offering this publication. I am looking forward to receiving my first copy.

(Carol Faulkenberry, 1308 Crest Avenue, Gadsden, AL 35904; e-mail


alncarol@internetpro.net)

Volume 1990 - 2002 Issue


Page 1148 of 2049
Skeptical Review Edited by Farrell Till

Skeptical Review
Volume Nine, Number Two
March/April 1998
Farrell Till, editor

• Archaeology and Biblical Accuracy


Responding to the inerrantist claim that archaeology has proven the accuracy of the
Bible, the editor shows that archaeology has confirmed the accuracy of very little of
the Bible and has in some cases shown that the Bible is in error.
• The Critics' Admissions Concerning Daniel
Inerrantist Everette Hatcher defends the traditional view that the book of Daniel was
written in the 6th Century B. C.
• The Inerrantist Way of Misrepresenting "Critics"
Farrell Till responds to Everette Hatcher and shows that he has misrepresented sources
in an attempt to show that even some "liberal" critics see evidence of an early
authorship of Daniel.
• How the Snake Slithered into Eden
William Sierichs, Jr., traces the origin of the Genesis snake myth back to Sumerian
and Babylonian mythology.
• The "Narrow View" vs. the "Larger Picture"
Fundamentalist Dr. Peter D. Righter accuses editor Farrell Till's "narrow view" of
keeping him from seeing the "larger picture" in the sins-of-the-fathers issue.
• Who's Really Looking at the "Narrow View"?
Farrell Till responds to Dr. Righter to show that his obsession with narrow
fundamentalist views actually impedes his ability to see the "larger picture."
• From the Mailbag
This four-page section contains an assortment of letters from readers and the editor's
comments.

Volume 1990 - 2002 Issue


Page 1149 of 2049
Skeptical Review Edited by Farrell Till

Archaeology and Biblical Accuracy


Farrell Till
Has archaeology proven the historical accuracy of the Bible? If you listened only to biblical
inerrantists, you would certainly think so. Amateur apologists have spread this claim all over
the internet, and in a letter published in this issue, Everett Hatcher even asserted that
archaeology supports that "the Bible is the inerrant word of God." Such a claim as this is
almost too absurd to deserve space for publication, because archaeology could prove the
inerrancy of the Bible only if it unearthed undeniable evidence of the accuracy of every single
statement in the Bible. If archaeological confirmation of, say, 95% of the information in the
Bible should exist, then this would not constitute archaeological proof that the Bible is
inerrant, because it would always be possible that error exists in the unconfirmed five percent.

Has archaeology confirmed the historical accuracy of some information in the Bible? Indeed
it has, but I know of no person who has ever tried to deny that some biblical history is
accurate. The inscription on the Moabite Stone, for example, provides disinterested,
nonbiblical confirmation that king Mesha of the Moabites, mentioned in 2 Kings 3:4-27, was
probably an actual historical character. The Black Obelisk provides a record of the payment of
tribute to the Assyrian king Shalmaneser III by Jehu, king of the Israelites (2 Kings 9-10; 2
Chron. 22:7-9). Likewise, the Babylonian Chronicle attests to the historicity of
Nebuchadnezzar, king of Babylon, and his conquest of Jerusalem as recorded in 2 Kings 25.
Other examples could be cited, but these are sufficient to show that archaeology has
corroborated some information in the Bible.

What biblicists who get so excited over archaeological discoveries like these apparently can't
understand is that extrabiblical confirmation of some of the Bible does not constitute
confirmation of all if the Bible. For example, the fact that archaeological evidence confirms
that Jehu was an actual historical character confirms only that he was an actual historical
character. It does not confirm the historical accuracy of everything that the Bible attributed to
him. Did a "son of the prophets" go to Ramoth-gilead and anoint Jehu king of Israel while the
reigning king was home in Jezreel recovering from battle wounds (2 Kings 9:1-10)? Did Jehu
then ride to Jezreel in a chariot and massacre the Israelite royal family and usurp the throne (2
Kings 9:16 ff)? We simply cannot determine this from an Assyrian inscription that claimed
Jehu paid tribute to Shalmaneser, so in the absence of disinterested, nonbiblical records that
attest to these events, it is hardly accurate to say that archaeology has proven the historicity of
what the Bible recorded about Jehu. Likewise, extrabiblical references to Nebuchadnezzar
may confirm his historical existence, but they do not corroborate the accuracy of such biblical
claims as his dream that Daniel interpreted (Dan. 2) or his seven-year period of insanity (Dan.
4:4-37). To so argue is to read entirely too much into the archaeological records.

The fact is that some archaeological discoveries in confirming part of the Bible
simultaneously cast doubt on the accuracy of other parts. The Moabite Stone, for example,
corroborates the biblical claim that there was a king of Moab named Mesha, but the
inscription on the stone gives a different account of the war between Moab and the Israelites
recorded in 2 Kings 3. Mesha's inscription on the stone claimed overwhelming victory, but the

Volume 1990 - 2002 Issue


Page 1150 of 2049
Skeptical Review Edited by Farrell Till
biblical account claims that the Israelites routed the Moabite forces and withdrew only after
they saw Mesha sacrifice his eldest son as a burnt offering on the wall of the city the Moabites
had retreated to (2 Kings 3:26-27). So the Moabite Stone, rather than corroborating the
accuracy of the biblical record, gives reason to suspect that both accounts are biased. Mesha's
inscription gave an account favorable to the Moabites, and the biblical account was slanted to
favor the Israelites. The actual truth about the battle will probably never be known.

Other archaeological discoveries haven't just cast doubt on the accuracy of some biblical
information but have shown some accounts to be completely erroneous. A notable example
would be the account of Joshua's conquest and destruction of the Canaanite city of Ai.
According to Joshua 8, Israelite forces attacked Ai, burned it, "utterly destroyed all the
inhabitants," and made it a "heap forever" (vs:26-28). Extensive archaeological work at the
site of Ai, however, has revealed that the city was destroyed and burned around 2400 B. C.,
which would have been over a thousand years before the time of Joshua. Joseph Callaway, a
conservative Southern Baptist and professor at Southern Baptist Theological Seminary, spent
nine years excavating the ruins of ancient Ai and afterwards reported that what he found there
contradicted the biblical record.

The evidence from Ai was mainly negative. There was a great walled city there
beginning about 3000 B. C., more than 1,800 years before Israel's emergence in
Canaan. But this city was destroyed about 2400 B. C., after which the site was
abandoned.

Despite extensive excavation, no evidence of a Late Bronze Age (1500-1200 B. C.)


Canaanite city was found. In short, there was no Canaanite city here for Joshua to
conquer (Biblical Archaeology Review, "Joseph A. Callaway: 1920-1988,"
November/December 1988, p. 24, emphasis added).

This same article quoted what Callaway had earlier said when announcing the results of his
nine-year excavation of Ai.
Archaeology has wiped out the historical credibility of the conquest of Ai as reported
in Joshua 7-8. The Joint Expedition to Ai worked nine seasons between 1964 and
1976... only to eliminate the historical underpinning of the Ai account in the Bible
(Ibid., p. 24).
The work of Kathleen Kenyon produced similar results in her excavation of the city of
Jericho. Her conclusion was that the walls of Jericho were destroyed around 2300 B.
C., about the same time that Ai was destroyed. Apparently, then, legends developed to
explain the ruins of ancient cities, and biblical writers recorded them as tales of
Joshua's conquests. Information like this, however, is never mentioned by inerrantists
when they talk about archaeological confirmation of biblical records.

Archaeological silence is another problem that biblical inerrantists don't like to talk
about. According to the Bible, the Israelite tribes were united into one nation that had
a glorious history during the reigns of king David and his son Solomon, yet the
archaeological record is completely silent about these two kings except for two
disputed inscriptions that some think are references to "the house of David." This is
strange indeed considering that references to Hebrew kings of much less biblical

Volume 1990 - 2002 Issue


Page 1151 of 2049
Skeptical Review Edited by Farrell Till
importance (Omri, Ahab, Jehu, Zedekiah, etc.) have been found in extrabiblical
records. This archaeological silence doesn't prove that David and Solomon did not
exist, but it certainly gives all but biblical inerrantists pause to wonder.

Another case in point is the biblical record of the exodus of the Israelites from Egypt
and their subsequent 40-year wandering in the Sinai wilderness. According to census
figures in the book of Numbers, the Israelite population would have been between 2.5
to 3 million people, all of whom died in the wilderness for their disobedience, yet
extensive archaeological work by Israeli archaeologist Eliezer Oren over a period of
10 years "failed to provide a single shred of evidence that the biblical account of the
Exodus from Egypt ever happened" (Barry Brown, "Israeli Archaeologist Reports No
Evidence to Back Exodus Story," News Toronto Bureau, Feb. 27, 1988). Oren
reported that although he found papyrus notes that reported the sighting of two
runaway slaves, no records were found that mentioned a horde of millions: "They
were spotted and the biblical account of 2.5 million people with 600,000 of military
age weren't?" Oren asked in a speech at the Royal Ontario Museum. That is certainly a
legitimate question. Up to 3 million Israelites camped in a wilderness for 40 years, but
no traces of their camps, burials, and millions of animal sacrifices could be found in
ten years of excavations. This may be an argument from silence, but it is a silence that
screams.

The Critics' Admissions Concerning Daniel


Everette Hatcher III
Farrell Till has asserted that reputable Bible scholars believe that the book of Daniel was not
written by an individual named Daniel during the sixth century B.C. (TSR, Vol 4.3, p. 12).
These scholars hold that the writer lived in the time of the Maccabees, and his "purpose was
to give his countrymen reason to believe that centuries earlier a prophet of Yahweh had
foreseen the rise of the Seleucid Empire and had predicted the triumph of the Maccabean
struggle for independence against Antiochus Epiphanes" (TSR, Vol. 7.3, p. 3). William
Sierichs, Jr. also takes this position in his article, "Daniel in the Historians' Den" (TSR, Vol.
7.4, p.8). Sierichs comments, "Daniel can't get Babylonian history straight, but he does pretty
well by the Hellenistic era. Obviously, whoever wrote the book was a very solid citizen of the
2nd century B.C.E., whose `prophecies' were wholly retroactive."

Both Till and Sierichs have been influenced by biblical scholars who have embraced the
higher critical views of the 1800's. However, most people have overlooked the fact that these
same scholars have made several admissions which are damaging to their Maccabean thesis.

The first admission concerns the conservative's view that Rome is the fourth kingdom
identified in Daniel's prophecy. Till states the critic's logic: "A flaw in this interpretation is the
obvious fact that the writer of Daniel considered the median and Persian kingdoms to be

Volume 1990 - 2002 Issue


Page 1152 of 2049
Skeptical Review Edited by Farrell Till
separate empires, because he had the Neo-Babylonian empire falling to `Darius the Mede'
(5:30-31). This is historically inaccurate (just one of many historical inaccuracies in the book
of Daniel), because reliable records of the time indicate that Cyrus the Great captured
Babylon and ended the Neo-Babylonian kingdom. Nevertheless, the writer of Daniel told of a
reign under "Darius the Mede: that preceded the reign of the Persian king, Cyrus the Great
(6:28; 10:1). So if the writer believed that the Neo-Babylonian Empire fell to the Medes and
then the Medes fell to the Persians, then the fourth kingdom in Daniel's interpretation would
have been Alexander's Hellenistic empire" (TSR, Vol. 4.3, p. 12).

Notice that Till bases his conclusion on the "obvious fact that the writer of Daniel considered
the Median and Persian kingdoms to be separate empires...." However, the famous Bible
critic, Dr. Samuel Driver, admitted, "In the book of Daniel the `Medes and Persians' are, it is
true, sometimes represented as united (Daniel 5:28; 6:8, 12, 15, cf. 8:20)" (The Book of
Daniel: Cambridge Bible for Schools and Colleges, Cambridge: University Press, 1900, p.
29). Conservative scholar Stephen Miller comments: "Such an admission seems fatal to
Driver's position, for if the author was aware at one point that the two nations were united into
one empire, he certainly would not have construed them as separate both physically and
chronologically elsewhere in the same book" (Daniel: The New American Commentary,
Nashville, TN, Broadman and Holman Publishers, 1994, p. 95).

Moreover, in Daniel 5:28, the word peres has the same consonants (only the consonants were
written in ancient Aramaic and Hebrew scripts) as the Aramaic term translated "Persians" and
likely was a paronomasia (a word play) hinting that the division of the kingdom would be
accomplished by the Persian armies. Bible critic Norman W. Porteous admits this hints at "the
victory of Persia over Babylon" (Daniel, The Old Testament Library, Philadelphia:
Westminster, 1965, p. 81). Furthermore, the Bible critic John A. Montgomery agrees ("A
Critical and Exegetical Commentary on the Book of Daniel," International Critical
Commentary, T. and T. Clark, Edinburgh, 1979, p. 263).

Arthur Jeffrey claims the author assumed from his reading of Old Testament prophecies
(Isaiah 13:17; 21:2, and Jeremiah 51:11, 28) that the Medes conquered Babylon before the
Persians (Arthur Jeffrey, "The Book of Daniel," Interpreter's Bible, Nashville: Abingdon,
1956, p. 434). However, Isaiah 21:2 blows this theory out of the water, because it speaks of
Elam and Media as the joint-conquerors of Babylon. The critic H. H. Rowley admits: "This
was doubtless written after Cyrus, king of Anshan, in southwest Elam, had brought the rest of
Elam under his sway, when to the Hebrew observer it appeared likely that these two powers
might unite in the destruction of Babylon. And since Elam is mentioned first, it is possible
that the passage dates from a time after the absorption of Media by Cyrus" (H. H. Rowley,
Darius the Mede and the Four World Empires in the Book of Daniel, 1935; reprint, Cardiff:
University of Wales, 1964, p. 58).

Till correctly notes that the writer of Daniel had "Darius the Mede" conquering Babylon, but
nowhere does the writer state that Darius was "the king of the Medes" or the "king of Media."
Dr. Robert H. Pfeiffer of Harvard University admitted the author of Daniel was "a very
learned man" and "a sage" (Robert H. Pfeiffer, Introduction to the Old Testament, New York:
Harper and Brothers, 1948, p. 757), but Pfeiffer must have assumed that this "sage" had never
read 2 Chronicles 36:20 where it is said that the Jews were servants to Nebuchadnezzar "and

Volume 1990 - 2002 Issue


Page 1153 of 2049
Skeptical Review Edited by Farrell Till
his sons until the reign of the kingdom of Persia." Clearly this indicates that the Persian reign
came immediately after the Babylonian reign.

The second admission concerns the madness of Nebuchadnezzar in Daniel chapter four.
William Sierichs, Jr., states that "the story of Nebuchadnezzar's insanity may be a reference to
a bout of insanity or lengthy depression in Nabonidus, who apparently was very unpopular in
Babylon..." (TSR, Vol. 7.4, p. 8). This is the position held by many modern critical scholars
today. Conservatives prefer a different explanation. Stephen Miller comments: "Some
scholars have deemed this chapter primarily a fictional account, likely derived from the same
source as the so-called `prayer of Nabonidus' (4QPrBab), an Aramaic fragment discovered at
Qumran in 1952 (D. N. Freedman, "The Prayer of Nabonidus," Bulletin of the American
Schools of Oriental Research, Vol. 145, 1957, pp. 31-32). Though affinities exist between
Daniel 4 and the "Prayer of Nabonidus," they are far outweighed by the differences (e.g.,
name of the king, nature of the illness, and location). It seems reasonable to categorize the
Nabonidus story as a distorted version or a later application of the biblical narrative" (p. 145).

Nevertheless, the critics insist there is no hint in the historical record that indicates it was
Nebuchadnezzar with this strange case of madness that resulted in a seven-year absence. R. H.
Pfeiffer called Daniel chapter four an "unhistorical tale," and "a confused reminiscence of the
years when Nabonidus spent at Tema in Arabia" (p. 758). Norman W. Porteous states,
"indeed there is no record of Nebuchadnezzar's having had leave of absence from his royal
duties on account of insanity" (p. 70). However, later on the same page Porteous admits that
fellow Bible critics Bevan, Montgomery, Bentzen, and Jeffrey have recorded such a story.
Abydenus's account is preserved by Eusebius (Praeparatio Evangelica, 9.41.1) and is
reproduced by John A. Montgomery (p. 221).

Abydenus says that in the last days of Nebuchadnezzar, the king was "possessed by some god
or other" while in his palace, and announced the coming of a Persian mule (i.e., Cyrus), who
would bring the people into slavery. Then says Abydenus, "He, when he had uttered this
prediction, immediately disappeared" (Praeparatio Evangelica, 9.41.1). Surely Porteous is
wrong to admit the existence of this story by the historian Abydenus, and at the same time
insist that "there is no record of Nebuchadnezzar's having had leave of absence from his royal
duties..."

The third and fourth admissions concern linguistic arguments. Farrell Till asserts: "Bible
fundamentalists like to think that Daniel was written in the sixth century B. C., shortly after
the events that the book closes with during the reign of Cyrus the Great, who had conquered
Babylon in 539 B.C. Few reputable Bible scholars, however, would fix the date that early,
because the book exhibits signs of a much later authorship. Scholars cite the writer's obvious
confusion about political events of the time that a contemporary would have surely been
familiar with, the linguistic style (especially the section written in Aramaic), and other factors
too numerous to discuss in detail as evidence that the book was written at the extreme end of
the Old Testament period (no sooner than the second century)" (TSR, Vol. 4.3, p. 13).

Dr. Samuel Driver also made much of the Aramaic of the Book of Daniel. he stated, "The
Aramaic of Daniel (which is all but identical with that of Ezra) is a Western Aramaic dialect,
of the type spoken in and about Palestine" (p. 59 of the introduction of Driver's commentary

Volume 1990 - 2002 Issue


Page 1154 of 2049
Skeptical Review Edited by Farrell Till
on Daniel), and he went on to suggest that archaeology had confirmed this. However, Jeffrey
admits that the Aramaic in the Book of Daniel "cannot be pressed as evidence for a particular
date, for it is that type of Aramaic which grew up for official use in the chancelleries and
came to be widely used in the ancient Near East" (p. 349). Jeffrey cites more recent
discoveries of fifth-century Aramaic texts that totally discredit Driver's view (Franz
Rosenthal, Die Aramaistische Forschung, [Leiden: E. J. Brill, 1939] pp. 66-71).

Till has highly recommended Jeffrey's work on Daniel (TSR, Vol. 7.3, p. 3, and Vol 7.4, p. 8).
According to Till, Jeffrey's material "gives a detailed analysis of the Book of Daniel to show,
first of all, that it was not written by its namesake who allegedly lived in Babylon during the
captivity, but by an unknown author during the time of the Seleucid Empire, which arose
from the partitioning of Alexander's kingdom after his death" (TSR, Vol. 7.3, p. 3). Does
Jeffrey's work accomplish this feat? Let's look at a couple of popular arguments that he uses.

The fourth admission by the critics concerns the term "Chaldeans." Jeffrey argues: "The use
of the word kasdim (Chaldeans), not in the proper ethnic sense which it has, for example, in
Jeremiah, but to mean a caste of wise men, points to a time when the word was commonly
used for a class of priestly astrologers, diviners, or magicians, a sense the word has in the
pages of Strabo or Diodorus Siculus, who wrote in the first century B.C. (p. 349).

Dr. Driver agrees that the argument concerning the use of the term "Chaldeans" is very
convincing. So much that he places it first in the list of his three strongest arguments that
show that the book of Daniel was composed in Palestine "during the persecution of Antiochus
Epiphanes" (pp. 47-56 of the Introduction).

How strong is this argument? On page 12 of Driver's commentary, Driver himself takes
exceptions to some of the assertions made by Jeffrey. Driver admits that in Daniel 5:30, and
9:1 the author of the book of Daniel did use the ethnic sense of the word "Chaldeans." Then
on the same page Driver admits this term "Chaldeans" is found "in Herodotus (Herodotus,
Histories, 1.181-183, c. 440 B.C.), and is common afterwards in the classical writers" (p. 12).
Furthermore, Driver also admits that evidence indicates that such a group of wise men as
pictured in the book of Daniel did exist as a group as early as 2000 B.C. (p. 14).

Francis Schaeffer summarized Driver's argument: "Remember this is his first strong
argument. he is going to take the book of Daniel and throw away its historical date on the
basis of these `so-called' strong arguments. Now we have defined this question in regard to
the term "Chaldeans." The writer knew the ethnic sense. This group did exist from a long time
before. About 90 years later everybody acknowledges that the word was used in this sense to
the wise men. And so he is going to throw away the book of Daniel and its dating and all that
it means on the basis that this specific group of wise men, who were well known from long
before and afterwards, were not called this term in this 90-year span (530 B.C. to 440 B.C.).
Now, once you word it this way, it doesn't look so strong" (Francis Schaeffer's five part series,
Dr. Driver's Criticism of the Book of Daniel, tape #2).

Is it any wonder that the bible critic J.J. Collins admits that the author's use of the term
"Chaldeans" cannot be used to date his material (Daniel, Hermeneia, Minneapolis: Fortress,
1994, pp. 137-138). In fact, Jeffrey makes a similar error in his commentary on Daniel 10:1.

Volume 1990 - 2002 Issue


Page 1155 of 2049
Skeptical Review Edited by Farrell Till
He states: "Cyrus is here called `king of Persia.' This may be merely a statement of fact, for he
was king of Persia, but if it is meant as an official title, it is an anachronism in the mouth of
Daniel. The title 'king of Persia', was Hellenistic usage and not the usage of the Achaemenid
kings at this time" (p. 500).

Jeffrey overlooked the fact that Robert Dick Wilson contradicted this view expressly with
what he found in the tablets of the Persian period (Robert Dick Wilson, "The Title `King of
Persia' in the Scriptures," The Princeton Theological Review, Vol. 15, 1917, pp. 90-145).
Wilson commented: "It is evident therefore, that there are thirty-eight distinct extra-biblical
instances of the use of this title from 545 to about 400 B.C.; and that these instances are found
in twenty different works by nineteen different persons (p. 100)."

This argument of Jeffrey's is completely put to flight concerning Daniel 10:1. It shows how
much many of these scholars continue to repeat the same old arguments. No doubt, Jeffrey
had read this argument in Driver's commentary (p. 152), but he had failed to read the
refutation provided by Wilson seventeen years later. I must admit that I have just repeated the
arguments of others on occasion without taking a closer look at both sides of the argument.
For example, I was guilty of making it appear that the interpretation of Daniel 5:7 is a simple
matter. Driver says the verse should read, "Shall rule as one of three in the kingdom" (p. 64),
but most conservative scholars claim the translation should be "the third highest ruler in the
kingdom" (NIV). Conservatives claim this is an indirect reference to Nabonidus while the
critics relate the passage to Daniel 6:2, which speaks of three rulers of equal rank and uses a
similar word.

At first I sided with the conservatives, but after further investigation I must admit that I am
undecided. What created doubts in my mind? It was the admission of three conservative
scholars that the critics may be right on this verse. On 4-26-96, I wrote Dr. Robert L. Alden
about this verse. He responded: "I looked up both Wood and Young as well as a couple [of]
other commentaries on Daniel. I also looked over the Aramaic original just to see what the
syntax of the verse was. I understand the reason for the differences of opinion that you have
discovered and am not sure which side of the log I fall off. Perhaps the concluding sentence at
the end of the first paragraph in E.J. Young's commentary is best, `but probably it is not to be
too dogmatic concerning the precise meaning of this word....' Incidently [sic] I had E. J.
Young when I was a student at Westminster Theological Seminary 1959- 1962. And I worked
with Leon Wood on the translation of the NIV."

Earlier I thought I had a lot of ammunition with the admission by critics W. Sibley Towner
and R.A. Anderson that the conservatives had the translation of Daniel 5:7 correct (W. Sibley
Towner, Daniel, Interpretation: A Bible Commentary for Teaching and Preaching, Atlanta:
John Knox, 1984, p. 73; R. A. Anderson, Signs and Wonders, International Theological
Commentary, Grand Rapids: Eerdmans, 1984, p. 57). However, now I don't think it is wise to
press the issue.

Nevertheless, there are two other issues in this chapter that I will press, and they both concern
Belshazzar. In the article "Daniel in the Historians' Den," William Sierichs, Jr., states that
Belshazzar was not the "son" of Nebuchadnezzar, and "Belshazzar was not the ruler as the
Book of Daniel claims, and he was never king" (TSR, Vol. 7.4, p. 8).

Volume 1990 - 2002 Issue


Page 1156 of 2049
Skeptical Review Edited by Farrell Till
These are two of the most common arguments used against the book of Daniel, but even the
radical critic, Dr. Philip R. Davies has admitted that both are "weak arguments" (Philip R.
Davies, Daniel, Sheffield: JSOT Press, 1985, p. 31). He stated: "Critical commentaries,
especially around the turn of the century, made much of the fact that Belshazzar was neither a
son of Nebuchadnezzar, nor king of Babylon. This is still sometimes repeated as a charge
against the historicity of Daniel, and resisted by conservative scholars. But it has been clear
since 1924 (J.A. Montgomery, Daniel, International Critical Commentary, Edinburgh: T. and
T. Clark, New York: C. Scribner's Sons, 1927, pp. 66-67) that although Nabonidus was the
last king of the Neo-Babylonian dynasty, Belshazzar was effectively ruling Babylon. In this
respect, then, Daniel is correct. The literal meaning of son should not be pressed" (pp. 30-31).

I call Davies a radical critic because he refuses to accept the archaeological evidence that
indicates that king David existed (Philip R. Davies, "`House of David' built on Sand," Biblical
Archaeology Review, July/August 1994, pp. 54-55), and more recently he suggested that
Hezekiah's tunnel was not dug by Hezekiah's men when the Bible claims, but was constructed
centuries later. However, several eminent archaeologists put this reinterpretation to rest
("Defusing Pseudo-Scholarship," Biblical Archaeology Review, March/April 1997, pp. 41-
50). For Davies to concede anything, it must really be self-evident. Therefore, I put forth his
admissions as especially meaningful. Furthermore, Davies does not accept the same view that
Till and Sierichs do concerning the date of the authorship of the first six chapters of Daniel.

In the 19th century the consensus among Bible critics was that all of the cheaters of Daniel
were written in Palestine during the 2nd century B.C. However, in the 20th century most of
the critics admit the first six chapters could have been written as early as the 6th century B.C.
in Babylon. Philip R. Davies comments, "According to nearly every modern commentator, the
tales of chapter 1-6 are originally products of a Jewish community in a Gentile environment"
(Philip R. Davies, "Eschatology in the Book of Daniel," Journal for the Study of the Old
Testament, Vol. 17, 1980, p. 33).

Could it be that the archaeological, linguistic, and historical evidence concerning Daniel will
lead next century's critics to consider the traditional theological view? This reminds me of an
amazing quote from the astronomer Dr. Robert Jastrow: "For the scientist who has lived by
his faith in the power of reason, the story ends like a bad dream. He has scaled the mountains
of ignorance; he is about to conquer the highest peak; as he pulls himself over the final rock,
he is greeted by a band of theologians who have been sitting there for centuries" (Robert
Jastrow, God and the Astronomers, New York: Warner Books, 1978, p. 111).

(Everette Hatcher III, P. O. Box 23416, Little Rock, AR 72221)

The Inerrantist Way of Misrepresenting


"Critics"

Volume 1990 - 2002 Issue


Page 1157 of 2049
Skeptical Review Edited by Farrell Till
Farrell Till
Anyone who has ever read much literature of Christian apologists and taken the time to check
the sources they quote in support of their claims has surely noticed that they often quote out
of context and distort their sources to leave the impression that science and scholarship are on
their side. Many of their distortions are so flagrant that they have to be intentional, but in the
case of Everette Hatcher's attempt to defend the fundamentalist view of the authorship of
Daniel, I doubt if the misrepresentations of his sources were deliberate. I suspect rather that
he pieced together his article from fundamentalist works without taking the time to check the
accuracy of the quotations cited in them, and so the mis representations are not his own but
those of the sources that he blindly trusted. At any rate, his article is a hodgepodge of half-
truths, misrepresentations, and out-of-context distortions, which I am publishing and taking
the time to respond to just to show the tactics that biblicists resort to in order to defend the
absurd position that the Bible is completely inerrant.

When I received the article, my first inclination was not to publish it because it is little more
than one appeal to authority after the other strung out over two and a half pages. In other
words, Hatcher basically argued throughout his article that the 2nd-century B. C. dating of the
book of Daniel is wrong and the 6th-century B. C. dating correct, because certain scholars say
so. In so doing, he pieced together various quotations, obviously lifted unchecked from
fundamentalist sources, and paraded them before us as if quoting a "scholar" necessarily
proves anything. I have said many times in TSR and its internet list that anyone committed to
a religious position can always find books published by authors who share that belief, so if
quoting "scholars" constituted proof of one's position, anyone could prove any belief to be
true. A Muslim could quote "scholars" who say that the Qur'an contains prophecies that have
been fulfilled, and a Mormon could quote "scholars" who say that the Book of Mormon is
historically accurate. There is much more to biblical apologetics than just citing "scholars,"
but apparently Hatcher does not realize this. If he does, he failed to indicate it in his article.

Along with his array of "scholarly" quotations put together in defense of a 6th-century B. C.
authorship of the book of Daniel, Hatcher sent me a packet of letters from Bible professors at
various seminaries and fundamentalist colleges, such as Southwestern Baptist Theological
Seminary, Dallas Theological Seminary, Mid-America Baptist Theological Seminary,
Covenant Theological Seminary, Wheaton College, and even a couple of British universities.
In other words, Hatcher has sent to me a bundle of letters from professors of Bible and
religion who have traditional views of the Bible, and presented them as apparent support for
his position. Not surprisingly, these professors all seem to agree with Hatcher's belief that
Daniel was written in the 6th-century B. C.

One of them stated that "(t)he discoveries of fragments of Daniel among the Dead Sea Scrolls
shows [sic] that it was written earlier than 164 B. C.," but that was all that he said on the
subject. He gave no evidence at all to support this assertion. Furthermore, saying that these
discoveries show that Daniel was written earlier than 164 B. C. is too imprecise to warrant
comment, for if it were written in 165 B. C., that would be earlier than 164 B. C. Referring
also to the copy of Daniel found in the Dead Sea Scrolls, another of Hatcher's professors said
in awkward syntax that "even the liberals say that this must have had several hundred years
before the second century" and that "(i)n that case, it would put it back at least to the fourth or

Volume 1990 - 2002 Issue


Page 1158 of 2049
Skeptical Review Edited by Farrell Till
fifth century, if not the sixth." The professor said nothing to explain why the discovery of a
copy of Daniel at Qumran would have to mean that it was written "several hundred years
before the second century." He simply alleged that this was so and further said, "I think this
effectively has nullified all the arguments for the second century writing of Daniel because
even the liberals cannot account for this Dead Sea Scrolls copy at the time it appeared and in
the form in which it appeared unless it had been in hand for several centuries." Hatcher needs
to explain why finding a manuscript from the Essene period at Qumran would have to mean
that the original was written "several centuries" earlier than 194 B. C., but neither he nor any
of his Bible professors bothered to do that. They simply asserted that it was so.

The letters that Hatcher sent me are too many to review in this article, but I must mention that
the word "liberal" was freely used throughout them to label those who date the book of Daniel
in the 2nd-century B. C. In the space of only a one-page letter, a professor of New Testament
at Covenant Theological Seminary, a Presbyterian college in St. Louis, used the word three
times in referring to those who hold the view of a late authorship. A professor at
Southwestern Baptist Theological Seminary said of these "liberals" that he "doubt[ed] if any
arguments you could offer would persuade adherents of the late date that they are wrong," as
if he and his cohorts who claim a 6th-century dating are the epitome of reasonableness in
considering the late-authorship view. These professors typically believe that labeling a
position with the word "liberal" somehow constitutes refutation, but mere labels don't prove
anything.

Hatcher's quoted sources: The appalling thing about the sources that Hatcher quoted in his
article is that most of them are from the works of scholars who accept the 2nd-century B. C.
authorship of Daniel, but Hatcher has lifted them out of context or distorted them to leave the
impression that they think the 2nd-century view is doubtful. Hatcher cited, for example, H. H.
Rowley's Darius the Mede and the Four World Empires in the Book of Daniel, but just a
cursory examination of this book will reveal that Rowley rejects the fundamentalist view that
Daniel was written by a 6th-century B. C. prophet and accepts a 2nd-century authorship. Even
the table of contents makes this obvious, where there are such chapter titles as, "There Is No
Reliable Evidence for Any Darius the Mede" and "Darius the Mede is a Conflation of
Confused Traditions." The writer of Daniel, purporting to be an official in the royal court of
Babylon at the time of its downfall, said in 5:30-31 that Belshazzar was slain and that "Darius
the Mede received the kingdom being about sixty-two years old," but historians, knowing that
the Persian king Cyrus the Great conquered Babylon, have seen this statement as a serious
flaw in the theory that Daniel was written by a member of the royal court at the time of
Babylon's fall to Cyrus. The problem has been complicated by the absence of any reference at
all to a Darius the Mede in ancient nonbiblical records. After discussing various critical
attempts to defend the obvious belief of the writer of Daniel that Babylon fell to a Median
king named Darius, Rowley said this in his concluding chapter:

The Book of Daniel is not a work of the sixth century B. C. The case against the
traditional date to which the composition of the book has been assigned rests on a
variety of considerations, but the single one we have examined in the first part of our
study would alone be sufficient to establish it. For [sic] a sixth-century person, who
not only lived through the events of the period, but took a leading part in them, could
not have made so gross an error as our author made in introducing Darius the Mede

Volume 1990 - 2002 Issue


Page 1159 of 2049
Skeptical Review Edited by Farrell Till
between Belshazzar and Cyrus. Nor could he have supposed that a Median empire
stood between the Babylonian and the Persian (University of Wales Press, 1935, p.
175).
After rejecting the 6th-century authorship of Daniel, Rowley went on to state that the book
was clearly written in the 2nd century:
As certainly can we say that the book of Daniel is a work of the second century B. C.
If the work is loosed from the sixth century by the inaccuracy of its knowledge of that
age, it is anchored in the second century by the accuracy of the knowledge of that age
which appears in its pages.... So long as the work was believed to be written in the
sixth century B. C., the accuracy of its descriptions of the second century but served to
establish the wonderful certainty of prophecy. But when the link with the sixth century
is broken by the proved historical errors in the part of the book that relates to that age,
the whole case is altered. It is impossible to believe that the mind of Daniel was
illumined with accurate knowledge of future times, while, at the same time,
thoroughly befogged as to the events in which he himself had played no mean
part, and we can only find in the limited range of the accurate knowledge the
indication of the author's period (Ibid., pp. 175-176).
So obviously Rowley did not believe that the book of Daniel was written by a 6th-century B.
C. prophet who personally witnessed many of the events recorded in the book, yet Hatcher
has taken Rowley's book and quoted it in a way that left the impression that this biblical
scholar saw serious problems in the critical views of those who reject a 6th-century B. C.
authorship. The snippet that Hatcher quoted, however, was never intended to leave that
impression, so Hatcher apparently puts more value on cherished beliefs than on scholastic
honor.

Critics like Arthur Jeffrey have surmised that the author of Daniel incorrectly assumed from
certain Old Testament prophecies that the "Medes conquered Babylon before the Persians,"
but Hatcher said that Isaiah 21:2 "blows this theory out of the water, because it speaks of
Elam and Media as the joint-conquerors of Babylon" (p. 2, this issue). For support of this
claim, Hatcher lifted a statement from Rowley out of context, which I will examine later, but
first I want to point out that Hatcher's reference to Isaiah 21:2 tacitly begs a crucial question.
If Hatcher believes in the 6th-century authorship of Daniel, he surely accepts the 8th-century
authorship of Isaiah. If so, he will be forced to say that Isaiah 21 was a prophecy of the fall of
Babylon that was made two centuries before Babylon was actually conquered. To say, then,
that Isaiah 21:2 shows that Elam and Media were "co-conquerors of Babylon" is to assume
that this prophecy proved true, but to so reason begs the question of whether predictive
prophecy accurately occurred in biblical times. Does Hatcher have any evidence that Elam,
with Media, was a "joint-conqueror" of Babylon? If so he needs to present it, because we
won't allow him simply to assume that if an Old Testament prophet predicted that Elam and
Media would be involved in the downfall of Babylon, it had to have happened as prophesied.
The fact is that Elam and Media combined forces in a failed attack on Babylon in 596 B. C.
(Eerdmans Bible Dictionary, 1987, p. 317), but this was 57 years before Babylon fell to Cyrus
the Great in 539, by which time both Elam and Media had been gobbled up by Persian
expansionism. Furthermore, Isaiah 21:2-9 says nothing at all about Persia; it mentions only
Elam and Media, so Hatcher needs to prove that Isaiah intended Elam to be understood as
Persia. Elam was located east of Mesopotamia in what is now Southwestern Iran, and Media
was a confederation of tribes that lived in a mountainous region east of Armenia and northeast

Volume 1990 - 2002 Issue


Page 1160 of 2049
Skeptical Review Edited by Farrell Till
of Mesopotamia, an area that is now located in Eastern Turkey and Northern Iran. They were
clearly separate territories and nations, but Media gained control of Elam prior to Persia's
westward expansion. By the time Babylon fell to the Persians in 539 B. C., Cyrus had
absorbed the Median empire, and so by then both Elam and Media were satrapies or provinces
in the Persian empire. In view of this, biblicists need to explain why they think that Isaiah
figuratively intended for Elam to represent Persia in his prophecy that Elam and Media would
jointly conquer Babylon. Since both empires had been absorbed by Persia at the time of
Babylon's fall, one could just as logically argue that Isaiah meant for Media to represent
Persia or that he meant for both Elam and Media to represent Persia. The fact is that Isaiah
named both Elam and Media as co-conquerers of Babylon, but it didn't happen; hence, this is
just a simple case of prophecy failure in which no help at all can be found to make the book of
Daniel historically accurate.

Of course, Hatcher and his fundamentalist cohorts are desperate to explain why the writer of
Daniel made such a grievous mistake as to say that Babylon fell to "Darius the Mede," a
person whom history knows nothing about, even though the fall of Babylon to Cyrus the
Great is attested to in reliable ancient records; hence, they must grasp at any and all straws in
sight to try to salvage their beloved biblical inerrancy doctrine. So they have reached for the
failed prophecy in Isaiah 21:2-9 that Elam and Media would jointly conquer Babylon. History
records that the Elamite-Median venture against Babylon failed, but inerrantists argue that it
must have succeeded because Isaiah said that it would, and Daniel said Babylon fell to Darius
the Mede. Hence, they are trying to prove the historical reliability of the book of Daniel by
assuming the inerrancy of the book of Isaiah. What can I say except that this is typical of how
inerrantist minds think? Why don't they just cut out a step and assume the inerrancy of Daniel
and be done with it?

Hatcher even lifted a quotation from H. H. Rowley out of context to make it appear that this
scholar, who unequivocally accepts a 2nd-century authorship of Daniel, believed that Isaiah
12:2 supports the view that the Medes and Persians "jointly" conquered Babylon. Below, I
will quote a longer passage from Rowley's book than the brief one that Hatcher cited. I will
italicize the part that Hatcher quoted so that readers can see the out-of-context distortion that
he gave to the quotation. The quotation will be long but necessary to expose the misleading
tactics of Bible fundamentalists.

More serious is the styling of Darius a Mede. For both Cyrus and Darius Hystaspis
were certainly Persians. But if our author [of Daniel] supposed that Darius preceded
Cyrus, this further mistake would naturally follow. For he might well know that Cyrus
established the Persian empire, and the knowledge that there had been a Median
empire earlier than the Persian would lead him to call Darius a Mede. It has been
pointed out, and especially by Charles [Critical Commentary on Daniel, pp. 141 ff],
that he would be influenced in this confusion by Scriptural prophecy. For Jeremiah
had prophesied that Babylon should fall before "an assembly of great nations from the
north" [50:9, 41], and had further specified them as the Medes [51:11, 28]. So, too,
Isaiah xiii [17] had predicted the overthrow of Babylon by the Medes. It was indeed
natural that the Hebrew prophets should look for the downfall of the Chaldaean
regime at the hands of the Medes. For it was by them that Nineveh was destroyed, and
though Nebuchadnezzar was then, and remained, the ally of the Medes, he seems to

Volume 1990 - 2002 Issue


Page 1161 of 2049
Skeptical Review Edited by Farrell Till
have felt a certain nervousness of them; and when he used his good offices to bring
about peace between Media and Lydia [Herodotus, Hist. i.74], his desire was probably
to prevent his neighbour and ally from becoming too powerful. And at the beginning
of the reign of Nabonidus we find that Monarch forming an alliance with Cyrus, who
was to revolt against Astyages. Apparently at a later date the author of Isaiah xxi
hailed Elam and Media as the prospective conquerors of Babylon. This was doubtless
written after Cyrus, king of Anshan, in southwest Elam, had brought the rest of Elam
under his sway, when to the Hebrew observer it appeared likely that these two powers
might unite in the destruction of Babylon. And since Elam is mentioned first, it is
possible that the passage dates from a time after the absorption of Media by Cyrus.
Already, then, hope was moving from the Medes to Cyrus, on whom they soon
became centred in the writings of deutero-Isaiah [Isa. 41:2, 25 ff; 44:28; 45:1 ff; 46:1
f; 47:1 ff; 48:14]. But the author of the book of Daniel, in the absence of any exact
history of the period, seems to have been misled by the earlier hopes, and to have
assumed that though the Persians shared in the overthrow of the Babylonian empire,
the throne of Babylon fell to the Medes (Darius the Mede and the Four World
Empires in the Book of Daniel, pp. 57-58).
So Hatcher's out-of-context quotation not only does not show Rowley admitting anything
favorable to the view that Daniel was written in the 6th-century B. C. but, to the contrary,
when examined in context, shows that Rowley considered Daniel's reference to the conquest
of Babylon by "Darius the Mede" to be a serious historical blunder that could not have been
made by someone who was a Babylonian official (as Daniel purportedly was) at the time of
the fall. In the paragraph from which Hatcher lifted his quotation, Rowley was merely
explaining that the author of Daniel, who actually lived much later than the 6th century, was
probably familiar with prophecies by Isaiah and Jeremiah that Babylon would be conquered
by the Medes and had incorrectly assumed that the prophecies had been fulfilled as predicted.
Furthermore, Rowley's reference to "the writings of deutero-Isaiah" shows that he accepts the
critical opinion that additions were made to the book of Isaiah by scribes or editors living in
Babylon during the 6th-century captivity and that these unknown redactors, who would have
been familiar with the threat that the Medes posed to Babylon, retrojected into Isaiah
predictions that Babylon would be overthrown by the Medes. These predictions proved
untrue, and by the time Babylon was conquered by Cyrus the Great, Media had been absorbed
into the Persian empire. So in order to manufacture scholarly evidence to support the view
that Daniel was written in the 6th century B. C., Hatcher has cited a passage from Isaiah and
quoted out of context an authority who didn't even believe that the book of Isaiah was written
in its entirety by the 8th-century prophet by that name, but this is the kind of extreme that
biblicists must resort to in their desperation to defend their inerrancy doctrine.

Hatcher's case is so weak that he has had to resort to claiming that an Aramaic pun in Daniel
5:28 "hints" that the writer knew about "the victory of Persia over Babylon." This is a strange
argument indeed, because we have just reviewed Hatcher's effort to show that Daniel was
correct in saying that Babylon fell to "Darius the Mede," so which way does Hatcher want it?
Does he want us to believe that Daniel knew something that history is silent about and that
Babylon really was conquered by the Medes, or does he want us to believe that Daniel knew
that the Persians actually overthrew Babylon and implied this via a pun in Daniel 5:28? He
can't have it both ways.

Volume 1990 - 2002 Issue


Page 1162 of 2049
Skeptical Review Edited by Farrell Till
In an attempt to give credibility to his pun argument, Hatcher cited Norman W. Porteous as a
proponent of the theory that Daniel intended his use of the word peres in 5:28 to be
understood as a pun. Porteous, however, is another biblical scholar who is firmly convinced
that the book of Daniel was written during the Maccabean era of the 2nd century B. C., yet
Hatcher has tried to enlist him as a proponent of the inerrantist view of a 6th-century
authorship. Hatcher cited (p. 2, this issue) Porteous's commentary on Daniel from The Old
Testament Library (Westminster Press, 1965) in an attempt to make a dubious pun in Daniel
5:28 imply that the writer of Daniel knew that Persia conquered Babylon. In other words,
Hatcher's case is so tenuous that he can't produce direct textual evidence that the writer of
Daniel knew that Babylon fell to Persia; he has had to resort to claiming that the writer of
Daniel "punningly" implied it. He expects us to believe that the writer of Daniel knew that
Babylon fell to Persian forces, but he couldn't just come right out and say so. The best he
could do was just "hint" at it in a pun.

Daniel 5:28 is in the context of the inscription that was written on the wall by a hand that
mysteriously appeared during a "great feast" that Belshazzar had given for a thousand of his
lords, during which Belshazzar had committed sacrilege by drinking wine from the golden
and silver vessels that Nebuchadnezzar had brought back from the temple in Jerusalem. The
hand had written mene, mene, tekel, upharsin, and Belshazzar was troubled to know the
meaning of the words. The historicity of this story is unlikely, because the inscription was
written in Aramaic, the language that was spoken by Babylonians at this time. Belshazzar,
being a Babylonian, would surely have been able to read the words, but according to the story
he couldn't, because he sent for "the astrologers, the Chaldeans, and soothsayers" and said to
them, "Whoever reads this writing and tells me its interpretation shall be clothed with purple
and have a chain of gold around his neck, and he shall be the third ruler in the kingdom" (v:7).
His "wise men," however, could not "read the writing or make known to the king its
interpretation" (v:8), so we are expected to believe that probably the best educated class of
people in Babylon could not read a four-word inscription in their native language. That's a
doubtful premise at best. We could accept the claim that they could not interpret the message
intended by the words, but to say that they could not "read" the words is a bit too much to
believe.

Finally, Daniel came to the rescue (just as he had done when none of the kingdom's wise men
could interpret Nebuchadnezzar's dream about the great image in chapter 2) and both read the
words and gave Belshazzar an interpretation:

"And this is the inscription that was written: Mene, Mene, Tekel, Upharsin. This is
the interpretation of each word. Mene: God has numbered your kingdom, and finished
it; Tekel: You have been weighed in the balances, and found wanting; Peres: Your
kingdom has been divided and given to the Medes and Persians (vs:25-28).
The word peres is where Hatcher has found his pun. Well, actually Hatcher didn't find the
pun; he was merely parroting what he has read in the works of biblical fundamentalists who
were desperate to find something in Daniel to defend the position of a 6th-century authorship.
At any rate, one has to wonder why Hatcher and cohorts are so desperate to find a pun in the
word peres, since Daniel in his interpretation said that the kingdom of Belshazzar had been
divided and given to the Medes and Persians. All the talk about a pun seems like an exercise
in redundant futility.

Volume 1990 - 2002 Issue


Page 1163 of 2049
Skeptical Review Edited by Farrell Till
So this brings us back to Hatcher's quotation from Porteous's commentary on Daniel. Porteous
himself, as we will soon see, was definitely not a proponent of 6th-century authorship. The
six-word "bite" that Hatcher has snipped from Porteous was from the context of a two-page
discussion of Daniel 5:24-28 (quoted above) in which Porteous focused mainly on textual
variations in different versions of the inscription on the wall. The discussion is too long to
quote in its entirety or even to summarize, but Porteous's approach was simply to state what
different scholars have thought about and concluded from the variations. All that Porteous did
was to note in passing that some have understood the last word in the inscription (peres) to
refer "punningly" to "the Aramaic word for `Persian,' and so hinting at the victory of Persia
over Babylon" (p. 81). Porteous himself did not say that he accepted this "understanding" of
the word. He just noted it and went on to discuss critical theories about the meaning of the
inscription. Most of these theories were that "Daniel" really intended the words to be veiled
references to the Babylonian kings who had preceded Belshazzar, and anyone can verify this
by reading pages 81-82 of Porteous's commentary.

Porteous's actual view was that the book of Daniel was written during the 2nd-century
Maccabean conflict and that many of the prophecies were aimed at the despised Seleucid king
Antiochus Epiphanes. As Porteous finished his comments on the inscription left by the
mysterious hand, he stated this critical opinion too plainly to misunderstand.

Daniel made the mysterious words immediately relevant to the situation of the king.
The author of the book doubtless means to hint that the pride and sacrilegious conduct
of Antiochus Epiphanes have invited immediate judgment. And so we press back to
the writer's conviction that, in the crisis which faced the loyal and sorely tried Jews of
his day, the Judge of all the earth could be counted on to do the right. The traditional
materials are freely and imaginatively handled to convey this truth in the most
impressive way possible. The masal or parable which he has produced is unforgettable
(emphasis added, p. 83).
If this is not enough to convince Hatcher that Porteous was not a biblical critic sympathetic to
the view of a 6th-century authorship, perhaps his concluding comments on verses 30 and 31
of Daniel 5 will. This was where Daniel said that Belshazzar was killed "that night," and
"Darius the Mede received the kingdom."
A single sentence describes the succession to the Babylonian throne of Darius the
Mede. This Darius is almost certainly a figment of the writer's imagination. As we
have seen, the prophets had foretold that Babylon would fall to the Medes and so there
had to be a Median kingdom between the Babylonia and the Persian and there had to
be a Median king to succeed Belshazzar. Every attempt to prove that there actually
was such a monarch has failed. Astyages, the last king of the Median Empire, will not
fit. He had been conquered by Cyrus in 549 B. C. Nor will Cyaxares II, the supposed
uncle of Cyrus, suit any better. He is no more than an invention of Xenophon's.
Gobryas, the renegade Babylonian who became a general of Cyrus, may have acted as
governor of Babylon but was certainly never king. The successor of the historical
Nabonidus was the historical Cyrus and there never was a Darius between them. But
there was a Darius, Darius Hystaspis, who captured Babylon after the death of
Cambyses in 520 B. C. and the tradition of this capture and the name of the conqueror,
with which may have been associated some memory of his achievements, may have
been attached to the apocryphal figure. There are thus actual historical reminiscences

Volume 1990 - 2002 Issue


Page 1164 of 2049
Skeptical Review Edited by Farrell Till
as possible ingredients in the story, but we are for all that once more in the realm of
legend" (emphasis added, pp. 83-84).
If Hatcher had bothered to check his source, he would have seen that Porteous is clearly a
proponent of a 2nd-century authorship of Daniel. In the very first paragraph of the
introduction to his commentary, Porteous said, "The linguistic evidence and the fact that the
visions reveal a vague knowledge of the Babylonian and Persian periods and an increasingly
accurate knowledge of the Greek period up to and including the reign of Antiochus
Epiphanes, with the exception of the closing events of that reign, suggest a date for the book
shortly before 164 B. C." (emphasis added).

Available space will not allow me to discuss all of Hatcher's distorted and misrepresented
sources in a single article, and so I will follow this one with at least two more, one to finish
analyzing what Hatcher perceives as damaging admissions that "critics" have made
"concerning Daniel" and a third one to look more in detail at the textual evidence that points
to a 2nd-century B. C. authorship. After I have run my responses, Hatcher may reply to them
if he wishes.

How the Snake Slithered into Eden


William Sierichs, Jr
Dan Barker's article on "Serpentine Logic" (TSR, November/December 1997) touches one of
Christianity's most embarrassing problems. All Christian theology boils down to a simple
statement: a talking snake made me do it.

It's fundamental to Christianity that because the snake misled Adam and Eve, the human race
became inherently corrupt, and all of our crimes and "sins" followed, which is why the loving,
merciful Christian god is going to torture the human race in the most agonizing way possible
forever and ever, amen.

Our only chance of escape, Christians tell us, is to believe without reservation that the vast
intellect that created and controls a universe stretching at least 12 billion light years in every
direction came to earth as a mortal, allowed himself to be killed, spent three days in hell, and
thereby redeemed those of us who believe in Jesus Christ... or who are baptized in his name...
or who do good deeds... or who are "born again"... or who were predestined by god from
before all creation... or....

Well, exactly how we are to be saved from endless, excruciating torture is not quite clear,
considering how important it's supposed to be. Different biblical passages give different
versions of "the only way to salvation." Let's hope god comes back in time to clear up for us
just exactly how to make the right bet in the great Casino of Religious Fates so that we can sit
and eat popcorn in the heavenly bleachers while watching everyone who bet wrong writhing
in agony far below in God's Gulag.

Volume 1990 - 2002 Issue


Page 1165 of 2049
Skeptical Review Edited by Farrell Till
Before I put my bet on Blaise Pascal's gigantic roulette wheel, though, I'm going to consider
an alternate origin of the talking snake story. Since 19th-century archaeologists dug up the
great, long-buried libraries of ancient Middle Eastern civilizations, scholars have known that
the two contradictory creation stories in Genesis are based on two separate Mesopotamian
myths.

The first chapter of Genesis, with its six days of creation, is an Israelite adaptation of an older
creation story called the "Enuma Elish," in which a Mesopotamian god--the high god Marduk
in the Babylonian version, the high god Ashur in the Assyrian version-- created the world.
This king-deity had fought and killed the primal mother goddess, Tiamat, and cut her body up
in six steps, using the parts to make the universe, earth, and its lifeforms. The "Enuma Elish"
steps parallel the six Genesis-1 steps of creation.

Genesis may contain a hint of its origin, since in 1:11 and again in 1:24, Yahweh did not
make life directly but commanded the (female) earth to create living things. Similarly, in
1:20, Yahweh told the waters to create sea life. The only lifeforms Yahweh and his associates
("Let us make man") actually made are the first humans, perhaps echoing Mesopotamian
stories that the gods made humans in other ways than Tiamat's corpse.

The second Genesis creation story borrows from a Mesopotamian story about the Sumerian
gods making a divine garden for themselves, and the god Enki fathering a gardener to tend it.
One version I have read named the gardener "Tagtug." He was trained to tend the garden and
could eat from all of its plants except possibly one. Some key lines are unclear, so the last
point is uncertain, but scholars say it fits in with a discussion of the garden's plants and
Tagtug's duties. The story later indicated that Tagtug was cursed for some reason--perhaps he
ate the plant or perhaps he was the victim of divine jealousy--but in any case, he was expelled
from this Sumerian paradise and became a mortal. Later, the gods made eight assistants for
him, each of whom had some specialty.

Scholars point out that, according to Genesis 4:17-22, Cain had eight descendants, several of
whom were associated with specific crafts: Tubalcain with bronze and iron tools; Jubal, the
first musician; and Jabal, the first herdsman. The father of these three was Lamech, the great-
great-great grandson of Cain. Scholars say that "Lamech" was a Sumerian name, a title of the
god Enki, who was the patron of singers as well as Tagtug's father. (The names of the eight
assistants of Tagtug are not said to be related to the names of Cain's eight descendants.) So at
least part of the second Genesis creation story was derived from much older Sumerian stories.
But where did that talking snake come from?

All across the Middle East, snakes were associated with the worship of goddesses, usually the
"Queen of Heaven" under her various names, such as Inanna (Sumerian) and Ishtar (Semitic).
The "Queen of Heaven" could have many powers, including love, sex, the fertility of life,
healing miracles, and control over life and death. These goddesses were also shown nude
sometimes, in association with a fruit tree and snakes.

It doesn't take much imagination to see that the story of Adam and Eve could be an attack on
goddess worship. To an ancient Israelite, the story had nothing to do with "original sin," and
the snake was not "Satan." The story was traditionally interpreted to explain how death and

Volume 1990 - 2002 Issue


Page 1166 of 2049
Skeptical Review Edited by Farrell Till
the hardships of life came into the original paradise. So the story also could be seen as
showing that, when the first man obeyed the chief goddess--a naked woman with a snake and
a magic fruit tree--the chief god cursed him.

This would be a purely speculative interpretation except for two things. First, the ancient
Israelites worshiped a goddess named Asherah as the wife of their high god, Yahweh, and
other goddesses. In The Hebrew Goddess, Raphael Patai sorts the evidence in the Jewish
scriptures to show that the Israelites worshiped Asherah along with Yahweh in the first
Jerusalem Temple for at least 236 years of its 370-year existence between the 10th and 6th
centuries B. C.

That goddess worship was popular among most Israelites is shown by the complaints of
Jeremiah and Ezekiel about the people's ritual observances, which included a form of
sacrament involving cakes and wine. The goddess worshiped may have been Asherah or
Anath (Patai's guess). In Jeremiah 44: 15-19, the women of Israel specifically blame their
nation's troubles on the failure to observe the rituals of the queen of Heaven:

Then all the men who knew that their wives had burned incense to other gods, with all
the women who stood by, a great multitude, and all the people who dwelt in the land
of Egypt, in Pathros, answered Jeremiah saying, "As for the word that you have
spoken to us in the name of Yahweh, we will not listen to you! But we will certainly
do whatever has gone out of our own mouth, to burn incense to the queen of heaven
and pour out drink offerings to her, as we have done, we and our fathers, our kings and
our princes, in the cities of Judah and in the streets of Jerusalem. For then we had
plenty of food, were well off, and saw no trouble. But since we stopped burning
incense to the queen of heaven and pouring out drink offerings to her, we have lacked
everything and have been consumed by the sword and by famine." The women also
said, "And when we burned incense to the queen of heaven and poured out drink
offerings to her, did we make cakes for her, to worship her, and pour out drink
offerings to her without our husbands' permission?"
In Ezekiel 8:14, the women were "weeping for Tammuz" at the gate to Yahweh's house. In
Mesopotamian belief, Tammuz was the husband of Ishtar, who once spent three days in death,
hanging on a wooden stake or wall-peg in the underworld, before being resurrected. She then
ascended to the earth's surface, where she appeared to various people and finally to her
husband. He then took her place in the underworld for part of the year, while she ascended to
her home in the sky. (Something about this story sounds very familiar, but I can't quite place
it.) Goddess worshipers--including the Israelites-- annually mourned Tammuz's temporary
death.

Patai and other scholars add that inscriptions have been found in several places in and near
Israel that refer to "Yahweh and his Asherah." The scriptural claims that radicals repeatedly
cleansed ancient Israel or Judea of idols, shrines, etc., to the goddesses are considered to be
exaggerated if not flat-out fictions invented much later, after monotheistic Judaism was
developed following the 6th-century B. C. E. return from the Babylonian exile. The Israelites
had always been polytheists, although the Yahweh cult was considered the royal cult and
thereby the state cult. The propaganda against Baal and Asherah was written after Yahweh

Volume 1990 - 2002 Issue


Page 1167 of 2049
Skeptical Review Edited by Farrell Till
worship developed either into a fanatical cult or a very greedy one that wanted all religious
revenues for itself.

The second piece of evidence and the clincher to this argument is that an iron-age inscription
gives Eve (Hawwah) as one name of the goddess Elat, which in turn is one of the names of
Asherah, and ancient art shows a Canaanite goddess named Qudsu, yet another name for
Asherah, as naked and associated with snakes and lotus plants (in Egypt).

So some Yahweh-only propagandist took an original creation story about the first man's being
expelled from a divine garden and inserted a venomous attack on goddess worship. ("Don't
give your donations to anyone but me and my god.") After monotheism was established in
Judea, and the Israelites' polytheistic past was obscured and forgotten, people no longer knew
that the story was simply anti-goddess propaganda that had slithered onto the scene and
instead took it as proof that women were responsible for the world's problems.

Ultimately, the snake-Eve story both created and nurtured a vicious misogyny in Bible-based
civilization, one that patriarchal versions of Christianity still implicitly endorse with their
male-only clergy. It's long overdue for civilized people to tell Bible-worshipers to hiss off and
take that talking-snake fable with them.

(William Sierichs, Jr., 316 Apartment Court Drive, Apt. 44, Baton Rouge, LA 70806)

The "Narrow View" vs. the "Larger


Picture"
Dr. Peter D. Righter
Concerning Farrell Till's elaborate expose of the innocent paying the price for the sins of their
fathers, I would address it this way. First, since I do not take the position that the Bible is
inerrant in its present form, I don't have to go into a lot of gyrations and hair splitting to
present my viewpoint. Rather, I will do my best to use concepts that are found in scripture.
The argument as I see it is that innocent children and other people suffer for the sins of their
parents, in apparent contradiction to Deuteronomy 24:16 (and Ezekiel 18:20), which states
that "(t)he fathers shall not be put to death for [the sins of] the children, neither shall the
children be put to death for [the sins of] the fathers." Are there other parallels in the Bible that
address the concept of one paying the price for the sins of others or even suffering when it
doesn't make any sense? And do these other parallels reflect the hand of God behind them?
Certainly Jesus Christ suffered for the sins of others, though He had done no wrong. Although
it wasn't for the sins of His heavenly Father that this occurred, it was because of the sins of
His earthly forefathers (speaking of His humanity and His Biblical ancestors--king David,
etc.). In addition, are we not all paying the price for the sin of Adam and Eve, including
physical death? One of the Biblical books that is overlooked in this study is the story of Job,

Volume 1990 - 2002 Issue


Page 1168 of 2049
Skeptical Review Edited by Farrell Till
whom God called a righteous man, and whom He allowed to go through the ringer in order to
show His sovereignty and counter Satan's contentions that if Job suffers, he will curse God.
Here, there was obviously a "higher" unseen purpose behind the suffering that we can't
disregard in considering other scripture. The concept that God sometimes allows the innocent
to suffer doesn't set well with a lot of people, but the theme is often repeated throughout the
Bible to serve God's greater purpose.

As an application in real life, Dr. Charles Stanley of the First Baptist Church in Atlanta, tells
the story about the late Dr. Barnhouse, who witnessed the effect of the suffering of an
innocent baby and God's use of it to bring others into confessing Christ as their savior. A baby
with Down Syndrome had been born to a young pastor, and the mother had not yet seen the
baby or knew of its condition. The young pastor was initially very worried about how to break
the news to his wife, and Dr. Barnhouse told him, "This is from the Lord!" What does Exodus
4:11 say? "Who gave man his mouth? Who makes him deaf or mute? Who gives him sight or
makes him blind? Is it not I, the Lord?" And remember Romans 8:28, "And we know that in
all things God works for the good of those who love him." So the young pastor was
encouraged. Meanwhile, the hospital switchboard operator (who might well have been a big
Skeptical Review fan, and who took pleasure in seeing Christians humbled), heard the news
about the baby and looked forward to seeing the young pastor and his wife crumble in the
face of adversity. But when the young pastor told his wife about the baby and the comments
of Dr. Barnhouse, she called her mother on the phone and told her, "Mother, the Lord has just
blessed us with a baby with Down Syndrome. We don't know the nature of this blessing yet,
but we're looking forward to finding out." The switchboard operator, who was listening in,
was dumbfounded, and she told many of the other workers and nurses, etc., about what the
wife had said. The next Sunday, unknown to the young pastor, thirty nurses and staff
members from the hospital were in the congregation, and accepted Christ as their savior. God
had used the "suffering" of an innocent baby to bring many into eternal life with Him. And we
see this concept repeated throughout the scripture.

Getting back to the argument at hand, Deuteronomy 24:16 and other laws were given to man
for the purpose of administering human justice, and the commandment in question is an
excellent precept. The basic concept was the people should not be held accountable for the
sins of others. The issue in scripture relates to justice. The unseen or unknown factor behind
the scenes is God's higher purpose (note Job). Certainly, human institutions of justice are
imperfect and often down right unjust. God, if you can indulge in the Biblical concept of His
omniscience, sees the broader perspective, and its effects on the future of mankind. Imagine
now a father telling his young daughter, "Always wait for the light to change in your favor
before crossing the street." Then, later, as the father is driving along with his daughter, trying
to rush his critically ill wife to the hospital, he looks both ways and proceeds to run a red light
so he can save his wife's life. The daughter admonishes him for going against his own
instructions and writes The Skeptical Review, but perhaps only the father sees the bigger
picture. And so it is with God. The innocent may sometimes suffer along with the
unrighteous, but only in the earthly realm and only to serve God's greater and perhaps unseen
purpose (as in the case of Job). You can rework the above analogy to show how justice can be
administered contrary to appointed judicial authority, which is imperfect, and which may on
occasion abrogate the spirit of the law in effecting the letter of the law. It is obvious from
scripture that God sometimes steps into human affairs and does something that seems totally

Volume 1990 - 2002 Issue


Page 1169 of 2049
Skeptical Review Edited by Farrell Till
contrary to our perceived views of His nature and character. Such is likely the case in Till's
illustrations, where God seems to abrogate the Law He gave man, and serves out His own
measure of justice on individuals and nations. Since David was King of Israel, and could
possibly thwart human measures of justice and retribution if he desired, God stepped in and
made David pay the price, whereas otherwise, there may well have been no justice for David
and his fellow Israelites to see. Why God selects some instances to step in and take things into
His own hands, while other times He doesn't, I don't claim to know, nor can anyone know the
hidden things of God. Obviously, mortal man would have never thought that God would send
His only begotten Son to "unjustly suffer and die" on the cross for us, so why do we have the
audacity to think we can "put God in a box" and completely understand His ways? You may
not like that answer, but according to Job, the crucifixion, and other such instances, it's
certainly "Biblical."

In addition to this, I think we have to realize that, Biblically speaking, physical death and
spiritual death are two different things. One can die innocently for the sins of others and
consider themselves very fortunate to gain entry into the kingdom of heaven. So when we
look at David and Bathsheba's son who was stricken by the Lord (as well as other examples),
we have to assume that that baby is in heaven, and a very happy camper at that! If we could
see him now, we might reconsider how unjust God might have seemed in taking him from the
earth. We tend to look at physical death as the "end-all" that defines the tenets of human and
divine justice, but the judgment seat of Christ is in fact the ultimate "terminus." If those who
were "unjustly" stricken with death had also innocently suffered spiritual death and wound up
in hell, then we could look at God as being unjust and unfair. So in that respect, the innocent
children do not ultimately suffer for the sins of their fathers, and God can remain a "good"
and just God.

By the way, regarding Till's clever little remark about the stoning of Achan ("Who ever said
that confession was good for the soul")--if Achan's confession was heartfelt, it likely saved
him from an eternity in hell, so in that respect, yes, it was good for his soul. Till only looked
at the narrow view, and not the "larger picture." If he's going to try to use the Bible against
itself, he should be aware of the totality of scripture and use the whole Bible, and he will save
himself some instances of embarrassment.

(Dr. Peter D. Righter, 148 Hattaway Drive, Altamonte Springs, FL 32701)

Who's Really Looking at the "Narrow


View"?
Farrell Till
After what amounted to a sermon more than an attempt to address the inconsistency of the
Bible on the issue of whether God punished the innocent for the "sins" of their fathers, Dr.

Volume 1990 - 2002 Issue


Page 1170 of 2049
Skeptical Review Edited by Farrell Till
Righter accused me of looking only at the "narrow view and not the `larger picture,'" but no
view of the Bible could be any narrower than the one he reflected in his article. That view is
that regardless of how inconsistent, contradictory, absurd, or incomprehensible the Bible may
appear in places, it is nevertheless the inspired word of God, and so any textual problems in it
have to be just apparent. Dr. Righter has proposed to explain the sins-of-the-fathers problem
by what he calls the "larger picture," but as we will see, the only thing larger than the
"picture" he speaks about is his naivety in accepting a thoroughly discredited view of the
Bible. That the Bible is not the "inerrant word of God" has been demonstrated so
convincingly that only diehard fundamentalists like Roger Hutchinson, Dr. James Price, and
now Dr. Righter continue to cling to the view. Biblical inerrancy is an outdated view of the
Bible that survives in fundamentalist Bible colleges and churches pastored by the products of
these colleges, but it is no longer taught in responsible seminaries or believed by scholars who
put academic integrity above religious bias.

Dr. Righter tried to give a semblance of respectability to his "closet" inerrantist views by
saying that he does not "take the position that the Bible is inerrant in its present form," but this
is only the old inerrant-only-in-the-original- autographs argument dressed up in new clothes
with no place to go. The Bible "in its present form" is all that we have. There are no original
autographs available for critical examination; thus, if Dr. Righter intends to defend the Bible,
he will have to defend it in its present form. There is no other "form" to defend.

From the very beginning of his article, Dr. Righter made it very clear that he was going to do
nothing but rehash tired, worn-out inerrancy arguments that have been repeatedly answered,
some of them in earlier TSR responses to Roger Hutchinson's attempt to justify Yahweh's
conduct in the matter of David's and Bathsheba's infant son. However, since Dr. Righter's
subscription began with the last issue of 1997, we can understandably excuse him for not
knowing how thoroughly Hutchinson has been trounced on some of the same arguments that
Righter has trotted out to defend the Bible "in its present form." For his benefit, I suppose we
can go through them again.

Other parallels in the Bible: Dr. Righter asked if there are "other parallels in the Bible that
address the concept of one paying the price for the sins of others or even suffering when it
doesn't make sense." The answer, of course, is that there are such parallels, and no one is
denying that. What Righter apparently can't see is that those other parallels harm rather than
help his case, for if the Bible clearly and unequivocally states that the fathers shall not bear
the iniquity of the sons and the sons shall not bear the iniquity of the fathers, it doesn't matter
how many "parallels" to the case of David's son inerrantists may find in the Bible. If they find
a hundred such parallels (which in my opinion would not be an exaggeration), all that they
will have proven is that the Bible is a hundred times more contradictory than what could be
claimed if David's son were the only example of such suffering that one could find in the
entire Bible. Deuteronomy 24:16 and Ezekiel 18:20 are quite clear in what they say, so let
Righter locate all of the "parallels" he wants to. In so doing, he will merely confirm that I am
right in saying that the Bible is inconsistent on this matter. If, for example, the Bible says in
Psalm 25:8 that God is "good," it doesn't matter how many texts parallel to this Dr. Righter
and other inerrantists may point too-- and admittedly they could find many others; if the Bible
in other places clearly teaches--as it does--that Yahweh was at times vicious and cruel, the
unbiased reader of the Bible is left with no other recourse except to conclude that it is

Volume 1990 - 2002 Issue


Page 1171 of 2049
Skeptical Review Edited by Farrell Till
inconsistent on this point. Ten thousand different scripture citations "parallel" to Psalm 25:8
will not make the word "good" appropriately descriptive of the Old Testament god who
ordered Joshua to leave nothing alive to breathe in his conquest of Canaan (Josh. 10:40;
11:11, 15) and commanded Saul to "utterly destroy" the Amalekites and "spare them not" but
to kill both men and women, infant and suckling (1 Sam. 15:2-3). No amount of
rationalizing or searching for "other parallels in the Bible" could make the character of such
an entity as this "good." Such acts as these are either not good or else the whole concept of
language becomes meaningless.

God's higher purpose: Dr. Righter, of course, has an explanation for such inconsistencies as
these, and he calls it "God's higher purpose." His argument here is nothing that we have not
heard many times before ad nauseam. The problem is that we are simply too stupid to see the
"larger picture," i. e., the "higher purpose" that God has in mind for his actions that seem
strange and even sometimes "bad" to us. It's just that he is so far above us that we can't
understand why he would kill a baby for something that the baby himself did not do, but if
our intelligence were equal to God's, it would make perfectly good sense to us. Yeah, right!
We have heard this all before, and what it amounts to is nothing more than inerrantist
desperation to defend the indefensible. If we are so intellectually inferior that we can't even
determine when the words "good" or "bad" should be used to describe acts or conduct, then
how are we to know whether God is indeed "good," as the psalmist claimed? The answer is
that we can't. If we cannot know what the words mean where the Bible says that the children
shall not be put to death for the sins of their fathers, then it becomes absurd to talk about
understanding that God has a "higher purpose" that surpasses our ability to comprehend. If
such an incomprehensible "higher purpose" is a fact, then God has no business trying to
communicate anything to us in human language. Revealing the Bible to us has turned out to
be a huge waste of divine time that could have been put to better use, because it was revealed
to us in languages whose vocabularies had acquired meaning through human experiences and
usages, but it turns out that God's vastly superior knowledge and "higher purpose" may very
likely have given meanings to words like "good," "bad," "love," "mercy," "kill," etc. that are
entirely different from ours, and so "good" may not be our kind of good and "love" may not
be our kind of love, etc., etc., etc. This is the kind of warped logic that leads inerrantists to
argue that even though killing children and babies would be a horrible sin if humans do it, the
act somehow becomes "good" if God does it. Why so? Well, because God's ways are higher
than our ways.

A Catch-All Explanation: What inerrantists have done is simply invent a catch-all


explanation that they can use to "explain" any objection that anyone may raise against the
Bible. Do you not understand why a perfect, impartial deity would select just one nation of
the multitude of nations to be his specially chosen people? Well, no problem. God's ways are
simply higher than our ways, and so he has a higher purpose that we just can't understand. Do
you have difficulty understanding why a kind, loving, merciful deity would require the
slaughter and incineration of millions of animals in homage to him? Well, no problem. His
ways are just higher than ours. Do you have difficulty understanding why an omnibenevolent,
omnijust deity would require the torture and sacrifice of his own son for the sins of humanity?
Again, no problem. This deity's ways are higher than our ways, so we are unable to
comprehend his "higher purpose." See how simple it is?

Volume 1990 - 2002 Issue


Page 1172 of 2049
Skeptical Review Edited by Farrell Till
The soundness of an argument can be determined by simply applying it to other situations,
and when this is done to the "higher-purpose" argument of biblical inerrantists, it becomes an
obvious reductio ad absurdum. Do Christians not understand why the Hindu deity Vishnu
would have revealed himself to humanity in so many avatars or incarnations? No problem.
His ways are simply higher than our ways, and if we were on his level of intelligence, we
would be able to understand his "higher purpose." Do mainstream Christians have difficulty
understanding why God would have revealed the Bible and then 1800 years after its
completion give humanity a latter day revelation through the Book of Mormon? Again, no
problem. We are so far below God's level of intelligence that we simply cannot understand his
"higher purpose." I could give many other examples, but these are sufficient to show the
absurdity of Dr. Righter's "higher-purpose" argument. If he continues this debate, I suggest
that he attend to what should be the first order of business, and that is to give us reasonable
evidence that the "higher purpose" of this god of his is fact and not just convenient fiction that
has been invented so that biblicists will have a catch-all explanation to hide behind. The
"convenience" of this dodge was very apparent in Righter's claim that he doesn't know, nor
does anyone know, why "God selects some instances to step in and take things into His own
hands," because that is just another mystery that we should file under "the hidden things of
God" (p. 9, this issue). The problem with this line of "argumentation" is as simple as
recognizing that that which explains everything explains nothing. If Dr. Righter has any kind
of logical defense to offer in the matter of David's son, I'd like to see it; otherwise, I will have
to ask him to refrain from sending us just more simplistic conjecture.

Anecdotal argumentation: Dr. Righter has so little to offer in defense of his position that he
spent almost a third of his article engaging in anecdotal argumentation. The story of the child
born with Down Syndrome has all the earmarks of what I call "pulpit legend." When I was a
young preacher, I was dismayed to hear different preachers tell the same anecdotes in their
sermons as if they had all personally experienced them, an unlikely scenario at best, and so I
recall being disillusioned by the realization that preachers were not above lying in order to
embellish their sermons. Whether Righter's anecdote has any basis in fact or not is irrelevant
to the matter of what the Bible says about the death of David's son. If the anecdote really
happened, it would prove only that a mother, blinded by religious indoctrination, mistakenly
considered the birth of a handicapped child to be a divine blessing, but it would in no way
prove that it actually was. From my wife's experience as a social worker and counselor, I have
had the occasion to meet many children with Down Syndrome and to know about special
circumstances that their families confront. So if the pastor's wife saw the birth of her child as
a divine blessing, I have nothing but pity for the warped sense of reasoning with which her
blind religious allegiance has afflicted her. Did this event cause 30 hospital workers to "accept
Christ as their savior" the very next Sunday? I doubt it, and this is the part of the anecdote that
strikes me as probable sermonic embellishment. If, however, it did happen, it would prove
only that there were 30 other people whose ability to think rationally was questionable. It
wouldn't prove that there is any savior to accept or that there is any divine "higher purpose"
that will explain away the biblical inconsistency in what it says about punishing the innocent
for the offenses of the guilty. Dr. Righter needs a logical argument to support his position.
Conjectures about divine higher purposes and babies camping happily in heaven won't do.

Volume 1990 - 2002 Issue


Page 1173 of 2049
Skeptical Review Edited by Farrell Till

From the Mailbag


The Null Hypothesis...

Thank you for once again producing a fine issue of The Skeptical Review. I've been following
the Jeremiah "prophecy" with interest. I read the previous issue in a sloop 60 miles off shore,
and my jaw hit the deck when I read where Dr. Price mangled the "null hypothesis" so
horribly. I was in no position (nautically) to mail my objections. Imagine my delight when
your next issue hammered the "null hypothesis" back into shape.

The "null hypothesis" is exactly that--null. It does not address the how, why, what, where,
and when of something that happened; it addresses the if of something having happened. One
need not explain an event that never occurred. The "resurrection of Jesus," Lazarus, and
assorted saints is an example. Where Atlantis existed is another. How Nazi flying saucers
enter and exit the hollow earth is a third. The default is the null hypothesis. There is nothing
to explain.

After plowing through the last three issues, I think all question of Jeremiah's "prophecy" can
be answered in the negative. Dr. Price cannot possibly reconcile the evidence against him that
you have presented to him. One can only wait and hope that Dr. Price has the grace,
intellectual honesty, and balls to concede his defeat.

Concerning the "sins of the fathers" topic, I am at a loss to understand why Roger Hutchinson
wrote what he did. Every Bible verse he quoted damns his position on the issue. The verses he
quoted did nothing to address the fact that the Bible claims that a child was punished for the
"sins" of the child's parents. The fact that Hutchinson can provide verses where the Bible also
says otherwise is the whole point: the Bible contradicts itself. Anyone who believes otherwise
has never read it.

My favorite part of TSR is "From the Mailbag." (I read this for entertainment and the rest for
education.) I take umbrage at Larry Laird's claim that Christians as a whole are not, and
cannot be, rational! TSR addresses a small subset of Christians who are compartmentally
irrational when it comes to their paper idol. Even this same subset must be rational at many or
most other times.

About "converted atheists," fundamentalist Christians have been using this ploy for decades.
(Creationists use it rather often.) Some claim to have been "Satanists," "witches," thieves,
rapists, blah, blah, blah... until they found fundamentalist Christianity and "got better." This
sells books, tapes, magazines, pamphlets, buttons, stickers, pins, and lectures. It should be
pointed out that every human being on the planet is born an atheist: atheism is the default.
"Committed atheist" is another matter, which you addressed well. Compared to inerrant
fundamentalist Christians, mainstream (i. e., rational) Christians would appear to be atheists.

(Rev. David Rice, 723 Calle Casita, San Clemente, CA 92673-2708)

Volume 1990 - 2002 Issue


Page 1174 of 2049
Skeptical Review Edited by Farrell Till
EDITOR'S NOTE: I apologize for the delay in publishing this letter, but the articles by Dr.
Price and Roger Hutchinson have generated more correspondence than space was available
for.

Convoluted Thinking? Whose?

Dr. Hector Avalos said that he hoped his letter to Dan Barker concerning Walter Kaiser's view
of the snake in Genesis 3 would help to "unravel the convoluted grammatical arguments that
some evangelicals like to use" (TSR, November/December 1997). However, it is Avalos who
is guilty of convoluted thinking if he believes the Bible but denies that the snake was Satan.

"All the days of your life" (3:14) shows that the serpent is treated as a personal being. In Job
1-2, the devil impugns the character of God again. In John 8:44, Jesus states, "[Satan] was a
murderer from the beginning, not holding to the truth, for there is no truth in him. When he
lies, he speaks his native language, for he is a liar and the father of lies" (NIV).

Martin Luther correctly noted, "The devil was permitted to enter beasts, as he here entered the
serpent. For there is no doubt that it was a real serpent in which Satan was and in which he
conversed with Eve" (Luther's Works, Lectures on Genesis, ed. J. Pelikan and D. Poellot,
trans. G. Schick, 1.511).

Is Genesis 3 a fable? Walter Kaiser answered Barker's logic with this reply: "You do have an
animal speaking, but that's not your only criteria for what constitutes a fable...." The real
question of evidence is answered by looking at the evidence available from historical records
and archaeology. This evidence confirms the accuracy of many parts of the Old Testament. In
fact, archaeology supports the view that the Bible is the inerrant word of God!

(Everett Hatcher III, P. O. Box 23416, Little Rock, AR 72221)

EDITOR'S NOTE: Hatcher's letter was timed perfectly. It arrived as I was writing a response
to his article on the authorship of Daniel (pages 2-4, this issue), which I have been holding
for several months until space was available for both it and my reply. In order to help readers
see the type of mindset that accepts the traditional, 6th-century B. C. dating of Daniel, I
bumped a couple of other letters, which I will publish later, in order to publish Hatcher's
letter simultaneously with his article.

Hatcher said that if Dr. Avalos believes the Bible, then he was the one who is guilty of
"convoluted thinking." I think that Hatcher must have fallen off the turnip truck just the day
before he wrote his letter, because Dr. Avalos's letter made it rather obvious that he doesn't
necessarily believe that something is true just because the Bible says that it is. This puts him
poles apart from Hatcher, who is obviously a biblical inerrantist and will therefore believe
anything the Bible says no matter how absurd it may be. Furthermore, Dr. Avalos did a
thorough linguistic analysis of the Genesis text to show that the only reasonable way to
interpret it is that the writer meant for readers to understand that a literal serpent talked to
the woman Eve. I wonder what Hatcher's linguistic credentials are that entitles him to dispute
Avalos's conclusion. I trust that everyone noticed that Hatcher made no attempt to analyze the
Genesis text. He just arbitrarily declared that Avalos was wrong.

Volume 1990 - 2002 Issue


Page 1175 of 2049
Skeptical Review Edited by Farrell Till
He did cite some scriptures, but their relevance completely escaped me. He seemed to be
arguing that because the Bible says elsewhere that Satan "impugned the character of God"
and that he was a murderer, a "liar and the father of lies," the talking serpent in Genesis 3
must have been Satan. I can't see that either scripture that he quoted sheds any light at all on
the identity of the "serpent" in Genesis 3, but the fact that Hatcher even quoted them shows
that he does not yet understand that what biblical writers B and C may have thought or
believed doesn't necessarily mean that writer A agreed with their theological views. Now if
Hatcher can find some textual indication in the Garden-of-Eden story that the Genesis writer
thought that the serpent was Satan or the devil, we want to see it. We aren't at all interested in
what Hatcher or other biblical writers may have thought he meant.

That applies even to what Martin Luther may have thought. Hatcher said that Luther
"correctly noted" that "(t)he devil was permitted to enter beasts, as he here entered the
serpent," but that was merely Luther's opinion, for which Hatcher offered no supporting
evidence. Space won't permit me to discuss them, but Luther had religious views that were
completely absurd. So the fact that he thought that the devil was in the serpent that conversed
with Eve neither surprises me nor impresses me. Hatcher will have to find something far more
substantial than this if he expects me to be convinced.

Hatcher said that "(t)he real question of evidence is answered by looking at the evidence," so
at last he said something that I can agree with. Now all he has to do is show us some evidence
that would prove that the Genesis writer meant for his readers to understand that the serpent
was Satan. In "How the Snake Slithered into Eden" on pages 8, 11 of this issue, William
Sierichs, Jr., traced the origin of the snake myth back to Babylonian and Sumerian mythology.
Perhaps Hatcher can tell us why we should not suspect that these myths explain how the
Garden-of-Eden story found its way into the Bible for reasons entirely different from the spin
that biblical fundamentalists have given to it. I recommend too that Hatcher read Josephus's
account of this story (Antiquities of the Jews, 1:1.4). If he will do that, he will find that as late
as the first century A. D. at least some Jews understood the story in Genesis 3 to be nothing
more than an explanation of why there is a natural enmity between humans and snakes. If the
meaning of the story is what Hatcher claims, I wonder why Josephus, having grown up in the
family of a prominent Jewish priest in Jerusalem and having been educated in Jewish culture
and traditions (The Life of Flavius Josephus, 2:7-12), would not have known that the serpent
in this story was Satan. Hatcher talks about "evidence" but gives none himself to support his
claims. Apparently, he believes that giving evidence is the duty of only his opposition.

Of all the bald assertions in his letter, none is more absurd than his claim that "archaeology
supports [that] the Bible is the inerrant word of God." This is an often-repeated claim of
Bible inerrantists, but I suggest that Hatcher read the article about what archaeology has
proven about the accuracy of the Bible (front page, this issue) and then tell us if he is willing
to defend this position in future exchanges.

Believe It or Not...

Regarding Dave Matson's reference to the veracity of Robert Ripley, despite claims of
accuracy, Ripley's Believe It or Not feature probably contained more questionable "facts" than
appear in the Christian Bible. Maurice Horn, in his World Encyclopedia of Cartoons, has this

Volume 1990 - 2002 Issue


Page 1176 of 2049
Skeptical Review Edited by Farrell Till
to say: "Its facts... are not always accurate (every critic has a favorite error or fallacy from
Believe It or Not... but this never fazed Ripley). Good newspaperman that he was, he knew
that unadorned facts and bare statistics make for poor copy, and his aim was to entertain, not
to enlighten. Indeed, much more accurate fact panels, such as John and Elsa Hiz's Strange as
It Seems and R. J. Scott's Did You Know? have not even approached the success of Ripley's
feature."

Richard S. Russell wrote, "(A)lmost every sentence you write or speak is unique. Neither you
nor anyone else has ever spoken that exact combination of words before or is likely to do so
again." To quote the immortal words of Rube Goldberg, "No matter how thin you slice it, it's
still baloney." The use of identical sentences occurs hundreds, if not thousands, of times
during the course of a year. To cite an example, cartoon editors frequently receive drawings
with identical gag lines from widely separated parts of the country. Although in some cases
plagiarism may be suspected, in others the duplication is purely coincidental as the drawings
in question have not yet been published and their creators did not know each other.

As for there being 800 thousand words in the English language, many hundreds of these are
peculiar to a particular discipline and, therefore, would not be used in everyday speech or
writing, even by writers considered erudite.

Finally, a few words regarding the Court-Matson discussion regarding the definition of
miracles. No doubt modern inventions such as the telephone and television would have been
regarded as "miracles" by the citizens of the ancient world. At least one of the biblical
miracles performed by Jesus, that of turning water into wine, can be explained by the use of a
container with a false bottom. Such vessels have been discovered by archaeologists during
excavations in the Near East.

A mathematician would be able to determine the "odds" against a certain event occurring,
ranging from 50-50 to 99-1 against. Raising the dead would rate in the latter category, but
considering the progress medical science has made since the days of Hippocrates, who is to
say that it absolutely could not happen.

(R. S. Craggs, 25 McMillian Avenue, West Hill, ON, Canada M1E 4B4)

EDITOR'S NOTE: It should be pointed out that Dr. James D. Price was the one who
introduced Ripley's Believe It or Not; Matson was merely reacting to Price's claim that the
extraordinary has been routinely accepted in the case of Ripley's famous cartoon series.
Cragg's comments show that gullibly accepting extraordinary claims like Ripley's will result
in mass deception, so it still seems like a good idea to demand extraordinary proof for
extraordinary claims.

On the matter of linguistic uniqueness in the formation of sentences, I recall having on my


desk, when I was still teaching, a copy of The American Heritage Dictionary, which contained
an introductory article about the extensiveness of the English vocabulary. The article
reported the results of an experiment that was conducted at a university. A professor
distributed to his classes copies of a cartoon that showed a bear in a countryside booth
talking on a telephone while an impatient motorist waited outside. The professor asked his

Volume 1990 - 2002 Issue


Page 1177 of 2049
Skeptical Review Edited by Farrell Till
students to write a brief description of what the cartoon was about. He found that all of his
students handed in papers that accurately described the scene but that no two students had
used sentences that were exactly like. The professor then had a computer expert to analyze the
content of the cartoon and the English language in terms of vocabulary, syntax, and grammar
rules and then predict how many students would have to describe the scene before two would
submit identical descriptions. The verdict of the computer was 19.5 billion, which would be
three times the population of the earth.

In the case of modern medical science perhaps being able some day to resurrect the dead, Mr.
Craggs should remember that even if this should ever happen, it wouldn't affect the Bible's
many claims of resurrection, because the claim is that these events happened before there was
anything even close to "modern medical science."

By the way, if Mr. Craggs or anyone else knows of sources I could consult to find information
on containers from ancient times with false bottoms, I'd like to have them.

A Little Logic Helps...

Enclosed is my check for another year's subscription. I read the latest issue yesterday in one
sitting. The article by Steve Carr about how the Bible has been changed over and over again
was especially interesting. I hope you print more articles on that subject. I read the article on
Bible miracles by inerrantist Roger Hutchinson with great humor. One of the best things I
ever did for myself was to learn logic. When I see fundamentalists like Hutchinson
continually make errors of logic, such as assuming the truth of what they seek to prove, I just
crack up. How can they expect us to take them seriously?

I too was a fundamentalist until I started asking questions about problems I noticed and
finding out that there were no answers, only dogma. Keep up the good work, because your
publication provides real answers to religious questions.

(Ed Unger, 13800 Toonigh Road, Canton, GA 30115)

EDITOR'S NOTE: How can fundamentalists expect us to take them seriously? Beats me.
Maybe Hutchinson or some other fundamentalist would like to answer this question.

Those who would like to investigate how the Bible has been changed over and over may like
to read a new booklet on this subject by A. J. Mattill, Jr. See Polluted Texts on the backside of
this issue for further details.

Correspondence Wanted...

I was pleasantly surprised to find an atheist/freethought publication so close to home. Please


begin my subscription to The Skeptical Review immediately. I would greatly enjoy
correspondence.

(Andrew Delunas, 3664 Neosho Street, St. Louis, MO 63116-4315)

Volume 1990 - 2002 Issue


Page 1178 of 2049
Skeptical Review Edited by Farrell Till
Books Recommended...

Enclosed is my renewal for 2 years. I really enjoy your publication. Recently, I've been doing
some reading about discoveries of the early days of religion and Christianity, the Dead Sea
Scrolls, etc. Two very interesting books are In the House of the Messiah by Ahmed Osman
and Who Wrote the New Testament? by Burton Mack (1995). I've gotten most of the books
from the Barnes & Noble mail-order catalog. Keep up the good work.

(Polly Dunne, P. O. Box 348, Blowing Rock, NC 28605-0348)

A Christian Reader's View...

Enclosed is a check for $6 to renew my subscription to The Skeptical Review. As a Christian


who is committed to the truth on any subject (John 8:32; 18:37), I look forward to receiving
every issue of TSR. I have been a subscriber since the summer 1995 issue, and I ordered all of
the back issues and all of your study aids. Your material is very informative, very interesting,
and the greatest challenge to Bible inerrancy I have seen.

As an oral debater you are very good, but as a writing debater you are the best I know of. (I
have 48 debate books in my library, and I have participated in some debates, so I know
something about the subject.) Vincent Lewis is the best oral debater I know of. In your
July/August issue you mentioned a possible debate with him. Please bring your readers up-to-
date on this.

(Don Robertson, 644 Walnut Street, Rock Hill, SC 29730)

EDITOR'S NOTE: An admirer of Vincent Lewis wrote and asked me to debate Lewis. I
accepted the invitation, and Lewis and I negotiated and signed propositions and debating
rules. The debate is to be organized by Lewis and a colleague, but I have heard nothing from
him or the colleague since the signing of the propositions several months ago. If this debate
materializes, I will publicize it in TSR.

Lot's Daughters...

Of special interest to me in your September/October issue was your lead essay, "Keep Them
Barefooted and Pregnant." In it, you recounted the story of Genesis 19, wherein god sends
two angels to Sodom to warn Lot of the impending destruction of the city. This reminded me
of a recent letter to the editor in the Denver Post from a fundamentalist outraged over a
previously published letter that stated being gay was not a biblical sin. The angry writer
offered the biblical story you mentioned in your column as proof of god's disapproval of
homosexuality: how the men of Sodom surrounded the house and insisted that Lot send out
the men "so that we can have sex with them," but Lot went out and offered his two virgin
daughters to the crowd instead. (What a swell dad!) The letter writer ended by stating
triumphantly that this was absolute proof that "clearly the `gay lifestyle' is viewed by God as a
perversity and a sin."

Volume 1990 - 2002 Issue


Page 1179 of 2049
Skeptical Review Edited by Farrell Till
I read his letter in amazement. Obviously, in his hatred of gays and inability to view the bible
objectively, the writer could not see the extreme disparagement of women that this anecdote
portrayed. I immediately sat down and wrote to the Denver Post:

"In his single-minded zeal to prove his god's hatred of gays, N. T. D. offers a bible
quote which, in reality, proves that the bible is not a repository of ethics and family
values. Amazingly, he is oblivious to the horror of Lot's offering up his own two
virginal daughters to a mob of men in order to save his male house guests. Anyone
who has taken the time to read the bible is aware that this treatment of women is
commonplace in its pages. Those who cling to the words of the long-dead barbarians
who wrote it either don't know, or don't care. If his quote proves god's hatred of gays,
as he is certain it does, by that rationale, it also proves god's hatred of women. It
justifies using rape to curb homosexuality, proving to rational people that biblical
quotes have no place in determining morality on either issue."

As a woman, I too am enormously frustrated at the seeming inability of women as a group to


cast off religious brainwashing. As a member of various groups for some years now, I find
more women willing to join us, but much more needs to be done. I hope you'll keep exposing
the misogyny in that tome known as "the good book," and your readers will copy those
articles and give them to the women in their lives. I have done just that with many articles
from various sources over the years, and now I'm happy to say that my 80-year-old mother is
an adamant atheist.

(Sandra Feroe, 1400 Pheasant Run, Berthoud, CO 80513-8419; e-mail SLFeroe@aol.com)

EDITOR'S NOTE: Lot's willingness to sacrifice his daughters to save his male visitors from
gang-rape was not just a one-time event in the Bible. Judges 19 tells the story of a man in the
town of Gibeah who offered to a crowd of homosexuals his virgin daughter and the concubine
of his male visitor to quell their demands to send the male visitor out to them. When the crowd
refused the "compromise," the male visitor sent his concubine out to the crowd. So here is
another biblical story that puts a higher value on the sexual honor of men than women. Like
the story of Lot, it also shows that fathers were willing to sacrifice the honor of their own
daughters in order to protect men from abuse.

Nontheistic Religions...

While going through the November/December 1997 issue of The Skeptical Review, one short
passage from Roger Hutchinson's article "Biblical Miracles" caught my eye. This was: "For
the religious person, there can only be one true religion and one true God; all other Gods, and
the miracles attributed to them, must be false."

This point of view (aka the fallacy of the false dilemma) may be valid among religions such
as the Abrahamic triad of Judaism, Christianity, and Islam, but it is by no means true of the
other "world" religions. In particular, it definitely isn't true of either Theravada Buddhism or
Jainism, two Indic religions which get along quite nicely without any conception of "one true
God" in the Western sense of a being or entity who is responsible for the creation and
continuing existence of the universe. Likewise, there is no perceived dichotomy of valuation

Volume 1990 - 2002 Issue


Page 1180 of 2049
Skeptical Review Edited by Farrell Till
between "brands" of miracles. They are all relegated to the manipulation of natural forces
operating (albeit infrequently) within the universe.

As far as the question of truth goes, neither of these two religions maintain that they have an
absolute monopoly on that commodity. The Buddha, for example, is often recorded as
pointing out that it is wrong for a wise man to say that "(t)his alone is truth, and everything
else is false" (Canki Sutta, #95 of the Majjhima Nikaaya, and elsewhere). At another place in
the canon, he says, "To be attached to one thing (to a certain view) and to look down on other
things (views) as inferior--this the wise men call a fetter" (Sutta Nipata, verse 798). The Jain
stance is remarkably similar.

I had thought that narrow-minded provincialism and cultural chauvinism were fast becoming
relics of the past as the 20th century draws to a close, but I find myself being disappointed,
time and again, as pockets of insularism, particularly in matters of religion, continue to crop
up where one would logically least expect to find them.

(Joseph Crea, P. O. Box 10576, Parker, FL 32404-1576; e-mail Joseph.


Crea@worldnet.att.net)

A Bone to Pick...

I've just read Roger Hutchinson's article, "Sins of the Fathers and Other Matters." His
reasoning reminds me of a dog chasing its tail--a lot of circular motion that gets nowhere and
accomplishes nothing. Regardless of whether the child was killed because David had
committed adultery or because David's deed had given the enemies of the Lord occasion to
blaspheme, the Bible says clearly that God forgave David but struck the child so that it
became sick and died and that God did this because of what David had done (2 Sam.12:13-
18). God punished a child because of the father's sin, and no amount of circumlocution can
change that.

Now I have a bone to pick with both Hutchinson and Till. Both referred to David and
Bathsheba committing adultery. Bathsheba did not commit adultery. She was raped! The
Bible (2 Sam. 11:4) says clearly that David sent messengers and took her. David was a king,
not a figurehead or constitutional monarch like kings and queens today. He could order the
death of people who displeased him. Bathsheba's husband was away at war, so she had no
protector. When a man who has the power of life and death over a woman orders her to have
intercourse with him, she is a rape victim and most definitely not an adulteress. Even the
Bible, which shows little regard for women's rights, did not speak of Bathsheba's sin but only
of David's sin. Show a little fairness, fellows

(Carol Faulkenberry, 1308 Crest Avenue, Gadsen, AL 35904; e-mail,


alncarol@internetpro.net)

EDITOR'S NOTE: Ms. Faulkenberry became a new subscriber with the January/February
1998 issue of TSR, so she didn't see my article "Inerrantist Tail-Chasing" in the last issue of
1997 in which I used the same analogy to describe the way that Roger Hutchinson thinks. I'm
tempted to say that our use of the same metaphor only goes to show that great minds think

Volume 1990 - 2002 Issue


Page 1181 of 2049
Skeptical Review Edited by Farrell Till
alike, but maybe I had better not, since she has a bone to pick with me in the matter of
Bathsheba's relationship with David. She prefers to think that David raped Bathsheba, but I
find this hard to justify solely on the grounds that the biblical text says that David sent
messengers and "took her." If we assume that there is any degree of historical accuracy in
this story, then we have to wonder why Bathsheba was bathing in a place where David could
see her from his palace roof (2 Sam. 11:2). One could easily surmise that she was familiar
enough with her surroundings to know that she could be seen from the palace and that, being
aware that she was "very beautiful to look upon," she was trying to send a message to David.
At any rate, whether David used force on Bathsheba or not, she certainly had no scruples
against using her influence on him to rise to the status of favorite wife. At the end of David's
life, she was powerful enough to persuade him to name their son Solomon his successor to the
throne rather than Adonijah, David's son by another wife, who had already taken steps to
secure the throne for himself. As this story is told in 1 Kings 1:5-40, we see that Nathan the
prophet (the one who had condemned David for his relationship with Bathsheba) understood
that Bathsheba had enough influence on David to make him act from his deathbed to keep
Adonijah from securing the kingship. The ploy worked when Bathsheba at Nathan's biding
went into David and made him promise that Solomon would be the king. David issued orders
for Zadok the priest to anoint Solomon as king in a coronation ceremony. When I read this
story, I find it hard to have much sympathy for Bathsheba. If she was truly forced into the
relationship with David, she adapted to her circumstances quite well.

Freedom from Ideological Constraints...

I was glad to see William Sierichs' article, "If Dr. Price Denies Zeus, He's Irrational," which
amplified the inconsistency of the inerrantist position. One of the reasons I abandoned
Christianity is that there is no reason, outside of faith, to justify believing one virgin birth over
another, believing one resurrection story over another, believing one holy book over another,
etc. Furthermore, when I observe the mental gymnastics and ra- tionalizations of inerrantists
in an attempt to maintain a belief in a god- inspired inerrant book, I am confident that I made
the right decision to abandon ship.

I don't think it's possible for an inerrantist to understand the intellectual freedom people
experience once they have released their minds from ideological constraints, including that of
an inerrant and literal interpretation of the Bible. Fundamentalists can never be true
freethinkers as long as they force upon themselves the limited framework of ancient writings.

The more inerrantists attempt to defend their position, the more inept and ridiculous they
appear. Of course, this comment is probably feeding some persecution syndrome of
inerrantists. Nevertheless, I think it would be safe to say that if a Muslim employed the type
of rationalizations and postulated absurdities of inerrantists, the inerrantist would waste no
time pointing out this fact to the Muslim.

(Michael Koller, 1995 Galeshead Circle, Germantown, MD 20876-6311; e-mail,


koller@freethinker.org)

The Result of Reading the Bible...

Volume 1990 - 2002 Issue


Page 1182 of 2049
Skeptical Review Edited by Farrell Till
Recently a subscriber to your magazine sat in on a debate between a coworker and me on the
subject of erroneous biblical scripture. He immediately brought to my attention your work.
Needless to say, after reading through his TSR collection, I am very impressed and eager to
join your mailing list. Please sign me up for your free one-year subscription trial offer. I find
your magazine very insightful and have already ordered several of the books you listed in
your arguments. It is refreshing to know that someone is promoting intelligent discussion and
encouraging like thought. I have all too often run into quibbling debates with biblical
apologists who would rather embrace blind faith than consider intelligently exploring their
faith and what they hold to be true. Many a skeptic has been proven wrong on a subject, but at
least we examine the evidence and make a fact-based decision. The nature of our race is
reasoning. Why do some choose to hide from this? Is the wise man the one who reads and
believes? Or reads and understands?

In closing, I will share with you my mother's question concerning my belief in biblical
errancy and my response. "How can you believe that the bible isn't true? You've read it... I
made you." "Exactly, did you expect me not to learn from it?"

(Aaron L. Ring, 1609 McGraft Street Muskegon, MI 49441; e-mail, Iwnt2BGod @aol.com)

EDITOR'S NOTE: If a mother wants her children to believe the Bible, the last thing she
should do is require them to read it. Those who read it just may see the absurdity of believing
it.

Disastrous Effects of Religion...

I enclose the amount of $20 for the renewal of my subscription as well as in payment of that
of the current year, giving up the free first year's subscription. As a Spanish citizen, I suffered
in my childhood the disastrous effects of a Catholic religious education, in the context, as
usual, of a political dictatorship.

My horrible English is not an obstacle, so I encourage you to continue in so necessary a work


as that of the Enlightenment in the way of the big thinkers of our Western culture, from
Montaigne to Popper or Bertrand Russell.

Please, I request that you correct the syntax of this letter if you decide to publish it.

(Dr. Rafael Sartorio, Jacinto Benavente 10, 8 B, 12580 Benicarlo, Castellon, Spain; e-mail,
rafasar@rocketmail.com)

EDITOR'S NOTE: Only a couple of syntactic changes were necessary in Dr. Sartorio's letter.
It's encouraging to hear from another person who has recognized the damaging effects that
religion has on young minds.

A Saint's View of Women...

Your very interesting article in the September/October 1997 issue of The Skeptical Review
entitled, "Keep Them Barefoot[ed] and Pregnant," brought to mind the remarks of St. Alban

Volume 1990 - 2002 Issue


Page 1183 of 2049
Skeptical Review Edited by Farrell Till
(4th century C. E.) on the difficulties of finding a wife: "To find a virtuous women is like
finding the one eel in a bag with 500 venomous snakes. And if you grope in the bag and find
the eel you have but a wet and slimy eel by the tail."

I look forward to further articles.

(Ross Firestone, 188 Mary Street, Winnetka, IL 60093; e-mail, rffphd@interaccess.com)

EDITOR'S NOTE: Obviously, St. Alban was a Bible reader.

Volume 1990 - 2002 Issue


Page 1184 of 2049
Skeptical Review Edited by Farrell Till

Skeptical Review
Volume Nine, Number Three
May/June 1998
Farrell Till, editor

• The Prophecy Farce


The editorial page discusses reasons why biblical prophecy fulfillment is impossible to
prove.
• Deliberate Misrepresentations After All
Editor Farrell Till continues his response to Everette Hatcher's claim that Modern
"critics" have made damaging admissions to their view that the book of Daniel was
written after the sixth-century B. C.
• Price Rides Again
After a long silence, Dr. James D. Price makes another attempt to defend his belief
that Jeremiah accurately predicted the length of time that the Judeans would be held in
Babylonian captivity.
• More Evasions and Misrepresentations
Farrell Till responds to Dr. Price's latest defense of Jeremiah's "prophecy."
• From the Mailbag
There are four pages of letters from subscribers and the editor's comments on many
issues raised in them.

The Prophecy Farce


Farrell Till

Volume 1990 - 2002 Issue


Page 1185 of 2049
Skeptical Review Edited by Farrell Till
What about all of the prophecy fulfillments? Biblicists almost always ask this question when
their belief in biblical inerrancy is challenged. No doubt those who ask the question sincerely
believe that prophecy fulfillment is irrefutable proof that the Bible was divinely inspired, but
in reality the question reflects a naive view of the Bible for which no credible evidence exists.
The "evidence" most often cited by prophecy-fulfillment proponents will usually fall into two
categories: (1) Unverifiable claims by biased biblical writers that certain events fulfilled
certain prophecies. (2) "Fulfillments" of prophecies that were probably written after the fact.
Anyone can successfully refute prophecy-fulfillment assertions by simply demanding clear
evidence when confronted with either category of claims. In other words, if a biblicist cites a
New Testament claim that such and such event fulfilled such and such prophecy, simply insist
on seeing reliable nonbiblical corroboration that the alleged fulfillment event actually
happened. Herod's massacre of the children in Bethlehem would be an example of an
uncorroborated event. The massacre allegedly fulfilled an Old Testament prophecy (Matt.
2:18), but no one has ever found an extrabiblical source that corroborates the lone biblical
reference to this event. If corroborating evidence of a fulfillment event should exist, then
demand evidence that the "prophecy" of this event was undeniably written before the event. In
the debate over Jeremiah's 70-year prophecy, which resumes in this issue of TSR (pp. 4-11),
the demand for clear, undeniable evidence that this prophecy was made before the fact has
proven to be an insurmountable hurdle for Dr. Price, who has yet to produce extrabiblical
corroboration of the prophecy.

Another--and even more effective-- counterargument to use against those who claim that
prophecy fulfillment proves the inspiration of the Bible requires sufficient knowledge of the
Bible to show that many Old Testament prophecies obviously failed. Anyone who is willing
to put the time into learning just a few of those failures will have no problems rebutting the
prophecy-fulfillment claims of any biblicists he/she may encounter. The prophetic tirades of
Isaiah (13-23) and Ezekiel (24-32) against the nations surrounding Israel provide a treasure
house of unfulfilled prophecies. Ezekiel, for example, prophesied that Nebuchadnezzar would
destroy Egypt and leave it utterly desolate for a period of 40 years, during which no foot of
man or beast would pass through it (chapter 20), but history recorded no such desolation of
Egypt during or after the reign of Nebuchadnezzar.

Ezekiel also prophesied that Nebuchadnezzar would destroy Tyre, which would never again
be rebuilt (26:7-14, but Nebuchadnezzar's siege of Tyre failed to take the city, and Tyre still
exists today. A curious thing about this prophecy against Tyre is that Isaiah also predicted that
Tyre would be destroyed, but, whereas Ezekiel predicted that Tyre would be permanently
destroyed and "nevermore have any being," Isaiah prophesied that it would be made desolate
only for a period of 70 years. A comparison of these two prophecies is an easy way to show
the silliness of claiming that prophecy fulfillment proves the inspiration of the Bible.

As noted in my exchanges with Matthew Hogan on Ezekiel's tirade against Tyre


(September/October 1997; November/December 1997), Ezekiel clearly predicted that Tyre
would be destroyed, become a bare rock and a place for spreading nets, and would be built no
more forever (26:7-14, 21; 27:28; 28:19). As Ezekiel did, Isaiah in his prophecies of
destruction against the nations around Israel also predicted the overthrow of Tyre. In 23:1, he
said, "The burden of Tyre. Howl, you ships of Tarshish; for it is laid waste, so that there is no
house, no entering in: from the land of Kittim it is revealed to them." The prophecy continued

Volume 1990 - 2002 Issue


Page 1186 of 2049
Skeptical Review Edited by Farrell Till
in typical fashion through the chapter, predicting waste and devastation, but beginning in
verse 13, Isaiah indicated that the destruction of Tyre would be only temporary, not
permanent:

Look at the land of the Chaldeans! This is the people; it was not Assyria. They
destined Tyre for wild animals. They erected their siege towers, they tore down her
palaces, they made her a ruin. Wail, O ships of Tarshish, for your fortress is destroyed.
From that day Tyre will be forgotten for seventy years, the lifetime of one king. At
the end of seventy years, it will happen to Tyre as in the song about the prostitute:
Take a harp, go about the city, you forgotten prostitute! Make sweet melody, sing
many songs, that you may be remembered. At the end of seventy years, Yahweh will
visit Tyre, and she will return to her trade, and will prostitute herself with all the
kingdoms of the world on the face of the earth. Her merchandise and her wages will
be dedicated to Yahweh; her profits will not be stored or hoarded, but her merchandise
will supply abundant food and fine clothing for those who live in the presence of
Yahweh.

So Ezekiel predicted a permanent destruction of Tyre that would last forever, but Isaiah
predicted just a temporary destruction that would last only 70 years or the estimated lifetime
of one king. The fact is that neither prophecy was ever fulfilled. Nebuchadnezzar did not
destroy Tyre forever, and it was never made desolate for a period of 70 years. Even when
Alexander the Great succeeded in his campaign against Tyre in 332 B. C., the city was soon
rebuilt (Wallace B. Fleming, The History of Tyre, Columbia University Press, p. 64) and has
existed ever since. Matthew Hogan was objective enough in his consideration of the evidence
to admit later that Ezekiel's prophecy against Tyre had failed ("From the Mailbag," TSR,
March/ April 1997, p. 12), but regardless of whether this prophecy failed or succeeded, it was
impossible for both Isaiah's and Ezekiel's prophecies against Tyre to succeed. At least one of
them had to fail, and so proponents of biblical prophecy fulfillment have a problem that they
must explain. If the Bible was really inspired by an omniscient, omnipotent deity, why would
he have directed one prophet to predict a temporary destruction of Tyre and then later direct
another prophet to predict that Tyre would be destroyed forever and never be rebuilt? A
likely answer is that neither prophet was divinely inspired; they both simply blustered in the
exaggerated rhetoric typical of biblical prophets and, working independently, contradicted
each other.

Deliberate Misrepresentation After All


Farrell Till
In my first response to Everette Hatcher's attempt to prove a 6th-century B. C. authorship of
the book of Daniel, I gave him the benefit of the doubt by assuming that he had not
deliberately misrepresented his sources but had only used secondhand references without
checking the context of the quotations they had cited. I have since learned that this was not

Volume 1990 - 2002 Issue


Page 1187 of 2049
Skeptical Review Edited by Farrell Till
the case, because when I was unable to locate some of his sources, Hatcher sent me
photocopies of the original works he had cited. Hence, I have to assume now that Hatcher
knew all along that such scholars as H. H. Rowley and Norman W. Porteous were firm
advocates of the Maccabean period of authorship and that the fragmented quotations that he
lifted out of context from their books were never intended to suggest in any way that the book
of Daniel was written in the 6th century B. C. by a Hebrew prophet who had gained
privileged status in the Babylonian court or that their view of a Maccabean authorship was in
any sense unsupportable.

A familiar type of inerrantist distortion results from the omission of a qualifying but that
follows a fragmented quotation. The first part of the quotation appears to favor the inerrantist
view until the qualifying but statement is read. By eliminating the buts and howevers,
inerrantists try to leave the impression that certain scientists and scholars agree with them.
Hatcher did this in response to my claim that the writer of Daniel obviously "considered the
Median and Persian kingdoms to be separate empires." He quoted Dr. Samuel Driver as
having admitted, "In the book of Daniel the `Medes and Persians' are, it is true, sometimes
represented as united" (March/April 1998, p. 2). I had seen this inerrantist tactic enough to
know that even without having read Driver's work, the parenthetical "it is true" indicated that
a qualifying but statement followed the fragment that Hatcher had quoted. When I was finally
able to check the context of the quotation, I found that I was right. Here is Driver's statement
in context:

In the book of Daniel the "Medes and Persians" are, it is true, sometimes represented
as united (v.28, vi.8, 12, 15, cf. viii. 20), but elsewhere they are represented as
distinct; after the fall of Babylon, Darius the Mede "receives the kingdom" (v.31), and
acts in it as king (vi.1, 2, 15, 25, 26); he reigns for a time--it is not said how long-- and
is succeeded by Cyrus, who is called pointedly "the Persian" (vi.28, cf. x.1, and
contrast ix.1, xi.1); the two horns of the ram in viii.3 are distinguished from each
other, one (representing the Persian empire) being higher (i.e., more powerful) than
the other (the Median empire), and coming up after it. Thus, in the view of the author
of the book, the more powerful rule of Persia is preceded by a "kingdom" of the
Medes, beginning immediately after the death of Belshazzar (S. R. Driver, D. D., The
Book of Daniel, Cambridge University Press, 1900, p. 29, emphasis added).
So when the fragmented quotation is read it context, one can clearly see that Driver did not
intend to express agreement with Hatcher's view that the second empire in Nebuchadnezzar's
vision was a combined Medo-Persian empire, which fundamentalists must contend for in
order to have a prophecy that an everlasting kingdom of God would be established in the days
of the Roman Empire. Driver was merely acknowledging that some verses in Daniel speak of
the Medes and the Persians as a unit, but this does not override the fact that the writer of
Daniel clearly indicated (as Driver noted above) a belief that the Medes conquered Babylon,
ruled over it for a time, and were then replaced by a Persian ruler. The few references to
Medes and Persians, as if they were a single entity or unit, could be easily explained by
merely assuming that the writer was aware that Persia had been a vassal state of Media until
Cyrus the Great rebelled against Astyages in 549 B. C. After this, Media was annexed and
made a satrapy (province) of Persia. Hence, the Persian Empire was in that sense a Medo-
Persian Empire, because it consisted of the territories of both Media and Persia. The problem
in the book of Daniel is that the writer seemed not to understand when the unification took

Volume 1990 - 2002 Issue


Page 1188 of 2049
Skeptical Review Edited by Farrell Till
place. He presented the Medes as the conquerers of Babylon in 539 B. C., when in actuality
the Median empire no longer existed at that time, having already been absorbed by Persia ten
years earlier.

If Hatcher had bothered to read Driver's comment immediately preceding the fragment that he
quoted, he would have seen where Driver pointed out that after the Persian absorption of
Media, it was even commonplace for the Persians to be referred to as Medes:

The second and third kingdoms are, in all probability, the Median and the Persian. The
home of the Medes was in the mountainous country N. and N. E. of Babylon, and S.
W. of the Caspian Sea; they are often mentioned in the Assyrian inscriptions from the
8th century B. C.; but they were first consolidated into an important power by
Cyaxares, B. C. 624-584, during whose reign, in 607, they were the chief instruments
in bringing about the destruction of Nineveh. Cyaxares was succeeded by Astyages,
whose soldiers deserted en masse to Cyrus (B. C. 549); and the empire of the Medes
thus passed into the hands of the Persians. Their name was however long remembered;
for the Greeks regularly spoke of the Persians as Medes (p. 28).
So a reason as simple as this could explain why the writer of Daniel sometimes referred to the
Medes and the Persians as a unit, but it isn't quite so easy to explain how that an official in the
royal court of Babylon at the time of its fall to Persia under the leadership of Cyrus the Great
could have made the mistake of saying that Babylon fell to "Darius the Mede." This is a
problem that Hatcher cannot explain by quoting "liberal" scholars out of context.

On the problem that this historical blunder poses to the fundamentalist view of a 6th-century
B. C. authorship of Daniel, Hatcher quibbled that "nowhere does the writer state that Darius
was `the king of the Medes' or the `king of Media'" (March/April 1998, p. 3). I assume that
Hatcher was resorting to the inerrantist claim that Darius may not have been a king but only
an official whom Cyrus had appointed as governor over Babylon, but this quibble won't work
either. As we will soon see, the word "king" was often applied to "Darius the Mede," and he
was addressed several times as "king." Even if the title of king had not been attributed to
Darius, we could determine that the writer of Daniel still thought that Darius was a king of
Media who had conquered Babylon and then ruled over it as king for a time. Let's notice, for
example, that no one--not even the inerrantists--dispute the fact that Cyrus was king of Persia.
In Daniel 10:1, the writer said, "In the third year of Cyrus king of Persia a thing was revealed
unto Daniel, whose name was called Belteshazzar." The writer here referred to Cyrus as king,
but he also dated his "vision" in this chapter by referring to the year of Cyrus's reign (the 3rd
year). Hence, we have reason to believe that when the writer dated his visions in chapters 9
and 11 as having occurred in the "first year of Darius the Mede" (9:1; 11:1), he was using the
reign of someone he thought was king as a method of dating. There isn't enough space
available to look at even a fraction of the biblical passages where writers used the year of a
king's reign as a way of dating events, so if this was a common way of dating biblical events,
why should we consider Daniel 9:1 and 11:1 to be any different?

Besides the method of dating, there are other indications that the writer considered "Darius
the Mede" to be a king. In 6:1-3, Darius was presented as having powers that could have been
exercised only by a king.

Volume 1990 - 2002 Issue


Page 1189 of 2049
Skeptical Review Edited by Farrell Till
It pleased Darius to set over the kingdom one hundred and twenty satraps, to be over
the whole kingdom; and over these, three governors, of whom Daniel was one, that
the satraps might give account to them, so that the king would suffer no loss. Then
this Daniel distinguished himself above the governors and satraps, because an
excellent spirit was in him; and the king gave thought to setting him over the whole
realm (emphasis added).
We could hardly imagine a "governor" with the power to reorganize an entire kingdom as
Darius allegedly did in this passage. Surely, only a reigning monarch could have made such
decisions, and besides that, the passage twice referred to Darius as "king," so perhaps Hatcher
would not be so wedded to his fundamentalist view of the authorship of Daniel if he would
just read the book a little more carefully. However, if the information above is not enough to
convince him, he should read on in chapter 6. After the reorganization of the kingdom, Daniel
gained so much favor with Darius that the other governors and satraps plotted against him.
So these governors and satraps thronged before the king, and said thus to him: "King
Darius, live forever! All the governors of the kingdom, the administrators and satraps,
the counselors and advisors, have consulted together to establish a royal statute and to
make a firm decree, that whoever petitions any god or man for thirty days, except you,
O king, shall be cast into the den of lions. Now, O king, establish the decree and sign
the writing, so that it cannot be changed, according to the law of the Medes and
Persians, which does not alter." Therefore King Darius signed the written decree (vs.
6-9, emphasis added).
So three times Darius was called king in this passage, and twice he was addressed as "King
Darius." Twenty-two more times in this chapter Darius was either referred to as king or was
called king, so there can be no doubt that the writer of Daniel thought that "Darius the Mede"
was a king who reigned over Babylon after its fall. The last verse in chapter 6 even says that
"this Daniel prospered in the reign of Darius." Kings reign, not governors.

Perhaps Hatcher will say that he said only that Darius was never called the king of the Medes
or the king of Media. That may be true, but the text of Daniel nevertheless indicates that this
is what the writer thought. Darius was twice called "the Mede" (5:31; 11:1), and in a text that
explicitly stated that he was made "king over the realm of the Chaldeans (Babylonians)," the
writer said that he was of the lineage or seed of the Medes (9:1). So in a book that repeatedly
referred to someone named Darius as "king" and that consistently referred to him as a Mede,
why should anyone think that the author of the book meant anything else but that this Darius
was a king of Media who ruled over Babylon for a time between the reigns of Belshazzar and
Cyrus the Great? Regardless, it still remains true that historical records show that Babylon fell
to Cyrus, who then reigned as king in Babylon until the end of his accession year (539-538 B.
C.) and then moved his royal residence to the former Median city of Ecbatana (see "Cyrus,"
Eerdmans Bible Dictionary, 1987, p. 251). So whether the writer of Daniel ever specifically
referred to Darius as "the king of Media" is irrelevant. The text of the book clearly referred to
him as a reigning king in Babylon, but the historical data leave no room for a King Darius
between Belshazzar and Cyrus.

In trying to shore up his quibble about the absence of a statement directly identifying Darius
as "the king of Media," Hatcher said that "Dr. Robert H. Pfeiffer of Harvard University
admitted the author of Daniel was `a very learned man' and `a sage'" (March/April 1998, p. 3).
I looked for the reference that Hatcher gave (Pfeiffer's Introduction to the Old Testament,

Volume 1990 - 2002 Issue


Page 1190 of 2049
Skeptical Review Edited by Farrell Till
Harper and Brothers, 1948, p. 757), and found that the statements alluded to were actually on
page 766. Here Pfeiffer did express the view that the author of Daniel had demonstrated "a
range of reading [that] was wide," but he went on to say that the writer's knowledge was "of
popular tales, circulating orally" (p. 766), so this was hardly a ringing endorsement of the
historical accuracy of the book. In fact, Pfeiffer went on to discuss some of the historical
inaccuracies in the book, including the confusion about "this imaginary Darius living before
Cyrus" (p. 757). Earlier, Pfeiffer had said that "the amount of historical information [in
Daniel] gradually improves as we move from the days of Nebuchadnezzar to those of
Antiochus Epiphanes" (p. 756), so Pfeiffer was clearly a proponent of the view that Daniel
was written during the Maccabean period of the 2nd century B. C.

Hatcher knew all of this, of course, because he later blasted his own critic for recognizing the
"knowledge" of the author of Daniel while rejecting the 6th-century B. C. dating of the book.
"Pfeiffer must have assumed that this `sage' had never read 2 Chronicles 36:20," Hatcher
complained, "where it is said that the Jews were servants to Nebuchadnezzar `and his sons
until the reign of the kingdom of Persia'" (March/April 1998, p. 3). Hatcher sees this as a clear
indication that "the Persian reign came immediately after the Babylonian reign."

There are so many flaws in Hatcher's argument that I hardly know where to begin responding
to them. First, even if we agree with Hatcher's assessment of the statement in 1 Chronicles,
that would mean only that the writer of this book was correctly informed about the order of
regnal succession in ancient Babylon, but it would not prove that the writer of Daniel was so
informed. Hatcher is using the same tactic that we have seen in the articles of Roger
Hutchinson and other inerrantists. He is attempting to prove biblical inerrancy by assuming
biblical inerrancy. At least indirectly, he is arguing that if the writer of 1 Chronicles knew that
a Persian reign came immediately after the Babylonian reign, then the writer of Daniel must
have known it too. In other words, Hatcher wants to assume the inerrancy of everything that
has been published under the title of Holy Bible, and so if it is known that a Persian rule
succeeded the Babylonian rule in ancient Babylon, then no matter how much the book of
Daniel may indicate that the writer thought otherwise, we have to assume that this writer must
not have meant what his text seems to indicate that he thought. To believe otherwise would be
to believe that the Bible is not inerrant, and we simply can't have that. Such is the way that
biblicists think.

There's an even bigger problem in Hatcher's assumption that 2 Chronicles 36:20 should
somehow prove that the writer of Daniel knew that a "Persian reign came immediately after
the Babylonian reign," and that problem is a known historical inaccuracy in the very passage
Hatcher has cited. The Chronicler said that "those [in Jerusalem] who escaped from the
sword" Nebuchadnezzar "carried away to Babylon, where they became servants to him and
his sons until the rule of the kingdom of Persia, but this is not true. Nebuchadnezzar died in
562 B. C. and was succeeded by his son Amel-Marduk, who reigned until 560 B. C., at which
time he was killed in a coup that was led by his brother-in-law Nergal-Sharezer ((Neriglissar).
At this time, Nebuchadnezzar's dynasty ended. Nergal-Sharezer ruled until 556 B. C. and was
succeeded by his son Labsi-Marduk, who was deposed by Nabonidus, who was the king of
Babylon at the time of its conquest by Cyrus. At this time, Nabonidus's son Belshazzar was
possibly serving in Babylon as co-regent, but the "sons" of Nebuchadnezzar had not ruled
Babylon for over 20 years. Hence, the Chronicler was incorrect in saying that the captives

Volume 1990 - 2002 Issue


Page 1191 of 2049
Skeptical Review Edited by Farrell Till
from Jerusalem served Nebuchadnezzar and his sons until the Persian conquest. This
inaccuracy hardly lends support to Hatcher's apparent attempt to prove that the book of Daniel
must be considered historically correct because 2 Chronicles 36:20 is.

An examination of the "critic's admissions" about Daniel shows that these admissions are
relatively minor and in no way damage the widely held scholastic view of a 2nd-century B. C.
authorship. In the next issue, I will discuss the problem of historical inaccuracies in Daniel,
which is one of the major reasons why scholars have assigned a 2nd-century B. C. date to this
book. Hatcher may then respond to my articles if he wishes.

Price Rides Again


Dr. James D. Price
Heavy demands have prevented me from responding promptly to Mr. Till's recent criticisms
of my defense of the fulfilled prophecy in Jeremiah 25:1-13 (TSR, September/October 1997).
Because Mr. Till wrote two articles to criticize my second response, I missed seeing some of
his criticism until I had submitted my most recent article. In this present article I endeavor to
answer the criticisms to which I have not fully responded previously. With this I bring my
defense of fulfilled prophecy to a close. The evidence and arguments, both good and bad,
have been sufficiently presented. The readers can weigh the evidence on both sides and decide
the issue for themselves. I begin first with a few loose ends that deal with some of Mr. Till's
quibbles, then I address the principal issues of the debate.

Nonexistent Prophet: Mr. Till asserted that no evidence exists outside of Israel that Jeremiah
was a real historical person: "There are no nonbiblical records from Syria or Egypt or
Babylon that mention a Hebrew prophet by the name of Jeremiah, who prophesied during the
first year of Nebuchadnezzar's reign that the Jews would be taken into bondage but released
after seventy years" [TSR, September/October 1997, p. 9]. This is another example of the
radical nature of Mr. Till's arguments. There is nothing extraordinary about the existence of
prophets among ancient peoples, so why must I produce extraordinary evidence for the
historicity of Jeremiah? Does he seriously believe that scholars, critics, and historians doubt
the existence of the Biblical Jeremiah? I have yet to read one who does. Even The
Interpreter's Bible, which Mr. Till cited as authority, accepts the reality of his existence: "We
are in the fortunate position of having more authentic information about Jeremiah than about
any other Hebrew prophet, perhaps than any other character in the Old Testament" [vol. 5, pp.
778]. "Yet the book contains a large amount of authentic material from and concerning the
prophet himself than some of the others, such as Isaiah" [vol. 5, p. 787]. Thus Mr. Till's own
authority indicates that much "authentic" information exists about the historical Jeremiah.

Double Standard: Mr. Till accused me of "demanding that we play on an unlevel field,
which would allow [me] to argue from the assumption that biblical miracles, in every case
without exception, happened exactly as recorded," whereas he claims that his "insistence has

Volume 1990 - 2002 Issue


Page 1192 of 2049
Skeptical Review Edited by Farrell Till
been only that biblical miracles should and must be evaluated by the same standards that are
applied to other literary works" [TSR, September/October, 1997, p. 8]. But those statements
are false on two counts. First, I have never demanded that we argue on the assumption that
miracles actually happen. Let Mr. Till cite any place in this debate where I have made that
demand. I have instead insisted that if we debate fulfilled prophecy, the presupposition must
be that miracles may be possible, rather than Mr. Till's assumption that miracles are
impossible. After all, a genuine fulfilled prophecy may be due to a supernatural revelation.
What purpose would there be to debate fulfilled prophecy if the assumption is that miracles
(fulfilled prophecy) are impossible. Under that assumption, there is no debate; the issue is
settled before discussion begins. But that's the even field on which Mr. Till insists we play.
That's stupid! I have insisted that the details of an alleged fulfilled prophecy must be
evaluated by the same standards that are applied to other literary works--that is, on an even
field. That sounds like the same standard that Mr. Till claims he follows. But let's consider
how he plays by those standards.

Mr. Till claims he insists on evaluating the evidence in the Bible by the same standard as he
uses for secular sources. But what modern historian doubts the existence of ancient writers
like Tacitus or Suetonius, Josephus or Herodotus? Yet Mr. Till doubts the existence of
Jeremiah unless there is extraordinary supporting evidence, contrary to almost all
contemporary historians and critics. What historian regards these ancient works to be the
fraudulent product of multiple authors, editors and redactors, produced over a long extent of
time? Yet Mr. Till and his skeptical critics make such extreme claims about the book of
Jeremiah. What historian questions the accuracy of the dates these authors assigned to their
writings? Yet Mr. Till questions the validity of a date in Jeremiah unless there is extraordinary
supporting evidence. Why? Because it is the date of a prophecy. Can anyone seriously believe
Mr. Till is really attempting to play on an even field, and that his evaluation of the evidence is
not driven by his theological agenda?

Mr. Till's Theology: My objection to Mr. Till's continuous harping on the issue of inerrancy
in this debate has resulted in his defense of that tactic. I have stated that the inerrancy of
Scripture is not crucial to my arguments. Fulfilled prophecy can be true even if presented in
an errant Bible. I have never appealed to the inerrancy of Scripture in this debate, nor do I
need to. That is indeed a nonissue in this debate. Mr. Till continues to poison the well with
such irrelevant tirades. On the other hand, the existence of God is relevant to the debate. How
can we debate the possibility of supernatural revelation of prophecy without considering the
possibility of God's existence? Mr. Till objected to my statement that his anti-supernatural
presupposition is based on a sequence of unverifiable fabulous claims. He stated that "I have
no theology, because I have no belief in gods." But when a person develops organized
opinions and arguments about God, revelation, and religion, whether positive or negative, that
is a theology. Only a person who takes no serious thought about God and religion has no
theology.

I assert again that Mr. Till's theological presupposition is based on a sequence of unverifiable
fabulous claims. One of those unverifiable fabulous claims is that God does not exist. Of
course he quibbled: "I do not assert that a deity does not exist; I simply say that I do not
believe in the existence of a deity" [TSR September/October 1997, p. 8]. But this is just a
game of words. Common sense reasoning sees no difference in those two statements in light

Volume 1990 - 2002 Issue


Page 1193 of 2049
Skeptical Review Edited by Farrell Till
of the fact that he identifies himself as an atheist, and he insists on the anti-supernatural
presupposition. If the evidence is insufficient for him to decide one way or the other, why not
remain an agnostic rather [than] opt for atheism? Agnosticism at least keeps the option open
to new evidence and better reasoning. But Mr. Till has clearly closed the door--and his mind.
He admitted that he, an atheist, demands supernatural evidence for supernatural claims--an
oxymoron!

Mr. Till justified his unbelief on the basis of a new hypothesis of the universe by Michio
Kaku, which views matter as eternal and "our universe only one bubble of many fluctuating
from `an infinite ocean frothing with universes,'" or as a "self-reproducing inflationary
universe" [TSR, September/October 1997, p. 8]. In order to avoid the necessary conclusion
demanded by the second law of thermodynamics and the objective evidence of a finite but
expanding universe, these atheistic physicists have expanded their conception into a mega-
universe that is infinite, eternal, and self-perpetuating, one that did not begin with a big-bang,
but itself endlessly and simultaneously "bubbles" many micro-universes in and out of
existence like a frothing ocean. Of course, this hypothesis is unverifiable, making use of such
terms as "immortal," "eternal," and "infinite." Evidently it is proper to apply these terms to a
physical system, but not to a supernatural deity. None of these attributes are verifiable by
physical measurements, but must, by their very nature, be part of the presuppositions of the
hypothesis. The existence of multitudes of other universes is also unverifiable, but must be
part of the presuppositions. Scientists have not probed the depths of our own universe, let
alone detected the existence of others. Contrary to the first law of thermodynamics, such an
eternal, self-perpetuating ocean must presuppose the existence of an infinite and inexhaustible
source of new energy within the system, or else postulate that the system "creates" the new
energy necessary to perpetuate the eternal frothing. Otherwise the second law of
thermodynamics would have forced the frothing ocean into equilibrium, and it would have
settled into a motionless nothingness by now, after the passage of an infinite period of time in
the past. This hypothesis looks very much like the method a physicist would use to describe
pantheism in physical and mathematical terms. All this hypothesis does is add a new
unverifiable fabulous claim to the chain of other unverifiable fabulous claims upon which Mr.
Till's theological presupposition hangs.

I wonder what unverifiable hypothesis Mr. Till accepted as true before this one came along,
and I wonder how gullible he will be to jump on the band wagon of the next unverifiable
hypothesis that comes down the pike. And he calls this common sense! Mr. Till appealed to
the authority of many physicists who are seeking naturalistic explanations for the existence of
the universe. What he failed to mention is that these physicist do not enjoy a consensus about
such naturalistic explanations, but are just as controversial among themselves as the radical
literary critics are. Another thing he failed to mention is that a recent survey in the journal
Nature "revealed that 40% of American physicists, biologists and mathematicians believe in
God--and not just in some metaphysical abstraction, but a deity who takes an active interest in
our affairs and hears our prayers: the God of Abraham, Isaac and Jacob" [The Wall Street
Journal, Wednesday, Dec. 24, 1997]. This does not include those who may be theistic, but not
of that persuasion, or those who are simply agnostics. It is unlikely that a large percentage of
physicists, biologists and mathematicians hold the extreme atheistic theology of Mr. Till,
Kaku, and Linde. I am, after all, in good company.

Volume 1990 - 2002 Issue


Page 1194 of 2049
Skeptical Review Edited by Farrell Till
Modern Miracles: Mr. Till scoffed at the two examples of modern miracles that I submitted
at his request. Particularly he was critical of the claim that Mrs. Marolyn Ford had her sight
instantaneously restored in response to prayer, after being blind for over eleven years due to
the disease of macular degeneration. According to The American Medical Association
Encyclopedia of Medicine, macular degeneration destroys the retinal nerve tissue and replaces
it with scar tissue, and the disorder is untreatable [Charles B. Clayman, ed. New York:
Random House, 1989, p. 658]. That is, once the retinal nerves are destroyed, spontaneous
remission is impossible. Mrs. Ford's sight has been near normal for about 25 years now. Her
restored sight is a miracle like the restoring of sight to the blind as recorded in the Bible. Mrs.
Ford is still living and presumably her medical records are available. Instead of investigating
the report, Mr. Till scoffed because the story was published by a publisher unknown to him,
rather than by a prominent one. What does the notoriety of the publisher have to do with
whether the story is true or not? Why would Mr. Till not investigate the alleged miracle and
expose it as a fraud or a natural recovery, if that's what he believes, rather than merely scoff?
Is this the behavior of one who wants to know the truth? Hardly. It is the behavior of a person
with a closed mind. Let him expose the alleged miracle as a fraud or let him accept what Mrs.
Ford accepts, that her restored sight is a miracle. We know why Mr. Till has not checked this
one out. He claims that he does not accept the possibility of miracles because he has never
seen one. However, if he looks at this one, he might have to change his mind and admit the
possibility. He might have to admit that since miracles actually occur today, they may also
have happened in Biblical times. That would destroy the whole foundation of his theology.

Other Misrepresentations: The following are a few more instances where Mr. Till has
ignorantly, or perhaps willfully, misrepresented the facts: (1) Mr. Till stated: "However, in the
Septuagint... chapter 51 is chapter 28, and 51:64 isn't even in the text" [TSR,
September/October 1997, p. 10]. He is right, chapter 51 in the Masoretic text is chapter 28 in
the Septuagint text. However, 51:64 is 28:54 in the Septuagint. It reads: "and shalt say, Thus
shall Babylon sink, and not rise, because of the evils which I bring upon it" (Jeremiah 28:64,
LXX). Note that what is missing is the words "Thus far are the words of Jeremiah," not the
whole verse. These words are missing because in the way the chapters are arranged in the
LXX, the words of Jeremiah do not end there. Mr. Till failed to correctly represent the
Septuagint here.

(2) Mr. Till stated "Furthermore, chapter 52 in the Septuagint begins with verses that aren't in
the Masoretic [text]: `The word which Jeremias the prophet spoke to Baruch son of Nerias,
when he wrote these words in the book from the mouth of Jeremias....'" This is false. These
words occur in the Masoretic text in 45:1. Masoretic chapter 45 occurs in the Septuagint as
51:31-35, which reads, "The word which Jeremias the prophet spoke to Baruch son of Nerias,
when he wrote these words in the book from the mouth of Jeremias...." Chapter 52 in the
Septuagint begins with the same words as found in the Masoretic text: "It was the twenty-first
year of Sedekias, when he began to reign, and he reigned eleven years in Jerusalem. And his
mother's name was Amitaal, the daughter of Jeremias, of Lobena" (Jeremiah 52:1 LXX). An
English translation of the Masoretic text of 52:1 reads: "Zedekiah was twenty-one years old
when he became king, and he reigned eleven years in Jerusalem. His mother's name was
Hamutal the daughter of Jeremiah of Libnah (Jeremiah 52:1). Mr. Till has erroneously
identified the start of chapter 52, and has contradicted his critical authority, The Interpreter's
Bible, which states that "Ch[apter] 52 is an appendix to the book of Jeremiah added in order to

Volume 1990 - 2002 Issue


Page 1195 of 2049
Skeptical Review Edited by Farrell Till
show how some of his prophecies were fulfilled" [vol. 5, p. 790]. Evidently Mr. Till did not
do his homework. The passage he said does not occur in the Masoretic text really does, and
the passage that he said begins chapter 52 in the Septuagint really doesn't.

(3) Mr. Till stated "that Dr. Price's 605 B. C. dating of the prophecy is clearly wrong and that
`Jeremiah' had directed the prophecy to the exiles who had been captured in 597 B. C." [TSR,
September/October, 1997, pp. 10, 16]. First of all, I did not date the prophecy in 605 B. C.,
Jeremiah did! He wrote: "The word that came to Jeremiah concerning all the people of Judah,
in the fourth year of Jehoiakim the son of Josiah, king of Judah" (Jeremiah 25:1). Historians
agree that the fourth year of Jehoiachim was 605 B. C. Let Mr. Till accuse the historical
authorities of error, not me. If Mr. Till insists that the date given in 25:1 is fraudulent, then let
him produce the proof, or accept the date that all objective evidence supports. Notice also that
the prophecy was directed to "all the people of Judah," not to exiles. Mr. Till has twisted the
words of the text to make it appear that this prophecy and the one in chapter 29 are the same.
But even a casual reading of chapter 29 indicates that his audience was different and the date
was different. Please, Farrell, it is not kosher to alter the text! Let's play the game by the rules.

Supernatural Proof: I was correct when I declared that Mr. Till believes that supernatural
claims demand supernatural proof. He admitted it when he stated: "I'm going to surprise him
and admit that in the cases of some extraordinary claims, this is exactly what I am saying"
[TSR September/October 1997, p. 7]. I am indeed surprised that he would admit what is
obvious from his arguments. By this he contradicts himself because he does not believe in the
supernatural, and he admits that no amount of objective evidence will convince him. His mind
is made up. Now the ordinary understanding of the statement "extraordinary claims require
extraordinary proof" is that such claims expect more than the normal convincing evidence, but
not supernatural evidence. So, for example, instead of a normal hypothesis being statistically
verified at the 95% level, an unusual one may require verification at the 99% level, or an
extraordinary one at the 99.9% level. But to admit that no amount of evidence would be
convincing is what most people call radical.

Poisoning the Well: Mr. Till accused me of ad hominem rhetoric because I referred to him as
a radical skeptic. I'm sure he does not object to the term "skeptic," so he must object to the
term "radical." But by his own admission his skepticism of supernatural events is radically
extreme. He refuses to even entertain the possibility of anything supernatural. No amount of
evidence will convince him otherwise. On the other hand he does not hesitate to refer to me
with rhetorically poisoned terms such as "fundamentalist" and "biblicist" and to my position
as "the Never-Never Land." No one is convinced that he meant the use of those terms in other
than a derogatory sense. His ad hominem rhetoric is admissible in this debate, but mine is not.
Come on, Farrell! Let's be consistent. You admitted that you're a radical.

Null Hypothesis: Now that I have covered Mr. Till's quibbles, it is time to get down to brass
tacks. Mr. Till has objected to my appeal to his failure to prove the null hypothesis, as though
that is a violation of the laws of logic. In this objection he reveals that he does not understand
the difference between deductive and inductive reasoning. Deductive reasoning is governed
by the laws of logic, and proceeds from postulates to conclusions. Such conclusions are
understood to have been proven absolutely, based on the assumed certainty of the postulates.
Such postulates are understood to be self-evident truths that require no proof and that are

Volume 1990 - 2002 Issue


Page 1196 of 2049
Skeptical Review Edited by Farrell Till
universally accepted. Mr. Till's anti-supernatural presupposition does not fall into this
category: it is not self-evident, and it is far from being universally accepted in domains
outside of natural science. Deductive reasoning is used in the disciplines of mathematics,
propositional calculus, and the like. But even in deductive reasoning the null hypothesis is
valid. It is one of the fundamental postulates of logic: the law of noncontradiction:
Propositions A and not-A cannot both be true at the same time. Therefore, if one proves not-A
by valid logical reasoning, then proposition A cannot be true in the same domain of
postulates.

In deductive logic one may not presuppose what is to be proven--that is a logical fallacy
known as circular reasoning. That is why, in this debate, I continue to insist that Mr. Till must
not presuppose that predictive prophecy is impossible. Predictive prophecy is what is to be
proved or disproved. It is circular reasoning to appeal to that supposition in this debate.
Further, in deductive reasoning, one cannot draw conclusions from the specific to the general,
from "some" to "all." That is, one cannot reason that because he has never observed X,
therefore X is impossible--that is the fallacy known as a hasty generalization. But that is the
type of reasoning Mr. Till wants to be accepted in this debate.

Inductive reasoning begins with observed facts and seeks to develop hypotheses and theories
that explain the data. Objective evidence is gathered and studied, and a hypothesis is
proposed. The hypothesis is subjected to statistical analysis to determine how well it explains
the observed data. A hypothesis that has strong statistical support is regarded as a reasonable
explanation of the observed data. Inductive reasoning is used in scientific investigations and
historical research. In this type of research, a common method for validating a hypothesis is to
disprove the null hypothesis, that is, to show that the opposing hypothesis is contrary to the
observed evidence. If the null hypothesis is statistically disproved beyond the 95% level of
certainty, the hypothesis is regarded as valid. This is a widely accepted scientific method. Mr.
Till objected that "there is about as much 'science' in the hermeneutical methods of prophecy-
fulfillment as there is in astrology and fortune-telling" [TSR, July/August 1997, p. 2]. Our
investigation is one of historical research, not of hermeneutics, one of evaluating objective
historical evidence, not of "interpreting" texts. This is the tactic of ridicule without dealing
with the issues.

Mr. Till's example of an alien spacecraft completely misses the point. He stated: "If I cannot
prove that an alien spacecraft did not crash near Roswell, NM, 50 years ago, my failure to
prove that this did not happen in no way constitutes proof that it did" [TSR,
September/October, p. 6]. This indicates that he either does not understand the significance of
a null hypothesis, or he has tried to confuse the readers. The question here is not whether
something crashed near Roswell--the discovered wreckage indicates that something did crash.
The real question is: was the crashed vehicle an alien spacecraft? The null hypothesis to prove
in this case is: The crashed vehicle was not an alien spacecraft, but one of known terrestrial
origin. Presumably objective evidence exists to answer that question. Why the evidence has
not been published and publicly evaluated is another question not relevant to this debate.

A null hypothesis must be proved or disproved on the basis of valid objective evidence, not
on a lack of evidence. To prove the null hypothesis means that the available objective
evidence strongly supports the null hypothesis instead of the hypothesis. To disprove the null

Volume 1990 - 2002 Issue


Page 1197 of 2049
Skeptical Review Edited by Farrell Till
hypothesis means that the available objective evidence does not support the null hypothesis
but the hypothesis. In this debate, the null hypothesis is: (1) Jeremiah did not write the
prediction in approximately 605 BC; (2) the Jews did not enter servitude to the king of
Babylon; (3) at the end of approximately 70 years, king of Babylon was not judged and the
Jewish servitude was not ended. I have presented objective evidence that strongly supports the
fulfillment of Jeremiah's prophecy. Mr. Till has not invalidated that evidence or produced
objective evidence to prove the null hypothesis. Instead, he has presented alternate
possibilities based on unverifiable, subjective theories. His alternate possibilities amount to
new claims. If my claims must be proved, then his claims must be proved. It is not sufficient
to present alternate possibilities without proof. His alternate possibilities must be shown to
satisfy the historical evidence better than my claim does. Otherwise, my claim stands
unrefuted. So far Mr. Till has failed to do that.

Burden of Proof: Mr. Till has accused me of shifting the burden of proof from my position to
his. That this is a false accusation should be obvious to any unbiased reader. I have presented
strong objective evidence to support my position and have shown that his objections are
unreasonable. Mr. Till has objected to my evidence on the basis of subjective theories which
he has offered as an alternative explanation. What I have demanded is that he show how his
alternative hypothesis better satisfies the available objective evidence; that is what I mean by
proving the null hypothesis. That is not shifting the burden of proof. It is demanding a
reasonable justification for a proposed alternative. Otherwise, without such justification, an
alternative remains merely that--an unverified alternative. What is his proposed alternative?--
Jeremiah did not write the prophecy in 605 B.C., but the prophecy is a fraud perpetrated by an
unknown group of unknown authors, editors, and redactors in an unknown sequence of
additions, deletions, and alterations at unknown times after the Jews returned from the
Babylonian Captivity. Now why shouldn't he be challenged to demonstrate how that
extraordinary claim better satisfies the objective evidence? Does he really think his common
sense readers are naive enough to accept that claim as reasonable without convincing
objective evidence? His complaint is an attempt to cover up the weakness of his position. I
don't regard my demand as a violation of the rules of debate. Instead, I regard it as a effort to
get to the truth.

Mr. Till insists that the burden of proof should be one-sided--on my position. He attempted to
justify that stance on the basis that my claim is extraordinary, and that extraordinary claims do
not require disproof. However, he admitted that "in debating an unextraordinary proposition,
both sides should share the burden of proof" [TSR, Sept./Oct. 1997, p. 7]. But let's consider
the extraordinary nature of my claim of fulfilled prophecy. What is extraordinary about a man
making a prediction on a certain date? That is an ordinary historical event which can be
supported by ordinary historical evidence. The fourth year of Jehoiakim is accepted as 605 B.
C. by most modern historians. Why not demand that ordinary historical evidence be produced
to disprove such an event and its date? What is extraordinary about an ancient people being
put into servitude to an alien king? It happened all the time. It is an ordinary historical event
which can be supported by ordinary historical evidence, and this ordinary historical event and
its date are almost universally accepted by modern historians. Why not demand that ordinary
historical evidence be produced to disprove such an ancient servitude? What is extraordinary
about a kingdom being deposed and people being released from servitude? This event and its
date are supported by ordinary historical evidence which is almost universally accepted by

Volume 1990 - 2002 Issue


Page 1198 of 2049
Skeptical Review Edited by Farrell Till
modern historians. Why not demand that ordinary historical evidence be produced to disprove
such an ancient release from servitude? These are all ordinary happenings that any unbiased
skeptic should require only ordinary historical evidence to validate, and should expect
ordinary historical evidence to refute. I have presented objective historical evidence to
substantiate the validity of all three ordinary aspects of my claim. It is not unreasonable for an
unbiased skeptic to expect Mr. Till to produce objective historical evidence to refute these
ordinary aspects of my claim.

Only two aspects of my position are extraordinary: (1) Jeremiah claimed that his prediction
came from God; and (2) the temporal orientation of these three ordinary events constitutes an
extraordinary happening--a fulfilled prediction that is beyond the normal expectations of
human foresight. It is the temporal orientation of the events to which Mr. Till objects. He
asserted that the book of Jeremiah "is the work of many writers over a long period of time"
[TSR, September/October, p. 10]. By such a statement Mr. Till has claimed that the text is
fraudulent. However, as I demonstrate later, a claim of fraud requires proof; such a claim is
invalid without proof.

There is nothing extraordinary about Jeremiah's claim itself; many similar claims are known.
What would be extraordinary is that God actually gave Jeremiah the prediction. However, no
direct evidence can be offered to support that claim. Jeremiah's claim is either true or false
without direct proof one way or the other. Mr. Till stated that I must prove that the prophecy
"was supernaturally revealed to Jeremiah" [TSR, September/October 1997, p. 7]. No, all I
must demonstrate is that a genuine prophecy was fulfilled and that there is a reasonable
possibility that it was supernaturally revealed. Mr. Till's three criteria for a genuine fulfilled
prophecy, which I accepted as valid for this debate, did not include proof that the prediction
came from a supernatural source.

One can allow for the possible truth of Jeremiah's claim based on the verified temporal
orientation of these three ordinary historical events. Assuming that the dates are valid, the
temporal orientation must be regarded as valid evidence of a genuine fulfilled prophecy. Since
all agree that such a fulfilled prophecy is beyond natural expectation, the explanation must be
either that it is a chance coincidence or that it is the result of something beyond mere natural
phenomenon. Both possibilities are equally improbable from a purely naturalistic perspective;
and for an unbiased skeptic, the evidence does not exclude the possibility of the supernatural
origin of the prophecy. The possibility can be excluded only by a fallacious appeal to Mr.
Till's anti-supernatural presupposition. But that presupposition is valid only in investigations
into the laws of nature. This debate is not about the laws of nature but about the possibility of
a supernatural origin of prophecy in the Bible. I have presented evidence that at least one
genuine prophecy in the Bible was fulfilled. This verifies the possibility of the supernatural
origin of prophecy in the Bible. Mr. Till will opt for the possibility that the fulfillment of this
prophecy is strictly due to coincidence. But he should admit that this choice is based strictly
on his preference of an unprovable theological presupposition, not on objective evidence. The
evidence does not deny but rather admits the possibility of a supernatural origin.

Till's Alternate Claim: In an attempt to refute my claim of fulfilled prophecy, Mr. Till
countered with a claim that the book of Jeremiah is a fraud, especially his prophecy.
However, such a claim must be proved. An alternative possibility remains merely a possibility

Volume 1990 - 2002 Issue


Page 1199 of 2049
Skeptical Review Edited by Farrell Till
without proof. Simon Greenleaf, who held the seat of the Royal Professorship of Law in
Harvard Law School and who was a recognized authority on jurisprudence, cited the
municipal law regarding ancient documents: "Every document, apparently ancient, coming
from the proper repository or custody, and bearing on its face no evident marks of forgery, the
law presumes to be genuine, and devolves on the opposing party the burden of proving it to be
otherwise" [The Testimony of the Evangelists, Grand Rapids: Baker Book House, 1965, p. 7].

Now the book of Jeremiah is known to be ancient, and its text comes from the proper custody
of succeeding generations of ancient authorities. These authorities regarded the book to be
genuine as opposed to other books that they dismissed as nonauthoritative. The book is
regarded as a reliable historical resource by both ancient and modern historians. Therefore,
since Mr. Till claims that the book is a forgery, he has the burden to prove that claim. In an
effort to support his claim of forgery, Mr. Till has made the following additional claims that
need proof: (1) the text of the book of Jeremiah is very unreliable; (2) the manuscript
evidence does not support a sixth-century origin of the book; (3) the book was not written by
Jeremiah in the sixth century B. C., but by a sequence of unknown authors, editors, and
redactors in postcaptivity times; (4) Jeremiah was an irrational deceiver; (5) Jeremiah was a
plagiarist; (6) the text was altered in the era of the second temple to give the appearance of a
fulfilled prophecy. Each of these alternative claims require proof, but Mr. Till has failed to
produce any convincing objective evidence for any, only unverifiable subjective theories.

(James D. Price, Ph.D., Professor of Hebrew and Old Testament, Temple Baptist Seminary,
Chattanooga, TN 37404; e-mail, drjdprice@aol.com)

More Evasions and Misrepresentations


Farrell Till
After a long silence, Dr. Price has returned to make another try at proving that the prophet
Jeremiah accurately predicted a 70-year Jewish captivity in Babylon. Price actually sent his
latest article to me in February, but by that time, I had already begun the exchanges with
Everette Hatcher on the authorship of Daniel, so I have had to wait until now to return to the
Jeremiah debate. I have even reshuffled editorial plans in order to resume the discussion with
this issue, Price's article was over 10,000 words in length, so I have had to divide it into two
sections. The nature of his wild assertions throughout the article will require more than just
one response for each section, so after I have made my replies to part one, I will publish part
two and reply to it. Dr. Price indicated at the beginning of his latest article that it will "bring
[his] defense of fulfilled prophecy to a close," but if he wishes to respond to my latest
rebuttals, I will be glad to publish it as long as he addresses arguments I have presented
against the authenticity of Jeremiah's 70-year prophecy. If, however, he just wants to rehash
his charges of "radical skepticism" and "anti-supernatural bias," I see no need to give him an
extended forum for that.

Volume 1990 - 2002 Issue


Page 1200 of 2049
Skeptical Review Edited by Farrell Till
The Nonexistent Claim of a Nonexistent Prophet: Dr. Price has wrongly accused me of
alleging that Jeremiah may not have been an actual historical person, but he either misread my
statement that he quoted in defense of this assertion or else he was setting up another straw
man (as he has done throughout our debate) in order to have something to kick around and
wrongly leave the impression that he is addressing the issues of the debate. Dr. Price quoted
the following statement that I made in the September/October 1997 issue: "There are no
nonbiblical records from Syria or Egypt or Babylon that mention a Hebrew prophet named
Jeremiah who prophesied during the first year of Nebuchadnezzar's reign that the Jews would
be taken into bondage but released after 70 years" (p. 9, emphasis added). I have emphasized
the restrictive "who" clause in this statement to show that I was not claiming that there is no
extrabiblical evidence to confirm the historicity of Jeremiah but that there is no extrabiblical
evidence to confirm that Jeremiah made the 70-year prophecy during the first year of
Nebuchadnezzar's reign. When the statement is examined in context (quoted below), it
becomes even more obvious that this was what I meant. To call attention to how he has
misrepresented its meaning, I have italicized the sentence that Dr. Price pulled out of context.

In the case of Jeremiah's prophecy, the evidence would not have to be supernatural. If
Dr. Price could produce undeniable corroborative evidence that Jeremiah wrote the
prophecy in 605 B. C. and that the text of his book was never revised or edited after
that date, that the Judean captives whom Jeremiah was referring to in chapter 25 were
taken to Babylon that same year, and that they were released from captivity in 535 B.
C., exactly 70 years later, then we would have the evidence that would be necessary to
confirm that an extraordinary prophecy-fulfillment did happen. But where is Dr. Price
going to find that kind of evidence? There are no nonbiblical records from Syria or
Egypt or Babylon that mention a Hebrew prophet named Jeremiah who prophesied
during the first year of Nebuchadnezzar's reign that the Jews would be taken into
bondage but released after 70 years. There were no archives of that time where
original documents were recorded so that they could be referred to later to verify the
accuracy of subsequent copies. In a word, there is simply no way to establish that
Jeremiah made his prophecy at the time that Dr. Price alleges. Hence, there is no way
for him to satisfy criterion number 2 in my list of valid prophecy requirements, which
states that one claiming a prophecy fulfillment must prove that the prophecy was made
before and not after the event (emphasis added).
So I did not assert that Jeremiah may not have existed. Dr. Price obviously misrepresented me
and then spent almost two columns in his article kicking around a straw man that has nothing
to do with the issues of this debate. He could have spent his time more productively by citing
extrabiblical evidence to confirm that Jeremiah made the 70-year prophecy at the time Price is
claiming. He didn't do this, of course, because there is no such evidence to cite. If there were,
he would have cited it long ago, but in the absence of such evidence he has to resort to calling
my position "stupid" (p. 4, this issue) and me a "radical skeptic."

A Reasonable Demand: To insist on extrabiblical confirmation of the date of Jeremiah's


prophecy is certainly reasonable, because, without it, we have only the word of a Hebrew text
that was transmitted by scribes and editors who were committed to the ethnocentric belief that
their people were the chosen ones of the god Yahweh, who had spoken to them in certain
"holy writings" of which Jeremiah was considered a part. Thousands of variations in the text
that they transmitted, not even to mention the existence of two distinct versions of the book of

Volume 1990 - 2002 Issue


Page 1201 of 2049
Skeptical Review Edited by Farrell Till
Jeremiah, are sufficient to justify the suspicion that these guardians of the sacred text had not
been above tampering with it to make it conform to prevailing views of the times. To think
otherwise is to assign a far higher degree of integrity to the Hebrew scribes than biblicists are
willing to grant to the scribal transmitters of other ancient religious documents.

As noted in "Archaeology and Biblical Accuracy" (March/April 1998, p. 1), many biblical
events and characters have been corroborated by extrabiblical records. The Moabite Stone
contains an inscription attributed to king Mesha, who is mentioned in 2 Kings 3; tablets
discovered in Babylonian archives refer to an allotment of sesame oil that was given to
Jehoiachin, the captive king of Ia-hu-da (Judah), thereby supporting the claim in Jeremiah
52:31-34 that the king of Babylon released Jehoiachin from prison and provided him with a
"continual allowance." The Israelite king Jehu was mentioned in Assyrian records, as were
Omri and Ahab. This is the type of extrabiblical corroboration that would give credibility to
the dating of Jeremiah's 70-year prophecy, but unfortunately for Dr. Price, no such
corroboration is known. The fact is that archaeological corroboration of biblical information
has always been confined to the ordinary, and nothing has ever been discovered to
corroborate any of the many extraordinary biblical claims, such as the parting of the Red Sea,
walking about unharmed in a fiery furnace, prolonged darkness at midday, resurrecting the
dead, etc., although these are the very types of claims one would expect to find in
extrabiblical records if such events had indeed happened. Their nature is such that, had they
actually occurred, at least some of them would have attracted enough attention to have been
recorded in secular records of the times. Since such records do not exist, this rationally
justifies the suspicion that they were only religious myths and legends that developed in
superstitious times. Dr. Price may call this view a "radical supernatural bias" if he wishes, but
reasonable people will recognize that the real radical is the person who gullibly accepts such
claims with nothing but religious writings that attest to them. Ironically enough, Dr. Price
himself noted that "(t)here is nothing extraordinary about the existence of prophets among
ancient peoples" (p. 4, this issue). That is exactly right. In ancient times, prophets, seers, and
mystics were as common as dirt, and so the very fact of their commonness is sufficient reason
to demand very convincing evidence before accepting the claim that some of them were
genuine prophets. In the case of Jeremiah, Dr. Price has presented no such evidence.

The Existence of Other Ancient Writers: While punching around on his straw man, Dr.
Price asked "what modern historian doubts the existence of ancient writers like Tacitus or
Suetonius, Josephus, or Herodotus." Since no one has said that Jeremiah was not a historical
character, the statement was irrelevant. I wonder, however, if Price is unaware that the
writings of these ancient authors have been subjected to historical criticisms just as rigid as
those that have been applied to biblical writers. The accuracy and authenticity of some of their
writings have been questioned for the same reason that scholars have challenged the accuracy
and authenticity of some biblical passages: they bear the earmarks of probable exaggeration
and forgery. In "Those Amazing Biblical Numbers" (TSR, Winter 1995, pp. 5-8), William
Sierichs, Jr., discussed probable exaggerations in the sizes of biblical armies, and in so doing
explained how that historical critics have recognized that ancient writers, including
Herodotus, were prone to exaggerate numbers. As for probable forgery, no better example can
be cited than the famous "testimonium" of Flavius Josephus in Antiquities of the Jews
(18:3.3). This passage refers to Jesus as "the Christ" and said that he had been resurrected
from the dead "as the divine prophets had foretold," as well as "ten thousand other wonderful

Volume 1990 - 2002 Issue


Page 1202 of 2049
Skeptical Review Edited by Farrell Till
things concerning him." Since these were unlikely statements to come from the pen of a
person who never converted to Christianity, scholars have long suspected that at least some of
the material in this statement was interpolated by Christian scribes who wanted readers to
think that the historicity of Jesus had been attested to by a famous Jewish writer. Besides this
problem, scholars knew that early Christian apologists like Origen had quoted Josephus in
support of their arguments but had never once cited this particular passage, which would
certainly have been one of their important proof texts had they known of its existence.
Furthermore, the passage concluded with the statement that the "tribe of Christians named
from him [Christ]" were not extinct "to this day." Many scholars, whom Dr. Price will
probably label "radical critics," have seen the expression "to this day" as a "give-away" of late
interpolation, because it implies a substantial time span between the events mentioned and the
date of authorship and, therefore, would not have been a likely phrase that Josephus, a first-
century author, would have used. The use of this same expression in the Pentateuch, by the
way, has led scholars to recognize that at least some sections were written by authors who
were more remote to the events than Moses would have been, so Dr. Price is very wrong if he
thinks that scholars just pick on the Bible but accept without question the authenticity of all
other ancient documents.

Dr. Price asked "what historian regards these ancient works [Tacitus, Suetonius, etc.] to be the
fraudulent product of multiple authors, editors and redactors, produced over a long period of
time." I have just shown that in some cases historians do regard some of these works as such,
but in setting up this straw man to kick around, Price has conveniently ignored a question
more relevant to the issue: What historian regards these ancient works to be accurate in
everything they reported? Does Dr. Price think for one moment that none of the "historical"
information reported in these works has ever been challenged? Since he is a contributing
writer to TSR, Dr. Price is the recipient of a free life-time subscription, so I have to wonder if
he has been reading it. In responding to Roger Hutchinson ("Inerrantist Tail-Chasing,"
November/December 1997, pp. 3-5, 11), I cited several examples of fabulous claims that both
Tacitus and Suetonius attested to and asked why Hutchinson does not accord to these claims
the same consideration that he grants to similar New Testament claims about miracles that
Jesus allegedly performed. Perhaps Price would like to answer that question and tell us if he
thinks that the emperor Vespasian's healing of a blind man and a man with a withered hand, as
both Tactitus and Suetonius claimed were done in the presence of many witnesses, were
actual historical events. If not, then why is he so willing to accept similar claims just because
they are recorded in the Bible? Does he expect us to believe there is no bias here?

Biblical Inerrancy: Price spoke disparagingly of the allusions that I have made to biblical
inerrancy during this debate. He said that it "is indeed a nonissue," but it certainly isn't. It is
paramount to understanding Price's motivation. He desperately wants us to believe that he is
the rational one, who has reached a conclusion about Jeremiah's prophecy solely on the basis
of "objective information" that he has examined, but this is as far from the truth as white is
from black. He believes that Jeremiah accurately predicted the 70-year captivity in Babylon
primarily because the Bible in its present forms indicates that he did, but the truth is that there
is no "objective evidence" that Jeremiah and/or his scribe Baruch wrote the book of Jeremiah
in its entirety and that it has been transmitted to us intact basically as it was originally written.
Without such evidence, Dr. Price cannot make this "prophecy" satisfy the list of criteria that
he himself has said is "satisfactory." As long as a substantial body of biblical scholars exist,

Volume 1990 - 2002 Issue


Page 1203 of 2049
Skeptical Review Edited by Farrell Till
who, after subjecting the book of Jeremiah to proven methods of textual criticism, conclude
that this book is a work of multiple authors over a long period of time, too much doubt about
its authenticity remains an obstacle that Price cannot overcome in his quest to prove prophecy
fulfillment. I will have more to say about this in my responses to the second part of Price's
latest defense of the 70-year prophecy.

A Simple Challenge: Price is upset over my "continuous harping on the issue of biblical
inerrancy in this debate" (p. 4), but he will just have to remain upset. I will not allow him to
deceive the readers of this debate into thinking that he is just the epitome of reasonableness
and objectivity and that his fundamentalist view of the Bible has had nothing to do with his
position on Jeremiah's 70-year prophecy. He wants us to think that he has sat down, with an
open mind, and thoroughly researched this issue and found that the evidence objectively
considered forced him to believe that Jeremiah made an amazing prophecy. "I have stated that
the inerrancy of Scripture is not crucial to my arguments," Price said. "Fulfilled prophecy can
be true even if presented in an errant Bible." Yes, I know that this is what Price has been
claiming, but I also know that it is the claim that Dr. Norman Geisler made about the
resurrection during our debate at Columbus, Georgia. It is the claim that many
pseudoobjective "apologists" like to parade before gullible debate audiences. "What, me
biased?" they love to boast. "Not on your life! I've just looked at the Bible objectively and
found that it is reasonable to believe it." But what objective person could read a book filled
with tales of people being raised from the dead, walking unharmed through a fiery furnace,
making the sun stand still, healing the lame and the blind, parting the water of a sea, routinely
seeing angels and even "God," etc., etc., etc. and actually believe that all of these claims are
historically factual, especially when it is known that the literature of other people living in the
same era was also filled with such tales? There is no objectivity at all in people who have
such beliefs. The truth is that they believe in the resurrection, the prophecies of Jeremiah, and
other extraordinary biblical claims primarily because they want to believe that the Bible is the
inspired, inerrant "word of God," and objectivity has little to do with it.

If Dr. Price truly believes that biblical inerrancy is a "nonissue" in this debate, he can prove
his sincerity very quickly. All he has to do is tell us just one biblical miracle claim that he
thinks is unhistorical or one statement in the Bible that he thinks is errant. This is a simple
challenge, but he will not accept it because he knows that if he makes a public admission that
there is something in the Bible that he doesn't believe is true, his position at Temple Baptist
Seminary will immediately be put in serious jeopardy. A nonissue indeed!

It is certainly true that truth is found in errant books. Who doesn't know this? But it is not true
that biblical inerrantists like Dr. Price believe in extraordinary claims like Jeremiah's
prophecy only because they have objectively examined the evidence and concluded that the
claims are true. Archaeologists could uncover tomorrow a shorter manuscript of the book of
Jeremiah, which was written in Hebrew and predated all other manuscripts of this book but
contained no reference at all to the 70-year prophecy, and Dr. Price and other inerrantists
would immediately go to work to find reasons why this discovery would not prove anything
about the authenticity of the prophecy and other information in the Masoretic text that the
new discovery did not contain.

Volume 1990 - 2002 Issue


Page 1204 of 2049
Skeptical Review Edited by Farrell Till
Till's Theology: After complaining about my references to biblical inerrancy, which he
described as a "nonissue," Price immediately claimed that the existence of God is relevant to
the debate. "How can we debate the possibility of supernatural revelation of prophecy," he
asked, "without considering the possibility of God's existence?" Well, I suppose we could do
it the same way that Price expects us to evaluate his position in this debate without
considering the fact that he is a biblical inerrantist. At any rate, we see how that Price speaks
out of both sides of his mouth. As just noted, he tries to argue that we cannot debate the
possibility of supernatural revelation of prophecy without considering the question of God's
existence, but later, he argued that the supernatural has nothing to do with whether genuine
prophecy occurs and that he has no obligation to demonstrate that it does. In reference to my
criteria of valid prophecy fulfillment, he said that he had accepted my criteria but that they
"did not include proof that the prediction came from a supernatural source" (p. 7, this issue).
So he apparently accepts the relevance of the supernatural when it suits his need but rejects it
when it doesn't.

As biblicists invariably do when they get painted into corners by their inconsistencies, Price
again desperately tried to shift the burden of proof to me, as if I am asserting anything that I
need to prove. He said that "when a person develops organized opinions and arguments about
God, revelation, and religion, whether positive or negative, that is a theology." And so, just
like that, Price turns me into a theologian with "organized opinions and arguments" about
God, revelation, and religion, but he does so without even considering my motivation. My so-
called theology is merely reactive. I am reacting to the absurdities of the real theologians, who
attempt to impose their opinions on others at risk to individual freedom and liberty. If no one
had ever asserted that "God" exists, no one would ever have opposed this view. If no one had
ever asserted that a certain book is "inspired of God," no one would ever have opposed this
view. Likewise, if no one had ever asserted that Jeremiah made an exceptionally accurate
prophecy statement, no one would ever have opposed that claim. Price apparently can't see
that the absurd assertions of the theologians are the reason why skeptics and atheists even
exist in the first place. However, the fact that theologians make assertions that rational
thinkers question, or even outright deny, puts no burden on them to prove that the assertions
of the theologians are not true. All it would take for Price to agree to this basic principle
would be for another religion like Islam or Hinduism to gain a majority in this country and
actively try to impose its religious views on everyone. In such a scenario, I don't think it
would take too long for Price to oppose that new majority, and I'm sure if any of them insisted
that he prove that their beliefs are not true, Price would quickly inform them that he was
under no such obligation. Yet when his religion is the one that is flying high, he expects
everyone who questions it to assume the burden of proving that it is not true.

The Unverifiable Fabulous Claims of Atheism: Price said that he asserts again that my
"theological presupposition is based on a sequence of unverifiable fabulous claims" (p. 4).
"One of those unverifiable fabulous claims," he said, "is that God does not exist." Price
realized that this is an outright falsehood, and so he quickly backpedaled and noted that I don't
assert that "a deity does not exist" but "simply say that I do not believe in the existence of a
deity." Price labeled this a quibble, and so he presumes to tell atheists what they are supposed
to say and what they really believe. If I should say that I do not believe that alien beings are
visiting the earth in spaceships, Price wouldn't have any problems understanding that I am not
flatly asserting that alien beings are not visiting the earth but only that I have no belief that

Volume 1990 - 2002 Issue


Page 1205 of 2049
Skeptical Review Edited by Farrell Till
they are. They may well be visiting the earth, but I have yet to see sufficient evidence to make
me believe that they are. Hence, I have no belief that aliens are visiting the earth. I suspect
that Price is upset because he knows that he makes an untenable assertion when he baldly
asserts that "God" exists, and so he seeks to push me into making the equally unverifiable
assertion that "God" does not exist. Because I refuse to play the game by rules that he wants
to impose, he cries that I'm not playing fairly.

In response to Price's claim that atheism ignores evidence that the universe is temporal, I
showed that some very reputable physicists like Michio Kaku and Andrei Linde, to name only
a few, have postulated theories about a self-reproducing, inflationary universe that has always
existed. In replying to this, Price had the gall to say, "(T)his hypothesis is unverifiable" (p. 5),
as if his hypothesis of an omniscient, omnipotent creator of the universe is verifiable. He went
on to castigate Kaku and Linde for "making use of such terms as `immortal,' `eternal,' and
`infinite,'" and he said this as if there were mountains of evidence to justify the constant
theistic use of such terms as "immortal," "eternal," and "infinite" in reference to the Hebrew
god Yahweh. He said that "(n)one of these attributes [of the universe] are verifiable by
physical measurements," a claim that I am not informed enough in physics to evaluate, but
even if the claim is true, it would put Kaku and Linde in no worse position than Price, because
he certainly cannot verify by physical measurements that his god is immortal, eternal, and
infinite, yet that doesn't keep him from asserting that he is.

Price wagged in the second law of thermodynamics, of course, and said that these "atheistic
physicists" had sought to "avoid the necessary conclusions demanded by" this law by
"expand[ing] their conception into a mega-universe" with characteristics of eternality that
cannot be verified. I suppose Price expects us to believe that a professor of Bible and Hebrew
at a fundamentalist seminary is in a better position to evaluate a scientific theory than are
some of the most renown physicists in the world. I hope he will excuse me for suspecting that
these physicists have given due consideration to the second law of thermodynamics and see
no conflict that it poses to the theory of an eternally existing, self-perpetuating universe.

Price appealed again to thermodynamics and said that "(c)ontrary to the first law of
thermodynamics, such an eternal, self-perpetuating ocean [multi-verse] must presuppose the
existence of an infinite and inexhaustible source of new energy within the system."
Otherwise, he went on to say, "the second law of thermodynamics would have forced the
frothing ocean into equilibrium, and it would have settled into a motionless nothingness by
now" (p. 5). I wonder if Price expected us to say, "Hey, I'll bet that those atheistic physicists
never thought of that!" But if he had read Kaku's article, he would have noticed that he dealt
with that problem:

Creating universes out of nothing may seem to violate cherished conservation


principles until we realize that it takes no energy to create a universe. If the universe is
closed like a bubble, then the energy content of its matter is positive, while the energy
of its gravity is negative: The sum is exactly zero (because it requires positive energy
to lift an object out of a gravitational well, the object's gravitational energy is
negative). Thus it takes no net energy to create new bubbles, which are constantly
being created in the sea of nothing. Universes are for free ("What Happened Before
the Big Bang," Astronomy, May 1996, p. 38).

Volume 1990 - 2002 Issue


Page 1206 of 2049
Skeptical Review Edited by Farrell Till
So I'll just let him argue with the renown physicists who espouse this theory as to whether it
violates any of the laws of thermodynamics. Before leaving this point, however, I will remind
Price that according to the first law of thermodynamics matter can be neither created nor
destroyed. Hence, if matter can be neither created nor destroyed and if matter now exists, that
suggests that matter has always existed, a conclusion that would be in perfect agreement with
the theory of an eternal, self-reproducing, inflationary universe.

Price's "Good Company": Price argued that the physicists I referred to "do not enjoy a
consensus about naturalistic explanations" of the universe "but are just as controversial among
themselves as the radical literary critics are," so we see again how Price responds to opinions
that conflict with his views. If biblical scholars disagree with his interpretation of the Bible,
Price dismisses them as just "radical literary critics," and if physicists suggest naturalistic
explanations for the existence of the universe, he labels them "atheistic physicists" who are
"just as controversial among themselves as the radical literary critics." He apparently believes
that labeling opponents as "liberal" or "atheistic" somehow constitutes refuting their
arguments.

Price offered in support of his opinion of "atheistic physicists" an article in The Wall Street
Journal that claimed "40% of American physicists, biologists and mathematicians believe in
God--and not just in some metaphysical abstraction, but... the God of Abraham, Isaac and
Jacob" (p. 5). From this he reasoned, without any supporting evidence, that probably a much
higher percentage of scientists are theists of some sort or at least "simply agnostics." I
checked the article cited but found only that it claimed that "a recent survey in the journal
Nature" had revealed that this percentage of scientists believed in a personal deity, but when I
looked in both Info-Tract and Readers' Guide, I was unable to find any such survey in Nature
or any other journal. I did find, however, that the author of the article that Price cited was
obviously a believer in the biblical god himself, so I have to wonder about his objectivity.

For the sake of argument, let's just assume that some such survey did report that 40% of
scientists believe in the god of Abraham, Isaac, and Jacob. That would mean that 60% of
them don't. Does Price see no significance in that? Moreover, the way the article stated the
results, one could reasonably conclude that only American or at least western scientists were
surveyed, because it would be unlikely that 40% of scientists in predominantly Hindu or
Buddhist or Shinto countries, such as Indian, China, Thailand, or Japan would be believers in
the biblical god. That would mean that the survey reflected a cultural bias and would therefore
not be an accurate gauge of objectivity in the matter of this god's existence. Furthermore, if
the evidence for the existence of Price's god is so compelling, we have to wonder why only
40% of scientists who grew up in a society in which Biblical religions are in the majority
believe in the personal god of Abraham. Somewhere along the way, 60% have found reasons
not to believe in this god, who must not be as scientifically obvious as Price seems to believe.
He may think that he is "in good company," but when the survey he cited is objectively
analyzed, his company really isn't very impressive. I think I prefer the company of Kaku,
Linde, and the Nobel laureates in physics whom they cited in their articles.

Modern Miracles: To show that he does not limit his acceptance of miracles to those
recorded in the Bible, Dr. Price cited the case of Marolyn Ford as an example of what he
believes is a genuine, modern-day miracle. The claim was that Mrs. Ford had been blind for

Volume 1990 - 2002 Issue


Page 1207 of 2049
Skeptical Review Edited by Farrell Till
eleven years from macular degeneration, which had destroyed the retinal nerve tissue in her
eyes, but, lo and behold, when she and her husband prayed one night, her vision was suddenly
restored, and she can now see with remarkable clarity. My reasons for doubting this claim
were detailed in my last response to Price, and, of course, he considers my rejection to be "the
behavior of a person with a closed mind" (p.5). He wondered why I had not investigated "the
alleged miracle and expose[d] it as a fraud," and the answer is simple: I didn't investigate this
claim for the same reason that I don't investigate the hundreds of other miracle claims that
religious zealots publicize every year. To check them all, I would have to do nothing but
investigate miracle claims. If a genuine miracle should actually occur, I'm sure that it would
create such notoriety that it would be publicized all over the world by responsible news
media. The fact that this did not happen in the case of Mrs. Ford's allegedly miraculous
healing is reason to suspect that it wasn't as miraculous as Dr. Price seems to believe. I might
also add that investigating miracle claims cuts both ways, so I wonder how much time Dr.
Price spent investigating the evidence in Mrs. Ford's healing. Did he check her medical
records? Did he interview her doctors? Or did he just accept her mere word that the miracle
had occurred? Somehow, I suspect the latter.

In the next issue, I will continue my response to Dr. Price's article. When I have addressed all
of his arguments, quibbles, and straw men, I will then publish part two and respond to it.

From the Mailbag


An Expression of Gratitude...

I wish to inform you of my feelings of gratitude for your free publication of TSR. I am still
uninformed of the source who submitted my name to you. However, I suspect it was someone
who wished me to have relief from my state of confusion.

I am in prison and have been for the past six years, and I was raised in a devout Christian
family. I was afraid to express my disbelief and the utter conflict of ideas on what life is all
about. When I first received your paper, earlier this year, it was in a way like touching a hot
potato. I scanned its contents, and the brainwashing in me made me put it down. I did not
throw it away, but I wouldn't allow myself to really read them until my third issue. Yes, it
took six months for me to let myself even read about my own doubts and secret ideas and
opinions of what I have so long been forced to believe. I really don't feel I ever truly believed.
Sure, there must have been a creator of some origin, but why all the myths? Greek myths are
as believable to me, and they too make little sense. The good old phrases "read your Bible"
and "put it in God's hands" are so often recited in prisons--and all over, I am sure. Total cop-
outs! What a way to be! I've found I get much better results and solutions by doing something
actively, not just by "praying" my fate will change.

I hope you will print my letter, because I have no access to e-mail or a web site. Actually,
after six years in this place, when I am released by December of 1998, I will have to discover

Volume 1990 - 2002 Issue


Page 1208 of 2049
Skeptical Review Edited by Farrell Till
what they are. Anyway, if you do choose to print my letter, please print my full address, and
perhaps I will receive some open-minded correspondence. I have written a few people who
have expressed opinions and viewpoints that I am very impressed and inspired by. However,
none have answered me as of yet.

I intend to start my new life of freedom, after paying my debt to society, a reborn, free person,
less burdened by doubt and brainwash, a totally free woman of the world! My liberation from
all the religious propaganda is so refreshingly honest to me and is a new lease on life. I am
actually experiencing a personal freedom within these walls. Praise progress!

Thank you again for all the wonderful work put into TSR and its printed wisdom and food for
thought. I promise you when I am able to pay for my subscription upon release (Texas
prisoners are not paid), I will make up the cost of your generosity of the free subscription I
now receive. Please keep me on your mailing list. TSR has opened many doors to my now
open mind.

My subscription expires 1-98. Please renew!

(Amy Smith, 620118, 2305 Ransom Road, Gatesville, TX 76528)

EDITOR'S NOTE: I can't remember who asked me to send TSR to Ms. Smith. There are about
60 prison inmates on the subscription list, and almost all of these came directly from the
inmates. I do remember receiving the request to add Ms. Smith to the list, but I don't recall
who sent it.

As for continuing her subscription, I send TSR free to all inmates who are unable to pay the
subscription cost. Therefore, she will continue to receive it. I thank her for taking the time to
notify me that it is appreciated.

The Lucifer Principle...

In reading a book called The Lucifer Principle by Howard Bloom, I was suddenly astonished
at something he related. His perspective concerning biological survival of all life, including
human, is that all species and divisions within the species want to propagate their genes and
in order to get rid of competing genes simply kill off the competition when given the chance.

He tells of the langur, an Asian monkey, that when the younger males topple the leader, will
go in and kill off all the baby langurs but leave unhurt the females in their sexual prime. Do
humans do this? Yes, the Yanomami of the Amazon region will raid a neighboring village,
kill or chase away the men, but leave the sexually capable women unharmed. Both the langur
and the Yanomami, after destroying the chance of their competition's genes being passed on,
then mate with the captive females and thus pass on their own genes.

Somewhere a bell began tinkling in my head. Hadn't I read something like that somewhere
else? Oh, my God, yes! The children of Israel would kill off the inhabitants as they went
rampaging through their land in order to reach the promised land. Sometimes they killed
everyone, even the suckling child; at other times they kept all the virgins alive for themselves.

Volume 1990 - 2002 Issue


Page 1209 of 2049
Skeptical Review Edited by Farrell Till
So that wasn't the voice of God after all telling them to do this. It was just the old animal brain
saying get rid of those competing genes. Surprise! Surprise!

(Marie Micheletti, Box 335, Tremont, IL 61568)

EDITOR'S NOTE: That is indeed an interesting hypothesis. Whether it is true or not, it is still
a disgrace to fundamentalist Christianity that its adherents will go to such extremes to justify
the barbarity of the ancient Israelites on the grounds that they were guided and directed in
their take-no-prisoners policy by an omniscient, omnipotent, omnibenevolent deity whose
"higher purpose" is simply beyond our comprehension. What is so hard to understand about
the barbarity of killing babies?

The 24-Hour Day...

Several times in your article "Is Roger Hutchinson for Real?" you used the term 24 hours as
referring to the time of the biblical "creation." However, I would like to point out that the 24-
hour day did not come into existence for millions of years after the world began. According to
the March 1998 issue of National Geographic, p. 74, 3.4 billion years ago each day "lasted
less than 18 hours." This was when the first-known life appeared on earth, which, presumably,
had been in existence for some time.

(R. S. Craggs, 25 McMillian Avenue, West Hill, ON, Canada M1E 4B4)

EDITOR'S NOTE: The intention in my article was to identify the way that the Genesis writer
used the Hebrew word yom, and the writer obviously had no awareness that the earth was
billions of years old. The writer spoke of evening and morning the first day, evening and
morning the second day, etc., etc., etc. Biblical chronology also indicates a belief that the
earth was only a few thousand years old when Genesis was written; hence, it is appropriate to
assume that the writer used yom in the sense of 24-hour periods divided into evening and
morning.

The Internet Again...

Please renew my subscription to The Skeptical Review. Enclosed is a check for $6. I always
read each issue from cover to cover. I've also been reading material from the Internet Infidels
page and the American Humanist Association. I wish I'd had access to all this back when I
was in college. I'm finding others who have the same viewpoint as I do (and are willing to
admit it).

(Julie Blevins, 125 Bercado Place, Apt. 34, Mishawaka, IN 46544-4132)

EDITOR'S NOTE: At the risk of sounding like a broken record, I will say again that the
internet is going to spell serious trouble for traditional religion. When Ms. Blevins was in
college, she didn't have access to information that is now on the internet because the religious
establishment could control what the public read by simply using its influence to keep
objectionable books off the shelves of public libraries. Needless to say, that kind of control

Volume 1990 - 2002 Issue


Page 1210 of 2049
Skeptical Review Edited by Farrell Till
can no longer be exercised. Anyone with an on-line computer can easily access information
that was previously censored.

Another Anonymous Tirade...

Is this all you can find in the whole Bible to disagree with? Then study John 3:16, John 14,
15, 16, 17.

Jesus died and was resurrected so we can live abundantly. If all you do is hunt things to
disagree with you only cheat yourself.

God is very real whether you believe it or not. Saying there is no God is like going into a deep
cave and saying there is no sun. Living with the sun is similar to living without God. How do
you imagine the sun has burned for eons and not burned up. Only God's power can make it so.

Hell is very real whether you believe it or not and everyone had the choice to believe in God
and His Son, Jesus and go to Heaven when you die or not believe and go to Hell. It's your
choice.

Plus if you give your self to Jesus, He will help you in every way.

Or you can struggle thru [sic] life without his help. That is your choice also.

I do not want any of your anti-Christ material. It is from the devil, who is very real also. If he
can keep you doubting God's word, he has you.

(Anonymous)

EDITOR'S NOTE: I receive many anonymous letters, most of which I laugh at and throw
away, but occasionally I like to publish them just to show the pathetic inability of biblicists to
respond intelligently to the evidence that disputes their claim that the Bible is the "inspired,
inerrant word of God." The letter above was sent to me on the back of a print-out of three
rather basic biblical contradictions that someone had posted on the internet with an attached
notice that a free subscription to The Skeptical Review was available on request. Hence, the
anonymous letter-writer incorrectly assumed that I had posted the alleged contradictions,
which incidentally I do agree are contradictions but of the type that can be "explained" by
alleging that they resulted from copyist errors.

On the chance that "Anonymous" may see this issue of TSR, I will try to comment on
some of the things that he said. No, these three contradictions are not "all" that I can
find "in the whole Bible to disagree with." They don't even come close to being all that
I can find to disagree with, but there is no way that I can demonstrate this to
"Anonymous" until he finds the courage to come out of the woodwork and put his
name and address on his attempts to reply to those who find inconsistencies in the
Bible.

Volume 1990 - 2002 Issue


Page 1211 of 2049
Skeptical Review Edited by Farrell Till
Why should I study the scripture citations that "Anonymous" listed? I know what they
say without even taking the time to look them up, because I have "studied" them many
times. I know of nothing in them that would in any way prove that the Bible does not
contain inconsistencies, contradictions, and other discrepancies.

Jesus was resurrected so that we can live abundantly? I don't suppose that
"Anonymous" has any real evidence that he can present in support of that claim, but
until he does produce convincing evidence that this unlikely event happened, he
should not expect me to swoon over another biblicist who is just parroting something
he was indoctrinated to believe but has never really taken the time to investigate.

God is very real whether I believe it or not? I don't suppose that "Anonymous" is even
capable of realizing that the mere assertion that "God is very real" doesn't make it so.
If there is any "real" evidence for the existence of this entity that "Anonymous" says is
"very real," why didn't he present it?

As for going into "a deep cave and saying there is no sun," I think it should be very
obvious to everyone who is living in a deep cave. "Anonymous" is in so deep, in fact,
that he has lost touch with common sense and basic reality.

How has the sun burned so long without burning up? Gee, I'm not even a physicist,
but I can easily explain that. Its size and the amount of fuel in its core accounts for
that. That is a far more reasonable explanation than the God-is-doing-it answer that
theists invariably resort to in matters that seem difficult to understand. In the case of
the sun, however, I see nothing at all difficult about understanding that it contains a
mass so large that it will require billions of years for it to burn out.

Hell is very real whether I believe it or not? If "Anonymous" should be reading this, I
invite him to read again my comments above about his assertion that God is very real
whether I believe it or not. Let's hope that he will begin to understand that reality can
never be determined by merely asserting that something is so.

If I give myself to Jesus, he will help me in every way? Well, I've been there and tried
that, but it didn't work. I'm not very likely to make the same mistake twice.

If "Anonymous" doesn't want any of my "anti-Christ material," all he has to do is let


me know who he is, and I will stop sending it to him if he is on the mailing list. At the
same time, I will inform him that I don't care to receive any more of his anonymous
letters from Albany, Georgia. If he has the courage to sign his name to his letters, I
will be glad to discuss with him whatever issues may be on his mind, but if he just
wants to fire anonymous potshots at me, I don't care to hear from him again. I don't
suppose he would even begin to understand that every anonymous letter I receive--and
I get a lot of them--merely convinces me that the writers of these letters found
themselves unable to respond to arguments against the doctrine of biblical inerrancy.

A Reaction to the "Larger Picture..."

Volume 1990 - 2002 Issue


Page 1212 of 2049
Skeptical Review Edited by Farrell Till
I received The Skeptical Review, Vol. 9, Number 2, in the mail this morning. It was, as
it always is, thoroughly delightful. I can't help but stop whatever it is that I am doing
and read your publication in its entirety immediately. That was the case today. Dr.
Peter Righter's "The `Narrow View' vs. the `Larger Picture'" was just wonderful. That
story he told about the down syndrome baby and the 30 converted hospital workers
had me howling with laughter. I live in a county that is 99% fundamentalist. We have
two weekly local newspapers here and there are stories like Dr. Righter's converted
hospital workers in every issue of both papers each week. Most of our local stories
like this are about a famous atheist (never named), who is speaking before "thousands"
of listeners. A reformed drunk in the front row then stands up and with one pithy
conundrum shuts that famous atheist up but good. (Sometimes the person who stands
up is an old lady.) These stories remind me of Jesus "shutting up" the Pharisees or
Sadducees of the Bible. I've been in plenty of arguments, and no one ever gets in the
last word. I should say that no one ever gets in the last word unless he is telling the
story ten years after its occurrence and is telling it to a group of like-minded persons.
Of course, in the Bible (or the Juniata Sentinel) arguments mostly seem to end with
Jesus (or a local fundamentalist) shooting his opponent down so forcefully that the
opponent is standing there with his tongue hanging out.

Well, I for one have my tongue hanging out in pleasure and gratitude for such a
publication as The Skeptical Review. Your publication is just wonderful.

(Robert L. Brown, HCR 67, Box OH-39, Mifflin, PA 17058)

EDITOR'S NOTE: As I said in my response to Dr. Righter, the tale about the 30
hospital converts has all the earmarks of what I call "pulpit legend," a tale that is told
so often by so many preachers that it acquires the status of an actual event.

The type of stories that Mr. Brown referred to are also legends that circulate and
recirculate, often from pulpit to pulpit, until the gullible come to accept them as real
events. A common type of this legend is the story of the famous biblical skeptic or
atheist, like Thomas Paine or Robert Ingersoll, who confessed his sins on his deathbed
and died a Christian. The way that some Christians circulate these tales, they must
believe that there never has been a famous skeptic or atheist who died without first
confessing his error. The truth is that these stories are invariably assertions for which
no supporting evidence exists.

Another version of the type of story that Mr. Brown encounters so often in his local
newspapers is the claim of the preacher who recalls the time that he silenced a
skeptical professor in his college days. In these tales, all it took was just a short
comment or question to leave the unidentified professor standing in embarrassed
silence unable to respond to his/her student's well chosen rebuttal comment. All I can
say is that if such inept professors actually exist anywhere, they deserve to be silenced.

At any rate. Dr. Righter is no longer with us. A few days after the March/April issue
was mailed, I found a message waiting for me on my answer machine when I returned
from out-of-down business. He informed me that he wanted his name removed from

Volume 1990 - 2002 Issue


Page 1213 of 2049
Skeptical Review Edited by Farrell Till
the subscription list, because he thought I had been "disrespectful" to him in my reply
to his article. Another opinion of his article is published below.

Twisted Logic...

I was appalled by the twisted logic of Dr. Peter D. Righter in his article "The `Narrow
View' vs. the `Larger Picture,'" in which he related the tale of the "conversions" of
thirty hospital nurses and staff members as the result of the birth of a baby with Down
Syndrome. To this horror story, one can only say, "God certainly works in mysterious
ways."

When I was a young, God-believing child and witnessing the great many deaths of
tiny babies and young children, I often asked why these infants had to die so young.
The answers I invariably received were something like, "Oh, God doesn't just want all
old people in heaven; that would be so depressing for him. God also wants young
people and babies with him in heaven to make it a more cheerful, happier place."

Having only an immature mind with which to work, I nevertheless managed to


question God's motives. I would think to myself, "If God is all powerful, why could he
not change old people into young people without having to inflict a lifetime of sorrow
on a young mother who has just given birth and now has to give up her baby?" And,
"If God controls the entire world and universe and already has everything imaginable,
why would he be lonely for one more dead child?"

Any questioning along these lines always brought sharp rebukes from the ministers.
As a child, I did not understand why these preachers were so defensive. As an adult, I
knew: the minister had no more knowledge than anyone else on these matters.

Today, if a father (Abraham of the Old Testament) were to take his son (Isaac), a pile
of wood, and a knife up to a mountain with the intention of stabbing and burning the
boy to death as a "sacrifice and burnt offering to God," the man would be hauled
before a court of law, charged with mental and physical abuse, judged insane, and
placed in a mental institution. The poor child undoubtedly would suffer nightmares for
the rest of his life, still imagining himself bound to this "altar" with his father standing
over him with a knife, intent on committing a horrendous deed. However, in the
Christian Bible, Abraham is praised as an exemplary man, possessing great faith in
God, and has "his seed multiplied as the stars of the heaven and as the sand which is
upon the sea shore" (Gen. 22:1-18).

As Robert Ingersoll so rightly stated, "If a man would follow, today, the teachings of
the Old Testament, he would be a criminal. If he would strictly follow the teachings of
the New, he'd be insane."

(Joseph Cunningham, 320 Grant Drive, Mascoutah, IL 62258-1031)

Enhancement of Sierichs' Article...

Volume 1990 - 2002 Issue


Page 1214 of 2049
Skeptical Review Edited by Farrell Till
There are a number of varying myths of ancient Sumer that have familiar resonances.
In one, Ea and Aruru made a man out of clay and animated him with the power of
divine word. In the later Enuma Elish, the human race was made of clay and animated
by the blood of the sacrificed god Kingu.

The original paradise myth was about the land of Dilmun. The mother goddess created
eight plants, and when Enki ate them, she cursed him. But she showed mercy by
creating eight healing goddesses, one for each of his eight wounded parts. One of
these was Nin-ti, who healed his rib. The name Nin-ti meant "lady who gives life," but
it could also be taken to mean "lady of the rib." This is why, in the Genesis creation
tale, the first woman was made from the rib of the first man. It was an idea suggested
by what was virtually a pun! Otherwise, there was no reason why Yahweh couldn't
have made woman the same way as he made Adam, whose name was a reference to
red clay.

Sumerian mythology also gave us the original story of Noah's ark. The gods regretted
making the human race and decided to destroy it by flood. One man, Ziusudra,
escaped it by building an ark in which he carried samples of all life. In the end, the
ship came down on a mountain in the land of Urartu (later called Ararat), and he
released a dove and a raven.

The Tower of Babel story, furthermore, was based on a real ziggurat at Babylon,
called Esagila, the largest ever built, whose construction was interrupted and
completion delayed for centuries. To this, the Hebrews appended an explanatory myth
to account for diversity in language.

Sumerian civilization is the seminal civilization of the world. Everything that followed
in the Near East is directly or indirectly dependent upon it and is probably more often
than not morally inferior. It's a shame we know so little about these vanished people,
other than that they were not Semites.

(Stephen Van Eck, Rural Route 1, Box 62, Rushville, PA 18839)

Just Who Is Surmising?

For shame, Till! Your remark, "One could easily surmise that... she (Bathsheba) was
trying to send a message to David," sounds like the reasoning of a redneck sheriff who
figures that any attractive rape victim was "just asking for it." I'd say you had to do a
lot of surmising.

Here are some questions to consider: (1) If Bathsheba was attempting to seduce David,
what was her motive? Had she been an unbetrothed virgin, she might have had an
excellent motive, for the law (Deut. 22:28-29) said that a man who had intercourse
with an unbetrothed virgin had to pay the bride price, marry her, and never divorce
her. The possibility of snaring a royal groom might have been enough to make a
young girl forget her modesty. But Bathsheba was a married woman, and the penalty
for committing adultery was death (Deut. 22:22). I know some men consider

Volume 1990 - 2002 Issue


Page 1215 of 2049
Skeptical Review Edited by Farrell Till
themselves irresistible, but I'm almost 60 years old, and I've never yet seen a man with
so much sex appeal that I'd risk death, probably by public stoning, just to have a romp
in the sack with him. Of course, the death penalty applied to men as well, but that
would have been considerably less a deterrent to David. Rich and powerful men often
get away with breaking the law, and David was the most powerful man anywhere
around.

(2) How did Bathsheba know just where and when to take that bath so she could send
her message? Bathing in that time and place was nowhere as simple as it is in modern
America. This is especially true if Bathsheba was performing the ritual purification at
the end of her period of "uncleanness." I understand that this ritual bath requires
submerging one's entire body in water--not the easiest thing to arrange when you have
no plumbing.

So we must ask whether it was David's habit to walk on his roof at a particular time in
the evening? We don't know. But if this wasn't his habit, Bathsheba wouldn't have
known when to take that fateful bath, and arranging a full-body bath on the spur of the
minute wouldn't have been all that easy. And how did Bathsheba know just what
David could see from the roof of the palace? Had she been up there to check it out?
Did he have a favorite place to stand? If not, how could she be sure when he would be
looking her way? Perhaps we should "surmise" that she bathed 24 hours a day for a
week or so, just hoping to catch the king's eye. It seems to me that David seeing
Bathsheba as she bathed was an accident.

Regardless of how it happened, an honorable man would have put the incident out of
his mind. Instead, David "sent messengers, and took her." From the time the
messengers arrived at her door, Bathsheba had no choice. And it makes no difference
whether David used physical violence or simply his authority as king. When a woman
is placed in a situation where she has no choice, where she has to acquiesce to sex, she
has been raped. Ask any judge.

As for Bathsheba using her charms to gain influence over David, it seems apparent to
me that the gal did what she had to do. Biblical women had few rights. All their lives,
they were dependent upon and subservient to men--first, their fathers; then, their
husbands; and, after they were widowed, their sons.

After her rape, Bathsheba found herself pregnant. She was in great peril. When her
husband returned from war, he would of course accuse her of adultery, and then he
could have her executed. Soon David saw to it that she was a widow. This did not
remove the possibility that she would be found guilty of adultery, and, even if she
should escape that fate, it left her with nobody to provide for her. She couldn't go on
welfare, and she couldn't go out and get a lucrative job, either. When David offered to
marry her, she had little choice but to accept, even though she may have hated the
fellow's guts. For that matter, when the king wanted to marry a woman, she probably
had little choice anyway.

Volume 1990 - 2002 Issue


Page 1216 of 2049
Skeptical Review Edited by Farrell Till
But her position still wasn't all that great, for she was not the senior wife. When her
son died, her position became more precarious, for men quite often favored the wives
who gave them sons. The only way she had to secure her position was to keep David
so besotted he would keep returning to her bed until she had another son. But even the
birth of Solomon did not bring real security to Bathsheba, for David had a number of
other sons who were older than Solomon, and it was normally the oldest surviving son
who inherited the father's position and property. Had another son succeeded David,
she and Solomon could have been treated as poor relations, even turned out of their
home. Her only security lay in keeping David so beguiled that she could persuade him
to leave the kingdom to Solomon.

Bathsheba was the victim of a powerful, selfish, thoroughly immoral king. She did
what she had to do to protect her interests and those of her child. I don't see how
anyone can condemn her for that. And I find it absolutely reprehensible that anyone
would take the sketchy Biblical account and use it as grounds for accusing her of
adultery or even immodesty. As I pointed out before, even the Bible, which has a
definite anti-woman bias, doesn't fault her for what she did.

(Carol Faulkenberry, 1308 Crest Avenue, Gadsden, AL 35904; e-mail,


alncarol@internetpro.net)

EDITOR'S REPLY: Why do I have the feeling that nothing I say is going to satisfy Ms.
Faulkenberry, unless I say that I am wrong, she is right, and Bathsheba beyond all
doubt was raped? However, I will not say that, because there is simply insufficient
evidence to support this claim. In the first place, we are talking about an event that
probably didn't happen. The actual historicity of David is very much in question.
Although archaeological records of comparatively minor kings like Omri, Ahab, and
Jehu have been discovered, only two widely disputed references to the "house of
David" have yet been found. This seems strange indeed when we consider that David
was allegedly the person who unified the tribes of Israel into one kingdom. One would
think that if any biblical person had left his mark in archaeological records, David
would have, but this is not the case. Many scholars therefore believe that he was just
the "King Arthur" of Israelite legends. Biblical inerrantists, however, are stuck with
what the Bible says about David, so they have to agree that everything the biblical
record attributes to David actually happened. But that does not explain why Ms.
Faulkenberry is so intent on making a rape victim of the female character in what may
well be only a legend.

Even if we assume that this story is historically accurate, there is really nothing in it
to support Ms. Faulkenberry's insistence that Bathsheba was raped. In comparing me
to a "redneck sheriff," who thinks that an attractive rape victim was just asking for it,
Ms. Faulkenberry begged the very question that is in dispute: if this story actually
happened, was Bathsheba raped? I find nothing in the biblical account to support that
view.

Ms. Faulkenberry asked what Bathsheba's motive could have been, if we assume that a
possible desire to entice David was involved when she bathed where he could see her,

Volume 1990 - 2002 Issue


Page 1217 of 2049
Skeptical Review Edited by Farrell Till
but I have to wonder if Faulkenberry is really that naive. With a little imagination,
couldn't she "surmise" what the motivation could have been? A husband away on
military duty, a king who had a history of adding women to his harem (1 Sam. 25:39-
44; 2 Sam. 5:13-15)--gee, I don't have to strain my imagination at all to guess what
Bathsheba's motivation could have been.

On the grounds that Bathsheba was married and knew that adultery was punishable
by death, Ms. Faulkenberry dismisses the possibility that Bathsheba had any interest
in a sexual relationship with David, but since when has marriage been a deterrent to
adultery? It happens all the time, and history has seen many cases of married women
who had sexual relationships with kings. If she doubts this, Ms. Faulkenberry should
read the works of Tacitus and Suetonius. As for the death penalty, it obviously was not
applied impartially, so why wouldn't a woman of that time have assumed that if she
became sexually involved with the king, she really wouldn't have had to worry too
much about being executed for adultery? The fact that the story indicates that neither
David nor Bathsheba was executed tells us that if there is any historical basis to this
tale, the threat of execution for adultery with the king was probably a minimal
concern to Bathsheba.

Ms. Faulkenberry seems to be basing her position primarily on the fact that the Bible
says that David sent messengers and "took" Bathsheba (2 Sam. 11:4), but before she
puts too much stock in this point, she should do an analysis of the Hebrew word
laqach (took), which was used throughout the Old Testament in the sense of "send for"
or "fetch." When Saul heard that young David had indicated a willingness to confront
the giant Goliath, Saul "sent" (laqach) for David, but there is no textual evidence that
force was involved in bringing David before the king. When Yahweh appeared to
Abraham at the oaks of Mamre, Abraham said to him that he would "fetch" (laqach)
him a morsel of bread (Gen. 18:5). Rebekah told Jacob to go to the flock and "fetch"
(laqach) two kids so that she could make "savory food" for his father (Gen. 27:9). In
Exodus 2:5, Pharaoh's daughter sent her maidens to "fetch" (laqach) the ark of
bulrushes in which the infant Moses had been set afloat. If Ms. Faulkenberry will
check Genesis 27:45; 42:16; Judges 11:5; 1 Samuel 16:11; and 1 Kings 17:10, she
will find just a few of the many places in the Old Testament where the word laqach
was used in the sense of "send for," "fetch," or "go get" with no suggestion of force
involved. Hence, there is no basis for her claim that Bathsheba was raped because
David send messengers and "took" her. The statement could well have meant only that
David sent messengers to let Bathsheba know that the king wanted to see her. In fact,
the NRSV, NIV, NEB, GNB, Jerusalem Bible, and others support this meaning in the
way that they translated the verse. "He sent messengers to fetch her," the Revised
English Bible says, "and when she came to him, he had intercourse with her...." So
where is any implication of force in this statement?

If this event actually happened, it may well be that Bathsheba was an unwilling
participant who submitted to David out of fear, but there is nothing in the text to
support this view. Ms. Faulkenberry objected to my surmising, but her letter contains
far more surmises than space will allow me to address. She surmised, for example,
that Bathsheba "beguiled" David as the only means of securing her position by

Volume 1990 - 2002 Issue


Page 1218 of 2049
Skeptical Review Edited by Farrell Till
producing another son. If so, she succeeded famously, because she actually produced
four sons for David (1 Chron. 3:5). Furthermore, the Bible claims that Yahweh
informed David that his kingdom would be established forever through a son who
would be born to him and named Solomon, so if David really believed that Yahweh
had made such a promise to him, Bathsheba, as the mother of this son, would hardly
have needed to beguile David in order to secure her position. Ms. Faulkenberry is
simply surmising far more than the Bible text justifies. It claims that Bathsheba had
sexual relations with Solomon and later became one of his wives but in no way
indicates that she was forced into either relationship. It claims that she had gained
such favor with David that she was able at the end of his life to manipulate him into
naming Solomon as the son who would succeed to the throne, although the custom of
primogeniture should have given the kingship to an older half-brother. It really is
hard for me to work up much sympathy for poor little Bathsheba. If she was raped, she
certainly adjusted well to a situation that was forced upon her.

Volume 1990 - 2002 Issue


Page 1219 of 2049
Skeptical Review Edited by Farrell Till

The Skeptical Review


Volume Nine, Issue Four
July/August 1998
Farrell Till, editor

• "Primary Colors of the Bible"


The critical methods that an English professor at Vassar College used to determine the
identity of the anonymous author of Primary Colors lends support to the views of
biblical critics who say that some books of the Bible are the works of multiple authors.
• "Where Is the Objective Evidence?"
Editor Farrell Till continues his responses to what Dr. James Price said would be his
"final word" in the matter of Jeremiah's 70-year prophecy.
• "Bad History in the Book of Daniel"
Editor Farrell Till discusses some of the cases of "bad" 6th-century B. C. history in the
book of Daniel that have led scholars to conclude that it was not written by a Jewish
captive who had become an important official in the Babylonian empire.
• "Theological Development Rather Than Revelation"
Robert Dornbush traces the rise of the Jewish view that descendants should not suffer
for the "sins" of their ancestors. He shows that the apparent textual conflicts on the
subject resulted from some writers having retained references to the earlier view that
was later preached against by prophets like Jeremiah and Ezekiel.
• "A Letter from...."
The popular biblical apologist Gleason L. Archer writes a letter in condemnation of
the efforts of editor Farrell Till to expose errors in the Bible.
• "The Best Inerrantists Have to Offer"
Farrell Till responds to Archer's letter.
• "From the Mailbag"
Readers express their views on a variety of subjects discussed in The Skeptical
Review.

Volume 1990 - 2002 Issue


Page 1220 of 2049
Skeptical Review Edited by Farrell Till

Primary Colors of the Bible


Farrell Till

When the anonymously written novel Primary Colors was riding high on the bestsellers' list,
speculation ran wild about the identity of the author. During the frenzy, an editor at New York
Magazine saw a story about a Shakespearean professor at Vassar College who had shown by
linguistic analysis that William Shakespeare was the author of an anonymously written poem.
The magazine then engaged Professor Don Foster to examine Primary Colors to see if he
could determine who had written it. Foster analyzed various samples of published writings,
and finally saw in the articles of Joe Klein, a columnist at Newsweek, various distinguishing
characteristics in his literary style that to his satisfaction identified Klein as the author of
Primary Colors. When Foster began his investigation, he had never even heard of Joe Klein,
but after he had failed to uncover the identity of Anonymous by analyzing the writings of
various political insiders, he broadened his search and finally discovered in Klein's articles the
clues he had been looking for.

After Foster announced his conclusion, Klein appeared on television on February 15, 1996,
and denied that he had written the book. "It's not me; I didn't do it," Klein said. "This is silly."
Foster, however, stuck by his guns and insisted that the stylistic evidence was too apparent to
deny. Finally, on July 17, 1996, Klein appeared on television again to announce that he was
indeed the author of Primary Colors. Literary sleuth Don Foster, who had found himself the
butt of ridicule after Klein's denial, had been vindicated.

In April, the TV news magazine Dateline interviewed Professor Foster, who explained to
reporter John Hockenberry the methods he had used to trace an anonymously written work to
Joe Klein. He explained that he first did a textual analysis of Primary Colors to identify
words, phrases, and expressions that were repeatedly used and to look for "quirky
expressions" and peculiarities of punctuation. Afterwards, he simply analyzed writing samples
until he located a consistent usage of the same "telltale" signs of authorship. Sample after
sample was rejected until finally he happened onto the writings of Joe Klein and found what
he had been looking for. The literary quirks and features that Foster had isolated in Primary
Colors occurred with such frequency in Klein's articles that despite Klein's initial denial,
Foster knew he was Anonymous.

To test Foster's literary forensic methods, Foster was challenged by Dateline correspondent
John Hockenberry to find a sample of his own writing in five samples that he submitted to
Foster. After analyzing a book that Hockenberry had written, Foster then looked for
Hockenberry's literary quirks in the five samples. He found not only the sample that
Hockenberry had written but also identified the other samples as the writings of P. J.
O'Rourke, Henry David Thoreau, Jack Kerouac, and an unknown person, possibly
Hockenberry's wife, who had been influenced by Hockenberry's style. He was right in every

Volume 1990 - 2002 Issue


Page 1221 of 2049
Skeptical Review Edited by Farrell Till
case. Dateline announced that Foster's work had been so impressive that he is now being
consulted by police departments to assist them in solving puzzling cases that require the
identification of writing styles. Foster has received so many of these requests that he is taking
a leave of absence from Vassar next academic year in order to work on police cases.

The work of scholars like Don Foster confirms that textual criticism is a valid science, yet
when biblical scholars who analyze the Bible and find linguistic signs of forgery,
interpolations, multiple authorships, and such like, biblical fundamentalists invariably dismiss
their opinions as the efforts of "liberal" or "radical critics" to discredit the Bible. On page 12
of this issue is a letter from Broderick D. Shepherd, the author of a book that disputes the
traditional dating of the book of Daniel, and he reports that he too often finds biblical
fundamentalists quick to dismiss his critical opinion of Daniel by simply labeling it
"liberalism." In past issues of TSR, fundamentalist views about the authorship of the books of
Jeremiah and Daniel have been challenged by documentation from the works of reputable
scholars only to be dismissed by fundamentalist spokesmen as just the conclusions of
"radical" or "liberal" or "atheistic" critics, as if these fundamentalists are completely unaware
of a universally recognized principle of logic that says the truth or falsity of a claim is
independent of its source. So whether the proponent of a critical theory is conservative,
liberal, or somewhere in between has nothing to do with whether his theory is true or false.
Critical theories must be evaluated on the basis of their merits or lack of the same and not on
the basis of who proposes them.

Professor Don Foster proved that writers leave in their works signs of their identity that are
just as telling as fingerprints or DNA that may be left behind at a crime scene. If the identify
of the person who left the fingerprints or DNA is not known, forensic experts could still
determine that the same person was involved in crimes A and B if the same prints or DNA
should be found at both scenes. If the fingerprints and DNA found at both scenes are
different, this would give investigators reasonable cause to conclude that the crimes were
committed by different persons. Literary analysis works the same way. If the identity of a
writer is not known, experts can still determine that documents A and B were written by the
same person if the same stylistic patterns and other literary imprints are found in both
documents. Likewise, if the style, vocabulary, and other literary imprints are very different in
both documents, scholars can be reasonably sure that the documents were written by different
persons.

These are the methods that biblical critics use to determine such matters as the probability that
the same person who wrote the epistles to the Corinthians and Romans did not write the
pastoral epistles, even though all of these documents claim that they were written by the
apostle Paul, or the likelihood that noticeable internal differences in style, vocabulary, and
theme within such books as Isaiah and Jeremiah point to multiple authorships rather than the
tradition that both books were each written by just one person. Although textual critics
undoubtedly reach some wrong conclusions, there is certainly no reason to doubt the general
reliability of the critical methods that they use to make these determinations. If it is true, as
Professor Don Foster proved, that those who write in English put identifying linguistic
markers in their works, there is no reason to believe that this is not true of those who write in
other languages. So if linguistic analysis of ancient Hebrew and Greek documents indicates
that there are "primary colors" in the Bible that dispute traditional views of authorship,

Volume 1990 - 2002 Issue


Page 1222 of 2049
Skeptical Review Edited by Farrell Till
Christian fundamentalists who don't like these scholarly conclusions are going to have to do
more in defense of their position than just yell, "Liberalism!"

Where Is the Objective Evidence?


Farrell Till

In defending his claim that Jeremiah accurately predicted that the Jews would be held in
Babylonian captivity for 70 years, Dr. Price has, in effect, argued that to disprove his claim I
must present "objective evidence" that Jeremiah did not make the prediction. In so doing, he
has flouted the widely accepted rule of evidence that requires the asserter to assume the
burden of proving his assertion. Basically, the extent of Price's evidence that Jeremiah's
prophecy is genuine has been that it is in the book of Jeremiah, the present form of which
religious traditionalists believe was written in its entirety by the prophet Jeremiah. In response
to this claim, I have shown that (1) two substantially different versions of Jeremiah have been
known to exist since the 3rd century B. C. and (2) many very reputable biblical scholars have
textually analyzed the book of Jeremiah and determined that it is not the work of just one
person but several. These facts cast serious doubt on the one thing that Dr. Price must
establish beyond reasonable doubt before he can claim that Jeremiah made an amazingly
accurate prophecy statement, and that one thing is that the book was written in its entirety at
the time he and other traditionalists claim that it was, i. e., prior to the end of the Babylonian
captivity of the Judean Jews.

The Null Hypothesis Again: Dr. Price spent an entire page talking again about my
responsibility to prove the "null hypothesis." This section was much too long to permit me to
respond to every minute point he tried to make, but a major premise in his argument seemed
to be a reiteration of his belief that his position will prevail unless I can prove that it is not
true or unless I can prove that a "null hypothesis" is more likely. Never mind that logicians
have long recognized that the absence of negative evidence does not constitute positive
evidence, just as the absence of positive evidence does not constitute negative evidence. No
matter how widely accepted this principle is, Dr. Price seems determined to win by default,
even though he hasn't even come close to proving his proposition. Perhaps it is the recognition
that he has failed to prove his proposition that has turned him to arguing that he has won by
default.

He appealed to the law of noncontradiction, which recognizes that a proposition cannot be


both A and not-A at the same time (May/June 1998, p. 6). Hence, he seems to be arguing that
if his proposition is A, and I cannot prove not-A, then it must be that A is true. That A and
not-A cannot both be simultaneously true is certainly a recognized principle of logic.
However, the failure of an opponent to prove not-A does not automatically make A true. It
may be that not-A is true, but the opponent was simply unable to establish its truth. Surely,

Volume 1990 - 2002 Issue


Page 1223 of 2049
Skeptical Review Edited by Farrell Till
Dr. Price can recognize this. Otherwise, he will have to agree that I am just as entitled to
argue that unless he can prove that not-A is untrue, then, by default, I will have established
that his proposition A is untrue.

In a long elaboration, he argued that a "hypothesis that has strong statistical support is
regarded as a reasonable explanation of the observed" and from this went on to argue that a
"common method for validating a hypothesis is to disprove the null hypothesis" or, in other
words, to show that "the opposing hypothesis is contrary to the observed evidence" (May/June
1998, p. 6). Dr. Price probably didn't recognize the serious damage that his own line of
reasoning has inflicted on his position, because his hypothesis is that Jeremiah, prior to the
end of the Judean captivity in Babylon, accurately predicted that the bondage would last for
70 years. There are various "null hypotheses" that could be offered as alternatives to this
proposition: (1) Jeremiah did write the prophecy before the end of the captivity, but he
intended the number 70 to have the figurative meaning of just a long, indefinite period of time
and not 70 literal years. (2) Jeremiah did write the prophecy before the end of the captivity
and used 70 in a literal sense, but the actual time of the captivity fell far short of 70 years. (3)
Postexilic editings of the book revised sections to make it appear that Jeremiah had accurately
predicted the length of the captivity.

In my previous responses, I have already discussed these alternatives to the traditional view
that Dr. Price espouses, but I don't recall that he has disproven any of them anywhere close to
the "95% level of certitude" that he indicated is necessary (May/June 1998, p. 6) to show that
the original hypothesis is "valid." By his own, standards, then he has not proven his
hypothesis is true because he has not disproven any of the null hypotheses beyond a 95%
level of certitude. How, for example, could he prove beyond a 95% level of certitude that
Jeremiah didn't intend the number 70 to be understood in a figurative sense? Many biblical
scholars think that this was the case, just as they think that Isaiah intended the number 70 to
represent only a long, indefinite period of time in his prophecy that Tyre would be destroyed
and forgotten for 70 years (23:15-17). How could Dr. Price or anyone else prove with a 95%
level of certitude that Jeremiah did not intend the number 70 to have the same figurative
meaning? If he did so intend, then it would not be true that he predicted the exact period of
captivity with amazing accuracy. It would only be true that Jeremiah had predicted that the
captivity would last for a long period of time, and there is nothing exceptional about a
prediction as indefinite as this.

The Second Alternative Hypothesis: As I have shown in earlier responses to Price (see "A
Figurative View," July/ August 1997, p. 6), if Jeremiah did intend for the 70 years to be
understood literally, then we have a prophecy failure rather than a prophecy fulfillment,
because the captivity fell far short of lasting 70 years. In his latest article, Price screamed, "I
did not date the prophecy in 605 B. C., Jeremiah did!" (May/June 1998, p. 5), but his problem
is that he can't seem to distinguish between when the prophecy was allegedly spoken and
when the captivity that it predicted began. He has fixed 605 B. C. as the date when the
captivity began, but I have shown (July/August 1997, p. 5) that 597 B. C. was the earliest date
that can be assigned to the beginning of the captivity. Jeremiah (25:1) said only that the "word
of Yahweh" came to him in the first year of Nebuchadnezzar's reign (which would have been
605 B. C.), but he did not say that the captivity would begin that year. He went on to predict
that a captivity in Babylon would happen and that it would last for 70 years (vs:11-12), but

Volume 1990 - 2002 Issue


Page 1224 of 2049
Skeptical Review Edited by Farrell Till
there is no way that Price can distort this passage to mean that Jeremiah predicted that the
captivity would begin in the first year of Nebuchadnezzar. Price can't seem to distinguish
between when Jeremiah said that the revelation came to him and the time that the captivity he
prophesied about actually began. The matter is as simple as if someone in our time had
claimed that in the first year of Bill Clinton's presidency a psychic insight came to him/her
warning that there would be 20 years of famine in Africa. If a famine in Africa should then
begin in 1998 and last until 2012, this could not in any sense be considered a remarkable
prophecy fulfillment, because the famine did not begin until 6 years after the prediction was
made and then lasted only 14 and not 20 years. In other words, 20 years after the time the
psychic prediction was made would not be the same as the actual duration of the famine,
which would have been only 14 years.

This is exactly the case with Jeremiah's 70-year prophecy. Even if we concede to Price for the
sake of argument that Jeremiah wrote this book in its entirety, it would still be true that the
captivity began in 597 and not 605 B. C. As I have shown by citing the letter that Jeremiah
sent to the captives in Babylon (29:1-14), the prediction of a 70-year captivity was made to
those who had been "carried away from Jerusalem to Babylon after that Jeconiah the king and
the queen-mother, and the eunuchs, and the princes of Judah and Jerusalem, and the
craftsmen, and the smiths were departed from Jerusalem" (29:1-2), and this event is described
in 2 Kings 24:10-17. All of the "objective" historical records that Price has talked about date
this event at 597 B. C. when Nebuchadnezzar sacked Jerusalem the first time. Since Babylon
fell in 539 B. C. to Cyrus, who granted in the first year of his reign permission for the Jews to
return to their homeland, even if we allow two years for them to return to Jerusalem (as Price
demanded), the time involved would still fall 10 years short of 70, not even to mention that
the two years the Jews were traveling back to their homeland could hardly be construed as
"captivity." Yet Price has leaned over backwards to try to get 70 years out of 58, so it is no
wonder that he has been distorting Jeremiah 25:1 to try to make it mean that the captivity
began in 605 B. C. He needs as many years as he can scrounge up, but even if we allowed
him to date the beginning of the captivity from 605 B. C. and then allowed him two years for
the captives to travel back to Jerusalem, he would still have only 68 years. Knowing this, he
has talked about "round numbers," but anyway he slices the material he has presented in this
debate, it still comes out baloney. He has come nowhere close to disproving this second "null
hypothesis" with a "95% level of certitude." The fact is that this is not even a hypothesis. It is
recognized as a historical fact supported by all the "objective evidence" that the Babylonian
captivity of the Judean Jews did not last 70 years. Some biblical scholars even date the
captivity from Nebuchadnezzar's second sacking of Jerusalem in 587 B. C., when he also
destroyed the city, left it in ruins, and "carried the population away captive" (2 Kings 25:1-
21). Even the book of Jeremiah itself records these two sackings of Jerusalem, dating the first
one in the 9th year of Nebuchadnezzar (52:4) and the second one in the 19th year of
Nebuchadnezzar (v:12). Needless to say, neither one of these could have happened in 605 B.
C., the first year of Nebuchadnezzar's reign. Hence, the "objective evidence" that Dr. Price
has been asking for shows that the Babylonian captivity lasted somewhere between 50 and 58
years, not 70. I would say, then, that if Dr. Price wants me to prove a "null hypothesis," I have
done so. That null hypothesis would be that the actual period of Babylonian captivity fell
several years short of 70. That being so, it cannot be claimed that Jeremiah was remarkably
accurate in predicting that the Judean Jews would be in captivity for 70 years. Dr. Price

Volume 1990 - 2002 Issue


Page 1225 of 2049
Skeptical Review Edited by Farrell Till
apparently has a hard time seeing that 58 is not 70 and isn't even close enough for 70 to be
considered just a "round number."

The Third Alternative Hypothesis: In my previous responses, I also showed that


responsible biblical scholarship has textually examined the book of Jeremiah and determined
that it was written not by one person but by several people over a long period of time. Hence,
another possible "null hypothesis" would be that the 70-year prophecy was a later addition
and that Jeremiah himself made no prediction about the duration of the captivity. Dr. Price's
response to this has basically been that it is just the opinion of "radical" or "atheistic" critics,
but as the front-page article of this issue shows, linguistic experts can use proven methods of
textual analysis to determine not just the authorship of documents but also sections where
forgery, revisions, and redactions have occurred. This type of literary criticism does not rely
on guesswork or wishful thinking but proven methods that use stylistic comparisons and
analyses of vocabulary, grammar, and theme to determine that ancient documents were
produced by multiple authors. If Professor Don Foster could use such methods to identify the
author of Primary Colors, there is no reason to think that experts in biblical Hebrew could not
use the same methods to determine that Jeremiah has undergone alterations and revisions and
was, consequently, written by more than one author. Furthermore, most of the biblical
scholars who accept this critical view of Jeremiah are not "radical atheists" but are in many
cases practicing Christians or Jews. At any rate, the body of scholarship that accepts this view
of Jeremiah is large enough and prestigious enough that Dr. Price is going to have to do more
than just scream, "Radical critics!" in order to disprove it beyond the 95% level of certitude
that he himself has said is necessary in order to establish the validity of the opposing
hypothesis.

Did I misrepresent the Septuagint? Dr. Price accused me of misrepresenting the Septuagint,
because I said that 51:64, which says, "Thus far are the words of Jeremiah," is not in the
Septuagint. I said this in response to Price's claim that chapter 52, which was obviously
written at least 37 years into the captivity (v:31), would in no way disprove that all of the
book up until this point had been written by Jeremiah or his scribe Baruch. He justified this
view by noting that 51:64, which is the last verse of Masoretic chapter 51, says, "Thus far are
the words of Jeremiah." In other words, he was arguing that the book of Jeremiah was
admitting that what Jeremiah himself spoke or wrote ended at 51:64 and what followed was
an addition that was made by someone else, possibly the scribe Baruch. Since the final
chapter (52) is essentially the same in both the Masoretic and Septuagint versions, all I
wanted to do was point out that chapter 51 in the Septuagint did not end with, "Thus far are
the words of Jeremiah." Therefore, we must wonder why if the intention of this statement was
to inform readers that someone besides Jeremiah had written the last chapter, it was not also
put before the last chapter in the Septuagint to let readers of this version know that Jeremiah's
words had ended and that someone else was writing the final chapter. After all, Dr. Price
seemed to be arguing that despite the many differences in the two versions, they were both
inspired accounts that Jeremiah had written.

Dr. Price argued that "(t)hese words are missing because in the way the chapters are arranged
in the LXX, the words of Jeremiah do not end there," but this is a quibble that I would have
thought was beneath the level of scholarship that Dr. Price claims to represent. If the purpose
of these words was to put a "disclaimer" before the final chapter of the Masoretic text to let

Volume 1990 - 2002 Issue


Page 1226 of 2049
Skeptical Review Edited by Farrell Till
readers know that Jeremiah's words had ended and someone else had written the last chapter,
then why wouldn't the omniscient, omnipotent Holy Spirit, who was presumably inspiring all
of this writing, have considered it appropriate to put the same disclaimer before the same final
chapter in the Septuagint version? After all, if the final chapter was written by someone
besides Jeremiah, chapter 51 in the Septuagint would have been the end of Jeremiah's words
just as a different chapter 51 was the end of Jeremiah's words in the Masoretic version. If not,
why not?

The same principle would apply to Dr. Price's response to what I had said about the beginning
verses of Septuagint chapter 52, which says, "The word which Jeremias the prophet spoke to
Baruch son of Nerias, when he wrote these words in the book from the mouth of Jeremias...."
I noted that Masoretic chapter 52 does not contain this statement, but Price has countered by
saying that these words are found in Masoretic 45:1. In so doing, he has again failed to see a
significant point. In Masoretic 45:1, they are separated from the final chapter, which Price
admits that Jeremiah did not write, but in Septuagint 52, they come immediately before a long
passage (chapter) that Price agrees was not written by Jeremiah, so rather than the
Septuagint's beginning with a "disclaimer" before the last chapter, it actually begins with,
"The words which Jeremias the prophet spoke to Baruch son of Nerias," a statement that
leaves the impression that what follows (the same text as in Masoretic 52) was indeed written
by Jeremiah. We, therefore, have one version (Masoretic) that says that Jeremiah's words
ended at chapter 52 but another (Septuagint) that leaves the impression that the chapter was
written by Jeremiah. Dr. Price doesn't see any problem in this? At any rate, if anything at all
can be concluded from all this confusion about the structure of the two different versions of
Jeremiah, it should be that a high degree of uncertainty about the contents of this book is very
much in evidence. Despite this uncertainty, Dr. Price expects us to believe that all of the
"objective evidence" indicates that Jeremiah made a remarkably accurate prediction about the
duration of the Babylonian captivity.

Poisoning the Well: Price accused me of referring to him with "rhetorically poisoned terms"
such as "fundamentalist" and "biblicist" and argued that this is substantially the same as his
use of "radical skeptic" in referring to those who reject traditional views of biblical
authorship. In logic, this is known as the tu quoque (you too) fallacy, because if I am
reasoning improperly, that would not justify his use of the same fallacious reasoning. I think,
however, that a reading of all the articles Price and I have exchanged will show that I have at
least tried to explain why I consider it proper to refer to him as a fundamentalist and a
biblicist. That is because his view of the Bible rests not on an objective, impartial examination
of its contents, as he would like for us to believe, but on a purely emotional religious belief
that it is the inspired, inerrant "word of God." Those who hold this view can best be described
as "fundamentalists" or "biblicists." On the other hand, I don't recall that Dr. Price has even
tried to explain why the fact that a person may be a "liberal" (one who breaks with tradition)
would necessarily make him wrong. He has simply implied that the opinions of those with
liberal views of the Bible should be rejected because... well, because they are liberals. Such a
position presupposes that tradition is always right, but we should know by now that this just
isn't true. Traditionalists once believed that the earth was flat, that it was the center of the
universe, and that the sun revolved around the earth, but we now know better. Traditionalists
once believed that there were many gods who each had a realm to preside over, but we now
know better. We now know better in these matters, because there were once "liberal" thinkers

Volume 1990 - 2002 Issue


Page 1227 of 2049
Skeptical Review Edited by Farrell Till
who were willing to swim against the tide of tradition. Jesus himself was a liberal, because we
find him saying in his sermons, "You have heard that it was said to them of old... but I say to
you...." Without liberals to rock the boat of societal stagnation, civilization would never
progress, and we would be doomed to live forever in the dark ages of ignorance.

More Straw Men: Price has again tried to evade his responsibility to provide extraordinarily
good evidence to support his position by arguing that his claim really isn't extraordinary. He
asked a series of questions: (1) What is extraordinary about a man making a prediction on a
certain date? (2) What is extraordinary about an ancient people being put into servitude to an
alien king? (2) What is extraordinary about a kingdom being deposed and people being
released from servitude? These are actually straw men that Price has set up to draw attention
from his inability to prove his actual proposition. That proposition is not whether the Israelites
were put into servitude to an alien king, because we all recognize that they were. That
proposition is not whether the Babylonian empire fell and the Israelites were then released
from captivity, because we also recognize that both these events probably happened. The
proposition isn't even whether Jeremiah made a prediction about the duration of the captivity,
because I am willing to admit that this could have happened. The real issue is whether
Jeremiah made a prediction that meant what Price claims and that was fulfilled exactly as he
had prophesied, and I have presented many reasons why this proposition is doubtful. The
reason most damaging to this proposition is the fact that even the "objective evidence" that
Price has touted so loudly simply will not add up to 70 years. If Jeremiah said that the
captivity would last 70 years but the "objective evidence" indicates that it didn't last that long,
then there was no remarkable prophecy fulfillment. Why is this so hard for Price to
understand?

Even Price realized that there is no way for him to hide from the fact that the extraordinary is
involved in his proposition, because after he had kicked around the straw men identified
above, he did admit that there are two extraordinary "aspects" in his proposition: (1) Jeremiah
claimed that his prediction came from God; and (2) the "fulfilled prediction" was "beyond the
normal expectations of human foresight" (May/ June 1998, p. 7). If, then, Dr. Price admits
that there are at least two "aspects" of his proposition that are extraordinary, we would
appreciate seeing him offer extraordinary evidence to support those two extraordinary aspects.
Needless to say, he has not done that yet. He has even been unable to offer convincing
evidence that the captivity lasted for 70 years, and he can never claim accurate prophecy
fulfillment until he establishes this.

Till's Additional Claims: Price ended his latest article with a list of six "additional claims"
that he thinks I must prove, as if it is my responsibility to assume the burden of proof in a
discussion that was prompted by an extraordinary claim that he has made. The first of these
"additional claims" that Price thinks I have the burden to prove is that "the text of the book of
Jeremiah is very unreliable." In truth, however, I have alleged only that the text of Jeremiah is
sufficiently unreliable to cast serious doubts on whether Jeremiah did indeed make the
prophecy that Price alleges. Since he is the one claiming that Jeremiah did make the
prophecy, he must assume the burden of showing that the textual confusion in this book does
no damage to his claim. Next, Price said that I must prove that "the manuscript evidence does
not support a sixth-century origin of the book," but this is also a misrepresentation of what I
have argued. I have said only that the existence of this book can be traced back only to the

Volume 1990 - 2002 Issue


Page 1228 of 2049
Skeptical Review Edited by Farrell Till
third century B. C., three centuries after it was allegedly written, and that the manuscript
shows that it experienced textual tampering even to the point that two entirely different
versions of varying length and organization co-existed. If such a situation as this existed as far
back as the manuscript evidence extends, what assurances can Price and his traditionalist
cohorts give us that the book didn't undergo even more radical changes between the 6th and
3rd centuries? Price further charged that I must prove that "the book was not written in the
sixth century B. C., but by a sequence of unknown authors, editors, and redactors in
postcaptivity times," but this is just another distortion of what I have said. I have never argued
that Jeremiah didn't write any of the book in the 6th century B. C. but that various scholars
(mentioned in previous articles) see textual evidence that revisions and redactions were made
during and after the captivity. These determinations have been made by sound methods of
textual criticism discussed in the front-page article of this issue, so it is again Price's
responsibility, as the claimant in this matter, to prove that the book was unquestionably
written in the 6th century B. C. by Jeremiah and has ever since remained essentially
unchanged. Needless to say, he cannot do this, or he would have already done it.

Price said that I have charged that Jeremiah was "an irrational deceiver." I don't recall using
the term "irrational" to describe Jeremiah, but I am willing to defend the charge that he
practiced deception. Price himself will have to admit that this is so or else surrender his
position that the Bible is inerrant. Jeremiah 38:14-28 relates an incident when King Zedekiah
conferred with him and then asked Jeremiah to lie to the princes about what was discussed if
they should ask him what he and the king had talked about. Verse 27 claims that the princes
did come to Jeremiah and ask him what the king had wanted with him, and Jeremiah "told
them according to all the words that the king had commanded him." Space doesn't allow me
to discuss the details, but this book presented Jeremiah as an opportunist who ingratiated
himself with the Babylonians after the conquest of Judea and referred to Nebuchadnezzar, the
very king who had enslaved the Israelites, as Yahweh's servant (25:9; 27:6; 43:10). That is
hardly honorable conduct.

Price said that I must prove that Jeremiah was a plagiarist, but I was under the impression that
I had already shown that either Jeremiah or other biblical writers had plagiarized sections of
the Bible. In "Testing the Null Hypothesis"(July/August 1997, pp. 2-3), I cited passages from
2 Kings 25:27-30 and Obadiah 1:2-5 that are almost identical in wording to passages in
Jeremiah. I don't recall that Price has even tried to explain how this could have happened
without at least some of these writers having plagiarized. Textual evidence of this nature is
exactly why many biblical scholars do not see the book of Jeremiah as the work of a single
writer. Price needs to address this textual problem.

Finally, Price said that I must prove that "the text was altered in the era of the second temple
to give the appearance of a fulfilled prophecy," but this is only one view that scholars have
suggested. Whether the alterations in the text occurred in the era of the second temple or at
some other time is really beside the point. The text of Jeremiah shows clear signs of having
been edited and revised, and Price has refused to address this problem beyond simply waving
it aside as a view of "liberal" or "radical" critics. If Price has some evidence that Jeremiah
wrote this book in its entirety in the early part of the 6th century and that it has ever since
remained essentially unchanged, then why doesn't he present it? He doesn't because he can't.
All he can do is ridicule "liberal" and "radical" critics who won't accept his claim that a

Volume 1990 - 2002 Issue


Page 1229 of 2049
Skeptical Review Edited by Farrell Till
prophecy that cannot be dated with certitude was accurately fulfilled by an event that did not
last as long as the prophecy had predicted.

Greenleaf's Principle: To support his claim that Jeremiah is an authentic work, Price cited
Simon Greenleaf's opinion about the genuineness of ancient documents. For those who may
not know, Greenleaf was a 19th-century professor of law, whose Testimony of the Evangelists
has become a popular reference work in Christian apologetics, because he took the position
that the "testimony" to the resurrection in the gospel accounts would be accepted as evidence
in any modern court of law. That any professor of law would seriously argue that second- and
thirdhand hearsay testimony would be acceptable in modern courts of law says more about
Greenleaf's determination to defend a personal religious belief than it does about his
credibility as an unbiased authority on rules of evidence, but his apparent willingness to
sacrifice legal scholarship to further personal religious biases has made him popular with
modern Christian apologists. Greenleaf's statement that Price quoted claimed that "[e]very
document, apparently ancient, coming from the proper repository or custody, and bearing on
its face no evident marks of forgery, the law presumes to be genuine," but what Price
conveniently ignores is that the book of Jeremiah does bear on its face "evident marks of
forgery." It is precisely because of those marks that a significant body of biblical scholars
consider the book of Jeremiah to be a collective work rather than the product of a single
writer. To prove his proposition, Dr. Price, by his own admission, must show beyond a 95%
level of certitude that the scholars who hold to a view of multiple authorship are wrong.

In the next issue, I will publish the last part of Dr. Price's latest article on this subject, which
he has said will bring his defense of fulfilled prophecy "to a close." After I have responded to
this part of his defense, if he wishes to, he may try again to prove the two points that are
absolutely necessary to sustain his position: (1) The book of Jeremiah as it exists today is the
work of a single 6th-century B. C. prophet. (2) The Judean Jews were held in Babylonian
captivity for exactly 70 years, just as the prophecy in 25:11 predicted. If he thinks he can
prove those two points, he is invited to try; however, no more space will be provided for him
to rant about "anti-supernatural biases" and "radical" or "atheistic" critics.

Bad History in the Book of Daniel


Farrell Till

In a phone conversation after the May/June issue was published, Everette Hatcher informed
me that I had apparently misunderstood his intentions. He said that he was not trying to leave
the impression that scholars like H. H. Rowley, Samuel Driver, and Norman Porteous were
advocates of a 6th-century B. C. authorship of Daniel but only that they had made some
admissions that were damaging to their position that this book was written in the 2nd century
B. C., during the Maccabean era. After this conversation, I reread Hatcher's article and

Volume 1990 - 2002 Issue


Page 1230 of 2049
Skeptical Review Edited by Farrell Till
noticed some sections that could be so interpreted, but for the most part, he left the impression
that these scholars were on his side. If, however, he assures me that his intention was only to
show that what he considers "liberal" scholarship has made some concessions that are
inconsistent with the view of a 2nd-century B. C. authorship, I am willing to take his word for
it. In that case, I would accuse Hatcher only of being unable to see the forest for the trees,
because the "concessions" that he thinks "liberal" scholars have made are rather minor
compared to the primary reasons why they date the authorship of Daniel in the Maccabean
period.

Various reasons have been given for rejecting the traditional view that Daniel was written by
a 6th-century Jew who was an official in the royal court of Babylon when the events in the
book were allegedly happening. Among these reasons are some that scholars don't agree on.
For example, some scholars think that the late authorship of Daniel was betrayed by its use of
Persian and Greek words that would not have been known by 6th-century residents of
Babylon, but other scholars see no particular value in this argument. Some scholars think that
the literary style of the Aramaic part of Daniel was the type that was used later than the 6th-
century B. C., but others disagree. Some scholars think that Daniel's use of the word
Chaldean to represent a caste of wise men, astrologers, and magicians rather than a
nationality indicates a late authorship, because the word was not generally used in this sense
in the 6th century, but others see no particular force to this argument. In other words, on
several rather minor points used to date the book, scholars obviously disagree. These were the
very points of disagreement that Hatcher focused on in his article, but he had little to say
about the major reasons why so-called "liberal scholars" date Daniel later than the 6th
century. On these major points, there is general agreement that they indicate that this book
was not written by a 6th-century official in the Babylonian court. These reasons can be
summarized by a single expression: bad history. The writer demonstrated an ignorance of 6th-
century Babylonian history that would not be expected of someone with the wisdom and
political position that the book attributed to Daniel, who identified himself in several places as
the author (7:2, 15; 8:1ff; 9:2; 10:2; etc).

Belshazzar's Father: We know from Babylonian records that Belshazzar was the son of
Nabonidus, the last king of Babylon. The book of Daniel presented Belshazzar as the
Babylonian king at the time of the empire's conquest, but I won't quarrel over this, because
even though Babylonian records never called him king, they indicate that Belshazzar may
have served as co-regent during his father's absence from Babylon. There is, however, reason
to quarrel with Daniel's references to Nebuchadnezzar as Belshazzar's "father" and to
Belshazzar as Nebuchadnezzar's "son." To focus attention on the extent to which this was
done, I will emphasize in bold print father and son in the text quoted below. The quotation is
long but necessary to show that the writer thought that Nebuchadnezzar and Belshazzar were
father and son.

King Belshazzar made a great festival for a thousand of his lords, and he was drinking wine in
the presence of the thousand. Under the influence of the wine, Belshazzar commanded that
they bring in the vessels of gold and silver that his father Nebuchadnezzar had taken out of
the temple in Jerusalem, so that the king and his lords, his wives, and his concubines might
drink from them. So they brought in the vessels of gold and silver that had been taken out of
the temple, the house of God in Jerusalem, and the king and his lords, his wives, and his

Volume 1990 - 2002 Issue


Page 1231 of 2049
Skeptical Review Edited by Farrell Till
concubines drank from them. They drank the wine and praised the gods of gold and silver,
bronze, iron, wood, and stone.
Immediately the fingers of a human hand appeared and began writing on the plaster of the
wall of the royal palace, next to the lampstand. The king was watching the hand as it wrote.
Then the king's face turned pale, and his thoughts terrified him. His limbs gave way, and his
knees knocked together. The king cried aloud to bring in the enchanters, the Chaldeans, and
the diviners; and the king said to the wise men of Babylon, "Whoever can read this writing
and tell me its interpretation shall be clothed in purple, have a chain of gold around his neck,
and rank third in the kingdom." Then all the king's wise men came in, but they could not read
the writing or tell the king the interpretation. Then King Belshazzar became greatly terrified
and his face turned pale, and his lords were perplexed.
The queen, when she heard the discussion of the king and his lords, came into the banqueting
hall. The queen said, "O king, live forever! Do not let your thoughts terrify you or your face
grow pale. There is a man in your kingdom who is endowed with a spirit of the holy gods. In
the days of your father he was found to have enlightenment, understanding, and wisdom like
the wisdom of the gods. Your father, King Nebuchadnezzar, made him chief of the
magicians, enchanters, Chaldeans, and diviners, because an excellent spirit, knowledge, and
understanding to interpret dreams, explain riddles, and solve problems were found in this
Daniel, whom the king named Belteshazzar. Now let Daniel be called, and he will give the
interpretation."
Then Daniel was brought in before the king. The king said to Daniel, "So you are Daniel, one
of the exiles of Judah, whom my father the king brought from Judah? I have heard of you
that a spirit of the gods is in you, and that enlightenment, understanding, and excellent
wisdom are found in you. Now the wise men, the enchanters, have been brought in before me
to read this writing and tell me its interpretation, but they were not able to give the
interpretation of the matter. But I have heard that you can give interpretations and solve
problems. Now if you are able to read the writing and tell me its interpretation, you shall be
clothed in purple, have a chain of gold around your neck, and rank third in the kingdom."
Then Daniel answered in the presence of the king, "Let your gifts be for yourself, or give your
rewards to someone else! Nevertheless I will read the writing to the king and let him know the
interpretation. O king, the Most High God gave your father Nebuchadnezzar kingship,
greatness, glory, and majesty. And because of the greatness that he gave him, all peoples,
nations, and languages trembled and feared before him. He killed those he wanted to kill, kept
alive those he wanted to keep alive, honored those he wanted to honor, and degraded those he
wanted to degrade. But when his heart was lifted up and his spirit was hardened so that he
acted proudly, he was deposed from his kingly throne, and his glory was stripped from him.
He was driven from human society, and his mind was made like that of an animal. His
dwelling was with the wild asses, he was fed grass like oxen, and his body was bathed with
the dew of heaven, until he learned that the Most High God has sovereignty over the kingdom
of mortals, and sets over it whomever he will. And you, Belshazzar his son, have not humbled
your heart, even though you knew all this! You have exalted yourself against the Lord of
heaven! The vessels of his temple have been brought in before you, and you and your lords,
your wives and your concubines have been drinking wine from them. You have praised the
gods of silver and gold, of bronze, iron, wood, and stone, which do not see or hear or know;
but the God in whose power is your very breath, and to whom belong all your ways, you have
not honored (Dan. 5:1-23, NRSV).

Volume 1990 - 2002 Issue


Page 1232 of 2049
Skeptical Review Edited by Farrell Till
According to the first part of this book, Daniel rose to prominence in the Babylonian court
during the reign of Nebuchadnezzar. This happened as a result of Daniel's interpretation of a
dream that Nebuchadnezzar's wise men were unable to interpret but which for Daniel, of
course, was a snap to decipher because Yahweh was his god (2:12-45). As a result, Daniel
was made "ruler over the whole province of Babylon and chief prefect over all the wise men
of Babylon" (2:48). If Daniel achieved such prominence in Nebuchadnezzar's kingdom, he
would have surely been familiar with the king's family, but in chapter five, the writer of this
story referred to Nebuchadnezzar five times as the "father" of Belshazzar. One of these
references was made by the writer himself in the narration of the story, two of the references
were attributed to the queen, one of them to Belshazzar himself, and the fifth to Daniel as he
addressed the king. In this address to the king, Daniel also referred to Belshazzar as
Nebuchadnezzar's "son," so in less than one chapter, six incorrect references were made to
Belshazzar's relationship to Nebuchadnezzar. How likely is it that a writer whom
Nebuchadnezzar had made ruler over the whole province of Babylon and the chief prefect of
all the wise men in Babylon would have repeatedly made a mistake like this?

Hatcher no doubt will parrot the inerrantist line and contend that the words father and son
were not being used literally in this story but only figuratively in the sense of "ancestor" and
"descendant," as when Abraham was referred to as the "father" of all Jews (Isaiah 51:2), and
as Jesus was called the "son of David" (Matt. 1:1). The examples are hardly parallel, however,
because Abraham was separated by centuries from the Jews of Isaiah's time, as Jesus was
separated in time from David sufficiently for readers of such texts as these to know beyond
reasonable doubt that father and son were being used figuratively. In the book of Daniel,
however, the reigns of Nebuchadnezzar and Belshazzar are related in consecutive chapters.
The account of Nebuchadnezzar's seven years of madness in fulfillment of a second dream
that Daniel had interpreted ends the 4th chapter, where Nebuchadnezzar praised Daniel's god
after he had regained his sanity: "Now I, Nebuchadnezzar, praise and extol and honor the
King of heaven, for all his works are truth, and his ways are justice; and he is able to bring
low those who walk in pride" (5:37). Then immediately the next chapter opens with an
account of the feast that King Belshazzar held to honor a thousand of his lords, so the writer
went directly from the reign of Nebuchadnezzar to the reign of Belshazzar without
mentioning any of the four kings who reigned between them. This within itself would indicate
an ignorance of 6th-century Babylonian history, because it at least implies that the writer
thought that Belshazzar's reign followed Nebuchadnezzar's. That would be an understandable
mistake for someone living centuries later, who thought that Belshazzar was
Nebuchadnezzar's son, but it would be very unlikely that the ruler of the entire province of
Babylon during Nebuchadnezzar's reign would have been so badly informed.

That Belshazzar wasn't Nebuchadnezzar's son has been established by the discovery of
Babylonian records. Information from these records, which can be found in almost any
general Bible dictionary, commentary, or encyclopedia, show that Nebuchadnezzar died in
562 B. C. and was succeeded by his son, but that son was Amel-Marduk, who was known as
"Evil-merodach" in the Old Testament: "In the thirty-seventh year of the exile of King
Jehoiachin of Judah, in the twelfth month, on the twenty-fifth day of the month, king Evil-
merodach of Babylon, in the year he began to reign, showed favor to King Jehoiachin of
Judah and brought him out of prison" (Jer. 52:31; 2 Kings 25:27). So even the Bible itself
acknowledges the reign of Amel-Marduk, who was assassinated in 560 B. C. during a coup

Volume 1990 - 2002 Issue


Page 1233 of 2049
Skeptical Review Edited by Farrell Till
led by his brother-in-law Nergal-Sharezer. As I noted in my previous reply to Hatcher, this
coup and the subsequent reign of Nergal-Sharezer ended the dynasty of Nebuchadnezzar and
his predecessor kings. So beginning with the reign of Nergal-Sharezer the "sons"
(descendants) of Nebuchadnezzar were no longer kings in Babylon. Nergal-Sharezer reigned
until 556 B. C., at which time he was succeeded by his son Labsi-Marduk, who very shortly
was deposed by Nabonidus. Belshazzar was the son of Nabonidus, who moved his capital to
Tema (on the Arabian peninsula) and apparently left Belshazzar in charge of Babylon. Thus,
from 560 B. C. until its fall to Persia in 539 B. C., Babylon was ruled by kings who were not
"sons" or descendants of Belshazzar. The fact that the writer of Daniel leaped from
Nebuchadnezzar to Belshazzar, passing over completely the reigns of four intervening kings,
certainly indicates a fuzzy knowledge of the history of this period. That lack of knowledge
provides the best explanation for why the writer would have called Nebuchadnezzar the
"father" of Belshazzar and Belshazzar the "son" of Nebuchadnezzar when the two were not
related. He called them father and son because he thought that they were.

That the writer of Daniel thought that Belshazzar was Nebuchadnezzar's son is supported by a
statement in the apocryphal book of Baruch. The statement purports to be a message that
accompanied a contribution that the captives in Babylon sent to the priests who were still in
Jerusalem.

They [the Babylonian captives] sent this message: The money we are sending you is to be
used to buy whole-offerings, sin-offerings, and frankincense, and to provide grain-offerings;
you are to offer them on the altar of the Lord our God, with prayers for king Nebuchadnezzar
of Babylon and for his son Belshazzar, that their life may last as long as the heavens are
above the earth. So the Lord will strengthen us and bring light to our eyes, and we shall live
under the protection of King Nebuchadnezzar of Babylon and of Belshazzar his son; we shall
give them service for many a day and find favour with them (Baruch 1:10-12, REB version).

The writer of this book claimed that he wrote it in Babylon "on the seventh day of the month,
in the fifth year after the capture and burning of Jerusalem by the Chaldaeans" (1:12). The
fifth year after the burning of Jerusalem would have been 582 B. C., at which time
Nebuchadnezzar was still reigning as king of Babylon. Obviously, the passage above was
written from the literary point of view of one who lived during the reign of Nebuchadnezzar,
so the fact that he twice referred to Nebuchadnezzar and his son Belshazzar and expressed the
hope that they would have long lives is a clear indication that the author of Baruch thought
that Belshazzar was literally the son of Nebuchadnezzar. This passage, of course, is in an
apocryphal work, which Hatcher and other inerrantists will argue that they are under no
obligation to accept as an inspired work, but whether they consider it inspired or not, it is
nevertheless a document that shows that during the 2nd century B. C. (the time that Baruch
was also written), for reasons that we may never know, some believed that Nebuchadnezzar
and Belshazzar were father and son. That a historical misimpression like this could have
circulated in the 2nd century B. C. is quite believable; however, it is unreasonable to think
that an important official in Nebuchadnezzar's royal court would have been so uninformed.

The Third Year of Jehoiakim: The writer of Daniel claimed that he and others were taken
captive in Jerusalem by Nebuchadnezzar in the "third year of the reign of Jehoiakim" and
carried away to Babylon (1:1-3), where he was selected to be educated in "the learning and

Volume 1990 - 2002 Issue


Page 1234 of 2049
Skeptical Review Edited by Farrell Till
the tongue of the Chaldeans" (v:4). Jehoiakim was king of Judah from 609 to 598 B. C.
(Eerdmans Bible Dictionary, 1987, p. 559), so if Nebuchadnezzar took him captive in the
"third year of Jehoiakim," this would have occurred in 606 B. C., which was a year before
Nebuchadnezzar became king. From Babylonian records previously mentioned, we know that
Nebuchadnezzar engaged the Egyptians under the command of Pharaoh Necho II and
defeated them at Carchemish in 605. Later that year, his father died, and he succeeded to the
throne. The next year he sacked the Philistine city of Ashkelon for refusing to pay tribute to
him, and from 604 to 601, he was kept busy securing the Egyptian front. He suffered a
setback in 601 at the hands of Pharaoh Hophra, but by 598, he had recovered sufficiently to
lay siege to Jerusalem. The Bible records this siege (2 Kings 24:10), and says that it was at
this time that captives from Jerusalem, along with treasures from the temple, were taken to
Babylon (2 Kings 24:13-16). This siege of Jerusalem happened not in the third year of
Jehoiakim but in his last year. The Bible is unclear about what happened to him, whether he
was killed during the siege or captured and taken to Babylon. Jeremiah 22:19 predicted that
he would be "buried with the burial of an ass, drawn and cast forth beyond the gates of
Jerusalem," but 2 Chronicles 36:6 claims that Nebuchadnezzar "bound him in fetters" and
took him to Jerusalem. Second Kings 24:6 merely says that Jehoiakim "slept with his fathers
and Jehoiachin his son reigned in his stead," after which the chapter describes Jehoiachin's
surrender to Nebuchadnezzar, who then carried him, the royal family, and other "chief men of
the land" to Babylon. So both biblical and Babylonian records indicate that Jerusalem fell to
Nebuchadnezzar in the last year of Jehoiakim's reign and not in his third year as the writer of
Daniel indicated. It seems rather strange that this man, who possessed all of the great wisdom
claimed in this book, did not even know what year he was taken captive to Babylon. It's
reasonable to think that someone living four centuries later could have been confused about
when Nebuchadnezzar captured Jerusalem and took captives back to Babylon, but it's hard to
believe that one of these captives would not have known when it happened.

The Son of Ahasuerus: In 9:1, the writer of Daniel described the mysterious "Darius the
Mede" as the "son of Ahasuerus, of the seed of the Medes," but Ahasuerus (better known as
Xerxes) was the king of Persia from 485-465 B.C., so it isn't at all possible that "Darius the
Mede," who allegedly reigned in Babylon in 539 B. C., was the son of someone who had not
yet been born. Ahasuerus was the Persian king who allegedly made Esther his queen in the
book named after this Jewish heroine. Since his father was Darius the Great, the writer of
Daniel may have confused his Dariuses and anachronistically made a son of Darius the Great
the king who had captured Babylon. At any rate, he made a historical mistake that would be
understandable for an author writing four centuries later, but it is not a mistake that we could
reasonably expect an important contemporary official of Babylon to make.

The Reign of Darius the Mede: The writer's confusion in matters like those discussed above
is one reason why many biblical scholars reject the tradition that Daniel was written by an
important Jewish official in the 6th-century Babylonian empire, but it is, of course, his
mistake in thinking that a Median empire intervened between the Babylonian and Persian
kingdoms that presents the most compelling reason why so many scholars think that this book
could not have been written by an official who was party to most of the events recorded in it.
Although "liberal" scholars whom biblical inerrantists deplore may disagree on minor points
concerning the dating of the book, they are in general agreement that this mistake was a major
blunder that would not have been made by someone who had been an important official in

Volume 1990 - 2002 Issue


Page 1235 of 2049
Skeptical Review Edited by Farrell Till
6th-century Babylon. Since this mistake has already been discussed in my previous responses
to Hatcher, I will only summarize the major points: (1) In 550 B. C., Cyrus conquered the
kingdom of the Medes and made it a province or satrapy of the Persian empire. (2) In 539 B.
C., Babylon fell to Cyrus, so by this time Media no longer existed. (3) Cyrus ruled in Babylon
from 539-538 B. C. and then moved his residence to Ecbatana, a city in territory that Cyrus
had taken in his conquest of Media. (4) The writer of Daniel clearly indicated that a "Darius
the Mede" reigned in Babylon for at least one year between the reigns of Belshazzar and
Cyrus the Persian, but by the time Daniel's mysterious "Darius the Mede" had finished his
reign in Babylon and Cyrus had begun his, Cyrus (according to contemporary Persian
records) had left Babylon and moved his official residence to Ecbatana. Who can believe that
a high government official living in those times could have written a book that contained such
historical inaccuracies as these?

A Peculiar Silence: To me, the important issue is not when the book of Daniel was written
but whether it is ac curate in what it reports, and I have pointed out several reasons why
readers of this book should be suspicious of its accuracy. The debate, however, concerns the
question of authorship, which Everette Hatcher has raised, so I will conclude this part of my
response with a point that I consider worthy of consideration. Appealing to historical silence
is considered a weak type of argumentation, but I am going to do it anyway, because I
personally believe that historical silence can sometimes be very compelling. The book of
Joshua, for example, claims that the sun once stood still and "hastened not to go down about a
whole day" so that the Israelites could complete their defeat of Amorite forces (Josh. 10:12-
14). If such an event as this had actually happened, it would have been noticed all over the
world and disinterested, unbiased records of it would surely have been left, but since no such
records exist, this is a compelling reason to doubt that this event ever happened (aside from
the fact that the sheer absurdity of the claim also makes it unbelievable). The same is true of
other biblical claims. The synoptic gospels allege that a three-hour period of darkness covered
"all the land" at midday, as the sun's light failed (Matt. 27:45; Mark 15:33; Luke 23:44). If
such a phenomenal event as this had actually happened, it would have produced such
widespread panic over "all the land" that many contemporary, disinterested records of it
would have been left, but no such records exist, except for some rather dubious references to
an "eclipse" that biblicists claim secular writers made in books that have not survived. This
secular silence about such a remarkable claim is sufficient reason for rational people to
assume that it never really happened. Silence, then, can sometimes be an important factor in
evaluating the credibility of historical claims that were reported only in biased sources.

There is a matter of silence about the person who allegedly wrote the book of Daniel that,
although not as compelling as the examples above, should nevertheless be considered in
evaluating the claim that this book was written by an important 6th-century Babylonian
official. The book of Ezekiel claims that it was written by a captive priest in Babylon who
began to receive his visions in the "fifth year of king Jehoiachin's captivity" (1:2). If this
claim is true, then the priest Ezekiel was in Babylon at the very time that Daniel allegedly
rose to prominence in the royal court, yet he made no unequivocal references to a Daniel, who
could have been the fellow captive that was appointed ruler over all the province of Babylon.
I say that he made no "unequivocal references" to this Daniel, because Ezekiel did mention
the name Daniel three times, but these were in contexts where this person was associated with
ancient biblical heroes like Noah and Job (14:14, 20; 28:3). Since the name is spelled "Danel"

Volume 1990 - 2002 Issue


Page 1236 of 2049
Skeptical Review Edited by Farrell Till
in some texts, this Daniel is thought to be the "Danel" of Ugaritic legend found on clay tablets
excavated at Ras Shamra, so it seems rather strange that Ezekiel would have written 48
chapters without once referring to a captive who had become a prominent Babylonian official.

In the next issue, I will discuss reasons why so many scholars date Daniel in the 2nd century
B. C.

Theological Development Rather Than


Revelation
Robert Dornbusch

I believe that the various contributions in the January/February 1998 issue of TSR have fairly
established the fact that contradictions do exist in the Bible with respect to the "Sins of the
Fathers" issue. However, I would buttress Jeff Shmura's point with one other New Testament
passage. In John 9:2, we find that even the disciples were inclined to think that children could
be afflicted by God because of the sins of the parents: "Rabbi, who sinned, this man or his
parents that he was born blind?" Where else would these Jews have come up with such a
notion if the "law," i. e., the Pentateuch, did not affirm that Yahweh could pursue punishment
to a third or fourth generation? Interestingly, we are not told that Jesus rebuked the disciples
for this kind of thinking by reminding them of Deuteronomy 14:16 or Ezekiel 18:20. Instead,
he merely claimed that this one case of blindness had happened so that "the works of God
should be made manifest in him."

I would suggest that these contradictions reflect a "theological development" among the
ancient Israelites rather than two established but contrary views held by 2 different authors or
sectarian camps. Contrary to traditional thinking, the Israelites were not "given" a complete
"law" in the 15th or 13th century B. C. E., and then were expected to maintain that same ethic
for centuries. Instead, the law and even the history of the Israelites betray a people who were
expanding upon and developing their theological and ethical standards as they became more
civilized. In other words, the "Sins of the Fathers" contradictions reflect a change in thinking
that involved a departure from a more primitive/traditional ethic toward a more enlightened
one. This did not occur at the start of the nation, nor did it take place overnight.

The reason that there are so many examples throughout the Bible where children (and
descendants) were punished for the previous sins of others is that Deuteronomy was not
written at the formation of the nation of Israel and that for the vast portion of the monarchy
the more primitive ethic held sway. (This primitive ethic, which punished descendants
alongside or in place of the parent can be seen in the punishment of Canaan for what Ham did
[Gen. 9:20-25]; in the household judgment against Korah [Num. 16:31-34]; and in Reuben's

Volume 1990 - 2002 Issue


Page 1237 of 2049
Skeptical Review Edited by Farrell Till
"magnanimous" offer to allow his sons to be killed if he failed to protect Jacob's youngest son
[Gen. 42:37]. Indeed, the near sacrifice of Isaac [Gen. 22] betrays a willingness on Abraham's
part to use his son as a means to gain divine favor for himself.) It was not until the reign of
Josiah (ca. 640-610 B. C. E.) or later that a radical theologian (Jeremiah) expressed a higher
ethic, which affirmed that each person would/should die for [or otherwise suffer the
consequences of] his sin alone (31:29-30). Despite the alleged existence of the Pentateuch
(and, thus, Deuteronomy) in his day, Jeremiah approached this new ethic somewhat
tentatively and even saw it as coming into vogue only in the future: "In those days people will
no longer say, `The fathers have eaten sour grapes, and the children's teeth are set on edge.'
Instead, everyone will die for his own sin; whoever eats sour grapes--his own teeth will be set
on edge" (Jer. 31:29-30). This is hardly what we would expect of a prophet who theoretically
was able to quote and proclaim this ethic as a Mosaic "law" that had already been in effect for
centuries. It was not until the exilic days of Ezekiel (593-571) that this particular ethic was
proclaimed with any kind of authority:

The word of the LORD came to me: "What do you people mean by quoting this
proverb about the land of Israel: `The fathers eat sour grapes, and the children's teeth
are set on edge'? As surely as I live, declares the Sovereign LORD, you will no longer
quote this proverb in Israel. For every living soul belongs to me, the father as well as
the son--both alike belong to me. The soul who sins is the one who will die.

"Suppose there is a righteous man who does what is just and right. He does not eat at
the mountain shrines or look to the idols of the house of Israel. He does not defile his
neighbor's wife or lie with a woman during her period. He does not oppress anyone
but returns what he took in pledge for a loan. He does not commit robbery but gives
his food to the hungry and provides clothing for the naked. He does not lend at usury
or take excessive interest. He withholds his hand from doing wrong and judges fairly
between man and man. He follows my decrees and faithfully keeps my laws. That man
is righteous: he will surely live, declares the Sovereign LORD.

"Suppose he has a violent son, who sheds blood or does any of these other things
(though the father has done none of them): He eats at the mountain shrines. He defiles
his neighbor's wife. He oppresses the poor and needy. He commits robbery. He does
not return what he took in pledge. He looks at the idols. He does detestable things. He
lends at usury and takes excessive interest. Will such a man live? He will not! Because
he has done all these detestable things, he will surely be put to death and his blood will
be on his own head.

"But suppose this son has a son who sees all the sins his father commits, and though
he sees them, he does not do such things. He does not eat at the mountain shrines or
look to the idols of the house of Israel. He does not defile his neighbor's wife. He does
not oppress anyone or require a pledge for a loan. He does not commit robbery but
gives his food to the hungry and provides clothing for the naked. He withholds his
hand from sin and takes no usury or excessive interest. He keeps my laws and follows
my decrees. He will not die for his father's sin; he will surely live. But his father will
die for his own sins, because he practiced extortion, robbed his brother and did what
was wrong among his people.

Volume 1990 - 2002 Issue


Page 1238 of 2049
Skeptical Review Edited by Farrell Till
"Yet you ask, `Why does the son not share the guilt of his father?' Since the son has
done what is just and right and has been careful to keep all my decrees, he will surely
live. The soul who sins is the one who will die. The son will not share the guilt of the
father, nor will the father share the guilt of the son. The righteousness of the righteous
man will be credited to him, and the wickedness of the wicked will be charged against
him" (Ezek. 18:1-20).

Interestingly, Ezekiel affirmed this more as a personal revelation rather than a legal truth that
could be found in the authoritative text! Furthermore, Ezekiel characterized the older
"Mosaic" ethic [Ex. 20:5; 34:7; Num. 14:18; Deut. 5:9] as a mere "proverb" (v:3). In Job, we
also find a central figure who does not quite approach the older ethic as if it was an
unquestionable "law" (21:19-21).

Of course, the reign of Josiah is notable for the mysterious discovery of a "book," which
Moses himself had presumably written (2 Kings 22)--as opposed to the stone tablets, which in
that day were still with the ark of the covenant, and any other oral and written commentary
that had for centuries been fobbed off as the teachings (but not the actual writings) of the
legendary Moses. We know that Josiah's "book" was unique because of how the Yahwists of
that day reacted to it and how they went about to authenticate it. In other words, it was not a
copy of the "law" as we know it today or even the legal autograph (beyond numerous copies
of it), which may have been temporarily "lost" after the reign of Hezekiah and during the
reign of Manasseh. In short, it was the first time that anyone had claimed to possess an actual
text of Moses. This, of course, casts uncertainty on the tradition that the Israelites possessed
the literary skills need to write, read, and copy the entire Pentateuch centuries earlier.
Actually, it was only during the previous century that the first "writing prophets" (Isaiah,
Micah, Hosea, Amos) had appeared, reflecting a budding literary society that was only
beginning to develop the skills necessary to produce a Pentateuch. Any previous writings (e.
g., court histories, the annals of the kings of Judah, the book of the Wars of Yahweh, Jashar,
or even that of Shemiah the prophet) were so archaic in scope and skill that they were merely
quoted/interpreted by later authors and allowed to fall into obscurity. In other words, we find
no work of the literary sophistication as that of the Pentateuch among the Israelites for seven
or more centuries after the time when they supposedly had the capability to emulate the
literary standard reflected in the "law."

Not surprisingly, the Israelites even as late as the 7th century B. C. E. betrayed a primitive
and superstitious regard for literature--an indication of just how novel writing was at that
point in their society. For example, they believed that their god used books in heaven (Ex.
32:32-33; Ezek. 2:9; 13:9; Dan. 7:10) and that a scroll could fly (Zech. 5:1-2). Similarly, they
had some peculiar notions of how to use written texts--other than reading them. We see this in
the ritualized tossing of a scroll into a river as part of a cursing incantation (Jer. 51:59-64); in
the wearing of small texts later called phylacteries on foreheads and arms as a form of amulet
(Deut. 6:4-9); in the eating of scrolls as a means to gain knowledge (Ezek. 3:2-4, itself based
on the primitive misconception that the seat of consciousness was in the bowels of a human
being); and in the adultery test where written curses were scraped off into a holy-mud
concoction and then drunk for affect (Num. 5:24, 26). None of these notions reflect a society
that had already possessed advanced literary skills for centuries or at least one whose chief
literary output had been authored by an omniscient mind! All of this goes to the point of

Volume 1990 - 2002 Issue


Page 1239 of 2049
Skeptical Review Edited by Farrell Till
showing that the Israelites were developing their texts as well as their ethics well after the
time that traditionalists believe both had been established.

Many textual scholars believe that Josiah's "book" comprised or included most of
Deuteronomy. That leaves us with the question of whether the ethic of 24:16 was included in
this "book," or if it was added later by another Deuteronomistic theologian during the exile. I
lean toward the latter view. It seems hardly credible that the young king having barely
finished hearing Deuteronomy 24:16 would turn around and say, "Great is Yahweh's anger
that burns against us, because our fathers have not obeyed the words of this book" (2 Kings
22:13). I believe the original "book" affirmed the old ethic (a great motivator for the king to
take his religious reforms seriously; after all, he had to make up for more than his own
generation's failures), which also explains its presence in Deuteronomy 5:9 and the reason
why Jeremiah (a contemporary of Josiah) spoke of the new ethic as a future development.

Whatever the case, it would be unlikely that the Deuteronomist invented all of the unique
ideas found in that book. Certainly some of these ideas had been percolating for some time
and had not yet reached critical mass. Thus, king Amaziah (796-767) may have (intentionally
or otherwise) acted a particular ethic (2 Kings 14:1-6) that would not become commonly
embraced and enforced as "Mosaic law" until Josiah's reign (or after). Much later, the
historian confidently portrayed this king as following the "law" only because by that date, this
ethic was written down as Mosaic law. (It is more than coincidence that the author was the
first to cite specifically the Deuteronomy 24:16 text.)

The die-for-your-own-sin ethic was advanced by progressive (but late) minds (e. g., Jeremiah)
despite the traditional and more barbaric ethic. However, the new ethic did not achieve the
status of law until the exile, and even then it did not totally supplant the old one. It was at this
time that the proto-Jews probably sought to distance themselves from the perceived sins of
their ancestors. Perhaps the inevitable injustices of exile prompted this ethical recalibration. (I
sincerely doubt that such a major change would have come about by a sudden introspection
over what they had been doing to each other for centuries, much less to outsiders such as the
Canaanites and Amalekites!) It is the kind of ethic we could expect of an exiled population,
especially of those born in exile, which as a mass could no longer abide an ethic that doomed
them for circumstances they had had no part in and that earlier had been so casually applied
against the individual, the minority, and the outsider. However, old religious notions rarely
die overnight. Tradition alone has a tendency to keep things in play long after their day.
Because the older ethic had long been in place, it could not help but crop up in ancient tales or
the historian's older source materials. It also appears that some had already accredited this
traditional value to Moses and, therefore, it showed up in various legal commentaries. Even as
late as the exile, the severity of the old ethic undoubtedly helped to explain national disaster.
It is even possible that its reference of a punishment to a "third or fourth generation" was
maintained (or reinterpreted) in the hope that Yahweh's anger over their forefathers' sins had a
limit. Whether because of the lingering strength of the old ethic or the newness of the higher
ethic, even the late historian could not break totally free of the old morality. Thus, he believed
that disaster meant for king Ahab could have been shunted over to his sons (1 Kings 21:20-
29), that judgment would be passed along to Hezekiah's descendants (2 Kings 20:16-19), or
that although Josiah could be spared, Yahweh was still so angry with the sin of Josiah's
grandfather (Manasseh) that destruction would be visited on later descendants (2 Kings 23:25-

Volume 1990 - 2002 Issue


Page 1240 of 2049
Skeptical Review Edited by Farrell Till
26; 24:3-4). Interestingly, Jerusalem fell to the Babylonians in the third generation after
Manasseh, and the initial defeat during the reign of Jehoiachin represented a fourth generation
disaster along a different genealogical branch.

Consequently, I see the two ethical positions not as separate theological stances but an
entanglement of a newer ethic with that of a traditional one-- without sufficient time or
theological development to break fully free of the older. Therefore, I am not surprised that
Jews even as late as the 1st century C. E. were yet influenced by the older ethic. After all, the
older gained a degree of immortality in that it was preserved in a written text, which came to
be viewed as "inspired." Furthermore, vicarious-atonement theology (an extension of the
older ethic that contradicted the die-for-your-own-sin morality) continued to be reflected in
animal sacrifices at the temple cult. Eventually, a corrupted Christianity would reinforce the
archaic ethic by the development of a mystic blood-atonement theology in the human
sacrifice of Jesus Christ.

Piecing together the events, much less the motivating factors of ancient history, always
involves a degree of speculation. (This is often true even of recent events.) However, I believe
the "sins of the fathers" contradictions are best explained as a developing theology that took
place late in Israelite history. Such an explanation finds not only support in the biblical text
but represents a plausible scenario with what we know about human development.
Nevertheless, this explanation would demand that we look at that text as the product of a
natural religion with all of its human trademarks (including contradictions, obscurities, and
misconceptions) instead of an inerrant revelation by an omniscient and holy god.

(Robert Dornbusch, P. O. Box 5000. OTF-160255, Carson City, MI 48811-5000)

A Letter From...
Gleason L. Archer

I am sorry to have been so delinquent over the recent weeks since you shared with me your
nihilistic view of the meaning of life or the significance of humanity. As you have been so
persistant [sic] in your attempt to prove that the Holy Scriptures of the Old Testament contain
errors of the gravest sort, it behoves [sic] me to point out to you that you thereby show
yourself to be absolutely meaningless, and all of life is nothing but an empty dream.

In your recent correspondence you have correctly dismissed the Arabic Qur'an as virtually
valuless [sic], a judgment with which I must concur. No other religious authority, whether
Sanskrit or Hindu or Ancient Egyptian religion yields a provable value for the ultimate
meaning or destiny of man. No other religious book or system appears to have any real or

Volume 1990 - 2002 Issue


Page 1241 of 2049
Skeptical Review Edited by Farrell Till
trustworthy authority from the standpoint of sound logic, this leaves the Tenach and the Greek
New Testament as the only answer to the question of humanity or life in any form.

Quite properly you have directed your polemic against me in my principal writings, the
Survey of Old Testament Introduction, and my Encyclopedia of Bible Difficulties. This radical
rejection of the most influential scripture and the authority of the enormous impact that has
spanned all five [sic] continents of the globe we live on, has left nothing for you to believe or
to trust except the total negitavism [sic] which you have chosen as the only tenable position to
hold.

Surely you must realize the consequence of dismissing God and facing with scornful rejection
of the fulfilment [sic] of more than 600 topics of prediction set forth in 8,352 verses of the
Bible. Rejection or dismissal of such an enormous body of data makes it crystal clear that the
radical skeptic has no reasonable ground on which to stand.

Or else, to put it more bluntly, your sweeping dismissal of all of these evidences indicates that
you are not really interested in any evidence that opposes your chosen conclusion. You have
decided in defiance of the firm conviction of the rest of the human race (apart from
communist atheists, of course) that there can be no creator, but only some creature who
somehow created himself. As I understand from your previous correspondence, that there may
be a god floating around above the universe, but completely disinterested [sic] in us, or in any
form of life or geological movement that may exist on our planet.

If this is indeed the case, it means that there are no measurements of activity or conscience in
which God is involved. The most degenerate terrorist stands on the same plane of
meaningless that you do. Therefore there is nothing that involves [sic] that has any meaning,
whether good or bad. There is no community or deity to pass judgment upon you, or to give
you any kind of recognition or praise; all your activity is essentially meaningless. You do not
have any concern about life or knowledge, and you do not even have to declaim against
people who believe that there is a God who sees and cares. Your proclamation to all the
human race is that nothing really matters; there is absolutely no yardstick for good or evil,
because neither good nor evil have [sic] any real existence. Life has no meaning and death has
no threat.

You may be perfectly content with your radical nihilism, but you will have to face the grim
truth when you die, that you have chosen the part of a fool, and you will join the great
company of those who have defied God and have found that not even Satan will be concerned
with your everlasting torment--as clearly set forth in the Scriptures of the Old and New
Testaments.

(Gleason L. Archer, Ph. D., 812 Castlewood Lane, Deerfield, IL 60015)

The Best Inerrantists Have to Offer

Volume 1990 - 2002 Issue


Page 1242 of 2049
Skeptical Review Edited by Farrell Till
Farrell Till

For the benefit of those who may not know, I should point out that Dr. Gleason Archer is the
guru of modern biblical inerrantists. In defending the inerrancy doctrine, they probably quote
his Encyclopedia of Bible Difficulties more often than any other apologetic reference work, so
I am delighted to publish a tirade that he directed at me in a letter that arrived unexpectedly in
mid-May. It will give TSR readers the opportunity to see that even one of the best apologists
that inerrantists have to offer really has no convincing evidence that the Bible is the inspired,
inerrant "word of God." About all he can do is preach.

I'm somewhat confused about Dr. Archer's references to my "recent correspondence," because
I have not written to him since 1993. Two years before, I sent him a complimentary
subscription to The Skeptical Review and requested his opinion of it, which after two follow-
up inquiries, he finally sent in a letter dated October 28, 1991. Needless to say, his opinion
wasn't complimentary. In response, I contacted him and proposed a debate on biblical
inerrancy at Trinity Evangelical Divinity School in Deerfield, Illinois, where he was a Bible
professor at the time and may still be. On February 16, 1993, he wrote me to reject the debate
proposal on the grounds that it would be "as pointless for [him] to debate [me] as it would be
for either of us to debate with Mary Baker Eddy about the reality of matter." My
correspondence files for that year show no response that I sent to this letter, which was quite
emphatic in its rejection of the debate proposal. I am therefore at a loss to understand Dr.
Archer's reference to my recent correspondence or why he would write to me after five years.

His letter, almost in its entirety, was a resort to the fallacy of undesirable consequences. This
fallacy results when one opposes a proposition on the grounds that if it is true, undesirable
consequences will ensue. Dr. Archer apparently thinks that if his god-view is not true, then
our existence is totally meaningless, so he seems to reason that the undesirability of this
consequence must mean that his god does exist. Aside from the absurdity of believing that a
proposition can be made true just by wanting it to be true, the proposition itself is absurd.
Even if Archer's god does exist, people live their entire lives without ever seeing him, yet they
are obviously able to live meaningful lives. I certainly believe that my life has meaning, and
that is because I have assumed the responsibility of giving it meaning. Except in cases where
there are debilitating handicaps, anyone else can do the same, because whether an individual's
life has meaning depends more on the individual than any other factor, so if Dr. Archer thinks
that his life would have no meaning without a god-crutch to lean on, I feel sorry for him. He
needs to find the personal initiative and imagination to give direction and purpose to his life
without leaning on some god-idea that may well be false.

No atheist bashing would be complete unless the theist regurgitated the usual threats and
denunciations, and so Dr. Archer didn't disappoint us. He said that the "most degenerate
terrorist stands on the same plane of meaningless that [I] do," but I would beg to differ with
him. Since many terrorists are motivated by beliefs that they are doing the will of whatever
gods they may believe in, it would be more accurate to say that the most degenerate terrorist
stands on the same plane of meaningless that Dr. Archer does. I personally prefer to stand on
a higher plane than that of a racist god who ordered the massacre of women, children, and

Volume 1990 - 2002 Issue


Page 1243 of 2049
Skeptical Review Edited by Farrell Till
infants who happened not to have been born in the nation that he had chosen to be his "special
people."

Dr. Archer ended his tirade with the usual Christian warning of what awaits those who "have
defied God," but he seems to have confused me with those who are credulous enough to
believe in vengeful gods and judgment days. Such rhetoric is wasted on those whose lives are
not directed by superstitious fear.

Meanwhile, I will renew my proposal that Dr. Archer and I debate biblical inerrancy at
Trinity Evangelical Divinity School so that the ministerial students there will have an
opportunity to consider both sides of the inerrancy question. We will eagerly await his
answer.

From the Mailbag


The Dating of Daniel...

I received a copy of the March/ April 1998 issue of The Skeptical Review courtesy of Mr.
Everette Hatcher. Mr. Hatcher purchased a copy of my book, Beasts, Horns and the
Antichrist: Daniel a Blueprint of the Last Days?

In this issue Mr. Hatcher wrote an article titled "The Critics' Admissions Concerning Daniel,"
and you wrote a rebuttal titled "The Inerrantist Way of Misrepresenting `Critics.'" Since I am
the author of a critical commentary on the apocalyptic visions in Daniel, I was intrigued by
this debate and subsequent articles in The Skeptical Review concerning biblical errancy.

My book is an attempt to combat the dogmatism of prophecy writers who chart the course of
future events from the apocalyptic visions in Daniel. Prophecy writers claim that Daniel
predicts future events such as a tribulation period, rise of Antichrist, Armageddon, etc.
However, as I'm sure you'll agree, the book of Daniel is a product of the second century B.C.
when the Jewish people were under the oppressive rule of one Antioch Epiphanes.

Falling on the side of biblical errancy, I can especially relate to your article titled "Who's
Really Looking at the `Narrow View'?" I too have repeatedly faced the "catch-all explanation"
that inerrantists put forth in their arguments that "God's ways are simply higher than our
ways, and so he has a higher purpose that we just can't understand."

I'd like to add another catch-all phrase that I've encountered. Inerrantists often claim that
errantists can't "believe" the Bible because they are approaching it with the wrong heart. In
other words, inerrantists claim that we don't approach the Bible with the proper spirit and
willingness to believe. Therefore, we are closing ourselves off to understanding this "higher
purpose" and can't see what the Bible is really saying beyond the written words.

Volume 1990 - 2002 Issue


Page 1244 of 2049
Skeptical Review Edited by Farrell Till
Getting back to Daniel, you stated that inerrantists often label critical interpreters "liberals" as
if this is some type of "catch-all" argument also. I have noted that in John Walvoord's work on
Daniel (Daniel: The Key to Prophetic Revelation), he continually dismisses scholarly research
as the conclusions of "unbelieving critics." His implication is that since the research came
from "unbelieving critics" it should be ignored.

In conclusion, I am enclosing a review copy of Beasts, Horns and the Antichrist and would
like to accept your invitation of a free first-year subscription to The Skeptical Review. I look
forward to reading more informative articles in your fine publication. Keep up the good work.

Beasts, Horns and the Antichrist is available in its entirety on the internet at
www.danielprophecy.com. Copies may be purchased by mail for $13.90 postpaid from
Cliffside Publishing House, P. O. Box 230, West Jefferson, NC 28694.

(Broderick D. Shepherd, address same as above; e-mail, author@danielprophecy.com)

EDITOR'S NOTE: Since receiving Mr. Shepherd's complimentary copy of Beasts, Horns and
the Antichrist, I have had the opportunity to read several chapters in it, and I have found it to
be an informative work on the controversy surrounding both the dating of this book and its
alleged prophecy of the "end times" and final "tribulation." It is a book that one can read
without needing to understand Hebrew and Aramaic. The layman who wants to know why
most scholars consider Daniel to be a product of the Maccabean period rather than the sixth-
century B. C. will find the reasons explained in simple language.

Another Opinion of Daniel...

Enclosed is my subscription for another year. It is in advance of the required time, but I do not
want to overlook this. Reading The Skeptical Review is one of my favorite occupations.
Compared to the cost of other periodicals these days, I cannot understand how you can deliver
TSR at the present subscription rate.

Like many of your other subscribers, I was taught the fundamentalist idea that the Bible came
from God. Of course, my elders taught me to study the Bible, and I studied seriously, so
seriously that, like so many other TSR fans, I began to see the contradictions, mistakes, and
obvious fabrications. As I grew older, I began to see the Bible as it is, a collection of ancient
texts, some profoundly beautiful and wise, such as Ecclesiastes, which reads almost like the
Rubaiyat of Omar Khayyam, or the message of love in 1 Corinthians 13. "Though I have faith
to move a mountain and have not love, I am nothing." Such things are a nice part of our
heritage but should be seen in the light of modern scholarship.

As to the recent TSR debates about the book of Daniel, I have long ago relegated Daniel to the
category of childlike fables along with Adam and Eve, Noah and the ark, and the amusing
stories of the tower of Babel and Jonah and the big fish. The exaggerations are so gross that
they bring a smile to my face. It is sad to see people try to defend such things as historical.

The scholars of the highly acclaimed Jerusalem Bible indicated in their preface to Daniel that
this book was written between 167 and 164 B. C. The second-century authorship of Daniel is

Volume 1990 - 2002 Issue


Page 1245 of 2049
Skeptical Review Edited by Farrell Till
widely accepted among serious students of Old Testament history, Catholic, Protestant, and
Hebrew. It is not an idea conceived by atheists and agnostics to destroy religious faith. In the
face of anachronisms and anomalies in the book of Daniel, the fundamentalists have an
unenviable position to defend. I am glad I do not have that task. As an old joke says, "I thank
God I'm an atheist."

(Clarence Sears, 11067 Scott Mill Road, Jacksonville, FL 32223; e-mail,


pennyuno@aol.com)

Fundamentalists in New Zealand...

Congratulations on another excellent edition. Enclosed is a cheque for a year's subscription


and disks of back issues.

It is interesting to note the thought patterns of fundamentalists. A short story should prove that
they are the same all around the world. Recently, at Easter, a current affairs programme in
New Zealand had a segment on a professor (I think) of religious studies, who was stating that
Jesus did not in all probability rise from the dead and only about 10% of the Bible is actually
true. In the short amount of time available to him, he provided an excellent account of his
reasoning. Basically, the gospels and other books of the New Testament are not and were not
meant to be true reports of events but rather a package of part fact, mostly stories that were
designed to gain converts to whatever early Christian sect had written the books. An integral
part of this was making the story appeal to whatever group was reading it, be they Hebrew,
Greek, or Roman, and so stories were fashioned to appeal to those groups. The gospels are
basically advertisements. In opposition to his view was shown an "Assemblies of God"
minister who "refuted" the professor on the grounds of "it's all new liberal theology," and, "If
Jesus didn't resurrect, then we have nothing to believe in." Neither of his arguments addressed
the issues the professor was raising.

Martin Luther may very well have noted that the Devil was allowed to enter serpents, but then
Martin Luther also believed that only humans have souls, a fact he demonstrated by nailing a
dog to a wall and saying that "even though only we have souls and can feel pain, God is so
merciful and gracious that he has allowed this dog without a soul to show the semblance of
feeling pain, even though he is not. Isn't God wonderful?" I don't trust Martin Luther
somehow.

Finally, I also disagree with Larry Laird's statement that Christians cannot be rational. The
statement is too broad. Christians can certainly be rational on a lot of subjects, as I am sure
they are, e. g., Reverend David Rice and also the professor whom I mentioned earlier.
However, I certainly agree to the point that Christians lose rational thought by belief in God,
which is untestable, unproven, and contradictory, and thus irrational.

(Hayden Wood, 14 Mays Street, Devonport, Auckland, New Zealand.)

EDITOR'S NOTE: I'm not surprised that biblical fundamentalists in New Zealand would have
the same disdain for biblical "liberals" as their cohorts in the United States. Through the
internet, I have had contacts with biblicists in various countries, and I have noticed that they

Volume 1990 - 2002 Issue


Page 1246 of 2049
Skeptical Review Edited by Farrell Till
all use the same illogical approaches to defending the Bible. On the internet, one will even
encounter Muslims using the same kind of arguments in support of Islam and the Qur'an.
This, however, doesn't seem to bother biblical inerrantists. They continue on their merry way
to attack "liberals" and "radical critics" and to propose ridiculous how-it-could-have-been
scenarios to "explain" biblical discrepancies. In this respect, they are irrational, but that
doesn't mean that they can't be rational in matters in which they have not been blinded by
their religious beliefs.

Thanks from Another Ex-Preacher...

I would like to thank you and your publication for doing what Jesus couldn't do, namely,
setting the captive free. I am a 26-year-old former ministerial student. I attended Trinity Bible
College, of the Assemblies of God denomination, in North Dakota and interned as a youth
pastor in Iowa, but I eventually left the Pentecostal ministry because of honest, painful doubt.
My main goal in life was to kill off my own personality (natural emotions, thoughts, etc.) and
try to be just like Jesus in obedience to "he must increase, I must decrease." I became addicted
to this Jesus inspired moral masochism. This created in me a strong persecution/martyr
complex, caused by the desire to be just like Jesus and the suffering servant of Isaiah 53.
Needless to say, after years of painful doubt and self-hatred, I realized enough is enough. This
doesn't feel right. It no longer had the "ring of truth." Your publication confirmed my doubts
and placed me on a new path of emotional recovery.

Thank you, Farrell. If you know any former Christians who are interested in corresponding
with other former Christians, through letter writing, will you please give them my address?

(Allen Nielsen, 290 SW Court Street, Apt. 1, Dallas, OR 97338)

EDITOR'S NOTE: Needless to say, I am always happy to receive letters like this. I can
understand the emotional turmoil that Mr. Nielsen has been through, and I wish him success
in adapting to a way of life that puts reason and reality above blind faith and wishful thinking.
I know from my own experience that the decision he made was not an easy one, but I also
know that his life will be a more satisfying one without the crutches that he was previously
leaning on.

The Hovind-Till Debate...

I recently watched a copy of a debate between you and Mr. Kent Hovind that took place a few
years ago. My friends that loaned me the tape expected that this would impress me with how
well Mr. Hovind could refute evolution. I have to admit, however, it had the opposite effect.
While I did not come away believing in evolution, I certainly agree that you were right in
stating that he did not scientifically prove the statement that he had agreed to debate. I do feel
that much of what he offered by way of explanation is very possible, but that is a far cry from
proving that it happened that way. I have a lot of doubts that evolution can be proved either
and think that either side has to rely a great deal on faith. I do have to admit, though, which I
hate doing, that you have shaken mine and I intend to investigate this issue further. I know
that you consider the Bible to be full of errors and contradictions, but do you believe that
there is a God and that we have any relationship with him? I hope I've not taken up too much

Volume 1990 - 2002 Issue


Page 1247 of 2049
Skeptical Review Edited by Farrell Till
space here, but I am the type who often displeases my friends because I question too much.
As well as the free subscription offer, I would like very much for you, Mr. Till, to write to me
personally to perhaps help me in my quest for the truth. Thank you for your time.

(Gladys Millett, RR 1 Box 1522, Lincoln, ME 04457; e-mail, 4743@telplus.net)

EDITOR'S NOTE: I'm often asked why I spend so much time on publishing TSR and working
on related projects, and the answer is in letters like the ones above. Having been trapped by
religious fundamentalism, I know the damage it can do to people's lives, so I get a great deal
of satisfaction from knowing that I have helped people to see that there is more happiness to
be gained from throwing away religious crutches and simply accepting reality as it is. Those
who squander their lives on wishful thinking won't understand this, of course, but those who
have abandoned religious superstition will. Ms. Millett had taken only the initial steps to
examine her religious beliefs when she sent me this letter, but in subsequent messages, she
has indicated that her search continues. I wish her luck and offer her any help that I can give.

The Good News Just Keeps Coming...

I have been reading The Skeptical Review on the internet and my opinion is that it is simply a
fantastic and marvelous magazine! You are to be greatly commended for your work and the
work of your contributors.

I was intrigued to discover that you were once associated with the Church of Christ
(noninstrument, I assume). I left that organization in 1991 (in favor of atheism) after I could
no longer honestly accept their dogmas of "verbal plenary inspiration" and "Biblical
inerrancy." Hume, Paine, McDonald, Holyoke, Bessant, Ingersoll, and a great many other
freethinkers and/or philosophers and scientists helped me along the path. In retrospect, I can
honestly say that my experience in the Church of Christ was perhaps the worst mental
experience of my life whereas atheism has been a most valuable and positive learning
adventure.

I do have a question. How do inerrantists respond to the problems of reverse chronologies


found in the Bible in parallel accounts? One example (of several) being that the book of
Matthew has the devil setting Jesus on the pinnacle of the temple, then taking him to a high
mountain (Mt. 4:5-8); whereas Luke 4:5-9 has the devil taking Jesus first to the high mountain
and then to the pinnacle of the temple.

Also, how have they responded to the specific problem of the robe placed upon Jesus, which
was colored scarlet in Matthew (Mt. 27:28) but turned purple in Mark and John (Mk. 15:17;
Jn. 19:2)? Was it two toned?

I apologize if I have wasted any of your time, which I know is already strained. Is it possible
for me to subscribe to TSR? At any rate, great job, Mr. Till. I appreciate you and your work.

(Donald Drye, 1523 Amberwood Circle, Murfreesboro, TN 37128; e-mail,


sdrye@bellsouth.net)

Volume 1990 - 2002 Issue


Page 1248 of 2049
Skeptical Review Edited by Farrell Till
EDITOR'S NOTE: I'm happy to hear when any religious fundamentalist abandons
superstition, but I'm particularly happy to hear from members of the Church of Christ who
take this step. Since that was the church that caught me in its grip, I'm more familiar with its
brand of biblical fundamentalism, so I can certainly understand why Mr. Drye would say that
his stint in this church was "the worst mental experience of his life." As others have so often
done in this column, he too notes that atheism has been a positive learning experience in his
life.

As for the sequential differences in Matthew's and Luke's accounts of the temptation of Jesus,
Luke's order is more logical than Matthew's, because it has the first two temptations
happening in the wilderness and then the third one in the city of Jerusalem, whereas
Matthew's order would have required Satan to take Jesus into the city for the second
temptation and then back into the wilderness for the third. Inerrantists, however, would never
agree that this is a sequential mistake. They will simply argue that Matthew didn't intend to
use a chronological order in relating the events. Literarily, this is a possibility, and so I
personally don't see this story as a major problem for biblical inerrantists.

To explain the discrepancy in the color of Jesus's robe, inerrantists will say that purple and
scarlet are tones or shades of the same color. In reality, purple and scarlet are distinct hues,
but keep in mind that inerrantists intent on defending the infallibility of the Bible will think
that the act of making a claim is the same as proving the claim.

Did Sierichs Quote Out of Context?

Overall, I enjoyed the copy of The Skeptical Review sent me. However, I noted that hypocrisy
seems to know no boundaries. It's as flagrant in the fundamentalist camps as well as in the
skeptical society camps. You pointed out in your article "The inerrantist way of
Misrepresenting `Critics'" that it is easy to quote scholars to support any error. You noted that
these scholars were often misquoted and taken out of context.

If you will read the article you posted in the same issue of your magazine entitled "How the
Snake Slithered into Eden" by William Sierichs, Jr., the same charges you rightfully made
against the fundamentalists could be used against this article. It cites "scholars" and doesn't
even bother to give us the names of these scholars, or their works, no references, nothing.
(See first page of article, second column, third paragraph, for example.)

I've been a humanist atheist and I've been a fundamentalist. I've found both groups quite
irrational when it comes to really testing their beliefs. And I've seen both groups often do the
very thing they accuse their opponents of doing.

(Gary Amirault, 118 Walnut, Herman, MO 65041; e-mail, gamirault@ktis. net; web site,
http://www.tentmaker.org)

EDITOR'S NOTE: Mr. Amirault tried to leave the impression that he is an objective inquirer,
who found both "humanist atheism" (whatever that is) and fundamentalist Christianity to be
"quite irrational." However, I took the time to investigate his web site and found that rather
than being an objective inquirer, he is just another Jesus peddler. In his introduction to this

Volume 1990 - 2002 Issue


Page 1249 of 2049
Skeptical Review Edited by Farrell Till
site, he said, "Tentmaker is Gary and Michelle Amirault and friends who have fallen in love
with our Father, the Creator of the universe. We are concerned with growing up into the
fullness of the stature of Christ and being about our Father's business. Knowing Who the
King of the Universe is and discovering He actually loves you, not just with words but in
reality, is something that is impossible to keep bottled up. It must be shared. That is what
Tentmaker is all about." Yeah, right! It sounds so "rational" that really objective people will
want to read more, won't they? The web page even has a picture of Michelle and Gary,
standing with arms entwined, smiling in youthful, blissful satisfaction that they have
discovered great truths that they feel compelled to share with the word. In other words, Gary
and Michelle are just a young, idealistic couple who have a lot to learn. I've been where they
are, and I would never trade it for where I am now.

Amirault accused me of hypocrisy for reprimanding Everette Hatcher's quoting sources out of
context but not saying anything about the same offense that he thinks he has found in William
Sierichs' article about Eden and the snake. In Hatcher's case, he was citing out of context in a
way that left the impression that his sources believed in a 6th-century B. C. authorship of
Daniel when in reality they are firm advocates of a 2nd-century B. C. dating of this book.
Sierichs, on the other hand, was simply relating Sumerian and Babylonian creation myths
that are generally known and can be found in various mythological reference works. Sierichs
did cite Raphael Patai's The Hebrew Goddess as a source (p. 8) but could have been more
specific in the column that Amirault cited, where twice "scholars say" was used without
specifying which scholars. However, this is hardly parallel to citing fragmented quotations
that leave impressions that are the opposite of what the quoted writers actually believed.

Was Jesus Born Twice?

One evening recently, having nothing better to do, I picked up the second volume of the
annual Gulf Coast Lectures, held in Portland, Texas, under the aegis of Pastor Jerry Moffitt.
In it, I found a paper given by a Church-of-Christ preacher entitled "Cleansing the Temple."
The author's argument was that the account of the cleansing in the synoptics and in John must
be different events and not a legendary account given two different treatments. John tells us
that Jesus found in the temple "those that sold oxen and sheep and doves, and the changers of
money sitting" (John 2:14). The preacher explained that the Greek word for found is heurisko.
"The grammatical construction found in John 2:14 (the verb heuren with the preposition en
and the dative case) means to find, to come upon accidentally, without seeking." Mark, for
instance, says, "And he entered into Jerusalem, into the temple; and when He had looked
round about upon all things, it being now eventide, He went out unto Bethany with the
twelve" (Mark 11:11). The preacher explained that the Greek word periblepo means to look
around. Thus, Jesus found this situation at the beginning of his ministry and sought to correct
it immediately. At the end of his ministry, Jesus went into the temple, saw the same problem
again, and returned the next day to correct it again.

Another difference in the two accounts is that in John the temple authorities asked Jesus by
what authority he did what he did. In the synoptics, the authorities sought to kill Jesus.

Well, you can see how different the two accounts are, so he had me convinced. I just love
irrefragable arguments!

Volume 1990 - 2002 Issue


Page 1250 of 2049
Skeptical Review Edited by Farrell Till
I wondered if there were other accounts that might be explained in a similar way, so I turned
to the beginning of Matthew that recorded the birth of Jesus. Matthew tells (1) Joseph
dreamed an angel of the Lord appeared to him; (2) Jesus was born in the days of Herod the
king; (3) wise men came from the east, bringing gifts; (4) Joseph, Mary, and Jesus fled to
Egypt; and (5) they returned after Herod died, and "turned aside" to go to Nazareth in Galilee.

Luke's account tells us (1) the angel Gabriel appeared to Mary to tell her she would be the
mother of Jesus; (2) Mary visited her cousin Elisabeth; (3) Mary went to Bethlehem with
Joseph; (4) forty-one days after Jesus's birth, Joseph, Mary, and Jesus went to Jerusalem for
Mary's purification; (5) they returned to Nazareth from where they went every year to
Jerusalem for Passover. This all happened after king Herod had died.

It is obvious that there are two events related in Matthew and Luke. There cannot be one
legendary event told in two different ways; therefore, Jesus was born twice, which is proper
for a leader who taught his followers to be born again.

Ain't Christian apologetics wonderful?

(Ernie Brennaman, 1601 Airline Road, Apt. 62, Corpus Christi, TX 78412-4434)

EDITOR'S NOTE: Mr. Brennaman assisted me as moderator and time keeper at my debates
in Portland, Texas, so he may recall that during one of my speeches, as I was rebutting an
argument Jerry Moffitt had based on an unsupported prophecy-fulfillment claim, Moffitt
interrupted me to announce that he would like to conduct a written debate with me on the
subject of prophecy fulfillment. When I reminded him that he was not even meeting his
obligations in our written debate on biblical inerrancy that was supposed to be in progress at
that time, he apologized for the delays that he had caused and promised that he would
arrange his schedule so that the debate on inerrancy could be completed reasonably soon and
allow us to begin one on prophecy fulfillment. That was four years ago, and the inerrancy
debate is still not complete, because Moffitt has not yet responded to the last manuscript that I
sent to him almost four years ago. The audience at this time was largely members of the
Church of Christ, who were attending the lectureship that the debate was a part of. They went
away probably thinking that "Brother Moffitt" had shown that atheist a thing or two by
defying him to debate prophecy fulfillment. I'm sure that Moffitt has never bothered to inform
them that the debate he challenged me to participate in has never materialized because he
will not respond to the materials I have submitted to him in the other debate.

Fundamentalist Christians in Prison...

I received the first issue of your newsletter today. I think it is great that there are people that
can be outspoken enough to voice the alternate views of this issue. I am in prison, and being
here I am used to the dogmatic arguments of Christians. You know, I've heard it said that
prison is one of the last places the devoutly religious are found. Well, all I can say is what
better place for a person to be brainwashed into various beliefs.

Without people like you, this world would not have advanced as it has. Please take note of my
prison ID number. Without it, I cannot receive your publication.

Volume 1990 - 2002 Issue


Page 1251 of 2049
Skeptical Review Edited by Farrell Till
(Brent Berry #B40916, P. O. Box 300, Vienna, IL 62995-0300)

EDITOR'S NOTE: There are now around 60 subscribers to TSR who are prison inmates.
Many of them write letters to me, which regrettably I rarely have time to answer, and Mr.
Berry is not the first one to report that having to contend with evangelical Christian inmates
is just a part of prison life.

Balaam...

In your article about Balaam in the November/December issue of The Skeptical Review, you
did not consider Numbers 31:8, 15-16. There it is written: "Among their victims were Evi,
Rekem, Zur, Hur and Reba--the five kings of Midian. They also killed Balaam son of Beor
with the sword.... "Have you allowed all the women to live?" he asked them. "They were the
ones who followed Balaam's advice and were the means of turning the Israelites away from
the LORD in what happened at Peor, so that a plague struck the LORD's people."

So the final verdict of the Old Testament on Balaam is quite negative.

(Thorbjorn Willoch, Landingsveien 24, N-0762 Oslo, Norway; e-mail, willoch@oslo.Geco-


Prakla.slb.com)

EDITOR'S NOTE: Deuteronomy 23:4-5 states that the Moabites hired Balaam to curse the
Israelites, but as the story is related in Numbers 22-24, Balaam consistently told Balak that
he would not prophesy anything contrary to what Yahweh told him even if he were offered a
houseful of silver and gold, so there is nothing in the "inspired" record to justify the NT claim
that Balaam taught Balak to cast a stumbling block before the children of Israel (Rev. 2:14)
or that he "loved the hire of wrong-doing" (2 Pet. 2:15). As for the charge that Balaam was
responsible for the orgy at Peor (Num. 31:16), this is inconsistent with the story of Balaam as
related in chapters 22-24. According to the account here, Balaam lived in Pethor, which was
located on the Euphrates River in Northern Mesopotamia several miles from Moab, and when
Balak sent emissaries with money to bribe him into coming to Moab to curse the Israelites,
Balaam refused the offer and did not go to Moab until Balak had sent a second group of
emissaries. Throughout his stay in Moab, however, he repeatedly told Balak that he would not
prophesy anything about the Israelites that Yahweh did not specifically direct him to say
(Num. 22:18, 38; 23:26; 24:12-13). Numbers 23 and 24 record only "blessings" that Balaam
pronounced upon Israelites, and after the last of these blessings (24:15-24), which were
spoken from a mountain where Balaam was able to see the encampment of the Israelites, the
text claims that "Balaam rose up, and went and returned to his place" (v:25). In the next
chapter, the orgy at Peor occurred, but by this time Balaam had returned to his "place." How
then could he have been involved in counseling the Israelites to "commit trespass against
Yahweh" (Num. 31:15)? The story gives no indication that Balaam had any contacts at all
with the Israelites. To say the least, what the Bible says about Balaam is highly inconsistent,
and that's a rather strange thing to find in a book that was presumably inspired in its entirety
by an omniscient, omnipotent deity.

Did Jeremiah Just Get Lucky?

Volume 1990 - 2002 Issue


Page 1252 of 2049
Skeptical Review Edited by Farrell Till
I was pleased to see the contribution from James D. Price in your May/June issue. Price's
writing is articulate, well organized, and largely jargon free--a refreshing change of pace from
some of the obtuse and convoluted material you get from other religionists.

Price cites the laws of thermodynamics. Your average reader may not be aware of what "law"
means in science. A "proposition" is a statement about how the world may or may not work.
A proposition is said to be "robust" if it's been tested many, many times and so far always
found to be valid. It's said to be "fundamental"-- though not in the biblical sense, of course--if
it's pretty basic to an understanding of how many other things work. Any proposition which
is both fundamental and robust gets elevated to the status of a "law."

The key words here are "so far." In science, all knowledge is tentative. Even something
which is as respectable as a "law" is subject to revision if better information comes along.
Unlike religion, science holds facts to be paramount. For example, about a century ago, we
had the law of conservation of matter, which said that matter may be neither created nor
destroyed, although it could be changed from one form of matter to another. There was a
similar law of conservation of energy. Most scientists would have bet the family jewels that
these laws were so reliable they would never be overturned.

Then along came this Swiss postal clerk who speculated that, in fact, matter and energy were
just 2 different versions of the same thing, and that they could be switched back and forth
according to the relationship E=MC2. It turned out to work, too. So all of a sudden, it was
possible to "destroy" matter while simultaneously "creating" energy, and vice versa.

So what did scientists do? Did they go into denial and say "No, that can't be right"? (Well,
actually, some did, but that didn't last long.) Did they throw up their hands and say "Geez, we
can't believe anything any more!"? Did they trash all references to their previously hallowed
laws?

No, they calmly restated their principles into an even more comprehensive, fundamental law,
the law of conservation of matter and energy. They built on knowledge already accumulated
to give us an even clearer understanding of nature. But they did change the explanations to fit
the facts.

I mention this because the 2nd law of thermodynamics, much beloved of fundamentalists, is
certainly respectable and time-tested with regard to the universe we see around us today. It is
not at all clear, however, that it applied to whatever may have existed prior to the Big Bang.
(Indeed, it's not at all clear that the phrase "prior to the Big Bang" has any meaning, if the Big
Bang was responsible for bringing time itself into existence.)

The short and only honest answer to the question "Where did everything come from?" is "We
just don't know." But I guarantee you that, if we ever find out, it's going to be because of
science, not religion.

Lastly, I'd like to cut through the page after page of clutter that accompanies your ongoing
dialog with Price to get at the heart of the issue of prophecy fulfillment: divine intervention.

Volume 1990 - 2002 Issue


Page 1253 of 2049
Skeptical Review Edited by Farrell Till
The odds against winning the PowerBall lottery are about a bazillion to 1 against. Yet, given
enough people buying tickets and enough time, despite all the odds, sooner or later someone
does win it. The following day, whose picture do you see on Page 1 of your local newspaper?
Hint: it's not any of the millions of losers. We remember the people who beat the odds, who
succeeded against all expectations. We forget the rest.

So here we have Jeremiah and his prophecy about the 70 years of Jewish captivity. Let us
grant Price, arguendo, virtually everything he asked for. Let's admit the possibility of miracles
or divine inspiration or whatever. Let's admit that Jeremiah was a real person who wrote
whatever Price claims he did, well in advance of any of the events foretold, and that the text
has been scrupulously preserved ever since. Let's say that the Jewish captivity did indeed last
70 years, or at least close enough for government work.

We now have 2 hypotheses to explain this set of circumstances: (A) God, who knows
everything, whispered in Jeremiah's ear what was going to happen in the years to come, and
Jeremiah wrote it down and passed it around. (B) Jeremiah, a more or less normal man, who
may or may not have been hearing little voices in his head, took a guess about what was going
to happen in the years to come, wrote it down, passed it around, and got lucky. We remember
him because he got lucky, while forgetting about the scores of other prophets (common as
dirt at the time, as Farrell has noted), because they were losers.

So, Dr. Price, why should we believe (A) rather than (B)?

(Richard S. Russell, Wisconsin Alumni Association, 650 N. Lake Street, Madison, WI, 53706;
e-mail, RichardRussell@badger.alumni.wisc.edu)

Price Is No Scientist...

One thing is clear. Price is not a scientist. There are many problems with a multi-universe
inflationary theory, but it is a genuine scientific theory.

I was also surprised by his claim that Andrei Linde was an atheist. According to Alan Guth's
book The Inflationary Universe, Linde, one of the architects of inflationary, multi-universe
theory, thought it was the simplest way for God to create a universe.

(Steven Carr, Flat C, 28 Keighley Road, Bradford BD8. England; e-mail,


steven@bowness.demon.co.uk; web page, http://www.bowness.demon.co.uk/)

Volume 1990 - 2002 Issue


Page 1254 of 2049
Skeptical Review Edited by Farrell Till

The Skeptical Review


Volume Nine, Issue Five
September/October 1998
Farrell Till, editor

• "Convenient Coincidences in the Book of Daniel"


The remarkable parallels in trials and ordeals faced by both Jewish captives in
Babylon and the Maccabean Jews support the view that the book of Daniel was
written in the 2nd century B. C.
• "Price's Last Stand"
In the final installment of his "final word" on Jeremiah's 70-year prophecy, Dr. James
Price continues to defend his claim of remarkable prophecy fulfillment.
• "Plowing the Same Ground"
Farrell Till shows that Price's "final word" on the 70-year prophecy was merely a
rehashing of arguments that had already been answered.
• "Good History in the Book of Daniel"
Farrell Till discusses the uncanny accuracy of many of Daniel's prophecies about 3rd-
and 2nd-century B. C. events. The remarkable accuracy of these prophecies contrasted
with very basic mistakes that the writer of Daniel made in 6th-century B. C. history
supports the view of a 2nd-century authorship.
• "From the Mailbag"
Readers of The Skeptical Review express their views on a variety of subjects.

Convenient Coincidences in the Book of


Daniel
Volume 1990 - 2002 Issue
Page 1255 of 2049
Skeptical Review Edited by Farrell Till
Farrell Till

Biblical fundamentalists claim that the book of Daniel was written in the 6th century B. C. by
an important official in the Babylonian empire, but in recent issues of The Skeptical Review,
including also this one, we have seen serious reasons to doubt this claim. As the textual
evidence has shown, a more likely view is that this book was written in the 2nd century B. C.
during the Maccabean era. If, however, it is indeed the case that Daniel was written in the 6th
century, there are some convenient coincidences in this book.

"Good History in the Book of Daniel" (pp. 9-11, 16, this issue) described the attempts of
Antiochus II Epiphanes to destroy Judaism during his reign. Many of his efforts to stamp out
Jewish traditions and religious practices are recorded in the apocryphal books of the
Maccabees. According to 1 Maccabees 1:41-61, Antiochus issued a decree ordering subjects
throughout his empire to "become one people and abandon their own customs" (v:41). The
traditional Jewish sacrifices were forbidden, and pagan altars were built on which "swine and
other unclean beasts" were to be sacrificed (v:47). The observance of Jewish dietary laws was
forbidden, and Jews were allegedly ordered under penalty of death to eat "unclean" meat,
including even pork (2 Macc. 6:7-9, 18-21). In defiance of this decree, "many in Israel found
strength to resist, taking a determined stand against the eating of any unclean food" (1 Macc.
1:62). Allegedly, "(t)hey welcomed death and died rather than defile themselves and profane
the holy covenant."

Oddly enough, the book of Daniel tells of another group of Jews who risked their lives rather
than to defile themselves with unclean food. Daniel and three other Judean captives,
Shadrack, Meshach, and Abednego, were selected to be educated in "the learning and the
tongue of the Chaldeans" (1:4). Their training program included "a portion of the royal rations
of food and wine" (v:5), but Daniel "resolved that he would not defile himself with the royal
rations" (v:8), and so he prevailed on the king's palace master to allow him and his friends to
eat for a period of ten days only food that was clean to them, after which their health would be
compared to the others in the training program. The palace master consented, and the young
Jewish captives were permitted to eat only clean foods. The results? At the end of 10 days,
they "appeared better and fatter than all the young men who had been eating the royal rations"
(v:15), and when they were later brought before the king, he found them in learning and
wisdom to be "ten times better than all the magicians and enchanters in his whole kingdom"
(vs:18-19). Jews living under the tyranny of Antiochus Epiphanes would get from this story
the obvious message that Yahweh will look after those who faithfully follow his dietary laws,
so in view of all the other evidence against the traditional view of Daniel's authorship, it's
hard to believe that a book actually written in the 6th century B. C. would have very
conveniently contained a story so clearly parallel to a religious dietary crisis that would
happen four centuries later.

Antiochus put pagan idols in the temple at Jerusalem and consecrated it to Zeus (2 Macc. 6:1-
2). Those who sacrificed in the temple had to do so on altars dedicated to pagan idols, and
anyone who disobeyed did so at the risk of death that had been decreed by royal command (1
Macc. 1:47-50). By another convenient coincidence, there is a story in Daniel that 2nd-

Volume 1990 - 2002 Issue


Page 1256 of 2049
Skeptical Review Edited by Farrell Till
century B. C. Jews suffering such persecution would have easily related to. Chapter 3 in
Daniel tells the story of a great image that Nebuchadnezzar set up in the province of Babylon
and decreed that on a signal from cornets, flutes, and other musical instruments, everyone was
to fall down and worship the idol. When Daniel's friends Shadrack, Meshach, and Abednego,
who also had apparently been appointed to high positions in the province (v:12), refused to
obey the king's decree, they were thrown into a fiery furnace that had been heated seven times
hotter than normal, but their god enabled them to walk through the fire unharmed. Second
Maccabees 7 tells the story of seven brothers who refused to obey the pagan decrees of
Antiochus. The king ordered "great pans and cauldrons to be heated," and when the first
brother refused to obey the king's decree, he was roasted alive in one of the pans. Then one by
one, the other brothers suffered the same fate, while their mother stood by watching and
urging them not to relent, promising that the god who had given them life "will in his mercy
give you back again breath and life" (v:23). To 2nd-century B. C. Jews who suffered such
persecutions as these, the moral in Daniel's story of the fiery furnace would have been
obvious: Serve Yahweh faithfully, and he, as the mother assured her sons, will deliver you.

According to Daniel, both Nebuchadnezzar and Belshazzar desecrated the sacred vessels from
the temple in Jerusalem, Nebuchadnezzar by taking them to Babylon to put in the house of his
god (1:2) and Belshazzar by sending for the vessels during the great festival for his lords so
that they could drink wine from them while praising their gods of gold, silver, brass, iron, and
wood (5:1-4). Like these Babylonian kings of old, Antiochus Epiphanes also profaned the
vessels of the temple. He had the "audacity to enter the most holy temple on earth" and lay
"his polluted hands on the sacred vessels" and gather up "the votive offerings which other
kings had made to enhance the splendor and fame of the shrine" (2 Macc. 5:15-16). The
arrogance of Belshazzar in profaning the vessels of the temple was immediately punished,
because that very night "Belshazzar the Chaldean king was slain" when the kingdom fell to
"Darius the Mede" (5:30-31). The implication of a story like this would have been readily
apparent to the writer's 2nd-century audience, which was surely aware of Antiochus's
desecration of the sacred vessels in the temple. It was a story intended to assure his readers
that Yahweh would eventually make Antiochus pay for his arrogance.

Such convenient coincidences as these in the story of a 6th-century B. C. captive who,


choosing to serve Yahweh faithfully, was rewarded with a position of prominence in the
kingdom of his captors complements the mountain of other evidence that indicates the author
of this book was actually a 2nd-century writer who wanted his contemporaries to believe that
a prophet living long ago in another difficult period of Jewish history had foreseen their
sufferings and predicted that they would triumph over oppression.

Price's Last Stand


Dr. James D. Price

Volume 1990 - 2002 Issue


Page 1257 of 2049
Skeptical Review Edited by Farrell Till
Mr. Till continues to attack the integrity of the text of the book of Jeremiah. He stated: "I have
shown that the existence of two very different versions of the book of Jeremiah (the
Septuagint and the Masoretic) is sufficient to cast suspicion on the integrity of the text" (TSR,
July/August 1997, p.2). His use of the term "very different" leads the readers to believe that
the two text traditions are contradictory with significantly different messages. This gives a
false impression. True, the order of the chapters is different in the two traditions. True, that
the Masoretic tradition is longer than the Septuagint tradition, but neither of these facts
suggest that the two traditions are contradictory or have divergent messages.

On the contrary, both text traditions record the same history, the same sermons, the same
oracles against the nations, and the same poems. The Masoretic text has all the characteristics
of an expanded second edition by the same author, with a new order, new material, and with
added details and explanatory glosses. For the most part, differences that go beyond these
characteristics are accounted for on the basis of the ordinary copyist variations that occur in
all ancient manuscripts. Particularly that is true for the text of the prophecy under debate.
Both text traditions record the same date, the same prediction, and the same seventy year time
period. Apart from minor details, textual critics can recover the earliest form of each text
tradition. That is true because the manuscript witnesses to Masoretic text of Jeremiah are no
more diverse than those of the Masoretic text of other Old Testament books. The same is true
for the text behind the Septuagint tradition. The text of Jeremiah is no more divergent than
any other ancient historical source for which there are multiple copies.

All this was discussed in an earlier article, yet Mr. Till continues to make this strong
accusation based on vague generalities which he has not supported with objective evidence.
Let him produce convincing evidence that the text of the book of Jeremiah is unreliable. Let
him produce one ancient Hebrew manuscript with a date for this prophecy other than the
fourth year of Jehoiakim. Then let him explain why historians regard the book as a reliable
resource for the history of that period.

Mr. Till misrepresented my argument for the validity of Jeremiah's text. He represented me as
saying "If we can't trust the Bible, then `all ancient history is invalid'" (TSR,
September/October 1997, p. 10.). That is not what I said at all, as all readers can verify. Come
on, Farrell! That is not Kosher. Play by the rules. I said, "Of course, this is not a problem
unique to the text of the Bible, but to all ancient historical records. If such evidence is
necessary before an ancient text has validity, then all ancient history is invalid" (p. 5). What
evidence was I referring to? The evidence from nonexistent ancient copyright agencies and
registered original texts in official archives-- the evidence Mr. Till said must be available
before the text of Jeremiah could be validated (TSR, May/June, 1997, p. 7). This is more
evidence that Mr. Till uses a double standard. Every one knows that such means for validating
the text of ancient documents did not exit in antiquity, yet he is willing to accept the validity
of the texts of ancient secular documents without such nonexistent evidence.

Perhaps Mr. Till does not understand what the term "validity" means when it applies to
ancient texts. The text of an ancient document is regarded as "valid" if the words preserved in
the existing copies of the document can be regarded as reliably representing the message and
intent of the original document. The text of an ancient document may be regarded as "valid"

Volume 1990 - 2002 Issue


Page 1258 of 2049
Skeptical Review Edited by Farrell Till
without one accepting the message of the document as true. A "valid" text may convey a false
message.

In this regard, the texts of the Biblical books are more valid than almost all of the texts of
ancient documents used for studying history. This is true because there are many more extant
ancient manuscript copies of the Biblical books than there are of the secular documents, and
the available textual evidence has greater consensus for validating the original words of the
books. Mr. Till may not want to accept the message of the Biblical books as true, but he is in
serious error to assert that he cannot do so because the texts of those books are less valid than
other documents of antiquity.

Sixth-Century Origin: Mr. Till has claimed that the evidence does not support a sixth-
century origin of the book. However, for him to make that claim, he must disregard the dates
in the text of the book, particularly the date assigned to the writing of that part of the book
containing the prophecy under debate. Jeremiah and his book are dated from the thirteenth
year of Josiah's reign (627 B. C.) to the captivity in 586 BC (1:1-3). The prophecy under
debate is dated in 605 B. C., the fourth year of Jehoiakim (25:1). That is the same year that
Jeremiah first wrote some of his book (36:1-4; 45:1) which clearly contained the prophecy
under debate (36:29). Other events are dated in the book (28:1; 32:1; 36:9; 39:1, 2; 46:2;
51:59; 52:4, 5, 12, 28, 29, 30, 31). The text records that Jeremiah was commanded to write
certain material (22:30; 30:2; 36:2; 36:28), and that Jeremiah (or his amanuensis, Baruch)
wrote certain material (36:4, 18, 32; 51:60). For Mr. Till to deny the sixth century date he
must regard all these passages as fraudulent, something that he must prove, not simply allege.

On the basis of objective evidence, the Septuagint translation was made in the third century B.
C. The Hebrew text behind the Septuagint translation must have been sufficiently well
established in the third century for the translators to have regarded it as authoritative. That
places that text tradition at least in the fourth and likely in the fifth century.

There are six manuscripts of Jeremiah from the Dead Sea Scroll collection which date in the
first or second century B. C., some of which are from the tradition behind the Septuagint, and
others from the Masoretic tradition. Emanuel Tov, Editor-in-Chief of the Dead Sea Scrolls
publication project, and a recognized authority on Old Testament textual criticism and one
"with no fundamentalist ax to grind," stated: "Between 1947 and 1956 fragments of more than
190 biblical scrolls were found in the eleven caves of Qumran" (Textual Criticism of the
Hebrew Bible, Minneapolis: Fortress Press, 1992, p. 103). Many of these were fragments but
with sufficient text to determine the textual tradition from which the manuscripts were copied.
Mr. Till quibbled about some of the manuscripts being fragmentary, as though reasonably
sized fragments cannot identify the text tradition to which they belong (TSR, July/ August
1997, p. 11). If the police find the skull and a few bones of a dead body, can they not
legitimately conclude that the whole living body once existed? And cannot an expert biologist
determine the original size, weight, race, gender, and general appearance of the dead person
from the evidence available in the extant fragments of the body? Cannot he study the DNA of
these remains and determine its significant genetic codes? So also, textual experts can
determine the textual family of a fragmentary text. If reasonably sufficient fragments of the
Masoretic text exist among the Dead Sea Scrolls, then common sense admits the existence of
the whole text. If the fragments confirm the essential details of the corresponding parts of the

Volume 1990 - 2002 Issue


Page 1259 of 2049
Skeptical Review Edited by Farrell Till
Masoretic text, then common sense admits that the missing fragments would also confirm the
essential details of the text tradition to approximately the same degree of agreement.

A complete manuscript of the book of Isaiah (1QIsaA) exists from the second century B. C.,
and it has about 95% agreement with the Masoretic text. Another manuscript of Isaiah
(1QIsaB) contains much of the text of 46 chapters of the book. This manuscript is almost
identical with the current form of the Masoretic text. Tov (pp. 31-32) listed a catalogue of the
types of differences between 1QIsaB and BHS (Biblia Hebraica Stuttgartensia), the accepted
form of the Masoretic text today: (1) Orthography (spelling differences), 107; (2) Added waw
conjunctive, 16; (3) Lack of waw conjunctive, 13; (4) Article (added/ omitted), 4; (5)
Difference in consonants 10; (6) Missing letter, 5; (7) Different grammatical number, 14; (8)
Differences in pronouns, 6; (9) Different grammatical form, 24; (10) Different proposition, 9;
(11) Different words, 11; (12) Omission of words, 5; (13) Addition of words, 6; (14) Different
sequence, 4.

That amounts to 234 differences of any kind "all of which concern minutiae" (Tov, p. 31).
However, items 1, 2, 3, 4 and 14 have little or no effect on meaning, so they may be
disregarded as insignificant. This leaves only 90 differences that may be regarded as of any
possible significance.

There are 66 chapters in the book of Isaiah, 1291 verses, 16,930 words, and 66,884 letters in
the current Masoretic text of Isaiah. If the number of words in 1QIsaB is estimated as 16,930
x 46/66 x .66 = 7,788 words, then 1QIsaB agrees with BHS (7,788 - 234) / 7,788 = 97.0%; or
if the insignificant variations are excluded, the texts agree (7,788 - 90)/ 7,788 = 98.8%. That
is about the kind of agreement that any manuscript of the Masoretic text has. Regarding the
Masoretic text in the era of the Dead Sea Scrolls, Tov, who is liberal in his approach the
Biblical text, wrote:

"Similar analysis is suggested by Andersen-Freedman... in their analysis of 4QSamB,


one of the earliest Qumran texts: `(I)nsofar as there is nothing un-Masoretic about the
spelling in 4QSamB, we can infer that the Masoretic system and set of spelling rules
were firmly in place in all principles and particulars by the third century BCE.'"

Because of the meticulous care of those who were involved in the copying of [the
Masoretic text], the range of differences between the members of the [Masoretic]
group was from the outset very small. One should remember that the temple employed
professional magihim, "correctors" or "revisers," whose task it was to safeguard
precision in the writing and transmission of the text (Tov, p. 32).

Such correctors or revisers were not responsible for altering the text, but for correcting or
revising manuscript copies that varied from the official exemplar in their care. It was this
meticulous care of the text that led scholars like these in the next generation to confirm that
the Masoretic text was the authentic tradition.

This places the textual tradition behind the Masoretic text at least in the fourth and likely in
the fifth century. But the witness of the Aramaic translation known as the Targum gives good
reason to place the Masoretic text of Jeremiah in at least the sixth century. Concerning the

Volume 1990 - 2002 Issue


Page 1260 of 2049
Skeptical Review Edited by Farrell Till
Aramaic Targum, Ernst Wurthwein, a recognized authority on Old Testament textual
criticism, stated: "The Jewish tradition associating it (the Targum) with Ezra (cf. Neh. 8:8)
may well be correct" (The Text of the Old Testament, Trans. by Erroll F. Rhodes Grand
Rapids: Wm. B. Eerdmans, 1979, p. 75). Now the importance of the Aramaic Targum of the
book of Jeremiah is that it was translated from a Hebrew text of the Masoretic tradition (Tov,
p. 149). If Wurthwein is correct, and there is no reason to doubt him, then the Masoretic
tradition of Jeremiah was already well established as authoritative in the fifth century B. C.
This gives reason to accept the sixth-century origin of the book with little reason to doubt it.
Not a shred of textual evidence exists that suggests that the date of Jeremiah's prophecy was
ever altered. If such evidence exists I'm sure Mr. Till would have called it to our attention.

This is supported by several fifth- or sixth-century witnesses to the existence of the book, and
the prophecy under debate in particular: (1) the author of the Chronicles (2 Chron. 36:22-23),
(2) the author of Ezra-Nehemiah (Ezra 1:1-5), (3) the prophet Zechariah (Zech. 1:12; 7:5), and
(4) the sixth-century prophet Daniel (Dan. 9:2). These very early witnesses knew Jeremiah's
book, and the prophecy under debate in particular. All of these witnesses accepted Jeremiah
as a historical person and the author of the prophecy. All regarded the prophecy as genuine,
not fraudulent.

In fact, Daniel read Jeremiah's prophecy before it was fulfilled (Dan. 9:1-2). This is evident
from the fact that Daniel did not record the fulfillment of the prophecy--something that would
have been significant to the content of his ninth chapter. I know Mr. Till rejects the date and
authorship of Daniel, and I am not interested in debating that question. But there is no reason
to late-date Daniel except Mr. Till's anti-supernatural presupposition. In my own opinion,
Daniel is a valid witness because his contemporary, the prophet Ezekiel, validated his date
and existence (Ezek. 14:14, 20; 28:3). This does not include the mention of the prophet
Jeremiah by the historian Josephus, the authors of some of the Apocryphal books (Sirach
49:6; 2 Macc 2:1, 5, 7; 15:14, 15; 1 Esdras 1:28, 32, 47, 57; 2:1; 4 Esdras 2:18), the Mishnah
and the Talmud. All these ancient sources regarded the prophet and his writings to be
authentic.

Now Mr. Till has objected to the cumulative force of all this evidence and asserted that I have
not provided sufficient evidence that Jeremiah wrote the prophecy before its fulfillment.
Instead he insisted that "What Dr. Price must find first is clear, incontestable evidence in
external sources that Jeremiah made this prophecy before the fact and that it was not put into
the text by revisionist scribes and editors" (TSR, Sept./October, p. 10). In other words, I am
supposed to prove that Mr. Till's claim of fraud is false. But as I have shown, the laws of
jurisprudence require the claim of fraud to be proved. The burden of that proof lies heavily on
Mr. Till. But if I were to find other clear, incontestable, external sources he would do with
them what he has already done with the four I have provided: he would label those sources as
fraudulent also. Remember, no amount of evidence will convince him. I have provided textual
evidence for the validity and early date of the book that is far more numerous and relatively
early than that for any secular historical document Mr. Till accepts without question. I have
provided four very early, independent, reputable witnesses to the existence of the book and
the prophecy. These witnesses were all recognized men of integrity, intelligence, and skill.
None would have accepted the authority of a known fraud, nor would they have been a part of
fraud. They would have known the textual evidence of their day, much like competent

Volume 1990 - 2002 Issue


Page 1261 of 2049
Skeptical Review Edited by Farrell Till
scholars do today. They would not have tolerated fraudulent tampering with the text. Yet Mr.
Till's response to these unimpeachable witnesses is that they too are frauds. They are guilty of
misrepresenting Jeremiah's prediction as a legitimate fulfilled prophecy when in fact it is a
fraud. Now, instead of one alleged fraudulent document we have five. But in so doing, Mr.
Till has multiplied his claims of fraud and multiplied his burden of proof! What has he offered
as proof? Unverifiable, subjective theories, and his anti-supernatural presupposition.

In addition, Mr. Till's denial of the sixth-century origin of the book of Jeremiah goes contrary
to the very critical scholars he cited as authority. As he indicated, The Interpreter's Bible
alleges that the 25th chapter of Jeremiah was written by the "Deuteronomist" (D), a later
scribe who added his material to Jeremiah's book. However, The Interpreter's Bible also
stated that "the D edition of Jeremiah was made about 550 B.C. The man (or group) who
made it may have lived in Egypt. D made use of the three preceding collections, adding his
own work to them or revising them at some points" (vol. 5, p. 788). Note that the date
assigned to the prophecy under debate was in the sixth century, and almost twenty years
before the prophecy was fulfilled. It seems that Mr. Till's critical authority does not support
his radical skepticism. Even if we accept the view of The Interpreter's Bible that the prophecy
was written by the "Deuteronomist" nearly 20 years before its fulfillment, the prophecy would
be genuine according to Mr. Till's three criteria, because foresight of 20 years is also beyond
human expectation. Mr. Till needs to find more radical critics to cite as authority.

Multiple Authors and Redactors: Mr. Till cited The Interpreter's Bible, which shares his
critical skepticism, as asserting that the book of Jeremiah "is not the product of one person or
of a small group of persons. It is the product of growth over a long period of time, to which
many contributed" (TSR, July/ August p. 2). Of course, such an assertion does not constitute
proof. It is only the opinion of another skeptic. Not a shred of objective evidence was
presented. He then cited another critic of the same stripe who stated: "style, grammar, syntax,
vocabulary, theme, and literary structure enables critics to recognize where redacting and
editing occurred in the biblical text" (TSR, July/August, p. 2). These are his defense of the
"fraud" hypothesis which he offers as an alternative to Jeremiah's authorship of the book. Of
course, such statements are based on the underlying hypothesis that writers are incapable of
writing in more than one style, vocabulary, theme, and literary structure. But just to articulate
such a hypothesis shows how ridiculous it is. Only a literary clod would have such
limitations. And if such a clod were to write something, who would publish or perpetuate his
literature? Are we to assume that the great prophets of the Bible were literary clods? Who is
to say that a given writer cannot produce a variety of literature such as history, poetry, novels,
biography, sermons, hymns, and theology? Would anyone dare to assume that such a writer
would be limited to one style, vocabulary, and literary structure for such a variety? Any
student of literature recognizes how artificial this hypothesis is.

Indeed it is a mystery
Why writers of ancient history
Could never take the time
To cast their thoughts in rhyme;
And poets could never compose
An essay in common prose;
And a master of ornithology

Volume 1990 - 2002 Issue


Page 1262 of 2049
Skeptical Review Edited by Farrell Till
Could never write theology,
But must use the very same words
To write of God and birds (JDP).
Surely Mr. Till, a former professor of English literature, knows that such an idea cannot be
supported. Obviously he can write in diverse styles and on manifold topics in vocabularies
suited for the occasion. Would he deny the same proficiency to the author of an ancient
classic?

According to John Bright, a scholar who shares Mr. Till's hypothesis of multiple authorship,
some critical scholars identify three types of literature in the book of Jeremiah: (1) Type A
consisting of prophetic oracles written primarily in poetry; (2) Type B consisting of
biography; and (3) Type C consisting of prose discourses in a rhetorical style similar to
Deuteronomy. (Jeremiah, The Anchor Bible, Garden City, NY: Doubleday & Company, Inc.,
1965, pp. lxiii-lxviii). According to Bright and other scholars of that ilk, each of these types
was written by a different author. Type A is usually attributed to the sixth-century prophet.
Note that, contrary to Mr. Till, the critical scholars really do believe there was a historical
Jeremiah. Types B and C are usually attributed to later authors who added their material to the
original work of Jeremiah--Type C being the product of a scribe or scribes of the
Deuteronomic school. Of course, no one explains why the poet Jeremiah could not have
written biography and prose; and no one explains how the disciples of Jeremiah, the early
rabbis, and the educated Jews would have tolerated anyone grossly tampering with the
original text of Jeremiah.

Regarding the Type C portions, the type to which some critics assign our prophecy, Bright
stated:

Suffice it here to say that the style of these discourses, though indeed closely akin to
that of the Deuteronomic literature, is a style in its own right with peculiarities and
distinctive expressions of its own; it is by no means glibly to be classified as
"Deuteronomistic." It is, moreover, not a late style, but a characteristic rhetorical prose
of the seventh/sixth centuries.... Though it may well have undergone some verbal
expansion after that time at the hands of editors and scribes, there is really no reason
to place any of it (or anything in the book, for that matter) after the Exile (Bright, p.
lxxi).
Although Bright is in general agreement with the critical approach to the book, he was honest
enough to acknowledge its limitations:
We are not, however, permitted to operate on this material by the methods of literary
criticism, as if by peeling off "non- genuine" accretions we could arrive at the
"genuine" words of the prophet. To attempt such a thing is a wholly subjective
procedure. Viewed as literature, the prose discourses are a unity and are to be treated
as such; they afford no stylistic canon for separating "genuine" from "nongenuine"
words. On the other hand, to form a priori notions of what the prophet could and
could not say, and then to attempt a separation on that basis, is indefensible and no
more than a projection of the critic's own predilections... In spite of undeniable verbal
differences, the contrast between the Jeremiah of the poetry and the Jeremiah of the
prose (and, one might add, between Jeremiah and the "Deuteronomists") has been, by
many scholars at least, badly exaggerated (Bright, p. lxxiii).

Volume 1990 - 2002 Issue


Page 1263 of 2049
Skeptical Review Edited by Farrell Till
In fact, some critics have identified the original "Deuteronomist" as Jeremiah (J. A.
Thompson, The Book of Jeremiah, The New International Commentary on the Old Testament,
R. K. Harrison, Gen. ed. Grand Rapids: Wm. B. Eerdmans, 1980, p. 44).

Beside the subjective nature of literary criticism, and the exaggerated manner in which critics
evaluate differences, one has only to read the various critical authors on the analysis of the
book of Jeremiah to observe one additional flaw: No two literary critics agree on the details of
their analysis. In fact, it is not unusual to find the critics being very critical of one another.
The Interpreter's Bible, which represents the critical view Mr. Till accepts, stated: "Though
the subject has long been studied, there is no consensus among scholars as to the stages in the
composition of the book of Jeremiah, or the process of growth by which it arrived at its
present state" (vol. 5, p. 787). This type of disagreement is evidence of the weakness and
subjectivity of the discipline as applied to Jeremiah. For a hypothesis to be eligible for
acceptance, there should be consistent agreement of the details when tested by different
investigators. Concerning the test of consistency of details, the literary criticism of Jeremiah
fails miserably. Mr. Till's claim of fraud cannot be proved by an appeal to the highly
subjective, overly exaggerated, and contradictory evidence from literary criticism.

Jeremiah the Deceiver: I have previously responded to Mr. Till's false accusation that
Jeremiah was a deceiver (TSR, September/October, pp. 4-5). No need exists for further
discussion of his attack on Jeremiah's veracity.

Jeremiah the Plagiarist: Mr. Till has accused Jeremiah of plagiarism in that a portion of
chapter 52 is almost verbatim with a similar passage in Second Kings. Of course, Mr. Till
admitted that he does not know whether Jeremiah copied Second Kings, or the author of
Second Kings copied Jeremiah, or whether both copied from some other source now
unknown. Nevertheless, even though Mr. Till does not know who copied who, he still accused
Jeremiah of plagiarism (TSR, July/August, p. 3). This is an example of his claimed "common
sense reasoning." The fact is that ancient writers often failed to acknowledge the source of
material they used. That was not illegal or unethical in those days. It is also very likely that
the kind of material common to Jeremiah and Second Kings was available through public
records. The use of public information cannot be regarded as plagiarism even under the
modern understanding of the term. Mr. Till has been so eager to malign Jeremiah's character
that he has allowed himself to be very careless with the evidence.

I have previously shown that Jeremiah did not write chapter 52, but that it was a historical
appendix written by a later author, probably Baruch. The Interpreter's Bible, which Mr. Till
cited as authority, declared: "Ch. 52 is an appendix to the book of Jeremiah added in order to
show how some of his prophecies were fulfilled" (vol. 5, p. 790). If that was the purpose of
the appendix, then it must have been added before the end of the Exile, because otherwise the
author would have mentioned the restoration of the Jews to their homeland, and the
fulfillment of the seventy-year prophecy.

Altered Text: Mr. Till's claim of fraud includes the allegation that the text of Jeremiah was
altered after the Exile in order to make it appear like a prophecy had been fulfilled. Of course,
the only evidence he had to offer is the unverifiable subjective hypothesis of literary criticism
discussed above. He appealed to the fact that this prophecy is part of the "Deuteronomic"

Volume 1990 - 2002 Issue


Page 1264 of 2049
Skeptical Review Edited by Farrell Till
segment of the book, and that this segment was allegedly written after the Exile, and so after
the fulfillment. However, critical scholar John Bright, as cited above, stated that from a
literary critical point of view the "Deuteronomic" prose is from the seventh/sixth century and
no literary reason exists to date it after the Exile (Bright, p. lxxi).

The part of the text that is crucial to fulfilled prophecy is the date the prophecy was given--the
fourth year of Jehoiakim (605 BC). Unfortunately for Mr. Till, not a single existing Hebrew
manuscript contains an alternate date. The same is true for all the ancient translations of
Jeremiah. The other part of the text that is important for fulfilled prophecy is the phrase
"seventy years." Mr. Till has quibbled extensively that the time between Jehoiakim's fourth
year and Cyrus' first year is not seventy years, but something less than that. Now, let's put Mr.
Till's claim to the test. If a post-exilic editor wanted to alter the text to make it look like a
fulfilled prophecy, why would he not put in the exact number of years instead of a round
number? After all, an exact number would be much more impressive, much more convincing!
Unfortunately for Mr. Till, no existing Hebrew manuscript contains an alternate number. The
same is true for all the ancient translations of Jeremiah. Thus, no objective evidence exists to
support Mr. Till's claim of a fraudulent alteration of the text. All objective textual evidence
supports the date and the reading "seventy years." So Mr. Till can only erroneously appeal to
his anti- supernatural presupposition. Sad to say, he has failed to prove his claim of fraud.

Round Number: Mr. Till objected to my interpretation of the "seventy years" as a round
number. According to him, the time of Israel's subjugation to the Babylonian kings has to be
exactly seventy years for the prophecy to be fulfilled. In this he disagrees with competent
critical scholars like John Bright who said: "This seems to be here no more than a round
number, i.e., a normal life span" (Bright, p. 160). Also he disagrees with The Interpreter's
Bible, his source of critical scholarship, which declared: "Seventy years constitute the period
during which the Jews were to serve Babylon, at the end of which Babylon itself was to be
punished. . . . The number was not to be taken literally, but rather as a round or `perfect'
number, perhaps the length of a man's lifetime (Ps. 90:10)" (vol. 5, p. 1000). This
understanding is supported by extrabiblical texts (Thompson, p. 513). For Mr. Till to demand
mathematical precision in statements of this type is to go beyond the normal principles of
interpretation used by the critical scholarship he claims to accept, and the "common sense
reasoning" he claims to follow.

Mr. Till complained that I did not provide documentation that the Persians followed the
accession-year method for reckoning the regnal years of their kings. Of course that would
have been an excellent opportunity for him to provide documentation to the contrary, which
he failed to do, primarily because he knows I'm right. This point is not critical for
understanding the seventy years of subjugation as a round number. However, for the record,
Jack Finegan, a recognized authority on ancient chronology and one who has no
"fundamentalist ax to grind," stated: "The system of reckoning which prevailed in Babylon,
Assyria, and Persia, may be called the accession-year system.... In the accession-year system
the portion of a year from the accession of the king to the end of the then current calendar
year is only his 'accession year' (and for chronological purposes remains a part of the last
numbered regnal year of his predecessor), and the new king's Year 1 begins only on the first
day of the new calendar year after his accession" (Handbook of Biblical Chronology,
Princeton: Princeton University Press, 1964, pp. 85-86). Among the Jews the practice was not

Volume 1990 - 2002 Issue


Page 1265 of 2049
Skeptical Review Edited by Farrell Till
consistent. Sometimes they followed the accession-year system and sometimes the non-
accession system. Likewise, they sometimes followed the calendar year that began with the
month of Nisan, and sometimes the calendar that began with the month Tishri (Finegan, pp.
87-92). In part, the method used by Jewish historians depended on the system used by the
nation which held political dominance over them at the time. It is very likely that Ezra, a
prominent citizen of Persia, and under orders from the Persian king, used the accession-year
system of the Persians.

Also Mr. Till complained that the return journey of the captives would not be as long as I
indicated. However, we have Ezra's record of how long it took his relatively small company
to travel to Jerusalem--four months (Ezra 7:9). It surely would take quite a bit longer than
four months for approximately 49,000 captives to travel that distance. Indeed, the available
evidence supports my estimation of the time involved. Mr. Till has extensively quibbled over
the interpretation of a round number. The fact that his critical scholars agree with me indicates
how radical and unreasonable his attempts at rebuttal are.

Conclusion: I have demonstrated that all the objective evidence supports the fact that the
prophet Jeremiah wrote the prophecy in 605 B.C., and that the prophecy was fulfilled at the
end of the predicted "seventy years," interpreted as a round number. The round-number
interpretation has the consensus of both conservative and liberal scholars. Mr. Till has
countered with his fabulous claim that the prophecy was a fraud written after its fulfillment
with the intent deceive. He failed to demonstrate that his claim of fraud satisfies the objective
evidence better than does the clear, unaltered statement of the text. I have presented numerous
lines of consistent objective evidence. Mr. Till has relied on unverifiable, inconsistent
subjective hypotheses to support his view. This ends my case, and my part of the debate. The
readers must evaluate the evidence for themselves and not let someone else do their thinking.
The verdict seems quite certain. Jeremiah 25:1-13 is a genuine fulfilled prophecy. I thank Mr.
Till for the opportunity to present the case for fulfilled prophecy, and I thank the readers for
patiently enduring the presentation.

(James D. Price, Ph.D., Professor of Hebrew and Old Testament, Temple Baptist Seminary,
Chattanooga, TN 37404; e-mail, drjdprice@aol.com)

Plowing the Same Ground


Farrell Till

What a relief to hear that Dr. Price is ending his part of this debate! We have sat patiently for
a year now and watched him plow the same ground over and over, through an endless maze of
quibbles and straw men, without ever trying to resolve the real difficulties in his claim that
Jeremiah accurately predicted the duration of the Babylonian captivity. He has been given

Volume 1990 - 2002 Issue


Page 1266 of 2049
Skeptical Review Edited by Farrell Till
well over a thousand dollars in free publishing space to present his case, and through it all he
has apparently thought that if he fought enough straw men, led us down enough tangent paths,
distorted enough of my rebuttal arguments, and kept parroting the same claims after they had
been answered, he might convince some readers that his position is right. To avoid boring
repetition, I will reply only briefly to his regurgitated complaints and "arguments" that I have
already responded to.

Jeremiah Not an Actual Person: Once again, Price has accused me of claiming that
Jeremiah was not an actual historical character (p. 4, this issue), but I showed in the May/June
edition that Price had distorted a section of my article in which I had said only that there were
no nonbiblical records that "mentioned a Hebrew prophet named Jeremiah who prophesied
during the first year of Nebuchadnezzar's reign that the Jews would be taken into bondage but
released after 70 years" (p. 8). So my claim was really that there were no nonbiblical records
to corroborate that Jeremiah had made such a prophecy and not that there were no nonbiblical
records to corroborate the historicity of Jeremiah. This is just a typical case of Price's fighting
straw men rather than trying to address the real problems in his prophecy-fulfillment claim.

The Sixth-Century Origin of Jeremiah: Price has again asserted that I have claimed "that
the evidence does not support a sixth-century origin of [Jeremiah]" (p. 2, this issue), but this is
another distortion of my position. I have never claimed that none of the book of Jeremiah was
written in the sixth century. I have said only that through the use of textual criticism, biblical
scholars have determined that the book was "the product of growth over a long period of time,
to which many contributed" (July/August 1997, p. 2). This point was discussed at length on
pages 2-4 of the issue just cited. In "Primary colors of the Bible" (July/August 1998), I
showed that responsible scholars, such as Don Foster of Vassar College, have established the
reliability of critical methods that textual critics use to determine forgery and multiple
authorship in biblical books, so as long as reputable scholars, using these proven methods,
express serious doubts that all of the book of Jeremiah was written by one person in the 6th-
century B. C., it is incumbent on Dr. Price as the affirmant to establish beyond doubt that his
claim meets all of the criteria of valid prophecy fulfillment. He certainly has not done that as
long as there are scores of reputable biblical scholars who, having critically examined the text
of Jeremiah, declare that this book is "the product of growth over a long period of time, to
which many contributed," for if this book was so written, that would undermine credibility in
the genuineness of the prophecy statement that is so crucial to Price's position. He must
establish that critical evaluations of the book, which question the authenticity of some of it,
are completely without merit, and he cannot do that by just quoting writers who disagree with
this view, as he did in this his "last stand." I'll have more to say later about his appeals to
writers who disagree with the principles of higher criticism.

The Dating of the Prophecy: Despite the amount of space that I have devoted to answering
Price's prating about the date of Jeremiah's prophecy, he has taken us down the same path
again without bothering even to acknowledge my response to this irrelevant matter. "The
prophecy under debate is dated in 605 B. C.," he said again, "the fourth year of Jehoiakim" (p.
2, this issue). But what has he said in reply to my observation that the passage he has cited in
support of this point said only that the word of Yahweh came to Jeremiah in the fourth year of
Jehoiakim (25:1)? This text in no way implies that Jeremiah was prophesying that the event
he was predicting, i. e., a Babylonian captivity of 70 years, was going to begin at that time.

Volume 1990 - 2002 Issue


Page 1267 of 2049
Skeptical Review Edited by Farrell Till
The best that Price can get from this text is a claim that Jeremiah prophesied in 605 B. C. that
there would be a captivity of 70 years (v:11), but the crucial matter is when the predicted
captivity actually began, and it did not begin in 605 B. C. Price likes to quote scholars (as if
finding scholars to quote in support of a religious belief is hard to do), so let's see him find
reputable scholars who would agree that the Babylonian captivity of the Judean Jews began in
605 B. C. He can't do it, yet he keeps insisting that the period of captivity began in 605 B. C.
and ended in 537 B. C., a span of time long enough to consider Jeremiah's 70 years to be an
accurate "round number." I have shown that Price's dates are incorrect on both ends, but a
biblicist with a pet theory to defend is not going to allow compelling evidence against it to
keep him from preaching it on the roof tops.

I have been over and over this matter, but Price has largely ignored it. I have even pointed out
that if we concede (incorrectly) that the captivity began in 605 B. C., Price still could not get
the 70 years that he needs to claim accurate prophecy fulfillment. Cyrus issued his decree in
539 B. C. that permitted the captives in Babylon to return to their homelands, so if Price will
just subtract 539 from 605, he should be able to see that the answer is 66. He has begged for a
two-year leeway that would have allowed time for the captives to travel from Babylon to
Jerusalem, but he has ignored my observation that Jeremiah's prediction was that the captive
nations would "serve the king of Babylon seventy years" (25:11). A people traveling back to
their homeland after having been granted their freedom by royal decree could hardly be
described as captives who were "serving the king of Babylon." Besides, Babylon had been
conquered by this time and absorbed into the Persian empire, and it was a Persian king who
had granted the Jews their freedom to return home. So in what sense could Price think that
they were "serving the king of Babylon" during this hypothetical two-year journey to
Jerusalem? Any way he looks at it, Price just can't get a Babylonian captivity of 70 years or
one even close enough to be considered a "round number." It does seem that a prophet guided
and inspired by the omniscient, omnipotent Yahweh in his prophecy utterances could have
performed a bit better than Jeremiah did in this matter.

The Integrity of Jeremiah's Text: Price tried desperately to show that "(t)he Hebrew text
behind the Septuagint translation must have been sufficiently well established in the third
century for the translators to have regarded it as authoritative" (p. 2, this issue), but he has
failed to give us a reasonable explanation for why two very different versions of the book of
Jeremiah were circulating simultaneously. A difference in theological views would be the
most likely explanation. Some groups apparently considered the short (Septuagint) version the
right one; others apparently considered the long (Masoretic) version the right one. In "The
Text Behind the Text of the Hebrew Bible," Harvard Professor Frank Moore Cross discussed
textual variations in biblical manuscripts and showed that many of them were the result of
"variant streams of tradition" that have been recognized as "`recensions' or `families' or `local
texts'" (Understanding the Dead Sea Scrolls, Random House, New York, 1992, p. 146). In
other words, it is the consensus of reputable scholars that variations in manuscripts developed
because of regional differences in theological views. Jeremiah was one of the books in which
Cross noted significant variations.

Sometimes one of these text types differs strikingly in detail from the traditional text
that has come down to us. In extreme instances we discover that a textual tradition is
preserved in a manuscript that stems not merely from textual changes in individual

Volume 1990 - 2002 Issue


Page 1268 of 2049
Skeptical Review Edited by Farrell Till
readings; it derives from an edition of a biblical work different broadly in content and
length from the edition used in the Rabbinic Recension. For example, there are two
editions of Jeremiah represented in manuscripts from Qumran: a long edition known
from our traditional Bible and a short edition that also differs in the order of the
prophetic oracles (p. 146).
Cross went on to explain that from "local texts," three primary textual traditions had
developed: the Hebrew textual base of the Old Greek (Septuagint) translation, the textual
background of the Samaritan Recension of the Pentateuch, and the text type that was utilized
in the Rabbinic (Masoretic) Recension (p. 147). He stated that these three forms of the text
"appear to have developed slowly between the fifth century B. C. and the first century B. C."
(p. 147). The "local texts" from which these textual traditions had developed were Palestinian,
Egyptian, and Babylonian. In other words, "standardization" of the texts didn't exist in the
four-century span during which the three textual traditions were developing. The texts that
groups used depended upon the geographical regions they lived in, and, according to Cross,
the Greek tradition enjoyed popularity in Egypt. Since the Septuagint was translated in
Alexandria, the translators' use of the short edition wouldn't necessarily indicate, as Price
supposes, that it was "sufficiently well established" among all Jewish groups for the
translators to have regarded it as "authoritative." It simply happened to be the preferred
version in the region where the translators did their work, but in other places, a longer,
significantly different version was preferred.

Perhaps the Hellenized Egyptian Jews had a penchant for brevity, because Cross pointed out
that Greek scholars in Alexandria had also produced a short edition of Homer (p. 150).
However, regardless of why some Jews of the time may have preferred a short edition of
Jeremiah while others in different regions preferred the longer version, there were clearly two
versions in use, and this fact doesn't help the popular biblicist view that men like Isaiah,
Ezekiel, and Jeremiah sat down and, while under the "inspiration" of God, wrote their books,
which were thereafter carefully preserved by conscientious, meticulous scribes. If Jeremiah
was so written, then why did its material need to be radically reorganized and why did
substantial additions need to be made to it? These are questions that will continue to cast
serious doubts on the integrity of the book of Jeremiah until better answers than Price's have
been given. Even he has admitted that "it is true that the text of Jeremiah has a greater portion
of variations than most of the other Old Testament books" (May/June 1997, p. 3), but he
brushes this fact aside with that claim that "it is not true that such variations compromise the
integrity of the book." He has yet to explain to us why so many variations in a book allegedly
written in the 6th century B. C., whose text cannot be traced farther back than the 3rd century
B. C., should not be considered sufficient reason to wonder what may have happened to the
text during the three-century "developmental period" Cross referred to, for which we have no
textual records. That's the problem that Price has not and cannot satisfactorily resolve, but
resolve it he must in order to sustain his claim that late in the 7th century B. C., Jeremiah
predicted that the Judean Jews would be held captive in Babylon for 70 years. If by chance he
could solve this problem, all he would prove is that Jeremiah made a prophecy that failed,
because I have shown that no chronological evidence supports Price's claim that the captivity
lasted close enough to 70 years to consider that it was accurately fulfilled even in "round-
number" terms.

Volume 1990 - 2002 Issue


Page 1269 of 2049
Skeptical Review Edited by Farrell Till
Another False Analogy: In search of evidence to show that the text of Jeremiah has been
preserved essentially as the prophet and/or his scribe Baruch wrote it, Price referred to six
manuscripts of Jeremiah that were found in the Dead Sea Scrolls, but even he admitted that
"some" of these were "from the tradition behind the Septuagint, and others from the Masoretic
tradition" (p. 2, this issue), so all that Price has shown by this is that as late as the time of the
Essene settlements at Qumran, two entirely different versions of this book existed. Hence,
there is certainly no proof in the Dead Sea discoveries to support the view that a text of
Jeremiah basically free of tampering has been passed along to us.

In a ridiculously false analogy, Price referred to 190 manuscript fragments that were found at
Qumran. From this he argued that scholars have been able to determine "the textual tradition
from which the manuscripts were copied" (p. 3). I don't disagree that the textual tradition of
fragments can be determined, but there is a substantial difference in determining what "textual
tradition" fragments may have been copied from and in determining exactly how the entire
book that a fragment was copied from may have read at the time of the copying. In other
words, if a fragment is discovered that turns out to be Jeremiah 25:9 as it reads in the
Masoretic version (which mentions Nebuchadnezzar by name, whereas the same verse in the
Septuagint does not), this fragment alone would show scholars only that Jeremiah 25:9 in the
Masoretic version used at Qumran read essentially as it does in the Masoretic text of Jeremiah
that has been transmitted to us. However, it would not prove that the entire Masoretic text of
Jeremiah as it was used then was the same as it is now.

In an attempt to show otherwise, Price compared the finding of fragmentary body parts to the
finding of manuscript fragments. "If the police find the skull and a few bones of a dead body,"
he asked, "can they not legitimately conclude that the whole living body once existed?" The
answer is certainly yes, but there would be many things about the person that the discovery of
a skull and a few bones could not determine. If the person whose body it is had been someone
whose left arm had been amputated below the elbow, the discovery of a "few bones" could
not determine this unless the left arm bones were included in the discovery. If this person had
had a birthmark on his forehead, the discovery of a skull and "few bones" would not reveal
this. If this person had had six digits on the right hand, the discovery of a skull and "few
bones" would not show this unless the bones of the right hand were included in the discovery.
If no hair was found with the bones, we could not know whether the person was a blond,
brunet, or redhead. We can therefore see that Price's analogy is false and that the discovery of
a few fragments of a biblical manuscript would not show that the entire manuscript from
which the fragments were copied was exactly like the version of that book as it exists today.

Another Straw Man: Price spent almost two columns on an irrelevant discussion of a
manuscript of Isaiah that was found at Qumran. He listed fourteen categories of variations in
this manuscript from the Masoretic text of today in order to make a point about the general
insignificance of the variations, but since the book of Isaiah is not the book of Jeremiah, this
entire section of his article was irrelevant. It was just another straw man that he could beat on
to draw attention away from the obvious fact that he cannot establish that the book of
Jeremiah--the one that contains the prophecy under discussion--has been transmitted to us
virtually unchanged from the way that Jeremiah wrote it.

Volume 1990 - 2002 Issue


Page 1270 of 2049
Skeptical Review Edited by Farrell Till
The Aramaic Targum: After the Isaiah tangent, Price turned next to the Aramaic Targum to
try to show that it "gives good reason to place the Masoretic text of Jeremiah in at least the
sixth century" (p. 3, this issue). Price didn't explain what this "good reason" is, so I guess that
he just expects us to take his word for it. Well, he did cite Ernest Wurthwein's opinion that
"(t)he Jewish tradition associating it [the Targum] with Ezra (cf. Neh. 8:8) may well be
correct" (p. 3, this issue). From this, I assume that Price expects us to accept as fact what
some writer thought "may well be correct." If it "may well be correct," what is the evidence
that it is? Nehemiah 8:8, which Wurthwein referred to, merely states that a group of men
whom Ezra had selected to read publicly the book of the law "gave the sense and caused them
to understand it." That's pretty flimsy evidence on which to date the making of the Aramaic
Targum.

For the sake of argument, let's assume that the Targum did date back to the time of Ezra. Price
would still have the same problem: How can he prove that a Targum written at that time
included the book of Jeremiah and has remained essentially unchanged ever since? If the
Masoretic text itself was undergoing changes, what reason do we have to suppose that the
Targum didn't? The translation of Wurthwein's book that Price quoted was published by
Eerdmans in Grand Rapids, MI, so maybe Price would be willing to consider another opinion
about the Targum that was published in an Eerdmans book: "(T)he further claim that some of
the extant written Targums are as a whole pre-Christian is vigorously debated" (Eerdmans
Bible Dictionary, 1987, p. 985). The article goes on to say that "despite the uncertainty with
regard to dating," the Targums are relied on extensively in various branches of biblical
studies. However, the fact that they cannot be reliably dated isn't much confirmation of Price's
claim that they show "good reason to place the Masoretic text of Jeremiah in at least the sixth
century." The oldest existing targumic writings date only to the 2nd century B. C., and these
are fragments that were found at Qumran. That's about four centuries too late to prove Price's
claim.

The Several Fifth- or Sixth-Century Witnesses to Jeremiah: Price cited the writers of 2
Chronicles, Ezra, Zechariah, and Daniel as "very early witnesses" who knew Jeremiah's book
"and the prophecy under debate in particular" (p. 3, this issue). That Dr. Price would resort to
such desperation as this is surprising even for him. I don't have the space to review the
literature on these books, but surely Price is aware that these are recognized as late additions
to the Hebrew scriptures. In this very issue of TSR, I am continuing a discussion of the book
of Daniel to show that, contrary to the biblicist view, this book was written in the second
century B. C. during the Maccabean era, so that would hardly make its author a "very early
witness" to Jeremiah's book and its 70-year prophecy. The Chronicles, Ezra, and Nehemiah
are recognized as the works of a single author who wrote them probably in the 3rd century B.
C. and possibly even as late as the midsecond century. The writer(s) of Zechariah dated some
of its prophecies in the late 6th century, but critical analysis has required parts of it to be dated
at least as late as the 4th century. Zechariah 9:13, for example, refers to the "sons" of Greece
as enemies against whom Yahweh would stir up the sons of Zion, but the Greeks were not
considered enemies of Zion until after the conquests of Alexander the Great in the 4th century
B. C. So Dr. Price's "very early witnesses" to Jeremiah and his prophecy turn out to be
witnesses who lived and wrote well after the time of Jeremiah.

Volume 1990 - 2002 Issue


Page 1271 of 2049
Skeptical Review Edited by Farrell Till
The Textual Integrity of Jeremiah: Dr. Price devoted two pages to defending the integrity
of the text of Jeremiah, but with only another page of space for me to finish my response to
him, I will make just three more comments on this issue: (1) Much of the "proof" that he cited
on this point involved only the quoting of authors who agree with his position, and as I have
repeatedly noted, no religious position is so absurd that one cannot find "scholars" who agree
with it. (2) Most of my rebuttals of Price's position have been aimed at showing that the
reliability of the book of Jeremiah cannot be established. (3) My citation of passages in
Obadiah that are almost identical in wording to passages in Jeremiah (July/August 1997, p. 3)
is sufficient to cast serious doubts on Price's claim that the book of Jeremiah has been reliably
transmitted through the centuries. I don't recall that Dr. Price even attempted to explain why
such doublets as these would exist in books that he believes were faithfully transmitted for
centuries by trustworthy scribes. The bottom line is that there are too many reasons why
rational people just cannot share Dr. Price's simplistic belief that the book of Jeremiah exists
today essentially as the 6th-century B. C. prophet wrote it; and as I have also pointed out,
even if Dr. Price could unequivocally establish the textual reliability of the book, he would
then have to explain why Jeremiah predicted that the captivity would last for 70 years when
clearly it didn't.

Changing Horses in Midstream: In this matter, Dr. Price has done an about face. He came
on board vigorously proclaiming that the 70-year prophecy was fulfilled with exceptional
accuracy in that Jeremiah predicted a captivity of 70 years and that the captivity lasted for 69
years, close enough to be considered a remarkable "round-number" fulfillment ("Prophecy of
Seventy Years of Servitude to Babylon," TSR, March/April 1997, p. 3). Now in his "last
stand," he has apparently decided to switch horses in midstream. He said that my insistence
that "the subjugation to the Babylonian kings had to be exactly seventy years for the prophecy
to be fulfilled" (p. 5, this issue) is in disagreement with "competent critical scholars like John
Bright who said: `This seems to be here no more than a round number, i. e., a normal life
span'" (ibid,). He proceeded to cite two more "scholars" who claim that the number "was not
to be taken literally, but rather as a round or `perfect' number, perhaps the length of a man's
lifetime." This is a flagrant reversal of his position, as anyone can verify by reading his initial
article in the March/April 1997 issue, which I quoted above. He defended the same position in
his second article, published in the May/ June 1997 issue (pp. 4-5), but now he is arguing that
the 70 years were figurative and are so understood by many scholars. A review of our articles
will show that I was, in fact, the one who first brought up the position that Dr. Price has now
switched to (see subtitle "A Figurative View," TSR, July/August 1997, p. 6). I restated this
view as an "alternative hypothesis" that Price must disprove in order to sustain his case
(July/August 1998, pp. 2-3), and now at the end of the debate, when Dr. Price has apparently
recognized his failure to prove his position, he seems to have decided that he likes the
"figurative" view and has decided to discard his original claim that the prophecy was fulfilled
with a remarkable "round-number" accuracy. Such wishy-washiness as this deserves no
further comment. Dr. Price has failed to make his case.

Good History in the Book of Daniel

Volume 1990 - 2002 Issue


Page 1272 of 2049
Skeptical Review Edited by Farrell Till
Farrell Till

In my previous responses to Everette Hatcher, I have noted that the general consensus of
biblical critics is that the book of Daniel was not written in the 6th century B. C. A major
reason why critics reject this traditional view is that serious errors in 6th-century history are
scattered throughout the book, a fact that is incompatible with the writer's claim that he was a
Hebrew captive in Babylon, who rose to a position of primary political importance in the
Babylonian empire. Many of these mistakes were noted in my last response to Hatcher "Bad
History in the Book of Daniel" (July/August 1998, pp. 6-8, 10), so there is no need to restate
them here.

A major reason why most critics who reject the 6th-century view of authorship date Daniel in
the 2nd century is also related to history. All of the history in Daniel is not bad; some of it is
accurate. Critics noticed that even though much of the 6th-century history in Daniel was
inaccurate, the most likely meanings of "prophetic" language concerning later events were
much more accurate and became even increasingly more accurate as the writer's prophecies
referred to 2nd-century B. C. events. This critical opinion of Daniel has become the
underpinning of the Maccabean view of its authorship. In the final chapter of his book Darius
the Mede and the Four World Empires in the Book of Daniel, H. H. Rowley stated this same
critical opinion of Daniel, which I quoted in an earlier response to Hatcher but is worth a
second consideration before we look at Daniel's "good" history, which is the keystone of the
Maccabean view of its authorship.

The Book of Daniel is not a work of the sixth century B. C. The case against the
traditional date to which the composition of the book has been assigned rests on a
variety of considerations, but the single one we have examined in the first part of our
study would alone be sufficient to establish it. For a sixth-century person, who not
only lived through the events of the period, but took a leading part in them, could not
have made so gross an error as our author made in introducing Darius the Mede
between Belshazzar and Cyrus. Nor could he have supposed that a Median empire
stood between the Babylonian and the Persian....

As certainly can we say that the book of Daniel is a work of the second century B. C.
If the work is loosed from the sixth century by the inaccuracy of its knowledge of that
age, it is anchored in the second century by the accuracy of the knowledge of that age
which appears in its pages.... So long as the work was believed to be written in the
sixth century B. C., the accuracy of its descriptions of the second century but served to
establish the wonderful certainty of prophecy. But when the link with the sixth century
is broken by the proved historical errors in the part of the book that relates to that age,
the whole case is altered. It is impossible to believe that the mind of Daniel was
illumined with accurate knowledge of future times, while, at the same time, thoroughly
befogged as to the events in which he himself had played no mean part, and we can
only find in the limited range of the accurate knowledge the indication of the author's
period (University of Wales Press, 1935, pp. 175-176).

Volume 1990 - 2002 Issue


Page 1273 of 2049
Skeptical Review Edited by Farrell Till
In addition to the mistakes that Rowley noted about Darius the Mede and the Median empire,
I discussed in my earlier responses "Daniel's" mistakes in saying that Jewish captives and
temple vessels were taken to Babylon during the third year of Jehoiakim's reign (1:1) and that
Nebuchadnezzar and Belshazzar were father and son (chapter 5). None of these claims
conform to Babylonian and other biblical records from this period. The writer of Daniel didn't
know the correct 6th-century B. C. history in basic matters like these, but his prophetic
statements showed an accurate knowledge of events that happened centuries later. Biblicists,
of course, will argue that Daniel's knowledge of events after the 6th century B. C. was the
result of divine inspiration, but it's hard to believe, as Rowley noted above, that Daniel's God
guided him into accurately reporting events that would happen in the distant future but
apparently had no interest in guiding him to report accurately the history of his own era. That
pill is a bit too large to swallow.

In chapter 8, for example, Daniel saw a "vision" of a male goat that came from the west,
"coming across the face of the whole earth without touching the ground" (v:5). This goat had
a great horn between its eyes, which it used to destroy a ram with two horns that had been
described in Daniel's vision just before the appearance of the goat. After trampling into the
ground the ram with two horns, the male goat "grew exceedingly great" (vs:7-8), but at the
"height of its power, the great horn was broken, and in its place there came up four prominent
horns toward the four winds of heaven." Admittedly, the language in this vision is typically
figurative, but scholars agree that it is an accurate description of Alexander the Great's
conquest of the territory that once belonged to the kings of Media and Persia and of the
breakup of Alexander's Grecian empire when he died at the height of his power. Upon
Alexander's death, his empire was divided into four smaller kingdoms by his generals, who
came to be known as the "Diadochi" (successors), so these would have been the four
prominent horns that came up when the great horn was broken. Macedonia and Greece were
allotted to Cassander, Pergamum and Asia Minor to Lysimachus, Syria and Babylon to
Antigonus, and Egypt and Palestine to Ptolemy.

That this was the probable meaning that the writer of Daniel intended in this vision is
supported by verses 20 and 21, where Daniel identified the horns on the ram as the "kings of
Media and Persia" and the male goat as the "king of Greece." The great horn between the eyes
of the goat was identified as "the first king" of Greece (v:21), and the four horns that arose
when the great horn was broken were identified as "four kingdoms [that] shall arise from his
nation, but not with his power" (v:22). In chapter 11, Daniel again referred to the breakup of
Alexander's kingdom. After "predicting" that "three more kings" would arise in Persia, he said
that the last one (who would be the fourth) would "stir up all against the kingdom of Greece"
(v:2) and that this would cause a "warrior king" to arise, "who shall rule with great dominion
and take action as he pleases" (v:3), but, once again, Daniel noted that "while still rising in
power, the dominion of this "warrior king" would be "broken and divided toward the four
winds of heaven, but not to his posterity," because the kingdom would be "uprooted and go to
others besides these" (v:4). The historical facts about Alexander's conquests and the
subsequent breakup of his empire fit the symbols in these "visions" too perfectly to suppose
that they could have been referring to anything else. Upon the death of Alexander, three of his
relatives wanted to take control of his kingdom (his sons Alexander and Herakles, and his
half-brother Philip Arrhidaeus), but none of them had the influence and power to do so;
hence, when Alexander died, his kingdom was "uprooted" and did not go to his "posterity."

Volume 1990 - 2002 Issue


Page 1274 of 2049
Skeptical Review Edited by Farrell Till
Hatcher expects us to believe that someone who didn't even know who ruled in the
Babylonian and Persian empires of his time could nevertheless look through time and make
an amazingly accurate prediction about who would rule over these same territories two
centuries later and what the ultimate fate of this future empire would be. That is also a pill too
large to swallow. An easier pill to take would be the more likely premise that the writer of
Daniel knew more about the Grecian empire than the Babylonian and Persian, because he
lived much closer to the time of the former.

Daniel's accuracy in his visions continued through chapter 11 as he "predicted" events that
would occur after the breakup of Alexander's empire. Space will not permit me to discuss the
interpretations of these predictions in detail, so I will show the "prophecies" in italicized print
and then give brief, undocumented "interpretations" whose facts can be verified in general
reference works on the history of these successor kingdoms.

Then the king of the south shall grow strong, but one of his officers shall grow stronger than
he and shall rule a realm greater than his own realm (11:5). The "king of the south" was
undoubtedly Ptolemy I Soter, who founded the Ptolemaic dynasty in Egypt after the breakup
of Alexander's empire. In 312 B. C., Seleucus I Nicator (a Ptolemaic general) defeated
Antigonus in a battle at Gaza. As noted above, Antigonus had received Syria and Babylon,
but after his defeat at Gaza, Seleucus ruled these territories and, with the exception of Egypt,
eventually expanded his kingdom into almost all of the regions Alexander had once
controlled. Thus, a former general of the "king of the south" grew stronger than Ptolemy and
eventually ruled over a realm greater than Ptolemy's.

After some years they shall make an alliance, and the daughter of the king of the south shall
come to the king of the north to ratify the agreement. But she shall not retain her power, and
his offspring shall not endure. She shall be given up, she and her attendants and her child and
the one who supported her (11:6). Seventy years later, when Antiochus II Theos ruled as the
"king of the north," a peace agreement was reached between the Seleucid (northern) and
Ptolemaic (southern) dynasties. Ptolemy II Philadelphus tried to seal the treaty by giving his
daughter Berenice in marriage to Antiochus II, who then divorced his wife Laodice and
disinherited their two sons by declaring that any son that Berenice had would succeed him to
the throne. Although Berenice did have a son, when Ptolemy Philadelphus died, Antiochus
"gave her up" and took back Laodice, who acted swiftly to have Antiochus, Berenice, and
their son all assassinated, along with Berenice's Egyptian servants and attendants. Hence,
Berenice was unable to "retain her power," and "the offspring" of the king of the north did not
"endure."

In those times a branch from her roots shall rise up in his place. He shall come against the
army and enter the fortress of the king of the north, and he shall take action against them and
prevail. Even their gods, with their idols and with their precious vessels of silver and gold, he
shall carry off to Egypt as spoils of war. For some years he shall refrain from attacking the
king of the north; then the latter shall invade the realm of the king of the south, but will return
to his own land (11:7-9). When Ptolemy Philadelphus died, Berenice's brother Ptolemy III
Euergetes succeeded him. In retaliation for his sister's murder, he attacked Syria, defeated
Laodice's son Seleucus II, who was now the "king of the north," and captured Seleucia (the
port city of Antioch), and so a "branch from Berenice's roots" had "entered the fortress of the

Volume 1990 - 2002 Issue


Page 1275 of 2049
Skeptical Review Edited by Farrell Till
king of the north." He "prevailed" by taking control of all of Syria, but unrest in Egypt
required Ptolemy Euergetes to withdraw from Syrian. In so doing, he took back gold, silver,
and images in precious metals that had been brought up from Egypt prior to the Ptolemaic
loss of Syria to Seleucid control. After the success of this excursion into Syria, Ptolemy III
"refrained from attacking the king of the north," but in 242 B. C., the "king of the north,"
Seleucus II, "invaded the realm of the king of the south," but his attack on Egypt was costly in
defeats on both land and sea, and so he withdrew and "returned to his own land."

Antiochus the Great: His sons shall wage war and assemble a multitude of great forces,
which shall advance like a flood and pass through, and again shall carry the war as far as his
fortress (v:10). Seleucus II Callinicus was succeeded by his son Seleucus III Ceraunus, who
"waged war against Egypt, but was assassinated in 223 B. C., at which time he was succeeded
by Antiochus the Great. Antiochus pursued the war against the "king of the south" with more
vigor than his predecessor. In 218 B. C., he "assembled a multitude of great forces" that
advanced easily through Palestine and "carried the war" into Gaza.

Moved with rage, the king of the south shall go out and do battle against the king of the north,
who shall muster a great multitude, which shall, however, be defeated by his enemy (11:11).
The king of the south, now Ptolemy IV Philopator, counterattacked Seleucid forces, and in a
battle at Raphia, near Gaza, smashed the "great multitude" that had been mustered by
Antiochus. "Defeated by his enemy," Antiochus had to retreat.

When the multitude has been carried off, his heart shall be exalted, and he shall overthrow
tens of thousands, but he shall not prevail. For the king of the north shall again raise a
multitude, larger than the former, and after some years he shall advance with a great army
and abundant supplies" (11:12-13). In a serious military blunder, Ptolemy IV, whose army
had slaughtered hundreds and routed "thousands" of Syrian troops, stopped his pursuit of the
retreating army of Antiochus and returned to Egypt. This mistake allowed Antiochus to
rebuild his army and attack Egypt again, so even though Ptolemy Philopator had scored a
great victory over the Seleucid forces, in a real sense he did not "prevail." A 13-year period of
"peace" between the Ptolemies and Seleucids followed, during which time Antiochus
solidified his military strength and turned his attention to the east to annex territories that had
once been in Alexander's empire. When Ptolemy Philopator died in 203 B. C., he was
succeeded by his son Ptolemy V Epiphanes, who was only five years old. Antiochus took
advantage of this situation to launch another attack on Egypt.

In those times many shall rise against the king of the south. The lawless among your own
people shall lift themselves up in order to fulfill the vision, but they shall fail (11:14). This
period was characterized by domestic unrest in Egypt as well as an alliance between
Antiochus and Philip V of Macedonia to conquer the Ptolemaic empire and divide it between
them. Jews in Palestine (Daniel's "own people"), apparently hoping to gain freedom from
Ptolemaic rule, also sided with Antiochus against Egypt, but the combined efforts failed. The
Ptolemaic dynasty survived, and Palestine remained under Egyptian domination.

Then the king of the north shall come and throw up siegeworks, and take a well-fortified city.
And the forces of the south shall not stand, not even his picked troops, for there shall be no
strength to resist (11:15). In another battle, Antiochus defeated the Ptolemaic general Scopas,

Volume 1990 - 2002 Issue


Page 1276 of 2049
Skeptical Review Edited by Farrell Till
who retreated to Sidon, a strongly fortified city at the time. Antiochus laid siege to Sidon and
took it in 198 B. C., so the "forces of the south didn't stand." Although Ptolemaic generals had
tried to lead Egyptian troops to the rescue of Scopas, these "picked troops" had "no strength to
resist" and could not penetrate the Seleucid siege of Sidon.

But he who comes against him shall take the actions he pleases, and no one shall withstand
him. He shall take a position in the beautiful land, and all of it shall be in his power. He shall
set his mind to come with the strength of his whole kingdom, and he shall bring terms of
peace and perform them. In order to destroy the kingdom, he shall give him a woman in
marriage; but it shall not succeed or be to his advantage (11:16-17). Undoubtedly, the
"beautiful land" was Palestine, which after Antiochus's victory over the Egyptians at Sidon
fell under Seleucid control, and no one could withstand Antiochus. Although the Ptolemaic
forces had been routed, Antiochus, who undoubtedly wanted to go against Egypt "with the
strength of his whole kingdom," was restrained by promises from Rome, a new political force
to contend with, that it would assist the Ptolemies if Antiochus invaded. Antiochus then
adopted a new strategy by giving his daughter Cleopatra in marriage to the 14-year-old
Ptolemy V Epiphanes, but Antiochus's plan to take control of the Egyptian throne through the
peaceful means of Cleopatra's influence did not "succeed or be to his advantage," because she
became loyal to her husband.

Afterward he shall turn to the coastlands, and shall capture many. But a commander shall put
an end to his insolence; indeed, he shall turn his insolence back upon him (11:18). After the
marriage plan failed, Antiochus tried to conquer the coastal areas of Asia Minor. He met with
initial success (captured many), but when he advanced toward Greece, "a commander," the
Roman general Scipio, "put an end to his insolence" in a defeat at Magnesia in 190 B. C.
Thus, the insolence of Antiochus was "turned back upon him."

Then he shall turn back toward the fortresses of his own land, but he shall stumble and fall,
and shall not be found (11:19). In his retreat to "the fortresses of his own land," Antiochus
was killed by an enraged mob while he was pillaging a temple in Elymais. He "stumbled and
fell and was not found."

Then shall arise in his place one who shall send an official for the glory of the kingdom; but
within a few days he shall be broken, though not in anger or in battle (11:20). Seleucus IV
Philopator succeeded Antiochus. Financially strapped because of his father's war debts,
Seleucus sent his finance minister Heliodorus to seize funds from the temple treasury in
Jerusalem. Not long afterwards, he died from suspected poisoning by Heliodorus, who then
tried to take the throne. So Seleucus died "not in anger" or violence or "in battle."

The Reign of the "Contemptible" Antiochus Epiphanes: This verse-by-verse explication


of Daniel's vision in chapter 11 shows that the writer, if indeed a 6th-century B. C.
Babylonian official, had "prophesied" events from the 4th to the 2nd century B. C. with
uncanny accuracy, even though he seemed not to know the history of his own era. This is why
modern biblical critics think it is more likely that in chapter 11, the writer, rather than looking
forward into time, was actually looking back on events that had happened far more recently
than the Babylonian captivity, and so that is why he performed much better in this chapter

Volume 1990 - 2002 Issue


Page 1277 of 2049
Skeptical Review Edited by Farrell Till
than in those that reported the activities of "Daniel" in 6th-century Babylon. The uncanny
accuracy of Daniel's 2nd-century B. C. prophecies, however, was about to change.

In his place shall arise a contemptible person on whom royal majesty had not been conferred;
he shall come in without warning and obtain the kingdom through intrigue (11:21). The death
of Seleucus IV brought the "contemptible" (v:21) Antiochus Epiphanes to power. Demetrius,
the eldest son of Seleucus, was heir to the throne, but Antiochus Epiphanes conspired to have
him sent to Rome, then murdered Heliodorus and an infant son of Seleucus, and took control
of the kingdom. Thus as verse 20 states, the title of "royal majesty" was not conferred on
Antiochus, who "without warning obtained the kingdom through intrigue."

Space will not allow a verse-by-verse analysis of Daniel's "prophecies" about Antiochus
Epiphanes, some of which are discussed on the front page of this issue, but such an analysis
would show the same uncanny accuracy. The prophecies "foretold" Antiochus's military
successes and his deposing of the High Priest Onias in 175 B. C. They "foretold" the attempts
of Antiochus to destroy the Jewish religion by banning dietary laws and desecrating the
temple in Jerusalem with pagan idols and sacrifices. Then, suddenly, the uncanny accuracy
stopped.

At the time of the end the king of the south shall attack him. But the king of the north shall
rush upon him like a whirlwind, with chariots and horsemen, and with many ships. He shall
advance against countries and pass through like a flood. He shall come into the beautiful
land, and tens of thousands shall fall victim, but Edom and Moab and the main part of the
Ammonites shall escape from his power. He shall stretch out his hand against the countries,
and the land of Egypt shall not escape. He shall become ruler of the treasures of gold and of
silver, and all the riches of Egypt; and the Libyans and the Ethiopians shall follow in his
train. But reports from the east and the north shall alarm him, and he shall go out with great
fury to bring ruin and complete destruction to many. He shall pitch his palatial tents between
the sea and the beautiful holy mountain. Yet he shall come to his end, with no one to help him
(11:44-45). None of this happened, and the probable explanation for the sudden prophecy
failures after a long string of uncanny successes is that the writer, who lived during the time
of Antiochus Epiphanes, was looking back on past events until he reached the time when he
was writing the book. In order to continue his "prophecies," he actually had to predict, and his
predictions proved incorrect. On the basis of past conflicts between the Seleucids and
Ptolemaic rulers that the writer was familiar with, he chose to predict that Antiochus, riding
high on a string of successes, would invade Egypt again, conquer it, and revel in its riches. It
was an intelligent guess, but it didn't happen.

The "prophecy" of the end of Antiochus was rather open-ended. It merely said that Antiochus
would "pitch his palatial tents between the sea and the beautiful holy mountain" (between the
Mediterranean Sea and Zion in Jerusalem) and "would come to an end with no one to help
him." Actually, Antiochus Epiphanes died in Persia (far removed from "palatial tents between
the sea and the beautiful holy mountain) of an illness that 1 Maccabees 6:1-6 attributed to
mental distress over news he had received that his armies in Judah had been put to flight.
Records of the date of his death vary from September 164 to December 164 B. C. Since the
writer of Daniel seemed to know everything about the history of the Seleucid kingdom except
the final days and death of its most despised tyrant, this is reasonable evidence that the book

Volume 1990 - 2002 Issue


Page 1278 of 2049
Skeptical Review Edited by Farrell Till
of Daniel was compiled sometime between the last accurate "prophecies," .i. e., the temple
desecrations and Jewish uprisings in 167-165, and the death of Antiochus Epiphanes in 164 B.
C. If Hatcher rejects this critical opinion, he must give a reasonable explanation not just for
the 6th-century B. C. historical errors in the book but also for the abrupt end to Daniel's
remarkable prophetic accuracies in chapter 11. Maybe Dr. Price would like to help him do
that.

From the Mailbag

Not What It's Cracked Up to Be...

I would like to express my deep appreciation to you for sending inmates The Skeptical Review
free of charge. This means a lot to me as I am a Texas inmate and have earned no kind of
wage since being here for the last five and three-fourth years. In the past, I had many
unanswered questions and lots of confusion from having religion forced down my throat as a
child. Yet when I had done my duty getting stuck reading the Bible, I've found nothing but
what seemed to me to be discrepancies and contradictions starting in the book of Genesis and
going all the way through it. Now thanks to someone leaving their copy of The skeptical
Review and my curiosity and love of literature, I had an opportunity to pick it up and see what
it was about. I've finally found someone else who has the same beliefs as I have and who can
actually see that the Bible is not all that it's cracked up to be! Not only this but you also have
the open-mindedness and gumption to speak out on your findings and own up to and publish
it too!

I am very grateful for you sharing your viewpoints, thoughts, and ideas on this subject. It's
been a relief to me to learn of others who also have trouble swallowing everything that was
written in the Bible. It's wonderful knowing I'm not alone. Please publish this letter and my
address in case anyone would like to write to me in the future. Thanks again for caring
enough about inmates to allow us to have free subscriptions.

(Luella Leever # 646663, M. V. Unit, 2305 Ransom Road, Gatesville, TX 76528)

EDITOR'S NOTE: I get more letters from prison inmates than I have time to answer. I have
published a few of them in the past, and in this issue, I am featuring several of them, which
will appear immediately below.

Prison, a Struggle for Reason...

I just thought I'd drop in with some feedback. I received my first issue of The Skeptical
Review, May/June 1998. This publication is great and definitely educational. For somebody
like me, incarcerated in the Texas prison system, it offers a breath of fresh air from the stifling

Volume 1990 - 2002 Issue


Page 1279 of 2049
Skeptical Review Edited by Farrell Till
propaganda of the prison ministries. I found your debate with Dr. Price to be hard-hitting and
direct. I see you didn't waste any time trying to set up smoke screens but instead got straight
to the point. I enjoy the mailbag section as well as the thoroughly written articles.

One viewpoint I'd like to share with you concerns the incarcerated freethinker. A journey
through the penal system can develop into a full-blown struggle for reason. The first stage of
this struggle begins in the county jail and continues on in prison, where these facilities are, in
general, pro-clergy. There always seem to be several types of religious services going on. For
the freethinker, there are no alternatives to offer sound advice and positive counseling in an
atmosphere of negativity. Without any type of nonreligious support services, perhaps as an
equal balance to religious services, many nonreligious people sit idle. The common
misconception feeding off this situation and perpetuated by the clergy is that freethinkers are
incorrigible evil-doers.

The best thing we can do is open our minds to reason and logic and concentrate on living a
positive life. If we want to improve our circumstances in life, then we need to take the
initiative to change things. The Skeptical Review not only provides valuable insight but is also
a tool with which to enlighten those who are confused. Thank you for the subscription. I
really appreciate it.

(Michael Garza #641928, Route 1, Box 150, Tennessee Colony, TX 75884)

Two Steps from Fanatics...

I am an inmate in a California state prison. I wrote to you a couple of days ago about my
family being two steps back from being fanatics. Do you remember? Well, I apologize for
sending you such a messy, sloppy letter. I do have much more respect for you and your work
to send such a hideous letter. I did fail to mention that I would really appreciate your putting
my brother on your mailing list. He is "saved" and lives (or tries to) as such.

I did mention that individuals come into this place and preach "the word" to us. They insist
that it is the word of God. I insist that the Bible is, if anything, only "a message." What I read
in your publication and the Freethought Perspective I try to ask my brother about. I know
well enough that he is a "holy roller" and nothing will open up his mind to be more rational in
his belief. I can only write so much in a letter, and I am limited to my resources to fully instill
a proper foundation to explain my points regarding the "far-fetched hoopla" this "God" lays
down by way of the Bible. If my brother could just read your publication, I more or less
already know that it will only ignite his fire. He will respond by expressing his views to you,
I'm sure. He will defend "J. C." to the fullest. You might ask then why waste a copy or an
issue if nothing will deter his close-mindedness? Well, he will take this issue to his
congregation, and when he does, no one there will be able to handle what you write. Thus,
they will have to research and learn more about their "invisible deity" to make some kind of
logical sense.

I am an agnostic, searching for logical sense rather than building my life around "faith." I
want proof; moreover, I need proof in order to wholeheartedly believe in something. I'm
sending you a letter from my brother that I received today, and hope you will understand what

Volume 1990 - 2002 Issue


Page 1280 of 2049
Skeptical Review Edited by Farrell Till
I'm trying to write in this letter. Please consider my brother to be placed on your mailing list.
His name and address is (deleted).

(Cesar Rodriguez #J78624, PBSP-SHU/C-6 C212, P. O. Box 7500, Crescent City, CA 95532)

EDITOR'S NOTE: The letter from Rodriguez's brother was basically a collage of scripture
quotations strung together in typical Christian fashion, as if the writer thinks that whatever is
in the Bible will be believed by those who read it. I have added Mr. Rodriguez's brother to the
mailing list, but I don't have much hope that TSR will bring any substantial change in the
brother's thinking. My experience has been that the majority of those who request the free
subscription themselves will renew after the first year, but most of those who receive it at
someone else's request won't. Quite often, they will either write or call and ask that their
names be removed from the mailing list. Christians think nothing at all about pushing tracts
and other religious literature onto others, but they don't like it when those with opposing
views return the favor.

Monopoly on Tracts...

I am a prisoner in Joliet Correctional Center, who has been thankfully receiving The Skeptical
Review for a few years. I want to "get the message out," so to speak, to the prisoners. You
wouldn't believe how much time and money religions spend to indoctrinate prisoners with the
fundamentalist drivel. Everywhere I turn I see a religious tract or a chaplain coming to try to
convert me. I am not going to take this attack on reality anymore. I am planning to hand out
anti-religious or anti-fundamentalist tracts. The religious groups have so far had a monopoly
on tracts in this prison, but I foresee a time when where there is a religious tract, an anti-
religious tract will sit right next to it. I was hoping you might have some tracts or information
you wouldn't mind me copying. This would be a great help to me and, I think, to many others.
I'm planning to do this campaign at much risk to myself, but the "real" truth must be heard at
all costs. I could pass out copies (with your permission) of The Skeptical Review, but it might
put a strain on you if you get a lot of requests for subscriptions. I believe it would do a lot of
good, but I'm sure there are ways of doing it without causing you any undue strain. I am
working with some material right now, but I desperately need more/different materials. I'm
not going to make much of an impact with what I've got to work with. I'm not sure what to do
here. If at all possible, please write me and let me know. Any help would be great.

(Thomas Olds #B00838, P. O. Box 515, Joliet, IL 60432)

EDITOR'S NOTE: Answering correspondence is a major problem for me, because I receive
far too many letters to give each one my individual attention. Publishing this letter will be my
way of responding to it.

Skepticism, Inc., has published and distributed written debates, but we have never distributed
tracts. We have back copies of each issue of TSR except one, but to use these in the
distribution program suggested by Mr. Olds would be cost prohibitive. Whereas tracts cost
pennies to print, a publication like TSR costs about $0.50 per copy just to publish. Our extra
back copies number from just a few copies to a couple of hundred per issue, and if we
distributed them as tracts, they would not last long. Then we would have none to meet the

Volume 1990 - 2002 Issue


Page 1281 of 2049
Skeptical Review Edited by Farrell Till
demand from new subscribers who want to have back issues. Also, if our back copies are
distributed as tracts, they will probably be treated as tracts; that is, many of them will just be
thrown away. I wish we could help, but I see no feasible way to do it.

We, of course, have no objections if the articles in TSR are copied locally. We have always
granted interested parties the right to do this provided that the name and address of The
skeptical Review are included on the copies, so that those who want additional information
will know how to contact us.

A Correction...

I hope you will run the following brief letter to fix a problem in my article "How the Snake
Slithered into Eden." I left something out of this article that could lead to confusion. I said
that the Mesopotamian epic the "Enuma Elish" tells how the king-deity cut up the body of the
primal goddess Tiamat to make the universe in six steps parallel to the six stages of Genesis. I
then pointed out that Genesis does not say that Yahweh directly created life on the earth, other
than the first humans, but ordered the earth and the sea to make the various lifeforms.

The significance of this reference was lost because of my oversight. The "Enuma Elish" does
not describe the actual creation of plant and animal life by the Mesopotamian gods, so it does
not parallel the Genesis creation account in that respect. This was my error, but then it
becomes significant that Genesis does not say that Yahweh directly created life either. If the
first chapter of Genesis is derived in some way from the "Enuma Elish," as scholars have
argued, this phrasing could be another hint of how the early Israelites kept to the basic format
of the original as they reshaped it to their needs. They said Yahweh made directly what the
Mesopotamian gods made directly and did not make directly what the gods did not make
directly in the "Enuma Elish."

The epic has a gap that might contain a description of the creation of life, but that's not
certain, and an argument has been made--a good one to my amateur ear--that the missing
passage did not contain such an account. However, Tiamat, the primal mother and sea, created
life, which could be the inspiration for the Genesis claim that Yahweh ordered the female
earth and sea to create life. It's speculation, of course, but that's all we can do in the absence of
solid information about how Genesis was composed.

I also should not have said that the "Enuma Elish" describes six steps of creation. The epic is
too vague to really break down into such a neat division. At least seven parallels, in identical
sequence, have been cited in the details of creation between the epic and Genesis 1.

I hope the preceding comments will make my article clearer and correct my error in saying
that the "Enuma Elish" matched the six stages of Genesis 1 explicitly. The parallelism is not
100 percent precise, yet in some ways is more intriguing.

(William Sierichs, Jr., 316 Apartment Court Drive, Apt. 44, Baton Rouge, LA 70806)

Comments on July/August Issue...

Volume 1990 - 2002 Issue


Page 1282 of 2049
Skeptical Review Edited by Farrell Till
As usual, many of the arguments went into very fine details, which are difficult for me to
remember or follow. I am not sure if I can remember anything that I can tell a believer.

Fulfilled prophecy is one of the usual foundations that Christians claim supports their faith as
if faith needed any help. The most that I can claim is that there are alternative explanations to
fulfilled prophecy. Trying to disprove a claimed fulfilled prophecy is beyond my knowledge
and training. If I try to prove to someone that Jeremiah did not make a fulfilled prophecy, I
will have to cite some historical experts. The Christian will cite his, and the debate might get
nowhere. Of course, showing that the Christian cannot prove fulfilled prophecy is very
important so that the true foundation of religion can be revealed.

I was surprised at the claim of Gleason L. Archer that life is meaningless without god and
without an infallible bible. There are many popular apologetic books that make the same
claim and seem to be very popular with the converted. Those who cannot live without their
god and infallible Bible are difficult to debate with, since they will cling to outrageous
explanations and will often tell people to accept mysteries. Some people (like me) easily see
biblical problems without much help; some people learn from the inerrancy debates, and most
people will need to be offered a good substitute before they can give up their god/infallible-
bible beliefs.

(John Delacruz, 3104 Del Monte Street, San Mateo, CA 94403-3819; e-mail
delacruz@visa.com)

EDITOR'S NOTE: Learning how to reply effectively to inerrantist arguments is like learning
any other subject. Time and repetition are needed to assimilate information, so Mr. Delacruz
should not think it unusual that he can't remember all major points in material he has read.
Additional reading and studying will increase ability to recall.

In earlier issues, I have pointed out that an easy way to respond to biblicists who claim
prophecy fulfillment is to demand reasonable proof that (1) the alleged prophecy statement
meant what the Christian is claiming, and (2) a fulfillment event actually occurred. Since
various interpretations have been given to most prophecy statements and since the only
evidence that biblicists can offer as evidence of fulfillment is the mere word of the Bible itself
that such and such events fulfilled such and such prophecies, these two demands will usually
put prophecy-fulfillment claimants into a defensive position from which they can't recover.

Gleason Archer is no different from the average would-be apologists whom skeptics
encounter so often. The fact is that most of these want-to-be apologists have gotten their
"arguments" and "explanations" from professional apologists like Archer, McDowell, Geisler,
etc. The shallowness and absurdity of their explanations, then, come from parroting what they
have heard the professionals say. Offering such explanations as "the mysteries of God can't
be understood by ordinary men" or "God's ways are higher than our ways" is something else
that amateur apologists have learned from the professionals. I have found that the best way to
show the inadequacies of such explanations is to point out that any religionist (Muslim,
Hindu, Mormon, etc.) could say the same thing to "explain" inconsistencies and discrepancies
in their "holy books." So what good are explanations that can be used to "prove" the truth of
any allegedly inspired book?

Volume 1990 - 2002 Issue


Page 1283 of 2049
Skeptical Review Edited by Farrell Till
A Letter to Gleason Archer...

I've read your letter to Mister Farrell Till in the latest issue (July/August 1998) of The
Skeptical Review. For humanitarian reasons, I felt moved to help you out by making some
suggestions: (1) Get a spelling checker. (2) Take a refresher course in grammar at a local
community college. (3) Abandon the false belief that the Bible does not contain gross errors.
It does. You are defending a falsehood, which cannot but make Christians appear foolish,
ignorant, and even wildly irrational. Inerrancy is bad theology! (4) Attack Mister Till's
scholarship, not him personally. Otherwise, you appear bitter and unable to counter an
argument rationally. (5) Cease attacking positions that Mister Till, nontheists, and Christians
with opposing views do not hold and thus do not defend. When you wail about "nihilism,"
you attack that which Mister Till does not assert, believe, support, or defend. Do you
understand how silly such an attack makes you appear? (6) Realize that right behavior, wrong
behavior, morality, and ethics are all not predicated upon belief in God. When you falsely
attack Mister Till for not having "a yardstick for good and evil," you fail to acknowledge the
fact that such a "yardstick" is independent of God-belief. Good is good because it is good: not
because some god or human being says it is; evil is evil because it is evil: not because some
god or human being says it is. (7) Cease insulting Mister Till and others by attributing to them
attributes they do not possess. When you falsely assert Mister Till has no "yardstick for good
and evil," you falsely assert Mister Till does not know good from evil. Such a person is called
"a sociopath," and is a grave danger to society and himself. Is it your contention that Mister
Till is a sociopath? (8) Agree to debate poor ole deluded Mister Till and show everyone how
wrong he is and how right you are, instead of wailing about how he "ignores" your
"evidence." [Warning: see suggestion number three above.] (9) Cease trying to threaten/scare
nontheists with your demonic God, your Satan, and your Hell. This just makes you appear
foolish. It is like a child making empty threats of the boogie-man to adults. It might be
considered cute and precocious the first dozen times or so that a nontheist hears it, but after a
few thousand times it is merely tedious. (10) Try reading the Bible instead of worshiping it.
(11) Subscribe to and study The Skeptical Review. Mister Till has much he can teach you.

(Rev. David Michael Rice, 723 Calle Casita, San Clemente, CA 92673-2708; e-mail,
shydavid@net999.com; http://page. net999.com/shydavid/index.htm)

Another Letter to Gleason Archer...

Having read with interest your letter published in The Skeptical Review (Volume 9, Number
4), I felt compelled to write you a few lines. First, let me congratulate you on both your
honesty and your courage. You are one of a very few fundamentalist leaders who will
confront the detractors of fundamentalism head on. If the Reverend Jerry Falwell is any
example, the preferred method is to avoid the detractors and hope (as he wrote quite honestly
in a letter to the Right Reverend John Shelby Spong regarding the latter, which Bishop Spong
quoted in the forward to Rescuing the Bible from Fundamentalism) to avoid bringing
articulate detractors to the attention of the public.

However, let me also say that I feel your defense of your position is fatally flawed, for a
number of reasons. For example, you draw specious conclusions that are warranted by your
belief and your suppositions but not by evidence. Take your basic contention regarding the

Volume 1990 - 2002 Issue


Page 1284 of 2049
Skeptical Review Edited by Farrell Till
centrality of "the Holy Scriptures" to moral philosophy. You make clear that you believe that
those who reject the authority of scripture must, prima facie, embrace nihilism and the belief
that all human activity is essentially meaningless. In my view, nothing could be further from
the truth.

I am an active participant in the Episcopal Church in America. I attend Eucharist regularly,


seek always to care for my fellow man, and am active almost continually in coalitions to help
bring economic and social justice to all people. When my life experience was assessed for
credit, while I was seeking my second bachelor's degree, I was awarded 10 credits for my
experiences in grassroots political activism. Whether you and I share the same moral beliefs
or not, we are both obviously moral creatures.

On the other hand, I certainly do not accept the idea that the Bible is inerrant or even
particularly inspired. It is sacred, because it is the accumulated story of people seeking God
within not one but two separate wisdom traditions. Clearly, however, not one of the three
pillars traditionally cited in support of an infallible text are applicable to the Bible. The Bible
is not perfectly historically accurate, it is not internally tangent, and it is not prophetically
perfect. Even if, for the sake of argument, one accepts that we cannot be certain enough about
history to prove that the Biblical record and the historical one are identical (a contention I
would argue is incorrect based on some of the Roman records, not to mention some of the
more outrageous scriptures that have thus far survived the censors; talk about the missing
link--know anyone who has seen the skeleton of a satyr?), the problems of internal tangency
and prophetic perfection remain, and are more than enough to prove the document a human
one.

Despite the shapes that one may contort one's intellect into, no one can reconcile large
portions of the text, and no one should know that better than you. As for prophecies, what
prophecies could be more important than those given by Christ himself? Yet, unless we have
2000-year-old people wandering around somewhere, at least one of his prophecies was
blatantly wrong--at least as given in the gospels--wasn't it?

The text was not, is not, and should not be the faith. We know from church history how Nicea
shaped the core doctrines of the church, doctrines that precious few who call themselves
Christian have ever departed from by more than a stone's throw. We also know how Nicea
and other church councils shaped the formation of canon. The received texts are not the
received texts because they were specially inspired but because they fit the theological and
ideological needs of people who had already determined what the doctrines of the church
were. They were, at best, convenient advertising material that fit the received orthodoxy of the
time.

Today's fundamentalists are, in all too many cases, not much more than biblio-idolaters. They
are people who place the Bible in the niche that idols occupied for many of the faiths of
ancient peoples, and worship its dictates, rather than examining the evidence, the
circumstances, and the conditions in order to better make ethical and moral decisions that
further the teachings we see in the life of the radical (and in his time, he was a radical) Jesus
Christ. They instead follow the teachings of His chroniclers, who wrote long after His
assumption.

Volume 1990 - 2002 Issue


Page 1285 of 2049
Skeptical Review Edited by Farrell Till
Please understand that I do not deny the importance of ideologies, whether they are ideologies
that I agree with or not. This century has proven how powerful ideologies, whether religious
or secular, can be. On the other hand, I deny that it is either necessary or desirable to follow a
fallible text, albeit a fallible sacred text, which is, at best, a blackened window--a window
through which we each interpret the shapes we see in a matter most convenient to us. Instead,
I suggest that as emerging understanding of the world and universe around us opens new
vistas of knowledge and new frontiers of science, we aim at helping the Christian churches
discard damaging anachronisms, face (as courageously as you have faced your detractors) a
future that will never reinforce first-century presumptions, and build a just and all-embracing
community, which will reject centuries-old prejudices and take pro-active stances on the
issues of the next century. Issues that will surely confront us, such as the legal standing of
clones, citizenship rights for our own colonists on other worlds, and quite possibly our
interactions with other intelligent species, I would submit, are not issues that the churches
today are universally ready to confront.

(Reynolds Jones, 443 Hicks Street, Apt 2G, Brooklyn, NY 11201; e-mail,
paragon2@bigfoot.com)

EDITOR'S NOTE: Dr. Archer has refused for a period of about 7 years to defend his
inerrancy belief in a public forum that would give seminary students where he teaches the
opportunity to hear the other side of the inerrancy controversy. For this reason, I can't agree
that Archer has shown "honesty and courage." I personally believe that he has done the
opposite by continuing to proclaim biblical inerrancy while knowing it wouldn't be wise to
allow his belief to be examined in public discussions by an informed opposition.

An African Perspective...

A quick comment about some of the articles I read in The Skeptical Review, May/June 1998.
A common twist that Christians use in their arguments that is simple to see, and easy to
squash and get them (if it's possible at all) to review their arguments, is the logic that says if
we do not understand something (a blind person seeing again, or how the universe started,
etc., etc.), then there must be a magical/godly explanation for it. Of course, this is quite
illogical and childish. No argument can ever follow that "logic." The only conclusion that can
be drawn from not understanding something, anything, is that we do not understand it. If
people had rather believe in the magic of miracles, rather than [god-given] science, then that
is their problem. I think that any reasonable investigation into any claim of miracles would
quickly show it to be either a farce and lie, or at best something that cannot be proven one
way or the other, or that all witnesses to it are coincidentally Christians.

I recently had a "chat" with a Christian who sounded quite convincing about a variety of
miracles that he had witnessed, and about the real existence of the spiritual world. At the
prompting of a friend, who suggested that I go and experience the miracles myself, the
Christian, upon some careful examination suddenly had no personal experience and could not
point me to any of the many miracle workers he claimed, so his stories fell apart quickly. It
was a real eye-opener to see just how he actually misrepresented the truth and lied about
everything. But that is religion for you. He has not changed, of course, has not phoned me to
follow up, and no doubt continues to lie to others.

Volume 1990 - 2002 Issue


Page 1286 of 2049
Skeptical Review Edited by Farrell Till
Another comment regarding Mr. Price's comment in column three, page four, about your
agnosticism vs atheism: often one can use a Christian's argument against him. Mr. Price has
no proof of any god, so why is he not an agnostic?

Thirdly, there is nothing wrong with being an atheist. There is something wrong (debatably)
with living without values, morals, principles which are based on reason and truth, rather than
dogma. There is no proof of any sort of any god (there are hundreds available on the market
and thousands of different interpretations), let alone Mr. Price's own interpretation of his
particular god. Always keep the debate open and refer only to what we know, i.e. belief
systems such as Christianity, rather than to the claimed god itself. I am a happy atheist. I try
to get involved in activities that will, in my opinion, add value to the world at large in my own
small way. I try to love my kids and teach them to add value as well. When I leave this world,
I hope it will be very slightly better than when I found it. With atheism, one does not need to
duck and dive, lie and twist the truth, change interpretations, etc. One can lead a pure and
clean life, focusing on the beauty of reality, no matter how complex and confusing it
sometimes seems.

Regarding the Faulkenberry article, note that you are both arguing from a modern moral
position. Up until very recently, women had few rights relative to men. Living in a country of
diverse ethnic cultures, I see daily the difference in attitude to a similar situation, where one
culture sees something as horrific and unacceptable, and another sees it as normal and totally
acceptable. (Normally it is only the losers that look for further and alternative relief and
justification.) I am certainly no historian or theologian, but I would assume that, if the actual
story is really accurate, in those days, (as still happens in many third-world countries), a
woman that the king sends for, has no right to refuse but every obligation and honour to obey
the king's every wish and command. There is definitely nothing unusual and abnormal about
this, even today. Of course, in modern first-world countries, morals have changed, and we
accept a different set of cultural standards. Indeed, we try to force them onto other cultures as
if they are necessarily superior. Something for you to question, Mrs. Faulkenberry.

Farrell, I compliment you on your work and efforts, and, most important, on your intentions.

(Peter Brossy, P. O. Box 41856, Craighall 2024, South Africa; e-mail,


peterbrossy@yahoo.com)

Primary Colors...

"Primary Colors of the Bible," your article concerning textual criticism in the July/August
issue of The Skeptical Review, was very interesting. In the article, you point out that biblical
critics do not think the apostle Paul wrote all the epistles attributed to him.

I can't locate the material to document the point (it is in one of the hundreds of books and
magazines in my library), but several years ago a textual critic used computer analysis to
indicate Paul did not write all of the 13 epistles that bear his name. In rebuttal, a Christian
textual critic used the same computer analysis of vocabulary, style, etc. to indicate Ian
Fleming did not write all of the 13 James Bond novels that bear his name.

Volume 1990 - 2002 Issue


Page 1287 of 2049
Skeptical Review Edited by Farrell Till
What does all of this prove? I think your statement in the article explains it best. You said,"
Textual critics undoubtedly reach some wrong conclusions...." Even if it could be established
that Paul did write (or compose, since he did use an amanuensis, see Romans 16:22) all 13 of
his epistles, this would not prove inerrancy. On the other hand, if Paul did not compose all
epistles attributed to him, your position of biblical errancy is firmly established.

(Don Robertson, 644 Walnut Street, Rock Hill, SC 29730)

EDITOR'S NOTE: I can't comment on the project that "proved" Ian Fleming didn't write all
13 of the James Bond novels, because I am not familiar with the data. Nevertheless, the
literary detective work of professor Don Foster certainly showed the validity of textual
criticisms, because he established the identity of an anonymous writer by looking for
distinguishing characteristics of the unknown author in the works of those who had published
under their real names. When he found the same literary characteristics, he announced who
"Anonymous" was, and he was right. I lay no claim to critical expertise comparable to
professor Foster's, but I know from my own career as a writing instructor that plagiarism was
easy to recognize in student writing. Stylistic changes in a student essay made plagiarism
stand out like a sore thumb. I will have to see more data than Mr. Robertson has given before
I will agree that literary criticism is unable to recognize forgery and redaction.

Subscription Cancellation...

Please cancel my subscription. I did not subscribe in the first place. I suspect some fool
(Psalm 14:1) athiest [sic] did.

(Fred L. Battles, 165 Cold Stream Trail, Felton, PA 17322-99245)

EDITOR'S NOTE: I suspect Mr. Battles is correct, but I also suspect that the "fool athiest
[sic]" who sent the subscription request knows how to spell the word "atheist." Mr. Battles
doesn't want anyone sending non-Christian literature to him, but he included with his note a
tract entitled "Are You Ready?" It was typically shallow and trite, and, of course, it relied on
the threat of damnation as its means of persuasion. I was able to read the first two
paragraphs before I felt the desire to vomit and threw it away. Does Mr. Battles think that I
am unaware of the biblical threat of eternal damnation? This is a device that will turn the
rational away from Christianity rather than draw them to it, but the simple-minded just aren't
able to realize this.

Volume 1990 - 2002 Issue


Page 1288 of 2049
Skeptical Review Edited by Farrell Till

The Skeptical Review


Volume Nine, Issue Six
November/December 1998
Farrell Till, editor

• "Silliness in the Book of Daniel"


Editor Farrell Till discusses various absurdities in the book of Daniel that cast
suspicion on the tradition that it is a divinely inspired work.
• "Lions 1, Daniel 0"
William Sierichs, Jr., discusses various mistakes and errors in the book of Daniel that
discredit the traditional claim of divine inspiration.
• "A Statement of Editorial Policy"
Editor Farrell Till announces that he will no longer accept articles from either
inerrantists or errantists that rely primarily on appeals to scholarly authority rather
than logical argumentation.
• "Clearing the Confusion over Babel"
Stephen Van Eck points out features in the tower of Babel story that identify it as just
an ancient myth.
• "Kosher Baloney"
Medical doctor Tim Gorski debunks the fundamentalist claim that the Levitical dietary
laws showed an awareness of scientific facts that were not known at the time.
• "Twenty-Four Hours in Time"
An anonymous writer circulates the urban myth that the long-day of Joshua has been
proven to be historical fact.
• "Fundamentalist Flapdoodle"
Farrell Till debunks the myth that NASA scientists have confirmed the long-day of
Joshua.

Volume 1990 - 2002 Issue


Page 1289 of 2049
Skeptical Review Edited by Farrell Till
• "Twenty-Four Hours in Time: A Rebuttal"
Charles N. Brennecke gives additional information to show that discovery of a
missing day in time is scientifically impossible.
• "A Letter to Gleason Archer"
Dave Matson responds to the letter from Gleason Archer that was published in the
July/August 1998 issue.
• "From the Mailbag"
Readers and Editor Farrell Till comment on a variety of religious issues.

Silliness in the Book of Daniel


Farrell Till

If Bible fundamentalists could prove that Daniel was written during the Babylonian captivity,
as its author claimed, that would do little to improve its credibility, because the date of
authorship is only one of many problems that have led rational people to reject the traditional
notion that the author of this book was "inspired of God." In addition to verifiable historical
mistakes in the book that have been discussed in response to Everette Hatcher's defense of the
traditional view of authorship, there are entirely too many absurdities in Daniel for rational
people to believe that it was a divinely inspired work.

The tale of Shadrach, Meshach, and Abednego in the fiery furnace is one of the silliest yarns
in the book, if not the entire Bible, and only those whose sense of reality has been warped by
religious indoctrination could believe that this is a historically accurate story. According to
this fanciful tale, King Nebuchadnezzar made an image of gold that was 60 cubits tall and six
cubits broad, set it up on the plain of Dura in the province of Babylon, and commanded all his
subjects to worship it (3:1-7). A cubit was about 18 inches in length, so the claim is that this
golden image was about 90 feet tall and nine feet wide. The story does not state the third
dimension of the image, but if its depth were equal to the width, the image would have
contained 270 cubic yards of gold, a quantity that would surely have impoverished the supply
of gold in the royal treasury for no purpose except to make a gigantic golden image that the
king would command all of his subjects to worship.

In all of the royal Babylonian archives that archaeologists have discovered, no reference to
such an image as this or any decree to worship it has been found. Since a penalty of death was
attached to any refusal to worship the image (3:6), it is unlikely that the erection of a
magnificent image like this and the laws attendant to it would have escaped the notice of the
royal archivists, who recorded such mundane matters as food allotments for captive kings and
less impressive religious ceremonies, but even if we assume that religious superstition was so
rampant at the time that a king would squander royal wealth on a project like this, the tale is
still too unlikely to believe.

Volume 1990 - 2002 Issue


Page 1290 of 2049
Skeptical Review Edited by Farrell Till
For one thing, the previous chapter had ended with Daniel's successful interpretation of a
dream that Nebuchadnezzar had had about another great image, and so impressed was the
king that he had said to Daniel, "Of a truth your God is the God of gods and the Lord of
kings" (2:47). Then out of gratitude for Daniel's service, Nebuchadnezzar gave Daniel "many
gifts" and made him ruler "over the whole province of Babylon" and "the chief governor over
all the wise men of Babylon" (v:48). The king also "appointed Shadrach, Meshach, and
Abednego over the affairs of the province of Babylon" (v:49). As in the case of the golden
image and the king's decree for all people to worship it, no record of "many gifts" given to
Daniel or of his and his three friends' promotions to positions of prominence have been found
in Babylonian archives, even though they contained meticulous records of royal distributions
to other Hebrew captives. Despite the absence of official corroboration of these claims,
biblicists expect us to believe that all this happened and that the same king who had been so
thoroughly convinced that Daniel's god was "the god of Gods" then turned around and made a
90-foot golden idol and commanded all of his subjects to worship it under pain of death for
disobeying the decree. His conversion to belief in Daniel's "true" god was short indeed.

These flaws in the basic premise of the story are minor indeed compared to the absurdities
that allegedly happened after the enactment of Nebuchadnezzar's decree. "Certain Chaldeans"
reported to the king that Shadrach, Meshach, and Abednego, whom he had "appointed over
the affairs of the province of Babylon" (v:12), had refused to serve the king's gods or to
worship the golden image that he had set up. Upon hearing this, Nebuchadnezzar had the
accused brought before him and offered them a chance to acquit themselves of blame. If they
would fall down and worship the golden image, the matter would be forgotten, but if they
refused to worship the image, they would be cast immediately into a fiery furnace. The three
loyal Yahwists refused, of course, and Nebuchadnezzar, in a rage of fury (v:19), commanded
that the furnace be heated seven times hotter than normal and that Shadrach, Meshach, and
Abednego be bound and cast into it. The heat was so intense that it killed the king's men who
carried the three to the furnace and threw them into the flames.

As the story unfolded beyond this point, silliness was piled onto silliness. The king rose up in
astonishment and said to his counselors," Did we not cast three men into the midst of the
fire?" Assured by his counselors that this was so, Nebuchadnezzar said, "Lo, I see four men
walking in the midst of the fire, and they have no hurt, and the aspect of the fourth is like a
son of the gods" (v:25). A little background music from the Twilight Zone series would be
appropriate here. Only three men were thrown into the fire, but four could be seen walking
about unharmed in the flames! What else could account for this but the hand of the almighty
Yahweh himself?

Nebuchadnezzar then "came near to the mouth of the burning, fiery furnace" (v:26) and called
to Shadrach, Meshach, and Abednego: "You servants of the Most High God, come forth"
(v:26). Never mind that the heat of the furnace was so intense that it had killed the men who
threw Shadrach, Meshach, and Abednego into the furnace. If the Bible says that
Nebuchadnezzar went to the mouth of this intensely hot furnace, without being hurt, and told
the men strolling about inside to come forth, we are supposed to believe that it happened. It's
in the book!

Volume 1990 - 2002 Issue


Page 1291 of 2049
Skeptical Review Edited by Farrell Till
When the men came out of the furnace, the king's satraps, deputies, governors, and
counselors, gathered around, saw that the fire had "had no power on their bodies" and that not
even "the hair of their heads [was] singed" (v:27). Their clothes had undergone no change and
didn't even have the smell of fire on them (v:27). If this tale were found in any ancient
document other than the Bible, even Christian fundamentalists would be amused at its
silliness, but since it is in the Bible, they believe that it actually happened-- and wonder why
skeptics don't.

But the absurdities continue. Nebuchadnezzar, who had already been convinced by Daniel's
dream interpretation that Yahweh was the "god of Gods and Lord of kings," got a case of true
religion again. After the men had exited the fiery furnace, he said to his official entourage,
"Blessed be the God of Shadrach, Meshach, and Abednego, who has sent his angel, and
delivered his servants that trusted in him" (v:28). His angel? Ah, so that's who the fourth party
in the fiery furnace was! An angel of Yahweh! We should have known. At any rate,
Nebuchadnezzar, slow learner that he was, had once again been convinced that Yahweh was
the only true god, and so he made a decree that "every people, nation, and language, which
speak anything amiss against the God of Shadrach, Meshach, and Abednego, will be cut to
pieces, and their houses will be made a dunghill, because there is no other god that is able to
deliver after this sort" (v:29). He then promoted Shadrach, Meshach, and Abednego in the
province of Babylon (v:30).

What they were promoted to is anyone's guess, for, as previously noted, Nebuchadnezzar had
already appointed them "over the affairs of the province of Babylon" (2:49), but what is even
stranger yet is that all of the hundreds of cuneiform tablets discovered in the Babylonian
archives of this period made no reference to these three Jewish captives who were promoted
to such prominence in the kingdom. Neither has any reference been found to any official
decree that Nebuchadnezzar issued, which, in effect, had declared Yahweh to be the national
god of the Babylonian empire. Biblicists may claim that this is an argument from silence, but
it is certainly a peculiar silence in records that made so many references to the religious
practices of the time.

The next chapter is related in the first person, as if Nebuchadnezzar himself was its author. It
underscores the king's incredible forgetfulness, for after twice having had the majesty and
greatness of the Hebrew god so forcefully demonstrated to him that he declared Yahweh to be
"the god of Gods," he apparently suffered another memory lapse, which brought a stern
warning from Yahweh in a dream. In the dream, Nebuchadnezzar saw a great tree in the midst
of the earth whose height reached to heaven (4:10-11), and as the dream continued, a watcher
(angel) descended from heaven and cut down the tree. Daniel, however, came to the rescue
again to interpret the dream and inform the king that he had gotten too big for his britches,
and so Yahweh was going to drive him from among men to live as a beast in the wild until
"seven times" had passed over him and he again realized what he had already recognized
twice before, *i. e.,* "the Most High rules in the kingdom of men and gives it to whomever he
will" (v:17). Sure enough, the dream came true, just as Daniel had predicted, and after
Nebuchadnezzar had boasted one day of his accomplishments, Yahweh drove him into the
wild to eat grass and live as a beast. At the end of the "seven times," assumed to be seven
years, Nebuchadnezzar lifted up his eyes to heaven (v:34), and his understanding returned to
him as he "blessed the Most High and praised and honored him that lives forever" (v:34). So

Volume 1990 - 2002 Issue


Page 1292 of 2049
Skeptical Review Edited by Farrell Till
once again, Nebuchadnezzar had come to realize the greatness of the Hebrew god, and his
kingdom was restored to him (v:36). At this point, Nebuchadnezzar dropped out of the book
of Daniel, so who knows? Maybe the third time was charmed, and the king remained
convinced the rest of his life that Yahweh was "the god of Gods."

Needless to say, Babylonian records made no reference to any seven-year period of insanity
or absence from his official duties during the reign of Nebuchadnezzar. As William Sierichs,
Jr., pointed out in his article (pp. 2-4, this issue), there are implications that Nabonidus, the
real father of Belshazzar, may have experienced a period of mental instability that contributed
to his absence from Babylon, but there is nothing in the records to indicate that
Nebuchadnezzar had had mental problems or that he was absent from his official duties for
any protracted period. Of course, none of this will matter to biblical inerrantists. If the book of
Daniel claims that Nebuchadnezzar was repeatedly convinced by miraculous demonstrations
that Yahweh was "the god of Gods" but somehow kept suffering lapses that caused him to
forget it, that's good enough for them. Never mind how silly the stories are or how silent the
extensive nonbiblical records of the time may have been about such matters. If it's in the
Bible, they are going to believe it.

Lions 1, Daniel 0
William Sierichs, Jr.

Farrell Till's response to Everette Hatcher III on the Book of Daniel covers most of the
problems I see with Everette's comments. I offer some complementary remarks, and in a later
article point out problems in literature related to Daniel that will help illustrate the problems
with that book.

Up until the 19th century C.E., no one was allowed to challenge openly the accuracy of the
Bible, outside of brief periods of liberalism such as the Enlightenment. The idea that Daniel
was a historically accurate account of events in 6th-century B.C.E. Babylon was not only
taught in schools but was backed up by the force of law. In the later 19th century, that
situation changed. Freedom of the press prevailed, and a few brave scholars took on the
established church by challenging the accuracy of the biblical texts. They were assisted by
archaeological discoveries of libraries of ancient literature.

After several decades of debate, the evidence had persuaded a majority of historians that
Daniel was not accurate about the fall of Babylon and that its so-called prophecies best
matched events leading up to the 2nd-century Seleucid-Judean war, better known as the
Maccabean War. No discoveries or scholarship since the late 19th century have changed that
new, majority consensus. Daniel is not on the radar screen of modern Assyriology. The
evidence led to this change, not any new beliefs or philosophies. In particular, several texts

Volume 1990 - 2002 Issue


Page 1293 of 2049
Skeptical Review Edited by Farrell Till
contemporary to the 6th B.C.E. trashed all claims to historical accuracy for Daniel. These
include the Cyrus Cylinder, The Babylonian Chronicle, The Dream Text of Nabonidus, and
the Persian Verse Account. As historians repeatedly warn, these accounts are not objective.
They were propaganda to varying degrees, but they were written by and for people who lived
just before, during, or immediately after the conquest of Babylon. With cautious reading of
their characterization of people and events, their information will be the most trustworthy
available. The people and places they name are going to be real. That's why scholars rely
upon them. Sources from later periods have identifiable problems that make them less reliable
than the contemporary accounts.

Historian T. Cuyler Young, Jr., also suggests that Second Isaiah reflects the "tone, if not some
of its actual words," of Persian propaganda in such passages as Isaiah 45:1-3 and 45:13 (The
Cambridge Ancient History, Vol. IV, Part 1, Chapter 1, Second Edition 1988, 1992 reprint, p.
37). My Oxford Annotated Bible notes that Cyrus is called a "messiah" in 45:1, the only
reference to a non-Israelite messiah in the Jewish Scriptures, emphasizing Cyrus's importance
to the Israelites. Second Isaiah doesn't mention "Darius the Mede." By contrast, no texts of
Daniel are contemporary with the events surrounding the fall of Babylon. All we really have
are copies of copies of copies of copies, etc., of an anonymous, undated text.

The Cambridge Ancient History also has a discussion of the sources of history of that period
in chapter 3a, written by Amelie Kuhrt. He mentions the sources above and material from the
ancient writer Berossus partially preserved by Josephus in the 1st century C.E. and Bishop
Eusebius in the 2nd-3rd century C.E., both of whom had propaganda concerns in their
selections. By also compressing Berossus's writings, they created even more distortions. In
The Antiquities of the Jews (10:11), Josephus repeats the stories about Daniel, including the
story of the insanity of Nebuchadnezzar (the biblical, not scholars' spelling) but with some
significant divergences. Josephus says that Nebuchadnezzar was followed not by Belshazzar
but by Evil-merodach ("his son"), then Neglissar, then Labosordacus--for a total period of
nearly 59 years--and finally by Belshazzar, who Josephus says was also called Nabonidus.
Josephus also identifies Darius the Mede and Cyrus, as the conquerors of Babylon. He says
Darius was the son of Astyages, a king of the Medes. Josephus apparently used another
source besides Daniel and combined the two, both expanding and contradicting parts of
Daniel.

Herodotus's The Persian Wars contains another account of Babylon's fall. He credits Cyrus,
who he claims was the grandson of Astyages by his daughter and a Persian prince, with
capturing Babylon but makes no mention of "Darius the Mede." Herodotus gives a detailed
account of how Astyages, warned by a prophecy, tried to have the infant Cyrus murdered but
was tricked, so that Cyrus grew to manhood and later rebelled against his grandfather. No
historian believes the infant-in-danger story, found in many accounts about great heroes
(Herakles, Perseus, Oedipus, Sargon I, Moses, and Jesus, for examples). Herodotus also
thought Nabonidus was the son of Nebuchadrezzar. In Book 1:188, Herodotus calls the last
king of Babylon Labynetus, and says he was the son of a Queen Nitocris (not known from any
records). Historians say that Herodotus's chronology elsewhere would make Nabonidus the
son of Nebuchadrezzar, also referred to in Book 1:74 as King Labynetus. Herodotus gathered
his material on a brief visit to the city before 425 B.C.E. Herodotus' story has other problems,
and appears to be based on a distorted oral version of the real events of Babylon's fall.

Volume 1990 - 2002 Issue


Page 1294 of 2049
Skeptical Review Edited by Farrell Till
Little more than a century after the conquest of Babylon, the story already was becoming
confused. Modern historians' version of these events are based upon the 6th-century B.C.E.
sources: Nebuchadrezzar was followed by Amel-Marduk, Neriglissar, Labashi-Marduk and
Nabonidus. Amel-Marduk, Neriglissar and Labashi-Marduk ruled for a total of only about
seven years, not 59. Belshazzar was the son of Nabonidus and was never the king. The story
of the insanity of Nebuchadrezzar was probably adapted erroneously from accounts of the
strange actions of Nabonidus. And as Till pointed out, no ruler named Darius the Mede was
involved in the conquest of Babylon. A Persian general named Ugbaru led the troops who
first entered the city. Nabonidus was captured soon after and was either executed or given a
minor official post. The accounts conflict. The records make no mention of Belshazzar in
connection with the city's occupation.

The ruler of the Persians and the Medes was Cyrus II the Great. He was the son of Cambyses,
grandson of Cyrus I, according to Cyrus's own account. He defeated the last king of the
Medes, Istuwigu (his Babylonian Chronicle name), called Astyages in later texts. The defeat
is recorded both by the Chronicle and by The Dream Text of Nabonidus, attributed to that
king. Neither text claims any family relationship between Cyrus and Astyages. Amelie Kuhrt
warns against other sources, such as Xenophon's Cyropaedia, as suspect. Finally, Kuhrt
describes the Persian Verse Account, a poem attacking Nabonidus and praising Cyrus. It
"lampoons Nabonidus's oppressive rule and lampoons his pretensions to wisdom to the point
of making him appear a crazed despot" (Cambridge Ancient History, Vol. IV, p. 123). Kuhrt
adds, "It is possible that the first part of the poem had already been composed in Nabonidus's
reign to express popular discontent and was later manipulated for their own purposes by the
Persians." The Babylonian Chronicle also mentions eccentric behavior by Nabonidus. So we
have two contemporary texts that claim strange behavior by Nabonidus, plus the later Qumran
text that Everette mentions that attributes a period of insanity to Nabonidus. Additionally, no
contemporary sources give any hints that Nebuchadrezzar was insane. Set against this is the
claim in Daniel, repeated in The Antiquities of the Jews, that Nebuchadnezzar was insane;
Nabonidus's eccentricity is never mentioned.

The errors of both Daniel and Antiquities raise very serious doubts about their credibility. The
fact that the origin of Daniel is unknown and that Antiquities was written 600 years later and
is based partially on Daniel, while contradicting it upon the city's last monarchs, makes them
worthless for purposes of historical scholarship. The lions just took big bites out of Daniel.
Biblical literalists have tried to argue that Belshazzar could be called the king of Babylon
because he ruled during the 10-year period that Nabonidus had abandoned the city and
performed some royal functions, but the contemporary texts say the Babylonians were angry
because their New Year Festival was not performed during that 10-year period. Only the king
could perform it. Since Belshazzar didn't do it, he could not have been the king, only the
crown prince. The failure of the Book of Daniel even to mention Nabonidus points to its
author's having a corrupt source of history. That Antiquities knows the name of Nabonidus but
confuses him with Belshazzar shows how corrupt the historical accounts of the 6th century
B.C.E. had become by Josephus's time. Daniel resembles The Persian Wars and Antiquities in
this respect, not the 6th-century accounts. The lions just ripped off Daniel's legs.

The supernatural miracle stories in Daniel are another huge problem. Believers cannot assert
the reality of magic for one incident, such as the three men who survived inside a blazing

Volume 1990 - 2002 Issue


Page 1295 of 2049
Skeptical Review Edited by Farrell Till
furnace, according to Daniel, without asserting the reality of all stories of magic. The Bible
then becomes simply a companion book to The Iliad, The Odyssey, Gilgamesh,The Enuma
Elish, The Book of the Coming Forth by Day, The Popul Vuh, The Eddas, The Thousand and
One Tales From the Arabian Nights, etc. If a believer insists that one set of magic stories is
true, such as those in the Bible, and denies the reality of all other such stories, what objective
basis is available for such discrimination? Christian fundamentalists have never offered any
support for their claims, other than to say they believe them because they believe them, but
the same argument comes from all other religions in support of their magic stories. Devout
ancient Greeks, Egyptians, Aztecs, etc. were just as certain of the existence of their gods and
miracles as any Christian fundamentalist is certain of the Bible today. Belief alone can never
be proof.

This type of sectarianism is intellectually corrupt. It's similar to the totally subjective beliefs
of Jews, Roman Catholics, Greek Orthodox Catholics, and Protestants as to which books are
"canonical." Why should one book from Jewish/Christian history be accepted into a canon
and another one rejected as "apocryphal"? Why is the book of Daniel canonical, while Bel
and the Dragon and Susanna--other sets of stories about Daniel--are "apocryphal" to
Protestants but found in the Roman Catholic, Greek, and Slavonic Bibles. The Hebrew
scriptures don't contain the Apocrypha, but all of them (except 2 Esdras) are in the Greek
Septuagint version.

The obvious fact is that defenders of one set of miracle traditions or one set of canonical
books are merely repeating what they were taught as children and have never been able to
escape, even when the evidence is against those teachings or when they encounter an alternate
set of traditions. When various groups of Christians and Jews can't even agree on something
as fundamental as just which stories are genuinely "holy" or "inspired" by the god that they all
claim to worship, why should anyone believe those fantastic stories or the bibliolaters' claims
for any of them? The book of Daniel has been weighed in the balance and found wanting. The
lions just severed Daniel's jugular vein, and he has bled to death.

Scholars can piece together part of the origin of Daniel. First, Dan'el was a folk-figure in
Bronze Age Canaanite literature (see endnote). Ezekiel mentions a Daniel in Ezekiel 14:14 as
someone so righteous that he, with Job and Noah, other Bronze Age figures, were the only
ones who would survive if Yahweh were to destroy a faithless land. Ezekiel 28:3 refers to
Daniel as a figure of wisdom. Ezekiel lived several decades before the Persians captured
Babylon, so his Daniel, a figure of the past to him, cannot be the Daniel of the book of Daniel,
who lived there during and after the city's fall.

Comments by three modern historians are relevant here: Gosta Ohlstrom offers a simple
explanation for why the seeming (or literal) madness of Nabonidus was transferred to
Nebuchadrezzar: "In later traditions, Nebuchadrezzar has also been seen as an `evil' king. It
seems that the tradition about the `evil' king Nabu-na'id (Nabonidus) `became caught and
confused in a web of hatred against Nebuchadrezzar' in later Jewish traditions, because he
devastated and destroyed Jerusalem and Judah." He partly quotes and endorses another
scholar's conclusions here (The History of Ancient Palestine, 1993, Sheffield Academic Press,
p. 811).

Volume 1990 - 2002 Issue


Page 1296 of 2049
Skeptical Review Edited by Farrell Till
Robin Lane Fox suggests the stories in Daniel "are only a selection from a wider group of
Daniel tales: we also find him in the story of Susanna and the Elders (now in the Apocrypha
of English Bibles), and in Bel and the Dragon, and no doubt we could have found him in
many more. Probably, some of these stories had led an earlier, separate life: twice they call
Daniel by the second name of Belteshazzar, a Babylonian name which perhaps was the name
of the stories' original hero" (The Unauthorized Version, Alfred A. Knopf, 1991, p. 332).

Finally, John J. Collins points out that Jewish "pseudepigrapha" began in the 3rd century
B.C.E. with apocalyptic writings attributed to "Enoch," a grandson of Adam and Eve via their
son Seth, i.e., someone who lived before the Genesis flood (Gen. 4:26). The writings of
Enoch are blatant fiction, written by an anonymous Jewish writer who used an ancient figure
to present his own ideas. A number of other pseudepigrapha followed, attributing various
writings to Moses, Ezra and Baruch. If Daniel was written in the 2nd century B.C.E., then it
follows this tradition. Collins says, "Since Daniel resembles the books of Enoch, Ezra, etc. in
so many respects, our initial assumption must be that Daniel is, like them, pseudepigraphic.
Nothing in the Book of Daniel requires us to abandon that assumption. The visions of Daniel,
like those of Enoch, can be quite satisfactorily explained as constructs of the Hellenistic age,
which are ascribed to an ancient figure to add to their authority" (The Book of Daniel, First
Maccabees, Second Maccabees, Michael Glazier, Inc., 1981, pp. 12-13). So there's no need to
jump through elaborate series of hoops to find punning hints in the text that the author knew
the real story and concealed it, or to justify ignoring the 17-year reign of King Nabonidus
while calling crown prince Belshazzar the king, or to verify the fantasies about the
disembodied hand, the lions den and the furnace, or to explain away the discrepancies
between Daniel and the 6th-century texts. Story-telling and propaganda-- human traits that
have been around for millennia--explain the problems quite well.

So Persian and Jewish propaganda merged to invent the madness of Nebuchadrezzar.


Distorted versions of Babylon's last days were floating around. Folk stories from the
Babylonian and Persian periods were rewritten to incorporate the legendary Semitic figure of
Daniel, and in the 2nd century, a Jewish propagandist syncretized and expanded this material
to give credibility to his "prophecies" of Jewish victory over the Seleucid Empire, intended to
hearten a desperate people. As ancient literature, it's fascinating. As a historically valid
account of Babylon's fall, much less the "inspired word of god," worthy of incorporation in a
"holy" book, it's pathetic. It's time to let the vultures eat what the lions left.

Endnote: The figure of "Dan'el the Rapha-man" appears in the Canaanite poem, "The Tale of
Aqhat." One scholar has offered the intriguing suggestion that "Rapha-man" may refer to the
legendary giant race of the Middle East referred to in Hebrew scriptures as the "Rephaim"
(Gen. 14:5; Deut. 2:11). Like the Anakim, they are usually reckoned as Rephaim, though the
Moabites called them Emim (Deut. 2:20-21). The gigantic King Og of Bashan (Deut. 3:11)
was the last of the Rephaim, who were also called the Anakim (Num 13:22, 33; Deut. 1:28)
and the "Nephilim" (Gen. 6:4), all of them the children of the mating of the "sons of god" and
mortal women in Genesis (The Ancient Near East, Vol. 1, edited by James B. Pritchard,
translation by H.L. Ginsberg, p. 118). These gigantic great-great-grandchildren of god should
have been destroyed in the great flood, yet they reappear to fight the Israelites in the time of
David (1 Chr. 20), who killed the chief giant, Goliath of Gath (1 Sam. 17:50-51); a separate
story claims an Israelite soldier named Elhannon killed Goliath of Gath (2 Sam. 21:19)--yet

Volume 1990 - 2002 Issue


Page 1297 of 2049
Skeptical Review Edited by Farrell Till
another little contradiction in the Bible, and one more proof of its basis in multiple, nondivine
sources.

(William Sierichs, Jr., 316 Apartment Court Drive, Apt. 44, Baton Rouge, LA 70806)

EDITOR'S NOTE: At least one historian (George Roux, Ancient Iraq, Pelican Books,
1966, p. 352) has claimed that Belshazzar died in a battle with the Persians at a place
called Opis before Babylon fell. Roux, who didn't cite his source of information, may
have inferred the death of Belshazzar from the fact that he disappeared from
Babylonian records after the battle of Opis. However, if Roux is correct, Belshazzar
could not have been in Babylon on the night of its fall. If Roux was not correct, his
conclusion would at least underscore the problems that historians confront in trying to
put together accurate accounts of what happened thousands of years ago. One would
think that the problems are sufficiently complex that biblicists would be a bit more
reluctant to declare the Bible inerrant in all historical details, especially when some of
those details obviously conflict with the extrabiblical records that have survived from
that era, but biblicists as a whole are famous for their determination to defend the Bible
on faith rather than evidence.

A Statement of Editorial Policy


Farrell Till

From the beginning of The Skeptical Review, I have maintained an "open-door" policy, which
allowed inerrantists to respond to articles that they disagreed with and even to submit their
own articles in defense of the biblical inerrancy doctrine, but this policy was never intended to
be what inerrantist writers have reduced it to within the last two years. In the articles of Dr.
James D. Price and Everette Hatcher, we have seen a defense strategy that depended primarily
on quoting or citing Christian writers who are in agreement with the positions that they were
defending, as if it would be startling news to readers that there are actually writers who agree
that Jeremiah's 70-year prophecy was remarkably fulfilled or that the book of Daniel was
written by a 6th-century B. C. Jewish captive who had risen to a position of political
importance in the Babylonian empire or that Jesus rose from the dead, and such like.

Since the quoting of books and articles that agree with the writer is something that the
defender of any religious belief can do, TSR will no longer publish articles either by errantists
or inerrantists whose supporting details rely primarily on quotations from books and
periodicals. This does not mean that writers will not be allowed to quote sources; they will
just not be allowed to let strings of quotations from author after author substitute for
argumentation. They will at least be expected to quote material that gives relevant information
to support the position they are defending, and the mere agreement of another writer doesn't

Volume 1990 - 2002 Issue


Page 1298 of 2049
Skeptical Review Edited by Farrell Till
constitute relevant supporting information. If there is reasonable evidence to prove the New
Testament claim that Jesus rose from the dead, then a writer supporting that claim should
understand his obligation to present evidence in support of it that goes beyond simply quoting
passages from the New Testament that say it happened or citing commentaries or apologists
who believe that it happened. If a defender of the resurrection knows of some unbiased,
disinterested contemporary of Jesus who wrote something that supports the New Testament
resurrection claim, it would certainly be appropriate to quote it. That would be real evidence,
but quoting the mere beliefs of writers who lived well after the alleged fact would prove
nothing except that there have been people all through the Christian era who believe that
Jesus was resurrected. Isn't that a big surprise! The same is true of such claims as the
scientific accuracy of the Genesis creation account or the stories of Noah's flood or plagues of
Egypt or the parting of the Red Sea or the fulfillment of Messianic prophecies, etc., etc., etc.
Finding writers who agree that such biblical claims as these are true would be as simple as
going to the religious section of any public library and looking through books and periodicals.
Would-be apologists who use this approach are proving nothing except that they have had
access to books that agree with them, so in the future I am going to insist that writers on both
sides of the biblical inerrancy issue do more than just cite writers who agree with them. They
will have to support their positions with logical argumentation.

Inerrantists who wonder exactly what I am looking for should go through back issues and
check the articles that Roger Hutchinson submitted. As simplistic as his logic often was, he
did at least make an attempt to defend his position with what he no doubt thought was logical
argumentation. Whenever he quoted a book or periodical, he usually did so to give
information in his source that was relevant to the argument he was presenting, and sometimes
he even submitted articles that contained no quotations at all except from the Bible or the
article in TSR that he was responding to (See "Jairus's Daughter: Dead But Raised to Live
Again," May/June 1996, pp. 4-5.) To a lesser degree, the same was true of Matt Perman's
articles submitted in defense of the resurrection. His arguments were simplistic regurgitations
of Josh McDowellian apologetics, but he did at least give the substance of the arguments and
the reasons why he thought they were sound. This is all I am asking inerrantists to do in their
future submissions, and their cooperation will be appreciated. In phone conversations with
Everette Hatcher, he has agreed that this request is reasonable, so in the articles that he says
he will submit in further defense of his traditional view of the book of Daniel, we will expect
to see more argumentation and fewer citations that contributed little more than just agreement
with his views.

Clearing the Confusion over Babel


Stephen Van Eck

Volume 1990 - 2002 Issue


Page 1299 of 2049
Skeptical Review Edited by Farrell Till
One of the most familiar Bible stories is the legend of the Tower of Babel. It's an epic tale that
the majority of Bible scholars would classify as an explanatory myth. Only hard-core
fundamentalists still take it as an accurate historical account, intent as they are on maintaining
a juvenile grasp of the scriptures. An examination of this familiar tale in the light of objective
scholarship and logic should prove insightful to the skeptical and perplexing to the pious. Not
many skeptics know that the story of the Tower of Babel is actually rooted in history. There
was a real tower that inspired a folk legend eventually incorporated into Genesis. This tower
was one of several ziggurats built as part of the temple complexes of the ancient Sumerians
(Shinar), the seminal civilization of the Near East. The one at Babel was the largest ever built,
and was later known as the Temple of Marduk. Located in the old city portion, it was called
Esagila by the Babylonians.

In the 24th century B. C. E., construction on at least one ziggurat was interrupted by the
conquests of Sargon of Akkad,* the first great conqueror of the region. Subsequently the
Sumerians were supplanted by the Babylonians, and it would remain unfinished for over a
millennium and a half. This became a wonder to various peoples, who devised folkloric
explanations for its status, among which (predictably) was the displeasure of some god or
gods. This much is mundane fact. However, the theological meaning appended to it by the
ancient Jews is not only unverified, but unverifiable.

An analysis of their account of the tower reveals some devastating logical and theological
problems. According to Genesis 11, God implicitly took offense at the arrogance of man in
attempting to build a tower reaching to heaven, and it says he "came down." First of all, this
contains the antiquated notion that heaven is literally "up," something no theologian today
would assert. This story is the product of an earlier time with a different conceptual universe.
Another problem is the fact that even the largest of ziggurats reached only a final height of
around 300 feet (very tall buildings were not really possible in Mesopotamia, where only sun-
dried bricks were available). The pyramids of Egypt were much taller, yet God did not object
to their intruding on his domain and thwart their construction. Today's skyscrapers are taller
still, yet neither God nor man has expressed any notion that they impiously infringe on God's
sacred realm. Genesis also offers their motive for building the tower: so that they wouldn't get
scattered all over the world. As a reason for building, this makes not the slightest bit of sense.
What's more, it exhibits a prior awareness of future events, serving as foreshadowing. Hence
its literary and mythic basis is apparent.

In this tale (11:7), "God" spoke to an apparent plurality of gods, an indication of pagan
borrowing that hadn't yet been entirely laundered away. He complained that "nothing will be
restrained from them, which they have imagined to do." This divine complaint would make a
lot more sense today than it did back then. We can fly through the skies (once thought
reserved for the angels), we've gone to the moon, and we've invented numerous technological
marvels. We've cured many diseases (once thought sent by God), we've lengthened the
lifespan, and we're unlocking the secrets of DNA, the consequences of which are potentially
staggering. (All this despite language barriers!) It's utterly illogical to think that God would
allow all this to occur without a hint of objection but once got upset when people build a
tower that couldn't have possibly reached heaven, one that was not so tall to begin with.
(Maybe he was just waiting for a flimsy excuse to do what he had had in mind all along--
scatter everyone.)

Volume 1990 - 2002 Issue


Page 1300 of 2049
Skeptical Review Edited by Farrell Till
It must also be pointed out that, contrary to Genesis 11:9, "Babel" means "gate of god." Both
Genesis and Exodus resort to folk etymology consistently, even when incorporating foreign
names. The emphasis was on finding a symbolic meaning in the context of the story, but in
application it was frequently a stretch. Here a vague correspondence was found between
"Babel" and the Hebrew word "balal," meaning confused. But it's clear they are not at all the
same term. (English speakers tend to insert their own etymology here for "babble.") This folk
etymology underscores again the literary basis of the story. Further legendary echoes are
found in the traditional attribution of the Tower of Babel to Nimrod. "Nimrod" (10:8-10) is
how the Jews rendered the name Ninurta, who was the first Assyrian conqueror, but he lived
in the 13th century B. C. E., over a thousand years after the tower's construction had been
initiated. His association with the tower, his legendary amplification, and his misplacement in
time are all explained by the fact that the Jews' grasp of history prior to Ninurta was hazy,
given that they themselves had just emerged as a self-aware cultural entity. In the 19th
century C. E., French archaeologists found in the ruins of Babylon an inscription apparently
by Nebuchadrezzar (Nabu-kudurri-usur). It referred to an old ziggurat that had long been
unfinished but that he had undertaken to complete. Apologists since then offer this as concrete
proof of the historical accuracy of the Genesis account. There are four serious problems with
this conclusion. First, the historical reality of an unfinished ziggurat does absolutely nothing
to verify the theological meaning attached to it by the Jews. The existence of a building, even
in ruins, is easy to verify, but whether God took offense at it, halted its construction, confused
people's language, and scattered them cannot be concluded from any inscription. This is
especially so, given that a plausible explanation exists for its interruption, namely the
inevitable disruption of conquest and the disappearance of the Sumerians who had started it.

The second problem poses a theological quandary. The text of the inscription refers in three
places to the tower's completion: "I have completed its magnificence," "I built and finished
it," and "I exalted its summit." Now if God had found it objectionable from the start, why
would he have allowed it to be completed? Wouldn't this have violated his will, and wouldn't
he therefore have prevented it? Otherwise, why would he have bothered to stop its
construction to begin with? Third, there is some confusion over which of two sites is that of
the actual tower. The Babel site is problematic. Nebuchadrezzar's inscription refers to the
temple at Borsippa, which was the temple of Nebo, and not to the temple of Marduk at Babel.
Since Borsippa and Babel are two different places, and there were temples at both (Babel's
being much taller), it's implausible that the former can accurately be referred to as "the Tower
of Babel."

The conclusion that the Borsippa site may have been the inspiration for the legend, later
mislocated to Babel, is supported by the fact that in ruins it was known as Birs Nimrud. This
is a subsequent reference to the prior legend. Additional evidence for Borsippa ironically
comes from one of the literalist's key pieces of evidence--the assertion that Borsippa means
"tongue tower." This name is derived from the Chaldean ("Barzippa"), indicating a linguistic
origin much more recent than the tower itself. Like Birs Nimrud, rather than confirming the
Genesis account, it merely suggests the use of existing legend as the source for a later name,
legend which also found its way into Genesis during or after the Babylonian Exile.

Further confusion is created by the inscription's reference to Marduk for its inspiration, not to
Nebo, whose temple it was, and whom Nebuchadrezzar's very name invokes. This

Volume 1990 - 2002 Issue


Page 1301 of 2049
Skeptical Review Edited by Farrell Till
discrepancy calls the inscription's credibility and authorship into question. (Let's hope the
mention of "Merodach" instead of "Marduk," and especially the erroneous "Nebuchadnezzar"
rather than "Nadu-kudurri-usur"--or even "Nebuchadrezzar"--is the result of translator
"license," and not the actual text!)

The Genesis story of the Tower of Babel is an explanatory myth, accounting for the diversity
of languages and the distribution of people after the legendary flood. It neatly served the
theological and narrative purposes of the Genesis author (Ezra?), but it has no historical basis
beyond the sketchy mention of an unfinished tower and some legendary resonances. And this
tower, it would even appear, was the wrong one! Die-hard literalists, however, always insist
on maintaining the fairy tale as reality, but in so doing they stand on shaky ground. To be
more precise, they're like those Warner Brothers cartoon characters who step off a ledge--they
won't fall until they look down and realize their lack of foundation. Now biblical literalists
can spend the rest of their lives not looking down, or they can get back on the ledge! Where
do you stand?

* According to an ancient legend, when Sargon of Akkad was born, his mother placed him in
a basket sealed with pitch, and set him adrift on a river. Sargon preceded Moses by more than
a thousand years, and his story was recorded in Babylonian archives prior to the composition
of Exodus (Grant R. Jeffrey, The Signature of God, Tyndale House, 1997).

(Stephen Van Eck, Route 1, Box 62, Rushville, PA 18839-9702)

Kosher Baloney
Tim Gorski, M. D.

Even the Old Testament kosher restrictions and other Hebrew practices are occasionally
defended by Bibliolaters as being eminently sensible and/or scientifically justified. They
imply--or explicitly claim--that this is more proof of the Bible's divine origin. Most often this
is done by Jewish fundamentalists, probably because Christian, "God's-Word" fanatics would
have to explain why, if keeping kosher is so great, they don't follow the strictures.

The arguments that I am talking about--that I have heard of--are as follows:

(1) The kosher dietary strictures against the eating of pork are based on the sound medical fact
that the parasitic disease of trichinosis can be transmitted through the consumption of pork
products.

(2) The kosher dietary strictures against the eating of shellfish are based on the sound medical
fact that these, too, can transmit disease.

Volume 1990 - 2002 Issue


Page 1302 of 2049
Skeptical Review Edited by Farrell Till
(3) The kosher restriction against mixing meat and milk was a prescient practice that reduced
or avoided the risk of typhoid.

(4) The circumcision of infant males was delayed until the eighth day after birth because it
takes this long for blood clotting factors to develop.

Even allowing that these may be instances of kosher practices having some practical utility,
this would hardly argue for their being of divine origin. The operation of natural selection on
the simple process of trial and error would be sufficient to result in the prevalence of useful,
and especially lifesaving traditions. Nor does anything in the Bible argue to the contrary. God
never tells his "chosen people," for example, that the reason they're to avoid pork or not mix
milk and meat products is because of "creeping things" too small to see that can make them
sick. Neither is there so much as a single account of someone disobeying "God's law" by
circumcising an infant before the seventh day and being "punished" by having their boy bleed
to death.

In addition, if it is going to be claimed that these kosher/Jewish biblical traditions were


established because of then-unknown scientific facts, then all the other idiosyncratic Jewish
strictures need to be shown to have a similar basis. Thus, it needs to be explained why it's OK
to eat ducks and geese but not swans (Deut. 14:16). An account needs to be given for why the
faithful were not permitted to eat, not just pigs, but also camels, hares, and hyraxes (Deut.
14:7). Old Testament, "God's-law" defenders should tell us the scientific basis for why their
Yahweh insists that we not sow a field with "mingled seed" or wear a garment "mingled of
linen and woolen (Lev. 19:19). They should justify the prohibition on eating blood (Lev.
19:26 and elsewhere) and on "round[ing] the corners of your heads" or "mar[ring] the corners
of thy beard" (Lev. 19:27). I would especially like to have the medical scientific facts that
support all the biblical teachings about diagnosing and treating leprosy, including the use of
bird blood as described at Leviticus 14:50-51.

Now with regard to the particular claims listed above concerning the Bible's supposed
science-based recommendations:

* Although pork can transmit trichinosis, there are lots of other parasitic nematodes that are
transmitted by other means, including the eating of beef. Poultry and just about everything
else can transmit diseases as well. Yet all of these, including trichinosis, can be avoided by
the simple expedient of cooking such foods thoroughly. So if God's intention was to issue
kosher rules to reduce the risk of food-borne illness, all he needed to have done was to insist
that all meats be well cooked. Handwashing by those handling the food would also have been
a plus, but I haven't been able to find any verses that commanded this simple step.

* Again, just about any food item can transmit disease. And, again, thorough cooking largely
if not completely eliminates the danger. There is no disproportionate risk from shellfish.

* Neither is there anything unique about mixtures of meat and milk. In fact, milk is a
relatively poor culture medium compared to, say, fruit juices, as far as disease-causing
bacteria are concerned. Besides, the Bible's prohibition [at Exodus 23:19 and elsewhere] is

Volume 1990 - 2002 Issue


Page 1303 of 2049
Skeptical Review Edited by Farrell Till
that "(t)hou shalt not seethe [emphasis added] a kid in his mother's milk." Seething, which is
to say boiling milk and meat together, destroys bacteria that can cause food-borne illnesses.

* Nor is there anything significant about the eighth day after birth as far as the safety of
circumcision is concerned. The eighth day was simply numerological superstition. Infants
have been routinely circumcised much earlier than that.[1] And if an infant is going to bleed
to death from a circumcision, it's much more likely to be as a result of an inherited
coagulation defect. The real question is: why circumcise at all? And the answer is that it was a
symbolic form of human sacrifice. In modern times, although it is arguable as to whether
there may be benefits from circumcision, the evidence is by no means overwhelming and
would almost certainly be outweighed by the risk of infection in a prescientific era in which
antibiotics were unknown.[2]

Clearly, "God's law" is not grounded in any kind of knowledge or understanding that
exceeded the rudimentary status of what people thought at the time that the Bible was written.
But fundamentalist Jews and Christians have yet to come to grips with this fact.
Fundamentalist Christians have an even tougher task unless they are going to disagree
completely with their fundamentalist Jewish colleagues and take the position that all the
kosher laws are nothing but empty ritual. For if it is to be believed that any of "God's law"
with regard to dietary strictures and the like has some basis in science, then one must
conclude that what was a good idea then is still a good idea. Moreover, it would also have to
be concluded that, unless he intended for Christians to be miraculously impervious to all the
food-borne diseases that kosher practices supposedly helped to prevent, Paul was mistaken
when he dismissed such traditions as childish.

[1] It is true that Vitamin K injections are now routinely given to newborns in order to hasten
the production of blood-clotting factors, but even before this measure became common,
infants were circumcised during the first 48 hours or so of life without any ill effects.

[2] Why didn't Yahweh teach the ancient Jews about bacteria and how to cultivate the
Pennicillium fungus?

(Tim Gorski, M. D., P. O. Box 202477, Arlington, TX 76006; e-mail, gorski@atheist.com)

Twenty-Four Hours in Time


An Anonymous Claim

Volume 1990 - 2002 Issue


Page 1304 of 2049
Skeptical Review Edited by Farrell Till
The following story has been called to our attention from many sources. Several readers have
sent us clippings from other newspapers. It has received far flung attention and by now you
may have seen it. However, the wonder of it is too much for us to neglect. Here is the story of
"The Missing Day in Time" as retold by the Jayton Chronicle.

This incident happened while our astronauts and space scientists at Green Belt, Maryland,
were checking the position of the sun, moon, and the planets out in space where they would
be 100 years and 1000 years from now. This is most important in order to keep the satellite
from colliding with another object in one of its orbits.

The necessary information was fed into the computer and it ran the measurement back and
forth over the centuries and it came to a halt. The red light came on and the computer stopped.

The information was rechecked and refed into the computer. The information was correct:
something must have been wrong with the computer. The service man was called. The
computer was checked. "It is perfect," he said. The head of operations asked, "What's
wrong?" The reply was, "We have found there is a day missing in space in elasped [sic] time."
Our space program had bogged down. There was no answer! One of the men on the team
spoke up, "You know, one time when I was in Sunday School and we talked about the sun
standing still." He took his Bible and turned to Joshua where the Lord said to Joshua, "Fear
them not. I have delivered them into they hand; there shall not a man of them stand before
thee." Joshua and his men were surrounded by the enemy and if darkness fell they would be
slaughtered. Joshua petitioned the Lord that the sun stand still. What was the result? "The sun
stood still and the moon stayed--and hasted not to go down about a whole day." The space
men queried, "That might be our missing day. Let's check the computer and see." They
checked the suns [sic] position in Joshua's day and found that it was close but not close
enough. The time elasped [sic] in Joshua's day was 23 hours and 20 minutes--not a whole
day. They read the Bible and there it was--"about a day," that is approximately.

These little words in the Bible were so important to our spacemen. But they were still in
trouble because if yor [sic] cannot account for 40 minutes, you will still be in trouble 1000
years from now. "Somewhere in the Bible it speaks of the sun moving backwards," said the
space scientist. They got out the Bible and read the words in II Kings 20:9-11: "Hezekiah, on
his deathbed, was visited by the prophet Isaiah, who told him that he was not going to die.
Hezekiah asked for a sign as proof. Isaiah asked, "Do you want the sun to go ahead 10
degrees?" Hezekiah said, "It is nothing for the sun to go ahead 10 degrees, but let the shadow
return backward ten degrees." Isaiah spoke to the Lord and the Lord brought the shadow ten
degrees backward. Ten degrees backward is exactly 40 minutes.

Twenty-three hours and 20 minutes in Joshua 10, plus 40 minutes in II Kings 20 making the
missing 24 hours the spacemen had to log in the logbook as being the missing day in the
universe!

What more can we add? "The grass withereth, the flower fadeth; but the word of our God
shall stand forever (Is. 40:8).

Volume 1990 - 2002 Issue


Page 1305 of 2049
Skeptical Review Edited by Farrell Till

Fundamentalist Flapdoodle
Farrell Till

The anonymous claim published above has been circulating for at least a half century, because
I first encountered it when I was a sophomore in 1952-53 at a Bible college in Arkansas.
Although my memory may be faulty, I seem to recall that I read it in Harmony of Science and
Scripture, a book in "Christian evidences" that was written by Harry Rimmer, the Josh
McDowell of his day. Over the years, I have seen variations of the claim, periodically updated
to adapt it to the latest technology and to give it the appearance of being just a recent
discovery. The version above was sent to me by Charles N. Brennecke, who stated in a cover
letter that someone left it at the Minnesota Geological Society's exhibition booth at the state
fair. Brennecke's response to the technological flaws in the claim appears on the following
page of this issue, but I too would like to make some comments not included in Brennecke's
reply.

Those who have participated in internet discussions about biblical inerrancy have no doubt
encountered at least some variation of this claim. It is so patently false that about two years
ago Reason and Revelation published a warning to its readers not to use it in their evangelistic
efforts. Reason and Revelation is published by Apologetics Press, Inc., a nonprofit
organization staffed by Church- of-Christ preachers and members, who pride themselves on
their scientific approach to biblical apologetics. Biblical inerrantists would do well to heed the
advice of the editors at Apologetics Press and refrain from using this discredited argument,
because those who don't take their advice will end up looking very foolish. Perhaps the
unknown author of the article above was aware of the risk, and so that was why he wrote it
anonymously.

To check the remote possibility that the reference to an article in the "Jayton Chronicle" may
have some basis in fact, I called directory assistance in Texas and asked if there was a phone
number for the Jayton Chronicle, and I was told that there was none. This didn't surprise me,
since I had already checked an atlas and found that the population of Jayton, Texas, is only
608. It is located in Kent County in West Texas, and the atlas shows only three other towns in
this county, all of which are evidently so small that no population figures were given. The
likelihood that any such article was ever published in the Jayton Chronicle seems remote
indeed.

Those who have access to the internet can find more information on this myth at
http://www.urbanlegends.com/science/missing_day.html, a web page written by Jim Lippard.
He points out that the NASA version of this legend dates back to the 1960s when it was
circulated by Harold Hill, who claimed that he was present at NASA's Goddard Space Flight
Center when all of this occurred. Lippard states that NASA has denied that it ever happened.

Volume 1990 - 2002 Issue


Page 1306 of 2049
Skeptical Review Edited by Farrell Till
Hill, who was formerly the president of the Curtis Engine Company of Baltimore, worked in
the diesel engine department at Goddard and had nothing to do with computer operations.

Lippard traced the pre-NASA version of this tale as far back as 1890 when Charles A. L.
Totten published it in Joshua's Long Day and the Dial of Ahaz: A Scientific Vindication. In
1974, Hill's version of the story was published in How to Live Like a King's Kid, by Logos
International, a Christian publisher with a history of publishing phony testimonies and
discredited claims. Among the National Enquirer types of books put out by Logos
International, Lippard cited Fernand Navarra's Noah's Ark: I Touched It, which, like all the
others in the list, has been debunked.

For those who have further interests in checking out this legend, Lippard recommended Jan
Harold Brunvand's The Choking Doberman and Other "New" Urban Myths, W. W. Norton &
Co., 1984, pp. 198-199; Robert W. Loftin's "Origin of the Myth about a Missing Day in
Time," Skeptical Inquirer, Summer 1991, pp. 350-351; and Tom McIver's "Ancient Tales and
Space-Age Myths of Creationist Evangelism," Skeptical Inquirer, Spring 1986, pp. 258-276.

Unfortunately for the anonymous author of the article above, there is no basis in fact for this
claim about a "missing day in time." Whoever he is, this anonymous author may have been
sincerely duped by a legend that keeps being circulated long after it has been debunked. This
should be a warning to TSR subscribers who believe in the traditional view of the Bible that
they should check carefully such claims as this one before they accept them and certainly
before they repeat them. Sad to say, in their zeal to defend the Bible, inerrantists are not above
circulating false information, and Bible-believers who don't engage in apologetics themselves
need to realize this.

Twenty-Four Hours in Time: A Rebuttal


Charles N. Brennecke

This yarn is without a doubt the most ridiculous piece of fiction I have seen in a long time.

Item: What is the source? The author clearly intends to remain anonymous--and with good
cause.

Item: The report is said to be from the Jayton Chronicle, but the anonymous writer doesn't
say if this is Jayton, Texas, which is the only Jayton shown in world atlases, or some other
unlisted Jayton (assuming even that Jayton, Texas, does have a newspaper name the
Chronicle). The story is said to have been reported in "other newspapers," but they are
carefully not identified.

Volume 1990 - 2002 Issue


Page 1307 of 2049
Skeptical Review Edited by Farrell Till
Item: No such incident ever happened in Greenbelt, Maryland. The author of the story
obviously knows nothing about how computers work or what astronometrics is all about.
NASA has a real computer center there, and one of their tasks is to calculate a table that is
know as an ephemeris (plural, ephemerides). These tables give us the positions of
astronomical bodies (sun, moon, stars, satellites) with respect to a given point of observation.
The point can be anywhere in space, even on the planet Pluto if necessary; it can go back (or
ahead) to any point in time.

The problem is that the calculations are based on the observed orbits of these bodies, and
accurate observations are not more than 300 years old. Observations accurate enough for long
term projections didn't even begin until fifty years ago. Based on these observations, the
computer can hustle back to Joshua's time and tell in which direction Joshua would have had
to look to see, say, the planet Venus, and how far above the horizon he would have had to
raise his eyes.

Item: Are those calculations correct, in the sense that an actual Joshua living over 3000 years
ago, would have actually found Venus at the predicted point? Not necessarily--both Earth and
Venus may have changed their orbital characteristics since that time. The only way to check
their correctness would be for NASA to send somebody back to Joshua's time and actually
measure! So far this is beyond the capability of rocket science. So when the story-teller says
that the Greenbelt people were "checking" their calculations, we have to ask, "Against what?"

Item: The problem of the computer stopping because it found a "missing year" is hilarious. A
computer knows nothing about "years" until the programmer tells it. What the computer does
is continuously repeat a particular calculation, each time with a different set of numbers
plugged into the calculation to get a result. The computer stops only when the programmer
tells it to; this can be done by telling the computer to do just so many repetitions or by telling
the computer to keep track of each result as it is calculated and then stop when a certain result
is achieved. The computer is totally insensitive to the facts underlying the numbers it is
crunching.

Item: Somehow the passage in Joshua, chapter 10, became converted from "about a day" to
the precise "23 hours and 40 minutes." If the computer couldn't tell Greenbelt that a day was
missing, how did it calculate the exact time of 23 hours and 40 minutes?

Item: The author relied on the KJV translation in 2 Kings, but he assumed a lot from a
questionable translation. The word translated here as "degrees" is a word that is elsewhere
translated as "steps" or "stairs" and is rendered "degrees" only in the story about the extension
of Hezekiah's life. Other translations besides the KJV and NKJV use "steps" instead of
"degrees" even in the tale about Hezekiah. Actually, then, Isaiah was asking Hezekiah if he
wanted the sun to go ahead 10 "steps" or back 10 "steps." Hezekiah made the interesting
observation: "No, anyone can make the shadow move ahead--instead let it move backwards
ten steps." We can't really know what "ten steps" meant in terms of measuring time in
Hezekiah's era or for that matter what kind of measuring device they were using. If they used
sundials like the modern type, it would have been easy to make the shadow move in any
direction without ever involving the sun at all, by just giving the gnomon a little shove with
an elbow.

Volume 1990 - 2002 Issue


Page 1308 of 2049
Skeptical Review Edited by Farrell Till
Of course, the measurement of angles in degrees, minutes, and seconds, as well as the
measurement of time in hours, minutes, and seconds is a relatively late invention. How the
author of this tract converted the angle measurement of "ten steps" into precisely 40 minutes
remains a mystery.

We haven't even addressed the physical consequences of stopping the earth's rotation for even
a few seconds, much less a 24-hour period. The energy requirements are so huge that it would
take the impact of a major celestial body, striking at just the right angle and the right spot on
the earth's surface, with exactly the correct velocity. Since the earth acts like an enormous
gyroscope, stopping the rotation would immediately make it tumble at right angles to its path
in orbit, so Joshua might have gotten a few seconds of darkness before the sun began to come
up in the north and set in the south! Dissipating all that rotational energy would have raised
the temperature substantially and perhaps fractured the mantle down to the iron core. All life
would probably have been exterminated within a few seconds, and poor ol' Josh would never
have gotten to savor his victory.

If this story is the sort of childish nonsense that appeals to Bible-believers, they are certainly
welcome to it. I do hope, for their sake, that they don't try to apply their insights to actual
projects!

(Charles N. Brennecke, 7992 Goodrich Avenue, St. Paul, MN 55105-3343)

A Letter to Gleason Archer


Dave Matson

Although your letter (TSR July/ August 1998) was intended for Farrell Till, it was, in a very
real sense, addressed to all of us who question biblical authority; thus, my reply to a letter that
very much deserves a lengthy reply.

Right off the bat, you have the gall to suggest that anyone who rejects the Bible is
"meaningless" and must partake in a life that "is nothing but an empty dream." That is your
theme throughout. By what right do you set the definition of a "meaningful life"? By what
stretch of logic is life "meaningful" only if it goes on and on without end in a Christian
heaven? How arrogant of you to dismiss all those other religions with the sweep of your hand!
(Many of them strike me as being far superior to Christianity.)

Let me present another viewpoint for you to consider. What meaning is there in a life that
goes on forever? Either that life must stagnate within certain parameters for an infinity or else
lose its identity in an endless sea of change. The latter is none other than a kind of death; the
former could be a meaningless hell.

Volume 1990 - 2002 Issue


Page 1309 of 2049
Skeptical Review Edited by Farrell Till
A meaningful life must be finite. A finite life may grow without completely losing its identity.
There is no prospect of going in circles forever. Were we to have the power of living as long
as we wished, I think that a time would come, sooner or later, however pleasant our existence,
when each and every one of us would say "Enough."

My life is meaningful. I ought to know. I'm the one who is living it! Its meaning is tied to the
"here and now" and not to some "Big Daddy" in the sky or to the prospect of living forever. If
life has no meaning in our short existence, then how can it have meaning if prolonged
forever? We can only live one day at a time. If there is no meaning in today, even though it
quickly passes and is forgotten, then where in an infinite life will meaning be found? Is mere
existence itself the ultimate meaning? I think not!

Each life lived well is like a story well told. It has a beginning, an end, and meaning. It does
not go on for ever and ever. Indeed, no story could truly go on forever and have meaning.
Either it would be trapped in an endless repetition of one sort or another, or it would wander
forever without settling on a clear meaning. The former would bore a reader to death; the
latter would have no point to it.

In an infinite story, 99.99999+% of the meaning must either be in the part already told (or will
be told) or in the remainder. If the former, then an infinite stretch of the story has no meaning;
we might as well cut it short. If the latter, then we are told a story with no meaning in that the
meaningful part is always beyond our reach. So, too, for an infinite life. We live only in the
"here and now," and if any meaning is to be had it must be there. Thus, a finite life may have
meaning.

Our "radical" rejection of so influential a work as the Bible is neither radical nor nihilistic. It
is a consequence of a vigorous exercise of reason, which in its higher forms is exhibited by
mainstream Bible scholars everywhere. Having influence is not the same thing as having
truth. The Bible may well be the most error-filled work known to man-- outside of those
voluminous commentaries purporting to prove its inerrancy. Neither the Koran nor the Vedas,
nor Hitler's book, nor Marx's book, nor Mao's little Red Book have anywhere near the
contradictions and absurdities found in the Bible. Understanding that fact is a positive step in
grasping reality, not a journey into nihilism. Our lives are quite meaningful and positive
without an inerrant Bible, thank you!

The skeptic scarcely needs to examine each and every one of those "600 topics of prediction
set forth in 8,352 verses of the Bible." It is enough to know that you and others, who, no
doubt, are familiar with each of those 600 topics, cannot produce a single one of note that
withstands a careful, scientific scrutiny. And, even if such a prophecy could be found in that
compilation of assorted, diverse works and multiple editing known as the Bible, that would
scarcely boost our confidence in those passages depicting talking snakes and donkeys--or
metal ax heads that float on water at the toss of a stick.

We rightfully reject those 600 topics of prediction, at least those requiring miracles, for we
have studied the best of them that have been put forward these last two hundred years. They
have holes big enough to drive a truck through! If, perchance, we have overlooked something,
do bring it to our attention. I assure you that we are interested in the truth, and that means

Volume 1990 - 2002 Issue


Page 1310 of 2049
Skeptical Review Edited by Farrell Till
looking at any serious evidence you may have. On a broader front, Till stands ready, even
now, to debate you should you change your mind and take him up. As for myself, I'll be
happy to examine any "conclusive" evidence you may have via correspondence. Impress us!
Present just one, miraculous prophecy that can withstand scientific scrutiny!

Why do you throw that tired, old, slanderous label of "communist atheists" at us? Don't you
know that only a few American atheists are communists? Such a cheap shot! Nor are we alone
in rejecting a personal creator-god. Have you forgotten about those religions of the Far East?
Do you think that every tribal religion believes in a personal creator-god? Where do you put
the Japanese in your scheme of things? Many religions do not have an all-powerful "creator-
god." You have a lot to learn!

Yes, we atheists do reject the firm conviction of godism, which is held by most of the human
race. Since when was truth determined by majority vote? Did not Columbus reject popular
belief when he sailed west in order to reach India? Did not Einstein do the same when he
developed his radical, new ideas of time and space? You speak as though it were a crime to
go against popular opinion! Where do you think new ideas come from? You would be
Catholic today had Luther and others not challenged the status quo!

Note well that the best and brightest are clearly on our side. According to a recent survey,
only about 7% of our scientists are believers. That, in a country that is a hotbed of Christian
fundamentalism! Statistics show that the better educated a person is, i. e., the more developed
his or her brain, the more likely that person will reject biblical authority. In the extreme case,
that of the Nobel prize-winners, devout, Bible-believing Christians are something of a rarity.
The Bible is criticized, of course, by both atheists and Christians. Devout (but intellectually
honest) Christians led the way.

Far from lacking a standard of morality, we understand its true meaning and source.
Therefore, unlike the religious terrorist, we cannot abandon morality in the name of some god
whenever we please. We will never be able to justify holy wars or burn dissenters at the stake,
in the name of Jesus. We will never have a theology that can justify massacring neighboring
peoples and taking their lands in the name of God. We will never burn our babies in ovens or
beat them to death to drive out "the devil," as is done surprisingly often by distraught
Christians who think that the rules of morality may be lifted for religious reasons. Neither are
we obliged to "justify" the atrocities found in the Bible. Trying to justify such atrocities is
bound to sow confusion in one's mind as to what is and is not moral.

Far from being morally rudderless, as you seem to think, we are at the, fountainhead of
morality! We don't need an intermediate, a "Big Daddy" in the sky, to tell us that love is good
and murder is bad! We have a pretty good idea of how to create a warm, friendly and
workable community. The proof of the pudding is in the eating of it. We have no need to
further justify our rules of morality. What works, works.

We have no need of a god to pass judgment on us. For the wayward among us, the long arm
of the law is much more effective. Christians can sin "on credit" and confess at the last
minute, thereby avoiding hell. What kind of system is that for preventing crime? The great

Volume 1990 - 2002 Issue


Page 1311 of 2049
Skeptical Review Edited by Farrell Till
bulk of locked-up criminals profess to be religious, but their beliefs didn't stop them from
committing their crimes.

We have no need of a god to praise us. Praise and recognition will flow from the just societies
that we create or live in. Since when does the biblical god praise individuals, anyway? Doesn't
the Bible say that God is no respecter of individuals?

Our thirst for knowledge is keen. We seek real knowledge, the truth, not mere justification of
dogmatic principles. We are free to seek real knowledge, whereas the conservative Christian
must support his or her literal belief in the Bible no matter how absurd it becomes.

Our reverence for life is deep. Unlike our fundamentalist brothers, who regard life on Earth as
a mere stop on the road to paradise, an unpleasant but necessary stop at that, a place to do
one's duty and be gone, we are free to embrace life in all its wonder and beauty. Unlike our
fundamentalist brothers, we know that such magnificence must be cared for. We will never
allow our natural resources to be used up or abused under the theory that the world is shortly
coming to an end, anyway. We suffer from no such illusion.

By all accounts, be it a firmer grip on morality or having life with a deeper meaning, the
atheist (who has adopted secular humanism) stands well. I would not trade my lot with any
Christian alive or dead.

Why should we fear death? Death is like leaving a banquet. You don't want to leave too early
as the food and company are at their best. On the other hand, you don't want to eat forever!
After a while, the food becomes bland and the pleasures blurred, and your stomach hurts.

Unlike many Christians, we do not view ourselves as unworthy worms hanging over a great
fire by a slender thread. How can anyone fully enjoy life with such poison in their brains? If
you teach a person that he or she is a worthless stench in God's nostrils, what kind of person
are you going to raise? That some fundamentalists do grow up healthy in mind and body is
more an attribute to human robustness and independence than anything else. The whole idea
of hell, no doubt invented for a complex of historical reasons--but especially useful for
keeping an ignorant flock in line, is one of the great contradictions of Christian theology. God
cannot be both a mass torturer par excellence and a loving father at the same time, period. No
amount of jumping backwards through flaming hoops, of postulating infinite crimes and other
such absurd notions, will ever justify the concept of hell. The atheist has the good sense
(along with some Christians) to dust it off as a primitive fairy tale.

If you prefer a sadist for a god, then keep your hell of fire and brimstone. You are welcome to
it. However, don't mistake us for simple-minded children who may be frightened by such talk.
Did you know that the history of hell may be traced back to Iran and other places? It's an
evolved, man-made concept. Thus, hell holds no terror for us. There is no "grim truth" there
to be owned up to at some future date, except in ignorant minds. We don't waste our time and
energy on such idiocy. We have far better things to do, such as enjoying a meaningful life.

(Dave Matson, The Oak Hill Free Press, P.O. Box 61274, Pasadena, CA 91116; e-mail,
103514.3640@compuserve.com)

Volume 1990 - 2002 Issue


Page 1312 of 2049
Skeptical Review Edited by Farrell Till
EDITOR'S NOTE: If Dr. Archer wishes to reply to Dave Matson, I will publish it.
However, I would prefer that Dr. Archer show the courage of his convictions and agree
to defend the Bible in public forum, preferably at the theological school where he
teaches. If he is so certain that the inspiration of the Bible has been shown by "more
than 600 topics of prediction," he should welcome the opportunity to defend this claim
in a forum that would give him the opportunity to vindicate the Bible before an audience
of aspiring clergymen, whose faith would surely be strengthened by a successful defense
of Dr. Archer's claim.

From the Mailbag

Take Me Off Your List...

I don't know where you got my name, but drop it off your list. I do not appreciate your
literature, which I believe is inspired by Satan, that old anti Christ [sic] himself. Very evident
and confirmed by 1 John chapter 4.

Do not send anymore, because it ends up in the trash, where it belongs.

(John Gramm, P. O. Box 456, Gridley, IL 61744)

EDITOR'S NOTE: As I have noted many times in comments on letters like this one, Christians
live by a double standard. Probably no religious group imposes more literature on others
than Christians. They go door to door, and they put tracts on car windshields, on bulletin
boards, in shopping centers, in rest rooms, and anywhere else that they think affords them the
opportunity to spread their doctrinal absurdities, yet many of them will react as Mr. Gramm
did when literature that opposes their beliefs is given to them. They want the right to press
their opinions on others, but they don't want others to have the same privilege. Mr. Gramm
received The Skeptical Review at the request of a friend. After Mr. Gramm's letter was
received, I had a phone conversation with the friend who had asked me to send TSR to
Gramm, and the conversation confirmed my suspicion that Gramm is probably a typical
Christian in that he wants others to read his literature. During the conversation, this person
referred to Gramm as "my former friend," so perhaps Mr. Gramm will want to take note of
that. If he expects the right to communicate his religious beliefs to his friends, he should
extend to them the same privilege if he expects to retain their friendship.

The shallowness of Gramm's comments was typical. He considers 1 John 4 to be confirmation


of the existence of an anti-Christ, and so we see that Gramm is only another Christian who
thinks that the mere citation of a scripture constitutes proof of his religious position. That's
too pathetic to deserve additional comment.

Volume 1990 - 2002 Issue


Page 1313 of 2049
Skeptical Review Edited by Farrell Till
A Complaint from England...

I'm returning your literature because I think you should be told that the way in which I was
"given" it wasn't correct. I'm a Christian, who had a little artistic display on Noah's ark at a
local library in England.

Everyone has the right to believe what they [sic] choose to believe and support. My complaint
to you is how one of your supporters "invaded" my display like an immboceil [sic], a really
pathetic act like an immature child. So an eye for an eye, you can have your literature back, I
could have thrown it in the bin but I wouldn't be bothered.

Where are your values in "playing it by the book"? If your supporters didn't like seeing an
exhibition on Noah's ark, why didn't they book themselves a display on what they believe in--
do their own display?

How low will you stoop?

(Jennifer Cowin, no address given)

EDITOR'S NOTE: This letter arrived with a postmark from Liverpool, England, but no return
address. With the letter were copies of some articles that had been copied from Douglas E.
Krueger's web page, which has republished with my permission some articles from The
Skeptical Review. The articles were the 1997 series on family values and "Jacob an Old
Geezer?" from the November/December 1996 issue. I, of course, have no idea who may have
left these articles at Ms. Cowin's display, but I do have some opinions about her complaint.

First, she shows that she is just another Christian who wants the right to publicize her beliefs
but objects to those with opposing views who exercise the same right. In all probability, the
library that she referred to is a public institution funded by tax money, so she evidently thinks
that it is appropriate for taxpayers to subsidize her religious beliefs. I heartily disagree with
this view, but it is one that has become deeply ingrained both here and abroad. If Ms. Cowin
wants to exhibit an artistic display about an absurd biblical story, she should have the right to
do it, but she should be willing to do so at her own expense and not expect to be subsidized by
public tax money.

Another interesting thing about her letter, which further confirms that Ms. Cowin seems to
think that British taxpayers should subsidize her religious beliefs, was that the letter came to
me without any postage affixed. Under my name in the address, she had written FREEPOST,
and the same was written in the upper righthand corner where postage stamps are normally
put, so this little bundle apparently came to me all the way from England by airmail at no
expense to Ms. Cowin. The cost of mailing was paid by the taxpayers of England and the
United States. I'm not familiar with whatever English postal regulation entitled Ms. Cowin to
send her package in this way, but if any subscribers in the United Kingdom read this column,
I would appreciate any information that you could give to me about "freepost" rights in
England. I have taken the envelope to the local postmaster and requested an investigation of
this matter. If any postal laws were violated, I would like to see the matter pursued both here
and in England.

Volume 1990 - 2002 Issue


Page 1314 of 2049
Skeptical Review Edited by Farrell Till
At the end of her letter, she asked how low I will stoop, and I have an answer to her question.
I will not stoop so low as to send semi-anonymous complaints to people whom I disagree
with. When I make such complaints, I put not only my name on my letters but also my mailing
address. Ms. Cowin is apparently too insecure in her beliefs to allow me the opportunity to
respond directly to her complaint. Furthermore, if I wanted to complain to someone through
the mail, I would pay the cost myself and not expect taxpayers to bear the expense for me.

Observations about the Daniel Series...

I confine myself mostly to developing new books and booklets in the fight against
fundamentalist nonsense and its twin, creation science, and my schedule is full. I can barely
imagine the work that goes into keeping TSR a topnotch periodical. You have done an
outstanding job! Every bit of praise you receive from your readers is deserved, and I enjoy
hearing from them almost as much as you do.

A couple of points can be added to your fine installment on Daniel. First, Belshazzar was
never a king, though clearly depicted as such in the Bible. Certain ceremonies could not be
held because only the legitimate king could hold them, and Nabonidus was away for some
years. Several sources report that Nabonidus had returned a few months before the final
defeat--only to die in a brief battle not far from Babylon. Thus, Belshazzar was not only never
a real king, but the main defeat of Babylonia did not involve him as suggested by the Bible.
His capture was more of a mop-up operation, in part because of Nabonidus' prior defeat in the
field and because the populace of Babylon saw no point in putting up resistance. Indeed, they
may have preferred Cyrus to their own government! I gather that Nabonidus was not very
popular.

Second, even if we were to take away all those fine points you made about Belshazzar not
being the son of Nebuchadnezzar, you would still have the better argument! Proper translation
demands that we stick with the most common meaning for a word unless driven to secondary
meanings by force of evidence. It doesn't matter that father or son might have 16 different
meanings. The obvious meaning (the common meaning) for each must be used in a good
translation unless the context clearly points to a secondary meaning. Anyone who reads the
passages you presented, assuming at least average intelligence, can see for himself that there
is hardly a hint for any other meaning. Clearly, father means father in the usual sense; son
means son in the usual sense. There is no conflict with the context in reading it that way, and
that is how it must be read. There is no escape--short of violating the principles of good
translation.

Dave Matson, Editor, The Oak Hill Free Press, P.O. 61274, Pasadena, CA 91116; e-mail
103514.3640@compuserve.com)

EDITOR'S NOTE: The principle of literary interpretation that Matson explained in his last
paragraph is taught in both hermeneutics and literary interpretation courses. Inerrantists like
Everette Hatcher ignore it only because acceptance of it will force them to admit that the
writer of Daniel made a historical error in referring to Nebuchadnezzar and Belshazzar as
father and son.

Volume 1990 - 2002 Issue


Page 1315 of 2049
Skeptical Review Edited by Farrell Till
Hovind-Till Debate...

My wife and I once viewed the Hovind-Till debate and we were surprised at how Hovind
almost completely ignored the topic of the debate, which was the Genesis Flood. While you
remained focused on the flood account, he continually expounded upon the flaws of
evolution. We have found that Hovind is like many fundamental Christians in that he simply
can not focus on any particular issue. By briefly touching upon a plethora of subjects, such
individuals are basically freed of going into the details of those subjects. That saying "the
devil is in the details" has some truth in that these people view reality as the devil.

(Christopher Boyce, 92-9 Chinkapin Drive, Stephens City, VA 22655; e-mail


orca@shentel.net)

EDITOR'S NOTE: What Mr. Boyce called "touching upon a plethora of subjects" is a typical
debating strategy used by creationists and fundamentalists. Debaters call it "stacking" or
"packing," which is the tactic of filling speeches with unsupported assertions and
generalizations that a creationist or biblical fundamentalist knows that an audience that is
predominantly Christian will be willing to accept without evidence to support it. A theistic
debater who asserts that design in the universe proves that there is a great designer knows
that most of his audience will accept this claim without supporting evidence, and so he can
say this and quickly go on to another assertion and then another and then another. In the
space of 30 minutes such a speaker can "stack" or "pack" into his speech 20 or 30 such
assertions, any one of which his opponent would need several minutes to refute. When the
opponent fails to address all such assertions, which will inevitably happen, the
creationist/fundamentalist will complain that his "arguments" are not being answered.

I have yet to debate an opponent who did not use this tactic to some degree. In the space of
just 20 minutes, for example, Dr. Norman Geisler put over 20 unsupported assertions into his
first speech at our debate at Columbus (Georgia) College in 1994 and then predictably
complained that I had not "addressed" most of his "lines of evidence." Since Geisler read
previously prepared manuscripts every time he spoke, he had to have put this comment into
his speech before the debate had even begun. In other words, Geisler knew that this was a
complaint that he could make, because he knew that I would not be able in just 20 minutes to
respond to over 20 unsupported assertions. He knew too that a predominantly Christian
audience would not complain that he had not really proven any of his assertions. After all,
how many Christians bother to ask for evidence before they become Christians? If it were in
the nature of Christians to insist upon evidence, they wouldn't be Christians in the first place.

To control "stacking" in debates I participate in, I have adopted a policy of demanding a rule
that will limit the affirmant of a proposition to just one argument per speech. In such a
format, the affirmant may present as much supporting material as he wants to within an
allotted time, but if he does not use that time, he must yield the floor to his opponent without
beginning a new argument. The negative speaker in turn will respond only to this one
argument within the specified time or else yield the floor back to his opponent. I have found
no one who will agree to this type of format, so obviously those on the side of theism and
biblicism want to retain the right to bombard their opponents with more assertions than they
can possibly respond to.

Volume 1990 - 2002 Issue


Page 1316 of 2049
Skeptical Review Edited by Farrell Till
Was Hatcher Misrepresented?

Having been quite impressed with your work (especially your humor and fairness) for several
years now, I feel compelled to call your attention to two instances where you have written
inaccurately about claims made by Everette Hatcher. And please understand that my views on
religion are the same as yours and thus in diametric opposition to those of Mr. Hatcher.

(1) TSR, March/April 1998, p. 7, you accuse Hatcher of misrepresenting the views of Norman
Porteous by claiming Porteous as an advocate of 6th century BCE authorship of Daniel, ^but^
on p. 2 (middle column) Hatcher calls Porteous a "Bible critic" who questions only one small
item about Daniel. Hatcher has never tried to pass Porteous off as anything other than a
proponent of the 2nd century BCE authorship view.

(2) TSR, July/August 1998, p. 14, you again accuse Hatcher of purposely leaving the
impression that certain scholars favor the 6th century BCE view. Knowing Everette Hatcher
as I do, I can state unequivocally that he is intellectually honest and would do no such thing.
Hatcher respects your scholarship and broad knowledge of the Bible and has stated these
views to me on three different occasions.

Please continue your instructive work.

(John George, College of Liberal Arts, University of Central Oklahoma, Department of


Political Science, 100 North University Drive, Edmond, OK 73034-5209)

EDITOR'S NOTE: In the June/ July 1998 issue, I explained that Everette Hatcher had
informed me during a phone conversation that I had misunderstood his intentions, because he
was not trying "to leave the impression that scholars like H. H. Rowley, Samuel Driver, and
Norman Porteous were advocates of a 6th-century B. C. authorship of Daniel" but was
claiming only that they "had made some admissions that were damaging to their position that
this book was written in the 2nd century B. C., during the Maccabean era" (p. 6). I went on to
say that after having reread Hatcher's article, I had noticed some sections "that could be so
interpreted." I noted, however, that Hatcher did at other times leave the impression that
"these scholars were on his side" but that I was "willing to take his word for it" if he claimed
that his intention was not to misrepresent. That issue, then, has already been settled, but I do
think that in future articles, Hatcher should be more careful in his citation of authorities. One
thing that he may want to keep in mind is that it isn't a good idea to quote just a brief
fragment of an author's statement, without giving the full context of the statement, especially
when the full context would clearly show a position that is contrary to the one that is being
argued. This tactic is so widespread in the literature of biblical fundamentalism that an
apologist with honest intentions who quotes only fragments and snatches from his sources
will run the risk of having his readers assume that they are seeing just another inerrantist
attempt to misrepresent.

Even Hatcher should realize this risk, because he has sent to me articles and published letters
that he has written to biblical fundamentalists like Tim LaHaye and Dr. James Kennedy in
which he took them to task for quoting out of context and even falsifying quotations in efforts
to make Bible-believing Christians out of so-called "founding fathers" like George

Volume 1990 - 2002 Issue


Page 1317 of 2049
Skeptical Review Edited by Farrell Till
Washington, John Adams, James Madison, and Thomas Jefferson. He should be aware, then,
of the danger of being misunderstood when fragmented quotations are lifted from a larger
context as support for a position that the quoted author does not himself defend.

My contacts with Professor John George and Everette Hatcher since the discussions of the
book of Daniel began have altered significantly my opinion of Mr. Hatcher. I have seen
enough of his letters to biblical fundamentalists to see that even though he is himself a
biblical inerrantist, he deplores the dishonest methods that many inerrantists use in defense of
their positions. What I have seen has, in fact, given me hope that Hatcher may some day see
that biblical inerrancy is a position that cannot be sustained even by honest methods of
argumentation. The recognition that there is no real evidence to support their position is
probably why so many inerrantists resort to dishonest apologetic methods.

Although the religious beliefs of the so-called founding fathers is not an issue that TSR
discusses, I will take the time to mention that Everette Hatcher sent to me an excellent
manuscript ("Misquotes, Fake Quotes, and Disputed Quotes of the Founders") on the subject.
It exposes the misrepresentations and distortions found in the works of Christian
fundamentalists who are trying hard to make their readers believe that men like Jefferson,
Adams, Washington, Madison, etc. were zealous, Bible-believing Christians. Those interested
in seeing the manuscript should contact Hatcher at P. O. Box 23416, Little Rock, AR 72221.

I have also learned that Hatcher and Professor George have worked together on this issue
and that George and Paul Boller, Jr., co-authored *They Never Said It,* a book published by
Oxford University Press, which exposes many misquotations that Christian fundamentalists,
in their zeal to make the United States a nation founded on biblical principles, have attributed
to the "founding fathers." Paul F. Boller, Jr., is a historian at Texas Christian University,
whose book George Washington & Religion (Southern University Press, 1963) demolished
the myth that Washington was a devout Christian.

Expiration Notices....

I have no idea whether my subscription has expired or not since your publication comes out
so infrequently. Would it be too much trouble for you to add the expiration date on the
address label so that we will know when to expect a renewal date?

(John C. Parker, 244 Williams Street, Meriden, CT 06450-4515)

EDITOR'S NOTE: The expiration date is printed on address labels and has been for two
years. The back page of each issue has had a notice entitled "Check Your Address Label." The
notice on page 11 of this issue explains that if 6-98 is printed on your label, then this will be
the last issue in your current subscription. In the past, I have also published notices to explain
how to interpret the numbers on the label. The last two numbers are the year of publication,
and the first number, which will always range from 1 to 6, is the number of the issue within
that year of publication. Hence, 6-98 means that the subscription expires with the 6th issue of
1998, which is the issue you are reading, 2-99 would mean that the subscription expires with
the second issue of 1999, and 3-00 would mean that the subscription expires with the 3rd
issue of 2000 (if I last that long).

Volume 1990 - 2002 Issue


Page 1318 of 2049
Skeptical Review Edited by Farrell Till
As for the frequency of publication, TSR is published bimonthly, and the scheduled mailing
dates are the first weeks of January, March, May, July, September, and November. Although
a few issues have been mailed late, TSR almost always goes out on schedule.

Pure Violence and Smut...

The letter from Reynolds Jones in the September/October edition of The Skeptical Review
baffles me. He admits the bible is not inerrant, or even inspired, yet he still thinks it's sacred.

Jones realizes that the book is neither perfect historically or philosophically and that it's not
internally tangent. He even refers to the "outrageous" scriptures and says the document is a
human one. Jones realizes that we atheists are as moral as Christians are. He also appears to
see how the bible was concocted to fit the dogmas that were already determined. As my poem
says, "The silly dogmas they already had in mind."

Indeed, Mr. Jones sees that fundamentalists actually idolize the bible, and he understands that
even the prophecies of Jesus weren't always fulfilled. For example, he told his followers that
they would not die before he returned to earth, yet, realizing all these things about the bible on
which Christianity is based, Mr. Jones says he still is active in church and attends Eucharist
regularly. How can Mr. Jones realize all those other things and not know that the Eucharist,
known by ancients as "eating the god," was stolen from other, much older religions. The
ancient idea was that by eating the flesh of a particular god, a follower was able to take on the
attributes of that god.

Mr. Jones speaks of his hope that all Christians will leave damaging anachronisms behind and
together build "a just and all-embracing community" that takes pro-active stances on the
issues of the next century, such as science and human and animal rights.

My response to that is dream on, Mr. Jones. That "sacred" book, the bible, is a conglomerate
book of blood-drenched atrocity. It considers women unclean and subhuman. It calls for death
for everything from getting raped in the city to eating blood, and 10,000 things in between.
The book calls for war after war in which the men are killed and their foreskins collected,
women and their children are killed, and only virgins are kept alive as sexual booty. It calls
for death of homosexuals.

The bible is pure violence and smut. The so-called ten commandments have no mention of
prohibiting rape, child abuse, unkindness, wife-beating, torture, etc. Just don't covet. Sacred,
you say? Nonsense!

(Dr. Dorothy B. Thompson, P. O. Box 562, Bandon, OR 97411)

EDITOR'S NOTE: I too can't understand why people who see the obvious myths and
superstitions in the Bible would continue the churchgoing habit. I suspect that this is done
primarily to socialize rather than to worship a god whose existence is very much in question.
One of the failings of skepticism/atheism is that it provides no comparable opportunity for
socialization, but we are probably too scattered, and certainly too unorganized to do so.

Volume 1990 - 2002 Issue


Page 1319 of 2049
Skeptical Review Edited by Farrell Till
Some readers may remember seeing examples of Dr. Thompson's religious satire in poetic
form that has been published in the mailbag column and other freethought papers. I have just
received a complimentary copy of a booklet of her poetry. Those interested in this type of
freethought literature should contact Leland W. Ruble at 833 Orchard Street, Toledo, OH
43609.

A Familiar Story...

As a former fundamentalist Christian turned skeptic, I continue debating and talking to my


Christian friends and family. When I present to them reasons for my unbelief, they will
invariably evade these reasons and postulate that some internal condition is to blame for my
current skepticism. In other words, they will accuse me of being angry with God because he's
not who I want him to be, or they will say I never truly understood what salvation, i. e., being
"born again," means, or they will claim that in the past I had some bad experiences that turned
me away from God. In most cases, this evasion is not purposeful, nor is it maliciously done;
instead, I believe that these people just cannot accept that someone actually left the faith
because of a rational examination of the claims of the Bible.

Others accuse me of leaving God because I harbored sin in my life, or they theorize that I've
allowed myself to be "brainwashed" by "the world" because I was not grounded firmly
enough in the Bible and in fellowship with God's people. Such evasions exasperate me,
especially when I try to redirect the topic back to my rational objections to the inerrancy of
the Bible. Most fundamentalist Christians just do not want to be bothered with the facts.

My more intellectual Christian friends (who insist they are not fundamentalist but
evangelical) try to "counsel" me to open my heart to God, give him my anger and doubt, and
he will open my eyes. "Seek and ye shall find," they firmly believe, and God will finally
touch my heart. But they insist I've got to rid myself of my anger and pride first.

Few of these people, no matter how intelligent, can imagine a life outside Christianity. That is
why they continually insist that the problem must be within me. But realizing how much
better my life has been since I cast off the chains of dogma, I continue to try to persuade my
Christian friends and family to listen and honestly think about the issues I raise.

That is why I am writing this letter to you. I am requesting first-year subscriptions to The
Skeptical Review for two of my friends whom I believe may be open to critical thought. Both
are in the ministry and both are committed bibliolaters. (I have their names and addresses
listed below.) Because I believe so firmly in your work, I wish to pay for their first-year
subscriptions. You have my permission to let them know I requested these subscriptions.

I also would like to purchase the videotape of the Till-Dobbs Debate. Enclosed is [a check]
for the subscriptions and the tape. You have my permission to apply anything left over to
other free subscriptions, overhead, etc.

(Mark Wenneborg, 2729 South Fourth Street, Apt. A, Springfield, IL 62703; e-mail,
lunadrach@aol.com)

Volume 1990 - 2002 Issue


Page 1320 of 2049
Skeptical Review Edited by Farrell Till
EDITOR'S NOTE: Probably any ex-Christian reader who has tried to discuss the Bible with
friends and relatives who are still believers could have written this letter, because it tells an
all-too-familiar story. It sounded just like a page out of my own past. I don't know how many
times people have asked me what terrible thing happened that made me hate God (as if I
could hate something that I don't believe even exists), or why I don't open my heart and let
God lead me back to the truth. The fact is that nothing "terrible" has ever happened to me. My
grandfathers died not long after my "conversion," but they were both 75 at the time, so there
was no reason to consider their deaths as horrible tragedies. My father died much younger,
but at the time, I had already decided that the Bible was not at all what I had grown up
believing. I have had a life rather free of events that could be called tragedies, yet friends and
associates of mine who remain Christians seem convinced that something "terrible" happened
that I am not being honest about. As Mr. Wenneborg noted above, people with this mindset
just can't accept the possibility that a person would rationally examine the Bible and conclude
that it is not "the inspired word of God," and so they have to find some way to explain people
like Wenneborg and me. Thus, they conclude that we have experienced some untold tragedy
or disappointment or that we were unable to live up to the standards of biblical morality, and
so we claim not to believe the Bible in order to have some justification for living "ungodly
lives."

I was glad that Mr. Wenneborg agreed to let me inform his friends that he requested the
subscriptions to TSR that they will receive. Many times, Christians who are added to the
mailing list at the request of friends who want them to read TSR will write or call in anger
demanding that their names be removed. Those who know that friends or acquaintances made
the requests on their behalf are less likely to do this.

Another Reaction to Archer's Letter...

This is just a note to tell you that I received another issue of The Skeptical Review, and I want
to thank you because I enjoy it very much. Your arguments are sound, your knowledge of the
subject is astounding and it is always a relief to be reminded that there is nothing wrong with
me just because that book doesn't make any sense to me. It doesn't make any sense.

I particularly wanted to comment on that message from Gleason Archer. You are obviously a
very honest person who works hard to help other people come to an understanding of the
world. I admire that in you and you do a very good job. Thanks again for the help that you
provide many people and I hope to support your activities by encouraging you and by
providing what information I can to help other people.

(Lisa Star, P. O. Box 428, Glen Echo, MD 20812-0428; e-mail, LisaStar@earthling.net)

Textual Criticism & Primary Colors...

I just reread the "Primary Colors" article in the July/August issue of TSR, and it occurred to
me that there is perhaps an even better example of "literary sleuthing" that could be
illustrated. That would be the case of the Unabomber. When the Unabomber's manifesto was
printed in the Washington Post, it was read by the younger brother of Theodore Kaczynski.
After comparing the manifesto to an assortment of Ted Kaczynski's old college essays and

Volume 1990 - 2002 Issue


Page 1321 of 2049
Skeptical Review Edited by Farrell Till
other writings, he found compelling similarities in both theme and word usage, so compelling,
in fact, that he used his own money to finance a private investigation and later reported his
hunch to the FBI. The end result, as we all know, was the arrest of Theodore Kaczynski. All
this from the art of textual analysis! Could any Christian now say that this is a fringe or
invalid science?

An interesting footnote to this, by the way, is that the Unabomb task force in the FBI wanted
the document published just as much as the Unabomber did. They hoped that someone would
recognize some of the Unabomber's quirky sentence structuring and repetitive "anti-
technology" themes as familiar. Their theory, and their faith in "textual fingerprinting," paid
off quite well.

I also have a question that I was hoping you could answer. I have just read The Age of Reason
by Thomas Paine for the first time. In it, he claims that the word "prophet," when used
throughout the Bible, is a mistranslation which should read "poet" in almost all cases. This
makes sense, and would be delightful if true, but Paine doesn't indicate how he knows this.
Since I've never seen this claim discussed before, I hoped you may know something about it
and could back up Paine's statement.

Thank you, and keep up the excellent work.

(Chad Kelly, P.O. Box 183 Greenwell Springs, LA 70739-0183; e-mail, constat@ibm.net)

EDITOR'S NOTE: The case of the unabomber is an excellent example of the reliability of
"literary sleuthing," so those who dismiss textual criticism of the Bible as just liberal efforts to
discredit the "word of God" have their heads in the sand. The validity of the methods used by
textual critics is widely recognized, and even biblicists will use them when it is to their
advantage to do so. Different terms were used in the Old Testament to designate "prophet,"
but the most commonly used one was nabi, which lexicons define as a word derived from a
root that meant "to call," and so a prophet was "one who was called." The term didn't
actually denote the idea of "prediction," but since those who were "called" (prophets) so often
engaged in predicting what would "come to pass," the word came to be associated with
foretelling the future. Many of the prophets wrote in poetic form, and so maybe this is why
Paine considered them poets.

Volume 1990 - 2002 Issue


Page 1322 of 2049
Skeptical Review Edited by Farrell Till

The Skeptical Review


Volume 10 – 1999
Farrell Till, Editor

• January/Feburary Volume Ten, Issue One


• March/April Volume Ten, Issue Two
• May/June Volume Ten, Issue Three
• July/August Volume Ten, Issue Four
• September/October Volume Ten, Iss Fiveue
• November/December Volume Ten, Issue Six

Volume 1990 - 2002 Issue


Page 1323 of 2049
Skeptical Review Edited by Farrell Till

Skeptical Review
Volume Ten, Number One
January/February 1999
Farrell Till, editor

• "Who's to Blame?"
The editor takes issue with those who claim that Christianity can't be blamed for the
actions of fanatics who murder abortion doctors and discriminate against and abuse
gays.
• "A Legend in His Own Time"
Farrell Till debunks the claim of a "modern Jonah" who survived being swallowed
alive by a whale.
• "Doubts But Questions about Prophecy"
Bruce Weston, a skeptical Christian, explains his reasons for being unable to abandon
entirely his biblical beliefs.
• "Where to Begin?"
Farrell Till explains why Weston's reasons for retaining some of his biblically based
beliefs are unwarranted.
• "The Sons of God and the 'Daughters of Men'"
Farrell Till ties Hebrew legend to the reference to the "sons of God and the daughters
of men" in Genesis 6:1-2.
• "From the Mailbag"
Readers and editor Farrell Till discuss a variety of religious issues.

Who's to Blame?

Volume 1990 - 2002 Issue


Page 1324 of 2049
Skeptical Review Edited by Farrell Till
When skeptics refer to the inquisitions, witch hunts, and other persecutions that Christianity
has left in its wake as reasons to oppose the religious right's intrusion into civil and political
affairs, many Christians will argue that those who were responsible for such atrocities as these
were not "real Christians." They do the same when terrorist acts are committed by those who
have links with organizations that have biblical terms like "God," "Yahweh," and "Phinehas"
in their names. We have seen this most recently in the assassination of Dr. Barnett Slepian
and the beating death of Matthew Shepard. Christians piously condemn these actions (while
silently approving them, we suspect) and declare that Christianity cannot be blamed for the
violent activities of radicals.

Christianity is not to blame? Then just who is to blame? Christianity is based on a book that
depicts racism, brutality, massacre, and homophobia as the will of the very god Christians
worship. If Bible-believers read in their "word of God" that Yahweh chose one nation to be
"his own possession above all peoples that are on the face of the earth" (Dt. 7:6), why should
it be surprising that some of these would see this as justification for preaching racial
superiority, which necessarily entails the preaching of racial inferiority? If Bible-believers
read in their holy book that their god many times commanded his "chosen ones" to
exterminate entire tribes and nations (Dt. 20:16; Josh. 11:20; 1 Sam. 15:1-3), why should it
surprise anyone that armies comprised of men who believe the Bible is "God's word" would
engage in "ethnic cleansing"? In such cases, the very things that these people are doing were
in times past approved by the god they worship. Well, of course, any rational person knows
that there was really no deity ordering such atrocities as these. The people who wrote the
books of the Bible merely thought that their god was so commanding them, because they were
barbaric people living in barbaric times, who had created their god in their own image, but try
telling that to someone who has been indoctrinated all of his/her life to believe that the Bible
is "the word of God."

In an article entitled "Don't Blame Pulpits for a Few Irrational Acts," Michael Miller,
religious editor of the Peoria (Illinois) Journal Star, said in reference to the murders of
Matthew Shepard and Dr. Barnett Slepian, "The people who blame these acts of violence on
the anti-abortion movement and on those who preach that homosexuality is wrong have no
evidence whatsoever to back up their claim" (Saturday, December 12, 1998, p. E5). Oh,
really? Mr. Miller sounds like the type who would say that the inquisitions, persecutions, and
witch hunts of the past cannot be blamed on those who preached that heresy and witchcraft
were wrong. Such simplistic reasoning as this ignores the obvious fact that hatred of this kind
begins with religion and is fanned by religion, and more than any factor, the direction that
religion takes is determined by what its leaders preach.

Why would anyone preach against witchcraft in the first place? Could it possibly be that
witchcraft was condemned from pulpits because preachers knew that the Bible said, "Thou
shalt not suffer a witch to live" (Ex. 22:18), and so they preached what their inspired word of
God said. In a time when superstitious people thought that witches were real and witchcraft
was being condemned from pulpits, is it any wonder that a colony under the control of a
government of Bible believers would have allowed the Salem witch trials to take place and 20
people to be killed and many others imprisoned for a sin that the Bible considered worthy of
death? To argue that such atrocities as these should not be blamed on "pulpits" is absurd.
Cotton Mather was probably the most influential clergymen in New England at the time of the

Volume 1990 - 2002 Issue


Page 1325 of 2049
Skeptical Review Edited by Farrell Till
Salem witch hunt, and anyone who reads his book Wonders of the Invisible World will see
that he approved of both the trials and the hangings that followed. One would have to be
hopelessly naive to think that what a preacher of his stature said from the pulpit had no
influence on events like the Salem witch trials. In the same way, it is ridiculous to think that
when preachers today rant and rave against abortion and homosexuality, they are completely
blameless when someone decides to give God a helping hand by planting bombs in abortion
clinics and gay bars.

If Christianity is not to blame for the activities of radical elements in our society, then why is
it that these elements are almost always associated with organizations that have Bible-based
beliefs? When has the bombing of a public or private building ever been traced to a
humanistic or atheistic organization? When has the assassination of an abortion doctor ever
been traced to an atheist or religious skeptic? Somehow it always seems to turn out that the
perpetrators of atrocities like these have connections with Christian organizations and the
buildings that were targeted for bombing were used for activities that are openly condemned
from Christian pulpits? Are we to think that all of these connections were purely coincidental?

In matters of public morality, it seems that Christians want to play both sides of the street.
They will point to the good that is being done by Christian organizations and people engaged
in charitable and humanitarian work and say, "See all of the good that is being done by those
who follow what the Bible teaches!" I have yet to hear a preacher or the religious editor of a
newspaper say, "Don't credit pulpits for the good works being done by those who do what is
preached in our churches," but when Fred Phelps and his Kansas-based Baptist flock parade
around the country with their God Hates Fags signs and even insensitively picket the funeral
of a gay man who was brutally beaten to death by homophobics or when a Paul Hill shoots an
abortion doctor in cold-blooded murder, Christian leaders quickly rise up and scream, "Don't
blame us! These are the violent acts of radicals." Well, they can't have it both ways. If they
are going to take the credit for the good that comes from preaching that people should love
their neighbors and help those in need, they should take the blame that comes from preaching
that focuses on the seamier aspects of the Bible. Some preachers see love, mercy, and
kindness taught in the Bible and teach their congregations to practice the same, but preachers
like Fred Phelps see where the Bible says that homosexuals should be killed (Lev. 20:13;
Rom. 1:27, 32) and think that this too should be preached. They're not responsible when
someone takes them seriously enough actually do what the Bible says? Why aren't they?

A Legend in His Own Time


Farrell Till

In the last issue of TSR, we discussed the myth about astronomical proof of Joshua's long day
that biblicists still recycle from time to time even though it has been repeatedly discredited

Volume 1990 - 2002 Issue


Page 1326 of 2049
Skeptical Review Edited by Farrell Till
and even rejected by scientifically informed inerrantists. As the mailbag column in this issue
shows, these articles created a lot of interest in the subject, an interest that is no doubt partly
due to the attention that this myth has received on the internet, where naive biblicists continue
to cite it as proof of biblical inerrancy. I mentioned in my article that I recalled encountering
this myth for the first time when as a Bible college student in the 50s, I dutifully read Harry
Rimmer's Harmony of Science and Scripture. Rimmer was a Baptist preacher whose stock in
trade was defending the Bible with appeals to pseudoscience. He was sort of the Josh
McDowell of that time, and no serious ministerial student was without at least some of
Rimmer's books in his personal library.

Over the years, I lost Rimmer's Harmony of Science and Scripture, but Don Robertson, a
subscriber from South Carolina, saw my reference to it and sent me some photocopied pages
from the chapter that presented the version of this urban myth that was circulating in the 50s.
Now that we have seen the latest "NASA" version of the myth, looking at the version that
Rimmer told in his book (copyrighted in 1936) should help even staunch biblicists see the
unreliability of the kind of information that their "apologists" use to defend the Bible. The
quotation is a bit long, but I'm going to publish it uncut so that no one can accuse me of
misrepresenting Rimmer.

The final testimony of science is that such a day [Joshua's long day] left its record for all time.
As long as time shall be, the record of this long day must remain. The fact is attested by
eminent men of science, two of whom I quote here.
Sir Edwin Ball, the great British astronomer, found that twenty-four hours had been lost out
of solar time. Where did that go, what was the cause of this strange lapse, and how did it
happen? The answer may be expected in vain from sources of human wisdom and learning!
There is a place, however, where the answer is found. And this place is attested by a scientist
of standing. There is a book by Prof. C. A. Totten of Yale, written in 1890, which establishes
the case beyond the shadow of a doubt. The condensed account of his book, briefly
summarized, is as follows:
Professor Totten wrote of a fellow-professor, an accomplished astronomer, who made the
strange discovery that the earth was twenty-four hours out of schedule! That is to say, there
had been twenty-four hours lost out of time. In discussing this point with his fellow-
professors, Professor Totten challenged this man to investigate the question of the inspiration
of the Bible. He said, "You do not believe the Bible to be the Word of God, and I do. Now
here is a fine opportunity to prove whether or not the Bible is inspired. You begin to read at
the very beginning and read as far as need be, and see if the Bible can account for your
missing time."
The astronomer accepted the challenge and began to read. Some time later, when the two men
chanced to meet on the campus, Professor Totten asked his friend if he had proved the
question to his satisfaction. His colleague replied, "I believe I have definitely proved that the
Bible is not the Word of God. In the tenth chapter of Joshua, I found the missing twenty-four
hours accounted for. Then I went back and checked up on my figures, and found that at the
time of Joshua there were only twenty-three hours and twenty minutes lost. If the Bible made
a mistake of forty minutes, it is not the Word of God!"
Professor Totten said, "You are right, in part at least. But does the Bible say that a whole day
was lost at the time of Joshua?" So they looked and saw that the text said, "about the space of
a whole day."

Volume 1990 - 2002 Issue


Page 1327 of 2049
Skeptical Review Edited by Farrell Till
The word "about" changed the whole situation, and the astronomer took up his reading again.
He read on until he came to the thirty-eighth chapter of the prophet Isaiah. In this chapter,
Isaiah has left us the thrilling story of the king, Hezekiah, who was sick unto death. In
response to his prayer, God promised to add fifteen more years to his life. To confirm the
truth of His promise, God offered a sign. He said, "Go out in the court and look at the sundial
of Ahaz. I will make the shadow on the sundial back up ten degrees!" Isaiah recounts that the
king looked, and while he looked, the shadow turned backward ten degrees, by which ten
degrees it had already gone down! This settles the case, for ten degrees on the sundial is forty
minutes on the face of the clock! So the accuracy of the Book was established to the
satisfaction of this exacting critic.
When the astronomer found his day of missing time thus accounted for, he laid down the
Book and worshipped its Writer, saying, "Lord, I believe!" (Harmony of Science and
Scripture, Books, Inc., 1960, pp. 236-238).

I naively lapped up such stuff as this in my Bible college days. I'm embarrassed to admit it,
because Rimmer's story almost reeks with the smell of phoniness. In his rebuttal of the NASA
version of this myth, Charles Brennecke explained why finding a "missing day" in the time of
Joshua would not be possible, but the scientifically impossible claims in the story are not the
only reasons to doubt it. Professor Totten, the source of Rimmer's version of the myth,
apparently didn't bother to give the name of his "fellow-professor," who was identified only
as an "accomplished astronomer," so there was no way that Rimmer's readers could have
checked the accuracy of Totten's claim that a professor's quest to find a missing day in time
had converted him from skeptic to believer. I'm always suspicious of tales about unnamed
skeptics who are instantly converted without even taking the time to evaluate whatever it was
that presumably impressed them so profoundly. I would say that Totten's "fellow-professor"
could not have been much of an "accomplished astronomer"--and certainly not an
accomplished skeptic--if immediately after reading the tale about Hezekiah's sundial, he laid
the Bible down and said, "Lord, I believe!" This sounds too much like those apologetic yarns
about the atheistic professor who was silenced in his tracks by a simplistic question that any
informed skeptic could easily answer, but pulpit audiences eat up this kind of stuff. That's
why preachers use it.

Furthermore, Rimmer, like his present-day counterparts, didn't even try to explain how the
"professor" knew that exactly 23 hours and 20 minutes were missing in the time of Joshua. He
simply said that the professor "checked up on [his] figures" and made this determination.
Exactly what calculations did the professor make, and what "figures" did he check? How was
he even able to make any calculations at all without knowing the exact orbital positions of the
earth and moon before and after the alleged long day? It's incredible that biblicists would
continue to circulate this yarn without even investigating to see if existing scientific
information can even establish its truth.

In many respects, however, the versions of this tale are alike. Rimmer tried to give his
account respectability by attributing the quest for a missing day in time to a professor at Yale
and a "fellow-professor" who was an "accomplished astronomer." Harold Hill adapted his
account to the space program and made NASA scientists the counterparts of the professor at
Yale and his friend, the "accomplished astronomer," but in both accounts, "scientists," who
would presumably be trustworthy people qualified to know, had discovered a missing day in

Volume 1990 - 2002 Issue


Page 1328 of 2049
Skeptical Review Edited by Farrell Till
time and then found the explanation for it in the Bible. The end result in the minds of gullible
inerrantists would be that science has confirmed the truth of the biblical account of Joshua's
long day. Never mind that the information in either version of the story was insufficient to
establish the truth of a myth that presumably proves the biblical myth. What biblical
inerrantists are going to know enough about science to see holes in the story? Those who do
know enough about science to see flaws in it (like the staff members of Apologetics Press,
Inc., mentioned in my earlier article on the subject) reject the myth and advise others not to
use it as an apologetic argument, but that Christians who have no background in science
would be so quick to believe such a tale as this should not be surprising. After all, their whole
religious lives have been based on a believe-anything approach to the Bible.

Everette Hatcher, whose defense of the book of Daniel will continue in the next issue,
apparently recognizes the phoniness of at least some of the information that Christian
apologists use to defend biblical inerrancy, because he has sent to me an article from
Perspectives on Science & Christian Faith that debunks another "scientific" proof that
Rimmer used in Harmony of Science and Scripture. Rimmer claimed that the story of an
English sailor, who was swallowed by a "whale shark" and then recovered alive forty-eight
hours later, proves that the story of Jonah is scientifically possible. The article that Hatcher
sent to me ("A Whale of a Tale: Fundamentalist Fish Stories," December 4, 1991, pp. 224-
237) was written by Dr. Edward B. Davis, a professor of science and history at Messiah
College in Grantham, Pennsylvania. It relates the author's detailed research into Rimmer's fish
story, a tedious effort that took Davis into the archives of various libraries and newspapers on
both sides of the Atlantic over a period of several months before leading him to the
conclusion that the story was a legend that never happened. The article is fascinating reading
that I highly recommend, because it shows not only how legends begin and grow but how that
they can even become full blown within the lifetimes of the principal parties in them. This
latter point is important, because a popular argument that Christian apologists use in defense
of the resurrection of Jesus is that a minimum of four generations is necessary for a legend to
develop, because if it begins earlier than this, people who lived at the time of the event or
person being legendized would be able to nip the story in the bud by testifying that they were
alive at the time and place and knew nothing about the events and people in the legend.
According to the argument, the same would be true of the second- and third-generation
descendants of people who had lived at the time. They could kill the legend by testifying that
their parents and grandparents living at the time had never mentioned any events or persons
involved in it. This is a commonly heard apologetic argument, but Davis's research into
Rimmer's fish story shows that it is without basis, because legends can and do develop over
much shorter periods of time than four generations and sometimes even within the very
lifetimes of the principal parties in the legends. In this country, we have only to consider the
legends that developed around such frontier heroes and outlaws as Wyatt Earp, Wild Bill
Hickok, Buffalo Bill, Jesse James, Billy the Kid, etc. to know that legends can develop much
sooner than four generations. I could cite other examples of rapidly emerging legends, but at
this point I am not as interested in debunking the fundamentalist argument about legends as in
showing how that a particular legend about a man and a whale began and was used in a
pseudoscientific attempt to vindicate the Bible.

Davis became interested in Rimmer's modern Jonah when he inherited books from a relative
of his wife and found inside one of them a copy of a sermon that Harry Rimmer had preached

Volume 1990 - 2002 Issue


Page 1329 of 2049
Skeptical Review Edited by Farrell Till
on "Jonah and the Whale." Between the book pages, there were also sermons on "Noah's Ark
and the Deluge" and "Modern Science and the Long Day of Joshua," all of which Rimmer had
later included in Harmony of Science and the Scripture. Attached to Rimmer's sermon on
Jonah was a tract about Jonah by an unknown Fred T. Fuge and also an article from The
Moody Bible Institute Monthly (September 1930), which had been written by Professor
Albertus Pieters of Western Theological Seminary in Holland, Michigan, to explore the
possibility of a man's being able to survive for three days in the stomach of a whale. Pieters'
article cited some sources that I will refer to later, but first I will present the version of the
modern-day Jonah as Davis found it in Rimmer's sermon folded between the pages of the
book he had inherited.

In the Literary Digest we noticed an account of an English sailor who was swallowed by a
gigantic Rhinodon [i. e., a whale shark] in the English Channel. Briefly, the account stated
that in the attempt to harpoon one of these monstrous sharks this sailor fell overboard, and
before he could be picked up again, the shark, feeding, turned and engulfed him. His horrified
friends made so much outcry that they frightened the fish, and it sounded and disappeared.
The entire trawler fleet put out to hunt the fish down, and forty-eight hours after the incident
occurred the fish was sighted and slain with a one-pound deck-gun. The winches on the
trawlers were too light to haul up the body of the mighty denizen of the deep, so they towed
the carcass to the shore and opened it, to give the body of their friend Christian burial. But
when the shark was opened, they were amazed to find the man unconscious but alive! He was
rushed to the hospital, where he was found to be suffering from shock alone, and a few hours
later was discharged as being physically fit. The account concluded by saying that the man
was on exhibit in a London Museum at a shilling admittance fee; being advertised as "The
Jonah of the Twentieth Century."
We corresponded with our representatives in London, and shortly afterward received
corroboration of this incident, and last year had the privilege of meeting this man in person.
His physical appearance was odd, in that his entire body was devoid of hair, and odd patches
of a yellowish-brown color covered his entire skin (Davis, pp. 229-230).

Davis noted that Rimmer's account was characteristically lacking in details that would enable
readers to verify the account. He didn't give the name of the sailor even though he claimed
that he had later met him in person. He said only that the sailor "was on exhibit in a London
Museum at a shilling admittance fee," but he didn't give the name of the museum. He didn't
give the name of the town along the English Channel where the incident happened and the
sailor was later retrieved alive from the whale shark's stomach. These are all strange
omissions in a story that the writer obviously intended as scientific verification of the biblical
story of Jonah.

Rimmer did say that he had read the story in the Literary Digest, but he didn't give the date of
publication. Davis explained that the Literary Digest was a "popular magazine from the late
19th and early 20th centuries that was rather like a cross between Reader's Digest and
Newsweek" (p. 230). Davis told how that he was able to find every issue of Literary Digest
from 1916 through 1927, with the exception of "a few issues just before 1920," and look
through each one for the article that Rimmer had alluded to in his sermon. He was unable to
find "anything even remotely like the story Rimmer printed" (p. 230). Davis noted that in
Harmony of Science and Scripture, which was published after the sermon, Rimmer's version

Volume 1990 - 2002 Issue


Page 1330 of 2049
Skeptical Review Edited by Farrell Till
of the modern-Jonah story was identical to the one he had told in his sermon except that he
omitted the reference to Literary Digest and said only that he had read the story in "a
magazine devoted to current affairs" (p. 230). Davis speculated that Rimmer had originally
encountered a version of the story that claimed the Literary Digest as its source, and had
accepted it without verification. When he perhaps learned later that no such story had ever
appeared in Literary Digest, he changed the source reference in Harmony of Science and
Scripture to a vague, imprecise "magazine devoted to current affairs." The end result was that
the version in Harmony contained no specific information at all that readers could have used
to verify Rimmer's claim.

Rimmer's imprecision did not deter Davis's determination to verify the story. Fuge's tract,
which Davis had found folded in the book with Rimmer's sermon, told a similar story that the
author attributed to a book written by a missionary to Iceland named Arthur Cook. (Davis
later found this book and learned that the author's name was really Gook, not Cook.) This
version of the story contained some specific details. The sailor's name was James Bartley, and
he was allegedly a crew member of a whaling ship named Star of the East. The incident had
happened not in the English Channel, but off the coast of the Falkland Islands (in the
Southern Atlantic). The "fish" was not a "whale shark" but an actual whale, which, after being
harpooned, had sounded, resurfaced, thrashing in a fit of agony, and capsized one of the boats.
Before the crew of the other boat could pick up the men, one had drowned, and another
named James Bartley could not be found. The whale was towed to the ship, where the crew
worked the rest of the day and part of the night to butcher and process it. Work resumed the
next day, and when the crew had stripped away all the fat and flesh, the stomach was hoisted
onto the ship, where Bartley was found unconscious but still alive inside it. As in Rimmer's
version of the story, Bartley survived the horrifying incident.

Armed now with at least a few specific details, Davis set out to check the story for accuracy.
He surmised that if an incident like this had happened, it would probably have been picked up
and published by the New York Times. Neither Rimmer nor Fuge had given a date for the
alleged incident, but fortunately the article by Pieters (cited above) had given February 1891
as the date for a similar event alleged in it. Davis checked the Times Index for that year, found
a lot of references to whales, but nothing about an incident like the one claimed by Rimmer,
Fuge, and Pieters. He continued checking through the index for the next year and the next and
the next, until in the volume for 1896, he found the entry: "Whale; man swallowed by...." On
the microfilm for that year, he found the story, which was almost identical to the version he
had read in Fuge's tract but with an additional note that "The Mercury of South Yarmouth,
England, October 1891" was the source of the information. In going through the other issues
of the Times on the same microfilm roll, Davis found more entries that referred to the Bartley
story. A Harlem preacher, for example, had claimed that he had verified the existence of a
ship named The Star of the East, which was a barque of 734 tons that had been "built in
Glasgow, based in London, and commanded by a Captain J. B. Killam" (p. 226).

At this point, Davis had begun to feel that this story was perhaps true, but he wasn't quite
ready to declare it a proven fact. Shortly afterwards, when he received a grant from the
National Science Foundation to do research on another project in the library of the Royal
Society in London, he saw it as an opportunity to kill two birds with one stone. In addition to

Volume 1990 - 2002 Issue


Page 1331 of 2049
Skeptical Review Edited by Farrell Till
doing his research project for NSF, he would also try to verify the information he had so far
uncovered on the James Bartley incident.

Few can read this article and lay it aside without feeling a profound admiration for Dr. Davis's
tenacious commitment to thorough research. Not many would have the time, patience, or
means to put into research the effort that Davis expended in his search to find if the Bartley
story was true. In England, he encountered all sorts of problems in his quest to verify the
information he had found about Bartley in the United States. These problems are too
numerous to summarize here (Davis's article was 14 pages long), so I will mention only the
major ones. At the British Library, he found copies of two sources of the Bartley story that
Pieters had mentioned in his article, and discovered in one of them a version of the story that
claimed that Bartley had been treated at a London hospital for injury to his skin. Knowing
now when the incident had allegedly occurred, Davis checked the *British Medical Journal*
for 1891-1895, but found no references at all to anyone who had ever been treated for the skin
problems that Bartley supposedly had suffered after his ordeal with the whale. Davis even
checked into the possibility of searching through hospital records for 1891 but inquiries about
the feasibility of trying this convinced him that it would be an impossible task even if records
from that time could be found.

He then turned to checking the claim of the New York Times that the Bartley story had first
been reported in an October 1891 issue of the South Yarmouth Mercury, but his research in
this direction found that no such newspaper had ever existed and that there wasn't even a
place called South Yarmouth listed on any maps of England. He did, however, find that there
was a Great Yarmouth on the seacoast a hundred miles north of London and that it even had a
local newspaper called the Yarmouth Mercury, which had been in print since 1890.

The specifics of Davis's research, some aspects of which I am even omitting, are too tediously
detailed to include here, but through correspondence he eventually managed to find a library
in Norwich County with microfilms of the Yarmouth paper and a librarian who was willing to
look for the whale story. When nothing was heard from the library after a few weeks, he
called to inquire about the status of the research and was told that just the day before the story
had been found in the June 1891 issue of the Yarmouth Independent. The Yarmouth
Independent? This was not the Yarmouth Mercury that had been mentioned in his source
article, so he asked for confirmation of the name and date and was assured that both were
right. The librarian, however, informed him that the story was different from the one he had
written about, because this was just a story about a whale that had been killed off the coast of
a nearby town named Gorleston, but there was no man inside it! "We've never heard such a
story as this," the librarian informed him.

Determined to follow every possible lead to the end, Davis made a three-hour train trip to
Great Yarmouth the next day to go through the library archives himself. The story he found
was about a 30-foot rorqual whale that had run into a pier at the town of Gorleston "just south
of Great Yarmouth." Several boats then pursued the whale, and after attempts to harpoon it
had failed, the men ran it aground and killed it. The whale was then hung up by its tail on the
shore and kept on exhibit for two days, where it was seen by an estimated crowd of over
2000. It was later dissected, and a taxidermist was hired to stuff the skin, which was then put
on exhibit in the London Westminster Aquarium.

Volume 1990 - 2002 Issue


Page 1332 of 2049
Skeptical Review Edited by Farrell Till
Davis's first reaction was that this could not have been the whale he was searching for, but
subsequent research made him suspect that it was. For one thing, he continued his search in
the archives of the Great Yarmouth library and found that two months after the story about the
Gorleston whale was published another story ran about a man who was swallowed by a whale
and later found alive, and the story was similar to the version that was in Fuge's tract, which
had been attached to Rimmer's sermon. The publication of two whale stories in the same
paper within the space of only two months was a coincidence that Davis wanted to pursue, so
plodding on in his research, he uncovered still other versions of the story that had been
published in the 1890s. One of them had even appeared in a French journal whose editor
(Henri de Parville) was a respected science journalist. In his article, Parville had said that after
having confronted this "entirely modern example," he had ended up "believing, this evening,
between ten and eleven o'clock, that Jonah really did come out of the whale alive" (Davis, p.
231). Besides Parville's article, Davis had found a copy of the book written by Arthur Gook,
the Icelandic missionary, whom Fuge had quoted in his tract. Upon learning that Gook had
even published an Icelandic version of the book, Davis searched until he found a copy of it, at
which time he learned that the version of the Bartley story was different in it. For one thing,
this version gave the date of the incident as August 25, 1891, whereas Gook had cited
February 1891 as the date in his English version.

So many different whale-swallows-man tales all published in the same decade was a
perplexing situation that Davis wanted to find an explanation for. Besides the many variations
in the various accounts, Davis was disturbed by an obvious absence of any indications that
real scientific research had gone into verification of the story before any of the accounts had
been published. In reference to two French accounts of the story that Davis had found, he
said, "(I)t isn't the least bit clear from anything I have found that either one made what could
be described as a careful investigation of the incident. I will state this more strongly: no one,
repeat, no one, has given the story the kind of careful investigation it warrants if it is to be
used as evidence for the reliability of scripture. Yet this is precisely what everyone citing the
story assumes--that its authenticity has been established beyond a reasonable doubt, at least
by de Parville if not also by others" (Davis p. 231).

The variations in the story and the obvious absence of serious scientific investigation before
publishing the accounts were just two problems that bothered Davis. His research had
uncovered other anomalies that led him to conclude that the Bartley story was actually a
legend that, oddly enough, had its roots in the Gorleston whale, which had had no man in its
stomach. Davis's research had discovered that there had been three vessels named Star of the
East in the 1890s, but since two of them were actually boats of less than 20 tons, Davis
concluded that the 734-ton Star of the East had to be the ship in the Bartley story, but it was a
cargo ship and not a whaler. Furthermore, Davis learned that even though Bartley had
allegedly been swallowed by the whale in 1891 near the Falkland islands, whaling in this area
had not even begun until 1909, almost two decades after the contradictory dates cited in the
different versions. Davis found sailing schedules for the 734-ton vessel and learned that it had
sailed from New York for Wellington, New Zealand, on June 25, 1890, under the command
of Captain J. B. Killam, whom the New York Times article had reported was the ship's captain.
The date of its arrival in New Zealand could not be found, but the schedules reported that it
left in early November for a return voyage to New York and had arrived there on April 17,
1891, a date that should have put it in the proximity of the Falkland Islands in February.

Volume 1990 - 2002 Issue


Page 1333 of 2049
Skeptical Review Edited by Farrell Till
^However,^ Davis had also obtained a copy of the "crew agreement" (contract) that had listed
"every member of the crew (including a few who signed on in Wellington and deserted just
six days later in Lyttleton), and there is no James Bartley on the list, nor anyone of similar
name, either for the entire voyage or any part of it" (Davis, p. 233).

Davis even learned that some apparent attempts were made to debunk this story at the very
time that it was developing into a legend. L. C. Allen's commentary on Jonah had cited a
correspondence between E. Konig (the author of an article on Jonah in Hastings' Dictionary of
the Bible) and a reader named Williams, which was published in 1906 and 1907 in The
Expository Times. Williams had requested Konig's opinion of the Bartley story, and in his
reply Konig had said that he would be interested in knowing "if the source and the certainty of
the above narrative could be established" (Davis p. 232). In his reply, Williams included
transcriptions of a letter from Mrs. John Killam, the wife of the captain of Star of the East. In
it, she said, "(T)here is not one word of truth in the whale story. I was with my husband all the
years he was in the Star of the East. There was never a man lost overboard while my husband
was in her. The sailor has told a great sea yarn" (Davis, p. 232).

The sailor had told a great sea yarn! This was an indication that Mrs. Killam was convinced
that someone had made up a tale that had never happened and had tried to put it into the
setting of the ship that her husband had commanded, probably as a ploy to give the story
credibility. This was what Davis suspected too. He put together facts that he had uncovered in
his research: (1) Harry Rimmer claimed in both his sermon and Harmony of Science and
Scripture that he had actually met this sailor, who was on exhibit in a London museum. (2)
The earliest version of the Bartley tale that he had found was published in the Yarmouth
newspaper only two months after the story of the Gorleston whale had been published in the
same paper. (3) The Gorleston whale was stuffed and put on display in a London museum. (4)
Tales of a man who had survived having been swallowed by a whale began to appear on both
sides of the Atlantic. (5) A man, so some versions of the story claimed, was on exhibit in a
London museum claiming that he had once been swallowed by a whale.

So was the Gorleston whale the source of this tale after all even though no man had been
found alive in this whale? Davis concluded that this was the probable explanation of a story
that had circulated around the world with no real scientific investigation having been made to
confirm it. Davis wondered if there had been a person, possibly even named James Bartley,
who suffered from a skin condition, and upon hearing about and maybe even seeing the
Gorleston whale, he thought that this was an opportunity to share the spotlight and maybe
even capitalize on his skin problem by putting himself on exhibit in circus side shows as a
man who had survived being swallowed by a whale. To make his story credible, in case
anyone bothered to investigate, he had put it into the setting of a real ship that actually had
been in the South Atlantic in 1891. Within two months, the story had been picked up by a
newspaper that had already shown an interest in whales, and from there it made its way
around the world and was made famous by such writers as Rimmer, Gook, Fuge, and Pieters.
Even though Mrs. Killam had tried to debunk the story, her denial didn't receive the notoriety
that the story, and so it grew into a full-blown legend. After all, a denial that the sensational
happened is never as appealing to human interest as the fantastic is, especially to Christians
wanting to prove that the Bible is "God's word."

Volume 1990 - 2002 Issue


Page 1334 of 2049
Skeptical Review Edited by Farrell Till
This, of course, is only a hypothesis that Davis formulated as a plausible explanation for the
contradictory and sometimes contrary-to-fact data that he had uncovered during his research.
The legend may not have begun this way, but however it started, the evidence indicates that
there is no historical basis for it. Davis's research is valuable not just because it established the
probable truth about this modern-Jonah story but because it also showed that legends can
indeed begin and grow quickly, even within the lifetimes of the parties and events that are
legendized. In this case, the legend of James Bartley had even fooled a respected scientific
journalist like Henri de Parville. When we see such as this happening in modern times, we
have to wonder just how many myths and legends found their way into the Jesus story that
became the foundation of Christianity. To think that such could not have happened in a time
of ancient superstition is incredibly naive when we know that legends can and do develop in
modern times.

Doubts But Questions About Prophecy


Bruce Weston

I have read a lot of your Skeptical Review, and I find that you make a lot of good points. As a
Christian in a Bible Church, I have found that the issues that you raise are questions that I
have often had but was just too scared to ask about. Lately, I have been rethinking what I
believe.

I can see now, for example, that Matthew definitely intended his narrative of the birth of Jesus
to explain how he got to be a Nazarene (because it must have been believed that the Christ
would be a Nazarene), but Luke had a different theory about this. According to Luke, Joseph
and Mary already lived in Nazareth, and so it was only natural that they would return there
after all things according to the law were accomplished.

In many respects I have found your material to be a breath of fresh air to allow me to think for
myself again. Now, I have to wonder why God would have given us a brain if He wanted us
not to really use it, but just to trust Him blindly. Of course, I know that the Christian response
to this is that God knows what's best for us and wants to keep us from bad influences.

Although I did enjoy fellowship and the singing of songs, it actually was tearing me apart
inside to think of many of my friends and family as being lost because they were not born-
again. I was teaching as a graduate assistant in the computer science department at Western
Michigan University, and I had immense difficulty keeping my mind on teaching computers.
Many of my students were from foreign countries, and many of them were Muslims. I had the
horrifying thought that they were all lost. I never did preach anything to them, but I often felt
like it just because the thought of others being lost made me feel this way. I eventually had to
drop out of teaching because I couldn't take the thought of it any more. I had a lot of Muslims

Volume 1990 - 2002 Issue


Page 1335 of 2049
Skeptical Review Edited by Farrell Till
in my own classes that I was taking there too, and when I should have been listening to the
lecture, I would find myself drifting off into wondering how it could be true that all of them
were lost. Instead of having a peace with my faith, which many Christians seem to have, I was
miserable thinking that about 80% of all the world was lost because they didn't believe (or just
had never had the Gospel presented to them). I find that reading some of the material in TSR
actually helps me to feel free from this thought which I had so much trouble dealing with.

I don't think that I could ever become an atheist like you though, Farrell. I see in you the same
kind of rigid desire to believe that there is no God as I see in the fundamentalist dogmatic
opinion that there is a God and that the Bible is His inerrant word and that those who don't
believe it are doomed to hell.

I don't see atheism as the only other option to turn to when leaving a fundamentalist Christian
belief. I still have a conviction (which you would call a superstition) that there is a God, even
though I can accept that there are definitely discrepancies in the Bible. There are false
prophecies, but I can't dismiss all of them as being false. In some of your articles, you have
stated that Christ taught that He would come again in the same generation. But I haven't seen
you address the Christian defense that by saying "this" generation, Jesus was talking about the
same generation that sees the fig tree put forth its leaves (and not the current generation of His
time). The fig tree is symbolic of Israel, which was reborn as a nation in 1948. This could be
significant when looking at many of the prophecies made concerning Israel in the Old
Testament, that the Jews would wander through the nations without losing their identity as
being Jews and that in the latter days God would bring them back to their land. The fall of
communism in the former Soviet Union has now allowed many Jews to return to Israel. And
many have returned from other areas also. Is this likely just a coincidence? See Isaiah 43:6;
Nehemiah 1: 8,9; Deuteronomy 4:27 and 28:64; Jeremiah 30:11 and 31:8,10; Ezekiel 20:33-
38; 34:11,13, 28, 30; and 36:10-11; Isaiah 11:11-12; and Amos 9:14,15.

In many ways it now looks as if prophecies concerning the last days could easily become true.
Someone could step forward in the middle east with a seven-year peace plan which he would
break after 3 and a half years and all hell would break loose on earth (as mentioned by Daniel
and repeated in Revelation). When I stop to think of how a person who lived back in Jesus's
day might have described a vision of a nuclear war, it comes pretty close to some of the things
described in parts of Revelation. There are also many prophecies in the old testament that
sound like they are talking about the effects of a nuclear bomb. When it is said that flesh will
rot while people stand on their feet (Zechariah 14:12), doesn't it sound like what the effects of
a neutron bomb would be? And isn't it believed that Israel currently has many of those at its
disposal? In Revelation, nuclear bombs could be thought of as stars falling from the sky. They
may look like that in a vision. An attack chopper that fires nerve gas may be described as a
locust from hell by someone who hadn't ever seen a chopper shooting people (Rev. 9:3-13).
The blades of the chopper could have been described as the wings of this strange locust (Rev.
9:9). Revelation 9:17 could be talking about tanks of some kind being described as horses that
shot fire from their mouths.

What are your thoughts on some of these things--and on these verses? You may just laugh at
all this, but I can still see much of this as very potential future prophecy (although often very
figurative). Your points about false prophecy like against Tyre are good, but I did note that

Volume 1990 - 2002 Issue


Page 1336 of 2049
Skeptical Review Edited by Farrell Till
Ezekiel has God saying that He would bring many nations against them as the waves of the
sea, i.e., one after the other (Ezek. 26:3). How would this be possible if we were to understand
that Tyre was never to be rebuilt after Nebuchadnezzar besieged it? Could it be that after all
the nations that would come upon Tyre like sea waves, then God would scrape her debris
from her and make her a bare rock? This would still allow for her to become a bare rock in the
future. I would still agree though that Ezekiel 26:12 contradicts Ezekiel 29:18.

I do have to agree with many of your points that you make in The Skeptical Review, and it
grieves my heart to think of people as being lost, but I don't think I would ever be able to just
throw off the entire Bible as you have done. I agree that it is not an inerrant collection of
books, but that doesn't mean that it's testimony should not be considered at all. Men will
always add their own ideas to stories to juice them up a bit, but my personal stance is that I
don't really want to take away the faith of believers. For many people that I know, their belief
in Christ means everything to them. If they are wrong and there is no God, then none of us
will ever know it, but if they are right and God is a righteous terror who is often hard to
understand but who also has provided free salvation through Christ then.... The end result of
rejecting Christ still seems to be much worse than the end result of rejecting most any of the
other religions of the world (if they were right).

I do agree though that many of the methods used by Christians to discredit other religions and
groups that they consider cults (if applied to Christianity) would discredit it in a short amount
of time. Sometimes in matters of faith, though, simple mathematical logic just doesn't apply.
There seems to be such things as religious truths where even though two stories contradict
they can both be considered true. This seems true, for example, in the two conflicting stories
of Jesus's birth as told in Matthew and also in Luke, but it isn't strictly necessary to believe in
an inerrant Bible to believe that Jesus could have risen from the dead. The apostle Paul was
sure of the resurrection of Christ long before the gospels were written. This is possible just as
it is also possible to believe that stories of the creation account are spiritually true without
demanding a literal 6-day creation. For example, It is hard for me to believe in outright
evolution. Even if gradual changes occurred to species over many years, I would still have to
see design in this. Perhaps in some way it could be compared to a complicated computer
program, (but still somebody had to write it). I can't answer why humans have wisdom teeth
(that need to be removed often) or why men have nipples, but at the same time what happened
to the humans with the upside down noses? Did they just drown when it rained and therefore
die out? It is still hard for me not to see the work of a designer when I look at the world, even
though I don't believe in a literal six day creation per se. I don't think I could ever just write
God off completely and be an atheist like you, Farrell.

(Bruce Weston, 1802 Fulford, Kalamazoo, MI 49001; 49001; e-mail; BruceWston@aol.com)

Where to Begin?
Farrell Till

Volume 1990 - 2002 Issue


Page 1337 of 2049
Skeptical Review Edited by Farrell Till

I am always happy to be of help to those who have taken their first steps away from Christian
fundamentalism but still have lingering doubts about whether they have made the right
decision. In Mr. Weston's case, he has raised so many questions that I hardly know where to
begin. Since most of his concerns seem related to biblical prophecies, I will focus primarily
on this issue, but first I want to assure him that after one has left a religion that he/she was
indoctrinated in while growing up, it is perfectly normal to wonder if the right decision was
made. I had such doubts myself for a period of time, and others who left religious
fundamentalism have told me that they did too. Just the night before I began writing this
article, I received a call from someone who had subscribed to The Skeptical Review after
deciding to leave a sect that taught demon possession. Former associates in this sect had been
bombarding him with warnings that he had put himself in danger of falling prey to demons
whom God would allow to possess him as punishment for having lost his faith. The caller had
been bothered with "what-if" concerns and wanted to know if I could give him advice on how
to deal with them.

Pascal's Wager: The members of this sect, of course, were using a variation of Pascal's
wager to try to scare a former member into coming back to the mire that he had pulled himself
out of. The term "Pascal's wager" has been applied to an argument that was made by Blaise
Pascal, a 17th-century French philosopher and mathematician, who argued that one should
"bet" on the existence of God rather than its alternative, because "if you gain, you gain all,"
but "if you lose, you lose nothing." Arguing from this false dichotomy, Pascal advised that
one should "wager, then, without hesitation that He is." Pascal's dichotomy was false, because
the conclusion that he reached cannot be justified by dividing the problem into just two
divisions, i. e., God exists or God does not exist. While it may be true that God either exists or
he does not exist, the problem of "gaining" or "losing" in the sense that Pascal was talking
about is far more complex than simply betting that "He is." If one is going to bet that God
exists, which religion based on that division of the dichotomy is one going to bet on? Should
one bet on Christianity or Islam? Hinduism or Zoroastrianism? If one chooses to bet on Islam
but Christianity turns out to be the true religion of the "God-is" division, we all know what
Christianity says awaits those who reject the "son of God." In this case, it would not be true
that the one who had wagered without hesitation that "He is" had "gain[ed] all." He would
have lost everything even though he had bet that "God is."

Let's suppose that there were some way to know that of all the religions based on the "God-is"
premise, Christianity is the correct one. What brand of Christianity should one then bet on?
Catholicism? Protestantism? If one bets on Catholicism, which kind of Catholicism should he
pick? Roman? Eastern Orthodox? Coptic? If one bets on Protestantism or some other non-
Catholic version of Christianity, which one should he/she wager on? Lutheran? Methodist?
Presbyterian? Baptist? Mormon? There would be hundreds of choices to make, and as anyone
who knows what the different churches--both Catholic and non-Catholic--teach, making the
wrong choice could result in dire eternal consequences. In other words, these religions
themselves teach that one could easily bet that "He is" and yet wind up losing everything. As
far as religion is concerned, there is no safe bet based solely on the premise that a god exists,
so those who are looking for the truth about the Bible should get out of their heads the notion
that maybe they should just play it safe and "bet" on God. Those who do this are really betting

Volume 1990 - 2002 Issue


Page 1338 of 2049
Skeptical Review Edited by Farrell Till
on the Bible and therefore begging the question of whether the Bible is the "word" of this god
who may exist.

Atheism: Before addressing his concerns about biblical prophecies, I should first comment on
an either-or fallacy that Mr. Weston seems to be applying to the question of whether he
should believe the Bible. Choosing not to believe the Bible does not require one to espouse
atheism. When I could no longer believe that the Bible was the inspired, inerrant word of God
(as I had been taught when I was growing up), I abandoned this former belief and left the
ministry; however, I did not abandon my belief in "God" at this time. I remained a theist for
several years, but as my biblical studies continued, my theistic belief weakened to the point
that I realized I was actually an agnostic. It wasn't until many years later that I decided that
atheism more accurately expressed my lack of belief in "God." I personally know many
people who have no belief at all that the Bible is a divine revelation, but they do maintain a
belief in theism. Mr. Weston should realize, then, that just because he has found good reasons
to question his traditional view of the Bible, this doesn't require him to become an atheist.

Biblical Prophecies: In Mr. Weston's case, his "what if" concerns prophecies instead of
demons or some other aspect of Christianity. He wonders if real prophecies in the Old
Testament would not prove that Christianity is the "true" religion. Personally, I don't think
they do for the simple reason that no "real" biblical prophecy fulfillment can be established
with even halfway certitude. I have debated biblical prophecy with many inerrantists, and
they all have tried to defend prophecy fulfillment by begging the question of biblical
accuracy. In other words, they assume that because New Testament writer A said that event B
fulfilled prophetic statement C, prophecy fulfillment has therefore been proven. Such a
simplistic view of prophecy fulfillment leaves too many questions unanswered. Did the
prophet who made statement C mean what New Testament writer A claimed was meant? Did
event B really happen? Even superficial analyses of the Old Testament prophecies that New
Testament writers claimed were fulfilled in their time will show that the so-called prophecies
existed only in the imaginations of the ones claiming fulfillment. Luke, for example, has Peter
claiming that Judas's fall from his apostolic office and his replacement by Matthias was
prophesied: "Let his dwelling place be desolate, and let no one live in it, and let another take
his office" (Acts 1:20). When the source of this alleged prophecy is examined, however, one
will see that Peter actually put together two statements from widely separated Psalms: (1) "Let
their dwelling place be desolate; let no one live in their tents [69:25]. (2) "Let his days be few,
and let another take his office" [109:8]. An examination of these statements in their original
contexts will show that they had no relationship to each other. In Psalm 69, the writer was
talking about enemies and adversaries who were trying to destroy him, and so his statement in
verse 25 was actually a prayer that God would punish them by making their dwelling places
desolate and then letting no one live in their tents. Luke took all sorts of liberties with the text
to twist it into a prophecy about Judas that he could put into Peter's mouth. The psalmist
spoke in the third person plural about his enemies and asked that their dwelling places be
made desolate, but Luke changed the pronoun to the third person singular (his) in a deceptive
attempt to make it seem that the prophecy was speaking of just one person, i.e., Judas, who
had abandoned his apostolic office by his betrayal of Jesus. Accordingly, Luke changed
dwelling places (plural) to the singular (dwelling place) and the plural tents was replaced
with the singular pronoun it to make it agree with the above mentioned switch from plural to

Volume 1990 - 2002 Issue


Page 1339 of 2049
Skeptical Review Edited by Farrell Till
singular in reference to the "dwelling place" that was abandoned. If one is allowed to take
such liberties as these with a text, he can make it into a prophecy of just about anything.

In the second part of this alleged prophecy, the psalmist was talking about the civil judgment
of a wicked man, who was representative of just any wicked person and not Judas
specifically. The writer asked God to enact certain judgments on the wicked man who is
"found guilty" (v:7). He asked God to "let his days be few and let another take his office"
(v:8), but he also asked God to "let his children be fatherless and let his wife be a widow"
(v:9), to "let his children continually be vagabonds and beg" (v:10), to "let the creditor seize
all that he has, and let strangers plunder his labor" (v:11), etc., etc., etc. The list of vindictive
requests for punishment that the psalmist made extends through 10 more verses, and anyone
who reads them with an open mind should be able to see that most of them could not be
stretched to apply to Judas.

A contextual analysis, then, will clearly show that Luke manufactured a prophecy in Acts
1:20 to try to make it appear that the Old Testament had foretold the fall of Judas and the
selection of his successor. This was nothing peculiar to Luke, because other New Testament
writers also strained to find prophecies of Judas in the Old Testament (Matt. 27:9; John
13:18), but when the Old Testament statements on which these fulfillment claims were based
are examined in their contexts, it should be obvious that they made no allusions to the events
that New Testament writers arbitrarily applied to them. As a matter of fact, many of these so-
called prophecies were really matter-of-fact statements and weren't even intended to be
prophecies of anything.

Besides the problem of obvious twisting and distorting to find prophecy where prophecy
never was intended, those who claim biblical prophecy fulfillment have another impossible
duty to perform. They must prove beyond reasonable doubt that the events really happened
that New Testament writers alleged were prophecy fulfillments, and that will be no easy task
for the apologist who tries to prove the divine origin of the Bible through claims of prophecy
fulfillment. Herod's massacre alleged in Matthew 2:16-18 illustrates both problems. Matthew
said that this event happened in fulfillment of Jeremiah 31:15, "A voice was heard in Ramah,
lamentation, weeping, and great mourning, Rachel weeping for her children, refusing to be
comforted, because they are no more," but when this statement is examined in the context of
Jeremiah 30-31, it is obvious that the prophet was speaking about the symbolic sorrow of
Rachel over the deportation of her "children" to Babylon during the captivity. This
"prophecy," then, was a prophecy only in Matthew's imagination as he desperately searched
through the Old Testament for predictions of events in the life of Jesus, but an even greater
problem for prophecy-fulfillment buffs is that they can't even prove with reasonable certitude
that any such event as Herod's massacre ever happened. Of all the New Testament writers,
Matthew is the only one who referred to it, and secular records of the time, some of which
treat Herod's reign very unfavorably, make no mention at all of any atrocity like this that was
committed under his orders. This silence of contemporary records and the similarities of
Matthew's claim to the "dangerous-child" myths of ancient times about babies that were
massacred to eliminate special children who were perceived as threats to kings make the
historicity of Herod's massacre very doubtful. Certainly, it cannot be established with a
certitude necessary to make a convincing case for Matthew's claim that this was a prophecy
fulfillment.

Volume 1990 - 2002 Issue


Page 1340 of 2049
Skeptical Review Edited by Farrell Till
Space is not available in this issue to address each of the "prophecies" about the return of the
Jews to their homeland and the destruction of Tyre, both of which Mr. Weston was
particularly concerned about, so I will reserve space in the next issue to discuss these with the
thoroughness that is necessary to show that they were not the successful prophecies that he
seems to think they were. In the space that I have left, I will comment on the apocalyptic
language in some Old Testament prophecies that he thinks we may be seeing fulfilled in
weapons of modern warfare.

Nuclear Weapons: Mr. Weston wonders if the language in Zechariah 14:12 could just be the
way that a person living in that time would have described a vision of the neutron bomb. The
verse says, "And this shall be the plague with which Yahweh will strike all the people who
fought against Jerusalem. Their flesh shall dissolve while they stand on their feet. Their eyes
shall dissolve in their sockets, and their tongues shall dissolve in their mouths" (Zech. 14:12,
emphasis added). Since the text clearly uses the word plague, I wonder why Mr. Weston
thinks that it may be an ancient description of the effects of a neutron bomb. Why not leprosy
or even bacterial warfare that would infect the enemies of Jerusalem with flesh eating
bacteria? I'm no expert on neutron bombs, but isn't it true that this weapon would cause death
from radiation? If so, wouldn't those exposed to it die long before their flesh would dissolve
while they were still standing on their feet?

Such language as this wasn't at all uncommon in the Old Testament. In the curses that were
pronounced upon the Israelites if they did not obey Yahweh's statutes after entering the
promised land was the warning that Yahweh would send upon them a "wasting disease" that
would "consume the eyes" (Lev. 26:16). In the warning that Moses gave to the Israelites
before they entered into Canaan, he said that if they did not obey Yahweh's commandments,
"Yahweh will make the plague cling to you until he has consumed you from the land which
you are going to possess. Yaheh will strike you with consumption, with the fever, with
inflammation, with severe burning fever, with the sword, with scorching, and with mildew;
they shall pursue you until you perish" (Dt. 28:21-22). In other words, there was nothing
unusual about this kind of rhetoric in biblical times, and in the cases just cited, these warnings
about plagues that would "consume" the flesh and eyes were made in reference to the
generation of Israelites about to enter the promised land. There is no reason to think that
similar language in Zechariah was not also intended for contemporary times. This kind of
language was just a scare tactic that prophets used to whip the people of their times into
shape. It's tragic that people in enlightened times still take it seriously.

Biblical Locusts = Attack "Choppers"? Mr. Weston wonders if the locusts in Revelation
9:3-13, which came out of the smoke from a bottomless pit were prophecies of modern attack
helicopters, but why not ask if they could have been fighter planes or cruise missiles? Why
helicopters? Whatever the writer may have meant in his hallucinatory madness, his
description isn't very accurate of attack helicopters. For one thing, these "locusts" were
"commanded not to harm the grass of the earth or any green thing, or any tree, but only those
men who have not the seal of God on their foreheads" (v:4). I've never seen a battle involving
helicopters, but if war movies are anywhere close to accurate, I can't imagine how attack
helicopters could engage in battle with such precision that only men and not trees, grass, and
any green thing would be harmed. Furthermore, the shape of these "locusts" was "like horses
prepared for battle" (v:7), but attack helicopters are hardly shaped like horses. Even if we

Volume 1990 - 2002 Issue


Page 1341 of 2049
Skeptical Review Edited by Farrell Till
stretch the word horses to mean tanks, it would still require imagination to see the shape of
tanks in helicopters.

The problem with such interpretations as these of biblical prophetic language is that it is
entirely speculative and arbitrary. Whereas one preacher looking for "end times" in
contemporary events will see neutron bombs and attack helicopters in the apocalyptic
language of biblical prophets, others will see space shuttles and submarines, while still others
will see germ warfare and the rise of Saddam Hussein. As the new millennium approaches,
the situation has gotten so ridiculous that no leader in the Near East can even sneeze without
someone seeing prophecy fulfillment in it.

In the next issue, I will address some of Mr. Weston's other prophetic concerns.

The Sons of God and the "Daughters of


Men"
Farrell Till

Embedded in Genesis 6:1-2 is the remnant of a myth about a time when angels married
earthly women and produced a race of giants that caused such corruption on earth that
Yahweh felt compelled to destroy all life except for that which Noah and his family saved
aboard the ark. I first wrote on this myth in the Autumn 1991 issue of The Skeptical Review
("If It Walks Like a Duck... pp. 2-6) and in the Winter 1992 issue ("Sons of God: Just the
Godly Lineage of Seth?"). These articles were written in response to a short article in the June
1991 edition of Christian Courier in which editor Wayne Jackson, a Church-of-Christ
preacher, claimed that the "sons of God" in Genesis 6:1-2 were merely the "godly"
descendants of Seth and made no allusion at all to angels. Although Jackson was offered
space to respond to these articles, neither he nor anyone else has attempted to reply to them.

In anticipation of a reply from someone whom I thought would surely come forth to defend
the historical accuracy of the Bible in this matter, I wrote a third article in 1992 about this
myth, which I have kept on hold. Since no one has yet accepted my offer of free publishing
space either to respond to my articles or to defend the accuracy of the Genesis 6 story, I have
decided to publish the third article. Those who wish to review the other two articles but don't
have all back issues of TSR can find them on its web site at
http://www.infidels.org/library/magazines/tsr/91 (and 92).

Some very striking parallels are evident when Genesis 6:1-2 is compared to two texts from
pseudepigraphic books that clearly presented the belief that angels had once consorted with

Volume 1990 - 2002 Issue


Page 1342 of 2049
Skeptical Review Edited by Farrell Till
human females and produced a race of giants that so corrupted the earth that Yahweh decided
to destroy all life except for that which Noah and his family saved aboard the ark.

Genesis: 6:1-2, "Now it came to pass, when men began to multiply on the face of the earth,
and daughters were born to them, that the sons of God saw the daughters of men, that they
were beautiful; and they took wives for themselves of all whom they chose" (NKJV).
1 Enoch 6:1-2, "In those days, when the children of man had multiplied, it happened that
there were born unto them handsome and beautiful daughters. And the angels, the children of
heaven, saw them and desired them; and they said one to another, `Come, let us choose wives
for ourselves from among the daughters of man and beget us children'" (Isaac Translation).
Jubilees 5:1, "And when the children of men began to multiply on the surface of the earth and
daughters were born to them, the angels of the LORD saw in a certain year of the jubilee that
they were good to look at. And they took wives for themselves from all of those whom they
chose" (Wintermute Translation).

The Problem: I have quoted three separate passages from Hebrew literature. The first is a
quotation from the book of Genesis, which Bible fundamentalists believe Moses wrote by
verbal inspiration of God; the other two are from pseudepigraphic books, written by unknown
Jewish authors purporting to be famous biblical characters. First Enoch presents itself as the
work of Enoch, the seventh generation descendant of Adam who was so righteous that God
translated him to heaven (Gen. 5:24; Heb. 11:5), but few contemporary Bible fundamentalists
believe that Enoch actually wrote this book, although many first-century Christians did
believe the book was authentic. As if he considered it an authentic work of Enoch, Jude (v:14)
quoted the ninth verse of chapter one and alluded to the fallen angels imprisoned beneath the
earth (v:6) that Enoch wrote about in great detail. The author of Jubilees, whoever he was,
also wrote extensively about fallen angels who had committed adultery with human women
and thereby corrupted the whole world (chapters 4-5,8).

The problem that 1 Enoch and Jubilees pose for believers in Bible inerrancy is the clarity with
which they described the carnal relationships that angels in antediluvian times established
with human women. The descriptions are so graphic and so frequent that no one can seriously
deny that the writers of 1 Enoch and Jubilees believed that angels had at one time cohabited
with earthly women and brought about the corruption of the world. The text in Genesis that
refers to this same myth is so brief that fundamentalists have been able to circumvent the
problem it poses for the inerrancy doctrine by giving it an interpretation that excludes angels.
The "sons of God" in Genesis 6 were not angels but simply the righteous descendants of Seth,
inerrantists argue, or men who were in covenant relationship with God and then fell away
through their sexual promiscuity or some such interpretation that would seem sensible to
gullible pulpit audiences who know very little about pseudepigraphic literature or even the
Bible itself. However, when Genesis 6:1-2 is juxtaposed with 1 Enoch 6:1-2 and Jubilees 5:1
the similarities in the three texts are so obvious that honest, objective readers will admit that
all three passages were referring to the same event. If the authors of the two pseudepigraphic
works were describing a period of sexual activity between angels and human women, as the
contexts from which the two quotations were taken will clearly show, then it is probably true
that the Genesis writer was referring to the same thing.

Volume 1990 - 2002 Issue


Page 1343 of 2049
Skeptical Review Edited by Farrell Till
Critical Analysis of the Texts: Just by reading straight through the passages as I have
presented them, without pausing to absorb meaning, one can see obvious similarities in the
three texts. For thirty years, I taught freshman writing at a community college, and I wouldn't
even try to estimate how many times I discovered plagiarism in the student essays I read.
Frequently, the discovery was made through similarities of expression that I found in different
essays. Whenever this happened, I would know that two students working independently of
each other had by chance gathered information from the same source. Without crediting their
source, they would use the information as if it were their own, but in so doing they would
retain enough of the original wording for a skilled reader to recognize that plagiarism had
occurred. Students whom I caught doing this often expressed surprise that I was able to detect
their plagiarism, but there was nothing at all mysterious about the way the discovery was
made. It involved only a simple process of critical analysis.

If we apply the same process to the three texts I quoted at the beginning, we can easily
determine that all three writers were relying on a common source, either written or oral. The
opening statement in all three texts is essentially the same. Whereas the biblical text says,
"(W)hen men began to multiply on the face of the earth," the other two say "children of men"
rather than just "men," and one (1 Enoch) leaves off the reference to "the face (or surface) of
the earth," and simply says that the "children of men had multiplied." Whoever or whatever
the beings who married the "daughters of men" were, the Bible calls them "sons of God,"
whereas the other two texts more specifically identify them as "angels." One describes them
as "angels, the children of heaven" (1 Enoch), and the other calls them "angels of the LORD"
(Jubilees). In addition, the text from Jubilees contains the expression "in a certain year of the
jubilee" that is not in the others. When these slight differences are removed, the reading of all
three texts becomes essentially the same.

To illustrate the striking parallel in wording without these minor variations, I am going to
substitute "sons of God" for "angels" and "men" for "children of men" in one of the
pseudepigraphic texts and then juxtapose it with the passage in Genesis. I will ask the readers
to lay their Bibles aside and then, without referring to them, try to identify the statement that
is Genesis 6:1-2:

(W)hen men began to multiply on the face of the earth, and daughters were born to them, the
sons of God saw the daughters of men, that they were beautiful; and they took wives for
themselves of all whom they chose.
(W)hen men began to multiply on the surface of the earth and daughters were born to them,
the sons of God saw that they were good to look at. And they took wives for themselves from
all of those whom they chose.

If you identified the first one as Genesis 6:1-2, you are right, and you apparently have a good
eye for detail. Before we leave this point, I want you to look at the two texts again and then
consider how slight the changes were that I made in Jubilees 5:1. I dropped "children of"
before men and "in a certain year of the jubilee" and then substituted "sons of God" for
"angels of the LORD." And that's it! That is how close the wording is in both passages. I now
urge you to obtain a copy of Jubilees, which you could probably get through interlibrary loan,
and read the context in which the statement in Jubilees 5:1 was made. If you do this, you will
see that the writer of Jubilees obviously believed that angels had married earthly women in

Volume 1990 - 2002 Issue


Page 1344 of 2049
Skeptical Review Edited by Farrell Till
antediluvian times and corrupted the whole world. You will see it so clearly that no
fundamentalist preacher will be able to explain it away.

Another Juxtaposition: If this is not enough to convince you, perhaps another juxtaposition
will help you see that the Genesis writer understood the marriages of the "sons of God" and
the daughters of men to mean the same as did the pseudepigraphic writers, who thought that
angels and earthly women had married:

Genesis 6:1-8, 11-12: Now it came to pass, when men began to multiply on the face of the
earth, and daughters were born to them, that the sons of God saw the daughters of men, that
they were beautiful; and they took wives for themselves of all whom they chose. And the
LORD said, "My Spirit shall not strive with man forever, for he is indeed flesh; yet his days
shall be one hundred and twenty years." There were giants on the earth in those days, and also
afterward, when the sons of God came in to the daughters of men and they bore children to
them. Those were the mighty men who were of old, men of renown.
Then the LORD saw that the wickedness of man was great in the earth, and that every intent
of the thoughts of his heart was only evil continually. And the LORD was sorry that He had
made man on the earth, and He was grieved in His heart. So the LORD said, "I will destroy
man whom I have created from the face of the earth, both man and beast, creeping thing and
birds of the air, for I am sorry that I have made them." But Noah found grace in the eyes of
the LORD.... The earth also was corrupt before God, and the earth was filled with violence.
So God looked upon the earth, and indeed it was corrupt; for all flesh had corrupted their way
on the earth.
Jubilees 5:1-5,8: And when the children of men began to multiply on the surface of the earth
and daughters were born to them, the angels of the LORD saw in a certain year of the jubilee
that they were good to look at. And they took wives for themselves from all of those whom
they chose. And they bore children to them; and they were giants. And injustice increased
upon the earth, and all flesh corrupted its way; man and cattle and beasts and birds and
everything which walks on the earth. And they all corrupted their way and their ordinances,
and they began to eat one another. And injustice grew upon the earth and every imagination
of the thoughts of all mankind was thus continually evil.
And the LORD saw the earth, and behold it was corrupted and all flesh had corrupted its
order and all who were on the earth had done every sort of evil in his sight. And he said, "I
will wipe out man and all flesh which I have created from upon the surface of the earth." But
Noah alone found favor in the sight of the LORD.... "And he said, "My spirit will not dwell
upon man forever; for they are flesh, and their days will be one hundred and ten years.

I have italicized certain expressions in the passage from Jubilees for obvious reasons. Each
italicized statement has its parallel in the Genesis passage! The order of the statements is not
always the same, but the substance is, and sometimes even the exact statement was made in
the Genesis text. In fact, if one took only the italicized statements in Jubilees, substituted
"sons of God" for angels, omitted the one reference to the jubilee, and rearranged some of the
expressions, he could almost reproduce the Genesis text. This is how it would look:

And when the children of men began to multiply on the surface of the earth and daughters
were born to them, the sons of God saw that they were good to look at. And they took wives
for themselves from all of those whom they chose. "And [the LORD] said, "My spirit will not

Volume 1990 - 2002 Issue


Page 1345 of 2049
Skeptical Review Edited by Farrell Till
dwell upon man forever; for they are flesh, and their days will be one hundred and ten years."
And they bore children to them; and they were giants.
And the LORD saw the earth, and behold it was corrupted and all flesh had corrupted its order
and all who were on the earth had done every sort of evil in his sight. And injustice grew upon
the earth and every imagination of the thoughts of all mankind was thus continually evil. And
he said, "I will wipe out man and all flesh which I have created from upon the surface of the
earth." But Noah alone found favor in the sight of the LORD.... And injustice increased upon
the earth, and all flesh corrupted its way; man and cattle and beasts and birds and everything
which walks on the earth. And they all corrupted their way....

This rearranged version contains everything that is in the Genesis passage except for verses 4a
and 6, where it was said, respectively, "There were giants on the earth in those days, and also
afterward, when the sons of God came in to the daughters of men," and "the LORD was sorry
that He had made man on the earth, and He was grieved in His heart." Anyone who can look
at this rearrangement and deny that the writers of Genesis and Jubilees relied on common
sources probably doesn't want to know the truth.

The Omitted Verses in the Jubilees Passage As the readers will notice by referring to the
middle column, the passage from Jubilees 5, from which I re-created the Genesis 6 narrative,
omitted a part where the ellipsis [...] was inserted. The two verses that were skipped clearly
indicate that the writer of Jubilees, like the writer of 1 Enoch, believed that God had
condemned angels who had consorted with human women to be imprisoned under the earth to
await judgment:

And against his [the LORD's] angels whom he had sent to the earth he was very angry. He
commanded that they be uprooted from all their dominion. And he told us to bind them in the
depths of the earth, and behold, they are bound in the midst of them, and they are isolated.
And against their children a word went forth from before his presence so that he might smite
them with the sword and remove them from under heaven (Jubilees 5:6-7).

If the readers will return momentarily to the re-created passage (bottom of the middle column
on this page) and insert these two verses at the first ellipsis (second paragraph), he will have,
in addition to the re-created Genesis passage, an expansion of the text that provides more
details about the myth or legend that the Genesis writer was alluding to in 6:1-12. Everything
in the two texts (Genesis and Jubilees) is remarkably parallel except for these two verses in
Jubilees that the Genesis account did not include. The additional verses leave no room to
doubt what the author of Jubilees thought that these "sons of God" were. Like "Enoch," he
clearly believed that they were fallen angels and that Yahweh had ordered them to be
imprisoned under the earth to await judgment. In the face of these facts, it is remarkable that
Bible fundamentalists will stubbornly deny that the Genesis writer was alluding to the fallen-
angel myth that figures so prominently in pseudepigraphic literature.

Genesis as the Source: Since scholars generally agree that Jubilees was written around 100
B.C., inerrantists may ask why it would not have been possible that Genesis was the source of
this section of Jubilees. In other words, it might be that someone with a wild imagination
lifted certain expressions from Genesis and wove around them a fanciful yarn about angels
and earthly women. Such is possible, of course, but then what do we say about the book of 1

Volume 1990 - 2002 Issue


Page 1346 of 2049
Skeptical Review Edited by Farrell Till
Enoch, the Testaments of the Twelve Patriarchs, and Qumran MS4? They all contain
narrative passages concerning a myth about the unnatural marriages of angels and human
women that corrupted the earth and caused God to destroy it with a great flood. Are we to
assume that the writers of these works all had wild imaginations that they used to turn the
Genesis flood into a fanciful tale about the marriages of angels and earthly women? Let's put
this speculation to the cutting edge of Occam's razor. Which is more likely, that different
writers working independently plagiarized a fragment of Genesis and turned it into an
imaginative story of angels and women marrying or that different writers working
independently wove their stories around a widely circulated myth about antediluvian
marriages of angels and women? To ask the question is to answer it.

Conclusion: True biblical scholars abandoned the myth of verbal inspiration long ago. They
did so because their studies of Mid-Eastern cultures that were contemporary to the Hebrews
of Bible times had destroyed the fanciful notion that the Bible was a special revelation from
God. They discovered that there was nothing at all "special" about it. It was based on the same
myths that had circulated in the Sumerian, Babylonian, Assyrian, Canaanite, Persian,
Egyptian, and Greek cultures that had surrounded the Hebrews. In fact, we now know that
there was even very little originality in the Hebrew myths. They had borrowed them from
neighboring cultures and adapted them to their society. The most obvious examples of this
adaptation can be found in the first 11 chapters of Genesis. The creation, the garden of Eden,
the flood, the tower of Babel--these myths had all originated in neighboring cultures.

And so did the Genesis 6 myth about the "sons of God" (angels) who married earthly women
and made the world so corrupt that God destroyed it with a great flood. Inerrantists will pooh-
pooh everything I have said in this analysis, because they know that their inerrancy doctrine
will collapse if it can be proven that at least part of the Bible is based on mythology. I believe
that I have established that Genesis 6:1-2 alludes to an ancient myth that was still believed by
some when the Bible was written. All I ask of you, the reader, is a fair hearing. Weigh
everything I have said and do the follow-up studies I have suggested, and I'm sure you will
agree that elements of mythology can be found in the Bible text.

From the Mailbag


Farrell Till

More Fundamentalist Flapdoodle..

In the November/December issue of TSR, you gave some guidelines for submitting articles for
publication. You said, "We are looking for materials that relate to the subject of biblical
inerrancy." In "From the Mailbag," the guidelines are not as narrow; the letters deal with a
variety of issues under the subject of religion. This provides a good balance to TSR. I find the

Volume 1990 - 2002 Issue


Page 1347 of 2049
Skeptical Review Edited by Farrell Till
letters and your replies very interesting and very informative. The best improvement to "From
the Mailbag" section would be this: add another page or two.

(Don Robertson, 644 Walnut Street, Rock Hill, SC 29730)

EDITOR'S NOTE: Although I do try (not always successfully) to restrict articles to the
subject of biblical inerrancy, I have allowed letters to discuss a variety of subjects. Since
readers seem to like this policy, I will continue it.

Mr. Robertson included with his letter some photocopied pages with this note attached: "This
should refresh your memory of Harry Rimmer's book and give you more fundamentalist
flapdoodle." The pages were from Rimmer's book The Harmony of Science and Scripture,
which I was sure was the source of my first exposure to the urban legend about the long-day
of Joshua. It turned out that my memory of something I had read over 40 years ago was
accurate. I recall Rimmer's book so well because he was the Josh McDowell of his day, and
nearly all Bible college student had copies of his books in their personal libraries. Mr.
Robertson sent only the pages from Rimmer's book (copyrighted in 1936) that dealt with the
long-day of Joshua, he did include a copy of the table of contents, which show that Rimmer
also included chapters that proved the scientific accuracy of the stories of Jonah, Noah's ark,
and other fabulous events. This goes to show that the fundamentalist flapdoodle going around
today as scientific proof of the Bible's divine origin is just the recycling of tales that were
answered and discredited long ago. On pages 2-5 of this issue, I discuss more in detail the
version of the long-day legend as Rimmer told it six decades ago.

More Nonsense Than Legend...

Regarding the November/December 1998 issue's articles on the infamous "Missing


Day/NASA" nonsense, I was introduced to this silliness in junior high school (Indian Springs,
Nevada) by a fundamentalist science teacher (a Mormon bishop) named "Bud" Hopkins
(circa 1974). Mister Hopkins brought up the "missing day" prank as "proof" of the Bible in
science class, in a public school (and thus US citizens were paying him to do so). He also
showed the class a reel-to-reel video tape produced by creationists that purported to "prove"
via computer simulation (albeit very crude, in those days) that the human red blood cell was
"designed." This religious indoctrination he called "science education." Since the room was
full of 12 and 13 year olds, I do not recall even a single objection being raised at the time.

I say "prank" because that's what the "missing day" nonsense is. It doesn't even qualify as a
hoax because it's blatantly false (like the "alien autopsy" prank), or an urban legend since it
isn't even remotely plausible. With no starting reference, there cannot be a method by which a
"missing day" can be calculated. If the prank were true, what is meant by a "day," anyhow? A
"day" can mean a roughly 24-hour rotational period of Earth, and it can mean 1/365.25 of an
orbit of Earth around Sol---the latter, given a reference point to start with, could show a
"missing day," but not the former: and the Hebrew Testament says Sol and Luna "stood still,"
not Earth.

And yet people still believe the "missing day" prank is true, which amazes me. Such a belief
would earn the holder a strait-jacket and medication if it had not been coupled with the

Volume 1990 - 2002 Issue


Page 1348 of 2049
Skeptical Review Edited by Farrell Till
Christian religion, and astonished derisive laughter by fundamentalist, inerrantist Christians if
it had been coupled to a non-Christian religion.

(Rev. David Michael Rice, 723 Casita, San Clemente, CA 92673-2708; e-mail,
shydavid@net999.com)

A Letter to Brennecke...

My sons are big fans of your new governor. I guess that might not impress you that I let my
children watch wrestling; however, I wanted to praise your article "Twenty-Four Hours in
Time: A Rebuttal." I wrote Harold Hill last year, but I received no reply. He may have passed
away. I am enclosing two items that may interest you. When I was growing up my pastor told
both the story concerning Joshua and NASA and this crazy story about the modern day Jonah
(James Bartley).

(Everette Hatcher III, Box 23416, Little Rock, AR 72221)

EDITOR'S NOTE: The enclosures that Mr. Hatcher referred to above were copies of a letter
to Harold Hill (the source of the NASA version of the urban legend about Joshua's long day)
and a 14-page article from Perspectives on Science and Christian Faith that was written by
Dr. Edward B. Davis of Messiah College to debunk Harry Rimmer's claim in Harmony of
Science and Scripture that a modern-day Jonah named James Bartley was swallowed by a
"whale shark" in 1891 and survived the ordeal. The research that Davis put into tracking
down the source of this yarn and completely discrediting it was so impressive that I used it as
a basis for the article on pages 2-5 of this issue, which shows how would-be apologists are
quick to grab just any tale that they think vindicates the Bible and circulate it as proven fact.

The Jayton Chronicle...

Regarding the Jayton Chronicle newspaper referred to in "Twenty-Four Hours in Time" in the
November/December 1998 issue of TSR, former editors of the Texas Almanac list a
newspaper called the chronicle in Jayton through the 1986/1987 edition, after which there is
no listing for a newspaper in Jayton.

There is a small public library in Jayton, the Kent Country Library (2000 volumes), which
would most likely have back issues of the Chronicle, but any research would probably have to
be done in person.

Given that there was at some point in the past a Jayton Chronicle and that the population of
Jayton is 600, I don't find it beyond the realm of possibility that "Twenty-Four Hours in
Time" was indeed printed in that periodical. Having lived in several small Texas towns with
weekly newspapers, I know that most of them will print almost anything in order to fill space.

Of course, if "Twenty-Four Hours" was printed in the Jayton Chronicle, that doesn't make it
true; it means only that the editor was a very gullible fundamentalist Christian or desperate to
fill space. Also note that "Twenty-Four Hours" says the story was "retold" in the Jayton
Chronicle, indicating that the newspaper was not the original source.

Volume 1990 - 2002 Issue


Page 1349 of 2049
Skeptical Review Edited by Farrell Till
(3625 South Hills Avenue, Fort Worth, TX 76109)

EDITOR'S NOTE: This letter indicated only the address of the writer. Since the subscription
files listed no one at this address, I could not identify the writer. I published the letter anyway
because the information in it relates directly to the "Twenty-Four Hours in Time" article that
Charles Brennecke responded to in the November/December issue. If the writer of this letter
will identify him-/herself, I will publish the name in a later issue.

Letters, by the way, should always show both the names and addresses of the writers
somewhere on the letters and not just on the envelopes. This can save me time looking for the
writers' names or addresses if the envelopes and letters become separated.

The Denial of Apologetics Press...

I received the most recent TSR issue yesterday, and saw the article alleging that NASA (or
whoever--I don't have the article at hand right now) had found a missing day and thus verified
the miraculous stories in Joshua and Isaiah. This is an urban legend, and it's so dumb that
even evangelical apologists have a hard time believing it. As proof of this, check out
http://www.apologeticspress.org/rr/rr1991/r&r9105a.htm.

The article at this URL is an online version of an article that originally appeared in the May
'91 edition of Reason and Revelation: A Monthly Journal on Christian Evidence. The article
thoroughly debunks the urban legend. You might want to use this article as a starting point in
writing a rebuttal to the legend. (I'd write one myself, but between my job and my family, I
simply don't have the time.)

(Rob Berry, 2344 Brisum Way, Hillard, OH 43026; e-mail, berry@coil.com)

EDITOR'S NOTE: The article that Rob Berry mentioned was the one that I referred to in my
response to this urban legend (November/December 1998, pp. 7-8). Apologetics Press is not
the only fundamentalist organization warning biblicists not to use this myth. Besides the
article in Perspectives on Science & Christian Faith that was my primary source of
information for the article on pages 2-5 of this issue, Everette Hatcher sent me copies of two
other articles from religious papers that debunked this myth about a missing day in time.

Fear and Some Trepidation...

It is with fear and some trepidation that I enclose my check to renew my subscription to The
Skeptical Review. I made the mistake, during a visit with my wife to her folks, of leaving the
November/December issue on the dining room table while I went to the bathroom. Upon my
return, I found my mother-in-law, of Southern Baptist extraction, waving the issue around in
the air and screaming about the blasphemy of your cover article on the book of Daniel, and
assuring me in no uncertain terms that this guy Till is going to join me in burning in hell and
"you can be sure of that," do you understand?

Now I realize the error of my ways in leaving such godawful materials carelessly exposed to
view, but I must also suggest that you be careful of what you write lest you cause undue upset

Volume 1990 - 2002 Issue


Page 1350 of 2049
Skeptical Review Edited by Farrell Till
to true believers and get them agitated. Nonetheless, I am pleased to renew, and rest assured
that your insightful publication is much welcome every two months in my private little world.

Finally, may I apologize for our misbehavior here in the 6th district of Illinois in reelecting
my congressman, Henry "Homewrecker" Hyde, back to his post by a 2 to 1 margin. He enjoys
a "safe seat" here in the Vatican of Evangelism, and there is not much we can do about it. As
it is, I must keep my drapes drawn at all times.

You all carry on, and keep the faith!

(Theodore M. Utchen, P. O. Box 1140, Wheaton, IL 60189-0140; e-mail


TMUtchen@compuServe.com)

EDITOR'S NOTE: Ah, yes, Henry Hyde, another contribution from Illinois to the Christian
right agenda. The state also added a senator to the cause in the last election. The reaction of
Mr. Utchen's mother-in-law is typical of what most of us who came from fundamentalist
backgrounds have to tolerate in our extended families. In fundamentalist families, it seems
that independent thinking is a definite no-no.

Like a Lamp unto My Feet...

A few years ago, the local Jehovah's Witnesses gave me a Bible. Although I had been raised
in the one true religion (First Congregationalist), I had never actually read the good book.
Now I must admit that before I began to read, I was skeptical; however, after reading only
Genesis, my skepticism proved to be grossly inadequate. I didn't know whether to laugh or
cry.

As I read, I couldn't help wondering how anyone who has graduated sixth grade could believe
this crap. After many conversations with believers, I began to realize that most of them don't
have the slightest idea what the book contains. For instance, one woman I talked to had
attended Jerry Falwell's Liberty College, but she was not aware of the story about Jephthah
and his unfortunate daughter.

Anyway, the reason I am writing is to thank you for your excellent publication. It is greatly
appreciated. It is most difficult to obtain "ungodly" literature, in our superstitious society. The
Skeptical Review is like "a lamp unto my feet." The simple man believeth every word, but the
prudent man subscribes to TSR. Each issue makes me feel "born again."

I would like to purchase all the back issues to 2-98. A check is enclosed.

(Gary Walker, P. O. Box 3, Compton, NH 03233)

EDITOR'S NOTE: Who hasn't disentangled himself from biblical fundamentalism and then
wondered, "How could I have ever believed this nonsense?" Once a person steps away from
it, the absurdity of it is as clear as daylight, but getting people to take the first steps away is
hard to do. Actually reading the Bible would probably be the easiest way for a Christian to

Volume 1990 - 2002 Issue


Page 1351 of 2049
Skeptical Review Edited by Farrell Till
realize that he/she has been duped. Too many Christians believe the Bible because... well, just
because they believe it.

Another Familiar Story...

Today, I received my first issue of The Skeptical Review, and found it to be fair and helpful.
Until last December, I was the leader of a university student organization for 12 years until I
hit the wall and left the ministry. My reasons for leaving began at the heart of my emotions.
After spending several summers ministering to Catholic Christians, Muslims, and atheists in
Europe and South America, I began to doubt seriously that such people were going to hell.
That led to a period of deep depression and questioning, followed by months of study, which
led to my rejection of Christianity for moral and rational reasons. Having built my life on
Christian beliefs, I now find myself alone in the world. I have lost my closest friends (who
feel sorry for me), and the tension this has brought to my family relationships is enormous. I
do not want to acknowledge Christmas, and I am finding it difficult to figure out which hills I
should die on. They are right in saying that my doubting came as a result of "battle fatigue,"
but I also believe that suffering is many times the exact agent we need to face reality. It has
been only 6 months or so since I gave up trying to reconcile Christianity and all its problems
and inconsistencies, but I am still battling depression and I would like to know if you have
some suggestions that might be helpful as I try to move on with my life. Thank you very
much.

(Craig Cunningham, Craig Cunningham, 6502 East Golf Links, Apt. R262, Tucson, AZ
85730; e-mail CCunn63@aol.com)

EDITOR'S NOTE: I may overdo the publication of letters like this one, but I like to run them
so that those who are in the initial stages of taking a rational look at the Bible can see that
what they experience when former "Christian" friends turn against them is usually the rule
rather than the exception. What has happened to Mr. Cunningham happened to me, and as
past letters to this column have shown, it happens to almost everyone who examines the
claims of Christianity and rejects them. For some reason, Christians see the Craig
Cunninghams of the world as threats, and in a sense I think they are right. In an age of
rapidly increasing knowledge and the technology to transmit it quickly, religion can no longer
keep people shackled in ignorance as it could it in the past. Every Craig Cunningham that
comes along must make Christians who don't want to think about their beliefs feel insecure.
At least something happens to make them angry at those who dare to renounce Christianity.

Reasons for Not Believing the Bible...

In a previous issue of The Skeptical Review, a letter from E. E. Brennaman asked, "Do
inerrantists really believe the Bible is inerrant, or do they just maintain it for the sake of
appearance?" This has been a question on my mind for years.

If someone asks me why I do not believe the Bible is inerrant (or infallible, which means the
same), my short answer is two examples. First, compare the two creation stories in Genesis
1:1-2:4a and Genesis 2:4b-25. The times of creation are 6 days (Gen. 1:31) vs. 1 day (Gen.
2:4b), the order of creation is different; male and female created last, both in the image of God

Volume 1990 - 2002 Issue


Page 1352 of 2049
Skeptical Review Edited by Farrell Till
(Gen 1:26:31) vs. man created first (Gen. 2:4b-7), then all the plants (Gen. 2:8-17), then all
the animals (Gen 2:18-20)--in an effort to find a partner for the man!--then woman last (Gen.
2:21-22). Note that Genesis 2 clearly states that God's first idea for a partner for man was not
a woman but an animal!

Second, compare the genealogies of Jesus in Matthew 1:1-17 and Luke 3:23-38. Note that
both Matthew and Luke listed the ancestors of Jesus only on his father's side. In Matthew, the
father of Joseph, the father of Jesus, is Jacob (v: 16), but in Luke he is Heli (v:23). In
Matthew, Jesus is descended from David's son Solomon (v. 6), but in Luke he is descended
from David's son Nathan (v. 31). Obvious errors.

Recently, I have heard fundamentalists on the radio, and read parts of books by "Evangelical
Apologists" trying to justify the belief in Bible inerrancy. They all seem to start with the
assumption that the Bible is inerrant and tell their patients(?) that even if they can't understand
why some "apparent error" is not an error, they must believe that there is some unknown
explanation. This is nothing more than self-delusion, or, as some have said, "parking your
brain at the church door."

Why would people believe this? The question in The Skeptical Review recalled a time when I
was a freshman in college (about 1941). This was long before TV Preachers or even TV. A
guest preacher at our Presbyterian Church said he believed (the creeds, I think) and that he
always would. I could not understand how any intelligent person could say such a thing. (I
knew about the change in belief in the churches; since the time of Galileo, they no longer
claim that the earth is the fixed center of the universe.) The Presbyterian creeds also state that
the earth, sun, stars, etc. were all created in 6 days, 6000 years ago. In my youthful way,
assuming then (but not now) that anyone who invoked the name of God must be a person of
integrity, I could see only one possible explanation for the preacher's statement: he considered
himself a member of the acting profession, and whereas, some actors played the roles in
Shakespearean plays, which people like to hear, he would play the role of a preacher saying
what people wanted to hear.

Now, however, I would give other reasons for some preachers claiming to believe in an
inerrant Bible: (1) They have been brainwashed or are under posthypnotic suggestion to
believe that if they don't affirm the inerrancy of the Bible they will go to a literal hell and
suffer through all eternity. (This from a God of love?) (2) It is a technique learned from
Madison Avenue advertising. The ad always says its product is perfect, never that it has an
error, and its main goal is to sell the product. (3) It aids in false advertising. First, they define
their theology, and then they find verses that support it. They ignore all verses that might
refute their theology. (4) Laziness: by claiming any verse they quote is from an inerrant Bible,
they imply that there can be no other verse that might refute it, thus making their job easy, if
they can find gullible people who will accept their assumptions. (5) It helps to select the least
intelligent and undiscerning as their followers, thus increasing their financial take. (6) They,
as I heard a radio fundamentalist telling a caller who couldn't understand why the two
conflicting stories of David or Elhanan killing Goliath were not an error, are the ones who
"are not playing with a full deck." (7) "Those people don't really believe anything, they just
like to say that they do," which was the remark of one person, after seeing the actions of one
preacher, and then hearing him say in a sermon that Christians should do just the opposite.

Volume 1990 - 2002 Issue


Page 1353 of 2049
Skeptical Review Edited by Farrell Till
These inerrantists may think that they are defending Christianity, but they are in the same
class as those who centuries ago condemned Copernicus and Galileo for opposing the then
orthodoxy of the churches that the earth was fixed at the center of the universe and did not
move. They harm Christianity rather than help it.

(Harold Curtis, P.O. Box 15626, Long Beach, CA 90815; HCurtis266@aol.com)

The Survival and Destruction of Books...

Certainly I have no biblical scholar credentials, but one really doesn't need them to see the
lack of credibility in one of Dr. Price's arguments. I get really tired of hearing this same
argument that many fundamentalists also use as a reason to consider the Bible to be correct
and the "word of God." They argue that the Bible is valid because there are "many more
extant ancient manuscripts of the biblical books than there are of secular documents."

They either don't know or ignore the historical reason that there are indeed fewer secular
ancient manuscripts. The reason is that early Christians destroyed as many secular books as
they could lay their hands on, including the large library at Alexandria. Knowing what a
strangle hold Christianity has had on the world, I find it a small wonder that any ancient
nonbiblical manuscripts survived.

Not satisfied, of course, with destroying what they considered pagan literature, the early
Christian church also began to destroy any early Christian literature that they didn't find
orthodox enough to suit them.

You can hear an echo of this same kind of "reasoning" down through the ages. Servetus was
burned at the stake with his "heretical" books neatly tucked into the chains that bound him to
the stake. Hear an echo also when Hitler burned books during the reign of the Third Reich.
Then how about even today when fundamentalist Christians have a book and record burning
of "satanist" materials?

(Marie Micheletti, 209 West Tazwell, Tremont, IL 61568)

Another Answer to Gleason Archer...

Gleason Archer claims that life is meaningless without god. If so, then god's existence would
be meaningless, since god has no god to give meaning to his existence. If god doesn't need a
god to validate his existence, then we don't either. It's just arbitrary, not logically necessary, to
say that an all powerful being doesn't need a god but we do. God is no better off, by Archer's
standards, than an atheist hermit (god has no god above him, remember) with an animal
collection.

Human lives aren't meaningless without god. We can lead ethical, excellent lives for their
own sake (see Exuberance and Eupraxophy by Dr. Paul Kurtz). It's own sake is its own
justification. Those who need a god to validate themselves have really insignificant lives.

Volume 1990 - 2002 Issue


Page 1354 of 2049
Skeptical Review Edited by Farrell Till
A god isn't needed for moral values, because godless people have formulated and practiced
good moral values, and a god's moral values are arbitrary and amoral ones--obeyed by
command and not due to effect. If they are good and practical, they don't need a god to
validate them. A god could command evil and contradictory things, and say they aren't evil
and contradictory, because he is a god.

I am without belief in an afterlife or consciousness after death, and I am quite comfortable and
not even the least uncomfortable with it. I live for those now living and the society that will
live after me. The good feeling I have now in knowing that the good I'm doing will live on
after I am no long conscious is justification enough for me. I'm part of something bigger than
myself, the line of human beings, Ingersoll included, who made things better for humanity
and expected no heavenly reward for it. This life was and is enough.

(Andrew Lutes, 658 Mans Lucas Road, Mansfield, OH 44907)

A Correction...

In my November/December 1998 letter, I erroneously reported that Nabonidus died before


Cyrus reached Babylon, so let me review the facts I have regarding the fall of Babylon. In 540
BCE Nabonidus returned from Tema to organize the defense of his kingdom. On April 4, 539
BCE, the New Year's Day Festival was properly celebrated once again. In that festival, the
king put his hands in the hands of the god Marduk as a symbol of renewal. Only the king
could perform this ceremony, which had been simply omitted during Nabonidus's 10-year
absence. A clay tablet (ANET 306-307) tells of the cancellation of that important ceremony
when Nabonidus was away.

While Nabonidus was living in Tema, in Northwestern Arabia, Belshazzar served as a co-
regent (crown prince), not as a king. Up until his death, inscriptions referred to Belshazzar as
"the king's Son." Needless to say, this 10-year neglect of the god Marduk did not make
Nabonidus popular with his priests. Some writers speculate that Nabonidus was supporting a
rival Moon cult in Tema.

Belshazzar was put in charge of troops guarding a defense rectangle including the cities of
Opis, Sippar, Cutha, and Borsippa, which were considered vital to the defense of Babylon.
Early in October 539 BCE, Cyrus attacked the city of Opis, aided by a turncoat Babylonian
governor. There was rioting within the defensive perimeter, and a battle was fought in which
Belshazzar was killed by Gubaru. Gubaru was one of Cyrus' commanders/governors. Within a
few more days, the remaining key cities were seized or else they surrendered forthwith, and
Nabonidus fled from Babylon. Perhaps, there was nowhere for him to go. Another source
speaks of his return to Babylon, where he was arrested. Beyond that, I have no further
information on him.

Herodotus enlivens his account of Babylon's capture with a story that Cyrus diverted the
Euphrates river into an old floodway, allowing his army to enter the city through a nearly dry
river bed. The older cuneiform record simply states that the army of Cyrus entered Babylon
without a battle to receive the acclaim of many citizens.

Volume 1990 - 2002 Issue


Page 1355 of 2049
Skeptical Review Edited by Farrell Till
Thus, the Bible is in gross error on almost every point! Belshazzar was never a king as the
Bible clearly claims that he was. Belshazzar was not in command when Babylon fell, let alone
surprised while feasting at night. He did not die by assassination but rather on the field of
battle. Babylon, itself, peacefully surrendered, contrary to numerous horrid prophecies in the
Bible. The kingdom of Babylon was not succeeded by a kingdom ruled by "Darius the Mede,"
about whom history knows nothing.

The sources of the information above were Great Events From History, Frank Magill (editor),
"Fall of Babylon," Ancient and Medieval Series, Vol. I, 1972, pp.179-183; History of the
Persian Empire, A. T. Olmstead, 1948; An Introduction to the Literature of the Old
Testament, Samuel R. Driver, 1957; and Anchor Bible, Book of Daniel.

(Dave Matson, editor, The Oak Hill Free Press, P.O. 61274, Pasadena, CA 91116; e-mail,
103514.3640@Compuserve.com)

EDITOR'S NOTE: This is the second time that I have encountered the claim that Belshazzar
was killed at the battle of Opis. The first reference was in Ancient Iraq by George Roux
(mentioned in my editorial note at the end of William Sierichs' article in the
November/December 1998 issue of TSR. Sierichs was reluctant to include it in his article,
because Roux did not explain how he had come by this information, so I cited it in my note as
an unverified claim. I have asked Dave Matson to identify which of his sources listed above
contained the reference to Belshazzar's death at Opis. If anyone else has information about
this, please contact me.

A Stereotypical Impression...

Now that I am corresponding with some prisoners, I've noticed that you are quite
disingenuous when it comes to publishing letters.

Some (and one in particular) of these prisoners are not very literate. Their spelling is not the
greatest, they confuse homonyms and also show distinct signs of learning disabilities.
However, when you print their letters in TSR, they read polished to spit-shined elegance. On
the other hand, when inerrants write in, every mistake they make is left in. This gives the
impression that the Free Thinker who writes in, just about always, has near-perfect literacy
while the inerrants all to often do not. I also get the impression that this practice might be
spilling over into the articles, too. Indeed, in the September/October TSR a letter from Rev.
David Michael Rice directed at Gleason Archer included these criticisms: (1) Get a spelling
checker. (2) Take a refresher course in grammar at a local community college. Somehow I get
the feeling that this would never be directed at anybody who is on your side of the argument.

While I realize that TSR is your magazine in every sense of the word, it is belief that this
practice should stop now.

(M. Paul Goldberg, 67-35 Yellowstone Boulevard, Apt 6T, Forest Hills, NY 11375-2610)

EDITOR'S NOTE: I didn't dare edit Mr. Goldberg's letter, so the errors in it are not
typographical. The letter was set up just as he submitted it, so he is the one who used to for

Volume 1990 - 2002 Issue


Page 1356 of 2049
Skeptical Review Edited by Farrell Till
too and omitted the word my in the last sentence. The punctuation and capitalization mistakes
are also his. If Mr. Goldberg disputes this, I will gladly send him a photocopy of his original.

I don't know what prison inmates Mr. Goldberg is corresponding with, but he seems to have
formed the stereotypical impression that prison inmates are semiliterate. While I was still
teaching college English, one of my duties was to evaluate the writing tests of inmates at a
local prison where the college provided educational services, and I learned from that
experience that the writing abilities of inmates were generally no better and no worse than
the students who attended the college on the outside. I can, in fact, recall some very well
written essays that came from prisoners applying for admittance to the college program. I can
recall some bad ones too, but I encountered both good and bad essays at the college campus
too.

Do I edit letters and articles before I publish them? You bet I do, because I prefer that TSR
reflect a better quality of writing than that which is generally found in other publications.
What Mr. Goldberg doesn't realize, however, is that I correct on both sides. I have corrected
the articles of Dr. James Price, Roger Hutchinson, Everette Hatcher, Matt Perman, Dr. Hugh
Ross, and many other inerrantists whose works have been published in TSR. There has
probably not been a single article published in TSR by anyone that I did not correct for
grammar, spelling, or punctuation. This includes my own articles, because sometimes in final
proofreadings, I spot mistakes that escaped my notice in earlier editings. The same is true of
letters in the mailbag column. I correct them on both sides, so why should I treat prison
inmates any differently?

There are times, of course, when I leave the mistakes in the articles or letters and mark them
with [sic], but I do this only when I think that showing the mistake will make a relevant point.
If I publish an arrogant or spiteful letter, I will leave the mistakes in the text and mark them
with [sic] to call attention to the probable level of education in the writer. If an inerrantist
shows a tendency to boast of personal scholarship or linguistic skills, I allow his mistakes in
English to reflect doubt on his claim. In the case of Dr. Archer, he is considered one of the
foremost biblical apologists of our generation, and many of his explanations of biblical
discrepancies are based on linguistics. I considered it appropriate to allow the mistakes in his
letter to give readers pause to wonder about his linguistic abilities.

Finally, I would like to suggest that Mr. Goldberg go to the back issues of TSR and read the
articles of Robert Dornbusch (July/August 1998, pp. 9-10) and Wilhelm Schmitt (May/June
1996, pp. 7-8 and September/October 1996, pp. 2-3). These were all written by prison
inmates, and I had to make very few corrections in them, far fewer than I had to make in the
articles of the inerrantists listed above. For Mr. Goldberg's benefit, I am publishing
immediately below a completely unedited letter that coincidentally arrived in the same mail
delivery with his. The author is an inmate, so I'll let Mr. Goldberg decide if his
correspondence with inmates has given him an accurate opinion about their writing abilities.

An Example of Inmate Literacy...

Volume 1990 - 2002 Issue


Page 1357 of 2049
Skeptical Review Edited by Farrell Till
Although I have only read a few issues of The Skeptical Review, I have found it to be an
invaluable resource. It's well researched and thought out, which is something that I'm not
wholly used to.

I often have read complaints from Christians who insist that your newsletter is "satanic" or
whatever in your "From the Mailbag" column. Your responses to these letters give me the
impression that you are getting increasingly agitated at dealing with such arrogance. I,
therefore, thought that I could share some news with you to lighten your spirits, so to speak.

Having dealt with many of these same kind of arrogant people, I have noticed that many of
them do not even read the bible that they proclaim so avidly as "the word of god!" Oft times,
the most obsessive and demanding of arguments against anything that could even remotely be
identified as destructive to their theology comes from people who don't fully understand what
they believe to begin with. They are generally just sitting around, waiting for some preacher
to explain to them what they believe.

Ralph Waldo Emerson said that "God offers to every mind its choice between truth and
repose. Take what you please, you cannot have both." Unfortunately, most people tend to
choose the latter, because there is less work involved. There is only a brief few who are
willing to work towards increasing our knowledge and discover the thrill of uncovering truth.

Of course, this is nothing new. There are still those who cannot believe that the universe does
not revolve around us. However, there are always those who are willing to point the way
towards a new horizon.

(Christopher Westervelt, 94107/2501, Prairie Correction Facility, P. O. Box 500, Appleton,


MN 56208)

Volume 1990 - 2002 Issue


Page 1358 of 2049
Skeptical Review Edited by Farrell Till

Skeptical Review
Volume Ten, Number Two
March/April 1999
Farrell Till, editor

• "Another Prophecy Failure"


Farrell Till examines Ezekiel's prophecy against Egypt (chapters 29-32) and shows
that the total destruction and devastation predicted by Ezekiel failed to happen.
• "Till Is Batting Around .250 on Daniel"
Inerrantist Everette Hatcher III continues his efforts to prove that the book of Daniel
was written in the 6th century B. C. by an important Jewish official in the Babylonian
and Persian royal courts.
• "Hatcher Can Quote and List"
In Farrell Till's reply to Hatcher, he shows that most of Hatcher's "arguments"
consisted of appeals to fundamentalist authors whom he quoted and listed with only
sketchy comments of his own in support of their claims.
• "The Tyre Prophecy Again"
Farrell Till continues his discussion of alleged prophecy fulfillments that continue to
concern a former fundamentalist who is trying to put aside his belief in biblical
inspiration.
• "From the Mailbag"
Readers and editor Farrell Till exchange comments on a variety of issues.

Another Prophecy Failure

Volume 1990 - 2002 Issue


Page 1359 of 2049
Skeptical Review Edited by Farrell Till
Any discussion with fundamentalist Christians that questions the divine inspiration of the
Bible will almost always elicit a challenge to explain all of the prophecy fulfillments. Bruce
Weston's article in the previous issue of TSR (pp. 6-7) shows just how far the wool has been
pulled over the eyes of the sheep on this issue. Weston has been able to see various flaws in
the claim that the Bible is God's inspired word, but his rejection of this belief is apparently
being held back by a concern that prophecy fulfillment just may indicate that there is some
truth in the claim. An article on pages 10-11 (this issue) discusses Ezekiel's prophecy against
Tyre, which was one of Weston's chief concerns, and shows that only a vivid imagination
could see this as an example of exact prophecy fulfillment. In coming issues, we will continue
to discuss the other prophecy concerns that Weston listed in his article until it has become
clear that apologetic arguments based on remarkable prophecy fulfillments are without merit.

The article in this issue on the Tyre prophecy referred to Ezekiel's promise that
Nebuchadnezzar would be "given" Egypt as compensation for his failure to take Tyre as the
prophecy had predicted, but when the ensuing prophecy against Egypt is analyzed, it becomes
clear that it failed too. In a four-chapter tirade against Egypt, Ezekiel said that Yahweh would
give Nebuchadnezzar Egypt as "wages" for the labor he had expended on Tyre in an
unsuccessful siege (29:19-20). The devastation of Egypt was to be complete. The land would
be an "utter waste and a desolation" from Migdol (in the north) to the border of Ethiopia (in
the south). So thorough would the devastation be that "neither foot of man nor foot of beast
would pass through it, and it would be uninhabited for 40 years and the Egyptians scattered
among the nations (29:9-12). At the end of the 40 years, Yahweh would gather the Egyptians
back to their country from where they had been scattered, but Egypt would forever be "the
lowliest of kingdoms" (v: 15). It would never "exalt itself above the nations" and would not
"rule over the nations anymore" (v:15).

Needless to say, none of this ever happened. There are no historical records of a 40-year
period when Egypt was so desolate that neither animals nor humans inhabited it, and the
population of Egypt was never scattered among the nations and then regathered to its
homeland. It's political influence has fluctuated through the centuries, but there has never
been a time when it could have been considered the "lowliest of kingdoms." No self-
respecting biblicist, however, would allow minor details like these to deter him in his
insistence that the Bible is inerrant, so all sorts of attempts have been made to show that this
is not a prophecy failure.

The fulfillment is yet future: Some inerrantists admit that this prophecy has not been
fulfilled, but they insist that it will be someday. This explanation ignores some rather explicit
language in the prophecy. It began with Yahweh telling Ezekiel to "set [his] face against
Pharaoh king of Egypt" and "to prophesy against him" and to say, "Behold I am against you,
O Pharaoh, king of Egypt" (29:2-3). Specific language is also directed to "Pharaoh king of
Egypt" in 30:21-22, 25; 31:2, 18; and 32:2, 31-32. Furthermore, the prophecy was very clear
in stating that this desolation of Egypt would be done by Nebuchadnezzar, who would be
"brought in to destroy the land" and to "fill the land with the slain" (30:10-11). Needless to
say, the rule of the pharaohs ended in Egypt centuries ago, and Nebuchadnezzar has been
dead even longer, so if the total desolation of Egypt and scattering of its population did not
happen in that era, it is reasonable to say that the prophecy failed. Inerrantists, however, are
not reasonable when the integrity of the Bible is at stake, so some will go so far as to say that

Volume 1990 - 2002 Issue


Page 1360 of 2049
Skeptical Review Edited by Farrell Till
even though the rule of the pharaohs has ended, it will be restored someday, at which time
Yahweh will bring about the fulfillment of Ezekiel's prophecy, possibly by a ruler who will
come from the same region as Nebuchadnezzar.

Although seriously proposed by some inerrantists, this "explanation" is such a resort to


desperation that it hardly deserves comment. It makes Yahweh a petty, vindictive deity who
will punish Egyptians in the distant future for something that their ancestors did, and it makes
possible the explanation of any prophecy failure in any religion. Believers in the prophecy
could simply say that even though it has not yet been fulfilled, it will be "someday." That type
of "logic" may impress biblical fundamentalists, but rational people will see it for exactly
what it is--desperation to cling to belief in prophecies that have been discredited by time.

The prophecy was figurative in its meaning: This "explanation" may take two forms: (1)
Some contend that this prophecy was fulfilled but that critics of the Bible have not recognized
it because they have interpreted literally what Ezekiel conveyed in figurative language. They
quibble that he meant only to say that great damage would be inflicted on Egypt and that this
was done when Nebuchadnezzar invaded Egypt in 568/7 B. C. The fact that total devastation
of Egypt obviously didn't happen at that time (or any other time) doesn't matter to those who
hold to this view. By rationalizing that plain language in the Bible was actually "figurative,"
they are able to convince themselves that the prophecy was fulfilled. (2) Other proponents of
the figurative view number themselves with the futurists. They accept that the prophecy was
obviously predicting a total devastation of Egypt, and they admit that this has not happened
yet. They use the figurative argument to explain away not the descriptions of destruction but
Ezekiel's references to Nebuchadnezzar and the pharaoh's of Egypt. To them, it doesn't matter
that Nebuchadnezzar and the pharaohs are long gone, because they contend that these were
only "figures" or "symbols" of the rulers who will be in power when Yahweh finally brings
about the fulfillment of Ezekiel's prophecy against Egypt. This "explanation" of the prophecy
is really no better than the one that sees a futuristic restoration of the Egyptian pharaohs and
Babylon's former empire. It reduces the god Yahweh to a petty, vindictive deity who will
punish future Egyptians for what their ancestors did. It's most obvious flaw, however, is that it
resorts to unlikely scenarios to try to make the Bible not mean what it obviously says. In
rather plain language, Ezekiel predicted a total destruction and desolation of Egypt that would
last for 40 years. It never happened, and no amount of rationalization can make that failure a
success.

Till Is Batting Around .250 on Daniel


Everette Hatcher III

Home-run hitters are always the strike-out kings too, and Farrell Till seems to have missed the
mark around 75% of the time in his series of articles on Daniel. However, Till did connect

Volume 1990 - 2002 Issue


Page 1361 of 2049
Skeptical Review Edited by Farrell Till
some of the time. For instance, he made some good points in his article "Good History in the
Book of Daniel" (September/October 1998, pp. 9-11, 16). I agree with the majority of what
Till said, and it is obvious that he has studied long and hard concerning the historical events
mentioned in Daniel chapter eleven.

I have also noticed that no one in the field of biblical errancy can hold a candle to Till. I was
amused when I read some of the peculiar errors of interpretation made by Dennis McKinsey.
For instance, concerning Daniel 9:24-25 McKinsey stated that "the weeks referred to are real
weeks of seven days, not years" (Encyclopedia of Biblical Errancy, Prometheus Books, 1995,
p. 164), but even those who hold the critical view accept that the author of Daniel was
speaking of years (Robert A. Anderson, Signs and Wonders, International Theological
Commentary, Eerdmans, 1984, pp. 111-115; Isaac Asimov, "The Book of Daniel," Asimov's
Guide to the Bible, Doubleday, 1969, p. 613; John J. Collins, Daniel, Fortress, 1994, pp. 352-
356; Samuel Driver, The Book of Daniel: Cambridge Bible for Schools and Colleges,
University Press, 1900, p. 135; John Goldingay, Daniel: Word Biblical Commentaries, Word,
1989, p. 262; Louis F. Hartman and Alexander A. DiLella, The Book of Daniel, Anchor Bible,
1978, p. 250; Arthur Jeffery, "The Book of Daniel," Interpreter's Bible, 1956, p. 493; Andre
Lacocque, The Book of Daniel, John Knox, 1979, p. 191; James A. Montgomery, A Critical
and Exegetical Commentary on the Book of Daniel, International Critical Commentary, T.
and T. Clark, 1927, reprint, 1979, p. 376; John Joseph Owens, "Daniel," Broadman Bible
Commentary, 1971, p. 439; Norman Porteous, Daniel, Old Testament Library, 1965, p. 141;
W. Sibley Towner, Daniel, Interpretation: A Bible Commentary for Teaching and Preaching,
1984, p. 143; Daniel L. Smith-Christopher, "The Book of Daniel," The New Interpreter's
Bible, Vol. 7, 1996, p. 128; Brodrick D. Shepherd, Beasts, Horns, and the Anti-Christ, 1994,
p. 78; Frank Zindler, "Daniel in the Debunker's Den," American Atheist, October 1986, p. 59).

For instance, the critic Jeffery states:

Its substance is that the seventy weeks are to be understood as seventy hebdomads or weeks
of years; i.e., they represent 490 years, the conclusion of which will see the coming of the
end.
Seventy weeks of years: Lit., seventy weeks, which the sequel shows means weeks of years.
The Greeks and Romans had a similar idea of a week-year (Aristotle, Politics, VII.16; Attic
Nights, III.10). It is commonly thought that the writer derived this from Lev. 25:2; 26:18-35
(Jeffery, p. 493).

When I examine Till's view concerning inerrancy, I must give him this compliment: I admire
his logic. Till found himself "on an irreversible trajectory toward agnosticism" (Edward T.
Babinski, Leaving the Fold, Prometheus Books, 1995, p. 294) when he no longer believed in
the doctrine of inerrancy. Now I believe that Till is incorrect in his conclusion concerning
inerrancy, but I cannot fault his logic. It amazes me that so many professing Christians accept
this idea that the Book of Daniel is a fraud, but they still worship the God of the Bible. Many
Christian scholars (e.g., DiLella, pp. 53-54; Smith-Christopher, p. 22; Owens, p. 377; Towner,
pp. 44-46; Collins, p. 56) claim that a forgery may be used to teach great moral lessons. If I
ever became convinced that the Bible contained fraud and false prophecies, I would leave my
Christianity behind just as Till did.

Volume 1990 - 2002 Issue


Page 1362 of 2049
Skeptical Review Edited by Farrell Till
I do find it strange that Till has avoided criticizing these liberal Christian scholars for not
carrying their views on Daniel to their logical conclusions. Maybe it is because Till has been
pre-occupied in criticizing those in the religious right for their inconsistencies. I have noticed
that Till has constantly been pointing out misrepresentations and misquotes used by many
inerrantists that he has debated. I agree that many in the religious right have been guilty in this
area, and I have attempted to confront dozens of these leaders myself concerning this
("Questionable Quotes," The Freedom Writer, May/June 1997, pp. 8-9; "Fake Quotes," letter
to the editor, Skeptic Magazine, Vol. 5, No. 4, 1997, p. 39; "The Bible Code," letter to the
editor, The Skeptical Inquirer, March/April 1998, p. 65). However, Till was incorrect when he
accused me of misrepresenting and misquoting the scholars who hold to the 2nd-century B. C.
view ("The Inerrantist Way of Misrepresenting `Critics,'" March/April 1998, pp. 4-7, 16). Till
stated, "Available space will not allow me to discuss all of Hatcher's distorted and
misrepresented sources in a single article, and so I will follow this one with at least two
more..." (March/April 1998, p. 16).

Nowhere did I indicate that the critic Norman Porteous was "a proponent of the inerrantist
view of the 6th-century authorship" (p. 7). Yet Till repeatedly accuses me of misrepresenting
several critics in just this fashion (March/April, p. 7; May/June, p. 2; July/August, p. 14), but I
made it clear in the second paragraph that I was examining the views of critics "who hold to
the "Maccabean thesis" ("The Critics' Admissions Concerning Daniel," March/April, p. 2). I
have always tried to confront those who have been guilty of misquotations and
misrepresentations. Therefore it was especially painful to endure the titles Till chose: "The
Inerrantist Way of Misrepresenting `Critics,'" (March/April 1998, p. 4); "Deliberate
Misrepresentation After All," (May/June 1998, p. 2). Neither did I misquote any of these
critics. Till commented:

A familiar type of inerrantist distortion results from the omission of a qualifying but that
follows a fragmented quotation. The first part of the quotation appears to favor the inerrantist
view until the qualifying but statement is read. By eliminating the buts and howevers,
inerrantists try to leave the impression that certain scientists and scholars agree with them.
Hatcher did this in response to my claim that the writer of Daniel obviously "considered the
Median and Persian kingdoms to be separate empires." He quoted Dr. Samuel Driver as
having admitted, "In the book of Daniel the `Medes and Persians' are, it is true, sometimes
represented as united" (March/ April 1998, p. 2). I had seen this inerrantist tactic enough to
know that even without having read Driver's work, the parenthetical "it is true" indicated that
a qualifying but statement followed the fragment that Hatcher had quoted. When I was finally
able to check the context of the quotation, I found that I was right (May/June, p. 2).

Till implied that I left out essential information that distorts Driver's quote. However, the
operative word sometimes is included in the quotation I used. The word sometimes does not
mean always, and I in no way implied that I thought that Driver was admitting that the author
of Daniel always represented the Medes and Persians as united. In fact, I listed the only five
scriptures that Driver considered as picturing a combined empire (Daniel 5:28; 6:8,12,15; cf.
8:20), and then I pointed out that this admission contradicts the critical position that Driver
still held. I did not imply that Driver expressed agreement with my view that "the second
empire in Nebuchadnezzar's vision was a combined Medo-Persian empire" (May/June, p. 2),
but I was merely observing that this admission was fatal to Driver's position.

Volume 1990 - 2002 Issue


Page 1363 of 2049
Skeptical Review Edited by Farrell Till
I cannot see how giving the full text of Driver's quotation would adjust the meaning at all. My
whole argument involved showing that the textual evidence in Daniel clearly points to a
combined kingdom being pictured and even a critic like Driver had to admit that some verses
indicated that.

Evidently Till realized the importance of this point because twice he quoted a passage from
the critic H. H. Rowley that addressed this very issue:

For [sic] a sixth-century person, who not only lived through the events of the period, but took
a leading part in them, could not have made so gross an error as our author made in
introducing Darius the Mede between Belshazzar and Cyrus. Nor could he have supposed that
a Median empire stood between the Babylonian and the Persian (University of Wales Press,
1935, p. 175, quoted in TSR, March/April 1998, p. 5) & September/October 1998, p. 9).

Till commented that "this critical opinion of Daniel has become the underpinning of the
Maccabean view of its authorship" (September/October 1998, p. 9). I would agree that many
critics have taken the position that the author of Daniel mistakenly had Babylon falling to a
Median empire (Anderson, pp. 22-23; Asimov, p. 602; Collins, pp. 166-167; Philip R. Davies,
Daniel, Sheffield: JSOT Press, 1985, p.26; Driver, p. 52 of introduction; Raymond Hammer,
The Book of Daniel, Cambridge University Press, 1976, p. 8; Hartman and DiLella, p. 50;
Jeffery, pp. 387-388; Pamela J. Milne, "The Book of Daniel," Harper Collins Study Bible, ed.
Wayne A. Meeks, 1993, p. 1318; Robert H. Pfeiffer, Introduction to the Old Testament, 1948,
p. 757; Porteous, p. 47; H. H. Rowley, Darius the Mede and the Four world Empires in the
Book of Daniel, 1935; p. 175; Daniel L. Smith-Christopher, p. 88; Towner, p. 70; Zindler, p.
58). These critics realize that if it can be demonstrated that the writer of Daniel envisioned a
rule by the Medes, then these critics can point to all the final "prophetic" fulfillments in the
Greek period since Greece would be the fourth kingdom. Moreover, they can also accuse
Daniel of a gross historical error. However, traditionalists claim some of the prophecies refer
to events that go past the Greek period, and there is only the problem of the missing person,
"Darius the Mede" and not a missing empire.

Traditionalists take the view that the author of the book of Daniel knew very well there was
no intermediate rule by the Medes. The conservative Stephen Miller correctly noted:

To suggest that any semi-educated Jew of the Maccabean period could be ignorant of the fact
that it was Cyrus the Persian who conquered the great Babylonian Empire and allowed the
Jewish captives to return to their homeland is not reasonable. Moreover, the Book of Ezra (cf.
1:1 ff.), which undoubtedly was at the writer's disposal, specifically declares that Cyrus
released the Jews from captivity in Babylon. It also understands Darius I to have ruled Persia
long after Cyrus (Ezra 4-5) (Daniel: The New American Commentary, Broadman and Holman
Publishers, 1994, p. 174).

The critics want us to believe that Daniel was written in the 2nd-century B. C., and that is the
reason it has inaccuracies concerning 6th-century B. C. events. However, any educated Jew in
the 2nd century would have known that Cyrus the Persian defeated Babylon.

Volume 1990 - 2002 Issue


Page 1364 of 2049
Skeptical Review Edited by Farrell Till
Let me address three of the historical situations that Till spends a great deal of time
discussing:

(1) Did the author of Daniel suppose that Darius Hystaspis preceded Cyrus? Till commented:

In 9:1, the writer of Daniel described the mysterious "Darius the Mede" as the "son of
Ahasuerus, of the seed of the Medes," but Ahasuerus, (better known as Xerxes) was king of
Persia from 485-465 B. C., so it isn't at all possible that "Darius the Mede," who allegedly
reigned in Babylon in 539 B.C., was the son of someone who had not yet been born.
Ahasuerus was the Persian king who allegedly made Esther his queen in the book named after
this Jewish heroine. Since his father was Darius the Great, the writer of Daniel may have
confused his Dariuses and anachronistically made a son of Darius the Great the king who had
captured Babylon. At any rate, he made a historical mistake that would be understandable for
an author writing four centuries later, but it is not a mistake that we could reasonably expect
an important contemporary official of Babylon to make (July/August 1998, p. 8; Rowley, pp.
57-58; DiLella, p. 36).

The critic John Goldingay admits that "Ahasuerus" probably is a title and not a personal name
(p. 239). Daniel 9:1 also discusses Darius the Mede, and many believe that "Darius the Mede"
is not a personal name but a title. This will be touched on later.

I would agree that if the author of Daniel made the historical blunder concerning the
intermediate reign by the Medes, in such a case, he could not be "an important contemporary
official of Babylon" (July/August 1998, p. 8). However, I would go one step further and insist
that he could not have been familiar with the other Old Testament books like Ezra. Remember
that the Dead Sea Scrolls include portions of both Daniel and Ezra. This indicates that the
critics who claim that the author of the book of Daniel had a Hasidic origin have a lot of
explaining to do (Towner, p. 7; DiLella, p. 45). DiLella states, "It is generally admitted that
the Essenes had their origin in the Hasidic movements that flourished in early 2nd-century B.
C. Judaism" (p. 45). These Essenes copied the Dead Sea Scrolls, and that indicates that they
had access to copies of the Old Testament scriptures for many generations. J.J. Collins
comments, "Fragments of eight mss of Daniel have been identified. The oldest of these,
4QDan. is dated by Frank Cross to `the late 2nd-century' B. C. E., `no more than about a half
century younger than the autograph'" (Collins, p. 2).

This presents two problems for the critics. How could the Qumran community accept Daniel
as Scripture if it incorrectly pictured Darius Hystaspis preceding Cyrus? Copies of Ezra they
possessed contradicted this. Also how could the Qumran community accept Daniel as
Scripture only fifty years after its composition? It is for this very reason that many of the
canonical psalms found there were redated.

The critic W. H. Brownlee asserted: "It would seem that we should abandon the idea of any of
the canonical psalms being of Maccabean date, for each song had to win its way in the esteem
of the people before it could be included in the sacred compilation of the Psalter. Immediate
entree for any of them is highly improbable" (The Meaning of the Qumran Scrolls for the
Bible, Oxford University Press, 1964, p. 30). Yet concerning Daniel, Brownlee accepts the
Maccabean date (p.36). The obvious question is: How can one theory push the date of a psalm

Volume 1990 - 2002 Issue


Page 1365 of 2049
Skeptical Review Edited by Farrell Till
back 200 years, but this same theory, when applied to Daniel, allow only 50 years? The
answer is that Brownlee was firmly committed to the critical assumption that Daniel could not
have been written before 164 B. C. His naturalistic presuppositions were getting in the way of
his ability to give the objective analysis.

(2) Is there a possible answer to the identity of "Darius the Mede"? Till wrongly assumed that
I hold the view that Darius was a governor appointed by Cyrus (May/June 1998, p. 2). While I
don't dismiss that possibility, I do favor a different view. I do not claim dogmatically that this
view is true, but it certainly is a realistic possibility. Many evangelicals have put forth the
theory that Darius is a title for Cyrus (D. J. Wiseman, "Some Historical Problems in the Book
of Daniel," Notes on Some Problems in the Book of Daniel, Tyndale, 1970, pp. 9-16; J.M.
Bulman, "The Identification of Darius the Mede," Westminster Theological Journal, Volume
35, 1973, pp. 247-267; J.G. Baldwin, Daniel, Tyndale Old Testament Commentaries,
InterVarsity, 1978, pp. 26-28, 127). Dual titles were not uncommon. Daniel and his friends
had dual names. Kings were known by two names at times. For instance, 1 Chronicles 5:26
reads, "So the God of Israel stirred up the spirit of Pul King of Assyria, even [Hebrew
conjunction waw] the spirit of Tiglath-Pileser King of Assyria." We now know that Assyrian
records indicate that Pul was Tiglath-Pileser's native name (James B. Pritchard [ed.], Ancient
Near Eastern Texts Relating to the Old Testament, Princeton University Press, 1950, p. 272).
Likewise, Wiseman translates Daniel 6:28, "Daniel prospered in the reign of Darius, even
[Aramaic conjunction waw] the reign of Cyrus the Persian" (Wiseman, p. 12). If Wiseman is
correct on this translation, then it may be a good explanation for this missing person case. The
critic Isaac Asimov did note that Cyrus was "indeed about 62 years old at this time" (p. 608),
and Daniel 5:31 says that Darius was 62.

The conservative Stephen Miller stated:

Bulman reasonably suggests that the author preferred the title Darius the Mede because it had
particular significance for the Jews (Bulman, p. 263). Both Isaiah (13:17) and Jeremiah
(51:11, 28) had predicted the downfall of Babylon to the Medes, and Daniel employed the
title to emphasize the fulfillment of these prophecies. Yet Daniel also used the title Cyrus the
Persian in order to explain the king's relationship to the world of that day he was ruler over
the whole Medo-Persian Empire. "The author may have assumed that 6:28 would make the
identification clear enough for the circle addressed" (Bulman, p. 252; Miller, pp. 175-176).

In fact, the critic Brian E. Colless concluded, "Everything seems to point to the same
conclusion: Darius the Mede is synonymous with Cyrus the Persian in the Book of Daniel"
("Cyrus the Persian as Darius the Mede in the Book of Daniel," Journal for the Study of the
Old Testament, Vol. 56, 1992, pp. 113-126).

I must admit that the argument concerning "Darius the Mede" is the most difficult problem
remaining for the inerrantist to resolve. However, this problem involves only the identity of
"Darius the Mede," and it does not concern the incorrect view that the Medes reigned between
the Babylonians and Persians. Also I must point out that Till himself admits that appealing "to
historical silence is considered a weak type of argumentation" (July/August 1998, p. 8). Yet
Till considers this type of evidence concerning Daniel "very compelling" partly because
Ezekiel makes no "unequivocal" reference to Daniel. Till asserted:

Volume 1990 - 2002 Issue


Page 1366 of 2049
Skeptical Review Edited by Farrell Till
Ezekiel did mention the name Daniel three times, but these were in contexts where this person
was associated with ancient biblical heroes like Noah and Job (14:14, 20; 28:3). Since the
name is spelled "Danel" in some texts, this Daniel is thought to be the "Danel" of Ugaritic
legend found on clay tablets excavated at Ras Shamra, so it seems rather strange that Ezekiel
would have written 48 chapters without once referring to a captive who had become a
prominent Babylonian official (July/August 1998, pp. 8, 10).

Till dismissed the three times Ezekiel mentions Daniel because Ezekiel is speaking of a
Daniel spelled "Danel" referred to in Ugaritic literature around the 14th century B. C. Other
critics agree (e.g., Hammer, p. 3; Owens, p. 374). However, the context in Ezekiel seems to
contradict this view. Ezekiel 14 is a message against the idolatrous elders. The conservative
H. Dressler asks, "Is it conceivable that the same prophet would choose a Phoenician-
Canaanite devotee of Baal as his outstanding example of righteousness? Within the context of
Ezekiel this seems to be a preposterous suggestion" (H. H. P. Dressler, "The Identification of
the Ugaritic Dnil with the Daniel of Ezekiel," Vetus Testamentum, Vol. 29, 1979, p. 159).
Furthermore, even the critic John Day admits "there are no linguistic objections to the
equation of the Daniel of Ezekiel XIV:14,20 and the hero of the book of Daniel. Ezekiel
simply spells the name without the vowel letter yodh." Day made these comments in an article
maintaining the critical conclusion that Ezekiel is referring to the Ugaritic Danel ("The Daniel
of Ugarit and Ezekiel and the Hero of the Book of Daniel," Vetus Testamentum, Vol. 30,
1980, pp. 174-184).

Critics seem never to learn. Earlier there was "very compelling" evidence from silence that
Belshazzar never existed. The conservative scholar Alan Millard stated:

Nebuchadnezzar had, of course, ruled over Babylon, but Belshazzar's name was nowhere to
be found outside the Biblical text. The Greek chroniclers who had preserved lists of ancient
kings identified Nabonidus, a successor to Nebuchadnezzar, as the last native ruler of
Babylon; Belshazzar was not even mentioned. Belshazzar, declared one commentator named
Ferdinand Hitzig in 1850, was "obviously a figment of the Jewish writer's imagination"
(Ferdinand Hitzig, Das Buch Daniel, Leipzig: Weidman, 1850, p. 75, as quoted by Millard,
"Daniel and Belshazzar in History," Biblical Archaeology Review, May/June 1985, pp. 74-
75).

It was in this atmosphere that Albert Barnes finished his commentary on December 26, 1851.
He could have put his faith in the current evidence of the day or the unchangeable word of
God. His choice was clear. He asserted:

The testimony of Daniel in the book before us should not be set aside by the statement of
Berosus, or by the other confused accounts which have come down to us. For anything that
appears to the contrary, the authority of Daniel is as good as that of Berosus, and he is as
worthy of belief. Living in Babylon and through a great part of the reigns of this dynasty;
present at the taking of Babylon, and intimate at court; honoured by some of these princes
more than any other man in the realm, there is no reason why he should not have had access
to the means of information on the subject, and no reason why it should not be supposed that
he has given a fair record of what actually occurred" (Notes, Critical, Illustrative, and
Practical on the Book of Daniel, Leavitt and Allen, 1858, p. 237).

Volume 1990 - 2002 Issue


Page 1367 of 2049
Skeptical Review Edited by Farrell Till
Barnes considered God's unchangeable word more reliable than historians, and Alan Millard
pointed out that historians soon after made some changes:

Then, in 1854, a British consul named J. G. Taylor explored some ruins in southern Iraq on
behalf of the British Museum. He dug into a great mud-brick tower that was part of the temple
of the moon god that dominated the city. Taylor found several small clay cylinders buried in
the brickwork, each about four inches long, inscribed with 60 or 50 lines of cuneiform
writing. When Taylor took the cylinders back to Baghdad, he showed them to his colleagues
(see E. Sollberger, "Mr. Taylor in Chaldaea," Anatolian Studies, Vol. 22, 1972, pp. 129-139).
Fortunately, his senior colleague was Sir Henry Rawlinson, who was one of those who had
deciphered the Babylonian cuneiform script. Rawlinson was able to read the writing on the
clay cylinders.
The inscriptions had been written at the command of Nabonidus, king of Babylon from 555 to
539 B.C. The king had repaired the temple tower, and the clay cylinders commemorated that
fact. The inscriptions proved that the ruined tower was the temple of the city of Ur. The words
were a prayer for the long life and good health of Nabonidus and for his eldest son. The name
of that son, clearly written, was Belshazzar!
Here was clear proof that an important person named Belshazzar lived in Babylon during the
last years of the city's independence. So Belshazzar was not an entirely imaginary figure (pp.
74-75).

Therefore, since the critics have been routed concerning the existence of Belshazzar, they
have decided to turn to other arguments concerning Belshazzar. Till picked up on one of the
weaker arguments when he commented:

If Daniel achieved such prominence in Nebuchadnezzar's kingdom, he would have surely


been familiar with the king's family, but in chapter five, the writer of the story referred to
Nebuchadnezzar five times as the "father" of Belshazzar....
Hatcher no doubt will parrot the inerrantist line and contend that the words father and son
were not being used literally in this story but only figuratively in the sense of "ancestor" and
"descendant," as when Abraham was referred to as the "father" of all Jews (Isaiah 51:2), and
as Jesus was called the "son of David" (Matt. 1:1). The examples are hardly parallel, however,
because Abraham was separated by centuries from the Jews of Isaiah's time, as Jesus was
separated in time from David sufficiently for readers of such texts as these to know beyond
reasonable doubt that father and son were being used figuratively (July/August 1998, p. 7).

Till is unaware of two biblical facts: (1) There is no word for grandfather in Hebrew or
Aramaic. The word father could refer to a grandfather as in the case of Abraham and Jacob
(Gen. 28:13; 32:9) or even to a great, great grandfather as in the case of David and Asa (1
Kings 15:10-13). (2) The term son can also mean successor. It is used this way in the Bible (1
Kings 20:35; 2 Kings 2:12; Robert Dick Wilson, Studies in the Book of Daniel, Grand Rapids:
Baker, reprint, 1979, Vol. 1, pp. 117-118). Also it is used this way in the "Black Obelisk" of
Shalmaneser III (c. 830 B.C.) when Jehu is called the "son of Omri" even though they were
not related (James B. Pritchard [ed.], Ancient Near Eastern Texts Relating to the Old
Testament, 2nd ed. Princeton University Press, 1955, p. 281). Similar usage in Egypt has been
found. In the Westcar Papyrus (dating from the Hyksos period), King Keb-ka of the Third
Dynasty is referred to as the father of King Khufu of the Fourth Dy nasty, a full century later.

Volume 1990 - 2002 Issue


Page 1368 of 2049
Skeptical Review Edited by Farrell Till
Daniel also followed this ancient custom of the time which was to recognize the king of
Babylon as the "son" (or successor) of Nebuchadnezzar. No wonder the critic Philip R.
Davies concluded, "The literal meaning of `son' should not be pressed..." (Davies, Daniel, p.
31).

Is there any direct textual evidence that indicates that the writer of Daniel knew Babylon fell
to Persia? Till stated:

Hatcher cited (p. 2, TSR Vol. 9.2) Porteous's commentary on Daniel from The Old Testament
Library (Westminster Press, 1965) in an attempt to make a dubious pun in Daniel 5:28 imply
that the writer of Daniel knew that Persia conquered Babylon. In other words, Hatcher's case
is so tenuous that he can't produce direct textual evidence that the writer of Daniel knew that
Babylon fell to Persia; he has to resort to claiming that the writer of Daniel `punningly'
implied it (TSR, Vol. 9.2, p. 7).

There is plenty of good textual evidence that the writer of Daniel knew that Babylon fell to a
combined empire made of the Medes and Persians. However, the critics cannot afford to
accept this evidence because they would have to admit there has been real prophecy. For
instance, many critics will admit that peres in Daniel 5:28 is a possible pun for Persia. Arthur
Jeffery states, "Moreover, since prs could also be pointed to mean `Persians,' it can refer to
the giving of the kingdom to the Persians; indeed, Bauer's suggestion allows him to give
Daniel's interpretation as `He has numbered! He has weighed! He has divided! The Persians!'"
(Jeffery, p. 432).

The critic James A. Montgomery noted:

Here a balanced phrase is obtained by finding a double paranomasia [sic] in the mystic word,
i. e., division and Persia. Were these ominous words first assembled and applied by our
narrator; or did he take them from some source and adapt them to his interpretation (so Bev.)?
It is to be noted that the play of words gives `Persia,' not `Media,' despite the fact that in
immediate sequence it is Darius the Mede who destroys the kingdom; the enigma is based on
the correct historical tradition of Cyrus' conquest" (p. 263).

Therefore, several critics will admit that Daniel 5:28 could be implying that the division of
Babylon would be done by the Persian armies (Porteous, p. 81). Nevertheless, the critics
usually give an alternative interpretation based on the work of Clermont-Ganneau in 1886
(Owens, p. 410; Collins, pp. 250-252; Montgomery, p. 263, Jeffery, p. 432; Porteous, p. 81;
Hartman, pp. 189-190; Driver, p. 69). The critic Robert A. Anderson commented:

Clermont-Ganneau advanced the thesis that the terms are measurements of weight, namely,
mina, tekel (the Aramaic equivalent of shekel), and peres. By this means the motif of
successful kingdoms already encountered in chapter 2, and which features so prominently in
the second half of the book, could be applied to the inscription, albeit in a modified form. The
subjects could be the last kings of the neo-Babylonian empire. It must be admitted that all this
is in the area of speculation. Fuller treatment is given in the commentaries of Hartman and
Lacocque. When we turn to the explanation in vv. 26-28 we are at least on firm ground (p.
61).

Volume 1990 - 2002 Issue


Page 1369 of 2049
Skeptical Review Edited by Farrell Till
Thus Anderson admits that the theory put forth by Clermont-Ganneau is "speculation." The
critic W. H. Brownlee goes even further. He observes, "There is one fatal weakness to this
method of interpreting the handwriting on the wall: It is not so interpreted in the Book of
Daniel itself" (Brownlee, p. 41)! Most critics don't want to admit the possibility that the
author of Daniel correctly thought that Babylon was conquered by a combined kingdom of the
Medes and the Persians. Therefore, they have to avoid taking Daniel 5:28 to its logical
conclusion. The conservative Gleason Archer stated:

The author of Daniel believed that Belshazzar was conquered by a coalition of Medes and
Persians; in Daniel 5:28 the whole point of the word play is that the Persians were about to
take over the kingdom directly from the Babylonians: "Peres: Your kingdom is divided
[prisat, from the verb pras, `separate'] and given to the Medes and Persians [paras]" (5:28). It
is quite apparent that only the Persians fit into this word play (P-R-S are the three consonants
involved in all three: PeReS, PeRiSat, PaRaS). The reason the Medes are mentioned first in
the phrase "the Medes and Persians" here is that historically the Persians had earlier been
subject to the Medes, until Cyrus defeated his uncle King Astyages of the Median Empire
back in 550 B.C. ("Daniel," Expositor's Bible Commentary, Grand Rapids: Zondervan, 1985,
pp. 16-17).

The passage that destroys the critical view completely is Daniel 8:1-20. The critic Raymond
Hammer admits that verse 3 "indicates a knowledge of the combined Medo-Persian Empire,
although elsewhere we have seen a tendency to think of Median and Persian empires as
separate entities" (Hammer, p. 84; Driver, p. 29). In Daniel 8:20 the ram with two horns is
"the kings of Media Persia." The critics do not want to admit there are many parallels between
the bear in chapter seven and the ram in chapter eight. Persia arose to be stronger than Media
in the alliance, and that is symbolized by both the bear and the ram being unbalanced (7:51,
8:3). Media-Persia's three major victories were over Babylon (539 B. C.), Lydia (546 B. C.),
and Egypt (525 B. C.). This is pictured by the three ribs in the bear's mouth (7:5b) while the
ram ran off in three directions to do battle (8:4a). The critics simply have no idea what the
three ribs symbolize (Jeffery, p. 454; Collins, p. 298; Driver, p. 82; Porteous, p. 105; R.A.
Anderson, p. 79). L. F. Hartman comments that "the effort of commentators to explain why
`three ribs should be in the mouth of a beast' have proved futile" (p. 205). Hartman and his
fellow critics have come up empty because they insist on making the bear a symbol of the
Median empire. There is no textual evidence to support this view. In a letter dated October 23,
1998, the conservative William Shea commented:

It is interesting to note that all of these arguments on the Daniel of the 2nd century B. C. go
back to the Neo-Platonist philosopher Porphyry in the 5th century A. D. Porphyry, however,
saw clearly that there was no separate Median kingdom, so his sequence was Babylon, Medo-
Persia, Greece I and Greece II. He had to shorten the sequence to get it to end up with Greece
and not Rome. The adaptation of dividing Media from Persia is a modern phenomenon,
worked out in the 18th and 19th centuries.

There is plenty of direct textual evidence in the book of Daniel that indicates that Babylon fell
to a combined Medo-Persian empire. Therefore, critics would be wise to stop insisting that
Daniel envisions a separate rule by the Medes.

Volume 1990 - 2002 Issue


Page 1370 of 2049
Skeptical Review Edited by Farrell Till
Another area of textual evidence that supports the Maccabean thesis according to the critics is
the late date "objective" scholars attribute to the languages used in the Book of Daniel. Till
stated:

In the very first paragraph of the introduction to his commentary, Porteous said, "The
linguistic evidence and the fact that the visions reveal a vague knowledge of the Babylonian
and Persian periods and an increasingly accurate knowledge of the Greek period up to and
including the reign of Antiochus Epiphanes, with the exception of the closing events of that
reign, suggest a date for the book shortly before 164 B. C. (March/April 1998, pp. 7,16,
emphasis added).

I wish Till would specifically indicate which linguistic evidence he would put forth as
significant. Earlier he cited the Aramaic (TSR, Vol. 4.3, p.13), but I dealt with that in my
previous article (March/April 1998, p. 3).

Gerhard F. Hasel noted that "several recent historical-critical commentaries have dropped the
argument from the Hebrew language for the late dating of the book of Daniel" (D. S. Russell,
A. Lacocque, J. J. Collins, W.S. Towner, and others; "Establishing a Date for the Book of
Daniel," Symposium on Daniel, ed., Frank B. Holbrook, Washington, D. C.: Biblical Research
Institute, 1986, p. 140).

William Shea has commented on Till's view concerning the date of authorship of the Book of
Daniel:

Till is behind the times in his view of the Aramaic in Daniel as Maccabean. No reputable
scholar that I know of at the present time holds that opinion. The reason for it is twofold.
First, the discovery of more and more Aramaic texts from Qumran. These have pushed the
date of Daniel backward, earlier, because Daniel writes a kind of Aramaic that is earlier than
Qumran's earliest Aramaic text, the Job Targum.
Second, more and more Aramaic inscriptions have been found and published and these have
been helpful in pulling Daniel's Aramaic earlier. So that now it is admitted that Daniel's
Aramaic is Imperial, not Maccabean. But that still leaves a range from the 7th to the 4th
century B. C. It does, however, rule out Till's late date (Letter dated October 23, 1998).

Therefore, many of the most respected Bible critics have moved to the position that only the
last six chapters definitely originated during the time of the Maccabees, and they hold that the
previous chapters initially were written during the Persian period.

The critic Philip R. Davies observed:

The progress of research on the book of Daniel in recent years has been marked by the
appearance of several major commentaries as well as articles and, especially, one very
important study. While these studies illustrate a variety of approaches to the book, they all
accept what has become a universally recognized distinction, namely between the two parts of
the book which contain respectively tales and visions. According to nearly every modern
commentator, the tales of chapters 1-6 are originally products of a Jewish community in a
Gentile environment, whose concerns were rather different from those of Jews who read these

Volume 1990 - 2002 Issue


Page 1371 of 2049
Skeptical Review Edited by Farrell Till
tales in Palestine in the Maccabean period (The most recent and detailed treatments are W.
Lee Humphreys, "A Life Style for Diaspora: A Study of the Tales of Esther and Daniel,"
Journal of Biblical Literature, Vol. 92, 1973, pp. 211-223; J. J. Collins, "The Court Tales in
Daniel and the Development of Jewish Apocalyptic, Journal of Biblical Literature, Vol. 94,
1975, pp. 218-234; H. P. Muller, "Marchen, Legende und Enderwartung," Vetus
Testamentum, Vol. 26, 1976, pp. 338-350); the visions, which were written during this period
are of a different genre, "apocalyptic...." We can be reasonably confident that the stories about
Daniel and his friends in chapters 1-6 were in existence before the visions were composed. To
begin with, the attitude to Gentiles and Gentile monarchs in particular hardly reflects a
Maccabean context (Philip R. Davies, "Eschatology in the Book of Daniel," Journal for the
Study of the Old Testament, Vol. 17, 1980, p. 33).

Therefore, it appears that Till is out of step with most of the modern critical scholarship
concerning the date of authorship of the first six chapters of Daniel because Till believes all of
Daniel was written during the Maccabean period. In "Convenient Coincidences in the Book of
Daniel," (September/October 1998, p. 1), Till makes the case that Daniel chapter one belongs
to the Maccabean period because "it's hard to believe that a book actually written in the 6th
century B. C. would have very conveniently contained a story so clearly parallel to a religious
dietary crisis that would happen four centuries later." According to Till, chapter three is late
because it is a story "that 2nd-century B. C. Jews suffering such persecution would have
easily related to" (p. 1), and chapter five is late because it involves the desecration of sacred
vessels. Till stated:

Such convenient coincidences as these in the story of a 6th-century B. C. captive who,


choosing to serve Yahweh faithfully, was rewarded with a position of prominence in the
kingdom of his captors complements the mountain of other evidence that indicates the author
of this book was actually a 2nd-century writer who wanted his contemporaries to believe that
a prophet living long ago in another difficult period of Jewish history had foreseen their
sufferings and predicted that they would triumph over oppression (p.1, 16).

I have clearly demonstrated that there is no mountain of legitimate "evidence that indicates
the author of this book [Daniel] was actually a 2nd-century writer." In Till's provocative
article "Primary Colors of the Bible" (July/August 1998, pp. 1, 5) he asserted, "In past issues
of TSR, fundamentalist views about the authorship of the books of Jeremiah and Daniel have
been challenged by documentation from the works of reputable scholars...." Till's article
argues that linguistic evidence should not be underrated. Yet Till has not offered any specific
linguistic evidence concerning Daniel in our current debate!! Instead, much of Till's focus is
on attacking my methods of writing. For instance, Till observed:

When I received the article, my first inclination was not to publish it because it is little more
than one appeal to authority after the other strung out over two and a half pages. In other
words, Hatcher basically argued throughout his article that the 2nd-century B. C. dating of the
book of Daniel is wrong and the 6th-century B. C. dating correct, because certain scholars say
so. In so doing, he pieced together various quotations, obviously lifted unchecked from
fundamentalist sources, and paraded them before us as if quoting a "scholar" necessarily
proves anything. I have said many times in TSR and its Internet list that anyone committed to
a religious position can always find books published by authors who share that belief, so if

Volume 1990 - 2002 Issue


Page 1372 of 2049
Skeptical Review Edited by Farrell Till
quoting "scholars" constituted proof of one's position, anyone could prove any belief to be
true.... There is much more to biblical apologetics than just citing "scholars," but apparently
Hatcher does not realize this (March/April 1998, pp. 4-5).

I am not an archaeologist or a linguist, but that doesn't stop me from discussing archaeology
or linguistics. I must quote experts in these fields, and in this sense I must use authorities in
my articles. Also many times other scholars articulate things in such a clear way that I would
rather quote them directly than put it in my own words. The real issue Till is getting at
concerns the strength of one's argument. Is there credible evidence to back up an argument or
not? Here I agree that one should not appeal to authority without having a credible argument.
However, my arguments are credible. Go back and closely examine the evidence I provided
for these following arguments: (1) Daniel does not picture the intermediate Median empire
that Till claims exists in the book of Daniel. (2) The Aramaic of Daniel does not point to a
2nd-century date of authorship. (3) Daniel did not necessarily err when he referred to
Belshazzar as Nebuchadnezzar's son because in the Near East the word son could also mean
successor.

I wish I had space to respond to Till's accusation concerning the apparent dating problems
found in Daniel 1:1-5. Also I wish I could spend more time on the archaeological evidence
that supports the 6th-century view. Today there is greater evidence than ever before that the
author of Daniel was an eyewitness of the events of the 6th-century B. C. Nevertheless, two
hundred years ago sufficient evidence existed that caused the critic Thomas Paine to note,
"Are they [the books of Ezekiel and Daniel] genuine? I am more inclined to believe that they
were than that they were not... in the manner which the books ascribed to Ezekiel and Daniel
are written agrees with the condition these men were in at the time of writing them" (The Age
of Reason, Secaucus, N. J.: Citadel Press, reprint, 1974, p. 150, emphasis added). Of course,
Paine denied there was any clarity concerning the prophecies in Daniel, but this is why I
enjoyed Till's last article so much ("Good History in the Book of Daniel," September/October
1998, pp. 9-11, 16). Till correctly observed that Daniel chapter eleven contains many
references to actual events that took place during the Greek period.

Comments like that brought Till's batting average up to .250 on Daniel. However, .250 is
nothing to brag about in the area of biblical interpretation.

(Everette Hatcher III, P. O. Box 23416, Little Rock, AR 72221)

Hatcher Can Quote and List


Farrell Till

Volume 1990 - 2002 Issue


Page 1373 of 2049
Skeptical Review Edited by Farrell Till
I feel compelled to begin my reply to Everette Hatcher's article with an apology for subjecting
readers to the tedious boredom of a long string of quotations and book lists with little
argumentation or comment of substance in between. Although Hatcher had agreed, as I
mentioned in the November/December 1998 issue, to refrain from argumentation by constant
appeals to authority, we saw in the foregoing article that nothing in his "apologetic" style has
changed. He responded to my articles as if he thought that bombarding us with citation after
citation and quotation after quotation from books and articles that agree with his view on the
authorship of Daniel would somehow prove that he is right. At times, he seemed to list books
purely for the sake of listing. In his second paragraph, for example, he mentioned in reference
to the 70 weeks of Daniel 9:24 what he thinks are "peculiar errors of interpretation made by
Dennis McKinsey" and then proceeded to list an entire column of nothing but books that
disagree with McKinsey's view on this issue, as if what McKinsey thinks about a subject that
hasn't even been a point of contention in my exchanges with Hatcher is relevant to this
discussion about the dating of the book of Daniel. Furthermore, the fact that Hatcher knows of
a long list of writers who disagree with McKinsey on this point proves only that there are
writers who disagree with McKinsey; it doesn't prove that these writers are right and
McKinsey is wrong.

The amount of space that Hatcher devoted to this tangent is rather mystifying. I suppose he
thought that he wasn't faring too well in debating relevant issues and so he would try to get in
a few licks at McKinsey in the space I am giving him to defend his position on Daniel.
Because my article outlining the new editorial policy on articles submitted to TSR
(November/December 1998, pp. 4, 9) was published after Hatcher had completed the article
above, I have extended him the courtesy of publishing it. If I were to talk to Hatcher face to
face, however, I would borrow an expression from George Bush and tell him to read my lips:
I will publish no more articles from him or anyone else that rely primarily on the fallacy of
appealing to authorities. I suggest that he review the editorial policy that I outlined in the
November/December issue and decide if he wants to respect it. If he does, I'll be glad to give
him a forum for his views. If he doesn't, he can look elsewhere for someone to publish his
articles. The appeal to authority, especially authorities who are intent on defending their
religious views, is a logical fallacy, and I will no longer accommodate those who have
nothing more than this to offer in support of their beliefs. Certainly, I will not subject my
subscribers to the boredom of an entire column containing nothing but book lists. They have
paid their money to read exchanges of arguments on the inerrancy/errancy views of the Bible
and not to wade through book lists and quotations from books, letters, and commentaries that
contain only expressions of opinion without argumentation to support those opinions.

An example of this type of forensic fallacy in Hatcher's articles is found in his quoting of
comments on Daniel that he has solicited from various conservative seminarians, who
(surprise! surprise!) agree with Hatcher's traditionalist view, but offer no real evidence to
support their assertions. One such quotation from William Shea, whom Hatcher described as a
"conservative," appears on page five of this issue:

It is interesting to note that all of these arguments on the Daniel of the 2nd century B. C. go
back to the Neo-Platonist philosopher Porphyry in the 5th century A. D. Porphyry, however,
saw clearly that there was no separate Median kingdom, so his sequence was Babylon, Medo-
Persia, Greece I and Greece II. He had to shorten the sequence to get it to end up with Greece

Volume 1990 - 2002 Issue


Page 1374 of 2049
Skeptical Review Edited by Farrell Till
and not Rome. The adaptation of dividing Media from Persia is a modern phenomenon,
worked out in the 18th and 19th centuries.

Before saying anything else about his statement, I have to point out how misinformed Shea is.
If Hatcher has quoted him correctly, he called Porphyry a 5th-century [A. D.] philosopher, but
actually Porphyry died in A. D. 305, so he barely survived the 3rd century. Hatcher
apparently quoted Shea as some kind of authority on the book of Daniel, but a dating mistake
of two centuries in a matter like when Porphyry lived and wrote doesn't give me much
confidence in his opinion on the dating of Daniel. Anyway, let's look at what we have in
Shea's statement. He seems to be arguing that Porphyry, a "5th-century" [sic] opponent of
Christianity, thought that the second kingdom of Daniel's vision was a unified Medo-Persian
kingdom, and so this must mean that the second kingdom actually was a unified Medo-
Persian kingdom, but the flaw in this kind of reasoning should be obvious even to Hatcher.
Shea noted, for example, that Porphyry also thought that the third kingdom was Greece I and
the fourth Greece II, so why didn't Shea accept Porphyry's opinion on this too? After all, if
Porphyry was right about what the second kingdom was, why should he not be considered
right about what the third and fourth kingdoms were? Well, those who are familiar with
various attempts by Christian apologists to make the visions of Daniel prophecies of the
coming of Christ and the establishment of his kingdom know that most of these theories are
based on a need to have the fourth kingdom of Daniel be the Roman Empire, so Shea has to
disagree with Porphyry on what the fourth kingdom was. Thus, Shea derogatorily noted that
Porphyry "had to shorten the sequence to get it to end up with Greece and not Rome." What
we have in Shea's letter to Hatcher, then, is not just a flagrant appeal to authority but also an
example of how biblicists will pick and choose from their "sources" whatever supports their
views but summarily reject that which disagrees with them. To a degree, everyone who
debates does this, but Christians are experts at it. Porphyry was a sharp critic of Christianity,
whose writings have survived only in the works of Christian writers who quoted him, and
Shea would probably disagree with 90% of what Porphyry said in those quoted fragments, but
when Shea finds something Porphyry said that supports a belief of his, he instantly seizes it
and cites it as if it should be accepted as "gospel truth." Then he immediately discredits his
own witness in another matter.

The reality is that Porphyry was just another person who, if Shea has cited him correctly, had
an opinion about the book of Daniel, and so he was no different from any other writer who
has expressed a view on this subject. In other words, Porphyry's opinion could be wrong, just
as any other person could well be wrong in an opinion. If Hatcher and Shea think that
Porphyry's view that the second kingdom of Daniel was a combined Medo-Persian empire,
then they should present whatever arguments Porphyry used in support of that opinion and
defend them. They can't just cite Porphyry's opinion and claim that this settles the issue. I
doubt, however, if they will be able to quote any reasons that Porphyry may have given in
support of his opinion, because, as I noted above, Porphyry's writings did not survive
(possibly because of Christian efforts to destroy them), and so it is unlikely that any
supporting arguments that he may have made still exist. If they did, I suspect that Shea would
have quoted them, especially if they had had any real substance to them.

Let's suppose for the sake of argument that Shea is right and that prior to the 18th and 19th
centuries no one had questioned the view that Daniel was written in the 6th century B. C. The

Volume 1990 - 2002 Issue


Page 1375 of 2049
Skeptical Review Edited by Farrell Till
same could be said of many other traditional views of the Bible. Until people felt free to
express their opinions of the Bible without fear of reprisals from the religious establishments
that controlled most governments where Christianity thrived, biblical criticism was practically
nonexistent. With a church empowered to persecute "heretics" and governments ready and
willing to imprison or execute them, who was going to express publicly the belief that Moses
may not have written the Pentateuch or that the exodus probably never happened or that the
resurrection of Jesus was unlikely? It wasn't until such prosecutorial powers were removed
from Christian-dominated governments that people felt free to question traditional views of
the Bible, so it shouldn't be surprising that disagreement with Christian traditions is far more
common today than it was in the heyday of Christianity.

To show the absurdity of Shea's argument, let's suppose that the nations that now forbid
public expression of views that conflict with traditional Islamic doctrines should remove these
restrictions and permit freedom of speech even in religious matters. Does Shea doubt for one
moment that opinions in conflict with Islamic traditions would begin to surface in Muslim
nations that allowed a free market of ideas? If there should then arise a body of Islamic critics
who expressed doubts that the Qur'an had originated as traditionally claimed, how logical
would Shea think an Islamic fundamentalist was if he should say, "Well, this opinion is just a
modern phenomenon that has been worked out in the 20th and 21st centuries"? Would Shea
not realize that this view had probably never been publicly proclaimed before because people
living in Islamic nations knew that they could suffer serious consequences for expressing such
opinions?

An additional problem in Shea's claim that the 6th-century B. C. authorship of Daniel was
never questioned until the 18th and 19th centuries is that there is really no way he can know
this, so it is an assertion that he is obligated to prove. I have already noted that Porphyry's
writings didn't survive, and this is also true of the works of Celsus and other opponents of
Christianity. How, then, could Shea possibly know that in a time when writings critical of the
Bible had a way of vanishing, no one ever expressed the view that the kingdoms of Media and
Persia were separate? That's an argument from silence if I ever heard one. Furthermore, royal
archives and other 6th-century B. C. records that have survived through archaeological
rediscovery recognized that the kingdoms of Media and Persia (as I have already noted in
earlier articles) were separate empires, so it just isn't true that the division of Media and Persia
into different kingdoms is a "modern phenomenon" that was "worked out in the 18th and 19th
centuries." It is a premise that has enough support from ancient records to give it the status of
historical fact. If Shea would consult just about any general biblical reference book, he would
see that Media was a separate empire in the 7th century B. C., which allied itself with
Babylon to capture the Assyrian strongholds of Nineveh in 612 and Haran in 610, but in 550
B. C., Cyrus conquered Media and absorbed it into his empire. This was over a decade before
Babylon fell to Cyrus, so why would Daniel, an official in the Babylonia court at the time,
have thought that Babylon fell to "Darius the Mede"? It is precisely because biblical critics
like Driver, Rowley, and Porteous are aware of these historical realities and recognize that the
writer of Daniel seemed unaware of them that they find it impossible to think that the author
of this book could have been a Babylonia official when the empire fell to Cyrus. The
historical ignorance of this author is not a problem that Shea can explain away by just
dismissing it as "a modern phenomenon, worked out in the 18th and 19th centuries." Sixth-
century B. C. records show that Media and Persia existed as separate empires until Persia

Volume 1990 - 2002 Issue


Page 1376 of 2049
Skeptical Review Edited by Farrell Till
absorbed Media, but the annexation took place a decade too late to make credible Daniel's
claim that Babylon fell to "Darius the Mede."

I have spent a lot of time on this point, but the time was necessary to drive home the flaws in
Hatcher's apologetic approach, which depends primarily on quoting writers who are in
agreement with his position. When he wrote letters to evangelical seminarians like William
Shea, did he expect even a remote possibility that any of them would write back and say,
"Well, I'm afraid that Till is right in this matter. The book of Daniel was definitely written
well after the 6th century B. C."? Certainly not! He wrote to them precisely because he knew
in advance that he would get answers in agreement with his position, which he could then
scatter through an article as if to say, "See what these men who teach in seminaries think.
They say I am right and you are wrong." What do you suppose Hatcher's reaction would be if
I had responded to him with constant allusions to what Dan Barker or William Sierichs, Jr., or
Dave Matson thinks about the dating of the book of Daniel. It would have taken Hatcher
about 10 seconds to scream, justifiably, that I am quoting sources who are already
predisposed to reject the 6th-century B. C. view.

If Barker or Sierichs or Matson should publish what I considered a particularly forceful


argument in agreement with my position, I wouldn't hesitate to quote it, but I would at the
same time include the material that they cited in support of the argument. I wouldn't just quote
an arbitrary assertion that they made and expect readers to accept it, and this is where Hatcher
needs to improve his apologetic method. For example, he quoted Shea again, who said that
"the discovery of more and more Aramaic texts from Qumran... have pushed the date of
Daniel backward earlier, because Daniel writes a kind of Aramaic that is earlier than
Qumran's earliest Aramaic text" (p. 6). What are these Aramaic texts that have been
discovered at Qumran? Shea didn't say; he simply asserted it as a fact that we presumably
should accept without question. He didn't even cite documentation so that we could check his
assertion for reliability. Exactly what are the linguistic characteristics of "the kind of
Aramaic" Daniel used that would establish that this book was written prior to the Maccabean
era? Again, Shea didn't say; he just declared that it was so. He didn't even tell us what
linguistic qualifications he has to reach a conclusion like this, and neither did Hatcher.

The most absurd part of this quotation from Shea's letter was his claim that "Till is behind the
times in his view of the Aramaic in Daniel as Maccabean" (p. 6), but he gave no justification
for making this statement, possibly because I have no views of the Aramaic in Daniel. I have
never studied Aramaic, so how could I have a view that the Aramaic in Daniel dated from the
Maccabean period? I have absolutely no qualifications at all to make such a statement, and so
I have never made any such claim. Apparently, Shea had assumed from a statement I had
quoted from Porteous's commentary on Daniel that I was making an argument that the
Aramaic in Daniel was stylistically the kind that was used in the 2nd century B. C., but that
wasn't the case at all. In the context of this quotation, I was criticizing Hatcher for having left
the impression that Porteous thought that Daniel dated from the 6th century, and it was never
my purpose to try to make an argument based on a language that I have never studied. An
examination of the quotation in context will show that this was clearly my intention.

If Hatcher had bothered to check his source, he would have seen that Porteous is clearly a
proponent of a 2nd-century authorship of Daniel. In the very first paragraph of the

Volume 1990 - 2002 Issue


Page 1377 of 2049
Skeptical Review Edited by Farrell Till
introduction to his commentary, Porteous said, "The linguistic evidence and the fact that the
visions reveal a vague knowledge of the Babylonian and Persian periods and an increasingly
accurate knowledge of the Greek period up to and including the reign of Antiochus
Epiphanes, with the exception of the closing events of that reign, suggest a date for the book
shortly before 164 B. C. (March/April 1998, pp. 7,16, emphasis added).

To verify Hatcher's misrepresentation of my statement, readers can check page 6 of this issue
(1st column, top) to see that by leaving off the first sentence of the quotation above, he left the
impression that I had quoted Porteous in order to argue that the style of the Aramaic in Daniel
shows that it was written during the 2nd century B. C., but that was not my intention. I quoted
the statement only to show that the part I italicized for emphasis clearly indicates that
Porteous believes the book dates from the Maccabean era, and this part just happened to
appear in a larger context where Porteous was commenting on what the Aramaic implies
about the date of authorship. So it was Porteous who expressed this view, not me.

After quoting me out of context, Hatcher then went on to say, "I wish Till would specifically
indicate which linguistic evidence he would put forth as significant," but since the statement
he had just quoted in no way indicates that I was basing an argument on the Aramaic of
Daniel, his intention to misrepresent me seems rather deliberate. If I personally have made no
claims based on the Aramaic text, why would Hatcher "wish" that I "would specifically
indicate which linguistic evidence [I] would put forth as significant"? At any rate, I found
Hatcher's "wish" absolutely ludicrous. Does he expect us to believe that he is knowledgeable
enough in Aramaic to speak with any authority on what the Aramaic text of Daniel indicates
about the probable date of authorship? If I should cite some specific Aramaic examples that
indicate a 2nd-century dating--and to do so, I would have to rely entirely on what Aramaic
experts say--I doubt that Hatcher would be linguistically qualified to dispute them. His
response would likely be a long list of books and articles whose authors think the opposite,
and the discussion would then take on a courtroom atmosphere where jurors sit and listen to
one group of experts give testimony that supports the prosecution's case and then another give
testimony that supports the defense. Hatcher may see some value to this kind of debating, but
I see it as a waste of time. If he can't present arguments for his case that he himself is
qualified to explain and defend, then he will have to find another medium for his articles. I
have more to do than wade through long, tedious articles that do nothing but string together
quotations from books and articles that agree with his position. Anyone with the patience to
pore over books in search of comments favorable to his position that he can string together in
an article can engage in Hatcher's style of "apologetics."

Before I leave the Aramaic issue, at least one more observation is in order. Hatcher and Shea-
-both of whom probably lack the linguistic credentials to speak with authority--claim that the
Aramaic in Daniel was stylistically a type that was used prior to the 2nd century B. C.
Whether this is true or not, I'm not qualified to say. I know only that some Aramaic scholars
dispute this claim, and I would think that scholars of the stature of Rowley and Porteous
would have good reasons for doing so. For the sake of argument, however, let's assume that
the traditionalists are right and that the Aramaic in Daniel was indeed a type that was used
earlier than the 2nd century B. C. Even if that were the case, it wouldn't prove that Daniel had
to have been written prior to the Maccabean period. To understand why, all one has to do is
realize that if Daniel were indeed written in the 2nd century, then it was a forgery intended to

Volume 1990 - 2002 Issue


Page 1378 of 2049
Skeptical Review Edited by Farrell Till
make readers believe that a 6th-century prophet had foreseen the events of that period. In
order to give credibility to the forgery, the writer would have been intelligent enough to
realize that he would have to imitate the language of the period in which the book claimed
that it was written. If, for example, I should undertake to forge a document that purported to
be from the early 17th century, I would know that I would have to us thou, thee, thy, and thine
instead of the current you, your, and yours. I would also know that I needed to use obsolete
words like divers in the sense of various or many, meet in the sense of fitting or appropriate,
espy in the sense of glimpse or catch sight of, etc. I would know that the spelling of words
still in use today and the styling of the letters would have to imitate 17th century orthography
and calligraphy. Without attention to these and other details, the forgery would fool no one.

Are we to assume that those who forged documents in ancient times didn't have the same
common sense? To ask this is not to beg the question of whether Daniel is indeed a forgery
but to point out that if it is, the forger would surely have known that he had to imitate the
linguistic features and quirks that 6th-century Aramaic was known to have. It certainly isn't
hard to imagine that a 2nd-century scribe undertaking to forge a document would have been
familiar with the language of the earlier period in which he wanted his readers to think he had
lived. It seems rather simplistic, then, to argue that because the language of Daniel resembles
pre-Maccabean Aramaic--if indeed it does--it had to have been written prior to the 2nd-
century B. C. This argument fails to take into consideration the possibility of deliberate
linguistic deception. At any rate, the Aramaic issue is one that Hatcher and I should leave to
those who are expert enough to speak with authority on the subject.

Available space in this issue will not allow me to reply to all of Hatcher's repeated appeals to
authority, so I am going to address just one more of his quotations that argued by mere
assertion and then reply in a later issue to the few that did attempt to support the assertions.
Hatcher quoted Stephen Miller's commentary on Daniel, which said, "To suggest that any
semi-educated Jew of the Maccabean period could be ignorant of the fact that it was Cyrus the
Persian who conquered the great Babylonian Empire and allowed the Jewish captives to
return to their homeland is not reasonable" (quoted p. 3, this issue). In support of this
assertion, all Miller said (at least in the part that Hatcher quoted) was that the book of Ezra
would have been "at the writer's disposal" and that it "specifically declares that Cyrus released
the Jews from captivity in Babylon." Yes, the book of Ezra does state that Cyrus released the
captives, but it does not specifically state that Cyrus had captured Babylon. Miller's proof,
then, amounts to no proof at all, because the issue is not whether a 2nd-century B. C. forger
knew that Cyrus had released the captives but whether this forger knew that Cyrus had
captured the Babylonian empire. Since Miller, Hatcher, *et al* have yet to produce any
convincing evidence that the writer of Daniel knew that Cyrus captured Babylon, they have
yet to give anything remotely resembling a satisfactory explanation to the problem of why a
member of the royal Babylonian court would have said that "Darius the Mede" had received
the kingdom [of Babylon]. So Miller winds up arguing that whereas it is "not reasonable" to
think that any semi-educated Jew of the Maccabean period could have been ignorant of the
fact that Cyrus the Persian had conquered Babylon, it is apparently quite reasonable to think
that a high official in the Babylonian court would not have known this or at least was so
linguistically inept that he could not clearly communicate who the conqueror was.

Volume 1990 - 2002 Issue


Page 1379 of 2049
Skeptical Review Edited by Farrell Till
I'll continue my reply to Hatcher in the next issue, after which he may respond if he agrees to
reply with argumentation and not appeals to inerrantist writers who agree with him. We didn't
need him to tell us that there are evangelical writers who agree with him; we already knew
that. What we want from him are arguments that give reasonable support for what these
writers claim.

The Tyre Prophecy Again


Farrell Till

The claim that Ezekiel's prophecy against Tyre in Ezekiel 26 was fulfilled with amazing
accuracy has been thoroughly refuted, yet uninformed biblicists keep repeating it. In the
article in Reason and Revelation, which executive editor Dr. Bert Thompson refused to let me
reprint in TSR (see "From the Mailbag," p. 12, this issue), Brad Bromling listed six details that
Ezekiel prophesied about Tyre and then said, "Each of these items came to pass exactly as
Ezekiel said" (December 1994, p. 96). In the preceding issue of TSR, I published an article by
Bruce Weston in which he described some of the struggles he is experiencing as he makes the
transition from Bible believer to skeptic. One of those struggles concerned prophecy
fulfillment and in particular Ezekiel's prophecy against Tyre, so even though I have already
discussed this prophecy in earlier issues of TSR, I am going to review it to show the absurdity
of trying to pawn this off as an example of remarkable prophecy fulfillment. Although
Apologetics Press would not permit me to reprint Bromling's article, copyright laws don't
prohibit quoting it, so I will have to rely on this method to show how wrong Bromling was in
his claim that Ezekiel's prophecy against Tyre was fulfilled.

Weston expressed doubt that Ezekiel had intended to prophesy that Tyre would be destroyed
and left desolate by Nebuchadnezzar, because "God has Ezekiel saying that He would bring
many nations against them as the waves of the sea" (Vol. 10, Num. 1, p. 6), and Bromling
stated the same position. The second of the "six specific predictions" that Bromling listed in
his article was that "(m)any nations would come against Tyre (v:3)." Before addressing the
claim that Ezekiel predicted that Tyre would be destroyed by "many nations," we should look
at the reference to these nations in its context.

In the eleventh year, on the first day of the month, the word of Yahweh came to me: Mortal,
because Tyre said concerning Jerusalem, "Aha, broken is the gateway of the peoples; it has
swung open to me; I shall be replenished, now that it is wasted." Therefore, thus says the Lord
GOD: See, I am against you, O Tyre! I will hurl many nations against you, as the sea hurls its
waves. They shall destroy the walls of Tyre and break down its towers. I will scrape its soil
from it and make it a bare rock. It shall become, in the midst of the sea, a place for spreading
nets. I have spoken, says the Lord GOD. It shall become plunder for the nations, and its
daughter-towns in the country shall be killed by the sword. Then they shall know that I am

Volume 1990 - 2002 Issue


Page 1380 of 2049
Skeptical Review Edited by Farrell Till
Yahweh. For thus says the Lord GOD: I will bring against Tyre from the north King
Nebuchadrezzar of Babylon, king of kings, together with horses, chariots, cavalry, and a great
and powerful army. Your daughter-towns in the country he shall put to the sword. He shall set
up a siege wall against you, cast up a ramp against you, and raise a roof of shields against
you. He shall direct the shock of his battering rams against your walls and break down your
towers with his axes. His horses shall be so many that their dust shall cover you. At the noise
of cavalry, wheels, and chariots your very walls shall shake, when he enters your gates like
those entering a breached city. With the hoofs of his horses he shall trample all your streets.
He shall put your people to the sword, and your strong pillars shall fall to the ground. They
will plunder your riches and loot your merchandise; they shall break down your walls and
destroy your fine houses. Your stones and timber and soil they shall cast into the water. I will
silence the music of your songs; the sound of your lyres shall be heard no more. I will make
you a bare rock; you shall be a place for spreading nets. You shall never again be rebuilt, for I
Yahweh have spoken, says the Lord GOD (Ezek. 26:1-14, NRSV with Yahweh substituted
for the LORD).

Bromling depicted the "exact" fulfillment of this prophecy as a series of events that happened
over a period of almost 1900 years.

Within a few years of Ezekiel's oracle, Nebuchadnezzar besieged the mainland city (586 B.
C.). When he finally defeated Tyre 13 years later, the city was deserted--most of the
inhabitants had already moved to the island. Things remained that way for about 241 years.
Then in 332 B. C., Alexander the Great took the island city for Greece. This was
accomplished by scraping clean the mainland city of its debris, and using those materials to
build a land-bridge to the island. Although Alexander brought much damage to the city, it still
stood. Tyre persisted for the next 1,600 years. Finally, in A. D. 1291, the Muslims thoroughly
crushed Tyre, and the city has remained in ruins ever since. Aside from a small fishing
community, nothing is left.

There are so many distortions and misrepresentations in this short paragraph that I hardly
know where to begin replying to them, but at least two important counterpoints must be made:
(1) Nebuchadnezzar didn't finally defeat Tyre as Bromling claimed. Tyre was a stronghold on
an off-shore island, and Nebuchadnezzar's 13-year siege (587-574 B. C.) against it failed, as
even general encyclopedias will inform those who bother to check. Nebuchadnezzar
succeeded only in capturing the mainland suburb, which was known as Ushu, but that
certainly didn't require 13 years. The mainland area was taken without difficulty, and then the
unsuccessful siege was directed against Tyre proper, which was the island stronghold.
Nebuchadnezzar finally withdrew his forces after securing a Tyrian agreement to pay annual
tribute to Babylon, but he did not capture the city much less destroy it permanently, as Ezekiel
predicted he would. (2) Bromling also erred in saying that Tyre has remained in ruins ever
since the Muslim conquest of 1291. The Interpreter's Dictionary of the Bible (Vol. 4, p. 721)
has an aerial photograph of modern Tyre, which shows the former island part covered by a
town that is much more than just the "small fishing village" that Bromling described above.
Also, the town is obviously much more than a "bare rock," where fishing nets are spread,
which Ezekiel predicted it would forever be (v:14). What the photograph shows agrees with
Harper's Bible Dictionary, which claims that an "important deep sounding in the 1970s"

Volume 1990 - 2002 Issue


Page 1381 of 2049
Skeptical Review Edited by Farrell Till
showed that "(t)he city has been almost continuously occupied from the third millennium B.
C. until the present, except for a major gap from 2000 to 1600 B. C." (1985, pp. 1101-1102).

Biblical inerrantists have resorted to all sorts of verbal gymnastics to try to explain how that
Ezekiel's prophecy could have been fulfilled if the site of ancient Tyre is still occupied by a
city. They say such things as it never regained its former splendor, but the prophecy didn't
predict this: it said that the city would never be rebuilt. Some inerrantists even say that
present-day Tyre is not built on the same site, but a look at modern maps of Lebanon will
show (as does the aerial photo mentioned above) that Sur [the modern name] is located on the
very site of the former island. It just isn't so that each of the items in this prophecy "came to
pass exactly as Ezekiel said." He predicted that the city would be destroyed and never built
again, but a city is there now on the same spot that Ezekiel said would be a bare rock forever.

That brings us to the matter of the "many nations" that Ezekiel said would be involved in the
destruction of Tyre. The literary organization of the prophecy (quoted above) seems rather
simple. It began with an introductory statement of what Yahweh intended to do to Tyre. He
said that he would (1) cause "many nations" to come against it, (2) destroy its walls, (3) break
down its towers, (4) scrape the dust from it and make it like a bare rock, and (5) slay its
"daughter villages" in the field. After describing in general terms what he was going to do to
Tyre, Yahweh then proceeded to state the specifics of how this would be done: "For thus says
the Lord GOD: I will bring against Tyre from the north King Nebuchadrezzar of Babylon...."
It seems rather clear, then, that Yahweh was saying that Nebuchadnezzar would be the
instrument that he would use to destroy Tyre as promised in the introductory statement, so he
proceeded to state the specifics of what Nebuchadnezzar would do to Tyre. He would put
Tyre's daughter-towns in the country [the mainland villages] to the sword, he would set up a
siege wall, he would cast up a ramp, etc., etc., etc. The prophecy listed a dozen specific
military actions that "he" would direct against Tyre, and the only reasonable antecedent of the
pronoun he is Nebuchadnezzar.

So if Ezekiel was declaring that Nebuchadnezzar would be the instrument that Yahweh would
use to destroy Tyre, why did he say that "many nations" would be sent against it? A
reasonable explanation of the prophet's reference to "many-nations" can be found in the ethnic
compositions of early empires. Empires like Babylonia formed from the conquest and
annexation of surrounding tribes and nations, so when an area was assimilated into an
adjoining kingdom, the soldiers of the conquered nations served the greater empire. The
Assyrian empire, for example, crumbled when the combined forces of the Medes,
Babylonians, and Scythians plundered Assur in 614 B. C. and Nineveh in 612. When Haran
fell to these allied forces in 610 and then Carchemish in 605, most of the Assyrian territory
was annexed by Babylon. In such cases, defeated armies swore allegiance to their conquerers,
so the armies of a king like Nebuchadnezzar were actually armies of "many nations."

Literally, then, when the armies of Nebuchadnezzar or Cyrus or Alexander attacked a city or
territory, it wasn't just the aggression of a single nation but of many nations. This reality of
ancient warfare was reflected in a familiar scenario in the Old Testament in which biblical
prophets and writers depicted battles against common enemies as the gathering of "many
nations." In 2 Chronicles 20:1-4, this allegedly happened when Jehoshaphat was king of
Judah.

Volume 1990 - 2002 Issue


Page 1382 of 2049
Skeptical Review Edited by Farrell Till
It happened after this that the people of Moab with the people of Ammon, and others with
them besides the Ammonites, came to battle against Jehoshaphat. Then some came and told
Jehoshaphat, saying, "A great multitude is coming against you from beyond the sea, from
Syria, and they are in Hazazon Tamar."

Psalm 2:1-2 depicted the "kings of the earth" as having set themselves against Yahweh and
his anointed. Isaiah 13:4 told of a "tumultuous noise of the kingdoms of nations" that were
gathered together against Yahweh of hosts. Zechariah 12:3 warned that "all nations of the
earth" that were gathered together against Jerusalem would be cut in pieces. Ezekiel himself
clearly used this same scenario at times. In the allegory of the two sisters (Oholah and
Oholibah), he warned Judah that Yahweh would send against it the Babylonians, Chaldeans,
Pekod, Shoa, and Koa, and all the Assyrians. The "many-nations" scenario was a
commonplace hyperbolic device that biblical prophets used in their vitriolic denunciations of
those who were enemies of Israel and Judah. This device was even used to denounce Judean
kings who "did evil in the sight of Yahweh." After Nebuchadnezzar had installed a puppet
king in Jerusalem and by a strange twist of thinking had come to be considered by some
biblical writers as God's servant, Jehoiakim (the puppet) rebelled, and "Yahweh sent against
him bands of Chaldeans, Syrians, Moabites, and Ammonites to destroy Judah" (2 Kings 24:1-
3), but the last two chapters of this book make it very clear that it was Nebuchadnezzar's army
that destroyed Judah and took the people captive to Babylon, but in a real sense it was
actually a conquest of "many nations," because Nebuchadnezzar's armies were comprised of
more than just Babylonians.

When inerrantists today look at Ezekiel's prophecy through the glasses of historical records,
they can clearly see that it was not fulfilled by Nebuchadnezzar, and so they must look for
some way to explain away the failure. Ezekiel's reference to "many nations" is a straw that
some inerrantists like Brad Bromling have grabbed to try to salvage the prophecy, and so they
have tried to make the prophecy mean that a series of attacks by many different nations spread
out over 1900 years would result in the eventual destruction but that Ezekiel never meant that
the total desolation of Tyre would be caused by Nebuchadnezzar. However, the literary
organization of the prophetic passage (which I analyzed above) and the facts just noted about
the multi-national composition of ancient armies like Nebuchadnezzar's make this
"explanation" questionable to say the least. It is more likely that Ezekiel meant that "many
nations" under the leadership of Nebuchadnezzar would bring about the total destruction of
Tyre.

A stubborn fact that inerrantists choose to overlook is that Ezekiel more or less apologized
later in his book for having erred in predicting that Nebuchadnezzar would destroy Tyre. This
passage clearly shows that Ezekiel meant that Nebuchadnezzar would destroy the island
stronghold and not just the mainland suburban area as Bromling has tried to distort the
prophecy into meaning.

In the twenty-seventh year, in the first month, on the first day of the month, the word of the
LORD came to me: Mortal, King Nebuchadrezzar of Babylon made his army labor hard
against Tyre; every head was made bald and every shoulder was rubbed bare; yet neither he
nor his army got anything from Tyre to pay for the labor that he had expended against it.
Therefore thus says the Lord GOD: I will give the land of Egypt to King Nebuchadrezzar of

Volume 1990 - 2002 Issue


Page 1383 of 2049
Skeptical Review Edited by Farrell Till
Babylon; and he shall carry off its wealth and despoil it and plunder it; and it shall be the
wages for his army. I have given him the land of Egypt as his payment for which he labored,
because they worked for me, says the Lord GOD (Ezek. 29:17-20).

The prophecy against Tyre in chapter 26 was dated in the "eleventh year" and this one in the
"twenty-seventh year," so the one immediately above was made after the other one. Notice
that Ezekiel said in the first italicized statement above that "neither [Nebuchadnezzar] nor his
army got anything from Tyre to pay for the labor that he had expended against it," but if
Bromling is right in his claim that Yahweh had intended for Nebuchadnezzar to defeat only
the mainland area, then it would not be true that he had received no pay for the labor he
expended. In reality, he would have gotten exactly what Yahweh had wanted him to have...
the mainland villages of Tyre. Obviously, then, Ezekiel was saying here that the original
prophecy had failed to materialize because Nebuchadnezzar did not accomplish what the
prophecy had promised, and so Yahweh was going to give him the land of Egypt "as his
payment for which he labored." In typical biblical vitriol, the next chapter launches into a
prophetic tirade against Egypt, which would presumably suffer the same sort of devastation
from Nebuchadnezzar that Ezekiel had earlier predicted for Tyre. This prophecy also failed,
but that's material for another article, some of which is discussed on the front page of this
issue.

In his zeal to praise the Bible where praise is not warranted, Bromling concluded his article
by asking, "How can we account for Ezekiel's precision regarding the history of this city?"
Well, that's an easy question to answer. There was no "precision" in Ezekiel's prophecy about
Tyre. It failed in various points, but Bromling's blind allegiance to biblical fundamentalism
will never allow him to admit it. He went on to ask, "How could [Ezekiel] look almost 1,900
years into the future and predict that Tyre would be a bald rock where fishermen would
spread their nets?" But as my reference to the aerial photograph of modern Tyre explained,
this site is not a bald rock. The major part of the former island is occupied by a town, and
even if this town were the insignificant fishing village that Bromling has tried to make it, that
would just be another point of prophecy failure, because Ezekiel said that Tyre would be
destroyed and "never more have any being"(26:14, 21; 27:36; 28:36). Bromling and other
biblicists may see the hand of God in this, but I see desperation in their attempts to find the
hand of God in an ancient prophecy that obviously failed.

From the Mailbag

A Proposal to Reason & Revelation...

You asked if it might be possible for you to receive permission from our offices to reprint in
its entirety Brad's article on Ezekiel's prophecy of Tyre in The Skeptical Review. Wayne and I
have discussed this, and we do not believe it appropriate to grant you permission to reproduce

Volume 1990 - 2002 Issue


Page 1384 of 2049
Skeptical Review Edited by Farrell Till
any of our material in The Skeptical Review. We are appreciative, however, of your desire to
abide by copyright laws.

(Dr. Bert Thompson, Executive Director, Apologetics Press, 230 Landmark Drive,
Montgomery, AL 36117; e-mail, mail@apologeticspress.org)

EDITOR'S NOTE: Written January 5, 1995, this letter was misfiled. By coincidence, it was
found in a search for something else right after the publication of the January/February 1999
issue of TSR in which a letter from Rob Berry referred to an internet site maintained by
Apologetics Press, where this conservative religious publication warned its readers that the
"missing day in time" was a myth that apologists should not use as proof of biblical
inerrancy. Apologetics Press publishes Reason and Revelation, a paper that describes itself
as "a monthly journal on Christian evidences." The staff of this organization are members of
the Church of Christ, and the "Wayne" referred to in Dr. Thompson's refusal of my request to
republish an article from their paper was undoubtedly a reference to Wayne Jackson, the
president of Apologetics Press. This is the same Wayne Jackson whose article on "The Sons of
God," I replied to in the Winter 1992 issue of TSR. Although I offered Jackson space to reply
to my article, he never accepted the invitation. As I recall, the articles in Reason &
Revelation, as well as those in Jackson's own paper (Christian Courier), were not copyrighted
at that time, but afterwards copyright notices began to appear in both papers.

The rejection of our offer of free rebuttal space and our request to republish an article on
prophecy fulfillment was mystifying to say the least, because the staff members of Apologetics
Press firmly believe that they have the truth, the whole truth, and nothing but the truth, so why
they would not want their articles to receive wider circulation and subsequent posting on our
internet site is hard to understand, since we always offer Christian fundamentalists the right
to rebut anything we publish in response to their articles.

A clue to the reluctance of Dr. Thompson and Mr. Jackson to allow the republication of their
articles may be in the December 1998 edition of Reason & Revelation. This issue contains an
article by Dr. Thompson entitled "Questions and Answers." In the article, Thompson cited
questions that unnamed "skeptics" had sent to his paper. The questions, of course, were
answered by the usual arbitrary how-it-could-have-been scenarios that internet users
encounter all of the time on lists and news groups that discuss biblical inerrancy. The
worthlessness of this method of resolving biblical discrepancies is evident from the fact that it
could be used by Muslims, Mormons, or any other religious group to "explain" discrepancies
in their religious books. If Dr. Thompson should undertake to defend biblical discrepancies in
a forum where he would confront an informed opposition that has an opportunity to rebut, he
would find that explaining biblical discrepancies is not quite as easy as it is when he's
preaching to the choir. The same is true of much of the material published in Reason &
Revelation. It is always aimed at people who are already inclined to believe that the Bible is
God's inspired, inerrant word, so that makes it easy to pawn off simplistic "explanations" as
real solutions to biblical discrepancies.

Through an exchange of subscriptions, Dr. Thompson receives The Skeptical Review, so I


assume that he will read this. When he does, he should consider it an invitation to exchange
not just subscriptions but also ideas. If he will grant me permission to republish articles from

Volume 1990 - 2002 Issue


Page 1385 of 2049
Skeptical Review Edited by Farrell Till
his paper, I will, as always, extend to him or any of his staff members the space to reply to
anything that I publish in response to their articles. I would be very interested in sending to
him some questions about biblical discrepancies for him to answer. If he will answer them, I
will publish his answers in TSR along with my replies to them, which he can then reply to if
he wishes. Of course, I would be delighted if he would publish exchanges like these in Reason
& Revelation, but I have no hope that this is going to happen.

More Comments on Archer's Letter...

First of all, I would like to thank those of you who encouraged me through your thoughtful
letters and e-mails. Regarding Gleason Archer's claim that life is meaningless without God,
my testimony suggests the exact opposite. I came to the point where I had to conclude that if
Christianity is the truth, then life is meaningless. The objective evidence to the contrary is
what brought me relief. If I knew the eternal destiny of even one loved one or friend, was
eternal horror, fire and torment, my life would be meaningless. Surely, a dog or a cat is the
one who has received "grace" in this deal. And to think that the purpose of humanity is to
punish most of it eternally! I ask you, Mr. Archer, and others like you, where do you find
meaning in such horror? Or is this yet another idea that you have chosen to conveniently tuck
away in your mental freezer awaiting further "revelation"?

(Craig Cunningham, 6502 East Golf Links, Apt. 262, Tucson, AZ 85730; e-mail,
CCunn63@aol.com)

EDITOR'S NOTE: Archer's claim that life has no meaning unless his god exists is a familiar
one to those who engage in internet discussions with Christians, because it is bandied about
as if it had been proven as absolute scientific fact. Those who have experienced going from
religious superstition to the personal satisfaction that comes with having found the courage to
face reality know better. Archer and those who parrot his claim don't know what they are
talking about. Before they repeat this claim, they should do a study of skeptics who were once
Christians; they might be surprised to learn that their opinion is not shared by those who are
more qualified to have an opinion on the subject.

Correspondence with Prisoners...

Concerning M. Paul Goldberg's letter published in the January/February 1999 "From the
Mailbag" column, I am a theist: while I am not a Christian, and certainly not an inerrantist, I
do believe that the Divine exists and that the Divine is immanent in the world. It is therefore
with a bit of chagrin that I note that theists in general (and fundamentalist in particular) appear
to be rather less literate than nontheists. Indeed, I find that education itself appears to diminish
religiosity. Judge for yourself: read the newsgroup "alt.atheism" and compare the spelling and
grammar of the non-theists' messages with the theists' messages. Yes, I believe in the Divine,
but as for correspondents I prefer atheists.

However, that is not why I'm taking this moment to write. I am writing because, firstly, Mr.
Goldberg used my name in vain, and, secondly, what he reports about prisoners is not what I
have observed. To date I correspond with three prisoners, and I have received letters from a
dozen or so others who are looking for people with whom to correspond. In no case can any

Volume 1990 - 2002 Issue


Page 1386 of 2049
Skeptical Review Edited by Farrell Till
of them be likened to people with "learning disabilities." I find such an accusation damn
insulting to my correspondents. Without exception the letters I receive from prisoners are the
literary equal to those of nonprisoners in spelling, grammar, punctuation, etc. (If any fault
could be mentioned, it would be that the penmanship is often rather poor.)

I do not have nearly enough time to correspond with all the prisoners who write to me: if
anyone wishes to verify Goldberg's opinion, or contrast it with what I report here, I urge them
to ask me for the names and addresses of the prisoners with whom I have not the time to
correspond. Do not take his word, or mine, for it: see for yourself. The folks in prison will
bless you for it, and I believe so will the Divine. (Yeah, I had to sneak that last bit in.)

(Rev. David Michael Rice, Mariner's Ministries [Dana Point, CA], 723 Calle Casita, San
Clemente, CA 92673-2708; e-mail, shydavid@ktb.net).

EDITOR'S NOTE: I am publishing below a sample of the type of letter that I often receive
from inmates. It is completely unedited.

Reading Materials Wanted...

I am an inmate at the State Correctional Institute at Greensburg, Pennsylvania. I receive no


support from family or friends and I remain indigent.

It is very difficult to obtain good books and reading materials while in prison. I would be so
grateful to you if you would send me a current or back issue of your publication. Also, if you
have any old, used or slightly damaged books not suitable for sale, please send to me what
you can find. Anything you are able to send will be received with open arms and an open
heart.

Thank you for helping me in my time of need and for sending light into this dark place.

(Lee Horwitz #CF5079, SCI-Greensburg, RD #10, Box 10, Greensburg, PA 15601)

EDITOR'S NOTE: Mr. Horwitz has been added to the mailing list to receive the free
subscription that I offer to all inmates who are unable to renew after the first year, but
providing the reading materials is another matter. Each press run of The Skeptical Review
will have some copies that are damaged, but finding the time to package and mail them to
inmates who may be interested in receiving them is a problem for me. As I write this, I am
trying to find time to finish mailing the previous issue to foreign addresses. These must be put
into catalog envelopes and marked according to what country they go to, so sending the 200
foreign subscriptions takes much more time than the domestic mailings.

Before anyone bundles reading materials and sends them to prisoners, I would suggest that
an inquiry be made first to see if they will be allowed into the prisons. Paul Goldberg, whom
David Rice mentioned above, has been involved in sending printed materials to prisoners,
and I know from correspondence he has sent me that he has at times encountered delivery
problems. He can be contacted for advice at 67-35 Yellowstone Boulevard, #6T, Forest Hills,
NY 11375-2610.

Volume 1990 - 2002 Issue


Page 1387 of 2049
Skeptical Review Edited by Farrell Till
TSR and Restrictions on Literature...

It is that time of year to write and thank you for sending TSR for the past year, and to ask that
you continue it through 1999, please. I am still indigent.

I see by the mailbag, page 16, November/December 1998, that you are gaining experience
dealing with prison systems in America's gulag. The Texas prison system bars some
freethought publications, including a publication named Thought out of Arizona, although
hard-core porn flows right in. The Texas prison system also bans the literary publication of
the University of Oklahoma, World Literature Today, while the hard-core porn flows right in,
as does all Jehovah's Witness literature, etc., etc., etc.

The worst thing about TSR in prison is that if you lend it out, it's a good bet you won't get it
back! No one has seen anything like it; it's not the corporate media/mainstream pablum they
were raised on and are used to. I quit lending and started giving because people don't want to
turn loose of them. Sad to say, there are Muslims who seek out TSR because they have heard
it debunks Christianity, but the Pentateuch is sacred scripture to Muslims, so I really don't see
their point.

You have the best thing going vis-a-vis inerrantists. Please keep me on your
mailing/subscription list. Some time in the future, I hope you delve into that emendation of
emendations, Luke 23:34.

(Robert J. Zani, No. 328938, Michael Unit, P. O. Box 4500, Tennessee Colony, TX 75886)

Another Inmate's Views...

I would like to apply for a free year's subscription to your magazine. I found your address in a
book called The Book Your Church Doesn't Want You to Read, and I read your chapter
"Nonexistent Prophecies: A Problem for Bible Inerrancy" and found it very interesting as well
as true. I'm not actually putting the Bible down; it's just that there are so many contradictions
in it I find it hard to accept as the so-called "word of the Lord." The Book Your Church
Doesn't Want You to Read has given me a whole lot of material to back up my theories
against all these "Bible pushers" and "reborn Christians" in prison here!!! Sir, to be honest
with you, it's damn funny how a lot of inmates in here find religion but once they hit the
"gates" it's back to the burglaries, crack pipe, shooting speed, etc., but, hey, while they are in
here they seem to have that need and desire to push the Bible off on me.

By the way, I can't wait to show my mother-in-law The Book Your Church Doesn't Want You
to Read. She's a Jehovah's Witness. I can't wait to see the expression on her face once she
reads it plus a book I have by David Reed Behind the Watchtower Curtain.

(Michael Beaudette, CDC K98580, Magnolia 9414 up, Chino State Prison, P. O. Box 600,
Chino, CA 91708)

Still Another...

Volume 1990 - 2002 Issue


Page 1388 of 2049
Skeptical Review Edited by Farrell Till
Walking down the tier, the other day, I discovered a most important publication, yours. I have
read the Bible. I am surrounded by "believers" who are "men of God" but haven't even read
the Bible and go to church daily. Please put me on the subscription list for prisoners and give
my address to anyone who might send back issues. I want very much to join you in helping
others learn that God also gave us a brain, that we must use it, and especially when reading
the scriptures, as indeed they themselves instruct.

(Dennis Alsin, C-38627, Box 8101-6112-X, SLO, CA 93409-8101)

Another Satisfied Ex-Christian...

I discovered your magazine at the Internet Infidels site and found it to be exactly what I've
been looking for. I live in a part of the country which is almost completely dominated by the
bibliolatrous mindset. Your publication will be a great resource. Please put me on your
subscription list. I'm sure I'll renew when my first year is up. I am probably in one of the
smallest minorities in the world. I was raised by atheist parents and attended the Unitarian
Universalist church. My parents taught me to respect science and skeptical thought from an
early age. For a brief time in my late teens, I attended a fundamentalist church at the behest of
a girlfriend. I actually became "born again" for all of three months, but the members of that
church made a large mistake: they insisted I now needed to study the Bible. Study it I did and
found so many ridiculous and unbelievable things I began to feel embarrassed that I had been
"suckered" into this bizarre belief system. I then began to attend more liberal churches and
gradually fell away from Christianity altogether. After an exploration into eastern religions
and some new age beliefs, my skeptical inclinations have won out. I feel I've come full circle.
I now consider myself to be Scientific Pantheist and am happier than I ever was when under
the delusion of religious belief. I love debating "fundies," and I find your magazine to be
loaded with great information. Keep up the good work.

(Tom Brookman, P. O. Box 201, Jellico, TN 37762; e-mail, tntbrookman@jellico.com)

EDITOR'S NOTE: Day in and day out, I read the messages that Christians post on the
internet in their efforts to defend the Bible, and a recurrent theme in their postings is that life
is meaningless without "God," but what we hear from those who have experienced life from
both sides of the "god issue" does not agree with the gloom-and-doom view of theists who
think that life can have no meaning without a god. Mr. Brookman is just one more witness to
dispute this often-repeated claim.

Worried about a Tower?

Here is my check for another year of your excellent publication. I especially enjoyed the latest
one with the Tower of Babel article by Stephen Van Eck. This was one of the first Bible
stories that I could see posed a problem relating to the existence of the Christian god. That he
was actually worried that men could build a tower reaching to heaven shows that god was no
better informed about astronomy and engineering than the ancient Hebrews. And if the Tower
of Babel upset him, why didn't he put a stop to our moon landing? Keep it up!

(Edward Unger, 1380 Toonigh Road, Canton, GA 30115; e-mail E.Unger@worldnet.att.net)

Volume 1990 - 2002 Issue


Page 1389 of 2049
Skeptical Review Edited by Farrell Till
EDITOR'S NOTE: Mr. Unger has asked a question that I have yet to see biblical inerrantists
give a sensible answer to. Most won't even try to answer it. If God was so concerned that the
building of a tower, which at best would have been only a few hundred feet high, might reach
into heaven, why does he tolerate modern space programs, where orbital shuttle flights have
become routine and probes are being sent to the far reaches of the solar system and even
beyond? That anyone would believe such nonsense as this is a sad commentary on human
intelligence.

So what does that say about me? I once believed it, but at least I outgrew it. What's the excuse
of biblical inerrantists my age and older who still cling to such ancient superstitions?

"Prophecies" in Daniel...

Regarding prophecy in the book of Daniel, according to Hermann Rausching's The Voice of
Destruction, a sect called "The Bible Searchers" was active in Germany during the 1930s, and
they identified the "King of the North" with Adolf Hitler, placing particular emphasis on
11:37-38: "Neither shall he regard the God of his Fathers, nor the desire of women, nor regard
any god; for he shall magnify himself above all. But in his estate shall honor the God of
force." Of course, they were consigned to the concentration camp for their predictions.

This was, of course, published before the end of the war. However, carrying the prediction to
its ultimate conclusion, the last verse of chapter 11 reads in the King James Version: "And he
shall plant the tabernacle of his palace between the seas (not the plural) in the glorious holy
mountain; yet he shall come to his end, and none shall help him."

A further interpretation (unfortunately, I don't remember the source) identified the two seas as
the Black and the Caspian, representing the German army's farthest thrust eastward during
Word War II.

No doubt the same text could be applied to a number of other rulers from Biblical times to the
present.

(R. S. Craggs, 25 McMillan Avenue, West Hill, ON, Canada M1E 4B4)

EDITOR'S NOTE: The final sentence in this letter explains why biblical prophecy fulfillment
claims should be viewed with skepticism. The same prophecies are often given various
interpretations, and the proponents of each interpretation are always sure that theirs is the
right one. There's not much value in prophetic writings that can mean whatever anyone wants
to make them mean.

Nebuchadnezzar and Belshazzar...

Thank you for responding to my request for Volume 9, number 6 of The Skeptical Review,
which I received on December 7th. The insight given on the various articles in it are
invaluable. I found particularly important in value the clearing up of certain fine points under
the discussion of the articles through comments and letters and just plain common sense. A
fine example, just to mention one, is the letter from Dave Matson, editor of The Oak Hill Free

Volume 1990 - 2002 Issue


Page 1390 of 2049
Skeptical Review Edited by Farrell Till
Press, in which he pointed out that there was no reason or indication for secondary meanings
of "father" and "son" in regards to the relationship of Nebuchadnezzar and Belshazzar,
therefore proving beyond reasonable doubt that the Hebrews mistook their relationship to be
actual father and son, an obvious error.

This kind of insight takes top priority. It is irrefutable, leaves no room for argument, and puts
any uncertainty to rest. I would like to subscribe to *TSR* and sincerely appreciate the one-
year free subscription. Thank you very much for this offer. I would like to comment on
articles, and if my own research warrants and merits it, I would be happy to submit any
articles to deal with the subject of biblical inerrancy.

(Gary Jollymour, 46493 Mayfair Avenue, Chilliwack, BC, Canada V2P 3J3)

EDITOR'S NOTE: I try to consider all articles that are submitted to TSR, but I receive more
of them than I can read. I try to look through them periodically to find those that best fit my
publishing goal, which is to discuss both sides of the biblical inerrancy issue. Unfortunately,
many submissions don't relate to inerrancy and have to be rejected even though they are often
interesting within the subjects they discuss. Writers should keep in mind that an article on the
subject of religion isn't necessarily an article about biblical inerrancy.

Renewing on Time...

Oops, I failed to renew on time, but you sent the July/August and September/October issues
anyway. Thanks. A check is enclosed to cover 2 years from June 1998 to June 2000, plus a
few bucks to cover special mailing of the November/December issue (for which I probably
missed the bulk mailing).

I continue to appreciate The Skeptical Review. I pass it on to my brother, who also reads it
with great interest, especially the stories of people who found their way out of the inerrancy
quagmire by reading your newsletter. It's very encouraging and inspiring. You are making not
only important contributions to biblical education but to people liberating themselves from a
prison of the mind.

(Paul A. Heffron, 4389 Hodgson Road, St. Paul, MN 55126)

EDITOR'S NOTE: Mr. Heffron made a common mistake in reading the expiration date on his
address label. He thought that 6-98 meant that his subscription expired in June 1998.
However, the first number refers not to the month of the year but to the number of the issue
within that year. Issue number 6 is the last one of the year (November/December), so the
other issues were mailed to him because his subscription had not actually expired. Issue
number 1 is mailed in January, number 2 in March, number 3 in May, number 4 in July,
number 5 in September, and number 6 in November. Each issue will have (usually on the back
page) a reminder of what the expiration date on the address label should read if the
subscription expires with that issue. You are now reading number two, so if your address
label has 2-99 on it, your subscription expires with this issue.

Incorrect Addresses...

Volume 1990 - 2002 Issue


Page 1391 of 2049
Skeptical Review Edited by Farrell Till
My nephew is a fanatical born-again believer in Christ. He loves to give out tracts and argue
about our country's founding fathers. I told him that he might be interested in your
publication; maybe it would give us something to talk about. He agreed to this. Please send a
year's subscription to him at (address deleted).

Thank you for all the work you do to help the deluded see things with a fresh perspective. My
nephew tells me that he has nothing to lose by believing, but I no longer swallow that line. (I
used it myself when I was a believer.) It is my firm conviction that it is always wrong to
believe a lie and that it is our responsibility to continually seek after the truth. I believed a lie
for fifteen years. What a waste! I felt such a load lifted when I was finally able to shed the
shackles of religion that had imprisoned me. I hope that your publication will be able to raise
some questions in Bill's mind so that he might be able to start thinking for himself instead of
constantly quoting what someone else told him.

Keep up the good work; it is sorely needed.

(Denise Schilling, S91 W22925 Milwaukee Avenue, Big Bend, WI 53103)

EDITOR'S NOTE: Ms. Schilling's nephew was added to the mailing list, but unfortunately the
first copy was returned with a notice that he had moved and that the forwarding order had
expired. If she will send me his new address, I will add him to the list.

This has happened before, so those who send requests for subscriptions to be sent to friends
or relatives should make sure that the addresses are current.

A Gift subscription...

I have thoroughly enjoyed reading The skeptical Review over the past few months as well as
reading the postings on the Errancy list. (I'm still in lurk mode.)

One of the many reasons I became an atheist some 12 years ago was because of
inconsistencies and contradictions I came across while reading the Bible, which I couldn't
reconcile with the (allegedly) omniscient, omnipotent, omnibenevolent being that (allegedly)
inspired its writing. Since, I've become a subscriber to TSR and your list, I've learned a great
deal concerning the issue, and it has further cemented by belief that no such being exists now
or ever did exist.

I've enclosed a check that should cover my next year's subscription to TSR as well as a little
extra to help defray the costs of sending out free issues to imprisoned inmates. You're doing a
great service there.

Oh, yeah, I also seem to remember a posting [on the internet] from Matt Bell requesting a free
subscription to TSR above and beyond the first year's free subscription. Well, I'd like to be of
help to Matt. Please take a portion of my check and renew his subscription for another year.

(Michael Zanussi, 421 Broadway Boulevard SE, Apt. 11, Albuquerque, NM 87102)

Volume 1990 - 2002 Issue


Page 1392 of 2049
Skeptical Review Edited by Farrell Till
EDITOR'S NOTE: The name Matt Bell will be familiar to subscribers who are also on the
Errancy internet list. Bell is a biblicist in Scotland who goes through the usual verbal
contortions to try to explain discrepancies in the Bible. When his free subscription to TSR
expired, he asked me to continue it at no cost because his conscience would not permit him to
give financial support to a Satanic work. (Those of you who aren't on the internet miss a lot of
fun.) In compliance with Mr. Zanussi's request, Bell's subscription was renewed for another
year.

Skeptic's Soapbox...

I'd like to thank you for the good work you are doing. I have been a born-again Christian and
then one of Jehovah's Witnesses for a total of over two decades. In the past two years or so, I
have been examining all of my beliefs.

I am now an atheist. I found The Skeptical Review on the internet, and it has been invaluable
in testing whether or not the Bible is inspired of God. I am much happier now that I am free
from arbitrary religious rules.

I think my subscription will be expiring soon, so I am including money for a two-year


renewal. I will be sending diskettes once I look up the policies about them. My web site
Skeptic's Soapbox is devoted to skeptical topics. Testing the Bible is a part of it. The URL is
http://www.geocities.com/Athens/Aegean/2260.

I recommend your magazine to all who will listen. You are making a real difference in
people's lives.

(Richard Nazar, 26 Jackson Court, Kanata, ON, Canada K2K 1B7; e-mail,
skepticsspbx@geocities.com)

EDITOR'S NOTE: So once again we see an atheist who says that he is happier now than
when he was a Christian. As I have pointed out before, I have yet to hear a skeptic or atheist
say, "I was much happier when I was a Christian." Nevertheless, I predict that we will
continue to hear Christians declare that life is meaningless without God or Christ.

Land of Superstition...

We received your brilliant journal The Skeptical Review (January/February 1999) and thank
you for the temporary subscription. The present letter is a humble request to bestow us with
all 43 issues of TSR via sea mail. It will be very difficult to get foreign exchange in India. If
you still prefer, we may try for some token payment. Kindly understand the urgent need of
skeptic literature in this land of superstition. India is the most superstitious country, where
anti-intellectualism is the political slogan of the ruling parties! Also we need the 47 page
Laws-Till and Jackson-Till debates. Thank you very much for your kind consideration and
precious assistance.

(R. Zekariya, Secretary, Vachanam, Centre for Comparative Religion, P. O. Box 3112, Kochi:
682030, India)

Volume 1990 - 2002 Issue


Page 1393 of 2049
Skeptical Review Edited by Farrell Till
EDITOR'S NOTE: As time is available to me, I will try to put together these materials and
send them to this organization. The letterhead indicated that this is an organization that exists
to study theology and philosophy and "their impact with modern science."

A Difficult Situation...

I received and enjoyed my first issue of the Skeptical Review.

I understand that you used to be in the ministry and I wondered if you could offer me some
help or advice with the situation I find myself in. Several months ago I concluded that the
bible was not inerrant, for all the obvious reasons. Now my faith in God is barely existent.
This might all be okay except for the fact that I am married to a youth minister. He and I met
at Xxxxxxxx Temple University.

Were you married at the time you lost your faith, and did this cause problems with your wife?
I'm sorry for the personal questions, but I am hoping that someone who has been down this
road already can help me. My husband and I get along great except for this issue. Now we
argue frequently not only about Christianity, but about what we want to teach our children
about God. Our children are xx and x.

I feel very guilty about the fact that I have changed the status quo. We had been going along
just fine for fifteen years until I "messed things up." I hate pretending to believe something I
don't believe, yet there I am involved in various ministries at church because I don't want to
hurt my husband or cause him trouble at his job.

Well, anyway. Do you know of marriages that have survived these types of crises? I can't
stand the thought of being separated from my husband but I can't stand the thought of
continuing this pretending, either.

(Anonymous)

EDITOR'S NOTE: Normally, I don't publish anonymous letters, but I am making an exception
in this case because of the special situation the letter writer is in. I have altered with x's any
words in the letter that might betray the identity of the writer.

I was unable to be of much help to her, because it so happened that when I told my wife that I
had lost faith in the Bible, I learned that she felt the same way. In case there may be readers
who have had the kind of experience this minister's wife is now coping with, I am publishing
her letter anonymously (with her permission). I suggested to her that she should not think that
she "messed things up" but that her husband messed up by not outgrowing superstitious
beliefs as she has done. If anyone has suggestions that may be helpful to her, I will pass them
along to her.

Volume 1990 - 2002 Issue


Page 1394 of 2049
Skeptical Review Edited by Farrell Till

Skeptical Review
Volume Ten, Number Three
May/June 1999
Farrell Till, editor

• "Biblical Anachronisms"
Biblical characters and events presented out of their proper time sequences show that
the Bible is not inerrant.
• "Still Grasping for Straws"
Farrell Till concludes his response to Everett Hatcher's last article in defense of a 6th-
century B. C. authorship of the book of Daniel.
• "When the Fig Trees Puts Forth Its Leaves"
Farrell Till concludes his discussion of alleged prophecy fulfillments that Bruce
Weston had identified as reasons why he is reluctant to reject belief in the inspiration
of the Bible.
• "Who's to Blame? More Hype Than Substance"
Biblical inerrantist Roger Hutchinson takes issue with Farrell Till's belief that the
Bible must bear some responsibility for modern-day violence perpetrated by Bible
believers.
• "The Two Faces of the Bible"
Farrell Till replies to Hutchinson to show that in addition to its "message of love" the
Bible also teaches bigotry and hatred.
• "Ezekiel's Prophecy against Egypt"
Dave Matson discusses absurdities in the belief that Ezekiel's prophecy against the
pharaohs of Egypt will yet be fulfilled at some future date.
• "From the Mailbag"
Readers and Editor Farrell Till discuss a variety of religious topics and issues.

Volume 1990 - 2002 Issue


Page 1395 of 2049
Skeptical Review Edited by Farrell Till

Biblical Anachronisms

Anachronisms occur in written documents when they make references to anything (persons,
places, events, etc.) that did not belong to the era in which the documents were set. If a history
of the Civil War made references to aerial bombardments and said that Thomas Jefferson was
the president at this time, these would be anachronisms, because airplanes didn't exist then
and Jefferson was president 50 years earlier. Anyone seeing such references in a history book
would immediately realize that the writer was poorly informed about some aspects of this
war. Certainly, no one would consider the author to be an infallible writer.

If we would react this way to anachronisms in a history book, consistency should require us to
react accordingly to anachronisms in the Bible. Inerrantists, of course, are loath to admit it,
but anachronisms are in the Bible. Genesis 36:31, for example, referred to kings that reigned
over Israel: "Now these were the kings who reigned in the land of Edom before any king
reigned over the children of Israel." Biblical fundamentalists believe that the book of Genesis
was written by Moses and will vigorously oppose any attempts to prove otherwise, but
according to Deuteronomy 34:1-7, Moses died and was buried in an unknown location before
the Israelites had even entered into the promised land. The first king of Israel was Saul, who
was anointed king in 1 Samuel 10:1, but if biblical chronology is correct, this would have
been approximately 400 years after Moses had died. If Moses did indeed write the book of
Genesis, how could he have known about kings who wouldn't exist for another four centuries?
Inerrantists will simplistically argue that he knew this by "inspiration," but a more reasonable
explanation would be that Genesis was written by someone else long after the time people
have traditionally believed.

Moses is the assumed author of the book of Exodus too, but another anachronism also casts
suspicion on the Mosaic authorship of this book. The writer, whoever he was, said that when
pharaoh allowed the Israelites to leave Egypt, "God did not lead them by way of the land of
the Philistines, although that was near" (Ex. 13:17). In other words, the writer was claiming
that God didn't lead the Israelites out through the land of the Philistines, even though that
would have been the shortest route, and the reason given was that the people might "see war,"
change their minds, and return to Egypt. So instead God took them the long way around,
where the Israelites still saw war with the Amalekites and other warring tribes, and constantly
bellyached and asked to return to Egypt, but that is another article. For now, we want to focus
on the anachronistic reference to the Philistines.

If biblical chronology is accurate, the exodus would have occurred around 1495 B. C., at
which time the Philistines had not yet settled into Western Canaan. Archaeological work has
confirmed that the Philistines were a "sea people," who migrated south, probably from Greece
and the Balkans, in the 13th and early 12th centuries B. C. They attempted to settle in the Nile
delta but were repulsed by Pharaoh Merneptah in the late 13th century. They settled on the
Palestinian coast, where they continued to harass Egypt, but near the beginning of the 12th
century, they were defeated by Ramses III in a combined land and sea battle that ended their
threat to Egypt. Dating from the 12th century, the annals of Ramses III and Egyptian stone

Volume 1990 - 2002 Issue


Page 1396 of 2049
Skeptical Review Edited by Farrell Till
reliefs at his temple to Ammon at Medinet Habu near Thebes tell of these battles with the
Philistines, so the southward migration of the Philistines actually occurred after the Israelite
exodus from Egypt. The reference to them in Exodus 13:17 is an anachronism that was either
penned by the real author of Exodus, who would have lived well after Moses, or else is a
textual insertion that was added to the original work. Either option does serious damage to the
inerrantist view of infallibly inspired biblical writers.

The Exodus writer also referred to Israelite priests before any priests had been appointed. In
Exodus 19, Moses had presumably been called to the top of Mount Sinai to chat with Yahweh
(v:22), at which time Yahweh gave a strict warning to Moses that the people should stay away
from the mountain: "Then Yahweh said to Moses, `Go down and warn the people, lest they
break through to gaze at Yahweh, and many of them perish. Also let the priests who come
near Yahweh consecrate themselves, lest Yahweh break out against them'" (vs:21-22). After
Moses had reminded the omniscient Yahweh that the people couldn't come to the mountain
because Yahweh had previously warned them that they would die if they touched it, Yahweh
said, "Away! Get down and then come up, you and Aaron with you. But do not let the priests
and the people break through to come up to Yahweh, lest he break out against them" (v:24).

So on this occasion, the inscrutable Yahweh twice referred to "the priests" and warned them
of dire consequences if they came near the mountain while the confab with Aaron and Moses
was in progress, but the problem for biblical inerrantists is that there were no priests at this
time for Moses to warn. Aaron and his sons were the first priests appointed to serve in the
new religion that Yahweh allegedly revealed to Moses on Mount Sinai, but their appointment
did not come until after Moses had returned from the mountain with intricate instructions on
how to build a tabernacle and the furniture and vessels that would be in it. Not until Exodus
28-29, after Moses had returned from the mountain, were Aaron and his sons consecrated as
priests, so it is hard to understand why an omniscient deity would not have known this before
telling Moses to warn nonexistent priests to stay away from the mountain. This is the kind of
anachronistic mistake that would be understandable in the writings of normal, fallible
humans, but we certainly wouldn't expect to find it in the writings of someone who was
presumably inspired by an omniscient, omnipotent deity.

That is exactly why rational people just can't believe that "God" inspired the writing of the
Bible. There are just too many mistakes like these that discredit the divine-inspiration claim.

FREE SUBSCRIPTION: For a free subscription to The Skeptical Review, e-mail


jftill@midwest.net or write to P. O. Box 717, Canton, IL 61520-0717.

Still Grasping for Straws


Farrell Till

Volume 1990 - 2002 Issue


Page 1397 of 2049
Skeptical Review Edited by Farrell Till
Near the end of his article in the March/April issue, Everette Hatcher justified his excessive
appeals to authority on the grounds that he is "not an archaeologist or a linguist" (p.6), and so
he "must use authorities in his articles." There is nothing at all wrong with "us[ing]
authorities" in an article if it is done in the right way. A skilled debater will present his case
through the development of his own arguments after which he may want to refer to a few
experts to show that his position is in agreement with what recognized authorities have said,
but if the debater does little more than just quote expert after expert and cite reference book
after reference book without attempting to show that the opinions of the experts are based on
sound evidence, the appeal to authorities becomes both abusive and fallacious. If Hatcher
doesn't consider himself qualified enough in archaeology or linguistics to articulate his own
arguments, then he should leave the apologetics to those who can.

That Hatcher made excessive appeals to authority without trying to support the opinions of his
"scholars" with logical argumentation can be seen by just glancing through his article. It is
filled from end to end with long columns of italicized book titles and the indented small-print
paragraphs where long quotations were cited. I did a computer analysis of Hatcher's article
and found that it contained 6,671 words. An article this long would be tedious reading even if
it contained no other distractions, but when I blocked off all of the places where Hatcher had
listed columns of books and articles or quoted from some authority he was appealing to, my
computer showed that 4,332 words were used in these sections. In other words, almost two
thirds of the article consisted of nothing but quoting authors or listing columns of books and
articles. The result was a very tedious and boring article to read, whose unconvincing
approach to persuasion was easily recognized by those who know that simply appealing to
authority is a logical fallacy. As some of the letters in this issue (pp. 12-13) will show, if
Hatcher had hoped to sway the opinion of TSR readers with an impressive array of quotations
and book titles, he failed.

I want Hatcher and other potential contributors of articles to understand that I am not
exercising editorial authority to forbid the use of quotations and the citation of reference
books and articles. I'm just asking those who submit articles to understand that there is more
to persuasive writing than the mere quoting of authorities in agreement with the position
being defended. If an authority is cited, the writer should, at the very least, attempt to show
why the quoted opinion should be considered correct. In going through Hatcher's article, I
found constant expressions like "the critic Jeffrey states," "the critic John Goldingay admits,"
"the critic W. H. Brownlee asserted," "the critic Brian E. Colless concluded," "even the critic
John Day admits," "other critics agree," "no wonder the critic Philip R. Davies concluded,"
etc., etc., etc. In most of these instances, Hatcher merely quoted what these critics had "stated"
or "admitted" or "concluded" or "agreed" without any attempt at all to justify through
argumentation what they had said, and in all but a few of these references, not enough was
said for readers to understand why critic so-and-so had said or concluded whatever was
quoted. That is not a proper use of references; it is a flagrant appeal to authorities intended to
sway readers into thinking that they should fall in line with Hatcher because so many
"scholars" seem to be on his side. But it didn't work. Most TSR readers understand that even
though Hatcher can cite authorities who think that the writer of Daniel understood that
Babylon fell to the Persians, there are nevertheless many reputable scholars who think that the
writer believed that Babylon fell to the Medes. The same is true of just about any other
authority he can cite. Can Hatcher cite authorities who think that the Aramaic style in Daniel

Volume 1990 - 2002 Issue


Page 1398 of 2049
Skeptical Review Edited by Farrell Till
indicates a 6th-century B. C. authorship? Well, I can cite authorities who think that the
Aramaic style indicates a later authorship. Can Hatcher cite authorities who think that the
writer of Daniel showed an accurate knowledge of Babylonian and Persian history? I can cite
authorities who think that he didn't. In other words, a war of "my scholars" against "your
scholars" is not going to settle anything. The readers of this debate are interested in seeing a
defense of Hatcher's reasons for accepting the 6th-century B. C. authorship of Daniel, not in
seeing how many "scholars" he can cite who agree with his position.

How long was required for manuscripts to be recognized as "scripture"? Here and there
in Hatcher's comments and occasionally even in his quotations, there were at least some weak
attempts to defend the traditional dating of Daniel with argumentation, and these deserve
replies, which I will now attend to. Taking a cue from his "critics," Hatcher argued that the
discovery of fragments of Daniel among the Dead Sea Scrolls recovered at Qumran must be
regarded as evidence that Daniel was written well before the 2nd century B. C. His argument
seemed to be that (1) the oldest of these fragments of Daniel has been dated by Frank Cross at
"the late 2nd century B. C.," (2) this would be "no more than about a half century younger
than the original autograph" [if it were indeed written around 164 B. C. as liberal critics
claim], but (3) the Qumran community would not have accepted Daniel "as Scripture only
fifty years after its composition" ( March/April 1999, p. 3).

When Hatcher quoted authorities, he usually just stated their opinions without making any
effort to defend their conclusions, and he has done the same here. He simply recycled an
argument that he found in the writings of DiLella and Collins, but he made no effort at all to
show that their conclusion necessarily follows from their premises. Why would writings have
to be older than 50 years before people would accept them as "scripture"? Hatcher didn't say;
he just made the assertion and then moved on. How long had the writings of Isaiah and
Jeremiah existed before they were accepted as scripture? We don't know, but if the book of
Jeremiah is to be trusted, he was a respected and feared prophet in his own lifetime, whose
opinion was sought by the rulers of Judah (38:24-28). Prior to this, Jeremiah claimed that a
scroll he had written so impressed a temple leader that he had it read in the presence of the
scribes and princes of Judah, who then sent it to the king and had it read in his presence
(chapter 36), so if the Bible is to be trusted, Hatcher should recognize that the writings of
some prophets were highly respected within the lifetimes of their authors.

On page 13 of this issue, there is a letter from Richard Packham, which responded to
Hatcher's 50-year argument by showing the rapidity with which the Book of Mormon was
accepted by thousands as scripture within a matter of just a few years. Now, only 170 years
later, there are millions all over the world who believe that the Book of Mormon is inspired
scripture, and this has all happened in a time when people were far more educated than they
were in the 2nd century B. C. I could cite other examples of people like Ellen G. White, Mary
Baker Eddy, and Ron Hubbard, whose writings were venerated almost immediately by their
followers, so if Hatcher expects us to see any merit in his argument, he is going to have to do
much more than just arbitrarily declare that Jews would not have accepted Daniel as
"scripture" in just 50 years.

Furthermore, he needs to explain why the presence of fragments from Daniel in the
discoveries at Qumran would necessarily prove that the community of Jews there had

Volume 1990 - 2002 Issue


Page 1399 of 2049
Skeptical Review Edited by Farrell Till
accepted this book as inspired scriptures. The Dead Sea Scrolls did not consist of just copies
of Old Testament books. Other works were in the Qumran library, so there is no reason to
believe that they considered everything they copied to be "scripture."

What did Maccabean Jews know about 6th-century Babylon? In an attempt to deny that
the writer of Daniel thought that Babylon had fallen to the Medes, Hatcher recycled Stephen
Miller's claim that it is "unreasonable" even to suggest that "any semi-educated Jew of the
Maccabean period could be ignorant of the fact that it was Cyrus the Persian who conquered
the great Babylonian Empire" ( March/April 1999, p. 3), but to see the flaw in this argument,
we have only to consider the political ignorance of many of our more educated
contemporaries. Does Hatcher think that just any "semi-educated" American living today
would know who was president of the United States during World War II? The Spanish
American War? The war with Mexico? The Great Depression? When I was teaching college
writing, I would sometimes give current-event tests and use the results to impress on students
that they could probably write more intelligently if they would try to stay informed by just
reading daily newspapers and watching TV news. Those tests would repeatedly show that
many students didn't even know simple things like who the current governor of the state or
vice president of the United States was, and these were people living in a time when mass-
media communication systems make information easily accessible. In the letters column of
this issue (pp. 12-13), David Mooney discussed this same issue and mentioned a survey he
recalled that showed many people didn't know basic facts about the American Revolution,
which happened only 223 years ago. Maccabean Jews would have been almost four centuries
removed from the fall of the Babylonian empire, so it is pure speculation to say that in a time
when there were no publishing companies or news media like what we have today, even
"semi-educated" Jews would have known that Cyrus had captured Babylon. The truth is that
there is no way that either Miller or Hatcher can know what "semi-educated" Maccabean Jews
would or would not have known about an empire that had ceased to exist 400 years earlier.
This is just another apologetic assertion for which they offer no supporting evidence.

The availability of the book of Ezra. Miller also argued that if Daniel had indeed been
written in the 2nd century B. C., the author would have had the book of Ezra at his "disposal"
to know that Cyrus was the Persian emperor who "had released the Jews from captivity in
Babylon" (p. 3). There are three problems with this "argument." First, the issue of historical
inaccuracy in the book of Daniel doesn't concern whether the author knew that Cyrus was the
emperor who had released the Jews from captivity, because the book of Daniel closed with no
mentioning of Cyrus's decree to allow the Jews to return to their homeland. The issue centers
on the writer's apparent belief that Babylon had fallen to the Medes rather than the Persians.
Second, the book of Ezra opens with a statement about a decree that Cyrus issued in the first
year of his reign to allow the Jews to return home, but I know of nothing in Ezra that refers to
the actual fall of Babylon. How, then could familiarity with this book have enabled the writer
of Daniel to know that the Persians (Cyrus) and not the Medes (Darius) were the actual
conquerors of Babylon? Third, it is merely an assumption that a 2nd-century B. C. author
would have had access to the book of Ezra. Hatcher seems not to be familiar with critical
works that consider the two Chronicles, Ezra, and Nehemiah the work of a single author, who
compiled them as a continuous narrative at some time between the 4th and 2nd centuries B. C.
Some think that this work was done even as late as the 1st century. I am making no claims
about the dating of Ezra and the other three books; I am simply pointing out that there are

Volume 1990 - 2002 Issue


Page 1400 of 2049
Skeptical Review Edited by Farrell Till
critical opinions about Ezra, which if right, dispute Miller's claim that a 2nd-century B. C.
author of Daniel would have had access to the book of Ezra. Hatcher has recycled this
argument, so it is his responsibility to prove that Ezra had been completed by 164 B. C. and
had been copied and circulated extensively enough that whoever wrote Daniel at this time
would have had the book available to him.

I see no way that Hatcher could possibly prove this, but there is textual evidence in Ezra and
Daniel that does serious damage to this speculative argument that Hatcher has borrowed from
Miller. As I noted above, the book of Ezra opens with a reference to the decree that Cyrus
issued in the first year of his reign to permit the Judean captives to return to their homeland to
rebuild the temple.

In the first year of King Cyrus of Persia, in order that the word of Yahweh by the mouth of
Jeremiah might be accomplished, Yahweh stirred up the spirit of King Cyrus of Persia so that
he sent a herald throughout all his kingdom, and also in a written edict declared: "Thus says
King Cyrus of Persia: Yahweh, the God of heaven, has given me all the kingdoms of the
earth, and he has charged me to build him a house at Jerusalem in Judah. Any of those among
you who are of his people--may their God be with them--are now permitted to go up to
Jerusalem in Judah, and rebuild the house of Yahweh, the God of Israel" (Ezra 1:1-3).

Ezra claimed that this decree was issued in the first year of Cyrus's reign, yet the writer of
Daniel made no reference to it. That's strange indeed, because if, as Hatcher alleges, this
writer knew that Babylon had fallen to Cyrus, why did the book of Daniel never refer to this
decree? The book claims that after Babylon fell, Daniel was made an important official in the
new government (Dan. 6:1-3), so if Darius the Mede was really Cyrus, as Hatcher absurdly
claimed (a point I will address later), then Daniel would have been an official in Cyrus's
government during the first year of his reign, the very time that Ezra said Cyrus issued the
decree noted above, but Daniel never referred to it. Why? Hatcher can't argue that the book of
Daniel closed before the issuance of this decree, because in 10:1, the writer of Daniel referred
to a vision that he received in the third year of "Cyrus king of Persia."

From Daniel's silence about Cyrus's decree, we can reasonably reach two conclusions: (1) It is
unlikely that this book was written by a 6th-century B. C. Jew who was an important official
in Cyrus's government, because such an official would have surely known about the royal
decree to let the Jewish captives return to Jerusalem, and would have mentioned it. (2) If the
author lived later than the 6th century, he probably was unfamiliar with Ezra, or he would
surely have made some reference to Cyrus's decree, which is not only mentioned but quoted
in the opening paragraph of this book. To say the least, Hatcher's argument based on
availability of the book of Ezra has some holes in it that he needs to plug.

Darius the Mede again: Daniel's reference to "Darius the Mede's" reign in Babylon prior to
the reign of Cyrus continues to be a fly in Hatcher's 6th-century B. C. ointment. In the
July/August 1998 issue of TSR, I pointed out that Daniel 9:1 referred to "Darius the Mede" as
the "son of Ahasuerus" and then noted the serious historical blunder that the writer made in so
describing Darius's ancestry, because Ahasuerus was the king of Persia from 485 to 465 B. C.,
which was over half a century after Daniel claimed that Darius the Mede had reigned in
Babylon. So how likely is it that the "son" of a king would have reigned over 50 years before

Volume 1990 - 2002 Issue


Page 1401 of 2049
Skeptical Review Edited by Farrell Till
his father? Even if Hatcher tries to make son mean only "descendant" in this passage, he still
has to explain how that a "descendant" could have reigned so many years before his
"ancestor."

Ahasuerus was the son of Darius the Great, who was the son of Cambyses II, who was the son
of Cyrus. In other words, Ahasuerus was the great-grandson of Cyrus, so if "Darius the Mede"
was the son of Ahasuerus, this would have made him the great-great-grandson of Cyrus, and
that would mean that the writer of Daniel somehow thought that the great-great-grandson of
Cyrus had reigned in Babylon before Cyrus did. That's how confused the writer was about
6th-century B. C. Babylonian and Persian history.

How did Hatcher respond to this? I hope readers who still have their March/April issue of
TSR will look on page 3 (middle column) and notice that he quoted the paragraph in which I
originally presented the information about Ahasuerus and then waved it aside with, "The critic
John Goldingay admits that `Ahasuerus' probably is a title and not a personal name." And that
was all he said! He didn't try to explain why Goldingay "admits" this; he simply declared that
he did and went on, as if he expected us to accept this just because a "critic" had said it. Well,
I have consulted several Bible dictionaries and reference books and found none that said
"Ahasuerus" was a title and not a personal name. They all identified Ahasuerus as the Persian
king, who was known as Xerxes in Greek and who reigned from 485 to 465 B. C.

Hatcher introduced the book of Ezra into this controversy, so let's notice a passage in this
book that shoots down his quibble that Ahasuerus was just a Persian title and not a name. The
writer of this book discussed the opposition from surrounding areas that the Jewish repatriates
experienced when they tried to rebuild the temple.

Then the people of the land tried to discourage the people of Judah. They troubled them in
building, and hired counselors against them to frustrate their purpose all the days of Cyrus
king of Persia, even until the reign of Darius King of Persia.
In the reign of Ahasuerus, in the beginning of his reign, they wrote an accusation against the
inhabitants of Judah and Jerusalem. In the days of Artaxerxes also, Bishlam, Mithredath,
Tabel, and the rest of their companions wrote to Artaxerxes king of Persia (Ezra 4:4-6).

The writer of Ezra referred to four separate Persian kings in this passage: Cyrus, Darius,
Ahasuerus, and Artaxerxes. The last three were direct descendant kings of the first. There is
no dispute about whether Cyrus, Darius, and Artaxerxes were names and not titles, so
Hatcher's claim that Ahasuerus was a title of Persian kings and not a name will require him to
give a plausible explanation for why the writer of Ezra in the passage above used names in
referring to three Persian kings but for some reason used a "title" in referring to the fourth.
Such a quibble as this only demonstrates the extremes that biblicists will go to in order to
defend their inerrancy belief.

Dual titles? Hatcher resorted to a similar quibble to explain the "Darius the Mede" problem.
"Many evangelicals," he said, "have put forth the theory that Darius is a title for Cyrus" (
March/ April, p. 4). He, of course, listed three writers who have "put forth" this theory, but the
only support that he gave for the argument was the citation of a couple of scriptures that show
that some biblical characters had two names. In both cases, however, the biblical text is rather

Volume 1990 - 2002 Issue


Page 1402 of 2049
Skeptical Review Edited by Farrell Till
clear in stating that these characters had dual names. Daniel and his three friends were one set
of examples, but the Bible is very explicit in stating that Daniel was also given the
Babylonian name Belteshazzar and that Hannaniah, Mishael, and Azariah were given the
names Shadrach, Meshach, and Abednego (Dan. 1:7). On the other hand, Hatcher can find no
text that unequivocally says that Cyrus was also named Darius.

The best that Hatcher could do to support this quibble was to cite Daniel 6:28, which says that
"Daniel prospered in the reign of Darius, and in the reign of Cyrus the Persian." The quibble
is that the Aramaic conjunction waw translated and in English versions really meant "even,"
and so the verse was actually saying that Daniel prospered in the reign of Darius, even in the
reign of Cyrus the Persian." This, however, is far from an explicit passage where Cyrus was
known by both the names Darius and Cyrus. I checked 25 different translations in my
personal library, including one French version and two literal translations, and I found none
that translated the waw conjunction with even. They all say that Daniel prospered in the reign
of Darius and in the reign of Cyrus the Persian. The version of the Jewish Publication Society
rendered the verse like this: "Thus Daniel prospered during the reign of Darius and during the
reign of Cyrus the Persian." If there is any merit at all to Hatcher's quibble, it is certainly
strange that so many translators failed to see the real meaning of this verse.

This "solution" to the Darius the Mede problem is weakened by three stubborn facts that
Hatcher must confront: (1) As I have already noted, Daniel described "Darius the Mede" as
the "son of Ahasuerus," and since his quibble that "Ahasuerus" was just a "title" failed to
carry the day, Hatcher must explain how that Cyrus, if he was indeed the same person as
"Darius the Mede," could have been the son of his great-grandson son. (2) Cyrus captured
Babylon in 539 B. C., and Ahasuerus (as previously noted) reigned as king of Persia from 485
to 465 B. C., so if Cyrus and Darius were one and the same and if Darius (Cyrus) was the son
of Ahasuerus, how was it possible that the son reigned as king of Persia over 50 years before
his father did? (3) The writer of Daniel dated his visions with reference to the reigns of Darius
and Cyrus. In 9:1 and 11:1, visions that Daniel had were both dated in the "first year of
Darius"; in 10:1, a vision came to Daniel in the "third year of Cyrus." How does Hatcher
explain this? If Darius and Cyrus were really the same person, why was there no explicit
statement to that effect, as there was in 4:19, where the dual names of Daniel were clearly
identified: "Then Daniel, whose name was Belteshazzar, was stricken dumb...." Surely, if
Darius and Cyrus were dual names for the same person, the writer would have given some
such indication, but it was never done. The writer always dated his visions with references to
the "first year of Darius" and the "third year of Cyrus," just as he had dated dreams prior to
Babylon's fall in the "first" and "third" years of Belshazzar (7:1; 8:1). There is no reasonable
explanation for why the writer of Daniel repeatedly referred to Darius and Cyrus by different
names except that he thought that they were separate individuals. To quibble that Darius and
Cyrus were dual names and "Ahasuerus" a title is not reasonable explanation; it is a grasping
for just any straw in sight to defend a cherished inerrancy belief.

The question of when the book of Daniel was written is really rather minor to the issue of
biblical inerrancy. What matters is whether the book is historically accurate, and in my final
reply to Hatcher, I will review some discrepancies in Daniel that he has yet to resolve
satisfactorily.

Volume 1990 - 2002 Issue


Page 1403 of 2049
Skeptical Review Edited by Farrell Till
FREE SUBSCRIPTION: For a free subscription to The Skeptical Review, e-mail
jftill@midwest.net or write to P. O. Box 717, Canton, IL 61520-0717.

When the Fig Tree Puts Forth Its Leaves


Farrell Till

In the past two issues of TSR, I have discussed various claims of prophecy fulfillment that
Bruce Weston expressed concern about in his article " Doubts But Questions about Prophecy"
in the January/February issue (pp. 6-7). In one case, he thought it possible that what skeptics
consider an example of prophecy failure wasn't necessarily a failure. This was the occasion of
an apparent promise that Jesus made that he would come again before "this generation" had
passed away. In Matthew 24, the disciples of Jesus asked him to tell them what would be the
sign of his coming and of the end of the world (v:3). Over the space of several verses (4-31),
Jesus answered their question and told of several "events" that would happen prior to his
return and the signs that would accompany his coming, after which he made the statement that
Weston inquired about.

Now learn this parable from the fig tree: When its branch has already become tender and puts
forth leaves, you know that summer is near. So you also, when you see all these things, know
that it is near--at the doors! Assuredly, I say to you, this generation will by no means pass
away till all these things take place (vs:32-34, emphasis added).

Weston wanted me to address "the Christian defense that by saying this generation, Jesus was
talking about the same generation that sees the fig tree put forth its leaves (and not to the
current generation of His time)" [TSR, January/February 1999, p. 6]. Weston then went on to
suggest that the fig tree was "symbolic of Israel, which was reborn as a nation in 1948," and
so this interpretation would mean that Jesus was saying that he would return before the
generation that witnessed the "rebirth" of Israel had passed away and not before the passing
away of the generation of his own time.

This is indeed an interpretation that modern apologists have resorted to in order to "explain"
what must otherwise be considered a flagrant prophecy failure. I hesitate to use the word
"interpretation" in reference to this view of the passage, because such a far-fetched "spin" as
this on a text that doesn't even mention the nation of Israel hardly qualifies to be called an
interpretation, but since Weston has obviously fallen under the influence of this
"explanation," I will, in compliance with his request, "address" it to show that it is too far-
fetched and unlikely to take seriously.

First, we should notice that this passage is not the only one where Jesus promised that he
would return before his own generation had passed away. One of the clearest statements of
this promise was made in Matthew 16:27-28.

Volume 1990 - 2002 Issue


Page 1404 of 2049
Skeptical Review Edited by Farrell Till
For the Son of Man will come in the glory of His Father with His angels, and then He will
reward each according to his works. Assuredly, I say to you, there are some standing here
who shall not taste death till they see the Son of Man coming in His kingdom.

Biblical inerrantists have tried to explain away the problem in this text by arguing that it was
a reference to the glorification of Jesus in the transfiguration, which followed immediately in
chapter 17. The best way to see the weakness of this explanation is to note that in the text
quoted above, Jesus was describing the final judgment when he would come with his angels
and reward every person according to his works, but the transfiguration scene (quoted below)
mentions nothing at all about angels or judgment.

Now after six days Jesus took Peter, James, and John his brother, led them up on a high
mountain by themselves; and He was transfigured before them. His face shone like the sun,
and His clothes became as white as the light. And behold, Moses and Elijah appeared to them,
talking with Him. Then Peter answered and said to Jesus, "Lord, it is good for us to be here; if
You wish, let us make here three tabernacles: one for You, one for Moses, and one for
Elijah." While he was still speaking, behold, a bright cloud overshadowed them; and suddenly
a voice came out of the cloud, saying, "This is My beloved Son, in whom I am well pleased.
Hear Him!" And when the disciples heard it, they fell on their faces and were greatly afraid.
But Jesus came and touched them and said, "Arise, and do not be afraid." When they had
lifted up their eyes, they saw no one but Jesus only" (Matt. 17:1-8).

That a final judgment in which all people will be rewarded according to their works will occur
when Jesus comes again is a well defined New Testament doctrine. The apostle Paul said,
"For we must all appear before the judgment seat of Christ that each one may receive the
things done in the body, according to what he has done, whether good or bad" (2 Cor. 5:10).
The book of Revelation closed with a warning of this final judgment: "Behold, I come quickly
and my reward is with me, to render to each man according to his works" (22:12). In his
interpretation of the parable of the tares, Jesus was very clear in saying to his disciples that the
final judgment would take place at the end of the world.

He that sows the good seed is the Son of Man; and the field is the world, and the good seed,
these are the sons of the kingdom; and the tares are the sons of the evil one; and the enemy
that sowed them is the devil: and the harvest is the end of the world; and the reapers are
angels. As therefore the tares are gathered up and burned with fire; so shall it be in the end of
the world. The Son of Man shall send forth his angels, and they shall gather out of his
kingdom all things that cause stumbling, and them that do iniquity, and shall cast them into
the furnace of fire: there shall be the weeping and the gnashing of teeth. Then shall the
righteous shine forth as the sun in the kingdom of their Father. He that has ears, let him hear
(Matt. 13:37-43).

Other scriptures could be cited, but these are sufficient to show that the New Testament
teaches that the second coming of Jesus will signal the end of the world, at which time there
will be a final, personal judgment on the basis of what each individual did in his/her lifetime.
The fanciful little yarn about the transfiguration of Jesus says nothing at all about angels
coming with Jesus or the rewarding of every person according to his works; therefore, it could
hardly be the fulfillment of his promise that some who were listening to him speak would not

Volume 1990 - 2002 Issue


Page 1405 of 2049
Skeptical Review Edited by Farrell Till
die until they had seen him return with his angels to reward every person according to his
works. No judgment took place on the "mount of transfiguration" and certainly this event
didn't usher in the end of the world, so how could it be a fulfillment of a promise that some
living then would be alive to see Jesus return to render judgment to all according to their
works? Just common sense should tell readers of this passage that when Jesus said that some
standing by (hearing him speak) would not die until they had seen him coming in his
kingdom, he was not talking about something that was going to happen the very next week
(after six days, Matt. 17:1), because that would mean that he was saying, in effect, that some
who were hearing him speak would not die until they had seen something that was going to
happen right away. The language of the text is such that it communicates the idea that some
hearing him speak would live to see an event that wasn't going to happen immediately but
would at least happen within their lifetimes.

Another interpretation of the promise in Matthew 16 is that Jesus was only saying that some
would not taste of death until they saw "the Son of Man coming in his kingdom" and that the
kingdom was the church, which was established on the day of Pentecost (Acts 2). Hence,
there were probably many listening to Jesus in Matthew 16 who also witnessed the coming of
the church (kingdom) in Acts 2, and so this promise was fulfilled. A major problem with this
interpretation is that it does not address the obvious fact that Jesus was speaking in Matthew
16 about a judgment that would be rendered to all people according to their works, and this
did not happen on the day of Pentecost. Furthermore, Jesus clearly said in this text that he
would "come in the glory of His father with His angels" to "reward every man according to
his works," and neither Jesus nor his angels were seen coming on the day of Pentecost. Such
interpretations as these, like the one that is troubling Weston, are far-fetched attempts to try to
explain away the obvious fact that Jesus taught that he would return soon, within the lifetime
of the people of his generation. That didn't happen, and so biblicists who are determined to
believe that the Bible is the inspired, inerrant word of God have resorted to all sorts of
fanciful speculations to try to explain away New Testament passages in which Jesus clearly
promised that his return would happen before his generation had passed away.

New Testament writers in general also taught that the end of the world was imminent. The
writer of 1 Peter said that "the end of all things is at hand" (4:7). James 5:8 admonished
Christians to "establish [their] hearts, for the coming of the Lord is at hand." The writer of 1
John said that the coming of "many Anti-Christs" enabled Christians to know that "it is the
last hour" (2:18). The writer of Hebrews said that Christ had appeared "once at the end of the
ages" to "put away sin by the sacrifice of Himself" (9:26). This was an obvious reference to
the crucifixion of Jesus, doctrinally considered to be a vicarious "atonement" for sin, and the
Hebrew writer said that it had happened at the "end of the ages," but if almost two thousand
years have since passed, how could this event be correctly described as something that had
happened at the end of the ages? The reasonable conclusion is that this writer sincerely
thought, like the writers of 1 Peter, James, and 1 John, that the end of the world was at hand,
but he was mistaken. After all, we see people today declaring their sincere beliefs that the end
of the world is at hand, as others have done throughout the Christian era, so it would not have
been unusual at all for religious alarmists in the early church to think that the end of the world
was "at hand." The remarkable thing is that so many people today are so determined to
believe that Jesus is coming again that they will ignore all of the false warnings of his return
that have been sounded in the past and resort to all sorts of unlikely interpretations to show

Volume 1990 - 2002 Issue


Page 1406 of 2049
Skeptical Review Edited by Farrell Till
that the New Testament warnings of an early return didn't really mean what they appear to
mean.

A popular quibble used to "explain" why Jesus hasn't returned yet, as some New Testament
passages seemed to promise, is based on 2 Peter 3:8, which says that "with the Lord one day is
as a thousand years, and a thousand years as once day," so die-hard inerrantists will argue that
when New Testament writers said that it was the "last hour" or that the end was "at hand,"
they were speaking in terms of how God perceives time. Yeah, right! We are supposed to
believe that God inspired certain writers to tell people that Jesus would come again someday,
but he chose to do so in language that had meaning only to an omniscient, omnipotent deity.
"Soon" didn't mean "soon," and "at hand" didn't mean "at hand," as humans understand these
terms, but as God understands them. Such an explanation makes God a devious entity who
chose to reveal important information in sort of a secret code that would have meaning only to
an omniscient, omnipotent deity. Biblicists may think that this quibble is a reasonable
explanation for the texts that said it was "the last hour" or that the end was "at hand," but they
know that it doesn't give a satisfactory explanation to the passages where Jesus said that he
would return before his generation had passed away, and so they resort to all sorts of
speculative interpretations to make these passages not mean what they clearly say.

That brings us back to the fanciful interpretation of Matthew 24:34, which apparently has
Bruce Weston troubled. "This generation" didn't mean the generation that Jesus was talking to
but the generation that would be alive when the nation of Israel was restored. In other words,
by arguing that Jesus didn't really mean what he seemed to be saying, biblicists have tried to
escape from the false prophecy that results when this passage is understood to mean exactly
what the language in it says: "(T)his generation will by no means pass away till all these
things take place."

Weston has apparently bought the idea that Jesus was referring to the generation that would
be living when the nation of Israel would be restored, but there is nothing in the text that even
remotely suggests that Jesus was referring to a restoration of the nation of Israel." Indeed,
Israel isn't even mentioned in the entire context of this conversation between Jesus and his
disciples. Furthermore, there are two other accounts of this conversation (Mark 13; Luke 21),
and, like Matthew, neither one of them mentioned the nation of Israel. That fact alone should
be sufficient for Weston and anyone else who has been taken in by this "interpretation" to
reject it.

An analysis of Matthew 24 shows that Jesus remarked to his disciples that the time would
come when the buildings of the temple would be destroyed and "not one stone would be left
upon another" (vs:1-2). His disciples responded with two questions: (1) When will these
things be? (2) What will be the sign of your coming and of the end of the world? (The second
question indicates that the disciples thought that the coming of Jesus would also be the end of
the world, and that view is consistent with scriptures I have already cited.) The rest of the
chapter consists of the answers that Jesus gave to these questions. When will these things be
or, in other words, when would the temple buildings be destroyed? Jesus's answer to this
question ran through verses 4-22, where he told the disciples that there would be wars and
rumors of war, that nation would rise against nation, that there would be famines, pestilences,
and earthquakes, which would be only "the beginnings of sorrow" (v:8). He told them that

Volume 1990 - 2002 Issue


Page 1407 of 2049
Skeptical Review Edited by Farrell Till
they would be persecuted and delivered up to tribulations and that the gospel would be
preached "in all the world as a witness to all nations" and that only then would the end come
(v:14).

In verses 15-22, he told the disciples to expect to see an "abomination of desolation" standing
in the holy place, which was to be a warning to those in Judea to flee to the mountains. He
then spoke of a "great tribulation, such as has not been since the beginning of the world" and
would never be thereafter (v:21). Most New Testament scholars understand this section to be
referring to the destruction of Jerusalem in A. D. 70 by the Romans, which was when the holy
place was desecrated by Gentiles who entered into it before the temple was destroyed. This
view receives strong support from Luke's parallel account in which he said, "When you see
Jerusalem surrounded by armies, then know that its desolation is near" (21:20). Jerusalem was
surrounded by the Roman army prior to the destruction of the city and temple.

At this point, Jesus had answered the first question of his disciples, who had asked when the
temple buildings would be destroyed, and so he turned to answering the second question:
"What will be the sign of your coming and of the end of the world?" To show that Jesus
meant for his disciples to understand that his "coming" would happen soon, within their
lifetime, we need to read exactly what he said in answering their second question.

Immediately after the tribulation of those days the sun will be darkened, and the moon will
not give its light; the stars will fall from heaven, and the powers of the heavens will be
shaken. Then the sign of the Son of Man will appear in heaven, and then all the tribes of the
earth will mourn, and they will see the Son of Man coming on the clouds of heaven with
power and great glory. And He will send His angels with a great sound of a trumpet, and they
will gather together His elect from the four winds, from one end of heaven to the other"
(vs:29-31, emphasis added).

I have emphasized two words, immediately and then, in this text to show how unlikely it is
that Jesus was referring to events that still haven't happened after almost two thousand years.
He said that immediately after the "tribulation of those days," certain phenomenal signs
would be seen, i. e., the darkening of the sun, the moon not giving its light, and the stars
falling from heaven (as if such were even possible). I have noted that there is general
agreement among New Testament scholars that in the verses prior to this one, Jesus was
telling his disciples of a great tribulation that would happen when Jerusalem was destroyed,
so in saying immediately after the "tribulation of those days," certain celestial signs would be
seen, he surely was saying that these signs would come immediately after "those days" of the
tribulation that accompanied the destruction of Jerusalem. Inerrantists, of course, will argue
that immediately didn't really mean immediately in the sense that humans use the word but in
the sense of immediacy in God's mind, so it would be possible for all of this to happen even
thousands of years from now and that still be "immediately" as far as God is concerned. In
other words, the inerrantists must again resort to arguing that even though God revealed the
Bible for human benefit, he nevertheless inspired it in some sort of secret code that would be
understood only by him. It's just another attempt to explain away biblical discrepancies by
arguing that the Bible doesn't really mean what it says. It presents inerrantists as arguing that
God couldn't inspire clear writing, and so modern inerrantists are needed to explain to people
what God really meant when he inspired his chosen writers to say whatever. It's an argument

Volume 1990 - 2002 Issue


Page 1408 of 2049
Skeptical Review Edited by Farrell Till
that makes sense only to those who are determined to go to any extreme to make the Bible
inerrant, but to those who read the text with no inerrancy axe to grind, verbal gymnastics
aren't needed to understand what it means. Jesus told his disciples that a "great tribulation"
would accompany the destruction of the temple, which happened when the Romans destroyed
Jerusalem, and that immediately after "the tribulation of those days" certain celestial signs
would be seen: the sun would be darkened, the moon would not give its light, the stars would
fall from heaven, and the powers of the heaven would be shaken.

The next key word in the text is then. Jesus said that after the appearance of the celestial
signs, which would follow immediately after the tribulation of those days, "the sign of the Son
of Man" would appear in heaven. All tribes of the earth would mourn, and they would see the
Son of Man coming on the clouds of heaven. He would send his angels forth with a great
sound of a trumpet, and they would gather together his elect from the four corners of the
earth. The language in this passage is too similar to other New Testament descriptions of the
second coming to deny that Jesus was here too speaking of his return. Revelation 1:7 says,
"Behold, He is coming with clouds, and every eye will see Him, even they who pierced Him.
And all the tribes of the earth will mourn because of him." The apostle Paul wrote in his first
epistle to the Thessalonians, "For the Lord himself will descend from heaven with a shout,
with the voice of an archangel, and with the trumpet of God. And the dead in Christ will rise
first" (4:16). In referring to the end of time when Christ would "deliver the kingdom to God
the Father" (1 Cor. 15:24), Paul also spoke of a trumpet that would sound at the final
resurrection, "Behold, I tell you a mystery: We shall not all sleep, but we shall all be changed-
-in a moment, in the twinkling of an eye, at the last trumpet. For the trumpet will sound, and
the dead will be raised incorruptible, and we shall be changed" (vs:51-52). This passage not
only uses the same terminology that is found in Jesus's description of his coming in Matthew
24:31 but it also presents the view that this sounding of the trumpet was an event that would
happen soon. Paul did not speak of this as something that would happen to those who would
be living at some time in the remote future but as something that was going to happen to him
and the Corinthian Christians he was writing to. "We shall not all sleep," he said, "but we
shall be changed." He said that the trumpet would sound and we--not people in the distant
future--will be changed. Clearly, Paul thought that the return of Jesus and the final
resurrection were events that would happen within his own life time, but since all of these
expectations have proven to be wrong, biblicists have been required to put far-fetched,
unlikely spins on the language of all New Testament passages that spoke about an early return
of Jesus.

Among the more popular resorts to absurd interpretations is the one that Bruce Weston
inquired about. He wondered if perhaps Jesus meant another generation besides his own when
he described his second coming and then said that "this generation" would not pass away until
all these things happen. Weston wondered if it is possible that Jesus was referring to the
generation that would be living when the nation of Israel was restored. I have already noted
that the context in which this statement was made never once referred to the nation of Israel,
so that fact alone should be enough to discredit this interpretation. How likely is it that Jesus
meant a generation of people that would be living when the nation of Israel was restored if the
nation of Israel was nowhere mentioned in the entire discourse that Jesus had with his
disciples? To make Jesus's analogy of the fig tree refer to something that wasn't even part of
the conversation on this occasion is literarily unsound. As I have pointed out, Jesus was

Volume 1990 - 2002 Issue


Page 1409 of 2049
Skeptical Review Edited by Farrell Till
simply answering two questions his disciples had asked: (1) When will the temple be
destroyed? (2) What will be the signs of your coming and of the end of the world? After he
had answered these questions, he used the analogy of the fig tree to help the disciples
understand how they could know when the end would come. "When you see a fig tree putting
forth its leaves, you know that summer is near, and so in the same way when you see all these
things, you should know that it [the end] is at the door," would be an accurate paraphrase of
what Jesus was saying, so the key to understanding what he meant lies in understanding what
"these things" meant. The biblicists who have influenced Mr. Weston want us to believe that
"these things" were the events that would surround the restoration of Israel, but as I have
already noticed, the restoration of Israel was not even a topic of conversation on this occasion.
A far more sensible view--and one that doesn't strain principles of literary interpretation--
would be to understand that "these things" referred to all of the events that Jesus had said
would accompany the destruction of the temple. Thus, he was saying to his disciples that
when they had seen all of "these things," they should know that the end (his second coming)
was "at the door" just as surely as they would know that summer was near when they saw a
fig tree putting forth its leaves. Since he went on immediately to say that "this generation"
would not pass away until all these things had happened, the only realistic interpretation of
the text is that he was telling his disciples that he would return soon, within the lifetime of
their generation. Since realistic interpretations sometimes cause serious problems for those
who want to believe that the Bible is "inerrant," biblicists constantly find it necessary to reject
realistic interpretations and search for far-fetched, unlikely ones in order to bring about a
phony "harmony" in their "inspired word of God."

We should also notice that in his analogy of the fig tree, Jesus was speaking in the second
person to his disciples. An examination of the chapter will show that he had done this
consistently all the way through his discourse. He said to them that you will hear of wars and
rumors of wars and went on to say that you should not be troubled (v:6). He told them that
persecutors would deliver you up to tribulation and to kill you (v: 9). He told them that you
will see the "abomination of desolation standing in the holy place" (v:15). He warned them
that if anyone should say to you that Christ is here or there, they should not believe it. In other
words, all through this chapter Jesus was speaking directly to his disciples and properly
addressing them in the second person as you, and no reasonable person can read this text and
deny that he was telling his disciples what they could personally expect to see and experience.
In the analogy of the fig tree, he continued to address his listeners in the second person:
"When its [the fig tree's] branch has already become tender, you know that summer is near.
So you also, when you see all these things, [you] know that it is near--at the doors." He wasn't
addressing this statement to a remotely distant generation of people, but he was speaking to
you, the disciples standing in his presence at the time. He was telling them that they would see
all of "these things" and thereby know that the end was near.

Part of Weston's inquiry cited certain Old Testament passages that have also been distorted to
give them a forced reference to the restoration of Israel in 1948, so in the next issue, I will
analyze some of these passages to show that they were never intended to mean what die-hard
prophecy-fulfillment buffs have tried to make them mean.

FREE SUBSCRIPTION: For a free subscription to The Skeptical Review, e-mail


jftill@midwest.net or write to P. O. Box 717, Canton, IL 61520-0717.

Volume 1990 - 2002 Issue


Page 1410 of 2049
Skeptical Review Edited by Farrell Till

Who's to Blame? More Hype than


Substance
Roger Hutchinson

Mr. Till proposes that we believe that the impetus behind evil is Christianity. He asks the
question, "If Christianity is not to blame for the activities of radical elements in our society,
then why is it that these elements are almost always associated with organizations that have
Bible-based beliefs" (Who's to Blame? TSR, January/February 1999)?

Actually, Mr. Till asks two questions, purposely slanted, that he joins together knowing that
he can easily confuse simple minded skeptics who tend to emote more than think. If
Christianity is not to blame for the activities of radical elements in our society, then who is?
The question deserves an answer. Why is it that these elements are almost always [according
to Till] associated with organizations that have Bible-based beliefs? Is this the conclusion of a
scientific investigation or the blustering of a disgruntled old man?

Is it true that radical elements are almost always associated with organizations that have
Bible-based beliefs?

When we look at the 20th century, we find that it is the skeptics who have rightfully earned a
reputation for gross atrocities against mankind. As examples, we can readily cite the
handiwork of Stalin in Russia, the communists in China, and the killing fields of Cambodia.
Collectively, those involved in just these three examples rejected the Bible and what it teaches
and accounted for the deaths of some 50 million innocent people. If one is honestly looking
for radical elements with a reputation for atrocities, one need look no further than to those
who reject the Bible--skeptics.

While I have not researched the atrocities committed by skeptics in previous centuries, I
suspect that their accomplishments parallel that which we observe in this century.
Christianity, by contrast, has been a force opposing the spread of a variety of evils devised by
skeptics who constantly denigrate the Bible in order to promote their atrocities. One can
easily see that Christianity has been a positive force in the world. It is difficult to comprehend
the atrocities that skeptics might have committed in the absence of the constraining influence
of Christianity.

Mr. Till imagines a relationship that he wants to exist but cannot prove. He does not provide
any data, other than a few anecdotes, to support his case nor does he attempt a scientific
investigation. Neither do I, but I am not making outlandish accusations. He mostly makes it
up as he goes. He fantasizes on a cause and proclaims fanatically that he must certainly be
right. The mentality of the skeptic seems well suited to the writing of such diatribes.

Volume 1990 - 2002 Issue


Page 1411 of 2049
Skeptical Review Edited by Farrell Till
There are rational explanations opposing the conclusion that Mr. Till puts forth. The most
obvious is that the relationships noted by Till are completely random. Maybe, the real
problem lies in a personality disorder unique to men. Perhaps the teaching of evolution in the
schools with its mantra, Survival of the Fittest, accounts for the Paul Hills of the world. Even
if it appears that something almost always seems to occur, no competent investigator would
jump to the conclusion that a causal relationship exists, without some form of proof. That Mr.
Till jumps to conclusions is warning enough that his pronouncements should not be taken
seriously.

Perhaps the relationship is one of perception and not reality. The President carries a large
Bible when he goes to church knowing that it will be prominently displayed when the
cameras roll. Does this mean that the President is a Bible believer, or does he merely want to
create the impression that he is?

Similarly, if a person wants to do evil, would it not be advantageous for that person to create
the perception that he seeks to do good? The successful con artist will always tell the truth
before he proposes a lie. Consequently, we should not be surprised that people who want to
do evil surround themselves with religious trappings in a clever attempt to disguise their true
plans. Mr. Till likes to tell us that he was once religious (by his definition) as if that should
mean something. Perhaps his religiosity was no more than a cloak to hide his true character
until he got what he wanted or was foiled in the attempt.

The infamous Fred Phelps parades around the country with signs that say God Hates Fags.
Certainly, God hates fags, but God also hates liars, adulterers, and thieves. Why doesn't Fred
Phelps, or anyone else, picket the White House with a sign that says God Hates Adulterers? If
Christianity is what prompts Mr. Phelps to act, then we should expect to see Christianity
being the impetus for other people to respond in a similar vein against liars and thieves and
everything else that the Bible says is evil? I find it suspicious that Mr. Till concludes that
Christianity inspires primarily the likes of the Fred Phelps' of the world.

Maybe Fred Phelps has always personally hated homosexuals and merely uses the Bible to
give an air of legitimacy to his hatred. He would not be the first to do so. Mr. Till would not
be the first skeptic to mistake a casual relationship for a causal relationship. Skeptics can be
heavy on emotion and short on logic.

If we survey the radical elements and the atrocities that they have committed in just the last
100 years, we see that the great majority of those elements share a skepticism of the Bible.
Christianity has not caused these people to act in this manner. Rather, Christianity has been an
important force limiting the damage that skeptics would otherwise have inflicted on their
innocent victims.

If Christianity is not to blame for the activities of radical elements in our society, then what
is? How do we explain the Paul Hills of the world and the Joseph Stalins? Were the killing
fields of Cambodia inspired by Christianity? One man kills several nurses in Chicago and
another climbs a tower in Texas and begins shooting people. Perhaps, such people are nothing
more than skeptics who could not stomach the Biblical injunction to Love Thy Neighbor and

Volume 1990 - 2002 Issue


Page 1412 of 2049
Skeptical Review Edited by Farrell Till
eventually went off the deep end. If so, should we, as Till suggests, prevent Christianity from
proclaiming its message of love?

Recalling the great atrocities of those who, like Mr. Till, proudly profess their skepticism of
what the Bible says, it hardly seems defensible to conclude that Christianity is the cause.
Skeptics of the Bible have proven themselves capable of great evil. Mr. Till may cite himself
as an exception, but that merely proves that his prior Biblical training was not entirely wasted.

So, what causes people to do evil? The Bible offers an explanation. It tells us that the heart is
deceitful above all things, and desperately wicked (Jer. 17:9). People are inherently wicked,
not inherently good. Every parent knows this. We do not have to teach our children to do
wrong, but we must constantly teach them the right thing to do. Further, the Bible says that
the wicked are estranged from the womb: they go astray as soon as they be born, speaking
lies. (Ps. 58:3) In other words, people are natural born skeptics who reject the Bible. The
world desperately needs Christianity and its proclamation of Biblical truth if it is to resist the
evil that skeptics will promote and be able to turn the wicked from their evil ways.

How do we account for the ability of skeptics like Stalin to accomplish such atrocities? The
Bible tells us that the wicked walk on every side, when the vilest men are exalted (Ps. 12:8).
Recent events in the United States offer a vivid example of this. This nation exalted a man,
Bill Clinton, by electing him to the Presidency. He then surrounded himself with men and
women who turned their heads and looked the other way at the evil he committed. So it was
with Stalin. He surrounded himself with people who were just as wicked as himself.

When confronted with the Bill Clintons and Paul Hills of the world, the Bible says of those
who justify the wicked and condemn the just that they are an abomination to the LORD (Prov.
17:15). This is the message of Christianity.

What does Christianity really advocate? Cease from anger, and forsake wrath: fret not thyself
in any wise to do evil (Ps. 37:8). Devise not evil against thy neighbor (Prov. 3:29). The
Biblical injunction and central teaching of Christianity has always been love thy neighbor.
There is no room in Christianity for people who carry a Bible to church on Sunday and
engage in sexual immorality on Monday. There is no room in Christianity for the Paul Hills
and Fred Phelps' who use Christianity as a cover for advancing their personal agendas.

Who is to blame? One thing is certain. It is not Christianity.

(Roger Hutchinson, 11904 Lafayette Drive, Silver Spring, MD 20902; e-mail


RHutchin@AOL.com)

FREE SUBSCRIPTION: For a free subscription to The Skeptical Review, e-mail


jftill@midwest.net or write to P. O. Box 717, Canton, IL 61520-0717.

The Two Faces of the Bible


Volume 1990 - 2002 Issue
Page 1413 of 2049
Skeptical Review Edited by Farrell Till
Farrell Till

After a long silence, Roger Hutchinson has apparently decided to try his luck again at
defending the indefensible. Long time subscribers will immediately recognize his name,
because Mr. Hutchinson has left behind him a long string of failures in trying to defend the
inerrancy of the Bible. These failures are a matter of record and speak for themselves in the
back issues of TSR. On the internet, he has left behind him an even longer string of dismal
failures, where at times his name has been used as a synonym for the silly speculations that
inerrantists resort to in order to "explain" biblical inconsistencies and discrepancies. If he
wishes to return to The Skeptical Review as a defender of the biblicist position, I am glad to
accord him the space to present his case. I am, however, disappointed to see that his ability to
reason rationally has not improved since his last appearance in this forum.

Anyone who will take the time to read in succession my article ( front page, January/February
1999) and his "reply" to it will see that he spent his time fighting straw men rather than
addressing the points that I made. I cited books, chapters, and verses to show that the Bible
teaches that Yahweh chose one nation to be his "own possession above all peoples that are on
the face of the earth" (Dt. 7:6), that Yahweh ordered his "chosen ones" to exterminate
systematically entire nations of people who either occupied a territory that this god
presumably wanted "his people" to have or were considered enemies of his "chosen ones" (Dt.
20:16; Josh. 11:20; 1 Sam. 15:1-3), that Yahweh had commanded "his people" not to "suffer a
witch to live" (Ex. 22:18), and that he had said that homosexuals should be killed (Lev. 20:13;
Rom. 1:27, 32).

I proposed that it is entirely reasonable to think that at least some people who have firm
convictions that the Bible is the "inspired word of God" will read such stuff as this and think
that it is God's will for them to behave accordingly. I have read Hutchinson's article several
times, but the only attempt that I saw him make to address these issues was to suggest that
Fred Phelps, the Baptist preacher who is infamous for his hatred of homosexuals, "has always
personally hated homosexuals and merely uses the Bible to give an air of legitimacy to his
hatred." I suppose that if he were pressed to explain away the Salem witch trials, he would
claim that the Puritan establishment, under whose auspices the trials were conducted, had
always personally hated witches and had merely used the Bible to give an air of legitimacy to
their hatred. He could do the same for the witch hunts and inquisitions in Europe. The leaders
of these persecutions had always personally hated witches and heretics and had merely quoted
the Bible and used the authority of the church just to give an air of legitimacy to their hatred.
In other words, we see that Hutchinson's tactics haven't changed. Just as he always seemed to
know exactly what Bible writers really meant to say in texts that have been cited as examples
of biblical discrepancy, so now he claims to know what the personal beliefs and intentions of
people like Fred Phelps really are. They aren't actually sincere Christians; they are phonies
who are parading about as Christians in order to give a semblance of "legitimacy" to their
hatred. Those who spend much time on the internet will recognize this as just another
variation of the not-real-Christians "explanation" of persecutions and atrocities committed in
the past by church officials. "Well, those people weren't real Christians," modern defenders of
the faith will argue. If, however, they can find an atrocity that was committed by leaders who

Volume 1990 - 2002 Issue


Page 1414 of 2049
Skeptical Review Edited by Farrell Till
at least seemed to be atheists, they have no problem considering these people to be "real
atheists," whose atheistic beliefs were what motivated them to commit atrocities.

Hutchinson went so far as to label those responsible for "the killing fields of Cambodia" as
"skeptics who could not stomach the Biblical injunction to love thy neighbor and eventually
went off the deep end." Several times he referred to such world leaders as "skeptics of the
Bible," as if we are supposed to believe that leaders in far eastern countries like Cambodia
and China, where infamous atrocities have been committed, had studied the Bible sufficiently
to know what it teaches and had rejected it. The fact is that Hutchinson cannot establish that
any of these leaders were "skeptics of the Bible" or that they even knew enough about the
Bible to know whether it was a believable book or not. I cannot be called a skeptic of the
Zoroastrian Avesta, because I simply don't know enough about it to describe myself as one
who is a skeptic of this book, and the same is true of other holy books. A more probable
explanation of the atrocities committed in China and Cambodia is that the leaders of these
nations were politicians who exterminated all potential opposition in order to secure their
dictatorial positions, and religious beliefs had nothing to do with it.

The same can be said of Joseph Stalin, who was another of Hutchinson's examples of those
whose "skepticism of the Bible" led them to commit atrocities. I'd personally like to know
how that Hutchinson knows that Stalin was motivated by biblical skepticism to commit
atrocities. The truth is that Stalin studied for the priesthood and was educated in a seminary,
so if Hutchinson has any evidence that Stalin turned to a kind of biblical skepticism that
motivated him to commit the atrocities attributed to him, then he should present it rather than
just speculate about a dictator's motivation for eradicating his opposition. Otherwise, it is
more reasonable to believe that Stalin was politically motivated to exterminate all those
whom he perceived as threats to the security of his political hold on the Soviet Union and that
his personal beliefs about the Bible had nothing to do with it. Indeed, if one were truly a
"Bible skeptic," he would probably be motivated to refrain from genocide and mass
executions, because tales of such deeds (presumably commanded by "God") fill the pages of
the Old Testament, and this aspect of the Bible has probably produced more biblical skeptics
than any other single factor. How reasonable is it, then, to think that a "real" Bible skeptic
would want to imitate something in the Bible that he had found to be morally repugnant?

Hutchinson's argument seems to be this: The leaders of China, Cambodia, and the Soviet
Union committed various atrocities. These leaders showed no indication of believing in the
Bible. Therefore, their atrocities were motivated by their lack of belief in the Bible. By the
same line of reasoning, I could prove that Hitler's atrocities were motivated by his belief in
Christianity. Hitler committed various atrocities, especially against European Jews. The
writings of Hitler made frequent references to his belief in God and Christianity. Therefore,
Hitler's atrocities were motivated by his belief in God and Christianity. If Hutchinson's
reasoning is sound, then so is mine. If not, why not? If Hutchinson doubts that Hitler held
Christian beliefs, I will gladly send him a long list of quotations from Hitler's speeches and
writings that show that he did.

In all probability, Hitler's atrocities had little to do with his religious beliefs, just as the
Cambodian, Chinese, and Soviet atrocities had little to do with the religious views of the
leaders of these nations. It is far more likely that they were all driven by political motivation

Volume 1990 - 2002 Issue


Page 1415 of 2049
Skeptical Review Edited by Farrell Till
and were willing to do whatever was necessary to secure their dictatorial holds over their
respective nations.

Political purges are quite different from inquisitions and religious persecutions. In the former,
the ones in power seek to preserve their political bases by removing their opposition; in the
latter, a religious establishment seeks to remove what is considered a threat to the "true faith"
or even to "save the souls" of those who have heretical views. Second Chronicles 21:4
claimed that when Jehoram of Judah succeeded his father Jehoshaphat as king, he killed all of
his brothers. We would hardly think that Jehoram's belief in Yahweh was the motivation
behind this atrocity but that it was instead a purely political act. He wanted to secure his
throne by eliminating the most likely threats to his reign. Second Kings 9-10 relates the story
of Jehu's massacre of the royal family of Israel, after which he became king of the Northern
Kingdom. As the story is related, Jehu committed the massacre in obedience to a divine
decree, but if any such incident as this actually happened, it is far more likely that it was
motivated by political ambition rather than divine injunction. Jehu may have been a believer
in Yahweh, who alleged divine decree in order to justify his actions, but it wasn't very likely
that his faith motivated him. He just wanted to be king like so many other tyrants who have
seized power by bloody force.

The apparent motivation behind inquisitions and persecutions was radically different, a basic
fact that Hutchinson could learn by consulting just a general encyclopedia. The "inquisition"
was a religious judicial institution that was established by papal decree in the middle ages. Its
purpose was to seek out, try, and sentence persons guilty of heresy. At first, the church didn't
approve of "coercion and physical penalties," but this opinion changed in the 12th century
with the rise of Albigensianism, which was viewed as threatening to matrimony and other
church-approved social institutions. Pope Innocent III began an organized crusade to wipe out
this heresy, and part of his effort was the enactment of punitive legislation. The atrocities that
are most often associated with the Inquisition began in 1231 when Pope Gregory IX issued
his Excommunicamus, which transferred much of the responsibility for "orthodoxy" from the
bishops to "inquisitors," who had been appointed with special papal powers that included
severe penalties for heresy. To summarize the general history of the Inquisition would be a
pointless waste of publishing space, since its history is well established in reference books
that Hutchinson could consult if he doubts its affiliation with the Christian establishment of
the eras in which it thrived. The inquisitors could and did prosecute any person they suspected
of heresy, and they elicited confessions by offering less severe punishments to those who
would confess voluntarily. Those who didn't were tried before clerical courts. They were
required under oath to answer all charges filed against them, and the testimony of two
witnesses was usually considered proof of their guilt. Those who were even suspected of
lying were imprisoned, and in 1252 Pope Innocent IV officially approved the use of torture to
extract confessions from the accused. Those who confessed were sometimes publicly
scourged and forced to submit to other public humiliations, such as carrying crosses and
wearing "tongues" of red cloth on their garments to signal that they had been found guilty of
lying. The worst penalty that could be imposed at this time was life imprisonment, but when
the church wanted a more severe punishment, the "guilty" were turned over to civil
authorities, who understood this transfer as a demand for execution. During the Spanish
Inquisition, however, the inquisitors were empowered by decree of King Ferdinand and
Queen Isabella, with papal approval, to impose the death penalty on the "guilty." Tomas de

Volume 1990 - 2002 Issue


Page 1416 of 2049
Skeptical Review Edited by Farrell Till
Torquemanda, the most notorious of the grand inquisitors, executed thousands of heretics.
The "grand inquisitor" was even given jurisdiction over the local tribunals in Spanish colonies
in Mexico and Peru. Holy Roman Emperor Charles V introduced the Inquisition into the
Netherlands in 1522 in a failed attempt to stamp out Protestantism. Funk and Wagnalls
Encyclopedia, which I accessed on the internet to gather most of this information, noted at the
end of its article on the Inquisition that "many Protestant lands had institutions as repressive
as the Spanish Inquisition, such as the consistory in Geneva at the time of the French reformer
John Calvin," but space won't allow me to discuss the heinous crimes of Protestantism.

That Hutchinson would even try to compare the acts of recent far eastern tyrants, who as far
as we know had no opinions about the Bible, to actions like the inquisitions and other
persecutions that were obviously religiously motivated is an act of desperation on his part.
The record is clear that the Puritan establishment of colonial New England persecuted and
executed Quakers and other dissidents because they were considered heretics. The modern
record is clear that preachers such as Fred Phelps publicly denounce homosexuality from their
pulpits and in some cases take their intolerance to the streets, waving signs that quote the
Bible in defense of their actions. The record is clear that Christian fundamentalists have gone
so far as to found the Christian Reconstruction movement, which calls for a national
government established on "biblical principles" that will provide the death penalty by stoning
(no less) for crimes of heresy, adultery, disobedience to parents, and all of the other offenses
that called for death under the Mosaic law.

But where are the public records of any organization of skeptics, agnostics, atheists, or
humanists who advocate such intolerance toward those whose beliefs are different from
theirs? Has anyone ever heard of an atheist organization that advocates the bombing or
burning of churches? Has anyone ever heard of any humanist organization that publicly
proclaims racial supremacy or hatred of homosexuals? Has anyone ever heard of an
organization of skeptics that advocates violence toward leaders of the Christian Coalition?
Has anyone ever heard of an organization of skeptics or atheists or humanists that advocates
bombing the offices of "pro-life" groups. If Hutchinson knows of any such movements made
up of skeptics, atheists, or humanists, why doesn't he call them to our attention?

The truth is that religion seems to foster intolerance. That appears to be true of other religions
as well as Christianity. The more fundamentalist a person is in his religious beliefs, the more
likely he/she is to be intolerant of those who disagree, and the less religious one is, the more
tolerant he/she is likely to be. Hutchinson made a great issue over Christianity's "message of
love," and it is undeniably true that some very lofty moral and social principles are taught in
the Bible. I would never deny that, but for every scripture that Hutchinson can quote that
teaches such ethics as "love thy neighbor," I can quote one that teaches hatred and intolerance.
This fact about the Bible is what makes so absurd the often-heard inerrantist claim that the
Bible is perfectly harmonious in its themes from cover to cover. That just isn't so. The Bible
has two faces. It teaches love, but it also teaches intolerance and hate. It's such a pity that so
many Christians neglect biblical teachings about love and practice what the other face of the
Bible teaches.

FREE SUBSCRIPTION: For a free subscription to The Skeptical Review, e-mail


jftill@midwest.net or write to P. O. Box 717, Canton, IL 61520-0717.

Volume 1990 - 2002 Issue


Page 1417 of 2049
Skeptical Review Edited by Farrell Till

Ezekiel's Prophecy Against Egypt


Dave Matson

The biblicist reassures us that Ezekiel's prophecy for Egypt (and certain other prophecies of
the Bible) will be fulfilled some time in the future. If necessary, a new pharaoh will arise, a
new Nebuchadnezzar will come into existence, and the first will be devastated by the second!
Is this sound reasoning?

How do we know that such things will happen? Doesn't it all boil down to the presumption
that the prophecy in question is true? Or, is it a case of claiming that something might happen
just because there is still time on the clock?

In the former case, why does the biblicist even bother with reason? Reasoning is like a long
anchor chain. All the links have to be there or else you lose the anchor! If you choose reason,
dear Bible-believer, then it's reason all the way. That is the only way that you can collect the
benefits of reason. If you are going to simply assume you are right at some difficult point,
then do so at the start. Stick your head in the sand, declare all biblical prophecy to be correct,
and let that be the end of the matter!

If we are dealing with the latter case, and if the Bible-believer has any interest in sound
reasoning, then he must understand that "might" is not good enough. Giant, green spiders
might be secretly ruling the city of San Diego! However, the intelligent mind does not stay up
at night worrying about such possibilities. The probability or likelihood of such a claim is
sufficiently low that we simply reject it. We might be wrong, but very likely we have made
the right decision.

We must weigh the evidence pro and con, and only then decide what the best conclusion is.
To accept the claim that a prophecy will be fulfilled in the future means this: The case for it
not happening is less likely than the case for it happening. Loopholes, as such, do not concern
us. The possibility that a prophecy might come true is not a valid defense; it must be weighed
against the possibility that it might not come true. To deny the latter by fiat, without recourse
to the evidence, is just another way to assume the truth of one's claim. You may not shut the
door to the possibility of prophecy failure without recourse to the evidence.

Anyone who defends a prophecy by insisting that it might come true, and there is always a
"might" in there somewhere, is really just assuming that it cannot fail. If reason is to prevail,
then we must weight the evidence and draw a less than certain conclusion. That is, we must
select the best case.

It may be urged that a good prophecy, by its very nature, must involve a certain degree of
unexpectedness. Thus, our procedure would reject every interesting prophecy. Fair enough.

Volume 1990 - 2002 Issue


Page 1418 of 2049
Skeptical Review Edited by Farrell Till
Instead of asking whether the prophecy will probably come true, let us ask whether the basic
conditions for that prophecy will probably exist.

Suppose that a certain prophecy states that someone will take a pot shot at a certain politician.
The politician is publicly available and guns are plentiful, as are nut cases. The prediction has
a low probability of being right, but the props are all in place. That unexpected twist of fate
could happen. Thus, we would not reject the prophecy as being false. It is reasonably viable
as far as prophecies go.

Compare Ezekiel's prophecy regarding Egypt. Millennia have passed. Egypt is no longer a
superpower ruled by pharaohs. It is a modern, political state; even its people are no longer
quite the same. Nebuchadnezzar is ancient history. Israel is not the same country; its people
are quite different. European emigrants have erected a modern state there. Religious freedom,
to some extent, is enjoyed.

Are we to believe, after yet more time has passed, that God will suddenly create another
Babylonia, which will launch its armies against Egypt and make it desolate?

Ridiculous!

The historical setting for that prophecy, the very props and stage that gives the play meaning,
has become obsolete. Aside from making God look like an idiot and a moral imbecile, the
fulfillment of such a prophecy would require another pharaoh and another Nebuchadnezzar.
The latter must attack the former without upsetting NATO or the Modern Israeli state, the
latter occupying the only practical route of attack. An attack on Israel, of course, would bring
in the United States. The whole scenario is highly improbable.

Our best conclusion is that even the basic conditions for this prophecy will fail to materialize.
Therefore, we are justified in calling it a false prophecy.

Some may object to such a conclusion, but their objection can only rest on that "might" we
discussed earlier. Uncertainty is unavoidable. The simple act of determining whether a
prophecy has succeeded or failed necessarily invokes some uncertainty. Absolute certainty is
lost at many different levels, practical and theoretical. Our conclusion about Ezekiel's
prophecy of Egypt is none the worse because we have not waited until the end of time. The
case for a successful prophecy, even given the unexpected nature of prophecy, has been ruled
out as our best conclusion, and that's all that matters.

The illusion that a prophecy cannot be judged until its allotted time has run its course is based
on another illusion-- that we can have absolute certainty. Absolute certainty says that we
cannot rule out a prophecy that's "still running," because it might come true. However, we
actually deal with probabilities in real life. An open-ended prophecy requiring a highly
improbable future to even set the stage is no better than a completed prophecy whose failure
has been judged highly probable. Both cases may be counted as "failed prophecy." The
former will likely fail; the latter will likely have been correctly rejected. We could be wrong
in either case, but if we have weighed both sides and found the prophecy highly improbable,
then we are justified in calling it a false prophecy. We just have to live with that bit of

Volume 1990 - 2002 Issue


Page 1419 of 2049
Skeptical Review Edited by Farrell Till
uncertainty, even as we must live with the notion that giant, green spiders might be ruling San
Diego. The burden is usually more imaginary than real.

(Dave Matson, editor, The Oak Hill Free Press, P.O. 61274, Pasadena, CA 91116; e-mail,
103514.3640@compuserve.com)

FREE SUBSCRIPTION: For a free subscription to The Skeptical Review, e-mail


jftill@midwest.net or write to P. O. Box 717, Canton, IL 61520-0717.

From the Mailbag

Fundamentalism Down Under...

As we move towards the year 2000, we are confronted almost daily with the onset of the "silly
season," where innumerable nutcases await the parousia. For this reason, also because of its
high quality, I would like to subscribe to The Skeptical Review for another year.

Religious fundamentalism does not necessarily have to be an overt danger. Hutterites and
Amish live quite peacefully and, for want of a better word, "fundamentally" without raising
much fuss. What I, and no doubt most of your readers, have a problem with are groups like
the "Concerned Christians" from Denver. Eight members of this group were arrested earlier
this year in Jerusalem. Their aim was to carry out extreme acts of violence in Jerusalem at the
end of the year, starting by provoking a shootout with Israeli police. Why? To hasten Christ's
return.

What is most irritating about groups like the "Concerned Christians" is not just the fact that
they are criminally insane but the fact that the grounds for their insanity is socially accepted.
They quote ad nauseam from the various books in the Bible, chopping and changing as if it
were a coherent whole. Instead, as we all know, there were many writers competing with one
another over many centuries. What percentage of the general population knows of the
Yahwist, Elohist, Priest, Deuteronomist, and Redactor who wrote the Hebrew scriptures?
How many know of the politics between Israel and Judah and which books were written by
the priests of which state? I am lucky in that I know someone with a doctorate in religious
studies, who gave a course in Christianity where I learned of the above facts, but for others it
is far more difficult to find good scholarship about the Bible. Bookstores are far more likely
to stock big-selling garbage like The Bible Code or Jesus the Man than books by Richard
Friedman or R. J. Hoffman.

What we all need to do, though, is not just read The Skeptical Review and books by reputable
scholars but write to newspapers whenever a religious debate is taking placed. When James
Veitch (Associate Professor of Religious Studies at Victoria University) challenged orthodox
scholarship about Jesus to show that it was more politics than truth intended (in his own

Volume 1990 - 2002 Issue


Page 1420 of 2049
Skeptical Review Edited by Farrell Till
words concerning the gospel of Luke) to say, "Our Jesus is better than your emperor," he was
rounded upon with claims that he is "absolutist and derisory," that he is writing 2,000 years
after Jesus while the gospels were written only 30 years afterwards, so the gospels are right,
that "skeptics come and go but he [Jesus] goes on for ever," and that they presently "grapple
with his living Spirit but they do not face him in person--yet." When a scholar is threatened
with eternal damnation for writing about a subject he loves, it is up to as many of us as
possible to defend him. It is one thing to have difficulty understanding subjects like quantum
physics--to do so requires formulae and coping with extremely abstract concepts--but having
difficulty understanding hermeneutics is quite inexcusable. The only prerequisites for good
scholarship are an open mind and literacy. If we all put this message forward in public as
often as possible, then hopefully the next "magic year" of 2033 will be less maddening for all
of us.

(Hayden Wood, 14 Mays Street, Devonport, Auckland, New Zealand)

EDITOR'S NOTE: The final paragraph of Mr. Wood's letter alludes to some newspaper
clippings that he enclosed with his letter. An associate professor of religious studies published
an article in which he discussed the mythological roots of the "nativity" stories in the New
Testament. The result was a flurry of letters expressing outrage at his audacity to question the
historicity of the New Testament accounts of Jesus's birth. The usual fundamentalist
comments were recycled: the apostles were so convinced of what they had seen and heard
that they went everywhere preaching it and were eventually killed for their faith, the
reliability of the gospel records has been confirmed, Sir William Ramsey [incorrectly
identified as Ransom in the letter] was a noted archaeologist who considered Luke "a
historian of the first rank," etc. All of these "apologetic" claims have been refuted over and
over, but gullible Christians still recycle them. Apparently the state of Christian
fundamentalism on the other side of the world is pretty much the same as it is here.

What Maccabean Jews Knew...

I read Hatcher's article and encountered this quotation from Stephen Miller in Daniel: The
New American Commentary: "To suggest that any semi-educated Jew of the Maccabean
period could be ignorant of the fact that it was Cyrus the Persian who conquered the great
Babylonian Empire and allowed the Jewish captives to return to their homeland is not
reasonable."

I do not have an exact parallel to draw on, but I can come close. First of all, keep in mind
there was a period of 375 years separating the time of Cyrus from the writing of Daniel (if the
critics are right). Now, just a little over 200 years ago we had our first president and just a
little before that King George III granted the thirteen colonies their independence. Our
country is far more literate than that of 2nd-century B. C. Judea, and we have far easier access
to information. But how many well educated Americans know the name of the king who
granted the thirteen colonies their independence? How many even know that George
Washington was our first president? This came to mind because a few years ago there was a
survey of supposedly well educated Americans that showed some could not even name the
first president! I think one in five got it wrong. Some said Thomas Jefferson, a few even said
Lincoln and others said Ben Franklin! According to this same survey over 20 percent did not

Volume 1990 - 2002 Issue


Page 1421 of 2049
Skeptical Review Edited by Farrell Till
know the name of the king or the country from which we gained our independence. Some said
France and others said Spain. All of the respondents were supposedly well educated people! I
am looking for the journal or article that contains this survey. I think I still have it stashed
somewhere. I will try to find this and send it to you. Won't Hatcher be surprised!

On page three, column three, he asked: "How could the Qumran community accept Daniel as
Scripture if it incorrectly pictured Darius Hystaspis preceding Cyrus?" My comment on this
is to ask him why so many Catholics accept the Apocrypha with its many inconsistencies,
which even fundamentalists acknowledge!

On page four, column 2, midway down, he quoted Dressler: "Is it conceivable that the same
prophet would choose a Phoenician-Canaanite devotee of Baal as his outstanding example of
righteousness? Within the context of Ezekiel this seems to be a preposterous suggestion."
Well, this may not be an exact parallel, but I have heard Christian ministers sometimes use
Muhammad, Gandhi, or even Buddha as good examples to follow. It depends on the context,
of course, and I am without a context to refer you to. Usually, they do it as a comparison to
Christ or as a witnessing tool to open a door with a convert of that particular religion.

On page five, column one, Hatcher referred to an Assyrian inscription that refers to Jehu as a
"son of Omri." He seems to claim this sets a precedent for Daniel's use of "father" in his book,
but this proves at best only an Assyrian custom, not a Jewish custom, and at worst it proves
only the Assyrian inscriber made a mistake. He fails to establish with certainty his case that
"in the near East the word son could also mean successor," unless, of course, there are more
examples from that era.

I am aware that Persia absorbed Media (page eight, column two, bottom) but would that make
it the "Medo-Persian Empire"? I see that Babylon absorbed Assyria, yet it isn't referred to as
the "Assyro-Babylonian Empire." How does one distinguish them as separate? Did Media
have a separate government though ruled by Persia? Was she one of her satraps?

I will try to find that survey for you!

(David Mooney, 1203 Mooney Way SW, Supply, NC 28462; e-mail, atheist89@hotmail.com)

EDITOR'S NOTE: In my second reply to Hatcher's article (pp. 2-5, this issue), I made some of
the same observations that Mooney did about Hatcher's attempts to defend the claim that
Daniel was written in the 6th century B. C. I'm glad that others can see through the illogical
arguments that Hatcher is using to defend his position, or rather I should say the illogical
arguments that Hatcher is parroting from fundamentalist books.

Rapid Acceptance of "Scriptures"...

One argument often made by inerrantists to defend the dating of the book of Daniel in the 6th
century BC is based on the evidence that this book was considered by many Jews as scripture
during the late 2nd century B. C., barely 50 years after the date which many higher critics
assign as its date of actual writing (ca. 165 B. C.). By the time of Jesus, ca. 170 years after its
writing, there is little doubt that most Jews accepted its prophetic and scriptural nature. The

Volume 1990 - 2002 Issue


Page 1422 of 2049
Skeptical Review Edited by Farrell Till
inerrantist argument asserts that it is extremely unlikely that a book could be accepted as
scripture so soon after its first appearance; it must therefore be much older.

That argument loses its force when one can point to a very similar phenomenon much closer
in time to us: The Book of Mormon, which appeared in 1830, claiming to have been sealed up
by divine command to appear in the last days, claiming to be divinely inspired, claiming to be
a history of ancient peoples, claiming to have been written centuries before by prophets of
God, and prophesying accurately of the very events that were occurring at the time of its
appearance. All very much like the Book of Daniel.

Like the Book of Daniel, the Book of Mormon was quickly accepted by many as the Word of
God. Within 50 years tens of thousands had read it and believed it and become Mormons, and
had established a thriving society in Utah. Today, 170 years later, there are over ten million
Mormons, all accepting the Book of Mormon as scripture, as written in ancient times by
prophets, in spite of overwhelming evidence that the alleged history contained in it is all
wrong, just like Daniel's Babylonian history.

So, if ten million Mormons can accept the Book of Mormon as scripture and history after 170
years, why is it so impossible that ten million Jews at the time of Jesus accepted the Book of
Daniel?

I think of the Book of Daniel as the "Book of Mormon" of its day.

(Richard Packham, 2145 Melton Road, Roseburg, OR 97470; e-mail,


packham@teleport.com)

EDITOR'S NOTE: I addressed this same issue in my latest reply to Hatcher, who has
parroted the claim of fundamentalist commentators that "scriptures" could not have been
accepted as scriptures within the short space of only 50 or so years, but how can anyone
know that? As I asked in my reply to Hatcher, how long did it take the ancient Jews to accept
the writings of Isaiah, Jeremiah, Ezra, and others as "scripture"? I dare say that neither
Hatcher nor any of his fundamentalist sources know. My goodness, if the omnimax Yahweh
inspired a person to write a book, I would think that such a god could arrange for this book to
be recognized immediately as "scripture." If not, why not?

Why Hatcher Parrots Other Biblicists...

I just got the latest edition of The Skeptical Review yesterday, and I would like to thank you
for renewing my subscription while I'm in high school. But I must say, that my favorite part is
not the articles (although those are good too). The best part of the magazine is the "From the
Mailbag" section, because in every issue there are several rebuttals to the myth that life is
meaningless without Christianity by people who have left the religion and now live happier
and more meaningful lives.

I would like to say that I was puzzled at your frustration with Everette Hatcher's method of
parroting people he agrees with and taking up so many pages with worthless citations. Maybe
you've been out of the lifestyle for so long that you've forgotten why inerrantists do this. They

Volume 1990 - 2002 Issue


Page 1423 of 2049
Skeptical Review Edited by Farrell Till
do it so that they can say to their readers, "Well, I've done a lot of research, and I still believe
what I believe, so you can too!" Of course, Hatcher knows that readers ready to lap up every
justification for their religion they can find will not bother to research the cited information.

People always wonder why I do not read much inerrantist literature and read a lot of skeptical
literature. Well, here's why: it's irritating to see people like Josh McDowell and D. James
Kennedy constantly recycle the same logical fallacies. I want to read something that actually
argues a point logically. Every time an inerrantist says, "Read this, it's good," I find the same
thing: "What if" scenarios, appeals to authority, or pure speculation. It's like a frustrated
parent who is arguing with a child about something and the child keeps recycling the same
arguments like "everybody else is doing it" or "why?" Sooner or later, the parent says: "End
of discussion!" Well, I've said to inerrantist literature, "End of discussion!" until they can
come up with better apologetic methods. Hatcher confirms my decision even more.

(Brian Rainey, 313 Tudor Place, Chesapeake, VA 23325; e-mail,


BRAINEY@norfacad.pvt.k12.va.us)

EDITOR'S NOTE: It's refreshing to learn that even a high school student can see through the
kinds of arguments that Hatcher has been subjecting us to. I think that Rainey is probably
right about Hatcher's motivation in spending so much time quoting and citing references. On
page 3 (March/April 1999), he devoted 16 lines to nothing but the listing of references that
agree that "Daniel mistakenly had Babylon falling to a Median Empire." Why else would he
have done this except to say between the lines, "Look at how many scholars I have read who
disagree with me, and I still believe that Daniel knew that Babylon fell to the Persians"? At
any rate, regardless of what his motivation may be, if he wants me to continue publishing his
articles, he is going to have to change his style and begin presenting arguments and
supporting them rather than just quoting authors. We have seen from him enough appeals to
dubious authority to last a lifetime.

Another Inerrantist Sees the Light...

I just received my first issue of TSR. Thank you for such an immediate response to my
request! I am excited about the journey I have now embarked upon. It is very refreshing and
encouraging to find others along the way who are not afraid to question their beliefs or those
of the so called majority. I have recently made the decision, because of the facts, to leave my
"God" thoughts behind. After over twenty-six years of accepting the so called "truth," I finally
found the courage to take the path less traveled, and believe me, it has made all the difference!
I have spent the last ten years in an ultra-fundamentalist church. I have been to the mission
field and served functions as an assistant minister, but four years ago after my son was born,
like a splash of cold water on my face, I awoke from my daze. I discovered that those who say
they love and support me don't really care. Their focus is on those who are "lost" rather than
those who are "saved." Anyhow, it was at this point I began to question the things I have been
taught and that I have taught others. My friends were not ready and definitely not supportive
of my questioning. I found that you can search for the truth within limits, but when I began
questioning what was behind the door they began accusing me of dissension and divisiveness.
I made the decision to move my family to a different state where the "church" was more
mature.

Volume 1990 - 2002 Issue


Page 1424 of 2049
Skeptical Review Edited by Farrell Till
It was challenging at first getting settled in, but once I did, I realized that though I had moved,
the thinking was the same, or should I say, the lack of it? This past November, I left the
church, although my wife continued to attend, and see no problem. I attended a few other
local churches and even began a house church, but the politics and those who think they know
all the answers to my questions were present. I began reading atheist literature and books, and
found what I have been searching for-- the freedom to question without having the feeling
that I'm a traitor or a vessel for satan.

Through all of this, my wife continues to attend the church. What has helped us in our
marriage and in raising our four-year-old son, is the openness with one another in discussing
what we are thinking. I do not make her do this or that. I want her to come to her own
conclusions. I want her to follow her own thoughts and convictions and not mine. I share with
her what I am learning and she reads parts of the books I am reading. I want her respect and
confidence that I continue to seek and have not given up. I express to her that I am more
happy now than I have ever been, and that I see my life progressing not regressing. I have
chosen a path less traveled and she is supportive, though at this point she does not agree with
what I am learning. There have been times I have wanted to ridicule or share my opinion
about religion and about what she is doing in the church, but I have decided to hold back my
comments. I believe that the time will come when she too will come to the same conclusions I
have in the past few months, but I want it to be her decision and her journey, not my own. I
challenge her to continue to question and seek and not to settle with what other men or
women tell her is the "truth." We have kept the lines of communication open, and it has
helped to make the transition not smooth but worth it!

There are those who will step out and walk while others watch and pass judgment. In the
military it is termed a review of the troops. Those who wear the tassels and medals do not
realize they are not in control. They watch not realizing they too are being watched and that
their so called majority and control are being challenged and reviewed. The Skeptical Review!

I look forward to future issues. Thanks for the support for those on the path to freedom!

(Brian Dube, 3300 Willow Crescent Drive, Apt. 11, Fairfax, VA 22030; e-mail,
bdube@nortelnetworks.com)

EDITOR'S NOTE: I have to think that traditional Christianity is in serious trouble. I receive
too many communications like this to believe otherwise. As I think back, I remember a phone
call from a student at my alma mater in his final year of study, who had heard about me and
wanted advice. He told me that his home congregation had paid his college expenses to
educate him for the ministry, but he had gradually come to realize that what he had been
taught to believe was not the truth. He was going through the usual psychological trauma of
trying to decide what to do. He didn't want to betray those who had trusted him, but he
couldn't bear the thought of trying to begin a profession that he knew was based on falsehood.
My advice to him was that, as psychologically difficult as it would be, he should make the
break now, because it was unlikely that he would ever again return to his former beliefs. I
knew exactly what mental turmoil he was going through, so my sympathy was with him.

Volume 1990 - 2002 Issue


Page 1425 of 2049
Skeptical Review Edited by Farrell Till
Just recently, I received a phone call from a Church-of-Christ missionary who is back in the
states, trying to decide what to do. He is scheduled to return to the mission field, but he knows
that he doesn't believe the things he will have to preach if he goes back. I have even received
phone calls from foreign countries, most of them from people who just can't cope any longer
with having to pretend that they believe what they know they don't believe. Most of these calls
and letters have come from preachers and members of the Church of Christ, but if that kind of
transition is taking place in a church as fundamentalist as this one, it must be happening at an
even faster pace in other churches, or perhaps it is the very absurdity of fundamentalist
Christianity that is waking these people up to reality. At any rate, I really believe that changes
are occurring that will radically alter the face of traditional Christianity in the next century.
Too much information about the other side of Christianity can now be accessed with just a
few keyboard strokes. This is bound to bring changes.

Jonah and the Whale...

The whale-Jonah debate raises a question: Is it physically possible for a large whale to
swallow a man whole? If so, is it possible for such a victim to remain in a whale's interior for
a limited length of time and emerge relatively unscathed? If the answer to the first question is
in the affirmative, then the stories of a whale swallowing a man must be given some credence.

In the '30s or '40s, the Family Herald and Weekly Star, a national agricultural journal, ran a
contemporary item about a man who was swallowed by a whale. He was a member of a
whaling fleet, and the whale was killed a short time later and the man released. Apparently, he
was in good condition except that his skin had turned paper-white from the creature's
digestive juices. This was a story of one or two paragraphs and was published as news. The
animal concerned was a whale, not a shark.

Obviously, such occurrences are rare, but the possibility that they do occur should not be
dismissed out of hand as a fabrication. Perhaps your diligent researcher would have more luck
researching an incident nearer our own time. If my memory is correct, it took place off the
coast of Africa.

(R. S. Craggs, 25 McMillan Avenue, West Hill, ON, Canada M1E 4B4)

EDITOR'S NOTE: Mr. Craggs' whale story sounds suspiciously like the one that Harry
Rimmer and others popularized in the mid-thirties. If Craggs will read my article again( "A
Legend in His Own Time," TSR, January/February 1999 , pp. 2-5, 11) and notice the details
more carefully this time, he will see that Rimmer popularized his modern-day Jonah story in
Harmony of Science and Scripture, which was published and copyrighted in 1936. Craggs
vaguely recalls that the modern-day Jonah story he read was published in the 30s or 40s. Are
the dates just coincidental? Furthermore, my analysis of Dr. Edward Davis's article in
Perspectives on Science & Christian Faith, which debunked Rimmer's fish story, pointed out
that Davis's research had uncovered several versions of the tale that had been published in
the 30s, the very time that Craggs remembers reading a similar story in an agricultural
journal. As my article showed, Davis eventually traced the story to an 1890 voyage of the Star
of the East, which had sailed through the southern Atlantic (between Africa and South
America), but it was not a whaling ship, and its log had reported no such incident. In all

Volume 1990 - 2002 Issue


Page 1426 of 2049
Skeptical Review Edited by Farrell Till
likelihood, Craggs remembers reading one of the many versions of this tale that was
circulated in the 1930s, but, regardless, we can hardly take seriously a story "of one or two
paragraphs" that he vaguely remembers reading in an obscure journal over six decades ago.
To "research" this incident, as he suggested, one would have to have much more information
than this.

Another View from Down Under...

I enjoy TSR enormously and have been more and more interested in your thoughtful articles
pointing out the impossibilities in the Bible. May I make one small contribution to the debate
as to why so many people believe in organized religions and in their texts such as the bible. It
is all to do with the satisfaction of intellectual security. It is nice to believe in God and in his
churches and bible because they explain fundamental questions so apparently clearly. It is
only when one starts to look at the explanations in detail that they fall apart so rapidly. I have
always strongly suspected that many, many people have started to look at the Bible critically
but have backed away from its incredibilities because they were disturbed by them and
therefore were much happier by backing off and not addressing them as intellectual problems.

I draw an analogy to the rearing of children. Almost everything that is written about
childrearing is claptrap because it is all so very simple. Children need security and they get
this in families of any size by love (I care for you. I will be secure for you) and by a clearly
established and generally fixed set of boundaries. Much of children's behavior is asking us to
set the boundaries for them. Once this is done they are content and happy because they have
the security of knowing where they fit in.

The same is surely true of many of those who believe the Bible and the word of
churchpersons. These set boundaries ("If you do this, you will go to heaven" or "if you do this
you will go to hell.") Like most of those who join military organizations, this is a wonderful
relief." Someone else is going to look after me. Therefore I will believe whatever they say to
believe and I will be secure not only in my life but in what they tell me will happen after
death." Thus these persons, when faced with questions about their beliefs, revert, like
children, to the concept that "it is so because my mother or father, told me it is so." While this
is quite understandable for children, it is inexcusable for those who consider themselves to be
adult.

I am thus suggesting that many attitudes and beliefs of the inerrantists are due to the security
they feel within their beliefs. Any questioning of them is not an intellectual challenge as much
as a threat against their security blankets... and this is how they respond to such questions.

Keep up your excellent work. I look forward eagerly to reading TSR.

(Richard Sadleir, 120 Sweetacres Drive, Belmont Lower Hutt, New Zealand 6009; e-mail,
rsadleir@paradise.net.nz)

EDITOR'S NOTE: I too have found that Christian fundamentalists are generally people of a
mindset that wants simple, black-or-white answers to questions, and so they are willing to
sacrifice their intellectual integrity for the sense of security that they derive from thinking that

Volume 1990 - 2002 Issue


Page 1427 of 2049
Skeptical Review Edited by Farrell Till
they know all of the answers to life's mysteries. I can remember that when I was a child, I
always wanted to know why. I'm sure this attitude was what doomed me to fail as a Christian
fundamentalist.

I hope that Mr. Sadleir, as well as Hayden Wood, whose letter appeared earlier in this issue,
won't object to my Americanization of their spelling.

Who Needs the Devil?

Bruce Weston's article "Doubts But Questions about Prophecy" in the January/February TSR
brought back memories of my own evolutionary journey from religious belief to the delicious
freedom of atheism. Many experiences along the way are stepping stones to rationality. Mr.
Weston's belief that he could never "just write off God completely and be an atheist like you"
is certainly familiar to me.

In my case, I was "born in a Baptist cradle and spanked with a Baptist paddle." I went to a
Baptist college, married a Baptist preacher, and gave birth to four p. k.'s (preacher's kids). I
was a Sunday school teacher, church pianist, choir director, and was deeply involved in every
aspect of church life.

It was perhaps 25 years later before I actually read all of the bible, rather than just the sound
bites I'd heard all my life. Reading the bible was the key that opened the door. And it was a
door that could never be closed again. It took another five or six years before I could think of
myself as an atheist, one who believes in no gods at all.

As soon as I admitted it, I was rejected by my three grown sons and most other relatives. Only
my daughter, who had also been studying the bible and history, did not withdraw her love and
support. So I fully understand the "fear and trepidation" heaped upon T. M. Utchen by his
Southern Baptist mother-in-law.

Yesterday I read again Emmett F. Field's lecture "Is the Bible the Word of God?" published
by Dave Matson at the Oakhill Free Press. In just 36 pages, Emmett Fields unravels the myths
of the Christian bible. The Christian god, as depicted in the primitive writings of the Old and
New Testaments is a murderous tyrant, brutal, vindictive, obscene, pornographic, who has
prepared a place of everlasting torment for almost every human being ever born. As Fields
says, "With a god like that, we don't need a devil!"

I am ever grateful to the great minds who have taught me along the way: Voltaire, Thomas
Paine, Dr. Madalyn O'Hair, Dave Matson, Farrell Till, Leland Ruble, Judith Hayes, Kersey
Graves, Elizabeth Cady Stanton, and others too numerous to mention. My life as a religionist
was one of fear, guilt, subjection, and misery, but life as an atheist is peaceful, happy,
exciting, and confident.

(Dr. Dorothy B. Thompson, P. O. Box 562, Bandon, OR 97411-0562)

EDITOR'S NOTE: I hate to sound like a broken record, but I can't help pointing out from time
to time that those who have tried Christianity and rejected it seem to be unanimous in their

Volume 1990 - 2002 Issue


Page 1428 of 2049
Skeptical Review Edited by Farrell Till
opinion that life has been much more satisfying to them as skeptics and atheists than it was
when they were Christians. This is completely contrary to the commonly expressed view that
we hear from Christians who think that life would be nothing but gloom and doom if they
didn't have their god to believe in.

I hope that Bruce Weston and all others who are presently experiencing the mental turmoil
that inevitably comes with "deconversion" will recognize another common thread in the
letters of those who have already traveled this route. Uncertainty and even fear are quite
normal during the transition stages from believer to nonbeliever. It's mentally painful while it
is happening, but after it's over, relief and satisfaction will replace the psychological anguish.

About Inmate Penmanship...

Thanks for carrying me on the free list. I am to be released from prison next week, so please
cancel my subscription. I should be a paying subscriber in a month or so. I cannot tell you
how edifying your paper has been, not just to me but to all who weren't afraid to read it.

Additionally, I loved the way you handled the guy who thought prisoners were "less than"
others. You stood up for us, and I appreciate it. "Penmanship" indeed! Try lying on your bunk
to write. I haven't seen a desk in a decade.

Anyway, thanks again for everything. Keep up the good work.

(Allen O'Conner....)

EDITOR'S NOTE: Mr. O'Conner gave me no new address. I would have continued his
subscription until he was resettled if I knew where to send it. At any rate, I appreciate his
telling me that his address was changing. Many subscribers don't, and it costs $0.55 each
time a paper is returned undeliverable.

FREE SUBSCRIPTION: For a free subscription to The Skeptical Review, e-mail


jftill@midwest.net or write to P. O. Box 717, Canton, IL 61520-0717.

Volume 1990 - 2002 Issue


Page 1429 of 2049
Skeptical Review Edited by Farrell Till

Skeptical Review
Volume 10, Number 4
July/August 1999
Farrell Till, editor

• "What Have These Sheep Done?"


The Bible declares that people should not be punished for the sins of others, but the
story of David's census of Israel shows that Yahweh didn't practice what he preached
to others.
• "What Daniel Didn't Know"
Farrell Till concludes his response to Everette Hatcher's attempt to defend the 6th-
century B. C. dating of the book of Daniel.
• "When Did the Wolf Dwell with the Lamb?"
Farrell Till concludes his examination of prophecy-fulfillment claims that Bruce
Weston had cited as reasons why he still wondered if the Bible might after all be
divinely inspired.
• "Hutchinson's Cheap Shots"
Dave Matson replies to Roger Hutchinson's claim that the Bible cannot influence
people to be intolerant or violent.
• "Two Faces of the Bible: Continued Confusion"
Roger Hutchinson replies to Farrell Till's article that discussed the two faces of the
Bible.
• "A Poor Selling Job"
Farrell Till replies to Roger Hutchinson's article.
• "From the Mailbag"
Readers and the editor exchange views on a variety of topics.

Volume 1990 - 2002 Issue


Page 1430 of 2049
Skeptical Review Edited by Farrell Till

What Have These Sheep Done?


One of the most puzzling tales of the Bible is told in 2 Samuel 24 and 1 Chronicles
21.Yahweh or Satan (depending on which account you want to believe) "moved David to
number Israel" (v:1).Since biblical inerrantists argue that the Bible is completely free of
errors, we will assume that in some sense Yahweh moved David to number Israel.One would
think that if Yahweh moved David to number Israel, Yahweh would have been pleased if
David did as he had been "moved" and took the census, but if you think this way, you are
reasoning like a rational person, and Bible stories aren't necessarily rational.In fact, they many
times tax the imagination of those who try to find rationality in them.

That's the case with this story about David.He conducted the census just as Yahweh had
"moved" him to do, but for some reason known only to Yahweh and the Gleason Archer type
of "apologists" who entertain us with verbal gymnastics that supposedly explain biblical
discrepancies, Yahweh was ticked off after David had done exactly what he had been
"moved" to do, and so he sent Nathan the prophet to call David on the carpet for taking the
census (2 Sam. 24:12).That wasn't really necessary, because David had already realized that
in doing what Yahweh had "moved"him to do, he had somehow sinned. That's what the
inspired word of God says: "But afterward, David was stricken to the heart because he had
numbered the people" (v:10).Why taking a census would be a sin, especially after God had
moved David to do it, is anyone's guess.Well, not anyone's guess, of course, because the
professional "apologists" like Gleason Archer, Norman Geisler, John Haley, etc. were
apparently blessed with special insights that enabled them to know that the Bible didn't really
mean what it plainly said. Just read their books, and you'll find all of the answers if you can
stop laughing long enough to read them all the way through.

At any rate, David realized he had sinned by taking the census that Yahweh had "moved" him
to take, and so he said, "I have sinned greatly in what I have done" (v:10). It's odd that David
would have thought that he had sinned and that even Yahweh's prophet Nathan would have
thought the same thing, because the inspired word of God claimed in 1 Kings 15:5, long after
he was dead, that "David did that which was right in the sight of Yahweh and did not turn
aside from anything that hecommanded him all the days of this life, except in the matter of
Uriah the Hittite," but this story about David in 2 Samuel 24 claims that David sinned in
conducting the census that Yahweh had "moved" him to take.But the census that David took
had nothing at all to do with the matter of Uriah the Hittite, whom David had murdered 12
chapters earlier, so if David never did anything wrong except in the matter of Uriah the
Hittite, how could his act of numbering Israel have been a sin?Oh, the confusion that comes
with trying to make sense out of the inspired, inerrant word of God!

But not to worry.The professional apologists have the problems all figured out.Just read their
books.If you do, you may encounter further confusion, because the "solutions" of the experts
don't always agree. John Haley, for example, talked about David's repentance and grief over
his sins (Alleged Discrepancies of the Bible, Baker House, pp. 222-223), but he never got
around to explaining how that if David had never done anything wrong except in the matter of
Uriah the Hittite, the census he took could have been a sin. Gleason Archer used an entirely
different approach, but he, like Haley, never did explain how that David always did what was

Volume 1990 - 2002 Issue


Page 1431 of 2049
Skeptical Review Edited by Farrell Till
right in the sight of Yahweh, except in the matter of Uriah the Hittite, yet had somehow
sinned in taking the census. Archer even listed other sins that David had committed but
excused them by saying that "David's heart was all there for God [whatever this means], and
God was his very reason for living" and that "David could never remain out of fellowship
with God for very long" (*Encyclopedia of Bible Difficulties,* Zondervan, p. 200).But just
how does any of this explain why the census could have been a sin if David had always done
what was right except in the matter of Uriah the Hittite?Maybe Roger Hutchinson, who is
presently on a crusade to enlighten us on the virtues of the Bible (see pp. 6-10, this issue), can
tell us what the real solution to this problem is.

The further one goes into this story of David's census, the more the confusion multiplies.We
learn not only that David sinned in doing exactly what Yahweh had moved him to do,
although he really did nothing wrong all of his days except in the matter of Uriah the Hittite,
but also that even the divinely inspired writers couldn't agree on the outcome of the
census.One inspired writer said that the number of valiant men was 800 thousand in Israel and
500 thousand in Judah (2 Sam. 24:9), but the other inspired writer said that the numbers were
1.1 million and 470 thousand (2 Chron. 21:5).But again we don't have to worry. The
professional "apologists" have this problem worked out, with different solutions, of course,
but "solutions" nevertheless.Haley thought that "oral tradition" or "copyist errors" may have
accounted for the differences, but Archer thought that the 800,000 Israelites in the one
account were only "battle-seasoned veterans," but the 1.1 million in the other account
included 300,000 men of military age who had had no battle experience.Yeah, right, the
biblical text gives plenty of reason to conclude that, doesn't it?Maybe Hutchinson can let us
know what the "real" solution is.

One would think that a fairly simple story could have no more inconsistencies than these, but
they continued right up to the end.The first account said that Yahweh sent Nathan the prophet
to offer David his choice of three punishments for his sin: "Shall seven years of famine come
to you in your land, or shall you flee three months before your enemies while they pursue
you, or shall there be three days' plague in your land" (2 Sam. 24:13)?On the other hand, the
second account says that Yahweh offered David just three years of famine rather than seven
(1 Chron. 21:12).If Nathan said three years, then he couldn't have said seven years, and if he
said seven years he couldn't have said three years, so what did he say, three or seven?This
time the professional "apologists" are getting help from modern versions of the Bible, which
solved the problem by just translating it away.The RSV, NRSV, NIV, GNB, and other recent
translations have put "three years" in the 2 Samuel account with footnotes to explain that the
Hebrew text actually says "seven years." How many people, who don't bother to read
footnotes, will pass over this white wash without even noticing it?

Translators have been a bit more honest with another glaring inconsistency in this story.The
first account of this affair in David's life ends with his buying a piece of ground that was later
used as the site of the temple.Here he paid 50 shekels of silver for property identified as a
team of oxen and a "threshing floor" (24:24), but the account in 1 Chronicles claims that he
paid 600 shekels of gold for "the place" (21:26). Once again the professionals have ridden to
the rescue.Haley speculated that the discrepancy could be just a copyist error (as if a scribe
could look at 50 shekels of silver and mistake it for 600 shekels of gold) or that David first
bought the oxen and only the top of the hill but then later decided to buy the whole hill of

Volume 1990 - 2002 Issue


Page 1432 of 2049
Skeptical Review Edited by Farrell Till
Moriah on which the threshing floor was located (p. 390). Archer gives a slightly different
twist to his "solution."The first account mentioned only the amount that David paid for two
oxen and the wooden cart that was used in threshing, butthe second account included not just
the price paid for the oxen and the cart but also the land too.In other words, David laid out 50
shekels of silver for the oxen and cart and then gave Ornan 600 shekels of gold for the
threshing floor. Readers will have to examine both accounts to see that either "solution" is
pure speculation. To show how unlikely these solutions are, let's imagine that both accounts
had given the same amount (either 50 shekels of silver or 600 shekels of gold) as the sum that
David paid the property owner.In that event, how would biblicists react if skeptics should try
to claim thatthe two accounts are discrepant because one said that David paid this amount for
the oxen and the threshing floor, but the other said that David paid the amount for "the
place"?We would be accused of all kinds of dishonesty, yet biblicists think they are entitled to
use the same kind of reasoning to prove that discrepancies are not in the Bible.Go figure.

Some biblicists have even argued that silver was more valuable than gold at that time, and so
50 shekels of silver was equivalent to 600 shekels of gold, but as unlikely as this is, it
wouldn't chance the fact that one inspired writer said that David gave the owner 50 shekels of
silver, and the other writer said that he gave the owner 600 shekels of gold.Both statements
can't be right. Maybe Hutchinson will come to the rescue and tell us what is the "right"
solution to this problem.

These inconsistencies in the two accounts are rather minor compared to the appalling
depiction of Yahweh's morality in this matter. As already noted, through the prophet sent to
rebuke David, Yahweh offered him his choice of three punishments.David's decision was to
leave the matter in Yahweh's hands: "I am in great distress.Please let us fall into the hand of
Yahweh, for his mercies are great" (1 Sam. 24:14).Well, if David thought that the mercies of
Yahweh were great, he was in for a big surprise.Yahweh, who was always a big one for
sending plagues (as we see in reading the wilderness wandering stories), chose the last
punishment he had offered David and "sent a plague upon Israel from the morning till the
appointed time" (v:15).Now keep in mind that whatever the "sin" was in taking the census,
David was the one who had done it, but Yahweh sent the plague upon the whole nation of
Israel "from Dan [in the north] to Beersheba [in the south]" (v:15).The result was that 70,000
[that's a seven followed by four zeroes] men of Israel died (v:15).So David committed the sin,
but Yahweh killed 70,000 other people because of the sin. Whatever happened to Yahweh's
law that said others should not be punished for the sins of others but that "a person shall be
put to death for his own sin" (Dt. 24:16; Ezek. 18:20).Well, apparently when Yahweh saw a
chance to send a plague, he never let it go by.

This David had to be the luckiest scoundrel who ever lived.He committed adultery with
Bathsheba, and then conspired to have her husband Uriah murdered (2 Samuel 11).The law of
Moses, under which David lived, commanded the death penalty for both offenses: "If a man
be found lying with a woman married to a husband, then both of them shall die" (Dt. 22:22),
and, "He who strikes a man so that he dies shall surely be put to death" (Ex. 21:12).David
himself didn't actually strike Uriah dead, but he had conspired to have someone kill him, and
the prophet Nathan said to David, "You have killed Uriah the Hittite with the sword" (2 Sam.
12:10).So David committed two offenses that called for the death penalty under the law of
Moses, but he escaped the penalty by saying just five words, "I have sinned against Yahweh"

Volume 1990 - 2002 Issue


Page 1433 of 2049
Skeptical Review Edited by Farrell Till
(v:13), upon which Nathan said, "Yahweh has also put away your sin; you shall not die."Sins
had nevertheless been committed, and so Yahweh had to have some satisfaction. He obtained
that satisfaction by killing the baby that was born to Bathsheba as a result of her relationship
with David (vs:14-18).Yes, that's right.David committed sins that called for the death penalty,
but Yahweh let him off and killed his child.

In the matter of the census, David's luck held out. Hetook the census (as Yahweh had moved
him to do) and was living in a society presumably governed by the principle that each person
should die for his own sins, but instead of punishing David, Yahweh sent a plague that killed
70,000 other people.Even David was able to see the appalling injustice of it. When he saw the
angel, who was striking the people,standing with his hand stretched out toward Jerusalem,
David said to Yahweh [in whatever way people talked to Yahweh in those days], "Surely I
have sinned, and I have done wickedly, but these sheep, what have they done?"So maybe
David was a man after Yahweh's own heart after all (1 Sam. 13:14; Acts 13:22), or at least he
was a man after what Yahweh's heart should have been.David could see the injustice of what
was happening.He had committed the offense, but Yahweh was killing thousands of others for
something that another person had done.David was outraged enough to ask, "What have these
sheep done?"

A principle is involved here that biblical inerrantists just can't see or at least refuse to admit
that they see.The Bible is filled with tales of people whom Yahweh killed or ordered killed
for the "sins" of others. If the Genesis flood actually happened as recorded, then there would
have been hundreds of children and babies drowned who were too young to be responsible for
whatever "wickedness" their parents may have been guilty of.In their march through Canaan,
the Israelites were presumably acting under orders to leave nothing alive to breathe (Dt.
20:16), and the book of Joshua claims that this command was carried out (10:10; 11:11-15).If
these stories are true, then the Israelites, under direct orders from Yahweh, massacred
thousands of children, who were too young to be morally guilty of anything.Whatever
"wickedness" their parents may have been guilty of could not be blamed on the children.

One of the most reprehensible tales like this in all of the Bible is found in 1 Samuel 15, where
Samuel the prophet said to King Saul, "Now therefore heed the voice of the words of
Yahweh.Thus says Yahweh of hosts, `I will punish Amalek for what he did to Israel, how he
ambushed him on the way when he came up from Egypt.Now go and attack Amalek, and
utterly destroy all that they have, and do not spare them.But kill both man and woman, infant
and nursing child, ox and sheep, camel and donkey" (vs:1-3).

The incident that Yahweh used as an excuse here to order Saul to massacre the Amalekitesis
recorded in Exodus 17:8-16), but if this was an actual historical event, it happened during the
Israelite journey through the Sinai wilderness, which would have been over 400 years before
the time of king Saul! Deuteronomy 25:17-19 made reference to this event and ordered the
Israelites to remember it and, after they were settled into the land that Yahweh was giving
them, to "blot out the remembrance of Amalek from under heaven."The tale in 1 Samuel 15
was just the pay off for this grudge that Yahweh had carried for four centuries.

What we are really dealing with here, of course, is the ancient superstitions of barbarous
times.If any of this happened, it wasn't done by any directions from God. It was action being

Volume 1990 - 2002 Issue


Page 1434 of 2049
Skeptical Review Edited by Farrell Till
taken by barbarous people who wanted to believe that their god, whom they had created in
their own image, was directing them to kill their enemies, including even children and
babies.Biblical inerrantists, of course, will never admit this.They cling to the myth that their
precious Bible is the inerrant word of God, and so they lean over backwards to try to justify
such barbaric stories as these.However, they will never justify them in the minds of rational
people until they come up with something better than their claims that if God gave life, he had
the right to take life or that God did those children and babies a favor by killing them at an
age when they would go to heaven rather than grow up to become wicked like their
parents.They're going to have to explain rationally why it was morally right to kill people for
what others had done.They're going to have to give a rational answer to David's question:
"What have these sheep done?"

What Daniel Didn't Know


Farrell Till
When the book of Daniel was written is not nearly so important as whether it contains errors.
If I should be able to prove that Daniel was not written in the 6th century B. C.--and I
certainly think I have given sufficient reason to suspect that it wasn't--all I would really have
established is that it contains at least one error in attributing its authorship to a 6th-century B.
C. Hebrew official in the Babylonian government. In this my final reply to Everette Hatcher
(until he is heard from again), I am going to reemphasize obvious historical errors in the text
of Daniel. In so doing, I will show that (1) the book is not inerrant and (2) the errors are such
that we cannot reasonably believe that a 6th-century B. C. Babylonian official would have
made them.

The father-son problem: In "Bad History in the Book of Daniel" (July/August 1998), I used
over one page (pp. 6-7) to show that the writer thought that Nebuchadnezzar and Belshazzar
were father and son. Hatcher's "rebuttal" of this argument consisted of a truncated quotation
from my article, in which he omitted with an ellipsis (...) the biblical passage that so presented
the relationship between Nebuchadnezzar and Belshazzar, followed by a dismissal of the
problem with a remark about the absence of a word for grandfather in both Hebrew and
Aramaic, but I didn't need Hatcher to tell me that Hebrew and Aramaic didn't have a word for
grandfather. This is common knowledge to anyone who has engaged in serious biblical
research. It is also common knowledge that when the Hebrew words for "father" and "son"
were being used to denote other than their primary meanings, the context made their
secondary meanings obvious. The context of Daniel 5 gives no indication at all that the words
for "father" and "son" were being used to denote anything but their primary meanings. I
suspect that Hatcher knows this and that this is why when he quoted my article, he chose to
omit with an ellipsis the passage from Daniel that I had quoted to show the writer's mistaken
impression that Nebuchadnezzar and Belshazzar were father and son. So that everyone can
see the force of my argument, I am going to quote this passage again. It is long, but as I said
before, necessary to show that the evidence is clearly against Hatcher in this matter.

Volume 1990 - 2002 Issue


Page 1435 of 2049
Skeptical Review Edited by Farrell Till
King Belshazzar made a great festival for a thousand of his lords, and he was drinking wine in
the presence of the thousand. Under the influence of the wine, Belshazzar commanded that
they bring in the vessels of gold and silver that his father Nebuchadnezzar had taken out of
the temple in Jerusalem, so that the king and his lords, his wives, and his concubines might
drink from them. So they brought in the vessels of gold and silver that had been taken out of
the temple, the house of God in Jerusalem, and the king and his lords, his wives, and his
concubines drank from them. They drank the wine and praised the gods of gold and silver,
bronze, iron, wood, and stone.
Immediately the fingers of a human hand appeared and began writing on the plaster of the
wall of the royal palace, next to the lampstand. The king was watching the hand as it wrote.
Then the king's face turned pale, and his thoughts terrified him. His limbs gave way, and his
knees knocked together. The king cried aloud to bring in the enchanters, the Chaldeans, and
the diviners; and the king said to the wise men of Babylon, "Whoever can read this writing
and tell me its interpretation shall be clothed in purple, have a chain of gold around his neck,
and rank third in the kingdom." Then all the king's wise men came in, but they could not read
the writing or tell the king the interpretation. Then King Belshazzar became greatly terrified
and his face turned pale, and his lords were perplexed.
The queen, when she heard the discussion of the king and his lords, came into the banqueting
hall. The queen said, "O king, live forever! Do not let your thoughts terrify you or your face
grow pale. There is a man in your kingdom who is endowed with a spirit of the holy gods. In
the days of your father he was found to have enlightenment, understanding, and wisdom like
the wisdom of the gods. Your father, King Nebuchadnezzar, made him chief of the
magicians, enchanters, Chaldeans, and diviners, because an excellent spirit, knowledge, and
understanding to interpret dreams, explain riddles, and solve problems were found in this
Daniel, whom the king named Belteshazzar. Now let Daniel be called, and he will give the
interpretation."
Then Daniel was brought in before the king. The king said to Daniel, "So you are Daniel, one
of the exiles of Judah, whom my father the king brought from Judah? I have heard of you
that a spirit of the gods is in you, and that enlightenment, understanding, and excellent
wisdom are found in you. Now the wise men, the enchanters, have been brought in before me
to read this writing and tell me its interpretation, but they were not able to give the
interpretation of the matter. But I have heard that you can give interpretations and solve
problems. Now if you are able to read the writing and tell me its interpretation, you shall be
clothed in purple, have a chain of gold around your neck, and rank third in the kingdom."
Then Daniel answered in the presence of the king, "Let your gifts be for yourself, or give your
rewards to someone else! Nevertheless I will read the writing to the king and let him know the
interpretation. O king, the Most High God gave your father Nebuchadnezzar kingship,
greatness, glory, and majesty. And because of the greatness that he gave him, all peoples,
nations, and languages trembled and feared before him. He killed those he wanted to kill, kept
alive those he wanted to keep alive, honored those he wanted to honor, and degraded those he
wanted to degrade. But when his heart was lifted up and his spirit was hardened so that he
acted proudly, he was deposed from his kingly throne, and his glory was stripped from him.
He was driven from human society, and his mind was made like that of an animal. His
dwelling was with the wild asses, he was fed grass like oxen, and his body was bathed with
the dew of heaven, until he learned that the Most High God has sovereignty over the kingdom
of mortals, and sets over it whomever he will. And you, Belshazzar his son, have not humbled
your heart, even though you knew all this! You have exalted yourself against the Lord of
heaven! The vessels of his temple have been brought in before you, and you and your lords,

Volume 1990 - 2002 Issue


Page 1436 of 2049
Skeptical Review Edited by Farrell Till
your wives and your concubines have been drinking wine from them. You have praised the
gods of silver and gold, of bronze, iron, wood, and stone, which do not see or hear or know;
but the God in whose power is your very breath, and to whom belong all your ways, you have
not honored (Dan. 5:1-23, NRSV).

Let's hope that Hatcher, if he replies again, will try to address this problem instead of just
waving it aside. He has before him an extended passage from the book of Daniel in which a
principal subject of the text was repeatedly referred to as the "father" of Belshazzar, who was
also referred to as the "son" of Nebuchadnezzar. Babylonian records, however, show that
Nebuchadnezzar was not Belshazzar's father, and no record of any kind has been recovered
that even indicates they were related. This is why the absence of a word for "grandfather" in
both Hebrew and Aramaic is irrelevant to this matter, because if Nebuchadnezzar wasn't even
an ancestor of Belshazzar, he certainly couldn't have been a distant "grandfather" to whom the
Hebrew word for "father" could have been appropriately used to express a genealogical
relationship between Nebuchadnezzar and Belshazzar.

I made another point in the same article that Hatcher didn't even wave at when he was passing
it by. I quoted from the apocryphal book of Baruch a passage that clearly shows that the
writer of this book also thought that Nebuchadnezzar and Belshazzar were father and son.
This mistake occurred in a context in which captives in Babylon were sending a contribution
to priests who were still in Jerusalem.

They [the Babylonian captives] sent this message:The money we are sending you is to be
used to buy whole-offerings, sin-offerings, and frankincense, and to provide grain-offerings;
you are to offer them on the altar of the Lord our God, with prayers for king Nebuchadnezzar
of Babylon and for his son Belshazzar, that their life may last as long as the heavens are
above the earth. So the Lord will strengthen us and bring light to our eyes, and we shall live
under the protection of King Nebuchadnezzar of Babylon and of Belshazzar his son; we shall
give them service for many a day and find favour with them (Baruch 1:10-12, REB version).

As I noted in my previous article (TSR, July/August 1998, p. 7), the author of this apocryphal
book claimed that he was writing "on the seventh day of the month, in the fifth year after the
capture and burning of Jerusalem by the Chaldaeans" (1:12). This would have been in 582 B.
C. while Nebuchadnezzar was still reigning but 26 years before Belshazzar's actual father
(Nabonidus) had usurped the throne via a military coup, so it is unlikely that Belshazzar
would have been known well enough at that time to have been regarded in any sense as the
"son" of Nebuchadnezzar. The only sensible conclusion to reach is that for some reason there
was a mistaken impression in the 2nd century B. C., when the apocryphal book of Baruch was
written, that Nebuchadnezzar and Belshazzar were father and son. This is compelling
evidence that the book of Daniel was also written in a time when this misimpression
prevailed, but even if it does nothing directly to date the book of Daniel, it certainly supports
my claim that the writer of Daniel made a significant error in chapter 5 by constantly referring
to Nebuchadnezzar and Belshazzar as father and son. Is it reasonable to think that an
important government official in 6th-century Babylon could have made such an error? This is
a problem that warrants far more attention than Hatcher has given it.

Volume 1990 - 2002 Issue


Page 1437 of 2049
Skeptical Review Edited by Farrell Till
In reply to the reference to Belshazzar as the "son" of Nebuchadnezzar, Hatcher said that
Daniel was just following an "ancient custom of the time which was to recognize the king of
Babylon as the `son' (or successor) of Nebuchadnezzar." His support for this quibble was to
cite a couple of archaeological records in which "son" seems to have been used in the sense of
a royal successor. One of these was an inscription on the Assyrian Black Obelisk of
Shalmaneser III, where Jehu was referred to as the "son of Omri, even though they were not
related" (March/April 1999, p. 5). We see, then, that Hatcher argues by assuming the
inerrancy of not just the Bible but also other ancient records. We have no way of knowing
whether the writer of this inscription was aware that Jehu was not a descendant of Omri, so as
far as we actually know, this reference could hardly constitute evidence that it was an "ancient
custom" to refer to successors of kings as "sons" even though they weren't really related. This
could have been just a mistaken impression that the writer of the inscription had. If Omri had
been a well known predecessor king of Jehu in a time when thrones were passed down from
father to son, a writer in another country, who was unaware of how Jehu had gained the
throne, could have easily thought that he was a descendant of Omri.

For the sake of argument, however, let's just assume that "father" and "son" were used in
ancient times exactly as Hatcher is arguing. He surely will admit that such uses were very
secondary and that the primary meanings of the words were the same as we use them today,
so when secondary senses were intended, these were recognizable by the context in which
they were used. When, for example, King Abimelech rebuked Abraham for trying to pass his
wife Sarah off as his sister, Abraham said, "Indeed she is truly my sister. She is the daughter
of my father, but not the daughter of my mother" (Gen. 20:12). This context in which "father"
was used easily enables the reader to recognize that it was intended to mean father in the
primary sense of the word. Likewise, the context shows that Moses was using the word
"fathers" in a secondary sense when he said to the Israelites, "So it shall be when Yahweh
your God brings you into the land of which he swore to your fathers, to Abraham, Isaac, and
Jacob, to give you large and beautiful cities." No one reading this would think that the word
fathers was being used in its primary sense.

On the other hand, those who read the 5th chapter of Daniel would certainly think that the
words father and son were being used in their primary senses to express the relationship
between Nebuchadnezzar and Belshazzar. If I am overstating my case here, I would like for
Hatcher to demonstrate it by citing a biblical passage comparable to this one in which the
writer went on verse after verse referring to someone as the "father" of so-and-so in a context
that clearly shows that father was being used to denote only ancestor or successor. Until he
can do that, he has no case.

Darius the son of Ahasuerus? Hatcher continues to skate on thin ice by sticking to his
premise that Darius and Cyrus were just different names for the same person. This premise is
based on a very flimsy possibility that the waw conjunction in Daniel 6:28 meant even instead
of and, so the verse could have been saying that "Daniel prospered in the reign of Darius,
even in the reign of Cyrus the Persian." What I wonder is why so many translators have failed
to realize this, because an extensive check of translations will show that they consistently
rendered the verse as quoted above.

Volume 1990 - 2002 Issue


Page 1438 of 2049
Skeptical Review Edited by Farrell Till
Another problem with this quibble has been ignored by Hatcher: if Darius and Cyrus were the
same person, then Cyrus was the son of someone who was born after him. Daniel 9:1 says
that Darius was "the son of Ahasuerus," but Ahasuerus was a Persian king whose reign didn't
begin until 485 B. C., 54 years after Daniel claimed that Babylon fell to Darius the Mede!
Cyrus's son Cambyses II probably served as a co-regent, but he was succeeded by Darius the
Great who wasn't a descendant of Cyrus. Darius the Great was then succeeded by his son
Ahasuerus. In other words, Ahasuerus was the third Persian king after Cyrus. How then could
Cyrus have been the "son of Ahasuerus"? Even son in the sense of descendant or successor
would make no sense in this context.

Hatcher, of course, is a biblical inerrantist, and so he has an answer to this problem.


"Ahasuerus probably is a title and not a personal name," he quibbled (March/April 1999, p.
3). What is his evidence to support this claim? He said that "the critic John Goldingay" said
so. What was Goldingay's proof of this? Well, if he had any proof, Hatcher didn't cite it.

Hatcher's reasoning: It's time, then, to take a look at the reasoning that Hatcher has used
throughout this debate. He has argued that the writer of Daniel really knew that the
Babylonian empire fell to the Persians and not the Medes, but instead of just coming out and
saying so, the writer chose instead to hint at it by punning on the word peres, which had the
same consonantal spelling as the word for Persian. To explain why the writer of Daniel
referred to Nebuchadnezzar and Belshazzar as father and son, he has argued that "father"
didn't really mean father and that "son" didn't really mean son. To explain how that Cyrus
could have been the son of Ahasuerus, a king who was third in line behind him, Hatcher has
argued that Ahasuerus, the name used several times in the Bible and ancient records in
reference to the same Persian king, wasn't really a name but just a title. To "prove" that Darius
the Mede and Cyrus were actually the same person, he has even argued that and didn't mean
and. Excuse the sarcasm, but that is somewhat like asking what does is mean. In a word,
Hatcher has quibbled throughout this discussion. Meanwhile, I have shown that what Daniel
didn't know is sufficient to cast deep suspicion on the claim that this book was written in the
6th century B. C. by a Babylonian official. Certainly, I have shown that the book is not
inerrant.

When Did the Wolf Dwell with the Lamb?


Farrell Till
In the past several issues I have been analyzing alleged prophecy fulfillments for Bruce
Weston, a subscriber whose biblical skepticism has been restrained by lingering feelings that
some prophecies may have been fulfilled. In my last article in this series, I will look at the
often-trumpeted claim that creation of the state of Israel in 1948 fulfilled biblical prophecies.

This one seems to be a favorite of prophecy-fulfillment buffs who are desperately looking for
something substantial on which to pin their hopes that the Bible is the inspired word of God.

Volume 1990 - 2002 Issue


Page 1439 of 2049
Skeptical Review Edited by Farrell Till
The history of Israel was checkered with periods of foreign captivities that began as early as
the time of the judges (Judges 3:7-11; 4:1-3; 6:1-6) and extended through the most famous
captivity of them all when Nebuchadnezzar removed many of the Judean captives to Babylon
(2 Kings 25). For an ethnocentric people who thought that they were the "chosen ones" of
their god Yahweh, these captivities were hard to comprehend. That they could be explained
by the simple fact of military science that the side with the largest army would usually win a
war was foreign to their way of thinking, and so the prophets of Israel always attributed
Israel's defeats to "evil" that the people had done "in the sight of Yahweh." This optimism that
the Israelites were the chosen people of the mightiest of all the gods inspired many of the
prophets to predict that although they were taken into captivity, Yahweh would one day bring
them back to their homeland.

After a unified Israel split following Solomon's reign, the northern kingdom (still known as
Israel), consisting of ten tribes, was defeated in 722 B. C. by the Assyrian king Shalmaneser,
and his successor Sargon II deported the population of the northern kingdom to Mesopotamia
and Media (2 Kings 17) and brought other captive people in to occupy the territory of Israel.
Although the southern kingdom Judah continued to exist, the Assyrian captivity ended the
existence of the northern kingdom. The deported Israelites were assimilated by the
Mesopotamians and became known as the "lost tribes of Israel," whose ethnic identity
remains "lost" until the present time.

This situation sparked a rash of restoration prophecies. Isaiah, who lived during these troubled
times and prophesied and wrote after the Assyrian deportation of the northern population,
made several predictions that the Israelites would be returned to their homeland. Isaiah 11:11-
12 is one of his better known "restoration" prophecies, and it is often quoted as a prophecy
that was fulfilled by the establishment of the state of Israel in 1948. In his article, Bruce
Weston cited this passage as one of the prophecies that troubled him. At this point, I would
recommend that those who have not yet read Dave Kroll's letter and my editorial comments
on pages 12-13 of this issue interrupt the reading of this article to notice what Kroll and I said
about the importance of considering the context of a "prophecy" before proclaiming that a
"fulfillment" has occurred. When the contexts of the so-called restoration prophecies are
analyzed, it is easy to see that the establishment of the state of Israel in 1948 fails to satisfy all
of the predictions in these "prophecies."

Isaiah 11:11 says, "It shall come to pass in that day that Yahweh shall set his hand again the
second time to recover the remnant of his people who are left." Biblicists desperately wanting
to see evidence that the Bible is the "inspired word of God" will quote this verse out of
context and proclaim that it is a wonderful prophecy fulfillment. Yahweh had called his
people out of Babylon to return them to their homeland, and now for the second time, he
restored their nation with the establishment of Israel in 1948.

A major problem with this fulfillment claim is that when this prophecy is analyzed within its
full context, too many other aspects of it were obviously not fulfilled in 1948 to consider the
establishment of Israel as a fulfillment of Isaiah's prediction that Yahweh would recover a
"remnant" of Israel a second time. The best way to see this is to examine what Isaiah
predicted in the verses before and after his "restoration" statement.

Volume 1990 - 2002 Issue


Page 1440 of 2049
Skeptical Review Edited by Farrell Till
There shall come forth a rod from the stem of Jesse, and a branch shall grow out of his roots.
The Spirit of Yahweh shall rest upon him, the Spirit of wisdom and understanding, the Spirit
of counsel and might, the Spirit of knowledge and of the fear of Yahweh. His delight is in the
fear of Yahweh, and he shall not judge by the sight of his eyes, nor decide by the hearing of
his ears; but with righteousness he shall judge the poor, and decide with equity for the meek
of the earth. He shall strike the earth with the rod of his mouth, and with the breath of his lips
he shall slay the wicked. Righteousness shall be the belt of his loins, and faithfulness the belt
of his waist (11:1-5).
Biblical references to a "branch" or "stem" from Jesse are almost universally seen as allusions
to a Messiah who would come from the lineage of David, the son of Jesse (Zech. 3:8).
Furthermore, the coming of this Messiah or branch from Jesse or David was invariably
associated with a regathering of the dispersed people of Yahweh.
Woe to the shepherds who destroy and scatter the sheep of my pasture," says Yahweh.
Therefore thus says Yahweh God of Israel against the shepherds who feed my people: "You
have scattered my flock, driven them away, and not attended to them. Behold, I will attend to
you for the evil of your doings," says Yahweh. "But I will gather the remnant of my flock out
of all countries where I have driven them, and bring them back to their folds; and they shall
be fruitful and increase. I will set up shepherds over them who will feed them; and they shall
fear no more, nor be dismayed, nor shall they be lacking," says Yahweh.
Behold the days are coming," says Yahweh, "that I will raise to David a branch of
righteousness; a king shall reign and prosper and execute judgment and righteousness in the
earth. In his days Judah will be saved, and Israel will dwell safely. Now this is his name by
which he will be called: Yahweh our righteousness (Jeremiah 23:1-8).

The context of Isaiah 11 also very clearly associated the coming of this branch from Jesse or
David with the second regathering of Yahweh's people predicted in verse 11. That is very
obvious to anyone who hasn't been blinded by fanatical desires to find fulfilled prophecies in
the Bible. Isaiah not only began this prophecy with the reference to the "stem of Jesse" but
alluded to the "root of Jesse" in the verse just before the one that predicted the second
regathering of the Israelites: "And in that day there shall be a root of Jesse, who shall stand as
a banner to the people; for the Gentiles shall seek him, and his resting place shall be glorious"
(v:10). Immediately after this verse, Isaiah made the prophecy that was allegedly fulfilled in
1948: "It shall come to pass in that day that Yahweh shall set his hand again the second time
to recover the remnant of his people who are left" (v:11). So if prophecy-fulfillment buffs are
going to claim that Isaiah's restoration prediction was fulfilled in 1948, they need to identify
for us the person who was the "branch" or "stem" of Jesse that accompanied this fulfillment
and stood "as a banner to the people" (v:10). When did this "branch of Jesse" "strike the earth
with the rod of his mouth" in 1948, and when did he "slay the wicked with the breath of his
lips" (v:5)? Biblicists could save themselves embarrassment if they would just examine the
full context of a "prophecy" before they shout, "Fulfillment!"

Another problem that they have to explain is why the apostle Paul quoted Isaiah 11:10 in a
context that clearly shows that he thought that Jesus was the "root of Jesse" that Isaiah
referred to: "Now I say that Jesus Christ has become a servant to the circumcision for the truth
of God, to confirm the promises made to the fathers, and that the Gentiles might glorify God
for his mercy, as it is written..." (Rom. 15:8-9). At this point Paul quoted four Old Testament
scriptures in support of his claim, the last of which was Isaiah 11:10, "And again, Isaiah says,

Volume 1990 - 2002 Issue


Page 1441 of 2049
Skeptical Review Edited by Farrell Till
`There shall be a root of Jesse, and he who shall rise to reign over the Gentiles, in him the
Gentiles shall hope.'" The reading is slightly different, but Paul was quoting from the
Septuagint, which rendered the verse as he wrote it. So if this "root of Jesse," whom Isaiah
said would come with the second regathering of Yahweh's people, was Jesus Christ, how can
prophecy-fulfillment proponents claim that the establishment of Israel in 1948, which came
almost 2000 years after Jesus, fulfilled a prophecy that said Israel would be restored "in the
day" that the "root of Jesse" came forth? I'll just let prophecy-fulfillment fanatics argue with
their own inspired, inerrant word of God.

A time of peace? Besides predicting that a "root of Jesse" would accompany the second return
of Yahweh's people, Isaiah also prophesied that this would be a period of unparalleled peace.

The wolf also shall dwell with the lamb, the leopard shall lie down with the young goat,the
calf and the young lion and the fattling together; and a little child shall lead them. The cow
and the bear shall graze; their young ones shall lie down together; and the lion shall eat straw
like the ox. The nursing child shall play by the cobra's hole, and the weaned child shall put his
hand in the viper's den. They shall not hurt nor destroy in all my holy mountain, for the earth
shall be full of the knowledge of Yahweh as the waters cover the sea (11:6-9).

So when did any of these events accompany the establishment of the state of Israel in 1948?
Prophecy-fulfillment buffs will be quick to point out that the language in this text is obviously
figurative, but that won't gain them anything. Even figurative language has intended meaning,
and the intention of the language here was to describe a time of perfect peace. The history of
Israel had been filled with oppression and struggles to maintain a hold on their homeland, and
at the time that Isaiah wrote this, their northern kingdom had been gobbled up by Assyrian
expansion, and ten of their tribes had been deported to Mesopotamian provinces. This, then,
was a prophecy that those who had been scattered abroad would not just be regathered to their
homeland but would also enjoy a time of peace so complete that it could be described as a
time when lambs, goats, and calves would dwell together with natural enemies like wolves,
leopards, and lions, and little children would be able to play unharmed with venomous
reptiles. Needless to say, no such period of peace accompanied the restoration of Israel in
1948. To the contrary, the history of modern Israel has been filled with wars and conflicts
with the Arab nations that surround it. Its very survival has depended on continuous vigilance
through military preparation. The children of modern Israel certainly don't play "on the
cobra's hole" and put their hands "into the viper's den." They live under a constant threat of
injury or death from terrorist activities of their nation's enemies.

A contemporary audience: The language that follows the prediction that the people of Israel
would be returned to their country clearly shows that Isaiah intended the prophecy for a
contemporary audience. The last part of verse 11 states that the people would be regathered
from Assyria, Egypt, Pathros, Cush, Elam, Shinar, and Hamath, but with the exception of
Egypt, none of these countries existed in 1948 when the nation of Israel was established.
Biblicists will quibble that Isaiah wasn't saying that these nations would still exist at the time
of the fulfillment but only that the people would be regathered from the geographical
locations once occupied by these countries. Using the names of the nations then in existence
was necessary, they say, in order for the people of that time to understand the prophecy, but in
so arguing they are admitting that Isaiah was speaking to a contemporary audience. What

Volume 1990 - 2002 Issue


Page 1442 of 2049
Skeptical Review Edited by Farrell Till
possible meaning or hope could the prophecy have had for that audience if it was referring not
to Isaiah's contemporaries but to Jews who would be living centuries later?

A context broader than just chapter 11 gives even more compelling evidence that Isaiah's
prophecy of Israel's restoration was directed to his contemporaries. In 10:5, he pronounced a
"woe to Assyria," the nation that had conquered the northern kingdom and sent its people into
foreign captivity, and then after a scathing denunciation of Assyria, he proclaimed that a
"remnant of Israel" would return and "never again depend on him who defeated them
[Assyria] but will depend on Yahweh, the holy one of Israel" (v:20). Then after repeating the
promise of the return of a "remnant of Jacob," he assured the people that they should "not be
afraid of the Assyrian" (v:24). "He [Assyria] shall strike you with a rod and lift up his staff
against you in the manner of Egypt," Yahweh said through Isaiah, but "yet a very little while
and the indignation will cease, as will my anger in their destruction" (vs:24-25). In addition to
his promises to bring the people back to their land, Yahweh also assured them that he would
"punish the fruit of the arrogant heart of the king of Assyria, and the glory of his haughty
looks" (10:12), but it would be difficult even for Yahweh to punish the king of Assyria after
there is no Assyria to have a king to punish, and certainly doing something 2700 years later
could hardly be called doing it in "yet a very little while." To argue that such passages as
these had reference to the distant future rather than to Isaiah's time is a desperate attempt to
find something to support an irrational belief in divine inspiration of the Bible.

The plundering of Israel's enemies: The context of Isaiah's "restoration" promise also
predicted that when Yahweh had "set his hand again the second time to recover the remnant
of his people," the enemies of Israel would be plundered and their resources destroyed.

They [the assembled outcasts of Israel, v. 12] shall fly down upon the shoulder of the
Philistines toward the west; together they shall plunder the people of the East. They shall lay
their hand on Edom and Moab, and the people of Ammon shall obey them. Yahweh will
utterly destroy the tongue of the Sea of Egypt, and strike it into seven streams and make men
cross over dry-shod (11:14-15).

The continued references to people and places that didn't exist in 1948 (Philistines, Edom,
Moab, and Ammon) give added indication that Isaiah was directing his prophecy to
contemporary times, but even if fulfillment buffs contend that these names were only
figurative references to territories that had these names in Isaiah's time, they must show us
when the reassembled outcasts of Israel plundered and destroyed these places in 1948 and
when the people of what was once Ammon obeyed them. They must also show us when
Yahweh "utterly destroyed" the tongue of the Egyptian Sea and struck the river of Egypt into
seven streams. None of this happened. Even when Israel later succeeded in its wars with the
Arabs, nothing of this magnitude happened.

This chapter of Isaiah closed with a final verse that conclusively shows that the prophet was
predicting a restoration of Israel that would happen in contemporary times: "There will be a
highway for the remnant of his [Yahweh's] people who will be left from Assyria, as it was for
Israel in the day that he came up from the land of Egypt" (11:16). Assyria was the nation that
had overrun the northern kingdom and taken its people into Mesopotamian captivity, and so
this chapter was prophesying an end of that captivity and the captives' return home, and not to

Volume 1990 - 2002 Issue


Page 1443 of 2049
Skeptical Review Edited by Farrell Till
a time of restoration so remotely distant in the future that it would have no meaning to Isaiah's
audience.

The prophecy failed even for that time, because no such restoration of the northern tribes ever
occurred, and they remain "lost" until this time. Biblical inerrantists, however, can't afford to
admit prophecy failure, and so they lifted two verses entirely out of context and assigned to
them an interpretation that the complete context will not support. If you are ever confronted
by a claim of biblical prophecy fulfillment, examine the statement in its full context, and you
will almost always find elements (like wolves that didn't dwell with lambs) that don't fit into
the fulfillment event. It's the easiest way to debunk fulfillment claims.

Hutchinson's Cheap Shots


Dave Matson
Roger Hutchinson's last article (May/June 1999) was one sorry affair. I promptly lost what
little respect I had for him as a seeker of truth. No cheap shot was too low for Roger. No Sir!
How shamelessly he flung those tired, old canards at us! At the very outset, we skeptics were
portrayed as duped pawns of Farrell Till.

I could not believe my eyes when he dredged up that old, sophomoric banality about atheist
killers of the 20th century! Perhaps a quotation from his article is in order "When we look at
the 20th century, we find that it is the skeptics who have rightfully earned a reputation for
gross atrocities against mankind. As examples, we can readily cite the handiwork of Stalin in
Russia, the communists in China, and the killing fields of Cambodia. Collectively, those
involved in just these three examples rejected the Bible and what it teaches and accounted for
the deaths of some 50 million innocent people" (May/June, p. 8).

When we look at the 20th century, we find that it is the anti-Islamics who have rightfully
earned a reputation for gross atrocities against mankind. We may readily cite the bloody
handiwork of Stalin in Russia, the communists in China, and those fanatical Khmer Rouge.
Collectively, they accounted for the deaths of some 50 million people--and the killers were all
anti-Islamic. Indeed, Hitler, himself, was anti-Islamic! But, more anti-Islamic than all of these
is Christianity, the leader of the pack, which has fought the Muslims for centuries. Therefore,
Christian philosophy is behind every major massacre of this century! Let Hutchinson, by his
own logic, acknowledge the moral bankruptcy of his religion.

Well, that's a horse of a different color! I guess it never occurred to Hutchinson that each of
his three groups held a number of beliefs, most of them having nothing to do with the killing
of innocents. That Roger so easily singles out skepticism as the culprit tells us more about his
prejudices than anything else.

Volume 1990 - 2002 Issue


Page 1444 of 2049
Skeptical Review Edited by Farrell Till
Just possibly, those Khmer Rouge fanatics might have been pursuing their fantasy of
"purifying" the populace to fit their ideal of an agricultural-peasant society. They were killing
anyone with a hint of education. Skeptics, because of their ability for independent thought,
would have been prime targets! Don't blame these murders on skepticism.

Just possibly, those communist Chinese might have been committed to ridding their country
of capitalist influences. Maybe they just got tired of the ritual of seeing dead people collected
off the streets in the large cities every morning. Possibly, they got tired of seeing foreigners
control China's ports for their own benefit while supplying dope to the people. Quite possibly,
they got tired of the endless corruption and brutal repression of the poor. When the cork
finally blew, there was plenty of fizz behind it--and it had very little to do with biblical
skepticism.

As for the Soviet Union, I've heard it said that some of Joe Stalin's friends were actually
Christians! Had Christianity worked to his favor, Stalin would have supported it even though
he, himself, was skeptical. However, Christianity was too closely tied to the czars, whom
communism had just overthrown. Hence, atheism became the official view of the Soviet
Union. Stalin was not about to compete with the church. The failure of early attempts at
collective farming led to millions of deaths by starvation. Don't blame those deaths on
skepticism. Stalin's endless purges and maneuvering accounted for still more deaths. Put those
deaths down to personal ambition and fear, not to skepticism. For all its drawbacks, Soviet
communism was probably an improvement on the serfdom of Christian, czarist Russia.

All in all, skepticism (atheism) had nothing to do with the above massacres. It was ^never^ a
case of skeptics murdering people just because they were religious.

Indeed, I cannot think of any historical incident of significance where people were killed for
not being good atheists! On the other hand, I cannot even begin to count those cases where
people have been murdered because they didn't accept some religious viewpoint. Religious
fanaticism, national fervor, personal ambition, greed, misplaced idealism and, the need for
scapegoats are reasons why people are massacred.

In his thrash and trash mode, Hutchinson couldn't resist a passing poke at evolution "Perhaps
the teaching of evolution in the schools with its mantra, Survival of the Fittest, accounts for
the Paul Hills of the world." To this, I can only say, "Get an education!" Anyone who bothers
to look at the past issues of Science or Nature, two of the world's leading scientific journals,
will quickly find that there is no scientific controversy over the fact of evolution. Scientific
opinion differs on how evolution proceeded, on the specific role of natural selection, or as to
whether evolution proceeds smoothly on the whole or in accelerated bursts or jumps. There is
no scientific disagreement over the fact that life has descended with modifications. I, for one,
am delighted that despite the meddling of people who share Roger's views, many of our
schools still teach real science in their science classrooms.

I'll leave it to someone else to supply some of the facts concerning the long, dismal history of
Christianity. It was not by accident that when Christianity was at its height, the period was
known as the dark age.

Volume 1990 - 2002 Issue


Page 1445 of 2049
Skeptical Review Edited by Farrell Till
(Dave Matson, editor, The Oak Hill Free Press, P.O. Box 61274, Pasadena, CA 91116; e-
mail,
103514.3640@compuserve.com)

Two Faces of the Bible: Continued


Confusion
Roger Hutchinson
After my rebuttal of his fanciful idea that Christianity is to blame for radical elements in our
society, Farrell Till manufactured another attempt to make Christianity the scapegoat for evil
("The Two Faces of the Bible," TSR, May/June 1999). Although he was forced to tone down
his rhetoric from the first article, he shows that he is still confused about this issue.

Mr. Till now proposes that "it is entirely reasonable to think that at least some people who
have firm convictions that the Bible is the `inspired word of God' will read such stuff as this
and think that it is God's will for them to behave accordingly." This is an example of the
"maybe, kinda, sort of, perhaps" reasoning that one finds skeptics making in their efforts to
denigrate the Bible when they cannot twist the Bible to say what they want.

It is certainly possible for any person to read any document and slice statements out of context
because it suits their purposes. Such people typically have no interest in the document itself,
and they extract only those parts that can be fit into their personal agendas. It happens despite
Till's denials, and it happens with respect to the Bible. When people do these things, there is
no logical basis to conclude that such people are influenced by what they have read. We stand
on more solid ground if we theorize that such people act in their own self-interest and use the
document to promote that interest.

Skeptics, like Till, are even known for doing this. They cite Biblical verses out of context to
create the illusion that the Bible is slanted in one direction or another. For example, in his
article, Till again trumpets those verses in the Bible about witches and homosexuals as if,
carved out of their context, these verses capture the essence of the Scriptures. They do not.
Till laments my failure to address his pious citations of Biblical verses. In doing this, he
reveals that he misunderstands his own argument. I accept the historical accuracy of the Bible
and agree that it says that God instructed Israel to destroy pagan nations. I also freely
acknowledge that the Bible condemns the actions of witches and homosexuals. However, Till
did not question the historical accuracy of the Bible but instead theorized a linkage between
what the Bible says and the actions of radical elements in society. Till, apparently, has
difficulty remembering his own arguments. Regardless, the mere citation of Biblical verses is
not sufficient to establish the linkage he sought.

Volume 1990 - 2002 Issue


Page 1446 of 2049
Skeptical Review Edited by Farrell Till
The Bible condemns evil. Within that context, it condemns various forms of evil including
sexual immorality. Thus, it should not be surprising to Till and other skeptics that different
forms of sexual immorality, including homosexuality, are specifically identified and
condemned. However, Till's implication that the Bible uniquely condemns homosexuality
above all other forms of sexual immorality, including, for example, what President Clinton
does with interns, is blatantly misleading. Perhaps he does it because he can find no logical
argument with which to advance his opinion that evidence will support.

In the same manner, the Bible describes Israel as a society that was to be built on a foundation
of submission to God. Consequently, it should not be surprising that, within that context, all
those who advocate idol worship and disobedience to God, such as witches, are condemned.
The Bible does not single out witches for special punishment but includes witches among
those who are to be similarly punished for promoting idol worship.

Despite this, we find Till excising verses that speak of witches and homosexuality and
claiming that they exert some mystical power over people even though this is inconsistent
with the context in which the verses are found. I then pointed out that such verses can be
easily abused by people who are pursuing a personal agenda against homosexuals or witches.
It is one thing to hate someone for personal reasons as this is contrary to Biblical teaching.
Seeing that a person is doing wrong and then seeking to limit that person's influence over the
innocent is a different matter and entirely consistent with Biblical teaching. Till seems unable
to grasp this simple distinction. Thus, he offers hyperbole rather than argument to make his
case.

Mr. Till has personal opinions about Christianity and the Bible. While he has a good deal of
knowledge about what the Bible says, he often fails to understand what he reads. We find that
he develops conclusions that he seeks to "prove" by pulling verses from the Bible, separating
them from their context, and weaving them together in a way that allows him to reach the
conclusion he had intended to reach all along. His conclusion that the Bible is to blame for the
actions of people like Fred Phelps is an example.

That Till argues by misrepresenting what he reads can be readily shown. One need only read
what he claimed I said in my earlier article and compare that to what I actually wrote. For
example, he has me labeling, as skeptics, those responsible for the killing fields of Cambodia.
He then takes off on a rampage seeking to prove that they were not skeptics. He finally and
piously concludes that I cannot prove that they were skeptics. His tirade was nothing more
than a rabbit trail that had nothing to do with my comments or his original argument.

In doing this, Till was unable to understand a very simple line of reasoning. By using the
examples of Cambodia and Russia, I was able to show very clearly that great atrocities have
been committed in places and under circumstances that cannot be linked to Christianity. Till's
whole premise that Christianity should be blamed for the activities of radical elements in
society is thereby shown to be bogus since these very same things, and worse, happen in
societies where Christianity cannot be cited as the cause. A very simple argument confounded
Till.

Volume 1990 - 2002 Issue


Page 1447 of 2049
Skeptical Review Edited by Farrell Till
In another case, Till created a straw man that even he would be able to knock down. He
framed my argument as: (1) people committed atrocities; (2) they did not believe the Bible;
consequently (3), they were motivated by their lack of belief in the Bible. Till committed a
very simple logical error. The proper logic here is that people who do not believe the Bible
cannot be motivated by the Bible to commit atrocities. The central issue raised by Till builds
on what people believe; not their lack of belief.

Remember, Till's basic fantasy is that Christianity is to blame for all kinds of evil. The
counter argument is that all kinds of evil occur in the absence of Christianity. Consequently, if
evil occurs in the absence of Christianity, how can we accept Till's argument that Christianity
must be the cause of evil just because the two may, by coincidence, be found together?

We also find that Till does not take kindly to people who argue as he does. As he often does,
he gets on his hobby horse and decries those who say that we must define a "real" Christian.
Then within a few paragraphs, he argues that I had misused the term, "skeptic," so that it was
necessary to define a "true" skeptic.

Many skeptics seem to think that any person who goes to church is a Christian. In the Mailbag
section of the same issue of TSR, one writer ended by saying, "My life as a religionist was one
of fear, guilt, subjection, and misery, but life as an atheist is peaceful, happy, exciting, and
confident." Very properly, this person labeled herself as a "religionist." Till, in his comments,
blunders by referring to the person as a Christian. One would think that Mr. Till, with his
educational background and religious experiences, would be able to differentiate between the
two. Apparently he is not.

Mr. Till also mocks the idea that I should claim "to know what the personal beliefs and
intentions of people like Fred Phelps really are." He quotes me as saying that Phelps "has
always personally hated homosexuals." That is not exactly what I said, but it is pretty close
for a skeptic, so why quibble. What is interesting is that he then turns around and devotes half
of his article to expounding on the true motivations of people. Such are the ways of skeptics,
it seems.

Till's reference to inquisitions and other persecutions adds nothing to his argument. One can
hypothesize a casual relationship between the Bible and Christianity and certain people
involved in these events but that does not prove a causal relationship. The witch trials in late
17th century Massachusetts do not seem much different to me than the persecution of day
care providers in late 20th century Massachusetts. Each was marked by hysteria and wild
accusations. To make Christianity the common denominator in these events or in one or the
other just because one finds a casual association is ludicrous. That Till does so reveals some
ineptness on his part in conducting a competent investigation.

Of skeptics such as Till, it could be said that they are ever learning and never able to come to
the knowledge of the truth. Mr. Till's original article was nothing more than a personal
opinion driven by an aversion to Christianity. He left religion but apparently cannot stop
preaching.

Volume 1990 - 2002 Issue


Page 1448 of 2049
Skeptical Review Edited by Farrell Till
(Roger Hutchinson, 11904 Lafayette Drive, Silver Spring, MD 20902; e-mail:
RHutchin@AOL.com)

A Poor Selling Job


Farrell Till
If Roger Hutchinson is hoping that we will buy his idea of a Bible that instills high moral
principles in those who follow it, the article by Dave Matson and letters in the "Mailbag"
column show that he is doing a poor selling job. Space for them in this issue was limited, but
more letters about Hutchinson will be published later.So far, all of them have firmly disagreed
with him, but if any readers care to defend his position, I will certainly give them equal
consideration.

Admittedly, the Bible does teach the "message of love" that Hutchinson spoke about, but this
is just another inconsistency in this book that so many fundamentalist Christians revere as the
inspired, inerrant word of God.Here it will proclaim that God is loving and merciful, but there
it will proclaim that he is a jealous, angry, vindictive god who sometimes killed people for
petty, trivial reasons.The front-page article in this issue "What Have These Sheep Done?"
discussed this aspect of the Hebrew god Yahweh, so there is no need to rehash it here.Suffice
it to say that it is hard to reconcile the image of a baby-killing god who ordered the death of
people for trivial offenses like picking up sticks on the sabbath (Num. 15:32-36) or not having
been circumcised (Gen. 17:13-14; Ex. 4:24-26) with the idea of a kind, loving, merciful god
that Hutchinson claims the Bible teaches. Anyone with even a particle of objectivity in his
brain should see the incompatibility of the two concepts. If I should say that John Doe kills
babies and inflicts torture and even death on those who disagree with him in trivial matters,
but John Doe is a kind, loving, and merciful person, even Roger Hutchinson could see the
incongruity in the description, but his blind allegiance to an idea that should have died with
the superstitions of the past will not allow him to see the inconsistencies in the nature of
"God" as he was depicted in the Bible.Yet he has the unmitigated gall to accuse me of acting
in my own "self-interest" and using "the document [Bible] to promote that interest" (p. 7, this
issue). I guess we are supposed to believe that Hutchinsonhas no interest in promoting his
belief that the Bible is the "inerrant word of God" and that when he twists himself into verbal
knots to "explain" that the Bible didn't really mean what it says, he isn't using the Bible to
"promote [his] self-interest."

Wanting it both ways: In debating biblical inerrantists, I often have to remind those
following the debate that my opponents are trying to play both sides of the street.By this I
mean, that they will reject an argument or principle if it is detrimental to their cause but will
turn around later and use the same argument or principle when it is beneficial to their
cause.We see Hutchinson doing that in this discussion.He wants to have it both ways.On the
one hand, he rejects the premise that people could be negatively influenced by the various
stories in the Bible in which Yahweh was depicted as a petty, jealous, vindictive god who

Volume 1990 - 2002 Issue


Page 1449 of 2049
Skeptical Review Edited by Farrell Till
ordered genocide, even to the point of killing children and babies,and also decreed the death
penalty for witchcraft, homosexuality, and various trivial offenses, but on the other hand, he
wants us to think that people can be positively influenced by the "message of love" and high
moral principles that Christianity teaches. Well, he can't have it both ways.If he is going to
insist that the "good" contained in the Bible can have positive influences on people, he must
admit the possibility that the "bad" contained in the Bible can have negative influences.If not,
why not?

We hear Christians today declaring that the violence that people, and especially children, see
in movies, TV, and video games is responsible for the steadily increasing violence in our
society, and I personally believe that they are at least partly right.However, they can't argue
that violence in the entertainment industry contributes to violence in our society but that
violence, hatred, and intolerance depicted in the Bible does not contribute to violence, hatred,
and intolerance in our society.Christian organizations maintain radio and TV stations that
specialize in so-called "family entertainment," and this is obviously done because they believe
that wholesome entertainment will contribute to the development of wholesome values, so to
be consistent, Christians need to acknowledge that if the unwholesome is contained in the
Bible, it can negatively influence moral values.

I recently saw evidence that they do realize that some Bible stories need to be cleaned up
before they are suitable for "family" audiences.While channel surfing a few days ago, I
happened onto a station in Central Illinois that promotes itself as a "family" channel.At the
time, Ancient Secrets of the Bible was on, so I paused out of curiosity to see if it had raised its
standards of scholarship since the last time I had watched one of its programs. Needless to
say, it hadn't.I quickly saw that it was up to its old tricks of showing brief dramatizations of
Bible stories and then switching to "scholars" who would relate one-sided accounts of recent
"discoveries" that prove the historical accuracy of the tales. On this particular night, the
scholars were presenting evidence that the ancient city of Sodom once existed.The
dramatization showed the angels entering the city and being invited by Lot to spend the night
in his home, and it showed the crowd of men, who came to Lot's house and demanded that the
strangers inside be sent out to them.However, it glossed over the reason why they wanted the
strangers and simply depicted the crowd as revelers who wanted the strangers to come out and
join in the fun, and it left out entirely Lot's offer to give his two virgin daughters to the men to
do with as they pleased if they would just spare the strangers the indignity they intended.The
dramatization depicted the departure of Lot's family from Sodom prior to its destruction and
showed Lot's wife looking back and being changed into a pillar of salt, but the story stopped
short of showing Lot's daughters getting him drunk and then engaging in sexual intercourse
with him, which resulted in their impregnation.

I saw these omissions as tacit recognition by the producers of this "family program" that even
certain Bible stories should not be dramatized for "familyentertainment" without leaving out
the unsavory parts. If the Bible is a book that will have only positive influences on people,
then why shouldn't movie producers show everything when they dramatize its stories?Maybe
Hutchinson can tell us.

Citing Biblical verses out of context: Hutchinson accused me of acting in my own self-
interest by quoting the Bible out of context "to create the illusion that the Bible is slanted in

Volume 1990 - 2002 Issue


Page 1450 of 2049
Skeptical Review Edited by Farrell Till
one direction or another" (p. 7, this issue). The only example of my promotion of this self-
interest that he cited was a remark about my "trumpet[ing] those verses in the Bible about
witches and homosexuals."Actually, I have said a lot more about verses in the Bible where
Yahweh ordered the systematic extermination of non-Hebraic tribes, but Hutchinson made
only a fleeting comment about this in a blink-of-the-eye admission that God did "instruct
Israel to destroy pagan nations."I suspect that he doesn't want to say too much about this
aspect of the Bible, because there is just a bit too much in it about Yahweh's instructions to
"destroy pagan nations" for even diehard biblicists to feel comfortable in arguing that the
Bible teaches an ennobling message of divine love and mercy and not intolerance and hatred.

I quoted Exodus 22:18, which simply says, "Thou shalt not suffer a witch to live,"but if
Hutchinson thinks that I have quoted out of context what the Bible says about killing witches,
I'll quote the "full context" and then let him explain to us just how my comments
misrepresented what the Bible teaches on the subject.

Exodus 22:16-20: And if a man entice a maid that is not betrothed, and lie with her, he shall
surely endow her to be his wife.If her father utterly refuse to give her unto him, he shall pay
money according to the dowry of virgins.Thou shalt not suffer a witch to live.Whosoever lieth
with a beast shall surely be put to death.He that sacrificeth unto any God, save unto the
LORD only, he shall be utterly destroyed.

It's obvious that the verses before and after verse 18 (italicized) shed no additional
information onthe commandment to kill witches.This statement was simply listed in a series
of unrelated commandments, and so by quoting only this one verse, I was not taking anything
"out of context" in order to promote my self-interest. In fact, by not quoting also the verses
around the commandment to kill witches, I actually made Yahweh look a bit better than he
does when he "full context" of the statement is read, because the verses before this
commandment clearly show that Yahweh had no consideration for the rights of women,
whose marital futures were putentirely into the hands of men if they made the mistake of
having pre-marital relations.The men could buy their way out of forced marriage, but
apparently the women were not given this choice. The verse after the commandment to kill
witches shows that Yahweh was religiously intolerant to the point of ordering the death of
those who would dare sacrifice to any god but him, yet Hutchinson expects us to believe that
the Bible will have only an ennobling influence on those who read it.

If I had had any devious desire to promote my own self-interests, I could have expanded the
Bible's condemnation of witches by quoting other scriptures besides Exodus 22:18.

Leviticus 20:27: A man also or woman that hath a familiar spirit, or that is a wizard, shall
surely be put to death: they shall stone them with stones: their blood shall be upon them.
Deuteronomy 18:10-12: There shall not be found among you any one that maketh his son or
his daughter to pass through the fire, or that useth divination, or an observer of times, or an
enchanter, or a witch, or a charmer, or a consulter with familiar spirits, or a wizard, or a
necromancer, for all that do these things are an abomination unto the LORD.

Now maybe Hutchinson can tell us how that I took anything out of context when I said that "it
is entirely reasonable to think that at least some people who have firm convictions that the

Volume 1990 - 2002 Issue


Page 1451 of 2049
Skeptical Review Edited by Farrell Till
Bible is the `inspired word of God' will read such stuff as this and think that it is God's will
for them to behave accordingly."Why is it so hard to imagine that people still living in the
superstitious times of Puritan New England could read what the Bible says about witchcraft
and think that they were doing God's will by hanging witches?Would he find it hard to believe
that if people read where the Bible says, "Love thy neighbor as thyself," they might think that
it is God's will for them to love their neighbors?If so, what criteria does he use to determine
that if the Bible says something that is considered morally "bad," it won't influence people to
do bad, but if it says something "good," it will influence them to do good?Talk about double
standards!

I don't have to quote "full contexts" to make the same point about what the Bible says
concerning homosexuality.If the Bible says--and it does--that "if a man lies with mankind, as
with womankind, both of them have committed abomination: they shall surely be put to
death" (Lev. 20:13), and if the Bible says--and it does--that men who leave the natural use of
the woman and burn in their lust one toward another are worthy of death (Romans 2:27, 32),
why do I have to quote a long catalog of other "sins" listed in the same context (for which the
infinitely merciful Yahweh also commanded the death penalty) before I can legitimately say
that the Bible taught fanatical intolerance of homosexuality?If I am distorting what the Bible
says by quoting out of context, let Hutchinson show that I am.

"Maybe, kinda, sort of, perhaps" reasoning: Hutchinson said that my suggesting that the
unsavory passages in the Bible could cause some people to think that God wants them to do
what these passages say is "the `maybe, kinda, sort of, perhaps' reasoning that one finds
skeptics making in their efforts to denigrate the Bible when they cannot twist the Bible to say
what they want,"but is it "maybe, kinda, sort of, perhaps reasoning" when he claims that
passages in the Bible that teach a "message of love" could cause people to think that they
should do what these scriptures say?If so, then he must explain to us the reason for the double
standard he is applying when he says that anything "bad" that the Bible says cannot influence
people to do bad, but the good that the Bible says can and does influence people to do good.

His charge that I am using a "maybe, kinda, sort of, perhaps reasoning" is downright comical
coming from him, because this kind of reasoning has been his stock in trade in both The
Skeptical Review and his "apologetic" attempts on the internet.In his first appearance in TSR
(Autumn 1995, pp. 4-5), he tried to resolve the chronological discrepancy in Exodus 12:40
and the Exodus 6 genealogy of Aaron by suggesting that generations may have been skipped
in the genealogy, because the Hebrew word ben (son) could have meant descendant.He
further argued that the word yalad used in reference to Jochebed's having borne Moses and
Aaron to Amram (Ex. 6:20) could have meant "borne" in the sense of being the ancestor of,
and so "Jochebed can be said to have borne Aaron and Moses even though she may have
actually given birth to their great-grandfather" (p. 5, emphasis added).To underscore the
"maybe, kinda, sort of, perhaps reasoning" that Hutchinson used in his article, I entitled my
reply to it "Just Another Far-Fetched How-It-Could-Have-Been" (pp. 5-6).

There isn't enough space in an entire issue for me to summarize all of the maybesand could-
have-beens that Hutchinson has postulated in his frantic efforts to defend biblical inerrancy,
but one more example bears looking at. In the May/June 1996 issue, he tried to resolve the
discrepancy in what Jairus said to Jesus by postulating that he could have said what Matthew

Volume 1990 - 2002 Issue


Page 1452 of 2049
Skeptical Review Edited by Farrell Till
claimed he said and also what Mark claimed he said.In other words, he proposed that Jairus
said to Jesus both that his daughter was dead and that she was at the point of death.He then
proceeded to write completely fabricated scenes of Jairus's encounter with Jesus in which
both statements were said.Even he admitted that the scenarios were improbable."We can only
speculate on what might have happened," he said (p.4) and later concluded the article by
saying, "The above scenarios serve to illustrate how events could have played out in a manner
consistent with the scriptures" (p. 5).Hutchinson writes articles riddled with speculations like
these and then has the gall to say that skeptics use a "maybe, kinda, sort of, perhaps" kind of
reasoning to "denigrate the Bible."I wonder if Hutchinson has ever heard the proverb about
the pot calling the kettle black?

Who labeled the Khmer Rouge skeptics? Hutchinson said that I incorrectly accused him of
"labeling, as skeptics, those responsible for the killing fields of Cambodia," but he needs to go
back and read his first article in this present exchange.As Matson noted, here is exactly what
he said:

When we look at the 20th century, we find that it is the skeptics who have rightfully earned a
reputation for gross atrocities against mankind.As examples, we can readily cite the
handiwork of Stalin in Russia, the communists in China, and the killing fields of
Cambodia.Collectively, those involved in just these three examples rejected the Bible and
what it teaches and accounted for the deaths of some 50 million innocent people.If one is
honestly looking for radical elements with a reputation for atrocities, one need look no further
than to those who reject the Bible--skeptics (May/ June, p.8, emphasis added).

If Hutchinson wasn't accusing the leaders of Russia, China, and Cambodia of being biblical
skeptics who were responsible for the deaths of 50 million people, then I am unable to
understand plain English.After citing only the Russians, Chinese, and Cambodian leaders as
examples of non-Christian atrocities, Hutchinson went on to say, "Skeptics of the Bible have
proven themselves capable of great evil" (p. 8).After I pointed out the absurdity of attributing
communist atrocities to skepticism of the Bible, Hutchinson is now trying to backpedal.He
claims that he had used the examples of Cambodia and Russia only "to show very clearly that
great atrocities have been committed in places and under circumstances that cannot be linked
to Christianity" (p. 7, this issue). I don't disagree that many atrocities have been committed
that cannot be linked to Christianity, but it was clearly not Hutchinson's intention to "show"
this in his original article.He was trying to do exactly what amateur apologists
repeatedlyclaim on internet sites, which is that atheism has resulted in more moral atrocities
than Christianity, and just as Hutchinson did, they invariably cite the Russian, Chinese, and
Cambodian leaders as examples of atheistic perpetrators of atrocities. Some of them will
throw in Hitler for good measure, but many have learned not to do this, since it is easy to
prove from Hitler's speeches and books that he was a believer in Catholicism.At any rate,
Hutchinson obviously got burned on this point, and now he is looking for a graceful way out.

What exactly did Hutchinson say about Fred Phelps? Hutchinson accused me of
misrepresenting what he said about the homophobic preacher Fred Phelps."He [Till] quotes
me as saying that Phelps `has always personally hated homosexuals,'" Hutchinson said, and
then added, "That is not exactly what I said, but it is pretty close for a skeptic, so why
quibble" (p. 7).Well, let's look at exactly what I said:

Volume 1990 - 2002 Issue


Page 1453 of 2049
Skeptical Review Edited by Farrell Till
I proposed that it is entirely reasonable to think that at least some people who have firm
convictions that the Bible is the "inspired word of God" will read such stuff as this and think
that it is God's will for them to behave accordingly. *I have read Hutchinson's article several
times, but the only attempt that I saw him make to address these issues was to suggest that
Fred Phelps, the Baptist preacher who is infamous for his hatred of homosexuals, "has always
personally hated homosexuals and merely uses the Bible to give an air of legitimacy to his
hatred"* ("The Two Faces of the Bible," May/June 1999,p. 9).

I have quoted a longer than necessary section so that Hutchinson can't accuse me of quoting
out of context, and I have emphasized in italics what I said in reference to his comment about
Phelps.Now here is exactly what he said about Phelps in his first article.

Maybe Fred Phelps has always personally hated homosexuals and merely uses the Bible to
give an air of legitimacy to his hatred.He would not be the first to do so.Mr. Till would not be
the first skeptic to mistake a casual relationship for a causal relationship. Skeptics can be
heavy on emotion and short on logic ("Who's to Blame? More Hype Than Substance,"
May/June 1999, p. 8, emphasis added).

So just how did I misrepresent Hutchinson, and why wasn't what I quoted "exactly what [he]
said"?I can see why he decided that it would be best not to quibble on this point.

Is Christianity responsible for "all kinds of evil"? Hutchinson accused me of claiming that
Christianity is responsible for "all kinds of evil,"but I have never said any such thing.I will
stick by my claim that it has been responsible for numerous moral atrocities throughout its
history, but certainly it has not been responsible for all of the atrocities committed in the part
of the world it has dominated for the past several centuries.Unfortunately, as long as there are
humans on the planet, there will always be atrocities regardless of what religions control the
regions where they occur. However, the fact that Christianity has not been responsible for all
atrocities in no way means that it has not been responsible for many atrocities.Hutchinson
cannot wave aside Christianity's bloody past with just a fleeting comment about the Salem
witch trials, like the "persecution of day care providers in late 20th century Massachusetts,"
having been "marked by hysteria and wild accusations."I would never dispute that the Salem
witch trials were marked by hysteria, but it was a religious hysteria that occurred at a time
when witchcraft was denounced from pulpits and the trials of "witches" were condoned by
influential clergymen like Cotton Mather, and in a region that came the closest to being a
Christian theocracy as any government ever established in the American colonies.This was a
colony that exiled religious dissidents like Roger Williams and Anne Hutchinson, openly
persecuted Quakers, and eventually banished them too, under penalty of death if they
returned.After she was banished for antinomian views and later became a Quaker, Mary Dyer
was hanged when she returned to Boston.Mary Latham, an 18-year-old girl, was hanged for
the crime of adultery.Are we to assume that it was merely coincidental that acts like these
happened in a colony where a book that prescribed the death penalty for adultery and "false
worship" was venerated?It's hard to study the history of early New England and exonerate
Christianity of any blame for the religious persecutions that happened when government was
under the control of a Puritan establishment.

Volume 1990 - 2002 Issue


Page 1454 of 2049
Skeptical Review Edited by Farrell Till
Hutchinson left untouched the information that I included about the persecutions and
executions of "heretics" that happened during the inquisition, except to say that my "reference
to inquisitions and other persecutions add nothing to [my] argument" (p. 7).The inquisitions
were sanctioned and authorized by official decrees of the church, but, of course, this adds
nothing to my argument.If Hutchinson should produce records that clearly tied Russian,
Chinese, and Cambodian atrocities to the atheistic views of communist leaders, you can bet
that he would soundly denounce me if I should say that his references to this evidence "add
nothing to his argument."

He ended with a biblically inspired barb: "Of skeptics such as Till, it could be said that they
are ever learning and never able to come to the knowledge of the truth."Needless to say, I
disagree.In my case, I'm thoroughly convinced that when I was a preacher, I was "ever
learning" and finally able to come to the truth. I have little hope that this will ever happen to
Hutchinson. He seems doomed to live out his life ever trying but never able to defend his
absurd belief in biblical inerrancy.

From the Mailbag


Farrell Till

How to Make the World a Better Place...

I am 24 years old, and I was born and raised as a Georgia Baptist. Until last year, I had never
heard of anyone who questioned the Bible's inspiration. I thought everybody accepted the
Bible as God's Word, but some people just chose to ignore it. However, like most Bible
believers, I had never actually read the Bible for myself. The bulk of my Bible knowledge
came from stories that were told to me as I was growing up, but a little over a year ago, I went
through some pretty hard times, and I made the mistake of deciding to read the Bible from
cover to cover. I was hoping to find out what I was doing wrong to bring on my hardships. I
had barely made it through the first few chapters of Genesis when I realized that something
was terribly wrong. Eventually, I turned to the internet to see if I could make sense of it all,
and as I was searching for "Bible," I ran across a little site called "The Secular Web." It was
then that I began to read some of your articles, which in turn led me to my free one-year
subscription to *TSR.* *The Skeptical Review* and your other writings have really helped
me to break free of the control that the Bible had over me. I considered myself to be a very
"good Christian," but I feel that I am a much better person now that "the Good Book" is out of
my life for good! I believe that the whole world would be a better place if all Bible believers
would just sit down and ^read^ the Bible and see it for what it is. Thanks for all that you do to
promote reason in our sometimes unreasonable world. I have enclosed a check for another
year of *TSR.*

Volume 1990 - 2002 Issue


Page 1455 of 2049
Skeptical Review Edited by Farrell Till
T. Carver Anderson, 75 Forest Lake Road, Pendergrass, GA 30567)

EDITOR'S NOTE:

Ever since my own deconversion, I have thought the same thing that Mr. Anderson just said.
People don't believe the Bible because they have seriously studied it, critically examined it,
and concluded that it is the "inspired word of God." They believe the Bible only because they
grow up in societies in which almost everyone accepts that the Bible is a divine revelation,
and those that don't believe this just keep quiet about it because they don't want to experience
the displeasure of those who will be outraged if anyone should dare question the foundation
of their religious beliefs. Like Mr. Anderson, when I was growing up, I didn't know a single
person who questioned that the Bible was "God's Word." I knew that some people didn't go to
church and certainly didn't live according to basic biblical teachings, but I thought that this
was just because they wanted to enjoy the pleasures of "sinning."

My journey to skepticism began when I embarked on a plan to study the Bible from end to
end so that I could be a knowledgeable, effective preacher. The result was that I read myself
into rejection of the Bible, because I encountered too much nonsense and inconsistency in it
to go on believing what I had been taught as a child, *i. e.,* the Bible is God's inspired word.
Like Mr. Anderson, I too believe that if everyone who professes Christianity would actually
sit down and study the Bible, this would do more damage to the Christian religion than all of
the articles posted on internet sites like the Secular Web. I have had too many people tell me
that reading the Bible destroyed their faith to believe otherwise.

Mr. Anderson noted that he was helped in his rejection of the Bible by materials that he read
on the internet. As I have said in this forum many times, the internet will prove to be the most
formidable foe that Christianity has ever confronted. In the past, people like Mr. Anderson
and me could grow up and be totally unaware that there were some who rejected the popular
view of the Bible, because preachers and Sunday School teachers would never mention them
and libraries would not stock books and articles that questioned the inspiration of the Bible,
but that time is long gone. Today, anyone with a computer can go to the internet and with a
search engine access articles like those published in *TSR* and read debates between
Christians and biblical skeptics. The information monopoly that Christianity enjoyed for so
long is gone, probably forever. That's bad news for traditional Christianity.

No Longer Frightened...

In the past few years, I've added religious studies to my list of hobbies. I'm trying to answer
questions like, "Why are religions so prevalent and held to so fiercely?" The Skeptical Review
has greatly assisted me. Its clear reasoning and quotations of appropriate biblical passages
have provided insights and inspiration. Combined with other scholarly criticisms of the
Christian Bible as well as other religions, I can now find replies that confound, if not
convince, those fervent believers who knock on my door.

Better prepared, I no longer fear a presentation of "God's plan." Now it is they whom I find
poorly prepared. In large part because of your efforts, I perhaps understand the Christian
Bible well enough to do a bit of comparative study. Recently, I invited Mormons to my house

Volume 1990 - 2002 Issue


Page 1456 of 2049
Skeptical Review Edited by Farrell Till
for a bit of data collection. A cursory reading of the Book of Mormon shows that it adds
nothing new to Christian thought other than a manifestly false history. To boot, it is
exceedingly tedious. Give me the Old Testament any day for its originality and variety.

Despite the lack of any new ideas or real differences with other mainline Christian sects, the
Book of Mormon does repeatedly warn that unbelievers will be damned. Thanks at least in
part to The Skeptical Review I'm not frightened.

(Michael O'Brien, 1017 Chevney Way, Shady Cove, OR 97539)

EDITOR'S NOTE: I often receive letters from people just recently deconverted who express
doubts about the decision they have made. They are bothered by the possibility that they made
the wrong decision, because the Bible just might be the "word of God" after all. I tell them
that such doubts are normal but that if they will stick to their decision, they will eventually
pass through this stage and experience the liberating confidence of knowing that they made
the right decision. I'm glad to see that Mr. O'Brien has apparently reached this point.

In a sense, the Old Testament may be "original," but in many ways it isn't. This can be seen
more clearly when the religions of the biblical era are studied and compared to Judaism. The
ancient Hebrews had a tribal god, but so did the Moabites, the Philistines, and other
surrounding tribes. The Hebrews offered animal sacrifices to appease their god, but so did the
nations around them. The Hebrews built a temple to their god, but the nations around them
built temples to their gods too. The Hebrews had a priesthood that they thought gave them
access to their god, but so did the nations around them. When the ancient Jewish religion is
compared to the religions of other nations of that time, many imitations in Judaism can be
identified. Critical scholars recognize, for example, that much of the material in the first
eleven chapters of Genesis was borrowed from Babylonian mythology.

Ezekiel's Prophecy against Tyre...

Thank you for continuing to publish The Skeptical Review. I find your material very helpful in
my continuing quest to arrive at a frame of reference that I can live with. Please renew my
subscription for another year for which I have enclosed the appropriate payment.

In regard to your recent article on Tyre, may I make the following point. When you simply
consider the context of this prophecy it becomes rather clear that any futuristic interpretation
of this prophecy becomes a moot point. In Ezekiel 26:2-3, the prophet gave the reason for
judgment to come against Tyre: "Because Tyre has said of Jerusalem, `Aha! The gate to the
nations is broken, and its doors have swung open to me; now that she lies in ruins I will
prosper,' therefore this is what the sovereign Lord says: I am against you, O Tyre." What
follows is God speaking in the first person, describing what would become of Tyre and
concluding with these words: "Then they will know that I am the Lord." It should be apparent
that the *they* referred to here were the Tyrians who were being punished for having designs
on Jerusalem.

The context clearly shows that it was because of the attitude of Tyre toward Jerusalem that it
was going to be destroyed. This attitude was being exhibited by the Tyrians at the time this

Volume 1990 - 2002 Issue


Page 1457 of 2049
Skeptical Review Edited by Farrell Till
prophecy was written. This attitude could not have been projected hundreds and thousands of
years into the future to necessitate a continuing judgment against Tyre. God said that as a
result of this destruction "they will know that I am the Lord." This statement was directed to
the Tyrians living at the time of this prophecy because it was those Tyrians that had an
"attitude" toward Jerusalem, which was the reason they were going to be destroyed. Tyrians
living during the time of Alexander and into the hundreds of years after Alexander would
have no idea that they were being invaded because of a prior attitude toward Jerusalem and
therefore they would not come to "know the Lord" through such invasions. To punish Tyrians
living hundreds and even thousands of years into the future for an attitude exhibited by
Tyrians living at the time of Ezekiel would place in question the administration of justice in
this matter.

Furthermore, in Ezekiel 27, the prophet listed the various nations that traded with Tyre and
described the lamenting of these nations over the destruction of Tyre. By simply reading
through this material, it becomes obvious that the nations spoken of, and the kind of trading
that was done, related to nations existing at the time of Ezekiel and not nations existing
hundreds and thousands of years in the future that would be lamenting the destruction of Tyre.

The context shows that the events of this prophecy would apply to the Tyrians living at the
time this prophecy was written. Since it is obvious from both secular and biblical history that
Nebuchadnezzar did not accomplish a great deal against Tyre, we can clearly see the
nonfulfillment of what Ezekiel claimed that God had ordered relative to the nation of Tyre.

(Dave Kroll, 6865 North Burbank, Milwaukee, WI 53224; e-mail dakbjk@aol.com)

EDITOR'S NOTE: Mr. Kroll has made some excellent points and in so doing has called
attention to one of the simplest ways to debunk prophecy-fulfillment claims. When so-called
prophecy fulfillments are analyzed in terms of the full context of the prophetic statement, it
will become apparent that the alleged fulfillment event failed to satisfy the full scope of the
original prediction. In my latest article in the series that has been examining prophecies that
Bruce Weston inquired about (pp. 6-10, this issue), I showed that prophecy-fulfillment buffs
have focused on a single verse (Isaiah 11:11) to claim that the formation of the nation of
Israel in 1948 fulfilled Yahweh's promise to return his people to their homeland. However,
when the context in which this single verse appeared is analyzed, one can easily see that there
are too many aspects of the original prophecy that did ^not^ happen in 1948 to consider the
formation of Israel as a fulfillment of this prophecy.

The same is true of Ezekiel's prophecy against Tyre. In addition to the contextual problems
that Mr. Kroll identified, there is the matter of Nebuchadnezzar's role in the "fulfillment." The
context of Ezekiel 26:7-14 clearly shows that Yahweh was prophesying that Nebuchadnezzar
would be the agent who would bring about a total destruction of Tyre. To circumvent the
problem posed by this context, biblicists have tried to claim that Nebuchadnezzar fulfilled the
prophecy in that he destroyed the mainland or suburban areas of Tyre, but the total context of
the prophecy is not at all friendly to this quibble. As the prophecy continued into verse 27, the
context showed that Ezekiel was speaking not of mainland villages but of the island
stronghold itself. He referred to Tyre as a city that "dwells at the entry of the sea" and whose
borders are "in the heart of the sea" (v:4). These would be accurate descriptions of an island

Volume 1990 - 2002 Issue


Page 1458 of 2049
Skeptical Review Edited by Farrell Till
city but not of mainland villages. In verses 4-9, Ezekiel used the imagery of men building a
ship to describe the way that the city of Tyre was originally built. This is an appropriate
metaphor for a city that covered an entire off-shore island but hardly an appropriate one for
mainland villages. Ezekiel warned that when its destruction came upon Tyre in the "day of its
ruin," it would "fall into the heart of the sea" (v:27). There are many other contextual
references like these that appropriately described a city on an island, but they would not be
accurate descriptions of villages built on the mainland. The full context of Ezekiel's prophecy
against Tyre shows that he was predicting the destruction of the island stronghold and not just
the mainland part. Since Nebuchadnezzar failed even to capture the island city, much less
destroy it, the only reasonable conclusion is that this prophecy failed.

Ezekiel's Many Nations...

I received The Skeptical Review today and have already read about the Tyre prophecy. There
is additional support for your position that many nations fought with Nebby. The 25th chapter
of Ezekiel has a tirade against Ammon, Moab, and Edom for their participation against Judah,
supporting your citing of 2 Kings 24:2. It was quite common in those days for the winners to
put the defeated in their armies (Assyria did it, as did the Romans). Even Tyre had
mercenaries in her forces (Ezek. 27:10). Nazi Germany copied this custom during WW2.
After overrunning Europe she instituted the Waffen SS. Europeans were invited to join these
forces to help fight communism in the East. Although fighting on the Russian front may have
been undesirable, it was far better than a concentration camp. Many joined. Anyway, I am
going to do some searching for just how prevalent this custom was in the time period in
question. I will let you know what I find, if its worth telling. You did well enough as it was; I
just like to have "overkill" of information when it comes to debating inerrantists. I am sure
you agree.

Although it's not an exact parallel, Desert Storm was "many nations" under one command.

(David Mooney, 1203 Mooney Way SW, Supply, NC 28462; e-mail, atheist89- @hotmail.com)

EDITOR'S NOTE: It does indeed require "overkill" at times in order to convince biblical
inerrantists that their position is indefensible. Even overkill will not convince most of them,
but it does at least silence some of them.

Encouraging Prison Inmates...

Just read the September/October issue. Excellent as usual. I saw an address for a freethinker
lady in a Texas prison. It so happens I already correspond with another lady in the same
prison. I have told them about each other and hope they will meet if they haven't already. I
take it upon myself to contact anyone listed in your mailbag section who seems to want
further contact with freethinkers, and provide them with a list of organization addresses,
"nontracts" from FFRF, etc. Iencourage all other freethinker readers of TSR to do likewise,
especially for inmates who obviously are more limited in their ability to connect than most
others. Since Dennis McKinsey has ceased publication of his Biblical Errancy, it appears it is
up to you to hold down the fort on the inerrancy issues. Keep up the great work. Should you

Volume 1990 - 2002 Issue


Page 1459 of 2049
Skeptical Review Edited by Farrell Till
need to contact me or the Alabama Freethought Association, temyb@gte.net or here
http://alfreethought.home.mindspring.com/

(Temy R. Beal, assistant director, Alabama Freethought Association, editor, *The Alabama
Freethinker,* P.O. Box 447, Ariton, AL 36311-0447: e-mail address noted above)

EDITOR'S NOTE: The date of this letter (October 1998) shows the difficulty that I have in
finding space to publish all letters that deserve to be read. This delay in publication will
perhaps explain to the writer of the next letter why some letters never appear in this column.

Witty, Clever, Trenchant Letters...

Although you haven't seen fit to publish any of the well considered, witty, clever, trenchant
letters that I've sent you over the years (the last one delineating why heaven is hotter than
hell), you nevertheless do a superb job in keeping the armies of the night at bay. As a result, I
cannot help but renew my subscription for yet another year, even as I bind up the wounds of a
lacerated ego.

(John Carver, 10517 Oklahoma Avenue, Chatsworth, CA 91311)

EDITOR'S NOTE: I appreciate Mr. Carver's sense of humor, and I hope that my editorial
comment above will explain why some letters are never published. For the same reason, I
cannot publish all articles that are submitted to TSR. Just trying to read everything requires a
big investment of time.

Who's to Blame...

I have enjoyed receiving your newsletter, since I am at this time thinking seriously about the
issue of Biblical inerrancy, and you always make me think. Concerning the article "Who's to
Blame?" in the January/February 1999 issue of TSR, I have a few comments in reply.

First, an inerrantist could turn the same argument around against atheism. Look at all the
officially atheist countries in this century, from the late Soviet Union to Cuba and Vietnam,
which have tortured and murdered millions in the name of Marxism or "Scientific
Materialism" or whatever. Western atheists such as Corliss Lamont have defended all of
them, blaming "excesses" such as show trials, persecution of all who dare to criticize the
glorious leader, and the Gulag, to outside interferences from foreigners. One of Joseph
McCabe's Blue Books, Atheist Russia Shakes the World (written about 1941, I believe),
claims "...the `Godless Bolsheviks,' as the Catholic press still called them only six months
ago, have created the greatest civilization of our time." Such statements would be laughable in
retrospect, if it were not so tragic that totalitarian regimes were defended by atheists so long
as they claimed to be atheist.

Second, the Old Testament prohibition against witchcraft (Ex. 22:18) would be reasonable if
one believed that some people really had power to harm others using supernatural means. If
one does not believe in the supernatural, then that command is another example of tragic

Volume 1990 - 2002 Issue


Page 1460 of 2049
Skeptical Review Edited by Farrell Till
consequences resulting from irrational beliefs. And of course the burden of proof is on those
who claim the supernatural exists.

Third, let's just admit that all human beings are capable of horrible actions against their fellow
human beings, whether due to "original sin" or primitive residues from our evolutionary past.
Their professed belief or philosophy seems to make no difference in their ability to do evil.

Fourth, in your reply to Gleason Archer (July/August 98), you criticized his use of the fallacy
of undesirable consequences. I am surprised to see you using this argument for your beliefs.
The articles that make the most sense to me are those that argue things such as scientific
problems in Biblical statements, unfulfilled prophecy, historical events that should be
mentioned in contemporary records but aren't, etc.

I would appreciate your reply. I look forward to many interesting and provoking articles in the
future.

(Robert S. Jackson, 159 Gaddis Road NW, Cartersville, GA 30120; e-mail,


rjac332730@aol.com)

EDITOR'S NOTE: In my reply to Roger Hutchinson in the May/June issue, I have addressed
Mr. Jackson's first point, so the only additional comment that I consider necessary concerns
Jackson's reference to the millions who were tortured or killed "in the name of Marxism." I
think it is an oversimplification to assume that all of these atrocities were committed "in the
name of Marxism," because I suspect that the leaders of these nations were just like other
tyrants, whose chief motivation was to secure their power hold on their respective countries,
and so their religious beliefs or lack of the same were probably incidental to their actions.
Furthermore, Marxism was primarily a political/economic philosophy, and so crimes
committed "in the name of Marxism" would have been political crimes and certainly not
crimes motivated by lack of religious beliefs.

As for Joseph McCabe's statement about the "Godless Bolsheviks," I feel no need to defend
what McCabe may have thought. I have a lot of respect for Joseph McCabe's works. As an ex-
priest turned skeptic, he wrote some books that were devastating to the Christian position, but
he died in 1955 and did not have an opportunity to examine communism "in retrospect," as
we have today, because its political and economic failures were not as obvious at that time as
they were later. People living when McCabe did could evaluate what communism had done
for the Soviet Union only in terms of how social conditions were under the communist regime
compared to life under czarist Russia. An impartial comparison at the time would surely have
concluded that they were no worse, if not much better. At any rate, I addressed the atrocities
of communist dictators in my reply to Hutchinson. If Mr. Jackson thinks that there is evidence
that such leaders as Joseph Stalin were motivated by atheistic beliefs to commit their
atrocities, I would like to see it. I am certainly not arguing that no atrocity was ever
committed as a direct result of a tyrant's atheism; I am arguing only that Christians rather
facilely associate crimes of nontheists with their atheism. If such reasoning is valid, then it
would be just as valid to argue that Adolf Hitler's atrocities resulted from his Catholic beliefs
or that the vast majority of prison inmates, who are theistic in their beliefs, committed their
crimes because of their belief in theism, and that would be patently fallacious reasoning.

Volume 1990 - 2002 Issue


Page 1461 of 2049
Skeptical Review Edited by Farrell Till
Since Jackson and I seem not to disagree on the irrationality of belief in witchcraft, no
comment on his second point is necessary. His third point was that human beings seem
capable of "horrible actions" regardless of their philosophical beliefs, and I certainly agree.
However, that does not remove the fact that an inordinate number of atrocious acts have been
religiously motivated, especially by belief in Christianity. To point to atrocities committed by
political leaders who seemed to have no religious beliefs and then claim that their lack of faith
led them to commit their crimes is a leap in logic that is hardly justified. However, when
decrees issued by popes and other religious leaders encouraged and authorized the purging of
"heretics," there can't be much doubt that whatever crimes followed in the wake of these
decrees were religiously motivated. That is the essential difference in inquisitions and other
religious persecutions and the atrocities committed by communist regimes.

I am puzzled by Jackson's surprise that I had pointed out the fallacy of undesirable
consequences in my reply to Gleason Archer, because a logical fallacy is a flaw in one's
reasoning that should be exposed in debating. Although exposure of scientific problems in
Biblical statements, unfulfilled prophecies, and silence in contemporary records of events that
should have been mentioned are certainly strong arguments against biblical inerrancy--after
all that's the very reason I so often refer to such problems in the Bible-- that is certainly no
reason not to point out logical fallacies in an opponent's line of reasoning.

Disunity in Freethought Groups...

Enclosed is a check for another year of TSR. I was wondering, how about 12 issues a year for
TSR? Since McKinsey's Biblical Errancy has bit the dust, there is a market for it. I would be
more than willing to pay $24 per year for such a publication. It would sure beat $25-30 for
each and every atheist group's asking price of membership and the denominational backbiting
it brings. In their search for new subscribers they tend to be very protective of their turf but
cannibalistic of other like organizations. Talk about your partisan politics.

The problem is that there aren't enough meals for each to feel satisfied or even able to do
more than pay legal fees to some lost cause rather than direct aggressive reeducation
programs at the general public. If all would bury the hatchet and combine forces and
resources, much could be done. Then again, 13 years of being out of the atheist closet and
having seen several monthly publications, I went away thinking that the few of which I was a
member were the only groups of their kind. I have since come to the conclusion that no matter
what two organizations I had been a member of, I would have had the same response. I would
never have been introduced to other freethought publications.

We criticize Christians for being so fragmented, but the freethought community is no better
and possibly even worse. And what do free thinkers have to show for it? I can name but have
never seen several of the reported atheists in Ohio. That number may be smaller or larger, but
how could I know? Where is the support system? Oh, I have to pay a fee of $25 to maybe see
who my fellow free thinkers are! Sorry, I didn't know the rules even to know that there were
others who wished me well. I'm of a substantial belief that I'm the only atheist that resides in
Seneca County. I have responded to various TSR mailbag writers, thanking them for a good
letter, but none have ever responded. Locally, I no longer have letters to the editors published.
I believe isolation is the proper term. No one else was near that I could use to hand off my

Volume 1990 - 2002 Issue


Page 1462 of 2049
Skeptical Review Edited by Farrell Till
information in order to circumvent the local paper's censorship. Where can I get help? Oh, I
can, but it may well cost me over $100 or more a year, and I'd still be no further ahead in my
search.

So without dynamic changes, even TSR may well be shelved. You may die or ill health may
bring it to a halt. Your voice will be lost. Sure, we can download all this stuff, but for how
long? Five years, maybe ten? The clock is ticking, but are we? Sidebar: I have been a member
of both AA and FFRF, and not once did I garner any information about members and those
geographically close to me. There seems to be no forum in any such organization. After all, if
group xx's information might be used to allow group zz a crack at their membership list, they
may "steal" them away, but to criticize leaves one open to personal attacks and
misinformation about one who is simply wanting to help the freethought cause. I'm not a
joiner; I hope I can be a doer. Within the freethought community it seems that free speaking is
heresy, and we think Christians are weird?

(Douglas L. Smith, P. O. Box 513, Tiffin, OH 44883; e-mail, imtruth@yahoo.com)

EDITOR'S NOTE: There is more to Mr. Smith's letter, which discussed advantages that
organized religion seems to have over freethought groups. I will publish it in the next issue.

I agree with many of his observations about disunity in freethought organizations, and I have
heard other skeptics express similar views. I have found it frustrating to read a letter in
another freethought publication in which the writer expressed concerns that I thought I could
help with by offering the writer the free subscription to TSR, but I couldn't contact the person,
because only initials and the name of a town were used to identify the writer. From the
beginning of TSR, I have published the names and addresses of all authors and letter writers
so that readers who wanted to contact them would know how to do so. I would think that this
is a minimum type of service that freethought publications could offer its readers. In doing
this, I have never worried about other organizations and editors stealing my subscribers,
because if TSR can't survive competition, maybe it should not be published in the first place.
Apparently, this policy hasn't hurt the paper, because the number of subscribers continues to
grow and has reached the point of becoming almost too much for me to manage myself.

So 12 issues per year? Bite your tongue, Mr. Smith! Six is almost more than I can manage,
and sometimes I find myself wondering why I ever changed from a quarterly format.

Natural-Born Skeptics...

The Skeptical Review was worth receiving this year just for "Who's to Blame?" in the
January/February issue, and the exchange between you and Roger Hutchinson in the
May/June issue. This illustrates the problem I've had with local Christians whenever I try to
show them the dark side of their faith. Even when I point out Bible passages where their god
and savior encourage believers to be bigoted and violent, they fold their arms and close their
ears. When I press further, they retreat into the not-real-Christians dodge and won't be
dissuaded from it. The next time I get into such a debate, I'll send my "opponents" copies of
the above articles; they can dismiss me because I have never been a Christian (although I
came scarily close), but they won't be able to dismiss a former minister so easily.

Volume 1990 - 2002 Issue


Page 1463 of 2049
Skeptical Review Edited by Farrell Till
I have to comment on two points Mr. Hutchinson made in his article. He wrote, "Perhaps the
teaching of evolution in the schools, with its mantra, Survival of the Fittest, accounts for the
Paul Hills of the world." What! How non sequitur can you get? Paul Hill is a Protestant
fundamentalist and has probably been one all his life (correct me if I'm mistaken about this).
He was also well known as a hardcore "pro-life" fanatic before his shooting of Britton and
Barrett. Even if he *was* taught the evolution theory, he probably never believed it, and it
had nothing to do with his picking up his gun. And I wager that same goes for all the other
"Paul Hills of the world."

Mr. Hutchinson also made a point that I agree with. He said, "People are natural born skeptics
who reject the Bible." Exactly, my dear sir! And when more people get over their early
conditioning and return to their natural skeptical state, the better off we all will be.

(Andrew C. Jones, 5399 Chaison Road, Gladstone, MI 49837-8812)

EDITOR'S NOTE: Hutchinson's jab at evolution is a typical fundamentalist tactic. My


experience with them on the internet has been that when they are at a loss to defend the
absurdity of their beliefs, they will lash out with something like, "My faith in the Bible is no
more ridiculous than your faith in evolution." It's fun to watch them try to defend the
existence of God with appeals to science: "Scientists have proven that the universe had a
beginning with the big bang," or "The second law of thermodynamics would prevent
evolution from occurring," or "Science has proven that something cannot come from
nothing," etc., etc., etc. These statements are not just inaccurate; they are inconsistent with the
fundamentalist position that the vast community of scientists worldwide are wrong in teaching
that evolution accounts for the diversity of life. After all, if scientists can't be trusted in what
they say about evolution, how can they be trusted in what they say about the big bang or the
laws of thermodynamics? Of course, consistency has never been a characteristic of Bible
believers.

Volume 1990 - 2002 Issue


Page 1464 of 2049
Skeptical Review Edited by Farrell Till

Skeptical Review
Volume 10, Number 5
September/October 1999
Farrell Till, editor

• "Lest They Die..."


The editor examines Old Testament warnings of death that would be inflicted on those
who dared to invade the sacred domain of the tabernacle where only priests were
permitted.
• "The Pentateuch: Not Wholly Moses or Even Partially"
Stephen Van Eck discusses clues in the Pentateuch that belie the traditional claim that
Moses wrote these books.
• "Grasshoppers, Locusts, and Other Four-Legged Creatures"
Gavin Steingo analyzes Leviticus 11:20-23 to show that the author erred in his
understanding of insect anatomy
• "Probability Applied to Biblical Inerrancy"
Kirk Mitchell applied the laws of probability to the Bible to show how much the odds
are against the fundamentalist claim that the Bible in its entirety is inerrant.
• "Biblical Discrepancies Explained"
Michael Bradford replies to the editor's May/June claim of anachronisms in the Bible.
• "Beware of Bible Fundamentalists 'Quoting' Sources"
Farrell Till shows that Michael Bradford has depended on unreliable sources to defend
the Bible against claims of anachronism.
• "Milosevic and Moses: Innocent or Guilty"
Sol Abrams compares the war crimes of Slobodan Milosevic to those attributed to
Moses in the Old Testament.
• "The Failure of Isaiah's Prophetic Rantings"
Farrell Till compares Isaiah's prophecy against Tyre to Ezekiel's to show that they
were inconsistent in substance and that both failed to materialize.
• "From the Mailbag"
The editor and readers exchange views on several biblical topics.

Volume 1990 - 2002 Issue


Page 1465 of 2049
Skeptical Review Edited by Farrell Till

Lest They Die....

Throughout human history ruling establishments have used religion to exploit ignorance and
superstition in order to keep the general population of tribes and nations under control. On a
small scale, we see this tactic at work today when presidents, governors, and other elected
officials call for a "day of prayer" or stand in legislative chambers in times of national tragedy
or crisis to appeal for a return to "biblical values" or the passage of laws to permit prayers in
school or the posting of the 10 commandments. More often than not, the personal lives of
those who make such appeals are far from exemplary, but that doesn't matter. They know that
this kind of rhetoric will mean votes in the next election, and so they exploit public ignorance
for all the political gain they can wring out of it.

When ignorance and superstition were more deeply ingrained than they are now--if one can
even imagine such a time--religious manipulation of the public by ruling classes was even
more overt. In biblical times, for example, priests and other officials used the threat of divine
wrath to keep the people in line. According to the book of Exodus, the so-called "law of
Moses" was revealed to Moses by face-to-face chats that Yahweh had with him on Mt. Sinai,
and Moses then passed the law along to the Israelites. A study of the Pentateuch shows that
the threat of Yahwistic wrath was a primary tool that the writer of this law used to ensure
general compliance with its complex maze of commandments and ceremonial requirements.

The center of Israelite worship during their wilderness-wandering years was the tabernacle, a
tent that they carried with them from camp to camp. As the story is told in the Bible, Yahweh,
the creator of the universe, who could have included in his divine revelation practical and
useful information that scientists have taken centuries to discover, chose instead to spend 15
chapters in the book of Exodus giving minute details about how to build this tent and make
the sacred furnishings that would be used inside it for various priestly ceremonies. When the
tabernacle was completed, the book of Exodus closed, but the detailed instructions continued
as Yahweh gave intricate commands about who was to do what in the disassembling and
transportation of the tent to the next camp site. When it was time to travel on, the "sons of
Aaron" (priests) were to prepare all of the furnishings in the tabernacle for transportation by
covering them with "sealskins." The Kohathite branch of the Levites would then carry the
furnishings on to the next camp site (Num. 4:4-15), but they were strictly warned not to touch
the tabernacle itself "lest they die" (v:15). They were also warned that they should never even
watch the priests covering the sacred vessels.

Then Yahweh spoke to Moses and Aaron, saying, "Do not cut off the tribe of the families of
the Kohathites from among the Levites; but do this in regard to them, that they may live and
not die when they approach the most holy things: Aaron and his sons shall go in and appoint
each of them to his service and task. But they the Kohathites shall not go in to watch while the
holy things are being covered, lest they die" (Num. 4:17-20).

Volume 1990 - 2002 Issue


Page 1466 of 2049
Skeptical Review Edited by Farrell Till
This was just the first of various warnings that ordinary people were never to intrude on the
sacred domain of the priests. Yahweh had commanded that the tabernacle be set up in the
middle of the encampments, with detailed instructions about where each tribe was to pitch its
tents with reference to the tabernacle 5(Num. 2). In selecting Aaron and his sons to officiate
in the priestly ceremonies conducted in the tabernacle, Yahweh had said, "So you shall
appoint Aaron and his sons, and they shall attend to their priesthood in the tabernacle; but the
outsider who comes near shall be put to death" (Num. 3:10). Dire warnings about obeying the
command for others to keep a safe distance from the tabernacle had already been given.

(Y)ou shall appoint the Levites over the tabernacle of Testimony, over all its furnishings, and
over all things that belong to it; they shall carry the tabernacle and all its furnishings; they
shall attend to it and camp around the tabernacle. And when the tabernacle is to go forward,
the Levites shall take it down; and when the tabernacle is to be set up, the Levites shall set it
up. The outsider who comes near shall be put to death. The children of Israel shall pitch their
tents, everyone by his own camp, every one by his own standard, according to their armies;
but the Levites shall camp around the tabernacle of Testimony, that there may be no wrath on
the congregation of the children of Israel; and the Levites shall keep charge of the tabernacle
of the Testimony" (Num. 1:50-53).

Repeated warnings were given to the Israelites that they were never to intrude on the sacred
domain of the Aaronic priests. They and they alone were to have access to the inner sanctuary
of the tabernacle. Their Levite brothers could attend to the needs of the priests and the
tabernacles, "but they shall not come near the articles of the sanctuary and the altar, lest they
die" (Num. 18:3). The Levites who weren't priests could join with the priests to attend to "the
work of the tabernacle, but an outsider shall not come near you" (v:4). Yahweh described the
nonpriestly Levites as "a gift" to the priests "to do the work of the tabernacle," but only the
priests could attend to the priesthood "for everything at the altar and behind the veil," but "the
outsider who comes near shall be put to death" (v:7). Within the space of just seven short
verses, the infinitely kind, loving Yahweh warned three times that only the priests were to
attend to duties inside the tabernacles and that others who dared to intrude on this holy
sanctuary would be put to death.

Biblicists, of course, believe that there was a divine purpose in these repetitious warnings of
death to those who would violate the sanctity of a tent serving as a sort of portable temple, a
purpose that surpasses the ability of mere humans to understand, but it all smacks too much of
primitive taboo and superstition for rational people to believe that this was the real reason
why the interior of the tabernacle was off limits to everyone but the priests. Let's just suppose
that there is a grain of truth in the wilderness-wandering tales and that the Israelites did
actually have a sacred tent that they carried with them in their nomadic jaunts through the
Sinai desert. What better way could ancient priests have protected their livelihood than by
instilling in the people a fear that those who intruded on their sacred turf did so at the risk of
their own lives? The Levitical code made specific provisions for how much the priests were
entitled to take from each animal and meal sacrifice that was brought to the tabernacle, so it
was to their economic benefit to keep the people coming with their daily offerings. If the
supply had ever dried up, the priests would have had to go to work herding sheep, making
pottery, or whatever else people did then to make a living.

Volume 1990 - 2002 Issue


Page 1467 of 2049
Skeptical Review Edited by Farrell Till
Maintaining a public awe and fear of what went on inside a holy sanctuary would have been
an easy way to keep the tabernacle economy stable, and what better way to do this than to
make the people afraid even to go near the sanctuary? After all, if the people had seen too
much of censers, candlesticks, lavers, showbread, and such like, the holy sanctuary would
surely have lost much of its mystique, but as long as people thought that this was a place so
holy that outsiders who intruded on it would be struck dead by divine wrath, the priestly hold
on the people was relatively secure.

There are stories in the Bible that undoubtedly developed from the general belief that
touching sacred vessels from the tabernacle would bring divine wrath on outsiders. First
Samuel 6 relates a tale about the calamities that befell the Philistines after they captured in
battle the ark of the covenant, in which the tablets containing the ten commandments were
kept. To end all of their misfortunes, the Philistines put the ark onto an ox-drawn cart and sent
it back to Israel. When the unguided oxen came to the town of Bethshemesh, curiosity
overcame religious taboo, and some of the townsmen looked inside the ark. Yahweh killed
57,070 men because of their violation of the sacred ark. As the story is told, Yahweh had
merely inflicted the Philistines with "tumors" because of their contact with the ark, so why
Yahweh treated the Israelites more severely than Philistines for touching the ark is anyone's
guess. The Hebrew god Yahweh can be accused of many things, but consistency is not one of
them.

Second Samuel 6 relates the story of a man who was killed for touching the ark when it was
being transported to Jerusalem. When oxen pulling the cart stumbled, a man named Uzzah
reached out to steady the ark, and when he touched it, Yahweh struck him dead "for his error"
(v:7). David, who was on the scene, was so stricken by fear that he canceled plans to take the
ark to Jerusalem and had it removed instead to the house of a man named Obededom.

Tales such as these undoubtedly developed from sincere beliefs that Yahweh had pronounced
death warnings on those who would intrude into matters that had been delegated to priests.
Instilling such fear was an excellent way to control people. The same tactic was used to scare
people into toeing the line in general matters that didn't involve tabernacle ceremonies, but
that's another article for another time.

The Pentateuch: Not Wholly Moses or Even


Partially
Stephen Van Eck

The religious conceptions of ordinary believers exist on the level of fairy-tale notions
inculcated in childhood, which resolutely persist into adulthood. Among these notions is the

Volume 1990 - 2002 Issue


Page 1468 of 2049
Skeptical Review Edited by Farrell Till
common belief that Moses wrote the first five books of the Bible. This view was rejected long
ago by such Jewish scholars as Aben-Ezra and Spinoza, and Maimonides doubted much of
Genesis. Most Bible scholars today, with the exception of hard-core zealots, also reject the
traditional view. They are pretty much forced to, since abundant anachronisms and other
discrepancies disprove Mosaic authorship. Moses, in fact, is quite possibly a fictional hero
concocted by the priests of the late monarchical period, but whether he existed or not, only an
obstinate disregard of facts can allow the fairy-tale notion of Mosaic authorship to survive
even elementary scholarship.

For starters, it must be acknowledged that no Egyptian record exists of Moses or any of the
remarkable things he allegedly did. Herodotus, who traveled extensively in Egypt gathering
information, made no mention of him. Christians, of course, need to assert that the resentful
and humiliated Egyptians had all references to Moses stricken from the official record, but
this is pure speculation unsupported by anything, not even the Bible. In fact, it contrasts with
the claim in Exodus 11:3, which says that "Moses himself was a man of great importance in
the land of Egypt, in the sight of Pharaoh's officials and in the sight of the people." His
miraculous works, even if not part of the official record, might have been preserved in
unofficial ones or in folk memory, either of which might have reemerged eventually after the
events were safely in the past. Indeed, God would have done well to have ensured that some
independent record existed to confirm Exodus--unless he somehow wanted to make faith as
problematic as possible.

Christians, however, are not persuaded by the witness (or lack thereof) of unbelievers, so we
will return to the Bible. Oddly enough, Moses scarcely figures at all in the historical books.
These include Judges, 1 and 2 Samuel, and Kings and cover approximately 500 years. Of the
very few references to him, mostly incidental, none can be shown to have been part of any
text prior to the time of Josiah. A man of Moses' stature would, if real, have figured
prominently in everything that followed his alleged times. He would have frequently been
cited by the religious leadership as their principal authority. He and his entire amazing story
could not possibly have become so utterly "forgotten," no matter how much Yahwism
allegedly fell into desuetude. So he becomes conspicuous by his absence from the pages of
postexodus Jewish history.

The name of Moses does not even appear in the Bible as that of any other person. After the
exile, though, the name began to be given to Jewish boys and is common to this day as
Moishe. (The same is true for all the big names of Genesis, such as Abraham, Isaac, and
Jacob.) One can only surmise that Jews were unaware of Moses or his books until postexilic
times.

Argument from silence has its place, but the best evidence against the Mosaic authorship is
contained in the Pentateuch itself, as well as the historical books of the Old Testament. The
former contains anachronistic references impossible to be the work of Moses, while the latter
describe religious practices among the Jews that demonstrate a complete ignorance of the Law
of Moses, even among the priests themselves. Let's examine the latter first.

Solomon had the first temple constructed, as described in 1 Kings 6-7, but neither he nor the
priests seemed to have any awareness of the second commandment's prohibition against

Volume 1990 - 2002 Issue


Page 1469 of 2049
Skeptical Review Edited by Farrell Till
making "any likeness of anything that is in heaven above or that is on the earth beneath" (Ex.
20:4), for the text describes images of oxen, lions, flowers, palm trees, and cherubim (6:23-
36; 7:23-50) without any condemnation. Zealous religionists going to the trouble and expense
of constructing the central temple for the entire faith would have made certain to build it in
conformity with their most important precepts. That these precepts did not yet exist is the
likeliest explanation of why they were ignored.

The fourth commandment, like the second, seems to have been unknown at a time long after it
had supposedly been given. The sabbath, as a seventh-day rest for devotional, is not
mentioned in any pre-Deuteronomic text and cannot, therefore, have been important, to say
the least, but after the exile, it had become of paramount importance. Prior to this time, the
only sabbaths the Jews apparently practiced were lunar ones for the new and full moons.
Isaiah (1:14) in the 8th century B. C. denounced lunar festivals, and Hosea (2:11), around the
same time, had God promising to destroy Israel's "new moons and her sabbaths." This must
refer to the lunar sabbath of the full moon, or it would hardly have been condemned.

Likewise, there is no record of Yom Kippur being observed until after the exile, but Leviticus
23:26-32, presumably written by Moses, commanded that this day of atonement be celebrated
yearly on the 10th day of the seventh month as a "holy convocation," but preexilic history is
conspicuously silent about this holy day. It, along with the sabbath and possibly passover (2
Kings 23:21-22), was probably stressed by the Jewish priests of the exile to reinforce their
religion among the people, lest they became assimilated.

Like the temple, skepticism is raised by the ark of the covenant itself. Exodus 37:1-9
described the ark with two prominent images of cherubim. This is the chest supposedly built
to contain the ten commandments, so right off the bat, the Jews chose to disregard the
injunction against graven images! It's more likely that the ten commandments did not exist
when the ark was built, originally as the seat of Yahweh, but when they were written later and
attributed to Moses, the ark retroactively became a receptacle for them.

According to Deuteronomy 31:24-26, the Law of Moses was also placed in the ark to be a
reminder to a future generation destined to become apostate, but the ark disappeared from
history after it was captured by the Philistines and then returned to the Hebrews to put an end
to a streak of bad luck the Philistines had experienced (1 Sam. 6:21; 7:1-2). It was later
returned to Jerusalem (2 Sam. 6:10-17) and then lost to history. It was probably among the
"treasures of the house of Yahweh" carried off by Shishak around 950 B. C. (1 Kings 14:26),
since no reference was made to it again, and it was not part of the restored temple of
Zerubbabel. If Hilkiah in 620 B. C. found "the book of the law in the house of Yahweh" as he
claimed (1 Chron. 34:14), then it cannot have been an actual book written by Moses that he
rather conveniently found. That book of the law had allegedly been put into the ark.

In addition to the historical books, the book of Jeremiah provides us with an important clue.
Reflecting the scarcely noted conflict between prophet and priest, Jeremiah had denied that
God instructed the Israelites about burnt offerings: "Thus says Yahweh of hosts, the God of
Israel: Add your burnt offerings to your sacrifices, and eat the flesh. For in the day that I
brought your ancestors out of the land of Egypt, I did not speak to them or command them
concerning burnt offerings and sacrifices" (7:21-22). Either Jeremiah refused to recognize

Volume 1990 - 2002 Issue


Page 1470 of 2049
Skeptical Review Edited by Farrell Till
Leviticus, making either him or it uncanonical, or (as most Bible scholars now believe)
Leviticus did not then exist! Exilic priests saw to writing it.

The Pentateuch contains telltale items that make Mosaic authorship impossible. Most
prominent are geographic discrepancies such as Genesis 11:28 and 15:7, which referred to
"Ur of the Chaldees." Ur was a Sumerian city during the time of Abraham. It had become a
Babylonia city before the time of Moses, who would have thought of it as such. The
Chaldeans did not enter history until several decades after Moses, and it was more years still
before they became dominant enough in the area to merit referring to it as Chaldea. Either
Shinar (Sumeria) or Babylonia would have been used by Moses to locate Ur rather than
Chaldea.

Genesis (14:14) also refers to the city of Dan, named after the tribe, which was named after
one of Jacob's sons, but it was not named Dan until after the Israelites had taken it over, which
all must admit was after the time of Moses. Judges 18:29 tells us that it was originally called
Laish. Genesis 35:19 identified Ephrath with its later name of Bethlehem, which also could
not have been renamed until after the time of Moses, who died before the Israelites entered
Canaan. Genesis 23:2 identified Kirjath-arba as Hebron, which it wasn't until it was renamed
after the Israelite conquest (Judges 1:10).

Genesis 21:32 and 26:1, 8 refer to the land of the Philistines, and 20:1 to Gerar, one of the
Philistine towns. At the time of Moses, Gerar was a Canaanite town (Gen. 10:19). The
Philistines didn't start colonizing the area until the mid-12th century B. C., so Moses would
have been unaware of them. (Eerdmans Bible Dictionary, 1987, pp. 828-829, is just one of
many reference works that recognize this as the time when the Philistines first entered this
region.) The later compilers of Genesis, however, would have thought of Gerar as a Philistine
town, and by placing these late comers into an earlier time, they betrayed themselves.

In addition to these discrepant geographic references, the use of such phrases as "until this
day" (Gen, 35:20; Deut. 34:6) necessarily entail some passage of time between the events and
the written accounts of them. Likewise, Deuteronomy 34:10 in saying, "(T)here has not arisen
a prophet since in Israel like unto Moses," would be meaningless and absurd if it had been
written by Moses himself. We should also not neglect Numbers 12:3, which stated that Moses
was the meekest guy in the world. If written by a man about himself, it would be boastful,
which is the exact opposite of meek.

Apologists will claim that these passages, like the famous self-obituary of Moses (Deut. 34:5-
7), are incidental additions by a later redactor and don't necessarily discredit the authenticity
of Deuteronomy, but if the original text could be added to or changed in any way, it only
leads to questions about what else was added, what might (unknown to us) have been
changed, and how we can be expected to trust its reliability.

Not as easy to explain away is Genesis 36:31-43: "These are the kings that reigned over
Edom, before there reigned any king over the children of Israel...." This cannot have been
written until some time during the period of Jewish kings, from 200 to 600 years after Moses.
it cannot be explained as a "prophecy," since it was written in the past tense, made no claim of
prognostication, and served no prophetic purpose anyway. Also indicating against the pen of

Volume 1990 - 2002 Issue


Page 1471 of 2049
Skeptical Review Edited by Farrell Till
Moses is the fact that this section is virtually identical with 1 Chronicles 1:43-54, which it
may have even been copied from. This would give a rather late date for Genesis, since the
Chronicles are recognized as postexilic works.

Something else that places Genesis rather late in composition is the story of Noah, who is
referred to in 7:2 as distinguishing between "clean" and "unclean" beasts. This distinction
wasn't made until Deuteronomy and was made for Jews only, which indicates not only a later
date for Genesis but discredits the reliability of the Noah story itself.

Finally, we should observe that the Pentateuch contains not one but several law codes (Ex.
20:1-17; 34:1-28; Deut. 5:6-21), presented in a highly chaotic manner. The very existence of
more than one code is an indication of more than one author and a diverse spread of time.
Moses would have had no need to issue other codes on the heels of a first, especially if it is
taken to be the word of an unchanging, perfect deity.

Given these problems with the text of the Pentateuch, the primary conclusion that it is
fundamentally the work of later authors is inescapable for all but the most obstinate zealot.
Other conclusions about the history of Judaism may be suggested by the same evidence. First
is the suspicion that Yahwism prior to the "reformation" of Josiah (2 Kings 22-23) was not in
decline, as is generally thought, but had actually not yet caught on! Second, when Hilkiah
claimed that he had found "the book of the law of Yahweh," the earliest appearance of
Deuteronomy, he or another priest had composed it at that time and passed it off as some
ancient authority as part of a successful scheme to gain status for Yahwism as the official
state religion. Third, Moses was a fictional character created as a legendary hero for the
people to look up to, the intermediary who brought what was allegedly God's word to Israel,
since its presentation merely by ordinary, contemporary men would not have been
convincing. This suspicion is supported by not only the absence of historical confirmation of
the existence of Moses but also by the apparently borrowed biographical details. Being placed
in a basket sealed with pitch and set adrift on a river is the story of Sargon of Akkad, the first
regional conqueror, who had lived more than a thousand years before Moses. The story of
how he was set adrift in a basket on the River Euphrates was inscribed on an Akkadian Stela.

Sargon, the great king of Akkad, am I. Of my father I know only his name.... Otherwise I know
nothing of him. My father's brother lived in the mountains. My mother was a priestess whom
no man should have known. She brought me into the world secretly.... She took a basket of
reeds, placed me inside it, covered it with pitch and placed me in the River Euphrates. And
the river, without which the land cannot live, carried me through part of my future kingdom.
The river did not rise over me, but carried me high and bore me along to Akki who fetched
water to irrigate the fields. Akki made a gardener of me. In the garden that I cultivated,
Inanna (the great goddess) saw me. She took me to Kish to the court of King Urzabala. There
I called myself Sargon, that is, the rightful king.

This translation of the inscription and a picture of the Stela of Sargon can be found in Behind
the Bible, Fredericton, NB, Canada, 1990, pp. 1-2. In addition to Sargon's account, this story
of the abandoned child who went on to be a great leader occurred elsewhere in mythology.
These coincidences, like the aforementioned discrepancies, strain credulity.

Volume 1990 - 2002 Issue


Page 1472 of 2049
Skeptical Review Edited by Farrell Till
(Stephen Van Eck, RR 1, Box 62, Rushville, PA 18839-9702)

Grasshoppers, Locusts, and Other Four-


Legged Creatures
Gavin Steingo

For much of the information in this discussion of errors in what the Bible teaches about
locusts and grasshoppers, I owe a special thanks to Dan Johnson, Ph. D., who is a senior
scientific researcher in Lethbridge, Canada, and a specialist in the anatomy and physiology of
these insects. He has reviewed my material and helped me with much of the technical
information.

In Leviticus 11:20-23, the Bible's description of locusts and grasshoppers is contrary to


known scientific facts.

All winged swarming things that go upon all fours are a detestable thing unto you. Yet these
may ye eat of all winged swarming things that go upon all fours, which have jointed legs
above their feet, wherewith to leap upon the earth; even these of them may ye eat: the locust
after its kinds, and the bald locust after its kinds, and the cricket after its kinds, and the
grasshopper after its kinds. But all winged swarming things which have four feet, are a
detestable thing unto you."

The first sentence of this passage has caused considerable controversy in the on-going debate
over biblical inerrancy. We know that locusts have six legs, as do all other insects. The word
"insect" is often defined (such as in the World Book Encyclopedia) as a "small six-legged
animal," and so biblical inerrantists have been challenged to explain why the biblical
description of locusts states that they have four legs when they have six. The text in Leviticus
clearly referred to insects as "all winged swarming things that go upon all fours."

Bibliolaters remain undaunted by this damaging description and assert that the verse is no
more than an idiomatic expression. According to J.H. Hertz (referred to in his commentary as
"the chief rabbi of the British Empire"), the phrase to "go upon all fours" cannot be taken to
mean that insects were possessed of only four legs but that the words probably referred to a
method of locomotion and signified only that they "move like quadrupeds" (Pentateuch and
Haftorahs, second edition, Soncino Press, 1960, p. 451).

The Chumash: Stone Edition offers a more technical and scientific explanation: "Hoffman
raises the difficulty that all insects have six legs not four. He explains that they have four legs
that are used for simple walking, while the other two are used for jumping" (Mesorah

Volume 1990 - 2002 Issue


Page 1473 of 2049
Skeptical Review Edited by Farrell Till
Publications, Ltd., 1993, p. 601). This argument will become important later when I examine
a central point in the inerrantist position.

Either way, bibliolaters remain unperturbed and find no reason to think that the writer of
Leviticus believed that locusts have only four legs, but to so think is to evade the problem. I
have yet to encounter an inerrantist who can convincingly explain away the real problem at
hand. Even if we accepted that the expression "go upon all fours" is not saying that locusts
have only four legs but just that they move about on four legs, what can one say about what
the passage in Leviticus says further along in verse 23: "But all winged swarming things
which have four feet are a detestable thing unto you"? Here, without any reference to
movement or motion, the passage described insects as "winged swarming things that have
four feet." It's rather hard to argue that this part of the passage was describing only the way
that insects move about.

Some translations like the King James Version and the Jerusalem Bible, have inserted the
word "other" in verse 23. The statement hence reads, "Even these of them ye may eat: the
locust after its kinds, and the bald locust after its kinds, and the cricket after its kinds, and the
grasshopper after its kinds. But all other winged swarming things which have four feet, are a
detestable thing unto you." The insertion of "other" makes the passage mean that one could
not eat winged swarming things with four feet, except the locust, bald locust, and
grasshopper. The word "other" clearly conveys the belief that locusts have four feet, and so do
other winged swarming things. "Other," however, is not present in the Hebrew text, so
translations that omit it accurately convey the reading of the original Hebrew, but it would be
ridiculous to argue that the expression "all winged swarming things which have four feet,"
coming as it does immediately after the commandment concerning locusts and grasshoppers,
should be seen as a statement completely unrelated to the preceding description of locusts and
grasshoppers. Indeed, with the word "but" beginning the reference to "all winged swarming
things which have four feet," in the context given, it seems entirely appropriate, and even
necessary, to assume that the word "other" was implied. "But" is a conjunction that shows
contrast, so if the statement permitting the eating of locusts was followed by a "but" that
prohibited the eating of "all winged swarming things which have four feet," it's rather obvious
that the writer meant to classify locusts and grasshoppers as "kinds" that belonged to a
category of all winged swarming things with four feet. No doubt, this was why many
translations have inserted the word "other."

Now that we know that "other" was not in the original Hebrew, let's just assume that the
statement "all winged swarming things which have four feet, are a detestable thing unto you"
was intended to stand alone and that it had nothing to do with the previous statement about
locusts. What creature could this sentence be talking about? There is no animal that exists that
is a winged swarming thing with four feet. The distinction had already been made between
these "winged swarming things" and birds, which were discussed in verses 13-20. There is no
"winged swarming" invertebrate with four feet, and there is no "winged swarming thing with
four feet" in any other animal group. The only possibility is the "bat," but the bat had also
been talked about completely separately in Leviticus 11:19. Besides this, the bat can hardly be
said to "swarm," from a Hebrew word that meant movement as in the swarming of "an active
mass of minute animals" (Strong #8318).

Volume 1990 - 2002 Issue


Page 1474 of 2049
Skeptical Review Edited by Farrell Till
There is the ridiculous example of the "flying squirrel," which has been proposed by some
inerrantists as a winged creature that goes about on all fours, but the flap of skin underneath
this animal's arms can hardly be considered a wing. Although this animal jumps really well, it
can hardly be considered a "flying" creature. Not only this, but unlike other squirrels, flying
squirrels are most active at night. They normally don't "fly" in packs and definitely don't
swarm!

So let us assume, as most do, that the phrase "all winged swarming things with four feet" in
verse 23 should have the word "other" and should read: "But all other winged swarming
things which have four feet are a detestable thing unto you." This would be saying that locusts
have four feet. This is not true, but an interesting argument has been used to explain why it
can be considered a true description of insects. Some bibliolaters claim that because the locust
sometimes uses primarily its front four legs to push itself along and its back legs are of
secondary importance when walking, the back legs of a locust were not considered legs or
feet. (This argument was used by Rashi, an 11th-century Jewish commentator). Thus, some
inerrantists quibble that maybe the biblical definitions of "foot" and "leg" were different from
our current definitions. Perhaps the Levitical writer did not call the "hind jumping
appendages" of a locust "legs" with "feet" but counted only the front four appendages. Thus,
by this definition, the Bible would be right, and the locust would have only four feet. Of
course, there is no anatomical blunder if the writer merely had a different definition from us
of "foot" and "leg," but this argument is problematic. The chief problem is that the writer's
definitions of "foot" and "leg" were obviously not different from ours. The passage described
these creatures (insects) as having legs above their feet, wherewith to leap upon the earth"
(v:21). This statement was clearly referring to the hind jumping appendages of the locust,
grasshopper, and cricket and in so doing called them "legs" with "feet." We can ascertain
from this statement that the writer of Leviticus did consider the back legs and feet of a
grasshopper, locust, and cricket to be "legs" and "feet," so the writer's definitions were the
same as ours. Therefore, it is highly probable that the writer truly did think that locusts,
grasshoppers, and crickets have only four legs.

People find it difficult to believe that even a fallible human could have made such an error
about the number of legs on a grasshopper. After all, anyone can pick up a grasshopper and
look at it directly. Grasshoppers, however, are often scarce at certain times of the year, and
this could account for the writer's ignorance. In fact, locusts have been known to be scarce for
several decades, which scarcity may have occurred at the time Leviticus was being written.
This may have been problematic for the Leviticus writer if he had wanted to observe a locust
more carefully.

To support the claim that people can be mistaken about the number of legs on a grasshopper, I
conducted my own study. I did a survey of fourteen and fifteen year olds at a private school in
South Africa. These students have already learned about electrostatics, algebra and Euclidian
geometry, and many aspects of geography. I asked eleven random students how many legs a
grasshopper has. Here are the answers that I received: (1) I don't know... six? (2) Six, it's an
insect. (3) 6 (4) 8 (5) 6... or 4? (6) 6 (7) I don't know. (8) 6 (9) 6 (10) I don't know. (11) 6. So
even some of these relatively educated people were not sure how many legs a grasshopper
has. The most likely explanation why the Leviticus writer erred is that he did not directly
observe a grasshopper before he wrote his description of insects.

Volume 1990 - 2002 Issue


Page 1475 of 2049
Skeptical Review Edited by Farrell Till
We must conclude that the Bible says, without any reasonable doubt, that locusts,
grasshoppers, and crickets have only four legs, but there is still another problem in this
passage that we should consider. Inerrantists obviously believe that the food laws in Leviticus
11 are the Word of God, so they believe that there are "benefits" that one will obtain from
following these dietary laws. The Christian Apologetic Society's Reasons to Believe has even
claimed that the Bible "foresaw" scientific information that could not have been known to
people living at the time that the Bible was being written, and so they argue that God inspired
biblical writers to "see" scientific information that would not be generally known until much
later. Reasons to Believe says that the "control of cancer and heart disease" is "forecasted" in
Leviticus chapters 11-19, so this is just an articulation of the inerrantist view that God made
the distinction between "clean" and "unclean" animals in order to protect "his people" from
health hazards. A frequent example bibliolaters will cite in support of this claim is that eating
insufficiently cooked pork can cause trichinosis, and so God protected the Israelites from this
disease by forbidding the eating of pork.

If there is anything to this argument, then it should follow that health factors were also the
reason why God prohibited the eating of all "unclean" animals, and this is where a serious
problem arises. Eating locusts is clearly a health hazard. Although there is a very specific way
of killing animals in Judaism (that is stated in the Gemera), no such directions were given for
locusts. As Rabbi Lauffer, a specialist in kosher laws in the Israeli organization Ohr
Somayach, said, "Not only do locusts not require ritual slaughter, they may even be eaten
alive." However, eating a live locust could cause infection and certain bacteria, and, although
rare, rickettsia could even infect the consumer. Rickettsia are parasitic microorganisms that
lived in the bodies of some insects and can be transmitted to animals and humans by the bite
of the hosts, or they can be transmitted when the host insects are eaten. The variety of
rickettsia that lives in locusts is not the type that causes typhus and spotted fever, but the
effect of eating them can nevertheless be disconcerting. When eating an uncooked dead
locust, one could be eating bacteria that rapidly multiply in the locust after it dies. This too
could lead to sickness.

This is not to deny the fact that locusts are generally quite nutritious. They have a high protein
content and are low in fat. Rabbi Lauffer has noted that the author of the Arichat Hashulchan
points out that locusts were never really considered a "delicacy" but rather a food for the
impoverished. Eating locusts, then, could provide a source of food in the desert or during a
famine, but we must not forget that eating uncooked locusts alive or dead can also cause
illness. This fact doesn't lend much support to the inerrantist claim that God instituted dietary
laws to protect his people from health hazards.

Of course, it is only one interpretation that the food laws in Leviticus were intended to have
health benefits. For example, there are some ardent scholars who believe that the food laws in
Leviticus were given for purely spiritual reasons. That is to say that the food laws were not
intended to have any physical benefits but were laws given for spiritual benefits that mere
humans are unable to understand, and so we should follow them because they are "God's
Word." I still believe, however, that it was necessary to have discussed the health hazards that
are posed by eating locusts, because so many bibliolaters believe that the food laws in
Leviticus were intended to have physical benefits like the prevention of cancer, heart disease,
and other illnesses.

Volume 1990 - 2002 Issue


Page 1476 of 2049
Skeptical Review Edited by Farrell Till
In "Kosher Insects," Isman and Cohen said, "That the rationale for the dietary restrictions lies
in the area of ancient hygiene and health regulations is a common misconception, having no
basis at all in the Biblical text" (American Entomologist, Summer 1995, p. 101). Some
disagree with this; others agree. Still others wonder why a law, presumably divine in origin,
was given to permit the eating of a few kinds of insects but forbid the eating of all others?
Bodenheimer has suggested that permitting the consumption of locusts "is nothing more than
a codification of a habit existing since the oldest times among the nomads of the Middle East,
which... has lasted down to our day" ("Insects as Human Food," The Ecology of Man, 1951).
Isman and Cohen acknowledged Bodenheimer's studies in saying, "Bodenheimer provides
numerous historical references to the human consumption of locusts; the practice continues
today in several tropical regions" (p. 102). The Food Insects Newsletter mentioned many
cultures that eat locusts today (July 1989). A strange thing about this practice is the fact that
many cultures that eat very few species of insects do eat orthopterans (e. g., locusts and
grasshoppers). The web page of the Food Insects Newsletter has a large picture of a
grasshopper on it, this and no other insect, so it seems that the consumption of orthopterans in
particular was (and still is) a widely practiced tradition in many cultures.

On its internet site, The Food Insects Newsletter republished an article by David Madsen that
told of the discovery of an unusual horde of grasshoppers:

In the spring of 1985, "millions" of grasshoppers (the migratory grasshopper, Melanoplus


sanguinipes) were found lying along the eastern shore of the Great Salt Lake. Madsen, state
archaeologist in the Antiquities Section of Utah's Division of State History, says, "enormous
numbers of the insects had flown or been blown into the salt water and had subsequently been
washed up, leaving neat rows of salted and sun-dried grasshoppers stretched for miles along
the beach" (July 1989: Vol 2, Issue #2).

The hoppers, coated with a thin veneer of sand, were in as many as five rows in some places,
with the widest rows ranging up to more than six feet in width and nine inches thick and
containing up to 10,000 grasshoppers per foot.

A year earlier, while digging in Lakeside Cave, which is at the western edge of the Great Salt
Lake, Madsen and co-workers had discovered thousands (even estimated millions) of
grasshopper fragments in the various strata of the cave floor. The hopper fragments, in a
matrix of sand, were also found in the majority of the samples of dried human feces found in
the cave. The connection between beach and cave was obvious. Lakeside Cave had been
visited by Great Basin hunter-gatherers intermittently for the past 5,000 years. It served only
as a temporary base because it is far from fresh water. Obviously, the cave was used as a
winnowing site for removing sand from the grasshoppers that were scooped up at the beach
and most of which were then hauled elsewhere. The same article said, "Madsen also
investigated the rate of return per unit of effort expended in collecting Mormon crickets
(Anabrus simplex), another food of early Native Americans.

From time to time, the Food Insects Newsletter even publishes insect recipes. One recipe
recommended ingredients of about 1000 grasshoppers (the younger the better); 1/2 cup chili
sauce; a pinch of salt; garlic; onion; 1 lemon; 1 cup of guacamole; and 6 tortillas. The
newsletter also said that recipes for crickets were now available and would be featured in a

Volume 1990 - 2002 Issue


Page 1477 of 2049
Skeptical Review Edited by Farrell Till
later issue. I have yet to see whether grasshoppers simply taste better than other insects, but it
does seem that, although many other insects were also eaten by various cultures in the past,
orthopterans were an obvious favorite. Isman and Cohen "favor another ecological
explanation" for why the Israelites chose to include locusts in their diet. They tasted better and
had become a dietary favorite of the Israelites before the writing of Leviticus, so as
Bodenheimer suggested, the classification of orthopterans as "clean" food was just a
codification of a longstanding dietary habit. This accounts for what seems like inconsistencies
to us in the dietary code, and "health benefits" or speculative "spiritual benefits" that surpass
human understanding had nothing to do with this law.

Various books have been written about Old Testament laws and stories that parallel laws and
stories of earlier literature. Although it is a controversial issue as to whether the Biblical
writers took stories from earlier cultures, many scholars, such as Victor Harold Matthews,
Don C. Benjamin, and Alexander Heidel, have spent their lives studying biblical customs that
paralleled practices in other cultures, so we must at least consider the possibility that the law
prohibiting the consumption of insects except for certain orthopterans was taken from
surrounding cultures. For example, the famous "eye for an eye" (Ex. 21:24) is clearly taken
from the Code of Hammurabi, which stated, "If a citizen blinds an eye of an official, then his
eye is to be blinded" (Article 196). It is very probable that (as Bodenheimer stated) the law to
eat orthopterans was based on common practice of the time, and divine revelation had nothing
to do with it.

To summarize, I have illustrated that the Leviticus writer believed that locusts, grasshoppers,
and crickets have four legs. I have shown that this was not an overly ridiculous mistake to
make. I have also explained that eating locusts, alive or dead but not cooked, can cause health
problems. Thus, I have shown that the food laws of the Bible could not have been put there
purely for health reasons. I have also shown that the eating of orthopterans was a common
practice in the Middle East during biblical times, as well as in various other cultures. To
attribute this dietary habit to divine revelation is stretching imagination a bit far.

(Gavin Steingo, P. O. Box 784496, Sandton 2146, South Africa; e-mail, gostein@iafrica.com)

Probability Applied to Biblical Inerrancy


Kirk Mitchell

Every now and then, creationists and biblical apologists trot out vastly large numbers
concerning the probability that atoms could join spontaneously to form life. Perhaps we could
use the same mathematical tools to answer the question of Bible inerrancy. This will entail a
short lesson in probability, so it is my hope that those of you with mathphobia can stick
around a little longer.

Volume 1990 - 2002 Issue


Page 1478 of 2049
Skeptical Review Edited by Farrell Till
Probability helps us measure "chance," as with the flip of a coin, the roll of dice, or the
dealing of a poker hand. It can also be adapted to events or situations where we are willing to
ascribe a numeric value as to the possibilities that an event is true or false. By definition, the
probability that a specific event will happen equals the number of favorable ways divided by
the number of possible ways. The probability of flipping a coin and coming up with heads is
therefore 1/2 (the one favorable event, heads, divided by the two possible outcomes, heads or
tails). This can be expressed as a decimal (0.5) or as a percentage (50%).

The probability of an event that is certain is 1, while the probability of an event that is
impossible is 0. We can then express an estimate of an event as a percentage between 0% and
100%. If we start adding independent events, things start to get very interesting. An
independent event is one in which the outcome of one event has no effect on the outcome of
the other. To find the probability of a sequence of events (for example, the probability of
flipping two coins and getting two heads) is found by multiplying the probability of each
independent event times the probability of the other. For example, we noticed above that the
chances are 1/2 of getting heads on the toss of a coin, so if two coins are simultaneously
tossed, the probability of getting heads on both coins is 1/2 x 1/2 = 1/4 or one out of four. (If
the two coins are tossed four times, the probability is that two heads would be obtained on at
least one of the tosses.) The probability then of getting all heads with three coins is 1/2 x 1/2 x
1/2 = 1/8 or one in eight.

Now we get to the good part. If we look at each of the explanations that Bible apologists give
us for the inconsistencies, interpretations of prophecies, scientific errors, mistranslations,
copyist errors and others as independent events (an explanation as to whether trees were
created before humans would not affect the outcome of one that explains how many women
went to Jesus's tomb), and assign a numeric value as to what we feel is the probability that
explanation is true, we can calculate the probability that the entire sequence of events is true,
and that the Bible is indeed inerrant.

Our limits on the number we choose has to fall between 0% (after all, if it's a possibility, it
must have a value greater than 0 or impossible) and 100% It obviously can't have a value of
100% unless there is absolute proof that it is true.

We can be charitable, and assign a value of near certainty that each and every one of their
arguments has a probability of 99% of being the correct explanation for each inconsistency. I
am choosing this value even though I realize it is extraordinarily high, mainly for simplifying
calculations later on. I will be using the decimal 0.99 to refer to 99% from this point.

Recall that we can find the probability of a sequence of events merely by multiplying the
probability of each event times the other. In this case, we wish to find the probability that the
Bible is indeed inerrant by multiplying the probability that each explanation of the
inconsistencies has a certainty of 99% (0.99) of being true.

I have no idea of the total number of inconsistencies in the Bible, but a count of those
compiled by Donald Morgan in a file I found on the internet called "Biblical Inconsistencies"
numbered more than 330. By multiplying the number 0.99 times itself 330 times, we come up
with a probability of 3.6%, or 1 chance out of nearly 28 that the Bible is inerrant! If your math

Volume 1990 - 2002 Issue


Page 1479 of 2049
Skeptical Review Edited by Farrell Till
skills haven't atrophied, you might recall that taking a number times itself is the same as
taking it to a certain power or exponent. For example 2 x 2 = 4 would be taking 2 to the
second power or squaring it; 2 x 2 x 2 x 2 would be taking 2 to the fourth power. Many
calculators have an exponent key that will enable you to try these calculations with relative
ease. My calculation was entered as 0.99^330.

If there are 1000 inconsistencies in the Bible that need explanations, then we would enter
0.99^1000 and get an answer of 0.0043% or 1 chance out of 23,164 that the Bible is inerrant!
Obviously, the higher the number of inconsistencies we find, the lower the probability that the
Bible is indeed inerrant.

Now, I chose 0.99 to simplify the math, but if there are two or more "how-it-could-have-
happened" explanations for a particular event, we might assume 100% certainty that at least
one of them is correct, but the probability that one particular one (say of three choices) was
correct, would have to be 33%, or 0.33. So for at least this event, we would have to multiply
our series times 0.33 with drastic effect! (Mind you, the entire series wouldn't consist of 0.33
to whatever power... merely that particular event!)

Because of this, each additional conflicting how-it-could-have-happened scenario that


apologists come up with shatters the probability that the Bible is inerrant. Each additional
interpretation of prophecies does the same. Oddly enough, it doesn't even matter how bizarre
their arguments might be! The more, the better (for us at least)!

Rational men and women are quite accustomed to uncertainty. Those proclaiming inerrancy
will have to present absolute proof of each of their tales if they're to avoid the slippery slopes
of probability. Without those absolute proofs, the rational person when asked "Is the Bible
inerrant?" could only answer, "Probably not."

(Kirk Mitchell, P. O. Box 204, Moscow, KS 67952-0204; e-mail, kmitchel@pld.com)

Biblical Discrepancies Explained


Michael Bradford

I have been receiving The Skeptical Review at no cost this year, and for this courtesy I thank
you. In this note I would simply like to submit some comments to you for your personal
consideration regarding the May/June 1999 issue.

Biblical Anachronisms: The Kings of Israel: Assuming Biblical chronology in the


Pentateuch to be correct, it is reasonable to assume that Moses composed the book shortly
prior to his death, i.e., after the majority of the dynamic events as recorded in the Pentateuch

Volume 1990 - 2002 Issue


Page 1480 of 2049
Skeptical Review Edited by Farrell Till
had transpired. The Israelites lived under the rule of Pharaoh in Egypt, and during the Exodus,
either passed through or around numerous kingdoms on their way to the Promised Land.
Thus, it is certainly no "political" jump for Moses to deduce that at one point in the future,
Israel too might have a "king." Moses would not have to be divinely inspired (although he
may have been) to make a logical deduction this simple.

The Priests: According to The New Strong's Concordance (Thomas Nelson Publishers,
Nashville, 1990), the primary word used for priest in the Old Testament is kthen, meaning one
officiating, priest, or acting priest (Hebrew #3548). The first use of this word in the
Pentateuch appears in Genesis 14:18, where the text mentions Melchizedek king of Salem, the
priest of God Most High. So, according to Bible chronology, there was a priest of the God
Most High approximately 600-700 years prior to the establishment of the Levitical priesthood
under Aaron. Throughout that time interval, as recorded in the Bible, many descendants of
Abraham continued to worship God. Considering the vast number of Jews that supposedly left
during the Exodus, it again is a relatively simple deduction that there was some form of
organized worship in their community. Therefore, it is not unreasonable to assume that, since
at least the time of Melchizedek, there were kthen among the people of Israel who facilitated
organized worship of God.

These brief examples illustrate that simple examination of the Bible in context can frequently
elucidate pseudoerrors, such as your proposed anachronisms.

Still Grasping for Straws: Among Bible "scholars" and "critics" there is an apparent
bimodal distribution regarding assignment of a date of composition to the book of Daniel, i.
e., either the 6th century or 2nd century B.C. Although the finding of fragments of the book of
Daniel among the Dead Sea Scrolls illustrates the relative antiquity of the text, the finding
does not necessarily validate the dating of the book of Daniel to either century B. C.
However, I have found a reference which claims that the Hebrew canon was first translated
into Greek during the Hellenization campaign of Ptolemy II Philadelphus prior to 270 B. C.
Chuck Missler in "The Unexpected King" cites The Encyclopedia Brittanica, Volume 10, p.
642, for this piece of secular history. See (if interested) his article on this topic for minor
details: www.direct.ca/trinity/king.html. I have not verified his reference and therefore cannot
state with certainty that it is correct. However, if the book of Daniel was indeed translated into
Greek for the Septuagint prior to 270 B. C., the "critical" claim that the book of Daniel was
written in the 2nd or 1st century B.C. would be clearly erroneous. Therefore, this reference
might be of interest for you to investigate.

(Michael Bradford, 75 Beaver Street, Waltham, MA 02543; e-mail,


cmnbradford@sprynet.com)

Beware of Bible Fundamentalists


"Quoting" Sources

Volume 1990 - 2002 Issue


Page 1481 of 2049
Skeptical Review Edited by Farrell Till
Farrell Till

With this issue of TSR, we welcome Michael Bradford into the ranks of those who have tried
to defend biblical inerrancy in this publication. In addition to his article above, he has also
submitted one on the issue of the sons of God and the daughters of men, which will probably
be published in the next issue of TSR. I am happy to welcome him, but I was disappointed to
see that he has nothing to offer but more of the same apologetic arguments that have been
repeatedly discredited in past issues. As long as biblical inerrantists continue to recycle these
"arguments," I will gladly publish them so that Christian subscribers can see that defenders of
their holy book have nothing substantial to offer as proof that it was divinely inspired.

Like inerrantists before him, Mr. Bradford has attempted to explain away the biblical
anachronisms that I had identified by simply offering how-it-could-have- been scenarios that
would put these anachronisms into their proper time slots, so before I address any of his
specific "solutions," I will first show that this popular apologetic method he is using is
logically flawed. Inerrantists argue that when a discrepancy is claimed in the Bible, the
problem will be automatically solved if they can just show how the passage could be
interpreted to remove the discrepancy, but this is a fallacious argument. If a discrepancy is
claimed in a written document, there are two possibilities: (1) It is a real discrepancy. (2) It is
only an apparent discrepancy that can be explained. In order to remove a claim of
discrepancy, then, the inerrantist must show an absolutely undeniable solution to the problem
and not just a "possible" solution, for if the biblicist shows only a possible solution, he has
done no more than show a possible solution. Positing only a "possible" explanation, then, will
always leave open the other possibility, which is that a real discrepancy does exist in the text.
Since inerrantists almost always assert with bold certitude that the Bible is the inspired,
inerrant "word of God," they owe us more evidence than just mere possibilities, because it's
possible that the Bible is every bit as errant as we have claimed in this publication. Indeed,
given the human tendency to err, I think I could say that it is very probable that the Bible is
errant, far more probable than the possibility that it is inerrant. Mere possibilities, then, are
inadequate to explain claims of discrepancy that are based on plain language in a biblical text.
An apologist who seeks to resolve such a discrepancy by arbitrarily declaring that the writer
could have meant such and so hasn't gone nearly far enough. He must show that the writer
unequivocally did mean such and so. Otherwise, the possibility of a discrepancy in the Bible
remains unresolved.

Many TSR subscribers also subscribe to my "errancy list" on the internet, so they know that I
have often pointed out on this site that there is no such thing as an unanswerable biblical
discrepancy, because no matter how obvious the discrepancy may be, there will always be an
inerrantist somewhere who will step forward to propose a how-it-could-have-been scenario to
"explain" the discrepancy. A serious problem with this method of "apologetics" is that if it is
in any way a sound method of resolving textual discrepancies, believers in any other holy
book (the Book of Mormon, the Qur'an, the Bhagavad-gita, the Avesta, etc.) must be accorded
the right to apply the same critical method to their sacred writings to "prove" that they are also
inerrant. Otherwise, biblicists who insist that this method can be applied only to the Bible will
be committing a logical fallacy known as special pleading.

Volume 1990 - 2002 Issue


Page 1482 of 2049
Skeptical Review Edited by Farrell Till
I have often challenged biblical inerrantists to cite what they consider to be an undeniable
example of discrepancy in some nonbiblical book so that I can use their same apologetic
methods to show that it really isn't a discrepancy. For some reason, inerrantists have evaded
this challenge like the plague, and I am sure I know why. They know that if they accept the
challenge, I will be able to show that the very apologetic methods they use to defend the Bible
can "disprove" any claim of errancy in other documents. Thus, biblical inerrantists expect us
to believe that a line of reasoning that would prove all written documents to be inerrant is a
sound way to show that the Bible contains no errors. The obvious fact that discrepancies in
writing can exist shows that this popular method of biblical apologetics is logically flawed.

The kings of Israel: Mr. Bradford's application of the how-it-could-have-been method of


apologetics has apparently proven to his satisfaction that no anachronism exists in the
reference to the "kings of Israel" in Genesis 36:31-43, because "Moses," knowing that other
countries had kings, could have guessed that there would one day be kings in the nation of
Israel. The weakness of this "explanation" can be seen when the text in Genesis 36 is
compared to passages where "Moses" referred to kings of Israel with clearly predictive
intentions. In at least two other places, the book of Deuteronomy said that the Israelites would
one day decide to have kings just like all the nations around them (17:14-17; 28:36-38), but in
both places, the language was clearly prophetic, and through the usage of the future tense, the
writer made clear that he knew that kings in Israel hadn't yet existed but would someday. As
Stephen Van Eck pointed out in his article, pages 2-3 in this issue, the language in Genesis
36:31ff is entirely different. It was not predictive but was stated in the past tense, obviously
written from the perspective of someone who knew that kings had reigned over Israel: "And
these are the kings that reigned in the land of Edom, before there reigned any king over the
children of Israel.

Compare this to the clearly predictive references that "Moses" made to Israelite kings: "When
you come to the land which Yahweh your God is giving you, and possess it and dwell in it,
and say, `I will set a king over me like all the nations that are around me,' you may indeed set
over you a king whom Yahweh your God will choose" (Dt. 17:14). This is a statement written
by someone who realized that he needed to present it from the perspective of someone who
knew nothing about Israelite kings at the time but knew by inspiration that someday there
would be such kings. The writer of Genesis 36:31ff carelessly forgot that he was supposed to
be someone who had lived before the era of Israelite kings, and so he slipped up and wrote the
passage from the perspective of one who knew that such kings had existed. This, as Van Eck
pointed out, is just one of many reasons to believe that the Pentateuch was written long after
the time of Moses.

The best that Mr. Bradford has done is present a possible but unlikely scenario to explain the
anachronism in Genesis 36:31, but not having established that his explanation is a fact and not
just a possibility, he has failed to resolve the problem, because the possibility still remains
that this is indeed an anachronism. A failure to establish his could-have-been scenario, then, is
a failure to prove that Genesis 36:31ff is an inerrant text.

In addition to the problem of the Israelite kings, Stephen Van Eck presented several other
reasons why the authorship of the Pentateuch must be dated long after the time of Moses.
When Mr. Bradford's "possible" explanation of an apparent anachronism in Genesis 36:31 is

Volume 1990 - 2002 Issue


Page 1483 of 2049
Skeptical Review Edited by Farrell Till
evaluated in terms of all of the problems that Van Eck identified, Bradford's "possible"
solution becomes unlikely indeed, so this brings us to the matter of Kirk Mitchell's article (p.
7, this issue) that applied probability factors to the biblical inerrancy claim. Even if we should
grant that Mr. Bradford's explanation of this anachronism is anywhere close to being 99%
certain as the reason why a book, which was presumably written centuries before Israel even
became a nation, referred to the kings who had reigned (past tense) over Israel, Mitchell's
article still shows how improbable it is that the Bible is inerrant. Anachronisms are just the tip
of the iceberg in the matter of biblical inerrancy, because there are compelling reasons to
suspect that biblical writers also made mistakes in science, history, geography, chronology,
prophecy, and other areas. Mitchell used 1,000 as an estimate of the number of possible
discrepancies in the Bible, but this number is surely conservative. The index in Gleason
Archer's Encyclopedia of Bible Difficulties has over 2,000 entries in its index of scriptures
that were referred to in discussing "explanations" of biblical discrepancies. John Haley's
Alleged Discrepancies in the Bible has over 2,500. Many possible discrepancies, however,
weren't even mentioned in either one of these apologetic books, so it is reasonable to consider
Mitchell's estimate of 1,000 discrepancies to be on the low side.

Mitchell also applied a factor of 99% certainty to all of the how-it-could-have-been scenarios
that biblicists use to "explain" biblical discrepancies, but a degree of certainty this high for all
alleged discrepancies is generous indeed. I won't go into specific examples, but I have heard
"explanations" of biblical discrepancies to which I wouldn't even assign a probability of one
percent, so if the number of possible discrepancies is increased and the probability of
correctness in the explanations is lowered to a more reasonable percentage, the likelihood that
the Bible is an inerrant book becomes slim indeed. This also is a factor that Mr. Bradford has
working against his faulty apologetic method.

Priests before Aaron and his sons? Bradford has applied the same how-it-could-have-been
reasoning to "explain" why there was no anachronism in Yahweh's warning in Exodus 19:21-
24 that the priests would be killed if they touched Mount Sinai while Yahweh and Moses
were powwowing on the summit, a warning that was given nine chapters before Aaron and
his sons were appointed the first priests of Israel. This is no problem, Bradford thinks,
because Genesis 14:18 referred to Melchizedek, the king of Salem, whom Abraham met
returning from his slaughter of the kings who had taken his nephew Lot captive, so if there
were priests in other tribes and nations, Bradford speculated, maybe the Hebrews also had
priests who were never mentioned in the Bible.

I find serious problems in this "resolution" of the anachronism. First, this Melchizedek, who
was presented in the Bible as a mysterious figure "without father and mother, without
genealogy, having neither beginning of days nor end of life" (Heb. 7:1-3), was the king of
Salem, thought to be Jerusalem, which didn't even become an Israelite city until David
captured it from the Jebusites (2 Sam. 6:1-15) in the 8th year of his reign, which would have
been about a thousand years after Abraham and 400 years after the time of Moses. Hence,
Bradford is arguing that because a non-Hebraic people had a vaguely described priest over
400 years before Aaron and his sons were appointed priests, then it *could have been* that
the Hebrews had had priests while they were slaves in Egypt who were just never mentioned
in the Bible. That's thin evidence indeed.

Volume 1990 - 2002 Issue


Page 1484 of 2049
Skeptical Review Edited by Farrell Till
In addition to this problem, I have to wonder why Yahweh selected only Aaron and his sons
to be the priests of Israel while they were trekking through the Sinai wilderness if he already
had priests who had been serving his "chosen ones" in Egypt. After all, didn't experience
count for anything? If Yahweh already had priests, then why didn't he stick with them and
appoint those who were already familiar with the duties of priests to continue in this service
or at least include them in the new priesthood. After all, the Israelite population in the
wilderness was about 2.5 to 3 million, if we are to consider the census figures in the book of
Numbers to be inerrant, and Aaron and his sons were only five. Right away, Yahweh reduced
this number by two when he consumed two of Aaron's sons for using "strange fire" in their
censors (Lev. 10:1-5), so this left only three priests to attend to the spiritual needs of 3 million
people. I realize that Yahweh's ways are higher than our ways, but it does seem a bit
confusing why Yahweh would have skimped so in appointing priests when there was a corps
of former priests from which he could have drawn emergency recruits. I'm sure there must be
some how-it-could-have-been explanation for this.

In the matter of the priests whom Yahweh seemed to have anachronistically referred to,
Bradford can't argue that these "possible priests" were just men who had presumed on their
own to become priests without authority from Yahweh, because the very passage that issued
the warning that the priests were to stay away from the mountain during Yahweh's confab
with Moses referred to them as "the priests... who come near to Yahweh" (Ex. 19:22). This
was the same terminology used later (Ex. 28:43; 30:20; 40:32) in reference to the priestly
duties of Aaron and his sons, so obviously the language of this passage, which I still believe
to be an anachronistic reference, was intended to convey that these "priests" were serving in
some official capacity in which they were to "come near to Yahweh," so this expression
conveys that these were men who had been sanctioned by Yahweh to serve as his priests. If
Yahweh had had such priests as this at the time, why were they not mentioned before this, and
why, as I have already asked, did Yahweh suddenly disenfranchise them and appoint an
entirely inexperienced line of priests? These are questions that Mr. Bradford needs to give
reasonably satisfactory answers to in order to make his explanation of the anachronism
credible.

The Greek (Septuagint) translation of the book of Daniel: Switching from anachronisms to
other issues being debated in TSR, Mr. Bradford has jumped into the dispute over the
authorship of the book of Daniel by citing a source that claims this book was translated into
Greek prior to 270 B. C., so since 270 B. C. obviously preceded 164 B. C., the approximate
date that many scholars assign to the authorship of Daniel, Bradford said that "the claim that
the book of Daniel was written in the 2nd or 1st century B. C. would be clearly erroneous."

Mr. Bradford is right, of course, because a book could not have been translated into another
language before it was even written in the original language, but I was glad to see that he
acknowledged that he had not yet checked for accuracy the source he cited. I'm going to give
him now what I hope will be a valuable lesson in trusting biblical fundamentalist sources. The
truth or falsity of a claim is always independent of its sources, but if Mr. Bradford ever takes
the time to check the claims made by biblical fundamentalists, he will find that distortion,
misrepresentation, and outright fabrications are rather commonplace. Chuck Missler is a
biblical fundamentalist whose writings would generally make the works of Josh McDowell
look downright scholarly. His purpose is to sell to his readers the notion that the Bible is the

Volume 1990 - 2002 Issue


Page 1485 of 2049
Skeptical Review Edited by Farrell Till
inspired word of God, so relying on the information in his articles to be trustworthy would be
somewhat like trusting the National Enquirer to give reliable evidence that alien beings are
visiting our planet.

It would be inappropriate of me to make a statement like this about Missler without


supporting it with concrete proof that his claims about the Bible should be viewed with
suspicion until they are verified. Mr. Bradford referred to a web page article in which Missler
had said that the "Hebrew canon" had been translated into Greek "prior to 270 B. C." I
checked the article and found that he did say this, but, as I will show, his claim is a distortion
of what his own quoted source said about the Septuagint (Greek) translation of the Old
Testament. I urge Mr. Bradford to research this subject more in depth. If he will do this, he
will find that there is a scholarly consensus that translation of the Septuagint only began in
the 3rd century B. C. but was not completed until much later, in the 2nd or 1st century B. C.
The first part of the Old Testament to be translated was the Torah or Pentateuch, and other
books were translated later. The whole process took over two centuries.

This is, in fact, exactly what Missler's source clearly states. Look back to the final paragraph
of Bradford's article, and you will see that Encyclopedia Brittanica,*volume 10, p. 642, was
the source that Missler referred to. I have taken the time to find this edition of the
encyclopedia, and this is what it said:

Septuagint, abbreviated LXX, the earliest extant Greek translation of the Old Testament from
the original Hebrew, presumably made for the use of the Jewish community in Egypt when
Greek was the lingua franca throughout the region. Analysis of the language has establish that
the Torah or Pentateuch (the first five books) was translated near the middle of the 3rd
century BC and that the rest of the Old Testament was translated in the 2nd century BC
(emphasis added).

This is the very first paragraph in the entry about the Septuagint. The article later relates that
the translation process began at the request of Ptolemy II Philadelphus, who had reigned from
285-246 B. C., but it does not say the work was finished during his reign. The statement in the
first paragraph, however, is so clear that I can only believe that Missler intentionally misled
his readers.

I have a copy of the Septuagint with Brenton's English translation in the margin, and the
introduction to this edition states the same opinion as noted in the source that Missler
misrepresented: the first books translated in the Septuagint were the Pentateuch, and scholars
know only that the work had commenced by the year 285 B. C. (p. i). Eerdmans Bible
Dictionary says that "in the first half of the third century B. C. the Pentateuch was translated
into Greek, with translation of the remaining portions completed during the next two
centuries" (1987, p. 154, emphasis added).

This is what Encyclopedia Americana said about the dating of the Septuagint:

The version was made in Alexandria, beginning in the reign of Ptolemy II, for the use of the
large numbers of Alexandrian Jews who were no longer familiar with Hebrew. The first
portion to be translated was the Pentateuch; the Prophets followed, and eventually the

Volume 1990 - 2002 Issue


Page 1486 of 2049
Skeptical Review Edited by Farrell Till
Writings, that is, the Wisdom literature and the other books of the Old Testament. The latest
parts were most likely translated in the 1st century B. C. (Vol. 24, 1998, p. 566, emphasis
added).

Obviously, then, Missler has distorted a scholarly reference to leave the impression that it
supports a fundamentalist view of the Bible. Scholarship, however, does not support Missler's
claim that the book of Daniel was translated into Greek as early as 285 B. C.

Why do fundamentalists distort information as Missler has done in this case? I think Mr.
Bradford's acceptance of Missler's distortion explains why so many biblicists twist
information to give the appearance that it supports their view of the Bible. They are
reasonably sure that their audience will not bother to check their claims for accuracy. In other
words, they do it because they know that they can get away with it. A few may check them,
but they don't care what these may think about their misrepresentation. They are interested in
those they can win to their side by distorting information. The lesson in all this for those who
are interested in truth is simple: beware of biblical fundamentalists quoting sources. Their
sources may not really say what the fundamentalists are claiming.

Milosevic and Moses: Innocent or Guilty?


Sol Abrams

If and when Slobodan Milosevic is brought before an international tribunal on charges of war
crimes, his defense team will be able to quote the Bible to exonerate him. Passages in
Numbers and Deuteronomy could easily be made relevant to what is happening in Kosovo
today by comparing atrocities there to what happened in the towns of Midian some 3400
years ago.

In Numbers 31:15-18, after his soldiers had killed all of the men among the Midianites,
Moses ordered his army officers to kill all of the male children, kill all of the nonvirgin
females but to save alive all of the virgin girls for his troops. Prior to this, the Israelites had
taken all of the animals and goods of the Midianites and then burned all of their towns. If
genocide or "ethnic cleansing" is a war crime, then this act of Moses was clearly a war crime.

What possible reason could Moses have given in order to justify this horrendous act of
genocide? After all, wasn't he the great "law giver"? He claimed that Yahweh, the God of
Israel, ordered him to do this, because the Midianites worshiped a deity named Baal Peor. The
Midianites felt that Baal Peor was nature's god, the creator of the universe, whereas the
Israelites believed that their god Yahweh was the creator. The current situation in Kosovo is
remarkably parallel. The Albanians in Kosovo worship a creator whom they call Allah; the
serbs worship this creator but call him the "Holy Trinity." So, in effect, what we have here is a

Volume 1990 - 2002 Issue


Page 1487 of 2049
Skeptical Review Edited by Farrell Till
demonization of those people who refer to the creator by a different name. These people are
accused of worshiping a false god.

Religion and ethnocentricity become the fuel that incites people, who are ordinarily moral
under normal circumstances, to commit the most horrendous acts of moral depravity without
any pangs of conscience, because they feel justified in killing the inhabitants of a land and
taking their property, because their holy book has set examples for them to do so. In chapters
6 and 7 of Deuteronomy, Moses told the Israelites that God wanted them to smash the altars
of the Canaanite tribes, utterly destroy them, and possess their land. This is exactly what
Milosevic was doing to the people of Kosovo, smashing their mosques, massacring them, and
taking their land. Neither Moses nor Milosevic believed in religious freedom.

It is interesting to note that Jethro, a Midianite priest, gave refuge to Moses when he fled from
Egypt in fear of his life (Ex. 2:14-23). Moses married his daughter, who had two sons by him.
We are not told in the book of Numbers what happened to the relatives of the priest who had
befriended Moses, but apparently Moses had felt no sense of obligation to them. There are
also stories in Numbers that show that Moses did not allow anyone to criticize him or his
brother Aaron. When anyone did so they suffered dire consequences, such as plagues and
even death by fire that Yahweh sent forth to consume them (Num. 11:33-35; 16:31-35). Once,
when Moses' own sister criticized him, she was stricken with leprosy (Num. 12:1-10). The
Serbian dictator Milosevic doesn't tolerate criticism either. In short, neither Moses nor
Milosevic permitted freedom of speech or religion.

This is quite a contrast to the beliefs and practices of our "founding fathers." Belief in
religious freedom was clearly stated in Thomas Jefferson's Notes on the State of Virginia,
1782: "The legitimate powers of government extend to such acts only as they are injurious to
others. But it does me no injury for my neighbor to say that there are 20 gods or no god, It
neither picks my pocket nor breaks my leg." Similar statements were made by Benjamin
Franklin and Thomas Paine, the unsung hero of the American Revolution. They believed that
to do good was their religion and discarded all divisive, demonizing dogmas. How strikingly
different the attitudes of Moses and Milosevic!

Do actions speak louder than words? Milosevic and Moses, innocent or guilty?

(Sol Abrams, 1759 Stuart Street, Brooklyn, NY 11229-2631; e-mail, heliosol@aol.com)

The Failure of Isaiah's Prophetic Rantings


Farrell Till

Volume 1990 - 2002 Issue


Page 1488 of 2049
Skeptical Review Edited by Farrell Till
Biblical inerrantists never seem to tire of looking for vindication of the Bible in prophecy
fulfillments. No skeptic discussing the Bible with a biblicist can question its divine origin for
very long without hearing the biblicist say, "Well, what about all of the prophecy
fulfillments?"

The best way to answer this question is with a question of your own: "What prophecy
fulfillments?" This alone may be enough to stop the biblicist in his tracks, because he may
well be a typical Christian who is uninformed in the Bible and is simply repeating something
he has heard but doesn't know enough about to discuss intelligently. If, however, the biblicist
is someone who does have specific prophecy fulfillment claims in mind, they can usually be
rebutted by just analyzing the alleged prophecy in context to point out parts of the prophecy
that seem to be missing in the fulfillment event. Such missing parts can almost always be
identified. I have discussed in past articles this approach to debating prophecy-fulfillment
claims, so I won't rehash it here.

Another effective method to use in such discussions is to turn the tables on the biblicist and
ask him, "Well, what about all of the prophecy failures?" To do this, of course, one would
have to be familiar with specific examples of prophecy failures. Several of these have been
discussed in past articles, the most frequently mentioned one being Ezekiel's prophecy against
Tyre. A simple way to make this prophecy failure more problematic for the biblicist is to
compare it to Isaiah's prophecy against Tyre and focus on the inconsistencies in the two. We
have seen in prior discussions of Ezekiel's prophecy that he predicted that Tyre would be
destroyed and never rebuilt (26:14,21; 27:36,19), but in his many tirades against the nations
around Israel, Isaiah uttered a prophecy against Tyre that predicted a destruction that wasn't
quite as harsh as Ezekiel's. In 23:1, he said, "The burden of Tyre. Howl you ships of Tarshish;
for it is laid waste, so that there is no house, no entering in: from the land of Kittim it is
revealed to them." The prophecy continued in typical fashion through the chapter, predicting
waste and devastation, but beginning in verse 13, Isaiah clearly said that the destruction of
Tyre would be only temporary, not permanent:

Look at the land of the Chaldeans! This is the people; it was not Assyria. They destined Tyre
for wild animals. They erected their siege towers, they tore down her palaces, they made her a
ruin. Wail, O ships of Tarshish, for your fortress is destroyed. From that day Tyre will be
forgotten for seventy years, the lifetime of one king. At the end of seventy years, it will
happen to Tyre as in the song about the prostitute: Take a harp, go about the city, you
forgotten prostitute! Make sweet melody, sing many songs, that you may be remembered. At
the end of seventy years, Yahweh will visit Tyre, and she will return to her trade, and will
prostitute herself with all the kingdoms of the world on the face of the earth. Her merchandise
and her wages will be dedicated to Yahweh; her profits will not be stored or hoarded, but her
merchandise will supply abundant food and fine clothing for those who live in the presence of
Yahweh (23:13-18, emphasis added).

So we see that Isaiah had a very different opinion of Tyre's destiny. He said that it would be
destroyed and forgotten 70 years but at the end of the 70 years, Yahweh would visit Tyre and
it would be restored. Obviously, one could make a much better case for the fulfillment of this
prophecy than for Ezekiel's. Nevertheless, Isaiah's prophecy against Tyre poses a serious

Volume 1990 - 2002 Issue


Page 1489 of 2049
Skeptical Review Edited by Farrell Till
problem for biblical inerrantists. They must explain why Isaiah predicted only a temporary
destruction of Tyre, whereas Ezekiel predicted an everlasting destruction.

Biblicists, of course, will claim that they see no problem. A favorite "explanation" of the
discrepancy is that Isaiah, who lived about 150 years before Ezekiel, was referring to a
destruction of Tyre that would happen before Ezekiel's time, but if this is so, when did the
destruction that Isaiah predicted happen? Even if we assume that Isaiah's prophecy was
fulfilled immediately after he made it, this would mean that the island stronghold didn't exist
for 70 of the 150 years between Isaiah and Ezekiel. This would have left only 80 years for the
city to be rebuilt before Ezekiel's era. If this happened, where is the record of it? Surely, the
total destruction for 70 years of such a great city would have found its way into some records
of the time, but none exist. Could we stretch imagination and by using Mitchell's formula
assume that it is 0.99% certain that the city was destroyed at this time but just wasn't
mentioned in contemporary records? It's seems far more probable that this was just another
prophetic ranting that failed to materialize.

From the Mailbag


Farrell Till

No more long quotations...

I agree with your observation that I quote too many long passages. Therefore, I wanted to
send you the enclosed article by the critics Brian Colless of Massey University, New Zealand,
entitled "Cyrus the Persian as Darius the Mede in the Book of Daniel," Journal for the Study
of the Old Testament, Vol. 56, 1992, pp. 113-116. I will refer to this article in my next paper
because I think Colless makes some excellent points. However, my paper will avoid using lots
of long quotations. I think you were right to point out that this produces tedious and boring
articles to read.

You also accuse me of writing about only those scholars who agree with me (TSR, Vol. 10, p.
7, column 2), but that is not right. I have corresponded with many critical scholars, and I will
gladly share these letters with you. You are probably aware of these critical scholars and their
commentaries on Daniel: J. J. Collins, Andre LaCocque, John Goldingay, W. Sibley Towner,
and D. S. Russell. My correspondence with them is enclosed. I have several other letters from
less known critics, and I will be glad to pass them on too.

Thanks you again for the opportunity to join you in this discussion on Daniel. I have learned
much as a result, and I hope you have picked up a few things along the way too.

Volume 1990 - 2002 Issue


Page 1490 of 2049
Skeptical Review Edited by Farrell Till
(Everette Hatcher III, P. O. Box 23416, Little Rock, AR 72221; e-mail,
everetteiii@aristotle.net)

EDITOR'S NOTE: This letter from Hatcher arrived after the July/August issue of TSR had
already been set up for press. I could not print it in that issue without reediting the "Mailbag"
column, and I didn't have time to do that.

In promising that he will use no more long quotations, Hatcher has missed the point I was
making about his appeals to authority. There is nothing wrong with "long quotations" if the
writer using them attempts to justify whatever major points the quotations make in support of
his case. In the Daniel debate, I have twice quoted almost the entire chapter of Daniel 5,
because I deemed it necessary to show that the writer of this passage obviously thought that
Nebuchadnezzar and Belshazzar were father and son. However, in addition to quoting this
passage, I also added my own commentary to explain why it is more reasonable to assign the
primary meanings to "father" and "son" in this text rather than unlikely secondary meanings.

My criticism of Hatcher's method has been that he too often quotes apparently for the sake of
showing that he has found some writers who agree with him, but he puts only minimal effort
into explaining why the positions of the writers he quotes should be accepted. When a writer
does this, he is making flagrant appeals to authority, which is a recognized logical fallacy,
and logical fallacies prove nothing.

Hatcher seems to think that I was objecting only to long quotations, but that wasn't the case at
all. There were sections in Hatcher's last article in which he strung together short quotations
without explaining why the positions stated in them were particularly convincing, and he also
strung together long lists of books and articles without quoting anything that they said. I
suspect his purpose in doing this was to say between the lines, "Look how much reading I
have done on this subject," but he doesn't need to convince me that he has put a lot of time
and effort into studying this subject. He should understand that a long list of references, such
as what we saw in column one on the first page of his last article, is actually self-defeating,
for rather than impressing readers, this will be more likely to bore them to the point of giving
up on the article.

I really saw nothing impressive in the letters that he sent to me from the "scholars" he has
been in contact with. Most of them were no more than terse notes that seemed intended to
make courtesy replies to correspondence that they didn't really have time to fool with. Some
of the replies were scribbled so hastily that I couldn't even decipher what they were saying.
The one typewritten reply from Andre LaCocque even expressed disagreement with Hatcher's
belief that the writer of Daniel lived in the 6th century B. C. "Whether the author of Daniel
lived in the sixth or in the second century BCE," LaCocque said in his letter, "it remains that
his visions and message are addressed specifically to people and circumstances of the 2nd
century. If you prefer to think of him as a 6th cent. man, I shall not argue with you. In that
case, however, there was no way for him and still less for his contemporaries to understand
anything that was revealed to him, as they all missed the point of reference of the
revelations." LaCocque's letter hardly lends support to Hatcher's position, and he would be
hard pressed to find a "scholar" outside the fundamentalist/evangelical community who
would agree that the author of Daniel lived in the 6th century B. C. At any rate, Hatcher

Volume 1990 - 2002 Issue


Page 1491 of 2049
Skeptical Review Edited by Farrell Till
indicated above that he intends to send another reply, so, if he submits an article that attempts
to argue logically instead of just appealing to authorities, we will have at least one more
opportunity to see if he has anything convincing to offer on this subject.

Two issues in a row!

I've been growing a little disturbed about the increasing abundance of pulpiteering
testimonials in the letters section from former congregationalists cooing about their
conversion to the bracing rigors of reason. Please understand that I fled the church not only
because I saw that the Bible was a compendium of contradictory nonsense and repulsive
superstition, but also because I hated getting spattered by the self-satisfied pious piddle of my
fellow worshipers--very similar to the sentiments that are coming to dominate "From the
Mailbag."

I realize that you're trying to ease the emotional trauma of some of these apparently fragile
children who are tentatively stepping out of the dark forest of ignorance into the sunlit fields
of truth; but please put a limit on these misty-eyed encomia and concentrate instead on the
many letters containing the biblical ammunition we so desperately need-- after all, we *are*
losing a country here.

"Biblical ammunition" reminds me-- I can find nothing in the Bible, Brewer's Dictionary of
Phrase and Fable, Wells (G. A.), Asimov, or Hoffmann about this: what started the tradition
of calling the three wise men Melchior, Gaspar, and Balthazar?

(John Carver, 10517 Oklahoma Avenue, Chatsworth, CA 91311)

EDITOR'S NOTE: Mr. Carver is going to think that he has hit the jackpot. After lamenting
that none of his letters had been published in the past, he has now made it into two
consecutive issues. He is right in supposing that one reason why I publish so many letters
from ex-Christians is to lend support to those who have reservations after having taken their
first steps away from their religious past. I receive many e-mail messages and phone calls
from people who are in this stage of skepticism, so such letters as the ones that Mr. Carver
complains about are published to help them see that those who choose rationality over
superstition eventually pass through the stage of doubt and reservation to find much more
satisfaction than they had ever had before.

Another reason for these letters is to counter the doom-and-gloom position of theists who so
emphatically state that skepticism/atheism has nothing to offer but pessimism and
unhappiness. If that is so, then why do ex-Christians almost universally agree that they are
far more happy now than when they were shackled to biblical superstitions?

I can't answer Mr. Carver's question about the wise men. I've never bothered to investigate
the matter.

Wrong issue received...

Volume 1990 - 2002 Issue


Page 1492 of 2049
Skeptical Review Edited by Farrell Till
Although I have never been a great believer in the Bible, I have only in the last few years
delved deeper into the whole issue of faith and believing. Your magazine has been a great
help in sorting out my own thoughts as well as in discussing these things with other people
(atheists as well as believers). Thank you.

May I point out that I received the January/February 1999 issue yesterday for the second time.
I suppose this was a mistake, so I hope to receive the March/April issue soon. Also, my first
name is Lidy, not Lindy (it sounds cute though).

(Lidy Mohr, Box 1598, Revelstoke, BC, Canada V0E 2S0)

EDITOR'S NOTE: Unfortunately, Mr. Mohr wasn't the only subscriber who received the
wrong issue. Somehow, some copies of the March/April issue got mixed with the May/June, so
in the mechanical way that envelopes are stuffed, the mistake was not noticed until some
mailings had gone out to Western Canada and possibly England too. A few have written to
notify us and have received replacement copies, but I have no way of knowing who else may
have gotten the wrong issue. If you were one who did, we will replace it upon receiving
notification.

I apologize for the misspelling of Mr. Mohr's first name. The mistake has been corrected in
the address files.

Allaying disquietudes...

I understand you are a skeptic, one who doubts the existence of God. I hope to be able to allay
your disquietude and bring a measure of peace to your life. To this end, I shall try to answer
the questions: Does God exist? Who is the best witness to His existence? How is this made
known to us?

First, I will attempt, with my limited knowledge, to answer, is there a God? The answer is an
emphatic yes. Who is the best witness to His existence? Ironically, it is communism. Consider
this for a moment: communism vehemently denies the existence of God, yet it expends its
energies to expunge all concept of God. How can you expunge someone that doesn't exist?
Does the underworld expend its energies to eradicate superman? No, of course not. It is
illogical to fight something that doesn't exist; therefore, I conclude that God does exist.

Now to answer the last question: who or what tells us humans, subconsciously, that there is a
God, an all-merciful and loving Being? The answer is our soul, that spirit with which we were
infused by our creator from the moment of conception, known as the soul. There are 6.5
billion of us on planet Earth, and each of us has a soul, an exact replica of our Creator in His
image, a weak one at that but nevertheless His image. That is the creature that tells us there is
a God and leaves no human in peace who denies Him, the Supreme Being.

I ask you, please do not believe anything I say but think about it and research it, then show me
where I am wrong.

(Frank P. Calderon, 4010 Josephine Street, Lynwood, CA 90262)

Volume 1990 - 2002 Issue


Page 1493 of 2049
Skeptical Review Edited by Farrell Till
EDITOR'S NOTE: Mr. Calderon need not worry. There isn't much chance that I will believe
anything he says, because his letter was one of the most illogical attempts to prove the
existence of God that I have ever read. The Skeptical Review exists to discuss the biblical
inerrancy doctrine, but in the "Mailbag" column, I have given readers and subscribers a
great deal of leeway to discuss topics other than this, so I will accommodate Calderon and, as
he requested, show him where he is wrong.

First, let me put Mr. Calderon's mind at ease and assure him that he doesn't need "to allay my
disquietude and bring a measure of peace to my life." The "disquietude" in my life ended
when I finally found the courage to admit that the Bible can't possibly be what I was taught to
believe about it as I was growing up and that the probability that some god exists who
monitors my every movement and thought is so remote that it doesn't even warrant serious
consideration. Furthermore, no words could adequately express just how much peace of mind
those decisions brought into my life. Mr. Calderon, then, should direct his concerns about the
disquietude of others to those who may have a need for his services. He is wasting his time
writing to me about his superstitious nonsense. He says that we all have "souls" that leave "no
human in peace who denies God," but I hate to tell him that my "soul" must be falling down
on the job, because I find myself perfectly at peace with my total and complete lack of belief
in this god Calderon is trying to peddle.

The proof he has offered that this god of his exists proves only that he has a serious need to
do a bit of studying in elementary logic. He should begin by reading about the fallacies of
begging the question and circular reasoning. "Is there a God?" he asked, and his answer was
an "emphatic yes." Well, boy, that's enough proof for me. Who could ask for more evidence
for this god's existence than Frank P. Calderon's emphatic yes. Obviously, Calderon doesn't
understand that argument by assertion proves nothing.

He then asked, "Who is the best witness to God's existence?" His answer? Communism is the
best witness, and he arrived at this conclusion by a very peculiar line of reasoning.
Communism vehemently denies the existence of God, so God must exist, since no one would
expend time and energy fighting something that doesn't exist. Oh, boy, where should I begin
to answer this ridiculous argument? First, I'll call to Calderon's attention that communism is
an economic/ political ideology that has nothing to do with theism. It teaches that all property
and means of production should belong to the state so that the output of production efforts
can be shared by the people in accordance with their individual needs, but it takes no position
on the existence of gods. As a matter of fact, the book of Acts indicates that the earliest
Christians practiced a form of communism by selling individually owned houses and lands
and keeping "all things in common" so that "distribution" could be made to "each, according
as anyone had need" (4:32-35), but, if we are to believe the Bible, this band of early
communists certainly didn't vehemently deny the existence of God. To the contrary, they
"ceased not to teach and to preach Jesus as the Christ" (5:42) and "went everywhere
preaching the word" (8:4). Mr. Calderon seems to be laboring under the impression that
because many communist leaders apparently had no belief in God, that necessarily made
communism an atheistic movement. That would be somewhat like arguing that because most
of our political leaders are Christians, the United States is a Christian nation, but most
conservative Christians constantly deplore the fact that the U. S. is not what they consider a

Volume 1990 - 2002 Issue


Page 1494 of 2049
Skeptical Review Edited by Farrell Till
Christian nation, and so they spend a great deal of their time trying to change it into a
country that would conform to their criteria of what a Christian nation should be.

Calderon then absurdly argued that it is "illogical to fight something that doesn't exist," so
opposition to God must mean that he exists. According to this logic, Calderon would have to
believe that a person who would try to convince a Mormon that the angel Moroni doesn't exist
would be proving by his opposition to the Mormon claims about Moroni that this Mormon
angel does indeed exist. Likewise, if someone in a Hindu society should undertake to convince
Hindus that the god Vishnu doesn't exist, he would by the very fact of his opposition to Vishnu
be proving that Vishnu does exist. I seriously doubt that Calderon would be willing to accept
these consequences of his own logic. People don't try to prove that Superman doesn't exist,
because there is no need to. Everyone realizes that Superman is a fictitious character. If,
however, movements should develop that were based on the belief that Superman was a real
person, there would certainly be people who would oppose this view, especially if the
"Superman" groups should seek to force that belief onto the rest of society as Christian
groups try to impose their God/ Jesus beliefs onto society in general.

Finally, Calderon informs us that "our soul" tells us that there is a "God." I am sorry to
disappoint him, but I haven't heard anything from my soul yet on the issue of whether some
vaguely defined god exists. On the other hand, my brain tells me that there is no real evidence
for the existence of gods, angels, demons, souls, and such like, so Calderon will have to go
back to the drawing board and look for more evidence than what he has shown us if he wants
me to abandon all common sense and believe in this god of his.

Not one of Hutchinson's virtues...

I just received the May/June issue of The Skeptical Review. Roger Hutchinson has now
convinced me of one thing, that objectivity is not one of his better virtues. I am surprised you
dignified his article with a response at all.

(Christopher Boyce, 706 Copperline Drive, Chapel Hill, NC 27516; e-mail


orcha@shentel.net)

EDITOR'S NOTE: As this column showed last issue, Mr. Boyce is not alone in his opinion of
Hutchinson's articles. More follows below, the first one a letter that a reader addressed to
Hutchinson.

By the way, I publish articles like Hutchinson's for reasons I have explained many times.
Biblicists are their own worst enemies. If I publish their nonsense, those whose minds are not
yet brain-dead from fundamentalist indoctrination will see the absurdity of biblicism from
what its own spokesmen say.

A Letter to Hutchinson...

I just read your article in the May/June 1999 issue of TSR, and I feel compelled to bring to
your attention several points that you overlooked in your article. First, I get the impression
that you wish to deny responsibility on behalf of your religion for some degree of

Volume 1990 - 2002 Issue


Page 1495 of 2049
Skeptical Review Edited by Farrell Till
wrongdoings. I do not think it was ever Till's intention to lay blame for all of the ills and
hatreds of the world on Christianity. Christianity has, of course, done its share. So have other
religions as well as those who have no religion. If you aren't prepared to admit that, then you
are very biased. Do you think that web sites such as "The Nuremberg Files" and Fred Phelps's
"http://www.godhatesfags.com/" do not cause social unrest and in some cases lead to extreme
acts? Of course, they do! I have no doubts that these sites influenced people such as the
unknown assassin who killed Dr. Barnett Slepian. I also have no doubt that the assassin in
question will never turn himself in, and probably sleeps quite well at night. Why shouldn't he?
On one hand, he has "Nuremberg Files" for guidance; while on the other hand, he feels that he
is enforcing his own version of Mosaic Law, and appeasing God simultaneously. There is
irresponsibility in those behind the pulpits. Washing your hands of it is not the answer. If you
took pride in your religion, you would take steps to vindicate its good name.

Second, in your list of modern day atrocities committed outside the grace of God, I find it
interesting that you did not mention Adolf Hitler. Had you done that, I could so easily point
out that Hitler's earliest ambition was to be a man of the cloth. He once told his people that
they should only breed with other Germans so that their children could be "born in the image
of God, and not a cross between man and beast." Could his faith in God have been the core of
his racism? Why not? If this sounds absurd, please allow me to illustrate the point for you.
Imagine a race of proud people who believe that God is on their side (which the Nazi party
stated time and time again) and eventually bring their bigotry to its ultimate conclusion by
annihilating (or attempting to annihilate) several other adjacent races. I have just accurately
described Nazi Germany as well as the Old Testament.

Finally, I take issue with you for referring to Till's writing as "the blustering of a disgruntled,
old man." No, dear sir, "disgruntled" is what would describe Till had he remained in the
Church of Christ (in my humble opinion). That is derogatory. That is a cheap shot. Till has
produced some of the best writing I have ever seen on the subject of errancy. In five years of
TSR, I have seen only two typographical errors ever made (discounting, of course, articles
submitted by Christians). There simply exists no body of work in all of Christian literature
that can be held to that standard.

All else aside, I feel your article was driven by anger more than reason. I feel it was a "hit and
run" styled literary attack, and doubt that a competent follow-up could even be written for it.
If I am wrong, prove me wrong. Respond to this letter.I don't think you can, or will.

(Chad D. Kelly, P. O. Box 183, Greenwell Springs, LA 70739-9998; e-mail, constat@ibm.net)

EDITOR'S NOTE: Unfortunately, I have made a few more than two typographical errors. I
proofread everything several times before sending it to press, but some errors manage to get
by. I groan every time I see one when I am reading through TSR after it has returned from the
printer. I guess I need the inspiration of the Holy Spirit to guide me to inerrancy.

I'm inclined to think that Hutchinson will send a reply to this letter. He's the type who just
doesn't recognize the truth of my often-repeated axiom: Give a biblical inerrantist enough
rope, and he will eventually hang himself.

Volume 1990 - 2002 Issue


Page 1496 of 2049
Skeptical Review Edited by Farrell Till
An insult to all skeptics...

Roger Hutchinson's article insulted all skeptics in general and you in particular. It indulged in
baseless speculation as to the motives of those who have considered the case for Christianity
and rejected it. Hutchinson also compounds the offense with his all "sweetness and light"
view of Christianity, calling it a "constraining influence" that prevented even worse atrocities
in the world, and, for bad measure, quite illogically gave the religion you've rejected credit for
whatever decency in you he may grudgingly concede.

Accepting such lofty credit but none of the blame when it comes to his religion is not only a
dishonest position but an indefensible one. Not only is there evil in Christianity, but decency
can be found outside it. Christians should hang their heads in shame in comparison to
Buddhism. Almost never does one hear of one sect of Buddhists murdering another over
doctrinal differences, something Christian sects spent several centuries doing. There have
been no religious wars in Buddhism, no inquisitions, no crusades, no witch hunts. Buddhism,
a human invention, is much closer to having achieved what Christianity only pretends to be.

Not only must Christians accept the blame for horrendous results, but Hutchinson would
certainly be miffed to hear that a case can be made for an assertion that even the atrocities of
communism derive in part from Christianity. The early Christians, after all, were communists.
Acts (2:44; 4:32) says that they "had all things in common," the very essence of communism.
They gave "every man according to his ability" (11:29) and received "according as he had
need" (4:35; 2:45), revealing a proverb appropriated by a French socialist, after Acts, and
quoted by Marx, who has always gotten the credit. Communism is a secularized form of
Messianism that bears all the aspects of a religion with an infallible founder, a holy writ,
sacred icons and relics, and persecution of heretics. The ideal communist state, like the
kingdom of heaven, is something that was always coming, never here. Without Judaic
Messianism and apostolic communism, Marxism never would have existed. So we see that as
a mindset, Christianity can be so potently dangerous that even its unintended offspring can
pose a menace, not the least to its unknown parent.

(Stephen Van Eck, RR 1, Box 62, Rushville, PA 18839)

More disagreement with Hutchinson...

I was looking forward to Roger Hutchinson's latest defense of his belief in inerrancy, and was
rather disappointed to see him degrade from mere sophistry, evasion, and ad hominem
attacks, to including parody as well. Three paragraphs into the article I stopped to make sure
that the author was indeed the inerrantist Hutchinson: the assertions he makes are identical to
the criticisms skeptics make of inerrantists. This appears to be a move of desperation on his
part.

He states that skeptics "denigrate the Bible when they cannot twist the Bible to say what they
want." Er, that is exactly what skeptics do not do. We assert that the Bible's authors meant
exactly what they wrote: it is the inerrantists who assert otherwise, and go through energetic
gyrations to make the Bible's errors and self-contradictions mean what they do not. I have
been reading TSR since early 1994, and I cannot recall any instance where skeptics insisted

Volume 1990 - 2002 Issue


Page 1497 of 2049
Skeptical Review Edited by Farrell Till
the Bible meant something other than what it says: indeed, skepticism of the Bible is
predicated upon taking the Bible exactly as it is written. I'm rather surprised that Hutchinson
has not realized this yet. Has he not been paying attention?

In the forth and fifth paragraphs, we see more of the same: Hutchinson takes valid criticisms
made by skeptics and asserts that the skeptics are guilty of those actions. This reminds me of
sophomoric elementary school yard taunts of "I know what you are but what am I?" That his
parody does not apply to the skeptics is not something he concerns himself with; his chief
motive appears to be a lack of anything better to say.

The rest of his article involves questions of "blame" about many social problems that have
nothing to do with Biblical errancy, so I am at a loss to understand why Hutchinson wasted so
much valuable space that he could have used to convince us that the Bible is inerrant. The
issue is inerrancy, not what Clinton does when his zipper is down. If one wishes to explore
the religious basis for racism, tribalism, authoritarianism, intolerance, and group violence, I
suggest he read the material available by Professor Bruce Hunsberger of the Wilfrid Laurier
University in Waterloo, Canada (see Prometheus Books). In my opinion (and the data tend to
show this opinion is valid), the greater the religious convictions one holds, the 'less' moral and
ethical the individual.

Those who love God the most tend to love their fellow human beings the least. What Jesus is
said to have taught and what Christians do is often diametrically opposite: love one's enemies;
bless those who curse you; if someone asks for your purse, give him also the cloak from off
your back. I am a neo-Pagan minister with a few Christian friends. When they question my
Paganism, I ask them in return: Why should I accept the Christian teachings when large
numbers of Christians refuse to follow them?

(David Rice, Mariner's Ministries, 723 Calle Casita, San Clemente, CA 92673-2708; e-mail,
shydavid@airspeed. com)

Who's to blame?

I just read the two faces of the bible and enjoyed it very much. It was predictable the day after
the Colorado school shootings that the Springfield News began receiving a flood of letters to
the editor blaming the violence on church/ state separation and demanding that prayer and
bible reading be allowed in schools. One letter even proposed arming teachers, janitors and
selected students. Wow, give a Bible in one hand and an AK47 in the other! Several letters
blamed the violence on abortion, and one minister said that scientists who teach a theory that
life developed from molecules were responsible for "children killing children." There has
never in history been a case of a scientific theory being the cause of a single death while one
verse in Exodus, "Thou shalt not suffer a witch to live," has been directly responsible for
many innocent women being cruelly burned at the stake.

My wife and I have just returned from Japan where the vast majority of people are agnostic
and Christians number about 1%. Abortion has been legal for many years, yet they have a
crime rate that is minuscule compared to the U.S. It seems to me that many Christians
proclaim moral superiority and then retreat behind a privileged sanctuary: "How dare you

Volume 1990 - 2002 Issue


Page 1498 of 2049
Skeptical Review Edited by Farrell Till
attack my religion" when you point out the evil many Christians have done such as Jim Jones,
David Koresh, and the many members of the clergy who have molested children! Every Ku
Klux Klan meeting was opened with a prayer just before they rode off to lynch our fellow
citizens. Three of the school shootings took place in the heart of the Bible belt, and at least
one perpetrator was described by his pastor as a Christian.... Oh, well, kind of out of your area
since you concentrate on biblical errancy, but thanks for letting me blow off steam.

(Bob Oliver, 607 North 38th Street, Nixa, MO 65714; e-mail, nakura@mocom.net)

EDITOR'S NOTE: The court decision banning school sponsored prayers in classrooms has
been blamed for just about every crime that has since been committed. People who argue so
irrationally should take the time to review the logical fallacy named post hoc, ergo propter
hoc (after this; therefore because of this). The truth is that crime was on the increase in the
United States well before this court decision was rendered. David Mooney, a longtime
subscriber to TSR, has written an article carefully documented from sources published prior
to the court decision that shows the growth of crime was already out of control before "God
was removed from the classrooms." I'll be glad to e-mail this article to anyone who would like
to see it.

Smith's letter continued...

Religious organizations control the venue. They don't have to come to us; it is we who have to
chase after them. There is nothing to get even close to them. If they want a tape or lecture
series, they seek out one of us who is weak, and they go after the "atheist." The commercially
produced piece (tape or transcript) just solidifies the evangelical's image of us. The other
more common ploy is to start a debate, after it is clear they are losing, declare to the faithful
that some cannot be saved and to listen to such as these would only lead the flock to
damnation.

They have the means to control their own content. Christian radio and TV stations have risen
by some accounts 40 to 60 percent over the last five years. In my state of Ohio, there isn't a
county that isn't covered. But think about it: Other people pay for your station, when you have
acquired enough capital, pay it off and then it's your station. Since the funds were from gifts
or donations, you don't owe anything or anyone and can do what you wish with the station;
it's yours. You could keep some of the Christian theme, go public during certain hours of the
day, keeping both religious funding (donations) and profit from advertising. You then have a
radio or TV empire at the nominal cost of next to nothing. Not a bad gig.

The national picture isn't much better. There are now two publishers of school textbooks,
eight major holders of radio, TV, satellite and cellular communications, and the ubiquitous
newspapers. Magazines are under fewer and fewer publishers. The flow of information is
being controlled ever so much by just a very few powerful organizations. Email and other
"on-line" services are similarly going in that direction. Communication of all forms will be
directed through these conduits, and there will be content management (censorship) of ideas
and expression. All this for the good of the children!

Volume 1990 - 2002 Issue


Page 1499 of 2049
Skeptical Review Edited by Farrell Till
Freethinkers had better start thinking about how we can increase our numbers, or our days
may well be numbered. There may well be a large number of us out here, but we are not
organized enough to merit any notice from the media or the public.

(Douglas L. Smith, P. O. Box 513, Tiffin, OH 44883-0513; e-mail, imtruth@excite.com)

EDITOR'S NOTE: The first part of Mr. Smith's letter, published in the issue before this one,
discussed (with some justification, I believe) the tendency of the freethought movement to
fragment into competing groups. No doubt, this hinders the growth of freethought in general;
however, much of what he says in this letter is not really the fault of a fragmented freethought
movement but an advantage that Christendom enjoys by virtue of a membership that it has for
the most part inherited from gains made in its past. I still believe that Christianity is steadily
losing ground and that the continued growth of the information age will only increase its
losses.

Volume 1990 - 2002 Issue


Page 1500 of 2049
Skeptical Review Edited by Farrell Till

Skeptical Review
Volume 10, Number 6
November/December 1999
Farrell Till, editor

• "Cassie and Jesus"


The editor exposes elements of urban legend in the claim that a student at Columbine
High School said yes when one of the gunmen put a gun in her face during the
massacre and asked if she believed in God.
• "What Men with David?"
Farrell Till exposes an error in Jesus's claim that David had men with him when he
went into the "house of God" and obtain loaves of showbread from the high priest.
• "A Poor Selling Job: Another Till Bankruptcy"
Roger Hutchinson continues his effort to whitewash the history of Christianity.
• "Where Up Is Down and Right Is Left"
Farrell Till replies to Roger Hutchinson's latest attempt to exonerate Christianity of all
blame in past atrocities in Western countries.
• "Observations on the Use of 'Genea" in the New Testament"
Brian Rainey discusses flaws in fundamentalist attempts to make the "generation" in
Matthew 24:34 refer to a time in the distant future.
• "From the Mailbag"
Readers and editor Farrell Till discuss a variety of religious issues.

Cassie and Jesus

Volume 1990 - 2002 Issue


Page 1501 of 2049
Skeptical Review Edited by Farrell Till
In recent issues, we have seen how that legends can begin and spread rapidly even in our time
when one would think that rapid-communication systems would be able to nip false tales in
the bud before they could take root and develop into legends, but, if anything, rapid-
communication media seem to increase the growth of legends and the speed with which they
can spread. The incidence of modern legends has become so commonplace that our language
now has a term for it, urban legend. The Skeptical Review has recently focused on three urban
legends that seem particularly tenacious, the claim that NASA has discovered a missing day
in time that confirms the biblical story of Joshua's long day; the tale of the verbally abusive
atheistic professor who is finally put in his place by a courageous Christian student that stands
up in class to cross-examine the professor and show by his own logic that he has no brain; and
tales of modern-day Jonahs, who have been swallowed by whales and later found alive. The
last two of these legends are discussed in the letters column of this issue, and the first one
keeps circulating on the internet no matter how many times it is debunked.

The tragedy at Columbine High School in Colorado has spawned a new urban legend that
seems destined to dwarf all of the others. Internet sites are now discussing daily the case of
Cassie Bernall, a student who was killed during the shooting incident there. According to
stories that circulated about Ms. Bernall after the massacre at Columbine High, one of the
gunmen at the school put a gun to her head and asked her if she believed in God, and when
she answered yes, the gunman killed her. The story spread quickly, especially on Christian
internet sites, and almost overnight Ms. Bernall became a modern-day Christian martyr. The
young lady's mother has since written a book She Said Yes: The Unlikely Martyrdom of
Cassie Bernall, which has become a popular seller.

After the story had spread, literally around the world, rumors began to circulate that Ms.
Bernall may not have been the person who was asked if she believed in God. On Tuesday,
September 28, 1999, the *Denver Post* ran a story in which Val Schnurr, an 18-year-old
student who survived the tragedy, claimed that she was the one who had been asked by the
gunman if she believed in God. According to Ms. Schnurr, a shotgun blast had struck her
while she was hiding under a library table. Bleeding from nearly three dozen wounds she had
received from the blast, Ms. Schnurr said that she then pled for her life, "Oh, my God, oh, my
God, don't let me die." She said that one of the gunmen then asked her if she believed in God,
and she said yes. While the gunman was reloading, she managed to crawl away, but a friend
who had been hiding with her was killed along with Cassie Bernall and ten others.

Investigators have since acknowledged that there is doubt about who the student was who said
yes when asked if she believed in God. "We have conflicting witness statements from several
kids who were in the library," Sheriff's spokesman Steve Davis said, "but this is not
something we're out to prove or disprove. It's not really a part of the investigation we're
doing." He said that determining who the real person was in this incident is not a "priority for
investigators."

Val Schnurr is now attending college, and she has said that she has no way of knowing if a
gunman asked Cassie Bernall the same question. "I don't want to be famous or deemed
anything," she said. "I said I believed in God out of respect for myself and respect for God.
That's it."

Volume 1990 - 2002 Issue


Page 1502 of 2049
Skeptical Review Edited by Farrell Till
This article is by no means an attempt to minimize the tragedy of what happened to Cassie
Bernall or the other students at Columbine High School. It was a tragedy that brought
inexpressible grief to the families of the victims, but this was a tragedy that just happened to
produce a story that relates directly to the issues that are addressed in The Skeptical Review.
The tragedy at Columbine High School happened just a few months ago, and within hours
reporters from news media all over the world were on the scene. Evidence has been examined
by professional investigators and witnesses who were on the inside have been questioned and
requestioned, but it has apparently been impossible to determine with certitude exactly what
happened. In particular, the investigation has not been able to determine if a gunman asked a
student if she believed in God and if the incident did occur, who the student was, yet despite
the conflicting evidence, the story of a modern-day martyr continues to circulate as if it were
unquestionably true.

The connection between Cassie and Jesus should now be obvious. In what is presently called
the "information age," we can't even determine the facts about an incident that happened in
our midst just a few months ago, and the facts in that case will probably never be known, yet
most people in our society would probably say that they believe everything that the New
Testament says about a man named Jesus, who lived almost two thousand years ago when
there were no newspapers, radio or TV stations, book publishers, public libraries, or national
archives to record the events of the time. Everything claimed about Cassie Bernall could have
easily happened. None of it involved the supernatural. A gunman could easily have shoved a
gun in her face and asked her if she believed in God, and she could easily have said yes, but
some are now recognizing that this may not have happened.

Jesus, on the other hand, allegedly spent much of his time involved in the supernatural. His
life even began supernaturally when he was born of a virgin--according to the only record of
his life--and from then on it was one supernatural event after the other. He changed water into
wine, he walked on water, he healed the blind and the lame, he fed multitudes of people with
just scraps of food, he calmed storms, he raised the dead and was finally resurrected from the
dead himself. No news reporters were present to record any of these events, and no
professional investigators were on the scene to question witnesses, yet it is all supposed to be
true in every detail, and relatively few people today question the truth of these claims.

In an age where urban legends have become commonplace, we should know better than this.
We should understand that if we cannot determine with reasonable certitude the facts about
events that happened right in our midst after they have been recorded and investigated by
professionals using methods that simply didn't exist in ancient times, and if we personally
witness the embellishment and romanticizing of events that happen in our time, then we
should realize that it was highly probable that such embellishment and exaggeration occurred
in ancient times when people like Abraham, Moses, David, and Jesus presumably walked the
earth. The person who says that he just knows that everything the Bible says about these
characters is the unembellished truth is someone whose sense of critical thinking has been
hopelessly impaired by religious indoctrination.

Volume 1990 - 2002 Issue


Page 1503 of 2049
Skeptical Review Edited by Farrell Till

What Men with David?


Farrell Till

Christians will sometimes ask me why most of my articles focus on "alleged" inconsistencies
and contradictions in the Old Testament. They are careful to use the word alleged, because
they, of course, contend that there are no inconsistencies or contradictions in the Old
Testament, but I suspect that the question betrays a recognition that there are many
troublesome passages in the Old Testament that believers in biblical inerrancy would just as
soon not have to think about. Their question implies a belief that the New Testament is above
reproach and that skeptics will be plumb out of luck if they should try to find discrepancies in
the New Testament. If this is what they think, they couldn't be more wrong, because the New
Testament, like the old, has plenty of inconsistencies and discrepancies. There are probably
no parallel accounts in the Old Testament that are as inconsistent as the resurrection narratives
in the gospels, but since I have already published a series of articles on those inconsistencies,
I'll feature another New Testament discrepancy in this article.

First, however, I'll just point out that even if the New Testament could be proven inerrant in
everything it says, biblical inerrantists would gain nothing by evading problem passages in the
Old Testament, because the Bible consists of both the Old and the New Testaments, and
biblical inerrantists claim that the Bible in its entirety is completely inerrant, in matters of
science, history, geography, chronology, prophecy, and all other aspects, as well as matters of
doctrine and faith. Hence, nothing is gained when inerrantists try to divert attention from the
Old Testament when skeptics are discussing biblical discrepancies with them. The same deity
allegedly inspired both the old and the new, and Yahweh, the god of the ancient Hebrews,
was allegedly the kind, merciful, loving "heavenly Father" of the new. If there are
discrepancies in the Old Testament, then, biblical inerrantists certainly don't solve anything by
just saying, "Well, why don't we talk about the New Testament?"

If, however, biblical inerrantists want to talk about the New Testament, I will be glad to
accommodate them. In Mark 2:23-28, there is a problem that has defied many inerrantist
attempts to explain it away. Readers should note carefully the parts that I have emphasized in
italic print and especially carefully the words emphasized in bold print.

Now it happened that he [Jesus] went through the grainfields on the sabbath; and as they went
his disciples began to pluck heads of grain. And the Pharisees said to him, "Look, why do
they do what is not lawful on the sabbath?" But he said to them, "Have you never read what
David did when he was in need and hungry, he and those with him; how he went into the
house of God in the days of Abiathar the high priest, and ate the showbread, which is not
lawful to eat except for the priests, and also gave some to those who were with him?" And he
said to them, "The sabbath was made for man, and not man for the sabbath. Therefore the Son
of Man is also lord of the sabbath."

Volume 1990 - 2002 Issue


Page 1504 of 2049
Skeptical Review Edited by Farrell Till
A matter incidental to this article but one that I plan to write about in a future article is the fact
that this story has Jesus teaching what biblicists cynically call "situational ethics." To most
biblical inerrantists, morality is absolute, and so they will claim that morality is a black-or-
white issue or, in other words, that an act is either moral or immoral. To them, the "situation"
of an act has nothing to do with whether it is moral or immoral. If the act is immoral, then it is
immoral. It was immoral yesterday, it is immoral today, and will be immoral tomorrow, and
there is no situation that can make an immoral act moral. In this story, however, we see Jesus
defending his disciples in an act that was a violation of the sabbath when they plucked grain
as they were walking through a field on the sabbath day. This constituted working, and so to
the Pharisees (according to the story), the disciples of Jesus were sinning by violating the
sabbath. Jesus, however, defended his disciples on the grounds that the situation they were in
justified their act of gathering grain. They were hungry and needed food, and so their situation
made an act that would otherwise have been wrong proper for them to do. "The sabbath was
made for man," Jesus said in defense of his disciples, "and not man for the sabbath."

Clearly, Jesus was teaching situational ethics. In defense of the position he had taken, Jesus
cited an incident in the life of David, and in so doing, he made two statements that were
inconsistent with the Old Testament account of the event that he referred to. Jesus said that
(1) David went into the house of God in the days of Abiathar the high priest, ate the
showbread that only priests were allowed to eat, and (2) gave some [of the showbread] to
those who were with him. This story is recorded in 1 Samuel 21:1-6, and as the story was told
there, Abiathar was not the high priest but his father Ahimelech was serving in this office, and
David was not in the company of other men at the time but was alone, fleeing from king Saul,
who was plotting to kill him. One has only to read the story in 1 Samuel to see that either
Jesus or the writer of Mark made two statements that were not consistent with the Old
Testament record. Before we look at the passage, some background information is necessary.

For some time before this incident, friction had been building between David and Saul, who
was king of Israel at the time. Because of David's increasing popularity with the people, Saul
had come to hate him and was plotting to kill him. David had become fearful for his life, and
so when the king's feast of the new moon was just a day away, David expressed his concern to
Jonathan, the king's son who had become David's best friend, that Saul would try to kill him if
David attended the feast. David and Jonathan devised a plan whereby David would hide in a
field while Jonathan went to the feast to evaluate his father's mood toward David. If Jonathan
sensed that Saul intended harm to David, Jonathan would come to the field on the third day
and send a signal to David to let him know that his life was in danger. Jonathan went to the
feast and, as anticipated, he found his father furious at David. As previously agreed, Jonathan
returned to the field two days later and sent the signal that let David know that Saul intended
to kill him. David came out of hiding, talked briefly to Jonathan, and then began a flight from
Saul that would take him into Philistine territory where he would live as a guerrilla marauder
until Saul was killed in battle with the Philistines (1 Sam. 20:5-42).

The first stop that David made in his flight from Saul was at Nob, where he sought help from
the priest Ahimelech and not Abiathar, whom Jesus in the text quoted above, said was the
high priest at the time. When Ahimelech saw him, he asked David, "Why are you alone and
no man with you?"

Volume 1990 - 2002 Issue


Page 1505 of 2049
Skeptical Review Edited by Farrell Till
In his reply, David lied and said that he was on a secret mission for the king and that the men
who were with him on the mission were hiding in an appointed place. Let's look at the full
text to get an overview of what the Old Testament says allegedly happened between David
and Ahimelech that day.

David came to Nob to the priest Ahimelech. Ahimelech came trembling to meet David, and
said to him, "Why are you alone, and no one with you?" David said to the priest Ahimelech,
"The king has charged me with a matter, and said to me, `No one must know anything of the
matter about which I send you, and with which I have charged you.' I have made an
appointment with the young men for such and such a place. Now then, what have you at
hand? Give me five loaves of bread, or whatever is here." The priest answered David, "I have
no ordinary bread at hand, only holy bread--provided that the young men have kept
themselves from women." David answered the priest, "Indeed women have been kept from us
as always when I go on an expedition; the vessels of the young men are holy even when it is a
common journey; how much more today will their vessels be holy?" So the priest gave him
the holy bread; for there was no bread there except the bread of the Presence, which is
removed from before Yahweh, to be replaced by hot bread on the day it is taken away (1
Samuel 21:1-6).

I have emphasized the name Ahimelech in bold print to direct attention to the obvious fact
that the priest who gave David assistance on this occasion was Ahimelech and not Abiathar,
whom Mark claimed Jesus had said was the high priest at the time of this incident. Abiathar
was Ahimelech's son, who succeeded his father after Saul killed him for having aided David
in his flight (1 Sam. 22:16-19), so this is just another discrepancy in the book that biblical
inerrantists claim is perfectly unified and harmonious from cover to cover.

The situation as recorded in the passage quoted above is rather obvious. David knew that to
make his story of a secret mission believable to Ahimelech, he would have to pretend that he
had men with him to assist in the mission. That was a lie, of course, because David had not
been sent on a secret mission by king Saul; he was, in fact, fleeing for his life from Saul. So
when Ahimelech asked why no men were with him, David told another lie and said that they
were in hiding. However, there were no men with David, because when he learned from
Jonathan that Saul was planning to kill him, he had made a rapid retreat from the field he had
been hiding in for three days. He had had no time to find men who would be willing to flee
with him. This, then, was the second lie that David told on this occasion (if we are to believe
in the accuracy of the Old Testament account). After lying about being on a secret mission for
the king, David, who was alone in his flight, told the priest Ahimelech that he had men with
him waiting in an appointed place.

Ahimelech, being a priest, wanted assurances that these men, whom David pretended were in
hiding, were pure enough to eat the sacred showbread, and so he asked David if they had been
with any women recently. The Levitical law required that a man who had had sexually
relations with a woman be considered ritually unclean until he had bathed (Lev. 15:18), and
men going into battle (as Ahimelech would have assumed that these "men" with David might
do) were also required to be sexually clean (Deut. 23:10ff). Hence, the priest wanted
assurances that if he gave something as ceremonially sacred as the showbread to David and
his men, they would also be ceremonially clean before they ate it. So David lied again and

Volume 1990 - 2002 Issue


Page 1506 of 2049
Skeptical Review Edited by Farrell Till
said that "women have been kept from us as always when I go on an expedition." These men,
however, were nonexistent; David had fabricated them as a part of his false scenario to make
it believable to Ahimelech that he was on a secret mission for the king. This, then, was a third
lie that David told in order to dupe Ahimelech into helping him in his flight from Saul.

The writer of Mark, however, was apparently so superficially knowledgeable in this Old
Testament story of David's flight that he thought the men whom David had fabricated were
real and had also eaten the showbread.

Mark 2:24-26: The Pharisees said to him [Jesus], "Look, why do they do what is not lawful
on the sabbath?" But he said to them, "Have you never read what David did when he was in
need and hungry, he and those with him: how he went into the house of God in the days of
Abiathar the high priest, and ate the showbread, which is not lawful to eat except for the
priests, and also gave some to those who were with him.

The same mistake occurred in Matthew's account.

Matthew 12:3-4: He said to them, "Have you not read what David did when he was hungry,
he and those who were with him: how he entered the house of God and ate the showbread,
which was not lawful for him to eat, nor for those who were with him, but only for the
priests?

And Luke made the same mistake.

Luke 6:3-4: Jesus answering them said, "Have you not even read this, what David did when
he was hungry, he and those who were with him: how he went into the house of God, took
and ate the showbread, and also gave some to those with him, which is not lawful for any but
the priests to eat?

So three verbally inspired writers made the same mistake and gave companions to David
when none were with him. Furthermore, they had the omniscient Jesus, through whom the
world was created, saying that companions were with David, whereas the context of 1 Samuel
20 and 21 make it clear that David was alone on his flight and fabricated companions in order
to get help from the priest Ahimelech. If Mark ever read this story, he must have read it so
superficially that he didn't understand that David had merely made up a contingent of men
waiting in an appointed place so that his lie about a secret mission would seem credible to the
priest. Matthew and Luke, in using Mark as their source, repeated the mistake.

At this point, I should be able to put a period and go on to another article, but inerrantists
won't let it be that simple. No matter how compelling the evidence for discrepancy may be,
they will always invent a how-it-could-have-been scenario to "explain" away the discrepancy.
In the matter of David's companions, a popular "explanation" is that other men were with
David; the biblical account just didn't mention them except when David referred to them in
his conversation with the priest Ahimelech, but to confirm that the "men" David referred to in
this conversation were nothing more than part of a lie that David told Ahimelech in order to
get help during his flight from Saul, I'm going to take the previous chapter, section by section,
and show that, as this story was told by a presumably inspired writer, it left no opportunity for

Volume 1990 - 2002 Issue


Page 1507 of 2049
Skeptical Review Edited by Farrell Till
David to gather men to accompany him on his flight from Saul. I'll begin with the section that
tells of David's and Jonathan's plan to find out if Saul was plotting to kill David. It is rather
long, but the best way to attack an inerrantist quibble like this is to walk the quibbler through
his own inspired text to show that it offers him no support.

1 Samuel 20:16-34: Thus Jonathan made a covenant with the house of David, saying, "May
Yahweh seek out the enemies of David." Jonathan made David swear again by his love for
him; for he loved him as he loved his own life.

Jonathan said to him, "Tomorrow is the new moon; you will be missed, because your place
will be empty. On the day after tomorrow, you shall go a long way down; go to the place
where you hid yourself earlier, and remain beside the stone there. I will shoot three arrows to
the side of it, as though I shot at a mark. Then I will send the boy, saying, `Go, find the
arrows.' If I say to the boy, `Look, the arrows are on this side of you, collect them,' then you
are to come, for, as Yahweh lives, it is safe for you and there is no danger. But if I say to the
young man, `Look, the arrows are beyond you,' then go; for Yahweh has sent you away. As
for the matter about which you and I have spoken, Yahweh is witness between you and me
forever."

Certainly, there is nothing in this text to indicate that anyone else was present except David
and Jonathan. Indeed, the subject of the conversation was such that we can reasonably assume
that they would not have wanted others to overhear it. Furthermore, this conversation took
place before David began his flight from Saul, so even if others were present, that would not
indicate that they were present three days later. The continuation of the story doesn't even hint
that David had anyone with him as he hid in the field.

Verses 24-34: So David hid himself in the field. When the new moon came, the king sat at
the feast to eat. The king sat upon his seat, as at other times, upon the seat by the wall.
Jonathan stood, while Abner sat by Saul's side; but David's place was empty.

Saul did not say anything that day; for he thought, "Something has befallen him; he is not
clean, surely he is not clean." But on the second day, the day after the new moon, David's
place was empty. And Saul said to his son Jonathan, "Why has the son of Jesse not come to
the feast, either yesterday or today?" Jonathan answered Saul, "David earnestly asked leave of
me to go to Bethlehem; he said, `Let me go; for our family is holding a sacrifice in the city,
and my brother has commanded me to be there. So now, if I have found favor in your sight,
let me get away, and see my brothers.' For this reason he has not come to the king's table."

Then Saul's anger was kindled against Jonathan. He said to him, "You son of a perverse,
rebellious woman! Do I not know that you have chosen the son of Jesse to your own shame,
and to the shame of your mother's nakedness? For as long as the son of Jesse lives upon the
earth, neither you nor your kingdom shall be established. Now send and bring him to me, for
he shall surely die." Then Jonathan answered his father Saul, "Why should he be put to death?
What has he done?" But Saul threw his spear at him to strike him; so Jonathan knew that it
was the decision of his father to put David to death. Jonathan rose from the table in fierce
anger and ate no food on the second day of the month, for he was grieved for David, and
because his father had disgraced him.

Volume 1990 - 2002 Issue


Page 1508 of 2049
Skeptical Review Edited by Farrell Till
We learn in this section of the story, that Jonathan and David had made a plan. David, in
accordance with the plan, hid in a field while Jonathan went to the feast. We would certainly
understand that people were present at the feast of the new moon that Saul had given, but
David wasn't there. He was hiding in a field, and the text mentioned nothing at all about
anyone else's being with him. Indeed, if David had taken others with him that would have
jeopardized the whole plan that he and devised, because the whole purpose of the plan was to
hide David in a place where he would be safe from Saul. One person could very likely hide
unseen in a field for three days, but a group of men would have increased the chances of
being seen, not even to mention the possibility that if David had taken a group with him, he
would have run the risk of having one or more of them sneak away to blow his cover. The
very nature of the plan that David and Jonathan devised certainly implied that David hid alone
in the field.

The day after the clash with his father, Jonathan came to tell David what he had learned at
Saul's banquet.

^Verses 35-40:^ In the morning Jonathan went out into the field to the appointment with
David, and with him was a little boy. He said to the boy, "Run and find the arrows that I
shoot." As the boy ran, he shot an arrow beyond him. When the boy came to the place where
Jonathan's arrow had fallen, Jonathan called after the boy and said, "Is the arrow not beyond
you?" Jonathan called after the boy, "Hurry, be quick, do not linger." So Jonathan's boy
gathered up the arrows and came to his master. But the boy knew nothing; only Jonathan and
David knew the arrangement. Jonathan gave his weapons to the boy and said to him, "Go and
carry them to the city."

In this section of the chapter, there were only three persons: David, hiding in the field;
Jonathan, shooting the arrows; and the boy retrieving the arrows. After the boy had retrieved
the arrows, Jonathan sent him back into the city. That left just Jonathan and David, and as the
text will later show, only after the boy was gone, did David rise up in the field and go to
Jonathan. David's waiting until the boy was gone before showing himself to Jonathan was just
another indication of the urgent need that David felt to keep his whereabouts a tight secret, so
everything related thus far in this story indicates that David was by himself in the field.

Verses 41-42: As soon as the boy had gone, David rose from beside the stone heap and
prostrated himself with his face to the ground. He bowed three times, and they kissed each
other, and wept with each other; David wept the more. Then Jonathan said to David, "Go in
peace, since both of us have sworn in the name of Yahweh, saying, `Yahweh shall be between
me and you, and between my descendants and your descendants, forever.'" He got up and left;
and Jonathan went into the city.

So as soon as David learned from Jonathan that Saul intended to kill him, David "got up and
left," and Jonathan returned to the city. This statement ended chapter 20, and the next chapter
began with David's encounter with Ahimelech in the town of Nob. There isn't a hint anywhere
in this entire story that anyone else besides Jonathan, the boy, and David were in the field in
the final scene, so if Jonathan and the boy returned to the city, who were these men that some
inerrantists claim were with David but just weren't mentioned?

Volume 1990 - 2002 Issue


Page 1509 of 2049
Skeptical Review Edited by Farrell Till
Some inerrantists will agree that there were no men in the field and that David took no men
with him when he fled, but they find a solution to the problem in the answer that David gave
to Ahimelech's question, "Why are you alone, and no one with you?" David's answer was that
he was on a secret mission for the king and that he had "made an appointment with the young
men for such and such a place" (21:2). In other words, these inerrantists claim that men had
been sent, probably by Jonathan, to link up with David at some place but that David and the
men had not yet reached the rendezvous point.

There are serious problems with this explanation. After apparently convincing Ahimelech that
he was on a secret mission for the king, David said, "What have you at hand? Give me five
loaves of bread or whatever is here" (v:3). It seems rather unlikely that David, knowing that
he was going to link up with a contingent of men who would continue on with him in his
flight, would have asked for just five loaves of bread. How long would that have lasted a
contingent of any size? The request for just five loaves of bread would suggest instead that the
writer of this account understood that David was by himself on his flight. Furthermore, David
also asked Ahimelech to give him a sword or spear (v:8), and Ahimelech gave him the sword
of the giant Goliath, which was taken to the tabernacle after David had killed Goliath with a
stone hurled from a sling (vs:8-9). That David even requested food and weapons from
Ahimelech was strange indeed if he knew that he was going to link up with a contingent that
Jonathan had sent to help him in his flight from Saul. If Jonathan had sent such a contingent,
it would surely have left with food and weapons, and David would have known this. Why
then would he have risked exposure by going to Ahimelech for food and weapons if he had
known that both would probably be available when he linked up with the men Jonathan had
sent to help him?

As the story continues, we find more evidence that the writer didn't just think that no men
were with David when he went to Ahimelech for help but that he also didn't know of any
contingent of men waiting to meet David in an "appointed place," for after telling about
Goliath's sword that Ahimelech gave to David, the writer said, "David rose and fled that day
for fear of Saul and went to Achish the king of Gath" (v:10).

David fled that day. David fled that day! It's noteworthy that no one else was mentioned in
David's flight, and the rest of the chapter shows why. No one else was mentioned, because the
way the story was told, no one else was with David. He was alone in his flight.

When David arrived in Gath, a city of the Philistines, he aroused concern among the people of
the city.

The servants of Achish said to him, "Is this not David the king of the land? Did they not sing
to one another of him in dances, `Saul has killed his thousands, and David his ten
thousands'?" David took these words to heart and was very much afraid of King Achish of
Gath (vs:10-12).

David was "very much afraid of King Achish." If David had had a contingent of men with
him, he would surely have commanded some respect from the people of the town, especially
since David was the one who had killed the giant Goliath, who was from the town of Gath (1
Sam. 17:4-23). On the other hand, an Israelite in Philistia with no men in his company would

Volume 1990 - 2002 Issue


Page 1510 of 2049
Skeptical Review Edited by Farrell Till
have had good reason to be afraid. That the writer did think that David was alone with no one
to depend on for his security is evident from the way that David, after having heard the words
of concern spoken by the servants of Achish, allegedly feigned insanity while he was in Gath.

Verses 13-15: So he [David] changed his behavior before them; he pretended to be mad when
in their presence. He scratched marks on the doors of the gate, and let his spittle run down his
beard. Achish said to his servants, "Look, you see the man is mad; why then have you brought
him to me? Do I lack madmen, that you have brought this fellow to play the madman in my
presence? Shall this fellow come into my house?"

In this conversation between Achish and his servants, the king mentioned no one but "the
man" and "this fellow." If a contingent of men was with David, why were they not
mentioned? We also need an explanation for why David pretended to be insane on this
occasion. The reasonable explanation is that he had left Jonathan in haste, stopped at Nob to
get help from Ahimelech, and hurried on to Gath, where he was alone in Philistine territory.
He pretended that he was an insane man because he had no one in his company to help defend
him if he should be threatened. This is a far more reasonable interpretation of this section of
the chapter than the assumption that David was traveling with other men but they were just
never mentioned.

After chapter 21 closed with Achish and his servants discussing "the man" and "this fellow,"
the next chapter finally mentioned men who joined David, but this was well after he had
stopped in Nob to ask Ahimelech for help.

22:1-2: David left there [Gath] and escaped to the cave of Adullam; when his brothers and all
his father's house heard of it, they went down there to him. Everyone who was in distress, and
everyone who was in debt, and everyone who was discontented gathered to him; and he
became captain over them. Those who were with him numbered about four hundred.

So apparently concerned about his security, David left Gath, at which time he was joined by
others, and after this point in his flight from Saul, the Bible writer made frequent references to
"men" who were with David. David survived in Philistia as a guerrilla marauder, and as the
writer told of these exploits, he made frequent references to "David's men," "David and his
men," or "the men of David" (23:3, 4, 5, 13; 24:3, 4, 7, etc., etc., etc.). Isn't it strange that
David had had men with him for two chapters, who were never mentioned in the narrative,
but as soon as 400 men joined him in 22:1-2, the writer suddenly couldn't get enough of
referring to the men who were with David?

Let's notice too that the second verse in this chapter (quoted above) says that David was
joined at the cave of Adullam by (1) everyone that was in distresss, (2) everyone who was in
debt, and (3) everyone who was discontented. Still with all of the everyones in these three
categories, David's troop at this time numbered only "about four hundred." So if David had
had men with him before he arrived at the cave of Adullam, they certainly couldn't have been
anywhere close to the contingent of troops that David implied in 21:5 when he was seeking
Ahimelech's help. This is just another indication that David was clearly lying to Ahimelech
when he said that he had men with him. It also explains why David asked for only five loaves
of bread. Not having any men at all with him, he had had no need for a larger quantity of

Volume 1990 - 2002 Issue


Page 1511 of 2049
Skeptical Review Edited by Farrell Till
bread, which would have encumbered him in his flight and undoubtedly molded before he
could eat it.

When the biblical account of David's flight from Saul is analyzed section by section, there is
no other conclusion to reach except that the writer of this story thought that David was alone
until he was joined by other men at the cave of Adullam, which was well after the time that he
went to the priest Ahimelech to ask for food and weapons. That conclusion requires also the
conclusion that the writer of Mark erred in having Jesus say that David ate the showbread and
also gave some of it to the men with him, because there were no men with David. Whoever
wrote this tale had superficially read 1 Samuel 20 and incorrectly interpreted David's lie to
mean that he had men waiting for him at an appointed place.

Despite all the textual evidence that supports this conclusion, dyed-in-the-wool inerrantists
will still insist that there is no inconsistency in this matter. "If Jesus said that David had men
with him," they will say, "that's good enough for me, so he had men with him whether the
story in the Old Testament mentioned them or not." This, however, is a flagrant resort to
begging the question at issue, because those who so argue are trying to prove biblical
inerrancy by assuming biblical inerrancy. By so reasoning a believer in just any holy book
(Book of Mormon, Qur'an, Avesta, etc.) could prove that it is inerrant.

In the matter of who was the high priest at this time, Jesus said that Abiathar was, but the Old
Testament account says that Ahimelech was. Is this a discrepancy? Well, not according to
confirmed inerrantists. They have actually argued that there is no discrepancy, because Jesus
said only that this had happened "in the days of Abiathar the high priest," and even though
Abiathar wasn't high priest at the time, it still happened "in his days." That would be like
claiming it is correct to say that World War II happened in the days of President Nixon. Is any
other comment necessary?

A Poor Selling Job: Another Till


Bankruptcy
Roger Hutchinson

In his first article, ("Who's to Blame?" TSR, January/February 1999), Farrell Till sought to
show that Christianity was to blame for the activities of radical elements in our society. His
argument: Guilt by association. In a second article ("The Two Faces of the Bible," TSR,
May/June 1999), he backtracked saying only that it was reasonable to think that some
Christians might be suspect. His argument: You are what you read.

Volume 1990 - 2002 Issue


Page 1512 of 2049
Skeptical Review Edited by Farrell Till
In his third attempt ("A Poor Selling Job," TSR, July/August 1999), desperate to prove
something, he proposed that he is right unless it can be proven that the Bible can instill high
moral principles in Christians. His argument: If the Bible causes Christians to do right, then it
must also cause them to do wrong. The argument is as bankrupt as those which preceded it.

The central theme in Till's argument is that Christians are equally influenced by the positive
or negative accounts that they read in the Bible. When Christians read the positive things (e.
g., God is love), they rush out and love people. When they read the negative things (e. g.,
homosexuality is sin), they supposedly run out and persecute people.

The Bible records many events in history. Those dealing with God's judgment against evil are
unpleasant. Paul tells us that they are examples to Christians so that they should not lust after
evil. On the other hand, the Bible speaks of God's love and how He readily shows mercy to
sinners. Christians are told to go into the world and spread this message.

The Bible clearly describes God as one who demands high moral character from those whom
He has created and judges those who revel in doing evil. The Bible just as clearly reveals God
to be merciful and quick to forgive those who sin.

This is not lost on Christians who read all the Bible and not just the few verses that capture
the imagination of skeptics. Till would have us believe that these two concepts are
incompatible. They are not. The proof is seen in the methods used by parents and teachers to
influence a child's behavior. Following the example that God has set, parents and teachers
punish children when they do bad and praise them when they do good. The effect is that
children, wanting to avoid punishment and desiring praise, seek to do good.

So it is with Christians. The parts of the Bible that describe the wrath of God serve as a
warning. Christians heed that warning and avoid activities that warrant punishment. However,
when Christians read those parts which depict God's mercy and forgiveness, they naturally
seek to incorporate such things into their lives.

Within the Biblical context, the unwholesome parts of the Bible do not negatively influence
moral values. Instead, they reinforce the wholesome parts and drive Christians to seek the
highest moral character.

Till's misunderstanding of Biblical context leads to his illogical application of the Scriptures.
In this respect, he is consistent. We can look at the comments from his latest article and see an
abundance of distorted arguments that he and skeptics employ to support their ideas.

Wanting it both ways, Till equates the contribution that violence on television is said to make
to violence in our society with that which he speculates is contributed by the historical
accounts of violent activities found in the Bible.

Violence on TV contributes to violence in society because kids watch TV constantly and


receive a steady diet of violence. By contrast, no one other than Mr. Till, and certainly not
kids, purposely and constantly immerse themselves in the Biblical accounts which depict
violence. Only the twisted mind of the skeptic could conclude that constant exposure to

Volume 1990 - 2002 Issue


Page 1513 of 2049
Skeptical Review Edited by Farrell Till
outside influences will have the same impact as rare and infrequent exposure to outside
influences.

Till then ponders why movie producers do not depict the more unsavory parts of the historical
record when they dramatize Bible stories. Are skeptics so naive? No one produces movies for
children that include the unsavory parts of life. Such things are not deemed fit for children
because children are innocent and impressionable.

Even TV shows for adults only go so far. Many have homosexual characters. However, while
these shows may depict somewhat explicit sexual activity between heterosexual couples, they
purposely do not depict or even suggest the unsavory sexual activities in which homosexuals
engage. Deviant behavior does not sell to parents who have children watching.

Certainly, the Bible can be explicit in its depiction of homosexuality and other activities. That
does not mean that such activities should be explicitly portrayed in movies watched by
children. One would think that Till, who has children, would be more sensitive to this issue.
Perhaps, skepticism has distorted his thinking.

Citing Biblical verses out of context, Till, like skeptics in general, never quite seems to grasp
the importance of context. He is able to accurately quote verses, yet understanding them
escapes him. He becomes fixated on certain verses, for example those which speak of
homosexuality or of witches. Even though these verses will appear in a section that lists a
number of activities that God says are wrong, Till claims that they exert unusual power and
influence over Christians.

Till concludes that the verses on homosexuality and witches will bring out the worst in
Christians while those verses nearby will engender little or no response. One verse supposedly
results in a fanatical intolerance while the next is met with indifference. Till's conclusions
possibly reflect that which he espoused in his early years when he hired himself out to preach.

Taken in context, we should expect Christians to react to each of the sins listed in the Bible in
the same manner. If they do not, then the normal scientific response would be to look for
some factor which would cause one to discriminate between one sin and another. We know
that skeptics like to determine right and wrong based on personal preference. Christians do
not have that option.

How, then, do we explain the actions of people like Fred Phelps. Is he a Christian who
righteously reacts to what the Bible says about homosexuals but then ignores everything else
the Bible says? Or, is he a man predisposed to hate homosexuals who uses the Bible to get
what he wants? If Phelps were to equally condemn all forms of sexual immorality, whether
homosexuality or a President's adultery, there might be stronger grounds for concluding that
the Bible is influencing his actions. He does not. There is no evidence that the Bible exerts
any influence over his actions.

The motives of any person must be suspect who labels himself a Christian and then condemns
sin without promoting the greater Biblical themes of mercy and forgiveness available from a
loving God. The clear context of the Bible is condemnation of sin enveloped by the

Volume 1990 - 2002 Issue


Page 1514 of 2049
Skeptical Review Edited by Farrell Till
forgiveness of God. Till quotes verses as if context did not exist and would have people
believe that Christians must be as ignorant as he.

^Maybe, kinda, sort of, perhaps reasoning:^ Till cites two examples ostensibly to prove his
point. They further illustrate weaknesses in his reasoning. Regarding the definition of the
Hebrew words, ben and yalad, referral to the Hebrew lexicons is sufficient to establish that
the words have the meaning I ascribed to them and are used in that manner. Further, few
people would conclude that a 430 year span could be bridged by four generations. It is
obvious that the genealogy in Exodus does not name every direct descendent from Levi to
Moses. Those who believe that the genealogy must be complete must also contrive to reduce
the Egyptian sojourn from 430 to about 200 years in order to be consistent. Translation from
one language to another can be difficult and context often determines which of several
possible meanings should be ascribed to a word. In this case, the 430 year context dictates
which definitions of ben and yalad to use.

In the second instance, Till confuses speculation and probability. On approaching the site of
an automobile accident, one may speculate on various scenarios to explain how the accident
occurred even though no evidence exists to substantiate any one scenario. That speculation
would not mean that the scenarios are improbable.

One may freely speculate on the unknown details of history so long as that speculation
accurately accounts for the known facts. Till says that such speculation would be a complete
fabrication. That may be, but people still do it. The scenario I proposed using the Biblical
accounts was consistent with the information that we have. While Till rails against my
scenario because I speculated on unknown details, he is unable to argue that my scenario was
inconsistent with that information which was given. Thus, his frustration.

Who labeled the Khmer Rouge skeptics? Here, Till failed to grasp the argument I presented
despite its simplicity. Till has me labeling the Khmer Rouge as skeptics (which I did), but he
then failed to grasp the significance of this. The label put on the Khmer Rouge is immaterial
even if it fits.

The point is that the Khmer Rouge are not Christians nor do they espouse Christian ideals. By
using the example of the Khmer Rouge, I was able to show that great atrocities have been
committed in places and under circumstances that cannot be linked to Christianity.
Consequently, the investigator who finds both Christians and non-Christians associated with
atrocities would be wrong to jump to the conclusion that Christianity is the cause of atrocities.

What exactly did Hutchinson say about Fred Phelps? In this instance, Till correctly
concludes that I accused him of misrepresenting my position. He then argues that it is only
necessary to accurately quote that which a person says in order to accurately convey what a
person meant. Only a skeptic would think this way.

Most people know that it is possible to misrepresent a person by accurately, but selectively,
quoting what he said. For example, a person could accurately quote the Bible as saying, "The
wages of sin is death," and then state that the Bible says everyone is going to hell. The
quotation is accurate; the conclusion is not.

Volume 1990 - 2002 Issue


Page 1515 of 2049
Skeptical Review Edited by Farrell Till
Till certainly quoted me accurately, but not completely. He neglected to include the first word
of that quote, the word "maybe." Could the omission of such a simple word have altered the
meaning of the quote? In this case, it did. The argument I conveyed was that any number of
factors could explain why Fred Phelps acts the way he does. Maybe (or one example is that)
he has always personally hated homosexuals. Till failed to grasp this very simple argument.

Is Christianity responsible for all kinds of evil? Till says that I falsely accused him of
saying that Christianity is responsible for all kinds of evil. Rather, he explains, he merely said
that Christianity has been responsible for numerous moral atrocities. He and Bill Clinton
share a common bond.

Till offers two examples, the Salem witch trials and the inquisition. Of the Salem witch trails,
we both agree that they were marked by hysteria. Till says it was religious hysteria. However,
we see a similar hysteria very recently in the prosecution of day-care providers. I would
theorize that child exploitation may have been at the heart of the Salem witch trials as they
were with the day care providers. For those unfamiliar with the case, it involved
psychological exploitation of children by police and prosecutors to get the children to say
things that were not true. The adult day-care providers were then convicted based on what the
children had been led to say. Certainly, Till did not conduct much of an investigation of the
events in Salem and not enough to even begin to judge what happened.

Regarding the inquisition, Till leaves out crucial points. The targets of the inquisition were
often those who had a different perspective on the Bible. So, was this a case of Christians
persecuting non-Christians or non-Christians persecuting Christians? Till does not seem to
know.

In addition, the inquisition was often secular in nature. Those in authority saw an advantage to
promoting the inquisition as a mechanism for getting rid of the opposition. Even those who
initiated the inquisition, a religious hierarchy, seem to have been more intent on preserving
their authority than in promoting the gospel. During the period of the inquisition, some
discovered that the preaching of the gospel was a more effective tool for rooting out heresy
than was hanging.

All in all, Till has again shown the bankruptcy of skeptical thinking when it comes to
criticizing the Bible.

(Roger Hutchinson, 11904 Lafayette Drive, Silver Spring, MD 20902; e-mail,


RHutchin@AOL.com)

Where Up Is Down and Right Is Left


Farrell Till

Volume 1990 - 2002 Issue


Page 1516 of 2049
Skeptical Review Edited by Farrell Till

As long as Roger Hutchinson persists in bending over backwards to defend the absurd belief
that the Bible is the inspired, inerrant word of God, I will continue to give him publishing
space to expose his ignorance. Presently, he is on a crusade to show that I was wrong in even
suggesting that the Bible could possibly be influencing some of the violent activities that have
unfortunately become a part of modern society. We live in a time when those with obvious
religious agendas are resorting to murder and terrorism to try to force compliance with what
they think is a code of objective morality that "emanates" from the "nature" of their morally
perfect God, so when peaceful methods of persuasion fail to convince abortion providers to
close their clinics, the more radical fringes of these guardians of public morality undertake to
force compliance with their demand by bombing clinics and killing doctors. When
homosexuals refuse to give up a lifestyle that in all probability is as natural to them as
heterosexuality is to others, the self-righteous ones seek to beat and bomb them into
compliance too. Hutchinson doesn't deny that such acts of violence happen, because, of
course, he can't deny them. He just denies that the perpetrators of this kind of violence were in
any way influenced by the Bible, but he also denies that the Bible is in any way inconsistent
or contradictory, so readers of my response to Hutchinson should keep in mind that I am
responding to someone who lives in a sort of fantasy land where in matters related to his
religious beliefs, he sees only what he wants to see. This is a world of self-imposed exile,
where up can be down and right can be left if it must be made that way in order to find unity
and harmony in a collection of ancient writings that the exiles desperately want to be the
"inspired, inerrant word of God."

I have probably had as much experience debating biblical inerrantists as anyone, but I have
rarely seen distortion and misrepresentation as flagrant as we are now seeing from Hutchinson
in his frantic attempts to save face after having shot himself in the foot by claiming that the
Bible can't in any way be considered responsible for modern-day bigotry and violence. In the
three pages available for my response in this issue, I intend to address all of his arguments. If
the three pages are not enough to answer them all, I will continue my reply in the next issue.

Flagrant Distortion: Almost every paragraph of Hutchinson's latest tirade contains a


distortion of what I have said on this issue. "In his first article," Hutchinson said, "Farrell Till
sought to show that Christianity was to blame for the activities of radical elements" (p. 6, this
issue). If I said this, I wish that Hutchinson would quote where I said it. The way that he has
worded the statement, he leaves the impression that I have claimed that Christianity is to
blame for all activities of radical elements, but I have said no such thing. In the opening
paragraph of my first article on this subject, I referred to terrorist acts "committed by those
who have links with organizations that have biblical terms like `God,' 'Yahweh,' and
`Phinehas' in their names" (January/February 1999, p. 1) and went on to say that if
Christianity is not to blame for the acts of organizations bearing such biblical names, then
who is to blame? The only correction I need to make to this statement is that in assessing
blame, I should have used Bible instead of Christianity, because it is certainly possible that
organizations like "the Army of God" or the "Phineas [sic] Priesthood" could have members
who are not Christians, but that would not remove the fact that the very names of such groups
as these are derived from the Bible.

Volume 1990 - 2002 Issue


Page 1517 of 2049
Skeptical Review Edited by Farrell Till
The Phineas [sic] Priesthood is a typical example of extremist organizations that have derived
their very names from the Bible. This is a white supremacist organization, whose latest poster
boy is Buford Furrow, who walked into a Jewish community center in California and opened
fire on children who had returned from an outing. He then fled the center and shot to death a
postman, whom he crossed paths with, for no apparent reason except that the postman wasn't
white. After he had been taken into custody, Furrow said he hoped that what he had done
would be a wake-up call for Americans to kill Jews. Investigation of Furrow's background
established that he had had connections with the Phineas [sic] Priesthood.

Where did this group get its name? Well, to Hutchinson's chagrin, he will never be able to
deny that this is a name derived directly from the Bible. Aaron, the first high priest of Israel,
had a grandson named Phinehas who rose to biblical fame by conducting a purge of
immorality during an Israelite/Midianite orgy recorded in Numbers 25. When Phinehas
learned that an Israelite man had taken a Midianite woman into his tent, he went into the tent
and thrust a spear through both the man and the woman while they were copulating. Yahweh,
who always seemed to enjoy a good bloodletting then said to Moses, "Phinehas son of
Eleazar, son of Aaron the priest, has turned back my wrath from the Israelites by manifesting
such zeal among them on my behalf that in my jealousy I did not consume the Israelites.
Therefore say, `I hereby grant him my covenant of peace. It shall be for him and for his
descendants after him a covenant of perpetual priesthood, because he was zealous for his God,
and made atonement for the Israelites'" (vs:10-13).

The Phineas [sic] Priests, then, are members of an extremist group that named itself after one
of Yahweh's favorite people, a man who killed those who didn't quite measure up to his moral
expectations. And why not select this name for a hate group? Anyone reading this story can
see that the Bible clearly praised a man for killing people whom he considered moral
inferiors. Further reading in this general area of the Bible will show that the god Yahweh was
also a racist, who had selected the Hebrews to be "a people for his own possession, above all
peoples on the face of the earth" (Deut. 7:6) and that he told his "chosen ones" to kill
everyone in the other ethnic groups in Canaan (Dt. 20:16). Why wouldn't a group with a
racial- superiority mentality not think it appropriate to name themselves after a man who had
received a "perpetual priesthood" for killing people whom his racist god didn't like? Is
Hutchinson so naive that he thinks the selection of this name by a white-supremacist group is
merely coincidental and that the Bible had no influence on their choice? Does he really think
that the activities of extremist groups are not at all influenced by the Bible in a society where
the Bible is a perennial best seller and where even the smallest towns will be dotted with
several Christian churches? I guess he expects us to think that none of these extremists ever
read where the Bible gave presumably divine orders to kill those of other ethnic groups and to
purge their ranks of false worshipers, and thought that it really wouldn't be such a big deal if
they went and did likewise.

High moral principles: Hutchinson distorted my position again when he said that I was so
"desperate to prove something" that I had "proposed that [I] am right unless it can be proven
that the Bible can instill high moral principles." His claim was that I had said this in my third
article ("A Poor Selling Job," July/ August 1999), but I have looked all through the article and
can't find where I said it. I did, however, find where I said that Hutchinson, who has been
holding the Bible up to us as a fountainhead of high moral principles and "messages of love,"

Volume 1990 - 2002 Issue


Page 1518 of 2049
Skeptical Review Edited by Farrell Till
can't have it both ways. If he is going to argue that people can be influenced positively by
high moral principles, which I admit are taught in some biblical passages, then he must also
agree that the low morality of the Bible can influence people negatively. "(H)e can't have it
both ways," I said, and contend that the stories of debauchery and barbarity in the Bible have
no negative influence on people but that its "high moral principles" do have a positive
influence. "If he is going to insist that the `good' contained in the Bible can have positive
influences on people," I said, "he must admit the possibility that the `bad' contained in the
Bible can have negative influences" (p. 8). That is quite different from the spin that he has
tried to put on this section of my article. I was, in fact, agreeing with him that the Bible can
and does have a positive influence on people but that consistency demands he admit that the
horrendous tales of divine cruelty and barbarity in the Bible can influence people negatively.
His stubborn fundamentalism, however, will never allow him to admit this, so he must vent
his frustration and anger on me for telling the truth about the Bible. He has an obligation to
show us why a book that contains both good and bad will influence people to do good but
won't influence them to do bad. I don't read Playboy magazine, but I have heard people who
do read it say that it contains some excellent articles that have nothing to do with nudity or
sex. I'm sure, however, that Hutchinson would never say that it is all right for Christians to
read this magazine on the grounds that even though it is sexually explicit at times, it also has
some good articles in it that don't pertain to nudity and sex and that the "bad" materials won't
negatively influence readers anyway.

Before Hutchinson counters here with a claim that the Bible is different from Playboy and
other such magazines in that the Bible condemned immoral conduct and discouraged it by
recording the punishment of those who engaged in such activities, I'll shoot this
rationalization down before he can even get it off the ground. Anyone who believes this is
either ignorant of the Bible or else has his head in the sand. Since I have already mentioned
the story of Lot in an earlier article, I can use it as an example of how the Bible didn't always
condemn immorality but even sometimes presented despicably immoral characters as people
who were worthy of emulation.

According to the story as it was told in Genesis 19, God had decided to destroy Sodom for its
extreme wickedness and sent two angels to warn Lot and his family to leave before the
destruction came. These angels had evidently appeared as ordinary men, for when word of
their presence in Lot's house had gotten around town, a group of apparently homosexual men
gathered outside Lot's house and demanded that he send his visitors out so that "we may know
them" (19:5). Rather than telling the crowd that this was immoral conduct that he would have
nothing to do with, what did the righteous Lot do? He went out to the men and said, "I beg
you, my brothers, do not act so wickedly! Look, I have two daughters who have not known a
man; let me bring them out to you, and do to them as you please; only do nothing to these
men, for they have come under the shelter of my roof" (v:8).

Of all the appalling immorality depicted in the Bible, this has to rank high on the list. Lot was
willing to turn his daughters over to a crowd and let them be gang raped in order to save his
male visitors the indignity of rape, as if protecting the virtue of complete strangers was more
important than protecting the virtue of one's own daughters. I would think that Hutchinson
would be hard pressed to find even in the ranks of the most immoral men alive today, anyone
who would have so little regard for his own daughters, but a character in the Bible could so

Volume 1990 - 2002 Issue


Page 1519 of 2049
Skeptical Review Edited by Farrell Till
conduct himself and, as we will see later, be considered a "righteous man." This story, of
course, mirrors the belief in biblical times that women were inferior to men, and so Lot, if
such a person ever lived, was simply reflecting this belief. However, it will do Hutchinson no
good to offer this as an excuse for why a "righteous man" would have done such a thing,
because Hutchinson is a biblical inerrantist, who believes that morality is absolute. If morality
is absolute, then he cannot argue that although it would be morally despicable for a father to
conduct himself like this today, back then such conduct would have been morally acceptable.
If he makes such an attempt, I will remind him that he is again trying to play both sides of the
street.

So did Yahweh withdraw his angels and inform Lot that he was no better than anyone else in
Sodom and would therefore have to die along with the others? Not at all. Lot was told to leave
with his family and not look back when destruction rained down on the city (vs:12-22). As the
story was told, Lot's wife turned and looked back when the fire and brimstone were coming
down on Sodom, so she was changed into a pillar of salt. Lot and his daughters went on and
escaped the destruction. Let's take a moment to look at what we have here in this treasury of
"high moral principles" that Hutchinson is praising. Lot offered his daughters to be victims of
gang rape and was divinely led to safety; Lot's wife looked back in curiosity when the fire and
brimstone were raining down, and she was changed into a pillar of salt. Who couldn't love this
book of "high moral principles" that Hutchinson cherishes so dearly?

The story didn't end with the fate of Lot's wife. Lot and his daughters went up into the
mountains and lived in a cave (v:30), where the most despicable part of this sordid story
happened.

Now the firstborn said to the younger, "Our father is old, and there is no man on the earth to
come in to us as is the custom of all the earth. Come, let us make our father drink wine, and
we will lie with him, that we may preserve the lineage of our father. So they made their father
drink wine that night. And the firstborn went in and lay with her father, and he did not know
when she lay down or when she arose.

It happened on the next day that the firstborn said to the younger, "Indeed I lay with my father
last night; let us make him drink wine tonight also, and you go in and lie with him, that we
may preserve the lineage of our father." Then they made their father drink wine that night
also. And the younger arose and lay with him, and he did not know when she lay down or
when she arose.

Thus both the daughters of Lot were with child by their father" (19:31-36).

Let's suppose that a best-seller was on the shelves of bookstores all across the land and that a
part of the plot included a character who engaged in drunken orgies with his own daughters.
Can you imagine the furor that such a book would create among the guardians of public
morality? How many demands would we hear to remove this book from store shelves and
certainly from public and school libraries? The people making such demands, however, don't
think anything at all about the presence of Bibles on the shelves of bookstores and libraries.

Volume 1990 - 2002 Issue


Page 1520 of 2049
Skeptical Review Edited by Farrell Till
God didn't approve of Lot's conduct, biblicists will argue, but if that is so, just where is God's
disapproval stated in the Bible? The text that I quoted above simply gave a matter-of-fact
account of the escapades of Lot and his daughters. There isn't a hint of divine disapproval in
the story. The last two verses of Genesis 19 merely related (matter-of-factly) that the firstborn
daughter gave birth to a son whom she named Moab and that the younger daughter gave birth
to a son whom she named Ben-ammi. There is no condemnation even implied in the story.
Elsewhere, however, Lot was described as a "righteous man" when the writer of 2 Peter said
that in destroying Sodom and Gomorrah, God had rescued "Lot, a *righteous* man" who was
"greatly distressed by the licentiousness of the lawless" and went on to say, "(T)hat righteous
man, living among them day after day, was tormented in his righteous soul by their lawless
deeds that he saw and heard" (2:7-8). Yes, indeed, this "righteous man" was so "greatly
distressed" by the lawless deeds of those around him that he offered to let a group of men
gang rape his own daughters and then later engaged in a drunken orgy with these same
daughters. When we read about such "high moral principles" as these, I think we have to
agree with Hutchinson that there isn't a chance in the world that the Bible could have a
negative influence on anyone.

In my July/August article (p. 8), I referred to a TV series called Ancient Secrets of the Bible,
which in one of its programs had dramatized the story of Lot but had omitted all of the sordid
details that I have just summarized. I cited these omissions as an example of how even
defenders of the Bible recognize that some of its material isn't fit to be related. In his latest
"contribution" to our discussions, Hutchinson said in reference

Вам также может понравиться